You are on page 1of 1944

INSIGHTS ON INDIA MOCK PRELIMINARY EXAM - 2015

INSIGHTS ON INDIA MOCK TEST - 1


GENERAL STUDIES

PAPER-I
Time Allowed: 2 Hours

Maximum Marks: 200

INSTRUCTIONS
1. IMMEDITELY AFTER THE COMMENCEMENT OF THE EXAMINATION, YOU SHOULD
CHECK THAT THIS TEST BOOKLET DOES NOT HAVE ANY UNPRINTED OR TORN OR MISSING PAGES OR
ITEMS, ETC. IF SO, GET IT REPLACED BY A COMPLETE TEST BOOKLET.
2. You have to enter your Roll Number on the Test I
Booklet in the Box provided alongside. DO NOT
write anything else on the Test Booklet.
4. This Test Booklet contains 200 items (questions). Each item is printed only in English. Each item comprises four responses
(answers). You will select the response which you want to mark on the Answer Sheet. In case you feel that there is more
than one correct response, mark the response which you consider the best. In any case, choose ONLY ONE response for each
item.
5. You have to mark all your responses ONLY on the separate Answer Sheet provided. See directions in the Answer Sheet.
6. All items carry equal marks.
7. Before you proceed to mark in the Answer Sheet the response to various items in the Test Booklet, you have to fill in some
particulars in the Answer Sheet as per instructions sent to you with your Admission Certificate.
8. After you have completed filling in all your responses on the Answer Sheet and the examination has concluded, you should
hand over to the Invigilator only the Answer Sheet. You are permitted to take away with you the Test Booklet.
9. Sheets for rough work are appended in the Test Booklet at the end.
10. Penalty for wrong answers :
THERE WILL BE PENALTY FOR WRONG ANSWERS MARKED BY A CANDIDATE IN THE OBJECTIVE TYPE
QUESTION PAPERS.
(i) There are four alternatives for the answer to every question. For each question for which a wrong answer has
been given by the candidate, one-third of the marks assigned to that question will be deducted as penalty.
(ii) If a candidate gives more than one answer, it will be treated as a wrong answer even if one of the given answers
happens to be correct and there will be same penalty as above to that question.
(iii)

If a question is left blank, i.e., no answer is given by the candidate, there will be no penalty for that question.
http://insightsonindia.com

INSIGHTS ON INDIA MOCK TEST SERIES FOR CIVIL SERVICES PRELIMINARY EXAM 2014

http://insightsonindia.com

INSIGHTS

Page 1

1. With reference to number of digits in


mobile numbers, consider the following
statements
1. Every country in the world follows the
standard where mobile numbers have
10 digits
2. If India had allowed 9 digits mobile
numbers, unique numbers wouldnt
have been sufficient for all its
population
3. In India, Mobile number digits are
determined by National Numbering
Plan (NNP)
Which of the above statements is/are correct?
a) 1 and 2 Only

b) Vedangas
c) Samaveda
d) Rig Veda
4. Consider the following statements
1. The languages used in majority of
King Ashokas rock edicts is Aramaic
2. The language used in the edicts found
in the eastern part of the sub-continent
is a type of Magadhi
Which of the above given statements is/are
correct?
a)
b)
c)
d)

1 Only
2 Only
Both
None

b) 2 and 3 Only
c) 1 and 3 Only
d) All

2. With reference to Garo Hills and people


living there, consider the following
statements,
1. It is one of the earliest places in India
where agriculture was practised
2. The society is matrilineal
3. Rice is the main food of Garo people
Which of the above given statements is/are
correct?
a)
b)
c)
d)

1 and 2 Only
2 and 3 Only
1 and 3 Only
All

3. The name Bharata was used for a group of


people who lived in the north-west of
present day India. In which of the
following texts it was first mentioned?
a) Upanishads
http://insightsonindia.com

INSIGHTS

5. Consider the following statements with


reference to why hunters-gatherers moved
from place to place in ancient times
1. Fear of eating up all the available plant
and animal resources if stayed at one
place
2. Because animals to be hunted move
from place to place
3. Plants and trees bear fruit in different
seasons
Which of the above statements is/are correct?
a)
b)
c)
d)

1 and 2 Only
2 and 3 Only
1 and 3 Only
All

6. Koldihwa in Uttar Pradesh is known for


a) Neolithic site
b) Palaeolithic site
c) Buddhist site
d) None of the above

Page 2

7. With reference to Catal Huyuk, a


historical place in Turkey, consider the
following statements
1. It is the largest and best-preserved
Palaeolithic site found to date
2. It is a UNESCO World Heritage Site
Which of the above sentences is/are correct?
a)
b)
c)
d)

1 Only
2 Only
Both
None

10. Inamgaon is a famous megalith site on the


river Ghod where ancient burial grounds
have been excavated. The river Ghod is a
tributary of
a) River Godavari
b) River Krishna
c) River Bhima
d) River Narmada

11. With reference to the ashvamedha


sacrifice ritual of the ancient times,
consider the following statements

8. With reference to citadels in Harappa and


Mohenjodaro sites, consider the following
statements
1. They were built to the east of the city
on large elevated regions
2. The Great Bath, found in Mohenjadaro
is built in one of these citadels
Which of the above statements is/are correct?
a)
b)
c)
d)

1 Only
2 Only
Both
None

wander freely and it was guarded by


the his men
2. If the horse was held by other rajas,
they had to fight the raja conducting
sacrifice
3. If the horse was allowed to pass
without being held, those rajas had to
conducting sacrifice
Which of the above statements is/are correct?
a) 1 and 2 Only
b) 2 and 3 Only
c) 1 and 3 Only
d) All
12. During the mahajanapada period, we hear
about kammakaras. Who were these

Which of the above statements is/are


INCORRECT?

people?

1 Only
2 Only
Both
None

http://insightsonindia.com

ashvamedha sacrifice was let loose to

surrender their kingdom to the raja

9. Consider the following statements


1. In Harappan cities, crafts made of
copper, bronze, iron and silver have
been found during excavations
2. Strangely, Harappan civilization did
not use gold

a)
b)
c)
d)

1. A horse belonging to a raja conducting

a) Watchmen of the forts


b) Watchmen of villages
c) Landless agricultural labourers
d) Slave men and women

INSIGHTS

Page 3

13. With reference to Buddha, consider the

16. With reference to Zoroastrianism,

following statements

consider the following statements

1. He belonged to a large sangha known

1. It is a major religion in Iran

as Vajji sangha

2. It was founded by Zoroaster, an

2. He was a Kshatriya

Iranian prophet
3. Present day Parsis in India are the

Which of the above statements is/are correct?


a) 1 Only

descendants of Zoroastrians
Which of the above statements is/are correct?

b) 2 Only
c) Both

a) 1 and 2 Only

d) None

b) 2 and 3 Only
c) 1 and 3 Only

14. With reference to Upanishads, consider

d) All

the following statements


1. They contain conversations between
teachers and students
2. These were part of the early Vedic
texts
Which of the above statements is/are correct?

17. Which of the following are is/are ways in


which the government can be made
accountable to the people in general?
1. Media
2. Social movements
3. Voting
Choose the correct answer using the codes below:

a) 1 only

a)
b)
c)
d)

b) 2 Only
c) Both
d) None

1 and 2
2 and 3
1 and 3
All of the above

15. With reference to Mahaviras teachings,


consider the following statements
1. He taught that men and women
should leave homes if they wished to
know the truth
2. Mahavira taught in Prakrit language
Which of the above statements is/are correct?
a) 1 Only
b) 2 Only
c) Both

18. Consider the following statements about


autocratic and Democratic form of
government:
1. The autocrat is not responsible to the
people.
2. Autocracy is also termed as Monarchy
since decision-making style is same in
both.
3. No other form of democracy except
representative democracies exist till
date.

d) None
http://insightsonindia.com

INSIGHTS

Page 4

Which of the above statements is/are true?


a)
b)
c)
d)

1 and 2
2 and 3
1 and 3
Only 1

a)
b)
c)
d)

19. Consider the following statements:


1. No Women had the right to vote in
elections in India before
Independence.
2. The first Woman suffrage movement
in India dates back to the immediate
aftermath of the Revolt of 1857.

1 and 2
2 and 3
1 and 3
All of the above

22. Consider the following statements about


Panchayati raj:
1. All the residents of the village are
members of Gram Sabha.
2. Panchayat is elected by the Gram
Sabha.
3. The Gram Panchayat is financially
accountable to the Gram Sabha.
Which of the above statements is/are true?

Which of these is/are true?


a)
b)
c)
d)

Which of the above statements is/are true?

a)
b)
c)
d)

1 only
2 only
Both 1 and 2
None

20. The manner in which the Cauvery river


water dispute between Tamilnadu and
Karnataka is being resolved shows the
a) Democratic nature of Indian state
b) Republican nature of Indian state
c) Parliamentary sovereignty in the
Indian state
d) Judicial sovereignty in the Indian
state

1 and 2
2 and 3
1 and 3
Only 2

23. Consider the following statements about


rural administration:
1. Each state has its own set of laws
regarding Panchayats.
2. The District or Zila Panchayat plays a
role in the sanctioning of money to
Gram Panchayats.
3. Panchayat secretary, who assists the
Panchayat, is chosen by the Sarpanch.
Which of the above statements is/are true?

21. Consider the following statements about


the region Ladakh:
1. It is a desert in mountains.
2. The famous pashmina shawls are
produced here.
3. Local versions of the Tibetan national
epic the Kesar Saga are performed and
sung by both Muslims and Buddhists
here.
http://insightsonindia.com

INSIGHTS

a)
b)
c)
d)

1 and 2
2 and 3
1 and 3
None of the above

24. In rural administration, which of the


following is/are usually the functions of
lekhpal/patwari/ kanungo?
1. Keeps record of the land in the village
Page 5

2. Keeps record of the crops grown in a


particular area
3. Collects land revenue from farmers.
Which of the above statements is/are true?
a)
b)
c)
d)

1 and 2
2 and 3
1 and 3
All of the above

25. In rural administration, which of the


following functions usually are performed
by the Tehsildar?
1. He supervises
patwari/kanungo/lekhpals work.
2. He hears land disputes.
3. He is the authority issuing land record
and caste certificates.
Which of the above statements is/are true?
a)
b)
c)
d)

1 and 2
2 and 3
1 and 3
All of the above

26. Landless laboureres in rural areas do not


get a constant stream of income
throughout the year. Why?
a) They migrate to urban areas.
b) Agriculture is seasonal in nature.
c) There do not have agricultural land.
d) No government policy/scheme
provides for social security measures
for landless labourers.

27. Which of the following problems is/are


associated with casual labourers in
India?
1. Low wages
2. Little or no social security
http://insightsonindia.com

INSIGHTS

3. Can be fired easily by the employer


Which of the above statements is/are true?
a)
b)
c)
d)

1 and 2
2 and 3
1 and 3
All of the above

28. Which of the following is/are true in


connection with the Mid-day meal
scheme?
1. It was first started in Tamilnadu.
2. It has adopted by other states after a
Supreme Court order in 2001.
3. It has helped decrease cases of
malnutrition in India.
Which of the above statements is/are true?
a)
b)
c)
d)

1 and 2
2 and 3
1 and 3
All of the above

29. In India, socio-economic inequality can be


across which of the following lines (as
suggested by several committees)?
1. Caste
2. Gender
3. Religion
4. Tribe
Choose the correct answer using the codes below:
a)
b)
c)
d)

1 and 2 only
All of the above
1, 2 and 3
2 and 4 only

30. The role of government in maintaining


public health is mandated/suggested in
which of the following?
Page 6

1. Constitution of India.
2. Government policies
3. Parliamentary enactments

2. State Human Rights Commission


3. State legislature
Choose the correct answer using the codes below:

Choose the correct answer using the code below:


a)
b)
c)
d)

1 and 2
2 and 3
1 and 3
All of the above

31. Consider the following health care centres:


1. Primary health care centre
2. Community health care centre
3. Sub-centres
Select the correct hierarchy based on the
population served by them in increasing order.
a)
b)
c)
d)

3,2,1
3,1,2
1,3,2
1,2,3

32. Consider the following statements:


Assertion (A): All states In India have the same
number of assembly constituencies.
Reason (R): Indian political system is federal in
nature.
In the context of the statements above, which of
these is correct?
a) Both A and R are correct and R is a
correct explanation for A.
b) Both A and R are correct and R is NOT
a correct explanation for A.
c) A is correct but R is incorrect.
d) A is incorrect but R is correct.

33. The term state government does NOT


refer to
1. The State High courts
http://insightsonindia.com

INSIGHTS

a)
b)
c)
d)

1 and 2
2 and 3
1 and 3
All of the above

34. Women related laws concerning which of


the following exist in India?
1. Domestic violence
2. Sexual harassment at workplace
3. Maternity benefits
Choose the correct answer using the codes below:
a)
b)
c)
d)

1 and 2
2 and 3
1 and 3
All of the above

35. Mass Media is considered as the fourth


pillar of democracy because of its
independence from the other three organs.
Which of the following may possibly
undermine the ability of media to be
unbiased, fair and represent pluralistic
points of view?
1. Ownership by large and important
corporate houses.
2. Strict censorship imposed by the
government
3. Separation of ownership and
editorial board of media houses
Choose the correct answer using the codes below:
a)
b)
c)
d)

1 and 2
2 and 3
1 and 3
All of the above

Page 7

36. A cooperative organization pools


resources from a group thus benefitting all
of its members. In India, which of the
following sectors have cooperatives?
1. Sugarcane
2. Milk
3. Banking
Choose the correct answer using the codes below:
a)
b)
c)
d)

1 and 2
2 and 3
1 and 3
All of the above

37. Which of these planets does not rotate and


revolve in the same direction?
a) Venus
b) Earth
c) Jupiter
d) Pluto

38. The bodies moving around the Sun and


found between the orbits of Mars and
Jupiter are
a) asteroids
b) meteorides
c) meteorites
d) comets

b) 2 only
c) Both 1 and 2
d) None

40. Which of the following would be the


consequences if the Earth stopped rotating
on its axis?
1. Some regions on earth would
experience darkness and extreme
cold for a very long period of time
during a year.
2. The duration of Sunlight would be
much longer than a day for some
regions of earth.
3. The Earth would eventually spiral
down close to the Sun and clash
with it after a long period of time.
Choose the correct answer using the codes
below:
a)
b)
c)
d)

1 and 2
2 and 3
1 and 3
All of the above

41. Consider the following statements:


Assertion (A): The level of sea remains almost the
same everywhere.
Reason (R): All the oceans of the world are
interconnected with each other.

39. Consider the following statements:


1. The mid-day sun is exactly
overhead at least once a year on all
latitudes.
2. The distance between successive
latitudes is the same whereas that
between successive longitudes (at
the same latitude) is not.
Which of these is/are true?

In the context of the statements above, which of


these is correct?
a) Both A and R are correct and R is a
correct explanation for A.
b) Both A and R are correct and R is NOT
a correct explanation for A.
c) A is correct but R is incorrect.
d) A is incorrect but R is correct.

a) 1 only
http://insightsonindia.com

INSIGHTS

Page 8

42. Which of the following are found in the


South American continent?
1. Andes mountain ranges
2. Great Victoria desert
3. Atakama deserts
Choose the correct answer using the codes below:
a)
b)
c)
d)

1 and 2
2 and 3
1 and 3
All of the above

45. Arrange these landforms found in India


from North to South.
1. Aravali range
2. Vindhya range
3. Satpura range
4. Godavari river
5. Krishna river
Choose the correct order from the codes given
below:

43. Consider the following statements:


Assertion (A): The coastlines of Atlantic ocean
provides ideal location for natural harbours and
ports.
Reason (R): The coastlines of Atlantic ocean are
highly indented and irregular.
In the context of the statements above, which of
these is correct?
a) Both A and R are correct and R is a
correct explanation for A.
b) Both A and R are correct and R is NOT
a correct explanation for A.
c) A is correct but R is incorrect.
d) A is incorrect but R is correct.

44. Which of the following are examples of


FOLD mountains?
1. Himalayas
2. Alps - Europe
3. Appalachians North America
4. Ural - Russia
5. Aravali - India
Choose the correct answer using the codes below:
a) 2, 3 and 4
b) 1 and 5
c) All of the above
http://insightsonindia.com

d) 1, 4 and 5

INSIGHTS

a) 1,2,3,4,5
b) 2,1,3,4,5
c) 1,3,2,5,4
d) 1,2,3,5,4

46. Consider the following statements:


1. No river passing through the Vindhya
and Satpura flows into the Arabian
Sea.
2. Eastern Ghats are broken and uneven
unlike the Western Ghats.
3. Eastern coastal plains are much
broader than that of the Western side.
Which of these is/are true?
a)
b)
c)
d)

1 and 2
2 and 3
1 and 3
All of the above

47. Coastal places like Kolkata and Mumbai


experience a humid and moderate
weather. Why?
a) Due to their continentality
b) Due to the flowing of sea and land
breezes
c) Due to their low elevation as
compared to nearby regions

Page 9

d) Due to the less frequency of eastern


cyclonic disturbances.

48. In India, tropical evergreen forests can be


found in
1. Andaman and Nicobar Islands
2. Sunderbans
3. Laskhadweep
4. Western slope of Western Ghats
5. In the north-Eastern states
Choose the correct answer using the codes below:
a)
b)
c)
d)

1, 2, 3 and 4
2 and 5 only
1, 4 and 5 only
4 and 5 only

1 and 2
2 and 3
1 and 3
Only 1

51. Sedimentary rocks can be directly formed


from which of the following?
1. Magma
2. Igneous rocks
3. Metamorphic rocks

a)
b)
c)
d)

Choose the correct answer using the codes below:


1 and 2 only
1, 4 and 5 only
3, 4 and 5 only
Only 1

50. Consider the following statements:


1. Continental crust is thicker than ocean
crust.
2. Continental crust is denser than ocean
crust.
3. The main constituents of continental
crust are Silica and Magnesium.
http://insightsonindia.com

a)
b)
c)
d)

Choose the correct answer using the codes below:

49. Which of the following places in India


houses both Mangroves and Tropical
Evergreen forests?
1. Andaman and Nicobar Islands
2. Sunderbans
3. Bhitarkanika
4. Pichavaram
5. Gulf of Katch

a)
b)
c)
d)

Which of these is/are true?

INSIGHTS

1 and 2
2 and 3
1 and 3
Only 2

52. Which of the following is/are responsible


for the formation of ox-bow lakes?
1. Meandering tendency of the river
2. Sediment load of the river
3. Potential of the soil/rock to get eroded
Choose the correct answer using the codes below:
a)
b)
c)
d)

1 and 2
2 and 3
1 and 3
All of the above

53. Arrange the following gases in an


decreasing order of their abundance in the
atmosphere.
1. Argon
2. Oxygen
3. Carbon dioxide
4. Hydrogen
Choose the correct answer using the codes below:
Page 10

a)
b)
c)
d)

2,4,3,1
2,3,4,1
2,3,1,4
2,1,3,4

Which of these is/are true?


a)
b)
c)
d)

54. What can be the consequences of global


warming?
1. Flooding of coastal areas
2. Conditions of drought and flood may
aggravate in certain regions of the
world.
3. Extinction of some species of plants
and animals
4. Albedo of the earth will increase
Choose the correct answer using the codes below:
a)
b)
c)
d)

55. Which of the following gases are taken


directly by the plants for making food?
1. Oxygen
2. Nitrogen
3. Carbon dioxide
4. Hydrogen
Choose the correct answer using the codes below:
a)
b)
c)
d)

Only 3
3 and 4
All of the above
2 and 3 only

Choose the correct answer using the codes below:


Only 1
2 and 3
1, 2 and 3 only
1 and 2 only

58. Consider the following statements:


1. As air gets warmer, its capacity to hold
moisture increases, if all other things
are constant.
2. The white trail left by jets is due to the
condensation of moisture from their
engines.
3. Cyclonic rainfall is caused in the
meeting of warm and cold air.
Which of these statements is/are true?

56. Consider the following statements:


1. All the weather phenomena like rain,
hailstorm, fog etc. happen in the
troposphere.
2. Stratosphere is free of clouds.
3. Temperature increases sharply with
increasing height in thermosphere.
http://insightsonindia.com

57. Which of the following are permanent


planetary winds?
1. Westerlies
2. Trade winds
3. Monsoon winds

a)
b)
c)
d)

1 and 2 only
All of the above
1, 2 and 3 only
3 and 4 only

1 only
1 and 2 only
All of the above
2 only

INSIGHTS

a)
b)
c)
d)

1 and 2
2 and 3
1 and 3
All of the above

59. A huge wave called Tsunami can be


result of which of the following under the
sea?
1. Volcanic eruptions
2. Earthquake
Page 11

3. Underwater landslides
Choose the correct answer using the codes below:
a)
b)
c)
d)

1 and 2
2 and 3
1 and 3
All of the above

63. These are found in the large parts of, inter


alia, Northern Australia and in Central
America. These regions experience
seasonal changes. Trees shed their leaves
in the dry season to conserve water. The
hardwood trees found in these forests are
sal, teak, neem and shisham. Hardwood.
Tigers, lions, elephants, langoors and
monkeys are the common animals of these
regions.

60. Tides in the ocean are highest in


a) Full moon days
b) New moon days
c) First quarter of moon
d) Both (a) and (b)
61. How can high tides be useful?
1. They help in navigation in the
harbours and ports.
2. They enable fishermen to get a
plentiful catch of fishes.
3. They help in marine upwelling due to
which the surface of the ocean water
become nutrient rich.
Choose the correct answer using the codes below:
a)
b)
c)
d)

1 and 2
2 and 3
1 and 3
All of the above

62. Consider the following statements about


warm and cold currents:
1. They affect the temperature conditions
of the region around which they pass.
2. Worlds best fishing grounds are
found where warm and cold currents
meet.
3. The areas where a warm and cold
current meet also experience foggy
weather making it difficult for
navigation.
Choose the correct answer using the codes below:
a) 1 and 2
http://insightsonindia.com

b) 2 and 3
c) 1 and 3
d) All of the above

INSIGHTS

Which type of vegetation suit the above


description?
a)
b)
c)
d)

Tropical deciduous forests


Temperate deciduous forests
Boreal forests
Mediterranean vegetation

64. Which of the following correctly


highlights the important differences
between Savanna and Steppes grasslands?
1. Steppes receive rainfall throughout the
year unlike Savanna.
2. Savannas lie nearer the equator and
are warmer than steppes.
3. Large plants and trees can grow more
easily in Savanna as compared to
Steppes.
Choose the correct answer using the codes below:
a)
b)
c)
d)

1 and 2
2 and 3
1 and 3
All of the above

65. Mosses and Lichens can be found in


which of the following?
a) Tundra vegetation
b) Tropical evergreen forests
c) Desertic vegetation
Page 12

d) Temperate evergreen forests

d) Ellora caves
70. Consider the following statements

66. Which of the following can be used as


inland waterways in India?
1. Navigable rivers
2. Navigable lakes
3. Navigable canals
Choose the correct answer using the codes below:
a)
b)
c)
d)

1. A great Tamil epic, Silappadikaram, is


older than another Tamil epic
Manimekalai
2. Silappadikaram was composed by
Sattanar
Which of the above statements is/are

1 and 2
2 and 3
1 and 3
All of the above

INCORRECT?
a) 1 Only
b) 2 Only
c) Both

67. Poduca, a port in South India, was a


Roman name. What is its modern name?

71. With reference to the National Cyber


Security and Coordination Centre (NCSC),

a) Chennai

a proposed body to be set up by the

b) Kaveripattanam

Ministry of IT and Communications,

c) Tuticorin

India, consider the following statements

d) Arikamedu
68. The grama bhojaka in ancient villages of
Northern India was a

1. It will analyse Internet traffic data


scanned and integrated from various
gateway routers at a centralised
location

a) Village Watchman

2. It will facilitate real-time assessment of

b) Village Headman

cyber-security threats and generate

c) Village priest

actionable reports for various agencies

d) Village treasurer
69. According to the Puranas Vishnu took the
shape of a boar in order to rescue the
earth, which had sunk into water. A
magnificent statue of a special form of
Vishnu, the Varaha or boar is located at
which of the following place?

Which of the above statements is/are


INCORRECT?
a) 1 Only
b) 2 Only
c) Both
d) None

a) Mount Abu

72. Water Puppetry, a famous theatre dance

b) Ajanta caves

form of South East Asia, originally

c) Eran (Madhya Pradesh)


http://insightsonindia.com

d) None

INSIGHTS

Page 13

belongs to and is widely performed in


which of the following country?

b) Countrys first satellite designed to be


sent to moon during next moon
mission

a) Laos

c) Countrys first mobile Antarctic

b) Malaysia

observatory

c) Vietnam

d) None of the above

d) Myanmar
73. The Space Exploration Technologies Corp
(SpaceX) which launches spacecrafts to
International Space Station, is a/an
a) Privately owned company

76. Spinosaurus aegyptiacus, whose skeletons


were recently discovered in Morocco, has
some special features. Consider the
following statements:
1. It is a species of dinosaurs

b) Subsidiary of NASA

2. It is the only known aquatic dinosaur

c) Autonomous agency under USA


government

Which of the above statements is/are correct?

d) None of the above


74. With reference to the Trans Pacific

a) 1 Only

Partnership (TPP), consider the following

b) 2 Only

statements

c) Both

1. It is an on-going operational free trade

d) None

agreement between USA and other


countried from Asia and Pacific region
2. India has participated in all TPPs
negotiations and is set to become its
member in 2015

77. With reference to Jaipur Foot, consider the


following statements
1. It is a leather-based prosthetic leg for
people with below-knee amputations
2. These are produced by the

Which of the above statements is/are

Government of Rajasthan and

INCORRECT?

distributed all around the world at a

a) 1 Only

subsidized price

b) 2 Only

Which of the above statements is/are correct?

c) Both

a) 1 Only

d) None

b) 2 Only

75. IndARC, developed by Indias Earth


System Science Organisation (ESSO) and
other research organizations, is

c) Both
d) None

a) Countrys first underwater moored


78. With reference to Uranium production,

observatory

consider the following statements


http://insightsonindia.com

INSIGHTS

Page 14

1. Australia has worlds largest Uranium


reserves
2. India imports its most of Uranium
from Russia, Kazakhstan and France
Which of the above statements is/are correct?
a) 1 Only
b) 2 Only

2. Ox-bow lakes
3. Mountains
4. Deltas
Choose the correct answer using the codes below:
a)
b)
c)
d)

1, 2 and 3
2, 3 and 4
1, 3 and 4
2 and 4 only

c) Both
d) None
79. With reference to Swachh Vidyalaya
scheme, consider the following statements
1. A Swachh Bharat Kosh will be
established to fund this scheme
2. Its mission is to make India a clean
country by 2019

Choose the correct answer using the codes below:

Which of the above statements is/are correct?


a) 1 Only
b) 2 Only
c) Both
d) None
80. The Global Ecosystem Dynamics
Investigation (GEDI) Lidar device
designed by NASA is used for
a) Mapping earth forests in 3D
b) Mapping density of ocean life
c) Mapping density of species in
rainforests

a)
b)
c)
d)

81. Which of the following features you are


likely to come across if you trace the
Ganga river from its origin to its
destination (sea)?
1. Plateaus
INSIGHTS

All of the above


1, 2 and 3 only
1 and 4 only
3, 4 and 5 only

83. Given below are some associations of


diseases with the deficiency that causes
them:
1. Copper Anemia
2. Vitamin B1 BeriBeri
3. Vitamin C Scurvy
Choose the code that represents the correct
associations.
a)
b)
c)
d)

d) None of the above

http://insightsonindia.com

82. Which of the following crops can be


grown in the Ganga Basin?
1. Maize
2. Wheat
3. Sugarcane
4. Millets
5. Gram

All of the above


1 and 2
2 and 3
1 and 3

84. Consider the following statements:


1. Algae prepare food by the process of
photosynthesis.
Page 15

2. Plants can synthesize not only


carbohydrates but also proteins and
fats.
3. All plants are autotrophs.
Which of these is/are true?
a)
b)
c)
d)

1 and 2 only
2 and 3 only
1 and 3 only
All of the above

85. Consuming sweet foods in excess can


cause tooth decay. The bacteria present in
the mouth convert the leftover sweet food
in mouth to X which cause the
degradation of teeth. What is X?
a) Acid
b) Base
c) Salt
d) None of the above

86. Oral Rehydration solution (ORS) can be


useful in which of the following diseases?
1. Diarrhoea
2. Cholera
3. Chickengunya
Choose the correct answer using the codes below:
a)
b)
c)
d)

1 and 2
2 and 3
1 and 3
All of the above

causes a fatal blood disease called sorters


disease? What is Y?
a) Anthrax
b) Clostridium
c) Salmonella typhi
d) Yersinia pestis

88. Constructing outer walls of buildings so


that they have trapped layers of air serves
which of the following purpose?
a) the temperature inside the building is
less affected by the temperature
outside
b) the temperature inside the building is
more affected by the temperature
outside
c) the building can tolerate earthquakes
more effectively
d) the building can take more load as
compared to a normal building

89. The most commonly used natural


indicator is litmus. It is extracted from?
a) Lichens
b) Mosses
c) Blue-green Algae
d) Red algae

90. Acid rain can contain which of the


following?
1. Carbonic acid
2. Sulphuric acid
3. Nitric acid
Choose the correct answer using the codes below:

87. Wool industry is an important means of


livelihood for many people in our country.
But sorters job is risky as sometimes they
get infected by a bacterium, Y, which

http://insightsonindia.com

INSIGHTS

a)
b)
c)
d)

1 and 2
2 and 3
1 and 3
All of the above

Page 16

91. When an ant bites, it injects a liquid P


into the skin. The effect of the sting can be
neutralised by rubbing Q. What are P
and Q respectively?
a) Formic Acid and moist baking soda
b) Moist baking soda and Formic Acid
c) Formic Acid and Vinegar
d) Moist Baking soda and vinegar

92. Some migratory birds travel thousands of


kilometres to escape the extreme climatic
conditions at home. Then they return back
to the same place where they started from.
Which of the following is a more complete
set of aids that may possibly guide them
in migration and return journey?
a) Sun, Moon and stars
b) Sun, stars and Magnetic field
c) Sun, Moon and Magnetic field
d) Stars, Moon and Magnetic field

93. Wheat, gram and paddy can be grown in


which kind of soil?
a) Sandy loam
b) Clay
c) Loam
d) Both (b) and (c)

94. Consider the following statements about


yeast:
1. They are single-celled organisms.
2. They respire anaerobically.
3. They are used in the production of
alcohol.

1 and 2
2 and 3
1 and 3
All of the above

http://insightsonindia.com

96. Consider the following statements about


plants produced by vegetative
propagation:
1. They take less time to grow and bear
flowers and fruits earlier than those
produced from seeds.
2. The new plants are exact copies of the
parent plant, as they are produced
from a single parent.
3. Cutting, budding and grafting come
under vegetative propagation
techniques.
Choose the correct answer using the codes below:
a)
b)
c)
d)

1 and 2
2 and 3
1 and 3
All of the above

97. Rear view mirrors used in automobiles are


a) Convex mirror
b) Concave mirror
c) Plane mirror
d) A combination of concave and
convex mirrors

Which of these is/are true?


a)
b)
c)
d)

95. Plants take water from soil in excess of


what they need and then release it by
transpiration. What purpose is served by
it?
a) Doing this generates a suction pull
which helps transport water to
greater heights in plants.
b) Doing this generates suction force
which keeps off excess water going
back to roots
c) Doing this keeps the leafs healthy
and be more efficient in
photosynthesis
d) Doing this keeps the phloem in the
stems healthier

INSIGHTS

Page 17

98. Excess of which of the following will


pollute water?
1. Nickel
2. Selenium
3. Cadmium
Choose the correct answer using the codes below:
a)
b)
c)
d)

1 and 2
2 and 3
1 and 3
All of the above

http://insightsonindia.com

99. It has been suggested that we should plant


X trees all along sewage ponds. These
trees absorb all surplus wastewater
rapidly and release pure water vapour
into the atmosphere. What can be X?
a) Eucalyptus
b) Teak
c) Mahogany
d) Rosewood

100.
In the eye donation, which part of
the eye is transplanted from the donor ?
(a) Cornea
(b) Lens
(c) Retina
(d) The whole eye

INSIGHTS

Page 18

INSIGHTS ON INDIA TEST SERIES - 2015


INSIGHTS ON INDIA MOCK PRELIMINARY EXAM 2015
INSIGHTS ON INDIA MOCK TEST - 1
GENERAL STUDIES
PAPER-I
SOLUTIONS
http://insightsonindia.com
INSIGHTS ON INDIA MOCK TEST SERIES FOR CIVIL SERVICES PRELIMINARY EXAM

1. Solution: b)
In the world due to technical and user base reasons the mobile numbers at present varied
from 10 to 11 digits. Due to technical reasons two countries, UK and China moved to 11
digits in mobile phone numbers. In India all mobile numbers have 10 digits under the
governments National Numbering Plan (NNP).
The number of digits in a mobile phone number describe the maximum mobile phones we
can have without dialling the country code, that is 91 (for India). If we had a 9 digit cell
number, the maximum number of cell numbers possible would have been 109, ie. a
maximum of 1000 million or 100 crore subscribers. Since our population is close to 125 crore,
obviously we cannot have a 9- digit cell phone number. Adopting a 10 digits cell number
offers a capacity to have 10 billion or 1000 crore subscribers and the total population would
get at least one mobile number each.
In 2003, the Department of Telecommunication ( DoT) In India had implemented the 10
digit mobile numbers which will meet the needs for 30 long years. On February 2009, India
registered a user base of 375.74 million wireless subscribers. 10-digit numbers can cater to an
additional 250 million users. Under the current 10-digit numbering scheme, only a
maximum of 1 billion mobile numbers can be issued and the mobile phone connections in
India would cross this mark in the next couple of years. DoT had previously proposed a
transfer of numbers from10 -digits to 11- digits in January 2010.
At present all mobile phone numbers are 10 digits long and the way to split the numbers is
defined in the National Numbering Plan 2003 as XXXX NNNNNN where XXXX is the
Network operator, NNNNNN is the subscriber numbers.

http://www.thehindu.com/sci-tech/question-corner/article6398015.ece

http://insightsonindia.com

INSIGHTS

Page 1

INSIGHTS ON INDIA TEST SERIES - 2015


2. Solution: d)
The Garo Hills) are part of the Garo-Khasi range in Meghalaya, India. They are inhabited
mainly by tribal dwellers, the majority of whom are Garo people. It is one of the wettest
places in the world. The chief meals of the Garos consist of rice with onions, capsicum and
salt thrice a day.
The society is matrilineal like the Khasis and the Jaintias. Till death, the new-born baby
belongs to the mother's family, irrespective of sex, even after marriage. Marriage within the
clan is completely prohibited and severely punishable for both the Khasis and the Garos. Till
now, the institution of Bachelors Dormitories which is gradually disappearing amongst the
tribes of North Eastern Region, are found in the Garo villages. In such dormitories young
people stay and live together till they are married. They receive various training in the
dormitories like protection of crops, construction of roads, organising festivals, sports and
ceremonies. This institution is similar to ancient agoges of Sparta.
3. Solution: d)
The Iranians and the Greeks who came through the northwest about 2500 years ago and
were familiar with the Indus, called it the Hindos or the Indos, and the land to the east of the
river was called India. The name Bharata was used for a group of people who lived in the
north-west, and who are mentioned in the Rigveda, the earliest composition in Sanskrit
(dated to about 3500 years ago). Later it was used for the country.

4. Solution: b)
Asoka's edicts are to be found scattered in more than thirty places throughout India, Nepal,
Pakistan and Afghanistan. Most of them are written in Brahmi script from which all Indian
scripts and many of those used in Southeast Asia later developed. The language used in the
edicts found in the eastern part of the sub-continent is a type of Magadhi, probably the
official language of Asoka's court. The language used in the edicts found in the western part
of India is closer to Sanskrit although one bilingual edict in Afghanistan is written in
Aramaic and Greek. Asoka's edicts, which comprise the earliest decipherable corpus of
written documents from India, have survived throughout the centuries because they are
written on rocks and stone pillars.

5. Solution: d)
There are at least four reasons why hunter-gatherers moved from place to place.
First, if they had stayed at one place for a long time, they would have eaten up all the
available plant and animal resources. Therefore, they would have had to go elsewhere in
search of food.

http://insightsonindia.com

INSIGHTS

Page 2

INSIGHTS ON INDIA TEST SERIES - 2015


Second, animals move from place to place either in search of smaller prey, or, in the case of
deer and wild cattle, in search of grass and leaves. That is why those who hunted them had
to follow their movements.
Third, plants and trees bear fruit in different seasons. So, people may have moved from
season to season in search of different kinds of plants.
Fourth, people, plants and animals need water to survive. Water is found in lakes, streams
and rivers. While many rivers and lakes are perennial (with water throughout the year)
others are seasonal. People living on their banks would have had to go in search of water
during the dry seasons (winter and summer). Besides, people may have travelled to meet
their friends and relatives.

6. Solution: a)
Neolithic archaeological site in Uttar Pradesh state of northern India, dated between 40001200 BC. Koldihwa was an agricultural village of circular uts, with stone axes, bone and
stone tools, pottery, and cattle pens; and early evidence of rice cultivation, found as
impressions in ceramic vessels.

7. Solution: b)
Catal Huyuk was a very large Neolithic and Chalcolithic proto-city settlement in
southern Anatolia, which existed from approximately 7500 BC to 5700 BC, and flourished
around 7000BC.[1] It is the largest and best-preserved Neolithic site found to date. In July
2012, it was inscribed as a UNESCO World Heritage Site.
atalhyk is located overlooking the Konya Plain, southeast of the present-day city
of Konya (ancient Iconium) in Turkey, approximately 140 km (87 mi) from the twin-coned
volcano of Mount Hasan. The eastern settlement forms a mound which would have risen
about 20 m (66 ft) above the plain at the time of the latest Neolithic occupation. There is also
a smaller settlement mound to the west and a Byzantine settlement a few hundred meters to
the east. The prehistoric mound settlements were abandoned before the Bronze Age. A
channel of the aramba river once flowed between the two mounds, and the settlement
was built on alluvial clay which may have been favourable for early agriculture.
http://www.catalhoyuk.com/history.html

8. Solution: a)
Many of these cities were divided into two or more parts. Usually, the part to the west was
smaller but higher. Archaeologists describe this as the citadel. Generally, the part to the east
was larger but lower. This is called the lower town. Very often walls of baked brick were
http://insightsonindia.com

INSIGHTS

Page 3

INSIGHTS ON INDIA TEST SERIES - 2015


built around each part. The bricks were so well made that they have lasted for thousands of
years. The bricks were laid in an interlocking pattern and that made the walls strong.
In some cities, special buildings were constructed on the citadel. For example, in
Mohenjodaro, a very special tank, which archaeologists call the Great Bath, was built in this
area. This was lined with bricks, coated with plaster, and made water-tight with a layer of
natural tar. There were steps leading down to it from two sides, while there were rooms on
all sides. Water was probably brought in from a well, and drained out after use. Perhaps
important people took a dip in this tank on special occasions.

9. Solution: c)
Most of the things that have been found by archaeologists in Harappa are made of stone,
shell and metal, including copper, bronze, gold and silver. Copper and bronze were used to
make tools, weapons, ornaments and vessels. Gold and silver were used to make ornaments
and vessels. Perhaps the most striking finds are those of beads, weights, and blades.

10. Solution: c)
Ghod River is located in Pune District, Maharashtra, western India.[1] It is a tributary of
the Bhima River.The Ghod originates on the eastern slopes of the Western Ghats at 1,090
metres (3,580 ft) above sea level. It flows in an east-southeast direction for approximately 200
kilometres (120 mi) before its confluence with the Bhima. It flows from the northern side of
the Bhimashankar hills.
The Kukadi River is one of the tributaries of the Ghod. Inamgaon, a post-Harappan agrarian
village located along the Ghod, has been studied for its archaeological finds.The river is
dammed by the Ghod Dam.

11. Solution: a)
Choosing leaders or rulers by voting is something that has become common during the last
fifty years or so. But, around 3000 years ago, we find some changes taking place in the ways
in which rajas were chosen. Some men now became recognised as rajas by performing very
big sacrifices. The ashvamedha or horse sacrifice was one such ritual. A horse was let loose
to wander freely and it was guarded by the rajas men. If the horse wandered into the
kingdoms of other rajas and they stopped it, they had to fight. If they allowed the horse to
pass, it meant that they accepted that the raja who wanted to perform the sacrifice was
stronger than them. These rajas were then invited to the sacrifice, which was performed by
specially trained priests, who were rewarded with gifts. The raja who organised the sacrifice
was recognised as being very powerful, and all those who came brought gifts for him.
http://insightsonindia.com

INSIGHTS

Page 4

INSIGHTS ON INDIA TEST SERIES - 2015


12. Solution: c)
There were two major changes in agriculture around Mahajanapada period. One was the
growing use of iron ploughshares. This meant that heavy, clayey soil could be turned over
better than with a wooden ploughshare, so that more grain could be produced. Second,
people began transplanting paddy. This meant that instead of scattering seed on the ground,
from which plants would sprout, saplings were grown and then planted in the fields. This
led to increased production, as many more plants survived. However, it was back breaking
work. Generally, slave men and women, (dasas and dasis) and landless agricultural
labourers (kammakaras) had to do this work.
13. Solution: b)
Siddhartha, also known as Gautama, the founder of Buddhism, was born about 2500 years
ago. This was a time of rapid change in the lives of people. As you saw in Chapter 6, some
kings in the mahajanapadas were growing more powerful. New cities were developing, and
life was changing in the villages as well .Many thinkers were trying to understand these
changes in society. They also wanted to try and find out the true meaning of life.
The Buddha belonged to a small gana known as the Sakya gana, and was a kshatriya. When
he was a young man, he left the comforts of his home in search of knowledge. He wandered
for several years, meeting and holding discussions with other thinkers. He finally decided to
find his own path to realisation, and meditated for days on end under a peepal tree at Bodh
Gaya in Bihar, where he attained enlightenment. After that, he was known as the Buddha or
the Wise One. He then went to Sarnath, near Varanasi, where he taught for the first time. He
spent the rest of his life travelling on foot, going from place to place, teaching people, till he
passed away at Kusinara.
Regarding Gana and Sangha
While Magadha became a powerful kingdom, Vajji, with its capital at Vaishali (Bihar), was
under a different form of government, known as gana o sangha.
In a gana or a sangha there were not one, but many rulers. Sometimes, even when thousands
of men ruled together, each one was known as a raja. These rajas performed rituals together.
They also met in assemblies, and decided what had to be done and how, through discussion
and debate. For example, if they were attacked by an enemy, they met to discuss what
should be done to meet the threat. However, women, dasas and kammakaras could not
participate in these assemblies.
Both the Buddha and Mahavira belonged to ganas or sanghas. Some of the most vivid
descriptions of life in the sanghas can be found in Buddhist books.
http://insightsonindia.com

INSIGHTS

Page 5

INSIGHTS ON INDIA TEST SERIES - 2015


Mahavira belonged to Vajji sangha.
14. Solution: a)
Around the time that the Buddha was preaching and perhaps a little earlier, other thinkers
also tried to find answers to difficult questions. Some of them wanted to know about life
after death, others wanted to know why sacrifices should be performed. Many of these
thinkers felt that there was something permanent in the universe that would last even after
death. They described this as the atman or the individual soul and the brahman or the
universal soul. They believed that ultimately, both the atman and the brahman were one.
Many of their ideas were recorded in the Upanishads. These were part of the later Vedic
texts. Upanishad literally means approaching and sitting near and the texts contain
conversations between teachers and students. Often, ideas were presented through simple
dialogues.
15. Solution: c)
The most famous thinker of the Jainas, Vardhamana Mahavira, also spread his message
around this time, i.e. 2500 years ago. He was a kshatriya prince of the Lichchhavis, a group
that was part of the Vajji sangha, about which you read in Chapter 6. At the age of thirty, he
left home and went to live in a forest. For twelve years he led a hard and lonely life, at the
end of which he attained enlightenment.
He taught a simple doctrine: men and women who wished to know the truth must leave
their homes. They must follow very strictly the rules of ahimsa, which means not hurting or
killing living beings. All beings, said Mahavira long to live. To all things life is dear.
Ordinary people could understand the teachings of Mahavira and his followers, because
they used Prakrit. There were several forms of Prakrit, used in different parts of the country,
and named after the regions in which they were used. For example, the Prakrit spoken in
Magadha was known as Magadhi.
16. Solution: b)
Zoroaster was an Iranian prophet. His teachings are contained in a book called the Avesta.
The language of the Avesta, and the practices described in it are very similar to those of the
Vedas. The basic teachings of Zoroaster are contained in the maxim Good thoughts, Good
Words and Good Deeds. Here is a verse from the Zend Avesta:
Lord, grant strength and the rule of truth and good thinking, by means of which one shall
create peace and tranquillity.

http://insightsonindia.com

INSIGHTS

Page 6

INSIGHTS ON INDIA TEST SERIES - 2015


For more than a thousand years, Zoroastrianism was a major religion in Iran. Later, some
Zoroastrians migrated from Iran and settled down in the coastal towns of Gujarat and
Maharashtra. They were the ancestors of todays Parsis.
Today, Shia Islam dominates religion in Iran.
17. Solution: d)
Accountability of the government is basically answerability for its deeds. If the activities of
the government can be communicated and questioned by the people, it shows that there is a
sense of answerability. Finally it is upto the people to decide the fate of the government by
voting.
All these are ways in which the government can be made accountable. You can also see
several rallies, marches and protests that keep the government of the day at its toe.
18. Solution: d)
Autocracy and Monarchy are two different political systems. An autocrat is an absolute
dictator for e.g. in Syria.
A democracy is termed as monarchy when the head of state is chosen on the hereditary
basis. Republic form of democracy is when the head of state is either directly or indirectly
elected.
19. Solution: d)
Women suffrage movement dates back to the First world war days (not the 1857 revolt). The
Women's Indian Association (WIA) was founded in 1917. It sought votes for women and the
right to hold legislative office on the same basis as men. These positions were endorsed by
the main political groupings, the Indian National Congress and the All-India Muslim
League. British and Indian feminists combined in 1918 to publish a magazine Stri
Dharma that featured international news from a feminist perspective. In 1919 in
the MontaguChelmsford Reforms, the British set up provincial legislatures which had the
power to grant women's suffrage. Madras in 1921 granted votes to wealthy and educated
women, under the same terms that applied to men. The other provinces followed, but not
the princely states (which did not have votes for men either).
20. Solution: a)
There is no term like Judicial sovereignty. In India no organ is supreme. There is separation
of powers between different organs. So even the Parliament is not supreme or sovereign. Its
authority is restricted by Fundamental rights and Judicial oversight.
21. Solution: d)
Ladakh is a desert in the mountains in the eastern part of Jammu and Kashmir. Very little
agriculture is possible here since this region does not receive any rain and is covered in
http://insightsonindia.com

INSIGHTS

Page 7

INSIGHTS ON INDIA TEST SERIES - 2015


snow for a large part of the year. There are very few trees that can grow in the region. For
drinking water, people depend on the melting snow during the summer months. So, people
here keep sheep and the sheep in this region are special because they produce pashmina
wool. This wool is prized and pashmina shawls cost a lot of money. The people in Ladakh
carefully collect the wool of the sheep and sell this to traders from Kashmir. Pashmina
shawls are chiefly woven in Kashmir.
Ladakh is also called Little Tibet. Islam was introduced in this region more than four
hundred years ago and there is a significant Muslim population here. Ladakh has a very rich
oral tradition of songs and poems. Local versions of the Tibetan national epic the Kesar Saga
are performed and sung by both Muslims and Buddhists.
22. Solution: b)
Any one who has the right to vote and lives in the premises if a Panchayat is a member of
the Gram Sabha. Panchayat is elected by it. The Sarpanch is usually elected by the Panchayat
it depends from state to state.
The Gram Sabha keeps a track of the work of Gram Panchayat and prevents misuse of
money. Some of the works by Gram Panchayat has to be approved by the Gram Sabha.

23. Solution: a)
The constitution via the 73rd amendment provides the skeleton for rural government. The
rest is decided by the states by laws enacted in the legislature.
The District Panchayat is the at the top of the hierarchy in rural government. It prepares
plan for the whole district with the help of Panchayat samitits and municipalities. In this
way it regulates the sanctioning of money to the gram Panchayats.
Panchayat secretary is a bureaucrat and selected by the Civil Services examination of the
concerned state. The state government appoints the Panchayat secretary.
24. Solution: d)
He provides all this information to the government thus keeping it up to date. He is
supervised by senior bureaucrats in the revenue department. He is also under the control of
District collector.
25. Solution: a)
The District collector or the BDO issues such important certificates. The only duty of the
Tehsildar is to make sure that these certificates are disbursed to the needy properly.
26. Solution: b)

http://insightsonindia.com

INSIGHTS

Page 8

INSIGHTS ON INDIA TEST SERIES - 2015


Option A is a consequence of seasonal income, not a reason. Not having agricultural land is
evident from them being called landless in the first place. That is not the cause of seasonal
income.
Policies for landless labourers do exist. But they do not have to do with their income except
schemes like MGNREGA.
The correct reason is that agriculture is seasonal in nature. So they do no get employment
throughout the year.
27. Solution: d)
The fact that unemployment and low skill labour are found easily in India leads to the easy
availability of casual labour.
The law does not regulate the hiring and firing of the casual labourers as they are part of the
informal economy.

28. Solution: d)
The Mid Day Meal Scheme is a programme of the Government of India designed to improve
the nutritional status of school-age children nation wide. The programme supplies free
lunches on working days for children in Primary and Upper Primary Classes in
Government, Government Aided, Local Body, Education Guarantee Scheme, and Alternate
Innovative Education Centres, Madarsa and Maqtabs supported under Sarva Shiksha
Abhiyan, and National Child Labour Project schools run by the Ministry of Labour. Serving
120,000,000 children in over 1,265,000 schools and Education Guarantee Scheme centres, it is
the largest such programme in the world.
It has also helped fight caste prejudice in rural areas as all students eat together. It has
generated kitchen employment for women and farmers.

29. Solution: c)
While tribals are socio-economically weaker than mainstream community, there is no
specific evidence that this inequality can also be seen across several tribal groups in India.
There are strong evidences for the other options like Sachar committee for Muslims
(religion).

30. Solution: d)

http://insightsonindia.com

INSIGHTS

Page 9

INSIGHTS ON INDIA TEST SERIES - 2015


The constitution of India prescribes in Directive principles of State policies that it shall be the
states endeavour to raise the level of nutrition and standard of living and to improve public
health.
The National health policy 2002 prescribes the role of government in public health.
Several laws related to health in India are:

The Medical Termination of Pregnancy Act and Rules


The Pre-Natal Diagnostic Techniques (PNDT) Act and Rules
Acts in Disability
Insecticides Act and Rules
Maternity Benefit Act and Rules
Narcotic Drugs and Psychotropic Substances Act and Rules
The Prevention of Food Adulteration Act, 1954
Drugs and Cosmetics Act, 1940
Bio-Medical Waste (Management and Handling) Rules, 1998
The Pharmacy Act, 1948
The Transplantation of Human Organs Act and Rules
Environmental Acts and Rules
Consumer Protection Act and Medical Profession
Mental Health Act, 1987
Food Safety and Standards Regulations
The Protection of Women From Domestic Violence Act, 2005
The Marriage Laws (Amendment) Bill, 2010
The Prohibition Of Sexual Harassment Of Women At Workplace Bill, 2010
Food Safety and Standards (Prohibition and Restrictions on Sales) Regulations, 2011
Food Safety and Standards (Contaminants, Toxins and Residues) Regulations, 2011

31. Solution: b)
Sub-centre is the first point of contact between the community an public health system. For
every 3000-4000 of population there is a sub-centre. For every 4-5 sub-centres, there is a
primary health centre. And for every 4-5 primary health centres, there is a community health
centre.

32. Solution: d)
All states do not have the same number of constituencies. India being a federal nation has
nothing to with the number of assembly seats in a state. The federal character of india has
more to do with the representation of states in the Rajya Sabha.

33. Solution: d)
http://insightsonindia.com

INSIGHTS

Page 10

INSIGHTS ON INDIA TEST SERIES - 2015


Government is formed from the legislature. So the whole legislature can NOT be called the
governemt. State judiciary and human rights commission keep a check on the activities of
the government. They can not be a part of the state government.

34. Solution: d)
The Prevention of Sexual harassment at Workplace Act, 2013 was recently enacted. Domestic
violence act is in place since 2005.

35. Solution: a)
Large bourses who control the money flow to media can prevent unbiased reporting from
media. They further may have connection with the political class and this may encourage
selective and distorted reporting.
Option 3 is in fact a way to make the functioning of media transparent and unbiased.

36. Solution: d)
In India, production of sugar from Sugarcane mostly takes place at Cooperative Sugar Mills
owned by local Farmers The Shareholders include all farmers, small and large,
supplying sugarcane to the mill.
Amul is an Indian dairy cooperative, based at Anand in the state of Gujarat, India. Amul
spurred India's White Revolution, which made the country the world's largest producer of
milk and milk products. In the process Amul became the largest food brand in India and has
ventured into markets overseas.
Both rural and urban cooperative banks exist which help the marginalized sections.

37. Solution: a)
Most planets also rotate on their axes (as well as revolve) in an anti-clockwise direction, but
Venus rotates clockwise (called "retrograde" rotation) once every 243 Earth days.

38. Solution: a)
Comet

http://insightsonindia.com

INSIGHTS

Page 11

INSIGHTS ON INDIA TEST SERIES - 2015

A comet is a relatively small solar system body that orbits the Sun. When close
enough to the Sun they display a visible coma (a fuzzy outline or atmosphere due to
solar radiation) and sometimes a tail.

Asteroid

Asteroids are small solar system bodies that orbit the Sun. Made of rock and metal,
they can also contain organic compounds. Asteroids are similar to comets but do not
have a visible coma (fuzzy outline and tail) like comets do.

Meteoroid

A meteoroid is a small rock or particle of debris in our solar system. They range in
size from dust to around 10 metres in diameter (larger objects are usually referred to
as asteroids).

Meteor

A meteoroid that burns up as it passes through the Earths atmosphere is known as


a meteor. If youve ever looked up at the sky at night and seen a streak of light or
shooting star what you are actually seeing is a meteor.

Meteorite

A meteoroid that survives falling through the Earths atmosphere and colliding with
the Earths surface is known as a meteorite.

39. Solution: d)
The mid-day sun is exactly overhead at least once a year on all latitudes in between the
Tropic of Cancer and the Tropic of Capricorn. This area, therefore, receives the maximum
heat and is called the Torrid Zone. The mid-day sun never shines overhead on any latitude
beyond the Tropic of Cancer and the Tropic of Capricorn. The angle of the suns rays goes
on decreasing towards the poles. As such, the areas bounded by the Tropic of Cancer and
the Arctic Circle in the Northern Hemisphere, and the Tropic of Capricorn and the Antarctic
Circle in the Southern Hemisphere, have moderate temperatures. These are, therefore, called
Temperate Zones.
The distance between latitudes keep increasing as one moves from equator to the poles. The
same between longitudes is constant at the same latitude.
40. Solution: a)
Options 1 and 2 can be logically deduced. Option 3 may be possible if Earth stops revolving
around the Sun. Stopping rotation would not affect its elliptical orbit.
41. Solution: a)

http://insightsonindia.com

INSIGHTS

Page 12

INSIGHTS ON INDIA TEST SERIES - 2015


Self-explanatory. However, at the equator the level of water is a little higher by 8cm. This is
because of greater solar insolation received at the equator. More heat causes expansion in
the sea water volume.

42. Solution: c)
The Atacama Desert is a plateau in South America, covering a 1,000-kilometre strip of land
on the Pacific coast, west of the Andes mountains. It is the driest non-polar desert in the
world.
The Great Victoria Desert, an interim Australian bioregion, is a sparsely populated desert
area in Western Australia and South Australia.
43. Solution: a)
The indented coastline as given in the figure below helps in two ways. One, it provides
natural sideways protection for the ships if they harbor there. And two, due to its shape, less
cost would have to be incurred even if artificial ports are to made there.

44. Solution: c)
There are three types of mountains- Fold Mountains, Block Mountains and the Volcanic
Mountains. The Himalayan Mountains and the Alps are young fold mountains with rugged
relief and high conical peaks. The Aravali range in India is one of the oldest fold mountain
systems in the world. The range has considerably worn down due to the processes of
erosion. The Appalachians in North America and the Ural mountains in Russia have
rounded features and low elevation. They are very old fold mountains.

45. Solution: a)
Self-explanatory.

http://insightsonindia.com

INSIGHTS

Page 13

INSIGHTS ON INDIA TEST SERIES - 2015


46. Solution: b)
The Vindhyas and the Satpuras are the important ranges in the Pennisular plateau. The
rivers Narmada and Tapi flow through these ranges. These are west-flowing rivers that
drain into the Arabian Sea.
The Western Ghats or Sahyadris border the plateau in the west and the Eastern Ghats
provide the eastern boundary. While the Western Ghats are almost continuous, the Eastern
Ghats are broken and uneven (Figure 7.3). The plateau is rich in minerals like coal and ironore. To the West of the Western Ghats and the East of Eastern Ghats lie the Coastal plains.
The western coastal plains are very narrow. The eastern Coastal plains are much broader.

47. Solution: b)
Continentality means that the more a place is surrounded by land, the more extreme its
weather will be .Because land heats faster as compared to sea.
Opposed to this, closeness to the sea moderates weather. Because in the day the sea heats
slower than land. This cool sea breeze blows across the land thus reducing the day
temperatures.
In the night the sea cools down slower. And the exact opposite happens.
This keeps the temperature moderate.

48. Solution: c)
Lakshadweep is a coral island. The Sundarbans is a natural region in Bengal. It is the largest
single block of tidal halophytic mangrove forest in the world.

49. Solution: d)
Sundarbans house littoral and swamp forests. Mangroves are found here, not evergreen
forests. However, the area surrounding Sundarbans have tropical evergreen forests.
Other places (except Andaman and Nicobar) have mangroves, but not evergreen forests.

50. Solution: d)
Continental crust is less denser than ocean crust as the later is made of basaltic rocks.
The main constituents of the former are Silica and Aluminium; and that of the later are Silica
and Magnesium.
http://insightsonindia.com

INSIGHTS

Page 14

INSIGHTS ON INDIA TEST SERIES - 2015

51. Solution: b)
Sedimentary rocks are formed when sediments (broken from a rock) are deposited and
consolidated under pressure or temperature or in other ways. Only igneous rocks can be
directly formed from magma, not sedimentary rocks.

52. Solution: c)
Sediment load has to do with formation of floodplains and levees.
This is how an ox-bow lake is formed.
As the river enters the plain it twists and turns forming large bends known as meanders.
Due to continuous erosion and deposition along the sides of the meander, the ends of the
meander loop come closer and closer. In due course of time the meander loop cuts off from
the river and forms a cut-off lake, also called an ox-bow lake.

53. Solution: d)

http://insightsonindia.com

INSIGHTS

Page 15

INSIGHTS ON INDIA TEST SERIES - 2015

54. Solution: c)
Albedo is the reflectivity of a surface. The more ice is there on the earth, the more it will
reflect sunrays as white ice is a good reflector. Therefore, albedo will decrease. So the
temperature of the earth will increase.
This and the melting of the snow will cause an expansion in the volume of sea water leading
to the flooding of the coastal areas.
Floods and drought will become more severe and frequent in certain pockets of the world.
This is because some regions will become hotter than usual. They will experience either
more intense rain leading to floods or more intense evaporation leading to hydrological
drought.
Such disturbance in the local weather, and climate later on, will disturb the local
environment. This will create problems for the species which will find it tough to adapt to
the sudden changing environment; flood; drought; high temperature; changing flora etc.
Some of them may go extinct too.

55. Solution: a)
While nitrogen is used by plants for making food, they do not obtain it directly from
atmosphere. Certain bacteria fix nitrogen from the atmosphere and make it available to the
plants in the form of nitrate etc.
Hydrogen and oxygen from the atmosphere are not used in making food.

56. Solution: c)
Self-explanatory. Refer to page no. 22 Structure of atmosphere Chapter 4- NCERT Geo
class 7th for more related details.

57. Solution: d)
Winds can be broadly divided into three types.
1. Permanent winds The trade winds, westerlies and easterlies are the permanent
winds.These blow constantly throughout the year in a particular direction.
2. Seasonal winds These winds change their direction in different seasons. For example
monsoons in India.
3. Local winds These blow only during a particular period of the day or year in a small area.
http://insightsonindia.com

INSIGHTS

Page 16

INSIGHTS ON INDIA TEST SERIES - 2015

58. Solution: d)
All are true. Jet planes flying in the sky leave a white trail behind them. The moisture from
their engines condenses. We see trails of this condensed moisture for some time when there
is no air movement to disturb it.

59. Solution: d)
During a storm, the winds blowing at very high speed form huge waves. These may cause
tremendous destruction. An earthquake, a volcanic eruption or underwater landslides can
shift large amounts of ocean water. As a result a huge tidal wave called tsunami, that may
be as high as 15m., is formed. The largest tsunami ever measured was 150m. high. These
waves travel at a speed of more than 700 km. per hour. The tsunami of 2004 caused wide
spread damage in the coastal areas of India. The Indira point in the Andaman and Nicobar
islands got submerged after the tsunami.

60. Solution: d)
The strong gravitational pull exerted by the sun and the moon on the earths surface causes
the tides. The water of the earth closer to the moon gets pulled under the influence of the
moons gravitational force and causes high tide. During the full moon and new moon days,
the sun, the moon and the earth are in the same line and the tides are highest. These tides are
called spring tides. But when the moon is in its first and last quarter, the ocean waters get
drawn in diagonally opposite directions by the gravitational pull of sun and earth resulting
in low tides. These tides are called neap tides.

61. Solution: a)
High tides help in navigation. They raise the water level close to the shores. This helps the
ships to arrive at the harbour more easily. The high tides also help in fishing. Many more
fish come closer to the shore during the high tide. This enables fishermen to get a plentiful
catch. The rise and fall of water due to tides is being used to generate electricity in some
places.
Marine upwelling is a different phenomen altogether.
Winds blowing across the ocean surface push water away. Water then rises up from beneath
the surface to replace the water that was pushed away. This process is known as
upwelling.

http://insightsonindia.com

INSIGHTS

Page 17

INSIGHTS ON INDIA TEST SERIES - 2015


Upwelling occurs in the open ocean and along coastlines. The reverse process, called
downwelling, also occurs when wind causes surface water to build up along a coastline
and the surface water eventually sinks toward the bottom.
Water that rises to the surface as a result of upwelling is typically colder and is rich in
nutrients. These nutrients fertilize surface waters, meaning that these surface waters often
have high biological productivity. Therefore, good fishing grounds typically are found
where upwelling is common.

62. Solution: d)
Self-explanatory. Refer to the last two pages of chapter 5 for more information on warm and
old currents.

63. Solution: a)
They are also found in large parts of India.
Mediterranean vegetation is mostly found in the areas around the Mediterranean sea in
Europe, Africa and Asia, hence the name.
Boreal forests are cold forests which have mostly coniferous trees.

64. Solution: d)
There most important difference between a steppe and a savanna is where it is located.
Savannas lie nearer the equator and are warmer than steppes. Being nearer the rainforest
means they have two major seasons: a hot, wet summer and a marginally cooler, but much
drier winter. Steppes, by contrast, lie further from the equator and in sheltered areas. This
means the precipitation they get is dispersed evenly throughout the year. Fewer large plants
can take root under such dry conditions. Being further from the equator also means colder
conditions, and in some northern steppes, snow, rather than rain, is common.
Read more at : http://www.ehow.com/info_8468978_differences-between-steppessavannas.html
65. Solution: a)
If you reach the polar region (Tundra type of vegetation) you will find the place extremely
cold. The growth of natural vegetation is very limited here. Only mosses, lichens and very
small shrubs are found here. It grows during the very short summer.

http://insightsonindia.com

INSIGHTS

Page 18

INSIGHTS ON INDIA TEST SERIES - 2015


66. Solution: d)
Waterways are the cheapest for carrying heavy and bulky goods over long distances. They
are mainly of two types inland waterways and sea routes. Navigable rivers, canals and
lakes are used as inland waterways. Some of the important inland waterways are the GangaBrahmaputra river system, the Great Lakes in North America and the river Nile in Africa.
Sea routes and oceanic routes are mostly used for transporting merchandise and goods from
one country to another. These routes are connected with the ports.

67. Solution: d)
Arikamedu is an archaeological site in Kakkayanthope, Ariyankuppam
Commune, Puducherry. It is located at a distance 7 km from its capital Pondicherry, in the
Indian territory of Puducherry, where Mortimer Wheeler conducted his best-known
excavation in the 1940s. Arikamedu - 'Arikan-medu or Poduke' which literally means,
'eroding mound'. Roman lamps, glassware and gems have also been found at the site.
For Map, refer to NCERT Class VI History - Page No. 85.
68. Solution: b)
In the northern part of the country, the village headman was known as the grama bhojaka.
Usually, men from the same family held the position for generations. In other words, the
post was hereditary. The grama bhojaka was often the largest landowner. Generally, he had
slaves and hired workers to cultivate the land. Besides, as he was powerful, the king often
used him to collect taxes from the village. He also functioned as a judge, and sometimes as a
policeman.
Apart from the gramabhojaka, there were other independent farmers, known as grihapatis,
most of whom were smaller landowners. And then there were men and women such as the
dasa karmakara, who did not own land, and had to earn a living working on the fields
owned by others. In most villages there were also some crafts persons such as the
blacksmith, potter, carpenter and weaver.
69. Solution: c)
Refer page 107, NCERT Class VI History Book.
70. Solution: b)
A famous Tamil epic, the Silappadikaram, was composed by a poet named Ilango, around
1800 years ago. It is the story of a merchant named Kovalan, who lived in Puhar and fell in
love with a courtesan named Madhavi, neglecting his wife Kannagi. Later, he and Kannagi
http://insightsonindia.com

INSIGHTS

Page 19

INSIGHTS ON INDIA TEST SERIES - 2015


left Puhar and went to Madurai, where he was wrongly accused of theft by the court
jeweller of the Pandya king. The king sentenced Kovalan to death. Kannagi, who still loved
him, was full of grief and anger at this injustice, and destroyed the entire city of Madurai.
Another Tamil epic, the Manimekalai was composed by Sattanar around 1400 years ago.
This describes the story of the daughter of Kovalan and Madhavi. These beautiful
compositions were lost to scholars for many centuries, till their manuscripts were
rediscovered, about a hundred years ago.
71. Solution: d)
Both are correct.
http://www.thehindu.com/sci-tech/super-cyber-intelligence-body-soon-announces-itminister/article6408263.ece
72. Solution: c)
It is a tradition that dates back as far as the 11th century when it originated in the villages of
the Red River Delta area of northern Vietnam. Today's Vietnamese water puppetry is a
unique variation on the ancient Asian puppet tradition.
The puppets are made out of wood and then lacquered. The shows are performed in a waistdeep pool. A large rod supports the puppet under the water and is used by the puppeteers,
who are normally hidden behind a screen, to control them. Thus the puppets appear to be
moving over the water. When the rice fields would flood, the villagers would entertain each
other using this form of puppet play.
http://www.thehindu.com/business/Industry/india-vietnam-to-sign-key-oil-searchpact/article6410313.ece (see the emage)

73. Solution: a)
Space Exploration Technologies Corporation, or SpaceX, is a space transport services
company headquartered in Hawthorne, California. It was founded in 2002 by
former PayPal entrepreneur and Tesla Motors CEO Elon Musk. SpaceX was founded with
the goal of reducing space transportation costs and enabling the colonization of Mars. It has
developed the Falcon 1 and Falcon 9 launch vehicles, both of which were designed from
conception to eventually become reusable. SpaceX also developed the Dragon spacecraft,
which are flown into orbit by the Falcon 9 launch vehicle, initially transporting cargo and
later planned to carry humans to the International Space Station and other destinations.

http://insightsonindia.com

INSIGHTS

Page 20

INSIGHTS ON INDIA TEST SERIES - 2015


In 2006, NASA awarded the company a Commercial Orbital Transportation Services (COTS)
contract to design and demonstrate a launch system to resupply cargo to the International
Space Station (ISS). SpaceX has since flown four missions to the ISS.
http://www.thehindu.com/sci-tech/science/nasa-fourth-spacex-cargo-mission-will-carryfirst-3d-printer-to-the-international-space-station/article6410118.ece

74. Solution: c)
The Trans-Pacific Partnership (TPP) is a proposed regional free-trade agreement. As of
2014 twelve countries throughout the Asia-Pacific region have participated in negotiations
on the TPP: Australia, Brunei Darussalam, Canada, Chile, Japan, Malaysia, Mexico, New
Zealand, Peru, Singapore, the United States, and Vietnam.
India is not involved in its negotiations.
http://www.ustr.gov/tpp

75. Solution: a)
The deployment of IndARC, the countrys first underwater moored observatory in the
Kongsfjorden fjord, half way between Norway and the North Pole, represents a major
milestone in Indias scientific endeavours in the Arctic region.
Designed and developed by scientists from the Earth System Science Organisation (ESSO),
National Centre for Antarctic and Ocean Research (NCAOR), National Institute of Ocean
Technology (NIOT) and Indian National Centre for Ocean Information Services (INCOIS),
IndARC was deployed from RV Lance, a research vessel belonging to the Norwegian Polar
Institute on July 23.
The observatory is anchored at a depth of 192 m and has an array of 10 state-of-the-art
oceanographic sensors strategically positioned at various depths in the water.
http://www.thehindu.com/sci-tech/energy-and-environment/indias-arctic-observatoryto-aid-climate-change-studies/article6405727.ece

76. Solution: c)
http://www.thehindu.com/sci-tech/science/spinosaurus-aegyptiacus-alienlike-giantwaterliving-dinosaur-unveiled/article6403739.ece

77. Solution: d)
Both are wrong.

http://insightsonindia.com

INSIGHTS

Page 21

INSIGHTS ON INDIA TEST SERIES - 2015


The Jaipur Foot, also known as the Jaipur Leg, is a rubber-based prosthetic leg for people
with below-knee amputations. Although inferior in many ways to the composite carbon
fibre variants, its variable applicability and cost efficiency make it an acceptable choice for
prosthesis. Ram Chander Sharma designed and developed it in 1968.
The Jaipur Foot is made of polyurethane, which at the time was the new material used in the
production of the prostheses. The material increases the durability and the convenience of
use. Now the government of India supports Bhagwan Mahavir Viklang Sahyata Samiti with
financial aid to carry out the work done by the organization.
http://www.thehindu.com/news/national/other-states/jaipur-foot-to-help-cowsnow/article6405736.ece

78. Solution: c)
At about 31 per cent of the world total, Australia has the largest known reserves of uranium.
Just before concluding the civil nuclear agreement with India, Australia had drawn the
curtain on its uranium trade with Russia, which is one of the largest suppliers of uranium to
India.
India is the only non-NPT signatory to which Australia will be supplying uranium. Besides
Russia, India imports most of its uranium from Kazakhstan and France.
http://www.thehindu.com/news/national/australiabanking-on-uranium-pact-to-gainedge-over-eurasian-rivals/article6401600.ece

79. Solution: c)
http://www.thehindu.com/news/national/centre-to-fund-swachh-vidyalayacampaign/article6401957.ece

80. Solution: a)
http://www.thehindu.com/sci-tech/nasa-to-map-earth-forests-in-3d/article6397048.ece

81. Solution: b)
Self-explanatory. Refer to the the section Life in Ganga-Brahmaputra basin- Chapter 8 7th
Geo NCERT

82. Solution: a)
http://insightsonindia.com

INSIGHTS

Page 22

INSIGHTS ON INDIA TEST SERIES - 2015


The main crop is paddy since cultivation of paddy requires sufficient water, it is grown in
the areas where the amount of rainfall is high. Wheat, maize, sorghum, gram and millets are
the other crops that are grown. Cash crops like sugarcane and jute are also grown. Banana
plantations are seen in some areas of the plain. In West Bengal and Assam tea is grown in
plantations (Fig. 8.10). Silk is produced through the cultivation of silk worms in parts of
Bihar and Assam.

83. Solution: b)

84. Solution: a)
Refer to sec 1.2 chapter 1 class 7th Science NCERT

85. Solution: a)
Normally bacteria are present in our mouth but they are not harmful to us. However, if we
do not clean our teeth and mouth after eating, many harmful bacteria also begin to live and
grow in it. These bacteria break down the sugars present from the leftover food and release
acids. The acids gradually damage the teeth. This is called tooth decay.

86. Solution: a)
http://insightsonindia.com

INSIGHTS

Page 23

INSIGHTS ON INDIA TEST SERIES - 2015


Refer to http://www.instructables.com/id/Oral-Rehydration-Salts-ORS/

87. Solution: a)
Refer to the link for more information on disease causing bacteria in humans
http://www.biotechnologyforums.com/thread-1945.html
88. Solution: a)
We often use electricity and fuels like coal and wood to keep our houses cool or warm. It is
possible to construct buildings, that are not affected much by heat and cold outside. This can
be done by constructing outer walls of buildings so that they have trapped layers of air. One
way of doing this is to use hollow bricks, which are available these days.

89. Solution: a)
The most commonly used natural indicator is litmus. It is extracted from lichens . It has a
mauve (purple) colour in distilled water. When added to an acidic solution, it turns red and
when added to a basic solution, it turns blue. It is available in the form of a solution, or in
the form of strips of paper, known as litmus paper. Generally, it is available as red and blue
litmus paper.

90. Solution: d)
The rain becomes acidic because carbon dioxide, sulphur dioxide and nitrogen dioxide
(which are released into the air as pollutants) dissolve in rain drops to form carbonic acid,
sulphuric acid and nitric acid respectively. Acid rain can cause damage to buildings,
historical monuments, plants and animals.

91. Solution: a)
Self-explnatory. The effect of an acid can be neutralized using a base.

92. Solution: b)
Self-explanatory.
93. Solution: d)

http://insightsonindia.com

INSIGHTS

Page 24

INSIGHTS ON INDIA TEST SERIES - 2015


94. Solution: d)
http://science.howstuffworks.com/life/fungi/yeast-info.htm

95. Solution: a)
Plants absorb mineral nutrients and water from the soil. Not all the water absorbed is
utilised by the plant. The water evaporates through the stomata present on the surface of the
leaves by the process of transpiration. The evaporation of water from leaves generates a
suction pull (the same that you produce when you suck water through a straw) which can
pull water to great heights in the tall trees. Transpiration also cools the plant.

96. Solution: d)
Refer to http://www.biology-resources.com/plants-vegetative-reproduction-01.html

97. Solution: a)
The images reflected in a convex mirror, then, look smaller than they are -- they're
compressed. This is why convex mirrors are used on cars: They reflect more in a smaller
space. In other words, a convex mirror has a wider field of view than a flat one, which can
only reflect the area right in front of it. With a wider field of view, the driver has more
information about the area to the right of the car.

98. Solution: d)
http://en.wikipedia.org/wiki/Water_pollution_in_India#Other_problems

99. Solution: a)
Refer to http://www.botanical-online.com/english/eucalyptus.htm

100.

Solution: a)

http://donateeyes.org/what-is-eye-donation/

http://insightsonindia.com

INSIGHTS

Page 25

INSIGHTS ON INDIA TEST SERIES - 2015

http://insightsonindia.com

INSIGHTS

Page 26

Facebook Group: Indian Administrative Service (Raz Kr)

INSIGHTS ON INDIA MOCK PRELIMINARY EXAM - 2014


INSIGHTS ON INDIA MOCK TEST - 2
GENERAL STUDIES

PAPER-I
Time Allowed: Two Hours

Maximum Marks: 200

INSTRUCTIONS
1. IMMEDIATELY AFTER THE COMMENCEMENT OF THE EXAMINATION, YOU SHOULD
CHECK THAT THIS TEST BOOKLET DOES NOT HAVE ANY UNPRINTED OR TORN OR
MISSING PAGES OR ITEMS, ETC. IF SO, GET IT REPLACED BY A COMPLETE TEST BOOKLET.
2. You have to enter your Roll Number on the Test I
Booklet in the Box provided alongside. DO NOT
write anything else on the Test Booklet.
4. This Test Booklet contains 100 items (questions). Each item is printed only in English. Each item
comprises four responses (answers). You will select the response which you want to mark on the Answer
Sheet. In case you feel that there is more than one correct response, mark the response which you consider
the best. In any case, choose ONLY ONE response for each item.
5. You have to mark all your responses ONLY on the separate Answer Sheet provided. See directions in the
Answer Sheet.
6. All items carry equal marks.
7. Before you proceed to mark in the Answer Sheet the response to various items in the Test Booklet, you
have to fill in some particulars in the Answer Sheet as per instructions sent to you with your Admission
Certificate.
8. After you have completed filling in all your responses on the Answer Sheet and the examination has
concluded, you should hand over to the Invigilator only the Answer Sheet. You are permitted to take away
with you the Test Booklet.
9. Sheets for rough work are appended in the Test Booklet at the end.
10. Penalty for wrong answers :
THERE WILL BE PENALTY FOR WRONG ANSWERS MARKED BY A CANDIDATE IN THE
OBJECTIVE TYPE QUESTION PAPERS.
i.

There are four alternatives for the answer to every question. For each question for which a wrong
answer has been given by the candidate, one-third of the marks assigned to that question will be
deducted as penalty.

ii.

If a candidate gives more than one answer, it will be treated as a wrong answer even if one of the given
answers happens to be correct and there will be same penalty as above to that question.

iii.

If a question is left blank, i.e., no answer is given by the candidate, there will be no penalty for that
question.

http://insightsonindia.com

INSIGHTS ON INDIA MOCK TEST SERIES FOR CIVIL SERVICES PRELIMINARY EXAM 2014

http://insightsonindia.com

Page 1

Facebook Group: Indian Administrative Service (Raz Kr)

1. Consider the following statements


on fish farming
1. Catla, Rohu and Mrigal are
famous marine fish breeds
of high economic value
which are farmed in sea
water
2. Finned fishes like mullets,
bhetki, and pearl spots are
farmed in freshwater farms
Which of the statements given above
is/are correct?
a.
b.
c.
d.

1 Only
2 Only
Both
None

2. On the basis of landmark judgments


passed by the Supreme Court, the Election
Commission of India came up with the
following mandates:
1. Every candidate has to make a legal
declaration, giving full details of
serious criminal cases pending
against the candidate
2. Details of the assets and liabilities
of the candidate and his or her
family
3. Education qualifications was not
mandatory
4. The information furnished by the
candidates has to be confidential.
5. The above mandates are in the
spirit of Article 19.
Which of the statements is/are correct?
a.
b.
c.
d.

1, 2 and 5
1, 2 and 3
1, 2, 3 and 4
1, 2, 3, 4 and 5

http://insightsonindia.com

3. Consider the following statements about


resource
1. Potential resources are those
resources whose quantity is known.
These resources are being used in
the present.
2. Actual resources are those whose
entire quantity may not be known
and these are not being used at
present. These resources could be
used in the future.
3. Resources
that
are
found
everywhere like the air we breathe,
are ubiquitous.
Which of the above statements is/are
correct?
a. 1 and 2 Only
b. 2 and 3 Only
c. 3 Only
d. 1,2 and 3
4.

Consider the following statements

1. Lord Hastings, Governor General


of India during 1813-23, initiated a
new policy of paramountcy. Now
the Company claimed that its
authority was paramount or
supreme, hence its power was
greater than that of Indian states.
2. Rani Channamma of Kitoor
(Karnataka) took to arms against
this policy and led an anti-British
resistance movement
Which of the statements given above
is/are correct?
a.
b.
c.
d.

1 Only
2 Only
Both
None
Page 2

Facebook Group: Indian Administrative Service (Raz Kr)

5.

Social Justice is an ideal cherished


by the founding fathers of the
Indian Constitution. With regard to
this, the Government of India had
appointed Mandal Commission.
1. Mandal Commision was asked to
determine the criteria to identify
the socially and educationally
backward classes and the status of
minorities in India.
2. One of the recommendations was
that 27 percentages of government
jobs be reserved for the socially and
educationally backward classes and
minorities.
3. In the Indira Sawhney and others
Vs Union of India case, it was
declared that the governments
order with respect to reservation
for other backward classes was
invalid.
Which of the statements is/are incorrect?
a.
b.
c.
d.

1 Only
3 Only
1, 2 and 3
None

6. Consider the following statements


about James Mill,
1. Suggested that the British should
conquer all the territories in India
to ensure the enlightenment and
happiness of the Indian people.
2. Mill thought that all Asian societies
were at a lower level of civilization
than Europe.
3. He published a massive threevolume work, A History of British
India in which he divided Indian
history into three periods Hindu,
Muslim and British.
Which of the above statement is/ are
correct?
a.
b.
c.
d.

1 and 2 only
2 and 3 only
All Three
None of them

http://insightsonindia.com

7. With regard to cropping patterns,


consider the following statements
1. Crop rotation is growing
two
or
more
crops
simultaneously on the same
piece of land,
2. Inter-cropping is growing
two
or
more
crops
simultaneously on the same
field in a definite pattern
3. The growing of different
crops on a piece of land in a
pre-planned succession is
known as mixed cropping.
Which of the statements given above
is/are correct?
a.
b.
c.
d.

1 Only
2 and 3 Only
2 Only
All Three

8. Which of the following diseases


is/are caused by virus?
1. Hepatitis-B
2. Polio
3. Measles
4. Cholera
5. Chicken Pox
Answer using the codes given below
a. 1,2 and 4 Only
b. 1,3 and 4 Only
c. 2,3 and 4 Only
d. 1,2,3 and 5 Only
9. According the parliamentary form
of Government adopted in India,
which of the following statements
are correct?
1. The Parliament is the
authority for making laws.

final
Page 3

Facebook Group: Indian Administrative Service (Raz Kr)

2. The Parliament control over those


who run the government is direct
and full.
3. The Parliament is the highest
forum of discussion and debate on
public issues and national policy.
4. The Parliament of India enjoys
absolute power and can seek
information about any matter.
Select the correct answer using the codes
given below
a.
b.
c.
d.

1 and 3
3 Only
1, 2 and 3
1, 2, 3 and 4

base and rapidly narrows towards


the top.
2. In countries where death rates
(especially
amongst the very
young) are decreasing, the pyramid
is broad in the younger age groups
Which of the statements above is/are
correct?
a. 1 Only
b. 2 Only
c. Both
d. None

10. Consider the following statements


with respect to the powers of the
Rajya Sabha:
1. Any ordinary law needs to be
passed by both the Lok Sabha and
Rajya Sabha. But if there is a
difference between the two
Houses, the final decision is taken
by Lok Sabha.
2. Once the Lok Sabha passes the
budget of the government or any
other money related law, the Rajya
Sabha cannot reject it.
3. The Rajya Sabha has power to pass
motion of no-confidence against
the Government.
Which of the statements is/are incorrect?
a.
b.
c.
d.

2 Only
1 and 3
1, 2 and 3
None

11. Read carefully


statements,

12. Consider the following statements


with reference to the formation of
Indian subcontinent
1. One of the oldest landmasses,
the Peninsula part, was a part of
the Gondwanaland.
2. The Gondwanaland included
India, Australia, South Africa,
Arabian Peninsula, Africa and
South America as one single
land mass
3. The collision of Indi-Australian
plate with the Eurasian plate
resulted in the formation of the
Himalayas
Which of the statements given above
is/are correct?

the

following

a.
b.
c.
d.

1 and 2 Only
1 and 3 Only
2 and 3 Only
All Three

1. The population pyramid of a


country in which birth and death
rates both are high is broad at the
http://insightsonindia.com

Page 4

Facebook Group: Indian Administrative Service (Raz Kr)

13. Which of the following not correct


about East India Company at 1600
a. In 1600, the East India Company
acquired a charter from the ruler of
England
b. No other trading group in England
could compete with the East India
Company.
c. The Company did not have to fear
competition from other English
trading companies.
d. None of them
14. Consider the following statements
about East India Company
1. The first English factory was set up
on the banks of the river Hugli in
1651.
2. This was the base from which the
Companys traders, known at that
time as factors,
Which of the above statement is/ are
correct?
a.
b.
c.
d.

1 only
2 only
Both
None

15. Carbon dioxide (CO2) is the best


extinguisher. Because,
1. CO2, being heavier than
oxygen, covers the fire like a
blanket. Since the contact
between the fuel and oxygen is
cut off, the fire is controlled.
2. The added advantage of CO2 is
that in most cases it does not
harm the electrical equipment.

a.
b.
c.
d.

1 Only
2 Only
Both
None

16. With reference to problems caused


by deforestation, consider the
following statements,
1. Deforestation
is
a
contributor
to global
warming, and is often cited
as one of the major causes
of the enhanced greenhouse
effect.
2. Deforestation reduces soil
cohesion, so that erosion,
flooding
and landslides ensue.
3. Deforestation on a human
scale results in decline in
biodiversity, and
on
a
natural global scale is
known to cause the
extinction of many species.
4. Ground water level also
gets lowered.
5. In deforested areas, the land
heats up faster and reaches
a
higher
temperature,
leading to localized upward
motions that enhance the
formation of clouds and
ultimately produce more
rainfall.
Which of the statements given above
is/are correct?
a.
b.
c.
d.

1, 2, 3 and 4 Only
1, 2,3 and 5 Only
1,2,4 and 5 Only
1,2,3,4 and 5

Which of the statements given above


is/are correct?

http://insightsonindia.com

Page 5

Facebook Group: Indian Administrative Service (Raz Kr)

17. With reference to the cloning of


Dolly the sheep, consider the
following statements
1. Two cells that were used to
clone Dolly were taken
from two female sheep
(ewes)
2. The nucleus of the egg cell
was removed and replaced
by the nucleus of the body
cell
3. Dolly was a clone of donor
of the egg cell
Which of the above statements is/are
correct?
a.
b.
c.
d.

1 and 2 Only
2 and 3 Only
1 and 3 Only
All Three

18. Consider the following statements


with regard to the Executives of
India:
1. Functionaries who take day-to-day
decisions but do not exercise
supreme power on behalf of the
people are collectively known as
the executive.
2. The temporary executives have
more power than the permanent
executives.
3. Secretary of Ministry of State and
Director General of Police are part
of temporary Executive.
4. The temporary executives are
experts in the matters of their
ministry.
Which of the statements is/are correct?
a.
b.
c.
d.

1 and 2
1, 2 and 3
1, 2, 3 and 4
None

http://insightsonindia.com

19. Prime Minister is the most


important political institution in
the country. Consider the following
statements:
1. There is no direct election to the
post of the Prime Minister.
2. The Prime Minister has a fixed
tenure.
3. A person who is not a member of
Parliament can also become a
minister provided he meets the
required conditions.
4. When the Prime Minister quits, the
entire ministry quits.
5. The rise of coalition politics has
imposed certain constraints on the
power of the Prime Minister.
Which of the statements is/are correct?
a.
b.
c.
d.

1, 2 and 5
2, 3 and 5
1, 3, 4 and 5
2, 3, 4 and 5

20. The Nachiketa Mor panel recently


gave several recommendations for
increasing the level of financial
inclusion in the country. What can
be the direct and indirect benefits
of financial inclusion?
1. Increase in domestic savings base
2. Higher economic growth for the
nation
3. Reduction in local-money lending
practices
4. Discouraging investment in Ponzi
schemes, non-productive assets etc.
5. Better public service delivery
Select the correct answer using the codes
given below
a.
b.
c.
d.

1 and 3
1,2, 3 and 4
4 and 5
All of the above
Page 6

Facebook Group: Indian Administrative Service (Raz Kr)

21. With reference to the national


Population Policy 2000, consider
the following statements
It imparts policy framework for,
1. Imparting
free
and
compulsory
school
education up to 14 years of
age.
2. Reducing infant mortality
rate to below 30 per 1000
live births
3. Achieving
universal
immunisation of children
against
all
vaccine
preventable diseases
4. Encouraging
delayed
marriage and child-bearing,
education of adolescents
about
the
risks
of
unprotected sex, making
contraceptive
services
accessible and affordable,
providing
food
supplements,
nutritional
services, strengthening legal
measures to prevent child
marriage.
Which of the statements given above
is/are correct?
a.
b.
c.
d.

following
Aurangzebs

1. He had granted only the Company


the right to trade duty free
http://insightsonindia.com

Which of the above statement is/ are


correct?
a.
b.
c.
d.

1 only
2 only
Both of Them
None of them
23. The Micro Finance Institutions
(MFIs)(Regulation
and
Development) Bill, 2012 intends
to regulate and develop the
MFIs sector. Who can be the
potential beneficiaries if it
becomes a law?

1.
2.
3.
4.

Small and Marginal farmers


Self-Help Groups
MFIs
Local money lenders

Select the correct answer using the codes


given below
a.
b.
c.
d.

1 and 2 only
3 and 4 only
1,2 and 3
All of the above

24. Consider the following statements


about forces

2 and 4 Only
2,3 and 4 Only
2 and 3 Only
1,2,3 and 4
22. Consider
the
statements about
farman,

2. For private trade, the officials


refused to pay the duty, causing an
enormous loss of revenue for
Bengal

1. Both electrostatic force and the


force exerted by magnets are
examples of non-contact forces
2. The force of gravity acts on all of us
all the time.
3. The force of friction always acts on
all the moving objects and its
direction is always opposite to the
direction of motion.
Page 7

Facebook Group: Indian Administrative Service (Raz Kr)

Which of the statements given above


is/are correct?
a.
b.
c.
d.

1 and 2 Only
2 and 3 Only
2 Only
All Three

25. Consider the following statements on


sound
1. The loudness of sound depends on
its frequency. When the frequency
of vibration is large, the sound
produced is loud.
2. The amplitude determines the
shrillness or pitch of a sound.
3. In humans, the sound is produced
by the larynx.
Which of the above statements is/are
correct?
a.
b.
c.
d.

1 and 2 Only
3 Only
All Three
None

26.
Assertion
(A):
Parliamentary
democracy in most countries is often
known as the Cabinet form of government.
Reason (R): It is not practical for all
ministers to meet regularly and discuss
everything; hence the decisions are taken in
Cabinet meetings
Which of the statements is correct?
a. Both A and R are True and R is the
correct explanation of A
b. Both A and R are True and R is not
the correct explanation of A
c. A is True and R is False
d. A is False and R is True
27. With reference to lightning, consider
the following statements
http://insightsonindia.com

1. During storms, negative and


positive charges meet in the clouds,
producing streaks of bright light
and sound. The process is called an
electric discharge.
2. During lightning coming in contact
with
open
vehicles,
like
motorbikes, tractors, construction
machinery, open cars are not safe.
3. During lightning it is not safe to use
mobile phone but it is safe to use a
landline.
4. Carrying umbrella is not a good
idea at all during thunderstorms
Which of the above statements is/are
correct?
a.
b.
c.
d.

1,2 and 3 Only


2 and 3 Only
1,2 and 4 Only
All Four

28.
The functions of the Indian
Government is carried out by a number of
Ministry and Departments. Consider the
following statements:
1. No minister can openly criticise any
decision of the government, even if
it is about another Ministry or
Department.
2. Every ministry has secretaries, who
are civil servants.
3. The secretaries provide the
necessary background information
to the ministers to take decisions.
4. The Prime Ministers decisions are
final in case disagreements arise
between Departments.
Which of
incorrect?
a.
b.
c.
d.

the

statements

is/are

1 and 4
1, 2 and 4
1, 2, 3 and 4
None

Page 8

Facebook Group: Indian Administrative Service (Raz Kr)

29. Free Trade Agreement (FTA) between


India and ASEAN can potentially lead to:
1. Increased
employment
opportunities in both nations
2. Violation of WTO global trading
norms
3. Higher Foreign Direct Investment
(FDI) from ASEAN into India

3. Shiwalik ranges are composed


of unconsolidated sediments
brought down by rivers from
the main Himalayan ranges
located farther north.
Which of the statements given above
is/are correct?
a.
b.
c.
d.

Select the correct answer using the codes


given below
a.
b.
c.
d.

1 and 3
1 and 2
2 and 3
Only 1

32. With reference to river Ganga, consider


the following statements
1. The Bhagirathi-Hooghly river is a
major tributary of river Ganga that
flows southwards through the
deltaic plains to the Bay of Bengal.
2. The headwaters of the Ganga,
called the Bhagirathi is fed by the
Gangotri Glacier and joined by the
Alaknanda at Devaprayag in
Uttaranchal.

30. An increase in natural gas prices in


India as suggested by the Rangrajan
formula can directly affect which of the
following sectors:
1.
2.
3.
4.

Manufacturing
Agriculture
Power
Mining and Exploration of Natural
Gas
5. Services
Select the correct answer using the codes
given below
a.
b.
c.
d.

3 and 4
1, 2 and 5
2, 3 and 4
1, 2, 3 and 4

31. Consider the following statements with


respect to the Himalayas
1. The northern most range is
known as the Great or Inner
Himalayas or the Himadri.
2. The core of Great Himalayas is
composed of sedimentary rocks.

http://insightsonindia.com

2 Only
2 and 3 Only
1 and 3 Only
3 Only

Which of the statements given above


is/are correct?
a.
b.
c.
d.

1 Only
2 Only
Both
None

33. Consider the following statements


about El Nino
1. It is a cold ocean current that flows
past the Peruvian Coast, in place of
the warm Peruvian current, every 2
to 5 years.
2. El Nino event affects the onset of
Monsoon
3. The presence of the El Nino leads
to a decrease in sea-surface
temperatures and weakening of the
trade winds in the region
Page 9

Facebook Group: Indian Administrative Service (Raz Kr)

Which of the statements given above


is/are correct?
a.
b.
c.
d.

1. Ground level ozone is a


major pollutant and is a
constituent of smog
2. Photochemical smog is
considered to be a problem
of modern industrialization
3. The smog created as a result
of a volcanic eruption is
often known as vog to
distinguish it as a natural
occurrence.

1 and 2 Only
2 and 3 Only
2 Only
3 Only

34. Consider the following statements


about The Battle of Plassey,
1. Robert Clive led the Companys
army against Sirajuddaulah at
Plassey.
2. Clive had managed to secure Mir
jafars support by promising to
make him nawab after crushing
Sirajuddaulah.
Which of the above statement is/ are
correct?
a.
b.
c.
d.

1 only
2 only
Both of Them
None of them

35. Consider the following statements


about indigo cultivation.
1. There were two main systems of
indigo cultivation nij and ryoti
2. Under nij system, farmers found it
easy to cultivate and expand their
cultivation

Which of the statements given above


is/are correct?
a.
b.
c.
d.

1 and 3 Only
2 and 3 Only
All Three
3 Only

37. Consider the following instances with


regard to exercise of a fundamental right?
1. Men
and
women
under
MGNERGA program get the same
salary
2. Parents property is inherited by
their children
3. Workers from Bihar go to the
Maharashtra in search of job
4. Christian missions set up a chain of
missionary schools and allure other
persons to convert to Christianity.
Which of the statements is/are correct?

Which of the above statement is/ are


correct?
a.
b.
c.
d.

only
only
All of Them
None of them

36. Consider the following statements


about smog

a.
b.
c.
d.

2 Only
1 and 3
2 and 4
3 Only

38. The foreign investments in the


National Manufacturing and Investment
zones (NMIZs) may
1. Increase Indias current account
deficit (CAD)

http://insightsonindia.com

Page 10

Facebook Group: Indian Administrative Service (Raz Kr)

2. Supplement domestic expertise and


resources
3. Result into de-congestion of nearby
urban areas
Select the correct answer using the codes
given below
a.
b.
c.
d.

2 and 3
Only 2
1 and 2
All of the above

39. Preferential market access (PMA)


policy adopted by India in the National
Electronics
Policy
2012
violates
international trade norms because:
1. Foreign Suppliers are not treated at
par with domestic suppliers
2. India will stop importing sensitive
electronic equipments for securityrelated reasons
3. WTO does not allow any
discrimination between foreign
goods from different countries

a.
b.
c.
d.

41. Consider the following:


1. Freedom to start a movement to
change the government
2. Freedom to oppose torture in
prisons.
3. Freedom
to
criticise
the
government on its policies.
4. Freedom to paint which would
incite others to rebel against the
government.
Which of the above is not available to
Indian citizens?
a.
b.
c.
d.

1. A change in temperature of
water
2. The removal of desirable
substances from waterbodies.
3. The addition of undesirable
substances to water-bodies.

1 and 2
1 and 3
2 only
Only 1

40. Consider the following statements


about The Munro system
1. Developed by Thomas Munro, this
system was gradually extended all
over south India.
2. British should act as paternal father
figures protecting the ryots under
their charge
Which of the above statement is/ are
correct?
http://insightsonindia.com

2 Only
2 and 3
3 and 4
4 Only

42. We use the term water-pollution to


cover which of the following effects?

Select the correct answer using the codes


given below
a.
b.
c.
d.

1 Only
2 Only
Both
None

Answer the question using the codes given


below
a.
b.
c.
d.

3 Only
2 and 3 Only
All Three
None

43. Consider the following statements


1. Atmospheric
nitrogen
is
converted to simpler forms
Page 11

Facebook Group: Indian Administrative Service (Raz Kr)

only by nitrogen fixing bacteria


and not by any other method
2. Nitrogen is essential in the
formation of carbohydrates and
fats in human bodies and plants
Which of the statements given above
is/are correct?
a.
b.
c.
d.

1 Only
2 Only
Both
None

44. With reference to various weather


phenomena occurring during hot weather
season in Northern part of India, consider
the following statements
1. Loo are strong, gusty, hot, dry
winds blowing during the day over
the north and northwestern India
2. Dust storms are very common
during the month of May in
northern India. These storms bring
temporary relief as they lower the
temperature and may bring light
rain and cool breeze.
3. Kaal Baisakhi or calamity for the
month of Baisakh, is a localised
thunderstorm, associated with
violent
winds,
torrential
downpours, often accompanied by
hail.
Which of the statements given above
is/are correct?
a.
b.
c.
d.

1 Only
1 and 3 Only
2 Only
All Three

http://insightsonindia.com

45. Which of the following statements


violates the Right to Freedom guaranteed
by the Constitution of India?
1. Indian citizens need permission to
visit some border areas of the
country.
2. Outsiders are not allowed to buy
property in some areas to protect
the interest of the local population.
3. The
government
bans
the
publication of a book that can go
against the ruling party in the next
elections.
Select the correct answer using the codes
given below
a.
b.
c.
d.

1 and 3
2 Only
3 Only
2 and 3

46. Which of the following statements


is/are correct with respect to the
Constitution of India?
1. It generates a degree of trust
among people of India
2. It limits the powers of the
government
3. It
determines
how
a
government should govern the
country
4. It determines the powers of the
government
Select the correct answer using the codes
given below
a.
b.
c.
d.

1 and 4
1, 3, and 4
1, 2 and 4
1, 2, 3 and 4

Page 12

Facebook Group: Indian Administrative Service (Raz Kr)

47. With reference to hybridization in


crops, consider the following statements
1. Only way of incorporating desirable
characters into crop varieties is by
hybridization.
2. Hybridisation refers to crossing
between genetically dissimilar
plants.
3. This crossing may be intervarietal
(between
different
varieties),
interspecific
(between
two
different species of the same genus)
or intergeneric (between different
genera).
Which of the statements above is/are
incorrect?
a.
b.
c.
d.

1 Only
2 and 3 Only
1 and 2 Only
None

48. With reference to the human eye


structure,
consider
the
following
statements
1. The iris is
the transparent front
part
of
the eye that
covers
the cornea, pupil, and anterior
chamber.
2. The cornea is a thin, circular
structure in the eye, responsible for
controlling the diameter and size of
the pupil and thus the amount of
light reaching the retina.
3. Behind the pupil of the eye is a lens
which is thicker in the centre. The
lens focuses light on the back of the
eye, on a layer called retina. Retina
contains several nerve cells.
Sensations felt by the nerve cells
are then transmitted to the brain
through the optic nerve.

http://insightsonindia.com

4. The pupil is a hole located in the


center of the iris of the eye that
allows light to enter the retina
Which of the statements above is/are
correct?
a.
b.
c.
d.

1,2 and 3 Only


2 and 3 Only
3 and 4 Only
None

49. The shaggy horned wild ox, the bharal


(blue sheep), wild sheep, and the kiang are
mainly found in which of the following
regions?
a.
b.
c.
d.

Arunachal Pradesh
Sikkim
Ladakh
Assam

50. Which of the following are biosphere


reserves in India?
1.
2.
3.
4.
5.

Sunderbans
Simlipal
Gulf of Mannar
Dihang-Dibang
Agasthyamalai

Answer the question using the codes given


below
a)
b)
c)
d)

1,2 and 4 Only


2,3 and 4 Only
3,4 and 5 Only
1,2,3,4 and 5

51. Which of the following statements is


correct?
a. Jet stream, a narrow belt of high
altitude (above 12,000 m) westerly
winds in the troposphere plays an
important role in causing Monsoon
rains.

Page 13

Facebook Group: Indian Administrative Service (Raz Kr)

b. The Coriolis force is responsible for


deflecting winds towards the left in
the northern hemisphere and
towards the right in the southern
hemisphere.
c. Western disturbance is nothing but
jet stream
d. None of the above.

a.
b.
c.
d.

54. Indias Constitution was drawn up


under very difficult circumstances. With
reference to the making of the constitution
of India, consider the following statements.

52. Consider the following statements


about National River Conservation Plan
(NRCP)

1. In 1928, Motilal Nehru and


other Congress leaders drafted a
Constitution for India.
2. In 1931, the resolution at
Lahore Session of the Indian
Nation Congress dwelt on how
independent Indias constitution
should look like.
3. Both
the
documents
(mentioned
above)
were
committed to the inclusion of
universal adult franchise and
protection
of
rights
of
minorities.

1. It is a wholly Centrally sponsored


Scheme
2. NRCP
essentially
addresses
pollution load from sewer systems
of town and cities and works
towards intercepting and diverting
sewage,
setting
up
electric
crematoria, low-cost sanitation,
river
front
development,
afforestation and solid waste
management.

Which of the statement is/are


incorrect?

Which of the statements given above


is/are correct?
1 Only
b. 2 Only
c. Both
d. None

a.
b.
c.
d.

a.

53. Which of the following recent factors


have caused ballooning of Indias trade
deficit with China?
1. Low demand of iron-ore in China
2. Indian ban on power equipments
from China
3. Protectionist measures adopted by
China
4. Eurozone economic slowdown
Select the correct answer using the codes
given below

1 Only
2 Only
2 and 3 Only
1 and 3 Only

55. Consider the following statements


regarding water pollution in India
1. Discharge of untreated sewage is
the single most important cause for
pollution of surface and ground
water in India.
2. A water sample with a 5 day BOD
greater than 20 mg O/L indicates
ecologically-unsafe polluted water
Which of the statements given above
is/are correct?
a.
b.

http://insightsonindia.com

3 and 4
1 and 2
1,2,3 and 4
1 and 3

1 Only
2 Only
Page 14

Facebook Group: Indian Administrative Service (Raz Kr)

c.
d.

Both
None

58. Consider the following statements with


reference to manure

56. With regard to making of the


constitution, consider the following
statements:
1. Our leaders were inspired by
socialist revolution in French
2. Our leaders were inspired by
the parliamentary democracy in
U.S.
3. The
Indian
Constitution
adopted many institutional
details and procedures from
Government of India Act, 1935
4. Much of the consensus
regarding the democratic setup
had evolved during the freedom
struggle
Which of the statement is/are correct?
a.
b.
c.
d.

1 and 3
3 and 4
1, 3 and 4
1, 2 and 4

57. Which of the following statements is


incorrect?
a. Organic farming is a farming system
with minimal or no use of
chemicals as fertilizers, herbicides,
pesticides etc. and with a
maximum input of organic
manures, recycled farm wastes, and
bio-agents, with healthy cropping
systems.
b. Mixed farming is a system of
farming on a particular farm which
includes crop production, rising of
livestock etc.
c. Composite fish culture system is
commonly used for fish farming.
d. None of the above

http://insightsonindia.com

1. Compost which is prepared by


using earthworms to hasten the
process of decomposition of plant
and animal refuse is called as
vermin-compost.
2. Some plants like sun hemp or guar
are grown and then mulched by
ploughing them into the soil which
becomes compost enriching the soil
in nitrogen and phosphorus.
3. The process in which farm waste
material like livestock excreta (cow
dung etc.), vegetable waste, animal
refuse, domestic waste, sewage
waste, straw, eradicated weeds etc.
is decomposed in pits is known as
green manuring.
Which of the statements given above
is/are correct?
a.
b.
c.
d.

1 Only
1 and 3 Only
2 and 3 Only
All Three

59. Consider the following statements


1. The elements of weather and
climate are the same, i.e.
temperature, atmospheric pressure,
wind, humidity and precipitation.
2. Weather refers to the state of the
atmosphere over an area at any
point of time.
3. Climate refers to the sum total of
weather conditions and variations
over a large area for a long period
of time (more than thirty years)
Which of the statements given above
is/are correct?
Page 15

Facebook Group: Indian Administrative Service (Raz Kr)

a.
b.
c.
d.

1 and 2 Only
1 and 3 Only
2 and 3 Only
All Three

60. Consider the following statements with


reference to the Preamble of the
Constitution?
1. Taking inspiration from the
American model, India has chosen
to begin its constitution with a
preamble.
2. Values that inspired and guided the
freedom struggle are embedded in
the Preamble of the Indian
Constitution.
3. It is the soul of the Indian
Constitution.
4. It provides a standard to examine
and evaluate any law and action of
government.
Which of the statement is/are correct?
a) 2 and 4
b) 2 and 3
c) 2, 3 and 4
d) 1, 2, 3 and 4
61. With reference to Ozone, consider the
following statements
1. Unlike the normal diatomic
molecule of oxygen, ozone is
poisonous.
2. Ozone is a powerful oxidant (far
more so than dioxygen).This makes
ozone a potent respiratory hazard
and pollutant near ground level.
Which of the statements given above
is/are correct?
a.
b.
c.
d.

1 Only
2 Only
Both
None

http://insightsonindia.com

62. India is growing slowly compared to


the previous decade. Economic growth of
India
may
be
revived
by
controlling/reducing/reforming which of
the following:
1.
2.
3.
4.
5.
6.

Corruption
Money-Laundering
Law and order situation
Red-tapism in the bureaucracy
Medical expenses by citizens
Gross Enrolment ratio (GER)

Select the correct answer using the codes


given below
a.
b.
c.
d.

1,2 and 4
1,2,3,4,5 and 6
5 and 6 only
3, 5 and 6

63. Which of the following is true about


Double Tax avoidance agreements
(DTAAs)?
1. It always lowers the annual gross
tax revenue generated in the host
country.
2. It is in the interest of investing
individuals/companies.
3. May encourage round-tripping of
money.
Select the correct answer using the below
given codes
a. Only 1
b. 2 and 3
c. Only 3
d. All of the above
64. With reference to climate, consider the
following statements
1. The sea exerts a moderating
influence on climate: As the
Page 16

Facebook Group: Indian Administrative Service (Raz Kr)

distance from the sea increases, its


moderating influence decreases and
the people experience extreme
weather conditions. This condition
is known as continentality.
2. Ocean currents along with onshore
winds affect the climate of the
coastal areas,
3. The pressure and wind system of
any area depend on the latitude and
altitude of the place. Thus it
influences the temperature and
rainfall pattern.

d. 1, 2, 3 and 4
66. With recent growing attractions to
Bitcoins across the globe. Consider the
following statements:
1. Bitcoins are illegal in India
2. Bitcoins can be used for illegal
activities.
3. Bitcoins have low transaction cost
4. Bitcoins are Non-Volatile
Which of the following statements is/are
correct?
a.
b.
c.
d.

Which of the statements given above


is/are incorrect?
a.
b.
c.
d.

1 Only
2 Only
3 Only
None

65. Constant rise in prices of goods and


services, has led to erosion of real wages of
workers. So the Government of India had
come up with a new Consumer Price
Index (CPI).With respect to the new
Consumer Price Index, consider the
following statements:
1. Consumer Price Index is released every
month
2. The Common base year for CPI is 2012
3. Consumer Price Index represents the
wholesale price of goods paid by the
households for a basket of goods and
services.
4. More weightage is given to the
manufacturing items under CPI
5. Housing forms one of the five major
groups for which Consumer Price
Index is available for both rural and
urban areas.
Which of the statements is/are correct?
a. 1 Only
b. 1, 3 and 5
c. 2, 4 and 5
http://insightsonindia.com

1 and 2
2 and 3
1, 2 and 3
1, 2, 3 and 4

67. Consider the following statements


about The Doctrine of Lapse
1. This was the final wave of
annexations, occurred under Lord
Dalhousie who was the GovernorGeneral from 1848 to 1856.
2. The doctrine declared that if an
Indian ruler died without a male
heir his kingdom would lapse,
that is, becomes part of neighboring
territory.
Which of the above statement is/ are
correct?
a.
b.
c.
d.

1 Only
2 Only
Both
None

68. Consider the following statements


about Western Disturbances
1. Western Disturbance causes winter
and pre monsoon season rainfall
across northwest India.
2. Over the Indo-Gangetic plains, it
brings cold wave conditions and
Page 17

Facebook Group: Indian Administrative Service (Raz Kr)

occasionally dense fog and cold day


conditions
3. This
is
a
nonmonsoonal precipitation
pattern
driven by the Easterlies.
Which of the statements given above
is/are correct?
a.
b.
c.
d.

1 Only
1 and 2 Only
2 and 3 Only
3 Only

69. What do you understand by the term Hot


Money?
a. Currency which has high value in
international market
b. Currency whose value increases rapidly
c. Currency whose value decreases
rapidly
d. Currency which has tendency to fly
out of the country
70. Recently six major banks made
currency swap pacts permanent. Which of
the following are correct with regard to
currency swap?
1. It helps in increasing liquidity
2. They are considered to be loans
provided by one country to another
3. They are risk free
4. India is yet to have any currency swap
arrangements with other countries.
Select the correct answer using the below
codes
a) 1 Only
b) 1 and 2
c) 1, 2 and 3
d) 1, 2, 3 and 4
71. Consider the following statements
about lakes in India

http://insightsonindia.com

1. The Wular lake in Jammu and


Kashmir is of glacier origin
2. The Chilika Lake, the Pulicat Lake,
the Kolleru Lake are lagoons.
Which of the statements above is/are
correct?
a.
b.
c.
d.

1 Only
2 Only
Both
None

72. Which of the following are the steps


taken by RBI to counter inflation?
1. Increasing interest rates
2. Introduction of Inflation-Indexed
National Saving Securities (IINSSs)
for retail investors
3. Increasing excise duty on gold
4. Permitting FDI in multi-band retail
trading
Answer using the given codes below
a.
b.
c.
d.

1 and 3
1 and 2
1, 3 and 4
1, 2, 3 and 4

73. Consider the following statements:


1. Leaders of the freedom movement
had an open mind about whether
the country should be democratic
or not after independence.
2. Members of the Constituent
Assembly of India held the same
views on all provisions of the
Constitution.
3. A country that has a constitution
must be a democracy.
4. Constitution cannot be amended
because it is the supreme law of a
country.

Page 18

Facebook Group: Indian Administrative Service (Raz Kr)

Which of the statements given above


is/are correct?
a.
b.
c.
d.

strongly
jointed
rocky
terrain.
3. The
dendritic
pattern
develops where the river
channel follows the slope of
the terrain
4. The radial pattern develops
when streams flow in
different directions from a
central peak or dome like
structure.

1 and 2
1 and 3
1 and 4
None

74. Consider the following statements with


reference to the Indus Water Treaty, 1960
1. Under the Treaty, the waters of
Western Rivers of the Indus river
system are allocated to India.
2. India is under obligation to let flow
the waters of the Eastern Rivers of
the Indus river system except for
the following uses: Domestic Use,
Non-consumptive use, Agricultural
use as specified, and Generation of
hydro-electric power as specified
3. India is under no obligation to
supply information of its storage
and hydroelectric projects as
specified.
4. This treaty was brokered by
International
Bank
for
Reconstruction and Development
between India and Pakistan.
Which of the statements given above
is/are correct?
a. 1, 2, and 3 Only
b. 2, 3 and 4 Only
c. 3 and 4 Only
d. 4 Only
75. With respect to river drainage patterns,
consider the following statements,
1. A trellis drainage pattern
develops where hard and
soft rocks exist parallel to
each other
2. A rectangular drainage
pattern develops on a
http://insightsonindia.com

Which of the above definitions is/are


correct?
a.
b.
c.
d.

1 and 2 Only
1,2 and 3 Only
2, 3 and 4 Only
1, 2, 3 and 4

76. In the context of India, which of these


is a provision that a democratic
constitution does not have?
1.
2.
3.
4.

Powers of the legislature


Name of the country
Powers of the head of the state
Name of the head of the state

Answer the question using the codes given


below
a.
b.
c.
d.

2 Only
4 Only
2, 3 and 4
2 and 4

77. Consider the following statements


1. The Pitli Island, an uninhabited
island in the Andaman and Nicobar
group of islands, has a bird
sanctuary.
2. Many of the Lakshadweep islands
are formed by corals and are a
major tourist attraction.

Page 19

Facebook Group: Indian Administrative Service (Raz Kr)

Answer the question using codes given


below
a.
b.
c.
d.

1 Only
2 Only
Both
None

80. With regard to elections consider the


following statements:

78. A recent CRISIL report suggested that


the quantity and quality of non-farm jobs
would shrink substantially in the coming
years despite a positive economic growth
rate in India. In such a scenario which of
the following can potentially lead to an
increase in employment levels in India?
1. Increasing the Cash Reserve Ratio
(CRR) and Interest rates
2. Relaxing stringent labour laws
3. Providing incentives to the Micro,
Small and Medium Enterprises
(MSMEs)
4. Relaxing FDI caps in Brownfield
projects in the manufacturing and
services sector
Which of the statements given above
is/are correct?
a.
b.
c.
d.

b. 2 only
c. Both
d. None

2 and 3
1 and 2
2, 3 and 4
1, 3 and 4

79. Consider the following statements


about census 2011
1. Maharashtra is the second most
populous state in India
2. The jump in percentage of literacy
from that of previous census report
was more for men than women
Which of the above statement is/ are
correct?
a. 1 only
http://insightsonindia.com

1. Some non-democratic countries


also hold elections.
2. Sometimes election is held only for
one constituency to fill the vacancy
caused by death or resignation of a
member. This is called a midelection.
3. In India we follow a system of
proportional representation for the
election of members of Lok Sabha.
4. Each constituency should have a
roughly equal population living
within it.
Which of the statements given above
is/are correct?
a. 2 and 4
b. 1 and 4
c. 1, 2 and 3
d. 1, 2, 3 and 4
81. India is divided into number of
constituency
for
electing
its
representatives. Consider the following
statements:
1. Each
Parliamentary
constituency has within it
several assembly constituencies
2. Each village or town is divided
into several wardsthat are like
constituencies.
3. Some
Parliamentary
constituencies are reserved for
people who belong to the
Scheduled
Castes
(SC),
Scheduled Tribes (ST) and
Other Backward Class (OBC)
Which of
correct?
a.
b.
c.
d.

the

statements

is/are

1 Only
1 and 3
1 and 2
1, 2 and 3
Page 20

Facebook Group: Indian Administrative Service (Raz Kr)

82. Consider the following statements


1. The Deccan Trap is a black soil
region and the numerous rivers
flowing in the region have
deposited this soil over thousands
of years
2. Barchans (a type of sand dunes)
cover larger areas of Thar Desert.
Which of the above statements given
above is/are correct?
a.
b.
c.
d.

1 Only
2 Only
Both
None

Which of the above statement is/ are


correct?
a.
b.
c.
d.

1 only
2 only
All of Them
None of them

85. Consider the following statements


about subsidiary alliance

83. Which of the following is an example


of a Forward or Future contract?
1. Mr. A pledging to sell his property
in Canada to Mr. B, at a fixed dollar
to rupee exchange rate at a fixed
price in dollars two years later from
now.
2. Mr. B pledging to sell his property
in Canada at a fixed price of 1
million Canadian dollars three years
later.
3. Mr. B pledging to sell 5 tonnes of
wheat to Mr. A at a fixed price
later in the day.
Which of the statements given above
is/are correct?
a.
b.
c.
d.

dream of ruling from Delhi was


shattered.
2. They were divided into many states
under different chiefs (sardars)
belonging to dynasties such as
Sindhia, Holkar, Gaikwad and
Bhonsle.

2 and 3 Only
1 Only
1 and 2 Only
All of the above

84. Consider the following statements


about Marathas

1. Indian rulers were not allowed to


have their independent armed
forces.
2. If the Indian rulers failed to make
the payment, then part of their
territory was taken away as penalty.
Which of the above statement is/ are
correct?
a.
b.
c.
d.

1 only
2 only
All of Them
None of them

86. Consider the following statements


1. The Garo, the Khasi and
the
Jaintia
Hills
in
Meghalaya are geologically
part of the peninsular
plateau
2. The Western Ghats are
higher than the Eastern
Ghats in elevation.
3. The Deccan Plateau is
higher in the east and slopes
gently westwards.

1. With defeat in the Third Battle of


Panipat in 1761, the Marathas
http://insightsonindia.com

Page 21

Facebook Group: Indian Administrative Service (Raz Kr)

Answer the above question using the codes


given below
a.
b.
c.
d.

1 and 2 Only
2 Only
1 and 3 Only
3 Only

87. Our Constitution entitles every citizen


to elect her/his representative and to be
elected as a representative. Which of the
following statements are correct with
respect to local government?
1. The system of reservation is
extended to other weaker sections
at the district and local level.
2. Local bodies are now reserved for
Other Backward Classes (OBC) as
well and one-fourth of the seats are
reserved in rural and urban local
bodies for women candidates.
3. 2% is reserved for disabled.
Select the correct answer using the codes
given below
a.
b.
c.
d.

1 Only
1 and 2
1, 2 and 3
None

88. Insider trading can be harmful to the


domestic economy and is illegal because,
a. It hurts the economic growth of a
nation, increases inflation and
Current account deficit (CAD)
b. It results in crash of the security
markets which are so vital for
channeling savings in the domestic
economy
c. It distorts the level-playing field
between investors in the stock
market and makes the markets
susceptible to heavy speculation
d. Speculation by individual investors
is illegal and hurts the retail
investors
http://insightsonindia.com

89. Which of the following measures was


not a change made after the 1857 revolt by
the British authorities?
a. The land and property of Muslims
was confiscated on a large scale and
they were treated with suspicion
and hostility.
b. Policies were made to protect
landlords and zamindars and give
them security of rights over their
lands.
c. The Governor-General of India was
given the title of Viceroy, that is, a
personal representative of the
Crown.
d. It was decided that the proportion
of Indian soldiers along with the
number of European soldiers would
be increased in the army.
90. India has a federal government, with
elected representatives at the central, state
and local levels. Which of the following
statements are incorrect with regard to
elections held in India?
1. In a democratic election, the list of
those who are eligible to vote is
prepared much before the election
and given to everyone. This list is
officially called the Electoral Roll
and is commonly known as the
Voters List.
2. A complete revision of the
Electoral Roll takes place every five
years.
3. Anyone who can be a voter can
also become a candidate in
elections and the candidates
minimum age should be 18 years.
Select the correct answer using the codes
given below
a.
b.
c.
d.

1 and 3
3 Only
2 Only
None
Page 22

Facebook Group: Indian Administrative Service (Raz Kr)

91. Which of the following statements is


incorrect with reference to menstruation?
a. At 45 to 50 years of age, the
menstrual cycle stops. Stoppage of
menstruation is termed menopause.
b. Menstruation occurs once in about
28 to 30 days. The first menstrual
flow begins at puberty and is
termed menarche
c. If fertilized egg does not develop in
the uterus after implantation, the
egg and the thickened lining of the
uterus along with its blood vessels
are shed off. This causes bleeding in
women
which
is
called
menstruation.
d. None of the above
92. The Micro Finance Institutions
(MFIs)(Regulation and Development) Bill,
2012 intends to regulate and develop the
MFIs sector. Who can be the potential
beneficiaries if it becomes a law?
1.
2.
3.
4.

Small and Marginal farmers


Self-Help Groups
MFIs
Local money lenders

Select the correct answer using the codes


given below
a.
b.
c.
d.

1 and 2 only
3 and 4 only
1,2 and 3
All of the above

93. Which of the following can be a form


of economic Protectionism by India?
1. Visa restriction for foreign nationals
2. Preferential market access policies
for domestic industries
3. Discouraging FII
4. Increasing custom duties on
imported goods and services
5. Increasing custom duties on
exported goods and services
Select the correct answer using the codes
below
a.
b.
c.
d.

94. The East India company found it


difficult to separate trade from politics.
Because,
1. Though the Royal Charter granted
the company the sole right to trade
with the East, it could not prevent
other European powers from
entering the Eastern markets.
2. Competition
amongst
the
European companies inevitably
pushed up the prices at which
these goods could be purchased,
and this reduced the profits that
could be earned.
3. The urge to secure markets
therefore led to fierce battles
between the trading companies
Which of the statements given above
is/are correct?
a.
b.
c.
d.

http://insightsonindia.com

1, 2 and 5
2 and 4
1, 3 and 4
1, 2 and 4

1 and 3 Only
2 and 3 Only
3 Only
All Three

Page 23

Facebook Group: Indian Administrative Service (Raz Kr)

95. With reference to the consequences of


granting Diwani of Bengal to the East India
Company,
consider
the
following
statements
1. The outflow of gold from Britain
increased after the Battle of Plassey,
and it increased more after the
assumption of Diwani
2. The new revenues could be used to
purchase cotton and silk textiles in
India, maintain Company troops,
and meet the cost of building the
Company fort and offices at
Calcutta
Which of the statements given above
is/are correct?
a.
b.
c.
d.

1 Only
2 Only
Both
None

96. Consider the following statements


about CITES
1. It lists several species of animals
and birds in which trade is
prohibited
2. It is also known as Washington
Convention
Which of the above statement is/are
correct?
a. 1 Only
b. 2 Only
c. Both
d. None
97. Which of the following statements
is/are correct?
1. To win back the loyalty of the
people, the British announced
rewards for loyal landholders would
http://insightsonindia.com

be allowed to continue to enjoy


traditional rights over their lands.
2. Those who had rebelled were told
that if they submitted to the
British, and if they had not killed
any white people, they would
remain safe and their rights and
claims to land would not be denied.
3. Nevertheless, hundreds of sepoys,
rebels, nawabs and rajas were tried
and hanged.
Answer the question using codes given
below
a.
b.
c.
d.

1 and 2 Only
2 and 3 Only
1 Only
All Three

98. Consider the following statements with


reference to regions of Northern plains of
India
1. The rivers, after descending from
the mountains deposit pebbles in a
narrow belt of about 8 to 16 km in
width lying parallel to the slopes of
the Shiwaliks. It is known as Terai.
2. South of Terai, the streams and
rivers re-emerge and create a wet,
swampy and marshy region known
as Bhabar.
3. The largest part of the northern
plain is formed of older alluvium.
They lie above the flood plains of
the rivers and present a terrace like
feature. This part is known as
Khadar.
4. The newer, younger deposits of the
flood plains are called Bhangar.
Which of the statements given above
is/are correct?
a.
b.
c.
d.

1 and 4 Only
2, 3 and 4 Only
1, 2, 3 and 4
None of the above
Page 24

Facebook Group: Indian Administrative Service (Raz Kr)

99. Which of the following pair is correctly


matched?
a) Dargah An open prayer place of
Muslims primarily meant for id
b) Khanqah The tomb of a Sufi saint
c) Idgah A sufi lodge, often used as a
rest house for travellers and a place
where
people come to discuss
spiritual matters, get the blessings
of saints, and hear sufi music
d) Cul-de-sac Street with a dead end

100.
Which of the following is/are
matched incorrectly?
1. Sarpagandha : Used to treat blood
pressure; it is found only in India.
2. Jamun : The juice from ripe fruit
is used to prepare vinegar which is
carminative and
diuretic, and
has digestive properties. The
powder of the seed is used for
controlling diabetes.
3. Arjun : The fresh juice of leaves is
a cure for earache. It is also used to
regulate blood pressure.
4. Babool : Leaves are used as a cure
for eye sores. Its gum is used as a
tonic.
Answer the above question using the codes
given below
a.
b.
c.
d.

1 and 2 Only
3 Only
4 Only
All are correct

http://insightsonindia.com

Page 25

Facebook Group: Indian Administrative Service (Raz Kr)

INSIGHTS MOCK TEST 2 - SOLUTIONS

1.
Solution: d)
Some marine fish of high economic value are also farmed in seawater. This includes finned fishes
like mullets, bhetki, and pearl spots, shellfish such as prawns. mussels and oysters as well as
seaweed. Oysters are also cultivated for the pearls they make.
In such a system, a combination of five or six fish species is used in a single fishpond. These
species are selected so that they do not compete for food among them having different types of
food habits. As a result, the food available in all the parts of the pond is used. As Catlas are
surface feeders, Rohus feed in the middle-zone of the pond, Mrigals and Common Carps are
bottom feeders, and Grass Carps feed on the weeds, together these species can use all the food in
the pond without competing with each other. This increases the fish yield from the pond.
2.
Solution (a)
Every person who wishes to contest an election has to fill a nomination form and give some
money as security deposit. Recently, a new system of declaration has been introduced on
direction from the Supreme Court. Every candidate has to make a legal declaration, giving full
details of : Serious criminal cases pending against the candidate; Details of the assets and liabilities
of the candidate and his or her family; and Education qualifications of the candidate. This
information has to be made public. This provides an opportunity to the voters to make their
decision on the basis of the information provided by the candidates.
Right to Information is a part of fundamental rights under Article 19(1) of the Constitution.
Article 19(1) says that every citizen has freedom of speech and expression. In 1976, the Supreme
Court said in the case of Raj Narain vs State of UP, that people cannot speak or express
themselves unless they know. Therefore, right to information is embedded in article 19.
Reference: Page No. 64 (Democratic Politics I)

3.
Solution: c)
On the basis of their development and use resources can be classified into two groups, actual
resources and potential resources.
Actual resources are those resources whose quantity is known. These resources are being used in
the present. The rich deposits of coal in Ruhr region of Germany and petroleum in the West
Asia, the dark soils of the Deccan plateau in Maharashtra are all actual resources.
http://insightsonindia.com INSIGHTS

Page 1

Facebook Group: Indian Administrative Service (Raz Kr)

INSIGHTS MOCK TEST 2 - SOLUTIONS


Potential resources are those whose entire quantity may not be known and these are not being
used at present. These resources could be used in the future. The level of technology we have at
present may not be advanced enough to easily utilise these resources. The uranium found in
Ladakh is an example of potential resource that could be used in the future. High speed winds
were a potential resource two hundred years ago. Today they are an actual resource and wind
farms generate energy using windmills like in Netherlands. You will find some in Nagercoil in
Tamil Nadu and on the Gujarat coast.

4.
Solution: c)
Lord Hastings, Governor General of India between 1813 - 1823, initiated a new policy of
paramountcy. Now the Company claimed that its authority was paramount or supreme, hence
its power was greater than that of Indian states. In order to protect its interests
it was justified in annexing or threatening to annex any Indian kingdom. This view continued to
guide later British policies as well.
This process, however, did not go unchallenged. For example, when the British tried to annex the
small state of Kitoor (in Karnataka today), Rani Channamma took to arms and led an anti-British
resistance movement. She was arrested in 1824 and died in prison in 1829. But Rayanna, a poor
chowkidar of Sangoli in Kitoor, carried on the resistance. With popular support he destroyed
many British camps and records. He was caught and hanged by the British in 1830.
5.
Solution (c)
The Government of India had appointed the Second Backward Classes Commission in 1979. It
was headed by B.P. Mandal. Hence it was popularly called the Mandal Commission. It was asked
to determine the criteria to identify the socially and educationally backward classes in India and
not minorities.
It recommended steps to be taken for their advancement. The Commission gave its Report in
1980 and made many recommendations. One of these was that 27 per cent of government jobs be
reserved for the socially and educationally backward classes.
Some persons and associations opposed to this order filed a number of cases in the courts. They
appealed to the courts to declare the order invalid and stop its implementation. The Supreme
Court of India bunched all these cases together. This case was known as the Indira Sawhney and
others Vs Union of India case. Eleven judges of the Supreme Court heard arguments of both
sides. By a majority, the Supreme Court judges in 1992 declared that this order of the

http://insightsonindia.com INSIGHTS

Page 2

Facebook Group: Indian Administrative Service (Raz Kr)

INSIGHTS MOCK TEST 2 - SOLUTIONS


Government of India was valid. It declared that well-to do persons among the backward classes
should be excluded from getting the benefit of reservation.
Reference: Page No. 80, 81 (Democratic Politics I)
6.
Solution: c
In 1817, James Mill, a Scottish economist and political philosopher, published a massive threevolume work, A History of British India. In this he divided Indian history into three periods
Hindu, Muslim and British. This periodisation came to be widely accepted. Mill thought that all
Asian societies were at a lower level of civilisation than Europe. According to his telling of
history, before the British came to India, Hindu and Muslim despots ruled the country.
Religious intolerance, caste taboos and superstitious practices dominated social life. British rule,
Mill felt, could civilise India. To do this it was necessary to introduce European manners, arts,
institutions and laws in India. Mill, in fact, suggested that the British should conquer all the
territories in India to ensure the enlightenment and happiness of the Indian people. For India was
not capable of progress without British help.
7.
Solution: c)
Mixed cropping is growing two or more crops simultaneously on the same piece of land, for
example, wheat + gram, or wheat + mustard, or groundnut + sunflower. This reduces risk and
gives some insurance against failure of one of the crops.
Inter-cropping is growing two or more crops simultaneously on the same field in a definite
pattern. A few rows of one crop alternate with a few rows of a second crop, for example,
soyabean + maize, or finger millet (bajra) + cowpea (lobia). The crops are selected such that
their nutrient requirements are different. This ensures maximum utilization of the nutrients
supplied, and also prevents pests and diseases from spreading to all the plants belonging to one
crop in a field. This way, both crops can give better returns.
The growing of different crops on a piece of land in a pre-planned succession is known as crop
rotation. Depending upon the duration, crop rotation is done for different crop combinations.
The availability of moisture and irrigation facilities decide the choice of the crop to be cultivated
after one harvest. If crop rotation is done properly then two or three crops can be grown in a year
with good harvests.

http://insightsonindia.com INSIGHTS

Page 3

Facebook Group: Indian Administrative Service (Raz Kr)

INSIGHTS MOCK TEST 2 - SOLUTIONS


8.
Solution: d)
Cholera is caused by bacteria whereas all other diseases are caused by virus.
9.
Solution (c)
Parliament is the final authority for making laws in any country. This task of law making or
legislation is so crucial that these assemblies are called legislatures. Parliaments all over the world
can make new laws, change existing laws, or abolish existing laws and make new ones in their
place.
Parliaments all over the world exercise some control over those who run the government. In
some countries like India this control is direct and full. Those who run the government can take
decisions only so long as they enjoy support of the Parliament.
Parliament is the highest forum of discussion and debate on public issues and national policy in
any country. Parliament can seek information about any matter but it does not enjoy absolute
power unlike Britain. The Parliament of India is bound by the provisions of the Constitution.
Reference: Page No. 83, 84 (Democratic Politics I)

10.
Solution (b)
Rajya Sabha some special powers over the states. But on most matters, the Lok Sabha exercises
supreme power. Any ordinary law needs to be passed by both the Houses. But if there is a
difference between the two Houses, the final decision is taken in a joint session in which
members of both the Houses sit together.
Lok Sabha exercises more powers in money matters. Once the Lok Sabha passes the budget of
the government or any other money related law, the Rajya Sabha cannot reject it. The Rajya
Sabha can only delay it by 14 days or suggest changes in it. The Lok Sabha may or may not accept
these changes.
Most importantly, the Lok Sabha controls the Council of Ministers. Only a person who enjoys the
support of the majority of the members in the Lok Sabha is appointed the Prime Minister. If the
majority of the Lok Sabha members say they have no confidence in the Council of Ministers, all
ministers including the Prime Minister, have to quit. The Rajya Sabha does not have this power.
Reference: Page No. 84, 85 (Democratic Politics I)

http://insightsonindia.com INSIGHTS

Page 4

Facebook Group: Indian Administrative Service (Raz Kr)

INSIGHTS MOCK TEST 2 - SOLUTIONS

11.
Solution: c)
The population pyramid of a country in which birth and death rates both are high is broad at the
base and rapidly narrows towards the top. This is because although, many children are born, a
large percentage of them die in their infancy, relatively few become adults and there are very few
old people. This situation is typified by the pyramid shown for Kenya.
In countries where death rates (especially amongst the very young) are decreasing, the pyramid is
broad in the younger age groups, because more infants survive to adulthood. This can be seen in
the pyramid for India. Such populations contain a relatively large number of young people and
which means a strong and expanding labour force.
12.

Solution: d)
The oldest landmass, (the Peninsula part), was a part of the Gondwana land. The Gondwanaland
included India, Australia, South Africa and South America as one single land mass. The
convectional currents split the crust into a number of pieces, thus leading to the drifting of the
Indo-Australian plate after being separated from the Gondwana land, towards north. The
northward drift resulted in the collision of the plate with the much larger Eurasian Plate. Due to
this collision, the sedimentary rocks which were accumulated in the geosyncline known as the
Tethys were folded to form the mountain system of western Asia and Himalayas.
The Himalayan uplift out of the Tethys sea and subsidence of the northern flank of the
peninsular plateau resulted in the formation of a large basin. In due course of time this depression,
gradually got filled with deposition of sediments by the rivers flowing from the mountains in the
north and the peninsular plateau in the south.
13.

Solution: d)
In 1600, the East India Company acquired a charter from the ruler of England, Queen Elizabeth I,
granting it the sole right to trade with the East. This meant that no other trading group in England
could compete with the East India Company. With this charter the Company could venture
across the oceans, looking for new lands from which it could buy goods at a cheap price, and
carry them back to Europe to sell at higher prices. The Company did not have to fear
competition from other English trading companies. Mercantile trading companies in those days
made profit primarily by excluding competition, so that they could buy cheap and sell dear. The
royal charter, however, could not prevent other European powers from entering the Eastern
markets.
http://insightsonindia.com INSIGHTS

Page 5

Facebook Group: Indian Administrative Service (Raz Kr)

INSIGHTS MOCK TEST 2 - SOLUTIONS


By the time the first English ships sailed down the west coast of Africa, round the Cape of Good
Hope, and crossed the Indian Ocean, the Portuguese had already established their presence in the
western coast of India, and had their base in Goa. In fact, it was Vasco da Gama, a Portuguese
explorer, who had discovered this sea route to India in 1498. By the early seventeenth century,
the Dutch too were exploring the possibilities of trade in the Indian Ocean. Soon the French
traders arrived on the scene. The problem was that all the companies were interested in buying
the same things. The fine qualities of cotton and silk produced in India had a big market in
Europe. Pepper, cloves, cardamom and cinnamon too were in great demand. Competition
amongst the European companies inevitably pushed up the prices at which these goods could be
purchased, and this reduced the profits that could be earned. The only way the trading companies
could flourish was by eliminating rival competitors. The urge to secure markets therefore led to
fierce battles between the trading companies. Through the seventeenth and eighteenth centuries
they regularly sank each others ships, blockaded routes, and prevented rival ships from moving
with supplies of goods. Trade was carried on with arms and trading posts were protected through
fortification. This effort to fortify settlements and carry on profitable trade also led to intense
conflict with local rulers. The company therefore found it difficult to separate trade from politics.
14.
Solution: c)

The first English factory was set up on the banks of the river Hugli in 1651. This was the base
from which the Companys traders, known at that time as factors, operated. The factory had a
warehouse where goods for export were stored, and it had offices where Company officials sat.
As trade expanded, the Company persuaded merchants and traders to come and settle near the
factory.
15.
Solution: c)
For fires involving electrical equipment and inflammable materials like petrol, carbon dioxide
(CO2) is the best extinguisher. CO2, being heavier than oxygen, covers the fire like a blanket.
Since the contact between the fuel and oxygen is cut off, the fire is controlled. The added
advantage of CO2 is that in most cases it does not harm the electrical equipment.
How do we get the supply of carbon dioxide? It can be stored at high pressure as a liquid in
cylinders. In what form is the LPG stored in cylinders? When released from the cylinder, CO2
expands enormously in volume and cools down. So, it not only forms a blanket around the fire, it
also brings down the temperature of the fuel. That is why it is an excellent fire extinguisher.
Another way to get CO2 is to release a lot of dry powder of chemicals like sodium bicarbonate
(baking soda) or potassium bicarbonate. Near the fire, these chemicals give off CO2.

http://insightsonindia.com INSIGHTS

Page 6

Facebook Group: Indian Administrative Service (Raz Kr)

INSIGHTS MOCK TEST 2 - SOLUTIONS


16.
Solution: d)
http://en.wikipedia.org/wiki/Deforestation#Environmental_problems

17.
Solution: a)
During the process of cloning Dolly, a cell was collected from the mammary gland of a female
Finn Dorsett sheep . Simultaneously, an egg was obtained from a Scottish blackface ewe. The
nucleus was removed from the egg. Then, the nucleus of the mammary gland cell from the Finn
Dorsett sheep was inserted into the egg of the Scottish blackface ewe whose nucleus had been
removed. The egg thus produced was implanted into the Scottish blackface ewe. Development of
this egg followed normally and finally Dolly was born.
Though Dolly was given birth by the Scottish blackface ewe, it was found to be absolutely
identical to the Finn Dorsett sheep from which the nucleus was taken. Since the nucleus from
the egg of the Scottish blackface ewe was removed, Dolly did not show any character of the
Scottish blackface ewe. Dolly was a healthy clone of the Finn Dorsett sheep and produced several
offspring of her own through normal sexual means. Unfortunately, Dolly died on 14th February
2003 due to a certain lung disease.

18.

Solution (a)
At different levels of any government we find functionaries who take day-to-day decisions but
do not exercise supreme power on behalf of the people. All those functionaries are collectively
known as the executive. Thus, when we talk about the government we usually mean the
executive.
In a democratic country, two categories make up the executive. One that is elected by the people
for a specific period, is called the political executive (temporary executive). Political leaders who
take the big decisions fall in this category. In the second category, people are appointed on a longterm basis. This is called the permanent executive or civil services.They remain in office even
when the ruling party changes.These officers work under political executive and assist them in
carrying out the day-to-day administration. Political executive have more power than the nonpolitical executive
In a democracy the will of the people is supreme. The minister is elected by the people and thus
empowered to exercise the will of the people on their behalf. She is finally answerable to the
people for all the consequences of her decision. That is why the minister takes all the final
http://insightsonindia.com INSIGHTS

Page 7

Facebook Group: Indian Administrative Service (Raz Kr)

INSIGHTS MOCK TEST 2 - SOLUTIONS


decisions. The minister decides the overall framework and objectives in which decisions on policy
should be made. The minister is not, and is not expected to be, an expert in the matters of her
ministry. The minister takes the advice of experts on all technical matters.
Reference: Page No. 86 (Democratic Politics I)
19.

Solution (c )
The President appoints the Prime Minister, no direct election to the Prime Minister. But the
President cannot appoint anyone she likes. The President appoints the leader of the majority
party or the coalition of parties that commands a majority in the Lok Sabha, as Prime Minister. In
case no single party or alliance gets a majority, the President appoints the person most likely to
secure a majority support.
The Prime Minister does not have a fixed tenure. He continues in power so long as he remains
the leader of the majority party or coalition. Sometimes, a person who is not a member of
Parliament can also become a minister. But such a person has to get elected to one of the Houses
of the Parliament within six months of appointment as minister.
When the Prime Minister quits, the entire ministry quits.
In recent years the rise of coalition politics has imposed certain constraints on the power of
the Prime Minister. The Prime Minister of a coalition government cannot take decisions as he
likes. He has to accommodate different groups and factions in his party as well as among alliance
partners. He also has to heed to the views and positions of the coalition partners and other
parties, on whose support the survival of the government depends.
Reference: Page No. 87, 88, 89 (Democratic Politics I)
20.

Solution: d)
http://articles.economictimes.indiatimes.com/2014-01-16/news/46264332_1_financial-inclusion-nachiketmor-bank-account
http://www.livemint.com/Opinion/VPzOA6NoPIXCynO9smJBiL/A-failed-attempt-at-financialinclusion.html
21.

Solution: d)
National Population Policy
Recognising that the planning of families would improve individual health and welfare, the
Government of India initiated the comprehensive Family Planning Programme in 1952. The
http://insightsonindia.com INSIGHTS

Page 8

Facebook Group: Indian Administrative Service (Raz Kr)

INSIGHTS MOCK TEST 2 - SOLUTIONS


Family Welfare Programme has sought to promote responsible and planned parenthood on a
voluntary basis. The National Population Policy 2000 is a culmination of years of planned efforts.
The NPP 2000 provides a policy framework for imparting free and compulsory school education
up to 14 years of age, reducing infant mortality rate to below 30 per 1000 live births, achieving
universal immunisation of children against all vaccine preventable diseases, promoting delayed
marriage for girls, and making family welfare a people-centered programme.
NPP 2000 and Adolescents
NPP 2000 identified adolescents as one of the major sections of the population that need greater
attention. Besides nutritional requirements, the policy put greater emphasis on other important
needs of adolescents including protection from unwanted pregnancies and sexually transmitted
diseases (STD). It called for programmes that aim towards encouraging delayed marriage and
child-bearing, education of adolescents about the risks of unprotected sex, making contraceptive
services accessible and affordable, providing food supplements, nutritional services, strengthening
legal measures to prevent child marriage.
22.

Solution: c
The Company tried continuously to press for more concessions and manipulate existing
privileges. Aurangzebs farman, for instance, had granted only the Company the right to trade
duty free. But officials of the Company, who were carrying on private trade on the side, were
expected to pay duty. This they refused to pay, causing an enormous loss of revenue for Bengal.
23.

Solution: c
http://www.prsindia.org/billtrack/the-micro-finance-institutions-development-and-regulation-bill-20122348/
24.

Solution: d)
A magnet can exert a force on another magnet without being in contact with it. The force
exerted by a magnet is an example of a non-contact force.

http://insightsonindia.com INSIGHTS

Page 9

Facebook Group: Indian Administrative Service (Raz Kr)

INSIGHTS MOCK TEST 2 - SOLUTIONS


The force exerted by a charged body on another charged or uncharged body is known as
electrostatic force. This force comes into play even when the bodies are not in contact. The
electrostatic force, therefore, is another example of a non-contact force.
The force of friction always acts on all the moving objects and its direction is always opposite to
the direction of motion. Example, A ball rolling along the ground gradually slows down and
finally comes to rest.
25.

Solution: b)
The loudness of sound depends on its amplitude. When the amplitude of vibration is large, the
sound produced is loud. When the amplitude is small, the sound produced is feeble.
The frequency determines the shrillness or pitch of a sound. If the frequency of vibration is higher
we say that the sound is shrill and has a higher pitch. If the frequency of vibration is lower, we say
that the sound has a lower pitch. For example, a drum vibrates with a low frequency. Therefore,
it produces a low-pitched sound. On the other hand, a whistle has a high frequency and therefore,
produces a sound of higher pitch.
26.

Solution (a)
Cabinet Ministers are usually top-level leaders of the ruling party or parties who are in charge of
the major ministries. Usually the Cabinet Ministers meet to take decisions in the name of the
Council of Ministers. Cabinet is thus the inner ring of the Council of Ministers.
Since it is not practical for all ministers to meet regularly and discuss everything, the decisions are
taken in Cabinet meetings. That is why parliamentary democracy in most countries is often
known as the Cabinet form of government.
Reference: Page No. 87 (Democratic Politics I)
27.

Solution: c)
During the development of a thunderstorm, the air currents move upward while the water
droplets move downward. These vigorous movements cause separation of charges. By a process,
not yet completely understood, the positive charges collect near the upper edges of the clouds
and the negative charges accumulate near the lower edges. There is accumulation of positive
charges near the ground also. When the magnitude of the accumulated charges becomes very
large, the air which is normally a poor conductor of electricity is no longer able to resist their
http://insightsonindia.com INSIGHTS

Page 10

Facebook Group: Indian Administrative Service (Raz Kr)

INSIGHTS MOCK TEST 2 - SOLUTIONS


flow. Negative and positive charges meet, producing streaks of bright light and sound. We see
streaks as lightning. The process is called an electric discharge.
Open vehicles, like motorbikes, tractors, construction machinery, open cars are not safe. Open
fields, tall trees, shelters in parks, elevated places do not protect us from lightning strokes.
Carrying umbrella is not a good idea at all during thunderstorms. If in a forest, take shelter under
shorter trees. If no shelter is available and you are
in an open field, stay far away from all trees. Stay away from poles or other metal objects. Do not
lie on the ground. Instead, squat low on the ground. Place your hands on your knees with your
head between the hands.
Inside the house
Lightning can strike telephone cords, electrical wires and metal pipes. During a thunderstorm
contact with these should be avoided. It is safer to use mobile phones and cordless phones.
However, it is not wise to call up a person who is receiving your phone through a wired phone.
Bathing should be avoided during thunderstorms to avoid contact with running water. Electrical
appliances like computers, TVs, etc., should be unplugged. Electrical lights can remain on. They
do not cause any harm.
28.

Solution (d)
Cabinet Ministers are usually top-level leaders of the ruling party or parties who are in charge of
the major ministries. Since it is not practical for all ministers to meet regularly and discuss
everything, the decisions are taken in Cabinet meetings. The cabinet works as a team. The
ministers may have different views and opinions, but everyone has to own up to every decision of
the Cabinet. No minister can openly criticise any decision of the government, even if it is about
another Ministry or Department.
Every ministry has secretaries, who are civil servants. The secretaries provide the necessary
background information to the ministers to take decisions.
29.

Solution: d)
http://www.eastasiaforum.org/2013/07/30/india-asean-fta-in-services-good-for-the-region-very-good-forindia-2/
30.

Solution: c)

http://insightsonindia.com INSIGHTS

Page 11

Facebook Group: Indian Administrative Service (Raz Kr)

INSIGHTS MOCK TEST 2 - SOLUTIONS


http://www.thehindubusinessline.com/industry-and-economy/rangarajan-panel-proposes-new-gaspricing-formula/article4264578.ece
31.

Solution: c)
The Himalaya consists of three parallel ranges in its longitudinal extent. A number of valleys lie
between these ranges. The northern most range is known as the Great or Inner Himalayas or the
Himadri. It is the most continuous range consisting of the loftiest peaks with an average height
of 6,000 metres. It contains all the prominent Himalayan peaks. The folds of Great Himalayas are
asymmetrical in nature. The core of this part of Himalayas is composed of granite. It is
perennially snow bound, and a number of glaciers descend from this range.
The range lying to the south of the Himadri forms the most rugged mountain system and is
known as Himachal or lesser Himalaya. The ranges are mainly composed of highly compressed
and altered rocks. The altitude varies between 3,700 and 4,500 metres and the average width is of
50 Km. While the Pir Panjal range forms the longest and the most important range, the Dhaula
Dhar and the Mahabharat ranges are also prominent ones. This range consists of the famous valley
of Kashmir, the Kangra and Kullu Valley in Himachal Pradesh. This region is well known for its
hill stations.
The outer most range of the Himalayas is called the Shiwaliks. They extend over a width of 10-50
Km and have an altitude varying between 900 and 1100 metres. These ranges are composed of
unconsolidated sediments brought down by rivers from the main Himalayan ranges located
farther north. These valleys are covered with thick gravel and alluvium. The longitudinal valley
lying between lesser Himalaya and the Shiwaliks are known as Duns. Dehra Dun, Kotli Dun and
Patli Dun are some of the well-known Duns.
32.

Solution: b)
Bhagirathi-Hooghly river is a distributary, not a tributary.
The headwaters of the Ganga, called the Bhagirathi is fed by the Gangotri Glacier and joined by
the Alaknanda at Devaprayag in Uttaranchal. At Haridwar the Ganga emerges from the
mountains on to the plains.
The main tributaries, which come from the peninsular uplands, are the Chambal, the Betwa and
the Son. These rise from semi arid areas, have shorter courses and do not carry much water in
them. Find out where and how they ultimately join the Ganga. Enlarged with the waters from its
right and left bank tributaries, the Ganga flows eastwards till Farakka in West Bengal. This is the
northernmost point of the Ganga delta. The river bifurcates here; the Bhagirathi-Hooghly (a
http://insightsonindia.com INSIGHTS

Page 12

Facebook Group: Indian Administrative Service (Raz Kr)

INSIGHTS MOCK TEST 2 - SOLUTIONS


distributary) flows southwards through the deltaic plains to the Bay of Bengal. The mainstream,
flows southwards into Bangladesh and is joined by the Brahmaputra. Further down stream, it is
known as the Meghna. This mighty river, with waters from the Ganga, and the Brahmaputra,
flows into the Bay of Bengal. The delta formed by these rivers is known as the Sunderban delta.
33.

Solution: c)
El Nino: This is a name given to the periodic development of a warm ocean current along the
coast of Peru as a temporary replacement of the cold Peruvian current. El Nino is a Spanish word
meaning the child, and refers to the baby Christ, as this current starts flowing during Christmas.
The presence of the El Nino leads to an increase in sea-surface temperatures and weakening of
the trade winds in the region.
The ENSO is known to have a pronounced effect on the strength of SW Monsoon over India
with the Monsoon being weak (causing droughts in India) during the El Nio years whereas La
Nia years had particularly good Monsoon strength over India
ENSO
Apart from this, it has also been noticed that changes in the pressure conditions over the southern
oceans also affect the monsoons. Normally when the tropical eastern south Pacific Ocean
experiences high pressure, the tropical eastern Indian Ocean experiences low pressure. But in
certain years, there is a reversal in the pressure conditions and the eastern Pacific has lower
pressure in comparison to the eastern Indian Ocean. This periodic change in pressure conditions is
known as the Southern Oscillation or SO. The difference in pressure over Tahiti (Pacific Ocean,
18S/149W) and Darwin in northern Australia (Indian Ocean, 1230S/131E) is computed to
predict the intensity of the monsoons. If the pressure differences were negative, it would mean
below average and late monsoons. A feature connected with the SO is the El Nino, a warm ocean
current that flows past the Peruvian Coast, in place of the cold Peruvian current, every 2 to 5
years. The changes in pressure conditions are connected to the El Nino. Hence, the phenomenon
is referred to as ENSO (El Nino Southern Oscillations).
34.

Solution: c
Pg 12: On hearing the news of the fall of Calcutta, Company officials in Madras sent forces under
the command of Robert Clive, reinforced by naval fleets. Prolonged negotiations with the Nawab
followed. Finally, in 1757, Robert Clive led the Companys army against Sirajuddaulah at Plassey.
One of the main reasons for the defeat of the Nawab was that the forces led by Mir Jafar, one of
http://insightsonindia.com INSIGHTS

Page 13

Facebook Group: Indian Administrative Service (Raz Kr)

INSIGHTS MOCK TEST 2 - SOLUTIONS


Sirajuddaulahs commanders, never fought the battle. Clive had managed to secure his support by
promising to make him nawab after crushing Sirajuddaulah. The Battle of Plassey became famous
because it was the first major victory the Company won in India.
35.

Solution : a
How was indigo cultivated?
There were two main systems of indigo cultivation nij and ryoti. Within the system of nij
cultivation, the planter produced indigo in lands that he directly controlled. He either bought the
land or rented it from other zamindars and produced indigo by directly employing hired
labourers.
The problem with nij cultivation
The planters found it difficult to expand the area under nij cultivation. Indigo could be cultivated
only on fertile lands, and these were all already densely populated. Only small plots scattered over
the landscape could be acquired. Planters needed large areas in compact blocks to cultivate indigo
in plantations. Where could they get such land from? They attempted to lease in the land around
the indigo factory, and evict the peasants from the area. But this always led to conflicts and
tension.
36.

Solution: c)
Traffic emissions such as from trucks, buses, and automobiles also contribute. Airborne byproducts from vehicle exhaust systems cause air pollution and are a major ingredient in the
creation of smog in some large cities.
The major culprits are from transportation sources are carbon monoxide (CO), nitrogen oxides
(NO and NOx), volatile organic compounds, sulfur dioxide, and hydrocarbons. These molecules
react with sunlight, heat, ammonia, moisture, and other compounds to form the noxious vapors,
ground level ozone, and particles that comprise smog.
Photochemical smog was first described in the 1950s. It is the chemical reaction of
sunlight, nitrogen oxides and volatile organic compounds in the atmosphere, which
leaves airborne particles and ground-level ozone.[14] This noxious mixture of air pollutants can
include the following:
Aldehydes
Nitrogen oxides, such as nitrogen dioxide
Peroxyacyl nitrates
Tropospheric ozone
http://insightsonindia.com INSIGHTS

Page 14

Facebook Group: Indian Administrative Service (Raz Kr)

INSIGHTS MOCK TEST 2 - SOLUTIONS


Volatile organic compounds
All of these harsh chemicals are usually highly reactive and oxidizing. Photochemical smog is
therefore considered to be a problem of modern industrialization. It is present in all modern
cities, but it is more common in cities with sunny, warm, dry climates and a large number of
motor vehicles. Because it travels with the wind, it can affect sparsely populated areas as well.
37.

Solution (c)
Men and women under MGNERGA program get the same salary- Right to Equality, the
government shall not discriminate against any citizen on grounds only of religion, race, caste, sex
or place of birth. All citizens have equality of opportunity in matters relating to employment or
appointment to any position in the government. No citizen shall be discriminated against or made
ineligible for appointment.
Parents property inherited by their children is not a fundamental right.
Workers from Bihar go to the Maharashtra in search of job comes under right to freedom. As
citizens we have the freedom to travel to any part of the country. We are free to reside and settle
in any party of the territory of India.
Christian missions can set up a chain of missionary schools (this comes under Right to Freedom of
Religion) but cannot allure other persons to convert to Christianity. Every person has a right to
profess, practice and propagate the religion he or she believes in. Every religious group or sect is
free to manage its religious affairs. A right to propagate ones religion, however, does not mean
that a person has right to compel another person to convert into his religion by means of force,
fraud, inducement or allurement. Of course, a person is free to change religion on his or her own
will.
38.

Solution: a)
http://www.gktoday.in/national-manufacturing-policy-india/
39.

Solution: d)
http://articles.economictimes.indiatimes.com/2013-07-08/news/40443725_1_pma-policy-preferentialmarket-access-policy-private-sector
40.

Solution : c)
The Munro system

http://insightsonindia.com INSIGHTS

Page 15

Facebook Group: Indian Administrative Service (Raz Kr)

INSIGHTS MOCK TEST 2 - SOLUTIONS


In the British territories in the south there was a similar move away from the idea of Permanent
Settlement. The new system that was devised came to be known as the ryotwar (or ryotwari ). It
was tried on a small scale by Captain Alexander Read in some of the areas that were taken over
by the Company after the wars with Tipu Sultan. Subsequently developed by Thomas Munro,
this system was gradually extended all over south India. Read and Munro felt that in the south
there were no traditional zamindars. The settlement, they argued, had to be made directly with
the cultivators (ryots) who had tilled the land for generations. Their fields had to be carefully and
separately surveyed before the revenue assessment was made. Munro thought that the British
should act as paternal father figures protecting the ryots under their charge.
41.

Solution (d)
Freedom of speech and expression is one of the essential features of any democracy. Our ideas
and personality develop only when we are able to freely communicate with others. You may
think differently from others. Even if a hundred people think in one way, you should have the
freedom to think differently and express your views accordingly. You may disagree with a policy
of government or activities of an association.
You are free to criticize the government or the activities of the association in your conversations
with parents, friends and relatives. You may publicise your views through a pamphlet, magazine
or newspaper. You can do it through paintings, poetry or songs. However, you cannot use this
freedom to instigate violence against others. You cannot use it to incite people to rebel against
government.
Freedom to start a movement to change the government is available. For instance, Anna Hazares
movement on corruption. The prisoners have the right to oppose torture, in this direction
National Human Rights Commission has played a major role.
42.

Solution: c)
WATER POLLUTION
Water dissolves the fertilisers and pesticides that we use on our farms. So some percentage of
these substances are washed into the water bodies. Sewage from our towns and cities and the
waste from factories are also dumped into rivers or lakes. Specific industries also use water for
cooling in various operations and later return this hot water to water-bodies. Another manner in
which the temperature of the water in rivers can be affected is when water is released from dams.
The water inside the deep reservoir would be colder than the water at the surface which gets
heated by the Sun.

http://insightsonindia.com INSIGHTS

Page 16

Facebook Group: Indian Administrative Service (Raz Kr)

INSIGHTS MOCK TEST 2 - SOLUTIONS


All this can affect the life-forms that are found in these water bodies in various ways. It can
encourage the growth of some life-forms and harm some other life-forms. This affects the balance
between various organisms which had been established in that system. So we use the term waterpollution to cover the
following effects:
1. The addition of undesirable substances to water-bodies. These substances could be the
fertilisers and pesticides used in farming or they could be poisonous substances, like mercury salts
which are used by paper-industries. These could also be disease-causing organisms, like the
bacteria which cause cholera.
2. The removal of desirable substances from water-bodies. Dissolved oxygen is used by the
animals and plants that live in water. Any change that reduces the amount of this dissolved
oxygen would adversely affect these aquatic organisms. Other nutrients could also be depleted
from the water bodies.
3. A change in temperature. Aquatic organisms are used to a certain range of temperature in the
water-body where they live, and a sudden marked change in this temperature would be
dangerous for them or affect their breeding. The eggs and larvae of various animals are
particularly susceptible to temperature changes.
43.

Solution: d)
Nitrogen gas makes up 78% of our atmosphere and nitrogen is also a part of many molecules
essential to life like proteins, nucleic acids (DNA and RNA) and some vitamins. Nitrogen is
found in other biologically important compounds such as alkaloids and urea too. Nitrogen is thus
an essential nutrient for all life-forms and life would be simple if all these life-forms could use the
atmospheric nitrogen directly. However, other than a few forms of bacteria, life-forms are not
able to convert the comparatively inert nitrogen molecule into forms like nitrates and nitrites
which can be taken up and used to make the required molecules.
Apart from bacteria, nitrogen is converted by a physical process. During lightening, the high
temperatures and pressures created in the air convert nitrogen into oxides of nitrogen. These
oxides dissolve in water to give nitric and nitrous acids and fall on land along with rain. These are
then utilized by life forms on the earth.
44.

Solution: d)
A striking feature of the hot weather season is the loo. These are strong, gusty, hot, dry winds
blowing during the day over the north and northwestern India. Sometimes they even continue
http://insightsonindia.com INSIGHTS

Page 17

Facebook Group: Indian Administrative Service (Raz Kr)

INSIGHTS MOCK TEST 2 - SOLUTIONS


until late in the evening. Direct exposure to these winds may even prove to be fatal. Dust storms
are very common during the month of May in northern India. These storms bring temporary
relief as they lower the temperature and may bring light rain and cool breeze. This is also the
season for localised thunderstorms, associated with violent winds, torrential downpours, often
accompanied by hail. In West Bengal, these storms are known as the Kaal Baisakhi calamity for
the month of Baisakh.
45.

Solution (c)
Every citizen has the right to freedom mentioned under Article 19. But one cannot exercise
his/her freedom in such amanner that it violates others right to freedom. Your freedoms should
not cause public nuisance or disorder.You are free to do everything which injures no one else.
Freedom is not unlimited license to do what one wants. Accordingly, the government can impose
certain reasonable restrictions on our freedoms in the larger interests of the society.
All citizens of India enjoy the freedom to move freely throughout the country. But under certain
circumstances, due to security reasons the freedom can restricted. For instance, entry in border
areas or army cantonment is restricted to ordinary citizens as it can be dangerous for the security
of India.
Outsiders are not allowed to buy property in some areas to protect the interest of the local
population. According to article 370, outsiders are not allowed to buy property and settle
permanently in the State of Jammu & Kashmir and also in tribal areas it is restricted inorder to
protect the ethnic and cultural identity of the tribal people.
The government banning publication of book motivated by political reasons is a violation of
freedom of expression (Article 19(1)).

46.

Solution (d)
The constitution is the supreme law of the land, it not only determines the powers of the
government but also lays down certain limitations on the powers of the government by providing
Fundamental rights to the citizens of India. It determines how a government should function and
generates a degree of trust and coordination that is necessary for different kind of people to live
together.

http://insightsonindia.com INSIGHTS

Page 18

Facebook Group: Indian Administrative Service (Raz Kr)

INSIGHTS MOCK TEST 2 - SOLUTIONS


47.

Solution: a)
Desirable characters can be introduced by genetically modifying the crops too.
One way of incorporating desirable characters into crop varieties is by hybridisation.
Hybridisation refers to crossing between genetically dissimilar plants. This crossing may be
intervarietal (between different varieties), interspecific (between two different species of the
same genus) or intergeneric (between different genera). Another way of improving the crop is by
introducing a gene that would provide the desired characteristic. This results in genetically
modified crops.
48.

Solution: c
The iris is a thin, circular structure in the eye, responsible for controlling the diameter and size of
the pupil and thus the amount of light reaching the retina.
The cornea is the transparent front part of the eye that covers the iris, pupil, and anterior
chamber. The cornea, with the anterior chamber and lens, refracts light, with the cornea
accounting for approximately two-thirds of the eye's total optical power.
The pupil is a hole located in the center of the iris of the eye that allows light to enter the retina
49.

Solution: c)
The Himalayas harbour a hardy range of animals, which survive in extreme cold. Ladhaks
freezing high altitudes are a home to yak, the shaggy horned wild ox weighing around one tonne,
the Tibetan antelope, the bharal (blue sheep), wild sheep, and the kiang (Tibetan wild ass).
Furthermore, the ibex, bear, snow-leopard and very rare red panda are found in certain pockets.
50.

Solution: d)
Reference: http://en.wikipedia.org/wiki/Biosphere_reserves_of_India

http://insightsonindia.com INSIGHTS

Page 19

Facebook Group: Indian Administrative Service (Raz Kr)

INSIGHTS MOCK TEST 2 - SOLUTIONS


51.

Solution: a)
Coriolis force: An apparent force caused by the earths rotation. The Coriolis force is responsible
for deflecting winds towards the right in the northern hemisphere and towards the left in the
southern hemisphere. This is also known as Ferrels Law.
Jet stream: These are a narrow belt of high altitude (above 12,000 m) westerly winds in the
troposphere. Their speed varies from about 110 km/h in summer to about 184 km/h in winter. A
number of separate jet streams have been identified. The most constant are the mid-latitude and
the sub tropical jet stream. These play an important role in bringing monsoon rains to India.
Western disturbances are different from Jet Streams. These are extratropical storms and are nonmonsoonal.
52.

Solution: b)
The Centrally sponsored Scheme of National River Conservation Plan (NRCP) is being
implemented by the central Government jointly with the State Government on a cost-sharing
basis. The pollution abatement works under NRCP presently cover identified polluted stretches
of 39 major rivers in 185 towns spread over 20 States in the country. The sanctioned cost of the
projects under NRCP is Rs . 8100.48 crore (including Rs.433 crore incurred under GAP-I). A
sewage treatment capacity of 4417 mld has been created under River Action Plans.
Pollution Abatement Works So far
The following pollution abatement works are taken up:
1. Interception and Diversion works to capture the raw sewage flowing into the river
through open drains and divert them for treatment . (I&D)
2. Sewage Treatment Plants for treating the diverted sewage. (STP)
3. Low Cost Sanitation works to prevent open defecation on river banks .(LCS)
4. Electric Crematoria and Improved Wood Crematoria and Improved Wood Crematoria to
conserve the use of wood and help in ensuring proper cremation of bodies brought to
the buring ghats. (EC&IWC)
5. River Front Development works such as improvement of bathing ghatsetc. (RED)
6. Other Measures like plantation, public awareness etc.

53.

Solution: d)

http://insightsonindia.com INSIGHTS

Page 20

Facebook Group: Indian Administrative Service (Raz Kr)

INSIGHTS MOCK TEST 2 - SOLUTIONS


54.

Solution (b)
As far back as in 1928, Motilal Nehru and eight other Congress leaders drafted a constitution for
India. In 1931, the resolution at the Karachi session of the Indian National Congress dwelt on
how Independent Indias constitution should look like. Both these documents were committed to
the inclusion of universal adult franchise, right to freedom and equality and to protecting the
rights of minorities in the constitution of independent India.

55.

Solution: c)
A 2007 study finds that discharge of untreated sewage is the single most important cause for
pollution of surface and ground water in India. There is a large gap between generation and
treatment of domestic waste water in India. The problem is not only that India lacks sufficient
treatment capacity but also that the sewage treatment plants that exist do not operate and are not
maintained.
In 2008, the water quality monitoring found almost all rivers with high levels of BOD. The worst
pollution, in decreasing order, were found in river Markanda (590 mg O/l), followed by river Kali
(364), river Amlakhadi (353), Yamuna canal (247), river Yamuna at Delhi (70) and river Betwa
(58). For context, a water sample with a 5 day BOD between 1 and 2 mg O/L indicates a very
clean water, 3 to 8 mg O/L indicates a moderately clean water, 8 to 20 indicates borderline water,
and greater than 20 mg O/L indicates ecologically-unsafe polluted water.
The levels of BOD are severe near the cities and major towns. In rural parts of India, the river
BOD levels were sufficient to support aquatic life.
Biochemical oxygen demand or B.O.D is the amount of dissolved oxygen needed by aerobic
biological organisms in a body of water to break down organic material present in a given water
sample at certain temperature over a specific time period. The term also refers to a chemical
procedure for determining this amount.
56.

Solution (b)
The familiarity with political institutions of colonial rule also helped develop an agreement over
the institutional design. The British rule had given voting rights only to a few. On that basis the
British had introduced very weak legislatures.
Elections were held in 1937 to Provincial Legislatures and Ministries all over British India.
These were not fully democratic governments. But the experience gained by Indians in the
working of the legislative institutions proved to be very useful for the country in setting up its
http://insightsonindia.com INSIGHTS

Page 21

Facebook Group: Indian Administrative Service (Raz Kr)

INSIGHTS MOCK TEST 2 - SOLUTIONS


own institutions and working in them. That is why the Indian constitution adopted many
institutional details and procedures from colonial laws like the Government of India Act, 1935.
Many of our leaders were inspired by the ideals of French Revolution, the practice of
parliamentary democracy in Britain and the Bill of Rights in the US. The socialist revolution in
Russia had inspired many Indians to think of shaping a system based on social and economic
equality.

57.

Solution: d)
All are correct.
58.

Solution: a)
Some plants like sun hemp or guar are grown and then mulched by ploughing them into the soil
which becomes green manure enriching the soil in nitrogen and phosphorus.
The process in which farm waste material like livestock excreta (cow dung etc.), vegetable waste,
animal refuse, domestic waste, sewage waste, straw, eradicated weeds etc. is decomposed in pits is
known as composting.
59.

Solution: d)
Climate refers to the sum total of weather conditions and variations over a large area for a long
period of time (more than thirty years). Weather refers to the state of the atmosphere over an
area at any point of time. The elements of weather and climate are the same, i.e. temperature,
atmospheric pressure, wind, humidity and precipitation. You may have observed that the
weather conditions fluctuate very often even within a day. But there is some common pattern
over a few weeks or months, i.e. days are cool or hot, windy or calm, cloudy or bright, and wet or
dry. On the basis of the generalised monthly atmospheric conditions, the year is divided into
seasons such as winter, summer or rainy seasons.
60.

Solution (d)
Values that inspired and guided the freedom struggle and were in turn nurtured by it, formed the
foundation for Indias democracy. These values are embedded in the Preamble of the Indian
Constitution. They guide all the articles of the Indian Constitution.
http://insightsonindia.com INSIGHTS

Page 22

Facebook Group: Indian Administrative Service (Raz Kr)

INSIGHTS MOCK TEST 2 - SOLUTIONS


Taking inspiration from American model, most countries in the contemporary world have chosen
to begin their constitutions with a preamble.
It contains the philosophy on which the entire Constitution has been built. It provides a standard
to examine and evaluate any law and action of government, to find out whether it is good or bad.
It is the soul of the Indian Constitution.
61.

Solution: c)
Elemental oxygen is normally found in the form of a diatomic molecule. However, in the upper
reaches of the atmosphere, a molecule containing three atoms of oxygen is found. This would
mean a formula of O3 and this is called ozone. Unlike the normal diatomic molecule of oxygen,
ozone is poisonous and we are lucky that it is not stable nearer to the Earths surface. But it
performs an essential function where it is found. It absorbs harmful radiations from the Sun. This
prevents those harmful radiations from reaching the surface of the Earth where they may damage
many forms of life.
62.

Solution: b)

63.

Solution: b)
64.

Solution: d)
All statements are correct; hence answer is (d) There are six major controls of the climate of any
place. They are: latitude, altitude, pressure and wind system, distance from the sea
(continentality), ocean currents and relief features.
Due to the curvature of the earth, the amount of solar energy received varies according to
latitude. As a result, air temperature decreases from the equator towards the poles. As one goes
from the surface of the earth to higher altitudes, the atmosphere becomes less dense and
temperature decreases. The hills are therefore cooler during summers. The pressure and wind
system of any area depend on the latitude and altitude of the place. Thus it influences the
temperature and rainfall pattern. The sea exerts a moderating influence on climate: As the
distance from the sea increases, its moderating influence decreases and the people experience
extreme weather conditions. This condition is known as continentality (i.e. very hot during
http://insightsonindia.com INSIGHTS

Page 23

Facebook Group: Indian Administrative Service (Raz Kr)

INSIGHTS MOCK TEST 2 - SOLUTIONS


summers and very cold during winters). Ocean currents along with onshore winds affect the
climate of the coastal areas, For example, any coastal area with warm or cold currents flowing
past it, will be warmed or cooled if the winds are onshore.
65.

Solution (a)
While WPI (Wholesale Price Index) represents the wholesale prices of goods, CPI indicates the
average price paid by households for a basket of goods and services.
Central Statistics Office (CSO), Ministry of Statistics and Programme Implementation releases
CPI with base year 2010 for all India and States/UTs separately for rural, urban and
combined(rural+urban)every month with effect from January, 2011.
These indices are available for five major groups namely Food, beverages and tobacco; Fuel and
light; Housing (Only for Urban; Rural areas are not included); Clothing, bedding and footwear,
and Miscellaneous.
Approximately 50% weightage is given to food items and total number of commodities in the
index is 200.

66.

Solution (b)
Bitcoin is a consensus network that enables a new payment system and a completely digital
money. It is the first decentralized peer-to-peer payment network that is powered by its users
with no central authority or middlemen.
While bitcoins are not illegal in India, the RBI is watching and learning about the developments
in bitcoin even though it has no intention of regulating it right now. But the Reserve Bank of
India (RBI) seems to have put a damper on things with its recent press release cautioning against
the use of virtual currencies. Since then, a number of bitcoin operators have shut shop. Although
the central bank has not established any regulatory framework applicable to bitcoins, it has
repeatedly stated that it is examining the issue.
Within Bitcoin network people can trade without actually disclosing who they are in real lives.It
is just like cash where nobody needs to disclose identity (ex. bank account details) , unlike in
other options like paypal or e-cheques where one needs to tie bank account details while
transacting.
Its not an income-generating asset class suitable for most investors. Its value, in dollars, fluctuates
wildly from one minute to the next. And while it can be a cheap way of transferring money,
there are too many glitches in its emerging network for bitcoin to be entirely reliable.
http://insightsonindia.com INSIGHTS

Page 24

Facebook Group: Indian Administrative Service (Raz Kr)

INSIGHTS MOCK TEST 2 - SOLUTIONS


67.

Solution: a
The Doctrine of Lapse - The final wave of annexations occurred under Lord Dalhousie who was
the Governor-General from 1848 to 1856. He devised a policy that came to be known as the
Doctrine of Lapse. The doctrine declared that if an Indian ruler died without a male heir his
kingdom would lapse, that is, become part of Company territory. One kingdom after another
was annexed simply by applying this doctrine: Satara (1848), Sambalpur (1850), Udaipur (1852),
Nagpur (1853) and Jhansi (1854).
68.

Solution: b)
Western Disturbance is the term used in India, Pakistan, Bangladesh and Nepal to describe
an extratropical storm originating in the Mediterranean, that brings sudden winter rain and snow
to the northwestern parts of the Indian subcontinent. This is a non-monsoonal precipitation
pattern driven by the Westerlies. The moisture in these storms usually originates over the
Mediterranean Sea and the Atlantic Ocean. Extratropical storms are a global, rather than a
localized, phenomena with moisture usually carried in the upper atmosphere (unlike tropical
storms where it is carried in the lower atmosphere). In the case of the subcontinent, moisture is
sometimes shed as rain when the storm system encounters the Himalayas.
Western Disturbances are important to the development of the Rabi crop in the northern
subcontinent, which includes the locally important staple wheat.
Western Disturbance causes winter and pre monsoon season rainfall across northwest India.
Winter months Rainfall has great importance in agriculture, particularly for the rabi crops. Wheat
among them is one of the most important crops, which helps to meet Indias food security.
During the season, normally 4-5 western disturbances in a month can be seen over northwest
India. Some of the western disturbances bring well-distributed and good rainfall, while some pass
with negligible rain or sometimes no rain. The Western disturbance affects day-to-day weather of
northwest India especially during winter season. It is usually associated with cloudy sky, higher
night temperatures, unusual rain etc. Over the Indo-Gangetic plains, it brings cold wave
conditions and occasionally dense fog and cold day conditions. These conditions remain stable
until it is disturbed by another Western Disturbances.
69.
Solution (d)
Hot money is a term that is most commonly used in financial markets to refer to the flow of
funds (or capital) from one country to another in order to earn a short-term profit on interest rate
differences and/or anticipated exchange rate shifts. These speculative capital flows are called "hot
money" because they can move very quickly in and out of markets, potentially leading to market
instability.
http://insightsonindia.com INSIGHTS

Page 25

Facebook Group: Indian Administrative Service (Raz Kr)

INSIGHTS MOCK TEST 2 - SOLUTIONS

70.

Solution (a)
Currency swaps were an important part of the policy response to the 2007-09 financial crisis,
keeping a lid on funding costs, which had spiralled due to fear over counter-party risk.
A currency swap is an exchange that occurs between two individuals or entities holding set
currencies for a specified period of time after which, they will be exchanged back.This can occur
between two or more parties. Upon the maturity of the exchange, each party returns to the other
the original amount of currency traded.
Due to laws of international accounting, currency swaps are not considered to be loans, and are
thus not required to be documented on a companys balance sheet. Instead of being considered a
loan, currency swaps are treated as foreign exchange transactions with the maturity date closed as
a forward contract, which is a simple agreement ensuring the date on which the amounts will be
repaid. The entirety of the cash flow contained within the swap is repaid upon maturity. This
includes the initial principal amount exchanged, the receipt or payment of interest in the same
currency, and the eventual return of the amount exchanged when the deal matures.
However, this investment also has some drawbacks. The first and foremost drawback can be that
the other party may fail to meet the obligation during the period of swap or upon maturity. The
parties exposed to the risk either one or both have chances to default on principal payment as
well as interest. The other drawback could be devaluation in the domestic currency, which can
happen when huge foreign debts are acquired. India has currency swap arrangement with Japan.
71.

Solution: b)
A meandering river across a flood plain forms cut-offs that later develop into ox-bow lakes. Spits
and bars form lagoons in the coastal areas, eg the Chilika lake, the Pulicat lake, the Kolleru lake.
Lakes in the region of inland drainage are sometimes seasonal; for example, the Sambhar lake in
Rajasthan, which is a salt water lake. Its water is used for producing salt.
Most of the fresh water lakes are in the Himalayan region. They are of glacial origin. In other
words, they formed when glaciers dug out a basin, which was later filled with snowmelt. The
Wular lake in Jammu and Kashmir, in contrast, is the result of the tectonic activity. It is the
largest freshwater lake in India. The Dal lake, Bhimtal, Nainital, Loktak and Barapani are some
other important fresh water lakes.
72.

Solution (b)

http://insightsonindia.com INSIGHTS

Page 26

Facebook Group: Indian Administrative Service (Raz Kr)

INSIGHTS MOCK TEST 2 - SOLUTIONS


RBI in India is incharge of the Monetary Policy and the Finance Ministry (Government of India)
is incharge of the Fiscal Policy. Increasing interest rates and Introduction of Inflation-Indexed
National Saving Securities (IINSSs) for retail investors are the steps taken by RBI.
RBI on consultation with the government plans to launch a 10-year savings instrument- InflationIndexed National Saving Securities (IINSSs) for retail investors that will offer inflation-linked
returns to small investors as an alternative to investing in gold. IINSS will be linked to the new
(combined) Consumer Price Index (CPI). The interest on these securities would comprise a fixed
rate plus inflation.
Interest would be compounded half-yearly and paid cumulatively at redemption. These securities
will be distributed through banks to reach out to the masses( Eligible investors : individuals,
Hindu Undivided Families, trusts and charitable institutions)
Whereas, increasing or decreasing taxes, duties, permitting FDI in retail are the steps taken by the
Finance Ministry (not RBI)to curb inflation.

73.

Solution (d)
The leaders of the freedom movement were clear in their mind that their country should be a
democracy. The different members had different views on different points, but ultimately they
resolved their differences.
All countries that have a constitution are not necessarily democratic but all countries which are
democratic must have their constitutions. The Constitution is amended by two-third majority of
the members of the Parliament if any change was required.

74.

Solution: d)
BRIEF PROVISIONS OF INDUS WATERS TREATY 1960
The Indus Waters Treaty is a water-sharing treaty between Pakistan and India, brokered by
the World Bank (then the International Bank for Reconstruction and Development).
i. The Indus system of rivers comprises three Eastern Rivers (Ravi, Beas and Sutlej and their
tributaries) and three Western Rivers (Indus, Jhelum and Chenab and their tributaries).
ii. The Indus Waters Treaty 1960 was signed on 19.09.1960 between India and Pakistan. It is
however effective from 01.04.1960.
iii. Under the Treaty, the waters of Eastern Rivers are allocated to India. India is under obligation
to let flow the waters of the Western Rivers except for the following uses:

http://insightsonindia.com INSIGHTS

Page 27

Facebook Group: Indian Administrative Service (Raz Kr)

INSIGHTS MOCK TEST 2 - SOLUTIONS


(a) Domestic Use,
(b) Non-consumptive use,
(c) Agricultural use as specified,
(d) Generation of hydro-electric power as specified
iv. India has been permitted to construct storage of water on Western Rivers upto 3.6 MAF for
various purposes. No storage has been developed so far.
v. India has been permitted Agricultural Use of 7, 01,000 acres over and above the Irrigated
Cropped Area (ICA) as on 1.4.60. Out of this additional ICA of 7,01,000 acres, only 2,70,000 can
be developed (i.e. a total ICA of 9,12,477 acres including that on 1.4.1960) till storages are
constructed and 0.5 MAF of water is released there from every year. ICA during 2008-09 was 7,
92,496 Acres.
vi. Under the Treaty, India and Pakistan have each created a permanent post of Commissioner for
Indus Waters. They together constitute the Permanent Indus Commission (PIC), which is
entrusted with the implementation of the Treaty. The PIC is required to hold meetings and tours
and submit report on its work to the two Governments every year. It has held 111 tours and 106
meetings so far.
vii. Both sides are required to exchange information related to river flows observed by them, not
later than three months of their observation and to exchange specified information on
Agricultural Use every year.
viii. India is under obligation to supply information of its storage and hydroelectric projects as
specified.
viii. India communicates as a gesture of goodwill, flood data to Pakistan from 1st July to 10th
October every year, to enable them to undertake advance flood relief measures. The arrangement
is reviewed every year.
ix. The Commissioners may discuss the questions arising under the Treaty under Article IX of
the Treaty related to Settlement of Differences and Disputes and in the case of non-resolution,
take further action under this Article for resolution through a Neutral Expert, negotiators or
Court of Arbitration. [A Neutral Expert appointed by World Bank on Pakistans request
delivered Expert Determination on Baglihar Hydroelectric Project in Feb. 2007. To resolve the
issues of Kishenganga H.E. Project, Pakistan has initiated proceedings for setting up a Court of
Arbitration.

http://insightsonindia.com INSIGHTS

Page 28

Facebook Group: Indian Administrative Service (Raz Kr)

INSIGHTS MOCK TEST 2 - SOLUTIONS


75.

Solution: d)
Drainage Patterns
The streams within a drainage basin form certain patterns, depending on the slope of land,
underlying rock structure as well as the climatic conditions of the area. These are dendritic, trellis,
rectangular, and radial patterns. The dendritic pattern develops where the river channel follows
the slope of the terrain. The stream with its tributaries resembles the branches of a tree, thus the
name dendritic. A river joined by its tributaries, at approximately right angles, develops a trellis
pattern. A trellis drainage pattern develops where hard and soft rocks exist parallel to each other.
A rectangular drainage pattern develops on a strongly jointed rocky terrain. The radial pattern
develops when streams flow in different directions from a central peak or dome like structure.
A combination of several patterns may be found in the same drainage basin
76.

Solution (b)
The Constitution of India has provisions for Powers of the legislature, Name of the country,
Powers of the head of the state but not the Name of the head of the state.
77.

Solution: a)
The Lakshadweep Islands group lying close to the Malabar coast of Kerala is composed of small
coral isalnds. Earlier they were known as Laccadive, Minicoy and Amindive. In 1973 these were
named as Lakshadweep. It covers small area of 32 sq km. Kavaratti island is the administrative
headquarters of Lakshadweep. This island group has great diversity of flora and fauna. The Pitli
island, which is uninhabited, has a bird sanctuary.
78.

Solution: a)
http://www.thehindu.com/business/Economy/nonfarm-jobs-to-shrink-by-25-pc-in-7-yearscrisil/article5550168.ece
79.

Solution: c)

http://insightsonindia.com INSIGHTS

Page 29

Facebook Group: Indian Administrative Service (Raz Kr)

INSIGHTS MOCK TEST 2 - SOLUTIONS


http://en.wikipedia.org/wiki/2011_census_of_India
80.

Solution (b)
Elections can be held in many ways. All democratic countries hold elections. But most nondemocratic countries also hold some kind of elections.
Sometimes election is held only for one constituency to fill the vacancy caused by death or
resignation of a member. This is called a by-election.
In India we follow a system of territorial representation/ area-based representation for the
election of members of Lok Sabha. And in Rajya Sabha we follow proportional representation
One of thefeatures of a democratic election is that every vote should have equal value. That is
why our Constitution requires that each constituency should have a roughly equal population
living within it.

81.

Solution (c)
In our country we follow an area based system of representation. The country is divided into
different areas for purposes of elections. These areas are called electoral constitutencies. The
voters who live in an area elect one representative.
Each Parliamentary constituency has within it several assembly constituencies. The same principle
applies for Panchayat and Municipal elections. Each village or town is divided into several wards
that are like constituencies. Each ward elects one member of the village or the urban local body.
Currently, in the Lok Sabha, 84 seats are reserved for the Scheduled Castes and 47 for the
Scheduled Tribes (as on 1 September 2012). This number is in proportion to their share in the
total population.
Thus the reserved seats for SC and ST do not take away the legitimate share of any other social
group. Seats are not reserved for OBCs & Minorities in Parliament.

82.

Solution: b)
One of the distinct features of the peninsular plateau is the black soil area known as Deccan Trap.
This is of volcanic origin hence the rocks are igneous. Actually these rocks have denuded over
time and are responsible for the formation of black soil.
Barchans (crescent shaped dunes) cover larger areas of the Thar Desert but longitudinal dunes
become more prominent near the Indo-Pakistan boundary
http://insightsonindia.com INSIGHTS

Page 30

Facebook Group: Indian Administrative Service (Raz Kr)

INSIGHTS MOCK TEST 2 - SOLUTIONS

83.

Solution: b)
http://www.thehindubusinessline.com/features/investment-world/market-watch/fundamentals-offorward-and-futures-contract/article3588996.ece
84.

Solution: c
From the late eighteenth century the Company also sought to curb and eventually destroy
Maratha power. With their defeat in the Third Battle of Panipat in 1761, the Marathas dream of
ruling from Delhi was shattered. They were divided into many states under different chiefs
(sardars) belonging to dynasties such as Sindhia, Holkar, Gaikwad and Bhonsle. These chiefs were
held together in a confederacy under a Peshwa (Principal Minister) who became its effective
military and administrative head based in Pune. Mahadji Sindhia and Nana Phadnis were two
famous Maratha soldiers and statesmen of the late eighteenth century.
85.

Solution: c
After the Battle of Buxar (1764), the Company appointed Residents in Indian states. They were
political or commercial agents and their job was to serve and further the interests of the
Company. Through the Residents, the Company officials began interfering in the internal affairs
of Indian states. They tried to decide who was to be the successor to the throne, and who was to
be appointed in administrative posts. Sometimes the Company forced the states into a subsidiary
alliance. According to the terms of this alliance, Indian rulers were not allowed to have their
independent armed forces. They were to be protected by the Company, but had to pay for the
subsidiary forces that the Company was supposed to maintain for the purpose of this
protection. If the Indian rulers failed to make the payment, then part of their territory was taken
away as penalty.
For example, when Richard Wellesley was Governor- General (1798-1805), the Nawab of
Awadh was forced to give over half of his territory to the Company in 1801, as he failed to pay
for the subsidiary forces. Hyderabad was also forced to cede territories on similar grounds.
86.

Solution: a)
The Deccan Plateau is a triangular landmass that lies to the south of the river Narmada. The
Satpura range flanks its broad base in the north while the Mahadev, the Kaimur hills and the
http://insightsonindia.com INSIGHTS

Page 31

Facebook Group: Indian Administrative Service (Raz Kr)

INSIGHTS MOCK TEST 2 - SOLUTIONS


Maikal range form its eastern extensions. Locate these hills and ranges in the physical map of
India. The Deccan Plateau is higher in the west and slopes gently eastwards. An extension of the
Plateau is also visible in the northeast locally known as the Meghalaya and Karbi-Anglong
Plateau. It is separated by a fault from the Chotanagpur Plateau. Three Prominent hill ranges from
the west to east are the Garo, theKhasi and the Jaintia Hills.
The Western Ghats and the Eastern Ghats mark the western and the eastern edges of the Deccan
Plateau respectively. Western ghats lie parallel to the western coast. They are continuous and can
be crossed through passes only. Locate the Thal, Bhor and the Pal Ghats in the Physical map of
India. The Western Ghats are higher than the Eastern Ghats. Their average elevation is 900
1600 metres as against 600 metres of the Eastern Ghats.
87.

Solution (a)
Our Constitution entitles every citizen to elect her/his representative and to be elected as a
representative. The Constitution makers, however, were worried that in an open electoral
competition, certain weaker sections may not stand a good chance to get elected to the Lok
Sabha and the state Legislative Assemblies. They may not have the required resources, education
and contacts to contest and win elections against others.
The system of reservation was extended to other weaker sections at the district and local level. In
many states, seats in rural (panchayat) and urban (municipalities and corporations) local bodies
are now reserved for Other Backward Classes (OBC) as well. However, the proportion of seats
reserved varies from state to state. Similarly, one-third of the seats are reserved in rural and urban
local bodies for women candidates. But not mandatory for disabled.
88.
Solution: c)
89.

Solution: d)
Given below are the important changes that were introduced by the British.
1. The British Parliament passed a new Act in 1858 and transferred the powers of the East India
Company to the British Crown in order to ensure a more responsible management of Indian
affairs. A member of the British Cabinet was appointed Secretary of State for India and made
responsible for all matters related to the governance of India. He was given a council to advise
him, called the India Council. The Governor-General of India was given the title of Viceroy, that
is, a personal representative of the Crown. Through these measures the British government
accepted direct responsibility for ruling India.

http://insightsonindia.com INSIGHTS

Page 32

Facebook Group: Indian Administrative Service (Raz Kr)

INSIGHTS MOCK TEST 2 - SOLUTIONS


2. All ruling chiefs of the country were assured that their territory would never be annexed in
future. They were allowed to pass on their kingdoms to their heirs, including adopted sons.
However, they were made to acknowledge the British Queen as their Sovereign Paramount. Thus
the Indian rulers were to hold their kingdoms as subordinates of the British Crown.
3. It was decided that the proportion of Indian soldiers in the army would be reduced and the
number of European soldiers would be increased. It was also decided that instead of recruiting
soldiers from Awadh, Bihar, central India and south India, more soldiers would be recruited from
among the Gurkhas, Sikhs and Pathans.
4. The land and property of Muslims was confiscated on a large scale and they were treated with
suspicion and hostility. The British believed that they were responsible for the rebellion in a big
way.
5. The British decided to respect the customary religious and social practices of the people in
India.
90.

Solution (b)
In a democratic election, the list of those who are eligible to vote is prepared much before the
election and given to everyone. This list is officially called the Electoral Roll and is commonly
known as the Voters List.
Anyone who can be a voter can also become a candidate in elections. The only difference is that
in order to be a candidate the minimum age is 25 years, while it is only 18 years for being a voter.
There are some other restrictions on criminals etc. but these apply in very extreme cases.
91.

Solution: c)
It is not if fertilized egg doesnt develop if fertilization doesnt occur at all, then the egg and
the thickened lining of the uterus along with its blood vessels are shed off. This causes bleeding in
women which is called menstruation.
Menstruation occurs once in about 28 to 30 days. The first menstrual flow begins at puberty and
is termed menarche. At 45 to 50 years of age, the menstrual cycle stops. Stoppage of menstruation
is termed menopause. Initially, menstrual cycle may be irregular. It takes some time to become
regular.
92.
Sorry repeat question
93.

Solution: d)
94.

Solution: d)
http://insightsonindia.com INSIGHTS

Page 33

Facebook Group: Indian Administrative Service (Raz Kr)

INSIGHTS MOCK TEST 2 - SOLUTIONS


In 1600, the East India Company acquired a charter from the ruler of England, Queen Elizabeth I,
granting it the sole right to trade with the East. This meant that no other trading group in England
could compete with the East India Company. With this charter the Company could venture
across the oceans, looking for new lands from which it could buy goods at a cheap price, and
carry them back to Europe to sell at higher prices. The Company did not have to fear
competition from other English trading companies. Mercantile trading companies in those days
made profit primarily by excluding competition, so that they could buy cheap and sell dear. The
royal charter, however, could not prevent other European powers from entering the Eastern
markets.
By the time the first English ships sailed down the west coast of Africa, round the Cape of Good
Hope, and crossed the Indian Ocean, the Portuguese had already established their presence in the
western coast of India, and had their base in Goa. In fact, it was Vasco da Gama, a Portuguese
explorer, who had discovered this sea route to India in 1498. By the early seventeenth century,
the Dutch too were exploring the possibilities of trade in the Indian Ocean. Soon the French
traders arrived on the scene. The problem was that all the companies were interested in buying
the same things. The fine qualities of cotton and silk produced in India had a big market in
Europe. Pepper, cloves, cardamom and cinnamon too were in great demand. Competition
amongst the European companies inevitably pushed up the prices at which these goods could be
purchased, and this reduced the profits that could be earned. The only way the trading companies
could flourish was by eliminating rival competitors. The urge to secure markets therefore led to
fierce battles between the trading companies. Through the seventeenth and eighteenth centuries
they regularly sank each others ships, blockaded routes, and prevented rival ships from moving
with supplies of goods. Trade was carried on with arms and trading posts were protected through
fortification. This effort to fortify settlements and carry on profitable trade also led to intense
conflict with local rulers. The company therefore found it difficult to separate trade from politics.
95.

Solution: b)
Finally, in 1765 the Mughal emperor appointed the Company as the Diwan of the provinces of
Bengal. The Diwani allowed the Company to use the vast revenue resources of Bengal. This
solved a major problem that the Company had earlier faced. From the early eighteenth century its
trade with India had expanded. But it had to buy most of the goods in India with gold and silver
imported from Britain. This was because at this time Britain had no goods to sell in India. The
outflow of gold from Britain slowed after the Battle of Plassey, and entirely stopped after the
assumption of Diwani. Now revenues from India could finance Company expenses. These
revenues could be used to purchase cotton and silk textiles in India, maintain Company troops,
and meet the cost of building the Company fort and offices at Calcutta.
96.
Solution: c)

The Convention on International Trade in Endangered Species of Wild Fauna and


Flora (CITES) entered into force in 1975, and became the only treaty to ensure that international
trade in plants and animals does not threaten their survival in the wild. Currently 179 countries
(called Parties), including the United States, implement CITES.
http://insightsonindia.com INSIGHTS

Page 34

Facebook Group: Indian Administrative Service (Raz Kr)

INSIGHTS MOCK TEST 2 - SOLUTIONS


The Convention on International Trade in Endangered Species of Wild Fauna and Flora, also
known as the Washington Convention
97.

Solution: d)
Just as victories against the British had earlier encouraged rebellion, the defeat of rebel forces
encouraged desertions. The British also tried their best to win back the loyalty of the people.
They announced rewards for loyal landholders would be allowed to continue to enjoy traditional
rights over their lands. Those who had rebelled were told that if they submitted to the British,
and if they had not killed any white people, they would remain safe and their rights and claims to
land would not be denied. Nevertheless, hundreds of
sepoys, rebels, nawabs and rajas were tried and hanged.
98.

Solution d)
According to the variations in relief features, the Northern plains can be divided into four regions.
The rivers, after descending from the mountains deposit pebbles in a narrow belt of about 8 to 16
km in width lying parallel to the slopes of the Shiwaliks. It is known as bhabar. All the streams
disappear in this bhabar belt. South of this belt, the streams and rivers re-emerge and create a
wet, swampy and marshy region known as terai. This was a thickly forested region full of wildlife.
The forests have been cleared to create agricultural land and to settle migrants from Pakistan after
partition. Locate Dudhwa National Park in this region.
The largest part of the northern plain is formed of older alluvium. They lie above the flood plains
of the rivers and present a terrace like feature. This part is known as bhangar. The soil in this
region contains calcareous deposites locally known as kankar. The newer, younger deposits of the
flood plains are called khadar. They are renewed almost every year and so are fertile, thus, ideal
for intensive agriculture.
99.

Solution: d)
Dargah The tomb of a Sufi saint
Khanqah A sufi lodge, often used as a rest house for travellers and a place where people
come to discuss spiritual matters, get the blessings of saints, and hear sufi music
Idgah
An open prayer place of Muslims primarily meant for id prayers
Cul-de-sac Street with a dead end
100.

Solution: d)

http://insightsonindia.com INSIGHTS

Page 35

Facebook Group: Indian Administrative Service (Raz Kr)

INSIGHTS ON INDIA MOCK PRELIMINARY EXAM - 2014


INSIGHTS ON INDIA MOCK TEST - 3
GENERAL STUDIES

PAPER-I
Time Allowed: Two Hours

Maximum Marks: 200

INSTRUCTIONS
1. IMMEDITELY AFTER THE COMMENCEMENT OF THE EXAMINATION, YOU SHOULD
CHECK THAT THIS TEST BOOKLET DOES NOT HAVE ANY UNPRINTED OR TORN OR
MISSING PAGES OR ITEMS, ETC. IF SO, GET IT REPLACED BY A COMPLETE TEST BOOKLET.
2. You have to enter your Roll Number on the Test I
Booklet in the Box provided alongside. DO NOT
write anything else on the Test Booklet.
4. This Test Booklet contains 100 items (questions). Each item is printed only in English. Each item
comprises four responses (answers). You will select the response which you want to mark on the Answer
Sheet. In case you feel that there is more than one correct response, mark the response which you consider
the best. In any case, choose ONLY ONE response for each item.
5. You have to mark all your responses ONLY on the separate Answer Sheet provided. See directions in the
Answer Sheet.
6. All items carry equal marks.
7. Before you proceed to mark in the Answer Sheet the response to various items in the Test Booklet, you
have to fill in some particulars in the Answer Sheet as per instructions sent to you with your Admission
Certificate.
8. After you have completed filling in all your responses on the Answer Sheet and the examination has
concluded, you should hand over to the Invigilator only the Answer Sheet. You are permitted to take away
with you the Test Booklet.
9. Sheets for rough work are appended in the Test Booklet at the end.
10. Penalty for wrong answers :
THERE WILL BE PENALTY FOR WRONG ANSWERS MARKED BY A CANDIDATE IN THE
OBJECTIVE TYPE QUESTION PAPERS.
(i)

There are four alternatives for the answer to every question. For each question for which a
wrong answer has been given by the candidate, one-third of the marks assigned to that
question will be deducted as penalty.

(ii) If a candidate gives more than one answer, it will be treated as a wrong answer even if one of the
given answers happens to be correct and there will be same penalty as above to that question.
(iii)

If a question is left blank, i.e., no answer is given by the candidate, there will be no penalty
for that question.

http://insightsonindia.com

INSIGHTS ON INDIA MOCK TEST SERIES FOR CIVIL SERVICES PRELIMINARY EXAM 2014

http://insightsonindia.com

Page 1

Facebook Group: Indian Administrative Service (Raz Kr)

1. With reference to waterways in


India, consider the following
statements
1. There are five national
waterways in India
2. The National WaterwayV connects Brahmani
river & Mahanadi delta
system along with East
Coast Canal
Which of the above statements is/are
correct?
a.
b.
c.
d.

1 Only
2 Only
Both
None

2. Power sharing is the very spirit


of democracy. In this context,
which
of
the
following
statements are correct?
1. Power sharing increases
conflict among different
communities
2. Power sharing increases
the
possibility
of
arbitrariness
3. Power sharing delays
decision making process
4. Power sharing increases
instability
and
divisiveness

http://insightsonindia.com

5. Power
sharing
undermines the unity of
a country
Select the correct answer using the
codes below
a.
b.
c.
d.

1, 2 and 5 Only
2 and 3 Only
2, 3 and 4 Only
None

3. India needs to be energy secure for


ensuring robust economic growth.
What are the characteristics of a good
source of energy:
1. Readily accessible
2. Easy to store and
transport
3. High calorific value
4. Should be renewable
Select the correct answer using the
codes below
a.
b.
c.
d.

1, 2, 3 and 4
1, 2 and 3 Only
3 and 4 Only
2 and 4 Only

4. Nineteenth century witnessed one


of the important phase of transition in
the history of the world. With regard
Page 2

Facebook Group: Indian Administrative Service (Raz Kr)

3. If taken on empty stomach,


these tablets can even cause
death in many cases of
children
4. This programme is restricted
to the children between
class 4th to class 12th as
children below 4th standard
are vulnerable to side effects.

to the plethora of novels that surfaced


during this period, consider the
following statements:
1. Novels focused more on the
lives of the great people
2. Novels
focused
on
the
problems faced by the working
class
3. Novels focused more on the
lives of the ordinary man
4. Oliver Twist was one of the
famous novel of this period
Which of the above is/are correct?
a) 1 and 4
b) 2 and 3
c) 3 and 4
d) 2, 3 and 4

5. Consider
statements
programme

the
on

following
WIFS

1. Iron folic acid tablet can be


taken during illness and even
during menstruation.
2. The weekly Iron Folic Acid
Supplementation (WIFS) is
an
evidence
based
programme for addressing
Iron Deficiency Anaemia
which is both safe and
effective.
http://insightsonindia.com

Which of the statements given above


is/are correct?
a.
b.
c.
d.

1 and 2 Only
1 and 3 Only
1,2 and 3 Only
All Four

6. With reference to the Rajiv


Gandhi Panchayat Sashaktikran
Abhiyan (RGPSA) scheme,
consider
the
following
statements,
1. The schemes of Rashtriya
Gram
Swaraj
Yojana(RGSY), e-Panchayat,
Panchayat
Empowerment
and
Accountability
Incentive Scheme(PEAIS)
and Panchayat Mahila Evam
Yuva
Shakti
Abhiyan
(PMEYSA)
will
be
subsumed in RGPSA from
2013-14
Page 3

Facebook Group: Indian Administrative Service (Raz Kr)

2. The funding of RGPSA for


State plans is envisaged on a
90:10 sharing basis by the
Central and all State
Governments respectively.
3. During the 12th plan, grants
to
State
Election
Commissions is linked to the
performance under RGPSA
4. Constitution
of
State
Finance Commission(SFC)
every
five years
and
placement of Action Taken
Report (ATR) on the
recommendations of the
SFC in the state Assembly is
a must to avail funds under
RGPSA
Which of the statements given above
is/are correct?
a.
b.
c.
d.

1,2 and 3 Only


1,3 and 4 Only
2, 3 and 4 Only
All Four

7. Sarojini Naidu was one of the


famous personality on the 20th
Century.
Which
of
the
following statements is/are
correct?
1. Sarojini Naidu was the first
Indian woman to become

http://insightsonindia.com

the President of the Indian


National Congress
2. Sarojini Naidu was the first
woman Governor of India
3. Sarojini Naidus birthday is
celebrated as "Women's
Day"
4. Sarojini Naidu was one of
the framers of Indian
Constitution.
Select the correct answer using the
codes below

a.
b.
c.
d.

1 and 4 Only
1, 3 and 4 Only
1, 2 and 4Only
1, 2, 3 and 4

8. With regard to Indian Literature


consider the following:
1.
2.
3.
4.

Stories in prose was new to India.


Panchatantra is a prose.
Prose overlooks asthetic appeal.
Dastan contains tales of adventure
and heroism in Persian and Urdu
and is regarded as a prose

Which of the statements given above


is/are correct?

a. 1, 2 and 3
b. 2, 3 and 4
Page 4

Facebook Group: Indian Administrative Service (Raz Kr)

c. 1 Only
d. 1, 3 and 4

9. With reference to resource


planning and development,
consider
the
following
statements
1. Resource planning includes
matching
the
resource
development plans with
overall
national
development plans.
2. The availability of resources
is a necessary condition for
the development of any
region, but mere availability
of resources in the absence
of corresponding changes in
technology and institutions
may hinder development.
Which of the statements given above
is/are correct?
a.
b.
c.
d.

1 Only
2 Only
Both
None

10. Consider
the
following
statements,
1. The
Limits
to
Growth is a 1972 book
http://insightsonindia.com

about the computer


modeling
of
exponentialeconomic
and population
growth with
finite
resource supplies
2. The Club of Rome is
a
global think
tank that deals with a
variety
of
international political
issues.
3. The purpose of The
Limits
to
Growth was to
explore
how
exponential growth
interacts with finite
resources.
Which of the statements given above
is/are correct?
a.
b.
c.
d.

1 and 2 Only
1 and 3 Only
2 and 3 Only
All Three

11. Power is shared among different


organs of government. Such a
separation of powers ensures
that none of the organs can
exercise unlimited
power.
Which the following system of
power sharing is called checks
and balances?
Page 5

Facebook Group: Indian Administrative Service (Raz Kr)

1. Horizontal division
powers
2. Separation of powers
3. Vertical
division
powers
4. Federal
division
powers

of

of
of

Which of the above answers is/are


correct?
a.
b.
c.
d.

1 Only
1 and 3
1 and 4
4 Only

12. In the parliamentary system of


Government, India has adopted the
principle of checks and balances.
Consider the following statements:
1. Even though ministers and
government
officials
exercise
power, they are responsible to the
Parliament or State Assemblies.
2. Powers shared among governments
at different levels
3. Although judges are appointed by
the executive, they can check the
functioning of executive or laws
made by the legislatures
4. Power shared among different
social groups, such as the religious
and linguistic groups.
http://insightsonindia.com

Which of
incorrect?
a.
b.
c.
d.

the

statements

is/are

2 and 4 Only
1 and 3 Only
1, 2 and 3 Only
1, 2, 3 and 4

13. Which of the following mirrors


are best suited to be used in
Solar cookers?
a. Concave and Convex
both
b. Only Plane Mirror
c. Convex mirror only
d. Concave mirror only

14. What is the basic force behind


the generation of Ocean
thermal energy?
a. Heat generated by the
seabed volcanoes.
b. Energy of waves and
currents
c. Temperature difference
between the surface and
deeper ocean
d. Rise and fall in the level
of tides

Page 6

Facebook Group: Indian Administrative Service (Raz Kr)

literature of the country


that could produce a
sense
of
national
belonging and cultural
equality
with
their
colonial masters.

15. Which of the following are


correct?
1. The modern novel form
developed in India in the
eighteenth
century,
as
Indians became familiar with
the Western novel.
2. Print and vernacular helped
in the development of the
modern novel.
3. The earliest novel in Marathi
was
Baba
Padmanjis
Yamuna Paryatan, which
spoke about the plight of
widows.
Select the correct answer using the
codes below
a.
b.
c.
d.

2 Only
1 and 2
2 and 3
1, 2 and 3

16. Consider the following:


1. Indian novelists of the
nineteenth century wrote
for a cause.
2. Colonial rulers regarded
the contemporary culture
of India as superior.
3. Indian novelists failed to
develop
a
modern
http://insightsonindia.com

Which of the above statements is/are


correct?
a.
b.
c.
d.

1 Only
1 and 2
3 Only
None

17. In the modern India, though


there was an increase in the
Indian novels, the spread of it
was declined because of which
of the following factors?
1. Absence of translations
of novels into different
regional languages.
2. Absence of Print
3. Vernacular languages did
not help in development
of the novels.
4. Fear of Western culture
invading Indian culture
Select the correct answers using the
codes below
a. 1, 2 and 3
Page 7

Facebook Group: Indian Administrative Service (Raz Kr)

b. 2, 3 and 4
c. 1, 3 and 4
d. None
18. Consider
the
following
statements about Agenda 21
1. Agenda 21 is a binding action
plan of the United Nations
with regard to sustainable
development
2. The "21" in Agenda 21 refers
to the 21 important points
that are need to be
implemented by all the
countries
3. In 2012, at the United
Nations Conference on
Sustainable
Development the attending
members reaffirmed their
commitment to Agenda 21
in their outcome document
called "The Future We
Want"
Which of the above statements is/are
correct?
a.
b.
c.
d.

1 and 2 Only
2 and 3 Only
3 Only
All Three

19. The First World War created a


new economic and political
http://insightsonindia.com

situation in India which.


Consider
the
following
statements related to the First
World War :
1. Customs duties were
introduced and income
tax was raised to finance
the war.
2. Through the war years
prices
increased

doubling between 1913


and 1918 leading to
extreme hardship for the
common people.
3. Soldiers were forcefully
recruited from the rural
areas
which
caused
widespread anger among
the people.
4. The end of the First
World War improved
the lives of the people.
Which of the above statement/s
is/are correct?
a.
b.
c.
d.

1 and 2 only
2 and 3 only
1, 3 and 4 only
1, 2 and 3

20. Which

of

the

following
Page 8

Facebook Group: Indian Administrative Service (Raz Kr)

statements about Mahatma


Gandhi's famous philosophy of
SATYAGRAHA is/are correct
?
1. Satyagraha is not physical
force. A satyagrahi does
not inflict pain on the
adversary; he does not
seek the destruction of
the enemy.
2. Mahatma Gandhi first
experimented
with
Satyagraha at Champaran
in Bihar.
3. Satyagraha
aims
at
appealing
to
the
conscience
of
the
oppressor.
4. Satyagraha
Theory
influenced Martin Luther
King, Jr.'s and James
Bevel's campaigns during
the civil rights movement
in the United States

21. On 13 April 1919, the infamous


Jallianwalla Bagh incident took
place in Amritsar. Consider the
following statements on the
Jallianwalla Bagh massacre.
1. The people had gathered
at JallianwallaBagh to
attend
the
annual
Baisakhi fair and there
wasn't any protest.
2. The
British
media
strongly
condemned
General
Dyer
for
ordering the killings of
the people.
3. The Government held an
inquiry
commision,
chaired by Lord Hunter.
4. This commission did not
have any members and
the report was uncritical
of General Dyer.
which of the above
is/are correct?

statement/s

Select the correct answer using the


codes below
a.
b.
c.
d.

1 and 2 only
1, 2 and 3 only
1, 3 and 4 only
1, 2, 3 and 4

http://insightsonindia.com

a.
b.
c.
d.

1 only
2 only
2 and 3 only
none of the above

Page 9

Facebook Group: Indian Administrative Service (Raz Kr)

22. Consider
Statements:

the

following

1. In India, citizens have the


right to seek enforcement of
the Fundamental Rights
2. The Right to Constitutional
Remedies
is
not
a
Fundamental Right.
3. In case of violation of a
Fundamental Right the
citizen can directly approach
Supreme Court or High
Court.
4. Only the High Courts have
the right to issue orders for
the enforcement of both
Fundamental Rights and
Legal rights.
5. The Courts can also enforce
the Fundamental Rights
against private individuals
and bodies.
Which of
correct?
a.
b.
c.
d.

the

statements

is/are

1 and 3
1, 3 and 5
1, 3, 4 and 5
1, 2, 3 and 5

23. The concept of sustainable


development focuses on the
needs of :
http://insightsonindia.com

a. Future generation of
humanity only
b. Conservation
of
environment irrespective
of the needs of economic
growth
c. Present
and
future
generations of humanity
d. Conservation of fossil
fuels to ensure energy
security of the nations

24. The fusion reaction is said to


the future source of energy.
What are the conditions needed
to achieve fusion reaction?
a. Extremely high pressure
and
extremely
low
temperature
b. Extremely low pressure
and
extremely
low
temperature
c. Extremely high pressure
and
extremely
high
temperature
d. Extremely low pressure
and
extremely
high
temperature

25. Which of the following


statements is/are true?

Page 10

Facebook Group: Indian Administrative Service (Raz Kr)

a. The hydrogen bomb is


based on thermonuclear
fission reaction.
b. The fundamental physics
of the fission chain
reaction in a nuclear
weapon is quite different
to the physics of a
controlled
nuclear
reactor. Also, the two
types of devices are
engineered differently.
c. No industrialized nation
has been able to meet
even one-fourth of its
total demand of energy
from nuclear reactors.
Select the correct answer using the
codes below
a.
b.
c.
d.

All of the above


3 only
1 and 2 only
None of the above

26. With reference to the Special


Industry
Initiative scheme
known as UDAAN, consider
the following statements,
1. It will be implemented by
the
National
Skills
Development Corporation
(NSDC) and Ministry of
http://insightsonindia.com

Home Affairs along with the


corporate sector
2. It targets the youth of J&K,
Naxal affected districts and
North East India, specifically
graduates and postgraduates,
who are seeking global and
local opportunities.
Which of the statements given above
is/are correct?
a.
b.
c.
d.

1 Only
2 Only
Both
None

27. A new campaign for the safety


of women titled I respect
women has been launched by
a. Ministry of tourism
b. Ministry of women and

family welfare
c. Ministry of Home Affairs
d. None of the above
28. Consider
the
following
statements on National Urban
Health Mission (NUHM),
which along with National
Rural Health Mission (NRHM)
1. Both the schemes have been

now clubbed under Sarva


Swasthya Abhiyan.
Page 11

Facebook Group: Indian Administrative Service (Raz Kr)

2. A NUHM is proposed for

the urban poor with focus


on those living in urban
slums.
3. One
Accredited
Social
Health Activist ASHA
(community link worker)
for 200 to 500 households
will work under NUHM
scheme
Which of the statements given above
is/are correct?
a.
b.
c.
d.

1 and 2 Only
2 and 3 Only
3 Only
2 Only

29. Consider the following statements:


1. The Concurrent List contains
forest, education and agriculture as
its subjects.
2. All States in the Indian Union do
not have identical powers.
3. The Union territories do not have
the powers of a State
4. The
Constitution
originally
provided for a two-tier system of
government,
the
Union
Government and the State
Government only.
Which of
correct?

the

statements

http://insightsonindia.com

is/are

a.
b.
c.
d.

1 and 4
1, 2 and 4
2, 3 and 4
1, 2, 3 and 4

30. Country A is said to be a


federal country. Which of the
following are the features of a
federal government?
1. There are four levels (or
tiers) of government.
2. The fundamental provisions
of the constitution can be
unilaterally changed by one
level of government.
3. Courts have the power to
interpret the constitution
and the powers of different
levels of government.
4. Sources of revenue for each
level of government are
clearly specified to ensure its
financial autonomy.
Select the correct answer using the
codes below
a.
b.
c.
d.

3 and 4
1, 3 and 4
1 and 3
1, 2, 3 and 4

Page 12

Facebook Group: Indian Administrative Service (Raz Kr)

31. During the 19th Century,


Bengal literature was one of the
important vernacular literature
which
has
contributed
immensely to the Indian
literature. Which of the
following is/are correct with
reference to Bengal literature?
1. Initially the Bengali novel
used a colloquial style
associated with urban life.
2. Initially the Bengali novel
used meyeli, the language
associated with womens
speech.
3. Bankim
Chandra
was
famous for verse-oriented
Indian writing.
4. Vandhe Mataram was taken
from Rabindranath tagores
Anandamath
Select the correct answer using the
codes below
a.
b.
c.
d.

1 and 2 Only
1, 3 and 4 Only
1, 2 and 3 Only
None

32. With regard to Vernacular


literature,
consider
the
following:
http://insightsonindia.com

1. Vernacular novels were


used
by
colonial
administrators
in
governing the Indian
society.
2. Vernacular
novels
inspired the spirit of
Nationalism
in
the
remote areas.
3. Vernacular novels failed
to revive the Indian
culture.
4. Vernacular novels were
translated into English by
British administrators.
Which of the above is/are correct?
a.
b.
c.
d.

2 Only
1 and 2 Only
1, 2 and 4 Only
1, 2, 3 and 4

33. The Himalayan Yew is a


a. Bird species known for
its long beautiful white
feathers
b. A sub species of Owl
which is bigger in size
than any other owl
species in the world
c. A medicinal plant known
from
which
world
Page 13

Facebook Group: Indian Administrative Service (Raz Kr)

famous anti-cancer drug


is produced
d. None of the above

34. Consider
the
following
statements about decline in
Indias biodiversity
1. Habitat
destruction,
hunting, poaching, overexploitation,
environmental pollution,
poisoning and forest fires
are factors, which have
led to the decline in
Indias biodiversity.
2. Other important causes
of
environmental
destruction are unequal
access,
inequitable
consumption of resources
and differential sharing of
responsibility
for
environmental
wellbeing.
Which of the statements given above
is/are correct?
a.
b.
c.
d.

1 Only
2 Only
Both
None

35. Consider
the
following
statements on forests,
1. Assam has the largest
area under permanent
forests, constituting 75
per cent of its total forest
area.
2. Reserved and protected
forests are also referred
to as permanent forest
estates maintained for the
purpose of producing
timber and other forest
produce,
and
for
protective reasons.
3. In India More than half
of the total forest land
has
been
declared
reserved forests. Reserved
forests are regarded as the
most valuable as far as
the conservation of forest
and wildlife resources are
concerned.
4. Almost one-third of the
total forest area is
protected
forest,
as
declared by the Forest
Department. This forest
land are protected from
any further depletion.
Which one of the statements given
above is/are correct?
a. 1, 2and 4 Only

http://insightsonindia.com

Page 14

Facebook Group: Indian Administrative Service (Raz Kr)

b. 2,3 and 4 Only


c. 1 and 4 Only
d. All Four

Select the correct answer using the


codes below

36. The first proper modern novel


was written by Srinivas Das,
which was titled Pariksha-Guru.
Which of the following is the
theme of Pariksha Guru?
a. It is based on the inner and
outer world of the newly
emerging middle classes.
b. It is based on Indias
freedom struggle.
c. It is based on the plight of
women.
d. It is based on the issues
between a master and a
student

37. What are the benefits


watershed management?

of

1. Boosts local communities


income
2. Rejunevates land and
water sources
3. Increases the life of
downstream dam and
reservoirs
4. Mitigates drought and
floods
http://insightsonindia.com

a.
b.
c.
d.

1 and 2 Only
1, 2 and 3 Only
All of the above
3 and 4

38. Storing water in form of


groundwater is often considered
to be a better option than
storing it on surface reservoirs.
What can be the possible
reasons behind it?
1. Unlike surface water,
groundwater can always
be free of contaminants.
2. Evaporation of surface
reservoirs is higher as
compared
to
groundwater.
3. Stagnated surface water
provides
a
breeding
ground
for
disease
vectors.
4. Even nearby wells can be
recharged and water need
not be conveyed to each
well.
Which of the statements given above
is/are correct?
a. All of the above
b. 2, 3 and 4
Page 15

Facebook Group: Indian Administrative Service (Raz Kr)

c. 1, 2 and 3
d. 1 and 4 only

39. Which of the


statements are true?

the Indian members.

following

1. The regions of water scarcity


are also the regions of high
poverty in India.
2. Narmada Bachao Aandolan
and Chipko Movement are
examples of movements
related to the conservation
of the environment.
3. Local irrigation solutions as
compared to Mega-irrigation
projects enable the local
communities to exercise
more control over their
fields and environment.
Select the correct answer
using the codes below
a.
b.
c.
d.

1 and 2 only
2 and 3 only
1 and 3 only
All of the above

40. The Rowlatt Act,1919 was an


act hurriedly passed through the
Imperial Legislative Council
despite the united opposition of
http://insightsonindia.com

Consider the following statements


with regard to this Act:
1. It gave the government
enormous powers to repress
only the political activities of
the Indians.
2. It allowed detention of political
prisoners without trial for three
or more years.
3. Opposition to this law was the
sole reason for Gandhiji to call
for
the
Non-Cooperation
Movement.
Which of the statements above
is/are correct?
a.
b.
c.
d.

1 only
1 and 2 only
1 and 3 only
1, 2 and 3

41. The Non-Cooperation-Khilafat


Movement began in January
1921. Various social groups
participated in this movement,
each with its own specific
aspiration in their own way.
Consider the following social
groups:

Page 16

Facebook Group: Indian Administrative Service (Raz Kr)

1. Students,
headmasters and
teachers,
and
lawyers
2. Peasants
3.Tribals
4.Plantation workers

b. 1,2,3,4 and 7 Only


c. 1,2,4 and 7 Only
d. 1,2 and 4 Only

43. The newly launched Seekho


aur Kamao scheme is,
1.

Which of the above group/groups


participated in the movement?

2.

a.
b.
c.
d.

1 and 2 only
1, 2 and 3 only
1, 3 and 4 only
1, 2, 3 and 4

42. Which of the following mission


mode projects are exclusively
implemented by the state
governments under National eGovernance Plan?
1.
2.
3.
4.
5.
6.
7.

Education
Health
PDS
Agriculture
Insurance
Banking
Road Transport

Select the codes given below to


answer the question
a. 1,2,3,5,and 7 Only
http://insightsonindia.com

a 100% Central Sector


Scheme
for
Skill
Development of minorities
Implemented
by
the
Ministry of Youth Affairs

Select the codes given below to


answer the question
a.
b.
c.
d.

1 Only
2 Only
Both
None

44. The Tory government in Britain


constituted
a
Statutory
Commission under Sir John
Simon which was to arrive in
India in 1928.
1. It did not have even a
single Indian member.
2. Lord
Reading,
announced in October
1929, a vague offer of
dominion status for
India in an unspecified
Page 17

Facebook Group: Indian Administrative Service (Raz Kr)

future.
3. All
the
political
parties in India except
the Muslim League
participated in Go
back
Simon
demonstrations.
Which
incorrect?
a.
b.
c.
d.

of

the

above

Which of the above statement/s


is/are correct?

a.
b.
c.
d.

is/are

1 and 2
2 and 3
2 only
3 only

46. Which of the following


statements is/are correct?
a. In India joint forest
management
(JFM)
programme e has been in
formal existence since
1988 when the state of
Orissa passed the first
resolution for joint forest
management
b. JFM depends on the
government to undertake
protection
activities
mostly on degraded forest
land managed by the
forest department.

45. Consider the statements on the


impact of the Great Depression
(which began in 1929 and lasted
till the mid-1930s) on India.
1. Both the exports
and imports of
India
nearly
halved between
1928 and 1934.
2. In
these
depression years,
India became an
exporter
of
precious metals,
notably gold.

http://insightsonindia.com

1 only
2 only
Both 1 and 2
Neither 1 nor 2

Select the right answer using the codes


below
a.
b.
c.
d.

1 Only
2 Only
Both
None

Page 18

Facebook Group: Indian Administrative Service (Raz Kr)

47. Consider
statements:

the

following

Technology and
Policy (STI) 2013

1. One of its key features is

1. Dr
B.R.
Ambedkar
organised the dalits into
the Depressed Classes
Association in 1930
2. He
clashed
with
Mahatma Gandhi at the
second Round Table
Conference
by
demanding reserved seats
for dalits in provincial
and central legislative
councils.
3. Gandhiji began a fast
unto death opposing
reservations to Depressed
Classes in 1932.

positioning India among the


top five global scientific
powers by 2020
2. Raising Gross Expenditure
in
Research
and
Development (GERD) to
2% from the present 1% of
the GDP in this decade by
encouraging
enhanced
private sector contribution.
3. India has declared 2010-20 as
the Decade of Innovation
4. A Strong and viable Science,
Research and Innovation
system for High Technology
led path for India (SRISHTI)
are the goal for the STI
policy.

Which of the above statement/s


is/are correct?

a.
b.
c.
d.

1 only
1 and 2 only
1 and 3 only
1 , 2 and 3

48. Consider
the
following
statements about the Science,

http://insightsonindia.com

Innovation

Which of the statements given above


is/are correct?
a.
b.
c.
d.

1and 2 Only
1,2 and 3Only
3 and 4Only
All Four

49. Consider
the
following
statements about the Jiyo Parsi
scheme
Page 19

Facebook Group: Indian Administrative Service (Raz Kr)

1. It has been launched in


order
to
reverse
the
declining trend of Parsi
population.
2. Financial assistance will be
provided under the scheme
for
investigation
and
detection
of
infertility,
counselling and fertility
treatment to couples.
3. This is a 100% Central
sector scheme

a.
b.
c.
d.

51. Consider
the
following
statements about the National
Urban Livelihoods Mission
(NULM)
1. It is an an improved version
of the earlier poverty
alleviation programme for
the urban poor titled Swarna
Jayanti
Shahari
Rozgar
Yojana (SJSRY)
2. It is being implemented by
the Ministry of Housing and
Urban Poverty Alleviation
(HUPA)

Which of the following statements


is/are correct?
a.
b.
c.
d.

1 and 2 Only
1 and 3 Only
3 Only
All Three

50. Consider the following state


about the Nai Roshni scheme
1. It is launched by the
ministry of women and
family welfare to impart
leadership skills to rural
women
2. This scheme is also known
as Scheme for Leadership
Development of Minority
Women.
Which of the following statements
is/are correct?
http://insightsonindia.com

1 Only
2 Only
Both
None

Which of the statements above is/are


correct?
a.
b.
c.
d.

1 Only
2 Only
Both
None

52. With reference to the Lifestyle


Disease
Education
and
Awareness Programme (LEAP),
consider
the
following
statements

Page 20

Facebook Group: Indian Administrative Service (Raz Kr)

1. The Programme is a joint


initiative of the National
Rural Health Mission and
the State Departments of
Health and Education of
Kerala government
2. The new initiative will
complement
a
peer
programme
titled
Amrutham, Arogyam for
adults in the State.
3. Training for students in
vegetable farm making and
making healthy diet is one of
its main objectives
Which of the statements given above
is/are correct?
a.
b.
c.
d.

1 and 2 Only
2 and 3 Only
All Three
None

53. Consider
statements:

the

following

1. Indian Constitution has


given the status of
national language to
Hindi.
2. Seventh
Schedule
contains the languages
recognized by Indian
Constitution.
http://insightsonindia.com

3. Languages in India are


grouped as scheduled and
non-scheduled languages.
4. In India, states have been
created on the linguistic
basis with the theory one
language- one state
Which of the statements is/are
correct?
a.
b.
c.
d.

3 Only
2 and 3
1, 2 and 4
1, 2 and 3

54. Rise of coalition government at the


centre has led to which of the
following?
1. More autonomy to the States.
2. The federal power sharing has
become more effective
3. Delay in decision making
4. It is one of the casual factors for
instability
Which of the statements is/are
correct?
a. 2 and 3
b. 2, 3 and 4
c. 1, 2 and 4
d. 1, 2, 3 and 4
Page 21

Facebook Group: Indian Administrative Service (Raz Kr)

55. Consider the following with regard


to 73rd and 74th Amendment Act.
1. The panchayat works under the
overall supervision of the gram
sabha.
2. Seats are reserved in the elected
bodies and the executive heads
of these institutions for the
Scheduled Castes, Scheduled
Tribes and Other Backward
Classes.
3. At least one-third of all
positions are reserved for
women and one-sixth for
minorities.
4. An independent institution
called the State Election
Commission has been created in
each
State
to
conduct
panchayat
and
municipal
elections. He can only be
removed by the President of
India.
Which of the above statements is/are
correct?
a.
b.
c.
d.

1 and 3
1, 2 and 4
2 and 3
2, 3 and 4

http://insightsonindia.com

56. The salient features of the


Scheme of Tourist Visa on
Arrival (TVOA) are:
1. The TVOA is available to
foreign nationals arriving
from Japan, USA, UK,
Singapore,
Finland,
Luxembourg, New Zealand,
Cambodia, Laos, Vietnam,
Philippines, Myanmar and
Indonesia.
2. The TVOA is allowed for a
maximum of two times in a
calendar year to a foreigner
with a minimum gap of two
months between each visit.
TVOA is non-extendable
and non-convertible.
3. The TVOA facility is even
applicable to the holders of
Diplomatic/Official
Passports.
Which of the statements given above
is/are correct?
a.
b.
c.
d.

1and 2 Only
2 and 3Only
2 Only
All Three

57. Consider
the
following
statements about Swavalamban
Yojana

Page 22

Facebook Group: Indian Administrative Service (Raz Kr)

1. "NPS -Swavalamban Model"


is designed to ensure ultralow administrative and
transactional costs, so as to
make small investments
viable
2. Open to eligible citizens of
India, in the age group of
4060 years. Subscriber is
free to choose the amount
he/she wants to invest every
year.
3. Regulated by Pension Fund
Regulatory
Development
Authorities (PFRDA)
Which of the statements given above
is/are correct?
a.
b.
c.
d.

1and 2 Only
1 and 3Only
2 Only
All Three

58. With reference to the proposed


the LPG Connection Portability
Scheme, consider the following
statements
1. An LPG consumer in these

markets can now switch to


the distributor of his/her
choice within a cluster of
LPG distributors in the
vicinity under this scheme
http://insightsonindia.com

2. This initiative is expected to

usher in improvement in
customer
service
by
fostering
healthy
competition among each
cluster of distributors.
3. No
Transfer
fee
or
additional security deposit
will be charged for transfer
of connection under the
portability scheme.
Which of the statements given above
is/are correct?
a.
b.
c.
d.

1and 2 Only
1 and 3Only
3 Only
All Three

59. Consider
the
following
statements on water scarcity
and water stress
1. Water scarcity is the lack
of sufficient available
water resources to meet
the demands of water
usage within a region
2. Water stress is the
difficulty of obtaining
sources of fresh water for
use during a period of
time and may result in
further depletion and
deterioration of available
water resources.
Page 23

Facebook Group: Indian Administrative Service (Raz Kr)

3. According to Falken
Mark, a Swedish expert,
water stress occurs when
water availability is less
than 1,000 cubic metre
per person per day.
4. The
per
capita
availability of water in
India is 1545 cubic meters
as per the 2011 census
compared to 1816 cubic
meters in 2011

Select the correct answer using the


codes below.
a.
b.
c.
d.

61. Consider
the
following
statements about pulses
1. India is the largest
producer as well as the
consumer of pulses in the
world.
2. Major pulses that are
grown in India are tur
(arhar), urad, moong,
masur, peas and gram.
3. Being leguminous crops,
all these crops including
arhar help in restoring
soil fertility by fixing
nitrogen from the air

Which of the statements given above


is/are correct?
a.
b.
c.
d.

1,2 and 3 Only


2,3 and 4 Only
3 and 4 Only
All Four

60. Which of the following


statements is/are correct?
1. Aus, Aman and Boro are
three crops of wheat
grown in and around
Punjab and Haryana
2. Cotton, jute, groundnut
and soyabean are mostly
grown during Zaid season
which is cropping season
between Rabi and Kharif
seasons.

http://insightsonindia.com

1 Only
2 Only
Both
None

Which of the statements given above


is/are correct?
a.
b.
c.
d.

1 and 2 Only
2 Only
3 Only
All Three

62. Consider
the
following
statements about the Printing
Press in India:
Page 24

Facebook Group: Indian Administrative Service (Raz Kr)

1. The printing press first came to


Kerala(Cochi) with Portuguese
missionaries in the midsixteenth century.
2. The Bengal Gazette, a weekly
newspaper,
is
the
first
published
Indian
newspaper, brought out by
Gangadhar Bhattacharya, who
was close to Rammohun Roy.

b. 4,5,3,2,1
c. 2,5,4,3,1
d. 2,4,5,3,1

64. To which crop does following


descriptions apply?
1. It
requires
high
temperature, light rainfall
or irrigation, 210 frostfree days and bright
sunshine for its growth.
2. It is a kharif crop and
requires 6 to 8 months to
mature.

Which of the above is/are correct?


a.
b.
c.
d.

1 only
2 only
both 1 and 2
neither 1 nor 2

63. The following energy sources


are the most widely used in
India.

Select the correct answer from


following choices
a.
b.
c.
d.

Sugarcane
Cotton
Tobacco
Jute

1. Coal
2. Petroleum and Natural
Gas
3. Hydroelectricity
4. Nuclear energy
5. Wind energy

65. With reference to the Rashtriya


Bal Swaasthya Karyakram,
consider
the
following
statements,

Which
option
shows
their
arrangement in increasing order of
consumption in India?

1. It is a newly launched
scheme by ministry of HRD
and Social Justice which
assures a package of health

a. 5, 4, 3, 2 ,1
http://insightsonindia.com

Page 25

Facebook Group: Indian Administrative Service (Raz Kr)

over
manufacturing,
packaging, testing and
quality standards, as their
patented/branded
equivalent.
3. Prices
of
the
branded/patented drugs
are generally quite high,
due to the temporary
monopoly
over
the
product, bestowed by the
patent.
4. At Jan Aushadhi Stores',
where quality generic
medicines, which are
equivalent
to
the
expensive branded drugs,
in terms of their potency
and efficacy, are sold at
cheaper prices.

services for children up to 18


years of age.
2. It is part of the National
Rural Health Mission
3. The services under this
programme aim to cover all
children of 0-6 years of age
group in rural areas and
urban slums, in addition to
children enrolled in classes
1st to 12th in Government
and Government aided
schools.
Which of the statements given above
is/are correct?
1.
2.
3.
4.

1 and 2 Only
1 and 3 Only
2 and 3 Only
All Three

66. Consider
the
following
statements on generic drugs
1. A generic drug is a
pharmaceutical product
that is manufactured
without a license from
the innovator company
and marketed after the
expiry date of the patent
or other exclusive rights.
2. Generic drugs are subject
to the same regulations
http://insightsonindia.com

Which of the statements given


above is/are correct?
a.
b.
c.
d.

1,2 and 4 Only


1,2 and 3 Only
1,2 and 4 Only
All Four

67. Consider
the
following
statements about the National
Electric Mobility Mission Plan
(NEMMP) 2020

Page 26

Facebook Group: Indian Administrative Service (Raz Kr)

1. It is a roadmap for
progressive induction and
expansion of efficient and
environmentally
friendly
electric and hybrid vehicles
in the country.
2. It aims to boost 6-7 million
units of new vehicle sales of
the full range of electric
vehicles by year 2020
3. The Mission Plan lays special
emphasis on progressive
indigenization of technology,
localization
of
manufacturing
and
eventually, job creation.
Which of the statements given
above is/are correct?
a.
b.
c.
d.

1 and 2 Only
1 and 3 Only
All Three
None

68. The beneficiaries under the


Antyodaya
Anna
Yojana"
(AAY) include
1. and persons earning their
livelihood on daily basis in
the informal sector like
porters, coolies, rickshaw
pullers, hand cart pullers,

http://insightsonindia.com

fruit and flower sellers,


snake charmers, rag pickers
2. Single women or single men
with no family or societal
support or assured means of
subsistence.
3. Families of HIV positive
persons and all primitive
tribal households
4. Landless
agriculture
labourers, marginal farmers,
rural artisans/craftsmen
Which of the statements given
above is/are correct?
a.
b.
c.
d.

1,2 and 3 Only


1 and 3 Only
1 and 4
All Four

69. Consider
the
following
statements about the Direct
Benefits
Transfer
(DBT)
scheme
1. The scheme is aimed at
cutting the bloated subsidy
bill of Rs.1, 64,000 crores.
2. Aims
to
eliminate
middlemen
in
various
government
sponsored
welfare
schemes
and
subsidized food, fuel and
fertiliser schemes.
Page 27

Facebook Group: Indian Administrative Service (Raz Kr)

3. On the basis of Aadhar cards


money is deposited in
beneficiaries accounts.
4. It would infuse financial
inclusion on a greater scale
in rural India.

a. 1 Only
b. 2 Only
c. Both
d. None
71. Consider
statements,

Which of the statements given


above is/are correct?
a.
b.
c.
d.

the

following

1. Operation Sadhbhavana is
evidence of the Indian
Army's
"organisational
innovation and operational
learning".
2. Youth Employment and
Guidance
Node
(YEGN) programme under
Operation Sadhbhavana was
launched a few years back
in border and remote areas
of Jammu and Kashmir to
enable the youth to get
employment
Which of the statements given above
is/are correct?

http://insightsonindia.com

following

1. The running water cuts


through the clayey soils
and makes deep channels
as gullies. It is known as
gully erosion.
2. Sometimes water flows
as a sheet over large areas
down a slope. In such
cases the top soil is
washed away. This is
known as sheet erosion.
3. Soil erosion is also caused
due to defective methods
of farming.
4. When the land becomes
unfit for cultivation it is
known as bad land.

1,2 and 3 Only


1,3 and 4 Only
1 and 4
All Four

70. Consider
statements

the

Which of the statements given above


is/are correct?
a.
b.
c.
d.

1,2 and 3
2,3 and 4
1,3 and 4
All Four

72. With reference to various


species and their status n the
Page 28

Facebook Group: Indian Administrative Service (Raz Kr)

wild in India, consider the


following statements
1. Great Indian Bustard,
Siberian Crane, Redheaded Vulture and
Forest Owlet are now
listed under critically
endangered birds
2. Among animals, Pygmy
Hog, Kondana Rat and
The
Sumatran
Rhinoceros are listed as
critically endangered
3. The Leatherback turtle is
listed
as
vulnerable
species
Which of the above statements is/are
correct?
a.
b.
c.
d.

1 and 2 Only
2 and 3 Only
1 and 3 Only
All Three

73. Consider the following criteria;


1. Populations
have
declined or will decrease,
by greater than 80% over
the last 10 years or three
generations.
2. Have
a
restricted
geographical range.
http://insightsonindia.com

3. Small population size of


less than 250 individuals
and continuing decline at
25% in 3 years or one
generation.
4. Very small or restricted
population of fewer than
50 mature individuals.
Which of the following categories
would be aptly described by the
above criteria for a species?
a.
b.
c.
d.

Vulnerable
Critically Endangered
Endangered
Near Risk Category

74. Social Justice is one of the


ideals
enshrined
in
the
preamble of the Constitution.
Which of the following are
correct with regard to the social
differences that occur in a
society?
1. All kinds of
social
differences are
based
on
accident
of
birth.
2. Some of the
differences are

Page 29

Facebook Group: Indian Administrative Service (Raz Kr)

based on our
choices.
3. Every social
difference
does not lead
to
social
division.
4. Democracy
always leads
to
disintegration
of society on
the basis of
social
divisions.
Select the correct answer using the
codes below
a.
b.
c.
d.

2 and 3
2, 3 and 4
1, 3 and 4
1, 2 and 4

75. Which of the following are the


measures taken by Government to
reduce social differences in the
society?
1. The government has provided
for 33% reservation for women
in the Parliament.
2. Hindi Language has been made
the national language of the
country.
http://insightsonindia.com

3. The State intervenes in


thematters of religion in order
to ensure equality within
religious communities.
4. Enactment of Mahatma Gandhi
National Rural Employment
Guarantee Act (MGNREGA)
Select the correct answer using the
codes below
a.
b.
c.
d.

1 and 4
3 and 4
1, 2 and 4
1, 2, 3 and 4

76. Communalism is based on the


belief that:
1. People who follow different
religions cannot belong to the
same social community.
2. Any difference that the
followers of a particular religion
may have is irrelevant or trivial
for community life.
3. The fundamental interests of a
particular community are the
same.
4. If the followers of different
religion
have
some
commonalities
these
are
superficial and immaterial.
Which of the statements is/are
correct?
Page 30

Facebook Group: Indian Administrative Service (Raz Kr)

a.
b.
c.
d.

78. Consider
the
statements with
Pressure Groups.

1 and 4
1, 2 and 4
1, 2 and 3
1, 2, 3 and 4

77. Consider
statements:

the

Which of the following statements


is/are correct?
a.
b.
c.
d.

1 Only
1 and 4
1,2 and 4
1, 2, 3 and 4

1. They have political ideology


and take positions on
political issues
2. Their decision making is
more formal and rigid.
3. Their principal concern is
the betterment and wellbeing of their members, not
society in general.
4. They
have
deepened
democracy.
5. These groups are formed
having
uncommon
objectives.

following

1. The Constitution of India


ensures
equality
of
citizens within religious
communities.
2. A
democratic
government is the most
legitimate government
3. Preamble is not an
integral part of the
Constitution
4. Democracy helps in
promoting dignity and
freedom of the individual

following
regard to

Which of
correct?
a.
b.
c.
d.

the

statements

is/are

1, 4
and 5
3, 4 and 5
1, 3 and 4
1, 2, 4 and 5

79. Which of the following is


associated with or true about
Ozone (O3) ?
1. Human exposure
to ozone is not
harmful.

http://insightsonindia.com

Page 31

Facebook Group: Indian Administrative Service (Raz Kr)

2. The
chemicals
used
in
fire
extinguishers
deplete
ozone
layer.
3.The
United
Nations
Environment
Programme
succeeded
in
freezing
the
production
of
ozone-depleting
substances
like
CFCs only at the
beginning of 21st
century.
Select the correct answer using the
codes below
a.
b.
c.
d.

2 and 3
2 only
1 and 3
1,2 and 3

80. The Ganga Action Plan was


being implemented from 1985
to make Ganga pollution-free.
Which of the following
qualities in the Ganga water
would ensure that it is safe for
human uses?

http://insightsonindia.com

1. High concentration of Coliform


bacteria which is found in
human intestine and kills other
harmful micro-organisms.
2. Low pH value of Ganga water
3. Complete absence of all metals
from the Ganga water.
Select the correct answer using the
codes below
a.
b.
c.
d.

1 and 2
2 and 3
2 only
None of the above

81. Who are the stakeholders in the


conservation of forests in India?
1. Manufacturing Industries
2. Tourists and wildlife enthusiasts
3. Forest
Department,
Government of India
4. Forest inhabitants
Select the correct answer using the
codes below
a.
b.
c.
d.

All of the above


2,3 and 4
3 and 4 only
4 only

82. Which of the following


statements have been realized

Page 32

Facebook Group: Indian Administrative Service (Raz Kr)

to be true by communities and


governments around the world?

last five years or more in


succession including the
current year for some
reason or the other .
2. Fallow lands other than
current fallows includes
all land which was taken
up for cultivation but is
temporarily
out
of
cultivation for a period of
not less than one year and
not more than five years.

1. Partnering
local
communities in the
conservation of forests
ensures decentralised
economic growth and
ecological
conservation.
2. Ecological stability of
the area may be
disturbed by banning
traditional methods of
exploitation of forest
produce by the local
communities.

3. Land which cannot be


brought under cultivation
except at an exorbitant
cost is classified as
unculturable
whether
such land is in isolated
blocks
or
within
cultivated holdings.

Select the correct answer using the


codes below
a.
b.
c.
d.

Both
1 only
2 only
None

83. With
reference
to
land
utilization in India, consider the
following definitions
1. Culturable waste land
includes land available
for cultivation, whether
taken up or not taken up
for cultivation once, but
not cultivated during the
http://insightsonindia.com

Which of the statements given above


is/are correct?
a.
b.
c.
d.

1 and 2 Only
2 and 3 Only
1 and 3 Only
All Three

84. Consider
the
following
statements about soils
1. Alluvial soils as a whole
are very fertile. Mostly
Page 33

Facebook Group: Indian Administrative Service (Raz Kr)

these
soils
contain
adequate proportion of
potash, phosphoric acid
and lime which are ideal
for the growth of
sugarcane, paddy, wheat
and other cereal and
pulse crops.
2. Black soils are black in
colour and are also
known as regur soils.
3. Black soils are rich in soil
nutrients, such as calcium
carbonate, magnesium,
potash and lime. These
soils are generally poor in
phosphoric contents.
4. The laterite soil develops
in areas with high
temperature and heavy
rainfall.
Which of the above statements is/are
correct?
a.
b.
c.
d.

1,2 and 3 Only


1,3 and 4 Only
2,3 and 4 Only
All Four

85. In the Indian context, which of


the following statements are
incorrect with respect to the
multi-party system adopted?

http://insightsonindia.com

1. It leads to political
instability.
2. It deepens democracy.
3. It creates a natural
system of checks and
balance
4. It may lead to hung
parliament
Select the correct answers using the
codes below
a.
b.
c.
d.

1, 2 and 3
3 Only
1, 2, 3 and 4
None

86. Democracies that follow a federal


system all over the world tend to have
two kinds of political parties; parties
that are present in only one of the
federal units and parties that are
present in several or all units of the
federation. Which of the following are
the necessary conditions for a party to
be recognized as a national party in
India?
1. The party should secure a
minimum 6% of total valid votes
in the Lok Sabha elections or the
Assembly elections in four or more
States.

Page 34

Facebook Group: Indian Administrative Service (Raz Kr)

2. The party should win a minimum


of 2% of Lok Sabha seats by
contesting from at least 3 states.
3. The party should be recognised as
a state party in minimum of 3
states.
4. The party should have won at least
4 seats in Lok Sabha elections
Select the correct answers using the
codes below
a.
b.
c.
d.

1 and 4
2 and 3
1 Only
None

87. Reforming political parties is one of


the aspects of good governance.
Which of the following is/are a step
in this direction?
1. The Parliament enacted antidefection law
2. On Supreme Courts orders, its
now
mandatory
for
every
candidate who contests elections to
file an Affidavit giving details of his
property and criminal cases
pending against him.
3. The Election Commission passed
an order making it necessary for
political parties to hold their
organizational elections and file
their income tax returns.
http://insightsonindia.com

4. The political parties do not come


under the ambit of Right to
Information Act.
Select the correct answers using the
codes below
a.
b.
c.
d.

1 and 4
1, 2 and 4
1, 2 and 3
1, 2, 3 and 4

88. Consider
the
following
statements about sustainable
development
1. Sustainable
development
ties together concern for
the carrying
capacity of natural systems
with
the
social
and
economic challenges faced
by humanity.
2. The carrying capacity of a
biological species in
an environment is
the
minimum population size of
the
species
that
the
environment can sustain
definitely,
given
the
food, habitat, water and
other necessities available in
the environment

Page 35

Facebook Group: Indian Administrative Service (Raz Kr)

3. Economic
development,
social development, and
environmental
protection
are the interdependent and
mutually reinforcing pillars
of sustainable development
Which of the statements given above
is/are correct?
a.
b.
c.
d.

1 and 2 Only
1 and 3 Only
2 and 3 Only
All Three

89. Which of the following


statements are true concerning
the construction of dams?
1. The vegetation that is
submerged under the
dam,
rots
under
anaerobic conditions and
gives rise to large
amounts of methane.
2. The construction of dams
is preferable in hilly
areas.
3. It disturbs the natural
eco-system
of
the
surrounding areas.
Select the correct answers using the
codes below
a. 1 and 3
http://insightsonindia.com

b. 1,2 and 3
c. 1 and 2
d. Only 3

90. What are the usual constraints


in promoting solar energy at a
large scale in India?
1. The manufacturing of solar
cells is very expensive given
the level of technology India
posses.
2. The energy efficiency of
solar cells is very low.
3. Solar cells require heavy and
costly maintenance.
Select the correct answers using the
codes below
a. 1 and 2
b. 1,2 and 3
c. 1 only
d. 2 and 3
91. Which of the following
statements is/are true?
1. The green plants in a
terrestrial
ecosystem
capture the majority of
the fraction of the solar
energy falling on their
leaves.
Page 36

Facebook Group: Indian Administrative Service (Raz Kr)

2. Only a fraction of the


energy is transferred from
one trophic level to the
other via the food chain.
3. The lower trophic levels
house greater number of
individuals than the
higher levels.
Select the correct answers using the
codes below
a.
b.
c.
d.

1 and 2
2 only
1 and 3
2 and 3

92. Which of the following is true


regarding the process of
Biological Magnification?
1. The
maximum
concentration of the
chemicals is found at the
highest trophic level.
2. The effect of biological
magnification
in
vegetables, fruits etc. can
always be overcome by
washing them properly.
3. The chemical must be
non-biodegradable
for
Biological magnification
to occur.
Select the correct answers using
the codes below
http://insightsonindia.com

a. 1 and 3
b. 1 only
c. 2 and 3
d. 3 only
93. With regard to the necessary
conditions for a party to be
recognized as a State party.
Consider
the
following
statements:
1. A political party should
secure at least 6% of the
total valid votes polled
during general election to a
State Legislative Assembly
and should, in addition, win
at least two seats in that
Assembly or the party
should win at least 3% of the
total number of seats or
three seats in the Legislative
Assembly, whichever is
more.
2. A political party should
secure at least 6% of the
total valid votes polled in a
State during a general
election to Lok Sabha and
win at least one seat in the
Lok Sabha from that State,
or the party should win at
least one seat in the Lok
Page 37

Facebook Group: Indian Administrative Service (Raz Kr)

Sabha for every 25 seats or


any fraction thereof allotted
to that State.
Which of
correct?
a.
b.
c.
d.

the

statements

is/are

1 Only
2 Only
Both 1 and 2
Neither 1 nor 2

c. 1, 3 and 4
d. 1, 2, 3 and 4
95. Which of the following nonmetals form oxides on burning
of fossil fuels:
1. Carbon
2. Nitrogen
3. Sulphur
Select the correct answer using the
codes below

94. With regard to registration of


political parties in India, consider the
following statements:
1. All political parties must register
themselves with the Election
Commission.
2. Not all parties that are registered
with the Election Commission are
recognized by it.
3. The registered parties will get
unique symbols.
4. The registered parties are entitled
for free broadcasting or telecasting
facilities.
Select the correct answers using the
codes below
a. 2 and 4

a. 1 and 2
b. 2 and 3
c. 1 and 3
d. 1,2 and 3
96. Which of the following
statements is/are true?
1. The
availability
of
electrical energy to each
individual in a country is
one of the parameters to
measure the growth of
the country.
2. The transmission of
electricity is less efficient
than transporting coal or
petroleum over the same
distance.

b. 1 and 3
http://insightsonindia.com

Page 38

Facebook Group: Indian Administrative Service (Raz Kr)

d. 1, 2 and 3

Select the correct answer using


the codes below
a. 1 only
b. 2 only
c. Both
d. None
97. Consider
the
following
statements
about
the
Constituent Assembly formed
in 1946:
1. The members of the
Constituent
Assembly
were elected in 1946
directly on the basis of
universal franchise.
2. The
Constituent
Assembly that came into
being was dominated by
the Congress party.
3. The
Congress
party
members
of
the
Constituent
Assembly
were
unanimous
in
theiropinion on all the
critical issues.
Which of the above is/are
incorrect?
a. 1 only
b. 1 and 2 only
c. 1 and 3 only
http://insightsonindia.com

98. In the nineteenth century,


hundreds of thousands of Indian
and Chinese labourers went to
work in various countries
around the world. Consider the
following statements with
regard to the migration of
Indian
labourers:
1. In India, indentured labourers
were hired under contracts
which promised good working
conditions and permanent
settlement in those countries.
2. Most Indian indentured
workers came from the presentday regions of Rajasthan,
Gujarat,
and
Punjab.
3. The labourers were hired to
work on plantations, in mines,
and in road and railway
construction projects around
the
world.
Which of the above is/are
correct?
a.
b.
c.
d.

1 and 2 Only
2 Only
3 Only
1 and 3 Only

99. India had a very rich and old


tradition
of
handwritten
Page 39

Facebook Group: Indian Administrative Service (Raz Kr)

manuscripts. Which of the


following statement/s about
the
manuscripts
is/are
incorrect?
1.The manuscripts were written
in Sanskrit, Arabic, Persian only
and never in any of the
vernacular
languages.
2.Manuscripts ceased to be
produced well before the
introduction
of
print.
3.Manuscripts were highly
expensive and were not widely
used
in
everyday
life.
Select the right answer using
the
codes
below
a.
b.
c.
d.

1 and 2Only
2 Only
3 Only
1
and

http://insightsonindia.com

100.
Consider the following
statements about Raja Ravi
Varma :
1. Raja Ravi Varma achieved
recognition for his depiction of
scenes from the epics of the
Mahabharata and Ramayana.
2. Raja Ravi Varma produced
images for mass circulation.
3. Ravi Varma's prints began
shaping popular ideas about
modernity
and
tradition,
religion and politics, and society
and culture.
Which of the above is/are correct?
a.
b.
c.
d.

2 only
2 and 3 only
1 and 3 only
1, 2 and 3

Only

Page 40

Facebook Group: Indian Administrative Service (Raz Kr)

INSIGHTS MOCK EXAM-3 SOLUTIONS


1. Solution: c)
There are Five waterways in India. For complete details and Maps, click the link
below.
http://iwai.nic.in/index1.php?lang=1&level=1&sublinkid=124&lid=143

2. Solution: d)
Two different sets of reasons can be given in favour of power sharing. Firstly,
power sharing is good because it helps to reduce the possibility of conflict between
social groups. Since social conflict often leads to violence and political instability,
power sharing is a good way to ensure the stability of political order (so no
arbitrariness, instability or divisiveness).
Power sharing is the very spirit of democracy. A democratic rule involves sharing
power with those affected by its exercise, and who have to live with its effects.
People have aright to be consulted on how they are to be governed. A legitimate
government is one where citizens, through participation, acquire a stake in the
system.
Reference: Page No. 6 (Democratic Politics Part 2)
3. Solution: b)
Chap14- page 243, NCERT 10th Science
4. Solution (d)
More than other forms of writing which came before, novels are about ordinary
people. They do not focus on the lives of great people or actions that change the
destinies of states and empires. Instead, they are about the everyday life of common
people. In the nineteenth century, Europe entered the industrial age. Factories came
up, business profits increased and the economy grew. But at the same time, workers
faced problems. Cities expanded in an unregulated way and were filled with
overworked and underpaid workers. The unemployed poor roamed the streets for
jobs, and the homeless were forced to seek shelter in workhouses. The growth of
industry was accompanied by an economic philosophy which celebrated the
pursuit of profit and undervalued the lives of workers. Deeply critical of these
http://insightsonindia.com

INSIGHTS

Page 1

Facebook Group: Indian Administrative Service (Raz Kr)

INSIGHTS MOCK EXAM-3 SOLUTIONS


developments, novelists such as Charles Dickens wrote about the terrible effects of
industrialisation on peoples lives and characters. Dickens focused on the terrible
conditions of urban life under industrial capitalism. His Oliver Twist (1838) is the
tale of a poor orphan who lived in a world of petty criminals and beggars.
Reference : Page No. 181 (India and Contemporary World II), Class X)

5. Solution: a)
Iron deficiency anaemia during adolescence can impair physical growth,
cognitive development, reduce physical fitness and energy levels and can affect
concentration and work performance. Iron deficiency in girls has more serious
health consequences. It can have impact on their entire life cycle. Anaemic girls
have lower pre pregnancy stores of iron. Anaemic adolescent girls have a higher
risk of preterm delivery and having babies with low weight. Anaemia in
adolescent girls also increases the risk of maternal death. One third of all the
material deaths take place in young women in the age group of 15 to 24 years.
Therefore regular consumption of iron folic acid supplements along with
diet rich in micronutrients is essential for prevention of iron deficiency anaemia in
adolescent girls and boys.
It is in this context that a nation-wide Weekly Iron and Folic Acid
Supplementation (WIFS) programme was launched in January this year to address
this critical health issue.
The Weekly Iron and Folic Acid Supplementation (WIFS) programme is
currently reaching out to 13 crore school-going girls and boys (class VI XII) and
out-of-school adolescent girls in government/ aided and municipal schools
and Anganwadi Centres across all states in India.
There have been a few reports of minor side affects like nausea and
vomiting in the recent past although guidelines have been issued on the
consumption of these iron folic acid tablets to prevent side affect like nausea.
When iron tablet is taken for the first time, the body may find it little difficult to
digest and symptoms such as stomachache and nausea may occur. However, if
taken after food, the absorption will be little low but stomach ache and nausea will
not occur. These side effects will eventually disappear once the tablet is regularly
http://insightsonindia.com

INSIGHTS

Page 2

Facebook Group: Indian Administrative Service (Raz Kr)

INSIGHTS MOCK EXAM-3 SOLUTIONS


taken for a few weeks as the body adjusts to the iron tablets. Hard stools after
consuming iron-folic acid tablet are harmless. The body takes the iron it needs and
the extra iron is removed through faeces. To reduce side affects IFA tablets should
be taken on full stomach. Taking any vitamin or nutrient is never restricted during
an illness. In fact, it helps speedy recovery from illness by improving immunity of
the body. Iron folic acid tablet can be taken during illness and even during
menstruation

6. Solution: b)
In order to strengthen the Pahchayati Raj system and also to address critical
gaps that constrain it, Ministry of Panchayati Raj has formulated the Scheme, Rajiv
Gandhi Panchayat Sashaktikran Abhiyan (RGPSA) which will be implemented
during the Twelfth Five Year Plan period. RGPSA aims to enable States to
strengthen their Panchayati Raj systems in their context by choosing from among a
menu of activities. States would have access to funds on the basis of perspective
and annual plans prepared under the scheme. States would be required to fulfill the
following essential conditions for accessing to funds:
Regular elections to panchayats or urban local bodies under the
superintendence of SEC
1/3 reservation for women in panchayats or other local bodies
Constitution of State Finance Commission(SFC) every five years and
placement of Action Taken Report (ATR) on the recommendations of the
SFC in the state Assembly
Constitution of District Planning Committees in all districts
RGPSA will provide performance linked funds from 2014-15 onwards.
Activities that would be supported under RGPSA include: Strengthening of State
Election Commission; Administrative and Technical Support at the Gram
Panchayat level; Construction /renovation of Gram Panchayat Buildings; Capacity
Building and Training of Elected Representatives and Functionaries; Institutional
Structure for Training at State, District & Block level; E-enablement of Panchayats,
etc.
The funding of RGPSA for State plans is envisaged on a 75:25 sharing basis by the
Central and State Governments respectively. For NE States, the ratio will be 90:10.

http://insightsonindia.com

INSIGHTS

Page 3

Facebook Group: Indian Administrative Service (Raz Kr)

INSIGHTS MOCK EXAM-3 SOLUTIONS


7. Solution (d)
Sarojini Naidu, also known by the sobriquet as The Nightingale of India. She was a
child prodigy, Indian independence activist and poet. Sarojini Naidu was one of the
framers of the Indian Constitution and the first Governor of the United Provinces
from 1947 to 1949, the first woman to become the governor of an Indian state. Her
birthday is celebrated as Women's Day all over India.
Reference: Wikipedia

8. Solution (b)
Stories in prose were not new to India. Banabhattas Kadambari, written in Sanskrit
in the seventh century, is an early example. The Panchatantra is another. There was
also a long tradition of prose tales of adventure and heroism in Persian and Urdu,
known as dastan. However, these works were not novels as we know them today.
Prose is a form of language which applies ordinary grammatical structure and
natural flow of speech rather than rhythmic structure (as in traditional
poetry).Prose benefits the more informal metrical structure of verse that is almost
always found in traditional poetry. Poems usually involve a meter and/or rhyme
scheme. Prose, instead, comprises full, grammatical sentences, which then
constitute paragraphs and overlook aesthetic appeal.

9. Solution: c)
Resource planning is a complex process which involves : (i) identification and
inventory of resources across the regions of the country. This involves surveying,
mapping and qualitative and quantitative estimation and measurement of the
resources. (ii) Evolving a planning structure endowed with appropriate technology,
skill and institutional set up for implementing resource development plans. (iii)
Matching the resource development plans with overall national development plans.
India has made concerted efforts for achieving the goals of resource planning right
from the First Five Year Plan launched after Independence.
The availability of resources is a necessary condition for the development of any
region, but mere availability of resources in the absence of corresponding changes in
http://insightsonindia.com

INSIGHTS

Page 4

Facebook Group: Indian Administrative Service (Raz Kr)

INSIGHTS MOCK EXAM-3 SOLUTIONS


technology and institutions may hinder development. There are many regions in
our country that are rich in resources but these are included in economically
backward regions. On the contrary there are some regions which have a poor
resource base but they are economically developed.

10. Solution: d)
The Limits to Growth is a 1972 book about the computer modeling of exponential
economic and population growth with finite resource supplies. The purpose of The
Limits to Growth was not to make specific predictions, but to explore how
exponential growth interacts with finite resources. Because the size of resources is
not known, only the general behavior can be explored.
The Club of Rome is a global think tank that deals with a variety of international
political issues. The Club of Rome raised considerable public attention with its
report Limits to Growth, which has sold 12 million copies in more than 30
translations, making it the best-selling environmental book in world history.

11. Solution (a)


Power is shared among different organs of government, such as the legislature,
executive and judiciary. This horizontal distribution of power because it allows
different organs of government placed at the same level to exercise different
powers. Such a separation ensures that none of the organs can exercise unlimited
power. Each organ checks the others. This results in a balance of power among
various institutions.
Reference: Page No. 8 (Democratic Politics Part 2)
12. Solution (a)
Power is shared among different organs of government, such as the legislature,
executive and judiciary. This horizontal distribution of power because it allows
different organs of government placed at the same level to exercise different
powers. Such a separation ensures that none of the organs can exercise unlimited
http://insightsonindia.com

INSIGHTS

Page 5

Facebook Group: Indian Administrative Service (Raz Kr)

INSIGHTS MOCK EXAM-3 SOLUTIONS


power. Each organ checks the others. This results in a balance of power among
various institutions.
Even though ministers and government officials exercisepower, they are
responsible to the Parliament or State Assemblies.
Similarly, although judges are appointed by the executive, they can check the
functioning of executive or laws made by the legislatures. This arrangement is
called a system of checks and balances.
Power can be shared among governments at different levels a general government
for the entire country and governments at the provincial or regional level. Such a
general government for the entire country is usually called federal
government.
Power may also be shared among different social groups, such as the religious and
linguistic groups. Community government in Belgium is a good example of this
arrangement.
Reference: Page No. 8, 9 (Democratic Politics Part 2)

13. Solution: d)
Concave mirrors converge the oncoming light beam to focus it on one point. It is
important to focus all the energy on one single point for generating the maximum
heat. Convex mirrors, on the other hand, diverge the energy beam. So, eneegy gets
de-concentrated instead of getting concentrated. Plane mirrors simply reflect the
energy beam without any divergence or convergence.

14. Solution: c)
Chap14- Page 250, NCERT 10th Science
15. Solution (c)
The modern novel form developed in India in the nineteenth century, as Indians
became familiar with the Western novel. The development of the vernaculars,
print and a reading public helped in this process. Some of the earliest Indian novels
http://insightsonindia.com

INSIGHTS

Page 6

Facebook Group: Indian Administrative Service (Raz Kr)

INSIGHTS MOCK EXAM-3 SOLUTIONS


were written in Bengali and Marathi. The earliest novel in Marathi was Baba
Padmanjis Yamuna Paryatan (1857), which used a simple style of storytelling to
speak about the plight of widows.
Reference : Page No. 187 (India and Contemporary World II, Class X)
16. Solution (a)
Indian novelists of the nineteenth century wrote for a cause. Colonial rulers
regarded the contemporary culture of India as inferior.
On the other hand, Indian novelists wrote to develop a modern literature of the
country that could produce a sense of national belonging and cultural equality with
their colonial masters.
Reference : Page No. 187(India and Contemporary World II, Class X)

17. Solution (d)


The modern novel form developed in India in the nineteenth century, as Indians
became familiar with the Western novel; there was no fear of Western culture
invading Indian culture which could hinder the development of novel in India.
The development of the vernaculars, print and a reading public helped in this
process.
Translations of novels into different regional languages helped to spread the
popularity of the novel and stimulated the growth of the novel in new areas.
In the north, Bharatendu Harishchandra, the pioneer of modern Hindi literature,
encouraged many members of his circle of poets and writers to recreate and
translate novels from other languages. Many novels were actually translated and
adapted from English and Bengali under his influence.
Reference : Page No. 187, 188 (India and Contemporary World II, Class X)
18. Solution: c)
Agenda 21 is a non-binding, voluntarily implemented action plan of the United
Nationswith regard to sustainable development It is a product of the UN
Conference on Environment and Development (UNCED) held in Rio de
http://insightsonindia.com

INSIGHTS

Page 7

Facebook Group: Indian Administrative Service (Raz Kr)

INSIGHTS MOCK EXAM-3 SOLUTIONS


Janeiro, Brazil, in 1992. It is an action agenda for the UN, other multilateral
organizations, and individual governments around the world that can be executed
at local, national, and global levels. The "21" in Agenda 21 refers to the 21st Century.
It has been affirmed and modified at subsequent UN conferences.
In 2012, at the United Nations Conference on Sustainable Development the
attending members reaffirmed their commitment to Agenda 21 in their outcome
document called "The Future We Want". 180 leaders from nations participated.

19. Solution. (b)


Customs duties were raised and income tax introduced to finance the war.
People hoped that their hardships would end after the war was over. But that did
not happen. In 1918-19 and 1920-21, crops failed in many parts of India, resulting in
acute shortages of food. This was accompanied by an influenza epidemic.
Reference: Page 54 India and the Contemporary World II 10th NCERT.

20. Solution.(c)
Mahatma Gandhi first experimented with Satyagraha in South Africa in his fight
against the racist regime. Satyagraha theory influenced Nelson Mandela's struggle in
South Africa under apartheid, Martin Luther King, Jr.'s and James Bevel's
campaigns during the civil rights movement in the United States, and many other
social justice and similar movements.
Reference: Page 55 India and the Contemporary World II 10th NCERT.
21. Solution. (d)
On that day a large crowd gathered in the enclosed ground of Jallianwalla Bagh.
Some came to protest against the governments new repressive measures. Others
had come to attend the annual Baisakhi fair.
The Government held an inquiry, chaired by Lord Hunter. While it was not
uncritical of Dyer, it held back from outright censure, and the Indian members of
his commission issued a more critical minority report.
http://insightsonindia.com

INSIGHTS

Page 8

Facebook Group: Indian Administrative Service (Raz Kr)

INSIGHTS MOCK EXAM-3 SOLUTIONS


British newspapers in India attacked the findings of the Hunter commission and
dismissed as mere politicking the views of Indians demanding justice and
compensation. The British media strongly defended General Dyer (Source:internet)
Reference: Page 56 India and the Contemporary World II 10th NCERT.
22. Solution (c)
The Fundamental Rights in the constitution are important because they are
enforceable. We have a right to seek the enforcement of the above mentioned
rights. This is called the Right to Constitutional Remedies. This itself is a
Fundamental Right. This right makes other rights effective. It is possible that
sometimes our rights may be violated by fellow citizens, private bodies or by the
government. When any of our rights are violated we can seek remedy through
courts. If it is a Fundamental Right we can directly approach the Supreme Court or
the High Court of a state.
Courts also enforce the Fundamental Rights against private individuals and bodies.
The Supreme Court and High Courts have the power to issue directions, orders or
writs for the enforcement of the Fundamental Rights. They can also award
compensation to the victims and punishment to the violators.
Only the High Courts have the right to issue orders for the enforcement of both
Fundamental Rights and Legal rights, Supreme Court can issue orders or writs only
for the enforcement of the Fundamental right and not other rights.
Reference: Page No. 107, 108, 109 (Democratic Politics -I)

23. Solution: c
Sustainable development concept never says that the imperatives of economic
growth be neglected altogether. What it says that the needs of the coming
generation too should be taken care of in the allocation and use of resources.

24. Solution: c
Chap-14- Page 252, NCERT 10th Science

http://insightsonindia.com

INSIGHTS

Page 9

Facebook Group: Indian Administrative Service (Raz Kr)

INSIGHTS MOCK EXAM-3 SOLUTIONS


25. Solution: d
Chap-14- Page 252, NCERT 10th Science

26. Solution: a)
Under the Special Industry Initiative of the Prime Minister, the National Skills
Development Corporation (NSDC) and Ministry of Home Affairs have been
mandated to work with the corporate sector in bringing about a positive change in
the employment and skills space of Jammu and Kashmir. The Special Industry
Initiative, known as Udaan, targets the youth of J&K, specifically graduates and
postgraduates, who are seeking global and local opportunities. Udaan thereby aims
to provide skills to 40 000 youth over a period of 5 years in high growth
sectors.Udaan has two objectives :
To provide exposure to the graduates and post graduates of Jammu and Kashmir to
the best of corporate India and
To provide corporate India with exposure to the rich talent pool available in the
state
27. Solution: a)
I respect women campaign:
The Ministry of Tourism organized a National Conference of State Tourism
Ministers on 18th July, 2013 in New Delhi.
The meeting passed the unanimous Resolution that the Departments of
Tourism of all States and Union Territories will work for ensuring safety and
security of the tourists especially of women tourists. Ministry of Tourism has also
launched a new campaign for the safety of women titled I respect women.
28. Solution: b)
The Eleventh Plan document mentions that the Plan will aim for inclusive growth
by introducing National Urban Health Mission (NUHM), which along with
National Rural Health Mission (NRHM) will form the Sarva Swasthya Abhiyan.
http://insightsonindia.com

INSIGHTS

Page 10

Facebook Group: Indian Administrative Service (Raz Kr)

INSIGHTS MOCK EXAM-3 SOLUTIONS


The Union Cabinet gave its approval to launch a National Urban Health
Mission (NUHM) as a new sub-mission under the over-arching National Health
Mission (NHM). Under the Scheme the following proposals have been approved :
1.
2.
3.
4.

One Urban Primary Health Centre (U-PHC) for every fifty to sixty
thousand population.
One Urban Community Health Centre (U-CHC) for five to six UPHCs in big cities.
One Auxiliary Nursing Midwives (ANM) for 10,000 population.
One Accredited Social Health Activist ASHA (community link
worker) for 200 to 500 households.

29. Solution (c)


Concurrent List includes subjects of common interest to both the Union
Government as well as the State Governments, such as education, forest, trade
unions, marriage, adoption and succession. Union List includes subjects of national
importance such as defence of the country, foreign affairs, banking,
communications and currency. They are included in this list because we need a
uniform policy on these matters throughout the country. State List contains
subjects of State and local importance such as police, trade, commerce, agriculture
and irrigation.
The Constitution originally provided for a two-tier system of government, the
Union Government or what we call the Central Government, representing the
Union of India and the State governments. Later, a third tier of federalism was
added in the form of Panchayats and Municipalities.
All States in the Indian Union do not have identical powers. Some States enjoy a
special status. Jammu and Kashmir has its own Constitution. Many provisions of
the Indian Constitution are not applicable to this State without the approval of the
State Assembly. Indians who are not permanent residents of this State cannot buy
land or house here. Similar special provisions exist for some other States of India as
well.
There are some units of the Indian Union which enjoy very little power.
These are areas which are too small to become an independent State but which
could not be merged with any of the existing States. These areas, like
Chandigarh, or Lakshadweep or the capital city of Delhi, are called Union
Territories. These territories do not have the powers of a State. The Central
Government has special powers in running these areas.
Reference: Page No. 16, 17 (Democratic Politics Part 2)
http://insightsonindia.com

INSIGHTS

Page 11

Facebook Group: Indian Administrative Service (Raz Kr)

INSIGHTS MOCK EXAM-3 SOLUTIONS

30. Solution (b)


Federalism is a system of government in which the power is divided between a
central authority and various constituent units of the country.
Key features of federalism:
1 There are two or more levels (or tiers) of government.
2 Different tiers of government govern the same citizens, but each tier has its own
jurisdiction in specific matters of legislation, taxation and administration.
3 The jurisdictions of the respective levels or tiers of government are specified in
the constitution. So the existence and authority of each tier of government is
constitutionally guaranteed.
4 The fundamental provisions of the constitution cannot be unilaterally changed by
one level of government. Such changes require the consent of both the levels of
government.
5 Courts have the power to interpret the constitution and the powers of different
levels of government. The highest court acts as an umpire if disputes arise between
different levels of government in the exercise of their respective powers.
6 Sources of revenue for each level of government are clearly specified to ensure its
financial autonomy.
Reference: Page No. 15 (Democratic Politics Part 2)

31. Solution (a)


Bankim read out Durgeshnandini (1865), his first novel, to such a gathering of
people who were stunned to realise that the Bengali novel had achieved excellence
so quickly. Besides the ingenious twists and turns of the plot and the suspense, the
novel was also relished for its language. The prose style became a new object of
enjoyment. Initially the Bengali novel used a colloquial style associated with urban
life. It also used meyeli, the language associated with womens speech. This style
was quickly replaced by Bankims prose which was Sanskritised but also contained
a more vernacular style. The novel rapidly acquired popularity in Bengal. By the
twentieth century, the power of telling stories in simple language made Sarat
http://insightsonindia.com

INSIGHTS

Page 12

Facebook Group: Indian Administrative Service (Raz Kr)

INSIGHTS MOCK EXAM-3 SOLUTIONS


Chandra Chattopadhyay (1876-1938) the most popular novelist in Bengal and
probably in the rest of India.
Vandhe Mataram was taken from Bankim Chandra Chattopadhyays Anandamath.
Reference : Page No. 190 (India and Contemporary World II, Class X)
32. Solution (c)
Colonial administrators found vernacular novels a valuable source of information on
native life and customs. Such information was useful for them in governing Indian
society, with its large variety of communities and castes. As outsiders, the British
knew little about life inside Indian households. The new novels in Indian languages
often had descriptions of domestic life. They showed how people dressed, their
forms of religious worship, their beliefs and practices, and so on. Some of these
books were translated into English, often by British administrators or Christian
missionaries. Indians used the novel as a powerful medium to criticise what they
considered defects in their society and to suggest remedies. Writers like
Viresalingam used the novel mainly to propagate their ideas about society among a
wider readership.
Novels also helped in establishing a relationship with the past. Many of them told
thrilling stories of adventures and intrigues set in the past. Through glorified
accounts of the past, these novels helped in creating a sense of national pride among
their readers. At the same time, people from all walks of life could read novels so
long as they shared a common language. This helped in creating a
sense of collective belonging on the basis of ones language.
Reference : Page No. 191 (India and Contemporary World II, Class X)

33. Solution: c)
The Himalayan Yew in trouble
The Himalayan Yew (Taxus wallachiana) is a medicinal plant found in various
parts of Himachal Pradesh and Arunachal Pradesh. A chemical compound called
taxol is extracted from the bark, needles, twigs and roots of this tree, and it has
been successfully used to treat some cancers the drug is now the biggest selling
http://insightsonindia.com

INSIGHTS

Page 13

Facebook Group: Indian Administrative Service (Raz Kr)

INSIGHTS MOCK EXAM-3 SOLUTIONS


anti-cancer drug in the world. The species is under great threat due to overexploitation. In the last one decade, thousands of yew trees have dried up in various
parts of Himachal Pradesh and Arunachal Pradesh.

34. Solution: c)
Habitat destruction, hunting, poaching, over-exploitation, environmental pollution,
poisoning and forest fires are factors, which have led to the decline in Indias
biodiversity. Other important causes of environmental destruction are unequal
access, inequitable consumption of resources and differential sharing of
responsibility for environmental well-being. Over-population in third world
countries is often cited as the cause of environmental degradation. However, an
average American consumes 40 times more resources than an average Somalian.
Similarly, the richest five per cent of Indian society probably cause more ecological
damage because of the amount they consume than the poorest 25 per cent. The
former shares minimum responsibilities for environmental well-being.

35. Solution: b)
In India, much of its forest and wildlife resources are either owned or managed by
the government through the Forest Department or other government departments.
These are classified under the following categories.
(i) Reserved Forests: More than half of the total forest land has been declared
reserved forests. Reserved forests are regarded as the most valuable as far as the
conservation of forest and wildlife resources are concerned.
(ii) Protected Forests: Almost one-third of the total forest area is protected forest,
as declared by the Forest Department. This forest land are protected from any
further depletion.
(iii) Unclassed Forests: These are other forests and wastelands belonging to both
government and private individuals and communities. Reserved and protected
http://insightsonindia.com

INSIGHTS

Page 14

Facebook Group: Indian Administrative Service (Raz Kr)

INSIGHTS MOCK EXAM-3 SOLUTIONS


forests are also referred to as permanent forest estates maintained for the purpose
of producing timber and other forest produce, and for protective reasons.
Madhya Pradesh has the largest area under permanent forests, constituting 75 per
cent of its total forest area.
Jammu and Kashmir, Andhra Pradesh, Uttaranchal, Kerala, Tamil Nadu, West
Bengal, and Maharashtra have large percentages of reserved forests of its total forest
area whereas Bihar, Haryana, Punjab, Himachal Pradesh, Orissa and Rajasthan have
a bulk of it under protected forests. All North-eastern states and parts of Gujarat
have a very high percentage of their forests as un-classed forests managed by local
communities.

36. Solution (a)


The first proper modern novel was written by Srinivas Das of Delhi. Srinivas Dass
novel, published in 1882, was titled Pariksha-Guru (The Master Examiner). It
cautioned young men of well-to-do families against the dangerous influences of bad
company and consequent loose morals. Pariksha-Guru reflects the inner and outer
world of the newly emerging middle classes. The characters in the novel are caught
in the difficulty of adapting to colonised society and at the same time preserving
their own cultural identity.
Reference : Page No. 188 (India and Contemporary World II, Class X)

37. Solution: c
Chap-16- Page 275, NCERT 10th Science

38. Solution: b
Chap-16- Page 276 , NCERT 10th Science
( Groundwater sources can not always be free of contaminants. However, it usually
contains lesser contaminants.)
http://insightsonindia.com

INSIGHTS

Page 15

Facebook Group: Indian Administrative Service (Raz Kr)

INSIGHTS MOCK EXAM-3 SOLUTIONS

39. Solution: d
Chap-16- Page 274-275, NCERT 10th Science

40. Solution.(a)
Explanation: Rowlatt Act gave the government enormous powers to repress
political activities, and allowed detention of political prisoners without trial for two
years.
Khilaf issue, the repressive Rowlatt act and the Jallianwalla Bagh massacre were
the 3 important causes for the Non-Cooperation Movement.
Reference : page No.55-56 chapter 3 (India and the Contemporary World II)

41.

Solution.(d)

Under the Inland Emigration Act of 1859, plantation workers were not
permitted to leave the tea gardens without permission. When they heard of the
Non-Cooperation Movement, thousands of workers defied the authorities, left the
plantations and headed home. They believed that Gandhi Raj was coming and
everyone would be given land in their own villages.
Tribal peasants interpreted the message of Mahatma Gandhi and the idea of swaraj
in yet another way. In the Gudem Hills of Andhra Pradesh, for instance, a militant
guerrilla movement spread in the early 1920s under the leadership of Alluri
Sitaramaraju.
The movement drew into its fold the struggles of peasants and tribals.In Awadh,
peasants were led by Baba Ramchandra a sanyasi led the movement against
talukdars and landlords who demanded from peasants exorbitantly high rents and a
variety of other cesses.
Reference page 58, chapter 3 (India and the Contemporary World II)
http://insightsonindia.com

INSIGHTS

Page 16

Facebook Group: Indian Administrative Service (Raz Kr)

INSIGHTS MOCK EXAM-3 SOLUTIONS

42. Solution: b)
The National e-Governance Plan (NeGP), being coordinated by
the Department of Electronics & Information Technology (DeitY), Ministry of
Communications & Information Technology, was approved in May 2006 with a
vision to Make all Government services accessible to the common man in his
locality, through common service delivery outlets and ensure efficiency,
transparency & reliability of such services at affordable costs to realise the basic
needs of the common man. Originally, the NeGP comprised of 27 Mission
Mode Projects (MMPs) and 8 components. However, in 2011 it was augmented
by adding 4 more projects - Health, Education, PDS and Posts to increase the
list of 27 MMPs to 31 MMPs.
An MMP is an individual project within the NeGP that focuses on one
aspect of electronic governance, such as banking, land records or commercial
taxes etc. Within NeGP, Mission Mode implies that projects have clearly
defined objectives, scopes, and implementation timelines and milestones, as
well as measurable outcomes and service levels.
The 31 MMPs comprising the NeGP are further classified as State,
Central or Integrated projects. Each state government can also define five MMPs
specific to its individual needs.
The List of the 3 types of MMPs is as follows:
Central MMPs under NeGP
Banking
Central Excise & Customs
Income Tax (IT)
Insurance
MCA21
Passport
Immigration,Visa and Foreigners Registration& Tracking
Pension
e-Office
Posts
UID

http://insightsonindia.com

INSIGHTS

Page 17

Facebook Group: Indian Administrative Service (Raz Kr)

INSIGHTS MOCK EXAM-3 SOLUTIONS


State MMPs under NeGP
Agriculture
Commercial Taxes
eDistrict
Employment Exchange
Land Records(NLRMP)
Municipalities
e-Panchayats
Police(CCTNS)
Road Transport
Treasuries Computerization
PDS
Education
Health
Integrated MMPs under NeGP
CSC
e-Biz
e-Courts
e-Procurement
EDI For eTrade
National e-governance Service Delivery Gateway
India Portal

43. Solution: a)
The scheme of Seekho aur Kamao is the Skill Development initiative. It is a 100%
Central Sector Scheme for Skill Development of minorities to be implemented
from the current financial year onwards. It is being implemented by the ministry of
minority affairs.

44. Solution.(b).
http://insightsonindia.com

INSIGHTS

Page 18

Facebook Group: Indian Administrative Service (Raz Kr)

INSIGHTS MOCK EXAM-3 SOLUTIONS

Lord Irwin announced in October 1929, a vague offer of dominion status for
India in an unspecified future.
All the political parties in India including the Muslim League participated in Go
back Simon demonstrations.
Reference: page 62, chapter 3 (India and the Contemporary World II)

45.

Solution.(c)

Reference: page 96-97 chapter 4 (India and the Contemporary World II)

46. Solution: a)
In India joint forest management (JFM) programme furnishes a good example for
involving local communities in the management and restoration of degraded forests.
The programme has been in formal existence since 1988 when the state of Orissa
passed the first resolution for joint forest management. JFM depends on the
formation of local (village) institutions that undertake protection activities mostly
on degraded forest land managed by the forest department. In return, the members
of these communities are entitled to intermediary benefits like non-timber forest
produces and share in the timber harvested by successful protection.

47. Solution.(a)
Dr B.R. Ambedkar, who organised the dalits into the Depressed Classes Association
in 1930, clashed with Mahatma Gandhi at the second Round Table Conference
by demanding separate electorates for dalits. When the British government
conceded Ambedkar s demand, Gandhiji began a fast unto death. He believed that
separate electorates for dalits would slow down the process of their integration into
society. Ambedkar ultimately accepted Gandhiji s position and the result
was
the Poona Pact of September 1932.It gave the Depressed Classes (later to be known
as the Schedule Castes) reserved seats in provincial and central legislative councils,
but they were to be voted in by the general electorate.
http://insightsonindia.com

INSIGHTS

Page 19

Facebook Group: Indian Administrative Service (Raz Kr)

INSIGHTS MOCK EXAM-3 SOLUTIONS


Reference:page 68 chapter 3 (India and the Contemporary World II)

48. Solution: d)
http://pib.nic.in/newsite/erelease.aspx?relid=91316

49. Solution: d)
http://minorityaffairs.gov.in/sites/upload_files/moma/files/jiyoparsis_0.pdf
50. Solution: b)
http://articles.timesofindia.indiatimes.com/2013-0106/lucknow/36173792_1_minority-women-muslim-women-sachar-committee
http://minorityaffairs.gov.in/sites/upload_files/moma/files/Revised%20Leadership.
pdf

51. Solution: c)
The NULM is actually an improved version of the earlier poverty alleviation
programme for the urban poor titled Swarna Jayanti Shahari Rozgar Yojana
(SJSRY) which was found to be wanting for a variety of reasons.
Having identified the problem areas in the implementation of the SJSRY and
recognising the need to address the needs of the growing population flocking to
cities in search of a better livelihood, the Ministry of Housing and Urban Poverty
Alleviation (HUPA) restructured the old scheme and unveiled it in its new avatar
as the NULM.
To begin with, the UPA government did not hold back on the funding. A
budget of a whopping Rs. 6404 crore has been provided for the remaining period of
the 12th five year plan for cities with a population of one lakh or more with Rs. 950
crore being allocated for 2013-2014.
http://insightsonindia.com

INSIGHTS

Page 20

Facebook Group: Indian Administrative Service (Raz Kr)

INSIGHTS MOCK EXAM-3 SOLUTIONS


It has also expanded the beneficiaries of urban poor to include the homeless
and street vendors who are invariably ignored in government programmes. A
special provision has been made for the funding of all-weather 24/7 shelters with all
essential facilities for the urban homeless. In addition, up to five per cent of the
NULM budget has been earmarked to provide support to urban street vendors
which will include skill upgradation and development of vendor markets.

52. Solution: c)
The Programme is a joint initiative of the National Rural Health Mission and the
State Departments of Health and Education. The new initiative will complement a
peer programme titled Amrutham, Arogyam for adults in the State. About 70 lakh
adults were screened as a part of Amrutham, Arogyam programme.
Changing Lifestyle and its Multiple Effects
The World Health Assembly organized by World Health
Organisation(WHO) in 2010 at Geneva cautioned in its report that non
communicable diseases -mainly cardiovascular diseases, cancers, chronic respiratory
diseases and diabetes, kill nearly 35 million people per year. The report also says
that almost 90% of fatalities before the age of 60 occur in developing countries and
are largely preventable. (Courtesy: WHO website)
Unhealthy diet, lack of exercise, stress and excess use of junk food are
the main causes that lead to lifestyle diseases at an early age. The cut-throat
competition in academics forces students to distance themselves from sporting
activities in and outside the school premises. India with its majority young
population is more vulnerable to such non-communicable diseases.
LEAP- An Introduction
The LEAP initiative will help educate teachers and students on lifestyle
diseases. Continuous awareness campaigns will be conducted across the schools as
a part of this programme. Medical Officers, dietitians and other health workers will
lead the classes which lay emphasis on healthy diet, exercise and good eating habits.
Main Objectives of LEAP:
Awareness classes for students and teachers on regular exercise and healthy
diet.
Facilities for exercise at schools.
http://insightsonindia.com

INSIGHTS

Page 21

Facebook Group: Indian Administrative Service (Raz Kr)

INSIGHTS MOCK EXAM-3 SOLUTIONS


Encourage students to reach schools either by walking or by cycling.
Training for students in vegetable farm making and making healthy diet.
Encourage Yoga and Sports in Schools.
Regular screening of school students to identify lifestyle diseases.
Awareness among students regarding lifestyle disease control.

53. Solution (a)


Indian Constitution did not give the status of national language to any one language.
Hindi was identified as the official language. But Hindi is the mother tongue of only
about 40 per cent of Indians. Therefore, there were many safeguards to protect
other languages. Besides Hindi, there are 21 other languages recognized as Scheduled
Languages by the Constitution (8th Schedule contains a list of 22 languages
recognized by the constitution).
According to the Constitution, the use of English for official purposes was to
stop in 1965. But the Central Government continues to use of English along with
Hindi for official purposes. Promotion of Hindi continues to be the official policy
of the Government of India. Promotion does not mean that the Central
Government can impose Hindi on States where people speak a different language.
Of the 22 languages that are included in the Eighth Schedule of the Indian
Constitution and are therefore called Scheduled Languages. Others are called nonScheduled Languages.
The States reorganization Commission under the chairmanship of Fazl Ali, which
created states on linguistic basis rejected the theory one language- one state
Reference: Page No. 20, 22 (Democratic Politics Part 2)
54. Solution (d)
After 1990- This period saw the rise of regional political parties in many States of
the country. This was also the beginning of the era of Coalition Governments at the
Centre. Since no single party got a clear majority in the Lok Sabha, the major
national parties had to enter into an alliance with many parties including several
http://insightsonindia.com

INSIGHTS

Page 22

Facebook Group: Indian Administrative Service (Raz Kr)

INSIGHTS MOCK EXAM-3 SOLUTIONS


regional parties to form a government at the Centre. This led to a new culture of
power sharing and respect for the autonomy of State Governments. This trend was
supported by a major judgement of the Supreme Court that made it difficult for
the Central Government to dismiss state governments in an arbitrary manner. Thus,
federal power sharing is more effective today than it was in the early years after the
Constitution came into force.
Reference: Page No. 20, 21 (Democratic Politics Part 2)
55. Solution (b)
A major step towards decentralization was taken in 1992. The Constitution was
amended to make the third-tier of democracy more powerful and effective. Now it
is constitutionally mandatory to hold regular elections to local government bodies.
Seats are reserved in the elected bodies and the executive heads of these
institutions for the Scheduled Castes, Scheduled Tribes and Other Backward
Classes.
At least one-third of all positions are reserved for women. No reservation for
minorities.
An independent institution called the State Election Commission (SEC) has been
created in each State to conduct panchayat and municipal elections and though
SEC is appointed by the Governor of the state, he can only be removed by the
President of India.
The panchayat works under the overall supervision of the gram sabha.
Reference: Page No. 24, 25 (Democratic Politics Part 2)

56. Solution: c)
The salient features of the Scheme of Tourist Visa on Arrival (TVOA) are:
The TVOA is allowed for a maximum validity of 30 days with single entry
facility.

http://insightsonindia.com

INSIGHTS

Page 23

Facebook Group: Indian Administrative Service (Raz Kr)

INSIGHTS MOCK EXAM-3 SOLUTIONS


The TVOA is available to foreign nationals arriving from Japan, Singapore,
Finland, Luxembourg, New Zealand, Cambodia, Laos, Vietnam, Philippines,
Myanmar and Indonesia at Delhi, Mumbai, Chennai and Kolkata
Airports. With effect from August 15, 2013 it has also been introduced at the
Kochi, Thiruvananthapuram, Hyderabad and Bangalore airports.
The fee for the TVOA is US $ 60/- or equivalent amount in Indian rupees
per passenger (including children).
The TVOA is allowed for a maximum of two times in a calendar year to a
foreigner with a minimum gap of two months between each visit. TVOA is
non-extendable and non-convertible.
Tourists from the above mentioned countries may also avail of TVOA for
up to 30 days for medical treatment, for casual business or to visit
friends/relatives, etc.
The TVOA facility is not applicable to the holders of Diplomatic/Official
Passports. Further, TVOA shall not be granted to the foreigners who have
permanent residence or occupation in India. Such persons can visit India on
normal visa, as applicable.

57. Solution: b)
Distinguishing Features of NPS -Swavalamban:
Voluntary - Open to eligible citizens of India, in the age group of 1860 years.
Subscriber is free to choose the amount he/she wants to invest every year.
Simple Eligible individuals in the unorganized sector can open an account
through their Aggregator and get an Individual subscriber (NPS - Swavalamban)
Account.
Safe - Regulated by Pension Fund Regulatory Development Authorities
(PFRDA), with transparent investment norms, regular monitoring and
performance review of fund managers by NPS Trust.
Economical Ultra-low cost structure with no minimum amount required per
annum or per contribution
"NPS -Swavalamban Model" is designed to ensure ultra-low administrative and
transactional costs, so as to make small investments viable.
Swavalamban Yojana is a scheme announced by the Government of India
under which Government will contribute Rs. 1000 per year to each NPS
Swavalamban account opened in year 2010-11,2011-12 ,2012-13 for five years as
under .
http://insightsonindia.com

INSIGHTS

Page 24

Facebook Group: Indian Administrative Service (Raz Kr)

INSIGHTS MOCK EXAM-3 SOLUTIONS


Account opened in 2010-2011 will get the benefit till 2014-2015
Account opened in 2011-2012 will get the benefit till 2015-2016
Account opened in 2012-2013 will get the benefit till 2016-2017
NPS Swavalamban account opened in the period 2013-2014 to 2016-2017 will
get the Swavalamban benefit up to 2016-17.
NPS - Swavalamban is a pension product to ensure a monthly income after the
retirement age has been attainted.
NPS - Swavalamban invests a portion of the contributions in the equity (stock)
market and hence there are possibilities of returns much higher than what banks
and similar financial institutions are able to offer. A portion of the corpus is
invested in equity markets which enables the corpus to grow quickly. However,
unlike other equity based investment schemes where risk of losing the money is
high, in NPS - Swavalamban; the risk is reduced considerably as up to 55% of
money is invested in Government securities and up to 40% in corporate bonds.

58. Solution: d)
The Minister of Petroleum & Natural Gas, Dr M. Veerappa Moily, has
approved the launch of Portability of LPG connections from today onwards across
Oil Marketing Companies/distributors covering 480 districts in the country. These
districts cover all possible LPG markets which have multiple LPG distributors of
various ratings.
With this, an LPG consumer in these markets can now switch to the
distributor of his/her choice within a cluster of LPG distributors in the vicinity
under the LPG Connection Portability Scheme. This measure will bring great relief
to those LPG consumers who are unhappy with the services of their current
distributor or want to move to an LPG distributor closer to their home.
As a pilot project, the Oil Marketing Companies (OMCs) together launched
LPG connection portability scheme in 24 districts covering 13 States in October
2013. Today, this scheme is being launched on a full-fledged all-India basis by
expanding its coverage to over 480 districts and a population of over 8.2 crore LPG
consumers across the country.

http://insightsonindia.com

INSIGHTS

Page 25

Facebook Group: Indian Administrative Service (Raz Kr)

INSIGHTS MOCK EXAM-3 SOLUTIONS


59. Solution: d)
Water scarcity is the lack of sufficient available water resources to meet the
demands of water usage within a region. It already affects every continent and
around 2.8 billion people around the world at least one month out of every year.
More than 1.2 billion people lack access to clean drinking water.
Water scarcity involves water stress, water shortage or deficits, and water crisis.
While the concept of water stress is relatively new, it is the difficulty of obtaining
sources of fresh water for use during a period of time and may result in further
depletion and deterioration of available water resources. Water shortages may be
caused by climate change, such as altered weather patterns
including droughts or floods, increased pollution, and increased human demand and
overuse of water. A water crisis is a situation where the available potable,
unpolluted water within a region is less than that region's demand. Water scarcity is
being driven by two converging phenomena: growing freshwater use and depletion
of usable freshwater resources.
The per capita availability of water in the country is 1545 cubic meters as per
the 2011 census. The average annual per capita availability of water in the country,
taking into consideration the population of the country as per the 2001 census,was
1816 cubic meters which reduced to 1545 cubic meters as per the 2011
census,denoting that the per capita water availability in the country is reducing
progressively due to increase in population.Also there are reports that demand for
water in India would rise dramatically to about 833 cubic kilometers in 2025 and
899 cubic kilometers in 2050 due to increase in population, rapid urbanization and
a growing GDP with significant lifestyle changes

60. Solution: d)
Kharif crops are grown with the onset of monsoon in different parts of the country
and these are harvested in September-October. Important crops grown during this
season are paddy, maize, jowar, bajra, tur (arhar), moong, urad, cotton, jute,
groundnut and soyabean. Some of the most important rice-growing regions are
Assam, West Bengal, coastal regions of Orissa, Andhra Pradesh, Tamil Nadu, Kerala
and Maharashtra, particularly the (Konkan coast) along with Uttar Pradesh and
Bihar.

http://insightsonindia.com

INSIGHTS

Page 26

Facebook Group: Indian Administrative Service (Raz Kr)

INSIGHTS MOCK EXAM-3 SOLUTIONS


Recently, paddy has also become an important crop of Punjab and Haryana. In
states like Assam, West Bengal and Orissa, three crops of paddy are grown in a year.
These are Aus, Aman and Boro.
In between the rabi and the kharif seasons, there is a short season during the
summer months known as the Zaid season. Some of the crops produced during
zaid are watermelon, muskmelon, cucumber, vegetables and fodder crops.

61. Solution: a)
Pulses: India is the largest producer as well as the consumer of pulses in the world.
These are the major source of protein in a vegetarian diet. Major pulses that are
grown in India are tur (arhar), urad, moong, masur, peas and gram. Pulses need less
moisture and survive even in dry conditions. Being leguminous crops, all these crops
except arhar help in restoring soil fertility by fixing nitrogen from the air.
Therefore, these are mostly grown in rotation with other crops. Major pulse
producing states in India are Madhya Pradesh, Uttar Pradesh, Rajasthan,
Maharashtra and Karnataka.

62. Solution.(b)
The printing press first came to Goa with Portuguese missionaries in the midsixteenth century.
Reference: Page 168 India and the Contemporary World II 10th NCERT
63. Solution: a)
Page no. 244, NCERT 10th Science

64. Solution: b)
http://insightsonindia.com

INSIGHTS

Page 27

Facebook Group: Indian Administrative Service (Raz Kr)

INSIGHTS MOCK EXAM-3 SOLUTIONS


Cotton: India is believed to be the original home of the cotton plant. Cotton is one
of the main raw materials for cotton textile industry. India is the third-largest
producer of cotton in the world. Cotton grows well in drier parts of the black
cotton soil of the Deccan plateau. It requires high temperature, light rainfall or
irrigation, 210 frost-free days and bright sunshine for its growth. It is a kharif crop
and requires 6 to 8 months to mature.

65. Solution: c)
The newly launched Rashtriya Bal Swaasthya Karyakram of the Ministry of
Health & Family Welfare assures a package of health services for children up to 18
years of age. The initiative, which is part of the National Rural Health Mission, was
launched on February 6, in Palghar, a tribal dominated block of Thane district in
Maharashtra. The programme will be extended to cover all districts of the country
in a phased manner.
Early Detection; Early Intervention
Rashtriya Bal Swaasthya Karyakram, also known as Child Health Screening
and Early Intervention Services aims at early detection and management
of 4Ds prevalent in children. These are Defects at birth, Diseases in children,
Deficiency conditions and Developmental delays including Disabilities. The health
screening of children is a known intervention under School Health Programme. It is
now being expanded to cover all children from birth to 18 years of age. The services
aim to cover all children of 0-6 years of age group in rural areas and urban slums, in
addition to children enrolled in classes 1st to 12th in Government and Government
aided schools. A set of 30 common ailments / health conditions have been
identified for screening and early intervention.

66. Solution: d
The Government of India is taking fresh measures to bolster the Jan
Aushadhi Campaign, as a public welfare programme, to supply quality medicines at
affordable prices to the common man through dedicated outlets. It is also a part of
direct market intervention strategy by making generic medicines easily available and
accessible in the market. A key initiative under the campaign is opening of Jan
Aushadhi Stores', where quality generic medicines, which are equivalent to the
http://insightsonindia.com

INSIGHTS

Page 28

Facebook Group: Indian Administrative Service (Raz Kr)

INSIGHTS MOCK EXAM-3 SOLUTIONS


expensive branded drugs, in terms of their potency and efficacy, are sold at cheaper
prices.
To know what these generic drugs are, we have the definition by the World Health
Organization (WHO). According to it, a generic drug is a pharmaceutical product
that is manufactured without a license from the innovator company and marketed
after the expiry date of the patent or other exclusive rights. These Generic drugs
are subject to the same regulations over manufacturing, packaging, testing and
quality standards, as their patented/branded equivalent. They have the same form,
strength, dosages, intended use, safety, and route of administration, performance
characteristics and quality on every count. Prices of the branded/patented drugs are
generally quite high, due to the temporary monopoly over the product, bestowed
by the patent. Once available in the market, without a patent right, they are
normally then sold by various pharmacy companies under different brand or
company given names. These are expected to be much cheaper due to the market
competition as they are sold by various manufacturers. These are normally referred
by its chemical or its salt name by the chemists/pharmacists.

67. Solution: c)
On 9th January 2013 the Prime Minister launched the National Electric
Mobility Mission Plan (NEMMP) 2020 which is a roadmap for progressive
induction and expansion of efficient and environmentally friendly electric and
hybrid vehicles in the country.
The National Electric Mobility Mission Plan 2020 is a culmination of
extensive and detailed exercise by the government and the industry covering a span
of one and a half years; also involving an international knowledge, that carried out a
primary data based survey among seven thousand respondents, to understand the
needs and aspirations of the Indian consumer with respect to clean, energy-efficient
and affordable transportation, and the state of preparedness of the Indian
automobile industry in this regard. The study took into consideration the current
international best practices, particularly in Europe, USA, Japan and China, and also
recent developments within the country, particularly steps being taken by the
government to encourage the use of electric and hybrid vehicles, for example the
subsidy scheme for electric vehicles ran by the Ministry of New and Renewable
Energy Resources.
http://insightsonindia.com

INSIGHTS

Page 29

Facebook Group: Indian Administrative Service (Raz Kr)

INSIGHTS MOCK EXAM-3 SOLUTIONS


The National Electric Mobility Mission Plan 2020 emanates from the
National Mission on Electric Mobility (NMEM) which was approved by the
Cabinet as the core vision document that has National energy security, mitigation
of the adverse impact of vehicles on the environment and growth of domestic
manufacturing capabilities as its principal end objectives.
The NEMMP 2020 is a well researched, comprehensive action plan which indicates
that high latent demand for environmentally friendly electric vehicle technologies
exists in the country. As per these projections, co-ordinated efforts are required by
the government and industry which would eventually result, by the year 2020, 6-7
million units of new vehicle sales of the full range of electric vehicles, along with
resultant liquid fuel savings of 2.2 2.5 million tones. This will also result in
substantial lowering of vehicular emissions and decrease in carbon di-oxide
emissions by 1.3% to 1.5% in 2020.

68. Solution: d)
Detailed guidelines were issued to the States/UTs for identification of
the Antyodaya families under the AAY and additional Antyodaya families under
the expanded AAY. In order to identify the households criteria adopted:- Landless
agriculture labourers, marginal farmers, rural artisans/craftsmen, such as potters,
tanners, weavers, blacksmiths, carpenters, slum dwellers and persons earning their
livelihood on daily basis in the informal sector like porters, coolies, rickshaw
pullers, hand cart pullers, fruit and flower sellers, snake charmers, rag pickers,
cobblers, destitute and other similar categories in both rural and urban areas.
Households headed by widows or terminally ill persons/disabled
persons/persons aged 60 years or more with no assured means of subsistence or
societal support.
Widows or terminally ill persons or disabled persons or persons aged 60
years or more or single women or single men with no family or societal support or
assured means of subsistence.
All primitive tribal households.

http://insightsonindia.com

INSIGHTS

Page 30

Facebook Group: Indian Administrative Service (Raz Kr)

INSIGHTS MOCK EXAM-3 SOLUTIONS


The above guidelines have further been amended vide letter dated 3rd June, 2009 to
include all eligible BPL, families of HIV positive persons in the AAY list of on
priority.

69. Solution: d)
Government believes that the Direct Cash Transfer or Direct Benefits
Transfer is likely to be a game-changer in more than one way.
The Centre releases as much as Rs 2, 00,000 crore as subsidies under various
schemes for the targeted sections across the country. Therefore it is within its right
to devise methods to reach beneficiaries the way it wants.
Firstly, the Direct Benefits Transfer (DBT) scheme is aimed at cutting the
bloated subsidy bill of Rs.1, 64,000 crore. Indias budget deficit was 5.8 per cent of
gross domestic product in the financial year ending 2012 March.
Secondly, unlike other welfare scheme launched so far by the Centre, DBT
helps in timely and quick transfer to intended beneficiaries.
Thirdly, the transfer of direct cash into account of targeted beneficiary is a
winning proposition for the recipients as it aims to eliminate middlemen in various
government sponsored welfare schemes and subsidized food, fuel and fertiliser
schemes. Take for instance, it's estimated that public coffers can be richer by
several crore yearly just by switching to cash handouts for LPG and kerosene, a
proposed move that would also curb diversion of subsidised cylinders for
commercial use and diesel adulteration with inexpensive kerosene. Bringing all
subsidies under DBT's ambit can be the major fiscal game-changer the economy
needs very much.
Fourthly, the Direct Benefits Transfer scheme is likely to be simple and error
free. On the basis of Aadhar cards money is deposited in beneficiaries accounts.
Fifthly indirect transfers are more prone to leakages than direct cash transfers.
So, that is why the Central Government has put in a mechanism of direct cash
transfer. According to Planning Commission the Public Distribution System has
become so inefficient that 58 per cent of the subsidized grains do not reach targeted
beneficiaries while one-third of it siphoned from the system.
Sixthly, the Aadhar based DBT helps eliminate duplicate cards and cards for
non-existent persons or ghost beneficiaries often found in schemes such as the PDS
and MNREGS.
http://insightsonindia.com

INSIGHTS

Page 31

Facebook Group: Indian Administrative Service (Raz Kr)

INSIGHTS MOCK EXAM-3 SOLUTIONS


Seventhly, with the actual transfer of cash taking place with the help of
micro automated teller machines (ATMs) it would infuse financial inclusion on a
greater scale in rural India. Quoting a World Bank Study the Reserve Bank of India
last year in its annual report has said, in India only 35 per cent have formal accounts
versus an average of 41 per cent in developing economies. With the implementation
of DBT, it could fuel financial inclusion.
Eighthly, aided by Aadhar technology Direct Benefits Transfer will not be a mere
welfare scheme but also the world's largest experiment in administrative reform. It
will revolutionise the delivery of welfare measures in worlds populous democracy.

70. Solution: c)
It was to involve the youth in constructive activities, enable them to get
employment that a major initiative Youth Employment and Guidance Node
(YEGN) programme was launched a few years back in border and remote areas of
Jammu and Kashmir .
The YEGN guidance and training to young men and women in various
skills to equip them to either get a job or start their own enterprise , prepare
them for recruitment in the army and other security agencies including paramilitary
forces and help them acquire some expertise for self employment .
Thanks to the YEGN under the Sadbhawna programme today many a
youth have been recruited in army, paramilitary forces, railways, banks and many
trained to earn a livelihood on their own.
The YEGN centres have computer training facilities, facilities for training in
different skills and competent personnel and expertees to guide the youth in getting
gainful employment or seeking higher education or sitting for exams for
recruitment in bank, railways and other fields.
The youth that include a large number of students have also access to
magazines and journals on general knowledge and competitive exams in various
sectors besides Employment News magazine.

71. Solution: d)
http://insightsonindia.com

INSIGHTS

Page 32

Facebook Group: Indian Administrative Service (Raz Kr)

INSIGHTS MOCK EXAM-3 SOLUTIONS


The denudation of the soil cover and subsequent washing down is described as soil
erosion. The processes of soil formation and erosion, go on simultaneously and
generally there is a balance between the two. Sometimes, this balance is disturbed
due to human activities like deforestation, over-grazing, construction and mining
etc., while natural forces like wind, glacier and water lead to soil erosion.
The running water cuts through the clayey soils and makes deep channels as gullies.
The land becomes unfit for cultivation and is known as bad land. In the Chambal
basin such lands are called ravines. Sometimes water flows as a sheet over large
areas down a slope. In such cases the top soil is washed away. This is known as
sheet erosion. Wind blows loose soil off flat or sloping land known as wind erosion.
Soil erosion is also caused due to defective methods of farming. Ploughing in a
wrong way i.e. up and down the slope form channels for the quick flow of water
leading to soil erosion.

72. Solution: a)
The Leatherback turtle is a critically endangered species.
The Leatherback Turtle (Dermochelys coriacea) is the largest of the living sea
turtles, weighing as much as 900 kg. Adult leatherback turtles are excellent
swimmers. They swim an average of 45-65 km a day, travel upto 15,000 km per
year and can dive as deep as 1200 m. Jellyfish is their primary food.
The population spikes of leatherbacks coincide with abundance of jellyfish,
making them important top-predators in marine environments.
Habitat: Tropical and subtropical oceans.
Distribution: Found in tropical and temperate waters of the Atlantic, Pacific, and
Indian Oceans.
Threats: High sea fishing operations, harvesting of eggs, destruction of nests by wild
predators and domesticated species such as cats, dogs and pigs. Artificial lighting
disorients hatchlings and adults and causes them to migrate inland rather
http://www.thehindubusinessline.com/news/15-bird-species-in-india-in-criticallyendangered-list/article5393581.ece
http://insightsonindia.com

INSIGHTS

Page 33

Facebook Group: Indian Administrative Service (Raz Kr)

INSIGHTS MOCK EXAM-3 SOLUTIONS


http://www.moef.nic.in/downloads/publicinformation/critically_endangered_booklet.pdf

73. Solution: b)
Critically endangered is the highest risk category assigned by the IUCN
(International Union for Conservation of Nature) red lIst to wild species. There are
five quantitative criteria to determine whether a taxon is threatened. A taxon is
critically endangered when the best availabile evidence indicates that it meets any
of the following criteria:
I. Populations have declined or will decrease, by greater than 80% over the last 10
years or three generations.
II. Have a restricted geographical range.
III. Small population size of less than 250 individuals and continuing decline at 25%
in 3 years or one generation.
IV. Very small or restricted population of fewer than 50 mature individuals.
V. High probability of extinction in the wild.
http://www.moef.nic.in/downloads/publicinformation/critically_endangered_booklet.pdf
Based on the International Union for Conservation of Nature and Natural
Resources (IUCN), we can classify as follows
Normal Species: Species whose population levels are considered to be normal for
their survival, such as cattle, sal, pine, rodents, etc.
Endangered Species: These are species which are in danger of extinction. The
survival of such species is difficult if the negative factors that have led to a decline
in their population continue to operate. The examples of such species are black

http://insightsonindia.com

INSIGHTS

Page 34

Facebook Group: Indian Administrative Service (Raz Kr)

INSIGHTS MOCK EXAM-3 SOLUTIONS


buck, crocodile, Indian wild ass, Indian rhino, lion tailed macaque, sangai (brow
anter deer in Manipur), etc.
Vulnerable Species: These are species whose population has declined to levels from
where it is likely to move into the endangered category in the near future if the
negative factors continue to operate. The examples of such species are blue sheep,
Asiatic elephant, Gangetic dolphin, etc.
Rare Species: Species with small population may move into the endangered or
vulnerable category if the negative factors affecting them continue to operate. The
examples of such species are the Himalayan brown bear, wild Asiatic buffalo,
desert fox and hornbill, etc.
Endemic Species: These are species which are only found in some particular areas
usually isolated by natural or geographical barriers. Examples of such species are the
Andaman teal, Nicobar pigeon, Andaman wild pig, mithun in Arunchal Pradesh.
Extinct Species: These are species which are not found after searches of known or
likely areas where they may occur. A species may be extinct from a local area,
region, country, continent or the entire earth. Examples of such species are the
Asiatic cheetah, pink head duck.

74. Solution (a)


These social differences are mostly based on accident of birth. Normally we dont
choose to belong to our community. We belong to it simply because we were born
into it. We all experience social differences based on accident of birth in our
everyday lives. People around us are male or female, they are tall and short, have
different kinds of complexions, or have different physical abilities or disabilities.
But all kinds of social differences are not based on accident of birth. Some of the
differences are based on our choices. For example, some people are atheists. They
dont believe in God or any religion. Some people choose to follow a religion other
than the one in which they were born. Most of us choose what to study, which
occupation to take up and which games or cultural activities to take part in. All
these lead to formation of social groups that are based on our choices.
Every social difference does not lead to social division. Social differences divide
similar people from one another, but they also unite very different people.
http://insightsonindia.com

INSIGHTS

Page 35

Facebook Group: Indian Administrative Service (Raz Kr)

INSIGHTS MOCK EXAM-3 SOLUTIONS


Democracy does not always leads to disintegration of society on the basis of social
divisions. Actually democracy is one of the best wayto accommodate social
diversity (as in India).
Reference: Page No. 32, 33 and 34 (Democratic Politics Part 2)

75. Solution (b)


There are various kinds of social differences that can take the form of social
divisions and inequalities. These are social differences based on gender, religion,
caste, wealth.
Womens reservation Bill has not yet been passed by the parliament. Hence it
cannot be considered as a step to reduce social difference.
Our Constitution did not give the status of nationallanguage to any one language.
Hindi was identified as the official language only and not National language. Hindi
as a national language will only create more social difference in the society.
Constitution allows the state to intervene in thematters of religion in order to
ensure equality within religious communities. For example, it bans untouchability.
Enactment of Mahatma Gandhi National Rural Employment Guarantee Act
(MGNREGA) would not only reduce the economic inequality but also social
inequality.
Reference: Based on the concepts of Chapter 4(Democratic Politics Part 2)
76. Solution (d)
Communalism involves thinking along the following lines. The followers of a
particular religion must belong to one community. Their fundamental interests are
the same. Any difference that they may have is irrelevant or trivial for community
life. It also follows that people who follow different religions cannot belong to the
same social community.
If the followers of different religion have some commonalities these are superficial
and immaterial. Their interests are bound to be different and involve a conflict. In
its extreme form communalism leads to the belief that people belonging to
http://insightsonindia.com

INSIGHTS

Page 36

Facebook Group: Indian Administrative Service (Raz Kr)

INSIGHTS MOCK EXAM-3 SOLUTIONS


different religions cannot live as equal citizens within one nation. Either, one of
them has to dominate the rest or they have to form different nations.
Reference: Page No. 47 (Democratic Politics Part 2)
77. Solution (c)
The Constitution of India ensures equality of citizens within religious communities
(as provided under Fundamental Rights Articles 25 to 28).
There is one respect in which democratic government is certainly better than its
alternatives: democratic government is legitimate government. It may be slow, less
efficient, not always very responsive or clean. But a democratic government is
peoples own government. That is why there is an overwhelming support for the
idea of democracy all over the world.
The preamble is not an integral part of the Indian constitution was first decided by
the Supreme Court of India in Beru Bari case therefore it is not enforceable in a
court of law. However, Supreme Court of India has, in the Kesavananda case,
recognised that the preamble may be used to interpret ambiguous areas of the
constitution where differing interpretations present themselves. In the 1995 case of
Union Government Vs LIC of India also the Supreme Court has once again held
that Preamble is the integral part of the Constitution.
Democracy stands much superior to any other form of government in promoting
dignity and freedom of the individual. Every individual wants toreceive respect
from fellow beings. Often conflicts arise among individuals because some feel that
they are not treated with due respect. The passion for respect and freedom are the
basis of democracy.
Reference: Page No. 92, 97 (Democratic Politics Part 2)
78. Solution (c)
Pressure groups are organisations that attempt to influence government policies.
But unlike political parties, pressure groups do not aim to directly control or share
political power. These organisations are formed when people with common
occupation, interest, aspirations or opinions come together in order to achieve a
common objective. Their decision-making is more informal and flexible.
http://insightsonindia.com

INSIGHTS

Page 37

Facebook Group: Indian Administrative Service (Raz Kr)

INSIGHTS MOCK EXAM-3 SOLUTIONS

Usually interest groups seek to promote the interests of a particular section or


group of society. Trade unions, business associations and professional (lawyers,
doctors, teachers, etc.) bodies are some examples of this type. They are sectional
because they represent a section of society: workers, employees, business persons,
industrialists, followers of a religion, caste group, etc. Their principal concern is the
betterment and well being of their members, not society in general.
On balance, however, pressure groups and movements have deepened democracy.
Putting pressure on the rulers is not an unhealthy activity in a democracy as long as
everyone gets this opportunity.
While interest groups and movements do not directly engage in party politics, they
seek to exert influence on political parties. Most of the movement groups take a
political stance without being a party. They have political ideology and political
position on major issues. The relationship between political parties and pressure
groups can take different forms, some direct and others very indirect.
Reference: Page No. 63, 64,67, 68 (Democratic Politics Part 2)

79. Solution: b
Chap-15, Pages 262, NCERT Class 10 Science
80. Solution: d
Chap-16, Page 266, 267 (Besides, acids have low pH value. Potable water has
moderate pH value between 6.5-7.5. Also, no natural water source can be metal
free. Metals like Iron, magnesium etc. are actually useful to the human body.)

81. Solution: b
Chap-16, Pages 270, NCERT Class 10 Science

82. Solution: a
http://insightsonindia.com

INSIGHTS

Page 38

Facebook Group: Indian Administrative Service (Raz Kr)

INSIGHTS MOCK EXAM-3 SOLUTIONS


Chap-16, Pages 273, NCERT Class 10 Science

83. Solution: d)
http://eands.dacnet.nic.in/PDF_LUS/Concepts_&_Definitions.pdf

84. Solution: d)
Page 8 and 10, Class X NCERT Geography.
85. Solution (d)
The multi-party system often appears very messy and leads to political instability.
At the same time, this system allows a variety of interests and opinions to enjoy
political representation and thereby deepens democracy. Multi-party system creates
a natural system of checks and balance, which helps to check the emergence of
dictatorial tendencies (which may happen in single or a two-party system). Since
there is no real control or limited control of number of parties in India, no single
party is able to get a majority and leads to hung parliament (where no party wins
the majority seats and thus cannot form a government).
Reference:Chapter 6 (Democratic Politics Part 2)
86. Solution (a)
The Answer is either the First statement and Fourth Statement together (or)
Second statement alone, but since the answer option does not have 2 Only, option
(a) is correct.
Conditions for recognition as a National party:
A political party shall be treated as a recognised National party, if, and only if,
either (A)(i) the candidates set up by it, in any four or more States, at the last
general election to the House of the People, or to the Legislative Assembly of the
State concerned, have secured not less than six percent of the total valid votes
polled in their respective States at that general election; and (ii) in addition, it has
http://insightsonindia.com

INSIGHTS

Page 39

Facebook Group: Indian Administrative Service (Raz Kr)

INSIGHTS MOCK EXAM-3 SOLUTIONS


returned at least four members to the House of the People at the aforesaid last
general election from any State or States; or (B) (i) its candidates have been elected
to the House of the People, at the last general election to that House, from at least
two percent of the total number of parliamentary constituencies in India, any
fraction exceeding one half being counted as one; and (ii) the said candidates have
been elected to that House from not less than three State
Reference: Election commission website
87. Solution (c)
The Constitution was amended to prevent elected MLAs and MPs from changing
parties. This was done because many elected representatives were indulging in
Defection in order to become ministers or for cash rewards. Now the law says that
if any MLA or MP changes parties, he or she will lose the seat in the legislature.
This new law has helped bring defection down. At the same time this has made any
dissent even more difficult. MPs and MLAs have to accept whatever the party
leaders decide.
The Supreme Court passed an order to reduce the influence of money and
criminals. Now, it is mandatory for every candidate who contests elections to file an
Affidavit giving details of his property and criminal cases pending against him.
The new system has made a lot of information available to the public. But there is
no system of check if the information given by the candidates is true.
The Election Commission passed an order making it necessary for political parties
to hold their organizational elections and file their income tax returns.
The parties have started doing so but sometimes it is mere formality.
When Political parties are brought under the ambit of Right to Information Act,
there is transparency and is a measure of good governance.
Reference:Page No. 85, 86 (Democratic Politics Part 2)

88. Solution: b)
Sustainable development ties together concern for the carrying capacity of natural
systemswith the social and economic challenges faced by humanity. As early as the
1970s, 'sustainability' was employed to describe an economy "in equilibrium with
basic ecological support systems."[1] Scientists in many fields have highlighted The
http://insightsonindia.com

INSIGHTS

Page 40

Facebook Group: Indian Administrative Service (Raz Kr)

INSIGHTS MOCK EXAM-3 SOLUTIONS


Limits to Growth,[2] and economists have presented alternatives, for example a
'steady state economy',[3] to address concerns over the impacts of expanding human
development on the planet.
The term 'sustainable development' rose to significance after it was used by
the Brundtland Commission in its 1987 report Our Common Future. In the report,
the commission coined what has become the most often-quoted definition of
sustainable development: "development that meets the needs of the present
without compromising the ability of future generations to meet their own needs."
The carrying capacity of a biological species in an environment is the maximum
population size of the species that the environment can sustain indefinitely, given
the food, habitat, water and other necessities available in the environment.
In population biology, carrying capacity is defined as the environment's maximal
load,[1] which is different from the concept of population equilibrium.
The United Nations 2005 World Summit Outcome Document refers to the
"interdependent and mutually reinforcing pillars" of sustainable development as
economic development, social development, and environmental
protection.[10] Based on this 'triple bottom line', numerous sustainability standards
and certification systems have been established in recent years, in particular in the
food industry.

89. Solution: b
Chap-14, Pages 246, NCERT Class 10 Science

90. Solution: a
Chap-14, Pages 250, NCERT Class 10 Science

91. Solution: d

http://insightsonindia.com

INSIGHTS

Page 41

Facebook Group: Indian Administrative Service (Raz Kr)

INSIGHTS MOCK EXAM-3 SOLUTIONS


Chap-15, Pages 259, NCERT Class 10 Science

92. Solution: a
Chap-15, Pages 260-61, NCERT Class 10 Science

93. Solution (c)


A political party shall be treated as a recognised political party in a State, if and
only if either the conditions satisfy:
a) A political party should secure at least six percent of the total valid votes
polled during general election to a State Legislative Assembly and should, in
addition, win at least two seats in that Assembly, or the party should win at
least three percent of the total number of seats or three seats in the
Legislative Assembly, whichever is more;
(Or)
b) A political party should secure at least six percent of the total valid votes
polled in a State during a general election to Lok Sabha and win at least one
seat in the Lok Sabha from that State, or the party should win at least one
seat in the Lok Sabha for every 25 seats or any fraction thereof allotted to
that State(or, at least one member to the Legislative Assembly of that State
for every 30 members of that Assembly or any fraction of that number)
Reference : Election Commission website

94. Solution (d)


Every party in the country has to register with the Election Commission. While the
Commission treats all parties equally, it offers some special facilities to large and
established parties. These parties are given a unique symbol only the official
candidates of that party can use that election symbol. Parties that get this privilege
and some other special facilities are recognised by the Election Commission for
this purpose. That is why these parties are called, recognised political parties. The
http://insightsonindia.com

INSIGHTS

Page 42

Facebook Group: Indian Administrative Service (Raz Kr)

INSIGHTS MOCK EXAM-3 SOLUTIONS


Election Commission has laid down detailed criteria of the proportion of votes and
seats that a party must get in order to be a recognised party.
All political parties must register themselves with the EC. However, not all parties
that are registered with the EC are 'recognised' by it preference in the matter of
allotment of free symbols. Further, registered political parties, in course of time, can
get recognition as `State Party or National Party subject to the fulfillment of the
conditions prescribed by the Commission in the Election Symbols (Reservation and
Allotment) Order, 1968, as amended from time to time.
Recognised `State and `National parties need only one proposer for filing the
nomination and are also entitled for two sets of electoral rolls free of cost and
broadcast/telecast facilities over Akashvani/Doordarshan during general elections.
Reference: Page No. 79, Election Commission website (Democratic Politics Part 2)

95. Solution: d,
Page no. 244, Chapter 14, NCERT 10th Science
96. Solution: a)
Page no 244-245, Chapter 14, NCERT 10th Science
97. Solution. (c)
The members of the Constituent Assembly were notelected on the basis of
universal franchise. In thewinter of 1945-46 provincial elections were held inIndia.
The Provincial Legislatures then chose the representatives to the Constituent
Assembly. The Constituent Assembly that came into being was dominated by one
party: the Congress.The Congress however was not a party with one voice. Its
members differed in their opinion on critical issues. Some members were inspired
by socialism while others were defenders of landlordism. Some were close to
communal parties while others were assertively secular.
http://insightsonindia.com

INSIGHTS

Page 43

Facebook Group: Indian Administrative Service (Raz Kr)

INSIGHTS MOCK EXAM-3 SOLUTIONS


Reference: Page 407-408 HISTORY-III 11th NCERT

98. Solution: c
99. Solution. (a)
India had a very rich and old tradition of handwritten manuscripts
in Sanskrit, Arabic, Persian, as well as in various vernacular languages. Manuscripts
continued to be produced till well after the introduction of print, down to the late
nineteenth century. Manuscripts, however, were highly expensive and fragile. They
had to be handled carefully, and they could not be read easily as the script was
written in different styles. So manuscripts were not widely used in everyday life.
100. Solution. (d)
Painters like Raja Ravi Varma produced images for mass circulation. Poor wood
engravers who made woodblocks set up shop near the letterpresses, and were
employed by print shops. Cheap prints and calendars, easily available in the bazaar,
could be bought even by the poor to decorate the walls of their homes or places of
work. These prints began shaping popular ideas about modernity and tradition,
religion and politics, and society and culture.
Reference: Page 171 India and the Contemporary World II 10th NCERT

http://insightsonindia.com

INSIGHTS

Page 44

Facebook Group: Indian Administrative Service (Raz Kr)

INSIGHTS ON INDIA MOCK PRELIMINARY EXAM - 2014


INSIGHTS ON INDIA MOCK TEST - 4
GENERAL STUDIES

PAPER-I
Time Allowed: Two Hours

Maximum Marks: 200

INSTRUCTIONS
1. IMMEDITELY AFTER THE COMMENCEMENT OF THE EXAMINATION, YOU SHOULD
CHECK THAT THIS TEST BOOKLET DOES NOT HAVE ANY UNPRINTED OR TORN OR
MISSING PAGES OR ITEMS, ETC. IF SO, GET IT REPLACED BY A COMPLETE TEST BOOKLET.
2. You have to enter your Roll Number on the Test I
Booklet in the Box provided alongside. DO NOT
write anything else on the Test Booklet.
4. This Test Booklet contains 100 items (questions). Each item is printed only in English. Each item
comprises four responses (answers). You will select the response which you want to mark on the Answer
Sheet. In case you feel that there is more than one correct response, mark the response which you consider
the best. In any case, choose ONLY ONE response for each item.
5. You have to mark all your responses ONLY on the separate Answer Sheet provided. See directions in the
Answer Sheet.
6. All items carry equal marks.
7. Before you proceed to mark in the Answer Sheet the response to various items in the Test Booklet, you
have to fill in some particulars in the Answer Sheet as per instructions sent to you with your Admission
Certificate.
8. After you have completed filling in all your responses on the Answer Sheet and the examination has
concluded, you should hand over to the Invigilator only the Answer Sheet. You are permitted to take away
with you the Test Booklet.
9. Sheets for rough work are appended in the Test Booklet at the end.
10. Penalty for wrong answers :
THERE WILL BE PENALTY FOR WRONG ANSWERS MARKED BY A CANDIDATE IN THE
OBJECTIVE TYPE QUESTION PAPERS.
(i)

There are four alternatives for the answer to every question. For each question for which a
wrong answer has been given by the candidate, one-third of the marks assigned to that question
will be deducted as penalty.

(ii) If a candidate gives more than one answer, it will be treated as a wrong answer even if one of the
given answers happens to be correct and there will be same penalty as above to that question.
(iii)

If a question is left blank, i.e., no answer is given by the candidate, there will be no penalty for
that question.

http://insightsonindia.com

INSIGHTS ON INDIA MOCK TEST SERIES FOR CIVIL SERVICES PRELIMINARY EXAM 2014

http://insightsonindia.com

Page 1

Facebook Group: Indian Administrative Service (Raz Kr)

1. Consider the following statements


about Hydrofluorocarbons (HFCs)
1. HFCs cause ozone depletion and
are banned under Montreal
protocol
2. HFCs are super-greenhouse gases
with an extremely high global
warming potential.
3. HFCs also figure in the basket of
six greenhouse gases under
the Kyoto Protocol
Which of the statements given above
is/are incorrect?
a.
b.
c.
d.

1 Only
2 and 3 Only
3 Only
None

2. Consider the following statements


with reference to the latitudinal and
longitudinal extent of India
1. The latitudinal and longitudinal
extent of India are roughly about
30 degrees
2. The distance from North to
South is shorter than the distance
from East to West
3. This difference is based on the
fact that the distance between
two longitudes increases towards
the poles whereas the distance
between two latitudes remains
the same everywhere.
Which of the statements given above
is/are correct?
a.
b.
c.
d.

1 Only
1 and 3 Only
2 and 3 Only
None

http://insightsonindia.com

3. Consider the following statements:


1. The Indian Constitution does not
make ethnic identity a criterion for
citizenship.
2. The members of the Indian
Constituent Assembly that was
formed pre-Independence were
indirectly elected.
3. The Constituent Assembly was
composed roughly along the lines
suggested by the Cripps Mission
plan.
4. The introduction of universal
suffrage was the only provision that
was passed without virtually any
debate in the Constituent Assembly.
Which of the statements given above
is/are correct?
a.
b.
c.
d.

1 and 2 Only
2, 3 and 4 Only
1, 2 and 4 Only
1, 2, 3 and 4

4. Consider the following statements about


the Harappan Civilization:
1. The Harappan seals which contained
animal motifs and signs, most of
them, were made of fine quality
metals.
2. The evidences show that Rice was
the main food grain consumed by
the people.
Which of the above statement/s is/are
correct?
a.
b.
c.
d.

1 only
2 only
Both
None

Page 2

Facebook Group: Indian Administrative Service (Raz Kr)

5. The purpose of Environmental


Impact Assessment (EIA) is:
To identify and evaluate the
potential impacts (beneficial and
adverse)of development and
projects on the environmental
system.
2. It is a useful aid for decision
making based on understanding
of the environment implications
including social, cultural and
aesthetic concerns which could
be integrated with the analysis of
the project costs and benefits.

Which of the reasons given above is /are


correct?
a.
b.
c.
d.

1.

Which of the above statements


is/are incorrect?
a.
b.
c.
d.

1 Only
2 Only
Both
None

6. December and January are the


coldest months in the northern plain.
The mean daily temperature remains
below 21C over most parts of
northern India. Consider the
following reasons,
1. States like Punjab, Haryana
and Rajasthan being far away
from the moderating influence
of sea experience continental
climate.
2. The snowfall in the nearby
Himalayan ranges creates cold
wave situation
3. Around February, the cold
winds coming from the
Caspian
Sea
and
Turkmenistan bring cold wave
along with frost and fog over
the northwestern parts of
India.

http://insightsonindia.com

1 and 3 Only
2 and 3 Only
3 Only
All Three

7. India has adopted bi-cameral


legislature, wherein the Rajya Sabha
forms the upper House and Lok
Sabha forms the lower House.
Consider the following with regard
to Rajya Sabha:
1. The number of members to be
elected from each State has been
fixed by the second schedule of the
Constitution.
2. It is an indirectly elected body.
3. Members of the Rajya Sabha are
elected for a term of six years and
they are also eligible for re-election.
4. The Rajya Sabha is never fully
dissolved; hence it is called a
permanent body.
Which of the above statements is/are
correct?
a.
b.
c.
d.

1 and 2 Only
2, 3 and 4 Only
2 and 3 Only
1, 2, 3 and 4 Only

8. Consider the following statements


about Jainism:
1. Vardhamana, who came to be
known as Mahavira, founded Jainism
in the sixth century BCE.
2. According to Jaina teachings, the
cycle of birth and rebirth is shaped
through karma.
Page 3

Facebook Group: Indian Administrative Service (Raz Kr)

3. Jaina scholars produced a wealth of


literature in a variety of languages
including Tamil.

a.
b.
c.
d.

1 Only
2 Only
Both
None

Which of the above is/are correct?


a.
b.
c.
d.

1 and 2 Only
1 and 3 Only
2 and 3 Only
All the above

9. Consider the following data


1. Total
Coliforms
Organism
MPN/100ml shall be 50 or less
2. pH between 6.5 and 8.5
3. Dissolved Oxygen 6mg/l or more
4. Biochemical Oxygen Demand 5
days 20C 2mg/l or less
The above criterion laid by central
pollution control board of India is meant
for?
a. Outdoor Bathing (organized)
b. Propagation of Wildlife and fisheries
c. Drinking water source without
conventional treatment but after
disinfection
d. Drinking water source after
conventional
treatment
and
disinfection

10. Consider the following statements


about Purusha sukta:
1. 1.Purushasukta is a hymn in the
Yajur veda
2. 2.The Brahmanas cited Purushasukta
to justify their dominance and
superiority in the society
Which of the above is/are correct?
http://insightsonindia.com

11. The constitution of India has


provided for certain qualifications in
order to become members of the
Parliament and State legislatures.
Which of the following statements
is/are correct?
1. Minimum and maximum age
requirements are provided in the
constitution for a candidate
contesting for elections.
2. A person who has undergone
imprisonment for two or more
years for some offence is
disqualified
from
contesting
elections.
3. A candidate standing for election
should have at least completed his
matriculation.
4. There is a reasonable restriction on
the income of the candidates
standing for elections.
Select the correct answer using the
codes below:
a.
b.
c.
d.

2 Only
1 and 2 Only
1, 2 and 3 Only
1, 2 and 4 Only

12. Consider the following statement:


These plains have characteristic
features of mature stage of fluvial
erosional
and
depositional
landforms such as sand bars,
meanders, ox-bow lakes and
braided channels.
Page 4

Facebook Group: Indian Administrative Service (Raz Kr)

To which part of the Northern


Plains of India, does the above
description refer to?
a.
b.
c.
d.

Tarai region
Bhangar and Khadar deposits
Bhabar region
None of the above

13. Consider the following statements


regarding the trends of change in the
Urbanization process in colonial
India :
1. Urbanization in India was sluggish all
through the 19th century and it
accelerated sharply from the first
decade of the 20th century with the
urban population increasing from
about 10 per cent of the total
population to about 15 per cent
between 1900 and 1910.
2. The smaller towns had little
opportunity to grow economically
and only a few cities developed as
the
new
commercial
and
administrative centres at the expense
of other existing urban centres..
Which of the above is/are correct?
a.
b.
c.
d.

given water sample at certain


temperature over a specific time
period.
2. An alternative to measure BOD is
the development of biosensors,
which are devices for the
detection of an analyte that
combines a biological component
with a physicochemical detector
component.

1 Only
2 Only
Both
None

14. With reference to Biological Oxygen


Demand, consider the following
statements
1. Biochemical
oxygen
demand or B.O.D is the amount
of dissolved oxygen needed by
aerobic biological organisms in a
body of water to break down
inorganic material present in a
http://insightsonindia.com

Which of the statements given above


is/are correct?
a.
b.
c.
d.

1 Only
2 Only
Both
None

15. Consider the following statements


with reference to proportional
representation system of election:
1. More than one representative may
be elected from one constituency
2. Candidate who wins the election
may not get majority votes.
3. Voter votes for the candidate rather
than the party in this system.
4. Under the system the entire country
may be considered as a single
constituency.
5. Every party gets seats in the
legislature in proportion to the
percentage of votes that it gets.
Select the correct answer using the
codes below:
a.
b.
c.
d.

2, 3 and 4 Only
1, 4 and 5 Only
1, 2, 3and 5 Only
1, 2, 3, 4 and 5

Page 5

Facebook Group: Indian Administrative Service (Raz Kr)

16. Consider the following statements:


1. Members other than ministers
cannot introduce bills.
2. Introduction of any new tax requires
the approval of the Lok Sabha and
Rajya Sabha
3. Budget is a mechanism for checks
and balance system adopted in India.
4. A bill proposed by a minister is
described as Private Bill
Which of the above statements is/are
correct?
a.
b.
c.
d.

2 Only
3 Only
2 and 3
1, 2, 3 and 4

17. Tamil Nadu witnessed some of the


earliest Bhakti movements (c. sixth
century) which were led by the
Alvars and Nayanars. Consider the
following in relation to the Bakti
movements in the Tamil region:
1. Alvars were the devotees of Shiva
while Nayanars were the devotees of
Vishnu.
2. The Alvars and Nayanars protested
against the caste system and the
dominance of
Brahmanas and
attempted to reform the system.
3. The Tamil bhakti
poets were
opposed to Buddhism and Jainism.
Which of the above is/are correct?
a.
b.
c.
d.

1 and 2 Only
2 and 3 Only
1 and 3 Only
All of the above

http://insightsonindia.com

18. Perhaps the most unique feature of


the Harappan civilisation was the
development of urban centres.
Consider the following statements
about Mohenjodaro, the most wellknown site:
1. The settlement is divided into two
sections, one smaller but higher and
the other much larger but lower.
2. The upper section (citadel) was
walled but the lower section was
not walled.
3. The bricks used for construction
were either sun-dried or baked and
were of a standardized ratio where
the length, breadth and height were
in the ratio 4:2:1 respectively.
Which of the above statement/s is/are
correct?
a.
b.
c.
d.

1 only
1 and 2 only
1 and 3 only
1, 2 and 3

19. Three major geological events in the


distant past have shaped the present
drainage systems of Peninsular India.
They are:
1. Subsidence of the western
flank of the Peninsula leading
to its submergence below the
sea during the early tertiary
period.
2. Upheaval of the Himalayas
when the northern flank of
the Peninsular block was
subjected to subsidence and
the
consequent
trough
faulting.

Page 6

Facebook Group: Indian Administrative Service (Raz Kr)

3. Slight tilting of the Peninsular


block from northeast to the
southwestern direction.
Which of the statements given above is
/are correct?
a.
b.
c.
d.

1 and 2 Only
2 Only
2 and 3 Only
All Three

20. The primary reasons behind deindustrializing India by the British


were,
1. To reduce India to the status of a
mere exporter of important raw
materials for the upcoming
modern industries in Britain.
2. To turn India into a sprawling
market for the finished products
of those industries so that their
continued expansion could be
ensured to the maximum
advantage of their home country
Britain.
3. To cater to the needs of few rich
businessmen from India who
supported the British policies
Which of the statements given above
is/are correct?
a.
b.
c.
d.

1 and 2 Only
2 and 3 Only
1 Only
2 Only

2. There are two all India Civil Services


of India, namely the Indian
Administrative Service (IAS) and the
Indian Police Service (IPS)
3. Power of creation of AIS is vested in
the lower house of the Central
Legislature
Which of the above statements is/are
correct?
a.
b.
c.
d.

1 Only
2 and 3 Only
1 and 3 Only
1, 2 and 3

22. Consider the following features of


the urban settlements in India before
the 20th century.
1. Money was raised for administering
towns through the systematic annual
collection of municipal taxes.
2. Surveys, gathering of statistical data,
and publishing of various official
reports was done regularly.
3. Towns dominated over the rural
population, thriving on the surplus
and taxes derived from agriculture.
Which of the above is/are the features
were non-existant in the pre-colonial India?
a.
b.
c.
d.

1 and 2 Only
1 and 3 Only
3 Only
None

21. With reference to All India Services


(AIS), consider the following
statements:

23. Consider the following statements


about financial sector reforms in
India

1. Only the central government can


take disciplinary action against them

1. One of the major aims of


financial sector reforms is to
reduce the role of RBI from

http://insightsonindia.com

Page 7

Facebook Group: Indian Administrative Service (Raz Kr)

regulator to facilitator of
financial sector
2. Financial
sector
reforms
include reforms of stock
exchange operations
and
foreign exchange market too.

25. Supreme Court is regarded as the


guardian of Fundamental Rights. For
the enforcement of fundamental
rights the Supreme Court issues
writs. Which of the following
statements is/are correct?

Which of the statements given above


is/are correct?

1. Quo warranto is issued when the


court finds that a particular office
holder is not doing legal duty and
thereby is infringing on the right of
an individual.
2. Certiorari is issued by a higher court
when a lower court has considered a
case going beyond its jurisdiction.
3. Habeas corpus means that the court
orders that the arrested persons
should be presented before it. It can
also order to set free an arrested
person if the manner or grounds of
arrest are not lawful or satisfactory.
4. Mandamus is issued when the court
finds that a person is holding office
but is not entitled to hold that office,
it issues the writ of mandamus and
restricts that person from acting as
an office holder.
5. Prohibition means the court orders a
lower court or another authority to
transfer a matter pending before it to
the higher authority or court.

a.
b.
c.
d.

1 Only
2 Only
Both
None

24. Consider the following statements


regarding air pollution in India
1. Adulterated
fuel
increases tailpipe
emissions of
hydrocarbons
(HC),
carbon
monoxide (CO), oxides of
nitrogen (NOx) and particulate
matter (PM)
2. Traffic congestion increases air
pollution
3. Central
Pollution
Control
Board now routinely monitors
four air pollutants namely
sulphur dioxide (SO2), oxides of
nitrogen
(NOx),
suspended
particulate matter (SPM) and
respirable particulate matter
(PM10).
Which of the statements given above
is/are correct?
a.
b.
c.
d.

1 and 3 Only
2 and 3 Only
1 and 3 Only
All

Select the correct answer using the


codes below:
a.
b.
c.
d.

3 Only
2, 3 and 4 Only
1, 3 and 4 Only
1, 2, 3, 4 and 5

26. Siricilla tragedy in Andhra Pradesh


where fifty powerloom workers
committed suicide was the outcome
of
a. Power sector reforms initiated
by the government

http://insightsonindia.com

Page 8

Facebook Group: Indian Administrative Service (Raz Kr)

b. A steep hike in power tariff.


c. Failure
of
the
power
producers in
providing
quality
power
to
the
powerloom industry.
d. All the above

3.

4.

27. Consider the following statements


about the response of government in
a socialist society
1. In a socialist society the
government decides what
goods are to be produced
in accordance with the
needs of society.
2. The government decides
how goods are to be
produced and how they
should be distributed.
3. Strictly, a socialist society
has no private property
since everything is owned
by the state.

Which of the statements given above


is/are correct?
a.
b.
c.
d.

1 and 2 Only
2Only
2 and 3 Only
All

28. Consider the following statements


about ITCZ - The Intertropical
Convergence Zone
1.

2.

It is known by sailors as the


doldrums, and in parts of Asia
as Monsoon trough.
By the middle of July, the low
pressure
ITCZ
shifts
northwards, roughly parallel

http://insightsonindia.com

to the Himalayas between 20


N and 25 N.
The ITCZ being a zone of low
pressure, attracts inflow of
winds
from
different
directions.
This
causes
Monsoon.
It is generally believed that
there is a cause and effect
relationship between the
ITCZ and Westerly Jet
Stream.

Which of the statements given above is


/are correct?
a.
b.
c.
d.

1, 2 and 3 Only
2 and 4 Only
1 and 4 Only
All Four

29. An insight into the social, economic,


cultural and political conditions of
the past are provided by the
descriptions and accounts of a
number of travelers who visited the
subcontinent. Consider the following
statements about some of the
travelers to India:
1. Al-Biruni
who
came
from
Uzbekistan in the eleventh century,
called Rihla in Arabic.
2. Ibn Battuta who came from
Morocco in the fourteenth century,
wrote Kitab-ul-Hind in Arabic.
3. Franois Bernier came from France
in the seventeenth century wrote
accounts of what he saw, frequently
comparing what he saw in India
with the situation in Europe.
Which of the above is/are incorrect?
Page 9

Facebook Group: Indian Administrative Service (Raz Kr)

a.
b.
c.
d.

1 and 2 Only
2 and 3 Only
1 and 3 Only
All the above

30. According to Dr. B.R. Ambedkar the


Constitution of Indian has been framed
after ransacking all the known
Constitutions of the World. With
reference to this statement, which of
the following statements
is/are
incorrect?
1. Principles of Liberty, Equality and
Fraternity were borrowed from
United States Constitution.
2. Written Constitution from United
States.
3. Both Fundamental Duties and Five
year plans were borrowed from
USSR
4. The Federal system was borrowed
from Canada.
5. Principles of Liberty, Equality and
Fraternity were borrowed from the
British Constitution.
Select the correct answer using the codes
below
a.
b.
c.
d.

1 and 5 Only
1, 2, 3 and 4 Only
2, 3, 4and 5 Only
3, 4 and 5 Only

31. Consider the following statements


with reference to carbon footprint
1. It is defined as the total sets
of greenhouse
gas emissions
caused by an organization, event,
product or person
2. Clean Development Mechanism
is
a
mandatory
market
http://insightsonindia.com

mechanism under Kyoto protocol


to reduce carbon footprint
3. The Greenhouse Gas Protocol
(GHG Protocol), a recent
initiative by the UN, is an
international accounting tool for
government and business leaders
to understand, quantify, and
manage greenhouse gas emissions
Which of the above statements is/are
incorrect?
a.
b.
c.
d.

1 and 2 Only
2 Only
3 Only
None

32. Consider the following statements


about the followers of Buddha:
1. Sangha was a body of disciples of the
Buddha to which women were never
allowed as members.
2. chaityas were the shrines which
were regarded as sacred by the
Bhikkus where the prayers were
made.
Which of the above is/are correct?
a.
b.
c.
d.

1 Only
2 Only
Both
None

33. Which of the following is incorrectly


matched?
a. An Isthumus is a narrow strip of
water, usually a small canal,
connecting two larger water
bodies

Page 10

Facebook Group: Indian Administrative Service (Raz Kr)

b. A Bay is a large body of water


connected to an ocean or sea
formed by an inlet of land
c. A Strait is a naturally formed,
narrow,
typically navigable waterway that
connects two larger, navigable
bodies of water
d. A Gulf is a large bay that is an
arm of an ocean or sea

4. Minority educational institutions


have the right to impart education in
their own language.

34. Soon after India had to play an


extensive role in promoting the
industrial sector. Consider the
following reasons,

36. Consider the following statements


about Buddhism:
1. Dhamma as taught by Buddha was
the path of righteous living.
2. According to Buddhist philosophy,
the world is transient and constantly
changing and does not have a soul.
3. According to Buddhist philosophy,
happiness is intrinsic to human
existence.
Which of the above is/are correct?

1. At the time of independence,


Indian industrialists did not
have the capital to undertake
investment
in
industrial
ventures required for the
development of our economy.
2. The market was not big
enough
to
encourage
industrialists to undertake
major projects even if they
had the capital to do so.
Which of the statements given above
is/are correct?
a.
b.
c.
d.

1 Only
2 Only
Both
None

35. With regard to rights of minorities


provided in India Constitution,
consider the following statements:
1. Right to minorities includes both
linguistic and cultural minorities.
2. All minorities can set up their own
educational institutions.
3. Members of the Sikh community
have the right to assemble at a
public place with their kirpans.
http://insightsonindia.com

Select the correct answer using the


codes below:
a.
b.
c.
d.

a.
b.
c.
d.

2 Only
1, and 2Only
1, 2, and 4 Only
1, 2, 3 and 4 Only

1 and 2 Only
1 and 3 Only
2 and 3 Only
All the above

37. Consider the following statements


1. Tariffs are a tax on imported
goods; they make imported
goods more expensive and
discourage their use.
2. Quotas specify the quantity of
goods which can be imported.
3. These are instruments by the
government to protect the
domestic industries from
foreign competition.
4. The policy aimed at replacing
or substituting imports with
domestic production is also
called as inward looking trade
strategy
Page 11

Facebook Group: Indian Administrative Service (Raz Kr)

Which of the statements given above


is/are correct?
a.
b.
c.
d.

1 and 2 Only
1,2and 3 Only
2, 3 and 4 Only
All

considered as a most potent


GHG
3. Water
vapor
and
clouds
contribute more to greenhouse
effect than ozone and carbon
dioxide combined
Which of the statements given above
is/are correct?

38. Consider the following statements


about capital goods
1. They are primary factors of
production
2. Capital goods industry means
industries which can produce
machine tools which are, in turn,
used for producing articles for
current consumption.
3. Aerospace, defense, construction
and machinery businesses make
up most of the capital goods
sector.
4. During the British period, there
was hardly any capital goods
industry to help promote further
industrialization in India.

Which of the statements given above


is/are correct?
a.
b.
c.
d.

1 and 2 Only
1, 2 and 3 Only
3 and 4 Only
All

39. Consider the following statements


about Green House Gases(GHG)
1. They absorb and emit radiation
within the thermal infrared range.
2. Along
with
Perfluorotributylamine (PFTBA),
Carbon Monoxide is also
http://insightsonindia.com

a.
b.
c.
d.

1 Only
1 and 3 Only
1 and 2 Only
All

40. Depletion of groundwater levels


in India can be attributed to which
of the following reasons?
1. Legally Groundwater is a
community resource in India.
2. Subsidized electricity is
given to farmers for irrigation
purposes.
3.
Increasing
commercialization of water
supply services in India.
Select the correct answer using codes
below.
a.
b.
c.
d.

1 only
1 and 3 only
2 and 3
All of the above

41. Consider
definitions

the

following

1. Geology is the science which


deals with the physical
structure and substance of the
earth, their history, and the
processes which act on them.
2. Physiography of an area is the
outcome of structure, process
and the stage of development.
3. Geography is
the branch
of knowledge that studies the
Page 12

Facebook Group: Indian Administrative Service (Raz Kr)

lands, the features,


inhabitants,
and
phenomena of the Earth.

the
the

Which of the statements given above


is/are incorrect?
a.
b.
c.
d.

2 Only
1 Only
None
3 Only

42. With regard to the First Past the


Post (FPTP) system adopted in
India, consider the following
statements:
1. The winning candidate need not
secure a majority of the votes.
2. This method is also called the
Plurality System.
3. The FPTP system offers voters a
choice not simply between parties
but specific candidates.
4. FPTP system is adopted in both the
houses of parliament and state
legislative assemblies.
Which of the above statements is/are
correct?
a.
b.
c.
d.

2 and 4 Only
1 and 4 Only
1, 2 and 3 Only
1, 2, 3 and 4
43. Consider the following statement
regarding
the
agricultural
technologies of the Harappan
settlers:

1. Evidences suggest that different


crops were grown together.
2. Oxen were not used for ploughing as
the bull was not known to them.

http://insightsonindia.com

Which of the above is/are correct?

a.
b.
c.
d.

1 Only
2 Only
Both
None
44. Consider
statements
growth

the
about

following
economic

1. It refers to increase in the


countrys capacity to produce
the output of goods and
services within the country.
2. A steady increase in GDP is a
good indicator of economic
growth
3. Economic growth always
brings
about
economic
development
Which of the statements given above
is/are correct?
a.
b.
c.
d.

1 Only
1 and 2Only
2 and 3 Only
All

45. The Harappans were strongly


devoted
towards
craft
production,
consider
the
following statements with regard
to it:
1. All the materials for the craft
production were procured locally.
2. Metals like copper, bronze and gold
were used but not steel.
3. Weight making was well known to
them with precise system of weights,
Page 13

Facebook Group: Indian Administrative Service (Raz Kr)

usually made of a stone called chert


and the weights did not have any
markings.
Which of the above is/are correct?
a.
b.
c.
d.

1 and 2 Only
2 and 3 Only
1 and 3 Only
All the above

youth stage and yet forms


meanders a typical feature
associated with the mature
stage in the evolution of
fluvial land form.
Which of the statements given above
is/are correct?
a.
b.
c.
d.

46. Consider
the
following
statements with reference to
functions
of
Election
Commission:
1. It determines the timing of elections
and prepares the election schedule.
2. It can postpone or cancel the
election in the entire country or a
specific State or constituency on the
grounds that a free and fair election
may not be possible.
3. During the election process, the
Election Commission can transfer or
even stop the transfers ofthe
administrative officers ofthe State
and central governments.
4. The
Election
Commission
implements model code of conduct
for both parties and candidates
Which of the statements is/are correct?
a.
b.
c.
d.

1, 2 and 4 Only
1, 3 and 4Only
1, 2 and 3 Only
1, 2, 3 and 4 Only
47. Consider
the
following
statements
1. Srinagar, capital city of the
state of Jammu and Kashmir is
located on the banks of
Chenab river.
2. Jhelum in Jammu and
Kashmir state is still in its

http://insightsonindia.com

1 Only
2 Only
Both
None

48. Consider
the
following
statements about Baba Guru
Nanak:
1. Baba Guru Nanak did not believe in
Hinduism and Islam and so he
wished to establish a new religion,
which he named as Sikhism
2. He rejected sacrifices, ritual baths,
image worship, austerities and the
scriptures.
3. He defined five symbols: uncut hair,
a dagger, a pair of shorts, a comb and
a steel bangle which need to be
worn by his followers.
Which of the above is/are correct?
a.
b.
c.
d.

1 Only
2 Only
1 and 3 Only
2 and 3 Only
49. Consider the following about the
recent Doha Conference on
Climate Talks :

1. Supported new ideas of geoengineering to tackle the climate


change
Page 14

Facebook Group: Indian Administrative Service (Raz Kr)

2. Supported implementation of simple


solutions like energy efficiency to
curb climate change.
Which of the following is true in this
connection?
a.
b.
c.
d.

Only 1
Only 2
Both
None

50. Consider the following with


regard to the Vice President of
India:
1. The Vice President is elected by the
electoral college consisting of
members of the Parliament, State
Legislatures.
2. The Vice President may be removed
from his office by a resolution of the
Lok Sabha passed by a majority and
agreed to by the Raj Sabha.
3. The Vice President acts as the exofficio Chairman of the Rajya Sabha
4. The decision of Supreme Court is
final with regard to any disputes in
the election of the Vice President
Which of the above statements is/are
correct?
a.
b.
c.
d.

the

following

1. Bhotias, inhabitants of valleys


of Greater Himalays, are a
nomadic group who migrate
to Bugyals (the summer
grasslands in the higher
reaches)
during
summer
http://insightsonindia.com

Which of the statements given above


is/are correct?
a.
b.
c.
d.

1 and 2 Only
2 and 3 Only
1 Only
All Three

52. Consider
the
following
statements about Kayals of Kerala
1. They are soft water lagoons
formed by the rivers just
before flowing into the
Arabian Sea.
2. Vembanad Kayal is the
largest one and unlike
others it is completely
made of brackish water
Which of the above statements is/are
incorrect?

2 and 3 Only
3 and 4 Only
1, 2 and 3 Only
1, 2, 3 and 4 Only
51. Consider
statements

months and return to the


valleys during winters.
2. The higher reaches of Sikkim
Himalayan
region
are
inhabited by Lepcha tribes.
3. The Dooars or Duars
formations,
the floodplains
and foothills, are prominent in
the Shiwalik ranges

a.
b.
c.
d.

1 Only
2 Only
Both
None

53. Consider the following saints:


1. Kabir
2. Guru Nanak
3. Mirabai
Who among the above used ideas and
terms drawn from Islam, Vedantic
Page 15

Facebook Group: Indian Administrative Service (Raz Kr)

traditions as well as yogic traditions in their


poems?
a.
b.
c.
d.

1. The Ritchies archipelago


and the Labrynth island are
two groups of island in the
Arabian Sea
2. Barren Island, the only active
volcano in India is situated
in the Andaman Islands.

1 Only
1 and 2 Only
1 and 3 Only
All the above

54. Which of the following can be


the causes of Coral bleaching :

Select the right answer using the codes


below?

1. Dumping of industrial waste in


ocean water
2. Increased sedimentation of coral
inhabited areas
3. Decline in Zooplankton levels
4. Elevation of sea water levels
5. Rise in ocean water temperature

a.
b.
c.
d.

57. The National Human Rights


Commission (NHRC) is the
watchdog of the human rights
guaranteed by the Constitution of
India. Consider the following
statements with reference to
NHRC:

Select the correct answer using codes


below
a.
b.
c.
d.

1, 3 and 5
1,2 and 3
4 and 5 only
All of the above

55. Consider
the
following
statements about subsidies
1. Subsidies provide an incentive
for efficient use of resources.
2. According
to
some
economists, subsidies do not
allow prices to indicate the
supply of a good.
Which of the statements given above
is/are correct?
a.
b.
c.
d.

1 Only
2 Only
Both
None

56. Which
of
the
following
statements is/are correct?
http://insightsonindia.com

1 Only
2 Only
Both
None

1. It is a constitutional body.
2. It has suo motu power.
3. It does not have the power of
prosecution.
4. It can intervene in any proceeding
involving allegation of violation of
human rights pending before a court.
Select the correct answer using the
codes below:
a.
b.
c.
d.

3 and 4 Only
2, 3 and 4 Only
1, 3 and 4 Only
1, 2, 3 and 4

58. Chistis were the most influential


group of sufis who migrated to
India in the late twelfth century.
Page 16

Facebook Group: Indian Administrative Service (Raz Kr)

Consider
the
following
statements related to the Chisti
saints:
1. They adopted several features of
Indian devotional traditions.
2. All the compositions of Chistis were
in Persian language.
3. The Sufis never accepted any grants
and donations from the political
elites.
Which of the above is/are correct?
a.
b.
c.
d.

1 Only
1 and 2 Only
1 and 3 Only
all the above

59. Consider
the
following
statements with reference to
antecedent rivers
1. An antecedent stream is a
stream that maintains its
original course and pattern
despite the changes in
underlying rock topography.
2. The Subansiri and the Kosi are
examples of antecedent rivers
in India
Which of the statements given above is
/are correct?
a.
b.
c.
d.

60. Consider
the
following
statements with reference to
Delimitation Commission of
India:
1. Delimitation Commission decides on
reservation of constituencies for
scheduled castes/scheduled tribes
both in house of the people and
Legislative Assemblies.
2. Delimitation Commission decides on
boundaries of constituencies all over
the country.
3. Delimitation Commission is an
independent body.
4. Delimitation Commissions orders
have the force of law and cannot be
called in question before any court.
Select the correct answer using the
codes below:
a.
b.
c.
d.

1 and 2 Only
2 and 3 Only
1, 2 and 3Only
1, 2, 3 and 4
61. The Save Our Species (SoS)
initiative by the IUCN will
support the conservation of
which of the following Indian
species:
1. Red-crowned roof turtle

1 Only
2 Only
Both
None

2. Four-toed terrapin
3. Gangetic Dolphins
Select the correct answer using the codes
below
a.
b.
c.
d.

http://insightsonindia.com

3 only
2 and 3
All of the above
1 only
Page 17

Facebook Group: Indian Administrative Service (Raz Kr)

62. In order to ensure free and fair


elections the constitution of
Indian has provide for an
independent
Election
Commission. Which of the
following
statements
is/are
correct
about
Election
Commission?
1. The Election Commission of India
can either be a single member or a
multi-member body.
2. The Chief Election Commissioner
(CEC) presides over the Election
Commission and has more powers
than
the
other
Election
Commissioners.
3. The Chief and the other Election
Commissioners are appointed by the
President on the recommendation of
the Prime Minister.
4. The removal of the Chief Election
commissioner requires absolute
majority
Select the correct answer using the
codes below:
a.
b.
c.
d.

1 Only
1 and 2 Only
1, 2 and 3 Only
1, 2 and 4 Only
63. The pattern of flow of water in a
river channel over a year is
known as its regime. Consider the
river regime of Ganga river,
1. The Ganga has its minimum
flow during the January-June
period.
2. The maximum flow is
attained either in August or in
September.
3. After September, there is a
steady fall in the flow.

http://insightsonindia.com

The main reason for the statement number


3 is,
a. The weakening of Monsoon
over the Ganga basin after
September
b. Arrival
of
Western
Disturbance
c. Excessive storage of water in
dams upstream
d. None of the above

64. The Sufis were a group of


religious minded people turned
to asceticism and mysticism in
protest against the growing
materialism of the Caliphate as a
religious and political institution.
Which of the following about
Sufism is incorrect?
a. The Sufi saints were critical of the
dogmatic definitions and scholastic
methods of interpreting the Quran
and sunna (traditions of the Prophet)
adopted by theologians.
b. Sufis regarded Prophet Muhammad
as a perfect human being.
c. Sufi silsilas were special rituals of
initiation in which the devotees took
an oath of allegiance, wore a patched
garment, and shaved their hair.
d. The Sufis sought an interpretation of
the Quran on the basis of their
personal experience.
65. Which of the following is true
about Permafrost:
1. It is soil frozen for a long period of
time.
2. Melting permafrost releases longfrozen carbon accelerating global
warming.
Page 18

Facebook Group: Indian Administrative Service (Raz Kr)

3. Most of the permafrost is found in


higher latitudes.
Select the correct answer using the codes
below
a.
b.
c.
d.

a.
b.
c.
d.

1 and 2 only
2 and 3 only
1 only
1,2 and 3

66. Hill stations were a distinctive


feature
of
colonial
urban
development. Consider the below
statements related to the Hill
stations:
1. 1.The founding and settling of hill
stations was initially connected with
the needs of the Governors-General
and viceroys.
2. Hill stations remained exclusive
racial enclaves for Europeans in India
till India got independence.
3. Darjeeling became the official
residence of the commander-in-chief
of the Indian army.
Which of the above is/are correct?
a.
b.
c.
d.

Select the correct answer using the codes


below

1 Only
2 Only
3 Only
None

67. Which of the following is true


about Marine Protected Areas
(MPAs) in India:
1. Except Karnataka, all coastal states
have MPAs.
2. Extraction of marine resources is
partially or fully prohibited here.
3. Marine biodiversity hotspots are
also found in MPAs.
http://insightsonindia.com

2 and 3 only
1 and 2 only
1,2 and 3
3 only

68. Consider
statements,

the

following

1. Land Reforms and Land


Ceiling are measures adopted
to bring equity in the
agricultural sector
2. Eliminating subsidies will
increase
the
inequality
between rich and poor
farmers and violate the goal of
equity.
3. The goal of equity was not
fully served by abolition of
intermediaries.
Which of the statements given above
is/are correct?
a.
b.
c.
d.

1 and 3 Only
1 and 2Only
3 Only
All

69. Below are the instances where


government has taken certain
steps or the individuals or the
society have taken certain actions.
Which of the following is/are
violation of Fundamental Rights?
1. The district court has banned the use
of loudspeakers by the temples,
mosques, and gurudwaras after 10
pm.
Page 19

Facebook Group: Indian Administrative Service (Raz Kr)

2. Karnataka society runs Kannada


medium schools outside Karnataka.
3. A director makes a documentary
film that criticises the policies of the
government.
Select the correct answer using the
codes below:
a.
b.
c.
d.

1 Only
2 and 3Only
1 and 3 Only
None
70. Consider
the
statements
about
cyclonic disturbances

following
Western

1. The
western
cyclonic
disturbances which enter the
Indian subcontinent from the
west and the northwest during
the winter months, originate
over the Mediterranean Sea.
2. An increase in the prevailing
night temperature generally
indicates an advance in the
arrival of these cyclones
disturbances.
3. They are brought into India by
the Easterly jet stream.
Which of the statements given above is
/are correct?
a.
b.
c.
d.

1 and 2 Only
2 and 3 Only
3 Only
All Three

India,
and
provided
the
framework within which the
work of constitution-making was
to
proceed.
consider
the
following statements with respect
to the "Objectives Resolution :
1. It was introduced by Sardar Patel in
the Constituent Assembly.
2. It proclaimed India to be an
Independent Sovereign, Socialist,
Secular Republic.
Which of the above is/are correct?
a.
b.
c.
d.

1 Only
2 Only
Both
None

72. Consider
statements

the

following

1. This river occupies the eastern


margins of the Chotanagpur
Plateau where it flows
through a rift valley.
2. The Barakar is its main
tributary.
To which river does the above description
refer to?
a.
b.
c.
d.

Damodar
Hugli
Subarnarekha
Brahmani

71. The "Objectives Resolution


introduced in the Constituent
Assembly on 13 December 1946
was a momentous resolution that
outlined the defining ideals of the
Constitution of Independent
http://insightsonindia.com

Page 20

Facebook Group: Indian Administrative Service (Raz Kr)

73. The government from time to


time has given effect to some
Directive Principles of State
Policy (DPSP). Which of the
following
statements
is/are
correct with reference to the
steps taken by government in this
regard?
1.
2.
3.
4.

Abolition of Zamindari system


Nationalisation of banks
Fixed minimum wages
Formation
of
Panchayat
institutions
5. Mid-day meal scheme.
6. Abolition of Privy Purses

raj

Select the correct answer using the


codes below:
a.
b.
c.
d.

1, 3, 4 and 6 Only
1, 2, 3 and 4 Only
1, 2, 3, 4 and 5 Only
1, 2, 3, 4, 5 and 6

74. Consider
statements

the

following

1. The Second Plan laid down


the basic ideas regarding goals
of Indian planning.
2. The essence of the Feldman
Mahalanobis model adopted
during the Second Plan is a
shift in the pattern of
industrial investment towards
building
up
a domestic consumption
goods sector.
3. The First Five Year Plan
stressed investment for capital
accumulation in the spirit of
the one-sector HarrodDomar
model.
http://insightsonindia.com

4. The Harrod Domar model


suggests that the economic
rate of growth depends on the
level of savings, and the
productivity of investment
Which of the statements given above
is/are correct?
a.
b.
c.
d.

1, 2 and 3 Only
1 and 2Only
2, 3 and 4 Only
All

75. Consider
the
following
statements about the Himalaya
mountains orientation
1. Himalayas in the Darjeeling
and Sikkim regions lie in an
east-west direction
2. While in Arunachal Pradesh
they are from southwest to
the northwest direction.
3. In Nagaland, Manipur and
Mizoram, they are in the
north-south direction.
Which of the statements given above
is/are correct?
a.
b.
c.
d.

1 and 2 Only
1 and 3 Only
None
All

76. Consider
statements

the

following

1. The Kashmir Himalayas are


famous
for
Karewa
formations, which are useful
for the cultivation of Zafran,
i.e. pure saffron.
2. Karewas are the thick deposits
of glacial clay and other
Page 21

Facebook Group: Indian Administrative Service (Raz Kr)

materials embedded with


moraines.
3. Karewas are predominantly
lacustrine in origin
4. The remains of Buddhist
Stupas and even artefacts f
stone age man have been
found in the Karewas

Which of the statements given above


is/are correct?
a.
b.
c.
d.

1 and 2 Only
1,2 and 3 Only
3 and 4 Only
All

77. In the Indian context, which of


the following situation demands
reasonable restriction on the right
to freedom?
1. Workers go on strike as their
demands are not met.
2. Dalits are denied entry in a
temple. A march is being
organised to forcibly enter the
temple.
3. Hundreds of tribals armed
with their traditional weapons
(bows arrows and axes) have
blocked the road. They are
demanding that the surplus
land taken for an industry be
returned to them.
4. People
of
a
minority
community have assembled
for a peace march after
communal riots in the city.
5. Government
officials
expressing their grievances
against
the
government
through media.

http://insightsonindia.com

Select the correct answer using the


codes below:
a.
b.
c.
d.

2 and 3 Only
1, 2, and 3 Only
1, 2, 3 and 5 Only
1, 2, 3, 4 and 5

78. The
Constituent
Assembly
deliberated on a number of
important issues with many
differences among the members
in their views and ideas.
Continuation
of
separate
electorates for Muslims was one
such issue. Consider the following
statements related to it:
1. 1.all the Muslim members of the
Constituent Assembly supported the
demand for separate electorates
which was rejected by others.
2. It was Begum Aizaas Rasul, the lone
Muslim woman member of the
Constituent Assembly who made a
powerful plea for continuing
separate electorates.
Which of the above is/are correct?
a.
b.
c.
d.

1 Only
2 Only
Both
None

79. Consider
statements

the

following

1. Rivers Mahanadi, Narmada,


Tapi and Yamuna flow
through rift valleys
2. A rift valley is a linear-shaped
lowland between several
highlands or mountain ranges
Page 22

Facebook Group: Indian Administrative Service (Raz Kr)

created by the action of a


geologic fault.
3. Lake Tanganyika, the second
largest freshwater lake in the
world by volume, and
the second deepest, in both
cases, is located in a rift valley
Which of the statements given above
is/are correct?
a.
b.
c.
d.

1 and 2 Only
1 and 3 Only
2 and 3 Only
All
80. Consider
the
following
statements on Northern Plains of
India
1. It was a geo-synclinal
depression
2. It has been gradually
filled by the sediments
brought
by
the
Himalayan
and
Peninsular rivers.
3. Average
depth
of
alluvial deposits in
these plains ranges from
1 2 km.

Which of the statements given above


is/are correct?
a.
b.
c.
d.

1 and 2 Only
1 and 3 Only
2 and 3 Only
All

http://insightsonindia.com

81. Consider the following statements with


reference to Fundamental Rights
provided in Constitution of India:
1. The Motilal Nehru committee had
demanded a bill of rights as far back
as in 1928.
2. Sometimes a person can be arrested
simply out of an apprehension that
he or she is likely to engage in
unlawful activity.
3. Right to Strike is a Fundamental
Right.
4. The government can impose
restrictions in certain areas declaring
the assembly of five or more persons
as unlawful.
Which of the statements given above
is/are correct?
a.
b.
c.
d.

3 Only
1 and 3 Only
1, 2 and 4 Only
1, 2, 3 and 4
82. Consider
the
following
statements with reference to the
Industrial Policy Resolution 1956,
1. This policy was used for
promoting
industry
in
backward regions; it was easier
to obtain a license if the
industrial unit was established
in an economically backward
area.
2. This resolution formed the
basis of the Third Five Year
Plan, the plan which tried to
build the basis for a socialist
pattern of society.
3. Under this policy even an
existing industry had to obtain
a license for expanding output
or for diversifying production.
Page 23

Facebook Group: Indian Administrative Service (Raz Kr)

Which of the statements given above


is/are correct?
a.
b.
c.
d.

1 and 2 Only
1 Only
1 and 3 Only
All

83. Consider
the
following
statements about Small Scale
Industries in India
1. An industrial undertaking in
which the investment in fixed
assets in plant and machinery
whether held on ownership
terms on lease or on hire
purchase does not exceed Rs 1
million is called a Small Scale
industry in India.
2. In 1955, the Village and Smallscale Industries Committee,
also
called
the
Karve
Committee,
noted
the
possibility of using small-scale
industries for promoting rural
development.
3. The production of a number
of products was reserved for
the small-scale industry to
protect them from big
industries.
Which of the statements given above
is/are correct?
a.
b.
c.
d.

1 and 2 Only
1 and 3 Only
2 and 3 Only
All

http://insightsonindia.com

84. Which of the following about the


Constituent
Assembly
is
incorrect?
a. Rajendra Prasad was the
President of the Constituent
Assembly and Dr. B R
Ambedkar served as the
Chairman of the Drafting
Committee
of
the
Constitution.
b. The Constituent Assembly
never debated the important
issues of cultural rights and
social justice on the floor of
the Assembly.
c. The discussions within the
Constituent Assembly were
also influenced by the
opinions expressed by the
public.
d. The Constituent Assembly
that came into being was
dominated by congress with
more than 80 per cent of the
members of the Constituent
Assembly being Congress
members.
85. Consider
statements:

the

following

1. First Past the Post (FPTP)system is


followed for all the elections in
India.
2. The Central Election Commission
does not supervise Panchayat and
Municipal elections.
3. The Election Commissioners can be
removed
by
Chief
Election
Commissioner.
4. Appointment of more than one
Election Commissioners in the
Election Commission is mandatory.

Page 24

Facebook Group: Indian Administrative Service (Raz Kr)

5. State Election Commission is


independent of Central Election
Commission.

3. Tipaimukh
Dam is
a
proposed embankment
dam on the river Barak in
Manipur

Which of the above statements is/are


correct?
a.
b.
c.
d.

1, and 4 Only
2, 4 and 5 Only
1, 4 and 5Only
1, 2 and 4 Only

Which of the statements given above


is/are correct?
a.
b.
c.
d.

1 and 2 Only
2 and 3 Only
1 and 3 Only
All Three

86. Consider the following with


regard to the President of India.
1. The President does not exercise
his/her discretion on the advice
given by the Council of Ministers.
2. Only elected members of the
Legislative Assemblies and both the
Houses of the Parliament take part
in electing the President.
3. The President can send the bill back
to the Parliament asking it to
reconsider the bill.
4. The Presidents ordinance making
power is not a discretionary power.
Which of the above statements is/are
correct?
a.
b.
c.
d.

1, and 4 Only
1, 2 and 3 Only
2 and 3 Only
2, 3 and 4 Only

87. Consider
the
following
statements about Barak river
1. It is an important river in
Manipur and Mizoram.
2. Silchar, important town of
Manipur is located on the
banks of Barak river
http://insightsonindia.com

88. Consider
the
following
statements with reference to
rivers
1. A river drains the water
collected from a specific area,
which is called its catchment
area.
2. An area drained by a river and
its tributaries is called a
drainage basin.
3. The boundary line separating
one drainage basin from the
other is known as the
watershed. T
4. Watersheds are small in area
while the basins cover larger
areas.
Which of the statements given above
is/are incorrect?
a.
b.
c.
d.

1 and 2 Only
1, 2 and 3 Only
1 and 3 Only
None

89. Consider the following,


On the basis of the size of the
watershed, the drainage basins of
India are grouped into three
categories:
Page 25

Facebook Group: Indian Administrative Service (Raz Kr)

1. Major river basins with more


than 20,000 sq. km of
catchment area.
2. Medium river basins with
catchment
area
between
2,000-20,000 sq. km
3. Minor river basins with
catchment area of less than
2,000 sq. km
To which of the categories do
rivers Tapi and Pennar belong to?
a.
b.
c.
d.

1
2
3
None

90. The set of policies under the


New Economic Policy of 1991-92
can broadly be classified into two
groups: the stabilization measures
and the structural reform
measures. Now consider the
following statements,
1. Stabilization measures are
short term
measures,
intended to correct some of
the weaknesses that have
developed in the balance of
payments and bring inflation
under control
2. Structural reform policies are
long-term measures, aimed at
improving the efficiency of
the economy and increasing its
international competitiveness
by removing the rigidities in
various segments of the Indian
economy
3. Maintaining sufficient foreign
exchange reserves was part of
structural reforms.

http://insightsonindia.com

Which of the statements given above


is/are correct?
a.
b.
c.
d.

1 and 2 Only
1,2and 3 Only
2 Only
All

91. The river Kosi frequently changes


its course. It is because,
1. The Kosi brings huge quantity
of sediments from its upper
reaches and deposits it in the
plains. The course gets
blocked, and consequently,
the river changes its course.
2. Extensive
soil erosion and landslides in
its upper catchment have
produced a silt yield of about
19 m/ha/year
(10 cu yd/acre/yr), one of the
highest in the world.
Which of the statements given
above is/are correct?
a.
b.
c.
d.

1 Only
2 Only
Both
None

92. Consider
the
following
statements with reference to
Indias peninsular bloc
1. The Peninsula is formed
essentially by a great
complex of very ancient
gneisses
and
granites,
which constitutes a major
part of it.
2. The Thar desert in
Rajasthan is also an
extension of the Peninsular
block
Page 26

Facebook Group: Indian Administrative Service (Raz Kr)

3. A part of the West coast


was submerged in the
ocean in the past

94. Which of the following are the


factors responsible for the land
degradation in India?

Which of the statements given above


is/are correct?

1. Unsustainable fuel wood and


fodder extraction
2. Shifting cultivation
3. Encroachment into forest
lands
4. Improper crop rotation
5. Improper
planning
and
management of irrigation
systems
6. Open access resource
7. Poverty of the agriculturedependent people

a.
b.
c.
d.

1 and 2 Only
1 and 3 Only
2 and 3 Only
All
93. Article 74 provides for a council
of ministers with the Prime
Minister at the head to aid and
advice the President. Which of
the following is/are correct with
regard to council of ministers?

1. The size of the Council of Ministers


should not exceed 15 percent of total
number of members of the House of
People and this is also applicable to
the Legislative Assembly in the
States.
2. The Council of Ministers is
collectively responsible to both the
Lok Sabha and Rajya Sabha.
3. Vote of no confidence even against a
single minister leads to the
resignation of the entire Council of
Ministers.
4. The death or resignation of the
Prime Minister automatically brings
about the dissolution of the Council
of Ministers.
Select the correct answer using the
codes below:
a.
b.
c.
d.

Select the right answer using the codes


below.
a.
b.
c.
d.

1,2,3,5,6 and 7 Only


2,3,4 and 5 Only
1,2,3,4 and 6 Only
All

95. Consider the following important


characteristics of Indias foreign
trade throughout the colonial
period,
1. Foreign trade generated a large
export surplus.
2. This export surplus did not result in
any flow of gold or silver into India.
3. This surplus was used to make
payments for the expenses incurred
by an office set up by the colonial
government in Britain, such as
expenses on war fought by the
British government itself.

1 Only
1, 2 and 4 Only
1, 3 and 4Only
1, 2, 3 and 4 Only

Which of the statements given above


is/are correct?

http://insightsonindia.com

Page 27

a.
b.
c.
d.

1 and 2 Only
1 and 3 Only
2 and 3 Only
All

Facebook Group: Indian Administrative Service (Raz Kr)

96. Consider
the
following
statements about the impact of
introduction of railways by the
British on Indian economy
1. It
enabled
people
to
undertake long distance travel
and
thereby
break
geographical and cultural
barriers
2. It expanded volume of Indias
exports and helped farmers
get profit
3. It fostered commercialization
of Indian agriculture which
adversely
affected
the
comparative self-sufficiency of
the village economies in India

b. 2 and 3Only
c. 1 and 3Only
d. 1, 2 and 3
98. Consider
the
following
statements on different types of
capital (economic)
1. Physical capital is completely
mobile between countries
except for some artificial trade
restrictions. Whereas Human
capital is not perfectly mobile
between countries.
2. Both forms of capital (human
and physical) depreciate with
time, but physical depreciates
fast whereas human capitals
depreciation can be reduced
through investment.
3. Human capital benefits the
society whereas, physical
capital benefits the owner

Which of the statements given above


is/are correct?
a.
b.
c.
d.

1 and 3 Only
1 and 2Only
2 and 3 Only
All

97. Consider
statements:

Which of the statements given above


is/are correct?
the

following

1. It is the constitutional obligation of


the Prime Minister to communicate
to the President all decisions of the
Council of Ministers relating to the
administration of the affairs of the
Union and proposals for legislation.
2. The
Governor
has
more
discretionary powers compared to
the President.
3. The writ jurisdiction of the Supreme
Court is narrower compared to the
High Court.
Which of the above statements is/are
correct?

a.
b.
c.
d.

1 and 2 Only
2 and 3 Only
3 Only
All

99. The power wielded by the Prime


Minister flows from various
sources. Which of the following
are such sources of power of the
Prime Minister?
1. Control over the Council of
Ministers
2. Leadership of the Lok Sabha
3. Command over the bureaucratic
machine
4. Access to media
5. Projection of personalities during
elections

a. 1 Only
http://insightsonindia.com

Page 28

Facebook Group: Indian Administrative Service (Raz Kr)

6. Projection as national leader during


international
summitry
and
foreign visits.
Select the correct answer using the
codes below:
a.
b.
c.
d.

1, 2, 3 and 4 Only
1, 2, 5 and 6Only
1, 2, 3, 5 and 6Only
1, 2, 3, 4, 5 and 6 Only
100. Consider
statements

the

Which of the statements given above


is/are incorrect?
following

1. The Environmental
Performance Index (EPI) is a

http://insightsonindia.com

method of quantifying and


numerically
marking
the
environmental performance of
a state's policies.
2. It was designed to supplement
the environmental targets set
forth
in
the
United
Nations Millennium
Development Goals.

a.
b.
c.
d.

1 Only
2 Only
Both
None

Page 29

Facebook Group: Indian Administrative Service (Raz Kr)

INSIGHTS MOCK PRELIMS TEST-4

SOLUTIONS

1. Solution: a)
Hydrofluorocarbons (HFCs)
Produced mostly in developed countries, HFCs replaced chlorofluorocarbons (CFCs) and
hydrochlorofluorocarbons (HCFCs), that were phased out under the Montreal Protocol on
Substances that Deplete the Ozone Layer. HFCs pose no harm to the ozone layer because, unlike
CFCs and HCFCs, they do not contain chlorine that depletes the ozone layer. But it has been
established that HFCs are not innocuous either.
They are super-greenhouse gases with an extremely high global warming potential. This means
they are capable of trapping enormous amounts of infrared radiations in the atmosphere and can
cause a greenhouse effect a thousand times stronger than carbon dioxide. It has been four years
since the issue of bringing HFCs under the Protocol's ambit was raised.
Developed countries say that since the rise in the emission of super-greenhouse gases is a
consequence of the phasing out of CFCs and HCFCs under the Montreal Protocol, the same
agreement should monitor them. Developing countries like India, China and Brazil, however, say
that the emission and regulation of greenhouse gases fall under the purview of the United Nations
Framework Convention on Climate Change (UNFCCC) and HFCs already figure in the basket of
six greenhouse gases under the Kyoto Protocol. Developed countries following the Kyoto
Protocol report their HFC emission data to UNFCCC; parties to the Montreal Protocol have no
such obligation. At the root of this argument is a doubt. Developing countries are apprehensive
about the high cost of transition from HFCs to a safer option.
2. Solution: a)
If you work out the latitudinal and longitudinal extent of India, they are roughly about 30
degrees, whereas the actual distance measured from north to south extremity is 3,214 km, and
that from east to west is only 2,933 km.
This difference is based on the fact that the distance between two longitudes decreases towards
the poles whereas the distance between two latitudes remains the same everywhere.

3. Solution c)
Most nations are an amalgamation of a complex set of historical traditions; they weave
together the diverse groups that reside within the nation in different ways. For example,
German identity was constituted by being ethnically German. The constitution gave
expression to this identity. The Indian Constitution, on the other hand, does not make ethnic
identity a criterion for citizenship.
http://insightsonindia.com

INSIGHTS

Page 1

Facebook Group: Indian Administrative Service (Raz Kr)

INSIGHTS MOCK PRELIMS TEST-4

SOLUTIONS

Formally, the Constitution was made by the Constituent Assembly which had been elected
for undivided India. It held its first sitting on 9 December1946 and re-assembled as
Constituent Assembly for divided India on 14 August 1947. Its members were elected by
indirect election by the members of the Provisional Legislative Assemblies that had been
established in 1935. The Constituent Assembly was composed roughly along the lines
suggested by the plan proposed by the committee of the British cabinet, known as the
Cabinet Mission.
There were legitimate differences of principle. And the differences were many: should India
adopt a centralised or decentralised system of government? What should be the relations
between the States and the centre? What should be the powers of the judiciary? Should the
Constitution protect property rights? Almost every issue that lies at the foundation of a
modern state was discussed with great sophistication. Only one provision of the Constitution
was passed without virtually any debate: the introduction of universal suffrage (meaning that
all citizens reaching a certain age, would be entitled to be voters irrespective of religion, caste,
education, gender or income). So, while the members felt no need at all to discuss the issue of
who should have the right to vote, every other matter was seriously discussed and debated.
Reference : Page 9, 15, 17(Indian Constitution at Work Class XI)
4.

Solution: d)

Explanation: The Harappan seal is possibly the most distinctive artefact of the Harappan or
Indus valley civilisation. Made of a stone called steatite, seals like this one often contain animal
motifs and signs from a script that remains undeciphered.
Metals were used in other artefacts, but for seals special clay was frequently used.
Archaeologists have been able to reconstruct dietary practices from finds of charred grains and
seeds. These are studied by archaeo-botanists, who are specialists in ancient plant remains. Grains
found at Harappan sites include wheat, barley, lentil, chickpea and sesame. Millets are found from
sites in Gujarat. Finds of rice are relatively rare.
Reference: Page No. 1,2 and 3 (THEMES IN INDIAN HISTORY PART I)
http://www.thehindu.com/books/archaeometallurgy-of-indus-civilisation/article2913312.ece

http://insightsonindia.com

INSIGHTS

Page 2

Facebook Group: Indian Administrative Service (Raz Kr)

INSIGHTS MOCK PRELIMS TEST-4

SOLUTIONS

5. Solution: d)
Both are correct statements.
http://moef.gov.in/citizen/specinfo/eia.html

6. Solution: d)
All are correct.

7. Solution b)
Each of the two Houses of the Parliament has different bases of representation. The Rajya
Sabha represents the States of India. It is an indirectly elected body. Residents of the State
elect members to State Legislative Assembly. The elected members of State Legislative
Assembly in turn elect the members of Rajya Sabha.
The number of members to be elected from each State has been fixed by the fourth schedule
of the Constitution.
Members of the Rajya Sabha are elected for a term of six years. They can get re-elected. All
members of the Rajya Sabha do not complete their terms at the same time. Every two years,
one third members of the Rajya Sabha complete their term and elections are held for those
one third seats only. Thus, the Rajya Sabha is never fully dissolved. Therefore, it is called the
permanent House of the Parliament.
Reference : Page 105, 106 (Indian Constitution at Work Class XI)

8.

Solution c)

The basic philosophy of the Jainas was already in existence in north India before the birth of
Vardhamana, who came to be known as Mahavira, in the sixth century BCE.
Jaina scholars produced a wealth of literature in a variety of languages Prakrit, Sanskrit and
Tamil. According to Jaina teachings, the cycle of birth and rebirth is shaped
through karma. Asceticism and penance are required to free oneself from the cycle of karma.
Reference: Page No. 88,89(THEMES IN INDIAN HISTORY PART I)

http://insightsonindia.com

INSIGHTS

Page 3

Facebook Group: Indian Administrative Service (Raz Kr)

INSIGHTS MOCK PRELIMS TEST-4

SOLUTIONS

9. Solution: c)
http://www.cpcb.nic.in/Water_Quality_Criteria.php

10. Solution b)
Purusha sukta is a hymn in the Rig veda. It describes the sacrifice of Purusha, the primeval man.
All the elements of the universe, including the four social categories, were supposed to have
emanated from his body: The Brahmana was his mouth, of his arms was made the Kshatriya.
His thighs became the Vaishya, of his feet the Shudra was born.
Reference: Page No. 61(THEMES IN INDIAN HISTORY PART I)
11. Solution a)
What is true of the right to vote is also true of right to contest election. All citizens have the
right to stand for election and become the representative of the people.
However, there are different minimum age requirements for contesting elections. For
example, in order to stand for Lok Sabha or Assembly election, a candidate must be at least 25
years old.
There are some other restrictions also. For instance, there is a legal provision that a person
who has undergone imprisonment for two or more years for some offence is disqualified from
contesting elections.
But there are no restrictions of income, education or class or gender on the right to contest
elections. In this sense, our system of election is open to all citizens.
Reference : Page 67(Indian Constitution at Work Class XI)

12. Solution: b)
The northern plains are formed by the alluvial deposits brought by the rivers the Indus, the
Ganga and the Brahmaputra. These plains extend approximately 3,200 km from the east to the
west. The average width of these plains varies between 150-300 km. The maximum depth of
alluvium deposits varies between 1,000-2,000 m. From the north to the south, these can be

http://insightsonindia.com

INSIGHTS

Page 4

Facebook Group: Indian Administrative Service (Raz Kr)

INSIGHTS MOCK PRELIMS TEST-4

SOLUTIONS

divided into three major zones: the Bhabar, the Tarai and the alluvial plains. The alluvial plains
can be further divided into the Khadar and the Bhangar.
Bhabar is a narrow belt ranging between 8-10 km parallel to the Shiwalik foothills at the breakup of the slope. As a result of this, the streams and rivers coming from the mountains deposit
heavy materials of rocks and boulders, and at times, disappear in this zone. South of the Bhabar is
the Tarai belt, with an approximate width of 10-20 km where most of the streams and rivers reemerge without having any properly demarcated channel, thereby, creating marshy and swampy
conditions known as the Tarai. This has a luxurious growth of natural vegetation and houses a
varied wild life. The south of Tarai is a belt consisting of old and new alluvial deposits known as
the Bhangar and Khadar respectively. These plains have characteristic features of mature stage of
fluvial erosional and depositional landforms such as sand bars, meanders, ox-bow lakes and
braided channels. The Brahmaputra plains are known for their riverine islands and sand bars. Most
of these areas are subjected to periodic floods and shifting river courses forming braided streams.

13. Solution b)
After 1800, urbanization in India was sluggish. All through the nineteenth century up to the first
two decades of the twentieth, the proportion of the urban population to the total population in
India was extremely low and had remained stagnant. In the forty years between 1900 and 1940
the urban population increased from about 10 per cent of the total population to about 13 per
cent. The smaller towns had little opportunity to grow economically. Calcutta, Bombay and
Madras on the other hand grew rapidly and soon became sprawling cities. In other words, the
growth of these three cities as the new commercial and administrative centres was at the
expense of other existing urban centres.
Reference: Page No. 322(THEMES IN INDIAN HISTORY PART III)
14. Solution: b)
Biochemical oxygen demand or B.O.D is the amount of dissolved oxygen needed by aerobic
biological organisms in a body of water to break down organic material present in a given water
sample at certain temperature over a specific time period.
Biochemical oxygen demand is the amount of oxygen required for microbial metabolism of
organic compounds in water. This demand occurs over some variable period of time depending
on temperature, nutrient concentrations, and the enzymes available to indigenous microbial
populations. The amount of oxygen required to completely oxidize the organic compounds to
carbon dioxide and water through generations of microbial growth, death, decay, and cannibalism
is total biochemical oxygen demand (total BOD).
http://insightsonindia.com

INSIGHTS

Page 5

Facebook Group: Indian Administrative Service (Raz Kr)

INSIGHTS MOCK PRELIMS TEST-4

SOLUTIONS

15. Solution b)
Larger geographical areas are demarcated as constituencies. The entire country may be considered
as a single constituency.
More than one representative may be elected from one constituency Candidate who wins the
election gets majority of votes unlike in First Past the Post (FPTP) system.
Voter votes for the party rather than the candidate (which is followed in First past the Vote
system) Every party gets seats in the legislature in proportion to the percentage of votes that it
gets.
In other electoral systems, especially PR systems, voters are often asked to choose a party and the
representatives are elected on the basis of party lists. As a result, there is no one representative
who represents and is responsible for one locality. In constituency based system like the FPTP,
the voters know who their own representative is and can hold him or her accountable.
Reference : Page 59 (Indian Constitution at Work Class XI)
16. Solution b)
No major bill is introduced in the Parliament without the approval of the Cabinet. Members
other than ministers can also introduce bills but these have no chance of being passed without
the support of the government.
If the Government of India proposes to introduce any new tax, it has to get the approval of
the Lok Sabha.
The Financial powers of the Parliament, involve grant of resources to the government to
implement its programmes. The government has to give an account to the Legislature about
the money it has spent and resources that it wishes to raise. The legislature also ensures that
the government does not misspend or overspend. This is done through the budget and annual
financial statements.
A bill is a draft of the proposed law. There can be different types of bills. When a nonminister proposes a bill, it is called private members Bill. A bill proposed by a minister is
described as Government Bill. Let us now see the different stages in the life of a bill.

http://insightsonindia.com

INSIGHTS

Page 6

Facebook Group: Indian Administrative Service (Raz Kr)

INSIGHTS MOCK PRELIMS TEST-4

SOLUTIONS

Reference : Page 108, 112(Indian Constitution at Work Class XI)

17. Solution b)
Alvars were the devotees of Vishnu while Nayanars were the devotees of Shiva. Some historians
suggest that the Alvars and Nayanars initiated a movement of protest against
the caste system and the dominance of Brahmanas or at least attempted to reform the system. To
some extent this is corroborated by the fact that bhaktas hailed from diverse social backgrounds
ranging from Brahmanas to artisans and cultivators and even from castes considered
untouchable. one of the major themes in Tamil Bhakti hymns is the poets opposition to
Buddhism and Jainism. This is particularly marked in the compositions of the Nayanars.
Reference: Page No. 143-146(THEMES IN INDIAN HISTORY PART II)
18. Solution c)
Explanation: Both the upper and the lower sections were walled and separated from each other.
Reference: Page No. 5 and 6(THEMES IN INDIAN HISTORY PART I)
19. Solution: a)
Three major geological events in the distant past have shaped the present drainage systems of
Peninsular India: (i) Subsidence of the western flank of the Peninsula leading to its submergence
below the sea during the early tertiary period. Generally, it has disturbed the symmetrical plan of
the river on either side of the original watershed.
(ii) Upheaval of the Himalayas when the northern flank of the Peninsular block was subjected to
subsidence and the consequent trough faulting. The Narmada and The Tapi flow in trough faults
and fill the original cracks with their detritus materials. Hence, there is a lack of alluvial and
deltaic deposits in these rivers. (iii) Slight tilting of the Peninsular block from northwest to the
southeastern direction gave orientation to the entire drainage system towards the Bay of Bengal
during the same period.

20. Solution: a)
As in the case of agriculture, so also in manufacturing, India could not develop a sound industrial
base under the colonial rule. Even as the countrys world famous handicraft industries declined,
http://insightsonindia.com

INSIGHTS

Page 7

Facebook Group: Indian Administrative Service (Raz Kr)

INSIGHTS MOCK PRELIMS TEST-4

SOLUTIONS

no corresponding modern industrial base was allowed to come up to take pride of place so long
enjoyed by the former.
The primary reasons behind de-industrializing India by the British were: Firstly, to reduce India to
the status of a mere exporter of important raw materials for the upcoming modern industries in
Britain. Secondly, o turn India into a sprawling market for the finished products of those
industries so that their continued expansion could be ensured to the maximum advantage of their
home country Britain.
Third statement is unfounded and unsubstantiated.

21. Solution (a)


There are three all India Civil Services of India, namely the Indian Administrative Service
(IAS), the Indian Forest Service (IFS) & the Indian Police Service (IPS).
AIS include the officer is governed by the service conditions laid down by the central
government? An IAS or IPS officer is assigned to a particular State, where he or she works
under the supervision of the State government. However, the IAS or IPS officers are
appointed by the central government, they can go back into the service of the central
government and most importantly, only the central government can take disciplinary action
against them.
Rajya Sabha is empowered to make laws creating one or more All India Services, which
would be common to the Union and State, if it is deemed to serve the national interest.
Reference : Page 95, 96(Indian Constitution at Work Class XI)
22. Solution a)
In the colonial India, money was raised for administering towns through the systematic annual
collection of municipal taxes. surveys, gathering of statistical data, and publishing of various
official reports was done regularly. But towns dominated over the rural population even in PreColonial times also, thriving on the surplus and taxes derived from agriculture.
Reference: Page No. 317,320 &321(THEMES IN INDIAN HISTORY PART III)
23. Solution: c)

http://insightsonindia.com

INSIGHTS

Page 8

Facebook Group: Indian Administrative Service (Raz Kr)

INSIGHTS MOCK PRELIMS TEST-4

SOLUTIONS

Financial Sector Reforms:


Financial sector includes financial institutions such as commercial banks, investment banks, stock
exchange operations and foreign exchange market. The financial sector in India is controlled by
the Reserve Bank of India (RBI). You may be aware that all the banks and other financial
institutions in India are controlled through various norms and regulations of the RBI. The RBI
decides the amount of money that the banks can keep with themselves, fixes interest rates,
nature of lending to various sectors etc. One of the major aims of financial sector reforms is to
reduce the role of RBI from regulator to facilitator of financial sector. This means that the
financial sector may be allowed to take decisions on many matters without consulting the RBI.

24. Solution: d)
Traffic congestion reduces average traffic speed. At low speeds, scientific studies reveal, vehicles
burn fuel inefficiently and pollute more per trip. For example, a study in the United States found
that for the same trip, cars consumed more fuel and polluted more if the traffic was congested,
than when traffic flowed freely. At average trip speeds between 20 to 40 kilometers per hour, the
cars pollutant emission was twice as much as when the average speed was 55 to 75 kilometers per
hour. At average trip speeds between 5 to 20 kilometers per hour, the cars pollutant emissions
were 4 to 8 times as much as when the average speed was 55 to 70 kilometers per hour. Fuel
efficiencies similarly were much worse with traffic congestion.
India's Central Pollution Control Board now routinely monitors four air pollutants namely
sulphur dioxide (SO2), oxides of nitrogen (NOx), suspended particulate matter (SPM) and
respirable particulate matter (PM10). These are target air pollutants for regular monitoring at 308
operating stations in 115 cities/towns in 25 states and 4 Union Territories of India. The monitoring
of meteorological parameters such as wind speed and direction, relative humidity and
temperature has also been integrated with the monitoring of air quality. The monitoring of these
pollutants is carried out for 24 hours (4-hourly sampling for gaseous pollutants and 8-hourly
sampling for particulate matter) with a frequency of twice a week, to yield 104 observations in a
year.

25. Solution a)
Habeas corpus: A writ of habeas corpus means that the court orders that the arrested person
should be presented before it. It can also order to set free an arrested person if the manner or
grounds of arrest are not lawful or satisfactory.

http://insightsonindia.com

INSIGHTS

Page 9

Facebook Group: Indian Administrative Service (Raz Kr)

INSIGHTS MOCK PRELIMS TEST-4

SOLUTIONS

Mandamus: This writ is issued when the court finds that a particular office holder is not doing
legal duty and thereby is infringing on the right of an individual.
Prohibition: This writ is issued by a higher court(High Court or Supreme Court) when a
lower court has considered a case going beyond its jurisdiction.
Quo Warranto: If the court finds that a person is holding office but is not entitled to hold that
office, it issues the writ of quo warranto and restricts that person from acting as an office
holder.
Certiorari: Under this writ, the court orders a lower court or another authority to transfer
amatter pending before it to the higher authority or court.
Reference : Page 41(Indian Constitution at Work Class XI)
26. Solution: d)
Siricilla Tragedy
As a part of liberalisation, privatisation and globalisation, the government started to reform the
power sector. The most important impact of these reforms has been a steep hike in power tariff.
Since the powerlooms, on which a large number of industrial workers in cottage and small-scale
sector depend, are driven by power energy, the impact of high tariff on them has been very
serious. Further, while the power sector reforms have led to hike in tariffs, the power producers
have failed in providing quality power to the powerloom industry. Since the wages of the
powerloom workers are linked to the production of cloth, power-cut means cut in wages of
weavers who were already suffering from hike in tariff. This led to a crisis in the livelihood of the
weavers and fifty powerloom workers committed suicide in a small town called Siricilla in
Andhra Pradesh.

27. Solution: d)
A socialist society answers the three questions in a totally different manner. In a socialist society
the government decides what goods are to be produced in accordance with the needs of society.
It is assumed that the government knows what is good for the people of the country and so the
desires of individual consumers are not given much importance. The government decides how
goods are to be produced and how they should be distributed. In principle, distribution under
socialism is supposed to be based on what people need and not on what they can afford to
purchase. Unlike under capitalism, for example, a socialist nation provides free health care to the
citizens who need it. Strictly, a socialist society has no private property since everything is owned
http://insightsonindia.com

INSIGHTS

Page 10

Facebook Group: Indian Administrative Service (Raz Kr)

INSIGHTS MOCK PRELIMS TEST-4

SOLUTIONS

by the state. With the collapse of the Soviet system in the last decades of the twentieth century,
socialist economies in the former Soviet Union and the socialist states in Eastern Europe ceased to
exist.

28. Solution: d)
The Intertropical Convergence Zone (ITCZ), known by sailors as the doldrums, is the area
encircling the earth near the equator where the northeast and southeast trade winds come
together.
As the summer sets in and the sun shifts northwards, the wind circulation over the subcontinent
undergoes a complete reversal at both, the lower as well as the upper levels. By the middle of
July, the low pressure belt nearer the surface [termed as Inter Tropical Convergence Zone
(ITCZ)] shifts northwards, roughly parallel to the Himalayas between 20 N and 25 N. By this
time, the westerly jet stream withdraws from the Indian region. In fact, meteorologists have
found an interrelationship between the northward shift of the equatorial trough (ITCZ) and the
withdrawal of the westerly jet stream from over the North Indian Plain. It is generally believed
that there is a cause and effect relationship between the two.
The ITCZ being a zone of low pressure, attracts inflow of winds from different directions. The
maritime tropical airmass (mT) from the southern hemisphere, after crossing the equator, rushes
to the low pressure area in the general southwesterly direction. It is this moist air current which is
popularly known as the southwest monsoon.
Inter Tropical Convergence Zone (ITCZ)
The Inter Tropical Convergence Zone (ITCZ) is a low pressure zone located at the equator where
trade winds converge, and so, it is a zone where air tends to ascend. In July, the ITCZ is located
around 20N-25N latitudes (over the Gangetic plain), sometimes called the monsoon trough. This
monsoon trough encourages the development of thermal low over north and northwest India. Due to
the shift of ITCZ, the trade winds of the southern hemisphere cross the equator between 40 and 60E
longitudes and start blowing from southwest to northeast due to the Coriolis force. It becomes
southwest monsoon. In winter, the ITCZ moves southward, and so the reversal of winds from
northeast to south and southwest, takes place. They are called northeast monsoons.

29. Solution a)
Al-Biruni who came from Uzbekistan in the eleventh century, called Kitab-ul-Hind in Arabic. Ibn
Battuta who came from Morocco in the fourteenth century, wrote Rihla in Arabic.

http://insightsonindia.com

INSIGHTS

Page 11

Facebook Group: Indian Administrative Service (Raz Kr)

INSIGHTS MOCK PRELIMS TEST-4

SOLUTIONS

Reference: Page No. 116,118(THEMES IN INDIAN HISTORY PART II)


30. Solution a)
Principles of Liberty, Equality and Fraternity were borrowed from French Constitution.
Fundamental Rights, Power of Judicial Review and independence of Judiciary, written
Constitution from U.S.
Quasi-federal form of government and residuary powers from Canada.
Reference : Page 22(Indian Constitution at Work Class XI)
31. Solution: c)
The Greenhouse Gas Protocol (GHG Protocol) is the most widely used international accounting
tool for government and business leaders to understand, quantify, and manage greenhouse gas
emissions. The GHG Protocol, a decade-long partnership between the World Resources
Institute and the World Business Council for Sustainable Development, is working with
businesses, governments, and environmental groups around the world to build a new generation
of credible and effective programs for tackling climate change.
It provides the accounting framework for nearly every GHG standard and program in the world from the International Standards Organization to The Climate Registry - as well as hundreds of
GHG inventories prepared by individual companies.

32. Solution b)
Initially, only men were allowed into the sangha, but later women also came to be admitted.
According to Buddhist texts, this was made possible through the mediation of Ananda, one of the
Buddhas dearest disciples, who persuaded him to allow women into the sangha. The Buddhas
foster mother, Mahapajapati Gotami was the first woman to be ordained as a bhikkhuni.
Reference: Page No.92, 95(THEMES IN INDIAN HISTORY PART I)
33. Solution: a)
An isthmus is a narrow strip of land connecting two larger land areas, usually with water on
either side. A tombolo is an isthmus where the strip of land consists of a spit or bar.

http://insightsonindia.com

INSIGHTS

Page 12

Facebook Group: Indian Administrative Service (Raz Kr)

INSIGHTS MOCK PRELIMS TEST-4

SOLUTIONS

A bay is a large body of water connected to an ocean or sea formed by an inlet of land due to the
surrounding land blocking some waves and often reducing winds.[1] Bays also exist in in-land
environments as an inlet to any larger body of water, such as a lake or pond, or the estuary of a
river, such as those found in and around the Great Lakes of North America, or in the estuary of
the Parramatta River in Australia. A large bay may be called a gulf, a sea, a sound, or a bight.
A cove is a circular or oval coastal inlet with a narrow entrance; some coves may be referred to as
bays. A fjord is a particularly steep bay shaped by glacial activity.
A strait is a naturally formed, narrow, typically navigable waterway that connects two larger,
navigable bodies of water. It most commonly refers to a channel of water that lies between
two land masses, but it may also refer to a navigable channel through a body of water that is
otherwise not navigable, for example because it is too shallow, or because it contains an
unnavigable reef or archipelago.
A gulf in geography is a large bay that is an arm of an ocean or sea.

34. Solution: c)

The Questions should have read Soon after independence, Indian had.
(regret the error)
Market and State in Indian Industrial Development:
The big question facing the policy makers was what should be the role of the government and
the private sector in industrial development? At the time of independence, Indian industrialists
did not have the capital to undertake investment in industrial ventures required for the
development of our economy; nor was the market big enough to encourage industrialists to
undertake major projects even if they had the capital to do so. It is principally for these reasons
that the state had to play an extensive role in promoting the industrial sector. In addition, the
decision to develop the Indian economy on socialist lines led to the policy of the state controlling
the commanding heights of the economy, as the Second Five Year plan put it. This meant that
the state would have complete control of those industries that were vital for the economy. The
policies of the private sector would have to be complimentary to those of the public sector, with
the public sector leading the way.

35. Solution d)

http://insightsonindia.com

INSIGHTS

Page 13

Facebook Group: Indian Administrative Service (Raz Kr)

INSIGHTS MOCK PRELIMS TEST-4

SOLUTIONS

Our Constitution believes that diversity is our strength. Therefore, one of the fundamental
rights is the right of the minorities to maintain their culture. This minority status is not
dependent only upon religion. Linguistic and cultural minorities are also included in this
provision.
All minorities, religious or linguistic, can set up their own educational institutions. By doing
so, they can preserve and develop their own culture.
Educational institutions set up by a minority are open all and not only to those children who
belong to that minority.
The first case : Rex - versus - Dhyan Singh (AIR 1952 Allahabad, 53) arose under the
provisions of U.P. Arms Act in 1948, and was decided by the Allahabad High Court in 1952.
Dhyan Singh had been convicted under the Arms Act for wearing and carrying more than one
kirpan without licence. When the Appeal came to the High Court, the Constitution of India
had come into operation, giving right to the Sikhs to wear and carry kirpans under Article 25.
The question to be decided before the High Court was : Whether a Sikh can wear and carry
more than one kirpan as the plural word kirpans has been used in Explanation I of the Article
25. After referring to the religious literature of the Sikhs, the High Court had held that a Sikh
is entitled to wear and carry only one kirpan, as Guru Gobind Singh had ordained the Sikhs to
wear always five Ks, i.e., kesh, kangha, kirpan, kachh and kara, which indicates only one of
these signs.
Right to minorities includes both linguistic and cultural minorities.
http://sikhinstitute.org/sikh_p_l/ch_5.htm
Reference : Page 39, 40 (Indian Constitution at Work Class XI)
36. Solution a)
According to Buddhist philosophy, the world is transient (anicca) and constantly changing; it is
also soulless ( anatta) as there is nothing permanent or eternal in it. Within this transient world,
sorrow (dukkha) is intrinsic to human existence.
Reference: Page No.90, 91(THEMES IN INDIAN HISTORY PART I)
37. Solution: d)

http://insightsonindia.com

INSIGHTS

Page 14

Facebook Group: Indian Administrative Service (Raz Kr)

INSIGHTS MOCK PRELIMS TEST-4

SOLUTIONS

The industrial policy that we adopted was closely related to the trade policy. In the first seven
plans, trade was characterized by what is commonly called an inward looking trade strategy.
Technically, this strategy is called import substitution. This policy aimed at replacing or
substituting imports with domestic production. For example, instead of importing vehicles made
in a foreign country, industries would be encouraged to produce them in India itself. In this policy
the government protected the domestic industries from foreign competition. Protection from
imports took two forms: tariffs and quotas.
Tariffs are a tax on imported goods; they make imported goods more expensive and discourage
their use. Quotas specify the quantity of goods which can be imported. The effect of tariffs and
quotas is that they restrict imports and, therefore, protect the domestic firms from foreign
competition. The policy of protection is based on the notion that industries of developing
countries are not in a position to compete against the goods produced by more developed
economies. It is assumed that if the domestic industries are protected they will learn to compete
in the course of time. Our planners also feared the possibility of foreign exchange being spent on
import of luxury goods if no restrictions were placed on imports. Nor was any serious thought
given to promote exports until the mid-1980s.

38. Solution: d)
A capital good is a durable good (one that does not quickly wear out) that is used in the
production of goods or services. Capital goods are one of the three types of producer goods, the
other two being land and labor, which are also known collectively as primary factors of
production. This classification originated during the classical economic period and has remained
the dominant method for classification.
The sector of the economy that includes capital-goods-producing businesses such as Boeing,
Caterpillar and Lockheed Martin. Aerospace, defense, construction and machinery businesses
make up most of the capital goods sector.
There was hardly any capital goods industry to help promote further industrialization in India.
Capital goods industry means industries which can produce machine tools which are, in turn,
used for producing articles for current consumption.
Factors of Production
In economics, factors of production are the inputs to the production process. Finished goods are
the output. Input determines the quantity of output i.e. output depends upon input. Input is the
starting point and output is the end point of production process and such input-output
relationship is called a production function. There are three basic factors of
production: land, labor, capital.
http://insightsonindia.com

INSIGHTS

Page 15

Facebook Group: Indian Administrative Service (Raz Kr)

INSIGHTS MOCK PRELIMS TEST-4

SOLUTIONS

39. Solution: b)
Although contributing to many other physical and chemical reactions, the major atmospheric
constituents, nitrogen (N2), oxygen (O2), and argon (Ar), are not greenhouse gases. This is
because molecules containing two atoms of the same element such as N2 and O2
and monatomic molecules such as argon (Ar) have no net change in their dipole moment when
they vibrate and hence are almost totally unaffected by infrared radiation. Although molecules
containing two atoms of different elements such as carbon monoxide (CO) or hydrogen
chloride (HCl) absorb IR, these molecules are short-lived in the atmosphere owing to their
reactivity and solubility. Because they do not contribute significantly to the greenhouse effect,
they are usually omitted when discussing greenhouse gases.
In 2013, this liquid was shown by researchers at the University of Toronto to be a greenhouse gas,
with warming properties more than 7,000 times that of carbon dioxide over a 100 year
period, and that, as such, it is the most potent greenhouse gas ever discovered. Its concentration in
the atmosphere is approximately 0.18 parts per trillion. The researchers also reported that the gas
can persist in the atmosphere for up to 500 years.
When ranked by their direct contribution to the greenhouse effect, the most important are:
Compound

Formula Contribution
(%)

Water vapor and clouds H


2O

36 72%

Carbon dioxide

CO2

9 26%

Methane

CH
4

4 9%

Ozone

O
3

7%

40. Solution: d)

http://insightsonindia.com

INSIGHTS

Page 16

Facebook Group: Indian Administrative Service (Raz Kr)

INSIGHTS MOCK PRELIMS TEST-4

SOLUTIONS

Groundwater is a critical resource in India, accounting for over 65% of irrigation water and 85%
of drinking water supplies. However, on current trends it is estimated that 60% of groundwater
sources will be in a critical state of degradation within the next twenty years.
Indias declining groundwater resources are the product of a number of drivers. Utilization of
groundwater facilitates irrigated agriculture in areas far from rivers; in fact, this was key to the
agricultural green revolution that occurred from the mid 1960s. In places where surface water is
available but unsafe for drinking or farmingmore than 70% of Indias surface water resources
are polluted by human waste or toxic chemicalsgroundwater has often been seen as a safe
alternative. Urban water supply infrastructure is often poor and unreliable: well drilling is
typically the most economical means of obtaining household water.
In rural areas, electricity subsidies allowing farmers to pump groundwater cheaply have become
entrenched in the political landscape. They are likely to become even more so as energy
requirements increase to extract water from greater depths. Low cost encourages excess water
withdrawal, an inefficient usage pattern commonly exacerbated by ineffective application
methods and the wastage of agricultural produce between farm and market. In order to feed a
growing and wealthier population, it is projected that agricultural water demand in the India of
2030 would need double to 1,200 billion m3 if these inefficient practices continued. The
problems are only going to get worse unless urgent changes occur.
Moreover, the groundwater belongs to the landowner as most of the state laws suggest. However,
the groundwater is actually a community resource. For, the depletion of groundwater beneath
ones own land is likely to deplete the levels in the areas around it too.
One of the critical challenges during the 12th Five-Year Plan is to evolve strategies to manage
ground water resources in a rational and sustainable manner.
In a major shift in the water sector, the 12th Plan clearly calls for more holistic aquifer
management strategies including separation of electricity feeders for agriculture and domestic use
to break the cycle of free use of electricity and excess use of groundwater.
It also focuses on rationalising the pricing of water for various categories of users, including
domestic users.
Besides, the new National water policy seeks to make the groundwater a community resource by
urging the states to amend their laws through financial incentives.
41. Solution: c)
All definitions are correct.

http://insightsonindia.com

INSIGHTS

Page 17

Facebook Group: Indian Administrative Service (Raz Kr)

INSIGHTS MOCK PRELIMS TEST-4

SOLUTIONS

42. Solution c)
The winning candidate need not secure a majority of the votes. This method is called the First
Past the Post (FPTP) system.
In the electoral race, the candidate who is ahead of others, who crosses the winning post first
of all, is the winner. This method is also called the Plurality System.
The FPTP system offers voters a choice not simply between parties but specific candidates..
In constituency based system like the FPTP, the voters know who their own representatives
and can hold him or her accountable.
The FPTP system is adopted in the lower house of the parliament and state legislative
assemblies; proportional representation by means of single transferable vote is carried out in
Rajya Sabha
Reference : Page 57(Indian Constitution at Work Class XI)
43. Solution a)
Representations on seals and terracotta sculpture indicate that the bull was known, and
archaeologists extrapolate from this that oxen were used for ploughing. Archaeologists have also
found evidence of a ploughed field at Kalibangan (Rajasthan), associated with Early Harappan
levels. The field had two sets of furrows at right angles to each other, suggesting that two
different crops were grown together.
Reference: Page No. 3 (THEMES IN INDIAN HISTORY PART I)

44. Solution: b)
Economic growth doesnt always bring development as is evident from various countries,
especially emerging economies including India.

45. Solution b)
While some such as clay were locally available, many such as stone, timber and metal had to be
procured from outside the alluvial plain. Recent archaeological finds suggest that copper was also
probably brought from Oman, on the southeastern tip of the Arabian peninsula. Steel was not
http://insightsonindia.com

INSIGHTS

Page 18

Facebook Group: Indian Administrative Service (Raz Kr)

INSIGHTS MOCK PRELIMS TEST-4

SOLUTIONS

known to the Harappans. Exchanges were regulated by a precise system of weights, usually made
of a stone called chert and generally cubical, with no markings. The lower denominations of
weights were binary (1, 2, 4, 8, 16, 32, etc. up to 12,800), while the higher denominations followed
the decimal system.
Reference: Page No. 12,13(THEMES IN INDIAN HISTORY PART I)
46. Solution d)
The Election Commission supervises the preparation of up-to-date voters list. It makes every
effort to ensure that the voters list is free of errors like nonexistence of names of registered
voters or existence of names of those non-eligible or non-existent.
It also determines the timing of elections and prepares the election schedule. The election
schedule includes the notification of elections, date from which nominations can be filed, last
date for filing nominations, last date of scrutiny, last date of withdrawal, date of polling and
date of counting and declaration of results.
During this entire process, the Election Commission has the power to take decisions to ensure
a free and fair poll. It can postpone or cancel the election in the entire country or a specific
State or constituency on the grounds that the atmosphere is vitiated and therefore, a free and
fair election may not be possible.
The Commission also implements a model code of conduct for parties and candidates. It can
order a re-poll in a specific constituency. It can also order a recount of votes when it feels that
the counting process has not been fully fair and just.
The Election Commission accords recognition to political parties and allots symbols to each of
them.
The Election Commission has very limited staff of its own. It conducts the elections with the
help of the administrative machinery. However, once the election process has begun, the
commission has control over the administration as far as election related work is concerned.
During the election process, the administrative officers of the State and central governments
are assigned election related duty and in this respect, the Election Commission has full control
over them. The EC can transfer the officers, or stop their transfers; it can take action against
them for failing to act in a non-partisan manner.
Reference : Page 67, 68, 69(Indian Constitution at Work Class XI)

http://insightsonindia.com

INSIGHTS

Page 19

Facebook Group: Indian Administrative Service (Raz Kr)

INSIGHTS MOCK PRELIMS TEST-4

SOLUTIONS

47. Solution: b)
Srinagar, capital city of the state of Jammu and Kashmir is located on the banks of Jhelum river.
A meander, in general, is a bend in a sinuous watercourse or river. A meander is formed when the
moving water in a stream erodes the outer banks and widens its valley and the inner part of the
river has less energy and deposits what it is carrying. A stream of any volume may assume a
meandering course, alternately eroding sediments from the outside of a bend and depositing them
on the inside. The result is a snaking pattern as the stream meanders back and forth across its
down-valley axis. When a meander gets cut off from the main stream, an oxbow lake is formed.

48. Solution b)
Baba Guru Nanak did not wish to establish a new religion, but after his death his followers
consolidated their own practices and distinguished themselves from both Hindus and Muslims.
Also he did not give the name Sikhism. He rejected sacrifices, ritual baths, image worship,
austerities and the scriptures of both Hindus and Muslims. It was Guru Gobind Singh who
defined its five symbols: uncut hair, a dagger, a pair of shorts, a comb and a steel bangle.
Reference: Page No. 163,164(THEMES IN INDIAN HISTORY PART II)
49. Solution: c)
The following was the outcome of the recent Doha conference
An eight year extension of the Kyoto Protocol until 2020 limited in scope to only
15% of the global carbon dioxide emissions due to the lack of participation of
Canada, Japan, Russia, Belarus, Ukraine, New Zealand and the United States and
due to the fact that developing countries like China (the world's largest emitter),
India and Brazil are not subject to any emissions reductions under the Kyoto
Protocol.
Language on loss and damage, formalized for the first time in the conference
documents.
The conference made little progress towards the funding of the Green Climate
Fund.
Focus on increasing the effectiveness of conventional means to tackle climate
change rather than going for risky and unexplored options like geo-engineering
etc.
http://insightsonindia.com

INSIGHTS

Page 20

Facebook Group: Indian Administrative Service (Raz Kr)

INSIGHTS MOCK PRELIMS TEST-4

SOLUTIONS

50. Solution b)
The Vice President is elected for five years. His election method is similar to that of the
President, the only difference is that members of State legislatures are not part of the electoral
college.
The Vice President may be removed from his office by a resolution of the Rajya Sabha passed
by a majority and agreed to by the Lok Sabha. The Vice President acts as the ex-officio
Chairman of the Rajya Sabha and takes overthe office of the President when there is a
vacancy by reasons of death, resignation, removal by impeachment or otherwise. The Vice
President acts as the President only until a new President is elected.
All disputes arising in connection with the election of the Vice President are petitioned to the
Supreme Court of India, which inquires into the matter. The petition is heard by a fivemember bench of the Supreme Court, which decides on the matter. The decision of the
Supreme Court is final.
Reference : Page 88(Indian Constitution at Work Class XI)
51. Solution: a)
As compared to the other sections of the Himalayas, these along with the Arunachal Himalayas
are conspicuous by the absence of the Shiwalik formations. In place of the Shiwaliks here, the
duar formations are important, which have also been used for the development of tea gardens.
Sikkim and Darjiling Himalayas are also known for their scenic beauty and rich flora and fauna,
particularly various types of orchids.
The Dooars or Duars are the floodplains and foothills of the eastern Himalayas in North-East
India around Bhutan. The region forms the gateway to Bhutan from India. There are 18 passages
or gateways through which the Bhutanese people can communicate with the people living in the
plains. This region is divided by the Sankosh river into the Eastern and the Western Dooars,
consisting of an area of 8,800 km (3,400 sq mi). The Western Dooars is known as the Bengal
Dooars and the Eastern Dooars as the Assam Dooars. Dooars is synonymous with the
term Terai used in Nepal and northern India and form the only nitrate rich soil in India.

52. Solution: c)

http://insightsonindia.com

INSIGHTS

Page 21

Facebook Group: Indian Administrative Service (Raz Kr)

INSIGHTS MOCK PRELIMS TEST-4

SOLUTIONS

Both are incorrect. Most kayals or backwaters are predominantly brackish i.e. saline. In certain
areas, such as the Vembanad Kayal, where a barrage has been built near Kumarakom, salt water
from the sea is prevented from entering the deep inside, keeping the fresh water intact. Such
fresh water is extensively used for irrigation purposes.

53. Solution a)
Reference: Page No. 161(THEMES IN INDIAN HISTORY PART II)

54. Solution: d)
When corals are stressed by changes in conditions such as temperature, light, or nutrients, they
expel the symbiotic algae living in their tissues, causing them to turn completely white
Warmer water temperatures can result in coral bleaching. When water is too warm, corals will
expel the algae (zooxanthellae) living in their tissues causing the coral to turn completely white.
This is called coral bleaching. When a coral bleaches, it is not dead. Corals can survive a bleaching
event, but they are under more stress and are subject to mortality.

When exposed to natural constraints (such as an increase/decrease in the temperature, high


tides) or anthropic factors (such as pollution or the sedimentation of terrrigenous substances),
coral can respond in various ways, depending on the species involved and on the extent of the
disturbance. When the temperature of the sea is greater than the tolerance threshold for some
time, the zooxanthellae carry out too much photosynthesis, and this leads the polyps to actively
reject these algae.
When the atolls are hit by "El Nio " or "La Nia", the climatic conditions are disturbed for a
short period and this leads to a change in the direction of the ocean currents, and in the case of "
El Nio", to an abnormally large increase in the temperature in the region of the coral reefs.
The global heating process which is affecting both the atmosphere and the oceans of the world
and might involve increases of as much as 2C over the next 50 years constitutes a serious threat
to the survival of the coral, since this would mean that the tolerance limits would be
overstepped.
The effects of the bleaching process
http://insightsonindia.com

INSIGHTS

Page 22

Facebook Group: Indian Administrative Service (Raz Kr)

INSIGHTS MOCK PRELIMS TEST-4

SOLUTIONS

The loss of symbiotic algae is causing the primary functions of the coral colonies (their growth,
their reproductive capacity, etc.) to regress, if not to stop altogether.
If this stress persists, the coral colonies will be at least partly if not completely destroyed and
invaded by algal meadows.
The disappearance of the coral would have the following consequences:
The fish population dependent on the corals for food would decrease thus hurting the marine
food chain and the biodiversity. This would affect fishermens catch and impact the local
economy.
The coastline would become eroded since it would no longer be protected by the reefs from sea
swell.

55. Solution: b)
Some economists point out that subsidies do not allow prices to indicate the supply of a good.
When electricity and water are provided at a subsidized rate or free, they will be used wastefully
without any concern for their scarcity. Farmers will cultivate water intensive crops if water is
supplied free, although the water resources in that region may be scarce and such crops will
further deplete the already scarce resources. If water is priced to reflect scarcity, farmers will
cultivate crops suitable to the region. Fertiliser and pesticide subsidies result in overuse of
resources which can be harmful to the environment. Subsidies provide an incentive for wasteful
use of resources.

56. Solution: d)
There are two major island groups in India one in the Bay of Bengal and the other in the
Arabian Sea. The Bay of Bengal island groups consist of about 572 islands/islets. These are situated
roughly between 6N-14N and 92E -94E. The two principal groups of islets include the
Ritchies archipelago and the Labrynth island.
The entire group of island is divided into two broad categories the Andaman in the north and
the Nicobar in the south. They are separated by a water body which is called the Ten degree
channel. It is believed that these islands are an elevated portion of submarine mountains.
However, some smaller islands are volcanic in origin. Barren island, the only active volcano in
India is also situated in the Nicobar islands.

http://insightsonindia.com

INSIGHTS

Page 23

Facebook Group: Indian Administrative Service (Raz Kr)

INSIGHTS MOCK PRELIMS TEST-4

SOLUTIONS

57. Solution b)
The government has established in 2000 an institution, the National Human Rights
Commission. The National Human Rights Commission (NHRC) is composed of a former
chief justice of the Supreme Court of India, a former judge of the Supreme Court, a former
chief justice of a High Court and two other members who have knowledge and practical
experience in matters relating to human rights.
The commissions functions include inquiry at its own initiative or on a petition presented to
it by a victim into complaint of violation of human rights; visit to jails to study the condition
of the inmates; undertaking and promoting research in the field of human rights etc.
The commission receives complaints in thousands every year related to custodial death,
custodial rape, disappearances, police excesses, failure in taking action, Indignity to women
etc.
The commission does not have the power of prosecution. It can merely make
recommendations to the government or recommend to the courts to initiate proceedings
based on the inquiry that it conducts.
Reference : Page 42 (Indian Constitution at Work Class XI)
58. Solution a)
Of the groups of sufis who migrated to India in the late twelfth century, the Chishtis were the
most influential. This was because they adapted successfully to the local environment and
adopted several features of Indian devotional traditions. Chistis used the local languages in their
compositions and communication. The sufis accepted unsolicited grants and donations from the
political elites.
Reference: Page No. 154,158,159(THEMES IN INDIAN HISTORY PART II)

59. Solution: c)
An antecedent stream is a stream that maintains its original course and pattern despite the
changes in underlying rock topography. A stream with a dendritic drainage pattern, for example,
can be subject to slow tectonic uplift. However, as the uplift occurs, the stream erodes through
the rising ridge to form a steep-walled gorge. The stream thus keeps its dendritic pattern even
though it flows over a landscape that will normally produce a trellis drainage pattern.
http://insightsonindia.com

INSIGHTS

Page 24

Facebook Group: Indian Administrative Service (Raz Kr)

INSIGHTS MOCK PRELIMS TEST-4

SOLUTIONS

The Himalayan Rivers are great examples of antecedent origin. These rivers originated well
before the Himalayan region was uplifted. The rivers Indus, Ganga and Sutlej originated on
Tibetan side and now traverse the existing mountain ranges, cutting deep gorges.

The Subansiri, tributary of the Brahmaputra and the Kosi, tributary of the Ganga are also
antecedent rivers.

60. Solution d)
Who decides which constituency is to be reserved? On what basis is this decision taken? This
decision is taken by an independent body called the Delimitation Commission. The Delimitation
Commission is appointed by the President of India and works in collaboration with the Election
Commission of India. It is appointed for the purpose of drawing up the boundaries of
constituencies all over the country. A quota of constituencies to be reserved in each State is fixed
depending on the proportion of SC or ST in that State. After drawing the boundaries, the
Delimitation Commission looks at the composition of population in each constituency. Those
constituencies that have the highest proportion of Scheduled Tribe population are reserved for
ST.
In the case of Scheduled Castes, the Delimitation Commission looks at two things. It picks
constituencies that have higher proportion of Scheduled Caste population. But it also spreads
these constituencies in different regions of the State. This is done because the Scheduled Caste
population is generally spread evenly throughout the country.
These reserved constituencies can be rotated each time the Delimitation exercise is undertaken.
The Delimitation Commission in India is a high power body whose orders have the force of law
and cannot be called in question before any court.
Reference : Page 59 and http://eci.nic.in(Indian Constitution at Work Class XI)

61. Solution: c)
6. Save Our Species (SOS) is a global coalition initiated by the three founding partners IUCN,
GEF and World Bank to build the biggest species conservation fund, supporting on-the-ground
field conservation projects all over the world. SOS will combine resources and funding
experience from the World Bank and GEF (Global Environment Facility), the authoritative
science of IUCN and the resources and ingenuity of the private sector to create a mechanism that
http://insightsonindia.com

INSIGHTS

Page 25

Facebook Group: Indian Administrative Service (Raz Kr)

INSIGHTS MOCK PRELIMS TEST-4

SOLUTIONS

ensures sufficient funding goes to species conservation projects where and when it will have the
most impact.
Apart from Red-crowned roof turtle, Four-toed terrapin and Gangetic Dolphins; white bellied
herons are also included in the initiative.

62. Solution a)
The Election Commission of India can either be a single member or a multi-member body.
Till 1989, the Election Commission was single member. Just before the 1989 general
elections, two Election Commissioners were appointed, making the body multi-member.
Soon after the elections, the Commission reverted to its single member status. In 1993, two
Election Commissioners were once again appointed and the Commission became multimember and has remained multi-member since then.
The Chief Election Commissioner (CEC) presides over the Election Commission, but does
not have more powers than the other Election Commissioners. The CEC and the two
Election Commissioners have equal powers to take all decisions relating to elections as a
collective body. They are appointed by the President of India on the advice of the Council of
Ministers.
The CEC is removed by the President if both Houses of Parliament make such are
commendation with a special majority. This is done to ensure that a ruling party cannot
remove a CEC who refuses to favour it in elections. The Election Commissioners can be
removed by the President of India.
Reference : Page 67, 68(Indian Constitution at Work Class XI)

63. Solution: a)
The discharge is the volume of water flowing in a river measured over time. It is measured either
in cusecs (cubic feet per second) or cumecs (cubic metres per second).
The Ganga has its minimum flow during the January-June period. The maximum flow is attained
either in August or in September. After September, there is a steady fall in the flow. The river,
thus, has a monsoon regime during the rainy season.
There are striking differences in the river regimes in the eastern and the western parts of the
Ganga Basin. The Ganga maintains a sizeable flow in the early part of summer due to snow melt

http://insightsonindia.com

INSIGHTS

Page 26

Facebook Group: Indian Administrative Service (Raz Kr)

INSIGHTS MOCK PRELIMS TEST-4

SOLUTIONS

before the monsoon rains begin. The mean maximum discharge of the Ganga at Farakka is about
55,000 cusecs while the mean minimum is only 1,300 cusecs

64. Solution c)
The word silsila literally means a chain, signifying a continuous link between master and disciple,
stretching as an unbroken spiritual genealogy to the Prophet Muhammad. It was through this
channel that spiritual power and blessings were transmitted to devotees.
Reference: Page No. 153 (THEMES IN INDIAN HISTORY PART II)

65. Solution: d)
Permafrost is permanently frozen soil, and occurs mostly in high latitudes. Permafrost comprises
24% of the land in the Northern Hemisphere, and stores massive amounts of carbon. As a result
of climate change, permafrost is at risk of melting, releasing the stored carbon in the form of
carbon dioxide and methane, which are powerful heat-trapping gases. In addition, permafrost is
structurally important, and its melting has been known to cause erosion, disappearance of lakes,
landslides, and ground subsidence. It will also cause changes in plant species composition at high
latitudes.

66. Solution d)
The founding and settling of hill stations was initially connected with the needs of the British
army. Hill stations became strategic places for billeting troops, guarding frontiers and launching
campaigns against enemy rulers. Simla became the official residence of the commander-in-chief of
the Indian army. Hill stations were important for the colonial economy. With the setting up of
tea and coffee plantations in the adjoining areas, an influx of immigrant labour from the plains
began. This meant that hill stations no longer remained exclusive racial enclaves for Europeans in
India.
Reference: Page No. 327,328(THEMES IN INDIAN HISTORY PART III)

http://insightsonindia.com

INSIGHTS

Page 27

Facebook Group: Indian Administrative Service (Raz Kr)

INSIGHTS MOCK PRELIMS TEST-4

SOLUTIONS

67. Solution: c)
4. Marine protected areas, like any protected area, are regions in which human activity has been
placed under some restrictions in the interest of protecting the natural environment, its
surrounding waters and the occupant ecosystems, and any cultural or historical resources that
may require preservation or management. Marine protected areas' boundaries will include some
area of ocean, even if it is only a small fraction of the total area of the territory.
Natural or historic marine resources are protected by local, state, territorial, native, regional, or
national authorities and may differ substantially from nation to nation. This variation includes
different limitations on development, fishing practices, fishing seasons and catch limits, moorings,
bans on removing or disrupting marine life of any kind.
Some people confuse marine reserves, where extraction of any resources is prohibited (no-take),
as the only type of MPA. MPAs may include marine reserves, as well as other zones in which
partial protection is afforded (seasonal closures, catch limits, etc.). Many MPAs are multiple-use
areas, where a variety of uses are allowed. For example, there are many different kinds of MPAs
in U.S. waters including national parks, wildlife refuges, monuments and marine sanctuaries,
fisheries closures, critical habitat, habitat areas of particular concern, state parks, conservation
areas, estuarine reserves and preserves, and numerous others. While a few sites exist as notake marine reserves, the vast majority of MPAs, both in terms of numbers and area, are open for
fishing, diving, boating, and other recreational and commercial uses.
In the case of India, the first MPA was designated in 1967 for the protection of wetlands and of
the birds migrating there, even before a specific legal framework for protected areas (PAs) was
put in place. According to the Government of India there are 31 MPAs in India, designated
mainly in 1980s and early 1990s. The main MPAs in mainland India are the Gulf of Mannar
national park and biosphere reserve, Gahirmatha wildlife sanctuary, Gulf of Kutch national park
and wildlife sanctuary, Malvan (marine) wildlife sanctuary, and Sunderbans national park and
biosphere reserve. There also are several PAs in Andaman and Nicobar islands.

68. Solution: d)
Equity in agriculture called for land reforms which primarily refer to change in the ownership of
landholdings. Just a year after independence, steps were taken to abolish intermediaries and to
make the tillers the owners of land. The idea behind this move was that ownership of land would
give incentives to the tillers to invest in making improvements provided sufficient capital was
made available to them.
Land ceiling was another policy to promote equity in the agricultural sector. This means fixing
the maximum size of land which could be owned by an individual. The purpose of land ceiling
was to reduce the concentration of land ownership in a few hands. The abolition of
intermediaries meant that some 200 lakh tenants came into direct contact with the government
http://insightsonindia.com

INSIGHTS

Page 28

Facebook Group: Indian Administrative Service (Raz Kr)

INSIGHTS MOCK PRELIMS TEST-4

SOLUTIONS

they were thus freed from being exploited by the zamindars. The ownership conferred on
tenants gave them the incentive to increase output and this contributed to growth in agriculture.
However, the goal of equity was not fully served by abolition of intermediaries. In some areas the
former zamindars continued to own large areas of land by making use of some loopholes in the
legislation.
The Debate Over Subsidies:
The economic justification of subsidies in agriculture is, at present, a hotly debated question. It is
generally agreed that it was necessary to use subsidies to provide an incentive for adoption of the
new HYV technology by farmers in general and small farmers in particular. Any new technology
will be looked upon as being risky by farmers. Subsidies were, therefore, needed to encourage
farmers to test the new technology.
Some economists believe that once the technology is found profitable and is widely adopted,
subsidies should be phased out since their purpose has been served. Further, subsidies are meant
to benefit the farmers but a substantial amount of fertilizer subsidy also benefits the fertilizer
industry; and among farmers, the subsidy largely benefits the farmers in the more prosperous
regions. Therefore, it is argued that there is no case for continuing with fertilizer subsidies; it does
not benefit the target group and it is a huge burden on the governments finances.
On the other hand, some believe that the government should continue with agricultural subsidies
because farming in India continues to be a risky business. Most farmers are very poor and they
will not be able to afford the required inputs without subsidies. Eliminating subsidies will
increase the inequality between rich and poor farmers and violate the goal of equity. These
experts argue that if subsidies are largely benefiting the fertilizer industry and big farmers, the
correct policy is not to abolish subsidies but to take steps to ensure that only the poor farmers
enjoy the benefits.

69. Solution d)
Banning of loudspeaker after 10 pm is in the interest of the larger public good, else this would
lead to sound pollution (loud speakers also disrupt peoples peace and concentration). Hence this
ban by the district court does not violate fundamental right.
All linguistic minorities can set up their own educational institutions to preserve and develop
their own culture. Thus the Karnataka Society has every right to run Kannada medium schools
outside Karnataka.
A director makes a documentary film that criticises the policies of the government. In this
situation the freedom of expression is being used.
http://insightsonindia.com

INSIGHTS

Page 29

Facebook Group: Indian Administrative Service (Raz Kr)

INSIGHTS MOCK PRELIMS TEST-4

SOLUTIONS

Reference : Chapter 2(Indian Constitution at Work Class XI)


70. Solution: a)
The western cyclonic disturbances which enter the Indian subcontinent from the west and the
northwest during the winter months, originate over the Mediterranean Sea and are brought into
India by the westerly jet stream. An increase in the prevailing night temperature generally
indicates an advance in the arrival of these cyclones disturbances.

71. Solution d)
"Objectives Resolution was introduced by Pandit Nehru in the Constituent Assembly, It
proclaimed India to be an Independent Sovereign Republic. Did not include the words Socialist
and Secular.
Reference: Page No. 411(THEMES IN INDIAN HISTORY PART III)
72. Solution: a)
The Damodar occupies the eastern margins of the Chotanagpur Plateau where it flows through a
rift valley and finally joins the Hugli. The Barakar is its main tributary.
It has a number of tributaries and subtributaries, such as Barakar, Konar, Bokaro,
Haharo, Jamunia, Ghari, Guaia, Khadia and Bhera.
The Barakar is the most important tributary of the Damodar. It originates near Padma in
Hazaribagh district and flows through Jharkhand before meeting the Damodar near Dishergarh in
West Bengal. The Damodar and the Barakar trifurcates the Chota Nagpur plateau.

73. Solution d)
The governments from time to time tried to give effect to some Directive Principles of State
Policy. They passed several zamindari abolition bills, nationalised banks, abolition of Privy Purses
enacted numerous factory laws, fixed minimum wages, cottage and small industries were
promoted and provisions for reservation for the uplift of the scheduled castes and scheduled
tribes were made.

http://insightsonindia.com

INSIGHTS

Page 30

Facebook Group: Indian Administrative Service (Raz Kr)

INSIGHTS MOCK PRELIMS TEST-4

SOLUTIONS

Directive Principles include the right to education, formation of Panchayati raj institutions all
over the country, partial right to work under employment guarantee programme and the mid-day
meal scheme etc.
Reference : Page 43,44(Indian Constitution at Work Class XI)
74. Solution: d)
Many distinguished thinkers contributed to the formulation of our five year plans. Among them,
the name of the statistician, Prasanta Chandra Mahalanobis, stands out. Planning, in the real sense
of the term, began with the Second Five Year Plan.
The Second Plan, a landmark contribution to development planning in general, laid down the
basic ideas regarding goals of Indian planning; this plan was based on the ideas of Mahalanobis. In
that sense, he can be regarded as the architect of Indian planning.
The FeldmanMahalanobis model is a Neo-Marxist model of economic development, created
independently by Soviet economist G. A. Feldman in 1928, and Indian statistician Prasanta
Chandra Mahalanobis in 1953. Mahalanobis became essentially the key economist
of India's Second Five Year Plan, becoming subject to much of India's most dramatic economic
debates.
The essence of the model is a shift in the pattern of industrial investment towards building up
a domestic consumption goods sector. Thus the strategy suggests in order to reach a high standard
in consumption, investment in building a capacity in the production of capital goods is firstly
needed. A high enough capacity in the capital goods sector in the long-run expands the capacity
in the production of consumer goods.

https://ib-economics.wikispaces.com/Harrod-Domar+Growth+Model

75. Solution: d)
The general orientation of these ranges is from northwest to the southeast direction in the
northwestern part of India. Himalayas in the Darjeeling and Sikkim regions lie in an east-west
direction, while in Arunachal Pradesh they are from southwest to the northwest direction. In
Nagaland, Manipur and Mizoram, they are in the north-south direction.

76. Solution: d)
http://insightsonindia.com

INSIGHTS

Page 31

Facebook Group: Indian Administrative Service (Raz Kr)

INSIGHTS MOCK PRELIMS TEST-4

SOLUTIONS

http://www.currentscience.ac.in/Volumes/100/06/0805.pdf

77. Solution d)
Fundamental rights primarily protect individuals from any arbitrary state actions, but some
rights are enforceable against individuals.
These provisions act as a check both on state action as well as the action of private
individuals. However, these rights are not absolute or uncontrolled and are subject to reasonable
restrictions as necessary for the protection of general welfare. They can also be selectively
curtailed.
Right to demonstration or picketing can be a Fundamental Right but not right to strike.
Every citizen has the right to assemble peacefully and without arms. There can be restrictions
on a peaceful march (For instance recent case on Anna Hazare) if the march breaches certain
laws or causes inconvenience to the general public.
Restrictions can be placed on Government officials expressing their grievances against the
government through media.
Reasonable restrictions can be imposed in the interest of public order, security of State,
decency or morality.
Reference : Chapter 2 (Indian Constitution at Work Class XI)

78. Solution d)
On 27 August 1947, B. Pocker Bahadur from Madras made a powerful plea for continuing separate
electorates. Not all Muslims supported the demand for separate electorates. Begum Aizaas Rasul,
for instance, felt that separate electorates were self destructive since they isolated the minorities
from the majority.
Reference: Page No. 416-419 (THEMES IN INDIAN HISTORY PART III)

79. Solution: c)

http://insightsonindia.com

INSIGHTS

Page 32

Facebook Group: Indian Administrative Service (Raz Kr)

INSIGHTS MOCK PRELIMS TEST-4

SOLUTIONS

Yamuna does not flow through a rift valley.


A rift valley is a linear-shaped lowland between several highlands or mountain ranges created by
the action of a geologic rift or fault. A Rift Valley is formed on a divergent plate boundary, a
crustal extension, a spreading apart of the surface, which is subsequently further deepened by the
forces of erosion. When the tensional forces were strong enough to cause the plate to split apart,
a center block dropped down between the two blocks at its flanks, forming a graben. This drop
of the center creates the nearly parallel steeply dipping walls of rift valley when it is new. This
feature is the beginning of the rift valley, but as the process continues, the valley widens, until it
becomes a large basin, that fills with sediment from the rift walls and the surrounding area. One
of the most well known examples of this process is the East African Rift.
Lake Tanganyika is an African Great Lake. It is estimated to be the second largest freshwater
lake in the world by volume, and the second deepest, in both cases, after only Lake
Baikal in Siberia; it is also the world's longest freshwater lake. The lake is divided among four
countries Tanzania, the Democratic Republic of the Congo, Burundi, and Zambia, with
Tanzania (46%) and the DRC (40%) possessing the majority of the lake. The water flows into
the Congo River system and ultimately into the Atlantic Ocean.
80. Solution: d)
The third geological division of India comprises the plains formed by the river Indus, the Ganga
and the Brahmaputra. Originally, it was a geo-synclinal depression which attained its maximum
development during the third phase of the Himalayan mountain formation approximately about
64 million years ago. Since then, it has been gradually filled by the sediments brought by the
Himalayan and Peninsular rivers. Average depth of alluvial deposits in these plains ranges from
1,000-2,000 m.
Geosyncline
It is a linear trough of subsidence of the Earths crust within which vast amounts of
sediment accumulate. The filling of a geosyncline with thousands or tens of thousands of feet of
sediment is accompanied in the late stages of deposition by folding, crumpling, and faulting of the
deposits. Intrusion of crystalline igneous rock and regional uplift along the axis of the trough
generally complete the history of a particular geosyncline, which is thus transformed to a belt of
folded mountains. The concept of the geosyncline was introduced by the American
geologist James Hall in 1859. Most modern geologists regard the concept as obsolete and largely
explain the development of linear troughs in terms of plate tectonics; the term geosyncline,
however, remains in use.

http://insightsonindia.com

INSIGHTS

Page 33

Facebook Group: Indian Administrative Service (Raz Kr)

INSIGHTS MOCK PRELIMS TEST-4

SOLUTIONS

81. Solution c)
Ordinarily, a person would be arrested after he or she has reportedly committed some offence.
However there are exceptions to this. Sometimes a person can be arrested simply out of an
apprehension that he or she is likely to engage in unlawful activity and imprisoned for some time
without following the above mentioned procedure. This is known as preventive detention. It
means that if the government feels that a person can be a threat to law and order or to the peace
and security of the nation, it can detain or arrest that person.
The government can impose restrictions in certain areas declaring the assembly of five or more
persons as unlawful. Freedom to assemble too is to be exercised peacefully and without arms.
Right to demonstration or picketing can be a Fundamental Right but not right to strike.
Reference : Page 35 (Indian Constitution at Work Class XI)
82. Solution: c)
Industrial Policy Resolution 1956 (IPR 1956):
In accordance with the goal of the state controlling the commanding heights of the economy, the
Industrial Policy Resolution of 1956 was adopted. This resolution formed the basis of the Second
Five Year Plan, the plan which tried to build the basis for a socialist pattern of society. This
resolution classified industries into three categories. The first category comprised industries which
would be exclusively owned by the state; the second category consisted of industries in which the
private sector could supplement the efforts of the state sector, with the state taking the sole
responsibility for starting new units; the third category consisted of the remaining industries
which were to be in the private sector.
Although there was a category of industries left to the private sector, the sector was kept under
state control through a system of licenses. No new industry was allowed unless a license was
obtained from the government. This policy was used for promoting industry in backward regions;
it was easier to obtain a license if the industrial unit was established in an economically backward
area. In addition, such units were given certain concessions such as tax benefits and electricity at a
lower tariff. The purpose of this policy was to promote regional equality.
Even an existing industry had to obtain a license for expanding output or for diversifying
production (producing a new variety of goods). This was meant to ensure that the quantity of
goods produced was not more than what the economy required. License to expand production
was given only if the government was convinced that the economy required the larger quantity of
goods.

83. Solution: c)
http://insightsonindia.com

INSIGHTS

Page 34

Facebook Group: Indian Administrative Service (Raz Kr)

INSIGHTS MOCK PRELIMS TEST-4

SOLUTIONS

In 1955, the Village and Small-scale Industries Committee, also called the Karve Committee,
noted the possibility of using small-scale industries for promoting rural development.
It was believed that small-scale industries are more labour intensive i.e., they use more labour
than the large-scale industries and, therefore, generate more employment. But these industries
cannot compete with the big industrial firms; it is obvious that development of small-scale
industry requires them to be shielded from the large firms. For this purpose, the production of a
number of products was reserved for the small-scale industry; the criterion of reservation being
the ability of these units to manufacture the goods. They were also given concessions such as
lower excise duty and bank loans at lower interest rates.
What is the definition of MSME?
The Government of India has enacted the Micro, Small and Medium Enterprises Development
(MSMED) Act, 2006 in terms of which the definition of micro, small and medium enterprises is
as under:
(a) Enterprises engaged in the manufacture or production, processing or preservation of goods as
specified below:
(i) A micro enterprise is an enterprise where investment in plant and machinery does not exceed
Rs. 25 lakh;
(ii) A small enterprise is an enterprise where the investment in plant and machinery is more than
Rs. 25 lakh but does not exceed Rs. 5 crore; and
(iii) A medium enterprise is an enterprise where the investment in plant and machinery is more
than Rs.5 crore but does not exceed Rs.10 crore.

84. Solution b)
Congress swept the general seats in the provincial elections, and the Muslim League captured
most of the reserved Muslim seats. But the League chose to boycott the Constituent Assembly,
pressing its demand for Pakistan with a separate constitution. The Socialists too were initially
unwilling to join, for they believed the Constituent Assembly was a creation of the British, and
therefore incapable of being truly autonomous. In effect, therefore, 82 per cent of the members of
the Constituent Assembly were also members of the Congress.

http://insightsonindia.com

INSIGHTS

Page 35

Facebook Group: Indian Administrative Service (Raz Kr)

INSIGHTS MOCK PRELIMS TEST-4

SOLUTIONS

Many of the linguistic minorities wanted the protection of their mother tongue, religious
minorities asked for special safeguards, while dalits demanded an end to all caste oppression and
reservation of seats in government bodies. Important issues of cultural rights and social justice
raised in these public discussions were debated on the floor of the Assembly.
Reference: Page No. 408(THEMES IN INDIAN HISTORY PART III)

85. Solution b)
First Past the Post (FPTP)system is followed in Lok Sabha and Assembly elections, whereas
proportional representation by means of single transferable vote is carried out in Rajya Sabha
and election of President and Vice-President.
Election Commission does not supervise Panchayat and Municipal elections, it is done by
State Election Commission.
The Election Commissioners are removed by the President not the Chief Election
Commissioner.
In 1993, two Election Commissioners were once again appointed and the Commission
became multi-member and has remained multi-member since then.
Reference : Page 68, 69 (Indian Constitution at Work Class XI), Election Commission website.

86. Solution d)
The President can send back the advice given by the Council of Ministers and ask the Council
to reconsider the decision. In doing this, the President acts on his (or her) own discretion.
When the President thinks that the advice has certain flaws or legal lacunae, or that it is not
in the best interests of the country, the President can ask the Council to reconsider the
decision. Although, the Council can still send back the same advice and the President would
then be bound by that advice, such a request by the President to reconsider the decision,
would naturally carry a lot of weight. So, this is one way in which the president can act in his
own discretion.
The President also has veto power by which he can withhold or refuse to give assent to Bills
(other than Money Bill) passed by the Parliament. Every bill passed by the Parliament goes to
the President for his assent before it becomes a law. The President can send the bill back to
http://insightsonindia.com

INSIGHTS

Page 36

Facebook Group: Indian Administrative Service (Raz Kr)

INSIGHTS MOCK PRELIMS TEST-4

SOLUTIONS

the Parliament asking it to reconsider the bill. This veto power is limited because, if the
Parliament passes the same bill again and sends it back to the President, then, the President
has to give assent to that bill.
Proportional representation by means of single transferable vote is carried out in the election
of President.
The Presidents ordinance making power is not a discretionary power, since it is based on the
advice of the council of ministers headed by the Prime Minister.
President is elected by the members of the electoral college consisting of Elected members of
both the Houses of the parliament, elected members of the legislative assemblies (States &
Union territories)
Reference : Page 84, 85, 86, 87(Indian Constitution at Work Class XI)

87. Solution: c)
In Mizoram State the Barak flows southwest then veers abruptly north when joined by a north
flowing stream and flows into Assam State where it turns westward again near Lakhipur as it
enters the plains. It then flows west past the town of Silchar where it is joined by the Madhura
River. Silchar is in Assam.
Tipaimukh Dam is a proposed embankment dam on the river Barak in Manipur state India, first
commissioned in 1984. The purpose of the dam is flood control and hydroelectric power
generation. It has been subject to repeated delays as the project developed, as there has been
controversy between India and Bangladesh over water rights, in addition to questions of
environmental effects of the huge project, as well as the need to relocate Manipuri people to
make way for a vast reservoir. In 2013, the governments of India and Bangladesh announced
further delays, as the latter nation undertakes additional studies about expected effects and
mitigating measures.

88. Solution: d)
All are correct.

http://insightsonindia.com

INSIGHTS

Page 37

Facebook Group: Indian Administrative Service (Raz Kr)

INSIGHTS MOCK PRELIMS TEST-4

SOLUTIONS

The catchments of large rivers are called river basins while those of small rivulets and rills are
often referred to as watersheds. There is, however, a slight difference between a river basin and a
watershed. Watersheds are small in area while the basins cover larger areas. River basins and
watersheds are marked by unity. What happens in one part of the basin or watershed directly
affects the other parts and the unit as a whole. That is why, they are accepted as the most
appropriate micro, meso or macro planning regions.

89. Solution: b)
(i)

(ii)
(iii)

Major river basins with more than 20,000 sq. km of catchment area. It includes 14
drainage basins such as the Ganga, the Brahmaputra, the Krishna, the Tapi, the
Narmada, the Mahi, the Pennar, the Sabarmati, the Barak, etc.
Medium river basins with catchment area between 2,000-20,000 sq. km
incorporating 44 river basins such as the Kalindi, the Periyar, the Meghna, etc.
Minor river basins with catchment area of less than 2,000 sq. km include fairly good
number of rivers flowing in the area of low rainfall.

90. Solution: a)
India agreed to the conditionalities of World Bank and IMF and announced the New Economic
Policy (NEP). The NEP consisted of wide ranging economic reforms. The thrust of the policies
was towards creating a more competitive environment in the economy and removing the barriers
to entry and growth of firms. This set of policies can broadly be classified into two groups: the
stabilisation measures and the structural reform measures. Stabilisation measures are short term
measures, intended to correct some of the weaknesses that have developed in the balance of
payments and bring inflation under control. In simple words, this means that there was a need to
maintain sufficient foreign exchange reserves and keep the rising prices under control.
On the other hand, structural reform policies are long-term measures, aimed at improving the
efficiency of the economy and increasing its international competitiveness by removing the
rigidities in various segments of the Indian economy.

91. Solution: c)

http://insightsonindia.com

INSIGHTS

Page 38

Facebook Group: Indian Administrative Service (Raz Kr)

INSIGHTS MOCK PRELIMS TEST-4

SOLUTIONS

River Kosi, also know as the sorrow of Bihar, has been notorious for frequently changing its
course. The Kosi brings huge quantity of sediments from its upper reaches and deposits it in the
plains. The course gets blocked, and consequently, the river changes its course.
Extensive soil erosion and landslides in its upper catchment have produced a silt yield of about
19 m/ha/year (10 cu yd/acre/yr), one of the highest in the world. Of major tributaries, the
Arun brings the greatest amount of coarse silt in proportion to its total sediment load. The river
transports sediment down the steep gradients and narrow gorges in the mountains and foothills
where the gradient is at least ten metres per km. On the plains beyond Chatra, the gradient falls
below one metre per km to as little as 6 cm per km as the river approaches the Ganges. Current
slows and the sediment load settles out of the water and is deposited on an immense alluvial
fan that has grown to an area of about 15 000 km. This fan extends some 180 km from its apex
where it leaves the foothills, across the international border into Bihar state and on to the Ganges.
The river has numerous interlacing channels that shift laterally over the fan from time to time.
Without channelization, floods spread out very widely. The record flow of 24 200 m/s is
equivalent to water a metre deep and more than 24 km wide, flowing at one metre per second.

92. Solution: d)
The northern boundary of the Peninsular Block may be taken as an irregular line running from
Kachchh along the western flank of the Aravali Range near Delhi and then roughly parallel to the
Yamuna and the Ganga as far as the Rajmahal Hills and the Ganga delta. Apart from these, the
Karbi Anglong and the Meghalaya Plateau in the northeast and Rajasthan in the west are also
extensions of this block. The northeastern parts are separated by the Malda fault in West Bengal
from the Chotanagpur plateau. In Rajasthan, the desert and other desertlike features overlay this
block.
The Peninsula is formed essentially by a great complex of very ancient gneisses and granites,
which constitutes a major part of it. Since the Cambrian period, the Peninsula has been standing
like a rigid block with the exception of some of its western coast which is submerged beneath
the sea and some other parts changed due to tectonic activity without affecting the original
basement.

93. Solution c)
Size of the Council of Ministers- Before the 91st Amendment Act (2003),the size of the
Council of Ministers was determined according to exigencies of time and requirements of the
situation.

http://insightsonindia.com

INSIGHTS

Page 39

Facebook Group: Indian Administrative Service (Raz Kr)

INSIGHTS MOCK PRELIMS TEST-4

SOLUTIONS

But then according to the amendment the Council of Ministers shall not exceed 15 percent of
total number of members of the House of People (or Assembly, in the case of the States).
The Council of Ministers is collectively responsible to the Lok Sabha. This provision means
that a Ministry which loses confidence of the Lok Sabha is obliged to resign. The principle
indicates that the ministry is an executive committee of the Parliament and it collectively
governs on behalf of the Parliament. Collective responsibility is based on the principle of the
solidarity of the cabinet. It implies that a vote of no confidence even against a single minister
leads to the resignation of the entire Council of Ministers.
In India, the Prime Minister enjoys a pre-eminent place in the government. The Council of
Ministers cannot exist without the Prime Minister. The Council comes into existence only
after the Prime Minister has taken the oath of office. The death or resignation of the Prime
Minister automatically brings about the dissolution of the Council of Ministers but the
demise, dismissal or resignation of a minister only creates a ministerial vacancy.
Reference : Page 90, 91, 92(Indian Constitution at Work Class XI)

94. Solution: d)
Some of the factors responsible for land degradation are
(i)
(ii)
(iii)
(iv)
(v)
(vi)
(vii)
(viii)
(ix)
(x)
(xi)
(xii)

loss of vegetation occurring due to deforestation


unsustainable fuel wood and fodder extraction
shifting cultivation
encroachment into forest lands
forest fires and over grazing
non-adoption of adequate soil conservation measures
improper crop rotation
indiscriminate use of agro-chemicals such as fertilizers and pesticides
improper planning and management of irrigation systems
extraction of ground water in excess of the recharge capacity
open access resource and
Poverty of the agriculture-dependent people.

95. Solution: d)
All are correct.

http://insightsonindia.com

INSIGHTS

Page 40

Facebook Group: Indian Administrative Service (Raz Kr)

INSIGHTS MOCK PRELIMS TEST-4

SOLUTIONS

Export surplus means an economic measure of a positive balance of trade, where a country's
exports exceeds its imports. A trade surplus represents a net inflow of domestic currency from
foreign markets, and is the opposite of a trade deficit, which would represent a net outflow.

96. Solution: a)
The British introduced the railways in India in 1850 and it is considered as one of their most
important contributions. The railways affected the structure of the Indian economy in two
important ways.
On the one hand it enabled people to undertake long distance travel and thereby break
geographical and cultural barriers while, on the other hand it fostered commercialization of
Indian agriculture which adversely affected the comparative self-sufficiency of the village
economies in India. The volume of Indias export trade undoubtedly expanded but its benefits
rarely accrued to the Indian people.

97. Solution d)
The Prime Minister acts as a link between the Council of Ministers on the one hand and the
President as well as the Parliament on the other. It is this role of the Prime Minister which led Pt.
Nehru to describe him as the linchpin of Government. It is also the constitutional obligation of
the Prime Minister to communicate to the President all decisions of the Council of Ministers
relating to the administration of the affairs of the Union and proposals for legislation.
At the State level, a similar parliamentary executive exists, though with some variations. The
most important variation is that there is a Governor of the State appointed by the President (on
the advice of the Central govt).Though the Chief Minister, like the Prime Minister is the leader of
the majority party in the Assembly, the Governor has more discretionary powers.
The writ jurisdiction of the Supreme Court is narrower compared to the High Court since writ
jurisdiction of High Court involves not only Fundamental Rights but also ordinary legal right.
Reference : Page 92, 93, 94(Indian Constitution at Work Class XI)

98. Solution: d)
Physical and Human Capital

http://insightsonindia.com

INSIGHTS

Page 41

Facebook Group: Indian Administrative Service (Raz Kr)

INSIGHTS MOCK PRELIMS TEST-4

SOLUTIONS

Both the forms of capital formation are outcomes of conscious investment decisions. Decision
regarding investment in physical capital is taken on the basis of ones knowledge in this regard.
The entrepreneur possesses knowledge to calculate the expected rates of return to a range of
investments and then rationally decides which one of the investments should be made. The
ownership of physical capital is the outcome of the conscious decision of the owner the
physical capital formation is mainly an economic and technical process. A substantial part of the
human capital formation takes place in ones life when she/he is unable to decide whether it
would maximize her/his earnings.
Children are given different types of school education and health care facilities by their parents
and the society. The peers, educators and society influence the decisions regarding human capital
investments even at the tertiary level, that is, at the college level. Moreover, the human capital
formation at this stage is dependent upon the already formed human capital at the school level.
Human capital formation is partly a social process and partly a conscious decision of the possessor
of the human capital.
You know that the owner of a physical capital, says a bus, need not be present in the place where
it is used; whereas, a bus-driver, who possesses the knowledge and ability to drive the bus, should
be present when the bus is used for transportation of men and materials. Physical capital is
tangible and can be easily sold in the market like any other commodity. Human capital is
intangible; it is endogenously built in the body and mind of its owner. Human capital is not sold
in the market; only the services of human capital are sold and hence the necessity of the owner of
the human capital to be present in the place of production. The physical capital is separable from
its owner, whereas, human capital is inseparable from its owner.
The two forms of capital differ in terms of mobility across space. Physical capital is completely
mobile between countries except for some artificial trade restrictions. Human capital is not
perfectly mobile between countries as movement is restricted by nationality and culture.
Therefore, physical capital formation can be built even through imports, whereas human capital
formation is to be done through conscious policy formulations in consonance with the nature of
the society and economy and expenditure by the state and the individuals.
Both forms of capital depreciate with time but the nature of depreciation differs between the
two. Continuous use of machine leads to depreciation and change of technology makes a machine
obsolete. In the case of human capital, depreciation takes place with ageing but can be reduced,
to a large extent, through continuous investment in education, health, etc. This investment also
facilitates the human capital to cope with change in technology which is not the case with
physical capital.
Nature of benefits flowing from human capital is different from that of physical capital. Human
capital benefits not only the owner but also the society in general. This is called external benefit.
An educated person can effectively take part in a democratic process and contribute to the sociohttp://insightsonindia.com

INSIGHTS

Page 42

Facebook Group: Indian Administrative Service (Raz Kr)

INSIGHTS MOCK PRELIMS TEST-4

SOLUTIONS

economic progress of a nation. A healthy person, by maintaining personal hygiene and sanitation,
stops the spread of contagious diseases and epidemics. Human capital creates both private and
social benefits, whereas physical capital creates only private benefit. That is, benefits from a
capital good flow to those who pay the price for the product and services produced by it.

99. Solution d)
The Prime Minister is involved in all crucial decisions of the government and decides on the
policies of the government. Thus, the power wielded by the Prime Minister flows from various
sources: control over the Council of Ministers, leadership of the Lok Sabha, command over the
bureaucratic machine, access to media, projection of personalities during elections, projection as
national leader during international summitry as well as foreign visits.
Reference : Page 92(Indian Constitution at Work Class XI)

100.

Solution: d)

The Environmental Performance Index (EPI) is a method of quantifying and numerically marking
the environmental performance of a state's policies. This index was developed from the Pilot
Environmental Performance Index, first published in 2002, and designed to supplement the
environmental targets set forth in the United Nations Millennium Development Goals.
The EPI was preceded by the Environmental Sustainability Index (ESI), published between 1999
and 2005. Both indexes were developed by Yale University (Yale Center for Environmental Law
and Policy) and Columbia University (Center for International Earth Science Information
Network) in collaboration with the World Economic Forum and the Joint Research Centre of
the European Commission. The ESI was developed to evaluate environmental sustainability
relative to the paths of other countries. Due to a shift in focus by the teams developing the ESI,
the EPI uses outcome-oriented indicators, then working as a benchmark index that can be more
easily used by policy makers, environmental scientists, advocates and the general public.
In the 2012 EPI ranking, the top five countries were Switzerland, Latvia, Norway, Luxembourg,
and Costa Rica.
The bottom five countries were South Africa, Kazakhstan, Uzbekistan, Turkmenistan,
and Iraq. The United Kingdom was ranked in 9th place, Japan 23rd place, Brazil 30th, the United
States 49th, China 116th, and India came in 125th.

http://insightsonindia.com

INSIGHTS

Page 43

Facebook Group: Indian Administrative Service (Raz Kr)

INSIGHTS ON INDIA MOCK PRELIMINARY EXAM - 2014


INSIGHTS ON INDIA MOCK TEST - 5
GENERAL STUDIES

PAPER-I
Time Allowed: Two Hours

Maximum Marks: 200

INSTRUCTIONS
1. IMMEDITELY AFTER THE COMMENCEMENT OF THE EXAMINATION, YOU SHOULD
CHECK THAT THIS TEST BOOKLET DOES NOT HAVE ANY UNPRINTED OR TORN OR
MISSING PAGES OR ITEMS, ETC. IF SO, GET IT REPLACED BY A COMPLETE TEST BOOKLET.
2. You have to enter your Roll Number on the Test I
Booklet in the Box provided alongside. DO NOT
write anything else on the Test Booklet.
4. This Test Booklet contains 100 items (questions). Each item is printed only in English. Each item
comprises four responses (answers). You will select the response which you want to mark on the Answer
Sheet. In case you feel that there is more than one correct response, mark the response which you consider
the best. In any case, choose ONLY ONE response for each item.
5. You have to mark all your responses ONLY on the separate Answer Sheet provided. See directions in the
Answer Sheet.
6. All items carry equal marks.
7. Before you proceed to mark in the Answer Sheet the response to various items in the Test Booklet, you
have to fill in some particulars in the Answer Sheet as per instructions sent to you with your Admission
Certificate.
8. After you have completed filling in all your responses on the Answer Sheet and the examination has
concluded, you should hand over to the Invigilator only the Answer Sheet. You are permitted to take away
with you the Test Booklet.
9. Sheets for rough work are appended in the Test Booklet at the end.
10. Penalty for wrong answers :
THERE WILL BE PENALTY FOR WRONG ANSWERS MARKED BY A CANDIDATE IN THE
OBJECTIVE TYPE QUESTION PAPERS.
(i)

There are four alternatives for the answer to every question. For each question for which a
wrong answer has been given by the candidate, one-third of the marks assigned to that question
will be deducted as penalty.

(ii) If a candidate gives more than one answer, it will be treated as a wrong answer even if one of the
given answers happens to be correct and there will be same penalty as above to that question.
(iii)

If a question is left blank, i.e., no answer is given by the candidate, there will be no penalty for
that question.

http://insightsonindia.com

INSIGHTS ON INDIA MOCK TEST SERIES FOR CIVIL SERVICES PRELIMINARY EXAM 2014

http://insightsonindia.com

Page 1

Facebook Group: Indian Administrative Service (Raz Kr)

b. 2 Only
c. Both
d. None

1. Factors affecting ocean salinity are,


1. The salinity of water in the
surface layer of oceans depend
mainly on evaporation and
precipitation.
2. Surface
salinity
is
greatly
influenced in coastal regions by
the fresh water flow from rivers,
and in polar regions by the
processes of freezing and thawing
of ice.
3. Wind, also influences salinity of
an area by transferring water to
other areas
Which of the above statements is/are
correct?
a.
b.
c.
d.

1 Only
1 and 2 Only
2 and 3 Only
All

2. Consider the following statements


1. The dominant thinking in
economics before Keynes was
that all the labourers who are
ready to work will find
employment and all the factories
will be working at their full
capacity.
2. Keynes approach was to examine
the working of the economy in its
entirety and examine the
interdependence of the different
sectors.
Which of the above statements is/are
correct?

3. Consider the following statements


about earthquakes
1. The point where the energy is
released is called the epicenter.
2. The energy waves travelling in
different directions reach the
surface. The point on the surface,
nearest to the epicenter, is called
hypocenter.
Which of the above statements is/are
incorrect?
a.
b.
c.
d.

1 Only
2 Only
Both
None

4. Consider the following:


Most Ashokan inscriptions were in
the Pali language while those in the
northwest of the subcontinet were
in Aramaic and Greek.
2. Most Ashokan inscriptions were
written in the Brahmi script;
however, some, in the northwest,
were written in Kharosthi.
1.

Which of the above is/are correct?


a.
b.
c.
d.

1 Only
2 Only
Both
None

5. Consider the following statements about


the recent Supreme Court order on

a. 1 Only
http://insightsonindia.com

Page 2

Facebook Group: Indian Administrative Service (Raz Kr)

creating a National Environment Appraisal


and Monitoring Authority (NEAMA) :
1. It would take away the
powers of forest clearances
from
the
Ministry
of
Environment and Forests
(MoEF).
2. NEAMA will be responsible
for implementing the Forest
Policy.
3. It will be independent from
the MoEF, but not from the
influence of the Cabinet
Committee on Economic
Affairs (CCEA).
Which of the following is/are true in this
connection?
a.
b.
c.
d.

Only 2
2 and 3
1 and 3
All of the above

6. According to Wegener, who gave


continental
drift
theory,
the
movement responsible for the
drifting of the continents was caused
by
1. The polar-fleeing force
2. The tidal force
3. The convection currents
Which of the above is/are correct?
a.
b.
c.
d.

1 Only
1 and 2 Only
2 Only
All Three

7. Consider the following with respect


to the powers of the Rajya Sabha :
1. Considers and approves non money
bills and suggests amendments to
money bills.
2. Approves
constitutional
amendments.
3. Exercises control over executive by
asking
questions,
introducing
motions and resolutions.
4. It can alone initiate the procedure
for removal of Vice President.
Which of the statements given above
is/are correct?
a.
b.
c.
d.

3 Only
2 and 3 Only
1, 2 and 3 Only
1, 2, 3 and 4 Only

8. Consider the following statements


about capital goods
1. While
they
make
production
of
other
commodities feasible, they
themselves
dont
get
transformed
in
the
production process.
2. They are the crucial
backbone
of
any
production process, in
aiding and enabling the
production to take place.
Which of the above statements is/are
characteristics of a capitalist economy?
a. 1 Only
b. 2 Only

http://insightsonindia.com

Page 3

Facebook Group: Indian Administrative Service (Raz Kr)

c. Both
d. None
9. Which of the followings is/are true
about National Ganga River Basin
Authority (NGBA)?
1. It is the monitoring and coordinating
authority for the Ganga river basin
protection.
2. It plays an important role in financial
planning for the Ganga river basin.
3. One of the objectives of NGBA is to
protect overuse of Ganga river water.
4. It is supported by World Bank.
Select the correct answer using the codes
below
a.
b.
c.
d.

1 and 3
1,2 and 4
All of the above
Only 3

10. Which of the following is/are correct


with reference to the Public Interest
Litigation (PIL)?
1. It has expanded the scope of Article
21
2. It has led to overburdening of courts
3. It may be introduced in a court of
law by the court itself.
4. It can be filed either in High Court
or Supreme Court
Select the correct code.
a.
b.
c.
d.

1 Only
1, 2 and 4 Only
2 and 4 Only
1, 2, 3 and 4

http://insightsonindia.com

11. When the availability of water in


different storages and reservoirs like
aquifers, lakes, reservoirs, etc. falls
below what the precipitation can
replenish, then it is called as
a.
b.
c.
d.

Meteorological drought
Hydrological drought
Agricultural drought
Ecological drought

12. Consider the following statements


about Consumer Price Index (CPI)
1. CPI is the index of prices
of a given basket of
commodities which are
bought
by
the
representative consumer.
2. CPI is generally expressed
in percentage terms.
3. CPI is one of the most
frequently used statistics
for identifying periods of
inflation or deflation.
Which of the above statements is/are
incorrect?
a.
b.
c.
d.

1 Only
2 and 3 Only
3 Only
None

13. Asoka was arguably the most famous


ruler of early India. Consider the
following statements:
In none of the Asokan
inscriptions, the name of the
ruler, Asoka, is mentioned.
What is mentioned instead
are titles adopted by the
ruler devanampiya and
piyadassi .
2. Asoka was the first ruler
1.

Page 4

Facebook Group: Indian Administrative Service (Raz Kr)

who inscribed his messages


to his subjects and officials
on stone surfaces.
Which of the above is/are correct?
a.
b.
c.
d.

1 Only
2 Only
Both
None

14. These clouds are formed at high


altitudes (8,000 - 12,000m). They
are thin and detached clouds having
a feathery appearance. They are
always white in colour.
The above description refers to which type
of cloud?
a.
b.
c.
d.

Cumulus
Cirrus
Cumulonimbus
Stratus

15. The Unionist party was a powerful


political party during the period
1923-47. Consider the following
about this party:
1. The

party represented the


interests of landholders
Hindu, Muslim and also the
Sikh.
2. The party was particularly
powerful in the Punjab region
and it did not favour the
creation of Pakistan as a
separate state.
Which of the above is/are correct?
a. 1 Only
http://insightsonindia.com

b. 2 Only
c. Both
d. None
16. The Census of India defines an urban
area as having:
1. A minimum population of 10000
2. At least 50 per cent of male working
population
engaged
in
nonagricultural occupations.
3. A density of population of at least
400 persons per sq.
Select the correct code.
a)
b)
c)
d)

3 Only
1 Only
1 and 2 Only
1 and 3 Only

17. The CAG recently unveiled a


scam relating to the violation of the
of compensatory afforestration
Consider the following statements
the policy:

major
policy
(CA).
about

1. It is compulsory to follow this


policy while awarding projects
which destroy forests in India.
2. Compensatory Afforestation
Fund
Management
and
Planning Authority (CAMPA)
implements this policy.
3. The
non-availability
of
suitable non-forest land for
CA has to be certified only by
the Chief Secretary of the
State/UT.
Which of the following is/are true
in this connection?
a) All of the above
Page 5

Facebook Group: Indian Administrative Service (Raz Kr)

b) None of the above

4.

c) 1 and 3
d) 1 and 2
18. Consider the following
about temperature

statements

1. Compared to land, the sea gets


heated slowly and loses heat
slowly. Land heats up and cools
down quickly.
2. The variation in temperature
over the sea is more compared to
land.
3. The places situated near the sea
come under the moderating
influence of the sea and land
breezes which moderate the
temperature.
Which of the above statements is/are
correct?
a.
b.
c.
d.

1 and 3 Only
1 and 2 Only
1 Only
2 and 3 Only

19. Arya Samaj was a North Indian Hindu


reform organisation of the late
nineteenth
and
early
twentieth
centuries, particularly active in the
Punjab.
Consider
the
following
statements about Arya Samaj:
Arya Samaj was founded
by M G Ranade in 1875.
2. It sought to revive Vedic
learning and combine it
with modern education in
the sciences.
3. It regards the Vedas as
infallible.
1.

http://insightsonindia.com

Arya Samaj professed


"Shuddhi" to bring back to
the Hindu fold those who
had recently converted to
Islam.

Which of the above is/are incorrect?


a. 1 Only
b. 2 and 4 Only
c. 1 and 2 Only
d. 1 and 4 Only
20. GDP of a country as an index of greater
well-being of the people of that country
may not be correct because,
1. Many activities in an
economy are not evaluated
in monetary terms.
2. The presence of positive or
negative
externalities
affects
the
actual
estimation of GDP
Which of the above statements is/are
correct?
a. 1 Only
b. 2 Only
c. Both
d. None
21. Which of the following statements
is/are correct with reference to the
powers of the Supreme Court of India?
1. It can transfer any Judge of a High
Court.
2. It can deal with any case within the
jurisdiction of the State.
3. It can move cases from any court to
itself.
4. Its decisions may be binding on all
courts.
Page 6

Facebook Group: Indian Administrative Service (Raz Kr)

5. It can transfer cases from one High


Court to another High Court

a.
b.
c.
d.

Select the correct code.


a.
b.
c.
d.

Which of the following is/are true


in this connection?

4 and 5 Only
1, 3 and 5 Only
1, 2, 3 and 5 Only
1, 2, 3, 4 and 5

Only 1
Only 2
Both
None

24. Consider the following statements


about the Sufi saints:

22. Consider the following:


1. The
1. The

Muslim League was


floated in Dhaka in 1906,
with a separate Muslim state
as their goal.
2. The name Pakistan or Pakstan (from Punjab, Afghan,
Kashmir,
Sind
and
Baluchistan) was coined by
Muhammad
Iqbal,
the
composer of Saare Jahan se
achcha.

Which of the above is/are correct?


a.
b.
c.
d.

1 Only
2 Only
Both
None

23. Consider the following


about Plant Succession :
1.

Which of the above is/are correct?


a.
b.
c.
d.

1 and 2 Only
1 and 3 Only
2 and 3 Only
All the above

25. Consider the


about cyclones

following

statements

statements

It is a directional nonseasonal
cumulative
change in the types
of plant species that
occupy a given area
through time.
2. Succession begins only
when an area is made
partially or completely
devoid of vegetation
because of a disturbance.
http://insightsonindia.com

Dargah of Shaikh
Nizamuddin Auliya is located
in Delhi.
2. The famous Dargah of Ajmer
is the tomb of Shaikh
Muinuddin Sijzi.
3. Both Shaikh Nizamuddin
Auliya and Shaikh Muinuddin
Sijzi belong to the Chisti
silsila.

1. The energy to the tropical


cyclone comes from the latent
heat released by the warm moist
air.
2. Hence, with the increase in
distance from the sea, the force of
the cyclone decreases.
Which of the above statements is/are
correct?
a. 1 Only
b. 2 Only
Page 7

Facebook Group: Indian Administrative Service (Raz Kr)

c. Both
d. None
26. Lord Dalhousie is one of the famous
Governers-General in India. Consider
the
following statements
about
Dalhousie:
Lord Dalhousie's policy of
"refrain
from
annexations"
creation affection towards him
among the people.
2. Dalhousie was a progressive
Governer-General,
he
was
responsible for the introduction
of post and telegraph system in
India.
3. Dalhousie
introduced many
administrative
as
well
as
educational reforms during his
period in office in India as the
Governer-General.
1.

Which of the above is/are correct?


a.
b.
c.
d.

1 and 2 Only
1 and 3 Only
2 and 3 Only
All the above

27. The following forms of relationships


exist between man and his environment
according to the development of
geographical thought
1. Environmental determinism
2. Possibilism
3. Neo-determinism
The correct sequence of the
development of these forms of
relationships is:
a. 2,1,3
b. 1,2,3
http://insightsonindia.com

c. 3,1,2
d. 1,3,2

28. The Constitution of India provides for a


single Integrated Judicial System. In this
context, which of the following statements
is/are correct?
1. District
Court
decides
cases
involving serious criminal offences.
2. Subordinate Courts considers cases
of both civil and criminal nature.
3. Both High Court and Supreme
Court have appellate jurisdiction
4. The Supreme Court is bound by its
own decision and cannot review it.
5. Both High Court and Supreme
Court have advisory jurisdiction
Select the correct code.
a.
b.
c.
d.

1, 2 and 3 Only
1, 3 and 5 Only
2, 3, and 4 Only
None

29. Consider the following


about bank deposits

statements

1. Time
deposit account
balances
are
usually
considered money and
form the greater part of the
narrowly defined money
supply of a country.
2. Fixed deposits, have a
fixed period to maturity
and are referred to as time
deposits.
Which of the above is/are correct?
a. 1 Only
Page 8

Facebook Group: Indian Administrative Service (Raz Kr)

b. 2 Only
c. Both
d. None
30. Consider the following statements
1. These volcanoes are characterised
by eruptions of cooler and more
viscous lavas than basalt.
2. These volcanoes often result in
explosive eruptions.
3. Along with lava, large quantities
of pyroclastic material and ashes
find their way to the ground.
The above characters best describe which
of the following type of volcanoes?
a.
b.
c.
d.

Caldera
Shield volcanoes
Composite volcanoes
Mid-Ocean ridge volcanoes

31. Consider the following statements


1. Reserve deposit ratio (rdr)
is the proportion of the
total deposits commercial
banks keep as reserves.
2. Commercial banks can
borrow money from RBI at
the bank rate when they
run short of reserves.
3. Maintaining
a
given
fraction of their total
demand and time deposits
in the form of specified
liquid assets by the banks is
called Cash Reserve Ratio.
Which of the above statements is/are
correct?
http://insightsonindia.com

a.
b.
c.
d.

1 and 2 Only
2 and 3 Only
3 Only
1 and 3 Only

32. Consider the following statements


related to the 1857 uprising:
During the 1857 uprising,
religious divisions between
Hindus and Muslim were
hardly
noticeable despite
British attempts to create such
divisions.
2. The fear that the British were
bent on converting Hindus and
Muslims to Christianity was
also a reason for the uprising.
1.

Which of the above is/are correct?


a.
b.
c.
d.

1 Only
2 Only
Both
None

33. Histories of the Gupta rulers have


been reconstructed from literature,
coins and inscriptions. Consider the
following statements about the
Gupta rule in India:
1. The Prayaga Prashasti

(also known as the


Allahabad
Pillar
Inscription) composed
by Banabhatta.
2. Banabhatta, was the
court
poet
of
Samudragupta.
3. The Prayaga Prashasti
was composed in praise
of SamudraGupta in
Prakrit language.
Which of the above is/are incorrect?
Page 9

Facebook Group: Indian Administrative Service (Raz Kr)

a.
b.
c.
d.

1 and 2 Only
1 and 3 Only
2 and 3 Only
All the above

34. Consider the following statements with


respect to special provisions provided to
Jammu and Kashmir:
1. The concurrence of the State is
required for making any law in
matters mentioned in both the
Union and Concurrent lists.
2. Emergency
due
to
internal
disturbances can be declared without
the concurrence of the State.
3. The Directive Principles of State
policy applies to the State.
4. Amendments
to
the
Indian
Constitution, under Article 368, can
only apply in concurrence with the
government of Jammu and Kashmir.
Which of the above statements is/are
incorrect?
a)
b)
c)
d)

2 and 3 Only
1, 2 and 4 Only
1, 3 and 4 Only
1, 2, 3 and 4

35. Consider the following statements:


1. Quantity of biomass in a
higher trophic level is lesser
than that of the immediate
lower levels.
2. In the phenomena of plant
succession, rate of succession
is
faster
in
secondary
succession.
Which of the following is/are true?
a. Only 1
http://insightsonindia.com

b. Only 2
c. Both
d. None
36. To promote peoples participation, the
parliament enactment 73rd and 74th
Amendment Act. With reference to 73rd
Amendment
Act,
consider
the
following statements:
1. The Gram Panchayat would
comprise of all the adult members
registered as voters in the Panchayat
area.
2. The roles and functions of the Gram
Sabha are decided by State
legislation.
3. All the three levels of Panchayati Raj
institutions are elected directly by
the people.
4. It is mandatory to create Gram
Sabha in all States.
Which of the above statements is/are
correct?
a)
b)
c)
d)

1 Only
1 and 2 Only
2, 3 and 4 Only
1, 2, 3 and 4

37. In the Geological Time Scale, the


extinction of dinosaurs took place in
a.
b.
c.
d.

Cretaceous period
Permian Period
Devonian Period
Triassic Period

38. Consider the following with regard to


the teachings of Buddha:
1. The Buddha taught
orally

through
discussion and debate.
2. Only a part of the
Page 10

Facebook Group: Indian Administrative Service (Raz Kr)

Buddhas speeches were


written down during his
lifetime.
3. The compilations of his
teachings were known
as Tipitaka. The Vinaya
Pitaka included rules
and regulations for
those who joined the
sangha or monastic
order
while
Abhidhamma
Pitaka
dealt with philosophical
matters.
Which of the above is/are correct?
a.
b.
c.
d.

1 Only
1 and 2 Only
1 and 3 Only
All the above

Which of the above statements is/are


correct?

39. Which of the following is/are true in


connection with micro plastics?
1.
It
environment the most.

affects

2. It is used
manufacturing of cosmetics.

marine
in

the

3. It can act as an agent to


transport
Persistent
Organic
Pollutants(PoPs]
Select the correct answer using the codes
below.
a.
b.
c.
d.

Only 2
1 and 3
All of the above
Only 3

40. Consider the following:


http://insightsonindia.com

1. One-third
Reservations
for
Scheduled Castes and Scheduled
Tribes are provided for at all the
three levels.
2. The States may or may not provide
reservations for the Other Backward
Castes (OBC)
3. In the areas inhabited by the
Adivasis, the Gram Sabha is more
powerful compared to the other
parts of the country.
4. A minimum age of 21 years is
necessary to become a member of a
Panchayat.
5. The power to appoint and remove
the State Finance commissioner is
vested in the Governor.

a)
b)
c)
d)

1 Only
2, 3 and 4 Only
1, 2, 3 and 5 Only
1, 2, 3 and 4 Only

41. An item is called a final good,


1. Because once it has been
sold it passes out of the
active economic flow
2. It will not undergo any
further transformation at
the hands of any producer.
3. A final good may never
undergo transformation by
the action of the ultimate
purchaser.
Which of the above statements is/are
correct?
a. 1 and 2 Only
b. 3 Only
Page 11

Facebook Group: Indian Administrative Service (Raz Kr)

c. 2 and 3 Only
d. All Three
42. Consider the following statements,
1. The government has legal
authority to issue currency
by printing in case of a
budget deficit
2. The government borrows
money by selling treasury
bills
or
government
securities to RBI, which
issues currency to the
government in return.
Which of the above statements is/are
correct?
a.
b.
c.
d.

1 Only
2 Only
Both
None

43. Consider the following about the The


Khilafat Movement, (1919-1920) :
1. The Khilafat Movement was a
movement of Indian Muslims,
led by Sir Syed Ahmed Khan.
2. It demanded that the Turkish
Sultan or Khalifa must retain
control over the Muslim sacred
places in the erstwhile Ottoman
Empire.
3. Mahatma Gandhi supported the
movement but the congress
didn't support it.
Which of the above is/are correct?
a.
b.
c.
d.

1 Only
2 Only
3 Only
2 and 3 Only

http://insightsonindia.com

44. With regard to the theory of basic


structure of the Constitution, consider
the following:
1. Parliament has powers to amend all
parts of the Constitution except the
basic structure.
2. Constitution specifies the basic
structure of the Constitution
3. It found its first expression in the
Golaknath case.
4. It has increased the powers of the
judiciary.
5. Federal system, fundamental duties,
social justice
form the basic
structure of the Constitution
Which of the above statements is/are
incorrect?
a)
b)
c)
d)

1, 2 and 3
1 and 4 Only
2 and 3 Only
2, 3 and 5 Only

45. Which of the following sources help in


studying about the interior of the earth?
1. Meteors that at times reach the
earth
2. Gravitation
3. Magnetic Field
4. Seismic Activity
Select the correct answer using the codes
below
a.
b.
c.
d.

1,2 and 3 Only


2 and 3 Only
3 and 4 Only
All
Page 12

Facebook Group: Indian Administrative Service (Raz Kr)

46. The term biotype means:


a. All individuals having
the same phenotype.
b. All individuals having
the same genotype.
c. All individuals having
different
phenotype
comprising a whole
group.
d. All individuals having
different
genotype
comprising a whole
group.

exchange inflow in the


economy, thereby keeping
the stock of high powered
money and total money
supply unchanged.
2. Its a tool to fight inflation
Which of the statements given above
is/are correct?
a.
b.
c.
d.

1 Only
2 Only
Both
None

49. Which of the following is true about


Profundal zone?
47. The Ocean Health Index ( OCI ) is a
tool that scientifically studies the health of
oceans from multiple dimensions. What
dimensions, other than biological, is/are
included in this analysis?
1.
2.
3.
4.

Physical
Economic
Social
Political

Select the correct answer using the codes


below
a.
b.
c.
d.

1 and 2
Only 3
1,2 and 3
All of the above

48. Consider the following


about sterilization by RBI

statements

1. Its an open market sale of


government securities of an
amount equal to the
amount
of
foreign
http://insightsonindia.com

a. It is a deep zone of inland


body of free standing water,
located below the range of
effective light penetration.
b. It is a deep zone of the sea
located below the range of
effective light penetration.
c. It is upper zone of inland
body of free standing water.
d. None of the above.
50. Consider the following statements and

choose the appropriate option:


1. Purusha sukta, describes
the sacrifice of Purusha,
the primeval man. It says
that all the elements of
the
universe,
were
supposed
to
have
emanated from his body.
2. Purusha sukta is a part of
Rigveda.
3. The rule that Kshatriyas
were supposed to engage
in warfare, protect people
and administer justice,
Page 13

Facebook Group: Indian Administrative Service (Raz Kr)

study the Vedas, get


sacrifices performed, and
make gifts is given in the
Dharmasutras
and
Dharmashastras.

2. Public Accounts Committee is


an example of Standing
Committee.
3. Members of the Standing
committees are selected from
both Houses.
4. Standing Committees are
permanent in nature.
5. Standing Committees are
means
of
parliamentary
control over the executive.

Which of the above is/are correct?


a. 1 Only
b. 1 and 3 Only
c. 2 and 3 Only
d. All the above
51. Consider the following:
1. Governor has the power to
recommend the dismissal of the
State government and suspension or
dissolution of State assembly.
2. The Lok Sabha consists of only
elected representatives.
3. The Vice President belongs to the
Upper House of the Parliament.
4. The 73rd and 74th amendments have
created uniformity in the structures
of Panchayati Raj and Nagarpalika
institutions across the country.
Which of the above statements is/are
correct?
a.
b.
c.
d.

1 and 4 Only
1 and 3 Only
2 , 3 and 4 Only
1, 2, 3 and 4

52. With reference to Parliamentary


Standing Committees consider the
following:
1. Standing Committees not only
supervise the work of various
departments but also their
budget.
http://insightsonindia.com

Which of the above statements is/are


correct?
a)
b)
c)
d)

1, 4 and 5 Only
1, 3 and 5 Only
1, 2, 4 and 5 Only
1, 2, 3, 4 and 5

53. Consider the following statements


with reference to GDP deflator and
CPI
1. GDP deflator includes
prices of goods consumed
by
the
representative
consumer,
hence
it
includes prices of imported
goods. CPI does not
include prices of imported
goods.
2. The goods purchased by
consumers
do
not
represent all the goods
which are produced in a
country. GDP deflator
takes into account all such
goods and services.
Which of the above sentences is/are
correct?
Page 14

Facebook Group: Indian Administrative Service (Raz Kr)

a.
b.
c.
d.

1 Only
2 Only
Both
None

54. Which one is supposed to be the most


stable environment?
a.
b.
c.
d.

Deep sea regions


Temperate Grasslands
Tropical rainforests
Hilly Regions

55. Subsidiary

Alliance was a system


devised by Lord Wellesley in 1798.
Consider the following terms and
conditions of the system:
1. The British would be responsible for

protecting their ally from both the


external as well as internal threats to
their power.
2. The ally could not enter into
agreements with other rulers or
engage in warfare.
3. The British would assist the ally in
maintaining an armed contingent.
Which of the above is/are incorrect?
a.
b.
c.
d.

3 Only
1 and 2 Only
2 and 3 Only
None

56. Consider

the following statements


about the evolution of the present
atmosphere

2. In the second stage, the hot


interior of the earth contributed
to the evolution of the
atmosphere.
3. Finally, the composition of the
atmosphere was modified by the
living world through the process
of photosynthesis.
Which of the above statement is/are
correct?
a.
b.
c.
d.

1 Only
2 and 3 Only
3 Only
All

57. Consider the following statements:


1. The influence of bordering
communities on each other is
known as the edge effect in
environmental science.
2. The edges are different to the
interior of the fragment in
climate, soil environment and
species composition.
Which of these is/are true?
a.
b.
c.
d.
58. Consider

Only 1
Only 2
Both
None

the
following
Virashaivism in Karnataka:

about

1. Basavanna founded Virashaivism as a

1. There are three stages in the


evolution
of
the
present
atmosphere. The first stage is
marked by the loss of primordial
atmosphere.
http://insightsonindia.com

separate caste within Hinduism.


2. Virashaivism
allowed
widow
remarriage.
3. Lingayats believe in rebirth after
death. So they practise funerary rites
such as cremation.
Page 15

Facebook Group: Indian Administrative Service (Raz Kr)

Which of the above is/are correct?


a.
b.
c.
d.

1 Only
2 Only
2 and 3 Only
1 and 3 Only

59. Consider the following statements with

reference to earthquake waves


1. Body waves are generated due to
the release of energy at the focus
and move in all directions
travelling through the body of the
earth.
2. The body waves interact with the
surface rocks and generate new
set of waves called surface waves.
3. The denser the material, the
lower is the velocity of these
waves.
Which of the above statements is/are
correct?
a.
b.
c.
d.

1 and 3 Only
2 and 3 Only
1 and 2 Only
All Three

60. The Indian Constitution has ensured the

independence of the judiciary through a


number of measures. Which of the
following are these measures provided
in the Constitution?
1. The Executive is not involved in the
process of appointment of judges.
2. The judges have a fixed tenure.
3. The expenses of the judiciary are
charged on the Consolidated Fund of
India.
http://insightsonindia.com

4. The Supreme court and the High


Court can punish any person for its
contempt
5. A judge of Supreme court can only
be removed by a special majority of
the Parliament.
Select the correct code.
a)
b)
c)
d)

2, 3, and 5 Only
1, 2, 3 and 5 Only
2, 3, 4 and 5 Only
1, 2, 3, 4 and 5

61. With reference to a fresh water lake,


consider the following characteristics:
1. Richness in nutrients
2. Rapid
turnover
of
phytoplankton
3. Depletion of dissolved oxygen
and nitrogen
4. Increase in zooplankton levels
Which of the following is/are
observed in a lake after its Eutrophication?
a.
b.
c.
d.

1,2 and 3
All of the above
1 and 3 only
2 and 4

62. Three Round Table Conferences held

between
1930-32.
Consider
following in this regard:

the

1. Mahatma Gandhi attended the first

Round table conference.


2. He attended only one Round table
conference out of the three Round
table conferences held.
3. He opposed the separate electorates
for the Untouchables in the Round
table conference he attended.
Page 16

Facebook Group: Indian Administrative Service (Raz Kr)

Which of the above is/are correct ?


a.
b.
c.
d.

1 and 2 Only
1 and 3 Only
2 and 3 Only
All the above

63. Consider the following statements


about Epiphytes a plant species:
4. They grow parasitically upon
another plant
5. They derive their moisture and
nutrients from the air and rain.
6. They may harm the forest trees.
Which of the following is/are true
in this connection?
a. 1 and 3
b. 1 and 2
c. Only 2
d. All of the above
64. "Quit India" campaign/movement was
launched in 1942. Consider the following
statements on the movement
1. The failure of the Cripps Mission

was the immediate trigger for


Mahatma Gandhi to launch Quit
India movement against British rule.
2. Mahatma Gandhi led the movement
through its course by travelling
extensively through the length and
breadth of the country.
Which of the above statement/s is/are
correct?
a.
b.
c.
d.

1 only
2 only
Both
None

65. Consider the following intrusive forms


of lava
1. Lacoliths - A large body of
magmatic material that cools in
the deeper depth of the crust
develops in the form of large
domes.
2. Batholiths - These are large
dome-shaped intrusive bodies
with a level base and connected
by a pipe-like conduit from
below
Which of the above is/are correct?
a.
b.
c.
d.

1 Only
2 Only
Both
None

66. Consider the following statements with


reference to GDP
1. Real
GDP is GDP
evaluated
at
current
market prices. Therefore,
real GDP will include all of
the changes in market
prices that have occurred
during the current year due
to inflation or deflation.
2. Nominal GDP is GDP
evaluated at the market
prices of some base year
3. The factor used to convert
GDP from current to
constant values in this way
is called the GDP deflator.
Which of the above statements is/are
correct?
a.
b.

http://insightsonindia.com

1 Only
2 and 3 Only
Page 17

Facebook Group: Indian Administrative Service (Raz Kr)

c.
d.

3 Only
All Three

67. The combined flora and fauna of a


region is called as
a. Ecotherm
b. Biota
c. Torpor
d. Hugosphere
68. It is argued by some scholars that the
separate electorates for Muslims,
created by the colonial government
crucially shaped the nature of
communal politics in India. In relation
to the Separate electorates, consider the
following:
1. With the Poona pact of 1916, the

Congress and the Muslim League


reached an agreement whereby the
Congress
accepted
separate
electorates.
2. The Indian Councils Act of 1909
provided separate electorates for the
Muslims .
Which of the above statement/s is/are
correct?
a.
b.
c.
d.

1 only
2 only
Both
None

69. The Manusmrti is one of the bestknown legal texts of early India, written
in Sanskrit and compiled between
second century BCE and second century
CE. Consider the following about
Manusmrti:
1. It gives equal rights to women on

the paternal property and resources


2. It strongly opposes the Varna
system.
http://insightsonindia.com

Which of the above is/are incorrect?


a.
b.
c.
d.

1 Only
2 Only
Both
None

70. Consider the following


about macroeconomics

statements

1. Macroeconomics deals with the


aggregate economic variables of an
economy.
2. It takes into account various
interlinkages which may exist
between the different sectors of an
economy.
3. Macroeconomics emerged as a
separate subject in the 1930s due to
Keynes.
Which of the above statements is/are
incorrect?
a.
b.
c.
d.

1 and 2 Only
3 Only
2 and 3 Only
None

71. Which of the following is implied by


Advisory jurisdiction of the Supreme
Court?
1. The President of India can refer any
matter that is of public importance
or that which involves interpretation
of Constitution to Supreme Court
for advice.
2. The advice of Supreme Court is
binding on the President.
3. This jurisdiction of the Supreme
Court may prevent unnecessary
litigations.
Page 18

Facebook Group: Indian Administrative Service (Raz Kr)

4. The President is not bound to accept


such an advice.
Select the correct code.
a)
b)
c)
d)

2 and 3 Only
1 and 4 Only
1, 3 and 4 Only
1, 2, 3 and 4

72. Consider the following statements


about sea-floor spreading concept
1. Constant eruptions at the crest of
oceanic ridges cause the rupture
of the oceanic crust and the new
lava wedges into it, pushing the
oceanic crust on either side. The
ocean floor, thus spreads.
2. The ocean floor that gets pushed
due to volcanic eruptions at the
crest, sinks down at the oceanic
trenches and gets consumed.
Which of the above statements is/are
correct?
a.
b.
c.
d.

1 Only
2 Only
Both
None

73. The MoEF has recently decided to


regulate developmental activities in
Western Ghats. Consider the following
in this regard:
1. The Gram Sabhas will have a role in
categorising the ecologically sensitive
areas where certain activities will be
regulated.
2. Only Red Category industries have
been banned.
http://insightsonindia.com

3. The Ministry has decided to


constitute
a
Western
Ghats
Development
and
Regulation
authority (WGDRA) to examine
developmental proposals and grant
clearance to projects in ESAs.

Which of the following is/are true in this


connection?
a.
b.
c.
d.

Only 2
2 and 3
All of the above
1 and 2

74. Consider the following statements


about the Coins issued in the ancient
India:
1. The earliest coins recovered from
excavations in India had symbols
punch-marked on them and were
made of silver and copper.
2. The first gold coins were issued in
India by the Guptas.
Which of the above is/are correct?
a.
b.
c.
d.

1 Only
2 Only
Both
None

75. Which of the following is incorrectly


matched?
a. Cocos plate : Between Central
America and Pacific plate
b. Nazca plate : Between North
America and Pacific plate

Page 19

Facebook Group: Indian Administrative Service (Raz Kr)

c. Arabian plate : Mostly the Saudi


Arabian landmass
d. Philippine plate : Between the
Asiatic and Pacific plate
76. Consider the following statements
about inventories and investments
1. In economics, the stock of unsold
finished goods, or semi-finished
goods, or raw materials which a
firm carries from one year to the
next is called inventory.
2. Inventories are treated as capital.
Addition to the stock of capital
of a firm is known as investment.
3. The addition to the machinery,
factory buildings, and equipments
employed by the firms is called as
the fixed business investment.
Which of the above statements is/are
correct?
a.
b.
c.
d.

1 and 2 Only
2 and 3 Only
3 Only
All Three

77. Consider the


about GDP

following

statements

1. It is a Sum total of gross value added


of all the firms in the economy.
2. If we calculate the GDP by looking
at the demand side of the products,
then it is referred to as the
expenditure method.
Which of the above statement is/are
correct?
a.
b.
c.
d.

1 Only
2 Only
Both
None

http://insightsonindia.com

78. Which of the following is true about


Deccan
Traps?
a. It is a large carbon deposit
located in Deccan plateau.
b. It is a large volcanic deposit
located in Deccan plateau of
west central India.
c. It is a large iron ore deposits
located in Deccan plateau of
west central India.
d. It is a large diamond deposit
located in Deccan plateau.
79. Which of the following was/were the
important agreements signed at the recent
Warsaw climate conference?
1. REDD+
2. Global e-sustainability Initiative
agreement
3. Loss and damage compensation
a. Only 1
b. 1 and 3
c. Only 3
d. All of the above
80. The Sanchi stupa is among the best
preserved ancient stupas. Consider the
following about the Sanchi stupa:
1. It was Emperor Ashoka who

commissioned the design of


Sanchi Stupa in the third century
BC.
2. The stupa has wonderful carvings
which interpret the life of
Buddha
and
his
various
incarnations from the Jataka tales.
3. Sanchi stupa is a UNESCO world
heritage site.
which of the above is/are correct ?
Page 20

Facebook Group: Indian Administrative Service (Raz Kr)

a.
b.
c.
d.

1. The scheme envisages the


development of brown
field and green field
projects in the food
processing sector in India.
2. The scheme addresses
environmental issues also.
3. It would encourage R&D
in the textile processing
sector.

1 and 2 Only
1 and 3 Only
2 and 3 Only
All the above

81. Consider the following statements


about Plastic Parks scheme:
1. Plastic Parks is a scheme which
envisages a cluster of plastic industry
where plastic reprocessing, recycling
plant and waste management system
is set up.
2. It will be set up through PPP model.

Select the correct answer using the codes


below.
a.
b.
c.
d.

Which of the following is/are true?


a) Only 2
b) Only 1
c) Both
d) None of the above

84.

Only 1
2 and 3
1 and 2
None of the above

Consider the following statements,

1. Net National Product at factor cost


is nothing but National Income.
2. To get NNP at factor cost we have
to deduct subsidies to the NNP
evaluated at market prices.

1. If we deduct depreciation
from GNP the measure of
aggregate income that we
obtain is called Net
National Product (NNP).
2. s Gross National Product
(GNP) is nothing but GDP
+ Net factor income from
abroad

Which of the above statements is/are


correct?

Which of the following statements is/are


correct?

82.

Consider the following statements

a.
b.
c.
d.
83.

1 Only
2 Only
Both
None

The Government has recently approved


the launching of a new Integrated
Processing
Development
Scheme
(IPCS). Which of the following is/are
correct in this connection?

http://insightsonindia.com

a.
b.
c.
d.
85.

1 Only
2 Only
Both
None

Consider the following statements with


respect to the jurisdiction of the
Supreme Court:
1. The government wanted to know if
it can pass a law about the
citizenship status of residents of
Page 21

Facebook Group: Indian Administrative Service (Raz Kr)

Pakistan occupied areas of Jammu


and Kashmir, comes under original
jurisdiction of the Supreme Court
2. In order to resolve the dispute about
river Cauvery the government of
Tamil Nadu wants to approach the
court, comes under appellate
jurisdiction of the Supreme Court
3. Court rejected the appeal by people
against the eviction from the dam
site, comes under writ jurisdiction of
the Supreme Court.

a.
b.
c.
d.
87.

86.

2 Only
1 and 2 Only
1, 2 and 3 Only
None

Consider the following statements


regarding measuring national income,
1. The
expenditure
approach measures
national income as total
spending on final goods and
services produced within
nation during an year.
2. The
expenditure
on
second hand goods should
not be included as they do
not contribute to the
current years production
of goods.
3. Expenditure on purchase
of
old
shares
and
bonds is included
in
computing national income

Which of the statements above is/are


correct?
http://insightsonindia.com

Consider the following statements


with respect to the youth stage of a
river
1. Streams are few during this stage
with poor integration and flow
over original slopes showing
shallow V-shaped valleys with no
floodplains or with very narrow
floodplains along trunk streams.
2. Streams divides are broad and flat
with marshes, swamp and lakes.
3. Waterfalls and rapids disappear.

Which of the above statements is/are


correct?
a.
b.
c.
d.

1 Only
2 and 3 Only
1 and 2 Only
All Three

Which of the above statements is/are


correct?
a.
b.
c.
d.
88.

1 and 2 Only
2 and 3 Only
A Only
All Three

Consider the features of limestone


caves
1. Stalactites hang as icicles of
different diameters.
2. Stalagmites rise up from the floor
of the caves.

Which of the above is/are incorrect?


a.
b.
c.
d.

1 Only
2 Only
Both
None

Page 22

Facebook Group: Indian Administrative Service (Raz Kr)

89.

90.

Western Himachal Pradesh (around


Dharamshala) and Kashmir Valley in
the Himalayan region and the
Kuchchh (Gujarat) fall under which
earthquake zone of India?
a. Very high damage risk zone
b. High damage risk zone
c. Moderate damage risk zone
d. Low damage risk zone

Article 368 provides for the procedure


for amendment of the Constitution.
With regard to amendment, consider
the following:
1. All
amendments
to
the
Constitution are initiated only in
the Parliament.
2. The Constitution Commission is
required
to
amend
the
Constitution.
3. After the passage of the
amendment bill in the Parliament
and in some cases, in State
legislatures,
referendum
is
required for ratification of the
amendment.
4. Sovereignty
of
elected
representatives is the basis of the
amendment procedure.
Which of the above statements is/are
correct?
a.
b.
c.
d.

91.

Indian
system
smooth
of the

1 and 4 Only
1 and 2 Only
2 and 3 Only
1, 2 and 4 Only
Constitution has adopted a
of Checks and Balance for
functioning of the three organs
government. Which of the

http://insightsonindia.com

following are
parliamentary
executive?

the instruments of
control
over
the

1. No Confidence Motion
2. Both Zero Hour and Half-anhour discussion
3. Comptroller and Auditor General
4. Amendment Provisions
5. Public Accounts Committee
6. Emergency provisions
Select the correct code.
a.
b.
c.
d.

1, 3, 4 and 5 Only
1, 2, 3 and 5 Only
1, 2, 4, 5 and 6 Only
1, 2, 3,4, 5 and 6

92.

Consider the following statements


about tsunamis
1. The speed of wave in the ocean
depends upon the depth of water.
It is more in the shallow water
than in the ocean deep.
2. As a result of this, the impact of
tsunami is less over the ocean and
more near the coast where they
cause large-scale devastations
3. Over deep water the tsunami has
very short wave-length and large
wave-height.

Which of the above statements is/are


correct?
a.
b.
c.
d.

1 Only
2 and 3 Only
1 and 2 Only
All Three

Page 23

Facebook Group: Indian Administrative Service (Raz Kr)

93.

Consider the following statements:

1. The Rajya Sabha cannot initiate,


reject or amend money bills.
2. The President cannot send back an
amendment bill for reconsideration
of the Parliament.
3. Fundamental Duties are known as
instrument of instructions in Indian
administration.
4. The President does not enjoy
constitutional discretion.
Which of the statements given above
is/are correct?

95.

In which of the following cases are


the powers of the Lok Sabha and
Rajya Sabha co-equal?

1. Election and impeachment of the


President
2. Approval of ordinances issued by the
President
3. Removal of Judges of Supreme
Court and High Court
4. Approval of proclamation of all
three types of emergencies
Select the correct code.

a.
b.
c.
d.

2 and 3 Only
1 and 2 Only
1, 2 and 4 Only
1, 2, 3 and 4 Only

a)
b)
c)
d)

2 and 4 Only
1 and 3 Only
1, 2 and 3 Only
1, 2, 3 and 4 Only

94.

Consider the following statements,

96.

Steel sheets used for making


automobiles and copper used for
making utensils is an example of

1. The monetary value of an


asset decreases over time
due to use, wear and tear
or
obsolescence.
This
decrease is measured as
depreciation.
2. Depreciation is the cost of
the good divided by
number of years of its
useful life.
Which of the above statements is/are
correct?
a.
b.
c.
d.

1 Only
2 Only
Both
None

a. Intermediate goods
b. Consumer durable
c. Capital goods
d. Consumer goods
97. Consider the following statements
about a capitalist economy
1. Production takes place for selling
the output in the market
2. There is sale and purchase of
labour services at a price which is
called the wage rate
Which of the above statements is/are
characteristics of a capitalist economy?
a. 1 Only
b. 2 Only

http://insightsonindia.com

Page 24

Facebook Group: Indian Administrative Service (Raz Kr)

c. Both
d. None
98.

Consider the following statements


about tropical cyclone
1. The centre of the
cyclone is mostly a
warm
and
highpressure
cloudless
core known as eye of
the storm.
2. Tropical cyclones are
characterized by large
pressure gradients
3. In case of the Bay of
Bengal,
cyclones
mostly develop during
the
months
of
October
and
November.

Which of the above statements is/are


correct?
a.
b.
c.
d.
99.

1 and 2 Only
2 and 3 Only
1 and 3 Only
All Three

Consider the following statements


related to biodiversity

2. Those species that spread widely


and create significant problems
are known as invasive species.
3. Most of the exotic species are
harmless and beneficial to the
ecosystem as they introduce
diversity in the region
Which of the above statements is/are
correct?
a.
b.
c.
d.

1 and 2 Only
2 and 3 Only
1 Only
All Three

Consider the following


statements with reference to genetic
diversity

100.

1. Humans differ in height, color


and physical appearance because
of genetic diversity
2. Genetic diversity refers to the
variation of genes within species
3. Genetic diversity is essential for a
healthy breeding of population of
species.
Which of the above statements is/are
correct?
a.
b.
c.
d.

1 and 2 Only
2 and 3 Only
3 Only
All Three

1. Species which are not the natural


inhabitants of the local habitat
but are introduced into the
system, are called exotic species.

http://insightsonindia.com

Page 25

Facebook Group: Indian Administrative Service (Raz Kr)

INSIGHTS MOCK PRELIMS TEST 5 - SOLUTIONS


1. Solution: d)
Factors affecting ocean salinity are mentioned below:
(i) The salinity of water in the surface layer of oceans depend mainly on evaporation and
precipitation.
(ii) Surface salinity is greatly influenced in coastal regions by the fresh water flow from rivers, and
in polar regions by the processes of freezing and thawing of ice.
(iii) Wind, also influences salinity of an area by transferring water to other areas.
(iv) The ocean currents contribute to the salinity variations. Salinity, temperature and density of
water are interrelated. Hence, any change in the temperature or density influences the salinity of
an area.
2. Solution: c)
Macroeconomics, as a separate branch of economics, emerged after the British economist John
Maynard Keynes published his celebrated book The General Theory of Employment, Interest and
Money in 1936. The dominant thinking in economics before Keynes was that all the labourers
who are ready to work will find employment and all the factories will be working at their full
capacity. This school of thought is known as the classical tradition. However, the Great
Depression of 1929 and the subsequent years saw the output and employment levels in the
countries of Europe and North America fall by huge amounts. It affected other countries of the
world as well. Demand for goods in the market was low, many factories were lying idle, workers
were thrown out of jobs. In USA, from 1929 to 1933, unemployment rate rose from 3 per cent to
25 per cent (unemployment rate may be defined as the number of people who are not working
and are looking for jobs divided by the total number of people who are working or looking for
jobs). Over the same period aggregate output in USA fell by about 33 per cent. These events
made economists think about the functioning of the economy in a new way. The fact that the
economy may have long lasting unemployment had to be theorised about and explained. Keynes
book was an attempt in this direction. Unlike his predecessors, his approach was to examine the
working of the economy in its entirety and examine the interdependence of the different sectors.
The subject of macroeconomics was born.
3. Solution: c)
Why does the earth shake?
The release of energy occurs along a fault. A fault is a sharp break in the crustal rocks. Rocks
along a fault tend to move in opposite directions. As the overlying rock strata press them, the
friction locks them together. However, their tendency to move apart at some point of time
overcomes the friction. As a result, the blocks get deformed and eventually, they slide past one
another abruptly. This causes a release of energy, and the energy waves travel in all directions.
http://insightsonindia.com

INSIGHTS

Page 1

Facebook Group: Indian Administrative Service (Raz Kr)

INSIGHTS MOCK PRELIMS TEST 5 - SOLUTIONS


The point where the energy is released is called the focus of an earthquake, alternatively, it is
called the hypocentre. The energy waves travelling in different directions reach the surface. The
point on the surface, nearest to the focus, is called epicentre. It is the first one to experience the
waves. It is a point directly above the focus.

4. Solution (b)
Most Ashokan inscriptions were in the Prakrit language
Reference: Page No.32(THEMES IN INDIAN HISTORY PART I)
5. Solution: A
A National Environmental Appraisal and Monitoring Authority will be created before March 31,
2014 following a Supreme Court order on Monday. The industry would be soon queuing up
before a new autonomous agency to seek environment clearances under the Environment
Protection Act, 1986.
Under the Environment Protection Act, 1986 almost all kind of projects require environment
clearance at the state or central level. These are at present processed for recommendation by state
or central level appraisal committees and then cleared by the state or central environment
ministries.
The projects coming up in forest areas require a forest clearance too under the Forest
Conservation Act, 1980. The Supreme Courts orders on Monday would not take away the
powers of the Ministrys forest bureaucracy to clear such proposals but the implementation of the
forest policy would come under the new authoritys domain.
The proposal is to make it operationally independent from the government.

6. Solution: b)
Wegener suggested that the movement responsible for the drifting of the continents was caused
by pole-fleeing force and tidal force. The polar-fleeing force relates to the rotation of the earth.
You are aware of the fact that the earth is not a perfect sphere; it has a bulge at the equator. This
bulge is due to the rotation of the earth. The second force that was suggested by Wegenerthe
tidal forceis due to the attraction of the moon and the sun that develops tides in oceanic
waters. Wegener believed that these forces would become effective when applied over many
million years. However, most of scholars considered these forces to be totally inadequate.
7. Solution (d)

http://insightsonindia.com

INSIGHTS

Page 2

Facebook Group: Indian Administrative Service (Raz Kr)

INSIGHTS MOCK PRELIMS TEST 5 - SOLUTIONS


Powers of Rajya Sabha: Considers and approves non money bills and suggests amendments to
money bills. Approves constitutional amendments. Exercises control over executive by asking
questions, introducing motions and resolutions. It can alone initiate the procedure for removal
of Vice President. It can give the Union parliament power to make laws on matters included
in the State list.
Reference : Page 109(Indian Constitution at Work Class XI)
8. Solution: c)
Of the final goods, we can distinguish between consumption goods and capital goods. Goods like
food and clothing, and services like recreation that are consumed when purchased by their
ultimate consumers are called consumption goods or consumer goods. (This also includes services
which are consumed but for convenience we may refer to them as consumer goods.)
Then there are other goods that are of durable character which are used in the production
process. These are tools, implements and machines. While they make production of other
commodities feasible, they themselves dont get transformed in the production process. They are
also final goods yet they are not final goods to be ultimately consumed. Unlike the final goods
that we have considered above, they are the crucial backbone of any production process, in aiding
and enabling the production to take place. These goods form a part of capital, one of the crucial
factors of production in which a productive enterprise has invested, and they continue to enable
the production process to go on for continuous cycles of production. These are capital goods and
they gradually undergo wear and tear, and thus are repaired or gradually replaced over time.
9. Solution: C
The Central Government, by a notification dated 20.2.2009, as set up National GangaRiver Basin
Authority (NGRBA)as an empowered planning, financing, monitoring and coordinating authority
for the Ganga River, in exercise of the powers conferred under the Environment (Protection)
Act,1986. The Prime Minister is ex-officio Chairperson of the Authority, and it has as its
members, the Union Ministers Concerned and the Chief Ministers of states through which Ganga
flows, viz., Uttarakhand, Uttar Pradesh, Bihar, Jharkhand and West Bengal, among others. The
objective of the Authority is to ensure effective abatement of pollution and conservation of the
river Ganga by adopting a holistic approach with the river basin as the unit of planning. The
functions of the Authority include all measures necessary for planning and execution of
programmes for abatement of pollution in the Ganga in keeping with sustainable development
needs. It is supported by the World Bank.
Functions of NGRBA
* The NGRBA would be responsible for addressing the problem of pollution in Ganga in a
holistic and comprehensive manner. This will include water quality, minimum ecological flows,
sustainable access and other issues relevant to river ecology and management.
http://insightsonindia.com

INSIGHTS

Page 3

Facebook Group: Indian Administrative Service (Raz Kr)

INSIGHTS MOCK PRELIMS TEST 5 - SOLUTIONS


* The NGRBA will not only be regulatory body but will also have developmental role in terms
of planning & monitoring of the river conservation activities and ensuring that necessary
resources are available.
* The NRGBA would work for maintaining the water quality of the river Ganga upto
acceptable standards. The pollution abetment activities will be taken through the existing
implementation mechanisms in the State and also through special Purpose Vehicles (SPVs0 at
the pollution hotspots.
* The NGRBA will ensure minimum ecological flow in the Ganga by regulating water
abstraction and by promoting water storage projects.
* The NGRBA will plan and monitor programmes for cleaning of Ganga and its tributaries. To
being with, it will concentrate on Ganga main stream.

10. Solution (d)


The chief instrument through which judicial activism has flourished in India is Public Interest
Litigation (PIL) or Social Action Litigation (SAL). PIL has become the most important vehicle of
judicial activism. It has expanded the scope of Article 21. In the first place it has overburdened
the courts.
A PIL may be introduced in a court of law by the court itself (suo motu), rather than the
aggrieved party or another third party.
A Public Interest Litigation (PIL) can be filed in any High Court or directly.
Reference : Pages 135, 136 (Indian Constitution at Work Class XI)

11. Solution: b)
Types of Droughts
Meteorological Drought : It is a situation when there is a prolonged period of inadequate rainfall
marked with mal-distribution of the same over time and space.
Agricultural Drought : It is also known as soil moisture drought, characterised by low soil
moisture that is necessary to support the crops, thereby resulting in crop failures. Moreover, if an
area has more than 30 per cent of its gross cropped area under irrigation, the area is excluded
from the drought-prone category.
Hydrological Drought : It results when the availability of water in different storages and
reservoirs like aquifers, lakes, reservoirs, etc. falls below what the precipitation can replenish.
Ecological Drought : When the productivity of a natural ecosystem fails due to shortage of water
and as a consequence of ecological distress, damages are induced in the ecosystem. Various parts

http://insightsonindia.com

INSIGHTS

Page 4

Facebook Group: Indian Administrative Service (Raz Kr)

INSIGHTS MOCK PRELIMS TEST 5 - SOLUTIONS


of India experience these droughts recurrently which result in some serious socio-economic and
ecological problems.
12. Solution: d)
Apart from GDP deflator, there is another way to measure change of prices in an economy which
is known as the Consumer Price Index (CPI). This is the index of prices of a given basket of
commodities which are bought by the representative consumer. CPI is generally expressed in
percentage terms. We have two years under consideration one is the base year, the other is the
current year. We calculate the cost of purchase of a given basket of commodities in the base year.
We also calculate the cost of purchase of the same basket in the current year. Then we express
the latter as a percentage of the former. This gives us the Consumer Price Index of the current
year vis-a-vis the base year. For example let us take an economy which produces two goods, rice
and cloth. A representative consumer buys 90 kg of rice and 5 pieces of cloth in a year. Suppose
in the year 2000 the price of a kg of rice was Rs 10 and a piece of cloth was Rs 100. So the
consumer had to spend a total sum of Rs 10 90 = Rs 900 on rice in 2000. Similarly, she spent
Rs 100 5 = Rs 500 per year on cloth. Summation of the two items is, Rs 900 + Rs 500 = Rs
1,400.
13. Solution (b)
Most of the inscriptions mentioned the king referred to as Piyadassi meaning pleasant to
behold; there were a few inscriptions which also referred to the king as Asoka.
Asoka was the first ruler who inscribed his messages to his subjects and officials on stone surfaces
natural rocks as well as polished pillars. He used the inscriptions to proclaim what he
understood to be dhamma.
Reference: Page No.32, 47 (THEMES IN INDIAN HISTORY PART I)
14. Solution: b)
Clouds
Cloud is a mass of minute water droplets or tiny crystals of ice formed by the condensation of the
water vapour in free air at considerable elevations. As the clouds are formed at some height over
the surface of the earth, they take various shapes. According to their height, expanse, density and
transparency or opaqueness clouds are grouped under four
types : (i) cirrus; (ii) cumulus; (iii) stratus; (iv) nimbus.
Cirrus
Cirrus clouds are formed at high altitudes (8,000 - 12,000m). They are thin and detatched clouds
having a feathery appearance. They are always white in colour.
Cumulus
http://insightsonindia.com

INSIGHTS

Page 5

Facebook Group: Indian Administrative Service (Raz Kr)

INSIGHTS MOCK PRELIMS TEST 5 - SOLUTIONS


Cumulus clouds look like cotton wool. They are generally formed at a height of 4,000 - 7,000 m.
They exist in patches and can be seen scattered here and there. They have a flat base.
Stratus
As their name implies, these are layered clouds covering large portions of the sky. These clouds
are generally formed either due to loss of heat or the mixing of air masses with different
temperatures.
Nimbus
Nimbus clouds are black or dark gray. They form at middle levels or very near to the surface of
the earth. These are extremely dense and opaque to the rays of the sun. Sometimes, the clouds
are so low that they seem to touch the ground. Nimbus clouds are shapeless masses of thick
vapour.
15. Solution (c)

Sikandar Hayat Khan, Punjab Premier and leader of the Unionist Party, who had drafted the
resolution, declared in a Punjab assembly speech on 1 March 1941 that he was opposed to a
Pakistan that would mean Muslim Raj here and Hindu Raj elsewhere ... If Pakistan means
unalloyed Muslim Raj in the Punjab then I will have nothing to do with it. Also the party
remained consistently loyal to the British.
Reference: Page No.386, 388(THEMES IN INDIAN HISTORY PART III)
16. Solution (a)
What is an urban area? It is very easy to identify a big city like Mumbai or Kolkata, but it is not
so easy to say this about some very small urban areas that are somewhere between a village and a
town.
The Census of India defines an urban area as having:
(i) a minimum population of 5000; (ii) at least 75 per cent of male working population engaged
in non agricultural occupations and (iii) a density of population of at least 400 persons per sq. km.
As per the 2001 census, nearly 28% of Indias population lives in urban areas.
Reference : Page 187 (Indian Constitution at Work Class XI)
17. Solution: A
As per the Forest (Conservation) Act 1980, as far as possible, the non-forest land for
Compensatory Afforestation (CA) was to be identified contiguous to or in the proximity of
Reserved Forest or Protected Forest. In case, non-forest land of CA was not available in the same
district, non-forest land for CA was to be identified anywhere else in the State/Union Territory.
If non forest land was unavailable in the entire State/ UT, funds for raising CA in double the area
in extent of the forestland diverted had to be provided by the user agency. The non-availability of
suitable non-forest land for CA in the State / Union Territory would be accepted by the Central
http://insightsonindia.com

INSIGHTS

Page 6

Facebook Group: Indian Administrative Service (Raz Kr)

INSIGHTS MOCK PRELIMS TEST 5 - SOLUTIONS


Government only on the Certificate of the Chief Secretary to the State/Union Territory
Government to that effect.
Based on the recommendations of the CEC, the Supreme Court of India in October 2002 directed
the creation of a Compensatory Afforestation Fundin which all the monies received from the
user agencies towards compensatory afforestation, additional compensatory afforestation, penal
compensatory afforestation, net present value (NPV) of forest land, Catchment Area Treatment
Plan Funds, etc. were to be deposited.

18. Solution: a)
Distance from the sea : Another factor that influences the temperature is the location of a place
with respect to the sea. Compared to land, the sea gets heated slowly and loses heat slowly. Land
heats up and cools down quickly. Therefore, the variation in temperature over the sea is less
compared to land. The places situated near the sea come under the moderating influence of the
sea and land breezes which moderate the temperature.
19. Solution (a)
Swami Dayananda Saraswati founded of Arya Samaj in 1875. The Shuddhi professed by Arya
Samajists angered the muslims. This can also be said to be one of the causes of rise of
Communalism in India post 1920s.
Reference: Page No.383, Internet(THEMES IN INDIAN HISTORY PART III)
20. Solution: c)
Can the GDP of a country be taken as an index of the welfare of the people of that country? If a
person has more income he or she can buy more goods and services and his or her material wellbeing improves. So it may seem reasonable to treat his or her income level as his or her level of
well-being. GDP is the sum total of value of goods and services created within the geographical
boundary of a country in a particular year. It gets distributed among the people as incomes
(except for retained earnings). So we may be tempted to treat higher level of GDP of a country as
an index of greater well-being of the people of that country (to account for price changes, we
may take the value of real GDP instead of nominal GDP). But there are at least three reasons why
this may not be correct
1. Distribution of GDP how uniform is it: If the GDP of the country is rising, the welfare may
not rise as a consequence. This is because the rise in GDP may be concentrated in the hands of
very few individuals or firms. For the rest, the income may in fact have fallen. In such a case the
welfare of the entire country cannot be said to have increased. For example, suppose in year
2000, an imaginary country had 100 individuals each earning Rs 10. Therefore the GDP of the
country was Rs 1,000 (by income method). In 2001, let us suppose the same country had 90
individuals earning Rs 9 each, and the rest 10 individual earning Rs 20 each. Suppose there had
http://insightsonindia.com

INSIGHTS

Page 7

Facebook Group: Indian Administrative Service (Raz Kr)

INSIGHTS MOCK PRELIMS TEST 5 - SOLUTIONS


been no change in the prices of goods and services between these two periods. The GDP of the
country in the year 2001 was 90 (Rs 9) + 10 (Rs 20) = Rs 810 + Rs 200 = Rs 1,010. Observe
that compared to 2000, the GDP of the country in 2001 was higher by Rs10. But this has
happened when 90 per cent of people of the country have seen a drop in their real income by 10
per cent (from Rs 10 to Rs 9), whereas only 10 per cent have benefited by a rise in their income
by 100 per cent (from Rs 10 to Rs 20). 90 per cent of the people are worse off though the GDP
of the country has gone up. If we relate welfare improvement in the country to the percentage of
people who are better off, then surely GDP is not a good index.
2. Non-monetary exchanges: Many activities in an economy are not evaluated in monetary terms.
For example, the domestic services women perform at home are not paid for. The exchanges
which take place in the informal sector without the help of money are called barter exchanges. In
barter exchanges goods (or services) are directly exchanged against each other. But since money is
not being used here, these exchanges are not registered as part of economic activity. In developing
countries, where many remote regions are underdeveloped, these kinds of exchanges do take
place, but they are generally not counted in the GDPs of these countries. This is a case of
underestimation of GDP. Hence GDP calculated in the standard manner may not give us a clear
indication of the productive activity and well-being of a country.
3. Externalities: Externalities refer to the benefits (or harms) a firm or an individual causes to
another for which they are not paid (or penalised). Externalities do not have any market in which
they can be bought and sold. For example, let us suppose there is an oil refinery which refines
crude petroleum and sells it in the market. The output of the refinery is the amount of oil it
refines. We can estimate the value added of the refinery by deducting the value of intermediate
goods used by the refinery (crude oil in this case) from the value of its output. The value added
of the refinery will be counted as part of the GDP of the economy. But in carrying out the
production the refinery may also be polluting the nearby river. This may cause harm to the
people who use the water of the river. Hence their utility will fall. Pollution may also kill fish or
other organisms of the river on which fish survive. As a result the fishermen of the river may be
losing their income and utility. Such harmful effects that the refinery is inflicting on others, for
which it does not have to bear any cost, are called externalities. In this case, the GDP is not taking
into account such negative externalities. Therefore, if we take GDP as a measure of welfare of the
economy we shall be overestimating the actual welfare. This was an example of negative
externality. There can be cases of positive externalities as well. In such cases GDP will
underestimate the actual welfare of the economy.
21. Solution (c)
The structure of the judiciary in India is pyramidal with the Supreme Court at the top, High
Courts below them and district and subordinate courts at the lowest level (see the diagram
below). The lower courts function under the direct superintendence of the higher courts.
Supreme Court of India:
Its decisions are binding on all courts. Can transfer Judges of High Courts.
Can move cases from any court to itself.
http://insightsonindia.com

INSIGHTS

Page 8

Facebook Group: Indian Administrative Service (Raz Kr)

INSIGHTS MOCK PRELIMS TEST 5 - SOLUTIONS


It can transfer cases from one High Court to another High Court
Can deal with cases within the jurisdiction of the State.
Reference : Pages 130 (Indian Constitution at Work Class XI)
22. Solution (d)
The goal of Muslim League at the time of its creation was to define and advance Muslim agendas
& protect the position of upper class Muslims in India. The demand for separate Muslim state
came later in the 1940s. The name Pakistan or Pak-stan (from Punjab, Afghan, Kashmir, Sind and
Baluchistan) was coined by a Punjabi Muslim student at Cambridge, Choudhry Rehmat Ali, who,
in pamphlets written in 1933 and 1935, desired a separate national status for this new entity.
Reference: Page No.386,Wikipedia(THEMES IN INDIAN HISTORY PART III)
23. Solution: C
Many plant communities are not self-sustaining. A field in the temperate deciduous forest
biome will remain a field only as long as it is grazed by animals or mowed regularly. If these
factors are eliminated, the balance tips in favor of other species. The newcomers will, in turn,
establish conditions that no longer favor them but promote the growth of still other species. The
revered naturalist and writer, Henry David Thoreau, called this process succession.
Primary Plant Succession
The process of plant succession begins just as soon as a land area capable of supporting plant life
is formed. Some examples:
accumulation of sand dunes at the edge of the ocean or a lake
cooling of a lava flow
exposure of rock by a retreating glacier
Secondary Plant Succession
Lumbering, farming, fires, and hurricanes interrupt the process of succession by removing the
dominant plants in the community. Their elimination sets the stage for a new succession to begin.
The many abandoned farms in New England (I live on one) illustrate this. People often wonder
why our pioneers built stone walls through the woods. The answer is that they did not. The walls
in the woods today once marked the boundaries of fields and pastures, but when cultivation and
grazing ceased, a secondary succession began. Where I live,
the grass of abandoned fields soon became invaded by low-growing, herbaceous species
like plantains, and golden rods.

http://insightsonindia.com

INSIGHTS

Page 9

Facebook Group: Indian Administrative Service (Raz Kr)

INSIGHTS MOCK PRELIMS TEST 5 - SOLUTIONS


These were quickly followed by woody shrubs like the common juniper, blueberries and
gray-stemmed dogwood
Soon sun-loving gray birch, poplars, and Eastern red cedar ("old-field cedar") became
established.
White pines or, in sandy well-drained locations, oaks have followed.
These will persist until some disturbance such as fire, hurricanes, or lumbering open up
the land, and the process of secondary succession begins again.
In general, plant succession is a reflection of the increasing efficiency of the community at
intercepting the energy of the sun and converting it into chemical energy. As one stage of
succession follows another,
the biomass of the community increases. This is the outcome of the increasing amount of
net productivity calories stored by the plant community.
This, in turn, provides calories for a larger community of consumers.
As succession continues, the diversity of species in the community increases at least for
a time.
When the system approaches its climax, the rate of increase in net productivity of the
plants is consumed by its own heterotrophs.
The system comes into equilibrium and reaches peak efficiency at channeling the energy
of the sun into the food web of the community.

24. Solution (d)


Reference: Page No.154(THEMES IN INDIAN HISTORY PART II)
25. Solution: c)
Consequences of Tropical Cyclones
The energy to the tropical cyclone comes from the latent heat released by the warm moist air.
Hence, with the increase in distance from the sea, the force of the cyclone decreases. In India, the
force of the cyclone decreases with increase in distance from the Bay of Bengal and the Arabian
Sea. So, the coastal areas are often struck by severe cyclonic storms with an average velocity of
180 km/h. Often, this results in abnormal rise in the sea level known as Storm Surge.

http://insightsonindia.com

INSIGHTS

Page 10

Facebook Group: Indian Administrative Service (Raz Kr)

INSIGHTS MOCK PRELIMS TEST 5 - SOLUTIONS


26. Solution (c)
Dalhousie followed the policy of Annexation through Doctrine of Lapse. Lord Dalhousies
annexations created disaffection in all the areas and principalities that were annexed. Also Awadh
was annexed under the pretext of maladministration.
Note: Read in detail about various reforms and policies of Dalhousie
Reference: Page No.296(THEMES IN INDIAN HISTORY PART III), Internet.
27. Solution: B
Throughout the study of geography, there have been a number of different approaches to
explaining the development of the world's societies and cultures. One that received much
prominence in geographic history but has declined in recent decades of academic study is
environmental determinism.
Environmental determinism is the belief that the environment (most notably its physical factors
such as landforms and/or climate) determines the patterns of human culture and societal
development. Environmental determinists believe that it is these environmental, climatic, and
geographical factors alone that are responsible for human cultures and individual decisions and/or
social conditions have virtually no impact on cultural development.
The main argument of environmental determinism states that an area's physical characteristics
like climate have a strong impact on the psychological outlook of its inhabitants. These varied
outlooks then spread throughout a population and help define the overall behavior and culture of
a society. For instance it was said that areas in the tropics were less developed than higher
latitudes because the continuously warm weather there made it easier to survive and thus, people
living there did not work as hard to ensure their survival.
Another example of environmental determinism would be the theory that island nations have
unique cultural traits solely because of their isolation from continental societies.
Possibilism in cultural geography is the theory that the environment sets certain constraints or
limitations, but culture is otherwise determined by social conditions. In Cultural ecology Marshall
Sahlins used this concept in order to develop alternative approaches to the environmental
determinism dominant at that time in ecological studies. Theory by Strabo in 64 BC that we,
humans, can make things happen by our own intelligence over time. Strabo cautioned against the
assumption that nature and actions of humans were determined by the physical environment
they inhabited. He observed that humans were the active elements in a human-environmental
partnership.
The controversy between geographical possibilism and determinism might be considered as one
of (at least) three dominant epistemologic controversies of contemporary geography. The other
two controversies are the "debate between neopositivists and neokantians about the

http://insightsonindia.com

INSIGHTS

Page 11

Facebook Group: Indian Administrative Service (Raz Kr)

INSIGHTS MOCK PRELIMS TEST 5 - SOLUTIONS


"exceptionalism" or the specificity of geography as a science the contention
between Mackinder and Kropotkin about what is - or should be - geography."
Possibilism in geography is, thus, considered as a distinct approach to geographical knowledge,
directly opposed to geographical determinism.

28. Solution: (a)


District Court deals with cases arising in the District. Considers appeals on decisions given by
lower courts. Decides cases involving serious criminal offences. Subordinate Courts consider cases
of civil and criminal nature.
Decisions made by the Supreme Court are binding on all other courts within the territory of
India. Orders passed by it are enforceable throughout the length and breadth of the country. The
Supreme Court itself is not bound by its decision and can at any time review it. Besides, if there is
a case of contempt of the Supreme Court, then the Supreme Court itself decides such a case.
The Supreme Courts and High Courts can hear appeals from lower courts and exercises
superintendence and control over courts below it. Both enjoy appellate jurisdiction.
Only Supreme Court has Advisory jurisdiction.
Reference : Pages 130, 134 (Indian Constitution at Work Class XI)
29. Solution: b)
In a modern economy money consists mainly of currency notes and coins issued by the monetary
authority of the country. In India currency notes are issued by the Reserve Bank of India (RBI),
which is the monetary authority in India. However, coins are issued by the Government of India.
Apart from currency notes and coins, the balance in savings, or current account deposits, held by
the public in commercial banks is also considered money since cheques drawn on these accounts
are used to settle transactions. Such deposits are called as demand deposits. Other deposits, e.g.
fixed deposits, have a fixed period to maturity and are referred to as time deposits.

30. Solution: c)
Composite Volcanoes
These volcanoes are characterised by eruptions of cooler and more viscous lavas than basalt.
These volcanoes often result in explosive eruptions. Along with lava, large quantities of
pyroclastic material and ashes find their way to the ground. This material accumulates in the
vicinity of the vent openings leading to formation of layers, and this makes the mounts appear as
composite volcanoes.
http://insightsonindia.com

INSIGHTS

Page 12

Facebook Group: Indian Administrative Service (Raz Kr)

INSIGHTS MOCK PRELIMS TEST 5 - SOLUTIONS


31. Solution: a)
Keeping reserves is costly for banks, as, otherwise, they could lend this balance to interest earning
investment projects. However, RBI requires commercial banks to keep reserves in order to ensure
that banks have a safe cushion of assets to draw on when account holders want to be paid. RBI
uses various policy instruments to bring forth a healthy rdr in commercial banks. The first
instrument is the Cash Reserve Ratio which specifies the fraction of their deposits that banks
must keep with RBI. There is another tool called Statutory Liquidity Ratio which requires the
banks to maintain a given fraction of their total demand and time deposits in the form of
specified liquid assets. Apart from these ratios RBI uses a certain interest rate called the Bank
Rate to control the value of rdr. Commercial banks can borrow money from RBI at the bank rate
when they run short of reserves. A high bank rate makes such borrowing from RBI costly and, in
effect, encourages the commercial banks to maintain a healthy rdr.
32. Solution (c)
It was remarkable that during the uprising religious divisions between Hindus and Muslim
were hardly noticeable despite British attempts to create such divisions. In Bareilly in western
Uttar Pradesh, in December 1857, the British spent Rs 50,000 to incite the Hindu population
against the Muslims. The attempt failed. There was a widespread fear that the British were bent
on destroying the caste and religions of Hindus and Muslims and converting them to Christianity
a fear that led people to believe many of the rumours that circulated at the
time. People were urged to come together and fight to save their livelihood, their faith, their
honour, their identity a fight which was for the greater public good.
Reference: Page No.301,304(THEMES IN INDIAN HISTORY PART III)
33. Solution (d)
The Prayaga Prashasti (also known as the Allahabad Pillar Inscription) composed by Harisena.
Harisena was the court poet of Samudragupta while Banabhatta was the court poet of
Harshavardhana. The Prayaga Prashasti was composed in praise of SamudraGupta in Sanskrit.
Reference: Page No.36 (THEMES IN INDIAN HISTORY PART I)
34. Solution (a)
According to Article 370, the concurrence of the State is required for making any laws in
matters mentioned in the Union and Concurrent lists. This is different from the position of
other States
The remaining differences between the other States and the Stateof J&K are that no
emergency due to internal disturbances can be declared in J&K without the concurrence of
the State. The union government cannot impose a financial emergency in the State andthe
Directive Principles do not apply in J&K. Finally, amendments to the Indian Constitution
(under Art. 368) can only apply in concurrence with the government of J&K.
http://insightsonindia.com

INSIGHTS

Page 13

Facebook Group: Indian Administrative Service (Raz Kr)

INSIGHTS MOCK PRELIMS TEST 5 - SOLUTIONS


Reference : Pages 170, 171 (Indian Constitution at Work Class XI)
35. Solution: B
Many plant communities are not self-sustaining. A field in the temperate deciduous forest
biome will remain a field only as long as it is grazed by animals or mowed regularly. If these
factors are eliminated, the balance tips in favor of other species. The newcomers will, in turn,
establish conditions that no longer favor them but promote the growth of still other species. The
revered naturalist and writer, Henry David Thoreau, called this process succession.
Primary Plant Succession
The process of plant succession begins just as soon as a land area capable of supporting plant life
is formed. Some examples:
accumulation of sand dunes at the edge of the ocean or a lake
cooling of a lava flow
exposure of rock by a retreating glacier
Secondary Plant Succession
Lumbering, farming, fires, and hurricanes interrupt the process of succession by removing the
dominant plants in the community. Their elimination sets the stage for a new succession to begin.
The many abandoned farms in New England (I live on one) illustrate this. People often wonder
why our pioneers built stone walls through the woods. The answer is that they did not. The walls
in the woods today once marked the boundaries of fields and pastures, but when cultivation and
grazing ceased, a secondary succession began. Where I live,
the grass of abandoned fields soon became invaded by low-growing, herbaceous species
like plantains, and golden rods.
These were quickly followed by woody shrubs like the common juniper, blueberries and
gray-stemmed dogwood
Soon sun-loving gray birch, poplars, and Eastern red cedar ("old-field cedar") became
established.
White pines or, in sandy well-drained locations, oaks have followed.
These will persist until some disturbance such as fire, hurricanes, or lumbering open up
the land, and the process of secondary succession begins again.

http://insightsonindia.com

INSIGHTS

Page 14

Facebook Group: Indian Administrative Service (Raz Kr)

INSIGHTS MOCK PRELIMS TEST 5 - SOLUTIONS


In general, plant succession is a reflection of the increasing efficiency of the community at
intercepting the energy of the sun and converting it into chemical energy. As one stage of
succession follows another,
the biomass of the community increases. This is the outcome of the increasing amount of
net productivity calories stored by the plant community.
This, in turn, provides calories for a larger community of consumers.
As succession continues, the diversity of species in the community increases at least for
a time.
When the system approaches its climax, the rate of increase in net productivity of the
plants is consumed by its own heterotrophs.
The system comes into equilibrium and reaches peak efficiency at channeling the energy
of the sun into the food web of the community.

36. Solution (c)


Provisions under 73rd Amendment ActAll States now have a uniform three tier Panchayati Raj structure. At the base is the Gram
Panchayat. A Gram Panchayat covers a village or group of villages. The intermediary level is
the Mandal (also referred to as Block or Taluka). These bodies are called Mandal or Taluka
Panchayats. The intermediary level body need not be constituted in smaller States. At the
apex is the Zilla Panchayat covering the entire rural area of the District.
The amendment also made a provision for the mandatory creation of the Gram Sabha. The
Gram Sabha would comprise of all the adult members registered as voters inthe Panchayat
area. Its role and functions are decided by State legislation.
All the three levels of Panchayati Raj institutions are elected directly by the people. The term
of each Panchayat body is five years.
Reference : Pages 183 (Indian Constitution at Work Class XI)
37. Solution: a)
Refer to Page No. 18, Fundamentals of Physical Geography class XI
38. Solution (c)
None of the Buddhas speeches were written down during his lifetime. After his death ( fifthfourth century BCE) his teachings were compiled by his disciples at a council of elders or senior
monks at Vesali (Pali for Vaishali in present-day Bihar). These compilations were known as
Tipitaka literally, three baskets to hold different types of texts. They were first transmitted
orally and then written and classified according to length as well as subject matter. The Vinaya
http://insightsonindia.com

INSIGHTS

Page 15

Facebook Group: Indian Administrative Service (Raz Kr)

INSIGHTS MOCK PRELIMS TEST 5 - SOLUTIONS


Pitaka included rules and regulations for those who joined the sangha or monastic order; the
Buddhas teachings were included in the Sutta Pitaka; and the Abhidhamma Pitaka dealt with
philosophical matters.
Reference: Page No.86 (THEMES IN INDIAN HISTORY PART I)
39. Solution: C
Microplastics which are produced either for direct use, such as for industrial abrasives, exfoliants,
cosmetics or roto milling or for indirect use as precursors (so called resin pellets or nurdles) for
the production of manifold consumer products ("primary microplastics").
Approximately half of the plastic material introduced to the marine environment is buoyant, but
fouling by organisms can induce the sinking of additional plastic debris to thesea floor, where it
may interfere with sediment-dwelling species and sedimental gas exchange processes. However,
this is of more importance for larger plastic debris.
Furthermore, plastic particles may highly concentrate and transport synthetic organic compounds
(e.g. persistent organic pollutants, POPs) commonly present in the environment and ambient sea
water on their surface through adsorption. It still remains unknown if microplastics can act as
agents for the transfer of POPs from the environment to organisms in this way, but
evidence suggest this to be a potential portal for entering food webs. Of further concern, additives
added to plastics during manufacture may leach out upon ingestion, potentially causing serious
harm to the organism. Endocrine disruption by plastic additives may affect the reproductive
health of humans and wildlife alike.

40. Solution (b)


One-third Reservations for Scheduled Castes and Scheduled Tribes are also provided for at all
the three levels, in proportion to their population. If the States find it necessary, they can also
provide for reservations for the backward castes (OBCs).
The provisions of the 73rd amendment were not made applicable to the areas inhabited by
the Adivasi populations in many States of India. In 1996, a separate act was passed extending
the provisions of the Panchayat system to these areas. Many Adivasi communities have their
traditional customs of managing common resources such as forests and small water reservoirs,
etc. For this purpose, more powers are given to the Gram Sabhas of these areas and elected
village panchayats have to get the consent of the Gram Sabha in many respects. The idea
behind this act is that local traditions of self government should be protected while
introducing modern elected bodies.
A minimum age of 21 years is necessary to become a member of a Panchayat.
The power to appoint the State Finance commissioner is vested in the Governor, but the
power to remove can be done only by the President of India.
Reference : Pages 184 (Indian Constitution at Work Class XI)
http://insightsonindia.com

INSIGHTS

Page 16

Facebook Group: Indian Administrative Service (Raz Kr)

INSIGHTS MOCK PRELIMS TEST 5 - SOLUTIONS

41. Solution: a)
In our modern economic setting this flow of production arises out of production of commodities
goods and services by millions of enterprises large and small. These enterprises range from giant
corporations employing a large number of people to single entrepreneur enterprises. But what
happens to these commodities after being produced? Each producer of commodities intends to
sell her output. So from the smallest items like pins or buttons to the largest ones like aeroplanes,
automobiles, giant machinery or any saleable service like that of the doctor, the lawyer or the
financial consultant the goods and services produced are to be sold to the consumers. The
consumer may, in turn, be an individual or an enterprise and the good or service purchased by
that entity might be for final use or for use in further production. When it is used in further
production it often loses its characteristic as that specific good and is transformed through a
productive process into another good. Thus a farmer producing cotton sells it to a spinning mill
where the raw cotton undergoes transformation to yarn; the yarn is, in turn, sold to a textile mill
where, through the productive process, it is transformed into cloth; the cloth is, in turn,
transformed through another productive process into an article of clothing which is then ready to
be sold finally to the consumers for final use. Such an item that is meant for final use and will not
pass through any more stages of production or transformations is called a final good.
Why do we call this a final good? Because once it has been sold it passes out of the active
economic flow. It will not undergo any further transformation at the hands of any producer. It
may, however, undergo transformation by the action of the ultimate purchaser. In fact many such
final goods are transformed during their consumption. Thus the tea leaves purchased by the
consumer are not consumed in that form they are used to make drinkable tea, which is
consumed. Similarly most of the items that enter our kitchen are transformed through the process
of cooking. But cooking at home is not an economic activity, even though the product involved
undergoes transformation. Home cooked food is not sold to the market. However, if the same
cooking or tea brewing was done in a restaurant where the cooked product would be sold to
customers, then the same items, such as tea leaves, would cease to be final goods and would be
counted as inputs to which economic value addition can take place. Thus it is not in the nature of
the good but in the economic nature of its use that a good becomes a final good.

42. Solution: b)
Apart from acting as a banker to the commercial banks, RBI also acts as a banker to the
Government of India, and also, to the state governments. It is commonly held that the
government, sometimes, prints money in case of a budget deficit, i.e., when it cannot meet its
expenses (e.g. salaries to the government employees, purchase of defense equipment from a
manufacturer of such goods etc.) from the tax revenue it has earned. The government, however,
has no legal authority to issue currency in this fashion. So it borrows money by selling treasury
http://insightsonindia.com

INSIGHTS

Page 17

Facebook Group: Indian Administrative Service (Raz Kr)

INSIGHTS MOCK PRELIMS TEST 5 - SOLUTIONS


bills or government securities to RBI, which issues currency to the government in return. The
government then pays for its expenses with this money. The money thus ultimately comes into
the hands of the general public (in the form of salary or sales proceeds of defense items etc.) and
becomes a part of the money supply. Financing of budget deficits by the governments in this
fashion is called Deficit Financing through Central Bank Borrowing.

43. Solution (b)


The Khilafat Movement,(1919-1920) was a movement of Indian Muslims, led by Muhammad Ali
and Shaukat Ali, that demanded the following: The Turkish Sultan or Khalifa must retain control
over the
Muslim sacred places in the erstwhile Ottoman empire; the jazirat-ul-Arab (Arabia, Syria, Iraq,
Palestine) must remain under Muslim sovereignty; and the Khalifa must be left with sufficient
territory to enable him to defend the Islamic faith The Congress supported the movement and
Mahatma Gandhi sought to conjoin it to the Non-cooperation Movement. Sir Syed Ahmed Khan
was an Indian Muslim philosopher and social activist of nineteenth century India.
Sir Syed founded the Muhammedan Anglo-Oriental College (Aligarh Muslim University) in 1875.
Reference: Page No.350(THEMES IN INDIAN HISTORY PART III)
44. Solution (d)
Judiciary has defined which aspects of the Constitution can be termed as the basic structure
and which cannot.
This theory found its first expression in the Kesavananda Bharati case and has been discussed
in subsequent judgments.
It has increased the powers of the judiciary and has come to be accepted by different political
parties and the government.
Though Federal system and social justice forms the basic structure of the Constitution,
fundamental duties does not.
Reference : Chapter 9 (Indian Constitution at Work Class XI)
45. Solution: d)
Indirect Sources
Analysis of properties of matter indirectly provides information about the interior. We know
through the mining activity that temperature and pressure increase with the increasing distance
from the surface towards the interior in deeper depths. Moreover, it is also known that the
density of the material also increases with depth. It is possible to find the rate of change of these
characteristics. Knowing the total thickness of the earth, scientists have estimated the values of
temperature, pressure and the density of materials at different depths. The details of these
characteristics with reference to each layer of the interior are discussed later in this chapter.
http://insightsonindia.com

INSIGHTS

Page 18

Facebook Group: Indian Administrative Service (Raz Kr)

INSIGHTS MOCK PRELIMS TEST 5 - SOLUTIONS


Another source of information are the meteors that at times reach the earth. However, it may be
noted that the material that becomes available for analysis from meteors, is not from the interior
of the earth. The material and the structure observed in the meteors are similar to that of the
earth. They are solid bodies developed out of materials same as, or similar to, our planet. Hence,
this becomes yet another source of information about the interior of the earth.
The other indirect sources include gravitation, magnetic field, and seismic activity. The
gravitation force (g) is not the same at different latitudes on the surface. It is greater near the
poles and less at the equator. This is because of the distance from the centre at the equator being
greater than that at the poles. The gravity values also differ according to the mass of material. The
uneven distribution of mass of material within the earth influences this value. The reading of the
gravity at different places is influenced by many other factors. These readings differ from the
expected values. Such a difference is called gravity anomaly. Gravity anomalies give us
information about the distribution of mass of the material in the crust of the earth. Magnetic
surveys also provide information about the distribution of magnetic materials in the crustal
portion, and thus, provide information about the distribution of materials in this part. Seismic
activity is one of the most important sources of information about the interior of the earth.
46. Solution: B
A group of organisms having the same genotype or identical genetic constitution. Organisms
having same genotype can be utilized in fighting against pests biologically.
47. Solution: C
The Ocean Health Index is a valuable tool for the ongoing assessment of ocean health. By
providing a means to advance comprehensive ocean policy and compare future progress, the
Index can inform decisions about how to use or protect marine ecosystems.
The Index is a collaborative effort, made possible through contributions from more than 65
scientists/ocean experts and partnerships between organizations including the National Center
for Ecological Analysis and Synthesis, Sea Around Us, Conservation International, National
Geographic, and the New England Aquarium. It is a multi-dimensional too measuring across
biological, social, economic and physical dimensions.
48. Solution: c)
Sterilisation by RBI: RBI often uses its instruments of money creation for stabilising the stock of
money in the economy from external shocks. Suppose due to future growth prospects in India
investors from across the world increase their investments in Indian bonds which under such
circumstances, are likely to yield a high rate of return. They will buy these bonds with foreign
currency. Since one cannot purchase goods in the domestic market with foreign currency, a
person who sells these bonds to foreign investors will exchange her foreign currency holding into
rupee at a commercial bank. The bank, in turn, will submit this foreign currency to RBI and its
http://insightsonindia.com

INSIGHTS

Page 19

Facebook Group: Indian Administrative Service (Raz Kr)

INSIGHTS MOCK PRELIMS TEST 5 - SOLUTIONS


deposits with RBI will be credited with equivalent sum of money. What kind of adjustments take
place from this entire transaction? The commercial banks total reserves and deposits remain
unchanged (it has purchased the foreign currency from the seller using its vault cash, which,
therefore, goes down; but the banks deposit with RBI goes up by an equivalent amount leaving
its total reserves unchanged). There will, however, be increments in the assets and liabilities on
the RBI balance sheet. RBIs foreign exchange holding goes up. On the other hand, the deposits of
commercial banks with RBI also increase by an equal amount. But that means an increase in the
stock of high powered money which, by definition, is equal to the total liability of RBI. With
money multiplier in operation, this, in turn, will result in increased money supply in the
economy.
This increased money supply may not altogether be good for the economys health. If the volume
of goods and services produced in the economy remains unchanged, the extra money will lead to
increase in prices of all commodities. People have more money in their hands with which they
compete each other in the commodities market for buying the same old stock of goods. As too
much money is now chasing the same old quantities of output, the process ends up in bidding up
prices of every commodity an increase in the general price level, which is also known as
inflation.
RBI often intervenes with its instruments to prevent such an outcome. In the above example, RBI
will undertake an open market sale of government securities of an amount equal to the amount
of foreign exchange inflow in the economy, thereby keeping the stock of high powered money
and total money supply unchanged. Thus it sterilises the economy against adverse external shocks.
This operation of RBI is known as sterilisation.

49. Solution: A
The profundal zone is a very cold zone, such as an ocean or a lake, located below the range of
effective light penetration. This is typically below the thermocline, the vertical zone in the water
through which temperature drops rapidly. The lack of light in the profundal zone determines the
type of biological community that can live in this region, which is distinctly different from the
community in the overlying waters. The profundal zone is part of the aphotic zone.

50. Solution (d)


Purusha sukta is a hymn in the Rigveda
Reference: Page No.61(THEMES IN INDIAN HISTORY PART I)

http://insightsonindia.com

INSIGHTS

Page 20

Facebook Group: Indian Administrative Service (Raz Kr)

INSIGHTS MOCK PRELIMS TEST 5 - SOLUTIONS


51. Solution (a)
The Governor has the power to recommend the dismissal of the State government and
suspension or dissolution of State assembly.
The Lok Sabha consists of elected representatives and also the President can nominate two
members to the Lok Sabha from the Anglo-Indian Community.
The Vice President should not be member of either House of the Parliament or State
Legislature.
73rd and 74th amendments have created uniformity in the structures of Panchayati Raj and
Nagarpalika institutions across the country.
Reference : Page 16, Chapter 4,Chapter 5, Chapter 8 (Indian Constitution at Work Class XI)
52. Solution (c)
Standing Committees not only supervise the work of various departments but also their
budget, their expenditure and bills that come up in the house relating to the department.
It is a permanent and regular committee which is constituted from time to time according to
the provisions of an Act of Parliament or Rules of Procedure and Conduct of Business. The
work done by the Indian Parliament is not only voluminous but also of a complex nature,
hence a good deal of its work is carried out in these Parliamentary Committees. There are two
types of Parliamentary Committee, the Standing Committee and the Ad hoc Committee.
Public Accounts Committee is an example of Standing Committee. Members of the Standing
committees are selected from both Houses, except for Estimates Committee where all the
members are from Lok Sabha Only.
Reference : Page 118, 119 (Indian Constitution at Work Class XI)& Wikipedia.
53. Solution: b)
It is worth noting that many commodities have two sets of prices. One is the retail price which
the consumer actually pays. The other is the wholesale price, the price at which goods are traded
in bulk. These two may differ in value because of the margin kept by traders. Goods which are
traded in bulk (such as raw materials or semi-finished goods) are not purchased by ordinary
consumers. Like CPI, the index for wholesale prices is called Wholesale Price Index (WPI). In
countries like USA it is referred to as Producer Price Index (PPI). Notice CPI (and analogously
WPI) may differ from GDP deflator because
1. The goods purchased by consumers do not represent all the goods which are produced in a
country. GDP deflator takes into account all such goods and services.
2. CPI includes prices of goods consumed by the representative consumer, hence it includes prices
of imported goods. GDP deflator does not include prices of imported goods.
3. The weights are constant in CPI but they differ according to production level of each good in
GDP deflator.

http://insightsonindia.com

INSIGHTS

Page 21

Facebook Group: Indian Administrative Service (Raz Kr)

INSIGHTS MOCK PRELIMS TEST 5 - SOLUTIONS


54. Solution: A

55. Solution: (c)


The British would be responsible for protecting their ally from external and internal threats to
their power. In the territory of the ally, a British armed contingent would be stationed and not
the contingent of the ally. The ally would have to provide the resources for maintaining this
contingent. The ally could enter into agreements with other rulers or engage in warfare only with
the permission of the British.
Reference: Page No.296(THEMES IN INDIAN HISTORY PART III)

56. Solution: d)
The present composition of earths atmosphere is chiefly contributed by nitrogen and oxygen.
You will be dealing with the composition and structure of the earths atmosphere in Chapter 8.
There are three stages in the evolution of the present atmosphere. The first stage is marked by
the loss of primordial atmosphere. In the second stage, the hot interior of the earth contributed to
the evolution of the atmosphere. Finally, the composition of the atmosphere was modified by the
living world through the process of photosynthesis.
The early atmosphere, with hydrogen and helium, is supposed to have been stripped off as a
result of the solar winds. This happened not only in case of the earth, but also in all the terrestrial
planets, which were supposed to have lost their primordial atmosphere through the impact of
solar winds.
During the cooling of the earth, gases and water vapour were released from the interior solid
earth. This started the evolution of the present atmosphere. The early atmosphere largely
contained water vapour, nitrogen, carbon dioxide, methane, ammonia and very little of free
oxygen. The process through which the gases were outpoured from the interior is called
degassing.
57. Solution: C
In ecology, edge effects refer to the changes in population or community structures that occur at
the boundary of two habitats. Areas with small habitat fragments exhibit especially pronounced
edge effects that may extend throughout the range. As the edge effects increase, the boundary
habitat allows for greater biodiversity.

58. Solution (b)


The Lingayats challenged the idea of caste and the pollution attributed to certain groups by
Brahmanas. They also questioned the theory of rebirth. These won them followers amongst those
who were marginalised within the Brahmanical social order. The Lingayats also encouraged
http://insightsonindia.com

INSIGHTS

Page 22

Facebook Group: Indian Administrative Service (Raz Kr)

INSIGHTS MOCK PRELIMS TEST 5 - SOLUTIONS


certain practices disapproved in the Dharmashastras,
such as post-puberty marriage and the remarriage of widows.
Reference: Page No.147(THEMES IN INDIAN HISTORY PART II)

59. Solution: c)
Earthquake waves are basically of two types body waves and surface waves. Body waves are
generated due to the release of energy at the focus and move in all directions travelling through
the body of the earth. Hence, the name body waves. The body waves interact with the surface
rocks and generate new set of waves called surface waves. These waves move along the surface.
The velocity of waves changes as they travel through materials with different densities. The
denser the material, the higher is the velocity. Their direction also changes as they reflect or
refract when coming across materials with different densities.

60. Solution (c)


The legislature is not involved in the process of appointment of judges. Thus, it was believed that
party politics would not play a role in the process of appointments. But the judges of the
Supreme Court and High Court are appointed by the President in consultation with the members
of the Judiciary. It should also be noted that while in making appointments, the executive plays a
crucial role; the legislature has the powers of removal. Hence the first statement is wrong.
In order to be appointed as a judge, a person must have experience as a lawyer and/or must be
well versed in law. Political opinions of the person or his/her political loyalty should not be the
criteria for appointments to judiciary.
The judges have a fixed tenure. They hold office till reaching the age of retirement. Only in
exceptional cases, judges may be removed. But otherwise, they have security of tenure. Security
of tenure ensures that judges could function without fear or favour.
The Constitution prescribes a very difficult procedure (special majority is required) for removal
of judges. The removal of judges of the Supreme Court and the High Courts is also extremely
difficult. A judge of the Supreme Court or High Court can be removed only on the ground of
proven misbehaviour or incapacity. A motion containing the charges against the judge must be
approved by special majority in both Houses of the Parliament.
The Constitution makers believed that a difficult procedure of removal would provide security of
office to the members of judiciary.
The judiciary is not financially dependent on either the executive or legislature. The Constitution
provides that the salaries and allowances of the judges are not subjected to the approval of the
legislature.
http://insightsonindia.com

INSIGHTS

Page 23

Facebook Group: Indian Administrative Service (Raz Kr)

INSIGHTS MOCK PRELIMS TEST 5 - SOLUTIONS


The actions and decisions of the judges are immune from personal criticisms. The judiciary
has the power to penalise those who are found guilty of contempt of court. This authority of
the court is seen as an effective protection to the judges from unfair criticism. Parliament
cannot discuss the conduct of the judges except when the proceeding toremove a judge is
being carried out. This gives the judiciary independence to adjudicate without fear of being
criticised.
Reference : Pages 125- 129 (Indian Constitution at Work Class XI)
61. Solution: A
Eutrophication is when the environment becomes enriched with nutrients. This can be a problem
in marine habitats such as lakes as it can cause algal blooms.
Fertilisers are often used in farming, sometimes these fertilisers run-off into nearby
water causing an increase in nutrient levels.
This causes phytoplankton to grow and reproduce more rapidly, resulting in algal blooms.
This bloom of algae disrupts normal ecosystem functioning and causes many problems.
The algae may use up all the oxygen in the water, leaving none for other marine life.
This results in the death of many aquatic organisms such as fish, which need the oxygen in
the water to live.
The bloom of algae may also block sunlight from photosynthetic marine plants under the
water surface.
Some algae even produce toxins that are harmful to higher forms of life. This can
cause problems along the food chain and affect any animal that feeds on them.
Zooplankton levels do not increase, but phytoplankton does suffer from rapid turnovers.

62. Solution (c)


He attended only the second RTC. At the Round Table Conference Mahatma Gandhi stated his
arguments against separate electorates for the Depressed Classes: Separate electorates to the
Untouchables will ensure them bondage in perpetuity.
Reference: Page No.360(THEMES IN INDIAN HISTORY PART III)

http://insightsonindia.com

INSIGHTS

Page 24

Facebook Group: Indian Administrative Service (Raz Kr)

INSIGHTS MOCK PRELIMS TEST 5 - SOLUTIONS


63. Solution: C
An epiphyte is a plant that grows non-parasitically upon another plant (such as a tree), and
derives its moisture and nutrients from the air, rain, and sometimes from debris accumulating
around it instead of the structure it is fastened to. An epiphytic organism that is not a plant is not
called an epiphyte. Epiphytes are usually found in the temperate zone (e.g.,
many mosses, liverworts, lichens, and algae) or in the tropics (e.g., many ferns, cacti, orchids,
and bromeliads).] Epiphytes provide a rich and diverse habitat for other organisms including
animals, fungi, bacteria, and myxomycetes. They do not harm trees instead support them.
64. Solution.(a)
After the failure of the Cripps Mission, Mahatma Gandhi decided to launch his third major
movement against British rule. This was the Quit India campaign, which began in August 1942.
Although Gandhiji was jailed at once (many other Congress leaders were also remained in the jail
for most part of the movement) , younger activists organised strikes and acts of sabotage all over
the country. Particularly active in the underground resistance were socialist members of the
Congress, such as Jayaprakash Narayan.
Reference: Page No.363(THEMES IN INDIAN HISTORY PART III)
65. Solution: d)
Batholiths
A large body of magmatic material that cools in the deeper depth of the crust develops in the
form of large domes. They appear on the surface only after the denudational processes remove
the overlying materials. They cover large areas, and at times, assume depth that may be several
km. These are granitic bodies. Batholiths are the cooled portion of magma chambers.
Lacoliths
These are large dome-shaped intrusive bodies with a level base and connected by a pipe-like
conduit from below. It resembles the surface volcanic domes of composite volcano, only these are
located at deeper depths. It can be regarded as the localised source of lava that finds its way to the
surface. The Karnataka plateau is spotted with domal hills of granite rocks. Most of these, now
exfoliated, are examples of lacoliths or batholiths.
66. Solution: c)
Nominal GDP is GDP evaluated at current market prices. Therefore, nominal GDP will include
all of the changes in market prices that have occurred during the current year due
to inflation or deflation. Inflation is defined as a rise in the overall price level, and deflation is
defined as a fall in the overall price level. In order to abstract from changes in the overall price
level, another measure of GDP called real GDP is often used. Real GDP is GDP evaluated at
http://insightsonindia.com

INSIGHTS

Page 25

Facebook Group: Indian Administrative Service (Raz Kr)

INSIGHTS MOCK PRELIMS TEST 5 - SOLUTIONS


the market prices of some base year. For example, if 1990 were chosen as the base year, then real
GDP for 1995 is calculated by taking the quantities of all goods and services purchased in 1995
and multiplying them by their 1990 prices.
GDP deflator. Using the statistics on real GDP and nominal GDP, one can calculate an
implicit index of the price level for the year. This index is called the GDP deflator and is given by
the formula

The GDP deflator can be viewed as a conversion factor that transforms real GDP into nominal
GDP. Note that in the base year, real GDP is by definition equal to nominal GDP so that the
GDP deflator in the base year is always equal to 100.
67. Solution: B

68. Solution.(b)
The Lucknow Pact of December 1916 was an understanding between the Congress and the
Muslim League (controlled by the UP-based Young Party) whereby the Congress accepted
separate electorates. The Poona Pact was signed in 1932 between Dr. Babasaheb Ambedkar and
Mahatma Gandhi by which seats were reserved for the Depressed Classes out of general
electorate.
Reference: Page No.383, Internet(THEMES IN INDIAN HISTORY PART III)
69. Solution (c)
According to the Manusmriti, the paternal estate was to be divided equally amongst sons after the
death of the parents, with a special share for the eldest. Women could not claim a share of these
resources. The Manusmriti laid down the duties of the chandalas. They had to live outside the
village, use discarded utensils, and wear clothes of the dead and ornaments of iron. They could
not walk about in villages and cities at night. They had to dispose of the bodies of those who had
no relatives and serve as executioners.
Reference: Page No.68 (THEMES IN INDIAN HISTORY PART I)
70. Solution: d)
Macroeconomics deals with the aggregate economic variables of an economy. It also takes into
account various interlinkages which may exist between the different sectors of an economy. This
is what distinguishes it from microeconomics; which mostly examines the functioning of the
particular sectors of the economy, assuming that the rest of the economy remains the same.
http://insightsonindia.com

INSIGHTS

Page 26

Facebook Group: Indian Administrative Service (Raz Kr)

INSIGHTS MOCK PRELIMS TEST 5 - SOLUTIONS


Macroeconomics emerged as a separate subject in the 1930s due to Keynes. The Great
Depression, which dealt a blow to the economies of developed countries, had provided Keynes
with the inspiration for his writings. In this book we shall mostly deal with the working of a
capitalist economy. Hence it may not be entirely able to capture the functioning of a developing
country. Macroeconomics sees an economy as a combination of four sectors, namely households,
firms, government and external sector.

71. Solution (c)


Under the Advisory Jurisdiction the President of India can refer any matter that is of public
importance or that which involves interpretation of Constitution to Supreme Court for advice.
However, the Supreme Court is not bound to give advice on such matters and the President is
not bound to accept such an advice.
Utility of the advisory powers of the Supreme Court- The utility is two-fold. In the first place, it
allows the government to seek legal opinion on a matter of importance before taking action on it.
This may prevent unnecessary litigations later. Secondly, in the light of the advice of the Supreme
Court, the government can make suitable changes in its action or legislations.
Reference : Pages 133 (Indian Constitution at Work Class XI)
72. Solution: c)
CONCEPT OF SEA FLOOR SPREADING
a detailed analysis of magnetic properties of the rocks on either sides of the mid-oceanic ridge led
Hess (1961) to propose his hypothesis, known as the sea floor spreading. Hess argued that
constant eruptions at the crest of oceanic ridges cause the rupture of the oceanic crust and the
new lava wedges into it, pushing the oceanic crust on either side. The ocean floor, thus spreads.
The younger age of the oceanic crust as well as the fact that the spreading of one ocean does not
cause the shrinking of the other, made Hess think about the consumption of the oceanic crust.
He further maintained that the ocean floor that gets pushed due to volcanic eruptions at the
crest, sinks down at the oceanic trenches and gets consumed.

73. Solution: A
http://articles.economictimes.indiatimes.com/2013-12-20/news/45418033_1_kasturiranganpanel-report-environment-ministry-western-ghats
http://articles.economictimes.indiatimes.com/2013-12-20/news/45418033_1_kasturiranganpanel-report-environment-ministry-western-ghats
http://insightsonindia.com

INSIGHTS

Page 27

Facebook Group: Indian Administrative Service (Raz Kr)

INSIGHTS MOCK PRELIMS TEST 5 - SOLUTIONS

74. Solution (a)


The first gold coins were issued in India by the Kushanas. Punch-marked coins made
of silver and copper (c. sixth century BCE onwards) were amongst the earliest to be minted and
used. These have been recovered from excavations at a number of sites throughout the
subcontinent. A punch-marked coin, so named because symbols were punched or stamped onto
the metal surface.
Reference: Page No.45 (THEMES IN INDIAN HISTORY PART I)
75. Solution: b)
Nazca plate is between South America and Pacific plate
76. Solution: d)
Inventories are treated as capital. Addition to the stock of capital of a firm is known as
investment. Therefore change in the inventory of a firm is treated as investment. There can be
three major categories of investment. First is the rise in the value of inventories of a firm over a
year which is treated as investment expenditure undertaken by the firm. The second category of
investment is the fixed business investment, which is defined as the addition to the machinery,
factory buildings, and equipments employed by the firms.
The last category of investment is the residential investment, which refers to the addition of
housing facilities. Change in inventories may be planned or unplanned. In case of an unexpected
fall in sales, the firm will have unsold stock of goods which it had not anticipated. Hence there
will be unplanned accumulation of inventories. In the opposite case where there is unexpected
rise in the sales there will be unplanned decumulation of inventories.
77. Solution: c)
If we sum the gross value added of all the firms of the economy in a year, we get a measure of the
value of aggregate amount of goods and services produced by the economy in a year (just as we
had done in the wheat-bread example). Such an estimate is called Gross Domestic Product
(GDP). Thus GDP Sum total of gross value added of all the firms in the economy.
An alternative way to calculate the GDP is by looking at the demand side of the products. This
method is referred to as the expenditure method.
78. Solution: B
The Deccan Traps are one of the largest volcanic provinces in the world. It consists of more than
6,500 feet (>2,000 m) of flat-lying basalt lava flows and covers an area of nearly 200,000 square
http://insightsonindia.com

INSIGHTS

Page 28

Facebook Group: Indian Administrative Service (Raz Kr)

INSIGHTS MOCK PRELIMS TEST 5 - SOLUTIONS


miles (500,000 square km) (roughly the size of the states of Washington and Oregon combined)
in west-central India.

79. Solution: A
Agreements at Warsaw included a significant set of decisions on ways to help developing
countries reduce greenhouse gas emissions from deforestation and the degradation of forests,
which account for around one fifth of all human-generated emissions. The Warsaw Framework
for REDD+ is backed by pledges of 280 million dollars financing from the US, Norway and the
UK.
The other two initiatives have been signed quite earlier.
80. Solution (d)
81. Solution: C
The Government of India proposes to support setting up of Plastic Parks for the promotion of
downstream plastic processing industries in the 12th plan under the PPP mode. The Scheme to be
implemented by the Department of Chemicals and Petrochemicals envisages setting up of a
Special Purpose Vehicle (SPV), which would provide infrastructure and common facilities for
the industries. State Government or its agency, which would set up such SPV, will have to
mandatorily have equity participation in it.
The Scheme is aimed at increasing investment in the sector for additions in capacity and for
increased exports in the sector. Adopting a clustered development approach, the scheme aims to
achieve environmentally sustainable growth through innovative methods of waste management,
recycling etc.
Currently the share of India in world trade for plastics is very low. The industry in the country is
very large but highly fragmented with the dominance of tiny, small and medium units.
82. Solution: a)

We have already noted that a part of the capital gets consumed during the year due to wear and
tear. This wear and tear is called depreciation. Naturally, depreciation does not become part of
anybodys income. If we deduct depreciation from GNP the measure of aggregate income that we
obtain is called Net National Product (NNP). Thus
NNP GNP Depreciation

http://insightsonindia.com

INSIGHTS

Page 29

Facebook Group: Indian Administrative Service (Raz Kr)

INSIGHTS MOCK PRELIMS TEST 5 - SOLUTIONS


It is to be noted that all these variables are evaluated at market prices. Through the expression
given above, we get the value of NNP evaluated at market prices. But market price includes
indirect taxes. When indirect taxes are imposed on goods and services, their prices go up. Indirect
taxes accrue to the government. We have to deduct them from NNP evaluated at market prices
in order to calculate that part of NNP which actually accrues to the factors of production.
Similarly, there may be subsidies granted by the government on the prices of some commodities
(in India petrol is heavily taxed by the government, whereas cooking gas is subsidised). So we
need to add subsidies to the NNP evaluated at market prices. The measure that we obtain by
doing so is called Net National Product at factor cost or National Income.
Thus, NNP at factor cost National Income (NI ) NNP at market prices (Indirect taxes
Subsidies) NNP at market prices Net indirect taxes (Net indirect taxes Indirect taxes
Subsidies)

83. Solution: B

The Cabinet Committee on Economic Affairs has approved the launching of a new Integrated
Processing Development Scheme (IPDS) with a total cost of Rs. 500 crore during the 12th Five
Year Plan and to establish four to six brown field projects and three to five green field projects
addressing the environmental issues faced by Textile Processing Units.
The eligible projects under the scheme would cover the following:
(i) Common Effluent Treatment Plant (CETP),
(ii) Captive power generation on technology preferably renewable/green technology,
(iii) Infrastructure such as storm water management, necessary roads and pipelines for water &
wastewater and
(iv) Facility for testing and R&D centres.
The Scheme will facilitate the textiles industry become globally competitive using
environmentally friendly processing standards and technology and create new processing parks.
This scheme will support the upgradation of existing processing clusters/centres specifically in
the area of water and waste water management and also encourage research and development
work in the textiles processing sector.
84. Solution: c)

http://insightsonindia.com

INSIGHTS

Page 30

Facebook Group: Indian Administrative Service (Raz Kr)

INSIGHTS MOCK PRELIMS TEST 5 - SOLUTIONS


Gross Domestic Product measures the aggregate production of final goods and services taking
place within the domestic economy during a year. But the whole of it may not accrue to the
citizens of the country. For example, a citizen of India working in Saudi Arabia may be earning
her wage and it will be included in the Saudi Arabian GDP. But legally speaking, she is an Indian.
Is there a way to take into account the earnings made by Indians abroad or by the factors of
production owned by Indians? When we try to do this, in order to maintain symmetry, we must
deduct the earnings of the foreigners who are working within our domestic economy, or the
payments to the factors of production owned by the foreigners. For example, the profits earned
by the Korean-owned Hyundai car factory will have to be subtracted from the GDP of India. The
macroeconomic variable which takes into account such additions and subtractions is known as
Gross National Product (GNP). It is, therefore, defined as follows
GNP GDP + Factor income earned by the domestic factors of production employed in the rest
of the world Factor income earned by the factors of production of the rest of the world
employed in the domestic economy
Hence, GNP GDP + Net factor income from abroad (Net factor income from abroad = Factor
income earned by the domestic factors of production employed in the rest of the world Factor
income earned by the factors of production of the rest of the world employed in the domestic
economy).
We have already noted that a part of the capital gets consumed during the year due to wear and
tear. This wear and tear is called depreciation. Naturally, depreciation does not become part of
anybodys income. If we deduct depreciation from GNP the measure of aggregate income that we
obtain is called Net National Product (NNP).
Thus NNP GNP Depreciation
85. Solution (a)

Original jurisdiction means cases that can be directly considered by the Supreme Court without
going to the lower courts before that. The Original Jurisdiction of the Supreme Court establishes
it as an umpire in all disputes regarding federal matters. In any federal country, legal disputes are
bound to arise between the Union and the States; and among the States themselves. The power
to resolve such cases is entrusted to the Supreme Court of India. It is called original jurisdiction
because the Supreme Court alone has the power to deal with such cases. Neither the High
Courts nor the lower courts can deal with such cases. In this capacity, the Supreme Court not
just settles disputes but also interprets the powers of Union and State government as laid down in
the Constitution.( Court rejected the appeal by people against the eviction from the dam site,
comes under original jurisdiction of the Supreme Court)
Writ Jurisdiction - any individual, whose fundamental right has been violated, can directly move
the Supreme Court for remedy. The Supreme Court can give special orders in the form of writs.
The High can also issue writs, but the persons whose rights are violated have the choice of either
http://insightsonindia.com

INSIGHTS

Page 31

Facebook Group: Indian Administrative Service (Raz Kr)

INSIGHTS MOCK PRELIMS TEST 5 - SOLUTIONS


approaching the High Court or approaching the Supreme Court directly. Through such writs, the
Court can give orders to the executive to act or not to act in a particular way.
Appellate Jurisdiction- the Supreme Court is the highest court of appeal. A person can appeal to
the Supreme Court against the decisions of the High Court. However, High Court must certify
that the case is fit for appeal, that is to say that it involves a serious matter of interpretation of
law or Constitution. In addition, in criminal cases, if the lower court has sentenced a person to
death then an appeal can be made to the High Court or Supreme Court. Of course, the Supreme
Court holds the powers to decide whether to admit appeals even when appeal is not allowed by
the High Court. Appellate jurisdiction means that the Supreme Court will reconsider the case
and the legal issues involved in it. If the Court thinks that the law or the Constitution has a
different meaning from what the lower courts understood, then the Supreme Court will change
the ruling and along with that also give new interpretation of the provision involved.
The High Courts too, have appellate jurisdiction over the decisions given by courts below
them.(Example- In order to resolve the dispute about river Cauvery the government of Tamil
Nadu wants to approach the court)
Advisory jurisdiction Under the Advisory Jurisdiction the President of India can refer any
matter that is of public importance or that which involves interpretation of Constitution to
Supreme Court for advice.
Example: The government wanted to know if it can pass a law about the citizenship status of
residents of Pakistan occupied areas of Jammu and Kashmir. Then the executive (President) seeks
advice of the Supreme Court.
Reference : Pages 132, 133 (Indian Constitution at Work Class XI)

86. Solution: c)

The Expenditure Method:


The expenditure approach measures national income as total spending on final goods and services
produced within nation during an year: The expenditure approach to measuring national income
is to add up all expenditures made for final goods and services at current market prices by
households, firms and government during a year. Total aggregate final expenditure on final output
thus is the sum of four broad categories of expenditures (i) consumption (ii) Investment (iii)
government and (iv) Net exports.
(i) Consumption expenditure: Consumption expenditure is the largest component of national
income. It includes expenditure on all goods and services produced and sold to the final consumer
during the year.
(ii) Investment expenditure: Investment is the use of todays resources to expand tomorrows
production or consumption. Investment expenditure is expenditure incurred on by business firms
on(a) new plants, (b) adding to the stock of inventories and (c) on newly constructed houses.

http://insightsonindia.com

INSIGHTS

Page 32

Facebook Group: Indian Administrative Service (Raz Kr)

INSIGHTS MOCK PRELIMS TEST 5 - SOLUTIONS


(iii)
Government expenditure: (G) it is the second largest component of national income. It
includes all government expenditure on currently produced goods and services but excludes
transfer payments while computing national income.
(iv) Net exports: Net exports are defined as total exports minus total imports.
National income calculated from the expenditure side is the sum of final consumption
expenditure, expenditure by business on plants, government spending and net exports.
NI = C + 1 +G + (X M)
Precautions
While estimating national income through expenditure method, the following precautions should be taken.
(i)
The expenditure on second hand goods should not be included as they do not contribute
to the current years production of goods.
(ii) Similarly, expenditure on purchase of old shares and bonds is not included as these also do
not represent expenditure on currently produced goods and
services.
(iii)
Expenditure on transfer payments by government such as unemployment benefit, old
age pensions, interest on public debt should also not be included because no productive service is
rendered in exchange by recipients of these payments.
87. Solution: a)

Youth
Streams are few during this stage with poor integration and flow over original slopes showing
shallow V-shaped valleys with no floodplains or with very narrow floodplains along trunk
streams. Streams divides are broad and flat with marshes, swamp and lakes. Meanders if present
develop over these broad upland surfaces. These meanders may eventually entrench themselves
into the uplands. Waterfalls and rapids may exist where local hard rock bodies are exposed.
Mature
During this stage streams are plenty with good integration. The valleys are still V-shaped but
deep; trunk streams are broad enough to have wider floodplains within which streams may flow
in meanders confined within the valley. The flat and broad inter stream areas and swamps and
marshes of youth disappear and the stream divides turn sharp. Waterfalls and rapids disappear.
Old

http://insightsonindia.com

INSIGHTS

Page 33

Facebook Group: Indian Administrative Service (Raz Kr)

INSIGHTS MOCK PRELIMS TEST 5 - SOLUTIONS


Smaller tributaries during old age are few with gentle gradients. Streams meander freely over vast
floodplains showing natural levees, oxbow lakes, etc. Divides are broad and flat with lakes,
swamps and marshes. Most of the landscape is at or slightly above sea level.

88. Solution: d)

Stalactites, Stalagmites and Pillars


Stalactites hang as icicles of different diameters. Normally they are broad at their bases and taper
towards the free ends showing up in a variety of forms. Stalagmites rise up from the floor of the
caves. In fact, stalagmites form due to dripping water from the surface or through the thin pipe,
of the stalactite, immediately below it.
Stalagmites may take the shape of a column, a disc, with either a smooth, rounded bulging end or
a miniature crater like depression. The stalagmite and stalactites eventually fuse to give rise to
columns and pillars of different diameters.
89. Solution: a)

National Geophysical Laboratory, Geological Survey of India, Department of Meteorology,


Government of India, along with the recently formed National Institute of Disaster Management,
have made an intensive analysis of more than 1,200 earthquakes that have occurred in India in
different years in the past, and based on these, they divided India into the following five
earthquake zones:
(i) Very high damage risk zone
(ii) High damage risk zone
(iii) Moderate damage risk zone
(iv) Low damage risk zone
(v) Very low damage risk zone.
Out of these, the first two zones had experienced some of the most devastating earthquakes in
India. areas vulnerable to these earthquakes are the North-east states, areas to the north of
Darbhanga and Araria along the Indo-Nepal border in Bihar, Uttaranchal, Western Himachal
Pradesh (around Dharamshala) and Kashmir Valley in the Himalayan region and the Kuchchh
(Gujarat). These are included in the Very High Damage Risk Zone. Similarly, the remaining parts
of Jammu and Kashmir, Himachal Pradesh, Northern parts of Punjab, Eastern parts of Haryana,
Delhi, Western Uttar Pradesh, and Northern Bihar fall under the High Damage Risk Zone.
Remaining parts of the country fall under moderate to very Low Damage Risk Zone. Most of the
areas that can be considered safe are from the stable landmass covered under the Deccan plateau.
http://insightsonindia.com

INSIGHTS

Page 34

Facebook Group: Indian Administrative Service (Raz Kr)

INSIGHTS MOCK PRELIMS TEST 5 - SOLUTIONS


90. Solution (a)

In this article (Article-368), there are two methods of amending the Constitution and they apply
to two different sets of articles of the Constitution. One method is that amendment can be made
by special majority of the two houses of the Parliament. The other method is more difficult: it
requires special majority of the Parliament and consent of half of the State legislatures. Note that
all amendments to the Constitution are initiated only in the Parliament. Besides the special
majority in the Parliament no outside agencylike a constitution commission or a separate
bodyis required for amending the Constitution.
Similarly, after the passage in the Parliament and in some cases, in State legislatures, no
referendum is required for ratification of the amendment.
After the passage in the Parliament and in some cases, in State legislatures, no referendum is
required for ratification of the amendment.
Only elected representatives of the people are empowered to consider and take final decisions on
the question of amendments. Thus, Sovereignty of elected representatives (parliamentary
sovereignty) is the basis of the amendment procedure.
Reference : page 201, 202 (Indian Constitution at Work Class XI)
91. Solution (b)

Instruments of Parliamentary Control


The legislature in parliamentary system ensures executive accountability at various stages: policy
making, implementation of law or policy and during and post implementation stage. The
legislature does this through the use of a variety of devices:
1. Deliberation and discussion- Zero Hour where members are free to raise any matter that
they think is important (though the ministers are not bound to reply), half-an hour
discussion on matters of public importance, adjournment motion etc. are some instruments of
exercising control.
2. Approval or Refusal of laws
3. Financial control- before granting money the Lok Sabha can discuss the reasons for which
the government requires money. It can enquire into cases of misuse of funds on the basis of
the report of the Comptroller and Auditor General and Public Accounts committees.
4. No confidence motion: The most powerful weapon that enables the Parliament to ensure
executive accountability is the no-confidence motion. As long as the government has the
support of its party or coalition of parties that have a majority in the Lok Sabha, the power of
the House to dismiss the government is fictional rather than real.
Reference : Page 116, 117 (Indian Constitution at Work Class XI)

http://insightsonindia.com

INSIGHTS

Page 35

Facebook Group: Indian Administrative Service (Raz Kr)

INSIGHTS MOCK PRELIMS TEST 5 - SOLUTIONS


92. Solution: c)

Tsunami
Earthquakes and volcanic eruptions that cause the sea-floor to move abruptly resulting in sudden
displacement of ocean water in the form of high vertical waves are called tsunamis (harbour
waves) or seismic sea waves. Normally, the seismic waves cause only one instantaneous vertical
wave; but, after the initial disturbance, a series of after-waves are created in the water that
oscillate between high crest and low trough in order to restore the water level.
The speed of wave in the ocean depends upon the depth of water. It is more in the shallow water
than in the ocean deep. As a result of this, the impact of tsunami is less over the ocean and more
near the coast where they cause large-scale devastations. Therefore, a ship at sea is not much
affected by tsunami and it is difficult to detect a tsunami in the deeper parts of sea It is so
because over deep water the tsunami has very long wave-length and limited wave-height.

93. Solution (c)

Only Lok Sabha can initiate, reject or amend money bills. If it is a money bill, the Rajya Sabha
can either approve the bill or suggest changes but cannot reject it. If it takes no action within
14 days the bill is deemed to have been passed. Amendments to the bill, suggested by Rajya
Sabha, may or may not be accepted by the Lok Sabha.
The Constitutional Amendment Bills can only be ratified i.e., it cannot be rejected or
returned.
The President does not enjoy constitutional discretion, he/she enjoys only situational
discretion like appointment of Prime Minister, dismissal of council of ministers.
Directive Principles of State Policy are known as instrument of instructions in Indian
administration.
Reference : Page 110, 113 (Indian Constitution at Work Class XI)
94. Solution: c)

Definition: The monetary value of an asset decreases over time due to use, wear and tear or
obsolescence. This decrease is measured as depreciation.
Description: Depreciation, i.e. a decrease in an asset's value, may be caused by a number of other
factors as well such as unfavorable market conditions, etc. Machinery, equipment, currency are
some examples of assets that are likely to depreciate over a specific period of time. Opposite of
http://insightsonindia.com

INSIGHTS

Page 36

Facebook Group: Indian Administrative Service (Raz Kr)

INSIGHTS MOCK PRELIMS TEST 5 - SOLUTIONS


depreciation is appreciation which is increase in the value of an asset over a period of time.
Accounting estimates the decrease in value using the information regarding the useful life of the
asset. This is useful for estimation of property value for taxation purposes like property tax etc.
For such assets like real estate, market and economic conditions are likely to be crucial such as in
cases of economic downturn.
95. Solution (d)

In all other spheres, including passing of non-money bills, constitutional amendments, and
impeaching the President and removing the Vice President the powers of Lok Sabha and
Rajya Sabha are co-equal. Participates in the election and removal of the President, Vice
President, Judges of Supreme Court and High Court.
Reference :Page 109,110 (Indian Constitution at Work Class XI)
96. Solution: a)

We may note here that some commodities like television sets, automobiles or home computers,
although they are for ultimate consumption, have one characteristic in common with capital
goods they are also durable. That is, they are not extinguished by immediate or even short
period consumption; they have a relatively long life as compared to articles such as food or even
clothing. They also undergo wear and tear with gradual use and often need repairs and
replacements of parts, i.e., like machines they also need to be preserved, maintained and renewed.
That is why we call these goods consumer durables.
Thus if we consider all the final goods and services produced in an economy in a given period of
time they are either in the form of consumption goods (both durable and non-durable) or capital
goods. As final goods they do not undergo any further transformation in the economic process.
Of the total production taking place in the economy a large number of products dont end up in
final consumption and are not capital goods either. Such goods may be used by other producers as
material inputs. Examples are steel sheets used for making automobiles and copper used for
making utensils.
These are intermediate goods, mostly used as raw material or inputs for production of other
commodities. These are not final goods.
97. Solution: c)

In a capitalist country production activities are mainly carried out by capitalist enterprises. A
typical capitalist enterprise has one or several entrepreneurs (people who exercise control over
major decisions and bear a large part of the risk associated with the firm/enterprise). They may
themselves supply the capital needed to run the enterprise, or they may borrow the capital. To
http://insightsonindia.com

INSIGHTS

Page 37

Facebook Group: Indian Administrative Service (Raz Kr)

INSIGHTS MOCK PRELIMS TEST 5 - SOLUTIONS


carry out production they also need natural resources a part consumed in the process of
production (e.g. raw materials) and a part fixed (e.g. plots of land). And they need the most
important element of human labour to carry out production. This we shall refer to as labour.
After producing output with the help of these three factors of production, namely capital, land
and labour, the entrepreneur sells the product in the market. The money that is earned is called
revenue. Part of the revenue is paid out as rent for the service rendered by land, part of it is paid
to capital as interest and part of it goes to labour as wages. The rest of the revenue is the earning
of the entrepreneurs and it is called profit. Profits are often used by the producers in the next
period to buy new machinery or to build new factories, so that production can be expanded.
These expenses which raise productive capacity are examples of investment expenditure.
In short, a capitalist economy can be defined as an economy in which most of the economic
activities have the following characteristics (a) there is private ownership of means of production
(b) production takes place for selling the output in the market (c) there is sale and purchase of
labour services at a price which is called the wage rate (the labour which is sold and purchased
against wages is referred to as wage labour).
98. Solution: b)

Tropical Cyclone
Tropical cyclones are intense low-pressure areas confined to the area lying between 30 N and
30 S latitudes, in the atmosphere around which high velocity winds blow. Horizontally, it
extends up to 500-1,000 km and vertically from surface to 12-14 km. A tropical cyclone or
hurricane is like a heat engine that is energised by the release of latent heat on account of the
condensation of moisture that the wind gathers after moving over the oceans and seas.
There are differences of opinion among scientists about the exact mechanism of a tropical
cyclone. However, some initial conditions for the emergence of a tropical cyclone are:
(i) Large and continuous supply of warm and moist air that can release enormous latent heat.
(ii) Strong Coriolis force that can prevent filling of low pressure at the centre (absence of Coriolis
force near the equator prohibits the formation of tropical cyclone between 0-5 latitude).
(iii) Unstable condition through the troposphere that creates local disturbances around which a
cyclone develops.
(iv) Finally, absence of strong vertical wind wedge, which disturbs the vertical transport of latent
heat.
Structure of Tropical Cyclone
Tropical cyclones are characterised by large pressure gradients. The centre of the cyclone is
mostly a warm and low-pressure, cloudless core known as eye of the storm. Generally, the
isobars are closely placed to each other showing high-pressure gradients. Normally, it varies
http://insightsonindia.com

INSIGHTS

Page 38

Facebook Group: Indian Administrative Service (Raz Kr)

INSIGHTS MOCK PRELIMS TEST 5 - SOLUTIONS


between 14-17mb/100 km, but sometimes it can be as high as 60mb/100km. Expansion of the
wind belt is about 10-150 km from the centre.
Spatio-temporal Distribution of Tropical Cyclone in India
Owing to its Peninsular shape surrounded by the Bay of Bengal in the east and the Arabian Sea
in the west, the tropical cyclones in India also originate in these two important locations. Though
most of the cyclones originate between 10-15 north latitudes during the monsoon season, yet in
case of the Bay of Bengal, cyclones mostly develop during the months of October and
November. Here, they originate between 16-2 N latitudes and to the west of 92 E. By July the
place of origin of these storms shifts to around 18 N latitude and west of 90E near the
Sunderban Delta. Table 7.4 and Figure 7.4 show the frequency and tracks of time of cyclonic
storms in India.
99. Solution: a)

An introduced, alien, exotic, non-indigenous, or non-native species, or simply anintroduction, is


a species living outside its native distributional range, which has arrived there by human activity,
either deliberate or accidental. Some introduced species are damaging to the ecosystem they are
introduced into, others have no negative effect and can, in fact, be beneficial as an alternative to
pesticides in agriculture for example. In some instances the potential for being beneficial or
detrimental in the long run remains unknown, for instance in New Zealand. A list of introduced
species is given in a separate article.
The effect of introduced species on natural environments is a controversial subject, and one that
has gained much scrutiny by scientists, governments, farmers and others. Not all introduced
species are problematic. Those species that spread widely and create significant problems are
known as invasive species.
100.

Solution: d)

Genetic Diversity
Genes are the basic building blocks of various life forms. Genetic biodiversity refers to the
variation of genes within species. Groups of individual organisms having certain similarities in
their physical characteristics are called species. Human beings genetically belong to the homo
sapiens group and also differ in their characteristics such as height, colour, physical appearance,
etc., considerably. This is due to genetic diversity. This genetic diversity is essential for a healthy
breeding of population of species.

http://insightsonindia.com

INSIGHTS

Page 39

Facebook Group: Indian Administrative Service (Raz Kr)

INSIGHTS ON INDIA MOCK PRELIMINARY EXAM - 2014


INSIGHTS ON INDIA MOCK TEST - 6
GENERAL STUDIES

PAPER-I
Time Allowed: One and Half Hours

Maximum Marks: 150

INSTRUCTIONS
1. IMMEDITELY AFTER THE COMMENCEMENT OF THE EXAMINATION, YOU SHOULD
CHECK THAT THIS TEST BOOKLET DOES NOT HAVE ANY UNPRINTED OR TORN OR
MISSING PAGES OR ITEMS, ETC. IF SO, GET IT REPLACED BY A COMPLETE TEST BOOKLET.
2. You have to enter your Roll Number on the Test I
Booklet in the Box provided alongside. DO NOT
write anything else on the Test Booklet.
4. This Test Booklet contains 100 items (questions). Each item is printed only in English. Each item
comprises four responses (answers). You will select the response which you want to mark on the Answer
Sheet. In case you feel that there is more than one correct response, mark the response which you consider
the best. In any case, choose ONLY ONE response for each item.
5. You have to mark all your responses ONLY on the separate Answer Sheet provided. See directions in the
Answer Sheet.
6. All items carry equal marks.
7. Before you proceed to mark in the Answer Sheet the response to various items in the Test Booklet, you
have to fill in some particulars in the Answer Sheet as per instructions sent to you with your Admission
Certificate.
8. After you have completed filling in all your responses on the Answer Sheet and the examination has
concluded, you should hand over to the Invigilator only the Answer Sheet. You are permitted to take away
with you the Test Booklet.
9. Sheets for rough work are appended in the Test Booklet at the end.
10. Penalty for wrong answers :
THERE WILL BE PENALTY FOR WRONG ANSWERS MARKED BY A CANDIDATE IN THE
OBJECTIVE TYPE QUESTION PAPERS.
(i)

There are four alternatives for the answer to every question. For each question for which a
wrong answer has been given by the candidate, one-third of the marks assigned to that question
will be deducted as penalty.

(ii) If a candidate gives more than one answer, it will be treated as a wrong answer even if one of the
given answers happens to be correct and there will be same penalty as above to that question.
(iii)

If a question is left blank, i.e., no answer is given by the candidate, there will be no penalty for
that question.

http://insightsonindia.com

INSIGHTS ON INDIA MOCK TEST SERIES FOR CIVIL SERVICES PRELIMINARY EXAM 2014

http://insightsonindia.com

Page 1

Facebook Group: Indian Administrative Service (Raz Kr)

1. Consider the following statements with


reference to human development.
1. Nobel Laureate Prof Amartya
Sen saw an increase in
freedom (or decrease in
unfreedom) as the main
objective of development.
2. According to Dr Mehbub-UlHaq, development is all about
enlarging peoples choices in
order to lead long, healthy
lives with dignity.
Which of the statements given above
is/are correct?
a.
b.
c.
d.

1 Only
2 Only
Both
None

2. Which of the following is/are true in


connection with Special Drawing Rights
(SDRs) ?
1. The SDR quota in IMF is different
for different countries.
2. Its value is not constant.
3. It derives its legitimacy from IMF
and other nations willing to use it.
Select the correct answer using the codes
below.
a.
b.
c.
d.

All of the above


1 and 3
2 Only
1 Only

http://insightsonindia.com

3. With reference
consider
the
statements

to AIDS,
following

1. AIDS is caused by the Human


Immuno deficiency Virus
(HIV), a member of a group
of viruses called retrovirus.
2. The
virus
enters
into
macrophages where RNA
genome of the virus replicates
to form infective new RNA
with the help of the enzyme
reverse transcriptase.
3. This
viral
DNA
gets
incorporated into host cells
DNA and directs the infected
cells to produce virus particles
Which of the above statements is/are
correct?
a.
b.
c.
d.

1 and 3 Only
1 and 2 Only
2 and 3 Only
All

4. Consider
the
following
statements about population
change
1. Population change is also called as
population growth
2. Natural Growth of Population
refers to the population increased
by difference between births and
deaths in a particular region
between two points of time.
3. Population change in an area is an
important indicator of economic
Page 2

Facebook Group: Indian Administrative Service (Raz Kr)

development, social upliftment


and historical and cultural
background of the region.
Which of the statements given above
is/are incorrect?
a.
b.
c.
d.

7. Productivity and sustainability are


two pillars of human development,
with reference to these two, consider
the following statements
1. Productivity
here
means
human labour productivity or
productivity in terms of
human
work.
Such
productivity
must
be
constantly
enriched
by
building capabilities in people.
2. Sustainability
means
continuity in the availability
of opportunities.
3. To have sustainable human
development, each generation
must
have
the
same
opportunities.

3 Only
1 Only
None
All

5. In which of the following cases of budget


deficits will the government have to
necessarily borrow not only to finance its
investment but also its consumption
requirements?
a) Fiscal deficit
b) Revenue deficit
c) Primary deficit
d) Current account deficit
6. The proportional income tax regime
acts as an automatic stabiliser a shock
absorber for the economy in which of the
following ways?
1. It makes disposable income of the
households less sensitive to fluctuations in
the GDP.
2. It redistributes the national
income and redirects government spending
in the right sectors.
a.
b.
c.
d.

1 Only
2 Only
Both
None of the above

http://insightsonindia.com

Which of the above statements is/are


correct?
a. 1 and 2 Only
b. 2 and 3 Only
c. 1 and 3 Only
d. All
8. Consider the following factors that
influence the distribution of
population around the world
1.
2.
3.
4.
5.
6.
7.

Availability of water
Soil
Industrialization
Climate
Landforms
Minerals
Urbanization

Which of the above factors have direct


influence on the distribution of
population?
Page 3

Facebook Group: Indian Administrative Service (Raz Kr)

a.
b.
c.
d.

1,2,4 and 6
1,2,4 and 5
1,2,3,4 and 7
1,2,4,5,6 and 7

9. Consider the following statements


about inbreeding which is used in
animal husbandry
1. Inbreeding refers to the
mating of more closely related
individuals within the same
breed for 4- 6 generations.
2. Inbreeding exposes harmful
recessive genes that are
eliminated by selection.
3. It helps in accumulation of
superior genes and elimination
of less desirable genes.
4. Continued
inbreeding,
especially close inbreeding,
usually reduces fertility and
even productivity. This is
called inbreeding depression.
Which of the statements given above
is/are incorrect?
a.
b.
c.
d.

1 and 2 Only
2 and 4 Only
4 Only
None

10. Consider the following factors:


1.
2.
3.
4.
5.

The size of the economy


Relief features of the country
Inheritance practices of the society
The composition of the economy
Traditional practices of the farmers

Which of the factors affects land use?


http://insightsonindia.com

a)
b)
c)
d)

1, 2 and 3 Only
2, 3 and 5 Only
1, 2, 3 and 5 Only
1, 2, 3, 4 and 5 Only

11. Consider the following statements


with reference to antibiotics
1. Antibiotics which are also
called as antimicrobial are an
agent
that
completely
eliminates microorganisms.
2. In
1928, Alexander
Fleming became the first to
discover
a
natural
antimicrobial fungus known
as penicillium rubens.
3. Ozone is an antimicrobial
agent.
Which of the above statements is/are
correct?
a.
b.
c.
d.

1 Only
1 and 2 Only
2 and 3 Only
1 and 3 Only

12. Which of the following factors affect


the value of the Indian currency in
international transactions?
1. Political stability in India
2. Economic potential of India
3. Demand of rupee in International
market
4. The difference between interest
rates between India and other countries.
Select the correct answer using the codes
below.
Page 4

Facebook Group: Indian Administrative Service (Raz Kr)

1. Growth is quantitative and


value neutral. It may have a
positive or a negative sign.
2. Development
means
a
qualitative change which is
always value positive.
3. Development occurs when
positive growth takes place.
4. Development occurs when
there is a positive change in
quality.

a) 1, 3 and 4
b) All of the above
c) 2, 3 and 4
d) 1 and 2
13. Consider the following crops:
1.
2.
3.
4.
5.

Barley
Rice
Cotton
Jute
Groundnut

Which of the above statements is/are


incorrect?

Which of these are Rabi crops?


a)
b)
c)
d)

a.
b.
c.
d.

1 and 5 Only
1, 2, and 4 Only
1, 4 and 5 Only
1, 3 and 5 Only

14. Which of the following statements


is/are true?
1. A change in nominal exchange rate
affects the real exchange rate.
2. Nominal exchange rate is affected
by the inflation rate prevailing in a country.

1 Only
2 Only
1 and 2 Only
None

16. Which of the following help stabilize


the economy in periods of slump in
economic growth?
1. Discretionary fiscal policy
2. Welfare transfers
3. Inelasticity in household spending

a) Only 1

a) 1 and 2

b) Only 2

b) All of the above

c) Both

c) Only 2

d) None

d) 1 and 3

15. Consider the following statements


about growth and development

http://insightsonindia.com

17. Consider the following statements


regarding methanogens
1. Methanogenic
populations
play an indispensable role in
Page 5

Facebook Group: Indian Administrative Service (Raz Kr)

anaerobic
wastewater
treatments
2. They live in the digestive
tracts of humans, where they
are responsible for the
methane content of flatulence
3. Methanogens are
microorganisms
that
produce methane as
a metabolic byproduct
in
anoxic conditions.
Which of the above statements is/are
correct?
a. 1 and 2 Only
b. 2 and 3 Only
c. 3 Only
d. All
18. Consider the following statements:
1. Enlarging the range of peoples
choices.
2. Political freedom.
3. A sound physical environment.
Which of the above is/are the aspect/s
of Human Development?
a)
b)
c)
d)

1 and 2 Only
1 and 3 Only
2 and 3 Only
All the above

19. Consider the following statements


about the Census data on Decadal
Population Growth Rate in India:
1. India has consistently witnessed a
Positive Decadal Population Growth
in every decade since 1901.
2. The Decadal Population Growth
Rate between 2001-11 as shown by
http://insightsonindia.com

the Census 2011 is the lowest


Decadal Population Growth Rate
since 1951.
Which of the above is/are correct?
a.
b.
c.
d.

1 Only
2 Only
Both
None

20. Consider the following statements


about beneficial microbes
1. The large holes in Swiss
cheese are due to production
of a large amount of CO2 by a
bacterium
named
Propionibacterium sharmanii.
2. The dough, which is used for
making bread, is fermented
using
a
bacteria
(Saccharomyces cerevisiae)
3. Micro-organisms
such
as
Lactobacillus
and
others
commonly called lactic acid
bacteria (LAB) grow in milk
and convert it to curd.
Which of the above is/are correct?
a.
b.
c.
d.

1 and 3 Only
2 and 3 Only
1 and 2 Only
All

21. Which of the following, inter alia, form


part of the Current Account Deficit
(CAD)?
1. Gifts and grants from abroad
2. Trade in invisibles
Page 6

Facebook Group: Indian Administrative Service (Raz Kr)

Unemployment
Poor living conditions
Political turmoil
Unpleasant climate
Natural disasters
Epidemics and socio-economic
backwardness.

useful to get rid of aphids and


mosquitoes, respectively.
4. In Indian the Government is
promoting organic farming
through various schemes like
National Project on Organic
Farming (NPOF), National
Horticulture Mission (NHM),
Horticulture Mission
for
North East & Himalayan
States (HMNEH), National
Project on Management of
Soil Health and Fertility
(NPMSH&F), Rashtriya Krishi
Vikas Yojana (RKVY) and
also Network Project on
Organic Farming of Indian
Council
of
Agricultural
Research (ICAR).

Which of the above factors can considered


as pull factors?

Which of the statements given above


is/are correct?

3. Sale and purchase of stocks


a) Only 2
b) 1 and 3
c) All of the above
d) 1 and 2
22. With
reference
to
factors
contributing to migration, consider
the following
1.
2.
3.
4.
5.
6.

a.
b.
c.
d.

1 and 2
3
5
None

23. With reference to organic farming,


consider the following statements
1. The organic farmer holds the
view that the eradication of
the creatures that are often
described as pests is not only
possible, but also undesirable.
2. A key belief of the organic
farmer is that biodiversity
furthers health.
3. The very familiar beetle with
red and black markings the
Ladybird, and Dragonflies are
http://insightsonindia.com

a. 1 and 4 Only
b. 1,2 and 4 Only
c. 1, 2 and 3 Only
d. All
24. Australias age-sex pyramid is bell
shaped and tapered towards the top.
This indicates that Australia has
a.
b.
c.
d.

Expanding populations
Constant populations
Declining populations
None of the above

25. Consider the following statements


with reference to a bioactive
molecule, cyclosporin A
1. It
is
used
as
an
immunosuppressive agent in
Page 7

Facebook Group: Indian Administrative Service (Raz Kr)

organ-transplant patients to
prevent rejection.
2. It is produced by the fungus
Trichoderma polysporum.
Which of
incorrect?
a.
b.
c.
d.

the

above

statements

is

a) 1 only
b) All of the above
c) None of the above

1
2
Both
None

26. Consider the following:


1. About 80 per cent of the coal
deposits in India is of anthracite type
and is of non-coking grade.
2. Crude
petroleum
occurs
in
sedimentary rocks of the tertiary
period
3. Uranium deposits occur in the
Laterite rocks.
4. Worlds richest monazite deposits
occur in Kerala, Andhra Pradesh and
also in Orissa
Which of the statements given above
is/are correct?
a)
b)
c)
d)

Select the correct answer using the codes


below

1 and 2 Only
2 and 4 Only
1, 3 and 4 Only
1, 2 and 4 Only

27. Fiscal deficit does not include which of


the following?
1. Non-debt creating capital receipts
2. Borrowing from RBI and other
financial institutions

d) 2 and 3

28. With reference to procedures used


in sewage water treatment, consider
the following statements.
1. The greater the Biological
Oxygen Demand of waste
water, more is its polluting
potential.
2. In the secondary treatment of
sewage water, flocs or masses
of bacteria is allowed to grow
and increase BOD levels
3. BOD refers to the amount of
the oxygen that would be
consumed if all the organic
matter in one liter of water
were oxidized by bacteria.
Which of the statements given above
is/are incorrect?
a. 1 and 2 Only
b. 2 and 3 Only
c. 2 Only
d. All
29. The Central and State Governments
have initiated many watershed
development
and
management
programmes in the country. One
such program is Haryali. Which of

3. Borrowing from abroad


http://insightsonindia.com

Page 8

Facebook Group: Indian Administrative Service (Raz Kr)

the following is/are correct regarding


Haryali?
1. It is State Government sponsored
program
2. The Project is executed by Gram
Sabha
3. Aims at enabling rural and urban
population to conserve water
4. It helps in employment generation
Select the correct code.
a.
b.
c.
d.

2 Only
2 and 4 Only
1, 2 and 3 Only
1, 2 and 4 Only

30. Which of the following can be the


possible implications of high revenue
deficit in the budget of the central
government for the Indian economy?
1. Stock of debt and interest
liabilities will pile up.
2. Reduction in social sector
expenditure
3. Low economic growth
Select the correct answer using the codes
below
a.
b.
c.
d.

1 and 2
2 and 3
1,2 and 3
1 and 3

31. Consider the following statements


about sex-ratio pattern around the
world

http://insightsonindia.com

1. A deficit of males in the


populations of many European
countries is attributed to
better status of women an
excessively male-dominated
out-migration to different
parts of the world in the past.
2. More
women
in
the
population does not mean
they have a better status. It
could be that the men might
have migrated to other areas
for employment.
3. In India, Haryana has lowest
sex ratio among states, and its
ratio has decreased further
compared to 2001 census.
Which of the statements given above
is/are correct?
a.
b.
c.
d.

1 Only
1 and 2 Only
1 and 3 Only
2 and 3 Only

32. Consider the following statements


about Single Cell Protein (SCP)
1. Microorganisms have a high
rate of multiplication and,
hence, rapid succession of
generations (algae: 26 hours,
yeast: 13 hours, bacteria: 0.5
2 hours) This principle is used
in production of SCP
2. Single-cell
protein (SCP)
typically refers to sources of
mixed protein extracted from
pure or mixed cultures
of algae, yeasts, fungi or
bacteria
Page 9

Facebook Group: Indian Administrative Service (Raz Kr)

3. They are used as a substitute


for protein-rich foods, in
human and animal feeds.
Which of the statements given above
is/are correct?
a.
b.
c.
d.

1 and 3 Only
2 and 3 Only
2 Only
All

33. Consider the following statements


about the Fiscal Responsibility and Budget
Management (FRBM) Act, 2003:
1. It strives for Inter-generational
equity in government finances.
2. It restricts borrowing from the RBI
by the Central government other
than in some specific cases.
3. It
strengthens
the
fiscal
accountability of the executive to
the legislature.
Which of the following is/are true in this
connection?
a.
b.
c.
d.

All of the above


1 and 2
2 and 3
Only 2

34. Consider the following statements


with reference to Tissue Culture
2. Micropropagation under tissue
culture refers to transfer of
desired genes into plant
saplings to produce identical
plants.
3. Explants refer to any part of a
plant taken out and grown in
http://insightsonindia.com

a test tube, under sterile


conditions in special nutrient
media.
4. Capacity to generate a whole
plant from any cell/explant is
called totipotency.
5. Each of the plants produced
by tissue culture will be
genetically identical to the
original plant from which they
were grown, i.e., they are
somaclones.
Which of the above statements
is/are correct?
a. 1,2 and 3 Only
b. 2,3 and 4 Only
c. 3 and 4 Only
d. All
35. Which of the following form part of
the non-tax revenue receipts of the
government budget?
1. Interest receipts
2. Dividend and returns from
investment made by the
government
3. Grants in aid from foreign
governments
Select the correct answer using the codes
below
a.
b.
c.
d.

1 and 2
2 and 3
All of the above
Only 1

36. Which one of the following is the


correct order of water allocation

Page 10

Facebook Group: Indian Administrative Service (Raz Kr)

priorities according to the National


Water policy, 2012?
1. Drinking
water,
Irrigation,
Hydropower,
Ecology,
Agro
industries
and
non-agricultural
industries, Navigation
2. Drinking water, Irrigation, Agro
industries
and
non-agricultural
industries, Hydropower, Ecology,
Navigation
3. Ecology, Drinking water, Irrigation,
Agro industries and non-agricultural
industries, Navigation, Hydropower
4. Drinking water, Ecology, Irrigation,
Agro industries and non-agricultural
industries, Hydropower, Navigation
37. Which of the following statements
is/are true?
1. The Capital Budget is an
account of the assets as well as
liabilities of the central
government.
2. The subsidies provided by the
Central government as a
percentage of GDP has been
declining from 1991 onwards.
3. Fiscal deficit is always
inflationary.
Select the correct answer using the codes
below
a.
b.
c.
d.

Only 1
2 and 3
1 and 3
None of the above

http://insightsonindia.com

38. Which of the following qualify to be a


part of capital receipts of the Central
government?
1. Borrowing from the RBI
2. Loans received from foreign
governments and institutions
3. PSU Disinvestment
Select the correct answer using the codes
below
a.
b.
c.
d.

1 and 3
1 and 2
2 and 3
All of the above

39. Consider the following statements


about biofertilizers
1. Biofertilisers are organisms
that enrich the nutrient
quality of the soil.
2. Bio-fertilizers do not contain
any chemicals which are
harmful to the living soil.
3. The
main
sources
of
biofertilisers are bacteria, fungi
and cyanobacteria.
4. Bio-fertilizers add nutrients
through the natural processes
of nitrogen
fixation,
solubilizing phosphorus, and
stimulating plant growth
through the synthesis of
growth-promoting
substances.
Which of the statements given above
is/are correct?
a. 2,3 and 4 Only
b. 3 and 4 Only
Page 11

Facebook Group: Indian Administrative Service (Raz Kr)

classified under the Dravidian family


of languages.

c. None
d. 2 Only
40. Consider the following statements
with reference to the population
theory put forward by Thomas
Malthus
1. He stated that the number of
people would increase faster
than the food supply
2. Any further increase would
result in a population crash
caused by famine, disease and
war.
3. The physical or positive
checks are better than the
preventive checks.
4. The preventive checks include
abortion,
birth
control,
prostitution, postponement of
marriage and celibacy
Which of the above statements is/are
correct?
a.
b.
c.
d.

1 and 2 Only
2 and 3 Only
1,2 and 4 Only
All

41. The Modern Indian Languages can


be classified into 4 Families, a
number of sub-families and groups.
Consider the following statements
about the Indian Languages:
1. The languages spoken in Central
India are classified under the Austric
Family.
2. Kashmiri language is classified under
the Indo-Aryan group.
3. Konkani language spoken in Goa is
http://insightsonindia.com

Which of the above is/are correct?


a.
b.
c.
d.

1 Only
2 Only
2 and 3 Only
1 and 2 Only

42. Expenditure on which of the following


is eligible to be a part of capital
expenditure of the Government of India?
1. Land Acquisition
2. Loans by the Central government
to the state governments and UTs.
3. Advances and loans made to the
PSUs.
Select the correct answer using the codes
below
a.
b.
c.
d.

All of the above


2 and 3
1 and 2
1 and 3

43. Which one of the following is NOT


the part of the definition of a town
as per the census of India?
1. Population density of 400 persons
per sq km.
2. Presence
of
municipality,
corporation, Cantonment etc.
3. More than 75% of the working
population engaged in non-primary
sector.
4. Population size of more than 5,000
persons.

Page 12

Facebook Group: Indian Administrative Service (Raz Kr)

44. Consider the following statements


about Multiple Ovulation Embryo
Transfer Technology (MOET)
1. It is a program for herd
improvement
for
the
successful
production
of
hybrids
2. In this method, a cow is
administered hormones, with
FSH-like activity (Follicle
Stimulating Hormone), to
induce follicular maturation
and super ovulation instead
of one egg, which they
normally yield per cycle, they
produce 6-8 eggs.
3. The animal is either mated
with an elite bull or artificially
inseminated. Fertilized eggs
are non-surgically removed
and transferred to surrogate
mothers.
4. The genetic mother is used for
another round of super
ovulation
Which of the statements given above
is/are incorrect?
a.
b.
c.
d.

2 Only
None
3 Only
3 and 4 Only

45. Which of the following relate to central


plan revenue expenditure?
1. Central Five year plans
2. Central assistance to state and
Union Territories (UTs)
http://insightsonindia.com

Select the correct answer using the codes


below
a.
b.
c.
d.

Only 1
Only 2
Both
None of the above

46. Consider the following statements


with reference to mutation breeding
1. It is the process of exposing
seeds
to chemicals or radiation in
order to generate mutants
with desirable traits to be bred
with other cultivars.
2. Mutation
breeding
is
commonly used to produce
traits in crops such as larger
seeds, new colors, or sweeter
fruits, that either cannot be
found in nature or have been
lost during evolution
3. In India, mutation induced
crops are produced by BARC
and have been released and
officially notified by the
Ministry
of
Agriculture,
Government of India for
commercial cultivation.
4. In developing and applying
mutation
breeding
using
ionising radiation, India has a
leading role among all nations.
Which of the above statements is/are
correct?
a. 1 and 3 Only
b. 1,2 and 3 Only
c. 2,3 and 4 Only
Page 13

Facebook Group: Indian Administrative Service (Raz Kr)

d. All
47. Consider the following statements
with reference to demographic
transition theory
1. In the last stage, both fertility and
mortality decline considerably. The
population is either stable or grows
slowly.
2. The first stage has low fertility and
high mortality because people
reproduce less for the fear of the
deaths due to epidemics and variable
food supply
3. Fertility remains high in the
beginning of second stage but it
declines with time. This is
accompanied by reduced mortality
rate.
4. Fertility is the natural capability to
produce offspring.
Which of the statements given above
is/are correct?
a.
b.
c.
d.

1,2,3 and 4
1,3 and 4
1,2 and 3
2,3 and 4

48. Consider the following with reference


to minerals and its distribution:
1. All minerals are exhaustible over
time.
2. Over 97 per cent of coal reserves
occur in the valleys of Damodar,
Sone, Mahanadi and Godavari
3. Bauxite is usually associated with
Sedimentary rocks
http://insightsonindia.com

4. Copper and Zinc form part of


Dharwar system of rocks
Select the correct code.
a.
b.
c.
d.

1 and 2 Only
2 Only
1, 3 and 4 Only
1, 2 and 4 Only

49. Which of the following activities would


be included in the revenue expenditure of
the central government?
1. Expenditure on building roads
2. Interest payments
3. Grants to state governments
Select the correct answer using the codes
below
a.
b.
c.
d.

1 and 3
All of the above
2 and 3
Only 2

50. Rural settlements in India can


broadly be put into four types:
Clustered,
Semi-clustered
or
fragmented, Hamleted and Dispersed
or isolated. Consider the following
statements with regard to the Rural
settlements in India:
1. In the Bundelkhand region of central
India and in Nagaland people live in
compact village for security or
defence reasons.
2. Dispersed settlements are often
caused by extremely fragmented
nature of the terrain and land
resource base of habitable areas.
Page 14

Facebook Group: Indian Administrative Service (Raz Kr)

3. In fertile alluvial plains of India, the


Dispersed settlements are commonly
found as there is a space constraint.
Which of the above is/are correct?
a.
b.
c.
d.

1 and 2 Only
1 and 3 Only
2 and 3 Only
All the above

51. With reference to commonly used


drugs by addicts, consider the
following statements
1. The
drugs,
which
are
commonly abused, opioids,
cannabinoids
and
coca
alkaloids, are obtained from
flowering plants.
2. Cocaine, commonly called
coke or crack is usually
snorted. It has a potent
stimulating action on central
nervous system, producing a
sense of euphoria and
increased energy.
3. Morphine is used to relieve
the pain for patients suffering
from terminal illnesses such as
cancer, HIV and thalassemia.
4. In India, as of now, use of
Morphine even for medical
use is completely banned
Which of the above statements is/are
correct?
a.
b.
c.
d.

1,2 and 3 Only


2 and 3 Only
3 and 4 Only
2 and 4 Only

http://insightsonindia.com

52. Consider the following statements


with reference to allergy
1. The exaggerated response of
the immune system to certain
antigens present in the
environment is called allergy.
2. Allergy is due to the release of
chemicals like histamine and
serotonin from the mast cells.
3. The use of drugs like adrenalin
and steroids quickly reduce
the symptoms of allergy.
Which of the above statements is/are
incorrect?
a.
b.
c.
d.

2 Only
3 Only
2 and 3 Only
None

53. National Income may include which


of the following:
1. Investment in the national
economy
2. External
Commercial
Borrowing
3. Indirect taxes imposed by the
government
Select the correct answer using the codes
below
a. All of the above
b. 1 only
c. 1 and 3
d. 2 and 3
54. Consider the following statements
about immunization and vaccination

Page 15

Facebook Group: Indian Administrative Service (Raz Kr)

1. The
principle
of
immunization or vaccination
is based on the property of
memory of the immune
system.
2. In vaccination, a preparation
of antigenic proteins of
pathogen
or
inactivated/weakened
pathogen
(vaccine)
are
introduced into the body.
3. The
vaccines
generate
memory B and T-cells that
recognize
the
pathogen
quickly
on
subsequent
exposure and overwhelm the
invaders with a massive
production of antibodies.
4. In cases of snakebites, the
injection which is given to the
patients, contain preformed
antigens against the snake
venom
Which of the statements given above
is/are incorrect?
a.
b.
c.
d.

3 and 4 Only
2 and 3 Only
4 Only
None

55. What amongst the following is/are


affected by high levels of government debt?
1. Inter-generational equity
2. Capital formation
3. Economic growth
Select the correct answer using the codes
below
http://insightsonindia.com

a.
b.
c.
d.

Only 3
2 and 3
1 and 2
1,2 and 3

56. Which of the following statements


is/are false?
1. The Real Exchange rate
captures
the
purchasing
power parity between two
nations.
2. The nominal exchange rate
does not represent the true
international competitiveness
of a nations economy.
Select the correct answer using the
codes below
a.
b.
c.
d.

Only 1
Only 2
Both
None

57. Consider the following statements


about immunity in human beings
1. Acquired
immunity
is
pathogen specific
2. Acquired immunity is carried
out with the help of two
special types of lymphocytes
present in our blood, BLymphocytes
and
TLymphocytes
3. The cell-mediated immune
response which is a type of
acquired
immunity
is
responsible for the graft
rejection.
Page 16

Facebook Group: Indian Administrative Service (Raz Kr)

Which of the above statements is/are


incorrect?
a.
b.
c.
d.

1 and 3 Only
3 Only
2 and 3 Only
None

58. Trade deficit is possibly affected by


which of the following factors?
1. Domestic Savings
2. Fiscal deficit
3. Investment in the economy
Select the correct answer using the codes
below

a.
b.
c.
d.

All of the above


2 and 3
1 and 2
1 and 3

59. Which of the following crops are


grown in semi-arid region?
1.
2.
3.
4.
5.
6.

Cotton
Wheat
Jowar
Maize
Pearl Millet
Groundnut

Select the correct code.


a)
b)
c)
d)

1, 3 and 4 Only
2, 3 and 5 Only
1, 3, 4 and 6 Only
1, 2, 3, 4, 5 and 6

http://insightsonindia.com

60. Which one of the following is the


correct sequence of availability of
fresh water in the decreasing order?
a. Glaciers, groundwater, freshwater
lakes, atmosphere and rivers
b. Glaciers,
freshwater
lakes,
groundwater, rivers and atmosphere
c. Glaciers, groundwater, freshwater
lakes, rivers and atmosphere
d. Glaciers,
groundwater,
rivers,
freshwater lakes and atmosphere
61. With reference to India, consider the
following Central Acts/Programs:
1. Environment Protection Act 1986
2. Water Cess Act
3. Mahatma Gandhi National Rural
Employment
Guarantee
Act
(MGNREGA)
4. Rural Drinking Water Program
Which of the above Acts have
relevance to/bearing on the water
conservation in the country?
a.
b.
c.
d.

2 Only
1 and 2 Only
1, 2 and 3 Only
1, 2, 3 and 4

62. Which of the following is/are part


of innate immunity in humans?
1. Gastrointestinal and urogenital
tracts that help in trapping
microbes entering our body
2. Acid in the stomach
3. Tears from eyes
4. Interferons
Page 17

Facebook Group: Indian Administrative Service (Raz Kr)

Select the correct answer using the codes


given below
a.
b.
c.
d.

1 and 2 Only
2 and 3 Only
1,2 and 3 Only
All

63. Consider the following statements


about Gambusia
1. It is kind of fish
2. It is introduced into ponds to
feed on mosquito larvae

65. Which one among the following sectors


has the highest proportion of the total
water used in India?
a)
b)
c)
d)

Domestic use
Irrigation
Industries
Power generation

66. Consider the following statements


about the life cycle of Plasmodium
1. Plasmodium enters the
human
body
as
sporozoites
(infectious
form) through the bite of
infected male Anopheles
mosquito.
2. The parasites initially
multiply within the liver
cells and then attack the
red blood cells (RBCs)
resulting in their rupture.
3. The rupture of RBCs is
associated with release of
a
toxic
substance,
haemozoin,
which
is
responsible for the chill
and high fever recurring
every three to four days.

Which of the statements given above


is/are correct?
a.
b.
c.
d.

1 Only
2 Only
Both
None

64. Consider the following statements


about chikungunya
1. It is a vector borne disease
2. It is caused by a virus
3. It is transmitted to humans
by virus-carrying domesticated
birds.
4. There is no specific treatment
with medications used to help
with symptoms
Which of the above statements is/are
correct?
a.
b.
c.
d.

1 and 2 Only
1,2 and 3 Only
2,3 and 4 Only
1,2 and 4 Only

http://insightsonindia.com

Which of the statements given above


is/are correct?
a.
b.
c.
d.

1 and 2 Only
2 Only
2 and 3 Only
All

Page 18

Facebook Group: Indian Administrative Service (Raz Kr)

67. Consider the following statements


about ringworms
1. Fungi are responsible for
ringworms which is one of the
most common infectious
diseases in man
2. Ringworms are generally
acquired from soil or by using
towels, clothes or even the
comb of infected individuals
3. Most fungi thrive in warm and
dry conditions
Which of the statements given above
is/are correct?
a. 1 and 2 Only
b. 2 and 3 Only
c. 1 and 3 Only
d. All
68. With reference to food grains grown in
India, consider the following statement :
1. Maize is a considered to be a food
and also a fodder crop.
2. Growing pulses increases soil
fertility.
3. Cereals occupy majority of the
cropped area
4. Maize is grown under semi-arid
conditions.
Which of the statements given above
is/are correct?
a)
b)
c)
d)

2 and 3 Only
2, 3 and 4 Only
1, 2 and 3 Only
1, 2, 3 and 4

69. Which one of the following is the main


form of degradation in irrigated areas?
http://insightsonindia.com

a)
b)
c)
d)

Wind erosion
Gully erosion
Siltation of land
Salinisation of soils

70. Consider the following statements


1. All parasites are pathogens
2. Pathogens have to adapt to life
within the environment of the
host.
3. Rhinoviruses, a pathogen, causes
common cold in humans
4. All cancers are non-pathogenic
diseases i.e. they are genetic in
origin
Which of the above statements is/are
correct?
a.
b.
c.
d.

1,2 and 3 Only


1,3 and 4 Only
2,3 and 4 Only
All

71. Consider the following statements


with reference to Malaria
1. Plasmodium, a tiny bacteria is
responsible for this disease.
2. Malignant malaria caused by
Plasmodium falciparum is the
most serious one and can even be
fatal.
3. Recently
researchers
in
Cambridge have identified a
single protein, AP2-G, which acts
as a master switch that triggers
the process in the malaria parasite
that spreads the disease.

Page 19

Facebook Group: Indian Administrative Service (Raz Kr)

Which of the above statements is/are


incorrect?
a.
b.
c.
d.

2 Only
1 Only
1 and 2 Only
None

72. Consider the following statements:

1. The area under non-agricultural uses


has increased at the expense of
wastelands and agricultural land.
2. The barren and wasteland are on the
decline.
3. The net sown area has increased.
4. The area under the forest has
increased.
Select the correct code.

1. Most of the area under permanent


pastures are owned by the village
panchayat.
2. Any land which is left uncultivated
for more than five years is known as
a culturable waste-Land.
3. The physical extent of land on
which crops are sown and harvested
is known as net sown area.
4. Land which is left without
cultivation for more than one
agricultural year is known as Current
Fallow.
Which of the statements given above
is/are correct?
a)
b)
c)
d)

3 Only
1 and 2 Only
1, 2 and 3 Only
1, 2, 3 and 4 Only

73. India has undergone major changes


within the economy over the past five
decades (1960-2012), and this has
influenced the land-use changes in the
country. With regard to this, what has
been the general trend in the land-use
pattern?

http://insightsonindia.com

a)
b)
c)
d)

1 Only
1, 2, and 4 Only
1, 2 and 3 Only
1, 2, 3 and 4 Only

74. In the context of India, Common


Property Resources (CPRs) type of land
ownership has been advocated since
decades. What is the intention behind
advocating CPR?
1. It provides fodder for the livestock
2. It provides more revenue to the
government
3. It provides livelihood for the weaker
section of the society
4. Every member can have property
rights over the land
5. Women are benefited
Select the correct code.
a)
b)
c)
d)

2, 3, and 4 Only
1, 3 and 5 Only
1, 2, 3 and 4 Only
1, 2, 3, 4 and 5 Only

Page 20

Facebook Group: Indian Administrative Service (Raz Kr)

75. With reference to spread of cancer


in human body, consider the
following statements
1. Normal cells show a property
called contact inhibition by
virtue of which contact with
other cells inhibits their
uncontrolled growth. Cancer
cells appear to have lost this
property. As a result of this,
cancerous cells just continue
to divide giving rise to masses
of cells called tumors.
2. Metastasis, the spread of
a cancer from
one organ or

http://insightsonindia.com

part to another non-adjacent


organ or part, is the most
feared property of malignant
tumors.
3. Cancer causing agents are
called as carcinogens
Which of the statements given above
is/are correct?
a.
b.
c.
d.

1 and 2 Only
1,2 and 3
2 and 3 Only
1 and 3 Only

Page 21

Facebook Group: Indian Administrative Service (Raz Kr)

INSIGHTS MOCK TEST 6 - SOLUTIONS

1. Solution: c)
Dr Mahbub-ul-Haq and Prof Amartya Sen were close friends and have worked
together under the leadership of Dr Haq to bring out the initial Human
Development Reports. Both these South Asian economists have been able to
provide an alternative view of development.
A man of vision and compassion, Pakistani economist Dr Mahbub-ul-Haq created
the Human Development Index in 1990. According to him, development is all
about enlarging peoples choices in order to lead long, healthy lives with dignity.
The United Nations Development Programme has used his concept of human
development to publish the Human Development Report annually since 1990.
Nobel Laureate Prof Amartya Sen saw an increase in freedom (or decrease in
unfreedom) as the main objective of development. Interestingly, increasing
freedoms is also one of the most effective ways of bringing about development. His
work explores the role of social and political institutions and processes in increasing
freedom.
The works of these economists are path breaking and have succeeded in bringing
people to the centre of any discussion on development.

2. Solution: A)
In 1967, gold was displaced by creating the Special Drawing Rights (SDRs), also
known as paper gold, in the IMF with the intention of increasing the stock of
international reserves. Originally defined in terms of gold, with 35 SDRs being equal
to one ounce of gold (the dollar-gold rate of the Bretton Woods system), it has
been redefined several times since 1974. At present, it is calculated daily as the
weighted sum of the values in dollars of four currencies (euro, dollar, Japanese yen,
pound sterling) of the five countries (France, Germany, Japan, the UK and the US).
It derives its strength from IMF members being willing to use it as a reserve
currency and use it as a means of payment between central banks to exchange for
national currencies. The original installments of SDRs were distributed to member

http://insightsonindia.com

INSIGHTS

Page 1

Facebook Group: Indian Administrative Service (Raz Kr)

INSIGHTS MOCK TEST 6 - SOLUTIONS


countries according to their quota in the Fund (the quota was broadly related to the
countrys economic importance as indicated by the value of its international trade)

3. Solution: a)
After getting into the body of the person, the virus enters into macrophages where
RNA genome of the virus replicates to form viral DNA with the help of the
enzyme reverse transcriptase. This viral DNA gets incorporated into host cells
DNA and directs the infected cells to produce virus particles. The macrophages
continue to produce virus and in this way acts like a HIV factory. Simultaneously,
HIV enters into helper T-lymphocytes (TH), replicates and produce progeny
viruses. The progeny viruses released in the blood attack other helper Tlymphocytes. This is repeated leading to a progressive decrease in the number of
helper T-lymphocytes in the body of the infected person.
During this period, the person suffers from bouts of fever, diarrhoea and weight
loss. Due to decrease in the number of helper T lymphocytes, the person starts
suffering from infections that could have been otherwise overcome such as those
due to bacteria especially Mycobacterium, viruses, fungi and even parasites like
Toxoplasma. The patient becomes so immuno-deficient that he/she is unable to
protect himself/herself against these infections. A widely used diagnostic test for
AIDS is enzyme linked immuno-sorbent assay (ELISA). Treatment of AIDS with
anti-retroviral drugs is only partially effective. They can only prolong the life of the
patient but cannot prevent death, which is inevitable.

4. Solution: c)
POPULATION GROWTH
The population growth or population change refers to the change in number of
inhabitants of a territory during a specific period of time. This change may be
positive as well as negative. It can be expressed either in terms of absolute numbers
or in terms of percentage. Population change in an area is an important indicator of
http://insightsonindia.com

INSIGHTS

Page 2

Facebook Group: Indian Administrative Service (Raz Kr)

INSIGHTS MOCK TEST 6 - SOLUTIONS


economic development, social upliftment and historical and cultural background of
the region.
Growth of Population : Change of population in particular area between two points
of time is known as growth of population. For example, if we deduct the
population of India 1991 (84.63 crore) from population of 2001 (102.70 crore) then
we shall get the growth of population (18.07 crores) in actual numbers.
Growth Rate of Population : This is the change of population expressed in
percentage.
Natural Growth of Population: This is the population increased by difference
between births and deaths in a particular region between two points of time.
Natural Growth = Births Deaths
Actual Growth of Population : This is Births Deaths + In Migration Out
Migration
Positive Growth of Population: This happens when the birth rate is more than the
death rate between two points of time or when people from other countries
migrate permanently to a region.
Negative Growth of Population: If the population decreases between two points of
time it is known as negative growth of population. It occurs when the birth rate
falls below the death rate or people migrate to other countries.

5. Solution: B)
The revenue deficit includes only such transactions that affect the current income
and expenditure of the government. When the government incurs a revenue
deficit, it implies that the government is dissaving and is using up the savings of the
other sectors of the economy to finance a part of its consumption expenditure. This
situation means that the government will have to borrow not only to finance its
investment but also its consumption requirements. This will lead to a build up of
stock of debt and interest liabilities and force the government, eventually, to cut
expenditure. Since a major part of revenue expenditure is committed expenditure,
it cannot be reduced. Often the government reduces productive capital
http://insightsonindia.com

INSIGHTS

Page 3

Facebook Group: Indian Administrative Service (Raz Kr)

INSIGHTS MOCK TEST 6 - SOLUTIONS


expenditure or welfare expenditure. This would mean lower growth and adverse
welfare implications.

6. Solution: A)
The proportional income tax, thus, acts as an automatic stabiliser a shock
absorber because it makes disposable income, and thus consumer spending, less
sensitive to fluctuations in GDP. When GDP rises, disposable income also rises but
by less than the rise in GDP because a part of it is siphoned off as taxes. This helps
limit the upward fluctuation in consumption spending. During a recession when
GDP falls, disposable income falls less sharply, and consumption does not drop as
much as it otherwise would have fallen had the tax liability been fixed. This
reduces the fall in aggregate demand and stabilises the economy.

7. Solution: d)
THE FOUR PILLARS OF HUMAN DEVELOPMENT
Just as any building is supported by pillars, the idea of human development is
supported by the concepts of equity, sustainability, productivity and
empowerment.
Equity refers to making equal access to opportunities available to everybody. The
opportunities available to people must be equal irrespective of their gender, race,
income and in the Indian case, caste. Yet this is very often not the case and happens
in almost every society.
For example, in any country, it is interesting to see which group the most of the
school dropouts belong to. This should then lead to an understanding of the reasons
for such behaviour. In India, a large number of women and persons belonging to
socially and economically backward groups drop out of school. This shows how the
choices of these groups get limited by not having access to knowledge.
Sustainability means continuity in the availability of opportunities. To have
sustainable human development, each generation must have the same
opportunities. All environmental, financial and human resources must be used
http://insightsonindia.com

INSIGHTS

Page 4

Facebook Group: Indian Administrative Service (Raz Kr)

INSIGHTS MOCK TEST 6 - SOLUTIONS


keeping in mind the future. Misuse of any of these resources will lead to fewer
opportunities for future generations.
A good example is about the importance of sending girls to school. If a community
does not stress the importance of sending its girl children to school, many
opportunities will be lost to these young women when they grow up. Their career
choices will be severely curtailed and this would affect other aspects of their lives.
So each generation must ensure the availability of choices and opportunities to its
future generations.
Productivity here means human labour productivity or productivity in terms of
human work. Such productivity must be constantly enriched by building
capabilities in people. Ultimately, it is people who are the real wealth of nations.
Therefore, efforts to increase their knowledge, or provide better health facilities
ultimately leads to better work efficiency.
Empowerment means to have the power to make choices. Such power comes from
increasing freedom and capability. Good governance and people-oriented policies
are required to empower people. The empowerment of socially and economically
disadvantaged groups is of special importance.

8. Solution: b)
FACTORS INFLUENCING THE DISTRIBUTION OF POPULATION
I. Geographical Factors
(i) Availability of water: It is the most factor for life. So, people prefer to live in
areas where fresh water is easily available. Water is used for drinking, bathing and
cooking and also for cattle, crops, industries and navigation. It is because of this
that river valleys are among the most densely populated areas of the world.
(ii) Landforms: People prefer living on flat plains and gentle slopes. This is because
such areas are favourable for the production of crops and to build roads and
industries. The mountainous and hilly areas hinder the development of transport
network and hence initially do not favour agricultural and industrial development.
So, these areas tend to be less populated. The Ganga plains are among the most
http://insightsonindia.com

INSIGHTS

Page 5

Facebook Group: Indian Administrative Service (Raz Kr)

INSIGHTS MOCK TEST 6 - SOLUTIONS


densely populated areas of the world while the mountains zones in the Himalayas
are scarcely populated.
(iii) Climate: An extreme climate such as very hot or cold deserts are
uncomfortable for human habitation. Areas with a comfortable climate, where
there is not much seasonal variation attract more people. Areas with very heavy
rainfall or extreme and harsh climates have low population. Mediterranean regions
were inhabited from early periods in history due to their pleasant climate.
(iv) Soils: Fertile soils are important for agricultural and allied activities. Therefore,
areas which have fertile loamy soils have more people living on them as these can
support intensive agriculture. Can you name some areas in India which are thinly
populated due to poor soils?
II. Economic Factors
(i) Minerals: Areas with mineral deposits attract industries. Mining and industrial
activities generate employment. So, skilled and semiskilled workers move to these
areas and make them densely populated. Katanga Zambia copper belt in Africa is
one such good example.
(ii) Urbanisation: Cities offer better employment opportunities, educational and
medical facilities, better means of transport and communication. Good civic (iii)
Industrialisation: Industrial belts provide job opportunities and attract large
numbers of people. These include not just factory workers but also transport
operators, shopkeepers, bank employees, doctors, teachers and other service
providers. The Kobe-Osaka region of Japan is thickly populated because of the
presence of a number of industries.
III. Social and Cultural Factors
Some places attract more people because they have religious or cultural
significance. In the same way people tend to move away from places where there
is social and political unrest. Many a times governments offer incentives to people
to live in sparsely populated areas or move away from overcrowded places.

http://insightsonindia.com

INSIGHTS

Page 6

Facebook Group: Indian Administrative Service (Raz Kr)

INSIGHTS MOCK TEST 6 - SOLUTIONS


9. Solution: d)
All are correct.
When breeding is between animals of the same breed it is called inbreeding, while
crosses between different breeds are called outbreeding.
Inbreeding : Inbreeding refers to the mating of more closely related individuals
within the same breed for 4- 6 generations. The breeding strategy is as follows
superior males and superior females of the same breed are identified and mated in
pairs.
The progeny obtained from such matings are evaluated and superior males and
females among them are identified for further mating. A superior female, in the
case of cattle, is the cow or buffalo that produces more milk per lactation. On the
other hand, a superior male is the bull, which gives rise to superior progeny as
compared to those of other males.
Inbreeding increases homozygosity. Thus inbreeding is necessary if we want to
evolve a pureline in any animal. Inbreeding exposes harmful recessive genes that are
eliminated by selection. It also helps in accumulation of superior genes and
elimination of less desirable genes. Therefore, this approach, where there is
selection at each step, increases the productivity of inbred population. However,
continued inbreeding, especially close inbreeding, usually reduces fertility and even
productivity. This is called inbreeding depression. Whenever this becomes a
problem, selected animals of the breeding population should be mated with
unrelated superior animals of the same breed. This usually helps restore fertility and
yield.

10. Solution (d)


Types of changes that an economy undergoes, which affect land-use.The size of
the economy (measured in terms of value for all the goods and services
produced in the economy)grows over time as a result of increasing population,
change in income levels, available technology and associated factors. As a result,
the pressure on land will increase with time and marginal lands would come
under use.
http://insightsonindia.com

INSIGHTS

Page 7

Facebook Group: Indian Administrative Service (Raz Kr)

INSIGHTS MOCK TEST 6 - SOLUTIONS


The composition of the economy would undergo a change over time. In other
words, the secondary and the tertiary sectors usually grow much faster than the
primary sector, specifically the agricultural sector. This type of change is
common in developing countries like India. This process would result in a
gradual shift of land from agricultural uses to non-agricultural uses. You would
observe that such changes are sharp around large urban areas. The agricultural
land is being used for building purposes.
Inheritance practices leads to land fragmentation and small size of the plots
makes it difficult for the farmer to use modern machinery.
Relief features influences the extent to which farm machinery can be used and
the rate of erosion of the soil. It also has influence over the climate.
Tradition, farmers are accustomed to doing things as they know it, therefore
they are less productive because the methods are outdated.
Reference: Page 41 (India People and Economy Class XII) &
http://prezi.com/dcgpsgprxfol/factors-affecting-land-use-physical-factorseconomic-factors-and-human-factors/
11. Solution: c)
Antibiotics do not completely eliminate microorganisms. They either kill or
inhibit their growth.

12. Solution: A)
Refer to Section 6.2 Foreign Exchange in Chapter-5, 12th Macroeconomics

13. Solution (a)


There are three distinct crop seasons in the northern and interior parts of
country, namely kharif, rabi and zaid. The kharif season largely coincides with
Southwest Monsoon under which the cultivation of tropical crops such as rice,
cotton, jute, jowar, bajra and tur is possible. The rabi season begins with the
onset of winter in October-November and ends in March-April. The crops are
Wheat, Gram, Rapeseeds Rice, Maize, Ragi, and Mustard, Barley Groundnut,
Jowar. The low temperature conditions during this season facilitate the
cultivation of temperate and subtropical crops such as wheat, gram and mustard.
http://insightsonindia.com

INSIGHTS

Page 8

Facebook Group: Indian Administrative Service (Raz Kr)

INSIGHTS MOCK TEST 6 - SOLUTIONS


Zaid is a short duration summer cropping season beginning after harvesting of
rabi crops. The cultivation of watermelons, cucumbers, vegetables and fodder
crops during this season is done on irrigated lands.
Reference: Page 44 (India People and Economy Class XII)
14. Solution: c)
Refer to Section 6.2 Foreign Exchange in Chapter 5 12th Macroeconomics

15. Solution: d)
All are correct.
Both growth and development refer to changes over a period of time. The
difference is that growth is quantitative and value neutral. It may have a positive or
a negative sign. This means that the change may be either positive (showing an
increase) or negative (indicating a decrease).
Development means a qualitative change which is always value positive. This
means that development cannot take place unless there is an increment or addition
to the existing conditions. Development occurs when positive growth takes place.
Yet, positive growth does not always lead to development. Development occurs
when there is a positive change in quality.
For example, if the population of a city grows from one lakh to two lakhs over a
period of time, we say the city has grown. However, if a facilities like housing,
provision of basic services and other characteristics remain the same, then this
growth has not been accompanied by development.

16. Solution: B)
Deliberate action to stabilise the economy is often referred to as discretionary fiscal
policy to distinguish it from the inherent automatic stabilising properties of the
fiscal system. As discussed earlier, proportional taxes help to stabilise the economy
against upward and downward movements. Welfare transfers also help to stabilise
http://insightsonindia.com

INSIGHTS

Page 9

Facebook Group: Indian Administrative Service (Raz Kr)

INSIGHTS MOCK TEST 6 - SOLUTIONS


income. During boom years, when employment is high, tax receipts collected to
finance such expenditure increase exerting a stabilising pressure on high
consumption spending; conversely, during a slump, these welfare payments help
sustain consumption. Further, even the private sector has built-in stabilisers.
Corporations maintain their dividends in the face of a change in income in the short
run and households try to maintain their previous living standards.
All these work as shock absorbers without the need for any decision-maker to take
action. That is, they work automatically. The built-in stabilisers, however, reduce
only part of the fluctuation in the economy, the rest must be taken care of by
deliberate policy initiative.

17. Solution: d)
Methanogens are microorganisms that produce methane as a metabolic byproduct
in anoxic conditions. They are classified as archaea, a domain distinct from bacteria.
They are common in wetlands, where they are responsible for marsh gas, and in the
digestive tracts of animals such as ruminants and humans, where they are
responsible for the methane content of belching in ruminants and flatulence in
humans. In marine sediments biomethanation is generally confined to
where sulfates are depleted, below the top layers. Moreover, the methanogenic
archaea populations play an indispensable role in anaerobic wastewater
treatments. Others are extremophiles, found in environments such as hot
springs and submarine hydrothermal vents as well as in the "solid" rock of the
Earth's crust, kilometers below the surface. Not to be confused
with methanotrophs which rather consume methane for their carbon and energy
requirements.

18. Solution (d)


Human development is a process of enlarging the range of peoples choices,
increasing their opportunities for education, health care, income and
empowerment and covering the full range of human choices from a sound
physical environment to economic, social and political freedom.
http://insightsonindia.com

INSIGHTS

Page 10

Facebook Group: Indian Administrative Service (Raz Kr)

INSIGHTS MOCK TEST 6 - SOLUTIONS


Reference: Page 24, Chapter 3, (India People and Economy Class XII)

19. Solution (b)


There was a negative Decadal Population Growth between 1911-1921, as shown
by the Census 1921 data.
Reference : Page 5, Chapter 1, (India People and Economy Class XII), Census 2011
data.

20. Solution: a)
The dough, which is used for making bread, is fermented using bakers yeast
(Saccharomyces cerevisiae).

21. Solution: D)
The current account records exports and imports in goods and services and transfer
payments. Trade in services denoted as invisible trade (because they are not seen to
cross national borders) includes both factor income (payment for inputs-investment
income, that is, the interest, profits and dividends on our assets abroad minus the
income foreigners earn on assets they own in India) and non-factor income
(shipping, banking, insurance, tourism, software services, etc.). Transfer payments
are receipts which the residents of a country receive for free, without having to
make any present or future payments in return. They consist of remittances, gifts
and grants. They could be official or private. The balance of exports and imports of
goods is referred to as the trade balance. Adding trade in services and net transfers
to the trade balance, we get the current account balance. The capital account
records all international purchases and sales of assets such as money, stocks, bonds,
etc. We note that any transaction resulting in a payment to foreigners is entered as a
debit and is given a negative sign. Any transaction resulting in a receipt from
foreigners is entered as a credit and is given a positive sign.
http://insightsonindia.com

INSIGHTS

Page 11

Facebook Group: Indian Administrative Service (Raz Kr)

INSIGHTS MOCK TEST 6 - SOLUTIONS


22. Solution: d)
People migrate for a better economic and social life. There are two sets of factors
that influence migration.
The Push factors make the place of origin seem less attractive for reasons like
unemployment, poor living conditions, political turmoil, unpleasant climate, natural
disasters, epidemics and socio-economic backwardness.
The Pull factors make the place of destination seem more attractive than the place
of origin for reasons like better job opportunities and living conditions, peace and
stability, security of life and property and pleasant climate.

23. Solution: d)
Biological control of pests and diseases: In agriculture, there is a method of
controlling pests that relies on natural predation rather than introduced chemicals.
A key belief of the organic farmer is that biodiversity furthers health. The more
variety a landscape has, the more sustainable it is. The organic farmer, therefore,
works to create a system where the insects that are sometimes called pests are not
eradicated, but instead are kept at manageable levels by a complex system of
checks and balances within a living and vibrant ecosystem. Contrary to the
conventional farming practices which often use chemical methods to kill both
useful and harmful life forms indiscriminately, this is a holistic approach that seeks
to develop an understanding of the webs of interaction between the myriad of
organisms that constitute the field fauna and flora. The organic farmer holds the
view that the eradication of the creatures that are often described as pests is not
only possible, but also undesirable, for without them the beneficial predatory and
parasitic insects which depend upon them as food or hosts would not be able to
survive. Thus, the use of biocontrol measures will greatly reduce our dependence
on toxic chemicals and pesticides. An important part of the biological farming
approach is to become familiar with the various life forms that inhabit the field,
predators as well as pests, and also their life cycles, patterns of feeding and the
habitats that they prefer. This will help develop appropriate means of biocontrol.
The very familiar beetle with red and black markings the Ladybird, and
Dragonflies are useful to get rid of aphids and mosquitoes, respectively. An
http://insightsonindia.com

INSIGHTS

Page 12

Facebook Group: Indian Administrative Service (Raz Kr)

INSIGHTS MOCK TEST 6 - SOLUTIONS


example of microbial biocontrol agents that can be introduced in order to control
butterfly caterpillars is the bacteria Bacillus thuringiensis (often written as Bt).
These are available in sachets as dried spores which are mixed with water and
sprayed onto vulnerable plants such as brassicas and fruit trees, where these are
eaten by the insect larvae. In the gut of the larvae, the toxin is released and the
larvae get killed. The bacterial disease will kill the caterpillars, but leave other
insects unharmed. Because of the development of methods of genetic engineering in
the last decade or so, the scientists have introduced B. thuringiensis toxin genes into
plants. Such plants are resistant to attack by insect pests. Bt-cotton is one such
example, which is being cultivated in some states of our country.
For more info:
http://www.business-standard.com/article/news-cm/organic-farming-beingpromoted-in-a-big-way-india-exporting-1-6-lakh-tonne-organic-products114022500332_1.html

24. Solution: b)
Australias age-sex pyramid is bell shaped and tapered towards the top. This shows
birth and death rates are almost equal leading to a near constant population.
Refer page no. 18 for Diagrams (Fundamentals of Human Geography, Class 12th,
NCERT)

25. Solution: d)
Cyclosporin A is an immunosuppressant drug widely used in organ
transplantation to prevent rejection. It reduces the activity of the immune system
by interfering with the activity and growth of T cells. It was initially isolated from
the fungus Tolypocladium inflatum (Beauveria nivea), found in a soil sample
obtained in 1969.
In addition to transplants, ciclosporin is also used in psoriasis, severe atopic
dermatitis,Kimura's disease, pyoderma gangrenosum, chronic autoimmune urticaria,
and, infrequently, in rheumatoid arthritis and related diseases, although it is only
http://insightsonindia.com

INSIGHTS

Page 13

Facebook Group: Indian Administrative Service (Raz Kr)

INSIGHTS MOCK TEST 6 - SOLUTIONS


used in severe cases. It is commonly prescribed in the US as an ophthalmic
emulsion for the treatment of dry eyes. It has been investigated for use in many
other autoimmune disorders, and is sometimes prescribed in veterinary cases,
particularly in extreme cases of immune-mediated hemolytic anemia. Inhaled
ciclosporin has been investigated to treat asthma and is being studied as a
preventive therapy for chronic rejection of the lungs. Ciclosporin has also been used
to help treat patients with acute severe ulcerative colitis that do not respond to
treatment with steroids. This drug is also used as a treatment of posterior or
intermediate uveitis with noninfective etiology. Ciclosporin is a drug currently used
to experimentally treat cardiac hypertrophy (an increase in cell volume).

26. Solution (b)


About 80 per cent of the coal deposits in India is of bituminous type and is of noncoking grade.
Crude petroleum occurs in sedimentary rocks of the tertiary period
Uranium deposits occur in the Dharwar rocks.
Worlds richest monazite deposits occur in Palakkad and Kollam districts of Kerala,
near Vishakhapatnam in Andhra Pradesh and Mahanadi river delta in Orissa
Reference: Pages 78, 80, 82 (India People and Economy Class XII)

27. Solution: D)
Fiscal deficit is the difference between the governments total expenditure and its
total receipts excluding borrowing
Gross fiscal deficit = Total expenditure (Revenue receipts + Non-debt creating
capital receipts)
Non-debt creating capital receipts are those receipts which are not borrowings and,
therefore, do not give rise to debt. Examples are recovery of loans and the proceeds
from the sale of PSUs. The fiscal deficit will have to be financed through
borrowing. Thus, it indicates the total borrowing requirements of the government
from all sources. From the financing side

http://insightsonindia.com

INSIGHTS

Page 14

Facebook Group: Indian Administrative Service (Raz Kr)

INSIGHTS MOCK TEST 6 - SOLUTIONS


Gross fiscal deficit = Net borrowing at home + Borrowing from RBI + Borrowing
from abroad

28. Solution: c)
Primary treatment : These treatment steps basically involve physical removal of
particles large and small from the sewage through filtration and sedimentation.
These are removed in stages; initially, floating debris is removed by sequential
filtration. Then the grit (soil and small pebbles) are removed by sedimentation. All
solids that settle form the primary sludge, and the supernatant forms the effluent.
The effluent from the primary settling tank is taken for secondary treatment.
Secondary treatment or Biological treatment : The primary effluent is passed into
large aeration tanks where it is constantly agitated mechanically and air is pumped
into it. This allows vigorous growth of useful aerobic microbes into flocs (masses of
bacteria associated with fungal filaments to form mesh like structures). While
growing, these microbes consume the major part of the organic matter in the
effluent. This significantly reduces the BOD (biochemical oxygen demand) of the
effluent. BOD refers to the amount of the oxygen that would be consumed if all
the organic matter in one liter of water were oxidised by bacteria. The sewage
water is treated till the BOD is reduced. The BOD test measures the rate of uptake
of oxygen by micro-organisms in a sample of water and thus, indirectly, BOD is a
measure of the organic matter present in the water. The greater the BOD of waste
water, more is its polluting potential.
Once the BOD of sewage or waste water is reduced significantly, the effluent is
then passed into a settling tank where the bacterial flocs are allowed to sediment.
This sediment is called activated sludge. A small part of the activated sludge is
pumped back into the aeration tank to serve as the inoculum. The remaining major
part of the sludge is pumped into large tanks called anaerobic sludge digesters.
Here, other kinds of bacteria, which grow anaerobically, digest the bacteria and the
fungi in the sludge. During this digestion, bacteria produce a mixture of gases such
as methane, hydrogen sulphide and carbon dioxide. These gases form biogas and can
be used as source of energy as it is inflammable.

http://insightsonindia.com

INSIGHTS

Page 15

Facebook Group: Indian Administrative Service (Raz Kr)

INSIGHTS MOCK TEST 6 - SOLUTIONS


29. Solution (b)
Haryali is a watershed development project sponsored by the Central
Government which aims at enabling the rural population to conserve water for
drinking, irrigation, fisheries and afforestation. The Project is being executed by
Gram Panchayats with peoples participation. One of the objective of Haryali is
Employment generation, poverty alleviation, community empowerment and
development of human and other economic resources of the rural areas. It aims
at enabling rural population to conserve water and not urban population.
Reference: Page 66, 67 (India People and Economy Class XII) &
http://dolr.nic.in/hariyaliguidelines.htm
30. Solution: c)
The revenue deficit includes only such transactions that affect the current income
and expenditure of the government. When the government incurs a revenue
deficit, it implies that the government is dissaving and is using up the savings of the
other sectors of the economy to finance a part of its consumption expenditure. This
situation means that the government will have to borrow not only to finance its
investment but also its consumption requirements. This will lead to a build up of
stock of debt and interest liabilities and force the government, eventually, to cut
expenditure. Since a major part of revenue expenditure is committed expenditure,
it cannot be reduced. Often the government reduces productive capital
expenditure or welfare expenditure. This would mean lower growth and adverse
welfare implications.

31. Solution: b)
The sex ratio is an important information about the status of women in a country.
In regions where gender discrimination is rampant, the sex ratio is bound to be
unfavourable to women. Such areas are those where the practice of female
foeticide, female infanticide and domestic violence against women are prevalent.
One of the reasons could be lower socio-economic status of women in these areas.

http://insightsonindia.com

INSIGHTS

Page 16

Facebook Group: Indian Administrative Service (Raz Kr)

INSIGHTS MOCK TEST 6 - SOLUTIONS


More women in the population does not mean they have a better status. It could be
that the men might have migrated to other areas for employment.
On an average, the world population reflects a sex ratio of 990 females per 1000
males. The highest sex ratio in the world has been recorded in Latvia which is 1187
females per 1000 males. In contrast, the lowest sex ratio occurs in U.A.E. which is
468 females per 1000 males.
The world pattern of sex ratio does not exhibit variations in the developed regions
of the world. The sex ratio is favourable for females in 139 countries of the world
and unfavourable for them in the remaining 72 countries listed by the United
Nations.
In general, Asia has a low sex ratio. Countries like China, India, Saudi Arabia,
Pakistan, Afghanistan have a lower sex ratio.
On the other extreme is greater part of Europe (including Russia) where males are
in minority. A deficit of males in the populations of many European countries is
attributed to better status of women, and an excessively male-dominated outmigration to different parts of the world in the past.
Haryana has lowest sex ratio. It showed improvement between 2001 and 2011
census from 861 to 877 females per thousand males.

32. Solution: d)
Single-cell protein (SCP) typically refers to sources of mixed protein extracted
from pure or mixed cultures of algae, yeasts, fungi or bacteria (grown on
agricultural wastes) used as a substitute for protein-rich foods, in human and animal
feeds.
It has been calculated that a 250 Kg cow produces 200 g of protein per day. In the
same period, 250g of a micro-organism like Methylophilus methylotrophus, because
of its high rate of biomass production and growth, can be expected to produce 25
tonnes of protein.
Large-scale production of microbial biomass has many advantages over the
traditional methods for producing proteins for food or feed.
http://insightsonindia.com

INSIGHTS

Page 17

Facebook Group: Indian Administrative Service (Raz Kr)

INSIGHTS MOCK TEST 6 - SOLUTIONS


1. Microorganisms have a high rate of multiplication and, hence, rapid
succession of generations (algae: 26 hours, yeast: 13 hours, bacteria: 0.52
hours)
2. They can be easily genetically modified for varying the amino acid
composition.
3. A very high protein content 4385% in the dry mass.
4. They can utilize a broad spectrum of raw materials as carbon sources, which
include even waste products. Thus, they help in the removal of pollutants
also.
5. Strains with high yield and good composition can be selected or produce
relatively easily.
6. Microbial biomass production occurs in continuous cultures and the quality
is consistent, since the growth is independent of seasonal and climatic
variations.
7. Land requirements is low and is ecologically beneficial.
8. A high solar-energy-conversion efficiency per unit area.
9. Solar energy conversion efficiency can be maximized and yield can be
enhanced by easy regulation of physical and nutritional factors.
10. Algal culture can be done in space that is normally unused and so there is no
need to compete for land.

33. Solution: A)
Refer to the section on FRBMA in Chapter 4, 12th Macroeconomics NCERT

34. Solution: b)
As traditional breeding techniques failed to keep pace with demand and to provide
sufficiently fast and efficient systems for crop improvement, another technology
called tissue culture got developed. What does tissue culture mean? It was learnt by
scientists, during 1950s, that whole plants could be regenerated from explants, i.e.,
any part of a plant taken out and grown in a test tube, under sterile conditions in
special nutrient media. This capacity to generate a whole plant from any
cell/explant is called totipotency. You will learn how to accomplish this in higher
classes. It is important to stress here that the nutrient medium must provide a
http://insightsonindia.com

INSIGHTS

Page 18

Facebook Group: Indian Administrative Service (Raz Kr)

INSIGHTS MOCK TEST 6 - SOLUTIONS


carbon source such as sucrose and also inorganic salts, vitamins, amino acids and
growth regulators like auxins, cytokinins etc. By application of these methods it is
possible to achieve propagation of a large number of plants in very short durations.
This method of producing thousands of plants through tissue culture is called
micropropagation. Each of these plants will be genetically identical to the original
plant from which they were grown, i.e., they are somaclones. Many important food
plants like tomato, banana, apple, etc., have been produced on commercial scale
using this method. Try to visit a tissue culture laboratory with your teacher to
better understand and appreciate the process.

35. Solution: c)
Revenue receipts are divided into tax and non-tax revenues.
Tax revenues consist of the proceeds of taxes and other duties levied by the central
government. Tax revenues, an important component of revenue receipts, comprise
of direct taxes which fall directly on individuals (personal income tax) and firms
(corporation tax), and indirect taxes like excise taxes (duties levied on goods
produced within the country), customs duties (taxes imposed on goods imported
into and exported out of India) and service tax.
Non-tax revenue of the central government mainly consists of interest receipts (on
account of loans by the central government which constitutes the single largest
item of non-tax revenue), dividends and profits on investments made by the
government, fees and other receipts for services rendered by the government. Cash
grants-in-aid from foreign countries and international organisations are also
included.

36. Solution (a)


Water Allocation Priorities: In the planning and operation of systems, water
allocation priorities should be broadly as follows: Drinking water, Irrigation,
Hydropower, Ecology, Agro industries and non-agricultural industries
Navigation and other uses.

http://insightsonindia.com

INSIGHTS

Page 19

Facebook Group: Indian Administrative Service (Raz Kr)

INSIGHTS MOCK TEST 6 - SOLUTIONS


Reference: Chapter 5 (India People and Economy Class XII)& Ministry of Water
resources.

37. Solution: A)
Fiscal deficit is not always inflationary. It depends whether the increased demand
for the good and services is matched by adequate new supplies.
The subsidies as a % of GDP has been decreasing till 2005-06, but has then picked
up to FY 2013-14.

38. Solution: D)
The main items of capital receipts are loans raised by the government from the
public which are called market borrowings, borrowing by the government from the
Reserve Bank and commercial banks and other financial institutions through the
sale of treasury bills, loans received from foreign governments and international
organisations, and recoveries of loans granted by the central government. Other
items include small savings (Post-Office Savings Accounts, National Savings
Certificates, etc), provident funds and net receipts obtained from the sale of shares
in Public Sector Undertakings (PSUs).

39. Solution: c)
A biofertilizer (also bio-fertilizer) is a substance which contains
living microorganisms which, when applied to seed, plant surfaces, or soil, colonizes
the rhizosphere or the interior of the plant and promotes growth by increasing the
supply or availability of primary nutrients to the host plant. Bio-fertilizers add
nutrients through the natural processes of nitrogen fixation,
solubilizing phosphorus, and stimulating plant growth through the synthesis of
growth-promoting substances. Bio-fertilizers can be expected to reduce the use
of chemical fertilizers and pesticides. The microorganisms in bio-fertilizers restore
the soil's natural nutrient cycle and build soil organic matter. Through the use of
bio-fertilizers, healthy plants can be grown, while enhancing the sustainability and
http://insightsonindia.com

INSIGHTS

Page 20

Facebook Group: Indian Administrative Service (Raz Kr)

INSIGHTS MOCK TEST 6 - SOLUTIONS


the health of the soil. Since they play several roles, a preferred scientific term for
such beneficial bacteria is "plant-growth promoting rhizobacteria" (PGPR).
Therefore, they are extremely advantageous in enriching soil fertility and fulfilling
plant nutrient requirements by supplying the organic nutrients through
microorganism and their byproducts. Hence, bio-fertilizers do not contain any
chemicals which are harmful to the living soil.

40. Solution: c)
Thomas Malthus in his theory (1793) stated that the number of people would
increase faster than the food supply. Any further increase would result in a
population crash caused by famine, disease and war. The preventive checks are
better than the physical checks. For the sustainability of our resources, the world
will have to control the rapid population increase.
Malthus argued that two types of checks hold population within resource
limits: positive checks, which raise the death rate; and preventive ones, which lower
the birth rate. The positive checks include hunger, disease and war; the preventive
checks, abortion, birth control, prostitution, postponement of marriage and
celibacy.

41. Solution (a)


Kashmiri language is classified under the Dardic-Aryan group of Indo-European
Family.
Konkani language spoken in Goa is classified under the Indo-European(Aryan)
family of languages.
Reference : Page 10, Chapter 1, (India People and Economy Class XII)

42. Solution: A)
This includes expenditure on the acquisition of land, building, machinery,
equipment, investment in shares, and loans and advances by the central government
http://insightsonindia.com

INSIGHTS

Page 21

Facebook Group: Indian Administrative Service (Raz Kr)

INSIGHTS MOCK TEST 6 - SOLUTIONS


to state and union territory governments, PSUs and other parties. Capital
expenditure is also categorised as plan and non-plan in the budget documents. Plan
capital expenditure, like its revenue counterpart, relates to central plan and central
assistance for state and union territory plans.
Non-plan capital expenditure covers various general, social and economic services
provided by the government.

43. Solution. (c)


Atleast 75 per cent of male working population engaged in non-agricultural
pursuits.
Reference : Any text on India's Urban Geography(Majid Husain), Internet.

44. Solution: b)
All are correct.
Refer page no. 168-169 Biology 12th NCERT

45. Solution: c)
Budget documents classify total revenue expenditure into plan and
non-plan expenditure. Plan revenue expenditure relates to central Plans
(the Five-Year Plans) and central assistance for State and Union Territory Plans.

46. Solution: d)
Mutation is the process by which genetic variations are created through changes in
the base sequence within genes resulting in the creation of a new character or trait
not found in the parental type. It is possible to induce mutations artificially through
use of chemicals or radiations (like gamma radiations), and selecting and using the
http://insightsonindia.com

INSIGHTS

Page 22

Facebook Group: Indian Administrative Service (Raz Kr)

INSIGHTS MOCK TEST 6 - SOLUTIONS


plants that have the desirable character as a source in breeding this process is
called mutation breeding. In mung bean, resistance to yellow mosaic virus and
powdery mildew were induced by mutations.
For more information:
http://www.thehindu.com/sci-tech/agriculture/mutation-breeding-of-oil-seedspulses-and-cereals/article5142401.ece

47. Solution: b)
DEMOGRAPHIC TRANSITION
Demographic transition theory can be used to describe and predict the future
population of any area. The theory tells us that population of any region changes
from high births and high deaths to low births and low deaths as society progresses
from rural agrarian and illiterate to urban industrial and literate society. These
changes occur in stages which are collectively known as the demographic cycle.
The first stage has high fertility and high mortality because people reproduce more
to compensate for the deaths due to epidemics and variable food supply. The
population growth is slow and most of the people are engaged in agriculture where
large families are an asset. Life expectancy is low, people are mostly illiterate and
have low levels of technology. Two hundred years ago all the countries of the world
were in this stage.
Fertility remains high in the beginning of second stage but it declines with time.
This is accompanied by reduced mortality rate. Improvements in sanitation and
health conditions lead to decline in mortality. Because of this gap the net addition
to population is high.
In the last stage, both fertility and mortality decline considerably. The population is
either stable or grows slowly. The population becomes urbanised, literate and has
high technical know-how and deliberately controls the family size.
This shows that human beings are extremely flexible and are able to adjust their
fertility.

http://insightsonindia.com

INSIGHTS

Page 23

Facebook Group: Indian Administrative Service (Raz Kr)

INSIGHTS MOCK TEST 6 - SOLUTIONS


In the present day, different countries are at different stages of demographic
transition.

48. Solution (d)


Minerals have certain characteristics. These are unevenly distributed over space.
All minerals are exhaustible over time. These take long to develop geologically
and they cannot be replenished immediately at the time of need. Thus, they
have to be conserved and not misused as they do not have the second crop.
Most of the metallic minerals in India occur in the peninsular plateau region in
the old crystalline rocks. Over 97 per cent of coal reserves occur in the valleys of
Damodar, Sone, Mahanadi and Godavari. Petroleum reserves are located in the
sedimentary basins of Assam, Gujarat and Mumbai High i.e. off-shore region in
the Arabian Sea. New reserves have been located in the Krishna-Godavari and
Kaveri basins.
Bauxite is found mainly in tertiary deposits and is associated with laterite rocks
occurring extensively either on the plateau or hill ranges of peninsular India and
also in the coastal tracts of the country.
Dharwar system of rocks. Copper, zinc have been major minerals.
Reference: Pages 73-76 (India People and Economy Class XII)

49. Solution: c)
Broadly speaking, revenue expenditure consists of all those expenditures of the
government which do not result in creation of physical or financial assets. It relates
to those expenses incurred for the normal functioning of the government
departments and various services, interest payments on debt incurred by the
government, and grants given to state governments and other parties (even though
some of the grants may be meant for creation of assets).

50. Solution. (a)


In fertile alluvial plains of India, Clustered settlements are found

http://insightsonindia.com

INSIGHTS

Page 24

Facebook Group: Indian Administrative Service (Raz Kr)

INSIGHTS MOCK TEST 6 - SOLUTIONS


Reference : Page 33, Chapter 4, (India People and Economy Class XII)

51. Solution: a)
Accessibility to morphine will improve further with the passing of The Narcotic
Drugs and Psychotropic Substances (Amendment) Bill, 2011. Morphine is essential
for patients suffering from terminal illnesses such as cancer, HIV and thalassemia.
Morphine is one of the cheapest and the best known pain-relieving drug. With the
new act, which was passed by the Rajya Sabha, there will be a single window
clearance to medical institutions wanting to store morphine.
Earlier 4-5 licenses were required to store morphine in hospitals for therapeutic use.
Drugs
The drugs, which are commonly abused are opioids, cannabinoids and coca
alkaloids. Majority of these are obtained from flowering plants. Some are obtained
from fungi.
Opioids are the drugs, which bind to specific opioid receptors present in our
central nervous system and gastrointestinal tract. Heroin, commonly called smack is
chemically diacetylmorphine which is a white, odourless, bitter crystalline
compound. This is obtained by acetylation of morphine, which is extracted from
the latex of poppy plant Papaver somniferum. Generally taken by snorting and
injection, heroin is a depressant and slows down body functions.
Cannabinoids are a group of chemicals , which interact with cannabinoid receptors
present principally in the brain. Natural cannabinoids are obtained from the
inflorescences of the plant Cannabis sativa. The flower tops, leaves and the resin of
cannabis plant are used in various combinations to produce marijuana, hashish,
charas and ganja. Generally taken by inhalation and oral ingestion, these are known
for their effects on cardiovascular system of the body.
Coca alkaloid or cocaine is obtained from coca plant Erythroxylum coca, native to
South America. It interferes with the transport of the neuro-transmitter dopamine.
Cocaine, commonly called coke or crack is usually snorted. It has a potent
stimulating action on central nervous system, producing a sense of euphoria and
increased energy. Excessive dosage of cocaine causes hallucinations. Other wellhttp://insightsonindia.com

INSIGHTS

Page 25

Facebook Group: Indian Administrative Service (Raz Kr)

INSIGHTS MOCK TEST 6 - SOLUTIONS


known plants with hallucinogenic properties are Atropa belladona and Datura.
These days cannabinoids are also being abused by some sportspersons.
Drugs like barbiturates, amphetamines, benzodiazepines, lysergic acid diethyl
amides (LSD), and other similar drugs, that are normally used as medicines to help
patients cope with mental illnesses like depression and insomnia, are often abused.
Morphine is a very effective sedative and painkiller, and is very useful in patients
who have undergone surgery. Several plants, fruits and seeds having hallucinogenic
properties have been used for hundreds of years in folk-medicine, religious
ceremonies and rituals all over the globe. When these are taken for a purpose other
than medicinal use or in amounts/frequency that impairs ones physical,
physiological or psychological functions, it constitutes drug abuse.

52. Solution: d)
The exaggerated response of the immune system to certain antigens present in the
environment is called allergy. The substances to which such an immune response is
produced are called allergens. The antibodies produced to these are of IgE type.
Common examples of allergens are mites in dust, pollens, animal dander, etc.
Symptoms of allergic reactions include sneezing, watery eyes, running nose and
difficulty in breathing. Allergy is due to the release of chemicals like histamine and
serotonin from the mast cells. For determining the cause of allergy, the patient is
exposed to or injected with very small doses of possible allergens, and the reactions
studied. The use of drugs like anti-histamine, adrenalin and steroids quickly reduce
the symptoms of allergy. Somehow, modern-day life style has resulted in lowering
of immunity and more sensitivity to allergens more and more children in metro
cities of India suffer from allergies and asthma due to sensitivity to the
environment. This could be because of the protected environment provided early
in life.

53. Solution: c)
National Income = Consumption + Investment + Government spending( It
includes all taxes and sources of income also.)
http://insightsonindia.com

INSIGHTS

Page 26

Facebook Group: Indian Administrative Service (Raz Kr)

INSIGHTS MOCK TEST 6 - SOLUTIONS


It does not include external commercial borrowing, which is included in the
external capital account.

54. Solution: c)
The principle of immunisation or vaccination is based on the property of memory
of the immune system. In vaccination, a preparation of antigenic proteins of
pathogen or inactivated/weakened pathogen (vaccine) are introduced into the
body. The antibodies produced in the body against these antigens would neutralise
the pathogenic agents during actual infection. The vaccines also generate memory
B and T-cells that recognize the pathogen quickly on subsequent exposure and
overwhelm the invaders with a massive production of antibodies. If a person is
infected with some deadly microbes to which quick immune response is required
as in tetanus, we need to directly inject the preformed antibodies, or antitoxin (a
preparation containing antibodies to the toxin). Even in cases of snakebites, the
injection which is given to the patients, contain preformed antibodies against the
snake venom. This type of immunisation is called passive immunisation.

55. Solution: D)
By borrowing, the government transfers the burden of reduced consumption on
future generations. This is because it borrows by issuing bonds to the people living
at present but may decide to pay off the bonds some twenty years later by raising
taxes. These may be levied on the young population that have just entered the
work force, whose disposable income will go down and hence consumption. Thus,
national savings, it was argued would fall. Also, government borrowing from the
people reduces the savings available to the private sector. To the extent that this
reduces capital formation and growth, debt acts as a burden on future generations.
Traditionally, it has been argued that when a government cuts taxes and runs a
budget deficit, consumers respond to their after-tax income by spending more. It is
possible that these people are short-sighted and do not understand the implications
of budget deficits. They may not realise that at some point in the future, the
government will have to raise taxes to pay off the debt and accumulated interest.

http://insightsonindia.com

INSIGHTS

Page 27

Facebook Group: Indian Administrative Service (Raz Kr)

INSIGHTS MOCK TEST 6 - SOLUTIONS


Even if they comprehend this, they may expect the future taxes to fall not on them
but on future generations.

56. Solution: D)
Refer to Section 6.2 Foreign Exchange in Chapter 5 12th Macroeconomics

57. Solution: d)
All are correct.
Acquired immunity, on the other hand, is pathogen specific. It is characterised by
memory. This means that our body when it encounters a pathogen for the first time
produces a response called primary response which is of low intensity. Subsequent
encounter with the same pathogen elicits a highly intensified secondary or
anamnestic response. This is ascribed to the fact that our body appears to have
memory of the first encounter.
The primary and secondary immune responses are carried out with the help of two
special types of lymphocytes present in our blood, B-Lymphocytes and TLymphocytes. The B-lymphocytes produce an army of proteins in response to
pathogens into our blood to fight with them. These proteins are called antibodies.
The T-cells themselves do not secrete antibodies but help B cells produce them.
Each antibody molecule has four peptide chains, two small called light chains and
two longer called heavy chains. Hence, an antibody is represented as H2L2.
Different types of antibodies are produced in our body. IgA, IgM, IgE, IgG are some
of them. Because these antibodies are found in the blood, the response is also called
as humoral immune response. This is one of the two types of our acquired immune
response antibody mediated. The second type is called cell-mediated immune
response or cell-mediated immunity (CMI). The T-lymphocytes mediate CMI.
Very often, when some human organs like heart, eye, liver, kidney fail to function
satisfactorily, transplatation is the only remedy to enable the patient to live a
normal life. Then a search begins to find a suitable donor Grafts from just any
source an animal, another primate, or any human beings cannot be made since the
grafts would be rejected sooner or later. Tissue matching, blood group matching are
http://insightsonindia.com

INSIGHTS

Page 28

Facebook Group: Indian Administrative Service (Raz Kr)

INSIGHTS MOCK TEST 6 - SOLUTIONS


essential before undertaking any graft/transplant and even after this the patient has
to take immunosuppresants all his/her life. The body is able to differentiate self
and nonself and the cell-mediated immune response is responsible for the graft
rejection.

58. Solution: A)
When a country runs a trade deficit, it is important to look at the right side of
equation to see whether there has been a decrease in saving, increase in investment,
or an increase in the budget deficit. There is reason to worry about a countrys longrun prospects if the trade deficit reflects smaller saving or a larger budget deficit
(when the economy has both trade deficit and budget deficit, it is said to be facing
twin deficits). The deficit could reflect higher private or government consumption.
In such cases, the countrys capital stock will not rise rapidly enough to yield
enough growth (called the growth dividend) it needs to repay its debt. There is
less cause to worry if the trade deficit reflects a rise in investment, which will build
the capital stock more quickly and increase future output. However, we must note
that since private saving, investment and the trade deficit are jointly determined,
other factors too must be taken into account.

59. Solution (d)


A semi-arid climate or steppe climate are climatic regions that receive precipitation
below potential evapotranspiration, but not extremely.
Cotton is a tropical crop grown in kharif season in semi-arid areas of the country.
Wheat being a rabi crop, it is mostly grown under irrigated conditions at high
altitudes and it is also grown as a rainfed crop in the semi-arid region.
Jowar is the main food crop in semi-arid areas of central and southern India.
Maharashtra alone produces more than half of the total jowar production of the
country.
Maize is a food as well as fodder crop grown under semi-arid climatic conditions
and over inferior soils.
http://insightsonindia.com

INSIGHTS

Page 29

Facebook Group: Indian Administrative Service (Raz Kr)

INSIGHTS MOCK TEST 6 - SOLUTIONS


Pearl Millet is one of the major rainfed cereal that is grown in semi-arid region.
Groundnut is also grown in semi-arid region.
Reference: Chapter 5 (India People and Economy Class XII)

60. Solution (a)


Glaciers, groundwater, freshwater lakes, salt lakes, atmosphere and rivers
Reference: Chapter 6 (India People and Economy Class XII)
61. Solution (c)
Percolation tanks would be constructed for all borewells owned by the
government. These works are done under MGNREGA, which helps in water
conservation.
Water (Prevention and Control of Pollution) Act 1974 and Environment
Protection Act 1986 help in control of water pollution. Water Cess Act 1977, is
also meant to reduce pollution.
Rural Drinking Water Program is not concerned with conservation of water.
Reference: Page 66, 67 (India People and Economy Class XII)

62. Solution: d)
Innate immunity is non-specific type of defence, that is present at the time of birth.
This is accomplished by providing different types of barriers to the entry of the
foreign agents into our body. Innate immunity consist of four types of barriers.
These are
(i) Physical barriers : Skin on our body is the main barrier which prevents entry of
the micro-organisms. Mucus coating of the epithelium lining the respiratory,
gastrointestinal and urogenital tracts also help in trapping microbes entering our
body.

http://insightsonindia.com

INSIGHTS

Page 30

Facebook Group: Indian Administrative Service (Raz Kr)

INSIGHTS MOCK TEST 6 - SOLUTIONS


(ii) Physiological barriers : Acid in the stomach, saliva in the mouth, tears from
eyesall prevent microbial growth.
(iii) Cellular barriers : Certain types of leukocytes (WBC) of our body like
polymorpho-nuclear leukocytes (PMNL-neutrophils) and monocytes and natural
killer (type of lymphocytes) in the blood as well as macrophages in tissues can
phagocytose and destroy microbes.
(iv) Cytokine barriers : Virus-infected cells secrete proteins called interferons
which protect non-infected cells from further viral infection.

63. Solution: c)
Gambusia is a large genus of fish in family Poeciliidae. Gambusia contains over 40
species, most of which are principally found in freshwater habitats, though some
species may also be found in brackish or saltwater habitats. Gambusiaspecies are
often called topminnows or simply gambusias; they are also known as mosquitofish,
which, however, refers more specifically to two species, G. affinis and G. holbrooki.
These can be introduced into ponds to eat mosquito larvae. As a consequence they
have been introduced widely outside their native range, and sometimes
become invasive, threatening the local species.They are very important in aquarium
trade, desired for small size, ease of breeding, and charming gracefulness. They are
viviparousthey have live young.

64. Solution: d)
Chikungunya virus (CHIKV) is an arthropod-borne virus, of the genus Alphavirus,
that is transmitted to humans by virus-carrying Aedes mosquitoes.There have been
recent breakouts of CHIKV associated with severe illness.
CHIKV infection causes an illness with a similar mode of transmission as dengue
fever, with an acute febrile phase lasting two to five days, followed by a longer
period of joint pains in the extremities. The pain associated with CHIKV infection
of the joints may persist for weeks or months, or in some cases years. Prevention is
via mosquito control and preventing bite by infected mosquitoes. There is no
specific treatment with medications used to help with symptoms.

http://insightsonindia.com

INSIGHTS

Page 31

Facebook Group: Indian Administrative Service (Raz Kr)

INSIGHTS MOCK TEST 6 - SOLUTIONS


65. Solution (b)
Reference: Chapter 6 (India People and Economy Class XII)
66. Solution: c)
Plasmodium enters the human body as sporozoites (infectious form) through the
bite of infected female Anopheles mosquito. The parasites initially multiply within
the liver cells and then attack the red blood cells (RBCs) resulting in their rupture.
The rupture of RBCs is associated with release of a toxic substance, haemozoin,
which is responsible for the chill and high fever recurring every three to four days.
When a female Anopheles mosquito bites an infected person, these parasites enter
the mosquitos body and undergo further development. The parasites multiply
within them to form sporozoites that are stored in their salivary glands. When these
mosquitoes bite a human, the sporozoites are introduced into his/ her body,
thereby initiating the events mentioned above. It is interesting to note that the
malarial parasite requires two hosts human and mosquitoes to complete its life
cycle; the female Anopheles mosquito is the vector (transmitting agent) too.

67. Solution: a)
Many fungi belonging to the genera Microsporum, Trichophyton and
Epidermophyton are responsible for ringworms which is one of the most common
infectious diseases in man. Appearance of dry, scaly lesions on various parts of the
body such as skin, nails and scalp. are the main symptoms of the disease. These
lesions are accompanied by intense itching. Heat and moisture help these fungi to
grow, which makes them thrive in skin folds such as those in the groin or between
the toes. Ringworms are generally acquired from soil or by using towels, clothes or
even the comb of infected individuals.

68. Solution (d)


The cereals occupy about 54 per cent of total cropped area in India. The country
produces about 11 per cent cereals of the world and ranks third in production
after China and U.S.A. India produces a variety of cereals, which are classified as
http://insightsonindia.com

INSIGHTS

Page 32

Facebook Group: Indian Administrative Service (Raz Kr)

INSIGHTS MOCK TEST 6 - SOLUTIONS


fine grains (rice, wheat) and coarse grains (jowar, bajra, maize, ragi), etc. Maize is
a food as well as fodder crop grown under semi-arid climatic conditions and
over inferior soils.Pulses are legume crops which increase the natural fertility of
soils through nitrogen fixation.
Reference: Page 47 (India People and Economy Class XII)

69. Solution (d)


Many irrigation projects, past and present, are severely affected by secondary
salinization or sodication and waterlogging. Secondary salinization of soil is the
main degradation process in arid and semi-arid areas.
Reference: Chapter 5 (India People and Economy Class XII)
70. Solution: a)
A wide range of organisms belonging to bacteria, viruses, fungi, protozoans,
helminths, etc could cause diseases in man. Such disease causing organisms are
called Pathogens. All parasites are therefore pathogens as they cause harm to the
host by living in (or on) them. The pathogens can enter our body by various means,
multiply and interfere with normal vital activities, resulting in morphological and
functional damage. Pathogens have to adapt to life within the environment of the
host. For example, the pathogens that enter the gut must know a way of surviving
in the stomach at low pH and resisting the various digestive enzymes.
Many viruses also cause diseases in human beings. Rhino viruses represent one such
group of viruses which cause one of the most infectious human ailments the
common cold. They infect the nose and respiratory passage but not the lungs. The
common cold is characterised by nasal congestion and discharge, sore throat,
hoarseness, cough, headache, tiredness, etc., which usually last for 3-7 days. Droplets
resulting from cough or sneezes of an infected person are either inhaled directly or
transmitted through contaminated objects such as pens, books, cups, doorknobs,
computer keyboard or mouse, etc., and cause infection in a healthy person.
An oncovirus is a virus that can cause cancer. This term originated from studies of
acutely transforming retroviruses in the 195060s, often called oncornaviruses to
http://insightsonindia.com

INSIGHTS

Page 33

Facebook Group: Indian Administrative Service (Raz Kr)

INSIGHTS MOCK TEST 6 - SOLUTIONS


denote their RNA virus origin. It now refers to any virus with a DNA or RNA
genome causing cancer and is synonymous with "tumor virus" or "cancer virus".
The vast majority of human and animal viruses do not cause cancer, probably
because of long-standing coevolution between the virus and its host.
Worldwide, the WHO International Agency for Research on Cancer estimated that
in 2002 17.8% of human cancers were caused by infection, with 11.9% being caused
by one of seven different viruses. The importance of this is that these cancers might
be easily prevented through vaccination (e.g., papillomavirus vaccines), diagnosed
with simple blood tests, and treated with less-toxic antiviral compounds.

71. Solution: b)
Some of the human diseases are caused by protozoans too. You might have heard
about malaria, a disease man has been fighting since many years. Plasmodium, a tiny
protozoan is responsible for this disease. Different species of Plasmodium (P. vivax,
P. malaria and P. falciparum) are responsible for different types of malaria. Of these,
malignant malaria caused by Plasmodium falciparum is the most serious one and
can even be fatal.
Article from The Hindu
Regarding third statement
http://www.thehindu.com/sci-tech/health/medicine-and-research/breakthroughin-malaria-research-claimed/article5722806.ece

72. Solution (c)


Area under Permanent Pastures and Grazing Lands : Most of this type land is
owned by the village Panchayat or the Government. Only a small proportion of
this land is privately owned. The land owned by the village panchayat comes under
Common Property Resources.
Culturable Waste-Land: Any land which is left fallow (uncultivated) for more than
five years is included in this category. It can be brought under cultivation after
improving it through reclamation practices.
Current Fallow: This is the land which is left without cultivation for one or less
than one agricultural year. Fallowing is a cultural practice adopted for giving the
land rest. The land recoups the lost fertility through natural processes.
http://insightsonindia.com

INSIGHTS

Page 34

Facebook Group: Indian Administrative Service (Raz Kr)

INSIGHTS MOCK TEST 6 - SOLUTIONS


Net Area Sown: The physical extent of land on which crops are sown and
harvested is known as net sown area.
Reference: Page 41 (India People and Economy Class XII) & online
73. Solution (b)
The area under non-agricultural uses is increasing at the expense of wastelands
and agricultural land. The rate of increase is the highest in case of area under
non-agricultural uses. This is due to the changing structure of Indian economy,
which is increasingly depending on the contribution from industrial and services
sectors and expansion of related infrastructural facilities. Also, an expansion of
area under both urban and rural settlements has added to the increase. Thus, the
area under non-agricultural uses is increasing at the expense of wastelands and
agricultural land.
The four categories that have registered a decline are barren and wasteland,
culturable wasteland, area under pastures and tree crops and net area sown.
As per the present assessment, the Forest and Tree cover of the country is 78.29
million ha, which is 23.81% of the geographical area of the country. In 1960 it
was around 18%.
Reference: Page 42 (India People and Economy Class XII) & PIB
74. Solution (b)
Land, according to its ownership can broadly be classified under two broad
heads private land and common property resources (CPRs). While the former
is owned by an individual or a group of individuals, the latter is owned by the
state meant for the use of the community. CPRs provide fodder for the livestock
and fuel for the households along with other minor forest products like fruits,
nuts, fibre, medicinal plants, etc. In rural areas, such land is of particular
relevance for the livelihood of the landless and marginal farmers and other
weaker sections since many of them depend on income from their livestock due
to the fact that they have limited access to land. CPRs also are important for
women as most of the fodder and fuel collection is done by them in rural areas.
They have to devote long hours in collecting fuel and fodder from a degraded
http://insightsonindia.com

INSIGHTS

Page 35

Facebook Group: Indian Administrative Service (Raz Kr)

INSIGHTS MOCK TEST 6 - SOLUTIONS


area of CPR. CPRs can be defined as communitys natural resource, where every
member has the right of access and usage with specified obligations, without
anybody having property rights over them. Community forests, pasture lands,
village water bodies and other public spaces where a group larger than a
household or family unit exercises rights of use and carries responsibility of
management are examples of CPRs.
Reference: Page 43 (India People and Economy Class XII)
75. Solution: b)
In our body, cell growth and differentiation is highly controlled and regulated. In
cancer cells, there is breakdown of these regulatory mechanisms. Normal cells show
a property called contact inhibition by virtue of which contact with other cells
inhibits their uncontrolled growth. Cancer cells appears to have lost this property.
As a result of this, cancerous cells just continue to divide giving rise to masses of
cells called tumors.
Tumors are of two types: benign and malignant. Benign tumors normally remain
confined to their original location and do not spread to other parts of the body and
cause little damage. The malignant tumors, on the other hand are a mass of
proliferating cells called neoplastic or tumor cells. These cells grow very rapidly,
invading and damaging the surrounding normal tissues. As these cells actively divide
and grow they also starve the normal cells by competing for vital nutrients. Cells
sloughed from such tumors reach distant sites through blood, and wherever they
get lodged in the body, they start a new tumor there. This property called
metastasis is the most feared property of malignant tumors.

http://insightsonindia.com

INSIGHTS

Page 36

Facebook Group: Indian Administrative Service (Raz Kr)

INSIGHTS ON INDIA MOCK PRELIMINARY EXAM - 2014


INSIGHTS ON INDIA MOCK TEST - 7
GENERAL STUDIES

PAPER-I
Time Allowed: Two Hours

Maximum Marks: 200

INSTRUCTIONS
1. IMMEDITELY AFTER THE COMMENCEMENT OF THE EXAMINATION, YOU SHOULD
CHECK THAT THIS TEST BOOKLET DOES NOT HAVE ANY UNPRINTED OR TORN OR
MISSING PAGES OR ITEMS, ETC. IF SO, GET IT REPLACED BY A COMPLETE TEST BOOKLET.
2. You have to enter your Roll Number on the Test I
Booklet in the Box provided alongside. DO NOT
write anything else on the Test Booklet.
4. This Test Booklet contains 100 items (questions). Each item is printed only in English. Each item
comprises four responses (answers). You will select the response which you want to mark on the Answer
Sheet. In case you feel that there is more than one correct response, mark the response which you consider
the best. In any case, choose ONLY ONE response for each item.
5. You have to mark all your responses ONLY on the separate Answer Sheet provided. See directions in the
Answer Sheet.
6. All items carry equal marks.
7. Before you proceed to mark in the Answer Sheet the response to various items in the Test Booklet, you
have to fill in some particulars in the Answer Sheet as per instructions sent to you with your Admission
Certificate.
8. After you have completed filling in all your responses on the Answer Sheet and the examination has
concluded, you should hand over to the Invigilator only the Answer Sheet. You are permitted to take away
with you the Test Booklet.
9. Sheets for rough work are appended in the Test Booklet at the end.
10. Penalty for wrong answers :
THERE WILL BE PENALTY FOR WRONG ANSWERS MARKED BY A CANDIDATE IN THE
OBJECTIVE TYPE QUESTION PAPERS.
(i)

There are four alternatives for the answer to every question. For each question for which a
wrong answer has been given by the candidate, one-third of the marks assigned to that question
will be deducted as penalty.

(ii) If a candidate gives more than one answer, it will be treated as a wrong answer even if one of the
given answers happens to be correct and there will be same penalty as above to that question.
(iii)

If a question is left blank, i.e., no answer is given by the candidate, there will be no penalty for
that question.

http://insightsonindia.com

INSIGHTS ON INDIA MOCK TEST SERIES FOR CIVIL SERVICES PRELIMINARY EXAM 2014

http://insightsonindia.com

Page 1

Facebook Group: Indian Administrative Service (Raz Kr)

1. With reference to buffer stock of


foodgrains, consider the following
statements
1. Buffer Stock is the stock of
foodgrains, namely wheat and rice
procured by the government
through Food Corporation of
India (FCI).
2. Buffer stocks are required to
stabilize prices during period of
production shortfall through
open market sales.
3. The seasonality of production and
procurement is a decisive factor
in determining the minimum
norm of food grains stocks
required in a particular quarter
of the year
4. In addition to buffer norms,
Government
has
prescribed
Strategic Reserve of wheat and
rice
Which of
incorrect?
a.
b.
c.
d.

the

above

statements

is

2 Only
3 Only
2 and 4 Only
None

2. With reference to unemployment,


consider the following statements
1. In case of rural areas, there is
seasonal
and
disguised
unemployment. Urban areas have
mostly educated unemployment.

http://insightsonindia.com

2. Disguised unemployment usually


happens among family members
engaged in agricultural activity.
3. According to ILO report,
unemployment rate in India is
increasing since 2011
Which of the statements given above
is/are correct?
a.
b.
c.
d.

1 and 2 Only
2 and 3 Only
2 Only
All

3. Consider the following statements


with reference to National Food for
Work programme
1. It was launched on November 14,
2004 in 150 most backward
districts of the country with the
objective of intensifying the
generation of supplementary
wage employment.
2. The programme is open to all
rural poor who are in need of
wage employment and desire to
do manual unskilled work.
3. This programme, along with
NREGA is still run in these 150
districts as a supplementary
program to benefit the poor
Which of the statements given above
is/are correct?
a.
b.
c.
d.

1 and 3 Only
1 and 2 Only
2 Only
All

Page 2

Facebook Group: Indian Administrative Service (Raz Kr)

4. Analysis of poverty based on


social exclusion and vulnerability
is now becoming very common.
With reference to this, consider
the following statements
1. According to social exclusion
concept, poverty must be seen
in terms of the poor having to
live only in a poor surrounding
with other poor people,
excluded from enjoying social
equality of better-off people
in better surroundings.
2. The working of the caste
system in India is an example
of social exclusion
3. Social exclusion may lead to,
but can cause more damage
than, having a very low
income.
4. The N. C Saxena Report
(from
the
National
Commission for Enterprises in
the Unorganised Sector),
based on data between the
period 199394 and 200405,
states that 77% of Indians live
on less than 20 a day
5. Vulnerability to poverty is a
measure, which describes the
greater probability of certain
communities or individuals of
becoming, or remaining, poor
in the coming years.
Which of the statements given above
is/are correct?
a. 1 and 3 Only
b. 1,2,3 and 4 Only
c. 1,2,3 and 5 Only
http://insightsonindia.com

d. All

5. With reference to Public


Distribution
System
(PDS),
consider the following statements
1. PDS, till 1992, was a general
entitlement scheme for all
consumers
without
any
specific target.
2. In
June
1997,
the
Government
of
India
launched the Targeted Public
Distribution System (TPDS)
with focus on the poor. The
identification of the poor
under the scheme is done by
the States as per State-wise
poverty estimates of the
Planning Commission for
1993-94
based
on
the
methodology of the Expert
Group on estimation of
proportion and number of
poor chaired by Late Prof
Lakdawala.
3. In order to make TPDS more
focused and targeted towards
poor, the Antyodaya Anna
Yojana (AAY) was launched
in December, 2000 for one
crore poorest of the poor
families.
4. Under national food security
act 2013, reforms include
door-step
delivery
of
foodgrains to the TPDS
outlets,
application
of
information
and
communication
technology
Page 3

Facebook Group: Indian Administrative Service (Raz Kr)

tools,
diversification
of
commodities
distributed
under the PDS over period of
time etc.
Which of the statements given above
is/are incorrect?
a.
b.
c.
d.

1 and 3 Only
3 and 4 Only
2 and 3 Only
None

6. With reference to jet streams,


consider the following statements
1. Over
India,
subtropical
easterly jet streams blow
south of the Himalayas, all
through the year except in
summer
2. In summer, the subtropical
easterly jet stream moves
north of the Himalayas with
the apparent movement of the
sun.
3. An easterly jet stream, called
the tropical westerly jet
stream blows over peninsular
India, approximately over
14N during the summer
months.
4. Jet streams are a narrow belt
of high altitude (above 12,000
m) westerly winds in the
troposphere. Their speed
varies from about 110 km/h
in summer to about 184 km/h
in winter.

http://insightsonindia.com

Which of the statements given above


is/are correct?
a.
b.
c.
d.

1,2 and 3 Only


All
2,3 and 4 Only
4 Only

7. With reference to population


density in India, consider the
following statements
1. The Northern Plains and
Kerala in the south have high
to very high population
densities because of the flat
plains with fertile soils and
abundant rainfall.
2. Assam and most of the
Peninsular
states
have
moderate population densities.
Hilly, dissected and rocky
nature
of
the
terrain,
moderate to low rainfall,
shallow and less fertile soils
have influenced population
densities in these areas.
3. India supports and sustains
17.5% of the world population
and there is an increase of
17.5% population in 2011 over
2001.
Which of the statements given above
is/are correct?
a.
b.
c.
d.

1 and 3 Only
2 and 3 Only
1 and 2 Only
All
Page 4

Facebook Group: Indian Administrative Service (Raz Kr)

8. Consider
statements

the

following

1. These
are
the
most
widespread forests of India.
They are also called the
monsoon forests and spread
over the region receiving
rainfall between 200 cm and
70 cm.
2. Teak is the most dominant
species
of
this
forest.
Bamboos,
sal,
shisham,
sandalwood, khair, kusum,
arjun, mulberry are other
commercially
important
species.
To which type of forests in India does
above description refer to?
a.
b.
c.
d.

Tropical deciduous forests


Tropical rain forests
Montane forests
Tropical rain forests

9. Consider
the
following
statements about The Inter
Tropical Convergence Zone
(ITCZ)
1. It is a broad trough of high
pressure
in
equatorial
latitudes.
2. This is where the northeast
and the southeast trade winds
converge.
3. This convergence zone lies
more or less parallel to the
http://insightsonindia.com

equator but moves north or


south with the apparent
movement of the sun.
Which of the statements given above
is/are incorrect?
a.
b.
c.
d.

1 Only
2 Only
3 Only
None

10. Which of the following biosphere


reserve in India is not a part of
the World Network of Biosphere
Reserves,
based
on
the UNESCO Man
and
the
Biosphere (MAB) Programme
list?
a.
b.
c.
d.

Nokrek
Panna
Panchmarhi
Nicobar Islands

11. Soon after Independence, India


faced many new challenges. From
among the following statements,
which were the most important
challenges that were important
and urgent for India?
1. To ensure the development
and well-being of the entire
society and not only of some
sections
2. To shape a nation that was
united, yet accommodative of
the diversity in our society.
Page 5

Facebook Group: Indian Administrative Service (Raz Kr)

3. To establish democracy.
4. To develop industries and
bring economic growth
Select the correct answer using the codes
below
a.
b.
c.
d.

1,2 and 3 Only


1,2,3 and 4
2,3 and 4
2 and 3 Only

12. Consider
the
statements about
Indian polity

following
Whips in

1. The main function of the


Whips is to keep members of
their party within sound of
the Division Bell whenever
any important business is
under consideration in the
House.
2. During sessions, the Whips of
different parties send to their
supporters periodic notices,
also sometimes called 'Whips',
apprising
them
when
important
Divisions
are
expected, telling them the
hour when a vote will
probably take place,
and
requesting them to be in
attendance at that time
3. The Chief Whip of
the
Government Party in the Lok
Sabha and Rajya Sabha is the
Minister of Parliamentary
Affairs.
http://insightsonindia.com

Which of the statements given above


is/are correct?
a.
b.
c.
d.

1 and 3 Only
2 and 3 Only
1 and 2 Only
All

13. Consider
the
following
statements with reference to
Dravidian movement
1. Though some sections of this
movement had ambitions of
creating a Dravid nation, the
movement took to arms and
violence was inevitable
2. It used democratic means like
public debates and the
electoral platform to achieve
its ends.
3. The
movement
strongly
opposed
the
Brahmins
dominance
and
affirmed
regional pride against the
political, economic
and
cultural domination of the
North.
Which of the statements given above
is/are correct?
a.
b.
c.
d.

1 and 2 Only
2and 3 Only
3 Only
1 and 3 Only

Page 6

Facebook Group: Indian Administrative Service (Raz Kr)

14. Consider
the
following
statements about Article 370 of
the Indian Constitution
1. Article 370 gives greater
autonomy to Jammu and
Kashmir compared to other
States of India.
2. The State has its own
Constitution.
3. All provisions of the Indian
Constitution are applicable to
the State.
4. Laws passed by the Parliament
apply to J&K only if the State
agrees.

Which of the statements given above


is/are correct?
a.
b.
c.
d.

1 and 4 Only
1,2 and 4 Only
1,3 and 4 Only
All

15. With reference to reorganization


of North Eastern states, consider
the following statements
1. Meghalaya, Manipur and
Tripura became Indian states
in 1972
2. Arunachal
Pradesh
and
Mizoram became separate
States only in 1986
3. Nagaland State was created
after
the
creation
of
Meghalaya, Manipur, Tripura
and Mizoram
http://insightsonindia.com

4. At independence the entire


region except Manipur and
Tripura comprised the State
of Assam.
Which of the above statements is/are
correct?
a.
b.
c.
d.

1,3 and 4 Only


2,3 and 4 Only
1,2 and 4 Only
All

16. Consider
the
following
statements with reference to Goa
1. In January 1967, the Central
Government held a special
opinion poll in Goa asking
people to decide if they
wanted
to be part of
Maharashtra
or
remain
separate.
2. This was the only time in
independent India that a
referendum-like
procedure
was used to ascertain peoples
wishes on a subject.
3. In 1967, Goa became a State
of the Indian Union.
Which of the statements given above
is/are incorrect?
a.
b.
c.
d.

3 Only
1 and 2 Only
2 and 3 Only
None

Page 7

Facebook Group: Indian Administrative Service (Raz Kr)

17. Consider
the
following
statements with reference to
population growth in India
1. The period from 1901-1921 is
referred to as a period of
stagnant or stationary phase of
growth of Indias population,
since in this period growth
rate was very low, even
recording a negative growth
rate during 1911-1921.
2. The decades 1921-1951 are
referred to as the period of
steady population growth.
3. The decades 1951-1981 are
referred to as the period of
population explosion in India,
which was caused by a rapid
fall in the mortality rate but a
high
fertility
rate
of
population in the country.
Which of the above statements is/are
correct?
a.
b.
c.
d.

1 and 2 Only
1 and 3 Only
2 and 3 Only
All

18. Consider
statements

the

a.
b.
c.
d.

1 Only
2 Only
None
Both

19. Data from Indian Remote Sensing


satellites are used for various
applications of resources survey
and management under the
National
Natural
Resources
Management System (NNRMS).
Which of the following is not one
of its applications?
1. Snow-melt
run-off
estimates for planning water
use in down -stream
projects
2. Land use and land cover
mapping
3. Urban planning
4. Forest survey
5. Wetland mapping
6. Environmental
impact
analysis
7. Mineral Prospecting
Select the correct answer using the codes
below

following

1. Languages
spoken
in
Meghalaya, Nicobar Islands
belong to the family of
Austric languages
2. Languages spoken in Assam
and Jammu and Kashmir
belong to the family of IndoEuropean
http://insightsonindia.com

Which of the above statements I incorrect?

a.
b.
c.
d.

1,2,3,4 and 7
1,2,4,5,6 and 7
2,3,4 and 7
All

20. With reference to Mumbai port,


consider the following statements
1. It is a natural harbour and the
biggest port of the country.
2. It has countrys largest oil
terminal
Page 8

Facebook Group: Indian Administrative Service (Raz Kr)

3. The port is administered by


the Mumbai
Port
Trust (MbPT),
an
autonomous
corporation
belonging to a private trust of
investors
Which of the above statements
is/are incorrect?
a.
b.
c.
d.

1 ad 3 Only
2 and 3 Only
3 Only
None

21. With
reference
to
water
pollution, consider the following
statements
1. Total dissolved solids (TDS) is
the term used to describe the
organic salts and small
amounts of inorganic matter
present in solution in water.
2. The principal constituents are
usually calcium, magnesium,
sodium, and potassium cations
and
carbonate,
hydrogencarbonate, chloride,
sulfate, and nitrate anions.
3. Primary sources for TDS in
receiving
waters
are agricultural and residential
runoff,
leaching
of soil
contamination and point
source water
pollution discharge
from
industrial
or sewage
treatment plants.
4. If
TDS
is
less
than
300mg/litre then its graded as
excellent for drinking
Which of the above statements is/are
correct?
http://insightsonindia.com

a.
b.
c.
d.

2 and 3 Only
1,2 and 3 Only
2,3 and 4 Only
All

22. With reference to latest Human


Development
Index
data,
consider the following statements
1. India has been ranked 136
among 187 countries evaluated
for human development index
(HDI)
2. Indias HDI value went up
from 0.345 to 0.554 between
1980 and 2012, an increase of
61 per cent or an average
annual increase of 1.5 per cent.
3. Life expectancy at birth and
mean years of schooling have
increased
4. As for the Multidimensional
Poverty Index (MPI), India
does worsse than Bangladesh
and Pakistan
Which of the above statements is/are
correct?
a.
b.
c.
d.

1 and 2 Only
2,3 and 4 Only
3 and 4 Only
1,2 and 3 Only

23. With reference to gandhijis


views on development, consider
the following statements
1. He was quite apprehensive
about
the
on-going
development particularly the
way
industrialisation
has
institutionalised the loss of
Page 9

Facebook Group: Indian Administrative Service (Raz Kr)

morality, spirituality, selfreliance, non-violence and


mutual
cooperation
and
environment.
2. In his opinion, austerity for
individual, trusteeship of
social wealth and non-violence
are the key to attain higher
goals in the life of an
individual as well as that of a
nation.
3. His views were also re-echoed
in the Club of Rome Report
Limits to Growth (1972),
Schumachers book Small is
Beautiful (1974), Brundtland
Commissions Report Our
Common Future (1987) and
finally in the Agenda-21
Report of the Rio Conference
(1993).
Which of
incorrect?
a.
b.
c.
d.

the

above

statements

is

2 Only
3 Only
1 Only
None

24. With reference to land usage


pattern in the country, consider
the following statements
1. Share of area under forest,
area under non agricultural
uses and current fallow lands
has shown a decrease between
1960s and 2000s (as per
reported area)
2. Barren
and
wasteland,
culturable wasteland, area
under pastures and tree crops
and net area sown have shown
an increase in their areas
during the same time
http://insightsonindia.com

3. The increase in land under


pastures and grazing lands can
be
explained
by
the
encroachment of forest land
and water bodies in villages
Which of the statements given above
is/are correct?
a.
b.
c.
d.

1 and 2 Only
1,2 and 3
3 Only
None

25. Consider
statements
cultivation

the
following
regarding
coffee

1. Coffee is a tropical plantation


crop.
2. India mostly grows superior
quality coffee, arabica, which
is in great demand in
International market
3. It has become the primary
export and backbone for
African countries like Uganda,
Burundi,
Rwanda,
and
Ethiopia
Which of the statements given above
is/are correct?
a.
b.
c.
d.

1 and 2 Only
1 and 3 Only
2 and 3 Only
All

26. Consider
the
following
statements about satyagraha
1. The idea of satyagraha
emphasised the power of
truth and the need to search
for truth.
Page 10

Facebook Group: Indian Administrative Service (Raz Kr)

2. It suggested that if the cause


was true, if the struggle was
against injustice, then physical
force was not necessary to
fight the oppressor.
3. According
to
Gandhiji,
satyagraha is a passive
resistance force
4. Without seeking vengeance
or
being
aggressive,
a
satyagrahi could win the battle
through non-violence
5. This could be done by
appealing to the conscience of
the oppressor. People
including the oppressors had
to be persuaded to see the
truth, instead of being forced
to accept truth through the
use of violence

Which of the above statements is/are


correct?
a.
b.
c.
d.

1,2,3 and 5 Only


1,2,4 and 5 Only
2,3,4 and 5 Only
All

27. Consider
the
following
statements with reference to
Rowalatt Act of 1919
1. This
act
effectively
authorized the government to
imprison for up to two years,
without trial, any person
suspected of terrorism living
in the Raj and gave the
imperial authorities power to
deal
with
revolutionary
activities.
http://insightsonindia.com

2. The accused was denied the


right to know the accusers and
the evidence used in the trial
3. The act was repealed in 1922
4. The infamous Jallianwala Bagh
massacre of 1919 was a result
of reaction against this act
5. The Rowlatt act was one of
the causes of the civil
disobedience movement
Which of the above statements is/are
correct?
a.
b.
c.
d.

1,3,4 and 5 Only


2,3,4 and 5 Only
1,2,3 and 4 Only
All

28. Consider
statements
Revolution

the
following
about
Green

1. This strategy of agricultural


development
made
the
country
self-reliant
in
foodgrain production. But
green revolution was initially
confined to irrigated areas
only.
2. This led to regional disparities
in agricultural development in
the country till the seventies,
after which the technology
spread to the Eastern and
Central parts of the country
3. The Planning Commission
initiated
agro-climatic
planning in 1988 to induce
regionally
balanced
Page 11

Facebook Group: Indian Administrative Service (Raz Kr)

agricultural development in
the country
Which of the above statements is/are
correct?
a.
b.
c.
d.

1 and 2 Only
1 and 3 Only
2 and 3 Only
All

29. Consider
the
following
statements
about
peasant
movement during 1920 in India
1. In Awadh, peasants were led
by Baba Ramchandra a
sanyasi who had earlier been
to Fiji as an indentured
labourer. The movement here
was against talukdars and
landlords who demanded from
peasants exorbitantly high
rents and a variety of other
cesses.
2. Peasant movements were
completely peaceful during
1920-21 and were integrated
with
Non-Cooperation
movement nationwide
3. Peasants had to do begar and
work at landlords farms
without any payment
4. The Oudh Kisan Sabha was
set up headed by Jawaharlal
Nehru, Baba Ramchandra and
a few others.
Which of the statements given above
is/are correct?
http://insightsonindia.com

a.
b.
c.
d.

1,2 and 4 Only


1,3 and 4 Only
2,3 and 4 Only
All

30. With reference to Inland


Emigration Act of 1859, consider
the following statements
1. According to this
act,
plantation workers were not
permitted to leave the tea
gardens without permission,
and in fact they were rarely
given such permission
2. When plantation workers
successfully participated in
Non-Cooperation movement
and left the plantations
altogether
Which of the above statements is/are
correct?
a.
b.
c.
d.

1 Only
2 Only
Both
None

31. Consider
statements

the

following

1. In the harvested crop, the


grain seeds need to be
separated from the chaff. This
process is called winnowing.
This is carried out with the
help of a machine called
combine which is in fact a
combined
harvester
and
winnower.
Page 12

Facebook Group: Indian Administrative Service (Raz Kr)

2. Farmers with small holdings of


land do the separation of grain
and chaff by winnowing
Which of the above statements is/are
incorrect?
a.
b.
c.
d.

1 Only
2 Only
Both
None

32. Consider
the
following
statements with reference to
revenue systems that were in
practice during the British rule in
India

d. 2 Only
33. With reference to some common
diseases
caused
by
microorganisms
in
plants,
consider the following statements
1. Rust of wheat is caused by a
virus
2. Citrus canker is caused by
fungi
Which of the above is/are correct?
a.
b.
c.
d.

1 Only
None
2 Only
Both

1. Among the three revenue


systems operational during
British period i.e. Mahalwari,
Ryotwari and Zamindari, the
last one was most exploitative
for the peasants
2. Under Mahalwari system, for
revenue purposes the name
was applied to any compact
area containing one or more
villages, which were called
estates.
The
revenue
settlement was made with the
estatehence
the
term mahalwariand there
were
distinct
types
of assessment.

34. Consider
the
following
statements with reference to civil
disobedience movment launched
by Gandhiji

Which of the above statements is/are


incorrect?

Which of the above statements is/are


correct?

a. Both
b. None
c. 1 Only
http://insightsonindia.com

1. Mahatma Gandhis letter of


31st January 1930 was, in a
way, an ultimatum. If the
demands were not fulfilled by
11 March, the letter stated, the
Congress would launch a civil
disobedience campaign.
2. The
letter
had
eleven
demands representing peasants
to industrialists
3. The most important demand
was to reduce salt tax.

a. 1 and 3 Only
b. 2 and 3 Only
c. 1 and 2 Only
Page 13

Facebook Group: Indian Administrative Service (Raz Kr)

b. 1 Only
c. 2 and 3 Only
d. All

d. All

35. Consider
statements

the

following

1. Acrylic is a synthetic fiber


made
of
acrylonitrile monomer
2. Cellulose is a natural polymer
3. Acrylic fiber is sometimes
called as artificial silk
Which of the above statements is/are
correct?
a.
b.
c.
d.

1 Only
2 Only
2 and 3 Only
All

36. Consider the statements about


Haryali programme
1. Haryali is a watershed
development
project
sponsored by the Central
Government which aims at
enabling the rural population
to
conserve
water
for
drinking, irrigation, fisheries
and afforestation.
2. The Project is being executed
by Gram Panchayats with
peoples participation.
3. It was launched by the UPA
government in 2005
Which of the statements given above
is/are correct?
a. 1 and 2 Only
http://insightsonindia.com

37. With reference to electroscope,


consider the following statements
1. It is an early scientific
instrument that is used to
detect the presence and
magnitude of electrons in
space.
2. Electroscopes detect electric
charge by the motion of a test
object due to the Coulomb
electrostatic force.
3. Electroscopes were used by
the Austrian scientist Victor
Hess in
the
discovery
of cosmic rays.
Which of the above statements is/are
incorrect?
a.
b.
c.
d.

1 and 2 Only
1 Only
2 and 3 Only
All

38. Revolution is the inalienable


right of mankind. Freedom is the
imprescriptible birthright of all.
The labourer is the real sustainer
of society To the altar of this
revolution we have brought our
youth as incense, for no sacrifice
is too great for so magnificent a
cause. Who said these words?
a. Subhash Chandra Bose
b. Surya Sen
c. Bhagat Singh
Page 14

Facebook Group: Indian Administrative Service (Raz Kr)

d. Aurabindo Ghosh
39. Consider
statements

the

following

1. Alluri Sitarama Raju led a


tribal uprising called Rampa
rebellion
2. After the passing of the 1882
Madras Forest Act, its
restrictions on the free
movement of tribal peoples in
the forest prevented them
from engaging in their
traditional "Podu" agricultural
system. This led to the
rebellion.
Which of the above statements is/are
correct?
a.
b.
c.
d.

Both
None
1 Only
2 Only

40. Gurumahisani,
Sulaipet,
Badampahar
(Mayurbhaj),
Kiruburu (Kendujhar) and Bonai
mines are famous for
a.
b.
c.
d.

Bauxite ore
Coal
Iron ore
Manganese

41. Consider
the
following
statements with reference to
bioenergy

http://insightsonindia.com

1. Bioenergy is non-renewable
energy made available from
materials
derived
from
biological sources.
2. Biomass is any organic
material which has stored
sunlight in the form of
chemical energy.
3. As
a
fuel
it
may
include wood,
wood
waste, straw, manure, sugarca
ne,
and
many
other
byproducts from a variety of
agricultural processes.
Which of
incorrect?
a.
b.
c.
d.

the

above

statements

is

1 Only
2 Only
3 Only
None

42. Consider
statements

the

following

1. Dr B.R. Ambedkar, who


organised the dalits into the
Depressed Classes Association
in
1930,
clashed
with
Mahatma Gandhi at the
second
Round
Table
Conference by demanding
separate electorates for dalits.
2. Gandhiji
believed
that
separate electorates for dalits
would slow down getting
poorna swaraj from the British
3. Ambedkar
ultimately
accepted Gandhijis position
and the result was the Poona
Pact of September 1932.
Page 15

Facebook Group: Indian Administrative Service (Raz Kr)

4. It gave the Depressed Classes


(later to be known as the
Schedule Castes) reserved
seats in provincial and central
legislative councils, but they
were to be voted in by the
general electorate.

2.

Which of the above statements is/are


correct?
a.
b.
c.
d.

3.

All
1,2 and 4 Only
1,3 and 4 Only
2,3 and 4 Only

43. Communalism in its higher


aspect, then, is indispensable to
the formation of a harmonious
whole in a country like India. The
units of Indian society are not
territorial as in European
countries The principle of
European democracy cannot be
applied
to
India
without
recognising the fact of communal
groups. The Muslim demand for
the creation of a Muslim India
within India is, therefore,
perfectly justified Who said
these words?
a.
b.
c.
d.

Muhammad Ali Jinnah


Liaquat Ali Khan
Muhammad Iqbal
Syed Ahmad Khan

4.

5.

which promised return travel


to India after they had worked
five years on their employers
plantation in foreign countries.
Indentured labour was a
bonded
labourer
under
contract to work for an
employer for a specific
amount of time within Indian
territory
Most
Indian
indentured
workers came from the
present-day regions of eastern
Uttar Pradesh, Bihar, central
India and the dry districts of
Tamil Nadu.
The main destinations of
Indian indentured migrants
were the Caribbean islands
(mainly Trinidad, Guyana and
Surinam), Mauritius and Fiji
Nineteenth-century indenture
has been described as a new
system of slavery.

Which of the above statements is/are


correct?
a.
b.
c.
d.

1,2,3 and 5
2,3 and 5
1,3,4 and 5
All

45. With reference to intercropping,


consider the following statements
1.

44. Consider
statements

the

following

Intercropping
gives
additional
yield
income/unit area than sole
cropping.

1. In India, indentured labourers


were hired under contracts
http://insightsonindia.com

Page 16

Facebook Group: Indian Administrative Service (Raz Kr)

2.

3.

4.

5.

It acts as an insurance
against failure of crops in
abnormal year
Inter-crops maintain the
soil fertility as the nutrient
uptake is made from both
layers of soil.
It helps in reduction in soil
runoff and controls weeds.
Higher amount of fertilizer
or irrigation water cannot
be utilized properly as the
component crops vary in
their response of these
resources.

Which of the above statements is/are


correct?
a.
b.
c.
d.

1,2,3 and 5
All
1,2,4 and 5
1,2,3 and 4

46. With reference to CITES,


consider the following statements
1. Its aim is to ensure that
international
trade
in
specimens of wild animals and
plants does not threaten their
survival.
2. CITES was drafted as a result
of a resolution adopted in
1963 at a meeting of members
of IUCN
3. Participation is compulsory.
Which of the statements given above
is/are correct?
a. 1,2 and 3
http://insightsonindia.com

b. 2 and 3
c. 1 and 2
d. 1 and 3
47. Which one of the following
methods is best to avoid soil
erosion on the steep slopes?
a.
b.
c.
d.

Contour barriers
Contour ploughing
Rock dam
Terrace farming

48. With reference to earthing


consider the following statements
1. The primary purpose of
earthing is to reduce the risk
of serious electric shock from
current
leaking
into
uninsulated metal parts of an
appliance, power tool, or
other electrical devices
2. Earthing is the process of
creating an alternative path for
the flow of fault/excessive
currents safely into the ground
in the presence of minimal
resistance or impedance.
3. Safe Earthing Electrode is a
metallic device mounted at
the highest point of the
building to capture lightning
strikes and direct it to the
earth via a safe path thereby
preventing it from flowing
through
the
building's
electrcial circuit.
Which of the above statements is/are
correct?
Page 17

Facebook Group: Indian Administrative Service (Raz Kr)

a.
b.
c.
d.

1 and 2 Only
2 and 3 Only
All
1 and 3 only

49. Regarding trade during the British


period in India, consider the
following statements
1. During 1820s, opium was the
largest product in Indias
exports basket
2. By helping Britain balance its
deficits, India played a crucial
role in the late-nineteenthcentury world economy
3. Britain had a trade deficit
with India
Which of the above statements is/are
incorrect?
a.
b.
c.
d.

3 Only
1 and 2 Only
2 and 3 Only
2 Only

50. With reference to Warli revolt ,


consider the following statements
1. Godavari Parulekar was the
prominent leader of the revolt
2. Warlis were primitive tribe
living in Madhya Pradesh
Which of the above statements is/are
correct?
a. 1 Only
b. 2 Only
c. Both
http://insightsonindia.com

d. None
51. With reference to the decline of
Mughal Dynasty, consider the
following statements
1. In 1849, Governor-General
Canning announced that after
the death of Bahadur Shah
Zafar, the family of the king
would be shifted out of the
Red Fort and given another
place in Delhi to reside in.
2. In 1856, Governor-General
Dalhousie
decided
that
Bahadur Shah Zafar would be
the last Mughal king and after
his death none of his
descendants
would
be
recognized as kings they
would just be called princes.
Which of the above statements is/are
incorrect?
a.
b.
c.
d.

1 Only
2 Only
None
Both

52. Regarding a constitution of a


country, consider the following
statements
1. It specifies the basic allocation
of power in a society. It
decides who gets to decide
what the laws will be.
2. It
decides
how
the
government
will
be
constituted.
Page 18

Facebook Group: Indian Administrative Service (Raz Kr)

3. It sets some limits on what a


government can impose on its
citizens.
4. Some constitutions enable and
empower the government to
take positive measures to
overcome forms of inequality
or deprivation.
Which of the statements given above
is/are correct?
a.
b.
c.
d.

1 and 2 Only
2 and 3 Only
3 and 4 Only
All

53. A constitution expresses the


fundamental identity of a people.
Which means,
1. The people as a collective
entity come into being only
through the basic constitution
2. It is by agreeing to a basic set
of norms about how one
should be governed, and who
should be governed that one
forms a collective identity.
3. It is by providing a framework
for positive discrimination, an
identity is given to the people
4. The
constitution
sets
authoritative constraints upon
what one may or may not do.
It defines the fundamental
values that we may not
trespass. This is moral identity.
Which of the above statements are correct?
a. 1,2, and 4
http://insightsonindia.com

b. 2,3 and 1
c. 3 and 4 Only
d. All
54. Amrita Devi Bishnoi Wildlife
Protection Award is given to
a. Tribal people of Rajasthan
who protect wildlife in the
desert regions
b. Individuals or communities
from rural areas that have
shown extraordinary courage
and dedication in protecting
wildlife
c. People
who
practice
transhumance in the forest
and hilly regions who protect
traditional knowledge
d. None
55. With reference to Joint Forest
Management (JFM), consider the
following statements
1. the provisions of National
Forest Policy 1988, the
Government of India, outlined
and conveyed to State
Governments a framework for
creating massive peoples
movement
through
involvement
of
village
committees for the protection,
regeneration and development
of degraded forest lands
2. Joint Forest Management
(JFM) programme in the
present form can be traced to
the
Arabari
experiment
Page 19

Facebook Group: Indian Administrative Service (Raz Kr)

initiated by foresters in the


state of West Bengal.
3. In this scheme, villagers agree
to assist in the safeguarding of
forest
resources
through
protection from fire, grazing,
and illegal harvesting in
exchange for which they
receive non-timber
forest
products and a share of the
revenue from the sale
of timber products
Which of the above statements is/are
correct?
a.
b.
c.
d.

1 and 2 Only
1 and 3 Only
2 and 3 Only
All

56. With
reference
to
cultivation,
consider
following statements

Jhum
the

1. It is also called as shifting


cultivation and slash and burn
cultivation
2. Shifting
cultivation is
an agricultural system
in
which plots of land are
cultivated temporarily, then
abandoned and allowed to
revert to their natural
vegetation while the cultivator
moves on to another plot.
3. The role of slash-and-burn is
significant
in
the
current Holocene extinction.
Which of the above statements is/are
incorrect?
http://insightsonindia.com

a.
b.
c.
d.

3 Only
2 Only
None
Both

57. The Indian Constitution was


never subject to such a
referendum, but nevertheless
carried
enormous
public
authority. This is because
1. It had the consensus and
backing of leaders who were
themselves popular
2. It was drawn up by people
who enjoyed immense public
credibility
3. It was drawn by a completely
elected
body
of
representatives
Which of the above statements is/are
correct?
a.
b.
c.
d.

1 and 3 Only
2 and 3 Only
1 and 2 Only
All

58. Indian our Constitution is a living


document. What do you mean by
that? Consider the following
statements
1. Like a living being, the
Constitution responds to
experience.
2. Even after so many changes in
the society, the Constitution
continues to work effectively
because of this ability to be
dynamic, to be open to
Page 20

Facebook Group: Indian Administrative Service (Raz Kr)

interpretations and the ability


to respond to the changing
situation
3. Almost like a living being, this
document keeps responding to
the
situations
and
circumstances arising from
time to time.
4. Like a living being, our
constitution is also prone to
destruction and its not
permanent
Which of the above aptly describe the
statement in the question in the context of
Indian constitution?
a.
b.
c.
d.

1,2 and 4
2 and 4
1,2, and 3
All

59. With reference to the effect of


the British rule on old Indian
cities, consider the following
statements
1. Old trading centres and ports
could not survive when the
flow of trade moved to new
centres
2. The British Rule caused rapid
urbanization of old cities
Which of the above statements is/are
correct?
a.
b.
c.
d.

1 Only
Both
2 Only
None

http://insightsonindia.com

60. With reference to collective


farming, consider the following
statements
1. A group of farmers form a
collective society by pooling
in their resources voluntarily
for more efficient and
profitable farming.
2. Collective farming or the
model of Kolkhoz was
introduced in erstwhile Soviet
Union to improve upon the
inefficiency of the previous
methods of agriculture and to
boost agricultural production
for self-sufficiency
3. The farmers have to pay taxes
on the farm produces, hired
machinery etc
Which of the above statements is/are
correct?
a.
b.
c.
d.

1 Only
2 and 3 Only
1 and 3 Only
All

61. With reference to waterlogging,


consider the following statements
1. Irrigation without proper
drainage of water leads to
waterlogging in the soil.
Besides affecting the crops,
waterlogging reduces the
salinity of the surface of the
soil
2. Waterlogging is one of the
problems that have come in
Page 21

Facebook Group: Indian Administrative Service (Raz Kr)

the wake of the Green


Revolution
3. Soil may be regarded as
waterlogged when the water
table of
the groundwater is
too high to conveniently
permit an anticipated activity,
like agriculture.
Which of the above statements is/are
correct?
a.
b.
c.
d.

1 and 3 Only
1 and 2 Only
2 and 3 Only
All

62. In which one of the following


regions is extensive commercial
grain cultivation not practised?
a. American Canadian prairies
b. Pampas of Argentina
c. European Steppes
d. Amazon Basin
63. Consider the statements with
reference to viticulture
1. It is a speciality of the
Mediterranean region
2. When the grapes are used
forwinemaking, it is also
known as viniculture
3. In Indian, viticulture is
prominent in far eastern
region

c. 1 and 3 Only
d. All

64. Dobson unit is a measurement of


a. The thickness of the ozone in
a column of air from the
ground to the top of the
atmosphere
b. The concentration of ozone
molecules in a litre of air
c. The intensity of breakdown of
ozone into oxygen and free
radicals
d. None of the above
65. Which of the following gases
contribute more towards global
warming?
a.
b.
c.
d.

N2O
CFCs
Methane
CO

66. Truck farming refers to


a. The regions where farmers
contribute
only
towards
transportation of agricultural
goods
b. The regions where farmers
specialize in vegetables only
c. The regions where farmers
specialize
in
agricultural
marketing
d. None
of
the
above

Which of the above statements is/are


correct?
a. 1 and 2 Only
b. 2 and 3 Only
http://insightsonindia.com

Page 22

Facebook Group: Indian Administrative Service (Raz Kr)

67. Consider
the
following
statements with reference to
eutrophication

supremacy of the Parliament


has to operate within this
framework.

1. It is the natural aging of a lake


by biological enrichment of its
water.
2. As the lakes fertility increases
with more incoming nutrients,
plant and animal life burgeons,
and organic remains begin to
be deposited on the lake
bottom
3. Eutrophication
increases
oxygen and hence contributes
to
overproduction
of
phytoplankton

3. The Judiciary, in its famous


Golaknath ruling found a way
out
of
the
existing
complications by turning to
the spirit of the Constitution
rather than its letter in which
it upheld that the balance of
powers
between
various
organs of the state was a basic
structure of the constitution.

Which of the above statements is/are


correct?
a.
b.
c.
d.

1 and 3 Only
1 and 2 Only
2 and 3 Only
All

68. With reference to the supremacy


of the Parliament, consider the
following statements
1. In a parliamentary democracy,
the Parliament represents the
people and therefore, it is
expected to have an upper
hand over both Executive and
Judiciary
2. The text of the Constitution
and it has given powers to
other
organs
of
the
government. Therefore, the
http://insightsonindia.com

Which of the above statements is/are


correct?
a.
b.
c.
d.

1 and 2 Only
2 and 3 Only
1 and 3 Only
3 Only

69. With reference to Sikhs, consider


the following statements
1. The Khalsa was inaugurated
on March 30, 1699, by Guru
Gobind Singh, the tenth Sikh
Guru
2. The Khalsa is the collective
body
of
all initiated Sikhs represented
by the five beloved-ones and
can be called the Guru Panth
3. TheKhalsa rose in revolt
against the Mughal authority
under
Banda
Bahadurs
leadership, declared their
sovereign rule by striking
coins in the name of Guru
Nanak and Guru Gobind
Page 23

Facebook Group: Indian Administrative Service (Raz Kr)

Singh, and established their


own administration between
the Sutlej and the Jamuna.
Which of the above statements is/are
correct?
a.
b.
c.
d.

1,2 and 3
3 Only
2 and 3 Only
None

70. With
reference
to
biomagnifications, consider the
following statements
1. Biomagnification refers to
increase in concentration of
the toxicant at successive
trophic levels.
2. This is caused by low or nonexistent rate of internal
degradation or excretion of
the substance often due to
water-insolubility
3. Species of fish that arelonglived and low on the food
chain,
such
as marlin, tuna, shark contain
more mercury.
4. High concentrations of DDT
disturb calcium metabolism in
birds, which causes thinning
of
eggshell
and
their
premature
breaking,
eventually causing decline in
bird populations.
Which of the above statements is/are
incorrect?
a. 2 Only
b. 3 Only
c. 4 Only
http://insightsonindia.com

d. None
71. With reference to CFL bulbs,
consider the following statements
1. Unlike other bulbs, CFL are
consumer friendly and dont
pose health hazard
2. The luminous efficacy of a
typical CFL is 5070 lumens
per watt (lm/W) and that of a
typical incandescent lamp
is 1017 lm/W
3. Theluminous
efficacy of
lamps is the number of
lumens produced for each
watt of electrical power used.
4. CFLs emit light from a mix
of phosphors inside the bulb,
each emitting one band of
color.
Which of the above statements is/are
correct?
a.
b.
c.
d.

1 and 4 Only
2,3 and 4 Only
3 and 4 Only
All

72. With reference to Harappan sites


and agriculture during its period,
consider the following statements
1. Most Harappan sites are
located in semi-arid lands,
where irrigation was probably
not required for agriculture as
they were located on the bank
of Indus river

Page 24

Facebook Group: Indian Administrative Service (Raz Kr)

2. It is likely that water drawn


from wells was used for
irrigation.
Besides,
water
reservoirs found in Dholavira
(Gujarat) may have been used
to store water for agriculture.
3. Traces of canals have been
found at the Harappan site of
Shortughai in Afghanistan, but
not in Punjab or Sind.
4. Archaeologists
have
also
found evidence of a ploughed
field
at
Kalibangan
(Rajasthan), associated with
Early Harappan levels
Which of the above statements is/are
correct?
a.
b.
c.
d.

1,2 and 4 Only


1,2 and 3 Only
2,3 and 4 Only
All

73. With reference to Bharat stage


emission standards , consider the
following statements
1. These are emission standards
instituted by the Government
of India to regulate the output
of air pollutants from internal
combustion
engine
equipment, including motor
vehicles.
2. The stoppage of production of
Maruti-800 is because of
regulation of emissions
3. At present India has adopted
Bharat stage V emission
norms
http://insightsonindia.com

4. Under Bharat stage III, two


wheelers were allowed to
emit up to 1 g/km of carbon
monoxide
Which of the statements given above
is/are correct?
a.
b.
c.
d.

1,2 and 3
1,3 and 4
1,2 and 4
All

74. Consider
the
following
statements about Indian systems
of medicine
1. It includes only five systems:
Ayurveda,
Yoga,
Unani,
Siddha and Naturopathy
2. Unani medicine first arrived in
India around 12th or 13th
century with establishment
of Delhi Sultanate(1206-1527)
and Islamic rule over North
India and
subsequently
flourished
under Mughal
Empire
3. Siddha
focused
to
"Ashtamahasiddhi," the eight
supernatural power. Those
who attained or achieved the
above said powers are known
as Siddhars.
Which of the above statements is/are
correct?
a. All
b. 2 and 3 Only
c. 1 and 2 Only
Page 25

Facebook Group: Indian Administrative Service (Raz Kr)

d. 3 Only
75. About whom did Ambedkar said
that his ability to put the most
intricate proposals in the simplest
and clearest legal form can rarely
be equalled during drafting of
Indian constitution?
a. B.N.Rau
b. S.N.Mukherjee
c. Alladi Krishnswamy Aiyar
d. K.M.Munshi
76. Which of the following are vital
functions of the environment?
1. it supplies resources
2. it assimilates waste
3. it sustains life by providing
genetic and bio diversity
4. it provides aesthetic services
like scenery
5. It serves human needs
Which of the statements given above
is/are correct?
a.
b.
c.
d.

1,2,3,4 and 5
2,3, and 4
1,2,3 and 4
1,3, and 4

77. With reference to judicial


activism, consider the following
statements
1. It has democratised the
judicial system by giving not
just to individuals but also
groups access to the courts
2. It has forced executive
accountability.
http://insightsonindia.com

3. Judicial activism may be


creating strains on this
democratic principle.
4. It has overburdened the
courts.
5. It has also made an attempt to
make the electoral system
much more free and fair
Which of
incorrect?
a.
b.
c.
d.

the

above

statements

is

4 Only
5 Only
None
2 Only

78. Consider
the
following
statements with regard to sharia
1. The sharia is the law
governing
the
Muslim
community
2. It is based on the Quran and
the hadis, traditions of the
Prophet including a record of
his remembered words and
deeds.
3. With the expansion of Islamic
rule outside Arabia, in areas
where customs and traditions
were
different,
qiyas
(reasoning by analogy) and
ijma (consensus of the
community) were recognised
as two other sources of
legislation. Thus, the sharia
evolved from the Quran,
hadis, qiyas and ijma
4. Every Mughal leader imposed
Sharia in the Indian sub
continent
Page 26

Facebook Group: Indian Administrative Service (Raz Kr)

Which of the above statements is/are


correct?
a.
b.
c.
d.

1,2 and 3 Only


2 and 3 Only
1 and 3 Only
All

a. They remove pollutants from


river water
b. They remove particulate
matter from thermal power
exhaust gases
c. They are used in air
conditioners
d. None of the above

79. With reference to Lingayats,


consider the following statements

81. Ex Situ conservation of wildlife


refers to

1. The Lingayats also encouraged


certain practices disapproved
in the Dharmashastras, such as
post-puberty marriage and the
remarriage of widows.
2. They believed in the theory of
rebirth.
3. Lingayats believe that on
death the devotee will be
united with Shiva and will not
return to this world.
4. Therefore they do not practise
funerary
rites
such
as
cremation, prescribed in the
Dharmashastras. Instead, they
ceremonially bury their dead.

a. Protection of threatened wild


animals in biosphere reserves
b. Conservation of endangered
species in zoological gardens,
botanical parks
c. Protection of animals through
domestication
d. None of the above

Which of the above statements is/are


correct?
a.
b.
c.
d.

1,2 and 3 Only


1,3, and 4 Only
A and 4 Only
All

80. What is the main function of


electrostatic
precipitator
electrostatic precipitator?
http://insightsonindia.com

82. With reference to groundnut,


consider the following statements
1. Groundnut is a rabi crop and
accounts for about half of the
major oilseeds produced in the
country
2. China, India, Nigeria, USA
and Myanmar are the major
groundnut growing countries.
3. Gujarat is the largest producer
of groundnut along with
Andhra Pradesh followed by
Tamil Nadu, Karnataka, and
Maharashtra
Which of the above statements is/are
correct?
a. 2 and 3 Only
b. 1,2 and 3
Page 27

Facebook Group: Indian Administrative Service (Raz Kr)

c. 1 and 2 Only
d. 1 and 3 Only

83. Which of the following are


narrowly utilitarian arguments for
conserving biodiversity?
1. Biodiversity plays a major role
in many ecosystem services
that nature provides
2. Humans derive countless
direct economic benefits from
nature -food (cereals, pulses,
fruits),
firewood,
fibre,
construction
material,
industrial products (tannins,
lubricants,
dyes,
resins,
perfumes ) and products of
medicinal importance.
3. With increasing resources put
into bioprospecting nations
endowed
with
rich
biodiversity can expect to
reap enormous benefits.
Select the correct answer using the codes
below
a.
b.
c.
d.

the

following

1. Lignite is a low grade brown


coal, which is soft with high
moisture content

http://insightsonindia.com

Which of the above statements is/are


correct?
a.
b.
c.
d.

2 and 3 Only
2 Only
1 Only
1 and 2 Only

85. The second trophic level in a lake


isa.
b.
c.
d.

Phytoplankton
Zooplankton
Benthos
Fishes

86. Which
of
the
following
statements are incorrect?

1 and 2
2 and 3
1 and 3
All

84. Consider
statements

2. Coal that has been buried


deep and subjected to
increased temperatures is
bituminous coal. It is the most
popular coal in commercial
use.
3. Damodar valley (West BengalJharkhand) Jharia, Raniganj,
Bokaro
are
important
coalfields where anthracite is
mined abundantly

1. Red
soil
develops
on
crystalline igneous rocks in
areas of low rainfall in the
eastern and southern parts of
the Deccan plateau
2. Red soils develop a reddish
colour due to diffusion of iron
in
crystalline
and
metamorphic rocks
Page 28

Facebook Group: Indian Administrative Service (Raz Kr)

Select the correct answer using the codes


below
a.
b.
c.
d.

1
2
Both
None

87. Watermelon,
muskmelon,
cucumber are mainly grown in
which season?
a.
b.
c.
d.

Rabi
Kharif
Zaid
None

Which of the above statements is/are


correct?

88. Intervention by the monetary


authority of a country in the
money
market to keep the
money supply stable against
exogenous or sometimes external
shocks such as an increase in
foreign exchange inflow is known
as
a.
b.
c.
d.

Revaluation
Sterilization
Open Market operation
None of the above

89. Consider
statements
Standard

the
following
regarding
Gold

1. From around 1870 to the


outbreak of the Second
World War in 1946, the
prevailing system was the gold
standard which was the

http://insightsonindia.com

epitome of the fixed exchange


rate system.
2. Each participant country
committed to guarantee the
free convertibility of its
currency into gold at a fixed
price.
3. Exchange
rates
were
determined by its worth in
terms of gold (where the
currency was made of gold, its
actual gold content).

a.
b.
c.
d.

1 and 2 Only
2 and 3 Only
1 and 3 Only
All

90. With reference to weathering


process, consider the following
statements
1. In
weathering,
oxidation
means a combination of a
mineral with oxygen to form
oxides or hydroxides.
2. Oxidation occurs where there
is ready access to the
atmosphere and oxygenated
waters
3. When oxidised minerals are
placed in an environment
where oxygen is absent,
reduction takes place.
4. Red colour of iron upon
reduction turns to brown or
yellow.

Page 29

Facebook Group: Indian Administrative Service (Raz Kr)

5. Red colour of iron upon


oxidation turns to greenish or
bluish grey.
Which of the above statements is/are
correct?
a.
b.
c.
d.

1,2 and 3 Only


4 and 5 Only
3,4 and 5 Only
All

91. The novel tells the moving story


of Hori and his wife Dhania, a
peasant
couple.
Landlords,
moneylenders,
priests
and
colonial bureaucrats all those
who hold power in society
form a network of oppression,
rob their land and make them
into landless labourers. Yet Hori
and Dhania retain their dignity to
the end. To which novel does
this description apply to?
a. Rangbhoomi (The Arena) by
Premchand
b. Godan (The Gift of Cow) by
Premchand
c. Chomana Dudi by Shivaram
Karanth
d. Anguriya Binimoy by Bhudeb
Mukhopadhyay
92. Cirque is a
a. A landform in glaciated
mountains
b. Landform in desert landscape
shaped by wind action
c. A landform in limestone cave
d. None
http://insightsonindia.com

93. The ozone layer is located at


a.
b.
c.
d.

Troposphere
Mesosphere
Ionosphere
Stratosphere

94. Consider
statements
movement

the
about

following
Khilafat

1. The Khilafat Movement,


(1919-1920) was a movement
of Indian Muslims, led by
Muhammad Ali and Shaukat
Ali
2. The demand was that the
Turkish Sultan or Khalifa
must retain control over the
Muslim sacred places in the
erstwhile Ottoman empire;
the jazirat-ul-Arab (Arabia,
Syria, Iraq, Palestine) must
remain
under
Muslim
sovereignty
3. Mahatma Gandhi sought to
conjoin it to the Noncooperation Movement.
Which of
incorrect?
a.
b.
c.
d.

the

above

statements

is

2
1
None
3

95. The temperature of air at any


place is influenced,
1. The latitude of the place
2. The longitude of the place
Page 30

Facebook Group: Indian Administrative Service (Raz Kr)

3. The altitude of the place


4. Distance from the sea, the airmass circulation;
5. The presence of warm and
cold ocean currents;
Select the correct answer using the codes
below
a.
b.
c.
d.

1,2,3,4 and 5
1,3,4 and 5
2,3,4 and 5
3,4 and 5

96. Consider
the
statements about
movement

following
Quit India

1. Quit India was genuinely a


mass movement, bringing into
its
ambit
hundreds
of
thousands of ordinary Indians.
2. It was in these years that the
League began to make a mark
in the Punjab and Sind,
provinces where it had
previously had scarcely any
presence.
3. It was launched after the
failure of the Cabinet mission
plan
Which of the above statements is correct?
a.
b.
c.
d.

1 and 2
1 and 3
2 and 3
All

http://insightsonindia.com

97. Consider
statements

the

following

1. The
continental
slope
connects the continental shelf
and the ocean basins
2. It begins where the bottom of
the continental shelf sharply
drops off into a steep slope
3. In this region massive
sedimentary deposits received
over a long time by the
continental shelves, become
the source of fossil fuels.
4. Canyons and trenches are
observed in this region.
Which of the above statements is/are
correct?
a.
b.
c.
d.

1,2 and 4
1,2,3 and 4
1,2 and 3
1,3 and 4

98. Guyots refer to


a. These are low islands found in
the tropical oceans consisting
of coral reefs surrounding a
central depression
b. These are deep valleys, some
comparable to the Grand
Canyon of the Colorado river.
c. It is a flat topped seamount
d. None of the above

Page 31

Facebook Group: Indian Administrative Service (Raz Kr)

99. Which
of
the
following
statements about ocean salinity
is/are correct?
1. The salinity of water in the
surface layer of oceans depend
mainly on evaporation and
precipitation.
2. Surface salinity is greatly
influenced in coastal regions
by the fresh water flow from
rivers, and in polar regions by
the processes of freezing and
thawing of ice.
3. Wind, also influences salinity
of an area by transferring
water to other areas
4. The ocean currents have
negligible role in affecting
ocean salinity

http://insightsonindia.com

Select the correct answer using the codes


below
a.
b.
c.
d.

1,2 and 3
2,3 and 4
3 and 4
1 and 3

100. Gel electrophoresis


technique used for

is

a. Cutting DNA molecules to


select only wanted fragments
b. Separating DNA fragments
c. Joining fragmented DNA
molecules
d. None of the above

Page 32

Facebook Group: Indian Administrative Service (Raz Kr)

INSIGHTS MOCK TEST 7 SOLUTIONS

1. Solution: d)
The buffer stocks are required to (i) feed TPDS and other welfare schemes, (ii)
ensure food security during the periods when production is short of normal
demand during bad agricultural years and (iii) stabilize prices during period of
production shortfall through open market sales.
The total annual stock of food grains in the Central Pool is distributed over
different quarters of the year depending upon offtake and procurement patterns.
The seasonality of production and procurement is thus a decisive factor in
determining the minimum norm of food grains stocks required in a particular
quarter of the year. For working out buffer stocking norms and making
recommendations for policy decisions, the Government has been setting up from
time to time Technical Groups under the Chairmanship of Union Food Secretary.

2. Solution: d)
All are correct.
In case of India we have unemployment in rural and urban areas. However, the
nature of the unemployed differs in rural and urban areas. In case of rural areas,
there is seasonal and disguised unemployment. Urban areas have mostly educated
unemployment.
Seasonal unemployment happens when people are not able to find jobs during
some months of the year. People dependant upon agriculture usually face such kind
of problem. There are certain busy seasons when sowing, harvesting, weeding,
threshing is done. Certain months do not provide much work to the people
dependant on agriculture.
In case of disguised unemployment people appear to be employed. They have
agricultural plot where they find work. This usually happens among family
members engaged in agricultural activity.
The work requires the service of five people but engages eight people. Three
people are extra. These three people also work in the same plot as five people. The
contribution made by the three extra people does not add to the contribution made
http://insightsonindia.com

INSIGHTS

Page 1

Facebook Group: Indian Administrative Service (Raz Kr)

INSIGHTS MOCK TEST 7 SOLUTIONS

by the five people. If three people are removed the productivity of the field will
not decline. The field requires the service of five people and the three extra people
are disguisedly employed.
In case of urban areas educated unemployment has become a common
phenomenon. Many youth with matriculation, graduation and post graduation
degrees are not able to find job. A study showed that unemployment of graduate
and post-graduate has increased faster than among matriculates. A paradoxical
manpower situation is witnessed as surplus of manpower in certain categories
coexist with shortage of manpower in others.
There is unemployment among technically qualified person on one hand, while
there is a dearth of technical skills required for economic growth. Unemployment
leads to wastage of manpower resource. People who are an asset for the economy
turn into a liability.
There is a feeling of hopelessness and despair among the youth. People do not have
enough money to support their family. Inability of educated people who are willing
to work to find gainful employment implies a great social waste. Unemployment
tends to increase economic overload. The dependence of the unemployed on the
working population increases. The quality of life of an individual as well as of
society is adversely affected. When a family has to live on a bare subsistence level
there is a general decline in its health status and rising withdrawal from the school
system.
Hence, unemployment has detrimental impact on the overall growth of an
economy. Increase in unemployment is an indicator of a depressed economy. It also
wastes the resource, which could have been gainfully employed. If people cannot
be used as a resource they naturally appear as a liability to the economy.
http://timesofindia.indiatimes.com/business/india-business/Unemploymentlevels-rising-in-India-experts-say/articleshow/29403619.cms

3. Solution: b)
The National Food for Work Programme(External website that opens in a new
window) was launched in November, 2004 in 150 most backward districts of the
http://insightsonindia.com

INSIGHTS

Page 2

Facebook Group: Indian Administrative Service (Raz Kr)

INSIGHTS MOCK TEST 7 SOLUTIONS

country, identified by the Planning Commission in consultation with the Ministry


of Rural Development and the State governments.
The objective of the programme was to provide additional resources apart from the
resources available under the Sampoorna Grameen Rozgar Yojana (SGRY)(File
referring to external site opens in a new window) to 150 most backward districts
of the country so that generation of supplementary wage employment and
providing of food-security through creation of need based economic, social and
community assets in these districts are further intensified. The scheme was 100 per
cent centrally sponsored. The programme has since been subsumed in National
Rural Employment Guarantee Act (External website that opens in a new
window) which has come in force in 200 identified districts of the country
including 150 NFFWP districts. The Act provides 100 days of work guarantee to
every rural household whose members volunteer to do unskilled manual work.
4. Solution: c)
Social exclusion
According to this concept, poverty must be seen in terms of the poor having to live
only in a poor surrounding with other poor people, excluded from enjoying social
equality of better-off people in better surroundings. Social exclusion can be both a
cause as well as a consequence of poverty in the usual sense. Broadly, it is a process
through which individuals or groups are excluded from facilities, benefits and
opportunities that others (their betters) enjoy. A typical example is the working
of the caste system in India in which people belonging to certain castes are
excluded from equal opportunities. Social exclusion thus may lead to, but can cause
more damage than, having a very low income.
Vulnerability
Vulnerability to poverty is a measure, which describes the greater probability of
certain communities (say, members of a backward caste) or individuals (such as a
widow or a physically handicapped person) of becoming, or remaining, poor in the
coming years. Vulnerability is determined by the options available to different
communities for finding an alternative living in terms of assets, education, health
and job opportunities. Further, it is analysed on the basis of the greater risks these
groups face at the time of natural disasters (earthquakes, tsunami), terrorism etc.
Additional analysis is made of their social and economic ability to handle these
risks. In fact, vulnerability describes the greater probability of being more adversely
http://insightsonindia.com

INSIGHTS

Page 3

Facebook Group: Indian Administrative Service (Raz Kr)

INSIGHTS MOCK TEST 7 SOLUTIONS

affected than other people when bad time comes for everybody, whether a flood or
an earthquake or simply a fall in the availability of jobs!

5. Solution: d)
http://dfpd.nic.in/?q=node/101

6. Solution: d)

Jet stream: These are a narrow belt of high altitude (above 12,000 m) westerly
winds in the troposphere. Their speed varies from about 110 km/h in summer to
about 184 km/h in winter. A number of separate jet streams have been identified.
The most constant are the mid-latitude and the sub tropical jet stream.
The pressure and wind conditions over India are unique. During winter, there is a
high-pressure area north of the Himalayas. Cold dry winds blow from this region to
the low-pressure areas over the oceans to the south. In summer, a low-pressure area
develops over interior Asia as well as over northwestern India. This causes a
complete reversal of the direction of winds during summer. Air moves from the
high-pressure area over the southern Indian Ocean, in a south-easterly direction,
crosses the equator, and turns right towards the low-pressure areas over the Indian
subcontinent. These are known as the Southwest Monsoon winds. These winds
blow over the warm oceans, gather moisture and bring widespread rainfall over the
mainland of India.
The upper air circulation in this region is dominated by a westerly flow. An
important component of this flow is the jet stream. These jet streams are located
approximately over 27-30 north latitude, therefore, they are known as subtropical
westerly jet streams.
Over India, these jet streams blow south of the Himalayas, all through the year
except in summer. The western cyclonic disturbances experienced in the north and
north-western parts of the country are brought in by this westerly flow. In summer,
the subtropical westerly jet stream moves north of the Himalayas with the
http://insightsonindia.com

INSIGHTS

Page 4

Facebook Group: Indian Administrative Service (Raz Kr)

INSIGHTS MOCK TEST 7 SOLUTIONS

apparent movement of the sun. An easterly jet stream, called the tropical easterly
jet stream blows over peninsular India, approximately over 14N during the
summer months.

7. Solution: d)
Assam and most of the Peninsular states have moderate population densities. Hilly,
dissected and rocky nature of the terrain, moderate to low rainfall, shallow and less
fertile soils have influenced population densities in these areas.
The Northern Plains and Kerala in the south have high to very high population
densities because of the flat plains with fertile soils and abundant rainfall. Identify
the three states of the Northern Plains with high population densities.
http://censusindia.gov.in/2011-provresults/data_files/india/Final_PPT_2011chapter7.pdf

8. Solution: a)
Tropical Deciduous Forests
These are the most widespread forests of India. They are also called the monsoon
forests and spread over the region receiving rainfall between 200 cm and 70 cm.
Trees of this forest-type shed their leaves for about six to eight weeks in dry
summer.
On the basis of the availability of water, these forests are further divided into moist
and dry deciduous. The former is found in areas receiving rainfall between 200 and
100 cm. These forests exist, therefore, mostly in the eastern part of the country
northeastern states, along the foothills of the Himalayas, Jharkhand, West Orissa
and Chhattisgarh, and on the eastern slopes of the Western Ghats. Teak is the most
dominant species of this forest. Bamboos, sal, shisham, sandalwood, khair, kusum,
arjun, mulberry are other commercially important species.
The dry deciduous forests are found in areas having rainfall between 100 cm and 70
cm. These forests are found in the rainier parts of the peninsular plateau and the
plains of Bihar and Uttar Pradesh. There are open stretches in which Teak, Sal,
http://insightsonindia.com

INSIGHTS

Page 5

Facebook Group: Indian Administrative Service (Raz Kr)

INSIGHTS MOCK TEST 7 SOLUTIONS

Peepal, Neem grow. A large part of this region has been cleared for cultivation and
some parts are used for grazing.
In these forests, the common animals found are lion, tiger, pig, deer and elephant. A
huge variety of birds, lizards, snakes, and tortoises are also found here.

9. Solution: a)
It is a broad trough of LOW pressure in equatorial latitudes. This is where the
northeast and the southeast trade winds converge. This convergence zone lies more
or less parallel to the equator but moves north or south with the apparent
movement of the sun.
10. Solution: b)
http://en.wikipedia.org/wiki/Biosphere_reserves_of_India

11. Solution: a)
Broadly, independent India faced three kinds of challenges. The first and the
immediate challenge was to shape a nation that was united, yet accommodative of
the diversity in our society. India was a land of continental size and diversity. Its
people spoke different languages and followed different cultures and religions. At
that time it was widely believed that a country full of such kinds of diversity could
not remain together for long. The partition of the country appeared to prove
everyones worst fears. There were serious questions about the future of India:
Would India survive as a unified country? Would it do so by emphasising national
unity at the cost of every other objective? Would it mean rejecting all regional and
sub-national identities? And there was an urgent question: How was integration of
the territory of India to be achieved?
The second challenge was to establish democracy. You know that the Constitution
granted fundamental rights and extended the right to vote to every citizen. India
adopted representative democracy based on the parliamentary form of
http://insightsonindia.com

INSIGHTS

Page 6

Facebook Group: Indian Administrative Service (Raz Kr)

INSIGHTS MOCK TEST 7 SOLUTIONS

government. These features ensure that the political competition would take place
in a democratic framework. A democratic constitution is necessary but not
sufficient for establishing a democracy. The challenge was to develop democratic
practices in accordance with the Constitution.
The third challenge was to ensure the development and well-being of the entire
society and not only of some sections. Here again the Constitution clearly laid down
the principle of equality and special protection to socially disadvantaged groups
and religious and cultural communities. The Constitution also set out in the
Directive Principles of State Policy the welfare goals that democratic politics must
achieve. The real challenge now was to evolve effective policies for economic
development and eradication of poverty.

12. Solution: c)
Whips
In the parliamentary form of Government, a party has its own internal organisation
inside Parliament and is served by a number of officials known as the Whips,
chosen from members of the party itself. The main function of the Whips is to
keep members of their party within sound of the Division Bell whenever any
important business is under consideration in the House.
During sessions, the Whips of different parties send to their supporters periodic
notices, also sometimes called 'Whips', apprising them when important Divisions
are expected, telling them the hour when a vote will probably take place, and
requesting them to be in attendance at that time. Realizing the vital role played by
the Chief Whips and leaders of parties and groups in Parliament as important
party functionaries, an Act namely the Leaders and Chief Whips of Recognized
Parties and Groups in Parliament (Facilities) Act {Act 5 of 1999) was passed during
the Twelfth Lok Sabha. The Act inter alia provides additional Secretarial
assistance and telephone facilities to the Leaders and Chief Whips.
Government Chief Whip: The Chief Whip of the Government Party in the Lok
Sabha is the Minister of Parliamentary Affairs. In the Rajya Sabha, the Minister of
State for Parliamentary Affairs holds this position. The Chief Whip is directly
responsible to the Leader of the House. It is part of his duties to advise the
http://insightsonindia.com

INSIGHTS

Page 7

Facebook Group: Indian Administrative Service (Raz Kr)

INSIGHTS MOCK TEST 7 SOLUTIONS

Government on parliamentary business and to maintain a close liaison with the


Ministers in regard to parliamentary business affecting their Departments. The
Chief Whip is assisted by one or two Ministers of State and at times by Deputy
Ministers also.
For extra information: http://archive.indianexpress.com/news/the-whiphand/589000/0

13. Solution: b)
Dravidian movement
Vadakku Vaazhkirathu; Therkku Thaeikirathu [The north thrives even as the
south decays]. This popular slogan sums up the dominant sentiments of one of
Indias most effective regional movements, the Dravidian movement, at one point
of time. This was one of the first regional movements in Indian politics. Though
some sections of this movement had ambitions of creating a Dravid nation, the
movement did not take to arms. It used democratic means like public debates and
the electoral platform to achieve its ends. This strategy paid off as the movement
acquired political power in the State and also became influential at the national
level.
The Dravidian movement led to the formation of Dravida Kazhagam [DK] under
the leadership of Tamil social reformer E.V. Ramasami Periyar. The organisation
strongly opposed the Brahmins dominance and affirmed regional pride against the
political, economic and cultural domination of the North. Initially, the Dravidian
movement spoke in terms of the whole of south India; however lack of support
from other States limited the movement to Tamil Nadu.

14. Solution: b)

Internally, there is a dispute about the status of Kashmir within the Indian union.
Article 370 gives greater autonomy to Jammu and Kashmir compared to other
http://insightsonindia.com

INSIGHTS

Page 8

Facebook Group: Indian Administrative Service (Raz Kr)

INSIGHTS MOCK TEST 7 SOLUTIONS

States of India. The State has its own Constitution. All provisions of the Indian
Constitution are not applicable to the State. Laws passed by the Parliament apply
to J&K only if the State agrees.
This special status has provoked two opposite reactions. There is a section of
people outside of J&K that believes that the special status of the State conferred
by Article 370 does not allow full integration of the State with India. This section
feels that Article 370 should therefore be revoked and J&K should be like any
other State in India.
Another section, mostly Kashmiris, believe that the autonomy conferred by
Article 370 is not enough. A section of Kashmiris have expressed at least three
major grievances. First, the promise that Accession would be referred to the
people of the State after the situation created by tribal invasion was normalised,
has not been fulfilled. This has generated the demand for a Plebiscite. Secondly,
there is a feeling that the special federal status guaranteed by Article 370, has been
eroded in practice. This has led to the demand for restoration of autonomy or
Greater State Autonomy. Thirdly, it is felt that democracy which is practiced in
the rest of India has not been similarly institutionalised in the State of Jammu and
Kashmir.

15. Solution: c)
In the North-East, regional aspirations reached a turning point in 1980s. This region
now consists of seven States, also referred to as the seven sisters. The region has
only 4 per cent of the countrys population but about twice as much share of its
area. A small corridor of about 22 kilometers connects the region to the rest of the
country. Otherwise the region shares boundaries with China, Myanmar and
Bangladesh and serves as Indias gateway to South East Asia.
The region has witnessed a lot of change since 1947. Tripura, Manipur and Khasi
Hills of Meghalaya were erstwhile Princely States which merged with India after
Independence. The entire region of North-East has undergone considerable
political reorganisation. Nagaland State was created in 1960; Meghalaya, Manipur
and Tripura in 1972 while Arunachal Pradesh and Mizoram became separate
http://insightsonindia.com

INSIGHTS

Page 9

Facebook Group: Indian Administrative Service (Raz Kr)

INSIGHTS MOCK TEST 7 SOLUTIONS

States only in 1986. The Partition of India in 1947 had reduced the North-East to a
land locked region and affected its economy. Cut off from the rest of India, the
region suffered neglect in developmental terms. Its politics too remained insulated.
At the same time, most States in this region underwent major demographic
changes due to influx of migrants from neighbouring States and countries.
The isolation of the region, its complex social character and its backwardness
compared to other parts of the country have all resulted in the complicated set of
demands from different states of the North-East. The vast international border
and weak communication between the North-East and the rest of India have
further added to the delicate nature of politics there. Three issues dominate the
politics of North-East: demands for autonomy, movements for secession, and
opposition to outsiders. Major initiatives on the first issue in the 1970s set the
stage for some dramatic developments on the second and the third in the 1980s.

16. Solution: a)
Goas liberation
Although the British empire in India came to an end in 1947, Portugal refused to
withdraw from the territories of Goa, Diu and Daman which were under its
colonial rule since the sixteenth century. During their long rule, the Portuguese
suppressed the people of Goa, denied them civil rights, and carried out forced
religious conversions. After Indias Independence, the Indian government tried very
patiently to persuade the Portuguese government to withdraw. There was also a
strong popular movement within Goa for freedom. They were strengthened by
socialist satyagrahis from Maharashtra. Finally, in December 1961, the Government
of India sent the army which liberated these territories after barely two days of
action. Goa, Diu and Daman became Union Territory.
Another complication arose soon. Led by the Maharashtrawadi Gomanatak Party
(MGP) one section desired that Goa, as a Marathi speaking area should merge with
Maharashtra. However, many Goans were keen to retain a separate Goan identity
and culture, particularly the Konkani language. They were led by the United Goan
Party (UGP). In January 1967, the Central Government held a special opinion poll
in Goa asking people to decide if they wanted to be part of Maharashtra or remain
http://insightsonindia.com

INSIGHTS

Page 10

Facebook Group: Indian Administrative Service (Raz Kr)

INSIGHTS MOCK TEST 7 SOLUTIONS

separate. This was the only time in independent India that a referendum-like
procedure was used to ascertain peoples wishes on a subject. The majority voted
in favour of remaining outside of Maharashtra. Thus, Goa continued as a Union
Territory. Finally, in 1987, Goa became a State of the Indian Union.
17. Solution: d)
Phase I : The period from 1901-1921 is referred to as a period of stagnant or
stationary phase of growth of Indias population, since in this period growth rate
was very low, even recording a negative growth rate during 1911-1921. Both the
birth rate and death rate were high keeping the rate of increase low. Poor health
and medical services, illiteracy of people at large and inefficient distribution system
of food and other basic necessities were largely responsible for a high birth and
death rates in this period.
Phase II : The decades 1921-1951 are referred to as the period of steady population
growth. An overall improvement in health and sanitation throughout the country
brought down the mortality rate. At the same time better transport and
communication system improved distribution system. The crude birth rate
remained high in this period leading to higher growth rate than the previous phase.
This is impressive at the backdrop of Great Economic Depression, 1920s and World
War II.
Phase III : The decades 1951-1981 are referred to as the period of population
explosion in India, which was caused by a rapid fall in the mortality rate but a high
fertility rate of population in the country. The average annual growth rate was as
high as 2.2 per cent. It is in this period, after the Independence, that developmental
activities were introduced through a centralised planning process and economy
started showing up ensuring the improvement of living condition of people at large.
Consequently, there was a high natural increase and higher growth rate. Besides,
increased international migration bringing in Tibetans, Bangladeshis, Nepalies and
even people from Pakistan contributed to the high growth rate.
Phase IV : In the post 1981 till present, the growth rate of countrys population
though remained high, has started slowing down gradually (Table 1.1). A downward
trend of crude birth rate is held responsible for such a population growth. This was,

http://insightsonindia.com

INSIGHTS

Page 11

Facebook Group: Indian Administrative Service (Raz Kr)

INSIGHTS MOCK TEST 7 SOLUTIONS

in turn, affected by an increase in the mean age at marriage, improved quality of life
particularly education of females in the country.
The growth rate of population is, however, still high in the country, and it has been
projected by World Development Report that population of India will touch 1,350
million by 2025.

18. Solution: c)
Refer page no. 10, India People and Economy, 12th NCERT

19. Solution: d)
Data from Indian Remote Sensing satellites are used for various applications of
resources survey and management under the National Natural Resources
Management System (NNRMS). Following is the list of those applications:
Preharvest crop area and production estimation of major crops.
Drought monitoring and assessment based on vegetation condition.
Flood risk zone mapping and flood damage assessment.
Hydro-geomorphological maps for locating underground water resources for
drilling well.
Irrigation command area status monitoring
Snow-melt run-off estimates for planning water use in down stream projects
Land use and land cover mapping
Urban planning
Forest survey
Wetland mapping
Environmental impact analysis
Mineral Prospecting
Coastal studies
Integrated Mission for Sustainable Development (initiated in 1992) for
generating locale-specific prescriptions for integrated land and water resources
development in 174 districts.

http://insightsonindia.com

INSIGHTS

Page 12

Facebook Group: Indian Administrative Service (Raz Kr)

INSIGHTS MOCK TEST 7 SOLUTIONS

20. Solution: c)
Mumbai is a natural harbour and the biggest port of the country. The port is
situated closer to the general routes from the countries of Middle East,
Mediterranean countries, North Africa, North America and Europe where the
major share of countrys overseas trade is carried out. The port is 20 km long and 610 km wide with 54 berths and has the countrys largest oil terminal. M.P.,
Maharashtra, Gujarat, U.P. and parts of Rajasthan constitute the main hinterlands of
Mumbai ports.
Mumbai Port, earlier known as Bombay Port, lies midway (Latitude 18 56.3' N,
Longitude 72 45.9' E) on the West coast of India, on the natural deepwaterharbour of Mumbai. The harbour spread over 400 square kilometres
(150 sq mi) is protected by the mainland of Konkan to its east and north and by the
island city of Mumbai to its west.The harbour opens to the south to the Arabian
Sea.
The port is administered by the Mumbai Port Trust (MbPT), an autonomous
corporation wholly owned by the Government of India. The port is primarily used
for bulk cargo, while most container traffic is directed to Nhava Sheva port across
the harbour.

21. Solution: c)
Total dissolved solids (TDS) is the term used to describe the inorganic salts and
small amounts of organic matter present in solution in water. The principal
constituents are usually calcium, magnesium, sodium, and potassium cations and
carbonate, hydrogencarbonate, chloride, sulfate, and nitrate anions.
The method of determining TDS in water supplies most commonly used is the
measurement of specific conductivity with a conductivity probe that detects the
presence of ions in water. Conductivity measurements are converted into TDS
values by means of a factor that varies with the type of water (2,3). The practical
quantitation limit for TDS in water by this method is 10 mg/litre (M. Forbes,
personal communication, 1988). High TDS concentrations can also be measured
gravimetrically, although volatile organic compounds are lost by this method (4).
The constituents of TDS can also be measured individually.
Primary sources for TDS in receiving waters are agricultural and residential runoff,
leaching of soil contamination and point source water pollution discharge from
industrial or sewage treatment plants.
http://insightsonindia.com

INSIGHTS

Page 13

Facebook Group: Indian Administrative Service (Raz Kr)

INSIGHTS MOCK TEST 7 SOLUTIONS

22. Solution: d)
http://www.thehindu.com/news/national/undp-brackets-india-with-equatorialguinea-in-human-development-index/article4510390.ece

23. Solution: d)
All are correct.

24. Solution: d)
India has undergone major changes within the economy over the past four or five
decades, and this has influenced the land-use changes in the country. These changes
between 1960- 61 and 2002-03. .
Three categories have undergone increases, while four have registered declines.
Share of area under forest, area under non- agricultural uses and current fallow
lands have shown an increase. The following observations can be made about these
increases:
(i) The rate of increase is the highest in case of area under non-agricultural uses.
This is due to the changing structure of Indian economy, which is increasingly
depending on the contribution from industrial and services sectors and expansion of
related infrastructural facilities. Also, an expansion of area under both urban and
rural settlements has added to the increase. Thus, the area under non-agricultural
uses is increasing at the expense of wastelands and agricultural land.
(ii) The increase in the share under forest, as explained before, can be accounted for
by increase in the demarcated area under forest rather than an actual increase in the
forest cover in the country.
(iii) The increase in the current fallow cannot be explained from information
pertaining to only two points. The trend of current fallow fluctuates a great deal
over years, depending on the variability of rainfall and cropping cycles.

http://insightsonindia.com

INSIGHTS

Page 14

Facebook Group: Indian Administrative Service (Raz Kr)

INSIGHTS MOCK TEST 7 SOLUTIONS

The four categories that have registered a decline are barren and wasteland,
culturable wasteland, area under pastures and tree crops and net area sown.
The following explanations can be given for the declining trends:
(i) As the pressure on land increased, both from the agricultural and nonagricultural sectors, the wastelands and culturable wastelands have witnessed
decline over time.
(ii) The decline in net area sown is a recent phenomenon that started in the late
nineties, before which it was registering a slow increase. There are indications that
most of the decline has occurred due to the increases in area under non- agricultural
use. (Note : the expansion of building activity on agricultural land in your village
and city).
(iii) The decline in land under pastures and grazing lands can be explained by
pressure from agricultural land. Illegal encroachment due to expansion of
cultivation on common pasture lands is largely responsible for this decline.

25. Solution: d)
Coffee is a tropical plantation crop. Its seeds are roasted, ground and are used for
preparing a beverage. There are three varieties of coffee i.e. arabica, robusta and
liberica. India mostly grows superior quality coffee, arabica, which is in great
demand in International market. But India produces only about 4.3 percent coffee
of the world and ranks sixth after Brazil, Vietnam, Colombia, Indonesia and
Mexico. Coffee is cultivated in the highlands of Western Ghats in Karnataka,
Kerala and Tamil Nadu. Karnataka alone accounts for more than two- third of total
production of coffee in the country.

Meanwhile, coffee had been introduced to Brazil in 1727, although its cultivation
did not gather momentum until independence in 1822. After this time, massive
tracts of rainforest were cleared first from the vicinity of Rio and later So Paulo for
coffee plantations.Cultivation was taken up by many countries in Central America
in the latter half of the 19th century, and almost all involved the large-scale
http://insightsonindia.com

INSIGHTS

Page 15

Facebook Group: Indian Administrative Service (Raz Kr)

INSIGHTS MOCK TEST 7 SOLUTIONS

displacement and exploitation of the indigenous people. Harsh conditions led to


many uprisings, coups and bloody suppression of peasants. The notable exception
was Costa Rica, where lack of ready labor prevented the formation of large farms.
Smaller farms and more egalitarian conditions ameliorated unrest over the 19th and
20th centuries.
Coffee has become a vital cash crop for many developing countries. Over one
hundred million people in developing countries have become dependent on coffee
as their primary source of income. It has become the primary export and backbone
for African countries like Uganda, Burundi, Rwanda, and Ethiopia, as well as many
Central American countries.
26. Solution: b)
Mahatma Gandhi returned to India in January 1915. As you know, he had come
from South Africa where he had successfully fought the racist regime with a novel
method of mass agitation, which he called satyagraha. The idea of satyagraha
emphasised the power of truth and the need to search for truth. It suggested that if
the cause was true, if the struggle was against injustice, then physical force was not
necessary to fight the oppressor. Without seeking vengeance or being aggressive, a
satyagrahi could win the battle through non- violence. This could be done by
appealing to the conscience of the oppressor. People including the oppressors
had to be persuaded to see the truth, instead of being forced to accept truth
through the use of violence. By this struggle, truth was bound to ultimately
triumph. Mahatma Gandhi believed that this dharma of non-violence could unite
all Indians.
After arriving in India, Mahatma Gandhi successfully organized satyagraha
movements in various places. In 1916 he travelled to Champaran in Bihar to inspire
the peasants to struggle against the oppressive plantation system. Then in 1917, he
organised a satyagraha to support the peasants of the Kheda district of Gujarat.
Affected by crop failure and a plague epidemic, the peasants of Kheda could not
pay the revenue, and were demanding that revenue collection be relaxed. In 1918,
Mahatma Gandhi went to Ahmedabad to organize a satyagraha movement amongst
cotton mill workers.
Mahatma Gandhi on Satyagraha

http://insightsonindia.com

INSIGHTS

Page 16

Facebook Group: Indian Administrative Service (Raz Kr)

INSIGHTS MOCK TEST 7 SOLUTIONS

It is said of passive resistance that it is the weapon of the weak, but the power
which is the subject of this article can be used only by the strong. This power is not
passive resistance; indeed it calls for intense activity. The movement in South
Africa was not passive but active
Satyagraha is not physical force. A satyagrahi does not inflict pain on the adversary;
he does not seek his destruction In the use of satyagraha, there is no ill-will
whatever.
Satyagraha is pure soul-force. Truth is the very substance of the soul. That is why
this force is called satyagraha. The soul is informed with knowledge. In it burns the
flame of love. Non- violence is the supreme dharma
It is certain that India cannot rival Britain or Europe in force of arms. The British
worship the war-god and they can all of them become, as they are becoming,
bearers of arms. The hundreds of millions in India can never carry arms. They have
made the religion of non-violence their own ...

27. Solution: c)
The Rowlatt Act passed by the Imperial Legislative Council in London on 10 March
1919, indefinitely extending "emergency measures" (of the Defence of India
Regulations Act) enacted during the First World War in order to control public
unrest and root out conspiracy, in India. Passed on the recommendations of
the Rowlatt Committee, named after its president, British judge Sir Sidney Rowlatt,
this act effectively authorized the government to imprison for up to two years,
without trial, any person suspected of terrorism living in the Raj and gave the
imperial authorities power to deal with revolutionary activities. The unpopular
legislation provided for stricter control of the press, arrests without warrant,
indefinite detention without trial, and juryless in camera trials for proscribed
political acts. The accused was denied the right to know the accusers and the
evidence used in the trial.[1] Those convicted were required to deposit securities
upon release and were prohibited from taking part in any political, educational, or
religious activities.[1]
Mohandas Karamchand Gandhi, among other Indian leaders, was extremely critical
of the Act and argued that not everyone should get punishment in response to
isolated political crimes. The Act annoyed many Indian leaders and the public,
which caused the government to implement repressive measures. Gandhi and
http://insightsonindia.com

INSIGHTS

Page 17

Facebook Group: Indian Administrative Service (Raz Kr)

INSIGHTS MOCK TEST 7 SOLUTIONS

others found that constitutional opposition to the measure was fruitless, so on April
6, a "hartal" was organised where Indians would suspend all business and fast as a
sign of their opposition. This event is known as theRowlatt satyagraha. Gandhiji
named rowlatt act as "black act".
However, the success of the hartal in Delhi, on 30 March, was overshadowed by
tensions running high, which resulted in rioting in thePunjab and other provinces.
Deciding that Indians were not ready to make a stand consistent with the principle
of ahimsa (non-violence), an integral part of satyagraha, Gandhi suspended the
resistance.
The Rowlatt Act came into effect in March 1919. In the Punjab the protest
movement was very strong, and on 10 April, two leaders of the congress, Dr. Satya
Pal and Dr. Saifuddin Kitchlew, were arrested and taken to an unknown place.
Army was called in Punjab and on 13 April people from neighbouring villages
gathered for Baisakhi Day celebrations in Amritsar, which led to the
infamous Jallianwala Bagh massacre of 1919.[2][3]
Accepting the report of the Repressive Laws Committee, the Government of
India repealed the Rowlatt Act, the Press Act and twenty-two other laws in March
1922

Among the significant causes of Non-Cooperation movement were colonial


oppression, exemplified by the Rowlatt Act and Jallianwala Bagh massacre,
economic hardships to the common man due to a large chunk of Indian wealth
being exported to Britain, ruin of Indian artisans due to British factory-made goods
replacing handmade goods, and popular resentment with the British over Indian
soldiers dying in World War I while fighting as part of the British Army, in battles
that otherwise had nothing to do with India.

28. Solution: d)
New seed varieties of wheat (Mexico) and rice (Philippines) known as high
yielding varieties (HYVs) were available for cultivation by mid-1960s. India took
advantage of this and introduced package technology comprising HYVs, along with
chemical fertilizers in irrigated areas of Punjab, Haryana, Western Uttar Pradesh,
Andhra Pradesh and Gujarat. Assured supply of soil moisture through irrigation was
a basic pre-requisite for the success of this new agricultural technology. This
http://insightsonindia.com

INSIGHTS

Page 18

Facebook Group: Indian Administrative Service (Raz Kr)

INSIGHTS MOCK TEST 7 SOLUTIONS

strategy of agricultural development paid dividends instantly and increased the


foodgrains production at very fast rate. This spurt of agricultural growth came to be
known as Green Revolution. This also gave fillip to the development of a large
number of agro-inputs, agro-processing industries and small-scale industries. This
strategy of agricultural development made the country self-reliant in foodgrain
production. But green revolution was initially confined to irrigated areas only. This
led to regional disparities in agricultural development in the country till the
seventies, after which the technology spread to the Eastern and Central parts of the
country.
The Planning Commission of India focused its attention on the problems of
agriculture in rainfed areas in 1980s. It initiated agro-climatic planning in 1 988 to
induce regionally balanced agricultural development in the country. It also
emphasized the need for diversification of agriculture and harnessing of resources
for development of dairy farming, poultry, horticulture, livestock rearing and
aquaculture.
Initiation of the policy of liberalisation and free market economy in 1990s is likely
to influence the course of development of Indian agriculture. Lack of development
of rural infrastructure, withdrawal of subsidies and price support, and impediments
in availing of the rural credits may lead to inter-regional and inter-personal
disparities in rural areas.

29. Solution: b)
From the cities, the Non-Cooperation Movement spread to the countryside. It
drew into its fold the struggles of peasants and tribals which were developing in
different parts of India in the years after the war.
In Awadh, peasants were led by Baba Ramchandra a sanyasi who had earlier been
to Fiji as an indentured labourer. The movement here was against talukdars and
landlords who demanded from peasants exorbitantly high rents and a variety of
other cesses. Peasants had to do begar and work at landlords farms without any
payment.
As tenants they had no security of tenure, being regularly evicted so that they could
acquire no right over the leased land. The peasant movement demanded reduction
http://insightsonindia.com

INSIGHTS

Page 19

Facebook Group: Indian Administrative Service (Raz Kr)

INSIGHTS MOCK TEST 7 SOLUTIONS

of revenue, abolition of begar, and social boycott of oppressive landlords. In many


places nai dhobi bandhs were organised by panchayats to deprive landlords of the
services of even barbers and washermen. In June 1920, Jawaharlal Nehru began
going around the villages in Awadh, talking to the villagers, and trying to
understand their grievances. By October, the Oudh Kisan Sabha was set up headed
by Jawaharlal Nehru, Baba Ramchandra and a few others. Within a month, over
300 branches had been set up in the villages around the region. So when the NonCooperation Movement began the following year, the effort of the Congress was to
integrate the Awadh peasant struggle into the wider struggle. The peasant
movement, however, developed in forms that the Congress leadership was unhappy
with. As the movement spread in 1921, the houses of talukdars and merchants were
attacked, bazaars were looted, and grain hoards were taken over. In many places
local leaders told peasants that Gandhiji had declared that no taxes were to be paid
and land was to be redistributed among the poor. The name of the Mahatma was
being invoked to sanction all action and aspirations.

30. Solution: a)
Workers too had their own understanding of Mahatma Gandhi and the notion of
swaraj. For plantation workers in Assam, freedom meant the right to move freely in
and out of the confined space in which they were enclosed, and it meant retaining a
link with the village from which they had come. Under the Inland Emigration Act
of 1859, plantation workers were not permitted to leave the tea gardens without
permission, and in fact they were rarely given such permission. When they heard of
the Non-Cooperation Movement, thousands of workers defied the authorities, left
the plantations and headed home. They believed that Gandhi Raj was coming and
everyone would be given land in their own villages.
They, however, never reached their destination. Stranded on the way by a railway
and steamer strike, they were caught by the police and brutally beaten up.

http://insightsonindia.com

INSIGHTS

Page 20

Facebook Group: Indian Administrative Service (Raz Kr)

INSIGHTS MOCK TEST 7 SOLUTIONS

31. Solution: a)
Harvesting of a crop is an important task. The cutting of crop after it is mature is
called harvesting. In harvesting, crops are pulled out or cut close to the ground. It
usually takes 3 to 4 months for a cereal crop to mature.
Harvesting in our country is either done manually by sickle or by a machine called
harvester. In the harvested crop, the grain seeds need to be separated from the
chaff. This process is called threshing. This is carried out with the help of a machine
called combine which is in fact a combined harvester and thresher Farmers with
small holdings of land do the separation of grain and chaff by winnowing

32. Solution: b)
Indian peasantry had been exploited for a long time as there had been unequal
distribution of land. Among the three revenue systems operational during British
period i.e. Mahalwari, Ryotwari and Zamindari, the last one was most exploitative
for the peasants
mahalwari system, one of the three main revenue systems of land tenure in British
India, the other two being the zamindar (landlord) and the ryotwari (individual
cultivator). The wordmahalwari is derived from the Hindi mahal, meaning a house
or, by extension, a district.
For revenue purposes the name was applied to any compact area containing one or
more villages, which were called estates. The revenue settlement was made with
the estatehence the term mahalwariand there were distinct types
of assessment. If a zamindar held the whole estate, the settlement was with him;
otherwise, payment was exacted from individual cultivators.
33. Solution: b)
Rust of wheat is caused by a Fungi
Citrus canker is caused by Bacteria

http://insightsonindia.com

INSIGHTS

Page 21

Facebook Group: Indian Administrative Service (Raz Kr)

INSIGHTS MOCK TEST 7 SOLUTIONS

34. Solution: c)
Mahatma Gandhi found in salt a powerful symbol that could unite the nation. On
31 January 1930, he sent a letter to Viceroy Irwin stating eleven demands. Some of
these were of general interest; others were specific demands of different classes,
from industrialists to peasants. The idea was to make the demands wide-ranging, so
that all classes within Indian society could identify with them and everyone could
be brought together in a united campaign. The most stirring of all was the demand
to abolish the salt tax. Salt was something consumed by the rich and the poor alike,
and it was one of the most essential items of food. The tax on salt and the
government monopoly over its production, Mahatma Gandhi declared, revealed the
most oppressive face of British rule.
Mahatma Gandhis letter was, in a way, an ultimatum. If the demands were not
fulfilled by 11 March, the letter stated, the Congress would launch a civil
disobedience campaign. Irwin was unwilling to negotiate. So Mahatma Gandhi
started his famous salt march accompanied by 78 of his trusted volunteers. The
march was over 240 miles, from Gandhijis ashram in Sabarmati to the Gujarati
coastal town of Dandi. The volunteers walked for 24 days, about 10 miles a day.
Thousands came to hear Mahatma Gandhi wherever he stopped, and he told them
what he meant by swaraj and urged them to peacefully defy the British. On 6 April
he reached Dandi, and ceremonially violated the law, manufacturing salt by boiling
sea water.

35. Solution: d)
Acrylic fibers are synthetic fibers made from a polymer (polyacrylonitrile) with an
average molecular weight of ~100,000, about 1900 monomer units. To be called
acrylic in the U.S, the polymer must contain at least 85% acrylonitrile monomer.
Typical comonomers arevinyl acetate or methyl acrylate. DuPont created the first
acrylic fibers in 1941 and trademarked them under the name Orlon. Acrylic is also
called acrilan fabric. As it resembles silk, it is also called artificial silk.

http://insightsonindia.com

INSIGHTS

Page 22

Facebook Group: Indian Administrative Service (Raz Kr)

INSIGHTS MOCK TEST 7 SOLUTIONS

36. Solution: a)
It was launched by NDA government in 20013. The Central and State
Governments have initiated many watershed development and management
programmes in the country. Some of these are being implemented by nongovernmental organisations also. Haryali is a watershed development project
sponsored by the Central Government which aims at enabling the rural population
to conserve water for drinking, irrigation, fisheries and afforestation. The Project is
being executed by Gram Panchayats with peoples participation. Neeru-Meeru
(Water and You) programme (in Andhra Pradesh) and Arvary Pani Sansad (in
Alwar, Rajasthan) have taken up constructions of various water-harvesting
structures such as percolation tanks, dug out ponds (Johad), check dams, etc.
through peoples participation. Tamil Nadu has made water harvesting structures in
the houses compulsory. No building can be constructed without making structures
for water harvesting.
Watershed development projects in some areas have been successful in
rejuvenating environment and economy. However, there are only a few success
stories. In majority of cases, the programme is still in its nascent stage. There is a
need to generate awareness regarding benefits of watershed development and
management among people in the country, and through this integrated water
resource management approach water availability can be ensured on sustainable
basis.

37. Solution: b)
An electroscope is an early scientific instrument that is used to detect the presence
and magnitude of electric charge on a body. It was the first electrical measuring
instrument. The first electroscope, a pivoted needle called the versorium, was
invented by British physicianWilliam Gilbert around 1600. The pith-ball
electroscope and the gold-leaf electroscopeare two classical types of electroscope
that are still used in physics education to demonstrate the principles
of electrostatics. A type of electroscope is also used in thequartz fiber radiation
dosimeter. Electroscopes were used by the Austrian scientist Victor Hess in the
discovery of cosmic rays.
Electroscopes detect electric charge by the motion of a test object due to
the Coulomb electrostatic force. The electric potential orvoltage of an object equals
http://insightsonindia.com

INSIGHTS

Page 23

Facebook Group: Indian Administrative Service (Raz Kr)

INSIGHTS MOCK TEST 7 SOLUTIONS

its charge divided by its capacitance, so electroscopes can be regarded as


crude voltmeters. The accumulation of enough charge to detect with an
electroscope requires hundreds or thousands of volts, so electroscopes are only used
with high voltage sources such as static electricity and electrostatic machines.
Electroscopes generally give only a rough, qualitative indication of the magnitude of
the charge; an instrument that measures charge quantitatively is called
an electrometer.

38. Solution: c)
Many nationalists thought that the struggle against the British could not be won
through non-violence. In 1928, the Hindustan Socialist Republican Army (HSRA)
was founded at a meeting in Ferozeshah Kotla ground in Delhi. Amongst its leaders
were Bhagat Singh, Jatin Das and Ajoy Ghosh. In a series of dramatic actions in
different parts of India, the HSRA targeted some of the symbols of British power.
In April 1929, Bhagat Singh and Batukeswar Dutta threw a bomb in the Legislative
Assembly. In the same year there was an attempt to blow up the train that Lord
Irwin was travelling in. Bhagat Singh was 23 when he was tried and executed by the
colonial government. During his trial, Bhagat Singh stated that he did not wish to
glorify the cult of the bomb and pistol but wanted a revolution in society:
Revolution is the inalienable right of mankind. Freedom is the imprescriptible
birthright of all. The labourer is the real sustainer of society To the altar of this
revolution we have brought our youth as incense, for no sacrifice is too great for so
magnificent a cause. We are content. We await the advent of revolution. Inquilab
Zindabad!

39. Solution: a)
http://en.wikipedia.org/wiki/Alluri_Sita_Rama_Raju

40. Solution: c)
India is endowed with fairly abundant resources of iron ore. It has the largest
reserve of iron ore in Asia. The two main types of ore found in our country are
http://insightsonindia.com

INSIGHTS

Page 24

Facebook Group: Indian Administrative Service (Raz Kr)

INSIGHTS MOCK TEST 7 SOLUTIONS

haematite and magnetite. It has great demand in international market due to its
superior quality. The iron ore mines occur in close proximity to the coal fields in
the north-eastern plateau region of the country which adds to their advantage.
The total reserves of iron ore in the country were about 20 billion tonnes in the
year 2004- 05. About 95 per cent of total reserves of iron ore is located in the States
of Orissa, Jharkhand, Chhattisgarh, Karnataka, Goa, Andhra Pradesh and Tamil
Nadu. In Orissa, iron ore occurs in a series of hill ranges in Sundergarh, Mayurbhanj
and Jhar. The important mines are Gurumahisani, Sulaipet, Badampahar
(Mayurbhaj), Kiruburu (Kendujhar) and Bonai (Sundergarh). Similar hill ranges,
Jharkhand has some of the oldest iron ore mines and most of the iron and steel
plants are located around them. Most of the important mines such as Noamundi
and Gua are located in Poorbi and Pashchimi Singhbhum districts. This belt further
extends to Durg, Dantewara and Bailadila. Dalli, and Rajhara in Durg are the
important mines of iron ore in the country. In Karnataka, iron ore deposits occur in
Sandur-Hospet area of Bellary district, Baba Budan hills and Kudremukh in
Chikmagalur district and parts of Shimoga, Chitradurg and Tumkur districts. The
districts of Chandrapur, Bhandara and Ratnagiri in Maharashtra, Karimnagar,
Warangal, Kurnool, Cuddapah and Anantapur districts of Andhra Pradesh, Salem
and Nilgiris districts of Tamil Nadu are other iron mining regions. Goa has also
emerged as an important producer of iron ore.

41. Solution: a)
Bioenergy is renewable energy made available from materials derived from
biological sources. Biomass is any organic material which has stored sunlight in the
form of chemical energy. As a fuel it may include wood, wood
waste, straw, manure, sugarcane, and many other byproducts from a variety of
agricultural processes. By 2010, there was 35 GW (47,000,000 hp) of globally
installed bioenergy capacity for electricity generation, of which 7 GW
(9,400,000 hp) was in theUnited States.[1]
In its most narrow sense it is a synonym to biofuel, which is fuel derived from
biological sources. In its broader sense it includes biomass, the biological material
used as a biofuel, as well as the social, economic, scientific and technical fields
associated with using biological sources for energy. This is a common
misconception, as bioenergy is the energy extracted from the biomass, as the
biomass is the fuel and the bioenergy is the energy contained in the fuel
http://insightsonindia.com

INSIGHTS

Page 25

Facebook Group: Indian Administrative Service (Raz Kr)

INSIGHTS MOCK TEST 7 SOLUTIONS

42. Solution: c)
Dr B.R. Ambedkar, who organised the dalits into the Depressed Classes Association
in 1930, clashed with Mahatma Gandhi at the second Round Table Conference by
demanding separate electorates for dalits. When the British government conceded
Ambedkars demand, Gandhiji began a fast unto death. He believed that separate
electorates for dalits would slow down the process of their integration into society.
Ambedkar ultimately accepted Gandhijis position and the result was the Poona
Pact of September 1932. It gave the Depressed Classes (later to be known as the
Schedule Castes) reserved seats in provincial and central legislative councils, but
they were to be voted in by the general electorate. The dalit movement, however,
continued to be apprehensive of the Congress- led national movement. Some of the
Muslim political organisations in India were also lukewarm in their response to the
Civil Disobedience Movement.
After the decline of the Non-Cooperation-Khilafat movement, a large section of
Muslims felt alienated from the Congress. From the mid-1920s the Congress came
to be more visibly associated with openly Hindu religious nationalist groups like
the Hindu Mahasabha. As relations between Hindus and Muslims worsened, each
community organised religious processions with militant fervour, provoking HinduMuslim communal clashes and riots in various cities. Every riot deepened the
distance between the two communities.

43. Solution: c)
In 1930, Sir Muhammad Iqbal, as president of the Muslim League, reiterated the
importance of separate electorates for the Muslims as an important safeguard for
their minority political interests. His statement is supposed to have provided the
intellectual justification for the Pakistan demand that came up in subsequent years.
This is what he said:
I have no hesitation in declaring that if the principle that the Indian Muslim is
entitled to full and free development on the lines of his own culture and tradition
in his own Indian home-lands is recognised as the basis of a permanent communal
settlement, he will be ready to stake his all for the freedom of India. The principle
http://insightsonindia.com

INSIGHTS

Page 26

Facebook Group: Indian Administrative Service (Raz Kr)

INSIGHTS MOCK TEST 7 SOLUTIONS

that each group is entitled to free development on its own lines is not inspired by
any feeling of narrow communalism A community which is inspired by feelings
of ill-will towards other communities is low and ignoble.
I entertain the highest respect for the customs, laws, religions and social institutions
of other communities. Nay, it is my duty according to the teachings of the Quran,
even to defend their places of worship, if need be. Yet I love the communal group
which is the source of life and behaviour and which has formed me what I am by
giving me its religion, its literature, its thought, its culture and thereby its whole
past as a living operative factor in my present consciousness
Communalism in its higher aspect, then, is indispensable to the formation of a
harmonious whole in a country like India. The units of Indian society are not
territorial as in European countries The principle of European democracy cannot
be applied to India without recognising the fact of communal groups. The Muslim
demand for the creation of a Muslim India within India is, therefore, perfectly
justified
The Hindu thinks that separate electorates are contrary to the spirit of true
nationalism, because he understands the word nation to mean a kind of universal
amalgamation in which no communal entity ought to retain its private
individuality. Such a state of things, however, does not exist. India is a land of racial
and religious variety. Add to this the general economic inferiority of the Muslims,
their enormous debt, especially in the Punjab, and their insufficient majorities in
some of the provinces, as at present constituted and you will begin to see clearly
the meaning of our anxiety to retain separate electorates.

44. Solution: c)
The example of indentured labour migration from India also illustrates the twosided nature of the nineteenth-century world. It was a world of faster economic
growth as well as great misery, higher incomes for some and poverty for others,
technological advances in some areas and new forms of coercion in others.
In the nineteenth century, hundreds of thousands of Indian and Chinese labourers
went to work on plantations, in mines, and in road and railway construction
projects around the world. In India, indentured labourers were hired under
http://insightsonindia.com

INSIGHTS

Page 27

Facebook Group: Indian Administrative Service (Raz Kr)

INSIGHTS MOCK TEST 7 SOLUTIONS

contracts which promised return travel to India after they had worked five years on
their employers plantation.
Most Indian indentured workers came from the present-day regions of eastern
Uttar Pradesh, Bihar, central India and the dry districts of Tamil Nadu. In the midnineteenth century these regions experienced many changes cottage industries
declined, land rents rose, lands were cleared for mines and plantations. All this
affected the lives of the poor: they failed to pay their rents, became deeply indebted
and were forced to migrate in search of work.
The main destinations of Indian indentured migrants were the Caribbean islands
(mainly Trinidad, Guyana and Surinam), Mauritius and Fiji.
Closer home, Tamil migrants went to Ceylon and Malaya. Indentured workers were
also recruited for tea plantations in Assam.
Recruitment was done by agents engaged by employers and paid a small
commission. Many migrants agreed to take up work hoping to escape poverty or
oppression in their home villages.
Agents also tempted the prospective migrants by providing false information about
final destinations, modes of travel, the nature of the work, and living and working
conditions. Often migrants were not even told that they were to embark on a long
sea voyage. Sometimes agents even forcibly abducted less willing migrants.
Nineteenth-century indenture has been described as a new system of slavery. On
arrival at the plantations, labourers found conditions to be different from what they
had imagined. Living and working conditions were harsh, and there were few legal
rights.
But workers discovered their own ways of surviving. Many of them escaped into
the wilds, though if caught they faced severe punishment. Others developed new
forms of individual and collective self- expression, blending different cultural forms,
old and new. In Trinidad the annual Muharram procession was transformed into a
riotous carnival called Hosay (for Imam Hussain) in which workers of all races and
religions joined. Similarly, the protest religion of Rastafarianism (made famous by
the Jamaican reggae star Bob Marley) is also said to reflect social and cultural links
with Indian migrants to the Caribbean. Chutney music, popular in Trinidad and
Guyana, is another creative contemporary expression of the post-indenture
http://insightsonindia.com

INSIGHTS

Page 28

Facebook Group: Indian Administrative Service (Raz Kr)

INSIGHTS MOCK TEST 7 SOLUTIONS

experience. These forms of cultural fusion are part of the making of the global
world, where things from different places get mixed, lose their original
characteristics and become something entirely new.

45. Solution: b)
Advantage of Intercropping:
Intercropping gives additional yield income/unit area than sole cropping.
2. It acts as an insurance against failure of crops in abnormal year.
3. Inter-crops maintain the soil fertility as the nutrient uptake is made from
both layers of soil.
4. Reduction in soil runoff and controls weeds.
5. Intercrops provide shade and support to the other crop.
6. Inter cropping system utilizes resources efficiently and their productivity is
increased (Reddy and Redid, 1992).
7. Intercropping with cash crops is higher profitable.
8. It helps to avoid inter-crop competition and thus a higher number of crop
plants are grown per unit area.
Disadvantages of intercropping:
Yield decreases as the crops differ in their competitive abilities.
2. Management of I/c having different cultural practices seems to be difficult
task.
3. Improved implements cannot be used efficiently.
4. Higher amount of fertilizer or irrigation water cannot be utilized properly
as the component crops vary in their response of these resources.
5. Harvesting is difficult.

46. Solution: c)
CITES (the Convention on International Trade in Endangered Species of Wild
Fauna and Flora, also known as the Washington Convention) is a multilateral
treaty to protect endangered plants and animals. It was drafted as a result of a
resolution adopted in 1963 at a meeting of members of the International Union for
Conservation of Nature (IUCN). The convention was opened for signature in 1973,
http://insightsonindia.com

INSIGHTS

Page 29

Facebook Group: Indian Administrative Service (Raz Kr)

INSIGHTS MOCK TEST 7 SOLUTIONS

and CITES entered into force on 1 July 1975. Its aim is to ensure that international
trade in specimens of wild animals and plants does not threaten the survival of the
species in the wild, and it accords varying degrees of protection to more than
35,000 species of animals and plants. In order to ensure that theGeneral Agreement
on Tariffs and Trade (GATT) was not violated, the Secretariat of GATT was
consulted during the drafting process
CITES is one of the largest and oldest conservation and sustainable use agreements
in existence. Participation is voluntary, and countries that have agreed to be bound
by the Convention are known as Parties. Although CITES is legally binding on the
Parties, it does not take the place of national laws. Rather it provides a framework
respected by each Party, which must adopt their own domestic legislation to
implement CITES at the national level. Often, domestic legislation is either nonexistent (especially in Parties that have not ratified it), or with penalties with the
gravity of the crime and insufficient deterrents to wildlife traders.[2] As of 2002, 50%
of Parties lacked one or more of the four major requirements for a Party:
designation of Management and Scientific Authorities; laws prohibiting the trade in
violation of CITES; penalties for such trade; laws providing for the confiscation of
specimens

47. Solution: d)
Soil erosion and depletion are the major threats to soil as a resource. Both human
and natural factors can lead to degradation of soils. Factors which lead to soil
degradation are deforestation, overgrazing, overuse of chemical feritilisers or
pesticides, rain wash, landslides and floods.
Some methods of soil conservation are
Mulching: The bare ground between plants is covered with a layer of organic
matter like straw. It helps to retain soil moisture.
Contour barriers: Stones, grass, soil are used to build barriers along contours.
Trenches are made in front of the barriers to collect water.
Rock dam: Rocks are piled up to slow down the flow of water. This prevents
gullies and further soil loss.
http://insightsonindia.com

INSIGHTS

Page 30

Facebook Group: Indian Administrative Service (Raz Kr)

INSIGHTS MOCK TEST 7 SOLUTIONS

Terrace farming: These are made on the steep slopes so that flat surfaces are
available to grow crops. They can reduce surface run-off and soil erosion.
Intercropping: Different crops are grown in alternate rows and are sown at
different times to protect the soil from rain wash.
Contour ploughing: Ploughing parallel to the contours of a hill slope to form a
natural barrier for water to flow down the slope.
Shelter belts: In the coastal and dry regions, rows of trees are planted to check the
wind movement to protect soil cover.

48. Solution: a)
Why Earthing ?
Earthing is the process of creating an alternative path for the flow of
fault/excessive currents safely into the ground in the presence of minimal
resistance or impedance.
The primary purpose of earthing is to reduce the risk of serious electric shock from
current leaking into uninsulated metal parts of an appliance, power tool, or other
electrical devices. In a properly earthed system, such leaking/fault current is carried
away harmlessly while tripping the fuse. Earthing also provides protection from
large electrical disturbances like lightning strikes and power surges. It also aids in
the dissipation of hazardous static electrical charges.
Although most electrical systems have fuses or circuit breakers for protection
against a fault current, the human body may be fatally electrocuted by a current of
less than one ampere which is well below the point at which a fuse or breaker will
operate. Earthing helps minimize such hazards from occurring.
The Earthing System
Safe Earthing Electrode
It is a metal electrode which goes into the ground near the building. It helps in the
efficient discharge of all the fault currents/ surge currents present in the electrical
system. It also helps in dissipating the high voltages which are passed on through
http://insightsonindia.com

INSIGHTS

Page 31

Facebook Group: Indian Administrative Service (Raz Kr)

INSIGHTS MOCK TEST 7 SOLUTIONS

the lightning arrestors atop buildings.


Back Fill Compounds
These are earth enhancement compounds which have different properties
depending on soil and other atmospheric conditions. Essentially, an ideal BFC has
high electrical conductivity, moisture capture and retention abilities and anti
corrosive properties. It works in tandem with the Safe Earthing Electrode. Together
both form the efficient earthing system.
Lightning Arrester
It is a metallic device mounted at the highest point of the building to capture
lightning strikes and direct it to the earth via a safe path thereby preventing it from
flowing through the building's electrcial circuit. In the absence of the lightning
arrester a lightning strike could destroy electrical equipment and cause harm to
human life through electrocutions.

49. Solution: a)
Historically, fine cottons produced in India were exported to Europe. With
industrialisation, British cotton manufacture began to expand, and industrialists
pressurised the government to restrict cotton imports and protect local industries.
Tariffs were imposed on cloth imports into Britain. Consequently, the inflow of
fine Indian cotton began to decline.
From the early nineteenth century, British manufacturers also began to seek
overseas markets for their cloth. Excluded from the British market by tariff barriers,
Indian textiles now faced stiff competition in other international markets. If we
look at the figures of exports from India, we see a steady decline of the share of
cotton textiles: from some 30 per cent around 1800 to 15 per cent by 1815. By the
1870s this proportion had dropped to below 3 per cent.
What, then, did India export? The figures again tell a dramatic story. While exports
of manufactures declined rapidly, export of raw materials increased equally fast.
Between 1812 and 1871, the share of raw cotton exports rose from 5 per cent to 35
per cent. Indigo used for dyeing cloth was another important export for many
decades. And, as you have read last year, opium shipments to China grew rapidly
http://insightsonindia.com

INSIGHTS

Page 32

Facebook Group: Indian Administrative Service (Raz Kr)

INSIGHTS MOCK TEST 7 SOLUTIONS

from the 1820s to become for a while Indias single largest export. Britain grew
opium in India and exported it to China and, with the money earned through this
sale, it financed its tea and other imports from China.
Over the nineteenth century, British manufactures flooded the Indian market. Food
grain and raw material exports from India to Britain and the rest of the world
increased. But the value of British exports to India was much higher than the value
of British imports from India. Thus Britain had a trade surplus with India. Britain
used this surplus to balance its trade deficits with other countries that is, with
countries from which Britain was importing more than it was selling to. This is how
a multilateral settlement system works it allows one countrys deficit with
another country to be settled by its surplus with a third country. By helping Britain
balance its deficits, India played a crucial role in the late-nineteenth-century world
economy.
Britains trade surplus in India also helped pay the so-called home charges that
included private remittances home by British officials and traders, interest
payments on Indias external debt, and pensions of British officials in India.

50. Solution: a)
http://www.indianetzone.com/50/indian_women_warli_movement.htm

51. Solution: d)
In 1849, Governor-General Dalhousie announced that after the death of Bahadur
Shah Zafar, the family of the king would be shifted out of the Red Fort and given
another place in Delhi to reside in.
In 1856, Governor-General Canning decided that Bahadur Shah Zafar would be the
last Mughal king and after his death none of his descendants would be recognized as
kings they would just be called princes.

http://insightsonindia.com

INSIGHTS

Page 33

Facebook Group: Indian Administrative Service (Raz Kr)

INSIGHTS MOCK TEST 7 SOLUTIONS

52. Solution: d)
Refer pages 2 to 5 Indian constitution at work, class 11th NCERT

53. Solution: a)
Fundamental identity of a people
Finally, and perhaps even most importantly, a constitution expresses the
fundamental identity of a people.
This means the people as a collective entity come into being only through the basic
constitution. It is by agreeing to a basic set of norms about how one should be
governed, and who should be governed that one forms a collective identity. One
has many sets of identities that exist prior to a constitution. But by agreeing to
certain basic norms and principles one constitutes ones basic political identity.
Second, constitutional norms are the overarching framework within which one
pursues individual aspirations, goals and freedoms. The constitution sets
authoritative constraints upon what one may or may not do. It defines the
fundamental values that we may not trespass. So the constitution also gives one a
moral identity. Third and finally, it may be the case that many basic political and
moral values are now shared across different constitutional traditions.
If one looks at constitutions around the world, they differ in many respects in
the form of government they enjoin in many procedural details. But they also share
a good deal. Most modern constitutions create a form of government that is
democratic in some respects, most claim to protect certain basic rights.
But constitutions are different in the way they embody conceptions of natural
identity. Most nations are an amalgamation of a complex set of historical traditions;
they weave together the diverse groups that reside within the nation in different
ways. For example, German identity was constituted by being ethnically G
German. The constitution gave expression to this identity.
The Indian Constitution, on the other hand, does not make ethnic identity a
criterion for citizenship. Different nations embody different conceptions of what
the relationship between the different regions of a nation and the central
http://insightsonindia.com

INSIGHTS

Page 34

Facebook Group: Indian Administrative Service (Raz Kr)

INSIGHTS MOCK TEST 7 SOLUTIONS

government should be. This relationship constitutes the national identity of a


country.

54. Solution: b)
Peoples participation has a long history in India. In 1731, the king of Jodhpur in
Rajasthan asked one of his ministers to arrange wood for constructing a new palace.
The minister and workers went to a forest near a village, inhabited by Bishnois, to
cut down trees. The Bishnoi community is known for its peaceful co-existence
with nature. The effort to cut down trees by the kings was thwarted by the
Bishnois. A Bishnoi woman Amrita Devi showed exemplary courage by hugging a
tree and daring kings men to cut her first before cutting the tree. The tree mattered
much more to her than her own life. Sadly, the kings men did not heed to her
pleas, and cut down the tree along with Amrita Devi. Her three daughters and
hundreds of other Bishnois followed her, and thus lost their lives saving trees.
Nowhere in history do we find a commitment of this magnitude when human
beings sacrificed their lives for the cause of the environment.
The Government of India has recently instituted the Amrita Devi Bishnoi Wildlife
Protection Award for individuals or communities from rural areas that have shown
extraordinary courage and dedication in protecting wildlife.

55. Solution: d)
http://agritech.tnau.ac.in/forestry/forest_jfm_index.html

http://en.wikipedia.org/wiki/Joint_Forest_Management

56. Solution: c)
http://en.wikipedia.org/wiki/Slash_and_burn#Ecological_implications

http://insightsonindia.com

INSIGHTS

Page 35

Facebook Group: Indian Administrative Service (Raz Kr)

INSIGHTS MOCK TEST 7 SOLUTIONS

57. Solution: c)
Mode of promulgation
This refers to how a constitution comes into being. Who crafted the constitution
and how much authority did they have? In many countries constitutions remain
defunct because they are crafted by military leaders or leaders who are not popular
and do not have the ability to carry the people with them. The most successful
constitutions, like India, South Africa and the United States, are constitutions
which were created in the aftermath of popular national movements. Although
Indias
Constitution was formally created by a Constituent Assembly between December
1946 and November 1949, it drew upon a long history of the nationalist movement
that had a remarkable ability to take along different sections of Indian society
together. The Constitution drew enormous legitimacy from the fact that it was
drawn up by people who enjoyed immense public credibility, who had the capacity
to negotiate and command the respect of a wide cross-section of society, and who
were able to convince the people that the constitution was not an instrument for
the aggrandisement of their personal power. The final document reflected the
broad national consensus at the time. Some countries have subjected their
constitution to a full-fledged referendum, where all the people vote on the
desirability of a constitution. The Indian Constitution was never subject to such a
referendum, but nevertheless carried enormous public authority, because it had the
consensus and backing of leaders who were themselves popular. Although the
Constitution itself was not subjected to a referendum, the people adopted it as
their own by abiding by its provisions. Therefore, the authority of people who
enact the constitution helps determine in part its prospects for success.

58. Solution: c)
Almost like a living being, this document keeps responding to the situations and
circumstances arising from time to time. Like a living being, the Constitution
responds to experience. In fact that is the answer to the riddle we mentioned at the
beginning about the durability of the Constitution. Even after so many changes in
the society, the Constitution continues to work effectively because of this ability to
http://insightsonindia.com

INSIGHTS

Page 36

Facebook Group: Indian Administrative Service (Raz Kr)

INSIGHTS MOCK TEST 7 SOLUTIONS

be dynamic, to be open to interpretations and the ability to respond to the changing


situation. This is a hallmark of a democratic constitution. In a democracy, practices
and ideas keep evolving over time and the society engages in experiments according
to these. A constitution, which protects democracy and yet allows for evolution of
new practices becomes not only durable but also the object of respect from the
citizens.
Indian constitution, as of now seems to be indestructible because of the strong
democracy and presence of checks and balances.

59. Solution: a)
In the late eighteenth century, Calcutta, Bombay and Madras rose in importance as
Presidency cities. They became the centres of British power in the different regions
of India. At the same time, a host of smaller cities declined. Many towns
manufacturing specialized goods declined due to a drop in the demand for what
they produced. Old trading centres and ports could not survive when the flow of
trade moved to new centres. Similarly, earlier centres of regional power collapsed
when local rulers were defeated by the British and new centres of administration
emerged. This process is often described as de-urbanisation. Cities such as
Machlipatnam, Surat and Seringapatam were de urbanised during the nineteenth
century. By the early twentieth century, only 11 per cent of Indians were living in
cities.

60. Solution: b)
Collective Farming
The basic principal behind this types of farming is based on social ownership of the
means of production and collective labour. Collective farming or the model of
Kolkhoz was introduced in erstwhile Soviet Union to improve upon the
inefficiency of the previous methods of agriculture and to boost agricultural
production for self-sufficiency.

http://insightsonindia.com

INSIGHTS

Page 37

Facebook Group: Indian Administrative Service (Raz Kr)

INSIGHTS MOCK TEST 7 SOLUTIONS

The farmers pool in all their resources like land, livestock and labour. However,
they are allowed to retain very small plots to grow crops in order to meet their
daily requirements.
Yearly targets are set by the government and the produce is also sold to the state at
fixed prices. Produce in excess of the fixed amount is distributed among the
members or sold in the market. The farmers have to pay taxes on the farm
produces, hired machinery etc. Members are paid according to the nature of the
work allotted to them by the farm management. Exceptional work is rewarded in
cash or kind. This type of farming was introduced in former Soviet Union under
the socialist regime which was adopted by the socialist countries. After its collapse,
these have already been modified.

61. Solution: c)
Waterlogging and soil salinity: Irrigation without proper drainage of water leads to
waterlogging in the soil. Besides affecting the crops, waterlogging draws salt to the
surface of the soil. The salt then is deposited as a thin crust on the land surface or
starts collecting at the roots of the plants. This increased salt content is inimical to
the growth of crops and is extremely damaging to agriculture. Waterlogging and soil
salinity are some of the problems that have come in the wake of the Green
Revolution

62. Solution: d)
Commercial grain cultivation is practised in the interior parts of semi-arid lands of
the mid- latitudes. Wheat is the principal crop, though other crops like corn,
barley, oats and rye are also grown. The size of the farm is very large, therefore
entire operations of cultivation from ploughing to harvesting are mechanized. There
is low yield per acre but high yield per person.

http://insightsonindia.com

INSIGHTS

Page 38

Facebook Group: Indian Administrative Service (Raz Kr)

INSIGHTS MOCK TEST 7 SOLUTIONS

63. Solution: a)
Viticulture (from the Latin word for vine) is the science, production, and study
of grapes. It deals with the series of events that occur in the vineyard. When the
grapes are used forwinemaking, it is also known as viniculture. It is a branch of the
science of horticulture.
While the native territory of Vitis vinifera, the common grape vine, ranges
from Western Europe to the Persian shores of the Caspian Sea, the vine has
demonstrated high levels ofadaptability and will sometimes mutate to
accommodate a new environment after its introduction. For this reason, viticulture
can be found on every continent except Antarctica.
Vineyards in India range from the more temperate climate of the northwestern
state ofPunjab down to the southern state of Tamil Nadu. Some of India's larger
wine producing areas are located in Maharashtra, Karnataka
near Bangalore and Andhra Pradesh nearHyderabad. Within the Maharashtra region,
vineyards are found on the Deccan Plateauand around
Baramati, Nashik, Pune, Sangli and Solapur. The high heat and humidity of the far
eastern half of the country limits viticultural activity.
64. Solution: a)
The thickness of the ozone in a column of air from the ground to the top of the
atmosphere is measured in terms of Dobson units (DU).

65. Solution: c)
http://www.nature.com/nature/journal/v344/n6266/abs/344529a0.html

66. Solution: b)
Truck farming, horticultural practice of growing one or more vegetable crops on a
large scale for shipment to distant markets. It is usually less intensive and diversified
than market gardening. At first this type of farming depended entirely on local or
regional markets. As the use of railroads and large-capacity trucks expanded and
refrigerated carriers were introduced, truck farms spread to the cheaper lands of the
West and South, shipping seasonal crops to relatively distant markets where their
http://insightsonindia.com

INSIGHTS

Page 39

Facebook Group: Indian Administrative Service (Raz Kr)

INSIGHTS MOCK TEST 7 SOLUTIONS

cultivation is limited by climate. The major truck-farming areas are in California,


Texas, Florida, along the Atlantic Coastal Plain, and in the Great Lakes area.
Centers for specific crops vary with the season. Among the most important truck
crops are tomatoes, lettuce, melons, beets, broccoli, celery, radishes, onions,
cabbage, and strawberries.

67. Solution: b)
Eutrophication is the natural aging of a lake by biological enrichment of its water.
In a young lake the water is cold and clear, supporting little life. With time, streams
draining into the lake introduce nutrients such as nitrogen and phosphorus, which
encourage the growth of aquatic organisms. As the lakes fertility increases, plant
and animal life burgeons, and organic remains begin to be deposited on the lake
bottom. Over the centuries, as silt and organic debris pile up, the lake grows
shallower and warmer, with warm-water organisms supplanting those that thrive in
a cold environment. Marsh plants take root in the shallows and begin to fill in the
original lake basin. Eventually, the lake gives way to large masses of floating plants
(bog), finally converting into land. Depending on climate, size of the lake and other
factors, the natural aging of a lake may span thousands of years. However,
pollutants from mans activities like effluents from the industries and homes can
radically accelerate the aging process. This phenomenon has been called Cultural or
Accelerated Eutrophication. During the past century, lakes in many parts of the
earth have been severely eutrophied by sewage and agricultural and industrial
wastes. The prime contaminants are nitrates and phosphates, which act as plant
nutrients. They overstimulate the growth of algae, causing unsightly scum and
unpleasant odors, and robbing the water of dissolved oxygen vital to other aquatic
life. At the same time, other pollutants flowing into a lake may poison whole
populations of fish, whose decomposing remains further deplete the waters
dissolved oxygen content. In such fashion, a lake can literally choke to death.
Heated (thermal) wastewaters flowing out of electricity-generating units, e.g.,
thermal power plants, constitute another important category of pollutants. Thermal
wastewater eliminates or reduces the number of organisms sensitive to high
temperature, and may enhance the growth of plants and fish in extremely cold
areas but, only after causing damage to the indigenous flora and fauna.
http://insightsonindia.com

INSIGHTS

Page 40

Facebook Group: Indian Administrative Service (Raz Kr)

INSIGHTS MOCK TEST 7 SOLUTIONS

Negative environmental effects include hypoxia, the depletion of oxygen in the


water, which causes a reduction in specific fish and other animals. Other species
(such as Nomura's jellyfish in Japanese waters) may experience an increase in
population that negatively affects otherspecies.

68. Solution: a)
In the last fifty five years some very critical situations arose in the politics and
constitutional development of the country. We have made a brief reference to
some of these in this chapter already. In terms of constitutional-legal issues, the
most serious question that came up again and again from 1950 was about the
supremacy of the Parliament. In a parliamentary democracy, the
Parliament represents the people and therefore, it is expected to have an upper
hand over both Executive and Judiciary. At the same time, there is the text of the
Constitution and it has given powers to other organs of the government. Therefore,
the supremacy of the
Parliament has to operate within this framework. Democracy is not only about
votes and peoples representation. It is also about the principle of rule of law.
Democracy is also about developing institutions and working through these
institutions. All the political institutions must be responsible to the people and
maintain a balance with each other.
Contribution of the Judiciary
During the controversy between the Judiciary and the Parliament, the Parliament
thought that it had the power and responsibility to make laws (and amendments)
for furthering the interests of the poor, backward and the needy. The Judiciary
insisted that all this has to take place within the framework provided by the
Constitution and pro-people measures should not bypass legal procedures, because,
once you bypass laws even with good intentions, that can give an excuse to the
power holders to use their power arbitrarily. And democracy is as much about
checks on arbitrary use of power as it is about the well-being of the people.
The success of the working of the Indian Constitution lies in resolving these
tensions. The Judiciary, in its famous Kesavananda ruling found a way out of the
http://insightsonindia.com

INSIGHTS

Page 41

Facebook Group: Indian Administrative Service (Raz Kr)

INSIGHTS MOCK TEST 7 SOLUTIONS

existing complications by turning to the spirit of the Constitution rather than its
letter. If you read the Constitution, you will not find any mention of the basic
structure of the Constitution. Nowhere does the Constitution say that such and
such are part of the basic structure. In this sense, the basic structure theory is the
invention of the Judiciary. How did it invent such a non- existent thing? And how
is it that all other institutions have accepted this during the past three decades?
Therein lies the distinction between letter and spirit. The Court came to the
conclusion that in reading a text or document, we must respect the intent behind
that document. A mere text of the law is less important than the social
circumstances and aspirations that have produced that law or document. The Court
was looking at the basic structure as something without which the Constitution
cannot be imagined at all. This is an instance of trying to balance the letter and the
spirit of the Constitution.

69. Solution: a)
The Sikhs
The organisation of the Sikhs into a political community during the seventeenth
century helped in regional state-building in the Punjab. Several battles were fought
by Guru Gobind Singh against the Rajput and Mughal rulers, both before and after
the institution of the Khalsa in 1699. After his death in 1708, the Khalsa rose in
revolt against the Mughal authority under Banda Bahadurs leadership, declared
their sovereign rule by striking coins in the name of Guru Nanak and Guru Gobind
Singh, and established their own administration between the Sutlej and the Jamuna.
Banda Bahadur was captured in 1715 and executed in 1716.
Under a number of able leaders in the eighteenth century, the Sikhs organized
themselves into a number of bands called jathas, and later on misls. Their combined
forces were known as the grand army (dal khalsa). The entire body used to meet at
Amritsar at the time of Baisakhi and Diwali to take collective decisions known as
resolutions of the Guru (gurmatas). A system called rakhi was introduced, offering
protection to cultivators on the payment of a tax of 20 per cent of the produce.
Guru Gobind Singh had inspired the Khalsa with the belief that their destiny was
to rule (raj karega khalsa). Their well-knit organization enabled them to put up a
http://insightsonindia.com

INSIGHTS

Page 42

Facebook Group: Indian Administrative Service (Raz Kr)

INSIGHTS MOCK TEST 7 SOLUTIONS

successful resistance to the Mughal governors first and then to Ahmad Shah Abdali
who had seized the rich province of the Punjab and the Sarkar of Sirhind from the
Mughals. The Khalsa declared their sovereign rule by striking their own coin again
in 1765. Significantly, this coin bore the same inscription as the one on the orders
issued by the Khalsa in the time of Banda Bahadur.
The Sikh territories in the late eighteenth century extended from the Indus to the
Jamuna but they were divided under different rulers. One of them, Maharaja Ranjit
Singh, reunited these groups and established his capital at Lahore in 1799.

70. Solution: a)
A few toxic substances, often present in industrial waste waters, can undergo
biological magnification (Biomagnification) in the aquatic food chain.
Biomagnification refers to increase in concentration of the toxicant at successive
trophic levels. This happens because a toxic substance accumulated by an organism
cannot be metabolised or excreted, and is thus passed on to the next higher trophic
level. This phenomenon is well-known for mercury and DDT. In this manner, the
concentration of DDT is increased at successive trophic levels; say if it starts at
0.003 ppb (ppb = parts per billion) in water, it can ultimately can reach 25 ppm
(ppm = parts per million) in fish-eating birds, through biomagnification. High
concentrations of DDT disturb calcium metabolism in birds, which causes thinning
of eggshell and their premature breaking, eventually causing decline in bird
populations.
Fish and shellfish concentrate mercury in their bodies, often in the form of
methylmercury, a highly toxic organic compound of mercury. Fish products have
been shown to contain varying amounts of heavy metals, particularlymercury and
fat-soluble pollutants from water pollution. Species of fish that arelong-lived and
high on the food chain, such as marlin, tuna, shark, swordfish,king
mackerel, tilefish (Gulf of Mexico), and northern pike contain higher
concentrations of mercury than others.

http://insightsonindia.com

INSIGHTS

Page 43

Facebook Group: Indian Administrative Service (Raz Kr)

INSIGHTS MOCK TEST 7 SOLUTIONS

71. Solution: d)

CFL contain mercury and is an health hazard.


http://en.wikipedia.org/wiki/Compact_fluorescent_lamp#Energy_efficiency

72. Solution: c)
Agricultural technologies While the prevalence of agriculture is indicated by finds
of grain, it is more difficult to reconstruct actual agricultural practices. Were seeds
broadcast (scattered) on ploughed lands?
Representations on seals and terracotta sculpture indicate that the bull was known,
and archaeologists extrapolate from this that oxen were used for ploughing.
Moreover, terracotta models of the plough have been found at sites in
Cholistan and at Banawali (Haryana). Archaeologists have also found evidence of a
ploughed field at Kalibangan (Rajasthan), associated with Early Harappan levels.
The field had two sets of furrows at right angles to each other, suggesting that two
different crops were grown together. Archaeologists have also tried to identify the
tools used for harvesting. Did the Harappans use stone blades set in wooden
handles or did they use metal tools?
Most Harappan sites are located in semi-arid lands, where irrigation was probably
required for agriculture. Traces of canals have been found at the Harappan site of
Shortughai in Afghanistan, but not in Punjab or Sind. It is possible that ancient
canals silted up long ago. It is also likely that water drawn from wells was used for
irrigation. Besides, water reservoirs found in Dholavira (Gujarat) may have been
used to store water for agriculture.

73. Solution: C)
http://en.wikipedia.org/wiki/Bharat_Stage_emission_standards

http://insightsonindia.com

INSIGHTS

Page 44

Facebook Group: Indian Administrative Service (Raz Kr)

INSIGHTS MOCK TEST 7 SOLUTIONS

http://articles.economictimes.indiatimes.com/2014-0324/news/48523863_1_quality-fuel-emission-norms-bs-iv

74. Solution: b)
It includes six systems: Ayurveda, Yoga, Unani, Siddha, Naturopathy and
Homeopathy (AYUSH).
http://en.wikipedia.org/wiki/Siddha_medicine

http://en.wikipedia.org/wiki/Unani

75. Solution: b)
The Constituent Assembly had 300 members in all. Of these, six members played
particularly important roles. Three were representatives of the Congress, namely,
Jawaharlal Nehru, Vallabh Bhai Patel and Rajendra Prasad. It was Nehru who
moved the crucial Objectives Resolution, as well as the resolution proposing that
the National Flag of India be a horizontal tricolour of saffron, white and dark green
in equal proportion, with a wheel in navy blue at the centre. Patel, on the other
hand, worked mostly behind the scenes, playing a key role in the drafting of several
reports, and working to reconcile opposing points of view. Rajendra Prasads role
was as President of the Assembly, where he had to steer the discussion along
constructive lines while making sure all members had a chance to speak.
Besides this Congress trio, a very important member of the Assembly was the
lawyer and economist B.R. Ambedkar. During the period of British rule, Ambedkar
had been a political opponent of the Congress; but, on the advice of Mahatma
Gandhi, he was asked at Independence to join the Union Cabinet as law minister.
In this capacity, he served as Chairman of the Drafting Committee of the
Constitution. Serving with him were two other lawyers, K.M. Munshi from Gujarat
and Alladi Krishnaswamy Aiyar from Madras, both of whom gave crucial inputs in
the drafting of the Constitution.
http://insightsonindia.com

INSIGHTS

Page 45

Facebook Group: Indian Administrative Service (Raz Kr)

INSIGHTS MOCK TEST 7 SOLUTIONS

These six members were given vital assistance b y two civil servants. One was B. N.
Rau, Constitutional Advisor to the Government of India, who prepared a series of
background papers based on a close study of the political systems obtaining in other
countries. The other was the Chief Draughtsman, S. N. Mukherjee, of whom
Ambedkar said that his ability to put the most intricate proposals in the simplest
and clearest legal form can rarely be equalled.

76. Solution: c)
Functions of the Environment: The environment performs four vital functions (i) it
supplies resources: resources here include both renewable and non-renewable
resources. Renewable resources are those which can be used without the possibility
of the resource becoming depleted or exhausted. That is, a continuous supply of
the resource remains available. Examples of renewable resources are the trees in the
forests and the fishes in the ocean. Non-renewable resources, on the other hand, are
those which get exhausted with extraction and use, for example, fossil fuel (ii) it
assimilates waste (iii) it sustains life by providing genetic and bio diversity and (iv)
it also provides aesthetic services like scenery etc.

77. Solution: c)
Judicial activism has had manifold impact on the political system. It has
democratised the judicial system by giving not just to individuals but also groups
access to the courts. It has forced executive accountability. It has also made an
attempt to make the electoral system much more free and fair. The court asked
candidates contesting elections to file affidavits indicating their assets and income
along with educational qualifications so that the people could elect their
representatives based on accurate knowledge.
There is however a negative side to the large number of PILs and the idea of a proactive judiciary. In the first place it has overburdened the courts. Secondly, judicial
activism has blurred the line of distinction between the executive and legislature on
the one hand and the judiciary on the other. The court has been involved in
resolving questions which belong to the executive. Thus, for instance, reducing air
or sound pollution or investigating cases of corruption or bringing about electoral
http://insightsonindia.com

INSIGHTS

Page 46

Facebook Group: Indian Administrative Service (Raz Kr)

INSIGHTS MOCK TEST 7 SOLUTIONS

reform is not exactly the duty of the Judiciary. These are matters to be handled by
the administration under the supervision of the legislatures. Therefore, some people
feel that judicial activism has made the balance among the three organs of
government very delicate. Democratic government is based on each organ of
government respecting the powers and jurisdiction of the others. Judicial activism
may be creating strains on this democratic principle.

78. Solution: a)
Refer page 149, Themes in Indian History, 12th NCERT

79. Solution: b)
The twelfth century witnessed the emergence of a new movement in Karnataka,
led by a Brahmana named Basavanna (1106-68) who was initially a Jaina and a
minister in the court of a Chalukya king. His followers were known as Virashaivas
(heroes of Shiva) or Lingayats (wearers of the linga). Lingayats continue to be an
important community in the region to date. They worship Shiva in his
manifestation as a linga, and men usually wear a small linga in a silver case on a loop
strung over the left shoulder. Those who are revered include the jangama or
wandering monks. Lingayats believe that on death the devotee will be united with
Shiva and will not return to this world. Therefore they do not practise funerary
rites such as cremation, prescribed in the Dharmashastras. Instead, they
ceremonially bury their dead.
The Lingayats challenged the idea of caste and the pollution attributed to certain
groups by Brahmanas. They also questioned the theory of rebirth. These won them
followers amongst those who were marginalised within the Brahmanical social
order. The Lingayats also encouraged certain practices disapproved in the
Dharmashastras, such as post-puberty marriage and the remarriage of widows. Our
understanding of the Virashaiva tradition is derived from vachanas (literally,
sayings) composed in Kannada by women and men who joined the movement.

http://insightsonindia.com

INSIGHTS

Page 47

Facebook Group: Indian Administrative Service (Raz Kr)

INSIGHTS MOCK TEST 7 SOLUTIONS

80. Solution: b)
There are several ways of removing particulate matter; the most widely used of
which is the electrostatic precipitator, which can remove over 99 per cent
particulate matter present in the exhaust from a thermal power plant. It has
electrode wires that are maintained at several thousand volts, which produce a
corona that releases electrons. These electrons attach to dust particles giving them a
net negative charge. The collecting plates are grounded and attract the charged dust
particles. The velocity of air between the plates must be low enough to allow the
dust to fall. A scrubber can remove gases like sulphur dioxide. In a scrubber, the
exhaust is passed through a spray of water or lime. Recently we have realised the
dangers of particulate matter that are very very small and are not removed by these
precipitators. According to Central Pollution Control Board (CPCB), particulate
size 2.5 micrometers or less in diameter (PM 2.5) are responsible for causing the
greatest harm to human health. These fine particulates can be inhaled deep into the
lungs and can cause breathing and respiratory symptoms, irritation, inflammations
and damage to the lungs and premature deaths.

81. Solution: b)
Ex situ Conservation In this approach, threatened animals and plants are taken out
from their natural habitat and placed in special setting where they can be protected
and given special care. Zoological parks, botanical gardens and wildlife safari parks
serve this purpose. There are many animals that have become extinct in the wild
but continue to be maintained in zoological parks. In recent years ex situ
conservation has advanced beyond keeping threatened species in enclosures. Now
gametes of threatened species can be preserved in viable and fertile condition for
long periods using cryopreservation techniques, eggs can be fertilised in vitro, and
plants can be propagated using tissue culture methods. Seeds of different genetic
strains of commercially important plants can be kept for long periods in seed banks.

82. Solution: a)
Groundnut in India
India ranks second in the world in groundnut production. In India, groundnut is
http://insightsonindia.com

INSIGHTS

Page 48

Facebook Group: Indian Administrative Service (Raz Kr)

INSIGHTS MOCK TEST 7 SOLUTIONS

considered as the most important oilseed followed by mustard. In India, groundnut


is mostly grown as a kharif crop, and contributes to around 45 percent of the total
oilseed production in the country. Gujarat, Andhra Pradesh, Tamil Nadu, Rajasthan
and Karnataka are the leading groundnut producing states in India accounting for
around 86 percent of the total domestic output. However, area under groundnut
cultivation in the country has gradually decreased from 6.7 million hectares (mha)
in 2005-06 to an estimated 4.9 mha in 2010-11. Total production of groundnut has
also declined from 8.0 million tonnes in 2005-06 to an estimated 5.6 million tonnes
in 2010-11

83. Solution: b)
The narrowly utilitarian arguments for conserving biodiversity are obvious; humans
derive countless direct economic benefits from nature- food (cereals, pulses, fruits),
firewood, fibre, construction material, industrial products (tannins, lubricants, dyes,
resins, perfumes ) and products of medicinal importance. More than 25 per cent of
the drugs currently sold in the market worldwide are derived from plants and
25,000 species of plants contribute to the traditional medicines used by native
peoples around the world. Nobody knows how many more medicinally useful
plants there are in tropical rain forests waiting to be explored.
With increasing resources put into bioprospecting (exploring molecular, genetic
and species-level diversity for products of economic importance), nations endowed
with rich biodiversity can expect to reap enormous benefits.
The broadly utilitarian argument says that biodiversity plays a major role in many
ecosystem services that nature provides. The fast- dwindling Amazon forest is
estimated to produce, through photosynthesis, 20 per cent of the total oxygen in
the earths atmosphere.
Can we put an economic value on this service by nature? You can get some idea by
finding out how much your neighborhood hospital spends on a cylinder of oxygen.
Pollination (without which plants cannot give us fruits or seeds) is another service,
ecosystems provide through pollinators layer bees, bumblebees, birds and bats.
What will be the costs of accomplishing pollination without help from natural
pollinators? There are other intangible benefits that we derive from naturethe
http://insightsonindia.com

INSIGHTS

Page 49

Facebook Group: Indian Administrative Service (Raz Kr)

INSIGHTS MOCK TEST 7 SOLUTIONS

aesthetic pleasures of walking through thick woods, watching spring flowers in full
bloom or waking up to a bulbuls song in the morning.

84. Solution: d)
Coal: In India, coal is the most abundantly available fossil fuel. It provides a
substantial part of the nations energy needs. It is used for power generation, to
supply energy to industry as well as for domestic needs. India is highly dependent
on coal for meeting its commercial energy requirements.
As you are already aware that coal is formed due the compression of plant material
over millions of years. Coal, therefore, is found in a variety of forms depending on
the degrees of compression and the depth and time of burial. Decaying plants in
swamps produce peat. Which has a low carbon and high moisture contents and low
heating capacity. Lignite is a low grade brown coal, which is soft with high moisture
content. The principal lignite reserves are in Neyveli in Tamil Nadu and are used
for generation of electricity. Coal that has been buried deep and subjected to
increased temperatures is bituminous coal.
It is the most popular coal in commercial use. Metallurgical coal is high grade
bituminous coal which has a special value for smelting iron in blast furnaces.
Anthracite is the highest quality hard coal. In India coal occurs in rock series of two
main geological ages, namely Gondwana, a little over 200 million years in age and in
tertiary deposits which are only about 55 million years old. The major resources of
Gondwana coal, which are metallurgical coal, are located in Damodar valley (West
Bengal-Jharkhand). Jharia, Raniganj, Bokaro are important coalfields. The Godavari,
Mahanadi, Son and Wardha valleys also contain coal deposits. Tertiary coals occur
in the north eastern states of Meghalaya, Assam, Arunachal Pradesh and Nagaland.
Remember coal is a bulky material, which loses weight on use as it is reduced to
ash. Hence, heavy industries and thermal power station are located on or near the
coalfields.

http://insightsonindia.com

INSIGHTS

Page 50

Facebook Group: Indian Administrative Service (Raz Kr)

INSIGHTS MOCK TEST 7 SOLUTIONS

85. Solution: b)
Refer page no. 246, biology class 12th NCERT

86. Solution: d)
Red soil develops on crystalline igneous rocks in areas of low rainfall in the eastern
and southern parts of the Deccan plateau. Yelllow and red soils are also found in
parts of Orissa, Chhattisgarh, southern parts of the middle Ganga plain and along
the piedmont zone of the Western Ghats. These soils develop a reddish colour due
to diffusion of iron in crystalline and metamorphic rocks. It looks yellow when it
occurs in a hydrated form.

87. Solution: c)
Rabi crops are sown in winter from October to December and harvested in
summer from April to June. Some of the important rabi crops are wheat, barley,
peas, gram and mustard. Though, these crops are grown in large parts of India,
states from the north and north- western parts such as Punjab, Haryana, Himachal
Pradesh, Jammu and Kashmir, Uttaranchal and Uttar Pradesh are important for the
production of wheat and other rabi crops. Availability of precipitation during
winter months due to the western temperate cyclones helps in the success of these
crops.
However, the success of the green revolution in Punjab, Haryana, western Uttar
Pradesh and parts of Rajasthan has also been an important factor in the growth of
the above- mentioned rabi crops. Kharif crops are grown with the onset of
monsoon in different parts of the country and these are harvested in SeptemberOctober.
Important crops grown during this season are paddy, maize, jowar, bajra, tur
(arhar), moong, urad, cotton, jute, groundnut and soyabean. Some of the most
important rice-growing regions are Assam, West Bengal, coastal regions of Orissa,
Andhra Pradesh, Tamil Nadu, Kerala and Maharashtra, particularly the (Konkan
coast) along with Uttar Pradesh and Bihar. Recently, paddy has also become an
http://insightsonindia.com

INSIGHTS

Page 51

Facebook Group: Indian Administrative Service (Raz Kr)

INSIGHTS MOCK TEST 7 SOLUTIONS

important crop of Punjab and Haryana. In states like Assam, West Bengal and
Orissa, three crops of paddy are grown in a year. These are Aus, Aman and Boro.
In between the rabi and the kharif seasons, there is a short season during the
summer months known as the Zaid season. Some of the crops produced during
zaid are watermelon, muskmelon, cucumber, vegetables and fodder crops.
Sugarcane takes almost a year to grow.

88. Solution: b)

89. Solution: b)
The Gold Standard: From around 1870 to the outbreak of the First World War in
1914, the prevailing system was the gold standard which was the epitome of the
fixed exchange rate system. All currencies were defined in terms of gold; indeed
some were actually made of gold. Each participant country committed to guarantee
the free convertibility of its currency into gold at a fixed price. This meant that
residents had, at their disposal, a domestic currency which was freely convertible at
a fixed price into another asset (gold) acceptable in international payments. This
also made it possible for each currency to be convertible into all others at a fixed
price. Exchange rates were determined by its worth in terms of gold (where the
currency was made of gold, its actual gold content). For example, if one unit of say
currency A was worth one gram of gold, one unit of currency B was worth two
grams of gold, currency B would be worth twice as much as currency A. Economic
agents could directly convert one unit of currency B into two units of currency A,
without having to first buy gold and then sell it. The rates would fluctuate between
an upper and a lower limit, these limits being set by the costs of melting, shipping
and recoining between the two Currencies 3. To maintain the official parity each
country needed an adequate stock of gold reserves. All countries on the gold
standard had stable exchange rates.
The question arose would not a country lose all its stock of gold if it imported
too much (and had a BoP deficit)? The mercantilist4 explanation was that unless
the state intervened, through tariffs or quotas or subsidies, on exports, a country
would lose its gold and that was considered one of the worst tragedies. David
http://insightsonindia.com

INSIGHTS

Page 52

Facebook Group: Indian Administrative Service (Raz Kr)

INSIGHTS MOCK TEST 7 SOLUTIONS

Hume, a noted philosopher writing in 1752, refuted this view and pointed out that
if the stock of gold went down, all prices and costs would fall commensurately and
no one in the country would be worse off. Also, with cheaper goods at home,
imports would fall and exports rise (it is the real exchange rate which will
determine competitiveness). The country from which we were importing and
making payments in gold would face an increase in prices and costs, so their now
expensive exports would fall and their imports of the first countrys now cheap
goods would go up. The result of this price- specie-flow (precious metals were
referred to as specie in the eighteenth century) mechanism is normally to improve
the BoP of the country losing gold, and worsen that of the country with the
favourable trade balance, until equilibrium in international trade is re-established at
relative prices that keep imports and exports in balance with no further net gold
flow. The equilibrium is stable and self-correcting, requiring no tariffs and state
action. Thus, fixed exchange rates were maintained by an automatic equilibrating
mechanism.

90.Solution: a)
Oxidation and Reduction
In weathering, oxidation means a combination of a mineral with oxygen to form
oxides or hydroxides. Oxidation occurs where there is ready access to the
atmosphere and oxygenated waters. The minerals most commonly involved in this
process are iron, manganese, sulphur etc. In the process of oxidation rock
breakdown occurs due to the disturbance caused by addition of oxygen. Red colour
of iron upon oxidation turns to brown or yellow. When oxidised minerals are
placed in an environment where oxygen is absent, reduction takes place. Such
conditions exist usually below the water table, in areas of stagnant water and
waterlogged ground. Red colour of iron upon reduction turns to greenish or bluish
grey.

91. Solution: b)
Refer page 198, NCERT history class 10th
http://insightsonindia.com

INSIGHTS

Page 53

Facebook Group: Indian Administrative Service (Raz Kr)

INSIGHTS MOCK TEST 7 SOLUTIONS

92. Solution: a)
Cirques are the most common of landforms in glaciated mountains. The cirques
quite often are found at the heads of glacial valleys. The accumulated ice cuts these
cirques while moving down the mountain tops. They are deep, long and wide
troughs or basins with very steep concave to vertically dropping high walls at its
head as well as sides. A lake of water can be seen quite often within the cirques
after the glacier disappears. Such lakes are called cirque or tarn lakes. There can be
two or more cirques one leading into another down below in a stepped sequence.

93. Solution: d)
The stratosphere is found above the tropopause and extends up to a height of 50
km. One important feature of the stratosphere is that it contains the ozone layer.
This layer absorbs ultra-violet radiation and shields life on the earth from intense,
harmful form of energy.

94. Solution: c)
The Khilafat Movement, (1919-1920) was a movement of Indian Muslims, led by
Muhammad Ali and Shaukat Ali, that demanded the following: The Turkish Sultan
or Khalifa must retain control over the Muslim sacred places in the erstwhile
Ottoman empire; the jazirat-ul-Arab (Arabia, Syria, Iraq, Palestine) must remain
under Muslim sovereignty; and the Khalifa must be left with sufficient territory to
enable him to defend the Islamic faith. The Congress supported the movement and
Mahatma Gandhi sought to conjoin it to the Non-cooperation Movement.

95. Solution: b)
Factors Controlling Temperature Distribution The temperature of air at any place
is influenced by (i) the latitude of the place; (ii) the altitude of the place; (iii)
distance from the sea, the air- mass circulation; (iv) the presence of warm and cold
ocean currents; (v) local aspects. The latitude : The temperature of a place depends
http://insightsonindia.com

INSIGHTS

Page 54

Facebook Group: Indian Administrative Service (Raz Kr)

INSIGHTS MOCK TEST 7 SOLUTIONS

on the insolation received. It has been explained earlier that the insolation varies
according to the latitude hence the temperature also varies accordingly.
The altitude : The atmosphere is indirectly heated by terrestrial radiation from
below. Therefore, the places near the sea-level record higher temperature than the
places situated at higher elevations. In other words, the temperature generally
decreases with increasing height. The rate of decrease of temperature with height is
termed as the normal lapse rate. It is 6.5C per 1,000 m.
Distance from the sea : Another factor that influences the temperature is the
location of a place with respect to the sea. Compared to land, the sea gets heated
slowly and loses heat slowly. Land heats up and cools down quickly. Therefore, the
variation in temperature over the sea is less compared to land. The places situated
near the sea come under the moderating influence of the sea and land breezes
which moderate the temperature.
Air-mass and Ocean currents : Like the land and sea breezes, the passage of air
masses also affects the temperature. The places, which come under the influence of
warm air-masses experience higher temperature and the places that come under
the influence of cold air- masses experience low temperature. Similarly the places
located on the coast where the warm ocean currents flow record higher
temperature than the places located on the coast where the cold currents flow.

96. Solution: c)
After the failure of the Cripps Mission, Mahatma Gandhi decided to launch his
third major movement against British rule. This was the Quit India campaign,
which began in August 1942. Although Gandhiji was jailed at once, younger
activists organised strikes and acts of sabotage all over the country. Particularly
active in the underground resistance were socialist members of the Congress, such
as Jayaprakash Narayan. In several districts, such as Satara in the west and
Medinipur in the east, independent governments were proclaimed.
The British responded with much force, yet it took more than a year to suppress
the rebellion. Quit India was genuinely a mass movement, bringing into its ambit
hundreds of thousands of ordinary Indians. It especially energised the young who, in
very large numbers, left their colleges to go to jail. However, while the Congress
http://insightsonindia.com

INSIGHTS

Page 55

Facebook Group: Indian Administrative Service (Raz Kr)

INSIGHTS MOCK TEST 7 SOLUTIONS

leaders languished in jail, Jinnah and his colleagues in the Muslim League worked
patiently at expanding their influence. It was in these years that the League began
to make a mark in the Punjab and Sind, provinces where it had previously had
scarcely any presence.

97. Solution: a)
Continental Shelf
The continental shelf is the extended margin of each continent occupied by
relatively shallow seas and gulfs. It is the shallowest part of the ocean showing an
average gradient of 1 or even less. The shelf typically ends at a very steep slope,
called the shelf break. The width of the continental shelves vary from one ocean to
another. The average width of continental shelves is about 80 km. The shelves are
almost absent or very narrow along some of the margins like the coasts of Chile, the
west coast of Sumatra, etc. On the contrary, the Siberian shelf in the Arctic Ocean,
the largest in the world, stretches to 1,500 km in width. The depth of the shelves
also varies. It may be as shallow as 30 m in some areas while in some areas it is as
deep as 600 m.
The continental shelves are covered with variable thicknesses of sediments brought
down by rivers, glaciers, wind, from the land and distributed by waves and currents.
Massive sedimentary deposits received over a long time by the continental shelves,
become the source of fossil fuels.
Continental Slope
The continental slope connects the continental shelf and the ocean basins. It begins
where the bottom of the continental shelf sharply drops off into a steep slope. The
gradient of the slope region varies between 2-5. The depth of the slope region
varies between 200 and 3,000 m. The slope boundary indicates the end of the
continents. Canyons and trenches are observed in this region.

http://insightsonindia.com

INSIGHTS

Page 56

Facebook Group: Indian Administrative Service (Raz Kr)

INSIGHTS MOCK TEST 7 SOLUTIONS

98. Solution: c)
Guyots
It is a flat topped seamount. They show evidences of gradual subsidence through
stages to become flat topped submerged mountains. It is estimated that more than
10,000 seamounts and guyots exist in the Pacific Ocean alone.

99. Solution: a)
Factors affecting ocean salinity are mentioned below:
(i) The salinity of water in the surface layer of oceans depend mainly on
evaporation and precipitation.
(ii) Surface salinity is greatly influenced in coastal regions by the fresh water flow
from rivers, and in polar regions by the processes of freezing and thawing of ice.
(iii) Wind, also influences salinity of an area by transferring water to other areas.
(iv) The ocean currents contribute to the salinity variations. Salinity, temperature
and density of water are interrelated. Hence, any change in the temperature or
density influences the salinity of an area.

100.

Solution: b)

Separation and isolation of DNA fragments : The cutting of DNA by restriction


endonucleases results in the fragmentes of DNA. These fragments can be separated
by a technique known as gel electrophoresis.
Since DNA fragments are negatively charged molecules they can be separated by
forcing them to move towards the anode under an electric field through a
medium/matrix. Nowadays the most commonly used matrix is agarose which is a
natural polymer extracted from sea weeds.
The DNA fragments separate (resolve) according to their size through sieving
effect provided by the agarose gel. Hence, the smaller the fragment size, the farther
it moves.
http://insightsonindia.com

INSIGHTS

Page 57

Facebook Group: Indian Administrative Service (Raz Kr)

INSIGHTS MOCK TEST 7 SOLUTIONS

http://insightsonindia.com

INSIGHTS

Page 58

Facebook Group: Indian Administrative Service (Raz Kr)

INSIGHTS ON INDIA MOCK PRELIMINARY EXAM - 2015


INSIGHTS ON INDIA MOCK TEST - 8
GENERAL STUDIES

PAPER-I
Time Allowed: 2 Hours

Maximum Marks: 200

INSTRUCTIONS
1. IMMEDITELY AFTER THE COMMENCEMENT OF THE EXAMINATION, YOU SHOULD
CHECK THAT THIS TEST BOOKLET DOES NOT HAVE ANY UNPRINTED OR TORN OR MISSING
PAGES OR ITEMS, ETC. IF SO, GET IT REPLACED BY A COMPLETE TEST BOOKLET.
2. You have to enter your Roll Number on the Test
Booklet in the Box provided alongside. DO NOT
Write anything else on the Test Booklet.
4. This Test Booklet contains 100 items (questions). Each item is printed only in English. Each item
comprises four responses (answers). You will select the response which you want to mark on the
Answer Sheet. In case you feel that there is more than one correct response, mark the response which
you consider the best. In any case, choose ONLY ONE response for each item.
5. You have to mark all your responses ONLY on the separate Answer Sheet provided. See directions in
the Answer Sheet.
6. All items carry equal marks.
7. Before you proceed to mark in the Answer Sheet the response to various items in the Test Booklet, you
have to fill in some particulars in the Answer Sheet as per instructions sent to you with your
Admission Certificate.
8. After you have completed filling in all your responses on the Answer Sheet and the examination has
concluded, you should hand over to the Invigilator only the Answer Sheet. You are permitted to take
away with you the Test Booklet.
9. Sheets for rough work are appended in the Test Booklet at the end.
10. Penalty for wrong answers :
THERE WILL BE PENALTY FOR WRONG ANSWERS MARKED BY A CANDIDATE IN THE
OBJECTIVE TYPE QUESTION PAPERS.
(i) There are four alternatives for the answer to every question. For each question for which a
wrong answer has been given by the candidate, one-third of the marks assigned to that
question will be deducted as penalty.
(ii) If a candidate gives more than one answer, it will be treated as a wrong answer even if one of
the given answers happens to be correct and there will be same penalty as above to that
question.
(iii)

If a question is left blank, i.e., no answer is given by the candidate, there will be no penalty
for that question.
http://insightsonindia.com

INSIGHTS ON INDIA MOCK TEST SERIES FOR CIVIL SERVICES PRELIMINARY EXAM 2015
http://insightsonindia.com

INSIGHTS

Page 1

Facebook Group: Indian Administrative Service (Raz Kr)

1. How will be the invention of Blue LED


significant to mankind?
1. Help reduce energy consumption
2. Help reduce carbon footprints
3. Help treat colour blindness
Choose the correct answer using the codes below:
a)
b)
c)
d)

1 and 2
2 and 3
1 and 3
All of the above

4. Consider the following statements about the


provision of compulsory licensing.
1. It cannot be given by any nation without
the permission of WTO.
2. It involves revoking the patent given to the
patent holder in the larger public interest.
3. Once the patent is revoked, the patent
holder cannot produce or manufacture the
good or service.
Choose the correct answer using the codes below:
a)
b)
c)
d)

1 and 2
2 and 3
1 and 3
None of the above

2. Consider the following statements about


International Thermonuclear Experimental
Reactor (ITER) being constructed presently.
1. It is established for studying the dynamics
of fission reaction in greater scientific
detail.
2. It is predicted that it will consume more
energy than it will intend to produce.
3. It is being constructed in France.

5. The technique of Integrated Pest


Management, apart from agriculture, can be
used in:
1. Horticulture
2. Human habitations
3. Preventive Conservation

Choose the correct answer using the codes below:

Choose the correct answer using the codes below:

a)
b)
c)
d)

1 and 2
2 and 3
1 and 3
3 only

3. Consider the following statements about Mars


Atmosphere and Volatile Evolution Mission
(MAVEN) mission of the U.S.
1. It will explore the surface and atmosphere
features of Mars.
2. It will look into the possibility of the loss of
Martian atmosphere.
3. It is intended to help NASA understand
how life developed on Mars.
Choose the correct answer using the codes below:
a) 1 and 2
b) 2 and 3
c) 1 and 3
d) All of the above
http://insightsonindia.com

a)
b)
c)
d)

1 and 2
2 and 3
1 and 3
All of the above

6. Consider the flowing statements:


1. Humans belong to the primate group
2. Lemurs and lorises do not belong to the
primate group
Which of the above statements is/are correct?
a)
b)
c)
d)

INSIGHTS

1 Only
2 Only
Both
None

Page 2

Facebook Group: Indian Administrative Service (Raz Kr)

7. Trap crops are growing in importance in


raising agricultural productivity. What is its
purpose?
a) Grown to attract insects and pests
b) Grown to trap pollen grains and foreign
particles from entering into the field to
stop cross-breeding of crops
c) Grown to trap moisture and nutrients
and then later release them when
mulched in soil
d) None of the above

3. Salt tolerant species of crops can be


cultivated here.
Choose the correct answer using the codes below:
a)
b)
c)
d)

1 and 2
2 and 3
1 and 3
All of the above

8. Recently Government of India launched the


Pentavalent Vaccine for few diseases. Which
of the following are included in it?
1. Polio
2. Diarrhoea
3. Hepatitis-B

11. Which of the following factors are considered


while declaring drought in a specific area by
the IMD?
1. More than 10% deficient rainfall for a
long period average.
2. More than 20% of agricultural area being
hit.
3. Ground water level going down by more
than 30%.

Choose the correct answer using the codes below:

Choose the correct answer using the codes below:

a)
b)
c)
d)

1 and 2
2 and 3
1 and 3
3 only

9. Which of the following is NOT a pillar of the


Digital India programme launched by the
government recently?
1. Free access to broadband to all by 2020.
2. Cradle to grave digital identity to all
3. Zero imports under Electronics
Manufacturing by 2020.
Choose the correct answer using the codes below:
a)
b)
c)
d)

1 and 2
2 and 3
1 and 3
1 only

a)
b)
c)
d)

12. Consider the following statements about the


wi-fi backscatter technology.
1. It is used to connect any far off devices.
2. It will helpful in making internet of
things.
3. The power requirement of the devices
that will be connected by it will be higher
than normal PCs.
Choose the correct answer using the codes below:
a)
b)
c)
d)

10. Consider the following statements about


Green Shield.
1. It is a tropical littoral ecosystem.
2. It reduces pressure from scarce
pasturelands.
http://insightsonindia.com

1 and 2
2 and 3
1 and 3
All of the above

INSIGHTS

1 and 2
2 and 3
1 and 3
2 only

Page 3

Facebook Group: Indian Administrative Service (Raz Kr)

13. Which of the following evidences suggest an


African origin for hominids?
1. African apes that are most closely related
to hominids
2. Australopithecus, have been found in
East Africa and date back to about 5.6
mya
Choose the correct answer using the codes below
a)
b)
c)
d)

1 Only
2 Only
Both
None

14. Supercomputers that are being built in India


or are presently exisiting can be used for
which of the following purposes?
1. Designing complicated drugs
2. Weather forecasting
3. Genetic analysis
Choose the correct answer using the codes below:
a)
b)
c)
d)

1 and 2
2 and 3
1 and 3
All of the above

15. Consider the following statements about


Intercontinental Ballistic Missiles (ICBM).
1. They can be used to deliver nuclear
weapons.
2. India is yet to develop an ICBM.
3. One ICBM can carry several warheads
that can strike different targets.
Choose the correct answer using the codes below:
a)
b)
c)
d)

1 and 2
2 and 3
1 and 3
All of the above

http://insightsonindia.com

16. The technology of Radio-Frequency


Identification Tag (RFID) can be used in
which of the following to reduce malpractices;
corruption and leakages?
1. Public Distribution System
2. Supply of coal to feeder plants
3. Tracking location of vechiles during
incidents like theft
Choose the correct answer using the codes below:
a)
b)
c)
d)

1 and 2
2 and 3
1 and 3
All of the above

17. Holograms have recently been used


intensively in election campaigns. On what
phenomena is holography based?
1. Interference of light
2. Diffraction of light
3. Rectilinear propagation of light
Choose the correct answer using the codes below:
a)
b)
c)
d)

1 and 2
2 and 3
1 and 3
All of the above

18. Consider the following statements about the


recent Meghraj Initiative of the Government
of India.
1. It is a part of the larger global initiative
Internet of Things
2. It will be used to develop technology that
will improve cloud seeding techniques in
India to achieve custom rainfall.
3. It has been adopted by all the states in
India.
Choose the correct answer using the codes below:
a) 1 and 2
b) 2 and 3
c) 1 and 3
d) None of the above
INSIGHTS

Page 4

Facebook Group: Indian Administrative Service (Raz Kr)

19. Olduvai Gorge or Oldupai Gorge in Tanzania


is one of the most important
paleoanthropological sites in the world. Its
known for
a) The discovery of first known civilization
b) The discovery of Homo habilis specimen
c) The discovery of first human settlements
d) None of the above
20. Consider the following statements about the
Near Field Communication (NFC)
Technology.
1. It is a long-range wireless connectivity
technology.
2. Data transfer rates are very fast as
compared to conventional broadband
internet.
3. The recent Smartchange card initiative
of RBI uses NFC technology.

Choose the correct answer using the codes below:


a)
b)
c)
d)

23. What is common with the following places?


1. Koobi Fora
2. West Turkana
3. Modjokerto
4. Sangiran
Choose the correct answer from below options
a) Discovery of earliest Australopithecus
specimen
b) Discovery of earliest Homo erectus
specimen
c) Discover of earliest Homo habilis
specimen
d) None of the above

Choose the correct answer using the codes below:


a)
b)
c)
d)

1 and 2
2 and 3
1 and 3
3 only

21. Pusa Hydrogel developed in an Indian


Institute is being used for which of the
following purposes?
a) Reducing Irrigation frequency in
agriculture
b) Fighting Climate Change
c) Clinical trials
d) Nano-technology in water purification
22. Consider the following statements about Bioreactors.
1. They can be used in meeting energy
needs of urban areas.
2. It can also be used for tissue growth and
bone grafts.
3. They can be used to provide clean
drinking water to the urban population in
India.
http://insightsonindia.com

1 and 2
2 and 3
1 and 3
All of the above

24. Consider the following statements about


sterilization a method of birth control.
1. Both men and women can be sterilized.
2. It is easily reversible.
3. It can be done through both surgical and
non-surgical methods.
Choose the correct answer using the codes below:
a)
b)
c)
d)

1 and 2
2 and 3
1 and 3
All of the above

25. Which of the following criteria have been laid


out for granting the Special category Status to
the Indian stats by the National Development
Council (NDC)?
1. Hilly and difficult terrain
2. Infrastructural backwardness
3. Bordering strategic international
boundaries

INSIGHTS

Page 5

Facebook Group: Indian Administrative Service (Raz Kr)

Choose the correct answer using the codes below:


a)
b)
c)
d)

1 and 2
2 and 3
1 and 3
All of the above

26. Recently the Production Sharing model (PSM)


and Revenue Sharing model (RSM) with
respect exploration in Oil and Gas sector were
in news. Consider the following statements
about them.
1. In PSM, the private party cannot recover
the costs of his investments.
2. In RSM, regulation and oversight by the
government is less than that in PSM.
3. Recently, Kelkar Panel has favoured the
PSM for deep-sea oil and gas exploration.
Choose the correct answer using the codes below:
a)
b)
c)
d)

1 and 2
2 and 3
1 and 3
All of the above

27. Consider the following statements


1. Monazite is the Thorium bearing mineral
2. As of 2014, in India, Kerala has largest
deposits on in situ monazite
Which of the above statements is/are correct?
a)
b)
c)
d)

1 Only
2 Only
Both
None

28. India is starting the Worlds largest floating


solar power panel project. Consider the
following statements about floating solar
power panels.
Assertion (A): It gives more power generation
yield than conventional ground based solar
power panels.
http://insightsonindia.com

Reason (R): Solar panels installed on land, face


reduction of yield as the ground heats up. In
water this problem is solved.
In the context of the statements above, which of
these is true?
a) A and R both are true, and R is the correct
explanation for A.
b) A and R both are true, and R is the NOT
the correct explanation for A.
c) A is correct, R is incorrect.
d) A and R both are incorrect.
29. Consider the following statements about
Green Seeker a device used in agriculture.
1. It can be used to ascertain the health of
plants.
2. It can also assess the nitrogen
requirements of the soil in the farms.
3. It uses infrared and red light.
Choose the correct answer using the codes
below:
a)
b)
c)
d)

1 and 2
2 and 3
1 and 3
All of the above

30. The Hadza are a small group of hunters and


gatherers, living in the vicinity of Lake Eyasi,
a salt, rift-valley lake, in the country of
a) Kenya
b) Malawi
c) Tanzania
d) Ethiopia
31. Consider the following statements about
IndARC Indian observatory in the Arctic.
1. It is the countrys first underwater
moored observatory.
2. It will collect real- time data on seawater
temperature, salinity, ocean currents etc.

INSIGHTS

Page 6

Facebook Group: Indian Administrative Service (Raz Kr)

3. Its objective is to understand the link


between melting of glaciers and loss of
bio-diversity in the Arctic.
Choose the correct answer using the codes below:
a)
b)
c)
d)

1 and 2
2 and 3
1 and 3
All of the above

32. The technology Sniffer Dog developed by


IISc recently to detect explosives is based on
a) Vapour releasing property of some
explosives
b) Magnetic properties of some explosives
c) Electrostatic properties of some explosives
d) Groundwater polluting tendency of the
underground explosives
33. BICEP2 recently announced the detection of a
distinct signature of cosmic gravitation
waves. In what ways can this discovery be
useful?
1. In understanding the Big Bang.
2. In understanding Cosmic inflation.
3. In giving all scientific information about
dark matter and dark energy.

c) 1 and 3
d) None of the above
35. Consider the following statements
1. The Dutch East India Company was
formed before the British East India
Company
2. The Dutch East India company was the
first to issue stocks in the world
Which of the above statements is/are correct?
a)
b)
c)
d)

36. Which of the following


policies/rules/guidelines/mechanisms are
related to E-waste management in India?
1. Hazardous waste management rules
1989.
2. E-waste management rules 2011
3. "Take back" mechanism
Choose the correct answer using the codes below:
a)
b)
c)
d)

Choose the correct answer using the codes below:


a)
b)
c)
d)

1 and 2
2 and 3
1 and 3
All of the above

34. Consider the following statements about Real


Sector Investment trusts (ReITS).
1. Small investors and institutions can not
invest into it.
2. Only private entities can invest in it.
3. ReITS will be government managed and
controlled.
Choose the correct answer using the codes below:
a) 1 and 2
b) 2 and 3
http://insightsonindia.com

1 Only
2 Only
Both
None

1 and 2
2 and 3
1 and 3
All of the above

37. E-waste has been reported by the Central


Pollution Control Board to be a major issue in
Solid Waste management in India. What can
be the ill-effects of improperly handled Ewaste?
1. Can sterilize the soil
2. Loss of bio-diversity
3. Groundwater Pollution
Choose the correct answer using the codes below:
a) 1 and 2
b) 2 and 3
c) 1 and 3
d) All of the above
INSIGHTS

Page 7

Facebook Group: Indian Administrative Service (Raz Kr)

38. Consider the following statements:


1. Bronze is an alloy of copper and iron
2. The earliest cities in Mesopotamia date
back to the bronze age, c.3000 BCE
Which of the above statements is/are correct?
a)
b)
c)
d)

1 Only
2 Only
Both
None

39. Reverse SEZs have been proposed by Indian


government to boost GDP growth in India.
Consider the following statements about
Reverse SEZs.
1. The industries located in reverse SEZs
will be able to source their raw material
without import duty and process it there
itself.
2. As of now, it will be limited to the
chemicals and fertilizers sector only.
Which of these is/are true?
a)
b)
c)
d)

1 only
2 only
Both 1 and 2
None

40. The Rakesh Mohan Committee on developing


National Transport Development Policy
recently submitted its report. Consider the
following statements.
1. It calls for a single unified Ministry to
deliver multi-modal transport.
2. It calls for National Pipeline Grid to be
established along the lines of the National
Electricity Grid.
3. It recommends privatization of all ports
in India.
Choose the correct answer using the codes below:
a) 1 and 2
b) 2 and 3
c) 1 and 3
d) All of the above
http://insightsonindia.com

41. The year 2014 is the year of Farmer Produce


Organizations (FPOs). Which of the following
organizations can be called as FPOs?
1. SHGs
2. Cooperatives in marketing
3. Farmer Clubs
Choose the correct answer using the codes below:
a) 1 and 2
b) 2 and 3
c) None of the above
d) All of the above
42. With reference to the Mask of Warka or the
Warka Head, a world famous piece of
sculpture, consider the following statements
1. It is a first accurate depiction of human
face on a stone
2. It belongs to the Sumerian culture
Which of the above statements is/are correct?
a) 1 Only
b) 2 Only
c) Both
d) None
43. Asian Infrastructure Investment Bank (AIIB)
was recently established along with 21 other
member countries, including India. Which of
the following countries are NOT parts of it?
1. Japan
2. Australia
3. Indonesia
4. Qatar
5. Pakistan
Choose the correct answer using the codes below:
a) None of the above mentioned countries
are a part of it.
b) All the above mentioned countries are a
part of it.
c) 4 and 5
d) 1, 2 and 3

INSIGHTS

Page 8

Facebook Group: Indian Administrative Service (Raz Kr)

44. Kailash Satyarthi recently won the Nobel


Peace Prize, 2014. He was the leader of the
Bachpan Bachao Andolan (BBA) that has
rescued more than 80,000 children from
exploitation. Which of the following
campaigns were NOT started by the BBA?
1. Child Labour Free India Campaign
2. Right to Education Campaign in 2001
3. Mukti Caravan
Choose the correct answer using the codes below:
a)
b)
c)
d)

1 and 2
2 and 3
All were started by it
All of the above are NOT started by it

45. Consider the following statements


1. Qiyas is the process of deductive
analogy in which the teachings of
the Hadith are compared and contrasted
with those of the Qur'an
2. Hadith are the reports of the teachings,
deeds and sayings of the Islamic
prophet Muhammad
Which of the above statements is/are correct?
a)
b)
c)
d)

1 Only
2 Only
Both
None

46. India and United Kingdom (UK) have


announced a Newton-Bhabha Fund
programme to scale up their
bilateral research & innovation relationship to
a greater level. Which of the following are the
provisions under it?
1. Setting up Centres for research in
Renewable Energy.
2. Centre Partnerships in Antimicrobial
Resistance.
3. Joint PhD student mobility programme.
Choose the correct answer using the codes below:
http://insightsonindia.com

a)
b)
c)
d)

1 and 2
2 and 3
1 and 3
All of the above

47. Union Government has re-launched Kisan


Vikas Patra scheme (KVP) to encourage the
habit of small savings among the citizens.
Consider the following statements about it.
1. There is no upper limit of ceiling on
investment.
2. Point of Contact for availing the scheme
will be local post-office.
3. The funds of the KVP will be utilized in
financing developmental plans of the
Centre and State Governments.
Choose the correct answer using the codes below:
a)
b)
c)
d)

1 and 2
2 and 3
1 and 3
All of the above

48. As per the Union Finance Ministry, some


states and UTs have achieved 100% inclusion
under the PM Jan Dhan Yojana (PMJDY).
Which are they?
1. Kerala
2. Goa
3. Punjab
4. Lakshadweep
5. Puducherry
Choose the correct answer using the codes below:
a)
b)
c)
d)

1 and 2 only
1, 3 and 5 only
2, 3 and 4 only
All except 3

49. Recently the Union cabinet has agreed to sign


SAARC Framework Agreement for Energy
Cooperation. Which of the following
countries are NOT its included in it?
1. Pakistan
INSIGHTS

Page 9

Facebook Group: Indian Administrative Service (Raz Kr)

2. Myanmar
3. Sri Lanka
Choose the correct answer using the codes below:
a)
b)
c)
d)

1 and 2
2 and 3
1 and 3
Only 2

50. Recently a Long Range Surface-to-Air Missile


(LRSAM) co-developed by India and Israel,
was successfully test-fired from a range in
Israel. Consider the following statements
about it.
1. It can counter drones as well as supersonic missiles.
2. It is an air-defence system.
3. It can be deployed on naval ships too.

b) industrial cooperatives were formed


c) trade unions were encouraged
d) small and medium enterprises were
jointly owned by the community
53. The reforms introduced in China in 1978
transformed the whole Chinese economy.
Which of the following were a part of it?
1. Public sector units were made to face
competition.
2. To attract FDI, SEZs were set up.
3. Agricultural lands were allocated to
individual households not for
ownership, only use.
Choose the correct answer using the codes below:
a)
b)
c)
d)

Choose the correct answer using the codes below:


a)
b)
c)
d)

1 and 2
2 and 3
1 and 3
All of the above

51. Which of the following will NOT be valid


argument(s)/reason(s) for the large economic
disparity between India and China?
1. India became independent much later
than China.
2. China opened its economy two decades
before India did.

54. In the normal course of development,


countries first shift their employment and
output from agriculture to manufacturing and
then to services. The trend was NOT followed
in which of these countries?
1. India
2. Pakistan
3. China
Choose the correct answer using the codes below:
a)
b)
c)
d)

Which of these hold correct?


a)
b)
c)
d)

1 only
2 only
Both 1 and 2
None

52. Under the Great Leap Forward Programme of


China, the commune system was developed
under which
a) people collectively cultivated lands
http://insightsonindia.com

1 and 2
2 and 3
1 and 3
All of the above

1 and 2
2 and 3
1 and 3
All of the above

55. Balance of Payments (BOP) is a statistical


statement summarising all the
1. Current Account transactions
2. Capital Account Transactions
3. Receipts and payments in external
transactions
Choose the correct answer using the codes below:
INSIGHTS

Page 10

Facebook Group: Indian Administrative Service (Raz Kr)

a)
b)
c)
d)

1 and 2
2 and 3
1 and 3
All of the above

56. As per the Indian Economic scenario in the


past decade, arrange the following in
increasing order of their magnitude
(generally).
1. Budgetary deficit
2. Fiscal Deficit
3. Revenue deficit
Choose the correct answer using the codes below:
a)
b)
c)
d)

123
132
213
321

57. Under the taxation terminology, the term


cascading effect refers to
a) When tax imposition leads to a
disproportionate rise in prices
b) When tax imposition leads to a
disproportionate fall in tax collection
c) When tax-GDP ratio falls to very low
levels during a recession
d) When tax imposition on one commodity
results in higher tax collection for
complementary commodities
58. Various items enter in the current account of
the balance of payments as invisibles. They
can be:
1. Tourism
2. Airways Travel
3. Remittances
Choose the correct answer using the codes below:
a)
b)
c)
d)

1 and 2
2 and 3
1 and 3
All of the above

http://insightsonindia.com

59. Some banks and financial institutions are


known as Merchant bankers. They
a) Advise companies and do portfolio
management
b) are larger than commercial banks and
mainly engage in large scale real estate
and infrastructure project dealings
c) are international banks dealing mainly in
risky instruments like credit default swap
d) None of the above
60. Government sometimes adopts stabilization
measures in the economy. They are aimed to
control
1. Balance of Payments
2. High rate of Inflation
3. Supply Side bottlenecks in the economy
Choose the correct answer using the codes below:
a)
b)
c)
d)

1 and 2
2 and 3
1 and 3
All of the above

61. Governments sometimes take large policy


measures like Structural Reform Policies.
Which of the following measures can be
included in it?
1. Privatization
2. Deregulation of several areas of the
Economy
3. Focussing more on industries than
agriculture
Choose the correct answer using the codes below:
a)
b)
c)
d)

1 and 2
2 and 3
1 and 3
All of the above

62. The Scheme Vanbandhu Kalyan Yojana


(VKY) aims at
a) The overall welfare of tribals in India
INSIGHTS

Page 11

Facebook Group: Indian Administrative Service (Raz Kr)

b) Increasing the forest cover in ecologically


sensitive regions
c) Implementing wildlife friendly initiatives
in all ecologically sensitive regions
d) None of the above
63. Resources in an economy are distributed
based on the economic theory the nation is
following. In capitalistic theory, which of the
following can be seen?
1. Resources are distributed solely on the
basis of market mechanism.
2. State intervention in the economy is kept
to the least.
3. The amount of goods to be produced is
dictated broadly by the state, and the rest
by the capitalists.
Choose the correct answer using the codes below:
a)
b)
c)
d)

Choose the correct answer using the codes below:


a)
b)
c)
d)

Choose the correct answer using the codes below:


1 and 2
2, 3 and 4
1 and 4 only
All of the above

65. After Independence, several land reform


measures were taken by the government.
Which of the following were part of it?
1. Putting a land ceiling
2. Abolition of intermediaries
http://insightsonindia.com

1, 2 and 3
2, 3 and 4
1 and 4 only
All of the above

66. With reference to agriculture in India,


consider the following statements
1. Agriculture is a Concurrent List subject
2. The agricultural produce market
committee (APMC) act is enacted and
implemented by the union government
Which of the above statements is/are correct?
a)
b)
c)
d)

1 and 2
2 and 3
1 and 3
All of the above

64. Which of the following features can be seen in


a socialistic model of economy?
1. Free health care for citizens
2. Subsidized public transport
3. Complete state control of all strategic
sectors like nuclear, defence etc.
4. State intervention in the market in terms
of regulatory policies etc.

a)
b)
c)
d)

3. Make the tillers the owners of land


4. State purchased private land and
distributed it to the poor

1 Only
2 Only
Both
None

67. Which of the following argument(s) can be


given for cutting or withdrawing or limiting
agricultural subsidies?
1. Subsidy largely benefits the farmers in
the more prosperous regions
2. Subsidies encourage non-judicious use of
natural resources.
3. Subsidies are market distorting.
Choose the correct answer using the codes below:
a)
b)
c)
d)

1 and 2
2 and 3
1 and 3
All of the above

68. Import-substitution has been one of the major


economic agendas of the government. In
which of the following ways can it be
achieved?
1. Attracting FDI in the country.
INSIGHTS

Page 12

Facebook Group: Indian Administrative Service (Raz Kr)

2. Attracting FII in the country.


3. Protectionism
4. Pushing for innovation and
indigenization

1. Public investment in market linkages,


research and extension etc. reduced.
2. Removal of fertilizer subsidy.
3. Reduction in import duties of agricultural
products.
4. Shift from production for the domestic
market towards production for the export
market focusing on cash crops in lieu of
production of food grains.

Choose the correct answer using the codes below:


a)
b)
c)
d)

1, 2 and 3
2, 3 and 4
1 and 4 only
1, 3 and 4

69. Which of the following factors had


contributed to the 1991 economic crisis in
India?
1. Rising prices of essential goods like oil.
2. Government running high fiscal deficit
3. High Current Account Deficit
Choose the correct answer using the codes below:
a)
b)
c)
d)

1 and 2
2 and 3
1 and 3
All of the above

70. After the 1991 economic crisis, the


government of India adopted the New
Economic Policy. Its components were
1. Financial Sector reforms
2. Deregulation of Industrial sector
3. Reduction in tax rates as a part of tax
reforms
4. Rupee was devalued against other
foreign currencies
Choose the correct answer using the codes below:
a)
b)
c)
d)

1, 2 and 3
2, 3 and 4
1 and 4 only
All of the above

71. The New Economic Policy 1991 undertook


some major reforms in agriculture. They were

Choose the correct answer using the codes below:


a)
b)
c)
d)

1, 2 and 3
2, 3 and 4
1 and 4 only
All of the above

72. The Financial sector was subjected to some


major reforms in the LPG policy of 1991.
Which of the following were included in it?
1. Financial sector was allowed to take
decisions on many matters without
consulting
the RBI.
2. Establishment of private sector banks,
Indian as well as foreign.
3. Banks were allowed to generate resources
from abroad with some restrictions.
Choose the correct answer using the codes below:
a)
b)
c)
d)

1 and 2
2 and 3
1 and 3
All of the above

73. In rural areas, which of the following are


predominant causes of poverty?
1. Seasonal unemployment
2. Underemployment
3. Continued fragmentation of small land
holdings
4. Migration to urban areas
Choose the correct answer using the codes below:
a) 1, 2 and 3

http://insightsonindia.com

INSIGHTS

Page 13

Facebook Group: Indian Administrative Service (Raz Kr)

b) 2 and 4 only
c) 1 and 3 only
d) All of the above
74. Which of the following factors make the
estimation of a uniform poverty line for all
citizens difficult?
1. Seasonality of poverty
2. Change in the consumption basket over
time
3. Drawing a line between chronic poverty
and transient poverty
Choose the correct answer using the codes below:
a)
b)
c)
d)

1 and 2
2 and 3
1 and 3
All of the above

75. The recently submitted report of the


Rangarajan panel on poverty estimates bring
about some major changes in the estimation of
poverty from earlier committees. What are
those changes or factors?
1. Household's ability to save
2. Behaviorally determined expenditure for
non-food items
3. Minimum basket separate for urban and
rural areas.
4. Included fats and proteins too in food
items to reflect present dietary habits.
Choose the correct answer using the codes below:
a)
b)
c)
d)

1, 2 and 3
2 and 4 only
1 and 3 only
All of the above

76. The following approaches have been applied


by the Government of India to alleviate
poverty. These approaches have changed and
evolved over the year in various five year
plans.
1. Growth trickle-down approach
http://insightsonindia.com

2. Expanding self-employment programmes


and wage employment programmes.
3. Providing minimum basic amenities to
people
Arrange them in the correct chronological order.
a)
b)
c)
d)

123
213
312
321

77. Investment in which of the following directly


leads to the formation of human capital of a
country?
1. Roads, power, transmission lines
2. Primary education
3. On-job training
4. Encouraging freedom of thought and
expression
Choose the correct answer using the codes below:
a)
b)
c)
d)

1, 2 and 3
2 and 4 only
1 and 3 only
2, 3 and 4 only

78. India wants to become a knowledge economy.


What does it mean?
a) Major share of GDP will come from highly
educated and skilled graduates (and
above)
b) Major share of GDP will come from the
information intensive sectors like IT,
information analysis etc.
c) Utilizing e-governance to its fullest
capacity for nations economic prosperity
d) All of the above
79. In which of the following sectors of education,
government spending is highest per capita
and overall (total) respectively?
a) Tertiary and Tertiary
b) Primary and tertiary
c) Tertiary and Primary
INSIGHTS

Page 14

Facebook Group: Indian Administrative Service (Raz Kr)

d) Secondary and Primary


80. Which of the following institutions exist till
date in the rural banking institutional
framework?
1. Commercial banks
2. Rural cooperatives
3. Regional Rural banks (RRBs)
4. Land Development banks
Choose the correct answer using the codes below:
a)
b)
c)
d)

1, 2 and 3
2 and 4 only
1 and 3 only
All of the above

81. Consider the following statements about


micro-credit.
1. It may consist of small loans without any
collateral.
2. SHGs can avail micro-credit both from
within and from outside credit
institutions.
3. No interest rate is charged at microcredit.
Choose the correct answer using the codes below:
a)
b)
c)
d)

1 and 2 only
2 and 3 only
1 and 3 only
All of the above

82. Consider the following statements about the


fishing sector in India.
1. Fishing from inland sources is more than
from the ocean.
2. Gujarat, Kerala and Maharashtra are
leading producers of fisheries.
3. Operation Flood was connected with
boosting fisheries production in India.
Choose the correct answer using the codes below:
a) 1 and 2 only
http://insightsonindia.com

b) 2 and 3 only
c) 1 and 3 only
d) All of the above
83. Which of the following benefits accrue on
following organic farming rather than
chemical farming?
1. Higher nutritional value of crops
2. Less labour input
3. Generates more income through exports
4. Organic inputs are cheaper leading to
better return on investment.
Choose the correct answer using the codes below:
a)
b)
c)
d)

1, 2 and 3
2 and 4 only
1 and 3 only
1, 3 and 4

84. Consider the following statements about the


horticulture sector in India.
1. It includes the production of medicinal
plants and aromatic species too.
2. Golden revolution is associated with
increase in productivity in horticulture.
But, India is yet to see Golden revolution.
3. Hybrid seed production and tissue
culture increase yield in horticulture.
Choose the correct answer using the codes below:
a)
b)
c)
d)

1 and 2 only
2 and 3 only
1 and 3 only
All of the above

85. Indian economy suffers with casualization of


workforce. It means that
a) A large number of labourers work in the
unorganised sector
b) A large number of labourers do not get a
regular salary
c) A large number of workers do not get any
social security benefit
d) All of the above
INSIGHTS

Page 15

Facebook Group: Indian Administrative Service (Raz Kr)

86. Which of the following would be included in


the informal sector of the Indian economy?
1. Farmers and agricultural labourers
2. Self-employed people
3. Owners of Small enterprises
Choose the correct answer using the codes below:
a)
b)
c)
d)

1 and 2 only
2 and 3 only
1 and 3 only
All of the above

87. Which factors may contribute to disguised


unemployment in India?
1. High population
2. Majority of farmers with small land
holding size
3. Seasonal nature of agricultural work

89. Fa Xian traveled in India around 399 CE.


Which kingdom did he describe about in his
writings?
a) Gupta
b) Nanda
c) Maurya
d) Shishunaga
90. Consider the following statements
1. Ancient Nalanda flourished under the
patronage of Mauryas
2. The Tang Dynasty Chinese
pilgrim Faxian left detailed accounts of
the school
Which of the above statements is/are correct?
a)
b)
c)
d)

Choose the correct answer using the codes below:


a)
b)
c)
d)

1 and 2 only
2 and 3 only
1 and 3 only
All of the above

88. Consider the following statements


1. The Sufis sought to renounce the world
and rely on God alone
2. The Sufis sought to achieve unity with
God through an intense love (ishq) for
God
3. Sufis used musical concerts (sama) to
induce ecstasy and stimulate emotions of
love and passion.
Which of the above statements is/are
INCORRECT?
a)
b)
c)
d)

1 and 2 Only
2 and 3 Only
1 and 3 Only
None

http://insightsonindia.com

1 Only
2 Only
Both
None

91. The Geniza records were


a) Collection of Arabic poetry
b) Collection of Jewish manuscripts
c) Collection of Arab translation of Sanskrit
works
d) None of the above
92. Zahiruddin Babur, the founder of Mughal
empire in India was a descendant of
a) Timur
b) Genghis Khan
c) Both Timur and Genghis Khan
d) None of the two
93. On 12 October 1492, Columbus sighted land
and he thought it was India. But in reality he
had reached
a) Philippines
b) Bahamas
c) Cuba
d) None of the above

INSIGHTS

Page 16

Facebook Group: Indian Administrative Service (Raz Kr)

94. Consider the following statements:


1. Aryabhata correctly insisted that the earth
rotates about its axis daily
2. Aryabhata described a geocentric model
of the solar system
3. Solar and lunar eclipses were scientifically
explained by Aryabhata
Which of the above statements is/are correct?
a)
b)
c)
d)

1 and 2 Only
2 and 3 Only
1 and 3 Only
All

95. Pedro Alvares Cabral who, who in 1500 sailed


from Portugal to find a route to India, is
credited as the first European to reach
a) South Africa
b) Brazil
c) Saudi Arabia
d) Madagascar
96. With reference to the Meiji Restoration in
Japan, consider the following statements
1. The Meiji Restoration refers to
establishment of practical powers and
consolidation of the political system
under the Emperor of Japan
2. The Meiji Restoration helped Japan
advance industrially
Which of the above statements is/are correct?
a)
b)
c)
d)

1 Only
2 Only
Both
None

c) It has a range of 3000 kms


d) None of the above
98. The newly launched Varishta Pension Bima
Yojana (VPBY) scheme is aimed at
a) Senior citizens aged 60 years and above
b) Senior citizens aged between 50 years to
60 years
c) Workers working in unorganized sector
d) Senior workers working in unorganized
sector
99. With reference to Chaudhary Ranbir Singh,
whose birth centenary were held recently,
consider the following statements
1. He was a Member of the Constituent
Assembly of India
2. Due to his initiative, freedom fighters
were given pension by the Government
Which of the above statements is/are correct?
a)
b)
c)
d)

100.
With reference to the Opium Trade in 19th
century, consider the following statements
1. Britain and other European nations
undertook the opium trade because of
their chronic trade imbalance with China.
2. The East India Company directly traded
opium with the Chinese rulers and it
bought silk and tea with the profits
Which of the above statements is/are correct?
a)
b)
c)
d)

97. With reference to Indias Pragati' tactical


missile, which of the following statements is
NOT correct?
a) It is a surface to surface missile
b) It is a variant of Prahaar missile meant for
export to friendly countries
http://insightsonindia.com

1 Only
2 Only
Both
None

INSIGHTS

1 Only
2 Only
Both
None

Page 17

Facebook Group: Indian Administrative Service (Raz Kr)

http://insightsonindia.com

INSIGHTS

Page 18

Facebook Group: Indian Administrative Service (Raz Kr)

INSIGHTS MOCK TESTS 2015 TEST 8 SOLUTIONS

1. Solution: a)
http://www.popsci.com/article/technology/why-blue-led-worth-nobel-prize
2. Solution: d)
ITER is an experimental Fusion Reactor being constructed presently in France for future
production of electricity from fusion energy. ITER is expected to produce at least ten times
more energy than the energy required to operate it. In future demo or commercial reactors
based on fusion, this energy can be converted to electricity. Nuclear energy based on fusion
may end energy crisis of the world and would revolutionize the energy scenario.
3. Solution: b)
MAVEN is the first spacecraft that will focus primarily on the state of the upper atmosphere,
the processes that control it, and the overall atmospheric loss that is currently occurring.
Specifically, MAVEN will explore the processes through which the top of the Martian
atmosphere can be lost to space. Scientists think that this loss could be important in
explaining the changes in the climate of Mars that have occurred over the last four billion
years.
MAVEN's data is specialized and of interest to smaller groups of researchers, but the project
fits one of NASA's "Big Questions" - specifically understanding how life developed on Earth.
Studying the atmosphere of Mars should provide insights to researchers interested in the
evolution of Earth's atmosphere.
4. Solution: d)

Compulsory licensing is when a government allows someone else to produce the


patented product or process without the consent of the patent owner. It is one of the
flexibilities on patent protection included in the WTOs agreement on intellectual
property the TRIPS (Trade-Related Aspects of Intellectual Property Rights)
Agreement.

The mechanism of CL is essentially about balancing patent rights with access to


medicine. The words social and economic welfare, public health, national
emergency and public health problems/crises used in the Act are all pointers to
the CL provisions being centred around access to medicine.

5. Solution: d)

Integrated Pest Management (IPM) is an ecosystem approach to crop production and


protection that combines different management strategies and practices to grow
healthy crops and minimize the use of pesticides.

Integrated pest management (IPM) is a broad based approach that integrates a range
of practices for control of pests.

http://insightsonindia.com

INSIGHTS

Page 1

Facebook Group: Indian Administrative Service (Raz Kr)

INSIGHTS MOCK TESTS 2015 TEST 8 SOLUTIONS


Significance

IPM programs use current, comprehensive information on the life cycles of pests and
their interaction with the environment.

This information, in combination with available pest control methods, is used to


manage pest damage by the most economical means, and with the least possible
hazard to people, property, and the environment.

IPM is used in agriculture, horticulture, human habitations, and preventive


conservation.

Globalization and increased mobility open allow increasing numbers of invasive


species to cross national borders.IPM poses the least risks while maximizing benefits
and reducing costs.

6. Solution: a)
Primate, in zoology, any mammal of the group that includes
the lemurs, lorises, tarsiers, monkeys, apes, and humans. The order Primates, with its 300 or
more species, is the third most diverse order of mammals, after rodents (Rodentia)
and bats (Chiroptera). Although there are some notable variations between some primate
groups, they share several anatomic and functional characteristics reflective of their common
ancestry. When compared with body weight, the primate brain is larger than that of other
terrestrial mammals, and it has a fissure unique to primates (the Calcarine sulcus) that
separates the first and second visual areas on each side of the brain. Whereas all other
mammals have claws or hooves on their digits, only primates have flat nails. Some primates
do have claws, but even among these there is a flat nailon the big toe (hallux). In all primates
except humans, the hallux diverges from the other toes and together with them forms a
pincer capable of grasping objects such as branches. Not all primates have similarly dextrous
hands; only the catarrhines (Old World monkeys, apes, and humans) and a few of the
lemurs and lorises have an opposable thumb.
7. Solution: a)
Trap crops are grown to attract insects or other organisms to protect main crops from pest
attack. Protection may be achieved either by preventing the pests from reaching the crop or
by concentrating them in certain parts of the field where they can be destroyed.
The principle of trap cropping rests on the fact that virtually all pests show a distinct
preference to a certain crop stage.
They are used in Integrated Pest Management.
8. Solution: d)
Pentavalent vaccine was launched recently under the National Immunisation Programme.
This one single vaccine shot will provide protection to infants from five life-threatening
http://insightsonindia.com

INSIGHTS

Page 2

Facebook Group: Indian Administrative Service (Raz Kr)

INSIGHTS MOCK TESTS 2015 TEST 8 SOLUTIONS


ailments, including Diphtheria, Pertussis, Tetanus (DPT), Hepatitis B and Hib (Haemophilus
influenza type b).

This is the most significant step in entire vaccination history in India.

It would ensure that all children are being provided with immunization for all five
lethal diseases in single vaccination and that too more conveniently.

9. Solution: d)

Digital India is a Programme to prepare India for a knowledge future.

It is an Umbrella Programme covering many departments.


It weaves together a large number of ideas and thoughts into a single,
comprehensive vision so that each of them is seen as part of a larger goal.
Each individual element stands on its own. But is also part of the larger
picture.
It is coordinated by DeitY, implemented by the entire government.
The weaving together makes the Mission transformative in totality

10. Solution: d)
What is green shield?

Mangroves are called green shield as they buffer the coastline against sea erosion
and devastating effects of cyclones and tsunami.

Since estuarine areas are highly populated areas, the slightest ecological imbalance
will take a heavy toll. They play a vital role in stabilizing these areas.

Importance of Mangroves:

Mangroves are buffers between the land and the sea. Coastlines throughout the
world are facing serious problems of coastal erosion and threat of rising sea levels
due to global warming has increased the threats by several folds. To control such

http://insightsonindia.com

INSIGHTS

Page 3

Facebook Group: Indian Administrative Service (Raz Kr)

INSIGHTS MOCK TESTS 2015 TEST 8 SOLUTIONS


assault of the sea on land the nature has provided what is called as Mangroves, a
tropical littoral ecosystem which is more dynamic than the sea itself.

Mangroves not only help in preventing soil erosion but also act as a catalyst in
reclaiming land from seas. This is a very unique phenomenon, since there is a
general tendency of water to engulf land.

Mangrove forests and estuaries are the breeding and nursery grounds for a number
of marine organisms including the commercially important shrimp, crab and fish
species. Hence, loss of mangroves not only affects us indirectly but there are direct
economic repercussions through loss of fishing industry.

In many coastal areas including Gulf of Kutch, mangroves are a substitute for fodder.
Thus mangroves reduce pressures from the scarce pasturelands.

Above all, Mangroves are now looked after by scientists as saviors in the today's
scenario of global warming. We all know that most of the coastal areas throughout
the world are going to be affected by sea level rise due to global warming. The effects
of which are already visible. Therefore, when most of the coastal areas will be
flooded, mangroves can possibly provide a gene bank for cultivating salt tolerant
species of crops that could be our future resource.

11. Solution: a)

Drought declaration has various socio-economic impacts in India. Indian


Meteorological Department (IMD) is entrusted with the task of drought declaration.
Despite of having a number of socio-economic fallouts, however, it is still a purely
geo-technical decision in India and IMD is the only authority in this regard.
IMD defines a drought year as one with more than 10% deficient rainfall for a long
period average and more than 20% of agricultural area being hit.
Meteorological Drought: Occurs when the seasonal rainfall is deficient more than
10% for long period average.
Moderate Drought: Rainfall deficit is in the range 25-50 %
Severe Drought: Rainfall deficit exceeds 50% of the normal rainfall.
However, for policy making purposes, only following two types of droughts are
concerned.
Agricultural Drought: Agricultural drought is the most significant drought category
about which policymakers and farmers concerned most. When a period of four
consecutive weeks with rainfall deficiency more than 50% or weekly rainfall less
than 5mm from mid-May to mid-October, a period when more than 80% of Kharif
crops are sown, are occurred, agricultural drought year is declared by IMD.
Hydrological Drought: it is defined as deficiencies in surface and sub-surface water
supplies leading to lack of water for normal and specific water needs.

12. Solution: d)
Wi-Fi backsetter technology is a novel invention in the field of Information Technology
(IT) that enables battery free devices to connect a network through existing wi-fi signals.
http://insightsonindia.com

INSIGHTS

Page 4

Facebook Group: Indian Administrative Service (Raz Kr)

INSIGHTS MOCK TESTS 2015 TEST 8 SOLUTIONS


It essentially solves the power problem for battery less devices regarding to connecting a
network.
Backsetter Wi-Fi technique has been seen as the first revolutionary step towards
realization of Internet of Things (IoT). This technique will not only be helpful to take
off the network of things but will broaden the base of such networks, as the billions
battery-less devices could also be connected to a network through a optional source of
power. This is indeed very first of its kind.
13. Solution: c)
Two lines of evidence suggest an African origin for hominids. First, it is the group of African
apes that are most closely related to hominids. Second, the earliest hominid fossils, which
belong to the genus Australopithecus, have been found in East Africa and date back to about
5.6 mya. In contrast, fossils found outside Africa are no older than 1.8 million years.
14. Solution: d)
A supercomputer is a computer with great speed and memory. This kind of computer can
do jobs faster than any other computer of its generation. They are usually thousands of times
faster than ordinary personal computers made at that time. Supercomputers can
do arithmetic jobs very fast, so they are used for weather forecasting, code-breaking, genetic
analysis and other jobs that need many calculations.

India has always strived to be technologically advanced in all aspects. Building of


supercomputers is no exception. PARAM, PARAMYUVA-2 etc are some of the
examples of supercomputers built by India for various purposes, like -

Precise weather forecasting, especially in times where frequent occurrence of natural


hazards is devastating.

Tapping of natural resources, like the KG basin sea fields.

Designing of complicated drugs.

Quantum mechanics and modeling etc

15. Solution: c)
http://en.wikipedia.org/wiki/Intercontinental_ballistic_missile
See introduction Only.
About Indias ICBM Programme

The Indian Ballistic Missile Defence Programme is an initiative to develop and


deploy a multi-layered ballistic missile defense system to protect India from ballistic
missile attacks.

http://insightsonindia.com

INSIGHTS

Page 5

Facebook Group: Indian Administrative Service (Raz Kr)

INSIGHTS MOCK TESTS 2015 TEST 8 SOLUTIONS

Introduced in light of the ballistic missile threat from Pakistan and China, it is a
double-tiered system consisting of two interceptor missiles, namely the Prithvi Air
Defence (PAD) missile for high altitude interception, and the Advanced Air Defence
(AAD) Missile for lower altitude interception.

16. Solution: d)
RFID (radio frequency identification) has likewise been around for decades. However, RFID
tends to require more technological hand-holding. RFID involves applying RFID tags to
items or boxes or pallets. Tags vary greatly in size, shape and capabilities, but one example
is pictured below. The tag with its small antenna emits a radio frequency signal that is
picked up and read by a special wireless RFID reader, conveying information from the tag
about the item it is affixed to.
Uses

RFID is adaptable to many of the same uses that barcodes are good for. But RFID is
especially useful in situations where vast quantities of goods must be moved or
tracked, or where tracking of item-specific information is necessary.

RFID has been mandated by some customers, such as Wal-Mart and the Department
of Defense, to track the vast quantities of items they require in their supply chains
and to supply much more detailed information. In such situations, RFID may be able
to do it more quickly, effectively and efficiently than barcodes.

RFID tags will enable the Motor Vehicles Department and the other law-enforcers to
easily trace the location of a vehicle, its expected time of arrival, locate vehicles
during incidents like theft and to enable hassle-free movement through the check
posts.

17. Solution: a)
A hologram is a physical structure that diffracts light into an image. The term hologram
can refer to both the encoded material and the resulting image.
A holographic image can be seen by looking into an illuminated holographic print or by
shining a laser through a hologram and projecting the image onto a screen.
Holography is a two stage process. The first stage is recording of hologram in the form of
interference pattern and in the second stage, the hologram acts as a diffraction grating for
the reconstruction beam and the image of the object is reconstructed for the hologram.
To understand the process check the sub-heading How holography works
http://holocenter.org/what-is-holography
18. Solution: d)

http://insightsonindia.com

INSIGHTS

Page 6

Facebook Group: Indian Administrative Service (Raz Kr)

INSIGHTS MOCK TESTS 2015 TEST 8 SOLUTIONS


Meghraj is the name given to the initiative of Government of India for its new program
which is going to take advantage of the Cloud Computing. Meghraj is just a name coined for
the purpose (Megh=Cloud, Raj=Rule i.e. Rule of Cloud Computing)
Another name for Meghraj is the GI Cloud Initiative.

It will enable the government to leverage cloud computing for effective delivery of eservices.

Components of Governments Cloud Computing:

Five essential characteristics (viz. on-demand self service, ubiquitous network access,
metered use, elasticity and resource pooling)

Three service models (infrastructure as a service, platform as a service and software


as a service)

Four deployment models (public cloud, private cloud, community cloud and hybrid
cloud)

19. Solution: b)
Olduvai Gorge or Oldupai Gorge in Tanzania is one of the most important
paleoanthropological sites in the world and has been instrumental in furthering the
understanding of early human evolution. This site was occupied by Homo habilis
approximately 1.9 million years ago, Paranthropus boisei 1.8 million years ago, and Homo
erectus 1.2 million years ago. Homo sapiens is dated to have occupied the site 17,000 years
ago. Olduvai Gorge is a steep-sided ravine in the Great Rift Valley that stretches through
eastern Africa. It is in the eastern Serengeti Plains in Arusha Region, Tanzania and is about
48 km (30 mi) long. It is located 45 km (28 mi) from the Laetoli archaeological site.
This site is significant in showing increased developmental and social complexities in
hominins. Evidence of this is shown in the production and use of stone tools, which
indicates the increase in cognitive capacities. Evidence also indicates the practices of both
scavenging and hunting, which are highlighted by the evidence of gnaw marks predating
cut marks, and comparisons on percentages of meat versus plant in the early hominid diet.
Furthermore, the collection of tools and animal remains in a central area is evidence of
increases in social interaction and communal activity.
20. Solution: d)

Near Field Communications (NFC) is a short-range wireless connectivity technology


that provides intuitive, simple and safe communication between electronic devices.
Communication occurs when two NFC-compatible devices are brought within four
centimetres of each other.

NFC operates at 13.56 Mhz and transfers data at up to 424 Kbits/ second. Because
the transmission range is short, the transactions are inherently secure.

http://insightsonindia.com

INSIGHTS

Page 7

Facebook Group: Indian Administrative Service (Raz Kr)

INSIGHTS MOCK TESTS 2015 TEST 8 SOLUTIONS

NFC is an upgrade of the existing proximity card standard (RFID) that combines the
interface of a smartcard and a reader into a single device. It allows users to
seamlessly share content between digital devices, pay bills wirelessly or even use
their cellphone as an electronic traveling ticket on existing contactless infrastructure
already in use for public transportation.

In India too, an NFC-enabled payment App was announced at the launch of


Blackberrys new Z10 mobile phone in February. PVR Cinemas launched what is
arguably the first commercial NFC-enabled payment App for the Blackberry10
platform that enables Z10 users to pay for their tickets across 15 of its theatre
complexes after topping up their Apps with their credit card accounts

Recently SBI and Mother Dairy launched SmartChange Card that works on Near
Field Communication.

E-commerce and E-payment systems in India are gaining ground with the
proliferation of NFC networks.

21. Solution: a)

Pusa Hydrogel is a product for increasing agricultural productivity primarily


through improvement in use efficiency of water. The technology includes bench scale
process for its production along with the concomitant use package under diverse
agricultural situations.

It is a granular product developed by IARI Delhi currently promoted by Ministry of


Agriculture. During irrigation the gel absorbs water like a sponge and during dry
periods releases it to the soil for plants.

Significance

Effective in soil for a minimum period of one crop season

Improves physical properties of soils and the soil less media

Improves seed germination and the rate of seedling emergence

Reduces leaching of agro-inputs such as herbicides and fertilizers

Reduces irrigation and fertigation requirements of crops

Reduces nursery establishment period

Helps plants withstand extended moisture stress

22. Solution: a)

A bioreactor may refer to any manufactured or engineered device or system that


supports a biologically active environment. In one case, a bioreactor is a vessel in
which a chemical process is carried out which

http://insightsonindia.com

INSIGHTS

Page 8

Facebook Group: Indian Administrative Service (Raz Kr)

INSIGHTS MOCK TESTS 2015 TEST 8 SOLUTIONS


involves organisms or biochemically active substances derived from such organisms.
This process can either be aerobic or anaerobic. These bioreactors are commonly
cylindrical, ranging in size from litres to cubic metres, and are often made of stainless
steel.

There are a number of types of bioreactors, and they are used for a variety of
purposes, from processing solid waste to manufacturing pharmaceuticals.

Advantages

Bioreactors are also used to promote growth, as for example in the production of
tissue cultures, or the cultivation of specific fungi utilized in pharmaceuticals.

Bioreactors could potentially be used to produce energy, or to grow tissue and bone
grafts.

Conversion of organic waste such as compost or solid waste is a common application


for bioreactors.

Chemical production can rely heavily on bioreactors, depending on the compounds


being manufactured, as can large-scale processing of compost and yard waste for
municipalities.

Recently scientists have developed a new kind of photobioreactor called as


Environmental Photobioreactor [ePBR]

The system produces oil [alternative energy source] by using algae.

23. Solution: b)
Homo is a Latin word, meaning man, although there were women as well! Scientists
distinguish amongst several types of Homo. The names assigned to these species are derived
from what are regarded as their typical characteristics. So fossils are classified as Homo
habilis (the tool maker), Homo erectus (the upright man), and Homo sapiens (the wise or
thinking man). Fossils of Homo habilis have been discovered at Omo in Ethiopia and at
Olduvai Gorge in Tanzania.
The earliest fossils of Homo erectus have been found both in Africa and Asia: Koobi Fora,
and west Turkana, Kenya, Modjokerto and Sangiran, Java. As the finds in Asia belong to a
later date than those in Africa, it is likely that hominids migrated from East Africa to
southern and northern Africa, to southern and north-eastern Asia, and perhaps to Europe,
some time between 2 and 1.5 mya. This species survived for nearly a million years.

24. Solution: c)
As many as 12 women died because of rat poisoning when state run sterilization program
failed in Chattisgarh. This raises a social issue where, as part of family planning program,
http://insightsonindia.com

INSIGHTS

Page 9

Facebook Group: Indian Administrative Service (Raz Kr)

INSIGHTS MOCK TESTS 2015 TEST 8 SOLUTIONS


India is resorting to forced sterilization method that is mostly targeted against poor women
barring men, because sterilization of men is a socially unacceptable thing in India.
Read the Gist only http://en.wikipedia.org/wiki/Sterilization_%28medicine%29
25. Solution: d)
There are five criterias for the Special Category States

Terrain = Difficult/Hilly

Population density = Low or tribal population= High

National Security due to strategic concern Like J& K / bordering strategic


international borders

Economic & Infrastructural Backwardness

Financial Situation= Non viable

Till now, eleven states have been awarded SCS and six are demanding it as Bihar.

Granted by National Development Council

Why given?

The main reason behind this categorization is the development of that particular
state where there are many problems due to hilly terrains, international borders and
strategic interests. Overall social and economic development cannot be done just as
other states.

What are the benefits?

Tax benefits, Corporate Tax Waiver, Excise Duty Waiver

90% funding for all Centrally sponsored schemes

Devolved funds under National Central assistance

26. Solution: b)
Developing the oil and gas sector needs huge investment. Therefore, the government invites
private companies under PPP to invest in oil and gas blocks along with providing
technology; and reap benefits.
There are two models that are followed worldwide a. Production-sharing model
b. Revenue Sharing Model

http://insightsonindia.com

INSIGHTS

Page 10

Facebook Group: Indian Administrative Service (Raz Kr)

INSIGHTS MOCK TESTS 2015 TEST 8 SOLUTIONS

India has recently shifted from a to b model on the recommendations of the


Rangarajan panel in New gas and exploration policy.

Production Sharing model

As per the model, OEGCs will first develop oil and gas fields. They will then first
recover their investment and operating expenditure. Then they will share profits
with the government.

Revenue Sharing model

OGECs will develop the oil and gas fields.

But the revenue will be shared between the government and OGECs from the first
batch of production of the oil and gas itself (irrespective of cost recovery or not). It
will be on the basis of level of output in the block, not on the level of investment.

Positives of the model

More transparent and less intervention of CAG and the government.

OEGCs will not be able to hoard or tamper with the production of oil and gas. So
cost recovery will be easier.

If the price of oil or gas increases, it will lead to a gain for both the OEGCs and the
government.

Therefore, Kelkar committee was of the view that the RSC should be followed only
for shallow and on-land blocks which have less risk. For deep-sea explorations, PSC
should be followed.

27. Solution: a)
http://pib.nic.in/newsite/PrintRelease.aspx?relid=112034
28. Solution: a)
Indias leading hydro power generator National Hydro Power Corporation (NHPC) is
planning to set up a 50 MW solar photovoltaic project over the water bodies in the southern
state of Kerala.
Beneficial due to:

Saves land
More yield
Saves water

The ecology of the water body is not likely to be affected much and it will also reduce
evaporation, thus helping preserve water levels during extreme summer. Solar panels

http://insightsonindia.com

INSIGHTS

Page 11

Facebook Group: Indian Administrative Service (Raz Kr)

INSIGHTS MOCK TESTS 2015 TEST 8 SOLUTIONS


installed on land, face reduction of yield as the ground heats up. When such panels are
installed on a floating platform, the heating problem is solved to a great extent.

29. Solution: d)
GreenSeeker

is a handheld gadget that works as a crop sensor.


The sensor emits brief bursts of red and infrared light.
By measuring the amount of each type of light that is reflected back from the plant,
the device can calculate and display the health of the crop.
Also, the farmer can use it to assess how much nitrogen the soil needs in each section
of land.

30. Solution: c)
The Hadza, or Hadzabe, are an ethnic group in north-central Tanzania, living around Lake
Eyasi in the central Rift Valley and in the neighboring Serengeti Plateau. The Hadza number
just under 1,000. Some 300400 Hadza live as hunter-gatherers, much as their ancestors have
for thousands or even tens of thousands of years; they are the last full-time hunter-gatherers
in Africa.
31. Solution: a)
IndARC

Scientists predict that melting of the Arctic glaciers will trigger changes in weather
patterns and ocean currents that could affect other parts of the world. The interaction
between the Arctic ice shelf and the deep sea and its influence on climate shift requires
detailed studies over an annual seasonal cycle.

Therefore, IndARC, the countrys first underwater moored observatory, has been
deployed in the Kongsfjorden fjord, half way between Norway and the North Pole.

It will collect real- time data on seawater temperature, salinity, ocean currents and other
vital parameters of the fjord.

So, it is expected that it will help scientists understand the Arctic climate process and its
influence on the Indian monsoon system.

It represents a major milestone in Indias scientific endeavours in the Arctic region.

32. Solution: a)
IISc has developed a molecular "sniffer dog" to detect explosives.

They have created a highly sensitive fluorescent polymer that scouts out a class of
commonly used explosives.

http://insightsonindia.com

INSIGHTS

Page 12

Facebook Group: Indian Administrative Service (Raz Kr)

INSIGHTS MOCK TESTS 2015 TEST 8 SOLUTIONS

TNT and other nitroaromatic compounds, which are used in a cocktail of chemicals in
landmines and plastic explosive devices, release vapours that quench the polymers,
reducing their glow.

While the drop in fluorescence is not visible to the naked eye, it is visually sensed and
interpreted by a high resolution spectrometer.

33. Solution: a)
Cosmic inflation refers to the idea that the cosmos experienced an exponential growth spurt
in its first trillionth, of a trillionth of a trillionth of a second. This theory holds that this
would have taken the infant universe from something unimaginably small to something
about the size of a marble. Space has continued to expand for the nearly 14 billion years
since.
Inflation was first proposed in the early 1980s to explain some aspects of the Big Bang
Theory that appeared to not quite add up, such as why deep space looks broadly the same
on all sides of the sky.
The theory came with a very specific prediction - that it would be associated with waves of
gravitational energy, and that these ripples in the fabric of space would leave an indelible
mark on the oldest light in the sky - the famous Cosmic Microwave Background.
The theory is in news because of the recent findings of BICEP2, which announced the
detection of a distinct signature of cosmic gravitational waves, possibly originating during
an explosive phase of expansion dubbed inflation, occurring at the earliest moments after
the big bang.
Discovery of gravitational waves, it is significant for two reasons.
First, this opens up a whole new way of studying the Universe, allowing scientists to infer
the processes at work that produced the waves.
Second, it proves a hypothesis called inflation. This can be used to give us information
about the origin of the universe, known as the big bang.
Gravitation waves are nothing but the waves that carry energy across the Universe.
34. Solution: d)
REITS:

Just as mutual funds do with equity and debt, REITs will pool money from investors
and invest them in income-generating (rental assets) and infrastructure offering them
a way to diversify their portfolios by investing in property.

http://insightsonindia.com

INSIGHTS

Page 13

Facebook Group: Indian Administrative Service (Raz Kr)

INSIGHTS MOCK TESTS 2015 TEST 8 SOLUTIONS

This will help establish a new asset class, and being a quasi debt-equity instrument,
be attractive for risk-averse investors get the twin benefits of yield as well as capital
appreciation.

For developers, it would improve property market transparency, smoothen volatile


property cycles, and potentially lower the cost of capital.

For small investors and institutions, REITs provides an opportunity to invest in


largescale commercial real estate projects which would have otherwise been only
possible for HNIs and wealthy individuals.

Further, tax concessions ensure that dividend payouts are healthy and less impacted
by changes in central tax laws.

35. Solution: b)
The Dutch East India Company was a chartered company established in 1602, when
the States General of the Netherlands granted it a 21-year monopoly to carry out trade
activities in Asia. It is often considered to have been the first multinational corporation in the
world and it was the first company to issue stock. It was a powerful company, possessing
quasi-governmental powers, including the ability to wage war, imprison and execute
convicts, negotiate treaties, strike its own coins, and establish colonies.
36. Solution: d)
Regulation of E-waste in India

Under the "Hazardous waste management rules 1989."

Article 243W - allows state legislatures to make laws on waste management - based
on this Municipal Solid Waste rules framed.

E-waste management rules 2011 - extended producer responsibility; registration of ewaste recyclers - checking parameters; technology; safe handling

Introduced "take back" mechanism for companies to collect e-waste from consumers.
The rationale is the recycling of e-waste.

37. Solution: d)
E-waste related Issues

Some studies estimate that annually 4 lakh tonnes of e-waste is generated in india.

The issue is not just the volume of wastes, but more their toxicity as they contain
mercury, lead, arsenic, cadmium and a blend of plastics that are difficult to remove
from the environment. Plus handling them. - nontoxic; carcinogenic; mutogenic;
corrosive; genotoxic effects.

Problem increased due to improper recycling

http://insightsonindia.com

INSIGHTS

Page 14

Facebook Group: Indian Administrative Service (Raz Kr)

INSIGHTS MOCK TESTS 2015 TEST 8 SOLUTIONS

The present capacity in India can handle only 10% of the e-waste produced today.
Mostly done by unorganised sector. e.g. Mayapuri incident Delhi - Cobalt 60
radiation

SPCBs do not have mechanisms and capacity to tackle the problem of increasing ewaste. Lack of Integration among agencies.

Leads to Groundwater pollution; esterification of soil; steriles the soil - impotent; biodiversity; damage to heart liver and splee; astahmtic bronchtis by bio-accumulation

38. Solution: b)
The earliest cities in Mesopotamia date back to the bronze age, c.3000 BCE. Bronze is an
alloy of copper and tin. Using bronze meant procuring these metals, often from great
distances. Metal tools were necessary for accurate carpentry, drilling beads, carving stone
seals, cutting shell for inlaid furniture, etc. Mesopotamian weapons were also of bronze.

39. Solution: c)
Reverse SEZs
The proposed reverse SEZs basically involve setting up industries in places where feedstock
(like for chemical industries) is available rather than importing it to India for e.g. Iran and
Myanmar.
It is based on the idea of SEZs only. But the difference is that it will be located abroad.
Benefits to India

As of now, it will be limited to the chemicals and fertilizers sector only.

Already, India is a leading player in the chemicals and fertiliser sector. The vision is
to make it amongst the largest and best to create employment and boost GDP growth
in India.

The industries located in reverse SEZs will be able to source their raw material
without import duty etc. and process it there itself. So, from this the Indian chemical
industry can source its material in a cost-effective manner.

The government has already set up specially delineated investment regions in the
form of PCPIRs (petroleum, chemicals & petrochemicals investment regions) at
Dahej, Gujarat, Visakhapatnam and Kakinada in Andhra Pradesh, Paradip in Orissa
and Cuddalore and Nagapattinam in Tamil Nadu.

It will also reduce pollution in India.

It will strengthen economic and diplomatic ties with the nations where reverse SEZs
will be established.

http://insightsonindia.com

INSIGHTS

Page 15

Facebook Group: Indian Administrative Service (Raz Kr)

INSIGHTS MOCK TESTS 2015 TEST 8 SOLUTIONS


40. Solution: a)
Following are some of the findings and recommendations contained in the Report of the
National Transport Development Policy Committee

Integrated development of ports, roads, rails etc - Integrated Transport Strategy Road, rail, port, aviation, NE transport, Urban transport
Optimal Model Mix
Develop Small ports - with 5-6 mega ports
Corporatization of Mega port authorities
Focus deep Inland waterways development, especially in NE
Expand PMGSY universally
Provide safety and traffic management as recommended by Sundar committee (road
committee)
International transport communication for NE region
Increase investment in railways 0.8% of GDP in the 12th Plan
Capacity expansion for both freight and passenger traffic
Present thinking is project centric - it should be integrated, developing human
resources; people-centric; with emphasis on NE region.
National level office for transport strategy
With respect to the movement of liquids and gases via pipeline, a National Pipeline
Grid could be established along the lines of the National Electricity Grid
Statutorily and financially empowered Metropolitan Urban Transport Authorities
(MUTAs) to be established at the metropolitan city level.
Bharat IV fuel quality standard should be implemented nationwide by the middle of
this decade, with a target to reach Bharat VI by 2020.
Single unified Ministry to deliver multi-modal transport
Independent regulatory authority with functional and financial autonomy
Legal structure in transport sector must be simplified

41. Solution: d)
FPOs cover organizations from lower tiers of formal cooperative structures in credit,
marketing, dairy and fishery, extending to self-help groups (SHGs), farmer clubs, joint
liability groups (JLGs) and, more recently, to producer companies.
Role in agriculture and poverty alleviation

Small and marginal farmers face several problems:

shrinking land holdings

Getting formal credit;

Difficulty in accessing critical inputs for agriculture such as quality seeds and timely
technical assistance.

http://insightsonindia.com

INSIGHTS

Page 16

Facebook Group: Indian Administrative Service (Raz Kr)

INSIGHTS MOCK TESTS 2015 TEST 8 SOLUTIONS


FPOs can help them in the following way:

It is a congregation of small and marginal farmers willing to pool resources and


share risks.

FPOs leave land titles with individual producers and use the strength of collective
planning for production, procurement and marketing.

It adds value to members produce through pooled resources of land and labour,
shared storage space, transportation and marketing facilities.

These improve bargaining power of small farmers.

Most importantly, transacting with a bank in a group like FPO can reduce
transactions costs of banks and buyers to deal them. Moreover, the group acts like
collateral for its members. So its easier to get a loan.

42. Solution: c)

43. Solution: d)
The Asian Infrastructure Investment Bank (AIIB) is an international financial
institution proposed by China. The purpose of the multilateral development bank is to
provide finance to infrastructure projects in the Asia Pacific region. AIIB is regarded by
some as a rival for the IMF, the World Bank and the Asian Development Bank (ADB), which
the AIIB says are dominated by developed countries like the United States and Japan.

http://insightsonindia.com

INSIGHTS

Page 17

Facebook Group: Indian Administrative Service (Raz Kr)

INSIGHTS MOCK TESTS 2015 TEST 8 SOLUTIONS


http://en.wikipedia.org/wiki/Asian_Infrastructure_Investment_Bank#Member_countries
Recently the US pressurized its close allies like Japan, Indonesia and Australia not to join the
AIIB. Owing to it, these nations stayed out of the membership.
44. Solution: c)
Bachpan Bachao Andolan
Success

The organization has rescued more than 80,000 children from bondage, trafficking
and exploitative labour

The BBA's Child Friendly Village program (in Hindi, Bal Mitra Gram, or BMG), has
been accepted as a best practice model for development and elimination of child
labour and trafficking.

It has established a immediate rehabilitation centre for rescued children - Mukti


Ashram in Delhi.

It has fought successfully several PILs in the SC.

Recent Campaigns

Child Labour Free India Campaign for total abolition on child labour till the age of 14
yrs

Right to Education Campaign in 2001

Child Domestic Labour campaign

Mukti Caravan (campaign against child trafficking for forced labour)

Missing Children Campaign: biggest ever research undertaken on missing children,


resulting in Supreme Court issuing notice to all states and union territories on
missing children

45. Solution: c)
In Islamic jurisprudence, qiys is the process of deductive analogy in which the teachings of
the Hadith are compared and contrasted with those of the Qur'an, in order to apply a
known injunction (nass) to a new circumstance and create a new injunction. Here the ruling
of the Sunnah and the Qur'an may be used as a means to solve or provide a response to a
new problem that may arise.
Hadith in Muslim religious use is often translated as prophetic 'traditions', meaning the
corpus of the reports of the teachings, deeds and sayings of the Islamic prophet Muhammad.
The hadith literature was compiled from oral reports that were in circulation in society
around the time of their compilation long after the death of Muhammad. Bukhari's
http://insightsonindia.com

INSIGHTS

Page 18

Facebook Group: Indian Administrative Service (Raz Kr)

INSIGHTS MOCK TESTS 2015 TEST 8 SOLUTIONS


collection is considered the most reliable by many traditional religious scholars who are
Sunnis. The Shi'as believe in an entirely separate body of Hadith.
46. Solution: d)
The Newton-Bhabha Fund is a programme which aims to build up strong interdisciplinary
dimension that will further strategise the science, research and innovation cooperation
between the UK and India. For this programme UK will provide 50 million pounds over 5
years period and India will provide similar matched efforts.
Key Features: Newton-Bhabha programme

Joint PhD student mobility programme: It aims to pave the way for the next
generation of researchers in the UK and India to interact with each other and build
long-term R&D linkages and collaborations. This mobility programme will be
administered by British Council India and the Department of Science and
Technology. It will start from March 2015 to sponsor UK and Indian PhD students to
spend a period of their study (3 to 6 months) in Premier Indian and UK
higher education institutions.

New research projects in Mental Health and Substance Abuse: These new projects
will be joint initiative of Medical Research Council (MRC) of UK and Indian Council
of Medical Research (ICMR). ICMR will fund the Indian component for this project
while UK will provide up to 2 million pounds.

Setting up Centres for research in Renewable Energy: These centres will be jointly
co-funded by RCUK Energy programme and Department of Science and Technology
(DST) of India. UK and the Department of Science & Technology of India will
provide support to business-led research and development aimed at tackling societal
challenges.

Centre Partnerships in Cancer Biology, Translational Regenerative Medicines in


Neuroscience and Antimicrobial Resistance: It will be funded jointly up to 7 million
pounds by Medical Research Council (MRC) of UK and Department
of Biotechnology (DBT), India.

47. Solution: d)
The previous KVP was discontinued in 2011. The scheme was very popular among the
investors and the percentage share of gross collections secured in KVP after its launch in
1988 was in the range of 9-29 per cent against the total collections received under
all National Savings Schemes in the country.
Key Features of KVP

Liquidity: Kisan Vikas Patra scheme has unique liquidity feature, where an investor
can encash his certificates after the lock-in period of 2 years and 6 months and
thereafter in any block of six months on pre-determined maturity value.

http://insightsonindia.com

INSIGHTS

Page 19

Facebook Group: Indian Administrative Service (Raz Kr)

INSIGHTS MOCK TESTS 2015 TEST 8 SOLUTIONS

Investment feature: The KVP will be available to the investors in the denomination
of 1000, 5000, 10000 and 50000 rupees with no upper ceiling on investment. The
scheme will provide facility of unlimited investment by way of purchase of
certificate from post office in various denominations.

KVP certificates: The certificates can also be issued in single or joint names and can
be transferred from one person to any other person. The investment made in the
certificate will double in 100 months. The facility of transfer from one post office to
another anywhere in India and of nomination will also be available under the
scheme.

Maturity period: With a maturity period of 8 years 4 months, the collections under
the scheme will be available with the Union Government for a fairly long period to
be utilized in financing developmental plans of the Centre and State Governments.

48. Solution: d)
Finance Ministry has declared States of Kerala and Goa, union territories of Chandigarh,
Puducherry and Lakshadweep and three districts of Gujarat- Porbandar, Mehasana,
Gandhinagar as 100 per cent saturated in terms of coverage of all households with at least
one bank account.
According to Finance Ministry

PMJDY so far managed to bring over 5000 crore rupees into the
normal banking system as account holders have started depositing cash. A large
chunk of this money would have been kept at home in the absence of accounts, with
little or no productive use.

According to the initial results, on an average about 80% of the households surveyed
have now got at least one bank account.

49. Solution: d)
In 2000, the process of regional cooperation in energy sector began between SAARC Member
States with the establishment of a Technical Committee on Energy.
Key facts of SAARC Framework Agreement for Energy Cooperation (Electricity)

It will enable greater cooperation in the power sector among SAARC countries.

It is expected to improve the power availability in the entire SAARC region.

It would facilitate integrated operation of the regional power grid among member
States.

Implications of this framework

http://insightsonindia.com

INSIGHTS

Page 20

Facebook Group: Indian Administrative Service (Raz Kr)

INSIGHTS MOCK TESTS 2015 TEST 8 SOLUTIONS

This agreement will boost the implementation of common power grid between
SAARC members. Thus help them to share the hydroelectric power generated in
North East India to Bangladesh, Nepal and other countries.

Excess electricity from India and Pakistan can be supplied to electricity


deficient Afghanistan.

Offshore wind projects could be set up in Sri Lankas coastal borders to power
southern part of India and Sri Lanka.

50. Solution: d)
LRSAM is called Barak 8 missile in Israel. Barak is the Hebrew word for Lightning.
It is designed to counter a wide variety of air-borne threats such as anti-ship missiles,
aircraft, UAVs and drones as well as supersonic cruise missiles.
Barak 8 is an advanced, long-range missile defence and air defence system with main
features being:

Long Range

Active Radar Seeker Missile

Vertical Launch

Multiple Simultaneous Engagements.

Importance of LRSAM

LRSAM will be deployed on the newly commissioned INS Kolkata, which had to be
inducted without the weapon system due to delays in testing.

It would also be deployed on other naval ships including recently commissioned INS
Kamrota.

The successful test marked a milestone in bilateral cooperation of the two countries in
developing advanced weapon systems.
51. Solution: a)
Argument 1
India, Pakistan and China have many similarities in their developmental strategies. All the
three nations have started towards their developmental path at the same time.
While India and Pakistan became independent nations in 1947, Peoples Republic of China
was established in 1949.
So Argument 1 is NOT valid.

http://insightsonindia.com

INSIGHTS

Page 21

Facebook Group: Indian Administrative Service (Raz Kr)

INSIGHTS MOCK TESTS 2015 TEST 8 SOLUTIONS


Argument 2
India and Pakistan adopted similar strategies such as creating a large public sector and
raising public expenditure on social development.
Till the 1980s, all the three countries had similar growth rates and per capita incomes.
However, china liberalised in 1979 and India did in 1991. From the data in the Economic
survey, it is found that presently India is at the same positions as China was two decades
ago. So the lag correlates with the liberalisation gap. So Argument 2 can possibly explain the
disparity.
52. Solution: a)
After the establishment of Peoples Republic of China under one party rule, all the critical
sectors of the economy, enterprises and lands owned and operated by individuals were
brought under government control.
The Great Leap Forward (GLF) campaign initiated in 1958 aimed at industrialising the
country on a massive scale. People were encouraged to set up industries in their backyards.
In rural areas, communes were started. Under the Commune system, people collectively
cultivated lands. In 1958, there were 26,000 communes covering almost all the farm
population.
53. Solution: d)
The present-day fast industrial growth in China can be traced back to the reforms
introduced in 1978.
China introduced reforms in phases. In the initial phase, reforms were initiated in
agriculture, foreign trade and investment sectors.
In agriculture, for instance, commune lands were divided into small plots which were
allocated (for use not ownership) to individual households. They were allowed to keep all
income from the land after paying stipulated taxes. In the later phase, reforms were initiated
in the industrial sector. Private sector firms, in general, and township and village
enterprises, i.e. those enterprises which were owned and operated by local collectives, in
particular, were allowed to produce goods. At this stage, enterprises owned by government
(known as State Owned EnterprisesSOEs), which we, in India, call public sector
enterprises, were made to face competition.
The reform process also involved dual pricing. This means fixing the prices in two ways;
farmers and industrial units were required to buy and sell fixed quantities of inputs and
outputs on the basis of prices fixed by the government and the rest were purchased and sold
at market prices. Over the years, as production increased, the proportion of goods or inputs
transacted in the market also increased. In order to attract foreign investors, special
economic zones were set up.

http://insightsonindia.com

INSIGHTS

Page 22

Facebook Group: Indian Administrative Service (Raz Kr)

INSIGHTS MOCK TESTS 2015 TEST 8 SOLUTIONS


54. Solution: a)
The proportion of workforce engaged in manufacturing in India and Pakistan were low at 19
and 20 per cent respectively. The contribution of industries to GDP is also just equal to or
marginally higher than the output from agriculture. In India and Pakistan, the shift is taking
place directly to the service sector.
Thus, in both India and Pakistan, the service sector is emerging as a major player of
development. It contributes more to GDP and, at the same time, emerges as a prospective
employer.

55. Solution: d)
Balance of Payments (BOP) : It is a statistical statement summarising all the external
transactions (receipts and payments) on current and capital account in which a country is
involved over a period of time, say, a year. As the BOP shows the total assets and
obligations over a time-period, it always balances.
56. Solution: b)
The fiscal deficit is the difference between the government's total expenditure and its total
receipts (excluding borrowing).
The elements of the fiscal deficit are (a) the revenue deficit, which is the difference between
the governments current (or revenue) expenditure and total current receipts (that is,
excluding borrowing) and (b) capital expenditure. The fiscal deficit can be financed by
borrowing from the Reserve Bank of India (which is also called deficit financing or money
creation) and market borrowing (from the money market, that is mainly from banks).
Budget deficit: A financial situation that occurs when an entity has more money going out
than coming in. The term "budget deficit" is most commonly used to refer to government
spending rather than business or individual spending. When it refers to federal government
spending, a budget deficit is also known as the "national debt." The opposite of a budget
deficit is a budget surplus, and when inflows are equal to outflows, the budget is said to be
balanced.
57. Solution: a)

http://insightsonindia.com

INSIGHTS

Page 23

Facebook Group: Indian Administrative Service (Raz Kr)

INSIGHTS MOCK TESTS 2015 TEST 8 SOLUTIONS


Cascading Effect : When tax imposition leads to a disproportionate rise in prices, i.e. by an
extent more than the rise in the tax, it is known as cascading effect.
http://www.youthkiawaaz.com/2012/03/the-cascading-effect-of-taxation-what-weunderstand-by-it/
http://www.thehindu.com/business/Economy/gst-will-remove-cascading-effect-of-leviespranab/article3446952.ece
58. Solution: d)
Invisibles : Various items enter in the current account of the balance of payments, some of
which are not visible goods. Invisibles are mainly services, like tourism, transport by
shipping or by airways, and financial services such as insurance and banking. They also
include gifts sent abroad or received from abroad and private transfer of funds, government
grants and interests, profits and dividends.
59. Solution: a)
Merchant Bankers : Banks or financial institutions, also known as investment bankers, that
specialise in advising the companies and managing their equity and debt requirement (often
referred to as portfolio management) through floatation and sale/purchase of stocks and
bonds.
60. Solution: d)
A macroeconomic strategy enacted by governments and central banks to keep economic
growth stable, along with price levels and unemployment. Ongoing stabilization policy
includes monitoring the business cycle and adjusting benchmark interest rates to control
aggregate demand in the economy. The goal is to avoid erratic changes in total output, as
measured by Gross Domestic Product (GDP) and large changes in inflation; stabilization of
these factors generally leads to moderate changes in the employment rate as well.
In essence, Fiscal and monetary measures adopted to control fluctuations in the balance of
payments and high rate of inflation come under stabilization meaasures.

61. Solution: a)
Long-term measures like liberalisation, deregulation and privatisation aimed to improve the
efficiency and competitiveness of the economy are termed as Structural Reform Policies.
For e.g. the 1991 reforms were one of the largest such measures undertaken in the economy.
Even the new government is taking a large number of structural reform measures like
opening up sectors e.g. FDI in defence; FDI in insurance to 49% etc.
62. Solution: a)

http://insightsonindia.com

INSIGHTS

Page 24

Facebook Group: Indian Administrative Service (Raz Kr)

INSIGHTS MOCK TESTS 2015 TEST 8 SOLUTIONS


http://pib.nic.in/newsite/PrintRelease.aspx?relid=110870
63. Solution: a)
In a market economy, also called capitalism, only those consumer goods will be produced
that are in demand, i.e., goods that can be sold profitably either in the domestic or in the
foreign markets. If cars are in demand, cars will be produced and if bicycles are in demand,
bicycles will be produced. If labour is cheaper than capital, more labour-intensive methods
of production will be used and vice-versa In a capitalist society the goods produced are
distributed among people not on the basis of what people need but on the basis of
Purchasing Powerthe ability to buy goods and services.
64. Solution: d)
In a socialist society the government decides what goods are to be produced in accordance
with the needs of society. It is assumed that the government knows what is good for the
people of the country and so the desires of individual consumers are not given much
importance. The government decides how goods are to be produced and how they should
be distributed. In principle, distribution under socialism is supposed to be based on what
people need and not on what they can afford to purchase.
Unlike under capitalism, for example, a socialist nation provides free health care to all its
citizens. Strictly, a socialist society has no private property since everything is owned by the
state. In Cuba and China, for example, most of the economic activities are governed by the
socialistic principles.
65. Solution: a)
At the time of independence ownership of land was concentrated in the hands of a few. This
led to the exploitation of the farmers and was a major hindrance towards the socio-economic
development of the rural population. Equal distribution of land was therefore an area of
focus of Independent India's government, and land reforms were seen as an important pillar
of a strong and prosperous country.
It did not include state purchase and distribution of land. Instead, the surplus land (above
ceiling) was taken from the big landlords and distributed to the poor. It has a limited
success.
Also refer to http://js2012.wordpress.com/why-jansatyagraha-2012/land-reforms-in-india/
66. Solution: d)
Agriculture is a state subject.
APMC act is implemented by the state governments.
http://www.niticentral.com/2014/09/06/apmc-act-amendment-freedom-frommiddlemen-for-farmers-237442.html

http://insightsonindia.com

INSIGHTS

Page 25

Facebook Group: Indian Administrative Service (Raz Kr)

INSIGHTS MOCK TESTS 2015 TEST 8 SOLUTIONS


67. Solution: d)
Some economists point out that subsidies do not allow prices to indicate the supply of a
good. When electricity and water are provided at a subsidised rate or free, they will be used
wastefully without any concern for their scarcity. Farmers will cultivate water intensive
crops if water is supplied free, although the water resources in that region may be scarce and
such crops will further deplete the already scarce resources. If water is priced to reflect
scarcity, farmers will cultivate crops suitable to the region. Fertiliser and pesticide subsidies
result in overuse of resources which can be harmful to the environment.
68. Solution: d)
Import substitution is aimed at replacing or substituting imports with domestic production.
For example, instead of importing vehicles made in a foreign country, industries would be
encouraged to produce them in India itself. In this policy the government protected the
domestic industries from foreign competition. Protection from imports took two forms:
tariffs and quotas. Tariffs are a tax on imported goods; they make imported goods more
expensive and discourage their use. Quotas specify the quantity of goods which can be
imported. The effect of tariffs and quotas is that they restrict imports and, therefore, protect
the domestic firms from foreign competition.
The policy of protection is based on the notion that industries of developing countries are
not in a position to compete against the goods produced by more developed economies. It is
assumed that if the domestic industries are protected they will learn to compete in the
course of time. Our planners also feared the possibility of foreign exchange being spent on
import of luxury goods if no restrictions were placed on imports. Nor was any serious
thought given to promote exports until the mid-1980s.
69. Solution: d)
In 1991, India met with an economic crisis relating to its external debt the government
was not able to make repayments on its borrowings from abroad; foreign exchange
reserves, which we generally maintain to import petrol and other important items, dropped
to levels that were not sufficient for even a fortnight.
Check the BACKGROUND section in Chapter 3 Indian Economy NCERT 11th.
70. Solution: d)
These reforms were part of the Liberalisation policy undertaken in 1991. For a detailed
discussion on these reforms, refer to Section 3.3 Liberalisation 11th NCERT Indian Economy
71. Solution: d)
Reforms in Agriculture: Reforms have not been able to benefit agriculture, where the
growth rate has been decelerating (till the 10th FYP).
Public investment in agriculture sector especially in infrastructure, which includes
irrigation, power, roads, market linkages and research and extension (which played a crucial
http://insightsonindia.com

INSIGHTS

Page 26

Facebook Group: Indian Administrative Service (Raz Kr)

INSIGHTS MOCK TESTS 2015 TEST 8 SOLUTIONS


role in the Green Revolution), has been reduced in the reform period. Further, the removal
of fertiliser subsidy has led to increase in the cost of production, which has severely affected
the small and marginal farmers.
This sector has been experiencing a number of policy changes such as reduction in import
duties on agricultural products, removal of minimum support price and lifting of
quantitative restrictions on agricultural products; these have adversely affected Indian
farmers as they now have to face increased international competition.
Moreover, because of exportoriented policy strategies in agriculture, there has been a shift
from production for the domestic market towards production for the export market focusing
on cash crops in lieu of production of food grains. This puts pressure on prices of food
grains.
72. Solution: d)
Financial Sector Reforms
Financial sector includes financial institutions such as commercial banks, investment banks,
stock exchange operations and foreign exchange market. The financial sector in India is
controlled by the Reserve Bank of India (RBI). All the banks and other financial institutions
in India are controlled through various norms and regulations of the RBI. The RBI decides
the amount of money that the banks can keep with themselves, fixes interest rates, nature of
lending to various sectors etc.
One of the major aims of financial sector reforms is to reduce the role of RBI from regulator
to facilitator of financial sector. This means that the financial sector may be allowed to take
decisions on many matters without consulting the RBI.
The reform policies led to the establishment of private sector banks, Indian as well as
foreign.
Foreign investment limit in banks was raised to around 50 per cent.
Those banks which fulfil certain conditions have been given freedom to set up new branches
without the approval of the RBI and rationalise their existing branch networks.
Though banks have been given permission to generate resources from India and abroad,
certain managerial aspects have been retained with the RBI to safeguard the interests of the
accountholders and the nation. Foreign Institutional Investors (FII) such as merchant
bankers, mutual funds and pension funds are now allowed to invest in Indian financial
markets.
73. Solution: c)
Migration is a consequence, not a reason.
Underemployment is a problem of urban areas. This results in getting underpaid not
causing poverty.
http://insightsonindia.com

INSIGHTS

Page 27

Facebook Group: Indian Administrative Service (Raz Kr)

INSIGHTS MOCK TESTS 2015 TEST 8 SOLUTIONS


Seasonal unemployment in agriculture threatens livelihood security.
74. Solution: d)
Problems in determining a uniform poverty line:
Poverty is a state of mind, a situation that people want to escape.
1. Seasonality of poverty
2. Non-distinction b/w antyodaya (poorest of the poor) and the just-above poor. In other
aspects, differentiating between chronic and transient poverty.
3. Many other factors like drinking water, sanitation etc also need to be taken into account
and their prices vary across states and regions
4. Factoring social indicators which lead to poverty
5. Change in the consumption basket over time and increase in prices beyond the levels of
inflation suggest
75. Solution: d)
Basis of methodology of Rangarajan panel on poverty
It adopted different yardsticks to measure poverty to reflect the changes in the Indian
economy

Consumption expenditure - food (calorie based); essential non-food like


education, health, clothing, conveyance, house rent etc.

Household's ability to save

Behaviorally determined expenditure for non-food items

It also introduced the concept of "relative poverty" by picking up for e.g. the bottom
quintile of population or using media consumption expenditure.

Difference from Tendulkar Committee

Stated that poverty in India was greater than suggested by Tendulkar Committee
(22.5%) in 2011-12 from 29.8% in 2009-10

Included fats and proteins too in food items to reflect present dietary habits.
Tendulkar - only calorie

Minimum basket separate for urban and rural areas. Tendulkar had only one basket.

Non-food consumption items were also included for the first time.

76. Solution: a)

http://insightsonindia.com

INSIGHTS

Page 28

Facebook Group: Indian Administrative Service (Raz Kr)

INSIGHTS MOCK TESTS 2015 TEST 8 SOLUTIONS


A very detailed and very useful discussion is given in Chapter 4 Poverty 11th NCERT
Economy under the section 4.6 - POLICIES AND PROGRAMMES TOWARDS POVERTY
ALLEVIATION. Please refer to it for a detailed understanding of the evolution of these
approaches.
77. Solution: d)
Just as a country can turn physical resources like land into physical capital like factories,
similarly, it can also turn human resources like students into human capital like engineers
and doctors.
Investment in education is considered as one of the main sources of human capital. There
are several other sources as well. Investments in health, on- thejob training, migration and
information are the other sources of human capital formation.
Freedom of thought and expression is one of the most fundamental factors in quality human
capital creation. It is the foundation of a nations intellect.
78. Solution: d)
The Indian software industry has been showing an impressive record over the past decade.
Entrepreneurs, bureaucrats and politicians are now advancing views about how India can
transform itself into a knowledge-based economy by using information technology (IT).
There have been some instances of villagers using e-mail which are cited as examples of
such transformation. Likewise, e-governance is being projected as the way of the future. The
value of IT depends greatly on the existing level of economic development.
Also refer to basics here:
http://en.wikipedia.org/wiki/Knowledge_economy
79. Solution: c)
Elementary education takes a major share of total education expenditure and the share of
the higher/tertiary education (institutions of higher learning like colleges, polytechnics and
universities) is the least. Though, on an average, the government spends less on tertiary
education, expenditure per student in tertiary education is higher than that of elementary.
This does not mean that financial resources should be transferred from tertiary education to
elementary education. As we expand school education, we need more teachers who are
trained in the higher educational institutions; therefore, expenditure on all levels of
education should be increased.
80. Solution: d)
The Green Revolution was a harbinger of major changes in the credit system as it led to the
diversification of the portfolio of rural credit towards production oriented lending. The
institutional structure of rural banking today consists of a set of multi-agency institutions,
namely, commercial banks, regional rural banks (RRBs), cooperatives and land development
banks. They are expected to dispense adequate credit at cheaperrates. Recently, Self-Help
http://insightsonindia.com

INSIGHTS

Page 29

Facebook Group: Indian Administrative Service (Raz Kr)

INSIGHTS MOCK TESTS 2015 TEST 8 SOLUTIONS


Groups (henceforth SHGs) have emerged to fill the gap in the formal credit system because
the formal credit delivery mechanism has not only proven inadequate but has also not been
fully integrated into the overall rural social and community development.
81. Solution: a)
Recently, Self-Help Groups (henceforth SHGs) have emerged to fill the gap in the formal
credit system because the formal credit delivery mechanism has not only proven inadequate
but has also not been fully integrated into the overall rural social and community
development. Since some kind of collateral is required, vast proportion of poor rural
households were automatically out of the credit network. The SHGs promote thrift in small
proportions by a minimum contribution from each member. From the pooled money, credit
is given to the needy members to be repayable in small instalments at reasonable interest
rates.
Such credit provisions are generally referred to as micro-credit programmes. SHGs have
helped in the empowerment of women. It is alleged that the borrowings are mainly confined
to consumption purposes.
82. Solution: a)
In India, after progressive increase in budgetary allocations and introduction of new
technologies in fisheries and aquaculture, the development of fisheries has come a long way.
Presently, fish production from inland sources contributes about 61 per cent to the total fish
production and the balance 39 per cent comes from the marine sector (sea and oceans).
Today total fish production accounts for 0.7 per cent of the total GDP. Among states, Kerala,
Gujarat, Maharashtra and Tamil Nadu are the major producers of marine products.
A large share of fishworker families are poor. Rampant underemployment, low per capita
earnings, absence of mobility of labour to other sectors and a high rate of illiteracy and
indebtedness are some of the major problems fishing community face today.
83. Solution: d)
Organic agriculture offers a means to substitute costlier agricultural inputs (such as HYV
seeds, chemical fertilisers, pesticides etc.) with locally produced organic inputs that are
cheaper and thereby generate good returns on investment.
Organic agriculture also generates income through exports as the demand for organically
grown crops is on a rise. Studies across countries have shown that organically grown food
has more nutritional value than chemical farming thus providing us with healthy foods.
Since organic farming requires more labour input than conventional farming, India will find
organic farming an attractive proposition. Finally, the produce is pesticide-free and
produced in an environmentally sustainable way.
84. Solution: c)

http://insightsonindia.com

INSIGHTS

Page 30

Facebook Group: Indian Administrative Service (Raz Kr)

INSIGHTS MOCK TESTS 2015 TEST 8 SOLUTIONS


Blessed with a varying climate and soil conditions, India has adopted growing of diverse
horticultural crops such as fruits, vegetables, tuber crops, flowers, medicinal and aromatic
plants, spices and plantation crops.
These crops play a vital role in providing food and nutrition, besides addressing
employment concerns. The period between 1991-2003 is also called an effort to heralding a
Golden Revolution because during this period, the planned investment in horticulture
became highly productive and the sector emerged as a sustainable livelihood option. India
has emerged as a world leader in producing a variety of fruits like mangoes, bananas,
coconuts, cashew nuts and a number of spices and is the second largest producer of fruits
and vegetables.
Flower harvesting, nursery maintenance, hybrid seed production and tissue culture,
propagation of fruits and flowers and food processing are highly remunerative employment
options for women in rural areas.
85. Solution: d)
Casualization of the workforce occurs whenever workers are employed in a casual,
temporary, or otherwise non-permanent and non-full-time capacity. In recent years,
casualization has become an increasingly visible problem, and those workers affected are
often subject to lower pay, barred from their right to join a union, and denied medical and
other benefits. Companies will often hire several part-time workers instead of one or two
full-time workers to avoid their obligation to provide benefits, to divide the workforce, and
to dissuade unionizing efforts.
In India the unorganised sector accounts for 90% of the total workforce and economic
activity.
86. Solution: d)
We classify workforce into two categories: workers in formal and informal sectors, which are
also referred to as organised and unorganised sectors.
All the public sector establishments and those private sector establishments which employ
10 hired workers or more are called formal sector establishments and those who work in
such establishments are formal sector workers.
All other enterprises and workers working in those enterprises form the informal sector.
Thus, informal sector includes millions of farmers, agricultural labourers, owners of small
enterprises and people working in those enterprises as also the self-employed who do not
have any hired workers. It also includes all non-farm casual wage labourers who work for
more than one employer such as construction workers and headload workers.
87. Solution: a)
Disguised unemployment is a situation where more labourers are working in a particular
economic activity than would be normally required.
http://insightsonindia.com

INSIGHTS

Page 31

Facebook Group: Indian Administrative Service (Raz Kr)

INSIGHTS MOCK TESTS 2015 TEST 8 SOLUTIONS


High population can be understood as excess of labour supply. More labour will be unable
to find jobs from rural area. So they shift to family or other farms.
Seasonal nature of agricultural work means even the well-off farmers can not employ full tie
labourers. This may contribute to unemployment, not disguised unemployment.
Small land holding size means the farmer can not deploy adequate technology. So labourers
have to be deployed (either from the family or outside). An incorrect estimation of work
based on scientific principles may result in more labourers than what may be required.
88. Solution: d)
A group of religious-minded people in medieval Islam, known as Sufis, sought a deeper and
more personal knowledge of God through asceticism (rahbaniya) and mysticism. The more
society gave itself up to material pursuits and pleasures, the more the Sufis sought to
renounce the world (zuhd) and rely on God alone (tawakkul). In the eighth and ninth
centuries, ascetic inclinations were elevated to the higher stage of mysticism (tasawwuf) by
the ideas of pantheism and love. Pantheism is the idea of oneness of God and His creation
which implies that the human soul must be united with its Maker. Unity with God can be
achieved through an intense love for God (ishq), which the woman-saint Rabia of Basra (d.
891) preached in her poems. Bayazid Bistami (d. 874), an Iranian Sufi, was the first to teach
the importance of submerging the self (fana) in God. Sufis used musical concerts (sama) to
induce ecstasy and stimulate emotions of love and passion.
89. Solution: a)
Faxian visited India in the early fifth century AD. He is said to have walked all the way from
China across icy desert and rugged mountain passes. He entered India from the north-west
and reached Pataliputra. He took back with him Buddhist texts and images sacred to
Buddhism.
Faxian's visit to India occurred during the reign of Chandragupta II. He is also renowned for
his pilgrimage to Lumbini, the birthplace of Gautama Buddha in modern Nepal. Faxian
claimed that demons and dragons were the original inhabitants of Ceylon (Sri Lanka)
90. Solution: d)
Nalanda was an acclaimed Mahvihra, a large Buddhist monastery in
ancient Magadha (modern-day Bihar), India. The site is located about 95 kilometres
southeast of Patna, and was a religious centre of learning from the fifth century CE to c.1197
CE.[3][4] Historians often characterize Nalanda as a university.
Nalanda flourished under the patronage of the Gupta Empire as well as emperors
like Harsha and later, the rulers of the Pala Empire. At its peak, the school attracted scholars
and students from as far away as Tibet, China, Korea, and Central Asia. It was ransacked
and destroyed by an army of the Muslim Mamluk Dynasty under Bakhtiyar Khilji in c.1197
CE.

http://insightsonindia.com

INSIGHTS

Page 32

Facebook Group: Indian Administrative Service (Raz Kr)

INSIGHTS MOCK TESTS 2015 TEST 8 SOLUTIONS


The Tang Dynasty Chinese pilgrim Xuanzang left detailed accounts of the school in the 7th
century. He described how the regularly laid-out towers, forest of pavilions, harmikas and
temples seemed to "soar above the mists in the sky" so that from their cells the monks "might
witness the birth of the winds and clouds." The pilgrim states: "An azure pool winds around
the monasteries, adorned with the full-blown cups of the blue lotus; the dazzling red flowers
of the lovely kanaka hang here and there, and outside groves of mango trees offer the
inhabitants their dense and protective shade."
91. Solution: b)

92. Solution: c)
Zahir-ud-din Muhammad Babur was a conqueror from Central Asia who, following a
series of setbacks, finally succeeded in laying the basis for the Mughal dynasty in the Indian
Subcontinent and became the first Mughal emperor. He was a direct descendant of Timur,
from the Barlas clan, through his father, and also a descendant of Genghis Khan through his
mother. Culturally, he was greatly influenced by the Persian culture and this affected both
http://insightsonindia.com

INSIGHTS

Page 33

Facebook Group: Indian Administrative Service (Raz Kr)

INSIGHTS MOCK TESTS 2015 TEST 8 SOLUTIONS


his own actions and those of his successors, giving rise to a significant expansion of
the Persianate ethos in the Indian subcontinent.
93. Solution: b)
Christopher Columbus (1451-1506) was a self-taught man who sought adventure and glory.
Believing in prophecies, he was convinced that his destiny lay in discovering a route to the
East (the Indies) by sailing westwards. He was inspired by reading Imago Mundi (a work
on astronomy and geography) by Cardinal Pierre dAilly written in 1410. He submitted his
plans to the Portuguese Crown, only to have them turned down. He had better luck with the
Spanish authorities who sanctioned a modest expedition that set sail from the port of Palos
on 3 August 1492. Nothing, however, prepared Columbus and his crew for the long Atlantic
crossing that they embarked upon, or for the destination that awaited them.
The fleet was small, consisting of a small nao called Santa Maria, and two caravels (small
light ships) named Pinta and Nina. Columbus himself commanded the Santa Maria along
with 40 capable sailors. The outward journey enjoyed fair trade winds but was long. For 33
days, the fleet sailed without sight of anything but sea and sky. By this time, the crew
became restive and some of them demanded that they turn back.
On 12 October 1492, they sighted land; they had reached what Columbus thought was
India, but which was the island of Guanahani in the Bahamas. (It is said that this name was
given by Columbus, who described the Islands as surrounded by shallow seas, baja mar in
Spanish.) They were welcomed by the Arawaks, who were happy to share their food and
provisions; in fact, their generosity made a deep impression upon Columbus. As he wrote in
his log-book, They are so ingenuous and free with all they have, that no one would believe
it who has not seen of it, anything they possess, if it be asked of them, they never say no, on
the contrary, they invite you to share it and show as much love as if their hearts went with
it.
94. Solution: d)
http://en.wikipedia.org/wiki/Aryabhata#Astronomy
95. Solution: b)
The Portuguese occupation of Brazil occurred by accident. In 1500, a grand procession of
ships set out from Portugal for India, headed by Pedro Alvares Cabral. To avoid stormy
seas, he made a wide loop around West Africa, and found to his surprise that he had
reached the coast of present-day Brazil. As it happened, this eastern part of South America
was within the section assigned on the map to Portugal by the Pope, so they regarded it as
indisputably theirs.
The Portuguese were more eager to increase their trade with western India than with Brazil,
which did not promise any gold. But there was one natural resource there which they
exploited: timber. The brazilwood tree, after which the Europeans named the region,
produced a beautiful red dye. The natives readily agreed to cut the trees and carry the logs
http://insightsonindia.com

INSIGHTS

Page 34

Facebook Group: Indian Administrative Service (Raz Kr)

INSIGHTS MOCK TESTS 2015 TEST 8 SOLUTIONS


to the ships in exchange for iron knives and saws, which they regarded as marvels. (For one
sickle, knife or comb [they] would bring loads of hens, monkeys, parrots, honey, wax, cotton
thread and whatever else these poor people had.)
96. Solution: c)
The Meiji Restoration also known as the Meiji Ishin, Renovation, Revolution, Reform,
or Renewal, was a chain of events that restored practical imperial rule to Japan in 1868
under Emperor Meiji. Although there were emperors of Japan before the Meiji Restoration,
the restoration established the practical abilities and consolidated the political system under
the Emperor of Japan. The goals of the restored government were expressed by the new
emperor in the Charter Oath. The Restoration led to enormous changes in Japan's political
and social structure, and spanned both the late Edo period (often called Late Tokugawa
shogunate) and the beginning of the Meiji period. The period spanned from 1868 to 1912 and
was responsible for the emergence of Japan as a modernized nation in the early twentieth
century.
97. Solution: c)
http://timesofindia.indiatimes.com/india/India-develops-new-tactical-missilePragati/articleshow/24882453.cms
http://en.wikipedia.org/wiki/Prahaar_(missile)
98. Solution: a)
The Government has revived the Varishta Pension Bima Yojana (VPBY) scheme for the
benefit of senior citizens aged 60 years and above. The scheme is being administered
through Life Insurance Corporation of India (LIC) and will remain open for a period of one
year from 15th August, 2014 to 14th August, 2015.
Under the scheme, all those who will subscribe to the VPBY during this period, will
receive a pension linked to an assured guaranteed return of 9% on the lump sum
contribution under the Yojana. The Salient features of the scheme are as under:

The scheme provides pension in the form of immediate annuity during the lifetime of
the pensioner with return of purchase price to the family/nominee on his/her death.

The mode of payment of pension can be monthly, quarterly, half-yearly or yearly.

The pension payment shall be through ECS or NEFT.

The minimum pension that can be subscribed to will be Rs. 500/- per month, Rs.
1500/- per quarter, Rs. 3000/- per half-year or Rs. 6000/- per year.

The maximum pension that can be subscribed to will be Rs. 5,000/- per month, Rs.
15,000/- per quarter, Rs. 30,000/- per half-year or Rs. 60,000/- per year.

http://insightsonindia.com

INSIGHTS

Page 35

Facebook Group: Indian Administrative Service (Raz Kr)

INSIGHTS MOCK TESTS 2015 TEST 8 SOLUTIONS

A lump sum purchase price for the desired pension shall be paid by the subscriber
under the scheme. The minimum and maximum purchase price for different modes of
pension corresponding to the maximum/minimum pensions given above will be a under:
99. Solution: c)
http://pib.nic.in/newsite/PrintRelease.aspx?relid=111998
100.

Solution: a)

Opium was first introduced to China by Turkish and Arab traders in the late 6th or early 7th
century. Taken orally to relieve tension and pain, the drug was used in limited quantities
until the 17th century, when the practice of smoking tobacco spread from North America to
China. The smoking of opium soon became popular throughout China; opium addiction
increased, and opium importations grew rapidly. By 1729 it had become such a problem that
the Yung-cheng emperor (ruled 172235) prohibited the sale and smoking of opium. This
failed to hamper thetrade, and in 1796 the Chia-ching emperor outlawed opium
importation and cultivation. In spite of such decrees, however, the opium trade continued to
flourish.
Early in the 18th century the Portuguese found that they could import opium from India
and sell it in China at a considerable profit. By 1773 the British had discovered the trade, and
that year they became the leading suppliers of the Chinese market. The British East India
Companyestablished a monopoly on opium cultivation in the Indian province of Bengal,
where they developed a method of growing opium poppies cheaply and abundantly. Other
Western nations also joined in the trade, including the United States, which dealt in Turkish
as well as Indian opium.
Britain and other European nations undertook the opium trade because of their chronic
trade imbalance with China. There was tremendous demand in Europe for Chinese tea,
silks, and porcelain pottery, but there was correspondingly little demand in China for
Europes manufactured goods and other trade items. Consequently, Europeans had to pay
for Chinese products with gold or silver. The opium trade, which created a steady demand
among Chinese addicts for opium imported by the West, solved this chronic trade
imbalance.
The East India Company did not carry the opium itself but, because of the Chinese ban,
farmed it out to country tradersi.e., private traders licensed by the company to take
goods from India to China. The country traders sold the opium to smugglers along the
Chinese coast. The gold and silver the traders received from these sales were then turned
over to the East India Company. In China the company used the gold and silver it received
to purchase goods that could be sold profitably in England.
The amount of opium imported into China increased from around 200 chests a year in 1729
to about 1,000 chests in 1767 and to around 10,000 a year between 1820 and 1830. By 1838 the
amount had grown to some 40,000 chests imported into China annually. The balance of
payments for the first time began to run against China and in favour of Britain.
http://insightsonindia.com

INSIGHTS

Page 36

Facebook Group: Indian Administrative Service (Raz Kr)

INSIGHTS MOCK TESTS 2015 TEST 8 SOLUTIONS

http://insightsonindia.com

INSIGHTS

Page 37

Facebook Group: Indian Administrative Service (Raz Kr)

INSIGHTS ON INDIA MOCK PRELIMINARY EXAM - 2015


INSIGHTS ON INDIA MOCK TEST - 9
GENERAL STUDIES

PAPER-I
Time Allowed: 2 Hours

Maximum Marks: 200

INSTRUCTIONS
1. IMMEDITELY AFTER THE COMMENCEMENT OF THE EXAMINATION, YOU SHOULD
CHECK THAT THIS TEST BOOKLET DOES NOT HAVE ANY UNPRINTED OR TORN OR MISSING
PAGES OR ITEMS, ETC. IF SO, GET IT REPLACED BY A COMPLETE TEST BOOKLET.
2. You have to enter your Roll Number on the Test
Booklet in the Box provided alongside. DO NOT
Write anything else on the Test Booklet.
4. This Test Booklet contains 100 items (questions). Each item is printed only in English. Each item
comprises four responses (answers). You will select the response which you want to mark on the
Answer Sheet. In case you feel that there is more than one correct response, mark the response which
you consider the best. In any case, choose ONLY ONE response for each item.
5. You have to mark all your responses ONLY on the separate Answer Sheet provided. See directions in
the Answer Sheet.
6. All items carry equal marks.
7. Before you proceed to mark in the Answer Sheet the response to various items in the Test Booklet, you
have to fill in some particulars in the Answer Sheet as per instructions sent to you with your
Admission Certificate.
8. After you have completed filling in all your responses on the Answer Sheet and the examination has
concluded, you should hand over to the Invigilator only the Answer Sheet. You are permitted to take
away with you the Test Booklet.
9. Sheets for rough work are appended in the Test Booklet at the end.
10. Penalty for wrong answers :
THERE WILL BE PENALTY FOR WRONG ANSWERS MARKED BY A CANDIDATE IN THE
OBJECTIVE TYPE QUESTION PAPERS.
(i) There are four alternatives for the answer to every question. For each question for which a
wrong answer has been given by the candidate, one-third of the marks assigned to that
question will be deducted as penalty.
(ii) If a candidate gives more than one answer, it will be treated as a wrong answer even if one of
the given answers happens to be correct and there will be same penalty as above to that
question.
(iii)

If a question is left blank, i.e., no answer is given by the candidate, there will be no penalty
for that question.
http://insightsonindia.com

INSIGHTS ON INDIA MOCK TEST SERIES FOR CIVIL SERVICES PRELIMINARY EXAM 2015
http://insightsonindia.com

INSIGHTS

Page 1

Facebook Group: Indian Administrative Service (Raz Kr)

1. Which of the following factors may have


helped the growth of population during
the 19th and 20th century?
1. Sanitation technology
2. Development and spread of
agriculture
3. Industrial revolution

4. A climax community is one where the


community is on the verge of
extinction
Which of the above statements is/are correct?
a)
b)
c)
d)

Choose the correct answer using the codes below:


a)
b)
c)
d)

1 and 2
2 and 3
1 and 3
All of the above

2. With reference to cave paintings in India,


consider the following statements
1. The Lakhudiyar cave paintings near
Barechhina in Jammu and Kashmir
belong to Palaeolithic age
2. Bhimbetka in Madhya Pradesh, where
rich cave paintings have been found, is
located in the Vindhya hills

4. Consider the following statements with


respect to the demography of a country.
Assertion (A): Developed countries are taking
more time to double their population as
compared to developing countries.
Reason (R): There is negative correlation between
economic development and population
growth.
In the context of the statements above, which of
these is true?
a) A and R both are true, and R is the
correct explanation for A.
b) A and R both are true, and R is the
NOT the correct explanation for A.
c) A is correct, R is incorrect.
d) A and R both are incorrect

Which of the above statements is/are correct?


a)
b)
c)
d)

1 Only
2 Only
Both
None

3. Consider the following statements


1. The species that invade a bare area are
called pioneer species
2. The gradual and fairly predictable
change in the species composition of a
given area is called ecological
succession
3. If succession takes place in areas that
somehow, lost all the living organisms
that existed there, then it is called as
secondary succession

http://insightsonindia.com

1,2 and 4 Only


2,3 and 4 Only
1,2 and 4 only
1,2 and 3 Only

5. Consider the following age pyramid and


some conclusions derived from it.

INSIGHTS

Page 2

Facebook Group: Indian Administrative Service (Raz Kr)

1. It is the age pyramid of a developing


country.
2. Population is constant.
3. A greater part of the population is
between the ages of 15-59 than the
other age groups.
Which of the following conclusions MAY be
correct?
a)
b)
c)
d)

Choose the correct answer using the codes below:


a)
b)
c)
d)

1 and 2
2 and 3
1 and 3
All of the above

6. Which of the following Indus Valley


Civilisation sites is NOT located in
Gujarat?
1. Dholavira
2. Ropar
3. Kalibangan
4. Rakhigarhi
5. Balathal
6. Lothal
Choose the correct answer using the codes below
(a) 1,2,3 and 4
(b) 2,3,4 and 6
(c) 2,3,4 and 5
(d) 2,3 and 4
7. Which of the following characteristics
differentiate between an urban and a rural
area?
1. Castesim is greater in rural areas.
2. Greater part of the population is
engaged in non-primary activities in
an urban area.
3. Density of population is higher in
rural areas.
Choose the correct answer using the codes below:
a)
b)
c)
d)

8. Which of the following is NOT a


necessary pillar of human development?
1. Equity
2. Empowerment
3. Sustainability

1 and 2
2 and 3
1 and 3
2 only

http://insightsonindia.com

1 and 2
2 and 3
1 and 3
All are necessary pillars

9. With reference to the Lion Capital found


at Sarnath, consider the following
statements
1. It was built in commemoration of the
historical event of the first sermon or
the Dhammachakrapravartana by the
Buddha at Sarnath
2. This was built by Ashoka
3. The crowning element, Dharamchakra,
a large wheel above the lions, was also
a part of this pillar
Which of the above statements is/are correct?
a)
b)
c)
d)

2 and 3 Only
1 and 2 Only
1 and 3 Only
All

10. Which of the following aspects of a


citizens life are included in Bhutans
Gross National Happiness (GNH) index?
1. Spiritual
2. Cultural
3. Economic
4. Good governance
Choose the correct answer using the codes below:
a) 3 and 4 only
b) 2, 3 and 4 only
c) 1 and 2 only
d) All of the above
INSIGHTS

Page 3

Facebook Group: Indian Administrative Service (Raz Kr)

11. Consider the following statements about


the Basic Structure of the Constitution of
India.
1. It is mentioned in Part III of the
Constitution.
2. The Supreme Court has interpreted
the Basic Structure in its various
judgements giving also the exhaustive
constituents of the basic structure.
3. A law or constitutional amendment
can be termed void if it violates the
Basic Structure.

14. Which of the following are the


characteristic of mixed farming?
1. Fodder crops are an important
component.
2. Crop rotation
3. Inter-cropping
4. Extensive use of chemical fertilizers
Choose the correct answer using the codes below:
a)
b)
c)
d)

Choose the correct answer using the codes below:


a)
b)
c)
d)

1 and 2
2 and 3
1 and 3
3 only

12. With reference to carrying capacity of the


earth, consider the following statements
1. Population size decreases above
carrying capacity of the earth
2. Ecological Footprint accounting helps
in estimating human demand
compared to ecosystem's carrying
capacity
Which of the above statements is/are correct?
a) 1 Only
b) 2 Only
c) Both
d) None
13. Which of the following factors are
common to countries with a lower HDI
value?
1. Political instability
2. Social unrest
3. Common religion
Choose the correct answer using the codes below:
a) 1 and 2
b) 2 and 3
c) 1 and 3
d) All of the above
http://insightsonindia.com

3 and 4 only
2, 3 and 4 only
1 and 2 only
All of the above

15. Truck farming is common in regions


where farmers specialize in growing
a) Cotton
b) Vegetables
c) Flowers
d) Fruits
16. Consider the following statements
1. The best example Gandhara style is the
stupa sculptures found at Sanghol in
the Punjab
2. In the Mathura school of sculpture,
only Buddhist sculptures are carved
Which of the above statements is/are correct?
a)
b)
c)
d)

1 Only
2 Only
Both
None

17. How are cooperative and collective


farming different?
1. In collective farming, farmers own the
pieces of land collectively, not
individually.
2. In collective farming, there is no
hierarchy all farmers have equal
power unlike cooperative farming.
INSIGHTS

Page 4

Facebook Group: Indian Administrative Service (Raz Kr)

3. A cooperative can be state enforced


unlike collective farming.
Choose the correct answer using the codes below:
a) 1 and 2
b) 2 and 3
c) 1 and 3
d) All of the above
18. Industries based on cheap and weight
losing material are located close to the
sources of raw material because
a) Further transportation cost will be less
b) Processing cost will be less
c) Raw material cost will be less
d) Labour cost will be less
19. Consider the following statements about
foot lose industries.
Assertion (A): They can be located in a
wide variety of places.
Reason (R): Because of their nondependence on raw materials.
In the context of the statements above, which of
these is true?
a) A and R both are true, and R is the
correct explanation for A.
b) A and R both are true, and R is the
NOT the correct explanation for A.
c) A is correct, R is incorrect.
d) A and R both are incorrect
20. Which of the following characteristics do
NOT belong to the Agri-businesses?
1. Highly mechanised
2. Large in Size
3. Reliance on chemicals
Choose the correct answer using the codes below:
a) 1 and 2
b) 2 and 3
c) 1 and 3
d) All of the above
http://insightsonindia.com

21. Which of the following perform Quinary


activities?
1. Secretary of Union Health
Department, Government of India.
2. CEO of an India based MNC
3. Research scientists
Choose the correct answer using the codes below:
a) 1 and 2
b) 2 and 3
c) 1 and 3
d) All of the above
22. Salmonella typhi which causes typhoid
fever in human beings is a
a) Bacteria
b) Protozoa
c) Virus
d) Fungus
23.

Consider the following statements


1. Commensalism is the interaction in
which one species benefits and the
other is neither harmed nor benefited
2. A grazing cattle is a classic example of
commensalism
Which of the above statements is/are correct?
a) 1 Only
b) 2 Only
c) Both
d) None
24. When outsourcing involves transferring
work to overseas locations, it is described
by the term off shoring. Why do
companies off-shore?
a) Non-availability of skilled manpower
in the country that off-shores
b) Availability of cheap skilled labour in
the country that is being off-shored to
c) Lack of good business climate in the
country that off-shores
d) To avoid issues of transfer pricing to
evade taxes

INSIGHTS

Page 5

Facebook Group: Indian Administrative Service (Raz Kr)

25. Railways in Russia are preferred much


more than Highways. What can be the
possible reason?
a) Huge geographical extent of Russia
b) The extremely cold climate of Russia
c) It is difficult to construct roads in the
Russian terrain
d) None of the above is an appropriate
reason
26. Before the construction of the Suez Canal
which route connected Liverpool (in UK)
and Colombo (in Sri Lanka) by sea?
a) The MediterraneanIndian Ocean
Route
b) The Cape of Good Hope Sea Route
c) The Northern Atlantic Sea Route
d) The South Pacific Sea Route
27. Which of the following routes fall under
the National Inland Waterways of India?
1. Bhagirathi Hooghly river route
2. SadiyaDhubri stretch of
the Brahmaputra River
3. Lakhipur-Bhanga stretch of Barak
River
Choose the correct answer using the codes below:
a)
b)
c)
d)

1 and 2
2 and 3
1 and 3
All of the above

28. Consider the following statements about


the Silk Road.
1. Trade on this route passed between
Europe and China, through India.
2. There is a recent proposal to establish
special economic zones and industrial
parks in the areas connected by Silk
Road.
3. Only Silk was traded on this route.

a)
b)
c)
d)

29. The wave of globalization has resulted in


an unprecedented growth of international
trade. International trade is based on
which of the following principles?
1. Comparative advantage
2. Complementarity and transferability
of goods and services
3. Sovereignty
Choose the correct answer using the codes below:
a) 1 and 2
b) 2 and 3
c) 1 and 3
d) All of the above
30. Arrange Indias total exports (in money
terms) with the following countries in
decreasing order.
1. USA
2. China
3. UAE
4. Brazil
Choose the correct order from the codes below:
a) 2134
b) 1234
c) 1342
d) 1324
31. In a country, a greater aged population is
problematic for which of the following
reasons?
1. Low demographic dividend
2. High social security expenditure
3. High death rates in population
Choose the correct answer using the codes below:
a)
b)
c)
d)

Choose the correct answer using the codes below:


http://insightsonindia.com

1 and 2
2 and 3
1 and 3
All of the above

INSIGHTS

1 and 2
2 and 3
1 and 3
All of the above
Page 6

Facebook Group: Indian Administrative Service (Raz Kr)

32. With reference to temple architecture in


India, consider the following statements
1. According to some scholars the Vesar
style of temples as an independent
style created through the selective
mixing of the Nagara and Dravida
orders
2. The vahan refers to the entrance to the
temple which may be a portico or
colonnaded hall that incorporates
space for a large number of
worshippers

1. Countries cannot normally


discriminate between their trading
partners.
2. A special privilege like lower customs,
if granted to one country has to be
extended to other countries too.
3. Countries can not set up a free trading
zone with regional countries.
Choose the correct answer from the codes below:
a)
b)
c)
d)

Which of the above statements is/are correct?


a) 1 Only
b) 2 Only
c) Both
d) None
33. The following commodities are the most
exported from India.
1. Petroleum Products
2. Pharmaceutical Products
3. Cotton, yarn and fabrics
4. Electronics items
Arrange them in decreasing order from the codes
below:
a)
b)
c)
d)

2134
1234
1243
2143

34. A narrow base and broad top in an age


pyramid signifies
a) Population ageing
b) Population is young
c) Death rates are very low.
d) Both (a) and (c)

1 and 2
2 and 3
1 and 3
All of the above

36. The practice of dumping concerning


international trade refers to
a) discrimination meted out by the host
country to the other investing (and
trading) country
b) Very high duties imposed by the host
country on the products of other
countries
c) Foreign countries selling goods at
unreasonably lower prices in
international markets.
d) None of the above
37. WTO does NOT cover which of the
following areas of international trade?
1. Intellectual Property Rights
2. Trade in Invisibles
3. Banking
Choose the correct answer from the codes below:
a)
b)
c)
d)

1 and 2
2 and 3
3 only
All of the above are covered by WTO

35. Which of the following are covered under


the Most-favoured nation (MFN)
provision of WTO?
http://insightsonindia.com

INSIGHTS

Page 7

Facebook Group: Indian Administrative Service (Raz Kr)

38. Which of the following components are


included in WHOs definition of a
Healthy City?
1. Meets the Basic Needs of All its
inhabitants.
2. Involves the Community in local
government.
3. Upgrading Energy use and
alternative Transport systems.
Choose the correct answer from the codes below:
a)
b)
c)
d)

Choose the correct answer from the codes below:

1 and 2
2 and 3
None of the above
All of the above

39. Consider the following statements about


Antacids.
1. They are used to treat acidity in the
liver and pancreas.
2. Excessive administration of Antacids
can trigger the production of more
acid in the concerned organ(s).
3. Antacids can also be used to treat ant
bites.
Choose the correct answer from the codes below:
a)
b)
c)
d)

1. They are used to treat depression,


anxiety and stress.
2. They can also be used to treat major
states of mental disturbance in
schizophrenics and other psychotic
patients.
3. They are highly useful in alleviating
the delusions, hallucinations, and
disordered thinking.

1 and 2
2 and 3
1 and 3
All of the above

40. You are suffering with nasal congestion


associated with common cold and allegy
due to pollen grains. Which of the
following is most likely to treat your
congestion?
a) Anti-histamines
b) Analgesics
c) Peptides
d) None of the above
41. Consider the following statements about
Tranquilizers.
http://insightsonindia.com

a)
b)
c)
d)

1 and 2
2 and 3
1 and 3
All of the above

42. Consider the following statements about


Public Interest Litigations (PILs).
1. Only the affected
individual/group/institution can file a
PIL in the court.
2. PILs are mentioned in Article 144 of
the constitution of India to ensure
social justice to the marginalised.
3. Judiciary can consider a case on its
own based on a newspaper report or
postal complaint received by the court.
Choose the correct answer using the codes below:
a)
b)
c)
d)

1 and 2
2 and 3
1 and 3
3 only

43. Chikungunya, an infection caused by the


Chikungunya virus, is transmitted to
humans by
a) Drinking infected water
b) Bite of specific species of infected
mosquitos
c) Coming in contact with secretions of
infected person
d) None of the above
INSIGHTS

Page 8

Facebook Group: Indian Administrative Service (Raz Kr)

44. Analgesics are used to reduce or abolish


pain. But their use in human beings can
lead to the impairment of
1. Mental Confusion
2. Incoordination in the nervous system
3. Impairment of Consciousness

2. Judges cannot be removed without an


inquiry by a bench of the Supreme
Court.
3. Absolute majority is required in the
Parliament to remove judges.
Choose the correct answer using the codes below:

Choose the correct answer from the codes below:


a)
b)
c)
d)

1 and 2
2 and 3
1 and 3
None of the above

45. Excessive and inappropriate use of antibiotics leads to anti-microbial resistance.


Which of the following processes may
lead to anti-microbial resistance after the
application of anti-biotics?
a) Natural Selection
b) Budding
c) Selective assimilation by the body
d) Selective assimilation by the bacteria
46. Consider the following statements
1. The exploitative appropriation of
indigenous forms of knowledge by
commercial actors is known as
biopiracy
2. The Nagoya Protocol to the
Convention on Biological Diversity
regulates biopiracy related issues
Which of the above statements is/are correct?
a)
b)
c)
d)

a)
b)
c)
d)

1 Only
2 Only
Both
None

47. Consider the following statements about


the removal of judges of Supreme Court
and High Courts.
1. Motion to impeach a judge is admitted
by the respective chairman/speaker of
the houses of Parliament.
http://insightsonindia.com

1 and 2
2 and 3
1 and 3
All of the above

48. Consider the following statements:


1. Disinfectants can be applied to both
human body and inanimate objects.
2. By varying the concentration, same
substance can act as a disinfectant or
antiseptic.
Which of these is/are true?
a)
b)
c)
d)

1 only
2 only
Both 1 and 2
None of the above

49. Antifertility drugs are used for birth


control for family planning. How do antifertility drugs work?
a) By suppressing sexual hormone
formation in the body
b) By suppressing ovulation in the body
c) By causing chemical sterilization
d) All of the above
50. Consider the following statements:
1. The Size of the territory of a country
plays an important role in determining
the level of human development.
2. The higher the per capita income the
higher will be the HDI of a nation.
3. In India, among states, higher per
capita income states always perform
better at HDI than other states with a
lower per capita income.
INSIGHTS

Page 9

Facebook Group: Indian Administrative Service (Raz Kr)

b) A and R both are true, and R is the


NOT the correct explanation for A.
c) A is correct, R is incorrect.
d) Both A and R are incorrect

Choose the correct answer using the codes below:


a)
b)
c)
d)

1 and 2
2 and 3
3 only
None of the above

51. Consider the following additives to food.


1. Fat emulsifiers and stabilising agents
2. Antioxidants
3. Preservatives
Which of the above has/have nutritional value?
a)
b)
c)
d)

1 and 2
2 only
1 and 3
None of the above

52. In a federation
1. No more than two autonomous tiers of
government can exist.
2. All tiers derive their authority from
the higher tier and the constitution.
3. Each tier has distinct set of powers and
responsibilities.
Choose the correct answer using the codes below:
a)
b)
c)
d)

1 and 2
2 and 3
1 and 3
3 only

54. Food preservatives prevent spoilage of


food due to microbial growth. Which of
the following can NOT be used as a food
preservative?
1. Sugar
2. Vegetable Oil
3. Sodium Benzoate
Choose the correct answer from the codes below:
a)
b)
c)
d)

55. Consider the following statements about


soaps used in daily life:
1. Shaving soaps contain ethanol to
prevent rapid drying.
2. Soaps containing glycerol are
transparent.
3. Beating tiny air bubbles before
hardening makes soaps float on water.
Choose the correct answer from the codes below:
a)
b)
c)
d)

53. Consider the following statements:


Assertion (A): No artificial sweetener can
be used for cooking (by heating) purposes.
Reason (R): Artificial sweeteners turn toxic
at higher temperatures.
In the context of the statements above, which of
these is true?
a) A and R both are true, and R is the
correct explanation for A.
http://insightsonindia.com

1 and 2
2 and 3
1 and 3
All of the above can be used as
preservatives

1 and 2
2 and 3
1 and 3
3 only

56. Consider the following statements


1. DNA molecules are positively charged
molecules
2. Gel electrophoresis technique is used
in separating the fragmented DNA
molecules
Which of the above statements is/are correct?

INSIGHTS

Page 10

Facebook Group: Indian Administrative Service (Raz Kr)

a)
b)
c)
d)

1 Only
2 Only
Both
None

57. Consider the following statements about


the powers of the Supreme Court of India.
1. Its decisions are binding on all courts.
2. It can transfer judges of High Courts.
3. It can move cases from any court in
India to itself.
4. It can transfer cases from one High
Court to another.
Choose the correct answer using the codes below:
a)
b)
c)
d)

1 and 2 only
2 and 3 only
1, 2 and 3 only
All of the above

58. Sometimes dyes do not get easily


absorbed on the clothe piece. This can be
due to
a) Clothe has been put in sunlight for
long.
b) Clothe has been washed with hard
water.
c) Clothe has been washed with alcohol
and then dried and dyed.
d) Clothe and Dye may not be at the
same temperature
59. Renewable energy can also play an
important role in resolving the energy
crisis in urban areas to a great extent.
Consider the following sources of
renewable energy in India.
1. Bio power
2. Small Hydro power
3. Solar power
4. Wind Power
Arrange the following in increasing order of their
present production in India.
http://insightsonindia.com

a)
b)
c)
d)

2134
1234
2143
1243

60. Consider the following statements about


the North Eastern Region Power System
Improvement Project (NERPSIP) recently
approved by the Government of India.
1. Half of the funding of the project is
from the World Bank.
2. The main aim of the project is to
increase the total power production in
the NE region.
3. It extends to Sikkim and Arunachal
Pradesh also.
Choose the correct answer from the codes below:
a)
b)
c)
d)

1 and 2
2 and 3
1 and 3
1 only

61. Consider the following statements


1. The caves of Ellora and Aurangabad
show the ongoing differences between
the two religionsBuddhism and
Brahmanical
2. Triple storey cave carving is found in
Ajanta whereas it is absent in Ellora
caves
Which of the above statements is/are correct?
a)
b)
c)
d)

1 Only
2 Only
Both
None

62. Which of the following is NOT captured


in the Human Development index (HDI)?
1. Sanitation coverage
2. Assets of a household
3. Women labour participation rate
INSIGHTS

Page 11

Facebook Group: Indian Administrative Service (Raz Kr)

d) All of the above

Choose the correct answer using the codes below:


a)
b)
c)
d)

1 and 2
2 and 3
1 and 3
None is captured

63. As some economists point out, India is on


a threshold of an Investment Recovery
Cycle. Which of the recent decisions of the
GoI would help in it?
1. Increasing FDI caps in defense and
insurance sectors
2. Recent ordinance on coal block
auctions
3. Make in India campaign
Choose the correct answer from the codes below:
a)
b)
c)
d)

1 and 2
2 and 3
1 and 3
All of the above

64. The traditional utensil making among


the Thatheras of Jandiala Guru, Punjab is
being inscribed on the UNESCO
Representative List of
the Intangible Cultural Heritage of
Humanity, 2014. Consider the following
statements about it.
1. The utensil is an alloy of copper, zinc
and tin.
2. The tradition of using the metals is
recommended by the ancient Indian
school of medicine, Ayurveda.
3. Skills of the Thatheras have been
transmitted in written instructions
from one generation to other
generations in Punjab.

65. The Cabinet has recently given its


approval to ratify Beijing Protocol and
introduce a bill in the light of the protocol.
The Beijing protocol concerns
a) Hijacking of planes
b) International Aviation operational
safety standards
c) Safety standards on International
airports
d) International cooperation for better
operation of international flights
66. Consider the following statements
1. Antibodies are produced by Tlymphocytes as part of primary
response to pathogens
2. Acquired immunity is pathogen
specific and is characterised by
memory
Which of the above statements is/are correct?
a)
b)
c)
d)

1 Only
2 Only
Both
None

67. Consider the following statements


1. Restriction endonucleases which cut
DNA at specific site are basically
enzymes
2. Restriction endonucleases are isolated
from bacteria
Which of the above statements is/are correct?
a)
b)
c)
d)

Choose the correct answer from the codes below:

1 Only
2 Only
Both
None

a) 1 and 2
b) 2 and 3
c) 1 and 3
http://insightsonindia.com

INSIGHTS

Page 12

Facebook Group: Indian Administrative Service (Raz Kr)

68. The Constitution (119th Amendment) Bill,


2013 aims to ratify the LBA between India
and Bangladesh under the Indira-Mujib
pact of 1974 to exchange areas and people
on either side of the border. Which of the
following states will be involved in the
land transfer?
1. Assam
2. Nagaland
3. Meghalaya
4. Tripura
Choose the correct answer using the codes below:
a)
b)
c)
d)

All of the above


1 only
1, 3 and 4
2 and 4 only

69. The Prime Minister recently inaugurated


the Hornbill festival in one of the Northeastern states. Consider the following
statements about it.
1. It is a tourism promotional
extravaganza of Nagaland.
2. It is also supported by the Centre as
one of the biggest
indigenous festivals of the country.
3. The festival is organized by the State
Tourism and Art & Culture
Departments.
Choose the correct answer using the codes below:
a)
b)
c)
d)

All of the above


1 and 2
1 and 3
2 and 3

1. Health food
2. Textile industry
3. Biodegradable plastics
Choose the correct answer using the codes below:
a)
b)
c)
d)

71. The RBI recently issued the final


guidelines for the Bharat Bill Payment
System (BBPS). Consider the following
statements about BBPS.
1. No agents are involved in BBPS as it is
a direct customer service.
2. Multiple bills can be paid at single
point of transaction.
3. Banking operations have been
excluded from BBPS.
Choose the correct answer using the codes below:
a)
b)
c)
d)

1 and 2
2 and 3
1 and 3
2 only

72. Consider the following statements about


Mount Aso.
1. It was an active volcano few decades
back, but is an inactive volcano now.
2. It is located in Japan.
3. The explosions in Mount Aso are of
very low intensity (if any).
Choose the correct answer using the codes below:
a)
b)
c)
d)

70. Indias first annual hemp conference was


held in New Delhi at Indian Habitat
Centre (IHC). The aim of the conference
was to provide an opportunity to
international agencies get idea of Indias
hemp agencies. In which of the following
can hemp be used?
http://insightsonindia.com

All of the above


1 and 2
1 and 3
2 and 3

INSIGHTS

1 and 2
2 and 3
1 and 3
2 only

Page 13

Facebook Group: Indian Administrative Service (Raz Kr)

73. The Lok Sabha has recently passed


the Constitution (Scheduled Castes)
Orders (Amendment) Bill, 2014. Which of
the following communities have been
added to the SC list?
1. Pulluvan and Thachar community
from Kerala
2. Dhobi community from Tripura
3. Majhi community in Sikkim

distribution of powers laid down by


the Constitution.
3. The review power extends to the laws
passed by State legislatures also.
Choose the correct answer using the codes below:
a)
b)
c)
d)

Choose the correct answer using the codes below:


a)
b)
c)
d)

1 and 2
2 and 3
1 and 3
All of the above

74. The World Trade Organization (WTO)


approved the first world-wide trade dealTrade Facilitation agreement (TFA).
Which of the following are included in
TFA?
1. Lowering import tariffs and
agricultural subsidies for developing
countries
2. Developed countries would abolish
hard import quotas on agricultural
products from the developing world.
3. Reduction in red tape
at international borders
Choose the correct answer using the codes below:
a)
b)
c)
d)

1 and 2
2 and 3
1 and 3
All of the above

75. Consider the following statements about


Judicial review.
1. The term judicial review is
mentioned nowhere in the
Constitution.
2. Supreme Court can use the review
powers if a law is inconsistent with the
http://insightsonindia.com

1 and 2
2 and 3
1 and 3
All of the above

76. Consider the following statements with


reference to plasmids which are used in
gene cloning
1. It is an independently replicating
extra-chromosomal DNA molecule
2. They are most commonly found in
bacteria as small, circular, doublestranded DNA molecules
Which of the above statements is/are correct?
a)
b)
c)
d)

1 Only
2 Only
Both
None

77. Exotic species can affect the local


ecosystem in which of the following
ways?
1. Compete with native species for
resources
2. Alter the ecosystem to the
disadvantage of other species
3. Cause loss of bio-diversity
Choose the correct answer using the codes
below:
a)
b)
c)
d)

INSIGHTS

1 and 2
2 and 3
1 and 3
All of the above

Page 14

Facebook Group: Indian Administrative Service (Raz Kr)

78. Which of the following are the major


points of contentions at the ongoing
Climate change conference at Lima, Paris?
1. Long-term financing for tackling
climate change
2. Principle of Common But
differentiated Responsibilities
3. Historic emissions issues of developed
countries
Choose the correct answer using the codes
below:
a)
b)
c)
d)

1 and 2
2 and 3
1 and 3
1 only

79. Watershed conservation is one of the


major interventions to fight the shortage
of water in many regions of India. What
else is included in Watershed
Conservation?
1. Forest protection
2. Agricultural best management
practices
3. Soil Conservation
Choose the correct answer using the codes
below:
a)
b)
c)
d)

1 and 2
2 and 3
1 and 3
All of the above

80. Recently Social Justice Bench has been set


up by the Judiciary to hear cases related to
social issues. Consider the following about
it.
1. It has been setup in the Supreme Court
as well as all the High Courts of India.
2. The Bench will dispose off PILs.
3. It will be headed by the Chief Justices
of the respective court(s).
http://insightsonindia.com

Choose the correct answer using the codes


below:
a)
b)
c)
d)

1 and 2
2 and 3
1 and 3
2 only

81. Few countries have recently banned Dirty


Coal. Consider the following statements
about Dirty Coal.
1. It has higher ash content than normal
coking coal.
2. It has lower sulphur content than
normal coking coal.
3. Dirty coal is one of the reasons behind
formation of smog in urban areas.
Choose the correct answer using the codes below:
a)
b)
c)
d)

1 and 2
2 and 3
1 and 3
All of the above

82. The Union Ministry of New and


Renewable Energy (MNRE) recently has
announced a scheme to develop 25 solar
parks across the country. Consider the
following statements.
1. There are no solar parks in India till
date.
2. In the scheme, no solar parks are
proposed in the North-Eastern region
of India.
3. If the scheme is implemented
successfully, the target under the
Jawaharlal Nehru National Solar
Mission will be accomplished before
target year.
Choose the correct answer using the codes below:
a) 1 and 2
b) 3 only
c) 1 and 3
INSIGHTS

Page 15

Facebook Group: Indian Administrative Service (Raz Kr)

d) 2 and 3
83. Tigers are at the apex of the foodweb in a
forest ecosystem. Consider the following
statements about the status of Tigers in
India.
1. As per the latest reports, highest
numbers of tigers have died recently in
the last four years than ever between
any two censuses.
2. There is no policy for protection of
tigers outside reserves from the
Central government.
Which of these is/are true?
a)
b)
c)
d)

1 only
2 only
Both 1 and 2
None

84. Consider the following statements


1. The rock-cut cave carved at Barabar
hills near Gaya in Bihar is known as
the Lomus Rishi cave was patronised
by Ashoka for the Ajivika sect
2. Most of the Buddhist stupas are
constructed over the relics of the
Buddha
Which of the above statements is/are
INCORRECT?
a)
b)
c)
d)

1 Only
2 Only
Both
None

2. Installing giant light reflecting mirrors


on earth
3. Cloud Seeding and cloud whitening
Choose the correct answer using the codes below:
a)
b)
c)
d)

86. Which of the following would be affected


if India does not have an Independent
Judiciary?
1. Rule of Law and Supremacy of Law
2. Enforcement of Fundamental Rights
3. Legitimacy of Indian democracy
Choose the correct answer using the codes below:
a)
b)
c)
d)

1 and 2
2 and 3
1 and 3
All of the above

87. With reference to BOD (biochemical


oxygen demand), consider the following
statements
1. The lesser the BOD of waste water,
more is its polluting potential
2. The sewage water is treated till the
BOD is increased to make it useful for
consumption
Which of the above statements is/are
INCORRECT?
a)
b)
c)
d)

85. Scientists have warned that


geoengineering, an emerging discipline
that seeks to tackle problems related to
climate change, could have catastrophic
consequences for the planets inhabitants.
Which of the following come under geoengineering?
1. Ocean iron fertilization
http://insightsonindia.com

1 and 2
2 and 3
1 and 3
All of the above

INSIGHTS

1 Only
2 Only
Both
None

Page 16

Facebook Group: Indian Administrative Service (Raz Kr)

88. Which of the following is/are a part of an


Independent Judicial system?
1. Judiciary is not accountable to the
legislature.
2. Judicial appointments are not affected
solely by political considerations.
3. The Executive is bound to follow and
implement the decisions of the
Judiciary even if it disagrees with it.

2. Square or rectangular copper tablets,


with an animal or a human figure on
one side and an inscription on the
other, or an inscription on both sides
have also been found
Which of the above statements is/are correct?
a)
b)
c)
d)

Choose the correct answer using the codes below:


a)
b)
c)
d)

1 and 2
2 and 3
1 and 3
All of the above

89. Consider the following statements about


the Judiciary in India.
1. The judiciary is not financially
dependent on either the executive or
legislature.
2. Parliament cannot discuss the conduct
of the judges except when the
proceeding to remove a judge is being
carried out.
3. The judiciary has the power to
penalise those who are found guilty of
contempt of court in case of unfair
criticism of its decisions.
Choose the correct answer using the codes below:
a)
b)
c)
d)

1 and 2
2 and 3
1 and 3
All of the above

90. With reference to seals and tablets found


at Indus Valley Civilization sites, consider
the following statements
1. The standard Harappan seal was a
square plaque 22 square inches,
usually made from mud and then
burnt to harden it
http://insightsonindia.com

1 Only
2 Only
Both
None

91. Which of the following would come under


the Original Jurisdiction of the Supreme
Court of India?
1. Advising the President on matters of
public importance and law.
2. Granting special leave to an appeal
from any judgement or matter passed
by any court in the territory of India.
3. Settling disputes between Union and
States and amongst States.
Choose the correct answer using the codes below:
a)
b)
c)
d)

1 and 2
2 and 3
3 only
1 and 3

92. When ready-made antibodies are directly


given to protect the body against foreign
agents, it is called
a) Active immunity
b) Passive immunity
c) Response immunity
d) None of the above
93. Indian model of federation is closer to that
of which of the following nations/nationstates?
1. USSR
2. Canada
3. USA
INSIGHTS

Page 17

Facebook Group: Indian Administrative Service (Raz Kr)

Choose the correct answer using the codes below:


a)
b)
c)
d)

1 and 2
2 and 3
1 and 3
2 only

94. Which of the following fall in the


Concurrent List under the Seventh
Schedule of the Constitution of India?
1. Forests
2. Education
3. Trade and Commerce
4. Ports
Choose the correct answer using the codes below:
a)
b)
c)
d)

a)
b)
c)
d)

Choose the correct answer using the codes below:


1 and 2
2 and 3
1 and 3
All of the above

Choose the correct answer using the codes below:


a)
b)
c)
d)

1 and 2
2 and 3
1 and 3
All of the above

98. Consider the following statements


1. Computed tomography (CT) uses Xrays to generate a three-dimensional
image of the internals of an object
2. MRI uses strong magnetic fields and
non-ionising radiations to accurately
detect pathological and physiological
changes in the living tissue
3. CT is useful in detecting cancer in
inner organs whereas MRI is not
useful in detecting cancers
Which of the above statements is/are correct?
a)
b)
c)
d)

96. Consider the following statements


1. In Widal test, bacteria causing typhoid
fever are mixed with serum containing
specific antibodies obtained from an
infected individual
2. Bacteria like Streptococcus pneumoniae
and Haemophilus influenza cause lung
diseases in humans

http://insightsonindia.com

1 Only
2 Only
Both
None

97. Under which of the following


circumstances can the Parliament legislate
on the subjects in the State list?
1. Rajya Sabha passing a resolution to
that effect.
2. Financial emergency
3. Presidents Rule.

1 and 2 only
2 and 3 only
1, 2 and 3 only
All of the above

95. Which of the following provisions of the


constitution point towards the Central
bias in the Indian federation?
1. Emergency provisions
2. The Position of Governor
3. Constitution provides Planning
under the Union List.

a)
b)
c)
d)

Which of the above statements is/are correct?

INSIGHTS

1 and 2 Only
1 and 3 Only
2 and 3 Only
All

Page 18

Facebook Group: Indian Administrative Service (Raz Kr)

99. With reference to nagara style of temple


architecture, consider the following
statements
1. It is popular in northern India
2. It does not usually have elaborate
boundary walls or gateways
3. The garbhagriha is always located
directly under the tallest tower
4. The valabhi type the rectangular
buildings with a roof that rises into a
vaulted chamber, is a sub-type of
nagara style
Which of the above statements is/are correct?
a) 1,2 and 3 Only
b) 2,3 and 4 Only
c) 1,3 and 4 Only
d) All

http://insightsonindia.com

100.
Which of the following processes
contribute in an increase of population of
a region?
1. Natality
2. Immigration
3. Emigration
4. Mortality
Choose the answer using the codes below
a)
b)
c)
d)

INSIGHTS

1 and 2 Only
1 and 3 Only
2 and 3 Only
2 Only

Page 19

Facebook Group: Indian Administrative Service (Raz Kr)

Insights Mock tests 2015: test 9 Solutions


1. Solution: c)

As can be clearly seen agriculture had developed and spread long back. It was a not a cause
for the rise in population. Low death rates due to better sanitation facilities; and
industrialization increased economic wealth that led to better living standards and a rise in
population.
2. Solution: b)
The rock shelters on banks of the River Suyal at Lakhudiyar, about twenty kilometres on the
Almora-Barechina road (Uttarkhand), bear these prehistoric paintings. Lakhudiyar literally
means one lakh caves. The paintings here can be divided into three categories: man, animal
and geometric patterns in white, black and red ochre. Humans are represented in stick-like
forms. A long-snouted animal, a fox and a multiple legged lizard are the main animal
motifs. Wavy lines, rectangle-filled geometric designs, and groups of dots can also be seen
here. One of the interesting scenes depicted here is of hand-linked dancing human figures.
There is some superimposition of paintings. The earliest are in black; over these are red
ochre paintings and the last group comprises white paintings.
The richest paintings are reported from the Vindhya ranges of Madhya Pradesh and their
Kaimurean extensions into Uttar Pradesh. These hill ranges are full of Palaeolithic and
Mesolithic remains, and they are also full of forests, wild plants, fruits, streams and creeks,
thus a perfect place for Stone Age people to live. Among these the largest and most
spectacular rock-shelter is located in the Vindhya hills at Bhimbetka in Madhya Pradesh.
Bhimbetka is located fortyfive kilometres south of Bhopal, in an area of ten square
kilometres, having about eight hundred rock shelters, five hundred of which bear paintings.
3. Solution: d)
An important characteristic of all communities is that composition and structure constantly
change in response to the changing environmental conditions. This change is orderly and
sequential, parallel with the changes in the physical environment. These changes lead finally
http://insightsonindia.com

INSIGHTS

Page 1

Facebook Group: Indian Administrative Service (Raz Kr)

Insights Mock tests 2015: test 9 Solutions


to a community that is in near equilibrium with the environment and that is called a climax
community.
The gradual and fairly predictable change in the species composition of a given area is called
ecological succession. During succession some species colonise an area and their populations
become more numerous, whereas populations of other species decline and even disappear.
The entire sequence of communities that successively change in a given area are called
sere(s).
The individual transitional communities are termed seral stages or seral communities. In the
successive seral stages there is a change in the diversity of species of organisms, increase in
the number of species and organisms as well as an increase in the total biomass. The present
day communities in the world have come to be because of succession that has occurred over
millions of years since life started on earth.
Actually succession and evolution would have been parallel processes at that time.
Succession is hence a process that starts where no living organisms are there these could be
areas where no living organisms ever existed, say bare rock; or in areas that somehow, lost
all the living organisms that existed there. The former is called primary succession, while the
latter is termed secondary succession.
Pioneer species:
The species that invade a bare area are called pioneer species. In primary succession on
rocks these are usually lichens which are able to secrete acids to dissolve rock, helping in
weathering and soil formation. These later pave way to some very small plants like
bryophytes, which are able to take hold in the small amount of soil. They are, with time,
succeeded by bigger plants, and after several more stages, ultimately a stable climax forest
community is formed. The climax community remains stable as long as the environment
remains unchanged. With time the xerophytic habitat gets converted into a mesophytic one.
4. Solution: a)
Self-explanatory. As the level of prosperity increases among nations; an increase in
awareness and a desire to have lesser off-springs leads to a lower growth rate of population.
5. Solution: c)
Narrow top means high death rates as the base is broad (which means the birth rates are
high). High death rate and high birth rate can be easily correlated with the demography of a
developing country. It leads to an increase in the population.
6. Solution: c)
The two major sites of the Indus Valley Civilisation, along the Indus riverthe cities of
Harappa in the north and Mohenjodaro in the southshowcase one of earliest examples of
civic planning. Other markers were houses, markets, storage facilities, offices, public baths,
etc., arranged in a grid-like pattern. There was also a highly developed drainage system.
http://insightsonindia.com

INSIGHTS

Page 2

Facebook Group: Indian Administrative Service (Raz Kr)

Insights Mock tests 2015: test 9 Solutions


While Harappa and Mohenjodaro are situated in Pakistan, the important sites excavated in
India are Lothal and Dholavira in Gujarat, Rakhigarhi in Haryana, Ropar in the Punjab,
Kalibangan and Balathal in Rajasthan, etc.
7. Solution: d)
The division of population into rural and urban is based on the residence. This division is
necessary because rural and urban life styles differ from each other in terms of their
livelihood and social conditions. The age-sex-occupational structure, density of population
(higher in urban) and level of development vary between rural and urban areas.
The criteria for differentiating rural and urban population varies from country to country. In
general terms rural areas are those where people are engaged in primary activities and
urban areas are those when majority of the working population is engaged in non-primary
activities.
8. Solution: d)
Just as any building is supported by pillars, the idea of human development is supported by
the concepts of equity, sustainability, productivity and empowerment.
Equity refers to making equal access to opportunities available to everybody. The
opportunities available to people must be equal irrespective of their gender, race, income
and in the Indian case, caste.
Sustainability means continuity in the availability of opportunities. To have sustainable
human development, each generation must have the same opportunities.
Empowerment means to have the power to make choices. Such power comes from
increasing freedom and capability. Good governance and people-oriented policies are
required to empower people. The empowerment of socially and economically
disadvantaged groups is of special importance.
9. Solution: d)
The Lion Capital discovered more than a hundred years ago at Sarnath, near Varanasi, is
generally referred to as Sarnath Lion Capital. This is one of the finest examples of sculpture
from the Mauryan period. Built in commemoration of the historical event of the first sermon
or the Dhammachakrapravartana by the Buddha at Sarnath, the capital was built by Ashoka.
The capital originally consisted of five component parts: (i) the shaft (which is broken in
many parts now), (ii) a lotus bell base, (iii) a drum on the bell base with four animals
proceeding clockwise, (iv) the figures of four majestic addorsed lions, and (v) the crowning
element, Dharamchakra, a large wheel, was also a part of this pillar. However, this wheel is
lying in a broken condition and is displayed in the site museum at Sarnath. The capital
without the crowning wheel and the lotus base has been adopted as the National Emblem of
Independent India.

http://insightsonindia.com

INSIGHTS

Page 3

Facebook Group: Indian Administrative Service (Raz Kr)

Insights Mock tests 2015: test 9 Solutions

10. Solution: d)
http://www.gnhbhutan.org/about/
11. Solution: d)
Refer to the section JUDICIARY AND PARLIAMENT 11the NCERT Polity for a detailed
discussion on the Basic Structure Doctrine and its evolution.
12. Solution: c)
http://en.wikipedia.org/wiki/Carrying_capacity
Below carrying capacity, populations typically increase, while above, they typically
decrease. A factor that keeps population size at equilibrium is known as a regulating factor.
Population size decreases above carrying capacity due to a range of factors depending on
thespecies concerned, but can include insufficient space, food supply, or sunlight. The
carrying capacity of an environment may vary for different species and may change over
time due to a variety of factors, including: food availability, water supply, environmental
conditions and living space.
http://science.howstuffworks.com/environmental/green-science/earth-carryingcapacity1.htm

http://insightsonindia.com

INSIGHTS

Page 4

Facebook Group: Indian Administrative Service (Raz Kr)

Insights Mock tests 2015: test 9 Solutions


13. Solution: a)
As many as 32 countries record low levels of human development. A large proportion of
these are small countries which have been going through political turmoil and social
instability in the form of civil war, famine or a high incidence of diseases. There is an urgent
need to address the human development requirements of this group through well thought
out policies.
International comparisons of human development can show some very interesting results.
Often people tend to blame low levels of human development on the culture of the people.
For example, X country has lower human development because its people follow Y religion,
or belong to Z community. Such statements are misleading.
To understand why a particular region keeps reporting low or high levels of human
development it is important to look at the pattern of government expenditure on the social
sector. The political environment of the country and the amount of freedom people have is
also important. Countries with high levels of human development invest more in the social
sectors and are generally free from political turmoil and instability. Distribution of the
countrys resources is also far more equitable.
14. Solution: d)
Mixed Farming is found in the highly developed parts of the world, e.g. North-western
Europe, Eastern North America, parts of Eurasia and the temperate latitudes of Southern
continents.
Mixed farms are moderate in size and usually the crops associated with it are wheat, barley,
oats, rye, maize, fodder and root crops.
Fodder crops are an important component of mixed farming. Crop rotation and
intercropping play an important role in maintaining soil fertility. Equal emphasis is laid on
crop cultivation and animal husbandry. Animals like cattle, sheep, pigs and poultry provide
the main income along with crops.
Mixed farming is characterised by high capital expenditure on farm machinery and
building, extensive use of chemical fertilisers and green manures and also by the skill and
expertise of the farmers.
15. Solution: b)
Truck farming is a horticultural practice of growing one or more vegetable crops on a large
scale for shipment to distant markets. It is usually less intensive and diversified than market
gardening. At first this type of farming depended entirely on local or regional markets. As
the use of railroads and large-capacity trucks expanded and refrigerated carriers were
introduce.

http://insightsonindia.com

INSIGHTS

Page 5

Facebook Group: Indian Administrative Service (Raz Kr)

Insights Mock tests 2015: test 9 Solutions


16. Solution: d)
The first century CE onwards, Gandhara (now in Pakistan), Mathura in northern India and
Vengi in Andhra Pradesh emerged as important centres of art production. Buddha in the
symbolic form got a human form in Mathura and Gandhara. The sculptural tradition in
Gandhara had the confluence of Bactria, Parthia and the local Gandhara tradition. The local
sculptural tradition at Mathura became so strong that the tradition spread to other parts of
northern India. The best example in this regard is the stupa sculptures found at Sanghol in
the Punjab. The Buddha image at Mathura is modelled on the lines of earlier Yaksha images
whereas in Gandhara it has Hellenistic features.
Images of Vaishnava (mainly Vishnu and his various forms) and Shaiva (mainly the lingas
and mukhalingas) faiths are also found at Mathura but Buddhist images are found in large
numbers. It may be noted that the images of Vishnu and Shiva are represented by their
ayudhas (weapons). There is boldness in carving the large images, the volume of the images
is projected out of the picture plane, the faces are round and smiling, heaviness in the
sculptural volume is reduced to relaxed flesh. The garments of the body are clearly visible
and they cover the left shoulder. Images of the Buddha, Yakshas, Yakshinis, Shaivite and
Vaishnavite deities and portrait statues are profusely sculpted. In the second century CE,
images in Mathura get sensual, rotundity increases, they become fleshier. In the third
century CE, treatment of sculptural volume changes by reducing the extreme fleshiness,
movement in the posture is shown by increasing distance.
between the two legs as well as by using bents in the body posture. Softness in the surface
continues to get refined. The trend continues in the fourth century CE but in the late fourth
century CE, the massiveness and fleshiness is reduced further and the flesh becomes more
tightened, the volume of the drapery also gets reduced and in the fifth and sixth centuries
CE, the drapery is integrated into the sculptural mass. Transparent quality in the robes of
the Buddha images is evident. In this period, two important schools of sculptures in
northern India are worth noting. The traditional centre, Mathura, remained the main art
production site whereas Sarnath and Kosambi also emerged as important centres of art
production. Many Buddha images in Sarnath have plain transparent drapery covering both
shoulders, and the halo around the head has very little ornamentation whereas the Mathura
Buddha images continue to depict folds of the drapery in the Buddha images and the halo
around the head is profusely decorated. One can visit museums at Mathura, Sarnath,
Varanasi, New Delhi, Chennai, Amaravati, etc. to study the features of early sculptures.
17. Solution: a)
Decision-making
Cooperatives: Most cooperatives require simple-majority to pass a vote. That is, only over
50% of workers have to approve a motion for it to be passed.
Collectives: Collectives, however, use a consensus model: this means that every worker
must vote yes in order to make a decision.

http://insightsonindia.com

INSIGHTS

Page 6

Facebook Group: Indian Administrative Service (Raz Kr)

Insights Mock tests 2015: test 9 Solutions


Hierarchies
While both cooperatives and collectives reject the hierarchical structure of typical
corporations (top-down, in which owners have authority over workers), the two take
different approaches to their structures.
Cooperatives: Every worker in a cooperative is also an owner. However, in some larger
cooperatives, there exists a form of hierarchy. This might mean that there is a president
(elected by the workers), a board of directors (also elected), and so on. However, all major
decisions are still made by votes taken in worker assemblies. Yet, most of the time, the
smaller the cooperative is the less formal its hierarchy will be if it has one at all.
Collectives: Collectives tend to strive to abolish any form of hierarchy. This means that no
one worker in the collective has any more authority than another worker.
18. Solution: a)
Raw material used by industries should be cheap and easy to transport. Industries based on
cheap, bulky and weight-losing material (ores) are located close to the sources of raw
material such as steel, sugar, and cement industries. Perishability is a vital factor for the
industry to be located closer to the source of the raw material. Agro-processing and dairy
products are processed close to the sources of farm produce or milk supply respectively.
Processing cost will be the same whether it is processed at the site of off the site.
19. Solution: a)
Foot loose industries can be located in a wide variety of places. They are not dependent on
any specific raw material, weight losing or otherwise. They largely depend on component
parts which can be obtained anywhere. They produce in small quantity and also employ a
small labour force. These are generally not polluting industries. The important factor in their
location is accessibility by road network.
20. Solution: d)
Agro based Industries
Agro processing involves the processing of raw materials from the field and the farm into
finished products for rural and urban markets. Major agro-processing industries are food
processing, sugar, pickles, fruits juices, beverages (tea, coffee and cocoa), spices and oils fats
and textiles (cotton, jute, silk), rubber, etc.
Agri-business is commercial farming on an industrial scale often financed by business
whose main interests lie outside agriculture, for example, large corporations in tea
plantation business. Agri-business farms are mechanised, large in size, highly structured,
reliant on chemicals, and may be described as agro-factories.

http://insightsonindia.com

INSIGHTS

Page 7

Facebook Group: Indian Administrative Service (Raz Kr)

Insights Mock tests 2015: test 9 Solutions


21. Solution: d)
The highest level of decision makers or policy makers perform quinary activities. These are
subtly different from the knowledge based industries that the quinary sector in general deals
with.
Quinary activities are services that focus on the creation, re-arrangement and interpretation
of new and existing ideas; data interpretation and the use and evaluation of new
technologies. Often referred to as gold collar professions, they represent another
subdivision of the tertiary sector representing special and highly paid skills of senior
business executives, government officials, research scientists, financial and legal consultants,
etc. Their importance in the structure of advanced economies far outweighs their numbers.
22. Solution: a)
Typhoid fever also known simply as typhoid is a common worldwide bacterial disease
transmitted by the ingestion of food or water contaminated with the feces of an infected
person, which contain the bacterium Salmonella enterica subsp. enterica,serovar Typhi.
The disease has received various names, such as gastric fever, enteric fever, abdominal
typhus, infantile remittant fever, slow fever, nervous fever and pythogenic fever. The
name typhoid means "resembling typhus" and comes from the neuropsychiatric symptoms
common to typhoid and typhus. Despite this similarity of their names, typhoid fever and
typhus are distinct diseases and are caused by different species of bacteria.
23.

Solution: c)

Commensalism: This is the interaction in which one species benefits and the other is neither
harmed nor benefited. An orchid growing as an epiphyte on a mango branch, and barnacles
growing on the back of a whale benefit while neither the mango tree nor the whale derives
any apparent benefit. The cattle egret and grazing cattle in close association, a sight you are
most likely to catch if you live in farmed rural areas, is a classic example of commensalism.
The egrets always forage close to where the cattle are grazing because the cattle, as they
move, stir up and flush out from the vegetation insects that otherwise might be difficult for
the egrets to find and catch. Another example of commensalism is the interaction between
sea anemone that has stinging tentacles and the clown fish that lives among them. The fish
gets protection from predators which stay away from the stinging tentacles. The anemone
does not appear to derive any benefit by hosting the clown fish.
24. Solution: b)
Outsourcing or contracting out is giving work to an outside agency to improve efficiency
and reduce costs. When outsourcing involves transferring work to overseas locations, it is
described by the term off - shoring, although both off - shoring and outsourcing are used
together. Business activities that are outsourced include information technology (IT), human
resources, customer support and call centre services and at times also manufacturing and
engineering.
http://insightsonindia.com

INSIGHTS

Page 8

Facebook Group: Indian Administrative Service (Raz Kr)

Insights Mock tests 2015: test 9 Solutions


Data processing is an IT related service easily be carried out in Asian, East European and
African countries, In these countries IT skilled staff with good English language skills are
available at lower wages than those in the developed countries. Thus, a company in
Hyderabad or Manila does work on a project based on GIS techniques for a country like
U.S.A or Japan. Overhead costs are also much lower making it profitable to get job-work
carried out overseas, whether it is in India, China or even a less populous country like
Botswana in Africa.
25. Solution: a)
In Russia, a dense highway network is developed in the industrialised region west of the
Urals with Moscow as the hub. The important Moscow-Vladivostok Highway serves the
region to the east. Due to the vast geographical area, highways in Russia are not as
important as railways.
26. Solution: a)
The MediterraneanIndian Ocean Route is a trade route that connects the highly
industrialised Western European region with West Africa, South Africa, South-east Asia and
the commercial agriculture and livestock economies of Australia and New Zealand. Before
the construction of the Suez Canal this was the route connecting Liverpool and Colombo
which was 6,400 km longer than the Suez Canal route. The volume of trade and traffic
between both East and West Africa is on the increase due to the development of the rich
natural resources such as gold, diamond, copper, tin, groundnut, oil palm, coffee and fruits.
27. Solution: d)
http://en.wikipedia.org/wiki/Inland_waterways_of_India
The significance of rivers as inland waterways for domestic and international transport and
trade has been recognised throughout the developed world. Despite inherent limitations,
many rivers have been modified to enhance their navigability by dredging, stabilising river
banks, and building dams and barrages for regulating the flow of water.
28. Solution: a)
Silk Road

More than 2,000 years ago trade passed through Silk Road between Europe and
China, through India

New route to cover China to South Asia, Central Asia and Europe

Infrastructural development to boost regional connectivity

Maritime connectivity

Ecological cooperation

http://insightsonindia.com

INSIGHTS

Page 9

Facebook Group: Indian Administrative Service (Raz Kr)

Insights Mock tests 2015: test 9 Solutions

Establishing special economic zones and industrial parks in the areas connected by
Silk Road

Routes - Yunnan (South western province China) - Myanmar - Kolkata - "Silk Road
economic belt." The BCIM will play a key role in this economic corridor.

South-eastern Chinese province Fujian to Chennai through littoral countries in the


region - Maritime Silk Road

Funding expected to come by BRICS New Development Bank and proposed Asian
Infrastructural Investment bank

29. Solution: a)
International trade is the result of specialisation in production. It benefits the world
economy if different countries practise specialisation and division of labour in the
production of commodities or provision of services. Each kind of specialisation can give rise
to trade. Thus, international trade is based on the principle of comparative advantage,
complimentarity and transferability of goods and services and in principle, should be
mutually beneficial to the trading partners. In modern times, trade is the basis of the worlds
economic organisation and is related to the foreign policy of nations. With well developed
transportation and communication systems, no country is willing to forego the benefits
derived from participation in international trade.
The principle of Sovereignty is in fact eroded by international trade.
30. Solution: d)
Look for the explanation in the next question.
31. Solution: d)
Age structure represents the number of people of different age groups. This is an important
indicator of population composition, since a large size of population in the age group of 1559 indicates a large working population. A greater proportion of population above 60
years represents an ageing population which requires more expenditure on health care
facilities. Similarly high proportion of young population would mean that the region has a
high birth rate and the population is youthful.
32. Solution: a)
The basic form of the Hindu temple comprises the following: (i) a cave-like sanctum
(garbhagriha literally womb-house), which, in the early temples, was a small cubicle with a
single entrance and grew into a larger chamber in time. The garbhagriha is made to house
the main icon which is itself the focus of much ritual attention; (ii) the entrance to the temple
which may be a portico or colonnaded hall that incorporates space for a large number of
worshippers and is known as a mandapa; (iii) from the fifth century CE onwards,
freestanding temples tend to have a mountain- like spire, which can take the shape of a
http://insightsonindia.com

INSIGHTS

Page 10

Facebook Group: Indian Administrative Service (Raz Kr)

Insights Mock tests 2015: test 9 Solutions


curving shikhar in North India and a pyramidal tower, called a vimana, in South India; (iv)
the vahan, i.e., the mount or vehicle of the temples main deity along with a standard pillar
or dhvaj is placed axially before the sanctum. Two broad orders of temples in the country
are known Nagara in the north and Dravida in the south. At times, the Vesar style of
temples as an independent style created through the selective mixing of the Nagara and
Dravida orders is mentioned by some scholars.
33. Solution: c)

34. Solution: d)
Population ageing is the process by which the share of the older population becomes
proportionally larger. This is a new phenomenon of the twentieth century. In most of the
developed countries of the world, population in higher age groups has increased due to
increased life expectancy. With a reduction in birth rates, the proportion of children in the
population has declined.
35. Solution: a)
Most-favoured-nation (MFN): treating other people equally - Under the WTO agreements,
countries cannot normally discriminate between their trading partners. Grant someone a
http://insightsonindia.com

INSIGHTS

Page 11

Facebook Group: Indian Administrative Service (Raz Kr)

Insights Mock tests 2015: test 9 Solutions


special favour (such as a lower customs duty rate for one of their products) and you have to
do the same for all other WTO members.
This principle is known as most-favoured-nation (MFN) treatment. It is so important that it
is the first article of the General Agreement on Tariffs and Trade (GATT), which governs
trade in goods. MFN is also a priority in the General Agreement on Trade in Services
(GATS) (Article 2) and the Agreement on Trade-Related Aspects of Intellectual Property
Rights (TRIPS) (Article 4), although in each agreement the principle is handled slightly
differently. Together, those three agreements cover all three main areas of trade handled by
the WTO.
Some exceptions are allowed. For example, countries can set up a free trade agreement that
applies only to goods traded within the group discriminating against goods from
outside. Or they can give developing countries special access to their markets. Or a country
can raise barriers against products that are considered to be traded unfairly from specific
countries. And in services, countries are allowed, in limited circumstances, to discriminate.
But the agreements only permit these exceptions under strict conditions. In general, MFN
means that every time a country lowers a trade barrier or opens up a market, it has to do so
for the same goods or services from all its trading partners whether rich or poor, weak or
strong.
36. Solution: c)
The practice of selling a commodity in two countries at a price that differs for reasons not
related to costs is called dumping.
In international trade, the export by a country or company of a product at a price that is
lower in the foreign market than the price charged in the domestic market. As dumping
usually involves substantial export volumes of the product, it often has the effect of
endangering the financial viability of manufacturers or producers of the product in the
importing nation.
37. Solution: d)
WTO is the only international organisation dealing with the global rules of trade between
nations. It sets the rules for the global trading system and resolves disputes between its
member nations. WTO also covers trade in services, such as telecommunication and
banking, and others issues such as intellectual rights.
The WTO has however been criticised and opposed by those who are worried about the
effects of free trade and economic globalisation.
38. Solution: a)
World Health Organisation (WHO) suggests that, among other things, a healthy city must
have:

A Clean and Safe environment.

http://insightsonindia.com

INSIGHTS

Page 12

Facebook Group: Indian Administrative Service (Raz Kr)

Insights Mock tests 2015: test 9 Solutions

39.

Meets the Basic Needs of All its inhabitants.


Involves the Community in local government.
Provides easily accessible Health service
Solution: b)

It's the acid in the ant's bite that cause the itch and sometimes more severe reactions.
Applying an antacid neutralizes the harmful acid effects.
Over production of acid in the stomach causes irritation and pain. In severe cases, ulcers are
developed in the stomach. Until 1970, only treatment for acidity was administration of
antacids, such as sodium hydrogencarbonate or a mixture of aluminium and magnesium
hydroxide. However, excessive hydrogencarbonate can make the stomach alkaline and
trigger the production of even more acid. Metal hydroxides are better alternatives because of
being insoluble, these do not increase the pH above neutrality. These treatments control only
symptoms, and not the cause. Therefore, with these metal salts, the patients cannot be
treated easily. In advanced stages, ulcers become life threatening and its only treatment is
removal of the affected part of the stomach.
40. Solution: a)
Histamine is a potent vasodilator. It has various functions. It contracts the smooth muscles
in the bronchi and gut and relaxes other muscles, such as those in the walls of fine blood
vessels. Histamine is also responsible for the nasal congestion associated with common cold
and allergic response to pollen. Synthetic drugs, brompheniramine (Dimetapp) and
terfenadine (Seldane), act as antihistamines. They interfere with the natural action of
histamine by competing with histamine for binding sites of receptor where histamine exerts
its effect.
41. Solution: d)
http://www.britannica.com/EBchecked/topic/602263/tranquilizer
42. Solution: d)
The chief instrument through which judicial activism has flourished in India is Public
Interest Litigation (PIL) or Social Action Litigation (SAL).
In normal course of law, an individual can approach the courts only if he/she has been
personally aggrieved. That is to say, a person whose rights have been violated, or who is
involved in a dispute, could move the court of law. This concept underwent a change
around 1979. In 1979, the Court set the trend when it decided to hear a case where the case
was filed not by the aggrieved persons but by others on their behalf. As this case involved a
consideration of an issue of public interest, it and such other cases came to be known as
public interest litigations. Around the same time, the Supreme Court also took up the case
about rights of prisoners.
This opened the gates for large number of cases where public spirited citizens and voluntary
organisations sought judicial intervention for protection of existing rights, betterment of life
http://insightsonindia.com

INSIGHTS

Page 13

Facebook Group: Indian Administrative Service (Raz Kr)

Insights Mock tests 2015: test 9 Solutions


conditions of the poor, protection of the environment, and many other issues in the interest
of the public. PIL has become the most important vehicle of judicial activism. Judiciary,
which is an institution that traditionally confined to responding to cases brought before it,
began considering many cases merely on the basis of newspaper reports and postal
complaints received by the court. Therefore, the term judicial activism became the more
popular description of the role of the judiciary.
43. Solution: b)
Chikungunya is an infection caused by the Chikungunya virus. It features sudden onset
fever usually lasting two to seven days, and joint pains typically lasting weeks or months
but sometimes years.
The virus is passed to humans by two species of mosquito of the genus Aedes: A.
albopictus and A. aegypti. The strain of chikungunya spreading to the US from the Caribbean
is most easily spread by A. aegypti. Concern exists that this strain of chikungunya could
mutate to make the A. albopictus vector more efficient. If this mutation were to occur,
chikungunya would be more of a public health concern to the US because the A. albopictus or
Asian tiger mosquito is more widespread in the US and is more aggressive than the A.
aegypti. Animal reservoirs of the virus include monkeys, birds, cattle, and rodents. This is in
contrast to dengue, for which only humans and primates are hosts.
44. Solution: d)
Analgesics reduce or abolish pain without causing impairment of consciousness, mental
confusion, incoordination or paralysis or some other disturbances of nervous system. These
are classified as follows:
(i) Non-narcotic (non-addictive) analgesics
(ii) Narcotic drugs
(i) Non-narcotic (non-addictive) analgesics: Aspirin and paracetamol belong to the class of nonnarcotic analgesics. Aspirin is the most familiar example. Aspirin inhibits the synthesis of
chemicals known as prostaglandins which stimulate inflammation in the tissue and cause
pain.
(ii) Narcotic analgesics: Morphine and many of its homologues, when administered in
medicinal doses, relieve pain and produce sleep. In poisonous doses, these produce stupor,
coma, convulsions and ultimately death. Morphine narcotics are sometimes referred to as
opiates, since they are obtained from the opium poppy. These analgesics are chiefly used for
the relief of postoperative pain, cardiac pain and pains of terminal cancer, and in child birth.
45. Solution: a)
http://en.wikipedia.org/wiki/Antibiotic_resistance#Cause

http://insightsonindia.com

INSIGHTS

Page 14

Facebook Group: Indian Administrative Service (Raz Kr)

Insights Mock tests 2015: test 9 Solutions


Natural selection is basically the Darwinian Principle. The bacteria that do not die after the
application of anti-biotics breed among themselves and form huge colonies later.
46. Solution: c)
http://www.cbd.int/abs/
http://en.wikipedia.org/wiki/Bioprospecting#Convention_on_Biological_Diversity_.28CB
D.29
Biopiracy is the term used to refer to the use of bio-resources by multinational companies
and other organisations without proper authorisation from the countries and people
concerned without compensatory payment. Most of the industrialised nations are rich
financially but poor in biodiversity and traditional knowledge. In contrast the developing
and the underdeveloped world is rich in biodiversity and traditional knowledge related to
bio-resources. Traditional knowledge related to bio-resources can be exploited to develop
modern applications and can also be used to save time, effort and expenditure during their
commercialisation.
47. Solution: a)
The removal of judges of the Supreme Court and the High Courts is also extremely difficult.
A judge of the Supreme Court or High Court can be removed only on the ground of proven
misbehaviour or incapacity. A motion containing the charges against the judge must be
approved by special majority in both Houses of the Parliament.
It is clear from this procedure that removal of a judge is a very difficult procedure and
unless there is a general consensus among Members of the Parliament, a judge cannot be
removed. It should also be noted that while in making appointments, the executive plays a
crucial role; the legislature has the powers of removal. This has ensured both balance of
power and independence of the judiciary. So far, only one case of removal of a judge of the
Supreme Court came up for consideration before the Parliament. In that case, though the
motion got two-thirds majority, it did not have the support of the majority of the total
strength of the House and therefore, the judge was not removed.
48. Solution: b)
Antiseptics and disinfectants are also the chemicals which either kill or prevent the growth
of microorganisms. Antiseptics are applied to the living tissues such as wounds, cuts, ulcers
and diseased skin surfaces. Examples are furacine, soframicine, etc. These are not ingested
like antibiotics. Commonly used antiseptic, dettol is a mixture of chloroxylenol and
terpineol. Bithionol (the compound is also called bithional) is added to soaps to impart
antiseptic properties. Iodine is a powerful antiseptic. Its 2-3 per cent solution in alcohol
water mixture is known as tincture of iodine. It is applied on wounds. Iodoform is also used
as an antiseptic for wounds. Boric acid in dilute aqueous solution is weak antiseptic for eyes.
Disinfectants are applied to inanimate objects such as floors, drainage system, instruments,
etc. Same substances can act as an antiseptic as well as disinfectant by varying the
http://insightsonindia.com

INSIGHTS

Page 15

Facebook Group: Indian Administrative Service (Raz Kr)

Insights Mock tests 2015: test 9 Solutions


concentration. For example, 0.2 per cent solution of phenol is an antiseptic while its one
percent solution is disinfectant. Chlorine in the concentration of 0.2 to 0.4 ppm in aqueous
solution and sulphur dioxide in very low concentrations, are disinfectants.
49. Solution: b)
Antibiotic revolution has provided long and healthy life to people. The life expectancy has
almost doubled. The increased population has caused many social problems in terms of food
resources, environmental issues, employment, etc. To control these problems, population is
required to be controlled. This has lead to the concept of family planning. Antifertility drugs
are of use in this direction. Birth control pills essentially contain a mixture of synthetic
estrogen and progesterone derivatives. Both of these compounds are hormones. It is known
that progesterone suppresses ovulation. Synthetic progesterone derivatives are more potent
than progesterone. Norethindrone is an example of synthetic progesterone derivative most
widely used as antifertility drug. The estrogen derivative which is used in combination with
progesterone derivative is ethynylestradiol (novestrol).
50. Solution: d)
International comparisons of human development are interesting. Size of the territory and
per capita income are not directly related to human development. Often smaller countries
have done better than larger ones in human development. Similarly, relatively poorer
nations have been ranked higher than richer neighbours in terms of human development.
For example, Sri Lanka, Trinidad and Tobago have a higher rank than India in the human
development index despite having smaller economies. Similarly, within India, Kerala
performs much better than Punjab and Gujarat in human development despite having lower
per capita income.
51. Solution: d)
Chemicals are added to food for (i) their preservation, (ii) enhancing their appeal, and (iii)
adding nutritive value in them. Main categories of food additives are as follows:
(i) Food colours
(ii) Flavours and sweeteners
(iii) Fat emulsifiers and stabilising agents
(iv) Flour improvers - antistaling agents and bleaches
(v) Antioxidants
(vi) Preservatives
(vii) Nutritional supplements such as minerals, vitamins and amino acids.
Except for chemicals of category (vii), none of the above additives have nutritive value.
These are added either to increase the shelf life of stored food or for cosmetic purposes.
http://insightsonindia.com

INSIGHTS

Page 16

Facebook Group: Indian Administrative Service (Raz Kr)

Insights Mock tests 2015: test 9 Solutions


52. Solution: d)
Federalism does not consist of a set of fixed principles, which are applied, to different
historical situations. Rather, federalism as a principle of government has evolved differently
in different situations. American federalism one of the first major attempts to build a
federal polity is different from German or Indian federalism. But there are also a few key
ideas and concepts associated with federalism.

Essentially, federalism is an institutional mechanism to accommodate two sets of


politiesone at the regional level and the other at the national level. Each
government is autonomous in its own sphere. In some federal countries, there is
even a system of dual citizenship. India has only a single citizenship.
The people likewise, have two sets of identities and loyaltiesthey belong to the
region as well as the nation, for example we are Gujaratis or Jharkhandis as well as
Indians. Each level of the polity has distinct powers and responsibilities and has a
separate system of government.
The details of this dual system of government are generally spelt out in a written
constitution, which is considered to be supreme and which is also the source of the
power of both sets of government. Certain subjects, which concern the nation as a
whole, for example, defence or currency, are the responsibility of the union or central
government. Regional or local matters are the responsibility of the regional or State
government.
To prevent conflicts between the centre and the State, there is an independent judiciary
to settle disputes. The judiciary has the powers to resolve disputes between the central
government and the States on legal matters about the division of power.
53. Solution: d)
Aspartame is the most successful and widely used artificial sweetener. It is roughly 100
times as sweet as cane sugar. It is methyl ester of dipeptide formed from aspartic acid and
phenylalanine. Use of aspartame is limited to cold foods and soft drinks because it is
unstable at cooking temperature.
Alitame is high potency sweetener, although it is more stable than aspartame, the control of
sweetness of food is difficult while using it.
Sucrolose is trichloro derivative of sucrose. Its appearance and taste are like sugar. It is
stable at cooking temperature. It does not provide calories.

54. Solution: d)
Food preservatives prevent spoilage of food due to microbial growth. The most commonly
used preservatives include table salt, sugar, vegetable oils and sodium benzoate,
C6H5COONa. Sodium benzoate is used in limited quantities and is metabolised in the body.
Salts of sorbic acid and propanoic acid are also used as preservatives.

http://insightsonindia.com

INSIGHTS

Page 17

Facebook Group: Indian Administrative Service (Raz Kr)

Insights Mock tests 2015: test 9 Solutions

55. Solution: d)
Basically all soaps are made by boiling fats or oils with suitable soluble hydroxide.
Variations are made by using different raw materials.
Toilet soaps are prepared by using better grades of fats and oils and care is taken to remove
excess alkali. Colour and perfumes are added to make these more attractive.
Soaps that float in water are made by beating tiny air bubbles before their hardening.
Transparent soaps are made by dissolving the soap in ethanol and then evaporating the excess
solvent.
In medicated soaps, substances of medicinal value are added. In some soaps, deodorants are
added. Shaving soaps contain glycerol to prevent rapid drying. A gum called, rosin is added
while making them.
It forms sodium rosinate which lathers well. Laundry soaps contain fillers like sodium
rosinate, sodium silicate, borax and sodium carbonate.

56. Solution: b)
The cutting of DNA by restriction endonucleases results in the fragmentes of DNA. These
fragments can be separated by a technique known as gel electrophoresis. Since DNA
fragments are negatively charged molecules they can be separated by forcing them to move
towards the anode under an electric field through a medium/matrix. Nowadays the most
commonly used matrix is agarose which is a natural polymer extracted from sea weeds. The
DNA fragments separate (resolve) according to their size through sieving effect provided by
the agarose gel. Hence, the smaller the fragment size, the farther it moves.

57. Solution: d)
The Constitution of India provides for a single integrated judicial system. This means that
unlike some other federal countries of the world, India does not have separate State courts.
The structure of the judiciary in India is pyramidal with the Supreme Court at the top, High
Courts below them and district and subordinate courts at the lowest level (see the diagram
below). The lower courts function under the direct superintendence of the higher courts.

http://insightsonindia.com

INSIGHTS

Page 18

Facebook Group: Indian Administrative Service (Raz Kr)

Insights Mock tests 2015: test 9 Solutions

58. Solution: b)
Hard water contains calcium and magnesium ions. These ions form insoluble calcium and
magnesium soaps respectively when sodium or potassium soaps are dissolved in hard
water.
These insoluble soaps separate as scum in water and are useless as cleansing agent. In fact
these are hinderance to good washing, because the precipitate adheres onto the fibre of the
cloth as gummy mass. Hair washed with hard water looks dull because of this sticky
precipitate. Dye does not absorb evenly on cloth washed with soap using hard water,
because of this gummy mass.

59. Solution: a)
The need to have a sustainable energy supply necessitates the exploration of available
energy resources. Among these, renewable energy resources are in the forefront. It is now an
established fact that renewable energy can be an integral part of sustainable development
because of its inexhaustible nature and environment friendly features. Renewable energy
can also play an important role in resolving the energy crisis in urban areas to a great
extend. So far , renewable energy projects with an aggregate capacity of about 33,200 MW
have been installed in the country which includes 22,168 MW of wind power, 2870 MW of
solar power, 4225 MW of bio power and 3938 MW of small hydro power.
http://insightsonindia.com

INSIGHTS

Page 19

Facebook Group: Indian Administrative Service (Raz Kr)

Insights Mock tests 2015: test 9 Solutions


http://pib.nic.in/newsite/efeatures.aspx

60. Solution: d)
The decision of the Government to approve the North Eastern Region Power System
Improvement Project (NERPSIP) is in tune with its commitment for the economic
development of north eastern States. As the Intra State Transmission and Distribution
systems in the north eastern states have remained very weak; the Central Electricity
Authority developed a comprehensive scheme for the North East Region (NER) in
consultation with the Power Grid Corporation of India Limited (PGCIL).
It is for the six States of Assam, Manipur, Meghalaya, Mizoram, Tripura and Nagaland.
The main aim of the project is to address the lacuna in the Intra-State Transmission &
Distribution Infrastructure in the north east region. A similar scheme was earlier approved
for Arunachal Pradesh and Sikkim at an estimated cost of Rs 4754.42 crore.
The Scheme is to be taken up under a new Central Sector Plan Scheme of Ministry of Power
and will be implemented with the assistance of World Bank loan and by the Government of
India. The NERPSIP will be funded on 50:50 (World Bank: GoI) basis except the component
of capacity building for Rs 89 crore, which will be fully funded by the Government of India.
This project is the first phase of the three phased development, for which the World Bank
will be extending loan in three tranches of US $ 500 million each.

61. Solution: a)
is located a hundred kilometres from Ajanta and has thirty-two Buddhist, Brahmanical and
Jain caves. It is a unique art-historical site in the country as it has monastries associated with
the three religions dating from the fifth century CE onwards to the eleventh century CE.
It is also unique in terms of stylistic eclecticism, i.e., confluence of many styles at one place.
The caves of Ellora and Aurangabad show the ongoing differences between the two
religionsBuddhism and Brahmanical. There are twelve Buddhist caves having many
images belonging to Vajrayana Buddhism like Tara, Mahamayuri, Akshobhya,
Avalokiteshwara, Maitrya, Amitabha, etc. Buddhist caves are big in size and are of single,
double and triple storeys. Their pillars are massive. Ajanta also has excavated doublestoreyed caves but at Ellora, the triple storey is a unique achievement. All the caves were
plastered and painted but nothing visible is left. The shrine Buddha images are big in size;
they are generally guarded by the images of Padmapani and Vajrapani. Cave No. 12, which
is a triple-storey excavation, has images of Tara, Avalokiteshwara, Manushi Buddhas and
the images of Vairochana, Akshobhya, Ratnasambhava, Amitabha, Amoghsiddhi,
Vajrasatva and Vajraraja. On the other hand, the only double-storey cave of the Brahmanical
faith is Cave No. 14. Pillar designs grow from the Buddhist caves and when they reach the

http://insightsonindia.com

INSIGHTS

Page 20

Facebook Group: Indian Administrative Service (Raz Kr)

Insights Mock tests 2015: test 9 Solutions


Jain caves belonging to the ninth century CE, they become very ornate and the decorative
forms gain heavy protrusion.
62. Solution: d)
http://hdr.undp.org/en/content/human-development-index-hdi
There is a separate Gender Index to capture female labour participation rates.

63. Solution: d)
On the manufacturing front, the Government launched a Make in India programme so
that the Indian products would get a leg-up in terms of being competitive both in price and
quality not only within the country but also abroad. In order to enable domestic
manufacturing to gain cost advantage in local production, the authorities took several steps
to improve the doing business ethos within the country through administrative actions so
that the pronounced pro-growth and pro-business approach is understood and used up by
entrepreneurial forces. In fact, the Union Finance Minister Mr. Jaitely has time and again
stated that pro-growth and pro-business approach does not stand in conflict with pro-poor
policies as the resultant gains accruing from higher growth would level the field by making
the government to increase its outlays on public programmes, including genuine welfare
measures that help poverty elimination. The government also opened up important sectors
like defense and railways for overseas investment and increasing the FDI cap in the
insurance segment and the foreign direct investment into these fields would definitely go a
long way in improving these sectors in terms of transfer of technology and best managerial
practices, besides bolstering their capital base for further expansion and modernization in
the short to medium-term. As manufacturing requires the motor of power, the government
swiftly took initiatives to improve the power situation for domestic users, both individuals
and industry.
Following a Supreme Court ruling invalidating the allocation of over 200 coal blocs since
1993 that left dozens of private companies in uncertainty over their power supplies, the
government came out with an Ordinance to sort out the mess on October 20. The
government can now reassign these blocs through a transparent, online auction than an
earlier practice that led to sordid happenings and allegations of corruption in allocations.
The latest action on the coal front would help the power, steel and cement companies that
rely on these coal supplies, besides benefitting the coal-rich States to get the revenue from
auctions directly instead through the Centre. The new Ordinance also contains an enabling
provision that recognizes the need to open up coal mining to private companies. All these
measures for revival of activities across a broad range of industries through swift
administrative action duly backed up by political class to secure the requisite seal of
approval from Parliament meant the restoration of the much-needed investors confidence.
http://pib.nic.in/newsite/efeatures.aspx

http://insightsonindia.com

INSIGHTS

Page 21

Facebook Group: Indian Administrative Service (Raz Kr)

Insights Mock tests 2015: test 9 Solutions


64. Solution: d)
Key facts about Thatheras

The craft of the Thatheras constitutes the traditional techniques of


manufacturing brass, copper and kansa (an alloy of copper, zinc and tin) utensils.

They have a unique ethnic and historical identity with an oral tradition that
underpins their skill.

The name of the community Thatheras is identical with the name of the element.

These craft utensils of Thatheras have both utilitarian and ritualistic value.

The tradition of using the metals is recommended by the ancient Indian school of
medicine, Ayurveda.

Skills of the Thatheras have been orally transmitted from one generation to other
generations.

During the reign of Maharaja Ranjit Singh (1883) the great 19thCentury Sikh
Monarch, the Thatheras crafts colony was established in Jandiala Guru, Punjab.
Thus, Jandiala Guru became an area of repute due to the skill of the Thatheras.

65. Solution: a)
Beijing Protocol: This Protocol has brought out new principal offences in hijacking. This
protocol is combined with ancillary offences, enlarged the scope of hijacking, expanded
jurisdiction and strengthened extradition and mutual assistance regimes in case hijacking.
The government has recently given nod to for introduction of the comprehensive AntiHijacking Bill, 2014 in the Parliament. The Cabinet has also given its approval to
ratify Beijing Protocol, 2010 of the UN body International Civil Aviation Organisation
(ICAO) to which India is a signatory.
66. Solution: b)
Acquired immunity, on the other hand, is pathogen specific. It is characterised by memory.
This means that our body when it encounters a pathogen for the first time produces a
response called primary response which is of low intensity. Subsequent encounter with the
same pathogen elicits a highly intensified secondary or anamnestic response. This is
ascribed to the fact that our body appears to have memory of the first encounter.
The primary and secondary immune responses are carried out with the help of two special
types of lymphocytes present in our blood, i.e., B-lymphocytes and T-lymphocytes. The Blymphocytes produce an army of proteins in response to pathogens into our blood to fight
with them. These proteins are called antibodies. The T-cells themselves do not secrete
antibodies but help B cells produce them.
http://insightsonindia.com

INSIGHTS

Page 22

Facebook Group: Indian Administrative Service (Raz Kr)

Insights Mock tests 2015: test 9 Solutions


67. Solution: c)
In the year 1963, the two enzymes responsible for restricting the growth of bacteriophage in
Escherichia coli were isolated. One of these added methyl groups to DNA, while the other
cut DNA. The later was called restriction endonuclease. The first restriction endonuclease
Hind II, whose functioning depended on a specific DNA nucleotide sequence was isolated
and characterised five years later. It was found that Hind II always cut DNA molecules at a
particular point by recognising a specific sequence of six base pairs. This specific base
sequence is known as the recognition sequence for Hind II. Besides Hind II, today we know
more than 900 restriction enzymes that have been isolated from over 230 strains of bacteria
each of which recognise different recognition sequences.
68. Solution: c)
A Parliamentary Standing Committee on External Affairs has asked the government to
present Constitution (119th Amendment) Bill, 2013 without any delay to give effect to
the India-Bangladesh Land Boundary Agreement (LBA).
The Constitution (119th Amendment) Bill, 2013 aims to ratify the LBA between India and
Bangladesh under the Indira-Mujib pact of 1974 to exchange areas and people on either side
of the border.
As per the provision of pact, India will exchange 111 enclaves measuring 17,160 acres with
Bangladesh and receive 51 enclaves covering 7,110 acres. Almost 51,000 people reside in
these enclaves. The territories involved in the exchange are in Assam, West
Bengal, Meghalaya and Tripura.
On 16th May 1974, soon after the independence of Bangladesh the LBA was signed between
India and Bangladesh in order to find a solution to the complex nature of border
demarcation. This pact was ratified by Bangladesh government in 1974.
In 1974, in India Cabinet had granted approval to this pact but was not ratified as it involved
cession of territory.
69. Solution: a)
PM Modi by wearing a traditional Naga headgear and by beating the traditional Naga gong
three times declared the festival open at the picturesque heritage village of Kisama, Kohima.
About Hornbill festival of Nagaland
Hornbill (name of a bird) Festival is celebrated in Nagaland every year in the first week of
December. This festival is 10-day tourism promotional extravaganza which showcases
rich culture of Naga tribes which includes shows of music and dance.
It was established on 1st December 1963 and was inaugurated by the then President Dr. S
Radhakrishnan. The festival is organized by the State Tourism and Art & Culture
Departments, at Naga Heritage Village, Kisama which is about 12 km from Kohima the state
http://insightsonindia.com

INSIGHTS

Page 23

Facebook Group: Indian Administrative Service (Raz Kr)

Insights Mock tests 2015: test 9 Solutions


capital. Notably, it is also supported by the Centre as one of the biggest
indigenous festivals of the country.
The Hornbill Festival provides a colourful mixture of dances, performances, crafts, parades,
games, sports, food fairs and religious ceremonies. The festival both exposes the culture and
tradition of tribal peoples, and reinforces Nagalands identity as a unique state in Indias
federal union.
70. Solution: a)
Hemp: It is the common name for plants of the entire cannabis. This term is often used to
refer only to cannabis strains cultivated for industrial (non-drug) use.
Industrial hemp: It has many uses, including paper, textiles, biodegradable plastics,
construction, health food, fuel and hemp oil, wax, resin, rope and pulp.
Implications of hemp industry: Industrial hemp can bring a revolution in the industry as
hemp fibre is the strongest one in the world. It has the potential to cure cancer and a next
generation crop to bring revolution to our Indian economy.
71. Solution: d)
It is an integrated bill payment system offering inter-operable and accessible bill payment
service to customers through a network of agents, enabling multiple payment modes, and
providing instant confirmation of payment.
About BBPS

It will help consumers pay multiple bills like electricity, telephone and school fees at
a single point of transaction.

The National Payment Corporation of India (NPCI) has been appointed as the nodal
body which will set the standards, and also take care of clearing and settlement as
the Bharat Bill Payment Central Unit (BBPCU).

In the two tiered BBPS set-up, there will be authorised operational units called
Bharat Bill Payment Operating Units (BBPOUs) with an agent network under the
BBPCU.

RBI has set a Rs 100-crore networth and domestic registration as qualifying


conditions for those seeking to be authorised collection agents.

Participants in the BBPS will include authorised entities such as BPCU, BBPOUs as
well as their authorised agents, payment gateways, banks, billers and service
providers, and other entities, including authorised prepaid payment instrument
issuers.

Thus, BBPS will help track all the payments being made in economy, including cash
payments to utilities, schools, and telcos among others.
http://insightsonindia.com

INSIGHTS

Page 24

Facebook Group: Indian Administrative Service (Raz Kr)

Insights Mock tests 2015: test 9 Solutions


72. Solution: d)
Mount Aso, a volcano in southern Japan has erupted. It is the first such eruption in 22 years,
causing flight cancellations and prompting warnings to stay away from its crater. This
eruption has spewed out lava debris and smoke, shooting plumes of ash a kilometre into the
sky.
Earlier, Japan Meteorological Agency (JMA) had reported earthquakes and other seismic
activity in late August on Mount Aso.
About Mount Aso
Mount Aso is the largest active volcano in Japan, and is among the largest in the world. It is
located in Aso Kuju National Park in Kumamoto Prefecture, on Kyushu Island. Its peak is
1592 m above sea level. Mount Aso has one of the largest calderas in the world (25 km northsouth and 18 km east-west). The caldera has a circumference of around 120 km.
73. Solution: a)

The bill includes certain communities in the list of Scheduled Castes


from Kerala, Madhya Pradesh, Odisha, and Tripura. They are

1. Kerala: Pulluvan, Thachar (other than Carpenter).


2. Odisha: Amata, Amath, Bajia, Jaggili, Jagli, Buna Pano.
3. Tripura: Chamar-Rohidas, Chamar-Ravidas, Dhobi, Jhalo-Malo.

This amendment Bill removes a Majhi (Nepali) community from the list of Scheduled
Castes in Sikkim.

74. Solution: d)
The Trade Facilitation Agreement (TFA) is the WTOs first-ever multilateral accord. It was
outcome of 9th Ministerial Conference of WTO for broader reform to boost global trade held
in Bali, Indonesia in December, 2013.
The agreement includes provisions for

Lowering import tariffs and agricultural subsidies: It will make it easier for
developing countries to trade with the developed world in global markets.

Abolish hard import quotas: Developed countries would abolish hard import quotas
on agricultural products from the developing world and instead would only be
allowed to charge tariffs on amount of agricultural imports exceeding specific limits.

Reduction in red tape at international borders: It aims to reduce red-tapism to


facilitate trade by reforming customs bureaucracies and formalities.

Implications
http://insightsonindia.com

INSIGHTS

Page 25

Facebook Group: Indian Administrative Service (Raz Kr)

Insights Mock tests 2015: test 9 Solutions


If this agreement is properly implemented, it may create US$1 trillion worth of global
economic activity which may add 21 million new jobs and lower the cost of doing
international trade by 1015 percent.
75. Solution: d)
Perhaps the most important power of the Supreme Court is the power of judicial review.
Judicial Review means the power of the Supreme Court (or High Courts) to examine the
constitutionality of any law if the Court arrives at the conclusion that the law is inconsistent
with the provisions of the Constitution, such a law is declared as unconstitutional and
inapplicable. The term judicial review is nowhere mentioned in the Constitution. However,
the fact that India has a written constitution and the Supreme Court can strike down a law
that goes against fundamental rights, implicitly gives the Supreme Court the power of
judicial review.
In the case of federal relations too, the Supreme Court can use the review powers if a law is
inconsistent with the distribution of powers laid down by the Constitution. Suppose, the
central government makes a law, which according to some States, concerns a subject from
the State list- then the States can go to the Supreme Court and if the court agrees with them,
it would declare that the law is unconstitutional. In this sense, the review power of the
Supreme Court includes power to review legislations on the ground that they violate
fundamental rights or on the ground that they violate the federal distribution of powers. The
review power extends to the laws passed by State legislatures also.
Together, the writ powers and the review power of the Court make judiciary very powerful.
In particular, the review power means that the judiciary can interpret the Constitution and
the laws passed by the legislature.
76. Solution: c)
A plasmid is a small DNA molecule within a cell that is physically separated from a
chromosomal DNA and can replicate independently. They are most commonly found in
bacteria as small, circular, double-stranded DNA molecules, however plasmids are
sometimes present in archaea and eukaryotic organisms. In nature, plasmids carry genes
that may benefit survival of the organism (e.g. antibiotic resistance), and can frequently be
transmitted from one bacterium to another (even of another species) via horizontal gene
transfer. While the chromosomes are big and contain all the essential information for living
(an adequate analogy is the hard-drive of a computer), plasmids usually are very small and
contain additional information (in this analogy, plasmids are the USB flash drives). Artificial
plasmids are widely used as vectors in molecular cloning, serving to drive the replication of
recombinant DNA sequences within host organisms.
77. Solution: d)
http://www.downtoearth.org.in/content/invasion-triggers-evolution

http://insightsonindia.com

INSIGHTS

Page 26

Facebook Group: Indian Administrative Service (Raz Kr)

Insights Mock tests 2015: test 9 Solutions


78. Solution: d)
http://www.downtoearth.org.in/content/varied-visions-new-climate-agreement-lima
79. Solution: d)
http://www.downtoearth.org.in/content/watershed-conservation-can-benefit-700-millionpeople-100-big-cities
80. Solution: d)
http://www.downtoearth.org.in/content/supreme-court-sets-social-justice-bench-dealissues
81. Solution: c)
http://www.downtoearth.org.in/content/japan-uses-climate-finance-lend-1-billionindonesia-dirty-coal
http://www.theguardian.com/world/2014/sep/17/chinas-ban-on-dirty-coal-could-costaustralian-mining-almost-15bn
82. Solution: b)
http://www.downtoearth.org.in/content/india-get-25-solar-parks
83. Solution: c)
http://www.downtoearth.org.in/content/274-tiger-deaths-four-years-india-recordshighest-toll-between-two-censuses
84. Solution: d)
The rock-cut cave carved at Barabar hills near Gaya in Bihar is known as the Lomus Rishi
cave. The facade of the cave is decorated with the semicircular chaitya arch as the entrance.
The elephant frieze carved in high relief on the chaitya arch shows considerable movement.
The interior hall of this cave is rectangular with a circular chamber at the back. The entrance
is located on the side wall of the hall. The cave was patronised by Ashoka for the Ajivika
sect. The Lomus Rishi cave is an isolated example of this period. But many Buddhist caves of
the subsequent periods were excavated in eastern and western India.
Due to the popularity of Buddhism and Jainism, stupas and viharas were constructed on a
large scale. However, there are also examples of a few Brahmanical gods in the sculptural
representations. It is important to note that the stupas were constructed over the relics of the
Buddha at Rajagraha, Vaishali, Kapilavastu, Allakappa, Ramagrama, Vethadipa, Pava,
Kushinagar and Pippalvina. The textual tradition also mentions construction of various
other stupas on the relics of the Buddha at several places including Avanti and Gandhara
which are outside the Gangetic valley.

http://insightsonindia.com

INSIGHTS

Page 27

Facebook Group: Indian Administrative Service (Raz Kr)

Insights Mock tests 2015: test 9 Solutions


85. Solution: d)
http://www.downtoearth.org.in/content/geoengineering-could-be-catastrophic-planet
http://en.wikipedia.org/wiki/Climate_engineering
86. Solution: d)
In any society, disputes are bound to arise between individuals, between groups and
between individuals or groups and government.
All such disputes must be settled by an independent body in accordance with the principle
of rule of law. This idea of rule of law implies that all individuals rich and poor, men or
women, forward or backward castes are subjected to the same law. The principal role of
the judiciary is to protect rule of law and ensure supremacy of law. It safeguards rights of
the individual, settles disputes in accordance with the law and ensures that democracy does
not give way to individual or group dictatorship. In order to be able to do all this, it is
necessary that the judiciary is independent of any political pressures.
87. Solution: c)
BOD refers to the amount of the oxygen that would be consumed if all the organic matter in
one liter of water were oxidised by bacteria. The sewage water is treated till the BOD is
reduced. The BOD test measures the rate of uptake of oxygen by micro-organisms in a
sample of water and thus, indirectly, BOD is a measure of the organic matter present in the
water. The greater the BOD of waste water, more is its polluting potential.
88. Solution: b)
Simply stated independence of judiciary means that

the other organs of the government like the executive and legislature must not
restrain the functioning of the judiciary in such a way that it is unable to do justice.
the other organs of the government should not interfere with the decision of the
judiciary.
judges must be able to perform their functions without fear or favour.

Independence of the judiciary does not imply arbitrariness or absence of accountability.


Judiciary is a part of the democratic political structure of the country. It is therefore
accountable to the Constitution, to the democratic traditions and to the people of the
country.
89. Solution: d)
The judges have a fixed tenure. They hold office till reaching the age of retirement. Only in
exceptional cases, judges may be removed. But otherwise, they have security of tenure.
Security of tenure ensures that judges could function without fear or favour. The
Constitution prescribes a very difficult procedure for removal of judges.

http://insightsonindia.com

INSIGHTS

Page 28

Facebook Group: Indian Administrative Service (Raz Kr)

Insights Mock tests 2015: test 9 Solutions


The Constitution makers believed that a difficult procedure of removal would provide
security of office to the members of judiciary.
The judiciary is not financially dependent on either the executive or legislature. The
Constitution provides that the salaries and allowances of the judges are not subjected to the
approval of the legislature. The actions and decisions of the judges are immune from
personal criticisms. The judiciary has the power to penalise those who are found guilty of
contempt of court. This authority of the court is seen as an effective protection to the judges
from unfair criticism. Parliament cannot discuss the conduct of the judges except when the
proceeding to remove a judge is being carried out. This gives the judiciary independence to
adjudicate without fear of being criticised.
90. Solution: b)
Archaeologists have discovered thousands of seals, usually made of steatite, and
occasionally of agate, chert, copper, faience and terracotta, with beautiful figures of animals,
such as unicorn bull, rhinoceros, tiger, elephant, bison, goat, buffalo, etc. The realistic
rendering of these animals in various moods is remarkable. The purpose of producing seals
was mainly commercial. It appears that the seals were also used as amulets, carried on the
persons of their owners, perhaps as modern-day identity cards. The standard Harappan seal
was a square plaque 22 square inches, usually made from the soft river stone, steatite.
Every seal is engraved in a pictographic script which is yet to be deciphered. Some seals
have also been found in gold and ivory. They all bear a great variety of motifs, most often of
animals including those of the bull, with or without the hump, the elephant, tiger, goat and
also monsters. Sometimes trees or human figures were also depicted.
The most remarkable seal is the one depicted with a figure in the centre and animals around.
This seal is generally identified as the Pashupati Seal by some scholars whereas some
identify it as the female deity. This seal depicts a human figure seated cross-legged. An
elephant and a tiger are depicted to the right side of the seated figure, while on the left a
rhinoceros and a buffalo are seen. In addition to these animals two antelopes are shown
below the seat. Seals such as these date from between 2500 and 1500 BCE and were found in
considerable numbers in sites such as the ancient city of Mohenjodaro in the Indus Valley.
Figures and animals are carved in intaglio on their surfaces. Square or rectangular copper
tablets, with an animal or a human figure on one side and an inscription on the other, or an
inscription on both sides have also been found. The figures and signs are carefully cut with a
burin. These copper tablets appear to have been amulets. Unlike inscriptions on seals which
vary in each case, inscriptions on the copper tablets seem to be associated with the animals
portrayed on them.
91. Solution: c)
Original jurisdiction means cases that can be directly considered by the Supreme Court
without going to the lower courts before that.
Form the diagram below, you will notice that cases involving federal relations go directly to
the Supreme Court. The Original Jurisdiction of the Supreme Court establishes it as an
http://insightsonindia.com

INSIGHTS

Page 29

Facebook Group: Indian Administrative Service (Raz Kr)

Insights Mock tests 2015: test 9 Solutions


umpire in all disputes regarding federal matters. In any federal country, legal disputes are
bound to arise between the Union and the States; and among the States themselves. The
power to resolve such cases is entrusted to the Supreme Court of India. It is called original
jurisdiction because the Supreme Court alone has the power to deal with such cases. Neither
the High Courts nor the lower courts can deal with such cases. In this capacity, the Supreme
Court not just settles disputes but also interprets the powers of Union and State government
as laid down in the Constitution.

92. Solution: b)
When a host is exposed to antigens, which may be in the form of living or dead microbes or
other proteins, antibodies are produced in the host body. This type of immunity is called
active immunity. Active immunity is slow and takes time to give its full effective response.
Injecting the microbes deliberately during immunisation or infectious organisms gaining
access into body during natural infection induce active immunity. When ready-made
antibodies are directly given to protect the body against foreign agents, it is called passive
immunity.
93. Solution: d)
US model is based on very strong states. USSR was based on an authoritarian centre.
Canada is a federal country with a Union bias same as India.

http://insightsonindia.com

INSIGHTS

Page 30

Facebook Group: Indian Administrative Service (Raz Kr)

Insights Mock tests 2015: test 9 Solutions


94. Solution: a)

95. Solution: a)
The very existence of a State including its territorial integrity is in the hands of Parliament.
The Constitution has certain very powerful emergency provisions, which can turn our
federal polity into a highly centralised system once emergency is declared.
Even during normal circumstances, the central government has very effective financial
powers and responsibilities.
Governor has certain powers to recommend dismissal of the State government and the
dissolution of the Assembly.
Planning is under the Concurrent List you can find it out because each State has a separate
Planning Department.
96. Solution: c)
Bacteria like Streptococcus pneumoniae and Haemophilus influenzae are responsible for the
disease pneumonia in humans which infects the alveoli (air filled sacs) of the lungs. As a
result of the infection, the alveoli get filled with fluid leading to severe problems in
http://insightsonindia.com

INSIGHTS

Page 31

Facebook Group: Indian Administrative Service (Raz Kr)

Insights Mock tests 2015: test 9 Solutions


respiration. The symptoms of pneumonia include fever, chills, cough and headache. In
severe cases, the lips and finger nails may turn gray to bluish in colour. A healthy person
acquires the infection by inhaling the droplets/aerosols released by an infected person or
even by sharing glasses and utensils with an infected person.
For Widal test, refer: http://en.wikipedia.org/wiki/Widal_test
97. Solution: c)
There are 5 conditions under which it can be done:
a) To give effect to an international treaty to which India is a signatory
b) Under Presidents rule
c) Under national emergency
d) When two or more states request Parliament to do so (applies to only those states)
e) When Rajya Sabha passes a resolution calling for Parliament to legislate in the state list.
98. Solution: a)
Cancer detection is based on biopsy and histopathological studies of the tissue and blood
and bone marrow tests for increased cell counts in the case of leukemias. In biopsy, a piece
of the suspected tissue cut into thin sections is stained and examined under microscope
(histopathological studies) by a pathologist. Techniques like radiography (use of X-rays), CT
(computed tomography) and MRI (magnetic resonance imaging) are very useful to detect
cancers of the internal organs. Computed tomography uses X-rays to generate a threedimensional image of the internals of an object. MRI uses strong magnetic fields and nonionising radiations to accurately detect pathological and physiological changes in the living
tissue.
99. Solution: d)
The style of temple architecture that became popular in northern India is known as nagara.
In North India it is common for an entire temple to be built on a stone platform with steps
leading up to it. Further, unlike in South India it does not usually have elaborate boundary
walls or gateways. While the earliest temples had just one tower, or shikhara, later temples
had several. The garbhagriha is always located directly under the tallest tower.
There are many subdivisions of nagara temples depending on the shape of the shikhara.
There are different names for the various parts of the temple in different parts of India;
however, the most common name for the simple shikhara which is square at the base and
whose walls curve or slope inward to a point on top is called the 'latina' or the rekhaprasada type of shikara.
The second major type of architectural form in the nagara order is the phamsana. Phamsana
buildings tend to be broader and shorter than latina ones. Their roofs are composed of
http://insightsonindia.com

INSIGHTS

Page 32

Facebook Group: Indian Administrative Service (Raz Kr)

Insights Mock tests 2015: test 9 Solutions


several slabs that gently rise to a single point over the centre of the building, unlike the
latina ones which look like sharply rising tall towers. Phamsana roofs do not curve inward,
instead they slope upwards on a straight incline. In many North Indian temples you will
notice that the phamsana design is used for the mandapas while the main garbhagriha is
housed in a latina building. Later on, the latina buildings grew complex, and instead of
appearing like a single tall tower, the temple began to support many smaller towers, which
were clustered together like rising mountain-peaks with the tallest one being in the centre,
and this was the one which was always above the garbhagriha.
The third main sub-type of the nagara building is what is generally called the valabhi type.
These are rectangular buildings with a roof that rises into a vaulted chamber. The edge of
this vaulted chamber is rounded, like the bamboo or wooden wagons that would have been
drawn by bullocks in ancient times. They are usually called wagonvaulted buildings. As
mentioned above, the form of the temple is influenced by ancient building forms that were
already in existence before the fifth century CE. The valabhi type of building was one of
them. For instance, if you study the ground-plan of many of the Buddhist rock-cut chaitya
caves, you will notice that they are shaped as long halls which end in a curved back. From
the inside, the roof of this portion also looks like a wagon-vaulted roof.

100.

Solution: a)

The density of a population in a given habitat during a given period, fluctuates due to
changes in four basic processes, two of which (natality and immigration) contribute an
increase in population density and two (mortality and emigration) to a decrease.
(i) Natality refers to the number of births during a given period in the population that are
added to the initial density.
(ii) Mortality is the number of deaths in the population during a given period.
(iii) Immigration is the number of individuals of the same species that have come into the
habitat from elsewhere during the time period under consideration.
(iv) Emigration is the number of individuals of the population who left the habitat and gone
elsewhere during the time period under consideration.

http://insightsonindia.com

INSIGHTS

Page 33

INSIGHTS ON INDIA MOCK PRELIMINARY EXAM - 2015


INSIGHTS ON INDIA MOCK TEST - 10
GENERAL STUDIES

PAPER-I
Time Allowed: 2 Hours

Maximum Marks: 160

INSTRUCTIONS
1. IMMEDITELY AFTER THE COMMENCEMENT OF THE EXAMINATION, YOU SHOULD
CHECK THAT THIS TEST BOOKLET DOES NOT HAVE ANY UNPRINTED OR TORN OR MISSING
PAGES OR ITEMS, ETC. IF SO, GET IT REPLACED BY A COMPLETE TEST BOOKLET.
2. You have to enter your Roll Number on the Test
Booklet in the Box provided alongside. DO NOT
Write anything else on the Test Booklet.
4. This Test Booklet contains 80 items (questions). Each item is printed only in English. Each item
comprises four responses (answers). You will select the response which you want to mark on the
Answer Sheet. In case you feel that there is more than one correct response, mark the response which
you consider the best. In any case, choose ONLY ONE response for each item.
5. You have to mark all your responses ONLY on the separate Answer Sheet provided. See directions in
the Answer Sheet.
6. All items carry equal marks.
7. Before you proceed to mark in the Answer Sheet the response to various items in the Test Booklet, you
have to fill in some particulars in the Answer Sheet as per instructions sent to you with your
Admission Certificate.
8. After you have completed filling in all your responses on the Answer Sheet and the examination has
concluded, you should hand over to the Invigilator only the Answer Sheet. You are permitted to take
away with you the Test Booklet.
9. Sheets for rough work are appended in the Test Booklet at the end.
10. Penalty for wrong answers :
THERE WILL BE PENALTY FOR WRONG ANSWERS MARKED BY A CANDIDATE IN THE
OBJECTIVE TYPE QUESTION PAPERS.
(i) There are four alternatives for the answer to every question. For each question for which a
wrong answer has been given by the candidate, one-third of the marks assigned to that
question will be deducted as penalty.
(ii) If a candidate gives more than one answer, it will be treated as a wrong answer even if one of
the given answers happens to be correct and there will be same penalty as above to that
question.
(iii)

If a question is left blank, i.e., no answer is given by the candidate, there will be no penalty
for that question.
http://insightsonindia.com

INSIGHTS ON INDIA MOCK TEST SERIES FOR CIVIL SERVICES PRELIMINARY EXAM 2015
http://insightsonindia.com

INSIGHTS

Page 1

1. In the post 1981 till present, the growth


rate of countrys population though
remained high, has started slowing down
gradually possibly due to
1. Increase in mean age of marriage
2. Increased literacy of females in the
country
3. Reduced international migration than
the previous periods

4. The recently released UN Human


Development report, 2014 mentions an
emphasis to be given on
1. Goal of Full employment
2. Universal social protections
3. Withdrawal of state from nonessentials
Choose the correct answer using the codes below:
a)
b)
c)
d)

Choose the correct answer using the codes below:


a)
b)
c)
d)

1 and 2
2 and 3
1 and 3
All of the above

2. Population in rural areas of India has been


affected by which of the following?
1. Relative degree of Urbanization
2. Lack of Public Health facilities in rural
areas
3. MGNREGA

5. In which of the following states, the


density of population is less than the
National average?
1. Madhya Pradesh
2. Chattisgarh
3. Kerala
4. Haryana
Choose the correct answer using the codes below:

Choose the correct answer using the codes below:


a)
b)
c)
d)

Choose the correct answer using the codes below:


a.
b.
c.
d.

a)
b)
c)
d)

1 and 2
2 and 3
1 and 3
All of the above

3. Jain or Buddhist population concentration


is NOT high in which of the following
states in India?
1. Maharashtra
2. Sikkim
3. Gujarat

1 and 2
2 and 3
1 and 3
It is high in all the above states.

http://insightsonindia.com

1 and 2
2 and 3
1 and 3
All of the above

All of the above


1 and 2
1, 2 and 4
3 and 4 only

6. As per the Human Development report,


2014 of UN, consider the following about
the status of gender inequality in India.
1. India fares the lowest in gender
development index in BRICS.
2. India has the worst gender inequality
in South Asia alongwith Pakistan.
3. Female literacy in India is the lowest
in South Asia.
Choose the correct answer using the codes below:
a)
b)
c)
d)
INSIGHTS

1 and 2
2 and 3
1 and 3
All of the above
Page 2

7. Consider the following statements about


intra-state and inter-state migration in
India:
1. Females predominate the streams of
short distance rural to rural migration
in both types of migration.
2. Men predominate the rural to urban
stream of inter-state migration.

10. Consider the following statements with


regard to the Census 2011, India.
1. Overall sex ratio has declined as
compared to Census 2001.
2. Child sex ratio has increased as
compared to Census 2001.
3. Population growth rate (per decade) is
the lowest since Independence.
Choose the correct answer using the codes below:

Which of these is/are true?


a)
b)
c)
d)

a)
b)
c)
d)

1 only
2 only
Both 1 and 2
None

8. Which of the following is associated with


migration in India, either domestic or
international?
1. Remittances
2. Change in Social composition of
regions
3. Development of slums

11. Consider the following statements.


1. Fossil fuels are always obtained from
non-metallic minerals.
2. There is generally an inverse
relationship between the quality and
quantity of minerals.
3. All minerals are exhaustible over time.
Choose the correct answer using the codes below:

Choose the correct answer using the codes below:


a)
b)
c)
d)

Choose the correct answer using the codes below:


a)
b)
c)
d)

a)
b)
c)
d)

1 and 2
2 and 3
1 and 3
All of the above

9. Which of the following factors have been


correlated with the high incidence of
poverty in India?
1. Jobless growth
2. Poor sanitation coverage
3. Poor reach of public health facilities

1 and 2
2 and 3
1 and 3
All of the above

12. Coal reserves in India can be found in


which of the following river valleys of
India?
1. Mahanadi
2. Godavari
3. Damodar
4. Sone
Choose the correct answer using the codes below:

1 and 2
2 and 3
1 and 3
All of the above

http://insightsonindia.com

1 and 2
2 and 3
1 and 3
All of the above

a)
b)
c)
d)

INSIGHTS

All of the above


2 and 3 only
1, 2 and 3 only
1 and 4 only

Page 3

13. Consider the following statements:


1. Most of the mineral reserves in India
occur to the east of a line joining
Mangalore and Kanpur.
2. No major mining activities have been
undertaken in the Himalayan regions.
3. Most of the metallic minerals in India
occur in the Pennisular Plateau region
in the old crystalline rocks.

a)
b)
c)
d)

16. Which of the following factors affect land


use changes in the economy of India?
1. Change in the composition of economy
2. Changes in population
3. Urbanization

Choose the correct answer using the codes below:


a)
b)
c)
d)

1 and 2
2 and 3
1 and 3
All of the above

14. Consider the following statements about


land reporting area in India.
1. Area under actual forest cover is
greater than area classified as forest.
2. There can be an increase in the forest
cover of India without any increase in
the actual forest cover.
3. Forest cover also includes plantations.
Choose the correct answer using the codes below:
a)
b)
c)
d)

1 and 2
2 and 3
1 and 3
All of the above

15. How are culturable waste-lands different


from barren wastelands, as shown in land
use categories in India?
1. Culturable wasteland can not be
brought to use even with the use of
modern technology.
2. Culturable wasteland is left fallow
more than five years.
3. Culturalble wastelands cultural
practice adopted for giving the land
rest.

2 only
2 and 3 only
1 and 2 only
All of the above

Choose the correct answer using the codes below:


a)
b)
c)
d)

1 and 2
2 and 3
1 and 3
All of the above

17. Common property resources (CPRs) in


rural areas are important for social
security because
1. they can be used as pasture grounds
by weak and landless communities
2. Increase in area under nonagricultural use
3. women from tribal communities can
extract fuel and fodder from CPRs
Choose the correct answer using the codes below:
a)
b)
c)
d)

1 and 2
2 and 3
1 and 3
All of the above

18. Which of the following social-economic


and cultural functions are performed by
land in rural areas?
1. Serves as security for credit
2. Lack of access to land is directly
correlated with incidence of poverty in
rural areas.

Which of these is/are true?


http://insightsonindia.com

INSIGHTS

Page 4

3. Security for natural hazards and other


contingencies
Choose the correct answer using the codes below:
a)
b)
c)
d)

Choose the correct answer using the codes below:

1 and 2
2 and 3
1 and 3
All of the above

19. Which of the following crops can be


grown in both kharif and rabi seasons in
India?
1. Rice
2. Wheat
3. Groundnut
4. Jowar
Choose the correct answer using the codes below:
a)
b)
c)
d)

3. Introduction of disease-resistant
varieties

a)
b)
c)
d)

22. Consider the following statements about


the Maize crop.
1. It can be grown on inferior soils.
2. It is sown all over India except eastern
and north-eastern regions.
3. Yield level of maize is higher than
other coarse cereals.
Choose the correct answer using the codes below:

1 and 2 only
2 and 3 only
1, 3 and 4 only
All of the above

20. There is a mismatch between the agroclimatic conditions and the cropping
pattern in the country. Which of the
following crop patterns show the same?
1. Sugarcane
2. Cotton
3. Groundnut
Choose the correct answer using the codes below:
a) 1 and 2
b) 2 and 3
c) 1 and 3
d) All of the above
21. The yield of Bajra has grown in Gujarat
and Haryana in recent years. This is due
to
1. Expansion of irrigation facilities
2. Introduction of drought resistant
varieties

http://insightsonindia.com

1 and 2
2 and 3
1 and 3
All of the above

a)
b)
c)
d)

1 and 2
2 and 3
1 and 3
All of the above

23. Consider the following statements about


some of the cash crops in India.
1. India lost both cotton and jute
growing areas to Pakistan after
partition.
2. All cash crops in India are grown only
in rainfed conditions.
3. No cash crop is grown in tropical areas
in the country.
Choose the correct answer using the codes below:
a)
b)
c)
d)

1 and 2
2 and 3
1 only
1 and 3

24. Consider the following statements about


tea and coffee crops in India.
1. Tea is an indigenous crop of India.
2. India grows inferior quality of coffee.
INSIGHTS

Page 5

3. Kerala produces more coffee than all


other states combined.
Choose the correct answer using the codes below:
a)
b)
c)
d)

1 and 2
2 and 3
None of the above
3 only

25. The yield of the crops in India is low in


comparison to the international level.
Why?
1. Agriculture is largely rainfed.
2. High pressure on land resources
3. Labour productivity is low in Indian
agriculture.
Choose the correct answer using the codes below:
a)
b)
c)
d)

1 and 2
2 and 3
All of the above
1 and 3

26. Which of the following challenges is being


faced by Indian agriculture in India
presently?
1. Lack of commercialization
2. Small and fragmented land holdings
3. Vast Under-employment
Choose the correct answer using the codes below:
a)
b)
c)
d)

1 and 2
2 and 3
All of the above
1 and 3

27. Why the share of forest has shown an


increase in the last four decades in India?
a) Better peoples participation in
managing forest area.
b) Increase in community forest land
c) Increase in notified area allocated for
forest growth
http://insightsonindia.com

d) Extensive and efficient efforts of


afforestation
28. Which of the following can NOT be the
use of Lagoons and backwaters?
1. Fishing
2. Irrigation of paddy crops
3. Navigation
Choose the correct answer using the codes below:
a)
b)
c)
d)

1 and 3
1 only
All can be the uses.
2 only

29. Arrange the following in ascending order


with respect to their use of ground water
in India.
1. Agriculture
2. Industries
3. Domestic
Choose the correct order from the codes below:
a)
b)
c)
d)

123
321
231
213

30. There has been an increase in arsenic


concentration in groundwater in West
Bengal and Bihar. This is due to
a) Excessive withdrawal of groundwater
b) Industrial pollution
c) Acid rain
d) Runoff from agricultural lands
31. Which of the following Parliamentary acts
deal with the issue of water pollution in
India?
1. Environment Protection Act, 1986
2. Water Cess Act, 1977
3. Air protection Act, 1975
Choose the correct answer using the codes below:
INSIGHTS

Page 6

a)
b)
c)
d)

1 and 3 only
1 and 2 only
2 and 3 only
All of the above

32. Which of the following aspects are


associated with effective watershed
management in India?
1. Community participation
2. Employment generation
3. Conservation of Groundwater
resources
Choose the correct answer using the codes below:
a)
b)
c)
d)

1 and 3 only
1 and 2 only
2 and 3 only
All of the above

33. Which of the following would result in the


phenomena of sea water intrusion in
coastal areas like Chennai?
1. Excessive withdrawal of Groundwater
2. Navigation channels which allow
which provide conduits for saltwater
to move inland
3. Sea level rise
Choose the correct answer using the codes
below:
a)
b)
c)
d)

1 and 3 only
1 and 2 only
2 and 3 only
All of the above

34. Rainwater harvesting can be beneficial in


addressing which of the following issues
related to water management?
1. Salt water intrusion
2. Arrest declining water table
3. Dilutes contaminants in groundwater
like arsenic, fluoride

a)
b)
c)
d)

35. Which of the following are the salient


features of the National water Policy,
2012?
1. Need for a National water framework
law
2. Treating water as an economic good
3. Puts water needs for food security
higher than that of sanitation
Choose the correct answer using the codes
below:
a)
b)
c)
d)

1 and 3 only
1 and 2 only
2 and 3 only
All of the above

36. The National water policy talks about


Water footprints. What is it?
a) Measure of water used per unit of
produced goods by industries
b) Measure of total water used in each
category of industry
c) Total amount of carbon emissions
caused in withdrawing and utilizing
water from natural sources
d) None of the above
37. The Hyogo framework for action, 2005
does NOT call for which of the following?
1. Disaster risk reduction as a part of
development planning
2. Community response in case of
disasters
3. Reducing death from disasters to 10%
by 2025 of the figure of the decade
1990-2000
Choose the correct answer using the codes below:

Choose the correct answer using the codes below:


http://insightsonindia.com

1 and 3 only
1 and 2 only
2 and 3 only
All of the above

a) 1 and 2 only
INSIGHTS

Page 7

b) 1 and 3 only
c) 2 and 3 only
d) All of the above
38. Arrange the following disasters in
increasing order of their geographical
spread/impact on India.
1. Earthquake
2. Cyclones
3. Drought
4. Floods
Choose the correct answer using the codes below:
a)
b)
c)
d)

1234
1342
2413
2431

39. India has been divided in five earthquake


zones. Which of the following states
comes under the Very High damage Risk
Zone?
1. Bihar
2. North-eastern states
3. Gujarat
Choose the correct answer using the codes below:
a)
b)
c)
d)

1 and 2 only
1 and 3 only
2 and 3 only
All of the above

40. Tsunamis in oceans can be caused due to


1. Earthquakes
2. Volcanic eruptions
3. Cyclones

41. Consider the following statements.


Assertion (A): The impact of tsunami is less over
the ocean and more near the coast where they
cause large-scale devastations.
Reason (R): The speed of wave in the ocean
depends upon the depth of water. It is more in
the shallow water than in the ocean deep.
In the context of the statements above, which of
these is true?
a) A and R both are true, and R is the
correct explanation for A.
b) A and R both are true, and R is the
NOT the correct explanation for A.
c) A is correct, R is incorrect.
d) A and R both are incorrect.
42. A tropical cyclone is like a heat engine
that is energised by
a) Latent heat of condensation
b) Latent heat of vaporisation
c) Presence of strong warm ocean
currents
d) Presence of warm and cold fronts
43. Which of the following pre-conditions are
essential for formation of tropical
cyclones?
1. Meeting of warm and cold fronts
2. Presence of Coriolis force
3. Unstable conditions in the troposphere
Choose the correct answer using the codes below:
a)
b)
c)
d)

Choose the correct answer using the codes below:


a)
b)
c)
d)

1 and 2 only
1 and 3 only
2 and 3 only
All of the above

http://insightsonindia.com

1 and 2 only
1 and 3 only
2 and 3 only
All of the above

44. Which of the following reasons can


possible explain more cyclones occurring
in bay of Bengal than in Arabian Sea?
1. Easterlies

INSIGHTS

Page 8

2. Bay of Bengal ocean has higher


temperature than Arabian sea
3. The narrow morphology of Bay of
Bengal
Choose the correct answer using the codes below:
a)
b)
c)
d)

1 and 2 only
1 and 3 only
2 and 3 only
All of the above

45. Which of the following may aggravate the


situation of floods?
1. Deforestation
2. Sedimentation of river basins
3. Colonization of flood-plains
Choose the correct answer using the codes below:
a)
b)
c)
d)

1 and 2 only
1 and 3 only
2 and 3 only
All of the above

46. Agricultural drought is a situation where


a) there is low soil moisture for the
growth of crops
b) there is a period of inadequate rainfall
c) availability of water in different
reservoirs depletes beyond normal
levels
d) None of the above
47. The National Disaster management Act,
2005 provides for which of the following?
1. Constitution of a Disaster response
force at central, state and district level
2. It provides for Disaster management
authorities at the central, state and
District levels
3. It fixes accountability for the disasters
with the District Magistrate/Collector

a)
b)
c)
d)

48. The Western coast of India has more sea


ports than the eastern coast. The
geographical explanation for it can be
a) Western coasts are high rocky coasts
b) Western coasts are low sedimentary
coasts
c) More cyclones are seen in Western
coasts
d) More trade occurs through the
Western coasts

49. The Delhi Police have recently launched a


scheme name Operation Milap. It is
aimed at
a) uniting missing children with their
parents
b) uniting the cross-border refugees with
their families
c) uniting the neglected old aged with
their families
d) uniting the surrendered naxals
(accused) with their families by
reducing their punishment
50. Consider the following statements about
the recently launched GSLV Mark-III X.
1. It uses cryogenic technology.
2. It is also fuelled by solid propellant.
3. It will help India become self-reliant in
launching communication satellites.
Choose the correct answer using the codes below:
a)
b)
c)
d)

Choose the correct answer using the codes below:

http://insightsonindia.com

1 and 2 only
1 and 3 only
2 and 3 only
All of the above

INSIGHTS

1 and 2 only
1 and 3 only
2 and 3 only
All of the above

Page 9

51. Recently some of the regions in districts in


Haryana were awarded Nirmal Gram
Puruskar (NGP). Consider the following
statements about NGP.
1. It is awarded to Gram Panchayats.
2. It is awarded on achieving open zero
defecation.
3. The award also includes the
maintaining of school hygiene.
Choose the correct answer using the codes below:
a)
b)
c)
d)

1 and 2 only
1 and 3 only
2 and 3 only
All of the above

52. It is said that the INS Arihant submarine


will help India achieve nuclear triad. What
are the other two elements of this triad?
1. Inter-continental ballistic missiles
2. Traditional strategic bombers
3. Nuclear plants
Choose the correct answer using the codes below:
a)
b)
c)
d)

1 and 2 only
1 and 3 only
2 and 3 only
1 only

53. The Rajasthan government has recently


launched Bhamashah Scheme card.
Consider the following statements about
it.
1. This scheme will provide woman head
of the family with a bank account.
2. This card will also be used as an
identity card for the people of
Rajasthan.
3. Workers under unorganized sector
will also be given insurance cover
under the card.
Choose the correct answer using the codes below:

http://insightsonindia.com

a)
b)
c)
d)

1 and 2 only
1 and 3 only
2 and 3 only
All of the above

54. Bangladeshs oil spill disaster is


considered as ecological catastrophe
which may threaten rare dolphins, other
wildlife species and worlds largest
mangrove forests in the Sundrbans. How
will affect the population of Dolphins?
a) It will cut down the dissolved oxygen
leading to suffocation of Dolphins.
b) It will reduce the population of
phytoplanktons thus directly affecting
Dolphins.
c) It will threaten the Mangroves and
coral reefs which are the breeding
grounds of Dolphins
d) None of the above
55. Government has given nod for the
implementation of Tea Development and
Promotion Scheme and approved it
under the outlay of the 12th Plan period.
Which of the following activities would
NOT be supported under the scheme?
1. Promoting Indian tea in overseas
markets alongwith domestic markets
2. Research on improving productivity
3. Plantation development
Choose the correct answer using the codes below:
a)
b)
c)
d)

1 and 2 only
1 and 3 only
2 and 3 only
All will be supported.

56. 21st June will be annually observed


as International Day of Yoga after United
Nations General Assembly (UN-GA)
adopted an India-led resolution. UN has
adopted this resolution under the agenda

INSIGHTS

Page 10

of Global Health and Foreign Policy.


Why was the date 21st June chosen?
a) It represents one of the two solstices, is
the longest day in the Northern
Hemisphere and has special
significance in many parts of the
world.
b) It falls in the middle of the year.
c) Other dates were already occupied
with important International days.
d) The date was choose randomly by the
Indian leadership and proposed to the
UNGA.
57. The Government of India has launched a
new schemes viz. Deendayal Upadhyaya
Gram Jyoti Yojana (DDUGJY). Consider
the following statements about it.
1. It aims to provide reliable and
adequate power supply to farmers by
separation of agriculture and nonagriculture feeders.
2. It also aims at strengthening of subtransmission and distribution
infrastructure in rural areas.
3. It will be in addition to the Rajiv
Gandhi Grameen Vidyutikaran Yojana
for rural electrification in villages
going now.
Choose the correct answer using the codes below:
a)
b)
c)
d)

1 and 2 only
1 and 3 only
2 and 3 only
All of the above.

In the context of the statements above, which of


these is true?
a) A and R both are true, and R is the
correct explanation for A.
b) A and R both are true, and R is the
NOT the correct explanation for A.
c) A is correct, R is incorrect.
d) A and R both are incorrect.

59. Which of the following


schemes/centres/laws are related to
agricultural development in India?
1. Krishi Vigyan Kendras (KVKs)
2. Grameen Bhandaran Yojana
3. Fertilizer (Control) Order, 1985
Choose the correct answer using the codes below:
a)
b)
c)
d)

1 and 2 only
1 and 3 only
2 and 3 only
All of the above.

60. Power crisis in India is a major issue. The


government has recently launched
Integrated Power Development Scheme
(IPDS) scheme. The scheme provides for
1. Strengthening of sub-transmission
network
2. IT enabled energy accounting /
auditing system
3. Customer Care Services
Choose the correct answer using the codes below:
a)
b)
c)
d)

58. Consider the following statements.


Assertion (A): Water resources/ irrigation
projects are planned, executed and maintained by
State Governments from their own resources and
as per their own priorities.

1 and 2 only
1 and 3 only
2 and 3 only
All of the above.

Reason (R): Water is a State subject.


http://insightsonindia.com

INSIGHTS

Page 11

61. Scientists say that the worlds oldest


water, found deep under Earths surface,
could be holding more water than the
volume of all of the worlds rivers,
swamps and lakes put together. Consider
the following statements about it.
1. It has been found in Canada,
Sandinavia and South Africa.
2. It can be found in Precambrian shield
rocks.
3. The water has been found to be fit for
drinking.
Choose the correct answer using the codes below:
a) 1 and 2 only
b) 1 and 3 only
c) 2 and 3 only
d) All of the above.
62. Recently Gigaton framework was created
by UNEP. Consider the following
statements about it.
1. It is aimed at saving coral reef
ecosystems across the world.
2. It is open to only littoral countries.
3. It is funded by World Bank.
Choose the correct answer using the codes below:
a)
b)
c)
d)

1 and 2 only
1 and 3 only
2 and 3 only
None of the above

63. Industrialists have been arguing that the


Forest Rights Act (FRA), 2006 is stalling
development in India. Consider the
following statements about it.
1. It provides land titles to landless
tribals in forest areas.
2. It provides tribals the right to extract
non-timber produce from their forests.
3. Gram Sabha recognizes the right of
tribals to land titles.

a)
b)
c)
d)

1 and 2 only
1 and 3 only
2 and 3 only
All of the above

64. The potential economic, social and


environmental gains from conserving
mangroves90 per cent of which are
found in developing countriesstill
remain largely under-exploited, says a
new United Nations (UN)
report. Consider the following statements.
1. Mangroves hold several times more
carbon than terrestrial forests.
2. As per the UN report, Conservation of
existing intact coastal wetlands is
simpler, more cost-effective with
greatest ecosystem benefits.
3. Mangroves grow in inter-tidal zones.
Choose the correct answer using the codes below:
a)
b)
c)
d)

1 and 2 only
1 and 3 only
2 and 3 only
All of the above

65. A recent notification from FSSAI proposes


to limit the maximum level of Transfats
(TFAs) to five per cent (by weight of total
quantity), from 2016. Consider the
following statements about Transfats.
1. They give food longer shelf life.
2. Transfats are also found naturally in
some foods.
3. Transfats can cause coronary heart
disease.
Choose the correct answer using the codes below:
a)
b)
c)
d)

1 and 2 only
1 and 3 only
2 and 3 only
All of the above

Choose the correct answer using the codes below:


http://insightsonindia.com

INSIGHTS

Page 12

66. Recently the use of anti-biotics in chicken


has been growing that may be an issue for
public health. Consider the following
statements about it.
1. There are no standards for use of antibiotics in chickens by FSSAI.
2. Quantity of bulk drug (antibiotics, in
kg) imported from China comprised
more than 90 per cent of the total bulk
drug imported by India in 2014.
3. Anti-microbial resistance in chicken
can spread to humans if anti-biotic use
in chicken is continued.
Choose the correct answer using the codes below:
a)
b)
c)
d)

1 and 2 only
1 and 3 only
2 and 3 only
All of the above

67. A recent study has established the


harmful heath effects of some of the
chemicals used in oil and gas exploration.
The adverse health effects are
1. Low birth weight and congenital birth
defects in infants
2. Abnormal menstrual cycle lengths in
women
3. Cardiovascular diseases in men
Choose the correct answer using the codes below:
a)
b)
c)
d)

1 and 2 only
1 and 3 only
2 and 3 only
All of the above

1. Directive principles of State Policy


2. Fundamental Rights
3. Preamble
Choose the correct answer using the codes below:
a)
b)
c)
d)

69. Which of the following reasons may


explain the discontent of the tribals with
the present economic development in
India?
1. The mineral bowl of India coincides
with the poverty bowl and tribal belts.
2. Limited Success of land reforms in
India
3. Ineffective implementation of tribalfriendly acts like PESA, FRA etc.
Choose the correct answer using the codes below:
a)
b)
c)
d)

1 and 2 only
1 and 3 only
2 and 3 only
All of the above

70. Which of the following cases would most


strengthen the argument that the
inequalities between man and women are
more social than natural?
1. Matrilineal societies in Kerala
2. Women as successful traders and
farmers in African societies
Choose the correct answer.

68. Caste based discrimination is one of the


menaces prevailing in Indian society.
However, the Constitution provides some
safeguards. Which of the following in the
constitution contain provisions that
mandate a non-discriminatory (caste)
society and the upliftment of the
discriminated castes?
http://insightsonindia.com

1 and 2 only
1 and 3 only
2 and 3 only
All of the above

a)
b)
c)
d)

INSIGHTS

1 only
2 only
Both 1 and 2
None

Page 13

71. Which of the following social reformers


did NOT work for the cause of women
upliftment in India?
1. Syed Ahmed Khan
2. Justice Ranade
3. Dayanand Saraswati
Choose the correct answer using the codes below:
a)
b)
c)
d)

1 and 2 only
1 and 3 only
All worked for the cause.
None worked for the cause.

72. In a multi-cultural democracy, which of


the following may lead to the alienation of
the minority?
1. Eliminating local autonomy
2. Adopting one national language
3. Lack of cultural safeguards and rights
Choose the correct answer using the codes below:
a)
b)
c)
d)

1 and 2 only
1 and 3 only
2 and 3 only
All of the above.

73. Consider a situation where a minority


tribal group worships the local forests and
treats them as their god. A private
company wishes to acquire the same
patch of forest. Imagining you are one of
the members of the tribal group, which of
the following constitutional articles can
you cite that will stop the private
company from acquiring the patch of
sacred forest?
a) Articles 14-16
b) Articles 25-28
c) Article 29-30
d) Article 39 (b) & (c) Directive
Principles of State Policy

http://insightsonindia.com

74. Which of the following ideologies can be


counted as Communal ideologies?
1. The interest of religion X is different
from that of religion Y.
2. The interests of religion X are
antagonistic to that of religion Y.
3. The interests of religion X can not be
achieved unless the other religion Y is
subordinated.
Choose the correct answer using the codes below:
a)
b)
c)
d)

1 and 2 only
1 and 3 only
2 and 3 only
All of the above.

75. In the Indian context, which of the


following practices of the government
would NOT be considered Secular?
1. Allowing the Punjabi community not
to wear helmets while driving
2. Acquiring religions trusts
3. Giving subsidies for Haz (to Mecca)
Choose the correct answer using the codes below:
a)
b)
c)
d)

1 and 2 only
3 only
2 and 3 only
None of the above.

76. With reference to ultra mega solar power


projects (UMSPPs), consider the following
statements
1. This scheme was introduced in 2010
by the Ministry of New and
Renewable Energy
2. Under the scheme, an UMSPP should
produce more than 500 MW of power
3. The first project at Sabhar lake in
Rajasthan was opposed by the
Rajasthan government on the ground
of not taking state government into
consideration in funding the project
INSIGHTS

Page 14

a)
b)
c)
d)

Which of the above statements is/are correct?


a)
b)
c)
d)

2 and 3 Only
3 and 1 Only
2 and 1 Only
2 Only

77. With reference to obesity and its


implications, consider the following
statements
1. According to WHO, a person having
Body Mass Index (BMI) of above 40 is
considered as morbidly obese
2. In USA and EU, obesity of all
categories is legally recognized as
disability on par with other disabilities

79. With reference to increasing antibiotic


resistance in the country, consider the
following statements
1. In poultry industry antibiotics is used
as growth promoter to increase weight
and grow fast chickens
2. Increased use of antibiotics in poultry
industry is one of the reasons for
increasing antibiotic resistance in India
Which of the above statements is/are correct?
a)
b)
c)
d)

Which of the above statements is/are correct?


a)
b)
c)
d)

1 Only
2 Only
Both
None

78. With reference to water on the earth,


consider the following statements
1. Scientists believe that the ancient
water found deep below the surface of
the holds more water than the volume
of all of the worlds rivers, swamps
and lakes put together
2. The surface water in all
lakes, swamps and rivers contribute to
10 per cent of all the freshwater on
earths surface. Remaining 90 per cent
of freshwater is stored as either or ice
or buried as groundwater

1 Only
2 Only
Both
None

1 Only
2 Only
Both
None

80. With reference to the Van Allen radiation


belts, consider the following statements
1. It is a zones of highly energetic
charged particles trapped at high
altitudes in the magnetic field of Earth
2. The Van Allen belts are most intense
over the Equator and are effectively
absent above the poles
Which of the above statements is/are correct?
a)
b)
c)
d)

1 Only
2 Only
Both
None

Which of the above statements is/are


INCORRECT?

http://insightsonindia.com

INSIGHTS

Page 15

http://insightsonindia.com

INSIGHTS

Page 16

Insights Mock Tests 2015: Test 10 Solutions


1. Solution: d)
Phase III (of demography of India):The decades 1951-1981 are referred to as the period of
population explosion in India, which was caused by a rapid fall in the mortality rate but a
high fertility rate of population in the country. The average annual growth rate was as high
as 2.2 per cent. It is in this period, after the Independence, that developmental activities were
introduced through a centralised planning process and economy started showing up
ensuring the improvement of living condition of people at large.
Consequently, there was a high natural increase and higher growth rate. Besides, increased
international migration bringing inTibetans, Bangladeshis, Nepalies and even people from
Pakistan contributed to the high growth rate.
Phase IV : In the post 1981 till present, the growth rate of countrys population though
remained high, has started slowing down gradually.
A downward trend of crude birth rate is held responsible for such a population growth. This
was, in turn, affected by an increase in the mean age at marriage, improved quality of life
particularly education of females in the country.
2. Solution: d)
A thorough examination of the pattern of distribution of rural population of India reveals
that both at intra-State and inter- State levels, the relative degree of urbanisation and extent
of rural-urban migration regulate the concentration of rural population.
Contrary to rural population, the proportion of urban population (27.8 per cent) in India is
quite low but it is showing a much faster rate of growth over the decades. In fact since 1931,
the growth rate of urban population has accelerated due to enhanced economic
development and improvement in health and hygienic conditions.
3. Solution: d)
Jains and Buddhists, the smallest religious groups in India have their concentration only in
selected areas of the country.
Jains have major concentration in the urban areas of Rajasthan, Gujarat and Maharashtra,
while the Buddhists are concentrated mostly in Maharashtra. The other areas of Buddhist
majority are Sikkim, Arunachal Pradesh, Ladakh in Jammu & Kashmir, Tripura, and Lahul
and Spiti in Himachal Pradesh.
4. Solution: a)
Suggestions by the UNDP report

The report makes a strong push for universal social protections, noting that countries
like Costa Rice, Ghana and South Korea as well as Scandinavian nations enacted
social security legislation at lower levels of income per capita than India is at right
now.

http://insightsonindia.com

INSIGHTS

Page 1

Insights Mock Tests 2015: Test 10 Solutions

Moreover, the report cites ILO estimates that a basic social floor - universal basic old
age and disability pensions, basic childcare benefits, universal access to essential
health care, social assistance and a 100-day employment scheme would cost India
less than 4 per cent of its GDP. At this stage, universal schemes make more sense for
India than targeted ones.

It says that, Evidence from around the world shows that universal delivery
mechanisms are the most efficient, pragmatic way of providing social services.
Targeting can also be effective, but really only makes sense when there are systems
in place that are advanced and performing optimally.

The UNDPs push for universalism applies to jobs too. It is advocating for countries to
return to the goal of full employment, a goal that it notes has disappeared from the global
agenda since the 1970s.
5. Solution: b)

6. Solution d)

Major gender pointers in UNHDR, 2014

http://insightsonindia.com

INSIGHTS

Page 2

Insights Mock Tests 2015: Test 10 Solutions

Worst gender inequality in South Asia -127th place with Pakistan

India lags far behind BRICS nations too.

Lower HDI than most neighbours. It was higher before 1990s - Since 1990s when
India has liberalised,. Countries like Nepal and Bangaldesh have improved their
social indicators at a faster pace than India.

The UNDP report argues - while progress has been made, it is on shaky grounds on
account of economic crises, social unrest, conflict and climate change.

7. Solution: c)
Migration of females is explained by marriage in the patriarchal setup of india. This is
further proved by evidence and migration patterns.
Migration of males from rural-urban is mostly due to economic reasons in search of
employment to feed their family.
8. Solution: d)
Consequences of Migration
Migration is a response to the uneven distribution of opportunities over space. People tend
to move from place of low opportunity and low safety to the place of higher opportunity
and better safety. This, in turn, creates both benefits and problems for the areas, people
migrate from and migrate to. Consequences can be observed in economic, social, cultural,
political and demographic terms.
Economic Consequences
A major benefit for the source region is the remittance sent by migrants. Remittances from
the international migrants are one of the major sources of foreign exchange. In 2002, India
received US$ 11 billion as remittances from international migrants. Punjab, Kerala and Tamil
Nadu receive very significant amount from their international migrants. The amount of
remittances sent by the internal migrants is very meagre as compared to international
migrants, but it plays an important role in the growth of economy of the source area.
Demographic Consequences
Migration leads to the redistribution of the population within a country. Rural urban
migration is one of the important factors contributing to the population growth of cities.
Age and skill selective out migration from the rural area have adverse effect on the rural
demographic structure. However, high out migration from Uttaranchal, Rajasthan, Madhya
Pradesh and Eastern Maharashtra have brought serious imbalances in age and sex
composition in these states.

http://insightsonindia.com

INSIGHTS

Page 3

Insights Mock Tests 2015: Test 10 Solutions


9. Solution: d)
The disaggregated data of poverty for the states show that there are States like Orissa and
Bihar which have recorded more than 40 per cent of their population living below the
poverty line. The States of Madhya Pradesh, Sikkim, Assam, Tripura, Arunachal Pradesh,
Meghalaya, Nagaland have more than 30 per cent of their population below poverty line.
Poverty is a state of deprivation. In absolute terms it reflects the inability of an individual
to satisfy certain basic needs for a sustained, healthy and reasonably productive living.
Employment rate for educated youth is 25 per cent. Jobless growth and rampant
unemployment are some of the important reasons for higher incidences of poverty in
India.Availability of pre and post natal health care facilities in order to reduce infant
mortality and post delivery deaths among mothers, old age health care, adequate nutrition
and safety of individual are some important measures of a healthy and reasonably long life.
Their lack results in malnutrition; disability etc. leading to poverty.
10. Solution: b)
http://timesofindia.indiatimes.com/india/Major-highlights-of-the-Census2011/articleshow/7833854.cms
11. Solution: d)
Non-metallic minerals are either organic in origin such as fossil fuels also known as mineral
fuels which are derived from the buried animal and plant life such as coal and petroleum.
Other type of non-metallic minerals are inorganic in origin such as mica, limestone and
graphite, etc.Minerals have certain characteristics. These are unevenly distributed over
space. There is inverse relationship in quality and quantity of minerals i.e. good quality
minerals are less in quantity as compared to low quality minerals. The third main
characteristic is that all minerals are exhaustible over time. These take long to develop
geologically and they cannot be replenished immediately at the time of need. Thus, they
have to be conserved and not misused as they do not have the second crop.
12. Solution: a)
http://en.wikipedia.org/wiki/Coal-mining_region#India
India has some of the largest reserves of coal in the world (approx. 267 billion tonnes). The
energy derived from coal in India is about twice that of energy derived from oil, whereas
worldwide, energy derived from coal is about 30% less than energy derived from oil. Over
97 per cent of coal reserves occur in the valleys of Damodar, Sone, Mahanadi and Godavari.
13. Solution: d)
Most of the metallic minerals in India occur in the peninsular plateau region in the old
rystalline rocks. Petroleum reserves are located in the sedimentary basins of Assam, Gujarat
and Mumbai High i.e. off-shore region in the Arabian Sea. New reserves have been located
in the Krishna-Godavari and Kaveri basins. Most of the major mineral resources occur to the
east of a line linking Mangalore and Kanpur.
http://insightsonindia.com

INSIGHTS

Page 4

Insights Mock Tests 2015: Test 10 Solutions


Minerals are generally concentrated in three broad belts in India. There may be some
sporadic occurrences here and there in isolated pockets. These belts are NE plateau region;
South-western plateau region; North-western region.
14. Solution: b)
Forests (land use category): It is important to note that area under actual forest cover is
different from area classified as forest.
The latter is the area which the Government has identified and demarcated for forest
growth. The land revenue records are consistent with the latter definition. Thus, there may
be an increase in this category without any increase in the actual forest cover.
15. Solution: a)
Culturable Waste-Land : Any land which is left fallow (uncultivated) for more than five
years is ncluded in this category. It can be brought under cultivation after improving it
through reclamation practices.
Barren and Wastelands : The land which may be classified as a wasteland such as barren
hilly terrains, desert lands, ravines, etc. normally cannot be brought under cultivation with
the available technology .
Current Fallow : This is the land which is left without cultivation for one or less than one
agricultural year. Fallowing is a cultural practice adopted for giving the land rest. The land
recoups the lost fertility through natural processes.
Fallow other than Current Fallow : This is also a cultivable land which is left uncultivated
for more than a year but less than five years. If the land is left uncultivated for more than
five years, it would be categorised as culturable wasteland.
16. Solution: d)
Land-use in a region, to a large extent, is influenced by the nature of economic activities
carried out in that region. However, while economic activities change over time, land, like
many other natural resources, is fixed in terms of its area. At this stage, one needs to
appreciate three types of changes that an economy undergoes, which affect land-use.
(i) The size of the economy (measured in terms of value for all the goods and services
produced in the economy) grows over time as a result of increasing population, change in
income levels, available technology and associated factors. As a result, the pressure on land
will increase with time and marginal lands would come under use.
(ii) Secondly, the composition of the economy would undergo a change over time. In other
words, the secondary and the tertiary sectors usually grow much faster than the primary
sector, specifically the agricultural sector. This type of change is common in developing
countries like India. This process would result in a gradual shift of land from agricultural
uses to non-agricultural uses.
http://insightsonindia.com

INSIGHTS

Page 5

Insights Mock Tests 2015: Test 10 Solutions


17. Solution: d)
Common Property Resources
Land, according to its ownership can broadly be classified under two broad heads private
land and common property resources (CPRs). While the former is owned by an individual
or a group of individuals, the latter is owned by the state meant for the use of the
community.
CPRs provide fodder for the livestock and fuel for the households along with other minor
forest products like fruits, nuts, fibre, medicinal plants, etc. In rural areas, such land is of
particular relevance for the livelihood of the landless and marginal farmers and other
weaker sections since many of them depend on income from their livestock due to the fact
that they have limited access to land. CPRs also are important for women as most of the
fodder and fuel collection is done by them in rural areas. They have to devote long hours in
collecting fuel and fodder from a degraded area of CPR.
CPRs can be defined as communitys natural resource, where every member has the right of
access and usage with specified obligations, without anybody having property rights over
them. Community forests, pasture lands, village water bodies and other public spaces where
a group larger than a household or family unit exercises rights of use and carries
responsibility of management are examples of CPRs.
18. Solution: d)
Land resource is more crucial to the livelihood of the people depending on agriculture:
(i) Agriculture is a purely land based activity unlike secondary and tertiary activities. In
other words, contribution of land in agricultural output is more compared to its contribution
in the outputs in the other sectors. Thus, lack of access to land is directly correlated with
incidence of poverty in rural areas.
(ii) Quality of land has a direct bearing on the productivity of agriculture, which is not true
for other activities.
(iii) In rural areas, aside from its value as a productive factor, land ownership has a social
value and serves as a security for credit, natural hazards or life contingencies, and also adds
to the social status.
19. Solution: d)

http://insightsonindia.com

INSIGHTS

Page 6

Insights Mock Tests 2015: Test 10 Solutions


20. Solution: d)
Cotton is grown more in Maharashtra. The yield in MH is lower as compared to Kahiwawad
Gujarat. The climatic conditions in Kathiawad Gujarat will yield more cotton.
The groundnut croplands in Gujarat have lower productivity than that in AP (which has
better agroclimatic conditions).
Similarly, Sugarcane is grown more in Bihar, UP; but better agroc-climatic conditions are
better present in MH, TN etc.
21. Solution: a)
Bajra is sown in hot and dry climatic conditions in northwestern and western parts of the
country. It is a hardy crop which resists frequent dry spells and drought in this region. It is
cultivated alone as well as part of mixed cropping. This coarse cereal occupies about 5.2 per
cent of total cropped area in the country.
Leading producers of bajra are the states of Maharashtra, Gujarat, Uttar Pradesh, Rajasthan
and Haryana. Being a rainfed crop, the yield level of this crop is low in Rajasthan and
fluctuates a lot from year to year. Yield of this crop has increased during recent years in
Haryana and Gujarat due to introduction of drought resistant varieties and expansion of
irrigation under it.
22. Solution: d)
Maize is a food as well as fodder crop grown under semi-arid climatic conditions and over
inferior soils. This crop occupies only about 3.6 per cent of total cropped area. Maize
cultivation is not concentrated in any specific region. It is sown all over India except eastern
and north-eastern regions. The leading producers of maize are the states of Madhya
Pradesh, Andhra Pradesh, Karnataka, Rajasthan and Uttar Pradesh. Yield level of maize is
higher than other coarse cereals. It is high in southern states and declines towards central
parts.
23. Solution: c)
Cotton is a tropical crop grown in kharif season in semi-arid areas of the country. India lost a
large proportion of cotton growing area to Pakistan during partition. Per hectare output of
cotton is high under irrigated conditions in north-western region of the country. Its yield is
very low in Maharashtra where it is grown under rainfed conditions.
India lost large jute growing areas to East Pakistan (Bangladesh) during partition. At
present, India produces about three-fifth of jute production of the world.
Sugarcane is a crop of tropical areas. Under rainfed conditions, it is cultivated in sub-humid
and humid climates. But it is largely an irrigated crop in India.

http://insightsonindia.com

INSIGHTS

Page 7

Insights Mock Tests 2015: Test 10 Solutions


24. Solution: c)
Tea is a plantation crop used as beverage. Black tea leaves are fermented whereas green tea
leaves are unfermented. Tea leaves have rich content of caffeine and tannin. It is an
indigenous crop of hills in northern China.
Coffee is a tropical plantation crop. Its seeds are roasted, ground and are used for preparing
a beverage. There are three varieties of coffee i.e. arabica, robusta and liberica. India mostly
grows superior quality coffee, arabica, which is in great demand in International market. But
India produces only about 4.3 per cent coffee of the world and ranks sixth after Brazil,
Vietnam, Colombia, Indonesia and Mexico. Coffee is cultivated in the highlands of Western
Ghats in Karnataka, Kerala and Tamil Nadu.
Karnataka alone accounts for more than twothird of total production of coffee in the
country.
25. Solution: c)
Per hectare output of most of the crops such as rice, wheat, cotton and oilseeds in India is
much lower than that of U.S.A., Russia and Japan. Because of the very high pressure on the
land resources, the labour productivity in Indian agriculture is also very low in comparison
to international level.
Irrigation covers only about 45 per cent of the cultivated area in India. The crop production
in rest of the cultivated land directly depends on rainfall.The vast rainfed areas of the
country, particularly drylands which mostly grow coarse cereals, pulses and oilseeds have
very low yields.
26. Solution: c)
The nature of problems faced by Indian agriculture varies according to agro-ecological and
historical experiences of its different regions.
Hence, most of the agricultural problems in the country are region specific. Yet, there are
some problems which are common and range from physical constraints to institutional
hindrances.
Refer to the section on Problems in Indian agriculture in NCERT 12th geography Chapter
5.
27. Solution: c)
Moreover, as per the Annual Forest survey report, MoEF, an increase in plantation area
resulted in the addition of total forest area in India. Platation crops includes tea, coffee,
rubber etc.
Therefore, the above clearly shows that the actual forest cover and reported cover are
different

http://insightsonindia.com

INSIGHTS

Page 8

Insights Mock Tests 2015: Test 10 Solutions


28. Solution: c)
India has a vast coastline and the coast is very indented in some states. Due to this, a
number of lagoons and lakes have formed. The States like Kerala, Orissa and West Bengal
have vast surface water resources in these lagoons and lakes. Although, water is generally
brackish in these water-bodies, it is used for fishing and irrigating certain varieties of paddy
crops, coconut, etc.
29. Solution: b)

30. Solution: a)
Some states utilise large proportion of their ground water potential which has resulted in
ground water depletion in these states. The over-use of ground water resources has led to
decline in ground water table in these states. In fact, over withdrawals in some states like
Rajasthan, and Maharashtra has increased fluoride concentration in ground-water, and this
practice has led to increase in concentration of arsenic in parts of West Bengal and Bihar.
31. Solution: b)
The legislative provisions such as the Water (Prevention and Control of Pollution) Act 1974,
and Environment Protection Act 1986 have not been implemented effectively. The result is
that in 1997, 251 polluting industries were located along the rivers and lakes. The Water Cess
Act, 1977, meant to reduce pollution has also made marginal impacts. There is a strong need
to generate public awareness about importance of water and impacts of water pollution. The
public awareness and action can be very effective in reducing the pollutants from
agricultural activities, domestic and industrial discharges.
32. Solution: d)
The concept of watershed development is an all encompassing one. Refer to:
http://chirag.org/what-we-do/natural-resource-management/conservation
33. Solution: d)
http://en.wikipedia.org/wiki/Saltwater_intrusion
http://insightsonindia.com

INSIGHTS

Page 9

Insights Mock Tests 2015: Test 10 Solutions


34. Solution: d)
Rainwater harvesting increases water availability, checks the declining ground water table,
improves the quality of groundwater through dilution of contaminants like fluoride and
nitrates, prevents soil erosion, and flooding and arrests salt water intrusion in coastal areas if
used to recharge aquifers.
35. Solution: b)
http://pib.nic.in/newsite/erelease.aspx?relid=90775
36. Solution: a)
The Water Footprint of a product is the volume of freshwater appropriated to produce the
product, taking into account the volumes of water consumed and polluted in the different
steps of the supply chain.
The NWP, 2012 calls for a system to evolve benchmarks for water uses for different
purposes, i.e., water footprints, and water auditing be developed to ensure efficient use of
water. Project financing has been suggested as a tool to incentivize efficient & economic use
of water.
37. Solution: a)
http://www.unisdr.org/we/coordinate/hfa
38. Solution: c)
Situation in India

India vulnerable to regular natural hazards due to unique geo-climatic conditions.

85% to single or multiple disasters and

About 57% of its area lies in high seismic zones.

Approximately 40 million hectares (10%) of the countrys land area is prone to flood,

About 8% of the total land mass is vulnerable to cyclone

68% of the area is susceptible to drought

World Bank- 2.25% of the GDP and 12.15% of the revenue of the country were lost due to
natural disasters during 1996-2000.
39. Solution: d)
National Geophysical Laboratory, Geological Survey of India, Department of Meteorology,
Government of India, along with the recently formed National Institute of Disaster Management,
have made an intensive analysis of more than 1,200 earthquakes that have occurred in India

http://insightsonindia.com

INSIGHTS

Page 10

Insights Mock Tests 2015: Test 10 Solutions


in different years in the past, and based on these, they divided India into the following five
earthquake zones:

40. Solution: a)
Earthquakes and volcanic eruptions that cause the sea-floor to move abruptly resulting in
sudden displacement of ocean water in the form of high vertical waves are called tsunamis
(harbour waves) or seismic sea waves.
Normally, the seismic waves cause only one instantaneous vertical wave; but, after the
initial disturbance, a series of afterwaves are created in the water that oscillate between high
crest and low trough in order to restore the water level.
41. Solution: a)
The speed of wave in the ocean depends upon the depth of water. It is more in the shallow
water than in the ocean deep. As a result of this, the impact of tsunami is less over the ocean
and more near the coast where they cause large-scale devastations. Therefore, a ship at sea is
not much affected by tsunami and it is difficult to detect a tsunami in the deeper parts of sea.
It is so because over deep water the tsunami has very long wave-length and limited waveheight. Thus, a tsunami wave raises the ship only a metre or two and each rise and fall takes
several minutes. As opposed to this, when a tsunami enters shallow water, its wave-length
gets reduced and the period remains unchanged, which increases the waveheight.
Sometimes, this height can be up to 15m or more, which causes large-scale destructions
along the shores. Thus, these are also called Shallow Water Waves.
42. Solution: a)
http://insightsonindia.com

INSIGHTS

Page 11

Insights Mock Tests 2015: Test 10 Solutions


Tropical cyclones are intense low-pressur areas confined to the area lying between 30 N
and 30 S latitudes, in the atmosphere around which high velocity winds blow.
Horizontally, it extends up to 500-1,000 km and vertically from surface to 12-14 km. A
tropical cyclone or hurricane is like a heat engine that is energised by the release of latent
heat on account of the condensation of moisture that the wind gathers after moving over the
oceans and seas.
43. Solution: c)
Some initial conditions for the emergence of a tropical cyclone are:
(i) Large and continuous supply of warm and moist air that can release enormous latent
heat.
(ii) Strong Coriolis force that can prevent filling of low pressure at the centre (absence of
Coriolis force near the equator prohibits the formation of tropical cyclone between 0-5
latitude).
(iii) Unstable condition through the troposphere that creates local disturbances around
which a cyclone develops.
(iv) Finally, absence of strong vertical wind wedge, which disturbs the vertical transport of
latent heat.
44. Solution: d)
More cyclones in Bay of Bengal occur due to:

Narrow bay of Bengal

Difference in salinity

Cyclones move east to west

Arabian sea is colder

Phallin came from Indo-pacific so frequency on east coast is greater

45. Solution: d)
Unlike other natural disasters, human beings play an important role in the genesis as well as
spread of floods.
Indiscriminate deforestation, unscientific agricultural practices, disturbances along the
natural drainage channels and colonisation of flood-plains and river-beds are some of the
human activities that play an important role in increasing the intensity, magnitude and
gravity of floods.

http://insightsonindia.com

INSIGHTS

Page 12

Insights Mock Tests 2015: Test 10 Solutions


46. Solution: a)
Types of Droughts
Meteorological Drought: It is a situation when there is a prolonged period of inadequate
rainfall marked with mal-distribution of the same over time and space.
Agricultural Drought: It is also known as soil moisture drought, characterised by low soil
moisture that is necessary to support the crops, thereby resulting in crop failures. Moreover,
if an area has more than 30 per cent of its gross cropped area under irrigation, the area is
excluded from the drought-prone category.
Hydrological Drought: It results when the availability of water in different storages and
reservoirs like aquifers, lakes, reservoirs, etc. falls below what the precipitation can
replenish.
47. Solution: a)
NDMA 2005 was adopted
The Act provides for establishment of:

National Disaster Management Authority (NDMA) - lays down broad policies,


guidelines etc.

Under it NEC(Secy heads) - coordinating, monitoring, assistance as well as


implementation of emergency relief and disaster response

State Disaster Management Authority (SDMA)

District Disaster Management Authority (DDMA)

The Act also provides for

Constitution of Disaster Response Fund and Disaster Mitigation Fund at National,


State and District levels.

Establishment of NIDM and NDRF.

Provides penalties for obstruction, false claims, misappropriation etc.

48. Solution: a)
High Rocky coast
The west coast of our country is a high rocky retreating coast. Erosional forms dominate in
the west coast. Whereas, the east coast of India is a low sedimentary coast. Depositional
forms dominate in the east coast.

http://insightsonindia.com

INSIGHTS

Page 13

Insights Mock Tests 2015: Test 10 Solutions


Features:

Highly indented

Formation of wave-cut platforms in front of the sea cliff due to constant water
erosional action

Material eroded in such manner, deposits along the off-shore forming wave terraces.

Several depositions like this results in the formation of barrier bars and spits that
break block the sea water forming a lagoon.

Low Sedimentary coast


Features:

Lagoons, deltas etc. found

Lagoons eventually turn into swamps which turn into coastal plains

Storm and tsunami waves cause darastic changes in the supply of depositional
material

Therefore, it is easy to construct seaports on western coasts.


49. Solution: a)
Delhi Police have launched Operation Milap scheme. This noble scheme aims at uniting
missing children with their parents.
Under the scheme

This scheme has been launched by the Anti-Human Trafficking Unit (AHTU) of the
Crime Branch of Delhi Police.

Under this scheme, AHTU will carry out special drive to check all the Children
Homes in the national capital. They will try to match the childrens record with the
available data of missing/kidnapped children with the police.

During this special drive, Crime Branch officials will try to make all out efforts to get
any clue about their parents/home so that they can be restored properly.

They will also take help of local police who have being asked to screen and
document all the children residing in the shelter homes, railway platforms, bus
stands, roads, religious places, etc.

Photographic database of these children will be undertaken by the concerned police


officials to link them properly.

http://insightsonindia.com

INSIGHTS

Page 14

Insights Mock Tests 2015: Test 10 Solutions


50. Solution: d)
Indian Space Research Organisation (ISROs) first sub-orbital flight and Indias latest
generation launch vehicle- GSLV Mark-III X was successfully lifted off from Satish Dhawan
Space Centre, Sriharikota,Andhra Pradesh.
About GSLV-Mark III
The GSLV-Mark III is a three stage/engine launch vehicle.
Its first stage comprises two identical S-200 large solid boosters with 200 tonne solid
propellant that are strapped on to the second stage, the L110 re-startable liquid stage.
The third stage/engine is the cryogenic which is more efficient as it provides more thrust for
every kilogram of propellant burnt.
Thus, this successful launch will help India in perfecting the cryogenic engine technology
and help to become self-reliant in launching communication satellites.
51. Solution: d)

This award is given by President of India to the heads of panchayats that did well in
maintaining hygiene in their respective villages by operating women sanitary
complexes well, preventing open area defecation, maintaining school hygiene etc.

It was first given in 2005 as a component of its flagship scheme- Total Sanitation
Campaign (TSC).

Till 2011, it was given by Union Ministry of Drinking Water and Sanitation
(MoDWS). But since 2012, it was taken up by the States, while selection of the Blocks
and District Panchayats continued with the Centre.

Gram Panchayats winning NGP usually functions as training centers for people
from other panchayat raj institutions aspiring to achieve full sanitation coverage.

52. Solution: d)
Indias first indigenously developed nuclear powered ballistic missile submarine- INS
Arihant was flagged off by the Defence Minister Manohar Parrikar from Vizag for sea trials.
In maiden sea trial, Arihants light water reactor will undergo comprehensive validation
before it is commissioned into the Navy.

Its successful implementation will help India to secure a seaborne nuclear deterrent.

Once it is inducted in navy will help India to complete its nuclear triad of delivering
nuclear weapons from land, sea and air.

A nuclear triad refers to a nuclear arsenal which consists of three components, traditionally
strategic bombers, intercontinental ballistic missiles (ICBMs), and submarine-launched
http://insightsonindia.com

INSIGHTS

Page 15

Insights Mock Tests 2015: Test 10 Solutions


ballistic missiles (SLBMs). The purpose of having a three-branched nuclear capability is to
significantly reduce the possibility that an enemy could destroy all of a nation's nuclear
forces in a first-strike attack; this, in turn, ensures a credible threat of a second strike, and
thus increases a nation's nuclear deterrence
53. Solution: a)
Rajasthan Government has started distribution of the Bhamashah cards among women
belonging to Below Poverty Line (BPL) category in Ajmer. The cards were distributed under
the Bhamashah scheme which was relaunched in August 2014.
This scheme aims to financially empower BPL women by providing them with monetary
benefits of various government schemes, as money will be transferred directly to their bank
accounts.

This scheme will provide woman head of the family with a bank account. Monetary
benefits from various schemes will be channelized in this account.

Beneficiaries will be provided with a card under this scheme. This card will also be
used as an identity card for the people of Rajasthan.

Rajasthan government aims to open around 1.5 crore bank accounts under this
scheme.

Earlier Bhamashah scheme


Previously it was launched in 2008, by Vasundhara Raje Government.
Under this scheme 8000 cards were issued and 29.07 lakh bank accounts were opened.
Government also had deposited 160 crore rupees in the banks for its implementation.
However, in 2009, it was discontinued by then Ashok Gehlot government.
54. Solution: a)
Sundrban has worlds largest mangrove forest which is home to wide range of fauna,
including 260 bird species, the Bengal tiger and other threatened species such as the
estuarine crocodile and the Indian python. It is also home to the rare Irrawaddy dolphin.
The oil spills has already blackened the shoreline and started degrading the water quality,
threatening trees, plankton, vast populations of small fishes.
Rare Irrawaddy dolphin may be the first victim of this oil spill as the thick layer of oil on the
surface of the river may cut down the dissolved oxygen, leading to suffocation of these
dolphins due to lack of oxygen.
It may even threaten Sunderban tigers as there feed on herbivores animals like deer and
other are going to suffer from oil spill as their vegetation may be covered by a thick layer of
oil and rinse into the soil once the water recedes during the tides.
http://insightsonindia.com

INSIGHTS

Page 16

Insights Mock Tests 2015: Test 10 Solutions


55. Solution: d)
It was approved by The Cabinet Committee on Economic Affairs (CCEA) chaired by Prime
Minister after it was proposed by the Union ministry of commerce & industry.
This scheme aims at

Supporting development of improved varieties and technologies for increasing


production, productivity and quality of tea.

Promote Indian tea in the overseas and domestic markets.

As per the scheme, the proposed fund will be spent on plantation development, quality upgradation and product diversification, market promotion and research and development.
56. Solution: a)
UN has adopted this resolution under the agenda of Global Health and Foreign Policy.
UN recognised that Yoga provides a holistic approach to health and well-being and its
wider dissemination of information about benefits of practising Yoga will be beneficial for
the health of the world population.
Resolution in this regard was introduced by Indias Ambassador to the United Nations
Asoke Mukerji and 175 nations as co-sponsors for it. It was the highest number ever for any
resolution in the 193-member UN General Assembly
Passing of this resolution also made history as it was for the first time that such an initiative
has been proposed and implemented by any country in the UN body in less than 90 days.
Background
Earlier in September 2014, Prime Minister had proposed this idea during his speech at UNGA. He had suggested 21st June because it represents one of the two solstices, is the longest
day in the Northern Hemisphere and has special significance in many parts of the world.
Significance
The main significance of the UN declaring an International Day is to focus the attention of
the international community on the topic and to encourage activities among the
member states to commemorate the day.
Till date, UN in its annual calendar has listed nearly 118 international days, years and
anniversaries for observance.

http://insightsonindia.com

INSIGHTS

Page 17

Insights Mock Tests 2015: Test 10 Solutions


57. Solution: a)
The Various measures taken for quality improvement and availability of essential factors
like fertilizers, seeds, electricity and irrigation facilities for agriculture in order to make it a
profitable business include the following:
The Government is implementing various Missions, Schemes and Projects which facilitate
production, availability & distribution of quality seeds and fertilizers to farmers. The
Government of India has launched a new schemes viz. Deendayal Upadhyaya Gram Jyoti
Yojana (DDUGJY) which aims to provide reliable and adequate power supply to farmers by
separation of agriculture and non-agriculture feeders and strengthening of sub-transmission
and distribution infrastructure in rural areas, among others. Rajiv Gandhi Grameen
Vidyutikaran Yojana for rural electrification will get subsumed in DDUGJY.
From PIB features.

58. Solution: a)
Water is a State subject; water resources/ irrigation projects are planned, executed and
maintained by State Governments from their own resources and as per their own priorities.
Government of India provides financial and technical assistance to the States under
Accelerated Irrigation Benefits Programme, Repair, Renovation and Restoration of Water
Bodies Scheme and CADWM Programme, to enhance the irrigation potential of the country
as well as to ensure its effective utilization. Financial assistance is also being provided to
farmers for micro irrigation and protected cultivation under various schemes viz., Mission
for Integrated Development of Horticulture (MIDH), On-Farm Water Management (OFWM)
under National Mission for Sustainable Agriculture (NMSA) and Rashtriya Krishi Vikas
Yojana (RKVY).
From PIB features.

59. Solution: d)
The Government is implementing many Missions/ Schemes such as Rashtriya Krishi Vikas
Yojana (RKVY), National Food Security Mission (NFSM), Mission for Integrated
Development of Horticulture (MIDH), National Mission on Oilseeds and Oil Palm
(NMOOP) and Grameen Bhandaran Yojana etc. for raising investments in agriculture. In
addition, Government has issued a framework for Public Private Partnership for Integrated
Agriculture Development (PPPIAD) for using RKVY allocation for bringing greater
association of private sector in agricultural development projects in the States.
The Krishi Vigyan Kendras (KVKs) also organise quality seed production and distribution to
farmers .During the last one year 1.57 lakh quintal seeds of improved varieties and hybrids
of cereals. Oilseeds, pulses, commercial crops, vegetables, flowers, fruits, spices, fodder,
http://insightsonindia.com

INSIGHTS

Page 18

Insights Mock Tests 2015: Test 10 Solutions


forest species, medicinal plants and fibre crops were produced and provided to 2.61 lakh
farmers by KVKs.
Fertilizer (Control) Order, 1985 has been promulgated for regulation of quality of fertilisers.
No person shall manufacture/import for sale, sell, offer for sale, stock or exhibit for sale or
distribute any fertiliser which is not notified in the said Order or not of standard prescribed
in the said Order. Samples of fertilizers are drawn periodically by notified fertiliser
inspectors of State Governments to check their quality whereas in case of imported
fertilisers, the fertiliser inspectors of the Central Government draw samples from
ships/containers for checking their quality.
PIB features.
60. Solution: d)
The IPDS announced in the Union Budget 2014-15 envisages strengthening of subtransmission network, Metering, IT application, Customer Care Services, provisioning of
solar panels and the completion of the ongoing works of Restructured Accelerated Power
Development and completion of the Reforms Programme (RAPDRP). The scheme will help
in reduction in AT&C losses, establishment of IT enabled energy accounting / auditing
system, improvement in billed energy based on metered consumption and improvement in
collection efficiency.
61. Solution: a)
http://www.downtoearth.org.in/content/estimate-world-s-oldest-water
62. Solution: d)
http://www.downtoearth.org.in/content/unep-launches-coalition-promote-renewableenergy
63. Solution: c)
Read the full page. This topic is a favourite of the UPSC.
http://www.forestrightsact.com/what-is-this-act-about
http://www.downtoearth.org.in/content/claims-forest-rights-act-holding-growth-arebogus
64. Solution: d)
http://www.downtoearth.org.in/content/economic-potential-carbon-rich-mangrovesuntapped-report

http://insightsonindia.com

INSIGHTS

Page 19

Insights Mock Tests 2015: Test 10 Solutions


65. Solution: d)
http://www.nhs.uk/chq/Pages/2145.aspx?CategoryID=51
http://www.downtoearth.org.in/content/fssai-notifies-draft-regulations-limit-trans-fats-5cent-2016
66. Solution: d)
http://www.downtoearth.org.in/content/antibiotics-chicken-mps-question-governmentaction-taken-after-cse-study
67. Solution: a)
http://www.downtoearth.org.in/content/chemicals-used-oil-and-gas-extraction-harmfulreproductive-health
68. Solution: d)
Preamble clearly says that the Indian constitution promotes the ideal of social justice.
DPSP Article 46 provides for the welfare of the backward classes.
Fundamental Rights Article 14-17 provide for non-discrimination against any particular
caste.
69. Solution: d)
Development and Extremism in tribals
1st phase

Failure of land reforms

Mineral bowl of India coincides with poverty and tribal bowl of India

Spread in 9 states
o

Political neglect

Mining -eco unsustainable and sociologically devastating

Insensitive forest administration and non implementation of PESA and FRA

Tribal displamcnemt, disconnect, deprivation and discontent

Leading to alienation

Why displacement

Wildlife sanctuaries

Mining

http://insightsonindia.com

INSIGHTS

Page 20

Insights Mock Tests 2015: Test 10 Solutions

Normal development projects like dams

70. Solution: c)
Because of the obvious biological and physical differences between men and women, gender
inequality is often treated as natural. However, despite appearances, scholars have shown
that the inequalities between men and women are social rather than natural. For example,
there are no biological reasons that can explain why so few women are found in positions of
public power. Nor can nature explain why women generally receive a smaller or no share in
family property in most societies. But the strongest argument comes from the societies that
were different from the normal or common pattern. If women were biologically unfit to be
inheritors and heads of families, how did matrilineal societies (as the Nairs of Kerala used to
be, and as the Khasis of Meghalaya still are) work for centuries? How have women managed
to be successful farmers and traders in so many African societies? There is, in short, nothing
biological about the inequalities that mark the relations between and men.
71. Solution: c)
Refer to the section 5.3 STRUGGLE FOR WOMENS EQUALITY AND RIGHTS Chapter 512th Indian society NCERT.
72. Solution: d)
Assimilationist and integrationist strategies try to establish singular national identities
through various interventions like:

Centralising all power to forums where the dominant group constitutes a majority,
and eliminating the autonomy of local or minority groups;
Imposing a unified legal and judicial system based on the dominant groups
traditions and abolishing alternative systems used by other groups;
Adopting the dominant groups language as the only official national language and
making its use mandatory in all public institutions;
Promotion of the dominant groups language and culture through nationa
linstitutions including state-controlled media and educational institutions;
Adoption of state symbols celebrating the dominant groups history, heroes and
culture, reflected in such things as choice of national holidays or naming of streets
etc.;
Seizure of lands, forests and fisheries from minority groups and indigenous people
and declaring them national resources

73. Solution: c)
Article 14-18 talk about non-discrimination and social and economic justice.
Articles 25-28 give religious rights.

http://insightsonindia.com

INSIGHTS

Page 21

Insights Mock Tests 2015: Test 10 Solutions


Articles 29-30 give the minorities a right to safeguard their cultural heritage- the forest in
this case. The Odisha Niyamgiri tribals also saved their sacred hills using this provision of
the constitution.
74. Solution: d)
All are communal ideologies.
In everyday language, the word communalism refers to aggressive chauvinism based on
religious identity. Chauvinism itself is an attitude that sees ones own group as the only
legitimate or worthy group, with other groups being seen by definition as inferior,
illegitimate and opposed. Thus, to simplify further, communalism is an aggressive political
ideology linked to religion. This is a peculiarly Indian, or perhaps South Asian, meaning that
is different from the sense of the ordinary English word. In the English language,
communal means something related to a community or collectivity as different from an
individual.
75. Solution: d)
One kind of difficulty is created by the tension between the western sense of the state
maintaining a distance from all religions and the Indian sense of the state giving equal
respect to all religions. Supporters of each sense are upset by whatever the state does to
uphold the other sense. Should a secular state provide subsidies for the Haj pilgrimage, or
manage the Tirupati-Tirumala temple complex, or support pilgrimages to Himalayan holy
places? Should all religious holidays be abolished, leaving only Independence Day, Republic
Day, Gandhi Jayanti and Ambedkar Jayanti for example? Should a secular state ban cow
slaughter because cows are holy for a particular religion? If it does so, should it also ban pig
slaughter because another religion prohibits the eating of pork?If Sikh soldiers in the army
are allowed to have long hair and wear turbans, should Hindu soldiers also be allowed to
shave their heads or Muslim soldiers allowed to have long beards? Questions of this sort
lead to passionate disagreements that are hard to settle.
Some of the above mentioned dilemmas are solved by the Indian model of Secularism by
positive discrimination and maintaining a principled distance from religion.
76. Solution: d)
The President of India has sanctioned (18 December 2014) the Scheme for Development of
Solar Parks and Ultra Mega Solar Power Projects in the country. The scheme envisages at
least 25 solar parks, each with a minimum capacity of 500 MW between 2014 and 2019. The
estimated financial support for the schemeRs 4,050 crorewas also approved.
Apart from solar parks, ultra mega solar power projects (UMSPPs) are also encouraged in
the scheme. A UMSPP is a single power project with capacity of over 500 MW. These
projects may be set up in some of these solar parks. In some cases, the full park may be one
UMSPP.

http://insightsonindia.com

INSIGHTS

Page 22

Insights Mock Tests 2015: Test 10 Solutions


MNRE wanted to set up four UMSPPs of 4,000 MW using solar photovoltaic (PV)
technology near Sambhar lake in Rajasthan, Kargil and Leh in Jammu and Kashmir and
Gujarat. The plant in Rajasthan was opposed by the state government itself (see Solar threat
to Sambhar). In January 2014, the state chief minister, Vasundhara Raje, wrote to the then
Union heavy industries minister, Praful Patel, expressing her reservations about the project
on the largest inland saline wetland in Asia.
http://www.downtoearth.org.in/content/scheme-large-scale-solar-projects-getspresidential-sanction
77. Solution: a)
http://www.downtoearth.org.in/content/obesity-can-be-counted-disability-says-eu-court
78. Solution: b)
Out of all the water on Earth, saline water in oceans, seas and saline groundwater make up
about 97% of it. Only 2.52.75% is fresh water, including 1.752% frozen
in glaciers, ice and snow, 0.70.8% as fresh groundwater and soil moisture, and less than
0.01% of it as surface water in lakes, swamps and rivers.[3][4] Freshwater lakes contain about
87% of this fresh surface water, including 29% in the African Great Lakes, 20% in Lake
Baikal in Russia, 21% in the North American Great Lakes, and 14% in other
lakes. Swamps have most of the balance with only a small amount in rivers, most notably
the Amazon River. The atmosphere contains 0.04% water. In areas with no fresh water on
the ground surface, fresh water derived from precipitationmay, because of its lower density,
overlie saline ground water in lenses or layers. Most of the world's fresh water is frozen
inice sheets. Many areas suffer from lack of distribution of fresh water, such as deserts.
http://www.downtoearth.org.in/content/estimate-world-s-oldest-water
79. Solution: c)
http://www.downtoearth.org.in/content/hatching-superbugs (Very important for Mains
too. See this years GS-3 question paper)
80. Solution: c)
Van Allen radiation belt, doughnut-shaped zones of highly energetic charged particles
trapped at high altitudes in the magnetic field of Earth. The zones were named for James A.
Van Allen, the American physicist who discovered them in 1958, using data transmitted by
the U.S. Explorer satellite.
The Van Allen belts are most intense over the Equator and are effectively absent above the
poles. No real gap exists between the two zones; they actually merge gradually, with the
flux of charged particles showing two regions of maximum density. The inner region is
centred approximately 3,000 km (1,860 miles) above the terrestrial surface. The outer region
http://insightsonindia.com

INSIGHTS

Page 23

Insights Mock Tests 2015: Test 10 Solutions


of maximum density is centred at an altitude of about 15,000 to 20,000 km (9,300 to 12,400
miles), though some estimates place it as far above the surface as six Earth radii (about
38,000 km [23,700 miles]).
The inner Van Allen belt consists largely of highly energetic protons, with energyexceeding
30,000,000 electron volts. The peak intensity of these protons is approximately 20,000
particles per second crossing a spherical area of one square cm in all directions. It is believed
that the protons of the inner belt originate from the decay of neutrons produced when highenergy cosmic rays from outside thesolar system collide with atoms and molecules of
Earths atmosphere. Some of the neutrons are ejected back from the atmosphere; as they
travel through the region of the belt, a small percentage of them decay into protons
and electrons. These particles move in spiral paths along the lines of force of Earths
magnetic field. As the particles approach either of the magnetic poles, the increase in the
strength of the field causes them to be reflected. Because of this so-called magnetic
mirroreffect, the particles bounce back and forth between the magnetic poles. Over time,
they collide with atoms in the thin atmosphere, resulting in their removal from the belt.
The outer Van Allen belt contains charged particles of both atmospheric and solar origin, the
latter consisting largely of helium ions from the solar wind (steady stream of particles
emanating from the Sun). The protons of the outer belt have much lower energies than those
of the inner belt, and their fluxes are much higher. The most energetic particles of the outer
belt are electrons, whose energies reach up to several hundred million electron volts.
Studies show that intense solar activity, such as a coronal mass ejection, may sometimes
diminish the outer region and produce a third fleeting zone of charged particles between the
outer and inner regions. Intense solar activity also causes other disruptions of the Van Allen
belts, which in turn are linked with such phenomena as auroras and magnetic storms

http://insightsonindia.com

INSIGHTS

Page 24

INSIGHTS ON INDIA MOCK PRELIMINARY EXAM - 2015


INSIGHTS ON INDIA MOCK TEST - 11
GENERAL STUDIES

PAPER-I
Time Allowed: 2 Hours

Maximum Marks: 200

INSTRUCTIONS
1. IMMEDITELY AFTER THE COMMENCEMENT OF THE EXAMINATION, YOU SHOULD
CHECK THAT THIS TEST BOOKLET DOES NOT HAVE ANY UNPRINTED OR TORN OR MISSING
PAGES OR ITEMS, ETC. IF SO, GET IT REPLACED BY A COMPLETE TEST BOOKLET.
2. You have to enter your Roll Number on the Test
Booklet in the Box provided alongside. DO NOT
Write anything else on the Test Booklet.
4. This Test Booklet contains 100 items (questions). Each item is printed only in English. Each item
comprises four responses (answers). You will select the response which you want to mark on the
Answer Sheet. In case you feel that there is more than one correct response, mark the response which
you consider the best. In any case, choose ONLY ONE response for each item.
5. You have to mark all your responses ONLY on the separate Answer Sheet provided. See directions in
the Answer Sheet.
6. All items carry equal marks.
7. Before you proceed to mark in the Answer Sheet the response to various items in the Test Booklet, you
have to fill in some particulars in the Answer Sheet as per instructions sent to you with your
Admission Certificate.
8. After you have completed filling in all your responses on the Answer Sheet and the examination has
concluded, you should hand over to the Invigilator only the Answer Sheet. You are permitted to take
away with you the Test Booklet.
9. Sheets for rough work are appended in the Test Booklet at the end.
10. Penalty for wrong answers :
THERE WILL BE PENALTY FOR WRONG ANSWERS MARKED BY A CANDIDATE IN THE
OBJECTIVE TYPE QUESTION PAPERS.
(i) There are four alternatives for the answer to every question. For each question for which a
wrong answer has been given by the candidate, one-third of the marks assigned to that
question will be deducted as penalty.
(ii) If a candidate gives more than one answer, it will be treated as a wrong answer even if one of
the given answers happens to be correct and there will be same penalty as above to that
question.
(iii)

If a question is left blank, i.e., no answer is given by the candidate, there will be no penalty
for that question.
http://www.insightsonindia.com

INSIGHTS ON INDIA MOCK TEST SERIES FOR CIVIL SERVICES PRELIMINARY EXAM 2015
http://insightsonindia.com

INSIGHTS

Page 1

1. Which of the following can be said to be


legacy of colonialism in India?
1. Tribals not being allowed in forest
pastoral lands
2. Growth of rationality and a scientific
temper in the masses
3. Indian police system

4. Which of the following were involved in


reducing or eliminating unjust social
discrimination in pre-colonial India?
1. Buddhism
2. Bhakti movement
3. Sufi movement
Choose the correct answer using the codes below:

Choose the correct answer using the codes below:


a) 1 and 2
b) 2 and 3
c) 1 and 3
d) All of the above
2. Consider the following statements:
1. Sanskrit works on astronomy,
mathematics and medicine had been
translated into Arabic fifth century
onwards
2. Al-Birunis book Rihla deals
extensively with the Hindu culture in
the region of then Punjab
Which of the above statements is/are correct?
a) 1 Only
b) 2 Only
c) Both
d) None
3. Interest rate differential between two
countries in the short-run can result in
1. Change of investment flows across
these two nations
2. Change in exchange rate of these two
countries
3. Change in trade flows across these
nations

a)
b)
c)
d)

5. Which of the following is/are NOT


associated with the cause of women social
reform movements in India?
1. All India Muslim Ladies Conference
2. Arya Samaj in Punjab
3. Pandita Ramabai
Choose the correct answer using the codes below:
a)
b)
c)
d)

Choose the correct answer using the codes below:


a)
b)
c)
d)

1 and 2 only
1 and 3 only
2 and 3 only
All of the above

http://insightsonindia.com

1 and 2
2 only
1 and 3
All are associated

6. The concept of Participatory democracy


in play can be seen in which of the
following bodies?
1. Gram Sabha
2. State Legislative assembly
3. District Planning Committee

Choose the correct answer using the codes


below:
a)
b)
c)
d)

1 and 2
2 and 3
1 and 3
All of the above

INSIGHTS

3 only
1 and 3
1 only
All of the above

Page 2

7. Consider the following statements about


Revenue expenditure.
1. It is provided for in every annual
budget of the government.
2. Some part of revenue expenditure may
be used for creation of physical assets.
3. Interest payments on debt incurred by
the government do NOT form part of
revenue expenditure.
Choose the correct answer using the codes below:
a)
b)
c)
d)

1 and 2 only
2 and 3 only
1 and 3 only
All of the above

8. In Germany, renewable energy-based


electricity generation has crossed 25 per
cent of the gross electricity produced.
Which of the following is the major source
of its renewable energy production?
a) Onshore wind energy
b) Offshore wind energy
c) Solar energy
d) Geothermal energy
9. Consider the following statements:
1. Ibn Battuta was lured by his reputation
as a generous patron of arts and
letters, and he set off for Delhi, passing
through Multan and Uch
2. Ibn Battuta was ordered in 1342 to
proceed to China as the Sultans envoy
to the Mongol ruler
In the above statements, the Sultan refers to
a)
b)
c)
d)

Muhammad Ghazni
Ghiyath al-Din Muhammad
Muhammad bin Tughlaq
None of the above

http://insightsonindia.com

10. Which of the following would most


clearly reflect the international
competitiveness of the Indian economy?
a) Nominal effective Exchange Rate
(NEER)
b) Real Effective Exchange Rate (REER)
c) Real Exchange Rate (RER)
d) Nominal Exchange Rate (NER)

11. Which of the following can be said to be


the sources of the core values of the Indian
constitution?
1. Karachi session of Congress, 1931
2. Cabinet Mission Plan 1946
3. 1916 Lucknow Pact
Choose the correct answer using the codes
below:
a) 3 only
b) 1 and 3
c) 1 only
d) All of the above
12. When the constitution was finalised
panchayats did not find a mention in it in
compulsory provisions. Which of the
following can be cited as the most
appropriate reason among the following?
a) Local elites and upper castes were so
well entrenched in society that local
self-government only meant a
continuing exploitation of the
downtrodden masses of Indian society
b) There was a fierce opposition in the
Constituent assembly that panchayats
as self-governing bodies will collapse
immediately on establishment
c) It was difficult to monitor the activities
of the local bureaucracy from the
Centre
d) In a federal model of polity, the
constitution makers did not want to
add a third tire because of the
complexities it will create.
INSIGHTS

Page 3

13. Capital expenditure of the government


results in creation of physical assets. It is
regulated by
1. Fiscal Responsibility and Management
(FRBM) Act
2. RBI
3. International treaties
Choose the correct answer using the codes below:
a)
b)
c)
d)

Choose the correct answer using the codes below:

1 and 2 only
2 and 3 only
1 only
All of the above

14. Which of the following national parks


would be affected if the proposed KenBetwa River linking project takes off?
a) Panna National Park
b) Bandhavgarh National Park
c) Kanha National Park
d) None of the above
15. Consider the following statements:
1. He was closely associated with the
Mughal court, as a physician to Prince
Dara Shukoh
2. He a Frenchman, was a doctor,
political philosopher and historian
3. He travelled to several parts of the
country, and wrote accounts of what
he saw, frequently comparing what he
saw in India with the situation in
Europe
In the above statements, He refers to?
a)
b)
c)
d)

1. By law women are supposed to have


an equal share of family property.
2. Small and marginal farmers make for
more than a majority in the rural
agricultural setup.
3. Most agricultural labourers are daily
wage workers.

Duarte Barbosa
Jean-Baptiste Tavernier
Manucci
Franois Bernier

16. Consider the following statements about


the rural society in India.
http://insightsonindia.com

a)
b)
c)
d)

1 and 2 only
2 and 3 only
1 and 3 only
All of the above

17. Gross investment in an economy


essentially means
a) Measure of final output comprising
capital goods
b) Total infrastructure buildup in the
economy
c) Aggregate monetary resources
available with all the banks in the
nation
d) Sum total value of the consumer and
capital goods in the economy
18. Benami transfers were one of the major
causes of the failure of the effective
implementation of the Land ceiling
provisions as a part of land reforms in
India. Benami transfers meant that
a) landowners managed to divide the
land among relatives and others which
allowed them to keep control over the
lands
b) Property was declared sold and
unnamed so that it escapes the
provisions of land ceiling act
c) The zamindars transferred the excess
land to the peasants first and then
bought it back from them again
d) None of the above

INSIGHTS

Page 4

19. If there is no current account deficit (CAD)


and fiscal deficit in an economy, then
ideally
a) investment in the economy should be
equal to domestic savings
b) savings will naturally be much greater
than investment in the economy
c) savings will be lesser than gross
investment in the economy
d) investment will be equal to the total
amount of capital goods and
infrastructure created in the economy
20. Consider the following statements
1. Francois Bernier constantly compared
Mughal India with contemporary
Europe, generally emphasising the
superiority of the latter
2. According to Bernier, one of the
fundamental differences between
Mughal India and Europe was the lack
of private property in land in the latter
Which of the above statements is/are
INCORRECT?
a)
b)
c)
d)

1 Only
2 Only
Both
None

21. Consider the following statements.


Assertion (A): RBI uses a certain interest rate
called the Bank Rate to control the value of
reserve deposit ratio in commercial banks.
Reason (R): A high bank rate discourages
borrowing by commercial banks from RBI.

b) A and R both are true, and R is the


NOT the correct explanation for A.
c) A is correct, R is incorrect.
d) A and R both are incorrect
22. With reference to Indian gharial, consider
the following statements
1. They are widely seen in the Indus
River, in the Brahmaputra of Bhutan
and Bangladesh, and in the Irrawaddy
River
2. These are infamous for killing humans
and eating them in many parts of
India, especially in delta regions
3. It is listed as critically endangered by
the IUCN
Which of the above statements is/are
INCORRECT?
a)
b)
c)
d)

1 and 2 Only
1,2 and 3 Only
2 and 3 Only
3 Only

23. Green revolution helped in


Commercialization of agriculture. It
meant that in the farms
1. Farmers produced more than mere
subsistence requirements to sell in
markets
2. Modern technology was adopted in
farms
3. Land holdings became consolidated to
produce more
Choose the correct answer using the codes below:
a)
b)
c)
d)

In the context of the statements above, which of


these is true?

1 and 2 only
2 and 3 only
1 and 3 only
All of the above

a) A and R both are true, and R is the


correct explanation for A.

http://insightsonindia.com

INSIGHTS

Page 5

24. Which of the following factors may


possibly have resulted in feminization of
agriculture in India?
1. Migration due to urbanization
2. Seasonal unemployment in rural areas
3. More women as agricultural labour
due to lack of land ownership in
women
Choose the correct answer using the codes below:
a) 1 and 2 only
b) 2 and 3 only
c) 1 and 3 only
d) All of the above
25. Contract farming is practiced in some of
the pockets in India. Consider the
following statements about it.
1. The company identifies the crop to be
grown, gives working capital and
know-how.
2. The farmers procure the seeds by
themselves.
3. The Company assures the farmer of
the purchase of produce at a predetermined price at a fixed date.
Choose the correct answer using the codes below:
a)
b)
c)
d)

1 and 2 only
2 and 3 only
1 and 3 only
All of the above

26. In a managed floating exchange rate


system the exchange rate of the domestic
currency is determined by
1. Market forces of Supply and Demand
2. The Central Bank
3. IMF
Choose the correct answer using the codes below:
a)
b)
c)
d)

27. With reference to birth and trend of Bhakti


traditions, consider the following
statements
1. The principal deities of the Vedic
pantheon, Agni, Indra and Soma,
become marginal figures, rarely visible
in textual or visual representations
2. The Vedas continued to be revered as
authoritative
3. Devotees often tended to project their
chosen deity, either Vishnu or Shiva,
as supreme
Which of the above statements is/are correct?
a) 1 and 3 Only
b) 3 and 2 Only
c) 2 and 1 Only
d) All
28. Consider the following statements about
the employment trends in India.
1. There are more self-employed people
than regular salaried people.
2. There are more regular salaried people
than casual labourers.
Which of these is/are true?
a) 1 only
b) 2 only
c) Both 1 and 2
d) None
29. Industrial workers in India are affected by
which of the following Industrial
Hazards?
1. Flooding of mines
2. Emission of toxic gases
3. Diseases like Silicosis and
Tuberculosis
Choose the correct answer using the codes below:
a)
b)
c)
d)

1 and 2 only
1 and 3 only
2 and 3 only
All of the above

http://insightsonindia.com

INSIGHTS

1 and 2 only
2 and 3 only
1 and 3 only
All of the above
Page 6

30. Consider the following statements about


Pandit Madan Mohan Malviya.
1. He had been the president of Indian
National Congress.
2. He was an important leader of Hindu
Mahasabha.
3. He is founder of Asias largest
residential university Banaras Hindu
University.
Choose the correct answer using the codes below:
a)
b)
c)
d)

1 and 2 only
2 and 3 only
1 and 3 only
All of the above

31. Which of the following powers of can be


devolved to the Panchayats as per the
Constitution of India?
1. Levying taxes
2. Preparing plans for economic
development
3. Managing local water supply and
ponds
Choose the correct answer using the codes below:
a)
b)
c)
d)

1 and 2 only
2 and 3 only
1 and 3 only
All of the above

32. Beetroot, improves blood flow and is


good for the heart. This because
1. It is rich in nitrites
2. Nitrite works as a signal to
haemoglobin, found in red blood cells,
to form nitric oxide which helps in less
clot formation
Which of the above statements is/are correct?
a) 1 Only
http://insightsonindia.com

b) 2 Only
c) Both
d) None

33. Which of the following arguments would


most weaken the case for an economic
policy of liberalisation?
a) Market alone would not be able to
look after the welfare of people,
especially the disadvantaged sections.
b) It is important to protect the
indigenous industries from outside
competition.
c) Greater integration into the global
market would be beneficial to Indian
economy.
d) Liberalisation requires a strong private
sector and resilient domestic industries
lack of which can negatively affect
the domestic economy.

34. Which of the following forces have


strengthened the tendencies of
Globalization?
1. Electronic economy
2. Knowledge economy
3. Globalization of finance
Choose the correct answer using the codes
below:
a)
b)
c)
d)

1 and 2 only
2 and 3 only
1 and 3 only
All of the above

35. Presently which of the following countries


is/are NOT member(s) of United Nations
Security Council (UNSC)?
1. Rwanda
2. Nigeria
3. Lithuania
INSIGHTS

Page 7

Choose the correct answer using the codes


below:
a)
b)
c)
d)

1 and 2 only
2 and 3 only
1 and 3 only
All are members

36. Consider the following statements about


the recently formed Eurasian Economic
Union (EEU).
1. It aims for a common currency among
member nations.
2. It aims for free trade.
3. It will regulate industrial and
agricultural policies of member
nations.
Choose the correct answer using the codes below:
a)
b)
c)
d)

1 and 2 only
2 and 3 only
1 and 3 only
All of the above

37. Consider the following statements:


1. Nirguna Bhakti form focused on the
worship of specific deities such as
Shiva, Vishnu and his avatars
(incarnations) and forms of the
goddess or Devi all in human forms
2. Suguna Bhakti form was worship of an
abstract form of god
Which of the above statements is/are
INCORRECT?
a)
b)
c)
d)

38. Filefish, also known as foolfish,


camouflages from its predators by
a) Changing its colour of the body to
match coral colours
b) They use a cologne they prepare from
the corals to smell just like corals
c) They act like dead fish
d) None of the above

39. The government has recently launched


Mission Indradhanush. The mission
intends to
a) Fight climate change in the tropical
rainforests of the country.
b) Use cloud seeding to improve rainfall
in arid areas
c) Immunise children against 7 vaccine
preventable diseases
d) Universal health coverage in select
backward blocks in rural areas

40. The Arms Trade treaty was recently


ratified. Consider the following
statements about it.
1. It is a legally-binding agreement.
2. It sets robust global standards for
cross-border import-export of
weapons.
3. It prohibits export of those weapons
that are to be used for genocide and
war crimes.
Choose the correct answer using the codes below:
a)
b)
c)
d)

1 Only
2 Only
Both
None

http://insightsonindia.com

INSIGHTS

1 and 2 only
2 and 3 only
1 and 3 only
All of the above.

Page 8

41. Which of the following would NOT be a


transaction related to the external sector of
the economy?
1. Joint production of helicopters by
India and Japan
2. Transfer of technology by Russia to
India
3. USA investment in Indian treasury
bills
Choose the correct answer using the codes below:
a)
b)
c)
d)

1 and 2 only
2 and 3 only
3 only
All can be external sector transactions

42. The resource rich Africa and Latin


America have some of the poorest
countries in the world, whereas many
prosperous countries have scarcely any
natural wealth. Which of the following
can be the strongest reason for the poverty
of resource rich African nations?
a) Little flow of production exists in the
poor African countries.
b) There is huge unemployment in the
poor African countries.
c) There is a lack of foreign investment in
these African countries.
d) The natural resources remain
unexploited in these poor countries
43. Consider the following statements:
1. Alvars were devotees of Vishnu
whereas Nayanars worshipped Shiva
2. The Nalayira Divyaprabandham,
anthology of compositions of
Nayanars is called as Tamil Vedas

44. Nyaya Panchayats have been setup in


some of the states in India. Their
jurisdiction extends to
a) Petty civil and criminal cases both
b) Petty civil cases only
c) All local civil cases
d) Petty criminal cases only

45. Consider the following statements about


the different kinds of goods produced in
an economy.
1. Consumption goods are not further
processed in the value addition chain
by producers.
2. Intermediate goods are the same as
capital goods.
3. Capital goods are able of
transformation at the hands of
producers.
Choose the correct answer using the codes below:
a.
b.
c.
d.

46. Which of the following changes in social


relations can be attributed to Green
Revolution?
1. Increase in use of agricultural labour
2. A shift from payment in kind to that in
cash
3. Loosening of bonded labour situation
Choose the correct answer using the codes below:
a)
b)
c)
d)

Which of the above statements is/are correct?


a)
b)
c)
d)

1 Only
2 Only
Both
None

http://insightsonindia.com

1 only
2 and 3 only
1 and 3 only
All of the above.

INSIGHTS

1 and 2 only
2 and 3 only
1 and 3 only
All of the above

Page 9

47. Consider the following statements about


Depreciation in an economy.
1. More depreciation will reduce the net
investment in the economy.
2. Depreciation takes natural calamities
and sudden destruction of property
into account.
Which of these is/are true?
a)
b)
c)
d)

1 only
2 only
Both 1 and 2
None

48. Recently the Maharashtra government


appointed the Kelkar Committee to study
the issue of balanced development of
various regions of Maharashtra. The
recommendations of the committee are
1. Vidarbha region should be given
autonomous status.
2. The committee recommended sticking
to the provisions of Article 371 of the
constitution for all the regions of
Maharashtra.
3. It also recommended increasing the
powers of the President over all the
backward regions of Maharashtra.

a)
b)
c)
d)

50. In an economy there is often a trade-off


between certain categories of goods. There
need NOT be a trade-off between which of
the following pairs?
a) Capital goods and intermediate goods
b) Capital goods and consumer goods
c) Intermediate goods and consumption
goods
d) Both (a) and (c)
51. Gross National Product involves net
factor income from abroad (NFIA) apart
from the national GDP. Which of the
following would come under NFIA?
1. Remittances from Indian nationals
working in Turkey
2. A reliance outlet in Ontario
3. A BMW car production unit in India
Choose the correct answer using the codes below:
a)
b)
c)
d)

Choose the correct answer using the codes below:


a)
b)
c)
d)

1 and 2 only
2 and 3 only
1 only
All of the above

49. Consider the following statements


1. One of the major themes in Tamil
bhakti hymns is the poets opposition
to Buddhism and Jainism
2. Nayanars and Alvars were patronized
by Chola rulers
Which of the above statements is/are correct?

http://insightsonindia.com

1 Only
2 Only
Both
None

1 and 2 only
2 and 3 only
1 and 3 only
All of the above

52. NASAs NuSTAR is a


a) It is a space based telescope that
studies high energy x-rays emanating
from starts and galaxies
b) It is a space-probe revolving around
Saturn
c) It is a probe sent to study the Sun
d) None of the above
53. Net national product (NNP) excludes
1. Subsidies
2. Indirect taxes
3. Depreciation

INSIGHTS

Page 10

Choose the correct answer using the codes below:


a)
b)
c)
d)

1 and 2 only
2 and 3 only
1 and 3 only
3 only

54. To compare GDP across nations, Real


GDP is used because
a) it calculates GDP at constant prices
b) it takes inflation into account
c) it excludes the subsidies given by a
particular country
d) it has been prescribed by IMF

55. The essential difference between using


GDP deflator or using CPI/WPI for
measuring level of inflation in the
economy is that
a) CPI/WPI takes overall inflation into
account
b) CPI/WPI takes only inflation in few
goods/services into account
c) GDP deflator does not include prices
of imported goods.
d) Both (b) and (c)

56. The speculative demand for money means


that
a) Money is used for investment in
securities that yield returns
b) Money is used for building of
infrastructure in the country
c) Money is demanded for immediate
consumption requirements
d) Money is saved for emergency
purposes by people

http://insightsonindia.com

57. Consider the following statements.


1. An increasing interest rate is beneficial
to people presenting holding bonds.
2. A lower inflation level is beneficial to
deposit holders.
3. A higher real interest rate is beneficial
to banks.
Choose the correct answer using the codes below:
a)
b)
c)
d)

1 and 2 only
2 and 3 only
1 and 3 only
All of the above

58. Currency notes and coins are called fiat


money because
a) They are legal tenders without any
intrinsic value
b) They are legal tenders with an intrinsic
value
c) They are non-binding tenders issued
by the Central bank
d) They are non-binding tenders issued
by the government

59. Consider the following statements with


reference to Lingayats in twelfth century
Karnataka,
1. Lingayats believe that on death the
devotee will be united with Shiva and
will not return to this world
2. Lingayats supported post-puberty
marriage and the remarriage of
widows
Which of the above statements is/are correct?
a)
b)
c)
d)

INSIGHTS

1 Only
2 Only
Both
None

Page 11

60. Consider the following statements about


broad money.
1. It is the least liquid.
2. It excludes time deposits.
3. It includes the reserve money of the
bank held by RBI.
Choose the correct answer using the codes below:
a)
b)
c)
d)

1 and 2 only
2 and 3 only
3 only
None of the above

61. Credit can be created by commercial


banks because
a) Banks are authorised to do so by RBI
b) Banks can re-lend the deposits held by
the public
c) Banks can lend the SLR and CRR after
authorisation from the Central bank
d) Repurchase of government securities
from Central bank makes it possible.

62. A high Cash Reserve Ratio (CRR) will


help the RBI in
1. Tackling inflation in short-run
2. Reduce the fiscal deficit of the
government in the short-run
3. Increase public money supply
Choose the correct answer using the codes below:
a)
b)
c)
d)

1 and 2 only
2 and 3 only
1 only
None of the above

http://insightsonindia.com

63. Consider the following statements about


Open Market Operations (OMOs).
1. They are undertaken both by RBI and
Government of India.
2. OMOs always increase money supply
in the economy.
3. OMOs can help tackle inflation in the
short-run in the economy.
Choose the correct answer using the codes below:
a)
b)
c)
d)

1 and 2 only
2 and 3 only
3 only
All of the above

64. Sterilization is used by RBI to


1. Fight external shocks to the economy
2. Counter inflation
3. Stabilize the depreciation in the Indian
rupee
Choose the correct answer using the codes below:
a)
b)
c)
d)

1 and 2 only
2 and 3 only
3 only
All of the above

65. Government needs to play an important


role in the economy other than the private
sector because
1. Public goods can not always be
provided by private sector
2. Government can take functions of
redistribution of wealth.
3. Stabilization of the economy can not
be done by the private sector
Choose the correct answer using the codes below:
a)
b)
c)
d)

INSIGHTS

1 and 2 only
2 and 3 only
3 only
All of the above

Page 12

66. Revenue receipts of the government do


NOT include which of the following?
1. Taxes imposed on goods imported
into and exported out of India
2. Income of PSUs
3. Profits on investments made by
government

following do NOT come under capital


receipts of the government?
1. Borrowing from RBI
2. Recovery of loans
3. Disinvestment in PSUs
Choose the correct answer using the codes below:
a)
b)
c)
d)

Choose the correct answer using the codes below:


a)
b)
c)
d)

1 and 2 only
2 and 3 only
1 and 3 only
All are part of revenue receipts of
government.

67. With reference to Sufi khanqah, consider


the following statements
1. It was a lodging facility for Sufi saints
and disciples
2. Series of Khanqahs were known as
silsilas

70. Which of the following measures would


come under the term Structural
Adjustment in the economy?
1. Cutting public subsidies
2. Liberalisation policy
3. Tight monetary policy
Choose the correct answer using the codes below:
a)
b)
c)
d)

Which of the above statements is/are correct?


a)
b)
c)
d)

1 Only
2 Only
Both
None

68. Plan revenue expenditure includes


1. Provisions made in five year plans
2. Central assistance to States
3. Central assistance to Union Territories
Choose the correct answer using the codes
below:
a)
b)
c)
d)

1 and 2 only
2 and 3 only
1 only
All form part of capital receipts.

1 and 2 only
2 and 3 only
1 and 3 only
All of the above

71. Arrange the following in increasing order


of value as reflected in the annual budget.
1. Budgetary deficit
2. Revenue deficit
3. Primary deficit
Choose the correct order from the codes below:
a)
b)
c)
d)

1 and 2 only
2 and 3 only
1 and 3 only
All of the above

123
132
321
Any of option 2 or 3 can be greater, but
1 will always be least.

69. Capital receipts form a major part of the


assets of the government. Which of the
http://insightsonindia.com

INSIGHTS

Page 13

72. The Sela River which was in news recently


for the Bay of Bengal oil spill tragedy, is
located in
a) India
b) Bangladesh
c) Myanmar
d) Sagar Island

73. With reference to Kabir and his


compositions. Consider the following
statements
1. The Kabir Bijak is preserved by the
Kabirpanth (the path or sect of Kabir)
in many parts of Rajasthan
2. the Kabir Granthavali is associated
with the Dadupanth in Uttar Pradesh
Which of the above statements is/are correct?
a)
b)
c)
d)

1 Only
2 Only
Both
None

74. The government can affect the level of


aggregate output of the economy by
which of the following?
1. Government purchases of goods and
services
2. Taxes
3. Foreign investment policies
Choose the correct answer using the codes below:
a)
b)
c)
d)

b) Excess spending is used to invest


abroad
c) Excess spending is used to provide
important public services like
transport, health services etc.
d) All of the above
76. Large fiscal deficit is discouraged at times
because private borrowers will get
crowded out of the financial markets. It
means that
a) There will be less financial resources
for the private sector
b) Private sector is not allowed to borrow
heavily from the securities market
c) The financial markets do not give good
returns to the private sector
d) Citizens find it difficult to invest in
private sector equity in securities
markets

77. Consider the following statements about


the FRBM act, 2003.
1. Actual deficits may exceed the targets
only on ground of national security,
calamity etc.
2. The Reserve Bank of India must not
subscribe to the primary issues of
central government securities.
3. It provides for greater transparency in
fiscal operations of the government.
Choose the correct answer using the codes below:

1 and 2 only
2 and 3 only
1 only
All of the above

75. Fiscal deficit need not always be


inflationary. This will be when
a) Excess spending will result in creation
of infrastructure and other useful
assets in the economy
http://insightsonindia.com

a)
b)
c)
d)

1 and 2 only
2 and 3 only
1 and 3 only
All of the above

78. The Government has recently released


commemorative postal stamp to highlight
heroic deeds of those engaged in 1857
Kuka movement. Consider the following
statements about it.
INSIGHTS

Page 14

1. It was initiated in Punjab.


2. This movement actively propagated
the principles of boycott and non-cooperation .
3. It became a part of the overall freedom
struggle.
Choose the correct answer using the codes below:
a)
b)
c)
d)

1 and 2 only
2 and 3 only
1 and 3 only
All of the above

b) 2 Only
c) Both
d) None
81. Rajasthan government has decided to fix
minimum educational qualification as
condition for aspiring candidates in
panchayats elections. It has been done at
the levels of
1. Zila Parishad
2. Block Panchayat
3. Panchayat
Choose the correct answer using the codes below:

79. Which of the following can be termed as


the limitations of GDP as an index of
welfare of a country?
1. It does not account for economic
inequalities.
2. Several non-monetary exchanges are
not accounted for
3. Environmental costs are neglected.
Choose the correct answer using the codes below:
a)
b)
c)
d)

1 and 2 only
2 and 3 only
1 and 3 only
All of the above

80. With reference to the Vijayanagar empire


and their relationship with the Sultanates,
consider the following statements
1. The adventurous policy of Rama Raya
that ultimately led to the decisive
defeat of Vijayanagar kingdom under
the hands of alliance of the Sultanates
2. The Sultans and the rayas were not
always or inevitably hostile to each
other
Which of the above statements is/are correct?
a) 1 Only
http://insightsonindia.com

a)
b)
c)
d)

1 and 2 only
2 and 3 only
1 only
All of the above

82. Presently, six sub-schemes are being


implemented as sub-schemes under
RKVY during 2014-15. These sub-schemes
and their allocations are
1. Bringing Green Revolution to Eastern
Region
2. Vidharbha Intensive Irrigation
Development Programme
3. National Mission for Protein
Supplements
4. Initiative on Vegetable Clusters
Choose the correct answer using the codes below:
a)
b)
c)
d)

1 and 2 only
3 and 4
1, 2 and 3 only
All of the above

83. The Current Account records transactions


in factor income also. Which of the
following /s/are included in factor
income?
1. Profits on investment abroad
2. Trade in services
3. Transfer payments
INSIGHTS

Page 15

Choose the correct answer using the codes below:


a)
b)
c)
d)

1 only
1 and 2 only
2 and 3 only
All of the above

84. The Vijayanagara empire was located on


the banks of river
a) Krishna
b) Tungabhadra
c) Cauvery
d) Tunga
85. A negative trade balance with a large
current account deficit (CAD) can have
which of the following consequences for
India?
1. High inflation in the economy
2. Depreciation of the domestic currency
3. Pulling out of FII from the economy
Choose the correct answer using the codes below:
a) 1 and 2 only
b) 1 and 3 only
c) 2 and 3 only
d) All of the above
86. If the Real Exchange rate of India with
respect to certain goods of USA is greater
than one, it means that
a) Those goods are cheaper in India.
b) Those goods are cheaper in USA.
c) No valuable comparison can be made
without taking GDP (PPP) into
account.
d) No valuable comparison can be made
without taking inflation into account.

2. Tea plantations were often located on


uninhabited hillsides
Which of these is/are true?
a) 1 only
b) 2 only
c) Both 1 and 2
d) None
88. Ricardian Equivalence is a proposition
that argues that consumers are forward
looking. It argues that
a) Taxation and borrowing are equivalent
means of financing expenditure.
b) Borrowing abroad and borrowing
domestically are equivalent means of
financing expenditure.
c) Saving or investment are equivalent
d) None of the above
89. In the jajmani system,
a) Zamindars paid small daily allowance
and diet money to artisans
b) Zamindars paid farmers daily wages
c) Zamindars mistreated and misused
lower caste people
d) None of the above
90. The value of the Indian currency will
appreciate if
1. Indians travel abroad more often
2. There is greater demand for Indian
goods
3. More FIIs find Indian market lucrative
to invest in
Choose the correct answer using the codes below:
a)
b)
c)
d)

87. Tea industry began in India in 1851. Most


of the tea gardens were situated in Assam.
But, bulk of the sorely needed labour had
to be imported from other provinces. This
may be due to
1. Sparse population of Assam
http://insightsonindia.com

INSIGHTS

1 and 2 only
1 and 3 only
2 and 3 only
All of the above

Page 16

91. Consider the following statements about


trade across nations and its impact on
prices.
1. If prices of domestic products fall,
while say foreign prices remain
constant, domestic exports will rise.
2. Changes in nominal exchange rates
would change the real exchange rate
and hence international relative prices.
Which of these is/are true?
a) 1 only
b) 2 only
c) Both 1 and 2
d) None
92. Which of the following economic policies
or activities of the British led to the
downfall of village based industries in
India?
1. Import substitution
2. Industrialization in Britain
3. Mainstreaming of western culture in
matters of clothing, food etc.
Choose the correct answer using the codes below:
a)
b)
c)
d)

1 and 2
2 and 3
1 and 3
All of the above

93. Farmer suicides in the country can be


linked to the agrarian distresses caused
due to structural changes in the economy
as well as changes in economic and
agricultural policies. Which of the
following can be counted among those?
1. Shift to cash crops
2. Exposure of Indian agriculture to
forces of globalization
3. Withdrawal of state from agricultural
extension activities
Choose the correct answer using the codes below:
a) 1 and 2 only
http://insightsonindia.com

b) 2 and 3 only
c) 1 and 3 only
d) All of the above
94. The Defence Ministry of India has
identified some strategic rail lines along
the Indo-China border. These are in the
states of
a) Jammu Kashmir
b) Assam
c) Arunachal Pradesh
Choose the correct answer using the codes below:
a) 1 and 2 only
b) 2 and 3 only
c) 1 only
d) All of the above
95. The Emperor Akbar classified the lands
and fixed different revenue to be paid by
each. Consider the following statements
1. Chachar is land that has lain fallow for
three or four years
2. Banjar is land uncultivated for five
years and more
3. Polaj is land which is annually
cultivated for each crop in succession
and is never allowed to lie fallow
4. Parauti is land left out of cultivation
for a time that it may recover its
strength
Which of the above are correctly matched?
a)
b)
c)
d)

1,2 and 3 Only


2,3 and 4 Only
1,3 and 4 Only
All

96. Cash grants-in-aid from foreign countries


and international organisations are
included in
a) Non-tax revenue receipts
b) Capital receipts
c) External borrowings of Government
d) Corporate Social responsibility
INSIGHTS

Page 17

97. Indias foreign exchange reserve is an


important mechanism against managing
external shocks. It consists of
a) Gold, SDR and Dollar only
b) SDR and foreign currency only
c) Gold, SDR and foreign currencies
d) Gold and foreign currencies only

98. Bridgmanite mineral is


a) The most abundant mineral in Earth
b) The rare earth mineral used in
semiconductor industry
c) The rarest mineral in earth
d) Invaluable mineral found on the ocean
surface
99. Which of the following cancers are mostly
caused by lifestyle changes?
a) Colon cancer
b) Lung cancer
c) Basal cell carcinoma
Choose the correct answer using the codes below
a)
1 and 3 Only
b)
2 and 3 Only
c) 3 and 2 Only
d)
All
100.
Particulates are the deadliest form
of air pollution. Which of the following
particulate categories are most dangerous
to human health?
a) Suspended particulate matter (SPM)
b) Respirable suspended particle (RSP)
c) Particulate Matter 2.5
d) None of the above

http://insightsonindia.com

INSIGHTS

Page 18

Insights Mock Tests 2015 Test 11 Solutions


1. Solution: d)
At one level, colonialism simply means the establishment of rule by one country over
another. In the modern period western colonialism has had the greatest impact. Every policy
of the British was geared towards the strengthening and expansion of British capitalism. For
instance it changed the very laws of the land. It changed not just land ownership laws but
decided even what crops ought to be grown and what ought not to be.
It meddled with the manufacturing sector. It altered the way production and distribution of
goods took place. It entered into the forests. It cleared trees and started tea plantations. It
brought in Forest Acts that changed the lives of pastoralists. They were prevented from
entering many forests that had earlier provided valuable forage for their cattle.
Moreover, the Indian police was established to suppress popular revolts via the Indian
Police Act 1861 which is in force even till date.
On the positive front, the British rule also inculcated a spirit of rationality and scientific
temper in the masses by their education, literature etc.

2. Solution: d)
Rihla is written by Ibn Battuta, not Al Biruni.
Al-Biruni was born in 973, in Khwarizm in presentday Uzbekistan. Khwarizm was an
important centre of learning, and Al-Biruni received the best education available at the time.
He was well versed in several languages: Syriac, Arabic, Persian, Hebrew and Sanskrit.
Although he did not know Greek, he was familiar with the works of Plato and other Greek
philosophers, having read them in Arabic translations. In 1017, when Sultan Mahmud
invaded Khwarizm, he took several scholars and poets back to his capital, Ghazni; Al-Biruni
was one of them.
He arrived in Ghazni as a hostage, but gradually developed a liking for the city, where he
spent the rest of his life until his death at the age of 70. It was in Ghazni that Al-Biruni
developed an interest in India. This was not unusual. Sanskrit works on astronomy,
mathematics and medicine had been translated into Arabic from the eighth century
onwards. When the Punjab became a part of the Ghaznavid empire, contacts with the local
population helped create an environment of mutual trust and understanding. Al-Biruni
spent years in the company of Brahmana priests and scholars, learning Sanskrit, and
studying religious and philosophical texts. While his itinerary is not clear, it is likely that he
travelled widely in the Punjab and parts of northern India.
Ibn Battutas book of travels, called Rihla, written in Arabic, provides extremely rich and
interesting details about the social and cultural life in the subcontinent in the fourteenth
century. This Moroccan traveller was born in Tangier into one of the most respectable and
educated families known for their expertise in Islamic religious law or sharia. True to the

http://www.insightsonindia.com

INSIGHTS

Page 1

Insights Mock Tests 2015 Test 11 Solutions


tradition of his family, Ibn Battuta received literary and scholastic education when he was
quite young.

3. Solution: a)
In the short run, another factor that is important in determining exchange rate movements is
the interest rate differential i.e. the difference between interest rates between countries.
There are huge funds owned by banks, multinational corporations and wealthy individuals
which move around the world in search of the highest interest rates. If we assume that
government bonds in country A pay 8 per cent rate of interest whereas equally safe bonds in
country B yield 10 per cent, the interest rate diferential is 2 per cent.
Investors from country A will be attracted by the high interest rates in country B and will
buy the currency of country B selling their own currency. At the same time investors in
country B will also find investing in their own country more attractive and will therefore
demand less of country As currency. This means that the demand curve for country As
currency will shift to the left and the supply curve will shift to the right causing a
depreciation of country As currency and an appreciation of country Bs currency. Thus, a
rise in the interest rates at home often leads to an appreciation of the domestic currency. Here, the
implicit assumption is that no restrictions exist in buying bonds issued by foreign
governments.

4. Solution: d)
A lot of social evils had plagued Indian society. The wellknown issues are that of sati,child
marriage, widow remarriage and caste discrimination. It is not that attempts were not made
to fight social discrimination in pre-colonial India. They were central to Buddhism, to Bhakti
and Sufi movements. Going ahead, what marked the post-colonial 19th century social reform
attempts was the modern context and mix of ideas. It was a creative combination of modern
ideas of western liberalism and a new look on traditional literature.

5. Solution: d)
Within India, social reformers from Punjab and Bengal exchanged ideas with reformers from
Madras and Maharashtra. Keshav Chandra Sen of Bengal visited Madras in 1864. Pandita
Ramabai travelled to different corners of the country. Some of them went to other countries.
Modern social organisations like the Brahmo Samaj in Bengal and Arya Samaj in Punjab were
set up. The All-India Muslim Ladies Conference (Anjuman-E-Khawatn-E-Islam) was founded
in 1914. Indian reformers debated not just in public meetings but through public media like
newspapers and journals. Translations of writings of social reformers from one Indian
language to another took place.
http://www.insightsonindia.com

INSIGHTS

Page 2

Insights Mock Tests 2015 Test 11 Solutions

6. Solution: c)
Both the concepts of participatory democracy and decentralised governance have thus become
popular. Participatory democracy is a system of democracy in which the members of a
group or community participate collectively in the taking of major decisions. Panchayati raj
is a major example where participatory democracy is being practiced. For e.g. in MGNREGA
major project decisions, approvals, social audit etc. are done by the local community. In
other bodies like State LA or DPC, it is an indirect or representative democracy.

7. Solution: a)
Broadly speaking, revenue expenditure consists of all those expenditures of the government
which do not result in creation of physical or financial assets. It relates to those expenses
incurred for the normal functioning of the government departments and various services,
interest payments on debt incurred by the government, and grants given to state
governments and other parties (even though some of the grants may be meant for creation
of assets).

8. Solution: a)
Among the renewable energy sources, onshore wind had the maximum contribution of 51.2
billion kWh, which comprised of one-third of the renewable energy contribution to the
energy mix. Gross electricity contribution of solar photovoltaic increased maximum, by 4.2
billion kWh, contributing 22.4 per cent of the renewable energy mix. Except for hydro
power, contribution from all renewable energy sources increased during 2014. Off-shore
wind has also seen rapid scale-up with energy production increasing by 37 per cent as
compared to 2013. Geo-thermal energy has also contributed to the renewable energy mix but
its share has been only about 0.1 per cent.
http://www.downtoearth.org.in/content/renewable-energy-share-germany-crosses-252014

9. Solution: c)
Muhammad bin Tughlaq
Unlike most other members of his class, Ibn Battuta considered experience gained through
travels to be a more important source of knowledge than books. He just loved travelling,
and went to far-off places, exploring new worlds and peoples. Before he set off for India in
1332-33, he had made pilgrimage trips to Mecca, and had already travelled extensively in
Syria, Iraq, Persia, Yemen, Oman and a few trading ports on the coast of East Africa.
http://www.insightsonindia.com

INSIGHTS

Page 3

Insights Mock Tests 2015 Test 11 Solutions


Travelling overland through Central Asia, Ibn Battuta reached Sind in 1333. He had heard
about Muhammad bin Tughlaq, the Sultan of Delhi, and lured by his reputation as a
generous patron of arts and letters, set off for Delhi, passing through Multan and Uch. The
Sultan was impressed by his scholarship, and appointed him the qazi or judge of Delhi. He
remained in that position for several years, until he fell out of favour and was thrown into
prison. Once the misunderstanding between him and the Sultan was cleared, he was
restored to imperial service, and was ordered in 1342 to proceed to China as the Sultans
envoy to the Mongol ruler.

10. Solution: b)
Since a country interacts with many countries, we may want to see the movement of the
domestic currency relative to all other currencies in a single number rather than by looking
at bilateral rates. That is, we would want an index for the exchange rate against other
currencies, just as we use a price index to show how the prices of goods in general have
changed.
This is calculated as the Nominal Effective Exchange Rate (NEER) which is a multilateral
rate representing the price of a representative basket of foreign currencies, each weighted by
its importance to the domestic country in international trade (the average of export and
import shares is taken as an indicator of this).
The Real Effective Exchange Rate (REER) is calculated as the weighted average of the real
exchange rates of all its trade partners, the weights being the shares of the respective
countries in its foreign trade. It is interpreted as the quantity of domestic goods required to
purchase one unit of a given basket of foreign goods.

11. Solution: c)
Even as India fought for its independence from British colonialism a vision of what Indian
democracy ought to look like emerged. As far back as in 1928, Motilal Nehru and eight other
Congress leaders drafted a constitution for India. In 1931, the resolution at the Karachi
session of the Indian National Congress dwelt on how independent Indias constitution
should look like. The Karachi Resolution reflects a vision of democracy that meant not just
formal holding of elections but a substantive reworking of the Indian social structure in
order to have a genuine democratic society.
The Karachi Resolution clearly spells out the vision of democracy that the nationalist
movement in India had. It articulates the values that were further given full expression in
the Indian Constitution.
Cabinet mission plan was about the post-independent political and territorial reorganization
of India.

http://www.insightsonindia.com

INSIGHTS

Page 4

Insights Mock Tests 2015 Test 11 Solutions


Lucknow pact was the merger of the league and Congress and the acceptance of separate
electorates (which is not at all a core value of the constitution).

12. Solution: a)
When the constitution was being drafted panchayats did not find a mention in it. At this
juncture, a number of members expressed their sorrow, anger and disappointment over this
issue. At the same time, drawing on his own rural experience Dr. Ambedkar argued that
local elites and upper castes were so well entrenched in society that local self-government
only meant a continuing exploitation of the downtrodden masses of Indian society. The
upper castes would no doubt silence this segment of the population further.
The concept of local government was dear to Gandhiji too. He envisaged each village as a
self-sufficient unit conducting its own affairs and saw gram-swarajya to be an ideal model to
be continued after independence.

13. Solution: c)
The budget provides for the capital expenditure of the government.
Since Independence, with the launching of the Five-Year Plans, it has also become a
significant national policy statement. The budget, it has been argued, reflects and shapes,
and is, in turn, shaped by the countrys economic life. Along with the budget, three policy
statements are mandated by the Fiscal Responsibility and Budget Management Act, 2003
(FRBMA). The Medium-term Fiscal Policy Statement sets a three-year rolling target for
specific fiscal indicators and examines whether revenue expenditure can be financed
through revenue receipts on a sustainable basis and how productively capital receipts
including market borrowings are being utilised.
RBI can not regulate governments fiscal policy, nor can any international treaty.

14. Solution: a)
http://en.wikipedia.org/wiki/Panna_National_Park
http://www.downtoearth.org.in/content/public-hearings-ken-betwa-river-link-projectsham-complain-activists
http://indiatoday.intoday.in/story/dam-in-panna-tiger-reserve-a-great-dream-with-somedeceit/1/383926.html

15. Solution: d)
http://www.insightsonindia.com

INSIGHTS

Page 5

Insights Mock Tests 2015 Test 11 Solutions


Once the Portuguese arrived in India in about 1500, a number of them wrote detailed
accounts regarding Indian social customs and religious practices. A few of them, such as the
Jesuit Roberto Nobili, even translated Indian texts into European languages. Among the best
known of the Portuguese writers is Duarte Barbosa, who wrote a detailed account of trade
and society in south India.
Later, after 1600, we find growing numbers of Dutch, English and French travellers coming
to India. One of the most famous was the French jeweller Jean-Baptiste Tavernier, who
travelled to India at least six times. He was particularly fascinated with the trading
conditions in India, and compared India to Iran and the Ottoman empire. Some of these
travellers, like the Italian doctor Manucci, never returned to Europe, and settled down in
India.
Franois Bernier, a Frenchman, was a doctor, political philosopher and historian. Like many
others, he came to the Mughal Empire in search of opportunities. He was in India for twelve
years, from 1656 to 1668, and was closely associated with the Mughal court, as a physician to
Prince Dara Shukoh, the eldest son of Emperor Shah Jahan, and later as an intellectual and
scientist, with Danishmand Khan, an Armenian noble at the Mughal court.

16. Solution: d)
Agricultural land is the single most important resource and form of property in rural
society. But it is not equally distributed among people living in a particular village or region.
Nor does everyone have access to land. In fact, the distribution of landholdings in most
regions is highly unequal among households. In some parts of India the majority of rural
households own at least some land usually very small plots. In other areas as much as 40
to 50 per cent of families do not own any land at all. This means that they are dependent on
agricultural labour or other kinds of work for their livelihoods. This of course means that a
few families are well-to-do. The majority live just above or below the poverty line.
In most regions of India, women are usually excluded from ownership of land, because of
the prevailing patrilineal kinship system and mode of inheritance. By law women are
supposed to have an equal share of family property. In reality they only have limited rights
and some access to land only as part of a household headed by a man.
Access to land largely determines what role one plays in the process of agricultural
production. Medium and large landowners are usually able to earn sufficient or even large
incomes from cultivation (although this depends on agricultural prices, which can fluctuate
greatly, as well as other factors such as the monsoon). But agricultural labourers are more
often than not paid below the statutory minimum wage and earn very little. Their incomes
are low. Their employment is insecure.
Most agricultural labourers are daily-wage workers. And do not have work for many days
of the year. This is known as underemployment. Similarly, tenants (cultivators who lease
their land from landowners) have lower incomes than owner-cultivators. Because they have
http://www.insightsonindia.com

INSIGHTS

Page 6

Insights Mock Tests 2015 Test 11 Solutions


to pay a substantial rent to the landowner often as much as 50 to 75 per cent of the income
from the crop.

17. Solution: a)
That part of our final output that comprises of capital goods constitutes gross investment of
an economy. These may be machines, tools and implements; buildings, office spaces,
storehouses or infrastructure like roads, bridges, airports or jetties. But all the capital goods
produced in a year do not constitute an addition to the capital stock already existing.
A significant part of current output of capital goods goes in maintaining or replacing part of
the existing stock of capital goods. This is because the already existing capital stock suffers
wear and tear and needs maintenance and replacement.

18. Solution: a)
Land ceiling act fixed the amount of land that can be owned by a particular family. The
ceiling varies from region to region, depending on the kind of land, its productivity, and
other such factors.
Very productive land has a low ceiling while unproductive dry land has a higher ceiling
limit. According to these acts, the state is supposed to identify and take possession of
surplus land (above the ceiling limit) held by each household, and redistribute it to landless
families and households in other specified categories, such as SCs and STs. But in most of
the states these acts proved to be toothless. There were many loopholes and other strategies
through which most landowners were able to escape from having their surplus land taken
over by the state.
While some very large estates were broken up, in most cases landowners managed to divide
the land among relatives and others, including servants, in so-called benami transfers
which allowed them to keep control over the land (in fact if not in name). In some places,
some rich farmers actually divorced their wives (but continued to live with them) in order to
avoid the provisions of the Land Ceiling Act, which allowed a separate share for unmarried
women but not for wives.

19. Solution: a)
The basic equation representing the economy is:
I+G+XMS+T
In other words
http://www.insightsonindia.com

INSIGHTS

Page 7

Insights Mock Tests 2015 Test 11 Solutions


(I S) + (G T) M X (2.8)
In the equation, G T measures by what amount the government expenditure exceeds the
tax revenue earned by it. This is referred to as budget deficit. M X is known as the trade
deficit it measures the excess of import expenditure over the export revenue earned by the
economy (M is the outflow from the country, X is the inflow into the country).
If there is no government, no foreign trade then G = T = M = X = 0.
Hence the equation yields I S, which means investments equal to savings.

20. Solution: b)
If Ibn Battuta chose to describe everything that impressed and excited him because of its
novelty, Franois Bernier belonged to a different intellectual tradition. He was far more
preoccupied with comparing and contrasting what he saw in India with the situation in
Europe in general and France in particular, focusing on situations which he considered
depressing. His idea seems to have been to influence policy-makers and the intelligentsia to
ensure that they made what he considered to be the right decisions.
Berniers Travels in the Mughal Empire is marked by detailed observations, critical insights
and reflection. His account contains discussions trying to place the history of the Mughals
within some sort of a universal framework. He constantly compared Mughal India with
contemporary Europe, generally emphasising the superiority of the latter. His
representation of India works on the model of binary opposition, where India is presented
as the inverse of Europe. He also ordered the perceived differences hierarchically, so that
India appeared to be inferior to the Western world.
According to Bernier, one of the fundamental differences between Mughal India and Europe
was the lack of private property in land in the former. He was a firm believer in the virtues
of private property, and saw crown ownership of land as being harmful for both the state
and its people. He thought that in the Mughal Empire the emperor owned all the land and
distributed it among his nobles, and that this had disastrous consequences for the economy
and society.
This perception was not unique to Bernier, but is found in most travellers accounts of the
sixteenth and seventeenth centuries. Owing to crown ownership of land, argued Bernier,
landholders could not pass on their land to their children. So they were averse to any longterm investment in the sustenance and expansion of production. The absence of private
property in land had, therefore, prevented the emergence of the class of improving
landlords (as in Western Europe) with a concern to maintain or improve the land. It had led
to the uniform ruination of agriculture, excessive oppression of the peasantry and a
continuous decline in the living standards of all sections of society, except the ruling
aristocracy.

http://www.insightsonindia.com

INSIGHTS

Page 8

Insights Mock Tests 2015 Test 11 Solutions


21. Solution: a)
The Reserve Deposit Ratio: Banks hold a part of the money people keep in their bank
deposits as reserve money and loan out the rest to various investment projects. Reserve
money consists of two things vault cash in banks and deposits of commercial banks with
RBI. Banks use this reserve to meet the demand for cash by account holders. Reserve deposit
ratio (rdr) is the proportion of the total deposits commercial banks keep as reserves.
Keeping reserves is costly for banks, as, otherwise, they could lend this balance to interest
earning investment projects. However, RBI requires commercial banks to keep reserves in
order to ensure that banks have a safe cushion of assets to draw on when account holders
want to be paid. RBI uses various policy instruments to bring forth a healthy rdr in
commercial banks. The first instrument is the Cash Reserve Ratio which specifies the
fraction of their deposits that banks must keep with RBI. There is another tool called
Statutory Liquidity Ratio which requires the banks to maintain a given fraction of their total
demand and time deposits in the form of specified liquid assets.
Apart from these ratios RBI uses a certain interest rate called the Bank Rate to control the value of
rdr. Commercial banks can borrow money from RBI at the bank rate when they run short of reserves.
A high bank rate makes such borrowing from RBI costly and, in effect, encourages the commercial
banks to maintain a healthy rdr.

22. Solution: a)
Gharials once thrived in all the major river systems of the Indian Subcontinent, spanning the
rivers of its northern part from the Indus River in Pakistan across the Gangetic floodplain to
theIrrawaddy River in Myanmar. Today, they are extinct in the Indus River, in
the Brahmaputra ofBhutan and Bangladesh, and in the Irrawaddy River. Their distribution
is now limited to only 2% of their former range.
In India, small populations are present and increasing in the rivers of the National Chambal
Sanctuary, Katarniaghat Wildlife Sanctuary, Son River Sanctuary and
the rainforest biome ofMahanadi in Satkosia Gorge Sanctuary, Odisha, where they
apparently do not breed.
Gharials do not kill and eat humans. Jewellery found in their stomachs may have been the
reason for the myth that gharials are man-eaters. They may have swallowed this jewellery
as gastroliths used to aid digestion or buoyancy management.
As the population has declined drastically in the past 70 years, the gharial is listed
as Critically Endangered by the IUCN.
http://www.downtoearth.org.in/content/science-technology-bytes-1

23. Solution: d)
http://www.insightsonindia.com

INSIGHTS

Page 9

Insights Mock Tests 2015 Test 11 Solutions


In most of the Green Revolution areas, it was primarily the medium and large farmers who were
able to benefit from the new technology. This was because inputs were expensive, and small and
marginal farmers could not afford to spend as much as large farmers to purchase these
inputs. When agriculturists produce primarily for themselves and are unable to produce for
the market, it is known as subsistence agriculture and they are usually termed peasants.
Agriculturists or farmers are those who are able to produce surplus, over and above the
needs of the family, and so are linked to the market. It was the farmers who were able to produce
a surplus for the market who were able to reap the most benefits from the Green Revolution and
from the commercialisation of agriculture that followed.

24. Solution: d)
The largescale circulation of labour has had several significant effects on rural society, in
both the receiving and the supplying regions. For instance, in poor areas where male family
members spend much of the year working outside of their villages, cultivation has become
primarily a female task. Women are also emerging as the main source of agricultural labour,
leading to the feminisation of agricultural labour force. The insecurity of women is greater
because they earn lower wages than men for similar work. Until recently, women were
hardly visible in official statistics as earners and workers. While women toil on the land as
landless labourers and as cultivators, the prevailing patrilineal kinship system, and other
cultural practices that privilege male rights, largely exclude women from land ownership.

25. Solution: c)
In some regions such as Punjab and Karnataka, farmers enter into contracts with
multinational companies (such as PepsiCo) to grow certain crops (such as tomatoes and
potatoes), which the companies then buy from them for processing or export. In such
contract farming systems, the company identifies the crop to be grown, provides the seeds
and other inputs, as well as the knowhow and often also the working capital.
In return, the farmer is assured of a market because the company guarantees that it will
purchase the produce at a predetermined fixed price. Contract farming is very common now
in the production of specialised items such as cut flowers, fruits such as grapes, figs and
pomegranates, cotton, and oilseeds.
While contract farming appears to provide financial security to farmers, it can also lead to
greater insecurity as farmers become dependent on these companies for their livelihoods.
Contract farming of export-oriented products such as flowers and gherkins also means that
agricultural land is diverted away from food grain production.
Contract farming has sociological significance in that it disengages many people from the
production process and makes their own indigenous knowledge of agriculture irrelevant. In
addition, contract farming caters primarily to the production of elite items, and because it
http://www.insightsonindia.com

INSIGHTS

Page 10

Insights Mock Tests 2015 Test 11 Solutions


usually requires high doses of fertilisers and pesticides, it is often not ecologically
sustainable.

26. Solution: a)
In a system of flexible exchange rates (also known as floating exchange rates), the exchange
rate is determined by the forces of market demand and supply. In a completely flexible
system, the central banks follow a simple set of rules they do nothing to directly affect the
level of the exchange rate, in other words they do not intervene in the foreign exchange
market (and therefore, there are no official reserve transactions).
In a fixed system, the Central bank completely determines the exchange rates.
In a managed system, the Central bank allows the currency to be affected by market forces.
At few times, the Bank intervenes directly in the forex market to buy and sell foreign
currencies in an attempt to moderate exchange rate movements whenever they feel that such
actions are appropriate. Official reserve transactions are, therefore, not equal to zero.

27. Solution: d)
All of these somewhat divergent and even disparate beliefs and practices would come to be
classified as Hindu over the course of the next millennium. The divergence is perhaps most
stark if we compare Vedic and Puranic traditions. The principal deities of the Vedic
pantheon, Agni, Indra and Soma, become marginal figures, rarely visible in textual or visual
representations. And while we can catch a glimpse of Vishnu, Shiva and the goddess in
Vedic mantras, these have little in common with the elaborate Puranic mythologies.
However, in spite of these obvious discrepancies, the Vedas continued to be revered as
authoritative. Not surprisingly, there were sometimes conflicts as well those who valued
the Vedic tradition often condemned practices that went beyond the closely regulated
contact with the divine through the performance of sacrifices or precisely chanted mantras.
On the other hand those engaged in Tantric practices frequently ignored the authority of the
Vedas. Also, devotees often tended to project their chosen deity, either Vishnu or Shiva, as
supreme. Relations with other traditions, such as Buddhism or Jainism, were also often
fraught with tension if not open conflict.
The traditions of devotion or bhakti need to be located within this context. Devotional
worship had a long history of almost a thousand years before the period we are considering.
During this time, expressions of devotion ranged from the routine worship of deities within
temples to ecstatic adoration where devotees attained a trance-like state. The singing and
chanting of devotional compositions was often a part of such modes of worship. This was
particularly true of the Vaishnava and Shaiva sects.

http://www.insightsonindia.com

INSIGHTS

Page 11

Insights Mock Tests 2015 Test 11 Solutions

28. Solution: a)
A major point of difference between developing and developed countries is the number of
people in regular salaried employment. In developed countries, the majority are formally
employed. In India, over 50% of the population is selfemployed, only about 14% are in
regular salaried employment, while approximately 30% are in casual labour (Anant 2005:
239). Economists and others often make a distinction between the organised or formal and
unorganised or informal sector.

29. Solution: d)
After mining has finished in an area, the company is supposed to cover up the open holes
and restore the area to its earlier condition. But they dont do this.
Workers in underground mines face very dangerous conditions, due to flooding, fire, the
collapse of roofs and sides, the emission of gases and ventilation failures. Many workers
develop breathing problems and diseases like tuberculosis and silicosis. Those working in
overground mines have to work in both hot sun and rain, and face injuries due to mine
blasting, falling objects etc. The rate of mining accidents in India is very high compared to
other countries.

30. Solution: d)
Former Prime Minister Atal Bihari Vajpayee and late educationist Madan Mohan Malviya
(posthumously) have been chosen for Bharat Ratna- countrys highest civilian award.
It was announced by the press communique from Rashtrapati Bhavan.
About Madan Mohan Malviya

He was born on December 25, 1861 and was an educationist and politician notable
for his role in Indias freedom struggle. He was popularly known Mahaman.

He is founder of Asias largest residential university Banaras Hindu University.

He was President of Indian National Congress in 1909 and 1918.

He is also remembered for his stellar role in the Independence movement and his
espousal of Hindu nationalism.

He is considered as one of the initial leaders of the right-wing Hindu Mahasabha and
died in 1946.

http://www.insightsonindia.com

INSIGHTS

Page 12

Insights Mock Tests 2015 Test 11 Solutions


31. Solution: d)
Panchayats should be given powers and authority to function as institutions of selfgovernment. It, thus, requires all state governments to revitalise local representative
institutions.
The following powers and responsibility were delegated to the Panchayats:

to prepare plans and schemes for economic development

to promote schemes that will enhance social justice

to levy, collect and appropriate taxes, duties, tolls and fees

help in the devolution of governmental responsibilities, especially that of finances to


local authorities

Social welfare responsibilities of the Panchayats include the maintenance of burning and
burial grounds, recording statistics of births and deaths, establishment of child welfare and
maternity centres, control of cattle etc.

32. Solution: c)
It IS known that nitrite-rich food, such as beetroot, improve blood flow and are good for the
heart. But how this happens was not clear. A study shows that nitrite works as a signal to
haemoglobin, found in red blood cells, to form nitric oxide. This reduces platelet activation
which causes blood clots. Researchers now aim to enhance this ability of haemoglobin to
treat conditions such as hypertension, sickle cell disease and stroke.
http://www.downtoearth.org.in/content/science-technology-bytes-1

33. Solution: d)
Argument A is more about the idea of a welfare state. A liberalised state too can look after
welfare needs of citizens.
Argument C strengthens the case for liberalisation.
Argument D is more complete and clear than Argument B. It is self-explanatory as to why
liberalisation may not be taken in an economy. Hence the answer is D.

34. Solution: d)
Refer to the section THE DIFFERENT DIMENSIONS OF GLOBALISATION in the
chapter 6 of Social Change and Development in India XII NCERT.
http://www.insightsonindia.com

INSIGHTS

Page 13

Insights Mock Tests 2015 Test 11 Solutions


It has been explained comprehensively there. This topic is important for Mains too. So lot of
points can be lifted for use in answers.

35. Solution: d)
This question is in the background of - UN Security Council (UNSC) has rejected a
resolution demanding an end to the Israeli occupation of the Palestinian territories within
three years i.e. 2017.
In this regard, Jordan earlier had submitted the motion in UN after it was agreed and backed
by 22 Arab states and the Palestinian Authority.
Eight members of the 15-strong Security Council voted for it

Russia, China, France, Argentina, Chad, Chile, Jordan and Luxembourg voted in
favour the resolution.

US and Australia voted against it.

UK, Lithuania, Nigeria, the Republic of Korea and Rwanda remained abstained.

This resolution failed to get support of at least nine members in order to pass.

36. Solution: b)

Russia and four ex-Soviet nations have finalized the formation of a new economic allianceEurasian Economic Union (EEU). Four ex-Soviet nations are Belarus, Kazakhstan, Armenia
and Kyrgyzstan.
It was finalized in EEU summit held in Moscow, Russia.
EEU will come into existence on 1 January 2015. It seeks to bolster integration of member
nations which includes Russia, Belarus, Kazakhstan, Armenia and Kyrgyzstan primarily
located in northern Eurasia.

It aims to provide free trade among member countries without any barriers.

In addition, it will co-ordinate the financial systems and regulates industrial and
agricultural policies along with labour markets and transportation networks of
member nations.

EEU will bring together around 170 million people from 5 member nations and will
have a combined economic output of $4.5 trillion.

http://www.insightsonindia.com

INSIGHTS

Page 14

Insights Mock Tests 2015 Test 11 Solutions

37. Solution: c)
At a different level, historians of religion often classify bhakti traditions into two broad
categories: saguna (with attributes) and nirguna (without attributes). The former included
traditions that focused on the worship of specific deities such as Shiva, Vishnu and his
avatars (incarnations) and forms of the goddess or Devi, all often conceptualised in
anthropomorphic forms. Nirguna bhakti on the other hand was worship of an abstract form
of god.

38. Solution: b)

Camouflage NEED not always be visual. Filefish use a cologne they prepare from the corals
they eat to hide from the nose of predators. A study in Australia found that crabs were
unable to smell filefish because they smelled like the corals. Insects are known to use the
technique but this is the first time that a vertebrate has been found to use chemical
camouflage. The find indicates that there could be more species which use it

39. Solution: c)
Government has launched Mission Indradhanush to immunise kids against 7 vaccinepreventable diseases.
Mission Indradhanush depicts 7 colours of the rainbow which aims to cover all those
children by 2020 who are either unvaccinated, or are partially vaccinated against 7 vaccine
preventable diseases.
http://www.insightsonindia.com

INSIGHTS

Page 15

Insights Mock Tests 2015 Test 11 Solutions

7 preventable diseases: diphtheria, whooping cough, tetanus, polio, tuberculosis,


measles and hepatitis B.

40. Solution: d)
This landmark treaty has officially became binding international law, which aims to regulate
around the USD 85 billion global arms trade.
It came into force after 60 nations ratified it, as of December 23 among the 130 signatories of
the treaty. In order to come in force, the treaty in total needed 50 nations ratification.

It is the first legally-binding multilateral agreement that prohibits nations from


exporting conventional weapons to countries that may use it for genocide, crimes
against humanity or war crimes.

It adds a new chapter in collective efforts of nations to bring responsibility,


accountability and transparency to the global arms trade.

It set robust global standards for cross-border transfers of conventional weapons


ranging from small firearms to tanks and attack helicopters.

It creates binding requirements for states to review cross-border contracts to ensure


weapons will not be used in human rights abuses, violations of humanitarian law or
organised crime

41. Solution: d)
Joint production would involve sharing and transfer or money and other resources across
nations.
Transfer of technology would also involve money and transfer of other resources. It is
essentially an external sector transaction of the economy.
Investment in treasury bills or government bonds of a country is a kind of FII. So an external
sector transaction.

42. Solution: a)
What makes countries rich or poor?
These are some of the central questions of economics. It is not that countries which are
endowed with a bounty of natural wealth minerals or forests or the most fertile lands are
naturally the richest countries. In fact the resource rich Africa and Latin America have some

http://www.insightsonindia.com

INSIGHTS

Page 16

Insights Mock Tests 2015 Test 11 Solutions


of the poorest countries in the world, whereas many prosperous countries have scarcely any
natural wealth.
There was a time when possession of natural resources was the most important
consideration but even then the resource had to be transformed through a production
process.
The economic wealth, or well-being, of a country thus does not necessarily depend on the
mere possession of resources; the point is how these resources are used in generating a flow
of production and how, as a consequence, income and wealth are generated from that
process.

43. Solution: a)
Some of the earliest bhakti movements (c. sixth century) were led by the Alvars (literally,
those who are immersed in devotion to Vishnu) and Nayanars (literally, leaders who were
devotees of Shiva). They travelled from place to place singing hymns in Tamil in praise of
their gods.
Some historians suggest that the Alvars and Nayanars initiated a movement of protest
against the caste system and the dominance of Brahmanas or at least attempted to reform
the system. To some extent this is corroborated by the fact that bhaktas hailed from diverse
social backgrounds ranging from Brahmanas to artisans and cultivators and even from
castes considered untouchable.
The importance of the traditions of the Alvars and Nayanars was sometimes indicated by
the claim that their compositions were as important as the Vedas. For instance, one of the
major anthologies of compositions by the Alvars, the Nalayira Divyaprabandham, was
frequently described as the Tamil Veda, thus claiming that the text was as significant as the
four Vedas in Sanskrit that were cherished by the Brahmanas.

44. Solution: a)
Nyaya Panchayats have been constituted in some states. They possess the authority to hear
some petty, civil and criminal cases. They can impose fines but cannot award a sentence.
These village courts have often been successful in bringing about an agreement amongst
contending parties. They have been particularly effective in punishing men who harass
women for dowry and perpetrate violence against them.

45. Solution: a)
Of the final goods, we can distinguish between consumption goods and capital goods.
Goods like food and clothing, and services like recreation that are consumed when
http://www.insightsonindia.com

INSIGHTS

Page 17

Insights Mock Tests 2015 Test 11 Solutions


purchased by their ultimate consumers are called consumption goods or consumer goods.
(This also includes services which are consumed but for convenience we may refer to them
as consumer goods.)
Then there are other goods that are of durable character which are used in the production
process. These are tools, implements and machines. While they make production of other
commodities feasible, they themselves dont get transformed in the production process.
They are also final goods yet they are not final goods to be ultimately consumed. They are
called capital goods.
Of the total production taking place in the economy a large number of products dont end
up in final consumption and are not capital goods either.
Such goods may be used by other producers as material inputs. Examples are steel sheets
used for making automobiles and copper used for making utensils. These are intermediate
goods, mostly used as raw material or inputs for production of other commodities. These are
not final goods.

46. Solution: d)
Several profound transformations in the nature of social relations in rural areas took place in
the post-Independence period, especially in those regions that underwent the Green
Revolution. These included:

an increase in the use of agricultural labour as cultivation became more intensive;

a shift from payment in kind (grain) to payment in cash;

a loosening of traditional bonds or hereditary relationships between farmers or


landowners and agricultural workers (known as bonded labour);

and the rise of a class of free wage labourers.

The change in the nature of the relationship between landlords (who usually belonged to the
dominant castes) and agricultural workers (usually low caste), was described by the
sociologist Jan Breman as a shift from patronage to exploitation.

47. Solution: a)
A part of the capital goods produced this year goes for replacement of existing capital goods
and is not an addition to the stock of capital goods already existing and its value needs to be
subtracted from gross investment for arriving at the measure for net investment. This
deletion, which is made from the value of gross investment in order to accommodate regular
wear and tear of capital, is called depreciation.

http://www.insightsonindia.com

INSIGHTS

Page 18

Insights Mock Tests 2015 Test 11 Solutions


So new addition to capital stock in an economy is measured by net investment or new
capital formation, which is expressed as
Net Investment = Gross investment Depreciation
So higher the depreciation, lower the net investment will be.
Depreciation is thus an annual allowance for wear and tear of a capital good. But it is NOT
related only to the wear and tear of a capital good.
Depreciation does not take into account unexpected or sudden destruction or disuse of
capital as can happen with accidents, natural calamities or other such extraneous
circumstances.
We are making a rather simple assumption here that there is a constant rate of depreciation
based on the original value of the asset. There can be other methods to calculate depreciation
in actual practice.

48. Solution: c)
Present Constitutional provisions for Vidarbha
Article 371 of Constitution says that President may provide governor of Maharashtra with
following special responsibility

Establish separate development boards for regions of Maharashtra like Vidarbha,


Marathwada and rest.

Every year, working reports of these boards will be placed before State Legislative
assembly.

Development expenditure funds must be equitable allocated for these regions.

These areas must be provided with equitable and adequate facilities for technical
education and vocational training. Even adequate employment opportunities in state
services must be provided to these regions.

Key Recommendations of Vijay Kelkar committee

Autonomous status must be granted to Vidarbha-eastern region of Maharashtra, as


people of this region feels neglected.

This region should get autonomous status on the lines of Meghalaya as it was
granted an autonomous status in 1969, when it was a part of Assam, by the 24th
constitutional amendment.

The people of Vidarbha feel that issues of the region are neglected by the Mumbaiheadquartered government.

http://www.insightsonindia.com

INSIGHTS

Page 19

Insights Mock Tests 2015 Test 11 Solutions

Shift the state secretariat (Mantralaya) to Nagpur- states second capital, for the
month i.e. from December 1 to 31 every year as a part of Nagpur pact (whereby
Vidarbha leaders agreed to merger of the region within Maharashtra in 1960)

All the important finance-related portfolios should be allocated to the leaders from
Vidarbha.

Some of the major directorates should be shifted to Aurangabad and Nagpur


districts.

Pending irrigation projects in Vidarbha should be completed on a priority basis.

Financial outlay of state: It should be classified into two categories- divisible and
non-divisible. The divisible outlay will be divided among three regions of Vidarbha,
Marathwada, and Rest of Maharashtra (RoM), excluding Mumbai. 45 per cent
divisible outlay for RoM, 33.24 per cent for Vidarbha and 25.31 per cent for
Marathwada.

49. Solution: c)
While Buddhism and Jainism had been prevalent in this region for several centuries,
drawing support from merchant and artisan communities, these religious traditions received
occasional royal patronage. Interestingly, one of the major themes in Tamil bhakti hymns is
the poets opposition to Buddhism and Jainism. This is particularly marked in the
compositions of the Nayanars. Historians have attempted to explain this hostility by
suggesting that it was due to competition between members of other religious traditions for
royal patronage. What is evident is that the powerful Chola rulers (ninth to thirteenth
centuries) supported Brahmanical and bhakti traditions, making land grants and
constructing temples for Vishnu and Shiva.

50. Solution: d)
There is output of consumer goods and services and output of capital goods. The consumer
goods sustain the consumption of the entire population of the economy. Purchase of
consumer goods depends on the capacity of the people to spend on these goods which, in
turn, depends on their income.
The other part of the final goods, the capital goods, are purchased by business enterprises
either for maintenance or addition to their capital stock so that they can continue to maintain
or expand the flow of their production. In a specific time period, say in a year, the total
production of final goods can thus be either in the form of consumption or investment and

http://www.insightsonindia.com

INSIGHTS

Page 20

Insights Mock Tests 2015 Test 11 Solutions


there is thus a trade-off. If an economy, out of its current production of final goods,
produces more of consumer goods, it is producing less of investment goods and vice-versa.
For the options B and C, If you apply common logic, you can understand that the more
intermediate goods are produced, the more capital and consumer goods can be produced.
For e.g. steel is an intermediate good machine is a capital good and car is a final
consumer good. There need not be a trade off in these pairs.

51. Solution: d)
GNP GDP + Factor income earned by the domestic factors of production employed in the
rest of the world Factor income earned by the factors of production of the rest of the world
employed in the domestic economy
Hence, GNP GDP + Net factor income from abroad
Remittances sent means Indian domestic labour being employed abroad.
Reliance and BMW outlet are domestic and foreign factors of production employed in
abroad and domestic country respectively.

52. Solution: a)
http://www.bbc.com/news/science-environment-30587002
http://en.wikipedia.org/wiki/NuSTAR

53. Solution: d)
NNP GNP Depreciation
It is to be noted that all these variables are evaluated at market prices.
Through the expression given above, we get the value of NNP evaluated at market prices.
But market price includes indirect taxes. When indirect taxes are imposed on goods and
services, their prices go up. Indirect taxes accrue to the government. We have to deduct
them from NNP evaluated at market prices in order to calculate that part of NNP which
actually accrues to the factors of production.
Similarly, there may be subsidies granted by the government on the prices of some
commodities (in India petrol is heavily taxed by the government, whereas cooking gas is
http://www.insightsonindia.com

INSIGHTS

Page 21

Insights Mock Tests 2015 Test 11 Solutions


subsidised). So we need to add subsidies to the NNP evaluated at market prices. The
measure that we obtain by doing so is called Net National Product at factor cost or National
Income.

Thus, NNP at factor cost National Income (NI ) NNP at market prices (Indirect taxes
Subsidies) NNP at market prices Net indirect taxes (Net indirect taxes Indirect taxes
Subsidies) .

54. Solution: a)
If prices change, then there may be difficulties in comparing GDPs. If we measure the GDP
of a country in two consecutive years and see that the figure for GDP of the latter year is
twice that of the previous year, we may conclude that the volume of production of the
country has doubled. But it is possible that only prices of all goods and services have
doubled between the two years whereas the production has remained constant.
Therefore, in order to compare the GDP figures (and other macroeconomic variables) of
different countries or to compare the GDP figures of the same country at different points of
time, we cannot rely on GDPs evaluated at current market prices. For comparison we take
the help of real GDP. Real GDP is calculated in a way such that the goods and services are
evaluated at some constant set of prices (or constant prices). Since these prices remain
fixed, if the Real GDP changes we can be sure that it is the volume of production which is
undergoing changes. Nominal GDP, on the other hand, is simply the value of GDP at the
current prevailing prices.

55. Solution: d)
The ratio of nominal GDP to real GDP gives us an idea of how the prices have moved from
the base year (the year whose prices are being used to calculate the real GDP) to the current
year. In the calculation of real and nominal GDP of the current year, the volume of
production is fixed. Therefore, if these measures differ it is only due to change in the price
level between the base year and the current year. The ratio of nominal to real GDP is a well
known index of prices. This is called GDP Deflator. Thus if GDP stands for nominal GDP
and gdp stands for real GDP then, GDP deflator = GDP/Gdp
There is another way to measure change of prices in an economy which is known as the
Consumer Price Index (CPI). This is the index of prices of a given basket of commodities
which are bought by the representative consumer. CPI is generally expressed in percentage
terms.

http://www.insightsonindia.com

INSIGHTS

Page 22

Insights Mock Tests 2015 Test 11 Solutions


56. Solution: a)
An individual may hold her wealth in the form of landed property, bullion, bonds, money
etc. For simplicity, let us club all forms of assets other than money together into a single
category called bonds. Typically, bonds are papers bearing the promise of a future stream
of monetary returns over a certain period of time. These papers are issued by governments
or firms for borrowing money from the public and they are tradable in the market. This
demand for money is speculative demand for money.

57. Solution: b)
If interest rates increase, more people would want to save in banks; and less people would
hold bonds. This would reduce the demand for bonds and their value. People holdings
bonds would suffer a loss.
For a deposit holder, lower inflation means better returns on deposit. Because inflation
erodes value of interest accrued to the holder.

58. Solution: a)
The value of the currency notes and coins is derived from the guarantee provided by the
issuing authority of these items. Every currency note bears on its face a promise from the
Governor of RBI that if someone produces the note to RBI, or any other commercial bank,
RBI will be responsible for giving the person purchasing power equal to the value printed
on the note. The same is also true of coins. Currency notes and coins are therefore called fiat
money.
They do not have intrinsic value like a gold or silver coin. They are also called legal tenders
as they cannot be refused by any citizen of the country for settlement of any kind of
transaction. Cheques drawn on savings or current accounts, however, can be refused by
anyone as a mode of payment. Hence, demand deposits are not legal tenders.

59. Solution: c)
The twelfth century witnessed the emergence of a new movement in Karnataka, led by a
Brahmana named Basavanna (1106-68) who was initially a Jaina and a minister in the court
of a Chalukya king. His followers were known as Virashaivas (heroes of Shiva) or Lingayats
(wearers of the linga). Lingayats continue to be an important community in the region to
date. They worship Shiva in his manifestation as a linga, and men usually wear a small linga
in a silver case on a loop strung over the left shoulder. Those who are revered include the
jangama or wandering monks.
http://www.insightsonindia.com

INSIGHTS

Page 23

Insights Mock Tests 2015 Test 11 Solutions


Lingayats believe that on death the devotee will be united with Shiva and will not return to
this world. Therefore they do not practise funerary rites such as cremation, prescribed in the
Dharmashastras. Instead, they ceremonially bury their dead. The Lingayats challenged the
idea of caste and the pollution attributed to certain groups by Brahmanas. They also
questioned the theory of rebirth. These won them followers amongst those who were
marginalised within the Brahmanical social order. The Lingayats also encouraged certain
practices disapproved in the Dharmashastras, such as post-puberty marriage and the
remarriage of widows.
60. Solution: d)
Money supply, like money demand, is a stock variable. The total stock of money in
circulation among the public at a particular point of time is called money supply. RBI
publishes figures for four alternative measures of money supply, viz. M1, M2, M3 and M4.
They are defined as follows
M1 = CU + DD
M2 = M1 + Savings deposits with Post Office savings banks
M3 = M1 + Net time deposits of commercial banks
M4 = M3 + Total deposits with Post Office savings organisations (excluding National
Savings Certificates)
where, CU is currency (notes plus coins) held by the public and DD is net demand deposits
held by commercial banks. The word net implies that only deposits of the public held by
the banks are to be included in money supply. The interbank deposits, which a commercial
bank holds in other commercial banks, are not to be regarded as part of money supply.
M1 and M2 are known as narrow money. M3 and M4 are known as broad money. These
gradations are in decreasing order of liquidity. M1 is most liquid and easiest for transactions
whereas M4 is least liquid of all. M3 is the most commonly used measure of money supply.
It is also known as aggregate monetary resources.

61. Solution: b)
The concept will be difficult to explain in limited words here. You can best refer to the
section 3.3.2 Chapter 3 12th Macroeconomics NCERT.
It has been explained with the help of a numerical example there.

62. Solution: c)

http://www.insightsonindia.com

INSIGHTS

Page 24

Insights Mock Tests 2015 Test 11 Solutions


High CRR would mean banks would park more funds with RBI. Hence they can lend less to
the public. This would reduce the overall money supply. It will also lower demand and
tackle short-term inflation.
It has no immediate coorelation with reducing fiscal deficit in the short-run. That is
something best tackled by a prudent fiscal policy.

63. Solution: c)
Open Market Operations: RBI purchases (or sells) government securities to the general
public in a bid to increase (or decrease) the stock of high powered money in the economy.
Suppose RBI purchases Rs 100 worth government securities from the bond market. It will
issue a cheque of Rs 100 on itself to the seller of the bond. The seller will deposit the cheque
in her bank, which, in turn, will credit the sellers account with a balance of Rs 100. The
banks deposits go up by Rs 100 which is a liability to the bank. However, its assets also go
up by Rs 100 by the possession of this cheque, which is a claim on RBI. The bank will deposit
this cheque to RBI which, in turn, will credit the banks account with RBI with Rs 100.

64. Solution: a)
Sterilisation by RBI: RBI often uses its instruments of money creation for stabilising the stock
of money in the economy from external shocks. Suppose due to future growth prospects in
India investors from across the world increase their investments in Indian bonds which
under such circumstances, are likely to yield a high rate of return. They will buy these bonds
with foreign currency. Since one cannot purchase goods in the domestic market with foreign
currency, a person who sells these bonds to foreign investors will exchange her foreign
currency holding into rupee at a commercial bank. The bank, in turn, will submit this
foreign currency to RBI and its deposits with RBI will be credited with equivalent sum of
money.
This increased money supply may not altogether be good for the economys health. If the
volume of goods and services produced in the economy remains unchanged, the extra
money will lead to increase in prices of all commodities RBI often intervenes with its
instruments to prevent such an outcome. In the above example, RBI will undertake an open
market sale of government securities of an amount equal to the amount of foreign exchange
inflow in the economy, thereby keeping the stock of high powered money and total money
supply unchanged. Thus it sterilises the economy against adverse external shocks. This
operation of RBI is known as sterilisation.

65. Solution: d)

http://www.insightsonindia.com

INSIGHTS

Page 25

Insights Mock Tests 2015 Test 11 Solutions


First, certain goods, referred to as public goods (such as national defence, roads,
government administration), as distinct from private goods (like clothes, cars, food items),
cannot be provided through the market mechanism, i.e. by transactions between individual
consumers and producers and must be provided by the government. This is the allocation
function.
Second, through its tax and expenditure policy, the government attempts to bring about a
distribution of income that is considered fair by society. The government affects the
personal disposable income of households by making transfer payments and collecting taxes
and, therefore, can alter the income distribution. This is the distribution function.
Third, the economy tends to be subject to substantial fluctuations and may suffer from
prolonged periods of unemployment or inflation. The overall level of employment and
prices in the economy depends upon the level of aggregate demand which is a function of
the spending decisions of millions of private economic agents apart from the government.

66. Solution: d)
Revenue receipts are divided into tax and non-tax revenues.
Tax revenues consist of the proceeds of taxes and other duties levied by the central
government. Tax revenues, an important component of revenue receipts, comprise of direct
taxes which fall directly on individuals (personal income tax) and firms (corporation tax),
and indirect taxes like excise taxes (duties levied on goods produced within the country),
customs duties (taxes imposed on goods imported into and exported out of India) and
service tax.
Non-tax revenue of the central government mainly consists of interest receipts (on account
of loans by the central government which constitutes the single largest item of non-tax
revenue), dividends and profits on investments made by the government, fees and other
receipts for services rendered by the government. Cash grants-in-aid from foreign countries
and international organisations are also included.

67. Solution: a)
By the eleventh century Sufism evolved into a welldeveloped movement with a body of
literature on Quranic studies and sufi practices. Institutionally, the sufis began to organise
communities around the hospice or khanqah (Persian) controlled by a teaching master
known as shaikh (in Arabic), pir or murshid (in Persian). He enrolled disciples (murids) and
appointed a successor (khalifa). He established rules for spiritual conduct and interaction
between inmates as well as between laypersons and the master.
Sufi silsilas began to crystallise in different parts of the Islamic world around the twelfth
century. The word silsila literally means a chain, signifying a continuous link between
http://www.insightsonindia.com

INSIGHTS

Page 26

Insights Mock Tests 2015 Test 11 Solutions


master and disciple, stretching as an unbroken spiritual genealogy to the Prophet
Muhammad. It was through this channel that spiritual power and blessings were
transmitted to devotees. Special rituals of initiation were developed in which initiates took
an oath of allegiance, wore a patched garment, and shaved their hair
68. PSolution: d)
Budget documents classify total revenue expenditure into plan and non-plan expenditure.
Plan revenue expenditure relates to central Plans (the Five-Year Plans) and central assistance
for State and Union Territory Plans. Non-plan expenditure, the more important component
of revenue expenditure, covers a vast range of general, economic and social services of the
government.
The main items of non-plan expenditure are interest payments, defence services, subsidies,
salaries and pensions.
69. Solution: d)
The main items of capital receipts are loans raised by the government from the public which
are called market borrowings, borrowing by the government from the Reserve Bank and
commercial banks and other financial institutions through the sale of treasury bills, loans
received from foreign governments and international organisations, and recoveries of loans
granted by the central government. Other items include small savings (Post-Office Savings
Accounts, National Savings Certificates, etc), provident funds and net receipts obtained
from the sale of shares in Public Sector Undertakings (PSUs).
70. Solution: a)
The process of liberalisation also involved the taking of loans from international institutions
such as the International Monetary Fund (IMF). These loans are given on certain conditions.
The government makes commitments to pursue certain kind of economic measures that
involve a policy of structural adjustments. These adjustments usually mean cuts in state
expenditure on the social sector such as health, education and social security. There is also a
greater say by international institutions such as the World Trade Organisation (WTO).
71. Solution: d)
Take the following example It depends on the levels of the following
Primary Deficit = 2(a+b+c) + 5 - 1-4 - 2(a) = (2(a+b+c) 1 ) or RD (5-1-4)
Revenue Deficit (RD) = 2 (a+b+c) 1
If (5-1-4) is positive, i.e. if the capital expenditure in a particular year is high, then PD<RD,
else at times PD>RD.

http://www.insightsonindia.com

INSIGHTS

Page 27

Insights Mock Tests 2015 Test 11 Solutions

72. Solution: b)
http://www.downtoearth.org.in/content/india-high-alert-after-massive-oil-spill-threatenssunderbans-ecology
http://en.wikipedia.org/wiki/2014_Sundarbans_oil_spill
73. Solutions: d)
Kabir (c. fourteenth-fifteenth centuries) is perhaps one of the most outstanding examples of
a poet-saint who emerged within this context. Historians have painstakingly tried to
reconstruct his life and times through a study of compositions attributed to him as well as
later hagiographies. Such exercises have proved to be challenging on a number of counts.
Verses ascribed to Kabir have been compiled in three distinct but overlapping traditions.
The Kabir Bijak is preserved by the Kabirpanth (the path or sect of Kabir) in Varanasi and
elsewhere in Uttar Pradesh; the Kabir Granthavali is associated with the Dadupanth in
Rajasthan, and many of his compositions are found in the Adi Granth Sahib (see Section 8.2).
All these manuscript compilations were made long after the death of Kabir. By the
nineteenth century, anthologies of verses attributed to him circulated in print in regions as
far apart as Bengal, Gujarat and Maharashtra

74. Solution: d)
The government directly affects the level of equilibrium income in two specific ways
government purchases of goods and services (G) increase aggregate demand and taxes, and

http://www.insightsonindia.com

INSIGHTS

Page 28

Insights Mock Tests 2015 Test 11 Solutions


transfers affect the relation between income (Y) and disposable income (YD) the income
available for consumption and saving with the households.
A liberal FDI policy will bring more investment and increase the output of the economy.
75. Solution: d)
The options are self-explanatory. Investment abroad will be used to avoid excess spending
in domestic economy that would have caused extra demand. Moreover, returns can be
generated from abroad which will be used to bridge fiscal deficit.
76. Solution: a)
It has been argued that there is a decrease in investment due to a reduction in the amount of
savings available to the private sector. This is because if the government decides to borrow
from private citizens by issuing bonds to finance its deficits, these bonds will compete with
corporate bonds and other financial instruments for the available supply of funds. If some
private savers decide to buy bonds, the funds remaining to be invested in private hands will
be smaller. Thus, some private borrowers will get crowded out of the financial markets as
the government claims an increasing share of the economys total savings.
77. Solution: d)

http://www.insightsonindia.com

INSIGHTS

Page 29

Insights Mock Tests 2015 Test 11 Solutions

78. Solution: d)

This movement marked the first major anti British reaction and its
new political order initiated in 1849 among the people in the Punjab in 1857.

The Namdhari Movement, aftermath of the Kuka Movement was the most important
phase as it aimed at overthrowing the British rule and played important role in
freedom struggle.

It had evoked the strong feelings of self-respect and sacrifice for the countrys
freedom struggle.

This movement actively propagated the principles of boycott and non-co-operation


given by Guru Ram Singh (founded the Namdhari sect) for the Namdharis.

Gurus Non-co-operation Movement actively propagated few things such as boycott


of education institutions of British and laws established by them.

At the time of movement, the Kuka followers were rigid in their clothing and wore
only hand-spun white attire in order not to reveal their identity as large number of
followers were in the police as well as army.

79. Solution: d)
Refer to the section GDP and WELFARE in chapter 2- 12th Macroeconomics NCERT. It has
been explained comprehensively.

80. Solution: c)
In 1565 Rama Raya, the chief minister of Vijayanagara, led the army into battle at RakshasiTangadi (also known as Talikota), where his forces were routed by the combined armies of
Bijapur, Ahmadnagar and Golconda. The victorious armies sacked the city of Vijayanagara.
The city was totally abandoned within a few years. Now the focus of the empire shifted to
the east where the Aravidu dynasty ruled from Penukonda and later from Chandragiri (near
Tirupati).
Although the armies of the Sultans were responsible for the destruction of the city of
Vijayanagara, relations between the Sultans and the rayas were not always or inevitably
hostile, in spite of religious differences. Krishnadeva Raya, for example, supported some
claimants to power in the Sultanates and took pride in the title establisher of the Yavana
kingdom. Similarly, the Sultan of Bijapur intervened to resolve succession disputes in
Vijayanagara following the death of Krishnadeva Raya. In fact the Vijayanagara kings were
keen to ensure the stability of the Sultanates and vice versa. It was the adventurous policy of
http://www.insightsonindia.com

INSIGHTS

Page 30

Insights Mock Tests 2015 Test 11 Solutions


Rama Raya who tried to play off one Sultan against another that led the Sultans to combine
together and decisively defeat him.

81. Solution: d)
Rajasthan Governor has approved the Rajasthan Panchayati Raj (Second Amendment)
Ordinance-2014 after it was notified by the state government.
This ordinance will amend the Rajasthan Panchayati Raj Act 1994.
Key Provisions of Ordinance

Contestant for Zila Parishad or Panchayat Samiti elections should have the
minimum qualification of secondary education i.e. Class 10 from the state board or
any approved institution or board.

Contestant for the Sarpanch elections, must have passed Class VIII from any school
in case of general category. In case of the scheduled area of panchayat, the contestant
should have passed Class 5 from a school to become a Sarpanch.

82. Solution: d)
These sub-schemes and their allocations are:i. Bringing Green Revolution to Eastern Region: - This programme was initiated in
2010-11 targeting the improvement in the rice based cropping system of Assam, West
Bengal, Orissa, Bihar, Jharkhand, Eastern Uttar Pradesh and Chhattisgarh. Allocation for
this scheme in 2010-11 & 2011-12 was Rs. 400 crore each, which has been enhanced to Rs.
1000.00 crore in 2012-13 & 2013-14. The allocation for the year 2014-15 is Rs.1000.00 crore.
ii. Initiative on Vegetable Clusters: - Growing demand for vegetables was proposed to
be met by a robust increase in the productivity and market linkage. For the purpose, an
efficient supply chain needed to be established, to provide quality vegetables at competitive
prices. The allocation for this sub-scheme was Rs.300.00 crore each in 2011-12 & 2012-13. The
allocation for the year 2013-14 was Rs. 200.00 crore and 2014-15 is Rs. 175.00 crore.
iii. National Mission for Protein Supplements: - National Mission for Protein
Supplements was launched with an allocation of Rs.300 crore during 2011-12 to take up
activities to promote animal based protein production through livestock development, dairy
farming, piggery, goat rearing and fisheries in selected blocks. During 2012-13 & 2013-14 an
amount of Rs. 500 crore & Rs. 400.00 crore were allocated for 2014-15, Rs. 300.00 crore has
been earmarked for this scheme.
iv. Saffron Mission: - The Scheme was initiated in 2010-11 with an overall Government of
India budgetary support of Rs.288.06 crore over four years. Allocation has been Rs. 39.44
http://www.insightsonindia.com

INSIGHTS

Page 31

Insights Mock Tests 2015 Test 11 Solutions


crore in 2010-11, Rs.50.00 crore each in 2011-12 & 2012-13. The mission was meant to bring
economic revival of J&K Saffron. Outlay for the year 2013-14 was Rs. 100.00 crore. An
amount of Rs.100.00 crore is earmarked for 2014-15.
v. Vidharbha Intensive Irrigation Development Programme: - The Scheme was initiated
in 2012-13 which seeks to bring in more farming areas under protective irrigation. The
allocation for the year 2012-13 & 2013-14 was Rs. 300.00 crore each. For 2014-15 Rs. 150.00
crore has been allocated for VIIDP.
vi. Crop Diversification: - The original Green Revolution States have the problem of
stagnating yields and over-exploitation of water resources. The answer lies in crop
diversification. An amount of Rs.500.00 Crore was allocated for 2013-2014 to the start a
programme of crop diversification that would promote technological innovation and
encourage farmers to choose crop alternatives. For 2014-15 Rs. 250.00 crore has been
allocated for this scheme.
From PIB features.

83. Solution: a)
The current account records exports and imports in goods and services and transfer
payments. Trade in services denoted as invisible trade (because they are not seen to cross
national borders) includes both factor income (payment for inputs-investment income, that
is, the interest, profits and dividends on our assets abroad minus the income foreigners earn
on assets they own in India) and non-factor income (shipping, banking, insurance, tourism,
software services, etc.). Transfer payments are receipts which the residents of a country
receive for free, without having to make any present or future payments in return. They
consist of remittances, gifts and grants. They could be official or private.

84. Solution: b)

http://www.insightsonindia.com

INSIGHTS

Page 32

Insights Mock Tests 2015 Test 11 Solutions

85. Solution: d)
High CAD results in currency devaluation directly. Currency devaluation can directly feed
into inflation.
Thus a high CAD fuels directly into the domestic economy. Along with fiscal deficit (which
combinedly is called twin deficits), it leads to high inflation in the economy.
An overall consequence is that FII and other investors will not find the domestic market
worthy enough to invest and pull off from it.

86. Solution: a)
For e.g. If one wants to plan a trip to London, she needs to know how expensive British
goods are relative to goods at home.
The measure that captures this is the real exchange rate the ratio of foreign to domestic
prices, measured in the same currency. It is defined as
Real exchange rate = ePf/P
where P and Pf are the price levels here and abroad, respectively, and e is the rupee price of
foreign exchange (the nominal exchange rate). The numerator expresses prices abroad
http://www.insightsonindia.com

INSIGHTS

Page 33

Insights Mock Tests 2015 Test 11 Solutions


measured in rupees, the denominator gives the domestic price level measured in rupees, so
the real exchange rate measures prices abroad relative to those at home. If the real exchange
rate is equal to one, currencies are at purchasing power parity.

87. Solution: c)
Most of the tea gardens were situated in Assam. In 1903, the industry employed 4,79,000
permanent and 93,000 temporary employees. Since Assam was sparsely populated and the
tea plantations were often located on uninhabited hillsides, bulk of the sorely needed labour
had to be imported from other provinces. But to bring thousands of people every year from
their far-off homes into strange lands, possessing an unhealthy climate and infected with
strange fevers, required the provision of financial and other incentives, which the teaplanters of Assam were unwilling to offer. Instead, they had recourse to fraud and coercion;
and they persuaded the government to aid and abet them in this unholy task by passing
penal laws.

88. Solution: a)
The consumer can be concerned about future generations because they are the children and
grandchildren of the present generation and the family which is the relevant decision
making unit, continues living. They would increase savings now, which will fully offset the
increased government dissaving so that national savings do not change. This view is called
Ricardian equivalence after one of the greatest nineteenth century economists, David
Ricardo, who first argued that in the face of high deficits, people save more.
It is called equivalence because it argues that taxation and borrowing are equivalent means
of financing expenditure. When the government increases spending by borrowing today,
which will be repaid by taxes in the future, it will have the same impact on the economy as
an increase in government expenditure that is financed by a tax increase today.
It has often been argued that debt does not matter because we owe it to ourselves. This is
because although there is a transfer of resources between generations, purchasing power
remains within the nation. However, any debt that is owed to foreigners involves a burden
since we have to send goods abroad corresponding to the interest payments.

89. Solution: a)
eighteenth-century records tell us of zamindars in Bengal who remunerated blacksmiths,
carpenters, even goldsmiths for their work by paying them a small daily allowance and
diet money. This later came to be described as the jajmani system, though the term was not
in vogue in the sixteenth and seventeenth centuries.

http://www.insightsonindia.com

INSIGHTS

Page 34

Insights Mock Tests 2015 Test 11 Solutions


http://en.wikipedia.org/wiki/Jajmani_system

90. Solution: c)
If Indians travel abroad more often, the reverse will happen. The demand for foreign
currency will increase at the expense of Indian currency. So there will be depreciation, not
appreciation.
If more investment or export occurs, then domestic currency will appreciate. Because it will
bridge the CAD, and even make it positive at times.

91. Solution: c)
Change in Prices: Consider the effects of changes in prices, assuming the exchange rate to be
fixed. If prices of domestic products fall, while say foreign prices remain constant, domestic
exports will rise, adding to aggregate demand, and hence will raise our output and income.
Analogously, a rise in prices of a countrys exports will decrease that countrys net exports
and output and income. Similarly, a price increase abroad will make foreign products more
expensive and hence again raise net exports and domestic output and income. Price
decreases abroad have the opposite effects.
Exchange Rate Changes: Changes in nominal exchange rates would change the real
exchange rate and hence international relative prices. A depreciation of the rupee will raise
the cost of buying foreign goods and make domestic goods less costly. This will raise net
exports and therefore increase aggregate demand. Conversely, a currency appreciation
would reduce net exports and, therefore, decrease aggregate demand. However, we must
note that international trade patterns take time to respond to changes in exchange rates. A
considerable period of time may elapse before any improvement in net exports is apparent.

92. Solution: b)
Unlike Britain where the impact of industrialisation led to more people moving into urban
areas, in India the initial impact of the same British industrialisation led to more people
moving into agriculture. The Census of India Report shows this clearly.
The extensive importation of cheap European piece goods and utensils, and the
establishment in India itself of numerous factories of the Western type, have more or less
destroyed many village industries. The high prices of agricultural produce have also led
many village artisans to abandon their hereditary craft in favour of agriculture .
People started buying and wearing imported or domestically manufactured western wears.
This led to the decline of traditional cottage based cotton enterprises.

http://www.insightsonindia.com

INSIGHTS

Page 35

Insights Mock Tests 2015 Test 11 Solutions


93. Solution: d)
The spate of farmers suicides that has been occurring in the different parts of the country
since 1997-98 can be linked to the agrarian distress caused by structural changes in
agriculture and changes in economic and agricultural policies. These include: the changed
pattern of landholdings; changing cropping patterns especially due to the shift to cash crops;
liberalisation policies that have exposed Indian agriculture to the forces of globalisation;
heavy dependence on high-cost inputs; the withdrawal of the state from agricultural
extension activities to be replaced by multinational seed and fertiliser companies; decline in
state support for agriculture; and individualisation of agricultural operations. According to
official statistics, there have been 8,900 suicides by farmers between 2001 and 2006 in
Andhra Pradesh, Karnataka, Kerala and Maharashtra.

94. Solution: d)
The Defence Ministry has identified 4 strategic rail lines along the Indo-China border.
Four strategic rail lines are

Arunachal Pradesh: 378 kilometre long Missamari Tenga -Tawang and 227 kilometre
long Pasighat -Tezu -Rupai railway projects.

Jammu and Kashmir: 498 kilometre long Bilaspur- Manali Leh. It also includes some
part of Himachal Pradesh.

Assam: 249-kilometre long North Lakhimpur-Along-Silapathar.

However, Defence ministry has not identified any strategic railway line along Pakistan
border.
After the approval of Defence Ministry, Railway Ministry will carry out the final location
survey at a cost of Rs 345 crore.
The time of completion and cost of these projects will be finalized after detailed geotechnical studies and final sanction.

95. Solution: d)
The Emperor Akbar classified the lands and fixed a different revenue to be paid by each.
Polaj is land which is annually cultivated for each crop in succession and is never allowed to
lie fallow. Parauti is land left out of cultivation for a time that it may recover its strength.
Chachar is land that has lain fallow for three or four years. Banjar is land uncultivated for
five years and more. Of the first two kinds of land, there are three classes, good, middling,
and bad. They add together the produce of each sort, and the third of this represents the
medium produce, one-third part of which is exacted as the Royal dues.
http://www.insightsonindia.com

INSIGHTS

Page 36

Insights Mock Tests 2015 Test 11 Solutions


96. Solution: a)
Non-tax revenue of the central government mainly consists of interest receipts (on account
of loans by the central government which constitutes the single largest item of non-tax
revenue), dividends and profits on investments made by the government, fees and other
receipts for services rendered by the government. Cash grants-in-aid from foreign countries
and international organisations are also included.

97. Solution: c)
While dollars are the reserve currency of the global financial system, Forex reserves contain
other currencies too like Yen, Euro, Pound etc. SDRs are also used to settle trade related
payments.

98. Solution: a)
The MINERAL that makes 38 per cent of Earth has finally been named-bridgmanite. For
around 50 years, the mineral, a form of magnesium iron silicate found 660 km inside Earth,
could not be studied or named because it did not survive the trip to the surface and its
properties remained unknown. However, meteorites that fall on Earth have the same
element in "frozen" form. X-ray analysis of Tenham, a meteorite that crashed in Australia in
1879 and had bridgmanite grains, revealed that mineral contains high amounts of ferric iron
and sodium
http://en.wikipedia.org/wiki/Silicate_perovskite

99. Solution: d)
http://www.bbc.com/news/health-30641833

100.

Solution: c)

http://www.downtoearth.org.in/content/so-we-can-breathe-easy
Atmospheric particulate matter also known as particulate matter (PM) or particulates is
microscopic solid or liquid matter suspended in the Earth's atmosphere. The term aerosol
commonly refers to the particulate/air mixture, as opposed to the particulate matter
alone.[3] Sources of particulate matter can be man-made or natural. They have impacts on
climate and precipitation that adversely affect human health. Subtypes of atmospheric
particle matter include suspended particulate matter (SPM), respirable suspended particle

http://www.insightsonindia.com

INSIGHTS

Page 37

Insights Mock Tests 2015 Test 11 Solutions


(RSP; particles with diameter of 10 micrometres or less), fine particles (diameter of 2.5
micrometres or less), ultrafine particles, and soot.
The IARC and WHO designate airborne particulates a Group 1 carcinogen. Particulates are
the deadliest form of air pollution due to their ability to penetrate deep into the lungs and
blood streams unfiltered, causing permanent DNA mutations, heart attacks, and premature
death.[4] In 2013, a study involving 312,944 people in nine European countries revealed that
there was no safe level of particulates and that for every increase of 10 g/m3 in PM10, the
lung cancer rate rose 22%. The smaller PM2.5 were particularly deadly, with a 36% increase
in lung cancer per 10 g/m3 as it can penetrate deeper into the lungs

http://www.insightsonindia.com

INSIGHTS

Page 38

INSIGHTS ON INDIA MOCK PRELIMINARY EXAM - 2015


INSIGHTS ON INDIA MOCK TEST - 12
GENERAL STUDIES

PAPER-I
Time Allowed: 2 Hours

Maximum Marks: 200

INSTRUCTIONS
1. IMMEDITELY AFTER THE COMMENCEMENT OF THE EXAMINATION, YOU SHOULD
CHECK THAT THIS TEST BOOKLET DOES NOT HAVE ANY UNPRINTED OR TORN OR MISSING
PAGES OR ITEMS, ETC. IF SO, GET IT REPLACED BY A COMPLETE TEST BOOKLET.
2. You have to enter your Roll Number on the Test
Booklet in the Box provided alongside. DO NOT
Write anything else on the Test Booklet.
4. This Test Booklet contains 100 items (questions). Each item is printed only in English. Each item
comprises four responses (answers). You will select the response which you want to mark on the
Answer Sheet. In case you feel that there is more than one correct response, mark the response which
you consider the best. In any case, choose ONLY ONE response for each item.
5. You have to mark all your responses ONLY on the separate Answer Sheet provided. See directions in
the Answer Sheet.
6. All items carry equal marks.
7. Before you proceed to mark in the Answer Sheet the response to various items in the Test Booklet, you
have to fill in some particulars in the Answer Sheet as per instructions sent to you with your
Admission Certificate.
8. After you have completed filling in all your responses on the Answer Sheet and the examination has
concluded, you should hand over to the Invigilator only the Answer Sheet. You are permitted to take
away with you the Test Booklet.
9. Sheets for rough work are appended in the Test Booklet at the end.
10. Penalty for wrong answers :
THERE WILL BE PENALTY FOR WRONG ANSWERS MARKED BY A CANDIDATE IN THE
OBJECTIVE TYPE QUESTION PAPERS.
(i) There are four alternatives for the answer to every question. For each question for which a
wrong answer has been given by the candidate, one-third of the marks assigned to that
question will be deducted as penalty.
(ii) If a candidate gives more than one answer, it will be treated as a wrong answer even if one of
the given answers happens to be correct and there will be same penalty as above to that
question.
(iii)

If a question is left blank, i.e., no answer is given by the candidate, there will be no penalty
for that question.
http://www.insightsonindia.com

INSIGHTS ON INDIA MOCK TEST SERIES FOR CIVIL SERVICES PRELIMINARY EXAM 2015
http://insightsonindia.com

INSIGHTS

Page 1

1. Which of the following were NOT the


subjects of the prehistoric cave paintings?
1. Human activities
2. Geometric designs
3. Excavations
Choose the correct answer using the codes given
below:
a)
b)
c)
d)

1 and 2
Only 3
Only 2
None were the concerned subjects

a)
b)
c)
d)

1 and 2
2 and 3
1 and 3
All of the above

4. Consider the following statements about


the International Monetary Fund (IMF).
1. It is an organ of United Nations (UN).
2. All members of the IMF do not have
equal say in its affairs.
3. It is also an infrastructural lending
financial institution.
Choose the correct answer using the codes below:

2. At places like Tekkalkota, in Karnataka


and Andhra Pradesh, rock paintings from
Neolithic age are often found. Which of
the following can be an appropriate
reason for it?
a) The population density of the
Neolithic man was one of the highest
in the region.
b) Granite rocks found in Karnataka and
AP are suitable for rock paintings.
c) Due to the rock structure of the region,
there was a lack of deep caves, so rock
paintings became abundant.
d) Paintings existed in other regions too,
but these regions were one of the most
excavated regions which made it
easier to find rock paintings
3. Consider the following statements about
the Bhimbetka rock paintings.
1. It is found in Vindhya Hills.
2. These paintings do not cover sacred
and royal images.
3. Artistic themes were covered in these
paintings.

Choose the correct answer using the codes given


below:

http://insightsonindia.com

a)
b)
c)
d)

1 and 2 only
2 and 3 only
2 only
1 and 3 only

5. International organizations and


cooperation is helpful because some
challenges transcend national boundaries.
For which of these challenges,
International cooperation is a must?
1. Preventing Cyber Crimes
2. Tackling Population explosion
3. Tackling Climate Change
4. Tackling epidemics
Choose the correct answer using the codes below:
a)
b)
c)
d)

2 and 4 only
1 and 3 only
1, 3 and 4 only
All of the above

6. Which of the following are NOT the


principal organs of the United Nations
(UN)?
1. International Court of Justice
2. Economic and Social Council
3. WTO
Choose the correct answer using the codes below:

INSIGHTS

Page 2

a)
b)
c)
d)

1 and 2 only
2 and 3 only
3 only
1 and 3 only

7. Consider the following statements about


the appointment of judges to the
International Court of Justice, Hague?
1. They are appointed for a term of five
years.
2. They are appointed solely by the
UNSC.
3. Veto in the UNSC is not applicable for
the appointment of these judges.
Choose the correct answer using the codes below:
a)
b)
c)
d)

1 only
2 and 3 only
3 only
None of the above

8. Consider the following statements about


the manner in which decisions are made
in the UN General Assembly (UNGA).
1. Each member state has one vote.
2. All decisions are passed by a twothirds majority.
3. Its decisions are not binding on all UN
members.

1. Its non-permanent members are


elected by the UNGA for a term of 2
years.
2. Its decision is binding on all UN
members.
3. Any expansion in the composition of
the UNSC will need the approval of
both the UNSC and UNGA.
Choose the correct answer using the codes below:
a)
b)
c)
d)

10. Apart from the changed power relations


in the World, which of the following
arguments would force a revisit of the
composition of the UNSC?
1. The economies of Asia are growing at
an unprecedented rate.
2. A whole new set of challenges
confronts the world like climate
change, ethnic conflicts.
3. Democratic global governance is only
a logical extension of the spread of
democracy across the globe.
Choose the correct answer using the codes below:
a)
b)
c)
d)

Choose the correct answer using the codes below:


a)
b)
c)
d)

1 only
2 and 3 only
1 and 3 only
All of the above

9. Consider the following statements about


the composition and working of the
United Nations Security Council (UNSC).

1 only
2 and 3 only
1 and 3 only
All of the above

1 and 2 only
2 and 3 only
1 and 3 only
All of the above

11. The World Bank is NOT mandated to


fund projects which fall in the ambit of
1. Disaster management
2. Environmental protection
3. Good governance
4. Rural public services
Choose the correct answer using the codes below:

http://insightsonindia.com

INSIGHTS

Page 3

a)
b)
c)
d)

1 and 3
1, 2 and 3
3 and 4 only
All the above can be funded by WB

12. The World Trade Organisation (WTO) is


an international organisation which sets
the rules for global trade. Consider the
following statements about it.
1. All its decisions are taken
unanimously.
2. Major economic powers have a veto
power on the major decisions.
3. It can punish the member nations for
violation of trade rules.
Choose the correct answer using the codes below:
a)
b)
c)
d)

1 and 2 only
2 and 3 only
1 and 3 only
All of the above

13. Statues in which of the following


materials were NOT found at Harappan
sites?
1. Bronze
2. Silver
3. Terracotta
Choose the correct answer using the codes given
below:
a)
b)
c)
d)

1 and 2
2 only
1 and 3
3 only.

14. A technique called lost-wax technique


was practised by the the Harappans to
caste Bronze statues. Which of the
following was/were NOT a part of the
technique?
1. Coating of clay
2. Draining wax
3. Use of Iron beads
http://insightsonindia.com

Choose the correct answer using the codes given


below:
a)
b)
c)
d)

1 and 2
3 only
1 only
2 only

15. Consider the following statements about


the seals found at the Harappan sites.
1. Gold and ivory was also used for
making these seals.
2. Even Unicorn and Rhinoceros were
depicted on these seals.
3. The purpose of producing seals was
solely artistic.
Choose the correct answer using the codes given
below:
a)
b)
c)
d)

1 and 2
2 and 3
1 and 3
All of the above

16. From archaeological finds it appears that


the people of the Indus Valley were
conscious of fashion. The use of which of
the following by them prompts towards
this proposition?
1. Different hairstyles and keeping of
beard
2. Use of cosmetics
3. Use of jewellery
Choose the correct answer using the codes given
below:
a)
b)
c)
d)

INSIGHTS

1 and 2
2 and 3
1 and 3
All of the above.

Page 4

17. Match the following traditional form of


murals with the states they are found in.
1. Pithoro Maharashtra
2. Warli Rajasthan
3. Mithila Uttar Pradesh
4. Nayaka paintings Tamil Nadu
Choose the correct answer using the codes below:
a)
b)
c)
d)

1 and 2 only
2 and 4 only
4 only
1 and 3 only

18. The Vesar style of temples as an


independent style was created through
the selective mixing of which of the
following orders?
1. Nagara
2. Phamsana
3. Dravida
4. Valabhi
Choose the correct answer using the codes below:
a)
b)
c)
d)

1 and 2 only
2 and 4 only
3 and 4 only
1 and 3 only

19. Consider the following depictions in


temple of Dashavtara Vishnu of Central
India.
1. Nara-Narayan Shows the discussion
between human soul and the eternal
divine.
2. Gajendramoksha Vishnus
discussion with the lord of elephants.
3. Rekha-Pasanda Vishnus
suppression of an asura who later
attains
moksha.
Which of the above associations are correct?
a) 1 and 2
http://insightsonindia.com

b) 2 and 3
c) 1 only
d) All of the above

20. Consider the following about the Odisha


style of temples.
1. It is a sub-style of the Dravida order.
2. Khakra is one of the main architectural
orders associated with+ it.
3. The exterior of the temples are lavishly
carved, their interiors are generally
quite bare.
Choose the correct answer using the codes below:
a)
b)
c)
d)

1 and 2
2 and 3
1 and 3
All of the above

21. A unique form of architecture developed


in the hills of Kumaon, Garhwal,
Himachal and Kashmir in India with the
use of wooden architecture, pagoda
shapes etc. This can be attributed to the
influence of which of the following?
1. Gandhara school of art
2. Buddhist traditions
3. Hindu traditions
Choose the correct answer using the codes below:
a)
b)
c)
d)

1 and 2
2 and 3
1 and 3
All of the above

22. Which of the following are cited as


possible reasons for the Cuban Missile
Crisis, 1962?
1. Spread of capitalism in Cuba
2. Impending USA led invasion of Cuba
3. Installation of nuclear missiles in Cuba
INSIGHTS

Page 5

2. For getting locations to spy on the


other superpower
3. For military bases
4. Ideological spread

Choose the correct answer using the codes below:


a)
b)
c)
d)

1 and 2
2 and 3
1 and 3
All of the above

23. Which of the following could have been


the possible effects and consequences of
the Cold war between USA and USSR?
1. Attempts by both superpowers to
promote their respective political and
economic ideologies
2. Nuclear arms proliferation in the
world
3. Formation of military alliances across
the globe
Choose the correct answer using the codes below:
a)
b)
c)
d)

1 and 2
2 and 3
1 and 3
All of the above

24. What were the possible advantages that


accrued to the smaller states that became
part of the power blocs formed by USA
and USSR during the Cold war?
1. Economic aid from superpowers
2. Military support
3. Greater voting rights in International
bodies
Choose the correct answer using the codes below:
a)
b)
c)
d)

1 and 2
2 and 3
1 and 3
All of the above

Choose the correct answer using the codes below:


a)
b)
c)
d)

26. Which of the following significant


events/actions were associated with the
Cold war in that period?
1. Truman Doctrine
2. Berlin Blockade
3. American intervention in Vietnam
4. Soviet intervention in Hungary
Choose the correct answer using the codes below:
a)
b)
c)
d)

1 and 4
2 and 3
1, 3 and 4
All of the above

27. Culmination of which of the following


factors might possibly have led to the start
of the Non-Alignment Movement (NAM)?
1. Growing cold war tensions
2. The international entry of newly
decolonized African nations
3. The economic impact of the Second
World War
Choose the correct answer using the codes below:
a)
b)
c)
d)

25. In the power blocs formed by the


superpowers, why were the smaller states
important to the superpowers?
1. For getting mineral resources
http://insightsonindia.com

1 and 4
2 and 3
1, 3 and 4
All of the above

INSIGHTS

1 and 2
2 and 3
1 and 3
All of the above

Page 6

28. The Non-Alignment Movement believed


in which of the following principles?
1. Isolationism from International
conflicts
2. Neutrality in International affairs
3. Equal opportunities for the economic
development of recently decolonized
nations

2. The Treaty of Friendship in 1971 with


the USSR for 20 years made India
virtually a member of the Soviet
Alliance system.
Choose the answer using the codes below.
a)
b)
c)
d)

Choose the correct answer using the codes below:


a)
b)
c)
d)

1 and 2 only
2 and 3 only
3 only
All of the above

29. During the Cold War, the idea of a New


International Economic Order (NIEO) as
proposed by the UNCTAD was
1. Giving the least developed countries
(LDCs) greater access to their own
natural resources.
2. Greater western market access for
LDCs
3. Providing the LDCs with a greater role
in the International institutions

1 only
2 only
Both 1 and 2
None strengthens the proposition

31. Several arms control treaties were signed


during the Cold War. Which of these were
from amongst those?
1. Limited Test Ban treaty
2. Nuclear Non-Proliferation treaty
3. Strategic Arms Reduction Treaty
Choose the correct answer using the codes below:
a)
b)
c)
d)

1 and 2 only
2 and 3 only
1 and 3 only
All of the above

Choose the correct answer using the codes below:


a)
b)
c)
d)

1 and 2 only
2 and 3 only
3 only
All of the above

30. Critics allege Indias non-alignment as


unprincipled during the Cold War.
Which of the following arguments would
strengthen the proposition?
1. India refused to take a firm stand on
crucial international issues like
Russian intervention of
Afghanistan.

http://insightsonindia.com

32. Which of these could NOT have been the


possible reasons for the breakdown of
Soviet Union?
1. A stagnant soviet economy
2. Authoritarian Soviet government
3. Coup in the communist party
Choose the correct answer using the codes below:
a)
b)
c)
d)

INSIGHTS

3 only
1 and 2 only
1 only
None of the options could be denied as
possible reasons.

Page 7

33. While reforms led by Soviet leader


Gorbachev were aimed at reforming the
Soviet Union, they also lead to the
breakdown of the Soviet Union. What
such reforms were introduced by
Gorbachev after 1985?
1. Freedom to the media
2. Making Soviet Communist party more
democratic
3. Allowing small private industries to
operate
Choose the correct answer using the codes below:
a)
b)
c)
d)

1 and 2 only
2 and 3 only
1 and 3 only
All of the above

34. The consequences of the breakdown of


Soviet Union had global proportions.
Which of the following could have been
the possible consequences?
1. End of cold war confrontations
2. Emergence of new countries in Europe
3. Shift in the balance of power in the
world
Choose the correct answer using the codes below:
a)
b)
c)
d)

1 and 2 only
2 and 3 only
1 and 3 only
All of the above

35. The model of transition in Russia, Central


Asia and east Europe that was influenced
by the World Bank and the IMF after the
breakdown of the Soviet Union came to be
known as shock therapy. It included
1. Collective farms were replaced by
private farming and capitalism in
agriculture.
2. Privatisation of state assets.
http://insightsonindia.com

3. Break up of the existing trade alliances


among the countries of the Soviet
bloc
Choose the correct answer using the codes below:
a)
b)
c)
d)

1 and 2 only
2 and 3 only
1 and 3 only
All of the above

36. In 2003, the US launched its invasion of


Iraq under the codename Operation Iraqi
Freedom. It was intended to
1. Prevent Iraq from developing
Weapons of Mass Destruction (WMD)
2. Contain the anti-democratic uprising
in Iraq
3. Protect Iraq from the attacks by the
neighbouring Gulf States
Choose the correct answer using the codes below:
a)
b)
c)
d)

1 and 2 only
2 and 3 only
1 only
All of the above

37. The USA has had a hegemonic position in


the international sphere since the end of
Cold War especially by exercising its
military influence. However, its
hegemonic power is subject to some
constraints. These may be
1. American Mass Media and Civil
Society
2. The moderating influence of NATO
3. Permanent members of the UNSC
Choose the correct answer using the codes below:
a)
b)
c)
d)

INSIGHTS

1 and 2 only
2 and 3 only
3 only
All of the above

Page 8

38. In what ways had the Indian government


involved itself in the problem of Sri
Lankan Tamils in Sri Lanka since the
origin of the problem?
1. Deploying Indian Peace Keeping
forces in SL
2. Pushing SL diplomatically on the 13th
amendment to its constitution
3. Taking up housing projects in war
torn areas

41. Nepal and India enjoy a very special


relationship that has very few parallels in
the world due to the Indo-Nepal Treaty of
friendship 1950. It includes
1. Porous borders
2. Work permit for Nepali nationals in
India
3. Allowing Nepali nations to acquire
land in India
Choose the correct answer using the codes below:

Choose the correct answer using the codes below:


a)
b)
c)
d)

1 and 2 only
2 and 3 only
1 and 3 only
All of the above

39. The conflicts between India and Pakistan


in 1948 and 1965 were possibly rooted in
the
1. Status of Kashmir
2. Status of Bangladesh
3. Control of the Siachen glacier
Choose the correct answer using the codes below:
a)
b)
c)
d)

a)
b)
c)
d)

1 and 2 only
2 and 3 only
1 and 3 only
All of the above

40. India and Pakistan are not in agreement


over the demarcation line in Sir Creek in
the Rann of Kutch. It is important for both
the nations because of
a) control of sea resources in the area
adjoining Sir Creek
b) its implications for the Indus water
treaty
c) the possibilities of effective
coordination over containing marine
piracy
d) the need to contain terrorism across
the sea lanes of communication
http://insightsonindia.com

1 and 2 only
2 and 3 only
1 and 3 only
All of the above

42. Lomus rishi caves are historically


important caves found at Barabar hills.
Consider the following statements about
it.
1. They are rock-cut caves found in
Gaya.
2. The cave was patronised by Ashoka
for the Ajivika sect.
3. The facade of the cave is decorated
with the semicircular chaitya arch as
the entrance.
Choose the correct answer using the codes given
below:
a)
b)
c)
d)

1 and 2
2 and 3
1 and 3
All of the above

43. Consider the following statements about


the Lion capital, Sarnath.
1. It was built in commemoration of the
historical event of the first sermon by
the Buddha at Sarnath.
2. It was discovered in the Medieval
period.
INSIGHTS

Page 9

3. The wheel inscribed in the Lion capital


is the Dharmachakra.
Choose the correct answer using the codes given
below:
a)
b)
c)
d)

3. Its influence spread to Northern India


too.
Choose the correct answer using the codes given
below:

1 and 2
2 and 3
1 and 3
All of the above

44. During the early phase of Buddhism,


Buddha is depicted symbolically through
footprints, stupas, lotus throne, chakra,
etc. This was because
a) Buddha had discouraged his
representation in any human form.
b) Buddhism was influenced from
particular practices from Hinduism.
c) These showed his Mahaparinirvana.
d) Construction of such symbols were
patronized by the Mauryan rulers

a)
b)
c)
d)

47. Consider the following statements about


the Gandhara school of art.
1. Images of Vaishnava and Shaiva are
also depicted alongwith the images of
Buddha.
2. Stupa sculpture was also a part of it.
3. It was developed owing to local
conditions only.
Choose the correct answer using the codes given
below:
a)
b)
c)
d)

45. Which of the following were NOT


depicted in Buddhist stupas?
1. Jatakas
2. Toranas
3. Paradakshas
Choose the correct answer using the codes given
below:
a)
b)
c)
d)

1 and 2
3 only
1 only
2 only

1 and 2
2 and 3
1 and 3
All of the above

1 and 2
2 and 3
1 and 3
All of the above

48. In South India, Buddhist monuments can


be found in these locations
1. Nagarjunkonda
2. Jagayyapetta
3. Amaravati
4. Goli
Choose the correct answer using the codes given
below:

46. Consider the following statements about


the Mathura school of art.
1. Buddha was depicted in human form.
2. It was a confluence of the Bactrian and
local Mathura tradition.

http://insightsonindia.com

a)
b)
c)
d)

INSIGHTS

2 and 3
1, 3 and 4
1 and 4
All of the above.

Page 10

49. Boddhisattva images were added as a part


of the personified representations of
certain virtues or qualities as propagated
by the Buddhist religious principles for
the welfare of the masses with the rise of
a) Mahayana Buddism
b) Vajrayana Buddhism
c) Gandhara School of art
d) Mathura school of art

50. Consider the following statements.


1. In Karla, the biggest rock-cut chaitya
hall was excavated.
2. Junnar has the largest Buddhist cave
excavations.
3. Roof Chaitya halls are found at Ajanta.
Choose the correct answer using the codes given
below:
a)
b)
c)
d)

1 and 2
2 and 3
1 and 3
All of the above

51. Consider the following sculptures, images


and idols depicted.
1. It contains a Buddhist sculpture seated
in a chaitya hall.
2. Idol sculpture of Gajasur Shiva is
found there.
3. Ravana shaking Mount Kailash is also
depicted alongwith other Vaishnavite
themes.
Which of the above are found in Ellora caves?
a)
b)
c)
d)

1 and 2
2 and 3
1 and 3
All of the above.

http://insightsonindia.com

52. Consider the following statements about


Elephanta caves located near Mumbai.
1. The entrance to Elephanta caves is
guarded by huge pillars.
2. It is a rock-cut cave.
3. Originally a Buddhist site, it came to
be dominated by the Shaivite faith.
Choose the correct answer using the codes below.
a)
b)
c)
d)

1 and 2
2 and 3
1 and 3
All of the above.

53. The International Atomic Energy Agency


(IAEA) was established in 1957. It has
been in news frequently. Consider the
following statements about it.
1. It is associated with the UN.
2. It seeks to prevent countries from
using any form of nuclear energy.
3. Nations which have signed the NonProliferation Treaty (NPT) are
required to allow regular inspections
by the IAEA.
Choose the correct answer using the codes below:
a)
b)
c)
d)

1 and 2 only
2 and 3 only
1 and 3 only
All of the above

54. Consider the following statements.


1. The members of the General Assembly
are automatically the members of all
other principal organs and specialised
agencies of the UN.
2. The UN Secretary-General can veto
the decision of the UN General
assembly when the decision goes
against international public interest.

INSIGHTS

Page 11

3. One veto can stall a Security Council


resolution.

a)
b)
c)
d)

Which of the above are correct?


a)
b)
c)
d)

1 and 2 only
2 and 3 only
1 and 3 only
3 only

55. Global Commons are areas or regions of


the world which are located outside the
sovereign jurisdiction of any one state,
and therefore require common
governance by the international
community. Which of the following are
related to managing the Global
Commons?
1. Montreal Protocol
2. 1991 Antarctic Environment protocol
3. Kyoto Protocol

1 and 2 only
2 and 3 only
1 and 3 only
All of the above

57. When we say India and China have


agreed on implementing some Confidence
Building measures (CBMs), we essentially
mean that
a) They have agreed for a total cease fire
across the international borders
b) They have agreed to improve the
relations between the armed personnel
of both countries across the borders
c) They have agreed to exchange
information on defence matters
between them on a regular basis
d) They have agreed to halt down
military modernization drives across
the border

Choose the correct answer using the codes below.


a)
b)
c)
d)

1 and 2 only
2 and 3 only
1 and 3 only
All of the above

56. Antarctica is a global common. In what


ways is Antarctica important to the
mankind?
1. It plays an important role in
maintaining climatic equilibrium as it
holds more than 90% of the terrestrial
ice.
2. Its deep ice cores provide an
important source of information about
greenhouse gas concentrations.
3. Its melting will provide an alternative
route of global trade.

58. Consider the following statements


1. The Permanent Settlement system was
introduced by the British to encourage
investment in agriculture
2. The British expected the Permanent
Settlement system would help the
emergence of a class of yeomen
farmers who would be loyal to the
Company
Which of the above statements is/are correct?
a)
b)
c)
d)

1 only
2 Only
Both
None

Which of the above are correct?


http://insightsonindia.com

INSIGHTS

Page 12

59. In the early decades after the Permanent


Settlement, zamindars regularly failed to
pay the revenue demand and unpaid
balances accumulated. Consider the
following reasons:
1. The revenue demand was very high
from the British
2. The revenue was invariable, regardless
of the harvest, and had to be paid
punctually
3. Zamindars lost their power to organise
local justice and the local police
Which of the above statements is/are correct?
a)
b)
c)
d)

2 and 3 Only
3 and 1 Only
1 and 2 Only
All

60. With reference to jotedars who emerged


during the Permanent Settlement days in
Bengal, consider the following statements
1. The power of jotedars was more
effective than that of zamindars in
whole of Bengal
2. Like zamindars who often lived in
urban areas, jotedars were also located
in the urban regions
Which of the above statements is/are correct?
a)
b)
c)
d)

2 Only
1 Only
None
Both

b) They raided the plains where settled


agriculturists lived during the years of
scarcity
c) Only option a is correct
d) Both a and b are correct

62. Consider the following statements


1. This country is home
to Escondida, which is the largest
copper mine in the world, producing
over 5% of global supplies
2. This country has Bolivia to the
northeast and the Drake Passage in the
far south
The above description refers to which of the
following countries?
a)
b)
c)
d)

Peru
Chile
Argentina
Venezuela

63. Recently the World Health Organization


allowed the use of meningitis vaccine
MenAfriVac on infants in Africa. Consider
the following statements with reference to
this vaccine
1. It is manufactured by the Serum
Institute of India Ltd
2. It is used to treat Measles and
Nephrotic Syndrome among children
Which of the above statements is/are correct?
a)
b)
c)
d)

61. In the 1770s the British embarked on a


brutal policy of extermination, hunting the
Paharias down and killing them.
Regarding these Paharis, which of the
following statements is correct?
a) They were hill folks who lived around
the Rajmahal hills
http://insightsonindia.com

INSIGHTS

1 Only
2 Only
Both
None

Page 13

64. With reference to Waves and Wave


Energy consider the following statements
1. Waves are generated by wind passing
over the surface of the sea
2. Wave energy is nothing but Tidal
energy which are basically same
3. Production of wave energy has zero
environmental impact
Which of the above statements is/are correct?
a) 1 and 2 Only
b) 3 and 2 Only
c) 1 and 3 Only
d) 1 Only
65. With reference to Blackbuck, consider the
following statements
1. It is classified as near
threatened by IUCN since 2003
2. The blackbuck is the only living
species of the genus Antilope
3. Blackbuck National Park at Velavadar
near Jaipur in Rajasthan is home to
largest number of Blackbucks in India
Which of the above is/are correct?
a)
b)
c)
d)

1 Only
1 and 2 Only
1 and 3 Only
2 and 3 Only

66. With reference to Masais, consider the


following statements
1. They are ethnic group of seminomadic people inhabiting southern
Kenya and northern Tanzania
2. They are primarily cattle herders and
all of their needs for food are met by
their cattle
Which of the above statements is/are correct?
a) 1 Only
b) 2 Only
c) Both
d) None
http://insightsonindia.com

67. Recently the Prime Minister of India


described the LED bulb as a Prakash
Path way to light. Why LED bulbs
are encouraged over incandescent and
fluorescent bulbs? Consider the following
statements:
1. LED lamps have a lifespan and
electrical efficiency that is several
times better than incandescent lamps,
and significantly better than most
fluorescent lamps
2. LEDs come to full brightness without
need for a warm-up time and consume
less power
3. LED lamps are insensitive to excessive
heat
Which of the above statements is/are correct?
a)
b)
c)
d)

All
3 and 2 Only
1 and 2 Only
3 and 1 Only

68. With reference to Santhals, consider the


following statements
1. They are the largest tribe in India to
retain a good language to the present
day
2. They are the largest tribal community
in India
3. They originally resided peacefully in
the hilly districts
of Mayurbhanj Chhotanagpur,
Palamau, Hazaribagh, but the British
policies pushed them towards
Rajmahal Hills
Which of the above statements is/are correct?
a)
b)
c)
d)
INSIGHTS

3 and 2 Only
All
1 and 2 Only
1 and 3 Only
Page 14

69. With reference to water storage capacity


in India and the functions of Central
Water Commission, consider the
following statements
1. Central Water Commission monitors
live storage status of 85 important
reservoirs of the country on weekly
basis
2. The total storage capacity of these 85
reservoirs is about 61% of the storage
capacity of 253.388 BCM which is
estimated to have been created in the
country
3. Among these 85 reservoirs, the Central
region under which Uttar Pradesh,
Uttarakhand, Madhya Pradesh and
Chhattisgarh come has more
reservoirs compared to other regions
Which of the above statements is/are correct?
a)
b)
c)
d)

Which of the above statements is/are correct?


a)
b)
c)
d)

72. Recently the government announced that


it has taken steps to encourage production
and availability of fortified and Neem
coated urea in the country. This is because
1. Neem coated urea reduces Nitrogen
loss by more than 10%
2. Neem coated urea will not only
increase crop yields but also lower
input cost to farmers
3. Its use will eliminate import of urea
resulting in huge foreign exchange
savings
Which of the above statements is/are correct?

1 and 3 Only
3 and 2 Only
All
1 and 2 Only

70. When the American Civil War broke out


in 1861, it had profound effect on cotton
growers in India. The cotton exports from
India
a) Increased
b) Decreased
c) Completely stopped
d) None of the above
71. With reference to Sepoy Mutiny of 1857,
consider the following statements
1. Proclamations were put up only in
Hindi (to reach common man) in the
cities calling upon the population,
both Hindus and Muslims, to unite,
rise and exterminate the firangis
2. Moneylenders and the rich were the
targets of common man who joined
the rebellion
http://insightsonindia.com

1 Only
Both
2 Only
None

a)
b)
c)
d)

1 and 3 Only
3 Only
2 and 3 Only
All

73. Which of the following leaders was/were


part of the 1857 Sepoy Mutiny?
1. Gonoo
2. Shah Mal
3. Birjis Qadr
4. Kunwar Singh
5. Nana Sahib
Choose the correct answer using the codes below
a)
b)
c)
d)

INSIGHTS

1,2 and 3
4 and 5
1,2, 4 and 5
All

Page 15

74. With reference to Indian citizenship and


related laws, consider the following
statements
1. At present two year continuous stay
in India is mandatory for Indian
Citizenship
2. The Indian Citizenship Act, 1955
provides for acquisition, termination,
deprivation, determination of Indian
Citizenship and other related aspects
3. Under the Citizenship act, a minor,
whose parents are Indian Citizens can
register as Overseas Citizen of India
(OCI)
Which of the above statements is/are correct?
a)
b)
c)
d)

1 Only
1 and 2 Only
1 and 3 Only
2 and 3 Only

75. With reference to Japanese Encephalitis,


consider the following statements
1. It is transmitted to humans through
bites from infected mosquitoes of
the Culex species
2. Domestic pigs and wild birds (herons)
are reservoirs of the virus
3. There is no cure for the disease.
Treatment is focused on relieving
severe clinical signs and supporting
the patient to overcome the infection
Which of the above statements is/are correct?
a)
b)
c)
d)

All
2 and 3 Only
1 and 3 Only
1 and 2 Only

76. With reference to events that took place


after Gandhiji returned from South Africa,
consider the following statements
1. On Tilaks advice, Gandhiji spent a
year travelling around British India,
http://insightsonindia.com

getting to know the land and its


peoples
2. His first major public appearance was
at the opening of the Banaras Hindu
University (BHU) in February 1916
Which of the above statements is/are correct?
a)
b)
c)
d)

1 Only
None
Both
2 Only

77. Which of the following countries have


been severely affected socio-economically
due to the Ebola infection?
a) Liberia and Sierra Leone
b) Liberia and Nigeria
c) Sierra Leone and Chad
d) Liberia and Democratic Republic of
Congo

78. Recently Idukki district of Kerala became


the first in India to be connected with high
speed rural broadband. Idukki is also
famous for
a) Location of Anamudi, the highest peak
in India south of the Himalayas
b) Location Idukki Dam - Asia's largest
Arch Dam
c) Both a and b
d) Only b
79. With reference to Swami Vivekananda,
consider the following statements
1. He founded the Ramakrishna
Math and the Ramakrishna Mission
2. Every year in India, his birthday is
celebrated there as National Youth
Day
3. He contributed to the concept of
nationalism in colonial India
Which of the above statements is/are correct?
INSIGHTS

Page 16

a)
b)
c)
d)

1 and 2 Only
2 and 3 Only
1 and 3 Only
All

80. Union Ministry of Agricultures Bringing


Green Revolution to Eastern India
(BGREI) programme was not launched in
which of the following states?
a) Assam
b) Bihar
c) West Bengal
d) Arunachal Pradesh
81. Which of the following is NOT one of the
Millennium Development Goals (MDGs)?
1. To combat HIV/AIDS, malaria, and
other diseases
2. To develop a global partnership for
development
3. To ensure environmental
sustainability
4. To achieve universal healthcare for all
Choose the answer using the codes below
a)
b)
c)
d)

1 and 2 Only
2 Only
2 and 4 Only
4 Only

82. Consider the following statements


1. Jinnah called for a Direct Action Day
to press the Leagues demand for

a) Both
b) None
c) 2 Only
d) 1 Only
83. Which among the following statements is
NOT correct about the contemporary
world order?
1. There is an absence of world
government, which could regulate a
States
behaviour.
2. States, which violate international law,
are severely punished by the UN.
3. The UNSC operates on democratic
principles of equity and fairness.
Choose the correct answer using the codes below:
a)
b)
c)
d)

1 and 2 only
2 and 3 only
1 only
All of the above

84. Consider the following statements about


the European Union.
1. It has its own constitution and a
Parliament.
2. It is a free trade area.
3. Its share of world trade is larger than
that of the USA.
4. It has a common currency.
Choose the correct answer using the codes below:

Pakistan after Cabinet Mission

a)
b)
c)
d)

initiated dialogue between the


Congress and the League failed
2. In 1945, a Conservative government
came to power in Britain and

1 and 2 only
2, 3 and 4 only
2 and 4 only
3 and 4 only

committed itself to granting


independence to India
Which of the above statements is/are correct?

http://insightsonindia.com

INSIGHTS

Page 17

85. China followed its own path in


introducing a market economy and
modernization programs step by step after
the 1970s. Consider the following
statements about it.
1. It introduced Special Economic Zones
(SEZs) before India did.
2. It did not focus on agricultural
modernization.
3. It did not rely on foreign investments.
Choose the correct answer using the codes below:
a)
b)
c)
d)

1 and 2 only
2 and 3 only
1 only
1 and 3 only

86. Which of the following religious and


social movements in the Gangetic valley
form part of the Shraman tradition?
1. Jainism
2. Vaishnavism
3. Bhakti movement
4. Buddhism
Choose the correct answer using the codes given
below:
a)
b)
c)
d)

a)
b)
c)
d)

88. According to the narrative, there is a


dialogue between the Buddha and Mara,
Buddha is shown his right hand inclining
towards showing
a) Mara that his place belongs to the
earth and he must dissolve in it.
b) Earth as a witness to his generosity.
c) that the Earth will be his final place of
dissolution.
d) that he has conquered Mara
89. India has a rich tradition of Mural
paintings. Which of the following scenes
have been depicted in the mural
traditions?
1. Shiva chasing the boara scene from
Kiratarjuniya, Lepaksh temple
2. Shiva killing Tripuraasura, Thanjavoor
3. Chola king Rajaraja and court poet
Karuvar Dever, Thanjavoor
Choose the correct answer using the codes below.
a)
b)
c)
d)

2 and 3
1, 3 and 4
1 and 4
All of the above.

87. Which of the following were the salient


features of the pillars constructed during
the Mauryan era?
1. They were rock-cut pillars.
2. They carried social and religious
messages.
3. The top portion of the pillar was
carved with capital figures like Bull
and elephant.
Choose the correct answer using the codes given
below:
http://insightsonindia.com

1 and 2
2 and 3
1 and 3
All of the above.

1 and 2
2 and 3
1 and 3
All of the above.

90. Badami is a famous mural painting site in


the state of Karnataka. Consider the
following statements about it.
1. The excavation of the Badami caves
was patronized by Chalukyas.
2. The cave is popularly known as
Vishnu cave.
3. Indra and his retinue are also painted
in the cave.
Choose the correct answer using the codes
below.
INSIGHTS

Page 18

a)
b)
c)
d)

1 and 2
2 and 3
1 and 3
All of the above

91. Chola paintings can NOT be found at


which of these places?
1. Gangaikonda temple
2. Thiruanathpuram
3. Brihdeshwara temple
4. Nartamalai
Choose the correct answer using the codes below:
a)
b)
c)
d)

Only 2
1 and 4
1 and 3
2 and 4

92. Kerala painters (during the period from


the sixteenth to the eighteenth century)
evolved a pictorial language and
technique of their own. Consider the
following about Kerala murals.
1. They adopted certain stylistic elements
from Nayaka and Vijayanagara
schools.
2. Paintings were also shown in threedimensionality.
3. Most of the narrations in them are
based on those episodes from Hindu
mythology which were popular in
Kerala.
Choose the correct answer using the codes below:
a) 1 and 2
b) 2 and 3
c) 1 and 3
d) All of the above
93. Which of the following factors could have
been responsible for the India-China
conflict of 1962?
1. Chinese takeover of Tibet in 1950
2. Competing territorial claims in the
Ladakh region
http://insightsonindia.com

3. Chinese support to Pakistani army


Choose the correct answer using the codes below:
a)
b)
c)
d)

1 and 2 only
2 and 3 only
1 and 3 only
All of the above

94. Governors rule has been imposed in the


state of Jammu and Kashmir (J&K)
recently. Consider the following
statements about Governors rule.
1. It is applicable as per the Indian
constitution.
2. It can be imposed only after the
consent of the President.
3. This is the first time Governors rule
has been imposed in J&K.
Choose the correct answer using the codes below:
a)
b)
c)
d)

1 and 2 only
2 only
1 and 3 only
All of the above

95. As per the recent amendment in the


Citizenship Act, 1955 the PIOs will get
which of the following benefits?
1. Exemption from appearing before the
local police station after every visit
2. Life-long Indian visa facility
3. Acquire property in India with more
flexibility
Choose the correct answer using the codes below:
a)
b)
c)
d)

INSIGHTS

1 and 2 only
3 only
1 and 3 only
All of the above

Page 19

96. The recently launched ICICI Banks


Digital Village Project involves
1. Digitised school teaching
2. Cashless banking for the village
3. E-health projects for the village

3. Allow government to reduce its stake


in Coal PSUs lower than the statutory
mandates
Choose the correct answer using the codes below:
a)
b)
c)
d)

Choose the correct answer using the codes below:


a)
b)
c)
d)

1 and 2 only
2 and 3 only
1 and 3 only
All of the above

97. Countries across the world are gradually


beginning to accept styrofoam (the
material most widely used to make
disposable cups and plates) as a serious
hazard to the environment. Why?
1. Hot foods and liquids actually start a
partial breakdown of the Styrofoam
becoming a toxin
2. It is non bio-degradable
3. It has been found to cause cancer in
animals

1 and 2 only
2 and 3 only
1 and 3 only
All of the above

99. ASEAN is seen as a major balancer in the


power relations in the World. The
ASEAN Way is called as an important
feature of ASEAN. It is?
a) The collective defence policy followed
by the ASEAN members
b) Reflects the life style and culture of the
ASEAN members
c) A form of interaction among ASEAN
members which is informal and
cooperative
d) The economic corridors being
developed by ASEAN

Choose the correct answer using the codes below:


a)
b)
c)
d)

1 and 2 only
2 and 3 only
1 and 3 only
All of the above

98. The government has recently cleared an


ordinance for amendments to the Mines
and Minerals Act. This will
1. Allow MNCs to directly bid for
mineral resources in India
2. Provide for a District Mineral Fund to
be used for rehabilitation

http://insightsonindia.com

100.
Which of the following countries
have experienced both civilian and
military rulers despite being a democracy
presently?
1. Pakistan
2. Nepal
3. Bangladesh
4. Sri Lanka
Choose the correct answer using the codes below:
a)
b)
c)
d)

INSIGHTS

2 and 4 only
1 and 3 only
1, 3 and 4 only
All of the above

Page 20

http://insightsonindia.com

INSIGHTS

Page 21

Insights Mock Tests 2015: Test 12 Solutions


1. Solution: b)
Prehistoric paintings have been found in many parts of the world. We do not really know if
Lower Palaeolithic people ever produced any art objects. But by the Upper Palaeolithic times
we see a proliferation of artistic activities. Around the world the walls of many caves of this
time are full of finely carved and painted pictures of animals which the cave-dwellers
hunted. The subjects of their drawings were human figures, human activities, geometric
designs and symbols. In India the earliest paintings have been reported from the Upper
Palaeolithic times.

2. Solution: b)
Remnants of rock paintings have been found on the walls of the caves situated in several
districts of Madhya Pradesh, Uttar Pradesh, Andhra Pradesh, Karnataka and Bihar. Some
paintings have been reported from the Kumaon hills in Uttarakhand also. The rock shelters
on banks of the River Suyal at Lakhudiyar, about twenty kilometres on the Almora
Barechina road, bear these prehistoric paintings.
Lakhudiyar literally means one lakh caves. The paintings here can be divided into three
categories: man, animal and geometric patterns in white, black and red ochre. Humans are
represented in stick-like forms. A long-snouted animal, a fox and a multiple legged lizard
are the main animal motifs. Wavy lines, rectangle-filled geometric designs, and groups of
dots can also be seen here. One of the interesting scenes depicted here is of hand-linked
dancing human figures.
THE ABOVE MAKES IT CLEAR that option D is not appropriate. For the other options
here is the argument and evidence.
There is some superimposition of paintings. The earliest are in black; over these are red
ochre paintings and the last group comprises white paintings. From Kashmir two slabs with
engravings have been reported. The granite rocks of Karnataka and Andhra Pradesh
provided suitable canvases to the Neolithic man for his paintings. There are several such
sites but more famous among them are Kupgallu, Piklihal and Tekkalkota. Three types of
paintings have been reported from herepaintings in white, paintings in red ochre over a
white background and paintings in red ochre.

3. Solution: c)
Richest paintings are reported from the Vindhya ranges of Madhya Pradesh and their
Kaimurean extensions into Uttar Pradesh. These hill ranges are full of Palaeolithic and
Mesolithic remains, and they are also full of forests, wild plants, fruits, streams and creeks,
thus a perfect place for Stone Age people to live. Among these the largest and most
spectacular rock-shelter is located in the Vindhya hills at Bhimbetka in Madhya Pradesh.

http://www.insightsonindia.com

INSIGHTS

Page 1

Insights Mock Tests 2015: Test 12 Solutions


Bhimbetka is located fortyfive kilometres south of Bhopal, in an area of ten square
kilometres, having about eight hundred rock shelters, five hundred of which bear paintings.
The caves of Bhimbetka were discovered in 195758 by eminent archaeologist V.S.
Wakankar and later on many more were discovered. Wakankar spent several years in
surveying these inaccessible hills and jungles to study these paintings.
The themes of paintings found here are of great variety, ranging from mundane events of
daily life in those times to sacred and royal images. These include hunting, dancing, music,
horse and elephant riders, animal fighting, honey collection, decoration of bodies, and other
household scenes.

4. Solution: c)

To be noted, it is not a UN body. It exists separately. The same is true for WTO, WB etc.
Also, only the WB, not the IMF, gives infrastructural loans.

5. Solution: c)
Nations can usually see that there are some things they must do together. There are issues
that are so challenging that they can only be dealt with when everyone works together.
Disease is an example. Some diseases can only be eradicated if everyone in the world
cooperates in inoculating or vaccinating their populations. Or take global warming and its
effects. As atmospheric temperatures rise because of the spread of certain chemicals called
chlorofluorocarbons (CFCs), there is a danger that sea levels will also rise, thereby
submerging many coastal areas of the world including huge cities. Of course, each country
can try to find its own solution to the effects of global warming. But in the end a more
effective approach is to stop the warming itself. This requires at least all of the major
industrial powers to cooperate.
http://www.insightsonindia.com

INSIGHTS

Page 2

Insights Mock Tests 2015: Test 12 Solutions

6. Solution: c)
Explanation for this and the next three questions are together.

7. Solution: d)
Explanation above.

8. Solution: c)
Explanation above.
http://www.insightsonindia.com

INSIGHTS

Page 3

Insights Mock Tests 2015: Test 12 Solutions

9. Solution: d)
Explanation above.

10. Solution: d)
After the Cold War, the realities of the globe are different that force a revisit of the
composition of the UNSC. Here are some of the changes that have occurred that revisit:

The Soviet Union has collapsed.


The US is the strongest power.
China is fast emerging as a great power, and India also is growing rapidly.
The economies of Asia are growing at an unprecedented rate.
Many new countries have joined the UN (as they became independent from the
Soviet Union or former communist states in eastern Europe).
A whole new set of challenges confronts the world (genocide, civil war, ethnic
conflict, terrorism, nuclear proliferation, climate change, environmental degradation,
epidemics).

11. Solution: d)
The World Bank was created immediately after the Second World War in 1945. Its activities
are focused on the developing countries. It works for human development (education,
health), agriculture and rural development (irrigation, rural services), environmental
protection (pollution reduction, establishing and enforcing regulations), infrastructure
(roads, urban regeneration, electricity) and governance (anti-corruption, development of
legal institutions). It provides loans and grants to the member-countries. In this way, it
exercises enormous influence on the economic policies of developing countries. It is often
criticised for setting the economic agenda of the poorer nations, attaching stringent
conditions to its loans and forcing free market reforms.

12. Solution: c)
This organisation was set up in 1995 as the successor to the General Agreement on Trade
and Tariffs (GATT) created after the Second World War. It has 150 members. All decisions
are taken unanimously but the major economic powers such as the US, EU and Japan have
managed to use the WTO to frame rules of trade to advance their own interests. The
developing countries often complain of non-transparent procedures and being pushed
around by big powers.

http://www.insightsonindia.com

INSIGHTS

Page 4

Insights Mock Tests 2015: Test 12 Solutions


13. Solution: b)
While Harappa and Mohenjodaro are situated in Pakistan, the important sites excavated in
India are Lothal and Dholavira in Gujarat, Rakhigarhi in Haryana, Ropar in the Punjab,
Kalibangan and Balathal in Rajasthan, etc. Statues whether in stone, bronze or terracotta
found in Harappan sites are not abundant, but refined.

14. Solution: b)
The art of bronze-casting was practised on a wide scale by the Harappans. Their bronze
statues were made using the lost wax technique in which the wax figures were first
covered with a coating of clay and allowed to dry. Then the wax was heated and the molten
wax was drained out through a tiny hole made in the clay cover. The hollow mould thus
created was filled with molten metal which took the original shape of the object. Once the
metal cooled, the clay cover was completely removed.

15. Solution: a)
Archaeologists have discovered thousands of seals, usually made of steatite, and
occasionally of agate, chert, copper, faience and terracotta, with beautiful figures of animals,
such as unicorn bull, rhinoceros, tiger, elephant, bison, goat, buffalo, etc. The realistic
rendering of these animals in various moods is remarkable. The purpose of producing seals
was mainly commercial. It appears that the seals were also used as amulets, carried on the
persons of their owners, perhaps as modern-day identity cards. The standard Harappan seal
was a square plaque 22 square inches, usually made from the soft river stone, steatite.
Every seal is engraved in a pictographic script which is yet to be deciphered. Some seals
have also been found in gold and ivory. They all bear a great variety of motifs, most often of
animals including those of the bull, Unicorn, Tiger etc.

16. Solution: d)
The Harappan men and women decorated themselves with a large variety of ornaments
produced from every conceivable material ranging from precious metals and gemstones to
bone and baked clay. While necklaces, fillets, armlets and finger-rings were commonly worn
by both sexes.
From archaeological finds it appears that the people of the Indus Valley were conscious of
fashion. Different hairstyles were in vogue and wearing of a beard was popular among all.
Cinnabar was used as a cosmetic and facepaint, lipstick and collyrium (eyeliner) were also
known to them. Many stone structural remains are also found at Dholavira which show how
the Indus Valley people used stone in construction.
http://www.insightsonindia.com

INSIGHTS

Page 5

Insights Mock Tests 2015: Test 12 Solutions

17. Solution: c)
Even today we observe that mural painting on interior and exterior walls of houses in
villages or havelis is prevalent in different parts of the country. These paintings are usually
made by women either at the time of ceremonies or festivals or as a routine to clean and
decorate the walls. Some of the traditional forms of murals are pithoro in parts of Rajasthan
and Gujarat, Mithila painting in northern Bihars Mithila region, warli paintings in
Maharashtra, or simply paintings on the walls, be it in a village of Odisha or Bengal,
Madhya Pradesh or Chhattisgarh.

18. Solution: d)
Two broad orders of temples in the country are known Nagara in the north and Dravida in
the south. At times, the Vesar style of temples as an independent style created through the
selective mixing of the Nagara and Dravida orders is mentioned by some scholars.
As temples grew more complex, more surfaces were created for sculpture through additive
geometry, i.e., by adding more and more rhythmically projecting, symmetrical walls and
niches, without breaking away from the fundamental plan of the shrine.
Phamsana and Valabhi are sub-types of Nagara.

19. Solution: c)

This temple is in the panchayatana style of architecture where the main shrine is built on a
rectangular plinth with four smaller subsidiary shrines at the four corners (making it a total
number of five shrines, hence the name, panchayatana). The tall and curvilinear shikhara also
corroborates this date.
Rekha-prasada type of shikhara is the presence of this curving latina in an early example of
a classic nagara style of temple.

20. Solution: b)
http://www.insightsonindia.com

INSIGHTS

Page 6

Insights Mock Tests 2015: Test 12 Solutions


The main architectural features of Odisha temples are classified in three orders, i.e.,
rekhapida, pidhadeul and khakra.
Most of the main temple sites are located in ancient Kalingamodern Puri District,
including Bhubaneswar or ancient Tribhuvanesvara, Puri and Konark. The temples of
Odisha constitute a distinct substyle within the nagara order. In general, here the shikhara,
called deul in Odisha, is vertical almost until the top when it suddenly curves sharply
inwards. Deuls are preceded, as usual, by mandapas called jagamohana in Odisha. Theground
plan of the main temple is almost always square, which, in the upper reaches of its
superstructure becomes circular in the crowning mastaka.
This makes the spire nearly cylindrical in appearance in its length. Compartments and
niches are generally square, the exterior of the temples are lavishly carved, their interiors
generally quite bare. Odisha temples usually have boundary walls.

21. Solution: d)
Kashmirs proximity to prominent Gandhara sites (such as Taxila, Peshawar and the
northwest frontier) lent the region a strong Gandhara influence by the fifth century CE. This
began to mix with the Gupta and post-Gupta traditions that were brought to it from Sarnath,
Mathura and even centres in Gujarat and Bengal.
Brahmin pundits and Buddhist monks frequently travelled between Kashmir, Garhwal,
Kumaon and religious centres in the plains like Banaras, Nalanda and even as far south as
Kanchipuram. As a result both Buddhist and Hindu traditions began to intermingle and
spread in the hills. The hills also had their own tradition of wooden buildings with pitched
roofs.
At several places in the hills, therefore, you will find that while the main garbhagriha and
shikhara are made in a rekha-prasada or latina style, the mandapa is of an older form of wooden
architecture. Sometimes, the temple itself takes on a pagoda shape.

22. Solution: b)
In 1961, the leaders of the Union of Soviet Socialist Republics (USSR) were worried that the
United States of America (USA) would invade communist-ruled Cuba and overthrow Fidel
Castro, the president of the small island nation off the coast of the United States. Cuba was
an ally of the Soviet Union and received both diplomatic and financial aid from it. Nikita
Khrushchev, the leader of the Soviet Union, decided to convert Cuba into a Russian base.
In 1962, he placed nuclear missiles in Cuba. The installation of these weapons put the US, for
the first time, under fire from close range and nearly doubled the number of bases or cities
in the American mainland which could be threatened by the USSR.

http://www.insightsonindia.com

INSIGHTS

Page 7

Insights Mock Tests 2015: Test 12 Solutions


Three weeks after the Soviet Union had placed the nuclear weapons in Cuba, the Americans
became aware of it.
The US President, John F. Kennedy, and his advisers were reluctant to do anything that
might lead to full-scale nuclear war between the two countries, but they were determined to
get Khrushchev to remove the missiles and nuclear weapons from Cuba. Kennedy ordered
American warships to intercept any Soviet ships heading to Cuba as a way of warning the
USSR of his seriousness. A clash seemed imminent in what came to be known as the Cuban
Missile Crisis.

23. Solution: d)
The Cold War was not simply a matter of power rivalries, of military alliances, and of the
balance of power.
These were accompanied by a real ideological conflict as well, a difference over the best and
the most appropriate way of organising political, economic, and social life all over the
world.
The western alliance, headed by the US, represented the ideology of liberal democracy and
capitalism while the eastern alliance, headed by the Soviet Union, was committed to the
ideology of socialism and communism.
The two superpowers and the countries in the rival blocs led by the superpowers were
expected to behave as rational and responsible actors. They were to be rational and
responsible in the sense that they understood the risks in fighting wars that might involve
the two superpowers. When two superpowers and the blocs led by them are in a deterrence
relationship, fighting wars will be massively destructive.

24. Solution: a)
The two superpowers were keen on expanding their spheres of influence in different parts
of the world. In a world sharply divided between the two alliance systems, a state was
supposed to remain tied to its protective superpower to limit the influence of the other
superpower and its allies.
The smaller states in the alliances used the link to the superpowers for their own purposes.
They got the promise of protection, weapons, and economic aid against their local rivals,
mostly regional neighbours with whom they had rivalries. The alliance systems led by the
two superpowers, therefore, threatened to divide the entire world into two camps. This
division happened first in Europe.
Most countries of western Europe sided with the US and those of eastern Europe joined the
Soviet camp. That is why these were also called the western and the eastern alliances.
http://www.insightsonindia.com

INSIGHTS

Page 8

Insights Mock Tests 2015: Test 12 Solutions

25. Solution: d)
Why the superpowers needed any allies at all? After all, with their nuclear weapons and
regular armies, they were so powerful that the combined power of most of the smaller states
in Asia and Africa, and even in Europe, was no match to that of the superpowers. Yet, the
smaller states were helpful for the superpowers in gaining access to
(i) vital resources, such as oil and minerals,
(ii) territory, from where the superpowers could launch their weapons and troops,
(iii) locations from where they could spy on each other, and
(iv) economic support, in that many small allies together could help pay for military
expenses.
They were also important for ideological reasons. The loyalty of allies suggested that the
superpowers were winning the war of ideas as well.

26. Solution: d)

27. Solution: a)
http://www.insightsonindia.com

INSIGHTS

Page 9

Insights Mock Tests 2015: Test 12 Solutions


The roots of NAM went back to the friendship between three leaders Yugoslavias Josip
Broz Tito, Indias Jawaharlal Nehru, and Egypts leader Gamal Abdel Nasser who held a
meeting in 1956. Indonesias Sukarno and Ghanas Kwame Nkrumah strongly supported
them. These five leaders came to be known as the five founders of NAM. The first nonaligned summit was held in Belgrade in 1961. This was the culmination of at least three
factors:
(i) cooperation among these five countries,
(ii) growing Cold War tensions and its widening arenas, and
(iii) the dramatic entry of many newly decolonised African countries into the international
arena. By 1960, there were 16 new African members in the UN.
The first summit was attended by 25 member states. Over the years, the membership of
NAM has expanded.

28. Solution: c)
As non-alignment grew into a popular international movement, countries of various
different political systems and interests joined it. This made the movement less
homogeneous and also made it more difficult to define in very neat and precise terms: what
did it really stand for? Increasingly, NAM was easier to define in terms of what it was not. It
was not about being a member of an alliance. The policy of staying away from alliances
should not be considered isolationism or neutrality.
Non-alignment is not isolationism since isolationism means remaining aloof from world
affairs. Isolationism sums up the foreign policy of the US from the American War of
Independence in 1787 up to the beginning of the First World War. In comparison, the nonaligned countries, including India, played an active role in mediating between the two rival
alliances in the cause of peace and stability. Their strength was based on their unity and
their resolve to remain non-aligned despite the attempt by the two superpowers to bring
them into their alliances.
Non-alignment is also not neutrality. Neutrality refers principally to a policy of staying out
of war. States practising neutrality are not required to help end a war. They do not get
involved in wars and do not take any position on the appropriateness or morality of a war.
Non-aligned states, including India, were actually involved in wars for various reasons.
They also worked to prevent war between others and tried to end wars that had broken out.

29. Solution: d)
Economic development was also vital for the independence of the new countries.

http://www.insightsonindia.com

INSIGHTS

Page 10

Insights Mock Tests 2015: Test 12 Solutions


Without sustained development, a country could not be truly free. It would remain
dependent on the richer countries including the colonial powers from which political
freedom had been achieved. The idea of a New International Economic Order (NIEO)
originated with this realisation. The United Nations Conference on Trade and Development
(UNCTAD) brought out a report in 1972 entitled Towards a New Trade Policy for Development.
The report proposed a reform of the global trading system so as to:
(i) give the LDCs control over their natural resources exploited by the developed Western
countries,
(ii) obtain access to Western markets so that the LDCs could sell their products and,
therefore, make trade more beneficial for the poorer countries,
(iii) reduce the cost of technology from the Western countries, and
(iv) provide the LDCs with a greater role in international economic institutions.
Gradually, the nature of nonalignment changed to give greater importance to economic
issues.

30. Solution: c)
Indias policy of non-alignment was criticised on a number of counts. Here we may refer to
only two criticisms:
First, Indias non-alignment was said to be unprincipled. In the name of pursuing its
national interest, India, it was said, often refused to take a firm stand on crucial international
issues.
Second, it is suggested that India was inconsistent and took contradictory postures.
Having criticised others for joining alliances, India signed the Treaty of Friendship in
August 1971 with the USSR for 20 years. This was regarded, particularly by outside
observers, as virtually joining the Soviet alliance system. The Indian governments view was
that India needed diplomatic and possibly military support during the Bangladesh crisis and
that in any case the treaty did not stop India from having good relations with other countries
including the US.

31. Solution: d)
LIMITED TEST BAN TREATY (LTBT)

http://www.insightsonindia.com

INSIGHTS

Page 11

Insights Mock Tests 2015: Test 12 Solutions


Banned nuclear weapon tests in the atmosphere, in outer space and under water. Signed by
the US, UK and USSR in Moscow on 5 August 1963. Entered into force on 10 October 1963.
NUCLEAR NON-PROLIFERATION TREATY (NPT)
Allows only the nuclear weapon states to have nuclear weapons and stops others from
aquiring them. For the purposes of the NPT, a nuclear weapon state is one which has
manufactured and exploded a nuclear weapon or other nuclear explosive device prior to 1
January 1967. So there are five nuclear weapon states: US, USSR (later Russia), Britain,
France and China. Signed in Washington, London, and Moscow on 1 July 1968. Entered into
force on 5 March 1970. Extended indefinitely in 1995.
STRATEGIC ARMS REDUCTION TREATY I (START-I)
Treaty signed by the USSR President Mikhail Gorbachev and the US President George Bush
(Senior) on the reduction and limitation of strategic offensive arms in Moscow on 31 July
1991.
STRATEGIC ARMS REDUCTION TREATY II (START-II)
Treaty signed by the Russian President Boris Yeltsin and the US President George Bush
(Senior) on the reduction and limitation of strategic offensive arms in Moscow on 3 January
1993.

32. Solutions: d)
There was no unemployment. State ownership was the dominant form of ownership: land
and productive assets were owned and controlled by the Soviet state.
The Soviet system, however, became very bureaucratic and authoritarian, making life very
difficult for its citizens. Lack of democracy and the absence of freedom of speech stifled
people who often expressed their dissent in jokes and cartoons. Most of the institutions of
the Soviet state needed reform: the one-party system represented by the Communist Party of
the Soviet Union had tight control over all institutions and was unaccountable to the people.
The party refused to recognise the urge of people in the fifteen different republics that
formed the Soviet Union to manage their own affairs including their cultural affairs.
Although, on paper, Russia was only one of the fifteen republics that together constituted
the USSR, in reality Russia dominated everything, and people from other regions felt
neglected and often suppressed.
In the arms race, the Soviet Union managed to match the US from time to time, but at great
cost. The Soviet Union lagged behind the West in technology, infrastructure (e.g. transport,
power), and most importantly, in fulfilling the political or economic aspirations of citizens.
The Soviet invasion of Afghanistan in 1979 weakened the system even further. Though
wages continued to grow, productivity and technology fell considerably behind that of the

http://www.insightsonindia.com

INSIGHTS

Page 12

Insights Mock Tests 2015: Test 12 Solutions


West. This led to shortages in all consumer goods. Food imports increased every year. The
Soviet economy was faltering in the late 1970s and became stagnant.

33. Solution: d)
The disintegration of Soviet Union started under the leadership of Micheal Gorbachev. A
discussion of his policies and their shortcomings would lead us to the break-up of the soviet
Union.
Gorbachev came to power in 1985. He was determined to transform and revitalize the Union
and for this purpose intended the modernizing and streamlining the communist party.
Gorbachev wanted to make communism practiced in Soviet more humane and democratic.

Reforms by Gorbachev perestroika (economic restructuring) and Glasnost (openness):


1. He made foreign affairs a priority, took initiatives for better relations with china; and
pushed Soviet withdrawal from Afghanistan.
2. Freedom was given to media to criticize the government.
3. For the first time, parties other than the Communist Party could participate in
elections.
4. In economic affairs government grip was loosened. Small scale private entrepreneurs
were allowed. Computerization was started which reduced the manual labor.
5. Workers were given the right to strike for better wages and conditions.
6. Gorbachev also encouraged foreign investment in Soviet enterprises.
7. Political changes were initiated that led to a move towards democracy within the
Communist party.

34. Solution: d)
The collapse of the second world of the Soviet Union and the socialist systems in eastern
Europe had profound consequences for world politics.
First of all, it meant the end of Cold War confrontations. The ideological dispute over
whether the socialist system would beat the capitalist system was not an issue any more.
Since this dispute had engaged the military of the two blocs, had triggered a massive arms
race and accumulation of nuclear weapons, and had led to the existence of military blocs, the
end of the confrontation demanded an end to this arms race and a possible new peace.

http://www.insightsonindia.com

INSIGHTS

Page 13

Insights Mock Tests 2015: Test 12 Solutions


Second, power relations in world politics changed and, therefore, the relative influence of
ideas and institutions also changed. The end of the Cold War left open only two possibilities:
either the remaining superpower would dominate and create a unipolar system, or different
countries or groups of countries could become important players in the international system,
thereby bringing in a multipolar system where no one power could dominate. As it turned
out, the US became the sole superpower.
Third, the end of the Soviet bloc meant the emergence of many new countries. All these
countries had their own independent aspirations and choices. Some of them, especially the
Baltic and east European states, wanted to join the European Union and become part of the
North Atlantic Treaty Organisation (NATO). Thus, the international system saw many new
players emerge, each with its own identity, interests, and economic and political difficulties

35. Solution: d)
The collapse of communism was followed in most of these countries by a painful process of
transition from an authoritarian socialist system to a democratic capitalist system. The
model of transition in Russia, Central Asia and east Europe that was influenced by the
World Bank and the IMF came to be known as shock therapy.
Each of these countries was required to make a total shift to a capitalist economy, which
meant rooting out completely any structures evolved during the Soviet period. Above all, it
meant that private ownership was to be the dominant pattern of ownership of property.
Privatisation of state assets and corporate ownership patterns were to be immediately
brought in. Collective farms were to be replaced by private farming and capitalism in
agriculture.
Shock therapy also involved a drastic change in the external orientation of these economies.
Development was now envisaged through more trade, and thus a sudden and complete
switch to free trade was considered essential. \
The free trade regime and foreign direct investment (FDI) were to be the main engines of
change. This also involved openness to foreign investment, financial opening up or
deregulation, and currency convertibility.
Finally, the transition also involved a break up of the existing trade alliances among the
countries of the Soviet bloc.

36. Solution: c)
On 19 March 2003, the US launched its invasion of Iraq under the codename Operation Iraqi
Freedom. More than forty other countries joined in the US-led coalition of the willing after
the UN refused to give its mandate to the invasion. The ostensible purpose of the invasion
http://www.insightsonindia.com

INSIGHTS

Page 14

Insights Mock Tests 2015: Test 12 Solutions


was to prevent Iraq from developing weapons of mass destruction (WMD). Since no
evidence of WMD has been unearthed in Iraq, it is speculated that the invasion was
motivated by other objectives, such as controlling Iraqi oilfields and installing a regime
friendly to the US.
Although the government of Saddam Hussein fell swiftly, the US has not been able to
pacify Iraq. Instead, a full-fledged insurgency against US occupation was ignited in Iraq.

37. Solution: d)
The first constraint is the institutional architecture of the American state itself. A system of
division of powers between the three branches of government places significant brakes upon
the unrestrained and immoderate exercise of Americas military power by the executive
branch.
The second constraint on American power is also domestic in nature, and stems from the
open nature of American society.
Although the American mass media may from time to time impose or promote a particular
perspective on domestic public opinion in the US, there is nevertheless a deep scepticism
regarding the purposes and methods of government in American political culture. This
factor, in the long run, is a huge constraint on US military action overseas.
However, it is the third constraint on the US that is perhaps the most important.
There is only one organisation in the international system that could possibly moderate the
exercise of American power today, and that is the North Atlantic Treaty Organisation
(NATO). The US obviously has an enormous interest in keeping the alliance of democracies
that follow the market economies alive and therefore it is possible that its allies in the NATO
will be able to moderate the exercise of US hegemony.

38. Solution: d)
The Sri Lankan problem involves people of Indian origin, and there is considerable pressure
from the Tamil people in India to the effect that the Indian government should protect the
interests of the Tamils in Sri Lanka. The government of India has from time to time tried to
negotiate with the Sri Lankan government on the Tamil question.
But in 1987, the government of India for the first time got directly involved in the Sri Lankan
Tamil question. India signed an accord with Sri Lanka and sent troops to stabilise relations
between the Sri Lankan government and the Tamils. Eventually, the Indian Army got into a
fight with the LTTE. The presence of Indian troops was also not liked much by the Sri
Lankans. They saw this as an attempt by India to interfere in the internal affairs of Sri Lanka.

http://www.insightsonindia.com

INSIGHTS

Page 15

Insights Mock Tests 2015: Test 12 Solutions


In 1989, the Indian Peace Keeping Force (IPKF) pulled out of Sri Lanka without attaining its
objective.

39. Solution: c)
Soon after the partition, the two countries got embroiled in a conflict over the fate of
Kashmir.
The Pakistani government claimed that Kashmir belonged to it. Wars between India and
Pakistan in 1947-48 and 1965 failed to settle the matter. The 1947-48 war resulted in the
division of the province into Pakistan-occupied Kashmir and the Indian province of Jammu
and Kashmir divided by the Line of Control. In 1971, India won a decisive war against
Pakistan but the Kashmir issue remained unsettled.
Indias conflict with Pakistan is also over strategic issues like the control of the Siachen
glacier and over acquisition of arms. The arms race between the two countries assumed a
new character with both states acquiring nuclear weapons and missiles to deliver such arms
against each other in the 1990s. In 1998, India conducted nuclear explosion in Pokaran.
Pakistan responded within a few days by carrying out nuclear tests in the Chagai Hills.
Since then India and Pakistan seem to have built a military relationship in which the
possibility of a direct and full-scale war has declined.

40. Solution: a)
India and Pakistan also have had problems over the sharing of river waters. Until 1960, they
were locked in a fierce argument over the use of the rivers of the Indus basin. Eventually, in
1960, with the help of the World Bank, India and Pakistan signed the Indus Waters Treaty
which has survived to this day in spite of various military conflicts in which the two
countries have been involved.
There are still some minor differences about the interpretation of the Indus Waters Treaty
and the use of the river waters. The two countries are not in agreement over the demarcation
line in Sir Creek in the Rann of Kutch. The dispute seems minor, but there is an underlying
worry that how the dispute is settled may have an impact on the control of sea resources in
the area adjoining Sir Creek. India and Pakistan are holding negotiations on all these issues.

41. Solution: d)
In 1950, Nepal signed India-Nepal Friendship treaty this treaty, in order to maintain the
special ties with independent India that it had with British India. Another reason was that
Nepals security concerns had been heightened by the Communist revolution in China and
its takeover of Tibet.
http://www.insightsonindia.com

INSIGHTS

Page 16

Insights Mock Tests 2015: Test 12 Solutions


The treaty provides for:

an open border between the two countries

allows Nepali nationals to work in India without a work permit, to apply for
government jobs and the civil services (except for the IFS, IAS, and IPS), to open
bank accounts and buy property.

42. Solution: d)
The rock-cut cave carved at Barabar hills near Gaya in Bihar is known as the Lomus Rishi
cave. The facade of the cave is decorated with the semicircular chaitya arch as the entrance.
The elephant frieze carved in high relief on the chaitya arch shows considerable movement.
The interior hall of this cave is rectangular with a circular chamber at the back. The entrance
is located on the side wall of the hall. The cave was patronised by Ashoka for the Ajivika
sect. The Lomus Rishi cave is an isolated example of this period. But many Buddhist caves of
the subsequent periods were excavated in eastern and western India.

43. Solution: c)
The Lion Capital discovered more than a hundred years ago at Sarnath, near Varanasi, is
generally referred to as Sarnath Lion Capital. This is one of the finest examples of sculpture
from the Mauryan period. Built in commemoration of the historical event of the first sermon
or the Dhammachakrapravartana by the Buddha at Sarnath, the capital was built by Ashoka.
The motif of the chakra becomes significant as a representation of the Dhammachkra in the
entire Buddhist art.

http://www.insightsonindia.com

INSIGHTS

Page 17

Insights Mock Tests 2015: Test 12 Solutions


Each animal figure, despite sticking to the surface, is voluminous, its posture creating
movement in the circular abacus. Despite having limited space between each chakra, these
animal figures display considerable command over the depiction of movement in a limited
space. The circular abacus is supported by an inverted lotus capital.

44. Solution: a)
During the early phase of Buddhism, Buddha is depicted symbolically through footprints,
stupas, lotus throne, chakra, etc. This indicates either simple worship, or paying respect, or at
times depicts historicization of life events. This was because Buddha said Digha Nikaya i.e.
he discouraged his representation in human forms after his death.
Gradually narrative became a part of the Buddhist tradition. Thus events from the life of the
Buddha, the Jataka stories, were depicted on the railings and torans of the stupas. Mainly
synoptic narrative, continuous narrative and episodic narrative are used in the pictorial
tradition.
While events from the life of the Buddha became an important theme in all the Buddhist
monuments, the Jataka stories also became equally important for sculptural decorations. The
main events associated with the Buddhas life which were frequently depicted were events
related to the birth, renunciation, enlightenment, dhammachakrapravartana, and
mahaparinibbana (death). Among the Jataka stories that are frequently depicted are Chhadanta
Jataka, Vidurpundita Jataka, Ruru Jataka, Sibi Jataka, Vessantara Jataka and Shama Jataka.
Stupa worship, Bharhut.

45. Solution: b)
After the developments at Bantur, the next phase of sculptural development at Sanchi Stupa1, Mathura, and Vengi in Andhra Pradesh (Guntur District) is noteworthy in the stylistic
progression. Stupa-1 at Sanchi has upper as well as lower pradakshinapatha or
circumambulatory path. It has four beautifully decorated toranas depicting various events
from the life of the Buddha and the Jatakas. Figure compositions are in high relief, filling up
the entire space.

46. Solution: a)
The first century CE onwards, Gandhara (now in Pakistan), Mathura in northern India and
Vengi in Andhra Pradesh emerged as important centres of art production. Buddha in the
symbolic form got a human form in Mathura and Gandhara. The sculptural tradition in
Gandhara had the confluence of Bactria, Parthia and the local Gandhara tradition. The local
sculptural tradition at Mathura became so strong that the tradition spread to other parts of

http://www.insightsonindia.com

INSIGHTS

Page 18

Insights Mock Tests 2015: Test 12 Solutions


northern India. The best example in this regard is the stupa sculptures found at Sanghol in
the Punjab.

47. Solution: a)
The sculptural tradition in Gandhara had the confluence of Bactria, Parthia and the local
Gandhara tradition.
The Buddha image at Mathura is modelled on the lines of earlier Yaksha images whereas in
Gandhara it has Hellenistic features. Images of Vaishnava (mainly Vishnu and his various
forms) and Shaiva (mainly the lingas and mukhalingas) faiths are also found at Mathura but
Buddhist images are found in large numbers. It may be noted that the images of Vishnu and
Shiva are represented by their ayudhas (weapons).

48. Solution: d)
Buddhist Monuments of South India - Vengi in Andhra Pradesh has many stupa sites like
Jagayyapetta, Amaravati, Bhattiprolu, Nagarjunkonda, Goli, etc. Amaravati has a
mahachaitya and had many sculptures which are now preserved in Chennai Museum,
Chatur Mukhlinga, Amaravati Site Museum, National Museum, New Delhi and the British
Museum, London. Like the Sanchi Stupa, the Amaravati Stupa also has pradakshinapatha
enclosed within a vedika on which many narrative sculptures are depicted.

49. Solution: b)
Along with the images of the Buddha, other Buddhist images of Boddhisattvas like
Avalokiteshvara, Padmapani, Vajrapani, Amitabha, and Maitreya Buddha started getting
sculpted.
However, with the rise of Vajrayana Buddhism many Boddhisattva images were added as a
part of the personified representations of certain virtues or qualities as propagated by the
Buddhist religious principles for the welfare of the masses.

50. Solution: d)
In western India, many Buddhist caves dating back to the second century BCE onwards
have been excavated. Mainly three architectural types were executed(i) apsidal vaultro of
chaitya halls (found at Ajanta, Pitalkhora, Bhaja); (ii) apsidal vault-roof pillarless hall (found
at Thana-Nadsur); and (iii) flat-roofed quadrangular hall with a circular chamber at the
back.

http://www.insightsonindia.com

INSIGHTS

Page 19

Insights Mock Tests 2015: Test 12 Solutions


Many cave sites have the standard first type of chaitya halls in the subsequent period. In
Karla, the biggest rock-cut chaitya hall was excavated.
The stupas in the fourth and fifth centuries CE have Buddha images attached. Junnar has the
largest cave excavations more than two hundred caves around the hills of the town
whereas Kanheri in Mumbai has a hundred and eight excavated caves. The most important
sites are Ajanta, Pitalkhora, Ellora, Nashik, Bhaja, Junnar, Karla, Kanheri. Ajanta, Ellora, and
Kanheri continue to flourish.

51. Solution: d)

There are twelve Buddhist caves having many images belonging to Vajrayana Buddhism
like Tara, Mahamayuri, Akshobhya, Avalokiteshwara, Maitrya, Amitabha, etc.
The brahamanical caves have many sculptures. Many caves are dedicated to Shaivism, but
the images of both Shiva and Vishnu and their various forms according to Puranic narrative
are depicted. Among the Shaivite themes, Ravana shaking Mount Kailash,
Andhakasurvadha, Kalyanasundara are profusely depicted whereas among the Vaishnavite
themes, the different avatars of Vishnu are depicted.

52. Solution: d)

http://www.insightsonindia.com

INSIGHTS

Page 20

Insights Mock Tests 2015: Test 12 Solutions

The Elephanta Caves located near Mumbai, were originally a Buddhist site which was later
dominated by the Shaivite faith. It is contemporary with Ellora, and its sculptures show
slenderness in the body, with stark light and dark effects. The other noteworthy cave site is
Bagh located near Indore in Madhya Pradesh.
The tradition of rock-cut caves continued in the Deccan and they are found not only in
Maharashtra but also in Karnataka, mainly at Badami and Aiholi, executed under the
patronage of the Chalukyas; in Andhra Pradesh in the area of Vijayawada; and in Tamil
Nadu, mainly at Mahabalipuram, under the patronage of the Pallavas.

53. Solution: c)
The International Atomic Energy Agency (IAEA) was established in 1957. It came into being
to implement US President Dwight Eisenhowers Atoms for Peace proposal. It seeks to
promote the peaceful use of nuclear energy and to prevent its use for military purposes.
IAEA teams regularly inspect nuclear facilities all over the world to ensure that civilian
reactors are not being used for military purposes.
It was in news due to the US action against the Irani Nuclear programme.
To be noted, even though India has not signed NPT, due to signing Indo-US civil nuclear
deal, we are supposed to allow for IAEA inspections.

54. Solution: d)
UN sec-Gen does not have any decision making powers with respect to the decisions taken
by the UNGA and UNSC. He is more of an executive officer of the UN who assists the UN in
its functioning.

http://www.insightsonindia.com

INSIGHTS

Page 21

Insights Mock Tests 2015: Test 12 Solutions


WHO, UNCTAD, UNECOSOC are organs of the UN which have lesser members than the
UNGA. So membership of the UNGA does not automatically qualify countries for their
membership of the other agencies and organs.

55. Solution: a)
Commons are those resources which are not owned by anyone but rather shared by a
community.
This could be a common room, a community centre, a park or a river. Similarly, there are
some areas or regions of the world which are located outside the sovereign jurisdiction of
any one state, and therefore require common governance by the international community.
These are known as res communis humanitatis or global commons. They include the earths
atmosphere, Antarctica, the ocean floor, and outer space. Cooperation over the global
commons is not easy. There have been many path-breaking agreements such as the 1959
Antarctic Treaty, the 1987 Montreal Protocol, and the 1991 Antarctic Environmental
Protocol.
A major problem underlying all ecological issues relates to the difficulty of achieving
consensus on common environmental agendas on the basis of vague scientific evidence and
time frames. In that sense the discovery of the ozone hole over the Antarctic in the mid1980s revealed the opportunity as well as dangers inherent in tackling global environmental
problems.

56. Solution: a)

http://www.insightsonindia.com

INSIGHTS

Page 22

Insights Mock Tests 2015: Test 12 Solutions

57. Solution: c)
In a slew of measures, India and China have agreed for some CBMs across the borders. It
includes sharing information on the location of troops; military modernization drives;
information exchange in case of violation of ceasefire; strengthening the communication
between the major generals of both the armies etc.

58. Solution: c)
In introducing the Permanent Settlement, British officials hoped to resolve the problems
they had been facing since the conquest of Bengal. By the 1770s, the rural economy in Bengal
was in crisis, with recurrent famines and declining agricultural output. Officials felt that
agriculture, trade and the revenue resources of the state could all be developed by
encouraging investment in agriculture. This could be done by securing rights of property
and permanently fixing the rates of revenue demand. If the revenue demand of the state was
http://www.insightsonindia.com

INSIGHTS

Page 23

Insights Mock Tests 2015: Test 12 Solutions


permanently fixed, then the Company could look forward to a regular flow of revenue,
while entrepreneurs could feel sure of earning a profit from their investment, since the state
would not siphon it off by increasing its claim. The process, officials hoped, would lead to
the emergence of a class of yeomen farmers and rich landowners who would have the
capital and enterprise to improve agriculture. Nurtured by the British, this class would also
be loyal to the Company.

59. Solution: d)
Company officials felt that a fixed revenue demand would give zamindars a sense of
security and, assured of returns on their investment, encourage them to improve their
estates. In the early decades after the Permanent Settlement, however, zamindars regularly
failed to pay the revenue demand and unpaid balances accumulated. The reasons for this
failure were various.
First: the initial demands were very high. This was because it was felt that if the demand
was fixed for all time to come, the Company would never be able to claim a share of
increased income from land when prices rose and cultivation expanded. To minimise this
anticipated loss, the Company pegged the revenue demand high, arguing that the burden
on zamindars would gradually decline as agricultural production expanded and prices rose.
Second: this high demand was imposed in the 1790s, a time when the prices of agricultural
produce were depressed, making it difficult for the ryots to pay their dues to the zamindar.
If the zamindar could not collect the rent, how could he pay the Company?
Third: the revenue was invariable, regardless of the harvest, and had to be paid punctually.
In fact, according to the Sunset Law, if payment did not come in by sunset of the specified
date, the zamindari was liable to be auctioned. Fourth: the Permanent Settlement initially
limited the power of the zamindar to collect rent from the ryot and manage his zamindari.
The Company had recognised the zamindars as important, but it wanted to control and
regulate them, subdue their authority and restrict their autonomy.
The zamindars troops were disbanded, customs duties abolished, and their cutcheries
(courts) brought under the supervision of a Collector appointed by the Company.
Zamindars lost their power to organise local justice and the local police. Over time the
collectorate emerged as an alternative centre of authority, severely restricting what the
zamindar could do. In one case, when a raja failed to pay the revenue, a Company official
was speedily dispatched to his zamindari with explicit instructions to take charge of the
District and to use the most effectual means to destroy all the influence and the authority of
the raja and his officers.

60. Solution: c)

http://www.insightsonindia.com

INSIGHTS

Page 24

Insights Mock Tests 2015: Test 12 Solutions


While many zamindars were facing a crisis at the end of the eighteenth century, a group of
rich peasants were consolidating their position in the villages. In Francis Buchanans survey
of the Dinajpur district in North Bengal we have a vivid description of this class of rich
peasants known as jotedars. By the early nineteenth century, jotedars had acquired vast
areas of land sometimes as much as several thousand acres. They controlled local trade as
well as moneylending, exercising immense power over the poorer cultivators of the region.
A large part of their land was cultivated through sharecroppers (adhiyars or bargadars) who
brought their own ploughs, laboured in the field, and handed over half the produce to the
jotedars after the harvest.
Within the villages, the power of jotedars was more effective than that of zamindars. Unlike
zamindars who often lived in urban areas, jotedars were located in the villages and
exercised direct control over a considerable section of poor villagers. They fiercely resisted
efforts by zamindars to increase the jama of the village, prevented zamindari officials from
executing their duties, mobilised ryots who were dependent on them, and deliberately
delayed payments of revenue to the zamindar. In fact, when the estates of the zamindars
were auctioned for failure to make revenue payment, jotedars were often amongst the
purchasers.

61. Solution: d)
Paharis lived around the Rajmahal hills, subsisting on forest produce and practising shifting
cultivation.
With their base in the hills, the Paharias regularly raided the plains where settled
agriculturists lived. These raids were necessary for survival, particularly in years of scarcity;
they were a way of asserting power over settled communities; and they were a means of
negotiating political relations with outsiders. The zamindars on the plains had to often
purchase peace by paying a regular tribute to the hill chiefs. Traders similarly gave a small
amount to the hill folk for permission to use the passes controlled by them.

62. Solution: b)
http://www.downtoearth.org.in/content/chile-copper-mine-water-use-risingdesalinitaion-industry-expected-gain-heavily
http://en.wikipedia.org/wiki/Chile

63. Solution: a)
The World Health Organization has allowed the use of meningitis vaccine MenAfriVac on
infants in Africa. The vaccine, which is inexpensive and effective, has been in use since 2010
http://www.insightsonindia.com

INSIGHTS

Page 25

Insights Mock Tests 2015: Test 12 Solutions


and is manufactured by the Serum Institute of India Ltd. The move would allow the use of
the vaccine for routine immunisation of children.
In a news release dated January 9, the World Health Organization (WHO) said, In the four
years since its introduction in Africa, MenAfriVac has had an immediate and dramatic
impact in breaking the cycle of meningitis A epidemics, leading the safe, effective
technology to be approved by WHO through its prequalification process for use in infants,
and paving the way for protecting millions more children at risk of the deadly disease.
Meningitis is a major killer in Africa, particularly in the meningitis belt of sub-Saharan
Africa which stretches from Ethiopia in the east to Senegal in the west.
http://www.downtoearth.org.in/content/who-approves-use-meningitis-vaccine-infants

64. Solution: d)
Wave power is distinct from the diurnal flux of tidal power and the steady gyre of ocean
currents. (http://www.rnp.org/node/wave-tidal-energy-technology)
There is a potential impact on the marine environment. Noise pollution, for example, could
have negative impact if not monitored, although the noise and visible impact of each design
vary greatly. Other biophysical impacts (flora and fauna, sediment regimes and water
column structure and flows) of scaling up the technology is being studied. In terms of socioeconomic challenges, wave farms can result in the displacement of commercial and
recreational fishermen from productive fishing grounds, can change the pattern of beach
sand nourishment, and may represent hazards to safe navigation.[86] Waves generate about
2,700 gigawatts of power. Of those 2,700 gigawatts, only about 500 gigawatts can be
captured with the current technology.
http://www.downtoearth.org.in/content/wave-energy-could-prove-be-more-steadycheaper-renewable-energy-source

65. Solution: b)
http://en.wikipedia.org/wiki/Blackbuck
http://en.wikipedia.org/wiki/Blackbuck_National_Park,_Velavadar
http://www.downtoearth.org.in/content/blackbuck-case-supreme-court-sets-aside-highcourt-stay-salman-khan-s-conviction

66. Solution: c)

http://www.insightsonindia.com

INSIGHTS

Page 26

Insights Mock Tests 2015: Test 12 Solutions


http://www.downtoearth.org.in/content/africa-s-masai-tribals-turn-slaughterhouse-wastebiogas
http://en.wikipedia.org/wiki/Maasai_people

67. Solution: c)
http://pib.nic.in/newsite/PrintRelease.aspx?relid=114328
http://en.wikipedia.org/wiki/LED_lamp#Comparison_to_other_lighting_technologies
http://en.wikipedia.org/wiki/LED_lamp#Limitations

68. Solution: b)
The insurrection of the Santhals was mainly against the British and their supporters like
moneylenders, bangali zamindars and their operatives. Before the advent of the British in
India the Santhals resided peacefully in the hilly districts of Mayurbhanj Chhotanagpur,
Palamau, Hazaribagh, Midnapur, Bankura and Birbhum. Their agrarian way of life was
based on clearing the forest; they also engaged themselves in hunting for subsistence. But, as
the agents of the new colonial rule claimed their rights on the lands of the Santhals, they
peacefully went to reside in the hills of Rajmahal. After a brief period of peace the British
operatives with their native counterparts jointly started claiming their rights in this new
land as well. The simple and honest Santhals were cheated and turned into slaves by
thezamindars and the money lenders who first appeared to them as business men and lured
them into debt, first by goods lent to them on loans. However hard the Santhals tried to
repay these loans, they never ended. Through corrupt measures of the money lenders, the
debts multiplied to an amount for which a generation of the Santhal family had to work as
slaves. Furthermore, the Santhal women who worked under labour contractors were
disgraced and abused.
http://en.wikipedia.org/wiki/Santhal_people
69. Solution: d)
The Water Storage available in 85 important reservoirs of the country as on January
01, 2015 was 89.947 BCM which is58% of total storage capacity of these reservoirs. This
storage is 85% of the storage of corresponding period of last year and 100% of storage of
average of last ten years. The present storage position during current year is less than the
storage position of last year but is equal to the storage of average of last ten years.
Central Water Commission monitors live storage status of 85 important reservoirs of the
country on weekly basis. These reservoirs include 37 reservoirs having hydropower benefit
with installed capacity of more than 60 MW. The total storage capacity of these reservoirs
is 155.046 BCM which is about 61% of the storage capacity of 253.388 BCM which is
estimated to have been created in the country.

http://www.insightsonindia.com

INSIGHTS

Page 27

Insights Mock Tests 2015: Test 12 Solutions


REGION WISE STORAGE STATUS :

NORTHERN REGION
The northern region includes States of Himachal Pradesh, Punjab and Rajasthan. There
are 6 reservoirs in this region having total storage capacity of 18.01 BCM. The total storage
available in these reservoirs is 8.88 BCM which is 49% of total storage capacity of these
reservoirs. The storage during corresponding period of last year was 63% and average
storage of last ten years during corresponding period was 54% of storage capacity of these
reservoirs. Thus, storage during current year is less than the corresponding period of last
year and also less than the average storage of last ten years during the corresponding
period.

EASTERN REGION
The Eastern region includes States of Jharkhand, Odisha, West Bengal and Tripura. There
are 15 reservoirs in this region having total storage capacity of 18.83 BCM. The total storage
available in these reservoirs is 13.61 BCM which is 72% of total storage capacity of these
reservoirs. The storage during corresponding period of last year was 86% and average
storage of last ten years during corresponding period was 68% of storage capacity of these
reservoirs. Thus, storage during current year is less than the corresponding period of last
year but better than the average storage of last ten years during the corresponding period.

WESTERN REGION
The Western region includes States of Gujarat and Maharashtra. There are 22 reservoirs
in this region having total live storage capacity of 24.54 BCM. The total storage available in
these reservoirs is 14.48 BCM which is 59% of total storage capacity of these reservoirs. The
storage during corresponding period of last year was 75% and average storage of last ten
years during corresponding period was 66% of storage capacity of these reservoirs. Thus,
storage during current year is less than the storage of last year and also less than the average
storage of last ten years.

CENTRAL REGION
The Central region includes States of Uttar Pradesh, Uttarakhand, Madhya Pradesh and
Chhattisgarh. There are 12 reservoirs in this region having total storage capacity of
42.30BCM. The total storage available in these reservoirs is 28.64 BCM which is 68% of total
storage capacity of these reservoirs. The storage during corresponding period of last year
was 74% and average storage of last ten years during corresponding period was 48% of live
storage capacity of these reservoirs. Thus, storage during current year is less than the storage
of last year but better than the average storage of last ten years.

SOUTHERN REGION
The Southern region includes States of Andhra Pradesh, Karnataka, Kerala and Tamil
Nadu. There are 30 reservoirs in this region having total live storage capacity of 51.37 BCM.
The total storage available in these reservoirs is 24.35 BCM which is 47% of total storage
capacity of these reservoirs. The storage during corresponding period of last year was 56%
and average storage of last ten years during corresponding period was 61% of storage
http://www.insightsonindia.com

INSIGHTS

Page 28

Insights Mock Tests 2015: Test 12 Solutions


capacity of these reservoirs. Thus, storage during current year is less than the corresponding
period of last year and also less than the average storage of last ten years during the
corresponding period.
States having better storage than last year for corresponding period
are Karnataka, Kerala and Tamil Nadu. States having equal storage than last year for
corresponding period is Uttarakhand. States having lesser storage than last year for
corresponding period are Himachal Pradesh, Punjab, Rajasthan, Jharkhand, Odisha, West
Bengal, Tripura, Gujarat, Maharashtra, Uttar Pradesh, Madhya Pradesh, Chattisgarh,
Andhra Pradesh.
http://pib.nic.in/newsite/PrintRelease.aspx?relid=114331

70. Solution: a)
When the American Civil War broke out in 1861, a wave of panic spread through cotton
circles in Britain. Raw cotton imports from America fell to less than three per cent of the
normal: from over 2,000,000 bales (of 400 lbs each) in 1861 to 55,000 bales in 1862. Frantic
messages were sent to India and elsewhere to increase cotton exports to Britain. In Bombay,
cotton merchants visited the cotton districts to assess supplies and encourage cultivation. As
cotton prices soared (see Fig. 10.15), export merchants in Bombay were keen to secure as
much cotton as possible to meet the British demand. So they gave advances to urban
sahukars who in turn extended credit to those rural moneylenders who promised to secure
the produce. When there is a boom in the market credit flows easily, for those who give out
loans feel secure about recovering their money.

71. Solution: c)
The sepoys began their action with a signal: in many places it was the firing of the evening
gun or the sounding of the bugle. They first seized the bell of arms and plundered the
treasury. They then attacked government buildings the jail, treasury, telegraph office,
record room, bungalows burning all records. Everything and everybody connected with
the white man became a target. Proclamations in Hindi, Urdu and Persian were put up in
the cities calling upon the population, both Hindus and Muslims, to unite, rise and
exterminate the firangis.
When ordinary people began joining the revolt, the targets of attack widened. In major
towns like Lucknow, Kanpur and Bareilly, moneylenders and the rich also became the
objects of rebel wrath. Peasants not only saw them as oppressors but also as allies of the
British. In most places their houses were looted and destroyed. The mutiny in the sepoy
ranks quickly became a rebellion.

http://www.insightsonindia.com

INSIGHTS

Page 29

Insights Mock Tests 2015: Test 12 Solutions


72. Solution: d)
http://pib.nic.in/newsite/PrintRelease.aspx?relid=114407

73. Solution: d)
To fight the British, leadership and organisation were required. For these the rebels
sometimes turned to those who had been leaders before the British conquest. One of the first
acts of the sepoys of Meerut, as we saw, was to rush to Delhi and appeal to the old Mughal
emperor to accept the leadership of the revolt. This acceptance of leadership took its time in
coming.
Bahadur Shahs first reaction was one of horror and rejection. It was only when some sepoys
had moved into the Mughal court within the Red Fort, in defiance of normal court etiquette,
that the old emperor, realising he had very few options, agreed to be the nominal leader of
the rebellion. Elsewhere, similar scenes were enacted though on a minor scale. In Kanpur,
the sepoys and the people of the town gave Nana Sahib, the successor to Peshwa Baji Rao II,
no choice save to join the revolt as their leader. In Jhansi, the rani was forced by the popular
pressure around her to assume the leadership of the uprising.
So was Kunwar Singh, a local zamindar in Arrah in Bihar. In Awadh, where the
displacement of the popular Nawab Wajid Ali Shah and the annexation of the state were still
very fresh in the memory of the people, the populace in Lucknow celebrated the fall of
British rule by hailing Birjis Qadr, the young son of the Nawab, as their leader. Not
everywhere were the leaders people of the court ranis, rajas, nawabs and taluqdars.
Often the message of rebellion was carried by ordinary men and women and in places by
religious men too. From Meerut, there were reports that a fakir had appeared riding on an
elephant and that the sepoys were visiting him frequently. In Lucknow, after the annexation
of Awadh, there were many religious leaders and self-styled prophets who preached the
destruction of British rule.
Elsewhere, local leaders emerged, urging peasants, zamindars and tribals to revolt. Shah
Mal mobilised the villagers of pargana Barout in Uttar Pradesh; Gonoo, a tribal cultivator of
Singhbhum in Chotanagpur, became a rebel leader of the Kol tribals of the region.

74. Solution: d)
http://pib.nic.in/newsite/PrintRelease.aspx?relid=114391

75. Solution: a)
Key facts
http://www.insightsonindia.com

INSIGHTS

Page 30

Insights Mock Tests 2015: Test 12 Solutions

Japanese encephalitis (JE) is a flavivirus related to dengue, yellow fever and West
Nile viruses, and is spread by mosquitoes.

JE is the main cause of viral encephalitis in many countries of Asia with nearly 68
000 clinical cases every year.

Although symptomatic JE is rare, the case-fatality rate among those with encephalitis
can be as high as 30%. Permanent neurologic or psychiatric sequelae can occur in
30%50% of those with encephalitis.

24 countries in the WHO South-East Asia and Western Pacific regions have endemic
JE transmission, exposing more than 3 billion people to risks of infection.

There is no cure for the disease. Treatment is focused on relieving severe clinical
signs and supporting the patient to overcome the infection.

Safe and effective vaccines are available to prevent JE. WHO recommends JE
vaccination in all regions where the disease is a recognised public health problem.

http://pib.nic.in/newsite/PrintRelease.aspx?relid=114402
http://www.who.int/mediacentre/factsheets/fs386/en/

76. Solution: d)
On Gokhales advice, Gandhiji spent a year travelling around British India, getting to know
the land and its peoples. His first major public appearance was at the opening of the Banaras
Hindu University (BHU) in February 1916. Among the invitees to this event were the
princes and philanthropists whose donations had contributed to the founding of the BHU.
Also present were important leaders of the Congress, such as Annie Besant. Compared to
these dignitaries, Gandhiji was relatively unknown. He had been invited on account of his
work in South Africa, rather than his status within India. When his turn came to speak,
Gandhiji charged the Indian elite with a lack of concern for the labouring poor.
The opening of the BHU, he said, was certainly a most gorgeous show. But he worried
about the contrast between the richly bedecked noblemen present and millions of the
poor Indians who were absent. Gandhiji told the privileged invitees that there is no
salvation for India unless you strip yourself of this jewellery and hold it in trust for your
countrymen in India. There can be no spirit of selfgovernment about us, he went on, if
we take away or allow others to take away from the peasants almost the whole of the results
of their labour. Our salvation can only come through the farmer. Neither the lawyers, nor
the doctors, nor the rich landlords are going to secure it. The opening of the BHU was an
occasion for celebration, marking as it did the opening of a nationalist university, sustained
by Indian money and Indian initiative. But rather than adopt a tone of self-congratulation,
http://www.insightsonindia.com

INSIGHTS

Page 31

Insights Mock Tests 2015: Test 12 Solutions


Gandhiji chose instead to remind those present of the peasants and workers who constituted
a majority of the Indian population, yet were unrepresented in the audience.

77. Solution: a)
Two new reports, released by the World Bank Group, have said that the socio-economic
impacts of Ebola in Liberia and Sierra Leone are far-reaching as both the countries continue
to experience job losses. The reports are based on mobile surveys carried out by the
international bank and its partners.
Following the economic slowdown, triggered by reduction in jobs, many households have
been forced to take short-term actions to cope with the changing socio-economic landscape.
For example, two-third of the people surveyed in Liberia have said that they were not able
to purchase enough rice to meet their needs in the previous two weeks, and nearly 80 per
cent of those cited a lack of money as the main reason.
At present, nearly half of Liberian household heads remain out of work. In Sierra Leone, an
estimated 9,000 wage workers and 170,000 self-employed workers outside of agriculture are
no longer working since the July/August 2014 baseline.
http://www.downtoearth.org.in/content/ebola-damaging-economies-two-countries-westafrica

78. Solution: c)
http://pib.nic.in/newsite/PrintRelease.aspx?relid=114566
http://en.wikipedia.org/wiki/Idukki_district

79. Solution: d)
http://pib.nic.in/newsite/PrintRelease.aspx?relid=114564
http://en.wikipedia.org/wiki/Swami_Vivekananda

80. Solution: d)
Union Ministry of Agricultures Bringing Green Revolution to Eastern India (BGREI)
programme was launched in Assam, Bihar, Chhattisgarh, Odisha, Purvanchal (eastern Uttar
Pradesh), Jharkhand and West Bengal. With this initiative, the government hoped to
improve production of crops, especially rice, and water resource management in the region.
http://www.insightsonindia.com

INSIGHTS

Page 32

Insights Mock Tests 2015: Test 12 Solutions


http://www.downtoearth.org.in/content/new-green-revolution-programme-has-notbenefited-jharkhand-farmers-says-report

81. Solution: d)
The Millennium Development Goals (MDGs) are eight international
development goals that were established following the Millennium Summit of the United
Nations in 2000, following the adoption of the United Nations Millennium Declaration. All
189 United Nations member states at the time (there are 193 currently), and at least
23 international organizations, committed to help achieve the following Millennium
Development Goals by 2015:
To eradicate extreme poverty and hunger
To achieve universal primary education
To promote gender equality and empower women
To reduce child mortality
To improve maternal health
To combat HIV/AIDS, malaria, and other diseases
To ensure environmental sustainability
To develop a global partnership for development
http://www.downtoearth.org.in/content/mdg-led-healthcare-policy-not-comprehensivesays-report
82. Solution: d)
In June 1944, with the end of the war in sight, Gandhiji was released from prison. Later that
year he held a series of meetings with Jinnah, seeking to bridge the gap between the
Congress and the League. In 1945, a Labour government came to power in Britain and
committed itself to granting independence to India. Meanwhile, back in India, the Viceroy,
Lord Wavell, brought the Congress and the League together for a series of talks. Early in
1946 fresh elections were held to the provincial legislatures.
The Congress swept the General category, but in the seats specifically reserved for
Muslims the League won an overwhelming majority. The political polarisation was
complete. A Cabinet Mission sent in the summer of 1946 failed to get the Congress and the
League to agree on a federal system that would keep India together while allowing the
provinces a degree of autonomy. After the talks broke down, Jinnah called for a Direct
Action Day to press the Leagues demand for Pakistan. On the designated day, 16 August
1946, bloody riots broke out in Calcutta. The violence spread to rural Bengal, then to Bihar,

http://www.insightsonindia.com

INSIGHTS

Page 33

Insights Mock Tests 2015: Test 12 Solutions


and then across the country to the United Provinces and the Punjab. In some places,
Muslims were the main sufferers, in other places, Hindus.

83. Solution: b)
UN is not the world government. It is only an international organization with hardly any
formal authority.
UNSC has 5 permanent members that yield veto power. So it does not work on the principle
of equity.
States can not be punished severely. Occasional sanctions may be put that too only when
there is a gross case of unauthorized military intervention; genocide etc. There are other
bodies like WTO which may take action against its members in case of violation of WTO
rules.

84. Solution: b)
The European Union has evolved over time from an economic union to an increasingly
political one. The EU has started to act more as a nation state. While the attempts to have a
Constitution for the EU have failed, it has its own flag, anthem, founding date, and currency.
It also has some form of a common foreign and security policy in its dealings with other
nations.
The European Union has tried to expand areas of cooperation while acquiring new
members, especially from the erstwhile Soviet bloc. The process has not proved easy, for
people in many countries are not very enthusiastic in giving the EU powers that were
exercised by the government of their country. There are also reservations about including
some new countries within the EU.
Its share of world trade is three times larger than that of the United States allowing it to be
more assertive in trade disputes with the US and China. Its economic power gives it
influence over its closest neighbours as well as in Asia and Africa. It also functions as an
important bloc in international

85. Solution: c)
The Chinese leadership took major policy decisions in the 1970s. China ended its political
and economic isolation with the establishment of relations with the United States in 1972.
Premier Zhou Enlai proposed the four modernisations(agriculture, industry, science and
technology and military) in 1973. By 1978, the then leader Deng Xiaoping announced the

http://www.insightsonindia.com

INSIGHTS

Page 34

Insights Mock Tests 2015: Test 12 Solutions


open door policy and economic reforms in China. The policy was to generate higher
productivity by investments of capital and technology from abroad.
China followed its own path in introducing a market economy. The Chinese did not go for
shock therapy but opened their economy step by step. The privatisation of agriculture in
1982 was followed by the privatisation of industry in 1998. Trade barriers were eliminated
only in Special Economic Zones (SEZs) where foreign investors could set up enterprises.

86. Solution: c)
Sixth century BCE marks the beginning of new religious and social movements in the
Gangetic valley in the form of Buddhism and Jainism which were part of the shraman
tradition. Both religions became popular as they opposed the varna and jati systems of the
Hindu religion. Magadha emerged as a powerful kingdom and consolidated its control over
the other regions.
By the fourth century BCE the Mauryas established their power and by the third century
BCE, a large part of India was under Mauryan control. Ashoka emerged as the most
powerful king of the Mauryan dynasty who patronised the shraman tradition in the third
century BCE.

87. Solution: d)
The tradition of constructing pillars is very old and it may be observed that erection of
pillars was prevalent in the Achamenian empire as well. But the Mauryan pillars are
different from the Achamenian pillars. The Mauryan pillars are rock-cut pillars thus
displaying the carvers skills, whereas the Achamenian pillars are constructed in pieces by a
mason.
Stone pillars were erected all over the Mauryan Empire with inscriptions engraved on them.
The top portion of the pillar was carved with capital figures like the bull, the lion, the
elephant, etc. The Mauryan pillar capital found at Sarnath popularly known as the Lion
Capital is the finest example of Mauryan sculptural tradition. It is also our national emblem.

88. Solution: b)

http://www.insightsonindia.com

INSIGHTS

Page 35

Insights Mock Tests 2015: Test 12 Solutions

The above event is part of the enlightenment where Buddha is surrounded by Maras army.
It is a personification of the commotion of mind which the Buddha went through at the time
of enlightenment. Mara represents desire.
According to the narrative, there is a dialogue between the Buddha and Mara, and the
Buddha is shown with his right hand indicating towards earth as a witness to his generosity.

89. Solution: d)

http://www.insightsonindia.com

INSIGHTS

Page 36

Insights Mock Tests 2015: Test 12 Solutions

90. Solution: d)

http://www.insightsonindia.com

INSIGHTS

Page 37

Insights Mock Tests 2015: Test 12 Solutions


Badami was the capital of the western Chalukyan dynasty which ruled the region from 543
to 598 CE. With the decline of the Vakataka rule, the Chalukyas established their power in
the Deccan. The Chalukya king, Mangalesha, patronised the excavation of the Badami
caves. He was the younger son of the Chalukya king, Pulakesi I, and the brother of
Kirtivarman I. The inscription in Cave No.4 mentions the date 578579 CE, describes the
beauty of the cave and includes the dedication of the image of Vishnu. Thus it may be
presumed that the cave was excavated in the same era and the patron records his Vaishnava
affiliation. Therefore, the cave is popularly known as the Vishnu Cave. Only a fragment of
the painting has survived on the vaulted roof of the front mandapa.
Paintings in this cave depict palace scenes. One shows Kirtivarman, the son of Pulakesi I and
the elder brother of Mangalesha, seated inside the palace with his wife and feudatories
watching a dance scene. Towards the corner of the panel are figures of Indra and his retinue.

91. Solution: a)
The tradition of building temples and embellishing them with carvings and paintings
continued during the reign of the Chola kings who ruled over the region from the ninth to
the thirteenth century. But it was in the eleventh century, when the Cholas reached their
zenith of power, that masterpieces of Chola art and architecture began to appear. The
temples of Brihadeswara at Thanjavur, Gangaikonda Cholapuram and Darasuram were
built during the reigns of Rajaraja Chola and his son, Rajendra Chola.
Though Chola paintings are seen in Nartamalai, the most important are those in
Brihadeswara temple. The paintings were executed on the walls of the narrow passage
surrounding the shrine. Two layers of paint were found when they were discovered. The
upper layer was painted during the Nayak period, in the sixteenth century. Thanks to the
cleaning of the surface painting, examples of the great tradition of painting during the Chola
Period were unveiled. The paintings show narrations and aspects related to Lord Shiva,
Shiva in Kailash, Shiva as Tripurantaka, Shiva as Nataraja, a portrait of the patron Rajaraja
and his mentor Kuruvar, dancing figures, etc.

92. Solution: d)
The painters in Kerala evolved a language taking cues from contemporary traditions like
Kathakali and kalam ezhuthu using vibrant and luminous colours, representing human
figures in three-dimensionality. Most of the paintings are seen on the walls of shrines and
cloister walls of temples and some inside palaces. Thematically too, paintings from Kerala
stand apart. Most of the narrations are based on those episodes from Hindu mythology
which were popular in Kerala. The artist seems to have derived sources from oral traditions
and local versions of the Ramayana and the Mahabharata for painted narration.

http://www.insightsonindia.com

INSIGHTS

Page 38

Insights Mock Tests 2015: Test 12 Solutions


More than sixty sites have been found with mural paintings which include three palaces
Dutch palace in Kochi, Krishnapuram palace in Kayamkulam and Padmanabhapuram
palace. Among the sites where one can see the mature phase of Keralas mural painting
tradition are Pundareekapuram Krishna temple, Panayanarkavu, Thirukodithanam,
Triprayar Sri Rama temple and Trissur Vadakkunathan temple.

93. Solution: a)
After India regained its independence from Britain, and China expelled the foreign powers,
there was hope that both would come together to shape the future of the developing world
and of Asia particularly. For a brief while, the slogan of Hindi-Chini bhaibhai was popular.
However military conflict over a border dispute between the two countries marred that
hope. Soon after independence, both states were involved in differences arising from the
Chinese takeover of Tibet in 1950 and the final settlement of the Sino-Indian border. China
and India were involved in a border conflict in 1962 over competing territorial claims
principally in Arunachal Pradesh and in the Aksai Chin region of Ladakh.

94. Solution: b)
Jammu Kashmir State assembly election held in December 2014 had given hung verdict as
no party has majority to form government in State.

In case of failure of constitutional machinery in any other state of India, the


Presidents Rule is imposed under Article 356 of the Constitution.

But in case of J&K, as per Section 92 of state Constitution, the Governors Rule can
be imposed in the state for a period of six months only after the consent of the
President of India in case of failure of constitutional machinery. During the
Governors rule, State Assembly is either suspended or dissolved.

If the Constitutional machinery is not restored before the expiry of this six month
period, the provision of Article 356 of the Constitution of India are extended to J&K
and the Presidents rule is imposed in the State.

It should be noted that, Governors rule has been imposed in the state for the 6 time
since 1977.

95. Solution: a)
Key amendments

http://www.insightsonindia.com

INSIGHTS

Page 39

Insights Mock Tests 2015: Test 12 Solutions

Citizenship Act amendments will benefit PIO and will give them benefits like lifelong visa and exemption from appearing before the local police station on every visit
etc.

It will also change the clause of parent Act which says that foreigners who marry
Indians must continuously stay in the country for a period of one year before they
get an Indian citizenship. In this regards they will get relief of 30 days in a year when
they can travel outside the country.

The amendment also clears the way for relaxed visa norms for minor children of OCI
cardholders.

96. Solution: d)

It is launched in lines with Governments flagship programme of Digital India.

It seeks to provide entire village with services ranging from cashless banking to
digitised school teaching.

As part of this project, banking platform at rural part will be digitized by covering all
aspects of banking like opening an account to sale of goods to purchase of products
including milk from the vendors or local kirana stores.

In case of school teaching, it will digitized all school records alongwith the Gujarat
syllabus and even teaching methods and tools.

It will also provide necessary infrastructure needed for digisiting entire village in
order to provide villager with the access required to data and information in a digital
format.

Besides basic banking facilities like ATMs and other digital banking services, other
services like e-health, e-milk producer group, Wi-Fi connectivity and schools with
digital black boards in the village, along with a host of other digital facilities will be
provided under this project.

97. Solution: d)
http://www.downtoearth.org.in/content/countries-rushing-ban-styrofoam-foodcontainers

98. Solution: a)
http://www.downtoearth.org.in/content/cabinet-clears-mineral-act-foreign-companiescan-mine-too
http://www.insightsonindia.com

INSIGHTS

Page 40

Insights Mock Tests 2015: Test 12 Solutions

99. Solution: c)
Unlike the EU there is little desire in ASEAN for supranational structures and institutions.
ASEAN countries have celebrated what has become known as the ASEAN Way, a form of
interaction that is informal, non-confrontationist and cooperative. The respect for national
sovereignty is critical to the functioning of ASEAN.
With some of the fastest growing economies in the world, ASEAN broadened its objectives
beyond the economic and social spheres. In 2003, ASEAN moved along the path of the EU
by agreeing to establish an ASEAN Community comprising three pillars, namely, the
ASEAN Security Community, the ASEAN Economic Community and the ASEAN SocioCultural Community.

100.

Solution: b)

Pakistan and Bangladesh have experienced both civilian and military rulers, with
Bangladesh remaining a democracy in the post-Cold War period. Pakistan began the postCold War period with successive democratic governments under Benazir Bhutto and Nawaz
Sharif respectively. But it suffered a military coup in 1999 and has been run by a military
regime since then. Till 2006, Nepal was a constitutional monarchy with the danger of the
king taking over executive powers. In 2006 a successful popular uprising led to the
restoration of democracy and reduced the king to a nominal position.

http://www.insightsonindia.com

INSIGHTS

Page 41

INSIGHTS ON INDIA MOCK PRELIMINARY EXAM - 2015


INSIGHTS ON INDIA MOCK TEST - 13
GENERAL STUDIES

PAPER-I
Time Allowed: 2 Hours

Maximum Marks: 200

INSTRUCTIONS
1. IMMEDITELY AFTER THE COMMENCEMENT OF THE EXAMINATION, YOU SHOULD
CHECK THAT THIS TEST BOOKLET DOES NOT HAVE ANY UNPRINTED OR TORN OR MISSING
PAGES OR ITEMS, ETC. IF SO, GET IT REPLACED BY A COMPLETE TEST BOOKLET.
2. You have to enter your Roll Number on the Test
Booklet in the Box provided alongside. DO NOT
Write anything else on the Test Booklet.
4. This Test Booklet contains 100 items (questions). Each item is printed only in English. Each item
comprises four responses (answers). You will select the response which you want to mark on the
Answer Sheet. In case you feel that there is more than one correct response, mark the response which
you consider the best. In any case, choose ONLY ONE response for each item.
5. You have to mark all your responses ONLY on the separate Answer Sheet provided. See directions in
the Answer Sheet.
6. All items carry equal marks.
7. Before you proceed to mark in the Answer Sheet the response to various items in the Test Booklet, you
have to fill in some particulars in the Answer Sheet as per instructions sent to you with your
Admission Certificate.
8. After you have completed filling in all your responses on the Answer Sheet and the examination has
concluded, you should hand over to the Invigilator only the Answer Sheet. You are permitted to take
away with you the Test Booklet.
9. Sheets for rough work are appended in the Test Booklet at the end.
10. Penalty for wrong answers :
THERE WILL BE PENALTY FOR WRONG ANSWERS MARKED BY A CANDIDATE IN THE
OBJECTIVE TYPE QUESTION PAPERS.
(i) There are four alternatives for the answer to every question. For each question for which a
wrong answer has been given by the candidate, one-third of the marks assigned to that
question will be deducted as penalty.
(ii) If a candidate gives more than one answer, it will be treated as a wrong answer even if one of
the given answers happens to be correct and there will be same penalty as above to that
question.
(iii)

If a question is left blank, i.e., no answer is given by the candidate, there will be no penalty
for that question.
http://www.insightsonindia.com

INSIGHTS ON INDIA MOCK TEST SERIES FOR CIVIL SERVICES PRELIMINARY EXAM 2015
http://insightsonindia.com

INSIGHTS

Page 1

1. Leaching of soil is detrimental to soil


productivity. This is because
a) Water-soluble inorganic nutrients
go down into the soil horizon and
get precipitated as unavailable
salts.
b) Bacterial and other biological
agents can no longer catabolise soil
c) Humification of the soil stops
d) None of the above is an
appropriate answer

2. Consider the following statements


about breeding of animals.
1. Biologically unrelated animals
cannot be breeded (bred)
2. The animals being breeded (bred)
should have common ancestors.
3. Artificial insemination does not
come under breeding practices.
Choose the correct answer using the codes
below.
a)
b)
c)
d)

1 and 2
2 and 3
1 and 3
None of the above

3. More than 3 billion people in the


World suffer from Hidden Hunger. It
can be caused due to the deficiencies
of
1. Micronutrients
2. Proteins
3. Vitamins

a)
b)
c)
d)

4. Which of the following factors may


possibly account for the greater
biological diversity at the tropics?
1. Tropical latitudes have remained
undisturbed for longer durations
than higher latitudes.
2. Tropical environments are less
seasonal.
3. There is more solar energy
available at the tropics.
Choose the correct answer using the codes
below.
a)
b)
c)
d)

1 and 2
2 and 3
3 only
All of the above

5. In agriculture, the biological method of


controlling pests that relies on natural
predation rather than introduced
chemicals. Which of the following
microbes can be used in the method?
1. Bacillus thuringiensis
2. Trichoderma
3. Lady Beetle
4. Dipteran
5. Cordyceps
Choose the correct answer using the codes
below.
a)
b)
c)
d)

Choose the correct answer using the codes


below.
http://insightsonindia.com

1 and 2
2 and 3
1 only
All of the above

INSIGHTS

1 and 4 only
1, 3 and 5 only
2 and 5 only
All of the above
Page 2

6. Which of the following features of the


Indian Constitution deal with
maintaining the social fabric, moral
values and national consciousness?
1. Fundamental Rights
2. Fundamental Duties
3. Directive Principles of State Policy
Choose the correct answer using the codes
below.
a)
b)
c)
d)

1 and 2 only
2 and 3 only
1 and 3 only
All of the above

7. A recent draft order of the Food Safety


and Standards Authority of Indias
(FSSAI) has proposed to put curbs on
the rampant use of antibiotics as a
growth promoter in food-producing
animals in India, such as chicken. It
will help in containing
1. Unregulated entry of antibiotics
into the food-producing industry
2. Antibiotic resistance
3. Import of chicken from the US to
the Indian market.
Choose the correct answer using the codes
below.
a)
b)
c)
d)

1 and 2
2 and 3
1 and 3
All of the above

8. A Delta 2 rocket carrying Soil Moisture


Active Passive (SMAP) satellite has
been recently launched by NASA.
How will it help humanity?
1. It may help residents in low-lying
regions brace for floods
2. It will help farmers to get ready for
drought conditions.
3. It will study the soil profile of
various regions on earth and relay
the data back to the scientific
community to be then transmitted
to the farmers of respective
countries.
Choose the correct answer using the codes
below.
a)
b)
c)
d)

9. Which of the following category of


constitutional amendments would
require the ratification of the states in
India?
1. Citizenship clauses
2. Changes in the Concurrent List
under Seventh Schedule
3. Changes in the state boundaries
4. Changing the number of puisine
judges in the Supreme Court
Choose the correct answer using the codes
below.
a)
b)
c)
d)

http://insightsonindia.com

1 and 2
2 and 3
1 and 3
All of the above

INSIGHTS

1 and 4 only
2 and 3 only
1, 2 and 3 only
2 only
Page 3

10. Which of the following possibilities


can be found in a Parliamentary
Democracy?
1. It can be a constitutional monarchy.
2. President can be the head of the
government.
3. The Prime Minister may belong to
the party without majority in the
legislature.
Choose the correct answer using the codes
below.
a)
b)
c)
d)

1 and 2
2 and 3
1 and 3
All of the above

11. Consider the following statements


about the position, powers, functions
and responsibilities of the President of
India.
1. He has a right to be informed of all
important matters and
deliberations of the Council of
Ministers.
2. The President need not accept the
advice of Council of Ministers if the
advice has been given for the first
time.
3. The only ground for his
impeachment is the violation of the
Constitution.
4. He scrutinizes the annual budget
proposal of the government before
placing it to the Parliament.
Choose the correct answer using the codes
below.

b) 1, 2 and 3
c) 2 and 3 only
d) All of the above

12. Consider a situation where the Prime


Minister wants to impose Presidents
rule in one State because the State
government has failed to effectively
curb atrocities against the Dalits in that
State. The President does not agree
wholeheartedly on signing the order.
Which of the following courses of
action are available to the President?
1. Tell the Prime Minister that he will
not sign on the order promulgating
Presidents rule.
2. Make a press statement about how
the Prime Minister is wrong.
3. Discuss the matter with the Prime
Minister and try to dissuade him
from taking this action, but if he
insists, agreeing to sign the said
order.
Choose the correct answer using the codes
below.
a)
b)
c)
d)

1 only
2 and 3
1 and 3
3 only

13. Consider the following statements


about the Mahalwari system.
1. The charge of collecting the
revenue and paying it to the
Company was given to the village
headman, rather than the
zamindar.

a) 1 and 2
http://insightsonindia.com

INSIGHTS

Page 4

2. The revenue demand was to be


revised periodically, not
permanently fixed.
3. It was not applicable to the
Northern part of the country.

In the context of the statements above, which


of these is true?
a) A and R both are true, and R is the
correct explanation for A.
b) A and R both are true, and R is the
NOT the correct explanation for A.
c) A is correct, R is incorrect.
d) A and R both are incorrect.

Choose the correct answer using the codes


below.
a)
b)
c)
d)

1 and 2
2 and 3
1 and 3
All of the above

14. Ancient Indian thinkers had arrived at


an elaborate system of ideas on
motion. It was in a way similar to the
laws of motion. In these Indian
systems which of the following
concepts comes closes to the concept of
inertia?
a) The notion of Vega in the
Vaisheshika theory
b) The notion of abhighat in the
Sankhya theory
c) The notion of nodan in the Sankhya
theory
d) The notion of sanskara in
Vaisheshika theory

15. Consider the following statements.


Assertion (A): A body like a ring or a sphere
rolling without slipping over a horizontal
plane will suffer no friction, in principle.
Reason (R): At every instant, there is just one
point of contact between the body and the
plane and this point has no motion relative to
the plane.
http://insightsonindia.com

16. In the Indian Judiciary, which of the


following factors may work as
constraints over the High Court judges
in giving their rulings?
1. Precedents set by Supreme Court
2. Constitution of India
3. Instructions of the Executive to the
courts
Choose the correct answer using the codes
below.
a)
b)
c)
d)

1 and 2 only
2 and 3 only
1 and 3 only
All of the above

17. Consider the following statements.


1. The Subordinate courts in India
deal with both civil and criminal
cases.
2. The High courts can not issue writs
for restoring fundamental rights.
3. Lok Adalats do not have the
jurisdiction to deal with criminal
cases.
Choose the correct answer using the codes
below.

INSIGHTS

Page 5

a)
b)
c)
d)

1 and 2 only
2 and 3 only
1 and 3 only
All of the above

18. Which of the following factors do NOT


justify a strong central government in
the Indian federal structure?
1. Balanced regional development
2. Territorial integrity of India
3. Growing financial needs of states
4. Increasing volume and funds
allotted to Social sector schemes
Choose the correct answer using the codes
below.
a)
b)
c)
d)

1 and 3 only
1, 3 and 4 only
3 and 4 only
1, 2 and 4 only

19. Consider the following about the


advisory jurisdiction of the Supreme
Court (SC) under Article 143 of the
constitution.
1. Being a constitutional organ, the SC
is bound to give advice when it is
referred.
2. The SC being the highest court of
Justice, legal advice tendered by
the court is legally binding on the
President.

c) Both 1 and 2
d) None

20. Lingayats do not practice funerary


rites such as cremation prescribed in
the Dharmashastras because
a) as per them such practices show
the domination of brahanism
b) they believe on death a devotee
will be united with Shiva and not
return to this world
c) such practices were seen to be
impure by them which dilutes the
sacredness of human body
d) None of the above

21. Consider the following statements


about the Virashiava tradition in
Karnataka in the 12 th century
1. They questioned the theory of
rebirth.
2. They encouraged certain practices
disapproved in the
Dharmashastras such as widow
remarriage.
3. Wandering monks were revered in
their traditions.
Choose the correct answer using the codes
below.
a)
b)
c)
d)

Which of the above is/are true?

1 and 2
2 and 3
1 and 3
All of the above

a) 1 only
b) 2 only
http://insightsonindia.com

INSIGHTS

Page 6

22. Consider the following statements


about Film Certification Appellate
Tribunal (FCAT).
1. It is a statutory body.
2. It comes under the aegis of
Ministry of Information and
Broadcasting.
3. Any applicant can file an appeal
before the tribunal for a certificate
in respect of a film who is
aggrieved by an order of the
Central Board of Film Certification
(CBFC).
Choose the correct answer using the codes
below.
a)
b)
c)
d)

1 and 2
2 and 3
1 and 3
All of the above

23. Block mountains are created when


a) There is intense volcanic activity in
nearby regions consistently for
longer durations
b) Large areas are broken and
displaced vertically
c) Tectonic plates converge and
displace each other marginally
d) Downward erosion of high plateau
regions
24. If the periods and timings of summer
and winter seasons change for the
earth, it may lead to which of the
following changes in the long-run?
1. Climate Change
2. Biodiversity
3. Geomorphological features
http://insightsonindia.com

Choose the correct answer using the codes


below.
a)
b)
c)
d)

1 and 2 only
2 and 3 only
1 and 3 only
All of the above

25. Consider the following statements


about the interior of earth.
1. The continental mass of the crust is
made up of silica and magnesium.
2. Crust is thicker at the ocean floors.
3. The innermost core of the earth is
made only of molten iron.
Choose the correct answer using the codes
below.
a)
b)
c)
d)

1 and 2 only
2 and 3 only
1 and 3 only
None of the above

26. The growth of natural vegetation


depends on which of the following
factors apart from temperature and
moisture?
1. Slope
2. Thickness of Soil
3. Presence of weeds and other
vegetation around
Choose the correct answer using the codes
below.
a)
b)
c)
d)

INSIGHTS

1 and 2 only
2 and 3 only
1 and 3 only
All of the above

Page 7

27. Consider the following statements.


1. They are commonly found along
the eastern margin of continents in
the mid-latitudinal coastal regions.
2. They comprise both hard and soft
wood trees.
3. These regions experience a lot of
seasonal changes.
Which of the above match the description of
Temperate Evergreen forests?
a)
b)
c)
d)

1 and 2
2 and 3
1 and 3
All of the above

28. The higher concentration of dust


particles is found in subtropical and
temperate regions than equatorial and
polar regions due to
a) Dry winds in comparison to
equatorial and Polar Regions.
b) Moist winds in comparison to
equatorial and polar regions
c) More seasonal rainfall in
comparison to equatorial and polar
regions.
d) More even temperature ranges in
comparison to equatorial and polar
regions

a)
b)
c)
d)

Chambal
Gandak
Luni
Ramganga

30. Arrange the following space


expeditions of India in correct
chronological order.
1. Aryabhatta
2. Rohini
3. INSAT
4. GSAT
Choose the correct answer using the codes
below.
a)
b)
c)
d)

1234
2143
2134
1243

31. Which of the following can be termed


as major causes of biodiversity loss?
1. Habitat fragmentation
2. Co-extinction
3. Alien species invasion
4. Global warming
Choose the correct answer using the codes
below.
a)
b)
c)
d)

29. Consider the following.


1. The river is famous for its badland
topography called ravines.
2. It flows through Rajasthan finally
joining Yamuna.
3. It rises in the Malwa plateau.

1 and 2 only
3 and 4 only
1, 2 and 4 only
All of the above

The river with the above description is


http://insightsonindia.com

INSIGHTS

Page 8

32. Catalytic converted is used in


automobiles for reducing emission of
poisonous gases. It converts
a) Unburnt hydrocarbons into carbon
dioxide and water
b) Unburnt hydrocarbons into vapour
and nitrogen dioxide
c) Poisonous sulphur in the fuel into
sulphur dioxide
d) Poisonous lead in the fuel into lead
oxide and water

33. Consider the following statements


about Ziyarat.
1. It is Pilgrimage to the tomb of sufi
saints.
2. This practice is an occasion for
seeking the sufis spiritual grace
(barakat).
3. The practice was restricted to a
certain class of Muslims.
Choose the correct answer using the codes
below.
a)
b)
c)
d)

1 and 2
2 and 3
1 and 3
All of the above

34. Consider the following statements


about the ideas of Kabir.
1. He described the ultimate reality as
the formless one.
2. His idea of God being shunya
(emptiness) relates to that of yogic
traditions.
3. His poems often attack Hindu
polytheism and idol worship.
http://insightsonindia.com

Choose the correct answer using the codes


below.
a)
b)
c)
d)

1 and 2
2 and 3
1 and 3
All of the above

35. Which of the following factors led to


the decline of the Vijayanagar Empire?
1. Strain in the imperial structure
following the death of Krishnadeva
Rayas
2. Rebelliousness in military chiefs
3. Invasion of the Mughals
Choose the correct answer using the codes
below.
a)
b)
c)
d)

1 and 2
2 and 3
1 and 3
All of the above

36. Optical fibres used for communication


purposes are based on the principle of
a) Total internal reflection
b) Rectilinear propagation of light
c) Diffraction of light
d) Dispersion of light

37. Consider the following statements


about bio-magnification.
1. It refers to an increase in
concentration of the toxicant at
successive trophic levels.
2. It can happen only in aquatic
bodies.

INSIGHTS

Page 9

b) 2 and 3
c) 1 and 3
d) All of the above

3. Only those toxic substances that


cannot be metabolised or excreted
by the organism cause biomagnification.
Choose the correct answer using the codes
below.
a)
b)
c)
d)

1 and 2
2 and 3
1 and 3
All of the above

38. The relation between the Biological


Oxygen Demand (BOD) and pollution
level of water is
a) More the BOD , more polluted the
water will be of biologically
degradable material
b) More the BOD, less polluted the
water will be of biologically
degradable material
c) More the BOD, more polluted the
water will be of both biologically
non-degradable material
d) More the BOD, less polluted the
water will be of biologically nondegradable material

39. The Poona pact between Ambedkar


and Gandhi gave the scheduled castes
reserved seats in
1. Provincial legislative councils
2. Central Legislative Councils
3. Local body councils

40. Before cooperating in the British war


effort during Second World War, the
Congress put some basic condition(s)
against the British. These were
1. After the war, a constituent
assembly should be convened to
determine the political structure of
a free India.
2. Immediately some form of
responsible government should be
established at the Centre.
Which of the above is/are correct?
a)
b)
c)
d)

1 only
2 only
Both 1 and 2
None

41. According to Buddhist philosophy


1. The world is soulless.
2. In the earliest forms of Buddhism,
whether or not god existed was
irrelevant.
3. Buddha advocates penance to get
salvation.
Choose the correct answer using the codes
below.
a)
b)
c)
d)

Choose the correct answer using the codes


below.

1 and 2
2 and 3
1 and 3
All of the above

a) 1 and 2
http://insightsonindia.com

INSIGHTS

Page 10

42. Algal bloom can affect water bodies in


which of the following ways?
1. By depleting the oxygen
availability
2. Fish morbidity
3. Reducing the amount of sunlight
available to the water body
Choose the correct answer using the codes
below.
a)
b)
c)
d)

1 and 2
2 and 3
1 and 3
All of the above

43. Consider the following statements.

3. They can minimize soil erosion.


The technique of soil conservation with the
above features is
a)
b)
c)
d)

45. Consider the following matches of the


minerals with the regions they are
largely mined in North America.
1. Iron Canadian Shield
2. Coal Appalachian regions
3. Copper Western Cordilleras
Which of the above matches is/are correct?

Assertion (A): Gravity is always attractive,


while electromagnetic force can be attractive
or repulsive.
Reason (R): Matter is mostly electrically
neutral (net charge is zero). Thus, electric
force is largely zero and gravitational force
dominates terrestrial phenomena.
In the context of the statements above, which
of these is true?
a) A and R both are true, and R is
the correct explanation for A.
b) A and R both are true, and R is
the NOT the correct explanation
for A.
c) A is correct, R is incorrect.
d) A and R both are incorrect.
44. Consider the following.
1. These are made on the steep slopes
so that flat surfaces are available to
grow crops.
2. They can reduce surface run-off.
http://insightsonindia.com

Contour ploughing
Terrace farming
Shelter belts
Contour barriers

a)
b)
c)
d)

1 and 2
2 and 3
1 and 3
All of the above

46. The crop of Jute requires


1. Light rainfall
2. Alluvial Soil
3. High Temperature
4. Bright Sunshine at time of harvest
Which of the above match the description of
Temperate Evergreen forests?
a)
b)
c)
d)

1 and 2
2 and 3
2, 3 and 4
1, 3 and 4

47. Consider the following statements


about the 1857 revolt.
1. It did not spread to South India.

INSIGHTS

Page 11

2. It was a reaction to the British


reforms in the religious and social
sphere of Indians.
3. The revolt did not receive any royal
patronage.
4. The Government of India Act 1858
was passed as a reaction by the
British government to the revolt.
Choose the correct answer using the codes
below.
a)
b)
c)
d)

1 and 2 only
3 and 4 only
1, 2 and 4 only
All of the above

48. Consider the following statements


about Jainism.
1. Jainism believes that the entire
world is animate.
2. According to Jaina teachings, the
cycle of birth and rebirth is shaped
through karma.
3. Monastic existence is a necessary
condition of salvation.
Choose the correct answer using the codes
below.
a)
b)
c)
d)

1 and 2
2 and 3
1 and 3
All of the above

49. The action of the endogenic forces are


not uniform and thus the tectonically
controlled original crustal surface is
uneven. This can be attributed to
factors like
1. Variation in crustal thickness
2. Variation in geothermal gradients
http://insightsonindia.com

3. Volcanism in lithosphere
Choose the correct answer using the codes
below.
a)
b)
c)
d)

1 and 2
2 and 3
1 and 3
All of the above

50. Which of the following can be the


adverse consequences of El-Nino?
1. The distortion of equatorial
atmospheric circulation.
2. Irregularities in the evaporation of
sea water.
3. Reduction in the amount of
planktons which further reduces
the number of fish in the sea.
Choose the correct answer using the codes
below.
a)
b)
c)
d)

1 and 2
2 and 3
1 and 3
All of the above

51. Genetic Engineering Appraisal


Committee (GEAC) under the
Environment Ministry had permitted
field trials of GM food in India.
Despite that states can either allow or
ban GM crop trials in their states. This
is because
a) Genetic research and development
comes under List II (State List) of
seventh schedule of the
Constitution.
b) The GEAC order had inserted a
mandatory condition of separate

INSIGHTS

Page 12

permission from states for GM crop


trials.
c) No agricultural scheme or policy
can be launched in India without
the consent of the states.
d) None of the above
52. Which of the following articles
(provisions) of the Indian Constitution
provides a safeguard against the
excesses of the legislature and the
executive?
1. Article 13
2. Article 19
3. Article 21
4. Article 32
Choose the correct answer using the codes
below.
a)
b)
c)
d)

1 and 4 only
2 and 4 only
1, 2 and 3 only
All of the above

53. Consider the following statements.


1. The Judiciary cannot initiate the
process of constitutional
amendment but can effectively
change the Constitution by
interpreting it differently.
2. Elected representatives alone have
the power to amend the
Constitution.
Which of the above is true?
a)
b)
c)
d)

1 only
2 only
Both 1 and 2
None of the above

http://insightsonindia.com

54. Consider the following statements


about the proportional representation
system of election.
1. The entire country may be a single
constituency in this system.
2. Candidate who wins the election
gets majority of votes.
3. A party may get more seats than
votes in the legislature.
Choose the correct answer using the codes
below.
a)
b)
c)
d)

1 and 2 only
2 and 3 only
1 and 3 only
All of the above

55. Consider the following tasks.


1. Finalisation of the list of people
below poverty line (BPL) in the
village
2. Constructing check-dams and
tanks
3. Scrutinizing the accounts of the
funds allotted under major social
sector schemes
Which of the above can be done by the
authorization of Gram Sabha?
a)
b)
c)
d)

1 and 2
2 and 3
1 and 3
All of the above

56. India has agreed to provide fund of Rs


1500 crore to Nepals Pancheshwar
multi-purpose project on Mahakali
river. The major benefits to India from
this project will be

INSIGHTS

Page 13

b) No offer of discussing about a


future constitution was made
c) Congress was not invited to the
conference
d) The British government had invited
Dalit and Muslim leaders to the
conference which were against the
Congress definition of swaraj

1. The containment of Kheri floods in


Uttar Pradesh
2. Procuring additional electricity
produced by the project
3. Irrigation of the Terai regions in
India
Choose the correct answer using the codes
below.
a)
b)
c)
d)

1 and 2
2 and 3
1 and 3
All of the above

57. Union government has launched


a Heritage City Development and
Augmentation Yojana
(HRIDAY) scheme to preserve and
rejuvenate the rich cultural heritage of
the country. Which of the following
cities are covered in the initial phase of
the scheme?
1. Amritsar
2. Puri
3. Warangal
4. Badami
5. Mathura
Choose the correct answer using the codes
below.
a)
b)
c)
d)

1, 4 and 5 only
2 and 5 only
1, 2 and 3 only
All of the above

58. Indian National Congress leaders did


not attend the first round table
conference because
a) It was called to discuss a vague
dominion status for India
http://insightsonindia.com

59. The Salt march and the Civil


Disobedience movement raised which
of the following issues about salt in
India?
1. Government monopoly of salt
production
2. High tax on salt
3. Government restrictions on
obtaining salt only from sea water
Choose the correct answer using the codes
below.
a)
b)
c)
d)

1 and 2
2 and 3
1 and 3
All of the above

60. Consider the following statements


about the Light Combat Aircraft (LCA)
recently handed over to the Indian Air
Force.
1. It is the first indigenously built
LCA.
2. It has a tailless design.
3. It has been manufactured by a joint
venture between public and private
defence research organizations.
Choose the correct answer using the codes
below.
a) 1 and 2
INSIGHTS

Page 14

b) 2 and 3
c) 1 and 3
d) All of the above
61. Consider the following statements
with reference to the status of J&K in
the Indian federal structure.
1. The Central legislation in the Union
list and Concurrent list applies to
J&K only after the consent of the
state government.
2. The Directive Principles of State
Policy do not apply to the state.
3. Amendments to the Indian
Constitution (under Art. 368) can
only apply in concurrence with the
government of J&K.

63. The economic justification of subsidies


in agriculture is, at present, a hotly
debated question. Which of the
following arguments would justify
agricultural subsidies?
1. Farming in India continues to a
risky business.
2. Most farmers are very poor and
they will not be able to afford the
required inputs without
subsidies.
3. Eliminating subsidies will increase
the inequality between rich and
poor farmers and violate the goal
of equity.
Choose the correct answer using the codes
below.

Choose the correct answer using the codes


below.
a)
b)
c)
d)

1 and 2 only
2 and 3 only
1 and 3 only
All of the above

62. Under the 123 agreement signed


recently between India and USA
1. India has waived nuclear liability
for Companies from the US.
2. US has waived the requirement of
tracking nuclear material provided
by it to India.

a)
b)
c)
d)

64. Tweaking the Minimum Support


Prices (MSP) for water-intensive crops
in India will affect which of the
following?
1. Food inflation levels
2. Groundwater levels
3. Crop diversification
Choose the correct answer using the codes
below.

Which of the above is correct?


a)
b)
c)
d)

a)
b)
c)
d)

1 only
2 only
Both 1 and 2
None of the above

http://insightsonindia.com

1 and 2
2 and 3
1 and 3
All of the above

INSIGHTS

1 and 2
2 and 3
1 and 3
All of the above

Page 15

65. In the policy of liberalisation 1991,


deregulation of the Industrial sector
meant that
1. Controls on price fixation would be
relaxed.
2. Some of the goods reserved for
small scale industries could be
produced by bigger corporations
too.
3. Industrial licensing was made
more relaxed and flexible.
Choose the correct answer using the codes
below.
a)
b)
c)
d)

Choose the correct answer using the codes


below.
a)
b)
c)
d)

1 and 2
2 and 3
1 and 3
All of the above

66. Consider the following statements


about the Beti Bachao-Beti Padhao
campaign.
1. It will be implemented all over the
country.
2. It will target improving the
efficiency of delivery of welfare
services meant for women.
3. The state governments will not
have any role to play in the
campaign.
Choose the correct answer using the codes
below.
a)
b)
c)
d)

67. Which of the following factors may


possibly have been responsible for
growing agrarian distress in the
country for cotton farmers?
1. Opening up of domestic markets
due to globalization
2. High production costs yet low and
unstable yields
3. Decline in public investment in
agriculture in the last two decades

1 and 2
2 and 3
1 and 3
2 only

1 and 2
2 and 3
1 and 3
All of the above

68. The woman-oriented organization


Kudumbshree is
a) Large informal bank
b) A self-help group to promote
income generation activities
amongst women
c) A local informal agency started by
NABARD in Kerala
d) A pressure group of woman
activists for the cause of financial
inclusion of women

69. Which of the following can be the


benefits of organic farming to the
farmers and consumers?
1. Good returns on locally produced
organic inputs
2. Land conservation
3. Income generation through exports
Choose the correct answer using the codes
below.

http://insightsonindia.com

INSIGHTS

Page 16

a)
b)
c)
d)

b) 2 and 3
c) 2 only
d) All are major issues

1 and 2
2 and 3
1 and 3
All of the above

70. Consider the following statements.


Assertion (A): When it comes to regular
salaried employment, men are found to be
engaged in greater proportion than women. It
may be explained by the fact that
Reason (R): Since regular salaried jobs require
skills and a higher level of literacy, women
might not have been engaged to a great
extent.
In the context of the statements above, which
of these is true?
a) A and R both are true, and R is the
correct explanation for A.
b) A and R both are true, and R is the
NOT the correct explanation for A.
c) A is correct, R is incorrect.
d) A and R both are incorrect
71. Which of the following are NOT the
major issues facing the electricity
sector in India presently?
1. State Electricity boards are facing
huge financial losses.
2. Indias installed capacity is not
sufficient to generate electricity to
feed an annual economic growth
of around 8-9 per cent.
3. Thermal power plants are facing
large shortages of coal supply.
Choose the correct answer using the codes
below.
a) 1 only
http://insightsonindia.com

72. Currency notes and coins cannot be


refused by any citizen of the country
for settlement of any kind of
transaction. This is because currency
notes and coins are
a) Legal tenders
b) They have an intrinsic value
c) The government makes it
compulsory by law to accept
payment by cash mode.
d) It has purchasing power equivalent
to the amount printed on it
73. If the Statutory Liquidity Ratio (SLR) is
made zero, how will it affect the
banking system and public finances?
1. Government may find it difficult to
raise money from the market.
2. Banks will have more money to
lend.
3. Banks may be able to reduce
interest rates on their loans to
customers.
Choose the correct answer using the codes
below.
a)
b)
c)
d)

1 and 2
2 and 3
1 and 3
All of the above

74. Consider the following statements.


Assertion (A): If all the account-holders of all
commercial banks in the country want their
deposits back at the same time, the banks will
INSIGHTS

Page 17

not have enough means to satisfy the need of


every accountholder and there will be bank
failures.
Reason (R): The total amount of deposits held
by all commercial banks in the country is
much larger than the total size of their
reserves..
In the context of the statements above, which
of these is true?
a) A and R both are true, and R is the
correct explanation for A.
b) A and R both are true, and R is the
NOT the correct explanation for A.
c) A is correct, R is incorrect.
d) A and R both are incorrect.

75. Consider the following statements.


Assertion (A): A high bank rate reduces the
potential for credit creation in the economy.
Reason (R): A high bank rate encourages
banks to keep smaller (or greater) proportion
of their deposits as reserves, since borrowing
from RBI is now costlier than before.
In the context of the statements above, which
of these is true?
a) A and R both are true, and R is the
correct explanation for A.
b) A and R both are true, and R is the
NOT the correct explanation for A.
c) A is correct, R is incorrect.
d) A and R both are incorrect.

http://insightsonindia.com

76. In which of these regions, the


thickness of the weathering mantle
would be the greatest?
a) Taiga zone
b) Semi-desert regions
c) Tropical forest zone
d) Grasslands
77. Kilkari, a IVRS based application is
being piloted by Government of India
under which
a) Information on maternal and child
health care is being disseminated to
the needy.
b) Information is sent by Primary
health care centres to the
panchayats about maternal health
care
c) A mother can register her
grievances against the institutional
delivery facilities under public
health care centres.
d) None of the above
78. In the state government, the Governor
exercises more discretion that the
President can exercise in the case of
Central government. This is because
a) Indian federalism provides for a
strong central government, and
comparatively weaker states.
b) The constitutional provisions limit
the overall discretion of the
President, which is not so clearly
defined for the Governor.
c) The Governor derives greater
authority from the orders of the
Central government with respect to
the states whereas no such system
exists for the President.
INSIGHTS

Page 18

d) The state politics is much more


vulnerable to Governors discretion
that the central politics is to
Presidents discretion.
79. Which of the following regions in
India comes under the Very High
damage risk zone for tropical
cyclones?
1. Eastern J&K
2. Western Kerala
3. Western coasts of Gujarat
4. Northern Uttar Pradesh

81. Which of the following are non-debt


creating capital receipts of the
Government of India?
1. Disinvestment in PSUs
2. Interest payments received on
loans previously forwarded by
Government of India
3. Borrowing from RBI
Choose the correct answer using the codes
below.
a)
b)
c)
d)

Choose the correct answer using the codes


below.
a)
b)
c)
d)

All of the above


1 and 3 only
2 and 4 only
3 only

80. In winter months, the weather


conditions over India are generally
influenced by
1. Distribution of pressure in Central
Asia
2. Rainfall in Mediterranean region
3. Tibetan highlands acting as a
barrier to jet streams
Choose the correct answer using the codes
below.
a)
b)
c)
d)

1 and 2
2 and 3
1 and 3
All of the above

82. Primary deficit in annual budget


(public finances) in India is the
a) Difference between fiscal deficit
and net interest liabilities
b) Difference between revenue deficit
and net interest liabilities
c) Difference between fiscal deficit
and budgetary deficit
d) Difference between revenue deficit
and budgetary deficit
83. Which of the following actors
participate in the Foreign Exchange
market?
1. Monetary authorities
2. Commercial banks
3. Sovereign governments
Choose the correct answer using the codes
below.
a)
b)
c)
d)

http://insightsonindia.com

1 and 2
2 and 3
1 and 3
1 only

INSIGHTS

1 and 2
2 and 3
1 and 3
All of the above
Page 19

84. Consider the following statements


about the Harappan civilization.
1. They practiced agriculture.
2. Most Harappan sites are located in
semi-arid lands.
3. There is evidence of use of water
storage structures.
Choose the correct answer using the codes
below.
a)
b)
c)
d)

1 and 2
2 and 3
1 and 3
All of the above

85. Consider the following statements.


1. Both Mahavira and the Gautam
Buddha questioned the authority of
the Vedas.
2. None of the Buddhas speeches
were written down during his
lifetime.
3. The basic philosophy of the Jainas
was already in existence in north
India before the birth of
Mahavira.
Choose the correct answer using the codes
below.
a)
b)
c)
d)

1 and 2
2 and 3
1 and 3
All of the above

86. Which of the following regions in the


World is NOT a bio-diversity hotspot?
1. Sri Lanka
2. Indo-Burma Region
3. Eastern European region
4. New Zealand
http://insightsonindia.com

Choose the correct answer using the codes


below.
a) 1 and 3 only
b) 2 and 3 only
c) 4 only
d) 3 only
87. There are many factors, other than
income and assets, which are
associated with poverty. These can be
1. Accessibility to basic education
2. Health care
3. Drinking water
4. Sanitation
Choose the correct answer using the codes
below.
a) 1 and 2 only
b) 2 and 3 only
c) 1, 3 and 4 only
d) All of the above
88. The High Level Committee (HLC) for
Restructuring of Food Corporation of
India (FCI) chaired by Shanta
Kumar has submitted its report to the
Prime Minister. The major
recommendations include
1. Outsourcing stocking operations of
FCI to private sector
2. FCI should hand over all
procurement operations of wheat,
paddy and rice to Andhra Pradesh,
Chhattisgarh, Haryana, Madhya
Pradesh, Odisha and Punjab .
3. Defer implementation of National
Food Security Act (NFSA) in states
that have not done end to end
computerization.
Choose the correct answer using the codes
below.
INSIGHTS

Page 20

a)
b)
c)
d)

1 and 2
2 and 3
1 and 3
All of the above

89. Consider the following statements


about the Hydroponics technique.
Which of these is NOT true?
1. Nutrients enter through the roots
of plants in this technique.
2. The technique cannot be used for
commercial production of
vegetables, fruits etc.
3. The nutrient solution used is
unaerated.
Choose the correct answer using the codes
below.
a)
b)
c)
d)

1 and 2
2 and 3
3 only
All of the above

90. Consider the following statements


about Plant Growth Regulators
(PGRs).
1. They play an important role in the
flowering and seed formation also
with plant growth.
2. They can help the plant in healing
with wounds.
3. Auxin and Cytokinin are widely
used PGRs.
Choose the correct answer using the codes
below.
a)
b)
c)
d)

1 and 2 only
2 and 3 only
1 and 3 only
All of the above

http://insightsonindia.com

91. Innate Immunity against non-specific


diseases in human beings can consist
of which of the following?
1. Tears in eyes
2. Acid in the Stomach
3. Saliva in the mouth
4. Facial hairs
Choose the correct answer using the codes
below.
a) 1 and 4 only
b) 2 and 3 only
c) 1, 2 and 3 only
d) 1 and 3 only
92. Francois Bernier was a traveller to
India in the 17th Century who wrote
detailed accounts of Indias social
customs and religious practices. As per
Bernier
1. Indias trading system was outdated.
2. In comparison to Europe, situation
in India was bleak.
3. Europeans were qualified enough
to rule India.
Choose the correct answer using the codes
below.
a)
b)
c)
d)

1 and 2
2 and 3
1 and 3
2 only

93. Consider the following statements


about Alvars and Nayanars.
1. They used Tamil for promoting
their messages and singing hymns
in praise of God.

INSIGHTS

Page 21

c) Organizing freedom struggle in


only the western regions of the
country
d) Getting political power and
entering the councils to break the
British political structure from
within

2. They did not believe in caste based


discrimination then prevailing in
the Hindu society.
3. They later merged with the
Vaishnavites and Shaivites.
Choose the correct answer using the codes
below.
a)
b)
c)
d)

1 and 2
2 and 3
1 and 3
2 only

94. The Health Ministry has introduced


web-based name based tracking
system called Mother & Child
Tracking System (MCTS) across all the
States and UTs. Consider the following
statements about it.
1. Immunization details of the child
are tracked.
2. The system captures personal
details of both the mother and child
up to five years of age.
3. The system ensures that the child
also gets proper adolescent care.
Choose the correct answer using the codes
below.
a)
b)
c)
d)

1 and 2
2 and 3
1 and 3
All of the above

95. The Home Rule league of Tilak aimed


at
a) a reform of the British
administration
b) Overthrowing the British
administration
http://insightsonindia.com

96. Which of the following points of


differences between the moderates and
extremists led to the Surat split?
1. Moderates were not in favour of
extending the boycott movement
beyond Bengal.
2. Moderates were totally opposed to
the boycott of councils, extremists
were not.
3. Different meanings of swaraj
Choose the correct answer using the codes
below.
a) 1 and 2
b) 2 and 3
c) 1 and 3
d) All of the above
97. In 1917, Gandhi organised a
satyagraha to support the peasants of
the Kheda district of Gujarat as they
were affected by
1. Crop failure
2. Plague epidemic
3. Inability to pay revenue to the
British government
Choose the correct answer using the codes
below.
a)
b)
c)
d)
INSIGHTS

1 and 2
2 and 3
1 and 3
All of the above
Page 22

98. The British had enacted the Rowlatt


Act and implemented it nationwide in
1919. Gandhiji decided to launch a
nationwide satyagraha against the act
because the Act provided for
1. Ban on political activities
2. Unreasonable detention of political
prisoners without trial
3. Allowed the government to ban all
the local language newspapers and
radio
Choose the correct answer using the codes
below.
a)
b)
c)
d)

1 and 2
2 and 3
1 and 3
All of the above

99. The belief of Dalit leaders during the


Freedom movement that Political
empowerment would resolve the
problems of their social disabilities
resulted in which of the following
demands?
1. Separate electorates
2. Reserved seats in education
institutions

http://insightsonindia.com

3. Organising dalits in major national


associations by dalit leaders
Choose the correct answer using the codes
below.
a)
b)
c)
d)

1 and 2
2 and 3
1 and 3
All of the above

100.
As per Mahanaya Buddhism,
Bodhisattvas were those who
a) Worked out their salvation with
their own efforts
b) Accumulated merit through their
efforts but did not attain salvation
c) Accumulated merit through their
efforts but did not attain salvation
to help others
d) None of the above

INSIGHTS

Page 23

Insights Mock Tests 2015: Test 13 Solutions


1. Solution: a)
Microorganisms break down detritus into smaller particles. This process is called
fragmentation. By the process of leaching, watersoluble inorganic nutrients go down
into the soil horizon and get precipitated as unavailable salts. Bacterial and fungal
enzymes degrade detritus into simpler inorganic substances. This process is called as
catabolism. It is important to note that all the above steps in decomposition operate
simultaneously on the detritus. Humification and mineralisation occur during
decomposition in the soil.

2. Solution: d)
When breeding is between animals of the same breed it is called inbreeding, while
crosses between different breeds are called outbreeding. Out-breeding is the
breeding of the unrelated animals, which may be between individuals of the same
breed but having no common ancestors for 4-6 generations (out-crossing) or between
different breeds (cross-breeding) or different species (inter-specific hybridisation).
Controlled breeding experiments are carried out using artificial insemination. The
semen is collected from the male that is chosen as a parent and injected into the
reproductive tract of the selected female by the breeder.

3. Solution: d)
More than 840 million people in the world do not have adequate food to meet their
daily food and nutritional requirements. A far greater number three billion people
suffer from micronutrient, protein and vitamin deficiencies or hidden hunger
because they cannot afford to buy enough fruits, vegetables, legumes, fish and meat.
Diets lacking essential micronutrients particularly iron, vitamin A, iodine and zinc
increase the risk for disease, reduce lifespan and reduce mental abilities.

4. Solution: d)
Ecologists and evolutionary biologists have proposed various hypotheses; some
important ones are
(a) Speciation is generally a function of time, unlike temperate regions subjected to
frequent glaciations in the past, tropical latitudes have remained relatively

http:www.insightsonindia.com

Insights

Page 1

Insights Mock Tests 2015: Test 13 Solutions


undisturbed for millions of years and thus, had a long evolutionary time for species
diversification,
(b) Tropical environments, unlike temperate ones, are less seasonal, relatively more
constant and predictable. Such constant environments promote niche specialisation
and lead to a greater species diversity and
(c) There is more solar energy available in the tropics, which contributes to higher
productivity; this in turn might contribute indirectly to greater diversity.

5. Solution: d)
http://en.wikipedia.org/wiki/Biological_pest_control#Micro-organisms
This method of controlling pests that relies on natural predation rather than
introduced chemicals. A key belief of the organic farmer is that biodiversity furthers
health. The more variety a landscape has, the more sustainable it is. The organic
farmer, therefore, works to create a system where the insects that are sometimes
called pests are not eradicated, but instead are kept at manageable levels by a
complex system of checks and balances within a living and vibrant ecosystem.

6. Solution: d)
A constitution is not a frozen and unalterable document. It is a document made by
human beings and may need revisions, changes and re-examination.
It is true that the constitution reflects the dreams and aspirations of the concerned
society. It must also be kept in mind that the constitution is a framework for the
democratic governance of the society. In this sense, it is an instrument that societies
create for themselves.
For e.g. Fundamental rights clearly establish a secular, equitable, just and nonexploitative social fabric in India.
Fundamental duties provide a secular, fraternistic and a patriotic aspect to the
Indian citizens by making them respect the constitution, national flag, asking them
to render service to the nation and mankind etc.
DPSP does so by providing for a more equitable work culture in men and women;
minimizing inequalities in social and economic status; establishing the ideal of a
welfare society etc.

http:www.insightsonindia.com

Insights

Page 2

Insights Mock Tests 2015: Test 13 Solutions


7. Solution: d)
http://www.downtoearth.org.in/content/india-s-move-curb-antibiotic-uselivestock-poultry-may-hit-imports-us

8. Solution: a)
http://www.thehindu.com/sci-tech/science/nasa-launches-earthobserving-soilmoisture-active-passive-satellite/article6844993.ece

9. Solution: d)
Citizenship clauses need a simple majority only in the Parliament.
The same is true for changing the boundary of states under Article 3; and for
changing the number of judges in the SC.

10. Solution: c)

http:www.insightsonindia.com

Insights

Page 3

Insights Mock Tests 2015: Test 13 Solutions


11. Solution: b)
The Constitution of India vests the executive power of the Union formally in the
President. In reality, the President exercises these powers through the Council of
Ministers headed by the Prime Minister. The President is elected for a period of five
years. But there is no direct election by the people for the office of President. The
President is elected indirectly. This means that the president is elected not by the
ordinary citizens but by the elected MLAs and MPs. This election takes place in
accordance with the principle of proportional representation with single transferable
vote.
The President can be removed from office only by Parliament by following the
procedure for impeachment. This procedure requires a special majority as explained
in the last chapter. The only ground for impeachment is violation of the
Constitution.
As per Article 74 (1): There shall be a Council of Ministers with the Prime Minister at
the head to aid and advise the President who shall in the exercise of his functions,
act in accordance with such advice. Provided that the President may require the
Council of Ministers to reconsider such advice.., and the President shall act in
accordance with the advice tendered after such reconsideration.
So he need not accept the advice if placed before him for the first time.

12. Solution: d)
Here again Article 74(1) comes in the picture. In view of the controversy about the
scope of the Presidents powers, a specific mention was made in the Constitution by
an amendment that the advice of the Council of Ministers will be binding on the
President. By another amendment made later, it was decided that the President can
ask the Council of Ministers to reconsider its advice but, has to accept the
reconsidered advice of the Council of Ministers (the PM representing them in this
case).

13. Solution: a)
In the North Western Provinces of the Bengal Presidency (most of this area is now in
Uttar Pradesh), an Englishman called Holt Mackenzie devised the new system which
came into effect in 1822. He felt that the village was an important social institution in
north Indian society and needed to be preserved. Under his directions, collectors
went from village to village, inspecting the land, measuring the fields, and recording
http:www.insightsonindia.com

Insights

Page 4

Insights Mock Tests 2015: Test 13 Solutions


the customs and rights of different groups. The estimated revenue of each plot
within a village was added up to calculate the revenue that each village (mahal) had
to pay. This demand was to be revised periodically, not permanently fixed. The
charge of collecting the revenue and paying it to the Company was given to the
village headman, rather than the zamindar. This system came to be known as the
mahalwari settlement.

14. Solution: a)

15. Solution: a)
A body like a ring or a sphere rolling without slipping over a horizontal plane will
suffer no friction, in principle. At every instant, there is just one point of contact
between the body and the plane and this point has no motion relative to the plane. In
this ideal situation, kinetic or static friction is zero and the body should continue to
roll with constant velocity. We know, in practice, this will not happen and some
resistance to motion (rolling friction) does occur, i.e. to keep the body rolling, some
applied force is needed. For the same weight, rolling friction is much smaller (even
by 2 or 3 orders of magnitude) than static or sliding friction.

16. Solution: a)
http:www.insightsonindia.com

Insights

Page 5

Insights Mock Tests 2015: Test 13 Solutions


A high court can not go against previous rulings of the SC which set a precedent. But
the SC and HCs can go against their own respective precedents. For e.g. the SC
changed its stance on constitutional amendment in Kesavananda Bharati case from
what it was in the Golaknath case.
But the same could not be done by the HC against an SC ruling.
The executive can not issue any instruction to the courts in India.

17. Solution: a)
http://apslsa.ap.nic.in/local_adalat_mechanism.html

18. Solution: c)
Besides the concern for unity, the makers of the Constitution also believed that the
socio-economic problems of the country needed to be handled by a strong central
government in cooperation with the States. Poverty, illiteracy and inequalities of
wealth were some of the problems that required planning and coordination. Thus,
the concerns for unity and development prompted the makers of the Constitution to
create a strong central government.
The central government has very effective financial powers and responsibilities. In
the first place, items generating revenue are under the control of the central
government. Thus, the central government has many revenue sources and the States
are mostly dependent on the grants and financial assistance from the centre.
Secondly, India adopted planning as the instrument of rapid economic progress and
development after independence. Planning led to considerable centralisation of
economic decision making.
But as the needs of states grow, a central control will become obsolete. Instead
Indian federalism will have to give more space and autonomy for the states.

19. Solution: d)
http:www.insightsonindia.com

Insights

Page 6

Insights Mock Tests 2015: Test 13 Solutions


In addition to original and appellate jurisdiction, the Supreme Court of India
possesses advisory jurisdiction also. This means that the President of India can refer
any matter that is of public importance or that which involves interpretation of
Constitution to Supreme Court for advice. However, the Supreme Court is not
bound to give advice on such matters and the President is not bound to accept such
an advice.
What then is the utility of the advisory powers of the Supreme Court? The utility is
two-fold. In the first place, it allows the government to seek legal opinion on a
matter of importance before taking action on it. This may prevent unnecessary
litigations later. Secondly, in the light of the advice of the Supreme Court, the
government can make suitable changes in its action or legislations.

20. Solution: b)
Lingayats continue to be an important community in the region to date. They
worship Shiva in his manifestation as a linga, and men usually wear a small linga in a
silver case on a loop strung over the left shoulder. Those who are revered include the
jangama or wandering monks. Lingayats believe that on death the devotee will be
united with Shiva and will not return to this world. Therefore they do not practise
funerary rites such as cremation, prescribed in the Dharmashastras. Instead, they
ceremonially bury their dead.

21. Solution: d)
The twelfth century witnessed the emergence of a new movement in Karnataka, led
by a Brahmana named Basavanna (1106-68) who was initially a Jaina and a minister
in the court of a Chalukya king. His followers were known as Virashaivas (heroes of
Shiva) or Lingayats (wearers of the linga).
The Lingayats challenged the idea of caste and the pollution attributed to certain
groups by Brahmanas. They also questioned the theory of rebirth. These won them
followers amongst those who were marginalised within the Brahmanical social
order. The Lingayats also encouraged certain practices disapproved in the
Dharmashastras, such as post-puberty marriage and the remarriage of widows. Our
understanding of the Virashaiva tradition is derived from vachanas (literally, sayings)
composed in Kannada by women and men who joined the movement.

22. Solution: d)
http:www.insightsonindia.com

Insights

Page 7

Insights Mock Tests 2015: Test 13 Solutions


Chairperson of Central Board of Film Certification (CBFC) Leela Samson has
resigned.
She resigned after Film Certification Appellate Tribunal (FCAT) cleared the
controversial film Messenger of God featuring Dera Saccha Sauda chief Gurmeet
Ram Rahim Singh in lead role.
The Censor Board headed by Ms. Samson had earlier denied the clearance certificate
to the film saying that the movie depicts Gurmeet Ram Rahim Singh as a God.
Film Certification Appellate Tribunal (FCAT)

FCAT is a statutory body. It has been constituted under the provisions of the
Cinematograph Act, 1952.

It comes under the aegis of Ministry of Information and Broadcasting.

Function: It hears the appeals filed as per provisions of Cinematograph Act.


Any applicant can file an appeal before the tribunal for a certificate in respect
of a film who is aggrieved by an order of the Central Board of Film
Certification (CBFC).

23. Solution: b)
There are three types of mountains- Fold Mountains, Block Mountains and the
Volcanic Mountains.
Block Mountains are created when large areas are broken and displaced vertically.
The uplifted blocks are termed as horsts and the lowered blocks are called graben.
The Rhine valley and the Vosges mountain in Europe are examples of such
mountain systems. Locate them on the world map in the atlas and find out some
more examples of this type of landforms.
Volcanic mountains are formed due to volcanic activity. Mt.Kilimanjaro in Africa
and Mt.Fujiyama in Japan are examples of such mountains.

24. Solution: d)

http:www.insightsonindia.com

Insights

Page 8

Insights Mock Tests 2015: Test 13 Solutions


Climate is made up of the essentially two elements temperature and moisture. Both
will chang if the duration of summers and winters will change.
Based on the above, the species which can not successfully adapt will become
extinct.
And finally higher temperatures, greater or lesser rainfall will also affect the land
features, river systems, erosion etc. leading to a change in geomorphological
features.

25. Solution: d)
Just like an onion, the earth is made up of several concentric layers with one inside
another. The uppermost layer over the earths surface is called the crust. It is the
thinnest of all the layers. It is about 35 km. on the continental masses and only 5 km
on the ocean floors.
The main mineral constituents of the continental mass are silica and alumina. It is
thus called sial (si-silica and al-alumina). The oceanic crust mainly consists of silica
and magnesium; it is therefore called sima (si-silica and ma-magnesium).
Just beneath the crust is the mantle which extends up to a depth of 2900 km. below
the crust.
The innermost layer is the core with a radius of about 3500 km. It is mainly made up
of nickel and iron and is called nife (ni nickel and fe ferrous i.e. iron). The central
core has very high temperature and pressure.

26. Solution: d)
With the change in height, the climate changes and that changes natural vegetation.
The growth of vegetation depends on temperature and moisture. It also depends on
factors like slope and thickness of soil.
The type and thickness of natural vegetation varies from place to place because of
the variation in these factors.
The competition for sunlight, water and other nutrients from soil affects the growth
of plants in the surrounding region. The more plants there are with limited
resources, the limited growth there will be unless there is some symbiotic relation
between those plants.

http:www.insightsonindia.com

Insights

Page 9

Insights Mock Tests 2015: Test 13 Solutions

27. Solution: a)
The temperate evergreen forests are located in the midlatitudinal coastal region.
They are commonly found along the eastern margin of the continents, e.g., In south
east USA, South China and in South East Brazil. They comprise both hard and soft
wood trees like oak, pine, eucalyptus, etc.
Tropical deciduous are the monsoon forests found in the large part of India,northern
Australia and in central America. These regions experience seasonal changes.

28. Solution: a)
Atmosphere has a sufficient capacity to keep small solid particles, which may
originate from different sources and include sea salts, fine soil, smoke-soot, ash,
pollen, dust and disintegrated particles of meteors. Dust particles are generally
concentrated in the lower layers of the atmosphere; yet, convectional air currents
may transport them to great heights. The higher concentration of dust particles is
found in subtropical and temperate regions due to dry winds in comparison to
equatorial and polar regions. Dust and salt particles act as hygroscopic nuclei
around which water vapour condenses to produce clouds.

29. Solution: a)
The Yamuna, the western most and the longest tributary of the Ganga, has its source
in the Yamunotri glacier on the western slopes of Banderpunch range (6,316 km). It
joins the Ganga at Prayag (Allahabad). It is joined by the Chambal, the Sind, the
Betwa and the Ken on its right bank which originates from the Peninsular plateau
while the Hindan, the Rind, the Sengar, the Varuna, etc. join it on its left bank.
The Chambal rises near Mhow in the Malwa plateau of Madhya Pradesh and flows
northwards through a gorge up wards of Kotain Rajasthan, where the Gandhisagar
dam has been constructed. From Kota, it traverses down to Bundi, Sawai Madhopur
and Dholpur, and finally joins the Yamuna. The Chambal is famous for its badland
topography called the Chambal ravines.

30. Solution: a)

http:www.insightsonindia.com

Insights

Page 10

Insights Mock Tests 2015: Test 13 Solutions

31. Solution: d)
Section 15.1.4 explains the case elaborately with many examples. Refer to the 12 th
Biology Chapter 15 NCERT.

32. Solution: a)
Catalytic converters, having expensive metals namely platinum-palladium and
rhodium as the catalysts, are fitted into automobiles for reducing emission of
poisonous gases. As the exhaust passes through the catalytic converter, unburnt
hydrocarbons are converted into carbon dioxide and water, and carbon monoxide
and nitric oxide are changed to carbon dioxide and nitrogen gas, respectively. Motor
vehicles equipped with catalytic converter should use unleaded petrol because lead
in the petrol inactivates the catalyst.

33. Solution: a)
Pilgrimage, called ziyarat, to tombs of sufi saints is prevalent all over the Muslim
world. This practice is an occasion for seeking the sufis spiritual grace (barakat). For
more than seven centuries people of various creeds, classes and social backgrounds
have expressed their devotion at the dargahs of the five great Chishti saints. Amongst
these, the most revered shrine is that of Khwaja Muinuddin, popularly known as
Gharib Nawaz (comforter of the poor).

34. Solution: d)
http:www.insightsonindia.com

Insights

Page 11

Insights Mock Tests 2015: Test 13 Solutions


Kabirs poems have survived in several languages and dialects; and some are
composed in the special language of nirguna poets, the sant bhasha.
These hint at the difficulties of capturing the nature of the Ultimate Reality in words:
expressions such as the lotus which blooms without flower or the fire raging in
the ocean convey a sense of Kabirs mystical experiences.
Also striking is the range of traditions Kabir drew on to describe the Ultimate
Reality. These include Islam: he described the Ultimate Reality as Allah, Khuda,
Hazrat and Pir. He also used terms drawn from Vedantic traditions, alakh (the
unseen), nirakar (formless), Brahman, Atman, etc. Other terms with mystical
connotations such as shabda (sound) or shunya (emptiness) were drawn from yogic
traditions.
Diverse and sometimes conflicting ideas are expressed in these poems. Some poems
draw on Islamic ideas and use monotheism and iconoclasm to attack Hindu
polytheism and idol worship; others use the sufi concept of zikr and ishq (love) to
express the Hindu practice of nam-simaran (remembrance of Gods name).

35. Solution: a)
Strain began to show within the imperial structure following Krishnadeva Rayas
death in 1529. His successors were troubled by rebellious nayakas or military chiefs.
By 1542 control at the centre had shifted to another ruling lineage, that of the
Aravidu, which remained in power till the end of the seventeenth century. During
this period, as indeed earlier, the military ambitions of the rulers of Vijayanagara as
well as those of the Deccan Sultanates resulted in shifting alignments.
Eventually this led to an alliance of the Sultanates against Vijayanagara. In 1565
Rama Raya, the chief minister of Vijayanagara, led the army into battle at RakshasiTangadi (also known as Talikota), where his forces were routed by the combined
armies of Bijapur, Ahmadnagar and Golconda. The victorious armies sacked the city
of Vijayanagara. The city was totally abandoned within a few years.

36. Solution: a)
Some important devices and their respective principles are listed below.

http:www.insightsonindia.com

Insights

Page 12

Insights Mock Tests 2015: Test 13 Solutions

37. Solution: c)
A few toxic substances, often present in industrial waste waters, can undergo
biological magnification (Biomagnification) in the aquatic food chain.
Biomagnification refers to increase in concentration of the toxicant at successive
trophic levels. This happens because a toxic substance accumulated by an organism
cannot be metabolised or excreted, and is thus passed on to the next higher trophic
level. This phenomenon is well-known for mercury and DDT.

38. Solution: a)
Biochemical oxygen demand (BOD) is the amount of dissolved oxygen needed by
aerobic biological organisms in a body of water to break down organic material
present in a given water sample at certain temperature over a specific time period. It
is possible to estimate the amount of biodegradable organic matter in sewage water
by measuring Biochemical Oxygen Demand (BOD). Higher the BOD will be, higher
the biodegradable material will be there in the water.

39. Solution: a)
Ambedkar clashed with Mahatma Gandhi at the second Round Table Conference by
demanding separate electorates for dalits. When the British government conceded
Ambedkars demand, Gandhiji began a fast unto death. He believed that separate
electorates for dalits would slow down the process of their integration into society.
Ambedkar ultimately accepted Gandhijis position and the result was the Poona Pact
of September 1932. It gave the Depressed Classes (later to be known as the Schedule
http:www.insightsonindia.com

Insights

Page 13

Insights Mock Tests 2015: Test 13 Solutions


Castes) reserved seats in provincial and central legislative councils, but they were to
be voted in by the general electorate. The dalit movement, however, continued to be
apprehensive of the Congress-led national movement.

40. Solution: c)
The governments response was negative. Instead the government said that these
demands may be a part of some future arrangement. It only decided to setup to a
consultative committee to discuss the issue.

41. Solution: d)
According to Buddhist philosophy, the world is transient (anicca) and constantly
changing; it is also soulless (anatta) as there is nothing permanent or eternal in it.
Within this transient world, sorrow (dukkha) is intrinsic to human existence. It is by
following the path of moderation between severe penance and self-indulgence that
human beings can rise above these worldly troubles. In the earliest forms of
Buddhism, whether or not god existed was irrelevant.

42. Solution: d)
Micro-organisms involved in biodegradation of organic matter in the receiving
water body consume a lot of oxygen, and as a result there is a sharp decline in
dissolved oxygen downstream from the point of sewage discharge. This causes
mortality of fish and other aquatic creatures.
Presence of large amounts of nutrients in waters also causes excessive growth of
planktonic (free-floating) algae, called an algal bloom which imparts a distinct colour
to the water bodies. Algal blooms cause deterioration of the water quality and fish
mortality. Some bloom-forming algae are extremely toxic to human beings and
animals.

43. Solution: a)
Gravity is always attractive, while electromagnetic force can be attractive or
repulsive. Another way of putting it is that mass comes only in one variety (there is
no negative mass), but charge comes in two varieties : positive and negative charge.
This is what makes all the difference. Matter is mostly electrically neutral (net charge
http:www.insightsonindia.com

Insights

Page 14

Insights Mock Tests 2015: Test 13 Solutions


is zero). Thus, electric force is largely zero and gravitational force dominates
terrestrial phenomena. Electric force manifests itself in atmosphere where the atoms
are ionised and that leads to lightning.

44. Solution: b)
You already know the features of Terrace farming as mentioned in the question.
Others are:
Contour barriers: Stones, grass, soil are used to build barriers along contours.
Trenches are made in front of the barriers to collect water.
Contour ploughing: Ploughing parallel to the contours of a hill slope to form a natural
barrier for water to flow down the slope.
Shelter belts: In the coastal and dry regions, rows of trees are planted to check the
wind movement to protect soil cover.

45. Solution: d)
The mineral deposits in North America are located in three zones: the Canadian
region north of the Great Lakes, the Appalachian region and the mountain ranges of
the west. Iron ore, nickel, gold, uranium and copper are mined in the Canadian
Shield Region, coal in the Appalachians region. Western Cordilleras have vast
deposits of copper, lead, zinc, gold and silver.

46. Solution: b)
Jute was also known as the Golden Fibre. It grows well on alluvial soil and requires
high temperature, heavy rainfall and humid climate. This crop is grown in the
tropical areas. India and Bangladesh are the leading producers of jute.
For cotton, bright sunshine is needed. For tea, a well-distributed rainfall is needed.

47. Solution: c)
The movement was confined to the Northern and Central India.

http:www.insightsonindia.com

Insights

Page 15

Insights Mock Tests 2015: Test 13 Solutions


The revolt was patronised by Mughal king Bahadurshah Jafar. Immediately after the
revolt, to curb such tendencies later on, the British government enacted the GoI Act,
1935.

48. Solution: d)
The most important idea in Jainism is that the entire world is animated: even stones,
rocks and water have life. Non-injury to living beings, especially to humans,
animals, plants and insects, is central to Jaina philosophy. In fact the principle of
ahimsa, emphasised within Jainism, has left its mark on Indian thinking as a whole.
According to Jaina teachings, the cycle of birth and rebirth is shaped through karma.
Asceticism and penance are required to free oneself from the cycle of karma. This
can be achieved only by renouncing the world; therefore, monastic existence is a
necessary condition of salvation. Jaina monks and nuns took five vows: to abstain
from killing, stealing and lying; to observe celibacy; and to abstain from possessing
property.

49. Solution: d)
The energy emanating from within the earth is the main force behind endogenic
geomorphic processes. This energy is mostly generated by radioactivity, rotational
and tidal friction and primordial heat from the origin of the earth.
This energy due to geothermal gradients and heat flow from within induces
diastrophism and volcanism in the lithosphere. Due to variations in geothermal
gradients and heat flow from within, crustal thickness and strength, the action of
endogenic forces are not uniform and hence the tectonically controlled original
crustal surface is uneven.

50. Solution: d)
EI-Nino is a complex weather system that appears once every three to seven years,
bringing drought, floods and other weather extremes to different parts of the world.
The system involves oceanic and atmospheric phenomena with the appearance of
warm currents off the coast of Peru in the Eastern Pacific and affects weather in
many places including India. EI-Nino is merely an extension of the warm equatorial
current which gets replaced temporarily by cold Peruvian current or Humbolt
current (locate these currents in your atlas). This current increases the temperature of
water on the Peruvian coast by 10C. This results in: (i) the distortion of equatorial
http:www.insightsonindia.com

Insights

Page 16

Insights Mock Tests 2015: Test 13 Solutions


atmospheric circulation; (ii) irregularities in the evaporation of sea water; (iii)
reduction in the amount of planktons which further reduces the number of fish in
the sea.
EI-Nino is used in India for forecasting long range monsoon rainfall. In 1990-91,
there was a wild EI-Nino event and the onset of southwest monsoon was delayed
over most parts of the country ranging from five to twelve days.

51. Solution: b)
Policies like RKVY, Seed missions, technology missions have been launched without
the consent of states. However, any major legal change will require the consent of
states.
Recently, Maharashtra government has given nod to confined genetically modified
(GM) food trails in state.
In this regard, state government has granted no-objection certificates (NOC) for open
field trials of 5 genetically modified (GM) crops. They are rice, chana (chickpeas),
maize, brinjal and cotton.

52. Solution: d)
Article 13 gives Judiciary the power to review legislative acts and administrative
rules.
Article 19 gives citizens the freedom of free speech, expression etc. which can be
used to expose the wrongdoings of the government.
Article 21 safeguards the lives of the citizens against arbitrary executive action.
Article 32 gives judiciary the power to correct excesses that have led to the violation
of the fundamental rights of a citizen by the state.

53. Solution: c)
First statement can be seen in cases like Kesavananda Bharati case Basic structure
Doctrine; in other cases which enhanced the scope of Article 21 etc.

http:www.insightsonindia.com

Insights

Page 17

Insights Mock Tests 2015: Test 13 Solutions


In all other cases of constitutional amendment, it is the Parliament and the state
legislatures that are involved in constitutional amendment via Article 368 of the
constitution.

54. Solution: a)
The following explains the essential differences between both systems.

55. Solution: d)
All the above functions can be devolved to the PRIs as per the 73 rd amendment to the
constitution.
Social audits, financial scrutiny, taking care of local matters etc. are the major works
being done by Gram Sabhas in the country. They hold the Gram Panchayat
accountable till their 5 year term.
In fact, finalising BPL list has become very effective with participation from Gram
Sabha. It ensures that no undeserving candidate is declared BPL.

http:www.insightsonindia.com

Insights

Page 18

Insights Mock Tests 2015: Test 13 Solutions


56. Solution: d)
India will provide fund of Rs 1500 crore to Nepals Pancheshwar multi-purpose
project on Mahakali river for countering the Kheri floods in Uttar Pradesh.
Pancheshwar multi-purpose project

It is located at Narayan Ghat of Dharchula on Mahakali River( also known


as Kali Ganga in Uttarakhand ) in Nepal.

After completion dam will be the second largest dam in the world with a
capacity of 6720 Mega Watt (MW) power production.

In case of power sharing, India has reached into an agreement with Nepal to
procure additional electricity produced by project.

This project will regulate and control the free flow of Karnali and Mohana
rivers which cause floods in Kheri, Pilibhit of Uttar Pradesh and other Terai
districts of northern India.

57. Solution: d)
In the initial phase, 12 heritage cities have been identified which will be rejuvenated
and developed under HRIDAY.
The 12 cities are: Amritsar, Varanasi, Gaya, Puri, Ajmer, Mathura, Dwarka, Badami,
Velankanni, Kanchipuram, Warangal and Amaravati.
HRIDAY- Heritage City Development and Augmentation Yojana

It aims to bring urban planning, economic growth and heritage conservation


together for heritage cities.

It also seeks beautification in an inclusive and integrated manner with focus


on cleanliness, livelihoods, skills, safety, security, accessibility and faster
service delivery of heritage cities.

Heritage management plan (HMP) will be prepared for the identified cities
which will outline heritage resources and develop policies to guide their
conservation, restoration, future use and development.

It will seek to improve last-mile connectivity heritage sites by documentation,


conservation of areas, providing more facilities for women, senior citizens
and differently abled citizens.

http:www.insightsonindia.com

Insights

Page 19

Insights Mock Tests 2015: Test 13 Solutions

HRIDAY will be dovetailed with the Tourism Ministrys Pilgrimage


Rejuvenation and Spiritual Augmentation Drive (PRASAD) scheme which has an
outlay of Rs 100 crore for augmentation of infrastructure at pilgrimage sites
across the country.

58. Solution: a)
When the Simon Commission arrived in India in 1928, it was greeted with the slogan
Go back Simon. All parties, including the Congress and the Muslim League,
participated in the demonstrations.
In an effort to win them over, the viceroy, Lord Irwin, announced in October 1929, a
vague offer of dominion status for India in an unspecified future, and a Round
Table Conference to discuss a future constitution. This did not satisfy the Congress
leaders. The radicals within the Congress, led by Jawaharlal Nehru and Subhas
Chandra Bose, became more assertive. The liberals and moderates, who were
proposing a constitutional system within the framework of British dominion,
gradually lost their influence.

59. Solution: a)
Mahatma Gandhi found in salt a powerful symbol that could unite the nation. On 31
January 1930, he sent a letter to Viceroy Irwin stating eleven demands. Some of these
were of general interest; others were specific demands of different classes, from
industrialists to peasants. The idea was to make the demands wide-ranging, so that
all classes within Indian society could identify with them and everyone could be
brought together in a united campaign. The most stirring of all was the demand to
abolish the salt tax. Salt was something consumed by the rich and the poor alike, and
it was one of the most essential items of food. The tax on salt and the government
monopoly over its production, Mahatma Gandhi declared, revealed the most
oppressive face of British rule.

60. Solution: a)
First indigenously-built Light Combat Aircraft (LCA) Tejas was handed over to the
Indian Air Force (IAF) by Defence Minister.
It was handed over after it was granted Initial Operational Clearance (IOC)-II, which
signifies that aircraft is airworthy to fly in different conditions.
http:www.insightsonindia.com

Insights

Page 20

Insights Mock Tests 2015: Test 13 Solutions


About Light Combat Aircraft (LCA) Tejas

Tejas is outcome of Indias LCA programme, which began in the 1983 to


replace countrys aging MiG-21 fighters.

It is a lightweight multi-role jet fighter indigenously-built by state-owned


Hindustan Aeronautics Limited (HAL).

It is a tailless and having compound delta wing design. It is powered by a


single engine.

It is mounted with inbuilt MultiMode Radar (MMR), Radar Warning Receiver


(RWR) systems.

As part of LCA programme, 20 aircraft will be built by 2017-2018, to make the


first squadron of the aircraft.

IOC-I was granted to the aircraft in January 2011 after it was built by HAL.
While, IOC- II was granted in September 2014, after the aircraft successfully
completed its maiden flight.

It is yet to receive the Final Operational Clearance (FOC) and excepted to


receive it by the year-end which includes testing mid-air refuelling and
its long-range missiles capabilities.

61. Solution: d)
According to Article 370, the concurrence of the State is required for making any
laws in matters mentioned in the Union and Concurrent lists.
The remaining differences between the other States and the State of J&K are that no
emergency due to internal disturbances can be declared in J&K without the
concurrence of the State. The union government cannot impose a financial
emergency in the State and the Directive Principles do not apply in J&K. Finally,
amendments to the Indian Constitution (under Art. 368) can only apply in
concurrence with the government of J&K.

62. Solution: b)
http://www.downtoearth.org.in/content/modi-obama-seal-nuclear-agreementdeal

http:www.insightsonindia.com

Insights

Page 21

Insights Mock Tests 2015: Test 13 Solutions


63. Solution: d)
Some economists believe that once the technology is found profitable and is widely
adopted, subsidies should be phased out since their purpose has been served.
Further, subsidies are meant to benefit the farmers but a substantial amount of
fertiliser subsidy also benefits the fertiliser industry; and among farmers, the subsidy
largely benefits the farmers in the more prosperous regions. Therefore, it is argued
that there is no case for continuing with fertiliser subsidies; it does not benefit the
target group and it is a huge burden on the governments finances.
On the other hand, some believe that the government should continue with
agricultural subsidies because farming in India continues to be a risky business.
Most farmers are very poor and they will not be able to afford the required inputs
without subsidies. Eliminating subsidies will increase the inequality between rich
and poor farmers and violate the goal of equity. These experts argue that if subsidies
are largely benefiting the fertiliser industry and big farmers, the correct policy is not
to abolish subsidies but to take steps to ensure that only the poor farmers enjoy the
benefits.

64. Solution: d)
Some economists point out that subsidies do not allow prices to indicate the supply
of a good. When electricity and water are provided at a subsidised rate or free, they
will be used wastefully without any concern for their scarcity.
Farmers will cultivate water intensive crops if water is supplied free, although the
water resources in that region may be scarce and such crops will further deplete the
already scarce resources. If water is priced to reflect scarcity, farmers will cultivate
crops suitable to the region. Fertiliser and pesticide subsidies result in overuse of
resources which can be harmful to the environment. Subsidies provide an incentive
for wasteful use of resources. Think about subsidies in terms of incentives and ask
yourself whether it is wise from the economic viewpoint to provide free electricity to
farmers.

65. Solution: d)
In India, regulatory mechanisms were enforced in various ways (i) industrial
licensing under which every entrepreneur had to get permission from government
officials to start a firm, close a firm or to decide the amount of goods that could be
produced (ii) private sector was not allowed in many industries (iii) some goods
http:www.insightsonindia.com

Insights

Page 22

Insights Mock Tests 2015: Test 13 Solutions


could be produced only in small scale industries and (iv) controls on price fixation
and distribution of selected industrial products. The reform policies introduced in
and after 1991 removed many of these restrictions. Industrial licensing was
abolished for almost all but product categories alcohol, cigarettes, hazardous
chemicals, industrial explosives, electronics, aerospace and drugs and
pharmaceuticals.
The only industries which are now reserved for the public sector are defence
equipments, atomic energy generation and railway transport. Many goods produced
by small scale industries have now been dereserved. In many industries, the market
has been allowed to determine the prices.

66. Solution: d)

It aims to spread awareness about the declining trend of Child Sex Ratio in
the country.

Objective: To prevent gender biased sex selective elimination by strict


enforcement of laws with stringent punishment to violators.

The Ministry of Women and Child Development (WCD), is the nodal agency
for the implementation of the scheme. While, Centre has provided initial
corpus of 100 crore rupees for it.

It will be implemented in 100 districts of the country, including 12 districts of


Haryana.

In this regard, union government will try to ensure access and availability of
essential requirements related to nutrition, health-care, education, protection
and elimination of discrimination of girl child.

This scheme will also generate awareness and improve the efficiency of
delivery of welfare services meant for women.

67. Solution: d)
India has the largest area under cotton cultivation in the world covering 8,300
hectares in 200203. The low yield of 300 kg per hectare pushes it into third position
in production. High production costs, low and unstable yields, decline in world
prices, global glut in production due to subsidies by the U.S.A. and other countries,
and opening up of the domestic market due to globalisation have increased the
http:www.insightsonindia.com

Insights

Page 23

Insights Mock Tests 2015: Test 13 Solutions


exposure of farmers and led to agrarian distress and suicides especially in the cotton
belt of Andhra Pradesh and Maharashtra. The issue is not one of profits and higher
returns but that of the livelihood and survival of millions of small and marginal
farmers who are dependent on agriculture.

68. Solution: a)
Kudumbashree is a women-oriented community-based poverty reduction
programme being implemented in Kerala. In 1995, a thrift and credit society was
started as a small savings bank for poor women with the objective to encourage
savings. The thrift and credit society mobilised Rs 1 crore as thrift savings. These
societies have been acclaimed as the largest informal banks in Asia in terms of
participation and savings mobilised.

69. Solution: d)
Organic agriculture offers a means to substitute costlier agricultural inputs (such as
HYV seeds, chemical fertilisers, pesticides etc.) with locally produced organic inputs
that are cheaper and thereby generate good returns on investment. Organic
agriculture also generates income through exports as the demand for organically
grown crops is on a rise. Studies across countries have shown that organically grown
food has more nutritional value than chemical farming thus providing us with
healthy foods. Since organic farming requires more labour input than conventional
farming, India will find organic farming an attractive proposition. Finally, the
produce is pesticide-free and produced in an environmentally sustainable way.

70. Solution: a)
Self-explanatory. Moreover, women are generally discouraged from taking up
regular salaried jobs that involve a stay out of the home in the Indian patriarchal
society.

71. Solution: d)
Some of the challenges that Indias power sector faces today are
(i) Indias installed capacity to generate electricity is not sufficient to feed an annual
economic growth of 9 per cent. In order to meet the growing demand for electricity,
http:www.insightsonindia.com

Insights

Page 24

Insights Mock Tests 2015: Test 13 Solutions


Indias commercial energy supply needs to grow at about 7 per cent. At present,
India is able to add only 20,000 MW a year. Even the installed capacity is underutilised because plants are not run properly
(ii) State Electricity Boards (SEBs), which distribute electricity, incur losses which
exceed Rs 500 billion. This is due to transmission and distribution losses, wrong
pricing of electricity and other inefficiencies. Some scholars also say that distribution
of electricity to farmers is the main reason for the losses; electricity is also stolen in
different areas which also adds to the woes of SEBs
(iii) private sector power generators are yet to play their role in a major way; same is
the case with foreign investors
(iv) there is general public unrest due to high power tariffs and prolonged power
cuts in different parts of the country
(v) thermal power plants which are the mainstay of Indias power sector are facing
shortage of raw material and coal supplies.

72. Solution: a)
Every currency notes bears on its face a promise from the Governor of RBI that if
someone produces the note to RBI, or any other commercial bank, RBI will be
responsible for giving the person purchasing power equal to the value printed on the
note. The same is also true of coins. Currency notes and coins are therefore called fiat
money. They do not have intrinsic value like a gold or silver coin. They are also
called legal tenders as they cannot be refused by any citizen of the country for
settlement of any kind of transaction. Cheques drawn on savings or current
accounts, however, can be refused by anyone as a mode of payment. Hence, demand
deposits are not legal tenders.

73. Solution: d)
Keeping reserves is costly for banks, as, otherwise, they could lend this balance to
interest earning investment projects. However, RBI requires commercial banks to
keep reserves in order to ensure that banks have a safe cushion of assets to draw on
when account holders want to be paid. SLR requires that a given fraction of their
total demand and time deposits in the form of specified liquid assets. Once its made
zero, they will have more to lend. But since SLR is generally held in form of
government issues bonds, government may face a scarcity of funds.
http:www.insightsonindia.com

Insights

Page 25

Insights Mock Tests 2015: Test 13 Solutions

74. Solution: a)
Look up at the long discussion in Section 3.3.2 of 12 th Macroeconomics NCERT.
From this you will understand that the total amount of money stock in the economy
is much greater than the volume of high powered money. Commercial banks create
this extra amount of money by giving out a part of their deposits as loans or
investment credits. The total amount of deposits held by all commercial banks in the
country is much larger than the total size of their reserves. If all the account-holders
of all commercial banks in the country want their deposits back at the same time, the
banks will not have enough means to satisfy the need of every accountholder and
there will be bank failures.

75. Solution: a)
RBI can affect the reserve deposit ratio of commercial banks by adjusting the value of
the bank rate which is the rate of interest commercial banks have to pay RBI if
they borrow money from it in case of shortage of reserves. A low (or high) bank rate
encourages banks to keep smaller (or greater) proportion of their deposits as
reserves, since borrowing from RBI is now less (or more) costly than before. As a
result banks use a greater (or smaller) proportion of their resources for giving out
loans to borrowers or investors, thereby enhancing (or depressing) the multiplier
process via assisting (or resisting) secondary money creation. In short, a low (or
high) bank rate reduces (or increases) rdr and hence increases (or decreases) the
value of the money multiplier, which is (1 + cdr)/(cdr + rdr). Thus, for any given
amount of high powered money, H, total money supply goes up.

76. Solution: c)
Weathering processes are conditioned by many complex geological, climatic,
topographic and vegetative factors. Climate is of particular importance. Not only
weathering processes differ from climate to climate, but also the depth of the
weathering mantle.

http:www.insightsonindia.com

Insights

Page 26

Insights Mock Tests 2015: Test 13 Solutions

77. Solution: a)
The IVRS based application called Kilkari is being piloted under which audio
messages on maternal and child health care are being sent to pregnant women and
parents of children. A series of 18 health promotion and awareness generation
messages for pregnant women and parents of infants which have been specially
customised for each stage of pregnancy and the age of infant have been
professionally recorded in simple Hindi dialect and would be disseminated as audio
messages through mobiles to lakhs of pregnant women and parents of infants
particularly those residing in high priority districts of high focus States.
Another IVRS based application called Mobile Academy had been tested for training
of ASHAs and ANMs. These applications are proposed to be nationally rolled out by
15th August, 2015 to increase awareness and improve the health-seeking behavior of
pregnant women, parents of infants and to provide training to the health workers.

78. Solution: b)
Its very simple to understand. Article 74(1) clearly prescribes that the President is
bound by the advice of the council of Ministers. Whereas in the case of Governor, the
respective provision does not make the advice of the council of ministers in state
binding on him.

79. Solution: b)

http:www.insightsonindia.com

Insights

Page 27

Insights Mock Tests 2015: Test 13 Solutions

80. Solution: c)
In winter months, the weather conditions over India are generally influenced by the
distribution of pressure in Central and Western Asia. A high pressure centre in the
region lying to the north of the Himalayas develops during winter. This centre of
high pressure gives rise to the flow of air at the low level from the north towards the
Indian subcontinent, south of the mountain range. The surface winds blowing out of
the high pressure centre over Central Asia reach India in the form of a dry
continental air mass. These continental winds come in contact with trade winds over
northwestern India. The position of this contact zone is not, however, stable.
Occasionally, it may shift its position as far east as the middle Ganga valley with the
result that the whole of the northwestern and northern India up to the middle Ganga
valley comes under the influence of dry northwestern winds. The variations in the
atmospheric pressure closer to the surface of the earth have no role to play in the
making of upper air circulation. All of Western and Central Asia remains under the
http:www.insightsonindia.com

Insights

Page 28

Insights Mock Tests 2015: Test 13 Solutions


influence of westerly winds along the altitude of 9-13 km from west to east. These
winds blow across the Asian continent at latitudes north of the Himalayas roughly
parallel to the Tibetan highlands. These are known as jet streams. Tibetan highlands
act as a barrier in the path of these jet streams. As a result, jet streams get bifurcated.

81. Solution: d)
Interest payments come under revenue receipts since they are regular payments that
the government received on loans forwarded.
Borrowing from RBI creates debt on government.

82. Solution: a)
The borrowing requirement of the government includes interest obligations on
accumulated debt. The goal of measuring primary deficit is to focus on present fiscal
imbalances. To obtain an estimate of borrowing on account of current expenditures
exceeding revenues, we need to calculate what has been called the primary deficit. It
is simply the fiscal deficit minus the interest payments
Gross primary deficit = Gross fiscal deficit Net interest liabilities
Net interest liabilities consist of interest payments minus interest receipts by the
government on net domestic lending.

83. Solution: a)
Foreign exchange market is the market in which national currencies are traded for
one another. The major participants in this market are commercial banks, foreign
exchange brokers and other authorised dealers and the monetary authorities. It is
important to note that, although the participants themselves may have their own
trading centres, the market itself is world-wide. There is close and continuous
contact between the trading centres and the participants deal in more than one
market.

84. Solution: d)
While the prevalence of agriculture is indicated by finds of grain, it is more difficult
to reconstruct actual agricultural practices.
http:www.insightsonindia.com

Insights

Page 29

Insights Mock Tests 2015: Test 13 Solutions


Most Harappan sites are located in semi-arid lands, where irrigation was probably
required for agriculture. Traces of canals have been found at the Harappan site of
Shortughai in Afghanistan, but not in Punjab or Sind. It is also likely that water
drawn from wells was used for irrigation. Besides, water reservoirs found in
Dholavira (Gujarat) may have been used to store water for agriculture.

85. Solution: d)
The basic philosophy of the Jainas was already in existence in north India before the
birth of Vardhamana, who came to be known as Mahavira, in the sixth century BCE.
According to Jaina tradition, Mahavira was preceded by 23 other teachers or
tirthankaras literally, those who guide men and women across the river of existence.
The Buddha (and other teachers) taught orally through discussion and debate. Men
and women (perhaps children as well) attended these discourses and discussed what
they heard. None of the Buddhas speeches were written down during his lifetime.
After his death (c. fifth-fourth century BCE) his teachings were compiled by his
disciples at a council of elders or senior monks at Vesali (Pali for Vaishali in
present-day Bihar). These compilations were known as Tipitaka literally, three
baskets to hold different types of texts.

86. Solution: d)
http://en.wikipedia.org/wiki/Biodiversity_hotspot#Distribution_by_region

87. Solution: d)
The existing mechanism for determining the Poverty Line also does not take into
consideration social factors that trigger and perpetuate poverty such as illiteracy, ill
health, lack of access to resources, discrimination or lack of civil and political
freedoms. The aim of poverty alleviation schemes should be to improve human lives
by expanding the range of things that a person could be and could do, such as to be
healthy and well-nourished, to be knowledgeable and participate in the life of a
community.
There are many factors, other than income and assets, which are associated with
poverty; for instance, the accessibility to basic education, health care, drinking water
and sanitation. They need to be considered to develop Poverty Line.

http:www.insightsonindia.com

Insights

Page 30

Insights Mock Tests 2015: Test 13 Solutions

88. Solution: d)
Some Recommendations of Shanta Kumar Committee
On procurement related issues:

FCI should hand over all procurement operations of wheat, paddy and rice to
Andhra Pradesh, Chhattisgarh, Haryana, Madhya Pradesh, Odisha and
Punjab as they have sufficient experience and reasonable infrastructure for
procurement.

FCI procurement should focus on eastern belt, where farmers do not get
minimum support price.

On stocking and movement related issues:

FCI should outsource its stocking operations to various agencies such as


Central Warehousing Corporation (CWC), State Warehousing Corporation
(SWC), Private Sector under Private Entrepreneur Guarantee (PEG) scheme.

It should be done on competitive bidding basis, inviting various stakeholders


and creating competition to bring down costs of storage.

Movement of grains should be containerized in order to reduce transit losses.


While, railways should have faster turn-around-time by having more
mechanized facilities.

NFSA and PDS related issues

Restructuring the National Food Security Act (NFSA) by virtually diluting its
scope and coverage from 67 per cent of population to about 40 per cent
population.

In order to curtail leakages in PDS, government should defer implementation


of NFSA in states that have not done end to end computerization.

End to end computerization:

It recommends end to end computerization of the entire food management


system, starting from procurement from farmers, to stocking, movement and
finally distribution through PDS.

It will help for real time basis monitoring in order to curb leakages

http:www.insightsonindia.com

Insights

Page 31

Insights Mock Tests 2015: Test 13 Solutions


89. Solution: b)
In 1860, Julius von Sachs, a prominent German botanist, demonstrated, for the first
time, that plants could be grown to maturity in a defined nutrient solution in
complete absence of soil. This technique of growing plants in a nutrient solution is
known as hydroponics. It has been successfully employed as a technique for the
commercial production of vegetables such as tomato, seedless cucumber and lettuce.
It must be emphasised that the nutrient solutions must be adequately aerated to
obtain the optimum growth.

90. Solution: d)
The plant growth regulators (PGRs) are small, simple molecules of diverse chemical
composition. They could be indole compounds (indole-3-acetic acid, IAA); adenine
derivatives (N6-furfurylamino purine, kinetin), derivatives of carotenoids (abscisic
acid, ABA); terpenes (gibberellic acid, GA3) or gases (ethylene, C2H4). Plant growth
regulators are variously described as plant growth substances, plant hormones or
phytohormones in literature.
The PGRs can be broadly divided into two groups based on their functions in a
living plant body. One group of PGRs are involved in growth promoting activities,
such as cell division, cell enlargement, pattern formation, tropic growth, flowering,
fruiting and seed formation. These are also called plant growth promoters, e.g.,
auxins, gibberellins and cytokinins. The PGRs of the other group play an important
role in plant responses to wounds and stresses of biotic and abiotic origin. They are
also involved in various growth inhibiting activities such as dormancy and
abscission. The PGR abscisic acid belongs to this group. The gaseous PGR, ethylene,
could fit either of the groups, but it is largely an inhibitor of growth activities.

http:www.insightsonindia.com

Insights

Page 32

Insights Mock Tests 2015: Test 13 Solutions


91. Solution: c)
Innate immunity is non-specific type of defence, that is present at the time of birth.
This is accomplished by providing different types of barriers to the entry of the
foreign agents into our body. Innate immunity consist of four types of barriers.
These are
(i) Physical barriers : Skin on our body is the main barrier which prevents entry of
the micro-organisms. Mucus coating of the epithelium lining the respiratory,
gastrointestinal and urogenital tracts also help in trapping microbes entering our
body.
(ii) Physiological barriers : Acid in the stomach, saliva in the mouth, tears from
eyesall prevent microbial growth.
(iii) Cellular barriers : Certain types of leukocytes (WBC) of our body like
polymorpho-nuclear leukocytes (PMNL-neutrophils) and monocytes and natural
killer (type of lymphocytes) in the blood as well as macrophages in tissues can
phagocytose and destroy microbes.
(iv) Cytokine barriers : Virus-infected cells secrete proteins called interferons which
protect non-infected cells from further viral infection.

92. Solution: d)
He was particularly fascinated with the trading conditions in India, and compared
India to Iran and the Ottoman empire.
Bernier travelled to several parts of the country, and wrote accounts of what he saw,
frequently comparing what he saw in India with the situation in Europe. He
dedicated his major writing to Louis XIV, the king of France, and many of his other
works were written in the form of letters to influential officials and ministers. In
virtually every instance Bernier described what he saw in India as a bleak situation
in comparison to developments in Europe. This assessment was not always accurate.
However, when his works were published, Berniers writings became extremely
popular.

93. Solution: a)
Some of the earliest bhakti movements (c. Sixth century) were led by the Alvars
(literally, those who are immersed in devotion to Vishnu) and Nayanars (literally,
http:www.insightsonindia.com

Insights

Page 33

Insights Mock Tests 2015: Test 13 Solutions


leaders who were devotees of Shiva). They travelled from place to place singing
hymns in Tamil in praise of their gods.
During their travels the Alvars and Nayanars identified certain shrines as abodes of
their chosen deities. Very often large temples were later built at these sacred places.
These developed as centres of pilgrimage. Singing compositions of these poet-saints
became part of temple rituals in these shrines, as did worship of the saints images.
Some historians suggest that the Alvars and Nayanars initiated a movement of
protest against the caste system and the dominance of Brahmanas or at least
attempted to reform the system. To some extent this is corroborated by the fact that
bhaktas hailed from diverse social backgrounds ranging from Brahmanas to artisans
and cultivators and even from castes considered untouchable.

94. Solution: a)
The Health Ministry has introduced web-based name based tracking system called
Mother & Child Tracking System (MCTS) across all the States and UTs in 2010 to
facilitate timely delivery of antenatal and postnatal care services to all the pregnant
women and immunization to all the children. The system captures personal details
such as name, address, mobile number etc., of every pregnant woman and child up
to five years of age.
A total of over 14 crore pregnant women and children, besides 2.24 lakh Auxiliary
Nurse Midwives (ANMs) and 9.31 Accredited Social Health Activists (ASHAs) have
been registered in MCTS till now. The MCTS aims to ensure that every pregnant
woman gets complete and quality antenatal and postnatal care, and every child
receives the full range of immunization services. Under MCTS, appropriate health
promotion messages to beneficiaries that are relevant according to the month of
pregnancy or date of birth of the child are being sent on mobiles of beneficiaries.
95. Solution: a)
Objectives of the Home Rule League movement

To establish self- government

To build up an agitation for home rule by promoting political education and


discussion.

To build the confidence of the Indians against the suppression of the British
government and to create an alternative movement to break the existing state
of stagnation and the inertia.

http:www.insightsonindia.com

Insights

Page 34

Insights Mock Tests 2015: Test 13 Solutions

To revive the political activity on their own while maintaining the principles
of congress.

To demand for greater political representation from the British government.

96. Solution: d)
http://www.facts-about-india.com/surat-split.php

97. Solution: d)
After arriving in India, Mahatma Gandhi successfully organised satyagraha
movements in various places. In 1916 he travelled to Champaran in Bihar to inspire
the peasants to struggle against the oppressive plantation system. Then in 1917, he
organised a satyagraha to support the peasants of the Kheda district of Gujarat.
Affected by crop failure and a plague epidemic, the peasants of Kheda could not pay
the revenue, and were demanding that revenue collection be relaxed. In 1918,
Mahatma Gandhi went to Ahmedabad to organise a satyagraha movement amongst
cotton mill workers.

98. Solution: a)
Gandhiji in 1919 decided to launch a nationwide satyagraha against the proposed
Rowlatt Act (1919). This Act had been hurriedly passed through the Imperial
Legislative Council despite the united opposition of the Indian members. It gave the
government enormous powers to repress political activities, and allowed detention
of political prisoners without trial for two years. Mahatma Gandhi wanted nonviolent civil disobedience against such unjust laws, which would start with a hartal
on 6 April.

99. Solution: d)

http:www.insightsonindia.com

Insights

Page 35

Insights Mock Tests 2015: Test 13 Solutions


Not all social groups were moved by the abstract concept of swaraj. One such group
was the nations untouchables, who from around the 1930s had begun to call
themselves dalit or oppressed. For long the Congress had ignored the dalits, for fear
of offending the sanatanis, the conservative high-caste Hindus. But Mahatma Gandhi
declared that swaraj would not come for a hundred years if untouchability was not
eliminated. He called the untouchables harijan, or the children of God, organised
satyagraha to secure them entry into temples, and access to public wells, tanks,
roads and schools.
He himself cleaned toilets to dignify the work of the bhangi (the sweepers), and
persuaded upper castes to change their heart and give up the sin of untouchability.
But many dalit leaders were keen on a different political solution to the problems of
the community.
They began organising themselves, demanding reserved seats in educational
institutions, and a separate electorate that would choose dalit members for
legislative councils. Political empowerment, they believed, would resolve the
problems of their social disabilities. Dalit participation in the Civil Disobedience
Movement was therefore limited, particularly in the Maharashtra and Nagpur
region where their organisation was quite strong.
Dr B.R. Ambedkar, who organised the dalits into the Depressed Classes Association
in 1930, clashed with Mahatma Gandhi at the second Round Table Conference by
demanding separate electorates for dalits.

100.

Solution: c)

Early Buddhist teachings had given great importance to self-effort in achieving


nibbana. Besides, the Buddha was regarded as a human being who attained
enlightenment and nibbana through his own efforts. However, gradually the idea of a
saviour emerged. It was believed that he was the one who could ensure salvation.
Simultaneously, the concept of the Bodhisatta also developed. Bodhisattas were
perceived as deeply compassionate beings who accumulated merit through their
efforts but used this not to attain nibbana and thereby abandon the world, but to help
others. The worship of images of the Buddha and Bodhisattas became an important
part of this tradition.
This new way of thinking was called Mahayana literally, the great vehicle.

http:www.insightsonindia.com

Insights

Page 36

Insights Mock Tests 2015: Test 13 Solutions

http:www.insightsonindia.com

Insights

Page 37

INSIGHTS ON INDIA MOCK PRELIMINARY EXAM - 2015


INSIGHTS ON INDIA MOCK TEST - 14
GENERAL STUDIES

PAPER-I
Time Allowed: 1.5 Hours

Maximum Marks: 150

INSTRUCTIONS
1. IMMEDITELY AFTER THE COMMENCEMENT OF THE EXAMINATION, YOU SHOULD
CHECK THAT THIS TEST BOOKLET DOES NOT HAVE ANY UNPRINTED OR TORN OR MISSING
PAGES OR ITEMS, ETC. IF SO, GET IT REPLACED BY A COMPLETE TEST BOOKLET.
2. You have to enter your Roll Number on the Test
Booklet in the Box provided alongside. DO NOT
Write anything else on the Test Booklet.
4. This Test Booklet contains 75 items (questions). Each item is printed only in English. Each item
comprises four responses (answers). You will select the response which you want to mark on the
Answer Sheet. In case you feel that there is more than one correct response, mark the response which
you consider the best. In any case, choose ONLY ONE response for each item.
5. You have to mark all your responses ONLY on the separate Answer Sheet provided. See directions in
the Answer Sheet.
6. All items carry equal marks.
7. Before you proceed to mark in the Answer Sheet the response to various items in the Test Booklet, you
have to fill in some particulars in the Answer Sheet as per instructions sent to you with your
Admission Certificate.
8. After you have completed filling in all your responses on the Answer Sheet and the examination has
concluded, you should hand over to the Invigilator only the Answer Sheet. You are permitted to take
away with you the Test Booklet.
9. Sheets for rough work are appended in the Test Booklet at the end.
10. Penalty for wrong answers :
THERE WILL BE PENALTY FOR WRONG ANSWERS MARKED BY A CANDIDATE IN THE
OBJECTIVE TYPE QUESTION PAPERS.
(i) There are four alternatives for the answer to every question. For each question for which a
wrong answer has been given by the candidate, one-third of the marks assigned to that
question will be deducted as penalty.
(ii) If a candidate gives more than one answer, it will be treated as a wrong answer even if one of
the given answers happens to be correct and there will be same penalty as above to that
question.
(iii)

If a question is left blank, i.e., no answer is given by the candidate, there will be no penalty
for that question.
http://www.insightsonindia.com

INSIGHTS ON INDIA MOCK TEST SERIES FOR CIVIL SERVICES PRELIMINARY EXAM 2015
http://insightsonindia.com

INSIGHTS

Page 1

1. The idea of Invisible Hand of the free


market in an economy means that
a) Markets work in tandem with the
state.
b) Market forces of supply and demand
bring equilibrium in the economy.
c) Competition in the markets helps the
state to fulfil its obligations.
d) Individuals pursuing their own
interest frequently promote the good
of society than when they really intend
to promote it

2. Many economic roles which were under


complete government monopolies were
now opened for participation by the
private sector. This change in a economy
can be called
a) Privatization
b) Deregulation
c) De-reservation
d) De-licensing

3. Consider the following statements about


the role of the state in an economy.
1. The state cannot earn profits like a
private enterprise.
2. The state regulates competitive
practices in the economy.
3. The state has the responsibility to
redistribute economic resources in the
economy.
Choose the correct answer using the codes below.
a)
b)
c)
d)

1 and 2
2 and 3
1 and 3
All of the above

http://insightsonindia.com

4. Consider the following statements about


the Indian constitution.
1. Public comments were invited while
finalizing the constitutional document.
2. It was put to referendum.
3. Some provisions of the constitution
came in force even before India
adopted the constitution fully on the
first republic day.
Choose the correct answer using the codes below.
a) 1 and 2 only
b) 2 and 3 only
c) 1 and 3 only
d) All of the above

5. The Constitution contains not only the


fundamental principles of governance but
also detailed administrative provisions.
This fact can be supported by looking up
in which of the following provisions of the
constitution?
1. Fifth and Sixth schedule
2. Centre-state relations
3. Appointment and functions of several
constitutional bodies
Choose the correct answer using the codes below.
a) 1 and 2 only
b) 2 and 3 only
c) 1 and 3 only
d) All of the above

6. Net Domestic Product (NDP) is arrived at


by
a) Subtracting the annual depreciation
and subsidies from the GDP.
b) Subtracting the annual depreciation
from the GDP.
c) Adding annual deprecation to the
GDP.
d) Adding the annual depreciation and
subsidies to the GDP.
INSIGHTS

Page 2

7. If the difference between the GDP and


NDP of an economy is becoming less year
over year, it may imply that
1. Research and development in the
economy is cutting down the levels of
depreciation in the economy.
2. The GDP growth rate has reduced
over years.
3. The government has reduced
subsidies and indirect taxes over the
years.
Choose the correct answer using the codes below.
a)
b)
c)
d)

1 and 2
2 and 3
1 only
All of the above

8. NDP is not used to compare the


economies of the world. This is because
a) The deprecation levels set by different
economies for goods are different.
b) NDP is also affected by the levels of
indirect taxes and subsidies offered by
an economy which can be different for
economies.
c) IMF and World Bank do not use NDP
for comparing economies.
d) None of the above

9. In which of the following cases, the Gross


National Product (GNP) of India would
increase?
1. More remittances are being sent by
Indians residing in Gulf countries.
2. India borrows less and lends more to
other World economies.
3. Software exports from India increase.

b) 2 and 3
c) 1 only
d) All of the above

10. In which of the following ways is the


concept of GNP used across the World?
1. IMF ranks nations as per their GNP in
terms of Purchasing Power parity
(PPP).
2. GNP also shows the external strength
of the economy.
3. It shows the financial dependence of
one economy over the other.
Choose the correct answer using the codes below.
a)
b)
c)
d)

1 and 2
2 and 3
1 and 3
All of the above

11. Consider the following statements about


Net National Product (NNP).
1. National Income of an economy is
counted in terms of NNP.
2. Per capita income of a nation is
obtained from NNP by dividing it
with the working population of the
country.
3. It is not influenced by factors external
to the economy.
Choose the correct answer using the codes below.
a)
b)
c)
d)

1 and 2
1 only
1 and 3
2 and 3

Choose the correct answer using the codes below.


a) 1 and 2
http://insightsonindia.com

INSIGHTS

Page 3

12. Factor Cost is an important component


while calculating national income. It is
a) input cost the producer has to incur
in the process of producing something
b) total market value of a product minus
subsidies given by the state
c) total market value of a product minus
indirect taxes imposed by the state
d) None of the above

13. Choose the correct equation of National


Income from the following.
a) National Income at Factor Cost = NNP
at Market Cost Indirect Taxes +
Subsidies
b) National Income at Factor Cost = GNP
at Market Cost Indirect Taxes +
Subsidies
c) National Income at Market Cost =
NNP at Factor Cost Indirect Taxes +
Subsidies
d) National Income at Market Cost =
GNP at Factor Cost Indirect Taxes +
Subsidies

14. Which of the following do NOT require a


constitutional amendment?
1. Settlement of a boundary dispute
between India and another country
2. Changing the name of a state
3. Creation of a new state

15. The Constitution of India confers which of


the following rights and privileges on the
citizens of India (and denies the same to
aliens)?
1. Right to Education
2. Right against discrimination on
grounds of religion, race, caste, sex or
place of birth
3. Freedom of speech and expression
4. Freedom of assembly
5. Right to conserve and safeguard
minority culture
Choose the correct answer using the codes
below.
a)
b)
c)
d)

16. Consider the following statements.


1. In India citizens by birth as well as a
naturalised citizen are eligible for the
office of President.
2. Foreigners staying in India do not
have to oblige to the fundamental
duties.
3. The Constitution does not prescribe
any qualifications for citizenship.
Choose the correct answer using the codes below.
a)
b)
c)
d)

Choose the correct answer using the codes below.


a)
b)
c)
d)

1 and 2 only
2 and 3 only
1 and 3 only
All of the above

http://insightsonindia.com

3 and 4 only
2 and 5 only
All except 1
All of the above

1 and 2 only
2 and 3 only
1 and 3 only
All of the above

17. The application of fundamental rights to


the citizens can be restricted in which of
the following situations?
1. Presidents rule
2. Financial emergency
INSIGHTS

Page 4

3. Martial Law
4. Internal emergency

Choose the correct answer using the codes below.


a)
b)
c)
d)

Choose the correct answer using the codes below.


a)
b)
c)
d)

1, 2 and 4 only
3 and 4 only
3 only
1 and 3 only

18. Consider the following statements about


the calculation of the Human
Development Index (HDI).
1. Life expectancy used in HDI data is
calculated at the time of death.
2. Under education, School enrolment
has a higher weightage than
educational attainment among adults.
3. Real per capita income used in the
calculation is adjusted for the differing
purchasing power parity among
nations.

1 and 2 only
2 and 3 only
1 and 3 only
All of the above

20. Consider the following statements about


the Government of India Act, 1935.
1. It successfully established an All-India
federation.
2. It abolished dyarchy in the provinces
and introduced provincial autonomy
in its place.
3. It extended franchise.
Choose the correct answer using the codes below.
a)
b)
c)
d)

1 and 2 only
2 and 3 only
1 and 3 only
All of the above

Choose the correct answer using the codes below.


a)
b)
c)
d)

1 and 2
1, 2 and 3
2 only
3 only

19. Which of the following correctly highlight


the difference between NITI Aayog and
the scrapped Planning Commission?
1. While Planning Commission enjoyed
the powers to allocate funds to
ministries and state
governments, NITI Aayog will be
an advisory body, or a think-tank.
2. States had a more direct and
authoritative role in NITI Aayog.
3. Planning Commission decided policies
for states and tied allocation of funds,
which NITI Aayog will not do.
http://insightsonindia.com

21. The Union Ministry of Youth Affairs &


Sports has revised the Scheme of
Rashtriya Khel Protsahan Puruskar.
According to the revised plan, this scheme
will be given in following four categories.
They are,
1. Encouragement of sports through
Corporate Social Responsibility (CSR)
2. Employment of sportspersons and
sports welfare measures
3. Sports for Development has been
provided for the NGOs in order to
recognize their contributions.
Choose the correct answer using the codes below.
a)
b)
c)
d)
INSIGHTS

1 and 2 only
2 and 3 only
1 and 3 only
All of the above
Page 5

22. An open competition system of selection


and recruitment of civil servants; and a
separate legislative wing for the
Governor-General was introduced by
a) Charter Act of 1853
b) Charter Act of 1833
c) Charter Act of 1861
d) Charter Act of 1892

23. Consider the following statements about


the Government of India Act 1858.
1. It abolished the East India Company.
2. It changed the entire structure of
government in India prevailing earlier.
3. As per the Act, Indian Government
was to be supervised and controlled in
England.
Choose the correct answer using the codes below.
a)
b)
c)
d)

1 and 2 only
2 and 3 only
1 and 3 only
All of the above

24. Which of the following acts are concerned


with changes to the legislative councils in
British India?
1. Act of 1909
2. Act of 1892
3. Act of 1861
Choose the correct answer using the codes below.
a)
b)
c)
d)

1 and 2 only
2 and 3 only
1 and 3 only
All of the above

1. Bicameralism
2. Direct elections
3. Dyarchy
Choose the correct answer using the codes below.
a)
b)
c)
d)

1 and 2 only
2 and 3 only
1 and 3 only
All of the above

26. Although the Simon Commission was


boycotted by all political parties in India,
some of the recommendations of the
commission were towards better
governance of India. These were
1. Abolition of dyarchy
2. Extension of responsible government
in the provinces
3. Establishment of a federation of British
India and princely states
Choose the correct answer using the codes below.
a)
b)
c)
d)

1 and 2 only
2 and 3 only
1 and 3 only
All of the above

27. The Jeevan Amrut Yojana of the


Maharashtra government is aimed at
providing the facility of
a) Blood supply to patients
b) Free piped drinking water to rural
Below poverty Line citizens
c) Free drinking water to the whole rural
Maharashtra
d) Organ donation facility to needy
patients

25. Which of the following were introduced


for the first time in India by the Act of
1919?
http://insightsonindia.com

INSIGHTS

Page 6

28. As per a recent United Nations


Conference of Trade and Development
(UNCTAD) report, China has overtaken
the United States (US) as the top
destination for foreign direct investment
(FDI). Arrange the following countries in
decreasing order of the FDI they receive.
1. Hong Kong
2. Singapore
3. Canada
4. India
Choose the correct order from the codes below.
a)
b)
c)
d)

1234
2431
1243
2134

29. Consider the following statements about


the Maritime Silk Road (MSR) project.
1. It has been initiated by China.
2. It involves building infrastructure in
strategically significant countries, inter
alia, in Africa and Europe.
3. No ports will be built in Pakistan or
Bangladesh under MSR.

3. There were to be no nominated


members in the CA as per the decided
scheme.
Choose the correct answer using the codes below.
a)
b)
c)
d)

1 and 2 only
2 and 3 only
1 and 3 only
All of the above

31. Consider the following statements.


1. If the secondary sector contributes 50
per cent or more to the total produce
value of an economy, it is an industrial
economy.
2. The tertiary sector always provides
livelihood to the largest number of
people in Industrial economies.
3. Shift from agrarian sector to the
service sector always happens
gradually with the growth of the
Industrial sector.
Choose the correct answer using the codes below.
a)
b)
c)
d)

1 and 2
2 and 3
1 and 3
1 only

Choose the correct answer using the codes below.


a)
b)
c)
d)

1 and 2 only
2 and 3 only
1 and 3 only
All of the above

30. Consider the following statements about


the constituent assembly (CA).
1. It was based on a scheme suggested by
the Cabinet mission.
2. All provinces and princely state were
to be represented in the CA as per the
decided scheme.
http://insightsonindia.com

32. Gross Domestic Product (GDP) is the


value of the all final goods and services
produced within the boundary of a nation
during one year. For India, this calendar
year period is from
a) 1st April to 31st March
b) 1st Jan to 31st December
c) 1st March to 28th February
d) There is no standard calendar year
period. GDP is calculated for different
annual periods

INSIGHTS

Page 7

33. Consider the following uses of the concept


of GDP.
1. It does not say anything about the
qualitative aspects of the produced
goods and services by the economy.
2. It is used by the IMF/WB in the
comparative analyses of its member
nations.
3. If the GDP is growing at the same rate
every successive year, it will add the
same
income to the income to the
economy in all these years.
Choose the correct answer using the codes below.
a)
b)
c)
d)

allowed. Now consider the following


courses of action.
1. Complain to the local police authority
about this discrimination and violation
of your fundamental right of free
movement
2. Approach the state High court of this
violation of fundamental right
3. Approach the Supreme Court with a
Public Interest Litigation (PIL) on the
same issue
Which of the above options are legally and
constitutionally appropriate?

1 and 2
2 and 3
1 and 3
All of the above

34. The structural part of the Constitution is,


to a large extent, derived from the
Government of India Act of 1935. This
argument is supported by which of the
following features/provisions of the
constitution?
1. Public Service Commission of Union
and States
2. Federal polity
3. Division of powers in lists between
Centre and states
4. Bicameralism in state assemblies
Choose the correct answer using the codes below.
a) 1 and 2 only
b) 2 and 3 only
c) 1 and 4 only
d) All of the above
35. Consider that you belong to the religion
Zoroastrianism. You are outside a Hindu
temple managed by the state and want to
enter. But somewhere at the entry you
find this written Only Hindus are
http://insightsonindia.com

a)
b)
c)
d)

2 only
3 only
All of the above
None of the above

36. India has adopted the parliamentary


system of Government. It is based on the
principle of
a) Doctrine of separation of powers
between the legislature and executive
organs
b) Cooperation and co-ordination
between the legislative and executive
organs
c) Doctrine of separation of powers
between the legislature and judicial
organs
d) Doctrine of separation of powers
between the executive and judicial
organs

37. Consider the following statements.


1. The term Federation has nowhere
been used in the Constitution.
2. The state and local governments
derive their authority from the
constitution rather than from the
Union government.
INSIGHTS

Page 8

Which of the above is/are true?


a)
b)
c)
d)

1 only
2 only
Both 1 and 2
None

38. Consider the following statements.


1. As per the Independence Act of 1947,
the constituent assembly was to
become the first dominion legislature.
2. The constituent assembly was not a
fully sovereign body.
Which of the above is/are true?
a)
b)
c)
d)

1. The government takes the sole


responsibility of providing goods and
services to the population.
2. No direct payments are made by the
consumer.
3. Prices of the goods and services are
determined in the open market.

1 only
2 only
Both 1 and 2
None

39. Among all the committees of the


Constituent Assembly, the most important
committee was the Drafting Committee.
Who among the following were NOT the
members of the committee?
1. N Gopalaswamy Ayyangar
2. Syed Mohammad Saadullah
3. T T Krishnamachari
4. Dr K M Munshi

Choose the correct answer using the codes below.


a)
b)
c)
d)

41. Which of the following arguments can be


forwarded to weaken the proposition of
choosing Industry as the prime mover of
the Indian economy post-independence?
1. Lack of investible capitalbe the case
of either the government or the private
sector.
2. Absence of the market for industrial
goods.
3. Industry as a prime mover cannot
work in a state led economy.
Choose the correct answer using the codes below.
a)
b)
c)
d)

Choose the correct answer using the codes below.


a)
b)
c)
d)

4 only
2 and 3 only
1 only
All were the members

40. Consider the following statements about a


State led distribution network in an
economy.

http://insightsonindia.com

1 and 2
2 and 3
1 and 3
All of the above

1 and 2
1, 2 and 3
2 only
3 only

42. For industrialisation and its success, every


economy needs the healthy presence of
some basic industries. These are
1. Electricity
2. Oil Refining
3. Coal
4. Crude Oil
Choose the correct answer using the codes below.
INSIGHTS

Page 9

a)
b)
c)
d)

All of the above


2 and 3 only
1 and 4 only
1, 2 and 3 only

43. Many Public Sector Units (PSUs) were


established post-independence. Which of
the following functions were NOT
performed by them?
1. Providing disposable income to the
government
2. Provision of Public goods
3. Cutting down the monopoly existing
in the private sector then

45. Which of the following arguments


support a market led economy?
1. Dynamic operation of market forces
brings a state of equilibrium to the
economy.
2. It values continuous improvement in
provision of goods and services as it is
based on competition.
3. Markets can fulfil all the needs of an
economy.

Choose the correct answer using the codes below.


a)
b)
c)
d)

Choose the correct answer using the codes below.


a)
b)
c)
d)

1 and 3
1 only
2 only
3 only

44. Consider the following statements about


the Charter Act of 1833.
1. The act created, for the first time, a
Government of India having authority
over the entire territorial area
possessed by the British in India.
2. It ended the activities of the East India
Company as a commercial body.
3. It separated, for the first time, the
legislative and executive functions of
the Governor-Generals council.

46. In India, all citizens irrespective of the


state in which they are born or reside
enjoy the same political and civil rights of
citizenship all over the country and no
discrimination is made between them
excepting in few cases in
1. Tribal areas
2. State of J&K
3. Areas mentioned under Article 371 of
the Constitution
Choose the correct answer using the codes below.
a)
b)
c)
d)

Choose the correct answer using the codes below.


a)
b)
c)
d)

1 and 2 only
2 and 3 only
1 and 3 only
All of the above

http://insightsonindia.com

1 and 2
2 and 3
1 and 3
All of the above

1 and 2 only
2 and 3 only
1 and 3 only
All of the above

47. Which of the following terminologies find


a mention in the Constitution of India?
1. Consolidated fund of India
2. State Public Service Commission
3. Financial emergency
4. Quasi-legislative
INSIGHTS

Page 10

Choose the correct answer using the codes below.


a)
b)
c)
d)

1. Territory of India is a wider


expression than the Union of India as
the former includes
only states &
UTs.
2. India can acquire foreign territories
according to the modes recognised by
international
law.
3. Parliament can establish new states
that were not a part of the Union of
India.

1 and 2 only
2 and 4 only
1, 3 and 4 only
1, 2 and 3 only

48. State emergency or Presidents rule can be


applied to an Indian state on which of the
following grounds?
1. Failure to comply with Centres
directions
2. Breakdown of constitutional
machinery in the state
3. Threat to the financial stability of the
state
4. Internal aggression in the state

Choose the correct answer using the codes below.


a)
b)
c)
d)

1 and 2 only
2 and 3 only
1 and 3 only
All of the above

Choose the correct answer using the codes below.


a)
b)
c)
d)

1 and 2 only
2 and 4 only
1, 3 and 4 only
1, 2 and 3 only

49. Consider the following statements about


the Preamble of the Constitution.
1. The Preamble of the source of
authority of the Constitution.
2. It declares the nature of the Indian
state.
3. It envisages political as well as social
ideals.

51. Article 3 of the Constitution authorizes the


Parliament to
1. Increase or diminish the area of any
state
2. Merge a state and Union Territory to
form a new state
3. Grant special status to the newly
created weak states
Choose the correct answer using the codes below.
a)
b)
c)
d)

1 and 2 only
2 and 3 only
1 and 3 only
All of the above

Choose the correct answer using the codes below.


a)
b)
c)
d)

1 and 2 only
2 and 3 only
1 and 3 only
All of the above

50. Consider the following statements.


http://insightsonindia.com

52. The Supreme Court of India is


1. A federal court
2. The guarantor of the fundamental
rights of the citizens
3. The guardian of the Constitution
4. The enforcer of Central as well as state
laws
INSIGHTS

Page 11

Choose the correct answer using the codes below.


a)
b)
c)
d)

Parliament only with the prior


recommendation of the President.
2. Before the bill is approved by the
Parliament, it refers the same to the
state legislature concerned for
expressing its views within a specified
period.

1 and 2 only
2 and 3 only
1 and 4 only
1, 2 and 3 only

53. That the Indian Constitution is founded


on the bedrock of the balance between the
Fundamental Rights and the Directive
Principles has been
a) Ruled by the Supreme Court
b) Provided for in the Constitution of
India
c) Affirmed by a resolution of the
Parliament
d) Affirmed by a resolution of the
Constituent assembly of India

54. The Constitution of India stands for a


secular state. Hence, it does not uphold
any particular religion as the official
religion of the Indian State. Which of the
following articles of the Constitution
reveal the secular character of the Indian
State?
1. Article 14
2. Article 15 and 16
3. Article 30
4. Article 44

Which of the above is/are correct?


a)
b)
c)
d)

1 only
2 only
Both 1 and 2
None

56. The term State has been used in different


provisions concerning the fundamental
rights against which the citizens are
protected. State can include which of the
following?
1. A Public Sector Undertaking
2. District Planning Committee
3. A Private party under a Public Private
Partnership (PPP) contract with the
government
Choose the correct answer using the codes below.
a)
b)
c)
d)

1 and 2 only
2 and 3 only
1 and 3 only
All of the above

Choose the correct answer using the codes below.


a)
b)
c)
d)

1 and 4 only
2 and 3 only
1, 2 and 3 only
All of the above

55. Consider the following statements about


the process of creation of new states.
1. A bill which intends to form a new
state can be introduced in the
http://insightsonindia.com

57. Article 14 of the constitution says that the


State shall not deny to any person
equality before the law or the equal
protection of the laws within the territory
of India. Which of the following would
come under equal protection of the
laws?
1. The equal subjection of all persons to
the ordinary law of the land
administered by ordinary law courts.
INSIGHTS

Page 12

2. Similar application of the same laws to


all persons who are similarly situated
3. Equality of opportunity to all under
the laws made by the Parliament
Choose the correct answer using the codes below.
a)
b)
c)
d)

2 only
2 and 3 only
1 and 3 only
1 and 2 only

58. Writ of Mandamus is a command issued


by the court. It can be issued to
1. Governors
2. Tribunals
3. Inferior courts
Choose the correct answer using the codes given
below.
a)
b)
c)
d)

1 and 2 only
2 and 3 only
1 and 3 only
All of the above

59. As per the Supreme Court, which of the


following are included in the Freedom of
Speech and expression?
1. Right against tapping of telephonic
conversation
2. Right to know about government
activities.
3. Right against bandh called by a
political party or organisation
Choose the correct answer using the codes below.
a)
b)
c)
d)

All of the above


2 and 3 only
1 and 3 only
1 and 2 only

60. Right to fair trial, Right to hearing, Right


to speedy trial and Right to free legal aid
are provided to citizens under which of
the following fundamental rights?
a) Equality before law and equal
protection of laws
b) Right to Life and Liberty
c) Right against arbitrary arrest and
detention
d) Right against exploitation

61. Article 23 of the Constitution prohibits


traffic in human beings, forced labour and
other similar forms of forced labour. It is
enforced by which of the following laws
made by the Parliament?
1. Minimum Wages Act, 1948
2. Contract Labour Act, 1970
3. Equal Remuneration Act, 1976
Choose the correct answer using the codes below.
a)
b)
c)
d)

All of the above


2 and 3 only
1 and 3 only
1 and 2 only

62. Consider the following provisions of the


Civil Nuclear Liability Act, 2010 that has
been in news for some time.
1. This Act limits the liability of the
operator in case of nuclear accident.
2. It secures the operator the right to
recourse under certain circumstances.
3. It provides a mechanism to
compensate victims of nuclear
damage.
Which of the above provisions have been cause(s)
of contention between India and other nuclear
supplier countries in recent times?
Choose the correct answer using the codes below.

http://insightsonindia.com

INSIGHTS

Page 13

a)
b)
c)
d)

1 and 2 only
2 and 3 only
1 and 3 only
All of the above

63. Recently, two beneficial algal species Ulva


paschima Bast and Cladophora goensis
Bast were discovered off the west coast of
India. Consider the following statements
about them.
1. They are endemic to India.
2. These species have excellent carbon
capture properties.
3. They can be used as bio-fuels.
Choose the correct answer using the codes below.
a)
b)
c)
d)

1 and 2 only
2 and 3 only
1 and 3 only
All of the above

64. The Indian constitution explicitly contains


which of the following provisions for
exercising direct democracy?
1. Referendum
2. Plebiscite
3. Recall
4. Initiative
Choose the correct answer using the codes below.
a)
b)
c)
d)

1 and 2 only
1 only
3 and 4 only
None of the above

3. Article 16
4. Article 19
Choose the correct answer using the codes below.
a)
b)
c)
d)

1 and 2 only
2 and 4 only
1, 3 and 4 only
1, 2 and 3 only

66. The status of the Preamble of the


Constitution has always been a question
of debate in the country. The present
opinion of the Supreme Court about the
status of the Preamble is that
a) It is a part of the constitution but
cannot be amended.
b) It is a not part of the constitution and
cannot be amended.
c) It is a part of the constitution and can
be amended.
d) It is not a part of the constitution but
can be amended.

67. Mass de-worming campaigns of school


children have been used both as a
preventive as well as a treatment method.
In this regard, the union ministry of
Health & Family Welfare launched the
National De-worming initiative. Which of
the following problems due to worming
will the scheme tackle?
1. Malnutrition
2. Wasting
3. Cognitive Impairment
4. Tissue damage that may require
corrective surgery
Choose the correct answer using the codes below.

65. Which of the following provisions of the


chapter on Fundamental Rights in the
Constitution ensure civic equality?
1. Article 13
2. Article 14
http://insightsonindia.com

a)
b)
c)
d)
INSIGHTS

1 and 4 only
2 and 3 only
1, 3 and 4 only
All of the above
Page 14

68. Consider the following statements about


the recently launched Deen Dayal
Upadhyaya Grameen Kaushalya Yojana.
1. The scheme lays Greater emphasis on
projects for poor rural youth in Jammu
and Kashmir
2. The scheme provides for greater
support for placed candidates in terms
of post-placement support, migration
support and alumni network.
3. Demand led skill training at no cost to
the rural poor

order, morality and the like. Which of the


following can be done by the state
constitutionally?
1. Regulate a religious institution
2. Throw open Hindu religious
institutions of a public character to all
classes and sections of
Hindus.
3. Restrict a religious practice
Choose the correct answer using the codes given
below.
a)
b)
c)
d)

Which of the above are the features of the


scheme?
a)
b)
c)
d)

1 and 2
2 and 3
1 and 3
All of the above

69. The vision of Digital India as launched


by the Prime Minister seeks of achieve
which of the following within the
stipulated time period?
1. Broadband for All rural panchayats
2. A digital cloud access to every Indian
citizen
3. One Common Service Centre in each
Gram panchayats
4. E-healthcare services

71. The writ jurisdiction of the Supreme Court


is narrower and less discretionary than
that of a high court in what respects?
1. High courts can enforce legal rights
too, SC cannot.
2. High court can issues it both against
persons and state, SC can issue it only
against the
later.
3. SC cannot refuse to issues writs when
it comes to Fundamental Rights, High
courts can.
Choose the correct answer using the codes given
below.
a)
b)
c)
d)

Choose the correct answer using the codes given


below.
a)
b)
c)
d)

1 and 2 only
2 and 3 only
1 and 3 only
All of the above

70. Although India is a secular country, the


state can take some steps to ensure public
http://insightsonindia.com

1 and 2 only
2 and 3 only
1 and 3 only
All of the above

1 and 2 only
2 and 3 only
1 and 3 only
All of the above

72. Consider the following statements about


the write of Habaes Corpus.
1. It can be issued by even District courts
in India.
2. It can be issued against private
individuals too.
INSIGHTS

Page 15

3. It cannot be issued when the detention


of the person is lawful.

Choose the correct answer using the codes given


below.

Choose the correct answer using the codes given


below.
a)
b)
c)
d)

1 and 2 only
2 and 3 only
1 and 3 only
All of the above

73. Consider the following statements about


the Right to Freedom from taxation for
Promotion of religion.
1. The state cannot use public tax money
for the promotion of any religion.
2. The state cannot impose a fee on any
religious activities.
Which of the above is/are true?
a)
b)
c)
d)

1 only
2 only
Both 1 and 2
None

a)
b)
c)
d)

1 and 2 only
2 and 3 only
1 and 3 only
None of the above

75. Judicial review in India is based on the


procedure established by law contained
in the Indian Constitution (Article 21).
What is meant by this principle?
a) Courts can review a Parliamentary
enactment if it is not reasonable.
b) Courts can review a Parliamentary
enactment if it goes against the goals
and objectives enshrined in the
constitution.
c) Courts can review a Parliamentary
enactment only if it was not enacted
following the right procedure.
d) None of the above

74. Consider the following statements about


the Right to Constitutional Remedies.
1. Any court in India can be moved for
enforcing this right.
2. The right to move to courts can never
be suspended under this right.
3. A citizen cannot go directly to the
Supreme Court to get this right
enforced. He has to go
by way of
appeal to higher courts.

http://insightsonindia.com

INSIGHTS

Page 16

Insights Prleims Test Series 2015: Mock Test 14 Solutions

1. Solution: d)
The invisible hand theory is a concept described by Adam Smith in the 18th century, and is
broadly accepted by todays economists as a means to explain the forces of a free
market. Smiths description of the invisible hand theory states that individuals pursuing
their own interest frequently promote the good of society than when he really intends to
promote it. Smith also provides a more direct scenario of the invisible hand theory, It is
not from the benevolence of the butcher, the brewer or the baker, that we expect our dinner,
but from their regard to their own self interest. We address ourselves, not to their humanity
but to their self-love, and never talk to them of our own necessities but of their advantages.
According to the invisible hand theory, each of us, acting in our own self-interests, generates
a demand for goods and services that compels others to deliver those goods and services in
the most efficient manner so that they may be able to receive compensation from others and
make a profit in doing so. In this process, resources are allocated in the most efficient
manner, in contrast to a process that relies on a centrally planned system.

2. Solution: c)
The kind of state-market mix for which India went was thought to be fit for the socioeconomic and political conditions of the time. Once the country started the process of
economic reforms in early 1990s, the prevailing state-market mix was redefined and a new
form of mixed economy began to be practised. As the socioeconomic conditions had
changed the state-market mix also changed. The redefined mixed economy for India had a
declared favour for the market economy. Many economic roles which were under complete
government monopolies were now opened for participation by the private sector.
Examples are many telecommunication, power, roads, oil and natural gas, etc. At the
same time, the responsibilities which were till date being shouldered by the state alone and
which could be taken up by the state only were given extra emphasis. In this category comes
the whole social sectoreducation, healthcare, drinking water, sanitation, nutrition, social
security, etc.

3. Solution: b)
If we look back into the economic history of the world, we see three possible roles for the
State/Government in the economy:
(i) As a regulator of the economic system (where the state takes important economic
decisions, announces the required kind of economic policies, takes the sole responsibility to
http://www.insightsonindia.com

INSIGHTS

Page 1

Insights Prleims Test Series 2015: Mock Test 14 Solutions


get them implemented and controlling and punishing those who dont oblige to those
economic decisions).
(ii) As a producer and/or supplier of the private goods and services (these include all those
goods and services which constitute the part of market and which will be distributed among
the needy according to the principles of the market mechanism. Here the state earns profit as
a private enterprise).
(iii) As a producer and/or supplier of the public goods or the social goods (these include
the goods and services which look essential from the social justice and well-being
perspective for the people. Education, healthcare, sanitation, drinking water, nutrition and
caring for the handicapped and old etc come under this category. These goods which are
generally distributed free of cost at times might reach the beneficiaries at the subsidised
prices. The loss incurred by the state in this way is paid out of the public exchequer which
means that the whole economy pays for the cause of a few people).

4. Solution: c)
The people of India were given eight months to discuss the draft and propose amendments.
In the light of the public comments, criticisms and suggestions, the Drafting Committee
prepared a second draft, which was published in October 1948.
Some provisions of the Constitution pertaining to citizenship, elections, provisional
parliament, temporary and transitional provisions, and short title contained in Articles 5, 6,
7, 8, 9, 60, 324, 366, 367, 379, 380, 388, 391, 392 and 393 came into force on November 26, 1949
itself.

5. Solution: d)
The Constitution of India is the lengthiest of all the written constitutions of the world. It is a
very comprehensive, elaborate and detailed document.
Originally (1949), the Constitution contained a Preamble, 395 Articles (divided into 22 Parts)
and 8 Schedules. Presently (2013), it consists of a Preamble, about 465 Articles (divided into
25 Parts) and 12 Schedules. The various amendments carried out since 1951 have deleted
about 20 Articles and one Part (VII) and added about 85 Articles, four Parts (IVA, IXA, IXB
and XIVA) and four Schedules (9, 10, 11 and 12). No other Constitution in the world has so
many Articles and Schedules.
Centre-state relations have been given in so much detail containing even the minutest of
provisions of taxation, administration and finances. For e.g. which taxes will be
appropriated by either centre or state has been given in the constitution.

http://www.insightsonindia.com

INSIGHTS

Page 2

Insights Prleims Test Series 2015: Mock Test 14 Solutions


Similarily, 5th and 6h schedule areas have not only been defined and demarcated by the
constitution, but also several administrative provisions like tribal advisory council; power of
Gram sabhas in those areas etc. have also been provided.
Four factors have contributed to the elephantine size of our Constitution. They are:
(a) Geographical factors, that is, the vastness of the country and its diversity.
(b) Historical factors, e.g., the influence of the Government of India Act of 1935, which was
bulky.
(c) Single Constitution for both the Centre and the states except Jammu and Kashmir4.
(d) Dominance of legal luminaries in the Constituent Assembly.

6. Solution: b)
Net Domestic Product (NDP) is the GDP calculated after adjusting the weight of the value of
depreciation. This is, basically, net form of the GDP, i.e. GDP minus the total value of the
wear and tear (depreciation) that happened in the assets while the goods and services were
being produced. Every asset (except human beings) go for depreciation in the process of
their uses, which means they wear and tear.
The governments of the economies decide and announce the rates by which assets
depreciate (done in India by the Ministry of Commerce and Industry) and a list is published,
which is used by the different sections of the economy to determine the real levels of
depreciations in different assets.

7. Solution: c)
NDP of an economy has to be always lower than its GDP for the same year, since there is no
way to cut the depreciation to zero. But mankind has achieved too much in this area by the
developments such as ball-bearing, lubricants, etc., all innovated to minimise the levels of
depreciation.
The different uses of the concept of NDP are as given below:
(a) For domestic use only to understand the historical situation of the loss due to
depreciation to the economy. Also used to understand and analyse the sectoral situation of
depreciation in industry and trade in comparative periods.
(b) To show the achievements of the economy in the area of research and development
which have tried cutting the levels of depreciation in a historical time period.

http://www.insightsonindia.com

INSIGHTS

Page 3

Insights Prleims Test Series 2015: Mock Test 14 Solutions


8. Solution: a)
NDP is not used in comparative economics, i.e., to compare the economies of the world.
Why this is so? This is due to different rates of depreciation which is set by the different
economies of the world. Rates of depreciation may be based on logic (as it is in the case of
houses in Indiathe cement, bricks, sand and iron rods which are used to build houses in
India can sustain it for the coming 100 years, thus the rate of depreciation is fixed at 1 per
cent per annum).
But it may not be based on logic all the time - for example, upto Feb 2000 the rate of
depreciation for the heavy vehicles (vehicles with 6-wheels and above) was 20 per cent while
it was done 40 per cent afterwards - to boost the sales of the vehicles. There was no logic in
doubling the rate! Basically, depreciation and its rates are used by the modern governments
as a tool of economic policy-making also, which is the third way how depreciation is used in
economics.

9. Solution: d)
Gross National Product (GNP) is the GDP of a country added with its income from abroad.
Here, the trans-boundary economic activities of an economy is also taken into account. The
items which are counted in the segment Income from Abroad are:
(i) Trade Balance: the net outcome at the year end of the total exports and imports of a
country may be positive or negative accordingly added with the GDP (in Indias case it has
always been negative except the three consecutive years 2000-03 when it was positive, due
to high levels of services sector export during the years courtsey the booming BPO
industry).
(ii) Interest of External Loans: the net outcome on the front of the interest payments i.e.
balance of the inflow (on the money lend out by the economy) and the outflow (on the
money borrowedby the economy) of the external interests. In Indias case it has been always
negative as the economy has been a net borrower from the world economies.
(iii) Private Remittances: the net outcome of the money which inflows and outflows on
account of the private transfers by the Indian nationals working outside India (to India)
and the foreign nationals working in India (to their home countries).

10. Solution: d)
The different uses of the concept GNP are as given below:
(a) This is the national income according to which the IMF ranks the nations of the world in
terms of the volumes at the Purchasing Power Parity (at PPP). For detailed discussion on
the PPP readers may search for it alphabetically in the Chapter- 24. India is ranked as the 4th
http://www.insightsonindia.com

INSIGHTS

Page 4

Insights Prleims Test Series 2015: Mock Test 14 Solutions


largest economy of the world (after the USA, Japan and China) - while as per the nominal/
prevailing exchange rate of rupee India is the 13th largest economy.
(b) It is the more exhaustive concept of national income than the GDP as it indicates about
the quantitative as well as the qualitative aspect of the economy, i.e., the internal as well
as the external strength of the economy.
(c) It enables us to learn several facts about the production behaviour and pattern of an
economy, such as, how much the outside world is dependent on its product and how much
it depends on the world for the same (numerically shown by the size and net flow of its
trade balance); what is the standard of its human resource in international parlance (shown
by the size and the net flow of its private remittances); what position it holds regarding
financial support from and to the world economies (shown by the net flow of interests on
external lending/borrowing).

11. Solution: b)
The different uses of the concept NNP are as given below:
(a) This is the National Income (NI) of an economy. Though, the GDP, NDP and GNP, all
are national income they are not written with capitalised N and I.
(b) This is the purest form of the income of a nation.
(c) When we divide NNP by the total population of nation we get the per capita income
(PCI) of that nation i.e. income per head per year. A very basic point should be noted here
that this is the point where the rates of dipreciation followed by the different nations make a
difference. Higher the rates of depreciation lower the PCI of the nation (whatever be the
reason for it- logical or artificial as in the case of depreciation being used as a tool of
policymaking)! Though, economies are free to fix any rate of depreciation for the different
assets the rates fixed by them make difference when the NI of the nations are compared by
the international financial institutions like the IMF, WB, ADB, etc.

12. Solution: a)
Income of an economy i.e. value of its total produced goods and services may be calculated
at either the factor cost or the market cost. What is the difference between them? Basically,
factor cost is the input cost the producer has to incur in the process of producing
something (such as cost of capital i.e. interest on loans, raw materials, labour, rent, power,
etc.). This is also termed as factory price or production cost/price. This is nothing but
price of the commodity from the producers side. While the market cost is derived after
adding the indirect taxes to the factor cost of the product, it means the cost at which the
goods reach the market i.e. showrooms (these are the Cenvat/central excise and the CST

http://www.insightsonindia.com

INSIGHTS

Page 5

Insights Prleims Test Series 2015: Mock Test 14 Solutions


which are paid by the producers to the Central government in India). This is also known as
the ex-factory price

13. Solution: a)
Similar to the indirect taxes, the various subsidies which are forwarded by the governments
need to be adjusted while calculating national income. They are added to the national
income at market cost, in case of India. Subsidies are added in the national income at the
market cost to derive the national income at factor cost.
This is because the prices at which the subsidised goods and services are made available by
the governments are not their real factor costs (subsidies are forwarded on the factor costs of
the goods and services) otherwise we will have a distorted value (which will be less than its
real value!).
Thus the formula will be:
National Income at Factor Cost = NNP at Market Cost + Subsidies
If the national income is derived at the market cost and governments forward no subsidies
there is no need to adjustments for the subsidies but after all there is not a single economy
in the world today which does not forward subsidies in one or the other form.
Putting indirect taxes and the subsidies both together Indias National Income will be
derived with the following formula (as India does it at the factor cost):
National Income at Factor Cost = NNP at Market Cost Indirect Taxes + Subsidies

14. Solution: d)
The Constitution (Article 4) itself declares that laws made for admission or establishment of
new states (under Article 2) and formation of new states and alteration of areas, boundaries
or names of existing states (under Articles 3) are not to be considered as amendments of the
Constitution under Article 368. This means that such laws can be passed by a simple
majority and by the ordinary legislative process. The Supreme Court in 1969 ruled that,
settlement of a boundary dispute between India and another country does not require a
constitutional amendment. It can be done by executive action as it does not involve cession
of Indian territory to a foreign country.

15. Solution: c)
These are the rights denied to alien citizens:
http://www.insightsonindia.com

INSIGHTS

Page 6

Insights Prleims Test Series 2015: Mock Test 14 Solutions

Right against discrimination on grounds of religion, race, caste, sex or place of birth
(Article 15).
Right to equality of opportunity in the matter of public employment (Article 16).
Right to freedom of speech and expression, assembly, association, movement,
residence and profession (Article 19).
Cultural and educational rights (Articles 29 and 30).
Right to vote in elections to the Lok Sabha and state legislative assembly.
Right to contest for the membership of the Parliament and the state legislature.
Eligibility to hold certain public offices, that is, President of India, Vice-President of
India, judges of the Supreme Court and the high courts, governor of states, attorney
general of India and advocate general of states.

16. Solution: a)
The Constitution deals with the citizenship from Articles 5 to 11 under Part II. However, it
contains neither any permanent nor any elaborate provisions in this regard. It only identifies
the persons who became citizens of India at its commencement (i.e., on January 26, 1950). It
does not deal with the problem of acquisition or loss of citizenship subsequent to its
commencement. It leaves the same to the Parliament.
In India both a citizen by birth as well as a naturalised citizen are eligible for the office of
President while in USA, only a citizen by birth and not a naturalised citizen is eligible for the
office of President.

17. Solution: b)
FRs can be suspended during the operation of a National Emergency except the rights
guaranteed by Articles 20 and 21. Further, the six rights guaranteed by Article 19 can be
suspended only when emergency is declared on the grounds of war or external aggression
(i.e., external emergency) and not on the ground of armed rebellion (i.e., internal
emergency).
Their application can be restricted while martial law is in force in any area. Martial law
means military rule imposed under abnormal circumstances to restore order (Article 34). It
is different from the imposition of national emergency.

18. Solution: d)

http://www.insightsonindia.com

INSIGHTS

Page 7

Insights Prleims Test Series 2015: Mock Test 14 Solutions


The HDI went on to select three broad parameters and allotted them an equal weightage on
the scale of one and measured the development of the countries included in the report. The
three parameters are as given below:
(i) Standard of living: to be indicated by the real per capita income adjusted for the differing
purchasing power parity (PPP).
(ii) Knowledge: to be measured by indicators related to the level of education: (a)
educational attainment among the adults (given 2/3 rd weightage). (b) school enrollment
(given 1/3 rd weightage).
(iii) Life Expectancy: to be calculated at the time of birth.
The UNDP ranks the economies in accordance of their achievements on the above-given
three parameters on the scale of one (i.e. 0.0001.000).

19. Solution: d)
Differences between NITI and Planning Commission
While Planning Commission enjoyed the powers to allocate funds to ministries and state
governments, NITI Aayog will be an advisory body, or a think-tank. Under Planning
Commission, States' role was limited to the National Development Council and annual
interaction during Plan meetings and the commission reported to National Development
Council that had state chief ministers and lieutenant governors of UTs. But Niti Aayogs
Governing Council has state chief ministers and lieutenant governors as the all powerful
body. Under Niti Aayog states are consulted while making policy and deciding on funds
allocation. Final policy would be a result of that consultations unlike under Planning
Commission when policy was formed by the commission and states were then consulted
about allocation of funds. While Niti Aayog is a think-tank and does not have the power to
impose policies, Planning Commission decided policies for states and tied allocation of
funds with projects it approved, a methodology driven by "one size fits all" concept.
PIB Features http://pib.nic.in/newsite/efeatures.aspx

20. Solution: b)
It provided for the establishment of an All-India Federation consisting of provinces and
princely states as units. The Act divided the powers between the Centre and units in terms
of three listsFederal List (for Centre, with 59 items), Provincial List (for provinces, with 54
items) and the Concurrent List (for both, with 36 items). Residuary powers were given to the
Viceroy. However, the federation never came into being as the princely states did not join it.

http://www.insightsonindia.com

INSIGHTS

Page 8

Insights Prleims Test Series 2015: Mock Test 14 Solutions


It abolished dyarchy in the provinces and introduced provincial autonomy in its place. The
provinces were allowed to act as autonomous units of administration in their defined
spheres.
Moreover, the Act introduced responsible governments in provinces, that is, the governor
was required to act with the advice of ministers responsible to the provincial legislature.
This came into effect in 1937 and was discontinued in 1939.
It extended franchise. About 10 per cent of the total population got the voting right.

21. Solution: d)
The revised Scheme, prescribes the eligibility criteria for all four categories.
Corporate Social Responsibility (CSR): Corporates now can spend and provide funds for
promotion and development of sports from the funds earmarked for CSR.
In this regard, training to promote rural sports, nationally recognized sports, Paralympic
sports and Olympic sports have been included in the Schedule VII of the Companies Act,
2013.
Non-governmental organizations (NGOs): In case of NGOs, who are working in promotion
and development of sports, a separate category, Sports for Development has been provided for
the NGOs in order to recognize their contributions.

22. Solution: a)
Features of the 1853 Act
1. It separated, for the first time, the legislative and executive functions of the GovernorGenerals council. It provided for addition of six new members called legislative councillors
to the council. In other words, it established a separate Governor-Generals legislative
council which came to be known as the Indian (Central) Legislative Council. This legislative
wing of the council functioned as a mini-Parliament, adopting the same procedures as the
British Parliament. Thus, legislation, for the first time, was treated as a special function of
the government, requiring special machinery and special process.
2. It introduced an open competition system of selection and recruitment of civil servants.
The covenanted civil service3 was thus thrown open to the Indians also. Accordingly, the
Macaulay Committee (the Committee on the Indian Civil Service) was appointed in 1854.
3. It extended the Companys rule and allowed it to retain the possession of Indian
territories on trust for the British Crown. But, it did not specify any particular period, unlike
the previous Charters. This was a clear indication that the Companys rule could be
terminated at any time the Parliament liked.
http://www.insightsonindia.com

INSIGHTS

Page 9

Insights Prleims Test Series 2015: Mock Test 14 Solutions


4. It introduced, for the first time, local representation in the Indian (Central) Legislative
Council. Of the six new legislative members of the governor-generals council, four members
were appointed by the local (provincial) governments of Madras, Bombay, Bengal and Agra.

23. Solution: c)
This significant Act was enacted in the wake of the Revolt of 1857also known as the First
War of Independence or the sepoy mutiny. The act known as the Act for the Good
Government of India, abolished the East India Company, and transferred the powers of
government, territories and revenues to the British Crown.
Features of the Act
1. It provided that India henceforth was to be governed by, and in the name of, Her Majesty.
It changed the designation of the Governor-General of India to that of Viceroy of India. He
(viceroy) was the direct representative of the British Crown in India. Lord Canning thus
became the first Viceroy of India.
2. It ended the system of double government by abolishing the Board of Control and Court
of Directors.
3. It created a new office, Secretary of State for India, vested with complete authority and
control over Indian administration. The secretary of state was a member of the British
cabinet and was responsible ultimately to the British Parliament.
4. It established a 15-member Council of India to assist the secretary of state for India. The
council was an advisory body. The secretary of state was made the chairman of the council.
5. It constituted the secretary of state-in-council as a body corporate, capable of suing and
being sued in India and in England.
The Act of 1858 was, however, largely confined to the improvement of the administrative
machinery by which the Indian Government was to be supervised and controlled in
England. It did not alter in any substantial way the system of government that prevailed in
India.

24. Solution: d)
Some Features of the Act of 1861
It made a beginning of representative institutions by associating Indians with the lawmaking process. It thus provided that the viceroy should nominate some Indians as nonofficial members of his expanded council. In 1862, Lord Canning, the then viceroy,

http://www.insightsonindia.com

INSIGHTS

Page 10

Insights Prleims Test Series 2015: Mock Test 14 Solutions


nominated three Indians to his legislative councilthe Raja of Benaras, the Maharaja of
Patiala and Sir Dinkar Rao.
Some Features of the Act of 1892
1. It increased the number of additional (non-official) members in the Central and provincial
legislative councils, but maintained the official majority in them.
2. It increased the functions of legislative councils and gave them the power of discussing
the budget and addressing questions to the executive.
Act of 1909 considerably increased the size of the legislative councils, both Central and
provincial. It retained official majority in the Central Legislative Council but allowed the
provincial legislative councils to have non-official majority.
It enlarged the deliberative functions of the legislative councils at both the levels. For
example, members were allowed to ask supplementary questions, move resolutions on the
budget, and so on.

25. Solution: d)
Features of the Act
1. It relaxed the central control over the provinces by demarcating and separating the central
and provincial subjects. The central and provincial legislatures were authorised to make
laws on their respective list of subjects. However, the structure of government continued to
be centralised and unitary.
2. It further divided the provincial subjects into two partstransferred and reserved. The
transferred subjects were to be administered by the governor with the aid of ministers
responsible to the legislative Council. The reserved subjects, on the other hand, were to be
administered by the governor and his executive council without being responsible to the
legislative Council. This dual scheme of governance was known as dyarchya term
derived from the Greek word di-arche which means double rule. However, this experiment
was largely unsuccessful.
3. It introduced, for the first time, bicameralism and direct elections in the country. Thus, the
Indian Legislative Council was replaced by a bicameral legislature consisting of an Upper
House (Council of State) and a Lower House (Legislative Assembly). The majority of
members of both the Houses were chosen by direct election.
4. It required that the three of the six members of the Viceroys executive Council (other than
the commander-in-chief) were to be Indian.
5. It extended the principle of communal representation by providing separate electorates
for Sikhs, Indian Christians, Anglo-Indians and Europeans.

http://www.insightsonindia.com

INSIGHTS

Page 11

Insights Prleims Test Series 2015: Mock Test 14 Solutions

26. Solution: d)
In 1927, the British Government announced the appointment a seven-member statutory
commission under the chairmanship of Sir John Simon to report on the condition of India
under its new Constitution. All the members of the commission were British and hence, all
the parties boycotted the commission.
The commission submitted its report in 1930 and recommended the abolition of dyarchy,
extension of responsible government in the provinces, establishment of a federation of
British India and princely states, continuation of communal electorate and so on. To consider
the proposals of the commission, the British Government convened three round table
conferences of the representatives of the British Government, British India and Indian
princely states.
On the basis of these discussions, a White Paper on Constitutional Reforms was prepared
and submitted for the consideration of the Joint Select Committee of the British Parliament.
The recommendations of this committee were incorporated (with certain changes) in the
next Government of India Act of 1935.

27. Solution: a)
Maharashtra Government is planning to revamp the Jeevan Amrut Yojana(blood-oncall) scheme to make it more beneficial to people.
State government is seeking to strengthen the network of blood storage units in rural areas
instead of current mechanism of transporting blood through motorcycles.
About Jeevan Amrut Yojana

It is first of its kind scheme in the country, launched in 2014.

It aimed at facilitating blood supply to patients scheduled to undergo surgeries and


medical procedures in remote areas.

The patients, who need the blood, have to pay 450 rupees towards the cost of blood
and 50 rupees to 100 rupees as transportation cost depending on the distance after
the request (for blood) is placed by the hospital concerned.

Upon receiving the call, the required blood and blood components are transported
on motorcycle to hospitals and nursing homes, in specially fitted cold storage boxes.

28. Solution: a)
Top 10 foreign investment destinations in 2014 ($ Billion)
http://www.insightsonindia.com

INSIGHTS

Page 12

Insights Prleims Test Series 2015: Mock Test 14 Solutions


Rank Country

FDI*

China

128

Hong Kong

111

United States

86

Singapore

81

Brazil

62

United Kingdom

61

Canada

53

Australia

49

Netherlands

42

10

Luxembourg

36

29. Solution: a)

The 21st century MSR project is an initiative by China to resurrect the ancient
maritime Silk Road. It is perceived to be an attempt by China to ameliorate relations
with South and Southeast Asia- in this case the focus is on maritime trade security.

The project will prioritize construction ports and infrastructure, industrial parks in
strategically significant countries in Southeast Asia, the Mideast, Africa, Europe and
the Indian Ocean region, which includes Bangladesh and Sri Lanka.

China has already begun building of port projects in Gwadar (Pakistan), Hambantota
(Sri Lanka) and Chittagong (Bangladesh) respectively.

China has announced a USD 40 billion Silk Road fund which became operational in
February 2015 for attracting small countries in the projects.

30. Solution: a)
In 1942, Sir Stafford Cripps, a member of the cabinet, came to India with a draft proposal of
the British Government on the framing of an independent Constitution to be adopted after
the World War II. The Cripps Proposals were rejected by the Muslim League which wanted
India to be divided into two autonomous states with two separate Constituent Assemblies.
http://www.insightsonindia.com

INSIGHTS

Page 13

Insights Prleims Test Series 2015: Mock Test 14 Solutions


Finally, a Cabinet Mission was sent to India. While it rejected the idea of two Constituent
Assemblies, it put forth a scheme for the Constituent Assembly which more or less satisfied
the Muslim League.
Composition of The Constituent Assembly
The Constituent Assembly was constituted in November 1946 under the scheme formulated
by the Cabinet Mission Plan. The features of the scheme were:
1. The total strength of the Constituent Assembly was to be 389. Of these, 296 seats were to
be allotted to British India and 93 seats to the Princely States. Out of 296 seats allotted to the
British India, 292 members were to be drawn from the eleven governors provinces2 and
four from the four chief commissioners provinces3, one from each.
2. Each province and princely state (or group of states in case of small states) were to be
allotted seats in proportion to their respective population. Roughly, one seat was to be
allotted for every million population.
3. Seats allocated to each British province were to be decided among the three principal
communitiesMuslims, Sikhs and general (all except Muslims and Sikhs), in proportion to
their population.
4. The representatives of each community were to be elected by members of that community
in the provincial legislative assembly and voting was to be by the method of proportional
representation by means of single transferable vote.
5. The representatives of princely states were to be nominated by the heads of the princely
states.

31. Solution: d)
If the secondary sector contributes 50 per cent or more to the total produce value of an
economy, it is an industrial economy. Higher the contribution, higher is the level of
industrialisation. The western economies who went for early industrialisation earning faster
and enough income and developing early were known as developed economies. Most of
these economies have crossed this phase once the process of industrialisation saturated.
As country after country successfully industrialised, a pattern of the population shift from
one to another sector was established, which was known as the stages of growth of an
economy. 7 With the intensification of industrialisation, dependency on primary sector for
livelihood decreased and dependency on secondary sector increased consistently. Similarly,
such economies saw a population shift from the secondary to the tertiary sectorand these
were known as the post-industrial societies or the service societies.
Almost the whole Euro-America falls under this categorythese economies are having over
50 per cent of their total produce value being contributed by their tertiary sectors and over
http://www.insightsonindia.com

INSIGHTS

Page 14

Insights Prleims Test Series 2015: Mock Test 14 Solutions


half of the population depends on the sector for their livelihood. Many other countries
which started industrialisation in the post-war period did show abberations in this shift of
the population and the incomeIndia being one among them.

32. Solution: a)
Option A is also the financial year of India. Since the annual budget is also prepared,
presented, approved and passed presented during the starting of this period, it is
appropriate to calculate GDP in that period also.

33. Solution: a)
The different uses of the concept GDP are as given below:
(i) Per annum percentage change in it is the growth rate of an economy. For example, if a
country has a GDP of Rs. 107 which is 7 rupees higher than the last year, it has a growth rate
of 7 per cent. When we use the term a growing economy, it means that the economy is
adding up its income i.e. in quantitative terms.
(ii) It is a quantitative concept and its volume/size indicates the internal strength of the
economy. But it does not say anything about the qualitative aspects of the produced goods
and services by the economy.
(iii) It is used by the IMF/WB in the comparative analyses of its member nations.

34. Solution: d)
The structural part of the Constitution is, to a large extent, derived from the Government of
India Act of 1935. The philosophical part of the Constitution (the Fundamental Rights and
the Directive Principles of State Policy) derives their inspiration from the American and Irish
Constitutions respectively. The political part of the Constitution (the principle of Cabinet
Government and the relations between the executive and the legislature) has been largely
drawn from the British Constitution.
In one of the explanations above, refer to the provision of the GoI act, 1935 to understand
how these provisions affected the Indian constitution.

35. Solution: d)
This case is not a violation of fundamental right. A hindu temple managed by the state is not
a public place where free movement is unrestricted. It is a place which belongs to a certain

http://www.insightsonindia.com

INSIGHTS

Page 15

Insights Prleims Test Series 2015: Mock Test 14 Solutions


religious community. It has all the constitutional rights to block the entry of people
belonging to other religious communities.
Hence, none of the above options would be appropriate as this case is not a violation of
fundamental right.

36. Solution: b)
The Constitution of India has opted for the British parliamentary System of Government
rather than American Presidential System of Government. The parliamentary system is
based on the principle of cooperation and co-ordination between the legislative and
executive organs while the presidential system is based on the doctrine of separation of
powers between the two organs.
The parliamentary system is also known as the Westminster model of government,
responsible government and cabinet government. The Constitution establishes the
parliamentary system not only at the Centre but also in the states. The features of
parliamentary government in India are:
(a) Presence of nominal and real executives;
(b) Majority party rule,
(c) Collective responsibility of the executive to the legislature,
(d) Membership of the ministers in the legislature,
(e) Leadership of the prime minister or the chief minister,
(f) Dissolution of the lower House (Lok Sabha or Assembly).

37. Solution: c)
The Constitution of India establishes a federal system of government. It contains all the
usual features of a federation, viz., two government, division of powers, written
Constitution, supremacy of Constitution, rigidity of Constitution, independent judiciary and
bicameralism.
However, the Indian Constitution also contains a large number of unitary or non-federal
features, viz., a strong Centre, single Constitution, single citizenship, flexibility of
Constitution, integrated judiciary, appointment of state governor by the Centre, all-India
services, emergency provisions, and so on.
Moreover, the term Federation has nowhere been used in the Constitution. Article 1, on the
other hand, describes India as a Union of States which implies two things: one, Indian

http://www.insightsonindia.com

INSIGHTS

Page 16

Insights Prleims Test Series 2015: Mock Test 14 Solutions


Federation is not the result of an agreement by the states; and two, no state has the right to
secede from the federation.
Hence, the Indian Constitution has been variously described as federal in form but unitary
in spirit,quasi-federal by K C Wheare, bargaining federalism by Morris Jones, cooperative federalism by Granville Austin, federation with a centralising tendency by Ivor
Jennings, and so on.

38. Solution: a)
After the acceptance of the Mountbatten Plan of June 3, 1947 for a partition of the country,
the representatives of most of the other princely states took their seats in the Assembly. The
members of the Muslim League from the Indian Dominion also entered the Assembly. The
Indian Independence Act of 1947 made the following three changes in the position of the
Assembly:
1. The Assembly was made a fully sovereign body, which could frame any Constitution it
pleased. The act empowered the Assembly to abrogate or alter any law made by the British
Parliament in relation to India.
2. The Assembly also became a legislative body. In other words, two separate functions were
assigned to the Assembly, that is, making of a constitution for free India and enacting of
ordinary laws for the country. These two tasks were to be performed on separate days.
Thus, the Assembly became the first Parliament of free India (Dominion Legislature).
Whenever the Assembly met as the Constituent body it was chaired by Dr. Rajendra Prasad
and when it met as the legislative body, it was chaired by G V Mavlankar. These two
functions continued till November 26, 1949, when the task of making the Constitution was
over.

39. Solution: d)
It was this committee that was entrusted with the task of preparing a draft of the new
Constitution. It consisted of seven members. They were:

Dr B R Ambedkar (Chairman)
N Gopalaswamy Ayyangar
Alladi Krishnaswamy Ayyar
Dr K M Munshi
Syed Mohammad Saadullah
N Madhava Rau (He replaced B L Mitter who resigned due to ill-health)
T T Krishnamachari (He replaced D P Khaitan who died in 1948)

http://www.insightsonindia.com

INSIGHTS

Page 17

Insights Prleims Test Series 2015: Mock Test 14 Solutions


The Drafting Committee, after taking into consideration the proposals of the various
committees, prepared the first draft of the Constitution of India, which was published in
February 1948. The people of India were given eight months to discuss the draft and
propose amendments. In the light of the public comments, criticisms and suggestions, the
Drafting Committee prepared a second draft, which was published in October 1948.

40. Solution: a)
In the arena of distribution network, we have three historically existing modelsstate,
market and statemarket mix. In the first kind of distribution system, the state (i.e. the
government) takes the sole responsibility of supplying the goods and services required by
the population with no payments being done by the consumerThe former Soviet Union
and Communist China being the best examples.
In the second categories comes the market mode of distribution which functions on the basis
of price mechanism. In this system, goods and services are made available in the market and
on the basis of their demand and supply, their prices are determined in the open market and
finally they get distributed to the population. This was the distribution system of the
capitalist economiesthe whole Euro-America till 1930s.

41. Solution: a)
The political leadership selected industry as the leading force of the economy after
independence this was already decided by the dominant group of the nationalist leaders
way back in the mid-1930s when they felt the need for economic planning in India before
setting up the National Planning Committee (1938). Given the available resource base it
seems an illogical decision as India lacked all those pre-requisites which could suggest the
declaration of industry as its prime mover:
(i) Almost no presence of infrastructure sector i.e. power, transportation and
communication.
(ii) Negligible presence of the infrastructure industries i.e. iron and steel, cement, coal, crude
oil, oil refining and electricity.
(iii) Lack of investible capitalbe the case of either the government or the private sector.
(iv) Absence of the required technology to support the process of industrialisation and no
research and development.
(v) Lack of skilled manpower.
(vi) Absence of entrepreneurship among the people.
(vii) Absence of the market for industrial goods.
http://www.insightsonindia.com

INSIGHTS

Page 18

Insights Prleims Test Series 2015: Mock Test 14 Solutions


(viii) Many other socio-psychological factors which were negative forces for the proper
industrialisation of the economy.

42. Solution: a)
There are six basic industries which every industrialising economy requires, namely
(i) Iron and Steel (ii) Cement (iii) Coal (iv) Crude oil (v) Oil refining and (vi) Electricity.
At present, there are eight Core Industries in India (with the Base: 2004-05=100), six existing
basic/infrastructure industries with two new additions i.e. Natural Gas and Fertilizer.
Similar to the infrastructure sector, these basic industries also require high level of capital,
technology, skilled manpower and articulation in entrepreneurship which was again
considered not feasible for the private sector of the time to manage. Even if the private sector
supplied goods from the basic industries they might not be able to sell their products in the
market due to the lower purchasing power of the consumers

43. Solution: d)
The investment to be made by the government in the PSUs was in the nature of asset
creation and these entities were to be involved in production activities. It was natural for the
government to gain control over the profits and dividends accruing from them. The goods
and services the PSUs were to produce and sell were going to provide disposable income to
the government. The government had a conscious policy of spending the income generated
by the PSUs. They were to be used in the supply of the social goods or what is called the
public goods. And thus, India was to have a developed social sector.
By social goods the government meant the universal supply of certain goods and services to
the Indian people. They included education, healthcare, nutrition, drinking water, social
security, etc. in India. It means that the PSUs were also visioned as the revenue generators
for the development of the social sector.
Due to many reasons the PSUs would not be able to generate as much profit as was required
for the healthy development of the social sector. This eventually hampered the availability
of public goods in the country. In place of giving profits back to the government, a very high
number of the PSUs started incurring huge losses and required budgetary supports as a
regular phenomenon.

44. Solution: a)
Charter Act of 1833
This Act was the final step towards centralisation in British India.
http://www.insightsonindia.com

INSIGHTS

Page 19

Insights Prleims Test Series 2015: Mock Test 14 Solutions


Features of the Act
1. It made the Governor-General of Bengal as the Governor-General of India and vested in
him all civil and military powers. Thus, the act created, for the first time, a Government of
India having authority over the entire territorial area possessed by the British in India. Lord
William Bentick was the first governor-general of India.
2. It deprived the governor of Bombay and Madras of their legislative powers. The
Governor-General of India was given exclusive legislative powers for the entire British
India. The laws made under the previous acts were called as Regulations while laws made
under this act were called as Acts.
3. It ended the activities of the East India Company as a commercial body, which became a
purely administrative body. It provided that the companys territories in India were held by
it in trust for His Majesty, His heirs and successors.
4. The Charter Act of 1833 attempted to introduce a system of open competition for selection
of civil servants, and stated that the Indians should not be debarred from holding any place,
office and employment under the Company. However, this provision was negated after
opposition from the Court of Directors.

45. Solution: a)
The capitalistic form of economy has its origin in the famous work of Adam SmithWealth
of Nations (1776). Adam Smith (17231790), the Scottish philosopher-economist professor at
Glasgow University whose writings formed the basis of classical economics had stressed
certain fine ideas which were to take fancy among some of the western countries and finally
capitalism took birth. He raised his voice against the heavy-handed government regulation
of commerce and industry of the time which did not allow the economy to tap its full
economic worth and reach the level of wellbeing.
Stressing division of labour, an environment of laissez faire (non-interference by the
government) he proposed that the invisible hand of the market forces (price mechanism)
will bring a state of equilibrium to the economy and a general well-being to the countrymen.
For such an economy to function for public well-being, he has acknowledged the need of
competition in the market.

46. Solution: d)
In tribal areas, fifth and sixth schedule give them greater autonomy, more political rights
and a different political structure alongwith regular special incentives from the Union and
State governments.

http://www.insightsonindia.com

INSIGHTS

Page 20

Insights Prleims Test Series 2015: Mock Test 14 Solutions


In J&K, acts like AFSPA, different constitution, special category status, Article 370 of the
constitution etc. create a difference in the status of citizens.
Article 371 contains special provisions for Maharashtra and Gujarat. Then follow a stream of
special provisions, including Articles 371A (for Nagaland), 371B (Assam), 371C (Manipur),
371D and E (Andhra Pradesh), 371F (Sikkim), 371G (Mizoram), 371H (Arunachal Pradesh),
and 371(I) (Goa).

47. Solution: d)
The Indian Constitution not only provides for the legislative, executive and judicial organs
of the government (Central and state) but also establishes certain independent bodies. They
are envisaged by the Constitution as the bulwarks of the democratic system of Government
in India. These are CAG, ECI, UPSC, state public service Commissions etc.
The Constitution ensures the independence of these bodies through various provisions like
security of tenure, fixed service conditions, expenses being charged on the Consolidated
Fund of India, and so on.

48. Solution: a)
The Indian Constitution contains elaborate emergency provisions to enable the President to
meet any extraordinary situation effectively. The rationality behind the incorporation of
these provisions is to safeguard the sovereignty, unity, integrity and security of the country,
the democratic political system and the Constitution.
The Constitution envisages three types of emergencies, namely:
(a) National emergency on the ground of war or external aggression or armed rebellion16
(Article 352);
(b) State emergency (Presidents Rule) on the ground of failure of Constitutional machinery
in the states (Article 356) or failure to comply with the directions of the Centre (Article 365);
and
(c) Financial emergency on the ground of threat to the financial stability or credit of India
(Article 360).

49. Solution: b)
The Preamble reveals four ingredients or components:

Source of authority of the Constitution: The Preamble states that the Constitution
derives its authority from the people of India.

http://www.insightsonindia.com

INSIGHTS

Page 21

Insights Prleims Test Series 2015: Mock Test 14 Solutions

Nature of Indian State: It declares India to be of a sovereign, socialist, secular


democratic and republican polity.
Objectives of the Constitution: It specifies justice, liberty, equality and fraternity as
the objectives.
Date of adoption of the Constitution: It stipulates November 26, 1949 as the date.

50. Solution: d)
Notably, the Territory of India is a wider expression than the Union of India because the
latter includes only states while the former includes not only the states but also union
territories and territories that may be acquired by the Government of India at any future
time. The states are the members of the federal system and share a distribution of powers
with the Centre. The union territories and the acquired territories, on the other hand, are
directly administered by the Central government.
Being a sovereign state, India can acquire foreign territories according to the modes
recognised by international law, i.e., cession (following treaty, purchase, gift, lease or
plebiscite), occupation (hitherto unoccupied by a recognised ruler), conquest or subjugation.
For example, India acquired several foreign territories such as Dadra and Nagar Haveli;
Goa, Daman and Diu; Puducherry; and Sikkim since the commencement of the Constitution.
Notably, Article 2 relates to the admission or establishment of new states that are not part of
the Union of India. Article 3, on the other hand, relates to the formation of or changes in the
existing states of the Union of India. In other words,
Article 3 deals with the internal re-adjustment inter se of the territories of the constituent
states of the Union of India.

51. Solution: a)
Article 3 authorises the Parliament to:
(a) form a new state by separation of territory from any state or by uniting two or more
states or parts of states or by uniting any territory to a part of any state,
(b) Increase the area of any state,
(c) Diminish the area of any state,
(d) Alter the boundaries of any state, and
(e) Alter the name of any state.

52. Solution: d)
http://www.insightsonindia.com

INSIGHTS

Page 22

Insights Prleims Test Series 2015: Mock Test 14 Solutions


The Supreme Court stands at the top of the integrated judicial system in the country. Below
it, there are high courts at the state level. Under a high court, there is a hierarchy of
subordinate courts, that is, district courts and other lower courts. This single system of
courts enforces both the central laws as well as the state laws, unlike in USA, where the
federal laws are enforced by the federal judiciary and the state laws are enforced by the state
judiciary.
The Supreme Court is a federal court, the highest court of appeal, the guarantor of the
fundamental rights of the citizens and the guardian of the Constitution. Hence, the
Constitution has made various provisions to ensure its independencesecurity of tenure of
the judges, fixed service conditions for the judges, all the expenses of the Supreme Court
charged on the Consolidated Fund of India, prohibition on discussion on the conduct of
judges in the legislatures, ban on practice after retirement, power to punish for its contempt
vested in the Supreme Court, separation of the judiciary from the executive, and so on.
Courts are not law enforcers; they are the upholders of the law. Laws are enforced by
executive agencies.

53. Solution: a)
According to Dr B R Ambedkar, the Directive Principles of State Policy is a novel feature of
the Indian Constitution. They are enumerated in Part IV of the Constitution. They can be
classified into three broad categoriessocialistic, Gandhian and liberalintellectual.
They impose a moral obligation on the state author-ities for their application. But, the real
force (sanction) behind them is political, that is, public opinion.
In the Minerva Mills case (1980), the Supreme Court held that the Indian Constitution is
founded on the bedrock of the balance between the Fundamental Rights and the Directive
Principles.

54. Solution: d)
The term secular was added to the Preamble of the Indian Constitution by the 42 nd
Constitutional Amendment Act of 1976.
(b) The Preamble secures to all citizens of India liberty of belief, faith and worship.
(c) The State shall not deny to any person equality before the law or equal protection of the
laws (Article 14).
(d) The State shall not discriminate against any citizen on the ground of religion (Article 15).
(e) Equality of opportunity for all citizens in matters of public employment (Article 16).

http://www.insightsonindia.com

INSIGHTS

Page 23

Insights Prleims Test Series 2015: Mock Test 14 Solutions


(f) All persons are equally entitled to freedom of conscience and the right to freely profess,
practice and propagate any religion (Article 25).
(g) Every religious denomination or any of its section shall have the right to manage its
religious affairs (Article 26).
(h) No person shall be compelled to pay any taxes for the promotion of a particular religion
(Article 27).
(i) No religious instruction shall be provided in any educational institution maintained by
the State (Article 28).
(j) Any section of the citizens shall have the right to conserve its distinct language, script or
culture (Article 29).
(k) All minorities shall have the right to establish and administer educational institutions of
their choice (Article 30).
(l) The State shall endeavour to secure for all the citizens a Uniform Civil Code (Article 44).

55. Solution: a)
Article 3 lays down two conditions in this regard: one, a bill contemplating the above
changes can be introduced in the Parliament only with the prior recommendation of the
President; and two, before recommending the bill, the President has to refer the same to the
state legistature concerned for expressing its views within a specified period.
Further, the power of Parliament to form new states includes the power to form a new state
or union territory by uniting a part of any state or union territory to any other state or union
territory3.
The President (or Parliament) is not bound by the views of the state legislature and may
either accept or reject them, even if the views are received in time. Further, it is not necessary
to make a fresh reference to the state legislature every time an amendment to the bill is
moved and accepted in Parliament.

56. Solution: d)
Article 12 has defined the term for the purposes of Part III. According to it, the State includes
the following:

Government and Parliament of India, that is, executive and legislative organs of the
Union government.

http://www.insightsonindia.com

INSIGHTS

Page 24

Insights Prleims Test Series 2015: Mock Test 14 Solutions

Government and legislature of states, that is, executive and legislative organs of state
government.
All local authorities, that is, municipalities, panchayats, district boards, improvement
trusts, etc.
All other authorities, that is, statutory or non-statutory authorities like LIC, ONGC,
SAIL, etc.

Thus, State has been defined in a wider sense so as to include all its agencies. It is the actions
of these agencies that can be challenged in the courts as violating the Fundamental Rights.
According to the Supreme Court, even a private body or an agency working as an
instrument of the State falls within the meaning of the State under Article 12.

57. Solution: a)
The term connotes: (a) the equality of treatment under equal circumstances, both in the
privileges conferred and liabilities imposed by the laws, (b) the similar application of the
same laws to all persons who are similarly situated, and (c) the like should be treated alike
without any discrimination. Thus, the former is a negative concept while the latter is a
positive concept. However, both of them aim at establishing equality of legal status,
opportunity and justice.
The Supreme Court held that where equals and unequals are treated differently, Article 14
does not apply. While Article 14 forbids class legislation, it permits reasonable classification
of persons, objects and transactions by the law. But the classification should not be arbitrary,
artificial or evasive. Rather, it should be based on an intelligible differential and substantial
distinction.
Equality of opportunity is a different fundamental right under Article 16.

58. Solution: b)
It literally means we command. It is a command issued by the court to a public official
asking him to perform his official duties that he has failed or refused to perform. It can also
be issued against any public body, a corporation, an inferior court, a tribunal or government
for the same purpose.
The writ of mandamus cannot be issued (a) against a private individual or body; (b) to
enforce departmental instruction that does not possess statutory force; (c) when the duty is
discretionary and not mandatory; (d) to enforce a contractual obligation; (e) against the
president of India or the state governors; and (f) against the chief justice of a high court
acting in judicial capacity.

http://www.insightsonindia.com

INSIGHTS

Page 25

Insights Prleims Test Series 2015: Mock Test 14 Solutions


59. Solution: a)
Freedom of Speech and Expression It implies that every citizen has the right to express his
views, opinions, belief and convictions freely by word of mouth, writing, printing, picturing
or in any other manner.

Right to propagate ones views as well as views of others.


Freedom of the press.
Freedom of commercial advertisements.
Right against tapping of telephonic conversation.
Right to telecast, that is, government has no monopoly on electronic media.
Right against bandh called by a political party or organisation.
Right to know about government activities.
Freedom of silence.
Right against imposition of pre-censorship on a newspaper.
Right to demonstration or picketing but not right to strike.

The State can impose reasonable restrictions on the exercise of the freedom of speech and
expression on the grounds of sovereignty and integrity of India, security of the state,
friendly relations with foreign states, public order, decency or morality, contempt of court,
defamation, and incitement to an offence.

60. Solution: b)
The Supreme Court has reaffirmed its judgement in the Menaka case in the subsequent cases.
It has declared the following rights as part of Article 21:

Right to live with human dignity.


Right to decent environment including pollution free water and air and protection
against
Hazardous industries.
Right to livelihood.
Right to privacy.
Right to shelter.
Right to health.
Right to free education up to 14 years of age.
Right to free legal aid.
Right against solitary confinement.
Right to speedy trial.
Right against handcuffing.
Right against inhuman treatment.
Right against delayed execution.
Right to travel abroad.
Right against bonded labour.

http://www.insightsonindia.com

INSIGHTS

Page 26

Insights Prleims Test Series 2015: Mock Test 14 Solutions

Right against custodial harassment.


Right to emergency medical aid.
Right to timely medical treatment in government hospital.
Right not to be driven out of a state.
Right to fair trial.
Right of prisoner to have necessities of life.
Right of women to be treated with decency and dignity.
Right against public hanging.
Right to hearing.
Right to information.
Right to reputation.

61. Solution: a)
The expression traffic in human beings include (a) selling and buying of men, women and
children like goods; (b) immoral traffic in women and children, including prostitution; (c)
devadasis; and (d) slavery. To punish these acts, the Parliament has made the Immoral Traffic
(Prevention) Act, 1956.
The term begar means compulsory work without remuneration. It was a peculiar Indian
system under which the local zamindars sometimes used to force their tenants to render
services without any payment. In addition to begar, the Article 23 prohibits other similar
forms of forced labour like bonded labour. The term forced labour means compelling a
person to work against his will. The word force includes not only physical or legal force
but also force arising from the compulsion of economic circumstances, that is, working for
less than the minimum wage. In this regard, the Bonded Labour System (Abolition) Act,
1976; the Minimum Wages Act, 1948; the Contract Labour Act, 1970 and the Equal
Remuneration Act, 1976 were made.

62. Solution: a)
http://indianexpress.com/article/india/india-others/explained-nuclear-deal-moves-butsome-way-still-to-go/

63. Solution: d)

Carbon capture properties- These species have excellent carbon capture properties
i.e. ability to absorb carbon dioxide from the atmosphere. So it can be used as biofuels and help reduce global warming.

http://www.insightsonindia.com

INSIGHTS

Page 27

Insights Prleims Test Series 2015: Mock Test 14 Solutions

Endemic- Both of them are endemic species to the western coast of India. So, their
cultivation is not going to cause any environmental harm and reduces the risk of bio
invasion in this region.

Bloom forming- Both species being bloom forming they have spontaneous growth.
So there is no need for any expensive cultivation systems such as photobioreactors or
extensive use of fertilizers/pesticides. They can grow sporadically at shorelines and
can sequester CO.

Medicinal values- Both species have similar characteristic like other algal species
which have Kahalalide-F , used in in clinical trials against prostate and breast cancers.
So they can be used in Pharmaceutical products.

It should be noted that, this discovery was the first molecular study on Indian algae, and
first algal species discovered in past 40 years.

64. Solution: d)
Democracy is of two typesdirect and indirect. In direct democracy, the people exercise
their supreme power directly as is the case in Switzerland. There are four devices of direct
democracy, namely, Referendum, Initiative, Recall and Plebiscite. In indirect democracy,
on the other hand, the representatives elected by the people exercise the supreme power and
thus carry on the government and make the laws. This type of democracy, also known as
representative democracy, is of two kindsparliamentary and presidential.
The Indian Constitution provides for representative parliamentary democracy under which
the executive is responsible to the legislature for all its policies and actions.
Referendum is a procedure whereby a proposed legislation is referred to the electorate for
settlement by their direct votes.
Initiative is a method by means of which the people can propose a bill to the legislature for
enactment.
Recall is a method by means of which the voters can remove a representative or an officer
before the expiry of his term, when he fails to discharge his duties properly.
Plebiscite is a method of obtaining the opinion of people on any issue of public importance.
It is generally used to solve the territorial disputes.

65. Solution: a)
The Preamble secures to all citizens of India equality of status and opportunity. This
provision embraces three dimensions of equalitycivic, political and economic.

http://www.insightsonindia.com

INSIGHTS

Page 28

Insights Prleims Test Series 2015: Mock Test 14 Solutions


The following provisions of the chapter on Fundamental Rights ensure civic equality:
(a) Equality before the law (Article 14).
(b) Prohibition of discrimination on grounds of religion, race, caste, sex or place of birth
(Article 15).
(c) Equality of opportunity in matters of public employment (Article 16).
(d) Abolition of untouchability (Article 17).
(e) Abolition of titles (Article 18).
There are two provisions in the Constitution that seek to achieve political equality. One, no
person is to be declared ineligible for inclusion in electoral rolls on grounds of religion, race,
caste or sex (Article 325). Two, elections to the Lok Sabha and the state assemblies to be on
the basis of adult suffrage (Article 326).

66. Solution: c)
Like any other part of the Constitution, the Preamble was also enacted by the Constituent
Assembly, but, after the rest of the Constitution was already enacted. The reason for
inserting the Preamble at the end was to ensure that it was in conformity with the
Constitution as adopted by the Constituent Assembly. While forwarding the Preamble for
votes, the president of the Constituent Assembly said, The question is that Preamble stands
part of the Constitution. The motion was then adopted.
Hence, the current opinion held by the Supreme Court that the Preamble is a part of the
Constitution, is in consonance with the opinion of the founding fathers of the Constitution.
However, two things should be noted:

The Preamble is neither a source of power to legislature nor a prohibition upon the
powers of legislature.
It is non-justiciable, that is, its provisions are not enforceable in courts of law.

67. Solution: d)
De-worming
It is administering an antihelmintic drug to a human or animal to rid them of parasites, such
as roundworm, hookworms, flukes and tapeworm. Mass deworming campaigns of school
children have been used both as a preventive as well as a treatment method
for helminthiasis which includes soil transmitted helminthiasis in children. Children can be
treated by administering for example Mebendazole and Albendazole. The cost is relatively
low. One tablet of Albendazole rids the child of parasitic worms which live in the childs
http://www.insightsonindia.com

INSIGHTS

Page 29

Insights Prleims Test Series 2015: Mock Test 14 Solutions


intestines and eat the nutrients the child needs for healthy mental and physical
development. This tablet is safe for both infected and non-infected children and has a
pleasant flavor.
Helminths
They are a group of parasites commonly referred to as worms and include schistosomes and
soil-transmitted helminths. Such infections are among the most common infections in
developing countries. While mild infections often go unnoticed, more severe worm
infections can lead to abdominal pain, listlessness, iron-deficiency anaemia, malnutrition,
stunting, and wasting.. Infections can also cause cognitive impairment as well as tissue
damage that may require corrective surgery.

68. Solution: d)
There are several challenges preventing Indias rural poor from competing in the modern
market, such as the lack of formal education and marketable skills. DDU-GKY bridges this
gap by funding training projects benchmarked to global standards, with an emphasis on
placement, retention, career progression and foreign placement.
Features of Deen Dayal Upadhyaya Grameen Kaushalya Yojana

Enable Poor and Marginalized to Access Benefits

Demand led skill training at no cost to the rural poor

Inclusive Program Design

Mandatory coverage of socially disadvantaged groups (SC/ST 50%; Minority 15%; Women
33%)

Shifting Emphasis from Training to Career Progression

Pioneers in providing incentives for job retention, career progression and foreign placements

Greater Support for Placed Candidates

Post-placement support, migration support and alumni network

Proactive Approach to Build Placement Partnerships

Guaranteed Placement for at least 75% trained candidates

Enhancing the Capacity of Implementation Partners

Nurturing new training service providers and developing their skills

Regional Focus

http://www.insightsonindia.com

INSIGHTS

Page 30

Insights Prleims Test Series 2015: Mock Test 14 Solutions


Greater emphasis on projects for poor rural youth in Jammu and Kashmir (HIMAYAT), the
North-East region and 27 Left-Wing Extremist (LWE) districts (ROSHINI)

Standards-led Delivery

All program activities are subject to Standard Operating Procedures that are not open to
interpretation by local inspectors. All inspections are supported by geo-tagged, time
stamped videos/photographs
http://pib.nic.in/newsite/efeatures.aspx

69. Solution: d)
Refer to the PIB e-features http://pib.nic.in/newsite/efeatures.aspx of 9 th Feb 2015

70. Solution: d)
An example of restricting a religious practice would be the Supreme Court banning
Jallikattu in TN.
Article 25 covers not only religious beliefs (doctrines) but also religious practices (rituals).
Moreover, these rights are available to all personscitizens as well as non-citizens.
However, these rights are subject to public order, morality, health and other provisions
relating to fundamental rights. Further, the State is permitted to:
(a) Regulate or restrict any economic, financial, political or other secular activity associated
with religious practice; and
(b) Provide for social welfare and reform or throw open Hindu religious institutions of a
public character to all classes and sections of Hindus.
Article 25 also contains two explanations: one, wearing and carrying of kirpans is to be
included in the profession of the Sikh religion; and two, the Hindus, in this context, include
Sikhs, Jains and Buddhists.

71. Solution: c)
The writ jurisdiction of the Supreme Court differs from that of a high court in three respects:

The Supreme Court can issue writs only for the enforcement of fundamental rights
whereas a high court can issue writs not only for the enforcement of Fundamental
Rights but also for any other purpose. The expression for any other purpose refers
to the enforcement of an ordinary legal right. Thus, the writ jurisdiction of the
Supreme Court, in this respect, is narrower than that of high court.

http://www.insightsonindia.com

INSIGHTS

Page 31

Insights Prleims Test Series 2015: Mock Test 14 Solutions

The Supreme Court can issue writs against a person or government throughout the
territory of India whereas a high court can issue writs against a person residing or
against a government or authority located within its territorial jurisdiction only or
outside its territorial jurisdiction only if the cause of action arises within its territorial
jurisdiction. Thus, the territorial jurisdiction of the Supreme Court for the purpose of
issuing writs is wider than that of a high court.
A remedy under Article 32 is in itself a Fundamental Right and hence, the Supreme
Court may not refuse to exercise its writ jurisdiction. On the other hand, a remedy
under Article 226 is discretionary and hence, a high court may refuse to exercise its
writ jurisdiction. Article 32 does not merely confer power on the Supreme Court as
Article 226 does on a high court to issue writs for the enforcement of fundamental
rights or other rights as part of its general jurisdiction. The Supreme Court is thus
constituted as a defender and guarantor of the fundamental rights.

72. Solution: b)
It is a Latin term which literally means to have the body of. It is an order issued by the
court to a person who has detained another person, to produce the body of the latter before
it. The court then examines the cause and legality of detention. It would set the detained
person free, if the detention is found to be illegal. Thus, this writ is a bulwark of individual
liberty against arbitrary detention.
The writ of habeas corpus can be issued against both public authorities as well as private
individuals. The writ, on the other hand, is not issued where the (a) detention is lawful, (b)
the proceeding is for contempt of a legislature or a court, (c) detention is by a competent
court, and (d) detention is outside the jurisdiction of the court.

73. Solution: d)
Article 27 lays down that no person shall be compelled to pay any taxes for the promotion or
maintenance of any particular religion or religious denomination. In other words, the State
should not spend the public money collected by way of tax for the promotion or
maintenance of any particular religion. This provision prohibits the State from favouring,
patronising and supporting one religion over the other. This means that the taxes can be
used for the promotion or maintenance of all religions.
This provision prohibits only levy of a tax and not a fee. This is because the purpose of a fee
is to control secular administration of religious institutions and not to promote or maintain
religion. Thus, a fee can be levied on pilgrims to provide them some special service or safety

http://www.insightsonindia.com

INSIGHTS

Page 32

Insights Prleims Test Series 2015: Mock Test 14 Solutions


measures. Similarly, a fee can be levied on religious endowments for meeting the regulation
expenditure.

74. Solution: d)
The Supreme Court has ruled that Article 32 is a basic feature of the Constitution. Hence, it
cannot be abridged or taken away even by way of an amendment to the Constitution. It
contains the following four provisions:
(a) The right to move the Supreme Court by appropriate proceedings for the enforcement of
the Fundamental Rights is guaranteed.
(b) The Supreme Court shall have power to issue directions or orders or writs for the
enforcement of any of the fundamental rights. The writs issued may include habeas corpus,
mandamus, prohibition, certiorari and quo-warranto.
(c) Parliament can empower any other court to issue directions, orders and writs of all kinds.
However, this can be done without prejudice to the above powers conferred on the Supreme
Court. Any other court here does not include high courts because Article 226 has already
conferred these powers on the high courts.
(d) The right to move the Supreme Court shall not be suspended except as otherwise
provided for by the Constitution. Thus the Constitution provides that the President can
suspend the right to move any court for the enforcement of the fundamental rights during a
national emergency (Article 359).
It is thus clear that the Supreme Court has been constituted as the defender and guarantor of
the fundamental rights of the citizens. It has been vested with the original and wide
powers for that purpose. Original, because an aggrieved citizen can directly go to the
Supreme Court, not necessarily by way of appeal. Wide, because its power is not restricted
to issuing of orders or directions but also writs of all kinds.

75. Solution: c)
Procedure established by law means that a law that is duly enacted by legislature or the
concerned body is valid if it has followed the correct procedure. Following this doctrine
means that, a person can be deprived of his life or personal liberty according to
the procedure established by law. So, if Parliament pass a law, then the life or personal
liberty of a person can be taken off according to the provisions and procedures of the that
law.

http://www.insightsonindia.com

INSIGHTS

Page 33

Insights Prleims Test Series 2015: Mock Test 14 Solutions


Due process of law doctrine not only checks if there is a law to deprive the life and personal
liberty of a person, but also see if the law made is fair, just and not arbitrary. If SC finds that
any law as not fair, it will declare it as null and void. This doctrine provides for more fair
treatment of individual rights.
Under due process, it is the legal requirement that the state must respect all of the legal
rights that are owed to a person and laws that states enact must confirm to the laws of the
land like fairness, fundamental rights, liberty etc. It also gives the judiciary to access the
fundamental fairness, justice, and liberty of any legislation.

http://www.insightsonindia.com

INSIGHTS

Page 34

INSIGHTS ON INDIA MOCK PRELIMINARY EXAM - 2015


INSIGHTS ON INDIA MOCK TEST - 15
GENERAL STUDIES

PAPER-I
Time Allowed: 2 Hours

Maximum Marks: 200

INSTRUCTIONS
1. IMMEDITELY AFTER THE COMMENCEMENT OF THE EXAMINATION, YOU SHOULD
CHECK THAT THIS TEST BOOKLET DOES NOT HAVE ANY UNPRINTED OR TORN OR MISSING
PAGES OR ITEMS, ETC. IF SO, GET IT REPLACED BY A COMPLETE TEST BOOKLET.
2. You have to enter your Roll Number on the Test
Booklet in the Box provided alongside. DO NOT
Write anything else on the Test Booklet.
4. This Test Booklet contains 100 items (questions). Each item is printed only in English. Each item
comprises four responses (answers). You will select the response which you want to mark on the
Answer Sheet. In case you feel that there is more than one correct response, mark the response which
you consider the best. In any case, choose ONLY ONE response for each item.
5. You have to mark all your responses ONLY on the separate Answer Sheet provided. See directions in
the Answer Sheet.
6. All items carry equal marks.
7. Before you proceed to mark in the Answer Sheet the response to various items in the Test Booklet, you
have to fill in some particulars in the Answer Sheet as per instructions sent to you with your
Admission Certificate.
8. After you have completed filling in all your responses on the Answer Sheet and the examination has
concluded, you should hand over to the Invigilator only the Answer Sheet. You are permitted to take
away with you the Test Booklet.
9. Sheets for rough work are appended in the Test Booklet at the end.
10. Penalty for wrong answers :
THERE WILL BE PENALTY FOR WRONG ANSWERS MARKED BY A CANDIDATE IN THE
OBJECTIVE TYPE QUESTION PAPERS.
(i) There are four alternatives for the answer to every question. For each question for which a
wrong answer has been given by the candidate, one-third of the marks assigned to that
question will be deducted as penalty.
(ii) If a candidate gives more than one answer, it will be treated as a wrong answer even if one of
the given answers happens to be correct and there will be same penalty as above to that
question.
(iii)

If a question is left blank, i.e., no answer is given by the candidate, there will be no penalty
for that question.
http://www.insightsonindia.com

INSIGHTS ON INDIA MOCK TEST SERIES FOR CIVIL SERVICES PRELIMINARY EXAM 2015
http://insightsonindia.com

INSIGHTS

Page 1

1. The Directive Principles of State Policy


denote the ideals that the State should keep in
mind while formulating policies and enacting
laws. These principles are applicable to which
of the following organs?
1. Legislative
2. Executive
3. Local authorities
4. Public Sector enterprises (PSUs)
Choose the correct answer using the codes given
below.
a)
b)
c)
d)

All of the above


1, 2 and 3 only
1 only
2 and 3 only

Choose the correct answer using the codes given


below.
a)
b)
c)
d)

All of the above


1, 2 and 3 only
1 and 2 only
3 and 4 only

4. Which of the following is NOT a Directive


Principle of State Policy?
1. Free legal aid to poor
2. Public assistance in cases of
unemployment
3. Secure the participation of workers in the
management of industries
Choose the correct answer using the codes below.

2. In the colonial legacy of government in India,


the Directive Principles in the present Indian
Constitution resemble which of the following
colonial enactments?
a) Codes enacted under Act of 1919
b) Instrument of Instructions enacted under
Independence Act, 1947
c) Instrument of Instructions enumerated in
the Government of India Act of 1935
d) Duties of State enumerated in the
Morley-Minto Act of 1909

3. As per the Supreme Court of India, if a law


may transgress certain fundamental rights,
but seeks to give effect to a Directive
Principle, the law may be saved from
unconstitutionality. Which of the
fundamental rights may be transgressed by
law this way?
1. Article 14
2. Article 19
3. Article 21
4. Article 30
http://insightsonindia.com

a)
b)
c)
d)

3 only
2 and 3 only
1 only
All are Directive Principles

5. In the constitution, the ideal of Economic


Equity is implied in
1. Fundamental Rights
2. Universal Adult franchise
3. Directive Principles of State Policy
4. Part IX and X of the Constitution
Choose the correct answer using the codes given
below.
a)
b)
c)
d)

All of the above


1, 2 and 3 only
1 and 2 only
3 only

INSIGHTS

Page 2

6. To secure for all citizens a uniform civil code


throughout the country is one of the
Directive Principles. It means that
a) All citizens will be under the same laws
irrespective of their caste
b) All citizens will be under the same laws
irrespective of their religion
c) All citizens will be under the same laws
irrespective of their political authority and
status
d) All citizens will be under the same laws
irrespective of them residing or not
residing in India

7. The 97th Amendment Act of 2011 added a


new Directive Principle relating to cooperative societies in the Constitution. The
amendment is concerned with which of the
following aspects of cooperative societies?
1. Autonomous functioning
2. Democratic control
3. Professional management
Choose the correct answer using the codes below.
a)
b)
c)
d)

1 and 2 only
2 and 3 only
1 and 3 only
All of the above

8. As per the Constitution, which of the


following is/are fundamental in the
governance of the country?
1. Fundamental Rights
2. Fundamental Duties
3. Directive Principles of State Policy
4. Preamble
Choose the correct answer using the codes given
below.

c) 1 and 2 only
d) 3 only

9. Directive Principles of State Policy help in


1. Amplifying the Preamble
2. Guiding courts in Judicial review of
legislative and executive acts
3. Supplementing fundamental rights of
citizens
Choose the correct answer using the codes below.
a)
b)
c)
d)

1 and 2 only
2 and 3 only
1 and 3 only
All of the above

10. Consider the following statements.


1. Fundamental Rights are subordinate to
Directive Principles of State Policy
(DPSP).
2. DPSP is not enforced automatically by
merely being a part of the Constitution; it
requires a legislation to enforce it.
Which of the above is/are correct?
a)
b)
c)
d)

1 only
2 only
Both
None

11. For a citizen of India, the duty to pay taxes is


a:
a) Fundamental Duty
b) Legal obligation
c) Constitutional obligation
d) Moral obligation

a) All of the above


b) 1, 2 and 3 only
http://insightsonindia.com

INSIGHTS

Page 3

12. Which of the following is a fundamental duty


mentioned in Article 51-A of the Constitution?
1. To develop a spirit of inquiry
2. To respect the National Flag and National
Anthem
3. To safeguard Public property
4. To do charitable activities which promote
social and economic equity
Choose the correct answer using the codes given
below.
a)
b)
c)
d)

All of the above


1, 2 and 3 only
1 and 2 only
3 and 4 only

13. Consider the following statements about


Fundamental duties.
1. They are enforceable by law.
2. They can be used by the courts for
determining the constitutionality of any
law that transgresses certain
fundamental rights.
Which of the above is/are true?
a)
b)
c)
d)

1 only
2 only
Both
None

14. As per Article 368 of the Constitution, the


amendment of the Indian constitution can be
initiated in
1. Lok Sabha
2. Rajya Sabha
3. The office of The President
4. State Legislatures
5. Council of Ministers
Choose the correct answer using the codes below.
http://insightsonindia.com

a)
b)
c)
d)

1, 3 and 5 only
1 and 2 only
2, 3 and 4 only
1, 2 and 5 only

15. Consider the following statements about


Constitutional amendment in India.
1. A private member of the Parliament
cannot introduce a constitutional
amendment bill
2. The prior permission of the President is
required for the introduction of every
constitutional amendment bill
3. Special days are reserved for introducing
constitutional amendment bills
Choose the correct answer using the codes below
a)
b)
c)
d)

1 and 2 only
2 only
1 and 3 only
None of the above

16. A constitutional amendment bill must be


passed in each house by a combination of
which of these conditions
1. Two-thirds of total membership of each
house
2. Majority of those present and voting in
each house
3. Two-thirds of those present and voting in
each house
4. Majority of total membership of each
house
Choose the correct answer using the codes below.
a)
b)
c)
d)
INSIGHTS

1 and 2
3 and 4
1 only
Either option (b) OR (c)
Page 4

17. When a constitutional amendment bill is


produced before the President, what are the
options available to him?
1. Withhold the bill
2. Return the bill for reconsideration of the
Parliament
3. End the bill
Choose the correct answer using the codes below.
a)
b)
c)
d)

1 and 2 only
2 only
1 and 3 only
None of the above

18. A number of provisions in the Constitution


can be amended by a simple majority of the
two Houses of Parliament outside the scope of
Article 368. These provisions include, inter
alia
1. Citizenship clauses
2. Abolition or creation of legislative
councils in states
3. Election of the President and its manner
Choose the correct answer using the codes below.
a)
b)
c)
d)

1 and 2 only
2 only
1 and 3 only
None of the above

19. As per a ruling of the Supreme Court, the


basic structure of the constitution can not be
amended by the Parliament. Which of the
following come under the basic structure?
1. Power of Supreme Court to issue writs in
case of violation of Fundamental Rights
2. Effective access to justice
3. Parliamentary system
Choose the correct answer using the codes below.
http://insightsonindia.com

a)
b)
c)
d)

1 and 2 only
2 and 3 only
1 and 3 only
All of the above

20. Collective Responsibility of the council of


ministers to the Parliament is the bedrock
principle of parliamentary government. It
implies that
a) The Parliament can remove the council of
ministers from office by passing a vote of
no confidence
b) The council of Ministers is appointed and
dismissed by the Parliament
c) All important decisions of the council of
Ministers have to be approved by the
Parliament
d) None of the above

21. If there is frequent disagreement on major


decisions in the Council of Ministers due to
just a few ministers, what option(s) does the
Prime Minister has/have?
1. Dismiss the Ministers who are causing
disagreement
2. Enforce his decision irrespective of the
view of the Council of Ministers
3. Convene a Parliamentary session to take
final view on the decision
Choose the correct answer using the codes below.
a)
b)
c)
d)

1 and 2 only
2 and 3 only
1 and 3 only
All of the above

22. In which of the following cases, the Lok Sabha


may be dissolved?
1. Prime Ministers recommending
dissolution of Lok Saha to the President
INSIGHTS

Page 5

2. No-confidence motion passed against the


incumbent government
3. Resignation of majority of cabinet
ministers
Choose the correct answer using the codes below.
a)
b)
c)
d)

1 and 2 only
2 and 3 only
1 and 3 only
All of the above

23. With reference to Indias physical setting,


consider the following statements:
1. India lies entirely in the northern
hemisphere
2. Distance between Indias two extreme
longitudes is larger than distance
between its two extreme latitudes
Which of the above statements is/are correct?
a)
b)
c)
d)
24.

1 Only
None
2 Only
Both

Which of the following states does NOT


share their borders with Mynamar?
1. Mizoram
2. Nagaland
3. Assam
4. Arunachal Pradesh
5. Tripura
6. Manipur

Choose the correct answer using the codes below


a)
b)
c)
d)

1,3 and 4
2, 3 and 6
3 and 5
1 and 6

25. With reference to Indias perennial rivers,


consider the following statements
1. The rivers Ramganga and Mahananda
are important tributaries of Yamuna
2. The rivers Manas and Dhansiri are
principal tributaries of river
Brahmaputra
Which of the above statements is/are correct?
a)
b)
c)
d)

1 Only
2 Only
Both
None

26. The entire India is divided into twenty river


basins. With reference to these river basins,
consider the following statements
1. Rivers with drainage area of 2000 sq.
km to 20000 sq. km are considered as
Major river basins
2. Narmada and Tapti river basins are
considered as Medium river basins
Which of the above statements is/are correct?
a)
b)
c)
d)

1 Only
2 Only
Both
None

27. With reference to the Himalayan flora,


consider the following statemnets
1. The oaks, laurels, maples,
rhododendrons and birch are found in
the temperate regions of Eastern
Himalayas
2. Silver fir, juniper and silver birch are
found at altitudes higher than 4500
metres in the Western Himalyan
region and are known as alpine trees
Which of the above statements is/are correct?
a) 1 Only

http://insightsonindia.com

INSIGHTS

Page 6

b) 2 Only
c) Both
d) None
28. With reference to Census 2011 and its
findings, consider the following statements
1. Census 2011 was the fourteenth census
since 1872
2. Overall sex ratio decreased by seven
points since 2001 census
3. The population of India since 1901 has
grown steadily with each passing
census without exception
4. According to 2011 census, West Bengal
is the thickly populated state in India
Which of the above statements is/are
INCORRECT?
a)
b)
c)
d)

1 and 2 Only
2 and 3 Only
1,2 and 4 Only
All

29. Consider the following statements


1. The design of the national flag was
adopted by the Constituent Assembly
of India after India became republic
2. The display of the National Flag is
solely governed by the non-statutory
instructions issued by the government
time to time
Which of the above statements is/are correct?
a)
b)
c)
d)

1 Only
2 Only
Both
None

30. Consider the following statements

http://insightsonindia.com

1. The words Satyameva Jayate inscribed on


national emblem are taken from Mundaka
Upanishad
2. Indias state emblem is an adaptation of
sculpture from Ashokas Sanchi pillar
Which of the above statements is/are correct?
a)
b)
c)
d)

1 Only
2 Only
Both
None

31. Consider the following statements:


1. The national song Vande Mataram
was composed by Bankimchandra
Chatterji in Bengali language
2. Vande Mataram was first sung on 27 th
December 1911, at the Kolkata session
of the Indian National Congress
Which of the above statements is/are correct?
a)
b)
c)
d)

1 Only
None
2 Only
Both

32. Consider the following statements


1. The national calendar is based on Saka
Era with Chaitra as its first month
2. According to Indias national calendar, 1
magha starts on 21st January of Gregorian
calendar
Which of the above statements is/are correct?
a)
b)
c)
d)

INSIGHTS

1 Only
2 Only
Both
None

Page 7

33. Consider the following statements


1. The President of India is the
constitutional head of executive of the
Union
2. The Council of Ministers are
collectively responsible to both the
houses of the Parliament
Which of the above statements is/are correct?
a)
b)
c)
d)

1 Only
2 Only
Both
None

34. With reference to some of the provisions of


the Constitution of India related to the
Fundamental rights and Directive principles
of state policy(DPSPs) , consider the following
statements
1. Prohibiting all forms of forced labour,
child labour and human trafficking is
enshrined in Directive Principles of
State Policy
2. The equal pay for equal work for men
and women is a fundamental right in
India
3. According to the Constitution of India,
the DPSPs are fundamental to the
governance of the country
Which of the above statements is/are
INCORRECT?
a)
b)
c)
d)

1 and 2 Only
2 Only
3 Only
2 and 3 Only

35. During an emergency, our political structure


undergoes a transformation from being
Federal to
a) Military
http://insightsonindia.com

b) Quasi-federal
c) Unitary
d) Being only a Single Government

36. Consider the following statements.


1. National emergency can be declared even
if security of India is not in threat, but
there is a case of imminent danger.
2. The operation of National Emergency
always applies to the whole of Indian
Territory.
Which of the above is/are correct?
a)
b)
c)
d)

1 only
2 only
Both 1 and 2
None

37. The President of India can declare a National


emergency only after the
1. Approval of the Cabinet
2. Approval by the Parliament
3. Approval by the State legislatures
involved
Choose the correct answer using the codes below.
a)
b)
c)
d)

1 and 2 only
2 and 3 only
1 and 3 only
1 only

38. The concept of Indicative Planning means


that
a) The private sector is involved directly in
the planning process for the nation
b) The state fixes long-term growth targets
for various sectors of the economy and
facilitates the private sector to achieve it

INSIGHTS

Page 8

c) The planning is decentralised with the


state and local governments being made
an integral part of the planning process
d) Allowing the market forces to work with
little regulation and no direction by the
state so that it results in optimum resource
utilization

39. Constitutional provisions which have a


bearing on planning in the country can be
found in
1. Seventh Schedule
2. Part IX
3. Sixth Schedule

41. Which of the following are monitorable


targets set to be achieved at the end of the 12 th
Five Year Plan?
1. Generating 50 million new work
opportunities in the non-farm sector
2. Completing Eastern and Western
Dedicated Freight Corridors
3. 50 per cent gram panchayats achieve
Nirmal Gram Status
Choose the correct answer using the codes below.
a)
b)
c)
d)

1 and 2 only
2 and 3 only
1 and 3 only
All of the above

Choose the correct answer using the codes below.


a)
b)
c)
d)

1 and 2 only
2 and 3 only
1 and 3 only
All of the above

40. Which of the following is/are NOT the


functions of the recently established NITI
Ayog?
1. Giving technical advice to the state
governments
2. Supervise the Implementation of various
social sector schemes
3. Allocating funds for the economic
projects approved by the Government
Choose the correct answer using the codes below.
a)
b)
c)
d)

1 and 2 only
2 and 3 only
1 only
None mentioned above are the functions
of NITI Aayog.

http://insightsonindia.com

42. Which of the following is/are NOT a


Fundamental Duty as enshrined in the
Constitution of India?
1. To value and preserve the rich
heritage of our composite culture
2. To safeguard public property and to
abjure violence
3. Who is a parent or guardian to
provide opportunities for education to
his child or ward, as the case may be,
between the age of six and fourteen
years
4. To uphold and protect the
sovereignty, unity and integrity of
India
5. To respect women and uphold Indias
rich culture
Choose the correct answer using the codes below
a)
b)
c)
d)

INSIGHTS

3 and 5 Only
3 Only
5 Only
1 and 5

Page 9

43. The President of India is elected by members


of an electoral college consisting of
1. Elected members of Lok Sabha

Which of the above statements is/are


INCORRECT?
a)
b)
c)
d)

2. Elected members of Legislative


Assemblies of states
3. Elected members of Rajya Sabha

1 Only
2 Only
3 Only
None

4. Elected members of Legislative


Councils of states
Choose the correct answer using the codes below
a) 1 and 2 Only
b) 1,2 and 3 Only
c) 1,2 and 4 Only
d) All the four

44. With reference to the office of Vice President


of India, consider the following statements:
1. When he is acting as President, he still

46. With reference to various Parliamentary


committees in India, consider the following
statements
1. Standing committee is permanent in
nature
2. Standing committees are formed only
by Lok Sabha
3. Ad hoc committees are temporary in
nature
4. The Committee on Petitions is an
example of ad hoc committee
Which of the above statements is/are correct?

performs the function of the Chairman

a)
b)
c)
d)

of the Rajya Sabha


2. The Vice-President is elected
indirectly by members of an electoral

1 and 2 Only
1,2 and 3 Only
2 and 3 Only
1 and 3 Only

college consisting of the members of


both Houses of Parliament
Which of the above statements is/are correct?
a) 1 Only
b) 2 Only
c) Both
d) None

47. Which of the following best describes the


function of committee on public
undertakings?
a) to examine the reports, if any, of the
Comptroller and Auditor General on the
Public Undertakings
b) to suggest alternative policies in order to
bring about efficiency and economy in

45. With reference to the Rajya Sabha, consider


the following statements
1. It is not subject to dissolution
2. One third of its members retire every
second year
3. All the elected members of the Rajya
Sabha are indirectly elected
http://insightsonindia.com

administration
c) to ascertain whether the money granted
by Parliament has been spent by
Government "within the scope of the
Demand"
d) None of the above

INSIGHTS

Page 10

48. Which of the following is NOT a function of


the Ministry of Parliamentary Affairs?
a) Planning and Coordination of legislative
and other official business in both Houses
b) Organisation of Youth Parliament
Competitions in Schools/Colleges
throughout the country
c) Appointment of Members of Parliament
on Committees and other bodies set up
by Government
d) None of the above

49. Which of the following features given in the


Constitution show that the division of powers
in the Constitution is in favour of the Centre?
1. Centre has overriding authority over the
Concurrent List
2. States in India have no right to territorial
integrity.
3. Veto of Parliament over State bills
4. Financial emergency
Choose the correct answer using the codes given
below.
a)
b)
c)
d)

All of the above


1, 2 and 3 only
1, 2 and 4 only
3 and 4 only

50. The legislative matters on which uniformity


of legislation throughout the country is
desirable but not essential are enumerated in
the (Constitution)
a) Residuary List
b) Concurrent List
c) Fifth and Sixth Schedule
d) Directive Principles of State Policy

http://insightsonindia.com

51. In case of a conflict between the Central law


and the state law on a subject enumerated in
the Concurrent List, which of the following is
possible?
1. Central law prevails over the state law.
2. State law prevails if it has received
Presidential assent
3. State law prevails of Governors of two
or more states have approved the
same legislation
Choose the correct answer using the codes below.
a)
b)
c)
d)

1 and 2 only
2 and 3 only
1 and 3 only
All of the above

52. The Constitution empowers the Parliament to


make laws on any matter enumerated in the
State List under which of the following
extraordinary circumstances?
1. When states make a request
2. To implement International agreements
3. When Rajya Sabha passes a resolution to
that effect
4. Presidents order in the Extraordinary
gazette of India
Choose the correct answer using the codes given
below.
a)
b)
c)
d)

INSIGHTS

All of the above


1, 2 and 3 only
1, 2 and 4 only
3 and 4 only

Page 11

53. In which of the following cases can the


Governor reserve a State bill for the approval
of the President?
1. During a financial emergency
2. If the bill restricts inter-state free trade
and commerce
3. If the Central Cabinet specifies in a
written order so.
Choose the correct answer using the codes below.
a)
b)
c)
d)

1 and 2 only
2 and 3 only
1 and 3 only
All of the above

54. Which of the following duties have been


constitutionally prescribed for the Centre to
exercise control over the state administration?
1. To protect every state against external
aggression and internal disturbance
2. To ensure that corruption and
maladministration do not affect state
governments
Which of the above is/are true?
a)
b)
c)
d)

1 only
2 only
Both 1 and 2
None

55. There are extra-constitutional devices to


promote cooperation and coordination
between the Centre and the states. These
include
1. Inter-state Councils
2. NITI Ayog
3. North-Eastern Council
4. Governors conference
Choose the correct answer using the codes given
below.
http://insightsonindia.com

a)
b)
c)
d)

1, 2 and 3 only
2, 3 and 4 only
1, 2 and 4 only
3 and 4 only

56. The Constitution provides for a division of


taxation powers between Centre and States.
Among the several taxes, service tax is
a) Levied by the Centre but Collected and
Appropriated by the States
b) Levied by the Centre but Collected and
Appropriated by the Centre and the States
c) Levied, collected and retained by the
Centre
d) Levied, collected and retained by the
States

57. The receipts from which of the following form


the major sources of non-tax revenues of the
Centre?
1. Broadcasting
2. Banking
3. Irrigation
Choose the correct answer using the codes below.
a)
b)
c)
d)

1 and 2 only
2 and 3 only
1 and 3 only
All of the above

58. The Constitution provides for specific grants


for promoting the welfare of the scheduled
tribes in a state or for raising the level of
administration of the scheduled areas. It
comes under
a) Statutory grants
b) Discretionary Grants
c) Backward area development grant
d) Grants-in-aid

INSIGHTS

Page 12

59. A bill which imposes or varies any tax or duty


in which states are interested can be
introduced
a) Only on the recommendation of the
President
b) Only with the consent of two or more
states
c) Only after Rajya Sabha passes a resolution
to that effect
d) Only on the recommendation of the
Finance Commission

62. The Union Government has recently imposed


an anti-dumping duty on imports of graphite
electrodes used in steel plants for five years.
Consider the following statements about antidumping duty
1. It is imposed by Ministry of Commerce.
2. It is imposed on products which are sold
in domestic markets at much higher
prices than international prices.
3. It is used to counter trade imbalance.
Choose the correct answer using the codes below.

60. Article 262 of the Constitution provides for


the adjudication of inter-state water disputes
by a separate tribunal. The need for an extra
judicial machinery to settle inter-state water
disputes is because
a) The courts being overburdened with
litigations and adjudicating slowly are
incapable of adjudicating sensitive water
disputes
b) water resources are not private property
and so rule of law is applied by ordinary
courts is not appropriate to deal with its
distribution
c) Division and distribution of any natural
resources is out of judicial scrutiny in
India
d) The courts cannot employ technical
committees to ascertain the distribution of
water resources

61. The festival Vanaj was organised for the first


time by the Union Government. It is related
with
a) Countering Wildlife extinction
b) Increasing green cover of India
c) Bridge the gaps of social divide and
promote tribal culture in India
d) Generating awareness about conserving
the ecosystem services humanity received
from forests
http://insightsonindia.com

a)
b)
c)
d)

1 and 2 only
1 and 3 only
2 and 3 only
None of the above

63. India and Sri Lanka have a Memorandum of


Understanding in which of the following?
1. On establishment of Nalanda University
2. On cooperation in peaceful uses of
Nuclear energy
3. On cooperation in Cultural
documentation
Choose the correct answer using the codes
below.
a)
b)
c)
d)

1 and 2 only
1 and 3 only
2 and 3 only
All of the above

64. The Government of India announces


Minimum Support Prices (MSP) from time to
time. Which of the following factors are taken
into account while calculating MSP?
1. Inter-crop parity
2. International Prices
3. Subsidy levels
4. Inflation levels
INSIGHTS

Page 13

Choose the correct answer using the codes below.


a)
b)
c)
d)

1 and 2 only
1, 3 and 4 only
2 and 3 only
All of the above

65. As per government norms, the Buffer stock of


grains with FCI can be used for which of the
following purposes?
1. Countering food inflation
2. Meeting grain shortages across
geographical regions
3. Disaster relief
Choose the correct answer using the codes below.
a)
b)
c)
d)

1 and 2 only
1 and 3 only
2 and 3 only
All of the above

66. Import of Edible oils in India has increased in


the last few decades. What factors are
responsible for it?
1. Competitive International Prices of Edible
oils
2. Very low import duty on Edible oil
3. India does not produce quality edible oil.
Choose the correct answer using the codes below.
a)
b)
c)
d)

1 and 2 only
1 and 3 only
2 and 3 only
All of the above

2. Grading and standardization of


agricultural produce
3. Direct sale of agricultural produce to
markets
Choose the correct answer using the codes below.
a)
b)
c)
d)

1 and 2 only
1 and 3 only
2 and 3 only
3 only

68. National Horticulture Mission (NHM) does


NOT include which of the following?
1. Aromatic plants
2. Medicinal plants
3. Spices
4. Plantation crops
Choose the correct answer using the codes below.
a)
b)
c)
d)

1 and 2 only
1, 3 and 4 only
2 and 3 only
All of these are included.

69. Consider the following statements about the


government interventions in Agricultural
insurance.
1. Private sector insurers have not been
allowed in agricultural insurance till date.
2. Not only post-harvest losses, but preharvest risks have also been covered.
3. Insurance has been restricted to incidents
of adverse weather situations in farms.
Choose the correct answer using the codes below.

67. Presently the Agricultural Produce Marketing


Committees (APMC) do NOT offer which of
the following facilities to farmers?
1. Regulation of remunerative prices to
farmers
http://insightsonindia.com

a)
b)
c)
d)

INSIGHTS

1 and 2 only
1 and 3 only
2 and 3 only
2 only

Page 14

70. Which of the following would come under


macro-stabilization measures?
1. Fiscal stimulus
2. Cutting Inflation levels
3. Reducing Current Account Deficit
Choose the correct answer using the codes below.
a)
b)
c)
d)

1 and 2 only
2 and 3 only
1 and 3 only
All of the above

71. In the context of economic reforms,


Privatization may mean
1. Facilitating expansion of the Private
sector by the state
2. Disinvestment in State owned enterprises
3. Increasing economic integration with
other State economies
Choose the correct answer using the codes below.
a)
b)
c)
d)

1 and 2 only
2 and 3 only
1 and 3 only
All of the above

72. Factor Market Reforms are considered to be


an important component of Second
Generation Economic Reforms in India. Factor
market reforms may include
1. Cutting down level of subsidies
2. Dismantling the Administered price
Mechanism
3. Rationalizing the taxation system

73. Which of the following are instruments of


Deflation in the economy?
1. Increase in tax rates
2. Increase in interest rates
3. Restricting from printing money
Choose the correct answer using the codes below.
a)
b)
c)
d)

1 and 2 only
2 and 3 only
1 and 3 only
All of the above

74. Cutting the Repo rate, cutting the Cash


Reserve Ratio (CRR) and tweaking other
monetary policy tools will NOT help fight
price rise in which of the following goods?
1. Salt
2. Wheat
3. Oil
4. Automobiles
Choose the correct answer using the codes below.
a)
b)
c)
d)

1 and 2 only
3 and 4 only
2 and 4 only
4 only

75. Core inflation shows price rise in all goods


and services excluding
a) Energy and food articles
b) Manufacturing goods
c) Unorganized sector services
d) Energy commodities

Choose the correct answer using the codes below.


a)
b)
c)
d)

1 and 2 only
2 and 3 only
1 and 3 only
All of the above

http://insightsonindia.com

INSIGHTS

Page 15

76. Inflationary spiral is a situation in an


economy which results out of a process of
interaction between
a) Wages and prices
b) Interest rates and prices
c) Taxes and prices
d) Employment levels and prices

a) The economy has seen price rise over the


last year
b) The economy has seen lowering of prices
over the last year
c) The purchasing power parity of masses
has increased over last year
d) The purchasing power parity of masses
has reduced over last year

77. Stagflation is a situation in the economy when


high X and low Y occur simultaneously. X
and Y respectively are
1. Inflation
2. Economic growth
3. Unemployment
4. Development

80. Consider the following effects of inflation in


the economy.
1. Lenders suffer and borrowers benefit.
2. With the same wages purchasing power
of people get reduced
3. Savings rate may increase

Choose the correct combination of X and Y.

Choose the correct answer using the codes below.

a)
b)
c)
d)

1 and 2
3 and 2
2 and 4
3 and 1

78. Consider the following statements about


Skewflation.
1. It denotes a highly irregular trend of
inflation in the economy.
2. Food articles in India have shown
Skewflation in India in the last 4-5 years.
3. Skewflation cannot be countered using
conventional monetary policy tools.
Choose the correct answer using the codes below.
a)
b)
c)
d)

1 and 2 only
3 only
2 and 3 only
2 only

a)
b)
c)
d)

1 and 2 only
1 and 3 only
2 and 3 only
All of the above

81. Wholesale Price Index (WPI) does NOT


calculate for price rise in which of the
following categories?
1. Services
2. Energy commodities
3. Intermediary goods
Choose the correct answer using the codes below.
a)
b)
c)
d)

1 and 2 only
1 and 3 only
2 and 3 only
All of the above

79. If the GDP deflator in an economy is greater


than 1, it implies that

http://insightsonindia.com

INSIGHTS

Page 16

82. With few exceptional years, India has been


facing the typical problem of structural
inflation. The factors responsible for this can
be
1. Shortfall in supply of goods
2. Infrastructure deficit
3. High fiscal deficit
Choose the correct answer using the codes below.
a)
b)
c)
d)

1 and 2 only
1 and 3 only
2 and 3 only
All of the above

83. The economy might face structural problems


like shortage of investible capital, lower
savings, falling standard of living, creation of
a sellers market in which of the following
phases of business cycle?
a) Recession and Boom
b) Recession only
c) Recovery and Recession
d) Depression and Recovery

84. Which of the following sectors of the Indian


economy accounts for the largest share of the
unorganised sector?
a) Agriculture
b) Construction sector
c) Services sector
d) Mining sector

85. Tenancy reforms is an important component


of Land reforms in India. It includes
1. Regulation of rent
2. Abolition of Intermediaries
3. Ownership rights to tenants
4. Security of tenure
Choose the correct answer using the codes below.
http://insightsonindia.com

a)
b)
c)
d)

3 and 4 only
1 and 3 only
1, 3 and 4 only
1, 2 and 4 only

86. Swine flu is NOT transmitted among humans


by
1. Touch
2. Vectors
3. Food
Choose the correct answer using the codes below.
a)
b)
c)
d)

1 and 2 only
1 and 3 only
2 and 3 only
All of the above

87. Department of Border Management of


government of India is part of which of the
following ministry?
a) Ministry of Defence
b) Ministry of Home Affairs
c) Ministry of Planning
d) Ministry of Earth Sciences

88. With reference to the Comptroller and


Auditor General of India, consider the
following statements
1. The reports of the Comptroller and
Auditor-General of India relating to the
accounts of the States shall be submitted
to the president, who shall cause them to
be laid before concerned Legislative
Assemblies
2. He shall only be removed from office in
like manner and on like grounds as a
Judge of the Supreme Court
Which of the above statements is/are correct?
a) 1 Only
INSIGHTS

Page 17

b) 2 Only
c) Both
d) None

89. The bhoochetana project in Karnataka is


related to
a) Enhancing availability of credit to
farmers
b) Enhancing productivity of rainfed
areas
c) Enhancing marketing facilities for
farmers
d) None of the above

90. Consider the following statements


1. The formation of tribunals in India
owes their origin to Constitutional
provisions
2. The administrative tribunals exercise
original jurisdiction in respect of
service matters of employees covered
by them

Council with duties identical with


those of the Inter-State Council
Which of the above statements is/are correct?
a)
b)
c)
d)

92. With reference to electoral rolls, consider the


following statements
1. The electoral roll is a list of all people
in the constituency who are registered
to vote in Indian Elections
2. Only those people with their names on
the electoral roll are allowed to vote
3. The electoral roll is revised once every
five years
Which of the above statements is/are correct?
a)
b)
c)
d)

Which of the above statements is/are correct?


a)
b)
c)
d)

1 Only
2 Only
Both
None

91. With reference to the Inter State Councils,


consider the following statements
1. A provision in the Constitution of
India governing formation of Inter
State Councils was added later
through an amendment
2. The genesis of the article can be traced
directly to Section 135 of the Govt. of
India Act, 1935 provided for
establishment of Inter-Provincial

http://insightsonindia.com

1 Only
2 Only
Both
None

1 and 3 Only
1 and 2 Only
2 and 3 Only
All

93. The Parliament exercises control over the


Government using which of the following
devices?
1. Question Hour
2. Adjournment motion
3. Discussions
4. Parliamentary committees
Choose the correct answer using the codes given
below.
a)
b)
c)
d)

INSIGHTS

All of the above


1, 2 and 3 only
1 and 2 only
3 and 4 only

Page 18

94. Consider the following statements about the


Parliamentary system in India.
1. Parliament is the supreme authority in
India.
2. In India, the Prime Minister can be from
either house of the Parliament.
3. A person who is not a Member of
Parliament is also eligible to become a
Minister.
Choose the correct answer using the codes below.
a)
b)
c)
d)

1 and 2 only
2 and 3 only
1 and 3 only
All of the above

c) Protect the observatory from any damages


from earthquake and other disasters
d) Save the observatory from the influence of
primordial gravitational waves

97. Consider the following statements about the


Soil Health Card scheme for farmers.
1. The card will carry crop wise
recommendations of nutrients/fertilizers
required for farms.
2. The card can be assessed without the aid
of technical or laboratory equipment.
3. Cards will be distributed by the State
governments.
Choose the correct answer using the codes below.

95. In a federal system of Government, there


must be
1. A Written Constitution
2. Independent Judiciary
3. Bicameral legislature
4. Local governments
Choose the correct answer using the codes given
below.
a)
b)
c)
d)

All of the above


1, 2 and 3 only
1 and 2 only
3 and 4 only

96. The objective of Indian Neutrino Observatory


(INO) is to conduct basic research on the
elementary particle called neutrino. The
observatory will be located underground so
as to
a) Safeguard the observatory from the
electro-magenetic waves travelling on
Earths surface
b) Provide adequate shielding to the
neutrino detector from cosmic
background radiation.
http://insightsonindia.com

a)
b)
c)
d)

1 and 2 only
1 and 3 only
2 and 3 only
All of the above

98. As per World Banks recently released


report Building Resilience for Sustainable
Development of the Sundarbans, which of
the following factors have been blamed for
the decline of Sundarbans ecosystem?
1. Mangrove destruction
2. Coral reefs destruction
3. Destructive impact of cyclones
4. Unsustainable fishing
Choose the correct answer using the codes below.
a)
b)
c)
d)

INSIGHTS

1 and 2 only
1, 3 and 4 only
2 and 3 only
2, 3 and 4 only

Page 19

99. The Union Government has recently launched


a Grievances Monitoring portal
named MADAD (Help) to register
grievances from
a) General public
b) Litigants stuck in court cases
c) Indians living abroad
d) Indian students

100. Which of the following Institutes/bodies


have a bearing on skill development in India?

http://insightsonindia.com

1. Khadi and Village Industries Commission


(KVIC)
2. Rural Self Employment Training Institute
(RSETI)
3. National Skill Commission
Choose the correct answer using the codes below.
a)
b)
c)
d)

INSIGHTS

1 and 2 only
1 and 3 only
2 and 3 only
2 only

Page 20

Insights Mock Tests 2015: Solutions Test - 15

1. Solution: a)
These are the constitutional instructions or recommendations to the State in legislative,
executive and administrative matters.
According to Article 36, the term State in Part IV has the same meaning as in Part III
dealing with Fundamental Rights. Therefore, it includes the legislative and executive organs
of the central and state governments, all local authorities and all other public authorities in
the country.

2. Solution: c)
The Directive Principles resemble the Instrument of Instructions enumerated in the
Government of India Act of 1935. In the words of Dr B R Ambedkar, the Directive
Principles are like the instrument of instructions, which were issued to the GovernorGeneral and to the Governors of the colonies of India by the British Government under the
Government of India Act of 1935. What is called Directive Principles is merely another name
for the instrument of instructions. The only difference is that they are instructions to the
legislature and the executive.

3. Solution: c)
The Directive Principles are non-justiciable in nature, that is, they are not legally enforceable
by the courts for their violation. Though non-justiciable in nature, they help the courts in
examining and determining the constitutional validity of a law. The Supreme Court has
ruled many a times that in determining the constitutionality of any law, if a court finds that
the law in question seeks to give effect to a Directive Principle, it may consider such law to
be reasonable in relation to Article 14 (equality before law) or Article 19 (six freedoms) and
thus save such law from unconstitutionality.

4. Solution: d)
Some of the Socialistic Principles are:

To promote equal justice and to provide free legal aid to the poor (Article 39 A).
To secure the right to work, to education and to public assistance in cases of
unemployment, old age, sickness and disablement (Article 41).
To make provision for just and humane conditions for work and maternity relief
(Article 42).
To secure a living wage, a decent standard of life and social and cultural
opportunities for all workers (Article 43).

http://www.insightsonindia.com

INSIGHTS

Page 1

Insights Mock Tests 2015: Solutions Test - 15

To take steps to secure the participation of workers in the management of industries


(Article 43 A).
To raise the level of nutrition and the standard of living of people and to improve
public health (Article 47).

5. Solution: d)
Fundamental rights ensure opportunities for economic non-discrimination. For e.g. all
should be eligible for public employment etc.It does not ensure economic equity/
Universal adult franchise ensures political equality and not economic equity.
DPSP alone ensures that all should have the right to work; at living wages; income
inequality should be minimized; women should get equal wages for equal work as men do
etc.
The last option deals with the subject of local government and not with economic equity.

6. Solution: b)
http://en.wikipedia.org/wiki/Uniform_civil_code_of_India
http://www.thehindu.com/opinion/open-page/uniform-civil-code-will-it-work-inindia/article6625409.ece

7. Solution: d)
The Constitution (Ninety Seventh Amendment) Act 2011 relating to the co-operatives is
aimed to encourage economic activities of cooperatives which in turn help progress of rural
India. It is expected to not only ensure autonomous and democratic functioning of
cooperatives, but also the accountability of the management to the members and other
stakeholders.

It makes Right to form cooperatives as a fundamental right.

Reservation of one seat for SC/ST and two seats for women on the board of every cooperative society.

Cooperatives could set up agency which would oversee election.

Uniformity in the tenure of Cooperative Board of Directors.

http://www.insightsonindia.com

INSIGHTS

Page 2

Insights Mock Tests 2015: Solutions Test - 15

Provisions for incorporation, regulation and winding up of co-operative societies


based on the principles of democratic process and specifying the maximum number
of directors as twenty-one.

Providing for a fixed term of five years from the date of election in respect of the
elected members of the board and its office bearers;

Providing for a maximum time limit of six months during which a board of directors
of co-operative society could be kept under suspension;

Providing for independent professional audit;

Providing for right of information to the members of the co-operative societies;

Empowering the State Governments to obtain periodic reports of activities and


accounts of co-operative societies; which have individuals as members from such
categories;

Providing for offences relating to co-operative societies and penalties in respect of


such offences.

8. Solution: d)
Though the Directive Principles are non-justiciable, the Constitution (Article 37) make it
clear that these principles are fundamental in the governance of the country and it shall be
the duty of the state to apply these principles in making laws. Thus, they impose a moral
obligation on the state authorities for their application, but the real force behind them is
political, that is, public opinion. As observed by Alladi Krishna Swamy Ayyar, no ministry
responsible to the people can afford lightheartedly to ignore the provisions in Part IV of the
Constitution. Similarly, Dr B R Ambedkar said in the Constituent Assembly that a
government which rests on popular vote can hardly ignore the Directive Principles while
shaping its policy. If any government ignores them, it will certainly have to answer for that
before the electorate at the election time.

9. Solution: d)
The Directive Principles, although confer no legal rights and creates no legal remedies, are
significant and useful in the following ways:

They are like an Instrument of Instructions or general recommendations addressed


to all authorities in the Indian Union. They remind them of the basic principles of the
new social and economic order, which the Constitution aims at building.
They have served as useful beacon-lights to the courts. They have helped the courts
in exercising their power of judicial review, that is, the power to determine the
constitutional validity of a law.

http://www.insightsonindia.com

INSIGHTS

Page 3

Insights Mock Tests 2015: Solutions Test - 15

They form the dominating background to all State action, legislative or executive and
also a guide to the courts in some respects.
They amplify the Preamble, which solemnly resolves to secure to all citizens of India
justice, liberty, equality and fraternity.

10. Solution: b)

11. Solution: b)
As per the Swaran Singh Committee (that suggested inclusion of fundamental rights);
including duty to pay taxes should have been a fundamental duty, but it was not agreed
upon.
So as per the Direct taxation laws; income and other kinds of taxes are a legal obligation for
an Indian citizen as he uses public services, receives social and political security from the
state etc.

12. Solution: b)
According to Article 51 A, it shall be the duty of every citizen of India:
to abide by the Constitution and respect its ideals and institutions, the National Flag
and the National Anthem;
http://www.insightsonindia.com

INSIGHTS

Page 4

Insights Mock Tests 2015: Solutions Test - 15

to cherish and follow the noble ideals that inspired the national struggle for freedom;
to uphold and protect the sovereignty, unity and integrity of India;
to defend the country and render national service when called upon to do so;
to promote harmony and the spirit of common brotherhood amongst all the people
of India transcending religious, linguistic and regional or sectional diversities and to
renounce practices derogatory to the dignity of women;
to value and preserve the rich heritage of the countrys composite culture;
to protect and improve the natural environment including forests, lakes, rivers and
wildlife and to have compassion for living creatures;
to develop scientific temper, humanism and the spirit of inquiry and reform;
to safeguard public property and to abjure violence;
to strive towards excellence in all spheres of individual and collective activity so that
the nation constantly rises to higher levels of endeavour and achievement; and
to provide opportunities for education to his child or ward between the age of six
and fourteen years. This duty was added by the 86th Constitutional Amendment
Act, 2002.

13. Solution: c)
Fundamental Duties help the courts in examining and determining the constitutional
validity of a law. In 1992, the Supreme Court ruled that in determining the constitutionality
of any law, if a court finds that the law in question seeks to give effect to a fundamental
duty, it may consider such law to be reasonable in relation to Article 14 (equality before
law) or Article 19 (six freedoms) and thus save such law from unconstitutionality.
They are enforceable by law. Hence, the Parliament can provide for the imposition of
appropriate penalty or punishment for failure to fulfil any of them.

14. Solution: b)
An amendment of the Constitution can be initiated only by the introduction of a bill for the
purpose in either House of Parliament and not in the state legislatures.
It is true that the proposal for amendment may come from any of the options mentioned, but
only after the executive clears it can it be put forth the Parliament. From here the formal
process starts.

15. Solution: d)
The bill can be introduced either by a minister or by a private member and does not require
prior permission of the president.

http://www.insightsonindia.com

INSIGHTS

Page 5

Insights Mock Tests 2015: Solutions Test - 15


A private member here means a MP who does not belong to the ruling coalition.
Special days in the Parliament are reserved for Private member bills not constitutional
amendment bills.

16. Solution: d)
The bill must be passed in each House by a special majority, that is, a majority (that is, more
than 50 per cent) of the total membership of the House and a majority of two-thirds of the
members of the House present and voting.
In case of two-third of total membership approving the bill, the second condition
automatically gets fulfilled.

17. Solution: d)
After duly passed by both the Houses of Parliament and ratified by the state legislatures,
where necessary, the bill is presented to the president for assent.
The president must give his assent to the bill. He can neither withhold his assent to the bill
nor return the bill for reconsideration of the Parliament.
After the presidents assent, the bill becomes an Act (i.e., a constitutional amendment act)
and the Constitution stands amended in accordance with the terms of the Act.

18. Solution: a)
These matters include:

Admission or establishment of new states.


Formation of new states and alteration of areas, boundaries or names of existing
states.
Abolition or creation of legislative councils in states.
Second Scheduleemoluments, allowances, privileges and so on of the president,
the governors, the Speakers, judges, etc.
Quorum in Parliament.
Salaries and allowances of the members of Parliament.
Rules of procedure in Parliament.
Privileges of the Parliament, its members and its committees.
Use of English language in Parliament.
Number of puisne judges in the Supreme Court.
Conferment of more jurisdiction on the Supreme Court.
Use of official language.

http://www.insightsonindia.com

INSIGHTS

Page 6

Insights Mock Tests 2015: Solutions Test - 15

Citizenshipacquisition and termination.


Elections to Parliament and state legislatures.
Delimitation of constituencies.
Union territories.
Fifth Scheduleadministration of scheduled areas and scheduled tribes.
Sixth Scheduleadministration of tribal areas.

19. Solution: d)
The following are part of the basic structure as has become clear from several rulings of
Supreme Court:

Supremacy of the Constitution


Sovereign, democratic and republican nature of the Indian polity
Secular character of the Constitution
Separation of powers between the legislature, the executive and the judiciary
Federal character of the Constitution
Unity and integrity of the nation
Welfare state (socio-economic justice)
Judicial review
Freedom and dignity of the individual
Parliamentary system
Rule of law
Harmony and balance between Fundamental Rights and Directive Principles
Principle of equality
Free and fair elections
Independence of Judiciary
Limited power of Parliament to amend the Constitution
Effective access to justice
Principle of reasonableness
Powers of the Supreme Court under Articles 32, 136, 141 and 142

20. Solution: d)
The ministers are collectively responsible to the Parliament in general and to the Lok Sabha
in particular (Article 75). They act as a team, and swim and sink together. The principle of
collective responsibility implies that the Lok Sabha (not Parliament) can remove the ministry
(i.e., council of ministers headed by the prime minister) from office by passing a vote of no
confidence.

21. Solution: a)
http://www.insightsonindia.com

INSIGHTS

Page 7

Insights Mock Tests 2015: Solutions Test - 15


The Prime Minister plays the leadership role in this system of government. He is the leader
of council of ministers, leader of the Parliament and leader of the party in power. In these
capacities, he plays a significant and highly crucial role in the functioning of the
government.
Although all decisions are supposed to be taken based on consensus in the Council, he has
the final say over all the decisions of the Council of Ministers. To this effect, he can even
dismiss a few ministers who are causing frequent disagreement in the council.

22. Solution: a)
The lower house of the Parliament (Lok Sabha) can be dissolved by the President on
recommendation of the Prime Minister. In other words, the prime minister can advise the
President to dissolve the Lok Sabha before the expiry of its term and hold fresh elections.
This means that the executive enjoys the right to get the legislature dissolved in a
parliamentary system.
Similarily, the legislature can also pass no confidence motion against the executive and
dismiss it. With it, the Lok Sabha is dissolved if there is no possibility of forming an
alternative government.
Resignation of any minister will not end Lok Sabha only that of the Prime Minister (that
too when no other person is chosen as PM) will bring the Lok Sabha to dissolution.

23. Solution: a)
Yes, India lies entirely in the northern hemisphere.
Second statement is wrong, because it implies that Indias east west distance is greater
than its north-south distance, which is incorrect. India measures 3,214 km (1,997 mi) from
north to south and 2,933 km (1,822 mi) from east to west.

24. Solution: c)

http://www.insightsonindia.com

INSIGHTS

Page 8

Insights Mock Tests 2015: Solutions Test - 15

25. Solution: b)
Tributaries of Ganga:
Tributaries
- left

Ramganga, Gomti, Ghaghara,Gandaki, Burhi


Gandak, Koshi, Mahananda

- right

Yamuna, Tamsa, Son, Punpun,Betwa, Chambal, Tons,


Ken, Sindh,Hindon, Sharda

Tributaries of Brahmaputra
- left

Dibang River, Lohit


River,Dhansiri
River, Kalang

http://www.insightsonindia.com

INSIGHTS

Page 9

Insights Mock Tests 2015: Solutions Test - 15


- right

Kameng River, Manas


River,Raidak
River, Jaldhaka
River,Teesta
River, Subansiri

26. Solution: b)
http://nca.gov.in/nb_basin.htm
India 2015 (Page no. 4)

27. Solution: c)
The Western Himalayan region extends from Kashmir to Kumaon. Its temperate zone is rich
in forests of chir, pine, other conifers and broad-leaved temperate trees. Higher up, forests of
deodar, blue pine, spruce and silver fir occur. The alpine zone extends from the upper limit
of the temperate zone of about 4,750 metres or even higher. The characteristic trees of this
zone are high-level silver fir, silver birch and junipers. The eastern Himalayan region
extends from Sikkim eastwards and embraces Darjeeling, Kurseong and the adjacent tract.
The temperate zone has forests of oaks, laurels, maples, rhododendrons, alder and birch.
Many conifers, junipers and dwarf willows also occur here. The Assam region comprises the
Brahmaputra and the Surma valleys with evergreen forests, occasional thick clumps of
bamboos and tall grasses. The Indus plain region comprises the plains of Punjab, western
Rajasthan and northern Gujarat. It is dry and hot and supports natural vegetation. The
Ganga plain region covers the area which is alluvial plain and is under cultivation for wheat,
sugarcane and rice. Only small areas support forests of widely differing types. The Deccan
region comprises the entire tableland of the Indian Peninsula and supports vegetation of
various kinds from scrub jungles to mixed deciduous forests. The Malabar region covers the
excessively humid belt of mountain country parallel to the west coast of the Peninsula.
Besides being rich in forest vegetation, this region produces important commercial crops,
such as coconut, betel nut, pepper, coffee and tea, rubber and cashew nut. The Andaman
region abounds in evergreen, mangrove, beach and diluvia forests. The Himalayan region
extending from Kashmir to Arunachal Pradesh through Nepal, Sikkim, Bhutan, Meghalaya
and Nagaland and the Deccan Peninsula is rich in endemic flora, with a large number of
plants which are not found elsewhere.

28. Solution: d)
All are incorrect. Please refer Pages 8 and 9 of India 2015 for more information.

http://www.insightsonindia.com

INSIGHTS

Page 10

Insights Mock Tests 2015: Solutions Test - 15


Census 2011 is 15th census since 1872
Overall sex ration increased by 7 points to 943.
Indias population growth decreased between 1911 and 21 - an exception.
Bihar is the thickly populated state in India according to census 2011.

29. Solution: d)
Apart from non-statutory instructions issued by the Government from time to time, display
of the National Flag is governed by the provisions of the Emblems and Names (Prevention
of Improper Use) Act, 1950 (No.12 of 1950) and the Prevention of Insults to National Honour
Act, 1971 (No. 69 of 1971). Flag Code of India, 2002 is an attempt to bring together all such
laws, conventions, practices and instructions for the guidance and benefit of all concerned.
For the sake of convenience, Flag Code of India, 2002, has been divided into three parts. Part
I of the Code contains general description of the National Flag. Part II of the Code is devoted
to the display of the National Flag by members of public, private organizations, educational
institutions, etc. Part III of the Code relates to display of the National Flag by Central and
State governments and their organisations and agencies.

30. Solution: a)
The emblem of India is an adaptation of the Lion Capital of Ashoka at Sarnath, preserved in
the Sarnath Museum in India. The actual Sarnath capital including four Asiatic lions
standing back to back - symbolizing power, courage, pride, and confidence - mounted on a
circular base.At the bottom it has 1 horse & a bull,at its centre it has a beautiful wheel
(Dharma chakra). The abacus is girded with a frieze of sculptures in high relief of an
elephant (of the east), a bull (of the west), a horse (of the south), and a lion (of the north),
separated by intervening wheels, over a lotus in full bloom, exemplifying the fountainhead
of life and creative inspiration. Carved out of a single block of polished sandstone, the
capital is crowned by the Wheel of the Law (Dharma Chakra). In the emblem adopted by
Madhav Sawhney in 1950 only three lions are visible, the fourth being hidden from view.
The wheel appears in relief in the centre of the abacus, with a bull on the right and a
galloping horse on the left, and outlines of Dharma Chakras on the extreme right and left.
The bell-shaped lotus beneath the abacus has been omitted.

31. Solution: b)
Vande Mataram was composed in Sanskrit. It was first sung at the 1896 session of the
Indian National Congress.

http://www.insightsonindia.com

INSIGHTS

Page 11

Insights Mock Tests 2015: Solutions Test - 15

32. Solution: c)

33. Solution: a)
According to the Part-V, Article-75 of the Indian Constitution, the Council of Ministers are
collectively responsible to only to the Lok Sabha.

34. Solution: a)
The Right against Exploitation, contained in Articles 2324, lays down certain provisions to
prevent exploitation of the weaker sections of the society by individuals or the State. Article
23 provides prohibits human trafficking, making it an offence punishable by law, and also
prohibits forced labour or any act of compelling a person to work without wages where he
was legally entitled not to work or to receive remuneration for it. However, it permits the
State to impose compulsory service for public purposes, including conscription and
community service. The Bonded Labour system (Abolition) Act, 1976, has been enacted by
Parliament to give effect to this Article. Article 24 prohibits the employment of children
below the age of 14 years in factories, mines and other hazardous jobs. Parliament has
enacted the Child Labour (Prohibition and Regulation) Act, 1986, providing regulations for
the abolition of, and penalties for employing, child labour, as well as provisions for
rehabilitation of former child labourers.
Article 39 lays down certain principles of policy to be followed by the State, including
providing an adequate means of livelihood for all citizens, equal pay for equal work for men
and women, proper working conditions, reduction of the concentration of wealth and means
of production from the hands of a few, and distribution of community resources to
"subserve the common good".
Despite being non-justiciable, the Directive Principles act as a check on the State; theorised
as a yardstick in the hands of the electorate and the opposition to measure the performance
http://www.insightsonindia.com

INSIGHTS

Page 12

Insights Mock Tests 2015: Solutions Test - 15


of a government at the time of an election. Article 37, while stating that the Directive
Principles are not enforceable in any court of law, declares them to be "fundamental to the
governance of the country" and imposes an obligation on the State to apply them in matters
of legislation.

35. Solution: c)
During an Emergency, the Central government becomes all powerful and the states go into
the total control of the Centre. It converts the federal structure into a unitary one without a
formal amendment of the Constitution. This kind of transformation of the political system
from federal during normal times to unitary during Emergency is a unique feature of the
Indian Constitution.
Since the state governments do exist even at the time of emergency, we can not call it single
government.

36. Solution: a)
Under Article 352, the President can declare a national emergency when the security of India
or a part of it is threatened by war or external aggression or armed rebellion. It may be noted
that the president can declare a national emergency even before the actual occurrence of war
or external aggression or armed rebellion, if he is satisfied that there is an imminent danger.
A proclamation of national emergency may be applicable to the entire country or only a part
of it. The 42nd Amendment Act of 1976 enabled the president to limit the operation of a
National Emergency to a specified part of India.

37. Solution: d)
The President can proclaim a national emergency only after receiving a written
recommendation from the cabinet. This means that the emergency can be declared only on
the concurrence of the cabinet and not merely on the advice of the prime minister. In 1975,
the then Prime Minister, Indira Gandhi advised the president to proclaim emergency
without consulting her cabinet.
The cabinet was informed of the proclamation after it was made, as a fait accompli. The 44th
Amendment Act of 1978 introduced this safeguard to eliminate any possibility of the prime
minister alone taking a decision in this regard.
Approval of the Parliament is required later to decide whether to continue the emergency or
not.

http://www.insightsonindia.com

INSIGHTS

Page 13

Insights Mock Tests 2015: Solutions Test - 15


38. Solution: b)
Identifying features of indicative planning may be summed up as under:

Every economy following the indicative planning were the mixed economies.
Unlike a centrally planned economy (countries following imperative planning)
indicative planning works through the market (price system) rather than replaces
it.15
Side by side setting numerical/quantitative targets (similar to the practice in the
imperative planning) a set of economic policies of indicative nature is also
announced by the economies to realise the plan targets.
The indicative nature of economic policies which are announced in such planning
basically encourage or discourage the private sector in its process of economic
decision making.

39. Solution: d)
7th schedule refers to the division of legislative subjects b/w Centre and states.
Economic and social planning is a concurrent subject. Also, while framing the Union, State
and Concurrent list, allocating subjects and other provisions, the Constitution vests power in
the Union to ensure co-ordinated development in essential fields of activity while preserving
the initiative and authority of the states in the spheres allotted to them.
Part IX refers to Panchayats which are responsible for local planning.
Sixth schedule gives elaborate guidelines on the political structure of the tribal areas; and
gives them the right to manage and redistribute their own resources and plan for economic
activities via several structures.

40. Solution: b)
http://en.wikipedia.org/wiki/NITI_Aayog

41. Solution: d)
Infrastructure, Including Rural Infrastructure targets under 12 th FYP

Increase investment in infrastructure as a percentage of GDP to 9 per cent by the end


of Twelfth Five Year Plan.
Increase the Gross Irrigated Area from 90 million hectare to 103 million hectare by
the end of Twelfth Five Year Plan.

http://www.insightsonindia.com

INSIGHTS

Page 14

Insights Mock Tests 2015: Solutions Test - 15

Provide electricity to all villages and reduce AT&C losses to 20 per cent by the end of
Twelfth Five Year Plan.
Connect all villages with all-weather roads by the end of Twelfth Five Year Plan.
Upgrade national and state highways to the minimum two-lane standard by the end
of Twelfth Five Year Plan.
Complete Eastern and Western Dedicated Freight Corridors by the end of Twelfth
Five Year Plan.
Increase rural tele-density to 70 per cent by the end of Twelfth Five Year Plan.
Ensure 50 per cent of rural population has access to 40 lpcd piped drinking water
supply, and 50 per cent gram panchayats achieve Nirmal Gram Status by the end of
Twelfth Five Year Plan.

42. Solution: c)
Except 5th option, all are fundmental duties
The Fundamental Duties noted in the constitution are as follows:
It shall be the duty of every citizen of India

to abide by the Constitution and respect its ideals and institutions, the National Flag and
the National Anthem;

to cherish and follow the noble ideals which inspired our national struggle for freedom;

to uphold and protect the sovereignty, unity and integrity of India;

to defend the country and render national service when called upon to do so;

to promote harmony and the spirit of common brotherhood amongst all the people of
India transcending religious, linguistic and regional or sectional diversities; to renounce
practices derogatory to the dignity of women;

to value and preserve the rich heritage of our composite culture;

to protect and improve the natural environment including forests, lakes, rivers and wild
life, and to have compassion for living creatures;

to develop the scientific temper, humanism and the spirit of inquiry and reform;

to safeguard public property and to abjure violence;

to strive towards excellence in all spheres of individual and collective activity so that the
nation constantly rises to higher levels of endeavour and achievement;

who is a parent or guardian to provide opportunities for education to his child or ward,
as the case may be, between the age of six and fourteen years

43. Solution: b)

http://www.insightsonindia.com

INSIGHTS

Page 15

Insights Mock Tests 2015: Solutions Test - 15


Article 54 of the constitution says:
"The President shall be elected by the members of an electoral college consisting of (a) The elected members of both Houses of Parliament and
(b) The elected members of the Legislative Assemblies of the States (including National
Capital Territory of Delhi and the Union Territory of Pondicherry vide the Constitution 70th
amendment Act, 1992)."
Thus in the election of the President the citizens play no direct part and he is elected
indirectly by the representatives or the people, like the American President but no special
electoral college is elected, as in the case of America. Another point of difference that may be
noted is that the election of the President of India is by the system of proportional
representation, by the single transferable vote, as provided by Article 55(3) of the
Constitution, while the American President is elected by the straight vote system.

http://pib.nic.in/archieve/others/pr.html
http://www.thehindu.com/todays-paper/tp-in-school/how-is-indias-presidentelected/article3617995.ece

44. Solution: b)
The Vice President shall act as President in the absence of the President due to death,
resignation, impeachment, or other situations. The Vice President of India is also ex
officio Chairperson of the Rajya Sabha. He ceases to perform as Chairman of Rajya Sabha
while acting as the President.
Article 66 of the Indian Constitution states the manner of election of the Vice-President. The
Vice-President is elected indirectly by members of an electoral college consisting of the
members of both Houses of Parliament in accordance with the system ofProportional
Representation by means of the Single transferable vote and the voting is by secret ballot

45. Solution: d)
The Rajya Sabha meets in continuous sessions, and unlike the Lok Sabha, the lower house of
Parliament, is not subject to dissolution. The Rajya Sabha has equal footing in all areas of
http://www.insightsonindia.com

INSIGHTS

Page 16

Insights Mock Tests 2015: Solutions Test - 15


legislation with Lok Sabha, except in the area of supply, where the Lok Sabha has overriding
powers.
Members sit for six-year terms, with one third of the members retiring every two years.

46. Solution: d)
Parliamentary Committees are of two kinds: Ad hoc Committees and the Standing
Committees. Ad hoc Committees are appointed for a specific purpose and they cease to exist
when they finish the task assigned to them and submit a report. The principal Ad
hoc Committees are the Select and Joint Committees on Bills. Others like the Railway
Convention Committee, the Committees on the Draft Five Year Plans and the Hindi
Equivalents Committee were appointed for specific purposes. Apart from the Ad
hoc Committees, each House of Parliament has Standing Committees like the Business
Advisory Committee, the Committee on Petitions, the Committee of Privileges and the Rules
Committee, etc.
Standing committee is a committee consisting of Members of Parliament. It is a permanent
and regular committee which is constituted from time to time according to the provisions of
an Act of Parliament or Rules of Procedure and Conduct of Business. The work done by
the Indian Parliament is not only voluminous but also of a complex nature, hence a good
deal of its work is carried out in these Parliamentary Committees. [1]
Both Houses of Parliament, Rajya Sabha and Lok Sabha, have similar Committee structures
with a few exceptions. Their appointment, terms of office, functions and procedures of
conducting business are broadly similar. These standing committees are elected or
appointed every year, or periodically by the Chairman of the Rajya Sabha or the Speaker of
the Lok Sabha, or as a result of consultation between them.

47. Solution: a)
Committee on Estimates
This Committee consists of 30 members who are elected by the Lok Sabha every year from
amongst its members. A Minister is not eligible for election to this Committee. The term of
the Committee is one year. The main function of the Committee on Estimates is to report
what economies, improvements in organisation, efficiency, or administrative reform,
consistent with the policy underlying the estimates may be effected and to suggest
alternative policies in order to bring about efficiency and economy in administration. From
time to time the Committee selects such of the estimates pertaining to a Ministry or a group
of Ministries or the statutory and other Government bodies as may seem fit to the
Committee. The Committee also examines matters of special interest which may arise or

http://www.insightsonindia.com

INSIGHTS

Page 17

Insights Mock Tests 2015: Solutions Test - 15


come to light in the course of its work or which are specifically referred to it by the House
or the Speaker.
Committee on Public Undertakings
The Committee on Public Undertakings consists of 15 members elected by the Lok Sabha
and 7 members of Rajya Sabha are associated with it. A Minister is not eligible for election to
this Committee. The term of the Committee is one year.
The functions of the Committee on Public Undertakings are(a) to examine the reports and
accounts of Public Undertakings; (b) to examine the reports, if any, of the Comptroller and
Auditor General on the Public Undertakings; (c) to examine in the context of the autonomy
and efficiency of the Public Undertakings whether the affairs of the Public Undertakings are
being managed in accordance with sound business principles and prudent commercial
practices; and (d) such other functions vested in the Committee on Public Accounts and the
Committee on Estimates in relation to the Public Undertakings as are not covered by clauses
(a), (b) and (c) above and as may be allotted to the Committee by the Speaker from time to
time. The Committee does not, however, examine matters of major Government policy and
matters of day-to-day administration of the Undertakings.
Committee on Public Accounts
This Committee consists of 15 members elected by the Lok Sabha and 7 members of the
Rajya Sabha are associated with it. A Minister is not eligible for election to this Committee.
The term of the Committee is one year.
The main duty of the Committee is to ascertain whether the money granted by Parliament
has been spent by Government "within the scope of the Demand". The Appropriation
Accounts of the Government of India and the Audit Reports presented by the Comptroller
and Auditor General mainly form the basis for the examination of the Committee. Cases
involving losses, nugatory expenditure and financial irregularities come in for severe
criticism by the Committee. The Committee is not concerned with questions of policy. It is
concerned only with the execution of the policy laid down by Parliament and its results.
Business Advisory Committee (Lok Sabha)
The Business Advisory Committee of Lok Sabha consists of 15 members including the
Speaker who is the ex-officio Chairman. The members are nominated by the Speaker. Almost
all sections of the House are represented on the Committee as per the respective strength of
parties in the House. The function of the Committee is to recommend the time that should
be allotted for the discussion of such Government legislative and other business as the
Speaker, in consultation with the Leader of the House, may direct to be referred to the
Committee. The Committee, on its own initiative, may also recommend to the Government
to bring forward particular subjects for discussion in the House and recommend allocation
of time for such discussions. The decisions reached by the Committee are always unanimous

http://www.insightsonindia.com

INSIGHTS

Page 18

Insights Mock Tests 2015: Solutions Test - 15


in character and representative of the collective view of the House. The Committee generally
meets at the beginning of each Session and thereafter as and when necessary.

48. Solution: d)
Functions assigned to the Ministry under the Government of India (Allocation of Business)
Rules, 1961 made by the President under Article 77(3) of the Constitution of India:

Dates of summoning and prorogation of the two Houses of Parliament, Dissolution


of Lok Sabha, President's Address to Parliament.

Planning and Coordination of legislative and other official business in both Houses.

Allocation of Government time in Parliament for discussion of motions given notice


of by Members.

Liaison with Leaders and Whips of various Parties and Groups represented in
Parliament.

Lists of Members of Select and Joint Committees on Bills.

Appointment of Members of Parliament on Committees and other bodies set up by


Government.

Functioning of Consultative Committees of Members of Parliament for various


Ministries.

Implementation of assurances given by Ministers in Parliament.

Governments stand on Private Members Bills and Resolutions.

Secretarial assistance to the Cabinet Committee on Parliamentary Affairs.

Advice to Ministries on procedural and other Parliamentary matters.

Coordination of action by Ministries on recommendations of general application


made by Parliamentary Committees.

Officially sponsored visits of Members of Parliament to places of interest.

Matters connected with powers, privileges and immunities of Members of


Parliament.

Parliamentary Secretaries - functions.

Organisation of Youth Parliament Competitions in Schools/Colleges throughout the


country.

Organisation of All India Whips Conference.

http://www.insightsonindia.com

INSIGHTS

Page 19

Insights Mock Tests 2015: Solutions Test - 15

Exchange of Government Sponsored Delegations of Members of Parliament with


other countries.

Determination of Policy and follow up action in regard to matters raised under rule
377 of the Rules of Procedure and Conduct of Business in Lok Sabha and by way of
Special Mentions in Rajya Sabha.

Manual for Handling Parliamentary work in Ministries/Departments.

The Salaries and Allowances of Officers of Parliament Act, 1953 (20 of 1953).

The Salary, Allowances and Pensions of Members of Parliament Act, 1954 (30 of
1954).

The Salary, and Allowances of Leaders of Opposition in Parliament Act, 1977 (33 of
1977).

The Leader and Chief Whips of Recognised Parties and Groups in Parliament
(Facilities) Act, 1998 (5 of 1999)

49. Solution: c)
The division of powers is in favour of the Centre and highly inequitable from the federal
angle. Firstly, the Union List contains more subjects than the State List. Secondly, the more
important subjects have been included in the Union List. Thirdly, the Centre has overriding
authority over the Concurrent List. Finally, the residuary powers have also been left with
the Centre, while in the US, they are vested in the states. Thus, the Constitution has made
the Centre very strong.
The Constitution stipulates three types of emergencies - national, state and financial. During
an emergency, the Central government becomes all powerful and the states go into the total
control of the Centre. It converts the federal structure into a unitary one without a formal
amendment of the Constitution. This kind of transformation is not found in any other
federation.
The governor is empowered to reserve certain types of bills passed by the state legislature
for the consideration of the President. The President can withhold his assent to such bills not
only in the first instance but also in the second instance. Thus, the President enjoys absolute
veto (and not suspensive veto) over state bills. But in US and Australia, the states are
autonomous within their fields and there is no provision for any such reservation.

50. Solution: b)

http://www.insightsonindia.com

INSIGHTS

Page 20

Insights Mock Tests 2015: Solutions Test - 15


The power to make laws with respect to residuary subjects (i.e., the matters which are not
enumerated in any of the three lists) is vested in the Parliament. This residuary power of
legislation includes the power to levy residuary taxes.
From the above scheme, it is clear that the matters of national importance and the matters
which requires uniformity of legislation nationwide are included in the Union List. The
matters of regional and local importance and the matters which permits diversity of interest
are specified in the State List. The matters on which uniformity of legislation throughout the
country is desirable but not essential are enumerated in the concurrent list. Thus, it permits
diversity along with uniformity.

51. Solution: a)
The Constitution expressly secures the predominance of the Union List over the State List
and the Concurrent List and that of the Concurrent List over the State List. Thus, in case of
overlapping between the Union List and the State List, the former should prevail. In case of
overlapping between the Union List and the Concurrent List, it is again the former which
should prevail. Where there is a conflict between the Concurrent List and the State List, it is
the former that should prevail.
In case of a conflict between the Central law and the state law on a subject enumerated in the
Concurrent List, the Central law prevails over the state law. But, there is an exception. If the
state law has been reserved for the consideration of the president and has received his
assent, then the state law prevails in that state. But, it would still be competent for the
Parliament to override such a law by subsequently making a law on the same matter.

52. Solution: b)
If the Rajya Sabha declares that it is necessary in the national interest that Parliament should
make laws on a matter in the State List, then the Parliament becomes competent to make
laws on that matter. Such a resolution must be supported by two-thirds of the members
present and voting. The resolution remains in force for one year; it can be renewed any
number of times but not exceeding one year at a time. The laws cease to have effect on the
expiration of six months after the resolution has ceased to be in force.
When the legislatures of two or more states pass resolutions requesting the Parliament to
enact laws on a matter in the State List, then the Parliament can make laws for regulating
that matter. A law so enacted applies only to those states which have passed the resolutions.
However, any other state may adopt it afterwards by passing a resolution to that effect in its
legislature. Such a law can be amended or repealed only by the Parliament and not by the
legislatures of the concerned states.

http://www.insightsonindia.com

INSIGHTS

Page 21

Insights Mock Tests 2015: Solutions Test - 15


53. Solution: a)
The Constitution empowers the Centre to exercise control over the states legislative matters
in the following ways:
(i) The governor can reserve certain types of bills passed by the state legislature for the
consideration of the President. The president enjoys absolute veto over them.
(ii) Bills on certain matters enumerated in the State List can be introduced in the state
legislature only with the previous sanction of the president. (For example, the bills imposing
restrictions on the freedom of trade and commerce).
(iii) The President can direct the states to reserve money bills and other financial bills passed
by the state legislature for his consideration during a financial emergency.

54. Solution: a)
The Constitution contains the following other provisions which enable the Centre to exercise
control over the state administration:
(i) Article 355 imposes two duties on the Centre: (a) to protect every state against external
aggression and internal disturbance; and (b) to ensure that the government of every state is
carried on in accordance with the provisions of the Constitution.
(ii) The governor of a state is appointed by the president. He holds office during the pleasure
of the President. In addition to the Constitutional head of the state, the governor acts as an
agent of the Centre in the state. He submits periodical reports to the Centre about the
administrative affairs of the state.
(iii) The state election commissioner, though appointed by the governor of the state, can be
removed only by the President.

55. Solution: b)
These include a number of advisory bodies and conferences held at the Central level.
The non-constitutional advisory bodies include the NITI AYOG, the National Development
Council, the National Integration Council, the Central Council of Health, the Central Council
of Local Government and Urban Development, the Zonal Councils, the North-Eastern
Council, the Central Council of Indian Medicine, Central Council of Homo-eopathy, the
Central Family Welfare Council, the Transport Development Council, the University Grants
Commission and so on.
The important conferences held either annually or otherwise to facilitate Centrestate
consultation on a wide range of matters are as follows: (i) The governors conference
(presided over by the President). (ii) The chief ministers conference (presided over by the
http://www.insightsonindia.com

INSIGHTS

Page 22

Insights Mock Tests 2015: Solutions Test - 15


prime minister). (iii) The chief secretaries conference (presided over by the cabinet
secretary). (iv) The conference of inspector-general of police. (v) The chief justices
conference (presided over by the chief justice of India). (vi) The conference of vice-cancellors.
(vii) The home ministers conference (presided over by the Central home minister). (viii) The
law ministers conference (presided over by the Central law minister).

56. Solution: b)
Service Tax Levied by the Centre but Collected and Appropriated by the Centre and the
States (Article 268-A): Taxes on services are levied by the Centre. But, their proceeds are
collected as well as appropriated by both the Centre and the states. The principles of their
collection and appropriation are formulated by the Parliament.
Taxes Levied and Collected by the Centre but Assigned to the States (Article 269): The
following taxes fall under this category:
(i) Taxes on the sale or purchase of goods (other than newspapers) in the course of inter-state
trade or commerce.
(ii) Taxes on the consignment of goods in the course of inter-state trade or commerce

57. Solution: a)
The Centre The receipts from the following form the major sources of non-tax revenues of
the Centre: (i) posts and telegraphs; (ii) railways; (iii) banking; (iv) broadcasting (v) coinage
and currency; (vi) central public sector enterprises; and (vii) escheat and lapse.19
The States The receipts from the following form the major sources of non-tax revenues of the
states: (i) irrigation; (ii) forests; (iii) fisheries; (iv) state public sector enterprise; and (v)
escheat and lapse.

58. Solution: a)
Article 275 empowers the Parliament to make grants to the states which are in need of
financial assistance and not to every state. Also, different sums may be fixed for different
states. These sums are charged on the Consolidated Fund of India every year.
Apart from this general provision, the Constitution also provides for specific grants for
promoting the welfare of the scheduled tribes in a state or for raising the level of
administration of the scheduled areas in a state including the State of Assam.
The statutory grants under Article 275 (both general and specific) are given to the states on
the recommendation of the Finance Commission.
http://www.insightsonindia.com

INSIGHTS

Page 23

Insights Mock Tests 2015: Solutions Test - 15


The Constitution also provided for a third type of grants-in-aid (other than Statutory and
Discretionary), but for a temporary period. Thus, a provision was made for grants in lieu of
export duties on jute and jute products to the States of Assam, Bihar, Orissa and West
Bengal. These grants were to be given for a period of ten years from the commencement of
the Constitution. These sums were charged on the Consolidated Fund of India and were
made to the states on the recommendation of the Finance Commission.

59. Solution: a)
To protect the interest of states in the financial matters, the Constitution lays down that the
following bills can be introduced in the Parliament only on the recommendation of the
President:

A bill which imposes or varies any tax or duty in which states are interested;
A bill which varies the meaning of the expression agricultural income as defined for
the purposes of the enactments relating to Indian income tax;
A bill which affects the principles on which moneys are or may be distributable to
states; and
A bill which imposes any surcharge on any specified tax or duty for the purpose of
the Centre.

60. Solution: b)
The Inter-State Water Disputes Act empowers the Central government to set up an ad hoc
tribunal for the adjudication of a dispute between two or more states in relation to the
waters of an inter-state river or river valley. The decision of the tribunal would be final and
binding on the parties to the dispute. Neither the Supreme Court nor any other court is to
have jurisdiction in respect of any water dispute which may be referred to such a tribunal
under this Act.
The need for an extra judicial machinery to settle inter-state water disputes is as follows:
The Supreme Court would indeed have jurisdiction to decide any dispute between states in
connection with water supplies, if legal rights or interests are concerned; but the experience
of most countries has shown that rules of law based upon the analogy of private proprietary
interests in water do not afford a satisfactory basis for settling disputes between the states
where the interests of the public at large in the proper use of water supplies are involved.

61. Solution: c)

http://www.insightsonindia.com

INSIGHTS

Page 24

Insights Mock Tests 2015: Solutions Test - 15


The first National Tribal Festival Vanaj concluded this month - February 2015 in New
Delhi.
It was organised by the Ministry of Tribal Affairs.
Key facts

The aim of weeklong National Tribal Festival was to bridge the gaps of social divide
and promote tribal culture in India.

The purpose of the festival was to uplift the quality of tribal lives of the country in
order to conserve and promote their distinctive cultures by providing them national
platform.

More than 900 folk and tribal artists from states like Assam, Chhattisgarh, Gujarat,
Himachal Pradesh, Jharkhand, Kerala, Madhya Pradesh, Maharashtra, Nagaland,
Odisha, Rajasthan, Sikkim, Tamil Nadu and West Bengal participated in it.

Maiden edition of the festival showcased the rich diversity of Tribal culture, art,
painting, music, dance, tribal medicine, tribal food, and a special attraction- tribal
cuisine.

62. Solution: d)

It is duty imposed by government on imported products which have prices less than
their normal values or domestic price.

Usually countries initiate anti-dumping probes to check if domestic industry has


been hurt because of a surge in below-cost imports.

Anti-Dumping Duty is imposed under the multilateral WTO regime and vary from
product to product and from country to country.

In India, anti-dumping duty is recommended by the Union Ministry of Commerce,


while the Union Finance Ministry imposes it.

63. Solution: d)
India and Sri Lanka have signed an agreement on civil nuclear cooperation, Cultural
cooperation during Sri Lankas newly elected President Maithripala Sirisen four-day visit to
India.
Sri Lanka has signed first such bilateral agreement on civil nuclear cooperation.
Following agreements were signed between India and Sri Lanka
http://www.insightsonindia.com

INSIGHTS

Page 25

Insights Mock Tests 2015: Solutions Test - 15

Agreement on Cooperation in the Peaceful Uses of Nuclear Energy- It will facilitate


the transfer and exchange of knowledge and expertise, resources in peaceful uses of
Nuclear energy between both nations. This agreement also includes capacity
building and training of personnel for nuclear security, radioactive waste
management and nuclear and radiological disaster mitigation and environmental
protection.

Programme of Cultural cooperation between both nations for years 2015-18 It


seeks to enhance cooperation in different fields like arts, visuals arts, cultural
documentation, archeology, handicrafts etc between both nations.

MoU on establishment of Nalanda University- It will enable Sri Lanka to


participate in Indias Nalanda University project.

Work Plan 2014-15 under MoU on Cooperation in the field of Agriculture- It will
facilitate bilateral cooperation in Agro Processing, Agricultural extension,
horticulture, agricultural machinery, training in farm mechanization, livestock
diseases, etc. between relevant institutes and organizations from both countries.

64. Solution: d)
Minimum Support Price (MSP) is a form of market intervention by the Government of India
to insure agricultural producers against any sharp fall in farm prices. The minimum support
prices are announced by the Government of India at the beginning of the sowing season for
certain crops on the basis of the recommendations of the Commission for Agricultural Costs
and Prices (CACP).
In formulating the recommendations in respect of the level of minimum support prices and
other non-price measures, the CACP takes into account a comprehensive view of the entire
structure of the economy of a particular commodity or group of commodities. Other Factors
include cost of production, changes in input prices, input-output price parity, trends in
market prices, demand and supply, inter-crop price parity, effect on industrial cost
structure, effect on cost of living, effect on general price level, international price situation,
parity between prices paid and prices received by the farmers and effect on issue prices and
implications for subsidy. The Commission makes use of both micro-level data and
aggregates at the level of district, state and the country

65. Solution: d)
The FCI has been constructing storage capacity for holding buffer and operational stocks of
food grains at nodal points in the country. The storage capacities available with FCI are
mainly used for storage of food grains and partly for other commodities and general
warehousing.

http://www.insightsonindia.com

INSIGHTS

Page 26

Insights Mock Tests 2015: Solutions Test - 15


The buffer stocks are required to
1. Feed TPDS and other welfare schemes,
2. Ensure food security during the periods when production is short of normal demand
during bad agricultural years
3. Stabilize prices during period of production shortfall through open market sales.

66. Solution: a)
Import dependence was about 3 per cent during 1992-93. The production of oilseeds, though
it has increased in recent years (from 184.40 lakh tons in 2000-01 to 297.99 lakh tons in 201112), it has not kept pace with the demand for edible oils in India. Imports have helped raise
the per capita availability of edible oils which has increased from 5.8 kg in 1992-93 increased
to 14.5 kg in 2010-11.
ii. One instrument for promoting future domestic production is calibration of the import
duty structure. Large imports of edible oils are primarily due to competitive prices of edible
oils in the international market and the import duty structure which has been sharply
reduced to near zero levels over time to protect consumers India has such a high market
share (in the world edible oil imports) that allows it to set some independent tariff policy
that can meet both goals better.

67. Solution: d)
The role of the agriculture market is to deliver agricultural produce from the farmer to the
consumer in the most efficient way. Agriculture markets are regulated in India through the
APMC Acts.
According to the provisions of the APMC Acts of the states, every APMC (Agricultural
Produce Marketing Committee) is authorised to collect market fees from the buyers/traders
in the prescribed manner on the sale of notified agricultural produce. The relatively high
incidence of commission charges on agricultural/horticultural produce renders their
marketing cost high, which is an undesirable outcome.
All this suggests that a single point market fee system is necessary for facilitating free
movement of produce, bringing price stabilisation, and reducing price differences between
the producer and consumer market segments. Another point to be highlighted is that the
cleaning, grading, and packaging of agricultural produce before sale by the farmers have not
been popularised by these market committees on a sufficient scale.

68. Solution: d)
http://www.insightsonindia.com

INSIGHTS

Page 27

Insights Mock Tests 2015: Solutions Test - 15


The horticulture sector includes a wide range of crops, such as fruits, vegetables, roots and
tuber crops, flowers, aromatic and medicinal plants, spices, and plantation crops, which
facilitate diversification in agriculture. It has been recognised that growing horticulture
crops is now an ideal option to improve livelihood security, enhance employment
generation, attain food and nutritional security, and increase income through value
addition. Over the years, there have been noticeable achievements and significant
improvement in the production and productivity of various horticulture crops.
The NHM scheme was launched during the Tenth Plan for holistic development of the
horticulture sector, duly ensuring forward and backward linkages by adopting a cluster
approach, with the active participation of all the stakeholders. The supply of quality planting
material through establishment of nurseries and tissue culture units, production and
productivity improvement programmes through area expansion and rejuvenation,
technology promotion, technology dissemination, human resource development, creation of
infrastructure for post-harvest management and marketing in consonance with the
comparative advantages of each state/region and their diverse agro-climatic conditions are
the major programmes of the Mission.

69. Solution: d)
There are various major crop insurance schemes under implementation in the country:
(i) National Agricultural Insurance Scheme (NAIS): The NAIS is a government-sponsored
central-sector crop insurance scheme being implemented in the country since 1999-2000
season (the erstwhile Comprehensive Crop Insurance Scheme-CCIS of 1985 was merged into it)
with the objective of providing financial support to farmers in the event of failure of crops
as a result of natural calamities, pests, and diseases. The Agriculture Insurance Company of
India Ltd. (AICIL) is the implementing agency for the Scheme. At present, the scheme is
being implemented by 25 states and two UTs.
(ii) Modified NAIS (MNAIS): With the aim of further improving crop insurance schemes, the
MNAIS is under implementation on pilot basis in 50 districts in the country from rabi 201011 season. Some of the major improvements made in the MNAIS are

Actuarial premium with subsidy in premium at different rates


All claims liability to be on the insurer
Unit area of insurance reduced to village panchayat level for major crops
Indemnity for prevented/sowing/planting risk and for post-harvest losses
due to cyclone
On account payment up to 25 per cent advance of likely claims as immediate
relief
More proficient basis for calculation of threshold yield, and
Allowing private sector insurers with adequate infrastructure

Other schemes too cover the aforementioned.


http://www.insightsonindia.com

INSIGHTS

Page 28

Insights Mock Tests 2015: Solutions Test - 15

70. Solution: d)
It includes all those economic policies which intend to boost the aggregate demand in the
economy be it domestic or external. For the enhanced domestic demand, the focus has to
be on increasing the purchasing power of the masses which entails an emphasis on the
creation of the gainful and quality employment opportunities.
Cutting down CAD and fiscal deficit affect the level of demand and inflation in the macroeconomy.

71. Solution: a)
Several connotations and meanings of the term privatisation have developed. We may see
them as follows:
(i) Privatisation in its purest sense and lexically means de-nationalisation i.e. transfer of the
state ownership of the assets to the private sector to the tune of 100 per cent. Such bold
moves took place only once anywhere in the world without any political fallouts - in the
early 1980s of the UK under the Thatcher regime. This route of privatisation has been
avoided by almost all democratic systems. In the mid-1990s some west European nations
Italy, Spain and Francebesides the USA went for such moves.21 India never ventured into
any such privatisation move.
(ii) The sense in which privatisation has been used is the process of disinvestment all over
the world. This process includes selling of the shares of the state-owned enterprises to the
private sector. Disinvestment is de-nationalisation of less than 100 per cent ownership
transfer from the state to the private sector. If an asset has been sold out by the Government
to the tune of only 49 per cent the ownership remains with the state though it is considered
privatisation. If the sale of shares of the state-owned assets has been to the tune of 51 per
cent, the ownership is really transferred to the private sector even then it is termed as
privatisation.
(iii) The third and the last sense in which the term privatisation has been used around the
world, is very wide. Basically, all the economic policies which directly or indirectly seem to
promote the expansion of the private sector or the market (economy) have been termed by
the experts and the governments as the process of privatisation. We may cite a few examples
from India de-licencing and dereservatrion of the industries, even cuts in the subsidies,
permission to foreign investment, etc.

72. Solution: a)

http://www.insightsonindia.com

INSIGHTS

Page 29

Insights Mock Tests 2015: Solutions Test - 15


Considered as the backbone for the success of the reform process in India itself, it consists
of dismantling of the Administered Price Mechanism (APM). There were many products in
the economy whose prices were fixed/regulated by the Government viz. petroleum, sugar,
fertilizers, drugs, etc.
Though a major section of the products under the APM were produced by the private sector,
they were not sold on the market principles which hindered the profitability of the
manufacturers as well as the sellers and ultimately the expansion of the concerned
industries leading to a demand-supply gap. Under market reforms these products were to
be brought into the market fold.
In the petroleum segment now only kerosene oil and the LPG remained under the APM
while petrol, diesel, lubricants have been phased out. Similarly, the income tax paying
families dont get sugar from the TPS on subsidies; only urea, among the fertilizers, remain
under APM while many drugs have also been phased out of the mechanism. Opening the
petroleum sector for private investment, cutting down the burden of levy on sugar, etc. are
now giving dividends to the economy. But we cannot say that the Factor Market Reforms
(FMRs) are complete in India. It is still going on. Cutting down subsidies on the essential
goods is a socio-political question in India. Till market-base purchasing power is not
delivered to all the consumers, it would not be possible to complete the FMRs.

73. Solution: d)
When the general level of prices is falling over a period of time this is deflation, the opposite
situation of inflation. It is also known as disinflation. But in contemporary economics,
deflation or disinflation not used to indicate fall in prices. Instead, a price rise is termed a
rise in inflation and a price fall is termed a fall in inflation. The terms deflation or
disinflation have become part of the macroeconomic policy of modern governments. In
policy terms, the terms show a reduction in the level of national income and output, usually
accompanied by a fall in the general price level.
Such a policy is often deliberately brought about by the governments with the objective of
reducing, inflation and improving the balance of payments (BoP) by reducing import
demand. As instruments of deflation, any policy includes fiscal measures (as for example,
tax increase) and monetary measures (as for example, increase in interest rate).

74. Solution: d)
The governments may take recourse to tighter monetary policy to cool down either the
demand-pull or the cost-push inflations. This is basically intended to cut down the money
supply in the economy by siphoning out the extra money (as RBI increases the Cash Reserve
Ratio of bank in India) from the economy and by making money costlier (as RBI increases

http://www.insightsonindia.com

INSIGHTS

Page 30

Insights Mock Tests 2015: Solutions Test - 15


the Bank Rate or Repo Rate in India). This is a short-term measure. In the long-run, the best
way is to increase production with the help of the best production practices.
Again, this measure does not work if the price rise is taking place in the items of everyday
use such as salt, onion, wheat, etc. (because nobody purchases such goods by borrowing
from the banks). This measure helps if the prices are rising due to extra demand of cement,
iron and steel, etc.
75. Solution: a)
Core Inflation is based on the inclusion or exclusion of the goods and services while
calculating inflation. Popular in western economies, core inflation shows price rise in all
goods and services excluding energy and food articles. In India, it was first time used in the
financial year 200001 when the Government expressed that it was under controlit means
the prices of manufactured goods were under control. This was criticised by experts on
account of excluding food articles and energy out of the inflation and feeling satisfied on the
inflation front. Basically, in the western economies, food and energy are not the problems for
the masses, while in India these two segments play the most vital role for them.

76. Solution: a)
An inflationary situation in an economy which results out of a process of wage and price
interaction when wages press prices up and prices pull wages up is known as the
inflationary spiral. It is also known as the wage-price spiral. This wage-price interaction was
seen as a plausible cause of inflation in the year 1935 in the US economy, for the first time.
It can be seen in Indias case also when the government revises the Dearness allowance
(addition to the basic salary w.r.t. to inflation levels), it adds to the existing demand in the
economy and leads to inflation.

77. Solution: a)
A situation in an economy when inflation and unemployment both are at higher levels, is
contrary to conventional belief. Such a situation first arose in 1970s in the US economy
(average unemployment rate above 6 per cent and the average rate of inflation above 7 per
cent)32a and in many Euro- American economies. This took place as a result of oil price
increases of 1973 and 1979 and anticipation of higher inflation. The stagflationary situation
continued till the early 1980s. Conventional thinking that a trade-off existed between
inflation and unemployment (i.e. Phillips curve) was falsified and several economies
switched over to alternative ways of economic policies such as monetaristic and supply-side
economics.

http://www.insightsonindia.com

INSIGHTS

Page 31

Insights Mock Tests 2015: Solutions Test - 15


When the economy is passing through the cycle of stagnation (i.e. long period of low
aggregate demand in relation to its productive capacity) and the government shuffles with
the economic policy, a sudden and temporary price rise is seen in some of the goodssuch
inflation is also known as stagflation. Stagflation is basically a combination of high inflation
and low growth.

78. Solution: c)
Economists usually distinguish between inflation and a relative price increase. Inflation
refers to a sustained, across-the-board price increase, whereas a relative price increase is a
reference to an episodic price rise pertaining to one or a small group of commodities. This
leaves a third phenomenon, namely one in which there is a price rise of one or a small group
of commodities over a sustained period of time, without a traditional designation.
Skewflation is a relatively new term to describe this third category of price rise.
In India, food prices rose steadily during the last months of 2009 and the early months of
2010, even though the prices of non-food items continued to be relatively stable. As this
somewhat unusual phenomenon stubbornly persisted, policymakers conferred on how to
bring it to an end. The term skewflation made an appearance in internal documents of the
Government of India, and then appeared in print in the Economic Survey 2009-10, GoI, MoF.
The skewedness of inflation in India in the early months of 2010 was obvious from the fact
that food price inflation crossed the 20 per cent mark in multiple months, whereas wholesale
price index (WPI) inflation never once crossed 11 per cent. It may be pointed out that the
skewflation has gradually given way to a lower-grade generalised inflation. (with the
economy in the middle of 2011 inflating at around 9 per cent with food and non-food price
increases roughly at the same level).

79. Solution: a)
This is the ratio between GDP at Current Prices and GDP at Constant Prices. If GDP at Current
Prices is equal to the GDP at Constant Prices, GDP deflator will be 1, implying no change in
price level. If GDP deflator is found to be 2, it implies rise in price level by a factor of 2, and
if GDP deflator is found to be 4 , it implies a rise in price level by a factor of 4. GDP deflator
is acclaimed as a better measure of price behaviour because it covers all goods and services
produced in the country (because the weight of services has not been equitably accounted in
the Indian headline inflation i.e. inflation at the WPI).

80. Solution: d)
Inflation redistributes wealth from creditors to debtors i.e. lenders suffer and borrowers
benefit out of inflation. The opposite effect takes place when inflation falls (i.e. deflation).
http://www.insightsonindia.com

INSIGHTS

Page 32

Insights Mock Tests 2015: Solutions Test - 15


Holding money does not remain an intelligent economic decision (because money loses
value with every increase in inflation) that is why people visit banks more frequently and
try to hold least money with themselves and put maximum with the banks in their saving
accounts. This is also known as the shoe leather cost of inflation (as it consumes the precious
time of the people visiting the bank frequently tagging their shoe!). It means that saving rate
increases. But this happens as a short-term effect of inflation. In the long-run, higher
inflation depletes the saving rate in an economy. Just the opposite situation arises when
inflation falls or shows falling traits with decreasing saving, in the short-run and increasing
saving in the long-run, respectively.

81. Solution: b)
WPI categories. Services are included in CPI. Price rise in intermediate goods is not counted.
Instead, the final price rise in the manufactured good is included.

82. Solution: d)
The typical problem of bottleneck inflation (i.e structural inflation) arises out of shortfalls in
the supply of goods, a general crisis of a developing economy, rising demand but lack of
investible capital to produce the required level of goods.
Whenever the Government managed to go for higher growths by managing higher
investible capital it had inflationary pressures on the economy (seen during 1970s and 1980s,
specially) and growth was sacrificed at the altar of lower inflation (which was politically
more justified). Thus the supply-side mismatch remained a long-drawn problem in India for
higher inflation. After some time even if the government managed higher expenditure, most
of it was eaten by the non-developmental areas which did show low growth with higher
inflation signs of a stagnating economy.

83. Solution: a)
Major traits of recession, to a great extent, are similar to that of depression may be summed
up as follows

there is a general fall in demand as economic activities takes a downturn;

http://www.insightsonindia.com

INSIGHTS

Page 33

Insights Mock Tests 2015: Solutions Test - 15

inflation remains lower or/and shows further signs of falling down;


employment rate falls/unemployment rate grows;
Industries resort to price cuts to sustain their business.

The major economic traits of boom may be listed as given below:

an accelerated and prolonged increase in the demand;


demand peaks up to such a high level that it exceeds sustainable output/production
levels;
the economy heats up and a demand-supply lag is visible;
the market forces mismatch (i.e. demand and supply inequilibrium) and tend to
create a situation where inflation start going upward;

(e) the economy might face structural problems like shortage of investible capital, lower
savings, falling standard of living, creation of a sellers market.

84. Solution: a)
Agriculture is not only the biggest sector of the economy (employment wise) but also the
most free private sector, too. It is the only profession which still carries no burden of
individual income tax.
This is the biggest unorganised sector of the economy accounting for more than 90 per cent
share in the total unorganised labour-force (93 per cent of the total labour force of the
economy i.e. 39.7 crores, is employed in the unorganised sector).

85. Solution: c)
Under the broader step of Tenancy Reforms, three inter-related reforms protecting the land
tenants were effected by the GoI:

Regulation of rent so that a fixed and rational rate of rent could be paid by the sharecroppers to the land owners;
Security of tenure so that a share-cropper could be feel secure about his future income
and his economic security; and
Ownership rights to tenants so that the landless masses (i.e. the tenants, the sharecroppers) could be transferred the final rights for the land they plough - land to the
tillers.

86. Solution: c)
http://www.insightsonindia.com

INSIGHTS

Page 34

Insights Mock Tests 2015: Solutions Test - 15


Prevention of pig-to-human transmission
The transmission from swine to humans is believed to occur mainly in swine farms, where
farmers are in close contact with live pigs. Although strains of swine influenza are usually
not able to infect humans, this may occasionally happen, so farmers and veterinarians are
encouraged to use face masks when dealing with infected animals. The use of vaccines on
swine to prevent their infection is a major method of limiting swine-to-human transmission.
Prevention of human-to-human transmission
Influenza spreads between humans when infected people cough or sneeze, then other
people breathe in the virus or touch something with the virus on it and then touch their own
face, eyes, nose or mouth. Swine flu cannot be spread by pork products, since the virus is
not transmitted through food. The swine flu in humans is most contagious during the first
five days of the illness, although some people, most commonly children, can remain
contagious for up to ten days.
PIB Features
http://pib.nic.in/newsite/efeatures.aspx

87. Solution: b)
The Ministry of Home Affairs extends manpower and financial support, guidance and
expertise to the State Governments for the maintenance of security, peace and harmony
without trampling upon the constitutional rights of the States.
The Ministry of Home Affairs shows[4][5] the has the following constituent Departments:
Department of Border Management
Department of Border Management, dealing with management of borders, including coastal
borders.
Department of Internal Security
Department of Internal Security, dealing with police, law and order and rehabilitation.
Department of Jammu & Kashmir Affairs
Department of Jammu & Kashmir (J & K) Affairs, dealing with the constitutional provisions
in respect of the State of Jammu & Kashmir and all other matters relating to the State
excluding those with which the Ministry of External Affairs is concerned.
Department of Home
Dealing with the notification of assumption of office by the President and Vice President,
notification of appointment of the Prime Minister and other Ministers, etc
http://www.insightsonindia.com

INSIGHTS

Page 35

Insights Mock Tests 2015: Solutions Test - 15


Department of Official Language
Dealing with the implementation of the provisions of the Constitution relating to official
languages and the provisions of the Official Languages Act, 1963.
Department of States
Dealing with Centre-State relations, Inter-State relations, Union Territories and Freedom
Fighters' pension.

88. Solution: b)
Comptroller and Auditor-General of India :(1) There shall be a Comptroller and Auditor-General of India who shall be appointed by the
President by warrant under his hand and seal and shall only be removed from office in like
manner and on like grounds as a Judge of the Supreme Court.
(2) Every person appointed to be the Comptroller and Auditor-General of India shall, before
he enters upon his office, make and subscribe before the President or some person
appointed in that behalf by him, an oath or affirmation according to the form set out for the
purpose in the Third Schedule.
(3) The salary and other conditions of service of the Comptroller and Auditor-General shall
be such as may be determined by Parliament by law and, until they are so determined, shall
be as specified in the Second Schedule:
Provided that neither the salary of a Comptroller and Auditor-General nor his rights in
respect of leave of absence, pension or age of retirement shall be varied to his disadvantage
after his appointment.
(4) The Comptroller and Auditor-General shall not be eligible for further office either under
the Government of India or under the Government of any State after he has ceased to hold
his office.
(5) Subject to the provisions of this Constitution and of any law made by parliament, the
conditions of service of persons serving in the Indian Audit and Accounts Department and
the administrative powers of the Comptroller and Auditor-General shall be such as may be
prescribed by rules made by the President after consultation with the Comptroller and
Auditor-General.
(6) The administrative expenses of the office of the Comptroller and Auditor-General
including all salaries, allowances and pensions payable to or in respect of persons serving in
that office, shall be charged upon the Consolidated Fund of India.
149. Duties and Powers of the Comptroller and Auditor-General :- The Comptroller and
Auditor-General shall perform such duties and exercise such powers in relation to the
accounts of the Union and of the States and of any other authority or body as may be
http://www.insightsonindia.com

INSIGHTS

Page 36

Insights Mock Tests 2015: Solutions Test - 15


prescribed by or under any law made by Parliament and, until provision in that behalf is so
made, shall perform such duties and exercise such powers in relation to the accounts of the
Union and of the States as were conferred on or exercisable by the Auditor-General of India
immediately before the commencement of this Constitution in relation to the accounts of the
Dominion of India and of the provinces respectively.
150. Form of accounts of the Union and of the States :- The accounts of the Union and of the
States shall be kept in such form as the President may, on the advice of the Comptroller and
Auditor-General of India, prescribe.
151. Audit Reports :(1) The reports of the Comptroller and Auditor-General of India relating to the accounts of
the Union shall be submitted to the president, who shall cause them to be laid before each
House of Parliament.
(2) The reports of the Comptroller and Auditor-General of India relating to the accounts of a
State shall be submitted to the Governor of the State, who shall cause them to be laid before
the Legislature of the State.

89. Solution: b)
http://www.icrisat.org/what-we-do/agro-ecosystems/Bhoo-Chetana/background.htm

90. Solution: c)
The enactment of Administrative Tribunals Act in 1985 opened a new chapter in the sphere
of administering justice to the aggrieved government servants. Administrative Tribunals Act
owes its origin to Article 323-A of the Constitution which empowers Central Government to
set-up by an Act of Parliament, Administrative Tribunals for adjudication of disputes and
complaints with respect to recruitment and conditions of service of persons appointed to the
public service and posts in connection with the affairs of the Union and the States. In
pursuance of the provisions contained in the Administrative Tribunals Act, 1985, the
Administrative Tribunals set-up under it exercise original jurisdiction in respect of service
matters of employees covered by it. As a result of the judgement dated 18 March 1997 of the
Supreme Court, the appeals against the orders of an Administrative Tribunal shall lie before
the Division Bench of the concerned High Court.
The Administrative Tribunals exercise jurisdiction only in relation to the service matters of
the litigants covered by the Act. The procedural simplicity of the Act can be appreciated
from the fact that the aggrieved person can also appear before it personally. The
Government can present its case through its departmental officers or legal practitioners.
Thus, the objective of the Tribunal is to provide for speedy and inexpensive justice to the
litigants.

http://www.insightsonindia.com

INSIGHTS

Page 37

Insights Mock Tests 2015: Solutions Test - 15


The Act provides for establishment of Central Administrative Tribunal (CAT) and the State
Administrative Tribunals. The CAT was set-up on 1 November 1985. Today, it has 17
regular benches, 15 of which operate at the principal seats of High Courts and the remaining
two at Jaipur and Lucknow. These Benches also hold circuit sittings at other seats of High
Courts. In brief, the tribunal consists of a Chairman, Vice-Chairman and Members. The
Members are drawn, both from judicial as well as administrative streams so as to give the
Tribunal the benefit of expertise both in legal and administrative spheres.

91. Solution: b)

92. Solution: b)
The electoral roll is a list of all people in the constituency who are registered to vote in
Indian Elections. Only those people with their names on the electoral roll are allowed to
vote. The electoral roll is normally revised every year to add the names of those who are to
turn 18 on the 1st January of that year or have moved into a constituency and to remove the
names of those who have died or moved out of a constituency. If you are eligible to vote and
are not on the electoral roll, you can apply to the Electoral Registration Officer of the
constituency, who will update the register. The updating of the Electoral Roll only stops
during an election campaign, after the nominations for candidates have closed.

93. Solution: a)
The greatest advantage of the parliamentary system is that it ensures harmonious
relationship and cooperation between the legislative and executive organs of the
government. The executive is a part of the legislature and both are interdependent at work.
As a result, there is less scope for disputes and conflicts between the two organs.
By its very nature, the parliamentary system establishes a responsible government. The
ministers are responsible to the Parliament for all their acts of omission and commission.
The Parliament exercises control over the ministers through various devices like question
hour, discussions, adjournment motion, no confidence motion, etc.

94. Solution: b)
The parliamentary system of government in India is largely based on the British
parliamentary system.
However, it never became a replica of the British system and differs in the following
respects:

http://www.insightsonindia.com

INSIGHTS

Page 38

Insights Mock Tests 2015: Solutions Test - 15

India has a republican system in place of British monarchical system. In other words,
the Head of the State in India (that is, President) is elected, while the Head of the
State in Britain (that is, King or Queen) enjoys a hereditary position.
The British system is based on the doctrine of the sovereignty of Parliament, while
the Parliament is not supreme in India and enjoys limited and restricted powers due
to a written Constitution, federal system, judicial review and fundamental rights.
In Britain, the prime minister should be a member of the Lower House (House of
Commons) of the Parliament. In India, the prime minister may be a member of any
of the two Houses of Parliament.
Usually, the members of Parliament alone are appointed as ministers in Britain. In
India, a person who is not a member of Parliament can also be appointed as minister,
but for a maximum period of six months.

95. Solution: b)

96. Solution: b)
The objective of INO is to conduct basic research on the elementary particle called neutrino.
Presently 21 research institutes, Universities and IITs from all over the country are involved
in this project. INO is expected to galvanise interest in basic science research in the whole
country and particularly in and around Theni and Madurai districts of Tamil Nadu. Science
students across the country will have opportunity to pursue cutting edge research in the
field of particle physics while being located in India.
The observatory will be located underground so as to provide adequate shielding to the
neutrino detector from cosmic background radiation. Tunnel construction is very common
and will not have any impact on environment, water sources or dams in the region.

http://www.insightsonindia.com

INSIGHTS

Page 39

Insights Mock Tests 2015: Solutions Test - 15


97. Solution: d)
A Soil Health Card is used to assess the current status of soil health and, when used over
time, to determine changes in soil health that are affected by land management. A Soil
Health Card displays soil health indicators and associated descriptive terms. The indicators
are typically based on farmers' practical experience and knowledge of local natural
resources. The card lists soil health indicators that can be assessed without the aid of
technical or laboratory equipment.
Though quite a few states including Tamil Nadu, Gujarat, Andhra Pradesh, and Haryana
are successfully distributing such cards, Centre plans to make it a pan India effort.
According to the data, up to March 2012 over 48 crore soil health cards have been issued to
farmers to make them aware about nutrient deficiencies in their fields. Tamil Nadu has
started issuing soil health cards from the year 2006 onwards. There are 30 Soil Testing
Laboratories (STLs) and 18 Mobile Soil Testing Laboratories functioning in the State. The
Laboratory at Kudumianmalai, Pudukottai District has been declared as central laboratory
and it monitors quality of analysis in all laboratories.

98. Solution: c)
Key facts of report

Losses- Annually the cost of environmental damage associated with ecosystem


degradation and biodiversity loss is about Rs. 670 crore. While, the cost of health
effects due to poor environment is estimated at Rs. 620 crore.

Factors- The losses are due to combination of factors associated with unsustainable
and inefficient economic activities in Sundarbans.

These factors include mangrove destruction, impact of cyclones, reduced agricultural


yields and unsustainable fisheries as well as destruction of ecosystem services.

Cyclones- Damage costs from cyclones are the highest and accounts for damages
worth Rs. 290 crore. It also includes damages to houses, agriculture, human injuries
and fatalities.

Health issues- Due to environmental degradation, villagers in Sundarbans are


suffering from poor health outcomes. This degradation is generally in the form of
adverse natural events, such as cyclones and storms and increases in soil salinity.

These risk factors contribute considerably to mortality and morbidity, particularly


among women and children.

99. Solution: c)

http://www.insightsonindia.com

INSIGHTS

Page 40

Insights Mock Tests 2015: Solutions Test - 15


MADAD an e-portal will help Indian citizens living abroad to file consular grievances online
on the services offered by the Indian Missions and Posts abroad.

It incorporates several innovative features including flexible architecture to handle a


variety of grievances, linking of similar grievances on the basis of passport number
to avoid duplication and automatic escalation and enhancement of priority.

It will also help to speed up forwarding and handling of complaints, improve


tracking and redressal and escalate unresolved cases.

100.

Solution: a)

There is no body like National Skill Commission.


The Khadi and Village Industries Commission (KVIC) has recently joined hands with Rural
Self Employment Training Institute (RSETI), to provide vocational training to unemployed
youth.
In this regard memorandum of understanding (MoU) was signed between the two bodies in
the presence of Micro, Small and Medium Enterprises Minster Kalraj Mishra. Along with
vocational training MoU also aims at providing loan linkages to deserving beneficiaries
linked to the Prime Ministers Employment Guarantee Programme (PMEGP).

http://www.insightsonindia.com

INSIGHTS

Page 41

INSIGHTS ON INDIA MOCK PRELIMINARY EXAM - 2015


INSIGHTS ON INDIA MOCK TEST - 16
GENERAL STUDIES

PAPER-I
Time Allowed: 2 Hours

Maximum Marks: 200

INSTRUCTIONS
1. IMMEDITELY AFTER THE COMMENCEMENT OF THE EXAMINATION, YOU SHOULD
CHECK THAT THIS TEST BOOKLET DOES NOT HAVE ANY UNPRINTED OR TORN OR MISSING
PAGES OR ITEMS, ETC. IF SO, GET IT REPLACED BY A COMPLETE TEST BOOKLET.
2. You have to enter your Roll Number on the Test
Booklet in the Box provided alongside. DO NOT
Write anything else on the Test Booklet.
4. This Test Booklet contains 100 items (questions). Each item is printed only in English. Each item
comprises four responses (answers). You will select the response which you want to mark on the
Answer Sheet. In case you feel that there is more than one correct response, mark the response which
you consider the best. In any case, choose ONLY ONE response for each item.
5. You have to mark all your responses ONLY on the separate Answer Sheet provided. See directions in
the Answer Sheet.
6. All items carry equal marks.
7. Before you proceed to mark in the Answer Sheet the response to various items in the Test Booklet, you
have to fill in some particulars in the Answer Sheet as per instructions sent to you with your
Admission Certificate.
8. After you have completed filling in all your responses on the Answer Sheet and the examination has
concluded, you should hand over to the Invigilator only the Answer Sheet. You are permitted to take
away with you the Test Booklet.
9. Sheets for rough work are appended in the Test Booklet at the end.
10. Penalty for wrong answers :
THERE WILL BE PENALTY FOR WRONG ANSWERS MARKED BY A CANDIDATE IN THE
OBJECTIVE TYPE QUESTION PAPERS.
(i) There are four alternatives for the answer to every question. For each question for which a
wrong answer has been given by the candidate, one-third of the marks assigned to that
question will be deducted as penalty.
(ii) If a candidate gives more than one answer, it will be treated as a wrong answer even if one of
the given answers happens to be correct and there will be same penalty as above to that
question.
(iii)

If a question is left blank, i.e., no answer is given by the candidate, there will be no penalty
for that question.
http://www.insightsonindia.com

INSIGHTS ON INDIA MOCK TEST SERIES FOR CIVIL SERVICES PRELIMINARY EXAM 2015
http://insightsonindia.com

INSIGHTS

Page 1

1. Consider the following statements about


the Indian economy immediately postindependence.
1. India decided to be a state led
economy.
2. All the industries were kept under the
compulsory licensing provision for
the private sector.
3. State governments did not have the
right to monopoly over any industry.
Choose the correct answer from the codes
below.
a)
b)
c)
d)

1 and 2
2 and 3
1 and 3
None of the above

2. To regulate monopolistic and unfair


practices in the market, the Competition
Act, 2002 had been introduced. It
regulates which of the following aspects
related to business firms in the market?
1. Merger and acquisition
2. Pricing
3. Investment in security markets
Choose the correct answer from the codes
below.
a)
b)
c)
d)

3. India was faced with a severe balance of


payment crisis in 1991. The factors
responsible for the crisis may be
1. Higher oil prices due to Gulf war
2. High Inflation
3. Unmanageable fiscal deficit
Choose the correct answer from the codes
below.
a)
b)
c)
d)

1 and 2
2 and 3
1 and 3
All of the above

4. Disinvestment is an important policy


thrust of the government in market
friendly reforms. The proceeds of
Disinvestment are used by government
for which of the following?
1. For Infrastructural development
2. For Social sector schemes
3. To privatize other PSUs
Choose the correct answer from the codes
below.
a)
b)
c)
d)

1 and 2
2 and 3
1 and 3
All of the above

1 and 2
2 and 3
1 and 3
All of the above

http://insightsonindia.com

INSIGHTS

Page 2

5. Token disinvestment in PSUs means that


a) Government will not lose ownership
and disinvest only a small share in the
PSU
b) Government will not lose ownership
but will disinvest till 26% stake for it
remains
c) Government will lose ownership and
will disinvest a majority of its stake in
the PSU
d) None of the above

6. As per the present Disinvestment policy


in PSUs
1. Citizens can also own shares of PSUs.
2. Government must retain majority
shareholding in the PSUs being
disinvested.
3. Profitable PSUs cannot be disinvested.
Choose the correct answer from the codes
below.
a)
b)
c)
d)

1 and 2
2 and 3
1 and 3
All of the above

7. If the Gross capital formation is


increasing in India, this means that
a) India is creating more and more
infrastructure
b) Indias financial markets are receiving
more and more investment
c) Banks in India are witnessing
gradually decreasing Non-performing
assets
d) Non-plan expenditure is increasing in
successive annual budgets
http://insightsonindia.com

8. Foreign Direct Investment (FDI) is NOT


allowed in which of the following sectors
in India?
1. E-Commerce
2. Housing and Real Estate Business
3. Coffee Plantations
Choose the correct answer from the codes
below.
a)
b)
c)
d)

1 and 2
2 and 3
1 and 3
All of the above

9. FDI in automatic route does NOT require


approval from the
1. Concerned State Government
2. RBI
3. Foreign Investment Promotion Board
(FIPB)
Choose the correct answer from the codes
below.
a)
b)
c)
d)

1 and 2
2 and 3
1 and 3
All of the above

10. FDI in select sectors beyond a certain


percentage of stake has to approved via
the Government Route. Which of the
following stakeholders may be involved
in the approval?
1. Ministry of Finance
2. Ministry of Commerce

INSIGHTS

Page 3

3. Foreign Investment Promotion Board


(FIPB)
4. RBI
Choose the correct answer from the codes
below.
a)
b)
c)
d)

1, 2 and 4 only
2 and 3 only
1, 3 and 4 only
All of the above

d) Manchester.

14. What was the name of the committee


which was set up to fight for the rights of
the passengers in Komaghata maru ship
in Vancouver?
a) Shore Committee.
b) Justice Committee.
c) Liberal Committee.

11. Who started the newspaper Free


Hindustan in North America?
a) Tarak Nath Das.
b) Lal Hardayal.
c) Lal Lajpar Rai.
d) Kudiram Bose.

12. Who set up a Swadesh Sevak Home in


Vancouver on the lines of the India House
in London?
a) Tarak Nath Das.
b) Lal Har Dayal.
c) Lal Lajpar Rai.
d) G.D. Kumar.

13. Yugantar Ashram was set up in


a) Punjab.
b) San Francisco.
c) London.
http://insightsonindia.com

d) Ghadar Committee

15. Consider the following statements:


1. Ghadar movement was a democratic
and egalitarian movement.
2. It was clearly stated by the Ghadarites
that their objective was the
establishment of an independent
republic of India.
Which of the above statements is/are true?
a) Only 1.
b) Only 2.
c) Both.
d) None

16. Consider the following statements:


1. The Ghadar Movement failed to
generate an effective and sustained
leadership that was capable of
integrating the various aspects of the
movement.
INSIGHTS

Page 4

2. The Ghadar movement also lacked


effective organization.
Which of the above statements is/are true?
a) Only 1.
b) Only 2.
c) Both.
d) None.

17. Delayed investments in last few years


resulted into an alarmingly high rate of
increase in stalled projects reaching upto
8.8 lakh crores. As per the latest Economic
Survey, the factor that is responsible for
delay in a large number of projects is:
a)
b)
c)
d)

Lack of speedy regulatory clearance


Unfavourable market Conditions
Low environmental clearances
Land clearances

18. The latest Economic Survey points out


that price subsidies are regressive. It
means that
a) A rich household benefits more from
the subsidy than a poor household
b) There is huge leakage in the
distribution of subsidies
c) Corporates are enjoying a much larger
share of subsidies than the poor
households
d) High price subsidies is pulling down
economic growth in India

19. The Eurozone crisis is one of the major


reasons of the global economic slowdown.
Which of the following factors made the
crisis difficult to resolve?
1. The eurozone lacks a single fiscal
authority capable of strict
enforcement
2. Economies with different levels of
competitiveness (and fiscal positions)
have a single currency.
3. There is no lender of last resort like a
Central bank.
Choose the correct answer from the codes
below.
a)
b)
c)
d)

1 and 2 only
2 and 3 only
1 and 3 only
All of the above

20. As per the World Bank the East Asian


Miracle Economies are the ones whose
economic growth was
a) export-led
b) state-led
c) Demographic dividend led
d) Services led

21. Indias energy dependence on imported


energy sources is in which of the
following?
1. Coal
2. Nuclear Fuel
3. Biofuels
4. Natural Gas
Choose the correct answer using the codes
below.

http://insightsonindia.com

INSIGHTS

Page 5

a)
b)
c)
d)

All of the above


1, 2 and 4 only
1 and 2 only
None of A, B or C options
combination is correct.

22. Which of the following does NOT form


part of the Indian money market?
1. Treasury bills
2. Mutual Funds
3. Share market
Choose the correct answer from the codes
below.
a)
b)
c)
d)

1 and 2 only
2 and 3 only
1 and 3 only
3 only

23. Which of the following bodies come


under the category of financial regulators?
1. Insurance Regulatory and
Development Authority (IRDA)
2. Forward Markets Commission (FMC)
3. NABARD
4. SIDBI
Choose the correct answer from the codes
below.
a)
b)
c)
d)

1, 2 and 4 only
2 and 3 only
1, 3 and 4 only
All of the above

24. Money market is that segment of financial


markets that
http://insightsonindia.com

a) satisfies the short-term capital needs


of corporations
b) satisfies the long-term capital needs of
corporations
c) provides short-term loans on
discounted interest rate to the
corporations
d) provides long-term loans on
discounted interest rate to the
corporations

25. The provision of repos and the reverse


repos help in
1. Getting banks the required amount of
funds
2. Allowing banks to park idle surplus
funds
3. Controlling inflation in the economy
Choose the correct answer from the codes
below.
a)
b)
c)
d)

1 and 2 only
2 and 3 only
1 and 3 only
All of the above

26. The call money market is an important


segment of the money market where
uncollateralized borrowing and lending of
funds take place on overnight basis.
Which of the following are NOT players
in the call money market?
1. Commercial banks
2. Regional Rural Banks
3. Cooperative banks
Choose the correct answer from the codes
below.
INSIGHTS

Page 6

a)
b)
c)
d)

2 only
2 and 3 only
1 and 3 only
All are players in call money market.

27. Open Market Operations (OMOs) are the


purchase and sale of securities between
a) RBI and the Government
b) Government and the Market
c) RBI and the market
d) RBI and Foreign Exchange Markets

28. The effectiveness of the Open Market


Operations (OMOs) is constrained by
which of the following factors
1. Fiscal deficit of the Government
2. Amount of Government Securities in
the market
3. Bank rate
Choose the correct answer from the codes
below.
a)
b)
c)
d)

2 only
2 and 3 only
1 and 2 only
All of the above

29. Liquidity Adjustment facility (LAF) is


a) The difference between repo and
reverse repo rate
b) The margin upto which banks can
reduce their Statutory Liquidity
requirements (SLR)
c) The difference between bank rate and
repo rate

http://insightsonindia.com

d) A special facility granted by RBI to


banks to cut down on Cash Reverse
Ratio (CRR) obligations

30. Under the Marginal Standing Facility


(MSL), commercial banks are allowed to
a) Park funds with RBI at a slightly
higher rate than official reverse repo
rates
b) Borrow funds overnight from RBI at a
slightly higher rate than repo rate
c) Cut down their obligations of SLR
upto a certain limit
d) None of the above

31. Consider the following statements about


Exchange-Traded Funds (ETFs).
1. They are traded like stocks.
2. They contain a basket of commodities
and assets.
3. ETFs are not allowed in India.
Choose the correct answer from the codes
below.
a)
b)
c)
d)

1 and 2 only
2 and 3 only
1 and 2 only
All of the above

32. A non-banking financial company (NBFC)


can NOT
1. Lend money to people
2. Have principal business as industrial
activity
3. Acquire stocks, bonds or securities
issued by Government

INSIGHTS

Page 7

Choose the correct answer from the codes


below.
a)
b)
c)
d)

1 and 2 only
2 and 3 only
2 only
3 only

33. Which of the following functions/roles


are performed by the RBI?
1. Agent of the Government of India in
the IMF
2. Developmental and promotional
functions
3. Keeper of the foreign currency
reserves
Choose the correct answer from the codes
below.
a)
b)
c)
d)

1 and 2 only
2 and 3 only
1 and 3 only
All are performed by RBI.

34. Consider the following statements about


Cash Reserve Ratio (CRR).
1. It is decided by the Government of
India.
2. It is necessary to consult the Boards of
all Public sector banks before deciding
CRR.
3. Increase in CRR increases the money
supply in the economy.
Choose the correct answer from the codes
below.
a) 1 and 2 only
b) 2 and 3 only
http://insightsonindia.com

c) 3 only
d) None of the above

35. Statutory Liquidity Ratio (SLR) can be


maintained by banks in the form of
1. Gold
2. Government Securities
3. Cash
Choose the correct answer from the codes
below.
a)
b)
c)
d)

1 and 2 only
2 and 3 only
1 and 3 only
All of the above

36. The interest rate which the RBI charges on


its long-term lendings is known as the
Bank Rate. The clients who borrow
through this route can be
1. Union Government
2. State Governments
3. NBFCs
4. Commercial banks
Choose the correct answer from the codes
below.
a)
b)
c)
d)

1 and 4 only
2 and 3 only
1, 2 and 4 only
All of the above

37. Inflation can be countered by increasing


which of the following?
1. CRR
2. Reverse Repo rate
3. SLR
INSIGHTS

Page 8

4. Bank rate
Choose the correct answer from the codes
below.
a)
b)
c)
d)

1 and 4 only
2 and 3 only
1, 2 and 3 only
All of the above

38. Which of the following Union Territories


participate in the election of the
President?
1. Delhi
2. Puducherry
3. Chandigarh
Choose the correct answer from the codes
below.
a)
b)
c)
d)

1 and 2 only
2 and 3 only
1 and 3 only
None of the above

39. The nominated members of both Lok


Sabha and Rajya Sabha do NOT
participate in which of the following?
1. Election of President
2. Passing of Constitutional Amendment
Bill
3. Impeachment of President
4. Election of Vice-President
Choose the correct answer from the codes
below.
a)
b)
c)
d)

1 and 4 only
2 and 3 only
1 and 3 only
1 only

http://insightsonindia.com

40. Consider the following statements


regarding the election of President.
1. The value of vote of a MLA is
equivalent to that of a MLC casting
vote in Presidential election.
2. The value of vote of a MLA is
equivalent to that of a MP casting vote
in Presidential election.
Which of the above is/are true?
a)
b)
c)
d)

1 only
2 only
Both 1 and 2
None

41. During the tenure of the Indian President,


he is immune from which of the
following?
1. Criminal proceedings
2. Civil Proceedings
3. Arrest and Imprisonment
Choose the correct answer from the codes
below.
a)
b)
c)
d)

1 and 2 only
2 and 3 only
1 and 3 only
All of the above

42. Consider the following statements about


the Impeachment of the Indian President.
1. Impeachment can only be initiated by
Lok Sabha.
2. Supreme Court is the authority that
investigates against the charges
levelled against the President.
INSIGHTS

Page 9

3. Representatives of Union Territories


in the Parliament do not participate in
the impeachment process.
Choose the correct answer from the codes
below.
a)
b)
c)
d)

2 only
2 and 3 only
1 and 3 only
None of the above

43. Which of the following appointments are


made by the President?
1. Attorney General of India
2. Chairman of Finance Commission
3. Comptroller and Auditor General of
India
Choose the correct answer from the codes
below.
a)
b)
c)
d)

1 and 2 only
2 and 3 only
1 and 3 only
All of the above

44. The President is an integral part of the


Parliament of India. Which of the
following are done by the President, with
reference to the sessions of the
Parliament?
1. Summoning
2. Proroguing
3. Dissolving Lok Sabha

c) 1 and 3 only
d) All of the above

45. Consider a case where the Parliament has


passed a legislation sent to the President
and the President returns it to the
Parliament for reconsideration. For the bill
to get passed now which of these should
happen?
a) Parliament should pass the bill again
but this time by a higher majority
b) Parliament should pass the bill again
by an ordinary majority
c) A joint sitting should be convened to
get the bill passed
d) Parliament should wait for 6 months
without taking any action on the
returned bill, after which it will be
deemed passed.

46. Which of the following bills can NOT be


returned for reconsideration or withheld
by the President?
1. Money Bills
2. Constitutional Amendment Bills
3. Private member bills
Choose the correct answer from the codes
below.
a)
b)
c)
d)

1 and 2 only
2 and 3 only
1 and 3 only
All of the above

Choose the correct answer from the codes


below.
a) 1 and 2 only
b) 2 and 3 only
http://insightsonindia.com

INSIGHTS

Page 10

47. Consider the following statements:


1. The Extremists wanted to extend the
Swadeshi Movement from Bengal to
the rest of the country.
2. The Extremists also wanted to
gradually extend the boycott from
foreign goods to every form of
association or cooperation with the
colonial Government.
3. The Moderates wanted to confine the
boycott part of the movement to
Bengal and were totally opposed to its
extension to the Government.
Which of the above statements is/are true?
a) Only 1 & 3.
b) Only 1.
c) Only 2 & 3.
d) All.

c) Only 2 & 3.
d) All.

49. Consider the following statements


regarding the Indian Councils Act of 1909:
1. The Indian Councils Act of 1909
increased the number of elected
members in both the imperial
Legislative Council and the provincial
legislative councils.
2. According to the Act said that, an
Indian was to be appointed a member
of the Governor-Generals Executive
Council.
3. The Act allowed to Vote on separate
budget items.
Which of the above statements is/are true?
a) Only 1 & 3.
b) Only 1.

48. Which of the following resolutions were


passed at the 1906 Calcutta session of the
Indian National Congress?

c) Only 2 & 3.
d) All.

1. Swadeshi.
2. Boycott.
3. National Education.
4. Self-Government.
Choose the correct answer using the codes
below:

50. Who organized Abhinav Bharat?


a) V.D. Savarkar.
b) Rash Behari Bose.
c) Subhas Chandra Bose.
d) B.G. Tilak.

a) Only 1 & 4.
b) Only 2, 3 & 4.
http://insightsonindia.com

INSIGHTS

Page 11

51. Consider the following statements.


1. An ordinance made when both the
Houses are in session is void.
2. Decision of the President to issue an
ordinance can be questioned in a
court.
Which of the above is/are true?
a)
b)
c)
d)

1 only
2 only
Both 1 and 2
None

52. Consider the following statements about


the pardoning power of the President.
1. He can exercise this power even
without the advice of the cabinet.
2. The President is not bound to give
reasons for his order.
3. Final decision of the President need
not rely on the evidences of the case.
Choose the correct answer from the codes
below.
a)
b)
c)
d)

1 and 2 only
2 and 3 only
1 and 3 only
All of the above

53. Though the President has no


constitutional discretion, he has some
situational discretion in which of the
following cases?
1. Appointment of Prime Minister when
no party has a clear majority in the
Lok Sabha.

http://insightsonindia.com

2. Dissolution of the Lok Sabha if the


council of ministers has lost its
majority.
3. Imposing Presidents rule in States in
case of constitutional breakdown
Choose the correct answer from the codes
below.
a)
b)
c)
d)

1 and 2 only
2 and 3 only
1 and 3 only
All of the above

54. Who among the following will not be


qualified for being a candidate for the
post of Vice-President?
1. Governor of any state
2. Union Cabinet Ministers
3. State cabinet Ministers
Choose the correct answer from the codes
below.
a)
b)
c)
d)

1 and 2 only
2 and 3 only
1 and 3 only
All of the above

55. Consider the following statements.


1. The Constitution does not contain any
specific procedure for the selection
and appointment of the Prime
Minister.
2. A person must prove his majority in
the Lok Sabha before he is appointed
as the Prime Minister.
3. The Prime Minister may be a member
of any of the two Houses of
parliament.
INSIGHTS

Page 12

Choose the correct answer from the codes


below.
a)
b)
c)
d)

1 and 2 only
2 and 3 only
1 and 3 only
All of the above

45. Consider the following statements about


the powers of the Prime Minister (PM).
1. The President can appoint only those
persons as ministers who are
recommended by the Prime Minister.
2. The PM can bring about the collapse
of the council of ministers by
resigning from office.
3. The council of Ministers cannot
function when the PM resigns.
Choose the correct answer from the codes
below.
a)
b)
c)
d)

1 and 2 only
2 and 3 only
1 and 3 only
All of the above

56. Consider the following statements.


1. The advice tendered by Ministers to
the President shall not be inquired
into by any court.
2. After the dissolution of the Lok Sabha,
the council of ministers does not cease
to hold office with immediate effect.
3. A minister who is a member of one
House of Parliament has the right to
speak and to take part in the
proceedings of the other House also.

http://insightsonindia.com

Choose the correct answer from the codes


below.
a)
b)
c)
d)

1 and 2 only
2 and 3 only
1 and 3 only
All of the above

57. Who among the following can be invited


to the meetings of the Cabinet?
1. Cabinet Secretary
2. Union Ministers of State
3. Deputy Ministers
Choose the correct answer from the codes
below.
a)
b)
c)
d)

1 and 2 only
2 and 3 only
1 and 3 only
All of the above

58. Which of the following is/are NOT


mentioned in the Constitution of India?
1. Powers and functions of the Union
Cabinet
2. Powers and functions of the State
Cabinets
3. Cabinet Committees
Choose the correct answer from the codes
below.
a)
b)
c)
d)

INSIGHTS

1 and 2 only
2 and 3 only
1 and 3 only
All of the above

Page 13

59. Consider the following statements about


Cabinet Committees.
1. They are setup by the President.
2. Non-cabinet ministers can not become
members of these committees.
3. All cabinet committees are headed by
the Home Minister.
Choose the correct answer from the codes
below.
a)
b)
c)
d)

1 and 2 only
2 only
1 and 3 only
None of the above

60. Who among the following can review the


decisions taken by the Cabinet
Committees?
1. Prime Minister
2. Cabinet
3. Parliament
4. Council of Ministers
Choose the correct answer from the codes
below.
a)
b)
c)
d)

1 and 2 only
2 and 4 only
2 only
1 and 3 only

61. The President of India is an integral part


of the Parliament. One of the reasons for it
is that
a) He has a constitutional right to attend
the meetings of the Parliament.
b) A bill passed by both the Houses of
Parliament cannot become law
without the Presidents assent.
http://insightsonindia.com

c) He has the constitutional authority to


both constitute and dissolve the
Parliament.
d) He is the head of the executive branch
of the Government in our
Parliamentary democracy.

62. Consider the following statements about


the representation in Rajya Sabha.
1. All the Union Territories (UTs) are
represented in Rajya Sabha.
2. Members in Rajya Sabha are
nominated by the president only on
the advice of the Council of Ministers.
3. Nominated members in Rajya Sabha
are more than those representing the
UTs.
Choose the correct answer using the codes
below.
a)
b)
c)
d)

1 and 2 only
2 and 3 only
1 and 3 only
All of the above

63. Consider the following statements.


Assertion (A): The simple majority system of
territorial representation does not represent
the whole electorate.
Reasons (R): It does not secure due
representation to minorities or other small
groups.
In the context of the statements above, which
of these is true?
a) A and R both are true, and R is the
correct explanation for A.
INSIGHTS

Page 14

b) A and R both are true, and R is the


NOT the correct explanation for A.
c) A is correct, R is incorrect.
d) A and R both are incorrect.

64. Out of the several kinds of proportional


representation system, which of the
following has been adopted in India?
1. List System
2. Single Transferrable Vote system
3. Mixed member proportional
representation
Choose the correct answer using the codes
below.
a)
b)
c)
d)

1 and 2 only
2 and 3 only
1 and 3 only
1 only

65. On the question whether a member is


subject to any of the disqualifications
mentioned under the Representation of
People Act (1951), whose decision is final?
a) President
b) Supreme Court
c) Election Commission of India
d) Lok Sabha Speaker

66. In the oath or affirmation by every


Member of Parliament (MP), s/he swears
to
1. Uphold the sovereignty and integrity
of India
2. Do right to all manner of people in
accordance with the Constitution and

http://insightsonindia.com

the law, without fear or favour,


affection or ill will.
3. Preserve, protect and defend the
Constitution and the law
Choose the correct answer using the codes
below.
a)
b)
c)
d)

1 and 2 only
2 and 3 only
1 only
All of the above

67. Consider the following statements about


the Lok Sabha speaker.
1. The speaker has to be elected from
amongst the Lok Sabha members.
2. When Lok Sabha is dissolved, the
speaker does not immediately cease to
hold office.
3. His decision in all Parliamentary
matters is final.
Choose the correct answer using the codes
below.
a)
b)
c)
d)

1 and 2 only
2 and 3 only
1 only
All of the above

68. The Speaker of the Lok Sabha derives his


powers and duties from
1. The Constitution of India
2. Parliamentary Conventions
3. Parliamentary laws
4. Rules of Procedure of Lok Sabha
Choose the correct answer using the codes
below.
INSIGHTS

Page 15

a)
b)
c)
d)

1, 2 and 4 only
2 and 3 only
1 and 4 only
All of the above

69. The decision of the Lok Sabha speaker is


considered in which of the following
cases?
1. Deciding whether a bill is money bill
or not
2. Disqualifying MPs under antidefection law
3. Deciding on calling a joint meeting of
both the Houses
Choose the correct answer using the codes
below.
a)
b)
c)
d)

1 and 2 only
2 and 3 only
1 and 3 only
All of the above

70. To impeach the Lok Sabha speaker


a) Both houses need to pass a resolution
to that effect by an absolute majority
b) Both houses need to pass a resolution
to that effect by an ordinary majority
c) Only Lok Sabha needs to pass a
resolution to that effect by an absolute
majority
d) Only Lok Sabha needs to pass a
resolution to that effect by an ordinary
majority

http://insightsonindia.com

71. Lok Sabha has recently passed Mines and


Minerals (Development and Regulation)
Amendment Bill, 2015. It covers which of
the following minerals?
1. Limestone
2. Bauxite
3. Manganese
4. Iron
Choose the correct answer using the codes
below.
a)
b)
c)
d)

1 and 4 only
2 and 3 only
1 and 3 only
All of the above

72. Election Commission of India (ECI) has


launched National Electoral Roll
Purification and Authentication
Programme (NERPAP). Under the
programme
a) Voter ID card of voters will be linked
with Aadhar data.
b) Voter ID card of voters will be reissued after redundancy check and
verification.
c) Voter ID card of voters will be stored
online and new cards will be allotted
online.
d) None of the above

73. Worlds first electric satellites were


successfully lifted off by a Space
Exploration (SpaceX) Technologies rocket.
What differences does an electric satellite
have with respect to conventional
satellites?

INSIGHTS

Page 16

1. It does not use a chemical propulsion


system.
2. Electric propulsion from these
satellites consumes relatively less fuel.
3. These satellites are heavier than
conventional satellites.
Choose the correct answer using the codes
below.
a)
b)
c)
d)

1 and 2 only
2 and 3 only
1 and 3 only
All of the above

74. With regard to taxation, in the Budget of


2015-16 which of the following items has
remained unchanged?
1. Service Tax
2. Income Tax Slab
3. Wealth Tax
4. Corporate Tax
Choose the correct answer using the codes
below.
a)
b)
c)
d)

1 only
2 and 3 only
1 and 4 only
All of the above

75. MUDRA bank announced in the Budget


2015-16 will provide finance wit highest
priority for
a) First generation SC/ST entrepreneurs
b) Women entrepreneurs
c) Entrepreneurs in Self-Help Groups
d) Young graduate entrepreneurs aged
less than 35 years

http://insightsonindia.com

76. Which of the following initiatives have


been taken in this Budget 2015-16?
1. Creation of a Universal Social Security
system for all Indians
2. Forward Markets Commission (FMC)
will be merged with the Securities and
Exchange Board of India (SEBI)
3. As an alternative to purchasing gold,
Sovereign Gold Bond will be
launched
Choose the correct answer using the codes
below.
a)
b)
c)
d)

1 and 2 only
2 and 3 only
1 and 3 only
All of the above

77. The Union Cabinet recently gave its


approval for establishing the New
Development Bank (NDB). Apart from
Infrastructure, the NDB will provide
capital for investment in which of these
areas?
1. E-governance
2. Sustainable Development
3. Border Modernization
Choose the correct answer using the codes
below.
a)
b)
c)
d)

INSIGHTS

1 and 2 only
2 and 3 only
1 and 3 only
All of the above

Page 17

78. Union government has accepted


recommendations of the 14th Finance
Commission (FC) as per its agenda of
cooperative federalism. The major
recommendations are?
1. Increase tax devolution to States to
42% from 32% previously.
2. Special grants for strengthening
duly elected gram panchayats
3. Larger financial role for states in
the NITI Ayog
Choose the correct answer using the codes
below.
a)
b)
c)
d)

1 and 2 only
2 and 3 only
1 and 3 only
All of the above

79. Which of the following brake systems are


being used presently in the Indian
railways?
1. Disc brake system
2. Air brake system
3. Rubber brake system
Choose the correct answer using the codes
below.
a)
b)
c)
d)

1 and 2 only
2 and 3 only
1 and 3 only
All of the above

b) Energy Sector
c) Mining Sector
d) Transportation sector

81. Which of the following papers were


launched by Annie Besant?
1. New India.
2. Commonwealth.
3. Yugaantar.
Choose the correct answer using the codes
below:
a)
b)
c)
d)

Only 1.
Only 1 & 2.
All.
None.

82. Consider the following statements


regarding Annie Besant:
1. Annie Besant founded the
Theosophical Society in London.
2. In 1898 she established the Central
Hindu College at Benares.
3. She was made the President of the
1917 Calcutta Session of the Indian
National Congress.
Which of the above statements is/are true?
a) Only 1.

80. In India, which of the following accounts


for the highest share of greenhouse
emissions as per recently released data by
the World Resource Institute?
a) Agriculture and Livestock
http://insightsonindia.com

b) Only 2 & 3.
c) Only 3.
d) All.

INSIGHTS

Page 18

83. Consider the following statements:


1. Congress and the Muslim League
supported Annie Besants decision to
set up Home Rule Leagues.
2. She actively took part in Civil
Disobedience Movement.
Which of the above statements is/are true?
a) Only 1.
b) Only 2.
c) Both.

2. Karnataka
3. Central Provinces
4. Berar
Choose the correct answer using the codes
below:
a) Only 1.
b) Only 1, 2 & 3.
c) Only 3 & 4.
d) All.

d) None.

84. Who among the following have been the


members of Home Rule Leagues?
1. J.L. Nehru.
2. Madan Mohan Malaviya.
3. M.A. Jinnah.
4. Surendranath Banerjea.
Choose the correct answer using the codes
below:
a) Only 1.
b) Only 2 & 4.

86. To ensure an environment friendly Indian


railway, which of the following initiatives
have been taken in the latest Railway
Budget 2015-16?
1. Expanding bio-toilets
2. Roof top solar energy generation
plants
3. Waste-to-energy plants for waste
generated from stations
Choose the correct answer using the codes
below.
a)
b)
c)
d)

1 and 2 only
2 and 3 only
1 and 3 only
All of the above

c) Only 1 2 & 3.
d) All.

85. Tilaks Home Rule League operated in


which of the following areas?
1. Maharashtra (excluding Bombay city)
http://insightsonindia.com

87. Majority of the existing farm credit corpus


in India goes to the
a) Agro-based industries
b) Farmers
c) Land improvement techniques
d) Food subsidies

INSIGHTS

Page 19

3. GIS devices
88. In the recently launched Economic Survey
2014-15, which of the following have been
pointed out as signs of worry for the
Indian rural economy?
1. Sharp decline in rural wages
2. Declining rate of agricultural
growth
3. Low expenditure in rural Public
health Infrastructure
Choose the correct answer using the codes
below.
a)
b)
c)
d)

1 and 2 only
2 and 3 only
1 and 3 only
All of the above

89. Natural cooling cycles in the X and the


Y have offset global warming in the past
decade, say researchers. X and Y are
a) Arctic and Antarctic
b) Arctic and Pacific
c) Antarctic and Pacific
d) Siberian and Pacific

90. A low cost mobile weather station made


recently in Sri Lanka can capture and
transmit near real-time data on rainfall. It
raises alarm in the event of extreme
rainfall and other natural disasters, which
can help farmers, disaster management
officials and the scientists plan ahead.
Which of the following technologies have
been used in the device?
1. Atomic Clock
2. GPS Sensor
http://insightsonindia.com

Choose the correct answer using the codes


below.
a)
b)
c)
d)

1 and 2 only
2 and 3 only
1 and 3 only
All of the above

91. With reference to Rashtriya Krishi Vikas


Yojana (RKVY), consider the following
statements:
1. Under this scheme, many sub-schemes
are selected and implemented by state
governments
2. Vidharbha Intensive Irrigation
Development Programme is one of its
sub-schemes
3. A sub-scheme called Nutri Farm has
been launched under RKVY which
provides subsidized fertilizers to BPL
farmers
Which of the above statements is/are correct?
a)
b)
c)
d)

1 and 3 Only
1 and 2 Only
2 and 3 Only
All

92. With reference to the National Food


Security Mission (NFSM), consider the
following statements:
1. Coarse cereals crops and commercial
crops (such as sugarcane, jute, cotton)
are not included in the missions
objective
2. The missions aims to help India
produce additional 10 million tonnes
INSIGHTS

Page 20

of rice, 8 million tonnes of wheat and 4


million tonnes of pulses by the end of
12th Five Year Plan.
Which of the above statements is/are correct?
a)
b)
c)
d)

1 Only
Both
2 Only
None

93. With reference to the National Mission for


Sustainable Agriculture (NMSA), consider
the following statements
1. It is a sub-mission under Rashtriya
Krishi Vikas Yojana (RKVY)
2. Comprehensive soil health
management; efficient on-farm water
management and mainstreaming
rainfed technologies are its main focus
areas
Which of the above statements is/are correct?
a)
b)
c)
d)

1 Only
2 Only
Both
None

94. With reference to agriculture cooperation


in India, consider the following
statements:
1. Agriculture cooperatives provide
around 25% of agricultural credit in
the country
2. Agriculture cooperatives produce
nearly 50% of Indias sugar
3. Agriculture cooperatives produce
nearly 25% of Indias fertilizer

http://insightsonindia.com

Which of the above statements is/are correct?


a)
b)
c)
d)

1 and 2 Only
3 and 2 Only
2 Only
1 and 3 Only

95. With reference to mechanization of


agriculture in India, consider the
following statements
1. Though India lacks far behind world
average rates in many farm
mechanization areas, but in case of
tractor density per thousand hectares,
it leads the world
2. Mechanization of farm is considered as
essential input in modern agriculture
Which of the above statements is/are correct?
a)
b)
c)
d)

1 Only
2 Only
Both
None

96. The National School of Drama was


established by
a) Lalit Kala Academy
b) Sangeet Nataka Academy
c) Sahitya Academy
d) Ministry of Culture, Government of
India

97. With reference to Ananda


Coomaraswamy, consider the following
statements:
1. He is described as a person who was
largely responsible for introducing
ancient Indian art to the West
INSIGHTS

Page 21

2. He was a noted Kannada scholar


3. Sahitya Academy has instituted a
Anand Fellowship in his honour
Which of the above statements is/are correct?
a)
b)
c)
d)

1 and 2 Only
2 and 3 Only
1 and 3 Only
All

98. Which of the following cities one of the


seven the Zonal Cultural Councils
established by the government of India is
NOT located?
a) Patiala
b) Dimapur
c) Thanjavur
d) Khajuraho
99. With reference to Visa on Arrival facility,
consider the following statements:
1. Some of the countries included in this
scheme include nationals from Finland
and New Zealand too

http://insightsonindia.com

2. Visa on Arrival is also applicable to


Diplomatic/Official Passports
Which of the above statements is/are correct?
a)
b)
c)
d)

1 Only
2 Only
Both
None

100. Which of the following is the function


of National Statistical Commission ?
a) To identify the core statistics, which
are of national importance and are
critical to the development of the
economy
b) To constitute professional committees
or working groups to assist the
Commission on various technical
issues
c) To evolve national policies and
priorities relating to the statistical
system
d) All the above

INSIGHTS

Page 22

Insights Mock Tests 2015: Solutions Test - 16


1. Solution: d)
The first economic policy of the country 1948 - major highlights of the policy are
given below:

India will be a mixed economy.


Some of the important industries were put under the Central List such as coal,
power, railways, civil aviation, arms and ammunition, defence, etc.

Some other industries (usually of medium category) were put under a State
List such as paper, medicines, textiles, cycles, rickshaws, two-wheelers, etc.

Rest of the industries (not covered by either the central or the State Lists) were
left open for private sector investmentwith many of them having the
provision of compulsory licensing.

2. Solution: a)
http://www.cci.gov.in/index.php?option=com_content&task=view&id=12
http://en.wikipedia.org/wiki/Competition_Commission_of_India

3. Solution: d)
Basically, in 1990 and 1991, there were several inter-connected events which were
growing unfavourable for the Indian economy.

Due to the Gulf War (199091), the higher oil prices were fastly23 depleting
Indias foreign reserves.

Sharp decline in the private remittances from the overseas Indian workers in
the wake of the Gulf War, specially from the Gulf region.

Inflation peaking at nearly 17 per cent.


The gross fiscal deficit of the central government reaching 8.4 per cent of the
GDP.
By the month of June 1991, Indias foreign exchange had declined to just two
weeks of import coverage.

4. Solution: a)
http://www.insightsonindia.com

INSIGHTS

Page 1

Insights Mock Tests 2015: Solutions Test - 16


Disinvestment is a process of selling government equities in public sector
enterprises. Disinvestment in India is seen connected to three major inter-related
areas, namely

A tool of public sector reforms


A part of the economic reforms started mid-1991. It has to be done as a
complementary part of the de-reservation of industries.

Initially motivated by the need to raise resources for the budgetary


allocations.

The approach towards public sector reforms in India has been much more cautious
than that of the other developing countries. India did not follow the radical solution
to itunder which outright privatisation of commercially viable PSUs is done and of
the unviable ones is completely closed.

5. Solution: a)
Disinvestment started in India with a high political cautionin a symbolic way
known as the token disinvestment. The general policy was to sell the shares of the
PSUs maximum upto the 49 per cent (i.e. maintaining government ownership of the
companies). But in practice, shares were sold to the tune of 510 per cent only.
This phase of disinvestment though brought some extra funds to the government
(which were used to fill up the fiscal deficit considering the proceeds as the capital
receipts) it could not initiate any new element to the PSUs which could enhance
their efficiency.
It remained the major criticism of this type of disinvestment, and the experts around
the world started suggesting the Government to go for it in the way the ownership
could be transferred from the government to the private sector.

6. Solution: a)
The present disinvestment policy is based on the main ideology that:

Citizens have every right to own part of the shares of Public Sector
Undertakings

Public Sector Undertakings are the wealth of the Nation and this wealth
should rest in the hands of the people, and

While pursuing disinvestment, Government has to retain majority


shareholding, i.e. at least 51% and management control of the PSUs.

http://www.insightsonindia.com

INSIGHTS

Page 2

Insights Mock Tests 2015: Solutions Test - 16


The action plan for disinvestment in profit making government companies is:

Already listed profitable PUSs (not meeting mandatory shareholding of 10%)


are to be made compliant by Offer for Sale by Government or by the PSUs
through issue of fresh shares or a combination of both;

Unlisted PSUs with no accumulated losses and having earned net profit in
three preceding consecutive years are to be listed;

Follow-on public offers would be considered taking into consideration the


needs for capital investment of PSUs, on a case by case basis, and
Government could simultaneously or independently offer a portion of its
equity shareholding;

In all cases of disinvestment, the Government would retain at least 51%


equity and the management control.

7. Solution: a)
In simple words Gross Capital Formation is Investment. When people save, they
tend to invest. The percentage of the investment made each year out of the total GDP
is called ross Capital Formation.
So, Rate of Gross Capital Formation is arrived as follows: Rate of Capital Formation
= (Investments /GDP) X 100

The importance of the Gross Capital formation lies in the fact that this is that
part of GDP which helps in the growth of the GDP itself. This is a must for
achieving high rate of production, capital formation, changes in production
techniques and changing in the outlook of the people themselves.

To achieve, the Optimum rate of economic growth, the rate of capital


formation should be above 40%. In India, the gross capital formation for the
year of 2009-10 was 36.5% of the GDP. It was composed of 9.2% in Public
Sector and 24.9% in Private sector. The investment from the Household sector
was 11.7%. Investment from the corporate sector was 13.2%.

8. Solution: d)
FDI is prohibited under Government as well as Automatic Route for the following
sectors:
Retail Trading
Atomic Energy
http://www.insightsonindia.com

INSIGHTS

Page 3

Insights Mock Tests 2015: Solutions Test - 16


Lottery Business
Gambling and Betting
Housing and Real Estate business
Agriculture (excluding Floriculture, Horticulture, Development of Seeds,
Animal Husbandry, Pisciculture and Cultivation of Vegetables, Mushrooms
etc. under controlled conditions and services related to agro and allied
sectors).
Plantations (Other than Tea plantations).

9. Solution: d)
FDI up to 100% is allowed under the automatic route in all activities/sectors except
the following which require prior approval of the Government:
Activities/items that require an Industrial License;
Proposals in which the foreign collaborator has an existing financial /
technical collaboration in India in the 'same' field,
Proposals for acquisition of shares in an existing Indian company in: Financial
services sector and where Securities & Exchange Board of India (Substantial
Acquisition of Shares and Takeovers ) Regulations, 1997 is attracted;
All proposals falling outside notified sectoral policy/caps or under sectors in
which FDI is not permitted.
FDI in sectors/activities to the extent permitted under automatic route does not
require any prior approval either by the Government or RBI. The investors are only
required to notify the Regional office concerned of RBI within 30 days of receipt of
inward remittances and file the required documents with that office within 30 days
of issue of shares to foreign investors.

http://www.insightsonindia.com

INSIGHTS

Page 4

Insights Mock Tests 2015: Solutions Test - 16


10. Solution: c)
FDI in activities not covered under the automatic route requires prior Government
approval and are considered by the Foreign Investment Promotion Board (FIPB),
Ministry of Finance. Application can be made in Form FC-IL; Plain paper
applications carrying all relevant details are also accepted. No fee is payable.
General permission of RBI under FEMA.
Indian companies having foreign investment approval through FIPB route do not
require any further clearance from RBI for receiving inward remittance and issue of
shares to the foreign investors. The companies are required to notify the concerned
Regional office of the RBI of receipt of inward remittances within 30 days of such
receipt and within 30 days of issue of shares to the foreign investors or NRIs.

11. Solution: a)

12. Solution: d)

13. Solution: b)

14. Solution: a)

15. Solution: c)

16. Solution: c)

17. Solution: b)
http://www.thehindu.com/business/Economy/economic-survey-projects-worthrs-88-lakh-crore-stalled/article6941227.ece?ref=sliderNews

http://www.insightsonindia.com

INSIGHTS

Page 5

Insights Mock Tests 2015: Solutions Test - 16

18. Solution: a)
http://www.thehindu.com/business/Economy/rs-377-lakh-crore-subsidy-reachesonly-very-few/article6941195.ece?ref=sliderNews

19. Solution: d)
In the aftermath of the global financial crisis in 2008, sovereign debt levels started to
mount. The revelation that the fiscal deficit in Greece was much higher than stated
earlier set off serious concerns in early 2010 about the sustainability of the debt. The
downgrade of ratings led to a spiral of rising bond yields and further downgrade of
government debt of other peripheral eurozone economies as well, that had high
public debt or a build-up of bank lending or both.
Concerns intensified in early 2010 as cross-border holdings of sovereign debt and
exposure of banks came to light. The financial markets quickly transmitted the
shocks which not only led to a sharp rise in credit default swap (CDS) spreads but
later impacted capital flows elsewhere.

The underlying weaknesses of the zone have made it difficult to resolve the
crisis

The eurozone lacks a single fiscal authority capable of strict enforcement;


Economies with different levels of competitiveness (and fiscal positions) have
a single currency;

These economies cannot adjust through a depreciation of the currency;


There is no lender of last resort, i.e. a full-fledged central bank (as is RBI in
India and other economies).

20. Solution: a)
The process of globalization has been marked by a rising share of exports (as also
imports) that reached 27.9 per cent for the world as a whole in 2010, with some
countries showing much higher dependence of exports. The Database of the World
Bank show that the so called East Asian Miracle Economies was that an export-led,

http://www.insightsonindia.com

INSIGHTS

Page 6

Insights Mock Tests 2015: Solutions Test - 16


investment-fuelled strategy propelled growth and helped them acquire
manufacturing capabilities.
This strategy was supported by a favourable exchange rate, cheap credit, and
relatively low wages which helped to gain competitive advantage. Global demand
for goods, particularly in the advanced markets, lent support to this growth strategy.
As a result, these economies moved up the value chain in manufacturing.

21. Solution: b)
India energy dependence on imported energy sources, appears modest at 25.7 per
cent in terms of total energy usage. (World Bank Database). However, this masks the
fact that around 80 per cent of the crude oil consumed is imported, whereas the bulk
of coal is domestically produced. Even with respect to coal, the country is importing
on the margin to meet domestic demand. On the other side, there is a large fraction
of population that has little or no access to commercial sources of energy and
depends on traditional sources.
Rise in the price of oil in international markets has been the main source of high
current account deficit. High international prices of fossil fuels also result in a higher
import bill, which either gets passed on to the consumers, or results in higher
subsidy thereby affecting fiscal health. That apart, the growing tensions in many oilproducing economies are a source of vulnerability for the energy security of India. In
this one area, the strategic advantage for India would lie in diversifying its energy
sources.

22. Solution: d)
Organised Indian Money Market - Present since independence, its real development
took place after the year 1985. Today there are eight instruments or components of
the Indian money market especially designed to fulfill the short-term fund
requirements of the different categories of the individuals, institutions or the firms
and companies:

Treasury Bills
Call Money Market
Certificate of Deposit
Commercial Bills
Commercial Papers
Mutual Funds

http://www.insightsonindia.com

INSIGHTS

Page 7

Insights Mock Tests 2015: Solutions Test - 16

Repo and Reverse Repo Markets.


Cash Management Bill

Share market is part of the Capital market, not money market.

23. Solution: d)
At present Indian financial market has a number of regulators, precisely eleven
RBI, SEBI, FMC, NABARD, IRDA, SIDBI, NHB, SFCs, LDBI, CLB and Registrar of
Cooperative Societies.
The Narasimhan Committee on Financial System (1991) has made a strong case for a
single regulator for banks, financial institutions and the non-banking financial
institutions in India.
Meanwhile, the Justice B N Srikrishna headed Financial Sector Legislative Reforms
Commission (FSLRC) handed over its report end-March 2013 in which it
recommended that this would be replaced by a horizontal structure whereby the
basic regulatory and monitoring functions of all areas would be done by a Unified
Financial Agency (UFA).

24. Solution: a)
For growth to take place, investments are required in the form of productive assets.
Such investments are long-term in nature. Funds for long-term purposes are raised
either through borrowings from the banks, or the financial institutions, or through
the security market by issuing shares or debentures.
Such funds are supplied by the long-term financial market, i.e. the capital market
and the firms or the productive assets are set up in an economy. But only the setting
up of firms does not guarantee production as these firms keep facing fund
mismatches in the process of continued production.
There is another required segment of financial market which could supply timely
funds to these firms so that they could continue their production process. Such funds
are required usually for a short period (days, fortnights, few months) of time and
thus are considered as their working capital requirements. The segment of financial
market which caters to the short-term requirements of such funds for the enterprises
is known as the money market or the working capital market of the economy.

http://www.insightsonindia.com

INSIGHTS

Page 8

Insights Mock Tests 2015: Solutions Test - 16

25. Solution: d)
Repos allow the banks and the financial institutions to borrow money from the RBI
for the short-term (by selling Government Securities to the RBI). In reverse repos, the
banks and financial institutions purchase Government securities from the RBI
(basically here the RBI is borrowing from the banks and the financial institutions).
All the Government securities are dated and the interest for the repo or reverse repo
transactions are announced by the RBI from time to time.
The provision of repos and the reverse repos have been able to serve the liquidity
evenness in the economy as the banks are able to get the required amount of funds
out of it, and they can park surplus dle funds through it. These instruments have
emerged as important tools in the management of the monetary and credit policy in
the recent years.

26. Solution: a)
The call money market is an important segment of the money market where
uncollateralized borrowing and lending of funds take place on overnight basis.
Participants in the call money market in India currently include scheduled
commercial banks (SCBs)- excluding regional rural banks), cooperative banks (other
than land development banks), and primary dealers, both as borrowers and lenders.
Prudential limits, in respect of both outstanding borrowing and lending transactions
in the call money market for each of these entities, are specified by the RBI.

27. Solution: c)
OMOs are conducted by the RBI via the sale/purchase of government securities (GSec) to/from the market with the primary aim of modulating rupee liquidity
conditions in the market. OMOs are an effective quantitative policy tool in the
armoury of the RBI, but are constrained by the stock of government securities
available with it at a point in time.

28. Solution: d)
http://www.goodreturns.in/classroom/2013/07/what-are-open-market-operationomos-191708.html
http://www.insightsonindia.com

INSIGHTS

Page 9

Insights Mock Tests 2015: Solutions Test - 16


OMOs are not related to Bank rate in any way since OMOs essentially depends on
how much securities RBI is issuing/purchasing, and how much the public is
selling/purchasing.
But, if bank rates are very low and there is a lot of money supply in the economy, the
effectiveness of the OMOs will reduce.
Similarly fiscal deficit too is not related to OMOs. But if there is high fiscal deficit,
OMOs can hardly control inflation.

29. Solution: a)
Liquidity Adjustment Facility (LAF) is the primary instrument of Reserve Bank of
India for modulating liquidity and transmitting interest rate signals to the market. It
refers to the difference between the two key rates viz. repo rate and reverse repo
rate. Informally, Liquidity Adjustment Facility is also known as Liquidity Corridor.
Under Repo, the banks borrow money from RBI to meet short term needs by putting
government securities (G-secs) as collateral. Under Reverse Repo, RBI borrows
money from banks by lending securities. While repo injects liquidity into the system,
the Reverse repo absorbs the liquidity from the system. RBI only announces Repo
Rate. The Reverse Repo Rate is linked to Repo Rate and is 100 basis points (1%)
below repo rate. RBI makes decision regarding Repo Rate on the basis of prevalent
market conditions and relevant factors.

30. Solution: b)
http://www.arthapedia.in/index.php?title=Marginal_Standing_Facility

31. Solution: a)
http://en.wikipedia.org/wiki/Exchange-traded_fund\

32. Solution: c)
A non-banking financial company (NBFC) is a company registered under the
Companies Act, 1956 and is engaged in the business of loans and advances,
acquisition of shares/ stock/ bonds/ debentures/ securities issued by government
or local authority or other securities of like marketable nature, leasing, hirehttp://www.insightsonindia.com

INSIGHTS

Page 10

Insights Mock Tests 2015: Solutions Test - 16


purchase, insurance business, chit business, but does not include any institution
whose principal business is that of agriculture activity, industrial activity,
sale/purchase/construction of immovable property.
A non-banking institution which is a company and which has its principal business
of receiving deposits under any scheme or arrangement or any other manner, or
lending in any manner is also a non-banking financial company (residuary nonbanking company i.e. RNBC).
NBFCs are doing functions akin to that of banks, however there are a few
differences:

An NBFC cannot accept demand deposits (which are payable on demand),


like the savings and current accounts.

It is not a part of the payment and settlement system and as such cannot issue
cheques to its customers

33. Solution: d)
RBI is the Central bank of India and performs these functions and fulfils the
following roles:

It is the issuing agency of the currency and coins other than rupee one
currency and coin (which are issued by the Ministry of Finance itself with the
signature of the Revenue Secretary on the note).
Distributing agent for the currency and coins issued by the Government.
Banker of the Government.
Bank of the banks/Bank of the last resort.
Announces the credit and monetary policy for the economy.
Stabilising the rate of inflation.
Stabilising the exchange rate of rupee.
Keeper of the foreign currency reserves.
Agent of the Government of India in the IMF.
Performing a variety of developmental and promotional functions under
which it did set up institutions like IDBI, SIDBI, NABARD, NHB, etc.

34. Solution: d)
Banks in India are required to hold a certain proportion of their deposits in the
form of cash. However, actually Banks dont hold these as cash with themselves,
http://www.insightsonindia.com

INSIGHTS

Page 11

Insights Mock Tests 2015: Solutions Test - 16


but deposit such case with Reserve Bank of India (RBI) / currency chests, which is
considered as equivalent to holding cash with RBI. This minimum ratio (that is the
part of the total deposits to be held as cash) is stipulated by the RBI and is known as
the CRR or Cash Reserve Ratio.
Thus, When a banks deposits increase by Rs100, and if the cash reserve ratio is 6%,
the banks will have to hold additional Rs 6 with RBI and Bank will be able to use
only Rs 94 for investments and lending / credit purpose. Therefore, higher the ratio
(i.e. CRR), the lower is the amount that banks will be able to use for lending and
investment. This power of RBI to reduce the lendable amount by increasing the
CRR, makes it an instrument in the hands of a central bank through which it can
control the amount that banks lend. Thus, it is a tool used by RBI to control liquidity
in the banking system.

35. Solution: a)
The statutory liquidity ratio (SLR) is the ratio (fixed by the RBI) of the total deposits
of a bank which is to be maintained by the bank with itself in non-cash form
prescribed by the Government.
The ratio of liquid assets to demand and time liabilities is known as Statutory
Liquidity Ratio (SLR). RBI is empowered to increase this ratio up to 40%. An
increase in SLR also restrict the banks leverage position to pump more money into
the economy.

36. Solution: d)
Bank Rate is the rate at which central bank of the country (in India it is RBI) allows
finance to commercial banks. Bank Rate is a tool, which central bank uses for shortterm purposes. Any upward revision in Bank Rate by central bank is an indication
that banks should also increase deposit rates as well as Base Rate / Benchmark
Prime Lending Rate. Thus any revision in the Bank rate indicates that it is likely that
interest rates on your deposits are likely to either go up or go down, and it can also
indicate an increase or decrease in your EMI.
The clients who borrow through this route are the GoI, State governments, Banks,
Financial Institutions, Co-operative Banks, NBFCs, etc. The rate has direct impact on

http://www.insightsonindia.com

INSIGHTS

Page 12

Insights Mock Tests 2015: Solutions Test - 16


the long-term lending activities of the concerned lending bodies operating in the
Indian financial system.

37. Solution: d)
The impact of CRR, SLR and bank rate has already been explained in some of the
explanations in previous questions.
For Reverse Repo rate - It is the rate of interest the RBI pays to its clients who offer
short term loan to it.
It is reverse of the repo rate and this was started in November 1996 as part of
Liquidity Adjustment Facility (LAF) by the RBI. In practice, financial institutions
operating in India park their surplus funds with the RBI for short-term period and
earn money. It has a direct bearing on the interest rates charged by the banks and the
financial institutions on their different forms of loans.
This tool was utilised by the RBI in the wake of over money supply with the Indian
banks and lower loan disbursal to serve twin purposes of cutting down banks losses
and the prevailing interest rate. It has emerged as a very important tool in direction
of following cheap interest regimethe general policy of the RBI since reform
process started.

38. Solution: a)
The President is elected not directly by the people but by members of electoral
college consisting of:

the elected members of both the Houses of Parliament;

the elected members of the legislative assemblies of the states; and


the elected members of the legislative assemblies of the Union Territories of
Delhi and Puducherry

39. Solution: d)
The nominated members of both of Houses of Parliament, the nominated members
of the state legislative assemblies, the members (both elected and nominated) of the
http://www.insightsonindia.com

INSIGHTS

Page 13

Insights Mock Tests 2015: Solutions Test - 16


state legislative councils (in case of the bicameral legislature) and the nominated
members of the Legislative Assemblies of Delhi and Puducherry do not participate
in the election of the President. Where an assembly is dissolved, the members cease
to be qualified to vote in presidential election, even if fresh elections to the dissolved
assembly are not held before the presidential election.
While electing and impeaching Vice-President, nominated members participate.

40. Solution: d)

41. Solution: c)
The President is entitled to a number of privileges and immunities. He enjoys
personal immunity from legal liability for his official acts. During his term of office,
he is immune from any criminal proceedings, even in respect of his personal acts. He
cannot be arrested or imprisoned. However, after giving two months notice, civil
proceedings can be instituted against him during his term of office in respect of his
personal acts.

http://www.insightsonindia.com

INSIGHTS

Page 14

Insights Mock Tests 2015: Solutions Test - 16

42. Solution: d)
The President can be removed from office by a process of impeachment for violation
of the Constitution. However, the Constitution does not define the meaning of the
phrase violation of the Constitution.
The impeachment charges can be initiated by either House of Parliament. These
charges should be signed by one-fourth members of the House (that framed the
charges), and a 14 days notice should be given to the President. After the
impeachment resolution is passed by a majority of two-thirds of the total
membership of that House, it is sent to the other House, which should investigate
the charges.
The President has the right to appear and to be represented at such investigation. If
the other House also sustains the charges and passes the impeachment resolution by
a majority of two-thirds of the total membership, then the President stands removed
from his office from the date on which the bill is so passed.
Thus, an impeachment is a quasi-judicial procedure in the Parliament. In this
context, two things should be noted: (a) the nominated members of either House of
Parliament can participate in the impeachment of the President though they do not
participate in his election; (b) the elected members of the legislative assemblies of
states and the Union Territories of Delhi and Puducherry do not participate in the
impeachment of the President though they participate in his election.
No President has so far been impeached.

43. Solution: d)
He appoints the prime minister and the other ministers. They hold office during his
pleasure.
He appoints the attorney general of India and determines his remuneration. The
attorney general holds office during the pleasure of the President.
He appoints the comptroller and auditor general of India, the chief election
commissioner and other election commissioners, the chairman and members of the
Union Public Service Commission, the governors of states, the chairman and
members of finance commission, and so on.
http://www.insightsonindia.com

INSIGHTS

Page 15

Insights Mock Tests 2015: Solutions Test - 16


He can seek any information relating to the administration of affairs of the Union,
and proposals for legislation from the prime minister.

44. Solution: d)
He enjoys the following legislative powers.

He can summon or prorogue the Parliament and dissolve the Lok Sabha. He
can also summon a joint sitting of both the Houses of Parliament, which is
presided over by the Speaker of the Lok Sabha.
He can address the Parliament at the commencement of the first session after
each general election and the first session of each year.

He can send messages to the Houses of Parliament, whether with respect to a


bill pending in the Parliament or otherwise.

He can appoint any member of the Lok Sabha to preside over its proceedings
when the offices of both the Speaker and the Deputy Speaker fall vacant.
Similarly, he can also appoint any member of the Rajya Sabha to preside over
its proceedings when the offices of both the Chairman and the Deputy
Chairman fall vacant.

45. Solution: b)
The President has the veto power over the bills passed by the Parliament10, that is,
he can withhold his assent to the bills. The object of conferring this power on the
President is two-fold(a) to prevent hasty and ill-considered legislation by the
Parliament; and (b) to prevent legislation which may be unconstitutional.
The veto power enjoyed by the executive in modern states can be classified into the
following four types:

Absolute veto that is, withholding of assent to the bill passed by the
legislature.

Qualified veto, which can be overridden by the legislature with a higher


majority.
Suspensive veto, which can be over ridden by the legislature with an ordinary
majority.
Pocket veto that is, taking no action on the bill passed by the legislature.

http://www.insightsonindia.com

INSIGHTS

Page 16

Insights Mock Tests 2015: Solutions Test - 16


Of the above four, the President of India is vested with threeabsolute veto,
suspensive veto and pocket veto. There is no qualified veto in the case of Indian
President; it is possessed by the American President.

46. Solution: a)
Suspensive Veto - The President exercises this veto when he returns a bill for
reconsideration of the Parliament.
However, if the bill is passed again by the Parliament with or without amendments
and again presented to the President, it is obligatory for the President to give his
assent to the bill. This means that the presidential veto is overridden by a re-passage
of the bill by the same ordinary majority (and not a higher majority as required in
USA).
The President does not possess this veto in the case of money bills. The President can
either give his assent to a money bill or withhold his assent to a money bill but
cannot return it for the reconsideration of the Parliament. Normally, the President
gives his assent to money bill as it is introduced in the Parliament with his previous
permission.

47. Solution: d)

48. Solution: d)

49. Solution: d)

50. Solution: a)

51. Solution: c)
The ordinance-making power is the most important legislative power of the
President. It has been vested in him to deal with unforeseen or urgent matters. But,
the exercise of this power is subject to the following limitations:

http://www.insightsonindia.com

INSIGHTS

Page 17

Insights Mock Tests 2015: Solutions Test - 16

He can promulgate an ordinance only when both the Houses of Parliament


are not in session or when either of the two Houses of Parliament is not in
session. An ordinance can also be issued when only one House is in session
because a law can be passed by both the Houses and not by one House alone.
An ordinance made when both the Houses are in session is void. Thus, the
power of the President to legislate by ordinance is not a parallel power of
legislation.

He can make an ordinance only when he is satisfied that the circumstances


exist that render it necessary for him to take immediate action. In Cooper case,
(1970), the Supreme Court held that the Presidents satisfaction can be
questioned in a court on the ground of malafide. This means that the decision
of the President to issue an ordinance can be questioned in a court on the
ground that the President has prorogued one House or both Houses of
Parliament deliberately with a view to promulgate an ordinance on a
controversial subject, so as to bypass the parliamentary decision and thereby
circumventing the authority of the Parliament.

52. Solution: b)
The Supreme Court examined the pardoning power of the President under different
cases and laid down the following principles:

The petitioner for mercy has no right to an oral hearing by the President.
The President can examine the evidence afresh and take a view different from
the view taken by the court.

The power is to be exercised by the President on the advice of the union


cabinet.

The President is not bound to give reasons for his order.


The President can afford relief not only from a sentence that he regards as
unduly harsh but also from an evident mistake.
There is no need for the Supreme Court to lay down specific guidelines for
the exercise of power by the President.
The exercise of power by the President is not subject to judicial review except
where the presidential decision is arbitrary, irrational, mala fide or
discriminatory.

Where the earlier petition for mercy has been rejected by the President, stay
cannot be obtained by filing another petition.

http://www.insightsonindia.com

INSIGHTS

Page 18

Insights Mock Tests 2015: Solutions Test - 16

53. Solution: a)
The President can act on his discretion (that is, without the advice of the ministers)
under the following situations:

Appointment of Prime Minister when no party has a clear majority in the Lok
Sabha or when the Prime Minister in office dies suddenly and there is no
obvious successor.
Dismissal of the council of ministers when it cannot prove the confidence of
the Lok Sabha.
Dissolution of the Lok Sabha if the council of ministers has lost its majority.

54. Solution: d)
To be eligible for election as Vice-President, a person should fulfil the following
qualifications:

He should be a citizen of India.


He should have completed 35 years of age.

He should be qualified for election as a member of the Rajya Sabha.


He should not hold any office of profit under the Union government or any
state government or any local authority or any other public authority.

But, a sitting President or Vice-President of the Union, the governor of any state and
a mini-ster for the Union or any state is not deemed to hold any office of profit and
hence qualified for being a candidate for Vice-President.

55. Solution: c)
The Constitution does not contain any specific procedure for the selection and
appointment of the Prime Minister. Article 75 says only that the Prime Minister shall
be appointed by the president.
http://www.insightsonindia.com

INSIGHTS

Page 19

Insights Mock Tests 2015: Solutions Test - 16


However, this does not imply that the president is free to appoint any one as the
Prime Minister. In accordance with the conventions of the parliamentary system of
government, the President has to appoint the leader of the majority party in the Lok
Sabha as the Prime Minister. But, when no party has a clear majority in the Lok
Sabha, then the President may exercise his personal discretion in the selection and
appointment of the Prime Minister.

In 1997, the Supreme Court held that a person who is not a member of either House
of Parliament can be appointed as Prime Minister for six months, within which, he
should become a member of either House of Parliament; otherwise, he ceases to be
the Prime Minister.
Constitutionally, the Prime Minister may be a member of any of the two Houses of
parliament. For example, three Prime Ministers, Indira Gandhi (1966), Deve Gowda
(1996) and Manmohan Singh (2004), were members of the Rajya Sabha. In Britain, on
the other hand, the Prime Minister should definitely be a member of the Lower
House (House of Commons).

45. Solution: d)
The Prime Minister enjoys the following powers as head of the Union council of
ministers:

He recommends persons who can be appointed as ministers by the president.


The President can appoint only those persons as ministers who are
recommended by the Prime Minister.

He allocates and reshuffles various portfolios among the ministers.


He can ask a minister to resign or advise the President to dismiss him in case
of difference of opinion.

He presides over the meeting of council of ministers and influences its


decisions.
He guides, directs, controls, and coordinates the activities of all the ministers.
He can bring about the collapse of the council of ministers by resigning from
office.

Since the Prime Minister stands at the head of the council of ministers, the other
ministers cannot function when the Prime Minister resigns or dies. In other words,
the resignation or death of an incumbent Prime Minister automatically dissolves the
http://www.insightsonindia.com

INSIGHTS

Page 20

Insights Mock Tests 2015: Solutions Test - 16


council of ministers and thereby generates a vacuum. The resignation or death of
any other minister, on the other hand, merely creates a vacancy which the Prime
Minister may or may not like to fill.

56. Solution: d)
Article 74 provides for a council of ministers with the Prime Minister at the head to
aid and advise the President in the exercise of his functions. The 42nd and 44th
Constitutional Amendment Acts have made the advice binding on the President.
Further, the nature of advice tendered by ministers to the President cannot be
enquired by any court. This provision emphasises the intimate and the confidential
relationship between the President and the ministers.
In 1971, the Supreme Court held that even after the dissolution of the Lok Sabha, the
council of ministers does not cease to hold office. Article 74 is mandatory and,
therefore, the president cannot exercise the executive power without the aid and
advise of the council of ministers. Any exercise of executive power without the aid
and advice will be unconstitutional as being violative of Article 74.
Again in 1974, the court held that wherever the Constitution requires the
satisfaction of the President, the satisfaction is not the personal satisfaction of the
President but it is the satisfaction of the council of ministers with whose aid and on
whose advice the President exercises his powers and functions.

57. Solution: a)
The ministers of state can either be given independent charge of
ministries/departments or can be attached to cabinet ministers. In case of
attachment, they may either be given the charge of departments of the ministries
headed by the cabinet ministers or allotted specific items of work related to the
ministries headed by cabinet ministers. In both the cases, they work under the
supervision and guidance as well as under the overall charge and responsibility of
the cabinet ministers. In case of independent charge, they perform the same
functions and exercise the same powers in relation to their ministries/departments
as cabinet ministers do.
However, they are not members of the cabinet and do not attend the cabinet
meetings unless specially invited when something related to their
ministries/departments are considered by the cabinet.

http://www.insightsonindia.com

INSIGHTS

Page 21

Insights Mock Tests 2015: Solutions Test - 16


Next in rank are the deputy ministers. They are not given independent charge of
ministries/departments. They are attached to the cabinet ministers or ministers of
state and assist them in their administrative, political, and parliamentary duties.
They are not members of the cabinet and do not attend cabinet meetings.

58. Solution: d)
The word cabinet; was inserted in Article 352 of the Constitution in 1978 by the 44th
Constitutional Amendment Act. Thus, it did not find a place in the original text of
the Constitution.
Now also, Article 352 only defines the cabinet saying that it is the council consisting
of the prime minister and other ministers of cabinet rank appointed under Article 75
and does not describe its powers and functions. In other words, its role in our
politico-administrative system is based on the conventions of parliamentary
government as developed in Britain.

59. Solution: d)
They are set up by the Prime Minister according to the exigencies of the time
and requirements of the situation. Hence, their number, nomenclature, and
composition varies from time to time.

Their membership varies from three to eight. They usually include only
Cabinet Ministers. However, the non-cabinet Ministers are not debarred from
their membership.
They not only include the Ministers in charge of subjects covered by them but
also include other senior Ministers.
They are mostly headed by the Prime Minister. Sometimes other Cabinet
Ministers, particularly the Home Minister or the Finance Minister, also acts as
their Chairman. But, in case the Prime Minister is a member of a committee,
he invariably presides over it.

60. Solution: a)

http://www.insightsonindia.com

INSIGHTS

Page 22

Insights Mock Tests 2015: Solutions Test - 16


Cabinet Committees not only sort out issues and formulate proposals for the
consideration of the Cabinet, but also take decisions. However, the Cabinet can
review their decisions. It means that the PM too can review their decisions.
They are an organisational device to reduce the enormous workload of the Cabinet.
They also facilitate in-depth examination of policy issues and effective coordination.
They are based on the principles of division of labour and effective delegation.

61. Solution: b)
Though the President of India is not a member of either House of Parliament and
does not sit in the Parliament to attend its meetings, he is an integral part of the
Parliament. This is because a bill passed by both the Houses of Parliament cannot
become law without the Presidents assent. He also performs certain functions
relating to the proceedings of the Parliament, for example, he summons and
prorogues both the Houses, dissolves the Lok Sabha, addresses both the Houses,
issues ordinances when they are not in session, and so on.
In this respect, the framers of the Indian Constitution relied on the British pattern
rather than the American pattern. In Britain, the Parliament consists of the Crown
(King or Queen), the House of Lords (Upper House) and the House of Commons
(Lower House). By contrast, the American president is not an integral part of the
legislature. In USA, the legislature, which is known as Congress, consists of the
Senate (Upper House) and the House of Representatives (Lower House).

62. Solution: b)
The maximum strength of the Rajya Sabha is fixed at 250, out of which, 238 are to be
the representatives of the states and union territories (4 members elected indirectly)
and 12 are nominated by the president.
Out of the seven union territories, only two (Delhi and Puducherry) have
representation in Rajya Sabha. The populations of other five union territories are too
small to have any representative in the Rajya Sabha.

http://www.insightsonindia.com

INSIGHTS

Page 23

Insights Mock Tests 2015: Solutions Test - 16

63. Solution: a)
Though the Constitution has adopted the system of proportional representation in
the case of Rajya Sabha, it has not preferred the same system in the case of Lok
Sabha. Instead, it has adopted the system of territorial representation for the election
of members to the Lok Sabha.
Under territorial representation, every member of the legislature represents a
geographical area known as a constituency. From each constituency, only one
representative is elected. Hence such a constituency is known as single-member
constituency. In this system, a candidate who secures majority of votes is declared
elected. This simple majority system of representation does not represent the whole
electorate. In other words, it does not secure due representation to minorities (small
groups).

64. Solution: d)
The system of proportional representation aims at removing the defects of territorial
representation. Under this system, all sections of the people get representation in
proportion to their number. Even the smallest section of the population gets its due
share of representation in the legislature.
There are two kinds of proportional representation, namely, single transferable vote
system and list system. In India, the first kind is adopted for the election of members
to the Rajya Sabha and state legislative council and for electing the President and the
Vice-President.
Though some members of the Constituent Assembly had advocated the system of
proportional representation for the election of members to the Lok Sabha, the
Constitution has not adopted the system due to two reasons.

Difficulty for the voters to understand the system (which is complicated) due
to low literacy scale in the country.

Unsuitability to the parliamentary government due to the tendency of the


system to multiply political parties leading to instability in government.

65. Solution: a)

http://www.insightsonindia.com

INSIGHTS

Page 24

Insights Mock Tests 2015: Solutions Test - 16


On the question whether a member is subject to any of the disqualifications under
RPA 1951, the presidents decision is final. However, he should obtain the opinion of
the election commission and act accordingly.
The question of disqualification under the Tenth Schedule is decided by the
Chairman in the case of Rajya Sabha and Speaker in the case of Lok Sabha (and not
by the president of India). In 1992, the Supreme Court ruled that the decision of the
Chairman/Speaker in this regard is subject to judicial review.

66. Solution: c)
Every member of either House of Parliament, before taking his seat in the House, has
to make and subscribe to an oath or affirmation before the President or some person
appointed by him for this purpose. In his oath or affirmation, a member of
Parliament swears:

to bear true faith and allegiance to the Constitution of India;


to uphold the sovereignty and integrity of India; and
to faithfully discharge the duty upon which he is about to enter.

Option 2 is part of oath taken by a Minister.


Option 3 is that taken by the President of India.

67. Solution: d)
The Speaker is elected by the Lok Sabha from amongst its members (as soon as may
be, after its first sitting). Whenever the office of the Speaker falls vacant, the Lok
Sabha elects another member to fill the vacancy. The date of election of the Speaker
is fixed by the President.
When a resolution for the removal of the Speaker is under consideration of the
House, he cannot preside at the sitting of the House, though he may be present.
However, he can speak and take part in the proceedings of the House at such a time
and vote in the first instance, though not in the case of an equality of votes.
It should be noted here that, whenever the Lok Sabha is dissolved, the Speaker does
not vacate his office and continues till the newly- elected Lok Sabha meets.
http://www.insightsonindia.com

INSIGHTS

Page 25

Insights Mock Tests 2015: Solutions Test - 16


The Speaker is the head of the Lok Sabha, and its representative. He is the guardian
of powers and privileges of the members, the House as a whole and its committees.
He is the principal spokesman of the House, and his decision in all Parliamentary
matters is final.

68. Solution: a)
The Speaker is the head of the Lok Sabha, and its representative. He is the guardian
of powers and privileges of the members, the House as a whole and its committees.
He is the principal spokesman of the House, and his decision in all Parliamentary
matters is final. He is thus much more than merely the presiding officer of the Lok
Sabha. In these capacities, he is vested with vast, varied and vital responsibilities and
enjoys great honour, high dignity and supreme authority within the House.
The Speaker of the Lok Sabha derives his powers and duties from three sources, that
is, the Constitution of India, the Rules of Procedure and Conduct of Business of Lok
Sabha, and Parliamentary Conventions (residuary powers that are unwritten or
unspecified in the Rules).

69. Solution: a)
The LS Speaker presides over a joint setting of the two Houses of Parliament. Such a
sitting is summoned by the President to settle a deadlock between the two Houses
on a bill.
He can allow a secret sitting of the House at the request of the Leader of the House.
When the House sits in secret, no stranger can be present in the chamber, lobby or
galleries except with the permission of the Speaker.
He decides whether a bill is a money bill or not and his decision on this question is
final. When a money bill is transmitted to the Rajya Sabha for recommendation and
presented to the President for assent, the Speaker endorses on the bill his certificate
that it is a money bill.
He decides the questions of disqualification of a member of the Lok Sabha, arising
on the ground of defection under the provisions of the Tenth Schedule. In 1992, the

http://www.insightsonindia.com

INSIGHTS

Page 26

Insights Mock Tests 2015: Solutions Test - 16


Supreme Court ruled that the decision of the Speaker in this regard is subject to
judicial review.

70. Solution: c)
The LS Speaker is provided with a security of tenure. He can be removed only by a
resolution passed by the Lok Sabha by an absolute majority (i.e, a majority of the
total members of the House) and not by an ordinary majority (i.e, a majority of the
members present and voting in the House).
This motion of removal can be considered and discussed only when it has the
support of at least 50 members.

71. Solution: d)
The bill amends the Mines and Minerals (Development and Regulation) Act, 1957 in
order to regulate the mining sector in the country.
Key provisions of the bill

Addition of a new 4th Schedule in to parent Act in order to include bauxite,


iron ore, limestone and manganese ore as notified minerals.

Creation of prospecting license-cum-mining lease as a new category of mining


license. It is defined as a two stage-concession for the purpose of undertaking
prospecting operations followed by mining operations.
Maximum area for mining- increase the area limits for mining, instead of
providing additional leases as per parent Acts area limitation.

Lease period- Mining leases will be granted for a lease period of 50 years for
all minerals other than coal, lignite and atomic minerals. Lease period for coal
and lignite remains unchanged.

72. Solution: a)
Objective is to bring out a totally error-free and authenticated electoral roll
throughout the country.
For the authentication purpose, Electoral Photo Identity Card (EPIC) data of
electors will be linked with Aadhar data.
http://www.insightsonindia.com

INSIGHTS

Page 27

Insights Mock Tests 2015: Solutions Test - 16

It also focuses to improve the image quality of electors along with sorting
issues like corrections of errors. Facility to link Aadhar number will be
provided to electors through sms, email, mobile application and National
Voters Service Portal using web services through ECI website.

Electors also can link their Aadhar number by making a call at 1950 to state
call centres. Under NERPAP, collection and feeding of Aadhar will also be
done by Electoral Registration Officer.

In this regard special Camps will be organized, Voter Facilitation Centres, eSeva centres and Citizen Service Centres. While Booth Level Officers will
conduct door-to-door surveys to collect the details.

73. Solution: a)
Eutelsats satellite is part of its 35-member commercial network. It will provide
services like mobile, internet, video and other communications services to expand its
reach into the Americas. While, ABS new satellite will serve its customers in Africa,
Europe and the Middle East.
These satellites are fitted with lightweight, all-electric engines rather than
conventional chemical propulsion systems. These electric engines allow satellites to
produce electric propulsion in order to reach and remain fixed in particular orbit.
Electric propulsion from these satellites consumes less fuel compared with satellite
having chemical propulsion. Thus making satellites lighter in weight and further
reducing cost of launch.

74. Solution: a)
Following has happened in the area of taxation in the recent Budget:

No change in the Tax slab on personal income.


Wealth tax abolished.
Additional 2% surcharge for the super-rich with income more than Rs. 1
crore.

For next 4 years, corporate tax has been reduced to 25 percent from present 30
percent. Service tax increased to14 per cent.

Apart from CSR, 100% tax exemption for contribution to Swachch Bharat
Abhiyan.

http://www.insightsonindia.com

INSIGHTS

Page 28

Insights Mock Tests 2015: Solutions Test - 16

75. Solution: a)
http://www.thehindu.com/business/budget/mudra-bankto-fund-the-missingmiddle/article6946797.ece

76. Solution: d)
Some of the Initiatives launched are:

Under the Swachh Bharat Abhiyan, six crore toilets across the country will be
built.

By 2020, housing for all.


Visa on Arrival (VoA) facility to be increased for 150 countries from present
43 countries.
As an alternative to purchasing gold, Sovereign Gold Bond will be launched.

In order to facilitate depositors of gold to earn interest and jewellers to obtain


loans on their metal accounts new scheme will be launched.
Forward Markets Commission (FMC) will be merged with the Securities and
Exchange Board of India (SEBI).

77. Solution: a)
http://www.washingtonpost.com/blogs/monkey-cage/wp/2014/07/17/what-thenew-bank-of-brics-is-all-about/

78. Solution: a)
http://www.finance.mp.gov.in/14fc_press_note_eng.pdf
http://www.insightsonindia.com

INSIGHTS

Page 29

Insights Mock Tests 2015: Solutions Test - 16


Refer to first two pages only.

79. Solution: a)
In the Air Brake system, compressed air is used for operating the brake system.
These brakes can be the conventional types or directly mounted on the bogies of the
coaches. The latest design is the Disc Brake System (DBS) found in LHB coaches and
is similar to what is found in automobiles.
DBS is micro processor controlled and an advanced version of Air Brake system. It is
essential for high speeds of 160kmph plus. It is superior in terms of reduced braking
distance, higher wheel life due to reduced frequency of wheel turning, and reduced
maintenance, less braking noise and higher efficiency due to simple brake rigging.
PIB FEATURES http://pib.nic.in/newsite/efeatures.aspx

80. Solution: b)
http://www.downtoearth.org.in/content/indias-climate-dilemma

81. Solution: b)

82. Solution: b)

83. Solution: d)

84. Solution: d)
http://www.insightsonindia.com

INSIGHTS

Page 30

Insights Mock Tests 2015: Solutions Test - 16

85. Solution: d)

86. Solution: d)
http://www.downtoearth.org.in/content/budget-2015-suresh-prabhu-promisesgreen-clean-indian-railways

87. Solution: a)
http://www.downtoearth.org.in/content/agriculture-wrong-prescription-rightdiagnosis
In the budget, Jaitley increased the farm credit corpus from Rs 8 lakh crore to Rs 8.5
lakh crore and announced the setting up of a Unified National Agriculture Market to
provide farmers better price for their produce.
But, around 94 per cent of the existing farm credit corpus goes directly to agro-based
industries and not to the farmers. So when the finance minister said Rs 8.5 lakh crore
will be given as farm credit, he actually meant that the farmers would get only Rs
50,000 crore or six per cent of the corpus.

88. Solution: a)
http://www.downtoearth.org.in/content/findings-economic-survey-2015-arecause-worry

89. Solution: c)
http://www.downtoearth.org.in/content/earth-s-natural-cooling-process-offsetglobal-warming-past-decade-0\
http://www.insightsonindia.com

INSIGHTS

Page 31

Insights Mock Tests 2015: Solutions Test - 16

90. Solution: a)
http://www.downtoearth.org.in/content/these-mobile-weather-stations-can-helpcope-floods-landslides

91. Solution: b)
Pursuant to the resolution adopted on 29-05-2007 by the National Development
Council (NDC), to reorient the current agricultural development strategies to meet
the needs of the farmers and for fresh efforts by the Central and State Governments
to rejuvenate the agricultural sector so as to achieve 4% annual growth during the
11th Five Year Plan, a new State Plan Scheme of Additional Central Assistance (ACA)
for agriculture and allied sectors, namely, Rashtriya Krishi Vikas Yojana (RKVY) was
launched during 2007-08 with an envisaged outlay of Rs. 25,000 crore for the Plan
period.
It requires the States to prepare District and State Agriculture Plans for creation of
such infrastructure, which are essential to catalyse the existing production scenario
for achieving higher production. Additional Central Assistance (ACA) is made
available to the States as 100% grants.
The RKVY Guidelines recognize and build on the need for convergence and
integration of the various programmes implemented at District/State level into
District Agriculture Plans (DAPs) and State Agriculture Plan (SAP). Each district is
required to formulate a District Agriculture Plan by including the resources available
from other existing schemes, District, State or Central Schemes such as Backward
Region Grant Fund (BRGF), Swarnajayanti Gram Swarozgar Yojana (SGSY),
National Rural Employment Guarantee Scheme (NREGS), Bharat Nirman and tied
and untied grants from the Central and State Finance Commissions etc. The District
Agriculture Plans are not to be the usual aggregation of the existing schemes but
would aim at moving towards projecting the requirements for development of
agriculture and allied sectors of the district. These plans present the vision for
agriculture and allied sectors within the overall development perspective of the
district. The District Agriculture Plans would reflect the financial requirement and
the sources of financing the agriculture development plans in a comprehensive way.
The DAP will include animal husbandry and fishery, minor irrigation projects, rural
development works, agricultural marketing schemes and schemes for water
harvesting and conservation, keeping in view the natural resources and
http://www.insightsonindia.com

INSIGHTS

Page 32

Insights Mock Tests 2015: Solutions Test - 16


technological possibilities in each district. Each State is further required to prepare a
comprehensive State Agricultural Plan (SAP) by integrating the DAPs. The State will
have to indicate resources that can flow from the State to the district.
During XII Plan, RKVY funding will be provided through three streams viz.
production growth (35%), infrastructure & Assets and sub-schemes (20%). The
remaining 10% will be is provisioned as flexi fund from which states can undertake
either production growth or infrastructure & assets projects depending upon States
needs & priorities. Looking at the requirement of increasing investment,
Government has recently done way with 35% requirement in production stream
thus paving the way for 100% allocation in investments for infrastructure buildings
& creation of assets.
The States have been provided flexibility and autonomy in the process of selection,
planning, approval and execution of schemes to make investments in interventions
as per their priorities and agro-climatic requirements so that the outcomes are as
envisaged in the RKVY objectives. The projects of the State Governments are
approved by the State Level Sanctioning Committees (SLSCs) under the
Chairmanship of Chief Secretary of the respective States. The funds are routed
through the State Agriculture Department, which is the nodal Department for the
scheme.

http://rkvy.nic.in/SubSchemes-Iframe.html
Refer Page No. 88-90 of India 2015 for more details.

92. Solution: a)
The Mission has been continued during 12th Five Year Plan with inclusion of coarse
cereals crops and commercial crops (sugarcane, jute, cotton). The Mission has target
of additional production of 25 million tonnes of foodgrains comprising 10 million
tonnes of rice, 8 million tonnes of wheat, 4 million tonnes of pulses and 3 million
tonnes of coarse cereals by the end of 12th Five Year Plan.

93. Solutions: c)
National Mission for Sustainable Agriculture (NMSA) seeks to transform Indian
agriculture into a climate resilient production system through suitable adaptation
and mitigation measures in domains of both crops and animal husbandry. NMSA as
http://www.insightsonindia.com

INSIGHTS

Page 33

Insights Mock Tests 2015: Solutions Test - 16


a programmatic intervention focuses on promotion of location specific
integrated/composite farming systems; resource conservation technologies;
comprehensive soil health management; efficient on-farm water management and
mainstreaming rainfed technologies.
NMSA identifies 10 key dimensions namely seed & culture water, pest, nutrient,
farming practices, credit, insurance, market, information and livelihood
diversification for promoting suitable agricultural practices that covers both
adaption and mitigation measures through four functional areas, namely, Research
and Development, Technologies, Products and Practices, Infrastructure and Capacity
building. During XII Five Year Plan, these dimensions have been embedded and
mainstreamed into Missions/Programmes/Schemes of Ministry of Agriculture
including NMSA through a process of restructuring of various schemes/missions
implemented during XI Five Year Plan and convergence with other related
programmes of Central/State Governments.

94. Solution: b)
Agriculture cooperatives distribute around 16% of agricultural credit in India. For
more info refer Page No. 95, India 2015

95. Solution: b)
The tractor density in India is 16 tractors per 1000 hectares whereas world average is
19.
Please refer page no. 98, India 2015
Some interesting articles on farm mechanization in India:
http://www.thehindubusinessline.com/opinion/farm-mechanisation-indianstyle/article4921271.ece
http://www.thehindu.com/business/Industry/talking-business-india-is-still-inearly-stages-of-farm-mechanisation/article6345284.ece

96. Solution: b)
The National School of Drama is one of the foremost theatre training institutions in
the world and the only one of its kind in India. It was set up by the Sangeet Natak
http://www.insightsonindia.com

INSIGHTS

Page 34

Insights Mock Tests 2015: Solutions Test - 16


Akademi as one of its constituent units in 1959. In 1975, it became an independent
entity and was registered as an autonomous organization under the Societies
Registration Act XXI of 1860, fully financed by the Ministry of Culture, Government
of India.
http://nsd.gov.in/delhi/index.php/about-nsd/

97. Solution: c)
Sahitya Akademi in 1996, instituted a Fellowship in the name of the great scholar
and aesthete Dr. Ananda Coomaraswamy to be offered to scholars from Asian
countries to pursue literary projects of their choice.
http://sahitya-akademi.gov.in/sahityaakademi/fellows/sahitya_akademi_fellowship.jsp#anand
http://en.wikipedia.org/wiki/Ananda_Coomaraswamy

98. Solution: d)
Other four cities where they are located are: Udaipur, Allahabad, Nagpur and
Kolkata
99. Solution:
In order to promote tourism, Tourist Visa on Arrival (TVOA) scheme was
introduced for the nationals of five countries, namely, Japan, Singapore, Finland,
Luxembourg and New Zealand with effect from 01.01.2010 for one year. The scheme
has been found to be useful by the foreign nationals.
Visa on Arrival Facility is available for holders of passport of following countries
Australia, Brazil, Cambodia, Cook Islands, Djibouti, Fiji, Finland, Germany, Guyana,
Indonesia, Israel, Japan, Jordan, Kenya, Kiribati, Laos, Luxembourg, Marshall
Islands, Mauritius, Mexico, Micronesia, Myanmar, Nauru,
New Zealand,Niue Island, Norway, Oman, Palau, Palestine, Papua New Guinea,
Philippines, Republic of Korea,
Russia, Samoa, Singapore, Solomon Islands,Thailand, Tonga, Tuvalu, UAE, Ukraine,
USA, Vanuatu, Vietnam.
Visa on Arrival is not applicable to Diplomatic/Official Passports.

http://www.insightsonindia.com

INSIGHTS

Page 35

Insights Mock Tests 2015: Solutions Test - 16


100.

Solution: d)

Background of the National Statistical Commission (NSC)


A Commission set up by the Government in January 2000 under the Chairmanship
of Dr. C. Rangarajan reviewed the statistical system and the entire gamut of Official
Statistics in the country. The Rangarajan Commission submitted its report to the
Government in August 2001. One of the key recommendations of this Commission
was to establish a permanent National Commission on Statistics to serve as a nodal
and empowered body for all core statistical activities of the country, evolve, monitor
and enforce statistical priorities and standards and to ensure statistical co-ordination
among the different agencies involved. The Rangarajan Commission also
recommended that the Commission be set up initially through a Government order.
Rangarajan Commission Report
In line with the recommendations of the Rangarajan Commission, the Government
of India ordered the setting up of a permanent National Statistical Commission
(NSC) vide Notification No. 85 dated 1st June 2005published in the Gazette of India,
Extraordinary, Part-III-Section 4.

The NSC is mandated to perform the following functions, namely: a. to identify the core statistics, which are of national importance and are critical
to the development of the economy;

b. to constitute professional committees or working groups to assist the


Commission on various technical issues;

c. to evolve national policies and priorities relating to the statistical system;

d. to evolve standard statistical concepts, definitions, classifications and


methodologies in different areas in statistics and lay down national quality
standards on core statistics;

e. to evolve national strategies for the collection, tabulation and dissemination of


core statistics, including the release calendar for various data sets;
http://www.insightsonindia.com

INSIGHTS

Page 36

Insights Mock Tests 2015: Solutions Test - 16

f. to evolve national strategies for human resource development on official


statistics including information technology and communication needs of the
statistical system;

g. to evolve measures for improving public trust in official statistics;

h. to evolve measures for effective co-ordination with State Governments and


Union Territory Administrations on statistical activities including
strengthening of existing institutional mechanisms;

i. to exercise statistical co-ordination between Ministries, Departments and


other agencies of the Central Government;

j.

to exercise statistical audit over the statistical activities to ensure quality and
integrity of the statistical products;

k. to recommend to the Central Government, or any State Government, as the


case may be, measures to effectively implement the standards, strategies and
other measures evolved under clauses (c) to (h);

l. to advise the Government on the requirement of legislative measures on


statistical matters including the statute for the National Statistical
Commission; and
m. to monitor and review the functioning of the statistical system in the light of
the laid down policies, standards and methodologies and recommend
measures for enhanced performance.
Powers of the NSC
The Commission has the requisite autonomy to discharge its functions effectively
and efficiently. In particular, the Commission has the powers to: -

http://www.insightsonindia.com

INSIGHTS

Page 37

Insights Mock Tests 2015: Solutions Test - 16

require production of any document which in the opinion of the


Commission will serve or may serve statistical purposes;

require statistical agencies and institutions to provide details of


statistical activities, including concepts and definitions used,
methodologies followed, quality standards adopted, sampling and
non-sampling errors, etc. in respect of core statistics;

require attendance of any person including any public servant on


matters connected with core statistics; and

issuing notices for examination of witnesses and documents or any


matters connected with core statistics.

http://www.insightsonindia.com

INSIGHTS

Page 38

INSIGHTS ON INDIA MOCK PRELIMINARY EXAM - 2015


INSIGHTS ON INDIA MOCK TEST - 17
GENERAL STUDIES

PAPER-I
Time Allowed: 2 Hours

Maximum Marks: 200

INSTRUCTIONS
1. IMMEDITELY AFTER THE COMMENCEMENT OF THE EXAMINATION, YOU SHOULD
CHECK THAT THIS TEST BOOKLET DOES NOT HAVE ANY UNPRINTED OR TORN OR MISSING
PAGES OR ITEMS, ETC. IF SO, GET IT REPLACED BY A COMPLETE TEST BOOKLET.
2. You have to enter your Roll Number on the Test
Booklet in the Box provided alongside. DO NOT
Write anything else on the Test Booklet.
4. This Test Booklet contains 100 items (questions). Each item is printed only in English. Each item
comprises four responses (answers). You will select the response which you want to mark on the
Answer Sheet. In case you feel that there is more than one correct response, mark the response which
you consider the best. In any case, choose ONLY ONE response for each item.
5. You have to mark all your responses ONLY on the separate Answer Sheet provided. See directions in
the Answer Sheet.
6. All items carry equal marks.
7. Before you proceed to mark in the Answer Sheet the response to various items in the Test Booklet, you
have to fill in some particulars in the Answer Sheet as per instructions sent to you with your
Admission Certificate.
8. After you have completed filling in all your responses on the Answer Sheet and the examination has
concluded, you should hand over to the Invigilator only the Answer Sheet. You are permitted to take
away with you the Test Booklet.
9. Sheets for rough work are appended in the Test Booklet at the end.
10. Penalty for wrong answers :
THERE WILL BE PENALTY FOR WRONG ANSWERS MARKED BY A CANDIDATE IN THE
OBJECTIVE TYPE QUESTION PAPERS.
(i) There are four alternatives for the answer to every question. For each question for which a
wrong answer has been given by the candidate, one-third of the marks assigned to that
question will be deducted as penalty.
(ii) If a candidate gives more than one answer, it will be treated as a wrong answer even if one of
the given answers happens to be correct and there will be same penalty as above to that
question.
(iii)

If a question is left blank, i.e., no answer is given by the candidate, there will be no penalty
for that question.
http://www.insightsonindia.com

INSIGHTS ON INDIA MOCK TEST SERIES FOR CIVIL SERVICES PRELIMINARY EXAM 2015
http://www.insightsonindia.com

INSIGHTS

Page 1

1. The Constitution does NOT contain


any specific procedure for the selection
and appointment of the
1. Chief Minister
2. Governor
3. Prime Minister
4. President
Choose the correct answer using the codes
below.
a)
b)
c)
d)

1, 2 and 3
All of the above
1 and 2 only
3 and 4 only

2. Consider the following statements.


1. The Chief Minister (CM) may be a
member of any of the two Houses
of a state legislature.
2. The CM may be dismissed by the
Governor even if he enjoys
majority support in state legislature

4. Chairman, State Public Service


Commission
Choose the correct answer using the codes
below.
a)
b)
c)
d)

1, 2 and 3 only
1, 2 and 4 only
3 and 4 only
1 and 2 only

4. With India adopting a decentralized


and participatory model of planning,
state planning boards have become
more relevant. The Chairman of the
State Planning board is
a) Minister of Planning, State
Government
b) Planning Secretary of the State
c) Chief Secretary
d) Chief Minister

Which of the above is/are true?


a)
b)
c)
d)

1 only
2 only
Both 1 and 2
None

3. The Governor requires advice of the


CM in appointing which of the
following important officials?
1. State election Commissioner
2. Advocate General
3. Chief Justice of High Court

5. With reference to the Akali Movement,


consider the following statements:
1. The main objective of the
movement was to reform the Sikh
religion and clean it from the
influence of Hinduism
2. The Shiromani Akali Dal was the
central body that organized the
movement and accepted nonviolence as the soul of the
movement
Which of the above statements is/are correct?

http://www.insightsonindia.com

INSIGHTS

Page 2

a)
b)
c)
d)

1 Only
2 Only
Both
None

6. In relation to the Council of Ministers


(CoM) of Centre and State, which of
the following powers are enjoyed by
both the President and the Governor
respectively?
3. Sending a decision of the CoM for
re-consideration
4. Dismissing the CoM
5. Getting the CoM to furnish desired
information to Governor/President

8. The Constitution provides for the


abolition or creation of legislative
councils in states. Accordingly, the
Parliament can abolish or create a
legislative council in a state when
1. The state assembly passes the
specific resolution with special
majority
2. It is passed in the Parliament by
ordinary majority.
3. A prior recommendation has been
made by the President.
Choose the correct answer using the codes
below.
a)
b)
c)
d)

Choose the correct answer using the codes


below.
a)
b)
c)
d)

1 and 2 only
2 and 3 only
1 and 3 only
All of the above

7. Article 163 provides for a council of


ministers with the chief minister at the
head to aid and advise the governor in
the exercise of his functions except the
discretionary ones. If any question
arises whether a matter falls within the
governors discretion or not, the final
decision lies with the
a) Governor
b) Chief Minister
c) State High Court
d) President

http://www.insightsonindia.com

1 and 2 only
2 and 3 only
1 and 3 only
All of the above

9. The size of the state council of


ministers and the ranking of ministers
is specified and determined by the
a) Constitution of India
b) Chief Minister
c) Governor
d) Laws enacted by the State
Legislative Assembly

10. Consider the following statements


about the legislative council in a state.
1. Its strength is fixed by the
Parliament.
2. The size of the council depends on
size of the assembly of the
concerned state.
INSIGHTS

Page 3

3. None of its members are directly


elected.
Choose the correct answer using the codes
below.
a)
b)
c)
d)

1 and 2 only
2 and 3 only
1 and 3 only
All of the above

13. The Chairman of the State Legislative


Council
1. Has to be a member of the Council
for being eligible
2. Elected by the Council
3. Appointment is approved by the
Governor
Choose the correct answer using the codes
below.

11. The famous Keys Affair during the


freedom struggle is related to:
a) The handover of the keys of the
Toshakhana of the Golden Temple
by the British to the Akalis
b) The Killing of the British officer
John Keys by the Akalis
c) The transfer of the corrupt British
officer John Keys from Amritsar
after stiff agitation by the Akalis
d) None of the above

12. In the election to the Legislative


Council of a State, which of the
following participate?
1. Municipalities
2. Members of Legislative Assembly
of the State
3. Important District Functionaries
Choose the correct answer using the codes
below.
a)
b)
c)
d)

1 and 2 only
2 and 3 only
1 and 3 only
All of the above

http://www.insightsonindia.com

a)
b)
c)
d)

1 and 2 only
2 and 3 only
1 and 3 only
All of the above

14. Consider the position with respect to


lapsing of bills on the dissolution of
the assembly.
1. A Bill passed by the assembly or
passed by both the houses but
pending assent of
the
governor or the President does not
lapse.
2. A Bill passed by the assembly but
pending in the council lapses.
3. A Bill pending in the council but
not passed by the assembly does
not lapse.
Choose the correct answer using the codes
below.
a)
b)
c)
d)

INSIGHTS

1 and 2 only
2 and 3 only
1 and 3 only
All of the above

Page 4

15. When a bill passed by the legislative


assembly transmitted to the legislative
council is rejected by it, then which of
the following is possible?
a) The assembly will pass the bill
again and the bill will be deemed to
be passed by the legislature.
b) The assembly will pass the bill
again, transmit it to the council and
only after the assent of the council
will the bill be deemed to be passed
by the legislature.
c) The assembly will pass the bill
again, transmit it to the council and
even without the assent of the
council will the bill be deemed to
be passed by the legislature.
d) The Governor will have to call a
joint sitting of both the houses.

16. With reference to the Vaikom


Satyagraha, consider the following
statements
1. E.V. Ramaswami Naicker was its
prominent leader
2. The demand of the movement to
seek entry to the temple for avarnas
(depressed classes)

17. Consider the following about the State


Legislature.
1. A Money Bill cannot be introduced
in the legislative council.
2. Money bills can be introduced only
on the recommendation of the
Governor.
3. The Governor cannot reserve a
money bill for the consideration of
the President.
Choose the correct answer using the codes
below.
a)
b)
c)
d)

18. In which of the following matters, the


powers and status of the council are
broadly equal to that of the assembly?
1. Approval of ordinances issued by
the governor.
2. Consideration of the reports of the
constitutional bodies.
3. Selection of ministers including the
chief minister.
Choose the correct answer using the codes
below.
a)
b)
c)
d)

Which of the above statements is/are correct?


a)
b)
c)
d)

1 Only
2 Only
Both
None

http://www.insightsonindia.com

1 and 2 only
2 and 3 only
1 and 3 only
All of the above

INSIGHTS

1 and 2 only
2 and 3 only
1 and 3 only
All of the above

Page 5

19. Concerning the High Court, which of


the following is done by the President?
1. Determining the Strength of High
Court
2. Appointing the Judges
3. Removal of Judges

Choose the correct answer using the codes


below.
a)
b)
c)
d)

1 only
2 and 4 only
1 and 3 only
1 and 4 only

Choose the correct answer using the codes


below.
a)
b)
c)
d)

1 and 2 only
2 and 3 only
1 and 3 only
All of the above

20. The Constitution has made which of


the following provisions to safeguard
and ensure the independent and
impartial functioning of a high court?
1. Conduct of Judges cannot be
discussed in state legislature
2. Their expenses are charged on
Consolidated Fund of India
3. Security of Tenure

22. The High courts are also called as


courts of record. It means that
1. The judgements, proceedings and
acts of the high courts are recorded
for
testimony.
2. The Judgements of HCs are
recognized as legal precedents.
3. The acts of the HCs cannot be
questioned if produced before any
subordinate
court in India.
Choose the correct answer using the codes
below.
a)
b)
c)
d)

Choose the correct answer using the codes


below.
a)
b)
c)
d)

1 and 2 only
2 and 3 only
1 and 3 only
All of the above

21. A high court does NOT enjoy which of


the following jurisdiction and powers?
1. Advisory Jurisdiction
2. Writ jurisdiction
3. Supervisory Jurisdiction
4. Original Jurisdiction

http://www.insightsonindia.com

1 and 2 only
2 and 3 only
1 and 3 only
All of the above

23. Judicial review is one the powers of a


high court. Under Judicial review
constitutionality of which of the
following can be examined by the HC?
1. Legislative enactments of
Parliament
2. Executive orders of State
governments
3. Executive orders of Central
government
Choose the correct answer using the codes
below.
INSIGHTS

Page 6

a)
b)
c)
d)

1 and 2 only
2 and 3 only
1 and 3 only
All of the above

24. With reference to the temple entry


movement and the Gandhian or
nationalist approach in fighting caste
oppression during the national
movement, consider the following
statements:
1. The Indian National Congress
lacked a clear strategy in ending
the caste system
2. The growth of casteism and the
continuous existence in practice of
oppression and discrimination
against the lower castes in post1947 India can be attributed to
failure of national movement in
ending this practice

Choose the correct answer using the codes


below.
a)
b)
c)
d)

1 and 2 only
2 and 3 only
1 only
None of the above

26. The appointment, posting and


promotion of district judges in a state
are made by the
a) Governor in consultation with the
High Court
b) Chief Justice of the concerned High
Court
c) Governor in consultation with the
High Court and the State Public
Service Commission
d) Chief Justice of the concerned High
Court in consultation with a
collegium of senior-most judges

Which of the above statements is/are correct?


a)
b)
c)
d)

1 Only
2 Only
Both
None

25. Consider the following statements.


1. The Constitution of India does not
recognize subordinate courts.
2. The Independence of Subordinate
courts from the executive is
secured via State laws.
3. Only the High Court, not the
District court, can exercise
superintendence over the lower
courts.
http://www.insightsonindia.com

27. Consider the following statements


about Jammu and Kashmir (J&K).
1. An amendment made to the
Constitution of India does not
apply to the state unless it is
extended by a presidential order.
2. It is the only state in the Indian
Union which has its own separate
state Constitution.
3. The President has no power to
suspend the Constitution of the
state on the ground of failure to
comply with the directions given
by him.
Choose the correct answer using the codes
below.
INSIGHTS

Page 7

a)
b)
c)
d)

1 and 2 only
2 and 3 only
1 and 3 only
All of the above

28. Which of the following are the


financial services offered by the Post
Offices in india?
1. Savings Bank account (SB).
2. Recurring Deposit account (RD).
3. Monthly Income Scheme (MIS).
4. Public Provident Fund (PPF).
5. Senior Citizens Savings Scheme
(SCSS).
Choose the correct answer using the codes
below:
a)
b)
c)
d)

Only 1.
Only 1, 2 , 3 & 5.
Only 2, 3, 4 & 5.
All.

29. Consider the following statements


regarding the National e-Governance
Plan (NeGP):
1. NeGP has been formulated by the
Department of Electronics and
Information Technology (DeitY)
and Department of Administrative
Reforms and Public Grievances
(DARPG).
2. Under the Plan, National e
governance fund has been created
under the Ministry of Finance to
provide funds at concessional rates.
3. There are 31 Mission Mode Projects
under NeGP.

Which of the above statements are true?


a)
b)
c)
d)

Only 1.
Only 2.
Only 1 & 3.
None.

30. MeghRaj Project is:


a) a Weather predicting application
prepared by the ISRO.
b) An app created by the Delhi
government for the safety of
women.
c) a new scheme for the development
North Eastern Areas.
d) a new project related to Cloud
Computing.

31. Consider the following statements


regarding the National Optical Fibre
Network:
1. It is a project to provide broadband
connectivity to over two lakh Gram
panchayats of India.
2. The project provides internet access
using existing optical fibre and
extending it to the Gram
panchayats.
3. A special purpose vehicle Bharat
Broadband Network Limited
(BBNL) is created as a Public Sector
Undertaking (PSU) under the
Companies Act of 1956 for the
execution of the project.
4. The project will be funded by the
Universal Service Obligation Fund
(USOF).
Which of the above statements are true?

http://www.insightsonindia.com

INSIGHTS

Page 8

a)
b)
c)
d)

Only 1.
Only 2, 3 & 4.
Only 2 & 4.
All.

32. Consider the following statements


regarding the Cyber Appellate
Tribunal:
1. It has been established under the
provisions of the Information
Technology Act, 2000.
2. It is the first and the only Cyber
Appellate Tribunal in the country.
3. The Cyber Appellate Tribunal has
powers to regulate its own
procedure including the place at
which it has its sittings.
Which of the above statements are true?
a)
b)
c)
d)

Only 1.
Only 2 & 3.
Only 1 & 3.
All.

33. Consider the following statements


regarding the Indian Computer
Emergency Response Team (CERT-In):
1. It is the Government organization
under Ministry of Communications
and Information Technology.
2. It is a nodal agency that deals with
cyber security threats like hacking
and phishing.

34. Governors rule is a unique feature of


the J&K polity. It is imposed when
a) The Central Government imposes
Presidents rule in the state
b) There is National emergency or
internal disturbance
c) The state administration is not
carried in accordance with the
provisions of the J&K constitution
d) The President is satisfied that the
state government is incapable of
administering the state and hands
over the governance to the
Governor of the state

35. The State of Nagaland is a special state


as per the Indian constitution.
Nagaland has relative political
autonomy in deciding which of the
following matters?
1. Administration of Civil and
Criminal Justice
2. Ownership of land and its transfer
3. Customary law and its practice
Choose the correct answer using the codes
below.
a)
b)
c)
d)

1 and 2 only
2 and 3 only
1 and 3 only
All of the above

Which of the above statements are true?


a)
b)
c)
d)

Only 1.
Only 2.
Both.
None.

http://www.insightsonindia.com

INSIGHTS

Page 9

36. Which of the following were the


implications of the 73 rd Constitutional
Amendment?
1. Constituting Panchayats became
mandatory for states.
2. Panchayats were brought under the
justiciable part of the Constitution.
3. The Structure of Panchayats across
the country was made uniform and
harmonised.
Choose the correct answer using the codes
below.
a)
b)
c)
d)

1 and 2 only
2 and 3 only
1 and 3 only
All of the above

37. As per the Constitution, every


Panchayat should have a Gram Sabha.
A Gram Sabha is a body consisting of
a) The whole population of the
villages under the Panchayat other
than Children of less than five
years
b) The adult population of the villages
under the Panchayat
c) The registered voters of the villages
under the Panchayat
d) All the Village committees
constituted by the Panchayat

38. Which of the following bodies were


constituted in pursuance of the 73 rd
and 74th constitutional amendment?
1. State Finance Commission
2. State Election Commission
3. District Planning Committees
http://www.insightsonindia.com

4. District Rural Development


Agency
Choose the correct answer using the codes
below.
a)
b)
c)
d)

1 and 2 only
2, 3 and 4 only
1, 3 and 4 only
1, 2 and 3 only

39. As per the Constitution, Which of the


following functional items can be kept
within the purview of the Panchayats?
1. Public Distribution System
2. Welfare of Weaker Sections
3. Primary Health Centres
4. Adult and non-formal education
Choose the correct answer using the codes
below.
a)
b)
c)
d)

1 and 2 only
2, 3 and 4 only
1, 3 and 4 only
All of the above

40. Consider the following provisions


under the 73rd amendment.
1. Direct elections to all seats in
panchayats at the village,
intermediate and district levels.
2. Indirect elections to the post of
chairperson of panchayats at the
intermediate and district levels.
3. Providing reservation of seats for
backward classes in panchayats at
any level

INSIGHTS

Page 10

the Scheduled Areas shall be


coordinated at the Panchayat level
3. For the exploitation of minor
minerals prior recommendation of
the Gram Sabha or the Panchayat is
needed.

Under the 73rd amendment, these above


provisions are
a) Compulsory for most states
b) Voluntary for most states
c) Some are compulsory, some
voluntary for states
d) Dependent on whether the state is
a special category state or not.

41. Provisions of the Panchayats


(Extension to the Scheduled Areas)
Act, 1996, popularly known as the
PESA Act or the Extension Act has
which of the following objectives?
1. To provide self-rule for the bulk of
the tribal population
2. To evolve a suitable administrative
framework consistent with
traditional practices
3. To safeguard and to preserve the
traditions and customs of tribal
communities

Choose the correct answer using the codes


below.
a)
b)
c)
d)

43. At the Central level, the subject of


urban local government is dealt with
by which of the following ministries?
1. Ministry of Home Affairs
2. Ministry of Urban Development
3. Ministry of Defence
Choose the correct answer using the codes
below.

Choose the correct answer using the codes


below.
a)
b)
c)
d)

1 and 2 only
2 and 3 only
1 and 3 only
All of the above

42. As per the PESA Act


1. Acquisition of land requires
consultation with the concerned
Gram Sabha
2. Actual planning and
implementation of the projects in

http://www.insightsonindia.com

1 and 2 only
2 and 3 only
1 and 3 only
All of the above

a)
b)
c)
d)

1 and 2 only
2 and 3 only
1 and 3 only
All of the above

44. The institutions of urban local


government originated and developed
in modern India during the period of
British rule. Which of the following
substantiate the argument?
1. Local self-government was
declared a provincial subject under
the Government of
India Act of
1935.

INSIGHTS

Page 11

2. Lord Ripons Resolution of 1882


has been hailed as the Magna
Carta of local self-government.
3. First panchayats and municipal
corporations in India were setup
under the British.

3. Members of State Legislative


Assembly
Choose the correct answer using the codes
below.
a)
b)
c)
d)

Choose the correct answer using the codes


below.
a)
b)
c)
d)

1 and 2 only
2 and 3 only
1 and 3 only
All of the above

45. Consider the following statements:


1. The Congress-Khilafat Swaraj Party
was advocated that the nationalists
should end the boycott of the
legislative councils and enter them
to expose them as sham
parliaments
2. The Swarajists came to be known
as no-changers
Which of the above statements is/are correct?
a)
b)
c)
d)

1 Only
2 Only
Both
None

46. The representation of the following


persons in a municipality can be
provided for by the State Legislature?
1. Persons having special knowledge
or experience in municipal
administration
2. Members of the Lok Sabha of that
constituency
http://www.insightsonindia.com

1 and 2 only
2 and 3 only
1 and 3 only
All of the above

47. Executive directors in Public Sector


banks are appointed
a) By Appointments Committee of the
cabinet on the recommendation of
the RBI Governor
b) By Appointments Committee of the
cabinet on the recommendation of a
selection panel headed by the RBI
Governor
c) By a selection panel headed by the
RBI Governor
d) By the Appointments Committee of
the Cabinet on the
recommendation of Department of
Personnel and Training

48. Consider the following statements


about the Attorney General.
1. He is a constitutional authority.
2. He comes under the purview of the
RTI Act.
3. He is appointed by the Chief Justice
of India.
Choose the correct answer using the codes
below.

INSIGHTS

Page 12

a)
b)
c)
d)

49. The term Blue Economy in Indias


MOUs with other countries refers to
a) Ocean based economy
b) Electronics based economy
c) An economy that gives greater
priority to bluechip companies
d) Fisheries based economy

50. A recent amendment to the Land


Acquisition Act removes the Consent
Clause of 80 per cent of land owners
for private projects and that of 70 per
cent of land owners for PPP projects in
case of acquiring land for which of the
following categories of land usage?
1. Rural Infrastructure
2. Industrial Corridors
3. Affordable Housing
Choose the correct answer using the codes
below.
a)
b)
c)
d)

1. It does not consume any


conventional fuel and runs solely
on solar energy.
2. It does not emit any Carbon
Dioxide.
3. It cannot fly at night.

1 and 2 only
2 and 3 only
1 and 3 only
All of the above

1 and 2 only
2 and 3 only
1 and 3 only
All of the above

51. Consider the following statements


about the Swiss long-range solarpowered aircraft project Solar Impulse
2.
http://www.insightsonindia.com

Choose the correct answer using the codes


below.
a)
b)
c)
d)

1 and 2 only
2 and 3 only
1 and 3 only
All of the above

52. Who formulates the Foreign Trade


Policy of India?
a) Department of Commerce.
b) Department of Economic Affairs.
c) Department of Industrial policy &
promotion.
d) Department of Publication.

53. Consider the following statements


regarding the Special Economic Zones
in India:
1. A foreign agency cannot set up
SEZs in India.
2. Currently, Tamil Nadu has the
second highest number of
operational SEZs in India.
3. Presently, there is no minimum
land requirement for setting up of
an IT/ITES SEZ in India.
Which of the above statements are true?
INSIGHTS

Page 13

a)
b)
c)
d)

54. Consider the following statements


regarding the Zero Duty Export
Promotion Capital Goods Scheme:
1. The scheme allows import of
capital goods on zero duty for pre
production, production and post
production without any subject to
an export obligation.
2. Spares for the existing plant and
machinery may also be imported
under the EPCG scheme.
3. Under the Scheme, EPCG
authorization holder cannot export
through a third party.

Which of the above statements are true?


a)
b)
c)
d)

a)
b)
c)
d)

Only 1.
Only 2 & 3.
Only 3.
None.

Only 1.
Only 2 & 3.
Only 2.
All.

55. Which of the following departments


come under the Ministry of
Information and Technology?
1. Department of Posts.
2. Department of
Telecommunications.
3. Department of Information
Technology.

56. Consider the following statements:


1. The modern postal system in India
was established by Lord Clive in
the year 1766.
2. The statute presently governing the
postal services in India is the
Indian Post Office Act, 1898.
Which of the above statements are true?
a)
b)
c)
d)

Only 1.
Only 2.
Both.
None.

57. Maharashtra government has


launched Bhagyashree scheme for
girls. Consider the following about it.
1. It is for a girl child born in a BPL
family.
2. After the girl completes 18 years of
age, the government will give
provide a certain maturity amount
to the family.
3. It has not been linked with the Beti
Bachao Beti Padhao Scheme of the
central Government.
Choose the correct answer using the codes
below.
a)
b)
c)
d)

Choose the correct answer using the codes


below:
http://www.insightsonindia.com

Only 1.
Only 2 & 3.
Only 1 & 3.
All.

INSIGHTS

1 and 2 only
2 and 3 only
1 and 3 only
All of the above
Page 14

58. North East Indias first Kisan Call


Centre (KCC) was opened in Agartala,
Tripura. A Kisan Call centre facilitates
information in which of the following?
1. Seed Procurement
2. Marketing of agricultural produce
3. Fertilizers and Soil Heath

60. NASAs Dawn space probe will help


in knowing
a) The limits of the Solar system
b) About the Solar systems
beginnings
c) All the unreached dwarf planets in
the solar system
d) The asteroids orbiting between
Mars and Jupiter

Choose the correct answer using the codes


below.
a)
b)
c)
d)

1 and 2 only
2 and 3 only
1 and 3 only
All of the above

59. Consider the following statements


about the Rotavirus vaccine- Rotavac
launched recently by the Union
Government.
1. It was developed under the joint
collaboration between India and
the United States.
2. It will help fight Diarrhoea.
3. This is the first vaccine available
globally against Rotavirus.
4. It is Indias first indigenously
developed rotavirus vaccine.
Choose the correct answer using the codes
below.
a)
b)
c)
d)

1, 2 and 4 only
2 and 4 only
2 and 3 only
1 and 4 only

http://www.insightsonindia.com

61. The state of Jammu & Kashmir is


culturally rich. Dej-Hor found in the
state is
a) an ornament worn by married
Kashmiri Pandit women
b) a festival celebrated by rural
Kashmiri Pandits
c) a place of worship for the Kashmiri
Pandits
d) the name of an idol worshipped by
Kashmiri Pandits

62. Thermal Plasma Technology


developed by BARC to fulfil the
Swacha Bharat Abhiyan is ideally
suited for the treatment of
1. Solid Waste
2. Liquid waste
3. Gaseous pollutants
Choose the correct answer using the codes
below.
a)
b)
c)
d)

INSIGHTS

1 and 2 only
2 and 3 only
1 and 3 only
1 only

Page 15

63. Environmental isotope techniques is


emerging as an important tool in the
water resources development and
management in the country. It can be
used for
1. Assessing groundwater salinity
2. Assess contamination in surface
water
3. Evaluate changes due to long-term
exploitation of groundwater
Choose the correct answer using the codes
below.
a)
b)
c)
d)

66. Refuse Derived Fuel (RDF) is an


emerging technology in solid waste
management. Consider the following
about it.
1. It is a processed form of Municipal
Solid Waste
2. It can act as a substitute for coal
energy.
3. It does not emit any carbon
dioxide.
Choose the correct answer using the codes
below.
a)
b)
c)
d)

1 and 2 only
2 and 3 only
1 and 3 only
All of the above

64. Which of the following leader was


NOT a swarajist?
a) Motilal Nehru
b) C. Rajagopalachari
c) C.R. Das
d) None of the above

65. AKRUTI ( Advance Knowledge of


Rural Technology Implementation)
Program run by Bhabha Atomic
Research Centre (BARC) is related
with
a) Solid waste management in rural
areas
b) Safe drinking water in rural areas
c) Stopping contamination of Soil in
rural areas
d) None of the above
http://www.insightsonindia.com

1 and 2 only
2 and 3 only
1 and 3 only
All of the above

67. The World Health Organization


(WHO) has started a unique initiative
First Embrace. It focuses on
a) Reducing new born and infant deaths
b) Reducing maternal deaths
c) Reducing diseases caused due to lack
of vaccination in African nations
d) Cutting down on the level of antibiotics to reduce the anti-bacterial
resistance
68. Consider the following statements
about Monkey fever reported to
spread in Kerala recently.
1. Large animals such as cattle play a
major role in the transmission of
the virus.
2. Those who visit forests regularly
are at a greater risk than those who
dont.
INSIGHTS

Page 16

c) 1, 3 and 4 only
d) 1, 2 and 3 only

3. The virus is transmitted to humans


by the bite of a tick.
Choose the correct answer using the codes
below.
a)
b)
c)
d)

1 and 2 only
2 and 3 only
1 and 3 only
All of the above

69. A report, published in the Journal of


Dental Research, says that more than
2.4 billion people across the world
have untreated tooth decay. India is
also becoming a Dental Tourism spot.
Which of these Hill stations in India is
known for dental tourism?
a) Idduki
b) Ponmudi
c) Ooty
d) Connor

70. The Patna High Court has recently


directed the Union government and
Bihar state government to restore the
original course of the Ganga in the
state capital. Which of the following
have been cited as causes for the
course diversion?
1. Illegal Sand Mining
2. Dredging
3. River Inter-linking Project
4. Climate Change

71. In the managed exchange rate system,


what are the ways in which the
exchange rate can be affected by the
Government or Central Bank?
1. Buying or Selling Foreign
Currencies
2. Raising the caps on Foreign Direct
Investment
3. Raising Interest rates on Foreign
Currency bank accounts
Choose the correct answer using the codes
below.
a)
b)
c)
d)

1 and 2 only
2 and 3 only
1 and 3 only
All of the above

72. Current Account of the External Sector


of the Indian economy includes which
of the following?
1. Export of Services from India
2. Foreign Aid
3. Private Remittances
Choose the correct answer using the codes
below.
a)
b)
c)
d)

Choose the correct answer using the codes


below.

1 and 2 only
2 and 3 only
1 and 3 only
All of the above

a) 1 and 2 only
b) 2 and 3 only
http://www.insightsonindia.com

INSIGHTS

Page 17

c) Weighted average of exchange


rates before the currencies of all of
Indias trading partners.
d) Simple average of exchange rates
before the currencies of all of
Indias trading partners.

73. A Balance of Payments (BoP) crisis is a


situation when
1. The forex reserves are not capable
of financing the BoP
2. Inflow of foreign funds and
investments is not bridging the BoP
Choose the correct answer from the codes
below.
a) 1 alone happens
b) 2 alone happens
c) Both 1 and 2 happen
simultaneously
d) Either of 1 or 2 happen

74. The issue of currency convertibility is


concerned with foreign currency
1. Outflow
2. Inflow
3. Investments

76. With reference to the Swarajist activity


in the legislatures, consider the
following statements:
1. The Swarajists lacked any policy of
coordinating their militant work in
the legislatures with mass political
work outside
2. Its activities inside legislatures
helped the Congress to wing many
municipal elections during 1923-24
Which of the above statements is/are correct?
a)
b)
c)
d)

Choose the correct answer using the codes


below.
a)
b)
c)
d)

1 alone
2 alone
3 alone
Both 2 and 3

75. The Nominal Effective Exchange Rate


(NEER) of the rupee is a
a) Weighted average of exchange
rates before the currencies of
Indias major trading partners.
b) Simple average of exchange rates
before the currencies of Indias
major trading partners.

http://www.insightsonindia.com

1 Only
2 Only
Both
None

77. A Hard currency is the one which:


1. Has a high level of liquidity
2. Belongs to the country with the
highest as well as highly
diversified exports
3. Is needed by every open economy
Choose the correct answer using the codes
below.
a)
b)
c)
d)

INSIGHTS

1 and 2 only
2 and 3 only
1 and 3 only
All of the above

Page 18

78. Special Economic Zones (SEZs) are


essentially industrial clusters which
are
1. Duty free enclaves
2. Treated as Foreign territory for all
purposes
3. Meant largely for exports
Choose the correct answer using the codes
below.
a)
b)
c)
d)

1 and 2 only
2 and 3 only
1 and 3 only
All of the above

79. The GAAR (General Anti-Avoidance


Rules), originally proposed in the
Direct Taxes Code 2010 , are targeted
at arrangements or transactions made
specifically to avoid taxes. The
objective of the GAAR provisions is to
codify the doctrine of substance over
form which means that
a) real intention of the parties and
purpose of an arrangement is
taken into account for
determining the tax consequences
b) Only those arrangements be
questioned which create rights
and obligations that are not at
arms length
c) Those transactions be questioned
which lack commercial substance
or are deemed to lack commercial
substance
d) Transactions will be questioned
irrespective of the Intention of the
parties and purpose of the
http://www.insightsonindia.com

agreement, if tax avoidance is


even one of the purposes of the
agreement

80. A High Current Account Deficit has


implications for
1. Rupee Volatility
2. Business Confidence in the
Economy
3. Stability of Macro-economic
indicators in the economy
Choose the correct answer using the codes
below.
a)
b)
c)
d)

1 and 2 only
2 and 3 only
1 and 3 only
All of the above

81. Foreign Direct Investment (FDI) is


preferred to the foreign portfolio
investments (PIS) because
1. FDI has a long-term nature of
investment.
2. It is not subject to immediate and
spontaneous fluctuations in the
market economy.
3. PIS does not bring benefits like
shared technology and managerial
practices.
Choose the correct answer using the codes
below.
a)
b)
c)
d)

INSIGHTS

1 and 2 only
2 and 3 only
1 and 3 only
All of the above

Page 19

82. Which of the following are NOT the


functions of the International
Monetary Fund (IMF)?
1. To facilitate international monetary
cooperation
2. To promote exchange rate stability
3. To assist in the establishment of a
multilateral system of payments

3. Interest Tax
4. Corporate Tax
Choose the correct answer using the codes
below.
a)
b)
c)
d)

Choose the correct answer using the codes


below.
a)
b)
c)
d)

1 and 2 only
2 only
3 only
All those mentioned are functions
of IMF.

83. What was common between the


following leaders of the national
movement?
a) Surya Sen
b) Jogesh Chandra Chatterjea
c) Bhagwati Charan Vohra
d) Chandrashekhar Azad
Choose the correct answer from the below
options:
a) All of them were from Bengal
b) All of them were part of nonviolent struggle during initial days
c) Except Bhagawati Chandra Vohra,
all were revolutionary terrorists
d) None of the above

85. VAT does not have a cascading effect


on the prices of goods and it does not
increase inflation. It is because
a) Tax is imposed and collected at
different points of value addition
chain
b) Tax is imposed and collected at one
point of the value addition chain
c) VAT is direct tax and is passed on
to the consumers
d) Both (a) and (c)

86. Consider the following statements


about the Goods and Services (GST)
Tax.
1. It will be levied based on the VAT
model.
2. It will be imposed all over the
country with the uniformity of rate.
3. It will subsume all Direct and
Indirect taxes.
Choose the correct answer using the codes
below.
a)
b)
c)
d)

84. Which of the following are examples


of Indirect taxation in the country?
1. Excise Duty
2. Sales Tax
http://www.insightsonindia.com

1 and 2 only
3 and 4 only
1, 2 and 3 only
All of the above

INSIGHTS

1 and 2 only
2 and 3 only
1 and 3 only
All of the above
Page 20

87. Commodities Transaction Tax (CTT)


was introduced for non-agricultural
commodity futures last year. Which of
the following are the objectives of
CTT?
1. Discouraging excessive speculation
2. Bring parity between Security
market and commodities market
3. Discouraging price discovery of the
underlying commodity
Choose the correct answer using the codes
below.
a)
b)
c)
d)

1 and 2 only
2 and 3 only
1 and 3 only
All of the above

88. Capital gains tax is a direct tax applied


on the profit made by the owner of an
asset. It can be applied on
1. Mutual Funds
2. Shares
3. Corporate Bonds
Choose the correct answer using the codes
below.
a)
b)
c)
d)

1 and 2 only
2 and 3 only
1 and 3 only
All of the above

89. The Government of India can borrow


long-term capital from which of the
following sources?
1. Foreign banks
2. Foreign Financial Institutions other
than banks
3. IMF
http://www.insightsonindia.com

Choose the correct answer using the codes


below.
a)
b)
c)
d)

1 and 2 only
2 and 3 only
1 and 3 only
All of the above

90. Consider the following statements


regarding the universal Service
Obligation Fund (USOF):
1. The fund is created under
department of telecommunications,
Ministry of communication and
information technology.
2. It is a non-lapsable fund.
3. The Fund has a statutory support
under Indian Telegraph
(amendment) act 2003.
Which of the above statements are true?
a)
b)
c)
d)

Only 1.
Only 2 & 3.
Only 3.
All.

91. Consider the following statements


regarding the Telecom Regulatory
Authority of India (TRAI):
1. It is a statutory body.
2. The Head of the authority should
either be a retired Supreme Court
Judge or a retired High Court Chief
Justice.
Which of the above statements are true?
a)
b)
c)
d)
INSIGHTS

Only 1.
Only 2.
Both.
None.
Page 21

92. Consider the following statements


regarding the Exclusive Economic
Zones:
1. It is a sea zone prescribed by the
United Nations Convention on the
Law of the Sea.
2. A country has special rights over
these zones regarding the
exploration and use of marine
resources, including energy
production from water and wind.
3. It stretches from the baseline out to
350 nautical miles (nmi) from the
coast.

94. Consider the following statements


regarding Nirbhay Missile, which was
recently tested:
1. It is India`s first indigenously
designed and developed long range
sub-sonic cruise missile.
2. The missile has a range of more
than 1000 km.
3. It is capable of being launched from
multiple platforms on land, sea and
air.
Which of the above statements are true?
a)
b)
c)
d)

Which of the above statements are true?


a)
b)
c)
d)

Only 1.
Only 2 & 3.
Only 1 & 2.
All.

93. Consider the following statements


regarding One Rank One Pension
Scheme:
1. This Scheme will ensure that
soldiers of the same rank and the
same length of service receive the
same pension, irrespective of their
retirement date.
2. The Scheme has already been
implemented by Indian Navy.

95. Consider the following statements


regarding Agni-V missile, which was
recently tested:
1. It has a strike range of over 5000
kms.
2. It can carry a nuclear warhead.
3. It can be configured to launch small
satellites.
4. It can be used to shoot down
enemy satellites in orbits.
Which of the above statements are true?
a)
b)
c)
d)

Which of the above statements are true?


a)
b)
c)
d)

Only 1.
Only 2.
Both.
None.

http://www.insightsonindia.com

Only 1.
Only 2.
Only 1 & 3.
All.

Only 1.
Only 2 & 3.
Only 1, 2 & 4.
All.

96. The capital expenditure of the


Government of India includes which
of the following?

INSIGHTS

Page 22

1. Loans forwarded by the GoI to the


PSUs
2. Loans to World Bank
3. Capital expenses to modernize the
defense forces
4. Interest payments received on
loans forwarded by GoI in the past
Choose the correct answer using the codes
below.
a)
b)
c)
d)

1, 2 and 3 only
2 and 4 only
1, 3 and 4 only
All of the above

97. Monetised deficit is a term being used


in India as a sign of fiscal prudence. It
is
a) The fiscal deficit minus the interest
liabilities
b) The part of fiscal deficit provided
by the RBI to the government
c) The part of fiscal deficit financed
from Internal borrowing
d) The part of revenue deficit financed
from Internal borrowing
98. Among the following, which is the
best way of deficit financing keeping
the health of macro-economic
indicators in mind?
a) External Borrowing
b) External Grants
c) Borrowing from RBI
d) Printing Currency

http://www.insightsonindia.com

99. Mundells impossible trinity is a


trilemma which recently has gained
substantial relevance in Economics.
Which of the following are part of this
trilemma?
1. Free capital flows
2. Fixed exchange rate
3. An Independent Monetary policy
Choose the correct answer using the codes
below.
a)
b)
c)
d)

1 and 2 only
2 and 3 only
1 and 3 only
All of the above

100.
Comparative Rating Index of
Sovereigns' (CRIS) on the basis of
sovereign ratings of various countries
is an innovation by the Ministry of
Finance, India. If India moves up on
the index, it means that
1. India has become a better
investment destination.
2. India has done better in its debt
ratings than other nations even if
its overall
absolute rating
has gone down.
Which of the above is/are true?
a)
b)
c)
d)

INSIGHTS

1 only
2 only
Both 1 and 2
None

Page 23

http://www.insightsonindia.com

INSIGHTS

Page 24

Insights Mock Test 2015: Test 17 Solutions

Q. No.

Solution

Q. No.

Solution

Q. No.

Solution

Q. No.

Solution

1
2
3
4
5
6
7
8
9
10
11
12
13
14
15
16
17
18
19
20
21
22
23
24
25

A
A
B
D
B
D
A
A
B
D
A
A
A
D
C
C
A
d
D
C
A
D
D
C
D

26
27
28
29
30
31
32
33
34
35
36
37
38
39
40
41
42
43
44
45
46
47
48
49
50

A
D
D
C
D
D
D
C
C
D
D
C
D
D
C
D
D
D
A
A
D
B
A
A
D

51
52
53
54
55
56
57
58
59
60
61
62
63
64
65
66
67
68
69
70
71
72
73
74
75

A
A
B
C
D
C
D
D
A
B
A
D
D
C
B
A
A
B
B
A
D
D
C
A
A

76
77
78
79
80
81
82
83
84
85
86
87
88
89
90
91
92
93
94
95
96
97
98
99
100

C
D
C
A
D
D
A
B
A
A
D
A
D
D
D
A
C
A
D
D
A
B
B
D
C

1. Solution: a)
The Constitution does not contain any specific procedure for the selection and
appointment of the Chief Minister. Article 164 only says that the Chief Minister shall
be appointed by the governor. However, this does not imply that the governor is free
to appoint any one as the Chief Minister. In accordance with the convections of the
parliamentary system of government, the governor has to appoint the leader of the
majority party in the state legislative assembly as the Chief Minister.
The same goes for the PM. Governors appointment and selection too is not detailed
in the constitution.

2. Solution: a)

http://www.insightsonindia.com

INSIGHTS

Page 1

Insights Mock Test 2015: Test 17 Solutions


A person who is not a member of the state legislature can be appointed as Chief
Minister for six months, within which time, he should be elected to the state
legislature, failing which he ceases to be the Chief Minister. According to the
Constitution, the Chief Minister may be a member of any of the two Houses of a
state legislature. Usually Chief Ministers have been selected from the Lower House
(legislative assembly), but, on a number of occasions, a member of the Upper House
(legislative council) has also been appointed as Chief Minister.
The term of the Chief Minister is not fixed and he holds office during the pleasure of
the governor. However, this does not mean that the governor can dismiss him at any
time. He cannot be dismissed by the governor as long as he enjoys the majority
support in the legislative assembly.6 But, if he loses the confidence of the assembly,
he must resign or the governor can dismiss him.

3. Solution: b)
The Chief Minister enjoys the following powers in relation to the governor:

He is the principal channel of communication between the governor and the


council of ministers. It is the duty of the Chief Minister:

to communicate to the Governor of the state all decisions of the council of


ministers relating to the administration of the affairs of the state and
proposals for legislation;
to furnish such information relating to the administration of the affairs of the
state and proposals for legislation as the governor may call for; and

if the governor so requires, to submit for the consideration of the council of


ministers any matter on which a decision has been taken by a minister but
which has not been considered by the council.
He advises the governor with regard to the appointment of important officials
like advocate general, chairman and members of the state public service
commission, state election commissioner, and so on.

4. Solution: d)
He is a member of the Inter-State Council and the National Development
Council, both headed by the prime minister.

He acts as a vice-chairman of the concerned zonal council by rotation, holding


office for a period of one year at a time.

The CM is the chairman of the State Planning Board.


He is the chief spokesman of the state government.

http://www.insightsonindia.com

INSIGHTS

Page 2

Insights Mock Test 2015: Test 17 Solutions

He is the crisis manager-in-chief at the political level during emergencies.


As a leader of the state, he meets various sections of the people and receives
memoranda from them regarding their problems, and so on.
He is the political head of the services.

5. Solution: b)
The Akali Movement developed on a purely religious issue but ended up as a
powerful episode of Indias freedom struggle. From 1920 to 1925 more than 30,000
men and women underwent imprisonment, nearly 400 died and over 2,000 were
wounded. The movement arose with the objective of freeing the Gurdwaras (Sikh
temples) from the control of ignorant and corrupt mahants (priests).
To control and manage the Golden Temple, the Akal Takht and other Gurdwaras, a
representative assembly of nearly 10,000 reformers met in November 1920 and
elected a committee of 175 to be known as the Shiromani Gurdwara Prabhandak
Committee (SGPC). At the same time, the need was felt for a central body which
would organize the struggle on a more systematic basis. The Shiromani Akali Dal
was established in December for this purpose. It was to be the chief organizer of the
Akali jathos whose backbone was provided by Jat peasantry while their leadership
was in the hands of the nationalist intellectuals. Under the influence of the
contemporary Non-Cooperation Movement and many of the leaders were
common to both the movements the Akali Dal and the SGPC accepted complete
non-violence as their creed.

6. Solution: d)
Article 164:

The Chief Minister shall be appointed by the governor and other ministers
shall be appointed by the governor on the advise of the Chief Minister;

The ministers shall hold office during the pleasure of the governor; and
The council of ministers shall be collectively responsible to the legislative
assembly of the state.

Article 167: It shall be the duty of the Chief Minister:

http://www.insightsonindia.com

INSIGHTS

Page 3

Insights Mock Test 2015: Test 17 Solutions

to communicate to the governor of the state all decisions of the council of


ministers relating to the administration of the affairs of the state and
proposals for legislation;
to furnish such information relating to the administration of the affairs of the
state and proposals for legislation as the governor may call for ; and
if the governor so requires, to submit for the consideration of the council of
ministers any matter on which a decision has been taken by a minister but
which has not been considered by the council.

7. Solution: a)
If any question arises whether a matter falls within the governors discretion or not,
the decision of the governor is final and the validity of anything done by him cannot
be called in question on the ground that he ought or ought not to have acted in his
discretion. Further, the nature of advice tendered by ministers to the governor
cannot be enquired by any court. This provision emphasises the intimate and the
confidential relationship between the governor and the ministers.
In 1971, the Supreme Court ruled that a council of ministers must always exist to
advise the governor, even after the dissolution of the state legislative assembly or
resignation of a council of ministers.

8. Solution: a)
Such a specific resolution must be passed by the state assembly by a special majority,
that is, a majority of the total membership of the assembly and a majority of not less
than two-thirds of the members of the assembly present and voting. This Act of
Parliament is not to be deemed as an amendment of the Constitution for the
purposes of Article 368 and is passed like an ordinary piece of legislation (i.e, by
simple majority)

9. Solution: b)
Like at the Centre, in the states too, the council of ministers consists of three
categories of ministers, namely, cabinet ministers, ministers of state, and deputy
ministers. The difference between them lies in their respective ranks, emoluments,
and political importance. At the top of all these ministers stands the chief minister
supreme governing authority in the state.
http://www.insightsonindia.com

INSIGHTS

Page 4

Insights Mock Test 2015: Test 17 Solutions


The Constitution does not specify the size of the state council of ministers or the
ranking of ministers. They are determined by the chief minister according to the
exigencies of the time and requirements of the situation.

10. Solution: d)
Unlike the members of the legislative assembly, the members of the legislative
council are indirectly elected. The maximum strength of the council is fixed at onethird of the total strength of the assembly and the minimum strength is fixed at 406.
It means that the size of the council depends on the size of the assembly of the
concerned state. This is done to ensure the predominance of the directly elected
House (assembly) in the legislative affairs of the state. Though the Constitution has
fixed the maximum and the minimum limits, the actual strength of a Council is fixed
by Parliament.

11. Solution: a)
A major victory was won by the Akalis in the Keys Affair in October 1921. The
Government made an effort to keep possession of the keys of the Toshakhana of the
Golden Temple. The Akalis immediately reacted, and organized massive protest
meetings; tens of Akali jathas reached Amritsar immediately. The SGPC advised
Sikhs to join the hartal on the day of the arrival of the Prince of Wales in India. The
Government retaliated by arresting the prominent, militant nationalist leaders of the
SGPC like Baba Kharak Singh and Master Tara Singh. But, instead of dying down,
the movement began to spread to the remotest rural areas and the army. The NonCooperation Movement was at its height in the rest of the country. The Government
once again decided not to confront Sikhs on a religious issue. It released all those
arrested in the Keys Affair and surrendered the keys of the Toshakhana to Baba
Kharak Singh, head of the SGPC. Mahatma Gandhi immediately sent a telegram to
the Baba: First battle for Indias freedom won. Congratulations.

12. Solution: a)
Of the total number of members of a legislative council:

1/3 are elected by the members of local bodies in the state like municipalities,
district boards, etc.,

http://www.insightsonindia.com

INSIGHTS

Page 5

Insights Mock Test 2015: Test 17 Solutions

1/12 are elected by graduates of three years standing and residing within the
state,

1/12 are elected by teachers of three years standing in the state, not lower in
standard than secondary school,

1/3 are elected by the members of the legislative assembly of the state from
amongst persons who are not members of the assembly, and
the remainder are nominated by the gover-nor from amongst persons who
have a special knowledge or practical experience of literature, science, art,
cooperative movement and social service.

Thus, 5/6 of the total number of members of a legislative council are indirectly
elected and 1/6 are nominated by the governor. The members are elected in
accordance with the system of proportional representation by means of a single
transferable vote.

13. Solution: a)
The Chairman is elected by the council itself from amongst its members.
As a presiding officer, the powers and functions of the Chairman in the council are
similar to those of the Speaker in the assembly. However, the Speaker has one
special power which is not enjoyed by the Chairman. The Speaker decides whether a
bill is a Money Bill or not and his decision on this question is final.

14. Solution: d)
The legislative council, being a permanent house, is not subject to dissolution. Only
the legislative assembly is subject to dissolution. Un-like a prorogation, a dissolution
ends the very life of the existing House, and a new House is constituted after the
general elections are held. The position with respect to lapsing of bills on the
dissolution of the assembly is mentioned below:

A Bill pending in the assembly lapses (whether originating in the assembly or


transmitted to it by the council).
A Bill passed by the assembly but pending in the council lapses.
A Bill pending in the council but not passed by the assembly does not lapse.
A Bill passed by the assembly (in a unicameral state) or passed by both the
houses (in a bicameral state) but pending assent of the governor or the
President does not lapse.

http://www.insightsonindia.com

INSIGHTS

Page 6

Insights Mock Test 2015: Test 17 Solutions

A Bill passed by the assembly (in a unicameral state) or passed by both the
Houses (in a bicameral state) but returned by the president for
reconsideration of House (s) does not lapse.

15. Solution: c)
If the council passes the bill without amendments or the assembly accepts the
amendments suggested by the council, the bill is deemed to have been passed by
both the Houses and the same is sent to the the governor for his assent. On the other
hand, if the assembly rejects the amendments suggested by the council or the council
rejects the bill altogether or the council does not take any action for three months,
then the assembly may pass the bill again and transmit the same to the council. If the
council rejects the bill again or passes the bill with amendments not acceptable to the
assembly or does not pass the bill within one month, then the bill is deemed to have
been passed by both the Houses in the form in which it was passed by the assembly
for the second time.
Therefore, the ultimate power of passing an ordinary bill is vested in the assembly.
At the most, the council can detain or delay the bill for a period of four months
three months in the first instance and one month in the second instance.

16. Solution: c)
Both are correct:
The Kerala Provincial Congress Committee (KPCC) took up the eradication of
untouchability as an urgent issue While carrying on a massive propaganda
campaign against untouchability and for the educational and social upliftment of the
Harijans, it was decided to launch an immediate movement to open Hindu temples
and all public roads to the avarnas or Harijans. This, it was felt, would give a
decisive blow to the notion of untouchability since it was basically religious in
character and the avarnas exclusion from the temples was symbolic of their
degradation and oppression. A beginning was made in Vaikom, a village in
Travancore. There was a major temple there whose four walls were surrounded by
temple roads which could not be used by avarnas like Ezhavas and Pulayas. The
KPCC decided to use the recently acquired weapon of Satyagraha to fight
unouchability and to make a beginning at Vaikom by defying the unapproachability
rule by leading a procession of savarnas (caste Hindus) and avarnas on the temple
roads on 30 March 1924.

http://www.insightsonindia.com

INSIGHTS

Page 7

Insights Mock Test 2015: Test 17 Solutions


E.V. Ramaswami Naicker (popularly known as Periyar later) led ajatha from
Madurai and underwent imprisonment

17. Solution: a)
The Constitution lays down a special procedure for the passing of Money Bills in the
state legislature.
A Money Bill cannot be introduced in the legislative council. It can be introduced in
the legislative assembly only and that too on the recommendation of the governor.
Every such bill is considered to be a government bill and can be introduced only by a
minister.
After a Money Bill is passed by the legislative assembly, it is transmitted to the
legislative council for its consideration. The legislative council has restricted powers
with regard to a Money Bill. It cannot reject or amend a Money Bill. It can only make
recommendations and must return the bill to the legislative assembly within 14 days.
The legislative assembly can either accept or reject all or any of the recommendations
of the legislative council.
If the legislative assembly accepts any recommendation, the bill is then deemed to
have been passed by both the Houses in the modified form. If the legislative
assembly does not accept any recommendation, the bill is then deemed to have been
passed by both the Houses in the form originally passed by the legislative assembly
without any change.
Finally, when a Money Bill is presented to the governor, he may either give his
assent, withhold his assent or reserve the bill for presidential assent but cannot
return the bill for reconsideration of the state legislature. Normally, the governor
gives his assent to a money bill as it is introduced in the state legislature with his
prior permission.
When a money bill is reserved for consideration of the President, the president ma y
either give his assent to the bill or withhold his assent to the bill but cannot return
the bill for reconsideration of the state legislature.

18. Solution: d)
In the following matters, the powers and status of the council are broadly equal to
that of the assembly:

http://www.insightsonindia.com

INSIGHTS

Page 8

Insights Mock Test 2015: Test 17 Solutions

Introduction and passage of ordinary bills. However, in case of disagreement


between the two Houses, the will of the assembly prevails over that of the
council.
Approval of ordinances issued by the governor.

Selection of ministers including the chief minister. Under the Constitution the,
ministers including the chief minister can be members of either House of the
state legislature. However, irrespective of their membership, they are
responsible only to the assembly.

Consideration of the reports of the constitutional bodies like State Finance


Commission, state public service commission and Comptroller and Auditor
General of India.
Enlargement of the jurisdiction of the state public service commission.

19. Solution: d)
Every high court (whether exclusive or common) consists of a chief justice and such
other judges as the president may from time to time deem necessary to appoint.
Thus, the Constitution does not specify the strength of a high court and leaves it to
the discretion of the president. Accordingly, the President determines the strength of
a high court from time to time depending upon its workload.
The judges of a high court are appointed by the President. The chief justice is
appointed by the President after consultation with the chief justice of India and the
governor of the state concerned.
A judge of a high court can be removed from his office by an order of the President.
The President can issue the removal order only after an address by the Parliament
has been presented to him in the same session for such removal.

20. Solution: c)
Expenses are charged on Consolidated fund of the State (not India).
The judges of a high court are provided with the security of tenure. They can be
removed from office by the president only in the manner and on the grounds
mentioned in the Constitution. This means that they do not hold their office during
the pleasure of the president, though they are appointed by him. This is obvious
from the fact that no judge of a high court has been removed (or impeached) so far.

http://www.insightsonindia.com

INSIGHTS

Page 9

Insights Mock Test 2015: Test 17 Solutions


The salaries and allowances of the judges, the salaries, allowances and pensions of
the staff as well as the administrative expenses of a high court are charged on the
consolidated fund of the state. Thus, they are non-votable by the state legislature
(though they can be discussed by it). It should be noted here that the pension of a
high court judge is charged on the Consolidated Fund of India and not the state.
The Constitution prohibits any discussion in Parliament or in a state legislature with
respect to the conduct of the judges of a high court in the discharge of their duties,
except when an impeachment motion is under consideration of the Parliament.

21. Solution: a)
Original Jurisdiction - The power of a high court to hear disputes in the first
instance, not by way of appeal. It extends to the following:

Matters of admirality, will, marriage, divorce, company laws and contempt of


court.

Disputes relating to the election of members of Parliament and state


legislatures.

Regarding revenue matter or an act ordered or done in revenue collection.


Enforcement of fundamental rights of citizens.
Cases ordered to be transferred from a subordinate court involving the
interpretation of the Constitution to its own file.

Article 226 of the Constitution empowers a high court to issue writs including habeas
corpus, mandamus, certiorari, prohibition and quo-warrento for the enforcement of the
fundamental rights of the citizens and for any other purpose.
A high court has the power of superintendence over all courts and tribunals
functioning in its territorial jurisdiction (except military courts or tribunals).

22. Solution: d)
As per Article 129 and 215 of the Constitution, the SC and HCs will be courts of
record.
It means that their judgement is basically a legal precedent before other courts. The
subordinate courts can cite and use these precedents in their judgements.
Moreover, as a court of record, a HC also has power to punish for contempt.

http://www.insightsonindia.com

INSIGHTS

Page 10

Insights Mock Test 2015: Test 17 Solutions


23. Solution: d)
Judicial review is the power of a high court to examine the constitutionality of
legislative enactments and executive orders of both the Central and state
governments. On examination, if they are found to be violative of the Constitution
(ultra-vires), they can be declared as illegal, unconstitutional and invalid (null and
void) by the high court. Consequently, they cannot be enforced by the government.
Though the phrase judicial review has no where been used in the Constitution, the
provisions of Articles 13 and 226 explicitly confer the power of judicial review on a
high court. The constitutional validity of a legislative enactment or an executive
order can be challenged in a high court on the following three grounds:

it infringes the fundamental rights (Part III),


it is outside the competence of the authority which has framed it, and
it is repugnant to the constitutional provisions.

24. Solution: c)
The main weakness of the temple entry movement and the Gandhian or nationalist
approach in fighting caste oppression was that even while amusing the people
against untouchability they lacked a strategy for ending the caste system itself. The
strength of the national movement in this respect was to find expression in the
Constitution of independent India which abolished caste inequality, outlawed
untouchability and guaranteed social equality to all citizens irrespective of their
caste. Its weakness has found expression in the growth of casteism and the
continuous existence in practice of oppression and discrimination against the lower
castes in post-1947 India.

25. Solution: d)
The state judiciary consists of a high court and a hierarchy of subordinate courts,
also known as lower courts. The subordinate courts are so called because of their
subordination to the state high court. They function below and under the high court
at district and lower levels.
Articles 233 to 237 in Part VI of the Constitution make the following provisions to
regulate the organization of subordinate courts and to ensure their independence
from the executive.

http://www.insightsonindia.com

INSIGHTS

Page 11

Insights Mock Test 2015: Test 17 Solutions


26. Solution: a)
The appointment, posting and promotion of district judges in a state are made by the
governor of the state in consultation with the high court.
A person to be appointed as district judge should have the following qualifications:

He should not already be in the service of the Central or the state government.
He should have been an advocate or a pleader for seven years.
He should be recommended by the high court for appointment.

Appointment of persons (other than district judges) to the judicial service of a state
are made by the governor of the state after consultation with the State Public Service
Commission and the high court.

27. Solution: d)
Under Article 1 of the Indian Constitution, the State of Jammu and Kashmir (J&K) is
a constituent state of Indian Union and its territory forms a part of the territory of
India. On the other hand, Article 370 in Part XXI of the Constitution grants a special
status to it.
Accordingly, all the provisions of the Constitution of India do not apply to it. It is
also the only state in the Indian Union which has its own separate state
Constitutionthe Constitution of Jammu and Kashmir.
For details go to Chapter 32 of Laxmikanth Indian Polity. It will not be possible to
list all the provisions.

28. Solution: d)
Explanation:
The post office offers a number of savings plans, including National Savings
Certificates, the Public Provident Fund, Recurring Deposit account (RD), savingsbank accounts, monthly-income plans, senior-citizens' savings plans and timedeposit accounts.

http://www.insightsonindia.com

INSIGHTS

Page 12

Insights Mock Test 2015: Test 17 Solutions


29. Solution: c)
Explanation:
National e-Governance Plan (NeGP) was initiated to make all government services
available to the citizens of India via electronic media. NeGP has been formulated by
the Department of Electronics and Information Technology (DeitY) and Department
of Administrative Reforms and Public Grievances (DARPG). The Government
approved the National e-Governance Plan, comprising 31 Mission Mode Projects
(MMPs) and eight components, on May 18, 2006.

30. Solution: d)
Explanation:
In order to utilise and harness the benefits of Cloud Computing, Government of
India has embarked upon a very ambitious and important initiative GI Cloud
which has been coined as Meghraj. The focus of this initiative is to evolve a Strategy
and implement various components including governance mechanism to ensure
proliferation of Cloud in government. Formulation of the Cloud Policy is one of the
primary steps that will facilitate large scale adoption of cloud by government.

31. Solution: d)
Explanation:
The National Optical Fibre Network (NOFN) is a project to provide broadband
connectivity to over two lakh Gram panchayats of India at a cost of Rs.20,000 crore.
The project provides internet access using existing optical fibre and extending it to
the Gram panchayats. The project was intended to enable the government of India to
provide e-services and e-applications nationally. A special purpose vehicle Bharat
Broadband Network Limited (BBNL) was created as a Public Sector Undertaking
(PSU) under the Companies Act of 1956 for the execution of the project. The project
will be funded by the Universal Service Obligation Fund (USOF).
All the Service Providers like Telecom Service Providers (TSPs), ISPs, Cable TV
operators etc. will be given non-discriminatory access to the National Optic Fibre
Network and can launch various services in rural areas. NOFN has the potential to
transform many aspects of our lives including video, data, internet, telephone
http://www.insightsonindia.com

INSIGHTS

Page 13

Insights Mock Test 2015: Test 17 Solutions


services in areas such as education, business, entertainment, environment, health
households and e-governance services.
National Optic Fibre Network uses Gigabit passive optical network (GPON)
technology indigenously developed by Centre for Development of Telematics (CDOT). A passive optical network (PON) brings fiber cabling and signals to the home
using a point-to-multipoint scheme that enables a single optical fiber to serve
multiple premises. Encryption maintains data security in this shared environment.
The architecture uses passive (unpowered) optical splitters, reducing the cost of
equipment compared to point-to-point architectures.

32. Solution: d)
Explanation:
Cyber Appellate Tribunal has been established under the Information Technology
Act under the aegis of Controller of Certifying Authorities (C.C.A.). The first and the
only Cyber Appellate Tribunal in the country has been established by the Central
Government in accordance with the provisions contained under Section 48(1) of the
Information Technology Act, 2000.
The Cyber Appellate Tribunal has, for the purposes of discharging its functions
under the I.T. Act, the same powers as are vested in a civil court under the Code of
Civil Procedure, 1908. However, the procedure laid down by the Code of Civil
Procedure, 1908 applies but at the same time the Tribunal is guided by the principles
of natural justice.
The Cyber Appellate Tribunal has powers to regulate its own procedure including
the place at which it has its sittings. Every proceeding before the Cyber Appellate
Tribunal shall be deemed to be a judicial proceeding within the meaning of sections
193 and 228, and for the purposes of section 196 of the Indian Penal Code and the
Cyber Appellate Tribunal shall be deemed to be a civil court for the purposes of
section 195 and Chapter XXVI of the Code of Criminal Procedure, 1973.

33. Solution: c)
Explanation:
http://www.insightsonindia.com

INSIGHTS

Page 14

Insights Mock Test 2015: Test 17 Solutions


Indian Computer Emergency Response Team (CERT-In) is the Government
organization under Ministry of Communications and Information Technology. It is a
nodal agency that deals with cyber security threats like hacking and phishing. It
strengthens security-related defence of the Indian Internet domain. In March 2014,
CERT reported a critical flaw in Android Jelly bean's VPN implementation.

34. Solution: c)
The constitution of J&K provides for Governors Rule. Hence, the governor, with the
concurrence of the President of India, can assume to himself all the powers of the
state government, except those of the high court. He can dissolve the assembly and
dismiss the council of ministers. The Governors Rule can be imposed when the state
administration cannot be carried on in accordance with the provisions of the J&K
Constitution. It was imposed for the first time in 1977. Notably, in 1964, Article 356
of the Indian Constitution (dealing with the imposition of Presidents Rule in a state)
was extended to the state of J&K.

35. Solution: d)
Article 371-A makes the following special provisions for Nagaland:

The Acts of Parliament relating to the following matters would not apply to
Nagaland unless the State Legislative Assembly so decides:

religious or social practices of the Nagas;


Naga customary law and procedure;
administration of civil and criminal justice involving decisions according to
Naga customary law; and
ownership and transfer of land and its resources.

The Governor of Nagaland shall have special responsibility for law and order in the
state so long as internal disturbances caused by the hostile Nagas continue. In the
discharge of this responsibility, the Governor, after consulting the Council of
Ministers, exercises his individual judgement and his decision is final. This special
responsibility of the Governor shall cease when the President so directs.

36. Solution: d)
The act gives a constitutional status to the panchayati raj institutions. It has brought
them under the purview of the justiciable part of the Constitution. In other words,
http://www.insightsonindia.com

INSIGHTS

Page 15

Insights Mock Test 2015: Test 17 Solutions


the state governments are under constitutional obligation to adopt the new
panchayati raj system in accordance with the provisions of the act. Consequently,
neither the formation of panchayats nor the holding of elections at regular intervals
depend on the will of the state government any more.
The provisions of the act can be grouped into two categoriescompulsory and
voluntary. The compulsory (mandatory or obligatory) provisions of the act have to
be included in the state laws creating the new panchayati raj system. The voluntary
provisions, on the other hand, may be included at the discretion of the states. Thus
the voluntary provisions of the act ensures the right of the states to take local factors
like geographical, politicoadministrative and others, into consideration while
adopting the new panchayati raj system.
The act is a significant landmark in the evolution of grassroot democratic institutions
in the country. It transfers the representative democracy into participatory
democracy. It is a revolutionary concept to build democracy at the grassroot level in
the country.

37. Solution: c)
The 73rd amendment act provides for a Gram Sabha as the foundation of the
panchayati raj system. It is a body consisting of persons registered in the electoral
rolls of a village comprised within the area of Panchayat at the village level. Thus, it
is a village assembly consisting of all the registered voters in the area of a panchayat.
It may exercise such powers and perform such functions at the village level as the
legislature of a state determines.
Three-Tier System - The act provides for a three-tier system of panchayati raj in
every state, that is, panchayats at the village, intermediate, and district levels. Thus,
the act brings about uniformity in the structure of panchayati raj throughout the
country. However, a state having a population not exceeding 20 lakh may not
constitute panchayats at the intermediate level.

38. Solution: d)
The governor of a state shall, after every five years, constitute a finance commission
to review the financial position of the panchayats.
The superintendence, direction and control of the preparation of electoral rolls and
the conduct of all elections to the panchayats shall be vested in the state election
http://www.insightsonindia.com

INSIGHTS

Page 16

Insights Mock Test 2015: Test 17 Solutions


commission. It consists of a state election commissioner to be appointed by the
governor. His conditions of service and tenure of office shall also be determined by
the governor.
DRDA have been abandoned now. They were not constituted in furtherance of the
provisions of these constitutional amendments.

39. Solution: d)
Some of the Subjects under the 11 th schedule that can be devolved to the Panchayats
are:

Poverty alleviation programme


Education, including primary and secondary schools
Technical training and vocational education
Adult and non-formal education
Libraries
Cultural activities
Markets and fairs

Health and sanitation including hospitals, primary health centres and


dispensaries
Family welfare
Women and child development

Social welfare, including welfare of the handicapped and mentally retarded


Welfare of the weaker sections, and in particular, of the scheduled castes and
the scheduled tribes

Public distribution system


Maintenance of community assets

40. Solution: c)
These are voluntary provisions:

Giving representation to members of the Parliament (both the Houses) and


the state legislature (both the Houses) in the panchayats at different levels
falling within their constituencies.

Providing reservation of seats (both members and chairpersons) for backward


classes in panchayats at any level.

http://www.insightsonindia.com

INSIGHTS

Page 17

Insights Mock Test 2015: Test 17 Solutions

Granting powers and authority to the panchayats to enable them to function


as institutions of self-government (in brief, making them autonomous bodies).

Devolution of powers and responsibilities upon panchayats to prepare plans


for economic development and social justice; and to perform some or all of
the 29 functions listed in the Eleventh Schedule of the Constitution.
Granting financial powers to the pachayats, that is, authorizing them to levy,
collect and appropriate taxes, duties, tolls and fees.

41. Solution: d)
The objectives of the PESA Act are as follows:

To extend the provisions of Part IX of the Constitution relating to the


panchayats to the scheduled areas with certain modifications
To provide self-rule for the bulk of the tribal population
To have village governance with participatory democracy and to make the
gram Sabha a nucleus of all activities
To evolve a suitable administrative framework consistent with traditional
practices

To safeguard and to preserve the traditions and customs of tribal


communities

To empower panchayats at the appropriate levels with specific powers


conducive to tribal requirements

To prevent panchayats at the higher level from assuming the powers and
authority of panchayats at the lower level of the gram Sabha

42. Solution: d)
http://pib.nic.in/newsite/PrintRelease.aspx?relid=108320

43. Solution: d)
The system of urban government was constitutionalised through the 74th
Constitutional Amendment Act of 1992. At the Central level, the subject of urban
local government is dealt with by the following three ministries: Ministry of Urban
Development, created as a separate ministry in 1985; Ministry of Defence in the case
of cantonment boards; and Ministry of Home Affairs in the case of Union Territories.
http://www.insightsonindia.com

INSIGHTS

Page 18

Insights Mock Test 2015: Test 17 Solutions

44. Solution: a)
The major events in this context are as follows:

In 1687-88, the first municipal corporation in India was set up at Madras.


In 1726, the municipal corporations were set up in Bombay and Calcutta.
Lord Mayos Resolution of 1870 on financial decentralisation visualised the
development of local self-government institutions.

Lord Ripons Resolution of 1882 has been hailed as the Magna Carta of local
self-government. He is called as the father of local-self government in India.

The Royal Commission on decentralisation was appointed in 1907 and it


submitted its report in 1909. Its chairman was Hobhouse.
Under the dyarchical scheme introduced in Provinces by the Government of
India Act of 1919, local self-government became a transferred subject under
the charge of a responsible Indian minister.

In 1924, the Cantonments Act was passed by the Central legislature.


Under the provincial autonomy scheme introduced by the Government of
India Act of 1935, local self-government was declared a provincial subject.

45. Solution: a)
A new line of political activity, which would keep up the spirit of resistance to
colonial rule, was now advocated by C.R. Das and Motilal Nehru. They suggested
that the nationalists should end the boycott of the legislative councils, enter them,
expose them as sham parliaments and as a mask which the bureaucracy has put
on, and obstruct every work of the council. This, they argued, would not be giving
up non-cooperation but continuing it in a more effective form by extending it to the
councils themselves. It would be opening a new front in the battle. C.R. Das as the
President of the Congress and Motilal as its Secretary put forward this programme
of either mending or ending the councils at the Gaya session of the Congress in
December 1922. Another section of the Congress, headed by Vallabhbhai Patel,
Rajendra Prasad and C. Rajagopalachari, opposed the new proposal which was
consequently defeated by 1748 to 890 votes. Das and Motilal resigned from their
respective offices in the Congress and on 1 January 1923 announced the formation of
the Congress-Khilafat Swaraj Party better known later as the Swaraj Party. Das was
the President and Motilal one of the Secretaries of the new party. The adherents of

http://www.insightsonindia.com

INSIGHTS

Page 19

Insights Mock Test 2015: Test 17 Solutions


the councilentry programme came to be popularly known as pro-changers and
those still advocating boycott of the councils as no changers.

46. Solution: d)
All the members of a municipality shall be elected directly by the people of the
municipal area. For this purpose, each municipal area shall be divided into territorial
constituencies to be known as wards. The state legislature may provide the manner
of election of the chairperson of a municipality. It may also provide for the
representation of the following persons in a municipality.

Persons having special knowledge or experience in municipal administration


without the right to vote in the meetings of municipality.

The members of the Lok Sabha and the state legislative assembly representing
constituencies that comprise wholly or partly the municipal area.
The members of the Rajya Sabha and the state legislative council registered as
electors within the municipal area.
The chairpersons of committees (other than wards committees).

47. Solution: b)
Recently Union Government has appointed 9 Executive Directors (EDs) in Public
Sector Banks (PSBs).
Their appointment was approved by Appointments Committee of the Cabinet on
recommendation of selection panel headed by the Reserve Bank of India (RBI)
Governor Raghuram Rajan.
Some of the Executive Directors (ED) of PSBs appointed are

K. Sahoo- ED of Allahabad Bank.

Pawan Kumar Bajaj- ED of Indian Overseas Bank.

Ravi Shankar Pandey- ED of Syndicate Bank.

http://www.insightsonindia.com

INSIGHTS

Page 20

Insights Mock Test 2015: Test 17 Solutions


48. Solution: a)
Delhi High Court in its recent ruling has held that Attorney General of India comes
under the ambit of Right to Information (RTI) Act, 2005.
This ruling reversed the previous decision of the Central Information Commission
(CIC) which had held that Attorney General does not come under RTI Act.
Delhi High Court held that

Attorney General is not merely a lawyer for the government, but is a


constitutional authority. This is because functions performed by the AG
under Article 76 (2) of the Constitution of India are in the nature of public
functions.

So the office of Attorney General is a public authority and comes under the
ambit of section 2(h) of the RTI Act.

The decision was given by Delhi High Court bench headed by Justice Vibhu Bakhru
while hearing on two separate pleas filed by RTI activists Subhash Chandra Agarwal
and RK Jain. In one plea, the petitioners had challenged the CIC ruling and urged
the court to declare AG office answerable to RTI Act.
In 2012, CIC bench had ruled that the Attorney General of India is only a person and
cannot be considered an authority under the RTI Act.

49. Solution: a)
Recently, Prime Minister Narendra Modi visited Seychelles as part of three-nation
tour for cementing strategic partnership with island courtiers of Seychelles, Maldives
and Sri Lanka.
Both nations agreed to set up joint working group to promote ocean based economy
i.e. on Blue Economy. In this regard, India committed to provide all help to
Seychelles.
Bluechip companies are nationally recognized, well-established and financially
sound company. Blue chips generally sell high-quality, widely accepted products
and services. Blue chip companies are known to weather downturns and operate
profitably in the face of adverse economic conditions, which helps to contribute to
their long record of stable and reliable growth.

http://www.insightsonindia.com

INSIGHTS

Page 21

Insights Mock Test 2015: Test 17 Solutions

50. Solution: d)
Lok Sabha has passed The Right to Fair Compensation and Transparency in Land
Acquisition, Rehabilitation and Resettlement (Amendment) Bill, 2015. The bill was
passed by voice vote along with incorporated 9 new amendments moved by Union
government.
The amendment bill replaces the ordinance promulgated in this regard and seeks to
amend the RFCTLARR Act, 2013.
Key provisions of Amendment Bill

The Bill creates five special categories of land use. They are 1. Rural
infrastructure, 2. Affordable housing, 3. Defence, 4. Industrial corridors, 5.
Infrastructure projects including Public Private Partnership (PPP) projects
where the Union government owns the land.

Removes the Consent Clause (CC) of 80 per cent of land owners for private
projects and that of 70 per cent of land owners for PPP projects in case of
acquiring land for above five categories from parent Act.

Removes of Social Impact Assessment (SIA) clause for 5 categories from


parent Act which is done to identify those affected and from the restrictions
on the acquisition of irrigated multi-cropped land.

The provisions for compensation, rehabilitation, and resettlement under 13


central Acts are in consonance with the LARR Act.

51. Solution: a)
Worlds first solar powered aircraft on global journey- Solar Impulse 2 successfully
landed in Ahmedabad, Gujarat.
The aircraft was piloted by Swiss pilots Bertrand Piccard and Andre Borschberg
from its first halt Muscat, Oman.
Key features of Solar Impulse 2 aircraft

It is a Swiss long-range solar-powered aircraft project and claimed to be first


aircraft to fly day and night without consuming conventional fuel.

Solely propelled by the solar energy and emits zero Carbon Dioxide (CO2).

http://www.insightsonindia.com

INSIGHTS

Page 22

Insights Mock Test 2015: Test 17 Solutions

Structure: Single-seater aircraft made of carbon fibre. It has a 72 metre


wingspan and weighs 2,300 kilo grams.

Solar cells: 17,248 solar cells inbuilt into its wings. These cells supply
renewable energy to propel the electric motors of aircraft.

These solar cells also recharge 4 lithium polymer batteries mounted on


aircraft which allows it fly at night.

The first test flight of the aircraft was conducted in the United States in 2013.

52. Solution: a)
Explanation: The Department of Commerce formulates, implements and monitors
the Foreign Trade Policy which provides the basic framework of policy and strategy
to be followed for promoting for promoting exports and trade.

53. Solution: b)
Explanation:
Who can set up SEZs?
Any private/public/joint sector or state government or its agencies can set up an
SEZ. Even, a foreign agency can set up SEZs in India.
State wise number of SEZs:

State/Union
Territory

Number of
operational
Special
Economic
Zones
(June 2012)

Andhra
36
Pradesh &Telangana

http://www.insightsonindia.com

Number
of SEZs
formally
approved
(June
2012)

Total
(Operational
+ Approved)

11

152

INSIGHTS

Page 23

Insights Mock Test 2015: Test 17 Solutions


Number of
operational
Special
Economic
Zones
(June 2012)

Number
of SEZs
formally
approved
(June
2012)

Total
(Operational
+ Approved)

Tamil Nadu

28

77

105

Karnataka

20

62

82

Maharashtra

18

119

137

Gujarat

13

53

66

Kerala

29

36

Uttar Pradesh

35

41

West Bengal

24

29

Rajasthan

11

15

Haryana

49

52

Chandigarh

Madhya Pradesh

17

18

Odisha

18

111

129

Punjab

State/Union
Territory

http://www.insightsonindia.com

INSIGHTS

Page 24

Insights Mock Test 2015: Test 17 Solutions


Number of
operational
Special
Economic
Zones
(June 2012)

Number
of SEZs
formally
approved
(June
2012)

Total
(Operational
+ Approved)

Goa

Chhattisgarh

Delhi

Dadra and Nagar


Haveli

Nagaland

Puducherry

Uttarakhand

Jharkhand

State/Union
Territory

For reference:
http://www.sezindia.nic.in/writereaddata/pdf/ListofoperationalSEZs.pdf.

54. Solution: c)
Explanation:

http://www.insightsonindia.com

INSIGHTS

Page 25

Insights Mock Test 2015: Test 17 Solutions


Export Promotion Capital Goods (EPCG) scheme allows import of capital goods
including spares for pre production, production and post production at zero duty
subject to an export obligation of 6 times of duty saved on capital goods imported
under EPCG scheme, to be fulfilled in 6 years reckoned from Authorization
issuedate.
EPCG scheme covers manufacturer exporters with or without supporting
manufacturer(s)/ vendor(s), merchant exporters tied to supporting manufacturer(s)
and service providers. The Scheme also covers a service provider who is designated
/ certified as a Common Service Provider (CSP). EPCG authorization holder can
export either directly or through third party (s). Export proceeds are to be realized in
freely convertible currency except for deemed exports. Import of capital goods
imported under the EPCG scheme shall be subject to Actual User condition till
export obligation is completed.
Spares for the existing plant and machinery may also be imported under the EPCG
scheme subject to an export obligation equivalent to 8 times of duty saved to be
fulfilled over a period of 8 years reckoned from the date of issuance of licence.

55. Solution: d)
Explanation:
The Ministry of Communication and Information Technology contains three
departments:
Department of Telecommunications
Department of Electronics and Information Technology
Department of Posts

56. Solution: c)
Explanation:
The postal service is under the Department of Posts, which is part of the Ministry of
Communications and Information Technology of the Government of India. The
modern postal system in India was established by Lord Clive in the year 1766 and it
was further developed by warren hastings in 1774.
http://www.insightsonindia.com

INSIGHTS

Page 26

Insights Mock Test 2015: Test 17 Solutions


The statute presently governing the postal services in India is the Indian Post Office
Act, 1898.

57. Solution: d)
Maharashtra government has launched Bhagyashree scheme for girls. The scheme
replaces state governments previous Sukanya scheme.
Bhagyashree scheme was launched by was launched by Chief Minister Devendra
Fadnavis along with the actress Bhagyashree who is brand ambassador of the
scheme.
Under the Sukanya scheme, the state government was to earmark an amount of Rs.
21,200 for each girl child born in a Below the Poverty Line (BPL) family.
While the Bhagyashree scheme aims at providing one lakh rupee on maturity after
the girl completes 18 years of age.
State government is seeking to link the Bhagyashree scheme with the Union
Governments Beti Bachao Beti Padhao scheme.

58. Solution: d)
North East Indias first Kisan Call Centre (KCC) was opened in Agartala, Tripura. It
was launched by Tripura Chief Minister Manik Sarkar.
The call centre is financed by Tripura government and will be run by experts of
agriculture, fisheries, horticulture and animal husbandry from a private enterprise.
About KCC

KCC Agartala will work daily from 6 am to 10 pm and provide a real time
advisory and immediate solution with expert consultation to farmers from
Tripura on various agriculture related issues.

It includes expert consultation seed procurement, agro-metric, fertilisers, soil


health, pesticides usage and marketing of the produces.

Farmers from neighbouring states of Tripura also will be provided by similar


services by Kisan Call Centre.

Tripura also launched farmers portal to issue advisory to farmers for farm
production under direct supervision of the experts.
http://www.insightsonindia.com

INSIGHTS

Page 27

Insights Mock Test 2015: Test 17 Solutions

59. Solution: a)
Prime Minister Narendra Modi recently launched the first indigenously developed
and manufactured Rotavirus vaccine- Rotavac.
The vaccine will boost Indias efforts to combat infant mortality due to diarrhoea.
This is the third such vaccine available globally against Rotavirus.
Development of the Vaccine

Rotavac vaccine was developed under the joint collaboration between India
and the United States in the area of medical research.

It was developed under the public-private partnership (PPP) model that


involved Union Ministry of Science and Technology, institutions of the US
Government and NGOs in India supported by the Bill and Melinda Gates
Foundation.

Union Government had funded the basic research in educational and


scientific along with USs National Institute of Health. While the Bharat
Biotech India Limited (BBIL) and the Gates Foundation had contributed
towards vaccine development and testing.

BBIL was selected for vaccine development by the India-U.S. Vaccine Action
Programme (VAP) in 1997-1998 by the standard government procedures.

BBIL, the manufacturer of vaccine has given undertaking to keep the cost of
the vaccine at US$ 1 per dose, thus making it cheapest vaccine available
globally.

60. Solution: b)
NASAs Dawn space probe has completed its orbit around dwarf planet Ceres which
is the largest object in the Solar System between Mars and Jupiter.
Dawn has taken 7.5 years to reach the orbit of Ceres. Scientists are hoping to glean
information from the Ceres which can tell them about the Solar Systems beginnings
after its formation four and a half billion ago.
Earlier during its journey from 2011 to 2012, the spacecraft also had explored and
surveyed the giant asteroid Vesta. It had delivered new insights and thousands of
images from the asteroid.
http://www.insightsonindia.com

INSIGHTS

Page 28

Insights Mock Test 2015: Test 17 Solutions


Dawn space probe is the first NASA exploratory mission to use ion propulsion to
enter orbits.

61. Solution: a)
The essence of Kashmir is defined by its rich culture and its warm people. It is
equally known for the magnificent ornaments and dresses worn by its people.
Ornaments in the valley are not worn for their intrinsic value and beauty alone but
for religious reasons as well. The beautiful golden ornament worn by married
Kashmiri Pandit women known as Dej-Hor is an important example.

PIB Features

62. Solution: d)
Solid waste dumping sites or landfill sites need more amount of land which is not
available in urban areas. Incineration of solid waste pollutes the environment if the
incinerators are not designed or operated properly.Thermal Plasma Technology is
ideally suited for waste treatment. By plasma technology Hazardous & toxic
compounds are broken down to elemental constituents at high temperatures;
Inorganic materials are converted to Vitrified Mass; and Organic materials are
Pyrolysed or Gasified, Converted to flue gases (H2 & CO) & Lower hydrocarbon
gases when operated at low temperature (500 600OC).Disposal of carcass is also
being thought of using plasma pyrolysis.
PIB Features

63. Solution: d)
http://www.insightsonindia.com

INSIGHTS

Page 29

Insights Mock Test 2015: Test 17 Solutions


Role of environmental isotope techniques in the water resources development and
management - There are two type of isotopes, stable isotopes and radioactive
isotopes. Isotope techniques are used to find out the type of contamination in surface
water and ground water, the sources and origin of contamination, pollutant
dispersion in surface water bodies, to assess the groundwater salinity, to assess the
changes due to long-term exploitation of groundwater, for hydro-chemical
investigation and to carry out geochemical evolution of groundwater.
PIB Features

64. Solution: c)
C.R. Das as the President of the Congress and Motilal as its Secretary put forward
this programme of either mending or ending the councils at the Gaya session of the
Congress in December 1922. Another section of the Congress, headed by
Vallabhbhai Patel, Rajendra Prasad and C. Rajagopalachari, opposed the new
proposal.

65. Solution: b)
Rural Human & Resource Development Facility is disseminating BARC
technologies, namely NisargrunaBiogas, Soil Organic Carbon Testing Kit, Seed Bank,
Domestic Water Purifier, Weather Forecasting, LLL, RIA, FSD, VTD; under the
AKRUTI( Advance Knowledge of Rural Technology Implementation) Program.
Activities carried out under the AKRUTI program aresurveys for safe drinking
water, Interaction with the villagers, Entrepreneurship development for domestic
water purifier production and Awareness programs for benefits of use purified
water. RHRDF has also launched a scheme for safe drinking water for village under
CSR.

66. Solution: a)
The Sewage is the waste water generated from domestic premises and consists
mainly of human waste. It typically contains 99.9% water and about 0.1% solid. The
solid waste in sewage is typically organic in nature and is broken down in the
sewage treatment plants resulting in sewage sludge as a byproduct. In Radiation
Hygienization process dry sludge generated at STPs is hygienized using radiation
technology using standard Gamma facility at a Dose of 10 kGs. Such radiation plants
are operating in India for sterilizing medical products.
http://www.insightsonindia.com

INSIGHTS

Page 30

Insights Mock Test 2015: Test 17 Solutions


Refuse Derived Fuel (RDF) is a processed form of Municipal Solid Waste (MSW) and
it can be a substitute to coal energy. The process of conversion of garbage into fuel
pellets involves primarily Drying, Separation of incombustible, Size reduction and
Pelletisation. It is being developed by BARC.
PIB Features

67. Solution: a)
http://www.downtoearth.org.in/content/who-launches-special-programmeprevent-infant-deaths

68. Solution: b)
http://www.downtoearth.org.in/content/monkey-fever-hits-kerala-health-workerfifth-victim-disease

69. Solution: b)
http://www.downtoearth.org.in/content/about-two-thirds-world-s-populationsuffers-tooth-decay
http://en.wikipedia.org/wiki/Ponmudi#Dental_Tourism

70. Solution: a)
http://www.downtoearth.org.in/content/bring-ganga-back-original-course-orderspatna-high-court

71. Solution: d)
A managed-exchange-rate system is a hybrid or mixture of the fixed and flexible
exchange rate systems in which the government of the economy attempts to affect
the exchange rate directly by buying or selling foreign currencies or indirectly,
through monetary policy6 (i.e., by lowering or raising interest rates on foreign
currency bank accounts, affecting foreign investment, etc.).

http://www.insightsonindia.com

INSIGHTS

Page 31

Insights Mock Test 2015: Test 17 Solutions


Today, most of the economies have shifted to this system of exchange rate
determination. Almost all countries tend to intervene when the markets become
disorderly or the fundamentals of economics are challenged by the exchange rate of
the time.

72. Solution: d)
A measurement of a countrys trade in which the value of goods and services it
imports exceeds the value of goods and services it exports. The current account also
includes net income, such as interest and dividends, as well as transfers, such as
foreign aid, though these components tend to make up a smaller percentage of the
current account than exports and imports. The current account is a calculation of a
countrys foreign transactions, and along with the capital account is a component of
a countrys balance of payment.

73. Solution: c)
The BoP of an economy is calculated on the principles of accountancy (double-entry
bookkeeping) and looks like the balance sheet of a companyevery entry shown
either as credit (inflow) or debit (outflow). If there is a positive outcome at the end of
the year, the money is automatically transferred to the foreign exchange reserves of
the economy. And if there is any negative outcome, the same foreign exchange is
drawn from the countrys forex reserves. If the forex reserves are not capable of
fulfilling the negativity created by the BoP, it is known as a BoP crisis and the
economy tries different means to solve the crisis in which going for forex help from
the IMF is the last resort.

74. Solution: a)
An economy might allow its currency full or partial convertibility in the current and
the capital accounts. If domestic currency is allowed to convert into foreign currency
for all current account purposes, it is a case of full current account convertibility.
Similarly, in cases of capital outflow, if domestic currency is allowed to convert into
foreign currency, it is a case of full capital account convertibility. If the situation is of
partial convertibility, then the portion allowed by the government can be converted
into foreign currency for current and capital purposes. It should always be kept in
mind that the issue of currency convertibility is concerned with foreign currency
outflow only.
http://www.insightsonindia.com

INSIGHTS

Page 32

Insights Mock Test 2015: Test 17 Solutions

75. Solution: a)
http://articles.economictimes.indiatimes.com/2003-01-28/news/27532328_1_realexchange-reer-nominal-effective-exchange-rate

76. Solution: c)
The Swarajist activity in the legislatures was spectacular by any standards. It
inspired the politicized persons and kept their political interest alive. People were
thrilled every time the allpowerful foreign bureaucracy was humbled in the councils.
Simultaneously, during 1923-24, Congressmen captured a large number of
municipalities and other local bodies. Das became the Mayor of Calcutta (with
Subhas Bose as his Chief Executive Officer), and Vithalbhai Patel. the President of
Bombay Corporation, Vallabhbhai Patel of Ahmedabad Municipality, Rajendra
Prasad of Patna Municipality, and Jawaharlal Nehru of Allahabad Municipality. The
no-changers actively joined in these ventures since they believed that local bodies
could be used to promote the constructive programme. Its preoccupation with
parliamentary politics also started telling on the internal cohesion of the Swaraj
Party. For one, the limits of politics of obstruction were soon reached. Having
repeatedly outvoted the Government and forced it to certify its legislation, there was
no way of going further inside the legislatures and escalating the politics of
confrontation. This could be done only by a mass movement outside. But the
Swarajists lacked any policy of coordinating their militant work in the legislatures
with mass political work outside. In fact, they relied almost wholly on newspaper
reporting.

77. Solution: d)
It is the international currency in which the highest faith is shown and is needed by
every economy.
The strongest currency of the world is one which has a high level of liquidity.
Basically, the economy with the highest as well as highly diversified exports that are
compulsive imports for other countries (as of high level technology, defence
products, life saving medicines and petroleum products) will also create high
demand for its currency in the world and become the hard currency. It is always
scarce.
http://www.insightsonindia.com

INSIGHTS

Page 33

Insights Mock Test 2015: Test 17 Solutions


Up to the Second World War, the best hard currency was the Pound Sterling () of
the UK but soon it was replaced by the US$at present some experts believe that
the Eurolands currency () might replace it, too. Some of the best hard currencies of
the world today are the US dollar, the Euro(), Japanese Yen () and the UK Sterling
Pound ().

78. Solution: c)
SEZ, or Special Economic Zone, is essentially an industrial cluster meant largely for
exports. An SEZ is governed by a special set of rules aimed at attracting direct
investment for export-oriented production. SEZs, earlier known as Export
Processing Zones or Free Trade Zones, are duty free enclaves which are treated as
foreign territory only for trade operations, duties, tariffs and typically marked by the
best infrastructure and least red tape. Other salient features of SEZs are:
Manufacturing or service activities are allowed;
Full freedom for sub-contracting;
No routine examination by customs authorities of export/import cargo;
Units in SEZs have to become net foreign exchange earners within three years; and
Domestic sales from them are subject to full customs duty and the import policy in
force.
The SEZ concept recognises the issues related to economic development and
provides for developing self-sustaining Industrial Townships so that the increased
economic activity does not create pressure on the existing infrastructure.

79. Solution: a)
The objective of the GAAR provisions is to codify the doctrine of substance over form
where the real intention of the parties and purpose of an arrangement is taken into
account for determining the tax consequences, irrespective of the legal structure of
the concerned transaction or arrangement. It essentially comes into effect where an
arrangement is entered into with the main purpose or one of the main purposes of
obtaining a tax benefit and which also satisfies at least one of the following four tests:

The arrangement creates rights and obligations that are not at arms length,

It results in misuse or abuse of provisions of tax laws,


Lacks commercial substance or is deemed to lack commercial substance, or

http://www.insightsonindia.com

INSIGHTS

Page 34

Insights Mock Test 2015: Test 17 Solutions

it is not carried out in a bona fide manner.

Thus, if the tax officer believes that the main purpose or one of the main purposes of
an arrangement is to obtain a tax benefit and even if one of the above four tests are
satisfied, he has powers to declare it as an impermissible avoidance arrangement
and re-characterise the entire transaction in a manner that is more conducive to
maximising tax revenues

80. Solution: d)
To some extent CAD is a good indicator for a developing country. For example, if
the industrialists borrow more from outside India to expand their business activities,
it leads to a growth in businesses and economic activity which in-turn leads to more
revenue from the businesses in the future. In this case a comfortable CAD indicates a
nation's intention to grow.
The major contributor to Indias CAD is imports of Gold and Crude Oil.The high
current account deficits stemming from imports which do not contribute to
economic growth but only are necessary to meet the demands of domestic
consumption coupled with a GDP which is not growing fast enough, has put the
economy on a dangerous footing. Some of the effects of sustained CAD are Currency
depreciation, Inflation, reduction of credit rating of the country affecting foreign
investment, Outflow of foreign currency due to fears of value erosion, leading to
further economic deterioration. This also aids depreciation of rupee.

81. Solution: d)
Foreign Direct Investment (FDI) is preferred to the foreign portfolio investments
(PIS) primarily because FDI is expected to bring modern technology, managerial
practices and has a longterm nature of investment. The government has liberalized
FDI norms overtime. As a result, only a handful of sensitive sectors now fall in the
prohibited zone and FDI is allowed fully or partially in the rest of the sectors.
Despite successive moves to liberalize the FDI regime, India is ranked fourth on the
basis of FDI Restrictiveness Index (FRI) compiled by OECD.

82. Solution: d)

http://www.insightsonindia.com

INSIGHTS

Page 35

Insights Mock Test 2015: Test 17 Solutions


The International Monetary Fund (IMF) came up in 1944 with the main functions as
exchange rate regulation, purchasing short-term foreign currency liabilities of the
member nations from around the world, allotting special drawing rights (SDRs) to
the member nations and the most important one as the bailor to the member
economies in the situation of any BoP crisis.
The main functions of the IMF are as given below:

to facilitate international monetary cooperation;

to promote exchange rate stability and orderly exchange arrangements;


to assist in the establishment of a multilateral system of payments and the
elimination of foreign exchange restrictions; and
to assist member countries by temporarily providing financial resources to
correct maladjustment in their balance of payments (BoPs).

The Board of Governors of the IMF consists of one Governor and one Alternate
Governor from each member country. For India, Finance Minister is the Ex-officio
Governor while the RBI Governor is the Alternate Governor on the Board.

83. Solution: b)
Revolutionary terrorism again became attractive after the abrupt end to the Non
Cooperation movement. It is not accidental that nearly all the major new leaders of
the revolutionary terrorist politics, for example, Jogesh Chandra Chatterjea, Surya
Sen, Jatin Das, Chandrashekhar Azad, Bhagat Singh, Sukhdev, Shiv Varma,
Bhagwati Charan Vohra and Jaidev Kapur, had been enthusiastic participants in the
non-violent Non-Cooperation Movement.

84. Solution: a)
The tax which has incidence and impact at the different points is the indirect tax
the person who is hit does not bleed6 someone else bleeds. As, for example, excise,
sales tax, etc which are imposed on either producers or the traders, but it is the
general consumers who bear the burden of tax.
http://business.gov.in/taxation/corporate.php

http://www.insightsonindia.com

INSIGHTS

Page 36

Insights Mock Test 2015: Test 17 Solutions


85. Solution: a)
The value added tax (VAT) is a method of tax collection as well as name of a state
level tax ( at present) in India. A tax collected at every stage of value addition, i.e.,
either by production or distribution is known as value added tax. The name itself
suggests that this tax is collected on the value addition (i.e., production).
Production of goods or services is nothing but stages of value additions where
production of goods is done by the industrialists or manufacturers. But these goods
require value addition by different service providers/ producers (the agents, the
wholesalers and the retailers) before they reach the consumers. From production to
the level of sale, there are many points where value is added in all goods. VAT
method of tax collection is different from the non-VAT method in the sense that it is
imposed and collected at different points of value addition chain, i.e., multi-point tax
collection.
That is why there is no chance of imposing tax upon tax which takes place in the
non-VAT method single point tax collection. This is why VAT does not have a
cascading effect on the prices of goods it does not increase inflationand is
therefore highly suitable for an economy like India where due to high level of
poverty large number of people lack the market level purchasing capacity. It is a
pro-poor tax system without being anti-rich because rich people do not suffer either.

86. Solution: d)
The Goods and Services Tax (GST) is a proposal18 of tax in India which will emerge
after merging many of the state and central level indirect taxes. Important points of
the proposed GST are as follows:

It will be a tax collected on the VAT methodhaving all the benefits of a VAT
kind of tax.

It will be imposed all over the country with the uniformity of rate and will
replace multiple central and state taxes (a single VAT it will be known). The
taxes to be withdrawn or merged into the GST are

Central Taxes: CENVAT, service tax, sales tax and stamp duty.
State Taxes: State excise, sales tax, entry tax, lease tax, works contract tax, luxury tax,
octroi, turnover tax and cess.

87. Solution: a)
http://www.insightsonindia.com

INSIGHTS

Page 37

Insights Mock Test 2015: Test 17 Solutions


The Union Budget 2013-14 introduced (basically, reintroduced) the CTT, however, only
for non-agricultural commodity futures at the rate of 0.01 per cent (which is
equivalent to the rate of equity futures on which a Securities Transaction Tax is
imposed in India). Alongwith this, transactions in commodity derivatives have been
declared to be made non-speculative; and hence for traders in the commodity
derivative segment, any losses arising from such transactions can be set off against
income from any other source (similar provisions are also applicable for the
securities market transactions).
Like all financial transaction taxes, CTT aims at discouraging excessive speculation,
which is detrimental to the market and to bring parity between securities market and
commodities market such that there is no tax/regulatory arbitrage. Futures contracts
are financial instruments and provide for price risk management and price discovery
of the underlying asset commodity / currency / stocks / interest. It is, therefore,
essential that the policy framework governing them is uniform across all the
contracts irrespective of the underlying assets to minimize the chances of regulatory
arbitrage. The proposal of CTT also appears to have stemmed from the general
policy of the government to widen the tax base.

88. Solution: d)
This is a direct tax and applies on the sales of all assets if a profit (gain) has been
made by the owner of the asset a tax on the gains one gets by selling assets. The
tax has been classified into two
(i) Short Term Capital Gain (STCG): It applies if the Asset has been sold within 36
months of owning it. In this case the rate of this tax is similar to the normal income
tax slab. But the period becomes 12 months in cases of shares, mutual funds, units
of the UTI and zero coupon bond in this case the rate of this tax is 15 per cent.
(ii) Long Term Capital Gain (LTCG): It applies if the asset has been sold after 36
months of owning it. In this case the rate of this tax is 20 per cent. In cases of
shares, mutual funds, units of the UTI and zero coupon bond there is exemption
(zero tax) from this tax (provided that such transaction is subject to Securities
Transaction Tax).

http://www.insightsonindia.com

INSIGHTS

Page 38

Insights Mock Test 2015: Test 17 Solutions


89. Solution: d)
Borrowings by the Government include all long-term loans raised by the
government inside the country (i.e., internal borrowings) and outside the country
(i.e., external borrowings). Internal borrowings might include the borrowings from
the RBI, Indian banks, financial institutions, etc. Similarly, external borrowings
might include the loans from the World Bank, the IMF, foreign banks, foreign
governments, foreign financial institutions, etc.

90. Solution: d)

Explanation:
Apart from the higher capital cost of providing telecom services in rural and remote
areas, these areas also generate lower revenue due to lower population density, low
income and lack of commercial activity. Thus normal market forces alone would not
direct the telecom sector to adequately serve backward and rural areas. Keeping in
mind the inadequacy of the market mechanism to serve rural and inaccessible areas
on one hand and the importance of providing vital telecom connectivity on the
other, most countries of the world have put in place policies to provide Universal
Access and Universal Service to ICT.
The New Telecom Policy - 1999 (NTP99) provided that the resources for meeting the
Universal Service Obligation (USO) would be raised through a Universal Access
Levy (UAL), which would be a percentage of the revenue earned by the operators
under various licenses. The Universal Service Support Policy came into effect from
01.04.2002. The Indian Telegraph (Amendment) Act, 2003 giving statutory status to
the Universal Service Obligation Fund (USOF) was passed by both Houses of
Parliament in December 2003. The Rules for administration of the Fund known as
Indian Telegraph (Amendment) Rules, 2004 were notified on 26.03.2004. As per the
Indian Telegraph Act 1885 (as amended in 2003, 2006 and 2008), the Fund is to be
utilized exclusively for meeting the Universal Service Obligation. This fund is non
lapsable.

91. Solution: a)
Explanation:
http://www.insightsonindia.com

INSIGHTS

Page 39

Insights Mock Test 2015: Test 17 Solutions


The Telecom Regulatory Authority of India (TRAI) was established with effect from
20th February 1997 by an Act of Parliament, called the Telecom Regulatory
Authority of India Act, 1997, to regulate telecom services, including
fixation/revision of tariffs for telecom services which were earlier vested in the
Central Government. TRAI is administered through a Secretariat headed by a
secretary.

92. Solution: c)
Explanation:
An exclusive economic zone (EEZ) is a sea zone prescribed by the United Nations
Convention on the Law of the Sea over which a state has special rights regarding the
exploration and use of marine resources, including energy production from water
and wind. It stretches from the baseline out to 200 nautical miles (nmi) from its
coast.

93. Solution: a)
Explanation:
One-rank one-pension scheme
This is a scheme which will ensure that soldiers of the same rank and the same
length of service receive the same pension, irrespective of their retirement date. In
simple words, it demands equal pensions for those who have retired in one
particular year, as those who retire in another year at the same position, and for the
same duration of services rendered.

The difference in the pension of present and past pensioners in the same rank
occurs on account of the number of increments earned by the defence
personnel in that rank.

So far, there was no such rule. While every pay commission bumps the
salaries of government servants, pensions of ex-servicemen remain the same.

http://www.insightsonindia.com

INSIGHTS

Page 40

Insights Mock Test 2015: Test 17 Solutions

Supreme Court in a ruling had stated that Pension is not a bounty nor a
matter of grace depending upon the sweet will of the employer. It is not an
ex-gratia payment, but a payment for past services rendered. The
significance of OROP is therefore justified.

The implementation of one rank, one pension is also expected to push up the
Centres defence pension payments by a record 40 per cent, posing fresh
challenges to keep the Centres fiscal deficit within the budgetary target of 4.1
per cent of the Gross Domestic Product.

94. Solution d)
Explanation:
Nirbhay is an all-weather low-cost long-range cruise missile with stealth and high
accuracy. The missile has a range of more than 1000 km. It weighs about one tonne
and has a length of 6 metres. Its relatively slow flight speed, just 1,000 km per hour,
allows it to navigate its way precisely to the target.
The Nirbhay cruise missile is an Indian version of the American Tomahawk, which
became an icon of high-tech warfare in the 1991 Gulf War through televised CNN
footage of Tomahawks flying through the streets of Baghdad and precisely entering
target buildings through open windows.
It carries a ring laser gyroscope for high-accuracy navigation and a radio altimeter
for the height determination. It is capable of being launched from multiple platforms
on land, sea and air and shall be inducted into Indian Navy, Army, and Air Force. In
particular, Nirbhay is being adapted for the Indo/Russian Su-30MKI. The missile is
capable of carrying nuclear warheads.
The two-stage missile Nirbhay is able to pick out a target and attack it among
multiple targets. The missile has a loitering capability, i.e., it can go round a target
and perform several manoeuvres and then re-engage it. Flying at treetop level and
navigating its way through heavily defended enemy airspace where a manned
fighter would be quickly shot down by anti-aircraft missiles and guns, the Nirbhay
is better equipped to survive the flight to its target.
The missile is capable of flying at different altitudes ranging from 500 m to 4 km
above the ground and can also fly at low altitudes to avoid detection by enemy
radar.

http://www.insightsonindia.com

INSIGHTS

Page 41

Insights Mock Test 2015: Test 17 Solutions

95. Solution: d)
Explanation:
Agni-V has a strike range of over 5000 kms and can carry a nuclear warhead of over
one tonne, off Odisha coast recently. It is a three stage, solid propellant missile. It
will extend Indias reach all over Asia, parts of Africa and parts of Europe.
Agni V can be configured to launch small satellites. It can also be used to shoot
down enemy satellites in orbits. Once fired, it cannot be stopped. It can, however, be
launched only after a decision by the Cabinet Committee on Security (CCS).

96. Solution: a)
All the areas which get capital from the government are part of the capital
expenditure. It includes so many heads in India

Loan Disbursals by the Government


The loans forwarded by the government might be internal (i.e., to the states,
UTs, PSUs, FIs, etc.) or external (i.e., to foreign countries, foreign banks,
purchase of foreign Bonds, loans to IMF and WB, etc.).
Loan Repayments by the Government of the Borrowings Made in the Past
Again loan payments might be internal as well as external. This consists of
only the capital part of the loan repayment as the element of interest on loans
are shown as a part of the revenue expenditure.
Plan Expenditure of the Government
This consists of all the expenditures incurred by the government to finance
the planned development of India as well as the central government financial
supports to the states for their plan requirements.
Capital Expenditures on Defence by the Government
This consists of all kinds of capital expenses to maintain the defence forces, the
equipment purchased for them as well as the modernisation expenditures. It
should be kept in mind that defence is a non-plan expenditure which has
capital as well as revenue expenditures element in its maintenance. The
revenue part of expenditure in the defence is counted in the revenue
expenditures by the government.

http://www.insightsonindia.com

INSIGHTS

Page 42

Insights Mock Test 2015: Test 17 Solutions

97. Solution: b)
The fiscal deficit excluding the interest liabilities for a year is the primary deficit, a
term India started using since the fiscal 199798.
The part of the fiscal deficit which was provided by the RBI to the government in a
particular year is Monetised Deficit, this is a new term adopted since 199798 in
India. This is shown in both the formsin quantitative as well as a percentage of the
GDP for that particular financial year.
It is an innovation in the fiscal management which brings in more transparency in
the governments expenditure behaviour and also in its capabilities concerning its
dependence on market borrowings by the RBI.

98. Solution: b)
Once deficit financing became an established part of public finance around the
world, the means of going for it were also evolved by that time. These means,
basically are the ways in which the government may utilise the amount of money
created as the deficit to sustain its budget for developmental or political needs.
Essentially the best mode of deficit financing should cause least inflation possible.
Refer to the section on means of deficit financing for details in Ramesh Singh Chap
18 Public Finance.

99. Solution: d)
This old trilemma asserts that a country cannot maintain, simultaneously, all three
policy goals of (a) free capital flows, (b) a fixed exchange rate, and (c) an
independent monetary policy. The impossible trinity, has seen enough waters
flowing down the time since it was articulated almost five decades ago which has a
strong theoretical foundation in the Mundell-Fleming Model developed in the 1960s.
Dani Rodrik argued that if a country wants more of globalisation, it must either give
up some democracy or some national sovereignty. Niall Ferguson highlighted the

http://www.insightsonindia.com

INSIGHTS

Page 43

Insights Mock Test 2015: Test 17 Solutions


trilemma of a choice between commitment to globalisation, to social order and to a
small state (meaning limited state intervention).

100.

Solution: c)

http://en.wikipedia.org/wiki/Comparative_Rating_Index_of_Sovereigns
http://www.thehindu.com/business/cris-is-indias-new-ratingindex/article2848509.ece

http://www.insightsonindia.com

INSIGHTS

Page 44

INSIGHTS ON INDIA MOCK PRELIMINARY EXAM - 2015


INSIGHTS ON INDIA MOCK TEST - 18
GENERAL STUDIES

PAPER-I
Time Allowed: 2 Hours

Maximum Marks: 200

INSTRUCTIONS
1. IMMEDITELY AFTER THE COMMENCEMENT OF THE EXAMINATION, YOU SHOULD
CHECK THAT THIS TEST BOOKLET DOES NOT HAVE ANY UNPRINTED OR TORN OR
MISSING PAGES OR ITEMS, ETC. IF SO, GET IT REPLACED BY A COMPLETE TEST BOOKLET.
2. You have to enter your Roll Number on the Test I
Booklet in the Box provided alongside. DO NOT
write anything else on the Test Booklet.
4. This Test Booklet contains 200 items (questions). Each item is printed only in English. Each item
comprises four responses (answers). You will select the response which you want to mark on the Answer
Sheet. In case you feel that there is more than one correct response, mark the response which you consider
the best. In any case, choose ONLY ONE response for each item.
5. You have to mark all your responses ONLY on the separate Answer Sheet provided. See directions in the
Answer Sheet.
6. All items carry equal marks.
7. Before you proceed to mark in the Answer Sheet the response to various items in the Test Booklet, you
have to fill in some particulars in the Answer Sheet as per instructions sent to you with your Admission
Certificate.
8. After you have completed filling in all your responses on the Answer Sheet and the examination has
concluded, you should hand over to the Invigilator only the Answer Sheet. You are permitted to take away
with you the Test Booklet.
9. Sheets for rough work are appended in the Test Booklet at the end.
10. Penalty for wrong answers :
THERE WILL BE PENALTY FOR WRONG ANSWERS MARKED BY A CANDIDATE IN THE
OBJECTIVE TYPE QUESTION PAPERS.
(i)

There are four alternatives for the answer to every question. For each question for which a
wrong answer has been given by the candidate, one-third of the marks assigned to that question
will be deducted as penalty.

(ii) If a candidate gives more than one answer, it will be treated as a wrong answer even if one of the
given answers happens to be correct and there will be same penalty as above to that question.
(iii)

If a question is left blank, i.e., no answer is given by the candidate, there will be no penalty for
that question.
http://insightsonindia.com

INSIGHTS ON INDIA MOCK TEST SERIES FOR CIVIL SERVICES PRELIMINARY EXAM 2015

http://insightsonindia.com

Page 1

1. Consider the following statements about


District Planning Committees.
1. It is constitutionally mandated for
each state to constitute a District
Planning Committee (DPC).
2. The DPC does not plan for the urban
based Municipalities.
3. All members of the DPC are elected
indirectly.
Choose the correct answer using the codes
below.
a)
b)
c)
d)

1 only
2 and 3 only
1 and 3 only
All of the above

2. Consider the following statements about


the composition and functioning of
Municipal Corporations (MCs).
1. The composition of the MCs is
governed solely by State laws.
2. The Municipal Commissioner heads
the council in the MCs.
3. All decisions in the MCs are subject to
the approval of the Municipal
Commissioner.
Choose the correct answer using the codes
below.
a)
b)
c)
d)

1 only
2 and 3 only
1 and 3 only
None of the above

3. Consider the following statements.


1. The constitution provides for a
uniform administrative system in all
Union Territories (UTs).
2. An administrator of a union territory
is an agent of the President and not
head of state like a Governor.
http://insightsonindia.com

3. The governor is to act independently


of his council of ministers if he is
administering a UT too.
Choose the correct answer using the codes
below.
a)
b)
c)
d)

1 and 2 only
2 and 3 only
1 and 3 only
All of the above

4. Which of the following statements is


correct about the legislative control of
Parliament over the Union Territories
(UTs)?
a) The Parliament cannot make laws in
State List for those UTs that have a
legislative assembly.
b) The Parliament can make laws in
State List for even those UTs that have
a legislative assembly but only after
prior recommendation of the
President.
c) The Parliament can make laws on any
subject of the three lists (including the
State List) for the union territories.
d) None of the above.

5. Consider the following statements with


reference to the National Tiger
Conservation Authority:
1. It is a statutory body under the
Ministry of Environment, Forests and
Climate Change.
2. It approves the tiger conservation plan
prepared by the State Government.
3. It is set up under the Chairmanship of
the Minister for Environment and
Forests.
Page 2

Which of the above statements are true?


a)
b)
c)
d)

Only 1.
Only 1 & 3.
Only 2 & 3.
All.

6. Consider the following statements


regarding the Karchi Session of Indian
National Congress, 1931:
1. Resolutions on Fundamental Rights
and the National Economic
Programme were passed and adopted
during this session
2. The session, which was under public
pressure after the execution of Bhagat
Singh, rejected a resolution on the
Gandhi-Irwin
Which of the above statements is/are correct?
a)
b)
c)
d)

1 Only
2 Only
Both
None

7. The Government has proposed a


Comprehensive Bankruptcy Code of global
standards to be brought in fiscal 2015-16.
This will help in improving
a) Ease of doing business in India
b) Banking standards in India
c) The situation of Micro-finance
Institutions in India
d) The situation of debt-ridden poor
families

8. The Government has announced Tax free


infrastructure bonds in the recent Budget
for which of the following sectors?
1. Rail
2. Road
3. Irrigation
http://insightsonindia.com

4. Energy
Choose the correct answer using the codes
below.
a) 1, 2 and 3 only
b) 2 and 3 only
c) All of the above
d) 1 and 4 only

9. SETU - Self-Employment and Talent


Utilization has been established in the
recent budget to support entrepreneurs in
which of the following domains?
1. Technology related issues
2. Financial issues
3. Facilitation to start-up the business
Choose the correct answer using the codes
below.
a) 1 and 2 only
b) 2 and 3 only
c) 1 and 3 only
d) All of the above

10. In the latest Annual Budget 2015-16,


highest tax revenue has been generated
from
a) Corporation tax
b) Excise duty
c) Service Tax
d) Income Tax

11. Arrange the following items in order of


increasing quantum of spending in the
recent Annual budget 2015-16.
1. Subsidies
2. Defense Services
3. Grants to State and UTs
4. Interest Payments
Choose the correct order from the codes
below.
Page 3

a) 1243
b) 4123
c) 4321
d) 3412

12. The Government proposes to set up 5 new


Ultra Mega Power Projects (UMPPs), each
of 4000 mw capacity in the plug-and-play
mode. What is meant by plug-and-play
mode in this context?
a) The UMPPs will be a high end
technology venture supported by the
government from the very initial
phases
b) All clearances and linkages will be in
place before the project is awarded by
the Government
c) The basic infrastructure of the UMPPs
will be setup by the government, and
then ownership will be transferred to
the private sector for further
upgradation
d) None of the above

13. The Finance Minister has proposed to set


up a Public Debt Management Agency
(PDMA) in the latest Budget. Consider the
following statements about it.
1. PDMA will take care of both Indias
external borrowing and domestic
debt.
2. It will purchase and re-issue various
bonds.
3. The agency will also regulate the
security markets in the country.
Choose the correct answer using the codes
below.
a) 1 and 2 only
b) 2 and 3 only
http://insightsonindia.com

c) 1 and 3 only
d) All of the above

14. In the recent Budget, the Government has


mooted to bring an India Financial Code.
Which of the following domains will be
included in the code?
1. Public Debt management
2. Financial Consumer protection
3. Monetary Policy
4. Reducing systemic risks in the Indian
financial system
Choose the correct answer using the codes
below.
a) 1 and 2 only
b) 2, 3 and 4 only
c) 1 and 3 only
d) All of the above

15. The Ministry of Finance is composed of:


1. Department of Economic Affairs.
2. Department of Expenditure.
3. Department of Revenue.
4. Department of Disinvestment.
5. Department of Financial Services.
Choose the answer using the codes below:
a)
b)
c)
d)

Only 1, 2, 3 & 5.
Only 2, 3, 4 & 5.
Only 1, 2, 3 & 4.
All.

16. Indias foreign exchange reserves


comprise:
1. Foreign Currency Assets.
2. Gold.
3. Special Drawing Rights (SDRs).
Page 4

4. Reserve Tranche Position in the IMF.

Choose the answer using the codes below:


a)
b)
c)
d)

Only 1, 2 & 3.
Only 2, 3 & 4.
Only 1 & 3.
All.

17. Which of the following functions are


performed by the Central Statistical
Office?
1. Compilation of National Accounts.
2. Conduct of Annual Survey of
Industries and Economic Censuses.
3. Compilation of Index of Industrial
Production.
Choose the correct answer using the codes
below:
a)
b)
c)
d)

Only 1.
Only 2 & 3.
Only 1 & 3.
All.

18. Consider the following statements about


the Election Commission of India (ECI).
1. Being an all-India body it serves both
the Central and State Governments.
2. It is not at all concerned with elections
to any level of Panchayats and
Municipalities.
3. The State election Commissions work
under the overall supervision of the
ECI.
Choose the correct answer using the codes
below.
a)
b)
c)
d)

1 and 2 only
2 and 3 only
1 and 3 only
All of the above

http://insightsonindia.com

19. Which of the following fiscal indicators


mentioned in the Annual budget has/have
increased as a percentage of GDP from
last year?
1. Revenue Expenditure
2. Gross Tax revenue
3. Tax devolution to states
4. Total outstanding liabilities of the
Government
Choose the correct answer using the codes
below.
a) 2 and 3 only
b) 1, 2 and 3 only
c) 2, 3 and 4 only
d) 1 and 4 only

20. As per the latest World Bank data, arrange


the following countries in decreasing
order of per capital GHG emissions.
1. USA
2. Australia
3. India
4. Indonesia
Choose the correct arrangement from the
codes below.
a) 1243
b) 1234
c) 2134
d) 2143

21. The PAT (perform - achieve - trade), as a


part of the National Action Plan for
Climate Change Mission, is a scheme for
a) trading energy-efficiency certificates in
large energy-intensive industries
b) trading carbon credits between
developing economies
c) trading GHG credits between energy
intensive industries within a country
d) None of the above
Page 5

22. A ordinance can only be issued by the


Lieutenant Governor (LG) in Delhi UT
when
a) The Chief Minister of Delhi has
endorsed it.
b) The CM along with the Council of
Ministers has endorsed it.
c) The President has given prior
recommendation to issue it.
d) The LG is satisfied that the situation
demands it and no approval is
required at the time of issuing the
ordinance.

23. Coral Reefs in India are found in:


1. Gulf of Mannar.
2. Gulf of Kachchh.
3. Lakshadweep.
4. Andaman and Nicobar Islands.
Choose the correct answer using the codes
below:
a)
b)
c)
d)

Only 1.
Only 1 & 2.
Only 1, 3 & 4.
All of the above.

24. Consider the following statements:


1. Nine of the eighteen biosphere
reserves in India are a part of the
World Network of Biosphere
Reserves, based on the UNESCO
Man and the Biosphere (MAB)
Programme list.
2. The Nilgiri Biosphere Reserve was
the first biosphere reserve in India
established in the year 1986.
3. In Biosphere Reserves protection is
granted not only to the flora and
fauna of the protected region, but
also to the human communities who
http://insightsonindia.com

inhabit these regions, and their ways


of life.
Which of the above statements are true?
a)
b)
c)
d)

Only 1.
Only 2 & 3.
Only 1 & 2.
All.

25. Consider the following statements with


reference to the National Biodiversity
Authority:
1. It is a statutory autonomous body
under the Ministry of Environment
and Forests.
2. The recommendations made by it
are advisory in nature.
3. It is headed by the Prime Minister
of India.
Which of the above statements are true?
a)
b)
c)
d)

Only 1 & 2.
Only 3.
Only 1 & 2.
All.

26. The Convention on Biological Diversity


(CBD):
1. Is a legally binding multilateral
environmental agreement.
2. Has 193 contracting Parties
(Countries) as its members.
3. Provides for equitable sharing of
benefits arising from the utilization
of traditional knowledge, and
practices, with holders of such
knowledge.
Which of the above statements are true?
a)
b)
c)
d)

Only 1.
Only 2 & 3.
Only 1 & 3.
All.
Page 6

d) None

27. Which of the following activities have


been recently brought under MGNREGA
as part of Green India Mission?
1. Water Harvesting
2. Afforestation
3. Farm Forestry
Choose the correct answer using the codes
below.
a)
b)
c)
d)

1 and 2 only
2 and 3 only
1 and 3 only
All of the above

30. NASAs Hubble Space Telescope has


recently spotted massive saltwater ocean
under the icy crust of Jupiters moon
Ganymede. Consider the following
statements about Ganymede.
1. It is the largest moon in our Solar
system.
2. It is the only moon with its own
magnetic field.
3. The moon also creates ribbons of hot
electrified gas called as auroras.
Choose the correct answer using the codes
below.

28. Researchers have recently found


Endophytic alga thriving in the Indian
Ocean. Its discovery is significant because
a) It can be used to make bio-fuels
b) It can be used to clean up oil spill
disasters in oceans
c) It contains anticancer compounds
such as Taxol
d) It can act as an effective phytoremediator

29. With reference to the Communal Award


announced by the British government in
August 1932, consider the following
statements:
1. According to it, Muslims would be
elected only by Muslims to the
legislatures
2. This Award declared that the
Depressed Classes as minorities and
would be treated on par with Muslims
or any other religious minorities

a)
b)
c)
d)

1 and 2 only
2 and 3 only
1 and 3 only
All of the above

31. Recently Pakistan has successfully tested


some of its first indigenously built defence
missiles and equipments. In this context,
what are Burraq and Barq?
a) All-weather armed drone and laser
guided missile
b) Laser guided missile and all-weather
armed drone
c) Inter-Continental Ballistic Missile
(ICBM) and Unarmed surveillance
drones
d) Unarmed surveillance drones and
Inter-Continental Ballistic Missile
(ICBM)

Which of the above statements is/are correct?


a) 1 Only
b) 2 Only
c) Both
http://insightsonindia.com

Page 7

32. Japan and France have recently signed an


arms transfer agreement for military
equipment, services and technology. This
will help Japan in developing
1. Drones
2. Unmanned technologies for
underwater search missions
3. Amphibian Flight technologies
Choose the correct answer using the codes
below.
a)
b)
c)
d)

1 and 2 only
2 and 3 only
1 and 3 only
All of the above

33. Consider the following statements with


reference to the Right of Children to Free
and Compulsory Education Act, 2009:
1. According to the Act, every child
in the age group of 6 to 14 years
has Right to elementary
education.
2. The Act makes provisions for a
non-admitted child to be admitted
to an age appropriate class.
3. Education in the Indian
constitution is a concurrent issue
and both centre and states can
legislate on the issue.

the National Policy on Education,


1986.
2. The Mahila Samakhya Scheme is
currently being implemented in all
states across the country except
the North Eastern States and
Sikkim.
Which of the above statements are true?
a)
b)
c)
d)

Only 1.
Only 2.
Both.
None

35. The Economic Survey points out that the


Indian banking system is afflicted by what
might be called double financial
repression. It means
1. Returns to savers and bankers are
reduced
2. Misallocation of capital to investors
3. Not enough capital for government to
borrow from the market
Choose the correct answer using the codes
below.
a) 1 and 2 only
b) 2 and 3 only
c) 1 and 3 only
d) All of the above

Which of the above statements are true?


a)
b)
c)
d)

Only 1.
Only 2 & 3.
Only 1 & 3.
All.

34. Consider the following statements with


reference to Mahila Samakhya
Programme:
1. The programme was launched in
1989 to pursue the objectives of
http://insightsonindia.com

36. Consider the following statements about


the role of UPSC.
Assertion (A): It has to be consulted in all
promotion and disciplinary matters
concerned with higher Public Services.
Reason (R): It is the Central personnel agency
in India.
In the context of the statements above, which
of these is true?
Page 8

a) A and R both are true, and R is the


correct explanation for A.
b) A and R both are true, and R is the
NOT the correct explanation for A.
c) A is correct, R is incorrect.
d) A and R both are incorrect.

37. Article 280 of the Constitution of India


provides for a Finance Commission.
Which kind of a body is Finance
Commission?
a) Quasi-judicial
b) Executive
c) Administrative
d) Quasi-legislative

38. As per the qualifications prescribed by


law, who among the following is/are fit to
become the Chairman of the Finance
Commission?
1. A high court Judge
2. A person who has special knowledge
in economics but has not worked for
the Government anytime before
3. A person who has served for two or
more years as Secretary to the
Government of India
Choose the correct answer using the codes
below.
a)
b)
c)
d)

1 only
2 and 3 only
1 and 3 only
3 only

39. Consider the following statements.


Assertion (A): The recommendations made by
the Finance Commission binding on the
government.
Reason (R): Finance Commission is a
constitutional body.
http://insightsonindia.com

In the context of the statements above, which


of these is true?
a) A and R both are true, and R is the
correct explanation for A.
b) A and R both are true, and R is the
NOT the correct explanation for A.
c) A is incorrect, R is correct.
d) A and R both are incorrect.

40. Which of these is/are Constitutional


bodies?
1. National Commission for SCs
2. National Commission for Minorities
3. National Commission for Protection
of Child Rights
4. National Human Rights Commission
Choose the correct answer using the codes
below.
a)
b)
c)
d)

1 only
1 and 4 only
All of the above
2, 3 and 4 only

41. Consider the following statements about


the National Commission for SCs.
1. The Chairman of the Commission is
appointed by a Selection panel which
includes he PM, Lok Sabha Speaker
and Leader of Opposition, Lok Sabha.
2. The commission presents an annual
report to the president which is
presented to Parliament.
3. The orders of the commission are
binding on the Government.
Choose the correct answer using the codes
below.
a) 1 only
b) 2 only
c) 2 and 3 only
d) 1 and 3 only
Page 9

42. As per the official view expressed in the


recent Economic Survey, which of the
following factors may have been
responsible for hindering private
investment in India in last few years?
1. Deficiencies of the public private
partnership (PPP) model in
infrastructure
2. Weak corporate balance sheets
3. High NPAs of banks
Choose the correct answer using the codes
below.
a) 1 and 2 only
b) 2 and 3 only
c) 1 and 3 only
d) All of the above

43. Consider the followings statements with


reference to Rajiv Gandhi Grameen
Vidyutikaran Yojana:
1. The scheme is being implemented
through the Rural Electrification
Corporation (REC) which has been
designated as Nodal Agency by
Ministry of Power.
2. The scheme provides for free of cost
connection to all rural households
living below poverty line.
3. It is a 100% centrally sponsored
scheme.
Which of the above statements are true?
a)
b)
c)
d)

Only 1.
Only 1 & 2.
Only 1 & 3.
All.

44. Consider the following statements about


the Climate Investment Fund (CIF).
1. It is for developing and middle-income
countries only.

http://insightsonindia.com

2. It also funds investment in clean


technologies.
3. It provides funding support for
development planning, including
public and private sector projects.
Choose the correct answer using the codes
below.
a) 1 and 2 only
b) 2 and 3 only
c) 1 and 3 only
d) All of the above

45. Consider the following statements.


1. As per the IMF, every fast-growing
Asian economy in recent years has
accelerated as it underwent a
demographic transition.
2. In India, the higher growth states have
witnessed a greater demographic
dividend than the lower growth states
in the last two decades.
Which of the above is/are true?
a) 1 only
b) 2 only
c) Both 1 and 2
d) None

46. Which of the following virtuous cycles of


economic growth is related with the
demographic dividend (DD)?
a) DD-> Lower dependency ratio (LDR) > High Growth-> Low fertility -> LDR
b) DD-> High employment -> High
growth -> LDR-> High employment
c) DD-> High Growth -> High
Employment -> DD
d) None of the above

Page 10

47. With reference to Gandhijis views on


untouchability, consider the following
statements:
1. According to him the very existence of
Hinduism was at risk if untouchability
continued to exist
2. He asked his followers to ignore
shastras if they preached
untouchability even in subtle form
3. Gandhiji not being hostile to Dr
Ambedkars or other Harijans
criticisms was a way of undergoing
penance for the mistakes committed by
caste Hindus against lower castes
4. It can be said that acceptance of
reservations for scheduled castes by
the caste Hindus during the early years
after gaining independence was a
reaction to Gandhijis please for
penance and reparation
Which of the above statements is/are correct?
a) 1, 2 and 3 Only
b) 2 and 3 Only
c) 1, 2, and 4 Only
d) All

48. Which of the following accurately defines


employment rate?
a) growth in those looking for work who
actually find it
b) growth in the fraction of those who can
work and that actually look for work
c) the total percentage of people
employed in the country divided by
the total working age population
d) None of the above

49. Total Factor Productivity (TFP) is the


portion of output not explained by the
a) amount of input used in production
b) investment put in production
c) wages of labour needed for production
http://insightsonindia.com

d) market supply and demand forces

50. Which of the following provisions have


been announced by the Government in the
recent Budget concerning agricultural
marketing?
1. Need to create a National Agriculture
Market
2. A new Central Warehousing agency
will be set up
3. Private sector will be allowed to
purchase grains directly from farmers
from their doorsteps.
Choose the correct answer using the codes
below.
a) 1 and 2 only
b) 2 and 3 only
c) 1 and 3 only
d) 1 only

51. Which of the following freedom fighter is


NOT considered as New Swarajist?
a) Bhulabhai Desai
b) Asaf Ali
c) M.A.Ansari
d) None of the above

52. Consider the following statements with


reference to the Accelerated Rural
Electrification Programme:
1. Assistance under the Scheme will
be available for electrification by
conventional and nonconventional sources of energy.
2. The interest subsidy under the
Scheme is 4%.
3. Funds would be provided under
the scheme on the basis of Net
Present Value of the interest
subsidies applicable on

Page 11

disbursement under Accelerated


Rural Electrification Programme.
Which of the above statements are true?
a)
b)
c)
d)

Only 1.
Only 1 & 2.
Only 1 & 3.
All.

53. Consider the following statements:


1. No sanction is required under the
provisions of the Electricity Act,
2003 for setting up captive power
plants.
2. The Electricity Act provides that
any person may construct,
maintain or operate a captive
generating plant and dedicated
transmission lines.
Which of the above statements are true?
a)
b)
c)
d)

Only 1.
Only 2.
Both.
None.

54. The Central Electricity Authority (CEA):


1. Is a Statutory Body.
2. The generating stations need not
compulsorily follow the CEA
technical standards specified for
construction of electrical plants and
electric lines.
3. Is headed by the Union Home
Minister.
Which of the above statements are true?
a)
b)
c)
d)

Only 1.
Only 1 & 2.
Only 1 & 3.
All.

http://insightsonindia.com

55. Satluj Jal Vidyut Nigam was incorporated


as a joint venture of:
a) The government of India and the
government of Himachal Pradesh.
b) The government of India and the
government of Jammu &
Kashmir.
c) The government of India and
NTPC Ltd.
d) The government of India and
NHPC Ltd.
56. Consider the following statements with
reference to the Jawaharlal Nehru
National Solar Mission:
1. The Mission has set the ambitious
target of deploying 20,000 MW of
grid connected solar power by
2022.
2. The Mission is under the aegis of
Ministry of New and Renewable
Energy.
Which of the above statements are true?
a)
b)
c)
d)

Only 1.
Only 2.
Both.
None.

57. India is a party to which of the following


international convention/conventions:
1. Convention on International Trade
in Endangered Species of Wild
Fauna and Flora (CITES).
2. International Whaling Commission
(IWC).
3. United Nations Educational,
Scientific, and Cultural
Organizations World Heritage
Convention (UNESCO WHC).
4. Convention on the Conservation of
Migratory Species of Wild Animals
(CMS).

Page 12

Choose the correct answer using the codes


below:
a)
b)
c)
d)

Only 1.
Only 2, 3 & 4.
Only 1, 2 & 4.
All.

is benefitting rich more than the poor. In


which of these sectors is the distortion
observed?
1. Electricity subsidies
2. Water Utilities subsidies
3. Railways fare
Choose the correct answer using the codes
below.

58. A per a recent study Preliminary


Modeling Studies on Carbon Taxes and
GDP Loss conducted by the Ministry of
Environment and Forest, GoI, carbon
taxes are revenue positive when
a) it involves no adjustment to other tax
rates in the economy.
b) other tax rates are adjusted so that the
revenue inflow from carbon tax is
exactly balanced by an equal reduction
in yields from reduced taxes.
c) it is in line with the carbon credits in
the economy
d) None of the above

a) 1 and 2 only
b) 2 and 3 only
c) 1 and 3 only
d) All of the above

61. Consider the following macroeconomic


indicators.
1. Current Account Deficit
2. Economic Growth
3. Savings
Which of the above has reduced in the last
couple of years as per the Economic Survey
2014-15?

59. Both the central and state governments


subsidise a wide range of products with
the expressed intention of making these
affordable for the poor. These include
1. Iron ore
2. Fertilizer
3. Sugar
4. Pulses

Choose the correct answer using the codes


below.

Choose the correct answer using the codes


below.

62. Consider the following statements.


1. Amongst the BRICS countries, India is
the most attractive foreign investment
destination.
2. Amongst the BRICS countries, China is
the biggest tourist attraction
destination and tourist economy.

a) All of the above


b) 1, 2 and 3 only
c) 2, 3 and 4 only
d) 1 and 4 only

a) 1 and 2 only
b) 2 and 3 only
c) 1 and 3 only
d) All of the above

Which of the above is/are true?


60. One of the important points mentioned in
the recent Economic Survey is the
inequitable distribution of subsidies that
http://insightsonindia.com

a) 1 only
b) 2 only
c) Both 1 and 2
Page 13

d) None

63. The Delhi Legislative assembly can make


laws on all the matters of the State List
and the Concurrent List except the three
matters of the State List, that is
1. Public Order
2. Police
3. Land
Choose the correct answer using the codes
below.
a)
b)
c)
d)

1 and 2 only
2 and 3 only
1 and 3 only
All of the above

64. The Lieutenant Governor is the


Administrator of the Delhi UT. In the case
of difference of opinion between the lt.
governor and his ministers, what option
can he chose?
a) Refer the matter to the president for
decision and act accordingly.
b) Refer the matter to the Legislative
Assembly for decision and act
accordingly.
c) The decision of the Council of
Ministers endorsed by the CM
prevails.
d) The decision of the Lieutenant
Governor, even though opposed by
CM, prevails.

65. With reference to the ideology of the Left


during the freedom struggle, consider the
following statements:
1. It considered the Indian National
Congress as an organisation led by
bourgeois
2. The Left advocated armed struggle
against the British to liberate India
http://insightsonindia.com

3. The Left was Convinced that the


masses were ever ready for struggles in
any form if only the leaders were
willing to initiate them.
Which of the above statements is/are correct?
a)
b)
c)
d)

1 and 3 Only
2 and 3 Only
1 and 2 Only
All

66. Indian banking system has been going


through several challenges. One of the
paradoxes of recent banking history is that
a) The share of the private sector in
overall banking aggregates barely
increased at a time when the country
witnessed its most rapid growth which
was fuelled by the private sector.
b) The share of the private sector in
overall banking aggregates heavily
increased at a time when the country
witnessed its slowest growth in a
decade due to lack of private
investment.
c) Banking NPAs increased at a time
when the private sector stopped
borrowing from banks and investing
d) None of the above

67. Under the scheme Mechanism for


Marketing of Minor Forest Produce
through Minimum Support Price and
development of Value chain for
MFP" Minimum Support Price (MSP) for
Minor Forest Produce (MFP) has been
fixed for some MFP which includes
1. Tamarind
2. Honey
3. Mahua Seed
4. Lac
5. Kusumi

Page 14

Choose the correct answer using the codes


below.
a)
b)
c)
d)

All of the above


2, 4 and 5 only
1, 3 and 4 only
1, 2, 3 and 5 only

68. Consider the following statements about


the Union Public Service Commission
(UPSC).
1. The Chairman and other members are
appointed by the President.
2. The constitution does not mention the
qualification for the post of UPSC
Chairman.
3. One-half of the members of the
Commission should have worked
under either the Government of India
or any state government.
Choose the correct answer using the codes
below.
a)
b)
c)
d)

1 and 2 only
2 and 3 only
1 and 3 only
All of the above

70. In the coming years, which of the


following factors would help increase
Indias economic growth rate?
1. Lowering of Oil Prices
2. Monetary easing
3. Lower Inflation
4. Increase in Savings
Choose the correct answer using the codes
below.
a) All of the above
b) 1, 2 and 3 only
c) 2, 3 and 4 only
d) 1 and 4 only

71. Consider the following statements with


reference to the Zoological Survey of
India:
1. It works under the Ministry of
Environment, Forest and Climate
Change (MoEFCC).
2. The ZSI participates in the Indian
Antarctic Program.
3. Similar to the Red Data Book
produced by IUCN, ZSI also releases a
Red Data Book on Indian Animals.
Which of the above statements are true?

69. If the UPSC tenders an advice to the


Government of India, which of these can
follow?
a) Only the Department of Personnel and
Training (DoPT) can reject the advice
of UPSC on reasonable grounds.
b) Only the Appointments Committee of
the Cabinet can reject UPSCs advice.
c) Any concerned Ministry can reject
UPSCs advice.
d) For UPSCs advice to be rejected,
approval from the Departmental
Standing Committees of the
Parliament is needed.

http://insightsonindia.com

a)
b)
c)
d)

Only 1.
Only 1 & 2.
Only 1 & 3.
All.

72. Consider the following statements:


1. The forest survey report, 2013,
notes down that the overall
Mangrove cover in the county in
2013 has decreased compared to
2011.
2. Mangroves in India account for
about 3% of the worlds mangrove
vegetation.
Page 15

3. Sundarbans in West Bengal


accounts for almost half of the total
area under mangroves in the
country.
Which of the above statements are true?
a)
b)
c)
d)

Only 1.
Only 1 & 2.
Only 1 & 3.
All.

73. Law Commission of India in its 255th


report on electoral reforms recently
submitted to the Government of Indian
has pitched for stronger Election
Commission of India (ECI). Its
recommendations include
1. Equal Constitutional protection to all
members of ECI in removability from
Office
2. Appointment of the Election
Commissioners by a three member
Selection panel
instead of the
present system of appointment by
President
3. The elevation of an EC should be on
the basis of seniority.
Choose the correct answer using the codes
below.
a)
b)
c)
d)

1 and 2 only
2 and 3 only
1 and 3 only
All of the above

74. Union Government has launched Pharma


Jan Samadhan scheme, for redressal of
grievances of consumers related to
1. Over-pricing of Drugs
2. Non-availability of Medicines
3. Non-availability of Doctors in Public
Hospitals
http://insightsonindia.com

Choose the correct answer using the codes


below.
a)
b)
c)
d)

1 and 2 only
2 and 3 only
1 and 3 only
All of the above

75. Which of the following statements is


correct with reference to the use of
Drones in India as of now?
a) The Aircraft Rules do not cover use of
drones as well as its sale and
purchase.
b) A license is required from Ministry of
Defense for sale and purchase of
drones; and a special permission is
required from the Indian Air Force
(IAF) to use drones in India
c) No special permission is needed to
manufacture drones. Only a special
permission is needed from the IAF to
use drones.
d) Drones are not treated as a special
defence equipment and are treated at
par with other defence instruments

76. In the recent years, Economic Survey


points towards a trend of declining
Employment Elasticity of Growth. This
means that
a) High growth has not been able to
create jobs.
b) Disproportionate growth across
various sectors has hindered job
creation.
c) Hiring and firing has become more
flexible and inevitable with higher
economic growth.
d) Share of unorganized sector
employment has been increasing with
high growth rates.

Page 16

77. Which of the following steps taken by the


Government in recent times are growthfriendly?
1. Passing an ordinance to make land
acquisition less onerous
2. Raising gas prices
3. Deregulating diesel prices
4. Passing the Mines and Minerals
(Development and Regulation)
(MMDR) Amendment Ordinance
Choose the correct answer using the codes
below.
a) All of the above
b) 1, 2 and 3 only
c) 2, 3 and 4 only
d) 1 and 4 only

78. In the pursuit of economic growth, ease of


doing business is a very important factor.
To achieve the same, in which of the
following areas, both the Centre and the
State can endeavour jointly?
1. Land Law reforms
2. Labour reforms
3. Reducing the cost of capital
Choose the correct answer using the codes
below.
a) 1 and 2 only
b) 2 and 3 only
c) 1 and 3 only
d) All of the above

79. The JAM number trinity Solution has


been hailed as an important driver of
growth and inclusion in India in the
coming years. It is essentially
a) Technology-enabled Direct Benefit
Transfers
b) M-banking
http://insightsonindia.com

c) Universalizing the Banking


Correspondent model supplemented
with Micro-ATMs
d) USSD based mobile transfers

80.Which of the following factors have been


held responsible for the present decline in
global crude oil prices?
1. Weak Global demand
2. Increase in crude-oil and shale gas
production in US
3. Iraq crisis
Choose the correct answer using the codes
below.
a) 1 and 2 only
b) 2 and 3 only
c) 1 and 3 only
d) All of the above

81. As per the recent Economic Survey,


Indias external sector economy is
vulnerable to which of the following
factors?
1. Risks from shift in US monetary policy
2. Eurozone slowdown
3. Rising non-oil, non-gold imports
Choose the correct answer using the codes
below.
a) 1 and 2 only
b) 2 and 3 only
c) 1 and 3 only
d) All of the above

82. In case of difference of opinion amongst


the Chief Election Commissioner and/or
two other election commissioners, how is
the matter decided?
a) It is decided by majority in the
Commission.
Page 17

b) The Chief Election Commissioners


decision prevails over other
commissioners.
c) The matter is referred to the
Parliamentary Standing Committee on
Subordinate Legislation the decision
of which is final and binding on ECI.
d) If two or more Election
Commissioners disagree on a matter,
then their disagreement will prevail.

83. The chief election commissioner is


provided with the security of tenure. He
cannot be removed from his office except
in same manner and on the same grounds
as that of
a) A Supreme Court Judge
b) UPSC Chairman
c) Comptroller and Auditor General
(CAG) of India
d) Either (a) or (c)

84. Though the constitution has sought to


safeguard and ensure the independence
and impartiality of the Election
Commission of India (ECI), which of the
following has not been specified explicitly
by the Constitution?
1. Qualification of the Election
Commissioners
2. Term of the members of the ECI
3. Manner of Appointment
Choose the correct answer using the codes
below.
a)
b)
c)
d)

1 and 2 only
2 and 3 only
1 and 3 only
All of the above

http://insightsonindia.com

85. Which of the following functions are NOT


performed by the Election Commission of
India?
1. Determining the territorial areas of
the electoral constituencies
2. Advising the president on matters
relating to the disqualifications of the
members of Parliament.
3. To cancel polls in the event of
Violence at election booths
Choose the correct answer using the codes
below.
a)
b)
c)
d)

1 and 2 only
2 and 3 only
1 only
All are performed by the ECI.

86. Elections are being conducted almost


every year in India. Elaborate machinery
is required for election management. Who
acts as the District returning Officer in the
Districts in these elections?
a) District Collector
b) Divisional Commissioner as the Chief
electoral Officer is incharge of the
Districts under him.
c) An Officer appointed by the District
Magistrate
d) An Officer other than the District
Magistrate appointed by the ECI.

87. As per the recent report of the


Intergovernmental Panel on Climate
Change (IPCC), adverse impacts of
Climate Change are likely to be felt more
acutely in
a) Tropical Zone countries
b) Sub-tropical zone countries
c) Temperate Zone countries
d) Polar zone countries

Page 18

88. With reference to the Congress Socialist


Party (CSP) which was formed at Bombay
in October 1934, consider the following
statements:
1. One of its main objectives was to give
the Indian National Congress a social
direction
2. CSP was formed by Jayaprakash
Narayan and others
3. CSPs long-term goal was to root out
Indian National Congress once the
British were driven out and establish a
government based on scialist
principles
Which of the above statements is/are correct?
a) 1 and 2 Only
b) 3 and 2 Only
c) 1 and 3 Only
d) All
89. The Census 2011 reported that the sex
ration in India declined from 927 in 2001,
to 918 in 2011. Alongwith India, which of
these countries witnessed a decline in Sex
ratio in this decade?
1. China
2. Bangladesh
3. Sri Lanka
Choose the correct answer using the codes
below.
a) 1 and 2 only
b) 2 and 3 only
c) 1 and 3 only
d) All of the above

90. For India to have a good fiscal


performance and a high growth rate, the
Economic Survey recommends
expenditure switching from consumption
to investment. It means
a) Directing more investment on gross
capital formation
http://insightsonindia.com

b) Manufacturing more durable goods


c) Producing more intermediate goods
d) Encouraging more savings habit in
India

91. Consider the following statements about


Scheduled Areas.
1. Scheduled areas are declared under
laws passed by the Parliament.
2. The concerned state can not exercise
executive power within the Scheduled
area.
3. The administration of these areas is
entirely under the control of the
President.
Choose the correct answer using the codes
below.
a)
b)
c)
d)

1 and 2 only
2 only
1 and 3 only
None of the above

92. Consider the following statements about


tribal areas in Sixth Schedule.
1. They are completely autonomous of
the State Government.
2. The autonomous districts are
governed by an elected District
Council.
3. The district and regional councils are
empowered to assess and collect land
revenue and to impose certain
specified taxes.
Choose the correct answer using the codes
below.
a)
b)
c)
d)

1 and 2 only
2 and 3 only
1 and 3 only
All of the above

Page 19

93. The district and regional councils under


Sixth Schedule administer the areas under
their jurisdiction. Consider the following
statements about it.
1. They can constitute village councils
or courts for trial of suits and cases
between the tribes.
2. Laws made by them do not require
the assent of either the Legislature or
the Governor.
Which of the above is/are true?
a)
b)
c)
d)

1 only
2 only
Both 1 and 2
None

94. A simple way to compare the relative


economic situation of countries is to
supplement the macro-economic
vulnerability index with a Rational
Investor Ratings Index (RIRI). Which of
the following components are accounted
in RIRI?
1. Inflation
2. Current Account Deficit
3. Fiscal Deficit
4. Currency Exchange Rate
Choose the correct answer using the codes
below.
a) All of the above
b) 1, 2 and 3 only
c) 2, 3 and 4 only
d) 1 and 4 only

95. Land acquisition appears to be a major


challenge. Different state governments are
adopting diverse approaches for dealing
with the issue. Gujarat has a land-pooling
model. Under the model
a) 50 per cent of the land is acquired
while the remaining 50 per cent is left
http://insightsonindia.com

with the original owners giving them a


stake in the upsides generated by land
monetisation.
b) Land is not acquired but pooled from
its owners, who get benefits out of
land monetisation
c) Government does not acquire land, but
leaves it to the industries themselves
who acquire land after a fair
settlement with a pool of owners
d) None of the above

96. HDR measures inequality in terms the


Gini Coefficient which measures
a) inequality in income or consumption
among individuals within a country
b) inequality in income or consumption
among individuals across countries
c) inequality in health and labour force
indicators among individuals within a
country
d) inequality in health and labour force
indicators among individuals across
countries

97. Consider the following statements with


reference to socio-economic development
comparison between Indian states.
1. Bihar had the highest decadal (200111) growth rate of population.
2. The poverty estimates indicate that the
highest poverty headcount ratio
(HCR) exists in Bihar.
3. Lowest poverty is in Himachal Pradesh
followed by Kerala.
Choose the correct answer using the codes
below.
a) 1 and 2 only
b) 2 and 3 only
c) 1 and 3 only
d) All of the above

Page 20

Which of the above is/are true?

98. Rashtriya Swasthya Bima Yojana (RSBY)


scheme provides smart card-based
cashless health insurance to
1. Landless labourers
2. BPL families
3. Unorganized sector workers
Choose the correct answer using the codes
below.
a) 1 in 2
b) 3 in 2
c) 2 only
d) 3 only

99. Which of the following statements


regarding Indias trade with the World is
correct?
1. Before the 2008 Financial crisis,
services exports growth rate outpaced
manufacturing exports growth rate.
2. After the 2008 Financial crisis,
manufacturing exports growth rate
outpaced services exports growth rate.

a) 1 only
b) 2 only
c) Both 1 and 2
d) None

100. The Economic Survey recommends


offsetting the Countervailing Duty for the
success of Make in India initiative.
Countervailing duty is
a) imposed in order to counter the
negative impact of import subsidies to
protect domestic producers
b) imposed on defense equipments for
strategic reasons
c) imposed on goods and services
produced from all SEZs in India
d) imposed on goods and services
produced from all NMIZs (Industrial
Corridors) in India

http://insightsonindia.com

Page 21

Insights Mock Test Series 2015: Test 18 Solutions


1. Solution: a)
Every state shall constitute at the district level, a district planning committee to
consolidate the plans prepared by panchayats and municipalities in the district, and
to prepare a draft development plan for the district as a whole. The state legislature
may make provisions with respect to the following:

The composition of such committees;


The manner of election of members of such committees;
The functions of such committees in relation to district planning; and
The manner of the election of the chairpersons of such committees.

The act lays down that four-fifths of the members of a district planning committee
should be elected by the elected members of the district Panchayat and
municipalities in the district from amongst themselves. The representation of these
members in the committee should be in proportion to the ratio between the rural and
urban populations in the district.
The chairperson of such committee shall forward the development plan to the state
government.

2. Solution: d)
Municipal corporations are created for the administration of big cities like Delhi,
Mumbai, Kolkata, Hyderabad, Bangalore and others. They are established in the
states by the acts of the concerned state legislatures, and in the union territories by
the acts of the Parliament of India. There may be one common act for all the
municipal corporations in a state or a separate act for each municipal corporation.
A municipal corporation has three authorities, namely, the council, the standing
committees and the commissioner.
The Council is the deliberative and legislative wing of the corporation. It consists of
the Councillors directly elected by the people, as well as a few nominated persons
having knowledge or experience of municipal administration. In brief, the
composition of the Council including the reservation of seats for SCs, STs and women
is governed by the 74th Constitutional Amendment Act.
The Council is headed by a Mayor. He is assisted by a Deputy Mayor. He is elected in
a majority of the states for a one-year renewable term. He is basically an ornamental
figure and a formal head of the corporation. His main function is to preside over the
meetings of the Council.

http://www.insightsonindia.com

INSIGHTS

Test - 18

The standing committees are created to facilitate the working of the council, which is
too large in size. They deal with public works, education, health, taxation, finance
and so on. They take decisions in their fields.
The municipal commissioner is responsible for the implementation of the decisions
taken by the council and its standing committees. Thus, he is the chief executive
authority of the corporation. He is appointed by the state government and is
generally a member of the IAS.

3. Solution: b)
Articles 239 to 241 in Part VIII of the Constitution deal with the union territories.
Even though all the union territories belong to one category, there is no uniformity in
their administrative system.
Every union territory is administered by the President acting through an
administrator appointed by him. An administrator of a union territory is an agent of
the President and not head of state like a governor. The President can specify the
designation of an administrator; it may be Lieutenant Governor or Chief
Commissioner or Administrator. At present, it is Lieutenant Governor in the case of
Delhi, Puducherry and Andaman and Nicobar Islands and Administrator in the case
of Chandigarh, Dadra and Nagar Haveli, Daman and Diu and Lakshadweep. The
President can also appoint the governor of a state as the administrator of an
adjoining union territory. In that capacity, the governor is to act independently of his
council of ministers.

4. Solution: c)
The Parliament can make laws on any subject of the three lists (including the State
List) for the union territories. This power of Parliament also extends to Puducherry
and Delhi, which have their own local legislatures. This means that, the legislative
power of Parliament for the union territories on subjects of the State List remain
unaffected even after establishing a local legislature for them. But, the legislative
assembly of Puducherry can also make laws on any subject of the State List and the
Concurrent List. Similarly, the legislative assembly of Delhi can make laws on any
subject of the State List (except public order, police and land) and the Concurrent
List.

5. Answer: d.
Explanation:

http://www.insightsonindia.com

INSIGHTS

Test - 18

Insights Mock Test Series 2015: Test 18 Solutions


The National Tiger Conservation Authority is a statutory body under the Ministry
of Environment, Forests and Climate Change constituted under enabling provisions
of the Wildlife (Protection) Act, 1972, as amended in 2006, for strengthening tiger
conservation, as per powers and functions assigned to it under the said Act. The
National Tiger Conservation Authority is set up under the Chairmanship of the
Minister for Environment and Forests.
The National Tiger Conservation Authority has been fulfilling its mandate within
the ambit of the Wildlife (Protection) Act, 1972 for strengthening tiger conservation
in the country by retaining an oversight through advisories/normative guidelines,
based on appraisal of tiger status, ongoing conservation initiatives and
recommendations of specially constituted Committees. Project Tiger is a Centrally
Sponsored Scheme of the Environment, Forests and Climate Change, providing
funding support to tiger range States, for in-situ conservation of tigers in
designated tiger reserves, and has put the endangered tiger on an assured path of
recovery by saving it from extinction, as revealed by the recent findings of the All
India tiger estimation using the refined methodology.
The authority approves the tiger conservation plan prepared by the State
Government. It also evaluates and assesses various aspects of sustainable ecology
and disallows any ecologically unsustainable land use such as, mining, industry
and other projects within the tiger reserves.

6. Solution: a)
The Congress met at Karachi on 29 March 1931 to endorse the Gandhi-Irwin or Delhi
Pact. Bhagat Singh, Sukhdev and Rajguru had been executed six days earlier. Even
though Gandhiji had made every attempt to save their lives, there was anger among
the people, especially the youth, as to why he had not refused to sign the Pact on this
question. All along Gandhijis route to Karachi he was greeted with black flag
demonstrations. The Congress passed a resolution drafted by Gandhiji by which it,
while dissociating itself from and disapproving of political violence in any shape or
form, admired the bravery and sacrifice of the three martyrs. The Congress
endorsed the Delhi Pact and reiterated the goal of Poorna Swaraj.
The Karachi session became memorable for its resolution on Fundamental Rights
and the National Economic Programme. Even though the Congress had from its
inception fought for the economic interests, civil liberties and political rights of the
people, this was the first time that the Congress defined what Swaraj would mean for
the masses. It also declared that, in order to end the exploitation of the masses,
political freedom must include real economic freedom of the starving millions.

http://www.insightsonindia.com

INSIGHTS

Test - 18

7. Solution: a)
http://www.thehindu.com/business/budget/comprehensive-bankruptcy-codeproposed/article6946327.ece

8. Solution: c)
http://www.thehindu.com/opinion/editorial/welcome-push-oninfrastructure/article6972017.ece

9. Solution: d)
http://www.business-standard.com/article/news-ani/budget-2015-16-selfemployment-and-talent-utilization-setu-to-be-established-115022800913_1.html

10.

Solution: a)

http://www.insightsonindia.com

INSIGHTS

Test - 18

Insights Mock Test Series 2015: Test 18 Solutions

11. Solution: b)

12. Solution: b)
http://www.dailypioneer.com/sunday-edition/sunday-pioneer/moneywise/govt-toset-up-5-umpps-via-plug-and-play-mode.html

13. Solution: a)
http://www.thehindu.com/business/budget/public-debt-management-agency-willbring-indias-external-and-domestic-debt-under-one-roof-finance-minister-arunjaitley/article6946273.ece

http://www.insightsonindia.com

INSIGHTS

Test - 18

http://businesstoday.intoday.in/story/union-budget-2015-16-public-debtmanagement-agency-proposed/1/216338.html

14. Solution: d)
http://www.prsindia.org/parliamenttrack/report-summaries/financial-sectorlegislative-reforms-commission-2792/
http://articles.economictimes.indiatimes.com/2014-07-10/news/51301270_1_fslrcfinancial-sector-indian-financial-code

15. Solution: d)
The Ministry comprises of all the above mentioned departments.

16. Solution: d)

17. Solution: d)
The Central Statistical Office which is one of the two wings of the National
Statistical Organisation (NSO) is responsible for coordination of statistical
activities in the country and for evolving and maintaining statistical standards. Its
activities include compilation of National Accounts; conduct of Annual Survey of
Industries and Economic Censuses, compilation of Index of Industrial Production,
as well as Consumer Price Indices. It also deals with various social statistics,
training, international cooperation, Industrial Classification etc.

18.Solution: a)
The Constitution of India directly to ensure free and fair elections in the country.
Article 324 of the Constitution provides that the power of superintendence, direction
and control of elections to parliament, state legislatures, the office of president of
India and the office of vice-president of India shall be vested in the election

http://www.insightsonindia.com

INSIGHTS

Test - 18

Insights Mock Test Series 2015: Test 18 Solutions


commission. Thus, the Election Commission is an all-India body in the sense that it
is common to both the Central government and the state governments.
It must be noted here that the election commission is not concerned with the
elections to panchayats and muncipalities in the states. For this, the Constitution of
India provides for a separate State Election Commission.

19. Solution: a)
It is important to keep track of important fiscal indicators in every years budget.

20.

Solution: a)

CO2 being the predominant GHG, an analysis (of the World Bank) of its emissions
across countries in per capita terms in 2009, compared to 2005 presents an
interesting picture. Although the G20 is referred to as a group, there are stark
disparities on the ground between member countries in terms of incomes, stages of
development as well as respective per capita CO2 emissions. In 2005
the USA had the highest CO2 emissions in metric tons per capita at 19.7, followed
by Australia (18.0).
the lowest per capita emitters in 2005 were Brazil (1.9), Indonesia (1.5), and India
(1.2) who continued to be the bottom three in 2009 as well.
In 2009, Australia ranked first within the G20, followed by the USA.

http://www.insightsonindia.com

INSIGHTS

Test - 18

21. Solution: a)
The PAT (perform - achieve - trade) is a scheme for trading energy-efficiency
certificates in large energy-intensive industries under the National Mission for
Enhanced Energy Efficiency
Identified industries are required to improve their specific energy consumption
(SEC) within the specified period of three years or face penalty provisions.
At the same time this mechanism facilitates efficient industries to trade their
additional certified energy savings (that go beyond the assigned target) with other
designated consumers who could use these certificates to comply with their SECreduction targets.
In the Twelfth Five Year Plan, the PAT scheme is likely to achieve about 15 million
tonnes oil equivalent of annual savings in coal, oil, gas, and electricity (including
6.686 million ton of oil-equivalent energy savings of first phase).

22.

Solution: c)

The Lt. Governor is empowered to promulgate ordinances during recess of the


assembly. An ordinance has the same force as an act of the assembly.
Every such ordinance must be approved by the assembly within six weeks from its
reassembly. He can also withdraw an ordinance at any time. But, he cannot
promulgate an ordinance when the assembly is dissolved or suspended. Further, no
such ordinance can be promulgated or withdrawn without the prior permission of
the President

23.

Solution: d)

24.

Solution: d)

There are 18 Biosphere Reserves in India. They protect larger areas of natural
habitat (than a National Park or Animal Sanctuary), and often include one or more
National Parks and/or preserves, along buffer zones that are open to some
economic uses. Protection is granted not only to the flora and fauna of the protected
region, but also to the human communities who inhabit these regions, and their
ways of life. Animals are protected and saved here.

http://www.insightsonindia.com

INSIGHTS

Test - 18

Insights Mock Test Series 2015: Test 18 Solutions

25.

Nine of the eighteen biosphere reserves are a part of the World Network of
Biosphere Reserves, based on the UNESCO Man and the Biosphere (MAB)
Programme list.
The Nilgiri Biosphere Reserve was the first biosphere reserve in India
established in the year 1986. It is located in the Western Ghats and includes 2
of the 10 biogeographical provinces of India.

Solution: c)

The National Biodiversity Authority (NBA) is a statutory autonomous body under


the Ministry of Environment and Forests, Government of India established in 2003
to implement the provisions under the National Biological Diversity Act, 2002,
after India signed Convention on Biological Diversity (CBD) in 1992.
It acts as a facilitating, regulating and advisory body to the Government of India
"on issues of conservation, sustainable use of biological resources and fair and
equitable sharing of benefits arising out of the use of biological resources."
Additionally, it advises State Governments in identifying the areas of biodiversity
importance (biodiversity hotspots) as heritage sites.

26.

Solution: d)

The Convention on Biological Diversity (CBD) is a legally binding multilateral


environmental agreement that has 193 contracting Parties (Countries) as its
members with three objectives conservation of biological diversity, sustainable
use of the diversity and ensuring fair and equitable sharing of benefits of such use.
It has entered into force on 29th December 1993.
The CBD provides sovereign rights over their biological resources, and requests
countries would facilitate access to the genetic resources by other parties subject to
national legislation and on mutually agreed terms. The CBD also provides for
equitable sharing of benefits arising from the utilization of traditional knowledge,
and practices, with holders of such knowledge. This has made it necessary for a
legislation to be put in place, which lays down the framework for providing access,
for determining the terms of such access and for ensuring the equitable sharing of
benefits.

27.

Solution: d)

http://www.insightsonindia.com

INSIGHTS

Test - 18

Union Government has merged Green India Mission (GIM) with the Mahatma
Gandhi National Rural Employment Guarantee Act (MGNREGA) Scheme.
By merging GIM with MGNREGA Scheme government seeks to increase 10 million
hectares of forest cover.

National Mission for a Green India aims for afforestation at 10 million


hectares of land over the next decade in order to increase and improve the
countrys forest cover. It also aims at improving ecosystems services, forest
based livelihood income of about three million households and to enhance
annual CO2 sequestration.

At present under MGNREGA Scheme green works such as water harvesting,


afforestation and farm forestry are undertaken which are implemented by
Union Ministry of Rural Development.

Government will also use modern technology like remote sensing to monitor
the progress of this initiative regularly.

Union Government also has set out convergence guidelines in this regard after
consulting both Ministry of Environment & Forest (MoEF) and Rural Development
Ministries.

28.

Solution: c)

This species of alga is originally inhabitant to the seas around the British Isles and
the East China Sea. It belongs to a type known as endophytic which means that they
are microscopic in size and generally found living inside macroscopic seaweeds.

Endophytic alga has ecological advantage to the host such as disease


resistance.

It contains anticancer compounds such as Taxol and can be used for treatment
of many cancers including ovarian cancer, breast cancer, lung cancer and
pancreatic cancer.

29.

Solution: c)

The British policy of Divide and Rule found another expression in the
announcement of the Communal Award in August 1932. The Award allotted to each
minority a number of seats in the legislatures to be elected on the basis of a separate
electorate that is Muslims would be elected only by Muslims and Sikhs only by Sikhs,
and so on. Muslims, Sikhs and Christians had already been treated as minorities. The
Award declared the Depressed Classes (Scheduled Castes of today) also to be a

http://www.insightsonindia.com

INSIGHTS

Test - 18

Insights Mock Test Series 2015: Test 18 Solutions


minority community entitled to separate electorate and thus separated them from
the rest of the Hindus.

30.

Solution: d)

Scientists believe that this finding could potentially support life. Hubble Space
Telescopes finding was published in the Journal of Geophysical Research: Space
Physics.
The finding was based on the results obtained after analyzing Ganymedes magnetic
field and aurorae created and controlled by it and its interaction with Jupiters
magnetic field.
Scientists estimate that

This ocean is 100 kms thick.

10 times deeper than Earths oceans.

It is buried under a 150-kilometre crust of mostly ice.

In 1970s, for first time scientists had suggested that ocean is present inside
Ganymede based on its models. NASAs Galileo mission also had provided first
evidence of ocean in Ganymede after measuring its magnetic field in 2002.
About Ganymede

Ganymede is the largest moon in our solar system. It is also the only moon
with its own magnetic field.

The magnetic field causes aurorae, which are ribbons of glowing, hot
electrified gas, in regions circling the north and south poles of the moon.

31. Solution: a)
Pakistan successfully tested its first indigenously built all-weather armed drone
named Burraq and a laser guided missile named Barq.
By successfully testing of Burraq, Pakistan became the 9th country in the World to
develop an unmanned combat aerial vehicle (UCAV) domestically.
About Burraq

Dveloped by a venture of the Pakistan Air Force (PAF) and the National
Engineering and Scientific Commission (NESCOM).

http://www.insightsonindia.com

INSIGHTS

Test - 18

It is named after the flying horse of Islamic tradition.

Burraq has ability to fly in all types of weather conditions.

It has capability to strike its targets with pinpoint accuracy.

With Barq laser-guided missile mounted on it, Burraq can hit moving targets
as well as stationary targets with pinpoint accuracy.

UCAV Burraq was first flight tested in May 2009.

At present, Pakistan has several types of unarmed surveillance drones in operation


but Burraq and Shahpar are its domestically developed UCAVs.

32.

Solution: a)

The defence pact seeks to boost cooperation and joint development between both
countries in order to strengthen security ties.
The agreement will also pave the way for Japan to develop drones and other
unmanned equipment together as it seeks to play a greater international military
role. It will also promote joint projects in several fields including unmanned
technologies for underwater search missions or surveillance.
Japan and France also have agreed to step up broader military cooperation and antiterrorism efforts. In the past, Japan had signed similar arms equipment and
technology transfer agreements with the Britain, United States and Australia.

33.

Solution: d.

Right to Education (RTE) Act


The Constitution (Eighty-sixth Amendment) Act, 2002 inserted Article 21-A in the
Constitution of India to provide free and compulsory education of all children in the
age group of six to fourteen years as a Fundamental Right in such a manner as the
State may, by law, determine. The Right of Children to Free and Compulsory
Education (RTE) Act, 2009, which represents the consequential legislation
envisaged under Article 21-A, means that every child has a right to full time
elementary education of satisfactory and equitable quality in a formal school which
satisfies certain essential norms and standards. Article 21-A and the RTE Act came
into effect on 1 April 2010.

With this, India has moved forward to a rights based framework that casts a
legal obligation on the Central and State Governments to implement this

http://www.insightsonindia.com

INSIGHTS

Test - 18

Insights Mock Test Series 2015: Test 18 Solutions

fundamental child right as enshrined in the Article 21A of the Constitution, in


accordance with the provisions of the RTE Act.
It is seen as the most historic development in universalisation of elementary
education in the country. It implies that every child in the age group of 6 to
14 years has Right to elementary education. They are entitled for free and
compulsory education.

The RTE Act provides for the:

Right of children to free and compulsory education till completion of


elementary education in a neighbourhood school.
It clarifies that compulsory education means obligation of the appropriate
government to provide free elementary education and ensure compulsory
admission, attendance and completion of elementary education to every
child in the six to fourteen age group. Free means that no child shall be
liable to pay any kind of fee or charges or expenses which may prevent him
or her from pursuing and completing elementary education.
It makes provisions for a non-admitted child to be admitted to an age
appropriate class.
It specifies the duties and responsibilities of appropriate Governments, local
authority and parents in providing free and compulsory education, and
sharing of financial and other responsibilities between the Central and State
Governments.
It lays down the norms and standards relating inter alia to Pupil Teacher
Ratios (PTRs), buildings and infrastructure, school-working days, teacherworking hours.
It provides for rational deployment of teachers by ensuring that the specified
pupil teacher ratio is maintained for each school, rather than just as an
average for the State or District or Block, thus ensuring that there is no
urban-rural imbalance in teacher postings. It also provides for prohibition of
deployment of teachers for non-educational work, other than decennial
census, elections to local authority, state legislatures and parliament, and
disaster relief.
It provides for appointment of appropriately trained teachers, i.e. teachers
with the requisite entry and academic qualifications.

The Sarva Shiksha Abhiyan (SSA) is the main vehicle for implementation of the RTE
Act. It is one of the largest programmes of its kind in the world. It is primarily
funded from central budget and it covers the whole country.

Under SSA, special attention has been given to urban deprived children,
children affected by periodic migration and children living in remote and
scattered habitations. Attempts have also been made to reach out to children
suffering from autism. It involves their identification, preparation of

http://www.insightsonindia.com

INSIGHTS

Test - 18

34.

individualized Education Plan, teacher training on Autism and therapeutic


support.
The programme has been implemented in order to narrow down gender and
social gaps in elementary education. Special efforts have been made to reach
out to girls and children belonging to SC/ST and Muslim minority
communities.

Solution: a)

Pursuant to the objectives of the NPE, 1986, the Mahila Samakhya Scheme was
started in 1989 to translate the goals enshrined in the NPE into a concrete
programme for the education and empowerment of women in rural areas
particularly those from socially and economically marginalized groups. The MS
scheme recognizes the centrality of education in empowering women to achieve
equality.
Salient features of the Programme:

The Mahila Sanghas or women's collectives at the village level provide the
women a space to meet, reflect, ask questions and articulate their thoughts
and needs and make informed choices.
The Mahila Sanghas though various programmes and awareness campaigns
have brought about a change in the outlook of rural women and the effects
can how be seen in various facets of life at home within the family, the
community and at the block and Panchayat levels.
The programme has also focused on awareness of the need to educate the
children, especially girls, to give the equal status and opportunities which
has resulted in a direct impact on enrolment and retention of girls in schools.

The Mahila Samakhya Scheme is currently being implemented in Ten States viz.,
Andhra Pradesh, Assam, Bihar, Chattisgarh, Jharkhand, Karnataka, Kerala,
Gujarat, Uttar Pradesh and Uttarakhand.

35.

Solution: a)

The problems in the Indian banking system lie elsewhere and fall into two categories:
policy and structure.
The policy challenge relates to financial repression.
The Indian banking system is afflicted by what might be called double financial
repression which reduces returns to savers and banks, and misallocates capital to

http://www.insightsonindia.com

INSIGHTS

Test - 18

Insights Mock Test Series 2015: Test 18 Solutions


investors. Financial repression on the asset side of the balance sheet is create by the
statutory liquidity ratio (SLR) requirement that forces banks to hold government
securities, and priority sector lending (PSL) that forces resource deployment in lessthan-fully efficient ways. Financial repression on the liability side has arisen from
high inflation since 2007, leading to negative real interest rates, and a sharp
reduction in household savings. As India exits from liabilityside repression with
declining inflation, the time may be appropriate for addressing its asset-side
counterparts.
Also refer to :
http://www.thehindu.com/business/budget/4d-solution-for-bankingindustry/article6941953.ece

36.

Solution: d)

The Constitution visualises the UPSC to be the watch-dog of merit system in India.
It is concerned with the recruitment to the all-India services and Central services
group A and group B and advises the government, when consulted, on promotion
and disciplinary matters. It is not concerned with the classification of services, pay
and service conditions, cadre management, training, and so on. These matters are
handled by the Department of Personnel and Trainingone of the three departments
of the Ministry of Personnel, Public Grievances and Pensions. Therefore, UPSC is
only a central recruiting agency while the Department of Personnel and Training is
the central personnel agency in India.
The role of UPSC is not only limited, but also recommendations made by it are only
of advisory nature and hence, not binding on the government. It is upto the Union
government to accept or reject that advise. The only safeguard is the answerability of
the government to the Parliament for departing from the recommendation of the
Commission. Further, the government can also make rules which regulate the scope
of the advisory functions of UPSC .

37.

Solution: a)

It is a quasi-judicial body as its work involves fixing the fair share of states in the
overall grant from Centre. Since it uses both legal principles and administrative
dictums in making decisions, it is called a quasi-judicial body. It is constituted by the
president of India every fifth year or at such earlier time as he considers necessary.
The Finance Commission consists of a chairman and four other members to be
appointed by the president. And since it is a Quasi-judicial body, the Chairman is a

http://www.insightsonindia.com

INSIGHTS

Test - 18

person having experience in public affairs. Other members are judges,


administrators or experts. So both administrative side and judicial sides are balanced
in the Commission.

38.

Solution: a)

The Constitution authorises the Parliament to determine the qualifications of


members of the commission and the manner in which they should be selected.
Accordingly, the Parliament has specified the qualifications of the chairman and
members of the commission. The chairman should be a person having experience in
public affairs and the four other members should be selected from amongst the
following:

39.

A judge of high court or one qualified to be appointed as one.


A person who has specialised knowledge of finance and accounts of the
government.
A person who has wide experience in financial matters and in administration.
A person who has special knowledge of economics.

Solution: c)

It must be clarified here that the recommendations made by the Finance


Commission are only of advisory nature and hence, not binding on the government.
It is up to the Union government to implement its recommendations on granting
money to the states.
To put it in other words, It is nowhere laid down in the Constitution that the
recommendations of the commission shall be binding upon the Government of India
or that it would give rise to a legal right in favour of the beneficiary states to receive
the money recommended to be offered to them by the Commission.

40.

Solution: a)

The National Commission for Scheduled Castes (SCs) is a constitutional body in the
sense that it is directly established by Article 338 of the Constitution.
On the other hand, the other national commissions like the National Commission for
Women (1992), the National Commission for Minorities (1993), the National
Commission for Backward Classes (1993), the National Human Rights Commission
(1993) and the National Commission for Protection of Child Rights (2007) are
statutory bodies in the sense that they are established by acts of the Parliament.

http://www.insightsonindia.com

INSIGHTS

Test - 18

Insights Mock Test Series 2015: Test 18 Solutions

41. Solution: b)
The separate National Commission for SCs came into existence in 2004. It consists of
a chairperson, a vice-chairperson and three other members. They are appointed by
the President by warrant under his hand and seal. Their conditions of service and
tenure of office are also determined by the President.
The commission presents an annual report to the president. It can also submit a
report as and when it thinks necessary.
The President places all such reports before the Parliament, along with a
memorandum explaining the action taken on the recommendations made by the
Commission. The memorandum should also contain the reasons for the nonacceptance of any of such recommendations.

42.

Solution: d)

Perspiration and inspiration, investment and efficiency, respectively, determine


long-run growth.
But the Indian private investment climate is clouded by the experience of the last
decade. A combination of factorsweak corporate balance sheets, an impaired
banking system, difficulty of exit, the deficiencies of the public private partnership
(PPP) model in infrastructurecould hold back private investment going forward.
Private investment must remain the main engine of long-run growth. But, in the
short to medium term, as the near-intractable problems get slowly resolved, public
investment, especially by the railways, will have to play a catalytic role.

43.

Solution: b)

Rajiv Gandhi Grameen Vidyutikaran Yojana


In order to attain National Common Minimum Programme (NCMP) goal of
providing access to electricity to all rural households and electrification of all
villages, Government of India, in April, 2005 conceived Rajiv Gandhi Grameen
Vidyutikaran Yojana (RGGVY) to electrify all un-electrified villages / habitations
and to provide access to electricity to all rural households in un-electrified and
electrified villages in the entire country.

http://www.insightsonindia.com

INSIGHTS

Test - 18

The scheme covers electrification of all the villages in the country except the
villages under the programme of Ministry of Non-conventional Sources
(MNES) for providing electricity from non-conventional energy sources
under their remote village electrification programme. With the launch of this
scheme the existing Accelerated Electrification of One lakh Villages and One
Crore Households and the Minimum Needs Programme for rural
electrification got merged with RGGVY.
The scheme has been launched to fulfil the commitment of the National
Common Minimum Programme (NCMP) of completing the household
electrification in next 5 years and modernizing the rural electricity
infrastructure.
The scheme is being implemented through the Rural Electrification
Corporation (REC) which has been designated as Nodal Agency by Ministry
of Power.
The scheme provides for free of cost connection to all rural households living
below poverty line. The programme aims at a qualitative transformation of
the rural electricity infrastructure. It envisages that there will not be any
discrimination between urban and rural areas in respect of hours of supply.
24 hours supply of good quality power would also enable dispersal of small
industries, khadi and village industries in the rural areas. It will also
facilitate delivery of modern health care, education and application of
information technologies. This is aimed at accelerated rural development,
employment generation and poverty alleviation.

SALIENT FEATURES
The scheme has the following important features:

Ninety per cent capital subsidy is provided towards overall cost of the
projects under the scheme. 10% of the project cost is contributed by States
through own resources / loan from financial institutions.
Prior commitment of the States has been obtained before sanction of projects
under the scheme for:
Guarantee by State Government for a minimum daily supply of 6- 8 hours.
Free of cost service connection to all families Below Poverty Lines.

Under the programme 90% grant is provided by Govt. of India and 10% as loan by
REC to the State Governments.

44.

Solution: d)

https://www.climateinvestmentfunds.org/cif/node/11900

http://www.insightsonindia.com

INSIGHTS

Test - 18

Insights Mock Test Series 2015: Test 18 Solutions

45.

Solution: c)

As per the IMF, every fast-growing Asian economy in recent years has accelerated as
it underwent a demographic transition. In India itself, the high growth states (Tamil
Nadu, Karnataka, and Gujarat) in the period 1991-2001 had a dependency ratio
which was 8.7 percentage points lower than that of the low growth states (Bihar,
Madhya Pradesh, and Uttar Pradesh) and an average annual growth rate that was 4.3
percentage points higher. Looking ahead, the low growth states will benefit more
from the demographic dividend, as higher incomes and lower fertility alter
demographics. Indeed, over the period 2001-11, the hitherto laggard states have
grown at an average of around 5 per cent annually.
The difference between their growth and that of the leaders in the period 2001-11 is
just 1.5 percentage. So demographic transition seems to be correlated with growth,
with some reasons to believe that causality flows both ways i.e., lower dependency
ratios increase growth and higher growth reduces fertility and consequently
dependency ratios.

46.

Solution: a)

Options B and C are wrong cycles. Only option A completes the cycle.
The low growth states will benefit more from the demographic dividend, as higher
incomes and lower fertility alter demographics. Indeed, over the period 2001-11, the
hitherto laggard states have grown at an average of around 5 per cent annually.
The difference between their growth and that of the leaders in the period 2001-11 is
just 1.5 percentage. So demographic transition seems to be correlated with growth,
with some reasons to believe that causality flows both ways i.e., lower dependency
ratios increase growth and higher growth reduces fertility and consequently
dependency ratios.

47.

Solution: d)

Gandhijis entire campaign was based on the grounds of humanism and reason. But
he also argued that untouchability, as practised at present, had no sanction in the
Hindu Shastras. But even if this was not so, the Harijan worker should not feel
daunted. Truth could -not be confined within the covers of a book. The Shastras
should be ignored if they went against human dignity.

http://www.insightsonindia.com

INSIGHTS

Test - 18

A major running theme in Gandhijis writings and speeches was the need for caste
Hindus to do penance and make reparations . . . for the untold hardships to which
we have subjected them (the Harijans) for centuries. For this reason, he was not
hostile to Dr. Ambedkar and other Harijans who criticized and distrusted him. They
have every right to distrust me, he wrote. Do I not belong to the Hindu section
miscalled superior class or caste Hindus, who have ground down to powder the so
called untouchables? At the same time, he repeatedly warned caste Hindus that if
this atonement was not made, Hinduism would perish: Hinduism dies if
untouchability lives, and untouchability has to die if Hinduism is to live. (This strong
theme of penance largely explains why caste Hindus born and brought up in pre1947 India so readily accepted large scale reservations in jobs, enrolment in
professional colleges and so on for the Scheduled Castes and Scheduled Tribes after
independence).

48.

Solution: a)

Growth in per capita income is driven by growth in labour productivity (what the
average worker produces), growth in working age population (fewer the people who
are in the dependent age group in the population, greater the output), growth in the
fraction of those who can work and that actually look for work (labour force
participation rate), and growth in those looking for work who actually find it
(employment rate).
Because accurate employment data are hard to find for developing countries, studies
typically ignore the employment rate in decomposing the sources of growth.

49.

Solution: a)

In economics, total-factor productivity (TFP), also called multi-factor productivity,


is a variable which accounts for effects in total output not caused by traditionally
measured inputs of labor and capital. If all inputs are accounted for, then total factor
productivity (TFP) can be taken as a measure of an economys long-term
technological change or technological dynamism.
The increase in the fraction of people working is probably not the main consequence
of the demographic dividend. Instead, the effects of the demographic dividend are
channelled through the increase in labour productivity, which comes from more
physical capital employed per worker (in turn resulting from greater saving and
investment), more human capital per worker (which comes from more education as
smaller families lead to greater spending on education per child), and greater TFP
(total factor productivity).

http://www.insightsonindia.com

INSIGHTS

Test - 18

Insights Mock Test Series 2015: Test 18 Solutions

50.

Solution: d)

Following initiatives have been taken wrt agriculture in the Budget

Pradhanmantri Gram Sinchai Yojana to provide Per Drop More Crop.


`5,300 crore to support micro-irrigation, watershed development and the
Pradhan Mantri Krishi Sinchai Yojana.
States urged to chip in. `25,000 crore in 2015-16 to the corpus of Rural
Infrastructure Development Fund (RIDF) set up in NABARD; `15,000 crore
for Long Term Rural Credit Fund; `45,000 crore for Short Term Co-operative
Rural Credit Refinance Fund; and `15,000 crore for Short Term RRB
Refinance Fund.
Target of `8.5 lakh crore of agricultural credit during the year 2015-16.
Focus on improving the quality and effectiveness of activities under
MGNREGA.
Need to create a National Agriculture Market for the benefit farmers, which
will also have the incidental benefit of moderating price rises. Government to
work with the States, in NITI, for the creation of a Unified National
Agriculture Market.

51. Solution: d)
A major debate on strategy occurred among the nationalists in the period following
the withdrawal of the Civil Disobedience Movement. In the first stage of the debate,
during 1934-35, the issue was what course the national movement should take in the
immediate future, that is, during its phase of non-mass struggle. How was the
political paralysis that it had sunk into to be overcome? There were two traditional
responses. Gandhiji emphasized constructive work in the villages, especially the
revival of village crafts.
Constructive work, said Gandhiji, would lead to the consolidation of peoples power,
and open the way to the mobilization of millions in the next phase of mass struggle.
Another section of Congressmen advocated the revival of the constitutional method
of struggle and participation in the elections to the Central Legislative Assembly to be
held in 1934. Led this time by Dr. M.A. Ansari, Asaf Ali, Satyamurthy, Bhulabhai
Desai and B.C. Roy, the new Swarajists argued that in a period of political apathy and
depression, when the Congress was no longer in a position to sustain a mass
movement, it was necessary to utilize elections and work in the legislative councils to
keep up the political interest and morale of the people. This did not amount, they
said, to having faith in the capacity of constitutional politics to achieve freedom.

http://www.insightsonindia.com

INSIGHTS

Test - 18

52.

53.

Solution: d)
The salient features of the Scheme are: assistance under AREP will be
available for electrification by conventional and non-conventional sources of
energy and the interest subsidy under the Scheme is four per cent.
Interest subsidy under AREP Scheme will be provided to State
Governments/Power Utilities on loan availed by it from any financial
institutions like Rural Electrification Corporation, Power Finance
Corporation, Rural Infrastructure Development Fund, National Agricultural
Bank and Rural Development for carrying out rural electrification
programmes which help to achieve national objectives of the Government for
complete electrification of villages by 2007 and coverage of all households by
2012.
Funds would be provided under the scheme on the basis of Net Present Value
of the interest subsidies applicable on disbursement under Accelerated Rural
Electrification Programme.

Solution: c)

Provisions related to Captive Power Plants under the Electricity Act, 2003:

54.

No sanction is required under the provisions of the Electricity Act, 2003 for
setting up captive power plants.
Under the Electricity Act, 2003, captive power plants, including group
captive, have been freely permitted.
The Act provides that any person may construct, maintain or operate a
captive generating plant and dedicated transmission lines.
Under the provisions of the Act, every person, who has constructed a captive
generating plant and maintains and operates such plant, shall have the right
to open access for the purposes of carrying electricity from his captive
generating plant to the destination of his use subject to the availability of
transmission capacity.

Solution: b)
The Central Electricity Authority of India (CEA) is a statutory organisation
constituted under section 3(1) of Electricity Supply Act 1948, which has been
superseded by section 70(1) of the Electricity Act 2003.
The CEA advises the government on matters relating to the National
Electricity Policy and formulates short-term and perspective plans for the
development of electricity systems.

http://www.insightsonindia.com

INSIGHTS

Test - 18

Insights Mock Test Series 2015: Test 18 Solutions

55.

Under the Electricity Act 2003, CEA prescribes the standards on matters
such as construction of electrical plants, electric lines and connectivity to the
grid, installation and operation of meters and safety and grid standards.
The CEA is also responsible for concurrence of hydro power development
schemes of central, state and private sectors taking into consideration the
factors which will result in efficient development of the river and its
tributaries for power generation, consistent with the requirement of
drinking water, irrigation, navigation and flood control.
Preparation of technical standards for construction of electrical plants,
electric lines and connectivity to the grid is the responsibility of CEA as per
section 73 (b) of the Electricity Act, 2003. However as per section 7 of this
Act, a generating company may establish, operate and maintain a
generating station if it complies with the technical standards only relating to
connectivity to the grid as given in clause (b) of section 73. This implies that
generating stations need not follow compulsory the CEA technical standards
specified for construction of electrical plants and electric lines.
The CEA plays a lead role in promoting the integrated operations of the
regional power grids and the evolution of a national grid. The eastern,
western and north-eastern grids have been integrated and are operating in
a synchronous mode. The eastern grid is connected to the northern as well as
southern grid through HVDC back to back links. The western grid is also
connected to the northern and southern grid through similar arrangements.
The CEA facilitates exchange of power within the country from surplus to
deficit regions and with neighbouring countries for mutual benefits.
The CEA advises central government, state governments and regulatory
commissions on all technical matters relating to generation, transmission
and distribution of electricity. It also advises state governments, licensees or
generating companies on matters which enable them to operate and
maintain the electricity system under their ownership or control in an
improved manner.

Solution: a)

SJVN Limited, a Mini Ratna & Schedule 'A' CPSU under the Ministry of Power,
Govt. of India, is a joint venture between the Govt. of India & Govt. of Himachal
Pradesh. Incorporated in the year 1988, the Company is fast emerging as a major
power player in the country. The present authorized capital of SJVN is INR 7000
crores.
SJVN is successfully operating the countrys largest 1500 MW Nathpa Jhakri
Hydropower Station and is setting new benchmarks in generation and

http://www.insightsonindia.com

INSIGHTS

Test - 18

maintenance year after year, after having tackled the silt erosion problems in
under-water turbine parts.

56.

Solution: c)

The Jawaharlal Nehru National Solar Mission was launched on the 11th January,
2010. The Mission has set the ambitious target of deploying 20,000 MW of grid
connected solar power by 2022 is aimed at reducing the cost of solar power
generation in the country through (i) long term policy; (ii) large scale deployment
goals; (iii) aggressive R&D; and (iv) domestic production of critical raw materials,
components and products, as a result to achieve grid tariff parity by 2022. Mission
will create an enabling policy framework to achieve this objective and make India a
global leader in solar energy. The Mission is under the aegis of Ministry of New and
Renewable Energy.

57.

58.

Solution: d)

Solution: a)

A recent study Preliminary Modeling Studies on Carbon Taxes and GDP Loss was
conducted by the Ministry of Environment and Forest, GoI. The results of the study
are given as below:
Undiscounted cumulative GDP loss: Carbon tax is revenue positive when it involves
no adjustment to other tax rates in the economy. It is revenue neutral when other tax
rates are adjusted so that the revenue inflow from carbon tax is exactly balanced by
an equal reduction in yields from reduced taxes.
The assessment and quantification of the costs of adaptation and mitigation is a
difficult task. However, it is clear that these costs are significant and will likely be
higher in the future as initiatives are taken in line with the goals outlined in the
NAPCC.

59.

Solution: a)

Rice, wheat, pulses, sugar, kerosene, LPG, naphtha, water, electricity, fertiliser, iron
ore, railways these are just a subset of the products and services that the
government subsidises. The estimated direct fiscal costs of these (select) subsidies
are about ` 378,000 crore or about 4.2 percent of GDP. This is roughly how much it
would cost to raise the expenditure of every household to that of a household at the
35th percentile of the income distribution.
http://www.insightsonindia.com

INSIGHTS

Test - 18

Insights Mock Test Series 2015: Test 18 Solutions

60.

Solution: d)

Price subsidies, no doubt provide help, but they may not have a transformative effect
on the economic lives of the poor. For many subsidies, only a small fraction of the
benefits actually accrue to the poor. For example, electricity subsidies benefit mainly
the (relatively wealthy) 67.2 percent of households that are electrified. A large
fraction of subsidies allocated to water utilities are spent on subsidising private taps
when 60 percent of poor households get their water from public taps.
Moreover, the implementation of subsidies can be fiendishly complex. In the case of
fertilizers, they are firm-specific and import-consignment specific, they vary by type
of fertilizer, and some are on a fixed-quantity basis while others are variable.

61. Solution: d)
The changing fortunes of India have been nothing short of dramatically positive.
Inflation has declined by over 6 percentage points since late 2013, and the current
account deficit has shrivelled from a peak of 6.7 percent of GDP (in Q3, 2012-13) to
an estimated 1.0 percent in the coming fiscal year. Foreign portfolio flows (of US$
38.4 billion since April 2014) have stabilized the rupee, exerting downward pressure
on long-term interest rates, reflected in the yield on 10-year government securities,
and contributed to the surge in equity prices (31 percent since April in rupee terms,
and even more in US dollars, ranking it the highest amongst emerging markets).
In a nearly 12-quarter phase of deceleration, economic growth averaged 6.7 percent
but since 2013-14 has been growing at 7.2 percent on average.

62.

Solution: b)

India ranks amongst the most attractive investment destinations, well above other
countries. It ranks well above the mean for its investment grade category, and also
above the mean for the investment category above it (on the basis of the new growth
estimates). Amongst BRICS (and other comparable countries) only China scores
above India.

http://www.insightsonindia.com

INSIGHTS

Test - 18

(Sources: European Travel Commission and World Tourism Organisation for the
following)

Brazil: in 2012 they issued a total of 8.6 million tourists. It is estimated that
long-distance trips will grow on an average of 9.5% per year. In 2012 tourist
spending in this market was 17 billion, ranking the country 12th in the world
in terms of tourist spending.

Russia: in 2012 they issued 33.8 million tourists, of which 79% travelled to
Europe. They occupy the 5th position regarding tourism expenditure
worldwide and in 2012 tourist spending reached 42.2 billion USD.

India: in 2012 they issued 10.9 million tourists, of which 95.1% went on long
distance trips. It is expected that the choice of remote destinations by
international tourists will increase by an annual average of 8.6% until 2017.

China: is one of the main global tourist markets, reaching a total of 83 million
tourists in 2012. They are also currently top of the table for tourism spending
with a figure of 102 billion USD.

During the first half of 2013, global tourism increased by 5% in terms of international
arrivals. This growth was more significant in emerging economy destinations. From
January to July 2013, China (up 31%) and Russia (up 22%) led the growth of foreign
travel spending.

63.

Solution: d)

The 69th Constitutional Amendment Act of 19915 provided a special status to the
Union Territory of Delhi, and re-designated it the National Capital Territory of Delhi
and designated the administrator of Delhi as the lieutenant (lt.) governor. It created
a legislative assembly and a council of ministers of Delhi. Previously, Delhi had a
metropolitan council and an executive council.
The strength of the assembly is fixed at 70 members, directly elected by the people.
The elections are conducted by the election commission of India. The assembly can
make laws on all the matters of the State List and the Concurrent List except the
three matters of the State List, that is, public order, police and land. But, the laws of
Parliament prevail over those made by the Assembly.

64.

Solution: a)

The chief minister of Delhi is appointed by the President (not by the lt. governor).
The other ministers are appointed by the president on the advice of the chief
http://www.insightsonindia.com

INSIGHTS

Test - 18

Insights Mock Test Series 2015: Test 18 Solutions


minister. The ministers hold office during the pleasure of the president. The council
of ministers is collectively responsible to the assembly.
The council of ministers headed by the chief minister aid and advise the lt. governor
in the exercise of his functions except in so far as he is required to act in his
discretion. In the case of difference of opinion between the lt. governor and his
ministers, the lt. governor is to refer the matter to the president for decision and act
accordingly.

65.

Solution: d)

The Left failed to make a deep study of Indian reality. With the exception of
Jawaharlal Nehru, the Left saw the dominant Congress leadership as bourgeois its
policy of negotiations as working towards a compromise with imperialism any resort
to constitutional work as a step towards the abandonment of the struggle for
independence. It took recourse to a simplistic model of analysing Indian social
classes and their political behaviour. It saw all efforts to guide the national
movement in a disciplined manner as imposing restrictions on the movement. It
constantly counterposed armed struggle to nonviolence as a superior form and
method of struggle, rather than concentrating on the nature of mass involvement and
mobilization and ideology. It was Convinced that the masses were ever ready for
struggles in any form if only the leaders were willing to initiate them. It constantly
overestimated its support among the people. Above all, the Left failed to grasp the
Gandhian strategy of struggle.

66.

Solution: a)

As the Economic Survey correctly points out - Indeed, one of the paradoxes of recent
banking history is that the share of the private sector in overall banking aggregates
barely increased at a time when the country witnessed its most rapid growth and one
that was fuelled by the private sector. It was an anomalous case of private sector
growth without private sector bank financing. Even allowing for the over- exuberance
of the PSBs that financed this investment-led growth phase, the reticence of the
private sector was striking

67.

Solution: a)

Minimum Support Price (MSP) for Minor Forest Produce (MFP) has been fixed for
ten MFP namely Tamarind, Honey, Gum Karaya, Karanj Seed, Sal Seed, Mahuwa
Seed, Sal Leaf, Chironjee, Myrobalan, and Lac. The Scheme has been implemented

http://www.insightsonindia.com

INSIGHTS

Test - 18

in eight States having Schedule areas as listed in the Fifth Schedule of the
constitution of India
The scheme provides for non-nationalization of MFP by the State concerned, so that
the collectors of MFP are free to sell the MFP in open market if the market price is
more than the MSP declared. States of Andhra Pradesh and Telangana are still
continuing with the practice of nationalization of MFP. Therefore, the scheme has
not been implemented in these two States.

68.

Solution: d)

The UPSC consists of a chairman and other members appointed by the president of
India. The Constitution, without specifying the strength of the Commission has left
the matter to the discretion of the president, who determines its composition.
Usually, the Commission consists of nine to eleven members including the chairman.
Further, no qualifications are prescribed for the Commissions membership except
that one-half of the members of the Commission should be such persons who have
held office for at least ten years either under the Government of India or under the
government of a state. The Constitution also authorises the president to determine
the conditions of service of the chairman and other members of the Commission.
The chairman and members of the Commission hold office for a term of six years or
until they attain the age of 65 years, whichever is earlier.

69.

Solution: b)

The UPSC presents, annually, to the President a report on its performance. The
President places this report before both the Houses of Parliament, along with a
memorandum explaining the cases where the advice of the Commission was not
accepted and the reasons for such non-acceptance. All such cases of non-acceptance
must be approved by the Appointments Committee of the Union cabinet. An
individual ministry or department has no power to reject the advice of the UPSC.

70.

Solution: a)

Impetus to growth will be provided by declining oil prices and increasing monetary
easing facilitated by ongoing moderation in inflation.

http://www.insightsonindia.com

INSIGHTS

Test - 18

Insights Mock Test Series 2015: Test 18 Solutions


Simulating the effects of tax cuts, declining oil prices will add spending power to
households, thereby boosting consumption and growth. Oil is also a significant input
in production, and declining prices will shore up profit margins and hence balance
sheets of the corporate sector.
Further declines in inflation and the resulting monetary easing will provide policy
support for growth both by encouraging household spending in interest-sensitive
sectors and reducing the debt burden of firms, strengthening their balance sheets.
The final favourable impulse will be the monsoon which is forecast to be normal
compared to last year.

71. Solution: d)
About ZSI:
Established in 1961, it is a premier organisation in zoological research and studies.

It was established to promote the survey, exploration and research of the


fauna in the region.
The activities of the ZSI are coordinated by the Conservation and Survey
Division in the Ministry of Environment and Forests, Government of India.

Main objectives:

Exploration and Survey of Faunal Resources


Taxonomic Studies
Status Survey of Endangered Species
Publication of Results through Departmental Journals
Publication of Fauna of India
Maintenance and Development of National Zoological Collections

Red Data Book:

Similar to the Red Data Book produced by IUCN, ZSI also releases a Red
Data Book on Indian Animals. It was first published in 1983.

The ZSI also participates in the Indian Antarctic Program, since its inception in
1989.

72.

Solution: d)

Mangroves in India:
Mangroves in India account for about 3% of the worlds mangrove vegetation.
Mangrove cover in India is 4,662 sq. km, which is 0.14% of the countrys total

http://www.insightsonindia.com

INSIGHTS

Test - 18

geographical area. Sundarbans in West Bengal accounts for almost half of the total
area under mangroves in the country. Mangrove in India is famous for its rich
variety of flora and fauna.
The forest survey report, 2013, notes down that the overall Mangrove cover in the
county in 2013 has decreased compared to 2011.The reasons for the decrease in the
mangrove cover may be:

Grazing by domestic cattle and exploitation of mangrove woods for fuel and
timber.
The neo-tectonic movement of river courses.
Abatement of upstream freshwater discharges due to construction of dams
and reservoirs.
Rapid trend of reclamation of mangrove forests for habitations.
Pollutant discharges from cities and industries etc.

Composition of Mangroves in India: The very dense mangrove comprises 1,403 sq.
km (30.10% of the total mangrove cover), moderately dense mangrove is 1,658.12
sq. km (35.57 %) while open mangroves cover an area of 1,600.44 sq. km (33%).

73.

Solution: d)

Recommendations of 255th Report of Law Commission with respect to ECI

Appointments of Chief Election Commissioners (CEC) and all Election


Commissioners (ECs) - It should be made by the President in consultation
with a three-member collegium or selection committee.

The collegium or committee shall consist of the Prime Minister, the Leader of
the Opposition of the Lok Sabha (or the leader of the largest opposition party
in the Lok Sabha) and the Chief Justice of India (CJI).

Seniority based elevation- The elevation of an EC should be on the basis of


seniority. If senior EC is not appointed due to unfit reasons, the three member
collegium or a committee give reasons in writing.

Equal constitutional protection to all members of the ECI- Equal


constitutional protection must be given to all members of the ECI in matters
of removability from office.

In this regard, Article 324(5) of the Constitution should be amended.

74.

Solution: a)

http://www.insightsonindia.com

INSIGHTS

Test - 18

Insights Mock Test Series 2015: Test 18 Solutions

The scheme is a web-enabled system created by National Pharmaceutical


Pricing Authority (NPPA).

It seeks to serve as a robust E-governance tool for protection of interests of


consumers through effective implementation of the price of drugs.

The scheme will provide consumers with an online facility to redress their
complaints related to over-pricing of medicines, non-availability of medicines
and refusal of supply for sale of any medicine without good and sufficient
reason.

After receiving the complaint, NPPA will initiate action on any complaint
within 48 hrs.

The scheme also seeks to create awareness among the people and act as a
deterrent against black-marketing, spurious medicines and inflated cost of
drugs by creating phama-literacy initiative.

75.Solution: a)
Directorate General of Civil Aviation has restricted civil use of drones/unmanned
aircraft systems (UAS) vide public notice dated 07.10.2014 available on DGCA`s
website www.dgca.nic.in. As of now, the Aircraft Rules do not cover use of drones as
well as its sale and purchase.
However, DGCA is in the process of developing an interim operations guideline for
civil use of UAS. Till such regulations are issued, no non government agency,
organization or an individual will launch a UAS in Indian Civil Airspace for any
purpose whatsoever.
PIB Releases

76.

Solution: a)

The power of growth to lift all boats will depend critically on its employment creation
potential. The data on longer-term employment trends are difficult to interpret
because of the bewildering multiplicity of data sources, methodology and coverage.
One tentative conclusion is that there has probably been a decline in long run
employment growth in the 2000s relative to the 1990s and probably also a decline in
the employment elasticity of growth: that is, a given amount of growth leads to fewer
jobs created than in the past.

http://www.insightsonindia.com

INSIGHTS

Test - 18

http://www.livemint.com/Opinion/zAcuIvXlzIfML7N2Q10L2H/Employmentelasticity-almost-zero.html

77. Solution: a)
Refer to Box 1.2 of the First Chapter Volume 1 of the Economic Survey 2014-15.
There are several other steps that have been taken to boost growth in the country.
These are important and you should be aware of it.

78.

Solution: d)

Reforms of labor and land laws and reducing the costs of doing business will need to
be a joint endeavor of the States and Center. The game-changing potential of
implementing the GST and moving to technology enabled Direct Benefit Transfers
which we call the JAM (Jan Dhan-Aadhaar-Mobile) Number Trinity solution-should
not be underestimated.
The objective should be to create a competitive, predictable, clean, and exemptionslight tax policy regime that will lower the cost of capital, incentivize savings, and
facilitate taxpayer compliance.

79.

Solution: a)

JAM is an abbreviation for Jan Dhan Yojana, Aadhaar and Mobile number. The
government is pinning its hopes on these three modes of identification to direct
benefits transfers to poor. It will help to stop and control the lekages in subsudy. The
Aadhaar Number is linked to an active bank account is key to implementing income
transfers. With the introduction of Jan Dhan Yojana, the number of bank accounts is
expected to increase further and offering greater opportunities to target and transfer
financial resources to the poor.
The Centre and States pay wages such as MGNREGA, supply rice, wheat, pulses,
cooking oil, sugar and kerosene at heavily subsidised prices through the PDS. Then,
sectors such as power, fertilisers and oil sell their products to people below market
prices. When all these are linked with JAM trinity then the corruption will be
reduced.

http://www.insightsonindia.com

INSIGHTS

Test - 18

Insights Mock Test Series 2015: Test 18 Solutions

80.

Solution: a)

The persistence of moderated oil prices seems highly probable for at least three
reasons: weaker global demand, increased supplies, and the global monetary and
liquidity environment.
Demand will remain soft because of slow growth in major areas of the world
economy, including China and Europe. Supply shifts are occurring related to the
increase in crude-oil and shale gas production in the US and the concomitant decline
in the oligopolistic power of OPEC, notably its swing producer, Saudi Arabia (which
decided not to react to the increase in supply from other sources). Going forward,
prices could increasingly be determined by the marginal cost of shale production
estimated at around US$ 60-65 per barrel.

81.Solution: d)
The outlook is favourable for the current account and its financing. A likely surfeit,
rather than scarcity, of foreign capital will complicate exchange rate management.
Risks from a shift in US monetary policy an turmoil in the Eurozone need to be
watched but could remain within control.
One source of concern is muted export growth and rising non-oil, non-gold imports
which could be affected by Indias deteriorating competitiveness, reflected in the
appreciation of the real effective exchange rate by 8.5 per cent since January 2014.

82.

Solution: a)

The Election Commission has been functioning as a multi-member body consisting


of three election commissioners. It can be more too.
The chief election commissioner and the two other election commissioners have
equal powers and receive equal salary, allowances and other perquisites, which are
similar to those of a judge of the Supreme Court. In case of difference of opinion
amongst the Chief Election Commissioner and/or two other election commissioners,
the matter is decided by the Commission by majority. They hold office for a term of
six years or until they attain the age of 65 years, whichever is earlier. They can resign
at any time or can also be removed before the expiry of their term.

http://www.insightsonindia.com

INSIGHTS

Test - 18

83.

Solution: d)

He cannot be removed from his office except in same manner and on the same
grounds as a judge of the Supreme Court. In other words, he can be removed by the
president on the basis of a resolution passed to that effect by both the Houses of
Parliament with special majority, either on the ground of proved misbehaviour or
incapacity. Thus, he does not hold his office till the pleasure of the president, though
he is appointed by him. The service conditions of the chief election commissioner
cannot be varied to his disadvantage after his appointment.

84.

Solution: a)

Some flaws can be noted in the Constitution with regard to the ECI, viz.,

85.

The Constitution has not prescribed the qualifications (legal, educational,


administrative or judicial) of the members of the Election Commission.
The Constitution has not specified the term of the members of the Election
Commission (the term of the Chief EC has been prescribed though).
The Constitution has not debarred the retiring election commissioners from
any further appointment by the government.

Solution: d)

In details, these powers and functions are:

To determine the territorial areas of the electoral constituencies throughout


the country on the basis of the Delimitation Commission Act of Parliament.
To prepare and periodically revise electoral rolls and to register all eligible
voters.
To notify the dates and schedules of elections and to scrutinise nomination
papers.
To grant recognition to political parties and allot election symbols to them.
To act as a court for settling disputes related to granting of recognition to
political parties and allotment of election symbols to them.
To appoint officers for inquiring into disputes relating to electoral
arrangements.

http://www.insightsonindia.com

INSIGHTS

Test - 18

Insights Mock Test Series 2015: Test 18 Solutions

86.

To determine the code of conduct to be observed by the parties and the


candidates at the time of elections.
To prepare a roster for publicity of the policies of the political parties on radio
and TV in times of elections.
To advise the president on matters relating to the disqualifications of the
members of Parliament.
To advise the governor on matters relating to the disqualifications of the
members of state legislature.
To cancel polls in the event of rigging, booth capturing, violence and other
irregularities.
To request the president or the governor for requisitioning the staff necessary
for conducting elections.
To supervise the machinery of elections throughout the country to ensure free
and fair elections.
To advise the president whether elections can be held in a state under
presidents rule in order to extend the period of emergency after one year.
To register political parties for the purpose of elections and grant them the
status of national or state parties on the basis of their poll performance

Solution: a)

The Election Commission is assisted by deputy election commissioners. They are


drawn from the civil service and appointed by the commission with tenure system.
They are assisted, in turn, by the secretaries, joint secretaries, deputy secretaries and
under secretaries posted in the secretariat of the commission.
At the state level, the Election Commission is assisted by the chief electoral officer
who is appointed by the chief election commissioner in consultation with the state
government. Below this, at the district level, the collector acts as the district
returning officer. He appoints a returning officer for every constituency in the district
and presiding officer for every polling booth in the constituency.

87.

Solution: a)

The Intergovernmental Panel on Climate Change (IPCC) in its recent report the
Fifth Assessment Report (AR5), published in 2014 has observed that, there has

http://www.insightsonindia.com

INSIGHTS

Test - 18

been an increasing trend in the anthropogenic emissions of greenhouse gases (GHG)


since the advent of the industrial revolution.
The change in the climate system is likely to have adverse impacts on livelihoods,
cropping pattern and food security. Extreme heat events are likely to be longer and
more intense in addition to changes in the precipitation patterns. Adverse impacts
are likely to be felt more acutely in tropical zone countries such as India, and within
India, the poor will be more exposed.

88.

Solution: a)

The move towards the formation of a socialist party was made in the jails during
1930-31 and 1932-34 by a group of young Congressmen who were disenchanted with
Gandhian strategy and leadership and attracted by socialist ideology. Many of them
were active in the youth movement of the late 1920s. In the jails they studied and
discussed Marxian and other socialist ideas. Attracted by Marxism, communism and
Soviet Union, they did not find themselves in agreement with the prevalent political
line of the CPI. Many of them were groping towards an alternative. Ultimately they
came together and formed the Congress Socialist Party (CSP) at Bombay in October
1934 under the leadership of Jayaprakash Narayan, Acharya Narendra Dev and
Minoo Masani. From the beginning, all the Congress socialists were agreed upon four
basic propositions: that the primary struggle in India was the national struggle for
freedom and that nationalism w..s a necessary stage on the way to socialism; that
socialists must work inside the National Congress because it was the primary body
leading the national struggle and, as Acharya Narendra Dev put it in 1934, It would
be a suicidal policy for us to cut ourselves 3ff from the national movement that the
Congress undoubtedly represents; that they must give the Congress and the national
movement a socialist direction; and that to achieve this objective they must organize
the workers and peasants in their class organizations, wage struggles for their
economic demands and make them the social base of the national struggle.

89.

Solution: a)

http://www.insightsonindia.com

INSIGHTS

Test - 18

Insights Mock Test Series 2015: Test 18 Solutions

90.

Solution: a)

In simple words Gross Capital Formation is Investment. When people save, they tend
to invest. The percentage of the investment made each year out of the total GDP is
called Gross Capital Formation.
The importance of the Gross Capital formation lies in the fact that this is that part of
GDP which helps in the growth of the GDP itself. This is a must for achieving high
rate of production, capital formation, changes in production techniques and
changing in the outlook of the people themselves.

91. Solution: d)
The various features of administration contained in the Fifth Schedule are as follows:

Declaration of Scheduled Areas: The president is empowered to declare an


area to be a scheduled area. He can also increase or decrease its area, alter its
boundary lines, rescind such designation or make fresh orders for such
redesignation on an area in consultation with the governor of the state
concerned.
Executive Power of State and Centre: The executive power of a state extends
to the scheduled areas therein. But the governor has a special responsibility

http://www.insightsonindia.com

INSIGHTS

Test - 18

92.

regarding such areas. He has to submit a report to the president regarding the
administration of such areas, annually or whenever so required by the
president. The executive power of the Centre extends to giving directions to
the states regarding the administration of such areas.
Tribes Advisory Council: Each state having scheduled areas has to establish a
tribes advisory council to advise on welfare and advancement of the scheduled
tribes. It is to consist of 20 members, three-fourths of whom are to be the
representatives of the scheduled tribes in the state legislative assembly. A
similar council can also be established in a state having scheduled tribes but
not scheduled areas therein, if the president so directs.

Solution: b)

The tribal areas in the four states of Assam, Meghalaya, Tripura and Mizoram have
been constituted as autonomous districts. But, they do not fall outside the executive
authority of the state concerned.
The governor is empowered to organise and re-organise the autonomous districts.
Thus, he can increase or decrease their areas or change their names or define their
boundaries and so on.
Each autonomous district has a district council consisting of 30 members, of whom
four are nominated by the governor and the remaining 26 are elected on the basis of
adult franchise.
The elected members hold office for a term of five years (unless the council is
dissolved earlier) and nominated members hold office during the pleasure of the
governor. Each autonomous region also has a separate regional council.

93.

Solution: a)

The district and regional councils administer the areas under their jurisdiction. They
can make laws on certain specified matters like land, forests, canal water, shifting
cultivation, village administration, inheritance of property, marriage and divorce,
social customs and so on. But all such laws require the assent of the Governor.
The district and regional councils within their territorial jurisdictions can constitute
village councils or courts for trial of suits and cases between the tribes. They hear
appeals from them. The jurisdiction of high court over these suits and cases is
specified by the governor.

http://www.insightsonindia.com

INSIGHTS

Test - 18

Insights Mock Test Series 2015: Test 18 Solutions

94.

Solution: b)

In assessing the risks and rewards of competing destinations, rational investors take
into account not just macroeconomic stability (which proxies for risks) but also
growth which crucially determines rewards and returns.
The RIRI is computed by averaging a countrys GDP growth rate and its macroeconomic indicators; the latter measured as the average of the fiscal deficit, current
account deficit, and inflation (all with negative signs). Thus, equal weight is given to
growth and macroeconomic stability. The greater the number, the better should be
its investor rating. Since, updated WEO forecasts are not publicly available for all
countries, data are from Citi Group and have been updated in January assuming an
oil price in the range of US$ 58-60 per barrel for 2015. Data from other sources yield
very similar estimates for the RIRI.

95.

Solution: a)

Gujarat has a land-pooling model whereby 50 per cent of the land is acquired while
the remaining 50 per cent is left with the original owners giving them a stake in the
upsides generated by land monetisation. Maharashtra allows for negotiated
purchase involving various stakeholders. In Haryana and Rajasthan, trunk and
industrial infrastructure are created by the state governments but private developers
directly participate in the other activities. The value increase is captured by the states
through development fees. Furthermore, in the initial DMIC master-planning
process, the attempt was made to identify large, easy-to-acquire land parcels that
were either barren or government owned

96.

Solution: a)

HDR measures inequality in terms of two indicators. The first indicator is the income
Gini
Coefficient which measures the deviation of distribution of income (or consumption)
among the individuals within a country from a perfectly equal distribution. For
India, the income Gini coefficient was 36.8 in 2010-11. In this respect, inequality in
India is lower than many other developing countries, e.g., South Africa (57.8), Brazil
(53.9), Thailand (53.6), Turkey (40.8), China (41.5), Sri Lanka (40.3), Malaysia
(46.2), Vietnam (37.6), as well as countries like USA (40.8), Hong Kong (43.4),

http://www.insightsonindia.com

INSIGHTS

Test - 18

Argentina (45.8), Israel (39.2), Bulgaria (45.3) etc., which are otherwise ranked very
high in terms of human development index.

97.

Solution: d)

The poverty estimates indicate that the highest poverty headcount ratio (HCR) exists
in Bihar at 53.5 per cent as against the national average of 29.8 per cent. In 2009-10
compared to 2004-5, Bihar has displaced Odisha as the poorest state, SocioEconomic and Caste Census (SECC) with Odishas situation improving considerably
in 2009-10. Lowest poverty is in Himachal Pradesh (9.5 per cent) followed by Kerala
(12 per cent).
Bihar has the highest decadal (2001-11) growth rate of population (25.07 per cent),
while Kerala has the lowest rate (4.86 per cent). Some big states like Gujarat,
Haryana, Madhya Pradesh, Rajasthan, and Uttar Pradesh also have high decadal
growth of population.

98.

Solution: b)

RSBY has been launched by Ministry of Labour and Employment, Government of


India to provide health insurance coverage for Below Poverty Line (BPL) families.
The objective of RSBY is to provide protection to BPL households from financial
liabilities arising out of health shocks that involve hospitalization. Beneficiaries
under RSBY are entitled to hospitalization coverage up to Rs. 30,000/- for most of
the diseases that require hospitalization. Government has even fixed the package
rates for the hospitals for a large number of interventions. Pre-existing conditions
are covered from day one and there is no age limit.
Coverage extends to five members of the family which includes the head of
household, spouse and up to three dependents. Beneficiaries need to pay only Rs.
30/- as registration fee while Central and State Government pays the premium to the
insurer selected by the State Government on the basis of a competitive bidding.

99.

Solution: c)

During Indias rapid growth phase between 2002-03 and 2008-09, the ratio of
exports of services to GDP increased dramatically, from 4.0 percent to nearly 9.0
percent. In contrast, manufacturing exports were less buoyant After the global
financial crisis, however, the roles seem to have been reversed; manufacturing
http://www.insightsonindia.com

INSIGHTS

Test - 18

Insights Mock Test Series 2015: Test 18 Solutions


exports seem to have done better than services exports. More worrisome, however,
both have slowed down in the last five years which does not augur well.

100. Solution: a)
Duties that are imposed in order to counter the negative impact of import subsidies
to protect domestic producers are called countervailing duties.
Description: In cases foreign producers attempt to subsidize the goods being
exported by them so that it causes domestic production to suffer because of a shift in
domestic demand towards cheaper imported goods, the government makes
mandatory the payment of a countervailing duty on the import of such goods to the
domestic economy.
This raises the price of these goods leading to domestic goods again being equally
competitive and attractive. Thus, domestic businesses are cushioned. These duties
can be imposed under the specifications given by the WTO (World Trade
Organization) after the investigation finds that exporters are engaged in dumping.
These are also known as anti-dumping duties.

http://www.insightsonindia.com

INSIGHTS

Test - 18

INSIGHTS ON INDIA MOCK PRELIMINARY EXAM - 2015


INSIGHTS ON INDIA MOCK TEST - 19
GENERAL STUDIES

PAPER-I
Time Allowed: 2 Hours

Maximum Marks: 200

INSTRUCTIONS
1. IMMEDITELY AFTER THE COMMENCEMENT OF THE EXAMINATION, YOU SHOULD
CHECK THAT THIS TEST BOOKLET DOES NOT HAVE ANY UNPRINTED OR TORN OR
MISSING PAGES OR ITEMS, ETC. IF SO, GET IT REPLACED BY A COMPLETE TEST BOOKLET.
2. You have to enter your Roll Number on the Test I
Booklet in the Box provided alongside. DO NOT
write anything else on the Test Booklet.
4. This Test Booklet contains 200 items (questions). Each item is printed only in English. Each item
comprises four responses (answers). You will select the response which you want to mark on the Answer
Sheet. In case you feel that there is more than one correct response, mark the response which you consider
the best. In any case, choose ONLY ONE response for each item.
5. You have to mark all your responses ONLY on the separate Answer Sheet provided. See directions in the
Answer Sheet.
6. All items carry equal marks.
7. Before you proceed to mark in the Answer Sheet the response to various items in the Test Booklet, you
have to fill in some particulars in the Answer Sheet as per instructions sent to you with your Admission
Certificate.
8. After you have completed filling in all your responses on the Answer Sheet and the examination has
concluded, you should hand over to the Invigilator only the Answer Sheet. You are permitted to take away
with you the Test Booklet.
9. Sheets for rough work are appended in the Test Booklet at the end.
10. Penalty for wrong answers :
THERE WILL BE PENALTY FOR WRONG ANSWERS MARKED BY A CANDIDATE IN THE
OBJECTIVE TYPE QUESTION PAPERS.
(i)

There are four alternatives for the answer to every question. For each question for which a
wrong answer has been given by the candidate, one-third of the marks assigned to that question
will be deducted as penalty.

(ii) If a candidate gives more than one answer, it will be treated as a wrong answer even if one of the
given answers happens to be correct and there will be same penalty as above to that question.
(iii)

If a question is left blank, i.e., no answer is given by the candidate, there will be no penalty for
that question.
http://insightsonindia.com

INSIGHTS ON INDIA MOCK TEST SERIES FOR CIVIL SERVICES PRELIMINARY EXAM 2015

http://insightsonindia.com

Page 1

1. If the Earths shape had been flat


instead of being spherical, which of the
following would happen?
1. The whole earth will have Sun rise
and Sun set at the same time.
2. Visible horizon remains the same
irrespective of altitude.
3. Diversity of seasons would not have
been possible on Earth.
Choose the correct answer using the codes
below.
a)
b)
c)
d)

1 and 2 only
2 and 3 only
1 and 3 only
All of the above

Choose the correct answer using the codes


below.
a)
b)
c)
d)

1 and 2 only
2 and 3 only
1 and 3 only
All of the above

4. Consider the following statements.


1. The Sun is never overhead at any
time of the year anywhere on earth
beyond tropics.
2. Beyond the tropics, latitude is
preferable to calculate distances
than longitude.
Which of the above is/are true?

2. Nikshay has been launched by the


Central Government which is a case
Based Web Based recording and
reporting system for
a) Tuberculosis cases
b) HIV Positive cases in LGBT
community
c) Hepatitis B cases in rural areas
d) Japanese Encephalitis cases

3. Indias first Polar Remotely Operated


Vehicle (PROVe) was successfully
operationalised for research in North
Antarctica by National Institute of
Ocean Technology (NIOT). Consider
the following about it.
1. It has been indigenously built
under the Union Ministry of Earth
Sciences.
2. It is also capable of probing the sea
bed.
3. PROVe will help understanding the
phenomenon of Monsoon better.

http://insightsonindia.com

a)
b)
c)
d)

1 only
2 only
Both 1 and 2
None

5. Which of these statements is true?


1. A traveller crossing the date line
from east to West loses a day (in
time).
2. There is a difference of one whole
day in time between the two sides
of 180 degree meridian.
3. The period of twilight is longer in
Britain than in Malaysia.
Choose the correct answer using the codes
below.
a)
b)
c)
d)

1 and 2 only
2 and 3 only
1 and 3 only
All of the above

Page 2

6. Savings as a percentage of GDP is an


important economic indicator.
Consider the following statistics with
regard to the last one decade.
1. Savings by households has
persistently declined over this
decade.
2. Physical savings by households has
always been greater than the
financial savings.
3. Household savings has always been
greater than savings from public
and private sector.
Choose the correct answer using the codes
below.
a)
b)
c)
d)

1 and 2 only
2 and 3 only
1 and 3 only
All of the above

7. Fold mountains are formed due to


large scale movements in the Earths
surface when stresses are setup in the
earths crust. These stresses that lead
to their formation can be induced by
which of the following?
1. Increased load of overlying rocks
2. Flow movements in the mantle
3. Magnetic intrusions in the crust
Choose the correct answer using the codes
below.
a)
b)
c)
d)

1 and 2 only
2 and 3 only
1 and 3 only
All of the above

8. If the Indian government wants to


amend the Companies Act 2013, which
of the following ministry will be
http://insightsonindia.com

primarily concerned with this


amendment and its implementation?
a) Ministry of Law and Justice
b) Ministry of Heavy Industries and
Public Enterprises
c) Ministry of Corporate Affairs
d) Ministry of Parliamentary Affairs

9. In humid highlands, several dissected


plateaus like Scottish Highlands are
found. Which of the forces are
responsible for the formation of such
dissected plateaus?
1. Stream action
2. Cuts done due to Glaciation
3. Abrasion by Wind
Choose the correct answer using the codes
below.
a)
b)
c)
d)

1 and 2 only
2 and 3 only
1 and 3 only
All of the above

10. Consider the following statements


about Loess.
1. These are blown from deserts and
barren surfaces and deposited upon
hills and valleys.
2. Areas covered by Loess are infertile
lands.
3. They are responsible for the
formation of peneplain.
Choose the correct answer using the codes
below.
a)
b)
c)
d)

1 and 2 only
2 and 3 only
1 and 3 only
All of the above

Page 3

11. Consider the following statements


about the treatise named
SoopaShastra.
1. It is the only compilation in a
regional language during the
medieval ages.
2. It is entirely devoted to cookery.
3. The treatise belongs to the period
of Hoysala dynasty.
Choose the correct answer using the codes
below.
a)
b)
c)
d)

1 and 2 only
2 and 3 only
1 and 3 only
All of the above

12. Consider the following statements


regarding usage of Urea
1. Excessive use of Urea encourages
climate change
2. Excessive use of urea cause
groundwater pollution
3. Use of Urea beyond recommended
rates enhances crop succulence,
thus making the plants prone to
disease and pest infestation, and to
lodging.
Which of the above statements is/are correct?
a)
b)
c)
d)

1 Only
1 and 2 Only
2 and 3 Only
All

13. The goals of RNTCP are to decrease


mortality and morbidity due to TB and
cut transmission of infection until TB
ceases to be a major public health
problem in India. Which of the
following are concerned with RNTCP?
1. DOTS (Directly Observed
treatment short course)
http://insightsonindia.com

2. ACSM (Advocacy, communication,


and social mobilization)
3. CB-NAAT (Cartridge-based nucleic
acid amplification)
Choose the correct answer using the codes
below.
a)
b)
c)
d)

1 and 2 only
2 and 3 only
1 and 3 only
All of the above

14. With reference to All India Kisan


Sabha, consider the following
statements:
1. Swami Sahajanand was its first
President and N. G. Ranga was its
first General Secretary
2. Its ideology was mainly leftist
3. Unlike other political organizations
of the time, Kisan Sabha worked
independently of the Indian
National Congress and never
associated with it in any form due
to ideological differences
Which of the above statements is/are correct?
a)
b)
c)
d)

1 and 2 Only
3 and 2 Only
1 and 3 Only
All

15. Consider the following about


Governments effort to contain
Tuberculosis in India.
1. TB now is a notifiable disease
under Government notification.
2. Govt. of India has banned the use
of serology in the diagnosis of TB.
3. Standardized treatment guidelines
have been made compulsory for the
private sector.
Page 4

Choose the correct answer using the codes


below.
a)
b)
c)
d)

1 and 2 only
2 and 3 only
1 and 3 only
All of the above

16. The Pacific Ring of Fire is a chain of


volcanoes that extends for more than
2,000 miles and includes locations
from
1. New Zealand
2. Alaska
3. Pacific islands of Solomon
Choose the correct answer using the codes
below.
a)
b)
c)
d)

1 and 2 only
2 and 3 only
1 and 3 only
All of the above

17. A Hot water spring can exist at any


part of the earth where
a) water sinks deep enough beneath
the surface to be heated by
interior forces
b) gases in the interior of the Earth
are powerful enough to eject hot
water at high pressure from deep
below the earths surface
c) volcanoes necessarily exist near a
groundwater source
d) None of the above

2. The Chairman is appointed by the


President after a recommendation
is made from the Ministry of
Social Justice and empowerment.
3. The Conditions of Service and
tenure of the members of the
Commission is determined by the
Constitution.
Choose the correct answer using the codes
below.
a)
b)
c)
d)

1 and 2 only
2 and 3 only
3 only
None of the above

19. Consider the following statements


about the power of the National
Commission for STs.
1. The Central government and the
state governments are required to
consult the Commission on all
major policy matters affecting the
STs.
2. Annual report submitted by the
commission is placed before the
Parliament by the President.
3. The commission has powers of
investigation into cases concerning
the rights and safeguards of STs.
Choose the correct answer using the codes
below.
a)
b)
c)
d)

1 and 2 only
2 and 3 only
3 only
All of the above

18. Consider the following statements


about the National Commission for
STs.
1. It comes under the Ministry of
Social Justice and empowerment.

http://insightsonindia.com

Page 5

20. Jawahrlal Nehrus admiration for


Soviet Russia was due to many
reasons. Which of the following is/are
true?
1. He admired it because of Soviet
Russias socialist ideology
2. He admired because he considered
Soviet Russia as an anti-imperial
force and hope for the future
3. He admired Stalins policies

3. At the Central level, the


Commissioner falls under the
Ministry of Minority Affairs.
Choose the correct answer using the codes
below.
a)
b)
c)
d)

1 and 2 only
2 and 3 only
1 and 3 only
All of the above

Choose the correct answer using the codes


below:
a)
b)
c)
d)

1 and 3 Only
2 and 3 Only
2 and 1 Only
All

21. With reference to the Rashtriya


Swasthya Bima Yojana (RSBY) ,
consider the following statements
1. As of March 2015, the scheme
since its inception is being
implemented by the Ministry of
Health and Family Welfare
2. It is a health insurance scheme for
BPL (below poverty line) families
Which of the above statements is/are correct?
a)
b)
c)
d)

Both
None
1 Only
2 Only

22. Consider the following statements


about the Special Officer for Linguistic
Minorities.
1. Originally the Constitution did not
make any provision for it.
2. The Constitution does not mention
qualifications for the Office.

http://insightsonindia.com

23. The Defence Acquisition Council


(DAC), chaired by Defence Minister,
recently approved the development of
an indigenous Airborne Warning and
Control Systems (AWACS). AWACS
will be a
a) radars mounted on an aircraft to
provide seamless 360-degree
coverage of the airspace
b) a modern radar system on ground
to provide seamless 360-degree
coverage of the airspace for flying
fighter aircrafts
c) a radar system that will be
attached with the IRNSS for geopositioning purposes for early
attack warnings
d) None of the above

24. The Union Agriculture Ministry


recently approved the setting up of a
price stabilisation fund (PSF). The
fund will
1. be used to support market
interventions for managing prices
of all agricultural commodities for
which MSP is announced
2. be used to forward loans to
governments for creating
warehousing infrastructure

Page 6

3. be used by APMC Committees for


stabilising prices of agri
commodities in local markets
Choose the correct answer using the codes
below.
a)
b)
c)
d)

1 and 2 only
2 and 3 only
1 and 3 only
2 only

25. The One belt One Road initiative is an


economic belt along the Silk Road.
Which of the following regions are
expected to be developed due to the
initiative?
1. Middle East
2. North Africa
3. East Asia
Choose the correct answer using the codes
below.
a)
b)
c)
d)

1 and 2 only
2 and 3 only
1 and 3 only
All of the above

26. Consider the following statements


about NITI Aayog.
1. It is a think-tank and does not have
the power to impose policies.
2. It has no power to allocate funds.
3. The CEO of NITI Aayog is
appointed by the PM.
Choose the correct answer using the codes
below.
a)
b)
c)
d)

27. The Chairman and members of the


National Human Rights Commission
(NHRC) should be
a) Retired chief justice of India; and
Serving judges of the Supreme
Court
b) Retired chief justice of India; and
Serving or retired judges of the
Supreme Court
c) Serving chief justice of India; and
Serving or retired judges of the
Supreme Court or Chief Justice of
High Court
d) Retired chief justice of India; and
Serving or retired judges of the
Supreme Court or Chief Justice of
High Court; and persons having
knowledge or practical experience
with respect to human rights

28. Which of the following sectors has the


highest Labour productivity in the
Indian Economy?
a) Real Estate and Business Services
b) Financial Services and Insurance
c) Registered manufacturing
d) Public Administration and defence

29. The Chairman of NHRC can be


removed on the same grounds and
procedure as that of the
a) Chairman, UPSC
b) CAG
c) Member, Election Commission of
India
d) Judge, High Court

1 and 2 only
2 only
1 and 3 only
All of the above

http://insightsonindia.com

Page 7

30. Nehru was critical of some of


Gandhian policies and strategies.
Which of the following did he criticise
most?
1. He criticized Gandhiji for
refusing to recognize the
conflict of classes
2. He criticized Gandhiji for
preaching harmony among the
British and the Indians and
broadly between the exploiters
and the exploited
3. He criticized Gandhiji for
putting forward the theories of
trusteeship by, and conversion
of, the capitalists and landlords
Choose the correct answer using the codes
below:
a)
b)
c)
d)

1 and 3 Only
3 and 2 Only
2 and 1 Only
All

31. The chairman and members of NHRC


are appointed by the president on the
recommendations of a committee
consisting of the prime minister as its
head and
1. Central Home Minister
2. Deputy Chairman of Rajya Sabha
3. Leader of Opposition Lok Sabha
4. Leader of Opposition Rajya
Sabha
Choose the correct answer using the codes
below.
a)
b)
c)
d)

32. Prime Minister of India recently


launched multi-purpose and multimodal platform PRAGATI (Pro-Active
Governance and Timely
Implementation). Consider the
following about it.
1. It is essentially about addressing
grievances of a common man.
2. PRAGATI will monitor and review
state government projects also.
3. Geo-spatial technology has been
used in PRAGATI.
4. It will be headed by Ministry of
Statistics and Programme
Implementation.
Choose the correct answer using the codes
below.
a)
b)
c)
d)

1 and 2 only
3 and 4 only
1, 2 and 3 only
All of the above

33. Venus is often considered as Earths


twin because
a) its spin and rotation time around
the Sun is almost the same as that
of Earth
b) of its close proximity in Size, mass
(weight) and density with Earth.
c) its surface and core is made of the
same gases as that of Earth
d) it shares its natural satellites with
Earth

1 and 3 only
2, 3 and 4 only
1 and 2 only
All of the above

http://insightsonindia.com

Page 8

34. Consider the following statements.


1. While all public investment in the
railways is undertaken by the
central government, public
investment in roads is undertaken
by the central government as well
as state governments.
2. Indian Railways has received more
public funding than all other
transportation infrastructure in
India put together.
Which of the above is/are true?
a)
b)
c)
d)

2 only
1 only
Both 1 and 2
None

35. Consider the following statements


about the railways coal freight
comparison between China and India.
1. China carries about thrice as much
coal freight per hour vis--vis India.
2. Coal is transported in India at more
than twice the cost vis--vis China.
3. It takes longer in India to transport
coal than China via railways.
Choose the correct answer using the codes
below.
a)
b)
c)
d)

1 and 2 only
2 and 3 only
1 and 3 only
All of the above

36. The Supreme Court has recently


declared Section 66A of the IT Act as
unconstitutional. Which of the
following activities that could have
been charged under the law earlier can
now be allowed based on the
judgment?
http://insightsonindia.com

1. A comment on Social networking


website that is viewed as offensive
by some individuals or community
2. A false declaration in public (not on
internet) about an incident or fact
that has not occurred
3. A private e-mail message that
causes insult to an individual
Choose the correct answer using the codes
below.
a)
b)
c)
d)

1 only
2 and 3 only
1 and 3 only
1 and 2 only

37. In this years budget, the government


proposed to institutionally separate
debt management from monetary
policy by way of a Public Debt
Management Agency. By this
separation which of the following
powers will be lost by RBI?
1. deciding how much of the old debt
RBI
wants to buy or sell (open market o
perations)
2. power to issue currency to the
Government
3. power to issue rules and guidelines
for debt management to public
sector banks
Choose the correct answer using the codes
below.
a)
b)
c)
d)

1 only
2 and 3 only
1 and 3 only
None of the above

Page 9

38. The Congress could not attempt a


complete overhaul of the agrarian
structure by completely eliminating
the zamindari system when it formed
provincial governments during 1930.
Consider the following reasons:
1. According to the constitutional
structure of the 1935 Act, the
provincial Ministries did not have
enough powers to do so
2. They suffered from an extreme
lack of financial resources, for the
lions share of Indias revenues
was appropriated by the
Government of India
Which of the above reasons is/are correct?
a)
b)
c)
d)

None
Both
2 Only
1 Only

39. Consider the following statements


about our Solar system.
1. All the planets revolve around the
Sun in elliptical orbits.
2. No gas that is found on Earth is
found on Jupiter and Saturn.
3. The only planet to rotate in
clockwise direction around the Sun
is Uranus.
Choose the correct answer using the codes
below.
a)
b)
c)
d)

a) All parts of the globe would have


equal days and nights at all times of
the year.
b) The temperature would be the same
at all points at a given time on the
Earth where there is daylight.
c) only the global distribution of
temperature would change a little;
and the rest would be the same as it
is now.
d) Seasonal changes would become
more diverse and rapid.

41. The Union Cabinet has recently given


its in-principle approval for the
concept and institutional framework of
Sagarmala Project. Consider the
following statements about it.
3. It seeks to develop a string of
ports around Indias coast.
4. It would start Coastal Economic
Zones (CEZ) around sea coasts in
India.
5. India and China will be partners
for this project.
Choose the correct answer using the codes
below.
a)
b)
c)
d)

1 and 2 only
2 and 3 only
1 only
All of the above

1 and 2 only
2 and 3 only
1 and 3 only
All of the above

40. The axis of the Earth is inclined to the


plane of the elliptic in which it rotates.
If its axis were perpendicular to this
plane then
http://insightsonindia.com

Page 10

42. The Jan Ausadhi Campaign was set


up with the intention of providing
access to affordable medicines for all.
Consider the following statements
about the new campaign and schemes
related to the same.
1. It seeks to dispel the myth that
quality of medicines is linked to
its price.
2. The scheme is highly dependent
on subsidies from Central and
State Governments.
3. As a part of the campaign, State
Government has to ensure
prescription of unbranded generic
medicines by the Government
doctors.
Choose the correct answer using the codes
below.
a)
b)
c)
d)

1 and 2 only
2 and 3 only
1 and 3 only
All of the above

43. With reference to the Competition


Appellate Tribunal (COMPAT) and the
Competition Commission of India
(CCI), consider the following
statements:
1. COMPAT is a statutory body
whereas CCI is a quasi judicial body
2. If CCI imposes penalty of X Rupees
on Y company, this company can
appeal to COMPAT for justice
Which of the above statements is/are
INCORRECT?
a)
b)
c)
d)

44. Consider the following statements


about Municipal bonds.
1. They are issued by the Central
government to finance municipal
projects.
2. Any interest or payment from the
bond is given only after the
municipal project has been
completed.
3. Municipal bonds cannot be tax
exempted as they relate to local
governments.
Choose the correct answer using the codes
below.
a)
b)
c)
d)

1 and 2 only
2 and 3 only
1 and 3 only
None of the above

45. Consider the following statements


about Pradhan Mantri Kaushal Vikas
Yojana (PMKVY).
1. It will be implemented with the
help of the private sector and
National Skill Development
Corporation (NSDC).
2. It is supported by the World Bank.
3. The State government would have a
role in monitoring the
implementation of the scheme.
Choose the correct answer using the codes
below.
a)
b)
c)
d)

1 and 2 only
2 and 3 only
1 and 3 only
All of the abov

2 Only
Both
1 Only
None

http://insightsonindia.com

Page 11

46. Mission Indradhanush launched by the


Central Government is concerned with
a) covering all those children who
are partially vaccinated or
unvaccinated.
b) providing insurance cover to
drought affected areas
c) covering nutritional support for
children suffering from
development related disabilities
d) tracking the mental health of
people from BPL families

47. Which of these sectors have shown


negative growth in overall employment
shares in the Economy in the postreforms period?
1. Registered Manufacturing
2. Public Administration and defence
3. Construction
Choose the correct answer using the codes
below.
a)
b)
c)
d)

1 and 2 only
2 and 3 only
1 and 3 only
All of the above

48. Consider the following statements:


1. He spearheaded the movement for
separate statehood for Gujarat
2. He was a prominent leftist who
worked outside the framework of
any organized left-wing party
during freedom struggle
To which of the following persons does above
description aptly apply?
a)
b)
c)
d)

Sardar Vallabhbhai Patel


Indulal Yagnik
Swami Sahajanand Saraswati
N. G. Ranga

http://insightsonindia.com

49. Consider the following statements


about Rajasthans human milk bank,
Jeevan Dhara.
1. It has been started by a NGO in
association with the Rajasthan
Government.
2. Under the scheme, free human
milk will be provided to
malnourished infants all over
Rajasthan.
3. The scheme is being funded by the
WHO.
Choose the correct answer using the codes
below.
a)
b)
c)
d)

1 and 2 only
2 and 3 only
3 only
None of the above

50. Which of the following are responsible


for creating the double financial
repression as pitched by the
Economic Survey?
1. Statutory Liquidity Ratio (SLR)
2. Priority Sector Lending (PSL)
3. Non Performing Assets (NPAs)
Choose the correct answer using the codes
below.
a)
b)
c)
d)

1 and 2 only
2 and 3 only
1 and 3 only
All of the above

51. Which of these rocks make up for large


portions of the crust of Earth?
1. Granitic rocks
2. Basaltic rocks
3. Pumic rocks
4. Obsidian rocks
Page 12

Choose the correct answer from the codes


given below.
a)
b)
c)
d)

2 and 3 only
1 and 2 only
1, 3 and 4 only
All of the above

52. I have never been able to take pride in


the fact that Burma has been made
part of British India. It never was and
never should be. The Burmese have a
civilization of their own.
Which of the following Indian freedom
fighters said the above lines?
a)
b)
c)
d)

Jawaharlal Nehru
Rabindranath Tagore
Mahatma Gandhiji
C Rajgopalchari

53. Consider the following statements


about igneous rocks.
1. They are generally crystalline in
nature.
2. They occur in layers and often
contain fossils.
3. Igneous rocks are always acidic in
nature.
Choose the correct answer using the codes
below.
a)
b)
c)
d)

1 only
2 and 3 only
1 and 3 only
All of the above

54. With reference to the Mid Day Meal


scheme, consider the following
statements:
1. It was launched by the Ministry of
Food and Civil Supplies in 1995
http://insightsonindia.com

2. The nutritional guidelines for


minimum amount of protein in the
food and calorie content per child
per day for Upper Primary school
students are 20 gram and 700
calories respectively
Which of the above statements is/are correct?
a) 2 Only
b) 1 Only
c) None
d) Both
55. Without a magnetic field, our planet
will lose its atmosphere as per
Researchers. How is Earths magnetic
field helping to hold the atmosphere?
a) it deflects ionizing particles from
the Sun which would otherwise
wear atmosphere away
b) The charged particles in the
Earths atmosphere are being held
by this magnetic field
c) Magnetic field supports the
gravitation forces in increasing
the escape velocity of gas particles
d) None of the above

56. Consider the following statements


about the CAG.
1. No minister can represent the CAG
in Parliament.
2. The Parliament decides upon the
extent of powers of the CAG.
3. CAG needs the prior permission of
the Governor before auditing the
concerned State Governments
accounts.
Choose the correct answer using the codes
below.
a)
b)
c)
d)

1 and 2 only
2 and 3 only
1 and 3 only
All of the above

Page 13

57. Arrange these financial accountability


processes in chronological order (i.e.
from start to end) in a financial year.
1. Detailed Budget allocations by the
Ministry of Finance
2. CAG Submitting audit report to the
President
3. Public Accounts Committee
submits report to Parliament
Choose the correct order.
a)
b)
c)
d)

123
321
132
312

58. In 1937, Japan launched an attack on


China. What was the opinion of the
Indian national Congress on this
event?
a) INC welcomed the development
as a tiny Asian country was able
to defeat mighty country
b) INC passed a resolution
condemning Japan and calling
upon the Indian people to
boycott Japanese goods
c) INC remained neutral
d) None of the above

59. Some public corporations account is


audited exclusively by private
professional auditors and the CAG
does not come into the picture at all.
These organizations are
1. LIC of India
2. RBI
3. Food Corporation of India
4. State Bank of India

a)
b)
c)
d)

1 and 4 only
2, 3 and 4 only
1 and 2 only
All of the above

60. Consider the following geological


locations.
1. Coastal mountain ranges
2. Off-shore islands
3. Midst of Deep Ocean beds
Which of the above are likely for the
occurrence of Volcanoes?
a)
b)
c)
d)

1 and 2 only
2 and 3 only
1 and 3 only
All of the above

61. Consider the following about the Office


of Attorney General of India.
1. The Attorney General (AG) is
appointed by the president after
consultation with the Chief Justice
of India.
2. He must be a person who is
qualified to be appointed a judge of
the Supreme Court.
3. The Constitution does not contain
the procedure and grounds for his
removal.
4. He may be removed by the
president at any time.
Choose the correct answer using the codes
below.
a)
b)
c)
d)

1 and 4 only
2, 3 and 4 only
1 and 2 only
All of the above

Choose the correct answer using the codes


below.
http://insightsonindia.com

Page 14

62. Consider the following statements


about the roles, powers and
responsibilities of the Attorney
General of India.
1. He does not fall in the category of
government servants.
2. He enjoys all the privileges and
immunities that are available to a
Member of Parliament.
3. He is not debarred from private
legal practice.
4. He has the right to speak and to
take part in the proceedings of both
the Houses of Parliament and any
committees thereof where he is a
member.
Choose the correct answer using the codes
below.
a)
b)
c)
d)

1 and 4 only
2, 3 and 4 only
1 and 2 only
All of the above

63. Consider the following statements.


1. The Attorney General (AG) is not a
member of the Central cabinet.
2. Just like the AG, other offices
subordinate to AG like Solicitor
General and others are provided for
by the Constitution.
3. The AG looks after all the legal
matters of the Government.
Choose the correct answer using the codes
below.
a)
b)
c)
d)

1 and 2 only
2 and 3 only
1 and 3 only
All of the above

1. is appointed by the President in


consultation with the Chief Justice
of High Court
2. is the Chief Law officer of the
Government
3. may not be a citizen of India.
Choose the correct answer using the codes
below.
a)
b)
c)
d)

1 and 2 only
2 only
1 and 3 only
All of the above

65. In October 1920 the Communist Party


of India was set up at
a) Kolkatta
b) Moscow
c) Tashkent
d) Kanpur

66. Consider the following statements


1. During freedom struggle,
Jawaharlal Nehru became the
President of Indian National
Congress sessions only once
2. The Independence for India League
was organized by Bhagat Singh and
his comrades
Which of the above statements is/are
INCORRECT?
a)
b)
c)
d)

None
Both
2 Only
1 Only

64. The Advocate General in the State


http://insightsonindia.com

Page 15

67. The NITI Aayog comprises which of


the following?
1. Chief Ministers of all the States
2. Experts, specialists and
practitioners with relevant domain
knowledge as special invitees
nominated by the Prime Minister
3. Chief Secretary of all the States
4. Lieutenant Governors of all UTs
Choose the correct answer using the codes
below.
a)
b)
c)
d)

1 and 4 only
2, 3 and 4 only
1 and 2 only
1, 2 and 4 only

68. In which of the following ways Block


mountains can be formed?
1. Faulting due to compression earths
crust
2. Faulting due to tension in earths
crust
3. Bending due to compression on
Earths crust
Choose the correct answer using the codes
below.
a)
b)
c)
d)

1 and 2 only
2 and 3 only
1 and 3 only
All of the above

69. To which of the following did the Floud


Commission report relate to?
a) It recommended that sharecroppers pay one third of their crop
to jotedars instead of one half
b) It recommended for the first time
for the abolition of zamindari
system in Bengal
c) Both a and b
d) Neither a nor b
http://insightsonindia.com

70. Tectonic plateaus are formed due to


earth movements that cause uplifts.
Which of the following are tectonic
plateaus found on Earth?
1. Meseta of Central Iberia
2. Harz of Germany
3. Tibetan Plateau
4. Bolivian plateau found between two
ranges of Andes
Choose the correct answer from the codes
given below.
a)
b)
c)
d)

2 and 3 only
1 and 2 only
1, 3 and 4 only
All of the above

71. As per the Economic Survey, some


peculiar reasons have been the major
factors behind the stalling of projects
in specifics sectors. Match accordingly.
1. Electricity - Fuel/feedstock/raw
material supply problem
2. Construction and Real Estate Lack of non-environmental
clearance
3. Manufacturing - Unfavourable
market conditions
Choose the correct answer using the codes
below.
a)
b)
c)
d)

1 and 2 only
2 and 3 only
1 and 3 only
All of the above

Page 16

72. As per the Economic Survey, which of


the factors unique to India have been
responsible for high and rising NPAs
for banks in India and weak balance
sheet for corporates?
1. Indias debt is almost exclusively
financed by public sector banks.
2. The public sector is exposed to
corporate risk in the form of public
private partnerships, and lending
by the public sector banks.
3. High inflationary situation
Choose the correct answer using the codes
below.
a)
b)
c)
d)

1 and 2 only
2 and 3 only
1 and 3 only
All of the above

73. All rocks whether igneous or


sedimentary can become metamorphic
rocks under great temperature and
pressure. In this manner, which of the
conversions to metamorphic rocks
below is correct?
1. Clay to Slate
2. Coal to graphite
3. Sandstone to Quartzite
4. Shale to Schist
Choose the correct answer from the codes
given below.
a)
b)
c)
d)

2 and 3 only
1 and 2 only
1, 3 and 4 only
All of the above

74. Gamma-ray bursts (GRBs) are formed


by intense explosions in space, for
instance when a giant star explodes or
two stars collide. How are GRBs a
threat to Earth?
http://insightsonindia.com

a) a long GRB can obliterate the


planets ozone layer, exposing all
life forms to ultraviolet radiation
from the sun.
b) a long GRB can wipe out Earths
entire atmosphere
c) a long GRB can cause disruptions
in Earths magnetic and
gravitation field causing
instability on earth
d) The electromagnetic waves sent by
GRBs interfere with the climatic
phenomena in troposphere and
can cause long term climate
change

75. The Congress Socialist Party (CSP)


which emerged in 1930s, subscribed to
which of the following political
ideologies?
a) Rightist
b) Leftist
c) Conservative
d) None of the above

76. The numbers of pollinating birds and


mammals are declining worldwide as
more such species are on the verge of
extinction, says a new study by IUCN.
Which of the following are key
pollinator bird species?
1. Hummingbird
2. Honeyeaters
3. Sunbirds
4. White-eyes
Choose the correct answer using the codes
below.
a)
b)
c)
d)

1 and 4 only
2, 3 and 4 only
1 and 3 only
All of the above

Page 17

77. To remove the CAG from his office,


which of the following would be
absolutely required?
1. Resolution passed with special
majority by Lok Sabha
2. Resolution passed with special
majority by Rajya Sabha
3. Enquiry by the Supreme Court
4. Order of the President
Choose the correct answer using the codes
below.
a)
b)
c)
d)

1 and 4 only
2, 3 and 4 only
1 and 2 only
All of the above

78. High SLR reduced Banks profitability.


So the Economic Survey recommends
reducing SLR. Which of the following
are viable options for government
financing other than SLR?
1. Foreign-Currency Dominated Debt
2. Bond market
3. Priority Sector Lending
Choose the correct answer using the codes
below.
a)
b)
c)
d)

1 and 2 only
2 and 3 only
1 and 3 only
All of the above

b) how efficiently debt has been


converted to profitability in the
company
c) the ratio of debt from internal and
external sources in a sovereign
bank
d) None of the above

80.Consider the following statements


1. The major aim of the newly
launched Deen Dayal Upadhyaya
Grameen Kaushalya Yojana is
bringing inclusive growth to the
country
2. Deen Dayal Upadhyaya Grameen
Kaushalya Scheme is implemented
by the Ministry of Rural
Development
Which of the above statements is/are correct?
a)
b)
c)
d)

1 Only
2 Only
None
Both

81. Capital to risk weighted assets ratio


(CRAR) is arrived at by dividing the
capital of the bank with aggregated
risk weighted assets for
1. Credit risk
2. Market Risk
3. Operational Risk
Choose the correct answer using the codes
below.

79. A leverage ratio is any one of several


financial measurements that look at
a) how much capital comes in the
form of debt (loans) in comparison
to equities of a company

http://insightsonindia.com

a)
b)
c)
d)

1 and 2 only
2 and 3 only
1 and 3 only
All of the above

Page 18

82. Gross capital formation as a


percentage of GDP, since the 1991
reforms has been the highest from
which of the following sector?
a) Household
b) Private Sector
c) Public Sector
d) Private Corporate Sector

83. The NHRC is vested with which of the


following powers under the Protection
of Human Rights Act, 1993?
1. Suo moto inquiry
2. Power to regulate its own
procedure
3. Powers of Civil Court
4. Powers to utilise the services of any
officer or investigation agency of
the Central government or any
state government
Choose the correct answer using the codes
below.
a)
b)
c)
d)

1 and 3 only
2, 3 and 4 only
1 and 2 only
All of the above

84. Consider the following statements


1. India is yet to succeed in
successfully testing indigenous
Cryogenic Upper Stage
2. Crew Module Atmospheric Reentry Experiment (CARE) was
successfully carried by GSLV Mk III
Which of the above statements is/are correct?
a)
b)
c)
d)

1 Only
Both
None
2 Only

http://insightsonindia.com

85. Consider that the NHRC has just now


completed inquired into a case of
women trafficking in India due to
gross neglect by a public servant.
Which of the following actions can be
taken by it?
1. Imposing a punishment on the
guilty public servant
2. recommending to the concerned
government or authority the
initiation of proceedings for
prosecution
3. approaching the Supreme Court or
the high court concerned for the
necessary directions, orders or
writs
Choose the correct answer using the codes
below.
a)
b)
c)
d)

1 and 2 only
2 and 3 only
1 and 3 only
All of the above

86. The recent Economic Survey points out


that a large number of projects in India
are stalled leading to a reduction in
economic growth. Consider the
following about it.
1. More number of projects are stalled
in the private sector than public.
2. Manufacturing dominates in total
value of stalled projects even over
infrastructure.
3. The stock of stalled projects has
been piling year on year since the
last five years.
Choose the correct answer using the codes
below.
a)
b)
c)
d)

1 and 2 only
2 and 3 only
1 and 3 only
All of the above
Page 19

87. Education is an important parameter


of average skill level of labourers and
their productivity. In which of these
sectors, majority of the labour has
completed at least secondary
education?
1. Health and Social Work
2. Transportation and
Communication
3. Real Estate and Business Services
Choose the correct answer using the codes
below.
a)
b)
c)
d)

1 and 2 only
2 and 3 only
1 and 3 only
All of the above

88. With reference to Differences between


NITI and Planning Commission,
consider the following statements
1. NITI Aayog will not have the
powers to allocate funds to
ministries and state governments
unlike erstwhile Planning
Commission
2. The role of Chief Ministers has
increased in the NITI Aayog which
was minimal in Planning
Commission
3. Niti Aayog is a think-tank and does
not have the power to impose
policies, whereas Planning
Commission decided policies for
states and tied allocation of funds
with projects it approved
Which of the above statements is/are correct?
a)
b)
c)
d)

1 and 3 Only
3 and 2 Only
1 and 2 Only
All

http://insightsonindia.com

89. Consider the following statements


about the river Jhelum that has been
caused floods in J&K twice within a
short period of time.
1. It is the largest of the five rivers of
Punjab.
2. The river Jhelum rises
from Verinag Spring situated at the
foot of the Pir Panjal.
3. No dams and barrages have been
built on the river so far.
4. It flows through Srinagar and
the Wular Lake before entering
Pakistan through a deep
narrow gorge.
Choose the correct answer using the codes
below.
a)
b)
c)
d)

1 and 2 only
3 and 4 only
1, 2 and 4 only
2, 3 and 4 only

90. The Enforcement Directorate (ED)


recently has provisionally attached
assets in connection with the Israeli
diplomat attack case. Which of the
following laws does the ED enforce?
1. Prevention of Money Laundering
Act, 2002
2. Foreign Exchange Management
Act, 1999
3. The Industrial Disputes Decision
Act, 1995
4. Competition Act, 2002
Choose the correct answer using the codes
below.
a)
b)
c)
d)

1 and 2 only
3 and 4 only
1, 2 and 4 only
2, 3 and 4 only

Page 20

91. The Defence Acquisition Council has


recently approved the Maitri project
with France. Consider the following
statements about it.
1. It is a next-generation quickreaction surface-to-air
missile (QRSAM) system .
2. It is long range and can be used to
target neighbouring countries too.
3. While the project is jointly
developed, the technology for the
missile is indigenous by DRDO.
4. The project concerns all three
wings of Indian armed forces
army, navy and air force.
Choose the correct answer using the codes
below.
a)
b)
c)
d)

1 and 2 only
3 and 4 only
1 and 4 only
2, 3 and 4 only

92. China has provided details about its


proposed Silk Road initiatives, which
would impact 4.4 billion people and,
within a decade, could generate trade
above 2.5 trillion dollars. Which of the
following are concerned with this plan?
1. Maritime Silk Road
2. Silk Road Economic Belt
3. Belt and road initiative
4. Sea lines of Communication Belt
Choose the correct answer using the codes
below.
a)
b)
c)
d)

1 and 2 only
3 and 4 only
1 and 4 only
1, 2 and 3 only

http://insightsonindia.com

93. The Food Safety and Standards


Authority of India (FSSAI) has been
established under the Food Safety and
Standards Act, 2006 as a statutory
body for laying down science based
standards for articles of food. What
other domains related to food are
being regulated by FSSAI?
1. Sale and import of food
2. Licensing Registration and Health
And Sanitary Permits
3. Packaging and Labelling
4. Provide training programmes for
those involved in food businesses
Choose the correct answer using the codes
below.
a)
b)
c)
d)

1 and 2 only
3 and 4 only
1, 3 and 4 only
All of the above

94. The Congress Ministries which were


formed in many provinces, resigned in
October 1939 primarily due to which of
the following reasons?
a) Opposition by Gandhiji and Nehru
for continuation in power
b) Political crisis brought by
beginning of the World War II
c) Their five year term ended in 1939
d) The British forced resignation of
ministries

95. Excessive use of urea leads to several


adverse implications on soil, crop
quality and overall ecosystem. Which
of the following may address the
harmful effects of Urea on soil?
1. Inclusion of legumes
2. Modified Nitrogen scheduling
using leaf color chart

Page 21

3. Neem oil coated urea should be


preferred over pilled urea
Choose the correct answer using the codes
below.
a)
b)
c)
d)

1 and 2 only
2 and 3 only
1 and 3 only
All of the above

96. With reference to The Rajiv Gandhi


Scheme for Empowerment of
Adolescent Girls (RGSEAG) Sabla ,
consider the following statements:
1. It is a centrally sponsored program
of Government of India under
Ministry of Women and Child
Development
2. Bringing into mainstream out of
school adolescent girls into
formal/non formal education is one
of its objectives
Which of the above statements is/are correct?
a)
b)
c)
d)

1 Only
Both
2 Only
None

97. Some European countries, including


Germany, France, Britain, and Russia
along with Australia have recently
decided to join the China-led Asian
Infrastructure Investment Bank
(AIIB). Consider the following
statements about AIIB.
1. AIIB provides finances to
infrastructure projects in the AsiaPacific region only.
2. India is the second largest
shareholder in the Bank after
China.
http://insightsonindia.com

3. AIIB and BRICS New Development


Bank have a joint funding facility
for needy nations.
Choose the correct answer using the codes
below.
a)
b)
c)
d)

1 and 2 only
2 and 3 only
1 and 3 only
All of the above

98. Socialist ideas became more popular


during the 1930s in India due to which
of the following developments?
1. Great Depression in America
2. Overthrowing of despotic Czarist
regime in Russia
3. Defeat of Germany in First World
War
Choose the correct answer using the codes
below:
a)
b)
c)
d)

1 Only
1 and 2 Only
2 and 3 Only
All

99. Which of the following were prominent


Congressmen and at the same time
rich industrialists?
1. Jamnalal Bajaj
2. G.D. Birla
3. Ambalal Sarabhai
4. Vadilal Lallubhai Mehta
Choose the correct answer using the codes
below:
a)
b)
c)
d)

1,2 and 3
2, 3 and 4
1, 2 and 4
1 and 4

Page 22

100.
With reference to stopping of
train when a chain is pulled by a
passenger, consider the following
statements:
1. It is based on the Disc Brake
System (DBS) and this system is
microprocessor controlled for more
efficient braking system
2. Disc Brake System is an advanced
version of Air Brake system

3. Alarm Chain Pulling contributes to


4% of the punctuality loss cases on
Indian Railways.
Which of the above statements is/are correct?
a)
b)
c)
d)

3, 2 and 1
2 and 3
2 and 1
1 and 3

http://insightsonindia.com

Page 23

INSIGHTS MOCK TEST SERIES 2015: TEST 19 SOLUTIONS


WWW.INSIGHTSONINDIA.COM

1. Solution: d)

2. Solution: a)
NIKSHAY is A web based solution for monitoring of TB patients to monitor Revised
National Tuberculosis Programme (RNTCP) effectively, a web enabled and case
based monitoring application called NIKSHAY has been developed by National
Informatics Centre (NIC). This is used by health functionaries at various levels across
the country in association with Central TB Division (CTD), Ministry of Health &
Family Welfare. NIKSHAY covers various aspects of controlling TB using
technological innovations. Apart from web based technology, SMS services have been
used effectively for communication with patients and monitoring the programme on
day to day basis.

3. Solution: d)
It has been deployed in Priyadarshini Lake located on the Schirmacher Oasis which
is a major source of water for Maitri, Indias second base in the Antarctica. Key facts
of PROVe Indigenously built by NIOT under the Union Ministry of Earth Sciences.
Capable of probing the sea bed under normal temperature and capable of exploring
up to 200 meters in inhospitable and tough regions like the ice clad Antarctica.
Operated remotely by scientists on board the ship and has inbuilt thrusters allowing
it for 360-degree movement. Will help researchers to study and find out details about

INSIGHTS MOCK TEST SERIES 2015: TEST 19 SOLUTIONS


WWW.INSIGHTSONINDIA.COM

dissolved oxygen in sea bottom, salinity and the quantity of sunlight hitting the
bottom of the sea.
The results and outcomes will help researchers in understanding the biological
activities taking place inside the sea. Implications of its operationalisation in
Antartica Its breakthrough is a great leap forward in forecasting with precision the
ever elusive Monsoon, a climatic phenomenon which determines Indias economy.
In case of forecasting Monsoon, PROVe will measure parameters like ocean currents,
temperature and salinity in the Arctic. It will especially help scientist to move away
from present Mathematical models for forecasting the Monsoon which many times
vary from initial forecasts.

4. Solution; c)

5. Solution: d)

INSIGHTS MOCK TEST SERIES 2015: TEST 19 SOLUTIONS


WWW.INSIGHTSONINDIA.COM

Case of Malaysia near the Equator; and Britain in temperate latitude.

6. Solution: d)
Household savings continue to be the largest contributor to gross capital formation.
Household savings has two components- financial and physical, where the latter
typically does not lend itself easily to financial intermediation in the economy. As can

INSIGHTS MOCK TEST SERIES 2015: TEST 19 SOLUTIONS


WWW.INSIGHTSONINDIA.COM

be seen from Table 5.1, the contribution of physical assets to household savings has
stood stubbornly above 60 per cent all through the last decade.

7. Solution: d)
http://www.bbc.co.uk/schools/gcsebitesize/geography/natural_hazards/fold_moun
tains_rev1.shtml

8. Solution: c)
The Ministry of Corporate Affairs is primarily concerned with administration of the
Companies Act 2013, the Companies Act 1956, the Limited Liability Partnership Act,
2008 & other allied Acts and rules & regulations framed there-under mainly for
regulating the functioning of the corporate sector in accordance with law.
The Ministry is also responsible for administering the Competition Act, 2002 to
prevent practices having adverse effect on competition, to promote and sustain
competition in markets, to protect the interests of consumers through the
commission set up under the Act.

9. Solution: d)
The natural agents, such as, the sun, the rain, the wind, the running water and the
glacier wear away the soft rocks of the upland areas. The resistant rock masses are
left there standing on the ground above the surroundings. The sometimes from
almost level surface, cut by rivers and streams. These upland areas together form
dissected plateau.
http://en.wikipedia.org/wiki/Dissected_plateau

INSIGHTS MOCK TEST SERIES 2015: TEST 19 SOLUTIONS


WWW.INSIGHTSONINDIA.COM

10.

Solution: c)

http://en.wikipedia.org/wiki/Loess#Fertility
Loess is a geologically recent deposit of silt or material which is usually yellowish or
brown in color and consisting of tiny mineral particles brought by wind to the places
where they now lie. It is a product of past glacial activity in an area. It is a
sedimentary deposit of mineral particles which are finer than sand but coarser than
dust or clay, deposited by the wind. Loess is a type of silt which forms fertile topsoil
in some parts of the world. Loess deposits are usually a few meters thick. One of the
key characteristics of these deposits is the cat steps'. The soil has few clay particles to
hold it together. It is composed mainly of quartz crystals which slide easily against
each other, and is therefore very subject to erosion. Because of this, there are miniearth slides, which form the steps.

INSIGHTS MOCK TEST SERIES 2015: TEST 19 SOLUTIONS


WWW.INSIGHTSONINDIA.COM

The above is a Loess deposition.


A peneplain is a low-relief plain representing the final stage of fluvial erosion during
times of extended tectonic stability.

11. Solution: d)
King Mangarasa III proved himself more of a chef than a king. His treatise named
SoopaShastra on cookery of medieval Karnataka stands out to be the only
compilation in a regional language during the medieval ages. King Mangarasa III
belonged to the Chengalvu dynasty who were subordinate kings ruling under the
suzerainty of Hoysala kings in the 16th century Karnataka. Mangarasas palace once
stood in the present day Kallali in HunsurTaluk in Karnataka.
Mangarasa being a well-knownpoet, versified SoopaShastra in 385 stanzas running
into six chapters during 15-16th century. The first chapter, Pistakadhyaya describes
as many as 50 wheat preparations. The next chapter called Panakadhyaya deals with
food-drinks. Mangarasa in old Kannadameant a good king (manga=good, beautiful;
rasa=king). So there were Kings with the same name ruling the same kingdom but
they were not related to each other - Mangarasa I wrote on poison; Mangarasa II
came up with a dictionary; and Mangarasa IV became a chronicler of Kings in
Kashmir.
Only two Sanskrit books - Bhima and Nala are devoted on cookery but fall short in
being called profound works. Mangarasa has obtained help from both these books
and besides got inspired by a character of an adept chef, Gouri, Hindu God Shivas
spouse as mentioned in SkandaPurana. Two earlier Kannada literatures,
Lingapurana and Lokopakara have more than fifty stanzas devoted on cookery.

INSIGHTS MOCK TEST SERIES 2015: TEST 19 SOLUTIONS


WWW.INSIGHTSONINDIA.COM

However, unlike Soopashastra they are not entirely devoted to cookery. Soopashastra
stands apart from all other work done till then in Kannada.
PIB Features

12. Solution: d)
Constant decline in soil fertility status, mainly due to nutrient removal by
intensive cropping systems in amounts far-exceeding their replenishment through
fertilizers and manures during past few decades, is considered one of the serious
second-generation problems of Green Revolution. Farmers often use nitrogenous
fertilizers (mostly urea) or nitrogenous and complex fertilizers (mostly urea and
DAP), ignoring the application of potash and other deficient nutrients.
On the other hand, multi-nutrient deficiencies have already emerged and
expanded in most of the soils. Soil analysis under different projects revealed
widespread deficiency of at least six nutrients viz., Nitrogen (N), Phosphorus (P),
Potash (K), Sulphur (S), Zinc (Zn) and Boron (B) in different parts of the
country. Some diagnostic surveys carried out in rice-wheat growing areas of northwestern India revealed that farmers often apply greater than recommended rates of
N to sustain the yield levels that were attained earlier with even less fertilizer use.
Urea, being most common N fertilizer, is indiscriminately used irrespective of
scientific prescriptions. Excessive use of urea leads to several adverse implications
on soil, crop quality and overall ecosystem. Some major disadvantages of
excessive/indiscriminate use of urea are listed as under:
It enhances mining of soil nutrients that are not applied or applied inadequately,
thus leading to deterioration of soil fertility. Such soils may require more fertilizers
over time to produce optimum yields.
Nitrogen applied in excess of crop demand is lost through volatilization,
denitrification and leaching.
Excessive use of N (urea) encourages climate change (when lost through
denitrification) and groundwater pollution (when lost through leaching). Increase in
nitrate content of groundwater in some intensively-cropped areas has been reported,
which is obviously due to leaching of nitrates beyond crop root zone. Increase in
nitrate content of groundwater is potentially harmful, as it is used for drinking
purposes in most of the rural areas.
Fertilizer N (urea) application beyond recommended rates enhances crop
succulence, thus making the plants prone to disease and pest infestation, and to
lodging.
Unbalanced use of urea decreases N use efficiency, thus leads to increase in cost of
production and lowering of net profits.

INSIGHTS MOCK TEST SERIES 2015: TEST 19 SOLUTIONS


WWW.INSIGHTSONINDIA.COM

For increasing use efficiency of N and other nutrients, profitability and


environmental safety, fertilizer N (urea) application needs to be rationalized. A few
guidelines for rational use of N fertilizers are indicated below:
Fertilizer N (urea) application should be invariably balanced not only with P and K
but also with deficient secondary and micronutrients.
Soil test-based fertilizer prescriptions have to be adopted. Farmers should insist
for S and micronutrient testing, as NPK alone (without S and micronutrients) is no
longer balanced fertilizer prescription.
Neem oil coated urea should be preferred over pilled urea, especially for basal
dressing.
Losses of N are usually less when urea is top-dressed before irrigation.
Modified N scheduling using leaf color chart (LCC) gives better N use efficiency in
crops. LCC-based real-time N application needs to be promoted in the crops (like
rice and wheat) for which LCC thresholds are available.
Conjoint use of organic manures and fertilizers may help curtailing the application
of fertilizers including that of urea.
Inclusion of legumes may curtail fertilizer N (urea) requirement by 25-50%.
Depending on cropping system and availability of irrigation, legumes could be
introduced as catch crop, green manures, forage crop, break crop or as short
duration grain crop.

13. Solution: d)
The basic strategy is DOTS (Directly Observed treatment short course) through
which treatment completion is ensured. Diagnosis and treatment is provided free of
cost to all individuals. The program operates through 3644 Tuberculosis Units (TUs)
and 13,306 designated microscopy centres (DMCs) throughout the country.
The program has a well-defined ACSM (Advocacy, communication, and social
mobilization) strategy based on Communication needs, Target Groups and
Communication tools/Media options to reach target groups. Roles and
responsibilities are defined at the central, state and district level.
Other strategies of RNTCP include TB-HIV management in collaboration with
NACO. It is a well-known fact that TB is more common in HIV cases and is one of the
most common causes of infection and mortality. TB patients are regularly tested for
HIV and vice versa. Patient with this dual infection receive co-trimoxazole
chemoprophylaxis. Both anti-TB drugs as well as antiretroviral drug therapy are
given to patients with both infections and disease. CB-NAAT (Cartridge-based

INSIGHTS MOCK TEST SERIES 2015: TEST 19 SOLUTIONS


WWW.INSIGHTSONINDIA.COM

nucleic acid amplification) test is done in priority basis to detect TB in Presumptive


TB cases among People living with HIV / AIDS in all CB-NAAT sites.

14. Solution: a)
Also, in 1934, with the formation of the Congress Socialist Party (CSP). the process of
the consolidation of the Left forces received a significant push forward. The
Communists, too, got the opportunity, by becoming members of the CSP to work in
an open and legal fashion. This consolidation of the Left acted as a spur to the
formation of an all-India body to coordinate the kisan movement, a process that was
already under way through the efforts of N.G. Ranga and other kisan leaders. The
culmination was the establishment of the All-India Kisan Congress in Lucknow in
April 1936 which later changed its name to the AllIndia kisan Sabha. Swami
Sahajanand, the militant founder of the Bihar Provincial Kisan Sabha (1929), was
elected the President, and N.G. Ranga, the pioneer of the kisan movement in Andhra
and a renowned scholar of the agrarian problem, the General Secretary. The first
session was greeted in person by Jawaharlal Nehru. Other participants included
Ram Manohar Lohia, Sohan Singh Josh, Indulal Yagnik, Jayaprakash Narayan,
Mohanlal Gautam, Kamal Sarkar, Sudhin Pramanik and Ahmed Din. The Conference
resolved to bring out a Kisan Manifesto and a periodic bulletin edited by Indulal
Yagnik. A Kisan Manifesto was finalized at the All-India Kisan Committee session in
Bombay and formally presented to the Congress Working Committee to be
incorporated into its forthcoming manifesto for the 1937 elections. The Kisan
Manifesto considerably influenced the agrarian programme adopted by the Congress
at its Faizpur session, which included demands for fifty per cent reduction in land
revenue and rent, a moratorium on debts, the abolition of feudal levies, security of
tenure for tenants, a living wage for agricultural labourers, and the recognition of
peasant unions.

15. Solution: d)
India is worlds highest MDR-TB burden country with 64,000 cases emerging
annually in notified Pulmonary TB cases.
Although RNTCP is being implemented in every part of country either through the
State government or through the local self- government wherever there is a separate
health system in corporations, focus is given for urban TB control. Exclusive
resources for urban areas included in RNTCP in terms of TB health visitors for every

INSIGHTS MOCK TEST SERIES 2015: TEST 19 SOLUTIONS


WWW.INSIGHTSONINDIA.COM

1 lakh aggregate urban population, additional funding norms in ACSM in urban


areas.
TB now is a notifiable disease under Government notification and the Govt. of India
has banned the use of serology in the diagnosis of TB. The program is trying to bring
all private sectors through case notification, persuading the private sector to follow
standardized treatment guidelines.

16. Solution: d)

INSIGHTS MOCK TEST SERIES 2015: TEST 19 SOLUTIONS


WWW.INSIGHTSONINDIA.COM

17. Solution: a)

Hot spring, also called thermal spring, spring with water at temperatures
substantially higher than the air temperature of the surrounding region. Most hot
springs discharge groundwater that is heated by shallow intrusions of magma
(molten rock) in volcanic areas. Some thermal springs, however, are not related to
volcanic activity. In such cases, the water is heated by convective circulation:
groundwater percolating downward reaches depths of a kilometre or more where the
temperature of rocks is high because of the normal temperature gradient of the
Earths crustabout 30 C (54 F) per kilometre in the first 10 km (6 miles).

18.Solution: d)
The National Commission for SCs and STs came into being consequent upon passing
of the 65th Constitutional Amendment Act of 1990. The Commission was established
under Article 338 of the Constitution with the objective of monitoring all the
safeguards provided for the SCs and STs under the Constitution or other laws.
Geographically and culturally, the STs are different from the SCs and their problems
are also different from those of SCs. In 1999, a new Ministry of Tribal Affairs was
created to provide a sharp focus to the welfare and development of the STs. It was
felt necessary that the Ministry of Tribal Affairs should co-ordinate all activities
relating to the STs as it would not be administratively feasible for the Ministry of
Social Justice and Empowerment to perform this role
The separate National Commission for STs came into existence in 2004. It consists of
a chairperson, a vice-chairperson and three other members. They are appointed by
the President by warrant under his hand and seal. Their conditions of service and
tenure of office are also determined by the President.

19. Solution: d)
The Commission presents an annual report to the President. It can also submit a
report as and when it thinks necessary.
The President places all such reports before the Parliament, along with a
memorandum explaining the action taken on the recommendations made by the
Commission. The memorandum should also contain the reasons for the nonacceptance of any of such recommendations.
The President also forwards any report of the Commission pertaining to a state
government to the state governor. The governor places it before the state legislature,

INSIGHTS MOCK TEST SERIES 2015: TEST 19 SOLUTIONS


WWW.INSIGHTSONINDIA.COM

along with a memorandum explaining the action taken on the recommendations of


the Commission. The memorandum should also contain the reasons for the nonacceptance of any of such recommendations.

20.

Solution: c)

In November 1927, Jawaharlal and Motilal visited the Soviet Union. On his return,
Jawaharlal wrote a series of articles for the Hindu which were also published in book
form. His reaction was very positive and idealistic and was reflected in the lines he
put on the title page of the book: Bliss was it in that dawn to be alive, but to be young
was very heaven. In 1928 and after Nehru 9 repeatedly praised the Soviet Union as
the greatest opponent of imperialism, this admiration for the Soviet Union was to
deepen as he came more and more under the influence of Marxism. At Lucknow, in
1936, he said that though he was pained and disagreed with much that was
happening in the Soviet Union, he looked upon that great and fascinating unfolding
of a new order and a new civilization as the most promising feature of our dismal
age. In fact, if the future is full of hope it is largely because of Soviet Russia. The
mass trials and purges of Stalins opponents in the 1930s repelled him, but he still
retained his faith in the Soviet regime, especially as, in his view, it stood as the one
real effective bulwark against fascism in Europe and Asia.

21. Solution: d)
Starting April 1, the Rashtriya Swasthya Bima Yojana (RSBY) of the Labour and
Employment Ministry will now be implemented by the Ministry of Health and
Family Welfare.
The RSBY, the health insurance scheme for BPL (below poverty line) families
provides for IT-enabled and smart-card-based cashless health insurance, including
maternity benefit cover up to Rs. 30,000 per annum on a family floater basis.
In India, of the estimated workforce of 47 crore, only eight crore are organised
workers and thirty nine crore are workers in the unorganised sector.

22.

Solution: d)

The Commissioner is assisted at headquarters by Deputy Commissioner and an


Assistant Commissioner. He maintains liaison with the State Governments and
Union Territories through nodal officers appointed by them.

INSIGHTS MOCK TEST SERIES 2015: TEST 19 SOLUTIONS


WWW.INSIGHTSONINDIA.COM

At the Central level, the Commissioner falls under the Ministry of Minority Affairs.
Hence, he submits the annual reports or other reports to the President through the
Union Minority Affairs Minister.
The Constitution does not specify the qualifications, tenure, salaries and allowances,
service conditions and procedure for removal of the Special Officer for Linguistic
minorities.

23.

Solution: a)

AWACS are radars mounted on an aircraft to provide seamless 360-degree coverage


of the airspace.
The Defence Acquisition Council (DAC), chaired by Defence Minister, recently
approved the development of an indigenous Airborne Warning and Control Systems
(AWACS) by the Defence Research and Development Organisation (DRDO). The Air
Force is currently operating three Israeli Phalcons based on Russian Il-76 aircraft. In
addition, the DRDO is developing two smaller Airborne Early Warning & Control
(AEW&C) based on the Embraer aircraft, scheduled to be delivered this year.

24.

Solution: d)

The fund, with a corpus of Rs. 500 crore, will be used to support market
interventions for managing prices of perishable agri-horticultural
commodities.

Initially, the fund is proposed to be used for onion and potato only. Losses
incurred, if any, in the operations will be shared between the Centre and the
States.

The fund will be used to advance interest-free loans to State governments and
Central agencies to support their working capital and other expenses on
procurement and distribution interventions for such commodities. These
commodities will be procured directly from farmers or farmers organisations
right at the farm gate or mandi levels and be made available at reasonable
prices to consumers.

For this purpose, the States will set up a revolving fund to which the Centre
and State will contribute equally. The ratio of Centre-State contribution to the
State-level corpus in respect of northeast States will, however, be 75:25.

The revolving fund is being mooted so that requirements for all future
interventions can be decided and met at the State level itself. The Central

INSIGHTS MOCK TEST SERIES 2015: TEST 19 SOLUTIONS


WWW.INSIGHTSONINDIA.COM

agencies will, however, set up their revolving fund entirely with advance from
the Centre.

25.

Solution: d)

http://www.chinausfocus.com/finance-economy/one-belt-and-one-road-farreaching-initiative/

26.

Solution: d)

http://en.wikipedia.org/wiki/NITI_Aayog#Difference_between_NITI_Aayog_and_
Planning_Commission

27.

Solution: d)

The commission is a multi-member body consisting of a chairman and four


members. The chairman should be a retired chief justice of India, and members
should be serving or retired judges of the Supreme Court, a serving or retired chief
justice of a high court and two persons having knowledge or practical experience
with respect to human rights. In addition to these full-time members, the
commission also has four ex-officio membersthe chairmen of the National
Commission for Minorities, the National Commission for SCs, the National
Commission for STs and the National Commission for Women.

28.

Solution: a)

Labour productivity has always been the focus of the government as it is the major
goal of all skill development missions and schemes.

INSIGHTS MOCK TEST SERIES 2015: TEST 19 SOLUTIONS


WWW.INSIGHTSONINDIA.COM

29.

Solution: a)

The president can remove the chairman or any member from the office under the
following circumstances:

If he is adjudged an insolvent; or
If he engages, during his term of office, in any paid employment outside the
duties of his office; or
If he is unfit to continue in office by reason of infirmity of mind or body; or
If he is of unsound mind and stand so declared by a competent court; or
If he is convicted and sentenced to imprisonment for an offence.

In addition to these, the president can also remove the chairman or any member on
the ground of proved misbehaviour or incapacity. However, in these cases, the
president has to refer the matter to the Supreme Court for an inquiry. If the Supreme
Court, after the inquiry, upholds the cause of removal and advises so, then the
president can remove the chairman or a member.

30.

Solution: d)

Nehru developed a complex relationship with Gandhiji during this period. He


criticized Gandhiji for refusing to recognize the conflict of classes, for preaching

INSIGHTS MOCK TEST SERIES 2015: TEST 19 SOLUTIONS


WWW.INSIGHTSONINDIA.COM

harmony among the exploiters and the exploited, and for putting forward the
theories of trusteeship by, and conversion of, the capitalists and landlords. In fact,
Nehru devoted a whole chapter in his Autobiography to gently combating some of the
basic aspects of Gandhian ideology.

31. Solution: d)
The chairman and members are appointed by the president on the recommendations
of a six-member committee consisting of the prime minister as its head, the Speaker
of the Lok Sabha, the Deputy Chairman of the Rajya Sabha, leaders of the Opposition
in both the Houses of Parliament and the Central home minister. Further, a sitting
judge of the Supreme Court or a sitting chief justice of a high court can be appointed
only after consultation with the chief justice of India.
The chairman and members hold office for a term of five years or until they attain the
age of 70 years, whichever is earlier. After their tenure, the chairman and members
are not eligible for further employment under the Central or a state government.

32.

Solution: c)

PRAGATI is a unique integrating and interactive platform.

The platform is aimed at addressing common mans grievances, and


simultaneously monitoring and reviewing important programmes and
projects of the Government of India as well as projects flagged by State
Governments.

Unique features:

The PRAGATI platform uniquely bundles three latest technologies: Digital


data management, video-conferencing and geo-spatial technology.

It also offers a unique combination in the direction of cooperative federalism


since it brings on one stage the Secretaries of Government of India and the
Chief Secretaries of the States.

With this, the Prime Minister is able to discuss the issues with the concerned
Central and State officials with full information and latest visuals of the
ground level situation. It is also an innovative project in e-governance and
good governance.

INSIGHTS MOCK TEST SERIES 2015: TEST 19 SOLUTIONS


WWW.INSIGHTSONINDIA.COM

It is a three-tier system (PMO, Union Government Secretaries, and Chief


Secretaries of the States).

Prime Minister will hold a monthly programme where he will interact with the
Government of India Secretaries, and Chief Secretaries through Videoconferencing enabled by data and geo-informatics visuals.

Issues to be flagged before the PM are picked up from the available database
regarding Public Grievances, on-going Programmes and pending Projects.

The system will ride on, strengthen and re-engineer the data bases of the
CPGRAMS for grievances, Project Monitoring Group (PMG) and the Ministry
of Statistics and Programme Implementation. PRAGATI provides an interface
and platform for all these three aspects.

It will also take into consideration various correspondences to PMs office by


the common people or from high dignitaries of States and/or developers of
public projects.

It is also a robust system for bringing e-transparency and e-accountability


with real-time presence and exchange among the key stakeholders.

33.

Solution: b)

Venus is sometimes called Earth's twin because Venus and Earth are almost the same
size, have about the same mass (they weigh about the same), and have a very similar
composition (are made of the same material). They are also neighbouring planets.
However, Venus and Earth are also very different. Venus has an atmosphere that is
about 100 times thicker than Earth's and has surface temperatures that are
extremely hot. Venus does not have life or water oceans like Earth does. Venus also
rotates backwards compared to Earth and the other planets.

34.

Solution: b)

INSIGHTS MOCK TEST SERIES 2015: TEST 19 SOLUTIONS


WWW.INSIGHTSONINDIA.COM

35.

Solution: d)

The cost of transportation of a ton of coal, for each country, is derived by multiplying
the average distance (in kilometers) travelled by the coal with the average cost (PPP
adjusted $) of transportation per ton kilometer. The average distance over which the
coal is transported divided by the average speed yields the time taken. Load carried
by the average freight train divided by the time taken yields capacity (tons carried per
hour). As the ratios reported in table 6.4 indicates, China carries about thrice as
much coal freight per hour vis--vis India. Coal is transported in India at more than
twice the cost vis--vis China, and it takes 1.3 times longer to do so.

36.

Solution: a)

http://www.ndtv.com/india-news/what-was-section-66a-749154
http://indianexpress.com/article/opinion/columns/the-judge-my-hero/

37.

Solution: d)

http://indianexpress.com/article/opinion/columns/getting-rbi-out-of-debt/

38.

Solution: b)

The Congress could not attempt a complete overhaul of the agrarian structure by
completely eliminating the zamindari system. This, for two reasons, According to the
constitutional structure of the 1935 Act, the provincial Ministries did not have

INSIGHTS MOCK TEST SERIES 2015: TEST 19 SOLUTIONS


WWW.INSIGHTSONINDIA.COM

enough powers to do so. They also suffered from an extreme lack of financial
resources, for the lions share of Indias revenues was appropriated by the
Government of India. The Congress Ministries could also not touch the existing
administrative structure, whose sanctity was guarded by the Viceroys and
Governors powers. What is more important, the strategy of class adjustment also
forbade it. A multi-class movement could develop only by balancing or adjusting
various, mutually clashing class interests. To unite all the Indian people in their
struggle against colonialism, the main enemy of the time, it was necessary to make
such an adjustment. The policy had to be that of winning over or at least neutralizing
as large a part of the landlord classes as possible so as to isolate the enemy and
deprive him of all social support within India. This was even more necessary because,
in large parts of the country, the smaller landlords were active participants in the
national movement

39.

Solution: c)

The vast majority of the system's mass is in the Sun, with most of the remaining
mass contained in Jupiter. The four smaller inner
planets, Mercury, Venus, Earth and Mars, also called the terrestrial planets, are
primarily composed of rock and metal. The four outer planets, the giant planets, are
substantially more massive than the terrestrials. The two largest, the gas
giants Jupiter and Saturn, are composed mainly of hydrogen and helium; the two
outermost planets, the ice giants Uranus and Neptune, are composed largely of
substances with relatively high melting points compared with hydrogen and helium,
called ices, such as water, ammonia and methane. All planets have almost circular
orbits that lie within a nearly flat disc called the ecliptic.

40.

Solution: a)

http://theweatherprediction.com/habyhints2/471/

41. Solution: c)
The Sagarmala project seeks to develop a string of ports around Indias coast. The
prime objective of the Sagarmala project is to promote port-led direct and indirect
development and to provide infrastructure to transport goods to and from ports
quickly, efficiently and cost-effectively. Therefore, the Sagarmala Project shall, inter
alia, aim to develop access to new development regions with intermodal solutions
and promotion of the optimum modal split, enhanced connectivity with main

INSIGHTS MOCK TEST SERIES 2015: TEST 19 SOLUTIONS


WWW.INSIGHTSONINDIA.COM

economic centres and beyond through expansion of rail, inland water, coastal and
road services.
The Sagarmala initiative will address challenges by focusing on three pillars of
development, namely

Supporting and enabling Port-led Development through appropriate policy


and institutional interventions and providing for an institutional framework
for ensuring inter-agency and ministries/departments/states collaboration
for integrated development,

Port Infrastructure Enhancement, including modernization and setting up of


new ports, and

Efficient Evacuation to and from hinterland.

In addition to strengthening port and evacuation infrastructure, it also aims at


simplifying procedures used at ports for cargo movement and promotes usage
of electronic channels for information exchange leading to quick, efficient,
hassle-free and seamless cargo movement.

The Sagarmala initiative would also strive to ensure sustainable development


of the population living in the Coastal Economic Zone (CEZ). This would be
done by synergising and coordinating with State Governments and line
Ministries of Central Government through their existing schemes and
programmes such as those related to community and rural development,
tribal development and employment generation, fisheries, skill development,
tourism promotion etc.

42.

Solution: c)

It is a scheme which seeks to make available quality medicines at affordable prices


for all, especially the poor and the disadvantaged.

Under this, less priced quality unbranded generic medicines will be made
available through Jan Aushadhi stores which inherently are less priced but are
of same and equivalent quality, efficacy and safety as compared to branded
generic medicines.

Under this Scheme, the State Government has to provide space in


Government Hospital premises for the running of the outlets (JAS).
Government hospitals, NGOs, Charitable Organisations and public societies
like Red Cross Society, Rogi Kalyan Samiti typically constituted for the
purpose can be operating agencies for the JAS.

INSIGHTS MOCK TEST SERIES 2015: TEST 19 SOLUTIONS


WWW.INSIGHTSONINDIA.COM

The operating agency for JAS is nominated on the basis of the


recommendations of the State government. Operational expenditure is met
from trade margins admissible for the medicines.

The State Government has to ensure prescription of unbranded generic


medicines by the Government doctors.

The Jan Aushadhi Programme is accordingly a self sustaining business model


not dependent on government subsidies or assistance. It is run on the
principle of Not for Profits but with Minimal Profits.

43.

Solution: c)

Both are statutory bodies i.e. established by an act of Parliament. Both were
established after amending the Competition Act 2002. COMPAT is a quasi judicial
body (a tribunal) whereas, CCI is not. CCI is a regulatory body.
http://www.thehindubusinessline.com/companies/competition-appellate-tribunalupholds-ccis-rs-630cr-fine-on-dlf/article6025288.ece

44.

Solution: d)

http://www.investopedia.com/articles/bonds/05/022805.asp

45.

Solution: c)

This is the flagship scheme for skill training of youth to be implemented by the
Ministry of Skill Development and Entrepreneurship through the National Skill
Development Corporation (NSDC). The scheme will cover 24 lakh persons.

Skill training would be done based on the National Skill Qualification


Framework (NSQF) and industry led standards.

Under the scheme, a monetary reward is given to trainees on assessment and


certification by third party assessment bodies. The average monetary reward
would be around Rs.8000 per trainee.

The scheme would be implemented through NSDC training partners.


Currently NSDC has 187 training partners that have over 2300 centres. In
addition, Central / State Government affiliated training providers would also
be used for training under the scheme.

INSIGHTS MOCK TEST SERIES 2015: TEST 19 SOLUTIONS


WWW.INSIGHTSONINDIA.COM

Focus under the PMKVY would be on improved curricula, better pedagogy


and better trained instructors. Training would include soft skills, personal
grooming, behavioral change for cleanliness, good work ethics. Sector Skill
Councils and the State Governments would closely monitor skill training that
will happen under PMKVY.

Skill Development Management System (SDMS) would be put in place to


verify and record details of all training centres a certain quality of training
locations and courses.

Biometric system and video recording of the training process would be put in
place where feasible.

A robust grievance redressal system would be put in place to address


grievances relating to implementation of the scheme. An online citizen portal
would be put in place to disseminate information about the scheme.

46.

Solution: a)

Mission Indradhanush launched on 25th December, 2014 with an aim to cover all
those children who are partially vaccinated or unvaccinated. Mission Indradhanush
is a nationwide initiative with a special focus on 201 high focus districts. These
districts account for nearly 50% of the total partially vaccinated or unvaccinated
children in the country.
Mission Indradhanush will provide protection against seven life-threatening
diseases (Diphtheria, Whooping Cough, Tetanus, Polio, Tuberculosis, Measles and
Hepatitis B). In addition, vaccination against Japanese Encephalitis and
Haemophilus influenza type B will be provided in selected districts of the country.
Vaccination against tetanus will be provided to the pregnant women.

47.

Solution: a)

INSIGHTS MOCK TEST SERIES 2015: TEST 19 SOLUTIONS


WWW.INSIGHTSONINDIA.COM

48.

Solution: b)

Indulal Kanaiyalal Yagnik (22 February 1892 17 July 1972) was an Indian
independence activist, a leader of the All India Kisan Sabha and one who lead
Mahagujarat Movement, which spearheaded the demand for the separate statehood
for Gujarat on 8 August 1956. He is also known as Indu Chacha. He was writer and
film maker also. Yagnik was elected to the 2nd Lok Sabha from Ahmedabad
constituency in the erstwhile Bombay state in 1957. He was re-elected to the 3rd, 4th
and 5th Lok Sabha from the same constituency from 19621972.
Left-wing individuals, such as Swami Sahajanand Saraswati, Professor N.G. Ranga,
and Indulal Yagnik, who worked outside the framework of any organized left-wing
party.

49.

Solution: d)

Rajasthans first State-run human milk bank, Jeevan Dhara, was inaugurated by
the Health Minister recently.

The first mothers milk bank in the State was started by a non-governmental
organisation in a government hospital in Udaipur.

Jeevan Dhara has been started in collaboration with the Norwegian government and
JK Lon Hospital, and aims to collect excess breast milk from lactating mothers and
provide it to infants deprived of it.

50.

Solution: d)

INSIGHTS MOCK TEST SERIES 2015: TEST 19 SOLUTIONS


WWW.INSIGHTSONINDIA.COM

Financial repression on the asset side of the balance sheet is created by the statutory
liquidity ratio (SLR) requirement that forces banks to hold government securities,
and priority sector lending (PSL) that forces resource deployment in less than fully
efficient ways. Financial repression on the liability side has arisen from high inflation
since 2007, leading to negative real interest rates, and a sharp reduction in
households financial savings.
As India exits from liability-side repression with declining inflation, the time may be
appropriate for addressing its asset-side counterparts.

51. Solution: b)

INSIGHTS MOCK TEST SERIES 2015: TEST 19 SOLUTIONS


WWW.INSIGHTSONINDIA.COM

52.

Solution: c)

In 1921, the Congress congratulated the Burmese people on their struggle for
freedom. Burma was at that time a part of India, but the Congress announced that
free India favoured Burmas independence from India. Gandhiji wrote in this context
in 1922: 1 have never been able to take pride in the fact that Burma has been made
part of British India. It never was and never should be. The Burmese have a
civilization of their own. In 1924, the Congress asked the Indian settlers in Burma to
demand no separate rights at the cost of the Burmese people.

53.

Solution: a)

Igneous rocks are formed deep inside Earth's crust (intrusive rocks) or at the surface
(extrusive rocks). As molten rock (known as magma) below the surface cools, the
liquid material begins to solidify into minerals of differing sizes and compositions.
Generally, the longer the cooling time, the larger the mineral crystals can grow.
Trapped deep in the Earth, magma is allowed to cool slowly. Examples of intrusive
rocks are granite and gabbro. Granite is made up of three common minerals - quartz,
mica and feldspar - and when viewed from a distance appear tan, pinkish or gray,
depending on the concentrations and grain sizes of the three minerals.
This rock is very widely used as a building material, due to its abundance and
strength. Gabbro has a lower quartz content than granite, and is therefore much
darker. Liquid rock that reaches the surface cools very quickly. As a result of this
immediate cooling, the crystals produced are extremely small. These tiny-grained
minerals tend to give these extrusive rocks a rather opaque, homogenous color.
All of the world's sea floors are produced in this manner of rapid cooling, and the
dark-gray, heavy rock produced is called basalt. Andesite (named after the Andes
Mountains in South America) is a cousin to basalt in that it is also volcanic in nature,
but is lighter-colored due to the decreased concentration of darker minerals.

54.

Solution: a)

The scheme was launched by the Ministry of Human Resources Development,


Government of India.

INSIGHTS MOCK TEST SERIES 2015: TEST 19 SOLUTIONS


WWW.INSIGHTSONINDIA.COM

The nutritional guidelines for minimum amount of food and calorie content per child
per day are:
Entitlement norm per child per day under MDM

Item

Primary (Class I to V) Upper Primary(Class VI to VIII)

Calories

450

700

Protein (in Grams )

12

20

Rice / Wheat (in Grams ) 100

150

Dal (in Grams )

20

30

Vegetables (in Grams )

50

75

Oil and Fat (in Grams )

7.5

In the case of micronutrients (vitamin A, iron, and folate) tablets and de-worming
medicines, the student is entitled to receive the amount provided for in the school
health programme of the National Rural Health Mission
http://mdm.nic.in/

55.

Solution: a)

http://www.downtoearth.org.in/content/how-long-will-life-earth-survive

56.

Solution: a)

INSIGHTS MOCK TEST SERIES 2015: TEST 19 SOLUTIONS


WWW.INSIGHTSONINDIA.COM

The Constitution (Article 149) authorises the Parliament to prescribe the duties and
powers of the CAG in relation to the accounts of the Union and of the states and of
any other authority or body.
Accordingly, the Parliament enacted the CAGs (Duties, Powers and Conditions of
Service) act, 1971. This Act was amended in 1976 to separate accounts from audit in
the Central government. The duties and functions of the CAG as laid down by the
Parliament and the Constitution are:
1. He audits the accounts related to all expenditure from the Consolidated Fund of
India, consolidated fund of each state and consolidated fund of each union territory
having a Legislative Assembly.
2. He audits all expenditure from the Contingency Fund of India and the Public
Account of India as well as the contingency fund of each state and the public account
of each state.

57.Solution: a)
In a financial year, CAGs report are submitted at the end of financial year. Then the
PAC receives CAGs reports, studies it; and send its own report to the Parliament.
Therefore, the first activity in a financial year is the examination of the budget by the
departmental standing committees after it has been presented and budget allocated
by the Ministry of Finance.

58.

Solution: b)

In 1937, Japan launched an attack on China. The Congress passed a resolution


condemning Japan and calling upon the Indian people to boycott Japanese goods as
a mark of their sympathy with the Chinese people. At its Haripura session in early
1938, the Congress reiterated this call while condemning the aggression of a brutal
imperialism in China and horrors and frightfulness that have accompanied it. It
warned that the invasion of China was fraught with the gravest consequences for the
future of the world peace and of freedom in Asia. As an expression of its solidarity
with the Chinese people, 12 June was celebrated throughout India as China Day. The
Congress also sent a medical mission, headed by Dr. M. Atal, to work with the
Chinese armed forces. One of its members, Dr. Kotnis, was to lay down his life
working with the Eighth Route Army under Mao ZeDongs command.

59.

Solution: d)

INSIGHTS MOCK TEST SERIES 2015: TEST 19 SOLUTIONS


WWW.INSIGHTSONINDIA.COM

The role of CAG in the auditing of public corporations is limited. Broadly speaking,
his relationship with the public corporations falls into the following three categories:
(i) Some corporations are audited totally and directly by the CAG, for example,
Damodar Valley Corporation, Oil and Natural Gas Commission, Air India, Indian
Airlines Corporation, and others.
(ii) Some other corporations are audited by private professional auditors who are
appointed by the Central Government in consultation with the CAG. If necessary, the
CAG can conduct supplementary audit. The examples are, Central Warehousing
Corporation, Industrial Finance Corporation, and others.
(iii) Some other corporations are totally subjected to private audit. In other words,
their audit is done exclusively by private professional auditors and the CAG does not
come into the picture at all. They submit their annual reports and accounts directly
to the Parliament. Examples of such corporations are Life Insurance Corporation of
India, Reserve Bank of India, State Bank of India, Food Corporation of India, and
others.

60.

Solution: d)

http://www.preservearticles.com/2011111717346/useful-notes-on-the-distributionof-volcanoes-throughout-the-world.html

61. Solution: b)
The Attorney General (AG) is appointed by the president. He must be a person who
is qualified to be appointed a judge of the Supreme Court. In other words, he must be
a citizen of India and he must have been a judge of some high court for five years or
an advocate of some high court for ten years or an eminent jurist, in the opinion of
the president.
The term of office of the AG is not fixed by the Constitution. Further, the
Constitution does not contain the procedure and grounds for his removal. He holds
office during the pleasure of the president. This means that he may be removed by
the president at any time. He may also quit his office by submitting his resignation to
the president. Conventionally, he resigns when the government (council of ministers)
resigns or is replaced, as he is appointed on its advice.
The remuneration of the AG is not fixed by the Constitution. He receives such
remuneration as the president may determine.

INSIGHTS MOCK TEST SERIES 2015: TEST 19 SOLUTIONS


WWW.INSIGHTSONINDIA.COM

62.

Solution: d)

In the performance of his official duties, the Attorney General has the right of
audience in all courts in the territory of India. Further, he has the right to speak and
to take part in the proceedings of both the Houses of Parliament or their joint sitting
and any committee of the Parliament of which he may be named a member, but
without a right to vote. He enjoys all the privileges and immunities that are available
to a member of Parliament.

Following limitations are placed on the Attorney General in order to avoid any
complication and conflict of duty:
o He should not advise or hold a brief against the Government of India.
o He should not advise or hold a brief in cases in which he is called upon
to advise or appear for the Government of India.
o He should not defend accused persons in criminal prosecutions without
the permission of the Government of India.
o He should not accept appointment as a director in any company or
corporation without the permission of the Government of India.

However, the Attorney General is not a full-time counsel for the Government. He
does not fall in the category of government servants. Further, he is not debarred from
private legal practice.

63.

Solution: c)

In addition to the AG, there are other law officers of the Government of India. They
are the solicitor general of India and additional solicitor general of India. They assist
the AG in the fulfilment of his official responsibilities. It should be noted here that
only the office of the AG is created by the Constitution. In other words, Article 76
does not mention about the solicitor general and additional solicitor general.
The AG is not a member of the Central cabinet. There is a separate law minister in
the Central cabinet to look after legal matters at the government level.

64.

Solution: b)

The advocate general is appointed by the governor. He must be a person who is


qualified to be appointed a judge of a high court. In other words, he must be a citizen
of India and must have held a judicial office for ten years or been an advocate of a
high court for ten years.

INSIGHTS MOCK TEST SERIES 2015: TEST 19 SOLUTIONS


WWW.INSIGHTSONINDIA.COM

As the chief law officer of the government in the state, the duties of the advocate
general include the following:

To give advice to the government of the state upon such legal matters which
are referred to him by the governor.
To perform such other duties of a legal character that are assigned to him by
the governor.
To discharge the functions conferred on him by the Constitution or any other
law.

In the performance of his official duties, the advocate general is entitled to appear
before any court of law within the state.

65.

Solution: c)

Attracted by the Soviet Union and its revolutionary commitment, a large number of
Indian revolutionaries and exiles abroad made their way there. The most well-known
and the tallest of them was M.N. Roy, who along with Lenin, helped evolve the
Communist Internationals policy towards the colonies. Seven such Indians, headed
by Roy, met at Tashkent in October 1920 and set up a Communist Party of India.
Independently of this effort, as we have seen, a number of left-wing and communist
groups and organizations had begun to come into existence in India after 1920. Most
of these groups came together at Kanpur in December 1925 and founded an all-India
organization under the name the Communist Party of India (CPI).

66.

Solution: b)

Nehru became the president of the historic Lahore Congress of 1929 at a youthful
forty. He was elected to the post again in 1936 and 1937
In 1928, Jawaharlal joined hands with Subhas to organize the Independence for
India League to fight for complete independence and a socialist revision of the
economic structure of society.

67.

Solution: d)

http://en.wikipedia.org/wiki/NITI_Aayog#Origin_and_formation

68.

Solution: a)

INSIGHTS MOCK TEST SERIES 2015: TEST 19 SOLUTIONS


WWW.INSIGHTSONINDIA.COM

http://en.wikipedia.org/wiki/Fault_block#Fault-block_mountains
http://www.mountainprofessor.com/fault-block-mountains.html

69.

Solution: c)

in British India, it was the tebhaga struggle in Bengal that held the limelight. in late
1946, the share-croppers of Bengal began to assert that they would no longer pay a
half share of their crop to the jotedars but only one-third and that before division the
crop would be stored in their khamars (godowns) and not that of the jotedars. They
were no doubt encouraged by the fact that the Bengal Land Revenue Commission,
popularly known as the Floud Commission, had already made this recommendation
in its report to the government.

INSIGHTS MOCK TEST SERIES 2015: TEST 19 SOLUTIONS


WWW.INSIGHTSONINDIA.COM

Source:
https://books.google.co.in/books?id=4jhNfZ4DtbEC&pg=PA26&lpg=PA26&dq=the
+Floud+Commission&source=bl&ots=jdPzXv91nz&sig=oTNYBjzjADFH1Wrl1rxn8jp

INSIGHTS MOCK TEST SERIES 2015: TEST 19 SOLUTIONS


WWW.INSIGHTSONINDIA.COM

Xlwg&hl=en&sa=X&ei=Vk8bVfMHIKKuwS_sYHAAQ&ved=0CCMQ6AEwAQ#v=onepage&q&f=false

70.

Solution: d)

One other such plateau is Deccan plateau, India. Some plateaus like these are found
between fold mountains such as the Bolivian plateau. These are distinct from the
volcanic plateaus that are formed due to the flow and consolidation of lava.

71. Solution: d)
It is clear that private projects are held up overwhelmingly due to market conditions
and non-regulatory factors whereas the government projects are stalled due to lack of
required clearances.

72.

Solution: d)

Many countries before, including Japan in the aftermath of the real estate and equity
boom of the late 1980s, have experienced over-extended corporate balance sheets.
However, there is something fundamentally Indian about this phenomenon.
First, the debt overhang of the corporate sector is accompanied by a relatively high
growth of around 6 to 7 per cent. Second, it has been accompanied by high inflation
(instead of the price deflation in the Japanese example). Third, the public sector is

INSIGHTS MOCK TEST SERIES 2015: TEST 19 SOLUTIONS


WWW.INSIGHTSONINDIA.COM

exposed to corporate risk in the form of public private partnerships, and lending by
the public sector banks. Fourth, unlike many other countries with high debt to equity
ratios currently, Indias debt is almost exclusively financed by public sector banks.
This has translated into high and rising non-performing assets for these banks.

73.

Solution: d)

http://en.wikipedia.org/wiki/Metamorphic_rock#Types_of_metamorphism

74.

Solution: a)

http://www.downtoearth.org.in/content/how-long-will-life-earth-survive

75.Solution: b)
A powerful left-wing group developed in India in the late 1920s and 1930s
contributing to the radicalization of the national movement. The goal of political
independence acquired a clearer and sharper social and economic content. The
stream of national struggle for independence and the stream of the struggle for social
and economic emancipation of the suppressed and the exploited began to come
together. Socialist ideas acquired roots in the Indian soil; and socialism became the
accepted creed of Indian youth whose urges came to be symbolized by Jawaharlal
Nehru and Subhas Chandra Bose. Gradually there emerged two powerful parties of
the Left, the Communist Party of India (CPI) and the Congress Socialist Party (CSP)

76.

Solution: d)

http://www.downtoearth.org.in/content/pollinating-species-declining-worldwide

77. Solution: d)
The CAG holds office for a period of six years or upto the age of 65 years, whichever
is earlier. He can resign any time from his office by addressing the resignation letter

INSIGHTS MOCK TEST SERIES 2015: TEST 19 SOLUTIONS


WWW.INSIGHTSONINDIA.COM

to the president. He can also be removed by the president on same grounds and in
the same manner as a judge of the Supreme Court.
In other words, he can be removed by the president on the basis of a resolution
passed to that effect by both the Houses of Parliament with special majority, either
on the ground of proved misbehaviour or incapacity.

78.

Solution: a)

Refer to the Sec Box 5.1 : Reducing the Statutory Liquidity Ratio in the recent
Economic Survey.
It has been explained very well in details there.

79.

Solution: a)

http://www.investopedia.com/terms/l/leverageratio.asp
Using the Leverage ratio is a tool for measuring the performance of the Indian
banking system.

80.

Solution: d)

According to Census 2011, India has 55 million potential workers between the ages
of 15 and 35 years in rural areas. At the same time, the world is expected to face a
shortage of 57 million workers by 2020. This presents a historic opportunity for
India to transform its demographic surplus into a demographic dividend. The
Ministry of Rural Development implements DDU-GKY to drive this national agenda
for inclusive growth, by developing skills and productive capacity of the rural youth
from poor families.
There are several challenges preventing Indias rural poor from competing in the
modern market, such as the lack of formal education and marketable skills. DDUGKY bridges this gap by funding training projects benchmarked to global standards,
with an emphasis on placement, retention, career progression and foreign
placement.
Features of Deen Dayal Upadhyaya Grameen Kaushalya Yojana

Enable Poor and Marginalized to Access Benefits


Demand led skill training at no cost to the rural poor

Inclusive Program Design


Mandatory coverage of socially disadvantaged groups (SC/ST 50%; Minority
15%; Women 33%)

INSIGHTS MOCK TEST SERIES 2015: TEST 19 SOLUTIONS


WWW.INSIGHTSONINDIA.COM

Shifting Emphasis from Training to Career Progression


Pioneers in providing incentives for job retention, career progression and foreign
placements

Greater Support for Placed Candidates


Post-placement support, migration support and alumni network

Proactive Approach to Build Placement Partnerships


Guaranteed Placement for at least 75% trained candidates

Enhancing the Capacity of Implementation Partners


Nurturing new training service providers and developing their skills

Regional Focus
Greater emphasis on projects for poor rural youth in Jammu and Kashmir
(HIMAYAT),
the North-East region and 27 Left-Wing Extremist (LWE) districts (ROSHINI)

Standards-led Delivery
All program activities are subject to Standard Operating Procedures that are
not open to interpretation by local inspectors. All inspections are supported by
geo-tagged, time stamped videos/photographs

81.Solution: d)
Capital to risk weighted assets ratio (CRAR) is arrived at by dividing the capital of
the bank with aggregated risk weighted assets for credit risk, market risk and
operational risk. Leverage ratio is defined by the RBI as ratio of total assets to total
capital. The international definition, for example as laid out by the Bank of
International Settlements, is typically the inverse.

INSIGHTS MOCK TEST SERIES 2015: TEST 19 SOLUTIONS


WWW.INSIGHTSONINDIA.COM

82.

Solution: b)

83.

Solution: d)

The commissions headquarters is at Delhi and it can also establish offices at other
places in India. It is vested with the power to regulate its own procedure. It has all
the powers of a civil court and its proceedings have a judicial character. It may call
for information or report from the Central and state governments or any other
authority subordinate thereto.
The commission has its own nucleus of investigating staff for investigation into
complaints of human rights violations. Besides, it is empowered to utilise the services
of any officer or investigation agency of the Central government or any state
government for the purpose. It has also established effective cooperation with the
NGOs with first-hand information about human rights violations.
NHRC can inquire into any violation of human rights or negligence in the prevention
of such violation by a public servant, either suo motu or on a petition presented to it
or on an order of a court.

84.

Solution: b)

With the successful Mars Orbit Insertion of Mars Orbiter Spacecraft on September
24, 2014, ISRO became the fourth Space Agency to successfully send a spacecraft to
Mars and the first Nation in the world to do so in its first attempt. The successful
launch of first experimental flight of Indias future heavy capacity launcher GSLV Mk
III, also known as LVM-3, on December 18, 2014, which carried Crew Module

INSIGHTS MOCK TEST SERIES 2015: TEST 19 SOLUTIONS


WWW.INSIGHTSONINDIA.COM

Atmospheric Re-entry Experiment (CARE) as its payload, marked Indias progress


towards self-reliance in launching four ton class of communication satellites into
Geosynchronous Transfer Orbit. Polar Satellite Launch Vehicle (PSLV-C26)
successfully launched Indias third navigational satellite IRNSS-1C, from Sriharikota
on October 16, 2014. French Remote Sensing Satellite SPOT-7 and four more copassenger satellites from Canada, Germany and Singapore were successfully
launched using PSLV-C23. Indias new communication satellite GSAT-16 was
successfully put into orbit on December 07, 2014 from French, Guiana. The year
2014 also saw the successful flight testing of GSLVs indigenous Cryogenic Upper
Stage, with the successful launch of GSLV-D5 placing the GSAT-14 communication
satellite into its intended orbit. Several major space missions are planned during
2015 which include next flight of Geo Synchronous Launch Vehicle GSLV-D6 with
Indigenous Cryogenic Stage to launch GSAT-6 communication satellite, launch of
Astronomical Satellite ASTROSAT, GSAT-15 communication satellite, four more
navigational satellites and two commercial launches of PSLV.

85.

Solution: b)

The commission is not empowered to inquire into any matter after the expiry of one
year from the date on which the act constituting violation of human rights is alleged
to have been committed. In other words, it can look into a matter within one year of
its occurrence4.
The commission may take any of the following steps during or upon the completion
of an inquiry:

it may recommend to the concerned government or authority to make


payment of compensation or damages to the victim;
it may recommend to the concerned government or authority the initiation of
proceedings for prosecution or any other action against the guilty public
servant;
it may recommend to the concerned government or authority for the grant of
immediate interim relief to the victim;
it may approach the Supreme Court or the high court concerned for the
necessary directions, orders or writs.

86.

Solution: a)

INSIGHTS MOCK TEST SERIES 2015: TEST 19 SOLUTIONS


WWW.INSIGHTSONINDIA.COM

The stalling rate of projects has been increasing at an alarmingly high rate in the last
five years, and the rate is much higher in the private sector.
The data shows that manufacturing and infrastructure dominate in the private
sector, and manufacturing dominates in total value of stalled projects even over
infrastructure. The governments stalled projects are predominantly in
infrastructure. Unfavourable market conditions (and not regulatory clearances) are
stalling a large number of projects in the private sector, and in contrast regulatory
reasons explain bulk of stalling in the public sector. Also, clearing the top 100 stalled
projects will address 83 per cent of the problem of stalled projects by value.

87.

Solution: c)

In order for a sector to offer transformational possibilities, it must not only be


characterised by high levels and growth rates of productivity, it must also absorb
resources from the rest of the economy. But in order to do so, the sectors use of
inputs must be aligned with the countrys comparative advantage.

INSIGHTS MOCK TEST SERIES 2015: TEST 19 SOLUTIONS


WWW.INSIGHTSONINDIA.COM

88.

Solution: d)

While Planning Commission enjoyed the powers to allocate funds to ministries and
state governments, NITI Aayog will be an advisory body, or a think-tank. Under
Planning Commission, States' role was limited to the National Development Council
and annual interaction during Plan meetings and the commission reported to
National Development Council that had state chief ministers and lieutenant
governors of UTs. But Niti Aayogs Governing Council has state chief ministers and
lieutenant governors as the all powerful body. Under Niti Aayog states are consulted
while making policy and deciding on funds allocation. Final policy would be a result
of thatconsultations unlike under Planning Commission when policy was formed by
the commission and states were then consulted about allocation of funds. While Niti
Aayog is a think-tank and does not have the power to impose policies, Planning
Commission decided policies for states and tied allocation of funds with projects it
approved, a methodology driven by "one size fits all" concept.

89.

Solution: c)

http://en.wikipedia.org/wiki/Jhelum_River#Course
http://en.wikipedia.org/wiki/Jhelum_River#Verinag_in_Kashmir

90.

Solution: a)

The Directorate General of Economic Enforcement is a law enforcement


agencyand economic intelligence agency responsible for enforcing economic laws
and fighting economic crime in India. It is part of the Department of
Revenue, Ministry of Finance. It comprises officers of the Indian administrative
services Indian Revenue Service, Indian police services. It was established on the 1st
day of June, 2000 by the Central Govt. of India to investigate provisions of
the Foreign Exchange Management Act, 1999.
For the trial of an offence punishable under section 4 of PMLA, the Central
Government (in consultation with the Chief Justice of the High Court), designates
one or more Sessions Court as Special Court(s). The court is also called "PMLA
Court". Any appeal against any order passed by PMLA court can directly be filed in
the High Court for that jurisdiction.
http://www.thehindu.com/todays-paper/tp-national/ed-attaches-assets-indiplomat-attack-case/article7050654.ece

INSIGHTS MOCK TEST SERIES 2015: TEST 19 SOLUTIONS


WWW.INSIGHTSONINDIA.COM

91. Solution: c)
http://www.thehindu.com/todays-paper/tp-national/india-france-to-codevelopnaval-missile/article7050655.ece
http://en.wikipedia.org/wiki/Maitri_%28missile%29

92.

Solution: d)

http://www.thehindu.com/todays-paper/tp-international/china-releases-details-ofsilk-road-plans/article7050633.ece
The initiative focuses on bringing together China, Central Asia, Russia and Europe
(the Baltic); linking China with the Persian Gulf and the Mediterranean Sea through
Central Asia and West Asia; and connecting China with Southeast Asia, South Asia
and the Indian Ocean.

93.

Solution: d)

http://www.fssai.gov.in/AboutFSSAI/introduction.aspx
http://www.thehindu.com/todays-paper/tp-opinion/safe-food-from-the-farm-tothe-plate/article7050630.ece

94.

Solution: b)

INSIGHTS MOCK TEST SERIES 2015: TEST 19 SOLUTIONS


WWW.INSIGHTSONINDIA.COM

The Congress Ministries resigned in October 1939 because of the political crisis
brought about by World War 11. But Gandhiji welcomed the resignations for another
reason they would help cleanse the Congress of the rampant corruption. He wrote
to C. Rajagopalachari on 23 October 1939: 1 am quite clear in my mind that what has
happened is best for the cause. It is a bitter pill I know. But it was needed. It will
drive away all the parasites from the body. We have been obliged to do wrong things
which we shall be able to avoid. The resignations produced another positive effect.
They brought the Left and the Right in the Congress closer because of a common
policy on the question of participation in the war.

95.

Solution: d)

Here are the steps

Fertilizer N (urea) application should be invariably balanced not only with P


and K but also with deficient secondary and micronutrients.
Soil test-based fertilizer prescriptions have to be adopted. Farmers should
insist for S and micronutrient testing, as NPK alone (without S and
micronutrients) is no longer balanced fertilizer prescription.
Neem oil coated urea should be preferred over pilled urea, especially for basal
dressing.
Losses of N are usually less when urea is top-dressed before irrigation.
Modified N scheduling using leaf color chart (LCC) gives better N use
efficiency in crops. LCC-based real-time N application needs to be promoted
in the crops (like rice and wheat) for which LCC thresholds are available.
Conjoint use of organic manures and fertilizers may help curtailing the
application of fertilizers including that of urea.
Inclusion of legumes may curtail fertilizer N (urea) requirement by 25-50%.
Depending on cropping system and availability of irrigation, legumes could be
introduced as catch crop, green manures, forage crop, break crop or as short
duration grain crop.

96.

Solution: b)

The Rajiv Gandhi Scheme for Empowerment of Adolescent Girls (RGSEAG) Sabla is
a centrally sponsored program of Government of India initiated on April 1, 2011
under Ministry of Women and Child Development.
The objectives of the program are:

INSIGHTS MOCK TEST SERIES 2015: TEST 19 SOLUTIONS


WWW.INSIGHTSONINDIA.COM

Enable the Adolescent girls for self-development and empowerment


Improve their nutrition and health status.
Promote awareness about health, hygiene, nutrition, adolescent reproductive and
sexual health (ARSH) and family and child care.
Upgrade home-based skills, life skills and integrate with the National Skill
Development Program (NSDP) for vocational skills.
Mainstream out of school adolescent girls into formal/non formal education.
Provide information/guidance about existing public services such as PHC, CHC,
Post Office, Bank, Police Station, etc.

97.

Solution: a)

The Asian Infrastructure Investment Bank (AIIB) is an international financial


institution proposed by China. The purpose of the multilateral development bank is
to provide finance to infrastructure projects in the Asia-Pacific region.

The AIIB was launched in Beijing last year to spur investment in Asia in
transportation, energy, telecommunications and other infrastructure.

AIIB is regarded by some as a rival for the IMF, the World Bank and the Asian
Development Bank (ADB), which the AIIB says are dominated by developed
countries like the United States and Japan.

Presently there are 22 members including China.

The authorised capital of AIIB will be $100 billion. AIIBs headquarters is to


be located in Beijing.

India is the second largest shareholder in the Bank after China.

It is being called as a very positive development in the sense that it opens up


more borrowing opportunities.

98.

Solution: a)

INSIGHTS MOCK TEST SERIES 2015: TEST 19 SOLUTIONS


WWW.INSIGHTSONINDIA.COM

Events (2) and (3) took place in late 1910s while the question is specific about 1930s.

99.

Solution: d)

Among the various groups that participated in the national movement were several
individual capitalists who joined the Congress. They fully identified with the
movement, went to jails and accepted the hardships that were the lot of
Congressmen in the colonial period. The names of Jamnalal Bajaj, Vadilal Lallubhai
Mehta, Samuel Aaron, Lala Shankar Lal, and others are well known in this regard.
There were other individual capitalists who did not join the Congress but readily gave
financial and other help to the movement. People like G.D. Birla, Ambalal Sarabhai
and Waichand Hirachand, fall into this category. There were also a large number of
smaller traders and merchants who at various points came out in active support of
the national movement. On the other hand, there were several individual capitalists
or sections of the class who either remained neutral towards the Congress and the
national movement or even actively opposed it.

100.

Solution: a)

How trains come to a stop?


We always wonder how trains run and how they are stopped. It is easy to
understand how a car comes to a stop when the driver applies the brake using the
brake pedal. Cars use hydraulic brakes whereas most of the trains in India use
pneumatic brakes or air brakes as they are commonly called. These coaches are
generally marked with an A or AB with the coach number. The trains can be stopped
by the driver or by the passengers by alarm chain pulling (ACP).
In the Air Brake system, compressed air is used for operating the brake system.
These brakes can be the conventional types or directly mounted on the bogies of the
coaches. The latest design is the Disc Brake System (DBS) found in LHB coaches and
is similar to what is found in automobiles. The atmospheric air is compressed in the
locomotive upto 10 kg/cm2 and supplied at a pressure of 6/5 kg/cm2 to the train. It
is carried across the whole length of the train through two pipes at the bottom of the
coaches. When the brakes are applied the pressure in the 5kg pipe (the brake pipe)
gets depleted proportionately and this causes the air to enter the brake cylinders
which push a piston. The brake blocks are connected through a linkage and the
piston movement causes them to stick to the wheels and stop the train by friction.

INSIGHTS MOCK TEST SERIES 2015: TEST 19 SOLUTIONS


WWW.INSIGHTSONINDIA.COM

The loss in pressure is made good by the air in the 6kg/cm2 pipe called the feed pipe.
In case of disc brake system the pads attached to the discs are used to stop the train.

DBS is micro processor controlled and an advanced version of Air Brake system. It
is essential for high speeds of 160kmph plus. It is superior in terms of reduced
braking distance, higher wheel life due to reduced frequency of wheel turning, and
reduced maintenance, less braking noise and higher efficiency due to simple brake
rigging. The main characteristics of DBS are 02 discs mounted on each axle as shown
in picture below:

Disc Brake Systm

Emergency Brake Handle

When the passenger pulls the alarm chain shown above located near the seat it
causes the pressure in the brake pipe to fall down giving an indication to the driver
and stopping the train. A valve on the coach called the PEASD (passenger
emergency alarm signal device) operates and the sound of the escaping air can be
heard from the coach. Also a light near the door glows and indicates the location of
the affected coach.
What does an unscheduled stoppage of a train by ACP mean to the public?
Have you ever wondered why trains are often delayed? Analysis shows that ACP
contributes to 4% of the punctuality loss cases on Indian Railways. Imagine the social
costs of the time delays to about 1500 passengers in a train when one pulls the chain
for a valid or often illegitimate reason. Apart from this time loss, calculations show
that for an extra stoppage of a 24 coach train travelling at 110 Kmph, the locomotive

INSIGHTS MOCK TEST SERIES 2015: TEST 19 SOLUTIONS


WWW.INSIGHTSONINDIA.COM

consumes an additional amount of approximately 130litres of diesel fuel. This


amounts to a loss of 8500 rupees as well as the increase in avoidable atmospheric
pollution. As responsible citizens we should abstain from using ACP and should
sensitize others not to do the same. The penalty for unauthorized use of the alarm
chain is Rs.1000 and/or imprisonment up to three months.
Apart from this we need to realize that the train requires about one kilometer to
stop due to its sheer momentum. It is for this reason that in spite of the driver being
vigilant there are accidents on unmanned level crossings when trespassers break the
rules and expect the train to stop immediately on seeing them.
As responsible citizens and enlightened green passengers we should refrain from
using the ACP unless there is a dire emergency. It may be worthwhile to contact the
railway staff on the train to solve the problem at hand.

INSIGHTS ON INDIA MOCK PRELIMINARY EXAM - 2015


INSIGHTS ON INDIA MOCK TEST - 20
GENERAL STUDIES

PAPER-I
Time Allowed: 1.5 Hours

Maximum Marks: 150

INSTRUCTIONS
1. IMMEDITELY AFTER THE COMMENCEMENT OF THE EXAMINATION, YOU SHOULD
CHECK THAT THIS TEST BOOKLET DOES NOT HAVE ANY UNPRINTED OR TORN OR
MISSING PAGES OR ITEMS, ETC. IF SO, GET IT REPLACED BY A COMPLETE TEST BOOKLET.
2. You have to enter your Roll Number on the Test I
Booklet in the Box provided alongside. DO NOT
write anything else on the Test Booklet.
4. This Test Booklet contains 75 items (questions). Each item is printed only in English. Each item comprises
four responses (answers). You will select the response which you want to mark on the Answer Sheet. In
case you feel that there is more than one correct response, mark the response which you consider the best.
In any case, choose ONLY ONE response for each item.
5. You have to mark all your responses ONLY on the separate Answer Sheet provided. See directions in the
Answer Sheet.
6. All items carry equal marks.
7. Before you proceed to mark in the Answer Sheet the response to various items in the Test Booklet, you
have to fill in some particulars in the Answer Sheet as per instructions sent to you with your Admission
Certificate.
8. After you have completed filling in all your responses on the Answer Sheet and the examination has
concluded, you should hand over to the Invigilator only the Answer Sheet. You are permitted to take away
with you the Test Booklet.
9. Sheets for rough work are appended in the Test Booklet at the end.
10. Penalty for wrong answers :
THERE WILL BE PENALTY FOR WRONG ANSWERS MARKED BY A CANDIDATE IN THE
OBJECTIVE TYPE QUESTION PAPERS.
(i)

There are four alternatives for the answer to every question. For each question for which a
wrong answer has been given by the candidate, one-third of the marks assigned to that question
will be deducted as penalty.

(ii) If a candidate gives more than one answer, it will be treated as a wrong answer even if one of the
given answers happens to be correct and there will be same penalty as above to that question.
(iii)

If a question is left blank, i.e., no answer is given by the candidate, there will be no penalty for
that question.
http://insightsonindia.com

INSIGHTS ON INDIA MOCK TEST SERIES FOR CIVIL SERVICES PRELIMINARY EXAM 2015

http://insightsonindia.com

Page 1

1. Consider the following statements.


Assertion (A): The Central Information
Commission is not a constitutional body.
Reason (R): It has been established under the
provisions of the Right to Information Act
(2005).
In the context of the statements above, which
of these is true?
a) A and R both are true, and R is the
correct explanation for A.
b) A and R both are true, and R is the
NOT the correct explanation for A.
c) A is correct, R is incorrect.
d) A and R both are incorrect.

2. Consider the following statements


about the Central Information
Commission (CIC).
1. The CIC consists of a Chairperson
and two other members.
2. MPs and MLAs are eligible to
become members of the CIC.
3. The Chairperson should be an
eminent justice in the opinion of
the President or eligible to be a
Supreme Court Judge.
Choose the correct answer from the codes
below.
a)
b)
c)
d)

1 and 2 only
2 and 3 only.
3 only
None of the above.

3. The Central Information Commission


is a/an
a) high-powered independent body
which inter alia looks into the
complaints made to it and decide
the appeals
http://insightsonindia.com

b) body that comes under Ministry of


Information & Broadcasting, which
inter alia looks into the complaints
forwarded to it by the Ministry and
decide the appeals
c) independent body that refers to
complaints only submitted to it by
the Department of Public
Grievances
d) quasi-judicial body that is
supervised by one of the special
benches of the Supreme Court, and
works independently of the
Government of India

4. The Chief Information Commissioner


is appointed by a selection committee
consisting of
1. Union Home Minister
2. Union Law Ministry
3. Chief Justice of India
4. Lok Sabha Speaker
5. Leader of Opposition, Rajya Sabha
Choose the correct answer using the codes
below.
a)
b)
c)
d)

1, 4 and 5 only
All of the above
None of the above
2, 3 and 4 only

5. Which of the following are NOT


eligible for reappointment to the
office?
1. Comptroller and Auditor General of
India
2. Chief Information Commissioner
3. Chairman, UPSC
4. Chief Election Commissioner
Choose the correct answer using the codes
below.
Page 2

a)
b)
c)
d)

1, 3 and 4 only
All of the above
1 and 2 only
2, 3 and 4 only

6. The Chief State Information


Commissioner can be removed by
a) the State Governor after an enquiry
by the Supreme Court upholds the
cause of removal
b) the State Governor after an enquiry
by the High Court upholds the
cause of removal
c) the President after an enquiry by
the Supreme Court upholds the
cause of removal
d) the President after an enquiry by a
Parliamentary Committee upholds
the cause of removal

7. Consider the following statements


about the Constituent assembly of
India.
1. Its constitution was finally subject
to the approval of the British
Parliament.
2. The selection of representatives of
various Indian states was flexible
and decided by consultation.
3. Members of the CA from Provinces
were elected directly from the
people by a very limited franchise.
Which of the above is/are true?
a)
b)
c)
d)

1 and 2 only
2 and 3 only
1 and 3 only
2 only

http://insightsonindia.com

8. Consider the following statements


about the appointment of CVC.
1. He is appointed by the President.
2. The recommendation of the
nominee of the post comes from the
Council of Ministers.
3. Appointment of CVC cannot be
challenged in court.
Choose the correct answer using the codes
below.
a)
b)
c)
d)

1 and 2 only
2 and 3 only
1 and 3 only
1 only

9. Consider the following procedures.


1. President referring the matter to
the Lok Sabha.
2. Inquiry by the Supreme Court.
3. Parliament passes the motion by a
simple majority in both the houses.
Which of the following procedures are
followed in the removal of CVC. Choose the
order that is chronological from the ones
given below.
a)
b)
c)
d)

123
213
2 only
13 only

10. Which of the following


ministries/departments/organs of the
Government of India (GoI) have a
bearing on the Central Bureau of
Investigation (CBI)?
1. Ministry of Law and Justice
2. Ministry of Home Affairs
3. CVC
4. UPSC
5. Department of Personnel and
Training
Page 3

Choose the correct answer using the codes


below.
a)
b)
c)
d)

1, 2 and 3 only
All of the above
None of the above
1, 3 and 5 only

11. Which of the following


powers/responsibilities are exercised
by CVC?
1. Part of the Selection Committee in
the appointment of the Director,
CBI
2. Instituting inquiries against
Union and State Ministers
3. Superintendence of Police in
matters of corruption
Choose the correct answer using the codes
below.
a)
b)
c)
d)

1 and 2 only
2 and 3 only
None of the above
1 only

12. Which of the following domains are


handled by the CBI for investigation
and reporting?
1. cyber and high technology crime
2. transnational organised crime
3. Economic crimes
Choose the correct answer using the codes
below.
a)
b)
c)
d)

1 and 2 only
2 and 3 only
All of the above
3 only

http://insightsonindia.com

13. Which of the following is/are NOT


National Investment and
Manufacturing Zones (NIMZs) that
have been approved in principle by the
Government of India (GoI)?
1. Kolar in Karnataka
2. Prakasam in Andhra Pradesh
3. Nagpur in Maharashtra
4. Chittor in Andhra Pradesh
Choose the correct matches using the codes
below.
a)
b)
c)
d)

1 and 3 only
2 and 4 only
4 only
All are NMIZs.

14. In order to boost the MSME sector,


several schemes are operational. Some
of the major initiatives taken for the
development of this sector are
1. Technology Centre Systems
Programme
2. India Inclusive Innovation Fund
3. Credit Linked Capital Subsidy
4. Credit Guarantee Scheme
5. MSE-Cluster Development
Programme
Choose the correct matches using the codes
below.
a)
b)
c)
d)

1, 3 and 5 only
2 and 3 only
2, 4 and 5 only
All of the above

15. According to the Financial Sector


Legislative Reforms Commission
(FSLRC), what are the core functions
of the Reserve Bank of India (RBI)?
1. Controlling inflation
2. Public debt management
3. Banking supervision
Page 4

Choose the correct answer using the codes


below.
a)
b)
c)
d)

1 and 2 only
2 and 3 only
1 and 3 only
All of the above

16. Who/Which among the following


comes under the jurisdiction of the
Lokpal?
1. Prime Minister
2. All categories of public servants.
3. Entities receiving donation from
foreign sources
Choose the correct answer using the codes
below.
a)
b)
c)
d)

1 and 2 only
2 and 3 only
All of the above
3 only

17. RESIDEX launched by the National


Housing Board of India measures the
a) movement of prices in the
residential housing segment.
b) growth rate of the construction of
residences in the residential
housing segment.
c) growth rate of the real state sector
in general
d) movement of prices in the real state
sector in general.

18. The Government of India Act, 1919


popularly known as Montague
Chelmsford Act was significant in
which of the following ways?
1. Established a responsible
Parliamentary system of
Government in the country
http://insightsonindia.com

2. The final decision on all important


questions was no more in the
hands of the Viceroy
3. It relaxed previously stringent
control of centre on provinces.
Which of the above is/are true?
a)
b)
c)
d)

1 and 2 only
2 and 3 only
1 and 3 only
3 only

19. The demand for a constituent


assembly was made officially by Indian
National Congress for the first time at
a) Second Round table conference
b) After the departure of Simon
Commission
c) the time of Gandhi-Irwin Pact
d) None of the above

20. Consider the following about the


power of inquiry of the Central
Information Commission. The CIC
1. can exercise suo-moto inquiry
powers
2. has powers similar to a civil court
during inquiry
3. may examine any record which is
under the control of the public
authority
Choose the correct answer using the codes
below.
a)
b)
c)
d)

1 and 2 only
2 and 3 only
1 and 3 only
All of the above

Page 5

21. The 97th Constitutional Amendment


Act of 2011 gave a constitutional status
and protection to co-operative
societies. In this context, which of the
following changes were made in the
constitution?
1. Right to form co-operative
societies became a fundamental
right
2. It included a new Directive
Principle of State Policy on
promotion of co-operative
societies
3. It established a Central
constitutional authority for
overseeing matters related to
cooperatives.
Choose the correct answer using the codes
below.
a)
b)
c)
d)

1 and 2 only
2 and 3 only
All of the above
3 only

22. The Constitution of India contains


explicit and specific provisions for
which of following language related
aspects?
1. Directive for development of the
Hindi language
2. Facilities for instruction in
mother-tongue at primary stage
for Children
3. Language to be used in
representation for redress of
grievances

23. Consider the following about the


composition of the Lokpal at the
Central Level.
1. 50 per cent of the members shall
be judicial members.
2. Seats are reserved for minority
and women in Lokpal members.
3. One member will be an eminent
jurist nominated by the President.
Choose the correct answer using the codes
below.
a)
b)
c)
d)

1 and 2 only
2 and 3 only
All of the above
3 only

24. Consider the following statements.


Assertion (A): Despite Agriculture being a
State Subject, the Centre can legislate on it
even in ordinary circumstances.
Reason (R): Agriculture is a means of
livelihood to a majority of population in
India. The Parliament legislates in national
interest.
In the context of the statements above, which
of these is true?
a) A and R both are true, and R is the
correct explanation for A.
b) A and R both are true, and R is the
NOT the correct explanation for A.
c) A is incorrect, R is correct.
d) A and R both are incorrect.

Choose the correct answer using the codes


below.
a)
b)
c)
d)

1 and 2 only
2 and 3 only
All of the above
1 and 2 only

http://insightsonindia.com

Page 6

25. Consider the following statements.


1. Wind gaps in the course of a river
do not allow the construction of
upland roads and railways.
2. Deltas are less preferred sites than
estuaries for the siting of large
ports.
Which of the above is/are true?
a)
b)
c)
d)

1 only
2 only
Both 1 and 2
None

26. Indias economic growth rate has


reduced over the last few years. Which
of the following sectors have been
unaffected by this reduction in growth?
1. Construction
2. Electricity
3. Mining
4. Manufacturing
Choose the correct matches using the codes
below.
a)
b)
c)
d)

1 and 3 only
2 and 4 only
4 only
All have been affected.

27. Offshore companies will now be


taxable in India, if key management
decisions are being taken from India.
This move by the Government was
necessitated because
a) Shell companies were being used in
business transactions to evade
taxes.
b) Such offshore companies extract
Indias oil resources; hence the tax.
c) Offshore companies were being
used for organized crime.
http://insightsonindia.com

d) there was a need to increase


investment in the Oil sector by
squeezing extra resources

28. Consider the following statements


about the recently undocked
submarine Kalvari.
1. It has been built indigenously.
2. It has stealth capability.
3. It cannot operate in any other
environment than tropics.
4. It can be used for intelligence
gathering purposes.
Choose the correct answer using the codes
below.
a)
b)
c)
d)

All of the above


1, 2 and 4 only
2 and 4 only
1 and 3 only

29. Consider the following about the


composition, powers and
appointments related to the Central
Administrative Tribunal (CAT).
1. The Chairman and members are
appointed by a collegiums
consisting of senior Supreme court
judges headed by the Chief Justice
of India.
2. The members are drawn from both
judicial and administrative stream.
3. CAT orders and decrees are final,
and no provision of appeal lies
whatsoever.
Choose the correct answer using the codes
below.
a)
b)
c)
d)

1 and 2 only
2 and 3 only
2 only
1 and 3 only
Page 7

30. DigiLocker can be used to securely


store e-documents as well as store
Uniform Resource Identifier (URI) link
of e-documents issued by various
issuer departments of the Government.
For a user, which of the things are
indispensable to register for
Digilocker?
1. Mobile number
2. Aadhaar number
3. E-mail ID
4. Bank Account
5. Residence proof
Choose the correct answer using the codes
below.
a)
b)
c)
d)

1; and either 2 or 3
2, 3 and 4 only
2; and either 1 or 3
2, 4 and 5 only

31. 4538 Vishyanand is a minor planet


named after Indias first chess
grandmaster Vishwanathan Anand.
Consider the following statements
about minor planets.
1. A large comet can be designated as
a minor planet.
2. It also includes dwarf planets that
orbit around the Sun.
3. International Astronomical Union
(IAU) is also responsible for
naming the minor planets.
Choose the correct answer using the codes
below.
a)
b)
c)
d)

1 and 2 only
2 and 3 only
All of the above.
1 and 3 only

http://insightsonindia.com

32. In recently released 2015 FM Global


Resilience Index, India has been
ranked 119 out of 130 countries. Which
of the following factors are taken into
account in the index?
1. Quality of fire risk management
2. Quality of natural hazard
management
3. Corruption control
4. GDP
5. Political risk
Choose the correct answer using the codes
below.
a)
b)
c)
d)

1, 2 and 3 only
2, 3 and 4 only
1, 4 and 5 only
All of the above

33. Union Government recently has


unveiled new Foreign Trade Policy
(FTP) to make India an exporting
powerhouse in the next five years.
Which of the following areas have been
addressed in the FTP?
1. E-commerce
2. Skill India programme
3. Paperless governance
Choose the correct answer using the codes
below.
a)
b)
c)
d)

1 and 2 only
2 and 3 only
All of the above.
1 and 3 only

34. The provisions of the 97th amendment


cover which of the following aspects
related to cooperatives?
1. Regulation
2. Winding-up
Page 8

3. Appointment of Board of
Directors
4. Auditing
Choose the correct answer using the codes
below.
a)
b)
c)
d)

1 and 2 only
2 and 3 only
All of the above
1 and 4 only

35. In deciding the official language of a


State, the choice of the state is
a) Limited to the languages
mentioned in the Eighth Schedule
of the Constitution
b) Limited to the languages officially
recognized by the State
Government
c) Either an aboriginal language or a
classical language or
English/Hindi
d) Not limited; any language can be
choose.

36. Which of the following is the criterion


for a language to be officially
recognized in a State of India?
a) If the Governor is satisfied that a
certain language should be
recognized; and declares so
officially
b) If the State Legislature passes a
resolution to that effect
c) When the President is satisfied
that a substantial population of
the state speaks that particular
language; and declares so
officially
d) None of the above

http://insightsonindia.com

37. Consider the following statements


about the fishery sector of India.
1. India ranks second in world fish
production.
2. More fishing volume comes from
the inland sector rather than the
marine sector.
3. Fisheries contribute more than
50% of the agricultural GDP.
Choose the correct matches using the codes
below.
a)
b)
c)
d)

1 and 2 only
2 and 3 only
1 and 3 only
All of the above

38. If the Model Agricultural Produce


Marketing Committee (APMC) Act is
implemented successfully all across
India, it will result in
1. Closure of government sponsored
mandis
2. barrier-free and tax-free national
market for agri commodities
3. freedom of trading in futures
market of agricultural commodities
Choose the correct matches using the codes
below.
a)
b)
c)
d)

1 and 2 only
2 and 3 only
3 only
None of the above

39. Consider the following statements


about food procurement in India.
1. Coarse grains are procured by state
governments and their agencies.
2. FCI is responsible both for openended procurement and storage of
foodgrains.
Page 9

3. Decentralized Procurement Scheme


(DCP) has been adopted by all state
governments.
Choose the correct matches using the codes
below.
a)
b)
c)
d)

1 and 2 only
2 and 3 only
3 only
All of the above

40. Leeves are formed in a flood plain


when the river
a) flows normally in a bed that rises
by the accumulation of deposits
b) flows rapidly in a bed that has been
eroded by large granular deposits
c) falls near a river cliff and
accumulates the deposit thereof
d) is just nearing the Delta plains

41. Deltas are formed when a river nears


the sea and starts depositing its
sediment load in the plains. Which of
the following can be the consequences
of delta formation?
1. Change of river course
2. Formation of several distributaries
of the original river
3. Fertility of the local soil increases.
Choose the correct answer using the codes
below.
a)
b)
c)
d)

1 and 2 only
2 and 3 only
1 and 3 only
All of the above

42. Which of the following conditions are


favourable for the formation of Deltas?
1. Coast should be sheltered,
preferably tideless.
http://insightsonindia.com

2. The sea adjoining the delta should


be shallow.
3. There should be no large lakes in
the river course.
Choose the correct answer using the codes
below.
a)
b)
c)
d)

1 and 2 only
2 and 3 only
1 and 3 only
All of the above

43. To ensure last-mile connectivity,


extension services need to be geared to
address emerging technological and
knowledge needs. Under this, the Sub
Mission on Agricultural Extension
(SMAE) under the National Mission on
Agricultural Extension and Technology
(NMAET) was revamped. Which of the
following has been subsumed under it?
1. District-level Agriculture
Technology Management Agencies
(ATMAs)
2. Mass Media and Kisan Call Centre
schemes
3. Central-sector Establishment of
Agri-Clinics and Agri-Business
Centres (ACABC) Scheme
4. SMS portal for farmers
Choose the correct matches using the codes
below.
a)
b)
c)
d)

All of the above


2 and 3 only
2, 3 and 4 only
1 and 4 only

44. Horticulture sector comprising a wide


array of crops. India is the largest
producer of which of the following?
1. Mango
2. Banana
Page 10

3.
4.
5.
6.

Coconut
Pomegranate
Papaya
Cashew

Choose the correct answer using the codes


below.
a)
b)
c)
d)

2 and 3 only
1, 4, 5 and 6 only
1, 2, 3 and 5 only
All of the above

45. Farmer Producer Organizations


(FPOs) have been identified as one of
the key strategies for achieving
inclusive agricultural growth during
the Twelfth Plan. In this regards,
consider the following statements with
reference to the role of The Small
Farmers Agribusiness Consortium
(SFAC).
1. It is the central procurement
agency for undertaking price
support operations under MSP for
pulses and oilseeds.
2. The SFAC is the nodal agency of
FPOs and will operate only through
FPOs at the farm gate.
Which of the above is correct?
a)
b)
c)
d)

1 only
2 only
Both 1 and 2
None

46. Consider the following statements.


1. Vertical erosion is dominant in the
upper courses of the river.
2. Incised meanders are features of
river rejuvenation.
3. Mass movement of earth is mainly
due to the lubricating action of
rain-water and gravitational forces.
http://insightsonindia.com

Choose the correct matches using the codes


below.
a)
b)
c)
d)

1 and 2 only
2 and 3 only
1 and 3 only
All of the above

47. Piedmont glaciers are formed when


1. Glaciers slowly erode the base soil
and reach the feet of the mountain
2. steep valley glaciers spill into
relatively flat plains
3. glaciers spread out in bulb-like
lobes
Choose the correct matches using the codes
below.
a)
b)
c)
d)

1 and 2 only
2 and 3 only
1 and 3 only
All of the above

48. Consider the following statements


about Central Services and All-India
Services.
1. Any disciplinary action (imposition
of penalties) against these officers
can only be taken by the Central
government.
2. The personnel of Central services
(even though are posted in states)
work under the exclusive
jurisdiction of the Central
government.
Which of the above is/are true?
a)
b)
c)
d)

1 only
2 only
Both 1 and 2
None

Page 11

49. Article 323 A empowers the


Parliament to provide for the
establishment of administrative
tribunals for the adjudication of
disputes relating to recruitment and
conditions of service of persons
appointed to public services of the
1. Other Public Authorities
2. State
3. Local Bodies
4. Public Corporations
Choose the correct answer using the codes
below.
a)
b)
c)
d)

All of the above


2 only
2, 3 and 4 only
3 and 4 only

50. In the globe, the possibility of finding a


desert is highest in the regions
1. which are within 15-30 degrees of
North-south latitude from the
equator
2. which lie on the western sides of
the continents where trade winds
are offshore
3. that near cold currents

2. By rift valleys due to faulting


3. By glaciations
4. By volcanic activity
Choose the correct answer using the codes
below.
a)
b)
c)
d)

All of the above


None of the above
1 and 3 only
2 and 4 only

52. Recently RBI seems to be concerned


with the Sacrifice ratio in deciding
monetary policy matters. Sacrifice
ratio is
a) cumulative output losses that an
economy must endure to reduce
average inflation, on a permanent
basis, by one percentage point
b) the multiplier effect of the
liquidity injected by the Central
bank in the economy divided for
every one percentage of deposit
ratio sacrificed by banks
c) the amount of fiscal stimulus
flexibility foregone by the
government for each percentage
point hike in the monetary policy
indicators
d) None of the above

Choose the correct matches using the codes


below.
a)
b)
c)
d)

1 and 2 only
2 and 3 only
1 and 3 only
All of the above

51. Lakes are among the most varied


features of the earths surface. In
which of the following ways can lakes
be formed?
1. By tectonic action
http://insightsonindia.com

53. The National Green Tribunal (NGT)


has recently ordered banning of diesel
vehicles older than 10 years in the
capital city of India. Why are old cars
with Diesel engines a problem for the
environment?
1. They emit more CO2 than normal
petrol cars.
2. Diesel cars emit several times more
NO2 than petrol cars.
3. They emit more hydrocarbons than
petrol engines.
Page 12

Choose the correct matches using the codes


below.
a)
b)
c)
d)

1 and 2 only
2 and 3 only
1 and 3 only
2 only

54. If a river is passing through an area


where rocks are composed of
homogenous beds of uniform
resistance to corrosion, then
1. The tributaries will join the main
valley obliquely as insequent
streams
2. The evolving drainage pattern will
be tree-like in appearance.
3. It will not form a catchment area.
Choose the correct answer using the codes
below.
a)
b)
c)
d)

1 and 2 only
2 and 3 only
1 and 3 only
All of the above

55. Consider the following features in a


drainage system.
1. Interlocking spurs
2. Meanders
3. River cliffs
4. Rapids and waterfalls
Which of the above will be found in the
middle course of the river system?
a)
b)
c)
d)

1 and 2 only
2 and 3 only
3 and 4 only
1, 2 and 3 only

1. Oder Germany & Poland


2. Mekong Laos & Thailand
3. Yalu North Korea & China
Choose the correct matches using the codes
below.
a)
b)
c)
d)

1 and 2 only
2 and 3 only
1 and 3 only
All of the above

57. India meets the production needs of


which of the following fertilizers
domestically (i.e. majority share need
not be imported)?
1. Urea
2. Potassic fertilizer
3. Phosphoric fertilizer
Choose the correct matches using the codes
below.
a)
b)
c)
d)

1 and 2 only
2 and 3 only
1 and 3 only
1 only

58. Ultra-processed junk foods are known


to be a health hazard as they are high
in
1. Trans fats
2. Salt
3. Hydrocarbons
4. Fats
Choose the correct answer using the codes
below.
a)
b)
c)
d)

1, 2 and 4 only
1 only
1 and 3 only
2 and 4 only

56. Consider the following matches of river


with the countries that they separate.
http://insightsonindia.com

Page 13

59. The government has recently launched


National Air Quality Index (NAQI).
Which of the following air pollutants
will be reported by the NAQI?
1. Ozone
2. Particulate matter
3. Hydrocarbons
4. CO2
Choose the correct answer using the codes
below.
a)
b)
c)
d)

2 and 3 only
4 only
1 and 2 only
1 and 4 only

60. India is NOT one the worlds top


producers of
1. Rice
2. Wheat
3. Milk
4. Fruits
5. Vegetables
Choose the correct answer using the codes
below.
a) All of the above
b) India is one of the worlds top
producers of all of these.
c) 2 and 4 only
d) 1, 3 and 5 only

61. In the functioning of the Central


Administrative Tribunal (CAT), it is
guided by and follows the
a) Civil Procedure Code of 1908
b) Principles of natural justice
c) Criminal Procedure Code
d) Indian Penal Code

62. Under Article 323 B, the Parliament


and the state legislatures are
http://insightsonindia.com

authorised to provide for the


establishment of tribunals for the
adjudication of disputes relating to
which of the following matters?
1. Food stuffs
2. Rent and tenancy rights
3. Industrial labour
4. Elections to State legislatures
Choose the correct answer using the codes
below.
a)
b)
c)
d)

All of the above


2 only
2, 3 and 4 only
3 and 4 only

63. Consider the following statements.


Statement (S): Under the Constitution, the
civil servants are conferred personal
immunity from legal liability for official
contracts.
Implication (I): The government (Central
or state) is liable for the contracts made by
Civil servants.
In the context of the statements above,
choose the correct option from the following.
a) Both S and I are correct, and I
follows correctly from S.
b) Both S and I are correct, but I does
not follow correctly from S.
c) S is correct, but I is incorrect.
d) Both S and I are incorrect.

64. With regards to the distribution and


procurement of food, which of the
following provisions have NOT been
provided in the National Food Security
Act, 2013?
1. grievance redressal mechanisms at
district and state levels
Page 14

2. In case of non-supply of entitled


foodgrains or meals, the
beneficiaries will receive a food
security allowance
3. doorstep delivery of foodgrains to
Targeted PDS outlets
Choose the correct matches using the codes
below.
a)
b)
c)
d)

1 and 2 only
2 and 3 only
3 only
All have been provided

65. As per the Public Procurement Policy


for Micro & Small Enterprises (MSEs)
1. Every central
ministry/department/public
sector-undertaking shall set a
minimum annual procurement
(purchase) goal from MSEs.
2. There is provision for mandatory
government purchase from MSEs
owned by SC and ST entrepreneurs.
Which of the above is/are true?
a)
b)
c)
d)

1 only
2 only
Both 1 and 2
None

b) 3 and 4 only
c) None of the above
d) All of the above
67. In a glacial landscape, arrange the
following in the order from which they
occur from the foot of the mountain
range?
1. Moraines
2. Crevasses
3. Arete
4. Corrie
Choose the correct order from the codes
below.
a)
b)
c)
d)

1234
4321
4213
1342

68. Consider the following statements


about the movement of glaciers.
1. If more snow and ice are added
than are lost through melting,
calving, or evaporation, glaciers
will advance.
2. Glaciers move by
internal deformation.
3. The middle of a glacier moves
faster than its sides.
Choose the correct matches using the codes
below.

66. In recent years, which of the following


sectors have attracted some of the least
Foreign Direct Investment (FDI) in
India?
1. Automobile industry
2. Metallurgical Industries
3. Drugs and pharmaceuticals
4. Power
Choose the correct answer using the codes
below.
a) 1 and 2 only
http://insightsonindia.com

a)
b)
c)
d)

1 and 2 only
2 and 3 only
1 and 3 only
All of the above

69. In India, the share of services sector in


the overall GDP of the state is the
lowest and highest respectively in
which of the following states?
a) Arunachal Pradesh & Chandigarh
Page 15

b) Sikkim & Delhi


c) Chattisgarh & Mizoram
d) Sikkim & Goa

70. Consider the following statements


about the Service Sector in India &
World.
1. Indias share in world services
exports has been increasing faster
than its share in world merchandise
exports.
2. Services Sectors share in the World
GDP is more than 50%.
3. Unlike major European and
Western countries, the shares of
both Service income (per capita)
and service sector employment is
low in both India and China.
Which of the above is/are true?
a)
b)
c)
d)

1 and 2 only
2 and 3 only
1 and 3 only
All of the above

71. Consider the following statements


about Central Vigilance Commission
(CVC).
1. Originally the CVC was neither a
constitutional body nor a statutory
body.
2. It is free of functional control from
any executive authority.
Which of the above is/are correct?
a)
b)
c)
d)

1 only
2 only
Both 1 and 2
None

frame her constitution, Cripps mission


was sent in 1942 to India. Which of the
following were a part of the agreement
offered by the mission?
1. The constitution of India was to be
framed by an elected constituent
assembly of people.
2. Any province has the rights not to
accept the constitution.
3. British government had rights to
enter into agreements with specific
Indian states.
Which of the above is/are true?
a)
b)
c)
d)

1 and 2 only
2 and 3 only
1 and 3 only
All of the above

73. Consider the following statements


about the impact and occurrence of Elnino in the Indian subcontinent in the
last decade?
1. While El-Nino has occurred every
year in the last decade, agricultural
productivity has not been affected
negatively every year.
2. El-Nino has affected only Kharif
crops, not Rabi crops.
3. The production of crops like
Sugarcane, groundnut and
soyabean also take a hit due to ElNino.
Choose the correct matches using the codes
below.
a)
b)
c)
d)

1 and 2 only
2 and 3 only
1 and 3 only
3 only

72. With the recognition by the British


government that India should herself
http://insightsonindia.com

Page 16

74. Consider the following statements


about the Gross Capital Formation
(GCF) in the agriculture sector of
India?
1. Public sector invests more in GCF
than private sector.
2. Public sector GCF is largely
subsidies.
3. Private sector GCF has not created
any irrigation or water-saving
infrastructure.
Choose the correct matches using the codes
below.
a)
b)
c)
d)

1 and 2 only
2 and 3 only
1 and 3 only
2 only

New Policy on Seed Development


(NPSD) that include
1. permitting 100 per cent foreign
direct investment (FDI) under the
automatic route
2. simplifying the procedure for
inclusion of new varieties in the
Organisation for Economic
Cooperation and Development
(OECD) Seeds Scheme
3. creation of a seed bank
Choose the correct matches using the codes
below.
a)
b)
c)
d)

1 and 2 only
2 and 3 only
1 and 3 only
All of the above

75. Several important policy initiatives


have been taken under the amended

http://insightsonindia.com

Page 17

INSIGHTS MOCK TESTS - 2015


TEST 20 SOLUTIONS

1. Solution: a)

The Central Information Commission (CIC) set up under the Right to Information
Act is the authorised body, established in 2005, under the Government of India to act
upon complaints from those individuals who have not been able to submit
information requests to a Central Public Information Officer or State Public
Information Officer due to either the officer not having been appointed, or because
the respective Central Assistant Public Information Officer or State Assistant Public
Information Officer refused to receive the application for information under the RTI
Act.
It was constituted through an Official Gazette Notification under the provisions of
the Right to Information Act (2005). Hence, it is not a constitutional body.

2. Solution: d)
The Commission consists of a Chief Information Commissioner and not more than
ten Information Commissioners. They are appointed by the President on the
recommendation of a selection committee.
They should be persons of eminence in public life with wide knowledge and
experience in law, science and technology, social service, management, journalism,
mass media or administration and governance. They should not be a Member of
Parliament or Member of the Legislature of any State or Union Territory. They
should not hold any other office of profit or connected with any political party or
carrying on any business or pursuing any profession.

3. Solution: a)
It is an independent body which inter alia looks into the complaints made to it and
decide the appeals. It entertains complaints and appeals pertaining to offices,
financial institutions, public sector undertakings, etc., under the Central Government
and the Union Territories.
The Department of Administrative Reforms and Public Grievances is the nodal
agency of the Government of India for administrative reforms as well as redressal of
public grievances relating to the states in general and those pertaining to Central
Government agencies in particular.

WWW.INSIGHTSONINDIA.COM

INSIGHTS MOCK TESTS - 2015


TEST 20 SOLUTIONS

4. Solution: c)
They are appointed by the President on the recommendation of a committee
consisting of the Prime Minister as Chairperson, the Leader of Opposition in the Lok
Sabha and a Union Cabinet Minister nominated by the Prime Minister. They should
be persons of eminence in public life with wide knowledge and experience in law,
science and technology, social service, management, journalism, mass media or
administration and governance.

5. Solution: b)
The Chief Information Commissioner and an Information Commissioner hold office
for a term of 5 years or until they attain the age of 65 years, whichever is earlier. They
are not eligible for reappointment.
The President can remove the Chief Information Commissioner or any Information
Commissioner from the office.
Same goes for CAG; Chairman, UPSC (5 year term); and CEC. Other members of
election Commissioners are re-eligible for the post of Chairman or Chief; not any
other.

6. Solution: a)
The State Chief Information Commissioner and a State Information Commissioner
hold office for a term of 5 years or until they attain the age of 65 years, whichever is
earlier. They are not eligible for reappointment.
The Governor can remove the State Chief Information Commissioner or any State
Information Commissioner from the office under the following circumstances:

if he is adjudged an insolvent; or
if he has been convicted of an offence which (in the opinion of the Governor)
involves a moral turpitude; or

WWW.INSIGHTSONINDIA.COM

INSIGHTS MOCK TESTS - 2015


TEST 20 SOLUTIONS

if he engages during his term of office in any paid employment outside the
duties of his office; or
if he is (in the opinion of the Governor) unfit to continue in office due to
infirmity of mind or body; or
if he has acquired such financial or other interest as is likely to affect
prejudicially his official functions.

In addition to these, the Governor can also remove the State Chief Information
Commissioner or any State Information Commissioner on the ground of proved
misbehaviour or incapacity4. However, in these cases, the Governor has to refer the
matter to the Supreme Court for an enquiry. If the Supreme Court, after the enquiry,
upholds the cause of removal and advises so, then the Governor can remove him.

7. Solution: d)
The Constituent Assembly was constituted in November 1946 under the scheme
formulated by the Cabinet Mission Plan.
The features of the scheme were:
1. The total strength of the Constituent Assembly was to be 389. Of these, 296 seats
were to be allotted to British India and 93 seats to the Princely States. Out of 296
seats allotted to the British India, 292 members were to be drawn from the eleven
governors' provinces and four from the four chief commissioners' provinces3, one
from each.
2. Each province and princely state (or group of states in case of small states) were to
be allotted seats in proportion to their respective population. Roughly, one seat was
to be allotted for every million population.
3. Seats allocated to each British province were to be decided among the three
principal communitiesMuslims, Sikhs and general (all except Muslims and Sikhs),
in proportion to their population.
4. The representatives of each community were to be elected by members of that
community in the provincial legislative assembly and voting was to be by the method
of proportional representation by means of single transferable vote.
5. The representatives of princely states were to be nominated by the heads of the
princely states. It is thus clear that the Constituent Assembly was to be a partly

WWW.INSIGHTSONINDIA.COM

INSIGHTS MOCK TESTS - 2015


TEST 20 SOLUTIONS

elected and partly nominated body. Moreover, the members were to be indirectly
elected by the members of the provincial assemblies, who themselves were elected on
a limited franchise.
The elections to the Constituent Assembly (for 296 seats allotted to the British Indian
Provinces) were held in JulyAugust 1946. The Indian National Congress won 208
seats, the Muslim League 73 seats, and the small groups and independents got the
remaining 15 seats.
However, the 93 seats allotted to the princely states were not filled as they decided to
stay away from the Constituent Assembly.
Although the Constituent Assembly was not directly elected by the people of India on
the basis of adult franchise, the Assembly comprised representatives of all sections of
Indian SocietyHindus, Muslims, Sikhs, Parsis, AngloIndians, Indian Christians,
SCs, STs including women of all these sections. The Assembly included all important
personalities of India at that time, with the exception of Mahatma Gandhi and M A
Jinnah.

8. Solution: d)
The CVC is a multi-member body consisting of a Central Vigilance Commissioner
(chairperson) and not more than two vigilance commissioners. They are appointed
by the president by warrant under his hand and seal on the recommendation of a
three-member committee consisting of the prime minister as its head, the Union
minister of home affairs and the Leader of the Opposition in the Lok Sabha.
They hold office for a term of four years or until they attain the age of sixty five years,
whichever is earlier. After their tenure, they are not eligible for further employment
under the Central or a state government.

9. Solution: c)
The president can also remove the Central Vigilance Commissioner or any vigilance
commissioner on the ground of proved misbehaviour or incapacity. However, in
these cases, the president has to refer the matter to the Supreme Court for an
enquiry.

WWW.INSIGHTSONINDIA.COM

INSIGHTS MOCK TESTS - 2015


TEST 20 SOLUTIONS

If the Supreme Court, after the enquiry, upholds the cause of removal and advises so,
then the president can remove him. He is deemed to be guilty of misbehaviour, if he
(a) is concerned or interested in any contract or agreement made by the Central
government, or
(b) participates in any way in the profit of such contract or agreement or in any
benefit or emolument arising there from otherwise than as a member and in
common with the other members of an incorporated company

10.

Solution: b)

The CBI is headed by a director, an IPS officer with a rank of Director General of
Police or Commissioner of Police (State). The director is selected based on the Lokpal
and Lokayuktas Act, 2013, and has a two-year term.
The CBI is subject to five ministries of the Government of India

Ministry of Home Affairs: Cadre clearance

DoPT: Administration, budget and induction of non IPS officers

Union Public Service Commission: Officers above the rank of Deputy SP

Law and Justice Ministry: Public prosecutors

Central Vigilance Commission: Anti-corruption cases

11. Solution: c)
The amended Delhi Special Police Establishment Act empowers a committee to
appoint the director of CBI. The committee consists the following people:

Prime Minister chairperson

Leader of Opposition member

Chief Justice of India or a Supreme Court Judge recommended by the Chief


Justice member

WWW.INSIGHTSONINDIA.COM

INSIGHTS MOCK TESTS - 2015


TEST 20 SOLUTIONS

When making recommendations, the committee considers the views of the outgoing
director.
Above Selection committee was constituted under The Lokpal and Lokayuktas Act,
2013. Before this central vigilance commissioner, under CVC act, had this power.
Police comes under the State governments. CVC is concerned only with the Central
government and its authorities.

12. Solution: c)
Based on its motto, mission and the need to develop professionalism, transparency,
adaptability to change and use of science and technology in its working, the CBI
focuses on

Combating corruption in public life, curbing economic and violent crimes


through meticulous investigation and prosecution
Evolving effective systems and procedures for successful investigation and
prosecution of cases in various law courts
Helping fight cyber and high technology crime
Creating a healthy work environment that encourages team-building, free
communication and mutual trust
Supporting state police organisations and law enforcement agencies in
national and international cooperation, particularly relating to enquiries and
investigation of cases
Playing a lead role in the war against national and transnational organised
crime
Upholding human rights, protecting the environment, arts, antiques and
heritage of our civilisation
Developing a scientific temper, humanism and the spirit of inquiry and reform
Striving for excellence and professionalism in all spheres of functioning so
that the organisation rises to high levels of endeavor and achievement.

13. Solution: d)
The Government of India had notif ied a National Manufacturing Policy (NMP) vide
Press Note dated 4 November 2011 with the objective of enhancing the share of
manufacturing in GDP to 25 per cent and creating 100 million jobs over a decade.
The Policy specially focuses on industries that are employment intensive, produce

WWW.INSIGHTSONINDIA.COM

INSIGHTS MOCK TESTS - 2015


TEST 20 SOLUTIONS

capital goods, have strategic significance, and where India enjoys a competitive
advantage besides small and medium enterprises and public-sector enterprises. The
NMP provides for promotion of clusters and aggregation, especially through the
creation of national investment and manufacturing zones (NIMZ). Till 2013-14, 16
NIMZs had been announced.
Of these, eight are along the Delhi-Mumbai Industrial Corridor (DMIC). Eight other
NIMZs have been given in-principle approval: (i) Nagpur in Maharashtra, (ii)
Chittoor in Andhra Pradesh, (iii) Medak in Andhra Pradesh (now Telengana), (iv)
Prakasam in Andhra Pradesh, (v) Tumkur in Karnataka, (vi) Kolar in Karnataka, (vii)
Bidar in Karnataka, and (viii) Gulbarga in Karnataka.

14. Solution: d)
In the recent past the Prime Ministers Task Force on MSMEs and the Twelfth Plan
Working Group on MSMEs have discussed issues related to the MSME sector.
Some of the major initiatives taken for the development of this sector are: (i)
Technology Centre Systems Programme; (ii) India Inclusive Innovation Fund; (iii)
Credit Linked Capital Subsidy; (iv) Credit Guarantee Scheme; (v) Prime Ministers
Employment Generation Programme; (vi) MSE-Cluster Development Programme;
and (vii) Scheme for Extension of non tax benef its to MSMEs for three years.

15. Solution: d)
http://www.moneylife.in/article/fslrc-recommendations-on-rbi-stop-look-andproceed/32137.html

16. Solution: c)
Following in the jurisdiction of Lokpal;
Prime Minister has been brought under the purview of the Lokpal.

Lokpals jurisdiction will cover all categories of public servants.

WWW.INSIGHTSONINDIA.COM

INSIGHTS MOCK TESTS - 2015


TEST 20 SOLUTIONS

All entities receiving donations from foreign source in the context of the
Foreign Contribution Regulation Act (FCRA) in excess of Rs 10 lakh per year
are brought under the jurisdiction of Lokpal.
Provides adequate protection for honest and upright public servants.
Lokpal will have power of superintendence and direction over any
investigation agency including CBI for cases referred to them by Lokpal.

17. Solution: a)
Keeping in view the prominence of housing and real estate as a major area for
creation of both physical and financial assets and its contribution in overall National
wealth, a need was felt for setting up of a mechanism, which could track the
movement of prices in the residential housing segment. Regular monitoring of the
house prices can be useful inputs for the different interest groups. Accordingly,
National Housing Bank, at the behest of the Ministry of Finance, undertook a pilot
study to examine the feasibility of preparing such an index at the National level.
The pilot study covered 5 cities viz. Bangalore, Bhopal, Delhi, Kolkata and Mumbai.
Besides, a Technical Advisory Group (TAG), with Adviser, Ministry of Finance, as its
Chairman and comprising of experts members form RBI, NSSO, CSO, Labour
Bureau, NHB and other market players, was constituted to deal with all the issues
relating to methodology, collection of data and also to guide the process of
construction of an appropriate index . Based on the results of the study and
recommendations of the TAG, NHB launched RESIDEX for tracking prices of
residential properties in India, in July 2007 by Shri P. Chidambram (then Honble
Finance Minister).

18.Solution: d)

WWW.INSIGHTSONINDIA.COM

INSIGHTS MOCK TESTS - 2015


TEST 20 SOLUTIONS

19. Solution: d)
The failure of the Simon commission and the Round table conference which led to
the Government of India Act, 1935, to satisfy the Indian aspirations accentuated the
demand for a constitution of India made by people of India without outside
interference. This was officially asserted by the INC in 1935.
This was reiterated at the working committee meeting of INC in Ludhiana, 1939.
Before this period, there were demands either only to review the constitution or
reform the administrative structures not to have a constituent assembly.

20.

Solution: d)

The Commission can order inquiry into any matter if there are reasonable grounds
(suo-moto power). While inquiring, the Commission has the powers of a civil court in
respect of the following matters:

summoning and enforcing attendance of persons and compelling them to give


oral or written evidence on oath and to produce documents or things;

WWW.INSIGHTSONINDIA.COM

INSIGHTS MOCK TESTS - 2015


TEST 20 SOLUTIONS

requiring the discovery and inspection of documents;


receiving evidence on affidavit;
requisitioning any public record from any court or office;
issuing summons for examination of witnesses or documents; and
any other matter which may be prescribed.

During the inquiry of a complaint, the Commission may examine any record which is
under the control of the public authority and no such record may be withheld from it
on any grounds. In other words, all public records must be given to the Commission
during inquiry for examination.

21. Solution: a)
The Constitution (Ninety Seventh Amendment) Act 2011 relating to the co-operatives
is aimed to encourage economic activities of cooperatives which in turn help progress
of rural India. It is expected to not only ensure autonomous and democratic
functioning of cooperatives, but also the accountability of the management to the
members and other stakeholders.
As per the amendment the changes done to constitution are:

In Part III of the constitution, after words "or unions" the words "Cooperative
Societies" was added.
In Part IV a new Article 43B was inserted, which says: The state shall
endeavour to promote voluntary formation, autonomous functioning,
democratic control and professional management of the co-operative
societies".
After Part IXA of the constitution, a Part IXB was inserted to accommodate
state vs centre roles.

Other features:

It makes Right to form cooperatives is a fundamental right.


Reservation of one seat for SC/ST and two seats for women on the board of
every co-operative society.

22.

Solution: c)

WWW.INSIGHTSONINDIA.COM

INSIGHTS MOCK TESTS - 2015


TEST 20 SOLUTIONS

The Constitution imposes a duty upon the Centre to promote the spread and
development of the Hindi language so that it may become the lingua franca of the
composite culture of India.
Further, the Centre is directed to secure the enrichment of Hindi by assimilating the
forms, style and expressions used in hindustani and in other languages specified in
the Eighth Schedule and by drawing its vocabulary, primarily on sanskrit and
secondarily on other languages.
Every aggrieved person has the right to submit a representation for the redress of any
grievance to any officer or authority of the Union or a state in any of the languages
used in the Union or in the state, as the case may be. This means that a
representation cannot be rejected on the ground that it is not in the official language.

23.

Solution: c)

http://www.thehindu.com/news/national/salient-features-of-lokpal-lokayuktasbill/article5474256.ece

WWW.INSIGHTSONINDIA.COM

INSIGHTS MOCK TESTS - 2015


TEST 20 SOLUTIONS

24.

Solution: c)

Agriculture being a state subject, the primary responsibility for increasing


agricultural production and productivity, exploiting untapped potential, and
enhancing incomes of the farming community, rests with state governments. Their
efforts are supplemented by many centrally sponsored and central sector schemes
(not legislation).
Therefore, the Union Minsitry of Agriculture supports the efforts to the state to bring
about greater coordination and reduce regional imbalances.

25.

Solution: b)

A wind gap is a valley through which a waterway once flowed but is now dry as a
result of stream capture. A water gap is a similar feature, but one in which a
waterway still flows. Water gaps and wind gaps often provide practical routes
for trails, roads, and railroads through mountainous terrain.
Examples of wind gaps in the Blue Ridge Mountains of Virginia include Swift Run
Gap, Rockfish Gap, and Buford's Gap.
Here is an image.

Since deltas have a large sediment deposition, they prevent ships from anchoring
close to the shores. There is not enough depth in the water.
WWW.INSIGHTSONINDIA.COM

INSIGHTS MOCK TESTS - 2015


TEST 20 SOLUTIONS

26.

Solution: d)

27.

Solution: a)

http://www.thehindu.com/business/black-money-in-stock-markets-sebi-tosuspend-shell-companies/article6871176.ece
Crooks are increasingly using shell companies for tax evasion, money laundering and
even bidding lucrative coal mines with good rail-road connectivity so they can be
resold to genuine companies at a higher price (similar to 2G scam). Therefore, SEBI
began an exercise to find and suspend such shell companies.

28.

Solution: b)

The first indigenously built submarine is part of the Indian Navys ambitious
submarine programme- Project 75. This programme is undertaken in collaboration
France to build fleet of six such submarines.
Features about Scorpene submarine - Kalvari

Has superior stealth capability.


Can undertake various operations including multifarious warfare, Antisubmarine warfare, Intelligence gathering, mine laying, area surveillance etc.
Ability to launch attack on the enemy using precision guided weapons.
The attack can be launched from underwater or on surface.
Designed to operate in all theatres including the tropics.

WWW.INSIGHTSONINDIA.COM

INSIGHTS MOCK TESTS - 2015


TEST 20 SOLUTIONS

Built using special steel that can withstand high yield stress that can withstand
high hydrostatic force and enabling to dive deeper.
Equipped with Weapons Launching tubes (WLT) that can carry weapons on
board and can be easily reloaded at sea.

29.

Solution: c)

The CAT is a multi-member body consisting of a chairman and members. Earlier, the
CAT consisted of a Chairman, Vice-Chairmen and members. With the amendment in
Administrative Tribunals Act, 1985 in 2006, the members have been given the status
of judges of High Courts. At present (2013), the sanctioned strength of the Chairman
is one and sanctioned strength of the Members is 65. They are drawn from both
judicial and administrative streams and are appointed by the president. They hold
office for a term of five years or until they attain the age of 65 years, in case of
chairman and 62 years in case of members, whichever is earlier.
Originally, appeals against the orders of the CAT could be made only in the Supreme
Court and not in the high courts. However, in the Chandra Kumar case2 (1997), the
Supreme Court declared this restriction on the jurisdict-ion of the high courts as
unconstitutional, holding that judicial review is a part of the basic structure of the
Constitution.

30.

Solution: c)

What is DigiLocker?
Dedicated personal storage space, linked to each residents Aadhaar number.
DigiLocker can be used to securely store e-documents as well as store Uniform
Resource Identifier (URI) link of e-documents issued by various issuer departments.
The e-Sign facility provided as part of DigiLocker system can be used to digitally sign
e-documents.
How does DigiLocker work?
To Sign-up for the DigiLocker you need to have an Aadhaar and mobile number
registered with Aadhaar. Type your Aadhaar number and the captcha code. After
clicking signup button, an OTP (One Time Password) will be sent to the registered
mobile number and email-id. Enter OTP and click on Validate OTP button to
complete the sign up and login.
How is DigiLocker going to help me?
WWW.INSIGHTSONINDIA.COM

INSIGHTS MOCK TESTS - 2015


TEST 20 SOLUTIONS

It will minimize the use of physical documents and will provide authenticity of the edocuments It will provide secure access to Govt. issued documents. It will also reduce
administrative overhead of Govt. departments and agencies and make it easy for the
residents to receive services.
From https://digitallocker.gov.in/

31. Solution: b)
His name was given to the minor planet which was previously identified as planet
4538. Now the planet will be called as 4538 Vishyanand and is located somewhere
between the Mars and Jupiter. Minor planet 4538 Vishyanand was discovered on 10
October 1988 by Japanese astronormer Kenzo Suzuki. Initially, the planet was
unnamed for almost 10 years.
A minor planet is a term used to describe an astronomical object that is neither a
dominant planet nor a comet and thus includes the dwarf planets that orbit around
the Sun. The orbital categories of these planets are the asteroids, Kuiper belt objects,
trojans, centaurs and other trans-Neptunian objects. The first minor planet
discovered was Ceres in 1801.
Naming of Minor Planets - Traditionally, the practice of naming a minor planet is left
to its discoverer. If the discovered minor planet remained unnamed for over 10 years,
than International Astronomical Union (IAU) names the planet.

32.

Solution: d)

About Indias ranking: India is ranked 115 position on the economic parameter that
takes into account GDP, political risk and oil intensity. This rank is same as in year
2014.
In the risk quality factor, India is ranked 109 for its quality of natural hazard and fire
risk management. In this parameter, India has shown slight improvement from its
113th rank in 2014.
In the supply chain category that looks at corruption control, local supplier quality
and infrastructure, India is ranked 89th position. In this category, India has fallen 11
spots from the previous year 2014.

WWW.INSIGHTSONINDIA.COM

INSIGHTS MOCK TESTS - 2015


TEST 20 SOLUTIONS

About FM Global Resilience Index - The Index measures business resilience of 130
nations. It is based on economic, risk quality and supply chain factors. The higher
ranking of country in this index indicates that it is best suited for companies seeking
to avoid disruptions in their global supply chain operations.

33.

Solution: c)

The new trade policy will incorporate government ambitious initiatives such as Make
in India, Digital India mission, Skill India programme, ease of business in order to
boost jobs and take Indias exports to 900 billion dollars by 2020.
Focus areas of the new FTP are:

Labour intensive sectors to generate more employment.


Boosting to environment-friendly products.
High tech value addition.
Involving skill India programme to promote entrepreneurship.
Moving towards branding of products.
Incentives for units located in SEZs.
Making the life of traders easier.
The new FTP also move towards paperless working. In this regard, a facility
has been created to upload documents in exporter/ importer profile in order
to avoid need of repeatedly submitting documents.

34.

Solution: c)

Following was included in the 97th amendment:

Cooperatives could set up agency which would oversee election.


Uniformity in the tenure of Cooperative Board of Directors.
Provisions for incorporation, regulation and winding up of co-operative
societies based on the principles of democratic process and specifying the
maximum number of directors as twenty-one.
Providing for a fixed term of five years from the date of election in respect of
the elected members of the board and its office bearers;
Providing for a maximum time limit of six months during which a board of
directors of co-operative society could be kept under suspension;

WWW.INSIGHTSONINDIA.COM

INSIGHTS MOCK TESTS - 2015


TEST 20 SOLUTIONS

Providing for independent professional audit;


Providing for right of information to the members of the co-operative
societies;

35.

Solution: d)

The legislature of a state may adopt any one or more of the languages in use in the
state or Hindi as the official language of that state.
Under this provision, most of the states have adopted the major regional language as
their official language. For example, Andhra Pradesh has adopted Telugu, Kerala
Malayalam, AssamAssamese, West BengalBengali, OdishaOdia. The nine
northern states of Himachal Pradesh, Uttar Pradesh, Uttarakhand, Madhya Pradesh,
Chhattisgarh, Bihar, Jharkhand, Haryana and Rajasthan have adopted Hindi.
Gujarat has adopted Hindi in addition to Gujarati.
Similarly, Goa has adopted Marathi in addition to Konkani Jammu and Kashmir has
adopted Urdu (and not Kashmiri). On the other hand, certain north-eastern States
like Meghalaya, Arunachal Pradesh and Nagaland have adopted English. Notably, the
choice of the state is not limited to the languages enumerated in the Eighth Schedule
of the Constitution.

36.

Solution: c)

When the President (on a demand being made) is satisfied that a substantial
proportion of the population of a state desire the use of any language spoken by them
to be recognised by that state, then he may direct that such language shall also be
officially recognised in that state. This provision aims at protecting the linguistic
interests of minorities in the states.
Also refer to http://en.wikipedia.org/wiki/Languages_of_India#State_level

37.

Solution: a)

Fisheries is an important source of livelihood and f ish, are an important source of


protein. There are 14.4 million fishermen in the country. India ranks second in world
f ish production, contributing about 5.4 per cent of global f ish production. It is also a
major producer of f ish through aquaculture. Total fish production during 2013-14 is
WWW.INSIGHTSONINDIA.COM

INSIGHTS MOCK TESTS - 2015


TEST 20 SOLUTIONS

estimated at 9.45 mt with 6.10 mt coming from the inland sector and 3.35 mt from
the marine sector. The sector contributes about 1 per cent to overall GDP and
represents 4.6 per cent of agri GDP.

38.

Solution: d)

39.

Solution: a)

The nodal agency that undertakes open-ended procurement, distribution, and


storage of foodgrains is the Food Corporation of India (FCI) with other central and
state agencies. Coarse grains are procured by state governments and their agencies.
The National Agricultural Cooperative Marketing Federation of India Limited
(NAFED), National Cooperative Consumers Federation of India Limited (NCCF),
CWC, and SFAC are the central nodal agencies that undertake procurement of
oilseeds and pulses under the Price Support Scheme (PSS) when the market rates of
these commodities fall below MSP. However, procurement operations are found to
be successful largely for rice and wheat and that too only in a few states like Punjab,
Haryana, Andhra Pradesh, and Madhya Pradesh.
WWW.INSIGHTSONINDIA.COM

INSIGHTS MOCK TESTS - 2015


TEST 20 SOLUTIONS

To enhance eff iciency in procurement and public distribution and to extend the
benef its of MSP to local farmers, the Decentralized Procurement Scheme (DCP) is
adopted by some state governments. For paddy it has been adopted by West Bengal,
Madhya Pradesh, Chhattisgarh, Uttarakhand, Andaman and Nicobar Islands,
Odisha, Tamil Nadu, Gujarat, Karnataka, Kerala, and Bihar; and by Andhra Pradesh
in KMS 2013-14 in 10 districts.

40.

Solution: a)

41. Solution: d)
A river delta is low-lying plain or landform that occurs at the mouth of a river near
where the river flows into the ocean or other body of water. Deltas are important to
both human activities and fish and other wildlife because they are normally home to
very fertile soil as well as a large amount of vegetation.
Most rivers begin at high elevations where snow, rain and other precipitation run
downhill into creeks and small streams. As these small waterways flow farther
downhill they eventually meet and form rivers.

WWW.INSIGHTSONINDIA.COM

INSIGHTS MOCK TESTS - 2015


TEST 20 SOLUTIONS

In many cases these rivers then flow toward larger the ocean or another body of
water and oftentimes they combine with other rivers. At the lowest part of the river is
the delta. It is in these areas where the river's flow slows and spreads out to create
sediment-rich dry areas and bio-diverse wetlands.
River deltas are some of the most biodiverse areas on the planet and as such it is
essential that they remain healthy to provide habitat for the many species of plants,
animals, insects and fish that live in them. There are many different species of rare,
threatened and endangered species living in deltas and wetlands.

42.

Solution: d)

Factors influencing formation of deltas:

Amount and type of sediment available.

Variations in volume of water discharged from river.

Aspect and geometry of coast.

Coastal processes in operation, for example, wave action.

Changes in coast level.

Impacts of climate on growth of vegetation and marine organisms.

Deltas are especially favourable for agriculture, due to the deposition of fine
sediment, but at the same time are places of high flood risk as shown by the Ganges
Delta.
Moreover, there should not streams that cut at right angles, that could wash
sediments away.
Also, if any large lake is present, all sediment on way be deposited in the lake itself
and the delta will not be formed.

43.

Solution: a)

The existing extension and IT schemes from the Eleventh Plan were strengthened,
expanded, and scaled up appropriately and implemented as components of the Sub
Mission on Agricultural Extension (SMAE) under the National Mission on
Agricultural Extension and Technology (NMAET).
WWW.INSIGHTSONINDIA.COM

INSIGHTS MOCK TESTS - 2015


TEST 20 SOLUTIONS

Greater role has been envisaged for the states in implementation and monitoring.
The schemes subsumed under the SMAE include: District-level Agriculture
Technology Management Agencies (ATMAs) that have been set up in 639 rural
districts of 28 states and 3 union territories (UT) across the country-these have benef
ited 28.5 million farmers, 25.6 per cent of whom were women; Mass Media and
Kisan Call Centre schemes, Central-sector Establishment of Agri-Clinics and AgriBusiness Centres (ACABC) Scheme; SMS portal for farmers.

44.

Solution: d)

A wide array of crops from fruits and vegetables to nuts, spices, medicinal plants,
flowers, and plantation crops, provides many opportunities for income generation.
Globally India is the second largest producer of fruits and vegetables; the largest
producer of mango, banana, coconut, cashew, papaya, and pomegranate; and the
largest producer and exporter of spices. Horticulture production, estimated at 265
million tonnes, exceeded the production of foodgrains and oilseeds in 2012-13 owing
to an 8.6 per cent increase in productivity of horticulture crops between 2008-09
and 2012-13.

45.

Solution: c)

46.

Solution: d)

WWW.INSIGHTSONINDIA.COM

INSIGHTS MOCK TESTS - 2015


TEST 20 SOLUTIONS

Vertical erosion is dominant in upper reaches because of


I.
II.
III.
IV.

high speed of river


low sediment load
high gradient
bed erosion

Basically river rejuvenation is a term used when the river starts becoming young
again (back from mature and old stage). Incised meanders are formed when river (in
its upper courses) develops deep valleys and gorges. And existing meanders are
vertically eroded by the rejuvenating stream.

47.

Solution: b)

https://nsidc.org/cryosphere/glaciers/gallery/piedmont.html
An image of a piedmont glacier

48.

Solution: c)

WWW.INSIGHTSONINDIA.COM

INSIGHTS MOCK TESTS - 2015


TEST 20 SOLUTIONS

The all-India services are controlled jointly by the Central and state governments.
The ultimate control lies with the Central government while the immediate control is
vested in the state governments. Any disciplinary action (imposition of penalties)
against these officers can only be taken by the Central government.
The personnel of Central services work under the exclusive jurisdiction of the Central
government. They hold specialised (functional and technical) positions in various
departments of the Central government.
The personnel of state services work under the exclusive jurisdiction of the state
government. They hold different positions (general, functional and technical) in the
departments of the state government.
However, they occupy lower positions (in the administrative hierarchy of the state)
than those held by the members of the all-India services (IAS, IPS and IFS).

49.

Solution: a)

The Centre, the states, local bodies, public corporations and other public authorities
can have the services of a dedicated tribunal. In other words, Article 323 A enables
the Parliament to take out the adjudication of disputes relating to service matters
from the civil courts and the high courts and place it before the administrative
tribunals.
In pursuance of Article 323 A, the Parliament has passed the Administrative
Tribunals Act in 1985.
The act authorises the Central government to establish one Central administrative
tribunal and the state administrative tribunals. This act opened a new chapter in the
sphere of providing speedy and inexpensive justice to the aggrieved public servants.

50.

Solution: d)

This link offers a comprehensive account of desert formation


https://faculty.unlv.edu/landau/desertgeography.htm

51. Solution: a)
WWW.INSIGHTSONINDIA.COM

INSIGHTS MOCK TESTS - 2015


TEST 20 SOLUTIONS

http://www.waterencyclopedia.com/Hy-La/Lake-Formation.html

52.

Solution: a)

http://www.livemint.com/Money/rUapzQTYbqwJi1yLAwI3jP/Reserve-Bankslongterm-game-plan.html

53.

Solution: d)

http://www.clickgreen.org.uk/product/directory/125055-petrol-vs-diesel-which-ismore-environmentally-friendly-.html
http://www.bbc.com/news/science-environment-30381223

54.

Solution: a)

http://en.wikipedia.org/wiki/Drainage_system_(geomorphology)#Dendritic_drain
age_pattern

55.

Solution: d)

WWW.INSIGHTSONINDIA.COM

INSIGHTS MOCK TESTS - 2015


TEST 20 SOLUTIONS

56.

Solution: d)

http://en.wikipedia.org/wiki/Yalu_River

57.Solution: d)
Increased fertilizer usage has played a significant role in improving agricultural
productivity. Urea, which is the main source of nitrogen (N), constitutes around 50
per cent of total fertilizer consumption. India meets 80 per cent of urea requirement
through indigenous production, but is largely import dependent for its potassic (K)
and phosphatic (P) fertilizer requirements.
A modified New Pricing Scheme (NPS)-III for existing urea units, notified to address
under-recoveries of existing urea units due to freezing of fixed cost at 2002-03 rates
will be implemented for one year from 2 April 2014.

58.

Solution: a)

http://www.downtoearth.org.in/content/pesticide-and-antibiotic-use-threatenfood-safety-india

WWW.INSIGHTSONINDIA.COM

INSIGHTS MOCK TESTS - 2015


TEST 20 SOLUTIONS

59.

Solution: c)

http://www.downtoearth.org.in/content/government-releases-national-air-qualityindex-cse-welcomes-it

60.

Solution: b)

With the shadow of the El Nio looming over the Indian monsoon, there are
legitimate concerns about its likely impact on agricultural production and
consequently prices of food products. What is significant is that over the last decade
Indian agriculture has become more robust with record production of foodgrains and
oilseeds.
Increased procurement, consequently, has added huge stocks of foodgrains in the
granaries. India is one of the worlds top producers of rice, wheat, milk, fruits, and
vegetables.
However, given that India is still home to a quarter of all undernourished people in
the world and since on an average almost half the total expenditure of about half the
households is on food, increasing the efficiency of the farm-to-fork value chain is
crucial for eliminating poverty and malnutrition.

61. Solution: b)
The CAT is not bound by the procedure laid down in the Civil Procedure Code of
1908. It is guided by the principles of natural justice. These principles keep the CAT
flexible in approach. Only a nominal fee of `50 is to be paid by the applicant. The
applicant may appear either in person or through a lawyer.
The Principles of Natural Justice have come out from the need of man to protect
himself from the excesses of organized power man has always appealed to someone
beyond his own creation. Such someone is the God and His laws, divine law or
natural law, to which all temporal laws and actions must confirm.
Natural Law is of the 'higher law of nature' or 'natural law'
Two core points in the concept of principles of natural justice

WWW.INSIGHTSONINDIA.COM

INSIGHTS MOCK TESTS - 2015


TEST 20 SOLUTIONS

1. Nemo in propria causa judex, esse debet - No one should be made a judge in
his own case, or the rule against bias.
2. Audi alteram partem - Hear the other party, or the rule of fair hearing, or the
rule that no one should be condemned unheard.

These two are the basic pillars of the Principles of Natural Justice. No system of law
can survive without these two basic pillars.

62.

Solution: a)

While Article 323 A contemplates establishment of tribunals for public service


matters only, Article 323 B contemplates establishment of tribunals for certain other
matters (mentioned above).
While tribunals under Article 323 A can be established only by Parliament, tribunals
under Article 323 B can be established both by Parliament and state legislatures with
respect to matters falling within their legislative competence.
Following matters can be adjudicated by separately established tribunals as per the
Article 323 B of the Constitution:
o
o
o
o
o
o
o
o

63.

Taxation
Foreign exchange, import and export
Industrial and labour
Land reforms
Ceiling on urban property
Elections to Parliament and state legislatures
Food stuffs
Rent and tenancy rights

Solution: a)

The civil servants are conferred personal immunity from legal liability for official
contracts. This means that the civil servant who made a contract in his official
capacity is not personally liable in respect of that contract but it is the government
(Central or state) that is liable for the contract. But, if the contract is made without
complying the conditions specified in the Constitution, then the civil servant who
made the contract is personally liable.
WWW.INSIGHTSONINDIA.COM

INSIGHTS MOCK TESTS - 2015


TEST 20 SOLUTIONS

Further, the civil servants also enjoy immunity from legal liability for their tortious
acts in respect of the sovereign functions of the government. In other cases, the
liability of the civil servants for torts or illegal acts is the same as of any ordinary
citizen.

64.

Solution: d)

65.

Solution: c)

The government has also notified the Public Procurement Policy for Micro & Small
Enterprises (MSEs) order 2012. The policy mandates that every central
ministry/department/public sector-undertaking shall set a minimum annual
procurement goal of 20 per cent of total product and service purchases from MSEs
from f inancial year 2012-13 onwards, in a period of three years. Further, the policy
has also earmarked a sub-target of 4 per cent of this 20 per cent for MSEs owned by
Scheduled caste (SC)/Scheduled tribe (ST)entrepreneurs.

WWW.INSIGHTSONINDIA.COM

INSIGHTS MOCK TESTS - 2015


TEST 20 SOLUTIONS

66.

Solution: c)

In fact, all these sectors received the maximum FDI.


During 2013-14, total FDI inflows (including equity inflows, reinvested earnings, and
other capital) were US$ 36.4 billion. FDI equity inflows were US$ 24.30 billion,
showing an increase of 8 per cent as compared to the previous year. Cumulative FDI
inflows from April 2000 to March 2014 stood at US$ 323.9 billion. Net FDI inflows
were US$ 21.6 billion during 2013-14. In recent years, services, construction,
telecommunications, computer software and hardware, drugs and pharmaceuticals,
automobile industry, power, metallurgical industries, and hotels and tourism are
sectors that have attracted maximum FDI inflows.

67.

Solution: a)

WWW.INSIGHTSONINDIA.COM

INSIGHTS MOCK TESTS - 2015


TEST 20 SOLUTIONS

http://en.wikipedia.org/wiki/Glacial_landform

68.

Solution: d)

https://www.asf.alaska.edu/blog/how-do-glaciers-move/

69.

Solution: a)

WWW.INSIGHTSONINDIA.COM

INSIGHTS MOCK TESTS - 2015


TEST 20 SOLUTIONS

70.

Solution: d)

Services share in world GDP was 65.9 per cent. But its share in employment was only
44.0 per cent in 2012. However, for the top 15 countries (in terms of services GDP),
except India and China,
the shares of both services GDP and services employment are high and close to each
other. In China, the shares of both services income and services employment are low.
In the Indian case the services sector had a high share in income at 56.9 per cent in
2012 with a lower share of 28.1 per cent in employment and the gap between the two
widening from 27.3 percentage points in 2001 to 28.8 percentage points in 2012.
Indias share in world services exports, which increased from 0.6 per cent in 1990 to
1.1 per cent in 2000 and further to 3.3 per cent in 2013, has been increasing faster
than its share in world merchandise exports. While the growth rate of services
exports of India has been higher than that of the world for all the years since 1996
(except 2009), in 2013 this has been reversed.

71. Solution: c)
The Central Vigilance Commission (CVC) is the main agency for preventing
corruption in the Central government. It was established in 1964 by an executive
resolution of the Central government. Its establishment was recommended by the
Santhanam Committee on Prevention of Corruption (196264).
Thus, originally the CVC was neither a constitutional body nor a statutory body.
Recently, in September 2003, the Parliament enacted a law conferring statutory
status on the CVC.
In 2004, the Government of India authorised the CVC as the Designated Agency to
receive written complaints for disclosure on any allegation of corruption or misuse of
office and recommend appropriate action.
The CVC is conceived to be the apex vigilance institution, free of control from any
executive authority, monitoring all vigilance activity under the Central Government
and advising various authorities in Central Government organisations in planning,
executing, reviewing and reforming their vigilance work.
WWW.INSIGHTSONINDIA.COM

INSIGHTS MOCK TESTS - 2015


TEST 20 SOLUTIONS

72.

Solution: d)

The main proposals of the mission were as follows:


--An Indian Union with a dominion status; would be set up; it would be free to decide
its relations with the Commonwealth and free to participate in the United Nations
and other international bodies.
--After the end of the war, a constituent assembly would be convened to frame a new
constitution. Members of this assembly would be partly elected by the provincial
assemblies through proportional representation and partly nominated by the
princes.
--The British Government would accept the new constitution subject to two
conditions.
(i) any province not willing to join the Union could have a separate constitution and
form a separate Union, and
(ii) the new constitution- making body and the British Government would negotiate a
treaty to effect the transfer of power and to safeguard racial and religious minorities.
--In the meantime, defence of India would remain in British hands and the governorgenerals powers would remain intact.

73.

Solution: d)

In the past decade, the El Nio occurred in 2002, 2004, 2006, and 2009. While
2002-03 was the only year that India showed negative agri sector growth with
average rainfall dropping 20 per cent below normal, 2009-10 experienced the most
severe drought in nearly 40 years with total rainfall being 23 per cent below normal.
A comparison of the changes in kharif and rabi production during the last four
occurrences of El Nio reveals that the impact was more in the kharif season.
The Extended Range Forecast System (ERFS provided by the IMD) seasonal forecast
for this monsoon season indicates the probability of lower rainfall in the rainfed
regions of central, south, and north-west India, which may affect crops like rice,
soybean, cotton, maize, jowar, groundnut, and sugarcane.
The f ifth South Asian Climate Outlook Forum (SASCOF-5) session forecast monsoon
def icit rainfall mainly in the southern peninsula and central India.

WWW.INSIGHTSONINDIA.COM

INSIGHTS MOCK TESTS - 2015


TEST 20 SOLUTIONS

74.

Solution: d)

Robustness of the agri sector can be attributed to the steady increase in GCF (both
public and private) from 16.1 per cent of its GDP in 2007-08 to 21.2 per cent in 201213 (2004-05 prices).
However, public expenditure (comprising public investments and input subsidies)
has been ceding its share in total GCF of the agri sector to the private sector and was
14.7 per cent in 2012-13. As a percentage of agri GDP also private investment has
been rising and was 18.1 per cent in 2012-13. The quality of public GCF, which is
largely directed towards subsidies, is also of concern.
The largest increase in private GCF was in labour-saving machines such as irrigation
and water-saving equipment , evidently a result of the declining rural workforce and
rising real wages.

75.Solution: d)
Seed quality accounts for 20-25 per cent of crop productivity.
As hybrid seeds in cross-pollinated crops give higher yields, greater emphasis was
placed on their production; thus, their availability has been higher than the
requirement. Important policy initiatives under the amended New Policy on Seed
Development (NPSD) include permitting 100 per cent foreign direct investment
(FDI) under the automatic route and simplifying the procedure for inclusion of new
varieties in the Organisation for Economic Cooperation and Development (OECD)
Seeds Scheme.
The thrust is also on creating a seed bank. A Seed Rolling Plan for the period up to
2016-17 is in place for all the states since 2013-14 for identification of good varieties
for the seed chain, and agencies responsible for production of seeds at every level.

WWW.INSIGHTSONINDIA.COM

INSIGHTS ON INDIA MOCK PRELIMINARY EXAM - 2015


INSIGHTS ON INDIA MOCK TEST - 21
GENERAL STUDIES

PAPER-I
Time Allowed: 1.5 Hours

Maximum Marks: 170

INSTRUCTIONS
1. IMMEDITELY AFTER THE COMMENCEMENT OF THE EXAMINATION, YOU SHOULD
CHECK THAT THIS TEST BOOKLET DOES NOT HAVE ANY UNPRINTED OR TORN OR
MISSING PAGES OR ITEMS, ETC. IF SO, GET IT REPLACED BY A COMPLETE TEST BOOKLET.
2. You have to enter your Roll Number on the Test I
Booklet in the Box provided alongside. DO NOT
write anything else on the Test Booklet.
4. This Test Booklet contains 85 items (questions). Each item is printed only in English. Each item comprises
four responses (answers). You will select the response which you want to mark on the Answer Sheet. In
case you feel that there is more than one correct response, mark the response which you consider the best.
In any case, choose ONLY ONE response for each item.
5. You have to mark all your responses ONLY on the separate Answer Sheet provided. See directions in the
Answer Sheet.
6. All items carry equal marks.
7. Before you proceed to mark in the Answer Sheet the response to various items in the Test Booklet, you
have to fill in some particulars in the Answer Sheet as per instructions sent to you with your Admission
Certificate.
8. After you have completed filling in all your responses on the Answer Sheet and the examination has
concluded, you should hand over to the Invigilator only the Answer Sheet. You are permitted to take away
with you the Test Booklet.
9. Sheets for rough work are appended in the Test Booklet at the end.
10. Penalty for wrong answers :
THERE WILL BE PENALTY FOR WRONG ANSWERS MARKED BY A CANDIDATE IN THE
OBJECTIVE TYPE QUESTION PAPERS.
(i)

There are four alternatives for the answer to every question. For each question for which a
wrong answer has been given by the candidate, one-third of the marks assigned to that question
will be deducted as penalty.

(ii) If a candidate gives more than one answer, it will be treated as a wrong answer even if one of the
given answers happens to be correct and there will be same penalty as above to that question.
(iii)

If a question is left blank, i.e., no answer is given by the candidate, there will be no penalty for
that question.
http://insightsonindia.com

INSIGHTS ON INDIA MOCK TEST SERIES FOR CIVIL SERVICES PRELIMINARY EXAM 2015

http://insightsonindia.com

Page 1

1. The list of castes or tribes that are to be


identified as SCs or STs are specified in
a) a Presidential order with respect to
each State and Union Territory
(UT)
b) the Fifth and Sixth Schedule of the
Constitution of India
c) the resolutions passed by the
respective state legislatures
d) the order issued by the State
Governor in the extra-ordinary
Gazette of India

2. As it does with respect to the SCs, the


National Commission for SCs is also
required to discharge similar functions
with regard to the
1. OBCs
2. STs
3. Anglo-Indian Community
Choose the correct answer from the codes
given below.
a)
b)
c)
d)

1 and 2 only
2 and 3 only
1 and 3 only
All of the above

3. The Election Commission registers


political parties for the purpose of
elections and grants them recognition
as national or state parties on the basis
of their
1. Assets and liabilities
2. Geographical presence in more
than four constituencies
3. Number of workers, volunteers and
election candidates
4. Poll performance

a)
b)
c)
d)

1 and 4 only
2 and 4 only
1 and 3 only
4 only

4. Consider the following statements.


1. There is only one general electoral
roll for every territorial
constituency for election to the
Parliament and the state
legislatures.
2. The state legislatures, apart from
Parliament can also make provision
with respect to all matters relating
to elections to the state legislatures.
Which of the above is/are true?
a)
b)
c)
d)

1 only
2 only
Both 1 and 2
None

5. Which of the following government


officials are a part of the machinery of
Election Commission of India that
conducts and supervises elections in
India?
1. Observers
2. Presiding officer
3. Returning Officer
4. Chief Electoral Officer
Choose the correct answer from the codes
given below.
a)
b)
c)
d)

1 and 4 only
2 and 4 only
1 and 3 only
All of the above

Choose the correct answer from the codes


given below.
http://insightsonindia.com

Page 2

6. The Model Code of Conduct for


guidance of candidates and political
parties comes immediately into effect
a) after the Election Commission of
India announces the election
schedule in a major press
conference
b) after the Election Commission of
India issues a specific order
enforcing the Model code of
conduct
c) automatically exactly one month
before the election dates
d) after the deadline for nominating
candidates ends

7. Every candidate seeking election to the


Parliament or a State Legislature has
to furnish on his nomination paper the
information on which of the following
matters?
1. Educational qualifications
2. Criminal Record
3. Assets and Liabilities
4. Details of political funding
Choose the correct answer from the codes
given below.
a)
b)
c)
d)

1 and 4 only
2, 3 and 4 only
1, 2 and 3 only
All of the above

8. Consider the following statements


about Elections to Rajya Sabha.
1. A candidate has to be an elector in
the state from where he wishes to
be elected in Rajya Sabha.
2. Secret Ballot system is used in
Rajya Sabha elections.
3. Both the Legislative assembly and
the legislative council take part in
the Rajya Sabha elections.
http://insightsonindia.com

Choose the correct answer from the codes


given below.
a)
b)
c)
d)

1 and 2 only
2 and 3 only
1 and 3 only
None of the above

9. Which of the following correctly points


the difference between Exit polls and
Opinion Polls?
1. Exit polls are not banned, but
opinion polls are banned in India.
2. Exit polls are taken after the elector
votes and exits from a particular
polling booth; opinion polls are
taken before the election.
3. Exit polls are conducted by the
Election Commission of India,
whereas opinion
polls are
conducted by private media
agencies.
Choose the correct answer from the codes
given below.
a)
b)
c)
d)

1 and 2 only
2 and 3 only
1 and 3 only
2 only

10. Disqualification on ground of defection


does not apply in cases of
1. Political Party mergers
2. Political party splits
3. A candidate quitting political party
Choose the correct answer from the codes
given below.
a)
b)
c)
d)

1 and 2 only
2 and 3 only
1 and 3 only
2 only
Page 3

11. Consider a case where a MP has voted


against his political party in the
Parliament. Which among the
following authorities will decide on his
disqualification pertaining to antidefection law?
a) Presiding Officer of that House of
the Parliament
b) President of India
c) Election Commission of India
d) Parliamentary Committee on Antidefection

12. Which of the following authorities


have a bearing on the decision of
Nuclear retaliation to a first strike on
India?
1. Nuclear Command Authority
2. Indian Armed Forces
3. Parliament
Choose the correct answer from the codes
given below.
a)
b)
c)
d)

1 and 2 only
2 and 3 only
1 and 3 only
1 only

c) 2 and 3 only
d) 1 and 4 only

14. Article 366 contains the definitions of


various terms used in different
provisions of the constitution. Which
of the following terms have NOT been
defined in the Constitution?
1. Indian State
2. Ruler
3. Anglo-Indian
Choose the correct answer from the codes
given below.
a)
b)
c)
d)

1 only
2 only
3 only
All have been defined.

15. Consider the following associations of


Individual islands and the Straits that
separate them from Mainland.
1. Newfoundland separated by Strait
of Belle isle
2. Tasmania by the Bass strait
3. Madagascar by the Mozambique
channel
Choose the correct answer using the codes
below.

13. In the Table of Precedence, which of


the following figures/authorities
precede the Chief Election
Commissioner?
1. Holders of Bharat Ratna decoration
2. Judges of Supreme Court
3. Attorney General of India
4. Leader of Opposition, Lok Sabha.
Choose the correct answer using the codes
below.
a) 1 and 2 only
b) 1, 2 and 4 only
http://insightsonindia.com

a)
b)
c)
d)

1 and 2 only
1 and 3 only
2 and 3 only
All of the above

16. The reef-building corals will NOT


flourish in conditions where
1. Cold currents frequently upwell
2. waters are very deep
3. there is absence of silt and water is
salty
Page 4

Choose the correct answer using the codes


below.
a)
b)
c)
d)

1 and 2 only
1 and 3 only
2 and 3 only
All of the above

17. The Continental shelves are of great


geographical significance for which of
the following reasons?
1. They are the richest fishing
grounds in the world as they are
rich in plankton growth.
2. They help increase the height of
tides.
3. They are rich in hydrocarbon
resources.
Choose the correct answer using the codes
below.
a)
b)
c)
d)

1 and 2 only
1 and 3 only
2 and 3 only
All of the above

18. A swimmer finds it easier to float in a


more saline denser ocean than a less
denser one. Which of the following
factors would have contributed to
making the ocean more saline making
it easy to swim?
1. High rate of evaporation
2. Degree of water mixing by currents
being less
3. Less precipitation over that ocean
Choose the correct answer using the codes
below.
a)
b)
c)
d)

19. The Norwegian coast, even at Latitude


60-70 degrees, is ice-free throughout
the year because of the influence of
1. Warm North Atlantic current
2. Continentality
3. Westerly trade winds
Choose the correct answer using the codes
below.
a)
b)
c)
d)

1 and 2 only
1 and 3 only
2 and 3 only
1 only

20. Which of the following factors decide


the movement of ocean currents?
1. Planetary Winds
2. Rotation of earth
3. Salinity of Ocean
4. Temperature of Ocean water
Choose the correct answer using the codes
below.
a)
b)
c)
d)

1 and 2 only
1, 3 and 4 only
2, 3 and 4 only
All of the above

21. In the North Indian ocean, there is a


complete reversal of the direction of
ocean currents between Summer and
Winter due to
a) Western tropical cyclones
b) Reversal in the direction of Jet
streams
c) Changes in the Monsoon winds
d) Cooling down of Tibetan plateau

1 and 2 only
1 and 3 only
2 and 3 only
All of the above

http://insightsonindia.com

Page 5

22. Consider the following statements:

Assertion (A): Death rate due to diseases is


normally high in tropical countries and low in
deserts.
Reason (R): Germs are not readily
transmitted in regions of high temperature
and low humidity.
In the context of the statements above, which
of these is true?
a) A and R both are true, and R is the
correct explanation for A.
b) A and R both are true, and R is the
NOT the correct explanation for A.
c) A is correct, R is incorrect.
d) A and R both are incorrect.

23. A barometer if subjected to the same


pressure will show different readings
when the following change
1. Altitude of the device
2. Gravitational force at different
latitudes
3. Temperature
Choose the correct answer using the codes
below.
a)
b)
c)
d)

1 and 2 only
1 and 3 only
2 and 3 only
All of the above

24. Relative humidity of the air in a


particular location governs which of
the following factors?
1. Cloud formation
2. Precipitation
3. Circulation of Ocean currents
Choose the correct answer using the codes
below.
http://insightsonindia.com

a)
b)
c)
d)

1 and 2 only
1 and 3 only
2 and 3 only
All of the above

25. The phenomenon of continentality


happens because
a) Ocean currents cannot reach inner
seas surrounded by land
b) land surfaces are heated and
cooled faster than the ocean water
around it
c) there is a larger landmass in the
Northern hemisphere than
Southern hemisphere
d) the effect of ocean waves and tides
is less pronounced in inner
continental regions

26. Consider the following.


1. Cold air underlies warmer air at
higher altitudes.
2. Passage of a cold front
3. Invasion of sea air by a cooler
onshore breeze
Which of the above will be seen when
temperature inversion occurs?
a)
b)
c)
d)

1 and 2 only
2 and 3 only
1 and 3 only
All of the above

27. Consider the following statements:


Assertion (A): During a temperature
inversion, air pollution released into the
atmosphere's lowest layer is trapped there
and can be removed only by strong horizontal
winds.

Page 6

Reason (R): Inversion of air temperature


restricts vertical mobility of wind.
In the context of the statements above, which
of these is true?
a) A and R both are true, and R is the
correct explanation for A.
b) A and R both are true, and R is the
NOT the correct explanation for A.
c) A is correct, R is incorrect.
d) A and R both are incorrect.

28. Consider the following sequence of


events.
1. Convergence of cold and warm air
2. Warm air rises above the cold air
3. Cold air pushes warm air
For a cyclonic or frontal rain, the above
activities should happen in which sequence?
a)
b)
c)
d)

123
213
132
321

29. Arrange the following planetary winds


in order of their occurrence from
North to South Pole.
1. South-Westerlies
2. North-East Trade winds
3. North-Westerlies
4. South-east trade winds
Choose the correct order from the codes
below.
a)
b)
c)
d)

1243
1234
3241
None of the orders above is correct.

http://insightsonindia.com

30. Consider the following statements.


1. Winds in the Northern hemisphere
are always deflected to their right.
2. Anticyclones are more frequent in
summer.
3. Cyclones do not occur in temperate
zones.
Choose the correct answer using the codes
below.
a)
b)
c)
d)

1 and 2 only
1 and 3 only
2 and 3 only
All of the above

31. Consider the following statements


about the Rubber plantation.
1. It was first discovered in the
Amazon Basin.
2. Brazil exports practically no natural
rubber.
3. Rubber has been transplanted in
several temperate regions too.
4. Thailand and Indonesia are leading
producers of rubber.
Choose the correct answer using the codes
below.
a)
b)
c)
d)

2 and 3 only
1 and 4 only
1, 2 and 4 only
All of the above

32. The Aranyakas or the treatises of the


forest
1. present secret explanations of the
rituals mentioned in Vedas
2. have their origin in the
philosophical discussions of the
Brahmanas
3. explain the meaning of Upanishads

Page 7

Choose the correct answer using the codes


below.
a)
b)
c)
d)

1 and 2 only
1 and 3 only
2 and 3 only
All of the above

33. As put in the Hindu Mahakavya (great


epics), Ramayana and Mahabharata,
which of the following are a part of
Purushartha (meaning of human life)?
1. Liberation (Moksha)
2. Fulfilment of all desires (Kama)
3. Worldly achievement (Artha)
4. Righteousness (Dharma)
Choose the correct answer using the codes
below.
a)
b)
c)
d)

2 and 3 only
1 and 4 only
1, 2 and 4 only
All of the above

34. The Puranas were written to illustrate


and expound the truth of the Vedas.
Out of these Mahapuranas became
more popular. Which of the following
subjects are dealt by Mahapuranas?
1. Sarga, the original creation of the
universe
2. Pratisarga, the periodical process of
destruction and re-creation
3. Manvantara, the different eras or
cosmic cycles
4. Surya Vamsha and Chandra Vamsa,
the histories of the solar and lunar
dynasties of Gods and sages
5. Vamshanucharita, the genealogies
of kings
Choose the correct answer using the codes
below.
http://insightsonindia.com

a)
b)
c)
d)

2, 3 and 5 only
1, 4 and 5 only
1, 2 and 4 only
All of the above

35. All the Buddhist canonical literature is


in Pali which includes Tipitaka
(threefold basket). Choose the right
matches among them with the subject
they deal with.
1. Vinaya Pitaka - Collection of the
speeches and dialogues of the
Buddha
2. Sutta Pitaka - Contains the
monastic rules of the Order of
Buddhist monks
3. Abhidhamma Pitaka - Elucidates
the various topics dealing with
ethics, psychology or theory of
knowledge
Choose the correct answer using the codes
below.
a)
b)
c)
d)

1 and 2 only
1 and 3 only
2 and 3 only
3 only

36. Consider the following statements


about early Dravidian Literature.
1. In Manimekalai (Tamil) there is an
elaborate exposition of the
doctrines of Buddhism.
2. Vaishnava bhakti literature was a
part of it.
3. There was no place for subjective
love poems or poetry in early
Dravidian literature.
Choose the correct answer using the codes
below.
a) 1 and 2 only
b) 1 and 3 only
Page 8

c) 2 and 3 only
d) All of the above

37. Which of the following were advocated


or being talked about by Nanak in
early Punjabi literature?
1. Truth is supreme, but above truth
is truthful living.
2. Unchanging reality
3. Cosmic law
4. Meditation
Choose the correct answer using the codes
below.
a)
b)
c)
d)

1 and 2 only
1 and 4 only
2 and 3 only
All of the above

38. The most powerful trend of medieval


Indian literature between 1000 and
1800 A.D. is devotional (bhakti)
poetry. The dominating note of bhakti
is
a) ecstasy and total identity with God
b) bliss in becoming the servant of
God
c) detachment of worldly identities
and service of God
d) detachment of worldly identities
and service of mankind

39. Which of the following were NOT


advocated by the Bhakti
movemement?
1. There is God in every human being.
2. Annihilation of the age-old caste
system
3. Conceptual erudition
Choose the correct answer using the codes
below.
http://insightsonindia.com

a)
b)
c)
d)

1 and 2 only
1 and 3 only
2 and 3 only
3 only

40. Consider the following about Sufism.


1. It was concerned with the mystical
dimensions of Islam.
2. It is a science whose objective is the
reparation of the heart and turning
it away from all else but God.
3. It also advocated the practice of
Dikhr reading religious scriptures
filled with the understanding of
Gods grace.
Choose the correct answer using the codes
below.
a)
b)
c)
d)

1 and 2 only
1 and 3 only
2 and 3 only
All of the above

41. The practice of Muraqaba in Sufism


can be likened to the practices of what
in other religions and faiths?
a) Prayer to God
b) Meditation
c) Dancing and whirling in the
expression of God
d) Religious pilgrimage to attain
liberation

42. Recently the Social progress index has


been in news that measure the all
round social development of a country.
It does NOT include
1. Access to advance education.
2. Tolerance and inclusion
3. Women participation
4. Personal safety
Page 9

Choose the correct answer from the codes


given below.
a)
b)
c)
d)

2 only
3 only
3 and 4 only
1 and 4 only

43. Ministry of Textile has taken a decision


to promote the use of geo-textiles in
North-eastern belts of India. Consider
the following statements about
Geotechnical textile products.
1. Geo-textiles are used most
commonly in the transportation
market for construction of
roadways.
2. Using these products consumes
more space in landfill projects.
3. India is the second largest producer
of this.
4. It is useful in desert area road
connectivity.
Choose the correct answer using the codes
below.
a)
b)
c)
d)

1 and 2 only
3 and 4 only
1 only
3 only

44. Which of following is NOT concerned


with the recommendations of the latest
reports of the 20th Law Commission?
1. Make it compulsory for parties to
declares all donations below a
certain limit
2. In Favor of compulsory voting.
3. Ban independent candidates from
contesting election.
4. The disqualification decision for
candidates concerning antidefection law should be made by
President and governor
respectively.
http://insightsonindia.com

Choose the correct answer using the codes


below.
a)
b)
c)
d)

1, 2 and 4 only
2 only
4 only
3 only

45. Which of the following are the features


of the recently launched Dhanush
Missile?
1. It is an air to air missile.
2. It is a liquid propelled missile.
3. It is capable to target both land
based and sea based target.
4. It cannot be used for intercontinental missile attacks.
Choose the correct answer using the codes
below.
a)
b)
c)
d)

1, 2 and 4 only
1 and 2 only
3 and 4 only
All of the above

46. Consider the following statements


about the International Renewable
Energy Agency (IRENA).
1. It is an intergovernmental
organization.
2. It has three governing bodies
(assembly, council, executives).
3. India is a permanent member of
it.
4. It is an organ of United Nations.
Which of the above is NOT correct about
IRENA?
a)
b)
c)
d)

1 and 4 only
3 only
2 and 4 only
3 and 4 only

Page 10

47. Consider the following statements


about bio-diversity.
1. Buddha peacock butterfly is
endemic to East Asia.
2. Malayan Green Banded peacock
butterfly also discovered in west
Bengal.
3. Sarus crane is the official bird of
Uttarakhand.

organization for any of the


purpose which it has to conduct.
4. Its objective is to generate
employment in rural India.
Which of the above is NOT true about the
KVIC?
a)
b)
c)
d)

2 and 3 only
1 only
3 only
1 and 2 only

Choose the correct answer using codes below.


a)
b)
c)
d)

1 and 2 only
2 and 3 only
2 only
1 only

48. Consider the following statements


about Green peace.
1. It is a non-profit organization.
2. Government of India has
suspended the registration of
Green peace India.
3. It is regulated under FCRA 2010.
Choose the correct answer from the codes
given below.
a)
b)
c)
d)

1 and 2 only
2 and 3 only
1 and 3 only
All of the above

49. Consider the following statements


about the Khadi and village industry
cCmmission.
1. It is an executive body.
2. It comes under the ministry of
micro, small and medium
enterprise.
3. The KVIC is authorized to
establish and maintain separate

http://insightsonindia.com

50. Rail budget 2015 announced


Kayakulp council. Consider the
following about it.
1. Its objective is to transform only
customer service in Indian
railways.
2. Government officials will head
this body.
3. It will have representatives from
rail trade unions as well.
Which of the above is correct?
a)
b)
c)
d)

1 and 2 only
2 and 3 only
1 and 3 only
3 only

51. Supreme court has recently given a


ruling over the growing cases of acid
attacks on women. Consider the
following with reference to it.
1. State will be responsible to
provide compensation to the
victim.
2. Victim will be provided free
reconstruction surgery.
3. Criminal action can be taken on
the hospitals that discriminate
with such victims.

Page 11

4. State will provide special


privileges for the livelihood of the
victim.
Which of the above is NOT correct?
a)
b)
c)
d)

1 and 3 only
3 only
4 only
2 and 4 only

52. Consider the following statements


about two species Chestnut breasted
partridge and Telangana crab spider.
1. Chestnut breasted partridge is
endemic to western Himalayas.
2. Chestnut breasted partridge is
Endangered according to IUCN.
3. Telangana crab spider do not
construct webs.
4. Telangana crab spider work as
bio-control agents against insects.
Which of the above is NOT correct?
a)
b)
c)
d)

1 and 2 only
1 and 3 only
4 only
3 and 4 only

53. Consider the following statements:


Assertion (A): Radar cannot be used to track
thing in water bodies.
Reason (R): Radar uses high electromagnetic
frequency which is absorbed by water and
thus cannot travel more.
In the context of the statements above, which
of these is true?

c) A is correct, R is incorrect.
d) A and R both are incorrect.

54. Consider the following with reference


to the principle of Net neutrality that
has been in news for long. As per the
principle
1. Free internet cannot be given.
2. All traffic on internet should be
treated equally.
3. For the same service, users can
not be charged differently.
Which of the above is correct?
a)
b)
c)
d)

1 and 2 only
1 and 3 only
2 and 3 only
All of the above

55. Consider the following statements


about Project 75, a bilateral joint
project.
1. It is to develop submarine with
joint India and Germany
collaboration.
2. No production has been
undertaken till date under the
project.
3. Vehicles designed under the
project will not be used for
intelligence gathering and stealth
capabilities.
Which of the above is true about Project 75?
a)
b)
c)
d)

1 and 2 only
2 and 3 only
1 and 3 only
None of the above

a) A and R both are true, and R is the


correct explanation for A.
b) A and R both are true, and R is the
NOT the correct explanation for A.
http://insightsonindia.com

Page 12

56. Which of the following nations provide


nuclear fuel to India for the purposes
of generating nuclear energy?
1. Russia
2. Kazakhstan
3. Canada
4. Australia
5. USA
Choose the correct answer using the codes
below.
a)
b)
c)
d)

1 and 5 only
2, 4 and 5 only
1, 3, 4 and 5 only
1, 2 and 3 only

57. Consider the following statements with


regard to the terms of reference of the
task force on Inter-linking of rivers set
up by the Government of India.
1. It will not take up Himalayan
rivers in the inter-linking project.
2. Only intra-state rivers can be
interlinked as per the government
orders.
3. Inter-linking work will be taken
under PPP mode of investment.
Which of the above is correct?
a)
b)
c)
d)

1 and 2 only
2 and 3 only
1 and 3 only
None of the above

58. Which of the following genres were


NOT parts of the trend of the medieval
literature?
1. Bhakti
2. Love ballads
3. Heroic poetry
4. Mystical poetry

http://insightsonindia.com

Choose the correct answer from the codes


below.
a)
b)
c)
d)

1, 2 and 4 only
2 and 3 only
All of the above were parts.
None of the above were parts.

59. The birth of writing and publishing of


novels in Indian literature is
associated with the
a) social reform-oriented movement
of the 19th century
b) writings of Amir Khusro and other
Medieval writers and poets
c) incoming of Christian
missionaries in India
d) early decades of postindependence when freedom of
speech was granted

60. Novels were found to be the most


appropriate medium to eulogize the
intellectual and physical richness of
the past, and reminded Indians about
their obligations and rights. In this
context, novels were written by
1. Bankim Chandra Chattarjee
2. Hari Narayan Apte
3. Rabindranath Tagore
Choose the correct answer from the codes
below.
a)
b)
c)
d)

1 and 2 only
2 and 3 only
1 and 3 only
All of the above

Page 13

61. Slowdown in the Indian economy is


associated and aggravated by
slowdown in which of these other
economies?
1. China
2. Japan
3. Europe
4. Russia
Choose the correct answer from the codes
below.
a)
b)
c)
d)

1, 2 and 3 only
2 and 3 only
1, 3 and 4 only
2 and 4 only

62. Which of the following economic


indicators have not grown since the
onset of the recent economic slowdown
in India?
1. Gross capital formation as a
percentage of GDP
2. Savings rate as a percentage of
GDP
3. Per capita Net National Income
Choose the correct answer from the codes
below.
a)
b)
c)
d)

1 and 2 only
2 and 3 only
1 and 3 only
All of the above

63. Consider the following about money


and credit flow in India.
1. With the recent actions of RBI,
broad money has reduced over the
last years as a percentage of GDP.
2. Scheduled commercial bank credit
has been reducing as a percentage
of GDP since the economic
slowdown in the recent years.
http://insightsonindia.com

Which of the above is/are true?


a)
b)
c)
d)

1 only
2 only
Both 1 and 2
None

64. Which of the following changes have


been made in the calculation of
headline growth in India?
1. Headline growth rate will now be
measured by GDP at constant
market prices instead of GDP at
factor cost at constant prices.
2. Sector-wise estimates of gross
value added (GVA) will now be
given at basic prices instead of
factor cost.
3. GDP growth data will be released
on a monthly basis for the
information of investors.
Choose the correct answer from the codes
below.
a)
b)
c)
d)

1 and 2 only
2 and 3 only
1 and 3 only
All of the above

65. The gap between domestic savings and


domestic investment in economics is
definitionally equal to the
a) Current account balance
b) Capital account balance
c) Domestic deficit
d) Reserves of all the scheduled
commercial banks in India

66. Which of the following can be included


in Gross Fixed Capital Formation
(GFCF)?
1. Dwellings
Page 14

2. Cultivated biological resources


3. Intellectual property products
4. Machinery
Choose the correct answer from the codes
below.
a)
b)
c)
d)

1 and 4 only
3 and 4 only
1, 2 and 3 only
All of the above

67. The decline in inflation during the last


financial year turned out to be much
faster than was anticipated in the
initial months of the year by the
government. Which of these factors
may have contributed to the same?
1. Persistent decline in crude oil
prices
2. Tight monetary policy
3. Relative stability of rupee
compared to other major
currencies
4. Base effect
Choose the correct answer from the codes
below.
a)
b)
c)
d)

1 and 2 only
3 and 4 only
1, 2 and 3 only
All of the above

68. Arrange the following in decreasing


order of their total value in Indian
exports.
1. Manufacturing goods
2. Crude and petroleum products
(including coal)
3. Agriculture and allied products

a)
b)
c)
d)

123
312
321
132

69. Consider the following statements


about the performance of the external
sector of India in the last one decade.
1. Net capital flows have never been
higher than the Current Account
Deficit (CAD).
2. Indias foreign exchange reserves
have not increased (rather
decreased).
3. Among the major economies with
a CAD, India is the second largest
foreign exchange reserve holder
after Brazil.
Choose the correct answer from the codes
below.
a)
b)
c)
d)

1 and 2 only
2 and 3 only
3 only
1 only

70. With a zero Current account Deficit,


which of the following can lead to an
increase in the Foreign exchange
reserves of India?
1. Higher export of invisibles
2. Higher portfolio investment
3. Higher External Commercial
Borrowing
Choose the correct answer from the codes
below.
a)
b)
c)
d)

1 and 2 only
2 and 3 only
1 and 3 only
All of the above

Choose the correct order from the codes


below.
http://insightsonindia.com

Page 15

71. Consider the following statements with


regard to External Commercial
Borrowings (ECBs).
1. Only private sector can use ECBs
to raise resources from abroad.
2. ECBs cannot be used for
investment in stock market or
speculation in real estate.
3. An infrastructure project can be
fully funded by an ECB.
4. A borrower cannot refinance its
existing rupee loan through ECB.
Choose the correct answer from the codes
below.
a)
b)
c)
d)

1 and 2 only
2, 3 and 4 only
1, 3 and 4 only
2 and 4 only

72. Indias external debt stock can increase


because of an increase in
1. Foreign Direct Investment
2. ECBs
3. Non-resident Indian Deposits
Choose the correct answer from the codes
below.
a)
b)
c)
d)

1 and 2 only
2 and 3 only
1 and 3 only
All of the above

73. Out of the four sources of taxation


listed below, which two have
contributed more to the total tax
revenue over last half decade?
1. Income Tax
2. Custom Duty
3. Service Tax
4. Corporation tax
http://insightsonindia.com

Choose the correct answer from the codes


below.
a)
b)
c)
d)

1 and 2
2 and 4
1 and 3
1 and 4

74. Consider the following sector-wise


comparisons with regard to the Central
Plan outlay in the last two annual
budgets. Plan outlay for
1. Energy is higher than that for
Industry and minerals.
2. Industry and minerals is lower
than that for transport.
3. Social services is higher than that
for energy.
Choose the correct answer from the codes
below.
a)
b)
c)
d)

1 and 2 only
2 and 3 only
1 and 3 only
All of the above

75. The introduction of the GST would be


a significant step in the field of indirect
tax reforms in India. Consider the
following statements about it.
1. GST would be a destination-based
tax as against the present concept
of originbased tax.
2. It would also be levied on interstate supplies.
3. Taxpayers with a turnover below a
certain threshold would be
exempt from GST.
4. GST will be levied only by the
Centre, and from there it will be
appropriated to the states.
Choose the correct answer from the codes
below.
Page 16

a)
b)
c)
d)

1, 2 and 3 only
2 and 4 only
1 and 3 only
1 and 4 only

76. The legendary struggle which also


became famous by the name of the
August Revolution in India was
a) Quit India movement
b) Civil Disobedience Movement
c) Non-cooperation movement
d) Swadeshi Movement

77. Which of the following


events/incidents were precursors to
the Quit India movement?
1. Failure of the Cripps Mission
2. British evacuation (leaving their
non-white subjects) from Malaya
and Burma
because of
Japanese invasion of Burma
3. British raising import duty on
various Indian manufactured
items
Choose the correct answer from the codes
below.
a)
b)
c)
d)

1 and 2 only
2 and 3 only
1 and 3 only
All of the above

78. The Quit India movement


1. was not ratified by Indian
National Congress
2. was spearheaded by Gandhi
3. started from Bombay.
4. was aimed at getting dominion
status for India
Choose the correct answer from the codes
below.

b) 2 and 4 only
c) 1 and 3 only
d) 2 and 3 only

79. Which of the following official appeals


were made by the leadership of the
Quit India movement before launching
the struggle?
1. Government servants to resign
from their posts
2. Soldiers to leave their companies
3. Princely states to declare that they
have withdrawn from the
suzerainty of the British
4. Peasants to refuse paying the land
revenue
Choose the correct answer from the codes
below.
a)
b)
c)
d)

1 and 2 only
All of the above
1, 3 and 4 only
4 only

80.A significant feature of the Quit India


Movement was the emergence of what
came to be known as parallel
governments in some parts of the
country. These were
1. Ballia in East U P
2. Tamluk in the Midnapur district
of Bengal
3. Satara in Maharashtra
Choose the correct answer from the codes
below.
a)
b)
c)
d)

1 and 2 only
2 and 3 only
1 and 3 only
All of the above

a) 1, 2 and 3 only
http://insightsonindia.com

Page 17

81. Consider the following statements


about the pre-Independence Indian
National Army (INA).
1. It was formed after the failure of
Quit India movement.
2. The idea of the INA was first
conceived in Malaya by Mohan
Singh.
3. The INA was raised by the
Japanese Army and supported by
them till our Independence.
Choose the correct answer from the codes
below.
a)
b)
c)
d)

1 and 2 only
2 and 3 only
1 and 3 only
2 only

82. The failure of the Imphal campaign


quashed any hopes of the INA
liberating the nation. The Imphal
campaign was
a) Japanese armies attempt to
destroy the Allied forces at Imphal
and invade India
b) INAs secret operation to invade
India with the help of Burmese
army
c) British armiesattempt to destroy
the Axis forces at Imphal and
invade Imphal
d) None of the above

83. Consider the following statements


about the Tebhaga movement.
1. It was initiated by the Kisan Sabha
in Bengal.
2. The demand of the movement was
to reduce the share given to
landlords.

http://insightsonindia.com

3. It was a non-violent movement


based on Satyagraha and
negotiation.
Choose the correct answer from the codes
below.
a)
b)
c)
d)

1 and 2 only
2 and 3 only
1 and 3 only
All of the above

84. The Warlis movement organized in


Western India was against
1. Forest contractors
2. Money lenders
3. British Bureaucracy
4. Rich farmers
5. Landlords
Choose the correct answer from the codes
below.
a)
b)
c)
d)

2, 4 and 5 only
1 and 2 only
3, 4 and 5 only
All of the above

85. The British were using political control


to subordinate the Indian economy
and society to the needs of the British
economy and society. Among them
Drain of wealth was caused by
1. Plunder and taxation
2. Employment of Englishmen in
India (Home charges)
3. Investing British capital in India
4. Free and unequal trade with India
Choose the correct answer from the codes
below.
a)
b)
c)
d)

1 and 2 only
All of the above
1, 3 and 4 only
4 only
Page 18

http://insightsonindia.com

Page 19

INSIGHTS MOCK TEST - 21


SOLUTIONS

WWW.INSIGHTSONINDIA.COM

1. Solution: a)
The constitution does not specify the castes or tribes which are to be called the SCs or
the STs. It leaves to the President the power to specify as to what castes or tribes in
each state and union territory are to be treated as the SCs and STs. Thus, the lists of
the SCs or STs vary from state to state and union territory to union territory. In case
of the states, the President issues the notification after consulting the governor of the
state concerned. But, any inclusion or exclusion of any caste or tribe from
Presidential notification can be done only by the Parliament and not by a subsequent
Presidential notification. Presidents have issued several orders specifying the SCs
and STs in different states and union territories and these have also been amended
by the Parliament.

2. Solution: c)
The President should set up a National Commission for the SCs to investigate all
matters relating to the constitutional safeguards for the SCs and to report to him
(Article 338). Similarly, the President should also set up a National Commission for
the STs to investigate all matters relating to the Constitutional safeguards for the STs
and to report to him (Article 338-A). The President should place all such reports
before the Parliament, along with the action taken memorandum. Previously, the
Constitution provided for a combined National Commission for SCs and STs. The
89th Amendment Act of 2003 bifurcated the combined commission into two separate
bodies.
The National Commission for SCs is also required to discharge similar functions with
regard to the OBCs and the Anglo-Indian Community as it does with respect to the
SCs. In other words, the commission has to investigate all matters relating to the
Constitutional and other legal safeguards for the OBCs and the Anglo-Indian
community and report to the President upon their working.

3. Solution: d)
The Election Commission registers political parties for the purpose of elections and
grants them recognition as national or state parties on the basis of their poll
performance. The other parties are simply declared as registered-unrecognised
parties.
The recognition granted by the Commission to the parties determines their right to
certain privileges like allocation of the party symbols, provision of time for political
broadcasts on the state-owned television and radio stations and access to electoral
rolls.

INSIGHTS MOCK TEST - 21


SOLUTIONS

WWW.INSIGHTSONINDIA.COM

4. Solution: c)
The Constitution (Article 324) provides for an independent Election Commission in
order to ensure free and fair elections in the country.
There is to be only one general electoral roll for every territorial constituency for
election to the Parliament and the state legislatures. Thus, the Constitution has
abolished the system of communal representation and separate electorates which led
to the partition of the country.
The state legislatures can also make provision with respect to all matters relating to
elections to the state legislatures including the preparation of electoral rolls and all
other matters necessary for securing their due constitution. But, they can make
provision for only those matters which are not covered by the Parliament. In other
words, they can only supplement the parliamentary law and cannot override it.

5. Solution: d)
The Chief Electoral Officer of a state/ Union Territory is authorised to supervise the
election work in the state/Union Territory subject to the overall superintendence,
direction and control of the Election Commission. The Election Commission of India
nominates or designates an Officer of the Government of the state / Union Territory
as the Chief Electoral Officer in consultation with that State Government / Union
Territory Administration.
The Election Commission of India nominates officers of Government as Observers
(General Observers and Election Expenditure Observers) for Parliamentary and
assembly constituencies. They perform such functions as are entrusted to them by
the Commission. They report directly to the Commission.
Refer to the section on election machinery in Chapter-65, Election in Laxmikanth.

6. Solution: a)
The Commission normally announces the schedule of elections in a major press
conference a few weeks before the formal process is set in motion. The Model Code of
Conduct for guidance of candidates and political parties comes immediately into
effect after such announcement.

INSIGHTS MOCK TEST - 21


SOLUTIONS

WWW.INSIGHTSONINDIA.COM

The formal process for the elections starts with the Notification or Notifications
calling upon the electorate to elect Members of a House. As soon as Notifications are
issued, candidates can start filing their nominations in the constituencies from where
they wish to contest. These are scrutinised by the Returning Officer of the
constituency concerned after the last date for the same is over after about a week.
The validly nominated candidates can withdraw from the contest within two days
from the date of scrutiny. Contesting candidates get at least two weeks for political
campaign before the actual date of poll.

7. Solution: c)
In 2003, the election Commission issued an order directing every candidate seeking
election to the Parliament or a State Legislature to furnish on his nomination paper
the information on the following matters.
(i) Whether the candidate has been convicted or acquitted or discharged in any
criminal offence in the past? Whether he/she was imprisoned or fined?
(ii) Prior to six months of filing nomination, whether the candidate is accused in any
pending case, of any offence punishable with imprisonment for two years or more,
and in which charges were framed or cognizance was taken by a court; if so, the
details thereof
(iii) The assets (immovable, movable, bank balances, etc.) of a candidate and his/her
spouse and that of dependents
(iv) Liabilities, if any, particularly whether there are any dues of any public financial
institution or government dues
(v) The educational qualifications of the candidate
Furnishing of any false information in the affidavit is now an electoral offence
punishable with imprisonment upto six months or fine or both.

8. Solution: d)
In 2003, the following two changes were introduced with respect to elections to the
Rajya Sabha:
(i) Domicile or residency requirement of a candidate contesting an election to the
Rajya Sabha was removed. Prior to this, a candidate had to be an elector in the state

INSIGHTS MOCK TEST - 21


SOLUTIONS

WWW.INSIGHTSONINDIA.COM

from where he was to be elected. Now, it would be sufficient if he is an elector in any


parliamentary constituency in the country.
(ii) Introducing open ballot system, instead of secret ballot system, for elections to
the Rajya
Sabha. This was done to curb cross-voting and to wipe out the role of money power
during Rajya Sabha elections. Under the new system, an elector belonging to a
political party has to show the ballot paper after marking his vote to a nominated
agent of that political party.

9. Solution: d)
Go through first few paragraphs only.
http://eci.nic.in/archive/instruction/recent/media/pnxitpoll_FINAL.html

10.

Solution: a)

A member of a House belonging to any political party becomes disqualified for being
a member of the House, (a) if he voluntarily gives up his membership of such
political party; or (b) if he votes or abstains from voting in such House contrary to
any direction issued by his political party without obtaining prior permission of such
party and such act has not been condoned by the party within 15 days.
The above disqualification on the ground of defection does not apply in the following
two cases:

If a member goes out of his party as a result of a merger of the party with
another party. A merger takes place when two-thirds of the members of the
party have agreed to such merger.
If a member, after being elected as the presiding officer of the House,
voluntarily gives up the membership of his party or rejoins it after he ceases to
hold that office. This exemption has been provided in view of the dignity and
impartiality of this office.

INSIGHTS MOCK TEST - 21


SOLUTIONS

WWW.INSIGHTSONINDIA.COM

11. Solution: a)
Any question regarding disqualification arising out of defection is to be decided by
the presiding officer of the House. Originally, the act provided that the decision of
the presiding officer is final and cannot be questioned in any court. However, in
Kihoto Hollohan case (1993), the Supreme Court declared this provision as
unconstitutional on the ground that it seeks to take away the jurisdiction of the
Supreme Court and the high courts.
It held that the presiding officer, while deciding a question under the Tenth
Schedule, function as a tribunal. Hence, his decision like that of any other tribunal, is
subject to judicial review on the grounds of mala fides, perversity, etc. But, the court
rejected the contention that the vesting of adjudicatory powers in the presiding
officer is by itself invalid on the ground of political bias.

12. Solution: d)
Nuclear retaliatory attacks can only be authorised by the civilian political leadership
through the Nuclear Command Authority.
The Nuclear Command Authority comprises a Political Council and an Executive
Council. The
Political Council is chaired by the Prime Minister. It is the sole body which can
authorise the use of nuclear weapons.
The Executive Council is chaired by the National Security Advisor. It provides inputs
for decision making by the Nuclear Command Authority and executes the directives
given to it by the Political Council.

13. Solution: b)
From the 7th number, here is the table of precedence.
The Table of Precedence is related to the rank and order of the officials of the Union
and State Governments.
7. Cabinet Ministers of the Union
Chief Ministers of States within their respective States
Deputy Chairman, Planning Commission

INSIGHTS MOCK TEST - 21


SOLUTIONS

WWW.INSIGHTSONINDIA.COM

Former Prime Ministers


Leaders of Opposition in Rajya Sabha and Lok Sabha

7A. Holders of Bharat Ratna decoration

8. Ambassadors Extraordinary and Plenipotentiary and High Commissioners of


Commonwealth countries accredited to India
Chief Ministers of States outside their respective States
Governors of States outside their respective States

9. Judges of Supreme Court

9A. Chairperson, Union Public Service Commission


Chief Election Commissioner
Comptroller & Auditor General of India

10. Deputy Chairman, Rajya Sabha


Deputy Chief Ministers of States
Deputy Speaker, Lok Sabha
Members of the Planning Commission
Ministers of State of the Union (and any other Minister in the Ministry of Defence
for defence matters)

11. Attorney General of India


Cabinet Secretary
Lieutenant Governors within their respective Union Territories.

INSIGHTS MOCK TEST - 21


SOLUTIONS

WWW.INSIGHTSONINDIA.COM

14. Solution: d)
Anglo-Indian means a person whose father or any of whose other male progenitors
in the male line is or was of European descent but who is domiciled within the
territory of India and is or was born within such territory of parents habitually
resident therein and not established there for temporary purposes only.
Indian State means any territory which the Government of the Dominion of India
recognized as such a State.
Ruler means the Prince, Chief or other person who, at any time before the
commencement of the Constitution (Twenty-sixth Amendment) Act, 1971, was
recognised by the President as the Ruler of an Indian State or any person who, at any
time before such commencement, was recognised by the President as the successor
of such Ruler.

15. Solution: d)
Individual islands lie just outside the continent, very much associated with the
characteristic features of the mainland of which they were once part.
Some other examples are Sri Lanka separated by Palk strait; Formosa by Formosa
strait.
Archipelago or island groups are different from such kind of islands.

16. Solution: d)
http://coral.org/coral-reefs-101/coral-reef-ecology/what-do-coral-reefs-need-tosurvive/
http://www.marinebio.net/marinescience/04benthon/crform.htm

INSIGHTS MOCK TEST - 21


SOLUTIONS

WWW.INSIGHTSONINDIA.COM

17. Solution: d)
Their shallowness allows the sunlight to penetrate the water which encourages the
growth of minute plants and microorganisms; hence, planktons, hence rich in fishes.
Their limited depth and gentle slope keep out cold currents and increase the height
of tides.
http://en.wikipedia.org/wiki/Continental_shelf#Economic_significance

18.Solution: d)
High rate of evaporation would take away water and leave more and more
concentration of salt in the ocean.
If continuously evaporated water is not mixed by currents, its salinity tends to
increase, thus becoming more denser.
Lesser precipitation ensures lesser inflow of fresh water and more salinity.

19. Solution: d)
http://en.wikipedia.org/wiki/Norwegian_Current#Effects_on_climate

20.

Solution: d)

Refer to the section on Movement of Ocean water in Goh Che Leong Chapter 12. It
has been explained in detail.
http://oceanexplorer.noaa.gov/facts/currents.html

INSIGHTS MOCK TEST - 21


SOLUTIONS

WWW.INSIGHTSONINDIA.COM

21. Solution: c)
It is due to Monsoon winds. In summer, June-October, when the dominant direction
is South-west Monsoon, the direction is south-westerly. In winter, its in the Northeastern direction.
This signifies the dominant effect of planetary winds on the circulation of ocean
currents.

22.

Solution: a)

The Intergovernmental Panel on Climate Change (IPCC) projections of increased


temperature and precipitation suggest the emergence of more disease-friendly
conditions in regions that did not previously host diseases or disease
carriers. Climate change accelerates the spread of disease primarily because warmer
global temperatures enlarge the geographic range in which disease-carrying animals,
insects and microorganisms--as well as the germs and viruses they carry--can
survive.
In addition to changing weather patterns, climatic conditions affect diseases
transmitted via vectors such as mosquitoes (vector-borne disease) or through
rodents (rodent-borne disease).

23.

Solution: a)

A barometer used to measure pressure and some aspects of the local weather is
affected by the altitude. The same column of water would read half if it is raised from
sea level to a height of 3.5 miles above the sea level.
Similarily, gravitational forces also affect the overall weight of the air and objects,
thus changing the pressure rating on the barometer.
http://weather.about.com/od/weatherinstruments/a/barometers.htm

24.

Solution: a)

INSIGHTS MOCK TEST - 21


25.

SOLUTIONS

WWW.INSIGHTSONINDIA.COM

Solution: b)

http://www.britannica.com/EBchecked/topic/135016/continentality
The effect of continentality may be moderated by proximity to the ocean, depending
on the direction and strength of the prevailing winds. Contrast with ocean
temperatures at the edges of each continent may be further modified by the presence
of a north- or south-flowing ocean current. For most latitudes, however,
continentality explains much of the variation in average temperature at a fixed
latitude as well as variations in the difference between January and July
temperatures.

26.

Solution: d)

Temperature inversion, is a condition in which the temperature of the atmosphere


increases with altitude in contrast to the normal decrease with altitude. When
temperature inversion occurs, cold air underlies warmer air at higher altitudes.
Temperature inversion may occur during the passage of a cold front or result from
the invasion of sea air by a cooler onshore breeze.

INSIGHTS MOCK TEST - 21


SOLUTIONS

WWW.INSIGHTSONINDIA.COM

Overnight radiative cooling of surface air often results in a nocturnal temperature


inversion that is dissipated after sunrise by the warming of air near the ground. A
more long-lived temperature inversion accompanies the dynamics of the large highpressure systems depicted on weather maps. Descending currents of air near the
center of the high-pressure system produce a warming (by adiabatic compression),
causing air at middle altitudes to become warmer than the surface air. Rising
currents of cool air lose their buoyancy and are thereby inhibited from rising further
when they reach the warmer, less dense air in the upper layers of a temperature
inversion.

27.

Solution: a)

A more long-lived temperature inversion accompanies the dynamics of the large


high-pressure systems depicted on weather maps. Descending currents of air near
the center of the high-pressure system produce a warming (by adiabatic
compression), causing air at middle altitudes to become warmer than the surface air.
Rising currents of cool air lose their buoyancy and are thereby inhibited from rising
further when they reach the warmer, less dense air in the upper layers of a
temperature inversion.
During a temperature inversion, air pollution released into the atmosphere's lowest
layer is trapped there and can be removed only by strong horizontal winds. Because
high-pressure systems often combine temperature inversion conditions and low wind
speeds, their long residency over an industrial area usually results in episodes of
severe smog.

28.

Solution: a)

INSIGHTS MOCK TEST - 21


29.

Solution: a)

30.

Solution: a)

SOLUTIONS

WWW.INSIGHTSONINDIA.COM

INSIGHTS MOCK TEST - 21


SOLUTIONS

WWW.INSIGHTSONINDIA.COM

Coriolis force is responsible for the deflection of winds in the Northern hemisphere
to the right.
For explanation of anti-cyclones in summer visit
http://www.lordgrey.org.uk/~f014/usefulresources/aric/Resources/Teaching_Packs
/Key_Stage_4/Weather_Climate/07.html
Cyclones occur in temperate zones too, wherever the conditions are met. An example
would be the hurricanes US faces.

31. Solution: c)
http://top5ofanything.com/list/7738a992/Rubber-Producing-Countries
http://en.wikipedia.org/wiki/Natural_rubber#Discovery_of_commercial_potential
Rubber plantations are not found in temperate regions. Right conditions do not exist
there.

32.

Solution: a)

Vedic ritual is preserved in literary texts called the Brahmanas. The main division of
the contexts of these extensive texts is twofold the ritualistic injuction and
discussions on the meaning of Vedic ritual and all that is related to it. The Aranyakas
or the treatises of the forest present secret explanations of the ritual, have their
origin in the philosophical discussions of the Brahmanas, find their culmination in
the Upanishads and represent the transitional phase between the ritualistic
symbolism of the Brahmanas and the philosophical doctrines of the Upanishads. The
Upanishads, written both in prose and poetry, are expressions of philosophical
concepts.
In the literal term, it means that knowledge which is imparted to the student who is
sitting very near to the teacher. That knowledge by which all ignorance is destroyed.
The knowledge of the identity of the self (Atman) with the eternal (Brahman). The
Upanishads are the end of the Vedas. This is the literature in which ancient sages
realised that in the final analysis, man has to know himself.

INSIGHTS MOCK TEST - 21


33.

SOLUTIONS

WWW.INSIGHTSONINDIA.COM

Solution: d)

The great epics (Mahakavya), the Ramayana (1500 BC) and the Mahabharata (1000
BC) are the, repositories of the ethnic memory of the Indian people. Valmiki, the poet
of the Ramayana, is known as Adikavi (first among the poets), and the story of Rama
is occasionally referred to in the Mahabharata. But both these epics were composed
over a long passage of time, not by one poet, but by many poets, for the purpose of
oral transmission by singers and story tellers. Both are epics of the people, and as
such, reflect the ethos and the psyche of a group of people, not only in a given
temporal frame, but have a universal human context. The Ramayana tells us about
how a man can achieve divineness, as Rama achieves divinity through righteous
action.
It also tells us about how to achieve the fourfold objectives (Purushartha) of human
life, Dharma (righteousness, or loosely, religion), Artha (worldly achievement,
mainly wealth and prosperity), Kama (fulfilment of all desires), and Moksha
(liberation). Inwardly it is a quest to know oneself.

34.

Solution: d)

The main Puranas are 18 encyclopaedic collections of legend and myth. Though the
archaic form of the genre might have existed as early as the fourth or the fifty century
B.C., the famous names of the 18 Mahapuranas were not discovered earlier than the
third century A.D. The phenomenal popularity of these Mahapuranas gave rise to yet
another sub-genre known as the Upapuranas or minor Puranas. They are also 19 in
number.
The Mahapuranas have five subjects. These are : (1) Sarga, the original creation of
the universe, (2) Pratisarga, the periodical process of destruction and re-creation, (3)
Manvantara, the different eras or cosmic cycles, (4) Surya Vamsha and Chandra
Vamsa, the histories of the solar and lunar dynasties of Gods and sages, (5)
Vamshanucharita, the genealogies of kings. Around this core skeleton of the five
subjects any Purana adds other diverse materials like matters of religious concern,
customs, ceremonies, sacrifices, festivals, the duties of various castes, different types
of donations, details of the construction of temples and images, and descriptions of
places of pilgrimage. The Puranas are the meeting point of diverse religious and
social beliefs, are linked with the vital spiritual and social needs and urges of the
people, and are a unique outcome of the ever-continuing synthesis based on an
understanding between various groups of vedic Aryans and non-Aryans.

INSIGHTS MOCK TEST - 21


35.

SOLUTIONS

WWW.INSIGHTSONINDIA.COM

Solution: d)

Buddha (500 B.C.) used Pali to give his sermons. All the Buddhist canonical
literature is in Pali which includes Tipitaka (threefold basket). The first basket,
Vinaya Pitaka, contains the monastic rules of the Order of Buddhist monks. The
second basket, Sutta Pitaka, is the collection of the speeches and dialogues of the
Buddha. The third basket, the Abhidhamma Pitaka, elucidates the various topics
dealing with ethics, psychology or theory of knowledge. The jataka Kathas are noncanonical Buddhist literature in which stories relating to the former births of the
Buddha (Bodhi-sattva or the would-be Buddha) are narrated.
These stories propagate Buddhist religious doctrines and are available in both
Sanskrit and Pali. As the jataka tales grew in bulk, they assimilated popular tales,
ancient mythology, stories from older religious traditions, etc. Jatakas are, in fact,
based on the common heritage of the Indian masses. Buddhist literature is also
abundantly available in Sanskrit, which includes the great epic Buddhacharita by
Aswaghosha (78 A.D.).

36.

Solution: a)

Dravidian literature mainly consists of the four languages, Tamil, Telugu, Kannada
and Malayalam. Early classical Tamil literature is known as Sangam literature
meaning fraternity, indicating mainly two schools of poets, aham (subjective love
poems), and puram (objective, public poetry and heroic). Aham deals purely with the
subjective emotions of the lover, and puram with all kinds of emotions, mainly the
valour and glory of kings, and about good and evil.
The Tamils, in all their 2000 years of literary effort, wrote nothing better. The
famous Thirukkural by Thiruvalluvar, in the 6th century A.D., serves as a manual of
precepts to guide one to noble living. It expounds a secular, moral and practical
attitude towards life. The twin epics, Silappadhikaram (the story of the anklet),
written by Ilango-Adigal, and Manimekalai (the story of Manimekalai) by Chattanar,
were written sometimes in A.D. 200-300 and give vivid accounts of Tamil society
during that period. These are valuable storehouses and epics of dignity and
sublimity, laying stress on the cardinal virtues of life. In Manimekalai there is an
elaborate exposition of the doctrines of Buddhism. If Tamil reveals a triumph of
Brahmanic and Buddhist knowledge, Kannada shows Jain ascendency in its ancient
phase. Malayalam absorbed a rich treasure contained in the Sanskrit language.
Nannaya (A.D.1100) was the first Telugu poet. In ancient times, Tamil and Telugu
spread to distant places.

INSIGHTS MOCK TEST - 21


SOLUTIONS

WWW.INSIGHTSONINDIA.COM

If one were to identify another striking feature of ancient Tamil literature, the
obvious choice would be Vaishnava (pertaining to Vishnu) bhakti (devotional)
literature. In Indian literature the effort has been to find out how a man can achieve
divinity.

37.

Solution: d)

The religious and cultural synthesis that was a special feature of medieval India finds
abundant expression in its literature. The Islamic element is all-pervasive, next only
to the Upanishadic Hindu element. Nanak, the first Sikh Guru, wrote in many
languages, but mostly in Punjabi, and was a great poet of inter-religious
communication. Nanak says truth is supreme, but above truth is truthful living. Guru
Nanak and other Sikh Gurus belong to the sant tradition, which believes in one
omnipresent God, and not in many gods like Rama and Krishna.
The poetry of the Sikh Gurus is collected in the Guru Granth Sahib (the Revered
Book), a multilingual text which talks about the unchanging one reality (Sat) the
cosmic law (Hukum), meditation (Satnam), compassion and harmony (Daya and
Santosh). Bulleh Shah, the most famous Muslim Punjabi poet, popularised Sufism
through Punjabi Kafi (verse-form). Kafi is a small poem in stanzas followed by
refrain and is sung in a dramatic way. Shah Latif, the famous Sindhi Muslim poet
(1689 A.D.) in his sacred work Risalo explained Sufi mystic love as the divine truth.

38.

Solution: a)

Unlike the dark middle ages of Europe, Indias middle ages brought about a very rich
tradition of devotional literature of remarkable merit which dispels the superstitious
assumption of a dark period of Indias history. Bhakti literature is the most
important development of the medieval period. It is love poetry. Love for ones Lord,
Krishna or Rama, the two main incarnations of the great God Vishnu. This love is
depicted as love between husband and wife, or between lovers, or between servant
and master, or between parents and child. This is personalisation of the godhood,
which means a truthful perception of God residing in you, and also harmony in life
which only love can bring.
Worldly love is Kama (Eros) and divine love is Prema (mystic Eros). The dominating
note in bhakti is ecstasy and total identity with God. It is a poetic approach to
religion and an ascetic approach to poetry.

INSIGHTS MOCK TEST - 21


39.

SOLUTIONS

WWW.INSIGHTSONINDIA.COM

Solution: d)

The conception of bhakti did away with the elite tradition of Sanskrit and accepted
the more acceptable language of the common man. Kabir (Hindi) says that Sanskrit
is like water of a well stagnant, Bhasa like flowing water. A seventh century Shaiva
Tamil writer Manikkarvachakar has something similar to say about in his book of
poetry Thiruvachakam. Bhakti also attacked the age-old caste system and devoted
itself to the worship of humanity, because the catch-word of bhakti is that God is
there in every human being. The movement was in essence subaltern, as most of its
poets belonged to the so-called lower castes. Bhakti is antitheology and against any
kind of conceptual erudition.

40.

Solution: a)

http://en.wikipedia.org/wiki/Sufism
http://en.wikipedia.org/wiki/Sufism#Dhikr

41. Solution: b)
The practice of muraqaba can be likened to the practices of meditation attested in
many faith communities. The word muraqaba is derived from the same root (r-q-b)
occurring as one of the 99 Names of God in the Qur'an, al-Raqb, meaning "the
Vigilant" and attested in the Qur'an. Through muraqaba, a person watches over or
takes care of the spiritual heart, acquires knowledge about it, and becomes attuned to
the Divine Presence, which is ever vigilant.
While variation exists, one description of the practice within a Naqshbandi lineage
reads as follows:
He is to collect all of his bodily senses in concentration, and to cut himself off from
all preoccupation and notions that inflict themselves upon the heart. And thus he is
to turn his full consciousness towards God Most High while saying three times: "Ilah
anta maqsd wa-ridka matlbmy God, you are my Goal and Your good
pleasure is what I seek". Then he brings to his heart the Name of the Essence
Allhand as it courses through his heart he remains attentive to its meaning, which
is "Essence without likeness". The seeker remains aware that He is Present,
Watchful, Encompassing of all, thereby exemplifying the meaning of his saying (may
God bless him and grant him peace): "Worship God as though you see Him, for if you

INSIGHTS MOCK TEST - 21


SOLUTIONS

WWW.INSIGHTSONINDIA.COM

do not see Him, He sees you". And likewise the prophetic tradition: "The most
favored level of faith is to know that God is witness over you, wherever you may be".

42.

Solution: c)

http://www.socialprogressimperative.org/data/spi
India has been ranked lower than Nepal in 2015.

43.

Solution: c)

Minister of State for Textiles (I/C), Shri Santosh Kumar Gangwar launched a scheme
to promote usage of Geotechnical Textiles in the North Eastern Region of India.
The ultimate objective is to introduce modern cost-effective technology pertaining to
the application and usage of geotechnical textiles in the development of
infrastructure of NE Region, which is likely to boost the regions economy.
Geotechnical Textiles is a proven technology world over. Geotechnical Textiles
provide the functional advantages of higher endurance and durability in roads and
infrastructure projects. Most of the developed countries have established empirical
evidence about these advantages through numerous studies as well as onsite trials. In
consideration of these benefits, Geotechnical Textiles have now become a way of life
in the developed parts of the world.
Geotechnical textile products are used for work related soil, rock e.g. laying down
roads and pavements in hilly areas, slope stabilization, protecting river
embankments-tunnels-rail-track etc; Marine Engineering Soil Erosion landfills and
waste management. Benefits of Geotextile products are less maintenance; need less
space; and more durability; ground stabilization etc. India is a leading producer of
geo-technical textiles.

44.

Solution: b)

Law Commission- Justice A.P. Shah: 255th report about electoral reforms

ban newspaper advertisements on the eve of elections be banned

INSIGHTS MOCK TEST - 21


SOLUTIONS

WWW.INSIGHTSONINDIA.COM

ban Independent candidates from contesting elections because mostly theyre


dummy candidates with no chances of winning, they only confuse the voters
and split vote share.
if any candidate indulges in paid news- disqualify him
ban election related broadcast from both print and electronic media for 48
hours before election
need to regulate exit polls and bring more transparency in their methodology.
make it compulsory for parties to declare all donations, even if less than
Rs.20,000, if total 20 crores or 20% of party contribution comes from such
small amounts. Upload all such data on ECI website, penalty for nondisclosure
need collegium / selection Committee to pick Election commissioners. It
should include PM, LoP in Lok Sabha/leader of largest opposition party and
CJI.
did not favor compulsory voting model of Gujarat and NGOs demand for right
to recall- right to proportional representation or even state funding of
election. However, indirect subsidies in-kind may be given to political parties.
introduce common electoral rolls for parliament, Vidhan Sabha and local
assembly elections by amending the relevant acts.
Must use EVM totalizer to prevent voter harassment after results.
setup more election benches in high court and they should pass order within
90 days after argument conclusion.
Amend the 10th schedule about anti-defection. The disqualification decision
should be made by President /governor with help of ECI, instead of
Speaker/chairman.
ECI should be given separate independent Secretariat.
if candidate fails to give proper record of election Expenditures, debar him for
5 years, instead of present 3 years.

45.

Solution: c)

India on 9 April 2015 successfully test fired nuclear weapons-capable Dhanush


missile from a ship, off the Odisha coast.Indigenously developed by Defence
Research and Development Organisation (DRDO).
The surface-to-surface missile is a naval variant of Prithvi missile. Capable of
carrying of 500 to 1000 kilogram conventional as well as nuclear payload. It is a
single-stage, liquid-propelled missile and has range of 350 km. Capable to target
both land-based and sea-based targets.

INSIGHTS MOCK TEST - 21


SOLUTIONS

WWW.INSIGHTSONINDIA.COM

It gives Indian Navy capability to strike enemy targets with great precision. It should
be noted that Dhanush is one of the 5 missiles developed by the DRDO under the
Integrated Guided Missile Development Programme (IGMDP).

46.

Solution: c)

IRENA is an intergovernmental organization to promote adoption and sustainable


use of renewable energy. It is the global hub for renewable energy cooperation and
information exchange within its member nations. Founded in 2009 and its statute
entered into force on 8 July 2010. Headquarters is in Abu Dhabi.
Two governing structures are present- IRENA Assembly which takes decisions at
macro level and gives policy guidance. IRENA Council is main governing body and is
responsible for implementing various decisions of assembly. Members - In total it
has 140 Members. India is one of the 33 Permanent and founder members of IRENA

47.

Solution: c)

Buddha / Malabar Peacock butterfly is endemic to South Asia. Remaining varities of


peacock butterflies found in West Bengal. Plus a new Malayan Green Banded
Peacock butterfly (Papilio palinurus) also discovered in W.Bengal recently. This
butterfly is also found in Mynmar, Borneo and Philippines.
Sarus Crane is the official bird of UP state. NGT orders declaring Dhanauri and
Parasaul as wetland to protect this species.
https://www.savingcranes.org/sarus-crane.html

48.

Solution: b)

Union government has suspended the registration of Greenpeace India, a non-profit


environmental organization for six months with immediate effect. Decision in this
regard was taken by Union Ministry of Home Affairs (MHA) after Greenpeace India
had prejudicially acted against the public interests and economic interests of the
country. As per MHA, Greenpeace India has used its funds received from foreign
sources for unauthorised purposes and has violated the rules mentioned Foreign
Contribution (Regulation) Act (FCRA), 2010. If Greenpeace India fails to make a

INSIGHTS MOCK TEST - 21


SOLUTIONS

WWW.INSIGHTSONINDIA.COM

satisfactory presentation to the Union government in this regard, its licence may be
cancelled permanently.

49.

Solution: b)

The Khadi and Village Industries Commission (KVIC) is a statutory body formed by
the Government of India, under the Act of Parliament, 'Khadi and Village Industries
Commission Act of 1956'. It is an apex organization under the Ministry of Micro,
Small and Medium Enterprises, with regard to khadi and village industries within
India, which seeks to - "plan, promote, facilitate, organise and assist in the
establishment and development of khadi and village industries in the rural areas in
coordination with other agencies engaged in rural development wherever
necessary".http://en.wikipedia.org/wiki/Khadi_and_Village_Industries_Commission -
cite_note-2 In April 1957, it took over the work of former All India Khadi and Village
Industries Board.http://en.wikipedia.org/wiki/Khadi_and_Village_Industries_Commission -
cite_note-3

Its head office is based in Mumbai, with its six zonal offices Delhi, Bhopal,
Bangalore, Kolkata, Mumbai and Guwahati. Other than its zonal offices, it has
offices in 29 states for the implementation of its various programs.

50.

Solution: d)

Rail budget 2015 announced Kayakulp council to transform Indian railways. Ratan
Tata will head this body.
http://timesofindia.indiatimes.com/india/Ratan-Tata-to-head-railways-Kaya-Kalpcouncil/articleshow/46623915.cms

51. Solution: c)
http://nlrd.org/resources-womens-rights/acid-attacks/acid-attacks-latestjudgements-of-supreme-courthigh-courts/supreme-court-order-on-acid-attacks
http://timesofindia.indiatimes.com/india/Treat-acid-attack-victims-for-freeSupreme-Court-orders-private-hospitals/articleshow/46884560.cms

INSIGHTS MOCK TEST - 21


52.

SOLUTIONS

WWW.INSIGHTSONINDIA.COM

Solution: a)

Chestnut-breasted Partridge (Arborophila mandellii) - This bird is endemic to the


Eastern Himalayas. IUCN status is Vulnerable. It is in news because after a long
time, photograph of the real bird has been taken in Arunanchal Pradesh.
Moreover, Osmania University researchers have found new species of spiders Telangana crab spider (Thomisus telanganensis) - at Nagnur in Karimnagar district.
They are called Crab spiders because they walk like crabs. They are also called Flower
spiders because they dont construct webs. They wait in flowers to catch insects by
venom sting. They benefit ecosystem by working as biocontrol agents against insects.

53.

Solution: a)

Radar uses high frequency radio waves. Theyre electromagnetic waves, can travel
long distance in air because theyre not much absorbed by air molecules. However,
Radar sent in water, then it can hardly travel beyond meter because water will absorb
those electromagnetic waves. Therefore, we cant use radar to track objects in water.
On the other hand, Sonar uses high frequency sound waves. Theyre mechanical
waves, they dont lose energy even after traveling long distance in water. So we use
Sonar to track objects in water.

54.

Solution: c)

http://en.wikipedia.org/wiki/Net_neutrality
TRAI consultation paper proposes that Viber, Whatsapp etc. are Over-the-Top (OTT)
Services. So then TRAI may try to regulate them by issuing license similar to
spectrum. This will kill startup companies and innovation. Ignored because its still a
consultation paper and no final verdict is reached

55.

Solution: d)

Project 75 aims to develop submarine with joint India-France collaboration.


http://www.globalsecurity.org/military/world/india/project-75.htm

INSIGHTS MOCK TEST - 21


SOLUTIONS

WWW.INSIGHTSONINDIA.COM

https://www.defenseindustrydaily.com/india-to-sign-multibillion-dollar-scorpenesub-contract-updated-01194/

56.

Solution: d)

http://www.tribuneindia.com/news/canada-to-supply-uranium-to-india-for-nextfive-years/67761.html\

57.Solution: d)
Union Government on 14 April 2015 constituted a task force on interlinking of rivers
in the country. The Task Force will be chaired by former Secretary of Ministry of
Water Resources B N Navalawala and comprises experts and senior officials.
Terms of References of Task Force

Take up all issues related to expediting the work on interlinking of rivers in


the country.
Consider alternative plans in case of infeasible links in the present National
Perspective Plan (NPP) apart from examining the existing links which are laid
out as per the NPP under both Himalayan and Peninsular components.
Facilitate interlinking of intra-basin and intra-state Rivers, along with that of
inter-state and inter-basin links.
Recommend time schedules for completing feasibility studies of all the river
links along with implementation schedule.
Suggest different means of funding mechanisms for the interlinking of rivers
project.
Provide guidance norms of appraisal for individual projects in respect of
economic viability, environmental impacts, socio-economic and preparation
of resettlement plans.
Device suitable mechanisms in order to bring speedy consensus among the
states. To implement the interlinking of Rivers, propose suitable
organizational structure.

58.

Solution: c)

Bhakti was not the only aspect of medieval literature. Love ballads and heroic poetry
in Punjabi, known as Kissa and Var, were popular Punjabi medieval forms. The most
famous Punjabi love ballad is Hir Ranjha, an immortal book by a Muslim poet called

INSIGHTS MOCK TEST - 21


SOLUTIONS

WWW.INSIGHTSONINDIA.COM

Warris Shah. A popular Punjabi heroic ballad, sung by village bards orally, is
Najabats Var of Nadir Shah. Var is the most popular form of Punjabi poetry, music
and drama, all rolled into one, and has been in vogue since the earliest times. In
Hindi, between 1700 and 1800 A.D., many poets like Bihari Lal and Keshav Das
created secular poetry of Sringara (erotic sentiment), and a large number of other
poets, wrote academic accounts of the entire range of poetry, in verse form.
During the medieval period, Urdu, as a language, came into being. It was Amir
Khusro (1253 A.D.), an early architect of Indias composite culture, and a great Sufi
poet, who first experimented with Persian and Hindi (then known as Hindavi) mixed
poetry, which was the genesis of a new language, subsequently recognised as Urdu.

59.

Solution: a)

The birth of the novel is associated with the social reform-oriented movement of the
19th century. This new genre, borrowed from the West, is characterized by a spirit of
revolt, right from its adoption into the Indian system.
The first Tamil novel, Pratap Mudaliyar Charitram (1879) by Samuel V. Pillai, the
first Telugu novel, Sri Ranga Raja Charitra (1872) by Krishnamma Chetty, and the
first Malayalam novel, Indu Lekha (1889) by Chandu Menon were written with
didactic intentions and to re-examine evil social customs and practices like
untouchability, caste distinctions, denial of remarriage of widows, etc.
In other first novels, like the Bengali novel, Phulmani O Karunar Bibaran (1852), by
an Englishwoman, H. Catherine Mullens, or the Hindi novel, Pariksha Guru (1882)
by Lala Sriniwas Das, one can discover shared patterns of response and articulation
towards social problems.

60.

Solution: c)

Historical novels were written by Bankim Chandra Chatterjee (Bengali), Hari


Narayan Apte (Marathi), and others, to describe the glorious past of India, and to
instill nationalist fervour in her people. Novels were found to be the most
appropriate medium to eulogize the intellectual and physical richness of the past,
and reminded Indians about their obligations and rights. In fact, in the 19th century,
the idea of national identity emerged from literature, and most Indian writings
turned into the voice of enlightment. This paved the way for India to understand the
real, factual position by the time it reached the threshold of the 20th century. It was
during this time that Tagore started writing the novel Gora (1910), to challenge
colonial rule, colonial criteria and colonial authority, and to give new meaning to
Indian nationalism.

INSIGHTS MOCK TEST - 21


SOLUTIONS

WWW.INSIGHTSONINDIA.COM

61. Solution: a)
Encouraged by the greater macro-economic stability and the reformist intent and
actions of the government, coupled with improved business sentiments in the
country, institutions like the IMF and the World Bank have presented an optimistic
growth outlook for India for the year 2015 and beyond.
The possible headwinds to such promising prospects, however, emanate from factors
like inadequate support from the global economy saddled with subdued demand
conditions, particularly in Europe and Japan, recent slowdown in China, and, on the
domestic front, from possible spill-overs of below normal agricultural growth and
challenges relating to the massive requirements of skill creation and infrastructural
upgradation. The encouraging results from the Advance Estimates for 2014-15
suggest that though the global sluggishness has partly fed into the lacklustre growth
in foreign trade; yet this downward pressure has been compensated by strong
domestic demand, keeping the growth momentum going.

62.

Solution: a)

INSIGHTS MOCK TEST - 21


63.

Solution: c)

64.

Solution: a)

SOLUTIONS

WWW.INSIGHTSONINDIA.COM

INSIGHTS MOCK TEST - 21


65.

SOLUTIONS

WWW.INSIGHTSONINDIA.COM

Solution: a)

Juxtaposing savings with investment, it becomes clear that it was the large savinginvestment gap of the consolidated public sector, complemented by a less
pronounced gap in the private corporate sector, which could not be fully defrayed by
the savings of households, that constituted the aggregate saving-investment gap. The
gap between domestic savings and domestic investment is definitionally equal to the
current account balance.
The general equation is that:
Investment Savings = Current account balance (i.e. exports imports etc.)
In view of the above, it is clear that household financial savings need to be raised to
keep the saving-investment gap at acceptable levels.

66.

Solution: d)

Gross fixed capital formation (GFCF) refers to the net increase in physical assets
(investment minus disposals) within the measurement period. It does not account
for the consumption (depreciation) of fixed capital, and also does not include land
purchases. It is a component of expenditure approach to calculating GDP.

INSIGHTS MOCK TEST - 21


67.

SOLUTIONS

WWW.INSIGHTSONINDIA.COM

Solution: d)

Global factors, namely persistent decline in crude prices, soft global prices of
tradables, particularly edible oils and even coal, helped moderate headline inflation.
The tight monetary policy was helpful in keeping the demand pressures contained,
creating a buffer against any external shock, and keeping volatility in the value of the
rupee under check. During the last one year, the rupee remained relatively stable vis-vis the major currencies, which too had sobering influence on inflation.
Moderation in wage rate growth reduced demand pressures on proteinbased items.
Base effect also contributed to the decline in headline inflation.

68.

Solution: a)

Over the last ten years, Indias merchandise trade (on customs basis) increased
manifold from US$ 195.1 billion in 2004-05 to US$ 764.6 billion in 2013-14 helping
Indias share in global exports and imports improve from 0.8 per cent and 1.0 per
cent respectively in 2004 to 1.7 per cent and 2.5 per cent in 2013.
Manufactured goods constituted the bulk of exportsover 63 per cent in recent
years followed by crude and petroleum products (including coal) with 20 per cent
share and agriculture and allied products with 13.7 per cent share.

69.

Solution: c)

With net capital flows remaining higher than the CAD, there was net accretion of
US$ 18.1 billion to Indias foreign exchange reserves (on BoP basis) in H1 of 2014- 15
as against a drawdown of US$ 10.7 billion in H1 of 2013-14. Among the major
economies with a CAD, India is the second largest foreign exchange reserve holder
after Brazil. Indias foreign exchange reserves at US$ 330.2 billion as on 6 February
2015 mainly comprised foreign currency assets amounting to US$ 305.0 billion,
accounting for about 92.5 per cent of the total.

70.

Solution: a)

INSIGHTS MOCK TEST - 21


SOLUTIONS

WWW.INSIGHTSONINDIA.COM

In the first half of 2014-15, Indias external-sector position was benign and
comfortable. Two important developments were: (i) lower trade deficit along with
moderate growth in invisibles that resulted in lower CAD and (ii) surge in capital
inflows, enabled by higher portfolio investment, foreign direct investment (FDI), and
external commercial borrowings (ECB).
Higher FDI or FII is directly credited to Indias foreign exchange reserves as it
bridges the CAD. Higher ECBs result in higher debts on the country, and not lead to
accretion in foreign exchange reserves.

71. Solution: d)
An external commercial borrowing (ECB) is an instrument used in India to
facilitate the access to foreign money by Indian corporations and PSUs (public
sector undertakings). ECBs include commercial bank loans, buyers' credit, suppliers'
credit, securitised instruments such as floating rate notes and fixed rate bonds etc.,
credit from official export credit agencies and commercial borrowings from the
private sector window of multilateral financial Institutions such as International
Finance Corporation (Washington), ADB, AFIC, CDC, etc. ECBs cannot be used for
investment in stock market or speculation in real estate.
The DEA (Department of Economic Affairs), Ministry of Finance, Government of
India along with Reserve Bank of India, monitors and regulates ECB guidelines and
policies. For infrastructure and greenfield projects, funding up to 50% (through ECB)
is allowed. In telecom sector too, up to 50% funding through ECBs is allowed.
Recently Government of India allowed borrowings in Chinese currency
yuan.Corporate sectors can mobilize USD 750 million via automatic route,whereas
service sectors and NGO's for microfinance can mobilize USD 200 million and 10
million respectively.
Borrowers can use 25 per cent of the ECB to repay rupee debt and the remaining 75
per cent should be used for new projects. A borrower cannot refinance its existing
rupee loan through ECB. The money raised through ECB is cheaper given near-zero
interest rates in the US and Europe, Indian companies can repay their existing
expensive loans from that.

72.

Solution: b)

INSIGHTS MOCK TEST - 21


SOLUTIONS

WWW.INSIGHTSONINDIA.COM

Indias external debt stock increased by US$ 13.7 billion (3.1 per cent) to US$ 455.9
billion at end-September 2014 over the end-March 2014 level. The rise in external
debt was on account of higher long-term debt particularly commercial borrowings
and non-resident Indian (NRI) deposits. The maturity profile of Indias external debt
indicates the dominance of long-term borrowings.
FDI does not lead to any debt on the country. It adds to the foreign exchange
reserves.
The case of ECBs has been explained in the previous few questions.

73.

Solution: d)

74.

Solution: a)

The broad sector-wise allocations of central Plan outlay (gross budgetary support in
central Plan plus internal and extra-budgetary resources of the CPSEs) indicate that
the energy, transport, social service, and industry and minerals, got the maximum
share in BE 2014-15.

INSIGHTS MOCK TEST - 21


75.Solution: a)

SOLUTIONS

WWW.INSIGHTSONINDIA.COM

INSIGHTS MOCK TEST - 21


76.

SOLUTIONS

WWW.INSIGHTSONINDIA.COM

Solution: a)

Quit India, Bharat Choro. This simple hut powerful slogan launched the
legendary struggle which also became famous by the name of the August
Revolution.) In this struggle, the common people of the country demonstrated an
unparalleled heroism and militancy. Moreover, the repression that they faced was
the most brutal that had ever been used against the national movement. The
circumstances in which the resistance was offered were also the most adverse faced
by the national movement until then using the justification of the war effort, the
Government had armed itself with draconian measures, and suppressed even basic
civil liberties. Virtually any political activity, however peaceful and legal, was at this
time an illegal and revolutionary activity.

77. Solution: a)
The failure of the Cripps Mission in April 1942 made it clear that Britain was
unwilling to offer an honourable settlement and a real constitutional advance during
the War, and that she was determined to continue Indias unwilling partnership in
the War efforts.
Combined with this was the impact of the manner of the British evacuation from
Malaya and Burma. It was common knowledge that the British had evacuated, the
white residents and generally left the subject people to their fate. /Letters from
Indians in South-East Asia to their relatives in India were full of graphic accounts of
British betrayal and their being left at the mercy of the dreaded Japanese.

78.

Solution: d)

Though Gandhiji himself had begun to talk of the coming struggle for some time
now, it was at the Working Committee meeting at Wardha on 14 July, 1942 that the
Congress first accepted the idea of a struggle. The All-India Congress Committee was
then to meet in Bombay in August to ratify this decision.
The historic August meeting at Gowalia Tank in Bombay was unprecedented in the
popular enthusiasm it generated. Huge crowds waited outside as the leaders
deliberated on the issue.

INSIGHTS MOCK TEST - 21


SOLUTIONS

WWW.INSIGHTSONINDIA.COM

And the feeling of anticipation and expectation ran so high that in the open session,
when the leaders made their speeches before the many thousands who had collected
to hear them, there was pin-drop silence.

79.

Solution: d)

Gandhijis speech also contained specific instructions for different sections of the
peop1e. Government servants would not yet be asked to resign, but they should
openly declare their allegiance to the Congress, soldiers were also not to leave their
posts, but they were to refuse to fire on our own people .
The Princes were asked to accept the sovereignty of your own people, instead of
paying homage to a foreign power. And the people of the Princely States were asked
to declare that they (were) part of the Indian nation and that they (would) accept the
leadership of the Princes, if the latter cast their lot with the People, but not
otherwise. Students were to give up studies if they were sure they could continue to
remain firm independence was achieved.
Gandhiji had placed the instructions he had drafted before the Waking Committee,
and in these he had proposed that peasants who have the courage, and are prepared
to risk their all should refuse to pay the land revenue. Tenants were told that the
Congress holds that the land belongs to those who work on it and to no one else.

80.

Solution: d)

The first one was proclaimed in Ballia, in East U P, in August 1942 under the
leadership of Chittu Pande, who called himself a Gandhian. Though it succeeded in
getting the Collector to hand over power and release all the arrested Congress
leaders, it could not survive for long and when the soldiers marched in, a week after
the parallel government was formed, they found that the leaders had fled.
In Tamluk in the Midnapur district of Bengal, the Jatiya Sarkar came into existence
on 17 December, 1942 and lasted till September 1944. Tamluk was an area where
Gandhian constructive work had made considerable headway and it was also the
scene of earlier mass struggles.
Satara, in Maharashtra, emerged as the base of the longest lasting and effective
parallel government. From the very beginning of the Quit India Movement, the
region played an active role. In the first phase from August 1942, there were marches

INSIGHTS MOCK TEST - 21


SOLUTIONS

WWW.INSIGHTSONINDIA.COM

on local government headquarters the ones on Karad, Tasgaon and Islampur


involving thousands.

81.Solution: d)
The idea of the INA was first conceived in Malaya by Mohan Singh, an Indian officer
of the British Indian Army, when he decided not to join the retreating British army
and instead went to the Japanese for help. The Japanese had till then only
encouraged civilian Indians to form anti-British organizations, but had no
conception of forming a military wing consisting of Indians.
Indian prisoners of war were handed over by the Japanese to Mohan Singh who then
tried to recruit them into an Indian National Army. The fall of Singapore was crucial,
for this brought 45,000 Indian POWs into Mohan Singhs sphere of influence. By the
end of 1942, forty thousand men expressed their willingness to join the INA.
The INA was also seen by many as a means of checking the misconduct of the
Japanese against Indians in South-East Asia and a bulwark against a future Japanese
occupation of India.
The outbreak of the Quit India Movement gave a fillip to the INA as well.
Anti-British demonstrations were organized in Malaya. On 1 September 1942 the
first division of the INA was formed with 16,300 men. The Japanese were by now
more amenable to the idea of an armed Indian wing because they were
contemplating an Indian invasion. But, by December 1942, serious differences
emerged between the Indian army officers led by Mohan Singh and the Japanese
over the role that the INA was to play. Mohan Singh and Niranjan Singh Gill, the
senior-most Indian officer to join the INA, were arrested. The Japanese, it turned
out, wanted only a token force of 2,000 men, while Mohan Singh wanted to raise an
Indian National Army of 20,000.

82.

Solution: a)

http://en.wikipedia.org/wiki/Battle_of_Imphal
One INA battalion commanded by Shah Nawaz was allowed to accompany the
Japanese Army to the Indo-Burma front and participate in the Imphal campaign. But
the discriminatory treatment which Included being denied rations, arms and being
made to do menial work for the Japanese units, completely demoralized the INA

INSIGHTS MOCK TEST - 21


SOLUTIONS

WWW.INSIGHTSONINDIA.COM

men. The failure of the Imphal campaign, and the steady Japanese retreat thereafter,
quashed any hopes of the INA liberating the nation.

83.

Solution: a)

The Tebhaga movement was an independence campaign initiated in Bengal by


the Kisan Sabha (peasants front of Communist Party of India) in 1946-1947. At that
time share-cropping peasants (essentially, tenants) had to give half of their harvest to
the owners of the land. The demand of the Tebhaga (sharing by thirds) movement
was to reduce the share given to landlords to one third.
In many areas the agitations turned violent, and landlords fled villages leaving parts
of the countryside in the hands of Kisan Sabha.
As a response to the agitations, the then Muslim League ministry in the province
launched the Bargadari Act, which provided that the share of the harvest given to the
landlords would be limited to one third of the total. But the law was not fully
implemented.

84.

Solution: d)

http://www.indianetzone.com/50/indian_women_warli_movement.htm

85.

Solution: b)

The leadership of the INM arrived at a clear and scientific understanding of


colonialism. The British were using political control to subordinate the Indian
economy and society to the needs of the British economy and society. This
understanding has given rise to certain ideological themes in the INM.
These themes came in because of the Drain of wealth from India due to the following,
plunder and taxation; Employment of Englishmen in India; Investing British capital
in India ; and Free and unequal trade.
This was the complex economic mechanism of modern imperialism.

INSIGHTS MOCK TEST - 21



SOLUTIONS

WWW.INSIGHTSONINDIA.COM

INSIGHTS ON INDIA MOCK PRELIMINARY EXAM - 2015


INSIGHTS ON INDIA MOCK TEST - 22
GENERAL STUDIES

PAPER-I
Time Allowed: 2 Hours

Maximum Marks: 200

INSTRUCTIONS
1. IMMEDITELY AFTER THE COMMENCEMENT OF THE EXAMINATION, YOU SHOULD
CHECK THAT THIS TEST BOOKLET DOES NOT HAVE ANY UNPRINTED OR TORN OR
MISSING PAGES OR ITEMS, ETC. IF SO, GET IT REPLACED BY A COMPLETE TEST BOOKLET.
2. You have to enter your Roll Number on the Test I
Booklet in the Box provided alongside. DO NOT
write anything else on the Test Booklet.
4. This Test Booklet contains 100 items (questions). Each item is printed only in English. Each item
comprises four responses (answers). You will select the response which you want to mark on the Answer
Sheet. In case you feel that there is more than one correct response, mark the response which you consider
the best. In any case, choose ONLY ONE response for each item.
5. You have to mark all your responses ONLY on the separate Answer Sheet provided. See directions in the
Answer Sheet.
6. All items carry equal marks.
7. Before you proceed to mark in the Answer Sheet the response to various items in the Test Booklet, you
have to fill in some particulars in the Answer Sheet as per instructions sent to you with your Admission
Certificate.
8. After you have completed filling in all your responses on the Answer Sheet and the examination has
concluded, you should hand over to the Invigilator only the Answer Sheet. You are permitted to take away
with you the Test Booklet.
9. Sheets for rough work are appended in the Test Booklet at the end.
10. Penalty for wrong answers :
THERE WILL BE PENALTY FOR WRONG ANSWERS MARKED BY A CANDIDATE IN THE
OBJECTIVE TYPE QUESTION PAPERS.
(i)

There are four alternatives for the answer to every question. For each question for which a
wrong answer has been given by the candidate, one-third of the marks assigned to that question
will be deducted as penalty.

(ii) If a candidate gives more than one answer, it will be treated as a wrong answer even if one of the
given answers happens to be correct and there will be same penalty as above to that question.
(iii)

If a question is left blank, i.e., no answer is given by the candidate, there will be no penalty for
that question.
http://insightsonindia.com

INSIGHTS ON INDIA MOCK TEST SERIES FOR CIVIL SERVICES PRELIMINARY EXAM 2015

http://insightsonindia.com

Page 1

1. Indian art is deeply rooted in religion


and it conduces to fulfilling the
ultimate aim of life, Liberation. It
means
a) A vision of the divine
b) Release from the cycle of birth and
death
c) out of body experience
d) Release from all earthly bondage
and resting in heaven

2. Miniature painting in India existed in


the form of illustrations to the religious
texts in which of the following cults
1. Buddhist
2. Vaishnav
3. Mughal
4. Jain

4. Lomas rishi cave in Bihar is a typical


example of early Indian cave
architecture. If you happen to visit
Lomas cave, which of these things may
come to your notice?
1. Entrance is a representation in
stone of a hut entrance
2. No timber has been used in the
cave
3. A carved frieze of stone-made
elephants
Choose the correct answer using the codes
below.
a)
b)
c)
d)

1 and 2 only
1 and 3 only
2 and 3 only
All of the above

Choose the correct answer using the codes


below.
a)
b)
c)
d)

1 and 4 only
3 only
2 and 3 only
All of the above

3. Consider the following statements.


1. Mughal paintings were not secular,
while other schools like Pahari had
secular paintings.
2. Art of stone masonry and stone
carving were imported in Ashoka's
times from Persia.
3. Timber was not used in Harappan
architecture.
Choose the correct answer using the codes
below.
a)
b)
c)
d)

1 and 2 only
1 and 3 only
2 and 3 only
2 only

http://insightsonindia.com

5. Consider the following statements


about Buddhist stupas.
1. One cannot enter the stupas.
2. They were constructed by royal
patronage when Buddha was
teaching.
3. Jataka stories have also been
engraved on stupas.
Choose the correct answer using the codes
below.
a)
b)
c)
d)

1 and 2 only
1 and 3 only
2 and 3 only
All of the above

6. With reference to ecosystem, consider


the following statements:
1. It is self regulating
2. It is self sustaining

Page 2

3. Biosphere can also be called as an


ecosystem
Which of the above statements is/are correct?
a)
b)
c)
d)

1 and 3 Only
All
2 and 3 Only
1 & 2 Only

7. Consider the following statements


about the inscriptions found at Aihole,
Karnataka.
1. Sanskrit as well as Tamil has been
used in the inscriptions.
2. The inscription contains the
description of historical defeats of
kingdoms.
3. There is mention about the
poet Kalidasa in the inscriptions.
Choose the correct answer using the codes
below.
a)
b)
c)
d)

1 and 2 only
1 and 3 only
2 and 3 only
All of the above

8. In Indian architecture,
Mahabalipuram is known to have
1. Rock cut temples
2. Monolithic structures
3. Structural temples
4. Limestone caves
Choose the correct answer using the codes
below.
a)
b)
c)
d)

1 and 2 only
1, 3 and 4 only
2 and 4 only
1, 2 and 3 only

http://insightsonindia.com

9. Consider the following statements


about Nalanda.
1. This place was visited by Buddha
and Mahavira.
2. Nalanda was an important centre of
Pala sculptures.
3. Nalanda was a principal centre of
Mahayana (Buddhism) learning.
4. The Chinese Pilgrims Huien Tsang
and Fa-hien studied at Nalanda.
Choose the correct answer using the codes
below.
a)
b)
c)
d)

1 and 2 only
1, 3 and 4 only
2 and 4 only
All of the above

10. Consider the following features of


temple architecture.
1. The barrel-shaped tower does not
crown the sanctum sanctorum or
thegarbha-griha
2. There is a Vimana - temple tower
over the main shrine
3. The style consists of three distinct
types of temples - Rekha Deula,
Pidha Deula and Khakhara Deula.
The above mentioned features are found in
the temple architecture of which state in
India?
a)
b)
c)
d)

Orissa
TamilNadu
Karnataka
Madhya Pradesh

11. Which of the following different styles


of architecture have been imbibed in
Hindu temple architecture?
1. Champa architecture
2. Gadag architecture
3. Maru-Gurjara architecture
Page 3

4. Khmer architecture
5. Sreekovil architecture
Choose the correct answer using the codes
below.
a)
b)
c)
d)

All except 1 and 4


All except 2 and 3
All except 5
All of the above

12. Consider the following about the


Dravidian style of temple architecture.
1. It is always square in plan.
2. Mandapas always cover and
precede the door leading to the cell.
3. Pillared halls are an invariable
feature of temples.
Choose the correct answer using the codes
below.
a)
b)
c)
d)

1 and 2 only
1 and 3 only
2 and 3 only
All of the above

13. Which of the following is NOT an


Abiotic component of an ecosystem?
1. Water
2. Soil
3. Humidity
4. Atmosphere
5. Radiation

14. Consider the following about the Pala


dynasty.
1. It ruled eastern-south Asia from 8th
-12th century.
2. Buddhism and Hinduism were
subjugated and suppressed under
Pala dynasty.
3. The Pala school of art flourished in
the Magadha region of Southern
Bihar.
Choose the correct answer using the codes
below.
a)
b)
c)
d)

1 and 2 only
1 and 3 only
2 and 3 only
All of the above

15. Which of the following features were


infused by Islamic architecture in
India?
1. Use of concrete as a factor of
construction
2. Use of lime mortar as a base for
decoration
3. Use of arches and vaults in
construction
Choose the correct answer using the codes
below.
a)
b)
c)
d)

1 and 2 only
1 and 3 only
2 and 3 only
All of the above

Choose the correct answer using the codes


below:
a)
b)
c)
d)

1 Only
3 Only
5 Only
None

http://insightsonindia.com

16. Which of these are the features of


Indo-Islamic architecture, but not
Hindu architecture?
1. utilisation of kiosks (chhatris)
2. tall towers (minars)
3. half-domed double portals
4. human worship in decorations and
embroidery
Page 4

Choose the correct answer using the codes


below.
a)
b)
c)
d)

1, 2 and 3 only
1 and 3 only
2 and 4 only
All of the above

17. Consider the following decorative


elements.
1. Pink sandstone
2. Octagonal forms
3. Shallow water-channels
4. Raising of platforms
5. Rhythm in structures
Which of the above are features of Persian
architecture?
a)
b)
c)
d)

3 and 4 only
1, 2, 3, 4 and 5
1, 2 and 5 only
2, 3 and 4 only

18. With reference to migration of an


animal species, consider the following
statements:
1. It is a phenomena of short distance
seasonal movement into new
region
2. A young Atlantic salmon leaving
the river of their birth when they
have reached a few inches in size
can be considered as true migration
Which of the above statements is/are correct?
a)
b)
c)
d)

1 Only
2 Only
Both
None

19. Which of these statements is NOT


correct about the Indus Valley
civilization?
1. People worshipped the Mother
Goddess or Goddess of fertility.
2. Dancing was a part of the culture at
Indus valley.
3. The trade and cultural contacts
were restricted to the other cultures
in the Indus valley.
Choose the correct answer using the codes
below.
a)
b)
c)
d)

20. The abacus of Lion capital at Sarnath is


supported by a bell-shaped base
consisting of a lotus
with dharmachakra. It may best
symbolize
a) Victory of the good over evil
b) Victory of righteousness over
physical force
c) Path of renunciation in Dharma
d) All life is governed by the same set
of laws

21. With reference to Saprotrophic


nutrition, consider the following
statements:
1. It involves processing of dead or
decayed organic matter
2. Organisms involved usually are
protozoa
Which of the above statements is/are correct?
a)
b)
c)
d)

http://insightsonindia.com

3 only
1 only
2 only
All of the above

1 Only
Both
None
2 Only
Page 5

22. Buddha is never represented in human


form in Buddhist art before the
Christian era, as his spirituality was
considered too abstract for the
purpose. Which of the following forms
was he represented in then?
1. Empty throne
2. Royal umbrella
3. Wheel of Law
4. Triratna symbol
Choose the correct answer using the codes
below.
a)
b)
c)
d)

2 and 3 only
1 and 3 only
1 and 4 only
All of the above

23. In the Gandhara school of art, Buddha


as seated in the typically Indian Yogic
posture is a sculpture style that has
been borrowed from
a) Hellenistic tradition of art
b) Gothic tradition of art
c) Roman sculpture style
d) None of the above

24. In Buddhism, the word Bodhisattva is


used for the one who has
a) Attained enlightenment but is not
ready for Nirvana
b) Attained enlightenment, but is not
seeking Nirvana and has decided to
teach others out of his compassion
c) Seeking enlightenment
d) Who became enlightened and then
liberated from the cycle of birth and
death

http://insightsonindia.com

25. Instead of the classical dignity,


sobriety and simplicity, the sculpture is
now more and more tending towards
ornamentation, creating highly ornate
art objects, with strange and unusual
imaginary creatures, such as half
human, half monsters, etc. This
transformation in sculpture style holds
true for the shift in
a) Gupta sculpture to Medieval
sculpture
b) Buddhist sculpture to Gupta
sculpture
c) Medieval sculpture to Modern
sculpture
d) Buddhist sculpture to Modern
sculpture

26. Consider the following statements


about the Kailasa temple at Ellora.
1. It was built by the Chalukyas.
2. It is based on Dravidian style of
architecture.
3. It is a rock-cut temple.
4. It is a part of the Ellora caves.
Choose the correct answer using the codes
below.
a)
b)
c)
d)

2 and 3 only
1 and 3 only
1 and 4 only
2, 3 and 4 only

27. Consider the following depictions.


1. Ravana shaking Mount Kailasha
and Siva is pressing down the
mountain with his foot.
2. King of the Naga and his queen
seated on a throne attended by a
hand maiden
3. Krishna teaching Arjuna in the
middle of Mahabharata

Page 6

Sculptures depicting which of the above can


be found at Ellora?
a)
b)
c)
d)

1 and 2 only
2 and 3 only
1 and 3 only
All of the above

28. Consider the following about Pala


paintings.
1. It is characterised by sinuous line
and subdued tones of colour.
2. It is totally disconnected from the
classical art of Ajanta.
3. The Pala art flourished under the
royal patronage of the Mughals.
Choose the correct answer using the codes
below.
a)
b)
c)
d)

1 and 2 only
2 and 3 only
1 only
All of the above

29. Consider the following statements:


1. Omnivores are considered as
secondary consumers in a food
chain
2. Herbivores are considered as
tertiary consumers
3. Carnivores are considered as
primary consumers in a food chain
Which of the above statements is/are correct?
a)
b)
c)
d)

2 and 3 Only
1 Only
1 & 2 Only
All

30. Students and pilgrims in from various


countries studies in Buddhist
monasteries. They took back to their
countries examples of Pala Buddhist
http://insightsonindia.com

art, in the form of bronzes and


manuscripts which helped to carry the
Pala style to
1. Nepal
2. Java
3. Tibet
4. Burma
5. Sri Lanka
Choose the correct answer using the codes
below.
a)
b)
c)
d)

1 , 3 and 4 only
2, 4 and 5 only
1, 3 and 5 only
All of the above

31. Consider the following statements.


Assertion (A): Despite Jainism being present
in Western India, no works motivated by
Jainism are found in Western Indian art.
Reason (R): Jainism did not receive any royal
patronage by the Kings of the Chalukya
Dynasty who ruled Gujarat and parts of
Rajasthan and Malwa.
In the context of the statements above, which
of these is true?
a) A and R both are true, and R is the
correct explanation for A.
b) A and R both are true, and R is the
NOT the correct explanation for A.
c) A is correct, R is incorrect.
d) A and R both are incorrect

32. Deccani school of painting made a


significant contribution to the
development of the Mughal style in
North India. The centres of Deccani
school were in
1. Ahmednagar
2. Bijapur
3. Golconda
Page 7

4. Hyderabad
5. Tanjore
Choose the correct answer using the codes
below.
a)
b)
c)
d)

1 , 3 and 4 only
2, 4 and 5 only
1, 3 and 5 only
All of the above

33. Consider the following statements


about the art of painting in Rajasthani
and Pahari regions.
1. These were largely secular in
character.
2. They drew inspiration from Mughal
school of paintings.
3. Paintings are marked by light
colours and bold drawings.
Choose the correct answer using the codes
below.
a)
b)
c)
d)

1 and 2 only
2 and 3 only
1 only
2 only

34. Consider the following statements


about the Wildlife institute of India.
1. It is an autonomous body.
2. It is under the Ministry of
Environment and Forests.
3. It has recently launched an
initiative for allowing PSU and
Companies to adopt endangered
species.
4. It has proposed that wildlife
conservation efforts spending be
counted under Corporate Social
responsibility (CSR).
Choose the correct answer using the codes
below.
http://insightsonindia.com

a)
b)
c)
d)

1 and 2 only
1, 3 and 4 only
2, 3 and 4 only
All of the above

35. Consider the following statements:


1. This biome is home to many
species of grasses such as purple
needlegrass, wild oats, foxtail,
ryegrass, and buffalo grass.
2. Rain in these biomes usually occurs
in the late spring and early summer
3. These are known as Pampas in
South America
To which of the following biomes, are the
above statements related to?
a)
b)
c)
d)

Savannah Biome
Tropical Grasslands
Temperate Grasslands
Taiga

36. 20th Law Commissions recent reports


provide several recommendations to
end discrimination against leprosy
patients. Consider the legal state of
affairs regarding leprosy.
1. Leprosy is a valid ground for
divorce under personal laws of
India.
2. Leprosy patients are banned from
Contesting election in local bodies
in some states.
3. Lepers Act permits segregation of
lepers into separate asylums.
Choose the correct statements using the codes
below.
a)
b)
c)
d)

1 and 2 only
1 and 3 only
2 and 3 only
All of the above
Page 8

37. Consider the following statements


about the recently launched Pradhan
Mantri Jeevan Jyoti Bima Yojana.
1. It is applicable to people of all age
groups.
2. Bank account is mandatory for
taking benefit from the scheme.
3. The insurance will be offered by
private insurance companies which
will be regulated by LCI and IRDA.
Choose the correct statements using the codes
below.
a)
b)
c)
d)

1 and 2 only
1 and 3 only
2 only
All of the above

38. RBI has recently mandated the banks


to keep side emergency fund called
Counter cyclical capital buffer. What
purposes will be served by this?
1. It will restrict the banking sector
from indiscriminate lending in the
periods of excess credit growth.
2. It will help RBI to manage foreign
exchange reserves better.
Which of the above is/are correct?
a)
b)
c)
d)

1 only
2 only
Both 1 and 2
None

39. With reference to desert plants,


consider the following statements:
1. They have a short life and the
capacity to leave behind very hardy
forms of propagation
2. They usually have small leaves and
tiny roots to save water
http://insightsonindia.com

3. Creosote bush is one of the


examples of desert plants
Which of the above statements is/are correct?
a)
b)
c)
d)

1 and 3 Only
2 and 3 Only
1 and 2 Only
All

40. Consider the following statements.


Assertion (A): Ocean acts as the largest
carbon sink on earth.
Reason (R): The difference in partial pressure
of the CO2 between seawater and air facilitate
gaseous exchange.
In the context of the statements above, which
of these is true?
a) A and R both are true, and R is the
correct explanation for A.
b) A and R both are true, and R is the
NOT the correct explanation for A.
c) A is correct, R is incorrect.
d) A and R both are incorrect

41. One way to increase the carbon


sequestration efficiency of the oceans
is to add micrometre-sized iron
particles in a certain form in the ocean.
The sequestration efficiency of oceans
increases because
a) Iron has the effect of stimulating
growth of plankton in the ocean.
b) It increases the rate of dissolution
of CO2 with ocean water.
c) Adding Iron increases the surface
density of the Ocean allowing for
easier gas exchange.
d) Iron helps increase the
photosynthesis rate of large marine
plants.
Page 9

42. The dissolved carbon dioxide is


important in the ocean because
1. It regulates PH of seawater and
thereby health of marine
organisms.
2. It helps marine organisms form
calcareous material.
3. It helps Phytoplanktons in
photosynthesis.
Choose the correct statements using the codes
below.
a)
b)
c)
d)

1 and 2 only
1 and 3 only
2 and 3 only
All of the above

43. With reference to atmospheric gases,


consider the following statements:
1. The percentage of carbon dioxide in
atmosphere is more than that of
argon gas
2. No other gas found above
troposphere zone of the
atmosphere
Which of the above statements is/are correct?
a)
b)
c)
d)

1 Only
2 Only
Both
None

44. The Critically Endangered Great


Indian Bustards is found in which of
the following states of India?
1. Rajasthan
2. Gujarat
3. Maharashtra
4. Haryana

a)
b)
c)
d)

1, 2 and 3 only
3 and 4 only
1 only
All of the above

45. Using bio-refining, It is possible to


obtain bio-ethanol from
1. Sea weeds
2. Corn
3. Rice straw
4. Coconut
5. Water hyacinth
Choose the correct statements using the codes
below.
a)
b)
c)
d)

1, 2 and 5 only
3 and 4 only
1 and 2 only
All of the above

46. Consider the following benefits.


1. Reducing coastal eutrophication
2. Cutting down on global warming
3. Obtaining fertilizers
Which of these can come from growing
extensive sea weeds in the ocean?
a)
b)
c)
d)

1 and 2 only
2 and 3 only
1 and 3 only
All of the above

47. The National Green Tribunal (NGT)


can deliver justice only in matters
pertaining to which of the following
subjects?
1. Air pollution
2. Climate change
3. Biodiversity
4. Conservation of Water

Choose the correct statements using the codes


below.
http://insightsonindia.com

Page 10

Choose the correct statements using the codes


below.
a)
b)
c)
d)

1, 3 and 4 only
3 and 4 only
1 and 2 only
All of the above

48. In high temperature high humidity


weather, why does it appear that we
are sweating more profusely as
compared to even hotter but dry
weather situations?
a) Sweat doesnt evaporate quickly
due to the high relative humidity
outside
b) In hot humid conditions,
hypothalamus (gland controlling
secretion) becomes more active
c) Human body releases more water
in conditions of high humidity
d) High humidity blocks the secretion
glands on the skin and the humidity
sticks on the skin

49. Consider the following statements


about 3D printing technology.
1. Objects produced cannot be
circular in shape.
2. Ultraviolet light lasers are used in
the printing process of adding
layers.
3. It cannot be used to produce
components used in aerospace.
4. Human organs can be produced
using 3D bio-printers.
Choose the correct statements using the
codes below.
a)
b)
c)
d)

2 and 4 only
3 and 4 only
1 and 2 only
All of the above

http://insightsonindia.com

50. The Rajya Sabha recently passed The


Rights of Transgender Persons Bill,
2014. It provides for
1. Reservation of transgenders in
public educational institutions and
government jobs.
2. Establishment of a special
transgender rights court.
3. Right for transgenders to adopt
children
Choose the correct statements using the codes
below.
a)
b)
c)
d)

1 and 2 only
1 and 3 only
2 and 3 only
All of the above

51. With reference to lentic ecosystem,


consider the following statements:
1. A wetland ecosystem is an example
of lentic ecosystem
2. In a lentic ecosystem, bacteria are
present only in the benthic zone i.e.
which comprises the bottom and
shore regions.
Which of the above stateents is/are correct?
a)
b)
c)
d)

None
Both
1 Only
2 Only

52. Consider the following about ESamiksha designed by National


Informatics Centre (NIC).
1. It is a part of the Digital India
program.
2. It is mandatory for all the states to
adopt and integrate this system in
their governance model.
Page 11

3. Using E-Samiksha, District


Magistrates can get directly in
touch with the panchayats and
block heads.
Choose the correct statements using the codes
below.
a)
b)
c)
d)

1 and 2 only
1 and 3 only
2 and 3 only
1 only

53. Consider the following about the INS


Visakhapatnam.
1. It is built entirely of Indian steel.
2. It is capable of launching BrahMos
missiles from its deck.
3. It has been designed and processed
in Israel.
4. The ship does not have a weapon
system of its own.
Choose the correct statements using the codes
below.
a)
b)
c)
d)

1 and 2 only
1 and 4 only
2 and 4 only
1, 2 and 3 only

54. The remora species that rides attached


to sharks and other fishes is a best
example of
a) Mutualism
b) Parasitism
c) Commensalism
d) None of the above

55. Consider the following about the


Indian missile Astra.
1. It is a land to air missile.
2. This is the smallest of the missiles
developed by DRDO.
http://insightsonindia.com

3. It can chase enemy aircraft at


supersonic speeds.
4. It has been developed indigenously.
Choose the correct statements using the codes
below.
a)
b)
c)
d)

1 and 3 only
1 and 4 only
2 and 4 only
2, 3 and 4 only

56. With reference to Profundal Zone of


pond ecosystem, consider the
following statements:
1. It is well lit zone
2. This is the main photosynthetic
body of the lake.
3. This zone produces food and
oxygen
Which of the above statements is/are
INCORRECT?
a)
b)
c)
d)

1 and 2 Only
2 and 3 Only
1 and 3 Only
All

57. As per a recent survey less than 30%


eligible women got the benefits of
Indira Gandhi Matritva Sahyog Yojana
(IGMSY). Consider the following about
IGMSY.
1. It provides partial wage
compensation to pregnant and
lactating women.
2. It is applicable only upto a
maximum of two children from a
mother.
3. It aims at promoting institutional
deliveries.
Choose the correct statements using the codes
below.
Page 12

a)
b)
c)
d)

1 and 2 only
1 and 3 only
2 and 3 only
All of the above

58. The Health minister recently launched


Mission Indradhanush for vaccination
against 7 preventable diseases. It
covers
1. Tuberculosis
2. Malaria
3. Whooping cough
4. Rotavirus
5. Rubella
6. Tetanus
Choose the correct statements using the codes
below.
a)
b)
c)
d)

1, 2, 4 and 6 only
2, 4 and 5 only
1, 3 and 6 only
All of the above

59. The earthquakes near the Himalayan


region are generally triggered due to
the collision/friction between which of
the following tectonic plates?
a) Indo-Eurasian plate and Central
Asian plate
b) Eurasian plate and Australian plate
c) Indian plate and Eurasian plate
d) Indian plate and Central Asian
plate

60. It is a biological interaction between


two or more organisms that is
detrimental to at least one of them; it
can also be an antagonistic association
between an organism and the
metabolic substances produced by
another.
http://insightsonindia.com

In the above questions, it refers to:


a) Antibiosis
b) Parasitism
c) Predation
d) Colonisation

61. Prime Minister recently inaugurated


Indias Healthcare Tourism Portal. It
will be useful for Medical tourism as it
provides information on
1. Ayurveda and wellness centres
2. Details such as costs related to
treatment in hospitals
3. Visa procedures
4. Travel and stay
Choose the correct statements using the codes
below.
a)
b)
c)
d)

1, 2, and 3 only
2 and 4 only
1 and 3 only
All of the above

62. Consider the following statements:


1. It is a partly enclosed coastal body
of brackish water with one or more
rivers or streams flowing into it.
2. They form a transition zone
between river environments and
maritime environments
The above statements describe which of the
following given geographical features?
a)
b)
c)
d)

Isthmus
Peninsula
Estuary
None of the above

Page 13

63. India and Bangladesh have decided to


take concerted conservation efforts to
protect the rare spectacled Langur.
Consider the following statements
about it.
1. It is found in Patharia Hills Reserve
Forest.
2. The primary focus of this
conservation effort is to preserve
the habitat for the survival of the
primates.
3. It is the ancestor of the endangered
Hoolock Gibbon and Golden
Langur.
Choose the correct statements using the codes
below.
a)
b)
c)
d)

1 and 2 only
2 and 3 only
1 and 3 only
All of the above

64. Consider the following statements:


1. In interactions in a biotic
community, colonization is
considered as a Positive interaction
between species
2. In interactions in a biotic
community, competition is
considered as a Negative
interaction between species
Which of the above statements is/are correct?
a)
b)
c)
d)

1 Only
2 Only
Both
None

65. India was placed at 117th position in


the recently released World Happiness
Report 2015 among the 158 studied
nations. Consider the following.
http://insightsonindia.com

1. The report was published by the


World Health Organization
(WHO).
2. The study for the report was done
by the United Nations
intergovernmental wing.
3. The happiness report takes account
of the social freedom and
corruption levels in
the country.
4. The report will be placed before the
United Nations general assembly
for a resolution and further
discussion.
Choose the correct statements using the codes
below.
a)
b)
c)
d)

1 and 2 only
3 and 4 only
1, 3 and 4 only
All of the above

66. With reference to micronutrients,


consider the following statements:
1. Iodine, vitamin and iron are some
of the micronutrients needed for
humans
2. Zinc, one of the micronutrients
needed by plants, is necessary
for osmosis and ionic balance
Which of the above statements is/are correct?
a)
b)
c)
d)

None
2 Only
Both
1 Only

67. Consider the following statements


about the occurrence of Volcanoes.
1. There are no active volcanoes in the
coastal belt of South America.
2. The Andes Mountains and
Indonesian region come under the
Ring of Fire.
Page 14

3. Volcanoes are also found in Midoceanic ridges.


Choose the correct statements using the codes
below.
a)
b)
c)
d)

1 and 2 only
1 and 3 only
2 and 3 only
All of the above

68. With reference to energy flow in an


ecosystem, consider the following
statements:
1. All energy needed by living
organisms is provided by reduction
process of sugar through
respiration and release of energy
2. The loss of energy through body
heat is far greater in warm-blooded
animals, which must eat much
more frequently than those that are
cold-blooded
3. The energy is passed on
from trophic level to trophic level
and each time about 10% of the
energy is lost
Which of the above statements is/are
INCORRECT?
a)
b)
c)
d)

1 and 2 Only
1 Only
3 Only
1 and 3 Only

69. India has emerged as the fourth largest


supplier of generic medicines to the
United States. Some countries impose
stringent regulatory measures on
import of generic medicines from
India. Which of these regions do not
allow such imports?
1. West Asia
2. Africa
3. Europe
http://insightsonindia.com

Choose the correct statements using the codes


below.
a)
b)
c)
d)

1 and 2 only
1 and 3 only
3 only
None of the above

70. Under the United Nations Framework


Convention on Climate Change
(UNFCCC) regime
1. Developing countries do not have
any obligation to share reports
about HFCs
2. Developed nations are bound to cut
green house gas emissions.
Which of the above is/are true?
a)
b)
c)
d)

1 only
2 only
Both 1 and 2
None

71. With reference to deciduous forests,


consider the following statements:
1. They lose their leaves to conserve
water or to better survive winter
weather conditions
2. They have xylem vessels with larger
diameters and therefore a greater
rate of transpiration (and hence
CO2 uptake as this occurs
when stomata are open) during the
summer growth period.
Which of the above statements is/are correct?
a)
b)
c)
d)

1 Only
2 Only
Both
None

Page 15

72. Mosquitoes are more attracted to some


human than others. Which of the
following factors play a role in it?
1. Body odour
2. Amount of CO2 in exhaled air
3. Body heat
4. Colour of clothing
Choose the correct statements using the codes
below.
a)
b)
c)
d)

1, 2 and 4 only
1 and 3 only
3 only
All of the above

73. The basic cause of Monsoon type of


climate is
a) Difference in the heating and
cooling of land and sea
b) Deflection of trade winds by coriolis
force
c) Change of seasons frequently in
various parts of a sub-continent
d) The periodic occurrence of warm
currents near oceans that border
continents

74. With reference to ecological pyramids,


cnsider the following statements:
1. The bottom of the pyramid
represents tertiary producers
2. The primary producers take energy
from the environment in the form
of sunlight and are located at the
apex of the pyramid
Which of the above statements is/are correct?
a) 1 Only
b) 2 Only
c) Both
d) None

75. For the survival of winter cereals sown


in Punjab, which of the following rains
is vital?
http://insightsonindia.com

a)
b)
c)
d)

North-east Monsoons
Cyclonic rainfall
Convectional rainfall
Both (a) and (c)

76. Consider the following statements


about Tropical Marine climate of
India.
1. It is found at eastern coasts of
India.
2. It receives steady rainfall from
trade winds all the time.
3. It is not prone to tropical cyclones.
4. Trees found in this climate region
are normally deciduous.
Choose the correct statements using the codes
below.
a)
b)
c)
d)

1, 2 and 3 only
1 and 3 only
2 only
1, 3 and 4 only

77. Consider the following statements.


1. The eastern coasts of continents
within the tropics have much
heavier rainfall than the interior or
the west coasts.
2. There is a marked difference in the
temperature between the east and
west coasts of countries in latitude
20 degrees to 35 degrees north.
The above is because of the effect of
a) Eastern cyclonic disturbances in
the tropical areas
b) Jet streams in tropical areas
c) Presence of warm currents on the
eastern coasts at the tropics
d) Continentality due to several
enclosed areas in the tropics

Page 16

78. Burmese Python in Florida state of


USA is considered as
a) Invasive species
b) Non-invasive species
c) Migratory species
d) None of the above

79. Consider the following statements


about rainfalls in various parts of the
world.
1. Both the length of the rainy season
and total amount of rainfall
decreases appreciably from the
equatorial region polewards
towards the desert fringes.
2. In the Southern hemisphere, rainy
season is during the Southern
Summer.
3. The Sudan type of climate has
heavy rainfalls due to extreme
diurnal ranges.
Choose the correct statements using the codes
below.
a)
b)
c)
d)

1 and 3 only
1 and 2 only
2 and 3 only
All of the above

80.The aridity of the hot deserts is mainly


due to the effects of
1. Off-shore trade winds
2. Off-shore cold currents
3. Continentality
Choose the correct statements using the codes
below.
a)
b)
c)
d)

1 and 3 only
1 and 2 only
2 and 3 only
All of the above

http://insightsonindia.com

81. Amongst the mid-latitude deserts,


many are found on plateaux and are at
a considerable distance from the sea.
Which of the following are such
deserts?
1. Gobi
2. Turkestan
3. Patagonian
4. Sahara
Choose the correct statements using the codes
below.
a)
b)
c)
d)

1 and 3 only
1 and 4 only
2 and 3 only
All of the above

82. The mid-latitude deserts that are


sheltered by high mountains around
them receive rainfall by
1. Depressions
2. Convectional rainfall
3. Orographic rainfall
Choose the correct statements using the
codes below.
a)
b)
c)
d)

1 and 3 only
1 and 2 only
2 and 3 only
All of the above

83. The Warm temperate western margin


climate is found in relatively few areas
of the world. The basic cause of this
type of climate is
a) Shifting of wind belts
b) Change in the direction of cold
currents
c) Presence of cyclonic depressions
d) Regular visit of Polar winds to the
regions

Page 17

84. Consider the following statements:


1. At the lowest trophic level (the
bottom of the food chain), plants
convert about 1% of the sunlight
they receive into chemical energy.
2. The efficiency with which energy or
biomass is transferred from one
trophic level to the next is called
the ecological efficiency.
3. Consumers at each level convert on
average only about 10% of the
chemical energy in their food to
their own organic tissue
Which of the above statements is/are correct?
a)
b)
c)
d)

1 and 3 Only
2 and 3 Only
1 and 3 Only
All

85. The prominence of local winds around


the Mediterranean Sea can be
explained by which of the following
topographic factors?
1. High Alps in the North
2. Open Atlantic on the West
3. Cold air masses from the
continental interiors
4. Sahara deserts in the South
Choose the correct statements using the codes
below.
a)
b)
c)
d)

1 and 3 only
1 and 4 only
2 and 3 only
All of the above

86. Which of the following kinds of


vegetation are NOT likely to be found
in the Mediterranean regions?
1. Evergreen coniferous forests
2. Grass
3. Deciduous forests
Choose the correct statements using the codes
below.
http://insightsonindia.com

a)
b)
c)
d)

1 and 3 only
1 and 2 only
2 and 3 only
3 only

87. Chinook, a famous local wind, comes


on the eastern slopes of Rockies in
Canada. It is useful because
1. It makes the snow covered pastures
grazable.
2. It is a hot wind and warms the local
climate.
3. It brings moderate amount of
rainfall with it.
Choose the correct statements using the codes
below.
a)
b)
c)
d)

1 and 3 only
1 and 2 only
2 and 3 only
All of the above

88. Which of the following is/are


autotrophs?
1. Cyanobacteria
2. Red Algae
3. Fungi
4. Green Algae
Choose the correct answer using the codes
below:
a)
b)
c)
d)

1 and 2 Only
2 and 3 Only
3 and 4 Only
1, 2 and 4 Only

89. The recent turnaround of the external


sector and the lowering of the current
Account deficit can be attributed to
which of the following factors?
1. Lowering of international prices of
crude petroleum
2. Restrictions on gold imports
3. Trade diversification process
Page 18

Choose the correct statements using the codes


below.
a)
b)
c)
d)

1 and 3 only
1 and 2 only
2 and 3 only
All of the above

90. Consider the following statements:


1. Predators are usually located at the
third trophic level in an ecological
pyramid
2. Decomposers are part of the first
trophic level of the food chain
Which of the above statemens is/are
INCORRECT?
a)
b)
c)
d)

1 Only
2 Only
Both
None

91. Since the recent economic slowdown in


India, which of the following imports
items has registered a decline in
growth?
1. Capital goods
2. Crude oil related products
3. Textile and allied products
Choose the correct statements using the codes
below.
a)
b)
c)
d)

1 and 3 only
1 and 2 only
2 and 3 only
1 only

92. With reference to CO2 in ecosystem,


consider the following statements:
1. The concentration of Carbon
Dioxide is more than oxygen in
water
2. Rising CO2 will stimulate plant
growth in an ecosystem
http://insightsonindia.com

Which of the above statements is/are correct?


a) 1 Only
b) 2 Only
c) Both
d) None
93. A keystone species is a species that
a) Has high biomass and crucial role
in a community
b) Has low biomass but huge role in a
community
c) Has minimal effect on an ecosystem
even if its loss is huge
d) Dwells as parasite on larger
animals
94. Consider the following statements
about the state of economic affairs
since the beginning of the millennium.
1. Increasing use of imported inputs
has generally caused a decline in
the domestic value added share of
total exports.
2. The foreign value added share in
exports has increased indicating
deepening of the process of
international production
fragmentation.
Which of the above is are/true?
a)
b)
c)
d)

1 only
2 only
Both 1 and 2
None

95. Which of the following schemes had


been implemented as a part of Indias
Foreign Policy?
1. Focus Market Scheme
2. Vishesh Krishi and Gram Udyog
Yoajana
3. Focus Product schemes
4. Market Development Assistance

Page 19

Choose the correct statements using the codes


below.
a)
b)
c)
d)

1 and 3 only
1 and 4 only
2 and 3 only
All of the above

96. An ecotone is a
a) Transition area between two
biomes
b) Region where a biome ends
c) Region where ecology is protected
from external influences
d) None of the above
97. Indian trade portal was launched at
the end of last year aimed at providing
vital information to Indian Industry.
Which of the following information
will be provided by the portal?
1. Most favoured nation (MFN) tariff
2. Details of all Trade agreements in
force
3. Platform for foreign buyers to
contact Indian suppliers
Choose the correct statements using the codes
below.
a)
b)
c)
d)

1 and 3 only
1 and 2 only
2 and 3 only
All of the above

98. The Trade Facilitation Agreement


(TFA), which was also endorsed by
India at the Ninth Ministerial
Conference of WTO, is basically aimed
at
1. Use of electronic payments
2. Risk management techniques
3. Giving all member nations of WTO
the Most favoured Nation (MFN)
Status

http://insightsonindia.com

4. Removal of ban on certain


commodities for all member
nations
Choose the correct statements using the codes
below.
a)
b)
c)
d)

1 and 3 only
1 and 4 only
2 and 3 only
1 and 2 only

99. If the Indian rupee depreciates against


Dollar persistently for a long period of
time, yet it will not affect Indias
overall trade scenario if
a) Effective exchange rate does not
change much
b) Real effective exchange rate does
not change much
c) Nominal effective exchange rate
does not change much
d) Foreign exchange reserves remain
constant
100.
The Capital account in the
external sector consists of
1. External assistance
2. External Commercial Borrowings
(ECBs)
3. Short-term debt
4. NRI deposits
5. Invisibles
6. Portfolio investment
Choose the correct statements using the codes
below.
a)
b)
c)
d)

All except 2, 5 and 6


All except 3 and 4
All except 1 and 6
All except 5

Page 20

http://insightsonindia.com

Page 21

Insights Mock Tests - 2015


TEST 22 SOLUTIONS

1. Solution: b)
In Indian religions and Indian philosophy, moksha also
called vimoksha, vimukti and mukti, means emancipation, liberation or release. In
the soteriological and eschatological sense, it connotes freedom from sasra, the
cycle of death and rebirth. In the epistemological and psychological sense, moksha
connotes freedom, self-realization and self-knowledge.
In Hindu traditions, moksha is a central concept and included as one of the four
aspects and goals of human life; the other three goals are dharma (virtuous, proper,
moral life), artha (material prosperity, income security, means of life),
and kama (pleasure, sensuality, emotional fulfillment). Together, these four aims of
life are called Pururtha in Hinduism.

2. Solution: d)
Miniature painting developed During the 10th century in western part of India that is
modern state of Rajasthan.These paintings are seen in hastprat [mini books written
by hands ] of Vaishnav sect and Jain cult. This paintings were made to make the
subject of the book more interested and to decorate the books.
Particular manuscript miniature painting developed in the western part of India that
is modern state of Rajasthan. The subjects of these miniature paintings are in
relation to the subjects of the manuscripts mostly religious and literary. Many
paintings are from Sanskrit and folk literature. It is on the subject of love stories.
Some paintings from Vaishnav sect of Hindu religion and some are from Jain cult.
The Paintings of Vaishnav sect are regarding various occasions of the life of Lord
Krishna and Gopies. Vaishnav paintings of "Gita Govinda" is about Lord Krishna.
The paintings of Jain cult is concerning to Jain Lords and religious subjects.
The earliest examples of miniature painting in India exist in the form of illustrations
to the religious texts on Buddhism executed under the Palas of the eastern India and
the Jain texts executed in western India during the 11th-12th centuries A.D.
The origin of the Mughal School of Painting is considered to be a landmark in the
history of painting in India. With the establishment of the Mughal empire, the
Mughal School of painting originated in the reign of Akbar in 1560 A.D.

3. Solution: d)
There does not appear to be any connection between the cities built in the 3rd
millennium B.C., with an astonishing civic sense, of first rate well-fired brick
structures, and the architecture of subsequent thousand years or so, of Indian art
history, after the decline and decay of the Harappan civilization and the beginning of
http://www.insightsonindia.com

Insights Mock Tests - 2015


TEST 22 SOLUTIONS

the historic period of Indian history, mainly the time of the great Mauryas of
Magadha. These thousands years or so were a period of tremendous, intellectual and
sociological activity and could not be barren of any artistic creations. However, due
to the fact that during this time sculpture and architecture was utilising organic and
perishable materials such as mud, mudbrick, bamboo, timber, leaves, straw and
thatch, these have not survived the ravages of time.
We know it for a fact that stone masonry and stone carving were imported in
Ashoka's times from Persia. There is abundant evidence of stone masons marks
similar to those at Persepolis. However, wood was still the dominant material and in
architectural remains of Ashokan times, the gradual transition from wood to stone is
apparent.

4. Solution: d)
A typical example of early cave architecture is the most datable cave of all, the socalled Lomas Rishi cave in the Barabar Hills of Bihar. An inscription proves that this
was excavated for the Ajivika sect in the time of Ashoka himself. The cave carved out
of the living rock, measures 55'x22'x20'. The entrance is a representation in stone of
a hut entrance, with the end of the roof constructed of bent timber supported by
cross beams, the ends of which are shown protruding. A carved frieze of elephants is
a stone imitation of similar work in wood along with a stone imitation of trellis work
made of small stick of bamboo. This is an excellent example showing the
development from earlier shapes in timber translated into stone. The period is the
3rd century B.C.

5. Solution: b)
The Buddhist Stupa is another form of architecture, comprising a hemispherical
dome, a solid structure into which one cannot enter. The stupa is a glorified,
beautified, enlarged funerary mound: what was once the resting place of the bones
and ashes of a holy man. Tradition has it that after the great demise of Lord Buddha,

http://www.insightsonindia.com

Insights Mock Tests - 2015


TEST 22 SOLUTIONS

Emperor Ashoka decided to construct a large number of stupas throughout his


dominion in memory of the Master and enshrine in them relics such as pieces of
bones, teeth, hair etc., over which the Stupas were constructed.
The railing and gateways at Bharhut, Sanchi and Bodh Gaya are the most famous in
the north and at Amravati and Nagarjunakonda in the South. Upright pillars and
cross bars, based on wooden construction, were made and provided the occasion for
dome of the finest low relief carvings to be found anywhere in Indian art. On these
surfaces are carved the favourite symbols of Buddhism, the lotus, elephant, bull, lion
and horse and some of the Jataka stories of the previous births of Buddha, depicted
in low relief.

6. Solution: b)
In a balanced condition, ecosystem functioning is self- regulating and self-sustaining.
This dynamic nature of ecosystem is dependent upon a number of factors including
flow of energy, cycling of materials and perturbations, both intrinsic and extrinsic.
Ecosystem is now recognized as a dynamic concept with structural heterogeneity
based on at least four functional compartments or phases.
Biosphere constitutes vast network of all ecosystems, hence, itself acts as a large
ecosystem.
According to Britannica encyclopedia, the biosphere is a global ecosystem composed
of living organisms (biota) and the abiotic (nonliving) factors from which they
derive energy and nutrients.

7. Solution: d)
http://en.wikipedia.org/wiki/Aihole_inscriptions
Aihole is a village having a historic temple complex in the Bagalkot
district of Karnataka, It is known for Chalukyan architecture, with about 125 stone
temples dating from 5th century CE. With its collection of architectural structures,
Aihoe has the potential to be included as a UNESCO World heritage site.

8. Solution: d)
Apart from structural temples the other variety of temples are rock cut, found at
Mahabalipuram, about 38 miles down south of Madras on the sea shore, datable to
the 5th century A.D. In local parlance they are known as Ratha or chariots and are
named after the five Pandava brothers and Draupadi but they neither have anything
to do with chariots nor probably with the Pandavas and these associations are purely
http://www.insightsonindia.com

Insights Mock Tests - 2015


TEST 22 SOLUTIONS

of a local character. The great Pallava rulers of Kanchipuram, were great builders and
the Pallava craftsmen, seized upon the long outcrop or rocks and boulders available
on the sea shore, carved them and gave to them the shape of temples (monolithic) as
well as colossal statues of lions, elephants and bulls, etc. carved out of smaller
boulders.
9. Solution: d)
According to literary tradition, Nalanda, 10 kilometres north of Rajgir and a suburb
of the ancient city, was visited by Buddha and Mahavira. Ashoka is said to have
worshipped at the chaitya-niches of Sariputra, Buddha's disciple, and erected a
temple. By the time of Harsha A.D. 606-648, Nalanda had become the principal
centre of Mahayana learning and a famed University town with numerous shrines
and monasteries which attracted scholars from far and near. The Chinese Pilgrims
Huien Tsang and Fa-hien studied at Nalanda and have left account of the settlement
and its life.
Temple 3 was more than 31 m. high and consisted of seven successive accumulations
of which the two latest belonged to the 11th and 12th centuries and the fifth one,
dating from circa 6th century, was notable for its sculptural wealth. The monasteries
were imposing rectangular buildings, each with an open courtyard, enclosed by a
covered verandah which leads into cells, arranged on the four sides. The cell facing
the entrance served as a shrine. Nalanda was an important centre of Pala sculptures
and bronzes and has also yielded seals and sealings of great historical significance.

10.

Solution: a)

Till about the 6th century A.D., the style of temple architecture was similar both in
the north as well as in the south. It is only after this date that each began to evolve in
its own different direction. For the present let it be understood clearly that the two
areas where temple architecture developed most markedly were the Deccan and
Orissa and in both these areas the northern and southern style temples can be found
side by side. The Vimana, the temple tower over the main shrine in Orissa is one of
the most glorious inventions of architecture in India and is functionally a much finer
conception than the south Indian Gopuram, where the barrel-shaped tower does not
crown the sanctum sanctorum or thegarbha-griha but is a glorified entrance gate.
We had suggested in our introduction that the architect wanted to impart to the
temple more importance, prominence than the other buildings in the
neighbourhood, because here lived his God in the garbha griha or the womb-house.
For more read the Introduction of
http://en.wikipedia.org/wiki/Kalinga_architecture

http://www.insightsonindia.com

Insights Mock Tests - 2015


TEST 22 SOLUTIONS

11. Solution: c)
Go through the introduction part and the different styles of architecture cursorily
http://en.wikipedia.org/wiki/Hindu_temple_architecture

12. Solution: d)
Dravidian style temples consist almost invariably of the four following parts,
differing only according to the age in which they were executed:

The principal part, the temple itself, is called the Vimana (or Vimanam). It is
always square in plan and surmounted by a pyramidal roof of one or more
stories; it contains the cell where the image of the god or his emblem is placed.

The porches or Mandapas (or Mantapams), which always cover and precede
the door leading to the cell.

Gate-pyramids, Gopurams, which are the principal features in the


quadrangular enclosures that surround the more notable temples.

Pillared halls or Chaultrisproperly Chawadis -- used for various purposes,


and which are the invariable accompaniments of these temples.

Besides these, a temple always contains temple tanks or wells for water (used for
sacred purposes or the convenience of the priests), dwellings for all grades of the
priesthood are attached to it, and other buildings for state or convenience.

Thanjavur Temple, Trichy, Tamilnadu

http://www.insightsonindia.com

Insights Mock Tests - 2015


TEST 22 SOLUTIONS

13. Solution: d)
All are Abiotic components:
In biology, abiotic factors can include water, light, radiation, temperature, humidity,
atmosphere, and soil. The macroscopic climate often influences each of the above.
Pressure and sound waves may also be considered in the context of marine or subterrestrial environments.[4]
All of these factors affect different organisms to different extents. If there is little or
no sunlight then plants may wither and die from not being able to get enough
sunlight to complete the cycle of photosynthesis. Many Archea require very high
temperatures, or pressures, or unusual concentrations of chemical substances, such
as sulfur, because of their specialization into extreme conditions. Certain fungi have
evolved to survive mostly at the temperature, the humidity, and stability of their
environment.

14. Solution: b)
From the eighth to twelfth centuries, the eastern portion of India was host to a
florescence of artistic activity. Under the Pala dynasty, which ruled large portions of
Eastern-South Asia for nearly four hundred years span, many centres of Buddhism
and Hinduism flourished.
The Pala dynasty came to power around 750 A.D. The Pala school of art first
flourished in the Magadha region of Southern Bihar, the homeland of Buddhist
religion. Not surprisingly, the majority of early Pala-period remains are Buddhist.
Due to intense religious activity during Pala Sena period, many religious structures
were built or renovated. Most of these buildings have vanished leaving no extant
architecture from this period and making it very difficult to reconstruct a systematic
overview of the architectural development. Inspite of non-availability of any building,
a huge corpus of sculpture and a few paintings survive from this period.
During the Pala-period, a number of monasteries and religious sites that had been
founded in earlier periods grew into prominence. The large cruciform stupa at
Paharpur (ancient Somapura) in Bengal (now Bangladesh), for example, measures
more than one hundred meters from North to South. It was built around the late
eighth or early ninth century. The walls of the courtyard contain 177 individual cells
that served as shrines.
Although the first two hundred or so years of Pala-period art were dominated by
Buddhist art, the Hindu remains also exist in some quantities in that phase and
clearly dominate in the last two hundred years of the Pala-period.

http://www.insightsonindia.com

Insights Mock Tests - 2015


TEST 22 SOLUTIONS

15. Solution: d)
In the Hindu style of construction spaces were spanned corbels, held together by
making courses project, each further than the one below, so that the open span was
gradually reduced to a size which could be covered with a single slab or brick.
Although there exists some evidence to suggest that the true arch may have been
known in India earlier, it is the Muslims who are believed to have brought the
principle of building a true arch so as to hold up the roof or ceiling or a top part of a
structure, the bricks or stones laid to reproduce a curve, held together by the keystone on the top of the rise. In many cases even if the true arch was familiar to
indigenous architects in ancient times, it was re-introduced by the Muslims. The
result was that flat lintels or corbelled ceilings were replaced by arches or vaults, and
the pyramidal roof or spire by the dome.
Among the architectural features introduced by them mention may be made of
arches, domes, minars and minarets, the pendentive, squinch arch, half domed
double portals, kiosks (chhatris) and the use of concrete as a factor of construction.
They also introduced gilding and painting in varied colours and designs. Muslim
decorative elements are usually of the nature of embroidery. Even though lime was
known and to certain extent used in construction work in India fairly early, mud was
generally used for brick work and large blocks of stones were laid one on top of the
other and held by means of iron clamps. The Muslims, like the Romans, were also
responsible for making extensive use of concrete and lime mortar as an important
factor of construction and incidentally used lime as plaster and a base for decoration
which was incised into it and held enamel work on tiles.

16. Solution: a)
http://asi.nic.in/asi_monu_whs_humayuntomb_char.asp
The introduction of decorative brackets, balconies, pendentive decorations, etc in the
architecture is an example in this regard. The other distinguishing features of IndoIslamic architecture are the utilisation of kiosks (chhatris), tall towers (minars) and
half-domed double portals. As human worship and its representation are not allowed
in Islam, the buildings and other edifices are generally decorated richly in
geometrical and arabesque designs. These designs were carved on stone in low relief,
cut on plaster, painted or inlaid. The use of lime as mortar was also a major element
distinct from the traditional building style.
17. Solution: b)
The first distinct example of proper Mughal architecture inspired by Persian
architecture, is the tomb of Humayun, in Delhi, built by his widow, Begha Begum.

http://www.insightsonindia.com

Insights Mock Tests - 2015


TEST 22 SOLUTIONS

This tomb is important for a proper study of the development of later Mughal
architecture and has provided the prototype, followed by architects who designed the
Mausoleum of Jahangir at Shahdara, Lahore, as well as the celebrated Taj Mahal, at
Agra. Although Sikander Lodi's tomb as the first garden tomb built in India, it is
Humayun's tomb which strikes a new note. It is a memorial erected by a devoted wife
for her imperial husband and is magnificent, grand and impressive. Raised on a vast
platform, the tomb proper stands in the centre of a square garden, divided into 4
main parts by causeways (Charbagh) in the centre of which ran shallow waterchannels.
The square, red, sandstone, double storeyed structure of the mausoleum rises over a
high square terrace, raised over a series of cells which are like a musical composition.
The octagonal form of the central chamber containing the cenotaph, is inspired by
Syrian and earlier Islamic models. It is for the first time that pink sandstone and
white are used with admirable effect, the white is used cleverly to emphasise,
surround and underline doors and windows, strengthening the design.
There is a certain rhythmic quality in the whole structure in its symmetrical design
and the repetition of the large dome in the similar pavillions with small but similar
domes.

18.Solution: c)
Animal migration is the relatively long-distance movement of individuals, usually on
a seasonal basis. It is found in all major animal groups, including birds,
mammals, fish, reptiles, amphibians, insects, and crustaceans.[1] The trigger for the
migration may be local climate, local availability of food, the season of the year or for
mating reasons.[2] To be counted as a true migration, and not just a local dispersal
or irruption, the movement of the animals should be an annual or seasonal
occurrence, such as birds migrating south for the winter; wildebeest migrating
annually for seasonal grazing; or a major habitat change as part of their life, such as
young Atlantic salmon leaving the river of their birth when they have reached a few
inches in size.

19. Solution: a)
The beginning of stone sculpture in India goes back to a very remote age. The
excavations carried out in 1924, at the ruins of Mohenjodaro on the Indus river and
Harappa in the Punjab, brought to light a highly developed urban civilization,
archaeologically known as the Indus Valley or Harappan Culture. It flourished from
C.2500 B.C. to 1500 B.C. These ancient cities had a systematic lay-out, wide roads,
spacious houses made of bricks, and an underground drainage system, somewhat
like our own. People worshipped the Mother Goddess or Goddess of fertility. Trade
http://www.insightsonindia.com

Insights Mock Tests - 2015


TEST 22 SOLUTIONS

and cultural contacts existed between these cities and those of Mesopotamia of which
the evidence is the occurrence of the seals, as well as similar carnelian beads,
knobbed pottery, etc., at both places. Clay was the earliest medium in which man
began to mould and we have discovered a large number of terracotta figurines from
these Indus Valley sites.

20.

Solution: b)

The great Buddhist Emperor Ashoka caused the erection of monolithic pillars of
sandstone, 30 to 40 feet high, crowned by animal figures like the bull, lion and
elephant, and had them inscribed with the Buddhist concepts of morality, humanity
and piety, which he wished his people to follow. Famous Ashokan pillars are from
Lauriya Nandangarh in Bihar, Sanchi and Sarnath.
The most remarkable of them all is the highly polished monolithic lion-capital found
at Sarnath, which is now the Emblem of the Government of India. It represents four
roaring lions back to back facing the four cardinal directions. The round abacus is
decorated with four dharmachakras or wheels of law, alternating with an elephant, a
bull, a horse and a lion, all carved with masterly skill. The abacus is supported by a
bell-shaped base consisting of a lotus with dharmachakra, which perhaps
symbolized the victory of righteousness over physical force. The superb modelling of
the figures executed in a realistic manner with a certain stylization, is invested with a
great power and dignity, and reveals the aristocratic and international nature of
Mauryan art.

21. Solution: a)
Saprotrophic nutrition is a process of chemoheterotrophic extracellular
digestion involved in the processing of dead or decayed organic matter. It occurs in
saprotrophs or heterotrophs, and is most often associated with fungi (for
example Mucor and Rhizobium) and soil bacteria. Saprotrophic microscopic fungi
are sometimes called saprobes; saprotrophic plants or bacterial flora are
called saprophytes (sapro- + -phyte, "rotten material" + "plant"). The process is most
often facilitated through the active transport of such materials
through endocytosis within the internal mycelium and its constituent hyphae.

22.

Solution: d)

http://www.insightsonindia.com

Insights Mock Tests - 2015


TEST 22 SOLUTIONS

Though it may seem strange, Buddha is never represented in human form in


Buddhist art before the Christian era, as his spirituality was considered too abstract
for the purpose. The adherents of the Buddhist faith followed the Hinayana path as a
means of attaining salvation. Buddha's presence in early Indian art is, therefore,
suggested by symbols like the Bodhi tree under which he attained enlightenment, the
wheel of law, his foot prints, the royal umbrella, the stupa and an empty throne, etc.
The relief-medallion from the fragment of a railing pillar of the stupa at Bharhut
datable to the 2nd Century B.C., shows the worship of the Bodhi tree by four figures.
Buddha had attained enlightenment under the Bodhi tree at Bodh Gaya. Here the
tree symbolizes the presence of Buddha.

23.

Solution: d)

The first century of the Christian era's revolutionary change, had far-reaching effects,
not only on the art of India, but also on the artistic development of Buddhist
countries of Asia. Buddha who was hitherto designated only by a symbol, was
conceived in human form. His person was given some of the 32 suspicious bodily
signs associated with the Mahapurushalakshana, such as the protuberance of the
skull, the hair-knot, bindi between the eyebrows and elongated ears. This change
came about as a result of the new changes that had crept into the religious outlook of
Buddhism due to the influence of the Devotional School of Hindu Philosophy,
requiring the worship of personal gods.
It must have exercised profound influence on the religious approach of the masses
towards Buddhism. The image becomes henceforth the main element of sculpture
and worship. Possibly, the emergence of the image of Buddha in Gandhara and in
Mathura was a parallel development. In each case, it was produced by the local artist
craftsmen working in the local tradition. At Mathura it clearly emerges from
the Yaksha tradition. The Gandhara image might seem to resemble Apollo in some
extraneous forms and does look characteristically Greco-Roman in drapery, but even
there most of the images represent Buddha as seated in the typically Indian Yogic
posture, a feature completely unknown to the Hellenistic tradition of art.

24.

Solution: c)

http://en.wikipedia.org/wiki/Bodhisattva#Bodhisattva_ideal
The Buddhist religion greatly flourished under the patronage of Kushan emperors,
and several images of the Buddha and Bodhisattavas were produced after the
earlier Yaksha types. Here, we may point out the difference between a Buddha and
Bodhisattava. Buddha is one who has attained the enlightenment of supreme
knowledge, while the Bodhisattava is still a candidate for it. A typical example of the

http://www.insightsonindia.com

Insights Mock Tests - 2015


TEST 22 SOLUTIONS

image of Buddha, as it was evolved by the Kushan sculptor in the 2nd century A.D.
shows him seated cross-legged on a

25.

Solution: a)

Medieval sculpture style came right after Gupta sculpture style.


The characteristic of this new form of style of art is the difference with classical art in
attitude, if not in skill and aptitude. Loveliness and idealisation are still the artist's
passion as they were for artists in the early classical period, but love of the ornate,
decorative details is now dominant over classic simplicity. There is more
complication, ornamentation and enrichment. There is an erroneous view that the
Indian artist was a strict conformist with the rules laid down in
the shilpasastras specifying how the gods of the Indian pantheon are to be shown in
images. One look at the variety and individuality of Indian sculpture will clearly
demonstrate that as styles went on developing the sculptors frequently departed
from the texts and rules laid down, and delighted in those departures and the
liberties they took with the bodies of humans and even of gods and goddesses.

26.

Solution: d)

In the middle of the 8th century the Rashtrakutas wrested power from the
Chalukyas. They created the greatest wonder of medieval Indian art in their Kailasa
temple at Ellora. Quarried out of a hill and solid rocks, it is sculptured on a grand
scale. The bold and magnificent carving in this temple shows the Rashtrakuta style of
tall and powerfully built figures, reflecting with spiritual and physical poise. The
beautiful architectural rock sculpture from Cave No.29 at Ellora shows the marriage
of Siva and Parvati. Siva holding the hand of the bashful Parvati occupies the centre
of the composition. To the right Brahma, the creator, is actively engaged in stirring
up the flames of the sacred fire. The parents of Parvati stand behind her to offer their
daughter to the great god. A number of gods assembled to witness the function are
shown hovering above the principal figures. The dignified grace of the divine couple
and the gentle solemnity of the occasion have been portrayed by the sculptor with a
masterly skill.
Since its an important part of Indian architecture, you may also consider reading the
introduction of http://en.wikipedia.org/wiki/Kailasa_temple,_Ellora

http://www.insightsonindia.com

Insights Mock Tests - 2015


27.

TEST 22 SOLUTIONS

Solution: a)

Another magnificent sculpture at Ellora is a panel depicting Ravana shaking mount


Kailasa. In this remarkable scene the quivering of the mountain can be felt, and
Parvati is shown greatly agitated, turning to Siva, grasping his hand in fear while her
maid takes to flight but the Great God is unmoved and holds on fast, pressing down
the mountain with his foot. The lower half of the composition exhibits Ravana
exerting all the force of his twenty arms against the mountain.
A classic panel showing the king of the Naga and his queen, belonging to Ajanta, 5th
century A.D. shows them seated on a throne attended by a hand maiden. The
sculptural work at Ajanta merits as great attention as the world famous wall
paintings.
The Vakataka traditions are derived from the earlier Satavahana which can be clearly
seen in the many carvings of Ajanta and in the painted and carved figures at Ajanta.
It is only the decorative elements, chiefly composed of pearls and ribbons, so
characteristic of the Gupta-Vakataka age, that distinguishes them from the simpler,
but notable sculpture of Amravati.

28.

Solution: d)

The earliest examples of miniature painting in India exist in the form of illustrations
to the religious texts on Buddhism executed under the Palas of the eastern India and
the Jain texts executed in western India during the 11th-12th centuries A.D.
The Pala painting is characterised by sinuous line and subdued tones of colour. It is a
naturalistic style which resembles the ideal forms of contemporary bronze and stone
sculpture, and reflects some feeling of the classical art of Ajanta. A fine example of
the typical Buddhist palm-leaf manuscript illustrated in the Pala style exists in the
Bodleian Library, Oxford, England. It is a manuscript of the Astasahasrika
Prajnaparamita, or the perfection of Wisdom written in eight thousand lines. It was
executed at the monastery of Nalanda in the fifteenth year of the reign of the Pala
King, Ramapala, in the last quarter of the eleventh century. The manuscript has
illustrations of six pages and also on the insides of both wooden covers.
The Pala art came to a sudden end after the destruction of the Buddhist monasteries
at the hands of Muslim invaders in the first half of the 13th century. Some of the
monks and artists escaped and fled to Nepal, which helped in reinforcing the existing
art traditions there.

29.

Solution: b)

http://www.insightsonindia.com

Insights Mock Tests - 2015


TEST 22 SOLUTIONS

Within an ecological food chain, consumers are categorized into three


groups: primary consumers, secondary consumers, and the tertiary
consumers.[2] Primary consumers are usually herbivores, feeding on plants and
fungus. Secondary consumers, on the other hand, are mainly carnivores, and prey on
other animals. Omnivores, who feed on both plants and animals, can also be
considered a secondary consumer. Tertiary consumers, sometimes also known
as apex predators, are usually at the top of food chains, capable of feeding on
secondary consumers and primary consumers. Tertiary consumers can be either fully
carnivorous or omnivorous. Humans are one such example of a tertiary consumer.

30.

Solution: d)

The Pala period (750 A.D. to the middle of the 12th century) witnessed the last great
phase of Buddhism and of the Buddhist art in India.
The Buddhist monasteries (mahaviharas) of Nalanda, Odantapuri,Vikramsila and
Somarupa were great centres of Buddhist learning and art. A large number of
manuscripts on palm-leaf relating to the Buddhist themes were written and
illustrated with the images of Buddhist deities at these centres which also had
workshops for the casting of bronze images. Students and pilgrims from all over
South-East Asia gathered there for education and religious instruction. They took
back to their countries examples of Pala Buddhist art, in the form of bronzes and
manuscripts which helped to carry the Pala style to Nepal, Tibet, Burma, Sri Lanka
and Java etc. The surviving examples of the Pala illustrated manuscripts mostly
belong to the Vajrayana School of Buddhism.

31. Solution: d)
The Western Indian style of painting prevailed in the region comprising Gujarat,
Rajasthan and Malwa. The motivating force for the artistic activity in Western India
was Jainism just as it was Buddhism in case of the Ajanta and the Pala arts. Jainism
was patronised by the Kings of the Chalukya Dynasty who ruled Gujarat and parts of
Rajasthan and Malwa from 961 A.D. to the end of the 13th century. An enormous
number of Jain religious manuscripts were commissioned from 12th to 16th
centuries by the princes, their ministers and the rich Jain merchants for earning
religious merit. Many such manuscripts are available in the Jain
libraries (bhandaras) which are found at many places in Western India.

32.

Solution: d)

Though no pre-Mughal painting from the Deccan are so far known to exist, yet it can
safely be presumed that sophisticated schools of painting flourished there, making a
http://www.insightsonindia.com

Insights Mock Tests - 2015


TEST 22 SOLUTIONS

significant contribution to the development of the Mughal style in North India. Early
centres of painting in the Deccan, during the 16th and 17th centuries were
Ahmednagar, Bijapur and Golconda. In the Deccan, painting continued to develop
independently of the Mughal style in the beginning. However, later in the 17th and
18th centuries it was increasingly influenced by the Mughal style.
Painting in Hyderabad started with the foundation of the Asafjhi dynasty by Mir
Qamruddin Khan (Chin Qulick Khan) Nizam-ul-Mulk in 1724 A.D. Influence of the
Mughal style of painting on the already existing early styles of Deccani paintings,
introduced by several Mughal painters who migrated to the Deccan during the period
of Aurangzeb and sought patronage there, was responsible for the development of
various styles of painting in the Deccan at Hyderabad and other centres.

33.

Solution: d)

Unlike Mughal painting which is primarily secular, the art of painting in Central
India, Rajasthani and the Pahari region etc. is deeply rooted in the Indian traditions,
taking inspiration from Indian epics, religious texts like the Puranas, love poems in
Sanskrit and other Indian languages, Indian folk-lore and works on musical themes.
The cults of Vaishnavism, Saivism and Sakti exercised tremendous influence on the
pictorial art of these places. Among these the cult of Krishna was the most popular
one which inspired the patrons and artists.
The themes from
theRamayana., the Mahabharata, the Bhagavata, the Siva Purana,
the Naishadacarita, the Usha Aniruddha, the GitaGovinda of Jayadeva,
the Rasamanjari of Bhanudatta, the Amaru Sataka, the Rasikapriya of Kesavadasa,
the Bihari Satasayee and the Ragamala etc., provided a very rich field to the painter
who with his artistic skill and devotion made a significant contribution to the
development of Indian painting.

34.

Solution: d)

The Wildlife Institute of India (WII) is an autonomous institution under the Ministry
of Environment and Forests, Government of India.
WII carries out wildlife research in areas of study like Biodiversity, Endangered
Species, Wildlife Policy, Wildlife Management, Wildlife Forensics, Spatial Modeling,
Ecodevelopment, and Climate Change.
launches initiative for allowing PSU and Companies to adopt endangered species
such as Great Indian bustard, Gharial, Lesser Florican, Snow Leopard etc. Spendings
will be counted under CSR in Companies Act 2013. This is because Budget 2015 has
reduced its funding by 25%.
http://www.insightsonindia.com

Insights Mock Tests - 2015


35.

TEST 22 SOLUTIONS

Solution: c)

http://kids.nceas.ucsb.edu/biomes/grassland.html

36.

Solution: d)

Under several personal laws of all religions, leprosy for more than two years serves as
a legitimate ground for divorce or separation between spouses. Under the State
Beggary Acts, persons with leprosy are put under the same category as those with
mental ailments, and medical examination, arrest and detention of persons affected
by leprosy is allowed.
The Life Insurance Corporation Act charges higher premium rates from persons with
leprosy. Several State Municipal and Panchayat Raj Acts bar persons with leprosy
from holding or contesting civic posts.
In its 256th report submitted to the Law Ministry on Tuesday, the commission listed
many laws that discriminate against leprosy patients.
The commission has prepared a draft Bill to eliminate discrimination against persons
affected with leprosy.
The commission recommended the repeal or amendment of all laws carrying
discriminatory provisions. The panel recommended an end to the segregation of
persons with leprosy, many of whom live in 850 leprosy colonies across India.
The Rehabilitation Council of India Act and the Persons with Disabilities Act do not
include all categories of persons with leprosy.
Recommendations are here:
http://pib.nic.in/newsite/PrintRelease.aspx?relid=118021
Also go through http://en.wikipedia.org/wiki/Leprosy_in_India
This is an important topic. Properly go through it.

37.

Solution: c)

Eligibility: Available to people in the age group of 18 to 50 and having a bank


account. People who join the scheme before completing 50 years can, however,
continue to have the risk of life cover up to the age of 55 years subject to payment of
premium.
Premium: Rs.330 per annum. It will be auto-debited in one instalment.

http://www.insightsonindia.com

Insights Mock Tests - 2015


TEST 22 SOLUTIONS

Payment Mode: The payment of premium will be directly auto-debited by the bank
from the subscribers account.
Risk Coverage: Rs.2 Lakh in case of death for any reason.
Terms of Risk Coverage: A person has to opt for the scheme every year. He can also
prefer to give a long-term option of continuing, in which case his account will be
auto-debited every year by the bank.
Who will implement this Scheme?: The scheme will be offered by Life Insurance
Corporation and all other life insurers who are willing to join the scheme and tie-up
with banks for this purpose.
Government Contribution:
(i) Various other Ministries can co-contribute premium for various categories of
their beneficiaries out of their budget or out of Public Welfare Fund created in this
budget out of unclaimed money. This will be decided separately during the year.
(ii)

Common Publicity Expenditure will be borne by Government.

38.

Solution: a)

The Internal Working Group of the Reserve Bank of India under the Chairmanship of
Shri B Mahapatra had submitted the final Report on the implementation of
Countercyclical Capital Buffer (CCCB) in July, 2014. The Report made
recommendations in areas such as indicators that may be used for CCCB decisions,
thresholds for activating the buffer, lead time for announcement of buffer, etc.
The aim of the Countercyclical Capital Buffer (CCCB) regime is twofold. Firstly, it
requires banks to build up a buffer of capital in good times which may be used to
maintain flow of credit to the real sector in difficult times. Secondly, it achieves the
broader macro-prudential goal of restricting the banking sector from indiscriminate
lending in the periods of excess credit growth that have often been associated with
the building up of system-wide risk.

39.

Solution: d)

Most desert species have found remarkable ways to survive by evading drought.
Desert succulents, such as cacti or rock plants (Lithops) for example, survive dry
spells by accumulating moisture in their fleshy tissues. They have an extensive
system of shallow roots to capture soil water only a few hours after it has rained.
Additionally, many cacti and other stem-succulent plants of hot deserts present
columnar growth, with leafless, vertically-erect, green trunks that maximize light

http://www.insightsonindia.com

Insights Mock Tests - 2015


TEST 22 SOLUTIONS

interception during the early and late hours of the day, but avoid the midday sun,
when excessive heat may damage plant tissues.
One of the most effective drought-survival adaptations for many species is the
evolution of an ephemeral life-cycle. An ephemeral life cycle is characterized by a
short life and the capacity to leave behind very hardy forms of propagation. This
ability is found not only in plants but also in many invertebrates. Desert ephemerals
are amazingly rapid growers capable of reproducing at a remarkably high rate during
good seasons.
http://en.wikipedia.org/wiki/Larrea_tridentata#Desert_adaptation

40.

Solution: a)

Carbon dioxide readily dissolves in water and the oceans provide a huge reservoir of
carbon. Across the world's oceans there is a continual cycle of equilibration of
dissolved carbon dioxide in water with carbon dioxide in the atmosphere.
Around 88 thousand million tonnes of carbon is released from the surface of the
world's oceans each year, with an annual uptake by the oceans of 90 thousand
million tonnes. Consequently, the net uptake of carbon dioxide by oceans is
estimated to be approximately 2 thousand million tonnes annually.
The carbon dioxide which dissolves in our oceans occurs in three main forms. Aside
from the normal carbon dioxide form, it is also found as bicarbonate and carbonate
ions. Most, about 90 percent, exists as bicarbonate with carbonate ions acting as the
link between carbon dioxide and bicarbonate. As concentrations of carbon dioxide
increase the supply of carbonate ions becomes limited and so the oceans become less
and less able to take up carbon dioxide from the atmosphere.

41. Solution: a)
One way to increase the carbon sequestration efficiency of the oceans is to add
micrometre-sized iron particles in the form of either hematite (iron oxide) or
melanterite (iron sulfate) to certain regions of the ocean. This has the effect of
stimulating growth of plankton. Iron is an important nutrient for phytoplankton,
usually made available via upwelling along the continental shelves, inflows from
rivers and streams, as well as deposition of dust suspended in the atmosphere.
Natural sources of ocean iron have been declining in recent decades, contributing to
an overall decline in ocean productivity (NASA, 2003) Yet in the presence of iron
nutrients plankton populations quickly grow, or 'bloom', expanding the base
of biomass productivity throughout the region and removing significant quantities of
CO2 from the atmosphere via photosynthesis.
http://www.insightsonindia.com

Insights Mock Tests - 2015


42.

TEST 22 SOLUTIONS

Solution: d)

The ocean plays an important part in the carbon cycle. Overall, the ocean is called a
carbon sink because it takes up more carbon from the atmosphere than it gives up.
Carbon dioxide from the atmosphere dissolves in the surface waters of the ocean.
Some of the carbon dioxide stays as dissolved gas, but much of it gets turned into
other things. Photosynthesis by tiny marine plants (phytoplankton) in the sunlit
surface waters turns the carbon into organic matter. Many organisms use carbon to
make calcium carbonate, a building material of shells and skeletons. Other chemical
processes create calcium carbonate in the water. The using up of carbon by biological
and chemical processes allows more carbon dioxide to enter the water from the
atmosphere.

43.

Solution: d)

The common name given to the atmospheric gases used


in breathing and photosynthesis is air. By volume, dry air contains 78.09% nitrogen,
20.95%oxygen,[1] 0.93% argon, 0.039% carbon dioxide, and small amounts of other
gases. Air also contains a variable amount of water vapor, on average around 1% at
sea level, and 0.4% over the entire atmosphere.
Stratosphere contains the ozone layer, which is the part of Earth's atmosphere that
contains relatively high concentrations of that gas.
44.

Solution: a)

Critically Endangered Great Indian Bustards (GIB) is the State bird of Rajasthan.
Total 150 GIB live in India out of them almost 100 are in Rajasthan alone, and
remaining in Maharashtra, Gujarat, and Andhra Pradesh.
But the new census says only 50 GIB left in Rajasthan. And zero GIB left in Uttar
Pradesh and Bihar. Thar desert of Rajasthan is the breeding ground for GIB.
Officially its called Desert National Park but legally its only a sanctuary (therefore
human activities permitted). Even Supreme court permitted school constructions
here. Other dangers are installation of windmills and shortage of forest guards.

45.

Solution: d)

The principle fuel used as a petrol substitute for road transport vehicles is
bioethanol. Bioethanol fuel is mainly produced by the sugar fermentation process,

http://www.insightsonindia.com

Insights Mock Tests - 2015


TEST 22 SOLUTIONS

although it can also be manufactured by the chemical process of reacting ethylene


with steam.
The main sources of sugar required to produce ethanol come from fuel or energy
crops. These crops are grown specifically for energy use and include corn, maize and
wheat crops, waste straw, willow and popular trees, sawdust, reed canary grass, cord
grasses, jerusalem artichoke, myscanthus and sorghum plants. There is also ongoing
research and development into the use of municipal solid wastes to produce ethanol
fuel.
Other sources are grass, rice straw, sea weed (macro algae) etc.

46.

Solution: d)

Using bio-refining, It is possible to turn macro algae (sea weeds) into bio-ethanol
(fuel), and byproducts like agar, pigment, lipids and fertilizers are added benefits.
Important seaweed species are Gelidiella acerosa and Gracilaria dura from Western
coast of India, and Gelidium pusillum collected from southeast coast.
But the challenge is that we need to grow millions of tones of sea weed. But the good
thing is seaweed farming can provide large scale employment to many fishermen,
and reduce coastal eutrophication and global warming.
Eutrophication is a common phenomenon in coastal waters. In contrast to
freshwater systems, nitrogen is more commonly the key limiting nutrient of marine
waters; thus, nitrogen levels have greater importance to understanding
eutrophication problems in salt water. Estuaries tend to be naturally eutrophic
because land-derived nutrients are concentrated where run-off enters a confined
channel. Upwelling in coastal systems also promotes increased productivity by
conveying deep, nutrient-rich waters to the surface, where the nutrients can be
assimilated by algae.
Sea grass can consume the extra mineral and consume carbon dioxide in
photosynthesis to reduce coastal eutrophication.
47.

Solution: a)

NGT can deliver justice only in matters pertaining to following acts- Water Act, The
Water Cess Act, The Forest Conservation Act, The Air Pollution Act, Environment
Protection Act, Public Liability Insurance Act and The Biological Diversity Act. Forest
ministry contends that NGT cant deliver any Verdict on matters related to climate
change. Because climate change subject is covered under international conventions
and protocols.

http://www.insightsonindia.com

Insights Mock Tests - 2015


TEST 22 SOLUTIONS

About NGT: The National Green Tribunal has been established in 2010 under the
National Green Tribunal Act 2010 for effective and expeditious disposal of cases
relating to environmental protection and conservation of forests and other natural
resources including enforcement of any legal right relating to environment and
giving relief and compensation for damages to persons and property and for matters
connected therewith or incidental thereto. It is a specialized body equipped with the
necessary expertise to handle environmental disputes involving multi-disciplinary
issues.

48.

Solution: a)

Relative Humidity is the ratio between amount of water vapour present in air vs. the
amount of water vapour the air can hold. During high temperature high relative
humidity scenario, human body tries to control temperature by evaporating moisture
as per commands from hypothalamus. But since outside air already has high relative
humidity, it cannot hold any more moisture, hence sweat doesnt evaporate quickly,
therefore it appears as if we are sweating more profusely during high humid climate.

49.

Solution: a)

http://en.wikipedia.org/wiki/3D_printing#Bio-printing
3D printing (or additive manufacturing, AM) is any of various processes used to
make a three-dimensional object. In 3D printing, additive processes are used, in
which successive layers of material are laid down under computer control. These
objects can be of almost any shape or geometry, and are produced from a 3D
model or other electronic data source. A 3D printer is a type of industrial robot.

50.

Solution: a)

The Rights of Transgender Persons Bill 2014 seeks to provide framework for the
formulation and implementation of a comprehensive national policy for ensuring
overall development of the transgender persons and their welfare. Moreover,

Two percent reservation in primary, secondary and higher education and in


government jobs.
Establishment of Employment Exchange, National and State Commissions for
Trasngender Persons and Special Transgender Rights Courts.
No child who is transgender will be separated from his or her parents on the
grounds of being a transgender except on an order of competent court.
Penalty for hate speech against transgender persons includes imprisonment
extending upto one year and with fine.

http://www.insightsonindia.com

Insights Mock Tests - 2015


TEST 22 SOLUTIONS

This bill will help government take necessary steps in order to ensure that
transgender persons enjoy the right to life with dignity and to personal liberty
guaranteed by the Constitution.
It should be noted that 29 nations and leading democracies in the world including
US, UK, Canada, France, Australia, Italy and Singapore have legislations protecting
rights of transgender persons.

51. Solution: c)
A lake ecosystem includes biotic (living) plants, animals and micro-organisms, as
well as abiotic (nonliving) physical and chemical interactions.
Lake ecosystems are a prime example of lentic ecosystems. Lentic refers to stationary
or relatively still water, from the Latin lentus, which means sluggish. Lentic waters
range from ponds to lakes to wetlands, and much of this article applies to lentic
ecosystems in general. Lentic ecosystems can be compared with lotic ecosystems,
which involve flowing terrestrial waters such as rivers and streams. Together, these
two fields form the more general study area of freshwater or aquatic ecology.
Lentic systems are diverse, ranging from a small, temporary rainwater pool a few
inches deep to Lake Baikal, which has a maximum depth of 1740 m. The general
distinction between pools/ponds and lakes is vague, but Brown states that ponds and
pools have their entire bottom surfaces exposed to light, while lakes do not. In
addition, some lakes become seasonally stratified (discussed in more detail below.)
Ponds and pools have two regions: the pelagic open water zone, and the benthic
zone, which comprises the bottom and shore regions. Since lakes have deep bottom
regions not exposed to light, these systems have an additional zone,
the profundal.[3] These three areas can have very different abiotic conditions and,
hence, host species that are specifically adapted to live there.
Bacteria are present in all regions of lentic waters. Free-living forms are associated
with decomposing organic material, biofilm on the surfaces of rocks and plants,
suspended in the water column, and in the sediments of the benthic and profundal
zones. Other forms are also associated with the guts of lentic animals as parasites or
in commensal relationships. Bacteria play an important role in system metabolism
through nutrient recycling, which is discussed in the Trophic Relationships section.

52.

Solution: d)

E-Samiksha is designed by National Informatics Centre (NIC). It has been developed


to make it a highly interactive web-based, user-friendly and customised system.

http://www.insightsonindia.com

Insights Mock Tests - 2015


TEST 22 SOLUTIONS

E-Samiksha will be used to monitor budget-related projects and also for monitoring
the infrastructure target and board meeting follow-up.
Currently, it is being used by Cabinet Secretariat, Prime Ministers Office and other
Ministries for monitoring progress implementation of various programmes and
follow-up of meetings.
It is not mandatory for states to follow this system.

53.

Solution: d)

The ship is designed by the Directorate of Naval Design, Indian Navys in-house
design organization and made of entirely of Indian steel. It is 163-metre-long vessel
and weighs approximately 3,000 tons. It is Propelled by four gas turbines, and
capable to achieve maximum speed of over 30 knots at a displacement of
approximately 7,300 tons. It is fitted with state-of-the-art weapons, advance Action
Information System (AIS), Integrated Platform Management system (IPMS),
sophisticated Power Distribution System (PDS) and a Combat Management System.
It is also equipped with the Israeli Multi-Function Surveillance Threat Alert Radar
(MF-STAR). MF-STAR will provide it with targeting information to 32 Barak 8 longrange surface-to-air missiles on board the warship. It will also be carrying 8 BrahMos
missiles. It has four 30 mm rapid-fire guns which will provide the ship with close-indefence capability and also has an MR gun that will enable ship to provide effective
naval gunfire support.

54.

Solution: c)

Commensalism, in biology, a relationship between individuals of two species in


which one species obtains food or other benefits from the other without either
harming or benefiting the latter.
The commensalthe species that benefits from the associationmay
obtainnutrients, shelter, support, or locomotion from the host species, which is
unaffected. The commensal relation is often between a larger host and a smaller
commensal. The host organism is essentially unchanged by the interaction, whereas
the commensal species may show great morphological adaptation. This relationship
can be contrasted with mutualism, in which both species benefit.
One of the best-known examples of a commensal is the remora (family Echineidae)
that rides attached to sharks and other fishes. Remoras have evolved on the top of
their heads a flat oval sucking disk structure that adheres to the bodies of their hosts.
Both remoras and pilot fishes feed on the leftovers of their hosts meals. A
commensal relationship based on shelter is seen in anemone fishes, such
http://www.insightsonindia.com

Insights Mock Tests - 2015


TEST 22 SOLUTIONS

as Amphiprion percula, which live unharmed among the stinging tentacles of sea
anemones, where they are protected from predators. Other examples of commensals
include bird species, such as the great egret (Ardea alba), that feed on insects turned
up by grazing mammals or on soil organisms stirred up by plowing. Various
biting lice, fleas, and louse flies are commensals in that they feed harmlessly on
the feathers of birds and on sloughed-off flakes ofskin from mammals.

55.

Solution: d)

http://indianexpress.com/article/india/india-others/astra-missile-launchedsuccessfully/
http://en.wikipedia.org/wiki/Astra_(missile)

56.

Solution: d)

The profundal zone is a deep zone of an inland body of freestanding water, such as
a lake or pond, located below the range of effective light penetration. This is typically
below the thermocline, the vertical zone in the water through
which temperature drops rapidly. The lack of light in the profundal zone determines
the type of biological community that can live in this region, which is distinctly
different from the community in the overlying waters. The profundal zone is part of
the aphotic zone.
The limnetic zone is the well-lit, open surface waters in a lake, away from the shore.
The vegetation of the littoral zone surrounds this expanse of open water and it is
above the profundal zone. This is the main photosynthetic body of the lake. This zone
produces the oxygen and food that support the lake's consumers.
It can be defined as the lighted surface waters in the area where the lake bottom is
too deep and unlit to support rooted aquatic plants. This area is occupied by a variety
of phytoplankton, consisting of algae and cyanobacteria, as well as zooplankton,
smallcrustaceans, and fish. Most photosynthesis takes place in this part of the lake.

57.Solution: a)
The Indira Gandhi Matritva Sahyog Yojana (IGMSY) was a flagship program of the
government of India introduced in 2010 under the Ministry of Women and Child
Development. It is a conditional cash transfer scheme.pregnant and lactating women
19 years of age and older for first two live births are eligible for the scheme.Its goal is

http://www.insightsonindia.com

Insights Mock Tests - 2015


TEST 22 SOLUTIONS

to partly compensate them for wage-loss during childbirth and childcare and to
provide conditions for safe delivery and good nutrition and feeding practices.
It envisions the achievement of its objectives by:

Promoting appropriate practice, care and institutional service utilization


during pregnancy, delivery and lactation

Encouraging the women to follow (optimal) nutrition and feeding practices,


including early and exclusive breast-feeding for the first six months; and

Providing cash incentives for improved health and nutrition to pregnant and
lactating mothers.

IGMSY provides financial assistance as grant-in-aid to state governments

58.

Solution: c)

http://en.wikipedia.org/wiki/Mission_Indradhanush

59.

Solution: c)

This recent earthquake in Nepal may have occurred because of two converging
tectonic plates, the India plate and the overriding Eurasian plate. These tectonic
plates are the large, thin, relatively rigid and are moving relative to one another on
the outer surface of the Earth. Both of these plates are slowly moving and sometimes
get stuck at their edges due to friction. Earthquake might have occured because of
these overriding plates after the stress on the edge overcomes the friction. It releases
tremendous energy in form of waves that travel through the Earths crust and causes
earthquake on the surface.

60.

Solution: a)

Antibiosis is a biological interaction between two or more organisms that is


detrimental to at least one of them; it can also be an antagonistic association between
an organism and the metabolic substances produced by another. Examples of
antibiosis include the relationship between antibiotics and bacteria and the
relationship between animals and disease-causing pathogens.
Antibiosis is commonly found and studied in host plants and the insects which feed
upon them.

http://www.insightsonindia.com

Insights Mock Tests - 2015


TEST 22 SOLUTIONS

"Antibiosis resistance affects the biology of the insect so pest abundance and
subsequent damage is reduced compared to that which would have occurred if the
insect was on a susceptible crop variety. Antibiosis resistance often results in
increased mortality or reduced longevity and reproduction of the insect."

61. Solution: d)
Prime Minister Narendra Modi on 24 April 2015 inaugurated Indias Healthcare
Tourism Portal www.indiahealthcaretourism.com at the SAAARC Trade Mart in the
Global Exhibition on Services (GES). It was inaugurated on the occasion of three-day
Global Exhibition on Services (GES)
The portal is developed by the Union Governments Department of Commerce and
the Services Export Promotion Council (SEPC). It is a comprehensive one-point
information site that covers hospital and travel-related information in India.
Presently, it covers 124 accredited medical facilities. It includes 93 medical centers,
30 Ayurveda and Wellness centers and 1 special category center. Among these
medical facilities 74 facilities are located in Tier I cites and the rest are in Tier II
cities. These covered accredited medical facilities on portal are easily locatable. It is
also easy to find by search options by location, medical specialty, key procedures,
language options etc. Details such as costs related to treatment in hospitals, visa and
travel formalities, tariff options on stay etc are also mentioned on the portal.

62.

Solution: c)

An estuary is a partly enclosed coastal body of brackish water with one or more rivers
or streams flowing into it, and with a free connection to the open sea.
Estuaries form a transition zone between river environments and maritime
environments. They are subject both to marine influences - such as tides, waves, and
the influx of saline water - and to riverine influences - such as flows of fresh water
and sediment. The inflows of both sea water and fresh water provide high levels of
nutrients both in the water column and in sediment, making estuaries among the
most productive natural habitats in the world
Most existing estuaries formed during the Holocene epoch with the flooding of rivereroded or glacially scoured valleys when the sea level began to rise about 10,00012,000 years ago.[4] Estuaries are typically classified according to
their geomorphological features or to water-circulation patterns. They can have
many different names, such as bays, harbors, lagoons, inlets, or sounds, although
some of these water bodies do not strictly meet the above definition of an estuary and
may be fully saline.

http://www.insightsonindia.com

Insights Mock Tests - 2015


TEST 22 SOLUTIONS

http://oceanservice.noaa.gov/education/kits/estuaries/media/supp_estuar04_delta
.html

63.

Solution: a)

These conservation efforts will be taken to protect this species of Langur found in
Patharia Hills Reserve Forest on either side of the fenced border along both nations
in Karimganj district of Assams Barak Valley. Decision in this regard was taken into
effect at the meeting of higher officials of both countries.
The primary focus of this conservation effort is to preserve the habitat for the
survival of the primates in the Patharia Hills Reserve Forest that is posing a major
threat to the wildlife present in the forest. It also seeks to spread awareness
campaigns which will be conducted in the fringe villages to urge people not to
destroy the forests for firewood. Spectacled Monkey is popularly known as the
Chasme vala bandar. They have white spectacle like patches around the eyes. The
Patheria Hills reserve Forest is spread over an area of around seven thousand
hectares on the Indian side and consists of two blocks Patheria A and B.
It is only the forest in Assam, where the Spectacled Monkey is found while Mizoram
and Tripura are the other two states in the country where it inhabits. Apart from
spectacled Langur, the reserve forest is also home to the endangered Hoolock Gibbon
and Golden Langur.
64.

Solution: c)

Both are correct. Please refer the book.

65.

Solution: b)

The report was published by the United Nations (UN) Sustainable Development
Solutions Network (SDSN).
Key facts from report: Top five countries: Switzerland (1st), Iceland (2nd), Denmark
(3rd), Norway (4th) and Canada (5th).
Bottom five countries: Rwanda (154th), Benin (155th), Syria (156th), Burundi (157th)
and Togo (158th).
Other countries: United States (15th), United Kingdom (21th), Japan (46th) and
China (84th). India is ranked below countries like Pakistan (81st), Bangladesh
(109th), Ukraine (111th) and Iraq (112th).

http://www.insightsonindia.com

Insights Mock Tests - 2015


TEST 22 SOLUTIONS

The report aims at influencing government policy. The report is prepared based upon
study undertaken by Gallup World Poll. The data is used to ranked nations based
upon variables such as real GDP per capita, healthy life expectancy, corruption levels
and social freedoms.

66.

Solution: a)

Micronutrients are nutrients required by humans and other organisms throughout


life in small quantities to orchestrate a range of physiological functions.[1] For people,
they include dietary trace minerals in amounts generally less than 100
milligrams/day - as opposed to macrominerals which are required in larger
quantities. The microminerals or trace
elements includes..atleast iron,cobalt, chromium, copper, iodine, manganese, seleniu
m, zinc and molybdenum. Micronutrients also include vitamins, which are organic
compounds required as nutrients in tiny amounts by an organism,[2] as well
as phytochemicals.
There are about seven nutrients essential to plant growth and health that are only
needed in very small quantities. Though these are present in only small quantities,
they are all necessary:

Boron is believed to be involved in carbohydrate transport in plants; it also


assists in metabolic regulation. Boron deficiency will often result in bud
dieback.

Chlorine is necessary for osmosis and ionic balance; it also plays a role
in photosynthesis.

Copper is a component of some enzymes. Symptoms of copper deficiency


include browning of leaf tips and chlorosis.

Iron is essential for chlorophyll synthesis, which is why an iron deficiency


results in chlorosis.

Manganese activates some important enzymes involved


in chlorophyll formation. Manganese deficient plants will
develop chlorosis between the veins of its leaves. The availability of
manganese is partially dependent on soil pH.

Molybdenum is essential to plant health. Molybdenum is used by plants to


reduce nitrates into usable forms. Some plants use it for nitrogen fixation,
thus it may need to be added to some soils before seeding legumes.

Zinc participates in chlorophyll formation, and also activates many enzymes.


Symptoms of zinc deficiency include chlorosis and stunted growth.

http://www.insightsonindia.com

Insights Mock Tests - 2015


67.

TEST 22 SOLUTIONS

Solution: c)

Calbuco volcano in southern Chile erupted on 23 April 2015 for the first time after
more than 42 years. It had erupted twice in the space of a few hours which led to
billowing of huge ash cloud over a sparsely populated, mountainous area in southern
Chile.
Calbuco Volcano is one of the most active volcanoes of the Southern Chilean Andes.
It should be noted that Chile has the second largest chain of volcanoes in the world
after Indonesia which has about 500 that are potentially active. Both countries fall on
the Ring of Fire.

68.

Solution: d)

In ecology, energy flow, also called the calorific flow, refers to the flow
of energy through a food chain. In an ecosystem,ecologists seek to quantify the
relative importance of different component species and feeding relationships.
A general energy flow scenario follows:

Solar energy is fixed by the photoautotrophs, called primary producers, like


green plants. Primary consumers absorb most of the stored energy in the plant
through digestion, and transform it into the form of energy they need, such
as adenosine triphosphate (ATP), through respiration. A part of the energy
received by primary consumers, herbivores, is converted tobody heat (an
effect of respiration), which is radiated away and lost from the system. The
loss of energy through body heat is far greater in warm-blooded animals,
which must eat much more frequently than those that are cold-blooded.
Energy loss also occurs in the expulsion of undigested food (egesta)
by excretion or regurgitation.

Secondary consumers, carnivores, then consume the primary consumers,


although omnivores also consume primary producers. Energy that had been
used by the primary consumers for growth and storage is thus absorbed into
the secondary consumers through the process of digestion. As with primary
consumers, secondary consumers convert this energy into a more suitable
form (ATP) during respiration. Again, some energy is lost from the system,
since energy which the primary consumers had used for respiration and
regulation of body temperature cannot be utilised by the secondary
consumers.

Tertiary consumers, which may or may not be apex predators, then consume
the secondary consumers, with some energy passed on and some lost, as with
the lower levels of the food chain.

http://www.insightsonindia.com

Insights Mock Tests - 2015


TEST 22 SOLUTIONS

A final link in the food chain are decomposers which break down the organic
matter of the tertiary consumers (or whichever consumer is at the top of the
chain) and release nutrients into the soil. They also break down plants,
herbivores and carnivores that were not eaten by organisms higher on the
food chain, as well as the undigested food that is excreted by herbivores and
carnivores. Saprotrophic bacteria and fungi are decomposers, and play a
pivotal role in the nitrogen andcarbon cycles.

The energy is passed on from trophic level to trophic level and each time about 90%
of the energy is lost, with some being lost as heat into the environment (an effect
of respiration) and some being lost as incompletely digested food (egesta). Therefore,
primary consumers get about 10% of the energy produced by autotrophs, while
secondary consumers get 1% and tertiary consumers get 0.1%. This means the top
consumer of a food chain receives the least energy, as a lot of the food chain's energy
has been lost between trophic levels. This loss of energy at each level limits typical
food chains to only four to six links.

69.

Solution: d)

India has emerged as the fourth largest supplier of generic medicines to the United
States. It was announced by Minister of State (MoS) for Chemicals and Fertilizers
Hansraj Ahir gave this information in the Rajya Sabha in reply to a question. Indias
generic medicines exports have touched over US 4 billion dollars in 2013-14 in spite
of stringent regulatory measures imposed by US. Union Government also announced
that Indian pharmaceutical companies are already exporting generic medicines to
Africa, West Asia and European countries.

70.

Solution: a)

http://www.downtoearth.org.in/content/hfc-regulation-countries-yet-reachconsensus
The United Nations Framework Convention on Climate Change (UNFCCC) is an
international environmental treaty (currently the only international climate policy
venue with broad legitimacy, due in part to its virtually universal membership)
negotiated at the United Nations Conference on Environment and Development
(UNCED informally known as the Earth Summit, held in Rio de Janeiro from 3 to 14
June 1992. The objective of the treaty is to "stabilize greenhouse gas concentrations
in the atmosphere at a level that would prevent
dangerous anthropogenic interference with the climate system"

http://www.insightsonindia.com

Insights Mock Tests - 2015


TEST 22 SOLUTIONS

71. Solution: c)
http://en.wikipedia.org/wiki/Deciduous

72.

Solution: d)

In the past, several studies have explored why some people are found to be mosquito
magnets while others remain relatively unattractive. Carbon dioxide and octenol in
exhaled air, amount of lactic acid emitted, body heat, secretor status, moisture,
physical movements and the colour of clothing are some factors that lead to
differences in attractiveness to mosquitoes.
Mosquitoes are attracted to carbon dioxide that humans exhale and it has been
observed that adults are more susceptible to mosquito bites than children due to the
larger amounts of carbon dioxide exhaled. Octenol, a chemical found in the human
breath, also attracts mosquitoes and forms a potent combination with carbon dioxide
in alluring mosquitoes. Additionally, the presence of larger amounts of lactic acid
that is emitted through the skin of humans and the production of which is linked
with physical activity as well as consumption of certain foods, are other factors
associated with an increased attractiveness for mosquitoes. This is why people who
sweat profusely tend to be ambushed by mosquitoes. A study of these three factors
was conducted in 2007 and was led by D F Hoel. It was published in the journal of
the American Mosquito Control Association.
http://www.downtoearth.org.in/content/why-mosquitoes-are-more-attracted-someus-others

73.

Solution: a)

The monsoon, which is essentially the seasonal reversal in wind direction, causes
most of the rainfall received in India and some other parts of the world. The primary
cause of monsoons is the difference between annual temperature trends over land
and sea. The apparent position of the Sun with reference to the Earth oscillates from
the Tropic of Cancer to the Tropic of Capricorn. Thus the low pressure region created
by solar heating also changes latitude.
The northeast and southeast trade winds converge in this low pressure zone, which is
also known as the Intertropical Convergence Zone or ITCZ. This low pressure regions
sees continuous rise of moist wind from the sea surface to the upper layers of the
atmosphere, where the cooling means the air can no longer hold so much moisture
resulting in precipitation. The rainy seasons of East Asia, sub-Saharan Africa,
Australia and the southern parts of North America coincide with the shift of ITCZ
towards these regions.

http://www.insightsonindia.com

Insights Mock Tests - 2015


TEST 22 SOLUTIONS

For more read http://timesofindia.indiatimes.com/india/Learning-with-the-TimesWhat-causes-the-monsoon/articleshow/6156778.cms

74.

Solution: d)

The bottom of the pyramid represents the primary producers (autotrophs). The
primary producers take energy from the environment in the form of sunlight or
inorganic chemicals and use it to create energy-rich molecules such as
carbohydrates. This mechanism is called primary production. The pyramid then
proceeds through the various trophic levels to the apex predators at the top.
When energy is transferred from one trophic level to the next, typically only ten
percent is used to build new biomass. The remaining ninety percent goes to
metabolic processes or is dissipated as heat. This energy loss means that productivity
pyramids are never inverted, and generally limits food chains to about six levels.
However, in oceans, biomass pyramids can be wholly or partially inverted, with more
biomass at higher levels.
75.Solution: b)
Western Disturbance is the term used in India, Pakistan, Bangladesh and Nepal to
describe an extratropical storm originating in the Mediterranean, that brings sudden
winter rain and snow to the north-western parts of the Indian subcontinent. This is a
non-monsoonal precipitation pattern driven by the Westerlies. The moisture in these
storms usually originates over the Mediterranean Sea and the Atlantic Ocean.
Extratropical storms are a global, rather than a localized, phenomena with moisture
usually carried in the upper atmosphere (unlike tropical storms where it is carried in
the lower atmosphere). In the case of the subcontinent, moisture is sometimes shed
as rain when the storm system encounters the Himalayas.
Western Disturbances are important to the development of the Rabi crop in the
northern subcontinent, which includes the locally important staple wheat.

76.

Solution: a)

This type of climate is experienced along the eastern coasts of tropical lands,
receiving steady rainfall from the Trade Winds all the time. The rainfall is both
orographic where the moist trades meet upland masses as in eastern Brazil and
convectional due to intense heating during the day and in summer. Its tendency is
towards a summer maximum as in monsoon lands, but without any distinct dry
period. The rhythm of climate as experienced in Cairns, on the eastern coast of
Queensland, under the constant influence of the South-East Trade Winds. and in
http://www.insightsonindia.com

Insights Mock Tests - 2015


TEST 22 SOLUTIONS

summer also affected by the tropical monsoons. Its wettest months are in January
(15.8 inches), February (16.4), March (17.7) and April (12.1), which is summer in the
southern hemisphere.
Approximately 70 per cent of the annual rainfall is concentrated in the four summer
months. There is not month without any rainfall. The range of temperature is typical
of the tropical latitudes with a maximum of 82 degree F. in January and a minimum
of 70degreeF. in July- a range of 12degree F for the year. Due to the steady influence
of the trades. the Tropical Marine Climate is more favourable for habitation, but it is
prone to severe tropical cyclones, hurricanes or typhoons.

77. Solution: c)
The warm current dwelling at the eastern coasts in the tropics makes the area
warmer and prepares it for more evaporation; low pressure zone and high rainfall.
The same phenomenon is found at the western coasts in higher latitudes where warm
currents dwell at the western coasts.
Cyclonic disturbances have nothing to do with the differences in temperature over
such a vast area 20 degrees to 35 degrees.

78.

Solution: a)

http://en.wikipedia.org/wiki/Burmese_pythons_in_Florida
http://www.nytimes.com/2015/04/06/us/the-burmese-python-snake-thats-eatingflorida.html?_r=0

79.

Solution: b)

The Sudan type of climate is characterized by an alternate hot, rainy season and cool,
dry season. In the northern hemisphere, the hot, rainy season normally begins in
May and lasts until September, as in Kano, Nigeria.
The rest of the year is cool and dry. The annual rainfall for Kano, which is located at a
height of 1,539 feet above sea level, 34 inches and is most entirely concentrated in the
summer. But the amount varies from 48 inches at Bathurst, in Gambia on the coast
to only 5 inches at Khartoum, in Sudan in the interior.
Both the length of the rainy season and the annual total rainfall decrease appreciably
from the equatorial region polewards towards the desert fringes. On the whole, the
annual precipitation is less than that of the Tropical Monsoon Climate and the length

http://www.insightsonindia.com

Insights Mock Tests - 2015


TEST 22 SOLUTIONS

of the wet and dry seasons differs with the locality. In the southern hemisphere, the
rainy season is from October to March (the southern summer). Its annual
precipitation of 32 inches also varies much from year to year.

80.

Solution: a)

Deserts are regions of scanty rainfall which may be hot like the hot deserts of the
Saharan type; or temperate as are the mid-latitude deserts like the Gobi. The aridity
of the hot deserts is mainly due to the effects of off-shore Trade Winds, hence they
are also called Trade Wind Deserts. The temperate deserts are rainless because of
their interior location in the temperate latitudes, well away from the rain-bearing
winds.
The major jot deserts of the world are located on the western coasts of continents
between latitudes 15degree and 30degreeN. and S. They include the Sahara Desert,
the largest single stretch of desert, which is 3,200 miles from east to west and at least
1,000 miles wide. Its total area of 3.5 million square miles is larger than all the 50
states of U.S.A. put together. The next biggest desert is the Great Australian Desert
which covers almost half other continent. The other hot deserts are the Arabian
Desert, Iranian Desert, Thar Desert, Kalahari and Namib Deserts. In North America,
the desert extends from Mexico into U.S.A. and is called by different names at
different places, e.g. the Mohave, Sonoran, Californian and Mexican Deserts. In
south America, the Atacama or Peruvian Desert is the driest of all desert with less
than 0.5 inches of rainfall annually.

81.Solution: d)
The Patagonian Desert is more due to its rain-shadow position on the leeward side of
the lofty Andes than to continentality.
The major jot deserts of the world are located on the western coasts of continents
between latitudes 15degree and 30degree N and S. They include the Sahara Desert,
the largest single stretch of desert, which is 3,200 miles from east to west and at least
1,000 miles wide. Its total area of 3.5 million square miles is larger than all the 50
states of U.S.A. put together.

82.

Solutions: b)

Climatic Conditions in the Mid-Latitude deserts is in many ways similar to those of


the hot deserts aridity is the keynote. These inland basins lie hundreds of miles from
the sea, and are sheltered by high mountains all around them. As a result they are cut
off from !he rain-bearing winds. Ocassionally depressions may penetrate the Asiatic
http://www.insightsonindia.com

Insights Mock Tests - 2015


TEST 22 SOLUTIONS

continental mass and bring light rainall in winter, or unexpected convectional may
bless the parched lands with brief showers in summer. For example Cashiar in
western China in the Gobi Desert has most of its 3.5 inches of annual precipitation in
summer. Due to their coldness and elevation snow falls in winter.

83.

Solution: a)

They are entirely confined to the western portion of continental masses, between 30
and 45 north and south of the equator. The basic cause of this type of climate is the
shifting of the wind belts. Though the area around the Mediterranean Sea has the
greatest extent of this type of ' 'winter rain climate', and gives rise to, the more
popular name Mediterranean Climate, the best developed form of this peculiar
climatic is found in Chile. Other regions include California, the south-western tip of
Africa (around Cape Town), southern Australia (in southern Victoria and around
Adelaide, bordering the St. Vincent and Spencer Gulfs), and south-west Australia
(Swanland).

84.

Solution: d)

In general, each trophic level relates to the one below it by absorbing some of the
energy it consumes, and in this way can be regarded as resting on, or supported by,
the next lower trophic level. Food chains can be diagrammed to illustrate the amount
of energy that moves from one feeding level to the next in a food chain. This is called
an energy pyramid. The energy transferred between levels can also be thought of as
approximating to a transfer in biomass, so energy pyramids can also be viewed as
biomass pyramids, picturing the amount of biomass that results at higher levels from
biomass consumed at lower levels.
The efficiency with which energy or biomass is transferred from one trophic level to
the next is called the ecological efficiency. Consumers at each level convert on
average only about 10% of the chemical energy in their food to their own organic
tissue (the ten-percent law). For this reason, food chains rarely extend for more than
5 or 6 levels. At the lowest trophic level (the bottom of the food chain), plants convert
about 1% of the sunlight they receive into chemical energy. It follows from this that
the total energy originally present in the incident sunlight that is finally embodied in
a tertiary consumer is about 0.001%[

85.

Solution: d)

http://www.1yachtua.com/Meditmarinas/Mediterranean_Sailing/mediterranean_winds.shtm
http://www.insightsonindia.com

Insights Mock Tests - 2015


86.

TEST 22 SOLUTIONS

Solution: d)

http://en.wikipedia.org/wiki/Mediterranean_forests,_woodlands,_and_scrub
This is a medium sized article contains all the important details of the
Mediterranean region vegetation.

87.

Solution: b)

On the eastern slopes of the Rockies in Canada and USA, a local wind similar to the
Fohn in Switzerland called the Chinook comes in a south-westerly direction to the
Prairies and has a considerable effect on the local pastures. It actually comes with the
depressions in winter or early spring from the pacific coast ascending the Rockies
and then descending to the Prairies. It is a hot wind and may raise the temperature
by 40F within a matter of 20 minutes. It melts the snow-covered pastures and
animals can be driven out of doors to graze in the open fields. The agricultural year is
thus accelerated. Local farmers welcome the Chinook for frequent Chinooks mean
mild winters.

88.

Solution: d)

http://www.brighthubeducation.com/science-homework-help/109894-examples-ofautotrophs-plants-algae-and-bacteria/

89.

Solution: d)

The robust external-sector outcome in the current year of moderate trade and
current account deficits, abundant financial flows, a build-up of foreign exchange
reserves and broadly stable exchange rate movement points to a return to the path of
strength and resilience that was in evidence before the global financial crisis of 2008.
This follows the improvement last year that was achieved in the face of an initial
phase of severe stress and on the strength of policy responses.
The correction in the international prices of crude petroleum in the second half of the
current fiscal has helped in the decontrol of diesel prices. The overall trade
performance signaled an opportune time for withdrawal of restrictions on gold
imports. The resilience also owed in part to the trade diversification process.
While trade and current account deficits are on even keel, the copious financial
inflows in excess of the financing requirement has helped shore up foreign exchange

http://www.insightsonindia.com

Insights Mock Tests - 2015


TEST 22 SOLUTIONS

reserves (US$ 328.7 billion at end-January 2015). These have helped allay the
vulnerability concerns that led to severe stress last year. These concerns, however,
remain a potent downside risk, should the global environment deteriorate for some
reason. The global economic outlook remains somewhat uncertain but stable and
likely to gain strength if lower global crude petroleum prices drive the demand
recovery process in key emerging market economies.

90.

Solution: c)

http://en.wikipedia.org/wiki/Trophic_level

91. Solution: d)
One of the major items in Indias import basket is the POL group, which accounted
for 36.6 per cent of Indias total imports in 2013-14. POL imports surged with a
growth of 46.2 per cent in 2011-12, mainly on account of significant increase in global
crude oil prices vis--vis 2010-11. The growth in imports of POL moderated to 5.9 per
cent and 0.4 per cent respectively in 2012-13 and 2013-14.
There was moderation in international crude oil prices (Brent) from US$109.8 per
barrel in the first quarter of 2014-15 to US$ 76.0 per barrel in the third quarter which
resulted in the value of POL imports declining by 7.9 per cent in 2014- 15 (AprilJanuary). Capital goods imports are another major group which declined
continuously from 2011-12 onwards. Within capital goods, imports of machinery
registered positive growth in 2014-15 (April-January).

92.

Solution: c)

Statement one is correct as CO2 more readily dissolves in water than O2.
It has been well established that rising CO2 will stimulate plant growth. Indeed,
climate change associated with rising atmospheric CO2 has already altered ecosystem
carbon balance through rising temperature, increased growing season, and increased
atmospheric water content. Studies in native ecosystems have shown that while
grasslands show a relative small stimulation of shoot growth, woody plants respond
vigorously to elevated CO2.
How much additional carbon will be added to terrestrial ecosystems as a result of the
CO2fertilization effect will depend on feedbacks of environmental factors on the
major resources of nutrients, water and light. Additionally the interaction between
plant physiological responses to elevated CO2 and environmental factors in native
species and in ecosystem processes is not well understood.
Rising atmospheric CO2 has the potential to stimulate carbon accumulation in
ecosystems through direct effects on photosynthesis and growth of plants. But

http://www.insightsonindia.com

Insights Mock Tests - 2015


TEST 22 SOLUTIONS

because growth of native species is often limited by the supply of water and
nutrients, particularly nitrogen, it is not clear whether, in the long-term of years to
decades, CO2 stimulation of plant growth would add significant amounts of
anthropogenic carbon as soil carbon to ecosystems. Moreover, the possibility that
additional carbon would be sequestered in long-term, carbon pools in the soil has not
been determined in native ecosystems.
93.

Solution: b)

A keystone species is a species that is connected to a disproportionately large number


of other species in the food-web. Keystone species have lower levels of biomass in the
trophic pyramid relative to the importance of their role. The many connections that a
keystone species holds means that it maintains the organization and structure of
entire communities. The loss of a keystone species results in a range of dramatic
cascading effects that alters trophic dynamics, other food web connections, and can
cause the extinction of other species.[90][91]
Sea otters (Enhydra lutris) are commonly cited as an example of a keystone species
because they limit the density of sea urchins that feed on kelp. If sea otters are
removed from the system, the urchins graze until the kelp beds disappear and this
has a dramatic effect on community structure.[92] Hunting of sea otters, for example,
is thought to have indirectly led to the extinction of the Steller's Sea
Cow (Hydrodamalis gigas).[93] While the keystone species concept has been used
extensively as a conservation tool, it has been criticized for being poorly defined from
an operational stance. It is difficult to experimentally determine what species may
hold a keystone role in each ecosystem. Furthermore, food web theory suggests that
keystone species may not be common, so it is unclear how generally the keystone
species model can be applied.
94.

Solution: c)

The changing contours of trade and the emergence of global production chains have
important implications for developing countries. Increasing use of imported inputs
has generally caused a decline in the domestic value added share of total exports.
The decomposition of value added by capital and different types of labour is an
important aspect of global fragmentation of the production process. It is often argued
that increasing trade and thereby integration with the world market will lead to new
opportunities for developing nations to employ their abundant medium and low
skilled workers. The aforementioned decomposition of domestic value added allows
examination of how the benefits of globalization are being distributed between
capital and different types of labour.
In the Indian context, the share of domestic value added exports in total exports has
witnessed a decline from 86.9 per cent in 1998-99 to 84.1 per cent in 2003-04 and
further to 78.5 per cent in 2007-08. The foreign value added share in exports,

http://www.insightsonindia.com

Insights Mock Tests - 2015


TEST 22 SOLUTIONS

however increased, indicating deepening of the process of international production


fragmentation. The domestic labour component is relatively higher in Indias service
exports than in merchandise exports. Further, the domestic value of exports based on
four components (unskilled, semi-skilled, skilled labour, and non-labour) shows that
the combined share of the skilled labour and non-labour components is significantly
high, which shows a pervasive process of technological change that is biased towards
the use of skilled labour and capital.
95.

Solution: d)

The government took various measures including those aimed at boosting the
performance of the export sectors which supplemented the announcements made in
the Budgets and in the Foreign Trade Policy (FTP) 2009 and its Annual
Supplements. Various schemes were strengthened, viz. Focus Product Schemes
(FPS), Focus Market Scheme (FMS), Market Linked Focus Product Scheme
(MLFPS), and Vishesh Krishi and Gram Udyog Yojana (VKGUY). In addition,
industry and trade bodies are given support for participation in buyer seller meets
(BSM), trade fairs, and exhibitions in various countries under the Market Access
Initiative (MAI) scheme and Market Development Assistance (MDA) scheme.
96.

Solution: a)

An ecotone is a transition area between two biomes.[1] It is where two communities


meet and integrate.[2] It may be narrow or wide, and it may be local (the zone
between a field and forest) or regional (the transition between forest and
grassland ecosystems).[3] An ecotone may appear on the ground as a gradual
blending of the two communities across a broad area, or it may manifest itself as a
sharp boundary line.
The word ecotone was coined from a combination of eco(logy) plus -tone, from
the Greek tonos or tension in other words, a place where ecologies are in tension.
97.

Solution: d)

India has entered into trading agreements with various countries of the world with
the objective of boosting its external trade. Foreign Trade Policy of India has always
focused on substantially increasing the country's share of global merchandise trade.
Accordingly the Government of India has been taking various steps towards boosting
its trade with the rest of the world by adopting policies and procedures which would
help to increase and facilitate both exports and imports with the other countries of
the world. To facilitate and thereby increase external trade activities with the rest of
the world, the Department of Commerce, Government of India has developed this
web portal. You can visit http://indiantradeportal.in/ just to get a broad idea.
98.

Solution: d)

https://www.wto.org/english/tratop_e/tradfa_e/tradfa_e.htm

http://www.insightsonindia.com

Insights Mock Tests - 2015


99.

TEST 22 SOLUTIONS

Solution: b)

Effective exchange rates are summary indicators of movement in the exchange rate
of home currency against a basket of currencies of trade partner countries and are
considered to be an indicator of international competitiveness. The real effective
exchange rate (REER) indices are used as indicator of external competitiveness of the
country over a period of time.
The nominal effective exchange rate (NEER) is the weighted geometric average of the
bilateral nominal exchange rates of the home currency in terms of foreign currencies.
REER is defined as a weighted geometric average of nominal exchange rates of the
home currency in terms of the foreign currencies adjusted for relative price
differential. Although the rupee has depreciated against the US dollar, in terms of
NEER (36 currencies) it appreciated by 2.8 per cent in December 2014 over March
2014. Similarly, REER also appreciated by 5.8 per cent during the same period
100.

Solution: d)

http://www.insightsonindia.com

Insights Mock Tests - 2015


http://www.insightsonindia.com

TEST 22 SOLUTIONS

INSIGHTS ON INDIA MOCK PRELIMINARY EXAM - 2015


INSIGHTS ON INDIA MOCK TEST - 23
GENERAL STUDIES

PAPER-I
Time Allowed: 1.5 Hours

Maximum Marks: 150

INSTRUCTIONS
1. IMMEDITELY AFTER THE COMMENCEMENT OF THE EXAMINATION, YOU SHOULD
CHECK THAT THIS TEST BOOKLET DOES NOT HAVE ANY UNPRINTED OR TORN OR
MISSING PAGES OR ITEMS, ETC. IF SO, GET IT REPLACED BY A COMPLETE TEST BOOKLET.
2. You have to enter your Roll Number on the Test I
Booklet in the Box provided alongside. DO NOT
write anything else on the Test Booklet.
4. This Test Booklet contains 75 items (questions). Each item is printed only in English. Each item comprises
four responses (answers). You will select the response which you want to mark on the Answer Sheet. In
case you feel that there is more than one correct response, mark the response which you consider the best.
In any case, choose ONLY ONE response for each item.
5. You have to mark all your responses ONLY on the separate Answer Sheet provided. See directions in the
Answer Sheet.
6. All items carry equal marks.
7. Before you proceed to mark in the Answer Sheet the response to various items in the Test Booklet, you
have to fill in some particulars in the Answer Sheet as per instructions sent to you with your Admission
Certificate.
8. After you have completed filling in all your responses on the Answer Sheet and the examination has
concluded, you should hand over to the Invigilator only the Answer Sheet. You are permitted to take away
with you the Test Booklet.
9. Sheets for rough work are appended in the Test Booklet at the end.
10. Penalty for wrong answers :
THERE WILL BE PENALTY FOR WRONG ANSWERS MARKED BY A CANDIDATE IN THE
OBJECTIVE TYPE QUESTION PAPERS.
(i) There are four alternatives for the answer to every question. For each question for which a
wrong answer has been given by the candidate, one-third of the marks assigned to that question
will be deducted as penalty.
(ii) If a candidate gives more than one answer, it will be treated as a wrong answer even if one of the
given answers happens to be correct and there will be same penalty as above to that question.
(iii)

If a question is left blank, i.e., no answer is given by the candidate, there will be no penalty for
that question.
http://insightsonindia.com

INSIGHTS ON INDIA MOCK TEST SERIES FOR CIVIL SERVICES PRELIMINARY EXAM 2015

http://insightsonindia.com

Page 1

1. Recently the Union government


cancelled licences of nearly 9,000
Non-Governmental Organisations in
an order issued by the Ministry of
Home affairs. This was done because
a) an off late IB report had
recommended banning all NGOs
that receive foreign funding
b) these NGOs were involved in antidevelopmental activities in various
parts of India
c) these NGOs flouted governmental
norms and accepted projects
radically different that what they
were licensed for
d) these NGOs failed to file their
annual returns and details of
foreign funds received when asked
by the government
2. Consider the following statements
about Swayambhunath Stupa located
in Kathmandu.
1. It is worlds youngest Buddhist
shrine.
2. It is one of the seven UNESCOlisted world heritage sites of South
Asia.
3. It has been built out of one single
large rock.
4. The Indian government funds the
conservation of this shrine.
Choose the correct answer using the codes
below.
a)
b)
c)
d)

1 and 2 only
2, 3 and 4 only
1 and 4 only
None of the above

3. Climate change will affect wildlife


directly and indirectly as it may lead to
change in
1. Weather patterns
2. Food that the wildlife feeds on
http://insightsonindia.com

3. Habitat of the Wildlife


4. Species composition of a particular
region
Choose the correct answer using the codes
below.
a)
b)
c)
d)

1 and 2 only
2, 3 and 4 only
1 and 4 only
All of the above

4. India and Japan have recently signed a


Five-point action agenda to increase
bilateral trade and investment between
the two nations. But, in some areas
cooperation is not there. These areas
are
1. Building Industrial townships in
India
2. Strategic sectors
3. IT Sector
4. Agriculture
Choose the correct answer using the codes
below.
a)
b)
c)
d)

1 and 2 only
3 and 4 only
4 only
All of the above

5. In which of the following forms/ways


FDI can be incentivized in the host
country?
1. Export Processing zones
2. Special Economic zones
3. Preferential tariffs
4. Liberalising security markets
Choose the correct answer using the codes
below.
a)
b)
c)
d)

1, 2 and 3 only
2 and 3 only
1, 3 and 4 only
1 and 2 only
Page 2

6. A Kathakali performance begins with


the kelikottu, calling the audience to
attention followed by the todayam. It is
a) a devotional number performed
where one or two characters invoke
the blessings of the gods
b) the musicians and drummers
holding the stage entertaining the
audience with an exhibition of their
skills in melappada
c) an orchestra which is also used in
other traditional performing arts of
Kerala
d) a pure dance sequence where the
actor is at great liberty to express
himself and display his skills
7. The Government has proposed to bring
certain changes in the Prevention of
Corruption Act, 1988. Consider the
following statements about the Act.
1. It covers only the Central
government agencies/bodies and
PSUs.
2. The offences punishable under this
act can be tried by special Judges
only.
3. Police does not have powers to
investigate offences under this
particular Act.
Choose the correct answer using the codes
below.
a)
b)
c)
d)

1 and 2 only
1 and 3 only
2 and 3 only
2 only

8. The total precipitation of Siberian


climate is determined by factors such
as
1. Altitude
2. Latitude
3. Proximity to poles
http://insightsonindia.com

4. Amount of exposure to influence by


westerlies
5. Temperate monsoons on the
eastern side of continents
Choose the correct answer from the codes
below.
a)
b)
c)
d)

1 and 3 only
2, 4 and 5 only
1, 2 and 3 only
All of the above

9. The soils of coniferous forests are


leached, acidic and poor because
1. The evergreen leaves of these
forests provide little leaves for
humus formation
2. decomposition rate of biomass is
low at cold temperatures in
coniferous forests
Which of the above is correct?
a)
b)
c)
d)

1 only
2 only
Both 1 and 2
None

10. The Cool temperate Eastern margin


(laurential) type climate is an
intermediate type of climate between
British and Siberian type climate. This
type of climate is not found in the
Southern hemisphere because
a) only a small section of the southern
hemisphere extends beyond 40
degrees latitude
b) the influence of westerlies is less
stronger because of the large sea
portion in southern hemisphere
c) a large number of warm currents
are present in the southern
hemisphere ocean in comparison to
the cold currents found at same
latitude in Northern hemispheres
d) None of the above
Page 3

11. The mission of the Organisation for


Economic Co-operation and
Development (OECD) is to
a) promote policies that will improve
the economic and social well-being
of people around the world.
b) promote policies that will improve
the economic and social well-being
of its member nations.
c) promote effective trade policies that
help lower down trade barriers for
its member nations across the
world
d) promote effective trade policies that
help lower down trade barriers for
international trade to flourish
12. Recently some of the eminent Indian
personalities have received highest
civilian awards from some foreign
government. These personalities are
1. CNR Rao
2. Yashwant Sinha
3. Kumar Mangalam Birla
4. LK Advani
Choose the correct answer using the codes
below.
a)
b)
c)
d)

1 and 2 only
1 and 3 only
2, 3 and 4 only
All of the above

13. Krishi Vigyaan Kendras (KVKs) work


to support agricultural extension
services to farmers in India. Which of
the following come under their
mandate?
1. Education & training of farmers,
entrepreneurs and voluntary
organizations
2. On farm demonstration of new
technologies

http://insightsonindia.com

3. Conducting research on Genetically


Modified Organisms (GMOs)
Choose the correct answer using the codes
below.
a)
b)
c)
d)

1 and 2 only
1 and 3 only
2 and 3 only
All of the above

14. Consider the following about the


Protection of Women from Domestic
Violence Act, 2005.
1. Minors are not entitled to relief
under the law.
2. Law can be applicable even to
unmarried couples.
3. The complaint can be filed against
both men and women.
Choose the correct answer using the codes
below.
a)
b)
c)
d)

1 and 2 only
1 and 3 only
2 and 3 only
All of the above

15. Changes in crude oil prices have a


bearing on Indian Economys
1. Current Account Deficit
2. Inflation levels
3. Interest rates
4. Exchange rate of Indian rupee
Choose the correct answer using the codes
below.
a)
b)
c)
d)

1, 3 and 4 only
1, 2 and 4 only
1 and 4 only
2 and 3 only

Page 4

16. Bharat Bill Payment System (BBPS) is


an integrated bill payment system
in India offering interoperable and
accessible bill payment service to
customers. Consider the following
about it.
1. Its nodal body will be a private
agency nominated by the
government.
2. It will also protect consumers from
frauds and mischief on e-commerce
transactions.
3. No agents will be involved in BBPS.
Choose the correct answer using the codes
below.
a)
b)
c)
d)

1 and 2 only
1 and 3 only
2 and 3 only
2 only

17. RBI recently allowed Indian


companies to issue rupee-denominated
bonds abroad. This is a move towards
a) Full capital account convertibility
b) Full current account convertibility
c) Liberalization of Industrial policy
in India
d) Great autonomy of foreign
exchange markets in India
18. Target Olympic Podium Scheme (TOP
Scheme) launched recently under the
National Sports Development Fund
(NSDF) will identify athletes from
which of the following disciplines?
1. Sailing
2. Wrestling
3. Archery
4. Boxing
5. Athletics
Choose the correct answer using the codes
below.

b) 2 and 4 only
c) 2, 3, 4 and 5 only
d) All of the above
19. Wind is caused due to difference in
temperature and pressure between two
regions. Factors affecting wind
movement can be
1. Latitude
2. Proximity of water bodies
3. Land undulations
4. Rotation of earth
Choose the correct answer using the codes
below.
a)
b)
c)
d)

1, 2, and 4 only
2 and 4 only
1 and 3 only
All of the above

20. Recently the Central Marine Fish


Research Institute (CMFRI), Kochi has
begun the genetic profiling of The
Indian Mackerel, an important food
fish commonly consumed in South
Asia. Consider the following about it.
1. The fish is commonly found in
warm shallow waters.
2. Regions of maximum population
are along the coasts of the Indian
and West Pacific oceans.
3. No inter-species genetic distinction
is found between these fishes
collected within the Northern
hemisphere.
Choose the correct answer using the codes
below.
a)
b)
c)
d)

1 and 2 only
2 and 3 only
1 and 3 only
All of the above

a) 1, 3, and 5 only
http://insightsonindia.com

Page 5

21. When you crack your knuckle it


produces popping sound. This happens
because
a) A cavity/bubble is formed in the
suddenly expanded joint volume
b) the fluid between the joint gets
compressed at high pressure and
produces sound
c) the bones of knuckles counter act
against each other
d) the tissues of knuckles counter act
against each other
22. Eastern Patagonia is a large arid
region. Its aridity can be attributed to
1. Climatic barrier of Southern Andes
2. Frequent and strong influence of
westerlies
3. nearby cold currents
Choose the correct answer from the codes
below.
a)
b)
c)
d)

1 and 2 only
2 and 3 only
1 only
All of the above

23. The meeting of the warm Gulf stream


and cold Labrador current on coastal
waters of the nearby island
(Newfoundland) produces
1. Mist and Fog
2. atmospheric storms
3. Apt fishing grounds
4. frontal systems
Choose the correct answer from the codes
below.
a)
b)
c)
d)

1 and 4 only
2 and 3 only
1 and 2 only
1, 2 and 3 only

http://insightsonindia.com

24. The Kathak dance combines the


elements of and is inspired from?
1. Vaishnavite cult of 15th century in
India
2. The emergence of Raslila in the
Braj region
3. Durbar style impetus from Hindu
courts
Choose the correct answer using the codes
below.
a)
b)
c)
d)

1 and 2 only
2 and 3 only
1 and 3 only
All of the above

25. Consider the following statements


about Kathak dance form.
1. It the only classical dance of India
having links with Muslim culture.
2. It is the only form of classical dance
wedded to Hindustani or the North
Indian music.
Which of the above is/are true?
a)
b)
c)
d)
26.

1 only
2 only
Both 1 and 2
None

Arrange these countries/entities in


descending order in their contribution
to cumulative global CO 2.
1. USA
2. China
3. India
4. EU
5. Russia

Choose the correct order from the codes given


below.
a)
b)
c)
d)

14253
41523
45123
12345
Page 6

27. In winter, a steep pressure gradient is


set up between the cold interiors of
Mongolia and Siberia, and the warmer
pacific coastlands. As a consequence of
this
1. The continental polar stream flows
outwards as the North-west
monsoon.
2. There is heavy rain on the
windward slopes.
3. Temperature inversion occurs near
the cold interiors.
Choose the correct answer from the codes
below.
a)
b)
c)
d)

1 and 2 only
2 and 3 only
1 and 3 only
1 only

28. The Southern continents also have


violent local storms, which though not
as severe as typhoons or cyclones, are
nevertheless significant. In this
context, the Southerly Buster is
1. a cold wind blowing along the coast
of Wales
2. accompanied by heavy thunder and
rain
3. leads to the formation of local
typhoons
Choose the correct answer from the codes
below.
a)
b)
c)
d)

1 and 2 only
2 and 3 only
1 and 3 only
1 only

29. Lai Haraoba is one of the main


festivals still performed in Manipur.
Consider the following about it.
1. It was conceived after the postvaishnavite period.
http://insightsonindia.com

2. It is also the earliest form of dance


which forms the basis of all stylised
dances in Manipur.
3. The principal performers in this reenact the theme of the creation of
the world.
Choose the correct answer using the codes
below.
a)
b)
c)
d)

1 and 2 only
2 and 3 only
1 and 3 only
All of the above

30. The cool temperate western margins


1. remain under the permanent
influence of westerlies
2. do not suffer from any cyclonic
activity
3. are not visited by heat waves
Choose the correct answer from the codes
below.
a)
b)
c)
d)

1 and 2 only
2 and 3 only
1 only
All of the above

31. Barley, a member of the grass family, is


a major cereal grain. It can be used as
1. Algicide
2. Fish feed
3. Preservative
4. a cure for cardiovascular diseases
Choose the correct answer from the codes
below.
a)
b)
c)
d)

1 and 2 only
2 and 4 only
1, 2 and 3 only
1 and 4 only

Page 7

32. Which of the following are related to


Hindustani Classical music?
1. Sruti
2. Saptak
3. Antara Gandhara
4. Moorcchanas
5. Gramas
Choose the correct answer using the codes
below.
a)
b)
c)
d)

1, 2 and 4 only
3 and 5 only
1 and 2 only
All of the above

33. A characteristic contribution of India


to musical rhythm is the tala. Tala is a
a) scale of melodies
b) cyclic arrangement of time units
c) subsidiary notes of a basic scale of
music
d) unit difference between the various
consecutive pitches within a scale
34. Consider the following about Thumri
form of singing.
1. It is not lyrical in its structure and
presentation.
2. It is essentially a love song.
3. Musical grammar is not adhered to
in Thumri.
Choose the correct answer using the codes
below.
a)
b)
c)
d)

1 and 2 only
2 and 3 only
1 and 3 only
2 only

35. Tappa is a popular type heard in


concerts in India. It consists of song
uttered in
1. fast note patterns
2. very simple compositions

http://insightsonindia.com

3. ragas where strong ragas like


bhairavi dont find a place
Choose the correct answer using the codes
below.
a)
b)
c)
d)

1 and 2 only
2 and 3 only
1 and 3 only
1 only

36. Which of the following musical forms


come under the Carnatic classical
music?
1. Gitam
2. Suladi
3. Svarajati
4. Kirtanam
5. Kriti
6. Pada
Choose the correct answer using the codes
below.
a)
b)
c)
d)

1 and 2 only
All except 2 and 3
All of the above
4 only

37. Consider the following about United


Nations Conference on Trade and
Development (UNCTAD).
1. It is governed by its member
states.
2. Member states are elected by UN
General Assembly.
3. UNCTAD organizes the World
Investment Forum every year to
bring greater investment in
developing economies.
4. It conducts cutting-edge research
and analysis in the field of
investment for sustainable
development.
Choose the correct answer using the codes
below.
Page 8

a)
b)
c)
d)

1 and 3 only
2 and 3 only
1, 3 and 4 only
1 and 4 only

38. In the Natya Shastra, compiled by


Bharat Muni dated 200 B.C.-200 A.D.,
musical instruments have been divided
into four main categories on the basis
of how sound is produced. These are
1. Ghana Vadya
2. Tata Vadya
3. Sushira Vadya
4. Reki Vadya
5. Adoor Vadya
Choose the correct answer using the codes
below.
a)
b)
c)
d)

2, 4 and 5 only
3 and 4 only
1, 2 and 3 only
All of the above

39. Consider the following about Bhavai a traditional theatre form.


1. It belongs to the region of Madhya
Pradesh.
2. It is an entirely romantic play
centred around love and affection.
3. It is associated with local deities
and their popular legends.
Choose the correct answer using the codes
below.
a)
b)
c)
d)

1 and 2 only
2 only
3 only
None of the above

40. Krishnattam, folk theatre of Kerala,


came into existence in the middle of
17th century A.D. under the patronage
of King Manavada of Calicut. Consider
the following about it.
http://insightsonindia.com

1. It is a cycle of eight plays


performed for eight consecutive
days.
2. The episodes are based on the
theme of Lord Krishna.
3. Themes depict victory of good over
evil.
4. Dashavatar performers wear masks
in this theatre.
Choose the correct answer using the codes
below.
a)
b)
c)
d)

1, 2 and 3 only
2 and 4 only
All of the above
None of the above

41. Consider the following features of folk


theatres.
1. performed by various castes to
appease and worship spirits
2. colourful costume and beautiful
headgears
3. Also called as Gods dance
Which local theatre has these features?
a)
b)
c)
d)

Koodiyaattam
Theyyam
Mudiyettu
Yakshagaana

42. Consider the following about


Therukoothu, the most popular form
of folk drama of Tamil Nadu.
1. It is performed to achieve rain
harvest.
2. It is based on the lives of Ravana.
3. It cannot be performed at temples.
4. It also draws inspiration from
Puranas.
Choose the correct answer using the codes
below.
Page 9

a)
b)
c)
d)

1, 2 and 3 only
2 and 4 only
1 only
None of the above

43. Consider the following about the art of


puppetry in India.
1. The earliest reference to the art of
puppetry is found in
Natyashashtra.
2. They are mostly based on epics and
legends.
3. Rod puppetry is not practiced in
India.
Choose the correct answer using the codes
below.
a)
b)
c)
d)

1 and 2 only
2 and 3 only
1 and 3 only
2 only

44. In Performance in Services, an


International Comparison (year on
year) tells us that
1. Indias service sector grew fastest
for the period 2001-13.
2. Service sector in U.S. has suffered
from instances of negative growth
in last one decade.
3. India is the only nation that has
able to maintain a positive growth
rate in services sector since the
economic recession of 2008.
Choose the correct answer using the codes
below.
a)
b)
c)
d)

1 and 2 only
2 and 3 only
1 and 3 only
All of the above

http://insightsonindia.com

45. As accepted internationally, which of


the following factors differentiates
Foreign Direct Investment (FDI) from
portfolio investment?
a) Control of ownership
b) Origin of investment
c) Entity of Investment
d) Outgoing and Incoming
46. Manipur dance has a large repertoire
of popular forms. Some of them are
1. Ras
2. Sankirtana
3. Thang-ta
4. Parengs
Choose the correct answer using the codes
below.
a)
b)
c)
d)

1, 3 and 4 only
2 and 4 only
1, 2 and 3 only
All of the above

47. Consider the following about the


Sattriya dance form.
1. It was conceived to propagate
Vaishnava faith.
2. It has not been influenced by
Assamese local folk dances.
3. The dance form is secular in
character.
Choose the correct answer using the codes
below.
a)
b)
c)
d)

1 and 2 only
2 and 3 only
1 and 3 only
1 only

Page 10

48. The Services sector had the highest


share in Gross capital formation (GCF)
in the Economy for the last two
financial years. Which of the following
sub-sectors contributed for GCF the
most?
1. Real estate
2. Ownership of dwelling
3. Professional services
Choose the correct answer using the codes
below.
a)
b)
c)
d)

1 and 2 only
2 and 3 only
1 and 3 only
All of the above

49. Decision-making in OECD is vested in


the OECD council which consists of
1. One representative from each
member country
2. Elected representatives from five
member countries by rotation
3. A representative of European
Commission
4. Secretary-general of UN
Choose the correct answer from the codes
below.
a)
b)
c)
d)

1 and 2 only
2 and 3 only
1 and 3 only
2 and 4 only

50. At present, India is engaged in


bilateral Free Trade Agreement (FTA)
negotiations with which of the
following countries?
1. Canada
2. Thailand
3. Zimbabwe
4. Israel
5. New Zealand
6. Australia

http://insightsonindia.com

Choose the correct answer from the codes


below.
a)
b)
c)
d)

All except 2 and 3


All except 1 and 6
All except 3
All except 4 and 5

51. Which of the following is NOT covered


under a Free Trade Agreement (FTA)?
1. Free movement of people across
the border of two countries
2. Reducing trade barrier for services
3. Coordination in fiscal and
monetary policy
Choose the correct answer from the codes
below.
a)
b)
c)
d)

1 and 2 only
2 and 3 only
1 and 3 only
All of the above

52. Consider the following statements


about the World Travel & Tourism
Council (WTTC).
1. It is an inter-governmental body.
2. It includes representation from the
private sector too.
3. WTTC campaigns for governments
to implement policies that ensure
the business environment is
conducive to the growth of Travel &
Tourism.
Choose the correct answer from the codes
below.
a)
b)
c)
d)

1 and 2 only
2 and 3 only
1 and 3 only
All of the above

Page 11

53. The Chhau dances of Orissa is a form


of Indian
a) tribal martial dance
b) narration of legends of local heroes,
kings and deities
c) folk theatre
d) dance in leaping and crouching in
twirling patterns
54. The folk music and dances of
agricultural communities celebrate the
rhythms of daily life. Together these
dances have formed a vast reservoir
from which the classical dances have
drawn sustenance. The classical dances
are
1. Sattriya
2. Kuchipudi
3. Odissi
4. Bhavai
Choose the correct answer using the codes
below.
a)
b)
c)
d)

1 and 2 only
2 and 3 only
3 and 4 only
1, 2 and 3 only

55. Consider the following about


Natyashashtra, an ancient Indian
treatise on performing arts.
1. It is traditionally attributed to
the Sage Bharata.
2. It deals with stage design, music,
dance, makeup, and primarily with
aspects of stagecraft.
3. The Natya Shastra is based upon
the Gandharva Veda.
Choose the correct answer using the codes
below.
a)
b)
c)
d)

1 and 2 only
2 and 3 only
1 and 3 only
All of the above

http://insightsonindia.com

56. In an international comparison of


tourist performance, the nations which
have the highest international tourist
arrivals and highest international
tourist receipts (income) respectively
are
a) USA and France
b) India and USA
c) India and Spain
d) France and USA
57. Consider the following about the
Intergovernmental Panel on Climate
Change (IPCC).
1. It is an agency under the UN.
2. Membership of IPCC is open to all
members of United Nations
Environment Program (UNEP).
3. The IPCC does not carry out its own
original research.
4. The ultimate objective of the
UNFCCC is to stabilize greenhouse
gas concentrations in the
atmosphere at a sustainable level.
Choose the correct answer from the codes
below.
a)
b)
c)
d)

1 and 2 only
2 and 3 only
1 and 4 only
All of the above

58. The ecological overshoot problem


essentially means
a) the ecological footprint is larger
than the biocapacity of the
population
b) unsustainable ecology due to
anthropogenic emissions
c) increasing population of lower
trophic levels in an ecosystem
overshooting the higher trophic
levels
d) None of the above
Page 12

59. Indian Space Research Organisation


(ISRO) recently successfully tested an
indigenous cryogenic engine. Consider
the following about a cryogenic engine.
1. It is more efficient as it provides
more thrust for every kilogram of
propellant
burnt.
2. Cryogenic fuels are extremely clean
as they give out only water while
burning.
3. It can be used in interplanetary
probes and manned space missions
by India.
Choose the correct answer using the codes
below.
a)
b)
c)
d)

1 and 2 only
1 and 3 only
2 and 3 only
All of the above

60. Consider the following about some of


the regional songs and their roles in
communities.
1. Bhuta song in Kerala is used to
ward away evil ghosts and spirits.
2. Bhakha form of folk music in
Jammu is sung when harvesting is
done.
3. Sana Lamok in Manipur is sung to
evoke the spirit of the presiding
deity.
Choose the correct answer using the codes
below.
a)
b)
c)
d)

1 and 2 only
2 and 3 only
1 and 3 only
All of the above

61. The Gulf type of climate is the best for


growing cotton because it has
1. more than six months frost free
2. frequent light rain showers
3. long bright sunshine periods
Choose the correct answer from the codes
below.
a)
b)
c)
d)

1 and 2 only
2 and 3 only
1 and 3 only
All of the above

62. Bharatnatyam dance is known to


be ekaharya because
a) one dancer takes on many roles in a
single performance
b) It is performed music and dance as
offerings to the deities, in the
temple courtyards
c) It is an abstract piece combining
pure dance with the recitation of
sound syllables.
d) the raga, singer and dancer flow in
one synchronic rhythm
63. The logo for International Day of Yoga
has recently been approved by the
Government of India. Consider the
following about the elements of the
logo.
1. Folding of both hands in the logo
reflects the union of individual
consciousness with universal
consciousness.
2. The brown leaves in the logo
symbolise the earth element.
3. The sun in the logo symbolise the
source of energy and inspiration.
4. The logo has also been adopted by
the United Nations General
Assembly (UNGA).
Choose the correct answer using the codes
below.

http://insightsonindia.com

Page 13

a)
b)
c)
d)

1, 2 and 3 only
1 and 3 only
2 and 4 only
All of the above

64. The National Green Tribunal (NGT)


has recently ordered a complete ban on
burning of any kind of garbage, leaves,
plastic waste and rubber in the open in
Delhi-National Capital Region (NCR).
What harmful gaseous emissions can
come out of such burning?
1. Dioxins
2. Furan
3. Nitrogen oxides
4. Methane
5. Hydrochloric acid
Choose the correct answer using the codes
below.
a)
b)
c)
d)

All except 3 and 5


All except 1 and 2
All except 5
All except 4

65. FDI equity inflows in Services sector


cumulatively since 2000 has been
highest in
a) Construction development
b) Hotel & Tourism
c) Telecommunications
d) Computer Software and Hardware
66. Which two sub-sectors had the highest
share in Indias major export services
in the last two financial years?
1. Computer services
2. Professional and consulting
services
3. R&D services
4. Business services
Choose the correct answer from the codes
below.
a) 1 and 2
http://insightsonindia.com

b) 2 and 3
c) 1 and 4
d) 2 and 4
67. Delhis Indira Gandhi International
Airport (IGIA) has been adjudged the
worlds best airport for the year 2014.
It is run by the Delhi International
Airport Limited (DIAL) which is
a) a joint venture between Delhi
Government and Central
government - Airport Authority of
India
b) run solely by the Central
government Directorate General
of Civil Aviation
c) a private agency licensed by the
Central government
d) a joint venture between the Airport
Authority of India and a private
sector firm
68. Consider the following statements
about the Nuclear energy status of
India.
1. India is among the leading
producers of Nuclear energy in the
World.
2. No nuclear power project in India
has been built with indigenous
technology.
3. While only Central government can
sanction nuclear projects, state
governments can also initiate
standalone nuclear projects.
4. There is no Thorium based
operational nuclear power plant in
India.
Choose the correct answer using the codes
below.
a)
b)
c)
d)

1 and 2 only
3 and 4 only
4 only
None of the above
Page 14

69. The outstanding feature of Carnatic


music is its X system, in the concept
of which, the ideal or 'absolute music'
is reached, and the highly developed
and intricate Y system, which has
made it extremely scientific and
systematic and unique in all respects.
Here X and Y are
a) Sruti and Tala
b) Raga and Tala
c) Sruti and Raga
d) Prabandhas and Raga
70. Consider the following matches of
regional music to the region they
belong to.
1. Rasiya Geet Rajasthan
2. Pankhida Punjab
3. Lotia Rajasthan
4. Mando Goa
Choose the correct matches from the above
using the codes below.
a)
b)
c)
d)

1 and 2 only
3 and 4 only
1 and 4 only
2 and 3 only

71. In the regional song Laman sung in


Himachal Pradesh
a) Local boys and girls sing the song
in replies to each other from
different mountain peaks.
b) women sing folk songs during early
rainy season
c) family members sing every time a
new one is born in the family
d) local deity is worshipped and
praised for better harvests the next
season

72. Powada is the traditional folk art from


Maharashtra. Consider the following
about it.
1. In this narratives are always odes in
praise of an individual hero or an
incident or place.
2. the tempo is fast and controlled by
the main singer
3. It is sung on the third day after the
new moon or amavasya of
shraavana month.
Choose the correct answer using the codes
below.
a)
b)
c)
d)

1 and 2 only
2 and 3 only
1 and 3 only
All of the above

73. India has been following actionoriented policies to bring rapid


development to its people while
purposefully addressing climate
change. Which of these stands have
been taken by India at global forums?
1. Historical responsibilities of
developed countries and equity in
access to global atmospheric
resources should continue to be the
basis of defining mitigation
commitments.
2. The responsibility of providing
climate related financial assistance
to the developing countries lies
with the developed countries.
3. The developed countries should
provide free climate related
technology transfer to developing
countries without the intellectual
property rights price-tag.
Choose the correct answer from the codes
below.

http://insightsonindia.com

Page 15

a)
b)
c)
d)

1 and 2 only
2 and 3 only
1 and 3 only
All of the above

74. Consider the following about the Clean


Development Mechanism (CDM).
1. It has been defined under the Kyoto
protocol.
2. Developed countries can get
emission reduction credits by
supporting environment friendly
projects in developing countries.
3. One company can trade emission
certificates with another company
in the same developing country
under CDM.
Choose the correct answer from the codes
below.

http://insightsonindia.com

a)
b)
c)
d)

1 and 2 only
2 and 3 only
1 and 3 only
All of the above

75. The Warm temperate Eastern Margin


(China type) climate is
1. strongly influenced by maritime
influence
2. a modified form of monsoonal
climate
3. typified by a cool dry winter
4. scarce in rainfall
Choose the correct answer from the codes
below.
a)
b)
c)
d)

1, 2 and 3 only
2 and 3 only
1, 3 and 4 only
All of the above

Page 16

INSIGHTS MOCK TEST 23 SOLUTIONS



http://www.insightsonindia.com

1. Solution: d)
The Union government has cancelled licences of nearly 9,000 Non-Governmental
Organisations for violation of Foreign Contribution Regulation Act (FCRA).
In this regards, Union Ministry of Home Affairs (MHA) had issued an order. The
order has mentioned that notices were issued to 10,343 NGOs for not filing annual
returns for the three years i.e. for 2009-10, 2010-11 and 2011-12.
These notices were served to the NGOs in October 2014 and had asked NGOs to file
their annual returns for mentioned years. They were also asked to specify amount of
foreign funds received, sources of funds and the purpose for which they were
received and manner in which they were utilised.
However, to this notices only 229 NGOs replied out of the 10,344 NGOs leading to
cancellation of their registration.

2. Solution: d)
Nepals devastating 7.9-magnitude earthquake on April 25 has partially damaged
Swayambhunath Stupa located in Kathmandu which is believed to be the worlds
oldest Buddhist shrine.
During this earthquake several other monuments situated in the vicinity of the 2,000
year-old Lichchhavi-era Buddhist shrine have been damaged but the main stupa has
remained intact during the earthquake.
Swayambhunath Stupa is one of the main Buddhist shrine and most visited tourist
sites in Nepal. The stupa is believed to have originated at a time when Kathmandu
was inhabited by a deity called Manjushri. It was drained out a big lake by cutting the
Chobar hill situated in southern Kathmandu thousands of years ago.
Three other Buddhist monasteries were also damaged including Jyoti Kirti Bihar,
Karmapa Bihar, Maha Bihar, Pratapur, Shantipur, Anantapur and Manjushree
temple.
The earthquake also has threatened the existence of one of the 7 UNESCO-listed
world heritage sites of Kathmandu.
Famous Hindu shrine Pashupati and House of Living Goddess Kumari in
Kathmandu has remained unharmed in the earthquake.
However, 300 year-old Kashtha Mandap temple situated in the heart of Kathmandu
city and 200 year-old Dharahara tower are among the important heritage sites which
were damaged by the earthquake.

http://www.insightsonindia.com

[Type text]

INSIGHTS MOCK TEST 23 SOLUTIONS



http://www.insightsonindia.com

3. Solution: d)
Recently published study in journal Science has found out that 1 out of 6 species on
Earth could possibly extinct by the end of the century as a result of increasing global
warming if the emission of carbon dioxide (CO2) stays the same.
The study published was titled Accelerating extinction risk from climate change and
is authored by Mark C. Urban. The study highlighted the urgency to take strong
action in order to address climate change.
Key facts from the report

16 percent of species in the world would face the risk of extinction because of
climatic factors.

The endemic plants and animals of Australia, South America and New
Zealand are at risk as for they would not be able to go to other place when
their only homeland becomes uninhabitable.

While, the species in North American and European regions have the lowest
extinction risk due to climate change.

The extinction risks from climate change will also accelerate for every degree
rise in global temperature if everyone didnt act now to limit future climate
change.

Climate change will affect wildlife directly and indirectly as it may lead to
change in weather patterns that may affect the food they rely on or the habitat
they live in.

4. Solution: c)
The action agenda was signed between Japans Minister for Economy, Trade and
Industry Yoichi Miyazawa and Minister of State (independent charge) for Commerce
and Industry Nirmala Sitharaman.
The five-point agenda includes

Development of selected townships in India as Japanese industrial townships.

Promotion of investment and infrastructure development.

Further development and cooperation in IT sector.

Enhancing cooperation in strategic sectors.

http://www.insightsonindia.com

[Type text]

INSIGHTS MOCK TEST 23 SOLUTIONS



http://www.insightsonindia.com

Asia-Pacific economic integration.

This five-point action agenda is a follow-up of Prime Minister Narendra Modis visit
to Japan in 2014 and will play important role in improving the trade relationship
between India and Japan.
It also comes in line with Union Governments flagship Make in India programme as
it will further increase investments from Japan into the Indias manufacturing sector.
It should be noted that Japan is the fourth largest foreign direct investment (FDI)
contributor to India. Its major FDI interests is in automobiles, pharmaceuticals and
services sectors which accounts for 7.46 per cent of total FDI equity inflows into
India.

5. Solution: a)
Foreign direct investment incentives may take the following forms:

low corporate tax and individual income tax rates

tax holidays

other types of tax concessions

preferential tariffs

special economic zones (investors have various sops to manufacture


domestically)

EPZ Export Processing Zones

Bonded warehouses

Maquiladoras

investment financial subsidies

free land or land subsidies

relocation & expatriation

infrastructure subsidies

R&D support

derogation from regulations (usually for very large projects)

All these encourage FDI.


http://www.insightsonindia.com

[Type text]

INSIGHTS MOCK TEST 23 SOLUTIONS



http://www.insightsonindia.com

6. Solution: a)
In no other dance style is the entire body used so completely as in Kathakali. The
technical details cover every part of the body from facial muscles to fingers, eyes,
hands and wrists.
Kalasams are pure dance sequences where the actor is at great liberty to express
himself and display his skills. The leaps, quick turns, jumps and the rhythmic coordination make kalasams, a joy to watch.
A Kathakali performance begins with the kelikottu, calling the audience to attention
followed by the todayam. It is a devotional number performed where one or two
characters invoke the blessings of the gods. Kelikottu is the formal announcement of
the performance done in the evening when drums and cymbals are played for a while
in the courtyard. A pure nritta piece known as the purappadu comes as a sequel to
this. Then the musicians and drummers hold the stage entertaining the audience
with an exhibition of their skills in melappada. Tiranokku is the debut on the stage of
all characters other than the pacha or minukku. Thereafter, the play or the particular
scene of the chosen play begins.

7. Solution: d)
Read about the Act here:
http://en.wikipedia.org/wiki/Prevention_of_Corruption_Act,_1988#Chapter_I:_Pr
eliminary
The recent Amendment The proposed amendments in the bill to amend parent Act
mainly aim at laying down more stringent measures to tackle corruption.
It also seeks to fill in the perceived gaps in the domestic anti-corruption law as per
the United Nations Convention Against Corruption (UNCAC).
Key Amendments

Proposes more stringent punishment for the offences of bribery, both for the
bribe giver and the bribe taker.

Enhances punishment from the minimum 6 months to 3 years and from the
maximum five years to seven years.

Extends the protection of prior sanction for prosecution to public servants


who cease to hold office due to retirement and resignation.

http://www.insightsonindia.com

[Type text]

INSIGHTS MOCK TEST 23 SOLUTIONS



http://www.insightsonindia.com

Non-monetary gratification is also be covered within the definition of the


word gratification in the PCA 1988.

8. Solution: d)
Read the intro of this article, then the mechanics.
http://en.wikipedia.org/wiki/Climate_of_Russia#Dynamics
It also depends on the penetration of cyclones and its continentality. SE monsoon
from the pacific ocean too affects the total precipitation.

9. Solution: c)
The soil of the coniferous forests are poor. They are excessively leached and very
acidic. The evergreen leaves provide little leaf-fall for humus formation, and the rate
of decomposition of leathery needles in a region of low temperature is slow. All these
factors are deterrents to the groifi of much under- growth. Absence of direct sunlight
and the short duration of summer are other contributary factors to a sparse
undergrowth, but where trees are widely spaced near the tree-line, heath and tundra
plants cover the intervening ground.

10.

Solution: a)

The reason is very simple. A current or westerly can only impact the weather systems
when there is a large landscape available around them. Since this is not the case here,
weather systems are not similar as found in the North.

11. Solution: a)
http://www.oecd.org/about/
The Organisation for Economic Co-operation and Development (OECD) is
an international economic organisation of 34 countries founded in 1961 to stimulate
economic progress and world trade. It is a forum of countries describing themselves
as committed to democracy and the market economy, providing a platform to

http://www.insightsonindia.com

[Type text]

INSIGHTS MOCK TEST 23 SOLUTIONS



http://www.insightsonindia.com

compare policy experiences, seeking answers to common problems, identify good


practices and coordinate domestic and international policies of its members.

12. Solution: a)
French Government has honoured former Union Finance Minister and veteran BJP
leader Yashwant Sinha with Officier de la Lgion dHonneur (Officer of the Legion of
Honour).
It is the highest French civilian distinction bestowed upon him in recognition of his
international action taken during his stint as Union Minister of Finance, Minister
External Foreign Affairs and for his invaluable contribution to international issues.
The honour was given to Sinha by French Ambassador to India Franois Richier in
Delhi.
Sinha as External Affairs (from 2002-2004) had chaired Indo-French Parliamentary
Friendship Group since its very inception (from 2009-2014).
He had contributed in deepening of the Indo-French strategic partnership which was
launched during the visit of them Prime Minister Atal Bihari Vajpayee in Paris in
September 1998.
For CNR rao http://articles.economictimes.indiatimes.com/2015-0501/news/61723780_1_cnr-rao-scientific-academies-linus-pauling-researchprofessor

13. Solution: a)
http://www.uasbangalore.edu.in/index.php/extension/krishi-vigyan-kendra
The Indian Council of Agricultural Research (ICAR) has created a network of 642
Krishi Vigyan Kendras (KVKs) in the country to facilitate farmers access to
agricultural technology generated by National Agricultural Research System. As part
of this facilitation process, the KVKs access, refine and demonstrate various
technologies to find out the suitability and to demonstrate the production potential
in farmers field.
From PIB Features

http://www.insightsonindia.com

[Type text]

INSIGHTS MOCK TEST 23 SOLUTIONS



http://www.insightsonindia.com

14. Solution: c)
The law simply provides protection from violence to all women whether they are
sisters, mothers, wives or partners living together in a shared household. To the
extent of providing protection, the law does not differentiate between married and
unmarried women.
A woman can file a complaint against any adult male perpetrator [Section 2 (q)] of an
act of violence. In cases where the woman is married, or lives in a relationship that is
in the nature of marriage, she can also file a complaint against the male or female
relatives of the husband/ male partner who have perpetrated the violence.
From PIB Features + The issue of Marital rape has been in news for some time; the
issue is connected with Domestic Violence Act.

15. Solution: b)
Typically in the literature, current account deficit (CAD) is viewed as foreign savings
that promote growth through higher investment given the level of domestic savings
in EMDEs; but in the context of Indias large oil import dependence and the sharp
rise in global crude oil prices, the widening of the CAD in 2011- 12 and 2012-13 may
be an atypical outcome. Changes in crude oil prices have direct bearing on Indias
CAD. Historically, crude oil imports accounted for a substantial portion of the
countrys total imports. Petroleum, oil, and lubricants (POL) imports accounted for
more than one-third of Indias total imports in recent years. In 2013-14, POL imports
accounted for 36.6 per cent of total imports.
Higher oil prices also fuel inflation in the economy as oil is vital to our energy needs.
Since CAD is affected, the exchange rate of rupee is also affected by changes in crude
oil price.

16. Solution: d)

http://www.insightsonindia.com

[Type text]

INSIGHTS MOCK TEST 23 SOLUTIONS



http://www.insightsonindia.com

http://en.wikipedia.org/wiki/Bharat_Bill_Payment_System
https://rbi.org.in/scripts/bs_viewcontent.aspx?Id=2866

17. Solution: a)
http://www.business-standard.com/article/finance/rbi-allows-companies-to-issuerupee-bonds-abroad-115040800016_1.html
http://www.livemint.com/Money/1ZEYenE7i9Os4dlLQ9MK2N/RBI-moves-toliberalize-bond-markets.html

18.Solution: d)
Ministry of Youth Affairs and Sports has established the National Sports
Development Fund (NSDF) with the objective of promotion of sports in general and
specific sports disciplines and individual sports persons in particular for achieving
excellence at the National and International level. Ministry has initiated the Target
Olympic Podium Scheme (TOP Scheme) under the NSDF. The scheme envisages
identification of athletes who are potential medal winners in Olympics and providing
support to them for preparation for Olympics.
As on 20th March, 2015, 45 athletes have been identified from 6 disciplines namely
athletics, badminton, boxing, shooting, wrestling and sailing have been identified as
potential medal winners for Olympics 2016. Around 30 more athletes are to be
identified from archery and athletics.

19. Solution: d)
These two sources explain the mechanism very well.
http://www.preservearticles.com/2011111217151/what-are-the-factors-that-affectwind-motion.html

http://www.insightsonindia.com

[Type text]

INSIGHTS MOCK TEST 23 SOLUTIONS



http://www.insightsonindia.com

http://study.com/academy/lesson/factors-that-affect-wind-pressure-gradientforces-coriolis-effect-friction.html

20.

Solution: a)

The Indian Mackerel Rastrelliger kanagurta is an important food fish commonly


consumed in South Asia. The fish is commonly found in warm shallow waters along
the coasts of the Indian and West Pacific oceans. But Indian Mackerel fishes
collected from Andaman waters are genetically distinctive from those caught from
the Indian mainland. Now Central Marine Fish Research Institute (CMFRI), Kochi
Has begin their genetic profiling. It will help in better fisheries management
http://www.thehindu.com/sci-tech/science/genetic-profiling-of-indianmackerel/article7105902.ece

21. Solution: a)
How knuckle cracking produces popping sound? New research says this happens
because when joint surfaces suddenly separate due to distraction force (or pulling
force) and there is less amount of synovial fiuid to fill the suddenly expanded joint
volume. Thus a cavity or bubble is formed, and sound is produced.
http://health.howstuffworks.com/humanbody/systems/musculoskeletal/question437.htm

22.

Solution: a)

The Patagonian Desert is the largest of the 40 parallel and is a large cold winter
desert, where the temperature rarely exceeds 12 C and averages just 3 C. The region
experiences about seven months of winter and five months of summer. Frost is not
uncommon in the desert but, due to the very dry condition year round, snow is. The
Andes, to the desert's west, are the primary reason for the Patagonian desert status
as they inhibit the westerly flow of moisture from the southern Pacific from reaching
inland. This creates a rain shadow that accounts for the formation of the desert and
is why, despite approximately half of the desert being only about 200 miles from the

http://www.insightsonindia.com

[Type text]

INSIGHTS MOCK TEST 23 SOLUTIONS



http://www.insightsonindia.com

ocean, such a large desert is found in the region. The cold Falkland Current off the
Atlantic coast of South America also contributes to the area's aridity.

23.

Solution: d)

The foggiest place in the world is the Grand Banks off the island of Newfoundland,
the meeting place of the cold Labrador Current from the north and the much warmer
Gulf Stream from the south.
As the Gulf Stream reaches Cape Hatteras, North Carolina, the cold Labrador
Current that flows from the north separates it from the coast. At this confluence, the
warm Gulf Stream waters combine with the cold winds accompanying the Labrador
Current, forming one of the densest concentrations of fog in the world. Because of
this immense heat transfer, atmospheric storms tend to intensify in this region.
In addition, the warm water and temperature contrast along the edge of the Gulf
Stream often increases the intensity of cyclones, tropical or otherwise.

24.

Solution: d)

The word Kathak has been derived from the word Katha which means a
story. Kathakars or story-tellers, are people who narrate stories largely based on
episodes from the epics, myths and legends. It probably started as an oral tradition.
Mime and gestures were perhaps added later on to make the recitation more
effective. Thus evolved a simple form of expressional dance, providing the origins of
what later developed into Kathak as we see it today.
The Vaishnavite cult which swept North India in the 15th century. and the resultant
bhakti movement contributed to a whole new range of lyrics and musical forms. The
Radha-Krishna theme proved immensely popular alongwith the works of Mirabai,
Surdas, Nandadas and Krishnadas.
The emergence of Raslila , mainly in the Braj region (Mathura in Western U.P.) was
an important development. It combined in itself music, dance and the narrative.
Dance in Raslila, however, was mainly an extension of the basic mime and gestures
of the Kathakars or story-tellers which blended easily with the existing traditional
dance.

http://www.insightsonindia.com

[Type text]

INSIGHTS MOCK TEST 23 SOLUTIONS



http://www.insightsonindia.com

With the coming of the Mughals, this dance form received a new impetus. A
transition from the temple courtyard to the palace durbar took place which
necessitated changes in presentation. In both Hindu and Muslim courts, Kathak
became highly stylised and came to be regarded as a sophisticated form of
entertainment. Under the Muslims there was a greater stress
on nritya and bhavagiving the dance graceful, expressive and sensuous
dimensions.

25.

Solution: c)

The nineteenth century saw the golden age of Kathak under the patronage of Wajid
Ali Shah, the last Nawab of Oudh. He established the Lucknow gharana with its
strong accent on bhava, the expression of moods and emotions. The
Jaipur gharana known for its layakari or rhythmic virtuosity and the
Benaras gharana are other prominent schools of Kathak dance. The technique of
movement in Kathak is unique to it.
Today, Kathak has emerged as a distinct dance form. Being the only classical dance
of India having links with Muslim culture, it represents a unique synthesis of Hindu
and Muslim genius in art. Further, Kathak is the only form of classical dance wedded
to Hindustani or the North Indian music. Both of them have had a parallel growth,
each feeding and sustaining the other.

26.

Solution: a)

http://www.insightsonindia.com

[Type text]

INSIGHTS MOCK TEST 23 SOLUTIONS



http://www.insightsonindia.com

27.

Solution: d)

The great landmass of Asiatic continent with its mountainous interior induces great
pressure changes between the summer and winter. Intense heating in the central
part of Asia sets up a region of low pressure in summer and the tropical air stream
Pacific air steam is drawn in as the rain bearing South- East Monsoon. Heavy
precipitation occurs in most parts of China decreasing inland.
In winter a steep pressure gradient is set between the cold interiors of Mongolia and
Siberia and the warmer Pacific Coastlands. The continental polar air mass flows
outwards as the North-West Monsoons, bitterly cold and very dry. There is a great
annual temperature range. Typhoons occur in this type of climate. Typhoons are
intense tropical cyclones that originate in the Pacific Ocean and move west wards to
the coastlands bordering the South China Sea. They are most frequent in late
summer, from July to September and can be very dangerous. The winds blow with
tremendous strength, the sky is overcast and there are torrential downpours.

http://www.insightsonindia.com

[Type text]

INSIGHTS MOCK TEST 23 SOLUTIONS



http://www.insightsonindia.com

28.

Solution: d)

http://media.bom.gov.au/social/blog/18/the-big-bust-southerly-busters-explained/
http://en.wikipedia.org/wiki/Southerly

29.

Solution: b)

Manipuri, one of the main styles of Indian Art or Classical Dances originated in
the picturesque and secluded state of Manipur in the north-eastern corner of India.
Because of its geographical location, the people of Manipur have been protected from
outside influences, and this region has been able to retain its unique traditional
culture.
The origin of Manipuri dance can be traced back to ancient times that go beyond
recorded history. The dance in Manipur is associated with rituals and traditional
festivals, there are legendary references to the dances of Shiva and Parvati and other
gods and goddesses who created the universe.
Lai Haraoba is one of the main festivals still performed in Manipur which has its
roots in the pre-Vaishnavite period. Lai Haraoba is the earliest form of dance
which forms the basis of all stylised dances in Manipur. Literally meaning - the
merrymaking of the gods, it is performed as a ceremonial offering of song and dance.
The principal performers are the maibas and maibis (priests and priestesses) who
re-enact the theme of the creation of the world.

30.

Solution: c)

The coal (Mid-latitude) West margin region having British type of climate lies
between 45 and 60 north and south of the equator on the western margins of the
continent in the belt of the Westerlies.
The summers are warm and the winters are cool. (Summers are net too hot and
winters are not very cold). These areas are affected by the permanent Westerlies
throughout the year. Cyclonic rainfall is experienced throughout the year. They are
affected by warm currents; hence, the temperature is not low despite the high
latitude.
These regions are visited by heat waves (not often) and is welcomed by locals as it
increases the overall temperature of the region.

http://www.insightsonindia.com

[Type text]

INSIGHTS MOCK TEST 23 SOLUTIONS



http://www.insightsonindia.com

31. Solution: a)
http://en.wikipedia.org/wiki/Barley#Uses
Barley is an important cereal grain raised in cool temperate regions. Hence the
question. Read the uses carefully, and also the patterns of weather, soil etc. required
for its growth.

32.

Solution: d)

The sruti thus is the unit of measure or small difference between the various
consecutive pitches within a grama or a scale. For all practical purposes they are said
to be twenty two. This is only as far as practical enumeration is concerned. just as we
would say that there are seven notes in an octave or saptak - from Sa to upper Sa.
But in reality the number of srutis employed in Indian music is infinite.
Getting back to gramas in Bharata's time, there were two, with seven notes each.
Bharata also mentions two other note: these were the antara
gandhara and kakali nishada.
Now, from each grama subsidiary scales are derived. These are
called moorcchanas. The notes are played or sung in a descending manner. There
are seven basic notes in a scale, hence there can be seven moorcchanas. There
were two gramas and each had seven standard notes and two auxiliary ones, as was
mentioned.

33.

Solution: b)

The sruti thus is the unit of measure or small difference between the various
consecutive pitches within a grama or a scale. For all practical purposes they are said
to be twenty two.
A characteristic contribution of India to musical rhythm is the tala. Tala is a cyclic
arrangement of time units. The basic units of time division are laghu, guru, and
pluta. These are actually derived from poetic prosody. Laghu comprises one
syllable,guru two, and plutathree. There are also larger units. Bharata's Natya
Shastra gives details of construction of tala out of various time units, how they
should be played and so on. Later authors developed a scheme of 108 talas.

http://www.insightsonindia.com

[Type text]

INSIGHTS MOCK TEST 23 SOLUTIONS



http://www.insightsonindia.com

34.

Solution: d)

Thumri and Tappa are popular types heard in concerts. The thumri is very
lyrical in its structure and presentation. These forms are termed as 'semi' or 'light'
classical. Thumri is a love song and hence the textual beauty is very important. This
is closely coordinated with the musical rendition. And keeping in mind its mood
a thumri is usually set to ragas like Khamaj, Kaphi, Bhairavi and so on and the
musical grammar is not strictly adered to. There are two styles of thumri singing: the
Poorab or Banaras which is fairly slow and staid and the Punjab style which is more
mercurial. Rasoolan Devi, Siddheshwari Devi are prominent musicians of this style.

35.

Solution: d)

Tappa is a form of Indian semi-classical vocal music. Its specialty is its rolling pace
based on fast, subtle and knotty construction. Its tunes are melodious, sweet and
depict the emotional outbursts of a lover. Tappe (plural) were sung mostly by
songstresses, known as Baigees, in royal courts.
The Tappa consists of the song uttered in fast note patterns. It is a difficult
composition and needs much practice. Both the Thumriand Tappa require special
training as do the Dhrupad and Khyal forms of singing. Ragas in
which Tappa compositions are set remain same as in Thumri style. Pt. L.K. Pandit,
Malini Rajurkar are names who specialize this form of singing.

36.

Solution: c)

Other forms are JATISVARAM, VARNAM, JAVALI, PADA.


There are musical forms belonging to Nibadha and Anibadha Sangeeta, that is,
Kalpita sangeeta and Manodharma sangeeta or improvised music. All these forms
are generally classified under different heads, as Sacred music, Art music, etc. The
several forms under these heads possess distinctive characteristics of their own. The
ancient musical forms like Prabandhas, etc. gradually gave away to the different
musicals forms that are in use in present day music, though the basic elements of the
ancient Prabandhas are still retained in the modern forms. The following musical
forms offer interesting study:
You can read more about them (not necessary however) here
http://www.ccrtindia.gov.in/carnaticclassicalmusic.php

http://www.insightsonindia.com

[Type text]

INSIGHTS MOCK TEST 23 SOLUTIONS



http://www.insightsonindia.com

37.

Solution: d)

http://unctad.org/en/Pages/AboutUs.aspx
UNCTAD, which is governed by its 194 member States, is the United Nations body
responsible for dealing with development issues, particularly international trade
the main driver of development.
Every two years, UNCTAD organizes the World Investment Forum, which brings
together major players from the international investment community to discuss
challenges and opportunities and to promote investment policies and partnerships
for sustainable development and equitable growth.

38.

Solution: c)

Musical instruments are the tangible and material representation of music which is
an auditory art. A study of these helps in tracing the evolution of music and also
explains many aspects of the material culture of the group of people to which these
instruments belong.
Natya Shastra divides them in four main categories on the basis of how sound is
produced.
(i) The Tata Vadya or Chordophones- Stringed instruments
(ii) The Sushira Vadya or Aerophones- Wind instruments
(iii) The Avanaddha Vadya or Membranophones- Percussion instruments
(iv) The Ghana Vadya or Idiophones- Solid instruments which do not require tuning.

39.

Solution: d)

Bhavai is the traditional theatre form of Gujarat. The centers of this form are Kutch
and Kathiawar. The instruments used in Bhavai are: bhungal, tabla, flute, pakhaawaj,
rabaab, sarangi, manjeera, etc. In Bhavai, there is a rare synthesis of devotional and
romantic sentiments.
Maach is the traditional theatre form of Madhya Pradesh. The term Maach is used
for the stage itself as also for the play. In this theatre form songs are given
prominence in between the dialogues. The term for dialogue in this form is bol and
rhyme in narration is termed vanag. The tunes of this theatre form are known as
rangat.

http://www.insightsonindia.com

[Type text]

INSIGHTS MOCK TEST 23 SOLUTIONS



http://www.insightsonindia.com

40.

Solution: a)

Krishnattam, folk theatre of Kerala, came into existence in the middle of 17th
century A.D. under the patronage of King Manavada of Calicut. Krishnattam is a
cycle of eight plays performed for eight consecutive days. The plays are Avataram,
Kaliamandana, Rasa krida, kamasavadha, Swayamvaram, Bana Yudham, Vivida
Vadham, and Swargarohana. The episodes are based on the theme of Lord Krishna his birth, childhood pranks and various deeds depicting victory of good over evil.
Dashavatar is the most developed theatre form of the Konkan and Goa regions. The
performers personify the ten incarnations of Lord Vishnu-the god of preservation
and creativity. The ten incarnations are Matsya (fish), Kurma (tortoise), Varaha
(boar), Narsimha (lion-man), Vaman (dwarf), Parashuram, Rama, Krishna (or
Balram), Buddha and Kalki. Apart from stylized make-up, the Dashavatar performers
wear masks of wood and papier mache.

41. Solution: b)
Theyyam is a traditional and extremely popular folk theatre form of Kerala. The
word 'Theyyam' is derived from the Sanskrit word 'Daivam' meaning God. Hence it is
called God's dance. The tradition of worshipping of spirits of ancestors, folk heroes,
and deities of various diseases and ailments can be traced back to ancient times in
South India. Theyyam is performed by various castes to appease and worship these
spirits. One of the distinguishing features of Theyyam is the colourful costume and
awe-inspiring headgears (mudi) nearly 5 to 6 feet high made of arecanut splices,
bamboos, leaf sheaths of arecanut and wooden planks and dyed into different strong
colours using turmeric, wax and arac.
Mudiyettu, traditional folk theatre form of Kerala is celebrated in the month of
Vrischikam (November-December). It is usually performed only in the Kali temples
of Kerala, as an oblation to the Goddess. It depicts the triumph of goddess
Bhadrakali over the asura Darika. The seven characters in Mudiyettu-Shiva, Narada,
Darika, Danavendra, Bhadrakali, Kooli and Koimbidar (Nandikeshvara) are all
heavily made-up.
Koodiyaattam, one of the oldest traditional theatre forms of Kerala, is based on
Sanskrit theatre traditions. The characters of this theatre form are: Chakyaar or
actor, Naambiyaar, the instrumentalists and Naangyaar, those taking on women's
roles. The Sutradhar or narrator and the Vidushak or jesters are the protagonists. It
is the Vidushak alone who delivers the dialogues. Emphasis on hand gestures and eye
movements makes this dance and theatre form unique.

http://www.insightsonindia.com

[Type text]

INSIGHTS MOCK TEST 23 SOLUTIONS



http://www.insightsonindia.com

42.

Solution: c)

Therukoothu, the most popular form of folk drama of Tamil Nadu, literally means
"street play". It is mostly performed at the time of annual temple festivals of
Mariamman (Rain goddess) to achieve rich harvest. At the core of the extensive
repertoire of Therukoothu there is a cycle of eight plays based on the life of Draupadi.
Kattiakaran, the Sutradhara of the Therukoothu performance, gives the gist of the
play to the audience and Komali entertains the audience with his buffoonery.
Yakshagaana, traditional theatre form of Karnataka, is based on mythological stories
and Puranas. The most popular episodes are from the Mahabharata i.e. Draupadi
swayamvar, Subhadra vivah, Abhimanyu vadh, Karna-Arjun yuddh and from
Ramayana i.e. Raajyaabhishek, Lav-kush Yuddh, Baali-Sugreeva yuddha and
Panchavati.

43.

Solution: d)

In Sanskrit terminology Puttalika and Puttika means little sons. The root of
Puppet is derived from the latin word Pupa meaning a doll. India is said to be the
home of puppets, but it is yet to awaken to its unlimited possibilities. The earliest
reference to the art of puppetry is found in Tamil classic Silappadikaaram
written around the 1st or 2nd century B.C.
Natyashastra, the masterly treatise on dramaturgy written sometime during 2nd
century BC to 2nd century AD., does not refer to the art of puppetry but the
producer-cum-director of the human theatre has been termed
as Sutradhar meaning the holder of strings. The word might have found its place
in theatre-terminology long before Natyashastra was written but it must come
from marionette theatre. Puppetry, therefore, must have originated in India more
than 500 years before Christ.
Almost all types of puppets are found in India.

44.

Solution: a)

http://www.insightsonindia.com

[Type text]

INSIGHTS MOCK TEST 23 SOLUTIONS



http://www.insightsonindia.com

45.

Solution: a)

A foreign direct investment (FDI) is a controlling ownership in a business


enterprise in one country by an entity based in another country.
Foreign direct investment is distinguished from portfolio foreign investment, a
passive investment in the securities of another country such as public stocks and
bonds, by the element of "control". According to the Financial Times, "Standard
definitions of control use the internationally agreed 10 percent threshold of voting
shares, but this is a grey area as often a smaller block of shares will give control in
widely held companies. Moreover, control of technology, management, even crucial
inputs can confer de facto control."
The origin of the investment does not impact the definition as an FDI, i.e., the
investment may be made either "inorganically" by buying a company in the target
country or "organically" by expanding operations of an existing business in that
country.

46.

Solution: c)

http://www.insightsonindia.com

[Type text]

INSIGHTS MOCK TEST 23 SOLUTIONS



http://www.insightsonindia.com

With the arrival of Vaishnavism in the 15th century A.D., new compositions based on
episodes from the life of Radha and Krishna were gradually introduced. It was in the
reign of King Bhagyachandra that the popular Rasleela dances of Manipur
originated. It is said, that this 18th century philosopher king conceived this complete
dance form along with its unique costume and music in a dream. Under successive
rulers, new leelas, and rhythmic and melodic compositions were introduced.
Manipur dance has a large repertoire, however, the most popular forms are the Ras,
the Sankirtanaand the Thang-Ta. There are five principal Ras dances of which
four are linked with specific seasons, while the fifth can be presented at any time of
the year. In Manipuri Ras, the main characters are Radha, Krishna and the gopis.

47.

Solution: d)

The Sattriya dance form was introduced in the 15th century A.D by the great
Vaishnava saint and reformer of Assam, Mahapurusha Sankaradeva as a powerful
medium for propagation of the Vaishnava faith. The dance form evolved and
expanded as a distinctive style of dance later on. This neo-Vaishnava treasure of
Assamese dance and drama has been, for centuries, nurtured and preserved with
great commitment by the Sattras i.e. Vaishnava maths or monasteries. Because of
its religious character and association with the Sattras, this dance style has been
aptly named Sattriya.
Sankaradeva introduced this dance form by incorporating different elements from
various treatises, local folk dances with his own rare outlook. There were two dance
forms prevalent in Assam before the neo-Vaishnava movement such as Ojapali and
Devadasi with many classical elements. Two varieties of Ojapali dances are still
prevalent in Assam i.e. Sukananni or Maroi Goa Ojah and Vyah Goa Ojah. Sukananni
Oja paali is of Sakti cult and Vyah Goa Oja paali is of Vaishnava cult. Sankaradeva
included Vyah Goa Ojah into his daily rituals in Sattra. Till now Vyah Goa Ojah is a
part of rituals of the Sattras of Assam. The dancers in a Oja paali chorus not only sing
and dance but also explain the narration by gestures and stylized movements. As far
as Devadasi dance is concerned, resemblance of a good number of rhythmic syllables
and dance postures along with footwork with Sattriya dance is a clear indication of
the influence of the former on the latter. Other visible influences on Sattriya dance
are those from Assamese folk dances namely Bihu, Bodos etc. Many hand gestures
and rhythmic syllables are strikingly similar in these dance forms.

48.

Solution: d)

http://www.insightsonindia.com

[Type text]

INSIGHTS MOCK TEST 23 SOLUTIONS



http://www.insightsonindia.com

Interestingly, the services sector has the highest share (54.6 per cent) in the gross
capital formation (GCF) of Rs. 35.4 lakhs in 2013-14. This is owing to the GCF in real
estate, ownership of dwelling and professional services at 20.1 per cent, though the
share has fallen in the last two years, followed by trade and repair services (10.6 per
cent) and public administration and defence (10.6 per cent) where there is
improvement in shares. The growth rate of services GCF at 3.1 per cent has also been
higher than the total GCF growth of 1.4 per cent. Infact, the positive GCF growth in
services led to positive growth in total GCF as GCF growth in agriculture and
industry was negative at - 0.3 per cent and - 0.6 per cent respectively. GCF growth in
manufacturing was even more negative at - 5.4 per cent.

49.

Solution: c)

Decision-making power is vested in the OECD Council. It is made up of one


representative per member country, plus a representative of the European
Commission.
The Council meets regularly at the level of permanent representatives to OECD and
decisions are taken by consensus. These meetings are chaired by the OECD
Secretary-General. The Council also meets at ministerial level once a year to discuss
key issues and set priorities for OECD work. The work mandated by the Council is
carried out by the OECD Secretariat.

50.

Solution: c)

http://www.insightsonindia.com

[Type text]

INSIGHTS MOCK TEST 23 SOLUTIONS



http://www.insightsonindia.com

India has signed Comprehensive Bilateral Agreements with the Governments of


Singapore, South Korea, Japan, and Malaysia. A Free Trade Agreement (FTA) in
services and investment was signed with the Association of South East Asian Nations
(ASEAN) in September 2014.
India has joined the Regional Comprehensive Economic Partnership (RCEP) plurilateral negotiations and is continuously engaged in the bilateral FTA negotiations
including Trade in Services with Canada, Israel, Thailand, the European Free Trade
Association (EFTA), Australia, New Zealand, and the EU. Negotiations with Canada
and Australia have not progressed much and modalities for the negotiations are still
being discussed. Negotiations with Thailand are at an advanced stage and with EFTA
are more or less over. India is also engaged in bilateral trade dialogues with the US
under the India-US Trade Policy Forum (TPF), with Australia under the IndiaAustralia Joint Ministerial Commission (JMC), with China under the India-China
Working-Group on Services, and with Brazil under the India-Brazil Trade
Monitoring Mechanism (TMM).

51. Solution: c)
A free-trade area is the region encompassing a trade bloc whose member countries
have signed a free trade agreement (FTA). Such agreements involve cooperation
between at least two countries to reduce trade barriersimport quotas and tariffs
and to increase trade of goods and services with each other. If people are also free to
move between the countries, in addition to FTA, it would also be considered an open
border. It can be considered the second stage of economic integration.
Unlike a customs union (the third stage of economic integration), members of a freetrade area do not have a common external tariff, which means they have different
quotas and customs, as well as other policies with respect to non-members. To avoid
tariff evasion (through re-exportation) the countries use the system of certification of
origin most commonly called rules of origin, where there is a requirement for the
minimum extent of local material inputs and local transformations adding value to
the goods. Only goods that meet these minimum requirements are entitled to the
special treatment envisioned by the free trade area provisions.

52.

Solution: b)

http://en.wikipedia.org/wiki/World_Travel_and_Tourism_Council
WTTC has made it a priority to raise awareness of the negative impact punitive
taxation has particularly aviation tax - on inbound and outbound tourism.

http://www.insightsonindia.com

[Type text]

INSIGHTS MOCK TEST 23 SOLUTIONS



http://www.insightsonindia.com

WTTC campaigns for governments to implement policies that ensure the business
environment is conducive to the growth of Travel & Tourism.
This means planning and investing in appropriate infrastructure and creating a tax
regime which allows the private sector to be competitive.

53.

Solution: a)

Martial art forms throughout the country have been stylized to quasi dance forms,
notable among which are the martial dances of the North-eastern hill tribes,
the Lazim dances of Maharashtra, the Kalaripayattu of Kerala, and the highly
stylized masked Chhau dances of Orissa, West Bengal and Bihar.
Chhau dance is a genre of Indian tribal martial dance which is popular in the Indian
states of Odisha, Jharkhand and West Bengal. There are three subgenres of the
dance, based on its places of origin and development, Seraikella Chhau, Mayurbhanj
Chhau and Purulia Chhau.

54.

Solution: d)

There are seven major classical dance styles Bharatnatyam from Tamil Nadu
and Karnataka, Kathakali, a classical dance-drama from Kerala, Manipuri from
Manipur, Kathak from Uttar Pradesh, Odissi from Orissa, and Kuchipudi from
Andhra Pradesh and Sattriya from Assam which has recently been included in the
fold of Classical Dances. In their present format, their history cannot be traced back
to over two to three hundred years, but they all have links with the ancient and
medieval literary, sculptural and musical traditions of India and of their particular
regions. They all adhere to the canons of classical dance laid down in the Natya
Shastra, a second century C.E. text ascribed to the sage Bharata, to whom it was
supposedly revealed by the Creator, Brahma.

55.

Solution: d)

The Natya Shastra is an ancient Indian treatise on the performing arts,


encompassing theatre, dance and music. It was written during the period between
200 BCE and 200 CE in classical India and is traditionally attributed to the Sage
Bharata.

The Natya Shastra is incredibly wide in its scope. While it primarily deals with
stagecraft, it has come to influence music, classical Indian dance, and literature as
well. It covers stage design, music, dance, makeup, and virtually every other aspect

http://www.insightsonindia.com

[Type text]

INSIGHTS MOCK TEST 23 SOLUTIONS



http://www.insightsonindia.com

of stagecraft. It is very important to the history of Indian classical music because it is


the only text which gives such detail about the music and instruments of the period.
Thus, an argument can be made that the Natya Shastra is the foundation of the fine
arts in India. The most authoritative commentary on the Natya
Shastra is Abhinavabharati by Abhinavagupta.

The text, which now contains 6000 slokas, is attributed to


the muni (sage) Bharata and is believed to have been written during the period
between 200 BCE and 200 CE. The Natya Shastra is based upon the much older
Gandharva Veda (appendix to Sama Veda) which contained 36000 slokas.
Unfortunately there are no surviving copies of the Natya Veda. Though many
scholars believe most slokas were transmitted only through the oral tradition, there
are scholars who believe that it may have been written by various authors at different
times.

56.

Solution: d)

57.Solution: d)
http://en.wikipedia.org/wiki/Intergovernmental_Panel_on_Climate_Change
Read the introduction part only.

http://www.insightsonindia.com

[Type text]

INSIGHTS MOCK TEST 23 SOLUTIONS



http://www.insightsonindia.com

58.

Solution: a)

It has been explained best in


http://wwf.panda.org/about_our_earth/all_publications/living_planet_report/201
2_lpr/demands_on_our_planet/overshoot/
You can refer to this article too
http://en.wikipedia.org/wiki/Overshoot_%28population%29

59.

Solution: d)

This engine will allow launch vehicles to carry satellites of up to capacity of four
tonnes in geostationary orbit. It will also give boost to Indias interplanetary probes
and manned space missions.
Facts about cryogenic engine

A cryogenic engine is more efficient as it provides more thrust for every


kilogram of propellant burnt.

Cryogenic fuels are extremely clean as they give out only water while burning.
The engine uses liquid oxygen at -2530C and liquid hydrogen at -1830C

It can develop the thrust needed in the final state of the rocket to put satellites,
weighing two tonnes or more, into a geosynchronous orbit.

It is essential to master this technology for any space power as launching


heavier satellites requires cryogenic engines even in the lower stages of the
rocket.

It will also make India self-reliant in sending heavier satellites to the required
orbits and eliminate need of dependence on foreign launch vehicles.

It should be noted that India sixth spacefarer after US, Russia, the European Space
Agency, China and Japan to develop a cryogenic engine.
Earlier in January 2015, India had successfully launched GSLV-D5, the first
successful launch vehicle with an indigenous cryogenic engine. But it had capacity to
launch satellites of up to two tonnes.
60.

Solution: d)

The basis of Bhuta song is rooted in superstitions. Some communities of Kerala do


Bhuta rituals to send away the evil ghost and spirits. This ritual is accompanied with
vigorous dancing and the music has a piercing and eerie character.

http://www.insightsonindia.com

[Type text]

INSIGHTS MOCK TEST 23 SOLUTIONS



http://www.insightsonindia.com

The Bhakha form of folk music is popular in Jammu region. Bhakha is sung by the
villagers when harvesting is done. It is considered to be the regional music with most
melodic and harmonious elements. It is sung to the accompaniment of instruments
like harmonium.
Manipurs hills and valley-both are fond of music and dance. Sana Lamok is sung at
the time of coronation ceremony by the Maaiba (priest). It may also be sung to
welcome the king. It is sung to evoke the spirit of Pakhangba, the presiding deity.
There is a belief that this song is potent with magical powers.

61. Solution: d)
First reason is that it has a long, hot growing season with 200 days as frost free.
Second, its moderately high temperature at around 75 degrees F, allows the crop to
slowly mature in six months.
Third, frequent light showers totalling around 40 inches is sufficient for the cotton
crop to grow.
Moreover, bright sunshine and regular irrigation facility enhances the yield of cotton
crops.

62.

Solution: a)

Bharatnatyam Dance is considered to be over 2000 years old. Several texts beginning
with Bharata Muni's Natya Shastra (200 B.C.E. to 200 C.E.) provide information on
this dance form. It is known to be ekaharya, where one dancer takes on many roles in
a single performance. In the early 19th century, the famous Tanjore Quartette, under
the patronage of Raja Serfoji are said to have been responsible for the repertoire of
Bharatnatyam dance as we see it today.
The style was kept alive by the devadasis, who were young girls 'gifted' by their
parents to the temples and who were married to the gods. The devadasisperformed
music and dance as offerings to the deities, in the temple courtyards. Some of the
renowned performers and gurus of the early part of the century belong to
the devadasi families, a well-known name is Bala Saraswati.

63.

Solution: a)

http://www.insightsonindia.com

[Type text]

INSIGHTS MOCK TEST 23 SOLUTIONS



http://www.insightsonindia.com

Facts about International Day of Yoga logo

Reflects peace and harmony for the humanity which is the essence of Yoga.

Folding of both hands in the logo reflects the union of individual


consciousness with universal consciousness.

It also reflects a perfect harmony between mind and body, man and nature
and the holistic approach to health and well-being.

The sun in the logo symbolise the source of energy and inspiration. While the
brown leaves symbolise the earth element.

The green leaves symbolise the nature and blue the fire element.

Background to the Yoga Day

The idea to observe 21 June as the International Day of Yoga was proposed
by Prime Minister Narendra Modi in September 2014 during his speech at
United Nations General Assembly (UN-GA).

In December 2014, UN-GA had unanimously had adopted an India-led


resolution to observe 21 June as the International Day of Yoga.

The resolution was passed within a record time of 75 days. It was also first
resolution in which 177 countries had become co-sponsors and was a world
record.

64.

Solution: d)

NGT also mentioned that burning of garbage and other material like plastic in open
accounts for nearly 30% of the air pollution in the capital and its suburbs.
Earlier, NGT had banned over 15-year-old petrol vehicles and over 10-year-old diesel
vehicles from plying on Delhi-NCR roads and also had issued detailed guidelines to
tackle air pollution from construction activity in the Delhi-NCR area.

http://www.insightsonindia.com

[Type text]

INSIGHTS MOCK TEST 23 SOLUTIONS



http://www.insightsonindia.com

The most publicized concerns from environmentalists about the incineration of


municipal solid wastes (MSW) involve the fear that it produces significant amounts
of dioxin and furan emissions. Dioxins and furans are considered by many to be
serious health hazards.
As for other complete combustion processes, nearly all of the carbon content in the
waste is emitted as CO2 to the atmosphere.
Other gaseous emissions in the flue gas from incinerator furnaces include nitrogen
oxides, sulfur dioxide, hydrochloric acid, heavy metals, and fine particles. Of the
heavy metals, mercury is a major concern due to its toxicity and high volatility, as
essentially all mercury in the municipal waste stream may exit in emissions if not
removed by emission controls.

65.

Solution: a)

66.

Solution: c)

http://www.insightsonindia.com

[Type text]

INSIGHTS MOCK TEST 23 SOLUTIONS



http://www.insightsonindia.com

67.

Solution: d)

Airports Council International (ACI) presented the Airport Service Quality (ASQ)
award to IGIA under the category of handling 25 to 40 million passengers per
annum. Airport Service Quality (ASQ) is the key to understanding how to increase
passenger satisfaction and improve business performance. While the ACI is a trade
association of the worlds airports and was founded in 1991.

IGIA is run by Delhi International Airport Limited (DIAL), a joint venture


between the Airports Authority of India (AAI) and GMR Group-led
consortium which include Fraport and Malaysia Airports Holdings Berhad.

The airport was developed under the public-private partnership (PPP) mode.
The mandate for DIAL was to finance, design, build, operate and maintain
IGIA for 30 years, along with an option to extend it for another 30 years.

Presently, IGIA hosts 6 domestic carriers, 56 international carriers and also


has the capacity to handle the gigantic aircrafts like Airbus A380.

68.

Solution: c)

http://en.wikipedia.org/wiki/Thorium-based_nuclear_power#India
India ranks at 13th position in the world in terms of generation electricity from
nuclear source.

http://www.insightsonindia.com

[Type text]

INSIGHTS MOCK TEST 23 SOLUTIONS



http://www.insightsonindia.com

This information was given by Minister of State (MoS) Dr Jitendra Singh in written
reply in the Rajya Sabha to an unstarred question.
As per the written reply

The ranking was based upon the data published by Power Reactor
Information System (PRIS) of International Atomic Energy Agency (IAEA) in
2014.

Indias current installed nuclear power capacity is 5780 MW. It is expected to


increase to 10080 MW by 2019 on progressive completion of projects.

Union Government also has accorded sanction for two more projects with a
total capacity of 3400 MW.

In future more nuclear power projects based both on indigenous technologies


and with international cooperation are planned.

The total requirement of nuclear fuel i.e. natural uranium is over 5 five years
is estimated to be around 5,940 tons.

It should be noted that India ranks 13th among the thirty one countries in the world
which are generating electricity from nuclear source.

69.

Solution: b)

Though clear cut demarcations in the style of musical presentation, similar to the
gharanas of Hindustani music are not seen in Carnatic music, yet, we do come across
different styles in rendering compositions.
Moving into the 17th century, the history of Carnatic music saw the epoch-making
scheme of the 72 Melakartas, introduced by Venkatamakhi and laid down in his
monumental work Chaturdandi Prakasika in the year 1620 A.D. The Melakarta
scheme is a highly comprehensive and systematic formula which includes within its
fold all the modes used in ancient as well as modern systems of music of the different
parts of the world. The scheme opened out new vistas of raga creation and later
composers like Tyagaraja invented many a beautiful raga by following it.
Many of the musicians and composers of the South were closely acquainted with the
Hindustani system of music also and wherever permissible, adopted Hindustani
ragas for their compositions. The ragas Yaman Kalyan, Hamir Kalyan, Malkaunsa,
Brindavani Sarang, Jaijaivanti, etc. were adopted in their compositions by the
Musical Trinity. Raga Kaphi, Kanada, Khamaj, Paraj, Purvi, Bhairav, etc. bear a very
close resemblance to their counterparts in the Hindustani system of music.
70.

Solution: b)

http://www.insightsonindia.com

[Type text]

INSIGHTS MOCK TEST 23 SOLUTIONS



http://www.insightsonindia.com

Refer to this page http://www.ccrtindia.gov.in/regionalmusic.php


Cultural traditions from various regions of the country reflect the rich diversity of
Regional Music of India. Each region has its own particular style.
Tribal and folk music is not taught in the same way that Indian classical music is
taught. There is no formal period of apprenticeship where the student is able to
devote their entire life to learning the music, the economics of rural life does not
permit this sort of thing. The musical practitioners must still attend to their normal
duties of hunting, agriculture or whatever their chosen profession is.

71. Solution: a)
In Laman a group of girls sing a stanza and a group of boys give reply in the song.
This continues for hours. Interesting is that the girls singing on one of the peaks of
the hill seldom see the faces of the boys singing on another peak. In between is the
hill which echoes their love song. Most of these songs are sung especially in Kullu
Valley.
Kajri is a folk song sung by women, from Uttar Pradesh and adjacent region, during
rainy season. On the third day in the second half of the bhadra, women sing Kajri
songs all through the night, while dancing in a semi-circle.
North India has a strong tradition of singing Sohar songs when a son is born in a
family. This has influenced the muslim culture and a form of Sohar song gained
currency in the muslim families living in some regions of Uttar Pradesh. Sohar
songs unmistakably point to the mingling of two cultures.

72.

Solution: a)

The word Powada itself means the narration of a story in glorious terms. The
narratives are always odes in praise of an individual hero or an incident or place. The
chief narrator is known as the Shahir who plays the duff to keep the rhythm. The
tempo is fast and controlled by the main singer who is supported by others in chorus.
The earliest notable Powada was the Afzal Khanacha Vadh (The Killing of Afzal
Khan) (1659) by Agnidas which recorded Shivajis encounter with Afzal Khan.
Teej is celebrated with great involvement by women of Rajasthan. This is a festival
celebrated on the third day after the new moon or amavasya of shraavana month.
The theme of the songs sung during this festival revolve around the union of Shiva
and Parvati, the magic of monsoon, greenery, peacock dance etc.

http://www.insightsonindia.com

[Type text]

INSIGHTS MOCK TEST 23 SOLUTIONS



http://www.insightsonindia.com

73.

Solution: a)

India has been one of the foremost advocates of long-term global cooperation in
combating climate change in accordance with the principles and provisions of the
UNFCCC. Climate change impacts being witnessed today are a result of the total
accumulated greenhouse emissions for which the major responsibility lies with the
developed nations. Moreover, despite the fast growth registered by some of the
developing countries, a large proportion of people in these countries still live in
extreme poverty. The Indian stance in the climate change negotiations has been
guided by the principle of Common but Differentiated (CBDR). India thus believes
that the climate change agreement of 2015 should take into consideration a whole
gamut of issues including adaptation, finance, technology development and transfer,
capacity building, transparency of action and support in a balanced manner, and loss
and damage in addition to mitigation.
The responsibility of providing financial assistance to the developing countries lies
with the developed
countries and this has been clearly articulated in the UNFCCC. India together with
other developing countries
continue to urge the developed countries to honour their obligation to provide new,
additional, and predictable financial support to developing countries in a
measurable, reportable, and verifiable manner. In this context ambitious
capitalization of the GCF assumes significance. Developed countries have been urged
to provide clear timelines and pathways to reach the US$ 100 billion annual
commitment made by them in 2010.
Technology transfer: Technology forms a major component of any move towards
combating climate change. The important issue in this regard is that while the
developed countries are the frontrunners in clean technology, the developing
countries do not possess either sufficient technical capability or the financial
resources to develop clean technologies. Appropriate mechanisms for smooth
transfer of technology from the developed to developing countries have to be agreed
upon. The intellectual property rights price-tag should not come in the way of such
technology transfer.

74.

Solution: a)

http://www.insightsonindia.com

[Type text]

INSIGHTS MOCK TEST 23 SOLUTIONS



http://www.insightsonindia.com

The Clean Development Mechanism (CDM) is one of the flexibility


mechanisms defined in the Kyoto Protocol (IPCC, 2007) that provides for emissions
reduction projects which generate Certified Emission Reduction units which may be
traded in emissions trading schemes.
The CDM addresses the second objective by allowing the Annex I countries to meet
part of their emission reduction commitments under the Kyoto Protocol by
buying Certified Emission Reduction units from CDM emission reduction projects in
developing countries (Annex I" parties are those countries that are listed in Annex I
of the treaty, and are the industrialized countries. Non-Annex I parties are
developing countries).

75.Solution: a)
This type of climate is found on the eastern margins of continents in warm temperate
latitudes, just outside the tropics.
It is, in fact, the climate of most part of China-a modified form of monsoonal climate.
It is thus also called the (Temperate Monsoon) or China Type of climate.
The Warm Temperate Eastern Margin Climate is typified by a warm moist summer
and a cool, dry winter.
The mean monthly temperature varies between 40F and 78F and is strongly
modified by maritime influence.
Rainfall is more than moderate, anything from 25 inches to 60 inches.

http://www.insightsonindia.com

[Type text]

http://www.insightsonindia.com

INSIGHTS ON INDIA MOCK PRELIMINARY EXAM - 2015


INSIGHTS ON INDIA MOCK TEST - 24
GENERAL STUDIES

PAPER-I
Time Allowed: 2 Hours

Maximum Marks: 200

INSTRUCTIONS
1. IMMEDITELY AFTER THE COMMENCEMENT OF THE EXAMINATION, YOU SHOULD
CHECK THAT THIS TEST BOOKLET DOES NOT HAVE ANY UNPRINTED OR TORN OR
MISSING PAGES OR ITEMS, ETC. IF SO, GET IT REPLACED BY A COMPLETE TEST BOOKLET.
2. You have to enter your Roll Number on the Test I
Booklet in the Box provided alongside. DO NOT
write anything else on the Test Booklet.
4. This Test Booklet contains 100 items (questions). Each item is printed only in English. Each item
comprises four responses (answers). You will select the response which you want to mark on the Answer
Sheet. In case you feel that there is more than one correct response, mark the response which you consider
the best. In any case, choose ONLY ONE response for each item.
5. You have to mark all your responses ONLY on the separate Answer Sheet provided. See directions in the
Answer Sheet.
6. All items carry equal marks.
7. Before you proceed to mark in the Answer Sheet the response to various items in the Test Booklet, you
have to fill in some particulars in the Answer Sheet as per instructions sent to you with your Admission
Certificate.
8. After you have completed filling in all your responses on the Answer Sheet and the examination has
concluded, you should hand over to the Invigilator only the Answer Sheet. You are permitted to take away
with you the Test Booklet.
9. Sheets for rough work are appended in the Test Booklet at the end.
10. Penalty for wrong answers :
THERE WILL BE PENALTY FOR WRONG ANSWERS MARKED BY A CANDIDATE IN THE
OBJECTIVE TYPE QUESTION PAPERS.
(i)

There are four alternatives for the answer to every question. For each question for which a
wrong answer has been given by the candidate, one-third of the marks assigned to that question
will be deducted as penalty.

(ii) If a candidate gives more than one answer, it will be treated as a wrong answer even if one of the
given answers happens to be correct and there will be same penalty as above to that question.
(iii)

If a question is left blank, i.e., no answer is given by the candidate, there will be no penalty for
that question.
http://insightsonindia.com

INSIGHTS ON INDIA MOCK TEST SERIES FOR CIVIL SERVICES PRELIMINARY EXAM 2015

http://insightsonindia.com

Page 1

http://www.insightsonindia.com

1. If a state government receives funding


under the National Wetland
Conservation Programme (NWCP), it
means that
1. The funds are to be used strictly for
conserving wetlands by the state
governments only.
2. Research work on wetlands by
states will not be financed under
the scheme.
3. Wetlands coming under Montreux
record only will be considered
eligible for state funding.
Select the correct answer using the codes
below.
a)
b)
c)
d)

1 and 2 only
2 and 3 only
3 only
None of the above

2. Consider the following statements:


1. If the intensity of an earthquake is
I (one) on the Mercelli Table, this
means there will be total
destruction including the
possibility of objects thrown into
air from the ground
2. Most aftershocks are considered
dangerous because of their higher
magnitude than that of the
mainshock
Which of the above statements is/are correct?
a)
b)
c)
d)

1 Only
2 Only
Both
None

http://insightsonindia.com

3. With reference to the aquatic plant


Water Hyacinth, consider the following
statements:
1. It grows only in tropical and
rainforest zones.
2. Leaves are killed by frost and salt
water.
3. It is considered as an invasive
species in lakes outside its native
area.
4. The roots of the plant naturally
absorb pollutants, including lead,
mercury and strontium.
Select the correct answer using the codes
below.
a)
b)
c)
d)

1 and 2 only
2, 3 and 4 only
3 and 4 only
2 and 4 only

4. Tackling the presence of detergents in


urban water bodies is becoming a
serious issue. Consider the following
with reference to it.
1. Influx of detergent water can result
in an explosion of algal and water
weeds.
2. It may deplete a water body of
oxygen.
3. Sea weeds if introduced gradually,
can act as phytoremediators in
degrading detergents present in a
water body.
Select the correct answer using the codes
below.
a)
b)
c)
d)

1 and 2 only
1 and 3 only
2 and 3 only
All of the above
Page 2

http://www.insightsonindia.com

5. Which of the following is/are the


reasons for the growing threat to Great
Indian Hornbills?
1. Habitat loss
2. Reduction in wild berries in forests
3. Global warming
4. Poaching
Select the correct answer using the codes
below.
a)
b)
c)
d)

1 and 2 only
1, 2 and 4 only
2 and 3 only
1, 3 and 4 only

6. An automobile company in Germany


has recently come up with synthetic
diesel that can be used as a fuel in cars.
This synthetic diesel will be made up of
1. Carbon dioxide
2. Water
3. Bio-ethanol
Select the correct answer using the codes
below.
a)
b)
c)
d)

1 and 2 only
1 and 3 only
2 only
3 only

7. Consider the following with reference


to the government imposed ban on
marine fishing during breeding period:
1. It does not apply to the Exclusive
Economic Zone (EEZ).
2. Banning fishing within the
territorial waters of India is in the
jurisdiction of the state
governments.
3. No foreign fishing vessel is allowed
in Indian waters.
http://insightsonindia.com

Select the correct answer using the codes


below.
a)
b)
c)
d)

1 and 2 only
1 and 3 only
2 and 3 only
3 only

8. Government has taken a number of


steps to protect confidential
information pertaining to Defence
Sector from Cyber-attack. These
include
1. Setting up of Cyber Operation
Centres for threat management and
mitigation
2. Operational networks of the Armed
Forces are air gapped from internet
3. Defence sector uses dedicated
servers connected directly to the
mains internet servers in the USA.
Select the correct answer using the codes
below.
a)
b)
c)
d)

1 and 2 only
1 and 3 only
2 and 3 only
3 only

9. Consider the following:


1. Vienna Convention (1963)
2. Paris Convention (1960)
3. Brussels Supplementary
Convention (1963)
All the three above conventions are related to
which of the following?
a) Eliminating chmeical weapons
b) Combating smuggling of parts of
wild animals
c) Nuclear liability
d) Dealing with illegal immigrants
into Europe
Page 3

http://www.insightsonindia.com

10. Adulteration of Milk and Milk


Products has been on the agenda of the
Food Safety and Standards Authority
of India (FSSAI) for the last few years.
As per FSSAI, inclusion of which of the
above have been used in the
adulteration of milk, which are
punishable under the law?
1. Presence of Glucose
2. Sub-standard Fat content
3. Skim milk powder
4. Urea

2. It includes Japan and South Korea


also.
3. It sponsors an exclusive ASEAN
bank dedicated to finance member
nations.
4. The ASEAN region is economically
a single market.
Select the correct answer using the codes
below.
a)
b)
c)
d)

1 and 4 only
1 and 3 only
2 and 3 only
1 only

Select the correct answer using the codes


below.
a)
b)
c)
d)

1 and 4 only
1 and 3 only
2 and 3 only
All of the above

11. Consider the following pairs:


1. Seshachalam hills Biosphere
Reserve: Himachal Pradesh
2. Lohabarrack Salt Water Crocodile
Sanctuary : Orissa
3. Talley Valley Wildlife Sanctuary :
Arunachal Pradesh
4. Betla National park : Jharkhand
Which of the above pair(s) is/are correctly
matched?
a)
b)
c)
d)

1 and 2 only
3 and 4 only
4 only
2 and 3 only

12. With reference to a grouping of


countries called as ASEAN, Consider
the following:
1. India is not a founding member of
ASEAN.
http://insightsonindia.com

13. Consider the following statements


1. The major part of basins of Bhavani
and Amaravati Rivers, both major
tributaries of River Kaveri, lies in
the state of Karnataka
2. Unlike other rivers of peninsular
India, River Kaveri is perennial
thanks to contribution of rainfall
both from Southwestern and
Northeastern monsoon winds
Which of the above statements is/are correct?
a)
b)
c)
d)

1 Only
2 Only
Both
None

14. The Union Cabinet approved the


flagship Namami Gange Program
which integrates the efforts to clean
and protect the Ganga river in a
comprehensive manner. Consider the
following statements about it.
1. The program will be implemented
by the Ministry of Water Resources.
2. The program will be 100% funded
by the Centre.
Page 4

http://www.insightsonindia.com

3. Bio-remediation and in-situ


treatment will not be adopted by
the program to clean Ganga as they
may degrade the water quality.
Select the correct answer using the codes
below.
a)
b)
c)
d)

1 and 2 only
1 and 3 only
2 and 3 only
2 only

15. Which of the following


terms/hypothesis/theories are
concerned with the evolution of species
on Earth?
1. Lamarckism
2. Pangenesis
3. Incrementalism
4. Genetic hitchhiking
Select the correct answer using the codes
below.
a)
b)
c)
d)

1 and 2 only
1 and 3 only
2 and 4 only
1, 2 and 4 only

17. The Union Cabinet has recently


approved for introduction of the
Compensatory Afforestation Fund Bill,
2015 in Parliament. The bill
1. will be applicable to state owned
forests too.
2. creates National Compensatory
Afforestation Fund (CAF) authority
3. excludes cases of compensatory
afforestation from the purview of
the National Green tribunal (NGT)
Select the correct answer using the codes
below.
a)
b)
c)
d)

1 and 2 only
1 and 3 only
2 and 3 only
All of the above

18. The skill of camouflage can be


observed in which of the following
species?
1. Stone flounder
2. Leopards
3. Leaf-tailed gecko
Select the correct answer using the codes
below.

16. The shale gas potential as estimated by


the government lies in which of the
following selected sedimentary basins/
sub basins?
1. Cambay
2. Damodar
3. Cauvery
4. Krishna-Godavari
Select the correct answer using the codes
below.
a)
b)
c)
d)

1 and 2 only
1 and 3 only
2 and 4 only
All of the above

http://insightsonindia.com

a)
b)
c)
d)

1 and 2 only
1 and 3 only
2 and 3 only
All of the above

19. A Minister is NOT eligible for election


to which of these Parliamentary
Committees?
1. Committee on Estimates
2. Public Accounts Committee
3. Committee on Public Sector
Undertakings (PSUs)

Page 5

http://www.insightsonindia.com

Select the correct answer using the codes


below.
a)
b)
c)
d)

1 and 2 only
1 and 3 only
2 and 3 only
All of the above

20. Which of the following add(s) nitrogen


or help in nitrogen fixation in the
nitrogen cycle on the planet earth?
1. Volcanic action
2. Lightning
3. Precipitation
4. Decomposition of plants and
animals

22. Oxygen in water is vital for aquatic


organisms. The level of oxygen in
water can be depleted by which of the
following?
1. Diffusion to air
2. Heat pollution
3. Photosynthetic activity of aquatic
plants
4. Presence of bio-degradable waste
Select the correct answer using the codes
below:
a)
b)
c)
d)

1, 2 and 3 only
2 and 4
1 and 3
2, 3 and 4 only

Select the correct answer using the codes


below.
a)
b)
c)
d)

1 and 2 only
1 and 3 only
2 and 4 only
All of the above

21. Scavengers are important in the food


web as they help clean the earth of
organic garbage keeping. Which of the
following are scavengers?
1. Bear
2. Seagulls
3. Racoons
4. Marine eels
Select the correct answer using the codes
below.
a)
b)
c)
d)

1 and 2 only
1 and 3 only
2 and 4 only
All of the above

http://insightsonindia.com

23. Ramya, a native of Tamil Nadu, is


moving into Sri Lanka to permanently
reside there and become its citizen in
the future. If she moves and settles in
Sri Lanka, she is a
a) Migrant
b) An immigrant
c) An emigrant
d) Both immigrant and emigrant

24. If you travel across Nilgiri Hills, you


are likely to see which of these native
vegetations?
1. Eucalyptus
2. Sholas
3. Wattle
Select the correct answer using the codes
below:
a)
b)
c)
d)

1 and 2 only
2 only
1 and 3 only
All of the above

Page 6

http://www.insightsonindia.com

25. Which of the following are major


pollutants released by Nuclear power
plants?
1. Strontium
2. Tritium
3. Mercury
4. Sulphur dioxides
Select the correct answer using the codes
below:
a)
b)
c)
d)

1 and 2 only
1, 2 and 3 only
1 and 4 only
2, 3 and 4 only

26. When you dine out at a restaurant,


these two taxes appear on your bill
other than the service charge.
1. Service Tax
2. VAT
Who collects these taxes?
a) Both by State government
b) Both by Central government
c) Service tax by Central government;
and VAT by state governments
d) Service tax by State government;
and VAT by both Central and state
governments

27. An Angle investor gives fund to


a) large industries at their time of
incurring losses
b) renovate and replace damaged
equipments in industries that
suffered from a natural disaster
c) finance budding start-ups usually
in exchange for ownership equity
d) help young companies as a
philanthropic grant

28. Which of the following changes in


India necessitated a shift from
Planning to Policy giving rise to the
NITI Ayog?
1. Expansion in Demography
2. Shift towards market economy
3. Increasing global integration
Select the correct answer using the codes
below:
a)
b)
c)
d)

1 and 2 only
2 only
1 and 3 only
All of the above

29. Which of these items are counted


under the Net Income from Abroad
(NIA) while calculating Gross National
Product (GNP)?
1. Foreign Institutional Investment
(FII)
2. Private remittances
3. Imports and Exports
4. Foreign Direct Investment (FDI)
Select the correct answer using the codes
below:
a)
b)
c)
d)

1 and 2 only
1 and 4 only
2 and 3 only
2, 3 and 4 only

30. The Current Account reflects the


position of the external sector of the
economy. The Current account deficit
includes
1. Foreign direct Investment
2. Foreign portfolio investment
3. Loans taken by the government
4. Grants given to the foreign nations
Select the correct answer using the codes
below:

http://insightsonindia.com

Page 7

http://www.insightsonindia.com

a)
b)
c)
d)

1 and 2 only
1 and 4 only
2 and 3 only
None of the above

31. The term Liquidity Adjustment


facility, frequently appearing in news,
is a
a) Monetary policy tool which allows
the central bank to tweak the liquid
money available with the banks
b) Monetary policy tools which allows
the Central bank to control the
issue of government securities in
the market
c) fiscal policy tool used by the
government to control upward
liquidity pressures in the economy
d) fiscal policy tool used by the
government to control credit issued
by the Central bank to the other
banks

32. Business Correspondents (BCs) or


Bank Mitras help in last mile
connectivity to further financial
inclusion. As per the Jan Dhan Yojana,
what is the eligibility criterion to
become a Bank Mitra?
a) The individual should be individual
owners of kirana shops/PCOs etc.
or agents of insurance companies in
India
b) The individual should be a retired
government employee or had
served in the local government
c) Any individual who has completed
18 years of age can become Bank
Mitra.
d) The individual should be above 18
years of age and should have at
least secondary education

http://insightsonindia.com

33. In the Indian economy, which of these


purposes is/are served by a Reverse
Repo rate?
1. Regulating the money supply in the
economy
2. Controlling inflation in the
economy
3. To prevent the banks from making
excessive profits
4. To help banks maintain a certain
level of cash with them for day to
day operations
Select the correct answer using the codes
below:
a)
b)
c)
d)

1 and 2 only
1 and 4 only
2 and 3 only
1, 2 and 4 only

34. The ISIS crisis, also known as the


Northern Iraq offensive, that has
affected several parts of Iraq began
with the capture of which city?
a) Mosul
b) Tirkit
c) Kirkuk
d) Baghdad

35. Consider the following statements


about the International Union for
Conservation of Nature (IUCN).
1. It is an inter-governmental
organization.
2. It works independently of nonprofit and non-governmental
organizations.
3. IUCN is not concerned with social
issues like Gender equality and
poverty alleviation.
4. Its principle tool is to mobilize the
public in support of nature
conservation.
Page 8

http://www.insightsonindia.com

Select the correct answer using the codes


below:
a)
b)
c)
d)

1 and 2 only
None of the above
3 and 4 only
2, 3 and 4 only

36. The present Yemeni Civil war has been


caused due to the
a) infighting between two groups to
constitute Yemeni government
b) some communal factions trying to
impose religious and legal order in
Yemeni society
c) spill over of the ISIS crisis in
Yemen
d) rebellion caused by the deprived
tribals from Northern Yemen for
civil rights and development

37. Consider the following about Namaste


Russia, a festival of Indian culture,
recently initiated by the Indian
President.
1. It will be a six month long festival.
2. It will be showcased and celebrated
both in India and Russia.
3. The festival is being hosted under
the patronage of Indian Council for
Cultural Relations (ICCR) from
Indias side.
Select the correct answer using the codes
below:
a)
b)
c)
d)

1 and 2 only
2 and 3 only
1 and 3 only
All of the above

38. World Health Organization (WHO),


the U.N. health agency, has recently
http://insightsonindia.com

declared Liberia Ebola-free. What is


the criterion for declaring a nation
Ebola-free?
a) no new Ebola cases or deaths
should be reported for three
months continuously
b) if no active viruses are found in a
special quarantine conducted by
WHO
c) no new Ebola cases should be
reported over two incubation
periods of the virus
d) If all the Ebola patients survive the
attack with no new cases being
reported

39. Consider the following statements:


1. Lithium chloride is used as a
relative humidity standard in the
calibration of hygrometers
2. Lithium chloride is widely used as
salt substitute to circumvent the
risk of high blood
pressure and cardiovascular
disease associated with a high
intake of sodium chloride
Which of the above statements is/are correct?
a)
b)
c)
d)

1 Only
2 Only
Both
None

40. India and Iran have signed a


Memorandum of Understanding
(MoU) for the development of the
strategically important Chabahar Port
in Iran. What will be the benefits for
India being able to use this port?
1. India will get sea-land access route
to Afghanistan bypassing Pakistan.
2. Saving transportation costs in
trading crude oil and urea
Page 9

http://www.insightsonindia.com

Which of the above is correct?


a)
b)
c)
d)

1 only
2 only
Both 1 and 2
None

41. With reference to the Chakma


community in India, consider the
following:
1. They live mainly in the state of
Assam.
2. They profess Theravada Buddhism.
3. They have been kept in the category
of scheduled tribes.
Select the correct answer using the codes
below:
a)
b)
c)
d)

1 and 2 only
2 and 3 only
1 and 3 only
All of the above

42. Production of bio-diesel is assuming


significance in agricultural
biotechnology. Which of these can be
used to produce Bio-diesel?
1. Microalgae
2. Jatropha
3. Waste plastic
4. Hemp
Select the correct answer using the codes
below:
a)
b)
c)
d)

1 and 2 only
2 and 3 only
1 and 4 only
All of the above

43. Which of the following schedules of the


Constitution have a bearing on the
governance of tribals in India?
http://insightsonindia.com

1.
2.
3.
4.

Fifth schedule
Sixth Schedule
Seventh Schedule
Tenth Schedule

Select the correct answer using the codes


below:
a)
b)
c)
d)

1 and 2 only
2 and 3 only
1 and 4 only
1, 2 and 3 only

44. Consider the statement:


Detached, generally dense clouds and
with sharp outlines that develop
vertically in the form of rising mounds,
domes or towers with bulging upper
parts often resembling a cauliflower.
The above description refers to which
of the following types of cloud?
a)
b)
c)
d)

Nimbostratus
Cumulus
Cirrus
Cirrocumulus

45. Consider the following statements


about National parks in India.
1. A state government cannot
establish National parks.
2. Human communities are allowed to
live in a National park.
3. Resource use is not allowed in the
buffer zones of the National parks.
Select the correct answer using the codes
below:
a)
b)
c)
d)

1 and 2 only
2 and 3 only
3 only
None of the above
Page 10

http://www.insightsonindia.com

46. Climate change is a change in the


statistical distribution
of weather patterns when that change
lasts for an extended period of time. It
can be caused due to factors like
1. Plate tectonics
2. Volcanic eruptions
3. Extreme variation in solar spots
4. Largescale death of phytoplanktons
Select the correct answer using the codes
below:
a)
b)
c)
d)

1, 2 and 4 only
2 and 3 only
1, 3 and 4 only
All of the above

47. The Directive Principles of State Policy


(DPSP) concern with which of the
following matters?
1. Conservation of environment
2. Protection of National Flag and
honour
3. Labour rights
Select the correct answer using the codes
below:
a)
b)
c)
d)

1 and 2 only
2 and 3 only
1 and 3 only
All of the above

48. The Atal Pension Yojana launched in


the Union Budget 2015-16 is focused
on
a) all citizens in the unorganized
sector
b) all income tax payer citizens of
India who contribute voluntarily
c) all BPL citizens in the unorganized
sector
http://insightsonindia.com

d) all women citizens and BPL citizens

49. Consider the following about the


Pradhan Mantri Jeevan Bima Yojana
launched recently.
1. It is applicable to bank account
holders between the ages of 18-50.
2. The scheme is restricted to rural
areas only.
3. It will cover insurance in case of
any disability arising out of a
natural hazard or accident.
Select the correct answer using the codes
below:
a)
b)
c)
d)

1 and 2 only
2 and 3 only
1 and 3 only
1 only

50. Which of the following bodies are


associated with Economic Planning in
India?
1. NITI Ayog
2. The Parliament
3. Zila Panchayats
4. State Planning departments
Select the correct answer using the codes
below:
a)
b)
c)
d)

1 and 2 only
2, 3 and 4 only
1 and 3 only
All of the above

51. Consider the following statements


about the Prime Ministers Office
(PMO)?
1. The PMO includes the anticorruption unit and the public wing
dealing with grievances.
Page 11

http://www.insightsonindia.com

2. Monitors the actions taken by the


PM in earlier meetings.
3. It generally handles all activities
not assigned to any particular
department.
Select the correct answer using the codes
below:
a)
b)
c)
d)

1 and 2 only
2 and 3 only
1 and 3 only
All of the above

52. A welfare state is a state


1. which cannot be run wholly on the
principles of capitalism
2. where public responsibility is for
those who are unable to avail
themselves of the minimal
provisions for a good life
3. which is necessarily democratic.
Select the correct answer using the codes
below:
a)
b)
c)
d)

1 and 2 only
2 and 3 only
1 and 3 only
All of the above

53. In appointing Ministers, which of the


following are followed by the President
and the Governor?
1. Constitution of India
2. Laws made by Parliament
3. Conventions
Select the correct answer using the codes
below:
a)
b)
c)
d)

1 and 2 only
2 and 3 only
1 and 3 only
All of the above

http://insightsonindia.com

54. If rupee starts depreciating too much


in the foreign exchange market, it may
influence
1. Exports from India
2. Inflation in India
3. Foreign Direct Investment in India
Select the correct answer using the codes
below:
a)
b)
c)
d)

1 and 2 only
2 and 3 only
1 and 3 only
All of the above

55. As per the constitution, the Indian


president has the power to
1. Ask for files relating to any matter
from Prime Minister that is in
deliberation in the Council of
Ministers
2. Send the decision of an individual
ministry for re-consideration
3. Suspend the council of ministers if
he feels it is violating constitutional
norms
Select the correct answer using the codes
below:
a)
b)
c)
d)

1 and 2 only
2 and 3 only
1 and 3 only
All of the above

56. In Parliament, the primary object of an


adjournment motion is to
a) suspend the proceedings of the
house for that particular session
b) draw the attention of the House to a
recent matter of urgent public
importance

Page 12

http://www.insightsonindia.com

c) censure the government for its lack


of action on sensitive issues
d) adjourn the introduction of a bill
for the next session of the house

57. As per Ayurveda, consuming water


stored in copper vessel for some time
is beneficial for the human body in
which of the following ways?
1. It balances all three doshas vata,
kapha and pitta.
2. It detoxifies the stomach.
3. clears the water of germs
Select the correct answer using the codes
below:
a)
b)
c)
d)

1 and 2 only
2 and 3 only
1 and 3 only
All of the above

58. Observe the following image:

The above picture is of a coral reef, but


of which of the following types?
a)
b)
c)
d)

Atoll
Barrier reef
Fringing reef
Peripheral reef

http://insightsonindia.com

59. Why do plants generally take oxygen


and not CO2 in night time?
a) Plant respiration is carried out only
in the night.
b) At night no photosynthesis takes
place
c) plants grow during night time and
require the oxygen for their growth
d) plants metabolize food only in the
night time

60. India's national ID program


called Aadhaar is the largest biometric
database in the world. Apart from face
photo, which other biometric features
are captured by it?
1. Fingerprint
2. Iris Scan
3. Voice tone recognition
Select the correct answer using the codes
below:
a)
b)
c)
d)

1 and 2 only
2 and 3 only
1 only
All of the above

61. Plants can be genetically engineered to


serve which of the following purposes?
1. Phytoremediation
2. Achieving salt tolerance
3. Biofuel research
Select the correct answer using the codes
below:
a)
b)
c)
d)

1 and 2 only
2 and 3 only
1 only
All of the above

Page 13

http://www.insightsonindia.com

62. Which of the following spacecrafts


are/were on trajectories that escape
our solar system for probe or research?
1. Pioneer 10
2. New Horizons
3. Voyager 1
4. Vostok 1
5. Endeavour
Select the correct answer using the codes
below:
a)
b)
c)
d)

1, 2 and 3 only
2, 3 and 5 only
1, 3 and 4 only
1, 2 and 5 only

63. Under Article 143 of the Constitution,


the Supreme Court can exercise
advisory jurisdiction to advise
1. President
2. Governors
3. Prime Minister
4. Chief Ministers
Select the correct answer using the codes
below:
a)
b)
c)
d)

1, 2 and 3 only
2, 3 and 4 only
1 and 2 only
1 only

64. Which of the following terms/practices


are concerned with genetic
modification/engineering in plants?
1. Promoter sequence
2. Gene Gun method
3. Agrobacterium method
Select the correct answer using the codes
below:
a)
b)
c)
d)

1 and 2 only
2 and 3 only
1 only
All of the above

http://insightsonindia.com

65. Consider the following organisms


1. Bacteria
2. Fungi
3. Angiosperms
4. Protozoa
Which of these statements is correct about
them?
a) They all belong to different classes,
but same phylum.
b) They all belong to different
Kingdoms.
c) They belong to the same Kingdom,
but different phylum.
d) They belong to the same Kingdom,
but different classes.

66. Changing the number of puisne


(regular) judges in the Supreme Court
or High Court requires legislation by
the Parliament with
a) Ordinary majority
b) Special majority
c) Absolute majority
d) Special majority with consent of
majority of states

67. Which of these places are famous for


producing one or the other traditional
handicrafts of India?
1. Vellore
2. Aligarh
3. Rajgir
4. Nagpur
Select the correct answer using the codes
below:
a)
b)
c)
d)

1 and 2 only
2 and 3 only
1 and 4 only
All of the above
Page 14

http://www.insightsonindia.com

68. Which of the following International


environmental agreements that are
either legally binding for nations or
carry legal authority?
1. Convention on the Conservation of
Antarctic Marine Living Resources
2. Cartagena Protocol on Biosafety
3. Stockholm Convention on
Persistent Organic Pollutants
Select the correct answer using the codes
below:
a)
b)
c)
d)

1 and 2 only
2 and 3 only
1 and 3 only
All of the above

69. What is the right order for the


following as primary, secondary and
tertiary consumers respectively in the
food web?
a) Algae, Krills, Lobster
b) Krill, Shrimp, Dolphin
c) Shrimp, Seastar, Shark
d) Oysters, Silverside, Humpback
whales
70. Which of the following is/are the
difference between the Hindu calendar
and Gregorian calendar system?
1. Hindu calendar is based on lunar
and solar cycles, while Gregorian is
based only on Solar cycles.
2. Hindu calendar system is rooted in
astronomical calculations and
predictions, while Gregorian
calendar is not.
Which of the above is/are correct?
a)
b)
c)
d)

1 only
2 only
Both 1 and 2
None

http://insightsonindia.com

71. With reference to El Nino, consider the


following statements:
1. During El Nino phase, the Western
Pacific will have high pressure
whereas Eastern Pacific will have
low pressure
2. The east coast of South America
receives increased rainfall due to El
Nino
Which of the above statements is/are
INCORRECT?
a)
b)
c)
d)

1 Only
2 Only
Both
None

72. Which of the following are NOT part(s)


of Indias National Seed Policy?
1. Encouragement to export of seeds
2. Abolition of licensing for seed
dealers
3. Development of new and improved
varieties of plants
Select the correct answer using the codes
below:
a)
b)
c)
d)

1 and 2 only
2 and 3 only
1 only
All of the above are parts of India
national seed policy.

73. Performed for the local forest deity


Umang-Lai, and songs on creation and
rituals are sung in which of these
festivals in the North-eastern part of
India?
a) Lai Haraoba
b) Chai Hia
c) Saikuti Zai
d) Bihu
Page 15

http://www.insightsonindia.com

74. A marine protected area (MPA) is


essentially a space in the ocean where
human activities are more strictly
regulated than the surrounding waters.
Which of these in India are MPAs?
1. Bhitarkanika
2. Gulf of Kachchh
3. Pulicat Lake
4. Lothian Island
5. Balukhand Konark
Select the correct answer using the codes
below:
a)
b)
c)
d)

1, 3 and 4 only
2, 3 and 5 only
1, 2 and 5 only
All of the above

75. Desertification of land has


consequences that include
1. Increased CO2 levels in the
atmosphere
2. Loss of biodiversity
3. Increased landslides
4. Lowering of Groundwater table
Select the correct answer using the codes
below:
a)
b)
c)
d)

1 and 3 only
2, 3 and 4 only
1, 2 and 4 only
All of the above

76. Ministry of Environment and Forests


has several subordinate and statutory
bodies functioning under it. These are
1. Central Pollution Control Board
2. Animal Welfare board of India
3. National Biodiversity Authority
4. Zoological Survey of India

http://insightsonindia.com

Select the correct answer using the codes


below:
a)
b)
c)
d)

1 and 3 only
2, 3 and 4 only
1, 2 and 4 only
All of the above

77. Consider the following about the


Global Competitiveness Report.
1. It is published by World Economic
Forum.
2. It compares competitiveness of
sovereign states as well as top 500
companies in the World.
3. It also ranks those civil society
organizations that have been most
effective in building a vibrant
business climate in a nation.
Select the correct answer using the codes
below:
a)
b)
c)
d)

1 and 2 only
2 only
1 only
1 and 3 only

78. Free radicals are highly reactive and


have the potential to cause damage to
cells, including damage that may lead
to cancer. Free radicals are formed
naturally in the body due to
a) an oxidation reaction in cells
b) a reduction reaction in cells
c) natural process of replacement of
cells
d) stress in body cells

Page 16

http://www.insightsonindia.com

79. Antioxidants are chemicals that block


the activity of other chemicals known
as free radicals. Which of these
Vitamins contain Antioxidants?
1. Vitamin E
2. Vitamin C
3. Vitamin B6
Select the correct answer using the codes
below:
a)
b)
c)
d)

1 and 2 only
2 only
1 only
1 and 3 only

80.The escape of nanoparticles into


atmosphere can be harmful for
humans. In which of the following
processed/ways can they escape?
1. Nanoparticles used in soil
regeneration
2. Emission with the exhaust in
automobiles
3. Washing of hydrophobic
nanocoatings (paint)
Select the correct answer using the codes
below:
a)
b)
c)
d)

1 and 2 only
2 only
1 only
All of the above

81. Non-physical savings in the Indian


economy will increase if
1. Inflation levels increase
2. Insurance cover expands
3. More number of bank branches
operate in rural and remote areas
4. Financial literacy is imparted on a
large scale basis

http://insightsonindia.com

Select the correct answer using the codes


below:
a)
b)
c)
d)

1, 2 and 3 only
2 and 3 only
2, 3 and 4 only
1 and 4 only

82. Consider the following about the


Samkhya school of Vedic thought.
1. It does not consider the existence of
God as relevant to human life.
2. It believes that only consciousness
exists and matter is illusion.
3. It emphasizes on meditation and
contemplation to realization.
Select the correct answer using the codes
below:
a)
b)
c)
d)

1 and 2 only
2 and 3 only
1 and 3 only
1 only

83. Ajivika, a heterodox school of Indian


philosophy believes that
a) Free will does not exists in human
beings
b) liberation or mukti is irrelevant to
human life
c) our universe is a material one
where humans can exercise free will
d) Both (a) and (b)

84. Samskara, a world classic, written by


U.R. Anathamaurthy portrays the
1. Spiritual struggle of man in terms
of urgency of lifes demands
2. frequent use of mythological
sequences as structural images in
an urban setting

Page 17

http://www.insightsonindia.com

3. the dogmas likely to faced by an


over growing cultural loss faced by
urban civilization
Select the correct answer using the codes
below:
a)
b)
c)
d)

1 and 2 only
2 and 3 only
1 and 3 only
1 only

85. Consider the following about the


theatre form Swang, performed in
the northern parts of India.
1. Music is not used in the
performance.
2. Dialogues are based mostly in
forms of poems.
3. Women do not take part in these
performances.
4. The plays contain a rare synthesis
of devotional and romantic
sentiments.
Select the correct answer using the codes
below:
a)
b)
c)
d)

1 and 4 only
2 and 3 only
All of the above
None of the above

86. Consider the following about the Pala


art.
1. The Pala painting is characterised
by straight lines and bright tones of
colour.
2. Pala style was transmitted chiefly
by means of bronze sculptures and
palm-leaf paintings.
3. It celebrates the human forms of
divinity.
4. The Pala art came to a sudden end
after the destruction of the
Buddhist monasteries at the hands
http://insightsonindia.com

of Muslim invaders in 13th


century.
Select the correct answer using the codes
below:
a)
b)
c)
d)

1 and 4 only
2, 3 and 4 only
All of the above
1, 2 and 4 only

87. There are several passes in the Northwestern Mountains of India. From
them, Alexander of Macedon came to
India through
a) Swat valley
b) Bolan pass
c) Khyber pass
d) Karakoram pass

88. In the Mesolithic age or middle stone


age period
1. domestication of animals was not
practiced
2. use of bows and arrows began
3. primitive cultivation was started
4. the practice of burial of human
bodies after death was not known
Select the correct answer using the codes
below:
a)
b)
c)
d)

1 and 4 only
2, 3 and 4 only
2 and 3 only
1, 2 and 4 only

89. With reference to ocean currents,


consider the following statements:
1. Their knowledge helps in reducing
the cost of shipping
2. Ocean currents helps in dispersal of
many life forms
Page 18

http://www.insightsonindia.com

3. Ocean currents affect temperatures


throughout the world
Which of the above statements is/are correct?
a)
b)
c)
d)

1 and 3 Only
2 and 3 Only
1 and 2 Only
All

90. Right faith is the belief in the teachings


and wisdom of Mahavira. As per
Mahavira
1. All objects whether animate or
inanimate have souls and feel pain
when they are injured
2. Asceticism and renunciation are
important in the path of realization
3. Vedas do not have divine authority.
Select the correct answer using the codes
below:
a)
b)
c)
d)

1 and 2 only
1 and 3 only
2 and 3 only
All of the above

91. Which of the following were parts of


the official policy followed by Lord
Wiilliam Bentick towards British
India?
1. Policy of non-intervention and nonaggression with the Indian states
2. welfare of Indian subjects
3. Indian education to be totally in
vernacular languages
Select the correct answer using the codes
below:
a)
b)
c)
d)

1 and 2 only
1 and 3 only
2 and 3 only
All of the above

http://insightsonindia.com

92. Henry Vivian Derozio was the founder


of the Young Bengal Movement. It was
concerned with
1. attacking old traditions and
decadent customs
2. Promoting womens rights and
education
3. Condition of farmers in the
countryside
Select the correct answer using the codes
below:
a)
b)
c)
d)

1 and 2 only
1 and 3 only
2 and 3 only
All of the above

93. After the perceived failure of the


moderates, the extremists continued
their expansion and attacks on the
British. Main objectives of the
extremists were
1. Self-government with dominion
status for India
2. Provincial autonomy from Central
legislature
3. More power for the local bodies
Select the correct answer using the codes
below:
a)
b)
c)
d)

1 and 2 only
1 and 3 only
2 and 3 only
1 only

94. The Swadeshi movement 1905 in


response to the division of Bengal
involved which of the following?
1. boycott of government service,
courts, schools and colleges

Page 19

http://www.insightsonindia.com

2. Promotion of National Education


through the establishment of
national schools and colleges
3. reviving age old local governments
and empowering them with legal
authority
4. formation of a volunteer armed
front that would fight against the
British
Select the correct answer using the codes
below:
a)
b)
c)
d)

1 and 2 only
1 and 4 only
2 and 4 only
2 and 3 only

95. The Lucknow pact of 1916 was


a) an agreement between the British
and Indian leaders about reforms
in Indian legislative council
b) an agreement between Muslim
League and Congress to merge as
one political body
c) an agreement where Congress
accepted the demand of separate
electorates for Muslims by the
Muslim league
d) an understanding reached between
British government and Indian
National Congress for selfgovernment, and separation of
electorates for communities

c) Indian resources will be drained


which has to paid by the Indian
public
d) The war will result in large scale
causalities of Indian soldiers who
were mostly peasants and poor
landless labourers

97. The chief cause(s)/event(s) leading to


the Khilafat Movement was/were?
1. defeat of Turkey in the First World
War.
2. partitioning of the ottoman empire
3. imposition of secularism on Turkey
by the British
Select the correct answer using the codes
below:
a)
b)
c)
d)

1 and 2 only
1 and 3 only
2 and 3 only
All of the above

98. Consider the following pairs:


1. Gramin Bhandaran Yojana Ministry of Food Processing
2. Mid-day meal scheme - Ministry of
Women and Child development
3. Bachat Lamp Yojana - Ministry of
Rural Development
Which of the above pairs is/are correctly
matched?

96. The Indian National Congress


vehemently opposed the Britishers
involving India in the Second World
war because
a) Indian were involved without their
consent and any consultation
b) British were fighting for a injust
cause

http://insightsonindia.com

a)
b)
c)
d)

1 only
2 and 3 only
2 only
None of the above

Page 20

http://www.insightsonindia.com

99. Consider the following about


Sustainable Agriculture Initiative
(SAI).
1. It is an inter-governmental
initiative targeted at developing
countries.
2. It receives funding from the World
Bank
3. The initiative gathers and develops
knowledge on sustainable
agriculture
4. It does not include the horticulture
and dairy sector.
Select the correct answer using the codes
below.
a)
b)
c)
d)

1 and 3 only
2 and 4 only
3 only
1, 2 and 4 only

understanding the origin and onset of


monsoon over the Indian
subcontinent, consider the following
statements:
1. The shift of the subtropical easterly
jet stream northwards from over
the plains of India towards the
Tibetan Plateau is said to trigger
the onset of Southwestern
Monsoon over India
2. The heating of Tibetan Plateau is
the reason why the jet stream
located on northern India shifts
towards the Plateau
Which of the above statements is/are correct?
a)
b)
c)
d)

1 Only
2 Only
Both
None

100.
With reference to Jet Stream
theory and its importnace to

http://insightsonindia.com

Page 21

INSIGHTS MOCK TESTS 2015 TEST


24 SOLUTIONS
http://www.insightsonindia.com

1. Solution: d)
About the program here http://envfor.nic.in/division/national-wetlandconservation-programme-nwcp
Financial assistance under NWCP is provided for two components i.e. Management
Action Plan (MAP) and Research Projects. Under the Scheme, 100% assistance is
provided for activities. Conservation and management of wetlands is primarily
vested with the State/UTs, who are in physical possession of the area. After
identification of wetlands under the Scheme, the State/UTs are to submit long-term
comprehensive Management Action Plans (MAPs) for a period of 3-5 years,
preferably 5 years, coinciding with the Plan period.
Under the Scheme, Ministry also sponsor multidisciplinary research projects by
academic/ managerial/ research institutions on various aspects of wetland
conservation to supplement execution of MAP in more realistic manner.

2. Solution: d)
First statement is wrong because if the vale is XII, then its considered as extreme.
Please check the table below:

I. Not felt

II. Weak

Not felt except by a very few under especially favorable


conditions.
Felt only by a few persons at rest, especially on upper floors of
buildings.
Felt quite noticeably by persons indoors, especially on upper

III. Weak

floors of buildings. Many people do not recognize it as an


earthquake. Standing motor cars may rock slightly. Vibrations
similar to the passing of a truck. Duration estimated.
Felt indoors by many, outdoors by few during the day. At night,

IV. Light

some awakened. Dishes, windows, doors disturbed; walls make


cracking sound. Sensation like heavy truck striking building.
Standing motor cars rocked noticeably.

V. Moderate

Felt by nearly everyone; many awakened. Some dishes,


windows broken. Unstable objects overturned. Pendulum clocks

http://www.insightsonindia.com

INSIGHTS MOCK TESTS 2015 TEST


24 SOLUTIONS
http://www.insightsonindia.com

may stop.

VI. Strong

Felt by all, many frightened. Some heavy furniture moved; a few


instances of fallen plaster. Damage slight.
Damage negligible in buildings of good design and construction;

VII. Very Strong

slight to moderate in well-built ordinary structures; considerable


damage in poorly built or badly designed structures; some
chimneys broken.
Damage slight in specially designed structures; considerable

VIII. Severe

damage in ordinary substantial buildings with partial collapse.


Damage great in poorly built structures. Fall of chimneys, factory
stacks, columns, monuments, walls. Heavy furniture overturned.
Damage considerable in specially designed structures; well-

IX. Violent

designed frame structures thrown out of plumb. Damage great in


substantial buildings, with partial collapse. Buildings shifted off
foundations.

X. Extreme

Some well-built wooden structures destroyed; most masonry and


frame structures destroyed with foundations. Rails bent.
Few, if any (masonry), structures remain standing. Bridges

XI. Extreme

destroyed. Broad fissures in ground. Underground pipe lines


completely out of service. Earth slumps and land slips in soft
ground. Rails bent greatly.

XII. Extreme

Damage total. Waves seen on ground surfaces. Lines of sight


and level distorted. Objects thrown upward into the air.

Regarding second statement, if the statement meant intensity, it could have been
considered partially correct (Because intensity refers to the amount of destruction
caused to lives and property). But as it is mentioned magnitude, majority of
aftershocks wil have lesser magnitude than the mainshocks. Hence the statement is
wrong.

http://www.insightsonindia.com

INSIGHTS MOCK TESTS 2015 TEST


24 SOLUTIONS
http://www.insightsonindia.com

3. Solution: b)
http://en.wikipedia.org/wiki/Eichhornia_crassipes#Invasive_species
The roots of Eichhornia crassipes naturally
absorb pollutants,including lead, mercury, and strontium-90, as well as some
organic compounds believed to be carcinogenic, in concentrations 10,000 times that
in the surrounding water. Water hyacinths can be cultivated for waste water
treatment.
Water hyacinth is reported for its efficiency to remove about 6080 % nitrogen (Fox
et al. 2008) and about 69% of potassium from water (Zhou et al. 2007). The roots of
water hyacinth were found to remove particulate matter and nitrogen in a natural
shallow eutrophicated wetland (Billore et al. 1998)

4. Solution: a)
http://www.thehindu.com/features/homes-and-gardens/tackling-detergents-inwater-bodies/article7161992.ece
After the washing of the hair or the clothes or the utensils the detergent is now in the
environment. This water contains phosphates which is a limiting nutrient for many
water bodies. Once the phosphates reach a lake or a stagnant water body it can result
in an explosion of algal, water hyacinth, water weeds and other growth, thus
eutrophying or killing a water body by depleting it of oxygen.

5. Solution: b)
http://www.thehindu.com/news/cities/Coimbatore/growing-threat-to-great-indianhornbills/article7137163.ece
The magnificent birds which were a common sight in the evergreen rain forests of the
Western Ghats are now forced, due to deforestation, to adapt themselves to hollows
in silver oak trees which form part of thick coffee plantations in Singara and Moyar
and some tea plantations in the lower part of the Nilgiri hills.

6. Solution: a)
http://news.nationalgeographic.com/energy/2015/04/150428-audi-ediesel-madefrom-water-air/

http://www.insightsonindia.com

INSIGHTS MOCK TESTS 2015 TEST


24 SOLUTIONS
http://www.insightsonindia.com

7. Solution: c)
To ban fishing within 12 nautical miles is subject of the State Governments, which is
imposed in the interests of the livelihood of fishermen. The Fishing ban of different
periods have been imposed by the coastal State Governments since last many years
or say decades; For example, West-Bengal, Andhra Pradesh, Tamil Nadu and Kerala
impose ban of 47-days (applicable from 1988-89); in Odisha it is 60 days (applied
since 2000); Gujarat, Maharashtra and Goa impose ban of 67 days (effective from
1989-90) and Daman & Diu are imposing ban of 75 days, whereas in Karnataka it is
applied for 57 days.
It was also decided that a uniform ban of 61 days will be applied in the area of the
Indian EEZ (beyond 12 nautical miles). The above proposal was agreed by all the
coastal states/UTs except Kerala.
No foreign fishing vessel is allowed in Indian waters. Besides, in case, any foreign
vessel is found engaged in illegal fishing in the Indian waters, the Indian Coast Guard
is taking strict action against such vessel and it is punished as per the law, which
includes seizure of foreign vessel(s) and its arrest of its crew members.

8. Solution: a)
To protect confidential information pertaining to Defence Sector from Cyber-attack ,
government has taken a number of steps including setting up of Cyber Operation
Centres for threat management and mitigation as part of Framework for Enhancing
Cyber Security of Indian Cyberspace. To protect important and confidential data
from Cyber-attack, the operational networks of the Armed Forces are air gapped
from internet. Further, Defence Services have established Cyber Emergency
Response Teams (CERTs) to prevent and react to cyber-attacks. Safeguards have
been instituted in the form of audits and physical checks. Policies, guidelines and
procedures are laid down and cyber security advisories are issued from time to time.

9. Solution: c)
https://www.oecd-nea.org/law/legal-documents.html

10.

Solution: d)

The Food Safety and Standards Authority of India (FSSAI) had conducted a
nationwide survey on milk adulteration in 2011 through its five regional offices. A
total of 1791 samples were drawn from 33 states and were tested in the Government
laboratories. 68.4% of the samples were found to be non-conforming to the

http://www.insightsonindia.com

INSIGHTS MOCK TESTS 2015 TEST


24 SOLUTIONS
http://www.insightsonindia.com

prescribed standards. Out of these samples, in 46.8% samples, milk found to be substandard in respect of Fat and Solid Not Fat (SNF) contents. Another 44.69% of the
samples (548) in respect of skim milk powder were found to be non-conforming to
the prescribed standards where presence of glucose was detected in 477 samples. A
total of 103 samples (5.75%) were found to be adulterated with detergents.

11. Solution: b)
Please scan through these lists twice thrice. You may not be able to remember all. But
it is important to have an idea, as UPSC keeps asking questions based on these.
http://en.wikipedia.org/wiki/List_of_national_parks_of_India
http://en.wikipedia.org/wiki/Wildlife_sanctuaries_of_India
http://en.wikipedia.org/wiki/Biosphere_reserves_of_India
The last one is the most important of all.

12. Solution: d)
The ASEAN Free Trade Area (AFTA) which was established on 28 January
1992 includes a Common Effective Preferential Tariff (CEPT) to promote the free
flow of goods between member states. When the AFTA agreement was originally
signed, ASEAN had only six members (Brunei, Indonesia, Malaysia, the Philippines,
Singapore, and Thailand). Vietnam joined in 1995, Laos and Burma in 1997, and
Cambodia in 1999. The newcomers have not fully met AFTA's obligations, but they
are officially considered part of the AFTA as they were required to sign the
agreement upon entry into ASEAN, and were given longer time frames in which to
meet AFTA's tariff reduction obligations.
The next steps are to create a:

single market and production base

competitive economic region

region of equitable economic development

region fully integrated into the global economy

Since 2007, ASEAN countries have gradually lowered their import duties with
member nations. The target is zero import duties by 2016.
http://en.wikipedia.org/wiki/Association_of_Southeast_Asian_Nations

http://www.insightsonindia.com

INSIGHTS MOCK TESTS 2015 TEST


24 SOLUTIONS
http://www.insightsonindia.com

13. Solution: d)
Both rivers Bhavani and Amaravati have their basins located in Tamil Nadu and
Kerala.
River Kaveri is not perennial as it doesnt receive rain throughout the year.

14. Solution: d)
Marking a major shift in implementation, the Government is focusing on involving
people living on the banks of the river to attain sustainable results. Drawing from the
lessons learnt from previous implementation, the program also focuses on involving
the States and grassroots level institutions such as Urban Local Bodies and
Panchayati Raj Institutions in implementation. The program would be implemented
by the National Mission for Clean Ganga (NMCG), and its state counterpart
organizations i.e., State Program Management Groups (SPMGs).
In order to ramp up progress, the Centre will now take over 100% funding of various
activities/ projects under this program.
Namami Gange will focus on pollution abatement interventions namely
Interception, diversion & treatment of wastewater flowing through the open drains
through bio-remediation / appropriate in-situ treatment / use of innovative
technologies / sewage treatment plants (STPs) / effluent treatment plant (ETPs);
rehabilitation and augmentation of existing STPs and immediate short term
measures for arresting pollution at exit points on river front to prevent inflow of
sewage etc.

15. Solution: d)
It is technical, but you just need to have a basic idea of it.
http://en.wikipedia.org/wiki/Evolution#Mechanisms
http://en.wikipedia.org/wiki/Inheritance_of_acquired_characteristics
The inheritance of acquired characteristics is a hypothesis that physiological changes
acquired over the life of an organism (such as the enlargement of a muscle through
repeated use) may be transmitted to offspring. It is also commonly referred to as
the theory of adaptation equated with the evolutionary theory
of French naturalist Jean-Baptiste Lamarck known as Lamarckism.
Pangenesis was Charles Darwin's hypothetical mechanism for heredity. The
pangenesis theory, similar to Hippocrates's views on the topic, imply that the whole
of parental organisms participate in heredity.

http://www.insightsonindia.com

INSIGHTS MOCK TESTS 2015 TEST


24 SOLUTIONS
http://www.insightsonindia.com

16. Solution: d)
The Minister of State (I/C) for Petroleum & Natural Gas Shri Dharmendra Pradhan
informed the Rajya Sabha in a written reply that various agencies have estimated the
shale gas and oil resource potential in selected sedimentary basins/ sub basins in
India as per the details below:
i) 300 to 2100 tcf of shale gas in the country (as available in public domain);
ii) 290 tcf of shale gas in four basins namely, Cambay, KG, Cauvery and Damodar;
iii) 6.1 tcf of technically recoverable shale gas in 3 basins, namely, Cambay, KG &
Cauvery.
iv) 584 tcf of sale gas and 87 Billion barrel of shale oil in 4 basins namely, Cambay,
KG, Cauvery and Damodar;
v) 187.5 tcf of shale gas in 5 basins namely, Cambay, KG, Cauvery, Ganga & Assam
and Assam - Arakan;
vi) 45 tcf of Shale Gas in Gondwana basin (including Damodar).

17. Solution: a)
The Bill provides for among other things:i. Establishment of the National CAF and the State CAFs to credit amounts collected
by State Governments and Union Territory Administrations to compensate loss of
forest land diverted for non-forest purpose.
ii. Constitution of a National Authority to manage and utilise amounts credited to the
National CAF.
iii. Constitution of a State Authority in each State and Union Territory to manage and
utilise the amounts credited to the State CAFs.
iv. Establishment of a Monitoring Group to assist the National Authority in
monitoring and evaluation of activities undertaken from amounts released from the
National CAF and State CAFs.
This would mitigate impact of diversion of such forest land.

http://www.insightsonindia.com

INSIGHTS MOCK TESTS 2015 TEST


24 SOLUTIONS
http://www.insightsonindia.com

18.Solution: d)
Many animals have evolved to exhibit some form of camouflage, which is an
adaptation that allows animals to blend in with certain aspects of their environment.
Camouflage increases an organism's chance of survival by hiding it
from predators. This gives the animal a higher probability of being able to
successfully reproduce and carry on the species. Camouflage is also used by some
predators as a tool for hunting.
Examples and explanations have been given here.
http://study.com/academy/lesson/what-is-camouflage-in-animals-definitionexamples-quiz.html
19. Solution: d)
http://www.parliamentofindia.nic.in/ls/intro/p21.htm
All these committees deal with the scrutiny of the government expenditure. If a
Minister is a part of such committees, it will create conflict of interest situation.
Hence, it is avoided. Moreover, the minister might influence the proceedings of the
meetings.
20.

Solution: c)

http://www.nature.com/scitable/knowledge/library/the-nitrogen-cycle-processesplayers-and-human-15644632

http://www.insightsonindia.com

INSIGHTS MOCK TESTS 2015 TEST


24 SOLUTIONS
http://www.insightsonindia.com

21. Solution: d)
Scavengers are a part of the food web, a description of which organisms eat which
other organisms in the wild. Organisms in the food web are grouped into trophic, or
nutritional, levels. There are three trophic levels. Autotrophs, organisms that
produce their own food, are the first trophic level. These include plants and algae.
Herbivores, or organisms that consume plants and other autotrophs, are the second
trophic level. Scavengers, other carnivores, and omnivores, organisms that consume
both plants and animals are the third trophic level.
Examples can be found here.
http://www.factmonster.com/ipka/A0768543.html

22.

Solution: d)

Dissolved oxygen enters water through the air or as a plant byproduct. From the air,
oxygen can slowly diffuse across the waters surface from the surrounding
atmosphere, or be mixed in quickly through aeration, whether natural or man-made.
The aeration of water can be caused by wind (creating waves), rapids, waterfalls,
ground water discharge or other forms of running water. Man-made causes of
aeration vary from an aquarium air pump to a hand-turned waterwheel to a large
dam.
http://www.fondriest.com/environmental-measurements/parameters/waterquality/dissolved-oxygen/#2

23.

Solution: d)

Emigration is the act of leaving one's native country with the intent to settle
elsewhere. Conversely, immigration describes the movement of persons into one
country from another. Both are acts of migration across national boundaries.

24.

Solution: b)

Over 2700 species of flowering plants, 160 species of fern and fern allies, countless
types of flowerless plants, mosses, fungi, algae, land lichens are found in the sholas of
the Nilgiris. No other Hill station has so many exotic species.
The Nilgiri tahr animal can be found in the hills.
Much of the Nilgiris natural Montane grasslands and shrublands interspersed
with sholas has been much disturbed or destroyed by extensive tea plantations, easy

http://www.insightsonindia.com

INSIGHTS MOCK TESTS 2015 TEST


24 SOLUTIONS
http://www.insightsonindia.com

motor vehicle access and extensive commercial planting and harvesting of nonnative eucalyptus and wattle plantations Acacia dealbata, Acacia mearnsii and cattle
grazing. In addition there is one large, and several smaller hydroelectric impoundments in the area. Scotch broom has become an ecologically
damaging invasive species.

25.

Solution: a)

Unlike coal-fired or oil-fired generation, nuclear power generation does not directly
produce any sulfur dioxide, nitrogen oxides, or mercury (pollution from fossil fuels is
blamed for 24,000 early deaths each year in the U.S. alone.
Large amounts of radioactive contamination were spread across Europe due to the
Chernobyl disaster, and cesium and strontium contaminated many agricultural
products, livestock and soil.
The greenhouse gas emissions from nuclear fission power are much smaller than
those associated with coal, oil and gas, and the routine health risks are much smaller
than those associated with coal.

26.

Solution: c)

This has been explained here properly. Please go through


http://www.charteredclub.com/service-tax-on-food-served-in-restaurant/

27.

Solution: c)

An angel investor or angel (also known as a business angel or informal


investor or angel funder) is an affluent individual who provides capital for a business
start-up, usually in exchange for convertible debt or ownership equity. A small but
increasing number of angel investors organize themselves into angel groups or angel
networks to share research and pool their investment capital, as well as to provide
advice to their portfolio companies.
Angel investors are usually found among an entrepreneur's family and friends. The
capital they provide can be a one-time injection of seed money or ongoing support to
carry the company through difficult times.
Angel investors give more favorable terms than other lenders, as they are usually
investing in the person rather than the viability of the business. They are focused on
helping the business succeed, rather than reaping a huge profit from their

http://www.insightsonindia.com

INSIGHTS MOCK TESTS 2015 TEST


24 SOLUTIONS
http://www.insightsonindia.com

investment. Angel investors are essentially the exact opposite of a venture capitalist.

28.

Solution: d)

This is a very comprehensive document released by the government. Please go


through the first few pages that discuss the rationale for Change in institutions in
india.
http://pib.nic.in/newsite/pdfdisplay.aspx?docid=404

29.

Solution: c)

You need to understand that FDI and FII are investments, not a source of income.
SO they cant be counted under NIA.
Remittances are earned by Indian nationals working abroad and sent to their family.
Same is done by foreign nationals working in India. So, the net is taken into account.
If you want to read in detail, it is explained here very comprehensively.
http://www.yourarticlelibrary.com/macro-economics/nfia-net-factor-income-fromabroad-nfia-significance-and-components/30266/

30.

Solution: d)

The current account is bridged by FDI and FII. It does not include them. Instead it
includes private remittances; trade in invisibles, and the trade deficit. Moreover,
short-term investments made by the government in the nations abroad also get
counted in the current account.
Loans and grants all come under the capital account.

31. Solution: a)
Liquidity Adjustment Facility (LAF) is the primary instrument of Reserve Bank of
India for modulating liquidity and transmitting interest rate signals to the market. It
refers to the difference between the two key rates viz. repo rate and reverse repo rate.
Informally, Liquidity Adjustment Facility is also known as Liquidity Corridor.
While repo injects liquidity into the system, the Reverse repo absorbs the liquidity
from the system. RBI only announces Repo Rate. The Reverse Repo Rate is linked to
Repo Rate and is 100 basis points (1%) below repo rate.

http://www.insightsonindia.com

INSIGHTS MOCK TESTS 2015 TEST


24 SOLUTIONS
http://www.insightsonindia.com

32.

Solution: c)

http://jandhanyojana.net/how-to-become-a-bank-mitra/

33.

Solution: a)

Reverse repo rate is the rate at which the central bank of a country (Reserve Bank of
India in case of India) borrows money from commercial banks within the country. It
is a monetary policy instrument which can be used to control the money supply in
the country.
An increase in the reverse repo rate will decrease the money supply and vice-versa,
other things remaining constant. An increase in reverse repo rate means that
commercial banks will get more incentives to park their funds with the RBI, thereby
decreasing the supply of money in the market.
So, the reverse repo takes away funds from the bank (cash is not with them).

34.

Solution: a)

The Northern Iraq offensive began on 5 June 2014, when the Islamic State of Iraq
and the Levant (ISIL; sometimes referred to as the Islamic State (IS)) and aligned
forces began a major offensive in northern Iraq against the Iraqi government,
following earlier clashes, which began in December 2013.
ISIL and aligned forces captured several cities and other territory, beginning with an
attack on Samarra on 5 June followed by the seizure of Mosul on 10 June
and Tikrit on 11 June. As Iraqi government forces fled south on 13 June,
the Kurdish forces took control of the oil hub of Kirkuk, part of the disputed
territories of Northern Iraq.

35.

Solution: b)

The International Union for Conservation of Nature (IUCN) is an international


organization working in the field of nature conservation and sustainable use
of natural resources. It is involved in data gathering and analysis, research, field
projects, advocacy, lobbying and education. IUCN's mission is to "influence,
encourage and assist societies throughout the world to conserve nature and to ensure
that any use of natural resources is equitable and ecologically sustainable."

http://www.insightsonindia.com

INSIGHTS MOCK TESTS 2015 TEST


24 SOLUTIONS
http://www.insightsonindia.com

Over the past decades, IUCN has widened its focus beyond conservation ecology and
now incorporates issues related to gender equality, poverty
alleviation and sustainable business in its projects. Unlike other international NGOs,
IUCN does not itself aim to mobilize the public in support of nature conservation. It
tries to influence the actions of governments, business and other stakeholders by
providing information and advice, and through lobbying and partnerships. The
organization is best known to the wider public for compiling and publishing the
IUCN Red List of Threatened Species, which assesses the conservation status of
species worldwide.
IUCN has a membership of over 1200 governmental and non-governmental
organizations.

36.

Solution: a)

http://www.nytimes.com/interactive/2015/03/26/world/middleeast/yemen-crisisexplained.html?_r=0
Its explained comprehensively in this article.

37.

Solution: c)

President Pranab Mukherjee inaugurated Namaste Russia, a festival of Indian


culture in Moscow during the last leg of his tour in Russia.
Namaste Russia will be six month long festival i.e. form May to November 2015. The
festival seeks to enthrall the Russian audience with cultural events from India.
The festival is being hosted under the patronage of Indian Council for Cultural
Relations (ICCR) and Ministry of Culture of the Russian Federation. As part of the
inauguration of this festival, Kathak dance performance was given by a team of
dancers choreographed by Padma Bhushan Kumudini Lakhia.
This festival seeks to enhance and boost cultural bonds that build solid bridges of
friendship and understanding between peoples of both countries.
The festival will be toured in various regions in Russia that will help people living in
different regions of Russia to deepen their engagement with contemporary and
classical Indian Culture. As part of this festival, an exhibition of Indian painting and
sculpture will be also organized.

http://www.insightsonindia.com

INSIGHTS MOCK TESTS 2015 TEST


24 SOLUTIONS
http://www.insightsonindia.com

38.

Solution: c)

WHO can declare a country Ebola free, if no new cases are reported over two
incubation periods of virus i.e. 21 days.
In case of Liberia, no new cases of Ebola were reported in past 42 days i.e. twice the
number of days the virus requires to incubate. The last confirmed death in Liberia
was reported on 27 March 2015.
The current outbreak is the deadliest in history. It was initially centred on Guineas
remote south-eastern region of Nzerekore in early 2014 and later had spread to
Liberia and Sierra Leone. Liberia was worst hit among the western African nations in
this outbreak.

39.

Solution: a)

Lithium chloride is mainly used for the production of lithium metal by electrolysis of
a LiCl/KCl melt at 450 C (842 F). LiCl is also used as a
brazing flux for aluminium in automobile parts. It is used as a desiccant for drying
air streams.[5] In more specialized applications, lithium chloride finds some use
in organic synthesis, e.g. as an additive in the Stille reaction. Also, in biochemical
applications, it can be used to precipitate RNA from cellular extracts.[8]
Lithium chloride is also used as a flame colorant to produce dark red flames.
Lithium chloride is used as a relative humidity standard in the calibration of
hygrometers.
Lithium salts affect the central nervous system in a variety of ways. While
the citrate, carbonate, and orotate salts are currently used to treat bipolar disorder,
other lithium salts including the chloride were used in the past. For a short time in
the 1940s lithium chloride was manufactured as a salt substitute, but this was
prohibited after the toxic effects of the compound were recognized.
It is Potassium chloride which is widely used as salt substitute but with certain
precautions (for ex, People with kidney failure, heart failure or diabetes should not
use this salt substitute without medical advice).

40.

Solution: c)

By signing of MoU Indian and Iranian commercial entities will be able to commence
negotiations for finalization of a commercial contract.

http://www.insightsonindia.com

INSIGHTS MOCK TESTS 2015 TEST


24 SOLUTIONS
http://www.insightsonindia.com

Under the contract, Indian firms will lease two existing berths at the Chabahar Port
and operationalize them as container and multi-purpose cargo terminals
The port is strategically located in Sistan-Balochistan Province on Irans
southeastern coast. It will act as strategic utility for India.
It will also help India to save transportation costs as it will be used to ship crude oil
and urea.
India will get sea-land access route to Afghanistan bypassing Pakistan.
Thus it will open opportunities for Indian companies to explore Afghanistans
mineral wealth, believed to be worth $1-3 trillion, for mutual benefit.
The availability of functional container and cargo terminal at the port will provide
Afghanistans garland road network system alternate access to a sea port.

41. Solution: b)
The Chakma also known as the Daingnet people , is an ethnic group concentrated in
the Chittagong Hill Tracts of Bangladesh. Today, the geographic distribution of
Chakmas is spread across Bangladesh and parts of northeastern India (mainly in
Mizoram), western Burma, China and diaspora communities in North
America, Europe, South Korea, Japan and Australia.
Within the Chittagong Hill Tracts, the Chakma are the largest ethnic group and make
up half of the region's population. The Chakma are divided into 46 clans or Gozas.
They have their own language, customs and culture, and profess Theravada
Buddhism. The Chakma Royal Family is one of the major Buddhist royal houses of
the South Asia.

42.

Solution: d)

The energy that we get from biofuels originally came from the sun. This solar energy
was captured through photosynthesis by the plants used as feedstocks (raw
materials) for biofuel production, and stored in the plants' cells.
Many different plant materials can be used for biofuel:

Sugar crops (such as sugar cane or sugar beet), or starch (like corn or maize)
can be fermented to produce ethanol, a liquid fuel commonly used for
transportation.

http://www.insightsonindia.com

INSIGHTS MOCK TESTS 2015 TEST


24 SOLUTIONS
http://www.insightsonindia.com

Natural oils from plants like oil palm, soybean, or algae can be burned directly
in a diesel engine or a furnace, or blended with petroleum, to produce fuels
such as biodiesel.

Wood and its byproducts can be converted into liquid biofuels, such as
methanol or ethanol, or into woodgas.

Wood can also be burned as solid fuel, like the familiar firewood. Chipped
waste biomass, such as the tops of trees discarded by logging operations, can
be burned in specially designed furnaces.

Researchers are working to improve biofuel production processes. Before bioenergy


can make a larger contribution to the energy economy, feedstocks, agricultural
practices, and technologies that are efficient in their use of land, water and fossil fuel
must be developed.

43.

Solution: d)

5th and 6th schedule directly concern themselves with tribal governance as all the
special provisions made under the constitution fall under it. For e.g. constituting a
tribal advisory council; giving tribals control over land and forests etc.
7th schedule is concerned with the division of legislative power between Centre and
States. It concerns with forests; welfare; mining and minerals which have a direct
bearing on the tribal community in India.
10th schedule is about anti-defection.

44.

Solution: b)

Cumulus

Detached, generally dense clouds and with sharp outlines that develop vertically in

http://www.insightsonindia.com

INSIGHTS MOCK TESTS 2015 TEST


24 SOLUTIONS
http://www.insightsonindia.com

the form of rising mounds, domes or towers with bulging upper parts often
resembling a cauliflower.
The sunlit parts of these clouds are mostly brilliant white while their bases are
relatively dark and horizontal.
Over land cumulus develops on days of clear skies, and is due diurnal convection; it
appears in the morning, grows, and then more or less dissolves again toward
evening.
For more, Please refer this link:
http://www.srh.noaa.gov/srh/jetstream/clouds/cloudwise/types.html

45.

Solution: d)

The protected area (PA) network in India has helped to conserve a significant part of
the country's biodiversity.
Most PAs in India have a core zone with national park status and a peripheral buffer
zone, which can be either a wildlife sanctuary or a reserve forest. Resource use has
been restricted to the buffer zones, where it has been regulated, while core areas are
completely closed. A 1991 amendment to the Wildlife Protection Act specifies that, in
wildlife sanctuaries, the chief wildlife warden must certify that any manipulation
does not harm wildlife, and that the manipulation be approved by the state
government.
State governments have constituted national parks with legislations such as
Kaziranga and Manas national park. This is because forests and their conservation
comes in the concurrent list.

46.

Solution: d)

Read the causes in detail (you can overlook technical details), because this is the
most comprehensive account of climate change you can get for UPSC.
http://en.wikipedia.org/wiki/Climate_change#Causes
To understand how plate tectonics can force climate change, consider this example:
The position of the continents determines the geometry of the oceans and therefore
influences patterns of ocean circulation. The locations of the seas are important in
controlling the transfer of heat and moisture across the globe, and therefore, in
determining global climate. A recent example of tectonic control on ocean circulation
is the formation of the Isthmus of Panama about 5 million years ago, which shut off

http://www.insightsonindia.com

INSIGHTS MOCK TESTS 2015 TEST


24 SOLUTIONS
http://www.insightsonindia.com

direct mixing between the Atlantic and Pacific Oceans. This strongly affected
the ocean dynamics of what is now the Gulf Stream and may have led to Northern
Hemisphere ice cover.

47.

Solution: c)

Protection of national flag and honour is a part of Fundamental rights for which laws
have been enacted.
Labour rights have been covered by mentioning equal wage for equal work for men
and women; ensuring a living wage for labourers; their participation in the
management of industries etc.

48.

Solution: a)

To address the longevity risks among the workers in unorganized sector and to
encourage the workers in unorganized sector to voluntarily save for their retirement

The APY is focussed on all citizens in the unorganized sector. All bank account
holders may join APY. APY is applicable to all citizen of India aged between
18-40 years.
The scheme is administered by the Pension Fund Regulatory and
Development Authority (PFRDA) through NPS architecture.
Under the APY, there is guaranteed minimum monthly pension for the
subscribers ranging between Rs. 1000 and Rs. 5000 per month. GoI will also
co-contribute 50% of the subscribers contribution or Rs. 1000 per annum,
whichever is lower. Government co-contribution is available for those who are
not covered by any Statutory Social Security Schemes and is not income tax
payer. The pension is guaranteed by GoI.

49.

Solution: d)

Highlights of The Pradhan Mantri Jeevan Jyoti Bima Yojana:

Eligibility: Available to people in the age group of 18 to 50 and


having a bank account. People who join the scheme before completing 50
years can, however, continue to have the risk of life cover up to the age of 55
years subject to payment of premium.

Premium: Rs 330 per annum. It will be auto-debited in one instalment.

http://www.insightsonindia.com

INSIGHTS MOCK TESTS 2015 TEST


24 SOLUTIONS
http://www.insightsonindia.com

Payment Mode: The payment of premium will be directly auto-debited by


the bank from the subscribers account.

Risk Coverage: Rs. 2 Lakh in case of death for any reason.

Terms of Risk Coverage: A person has to opt for the scheme every
year. He can also prefer to give a long-term option of continuing, in which
case his account will be auto-debited every year by the bank.

Who will implement this Scheme?: The scheme will be offered by Life
Insurance Corporation and all other life insurers who are willing to join the
scheme and tie-up with banks for this purpose.

Accidental insurance is covered under PM Suraksha Bima Yojana.

50.

Solution: b)

NITI Ayog is not concerned with economic planning, it is concerned with economic
policy. That is the major difference between this and Planning Commission.
Parliament used to approve the five year plans and even now the annual budget (an
economic plan) is approved by the Parliament.
Zila panchayats consolidate the plans received by the lower tiers and forward it to the
District Planning Committee.
51. Solution: d)
The Prime Minister's Office (PMO) consists of the immediate staff of the Prime
Minister of India, as well as multiple levels of support staff reporting to the Prime
Minister.

The PMO provides secretarial assistance to the Prime Minister - virtual 'think
tank' of the PM.

It is headed by the Principal Secretary to the Prime Minister.

The PMO includes the anti-corruption unit and the public wing dealing with
grievances.

The office houses the Prime Minister and few selected officers of Indian Civil
Service who work with him to manage and coordinate government and his
office.

The Prime Minister through his office coordinates with all ministers in the
central union cabinet, minister of independent charges and governors and
ministers of state government.

http://www.insightsonindia.com

INSIGHTS MOCK TESTS 2015 TEST


24 SOLUTIONS
http://www.insightsonindia.com

Monitors the actions taken by the PM in earlier meetings.

It generally handles all activities not assigned to any particular department.

Its importance increases wrt to cabinet too as PM is the coordinator of the


Cabinet.

52.

Solution: a)

A welfare state is a concept of government in which the state plays a key role in the
protection and promotion of the economic and social well-being of its citizens. It is
based on the principles of equality of opportunity, equitable distribution of wealth,
and public responsibility for those unable to avail themselves of the minimal
provisions for a good life. The general term may cover a variety of forms of economic
and social organization. The sociologist T.H. Marshall identified the modern welfare
state as a distinctive combination of democracy, welfare, and capitalism (not fully).
Even though Kautilyas and Ashokas state were not democratic, they were welfare
states.

53.

Solution: d)

Constitution of India prescribes that Minister will be appointed on the advice of PM


and CM respectively in Centre and States.
The 91st amendment restricts the Council of Ministers to 15% of the total strength of
Lok Sabha.
Other conventions like Majority party leader; coalition leader etc. are also followed
when appointing Ministers.

54.

Solution: d)

It has been explained beautifully in the later half of the article


http://zeenews.india.com/business/news/finance/rupees-free-fall-how-it-willimpact-you_82322.html
FDI will be affected because foreign investors are sensitive to a) the value of the
currency that determines the actual value of their investment; and b) the confidence
in the Indian economy because of a depreciating rupee. In the long run however,
these investors may be more interested in investing due to the lower value of rupee in
the exchange market.

http://www.insightsonindia.com

INSIGHTS MOCK TESTS 2015 TEST


24 SOLUTIONS
http://www.insightsonindia.com

55.

Solution: a)

If an individual ministry has taken the decision, which has not been considered by
the cabinet, the president can send the file for re-consideration.
Also, the president can not just ask for the files, but also ask details about the
decisions that are to be taken in the council of ministers.
However, he cannot suspend it, if it enjoys confidence in the Lok Sabha.

56.

Solution: b)

The primary object of an adjournment motion is to draw the attention of the House
to a recent matter of urgent public importance having serious consequences and in
regard to which a motion or a resolution with proper notice will be too late. The
matter proposed to be raised should be of such a character that something very grave
which affects the whole country and its security has happened and the House is
required to pay its attention immediately by interrupting the normal business of the
House.
The adjournment motion is thus an extraordinary procedure which, if admitted,
leads to setting aside the normal business of the House for discussing a
definite matter of urgent public importance.

57.Solution: d)
According to Ayurveda, water stored in a copper vessel has the ability to balance all
the three doshas in your body, (vata, kapha and pitta) and it does so by positively
charging the water. The water stored in a copper vessel is known as tamara jal and
is supposed to be consumed after storing the water in a copper vessel for at least
eight hours.
Copper also helps cleanse and detox your stomach, regulate the working of your liver
and kidneys, and proper elimination of waste and ensures the absorption of nutrients
from food.
58.

Solution: c)

Fringing reef is distinguished from the other two main types (barrier reefs and atolls)
in that it has either an entirely shallow backreef zone (lagoon) or none at all. If a
fringing reef grows directly from the shoreline (see photo, right) the reef flat extends
right to the beach and there is no backreef. In other cases (e.g., most of the

http://www.insightsonindia.com

INSIGHTS MOCK TESTS 2015 TEST


24 SOLUTIONS
http://www.insightsonindia.com

Bahamas), fringing reefs may grow hundreds of yards from shore and contain
extensive backreef areas with numerous seagrass meadows and patch reefs.
This type of coral reef is the most common type of reef found in
the Caribbeanand Red Sea. Darwin believed that fringing reefs are the first kind of
reefs to form around a landmass in a long-term reef growth process.
59.

Solution: b)

Respiration( taking in oxygen) is carried out both at night and in the day. But, at
night no photosynthesis takes place, so the plant is only taking in oxygen, not CO2.
In the day, photosynthesis also takes place, which counters the effect of respiration
on gases around the plant. So, during the night, concentration of CO2 is more, and
oxygen is less around the plant.
During the day, more oxygen and less CO2 is present around the plant as compared
to at night.
60.

Solution: a)

Aadhar is a biometrics-based digital identity assigned for a person's lifetime,


verifiable online instantly in the public domain, at any time, from anywhere, in a
paperless way. It is designed to enable government agencies to deliver a retail public
service securely based on biometric data (fingerprint, iris scan and face photo), along
with demographic data (name, age, gender, address, parent/spouse name, mobile
phone number) of a person. The data is transmitted in encrypted form over the
internet for authentication, aiming to free it from the limitations of physical presence
of a person at a given place.
61. Solution: d)
Plant biotechnology continues to create new opportunities in agriculture,
environmental biology, chemical production, and medicine. The development
of genetically modified crops with valued traits like herbicide and pest
resistance, drought and salt tolerance, or increased yield requires different
combinations of expression elements to achieve optimum effects.
Applications like engineering plants for phytoremediation or biofuel research often
involve complex design and assembly of DNA elements that require advanced
cloning tools.
62.

Solution: a)

Read the intro only that too generally.


http://en.wikipedia.org/wiki/Pioneer_10
http://en.wikipedia.org/wiki/Voyager_1
http://en.wikipedia.org/wiki/New_Horizons

http://www.insightsonindia.com

INSIGHTS MOCK TESTS 2015 TEST


24 SOLUTIONS
http://www.insightsonindia.com

63.

Solution: d)

Article 143 read verbatim: Power of President to consult Supreme Court ( 1 ) If at


any time it appears to the President that a question of law or fact has arisen, or is
likely to arise, which is of such a nature and of such public importance that it is
expedient to obtain the opinion of the Supreme Court upon it, he may refer the
question to that Court for consideration and the Court may, after such hearing as it
thinks fit, report to the President its opinion thereon
(2) The President may, notwithstanding anything in the proviso to Article 131, refer
a dispute of the kind mentioned in the said proviso to the Supreme Court for opinion
and the Supreme Court shall, after such hearing as it thinks fit, report to the
President its opinion thereon.
64.

Solution: d)

Although it is a technical topic, UPSC keeps asking questions on this. Go through this
link and get a general idea of how these crops are made, and why they are made.
http://cls.casa.colostate.edu/transgeniccrops/what.html
http://cls.casa.colostate.edu/transgeniccrops/how.html
65.

Solution: b)

http://www.insightsonindia.com

INSIGHTS MOCK TESTS 2015 TEST


24 SOLUTIONS
http://www.insightsonindia.com

66.

Solution: a)

For changing powers, special majority is required; for number of judges it is not.
The original Constitution of 1950 envisaged a Supreme Court with a Chief Justice
and 7 puisne Judges - leaving it to Parliament to increase this number. In the early
years, all the Judges of the Supreme Court sat together to hear the cases presented
before them. As the work of the Court increased and arrears of cases began to
cumulate, Parliament increased the number of Judges from 8 in 1950 to 11 in 1956,
14 in 1960, 18 in 1978 and 26 in 1986. As the number of the Judges has increased,
they sit in smaller Benches of two and three - coming together in larger Benches of 5
and more only when required to do so or to settle a difference of opinion or
controversy.

67.

Solution: d)

This link contains the places famous for all types of handicraft in India. Just scan
through it once.
http://www.mapsofindia.com/maps/crafts/

68.

Solution: d)

The Convention on the Conservation of Antarctic Marine Living Resources,


also Commission on the Conservation of Antarctic Marine Living Resources,
and CCAMLR, is part of the Antarctic Treaty System. The Convention was opened for
signature on 1 August 1980 and entered into force on 7 April 1982 by the
Commission for the Conservation of Antarctic Marine Living Resources,
headquartered in Tasmania, Australia. The goal is to preserve marine life and
environmental integrity in and near Antarctica.
It was established in large part to concerns that an increase in krill catches in
the Southern Ocean could have a serious impact on populations of other marine life
which are dependent upon krill for food.
The Cartagena Protocol on Biosafety to the Convention on Biological Diversity is an
international agreement on biosafety, as a supplement to the Convention on
Biological Diversity. The Biosafety Protocol seeks to protect biological diversity from
the potential risks posed by genetically modified organisms resulting from modern
biotechnology.

http://www.insightsonindia.com

INSIGHTS MOCK TESTS 2015 TEST


24 SOLUTIONS
http://www.insightsonindia.com

69.

Solution: c)

Consider this food chain. With this you can draw common links to other type of
marine food chains.

primary producer/autotrophsorganisms, like plants, that produce food.


Examples: phytoplankton, algae

primary consumer/heterotrophan animal that eats primary producers.


Examples: mussels, oysters, krill, copepods, shrimp

secondary consumer/heterotrophan animal that eats primary


consumers. Examples: blue claw crab, lobster, seastar, humpback whale,
silverside

tertiary consumer/heterotrophan animal that eats secondary


consumers. Examples: shark, dolphin

apex predator/heterotrophan animal at the top of the food chain with


no predators. Examples: shark, dolphin

decomposer/detritivoresorganisms that break down dead plant and


animal material and wastes and release it again as energy and nutrients in the
ecosystem. Examples: bacteria, fungi, worms, crabs

70.

Solution: c)

The Gregorian calendar is based upon the solar system and hence it has 365 days.
The Hindu calendar is based upon the lunar and solar systems. With the new moon
appearing after every 27.1/2 days, the 12 lunar months take 354 days. The difference
of 11 days is adjusted every three years when an additional month called adhik-mas is
added. The difference between the two calendars explains why the dates of Hindu
festivals vary every year. The names of the 12 months are Chaitra, Vaishakh, Jyeshth,
Ashadh, Shravan, Bhadrapad (Bhadon), Ashvin, Kartik, Margashirsha (Aahan),
Poush, Magh and Phagun. Hindus sometimes use abbreviated forms for some
months.

71. Solution: b)
Second statement is wrong. It is West coast which receives abnormal rainfall during
El Nino.
It is because of low pressure in eastern Pacific (i.e. western coast of South and North
America) the region receives heavy rainfall. Whereas, its western counterpart,

http://www.insightsonindia.com

INSIGHTS MOCK TESTS 2015 TEST


24 SOLUTIONS
http://www.insightsonindia.com

including India receive low rainfall thanks to high pressure created by El Nion in the
region.

72.

Solution: d)

The National Seeds Policy provides for intellectual property protection to new
varieties and set up institutes for the planned development of the sector. The policy
has nine major thrust areas including the production of quality seeds by the private
sector, establishing state seed testing laboratories, access to breeder seed, etc
The main features of the National Seeds Policy, 2002

Development of new and improved varieties of plants


Timely availability of quality seeds
Compulsory registration of seeds
Creation of infrastructure facilities
Quality assurance, promotion of seed industry,
Abolition of licensing for seed dealers,
Facility for import of best quality seeds,
Encouragement to export of seeds
Creation of Seed Banks and National Seed Grid.

These initiatives will encourage investment in research and development, thereby


ensuring availability of high yielding varieties of seeds which will lead to higher
production and improving the economic condition of the farmers in the country.

73.

Solution: a)

The meaning of Lai Haraoba is the festival of gods and goddesess. It is performed for
the Umang-Lai (forest deity). Ougri Hangen, song of creation and Heijing Hirao a
ritualistic song is sung on the last day of Lai Haraoba festival.
Mizo are traditionally known as a singing tribe. The regional folk songs of Mizoram
constitute the richest heritage of Mizos. Saikuti, a poetess of Mizoram composed
songs in praise of warriors, brave hunters, young men aspiring to be great warriors
and hunters etc.
As per Mizo custom during the Chapchar Kut festival not only singing, dance should
also continue throughout the festival. Special occasion for singing and dancing is
called chai and songs are known as chai hia (chai songs).

http://www.insightsonindia.com

INSIGHTS MOCK TESTS 2015 TEST


24 SOLUTIONS
http://www.insightsonindia.com

74.

Solution: d)

75.Solution: d)
It has been explained beautifully in this article.
http://www.unesco.org/mab/doc/ekocd/chapter11.html

76.

Solution: d)

UPSC keeps asking questions based on Environment related organizations in India.


Here is the complete list with links to relevant Wikipedia pages.
Subordinate and statutory bodies

Andaman & Nicobar Islands Forest and Plantation Development


Corporation (Public Sector Undertaking)

Animal Welfare Board of India

http://www.insightsonindia.com

INSIGHTS MOCK TESTS 2015 TEST


24 SOLUTIONS
http://www.insightsonindia.com

Botanical Survey of India (BSI), Kolkata

Central Pollution Control Board

Central Zoo Authority, New Delhi

Directorate of Forest Education (DFE), Dehradun

Forest Survey of India (FSI), Dehradun

Indira Gandhi National Forest Academy (IGNFA), Dehradun

National Afforestation and Eco-Development Board

National Biodiversity Authority, Chennai

National Ganga River Basin Authority

National Institute of Animal Welfare

National Museum of Natural History (NMNH), New Delhi

National Tiger Conservation Authority

National Zoological Park (NZP), New Delhi

Zoological Survey of India (ZSI), Kolkata

Autonomous bodies

Govind Ballabh Pant Institute of Himalayan Environment and Development,


Almora

Indian Council of Forestry Research and Education (ICFRE), Dehradun

Arid Forest Research Institute

Forest Research Institute (India)

Himalayan Forest Research Institute and many others

Indian Institute of Forest Management

Indian Plywood Industries Training and Research Institute

Wildlife Institute of India (WII)

77. Solution: c)
The Global Competitiveness Report 2014-2015 assesses the competitiveness
landscape of 144 economies, providing insight into the drivers of their productivity

http://www.insightsonindia.com

INSIGHTS MOCK TESTS 2015 TEST


24 SOLUTIONS
http://www.insightsonindia.com

and prosperity. The report remains the most comprehensive assessment of national
competitiveness worldwide, providing a platform for dialogue between government,
business and civil society about the actions required to improve economic
prosperity. Competitiveness is defined as the set of institutions, policies and factors
that determine the level of productivity of a country. The level of productivity, in
turn, sets the level of prosperity that can be earned by an economy.
The different aspects of competitiveness are captured in 12 pillars, which compose
the Global Competitiveness Index that include terms like innovation; skills etc.

78.

Solution: a)

Skim through the introduction part.


http://en.wikipedia.org/wiki/Free-radical_theory_of_aging
The chain reaction caused by free radicals can lead to cross-linking of atomic
structures. In cases where the free radical-induced chain reaction involves base
pair molecules in a strand of DNA, the DNA can become cross-linked. Such cross
linking and damage to cells can even lead to cancer.

79.

Solution: a)

Vitamin B6 - Part of an enzyme needed for making new cells; important to nerve
function
Vitamin C - Antioxidant; part of an enzyme needed for protein metabolism;
important for immune system health; aids in iron absorption
Vitamin E- Antioxidant; protects cell walls
You can read more here
http://www.emedicinehealth.com/vitamins_their_functions_and_sourceshealth/article_em.htm
80.

Solution: d)

Some types of nanoparticles are expected to be benign and used for making paints
and sunscreen lotion etc. However, there are also dangerous nanosized particles and
chemicals that are known to accumulate in the food chain and have been known for
many years. The list and processes are mentioned in the link below.
http://en.wikibooks.org/wiki/Nanotechnology/Health_effects_of_nanoparticles#N
anotoxicology:_Health_effects_of_nanotechnology

http://www.insightsonindia.com

INSIGHTS MOCK TESTS 2015 TEST


24 SOLUTIONS
http://www.insightsonindia.com

81.Solution: c)
Non-physical savings essentially mean financial savings by households; corporate
and the government. Major portion comes from the households and private sector.
This saving is mostly in the form of bank deposits, securities, insurance or
investment.
If inflation levels are high, the returns on both deposit and investment will be low.
This will discourage saving.
However, if all other options happen, they will push the volume and base of savings
up and will help in overall growth of the Indian economy.

82.

Solution: d)

Samkhya is strongly dualist. Smkhya philosophy regards the universe as consisting


of two realities; Purua (consciousness) and prakriti (matter). Jiva (a living being) is
that state in which purua is bonded to prakriti in some form. This fusion, state the
Samkhya scholars, led to the emergence of buddhi (spiritual awareness)
andahankara (individualized ego consciousness, I-maker). The universe is
described by this school as one created by Purusa-Prakriti entities infused with
various permutations and combinations of variously enumerated elements, senses,
feelings, activity and mind. During the state of imbalance, one of more constituents
overwhelm the others, creating a form of bondage, particularly of the mind. The end
of this imbalance, bondage is called liberation, or moksha by Samkhya school of
Hinduism.
The existence of God or supreme being is not directly asserted, nor considered
relevant by the Samkhya philosophers. Skhya denies the final
cause of Ishvara (God). While Samkhya school of Hinduism considers the Vedas as a
reliable source of knowledge, it is an atheistic philosophy

83.

Solution: d)

The jvika school is known for its Niyati doctrine of absolute determinism, the
premise that there is no free will, that everything that has happened, is happening
and will happen is entirely preordained and a function of cosmic principles. jvika
considered the karma doctrine as a fallacy. Ajivika metaphysics included a theory of
atoms similar to the Vaisheshika school of Hinduism, where everything was
composed of atoms, qualities emerged from aggregates of atoms, but the aggregation
and nature of these atoms was predetermined by cosmic forces. jvikas were

http://www.insightsonindia.com

INSIGHTS MOCK TESTS 2015 TEST


24 SOLUTIONS
http://www.insightsonindia.com

atheists and rejected the authority of the Vedas, but they believed that in every living
being is an Atman (soul) a central premise of Hinduism and Jainism

84.

Solution: d)

The novel Samskara is an accurate estimate of brahmin society in the sixties or more
correctly the brahmin societies of all times which suffer the serious problems of
backwardness despite having intellectuals among them. Reason behind all silly
problems that emerge is that their energy is directed by age-old convictions, beliefs,
customs, traditions and superstition. Ananthamurthy raises questions on very
sensitive issues like rituals, samskara, untouchability, sex, community feeling. He
scans human weakness such as greed, lust and lack of human concern in the
Brahmin community. Ananthamurthys characters favour freedom from the life in
shackles of ritualistic performances. Naranappa the rebel character is anti
brahminical in deeds ,but Praneshacharya who earned the title 'the crest jewel of
vedic learning' rejects the double standard thinking after the death of his wife
Bhagirathi. He wants to settle his life with Chandri a prostitute. The sudden death of
Naranappa brings the real examination of Acharya's ideals, learning, and wisdom.
This event brings a real man out of Acharya's being, burdened with suffocating
scriptural knowledge. He wants to live like an ordinary man neither a righteous
brahmin nor the crest jewel of vedic learning.

85.

Solution: d)

Originally the theatre form Swang, was mainly music-based. Gradually, prose too,
played its role in the dialogues. The softness of emotions, accomplishment of rasa
alongwith the development of character can be seen in this theatre form. The two
important styles of Swang are from Rohtak and Haathras. In the style belonging to
Rohtak, the language used is Haryanvi (Bangru) and in Haathras, it is Brajbhasha.
Bhavai is the traditional theatre form of Gujarat. The centers of this form are Kutch
and Kathiawar. The instruments used in Bhavai are: bhungal, tabla, flute, pakhaawaj,
rabaab, sarangi, manjeera, etc. In Bhavai, there is a rare synthesis of devotional and
romantic sentiments.

86.

Solution: b)

Pala art, also called Pala-Sena art or Eastern Indian art, artistic style that flourished
in what are now the states of Bihar and West Bengal, India, and in what is now
Bangladesh. Named for the dynasty that ruled the region from the 8th to the 12th
century ce, Pala style was transmitted chiefly by means of bronze sculptures and
palm-leaf paintings, celebrating the Buddha and other divinities.

http://www.insightsonindia.com

INSIGHTS MOCK TESTS 2015 TEST


24 SOLUTIONS
http://www.insightsonindia.com

Pala-period bronzes, which were cast by the lost-wax process, consist of an alloy of
eight metals. They represent various divinities and, being mainly small in size and
thus portable, were intended for private worship. In terms of style, the metal images
largely continued the Gupta tradition of Sarnath but endowed it with a certain heavy
sensuousness. They differ little from contemporary stone sculptures of the region but
surpass them in the precise definition of ornamental detail, in a certain elegant
virtuosity, and in their emphasis on plasticity. The bronze sculptures from this area
played an important part in the diffusion of Indian influence in Southeast Asia.

87.

Solution: a)

It was considered for a long time that the Himalayas stood as a natural barrier to
protect India against invasions. But, the passes in the northwest mountains such as
the Khyber, Bolan, Kurram and Gomal provided easy routes between India and
Central Asia. These passes are situated in the Hindukush, Sulaiman and Kirthar
ranges. From prehistoric times, there was a continuous flow of traffic through these
passes. Many people came to India through these passes as invaders and immigrants.
The Indo-Aryans, the Indo-Greeks, Parthians, Sakas, Kushanas, Hunas and Turks
entered India through these passes. The Swat valley in this region formed another
important route. Alexander of Macedon came to India through this route. Apart from
invading armies, missionaries and merchants came to India using these routes.
Therefore, these passes in the northwest mountains had facilitated trade as well as
cultural contacts between India and the Central Asia.

88.

Solution: c)

Mesolithic or Middle Stone Age falls roughly from 10000 B.C. to 6000 B.C. It was the
transitional phase between the Paleolithic Age and Neolithic Age. Mesolithic remains
are found in Langhanj in Gujarat, Adamgarh in Madhya Pradesh and also in some
places of Rajasthan.
In the sites of Mesolithic Age, a different type of stone tools is found. These are tiny
stone artifacts, often not more than five centimeters in size, and therefore called
microliths. The hunting-gathering pattern of life continued during this period.
However, there seems to have been a shift from big animal hunting to small animal
hunting and fishing. The use of bow and arrow also began during this period. Also,
there began a tendency to settle for longer periods in an area. Therefore,
domestication of animals, horticulture and primitive cultivation started. Animal
bones are found in these sites and these include dog, deer, boar and ostrich.
Occasionally, burials of the dead along with some microliths and shells seem to have
been practiced.

http://www.insightsonindia.com

INSIGHTS MOCK TESTS 2015 TEST


24 SOLUTIONS
http://www.insightsonindia.com

89.

Solution: d)

Knowledge of surface ocean currents is essential in reducing costs of shipping, since


traveling with them reduces fuel costs. In the wind powered sailing-ship era,
knowledge was even more essential. A good example of this is the Agulhas Current,
which long prevented Portuguese sailors from reaching India. In recent times,
around-the-world sailing competitors make good use of surface currents to build and
maintain speed. Ocean currents are also very important in the dispersal of many life
forms. An example is the life-cycle of the European Eel.
Ocean currents are important in the study of marine debris, and vice versa. These
currents also affect temperatures throughout the world. For example, the ocean
current that brings warm water up the north Atlantic to northwest Europe also
cumulatively and slowly blocks ice from forming along the seashores, which would
also block ships from entering and exiting inland waterways and seaports, hence
ocean currents play a decisive role in influencing the climates of regions through
which they flow. Cold ocean water currents flowing from polar and sub-polar regions
bring in a lot ofplankton that are crucial to the continued survival of several key sea
creature species in marine ecosystems. Since plankton are the food of fish, abundant
fish populations often live where these currents prevail.
Ocean currents can also be used for marine power generation, with areas off of
Japan, Florida and Hawaii being considered for test projects.

90.

Solution: d)

Mahavira regarded all objects, both animate and inanimate, have souls and various
degrees of consciousness. They possess life and feel pain when they are injured.
Mahavira rejected the authority of the Vedas and objected to the Vedic rituals. He
advocated a very holy and ethical code of life. Even the practice of agriculture was
considered sinful as it causes injury to the earth, worms and animals. Similarly the
doctrine of asceticism and renunciation was also carried to extreme lengths by the
practice of starvation, nudity and other forms of self-torture.
91. Solution: a)
Bentinck took effective steps to prevent the ritual of child sacrifice at Saugar Island in
Bengal. He not only prohibited female infanticide but declared them as punishable
crime. Moreover, he declared the welfare of subjects was his cause in India. This was
also seen in the several laws and regulations he enacted.

http://www.insightsonindia.com

INSIGHTS MOCK TESTS 2015 TEST


24 SOLUTIONS
http://www.insightsonindia.com

William Bentinck adopted a policy of non-intervention and non-aggression with


Indian states. If at all he interfered in the affairs of the Indian states, it was only to
end any form of misgovernment and never to annex any territory.
The introduction of English Education was a significant event of Lord William
Bentincks administration. He appointed a committee headed by Lord Macaulay to
make recommendations for the promotion of education.

92.

Solution: a)

Henry Vivian Derozio - founder of the Young Bengal Movement - was born in
Calcutta in 1809 and taught in the Hindu College, Calcutta. He died of cholera in
1833. His followers were known as the Derozians and their movement the Young
Bengal Movement.
They attacked old traditions and decadent customs. They also advocated womens
rights and their education. They founded associations and organized debates against
idol worship, casteism and superstitions.

93.

Solution: d)

The Extremists had no faith in the British sense of justice and fair play. They pointed
out the forceful means by which the British had taken control of India. They believed
that political rights will have to be fought for. They had the spirit of self-reliance and
self-determination.
Their main objective was to attain Swaraj or complete independence and not just
self-government. In contrast, the main Demands of Moderates

Expansion and reform of legislative councils.


Greater opportunities for Indians in higher posts by holding the ICS
examination simultaneously in England and in India.
Separation of the judiciary from the executive.
More powers for the local bodies.
Reduction of land revenue and protection of peasants from
unjust landlords.
Abolition of salt tax and sugar duty.
Reduction of spending on army.
Freedom of speech and expression and freedom to form associations

http://www.insightsonindia.com

INSIGHTS MOCK TESTS 2015 TEST


24 SOLUTIONS
http://www.insightsonindia.com

94.

Solution: a)

The Swadeshi Movement involved programmes like the boycott of government


service, courts, schools and colleges and of foreign goods, promotion of Swadeshi
goods, Promotion of National Education through the establishment of national
schools and colleges. It was both a political and economic movement.
The Swadeshi Movement was a great success. In Bengal, even the landlords joined
the movement. The women and students took to picketing. Students refused using
books made of foreign paper.
The government adopted several tough measures. It passed several Acts to crush the
movement. The Swadeshi volunteers were beaten badly. The cry of Bande Mataram
was forbidden. Schools and colleges were warned not to allow their students to take
part in the movement or else their, aid would be stopped. Some Indian government
employees lost their jobs. Extremist leaders Bala Gangadhar Tilak, Lala Lajpat Rai,
Bipin Chandra Pal and Aurobindo Ghosh were imprisoned and deported.

95.

Solution: c)

Lucknow Pact, (December 1916), agreement made by the Indian National


Congressheaded by Maratha leader Bal Gangadhar Tilak and the All-India Muslim
League led by Muhammad Ali Jinnah; it was adopted by the Congress at
its Lucknow session on December 29 and by the league on Dec. 31, 1916. The meeting
at Lucknow marked the reunion of the moderate and radical wings of the Congress.
The pact dealt both with the structure of the government of India and with the
relation of the Hindu and Muslim communities.
On the former count, the proposals were an advance on Gopal Krishna Gokhales
political testament. Four-fifths of the provincial and central legislatures were to be
elected on a broad franchise, and half the executive council members, including those
of the central executive council, were to be Indians elected by the councils
themselves. Except for the provision for the central executive, these proposals were
largely embodied in the Government of India Act of 1919. The Congress also agreed
to separate electorates for Muslims in provincial council elections and for weightage
in their favour (beyond the proportions indicated by population) in all provinces
except the Punjab and Bengal, where they gave some ground to the Hindu and Sikh
minorities. This pact paved the way for Hindu-Muslim cooperation in the Khilafat
movement and Mohandas Gandhis non-cooperation movement from 1920.

96.

Solution: a)

In 1937 elections were held under the provisions of the Government of India Act of
1935. Congress Ministries were formed in seven states of India. On 1 September 1939

http://www.insightsonindia.com

INSIGHTS MOCK TESTS 2015 TEST


24 SOLUTIONS
http://www.insightsonindia.com

the Second World War broke out. The British Government without consulting the
people of India involved the country in the war. The Congress vehemently opposed it
and as a mark of protest the Congress Ministries in the Provinces resigned on 12
December 1939. The Muslim League celebrated that day as the Deliverance Day. In
March 1940 the Muslim League demanded the creation of Pakistan.

97.

Solution: a)

The Khilafat movement was a very important event in the political history of India.
The Muslims of India had a great regard for the Khilafat (Caliphate) which was held
by the Ottoman Empire. During World War I, the Ottoman Empire (Turkey) joined
the war in favour of Germany. But Turkey and Germany lost the war and a pact
commonly known as Istanbul Accord was concluded between the Allied Forces on
3rdNovember 1918. According to this Pact the territories of Turkey were to be divided
among France, Greece and Britain.
During the war the Indian Muslims were in a very awkward position, because they
had a deep-rooted devotion to the caliphate. They had profound respect for this holy
institution. Therefore, their support to the British Government was subject to the
safeguard and protection of the holy places of Turkey and on the condition that
Turkey will not to be deprived of its territories. But the British Government could not
fulfill both of these promises. The Treaty of Savers 1920 was imposed on Turkey and
its territories like Samarna, Thrace and Anatolia were wrested from it and
distributed among European countries. A wave of anger swept across the Muslin
World and the Indian Muslims rose against the British Government. Muslim leaders
like Maulana Abdul Kalam Azad, Moulana Muhammad Ali Johar, Moulana Shoukat
Ali and others reacted against the British Government policy and were put behind
the bars.
98.

Solution: d)

Here is a list of schemes. Please go through the minor descriptions and the ministry
who is running the scheme.
http://en.wikipedia.org/wiki/List_of_government_schemes_in_India

99.

Solution: c)

The Sustainable Agriculture Initiative (SAI) Platform is the main food and
drink industry initiative supporting the development of sustainable agriculture
worldwide.
In 2002 Nestl, Unilever and Danone created the Sustainable Agriculture Initiative
(SAI) Platform, a non-profit organization to facilitate sharing, at precompetitive

http://www.insightsonindia.com

INSIGHTS MOCK TESTS 2015 TEST


24 SOLUTIONS
http://www.insightsonindia.com

level, of knowledge and initiatives to support the development and implementation


of sustainable agriculture practices involving the different stakeholders of the food
chain.
SAI Platform today counts over 60 members, which actively share the same view on
sustainable agriculture seen as "the efficient production of safe, high quality
agricultural products, in a way that protects and improves the natural environment,
the social and economic conditions of farmers, their employees and local
communities, and safeguards the health and welfare of all farmed species".
Among the latest services and deliverables produced, the SAI Platform developed (or
co-developed) Principles and Practices for sustainable water management at farm
level; recommendations for Sustainability Performance Assessment (SPA); a
standardised methodology for the dairy sector to assess green house gas
emissions; an Executives Training on Sustainable Sourcing; and many more.

100.

Solution: b)

According to the theory the onset of SW Monsoon over Indian subcontinent is driven
by the shift of the subtropical westerly jet northwards from over the plains of India
towards the Tibetan Plateau. This shift is due to the intense heating of the Plateau
during the summer months. This shift of the westerly jet to the north of the
Himalayas is not a slow and gradual process, as expected for most changes in
weather pattern. The primary cause of these is believed to be the height of the
Himalayas. As the Tibetan Plateau heats up the low pressure created over it pulls
the westerly jet northwards. Due to lofty Himalayas, the westerly jet is inhibited
from moving northwards. However, with continuous dropping pressure, sufficient
force is created for the movement of the westerly jet across the Himalayas after a
significant period. As such, the shift of the jet is sudden and abrupt causing
the bursting of SW Monsoon rains onto the Indian plains. The reverse shift happens
for the NE Monsoon. (Source Wikipedia)

http://www.insightsonindia.com

INSIGHTS MOCK TESTS 2015 TEST


24 SOLUTIONS
http://www.insightsonindia.com


http://www.insightsonindia.com

http://www.insightsonindia.com

INSIGHTS ON INDIA MOCK PRELIMINARY EXAM - 2015


INSIGHTS ON INDIA MOCK TEST - 25
GENERAL STUDIES

PAPER-I

Time Allowed: 2 Hours

Maximum Marks: 200

INSTRUCTIONS
1. IMMEDITELY AFTER THE COMMENCEMENT OF THE EXAMINATION, YOU SHOULD
CHECK THAT THIS TEST BOOKLET DOES NOT HAVE ANY UNPRINTED OR TORN OR MISSING
PAGES OR ITEMS, ETC. IF SO, GET IT REPLACED BY A COMPLETE TEST BOOKLET.
2. You have to enter your Roll Number on the Test
Booklet in the Box provided alongside. DO NOT
Write anything else on the Test Booklet.
4. This Test Booklet contains 100 items (questions). Each item is printed only in English. Each item
comprises four responses (answers). You will select the response which you want to mark on the
Answer Sheet. In case you feel that there is more than one correct response, mark the response which
you consider the best. In any case, choose ONLY ONE response for each item.
5. You have to mark all your responses ONLY on the separate Answer Sheet provided. See directions in
the Answer Sheet.
6. All items carry equal marks.
7. Before you proceed to mark in the Answer Sheet the response to various items in the Test Booklet, you
have to fill in some particulars in the Answer Sheet as per instructions sent to you with your
Admission Certificate.
8. After you have completed filling in all your responses on the Answer Sheet and the examination has
concluded, you should hand over to the Invigilator only the Answer Sheet. You are permitted to take
away with you the Test Booklet.
9. Sheets for rough work are appended in the Test Booklet at the end.
10. Penalty for wrong answers :
THERE WILL BE PENALTY FOR WRONG ANSWERS MARKED BY A CANDIDATE IN THE
OBJECTIVE TYPE QUESTION PAPERS.
(i)

There are four alternatives for the answer to every question. For each question for which a
wrong answer has been given by the candidate, one-third of the marks assigned to that
question will be deducted as penalty.

(ii) If a candidate gives more than one answer, it will be treated as a wrong answer even if one of
the given answers happens to be correct and there will be same penalty as above to that
question.
(iii)

If a question is left blank, i.e., no answer is given by the candidate, there will be no penalty
for that question.
http://www.insightsonindia.com

INSIGHTS ON INDIA MOCK TEST SERIES FOR CIVIL SERVICES PRELIMINARY EXAM 2015

http://www.insightsonindia.com INSIGHTS

Page 1

http://www.insightsonindia.com

1. The PSLV programme of the ISRO


launches satellites for which of the
following purposes?
1. Earth observation
2. Navigation
3. Studying space sciences
4. Disaster management
Choose the correct answer using the codes
below.
a)
b)
c)
d)

1 and 2 only
2 and 3 only
1 and 4 only
All of the above

2. The government has recently taken a


decision to revive some Urea factories.
Consider the following about Urea.
1. Urea is naturally produced in
humans and animals.
2. CO2 is used in the manufacturing of
Urea.
3. It also contains NPK (nitrogenphosphorus-potassium).
Choose the correct answer using the codes
below.
a)
b)
c)
d)

1 and 2 only
2 and 3 only
1 and 3 only
All of the above

3. Investments made by which of the


following are considered as domestic
investment in India?
1. A foreign company investing in
India which also has a subsidiary in
India.

2. Investments made by NRIs and


Persons of Indian origin (PIOs) in
India.
3. Profits earned on investments
made by an Indian firm abroad
Choose the correct answer using the codes
below.
a)
b)
c)
d)

1 and 2 only
2 and 3 only
1 and 3 only
2 only

4. What are non-repatriable investments,


in the context of NRIs?
a) It refers to the investment wherein
NRI cannot convert invested money
back to his home country.
b) It refers to the investments where a
foreign currency can be converted
back into the currency of the home
country by an NRI.
c) The investments that come in the
form of remittances where no
return is earned by the NRI.
d) Investment made in government
bonds of home country.

5. A new species of fish called Pethia


striata was discovered by ecologists in
Tunga River in the Kudremukh
National Park, Karnataka. Consider
the following in regard to it.
1. The fish is native to Western Ghats.
2. The fish has been put under the
category of vulnerable as per
IUCN.

http://www.insightsonindia.com INSIGHTS

Page 2

http://www.insightsonindia.com

3. The fish is useful for purposes of


phyto-remediation in the local
rivers.
Choose the correct answer using the codes
below.
a)
b)
c)
d)

1 and 2 only
2 and 3 only
1 and 3 only
1 only

6. The ancient city of Palmyra was


recently in news because of
a) excavation of man made rocks
dating back even the pre-stone age
period
b) its take over by the militant group
ISIS
c) its impending inclusion in the
Chinese One road One Belt
initiative
d) it being the first UNESCO world
heritage site that has been brought
to ruins
7. Consider the following statements:
1. The Vernacular Press Act was
passed in 1878 under the Governor
Generalship and Viceroyalty
of Lord Ripon
2. The Vernacular Press Act of 1878
was directed only against Indian
language newspapers
Which of the above statements is/are correct?
a)
b)
c)
d)

1 Only
2 Only
Both
None

8. Recently released UN World Economic


Situation and Prospects (WESP)
Report for Mid update -2015 has
projected that Indias economic growth

will surpass Chinas growth in 2016. As


per the report this will be made
possible by
1. Strong double digit growth in
agriculture
2. Increase in Indian exports
3. Stronger domestic consumption
Choose the correct answer using the codes
below.
a)
b)
c)
d)

1 and 2 only
2 and 3 only
1 and 3 only
All of the above

9. Which of these states have achieved


100% financial inclusion as per the PM
Jan Dhan Yojana?
1. Meghlaya
2. Kerala
3. Assam
4. Goa
5. Punjab
Choose the correct answer using the codes
below.
a)
b)
c)
d)

2, 3 and 4 only
2, 3, 4 and 5 only
1, 2 and 4 only
1, 3 and 5 only

10. Researchers have recently discovered


the driest location on Earth which is as
dry as the Martian surface. The
location is
a) Yungay in Atacama Desert, Chile
b) Maria Elena south in Atacama
Desert, Chile
c) Erg Chebbi in Sahara Desert
d) Sahel in Sahara desert

http://www.insightsonindia.com INSIGHTS

Page 3

http://www.insightsonindia.com

11. Electrostatic precipitators can solve


the urban pollution in India with
regard to
1. Particulate matter
2. NO2
3. Hydrocarbons
Choose the correct answer using the codes
below.
a)
b)
c)
d)

1 and 2 only
1 only
2 and 3 only
1 and 3 only

12. Catalytic converters, having expensive


metals namely platinum-palladium
and rhodium as the catalysts, are fitted
into automobiles for
a) enhancing their energy efficiency
b) reducing their fuel consumption
c) reducing emission of poisonous
gases
d) improving their transmission
system and speed

13. Genetic modification of plants can be


helpful in
1. increased efficiency of mineral
usage by plants
2. enhanced nutritional value of food
3. making tailor-made plants to
supply alternative resources to
industries
Choose the correct answer using the codes
below.
a)
b)
c)
d)

1 and 2 only
All of the above
2 and 3 only
1 and 3 only

14. The Bt protein in the Bt Cotton is a


toxin. When a cotton plant is made
into Bt Cotton, why isnt it harmed by
this toxin?
a) The toxin is harmful only to insects
not plants
b) The toxin is present in an inactive
form in the plant
c) The genetic modification is done in
a way that the plant is made
immune to such toxins
d) Both (a) and (c)

15. Tropics are known for greater


biological diversity than temperate
regions. Which of the following can
NOT be reason(s) for the same?
1. With the geological stability in
tropics, it had more time for species
evolution and diversification.
2. There is greater solar energy
available at the tropics.
Choose the right option from the code below.
a)
b)
c)
d)

1 only
2 only
Both 1 and 2
Both can be the correct reasons.

16. Consider the following differences


between active and passive euthanasia.
1. Active euthanasia is illegal but
passive euthanasia is legal in India.
2. Unlike Active euthanasia, passive
euthanasia is done by withdrawing
life support.
3. In active euthanasia, instant death
occurs for the patient, unlike
passive euthanasia.

http://www.insightsonindia.com INSIGHTS

Page 4

http://www.insightsonindia.com

Choose the correct answer using the codes


below.
a)
b)
c)
d)

1 and 2 only
All of the above
2 and 3 only
1 and 3 only

17. India and Mongolia recently decided to


elevate their ties to the level of
Strategic partnership. What does the
term strategic partnership mean?
a) Strategic partnership between two
nations is equivalent of joining an
alliance and becoming allies in
international affairs.
b) Its a long-term
interaction/committment between
two countries based on political,
economic, social and historical
factors.
c) Strategic partners are obstructed
with least restrictions in the
economic sphere like tarrifs,
investments and duties.
d) Its a long term mutual agreement
between two nations to support
each other militarily in the
outbreak of a war or aggression.

18. Balance of Payments (BoP) does NOT


include which of the following?
1. Foreign portfolio Investment
2. External commercial borrowing
3. Foreign Direct Investment
4. Trade in invisibles
5. Private remittances
Choose the correct answer using the following
codes:
a) 1, 2 and 5 only
b) 2, 3 and 4 only

c) 1 and 4
d) All are included in BoP.

19. The book Economic History of India


which examined in minute detail the
entire economic record of colonial rule
since 1757, was written by?
a) Justice Mahadev Govind Ranade
b) Dadabhai Naoroji
c) Romesh Chandra Dutta
d) M. N. Roy

20. The Triffin dilemma or paradox is the


conflict of economic interests that
arises between short-term domestic
and long-term international objectives
when
a) a national currency also serves as a
world reserve currency
b) a nation is forced to go for trade
deficit even though it wants trade
surplus
c) a nations monetary policy is largely
in conflict with that of the global
monetary policy
d) free trade agreements clash with
the objective of reviving domestic
industries

21. In his recent speeches, the RBI


governor has pointed out that growth
should not be diverted from other
markets due to quantitative easing.
What does it imply?
1. In periods of slump growth
economies are often resorting to
lose monetary policy to boost
growth in short-run
2. Fiscal stimulus and tax sops by
governments are being used to
attract investment without under

http://www.insightsonindia.com INSIGHTS

Page 5

http://www.insightsonindia.com

taking structural reforms and


policy correction
Which of the above is/are correct?
a)
b)
c)
d)

1 only
2 only
Both 1 and 2
None

22. Consider the following statements


about the Controller general of
Accounts (CGA) in India.
1. He is appointed by the President of
India.
2. He is the principal accounts adviser
to the Union government.
3. Union finance account prepared by
him is presented to the Parliament.
Choose the correct answer using the codes
below.
a)
b)
c)
d)

1 and 2 only
All of the above
2 and 3 only
1 and 3 only

23. Consider the following statements:


1. The Shell gas and oil compnay is
headquartered in London
2. The Netherlands government is the
largest shareholder in the Shell
company
Which of the above statements is/are correct?
a)
b)
c)
d)

1 Only
2 Only
Both
None

24. Worlds first fully warm-blooded fish


named Opah or moonfish was
discovered recently. Beinf a fully
warm-blooded fish means that
a) ability to generate heat
independent of the environment
and circulate heated blood
throughout its body
b) ability to live in all seasons in warm
water irrespective of the
surrounding temperature
c) ability to survive in chilly cold
waters by solely internal body
mechanisms
d) ability to generate heat in the body
without any use of external body
parts independent of the
surrounding environment

25. State Bank of India (SBI) has launched


contact-less credit and debit cards
sbiINTOUCH using Near Field
technology (NFC). Consider the
following about NFC technology.
1. Its a short range low powered
Bluetooth based link technology.
2. A pairing code is needed every time
a device connected via NFC.
3. It operates on radio frequency.
Choose the correct answer using the codes
below.
a)
b)
c)
d)

http://www.insightsonindia.com INSIGHTS

1 and 2 only
All of the above
2 and 3 only
3 only

Page 6

http://www.insightsonindia.com

26. The scheme USTAAD launched by


Union Ministry of Minority affairs is
concerned with
a) upgrading Skills and Training of
minority communities by
preservation of traditional ancestral
arts and crafts
b) skill development of minority
population to make them more
employable in small scale
industries
c) providing financial and technical
support to the traditional craft
based businesses run by minority
for reviving Indian culture
d) providing scholarships to the
talented child artists from the
minority community and showcase
their talent to the whole of India

27. As per Jainism, to attain purity of soul,


one must fight against
1. Worldly passions
2. Bodily senses
3. Ones own soul
Choose the correct answer using the codes
below.
a)
b)
c)
d)

1 and 2 only
2 and 3 only
1 and 3 only
All of the above

28. Consider the following statements


about the Civil Disobedience
movement (CDM).
1. The movement officially was not
approved by the congress before its
launch.
2. 26th January as Independence Day
all over the country was decided
during CDM.

3. All the round table conferences


between the British government
and Indian leaders
followed
only after this movement.
4. The movement was withdrawn
because of violence.
Choose the correct answer using the codes
below.
a)
b)
c)
d)

1 and 2 only
1 and 4 only
2 and 3 only
1 and 3 only

29. The Moro people or Bangsamoro are a


population of indigenous Muslims
living in which of the following
countries?
a) Malaysia
b) Indonesia
c) Philippines
d) Myanmar

30. Capillary action is the ability of a


liquid to flow in narrow spaces without
the assistance of
1. Gravity
2. Inter-molecular forces
3. Surface tension of water
Choose the correct answer using the codes
below.
a)
b)
c)
d)

http://www.insightsonindia.com INSIGHTS

1 and 2 only
2 and 3 only
1 and 3 only
1 only

Page 7

http://www.insightsonindia.com

31. Mangroves grow best in Inter-tidal


zones due to
1. shallowness of water there
2. high nutrient content in inter-tidal
zones
3. lesser sediment load
Choose the correct answer using the codes
below.
a)
b)
c)
d)

1 and 2 only
2 and 3 only
1 and 3 only
All of the above

32. Global strategic petroleum


reserves (GSPR) refer to
a) untapped petroleum reserves that
lie in the global commons like
Antarctica
b) untapped petroleum reserve in the
ocean basins of international
waters
c) crude oil inventories held by
governments to tackle an
emergency of energy crisis.
d) petroleum reserves that lie in zones
of international territorial disputes

c) 2 and 3 only
d) All of the above

34. Which of the following are sources of


methane emission?
1. Wetland rice cultivation
2. Livestock rearing
3. Oceans
Choose the correct answer using the codes
below.
a)
b)
c)
d)

1 and 2 only
1 and 3 only
2 and 3 only
All of the above

35. As per a recent ruling of Supreme


Court regulating public
advertisements, whose images can
feature in a government
advertisement?
1. Chief Minister
2. Prime Minister
3. President and Vice-President
4. Late Prime Ministers
Choose the correct answer using the codes
below.

33. Thickness of atmosphere is greatest at


the equator due to
1. Air being less dense at equator
2. High insolation and strong
convectional currents in
troposphere over equator
3. At equator gravitational pull on
gases is more
Choose the correct answer using the codes
below.
a) 1 and 2 only
b) 1 and 3 only

a)
b)
c)
d)

1 and 2 only
2 and 3 only
2 only
3 and 4 only

36. Which of the following come under the


concept of Ecological Engineering?
1. pest management through
biocontrol agents
2. using geo-engineering to stall
climate change

http://www.insightsonindia.com INSIGHTS

Page 8

http://www.insightsonindia.com

3. conservation of non-renewable
energy
Choose the correct answer using the codes
below.
a)
b)
c)
d)

1 and 2 only
2 and 3 only
2 only
1 and 3 only

37. Why does milk overflow when it starts


boiling in a vessel, and water does not?
a) Milks boiling point is lower than
that of water.
b) Milk is denser than water, so more
heat makes it lighter for it to
overflow easily.
c) When milk is heated, its denser
contents like fat start depositing on
the top layer stoping the vapour
from exiting
d) milk is heavier than water, so
escaping of vapour takes place at
much grater force pushing milk out
of the container

38. Consider the following statements:


1. An individual on the path to
becoming a buddha is known as
Bodhisattva
2. Avalokiteshvara is an embodiment
of Buddha and Shiva worshipped
by converts from Buddhism to
Hinduism during the reign of
Gupta empire
Which of the above statements is/are correct?
a)
b)
c)
d)

1 Only
2 Only
Both
None

39. With reference to permaculture,


consider the following stattements
1. It is a systems approach to
agriculture which encompasses
elements and processes aimed to
making agriculture sustainable and
efficient
2. Agroforestry and rainwater
harvesting are some of the
elements of permaculture
Which of the above statements is/are correct?
a)
b)
c)
d)

1 Only
2 Only
Both
None

40. NASAs Messenger spacecraft crashed


into Mercury recently. This crash was
due to
a) excessive gravitation pull of the Sun
b) Change in Messengers orbital
positions with respect to Mercurys
c) Messenger running out of fuel
d) it being hit by an asteroid orbiting
Mercury

41. Internet.org, a partnership between a


Social networking company and some
mobile companies, aims at
a) providing seamless wireless
internet connectivity to every
person in developing countries
b) providing free web access in less
developed countries
c) providing affordable or free access
to select internet services in less
developed countries
d) providing basic digital literacy
about internet to every person in
the less developed countries

http://www.insightsonindia.com INSIGHTS

Page 9

http://www.insightsonindia.com

42. Consider the following statements


regarding the administrator of UTs in
India.
1. An administrator of a union
territory is an agent of the
President and not head of state like
a governor.
2. The administrator of a UT has
absolute veto power over the laws
enacted by the legislative assembly
of that UT.
3. In a UT having an elected
government, administrator is
appointed after consultation with
the CM of the UT.
Choose the correct answer using the codes
below.
a)
b)
c)
d)

1 and 2 only
2 and 3 only
1 only
1 and 3 only

43. In the case of difference of opinion


between the Lt. governor (LG) of Delhi
and his ministers
a) the matter is to be referred to the
Central government for resolution
and its decision will prevail
b) the matter is referred to the
President and the LG will act
accordingly.
c) the matter is referred to the Delhi
assembly which decides the issues
on the basis of special majority
d) the matter is referred a bench of
SC, the decision of which is binding
on the LG and ministers

44. Which of the following participate in


the electing the President of India?
1. Elected Members of Legislative
Councils
2. Nominated members of both
houses of Parliament
3. Elected members of the legislative
assembly of the UTs
Choose the correct answer using the codes
below.
a)
b)
c)
d)

1 and 2 only
2 and 3 only
3 only
1 and 3 only

45. If Parliament has repeatedly failed to


perform its duty, in the opinion of the
President, then as per his
constitutional mandate he can
1. Dissolve the Lok Sabha
2. Dismiss the Council of Ministers
3. Promulgate ordinance to keep the
legislative business on track
Choose the correct answer using the codes
below.
a)
b)
c)
d)

1 and 2 only
2 and 3 only
3 only
None of the above

46. Consider the following statement


This school drew upon the
anthropomorphic traditions of
Roman religion and represented
the Buddha with a youthful Apollolike face, dressed in garments
resembling those seen on Roman
imperial statues.

http://www.insightsonindia.com INSIGHTS

Page 10

http://www.insightsonindia.com

The above statement refers to which of the


following schools of art?
a)
b)
c)
d)

Mathura school of art


Gandhara school of art
Taxila School of art
Both a and b

47. Which of the following bodies do NOT


find a mention in the constitution of
India?
1. Cabinet
2. Parliamentary Committees
3. NITI Ayog
4. Prime Ministers Office (PMO)
Choose the correct answer using the codes
below.
a)
b)
c)
d)

1 and 4 only
2 and 3 only
3 and 4 only
All of the above

48. Which of the following announcements


were made at the Delhi durbar, 1911?
1. Capital to be Delhi
2. Annulment of partition of Bengal
3. Provision of separate electorates for
minorities
Choose the correct answer using the codes
below.
a)
b)
c)
d)

1 and 2 only
1 and 3 only
2 only
All of the above

49. The Government of India Act 1919 led


to the adoption of an important
tradition into India. This is
a) the finance minister presenting the
whole budget by reading
b) Presidents address to both houses
of Parliament
c) Use of English language in
legislatures
d) All of the above

50. With reference to spacewalks, consider


the following statements:
1. Before going out for spacewalk,
astronauts breathe pure oxygen for
a few hours to get rid of all the
nitrogen in an their body
2. Astronauts practice spacewalks
underwater in a large swimming
pool which is called as the Neutral
Buoyancy Laboratory, or NBL
Which of the above statements is/are correct?
a)
b)
c)
d)

1 Only
2 Only
Both
None

51. Laterite soils found in India are


rendered infertile because of
1. excessive iron in the soil
2. lack of organic matter
3. little nitrogen content in the soil
Choose the correct answer using the codes
below.
a)
b)
c)
d)

http://www.insightsonindia.com INSIGHTS

1 and 2 only
1 and 3 only
2 only
None of the above

Page 11

http://www.insightsonindia.com

52. Which of the following factors


contribute to the formation of the
Indian Monsoon?
1. Land-sea frontier of India and its
differential heating
2. High temperature and shifting of
the Sun in summer
3. Trade winds
Choose the correct answer using the codes
below.
a)
b)
c)
d)

1 and 2 only
1 and 3 only
2 only
All of the above

53. Consider the following regions.


1. Slopes of the Western Ghats
2. North-eastern regions of Arunachal
Pradesh, Meghalaya, Assam
3. Tarai areas of the Himalayas
4. Andaman groups of Islands
What kind of vegetation are you most likely to
find common in all these areas?
a)
b)
c)
d)

Tropical Deciduous forests


Tropical evergreen forests
Monsoon type of Forests
Mountain vegetation

54. The Badami cave temples are a


complex of temples located at Badami,
a town in the Bagalkot District in the
north part of Karnataka, India. Which
of the following dynasties does Badami
cave architecture belong to?
a) Chalukya
b) Rashtrakuta
c) Hoysala
d) Pallava

55. With reference to Rohingya People


consider the following statements
1. They are Indo-Aryan peoples living
in northern Rakhine State,
Myanmar, who predominantly
speak Urdu mixed Bengali language
2. Next to Myanmar, significant
population of Rohingya people live
in Saudi Arabia
Which of the above statements is/are correct?
a)
b)
c)
d)

1 Only
2 Only
Both
None

56. Choosing people for constitutional


positions in legislative bodies in India
can NOT happen by which of the
following systems?
1. First past the post (FPTP) election
system
2. Proportional Representation
election System
3. Nomination
4. Competitive Examinations
Choose the correct answer using the codes
below.
a)
b)
c)
d)

1 and 4 only
2 and 3 only
3 and 4 only
4 only

57. With reference to Deep Space Climate


Observatory, consider the following
statements:
1. It was launched by NASA for the
purpose of Earth observation
2. Its functions include observing
phenomena on Earth including

http://www.insightsonindia.com INSIGHTS

Page 12

http://www.insightsonindia.com

changes in ozone, aerosols, dust


and volcanic ash, cloud height,
vegetation cover and climate
Which of the above stateents is/are correct?
a)
b)
c)
d)

1 Only
2 Only
Both
None

1. Constitutionally, he is under the


control and subordinate to the
Central government.
2. Consultation with the CM of the
state is a must before appointing
Governor of that state.
3. Domicile residents of a state are not
eligible for being the Governor of
that state.
Choose the correct answer using the codes
below.

58. The proceedings in Lok Sabha always


starts with X, move to Y and then Z.
X, Y and Z here are
a) Zero hour, Business hour and
Agenda of the house
b) Zero hour, Question hour and
Agenda of the house
c) Question hour, Zero hour and
Agenda of the house
d) Business hour, Zero hour and
Question hour

59. Any withdrawal of money from the


consolidated fund of India by the
government has to go through the
same procedure as that of a
a) Regular government budget
b) Regular government budget only in
case of major expenditures that
exceed the annual budget
c) Ordinary withdrawal from the
Public Account of India
d) A withdrawal from the Public
Account of India in emergency
cases

a)
b)
c)
d)

1 and 2 only
1 only
3 only
None of the above

61. If the Legislative assembly (LA) of a


state passes a bill, which is then
rejected by the legislative council (LC),
which of the following may follow?
1. The bill will stand repealed.
2. The bill will be referred to a joint
sitting of both the houses.
3. The bill will be referred to the
Governor for a final view.
4. The bill can be again passed by the
LA and re-sent to the LC.
5. The LA can directly reject the view
of the LC, and go ahead with
enacting the bill.
Choose the correct answer using the codes
below.
a)
b)
c)
d)

4 or 5 only
1 or 2 or 3 only
2 or 5 only
4 only

60. As per the Constitution, which of these


statements regarding the Office of
Governor is/are true?

http://www.insightsonindia.com INSIGHTS

Page 13

http://www.insightsonindia.com

62. With reference to natural rubber,


consider the following statements
1. It consists of polymers of the
organic compound isoprene, with
minor impurities of other organic
compounds plus water
2. Rubberwood plants requires welldrained, weathered soil consisting
of laterite, lateritic types,
sedimentary types, nonlateritic red,
or alluvial soils
Which of the above statements is/are correct?
a)
b)
c)
d)

1 Only
2 Only
Both
None

63. With reference to child labour problem


in India, consider the following
statements
1. At present there is complete ban on
child labour under the age of 14 in
all types of occupation except farm
and domestic work after school
hours
2. The Child Labour (Prohibition &
Regulation) Act is being
implemented by the Ministry of
Women and Child Welfare
Which of the above statements is/are
INCORRECT?
a)
b)
c)
d)

1 Only
2 Only
Both
None

64. Consider the following statements.

1. Transfer of judges of the high


courts is the sole prerogative of the
Chief Justice of India (CJI).
2. The state government can take
disciplinary action against erring
judges of the High Court
in
that respective state.
3. The CM of the state is consulted
before appointing the judges of the
concerned High Court.
Choose the correct answer using the codes
below.
a)
b)
c)
d)

1 and 2 only
1 only
3 only
None of the above

65. Union Cabinet has given its approval


to Benami Transactions (Prohibition)
(Amendment) Bill, 2015. The bill aims
to
a) tackle black money generation in
India
b) regulate unauthorized transactions
in the security markets
c) regulate all private institutions
where the owners identity has not
been officially verified
d) regulate money laundering
activities on cyber networks

66. Minimal Alternate Tax has been in


news for some time. It is basically
a) a minimum amount of tax that
companies have to pay for
diversifying their business portfolio
b) the minimum tax that companies
have to pay even if their book
profits are zero

http://www.insightsonindia.com INSIGHTS

Page 14

http://www.insightsonindia.com

c) the minimum tax that companies


need to pay even if their net income
is shown to be zero
d) the minimum tax that companies
need to pay to start doing business
outside India

67. The ultimate goal of Buddhism is the


attainment of the sublime state of
Nirvana, which as per it can be
attained by
a) following the eightfold path taught
by the Buddha
b) realizing directly the four noble
truths taught by the Buddha
c) contemplating on existence,
meditation and Buddhist spiritual
processes
d) doing good to all selflessly

68. Consider the following statements with


reference to a River in India
1. This river rises on the western
slopes of the Aravalli Range near
Ajmer
2. This river is known as Sagarmati
near its origin place
3. This river dissipates into the wastes
of the northeastern part of the
marsh called the Rann of Kachchh
(Kutch) in Gujarat state

69. With reference to Varishtha Pension


Bima Yojana (VPBY), consider the
following statements
1) The scheme will be operated by all
the nationalized banks
2) This scheme aims to provide
monthly pension ranging from Rs
500/ to Rs 5,000/ per month to
senior citizens of the country
3) Citizens aged 60 years and above
can avail the benefits of this
scheme
Which of the above statements is/are correct?
a)
b)
c)
d)

1 and 2 Only
1 and 3 Only
2 and 3 Only
All

70. Which of the following were the factors


behind the decline of the Gupta
empire?
1. Foreign invasion
2. Rise of the feudatories in the
kingdom
3. Immense economic loss due to
lowered revenue from trade and
commerce
Choose the correct answer using the codes
below.
a)
b)
c)
d)

1 and 2 only
2 and 3 only
1 and 3 only
All of the above

Identify the river from above clues:


a)
b)
c)
d)

River Banas
River Luni
River Kali Sindh
River Gomti

71. Din-e Illahi, started by Mughal


emperor Akbar was
a) a system of governance in the
provinces to keep the subedars
under check

http://www.insightsonindia.com INSIGHTS

Page 15

http://www.insightsonindia.com

b) a system of tax collection that was


based on concessions for poor and
equity for landless farmers
c) a religion where the soul is
encouraged to purify itself by
yearning for God.
d) a set of moral precepts propounded
by Akbar that carried the force of
law
72. During autumn and spring seasons,
there is a bloom of phytoplankton
growth in lakes. This is because in
these seasons
a) The biotic and nutrient load of lake
water is at its peal
b) Lake waters are least disturbed by
rain and surface run-offs that could
have harmed phytoplanktons
c) Temperature is optimal for the
growth of phytoplanktons
d) Lake water churns and
redistributes water and oxygen
which are vital for the growth of
phytoplanktons
73. Which of the following committees
is/are NOT related to Panchayat Raj
institutions?
1. Ashok Mehta committee
2. Balwant Rai Mehta committee
3. Dr.L.M.Singhvi Committee
4. G.V.K Rao committee
Choose the correct answer using the codes
below:
a)
b)
c)
d)

3 Only
4 Only
Both 3 and 4
All are related

74. Your friend tells you that you can


increase your weight without

increasing your mass. Which of these


factors is your friend referring to, that
can do so?
1. Change in the rotation period of the
earth
2. Change in the revolution time of
the earth around the sun
3. Change in thickness (radius) of the
earth
Choose the correct answer using the codes
below.
a)
b)
c)
d)

1 and 2 only
2 and 3 only
1 and 3 only
3 only

75. Micro-propagation is a technique of


a) Producing a large number of plants
through tissue culture
b) Large scale genetic modification in
the micro components of plant
c) Growing plants on a large scale in
nutrient medium which has only
micro-nutrients
d) None of the above

76. With reference to Magna Carta,


consider the following statements
1. It promised the protection of
church rights and other rights to
rebelling barons
2. It was a charter agreed and
enacted by the Pope of Vatican in
1215
Which of the above statements is/are
correct?
a)
b)
c)
d)

http://www.insightsonindia.com INSIGHTS

1 Only
2 Only
Both
None
Page 16

http://www.insightsonindia.com

77. Which of the following diseases in


India can be tackled by improving
sanitation?
1. Anaemia
2. Cholera
3. Japanese encephalitis (JE)
4. Hepatitis

3. Reducing debt to GDP ratio of the


country
Choose the correct answer using the codes
below.
a)
b)
c)
d)

1 and 2 only
2 and 3 only
1 and 3 only
3 only

Choose the correct answer using the codes


below.
a.
b.
c.
d.

1 and 2 only
2, 3 and 4 only
3 only
All of the above

78. Consider the following statement:


Its purpose is to contribute to peace and
security by promoting international
collaboration through education, science, and
culture in order to further universal respect
for justice, the rule of law, and human rights
along with fundamental freedom proclaimed
in the United Nations Charter.
Which of the following United Nations body
has the above statement as its main purpose?
a)
b)
c)
d)

UNICEF
UNHCR
UNESCO
UNDP

79. In budget documents, the term fiscal


prudence is often mentioned. What
does it imply?
1. Not taking any new government
initiative to lower government costs
2. Harmonization of monetary and
fiscal targets

80.With reference to LED lamp, consider


the following statements:
1. LEDs are not affected by high
temperature unlike fluorescent
lamps
2. LED chips need controlled direct
current (DC) electrical power in
order to function smoothly
3. Unlike other lamps, most LEDs do
not emit light in all directions
Which of the above statements is/are
INCORRECT?
a)
b)
c)
d)

1 and 3 Only
1 and 2 Only
2 and 3 Only
All

81. Increasing the Cash Reserve Ratio


(CRR) and interest rates in the
economy will lead to
a) greater money supply and higher
investments in the economy
b) lesser money supply and lower
investments in the economy
c) constant money supply but lower
investments in the economy
d) lesser money supply but higher
investment in the economy

http://www.insightsonindia.com INSIGHTS

Page 17

http://www.insightsonindia.com

82. The main items of capital receipts of


the Government of India are
1. Market borrowings
2. Borrowings from the RBI
3. Borrowings by sale of treasury bills
4. Sale of share in PSUs
Choose the correct answer using the codes
below.
a)
b)
c)
d)

1 and 2 only
All of the above
1, 3 and 4 only
3 and 4 only

83. Consider the phenomenon of


casualization of workforce.
1. It is the movement of labour from
formal sector to the self-employed
and informal sectors
2. Casualization has only increased
after the 1991 economic reforms.
3. Casualization leads to lower
economic growth of a country.
Choose the correct answer using the codes
below.
a)
b)
c)
d)

1 and 2 only
2 and 3 only
1 and 3 only
2 only

84. Consider the following statements:


1. It is a national park located in West
Garo Hills district of Meghalaya
2. UNESCO added this National park
to its list of Biosphere Reserves in
May 2009
3. This biosphere reserve has a
remnant population of the Red
panda

To which of the following national


parks/biosphere reserves does the above
description apply?
a)
b)
c)
d)

Manas
Nokrek
Dihang-Dibang
Dibru Saikhowa

85. A reduction in Per capita income of


India can happen by
1. High rates of depreciation in the
economy
2. Negative net income from abroad
3. Lowering of Indias GDP
Choose the correct answer using the codes
below.
a)
b)
c)
d)

1 and 2 only
2 and 3 only
1 and 3 only
All of the above

86. The upstream industries in the


petroleum sector are largely concerned
with
1. Marketing of crude oil
2. Production of crude oil
3. Exploration of crude oil
4. Refining of crude oil
Choose the correct answer using the codes
below.
a)
b)
c)
d)

http://www.insightsonindia.com INSIGHTS

1, 2 and 3 only
1 and 4 only
2 and 3 only
All of the above

Page 18

http://www.insightsonindia.com

87. With reference to the steel sector in


India, consider the following
statements:
1. In 2014, India was the second
largest steel producer in the
world, next only to China
2. India is the worlds largest
producer of sponge iron with a
host of coal based units
3. India is a net importer of steel
from other countries
Which of the above statements is/are correct?
a)
b)
c)
d)

1 and 3 Only
2 and 3 Only
1 and 2 Only
All

88. Ocean currents are at the surface as


well as in the deeper ocean. Surface
and deep ocean currents are driven
respectively by which of the following
factors?
a) Wind and density differences in the
ocean
b) Wind and temperature differences
in the ocean layers
c) Gravity and temperature
differences in the ocean layers
d) Coriolis force and density
differences in the ocean

89. Little or no ocean tides occur at the


equator due to
1. the regular heating of the equators
2. the peculiar orbital plane of the
moon
3. presence of heavy surface water
currents
Choose the correct answer using the codes
below.

a)
b)
c)
d)

1 and 2 only
1 and 3 only
2 and 3 only
2 only

90. Subduction of the earth can lead to


which of the following?
1. Cyclones
2. Tsunami
3. Volcanism
4. Earthquakes
5. Orogeny
Choose the correct answer using the codes
below.
a)
b)
c)
d)

1, 2 and 3 only
2, 3 and 4 only
2, 3, 4 and 5 only
3, 4 and 5 only

91. In which of the following ways, the


earths atmosphere is heated?
1. Directly by solar radiation
2. Indirectly by radiation reflected
from earth
3. Transfer of heat across earth by
advection
Choose the correct answer using the codes
below.
a)
b)
c)
d)

1 and 2 only
1 and 3 only
2 and 3 only
All of the above

92. Mimansa, a school of Vedanta


Philosophy, recognizes which of the
following as the means of valid
knowledge?
1. Perception

http://www.insightsonindia.com INSIGHTS

Page 19

http://www.insightsonindia.com

2. Postulation
3. Inference
4. Comparison and analogy

c) 1 and 3 only
d) All of the above

Choose the correct answer using the codes


below.
a)
b)
c)
d)

1 and 2 only
2 and 4 only
1, 3 and 4 only
All of the above

93. With reference to tropical cyclones,


consider the following statements
4. They are characterized by a lowpressure center and numerous
thunderstorms that produce
strong winds and flooding rain
5. Tropical cyclones help to maintain
equilibrium in the Earth's
troposphere
Which of the above statements is/are correct?
a)
b)
c)
d)

1 Only
2 Only
Both
None

94. Which of the following could be


verified for the Indus Valley
Civilization?
1. They had uniform measure of
weights and measures.
2. They had monarchial style of
governance.
3. Gold jewelry was popular in the
civilization.
Choose the correct answer using the codes
below.
a) 1 and 2 only
b) 2 and 3 only

95. Which of the following play a role in


the formation of tropical cyclones?
1. Westerlies
2. Tropical jet streams
3. Roaring forties
Choose the correct answer using the codes
below.
a)
b)
c)
d)

1 and 2 only
1 and 3 only
2 only
None of the above

96. The Indigo movement, 1860, was


initiated by the peasants because
a) european planters forced peasants
to grow indigo
b) peasants were not being given
market price for indigo produced
by the government
c) the British government imposed
heavy tax on the production of
Indigo
d) farmers growing Indigo were not
allowed to grow any other crop

97. Consider the following statements


about Bardoli Satyagraha.
1. It was started after Gandhi joined
the cause of national freedom
movement.
2. It was centred around increased
land revenue by the British
government.
3. The movement was led by
Vallabhbhai Patel.

http://www.insightsonindia.com INSIGHTS

Page 20

http://www.insightsonindia.com

4. It was restricted to Gujarat.

Choose the correct answer using the codes


below.
a)
b)
c)
d)

1 and 2 only
1 and 3 only
2 and 4 only
All of the above

and West Antarctica become


unstable and melt at an
unstoppable rate
2. According to the latest consensus
science reported in September by
the Intergovernmental Panel on
Climate Change (IPCC), the Earth
has already warmed 2C from 1880
(preindustrial times) to 2012
Which of the above statements is/are correct?

98. Consider the following statements


1. A Rubiks Cube is a combination
puzzle toy
2. A normal (3x3x3) Rubiks Cube
consists of 21 pieces

a)
b)
c)
d)

1 Only
2 Only
Both
None

Which of the above statements is/are correct?


a)
b)
c)
d)

1 Only
2 Only
Both
None

99. The de facto assumption of climate


change policy is that the world must
limit the increase in global
temperatures to 2 degrees Celsius
above pre-Industrial levels. With
reference to this, consider the
following statements
1. It is assumed that if the limit is
crossed, ice sheets in Greenland

100.
Which of the following does
NOT find mention in the Preamble to
Indias Constitution?
1. Social justice
2. Liberty of worship
3. Equality of status
4. Liberty of religion
Choose the correct answer using the codes
below:
a)
b)
c)
d)

http://www.insightsonindia.com INSIGHTS

2 Only
3 Only
2 and 4 Only
4 Only

Page 21


Insights Mock Tests 2015: Test 25 Solutions

1. Solution: d)
Union Cabinet has approved continuation of Polar Satellite Launch Vehicle (PSLV)
programme .
The continuation PSLV programme will enable Indian Space Research Organisation
(ISRO) to launch of satellites required for earth observation & navigation and Space
Sciences. It will also help government to clinch commercial launch service contracts
and meet the demand for the launch of satellites at a frequency of four to five
launches per year.
It was sanctioned by Union Government in 2008 to use PSLV as a workhorse vehicle
to meet the countrys satellite launch requirements. The operationalisation of this
programme has made India self-reliant in the launching capability of satellites for
the purpose of earth observation, disaster management, space sciences and
navigation.

2. Solution: d)
Urea is an inexpensive form of nitrogen fertilizer with an NPK (nitrogenphosphorus-potassium) ratio of 46-0-0. Although urea is naturally produced in
humans and animals, synthetic urea is manufactured with anhydrous ammonia.
Although urea often offers gardeners the most nitrogen for the lowest price on the
market, special steps must be taken when applying urea to the soil to prevent the loss
of nitrogen through a chemical reaction.
Read more at http://homeguides.sfgate.com/urea-fertilizer-48588.html

3. Solution: d)
Union Government has decided to amend Foreign Direct Investment (FDI) norms
for NRIs, Persons of Indian Origin (PIOs) and Overseas Citizens of India (OCI) to
increase capital flows into the country.
Government will amend FDI policy on investments by NRIs, PIOs & OCIs which will
give them parity in economy and education. Now non-repatriable investments of
NRIs, PIOs & OCIs under under Schedule 4 of FEMA regulations will considered as
domestic investment.

4. Solution: a)
http://www.insightsonindia.com INSIGHTS

Page 1


Insights Mock Tests 2015: Test 25 Solutions

Word Repatriation is derived from the latin word Repatriare which means go
home. Repatriation is the process of returning a person back to ones place of origin
or citizenship. The term is also used to refer to the process of converting a foreign
currency back into the currency of the home country.
Repatriable NRI Investment : It refers to the investment wherein NRI can take the
invested money back to foreign currency i.e., amount invested can be converted to an
investors home country.
Non-Repatriable NRI Investment: It refers to the investment wherein NRI cannot
convert invested money back to his home country.

5. Solution: d)
The fish is native to Western Ghats, a biodiversity hotspot. It is generally found in
two streams of Mudba and Turad of Tunga River. The male of the Pethia striata
species is red in colour and measures 3-4cm in length, while the female is grey with a
striking pattern of scales. They are usually are found in small groups of 3-4
individuals in shallow pools.
he discovery of species is addition to the largest fish family which is scientifically
called as Cyprinidae that includes goldfish, common carp and zebrafish.

6. Solution: b)
Ancient ruins of Palmyra dates back to the first and second Century when it was
under Roman rule. Palmyra was once a Silk Road hub and one of the cultural centres
of the ancient world. The city occupies mythological status in Syria and is home to
some of the most beautiful and well-preserved ruins of antiquity, including the
Temple of Bel, built in 1st century.
Islamic State of Iraq and Syria (ISIS) militants have taken complete control of the
ancient Syrian city of Palmyra. The city is a UNESCO designated World Heritage site
and home to some of the worlds most magnificent ancient ruins. With the seizure of
entire ancient city, there are fears that the militants will destroy the ancient ruins as
there is no presence of government troops who had entirely withdrawn from the city
following an IS advance. It should be noted that the ancient ruins are situated in a
strategically important area on the road between Syrias capital Damascus and the
contested eastern city of Deir al-Zour.

http://www.insightsonindia.com INSIGHTS

Page 2


Insights Mock Tests 2015: Test 25 Solutions

7. Solution: b)

The Vernacular Press Act was passed in 1878 under the Governor Generalship and
Viceroyalty of Lord Lytton, for better control of Indian language newspapers. The
purpose of the Act was to control the printing and circulation of seditious material,
specifically that which could produce disaffection against the British Government in
India in the minds of the masses.

8. Solution: b)
The report has projected Indias economy to grow by 7.6 per cent in year 2015 and
7.7 per cent in 2016. While, it has projected Chinas economic growth will be 7 per
cent in year 2015 and 6.8 per cent in 2016.
The main reasons for strengthening of growth in this region will be because of
stronger domestic consumption and investment, and increase in exports.
The report has forecasted South Asian regions GDP will grow by 6.7 per cent in 2015
and 6.9 per cent in 2016.

9. Solution: c)
Union Finance Ministry on 14 November 2014 announced that Goa and Kerala
became the first states in the country to achieve 100 percent Financial Inclusion
under Pradhan Mantri Jan Dhan Yojana (PMJDY).
Also, three Union Territories (UTs) namely Chandigarh, Puducherry and
Lakshadweep achieved 100 percent financial inclusion.
And recently, Meghalaya has achieved 100 percent success in implementing Union
Governments flagship scheme for financial inclusion-Pradhan Mantri Jan Dhan
Yojna (PMJDY).

10.

Solution: b)

This discovered site is dry as Mars and has a mean atmospheric relative humidity
(RH) of 17.3 percent and a soil RH of a constant 14 percent at a depth of one meter.
Soil value taken form the MES site matches the lowest RH measurements taken on
Mars by the Mars Science Laboratory at Gale Crater. Thus, making these conditions
to establish the fact that this site is as dry as those found recently on the Martian
surface. Apart from this scientist also have found some bacteria species living there
in the soil profile. This discovery also has toppled the Yungay region which was the
http://www.insightsonindia.com INSIGHTS

Page 3


Insights Mock Tests 2015: Test 25 Solutions

driest area of the hyper-arid Atacama desert. Yungay region conditions were close to
the so-called dry limit for life on Earth. However, with this discover MES is much
drier than Yungay.

11. Solution: b)
There are several ways of removing particulate matter; the most widely used of which
is the electrostatic precipitator (Figure 16.1), which can remove over 99 per cent
particulate matter present in the exhaust from a thermal power plant. It has
electrode wires that are maintained at several thousand volts, which produce a
corona that releases electrons. These electrons attach to dust particles giving them a
net negative charge. The collecting plates are grounded and attract the charged dust
particles. The velocity of air between the plates must be low enough to allow the dust
to fall.

12. Solution: c)
The catalysts, are fitted into automobiles for reducing emission of poisonous gases.
As the exhaust passes through the catalytic converter, unburnt hydrocarbons are
converted into carbon dioxide and water, and carbon monoxide and nitric oxide are
changed to carbon dioxide and nitrogen gas, respectively. Motor vehicles equipped
with catalytic converter should use unleaded petrol because lead in the petrol
inactivates the catalyst.

13. Solution: b)
Plants, bacteria, fungi and animals whose genes have been altered by manipulation
are called Genetically Modified Organisms (GMO). GM plants have been useful in
many ways. Genetic modification has:

made crops more tolerant to abiotic stresses (cold, drought, salt, heat).
reduced reliance on chemical pesticides (pest-resistant crops).
helped to reduce post harvest losses.
increased efficiency of mineral usage by plants (this prevents early exhaustion
of fertility of soil).
enhanced nutritional value of food, e.g., Vitamin A enriched rice.

In addition to these uses, GM has been used to create tailor-made plants to supply
alternative resources to industries, in the form of starches, fuels and
pharmaceuticals.
http://www.insightsonindia.com INSIGHTS

Page 4


Insights Mock Tests 2015: Test 25 Solutions

14. Solution: b)
Some strains of Bacillus thuringiensis produce proteins that kill certain insects such
as lepidopterans (tobacco budworm, armyworm), coleopterans (beetles) and
dipterans (flies, mosquitoes). B. Thuringiensis forms protein crystals during a
particular phase of their growth. These crystals contain a toxic insecticidal protein.
Why does this toxin not kill the Bacillus? Actually, the Bt toxin protein exist as
inactive protoxins but once an insect ingest the inactive toxin, it is converted into an
active form of toxin due to the alkaline pH of the gut which solubilise the crystals.

15. Solution: d)
Ecologists and evolutionary biologists have proposed various hypotheses; some
important ones are (a) Speciation is generally a function of time, unlike temperate
regions subjected to frequent glaciations in the past, tropical latitudes have remained
relatively undisturbed for millions of years and thus, had a long evolutionary time for
species diversification, (b) Tropical environments, unlike temperate ones, are less
seasonal, relatively more constant and predictable. Such constant environments
promote niche specialisation and lead to a greater species diversity and (c) There is
more solar energy available in the tropics, which contributes to higher productivity;
this in turn might contribute indirectly to greater diversity.

16. Solution: b)
Active euthanasia: In this case medical professionals or another person deliberately
do something that causes instant death of patient.
Passive euthanasia: In this case, the common treatments such as antibiotics or life
support of patient are withhold which in turn leads to death.
Passive euthanasia is legal in India. On 7 March 2011 the Supreme Court of India
legalised passive euthanasia by means of the withdrawal of life support to patients in
a permanent vegetative state. The decision was made as part of the verdict in a case
involving Aruna Shanbaug, who had been in a vegetative state for 42 years at King
Edward Memorial Hospital until her death recently in 2015.

17. Solution: b)
A strategic partnership is a long-term interaction between two countries based on
political, economic, social and historical factors. Such a partnership manifests itself
http://www.insightsonindia.com INSIGHTS

Page 5


Insights Mock Tests 2015: Test 25 Solutions

in a variety of relationships. India has signed strategic partnerships with more than
30 countries.
You can read this beautiful article for a better understanding.
http://thediplomat.com/2013/11/why-does-india-have-so-many-strategic-partnersand-no-allies/

18.Solution: d)
BoP is a statement that summarizes an economys transactions with the rest of the
world for a specified time period. The balance of payments, also known as balance of
international payments, encompasses all transactions between a countrys residents
and its nonresidents involving goods, services and income; financial claims on and
liabilities to the rest of the world; and transfers such as gifts. The balance of
payments classifies these transactions in two accounts the current account and the
capital account. The current account includes transactions in goods, services,
investment income and current transfers, while the capital account mainly includes
transactions in financial instruments. An economys balance of payments
transactions and international investment position (IIP) together constitute its set of
international accounts. All the four options form a part of the capital account.

19. Solution: c)
Romesh Chandra Dutt, a retired ICS officer, published The Economic History of
India at the beginning of the 20th century in which he examined in minute detail the
entire economic record of colonial rule since 1757.

20.

Solution: a)

The Triffin dilemma or paradox is the conflict of economic interests that arises
between short-term domestic and long-term international objectives when a national
currency also serves as a world reserve currency. The dilemma of choosing between
these objectives was first identified in the 1960s by Belgian-American economist
Robert Triffin. He pointed out that the country whose currency, being the global
reserve currency, foreign nations wish to hold, must be willing to supply the world
with an extra supply of its currency to fulfill world demand for these foreign
exchange reserves, and thus cause a trade deficit.
The use of a national currency, e.g., the U.S. dollar, as global reserve currency leads
to tension between its national and global monetary policy. This is reflected in
http://www.insightsonindia.com INSIGHTS

Page 6


Insights Mock Tests 2015: Test 25 Solutions

fundamental imbalances in the balance of payments, specifically the current account:


some goals require an overall flow of dollars out of the United States, while others
require an overall flow of dollars into the United States.

21. Solution: c)
Read this article to understand the context of the question. Explanation has already
been given in the statements.
http://www.newindianexpress.com/business/news/Emerging-Markets-NeedStronger-Safety-Net-to-Grow-Rajan/2015/05/23/article2828352.ece
Underline the term prisoners dilemma in the article.

22.

Solution: c)

CGA is the principal Accounts Adviser to the Union Government. He is responsible


for maintaining and establishing a technically sound management accounting
system. Every month he prepares a critical analysis of revenues, expenditures,
borrowings and the deficit for Union Finance Minister. He also prepares annual
Union Finance Accounts and Appropriation Accounts (Civil) for presentation to the
Parliament.
Union Ministry of Finance has recently appointed M J Joseph as the Controller
General of Accounts (CGA). He will succeed Jawahar Thakur, who was recently
appointed as OSD (Accounting Reforms) in the office of CGA.

23.

Solution: d)

Royal Dutch Shell plc commonly known as Shell, is anAnglo


Dutch multinational oil and gas company headquartered in the Netherlands and
incorporated in the United Kingdom. Created by the merger of Royal Dutch
Petroleum and UK-based Shell Transport & Trading, it is the fourth largest company
in the world as of 2014, in terms of revenue, and one of the six oil and gas
"supermajors".
As of January 2013 the largest shareholder is Capital Research Global Investors with
9.85% ahead of BlackRock in second with 6.89%.
Recently the United Steelworkers union striked against Royal Dutch Shell Plc at nine
U.S. oil and chemical plants. Up until recently oil prices had undergone steep drops

http://www.insightsonindia.com INSIGHTS

Page 7


Insights Mock Tests 2015: Test 25 Solutions

since June, but the union's action as part of the supply chain caused the market price
to soar more than eight percent.

24.

Solution: a)

Warm-blooded animals generate their own heat and maintain a body temperature
independent of the environment. They are also known as endotherms. It should be
noted most of mammals and birds are warm-blooded animals.
Opah or moonfish has ability to circulate heated blood throughout its body which
makes it first fish known to be fully warm-blood.
It can internally generate heat through constant flapping of wing-like pectoral fins.
Its unique structure also help to prevent this heat from being lost to the environment
as its gills lets warm blood to heat up cold blood returning from the gills respiratory
surface. This generated heat has an average muscle temperature about 4 to 50 C
above the surrounding water temperature.

25.

Solution: d)

http://www.nearfieldcommunication.org/about-nfc.html
http://www.in.techradar.com/news/phone-and-communications/What-is-NFCand-why-is-it-in-your-phone/articleshow/38919446.cms

26.

Solution: a)

Union Government formally launched Upgrading the Skills and Training in


Traditional Arts/Crafts for Development (USTAAD) Scheme.
The Scheme aims at upgrading Skills and Training of minority communities by
preservation of traditional ancestral Arts and Crafts. It also envisages boosting the
skill of craftsmen, weavers and artisans who are already engaged in the traditional
ancestral work. Under the scheme, assistance will be provided to traditional artisans
to sell their products in order to make them more compatible with modern markets.
It is fully funded by Union Government and Union Ministry of Minority Affairs is
nodal agency in implementing it.
27.

Solution: a)

The name Jainism derives from the Sanskrit verb ji, to conquer. It refers to the
ascetic battle that, it is believed, Jain renunciants (monks and nuns) must fight
http://www.insightsonindia.com INSIGHTS

Page 8


Insights Mock Tests 2015: Test 25 Solutions

against the passions and bodily senses to gain omniscience and purity of soul or
enlightenment. The most illustrious of those few individuals who have achieved
enlightenment are called Jina (literally, Conqueror), and the traditions monastic
and lay adherents are called Jain (Follower of the Conquerors), or Jaina.

28.

Solution: c)

Mahatma Gandhi led the Civil Disobedience Movement that was launched in the
Congress Session of December 1929. The aim of this movement was a complete
disobedience of the orders of the British Government. During this movement it was
decided that India would celebrate 26th January as Independence Day all over the
country. On 26th January 1930, meetings were held all over the country and the
Congress tricolour was hoisted. The British Government tried to repress the
movement and resorted to brutal firing, killing hundreds of people. Thousands were
arrested along with Gandhiji and Jawaharlal Nehru.
But the movement spread to all the four corners of the country Following this, Round
Table Conferences were arranged by the British and Gandhiji attended the second
Round Table Conference at London. But nothing came out of the conference and the
Civil Disobedience Movement was revived.
During this time, Bhagat Singh, Sukhdev and Rajguru were arrested on the charges
of throwing a bomb in the Central Assembly Hall (which is now Lok Sabha) in Delhi,
to demonstrate against the autocratic alien rule. They were hanged to death on
March 23, 1931.

29.

Solution: c)

The Moro people or Bangsamoro are a population of indigenous Muslims in the


Philippines, forming the largest non-Catholic group in the country, and comprising
about 5% of the total Philippine population.
The Moro Insurgency in the Philippines is an ongoing insurgency in the
predominantly-Muslim Southern Philippines. Recently it derailed a peace treaty and
caused millions of dollars of revenue loss to Philippines government.

30.

Solution: d)

The effect can be seen in the drawing up of liquids between the hairs of a paintbrush, in a thin tube, in porous materials such as paper, in some non-porous
materials such as liquefied carbon fiber, or in a cell. It occurs because
http://www.insightsonindia.com INSIGHTS

Page 9


Insights Mock Tests 2015: Test 25 Solutions

of intermolecular forces between the liquid and surrounding solid surfaces. If the
diameter of the tube is sufficiently small, then the combination of surface
tension (which is caused by cohesion within the liquid) and adhesive forces between
the liquid and container act to lift the liquid. In short, the capillary action is due to
the pressure of cohesion and adhesion which cause the liquid to work against gravity

31. Solution: a)
Mangroves are defined as assemblages of salt tolerant trees and shrubs that grow in
the intertidal regions of the tropical and subtropical coastlines. They grow
luxuriantly in the places where freshwater mixes with seawater and where sediment
is composed of accumulated deposits of mud.
Inter-tidal zone is the space on the shores between the areas of high tide and low
tide. It is very rich in nutrients and hence supports the growth of Mangroves the best.

32.

Solution: c)

http://en.wikipedia.org/wiki/Global_strategic_petroleum_reserves

33.

Solution: a)

The troposphere is thicker over the equator than the poles because the equator is
warmer. Heat differential on the planet's surface causes convection currents to flow
from the equator to the poles. This implies that the warmer the weather, the thicker
is the troposphere. Thus the simple reason is thermal expansion of the atmosphere at
the equator and thermal contraction near the poles.

34.

Solution: d)

There are both natural and human sources of methane emissions. The main natural
sources include wetlands, termites and the oceans. Natural sources create 36% of
methane emissions. Important human sources come from landfills, livestock
farming, as well as the production, transportation and use of fossil fuels. Humanrelated sources create the majority of methane emissions, accounting for 64% of the
total
Methane producing microbes living in the ocean create these emissions. This creates
10% of natural methane emissions. Globally, oceans create 19 million tonnes of
methane annually.
http://www.insightsonindia.com INSIGHTS

Page 10


Insights Mock Tests 2015: Test 25 Solutions

The majority of oceanic methane emissions gets produced in deeper sediment layers
of productive coastal areas.

35.

Solution: c)

Supreme Court (SC) on 13 May 2015 issued a slew of guidelines on regulation of


public advertisements. This guidelines seeks to prevent ruling politicians at all levels
from misusing tax payers money to enhance their image or of their parties as it is
against democratic principles.
Only photographs of the President, Prime Minister (PM) and Chief Justice of India
(CJI) can feature in the government advertisements with their prior approval.
Besides, the photos of late national leaders like Mahatma Gandhi and Jawaharlal
Nehru can be used in these advertisements. Henceforth, official advertisements
should not carry the name of any political party, political symbol, logo or flag. In
order to regulate the issue of public advertisement and to keep check on framed
guidelines to prevent its violation, Union government must constitute a threemember committee.
36.

Solution: d)

Geo-engineering is still being debated. It includes activites like cloud seeding,


installing large sunlight reflecting mirrors, feeding oceans with more minerals to
increase phytoplanktons and carbon sinking. But these are not part of ecological
engineering.
For the rest just scan through
http://en.wikipedia.org/wiki/Ecological_engineering#Overview

37.

Solution: c)

The boiling point of milk (100.17C) is slightly higher than of water (100C). When
milk is heated, its relatively less dense constituents like the fat/proteins begin to float
on top cream layer. So, milk vapor cant pass through this upper layer. Due to this, at
boiling point milk will overflow due to large quantity of milk vapor rupturing the
cream layer. However, this does not happen in a Milk cooker which is a doublewalled vessel. Its gap has water, which boils before milk. So, inner vessel remains at
100C. So, cream layer forms but milk vapor doesnt form. Thats why milk does not
overflow in a milk cooker.

http://www.insightsonindia.com INSIGHTS

Page 11


Insights Mock Tests 2015: Test 25 Solutions

38.

Solution: a)

Avalokiteshvara, ( Sanskrit: avalokita, looking on; ishivara, lord) Chinese


Guanyin, Japanese Kannon, the bodhisattva (Buddha-to-be) of infinite compassion
and mercy, possibly the most popular of all Buddhist deities, beloved throughout the
Buddhist world. He supremely exemplifies the bodhisattvas resolve to postpone his
own Buddhahood until he has helped every being on earth achieve emancipation.
Bodhisattva, (Sanskrit), Pali bodhisatta (one whose goal is awakening), in
Buddhism, one who seeks awakening (bodhi)hence, an individual on the path to
becoming a buddha.
http://en.wikipedia.org/wiki/Avalokite%C5%9Bvara

39.

Solution: c)

Elements of sustainable agriculture includepermaculture, agroforestry, mixed


farming, multiple cropping, and crop rotation.
Permaculture is a systems approach. It has many branches that include but are not
limited toecological design, ecological engineering, environmental
design, construction and integrated water resources management that
develops sustainable architecture, regenerative and selfmaintainedhabitat and agricultural systems modeled from natural ecosystems.
http://en.wikipedia.org/wiki/Permaculture#Common_practices

40.

Solution: c)

http://timesofindia.indiatimes.com/home/science/Nasas-Messenger-spacecraftmakes-crashing-finale-into-Mercury/articleshow/47119133.cms
http://www.space.com/29281-messenger-spacecraft-mercury-crash.html

41. Solution: c)
Internet.org is a partnership between social networking services
company Facebook and six mobile phone companies
(Samsung, Ericsson, MediaTek, Opera Software, Reliance and Qualcomm) that plans
to bring affordable access to selected Internet services to less developed countries by
increasing efficiency, and facilitating the development of new business models
around the provision of Internet access.
http://www.insightsonindia.com INSIGHTS

Page 12


Insights Mock Tests 2015: Test 25 Solutions

It has been criticized for violating net neutrality and favoring Facebook's own
services over its rivals. An Indian journalist, in his reply to Mark Zuckerberg's article
defending Internet.org in India, criticized Internet.org as "being just a Facebook
proxy targeting India's poor" as it provides restricted Internet access to Reliance
Telecom's subscribers in India. Until April 2015, Internet.org users could access (for
free) only a few websites, and Facebook's role as gatekeeper in determining what
websites were in that list was criticised for violating net neutrality.

42.

Solution: c)

Every union territory is administered by the President acting through an


administrator appointed by him. An administrator of a union territory is an agent of
the President and not head of state like a governor. He does not have absolute veto
power. Only in few cases like in concurrent list legislations, he needs to refer the bill
to the President of India. At most, he can delay a bill.

43.

Solution: b)

The council of ministers headed by the chief minister aid and advise the lt. governor
in the exercise of his functions except in so far as he is required to act in his
discretion. In the case of difference of opinion between the lt. governor and his
ministers, the lt. governor is to refer the matter to the president for decision and act
accordingly.
When a situation arises in which the administration of the territory cannot be carried
on in accordance with the above provisions, the president can suspend their (above
provisions) operation and make the necessary incidental or consequential provisions
for administering the territory.

44.

Solution: c)

The electoral college of the President consists of : a) Elected members of both the
houses of the parliament b) Elected members of Legislative assemblies of the states
and c) Elected members of assemblies of Delhi and Pudducherry.

45.

Solution: d)

The Presidents powers with respect to the Parliament are restricted by:

http://www.insightsonindia.com INSIGHTS

Page 13


Insights Mock Tests 2015: Test 25 Solutions

The council of Ministers can be dismissed only when it has lost the confidence
of the house.
Lok Sabha can be dissolved only when no government can be formed.
And, there are only three grounds to issue an ordinance:
o The parliament should not be in session, whether one or both the houses
o The subject must be very urgent that any delay in passing such a bill may
be against public interest, like in the recent case of Criminal law
(amendment) ordinance, 2013.
o The council of ministers must advice the President to do so.

46.

Solution: b)

http://www.britannica.com/EBchecked/topic/225187/Gandhara-art

47.

Solution: c)

NITI Ayog replaces Planning Commission as another executive body. It is not even a
statutory body.
PMO is also an executive body set up under an order of cabinet.
Cabinet however finds mention under the constitution. Also, the constitution
mentioned about the important parliamentary committee like PAC.

48.

Solution: a)

On 22 March 1911, a royal proclamation announced that the Durbar would be held in
December to commemorate the coronation in Britain a few months earlier of King
George Vand Queen Mary and allow their proclamation as Emperor and Empress of
India. Without public forewarning, the announcement of the move of India's capital
from Calcutta to Delhi was also made at the Durbar.
Moreover, partition of Bengal was annlued seeing popular opposition; and Bihar,
Orrisa and Chotanagpur were separated from Bengals jurisdiction.

49.

Solution: b)

Provision for Address by the Head of State to Parliament goes back to the year 1921
when the Central Legislature was set up for the first time under the Government of
India Act, 1919.
http://www.insightsonindia.com INSIGHTS

Page 14


Insights Mock Tests 2015: Test 25 Solutions

Today this can be seen when a) the house assembles for the first time; b) when a new
government is sworn in; c) before the budget is presented etc.

50.

Solution: c)

http://www.nasa.gov/audience/forstudents/k-4/stories/what-is-a-spacewalkk4.html#.VWXp8a2qqko

51. Solution: b)
These soils are red to reddish yellow in colour and low in N, P, K, lime and magnesia.
These soils are formed in-situ under conditions of high rainfall with alternation dry
and wet periods. On account of heavy rainfall there is an excessive leaching of soil
colloids and silica hence the soils are porous.
Due to high rainfalls, the organic matter created humic acid which aids the process of
Desilication.

52.

Solution: d)

The origin of India monsoon has traditionally been ascribed to the differential
heating and cooling of the continent of Asia and the Indian Ocean and Monsoon were
considered large scale land and sea breezes. This simplistic model fails to explain
many of the complex issues associated with monsoon.
According to recent dynamic theories, the Monsoon is a highly complex phenomenon
that happens due to seasonal migration of planetary winds and pressure belts
following the sun and is closely influenced by the middle and upper tropospheric
circulation. The occurrence and changing position of jet stream influence the origin
and development of monsoons.
The presence of Himalayan ranges and Tibetan Plateau also have a close bearing on
the origin of Monsoon. The jet streams that lie about 12 km in the troposphere are
bifurcated by the Himalayan Mountains and the Tibetan Plateau. Moreover other
global climatic phenomenon like El Nino and La Nina also affect rainfall pattern in
Monsoon.
And it is ultimately circulated with the trade winds.

http://www.insightsonindia.com INSIGHTS

Page 15


Insights Mock Tests 2015: Test 25 Solutions

53.

Solution: b)

Tropical Evergreen Rain Forests grow in areas where rainfall is more than 200 cm.
They are mainly found on the slopes of the Western Ghats and the north-eastern
regions of Arunachal Pradesh, Meghalaya, Assam, Nagaland, the Tarai areas of the
Himalayas and the Andaman groups of Islands. The trees in these forests never shed
their leaves all at a time in any part of the year. Under humid tropical condition, subsoil water never dries up completely. So that during dry season, trees in these forests
do not shed their leaves due to lack of sub-soil water supply. The trees in these belts
have dense growth. Important varieties of trees are sisthu, chaplash, rosewood,
mehogeny, bamboos, garjan and sandal wood.

54.

Solution: a)

The Badami cave temples are a complex of temples located at Badami, a town in the
Bagalkot District in the north part of Karnataka, India. They are considered an
example of Indian rock-cut architecture, especially Badami Chalukya Architecture
initiated during the 6th century. Badami previously known as Vapati Badami, the
capital of the Early Chalukyas, who ruled much of Karnataka from middle of 6th until
the middle of 8th centuries is suitated on the west bank of artificial lake filed with
greenish water, dammed by an earthen wall faced with stone steps.

55.

Solution: b)

Rohingya people speak Rohingya language.

Regions with significant populations

Burma (Arakan), Bangladesh, Malaysia,Pakistan, Saudi


Arabia, Thailand, Indonesia,India

Burma

735,000800,000

Saudi Arabia

400,000

Bangladesh

300,000 - 500,000

Pakistan

200,000

http://www.insightsonindia.com INSIGHTS

Page 16


Insights Mock Tests 2015: Test 25 Solutions

Thailand

100,000

Malaysia

40,070[

56.

Solution: d)

All posts whether that of a MP, MLA, Speaker, Vice-President, President etc. are
concerned with legislative bodies/business in India.
FPTP is used for electing MPs. PR system is used for electing President. Nomination
is used by President for nominating MPs to LS and RS.

57.Solution: b)
Deep Space Climate Observatory (DSCOVR) (formerly known as Triana, unofficially
known as GoreSat) is a NOAA Earth observation and space weather satellite
launched by SpaceX on a Falcon 9 launch vehicle on 11 February 2015 from Cape
Canaveral.
It was originally developed as a NASA satellite proposed in 1998 by then-Vice
President Al Gore for the purpose of Earth observation. It is intended to be
positioned at the Sun-Earth L1 Lagrangian point, 1,500,000 km (930,000 mi) from
Earth, to monitor variable solar wind condition, provide early warning of
approaching coronal mass ejections and observe phenomena on Earth including
changes in ozone, aerosols, dust and volcanic ash, cloud height, vegetation cover and
climate. At this location it will have a continuous view of the Sun and the sunlit side
of the Earth. The satellite is planned to orbit the Sun-Earth L1 point in a six-month
period, with a spacecraft-Earth-Sun angle varying from 4 to 15 degrees. It will take
full-Earth pictures about every two hours and be able to process them faster than
other Earth observation satellites.

58.

Solution: c)

http://www.insightsonindia.com INSIGHTS

Page 17


Insights Mock Tests 2015: Test 25 Solutions

Question Hour: The first hour of every sitting of Parliament is generally reserved
for the asking and answering of questions.
Zero Hour: The time immediately following the Question Hour has come to be
known as "Zero Hour". It starts at around 12 noon (hence the name) and members
can, with prior notice to the Speaker, raise issues of importance during this time.
Typically, discussions on important Bills, the Budget, and other issues of national
importance take place from 2pm onwards.

59.

Solution: a)

The constitution lays down three conditions without fulfilling which "No money out
of the Consolidated Fund of India shall be appropriated" or withdrawn. These
conditions are:
o the appropriation must be in accordance with a law authorising appropriation
of money from the Consolidated Fund; and
o the appropriation should be for a "purpose provided in this Constitution"; and
o the appropriation must be made "in the manner provided in this
Constitution" as the manner for appropriation of money from the
Consolidated Fund of India (which is that of a regular budget)

60.

Solution: d)

http://www.insightsonindia.com INSIGHTS

Page 18


Insights Mock Tests 2015: Test 25 Solutions

The governor holds an independent office under the constitution. Conventions like
consulting the CM; Governor being from a different state etc. are not mentioned in
the constitution. It is solely on the discretion of the President on how he wants to
appoint the Governor (given some qualifications mentioned in the constitution of
India)

61. Solution: d)
When a bill is passed by the legislative assembly and transmitted to the legislative
council, the latter has four alternatives before it:
o it may pass the bill as sent by the assembly (i.e., without amendments);
o it may pass the bill with amendments and return it to the assembly for
reconsideration;
o it may reject the bill altogether; and
o it may not take any action and thus keep the bill pending.
If the council passes the bill without amendments or the assembly accepts the
amendments suggested by the council, the bill is deemed to have been passed by both
the Houses and the same is sent to the the governor for his assent. On the other
hand, if the assembly rejects the amendments suggested by the council or the council
rejects the bill altogether or the council does not take any action for three months,
then the assembly may pass the bill again and transmit the same to the council. If the
council rejects the bill again or passes the bill with amendments not acceptable to the
assembly or does not pass the bill within one month, then the bill is deemed to have
been passed by both the Houses in the form in which it was passed by the assembly
for the second time. Therefore, the ultimate power of passing an ordinary bill is
vested in the assembly

62.

Solution: c)

http://www.unctad.info/en/Infocomm/Agricultural_Products/Caoutchouc/Crop/Pl
ant-profile-Hevea-brasiliensis/

http://en.wikipedia.org/wiki/Natural_rubber

63.

Solution: c)

http://www.insightsonindia.com INSIGHTS

Page 19


Insights Mock Tests 2015: Test 25 Solutions

The scheme is being implemented by the Ministry of Labour and Employment


First statement is wrong as it is still a proposal put forward recently by the new
government at the centre.
http://timesofindia.indiatimes.com/india/Govt-proposes-complete-ban-on-childlabour-till-14-years/articleshow/47195609.cms
http://labour.gov.in/content/division/about-child-labour.php

64.

Solution: d)

The Chief Justice of India must consult a collegium of judges. After this the President
of India gives orders for the transfer of judges, even while the high courts come
under the administrative control of the SC.
The state governments have no control over the HC judges. Only the Governor is
consulted before appointing the judges and CJI of the HCs.

65.

Solution: a)

The Bill seeks to amend the Benami Transactions (Prohibition) Act, 1988 by adding
additional provisions that provides for stringent measures against violators in order
to curb and check the generation of black money in the country. It also adds
provisions for attachment and confiscation of benami properties and imposes fine
with imprisonment. The Bill also has provision for prosecution and aims to act as a
major avenue for blocking benami property, which leads to generation and holding of
black money especially in real estate.

66.

Solution: c)

There had been a lot of discussion and lobbying on MAT both in the budget and in
the recently setup committee on MAT. Union Government has recently constituted
Committee headed by Justice A.P. Shah to look into the issue of Minimum Alternate
Tax (MAT) on Foreign Institutional Investors (FIIs).
What is MAT?
In the past, a large number of companies showed book profits on their profit and loss
account and at the same time distributed huge dividends. However, these companies
http://www.insightsonindia.com INSIGHTS

Page 20


Insights Mock Tests 2015: Test 25 Solutions

didnt pay any tax to the government as they reported either nil or negative income
under provisions of the Income-Tax Act. These companies were showing book profits
and declaring dividends to their shareholders but were not paying any tax. These
companies are popularly known as zero tax companies.
More exemptions, deductions, and other incentives under the Income-Tax Act in the
form of liberal rates of depreciation gradually led to the emergence of zero tax
companies, which in spite of having high book profit were able to reduce their
taxable income to nil. In order to bring such companies under the I-T net, Section
115JA was introduced from assessment year 1997-98.
Then, all companies having book profits under the Companies Act shall have to pay a
minimum alternate tax at 18.5%. MAT is a way of making companies pay minimum
amount of tax. It is applicable to all companies except those engaged in
infrastructure and power sectors.

67.

Solution: a)

Read just the introduction from the link below it includes right speech, right
conduct, right livelihood, right effort, right view, right intent, right mindfulness and
right concentration.
http://en.wikipedia.org/wiki/Noble_Eightfold_Path

68.

Solution: b)

http://www.britannica.com/EBchecked/topic/351624/Luni-River

69.

Solution: c)

Varishtha Pension Bima Yojana (VPBY) will benefit the vulnerable section of society
with limited resources as it will provide monthly pension ranging from Rs 500/ to Rs
5,000/ per month to senior citizens of the country. This scheme is for the benefit of
citizens aged 60 years and above.
VPBY is like reverse of a normal insurance policy as in case of VPBY, the beneficiary
gets an income at the overall rate of 9.38 % per annum on their deposits as they are
being paid on monthly basis.

http://www.insightsonindia.com INSIGHTS

Page 21


Insights Mock Tests 2015: Test 25 Solutions

Under this revived Varishtha Pension Bima Yojana (VPBY), the senior citizens would
get pension on fixed basis either on yearly or monthly basis which will provide social
security to senior citizens.
LIC of India has been given the sole privilege to operate this scheme.

70.

Solution: d)

The last great king of the Gupta was Skanda Gupta was ascended the throne about
455 A.D. Even during the later years of Kumar Gupta's reign, the empire was
attacked by a tribe called Pushyamitra but it was repulsed, And immediately after the
accession of Skanda Gupta, Hunas made inroads, but they too were repelled.
However, fresh waves of Invaders arrived and shattered the fabric of the Gupta
Empire. Although in the beginning the Gupta king Skanda Gupta tried effectively to
stem the march of the Hunas into India, his successors proved to be weak and could
not cope with the Huna invaders, who excelled in horsemanship and who possibly
used stirrups made of metal,
The Gupta empire was further undermined by the rise of the feudatories. The
governors appointed by the Gupta kings in north Bengal and their feudatories in
Samatata or south-east Bengal broke away from the Gupta control.
After the middle of the fifth century the Gupta kings made desperate attempts to
maintain their gold currency by reducing the content of pure gold in it. The loss of
western India complete by the end of the fifth century, must have deprived the
Gutpas of the rich revenues from trade and commerce and crippled them
economically, and the princes of Thaneswar established their power in Haryana and
then gradually moved on to Kanauj.

71. Solution: c)
The Dn-i Ilh was a syncretic religion propounded by the Mughal emperor Akbar
the Great in 1582 AD, intending to merge the best elements of the religions of his
empire, and thereby reconcile the differences that divided his subjects. The elements
were primarily drawn from Islam and Hinduism, but some others were also taken
from Christianity, Jainism and Zoroastrianism.
Din-i-Ilahi prohibits lust, sensuality, slander and pride, considering them sins. Piety,
prudence, abstinence and kindness are the core virtues. The soul is encouraged to
purify itself through yearning of God. Celibacy is respected and the slaughter of
animals is forbidden.

http://www.insightsonindia.com INSIGHTS

Page 22


Insights Mock Tests 2015: Test 25 Solutions

72.

Solution: d)

In summer and winter, water across layers of lake hardly mixes. So, either the upper
layer or deep water is cold or warm. Hence, there is little churning of oxygen and
nutrients. But, due to similar water temperatures at both surface and under water in
autumn and spring, water tends to mix.

73.

Solution: d)

Various Committees on Panchayati Raj :

74.

Balwant Rai Mehta : Estd 1957


V.T.Krishnammachari : 1960
Takhatmal Jain Study Group: 1966
Ashok Mehta Committee : 1977
G.V.K Rao committee :1985
Sarkaria Commission : 1986
Dr.L.M.Singhvi Committee:1986

Solution: c)

Rotation of earth is maximum at the equator and minimum at the poles. The rotation
of the earth pushes objects away from its surface. The gravitational force pulls it
inwards. The difference is the weight of the object. So the higher the rotation speed of
the earth, the lesser weight you will experience.
The period of revolution of the earth does not concern itself with gravity and weight.
Moreover, if the radius of the earth changes, the gravitational force acting on you will
change, hence your weight.

75.Solution: a)
As traditional breeding techniques failed to keep pace with demand and to provide
sufficiently fast and efficient systems for crop improvement, another technology
called tissue culture got developed. It was learnt by scientists, during 1950s, that
whole plants could be regenerated from explants, i.e., any part of a plant taken out
and grown in a test tube, under sterile conditions in special nutrient media.

http://www.insightsonindia.com INSIGHTS

Page 23


Insights Mock Tests 2015: Test 25 Solutions

This method of producing thousands of plants through tissue culture is called micropropagation. Each of these plants will be genetically identical to the original plant
from which they were grown, i.e., they are somaclones. Many important food plants
like tomato, banana, apple, etc., have been produced on commercial scale using this
method.

76.

Solution: a)

Magna Carta (Latin for "the Great Charter"), also called Magna Carta Libertatum
(Latin for "the Great Charter of the Liberties"), is a charter agreed by King John of
England at Runnymede, near Windsor, on 15 June 1215. First drafted by the
Archbishop of Canterbury to make peace between the unpopular King and a group of
rebel barons, it promised the protection of church rights, protection for the barons
from illegal imprisonment, access to swift justice, and limitations on feudal payments
to the Crown, to be implemented through a council of 25 barons. Neither side stood
behind their commitments, and the charter was annulled by Pope Innocent III,
leading to the First Barons' War.
The four remaining original copies of the Magna Carta go on public display at the
British Library in London to commemorate the 800th anniversary signing of the
document in June 1215.

77. Solution: d)
Refer to this link. It has been explained very well. It mentions other diseases too.
http://www.hindustantimes.com/india-news/15-diseases-india-can-stamp-out-byimproving-sanitation/article1-1270690.aspx

78.

Solution: c)

http://en.wikipedia.org/wiki/UNESCO

79.

Solution: d)

It is basically rational and non-populist public expenditure that does not rely solely
on cutting public expenditure (but also relies on increasing revenue). Refer to
http://www.thehindu.com/business/budget/fiscal-prudence-needed-to-puteconomy-on-track/article6191798.ece

http://www.insightsonindia.com INSIGHTS

Page 24


Insights Mock Tests 2015: Test 25 Solutions

80.

Solution: a)

Some LED lamps are made to be a directly compatible drop-in replacement for
incandescent or fluorescent lamps. An LED lamp packaging may show
the lumen output, power consumption in watts, color temperature in kelvins or
description (e.g. "warm white"), operating temperature range, and sometimes the
equivalent wattage of an incandescent lamp of similar luminous output.
Most LEDs do not emit light in all directions, and their directional characteristics
affect the design of lamps. Although through the progression of time,
omnidirectional lamps are becoming more common, allowing for 360 light spread.
The light output of single LEDs is less than that of incandescent and compact
fluorescent lamps; in most applications multiple LEDs are used to form a lamp,
although high-power versions (see below) are becoming available.
LED chips need controlled direct current (DC) electrical power; an
appropriate circuit is required to convert alternating current from the supply to the
regulated low voltage direct current used by the LEDs. LEDs are adversely affected
by high temperature, so LED lamps typically include heat dissipation elements such
as heat sinks and cooling fins.

81.Solution: b)
CRR is the amount of cash that banks keep with RBI. If CRR is more, they have less
to give as loan. This reduced money supply in the market.
In addition to this higher interest rates will further discourage investments.
Therefore, the net result will be a tight monetary policy leading to lower investments.

82.

Solution: b)

These are loans raised by the government from the public which are called market
borrowings, borrowing by the government from the Reserve Bank and commercial
banks and other financial institutions through the sale of treasury bills, loans
received from foreign governments and international organisations, and recoveries
of loans granted by the central government. Other items include small savings (PostOffice Savings Accounts, National Savings Certificates, etc), provident funds and net
receipts obtained from the sale of shares in Public Sector Undertakings (PSUs).

83.

Solution: d)

http://www.insightsonindia.com INSIGHTS

Page 25


Insights Mock Tests 2015: Test 25 Solutions

In the last four decades (1972-2010), people have moved from self-employment and
regular salaried employment to casual wage work. Yet self-employment continues to
be the major employment provider. Scholars call this process of moving from selfemployment and regular salaried employment to casual wage work as casualisation
of workforce. This makes the workers highly vulnerable due to lack of proper legal
protection, tough work conditions and social security schemes.
The last statement is exaggerated. There is no concrete evidence to show that it leads
to lower economic growth of a country.

84.

Solution: b)

http://en.wikipedia.org/wiki/Biosphere_reserves_of_India

http://en.wikipedia.org/wiki/Nokrek_National_Park

85.

Solution: d)

Per capita Income = NNP = GDP Depreciation + Net Income from abroad.
As you vary these three factors, NNP will change.
Depreciation can be logical or artificial ( state dictated). Though economies fix their
rates of depreciation, international bodies fix their own rates of depreciation while
ranking nations on the basis of national income.

86.

Solution: c)

The oil and gas industry is usually divided into three major
sectors: upstream, midstream and downstream. The upstream oil sector is also
commonly known as the exploration and production (E&P) sector.
The upstream sector includes the searching for potential underground or
underwater crude oil and natural gas fields, drilling of exploratory wells, and
subsequently drilling and operating the wells that recover and bring the crude oil
and/or raw natural gas to the surface.
The midstream sector involves the transportation (by pipeline, rail, barge, oil
tanker or truck), storage, and wholesale marketing of crude or refined petroleum
products.
http://www.insightsonindia.com INSIGHTS

Page 26


Insights Mock Tests 2015: Test 25 Solutions

The downstream sector commonly refers to the refining of petroleum crude oil and
the processing and purifying of raw natural gas, as well as the marketing and
distribution of products derived from crude oil and natural gas.

87.

Solution: b)

http://steel.gov.in/overview.htm

88.

Solution: a)

Ocean currents are of two types viz. Surface Currents and Deep Currents. Surface
currents affect surface water above the pycnocline (<10% of ocean water). These
currents are primarily driven by major wind belts. The Deep currents affect deep
water below pycnocline (90% of ocean water) and are primarily driven by density
differences. The deep currents are larger and slower than surface currents.

89.

Solution: d)

Tides are a very complex phenomenon. For any particular location, their height and
fluctuation in time depends to varying degrees on the location of the Sun and the
Moon, and to the details of the shape of the beach, coastline, coastline depth and
prevailing ocean currents.
The tidal bulge of the Moon follows along the path on the earth's surface which
intersects with the orbital plane of the Moon. This plane is tilted about 23 degrees
with respect to the equatorial plane of the earth. The result is that near the equator,
the difference between high tide and low tide is actually rather small, compared to
other latitudes. To see this, draw a circle inscribed in an ellipse, with the major axis
of the ellipse rotated by 23 degrees with respect to the circle's horizontal diameter.
Now measure the height of the elliptical contour just above the 'equator' of the circle.
You will see that it is quite small compared to other positions on earth, particularly at
latitudes of 23 degrees or so. Even larger differences can occur depending on the
shape of a bay or inlet or continental shelf.

90.

Solution: c)

Subduction is responsible for high rates of volcanism, earthquakes, and mountain


building. When the large pieces of material on the subducting plate are pressed into
the overriding plate, it results in the Orogeny or Mountain formation. These areas
are subject to many earthquakes.
http://www.insightsonindia.com INSIGHTS

Page 27


Insights Mock Tests 2015: Test 25 Solutions

In the ocean bed, earthquake can lead to a s tsunami because of displacement of a


large quantity of water in a very short period of time.

91. Solution: c)
There are different ways of heating and cooling of the atmosphere.
The earth after being heated by insolation transmits the heat to the atmospheric
layers near to the earth in long wave form. The air in contact with the land gets
heated slowly and the upper layers in contact with the lower layers also get heated.
This process is called conduction.
Conduction takes place when two bodies of unequal temperature are in contact with
one another, there is a flow of energy from the warmer to cooler body. The transfer of
heat continues until both the bodies attain the same temperature or the contact is
broken. Conduction is important in heating the lower layers of the atmosphere.
The air in contact with the earth rises vertically on heating in the form of currents
and further transmits the heat of the atmsphere. This process of vertical heating of
the atmosphere is known as convection. The convective transfer of energy is confined
only to the troposphere.
The transfer of heat through horizontal movement of air is called advection.
Horizontal movement of the air is relatively more important than the vertical
movement.

92.

Solution: d)

Just scan through this link. No need to read in detail. Just grasp the central idea of
the philosophy. It is important for UPSC.
http://en.wikipedia.org/wiki/M%C4%ABm%C4%81%E1%B9%83s%C4%81#Epistem
ology

93.

Solution: c)

http://en.wikipedia.org/wiki/Cyclone#Tropical
Although their effects on human populations can be devastating, tropical cyclones
can also relieve drought conditions. They also carry heat and energy away from the
tropics and transport it toward temperate latitudes, which makes them an important

http://www.insightsonindia.com INSIGHTS

Page 28


Insights Mock Tests 2015: Test 25 Solutions

part of the global atmospheric circulation mechanism. As a result, tropical cyclones


help to maintain equilibrium in the Earth's troposphere.

94.

Solution: c)

Archaeological records provide no immediate answers for a centre of power or for


depictions of people in power in Harappan society. But, there are indications of
complex decisions being taken and implemented. For instance, the extraordinary
uniformity of Harappan artefacts as evident in pottery, seals, weights and bricks.
These are the major theories:

There was a single state, given the similarity in artefacts, the evidence for
planned settlements, the standardised ratio of brick size, and the
establishment of settlements near sources of raw material.

There was no single ruler but several: Mohenjo-daro had a separate ruler,
Harappa another, and so forth.

Harappan society had no rulers, and everybody enjoyed equal status.

95.

Solution: d)

The conditions for tropical cyclones have been covered in NCERT textbooks but this
link below explains it even better.
http://www.aoml.noaa.gov/hrd/tcfaq/A15.html

96.

Solution: a)

In Bengal, the indigo planters, nearly all Europeans, exploited the local peasants by
forcing them to grow indigo on their lands instead of the more paying crops like rice.
The planters forced the peasants to take advance sums and enter into fraudulent
contracts which were then used against the peasants.
The planters also tried methods like evictions and enhanced rents. The ryots replied
by going on a rent strike by refusing to pay the enhanced rents and by physically
resisting the attempts to evict them.
Gradually, they learned to use the legal machinery and initiated legal action
supported by fund collection.

http://www.insightsonindia.com INSIGHTS

Page 29


Insights Mock Tests 2015: Test 25 Solutions

The Bengali intelligentsia played a significant role by supporting the peasants cause
through newspaper campaigns, organisation of mass meetings, preparing
memoranda on peasants grievances and supporting them in legal battles.
The Government appointed an indigo commission to inquire into the problem of
indigo cultivation. Based on its recommendations, the Government issued a
notification in November 1860 that the ryots could not be compelled to grow indigo
and that it would ensure that all disputes were settled by legal means.

97.

Solution: d)

The Bardoli taluqa in Surat district had witnessed intense politicisation after the
coming of Gandhi on the national political scene. The movement sparked off in
January 1926 when the authorities decided to increase the land revenue by 30 per
cent.
The Congress leaders were quick to protest and a Bardoli Inquiry Committee was set
up to go into the issue. The committee found the revenue hike to be unjustified. In
February 1926, Vallabhbhai Patel was called to lead the movement.
It set the condition that first the enhanced rent be paid by all the occupants (not
actually done). Then, a committee went into the whole affair and found the revenue
hike to be unjustified and recommended a rise of 6.03 per cent only.
During the 1930s, the peasant awakening was influenced by the Great Depression in
the industrialised countries and the Civil Disobedience Movement which took the
form of no-rent, no-revenue movement in many areas.

98.

Solution: c)

https://www.speedsolving.com/wiki/index.php/Rubik%27s_Cube_Fact_sheet
http://www.mathematische-basteleien.de/rubikscube.htm

99.

Solution: a)

http://news.nationalgeographic.com/news/energy/2014/02/1402277-globalwarming-2-degree-target/

http://www.insightsonindia.com INSIGHTS

Page 30


Insights Mock Tests 2015: Test 25 Solutions

100.

Solution: d)

These are the opening words of the preamble to the Indian Constitution

WE, THE PEOPLE OF INDIA, having solemnly resolved to constitute India into
a SOVEREIGN SOCIALIST SECULAR DEMOCRATIC REPUBLIC and to secure
to all its citizens:
JUSTICE, social, economic and political;
LIBERTY of thought, expression, belief, faith and worship;
EQUALITY of status and of opportunity;
and to promote among them all
FRATERNITY assuring the dignity of the individual and the unity and integrity
of the Nation;
IN OUR CONSTITUENT ASSEMBLY this twenty-sixth day of November, 1949,
do HEREBY ADOPT, ENACT AND GIVE TO OURSELVES THIS
CONSTITUTION.

http://www.insightsonindia.com INSIGHTS

Page 31

INSIGHTS ON INDIA MOCK PRELIMINARY EXAM - 2015


INSIGHTS ON INDIA MOCK TEST - 26
GENERAL STUDIES

PAPER-I
Time Allowed: 2 Hours

Maximum Marks: 200

INSTRUCTIONS
1. IMMEDITELY AFTER THE COMMENCEMENT OF THE EXAMINATION, YOU SHOULD
CHECK THAT THIS TEST BOOKLET DOES NOT HAVE ANY UNPRINTED OR TORN OR MISSING
PAGES OR ITEMS, ETC. IF SO, GET IT REPLACED BY A COMPLETE TEST BOOKLET.
2. You have to enter your Roll Number on the Test
Booklet in the Box provided alongside. DO NOT
Write anything else on the Test Booklet.
4. This Test Booklet contains 100 items (questions). Each item is printed only in English. Each item
comprises four responses (answers). You will select the response which you want to mark on the
Answer Sheet. In case you feel that there is more than one correct response, mark the response which
you consider the best. In any case, choose ONLY ONE response for each item.
5. You have to mark all your responses ONLY on the separate Answer Sheet provided. See directions in
the Answer Sheet.
6. All items carry equal marks.
7. Before you proceed to mark in the Answer Sheet the response to various items in the Test Booklet, you
have to fill in some particulars in the Answer Sheet as per instructions sent to you with your
Admission Certificate.
8. After you have completed filling in all your responses on the Answer Sheet and the examination has
concluded, you should hand over to the Invigilator only the Answer Sheet. You are permitted to take
away with you the Test Booklet.
9. Sheets for rough work are appended in the Test Booklet at the end.
10. Penalty for wrong answers :
THERE WILL BE PENALTY FOR WRONG ANSWERS MARKED BY A CANDIDATE IN THE
OBJECTIVE TYPE QUESTION PAPERS.
(i) There are four alternatives for the answer to every question. For each question for which a
wrong answer has been given by the candidate, one-third of the marks assigned to that
question will be deducted as penalty.
(ii) If a candidate gives more than one answer, it will be treated as a wrong answer even if one of
the given answers happens to be correct and there will be same penalty as above to that
question.
(iii)

If a question is left blank, i.e., no answer is given by the candidate, there will be no penalty
for that question.
http://www.insightsonindia.com

INSIGHTS ON INDIA MOCK TEST SERIES FOR CIVIL SERVICES PRELIMINARY EXAM 2015
http://www.insightsonindia.com

INSIGHTS

Page 1

1. The President recently gave his assent


to re-promulgate the Land Acquisition
ordinance as per provisions of Article
123 of Indian Constitution. When can
the President re-promulgate an
ordinance?
a) when it is in public interest and any
of the houses fail to pass the bill
b) when any of the houses has not
reassembled before the expiry of
the ordinance
c) when in an emergency, it is
required in public interest
d) when the bill is rejected by both the
houses despite the government
pushing for the bill

2. The WHO Framework Convention on


Tobacco Control (WHO
FCTC) provides for
1. Banning of tobacco products in all
countries party to the convention
2. Eliminating illicit trade of tobacco
products
3. Alternative livelihoods to tobacco
farmers
Choose the correct answer using the codes
below.
a)
b)
c)
d)

1 and 2 only
3 only
2 only
1 and 3 only

3. Monosodium glutamate (MSG) is


added artificially to packaged food to
enhance flavour, also found in the
popular product Maggi. Consider the
following about it.
1. It occurs naturally in many
agricultural products.
2. Large doses of MSG can cause
headaches.
3. It is a major factor in causing
obesity

Choose the correct answer using the codes


below.
a)
b)
c)
d)

1 and 2 only
3 only
2 only
1 and 3 only

4. Consider the following about OPCW


(Organisation for the Prohibition of
Chemical Weapons).
1. It is an inter-governmental
organization headquartered in
Hague.
2. One of its mandates is to destroy
the global chemical weapons
stockpile.
3. It operates directly under the aegis
of United Nations Security Council
(UNSC).
4. India is not a signatory to the
Convention on Chemical weapons

Choose the correct answer using the codes


below.
http://www.insightsonindia.com

INSIGHTS

Page 2

a)
b)
c)
d)

1 and 2 only
3 and 4 only
2 and 4 only
1 and 3 only

5. Tripura government has recently


decided to withdraw the Armed
Forces Special Powers Act (AFSPA),
1958 entirely from the state. Consider
the following about AFSPA.
1. Only the Governor of a state can
declare an area to come under
AFSPA.
2. The Act makes local civilian
administration accountable to the
military.
3. It can be withdrawn from an area
only after the consent of the Central
government and Indian armed
forces.
4. AFSPA cannot be applicable in a
state during emergency.
Choose the correct answer using the codes
below.
a)
b)
c)
d)

1 and 2 only
3 only
1 only
1 and 4 only

6. A new study has warned that Glaciers


in the Everest region of the Himalayas
could shrink drastically due to global
warming. What consequences can this
have?
1. It will affect snowfall and rain in
the Himalayan region.

http://www.insightsonindia.com

2. It will greatly affect the farming


and hydropower generation
downstream of the Himalayan
peaks.
3. It can lead to avalanches in this
region.
4. It will reduce the albedo of the
earth.
Choose the correct answer using the codes
below.
a)
b)
c)
d)

1 and 2 only
1, 3 and 4 only
2 and 4 only
All of the above

7. Consider the following about the


Foreign Investment Promotion
Board (FIPB), India.
1. It approves all FDIs under the
automatic route.
2. It is headed by the Union Minister
of Finance.
3. It is a sub-committee of the Cabinet
Committee on Economic Affairs.
Choose the correct answer using the codes
below.
a)
b)
c)
d)

INSIGHTS

1 and 2 only
3 only
1 only
None of the above

Page 3

8. Consider the following statements:


1. A double layer Blu-Ray disc can
hold upto 50 GB of data
2. Unlike DVDs, which use a red laser
to read and write data, Blu-ray uses
a blue laser
3. Unlike DVDs, where data is stored
between layers of two
polycarbonate layers, in Blu-ray
data is stored on top of a 1.1-mmthick polycarbonate layer
4. Blu-ray discs are far thinner than
DVDs
Which of the above statements is/are
incorrect?
a)
b)
c)
d)

1 Only
2 Only
3 Only
4 Only

9. With reference to Poona Pact, consider


the following statements:
1. Mahatma Gandhi objected to the
provision of separate electorates for
the Scheduled (formerly
untouchable) Castes
2. The Pact was the result of the
communal award of Aug. 4, 1932,
made by the British government
Which of the above statements is/are correct?
a)
b)
c)
d)

1 Only
2 Only
Both
None

10. Consider the following about cabinet


committees.
1. They are set up by the Prime
Minister.
2. All committees are ad hoc in
nature.
3. They cannot take a final decision
on matters referred to them by
cabinet.
4. Cabinet committees are also known
as Empowered Group of Ministers
(EGoM).
Choose the correct answer using the codes
below.
a)
b)
c)
d)

1 and 3 only
2, 3 and 4 only
1 only
3 only

11. Triticale, a high yield and rich protein


content cereal is a
a) Wheat and Rye hybrid
b) Wheat and Maize hybrid
c) Maize and Rye Hybrid
d) Rice and Rye Hybrid

12. Consider the following statements:


1. The Lion Capital represents four
roaring lions back to back facing
the four cardinal directions
2. The round abacus in the Lion
Capital is decorated with four
dharmachakras or wheels of law,
alternating with an elephant, a bull,
a horse and a tiger
Which of the above statements is/are correct?

http://www.insightsonindia.com

INSIGHTS

Page 4

a)
b)
c)
d)

1 Only
2 Only
Both
None

13. Primary sector of the economy


includes activities
a) That are based on natural resources
b) Where a good is produced by
exploiting natural resources
c) That do not require skilled
manpower
d) That produce only intermediary
goods

14. Game theory concerns itself with the


a) Study of conflict and cooperation
between intelligent rational
decision-makers
b) Study of irrational behaviour of
human beings
c) Study of computer simulation
models of military strategy
d) Defence strategy of nations in
maritime zones

15. Ministry of Earth Sciences has


developed a SMS-based alert system
for which of the following disasters?
1. Heavy rains
2. Cyclones
3. Storms
4. Earthquakes
5. Avalanche
Choose the correct answer using the codes
below.
http://www.insightsonindia.com

a)
b)
c)
d)

1, 2 and 3 only
2, 3 and 4 only
1 and 4 only
All of the above

16. Consider the following statements


1. Gandhara art was a product of the
combination of Hellenistic, West
Asiatic and native elements
2. In Gandhara Art, most of the
images represent Buddha as seated
in the typically Indian Yogic
posture
Which of the above statements is/are correct?
a)
b)
c)
d)

1 Only
2 Only
Both
None

17. Urban lake pollution has become a


major problem in metro cities. Which
of the following biological measures
can be used to counter urban lake
pollution?
1. Spraying enzymes
2. Using bacteria that consume the
biological load in water
3. Introducing exotic fishes in the lake
Choose the correct answer using the codes
below.
a)
b)
c)
d)

INSIGHTS

1 and 2 only
2 and 3 only
1 and 3 only
All of the above

Page 5

18. Periodically, banks have to submit


information to the RBI on
1. How much they are lending
2. Who are they lending to
3. What interest rate are they lending
at
4. How much deposits do they have
Choose the correct answer using the codes
below.
a)
b)
c)
d)

1, 2 and 3 only
3 and 4 only
2 and 4 only
All of the above

19. With reference to Jainism and its


principles, consider the following
statements:
1. Samyak Darshana means seeing
(hearing, feeling, etc.) things
properly, and avoiding
preconceptions and superstitions
that get in the way of seeing clearly
2. Samyak jnana means having an
accurate and sufficient knowledge
of the real universe
Which of the above statements is/are correct?
a)
b)
c)
d)

1 Only
2 Only
Both
None

20. Honeybees are critical to food


production because
a) They are responsible for pollinating
a large number of crops
b) They act as bio-pesticides killing
harmful bacteria in the fields
http://www.insightsonindia.com

c) Their secretion acts as bio-fertilizer


for the crops
d) Beehives can be used to ward off
unwanted insects from the fields

21. Pankhida and Lotia are two song


traditions sung by peasants and
women respectively in which of the
following states?
a) Maharashtra
b) Punjab
c) Rajasthan
d) Bihar

22. The ozone hole over the Arctic would


have been 40 per cent bigger today if
ozone-depleting chemicals had not
been banned in the 1980s, suggests a
new research. Ozone depletion can
affect
1. Plant development and growth
2. Amount of phytoplankton in the
oceans
3. The natural balance of greenhouse
gases in the atmosphere
Choose the correct answer using the codes
below.
a)
b)
c)
d)

1 and 2 only
2 and 3 only
1 and 3 only
All of the above

23. Urban heat island effect that


increases the effective temperature of
cities is caused due to
1. Paved ground
2. Lack of tree cover
INSIGHTS

Page 6

3. Air pollution
4. Tall buildings
Choose the correct answer using the codes
below.
a)
b)
c)
d)

1, 2 and 3 only
3 and 4 only
1 and 2 only
All of the above

24. Money is accepted as a medium of


exchange for goods and services in
India because
1. It carries intrinsic value.
2. It is used by banks.
3. It carries legal authority.
4. It is the most abundant resource.
Choose the correct answer using the codes
below.
a)
b)
c)
d)

1, 2 and 3 only
3 only
2 and 4 only
All of the above

25. The unseasonal rain and hailstorm in


India this year damaged rabi crops,
wheat especially. This is speculated to
increase international wheat prices.
How?
1. Better quality wheat is available
internationally at prices that are
cheaper than in India.
2. India contributes substantially to
global wheat exports.

c) Both 1 and 2
d) None
26. Consider the following statements
1. She helped to establish the
Women's Indian Association (WIA)
in 1917
2. In 1925, she presided over the
annual session of Indian National
Congress at Cawnpore (now
Kanpur)
3. In 1931, she participated in the
Round table conference with
Gandhi and Madan Mohan
Malaviya
In the above statements, She refers to which
of the following freedom fighters?
a)
b)
c)
d)

Annie Besant
Kamala Nehru
Sarojini Naidu
Vijaya Lakshmi Pandit

27. In the context of environment


conservation, what is rebound effect?
a) Supposedly environment friendly
techniques causing damage to the
environment
b) Reduction in expected gains from
new technologies that increase the
efficiency of resource use
c) Lack of adaptation of ecological
systems to the changes introduced
by human beings
d) None of the above

Which of the above is/are correct reason(s)?


a) 1 only
b) 2 only
http://www.insightsonindia.com

INSIGHTS

Page 7

28. With reference to the Third Battle of


Panipath, consider the following
statements:
1. It was fought between Marathas
and force of Ahmad Shah Abdali
2. The outcome of the War resulted in
restoration of Mughal Empire to
Shah Alam II
Which of the above statements is/are correct?
a)
b)
c)
d)

1 Only
2 Only
Both
None

29. The recent recovery in Indias growth


rate can be attributed to which of the
following factors?
1. Revision in the methodology of
GDP calculation
2. Substantial increase in gross capital
formation
3. Increase in manufacturing growth
rate
Choose the correct answer using the codes
below.
a)
b)
c)
d)

1 and 2 only
2 and 3 only
1 and 3 only
All of the above

30. The incidence of cancer has grown


over the last decade in India. The
following regions lead in a particular
type of cancer. Match these regions
with the possible major causes of
cancer in them.

http://www.insightsonindia.com

1. Punjab Pollution and use of


pesticides
2. West Bengal High use of tobacco
products
3. North-eastern region Household
burning of firewood
4. Coastal India diet rich in spice
and salt
Choose the correct answer using the codes
below.
a)
b)
c)
d)

1 and 2 only
1, 3 and 4 only
2 and 4 only
All of the above

31. The Union government will be


implementing the recommendations
made by the Kasturirangan panel on
conservation of Western Ghats soon.
Which of the following important
recommendations were made by the
panel?
1. Polluting industries should be
banned in entire Western Ghats
region.
2. All projects under Ecologically
Sensitive Areas (ESA) will require
prior-informed consent and noobjection from the Gram Sabha.
3. Establishment of an over-arching
authority to regulate development
in the region.
Choose the correct answer using the codes
below.
a)
b)
c)
d)
INSIGHTS

1 and 2 only
1 and 3 only
2 only
None of the above
Page 8

32. In banking terminology, what does


collateral mean?
a) the person who takes the guarantee
for the lenders repayment ability
b) amount of loan that goes in
purchasing physical assets
c) an asset against which the lender
obtains the loan
d) credit history of the lender

33. Hatha Yoga is a popular system in


Indian Yogic practices. In this Hatha
refers to
a) The two symmetrical sides of the
human body
b) Concentration of the mind in the
kundalini
c) The two opposite currents that
regulate all processes in our body
d) The essence of all physical postures
that lead to greater human health

34. Car provides protection during a


thunderstorm and lightning because of
a) The elevation of the car from the
ground soil
b) Its rubber tyres that insulate the car
from ground
c) The metallic body of car
d) The insulating material present in
seats, handles and window

35. Which of the following are examples


of a physical process, denaturation?
1. Boiling of egg
http://www.insightsonindia.com

2. Curdling of milk
3. Formation of tea
Choose the correct answer using the codes
below.
a)
b)
c)
d)

1 and 3 only
2 and 3 only
1 and 2 only
All of the above

36. The National Highway 3 rund


through which of the following states?
1. Uttar Pradesh
2. Madhya Pradesh
3. Maharashtra
4. Chhattisgarh
5. Rajasthan
Choose the correct answer from the codes
given below:
a)
b)
c)
d)

1,2,3 and 4 Only


1,2 and 3 Only
1,2,3 and 4 Only
All

37. Consider the following about the role


and powers of the Speaker, Lok Sabha.
1. He is the head and representative
of the Lok Sabha.
2. His decision in all Parliamentary
matters is final.
3. He is the final interpreter of the
provisions of the Constitution of
India within the house.
4. He appoints the chairman of all the
parliamentary committees of the
Lok Sabha and supervises their
functioning.
INSIGHTS

Page 9

Choose the correct answer using the codes


below.
a)
b)
c)
d)

2 only
2, 3 and 4 only
1 and 3 only
All of the above

38. If too much of organic matter is added


to a water body, it may lead to
1. Reduction of dissolved oxygen
2. Algal boom
3. Greenhouse emission from the
water body
4. Loss of aquatic life in the water
body
Choose the correct answer using the codes
below.
a)
b)
c)
d)

1, 2 and 4 only
2 and 3 only
1 and 4 only
All of the above

39. Recently inflation is at a five year low


in the Indian economy. This can be
attributed to
1. Good rainfall
2. Fall in price rise rate of food items
3. Cheaper manufactured goods
4. Fall in prices of fuel and power
Choose the correct answer using the codes
below.
a)
b)
c)
d)

1, 2 and 4 only
3 and 4 only
1 and 2 only
2, 3 and 4 only

http://www.insightsonindia.com

40. Which of the following is/are forms of


dance-drama or folk theatre performed
in various parts of India?
1. Yakshagana of Karnataka
2. Theyyam of Kerala
3. Tamasha of Maharashtra
4. The Bengali Jatra
Choose the answer using the codes below:
a)
b)
c)
d)

1,2 and 3 Only


2,3 and 4 Only
1, 3 and 4 Only
All

41. The new way of calculating headline


GDP growth rate in India is based on
a) GDP at factor price at market prices
b) GDP at market prices
c) GDP at constant prices
d) GDP at factor price at constant
prices

42. You are suffering with acidity in the


stomach. Which of the following
is/are most likely to cure your acidity?
1. Antacid
2. Amla
3. Curd
4. Spinach
Choose the correct answer using the codes
below.
a)
b)
c)
d)

INSIGHTS

1 and 3 only
2 and 4 only
1, 3 and 4 only
1 only

Page 10

43. Consider the following statements:


1. He is widely regarded as a key
figure in literary renaissance of
Bengal as well as India
2. From 1858, until his retirement in
1891, he served as a deputy
magistrate and deputy collector in
the Government of British India
To whom do the above descriptions refer to?
a)
b)
c)
d)

Ishwar Chandra Vidyasagar


Rabindranath Tagore
Bankim Chandra Chattopadhyay
Ram Mohan Roy

44. With reference to Non-Cooperation


movement, consider the following
statements:
1. The movement was successful in
attaining its main objective
2. The main trigger for the launch of
this movement was the massacre at
Amritsar in April 1919
3. The movement marked the
transition of Indian nationalism
from a middle-class to a mass basis

2. Committee consisting of the PM,


Lok Sabha speaker and Leader of
Opposition
3. Former CAG of India
Choose the correct answer using the codes
below.
a)
b)
c)
d)

1 and 2 only
2 only
1 and 3 only
1 only

46. The Central Information Commission


of India performs which of the
following kind of tasks?
1. Quasi-judicial
2. Delegated legislation
3. Administrative
Choose the correct answer using the codes
below.
a)
b)
c)
d)

1 and 2 only
2 and 3 only
1 and 3 only
3 only

Which of the above statements is/are correct?


a)
b)
c)
d)

1 and 2 Only
1 and 3 Only
2 and 3 Only
All

45. The Comptroller and Auditor General


of India is appointed by the President
of India based on the advice by
1. Council of Ministers

http://www.insightsonindia.com

47. Resignation or dissolution of which of


the following may bring a collapse of
the Council of Ministers at the state
level?
1. Governor
2. Chief Minister
3. State Legislative assembly
4. State Legislative Council
Choose the correct answer using the codes
below.
INSIGHTS

Page 11

a)
b)
c)
d)

2, 3 and 4 only
2 and 3 only
1, 2 and 4 only
3 only

48. With reference to Satyagraha as


preached and practised by Gandhiji,
consider the following statements:
1. The satyagrahi asserts that truth by
refusing to submit to the wrong or
to cooperate with it in any way
2. Satyagraha forbids any tactic
suggesting the use of secrecy to
ones advantage

50. If the Rajya Sabha declares that it is


necessary in the national interest that
Parliament should make laws on a
matter in the State List, then the
Parliament becomes competent to
make laws on that matter. Such a
resolution must be supported by
a) two-third of members present and
voting
b) two-third of members present and
voting, and majority of the
membership of the house
c) simple majority of the house
d) majority of the membership of the
house

Which of the above statements is/are correct?


a)
b)
c)
d)

1 Only
2 Only
Both
None

49. Fundamental rights in the constitution


are available against the arbitrary
action of
1. The State
2. Private individuals in some cases
3. Societies and associations
4. International agencies
Choose the correct answer using the codes
below.
a)
b)
c)
d)

2, 3 and 4 only
1, 2 and 3 only
1 and 2 only
1 only

51. In 1904 Gandhiji set up a farm at


Phoenix near Durban where he and his
friends could live in austerity and hard
work. Which of the following books
had an immediate influence on
Gandhiji to set up this farm?
a) War and Peace by Leo Tolstoy
b) Walden by Henry David Thoreau
c) Unto The Last by John Ruskin
d) The Bible

52. Average income a country can be a


good measure of economic progress,
but not economic development. Why?
1. It does not capture income
disparities.
2. It does not capture situation of
unemployment in the country.
3. It cannot capture the efficiency of
resource use of the country.
Choose the correct answer using the codes
below.

http://www.insightsonindia.com

INSIGHTS

Page 12

a)
b)
c)
d)

1 and 2 only
2 and 3 only
1 and 3 only
All of the above

53. One way to find out if adults are


undernourished is to calculate what
nutrition scientists call Body Mass
Index (BMI). It is the
a) ratio of body weight and body
height
b) ratio of square of body weight; and
body height
c) ratio of body height; and square of
body weight
d) ratio of body height and body
weight
54. With reference to Round Table
Conferences that were held in London
during Indias struggle for its
independence from the British,
consider the following statements:
1. The first session had 73
representatives, from all Indian
states and all parties except the
Indian National Congress
2. The second session was attended
by Mahatma Gandhi as the
Congress representative
3. The Government of India Act -1935
was the result of deliberations held
at these Round Table Conferences.
Which of the above statements is/are correct?
a)
b)
c)
d)

1 and 3 Only
2 and 3 Only
1 and 2 Only
All

http://www.insightsonindia.com

55. In India, the growth of the services


sector can be attributed to
1. Rising income of people
2. Urbanization
3. Developments in Science and
Technology
4. Development of agriculture and
industry sectors
Choose the correct answer using the codes
below.
a)
b)
c)
d)

1, 2 and 3 only
2 and 3 only
1 and 4 only
All of the above

56. Disguised unemployment is caused


essentially by
a) Under-employment
b) Unemployment
c) Casual employment
d) Over-Employment

57. A constitutional amendment bill may


need the consent of
1. Lok Sabha
2. Rajya Sabha
3. President of India
4. State Legislative assemblies
5. State legislative councils
Choose the correct answer using the codes
below.
a)
b)
c)
d)

INSIGHTS

1, 2 and 3 only
1, 2, 3 and 4 only
4 and 5 only
All of the above

Page 13

58. In 2014, the Nobel Prize in Physics was


awarded to scientists who invented
Efficient Blue Light-Emitting Diodes.
With reference to these Blue LEDs,
consider the following statements:
1. High-quality gallium phosphide is
used to create Blue LED
2. Invention of Blue LED was
essential to create a White LED
3. A modern white LED lightblub
converts more than 50 percent of
the electricity it uses into light
Which of the above statements is/are correct?
a)
b)
c)
d)

1 and 2 Only
2 and 3 Only
1 and 3 Only
All

59. The rivers Indravati River, Manjira


River, Bindusara River and Sabari
River are the tributaries of which of
the following river?
a) Krishna River
b) Godavari River
c) Narmada River
d) Damodar River

60. For rural households, arrange the


following sources of credit in
descending order with respect to their
credit volume?
1. Moneylenders
2. Cooperative banks
3. Government
Choose the correct order from the codes
below.
http://www.insightsonindia.com

a)
b)
c)
d)

123
231
213
132

61. As per the 73rd Constitutional


amendment, the power to expand the
responsibilities of the panchayati raj
bodies rests with
a) State legislatures
b) Governor
c) President of India
d) Parliament

62. With reference to the Lucknow Pact,


consider the following statements:
1. The pact dealt both with the
structure of the government of
India and with the relation of the
Hindu and Muslim communities.
2. This pact paved the way for
Hindu-Muslim cooperation in the
Khilafat movement and Gandhijis
non-cooperation movement from
1920
Which of the above statements is/are correct?
a)
b)
c)
d)

1 Only
2 Only
Both
None

63. Consider the following about the areas


under the Sixth schedule of the
Constitution.
1. The governor is empowered to
organise and re-organise the

INSIGHTS

Page 14

autonomous districts under these


areas.
2. The district and regional councils
can make laws on certain specified
matters.
3. The acts of Parliament or the state
legislature necessarily do not apply
to autonomous districts.
4. The village councils can do trial of
suits and cases between the tribes.
Choose the correct answer using the codes
below.
a)
b)
c)
d)

1 and 2 only
2, 3 and 4 only
1 and 3 only
All of the above

64. Consumers in India have the legal


right to information about the goods
being sold to them. For a medical
commodity, displaying which of the
following information is mandatory
for the producers?
1. Ingredients used
2. Information relating to side effects
and risks
3. Address of the manufacturer
4. Directions for proper use
Choose the correct answer using the codes
below.
a)
b)
c)
d)

1, 2 and 4 only
3 and 4 only
1 and 2 only
All of the above

http://www.insightsonindia.com

65. With reference to the World Food


Prize, consider the following
statements:
1. It was created by Norman E
Borlaug
2. It was created to honour
individuals who have contributed
to improving the availability and
quality of food worldwide
3. It is given by the United Nations
World Food Programme
Which of the above statements is/are correct?
a)
b)
c)
d)

1 and 3 Only
1 and 2 Only
2 and 3 Only
All

66. Which of the following are included in


the non-tax revenue receipts of the
government of India?
1. Profits and dividends which the
government gets from its public
sector undertakings
2. External grants received by the
government
3. Interests received by the
government out of all loans
forwarded by it
Choose the correct answer using the codes
below.
a)
b)
c)
d)

INSIGHTS

1 and 2 only
1 and 3 only
2 and 3 only
All of the above

Page 15

67. To secure for all citizens a uniform


civil code throughout the country is
one of the Directive Principles of state
policy in our constitution. If this is
implemented, it will lead to
1. common laws for all religious
communities in India
2. common laws for both men and
women
3. centralization of justice delivery
system in courts rather than local
bodies
Choose the correct answer using the codes
below.
a)
b)
c)
d)

1 and 2 only
2 and 3 only
1 only
All of the above

68. With reference to a National Park


located in Jammu and Kashmir,
consider the following statements:
1. It is located 22 kilometers from
Srinagar, Jammu and Kashmir
2. This National park is located in the
Zabarwan Range of the western
Himalayas
3. The main animal species that this
National Park is most famous for is
the hangul, or the Kashmir stag
To which National Parks given below does
above descriptions refer ?
a)
b)
c)
d)

Hemis National Park


Kazinag National Park
Kishtwar National Park
Dachigam National Park

http://www.insightsonindia.com

69. Which of the following processes lead


to loss of vitamins/nutrients from
vegetables?
1. mixing with other vegetables
2. if excess water used in the cooking
is thrown away
3. continuous heating
Choose the correct answer using the codes
below.
a)
b)
c)
d)

1 and 2 only
2 and 3 only
1 and 3 only
All of the above

70. Consider the following statements.


Assertion (A): Farmers do not need to add
nitrogen fertiliser to the soil in
which
leguminous plants are grown.
Reason (R): Rhizobium bacteria that can take
atmospheric nitrogen and convert it into a
soluble form lives in the roots of leguminous
plants.
In the context of the statements above, which
of these is true?
a) A and R both are true, and R is the
correct explanation for A.
b) A and R both are true, and R is the
NOT the correct explanation for A.
c) A is correct, R is incorrect.
d) A and R both are incorrect.

INSIGHTS

Page 16

71. Which of the following is/are the same


for both the Supreme Court and High
Courts?
1. Appointment procedure of Judges
2. Extent of writ issuing authority
3. Power to interpret the constitution
4. Power of Judicial review of Central
laws
Choose the correct answer using the codes
below.
a)
b)
c)
d)

2 and 3 only
1, 3 and 4 only
1, 2 and 4 only
3 and 4 only

72. With reference to the Khilafat


Movement, consider the following
statements:
1. One of the causes of this movement
was the Treaty of Svres (August
1920), which gave parts of the
Turkish homeland to Greece and
other non-Muslim powers
2. The movement collapsed when
Mustafa Kemal Atatrk abolished
the caliphate altogether in 1924
Which of the above statements is/are correct?
a)
b)
c)
d)

1 Only
2 Only
Both
None

73. Due to temperature differences, warm


air at equator rises up and cold wind
from the Northern and Southern Polar
Regions rush in, to take its place. But,
http://www.insightsonindia.com

the wind movement is not exactly in


the North-South Direction because of
a) Rotation of earth
b) Revolution of earth around the Sun
c) Direction change by ocean currents
and obstruction by clouds
d) Obstruction by landscapes

74. Design of the Vermi-processing toilets


has been tested in India. Consider the
following about it.
1. Excreta are treated by earthworms.
2. It is a totally dry technology.
3. Processed material is suitable for
use as herbicide in farms.
4. Pilot projects in trains have been
implemented with this toilet.
Choose the correct answer using the codes
below.
a)
b)
c)
d)

1 and 3 only
2 and 4 only
1 only
2, 3 and 4 only

75. Turning and loosening of soil is very


important for cultivation of crops.
Why?
1. Brings the nutrient-rich soil to the
top
2. Helps in the growth of earthworms
and microbes present in the soil
3. Reduces the harm caused by
pesticides
Choose the correct answer using the codes
below.
INSIGHTS

Page 17

a)
b)
c)
d)

1 and 3 only
2 and 3 only
1 and 2 only
All of the above

76. Which of the following affects the


propagation of electromagnetic
waves?
1. Composition of the earths
atmosphere
2. Shape of earth
3. Amount of humidity present in the
surrounding environment
Choose the correct answer using the codes
below.
a)
b)
c)
d)

1 and 3 only
2 and 3 only
1 and 2 only
All of the above

77. Hair washed with hard water looks


dull because of a sticky precipitate
which also makes
1. Clothe fibres hard
2. Human intestines more prone to
blockages
3. Foams in polluted urban lakes
Choose the correct answer using the codes
below.
a)
b)
c)
d)

1 and 3 only
2 and 3 only
1 and 2 only
None of the above

a) Rate of production of organic


matter during photosynthesis
b) Rate of diversification of the
species base every year
c) Rate of gathering of organic matter
by all possible natural processes
d) Rate of nutrient cycling to and fro
from soil
79. With reference to the Parliamentary
seats, the Constitution ensures that
there is uniformity of representation
1. between the states
2. between the different
constituencies of the state
Which of the above correctly complete the
above statement?
a)
b)
c)
d)

1 only
2 only
Both 1 and 2
None

80. Which of the following species found


in India has been kept in the critically
endangered in the IUCN red list?
1. Himalayan Quail
2. Indian Vulture
3. Siberian Crane
4. Namdapha Flying Squirrel
Choose the correct answer using the codes
below.
a)
b)
c)
d)

1 and 4 only
2 and 3 only
1 and 2 only
All of the above

78. Gross primary productivity of an


ecosystem is
http://www.insightsonindia.com

INSIGHTS

Page 18

81. Consider the following statements.


1. A given species may occupy more
than one trophic level in the same
ecosystem at the same time.
2. In most ecosystems, pyramids of
energy and biomass are upright.
Which of these is/are true?
a)
b)
c)
d)

1 only
2 only
Both 1 and 2
None

82. Panch Mahal, located in Agra, Uttar


Pradesh is an example of
a) Indo-Islamic Architecture
b) Indo Roman Architecture
c) Indo Saracenic Architecture
d) None of the above

83. In ecological terminology, what are


seres?
a) Edge species that show greater
diversity in the centre of an
ecosystem
b) Entire sequence of communities
that successively change in a given
area
c) A sequence of communities that
remains constant despite the
change in the type of ecosystem in
surrounding areas
d) None of the above

84. Which are the major differences


between carbon and phosphorus
cycles in the atmosphere?
http://www.insightsonindia.com

1. Atmospheric inputs of phosphorus


through rainfall are much smaller
than carbon inputs.
2. Gaseous exchanges of phosphorus
between organism and
environment are negligible.
Which of these is/are true?
a)
b)
c)
d)

1 only
2 only
Both 1 and 2
None

85. Which of the following are ecosystem


services that are hard and exorbitantly
costly to replicate technologically?
1. Mitigating droughts
2. Pollinating crops
3. Providing carbon sinks
4. Nutrient cycling
Choose the correct answer using the codes
below.
a)
b)
c)
d)

1 and 4 only
2, 3 and 4 only
1 and 2 only
All of the above

86. Bio-magnification occurs when


1. The substance can't be broken
down by environmental processes
2. The substance concentration
increases progressively as it moves
up a food chain
3. Low or non-existent rate of internal
degradation or excretion of the
substance by living organisms

INSIGHTS

Page 19

Choose the correct answer using the codes


below.
a)
b)
c)
d)

1 and 3 only
2 and 3 only
1 and 2 only
All of the above

87. Purchasing power parity between two


currencies means that
a) both currencies have equal
exchange rate
b) same goods in both currencies will
effectively cost the same
c) both currencies can be replaced
without buying and selling in the
Forex market
d) All of the above
88. Which of the following can be the uses
of DNA fingerprinting?
1. To determine paternity of an
individual.
2. To identify the dead bodies in any
accident by comparing the DNAs
of parents or children
3. To identify racial groups to rewrite
biological evolution
Choose the correct answer using the codes
below.
a)
b)
c)
d)

1 and 3 only
2 and 3 only
1 and 2 only
All of the above

89. Atmospheric pollution is generally


studied at
http://www.insightsonindia.com

1. Stratospheric levels
2. Tropospheric levels
3. Ionospheric levels
Choose the correct answer using the codes
below.
a)
b)
c)
d)

1 and 3 only
2 and 3 only
1 and 2 only
1 only

90. In a greenhouse, visible light passes


through the transparent glass and
heats up the soil and the plants. Apart
from the sun rays, greenhouse is also
heated by
a) Infrared emissions from warm Soil
b) Infrared emissions from warm
plants
c) Infrared emissions from both warm
Soil and plants
d) Infrared emissions from humidity
present in the greenhouse
91. Acid rain can be controlled by
1. Using low sulphur content diesel in
vehicles
2. Switching over to natural gas as
fuels instead of coal or oil
3. Encouraging use of firewood rather
than Kerosene
Choose the correct answer using the codes
below.
a)
b)
c)
d)

INSIGHTS

1 and 3 only
2 and 3 only
1 and 2 only
All of the above

Page 20

92. Photochemical smog, a growing


problem in urban areas is most likely
to happen in
a) Warm, sunny and dry climates
b) Hot and humid climates
c) Cold and dry climates
d) Cold and humid climates

93. Certain plants can metabolize nitrogen


oxide and help control urban
pollution. These plants are
1. Pinus
2. Quercus
3. Vitis
4. Juniparus
Choose the correct answer using the codes
below.
a)
b)
c)
d)

1, 3 and 4 only
2 and 3 only
1 and 2 only
All of the above

94. A patent is a set of exclusive rights


granted by a sovereign state to an
inventor or assignee for a limited
period of time in exchange for
a) detailed public disclosure of an
invention
b) royalty from the state
c) monetary incentives from the users
d) commercial availability of the
invention

1. State governments are subservient


to the Union government
2. State governments derive their
power from the Union government
3. Union government is primarily
responsible for the administration
in the states
4. All major decision concerning
states are taken by the Union
government
Choose the correct answer using the codes
below.
a)
b)
c)
d)

1, 2 and 3 only
2 and 4 only
1 and 3 only
None of the above

96. In the process of transpiration, the


evaporation of water from leaves also
helps in
1. Transportation of water to great
heights in the plant
2. Regulating the temperature of the
plant
3. Absorbing more nutrients from the
roots
Choose the correct answer using the codes
below.
a)
b)
c)
d)

1 and 3 only
2 and 3 only
1 and 2 only
All of the above

95. The Indian constitution is said to have


a Federal System with Unitary Bias. It
means that
http://www.insightsonindia.com

INSIGHTS

Page 21

97. Consider the following about the


recently launched National
Supercomputing Mission of India.
1. Under this, national academic and
R&D institutions will get super
computing grids.
2. It provides for development of
professional High Performance
Computing (HPC) aware human
resource.
3. Supercomputing centres will be
opened in every state in India.
Choose the correct answer using the codes
below.
a)
b)
c)
d)

100.
The Dhamekh stupa is an
imposing cylindrical structure of the
Gupta age, partly built of stone and
partly of brick. This site is looked upon
with, greatest sanctity and became a
flourishing Buddhist establishment
with numerous temples, stupas and
monasteries. Where is this stupa
located?
a) Bodh Gaya
b) Sarnath
c) Kushinagar
d) Rajgir

1 and 2 only
3 only
1 only
All of the above

98. What was common between Ghosha,


Lopamudra, Apala and Romasha
Brahmavadini
a) They were ancient Indian women
authors
b) They were ancient Indian women
painters
c) They were ancient Indian women
dancers
d) None of the above

99. The apex court of India, Supreme


Court, was established under the
a) Regulating Act of 1773
b) Pitts India Act 1784
c) Charter Act 1833
d) Queens Durbar Proclamation 1911

http://www.insightsonindia.com

INSIGHTS

Page 22

Insights Mock Tests 2015: Test 26 Solutions


1. Solution: b)
Article 123 of the constitution gives legislative power to President. He can issue
ordinances when Parliament is in recess i.e. not in session if there is urgent need to
have a law on some urgent public matter. The promulgated ordinance has similar
effect to an act of parliament. However, every ordinance must be laid and approved
by both houses of the parliament within 6 weeks from the reassembling. If not
placed and approved by both houses of the parliament after reassembling it becomes
invalid or lapses.

2. Solution: c)
WNTD is one of eight official global public health campaigns marked by the WHO.
The FCTC, one of the most quickly ratified treaties in United Nations history, is a
supranational agreement that seeks "to protect present and future generations from
the devastating health, social, environmental and economic consequences of tobacco
consumption and exposure to tobacco smoke" by enacting a set of universal
standards stating the dangers of tobacco and limiting its use in all forms worldwide.
Even though it talks about regulating the use of tobacco, it does not provide for a
tobacco ban; neither about alternative livelihoods for farmers.

3. Solution: a)
Uttar Pradesh Food Safety and Drug Administration (UP FDA) had recently asked
Nestle India to withdraw a batch of Maggi noodles from the market after they had
found that it contained high level of lead and Monosodium glutamate (MSG), a taste
enhancer.
Monosodium glutamate is a kind of amino acid. It occurs naturally in many
agricultural products. Monosodium glutamate is added artificially to packaged food
to enhance flavour. Large doses of MSG can cause headaches and other feelings of
discomfort collectively known as Chinese Restaurant Syndrome.

4. Solution: a)
http://www.insightsonindia.com

INSIGHTS

Page 1

Insights Mock Tests 2015: Test 26 Solutions


Recently, Nobel peace prize winning organisation OPCW (Organisation for the
Prohibition of Chemical Weapons) has achieved major milestone of destroying 90
per cent of the global chemical weapons stockpile.
OPCW promotes, administers and verifies the adherence to the Convention on the
Prohibition of the Development, Production, Stockpiling and Use of Chemical
Weapons and on their Destruction (CWC).
This convention outlaws the production, stockpiling, and use of chemical weapons
and their precursors. 190 member-states have signed and ratified this convention
including India. Six states- Angola, Egypt, Israel, Myanmar, North Korea and South
Sudan are still outside the CWC.

5. Solution: c)
AFSPA initially empowered only the Governors of the States and the Administrators
of the Union Territories to declare areas in the concerned State or the Union
Territory as 'disturbed'. But, since 1972 AFSPA can be enforced by the Central
government too.
The Act gives the military special powers over disturbed areas where it can use
force (without formal prosecution etc.) if it deems necessary.
The Act has received criticism from several sections for alleged concerns about
human rights violations in the regions of its enforcement alleged to have happened.
Hence, the Tripura government has decided to withdraw the Armed Forces Special
Powers Act (AFSPA), 1958 from the remaining areas of the state. Consent of military
is not required to withdraw the act from disturbed areas.

6. Solution: d)
The rate of disappearance will depend on how much greenhouse-gas emissions
continue to rise and indirectly will affect temperature, snowfall and rainfall in this
region. Increased temperatures due to global warming will not only increase the
rates of snow and ice melt, but can also result in a change of precipitation from snow
to rain at critical elevations, where glaciers are concentrated. It will greatly affect the
farming and hydropower generation downstream of the Himalayan peaks. While,
http://www.insightsonindia.com

INSIGHTS

Page 2

Insights Mock Tests 2015: Test 26 Solutions


avalanches and earthquakes in this region also can breach the dams, causing
catastrophic floods. It can also result in the creation and growth of lakes dammed by
glacial debris.
Albedo is the fraction of solar energy (shortwave radiation) reflected from the Earth
back into space. It is a measure of the reflectivity of the earth's surface. Ice, especially
with snow on top of it, has a high albedo: most sunlight hitting the surface bounces
back towards space.

7. Solution: d)
The Foreign Investment Promotion Board (FIPB) is a national agency of Government
of India, with the remit to consider and recommend foreign direct investment (FDI)
which does not come under the automatic route. It provides a single window
clearance for proposals on FDI in India.
It comprises of secretaries of different ministries with Secretary, Department of
Economic Affairs, Ministry of Finance being the chairman.
In a significant move aimed at expediting flow of foreign investment into the
country, the Union Cabinet liberalised the FDI policy further by allowing the FIPB to
clear proposals from overseas entities worth up to 30 billion, against the existing
limit of 12 billion. Recommendations of FIPB for proposals up to 30 billion are
approved by Minister of Finance. While recommendations for proposals of more
than 3000 crore need to be approved by Cabinet Committee on Economic Affairs
(CCEA).

8. Solution: d)
http://electronics.howstuffworks.com/blu-ray2.htm
Blu-ray discs are almost as thick as DVDs.

9. Solution: c)
Poona Pact, (Sept. 24, 1932), agreement between Hindu leaders in India granting
new rights to untouchables (low-caste Hindu groups). The pact, signed at Poona
(now Pune, Maharashtra), resulted from the communal award of Aug. 4, 1932, made
http://www.insightsonindia.com

INSIGHTS

Page 3

Insights Mock Tests 2015: Test 26 Solutions


by the British government on the failure of the India parties to agree, which allotted
seats in the various legislatures of India to the different communities. Mahatma
Gandhi objected to the provision of separate electorates for the Scheduled (formerly
untouchable) Castes, which in his view separated them from the whole Hindu
community. Though in prison, Gandhi announced a fast unto death, which he began
on September 18.
Bhimrao Ramji Ambedkar, the untouchable leader, who felt that his groups special
interests might be advanced by the governments system, resisted concessions until
Gandhi was near death. He and the Hindu leaders then agreed to the pact, which
withdrew separate electorates but gave increased representation to the Scheduled
Castes for a 10-year period.

10. Solution: c)
They are extra-constitutional in emergence. In other words, they are not mentioned
in the Constitution. However, the Rules of Business provide for their establishment.
They are of two typesstanding and ad hoc. They are set up by the Prime Minister
according to the exigencies of the time and requirements of the situation.
They not only sort out issues and formulate proposals for the consideration of the
Cabinet, but also take decisions. However, the Cabinet can review their decisions.

11. Solution: a)
Triticale, Wheat-rye hybrid that has a high yield and rich protein content. The first
cross was reported in 1875, the first fertile cross in 1888. The name triticale first
appeared in scientific literature in 1935 and is attributed to Erich Tschermak von
Seysenegg. In favourable environmental conditions its yield equals that of wheat;
under poor conditions its yield exceeds that of wheat. Its flour is not very suitable for
breadmaking but can be blended with wheat flour. Major producers are Russia, the
U.S., and Australia.

12. Solution: a)
The most remarkable among Mauryan Art is the highly polished monolithic lioncapital found at Sarnath, which is now the Emblem of the Government of India. It
http://www.insightsonindia.com

INSIGHTS

Page 4

Insights Mock Tests 2015: Test 26 Solutions


represents four roaring lions back to back facing the four cardinal directions. The
round abacus is decorated with four dharmachakras or wheels of law, alternating
with an elephant, a bull, a horse and a lion, all carved with masterly skill. The abacus
is supported by a bell-shaped base consisting of a lotus with dharmachakra, which
perhaps symbolized the victory of righteousness over physical force. The superb
modelling of the figures executed in a realistic manner with a certain stylization, is
invested with a great power and dignity, and reveals the aristocratic and
international nature of Mauryan art.

13. Solution: b)
Here, no definition is absolutely appropriate. UPSC will often give such options to
confuse you.
Best approach would be to eliminate and select. Option A is a very general
statement, and thus cannot be true.
Option C and D can be true for any of the sectors of the economy.
So, option B is the most appropriate choice here.
Right definition would be - The primary sector of the economy is the sector of an
economy making direct use of natural resources. This includes agriculture, forestry,
fishing and mining. This is contrasted with the secondary sector, producing
manufactured goods, and the tertiary sector, producing services.

14. Solution: a)
Renowned American mathematician and Nobel laureate John Nash passed away
recently. He was renowned for works in game theory.
Game theory is the study of strategic decision making. Specifically, it is "the study of
mathematical models of conflict and cooperation between intelligent rational
decision-makers." An alternative term suggested "as a more descriptive name for the
discipline" is interactive decision theory.
Game theory is mainly used in economics, political science, and psychology, as well
as logic, computer science, and biology. The subject first addressed zero-sum games,
such that one person's gains exactly equal net losses of the other participant or
participants. Today, however, game theory applies to a wide range of behavioral
http://www.insightsonindia.com

INSIGHTS

Page 5

Insights Mock Tests 2015: Test 26 Solutions


relations, and has developed into an umbrella term for the logical side of decision
science, including both humans and non-humans (e.g. computers, animals).

15. Solution: a)
During the past one year, the Ministry of Earth Sciences has provided agrometeorological advisories through mobile phones to 7.1 million farmers in 505
districts, launched customized weather forecast service for the holy yatra to
Amarnath Cave and commissioned a weather forecasting system at the heliport
serving pilgrim going to Vaishnodevi.
In addition, a SMS-based disaster alert system for cyclones, storms and heavy rains
was also developed for the nation and two mobile applications for android phones
and tablets called India weather` and SAFAR-Air has been launched. India Weather
is designed to give current day and four-days advance forecast of weather in 310
cities across the country, while Safar-Air provides air quality information for the
metro cities in the country.

16. Solution: c)
After Alexander's invasion of India in 326 B.C., the IndoGreek, Indo Scythian and Kushan kings ruled over its
north-western territories and under their patronage
emerged a distinct style of sculpture, popularly known as
the Greco-Roman, Buddhist or Gandhara art. It was a
product of the combination of Hellenistic, West Asiatic
and native elements. Greek and Roman techniques,
modified according to Indian requirements, were
employed in fashioning the Gandhara sculpture which
truly represents Indian culture in a Western garb. The
subject-matter treated is predominantly Buddhist. Its area
extended from Takshila in India to the Swat Valley in
Pakistan and northwards to areas in Afghanistan.
The first century of the Christian era's revolutionary
change, had far-reaching effects, not only on the art of
India, but also on the artistic development of Buddhist
http://www.insightsonindia.com

INSIGHTS

Head of Buddha,
Gandhara period, 2nd
century A.D., Uttar
Pradesh

Page 6

Insights Mock Tests 2015: Test 26 Solutions


countries of Asia. Buddha who was hitherto designated
only by a symbol, was conceived in human form. His
person was given some of the 32 suspicious bodily signs
associated with the Mahapurushalakshana, such as the
protuberance of the skull, the hair-knot, bindi between the
eyebrows and elongated ears. This change came about as a
result of the new changes that had crept into the religious
outlook of Buddhism due to the influence of the
Devotional School of Hindu Philosophy, requiring the
worship of personal gods. It must have exercised profound
influence on the religious approach of the masses towards
Buddhism. The image becomes henceforth the main
element of sculpture and worship. Possibly, the emergence
of the image of Buddha in Gandhara and in Mathura was a
parallel development. In each case, it was produced by the
local artist craftsmen working in the local tradition. At
Mathura it clearly emerges from the Yaksha tradition. The
Gandhara image might seem to resemble Apollo in some
extraneous forms and does look characteristically GrecoRoman in drapery, but even there most of the images
represent Buddha as seated in the typically Indian Yogic
posture, a feature completely unknown to the Hellenistic
tradition of art.

17. Solution: a)
Enzymes are biological substances that help break down proteins. Basically they are
natural catalysts for control of metabolism and growth. Enzymes can be produced in
an artificial way and added to all kinds of biological processes in orer to control the
rate in which chemical reactions occur.
A common use of enzymes is in washing powder, as washing clothes often involves
removing biological dirt like sweat, blood or other human excrements. International
rehabilitation and soil stabilisation services (IRASSS) has found a way to use
enzymes to clean soil and water on a large scale, like cleaning golf courses and lakes.
Exotic fishes would only introduce more competition for resources in the lake. This
may further affect the biodiversity of the lake.

http://www.insightsonindia.com

INSIGHTS

Page 7

Insights Mock Tests 2015: Test 26 Solutions

18. Solution: d)
RBI lays down the working norms for banks in India.
To ensure that its policies and schemes are being operated correctly, RBI asks
information from banks on periodic intervals.
To ensure that priority sector lending norms are not being violated, banks have to
show amount of credit and customers.
Similarily, information on interest rate allows RBI to check whether banks are
lending much below or much above the prescribed interest rates.
To enforce CRR and SLR norms, RBI asks information on deposits.

19. Solution: c)
The Three Jewels of Jainism
The aim of Jain life is to achieve liberation of the soul.
This is done by following the Jain ethical code, or to put it simply, living rightly by
following the three jewels of Jain ethics.
There are three parts to this: right faith, right knowledge and right conduct. The first
two are very closely connected.
Right faith - Samyak darshana
This doesn't mean believing what you're told, but means seeing (hearing, feeling,
etc.) things properly, and avoiding preconceptions and superstitions that get in the
way of seeing clearly.
Some books call samyak darshana "right perception". You can't achieve this unless
you are determined to find the truth, and distinguish it from untruth.
Right knowledge - Samyak jnana
This means having an accurate and sufficient knowledge of the real universe - this
requires a true knowledge of the five (or six) substances and nine truths of the
universe - and having that knowledge with the right mental attitude.

http://www.insightsonindia.com

INSIGHTS

Page 8

Insights Mock Tests 2015: Test 26 Solutions


One writer puts it like this: "if our character is flawed and our conscience is not clear,
knowledge alone will not help us achieve composure and happiness".
Today this means having a proper knowledge of the Jainscriptures.
Some writers describe right knowledge as meaning having a pure soul; a soul that is
free from attachment and desire... others say that a person who has right knowledge
will naturally free themselves from attachment and desire, and so achieve peace of
mind.
Right conduct - Samyak charitra
This means living your life according to Jain ethical rules, to avoid doing harm to
living things and freeing yourself from attachment and other impure attitudes and
thoughts.
Jains believe that a person who has right faith and right knowledge will be
motivated and able to achieve right conduct.
Many Jains believe that a person without right faith and right knowledge cannot
achieve right conduct - so it's no use following scripture and ritual for the wrong
reasons (e.g. so that other people will think you are a good person). Not all Jains
hold this view. (Source BBC)

20. Solution: a)
As honey bees gather pollen and nectar for their survival, they pollinate crops such
as apples, cranberries, melons and broccoli. Some crops, including blueberries and
cherries, are 90-percent dependent on honey bee pollination; one crop, almonds,
depends entirely on the honey bee for pollination at bloom time.

http://www.downtoearth.org.in/content/united-states-environmental-bodyproposes-temporary-pesticide-free-zones-protect-honeybees

21. Solution: c)
Pankhida, Rajasthan Sung by the peasants of Rajasthan while doing work in the
fields, the peasants sing and speak while playing algoza and manjira. The literal
meaning of the word Pankhida is lover.
http://www.insightsonindia.com

INSIGHTS

Page 9

Insights Mock Tests 2015: Test 26 Solutions


Lotia, Rajasthan Lotia is sung in the chaitra month during the festival - Lotia.
Women bring lotas (a vessel to fill water) and kalash (a vessel considered to be
auspicious to fill water during worship) filled with water from ponds and wells.
They decorate them with flowers and come home.

22. Solution: d)
Plant growth, as well as its physiological and developmental processes are all
affected negatively. These include the way plants form, timing of development and
growth, distribution of plant nutrients and metabolism, etc. These changes can have
important implications for plant competitive balance, animals that feed on these
plants, plant diseases, and biogeochemical cycles.
Phytoplankton form the foundation of aquatic food webs. These usually grow closer
to the surface of water, where there is enough sunlight. Changes in UV levels is
known to affect the development and growth of phytoplankton, and naturally, the
fish that feed on them. UV radiation is also know to have affect the development
stages of of fish, shrimp, crab, amphibians and other animals. When this happens,
animals in the upper food chain that feed on these tiny fishes are all affected.
The power of higher UL levels affect the natural balance of gasses (and greenhouse
gases) in the biosphere: e.g., carbon dioxide (CO2), carbon monoxide (CO), carbonyl
sulfide (COS) and ozone. Changes in UV levels can cause biosphere-atmosphere
feedback resulting from the atmospheric buildup of these gases.
You can read more about this at http://www.conserve-energy-future.com/causesand-effects-of-ozone-hole.php

23. Solution: d)
The tall buildings within many urban areas provide multiple surfaces for the
reflection and absorption of sunlight, increasing the efficiency with which urban
areas are heated. This is called the "urban canyon effect". Another effect of buildings
is the blocking of wind, which also inhibits cooling by convection and pollution from
dissipating.
Read this properly in detail here
http://en.wikipedia.org/wiki/Urban_heat_island#Causes

http://www.insightsonindia.com

INSIGHTS

Page 10

Insights Mock Tests 2015: Test 26 Solutions


Implications - http://www.downtoearth.org.in/content/heat-wave-death-tollnears-2000-andhra-telangana-0

24. Solution: b)
Unlike the things that were used as money earlier, modern currency is not made of
precious metal such as gold, silver and copper. And unlike grain and cattle, they are
neither of everyday use. The modern currency is without any use of its own.
It is accepted as a medium of exchange because the currency is authorised by the
government of the country.

25. Solution: a)
Indian traders, especially from south India, have started importing wheat from
Australia and other parts of world in the wake of poor yields in India and lower
international price.
Officials in the agriculture ministry say the government is closely watching
international wheat prices.
Read this article to understand the whole situation.
http://www.downtoearth.org.in/content/will-india-s-wheat-imports-lead-globalfood-crisis

26. Solution: c)
http://en.wikipedia.org/wiki/Sarojini_Naidu
27. Solution: b)
In conservation and energy economics, the rebound effect (or take-back effect) is the
reduction in expected gains from new technologies that increase the efficiency
of resource use, because of behavioral or other systemic responses. These responses
usually tend to offset the beneficial effects of the new technology or other measures
taken.

http://www.insightsonindia.com

INSIGHTS

Page 11

Insights Mock Tests 2015: Test 26 Solutions


For instance, if a 5% improvement in vehicle fuel efficiency results in only a 2% drop
in fuel use, there is a 60% rebound effect (since (5-2)5 = 60%). The 'missing' 3% might
have been consumed by driving faster or further than before.

28. Solution: c)
http://en.wikipedia.org/wiki/Third_Battle_of_Panipat#Aftermath

29. Solution: c)
A recently released data by the Central Statistics Office shows that Indias economy
grew by 7.3% during 2014-15. Gross fixed capital formation a barometer for
investments slowed for the second straight year.
The manufacturing sector grew by 7.1% against the 2013-14 growth of 5.3%.
The prime drivers of the growth were the significantly stronger performance of
manufacturing, electricity, gas, water supply and other utility services and the
financial, real estate and professional services.
30. Solution: b)
An international consortium of researchers led by the Institute for Health Metrics
and Evaluation at the University of Washington, in its recently released data, has
said that the incidence of cancer has grown over the last decade in India although
the rate of mortality has fallen. The study is named The Global Burden of Cancer
2013.

http://www.insightsonindia.com

INSIGHTS

Page 12

Insights Mock Tests 2015: Test 26 Solutions

31. Solution: d)
The Kasturirangan panel was set up to study the Gadgil committee report on the
Western Ghats. The Gadgil panel report had faced unanimous opposition from state
governments for recommending that almost three-fourth of the hills, including
plantations, cultivated lands and large habitations, be turned into a restricted
development zone with an over-arching authority to regulate the region superseding
the elected authorities role.

http://www.insightsonindia.com

INSIGHTS

Page 13

Insights Mock Tests 2015: Test 26 Solutions


Read the recommendations here http://timesofindia.indiatimes.com/india/37-ofWestern-Ghats-ecologically-sensitive-Kasturirangan-panelreport/articleshow/19607237.cms
32. Solution: c)
Every loan agreement specifies an interest rate which the borrower must pay to the
lender along with the repayment of the principal. In addition, lenders may demand
collateral (security) against loans.
Collateral is an asset that the borrower owns (such as land, building, vehicle,
livestocks, deposits with banks) and uses this as a guarantee to a lender until the
loan is repaid.
If the borrower fails to repay the loan, the lender has the right to sell the asset or
collateral to obtain payment. Property such as land titles, deposits with banks,
livestock are some common examples of collateral used for borrowing.

33. Solution: c)
The Prime Minister, Shri Narendra Modi, has called for making Yoga a mass
movement, connecting people and strengthening the bonds of humanity.
Hatha Yoga is known for the asanas or postures. Ha and tha, the sun and moon,
refer to the two opposite currents that regulate all processes in our body.
The Hatha Yoga Pradipika names sana as the first accessory and its benefits as the
attainment of steadiness, freedom from disease, and lightness of body.
Traditional Hatha Yoga consists of: Asanas (postures); Shat Karmas (six cleansing
techniques, also known as Shat Kriyas); Pranayama (control of breathing with
retention); Bandhas (locks) and Mudras (seals) for the regulation of Prana (life-force)
and kundalini; and Samadhi (Union with God, realization of the Self, ecstasy,
nirvana).
34. Solution: c)
Read the link carefully. The concept of car protection has been explained beautifully.
http://www.weather.com/storms/tornado/news/what-happens-when-lightninghits-car-20140625

http://www.insightsonindia.com

INSIGHTS

Page 14

Insights Mock Tests 2015: Test 26 Solutions


35. Solution: c)
When a protein in its native form, is subjected to physical change like change in
temperature or chemical change like change in pH, the hydrogen bonds are
disturbed.
Due to this, globules unfold and helix gets uncoiled and protein loses its biological
activity. This is called denaturation of protein.
The coagulation of egg white on boiling is a common example of denaturation.
Another example is curdling of milk which is caused due to the formation of lactic
acid by the bacteria present in milk.

36. Solution: c)
http://en.wikipedia.org/wiki/National_Highway_3_(India)(old_numbering)
National Highway 3, or NH 3, commonly referred to as the MumbaiAgra Highway
or AB Road, is a major Indian National Highway that runs through the states of
Uttar Pradesh, Rajasthan, Madhya Pradesh and Maharashtra in India.

37. Solution: d)
He is the guardian of powers and privileges of the members, the House as a whole
and its committees. He is the principal spokesman of the House, and his decision in
all Parliamentary matters is final. He is thus much more than merely the presiding
officer of the Lok Sabha. In these capacities, he is vested with vast, varied and vital
responsibilities and enjoys great honour, high dignity and supreme authority within
the House.
He is the final interpreter of the provisions of (a) the Constitution of India, (b) the
Rules of Procedure and Conduct of Business of Lok Sabha, and (c) the parliamentary
precedents, within the House.
He appoints the chairman of all the parliamentary committees of the Lok Sabha and
supervises their functioning. He himself is the chairman of the Business Advisory
Committee, the Rules Committee and the General Purpose Committee.

38. Solution: d)
http://www.insightsonindia.com

INSIGHTS

Page 15

Insights Mock Tests 2015: Test 26 Solutions


If high quantity of organic matter is added to water, all the available oxygen is used
up. This causes oxygen dependent aquatic life to die.
Thus, anaerobic bacteria (which do not require oxygen) begin to break down the
organic waste and produce chemicals that have a foul smell and are harmful to
human health. Aerobic (oxygen requiring) bacteria degrade these organic wastes and
keep the water depleted in dissolved oxygen.
They also produce carbon dioxide which can be released in the atmosphere. High
organic matter may also contain nutrients supportive of eutrophication of the water
body.

39. Solution: d)
http://www.thehindu.com/business/Economy/wholesale-price-index-basedinflation-dips-to-record-low/article7105132.ece

40. Solution: d)

http://www.insightsonindia.com

INSIGHTS

Page 16

Insights Mock Tests 2015: Test 26 Solutions


There are several forms of dance-drama or folk theatre, such as the Nautanki of
Rajasthan, Uttar Pradesh and Bihar, the Bhavai of Gujarat, the irreverent Tamasha of
Maharashtra , the Bengali Jatra, the spectacular Yakshagana of Karnataka and
Theyyam of Kerala, all of which narrate legends of local heroes, kings and deities.

41. Solution: b)
India now measures GDP by market prices instead of factor costs, to take into
account gross value addition in goods and services as well as indirect taxes. The base
year has been shifted to 2011/12 from 2004/05 earlier.
The governments statistics department says the new method is more in line with
global practices and gives a better picture of economic activity.

http://www.livemint.com/Politics/xziKtmtOxBJntZb41p2hDL/India-GDP-seensurging-74-in-data-that-has-puzzled-economi.html

42. Solution: d)
To cure acidity you would need to take a base. All options except antacid are acids.
Amla is a citrous fruit.
Curd contains lactic acid. And spinach contains oxalic acid.
All these acids occur in nature.

43. Solution: c)
http://en.wikipedia.org/wiki/Bankim_Chandra_Chattopadhyay

44. Solution: c)

http://www.insightsonindia.com

INSIGHTS

Page 17

Insights Mock Tests 2015: Test 26 Solutions


Noncooperation movement, (September 1920February 1922), unsuccessful attempt,
organized by Mohandas Gandhi, to induce the British government of India to grant
self-government, or swaraj, to India. It arose from the outcry over the massacre at
Amritsar in April 1919, when the British killed several hundred Indians, and from
later indignation at the governments alleged failure to take adequate action against
those responsible. Gandhi strengthened the movement by supporting (on nonviolent
terms) the contemporaneous Muslim campaign against the dismemberment of
Turkey after World War I.
The movement was to be nonviolent and to consist of the resignations of titles; the
boycott of government educational institutions, the courts, government service,
foreign goods, and elections; and the eventual refusal to pay taxes. Noncooperation
was agreed to by the Indian National Congress at Calcutta (now Kolkata) in
September 1920 and launched that December. In 1921 the government, confronted
with a united Indian front for the first time, was visibly shaken, but a revolt by the
Muslim Moplahs of Kerala (southwestern India) in August 1921 and a number of
violent outbreaks alarmed moderate opinion. After an angry mob murdered police
officers at Chauri Chaura (February 1922), Gandhi himself called off the movement;
the next month he was arrested without incident. The movement marks the
transition of Indian nationalism from a middle-class to a mass basis.

45. Solution: d)
The CAG is appointed by the president of India by a warrant under his hand and
seal. The recommendation is passed by the Council of Ministers.
Recently, The Supreme Court rejected a plea seeking to quash appointment of Shashi
Kant Sharma as the Comptroller and Auditor General of India (CAG) on the
grounds of conflict of interest.
Read this important article.
http://www.ibnlive.com/news/india/cag-appointment-2-966864.html

http://www.insightsonindia.com

INSIGHTS

Page 18

Insights Mock Tests 2015: Test 26 Solutions

46. Solution: c)
It is a quasi-judicial body since it decides and disposes complaints relating to the RTI
Act.
It does not do delegated legislation because that would mean it is framing rules and
regulations. It only enforces the existing rules and regulations pertaining to the RTI
act.
It is also an administrative body as it provides information services to citizens of
India.

47. Solution: b)
Since the
Chief Minister is the head of the council of ministers, his resignation or death
automatically dissolves the council of ministers. The resignation or death of any
other minister, on the other hand, merely creates a vacancy, which the Chief Minister
may or may not like to fill.
Even though the ministers may be appointed from any of the houses, only by the
dissolution of the assembly does the council of minister dissolves. The Legislative
council is a permanent body is never dissolved.

48. Solution: c)
Satyagraha, ( Sanskrit and Hindi: holding onto truth) concept introduced in the
early 20th century by Mahatma Gandhi to designate a determined but nonviolent
resistance to evil. Gandhis satyagraha became a major tool in the Indian struggle
against British imperialism and has since been adopted by protest groups in other
countries. According to this philosophy, satyagrahispractitioners of satyagraha
achieve correct insight into the real nature of an evil situation by observing a
nonviolence of the mind, by seeking truth in a spirit of peace and love, and by
undergoing a rigorous process of self-scrutiny. In so doing, the satyagrahi
encounters truth in the absolute.

http://www.insightsonindia.com

INSIGHTS

Page 19

Insights Mock Tests 2015: Test 26 Solutions


By refusing to submit to the wrong or to cooperate with it in any way, the
satyagrahi asserts that truth. Throughout the confrontation with evil, the satyagrahi
must adhere to nonviolence, for to employ violence would be to lose correct insight.
Satyagrahis always warn their opponents of their intentions; satyagraha forbids any
tactic suggesting the use of secrecy to ones advantage. Satyagraha includes more
than civil disobedience. Its full range of application extends from the details of
correct daily living to the construction of alternative political and economic
institutions. Satyagraha seeks to conquer through conversion: in the end, there is
neither defeat nor victory but rather a new harmony.

49. Solution: b)
Recently, a Muslim MBA grad was denied a job in a diamond firm in Mumbai. The
question has been framed in that regard only.
http://timesofindia.indiatimes.com/india/Adani-hires-MBA-grad-who-wasdenied-job-for-being-a-Muslim/articleshow/47486173.cms

50. Solution: a)
Such a resolution must be supported by two-thirds of the members present and
voting. The resolution remains in force for one year; it can be renewed any number
of times but not exceeding one year at a time. The laws cease to have effect on the
expiration of six months after the resolution has ceased to be in force.
This provision does not restrict the power of a state legislature to make laws on the
same matter. But, in case of inconsistency between a state law and a parliamentary
law, the latter is to prevail.

51. Solution: c)
Gandhi felt an irresistible attraction to a life of simplicity, manual labour, and
austerity. In 1904after reading John Ruskins Unto This Last, a critique of
capitalismhe set up a farm at Phoenix near Durban where he and his friends could
live by the sweat of their brow. Six years later another colony grew up under
Gandhis fostering care near Johannesburg; it was named Tolstoy Farm for the
Russian writer and moralist, whom Gandhi admired and corresponded with. Those
two settlements were the precursors of the more-famous ashrams (religious retreats)
http://www.insightsonindia.com

INSIGHTS

Page 20

Insights Mock Tests 2015: Test 26 Solutions


in India, at Sabarmati near Ahmedabad (Ahmadabad) and at Sevagram near
Wardha. (Source: Britannica)

52. Solution: d)

53. Solution: c)
Take the weight of the person in kg. Then take the height in metres.
Divide the weight by the square of the height. If this figure is less than 18.5 then the
person would be considered undernourished. However, if this BMI is more than 25,
then a person is overweight.
Do remember that this criterion is not applicable to growing children.

54. Solutions: d)

http://www.insightsonindia.com

INSIGHTS

Page 21

Insights Mock Tests 2015: Test 26 Solutions


Round Table Conference, (193032), in Indian history, a series of meetings in three
sessions called by the British government to consider the future constitution of India.
The conference resulted from a review of the Government of India Act of 1919,
undertaken in 1927 by the Simon Commission, whose report was published in 1930.
The conference was held in London.
The first session (Nov. 12, 1930Jan. 19, 1931) had 73 representatives, from all Indian
states and all parties except the Indian National Congress, which was waging a civil
disobedience campaign against the government. Its principal achievement was an
insistence on parliamentarianisman acceptance by all, including the princes, of the
federal principleand on dominion status as the goal of constitutional development.
The second session (SeptemberDecember 1931) was attended by Mahatma Gandhi
as the Congress representative; it failed to reach agreement, either constitutionally or
on communal representation. The third session (Nov. 17Dec. 24, 1932) was shorter
and less important, with neither the Congress nor the British Labour Party attending.
The result of these deliberations was the Government of India Act, 1935, establishing
provincial autonomy and also a federal system that was never implemented.

55. Solution: d)
For the growth of the tertiary sector in India, there can be many reasons.
First, in any country several services such as hospitals, educational institutions, post
and telegraph services, police stations, courts, village administrative offices,
municipal corporations, defence, transport, banks, insurance companies, etc. are
required. These can be considered as basic services. In a developing country the
government has to take responsibility for the provision of these services.
Second, the development of agriculture and industry leads to the development of
services such as transport, trade, storage and the like, as we have already seen.
Greater the development of the primary and secondary sectors, more would be the
demand for such services.
Third, as income levels rise, certain sections of people start demanding many more
services like eating out, tourism, shopping, private hospitals, private schools,
professional training etc. You can see this change quite sharply in cities, especially in
big cities.
Fourth, over the past decade or so, certain new services such as those based on
information and communication technology have become important and essential.

http://www.insightsonindia.com

INSIGHTS

Page 22

Insights Mock Tests 2015: Test 26 Solutions

56. Solution: a)
Consider a firm which has employed more than required number of labour.
Each labour is doing some work but no one is fully employed. This is the situation of
underemployment, where people are apparently working but all of them are made
to work less than their potential.
This kind of underemployment is hidden in contrast to someone who does not have
a job and is clearly visible as unemployed. Hence, it is also called disguised
unemployment.
Over employment is the exact opposite of this situation. Workers are doing overtime
work here and producing more than they can in an ideal case. They are even ready
to sacrifice their income for some leisure.

57. Solution: b)
The majority of the provisions in the Constitution need to be amended by a special
majority of the Parliament, that is, a majority (that is, more than 50 per cent) of the
total membership of each House and a majority of two-thirds of the members of each
House present and voting.
Those provisions of the Constitution which are related to the federal structure of the
polity can be amended by a special majority of the Parliament and also with the
consent of half of the state legislatures by a simple majority.

58. Solution: b)
Gallium nitride is used in Blue LED whereas gallium phosphide is used in preparing
Red and Green LED.
http://www.popsci.com/article/technology/why-blue-led-worth-nobel-prize

59. Solution: b)
The Godavari River has a drainage area of 312,812 km2 (120,777 sq mi), which is
nearly one-tenth of the area of India and is greater than the areas of England and
http://www.insightsonindia.com

INSIGHTS

Page 23

Insights Mock Tests 2015: Test 26 Solutions


Ireland put together. Major tributaries of the river include the Purna (South),
Pravara, Indravati, Manjira River, Bindusara River, Sabari River, Wainganga, and
Wardha River.

60. Solution: a)

The various types of loans can be conveniently grouped as formal sector loans and
informal sector loans. Among the former are loans from banks and cooperatives. The
informal lenders include moneylenders, traders, employers, relatives and friends,
etc.
So, formal credit is more than the informal credit.

61. Solution: a)
The provisions of the act can be grouped into two categoriescompulsory and
voluntary. The compulsory (mandatory or obligatory) provisions of the act have to
be included in the state laws creating the new panchayati raj system. The voluntary
provisions, on the other hand, may be included at the discretion of the states. Thus
the voluntary pr-ovisions of the act ensure the right of the states to take local factors
http://www.insightsonindia.com

INSIGHTS

Page 24

Insights Mock Tests 2015: Test 26 Solutions


like geographical, politicoadministrative and others, into consideration while
adopting the new panchayati raj system.

62. Solution: c)
Lucknow Pact 1916, IndiaREAD VIEW HISTORY EDIT FEEDBACK Lucknow Pact,
(December 1916), agreement made by the Indian National Congress headed by
Maratha leader Bal Gangadhar Tilak and the All-India Muslim League led by
Muhammad Ali Jinnah; it was adopted by the Congress at its Lucknow session on
December 29 and by the league on Dec. 31, 1916. The meeting at Lucknow marked
the reunion of the moderate and radical wings of the Congress. The pact dealt both
with the structure of the government of India and with the relation of the Hindu and
Muslim communities.
On the former count, the proposals were an advance on Gopal Krishna Gokhales
political testament. Four-fifths of the provincial and central legislatures were to be
elected on a broad franchise, and half the executive council members, including
those of the central executive council, were to be Indians elected by the councils
themselves. Except for the provision for the central executive, these proposals were
largely embodied in the Government of India Act of 1919. The Congress also agreed
to separate electorates for Muslims in provincial council elections and for weightage
in their favour (beyond the proportions indicated by population) in all provinces
except the Punjab and Bengal, where they gave some ground to the Hindu and Sikh
minorities. This pact paved the way for Hindu-Muslim cooperation in the Khilafat
movement and Mohandas Gandhijis noncooperation movement from 1920.

63. Solution: d)
The tribal areas in the four states of Assam, Meghalaya, Tripura and Mizoram have
been constituted as autonomous districts. But, they do not fall outside the executive
authority of the state concerned. The district and regional councils within their
territorial jurisdictions can constitute village councils or courts for trial of suits and
cases between the tribes. They hear appeals from them.
The jurisdiction of high court over these suits and cases is specified by the governor.
http://www.insightsonindia.com

INSIGHTS

Page 25

Insights Mock Tests 2015: Test 26 Solutions

64. Solution: d)
When you buy garments, you will find information on instructions for washing.
When we buy medicines, on the packets, you might find directions for proper use
and information relating to side effects and risks associated with usage of that
medicine.
Rules have been made so that the manufacturer displays this information. It is
because consumers have the right to be informed about the particulars of goods and
services that they purchase. Consumers can then complain and ask for compensation
or replacement if the product proves to be defective in any manner.

65. Solution: b)
It is given by the World Food Prize Foundation.
http://www.britannica.com/EBchecked/topic/74242/Norman-Ernest-Borlaug

66. Solution: d)
Interests received can be inside the country (i.e., internal lending) or outside the
country (i.e., external lending). It means this income might be in both domestic and
foreign currencies.
Fiscal services also generate incomes for the government, i.e., currency printing,
stamp printing, coinage and medals minting, etc.
General Services also earn money for the government as the power distribution,
irrigation, banking, insurance, community services, etc.
Fees, Penalties and Fines received by the government also form a part of non-tax
revenue receipt.

67. Solution: c)
Uniform civil code in India is the debate to replace the personal laws based on the
scriptures and customs of each major religious community in the country with a

http://www.insightsonindia.com

INSIGHTS

Page 26

Insights Mock Tests 2015: Test 26 Solutions


common set governing every citizen. These laws are distinguished from public law
and cover marriage, divorce, inheritance, adoption and maintenance.
The laws already apply similarly for both men and women. In specific cases of
property inheritance etc., a uniform civil code will have no effect. However, there
will be a common law for all the communities in India.

68. Solution: d)
http://en.wikipedia.org/wiki/Dachigam_National_Park

69. Solution: b)
Cooking results in the loss of certain nutrients. Many useful proteins and
considerable amounts of minerals are lost if excess water is used during cooking and
is then thrown away.
Vitamin C gets easily destroyed by heat during cooking.
Repeated washing and peeling off of the skin also lead to loss of nutrients.

70. Solution: a)
Usually crops require a lot of nitrogen to make proteins. After the harvest, the soil
becomes deficient in nitrogen. Though nitrogen gas is available in plenty in the air,
plants cannot use it in the manner they can use carbon dioxide. They need nitrogen
in a soluble form.
The bacterium called Rhizobium can take atmospheric nitrogen and convert it into a
soluble form. But Rhizobium cannot make its own food. So it lives in the roots of
gram, peas, moong beans and other legumes and provides them with nitrogen. Most
of the pulses (dals) are obtained from leguminous plants. In return, the plants
provide food and shelter to the bacteria. They have a symbiotic relationship.

http://www.insightsonindia.com

INSIGHTS

Page 27

Insights Mock Tests 2015: Test 26 Solutions


71. Solution: d)
Unlike the SC, in the appointment to HC judges, the Governor of the state is also
consulted.
HCs can issue writs for legal rights as well as fundamental rights. The SC can issue
writs only for enforcing fundamental rights.
Both can interpret the constitution, but the version of the SC will prevail over the
HCs.
Both can review central as well as state laws. Even here the view of the SC will
prevail over HCs.

72. Solution: c)
Khilafat movement, force that arose in India in the early 20th century as a result of
Muslim fears for the integrity of Islam. These fears were aroused by Italian (1911)
and Balkan (191213) attacks on Turkeywhose sultan, as caliph, was the religious
head of the worldwide Muslim communityand by Turkish defeats in World War I.
They were intensified by the Treaty of Svres (August 1920), which not only
detached all non-Turkish regions from the empire but also gave parts of the Turkish
homeland to Greece and other non-Muslim powers.
A campaign in defense of the caliph was launched, led in India by the brothers
Shaukat and Muammad Al and by Abul Kalam Azad. The leaders joined forces
with Mahatma Gandhis noncooperation movement for Indian freedom, promising
nonviolence in return for his support of the Khilafat movement. In 1920 the latter
movement was marred by the ijrat, or exodus, from India to Afghanistan of about
18,000 Muslim peasants, who felt that India was an apostate land. It was also
tarnished by the Muslim Moplah rebellion in south India (Malabar) in 1921, the
excesses of which deeply stirred Hindu India. Gandhis suspension of his movement
and his arrest in March 1922 weakened the Khilafat movement still further. It was
further undermined when Mustafa Kemal Atatrk drove the Greeks from western
Asia Minor in 1922 and deposed the Turkish sultan in the same year; it finally
collapsed when he abolished the caliphate altogether in 1924.

http://www.insightsonindia.com

INSIGHTS

Page 28

Insights Mock Tests 2015: Test 26 Solutions

73. Solution: a)
The winds would have flown in the north-south direction from north to south, or
from south to north. A change in direction is however, caused by the rotation of the
earth which results in the generation of the coriolis force.

74. Solution: c)

http://www.insightsonindia.com

INSIGHTS

Page 29

Insights Mock Tests 2015: Test 26 Solutions


Some toilets use earthworms to decompose faecal matter and kitchen organic waste.
A design of this toilet in which humans excreta is treated by earthworms has been
tested in India.
It has been found to be a novel, low water-use toilet for safe processing of human
waste. The operation of the toilet is very simple and hygienic. The human excreta is
completely converted to vermi cakes a resource much needed for soil.
There is no heating element and the system is not intended to operate above 35 C, to
protect the worms.

75. Solution: a)
Earthworms and microbes present in the soil are friends of the farmer since they
further turn and loosen the soil and add humus to it.
In addition, dead plants and animals get decomposed by soil organisms. In this way,
various nutrients held in the dead organisms are released back into the soil. These
nutrients are again absorbed by plants.

76. Solution: a)

http://www.insightsonindia.com

INSIGHTS

Page 30

Insights Mock Tests 2015: Test 26 Solutions


In this diagram, all the factors that can possibly affect the transmission of the EM
waves are given.

77. Solution: d)
Hard water contains calcium and magnesium ions. These ions form insoluble
calcium and magnesium soaps respectively when sodium or potassium soaps are
dissolved in hard water.
These insoluble soaps separate as scum in water and are useless as cleansing agent.
In fact these are hinderance to good washing, because the precipitate adheres onto
the fibre of the cloth as gummy mass. Hair washed with hard water looks dull
because of this sticky precipitate. Dye does not absorb evenly on cloth washed with
soap using hard water, because of this gummy mass.
78. Solution: a)
The rate of biomass production is called productivity. It can be divided into gross
primary productivity (GPP) and net primary productivity (NPP). Gross primary
productivity of an ecosystem is the rate of production of organic matter during
photosynthesis. A considerable amount of GPP is utilised by plants in respiration.
Gross primary productivity minus respiration losses (R), is the net primary
productivity (NPP).
Net primary productivity is the available biomass for the consumption to
heterotrophs (herbiviores and decomposers).
Secondary productivity is defined as the rate of formation of new organic matter by
consumers.

79. Solution: c)
For the purpose of holding direct elections to the Lok Sabha, each state is divided
into territorial constituencies. In this respect, the Constitution makes the following
two provisions:

http://www.insightsonindia.com

INSIGHTS

Page 31

Insights Mock Tests 2015: Test 26 Solutions

Each state is allotted a number of seats in the Lok Sabha in such a manner that
the ratio between that number and its population is the same for all states.
This provision does not apply to a state having a population of less than six
millions.
Each state is divided into territorial constituencies in such a manner that the
ratio between the population of each constituency and the number of seats
allotted to it is the same throughout the state.

80. Solution: d)
For a list (and some basic info) on all critically endangered animal species of India,
please refer to this document prepared by the MoEF. Its very comprehensive.
http://www.moef.nic.in/downloads/publicinformation/critically_endangered_booklet.pdf
For fishes/birds/reptiles etc. refer to
http://en.wikipedia.org/wiki/List_of_endangered_animals_in_India#Critically_en
dangered
81. Solution: c)
The trophic level represents a functional level, not a species as such. A given species
may occupy more than one trophic level in the same ecosystem at the same time; for
example, a sparrow is a primary consumer when it eats seeds, fruits, peas, and a
secondary consumer when it eats insects, worms etc.
In most ecosystems, all the pyramids, of number, of energy and biomass are upright,
i.e., producers are more in number and biomass than the herbivores, and herbivores
are more in number and biomass than the carnivores. Also energy at a lower trophic
level is always more than at a higher level.
There are exceptions to this generalisation: The pyramid of biomass in sea is
generally inverted because the biomass of fishes far exceeds that of phytoplankton.

82. Solution: a)

http://www.insightsonindia.com

INSIGHTS

Page 32

Insights Mock Tests 2015: Test 26 Solutions


The most typical and the most well known
building in Indo Islamic is Panch Mahal, the
highest and the most impressive structure,
called the palace of five storeys. It is based on
the Hindu system of trabeate structure,
consisting of pillar, architrave, and brackets,
with the only exception of the topmost domed
pavilion, purposefully thrown out of the centre
that crowns the entire building. The tower was
perhaps used for recreation by the emperor
and members of the royal household. The idea
behind this impressive structure of
diminishing storeys, one on top of the other,
consisting of open, terraces in front of the
covered areas, was comfort, shade and airy
open pillared verandahs provided by
perforated railings, constructed with an eye to
providing shade and fresh air, to the
inhabitants seated on the cool floors.
The Diwan-i-Khas or Hall of Private Audience,
is of a unique design. It is a square chamber
with three openings on each side and a richly
Panch Mahal, Fatehpur Sikri,
carved column in the centre supporting a
Agra, Uttar Pradesh
magnificent flower shaped capital. Thorough
ventilation is provided by placing on all sides
perforated windows opposite each other on
every wall. The charming balcony supported
by a circular top capital, runs round the halls
whole length of the 4 sides on the first floor
level, supported by brackets. It is believed that
the central place was occupied by the
Emperor's throne while his Ministers sat at the
corners or on the peripheral passage.
The Turkish Sultana's house consists of a small
chamber surrounded by a verandah. It is
beautifully carved both on the outside and
inside; particularly remarkable being the wide
dado carved with panels, portraying jungle
http://www.insightsonindia.com

INSIGHTS

Page 33

Insights Mock Tests 2015: Test 26 Solutions


scenes with animals, birds and trees. It is the
most ornamented building in a "gigantic jewel
casket" says Fergusson.

83. Solution: b)
The individual transitional communities are termed seral stages or seral
communities.
In the successive seral stages there is a change in the diversity of species of
organisms, increase in the number of species and organisms as well as an increase in
the total biomass.
The present day communities in the world have come to be because of succession
that has occurred over millions of years since life started on earth. Actually
succession and evolution would have been parallel processes at that time.

84. Solution: c)
Moreover, when rocks are weathered, minute amounts of these phosphates dissolve
in soil solution and are absorbed by the roots of the plants.
Herbivores and other animals obtain this element from plants. The waste products
and the dead organisms are decomposed by phosphate-solubilising bacteria
releasing phosphorus.
So, unlike carbon cycle, there is no respiratory release of phosphorus into
atmosphere.

85. Solution: d)

http://www.insightsonindia.com

INSIGHTS

Page 34

Insights Mock Tests 2015: Test 26 Solutions


Healthy ecosystems are the base for a wide range of economic, environmental and
aesthetic goods and services. The products of ecosystem processes are named as
ecosystem services, for example, healthy forest ecosystems purify air and water,
mitigate droughts and floods, cycle nutrients, generate fertile soils, provide wildlife
habitat, maintain biodiversity, pollinate crops, provide storage site for carbon and
also provide aesthetic, cultural and spiritual values.
Though value of such services of biodiversity is difficult to determine, it seems
reasonable to think that biodiversity should carry a hefty price tag.

86. Solution: d)
Biological magnification often refers to the process whereby certain substances such
as pesticides or heavy metals move up the food chain, work their way into rivers or
lakes, and are eaten by aquatic organisms such as fish, which in turn are eaten by
large birds, animals or humans. The substances become concentrated in tissues or
internal organs as they move up the chain.
Bio-accumulants are substances that increase in concentration in living organisms as
they take in contaminated air, water, or food because the substances are very slowly
metabolized or excreted.
The following is an example showing how bio-magnification takes place in nature:
An anchovy eats zoo-plankton that have tiny amounts of mercury that the zooplankton has picked up from the water throughout the anchovies lifespan. A tuna
eats many of these anchovies over its life, accumulating the mercury in each of those
anchovies into its body. If the mercury stunts the growth of the anchovies, that tuna
is required to eat more little fish to stay alive. Because there are more little fish being
eaten, the mercury content is magnified

87. Solution: b)
Purchasing power parity is used worldwide to compare the income levels in
different countries. PPP thus makes it easy to understand and interpret the data of
each country.
Let's say that a pair of shoes costs Rs 2500 in India. Then it should cost $50 in
America when the exchange rate is 50 between the dollar and the rupee.
http://www.insightsonindia.com

INSIGHTS

Page 35

Insights Mock Tests 2015: Test 26 Solutions


The theory aims to determine the adjustments needed to be made in the exchange
rates of two currencies to make them at par with the purchasing power of each other.

88. Solution: d)
It is known that every individual has unique fingerprints. These occur at the tips of
the fingers and have been used for identification for a long time but these can be
altered by surgery.
A sequence of bases on DNA is also unique for a person and information regarding
this is called DNA fingerprinting. It is same for every cell and cannot be altered by
any known treatment. DNA fingerprinting is now also used in forensic laboratories
for identification of criminals.

89. Solution: c)
Above the troposphere, between 10 and 50 km above sea level lies stratosphere.
Troposphere is a turbulent, dusty zone containing air, much water vapour and
clouds. This is the region of strong air movement and cloud formation. The
stratosphere, on the other hand, contains dinitrogen, dioxygen, ozone and little
water vapour.
Atmospheric pollution is generally studied as tropospheric and stratospheric (also
because of the presence of ozone) pollution.

90. Solution: c)
Just as the glass in a greenhouse holds the suns warmth inside, atmosphere traps
the suns heat near the earths surface and keeps it warm. This is called natural
greenhouse effect because it maintains the temperature and makes the earth perfect
for life.
In a greenhouse, visible light passes through the transparent glass and heats up the
soil and the plants. The warm soil and plants emit infrared radiations. Since glass is
http://www.insightsonindia.com

INSIGHTS

Page 36

Insights Mock Tests 2015: Test 26 Solutions


opaque to infrared (heat) radiations, it partly reflects and partly absorbs these
radiations. This mechanism keeps the energy of the sun trapped in the greenhouse.

91. Solution: c)
The air around the city of Agra, where the Taj Mahal is located, contains fairly high
levels of sulphur and nitrogen oxides. This caused heavy acid rains.
The Government of India announced an action plan that aimed at clearing the air in
the Taj Trapezium an area that includes the towns of Agra, Firozabad, Mathura
and Bharatpur. Under this plan more than 2000 polluting industries lying inside the
trapezium would switch over to the use of natural gas or liquefied petroleum gas
instead of coal or oil. A new natural gas pipeline would bring more than half a
million cubic metres of natural gas a day to this area.
People living in the city will also be encouraged to use liquefied petroleum gas in
place of coal, kerosene or firewood. Vehicles plying on highways in the vicinity of
Taj would be encouraged to use low sulphur content diesel.

92. Solution: a)
The word smog is derived from smoke and fog.
This is the most common example of air pollution that occurs in many cities
throughout the world.
Photochemical smog occurs in warm, dry and sunny climate. The main components
of the photochemical smog result from the action of sunlight on unsaturated
hydrocarbons and nitrogen oxides produced by automobiles and factories.
Photochemical smog has high concentration of oxidising agents and is, therefore,
called as oxidising smog.

93. Solution: d)
Many techniques are used to control or reduce the formation of photochemical
smog. If we control the primary precursors of photochemical smog, such as NO 2 and
hydrocarbons, the secondary precursors such as ozone and PAN, the photochemical
smog will automatically be reduced.

http://www.insightsonindia.com

INSIGHTS

Page 37

Insights Mock Tests 2015: Test 26 Solutions


Usually catalytic converters are used in the automobiles, which prevent the release
of nitrogen oxide and hydrocarbons to the atmosphere. Certain plants e.g., Pinus,
Juniparus, Quercus, Pyrus and Vitis can metabolise nitrogen oxide and therefore,
their plantation could help in this matter.

94. Solution: a)
A patent is an exclusive right granted for an invention, which is a product or a
process that provides, in general, a new way of doing something, or offers a new
technical solution to a problem.
To get a patent, technical information about the invention must be disclosed to the
public in a patent application.
In principle, the patent owner has the exclusive right to prevent or stop others from
commercially exploiting the patented invention. In other words, patent protection
means that the invention cannot be commercially made, used, distributed, imported
or sold by others without the patent owner's consent.

95. Solution: d)
The Constitution of India establishes a federal system of government. It contains all
the usual features of a federation, viz., two government, division of powers, written
Constitution, supremacy of Constitution, rigidity of Constitution, independent
judiciary and bicameralism.
However, the Indian Constitution also contains a large number of unitary or nonfederal features, viz., a strong Centre, single Constitution, single citizenship,
flexibility of Constitution, integrated judiciary, appointment of state governor by the
Centre, all-India services, emergency provisions, and so on.
But, all these do not mean that state governments draw power from Central
government or are subservient to the Union government. The Union government is
ultimately responsible for the overall development of the nation; its unity, territorial
integrity, and security. Hence, it has been given more powers than the state
governments.

96. Solution: a)
http://www.insightsonindia.com

INSIGHTS

Page 38

Insights Mock Tests 2015: Test 26 Solutions


Plants absorb mineral nutrients and water from the soil. Not all the water absorbed
is utilised by the plant. The water evaporates through the stomata present on the
surface of the leaves by the process of transpiration.
The evaporation of water from leaves generates a suction pull (the same that you
produce when you suck water through a straw) which can pull water to great
heights in the tall trees. Transpiration also cools the plant.

97. Solution: a)
The Mission has been conceptualized and evolved keeping in view the ever
increasing computing demand of the scientific and academic community in the
country, international technology trends and roadmaps of leading countries in the
area, strategic importance and emergence of supercomputing as a benchmark for
Scientific & Technological advancements. Two key departments of the Government
of India, DeitY and DST will be implementing the mission jointly through two
leading organizations. These are the Centre for Development of Advanced
Computing (C-DAC) and the Indian Institute of Science (IISc), Bangalore.
The Mission envisages empowering our national academic and R&D institutions
spread over the country by installing a vast supercomputing grid comprising of
more than 70 high-performance computing facilities. These supercomputers will also
be networked on the National Supercomputing grid over the National Knowledge
Network (NKN). The NKN is another programme of the government which
connects academic institutions and R&D labs over a high speed network. Academic
and R&D institutions as well as key user departments/ministries would participate
by using these facilities and develop applications of national relevance. The Mission
also includes development of highly professional High Performance Computing
(HPC) aware human resource for meeting challenges of development of these
applications.

98. Solution: a)

http://www.insightsonindia.com

INSIGHTS

Page 39

Insights Mock Tests 2015: Test 26 Solutions


Women writers like Ghosha, Lopamudra, Gargi, Maitreyi, Apala, Romasha
Brahmavadini, etc., right from the days of the Vedas (6000 B.C. 4000 B.C.), focused
on the image of women in mainstream Sanskrit literature.

99. Solution: a)
The Supreme Court has been consistently in news for its heightened activism since
last few years. While this question may seem very factual, but UPSC is very
unpredictable and such questions can be very easily expected in the examination.
Moreover, Regulating Act of 1773 is of great constitutional importance as (a) it was
the first step taken by the British Government to control and regulate the affairs of
the East India Company in India; (b) it recognised, for the first time, the political and
administrative functions of the Company; and (c) it laid the foundations of central
administration in India.

100.

Solution: b)

The Dhamekh stupa at Sarnath is an imposing


cylindrical structure (ht. 43.5 m., dia at base 28.3 m.) of
the Gupta age, partly built of stone and partly of brick.
Its stone basement has eight projecting faces with large
niches for statuary and is further adorned with
delicately-carved floral and geometrical patterns.
Making the holy spot of the enlightenment of the
Master, this site is looked upon with, greatest sanctity
and became a flourishing Buddhist establishment with
numerous temples, stupas and monasteries. According
to tradition a large number of shrines and memorials
were created at the site to commemorate the incidents
before and after enlightenment.
The main brick built shrine known as the Mahabodhi
temple which appears to have been originally erected
in circa 2nd century A.D. is encumbered with heavy
renovation, the four corner-towers being an arbitrary
addition of circa 14th century A.D. Its central tower,
http://www.insightsonindia.com

INSIGHTS

Dhamekh Stupa at
Sarnath,
Utter Pradesh

Page 40

Insights Mock Tests 2015: Test 26 Solutions


standing on a high plinth, is about 55m. high and is a
straight-edged pyramid of seven storeys, by pilasters
and chaitya niches.

http://www.insightsonindia.com

INSIGHTS

Page 41

INSIGHTS ON INDIA MOCK PRELIMINARY EXAM - 2015


INSIGHTS ON INDIA MOCK TEST - 27
GENERAL STUDIES

PAPER-I
Time Allowed: 2 Hours

Maximum Marks: 200

INSTRUCTIONS
1. IMMEDITELY AFTER THE COMMENCEMENT OF THE EXAMINATION, YOU SHOULD
CHECK THAT THIS TEST BOOKLET DOES NOT HAVE ANY UNPRINTED OR TORN OR MISSING
PAGES OR ITEMS, ETC. IF SO, GET IT REPLACED BY A COMPLETE TEST BOOKLET.
2. You have to enter your Roll Number on the Test
Booklet in the Box provided alongside. DO NOT
Write anything else on the Test Booklet.
4. This Test Booklet contains 100 items (questions). Each item is printed only in English. Each item
comprises four responses (answers). You will select the response which you want to mark on the
Answer Sheet. In case you feel that there is more than one correct response, mark the response which
you consider the best. In any case, choose ONLY ONE response for each item.
5. You have to mark all your responses ONLY on the separate Answer Sheet provided. See directions in
the Answer Sheet.
6. All items carry equal marks.
7. Before you proceed to mark in the Answer Sheet the response to various items in the Test Booklet, you
have to fill in some particulars in the Answer Sheet as per instructions sent to you with your
Admission Certificate.
8. After you have completed filling in all your responses on the Answer Sheet and the examination has
concluded, you should hand over to the Invigilator only the Answer Sheet. You are permitted to take
away with you the Test Booklet.
9. Sheets for rough work are appended in the Test Booklet at the end.
10. Penalty for wrong answers :
THERE WILL BE PENALTY FOR WRONG ANSWERS MARKED BY A CANDIDATE IN THE
OBJECTIVE TYPE QUESTION PAPERS.
(i) There are four alternatives for the answer to every question. For each question for which a
wrong answer has been given by the candidate, one-third of the marks assigned to that
question will be deducted as penalty.
(ii) If a candidate gives more than one answer, it will be treated as a wrong answer even if one of
the given answers happens to be correct and there will be same penalty as above to that
question.
(iii)

If a question is left blank, i.e., no answer is given by the candidate, there will be no penalty
for that question.
http://www.insightsonindia.com

INSIGHTS ON INDIA MOCK TEST SERIES FOR CIVIL SERVICES PRELIMINARY EXAM 2015
http://www.insightsonindia.com

INSIGHTS

Page 1

1. Consider the following about the Food


Safety and Standards Authority of
India (FSSAI).
1. It is a statutory body.
2. Union Ministry of Health & Family
Welfare is the Administrative
Ministry for the implementation of
FSSAI.
3. It provides scientific and technical
advice to Central and state
governments on food safety.
4. Only FSSAI can ban an unsafe food
from market.
Choose the correct answer using the codes
below.
a)
b)
c)
d)

2 and 4 only
1 and 3 only
1, 2 and 3 only
All of the above

2. India has recently slapped antidumping duty on imports of certain


steel products from China, Malaysia
and South Korea. Anti-dumping duty
is applied when
a) Imports are much cheaper than
their actual market price
b) Imports are much costlier than
their actual market price
c) Imports exceed the allowed quota
by the government
d) Countries collude to reduce the
quantity of imports to a particular
nation

3. Which of the following is/are the


benefits to India by ratifying the IndiaBangladesh Land Boundary
agreement?
http://www.insightsonindia.com

1. It will allow Indian security forces


seamless access to both sides of the
land border.
2. It will settle the question of
citizenship for various people in the
disputed enclaves around the land
boundary.
3. It would improve access to the
north-eastern states.
Choose the correct answer using the codes
below.
a)
b)
c)
d)

1 and 2 only
1 and 3 only
2 and 3 only
All of the above

4. Members of which of the following


security forces in India are known as
Black Cats?
a) National Security Guard
b) Central Reserve Police Force
c) Rapid Action Force
d) None of the above

5. In recorded human history, which of


the following human organs have been
transplanted?
1. Heart
2. Skull and scalp partially
3. Intestine
4. Brain
Choose the correct answer using the codes
below.
a)
b)
c)
d)

INSIGHTS

1, 2 and 3 only
1 and 3 only
2, 3 and 4 only
1 and 2 only

Page 2

6. Recently, the Union Government has


launched nationwide Jal Kranti
Abhiyan. It aims at
a) Providing all villages with piped
drinking water facility
b) Turning one water scared village in
each district of the country into
water surplus village
c) Rejuvenating the natural water
sources in all Indian villages to
reduce water deficit and tackle
drought
d) Water saving in all the Indian
villages through awareness
generation and financial incentives

7. Kitab-ul-Hind which recorded various


aspects of India among other things as
seen by a traveller, was written by
a) Ibn Battuta
b) Al Biruni
c) Ibn Juzayy
d) Shaikh Ali Hazin

8. Under WTO rules, a nation can ban


imports of a particular commodity
from a nation in which of the following
cases?
1. To protect domestic industries
2. On grounds that the imported
commodity does not comply with
international standards
3. On ground of public health
Choose the correct answer using the codes
below.
a)
b)
c)
d)

1 and 2 only
1 and 3 only
2 and 3 only
All of the above

http://www.insightsonindia.com

9. In banking, the difference between real


and nominal interest rates reflects
a) Cost of loan to the banks
b) Rate of Inflation
c) Profit to banks
d) Difference between repo and
reverse repo rate

10. Corporate farming refers to a practice


where
1. Land is owned by the corporate
2. Farmer undertakes a contract
based on a payment to grow crops
on his land
3. Corporate involve themselves in
front end agricultural marketing
activities
Choose the correct answer using the codes
below.
a)
b)
c)
d)

1 and 3 only
2 and 3 only
1 and 2 only
2 only

11. With reference to Franois Bernier,


consider the following statements:
1. He extensively travelled in India
during the rule of Aurangzeb and
Dara Shikoh
2. One of his books is considered the
first published post-Classical
classification of humans into
distinct races
Which of the above statements is/are correct?
a)
b)
c)
d)

INSIGHTS

1 Only
2 Only
Both
None

Page 3

12. The Tripartite Free Trade Area (TFTA)


has been signed between
a) African Nations
b) Asia-Pacific rim nations USA,
Japan, South Korea
c) West European nations France,
Germany and Greece
d) South-east Asian nations, Russia
and Australia

13. The Constitution confers which of the


following rights and privileges on the
citizens of India, and denies the same
to aliens?
1. Cultural and educational rights
2. Right to freedom
3. Right against exploitation
4. Right to equality of opportunity
Choose the correct answer using the codes
below.
a)
b)
c)
d)

1 and 2 only
2 and 3 only
3 and 4 only
1, 2 and 4 only

14. Consider the following statements:


1. Climate refers to the sum total of
weather conditions and variations
over a large area for a long period
of time (more than thirty years)
2. The elements of weather and
climate are the same
3. Weather refers to the state of the
atmosphere over an area at any
point of time
Which of the above statements is/are correct?
a)
b)
c)
d)

1 and 2 Only
3 and 1 Only
2 and 3 Only
All

http://www.insightsonindia.com

15. The Government of India Act of 1919,


also known as Montagu-Chelmsford
Reforms provided for the system of
diarchy. This meant that
a) The legislative sphere of central and
provincial government was clearly
demarcated
b) Instead of a single legislative
chamber, two councils
(bicameralism) will take decisions
c) A part of provincial legislation will
be handled by the Governor and the
rest by the legislative council
d) None of the above

16. During Presidents rule in a state


1. State council of ministers is
dissolved
2. Central government takes over the
executive machinery of the state
3. The President becomes empowered
to make laws with respect to any
matter in state list
Choose the correct answer using the codes
below.
a)
b)
c)
d)

1 and 2 only
2 and 3 only
1 and 3 only
1 only

17. With reference to Permanent


Settlement system of land revenue,
consider the following statements:
1. It was an agreement between the
East India Company and Bengali
landlords to fix revenues to be
raised from land
2. The British officials hopes to
encourage investment in
agriculture through this initiative
INSIGHTS

Page 4

3. By introducing this system, the


British hoped to establish a class of
yeomen farmers and rich
landowners who would be loyal to
the Company
Which of the above statements is/are
INCORRECT?
a)
b)
c)
d)

1 and 2 Only
1 and 3 Only
2 and 3 Only
All

18. Consider the following about IAP


HealthPhone programme recently
launched by Union government.
1. It is a digital mass education
programme.
2. It will address malnutrition in
women and children.
3. It is supported by UNICEF.
4. It is a Public-Private Partnership
(PPP) initiative.
Choose the correct answer using the codes
below.
a)
b)
c)
d)

1 and 2 only
1, 3 and 4 only
2, 3 and 4 only
All of the above

19. Under which of the following writs a


court issue command to a public
official asking him to perform his
official duties that he has failed or
refused to perform?
a) Certiorari
b) Mandamus
c) Prohibition
d) Quo-Warranto

http://www.insightsonindia.com

20. The Gross National Product (GDP) of a


nation is NOT affected in the short-run
by
1. Growing trade deficit
2. Growing flow of remittances
3. Cutting down on interest rates in
the economy
Choose the correct answer using the codes
below.
a)
b)
c)
d)

1 and 3 only
2 and 3 only
1 and 2 only
It is affected by all of the above.

21. Concerning the Central government,


which of the following are
constitutional mandates/duties?
1. To devolve a share of income
proceeds to states
2. To protect states from
constitutional breakdown
3. To protect states from external and
internal aggression
Choose the correct answer using the codes
below.
a)
b)
c)
d)

1 and 2 only
2 and 3 only
1 and 3 only
All of the above

22. To whom does the below description


refer to?
1. He oversaw the consolidation of
British control over much of
peninsular India, setting the stage
for the British Raj
2. He was instrumental in enacting
administrative and legal reforms
that fundamentally altered civil
INSIGHTS

Page 5

administration and land


management practices in India
3. He introduced a system of circuit
courts with a superior court that
met in Calcutta and had the power
of review over circuit court
decisions
Choose the correct answer using the codes
below:
a)
b)
c)
d)

Lord Dalhousie
Lord Cornwallis
Lord William Bentinck
Lord Ripon

23. Under the system of Scientific


Forestry in colonial times
a) New trees are added to a region as
per its ecology
b) Old trees are cut and new monovariety of trees are grown in its
place
c) Biodiversity of a forest is enhanced
by adding exotic and native species
d) None of the above

24. With reference to Nader Shahs


invasion of Mughal India and the
subsequent plunder, consider the
following statements:
1. At the time of Nader Shahs
invasion, Aurangzeb was old and
facing succession wars from his
sons
2. He defeated Mughal army at the
Battle of Karnal
3. Mughals defeat paved the way for
the British to fill the power vacuum

b) 1 and 2 Only
c) 1 and 3 Only
d) All
25. The Ashobaa cyclonic depression over
the Arabian Sea has shortly disrupted
the normal flow of the southwest
monsoon winds because
1. Southwest monsoon winds have
now turned towards the north and
the west after entering the
Southern Peninsula
2. The cyclonic depression has
delayed the heating, condensation
and precipitation in the Indian
ocean
Which of the above is/are true?
a)
b)
c)
d)

1 only
2 only
Both 1 and 2
None

26. Consider the following about the role


and functions of the RBI.
1. It is the custodian of the stipulated
cash reserves of the Commercial
Banks.
2. It manages the banking needs of
the government.
3. It controls the money supply in the
economy.
4. It manages the Foreign Exchange
Management Act.
Choose the correct answer using the codes
below.
a)
b)
c)
d)

1, 2 and 3 only
2, 3 and 4 only
1, 2 and 4 only
All of the above

Which of the above statements is/are correct?


a) 2 and 3 Only
http://www.insightsonindia.com

INSIGHTS

Page 6

27. If the Ministry of Environment and


Forests (MoEF) declares some wild
animals as vermin it would
1. Allow their liberal hunting in the
forests
2. Allow their movement from one
forest to other
3. Declare them as no beneficial to
humans
Choose the correct answer using the codes
below.
a)
b)
c)
d)

1 and 2 only
2 and 3 only
1 and 3 only
All of the above

28. With reference to Manikkavachakar,


consider the following statements:
1. He was a Sangam poet and wrote
Vaishnav hymns in Tamil
2. His work is a poetic expression of
the joy of God-experience, the
anguish of being separated from
God
Which of the above statements is/are correct?
a)
b)
c)
d)

1 Only
2 Only
Both
None

29. Consider the following statements:


1. India lies in the region of north
easterly winds
2. North-easterly winds originate
from the subtropical high-pressure
belt of the northern hemisphere
3. North-easterly winds are known as
doldrums

a)
b)
c)
d)

3 and 1 Only
2 and 1 Only
3 and 2 Only
All

30. If the tax expenditure of the


government increases, it implies that
1. Government is giving greater tax
exemptions
2. Revenue expenditure of the
government is increasing
3. Government is spending more tax
money in non-plan sector of the
economy
Choose the correct answer using the codes
below.
a)
b)
c)
d)

1 and 3 only
2 and 3 only
1 and 2 only
1 only

31. In India, which of these might lead to


inflationary pressures in the shortrun?
1. Printing more money
2. Rupee Depreciation
3. High tax rebates to Indian citizens
Choose the correct answer using the codes
below.
a)
b)
c)
d)

1 and 3 only
2 and 3 only
1 and 2 only
All of the above

Which of the above statements is/are correct?


http://www.insightsonindia.com

INSIGHTS

Page 7

32. The Greek depression refers to the


phenomenon which started with
a) Greece nearly defaulting on
sovereign debt
b) Greece suffering very low economic
growth rates immediately after the
2008 recession
c) Greece being pushed out of the
European Monetary union because
of debt default
d) The massive austerity programs in
Greece leading to high debt levels
and unemployment

33. With reference to Subsidiary Alliance,


consider the following statements:
1. Subsidiary Alliance was a system
devised by Lord Dalhousie in 1798.
2. According to this, the British would
be responsible for protecting their
ally from external and internal
threats to their power
3. According to this, the ally could
enter into agreements with other
rulers or engage in warfare only
with the permission of the British
Which of the above statements is/are correct?
a)
b)
c)
d)

1 and 2 Only
2 Only
2 and 3 Only
All

34. Which of these activities are legally


permitted for banks, but NOT for NonBanking Financial Institutions
(NBFIs)?
1. Offering loans
2. Offering deposits
3. Offering cheque withdrawal facility

http://www.insightsonindia.com

Choose the correct answer using the codes


below.
a)
b)
c)
d)

1 and 3 only
2 and 3 only
1 and 2 only
All of the above

35. Which of the following may force


marine life to poles from the equator?
1. Rising sea levels
2. Rising sea temperatures
3. Increased mineralization of sea
water
4. Increased population of
phytoplanktons in the ocean
Choose the correct answer using the codes
below.
a)
b)
c)
d)

1 and 2 only
2 and 3 only
2 and 4 only
1 and 3 only

36. Consider the following statements:


1. The western cyclonic disturbances
experienced in the north and northwestern parts of India are brought
in by this westerly flow
2. Western disturbances are
subtropical storms
3. Western Disturbances are
important to the development of
the Kharif crop in the northern
subcontinent
Which of the above statements is/are correct?
a)
b)
c)
d)
INSIGHTS

1 and 2 Only
2 and 3 Only
3 and 1 Only
All
Page 8

37. Which one of the following bioreserves of India is not included in the
world network of bioreserve?
a) Manas
b) Gulf of Mannar
c) Nokrek
d) Nanda devi

38. Which of these statements is/are true


about Endemic species?
1. They are naturally found only in an
exclusive area.
2. They cannot be introduced in areas
other than where they are found.
3. Exotic species and endemic species
cannot coexist.
Choose the correct answer using the codes
below.
a)
b)
c)
d)

1 and 2 only
2 and 3 only
1 and 3 only
1 only

39. Consider the following about the


scientific technique of growing testtube babies.
1. It is done when the female is totally
incapable of giving birth.
2. Baby is fertilized and grown under
laboratory conditions.
3. It also leads to genetic
modifications in the baby.

40. Consider the following statements:


1. In Mahayana and Vajrayana
Buddhism, she is a deva or
bodhisattva associated with light
and the sun
2. She has been depicted with one,
three, five or six faces and two, six,
eight, ten or twelve arms; three
eyes; in her many-faced
manifestations one of her faces is
that of a sow
To which God/Goddess does above
description refer to?
a)
b)
c)
d)

Vajravrh
Varahi
Marichi
Vajrayogini

41. An increase in the carbon dioxide


content in the atmosphere would cause
1. More heat to be retained in the
atmosphere
2. Less moisture to be retained in the
air
3. Erosion of the ozone in the
atmosphere
Choose the correct answer using the codes
below.
a)
b)
c)
d)

1 and 2 only
2 and 3 only
1 only
None of the above

Choose the correct answer using the codes


below.
a)
b)
c)
d)

1 and 2 only
2 and 3 only
1 and 3 only
None of the above

http://www.insightsonindia.com

INSIGHTS

Page 9

42. As per recent study, a bird species


Yellow-breasted buntings is near to
extinction. Consider the following
about it.
1. It is a songbird.
2. It is a migratory bird wintering in
India.
3. The main reason for the decline of
the species is habitat destruction.
Choose the correct answer using the codes
below.
a)
b)
c)
d)

1 and 2 only
2 and 3 only
1 and 2 only
All of the above

43. In the Fourth Anglo-Mysore War,


which combined forces defeated Tipu
Sultan?
a) The British East India Company
and the Nizam of Hyderabad
b) The British East India and the
French
c) The British East India and the
Marathas
d) The British East India and the
Dutch
44. The Central Vigilance Commissioner
and other Vigilance commissioners are
appointed by the President on the
recommendation of a select committee
including
1. Prime Minister
2. Union Home Minister
3. Leader of Opposition, Lok Sabha
4. Lok Sabha speaker
5. Chief Justice of India
Choose the correct answer using the codes
below.

b) 1, 3 and 4 only
c) 1, 2 and 3 only
d) All of the above
45. In the regional music Pandavani of
Chattisgarh
a) Tales from Mahabharata are sung
as a ballad
b) Farm songs are recited in unison by
peasants in harvest season
c) War gods are celebrated by
powerful group singing and
dancing in temples
d) A local poetic version of Bhagwat
Gita is recited on auspicious
religious occasions

46. To which of the following forest types


does Bamboos, sal, shisham,
sandalwood tree species
predominantly belong to?
a) Tropical evergreen
b) Tropical dry deciduous
c) Tropical moist deciduous
d) Tropical semi-evergreen

47. In the mid-oceanic ridges, sea floor


spreading leads to
1. Increases in age of rocks as one
moves away from the crest.
2. Thin layers of sediments on the
ocean floor
3. Occurrence of lava near the ridges
Choose the correct answer using the codes
below.
a)
b)
c)
d)

1 and 3 only
2 and 3 only
1 and 2 only
All of the above

a) 2, 4 and 5 only
http://www.insightsonindia.com

INSIGHTS

Page 10

48. Elite Panel of International Cricket


Council (ICC) Umpires is chosen by
a) The respective national cricket
boards
b) The respective international cricket
clubs to which the empires belong
c) The ICC Umpires Selection Panel
d) A committee consisting of the elite
panel of empires of ICC

51. Consider the following statements:


1. The name Pakistan or Pak-stan was
coined by Urdu poet Mohammad
Iqbal
2. The Urdu poet Mohammad Iqbal
apoke of the need for a NorthWest Indian Muslim state as an
autonomous unit within a single,
loose Indian federation
Which of the above statements is/are correct?

49. Hydrogen is an important nutrient for


plant growth and development. Plants
obtain hydrogen from
a) Air
b) Water
c) Soil
d) Manures

50. With reference to a particular school of


painting that originated in India,
consider the following statements:
1. The paintings, usually 17 by 11
inches (43 by 28 centimetres), were
done on blank sheets, with no
attempt made to fill in the
backgrounds
2. It was a watercolour painting
produced in the 19th century in
India by artists in the Calcutta
marketplace for sale to pilgrims
3. Most usually depicted were the
popular Hindu deities, but scenes
of contemporary life are also found
Identify the school of painting from below
options:
a)
b)
c)
d)

Pahari School of Painting


Bengal School of Painting
Kalighat School of Painting
None of the above

a)
b)
c)
d)

1 Only
2 Only
Both
None

52. The Khoya Paya web platform lauched


by Union government is for people to
a) Report, track and upload
information on missing children
b) Report, track and upload
information on
missing/stolen/mishandled goods
c) Report and track missing women in
local regions
d) Report and track women who are
victims of trafficking and
prostitution

53. Recent lowering of Repo rate by the


RBI may
1. Immediately reduce money supply
in the economy
2. Make lending difficult for banks
3. Lead to immediate reduction in
deposit interest rates for people
Choose the correct answer using the codes
below.

http://www.insightsonindia.com

INSIGHTS

Page 11

a)
b)
c)
d)

1 and 2 only
2 and 3 only
3 only
None of the above

54. Environmental impact


assessment (EIA) is NOT concerned
with
1. Mitigating the developmental
effects of a project
2. Public participation in evaluating
projects
3. Social impact of a project
Choose the correct answer using the codes
below.
a)
b)
c)
d)

1 and 2 only
2 and 3 only
3 only
1 and 3 only

55. Consider the following statements:


1. Defence was one of the many
reasons why the British took upon
themselves the task of town
planning from the early years of
their rule in Bengal
2. The Fort William was Calcuttas
first significant town planning
measure
Which of the above statements is/are correct?
a)
b)
c)
d)

1 Only
2 Only
Both
None

http://www.insightsonindia.com

56. Which of the following differentiate an


ordinary committee from a
Parliamentary committee?
1. It is appointed or elected by the
House or nominated by the
Speaker/Chairman
2. It works under the direction of the
Speaker/Chairman.
3. It presents its report directly to the
President of India.
4. It is assisted by the Central
Secretariat.
Choose the correct answer using the codes
below.
a)
b)
c)
d)

1 and 2 only
2 and 3 only
1 and 4 only
All of the above

57. Consider the following about Green


Climate Fund (GCF).
1. It was established under United
Nations Environment Programme
(UNEP).
2. It is aimed at achieving the goal set
out by United Nations Framework
Convention on Climate Change
(UNFCCC).
3. It is a mechanism to redistribute
money from the developed to the
developing world.
Choose the correct answer using the codes
below.
a)
b)
c)
d)

INSIGHTS

1 and 2 only
2 and 3 only
1 and 3 only
All of the above

Page 12

58. Which one of the following lakes is a


salt water lake?
a) Sambhar
b) Wular
c) Dal
d) Loktak

59. As per the Constitution, which of the


following is/are NOT a part of the
Union Executive?
1. Vice-President
2. Attorney General of India
3. Non-cabinet Ministers
4. Cabinet Secretary
Choose the correct answer using the codes
below.
a)
b)
c)
d)

2, 3 and 4 only
1 and 4 only
2 and 3 only
4 only

60. Which of the following is/are the


eligibility criteria for the office of the
Prime Minister?
1. He should not have been a former
President of India.
2. He should have been directly
elected by the people of India.
3. He should be the leader of a
political party.
Choose the correct answer using the codes
below.
a)
b)
c)
d)

1 and 2 only
2 and 3 only
3 only
None of the above

61. Consider the following statements.


1. Fiscal deficit always leads to
inflation in the economy.
2. Supply side bottlenecks in the
economy are caused by low fiscal
deficit.
Which of the above is/are true?
a)
b)
c)
d)

1 only
2 only
Both 1 and 2
None

62. Gene therapy is a collection of methods


that allows correction of a gene defect.
It can NOT be administered in
1. Children
2. Old age people
3. Human embryos
Choose the correct answer using the codes
below.
a)
b)
c)
d)

1 and 3 only
2 and 3 only
1 and 2 only
It can be administered in all.

63. Consider the following about the


Middle East Respiratory Syndrome
(MERS) recently in news.
1. The virus causing it comes from
bats.
2. People can be infected after close
contact with victims via a
respiratory route.
3. The disease vector has not been
firmly established.
4. Till date there is no vaccine
available to prevent it.
Choose the correct answer using the codes
below.

http://www.insightsonindia.com

INSIGHTS

Page 13

a)
b)
c)
d)

1 and 4 only
2 and 3 only
1, 2 and 4 only
All of the above

64. Which one of the following was/were


prominent among the Kannada
Vachana Sahitya movement?
1. Basavanna
2. Akka Mahadevi
3. Allama Prabhu
4. Kannappa Nayanar
Choose the correct answer using the codes
below:
a)
b)
c)
d)

1,2 and 4 Only


1 and 3 Only
1,2 and 3 Only
All

65. Consider the following about the


effects of Fluoride in water.
1. Its deficiency in drinking water can
cause tooth decay.
2. Excess Fluoride concentration in
water causes brown mottling of
teeth and damage to bones.
3. Fluoride concentration in water is
also related to blue-baby
syndrome.

the energy of sunlight that falls on


their leaves and convert it into food
energy. When the plants are consumed
at the higher trophic levels, plants
stored energy is lost in
1. Heat to environment
2. Digestion of the food
3. Growth and development of the
organism
Choose the correct answer using the codes
below.
a)
b)
c)
d)

1 and 3 only
2 and 3 only
1 and 2 only
All of the above

67. The government can stabilize or boost


the economy by which of the following
measures?
1. Tweaking monetary policy
2. Providing fiscal stimulus
3. Welfare transfers
Choose the correct answer using the codes
below.
a)
b)
c)
d)

1 and 3 only
2 and 3 only
2 only
All of the above

Choose the correct answer using the codes


below.
a)
b)
c)
d)

1 and 3 only
2 and 3 only
1 and 2 only
All of the above

66. The green plants in a terrestrial


ecosystem capture a little fraction of
http://www.insightsonindia.com

68. The legitimacy of Special Drawing


Rights (SDR) being used for
international transactions comes from
1. IMF authorising its use
2. Nations willing to use it
3. Nearly constant value of the SDR
Choose the correct answer using the codes
below.
INSIGHTS

Page 14

a)
b)
c)
d)

1 and 3 only
2 and 3 only
1 and 2 only
All of the above

69. Consider the following.


1. Growth in the fraction of those who
can work and are looking for it
2. Growth in the fraction of those who
find work
3. Growth in the fraction of those
employed in organized sector
Which of the above correspond correctly with
Labour Force Participation rate and
Employment Rate?
a)
b)
c)
d)

1 and 2 respectively
2 and 1 respectively
1 and 3 respectively
2 and 3 respectively

70. The system of progressive taxation in


the country is in accordance with the
a) Directive principles of State Policy
b) Fundamental Rights
c) Preamble
d) All of the above

71. Consider the following statements:


1. Jinnah called for a Direct Action
Day to press the Leagues demand
for Pakistan after the failure of
Cabinet Mission talks
2. The Cabinet Mission plan
envisaged creation of Muslimmajority provinces grouped as Sind, Punjab and North-West
Frontier Province into one group,
and Bengal and Assam into another

http://www.insightsonindia.com

Which of the above statements is/are


INCORRECT?
a)
b)
c)
d)

1 Only
2 Only
Both
None

72. Crop rotation can be done by


1. Growing different crops in different
seasons
2. Growing different crops in different
fields
3. Growing alternate crops in
alternate rows in the same field
Choose the correct answer using the codes
below.
a)
b)
c)
d)

1 and 2 only
2 and 3 only
1 and 3 only
1 only

73. To protect grains from degradation


and pests like rats and insects, grains
may be stored in
1. Jute bags
2. Metallic bins
3. Clay pots
4. Grass baskets
Choose the correct answer using the codes
below.
a)
b)
c)
d)

INSIGHTS

1 and 2 only
2 and 4 only
1, 3 and 4 only
All of the above

Page 15

74. The Indian model of government is


also called as the Westminster model
of government because
a) Indian constitution was made with
the assistance of the British
b) Indian constitution was modelled
very closely on the lines of the
British constitution
c) India follows parliamentary form of
government
d) The popular parliamentary house is
elected

75. Which of the following provisions


ensure the independence and
impartiality of the office of the Lok
Sabha Speaker?
1. He can be removed only by a
resolution passed by the Lok Sabha
by a special majority.
2. His work and conduct cannot be
discussed and criticised in the Lok
Sabha except on a substantive
motion.
3. His salary and allowances are not
subject to the annual vote of
Parliament.
4. He cannot vote in the first instance.
Choose the correct answer using the codes
below.
a)
b)
c)
d)

1 and 2 only
2, 3 and 4 only
None of the above
All of the above

76. The High court has original


jurisdiction with respect to which of
these matters?
1. Dispute between two adjoining
states
http://www.insightsonindia.com

2. Selective cases involving


Constitutional Interpretation
3. Disputes relating to the election of
MPs and MLAs
4. Enforcement of fundamental rights
of citizens
Choose the correct answer using the codes
below.
a)
b)
c)
d)

1 and 2 only
2, 3 and 4 only
3 and 4 only
All of the above

77. In 1946, Mahatma Gandhi visited


Noakhali and other riot-torn areas to
stop communal violence. Today
Noakhali is located in which of the
following states?
a) Bihar
b) West Bengal
c) Assam
d) None of the above

78. The Nubra, the Shyok and the Hunza


are the tributaries of which of the
following rivers?
a) Brahmaputra
b) Indus
c) Jhelum
d) Chenab

79. The Finance Commission is required to


make recommendations to the
president of India in which of the
following matters?
1. Augmenting the resources of the
PSUs
2. Augmenting the resources of the
states to support local bodies
INSIGHTS

Page 16

3. Principles that should govern the


grants-in-aid to the states by the
Centre
Choose the correct answer using the codes
below.
a)
b)
c)
d)

1 and 2 only
2 and 3 only
1 and 3 only
All of the above

80. Consider the following about Manipuri


Dances.
1. It is a classical dance of India.
2. In this, there are references to the
dances of gods and goddesses who
created the universe.
3. It was introduced by a Vaishnava
saint and reformer of Manipur.
Choose the correct answer using the codes
below.
a)
b)
c)
d)

1 and 3 only
2 and 3 only
1 and 2 only
All of the above

81. Which of the following


travellers/scholars was/were
associated with the Vijayanagar
Empire and Hampi?
1. Colin Mackenzie
2. John Marshall
3. Alexander Greenlaw
4. J.F. Fleet
Choose the correct answer using the codes
below:

d) All

82. Consider the following about


adjournment and prorogation of the
house.
1. Unlike Adjournment, Prorogation
of the house not only terminates
not only the sitting but also the
session of the House.
2. Adjournment and prorogation both
are done by the Speaker.
3. Both do not affect the bills or any
other business pending before the
House.
Choose the correct answer using the codes
below.
a)
b)
c)
d)

1 and 2 only
2 and 3 only
1 and 3 only
All of the above

83. Consider the following about the


Union Public Service Commission.
1. It is an advisory body to the
Government of India.
2. It presents its annual report
directly to the Parliament.
3. An individual ministry or
department has no power to reject
the advice of the UPSC.
Choose the correct answer using the codes
below.
a)
b)
c)
d)

1 and 2 only
2 and 3 only
1 and 3 only
3 only

a) 1 and 2 Only
b) 1,2 and 3 Only
c) 1,3 and 4 Only
http://www.insightsonindia.com

INSIGHTS

Page 17

84. Which of the following factors affect


the amount of insolation received by
the Earth at different latitudes?
1. Tilt of Earths axis
2. Rotation of earth
3. Atmospheric circulation
4. Cloudiness
Choose the correct answer using the codes
below.
a)
b)
c)
d)

1 and 4 only
2 and 3 only
1, 3 and 4 only
All of the above

85. Consider the following statements.


1. In general, as we proceed from the
equator towards the poles, rainfall
goes on decreasing steadily.
2. The coastal areas of the world
receive greater amounts of rainfall
than the interior of the continents.
3. The rainfall is more over the oceans
than on the landmasses of the
world.
Choose the correct answer using the codes
below.
a)
b)
c)
d)

1 and 3 only
2 and 3 only
1 and 2 only
All of the above

86. Consider the following statements.


1. A species with a high genetic
diversity has greater chance to
survive climate change.
2. An ecosystem with high
biodiversity has a greater chance of
adapting to environmental change.

a)
b)
c)
d)

1 only
2 only
Both 1 and 2
None

87. The process in Eutrophication that is


central to reducing biodiversity in a
lake is?
a) Decomposition of organic matter
into inorganic matter by algae
b) Breakdown of organic matter that
consumes oxygen from water
c) Reduction of organic matter that is
the feedstock of aquatic animals
d) Abundance of flora in the lake
leading to habitat fragmentation
and reduced food and space

88. Over half of the e-wastes generated in


the developed world are exported to
developing countries where some of
the important metals can be obtained
in the recycling process. These are
1. Copper
2. Iron
3. Silicon
4. Nickel
5. Gold
Choose the correct answer using the codes
below.
a.
b.
c.
d.

All except 3 and 4


All except 1 and 2
All except 5
All of the above

Which of the above is/are true?


http://www.insightsonindia.com

INSIGHTS

Page 18

89. Snow-blindness can be caused by


exposure to
1. Ultra-violet rays
2. High intensity gamma rays
3. High frequency X-rays
Choose the correct answer using the codes
below.
a)
b)
c)
d)

1 and 3 only
2 and 3 only
1 only
3 only

90. Consider the following about


Petroleum.
1. It is formed in absence of oxygen.
2. While extracting from rocks, it is
found above water.
3. Petroleum deposits usually contain
natural gas deposits too.
Choose the correct answer using the codes
below.
a)
b)
c)
d)

1 and 2 only
2 and 3 only
1 and 3 only
All of the above

91. Consider the following about reserved


and protected forests.
1. Human activities like hunting and
grazing are permitted here.
2. Reserved forests have more
stringent environmental
regulations that protected forests.
3. These areas are declared by the
state governments.

b) 2 and 3 only
c) 1 and 3 only
d) All of the above

92. Which of the following rights have


been granted to the Attorney General
of India?
1. Right to take part in the
proceedings in any of the houses of
Parliament
2. Right to be a member of a
Parliamentary Committee
3. Right to all the privileges and
immunities granted to the MPs
Choose the correct answer using the codes
below.
a)
b)
c)
d)

1 and 2 only
2 and 3 only
1 and 3 only
All of the above

93. Waterlogging is a major problem in


agriculture because
1. It draws salt to the surface of the
soil.
2. It leads to salt being collected
around the roots of the plants.
3. It reduces the biological activity of
microorganisms in the soil.
Choose the correct answer using the codes
below.
a)
b)
c)
d)

1 and 3 only
2 and 3 only
1 and 2 only
All of the above

Choose the correct answer using the codes


below.
a) 1 and 2 only
http://www.insightsonindia.com

INSIGHTS

Page 19

94. Which of the following would


constitute a case of bio-piracy?
1. MNCs using the bio-resources of a
nation without proper
authorization
2. Individuals and companies
patenting indigenous traditional
community knowledge
3. Poaching and hunting of animals to
extract economically expensive
parts

3. Bi-annual Parliamentary financial


review
4. Motions and resolutions in the Lok
Sabha
Choose the correct answer using the codes
below.
a)
b)
c)
d)

1, 2 and 4 only
2 and 4 only
1 and 3 only
All of the above

Choose the correct answer using the codes


below.
a)
b)
c)
d)

1 and 3 only
2 and 3 only
1 and 2 only
All of the above

95. Tadoba National Park which is known


as Maharashtra's oldest and largest
National Park is famous for
a) Tiger Reserve
b) Elephant Reserve
c) Chinkara Reserve
d) Crocodile Reserve

96. The Caral civilization was recently in


news due to discovery of ancient
statues. The civilization was present in
a) Columbia
b) Mexico
c) Peru
d) Argentina

97. Fiscal accountability of the executive to


the legislature is ensured by which of
these?
1. FRBM Act, 2003
2. Annual Budgets
http://www.insightsonindia.com

98. The government will have to


compulsorily borrow not only to
finance its plan expenditure but also
its non-plan expenditure
(consumption requirements) in cases
of
1. Fiscal deficit
2. Revenue deficit
3. Primary deficit
Choose the correct answer using the codes
below.
a)
b)
c)
d)

1 and 2 only
2 and 3 only
2 only
3 only

99. Consider the following about carrier


INS Vikrant.
1. It has been made in India after
importing Russian technology.
2. It is Indias largest aircraft carrier.
3. It will be positioned in the South
China Sea region.
4. It is capable of being used as a
stealth submarine also.
Choose the correct answer using the codes
below.
INSIGHTS

Page 20

a)
b)
c)
d)

2 only
1 and 3 only
1 and 4 only
2 and 3 only

100.
Adipurana, a 10th-century
Kannada text written in Champu style,

http://www.insightsonindia.com

a mix of prose and verse, dealing with


the ten lives of the first tirthankara,
Adinatha, was written by
a) Ranna
b) Janna
c) Pampa
d) Ponna

INSIGHTS

Page 21

Insights Mock tests 2015: Test 27 Solutions


1. Solution: c)
http://www.fssai.gov.in/AboutFSSAI/introduction.aspx
http://www.ndtv.com/india-news/nestle-approaches-bombay-high-court-againstfssai-maharashtra-fda-order-770688
It has been established under Food Safety and Standards Act, 2006 which
consolidates various acts & orders that have hitherto handled food related issues in
various Ministries and Departments. FSSAI has been created for laying down science
based standards for articles of food and to regulate their manufacture, storage,
distribution, sale and import to ensure availability of safe and wholesome food for
human consumption.
Various central Acts like Prevention of Food Adulteration Act, 1954 , Fruit Products
Order , 1955, Meat Food Products Order , 1973, Vegetable Oil Products (Control)
Order, 1947,Edible Oils Packaging (Regulation)Order 1988, Solvent Extracted Oil,
De- Oiled Meal and Edible Flour (Control) Order, 1967, Milk and Milk Products
Order, 1992 etc were repealed after commencement of FSS Act, 2006.

2. Solution: a)
Anti-dumping duty is imposed by government on imported products which have
prices less than their normal values or domestic price.
Usually countries initiate anti-dumping probes to check if domestic industry has
been hurt because of a surge in below-cost imports.
Anti-Dumping Duty is imposed under the multilateral WTO regime and varies from
product to product and from country to country.
In India, anti-dumping duty is recommended by the Union Ministry of Commerce,
while the Union Finance Ministry imposes it.
3. Solution: c)
The Constitution (119th Amendment) Bill has been passed by the Parliament of India
on 7th May 2015.While India will gain 510 acres of land, ten thousand acres of land
will notionally go to Bangladesh. However, these are remote enclaves which India
cannot access. This legislation will redraw Indias boundary with Bangladesh by
exchanging enclaves in Assam, West Bengal, Tripura and Meghalaya.
Implications of the agreement are:
http://www.insightsonindia.com

INSIGHTS

Page 1

Insights Mock tests 2015: Test 27 Solutions

It will secure the long stranded boundary and enable to curb the illegal
migration, smuggling and criminal acts cross the border.
It would help those stateless citizens by granting the citizenship from their
respective countries.
It would help settle the boundary dispute at several points in Meghalaya,
Tripura, Assam, and west Bengal.
It would improve the access to the underdeveloped north-eastern state and
would further enhance the developmental works in the region.
It would help to increase the connectivity with the south-east Asia as part of
Indias North-eastern policy.

4. Solution: a)
Recently Union Home Ministry has appointed Senior IPS Officer RC Tayal as the
new Director General (DG) of Indias elite counter-terror force National Security
Guard (NSG).
It also nodal agency for counter-terror and counter-hijack operations and later was
tasked to protect high-risk VVIPs. It operates under the oversight of the Ministry of
Home Affairs and is headed by the Director General of the Indian Police Service
(IPS).
The NSG members are also known as Black Cats because of the black drill cotton
coveralls and balaclavas or helmets they wear.
5. Solution: a)
http://www.thehindu.com/sci-tech/health/us-doctors-perform-worlds-firstskullscalp-transplant/article7285933.ece
Organ transplantation is the moving of an organ from one body to another or from
a donor site to another location on the person's own body, to replace the recipient's
damaged or absent organ.
Organs that can be transplanted are
the heart, kidneys, liver, lungs, pancreas, intestine, and thymus. Tissues
include bones, tendons (both referred to as musculoskeletal
grafts),cornea, skin, heart valves, nerves and veins.
Organ donors may be living, brain dead, or dead via circulatory death.

6. Solution: b)
http://www.insightsonindia.com

INSIGHTS

Page 2

Insights Mock tests 2015: Test 27 Solutions


Under the nationwide campaign a village with acute water scarcity on a pilot project
will be selected as Jal Gram.
In this identified village a comprehensive integrated development plan will be
framed and several water conservation activities will be launched.
Activities proposed under the campaign include rain water harvesting, recycling of
waste water, micro irrigation for using water efficiently and mass awareness
program. Along with it, a cadre of local water professional Jal Mitra will be created
and they will be given training to create mass awareness.
As pilot project, activities under this nation campaign will be run in one water
stressed village in each 672 districts in the country.

7. Solution: b)
Al-Birunis Kitab-ul-Hind, written in Arabic, is simple and lucid. It is a voluminous
text, divided into 80 chapters on subjects such as religion and philosophy, festivals,
astronomy, alchemy, manners and customs, social life, weights and measures,
iconography, laws and metrology. Generally (though not always), Al-Biruni adopted
a distinctive structure in each chapter, beginning with a question, following this up
with a description based on Sanskritic traditions, and concluding with a comparison
with other cultures. Some present-day scholars have argued that this almost
geometric structure, remarkable for its precision and predictability, owed much to
his mathematical orientation. Al-Biruni, who wrote in Arabic, probably intended his
work for peoples living along the frontiers of the subcontinent. He was familiar with
translations and adaptations of Sanskrit, Pali and Prakrit texts into Arabic these
ranged from fables to works on astronomy and medicine. However, he was also
critical about the ways in which these texts were written, and clearly wanted to
improve on them.

8. Solution: c)
Recently, India has lost a poultry case against United States at World Trade
Organization (WTO) after WTOs Dispute Settlement Board (DSB) ruled that Indias
ban on import of poultry meat, eggs and live pigs from US is inconsistent with the
international norms.
India had banned imports of various agriculture products including poultry meat and
eggs from the US in 2007.

http://www.insightsonindia.com

INSIGHTS

Page 3

Insights Mock tests 2015: Test 27 Solutions


These poultry products were banned as a precautionary measure to prevent
outbreaks of Avian Influenza and bird flu fears. In March 2012, US had dragged
India to the WTO for banning these products and had contended Indias import
prohibition saying that it was not based on the relevant international standard or on
a scientific risk assessment.
Earlier in October 2014 WTO Appellate Body had ruled in favour of US and clearly
mentioned that Indias ban measures are arbitrarily and unjustifiably discrimination
on international trade and constitutes a disguised restriction on it.
Protectionism (saving domestic industries) is a clear violation of WTO rules.
9. Solution: b)
Refer to this article. It has been explained beautifully
http://www.investopedia.com/articles/investing/082113/understanding-interestrates-nominal-real-and-effective.asp
You should also know that the higher is the difference between the real and nominal
rates; the less profitable is the lending business to the bank.

10.

Solution: a)

Corporate farming is a term used to describe companies that own or influence farms
and agricultural practices on a large scale. This includes not only corporate
ownership of farms and selling of agricultural products, but also the roles of these
companies in influencing agricultural education, research, and public policy through
funding initiatives and lobbying efforts.
Contract farming- In contrast, Farming contracts are agreements between a farmer
and a buyer that stipulates what the farmer will grow and how much they will grow
usually in return for guaranteed purchase of the product or financial support in
purchase of inputs (e.g. feed for livestock growers).

11. Solution: c)
Franois Bernier (25 September 1620 22 September 1688) was a French physician
and traveller. He was born at Jou-Etiau in Anjou. He was briefly personal physician
to Prince Dara Shikoh, the elder son of Shah Jahan, and after Dara Shikoh's fall was
attached to the court of the Emperor Aurangzeb for around 12 years during his stay
in India.

http://www.insightsonindia.com

INSIGHTS

Page 4

Insights Mock tests 2015: Test 27 Solutions


His 1684 publication Nouvelle division de la terre par les diffrentes espces ou
races qui l'habitent is considered the first published post-Classical classification of
humans into distinct races. He also wrote Travels in the Mughal Empire, which is
mainly about the reigns of Dara Shikoh and Aurangzeb. It is based on his own
extensive journeys and observations, and on information from eminent Mughal
courtiers who had witnessed the events at first hand.

12. Solution: a)
African countries have signed a historic The Tripartite Free Trade Area (TFTA) that
facilitates creation of Africas largest free-trade zone.

http://www.insightsonindia.com

INSIGHTS

Page 5

Insights Mock tests 2015: Test 27 Solutions

13. Solution: d)
The following are denied to foreign citizens, also called aliens:

Right against discrimination on grounds of religion, race, caste, sex or place of


birth (Article 15).
Right to equality of opportunity in the matter of public employment (Article
16).
Right to freedom of speech and expression, assembly, association, movement,
residence and profession (Article 19).
Cultural and educational rights (Articles 29 and 30).
Right to vote in elections to the Lok Sabha and state legislative assembly.
Right to contest for the membership of the Parliament and the state
legislature.
Eligibility to hold certain public offices, that is, President of India, VicePresident of India, judges of the Supreme Court and the high courts, governor
of states, attorney general of India and advocate general of states.

14. Solution: d)
Climate refers to the sum total of weather conditions and variations over a large area
for a long period of time (more than thirty years). Weather refers to the state of the
atmosphere over an area at any point of time. The elements of weather and climate
are the same, i.e. temperature, atmospheric pressure, wind, humidity and
precipitation.

15. Solution: c)
The 1919 act divided the provincial subjects into two parts transferred and
reserved.
The transferred subjects were to be administered by the governor with the aid of
ministers responsible to the legislative Council.
The reserved subjects, on the other hand, were to be administered by the governor
and his executive council without being responsible to the legislative Council.

http://www.insightsonindia.com

INSIGHTS

Page 6

Insights Mock tests 2015: Test 27 Solutions


This dual scheme of governance was known as - dyarchy - a term derived from the
Greek word di-arche which means double rule. However, this experiment was largely
unsuccessful.

16. Solution: a)
When the Presidents rule is imposed in a state, the Parliament becomes empowered
to make laws with respect to any matter in the State List in relation to that state. A
law made so by the Parliament continues to be operative even after the presidents
rule.
All the decisions concerning the state emanate from the President based on the
advice given by Council of Ministers.
The state legislature is also either suspended or dissolved.

17. Solution: d)
In introducing the Permanent Settlement, British officials hoped to resolve the
problems they had been facing since the conquest of Bengal. By the 1770s, the rural
economy in Bengal was in crisis, with recurrent famines and declining agricultural
output. Officials felt that agriculture, trade and the revenue resources of the state
could all be developed by encouraging investment in agriculture. This could be done
by securing rights of property and permanently fixing the rates of revenue demand. If
the revenue demand of the state was permanently fixed, then the Company could
look forward to a regular flow of revenue, while entrepreneurs could feel sure of
earning a profit from their investment, since the state would not siphon it off by
increasing its claim. The process, officials hoped, would lead to the emergence of a
class of yeomen farmers and rich landowners who would have the capital and
enterprise to improve agriculture. Nurtured by the British, this class would also be
loyal to the Company.

18.Solution: d)
Union Government has launched IAP HealthPhone programme, worlds largest
digital mass education programme to address malnutrition in women and children.
This programme is initiative of Indian Academy of Pediatrics (IAP) in partnership
with Union Ministry of Women and Child Development (WCD); United Nations
http://www.insightsonindia.com

INSIGHTS

Page 7

Insights Mock tests 2015: Test 27 Solutions


Childrens Fund (UNICEF); Supported by Vodafone India. It is a public-private
partnership (PPP) initiative that seeks to educate over 6 million girls and women
between 13 and 35 years of age and their families on better health and nutrition
practices by 2018 by leveraging penetration of mobile phones in the country. It will
promote and distribute four re-edited videos from the Poshan (nutritional videos)
series in 18 Indian languages, which are jointly produced by Union Ministry WCD
and UNICEF.
The three-year intensive awareness campaign under IAP HealthPhone programme is
expected to benefit to improve health of around 60 million children born by 2025 in
India and also play important transformative role in Indias fight against
malnutrition.

19. Solution: b)
Mandamus literally means we command. It is a command issued by the court to a
public official asking him to perform his official duties that he has failed or refused to
perform. It can also be issued against any public body, a corporation, an inferior
court, a tribunal or government for the same purpose.
The writ of mandamus cannot be issued (a) against a private individual or body; (b)
to enforce departmental instruction that does not possess statutory force; (c) when
the duty is discretionary and not mandatory; (d) to enforce a contractual obligation;
(e) against the president of India or the state governors; and (f) against the chief
justice of a high court acting in judicial capacity.

20.

Solution: d)

GNP is basically GDP plus net factor income from abroad (NFIA).
NFIA includes imports, exports, remittances etc. So any change in them will
immediately change the GNP.
Moreover, lower interest rates in the economy encourage consumption and boost
capital investment leading to a higher GDP (hence GNP) in the short-run.

21. Solution: d)
Article 355: Duty of the Union to protect States against external aggression and
internal disturbance - It shall be the duty of the Union to protect every State against
http://www.insightsonindia.com

INSIGHTS

Page 8

Insights Mock tests 2015: Test 27 Solutions


external aggression and internal disturbance and to ensure that the government of
every State is carried on in accordance with the provisions of this Constitution.
Article 365: Article 365 says that where any state has failed to comply with (or to give
effect to) any directions given by the Centre, it will be lawful for the President to hold
that a situation has arisen in which the government of the state cannot be carried on
in accordance with the provisions of the Constitution. It means that, in such a
situation, the Presidents rule can be imposed in the state under Article 356.

22.

Solution: b)

British General Charles Cornwallis, the 2nd Earl Cornwallis, was appointed in
February 1786 to serve as both Commander-in-Chief of British India and Governor of
the Presidency of Fort William, also known as the Bengal Presidency. Based in
Calcutta, he oversaw the consolidation of British control over much of peninsular
India, setting the stage for the British Raj. He was also instrumental in enacting
administrative and legal reforms that fundamentally altered civil administration and
land management practices in India. According to historian Jerry Dupont,
Cornwallis was responsible for "laying the foundation for British rule throughout
India and setting standards for the services, courts, and revenue collection that
remained remarkably unaltered almost to the end of the British era."

23.

Solution: b)

Scientific forestry is the science of managing forests. It was a system introduced in


the colonial times.
Plantation crops were introduced by replacing the local forests with one type of trees
introduced in a row.
Moreover, the forests were surveyed by officials and various plans were made for
better forest management.
Essentially, the main goal is to create and implement systems that allow forests to
continue a sustainable continuation of environmental supplies and services. The
challenge is to create systems that are socially accepted while sustaining the resource
and any other resources that might be affected.

24.

Solution: a)

The Battle of Karnal (February 24, 1739 was a decisive victory for Nader Shah, the
emperor of the Iranian Afsharid dynasty during his invasion of India. The Shah's
http://www.insightsonindia.com

INSIGHTS

Page 9

Insights Mock tests 2015: Test 27 Solutions


forces defeated the army of Muhammad Shah which was roughly six times the size of
his own in what is regarded as the crowning achievement in a spectacular military
career. the Indian army was lured into battle in segments and defeated in detail, in
little more than three hours, paving the way for the Afsharid sack of Delhi. The battle
took place near Karnal, 110 kilometres (68 mi) north of Delhi, India.
Aurangzeb had died in 1707 itself. Hence statement 1 is wrong.

25.

Solution: a)

Read the mechanism here http://www.downtoearth.org.in/content/cycloneashobaa-slows-monsoon-progress-india


The southwest monsoon winds have entered east and northeast India. Heavy rains
have already been reported in Arunachal Pradesh, Assam and Meghalaya.

26.

Solution: d)

Monetary Authority:

Formulates, implements and monitors the monetary policy.

Objective: maintaining price stability and ensuring adequate flow of credit to


productive sectors.

Regulator and supervisor of the financial system:

Prescribes broad parameters of banking operations within which the country's


banking and financial system functions.

Objective: maintain public confidence in the system, protect depositors'


interest and provide cost-effective banking services to the public.

Manager of Foreign Exchange

Manages the Foreign Exchange Management Act, 1999.

Objective: to facilitate external trade and payment and promote orderly


development and maintenance of foreign exchange market in India.

Issuer of currency:

Issues and exchanges or destroys currency and coins not fit for circulation.

Objective: to give the public adequate quantity of supplies of currency notes


and coins and in good quality.

http://www.insightsonindia.com

INSIGHTS

Page 10

Insights Mock tests 2015: Test 27 Solutions


Developmental role

Performs a wide range of promotional functions to support national


objectives.

Related Functions

Banker to the Government: performs merchant banking function for the


central and the state governments; also acts as their banker.

Banker to banks: maintains banking accounts of all scheduled banks.

27.

Solution: d)

Read the link for understanding the issue.


http://www.downtoearth.org.in/content/environment-ministry-may-allow-huntingnuisance-wild-animals
The Ministry of Environment, Forests and Climate Change (MoEFCC) is planning to
allow the legitimate hunting of wild animals such as blue bulls (nilgai) and wild boars
to tackle man-animal conflict.
According to environment minister Prakash Javadekar, the ministry is mulling the
implementation of an office memorandum passed in December last year, which
states that wild animals which destroy crops should be treated as vermins. However,
the ministrys move has drawn flak from animal rights groups.

28.

Solution: b)

Maanikavasagar was a 9th-century Tamil poet who wrote Tiruvasakam, a book of


Shaiva hymns. Manikkavasakar was one of the main authors of saivite tirumurai: his
work forms one volume of the Tirumurai, the key religious text of Tamil Shaiva
Siddhanta. A minister to the Pandya king Varagunavarman II (c. 862 C.E. 885
C.E.) (also called Arimarthana Pandiyan), he lived in Madurai. His work is a poetic
expression of the joy of God-experience, the anguish of being separated from God.
Although he is a prominent saint in Southern India, he is not counted among the
sixty-three nayanars.

29.

Solution: b)

The trade winds are the prevailing pattern of easterly surface winds found in the
tropics, within the lower portion of the Earth's atmosphere, in the lower section of
http://www.insightsonindia.com

INSIGHTS

Page 11

Insights Mock tests 2015: Test 27 Solutions


the troposphere near the Earth's equator. The trade winds blow predominantly from
the northeast in the Northern Hemisphere and from the southeast in the Southern
Hemisphere, strengthening during the winter and when the Arctic oscillation is in its
warm phase. Historically, the trade winds have been used by captains of sailing ships
to cross the world's oceans for centuries, and enabled European empire expansion
into the Americas and trade routes to become established across the Atlantic and
Pacific oceans.

30.

Solution: d)

Tax expenditures are revenue losses attributable to tax provisions that often result
from the use of the tax system to promote social goals without incurring direct
expenditures.
Income tax provisions generally seek to promote one or more of three broad
objectives: measuring income accurately, distributing fiscal benefits and burdens
based on a households ability to pay, and promoting activities or behavior that are
considered socially desirable. Tax expenditures are tax provisions that are not
structural features of the income tax or necessary to measure income accurately.

31. Solution: d)
Printing money is more or less always inflationary if not backed up immediately with
supply side augmentations.
Rupee depreciation feeds in the cost of our imports especially making oil costlier.
This impacts the vital transportation sector from where inflation is fed in other
sectors of the economy too quickly.
High tax rebates quickly increase the purchasing power of the citizens. This leads to
higher demand, and if not backed by greater supply, will lead to inflationary
pressures.

32.

Solution: a)

It has been explained here. Giving the account of the crisis here would be difficult.
http://www.educationworld.com/a_lesson/explaining-the-greek-economic-crisiswith-students.shtml
Also read the introduction of this article
https://en.wikipedia.org/wiki/Greek_government-debt_crisis
http://www.insightsonindia.com

INSIGHTS

Page 12

Insights Mock tests 2015: Test 27 Solutions


No need to read in-depth. Just get a general idea.

33.

Solution: c)

Subsidiary Alliance was a system devised by Lord Wellesley in 1798. All those who
entered into such an alliance with the British had to accept certain terms and
conditions: (a) The British would be responsible for protecting their ally from
external and internal threats to their power. (b) In the territory of the ally, a British
armed contingent would be stationed. (c) The ally would have to provide the
resources for maintaining this contingent. (d) The ally could enter into agreements
with other rulers or engage in warfare only with the permission of the British.

34.

Solution: b)

A non-bank financial institution (NBFI) is a financial institution that does not have a
full banking license or is not supervised by a national or international banking
regulatory agency. NBFIs facilitate bank-related financial services, such
as investment, risk pooling, contractual savings, and market brokering. Examples of
these include insurance firms, pawn shops, cashier's check issuers, check
cashing locations, payday lending, currency exchanges, and microloan organizations.
They provide "multiple alternatives to transform an economy's savings into capital
investment [which] act as backup facilities should the primary form of
intermediation fail.

35.

Solution: a)

http://www.downtoearth.org.in/content/rising-sea-levels-could-force-marine-lifetowards-poles
Rising sea temperatures caused by global climate change could drive many marine
creatures away from the Equator, but their move toward the poles promises to put
them in peril in habitats that are smaller and less hospitable.

36.

Solution: a)

The western cyclonic disturbances are weather phenomena of the winter months
brought in by the westerly flow from the Mediterranean region. They usually
influence the weather of the north and north-western regions of India.

http://www.insightsonindia.com

INSIGHTS

Page 13

Insights Mock tests 2015: Test 27 Solutions


https://en.wikipedia.org/wiki/Western_Disturbance
http://www.skymetweather.com/content/weather-faqs/what-is-westerndisturbance/

37.

Solution: a)

https://en.wikipedia.org/wiki/World_Network_of_Biosphere_Reserves_in_Asia_a
nd_the_Pacific#India
https://en.wikipedia.org/wiki/World_Network_of_Biosphere_Reserves

38.

Solution: d)

An 'Endemic Species' is one that is only found in that region and nowhere else in the
world. As such they are of conservation concern because they are not widespread and
may be confined to only one or two protected areas.
Sal and wild mango are two examples of endemic flora of the Pachmarhi Biosphere
Reserve. Bison, Indian giant squirrel and flying squirrel are endemic fauna of this
area.
Destruction of their habitat, increasing population and introduction of new species
may affect the natural habitat of endemic species and endanger their existence.

39.

Solution: d)

In some women oviducts are blocked. These women are unable to bear babies
because sperms cannot reach the egg for fertilization. In such cases, doctors collect
freshly released egg and sperms and keep them together for a few hours for IVF or in
vitro fertilization (fertilization outside the body).
In case fertilization occurs, the zygote is allowed to develop for about a week and
then it is placed in the mothers uterus.
Complete development takes place in the uterus and the baby is born like any other
baby. Babies born through this technique are called test-tube babies. This term is
actually misleading because babies cannot grow in test tubes.

http://www.insightsonindia.com

INSIGHTS

Page 14

Insights Mock tests 2015: Test 27 Solutions


40.

Solution: c)

In Mahayana and Vajrayana Buddhism, Marici is a deva or bodhisattva associated


with light and the sun. She is known as Molizhitian or Molizhitian Pusa in China and
Marishi-ten in Japan and in Tibetan as 'Odzer Canma, "Woman Endowed with Rays
of Light" (Wylie: 'od zer can ma). She is one of the 20 (or 24). In Taoism and Chinese
folk religion, Marici is known under the epithet of the Lady of Dipper Mother; or
Doumu, a name sometimes used by Buddhists.

41. Solution: c)
Some gases prevent the escape of heat from the Earth. An increase in the percentage
of such gases in the atmosphere would cause the average temperatures to increase
worldwide and this is called the greenhouse effect.
Carbon dioxide is one of the greenhouse gases. An increase in the carbon dioxide
content in the atmosphere would cause more heat to be retained by the atmosphere
and lead to global warming.
Ozone concentration has no relation with the amount of carbon dioxide.

42.

Solution: a)

The study has mentioned that population this bird species which is also known as the
rice bird in China has plunged by 90 percent since 1980 and disappearing from
Eastern Europe, Japan and parts of Russia.

The main reason mentioned in the study for decline of songbirds is illegal hunting for
food and sold on the black market in China.
It should be noted that Yellow-breasted buntings is Eurasian passerine bird that
belongs to the bunting family (Emberizidae). It is migratory bird wintering in southeast Asia, India, and southern China.

http://www.insightsonindia.com

INSIGHTS

Page 15

Insights Mock tests 2015: Test 27 Solutions


43.

Solution: a)

In the Fourth Anglo-Mysore War, the combined forces of the British East India
Company and the Nizam of Hyderabad defeated Tipu, and he was killed on 4 May
1799 while defending his fort of Srirangapatna.

44.

Solution: c)

The CVC is headed by a Central Vigilance Commissioner and has two Vigilance
Commissioners.
President Pranab Mukherjee has appointed KV Chowdary new Central Vigilance
Commissioner.
Central Vigilance Commissioner and has two Vigilance Commissioners are appointed
by President on the recommendations of select committee comprising of Prime
Minister as Chairperson, Union Minister of Home Affairs and Leader of the second
largest party in the Lok Sabha or majority group leader in parliament.
Its function is to monitor all vigilance activity under the Union Government and
advise various authorities in Union Government organizations in planning,
executing, reviewing and reforming their vigilance work.

45.

Solution: a)

One or two episodes from Mahabharata are chosen for the nights performance. The
main singer continuously sits throughout the performance and with powerful singing
and symbolic gestures he assumes all the characters of the episode one after another.
Pankida is sung by the peasants of Rajasthan while doing work in the fields, the
peasants sing and speak while playing algoza and manjira. The literal meaning of
the word Pankhida is lover.

46.

Solution: c)

On the basis of the availability of water, these forests are further divided into moist
and dry deciduous. The former is found in areas receiving rainfall between 200 and
100 cm. These forests exist, therefore, mostly in the eastern part of the country
northeastern states, along the foothills of the Himalayas, Jharkhand, West Orissa
and Chhattisgarh, and on the eastern slopes of the Western Ghats. Teak is the most
dominant species of this forest. Bamboos, sal, shisham, sandalwood, khair, kusum,
arjun, mulberry are other commercially important species.
http://www.insightsonindia.com

INSIGHTS

Page 16

Insights Mock tests 2015: Test 27 Solutions

47.

Solution: d)

The ocean crust rocks are much younger than the continental rocks. The age of rocks
in the oceanic crust is nowhere more than 200 million years old. Some of the
continental rock formations are as old as 3,200 million old.
This is due to sea floor spreading. Lava from deep in the ocean comes out on the
surface creating new sediments on the ocean floor.
Constant eruptions at the crest of oceanic ridges cause the rupture of the oceanic
crust and the new lava wedges into it, pushing the oceanic crust on either side. The
ocean floor, thus spreads.
So, Rocks closer to the mid-oceanic ridges are normal polarity and are the youngest.
The age of the rocks increases as one moves away from the crest.

48.

Solution: c)

After 11 years, Sundaram Ravi became the first Indian after S. Venkataraghavan to be
inducted in the International Cricket Councils (ICC) Elite Panel of Umpires
following an annual review and selection process.
The Emirates Elite Panel of ICC Umpires was selected by the ICC Umpires Selection
Panel consisting of Chairman, Geoff Allardice, ICCs General Manager Cricket, ICC
Chief Match Referee Ranjan Madugalle, ex-England player and coach and umpire,
David Lloyd, and Srinivas Venkataraghavan, the former India captain and
international umpire.
http://www.icc-cricket.com/news/2015/media-releases/88126/icc-announcesemirates-elite-panel-of-icc-umpires-for-2015-2016
http://www.thehindu.com/sport/cricket/ravi-inducted-into-icc-elite-panel-ofumpires/article7282542.ece

49.

Solution: b)

There are sixteen nutrients which are essential for plants.


Air supplies carbon and oxygen, hydrogen comes from water, and soil supplies the
other thirteen nutrients to plants.
http://www.insightsonindia.com

INSIGHTS

Page 17

Insights Mock tests 2015: Test 27 Solutions


As far as other nutrients are concerned, here is a very important table.

50.

Solution: c)

Klgh painting, short-lived style of watercolour painting produced in the 19th


century in India by artists in the Calcutta marketplace for sale to pilgrims visiting the
Klgh temple. The style is characterized by broad sweeping brush lines, bold
colours, and simplification of forms suitable for their mass production. The
paintings, usually 17 by 11 inches (43 by 28 centimetres), were done on blank sheets,
with no attempt made to fill in the backgrounds.
Most usually depicted were the popular Hindu deities, but scenes of contemporary
life are also found. The school, which rose in response to the competition of cheap
coloured lithographs, soon lost the contest and disappeared rapidly. The charm and
vigour of Klgh painting had an influence on a number of modern Indian painters,
as can be seen in the work of Jamini Roy.

51. Solution: b)
The name Pakistan or Pak-stan is coined by a Punjabi Muslim student at Cambridge,
Choudhry Rehmat Ali.

52.

Solution: a)

http://khoyapaya.gov.in/mpp/home
Khoya Paya is joint initiative of the Union Ministry of Women and Child
Development (WCD) and the Department of Electronics and Information
Technology (DeitY). The first-of-its-kind web portal will serve as a platform for
people to not only report a missing child but also track the efforts towards his or her
recovery.

http://www.insightsonindia.com

INSIGHTS

Page 18

Insights Mock tests 2015: Test 27 Solutions


The website will also provide links to court orders wherein police have been
mandated to register an FIR in the matter of a child gone missing and treat it as
either abduction or trafficking unless it is proved otherwise in the investigation.

53.

Solution: d)

Reserve Bank of India (RBI) has reduced the policy repo rate under the liquidity
adjustment facility (LAF) by 25 basis points from 7.5 per cent to 7.25 per cent with
immediate effect.
The concept of repo rate has been covered in previous tests. You should understand
its consequences here http://blog.bankbazaar.com/rbi-repo-cut-how-it-affects-yourloans-and-deposits/

54.

Solution: c)

An environmental impact assessment (EIA) is an assessment of the possible impacts


that a proposed project may have on the environment, consisting of
the environmental, social and economic aspects. The purpose of the assessment is to
ensure that decision makers consider the environmental impacts when deciding
whether or not to proceed with a project.
The International Association for Impact Assessment (IAIA) defines an
environmental impact assessment as "the process of identifying, predicting,
evaluating and mitigating the biophysical, social, and other relevant effects of
development proposals prior to major decisions being taken and commitments
made."
EIAs are unique in that they do not require adherence to a predetermined
environmental outcome, but rather they require decision makers to account for
environmental values in their decisions and to justify those decisions in light of
detailed environmental studies and public comments on the potential environmental
impacts.
PIB Features

55.

Solution: c)

There were many reasons why the British took upon themselves the task of town
planning from the early years of their rule in Bengal. One immediate reason was
defence. In 1756, Sirajudaula, the Nawab of Bengal, attacked Calcutta and sacked the
http://www.insightsonindia.com

INSIGHTS

Page 19

Insights Mock tests 2015: Test 27 Solutions


small fort which the British traders had built as their depot for goods. The English
East India Company traders had been continuously questioning the sovereignty of
the Nawab. They were reluctant to pay customs duties, and refused to comply with
the terms on which they were expected to operate. So Sirajudaula wanted to assert
his authority.
Subsequently, in 1757, when Sirajudaula was defeated in the Battle of Plassey, the
East India Company decided to build a new fort, one that could not be easily
attacked. Calcutta had grown from three villages called Sutanati, Kolkata and
Govindapur. The Company cleared a site in the southernmost village of Govindapur
and the traders and weavers living there were asked to move out. Around the new
Fort William they left a vast open space which came to be locally known as the
Maidan or garer-math.
This was done so that there would be no obstructions to a straight line of fire from
the Fort against an advancing enemy army. Once the British became more confident
about their permanent presence in Calcutta, they started moving out of the Fort and
building residences along the periphery of the Maidan. That was how the English
settlement in Calcutta gradually started taking shape. The vast open space around
the Fort (which still exists) became a landmark, Calcuttas first significant town
planning measure.

56.

Solution: a)

These committees present their report to the House or to the Speaker / Chairman.
They have a secretariat provided by the Lok Sabha/Rajya Sabha.
The consultative committees, which also consist of members of Parliament, are not
parliamentary committees as they do not fulfill these conditions.
The Constitution of India makes a mention of these committees at different places,
but without making any specific provisions regarding their composition, tenure,
functions, etc. All these matters are dealt by the rules of two Houses.

57. Solution: b)
http://www.downtoearth.org.in/content/green-climate-fund-racing-get-its-showroad-cop-21
The Green Climate Fund (GCF) is a fund within the framework of
the UNFCCC founded as a mechanism to redistribute money from the developed to
the developing world, in order to assist the developing countries
in adaptation and mitigation practices to counter climate change. The GCF is based
http://www.insightsonindia.com

INSIGHTS

Page 20

Insights Mock tests 2015: Test 27 Solutions


in the new Songdo district of Incheon, South Korea. It is governed by a Board of 24
members and initially supported by an Interim Secretariat.

58.

Solution: a)

The Sambhar Salt Lake, India's largest inland salt lake, a bowl shape lake encircles
historical Sambhar Lake Town located 96 km south west of the city of Jaipur
(Northwest India) and 64 km north east of Ajmer along National Highway 8 in
Rajasthan.

59.

Solution: d)

Articles 52 to 78 in part V of the Constitution deal with the Union Executive.


The Union executive consists of the President, the Vice-President, the Prime
Minister, the council of ministers and the attorney general of India.
Even though other officials are a part of the executive branch of the government, the
constitution does not mention them explicitly.

60.

Solution: d)

Article 75 says only that the Prime Minister shall be appointed by the president.
However, this does not imply that the president is free to appoint any one as the
Prime Minister. In accordance with the conventions of the parliamentary system of
government, the President has to appoint the leader of the majority party in the Lok
Sabha as the Prime Minister. But, when no party has a clear majority in the Lok
Sabha, then the President may exercise his personal discretion in the selection and
appointment of the Prime Minister.
Moreover, the PM can be a member of any of the houses.

61. Solution: d)
Fiscal deficit need not always be inflationary.
If enhanced government spending is supplemented by supply increases, it can
actually lead to greater economic growth and low inflation.

http://www.insightsonindia.com

INSIGHTS

Page 21

Insights Mock tests 2015: Test 27 Solutions


Supply side bottlenecks are caused by low government spending or neglect; low gross
capital formation; and lack of private sector participation in creating infrastructure.

62.

Solution: d)

Gene therapy is an experimental technique that uses genes to treat or prevent


disease. In the future, this technique may allow doctors to treat a disorder by
inserting a gene into a patients cells instead of using drugs or surgery. For e.g. it can
be used to treat developmental defects in a child/embryo.
Correction of a genetic defect involves delivery of a normal gene into the individual
or embryo to take over the function of and compensate for the non-functional gene.
Although gene therapy is a promising treatment option for a number of diseases
(including inherited disorders, some types of cancer, and certain viral infections), the
technique remains risky and is still under study to make sure that it will be safe and
effective. Gene therapy is currently only being tested for the treatment of diseases
that have no other cures.

63.

Solution: d)

As per recently published study, camels aged less than four years might be a major
source of Middle East Respiratory Syndrome (MERS). The study was published in
the journal Emerging Infectious Diseases.
Refer to
https://en.wikipedia.org/wiki/Middle_East_respiratory_syndrome_coronavirus#E
volution
http://www.medicalnewstoday.com/articles/262538.php

64.

Solution: c)

https://en.wikipedia.org/wiki/Vachana_sahitya
https://en.wikipedia.org/wiki/Nayanars

65.

Solution: c)

For drinking purposes, water should be tested for fluoride ion concentration.

http://www.insightsonindia.com

INSIGHTS

Page 22

Insights Mock tests 2015: Test 27 Solutions


Its deficiency in drinking water is harmful to man and causes diseases such as tooth
decay etc. The F ions make the enamel on teeth much harder.
However, F ion concentration above 2 ppm causes brown mottling of teeth. At the
same time, excess fluoride (over 10 ppm) causes harmful effect to bones and teeth, as
reported from some parts of Rajasthan.
The maximum limit of nitrate in drinking water is 50 ppm. Excess nitrate (not
fluoride) in drinking water can cause disease such as methemoglobinemia (blue
baby syndrome).

66.

Solution: d)

The autotrophs capture the energy present in sunlight and convert it into chemical
energy. This energy supports all the activities of the living world.
From autotrophs, the energy goes to the heterotrophs and decomposers when one
form of energy is changed to another, some energy is lost to the environment in
forms which cannot be used again.
When green plants are eaten by primary consumers, a great deal of energy is lost as
heat to the environment, some amount goes into digestion and in doing work and the
rest goes towards growth and reproduction. An average of 10% of the food eaten is
turned into its own body and made available for the next level of consumers.

67.

Solution: b)

Monetary policy review falls under the RBI, which is not the government. It acts
independently of the government in deciding monetary policy.
Fiscal stimulus means greater flow of government spending in productive sectors. It
boosts growth.
Welfare transfers augment the earnings of poor households providing them with
economic security as well as enhanced purchasing power which leads to more
demand and higher economic growth.

68.

Solution: d)

The SDR is an international reserve asset, created by the IMF in 1969 to supplement
its member countries official reserves.

http://www.insightsonindia.com

INSIGHTS

Page 23

Insights Mock tests 2015: Test 27 Solutions


Its value is based on a basket of four key international currencies, and SDRs can be
exchanged for freely usable currencies.
The SDR is neither a currency, nor a claim on the IMF. Rather, it is a potential claim
on the freely usable currencies of IMF members. Holders of SDRs can obtain these
currencies in exchange for their SDRs in two ways: first, through the arrangement of
voluntary exchanges between members; and second, by the IMF designating
members with strong external positions to purchase SDRs from members with weak
external positions.
In addition to its role as a supplementary reserve asset, the SDR serves as the unit of
account of the IMF and some other international organizations.

69.

Solution: a)

The Labour force participation rate refers to the number of people who are either
employed or are actively looking for work. The number of people who are no longer
actively searching for work would not be included in the participation rate.
During an economic recession, many workers often get discouraged and stop looking
for employment, as a result, the participation rate decreases.

70.

Solution: a)

The DPSP clearly mentions in the articles 39 (a) and (b) that it is the states goal to
reduce income inequalities and ensure that surplus income is redistributed to avoid
concentration of money in the economy.
Preamble merely talks about economic and social equality in general terms.
Hence, DPSP will be a preferred option to Preamble.

71. Solution: d)
Early in 1946 fresh elections were held to the provincial legislatures. The Congress
swept the General category, but in the seats specifically reserved for Muslims the
League won an overwhelming majority. The political polarisation was complete. A
Cabinet Mission sent in the summer of 1946 failed to get the Congress and the
League to agree on a federal system that would keep India together while allowing
the provinces a degree of autonomy. After the talks broke down, Jinnah called for a
http://www.insightsonindia.com

INSIGHTS

Page 24

Insights Mock tests 2015: Test 27 Solutions


Direct Action Day to press the Leagues demand for Pakistan. On the designated
day, 16 August 1946, bloody riots broke out in Calcutta. The violence spread to rural
Bengal, then to Bihar, and then across the country to the United Provinces and the
Punjab. In some places, Muslims were the main sufferers, in other places, Hindus.

https://en.wikipedia.org/wiki/1946_Cabinet_Mission_to_India

72.

Solution: c)

One method of replenishing the soil with nutrients is through crop rotation. This can
be done by growing different crops alternately.
Earlier, farmers in northern India used to grow legumes as fodder in one season and
wheat in the next season. This helped in the replenishment of the soil with nitrogen.
Farmers are being encouraged to adopt this practice.
Moreover, in the alternate rows in the field, two different crops can be grown in
different seasons alternatively.

73.

Solution: d)

http://www.nda.agric.za/docs/grain/grain.htm
But, large scale storage of grains is done in silos and granaries to protect them
from pests like rats and insects.
Dried neem leaves are used for storing food grains at home. For storing large
quantities of grains in big godowns, specific chemical treatments are required to
protect them from pests and microorganisms.
http://www.insightsonindia.com

INSIGHTS

Page 25

Insights Mock tests 2015: Test 27 Solutions

74.

Solution: c)

Westminster is a place in London where the British Parliament is located. It is often


used as a symbol of the British Parliament.
The Parliament is the legislative organ of the Union government. It occupies a preeminent and central position in the Indian democratic political system due to
adoption of the parliamentary form of government, also known as Westminster
model of government.
Indian constitution departs on a number of principles from the British constitution
namely federalism v/s unitary government; monarchy v/s republic; judicial review
etc.

75. Solution: b)
He is provided with a security of tenure. He can be removed only by a resolution
passed by the Lok Sabha by an absolute majority (ie, a majority of the total members
of the House) and not by an ordinary majority (ie, a majority of the members present
and voting in the House). This motion of removal can be considered and discussed
only when it has the support of at least 50 members.
Moreover, his powers of regulating procedure or conducting business or maintaining
order in the House are not subject to the jurisdiction of any Court.
He cannot vote in the first instance. He can only exercise a casting vote in the event
of a tie. This makes the position of Speaker impartial.

76.

Solution: b)

Original jurisdiction means the power of a high court to hear disputes in the first
instance, not by way of appeal.
Some other matters that come under this are:

Matters of admirality, will, marriage, divorce, company laws and contempt of


court.
Cases ordered to be transferred from a subordinate court involving the
interpretation of the Constitution to its own file

Before 1973, the Calcutta, Bombay and Madras High Courts also had original
criminal jurisdiction. This was fully abolished by the Criminal Procedure Code, 1973.
http://www.insightsonindia.com

INSIGHTS

Page 26

Insights Mock tests 2015: Test 27 Solutions

77. Solution: d)
It is located in Bangladesh.
https://en.wikipedia.org/wiki/Noakhali_District
https://en.wikipedia.org/wiki/Noakhali_riots

78.

Solution: b)

The river Indus rises in Tibet, near Lake Mansarowar. Flowing west, it enters India in
the Ladakh district of Jammu and Kashmir. It forms a picturesque gorge in this part.
Several tributaries, the Zaskar, the Nubra, the Shyok and the Hunza, join it in the
Kashmir region. The Indus flows through Baltistan and Gilgit and emerges from the
mountains at Attock. The Satluj, the Beas, the Ravi, the Chenab and the Jhelum join
together to enter the Indus near Mithankot in Pakistan.

79.

Solution: b)

It makes recommendations on the following matters:

The distribution of the net proceeds of taxes to be shared between the Centre
and the states, and the allocation between the states of the respective shares of
such proceeds.
The principles that should govern the grants-in-aid to the states by the Centre
(i.e., out of the consolidated fund of India).
The measures needed to augment the consolidated fund of a state to
supplement the resources of the panchayats and the municipalities in the state
on the basis of the recommendations made by the state finance commission.
Any other matter referred to it by the president in the interests of sound
finance.

80.

Solution: c)

Manipuri is one of the main styles of Indian Art or Classical Dances.


The origin of Manipuri dance can be traced back to ancient times that go beyond
recorded history. The dance in Manipur is associated with rituals and traditional
festivals, there are legendary references to the dances of Shiva and Parvati and other
gods and goddesses who created the universe.
http://www.insightsonindia.com

INSIGHTS

Page 27

Insights Mock tests 2015: Test 27 Solutions


Lai Haraoba is one of the main festivals still performed in Manipur which has its
roots in the pre-Vaishnavite period.
The Sattriya dance form was introduced in the 15th century A.D by the great
Vaishnava saint and reformer of Assam, Mahapurusha Sankaradeva as a powerful
medium for propagation of the Vaishnava faith.

81.Solution: d)
Landmarks in the Discovery and Conservation of Vijayanagara
1800 Colin Mackenzie visits Vijayanagara
1856 Alexander Greenlaw takes the first detailed photographs of archaeological
remains at Hampi
1876 J.F. Fleet begins documenting the inscriptions on the temple walls at the site
1902 Conservation begins under John Marshall
1986 Hampi declared a World Heritage site by UNESCO
Source: NCERT Class XII, History

82.

Solution: c)

83.

Solution: d)

The Union Public Service Commission (UPSC) is the central recruiting agency in
India. It is an independent constitutional body. Even though its advice is not binding
http://www.insightsonindia.com

INSIGHTS

Page 28

Insights Mock tests 2015: Test 27 Solutions


on the government, it is NOT merely an advisory body (that would make its position
very weak, which is not the case).
The UPSC presents, annually, to the presi-dent a report on its performance. The
President places this report before both the Houses of Parliament, along with a
memorandum explaining the cases where the advice of the Commission was not
accepted and the reasons for such non-acceptance.
All such cases of non-acceptance must be approved by the Appointments Committee
of the Union cabinet. An individual ministry or department has no power to reject
the advice of the UPSC.

84.

Solution: a)

Rotation of earth causes Coriolis force which only leads to the redistribution of heat
(insolation) by way of trade winds, cyclones etc. It does not affect the amount of
insolation received by the Earth.
Similarly, atmospheric circulation only redistributes heat on earth.
On the other hand, if earth was more tilted, it would receive even lesser solar
insolation at the poles.
Clouds reflect solar insolation back in the space. So it reduces the amount received to
earth.

85.

Solution: d)

Reason for statement 1: Generally the amount of solar insolation reduces which leads
to lesser temperature and thus lesser evaporation -> lesser precipitation. This is a
general rule. There can be exceptions to it when the area is given rainfall by cyclonic
depressions etc.
Reason for statement 2: More moisture laden winds are found in the coastal areas
due to the proximity of the ocean, hence more rainfall.
Reason for statement 3: Same as above. Oceans are rich in water -> more
evaporation -> more precipitation.

86.

Solution: c)

Species capture and store energy, produce and decompose organic materials, help to
cycle water and nutrients throughout the ecosystem, fix atmospheric gases and help
http://www.insightsonindia.com

INSIGHTS

Page 29

Insights Mock tests 2015: Test 27 Solutions


regulate the climate. These functions are important for ecosystem function and
human survival.
The more diverse an ecosystem, better are the chances for the species to survive
through adversities and attacks, and consequently, is more productive. Hence, the
loss of species would decrease the ability of the system to maintain itself.
Hence, the more the variety of species in an ecosystem, the more stable the
ecosystem is likely to be.

87.

Solution: b)

When algae die, they decompose and the nutrients contained in that organic matter
are converted into inorganic form by microorganisms. This decomposition process
consumes oxygen, which reduces the concentration of dissolved oxygen. The
depleted oxygen levels in turn may lead to fish kills and a range of other effects
reducing bio-diversity. Nutrients may become concentrated in an anoxic zone and
may only be made available again during autumn turn-over or in conditions of
turbulent flow.
Enhanced growth of aquatic vegetation or phytoplankton and algal blooms disrupts
normal functioning of the ecosystem, causing a variety of problems such as a lack
of oxygen needed for fish and shellfish to survive. The water becomes cloudy,
typically coloured a shade of green, yellow, brown, or red. Eutrophication also
decreases the value of rivers, lakes and aesthetic enjoyment. Health problems can
occur where eutrophic conditions interfere with drinking water treatment.

88.

Solution: d)

Irreparable computers and other electronic goods are known as electronic wastes (ewastes). E-wastes are buried in landfills or incinerated. Over half of the e-wastes
generated in the developed world are exported to developing countries, mainly to
China, India and Pakistan, where metals like copper, iron, silicon, nickel and gold are
recovered during recycling process.
Unlike developed countries, which have specifically built facilities for recycling of ewastes, recycling in developing countries often involves manual participation thus
exposing workers to toxic substances present in e-wastes. Recycling is the only
solution for the treatment of e-waste, provided it is carried out in an environment
friendly manner.

http://www.insightsonindia.com

INSIGHTS

Page 30

Insights Mock tests 2015: Test 27 Solutions


89.

Solution: c)

UV radiation of wavelengths shorter than UV-Blue, are almost completely absorbed


by Earths atmosphere, given that the ozone layer is intact.
But, UV-B damages DNA and mutation may occur. It causes aging of skin, damage to
skin cells and various types of skin cancers.
In human eye, cornea absorbs UV-B radiation, and a high dose of UV-B causes
inflammation of cornea, called snow-blindness, cataract, etc. Such exposure may
permanently damage the cornea.

90.

Solution: d)

Petroleum was formed from organisms living in the sea. As these organisms died,
their bodies settled at the bottom of the sea and got covered with layers of sand and
clay. Over millions of years, absence of air, high temperature and high pressure
transformed the dead organisms into petroleum and natural gas.

91. Solution: d)
You should know three things by reserved and protected forests:

Declared by the state governments (unlike Wildlife Sanctuaries or National


Parks)

These are forested lands where human activity (like hunting, logging, grazing
etc) may be permitted on a sustainable basis

http://www.insightsonindia.com

INSIGHTS

Page 31

Insights Mock tests 2015: Test 27 Solutions

The key difference between Reserved Forests and Protected Forests is that in
Reserved Forests explicit permission is required for such activities whereas in
Protected Forests such activities are permitted unless explicitly prohibited

92.

Solution: d)

In the performance of his official duties, the Attorney General has the right of
audience in all courts in the territory of India.
Further, he has the right to speak and to take part in the proceedings of both the
Houses of Parliament or their joint sitting and any committee of the Parliament of
which he may be named a member, but without a right to vote.
He enjoys all the privileges and immunities that are available to a member of
Parliament.

93.

Solution: c)

Irrigation without proper drainage of water leads to waterlogging in the soil. Besides
affecting the crops, waterlogging draws salt to the surface of the soil.
The salt then is deposited as a thin crust on the land surface or starts collecting at the
roots of the plants.
This increased salt content is inimical to the growth of crops and is extremely
damaging to agriculture. Waterlogging and soil salinity are some of the problems
that have come in the wake of the Green Revolution.

94.

Solution: d)

Biopiracy is a situation where indigenous knowledge of nature, originating with


indigenous peoples, is used by others for profit, without permission from and with
little or no compensation or recognition to the indigenous people themselves.
Moreover, it is also used to refer to the use of bio-resources by multinational
companies and other organisations without proper authorisation from the countries
and people concerned without compensatory payment.
Traditional knowledge related to bio-resources can be exploited to develop modern
applications and can also be used to save time, effort and expenditure during their
commercialisation.

http://www.insightsonindia.com

INSIGHTS

Page 32

Insights Mock tests 2015: Test 27 Solutions


95.

Solution: a)

https://en.wikipedia.org/wiki/Tadoba_Andhari_Tiger_Project

96.

Solution: c)

Researchers in Peru have discovered trios of statuettes which belong to some 3,800
years ago. These statuettes are believed to be created by the ancient Caral civilization
some 3,800 years ago.
The Caral civilization had emerged some 5,000 years ago and had lived in Perus
Supe Valley. They have left behind impressive architecture including pyramids and
sunken amphitheaters.
Among discovered statuettes, two are of the figures consisting of a naked man and
woman painted in white, black and red. It is believed that they represent political
authorities. The third statue is of women with 28 fingers and red dots on her white
face. It is believed that she represent a priestess. Researchers also have unearthed
two mud figurines of womens faces wrapped in cloth and covered with yellow, blue
and orange feathers.

97.

Solution: a)

FRBM Act, 2003 strives for Inter-generational equity in government finances. So, it
restricts borrowing from the RBI by the Central government other than in some
specific cases.
Moreover, it puts a rational limit on the fiscal deficit as a percentage of GDP to tackle
political populism.
Annual budgets are scrutinized by various Parliamentary committees and then voted
by the Parliament. Hence, the legislature ensures the executive is spending right.

98.

Solution: c)

Gross Primary Deficit is Gross Fiscal Deficit less interest payments. Net Primary
Deficit is Net Fiscal Deficit minus net interest payments. Net interest payment is
interest paid minus interest receipt.
Even though government runs a primary deficit, it need not borrow to fund its
consumption requirements.
http://www.insightsonindia.com

INSIGHTS

Page 33

Insights Mock tests 2015: Test 27 Solutions


Revenue deficit (revenue receipts revenue expenditure) will mean that the
government is falling short even for regular budget expenditures.

99.

Solution: a)

It is Indias first indigenously-built largest aircraft carrier. On completion it will carry


up to 36 fixed-wing jets, including MiG-29 K and indigenous Light Combat Aircraft
(LCA) Tejas.
The basic design of the carrier is done by Indian Navys Directorate of Naval Design
and was developed into detailed design by the design team of CSL.
After induction into Indian Navy its primary role will only be to defend the naval
fleet and will be positioned in the Indian Ocean region where the worlds commercial
and economic interests conflate.

100.

Solution: c)

Adipurana is a 10th-century Kannada text written in Champu style, a mix of prose


and verse, dealing with the ten lives of the first tirthankara, Adinatha, also known as
Rishabhanatha . This work is known to be the first work of Kannada poet Adikavi
Pampa (941 CE). It is based on the story narrated by Jinasenacharya in his earlier
Sanskrit work Purvapurana. Vikramarjuna Vijaya and Adipurana are, in some
respects, comparable to Milton's Paradise Lost and Paradise Regained.

http://www.insightsonindia.com

INSIGHTS

Page 34

TEST 28 QUESTION PAPER (TEST SERIES 2015)


INSIGHTS ON INDIA MOCK PRELIMINARY EXAM - 2015


INSIGHTS ON INDIA MOCK TEST - 28
GENERAL STUDIES

PAPER-I

Time Allowed: 2 Hours

Maximum Marks: 200

INSTRUCTIONS
1. IMMEDITELY AFTER THE COMMENCEMENT OF THE EXAMINATION, YOU SHOULD
CHECK THAT THIS TEST BOOKLET DOES NOT HAVE ANY UNPRINTED OR TORN OR MISSING
PAGES OR ITEMS, ETC. IF SO, GET IT REPLACED BY A COMPLETE TEST BOOKLET.
2. You have to enter your Roll Number on the Test
Booklet in the Box provided alongside. DO NOT
Write anything else on the Test Booklet.
4. This Test Booklet contains 100 items (questions). Each item is printed only in English. Each item
comprises four responses (answers). You will select the response which you want to mark on the
Answer Sheet. In case you feel that there is more than one correct response, mark the response which
you consider the best. In any case, choose ONLY ONE response for each item.
5. You have to mark all your responses ONLY on the separate Answer Sheet provided. See directions in
the Answer Sheet.
6. All items carry equal marks.
7. Before you proceed to mark in the Answer Sheet the response to various items in the Test Booklet, you
have to fill in some particulars in the Answer Sheet as per instructions sent to you with your
Admission Certificate.
8. After you have completed filling in all your responses on the Answer Sheet and the examination has
concluded, you should hand over to the Invigilator only the Answer Sheet. You are permitted to take
away with you the Test Booklet.
9. Sheets for rough work are appended in the Test Booklet at the end.
10. Penalty for wrong answers :
THERE WILL BE PENALTY FOR WRONG ANSWERS MARKED BY A CANDIDATE IN THE
OBJECTIVE TYPE QUESTION PAPERS.
(i)

There are four alternatives for the answer to every question. For each question for which a
wrong answer has been given by the candidate, one-third of the marks assigned to that
question will be deducted as penalty.

(ii) If a candidate gives more than one answer, it will be treated as a wrong answer even if one of
the given answers happens to be correct and there will be same penalty as above to that
question.
(iii)

If a question is left blank, i.e., no answer is given by the candidate, there will be no penalty
for that question.
http://www.insightsonindia.com

INSIGHTS ON INDIA MOCK TEST SERIES FOR CIVIL SERVICES PRELIMINARY EXAM 2015

http://www.insightsonindia.com INSIGHTS

Page 1

TEST 28 QUESTION PAPER (TEST SERIES 2015)


1. Which of the following


benefits/applications may be derived from
the national project, India-based Neutrino
Observatory (INO)?
1. Remote monitoring of nuclear
reactors
2. Detecting mineral and oil deposits
deep in the earth
3. Finding out early geological defects in
earth
4. Understanding dark matter
Choose the correct answer using the codes
below.
a)
b)
c)
d)

1 and 2 only
3 and 4 only
2, 3 and 4 only
All of the above

2. Consider the following about Neutrinos.


1. They are one of the fundamental
particles.
2. They can be harmful when exposed to
bare human body.
3. They can be weaponised for mass
destruction (WMD).
4. They are the information bearers of
the universe.
Choose the correct answer using the codes
below.
a)
b)
c)
d)

1 and 2 only
3 and 4 only
1 and 4 only
2 and 3 only

3. C N R Rao, became the first Indian to


reach the h-index of 100. The h-index is a
measure of

a) level of Scientific productivity


b) level of International presence of a
scientist
c) number of awards won at International
level
d) number of patents in the name of the
scientist

4. Consider the following about GPS Aided


Geo Augmented Navigation (GAGAN)
system.
1. It is Indias first indigenous
navigational guide system.
2. It is being used only for military
purposes in India.
3. It is supported by Polar satellites.
Choose the correct answer using the codes
below.
a)
b)
c)
d)

1 and 2 only
2 and 3 only
1 and 3 only
1 only

5. The Union Government has recently


launched NOWCAST weather alert
system. It will serve
a) flood and drought sensitive rural areas
in India for mitigation and
rehabilitation purposes
b) farmers giving alerts on extreme
weather conditions
c) Indian shipping industry by providing
details on tides, wave height, tsunami
and cyclones
d) coastal areas in the high risk zones of
tsunamis, cyclones and hailstorms

http://www.insightsonindia.com INSIGHTS

Page 2

TEST 28 QUESTION PAPER (TEST SERIES 2015)


6. In view of the farmer suicides in the


country, the government has been running
some insurance schemes. Which of the
following crop insurance schemes are
running till date?
1. Modified National Insurance Scheme
(MNAIS)
2. Farm Income Insurance Scheme
(FIIS)
3. Weather based Crop Insurance
Scheme (WBCIS)
4. Comprehensive Crop Insurance
Scheme (CCIS)
Choose the correct answer using the codes
below.
a)
b)
c)
d)

1 and 4 only
2 and 3 only
1 and 4 only
1 and 3 only

7. By government support, farmers in India


can be insured against
1. Shortfall in crop yield
2. Crop loss due to adverse events
3. Crop loss due to pest attacks
4. Post-harvest crop losses within a
specified period
Choose the correct answer using the codes
below.
a)
b)
c)
d)

1 and 4 only
2 and 3 only
1 and 4 only
1, 2 and 4 only

8. A state political party can be derecognized by the Election Commission of


India when
1. It fails to submit its poll expenses

2. It does not reveal information under


the RTI Act
3. It fails to win any seat in state
elections.
Choose the correct answer using the codes
below.
a)
b)
c)
d)

1 and 2 only
2 and 3 only
1 only
1 and 3 only

9. Researchers have recently created the


worlds thinnest light bulb using graphene
as filament. Graphene is
a) a crystalline form of carbon allotrope
b) a condensed form of silicon that is
used in making thin sheets
c) an alloy of tungsten popularly used as
high melting points metals
d) a mixed form of glass and carbon that
has greater agility and durability

10. Charles Correa played a pivotal role in the


creation of architecture for postIndependence India. He was also
1. the first Chairman of the National
Commission on Urbanization
2. known for his use of traditional
methods and materials for
construction
3. Indias first architect to have designed
structures for other sovereign nations.
Choose the correct answer using the codes
below.
a)
b)
c)
d)

1 and 2 only
2 and 3 only
1 and 3 only
All of the above

http://www.insightsonindia.com INSIGHTS

Page 3

TEST 28 QUESTION PAPER (TEST SERIES 2015)


11. The Battle of Waterloo was a pivotal


moment in modern History of Europe. In
this battle
a) Napolean was defeated by Angloallied and Prussian army
b) Prussia was defeated in Belgium by an
army led by Napolean Bonaparte
c) British army took control of several
major provinces of Belgium and ruled
thereafter alongwith Prussia
d) France became divided in several
smaller provinces which then came to
be ruled by imperial powers from
Netherlands and Prussia

12. Haryana Government has recently


launched a Village Mentoring Project
(VMP) in a bid to improve healthcare and
education of the villagers with a special
emphasis on girl child. Under VMP,
villages will be mentored by
1. MLAs
2. Corporate houses
3. Educational institutions
4. Eminent individuals

14. Jharkhand government has recently


banned the sale of thermocol plates in the
state because
a) food consumption in thermocol plates
was found hazardous for human
health
b) thermocol plate producing industries
came under the Red zone industries
c) it will promote leaf plates made by
tribals in the state so as to uplift them
economically
d) All of the above

15. The famous historical Magna Carta had


laid the first steps towards
1. Parliamentary Democracy
2. Rule of Law
3. Liberty and rights
Choose the correct answer using the codes
below.
a)
b)
c)
d)

1 and 2 only
2 and 3 only
1 and 3 only
All of the above

Choose the correct answer using the codes


below.
a)
b)
c)
d)

1 and 2 only
1 and 3 only
2 and 4 only
2 and 3 only

13. Lake Victoria pipeline project was recently


in news because of an agreement between
a) India and Kenya
b) India and Tanzania
c) USA and Russia
d) USA and India

16. Liquidity coverage ratio (LCR) is one of


the criteria in the Basel III regulations. If
it is high for a bank it means that
a) the bank is lending less than its
potential assets
b) the bank has a large amount of liquid
assets
c) the bank has kept a large amount of
deposit with RBI
d) the bank is unable of pay its debts

http://www.insightsonindia.com INSIGHTS

Page 4

TEST 28 QUESTION PAPER (TEST SERIES 2015)


17. The Indian Nuclear Insurance Pool


(INIP) as per the mandatory provision
under the Civil Liability for Nuclear
Damage Act (CLND), 2010 setup by the
Union government will insure
a) only operators of nuclear reactors
b) both operators and suppliers related
to nuclear projects
c) people living nearby the nuclear
reactor in case of a nuclear accident
d) operators, suppliers and people who
are affected in case of a nuclear
accident

18. Bibek Debroy Committee has recently


submitted its report on the restructuring
of Indian Railways to the Union
Government. Which of the following
is/are major recommendations suggested
by the committee?
1. Establishing an independent rail
regulator
2. Privatization of Indian railways
3. Scrapping of the Railways Board
4. State governments to have financial
equity in Railways
Choose the correct answer using the codes
below.
a)
b)
c)
d)

1, 2 and 4 only
2 and 3 only
1 only
All of the above

19. One Rank One Pension (OROP) is one of


the widely debated issues in the country
presently. It essentially means that
a) all ex-servicemen with similar lengths
of service will get the same pension
and similar ranks irrespective of their
retirement rank
b) all ex-servicemen who held the same
rank and similar lengths of service will

get the same pension irrespective of


their retirement year
c) all ex-servicemen belonging to a
particular rank will have their
pensions revised upwards irrespective
of their retirement year
d) all ex-servicemen who held the same
rank will get the same pension
irrespective of their retirement year
and length of service

20. Which of the following is/are NOT forms


of Yoga?
1. Laya Yoga
2. Raja Yoga
3. Jnana Yoga
4. Mantra Yoga
Choose the correct answer using the codes
below.
a)
b)
c)
d)

All are forms of Yoga.


2 and 3 only
1 and 3 only
1 and 4 only

21. Union Government has launched an


Augmented Reality technology based
mobile application. Consider the following
about augmented reality technology.
1. Artificial information about the
environment and its objects can be
overlaid on the real world.
2. It makes information about the
surrounding real world of the user
interactive and digitally manipulable.
Which of the above is true?
a)
b)
c)
d)

1 only
2 only
Both 1 and 2
None

http://www.insightsonindia.com INSIGHTS

Page 5

TEST 28 QUESTION PAPER (TEST SERIES 2015)


22. Consider the following about Advertising


Standards Council of India (ASCI).
1. It is a statutory body.
2. It is not under the control of
Government of India.
3. Advertising industries and media are
also included as its members.
4. It submits an annual report to the
Ministry of Information and
Broadcasting.
Choose the correct answer using the codes
below.
a)
b)
c)
d)

2, 3 and 4 only
1, 2 and 4 only
3 only
3 and 4 only

23. The Source of authority of the Indian


Constitution can be found in
a) Directive Principles of State Policy
b) Preamble of the Constitution
c) Fundamental Rights
d) Fundamental Duties

24. In case of a disagreement between the two


Houses regarding a constitutional
amendment bill, which of the following
can be recourse(s) to resolve the
deadlock?
1. A joint sitting of both the houses
2. Vote by majority of state legislatures
3. If the bill is passed again by the Lok
Sabha alone with a higher majority
Choose the correct answer using the codes
below.
a)
b)
c)
d)

25. The Constitution establishes the


parliamentary system not only at the
Centre but also in the states. This is
evident by
1. Membership of the ministers in the
legislature
2. Collective responsibility of the
executive to the legislature
3. The provision of dissolution of the
lower House
Choose the correct answer using the codes
below.
a)
b)
c)
d)

1 and 2 only
2 and 3 only
1 and 3 only
All of the above

26. Consider the following about the


provisions for Citizenship in India.
1. Any person born in India will be
automatically deemed to be a citizen
of India.
2. An illegal migrant cannot be accepted
as Indian citizen only by virtue of him
living in India for long.
3. Acquiring the citizenship of a foreign
nation will terminate Indian
citizenship.
Choose the correct answer using the codes
below.
a)
b)
c)
d)

1 and 2 only
2 and 3 only
1 and 3 only
All of the above

1 and 2 only
2 only
1 and 3 only
None of the above

http://www.insightsonindia.com INSIGHTS

Page 6

TEST 28 QUESTION PAPER (TEST SERIES 2015)


27. Which of these constitutional provisions


under 73rd amendment is/are compulsory
for every Indian state to follow?
1. Organisation of Gram Sabha in a
village or group of villages
2. Direct elections to all seats in
panchayats at the village, intermediate
and district levels
3. Granting financial powers to the
panchayats
Choose the correct answer using the codes
below.
a)
b)
c)
d)

1 and 2 only
2 and 3 only
1 and 3 only
All of the above

28. A federal government means a


government in which
a) all the powers are vested in the
national government and the regional
governments derive their authority
from the national government.
b) powers are divided between the
national government and the regional
governments by the Constitution and
both operate in their respective
jurisdictions independently
c) there is division of powers between
the Central and state governments;
and also between the federal and state
judiciaries
d) a large number of powers are vested in
the national government and the
regional governments, with some
independent powers, derive their
authority from the national
government

29. The Indian constitution is a written one


unlike in some of the other democracies.
What does it imply?
1. The form of government in India has
been codified in the constitution to
reduce political and administrative
conflicts.
2. All the laws made by Parliament are to
be written down as a part of the
constitution.
3. Only because of a written constitution,
citizens are able to enjoy fundamental
rights.
Choose the correct answer using the codes
below.
a)
b)
c)
d)

1 and 2 only
2 and 3 only
1 and 3 only
1 only

30. Elections to the Rajya Sabha from the


State legislative assemblies are done on
the basis of proportional representation.
This means
a) every district in the state is given due
representation in Rajya Sabha from
that state
b) all the states are given representation
in Rajya Sabha
c) that all the states irrespective of their
size and population get due
representation in Rajya Sabha
d) None of the above

http://www.insightsonindia.com INSIGHTS

Page 7

TEST 28 QUESTION PAPER (TEST SERIES 2015)


31. Consider the following about Speaker Pro


Tem.
1. The President can appoint anyone as
the Speaker Pro Tem of Lok Sabha.
2. The Speaker Pro Tem has all the
powers of the Speaker.
3. Office of the Speaker Pro Tem is
temporary in nature.
Choose the correct answer using the codes
below.
a)
b)
c)
d)

1 and 2 only
2 and 3 only
1 and 3 only
All of the above

32. Consider the following statements.


1. Tax cannot be levied or collected
except by authority of law.
2. A new tax cannot be imposed unless
asked for by the executive
government.
3. No expenditure can be sanctioned
unless asked for by the executive
government.
Choose the correct answer using the codes
below.
a)
b)
c)
d)

1 and 2 only
2 and 3 only
1 and 3 only
All of the above

33. The principle of the rule of lapse leads to


rush of expenditure towards the end of a
financial year because
a) unspent money will be accounted as
insufficient and inefficient funding by
the auditor
b) unspent voted expenditure would
lapse by the end of the financial year

c) unspent money will be allocated to the


next years financial budget
d) unspent money will increase the fiscal
burden of the government

34. Which of the following limits the


legislative sphere and authority of the
State Legislatures?
1. A written constitution
2. Division of powers in the Seventh
Schedule
3. Judicial review of State enactments by
the High courts
4. Veto powers of the Governor and
President over state legislations
Choose the correct answer using the codes
below.
a)
b)
c)
d)

2, 3 and 4 only
2 and 3 only
1 and 4 only
All of the above

35. Without the prior approval of the


President of India, which of the following
can NOT be done?
1. Abolition of a certain tax by the
Parliament
2. Introduction of State reorganization
bill in the Parliament
3. Introduction of money bill in the
Parliament.
Choose the correct answer using the codes
below.
a)
b)
c)
d)

1 and 2 only
2 and 3 only
1 and 3 only
All of the above

http://www.insightsonindia.com INSIGHTS

Page 8

TEST 28 QUESTION PAPER (TEST SERIES 2015)


36. Consider the following about Western


cyclonic disturbances in India.
1. They usually occur in winter months.
2. They are brought by westerly flow
from Central Asian region.
3. They influence the weather of the
north and north-western regions of
India.

Choose the correct statements using the codes


below.

Choose the correct statements using the codes


below.

39. Which of the following factors influence


the evolution of landforms on earth?
1. Stability of sea level
2. Tectonic stability of landmasses
3. Climate
4. Groundwater
5. Volcanism

a)
b)
c)
d)

1 and 2 only
2 and 3 only
1 and 3 only
All of the above

37. The stars are localised lumps of gas within


a nebula. Which of the following is/are
considered as some of the stages in the
development of a planet from stars?
1. A dust cloud develops around the core
gas particles.
2. Collision of large gas clouds
3. Bursting of condensed gas clouds
Choose the correct statements using the codes
below.
a)
b)
c)
d)

1 and 2 only
2 and 3 only
1 and 3 only
All of the above

38. Which of the following correctly show the


significance of weathering in rocks?
1. Helps in enrichment of minerals in
rocks
2. Leads to mass wasting and reduction
of relief
3. Aids the formation of soil and thus
biodiversity

a)
b)
c)
d)

1 and 2 only
2 and 3 only
1 and 3 only
All of the above

Choose the correct statements using the codes


below.
a)
b)
c)
d)

1 and 4 only
2, 3 and 5 only
2, 3, 4 and 5 only
All of the above

40. Hari is dependent on a pendulum based


clock for timing kept in a room much
above ordinary summer temperatures. He
is invited for a meeting by his friend (who
stays at a relatively cooler place) at sharp
2o clock. Considering Haris total
dependence on the pendulum clock for
time, it is likely that
a) Hari will be late for the meeting
b) Hari will on time for the meeting
c) Hari will be arrive early for the
meeting
d) The pendulum clock will show a
highly incorrect time and Hari will
never arrive for the meeting

http://www.insightsonindia.com INSIGHTS

Page 9

TEST 28 QUESTION PAPER (TEST SERIES 2015)


41. When a TV signal is broadcasted, why is


its range confined to a limited distance?
a) TV signal transmission uses long
waves which cannot be transmitted for
long distances.
b) TV signal transmission uses short
waves which are absorbed by the
atmosphere
c) TV signal waves carry limited energy
waves which get dissipated is short
period of time
d) The spherical shape of earth does not
allow for very long transmissions

42. When the soil is subjected to Mulching a process which of these is done?
1. Some loose material is laid on the
ground to prevent excessive
evaporation or erosion of the soil
2. The field is irrigated at regular
intervals
3. Seedlings are transplanted
4. Big pieces of soil are broken down into
smaller pieces
Choose the correct answer using the codes
below.
a)
b)
c)
d)

1 and 4 only
1 and 3 only
2 and 3 only
1 only

43. Why do tropical deciduous trees shed


their leaves in summer?
a) Because rainfall is inadequate to cover
up evaporation in summers
b) So that they can prevent excessive loss
of water through evaporation
c) So as to prevent excessive loss of
water through transpiration

d) Because tree leaves generally get dried


up due to the excessive heat

44. Comets orbiting the Sun have glowing


tails because?
1. A comet tail and coma having dust
particles are illuminated by the Sun
2. Gases in the comets tail glow
from ionization.
Which of the above is/are true?
a)
b)
c)
d)

1 only
2 only
Both 1 and 2
None

45. In the practice of out-breeding, which of


the following is/are possible?
1. Different breed of animals can be bred
together
2. Different species of animals can be
bred
3. Superior male of one breed is mated
with superior female from another
breed
Choose the correct answer using the codes
below.
a)
b)
c)
d)

1 and 2 only
1 and 3 only
2 and 3 only
All of the above

46. Bio-fortification leads to increase in the


nutritional value of crops. This can be
done through
1. Conventional breeding of crops
2. Genetic Engineering
3. Injecting crops with nutritive
components

http://www.insightsonindia.com INSIGHTS

Page 10

TEST 28 QUESTION PAPER (TEST SERIES 2015)


4. Grafting crops with each other

Choose the correct answer using the codes


below.
a)
b)
c)
d)

1 and 2 only
1 and 3 only
2 and 4 only
2 and 3 only

47. Consider the following about


Chandrayaan 2, Indias second lunar
exploration mission.
1. The missions primary objective is to
find possible sources of water and
useful minerals on moon.
2. Exploration will not be done on the
lunar surface and instead the lunar
orbit will be used.
3. The mission is being undertaken in
collaboration with Germany and
Russia.
Choose the correct statements using the codes
below.
a)
b)
c)
d)

1 and 2 only
2 and 3 only
1 and 3 only
None of the above

48. Which of these is the most appropriate


evidence that shows Harappan civilization
believed in evil forces?
a) They worshipped Pasupati, which was
considered a powerful deity
b) They used metal objects as seals and
in house entrances
c) They used amulets as protection
against evil forces
d) Animals were sacrificed at regular
intervals in Harappa

49. In the Early Vedic period, who was known


as Vishayapati?
a) One who had great knowledge of all
Vedas and was proficient in teaching
them
b) Head of a group of villages
c) local ruler who patronised vedic
worship and sacrifices
d) The one who headed the military unit
in case of major incidents

50. As per Gautam Buddha


1. No entity in the Universe is as
omnipotent as soul.
2. Human suffering is caused because of
past karmas and there is a way to
transcend it.
3. Transmigration of soul is affected by
the consciousness a person carries in
this birth.
4. God can be attained by living a
balanced life.
Choose the correct statements using the codes
below.
a)
b)
c)
d)

1, 2 and 3 only
2 and 4 only
1 and 3 only
None of the above

51. Which of the following can said to be the


immediate effects of the invasion of
Alexander in India?
1. Stopped political unification of North
India under the Mauryas
2. Direct contact between India and
Greece
3. Annexation of North-western India by
Greece
Choose the correct statements using the codes
below.

http://www.insightsonindia.com INSIGHTS

Page 11

TEST 28 QUESTION PAPER (TEST SERIES 2015)


a)
b)
c)
d)

1 and 2 only
2 and 3 only
1 and 3 only
2 only

52. Horizontal distribution of pressure is


studied by drawing isobars which are lines
connecting places having equal pressure.
How is the effect of altitude in a region
eliminated while studying isobars?
a) by reducing the place being studied to
sea level for comparison
b) by ignoring the nearby high grounds
and mountains and considering the
ground level pressures
c) by measuring the sea level
temperature and pressure of the
nearby regions
d) None of the above

53. RBI uses the tool of sterilization to protect


the economy when
1. Foreign funds and investments pump
in large amounts of money in the
economy
2. Domestic investors lose confidence in
the economy due to falling investment
rate
3. There is a shortage of foreign
exchange reserves of India
Choose the correct answer using the codes
below.
a)
b)
c)
d)

1 and 2 only
2 and 3 only
1 and 3 only
1 only

54. Consider the following examples related to


the opportunity costs between two
options in each of the statements below.
1. An entrepreneur is making zero
profits on both production and nonproduction of goods
2. A labourer earns as much money in a
given time as much he values leisure
In which of the above case(s) is the
opportunity cost between the two options
non-zero?
a)
b)
c)
d)

1 only
2 only
Both 1 and 2
None

55. Which of the following is/are


redistributive tools of the government?
1. Progressive taxation
2. Welfare spending
3. Public employment programmes
4. Subsidies
Choose the correct answer using the codes
below.
a)
b)
c)
d)

1, 2 and 3 only
2, 3 and 4 only
1 and 4 only
All of the above

56. Consider the following statements.


1. High public debt is always bad for an
economy.
2. Inflation can never be good for a
growing economy.
3. Only a nation that has a positive trade
balance can become a developed
country.
Choose the INCORRECT statements using
the codes below.

http://www.insightsonindia.com INSIGHTS

Page 12

TEST 28 QUESTION PAPER (TEST SERIES 2015)


a)
b)
c)
d)

1 and 2 only
2 and 3 only
1 and 3 only
All of the above

57. The issue of Black money has been in


news frequently. Black money can be
generated from which of the following
activities?
1. Tax evasion
2. Tax avoidance
3. Illegal economic activities
4. Foreign investment
Choose the correct answer using the codes
below.
a)
b)
c)
d)

1, 2 and 3 only
2, 3 and 4 only
1 and 4 only
2 and 3 only

58. Consider that a government has been


struggling with high fiscal deficits and
high inflation in the economy. Which of
the following borrowing sources is the
most favourable for it to stabilize the
economy?
a) Borrowing from the public via
treasuries
b) Printing fresh currency by RBI
c) Grants from foreign nations
d) Borrowing from sovereign nations

59. Some of the species below have been in


news. Which of these species is/are
endemic to India?
1. Blue Mormon

2. Yellow-footed green pigeon


3. Indian darter
4. Southern birdwing
Choose the correct answer using the codes
below.
a)
b)
c)
d)

1, 2 and 3 only
2, 3 and 4 only
1 and 4 only
All of the above

60. The Nathu la pass was recently in news.


Consider the following about it.
1. It is a mountain pass in the Northern
Himalayas.
2. It is one of the open trading border
posts between China and India.
3. It was sealed by India after the
1962 Sino-Indian War.
Choose the correct statements using the codes
below.
a)
b)
c)
d)

1 and 2 only
2 and 3 only
1 and 3 only
1 only

61. The District and sessions Judge in a state


works directly under the control of which
of the following authorities?
1. District Collector
2. State Governor
3. State Law Minister
Choose the correct answer using the codes
below.
a)
b)
c)
d)

1 only
2 and 3 only
3 only
None of the above

http://www.insightsonindia.com INSIGHTS

Page 13

TEST 28 QUESTION PAPER (TEST SERIES 2015)


62. Which of the following is NOT a teaching


in Sufism?
1. Turning away heart from all else but
God
2. Purify ones inner self
3. Practicing asceticism
4. Repeating the name of God

3. Mahayana Buddhism believes in idol


worship.
Choose the correct answer using the codes
below.
a)
b)
c)
d)

1 and 2 only
1 and 3 only
2 and 3 only
All of the above

Choose the correct answer using the codes


below.
a)
b)
c)
d)

3 and 4 only
1 and 3 only
2 and 4 only
All are teachings in Sufism.

63. Destruction of Ozone layer can be caused


by
1. All greenhouse gases
2. Supersonic jets
3. Missile launches
4. Heating of the stratosphere
Choose the correct answer using the codes
below.
a)
b)
c)
d)

3 and 4 only
1 and 3 only
2 only
All of the above

64. Consider the following statements about


idols and idol worship in India.
1. Worship of idols in India started
during the Gupta period.
2. The sculpture with the three faces of
Brahma, Vishnu and Mahesh known
as Trimurti is found in Elephanta
caves.

65. You will observe that in cloudy nights less


dew is formed. Why?
a) when there are more clouds there is
less of particulate matter in the
atmosphere
b) clouds allow the heat in the
atmosphere to pass through and
radiation takes place quickly
c) dew is not formed when the
conditions in the environment are that
of precipitation
d) clouds retain the heat in the
atmosphere and radiation takes place
slowly

66. Who among the following is/are the


member(s) of NITI Ayog?
1. Chief Ministers of all States
2. Chief Minister of UTs (with
legislature)
3. Nominated Union Cabinet Ministers
4. Union Secretary of Defence, Finance,
Planning and Home
5. State Finance Ministers
Choose the correct answer using the codes
below.
a)
b)
c)
d)

1, 2 and 5 only
1, 3, 4 and 5 only
3 only
1, 2 and 3 only

http://www.insightsonindia.com INSIGHTS

Page 14

TEST 28 QUESTION PAPER (TEST SERIES 2015)


67. Under the Panchayats (Extension to


Scheduled areas) - PESA, 1996 Act, Gram
Sabhas are given the authority to stop land
alienation by
a) State Legislatures
b) District Collector
c) State Forest department
d) Central government

68. It is possible for two different branches of


a plant to yield two different fruits by
1. Hybridization
2. Grafting
3. Tissue Culture
Choose the correct answer using the codes
below.
a)
b)
c)
d)

1 and 2 only
1 and 3 only
2 only
None of the above

69. If light is entering through a small hole in


a closed room through a closed door, the
image of the opposite house appears as
inverted on the opposite wall of the hole.
This can be explained on the basis of
a) the rectilinear propagation of light
b) the shortest path of light property
c) the hole acts as a lens and magnifies
the object in front of it
d) the light taking turns at the hole and
splitting in N number of different rays

70. In Indian literature, Mucchangam is


a) Buddhist literature patronized under
Satvahanas that emerged after the
Buddhist councils

b) Three Sangams of Tamil poets in


ancient Tamil Nadu
c) Compilation of all the work of the poet
Kalidasa that describes the character
of man
d) a collection of mantras and rituals
(part of Vedas) that are to be
performed on special occasions

71. Consider the following statements about


the early Medieval period.
1. The first Muslim kingdom in India
was established at Ajmer.
2. The Battles of Tarain and Chandawar
contributed to the establishment of
Turkish rule in India.
3. In the first century of the early
medieval period, Rajputs and Mughals
ruled India together.
Choose the correct statements using the codes
below.
a)
b)
c)
d)

1 and 2 only
2 and 3 only
1 and 3 only
All of the above

72. Population III stars were recently in


news because of the discovery of COSMOS
Redshift 7. These Population III stars are
the
a) third generation of stars which are the
youngest and brightest in the Universe
b) first generation of stars which are
born out of the primordial material of
the Big Bang
c) third generation stars which are the
brightest and nearest to the Solar
system
d) stars belonging to the brightest distant
galaxy known to mankind which were

http://www.insightsonindia.com INSIGHTS

Page 15

TEST 28 QUESTION PAPER (TEST SERIES 2015)


born right after the formation of the


gaseous universe

73. Tropical wet and dry climates are those


where
1. There is abundance of Monsoon rain
showers
2. Temperature is high throughout the
year
3. Diurnal range of temperature is
almost nil.
Choose the correct answer using the codes
below.
a)
b)
c)
d)

1 and 2 only
1 and 3 only
2 only
2 and 3 only

Choose the correct statements using the codes


below.
a)
b)
c)
d)

1 and 2 only
2 and 3 only
1 and 3 only
All of the above

76. Prarthana Samaj founded by Keshub


Chandra Sen in Bombay worked on which
of the following socio-religious reforms?
1. Disapproval of the caste system
2. Raising the age of marriage in both
males and females
3. Land reforms for poor landless
peasants
4. Revival of ancient Vedic practices to
educate the India society
Choose the correct statements using the codes
below.

74. The idea of Satyagraha as propounded by


Gandhi includes Satyagraha as
1. A passive resistance
2. Battle with a distance
3. Non-violence which is supreme
Dharma
Choose the correct statements using the codes
below.
a)
b)
c)
d)

1 and 2 only
2 and 3 only
1 and 3 only
3 only

75. The first Round table conference was


announced by Lord Irwin to discuss
1. Dominion status for India
2. Future constitution of India
3. Rights of Princely states in India

a)
b)
c)
d)

1, 2 and 3 only
2 and 4 only
3 and 4 only
1 and 2 only

77. The partition of Bengal was annulled in


1911 by the British because
1. They wanted to curb revolutionary
terrorism
2. Muslim leaders had protested sharply
against a division of Bengal.
3. A divided Bengal was becoming
difficult to administer.
Choose the correct statements using the codes
below.
a)
b)
c)
d)

1 and 2 only
2 and 3 only
1 and 3 only
1 only

http://www.insightsonindia.com INSIGHTS

Page 16

TEST 28 QUESTION PAPER (TEST SERIES 2015)


78. Which of the following most appropriately


shows the impact of the Russian
revolution in India?
a) Indian masses understood that
Imperials can be challenged with
unity.
b) Indian leaders understood the
exploitative nature of
authoritarianism
c) Indian soldiers grasped how the
British were able to use their vast
military influence to rule India
d) Indian leaders understood that to
start a political revolution, an
economic revolution is a must

79. Consider the following about the Eka


movement that surfaced in the Northern
provinces at the end of 1921.
1. It was centred on tenant security and
high land rents.
2. The grassroot leadership of the
movement came from the middle class
intellectuals also leading the Indian
national movement.
3. Severe repression by British
authorities brought an end to this
movement.
Choose the correct statements using the
codes below.
a)
b)
c)
d)

1 and 2 only
2 and 3 only
1 and 3 only
All of the above

3. Rate of Humus formation


4. Colour and texture
Choose the correct statements using the codes
below.
a)
b)
c)
d)

1 and 3 only
2 and 3 only
1, 2 and 4 only
All of the above

81. You are on a visit to a mining site. Out of a


small heap of rocks, you can easily identify
those rocks that contain copper because
a) those rocks will be more lustrous than
the other non-copper containing rocks
b) those rocks will appear blue in colour
c) those rocks are very fine in texture
and have a glassy appearance
d) those rocks will be highly reddish in
colour and can be scratched even with
a nail

82. Consider the following geographical


regions.
1. Western Cordilleras
2. Appalachians
3. Guiana Highlands
4. Kanin Peninsula
Which of the above belong to the North
American region?
a)
b)
c)
d)

1 and 2 only
3 and 4 only
1, 2 and 3 only
All of the above

80.The parent rock of soil controls which of


the following properties of soils?
1. Mineral content
2. Permeability
http://www.insightsonindia.com INSIGHTS

Page 17

TEST 28 QUESTION PAPER (TEST SERIES 2015)


83. Millets and Maize are high nutrition crops


and can be cultivated on a large scale.
Which of these conditions are common for
their cultivation?
1. Moderate temperature
2. Sandy soils
3. Low rainfall
4. Prolonged periods of sunshine
Choose the correct statements using the codes
below.
a)
b)
c)
d)

1 only
2 and 3 only
1, 2 and 4 only
All of the above

84. Kumaon Himalayas is that region of


Himalayas that lies between
a) Satlaj and Kali rivers
b) the base of Shiwaliks and bangar
deposits
c) Pir panjal and Zaskar Himalayas
d) Brahmaputra river basin and Garo
mountains

85. Consider the following about Indian


climate.
1. There is decrease in rainfall generally
from east to west in the Northern
Plains.
2. In general, coastal areas experience
less contrasts in temperature
conditions.
3. Seasonal contrasts are more in the
interior of the country.
Choose the correct statements using the codes
below.
a) 1 and 2 only
b) 2 and 3 only
c) 1 and 3 only

d) All of the above

86. The Union Cabinet has recently approved


the launch of the Housing for All by
2022 programme. It aims at
1. Rehabilitation of slum-dwellers
2. Promotion of affordable housing for
the urban poor
3. Improving existing housing
infrastructure in the country
Choose the correct answer using the codes
below.
a)
b)
c)
d)

1 and 2 only
2 and 3 only
1 and 3 only
All of the above

87. Consider the following about regulation in


medical industry.
1. Government presently does not
regulate online sale of medicines.
2. Government regulates the price of all
drugs sold or imported in India.
Which of the above is/are true?
a)
b)
c)
d)

1 only
2 only
Both 1 and 2
None

88. During the 1st millennium BCE,


the Vaisheshika School of atomism was
founded. As per the school of philosophy
1. All objects in the physical universe are
reducible to atoms.
2. Atoms are indivisible and eternal.
3. Each atom possesses its own distinct
individuality.

http://www.insightsonindia.com INSIGHTS

Page 18

TEST 28 QUESTION PAPER (TEST SERIES 2015)


4. Knowledge and liberation is


achievable by complete understanding
of the world of experience.
Choose the correct answer using the codes
below.
a)
b)
c)
d)

1 and 2 only
2 and 3 only
3 and 4 only
All of the above

89. Consider the following statements.


Assertion (A): The Coriolis force is maximum
at the poles and is absent at the equator.
Reason (R): Coriolis force is directly
proportional to the angle of latitude.
In the context of the statements above, which
of these is true?
a) A and R both are true, and R is the
correct explanation for A.
b) A and R both are true, and R is the
NOT the correct explanation for A.
c) A is incorrect, R is correct.
d) A and R both are incorrect.

90. If the value of rupee starts depreciating in


International market, it will benefit
1. Indian exporters
2. Foreign Institutional Investors (FII)
3. Domestic borrowers
Choose the correct answer using the codes
below.
a)
b)
c)
d)

1 and 2 only
1 and 3 only
2 and 3 only
1 only

91. Indian government has been attempting


to diversify its oil imports portfolio. How
will this benefit India?
1. It is a guard against oil related geopolitical risks.
2. It may reduce shipping time and cost
of crude oil.
3. India will be in a better bargaining
position with oil exporting nations.
Choose the correct answer using the codes
below.
a)
b)
c)
d)

1 and 2 only
1 and 3 only
2 and 3 only
All of the above

92. National Disposable Income includes


which of the following?
1. Net National Product at market prices
2. Current transfers from the rest of the
world
3. Current transfers from government
Choose the correct answer using the codes
below.
a)
b)
c)
d)

1 and 2 only
2 and 3 only
1 and 3 only
All of the above

93. Consider the following statements about


government bonds in the economy.
1. If the interest rates are high, people
would like to keep more money in the
form of government bonds.
2. If there is reduced money supply in
the economy, central bank issues
more government bonds in the
market.
Which of the above is/are correct?

http://www.insightsonindia.com INSIGHTS

Page 19

TEST 28 QUESTION PAPER (TEST SERIES 2015)


a)
b)
c)
d)

1 only
2 only
Both 1 and 2
None

94. Shilpa Shastras is an umbrella term for


numerous Hindu texts that describe arts,
crafts, and their design rules, principles
and standards. They did NOT deal with
1. Painting
2. Metallurgy
3. Architecture
4. Medicine

a) It is a type of monetary policy used by


central banks to stimulate the
economy when standard monetary
policy has become ineffective.
b) It is a type of fiscal stimulus given by
federal/central government when the
economy is going through stagflation
c) It is a type of monetary policy when
government securities are sold
extensively in the market by the
Central bank
d) It is a type of fiscal policy where
growth is diverted from other nations
by providing several sops and
incentives to foreign investors

Choose the correct answer using the codes


below.
a)
b)
c)
d)

2, 3 and 4 only
4 only
1 and 3 only
They dealt with all of the above

95. Take-out finance in India is being used


for
1. Providing funding for long duration
projects
2. Providing bailout funds to sick and
underperforming companies
3. Reducing the Non-performing assets
of banks
Choose the correct answer using the codes
below.
a)
b)
c)
d)

1 and 2 only
1 only
2 and 3 only
All of the above

96. Quantitative easing has been in news for


some time. What does it mean?

97. A nation which is rich in natural resources


and skilled labourers is not economically
very poor with a low GDP per capita.
Which of these can be the most
appropriate explanations for the same?
a) the flow of production in the economy
is very weak
b) the nation is not relying on
sustainable growth models
c) the governments intervention in the
economy is hampering the
development of competitive markets
d) the nation has to rely on imports
excessively for production process

98. Capital goods are those goods that are


1. Used for the production of
intermediate goods
2. Used for the production of consumer
goods
3. Used for generation of capital
4. a part of the nations fixed capital
formation
Choose the correct answer using the codes
below.

http://www.insightsonindia.com INSIGHTS

Page 20

TEST 28 QUESTION PAPER (TEST SERIES 2015)


a)
b)
c)
d)

1, 2 and 4 only
3 and 4 only
2 and 4 only
1 only

99. One of the ways of measuring the income


of a nation is to add all the factor
payments made in the economy. Here
factor payments mean
a) payments made to resources used in
the production process
b) payments used for deploying human
capital in the industries

c) payments made to the fixed asset and


capital used in the production process
in the economy
d) the collective income of all the firms in
the economy

100. What is called Directive Principles in


the present Indian constitution is merely
another name for the instrument of
instructions issued in
a) Indian Independence Act, 1947
b) Government of India Act, 1935
c) 1858 Charter Act
d) Morley-Minto reforms Act, 1919

http://www.insightsonindia.com INSIGHTS

Page 21


INSIGHTS MOCK TEST SERIES 2015: TEST 28 SOLUTIONS

1. Solution: d)
This recent article is The Hindu is the most comprehensive article written on INO by Kalam and
Srijan Pal Singh. It will address all aspects of Neutrino related studies too.
Its an important topic for this year Prelims.
http://www.thehindu.com/opinion/lead/going-all-out-for-neutrino-research/article7322627.ece

2. Solution: c)
Neutrinos are the information bearers of the universe which are almost never lost in their path.
Indias effort in studying neutrinos at INO may help us unravel the deepest mystery of the universe
why there is more matter than antimatter in the universe.
Neutrinos are the least harmful of all elementary particles, as they almost never react with solid
bodies. The mean free path for iron, or the average distance a neutrino will travel in say an iron rod,
before interacting with an atom, is about 1 light year. Needless to say, with the human body being less
than 2 metres in height, any harmful effect of neutrino is near impossible.
Also, people tend to confuse the neutrino for the neutron. This has also led to the confusion that
neutrinos can be weaponised, which is far from the truth. The neutron bomb, which many discuss, is
dangerous but has nothing to do with harmless neutrinos and is made based on a technology around
the neutrons, particles which are much heavier.

3. Solution: a)
The h-index is an index that attempts to measure both the productivity and citation impact of
the published body of work of a scientist or scholar. The index is based on the set of the scientist's
most cited papers and the number of citations that they have received in other publications. The index
can also be applied to the productivity and impact of a scholarly journal as well as a group of
scientists, such as a department or university or country. The index was suggested in 2005 by Jorge E.
Hirsch, a physicist at UCSD, as a tool for determining theoretical physicists' relative quality and is
sometimes called the Hirsch index or Hirsch number.
http://www.saharasamay.com/nation-news/676542754/c-n-r-rao-s-first-indian-scientist-reach-hindex.html
http://www.insightsonindia.com INSIGHTS

Page 1


INSIGHTS MOCK TEST SERIES 2015: TEST 28 SOLUTIONS

4. Solution: d)
GAGAN was jointly developed by the ISRO and Airports Authority of India (AAI) for the advancement
of air navigation. It is an indigenous navigational guide system developed on the lines of GPS system
of the US. Presently, GAGAN signal is broadcasted through two Geostationary Earth Orbit (GEO)
satellites-GSAT8 and GSAT10.
Recently, ISRO has announced to provide navigational support to Indian Railways through GPS
Aided Geo Augmented Navigation (GAGAN) system. The navigational support along with space
technology-based tools will be provided to Indian Railways for unmanned level crossings safety.
By using GAGAN software system, a train will get to know the location of any unmanned level
crossing by a warning signal. Once this warning signal is given, the trains hooter will automatically
start when it comes near an unmanned crossing.

5. Solution: b)
NOWCAST weather alert system will provide free SMS service alerts on extreme weather
conditions (ex-hailstorm) to over one crore registered farmers on their mobile phones. The free
SMS alerts will be sent in Hindi, English and regional languages as per farmers request. It will be
issued to farmers whose mobile phones are already registered on the governments mKisan portal.
Under this initiative, the extreme weather data originated from IMD will be updated on mKisan
portal using a web service. Later, from mKisan portal the warnings regarding extreme weather
conditions will be automatically transmitted by SMS to farmers located at district and block level.
Insurance Web-Portal (www.farmer.gov.in/insurance) will be used to create awareness and
encourage farmers to insure their crops by taking advantage of existing schemes.

6. Solution: d)
Agricultural Insurance in India is covered by National Crop Insurance Programme by merging three
schemes viz. Modified National Agricultural insurance Scheme (MNAIS), Weather Based Crop
insurance Scheme (WBCIS) and Coconut Palm Insurance Scheme (CPIS).
These three schemes now serve as components of the NCIP. The FIIS and CCIS were discontinued
before 2000.
http://www.insightsonindia.com INSIGHTS

Page 2


INSIGHTS MOCK TEST SERIES 2015: TEST 28 SOLUTIONS

Also, read this very insightful article on agricultural insurance related issues in India.
http://articles.economictimes.indiatimes.com/2015-04-26/news/61542788_1_crop-insurancescheme-insurance-fraud-kisan-credit-card

7. Solution: d)
Broadly, the National Crop Insurance Scheme in India covers three stages of crop production.

First stage is of planting or sowing, if the crop was prevented from planting / sowing due to
deficit rainfalls and adverse seasonal conditions.
Second stage is to cover standing crops which got damaged due to unpreventable risks such as
drought, flood, Inundation, Pests, Landslides, Natural Fire, Storms, Cyclones etc.
Third stage is the time period of two weeks after harvesting when the crops are allowed to dry
in field and something wrong happens which destroys the crop such as cyclonic rains/
hailstorms etc.

There are some key differences between its sub-schemes - Modified National Agricultural insurance
Scheme (MNAIS) and Weather Based Crop insurance Scheme (WBCIS).

MNAIS specifically indemnifies the cultivator against shortfall in crop yield, WBCIS
indemnifies against the crop loss due to adverse weather incidences.

MNAIS is on Individual approach, where claim assessment is made for every individual
insured farmer who has suffered a loss. WBCIS is on area approach where sum assured is predefined.

8. Solution: c)
Election Commission of India (ECI) has suspended recognition of the PA Sangma-led National
Peoples Party (NPP) which was a recognized state party in Meghalaya.
ECI decision in this regard comes in line with Supreme Courts judgement in the case of Common
Cause Vs Union of India and others. In this judgement SC had made compulsion for all political
parties to file their election expenditure statement within 75 days of assembly elections and 90 days of
Lok Sabha elections.
The de-recognition of a party does not mean its banned.

http://www.insightsonindia.com INSIGHTS

Page 3


INSIGHTS MOCK TEST SERIES 2015: TEST 28 SOLUTIONS

9. Solution: a)
The new type of thinnest bulb can be used as broadband light emitter. It can also be integrated into
chips which will pave the way towards the realisation of atomically flexible, thin and transparent
displays. It will also help to boost graphene-based on-chip high speed optical communications.

Graphene
Graphene has many extraordinary properties. It is about 200 times stronger than steel by weight,
conducts heat and electricity with great efficiency and is nearly transparent.
The global market for graphene is reported to have reached $9 million by 2014 with most sales in the
semiconductor, electronics, battery energy and composites industries.

10.

Solution: a)

Charles Correa recently passed away. He was Indias greatest contemporary architect.
He was Chief Architect for Navi Mumbai along with Shirish Patel and Pravina Mehta from 197075.
He also had served as first Chairman of the National Commission on Urbanization and was appointed
by then Prime Minister Rajiv Gandhi in 1985.
He is founder of Mumbais Urban Design Research Institute (established in 1984) dedicated to protect
the environment and improve lives of urban communities.

11. Solution: a)
A commemoration of the 200th anniversary of the Battle of Waterloo was held in Belgium.
http://www.insightsonindia.com INSIGHTS

Page 4


INSIGHTS MOCK TEST SERIES 2015: TEST 28 SOLUTIONS

In this battle around ninety thousand French troops led by Napoleon Bonaparte were defeated which
had ended their quest of conquering major parts this region. The battle was fought on 18 June 1815
(Sunday) near Waterloo i.e. in present-day Belgium, then part of the United Kingdom of the
Netherlands.
French army was defeated by the armies of the Seventh Coalition, comprising Anglo-allied army and
Prussian arm led by the Duke of Wellington and Field Marshal Bluecher.
After the defeat in this war, Napoleon was exiled to the island of Saint Helena in the South Atlantic
Ocean, where he died in 1821. The battle had played important role in redrawing the map of Europe
and maintaining the peace in this continent for almost a century until the World War I (started in
1914) tore it apart again.

12. Solution: d)
The project was launched in Rajjipur village of Panchkula district of state in collaboration with Indian
School of Business (ISB). VMP is the first of its kind initiative in Haryana under which villages in the
state would be mentored by leading educational institutions, corporate houses and other
organizations.
It will be launched in a phased manner in 5 villages each of the 22 districts in the state. In case of
Rajjipur village of Panchkula district, faculty and students of ISB will visit the village. They will
present a street play on the theme of Continuing girl education, sanitation, hygiene and nutrition and
also conduct a counseling session, health camps engaging with the school kids and village residents.

13. Solution: b)
President of Tanzania is on a five-day tour to India from 17 June to 21 June 2015.
India and Tanzania have inked various agreements including for cooperation in the field of statistics,
hydrography, agricultural science and tourism. Among them this deserves mention:

Loan agreement to operationalise the LoC: Loan agreement on LOC for US 268.35 million
dollars for extension of Lake Victoria pipeline project
Agreement for appointing the former as PMC: The agreement will pave way for preparing
Detailed Project Report (DPR) for Lake Victoria pipeline project.

14. Solution: c)
http://www.insightsonindia.com INSIGHTS

Page 5


INSIGHTS MOCK TEST SERIES 2015: TEST 28 SOLUTIONS

http://www.financialexpress.com/article/economy/companies/jharkhand-cm-raghubar-das-bansthermocol-plates/87593/
At the same time it will ensure that the plates used for mass consumption are environment friendly.
Even though its a state related matter, it was covered in national dailies and UPSC likes to give such
twisters. Be prepared for all.

15. Solution: a)
United Kingdom (UK) recently celebrated 800 years of signing of Magna Carta on 15 June 2015. The
celebrations were for the 8 century year old historic document which had laid foundation for modern
democratic society.
The document had written promises between King John and his subjects mentioning that the king
will govern England and deal with its people according to the customs of feudal law. Magna Carta had
laid the first steps towards parliamentary democracy and the principle supremacy of law i.e. no one is
above the law.
It also laid the principle of balance of power between the governed and government. Its influence can
be seen in other documents across the world including the UN Universal Declaration of Human
Rights and on US Constitution and Bill of Rights.
In case of India, its influence also can be seen on Part III of constitution which contains fundamental
rights, described as the magna carta of India which has been derived inspiration from Bill of Rights.

16. Solution: b)
http://www.livemint.com/Politics/RRzx6QaPMv5QHdIsHBNEQM/RBIs-action-on-BaselIII-normsdraws-praise.html
LCR is basically highly liquid assets held by financial institutions in order to meet short-term
obligations
The liquidity coverage ratio is an important part of the Basel Accords, as they define how much liquid
assets have to be held by financial institutions. Because banks are required to hold a certain level of
highly-liquid assets, they are less able to lend out short-term debt.

http://www.insightsonindia.com INSIGHTS

Page 6


INSIGHTS MOCK TEST SERIES 2015: TEST 28 SOLUTIONS

17. Solution: b)
A clause in the CLND Act empowers the operator the Right to Recourse and allows it to sue the
suppliers in case of any accident. This was seen as a major hindrance to the growth of the nuclear
industry. These concerns led to the formation of the nuclear insurance pool.
Benefits of INIP

To deal with management of cover to both operators and suppliers on behalf of all direct
insurance companies participating in the pool.
INIP will offer policies on the nuclear operators liability insurance policy and a nuclear
suppliers special contingency (against right to recourse) insurance policy.

18.Solution: c)
The committee has suggested measures for restructuring the Railway Board and its departments so
that policy making is separated from day-to-day operations.

Establishment of an independent regulator Railway Regulatory Authority of India (RRAI) with


a separate budget and to be independent of the Ministry.
RRAI will decide on tariffs to revamp the cash-strapped railways.

The panel says that the Regulator will work under the policy framed by the Ministry, while the
present Railway Board will become a board of Indian Railways the government-run operator
alone. The Regulator can recommend fare revisions but these will not be binding on the
Railway Ministry leaving scope, presumably, for the political dispensation of the day to take a
call.

Railway Budget should be phased out with gross budgetary support to Indian Railways.

There is need to improve the internal resource generation and explore varied methods of
financing but also to improve utilisation of available resources.
No privatisation of Indian Railway but allowed participation of private sector in the railway
projects.
Separation of activities like running of hospitals, schools, real estate development, catering,
manufacturing of locomotives, coaches and wagons from the core business of running trains.

19. Solution: b)
http://www.insightsonindia.com INSIGHTS

Page 7


INSIGHTS MOCK TEST SERIES 2015: TEST 28 SOLUTIONS

One Rank One Pension aims at dispensing the same amount of pension for each rank, with similar
length of service, and with the benefits of future enhancements passed on to past retirees of the
Indian army.
For example, a Subedar who has retired from the army in 1979 is supposed to get the same pension as
a Subedar who has retired in 2013, with the same number of years of service.
OROP will benefit more than 20 lakh ex-servicemen, and about six lakh war widows.

20.

Solution: a)

The question has been made in light of the International Yoga day. Some of the previous tests cover
other aspects.
Yoga Philosophy is one of the six systems of Hindu Philosophy which exist in India. Unlike so many
other philosophies of the world, it is a philosophy that is wholly practical. Yoga is an exact science
based on certain immutable Laws of Nature.
There are four main forms of Yoga, according to one school of thought, namely Mantra Yoga, Hatha
Yoga, Laya Yoga and Raja Yoga; Kundalini Yoga is really Laya Yoga. There is another classification:
Jnana Yoga, Raja Yoga, Laya Yoga, Hatha Yoga and Mantra Yoga.
Yoga is a science that teaches us the method of joining the individual soul and the Supreme Soul. It is
the merging of the individual will with the Cosmic or Universal Will.
Yoga is that state of Absolute Peace wherein there is neither imagination nor thought. Yoga is control
of mind and its modifications. Yoga teaches us how to control the modifications of the mind and
attain liberation.

21. Solution: c)
Augmented reality (AR) is a live direct or indirect view of a physical, real-world environment whose
elements are augmented (or supplemented) by computer-generated sensory input such as sound,
video, graphics or GPS data.
It is related to a more general concept called mediated reality, in which a view of reality is modified
(possibly even diminished rather than augmented), by a computer. As a result, the technology
functions by enhancing ones current perception of reality.

http://www.insightsonindia.com INSIGHTS

Page 8


INSIGHTS MOCK TEST SERIES 2015: TEST 28 SOLUTIONS

By contrast, virtual reality replaces the real world with a simulated one. Augmentation is
conventionally in real-time and in semantic context with environmental elements, such as sports
scores on TV during a match.
With the help of advanced AR technology (e.g. adding computer vision and object recognition) the
information about the surrounding real world of the user becomes interactive and digitally
manipulable. Artificial information about the environment and its objects can be overlaid on the real
world
22.

Solution: c)

ASCI is a self-regulatory voluntary non-governmental organization (NGO) of the advertising industry


in India. Its aim is to maintain and enhance the publics confidence in advertising. It was established
in 1985 by three main constituents of advertising industry viz advertisers, advertising agencies and
media.
It has recently launched an initiative (mobile application) that will provide consumers/users platform
to report complaint services for misleading advertisements on a mobile apart from ASCIs online
complaint facility. It is Indias first consumer complaint mobile app.

23.

Solution: b)

The Preamble reveals four ingredients or components:

24.

Nature of Indian State: It declares India to be of a sovereign, socialist, secular democratic and
republican polity.
Source of authority of the Constitution: The Preamble states that the Constitution derives its
authority from the people of India.
Objectives of the Constitution: It specifies justice, liberty, equality and fraternity as the
objectives.
Date of adoption of the Constitution: It stipulates November 26, 1949 as the date.

Solution: d)

An amendment of the Constitution can be initiated only by the introduction of a bill for the purpose in
either House of Parliament and not in the state legislatures.
http://www.insightsonindia.com INSIGHTS

Page 9


INSIGHTS MOCK TEST SERIES 2015: TEST 28 SOLUTIONS

The bill must be passed in each House by a special majority, that is, a majority (that is, more than 50
per cent) of the total membership of the House and a majority of two-thirds of the members of the
House present and voting.
Each House must pass the bill separately. In case of a disagreement between the two Houses, there is
no provision for holding a joint sitting of the two Houses for the purpose of deliberation and passage
of the bill. The bill ends.

25.

Solution: d)

The parliamentary system is based on the principle of cooperation and co-ordination between the
legislative and executive organs while the presidential system is based on the doctrine of separation of
powers between the two organs. The features of parliamentary government in India are:

26.

Presence of nominal and real executives;


Majority party rule,
Collective responsibility of the executive to the legislature,
Membership of the ministers in the legislature,
Leadership of the prime minister or the chief minister,
Dissolution of the lower House (Lok Sabha or Assembly).

Solution: b)

Just scan through some of the provisions. This issue has been in news for sometime about giving
Hindu migrants from Bangladesh and Pakistan Indian citizenship.
http://indiancitizenshiponline.nic.in/acquisition1.htm
http://www.thehindu.com/news/national/govt-set-to-grant-citizenship-to-hindu-immigrants-frombangladesh/article7190826.ece

27.

Solution: a)

The compulsory (obligatory or mandatory) and voluntary (discretionary or optional) provisions


(features) of the 73rd Constitutional Amendment Act (1992) as in the Part IX of the Constitution are
separately given for the states.
Some of the voluntary provisions are:
http://www.insightsonindia.com INSIGHTS

Page 10


INSIGHTS MOCK TEST SERIES 2015: TEST 28 SOLUTIONS

28.

Giving representation to members of the Parliament (both the Houses) and the state legislature
(both the Houses) in the panchayats at different levels falling within their constituencies.
Providing reservation of seats (both members and chairpersons) for backward classes in
panchayats at any level.
Granting powers and authority to the panchayats to enable them to function as institutions of
self-government (in brief, making them autonomous bodies).
Devolution of powers and responsibilities upon panchayats to prepare plans for economic
development and social justice; and to perform some or all of the 29 functions listed in the
Eleventh Schedule of the Constitution.

Solution: b)

Political scientists have classified governments into unitary and federal on the basis of the nature of
relations between the national government and the regional governments.
By definition, a unitary government is one in which all the powers are vested in the national
government and the regional governments, if at all exist, derive their authority from the national
government. A federal government, on the other hand, is one in which powers are divided between
the national government and the regional governments by the Constitution itself and both operate in
their respective jurisdictions independently. Britain, France, Japan, China, Italy, Belgium, Norway,
Sweden, Spain and so on have the unitary model of government while the US, Switzerland, Australia,
Canada, Russia, Brazil, Argentina and so on have the federal model of government.
In a federal model, the national government is known as the Federal government or the Central
government or the Union government and the regional government is known as the state government
or the provincial government.
29.

Solution: d)

Constitution specifies the structure, organisation, powers and functions of both the Central and state
governments and prescribes the limits within which they must operate. Thus, it avoids the
misunderstandings and disagreements between the two.
All the laws made in India are codified separate from the constitution and maintained in a law book.
They need not be a part of the constitution.
Even in the UK where there is no written constitution, people enjoy several fundamental rights.
However, only because our fundamental rights are written in the constitution, it is difficult to amend
and change them as per the wishes of the political executive.

http://www.insightsonindia.com INSIGHTS

Page 11


INSIGHTS MOCK TEST SERIES 2015: TEST 28 SOLUTIONS

30.

Solution: c)

Option C is more preferable to option B because the former is more complete and correct.
Proportional representation essentially means representation that is proportionate to some factor e.g.
population here.
The system of proportional representation aims at removing the defects of territorial representation.
Under this system, all sections of the people get representation in proportion to their number. Even
the smallest section of the population gets its due share of representation in the legislature.

31. Solution: b)
As provided by the Constitution, the Speaker of the last Lok Sabha vacates his office immediately
before the first meeting of the newly- elected Lok Sabha. Therefore, the President appoints a member
of the Lok Sabha as the Speaker Pro Tem. Usually the senior most member is selected for this. The
President himself administers oath to the Speaker Pro Tem.
The Speaker Pro Tem has all the powers of the Speaker. He presides over the first sitting of the newlyelected Lok Sabha. His main duty is to administer oath to the new members. He also enables the
House to elect the new Speaker.
When the new Speaker is elected by the House, the office of the Speaker Pro Tem ceases to exist.
Hence, this office is a temporary office, existing for a few days.

32.

Solution: d)

Article 113 of the constitution says that no new expenditure can be sanctioned unless asked for by the
executive government. It means that no other branch of the government has the authority to sanction
public expenditure.
Article 117 of the constitution says that no other branch of the government can ask for a new tax other
than the executive.
But even if the executive wants to impose a new tax, it cannot be done without the authorization of the
legislature.

33.

Solution: b)

http://www.insightsonindia.com INSIGHTS

Page 12


INSIGHTS MOCK TEST SERIES 2015: TEST 28 SOLUTIONS

Every government budget is on an annual basis.


Therefore, the "rule of lapse" will apply at the end of the year, i.e., if the voted expenditure (by the
Parliament) is not spent by the end of the financial year (i.e., March 31), then the unspent balance
would lapse.
Observance of the rule of lapse has led to a heavy "rush of expenditure" towards the close of the
financial year.

34.

Solution: d)

Since legislative division is made in 7th schedule Union, State and Concurrent list, the state
legislatures are bound to their allocated subjects.
Both the Supreme and High courts can review the acts of the state legislatures and declare them null
and void if they are ultra vires the constitution or any fundamental rights.
The Governor has every right to reserve state bill for President in certain cases. The president may or
may not sign the bills in which case they lapse.
The governor can also exercise his veto powers over state enactments. He can also delay a bill.
35.

Solution: b)

Article 3 lays down two conditions in the event of states reorganization: one, a bill contemplating the
above changes can be introduced in the Parliament only with the prior recommendation of the
President; and two, before recommending the bill, the President has to refer the same to the state
legislature concerned for expressing its views within a specified period.
The Constitution lays down a special procedure for the passing of money bills in the Parliament. A
money bill can only be introduced in the Lok Sabha and that too on the recommendation of the
president. Every such bill is considered to be a government bill and can be introduced only by a
minister.

36.

Solution: c)

The western cyclonic disturbances are weather phenomena of the winter months brought in by the
westerly flow from the Mediterranean region.
They usually influence the weather of the north and north-western regions of India. Tropical cyclones
occur during the monsoon as well as in October - November, and are part of the easterly flow.
http://www.insightsonindia.com INSIGHTS

Page 13


INSIGHTS MOCK TEST SERIES 2015: TEST 28 SOLUTIONS

These disturbances affect the coastal regions of the country. The disasters caused by them on Orissa
and Andhra Pradesh coast are notable.

37.

Solution: a)

The following are considered to be the stages in the development of planets:


The stars are localised lumps of gas within a nebula. The gravitational force within the lumps
leads to the formation of a core to the gas cloud and a huge rotating disc of gas and dust
develops around the gas core.
In the next stage, the gas cloud starts getting condensed and the matter around the core
develops into small-rounded objects. These small-rounded objects by the process of cohesion
develop into what is called planetesimals. Larger bodies start forming by collision, and
gravitational attraction causes the material to stick together. Planetesimals are a large number
of smaller bodies.
In the final stage, these large number of small planetesimals accrete to form a fewer large
bodies in the form of planets.
38. Solution: d)

Weathering processes are responsible for breaking down the rocks into smaller fragments and
preparing the way for formation of not only regolith and soils, but also erosion and mass movements.
Biomes and biodiversity is basically a result of forests (vegetation) and forests depend upon the depth
of weathering mantles.
Erosion cannot be significant if the rocks are not weathered. That means, weathering aids mass
wasting, erosion and reduction of relief and changes in landforms are a consequence of erosion.
Weathering is an important process in the formation of soils.
Weathering of rocks and deposits helps in the enrichment and concentrations of certain valuable ores
of iron, manganese, aluminium, copper etc., which are of great importance for the national economy.
When rocks undergo weathering, some materials are removed through chemical or physical leaching
by groundwater and thereby the concentration of remaining (valuable) materials increases. Without
such a weathering taking place, the concentration of the same valuable material may not be sufficient
and economically viable to exploit, process and refine. This is what is called enrichment.

39.

Solution: d)

As the geomorphic agents are capable of erosion and deposition, two sets Erosional or destructional
and depositional or constructional of landforms are produced by them.
http://www.insightsonindia.com INSIGHTS

Page 14


INSIGHTS MOCK TEST SERIES 2015: TEST 28 SOLUTIONS

Many varieties of landforms develop by the action of each of the geomorphic agents depending upon
especially the type and structure i.e. folds, faults, joints, fractures, hardness and softness,
permeability and impermeability, etc. come under structure of rocks. There are some other
independent controls like (i) stability of sea level; (ii) tectonic stability of landmasses; (iii) climate,
which influence the evolution of landforms.
Any disturbance in any of these three controlling factors can upset the systematic and sequential
stages in the development and evolution of landforms. Moreover, geomorphic regimes i.e. running
water; groundwater, glaciers, waves, and winds also affect landforms on earth.

40.

Solution: a)

An ordinary clock loses time in summer. This is because the length of the pendulum increases by the
expansion of the metal due to the additional heat. Since the pendulum clock depends on the length of
the pendulum for accuracy of the time, its time period is denoted by 2 (l/g) where l is the length of
the pendulum and g is value of the gravity. So, its time period increases and the clock slows down and
gets late.
So Hari will arrive late for the meeting.

41. Solution: d)
Surface Waves are the principle waves used in AM, FM and TV broadcast. Objects such as buildings,
hills, ground conductivity, etc. have a significant impact on their strength.

Space waves are also used. The other name of space wave propagation is line of sight propagation.
There are some limitations of space wave propagation.
1. These waves are limited to the curvature of the earth.
2. These waves have line of sight propagation, means their propagation is along the line of sight
distance.

http://www.insightsonindia.com INSIGHTS

Page 15


INSIGHTS MOCK TEST SERIES 2015: TEST 28 SOLUTIONS

The line of sight distance is that exact distance at which both the sender and receiver antenna are in
sight of each other. So, from the above line it is clear that if we want to increase the transmission
distance then this can be done by simply extending the heights of both the sender as well as the
receiver antenna. This type of propagation is used basically in radar and television communication.
The frequency range for television signals is nearly 80 to 200MHz. These waves are not reflected by
the ionosphere of the earth. The property of following the earths curvature is also missing in these
waves. So, for the propagation of television signal, geostationary satellites are used. The satellites
complete the task of reflecting television signals towards earth. If we need greater transmission then
we have to build extremely tall antennas.

42.

Solution: d)

Mulch is simply a protective layer of a material that is spread on top of the soil. Mulches can either be
organic -- such as grass clippings, straw, bark chips, and similar materials -- or inorganic -- such as
stones, brick chips, and plastic.
Organic mulches also improve the condition of the soil. As these mulches slowly decompose, they
provide organic matter which helps keep the soil loose. This improves root growth, increases the
infiltration of water, and also improves the water-holding capacity of the soil. Organic matter is a
source of plant nutrients and provides an ideal environment for earthworms and other beneficial soil
organisms.

43.

Solution: c)

In tropical and subtropical forests, trees shed their leaves at the onset of the dry season. Trees that
lose all of their leaves for part of the year are known as deciduous trees.
Most deciduous trees have broad leaves that are susceptible to being damaged during cold or dry
weather. In contrast, most evergreen trees either live in warm, wet climates or they have weatherresistant needles for leaves. However, there are exceptions in nature, such as tamarack trees that shed
their needles every autumn and live oaks that retain their broad leaves for the entire year even in
relatively cool climates.
Shedding leaves helps trees to conserve water and energy.

44.

Solution: c)

http://www.insightsonindia.com INSIGHTS

Page 16


INSIGHTS MOCK TEST SERIES 2015: TEST 28 SOLUTIONS

A comet tail and coma are illuminated by the Sun and may become visible from Earth when
a comet passes through the inner Solar System.
Separate tails are formed of dust and gases, becoming visible with through different phenomena; the
dust reflects sunlight directly and the gases glow from ionisation.

The bright white tail is called the dust tail because it is made of the dust coming off the nucleus as the
comet heats up near the Sun. The dust tail doesn't actually glow with its own light. The dust grains
reflect the Sunlight, making the tail look very bright.
Most comets are too faint to be visible without the aid of a telescope, but a few each decade become
bright enough to be visible to the naked eye.

45.

Solution: a)

Out-breeding is the breeding of the unrelated animals, which may be between individuals of the same
breed but having no common ancestors for 4-6 generations (out-crossing) or between different breeds
(cross-breeding) or different species (inter-specific hybridisation).
In this method, superior males of one breed are mated with superior females of another breed. Crossbreeding allows the desirable qualities of two different breeds to be combined.
Controlled breeding experiments are carried out using artificial insemination

46.

Solution: a)

Fortification is the practice of deliberately increasing the content of an essential micronutrient, i.e.
vitamins and minerals (including trace elements) in a food, so as to improve the nutritional quality of
the food supply and provide a public health benefit with minimal risk to health.
http://www.insightsonindia.com INSIGHTS

Page 17


INSIGHTS MOCK TEST SERIES 2015: TEST 28 SOLUTIONS

Biofortification is the process by which the nutritional quality of food crops is improved through
biological means such as conventional plant breeding. Biofortification differs from conventional
fortification in that biofortification aims to increase nutrient levels in crops during plant growth
rather than through manual means during processing of the crops. Biofortification may therefore
present a way to reach populations where supplementation and conventional fortification activities
may be difficult to implement and/or limited.
Examples of biofortification projects include:

iron-biofortification of rice, beans, sweet potato, cassava and legumes;

zinc-biofortification of wheat, rice, beans, sweet potato and maize;

provitamin A carotenoid-biofortification of sweet potato, maize and cassava; and

amino acid and protein-biofortification of sourghum and cassava.

47.

Solution: d)

The Chandrayaan-2 is an ISRO mission aimed at placing an orbiter around the moon and sending a
lander and a rover to its surface in 2017-18.
Here is the mission overview http://isp.justthe80.com/moon-exploration/chandrayaan---2#TOCMission-Overview
Unlike Chandrayaan-1's lunar probe, which impacted the Moon's surface, the lander will make a soft
landing. It will use a rover to explore its surface.
Initially, the lander was slated to be developed by Russia in collaboration with India. When Russia
stated its inability provide the lander to meet even the revised time frame of 2015, Indian officials
decided to develop the lander independently.

48.

Solution: c)

From the seals, terracotta figurines and copper tablets we get an idea on the religious life of the
Harappans. The chief male deity was Pasupati, (proto-Siva) represented in seals as sitting in a yogic
posture with three faces and two horns. Trees and animals were also worshipped by the Harappans.
They believed in ghosts and evil forces and used amulets as protection against them.
The chief female deity was the Mother Goddess represented in terracotta figurines. In latter times,
Linga worship was prevalent.
http://www.insightsonindia.com INSIGHTS

Page 18


INSIGHTS MOCK TEST SERIES 2015: TEST 28 SOLUTIONS

49.

Solution: b)

In the Rig Vedic or early Vedic period, the basic unit of political organization was kula or family.
Several families joined together on the basis of their kinship to form a village or grama. The leader of
grama was known as gramani. A group of villages constituted a larger unit called visu. It was headed
by vishayapati. The highest political unit was called jana or tribe.
There were several tribal kingdoms during the Rig Vedic period such as Bharatas, Matsyas, Yadus and
Purus. The head of the kingdom was called as rajan or king. The Rig Vedic polity was normally
monarchical and the succession was hereditary. The king was assisted by purohita or priest and
senani or commander of the army in his administration.

50.

Solution: d)

Buddha did not involve himself in fruitless controversies regarding metaphysical questions like god,
soul, karma, rebirth, etc., and concerned himself with the practical problems confronting man.
Buddha neither accepts god nor rejects the existence of god. He laid great emphasis on the law of
karma. He argued that the condition of man in this life depends upon his own deeds in present life.
He taught that the soul does not exist. However, he emphasized Ahimsa. By his love for human beings
and all living creatures, he endeared himself to all.

51. Solution: d)
The system of small independent states came to an end with Alexanders invasion. The immediate
effect of Alexanders invasion was that it encouraged political unification of north India under the
Mauryas.
Alexanders invasion had also paved the way for direct contact between India and Greece. The routes
opened by him and his naval explorations increased the existing facilities for trade between India and
West Asia.
However, his aim of annexing the northwestern India to his empire was not fulfilled due his
premature death. His authority in the Indus valley was a short-lived one because of the expansion of
Mauryan Empire under Chandragupta Maurya.

http://www.insightsonindia.com INSIGHTS

Page 19


INSIGHTS MOCK TEST SERIES 2015: TEST 28 SOLUTIONS

52.

Solution: a)

In order to eliminate the effect of altitude on pressure, it is measured at any station after being
reduced to sea level for purposes of comparison. The sea level pressure distribution is shown on
weather maps.
The figure above (from NCERT) shows the patterns of isobars corresponding to pressure systems.
Low pressure system is enclosed by one or more isobars with the lowest pressure in the centre.
High-pressure system is also enclosed by one or more isobars with the highest pressure in the centre.

53.

Solution: d)

It refers to the process by which the RBI takes away money from the banking system to neutralise the
fresh money that enters the system.
Suppose the RBI decides to buy US dollars (USD) from the market. Now, the money held by the RBI
does not form part of the banking system. So, if the RBI releases rupees from its coffers to buy dollars,
the money supply in the banking system increases. That can be a problem leading to inflation.
If you need more explanation and details go to
http://www.thehindubusinessline.com/iw/2003/02/02/stories/2003020201001400.htm
54.

Solution: d)

Opportunity cost is the cost that you pay the loss of other alternatives when one alternative is chosen.
In statement 1, no matter what the entrepreneur chooses, he has to suffer the same loss or profit. In
other words, both the options are same.
In statement 2, no matter what the labourer chooses, his benefit from both the activities is the same.
http://www.insightsonindia.com INSIGHTS

Page 20


INSIGHTS MOCK TEST SERIES 2015: TEST 28 SOLUTIONS

55.

Solution: d)

Redistribution of income and redistribution of wealth are respectively the transfer of income and
of wealth (including physical property) from some individuals to others by means of a social
mechanism such as taxation, monetary policies, welfare, land
reform, charity, confiscation,divorce or tort law.
Wealth redistribution can be implemented through land reform that transfers ownership of land from
one category of people to another, or through inheritance taxes or direct wealth taxes.

56.

Solution: d)

High public debt is not a problem when it is being used for creation of infrastructure, employment
and other productive areas.
Inflation is needed for any economy to grow because inflation suggests that the demand for goods has
outstripped the supply. In this case more production is required which leads to higher growth.
A positive trade balance has little relation with a criterion of development. USA has a large trade
deficit, yet is is a developed country. However, having a positive trade balance is good for the economy
because it means we are competitive in the world export market.

57.Solution: d)
Foreign investment does not generate black money. It only circulates the existing black money supply.
Tax avoidance is not exactly illegal. It basically means to bypass tax laws and rules to reduce the tax
burden on an individual or firm. The issue of GAAR is one of issues related with it.
On the other hand, tax evasion is the illegal evasion of taxes by individuals, corporations and trusts.
Tax evasion often entails taxpayers deliberately misrepresenting the true state of their affairs to the
tax authorities to reduce their tax liability and includes dishonest tax reporting, such as declaring less
income, profits or gains than the amounts actually earned, or overstating deductions. Tax evasion is
an activity commonly associated with the informal economy.

58.

Solution: a)

http://www.insightsonindia.com INSIGHTS

Page 21


INSIGHTS MOCK TEST SERIES 2015: TEST 28 SOLUTIONS

Grants from foreign nations are preferred only when the government does not want to increase the
money supply and inflation by infusion of fresh money from abroad.
Borrowing from sovereigns nations also suffers from this defect.
Public borrowing will keep the money supply constant allowing the government to finance its deficit
as well as tackle inflation.
Printing fresh currency is highly inflationary as it increases the money supply without adding
anything to the production process.

59.

Solution: d)

The Southern Birdwing (Troides minos) is a large & Swallowtail butterfly endemic to Peninsular
India. With a wingspan of 140190 mm, it is the largest butterfly of India, found in southern India.

Southern Birdwing
Blue Mormon (Papilio polymnestor) is the second largest butterfly species in terms of size, after the
southern birdwing (Troides minos).
Maharashtra has officially declared Blue Mormon species as State butterfly as part of state symbols.
With this, Maharashtra became the first State in the country to have a State butterfly.

Blue mormon

http://www.insightsonindia.com INSIGHTS

Page 22


INSIGHTS MOCK TEST SERIES 2015: TEST 28 SOLUTIONS

It is only found (endemic to) in Sri Lanka, Western Ghats of Maharashtra, South India and coastal
belts of the country.
Kerala Government has declared June 22 as Snake Bird Day in a bid to protect the endangered
species which is also known as Indian darter.
It is water bird which is also known as Indian darter (Anhinga melanogaster). It has large slender
long-necked water bird and is mostly black with silver wing markings. As per IUCN Red List of
Threatened Species, Snake Bird is classified as Near Threatened species.
60.

Solution: b)

China has opened 2nd land border crossing in Tibet via Nathu La, Sikkim to allow the Indian pilgrims
for Kailash-Mansarovar yatra. The new route is in addition and alternative to the existing Lipulekh
Pass route located in Uttarakhand which was only route earlier. This route was damaged in the
devastating Uttarakhand floods in 2013.
Nathu La is a mountain pass in the Himalayas. It connects the Indian state of Sikkim with China's
Tibet Autonomous Region. The pass, at 4,310 m (14,140 ft) above mean sea level, forms a part of an
offshoot of the ancient Silk Road. Nathu means "listening ears" and La means "pass" in Tibetan. On
the Indian side, the pass is 54 km (34 mi) east of Gangtok, the capital of Indian state of Sikkim. Only
citizens of India can visit the pass, and then only after obtaining a permit in Gangtok.
Nathu La is one of the second open trading border posts between China and India; the next is Shipkila
in Himachal Pradesh and Lipulekh (or Lipulech) in Nepal is also an other way. Sealed by India after
the 1962 Sino-Indian War, Nathu La was re-opened in 2006 following numerous bilateral trade
agreements.

61. Solution: d)
The District courts are subordinate courts that work under the authority of the High court and
Supreme Court. Immediate control is exerted by the HC, and final control by the SC.
The HCs are responsible for appointment, postings, transfers and general administration of the
district courts.
The district courts are free from interference by the state or central executive.

62.

Solution: d)

http://www.insightsonindia.com INSIGHTS

Page 23


INSIGHTS MOCK TEST SERIES 2015: TEST 28 SOLUTIONS

Classical Sufi scholars have defined Sufism as "a science whose objective is the reparation of the heart
and turning it away from all else but God". Alternatively, in the words of the Darqawi Sufi
teacher Ahmad ibn Ajiba, "a science through which one can know how to travel into the presence of
the Divine, purify one's inner self from filth, and beautify it with a variety of praiseworthy traits".
Classical Sufis were characterised by their attachment to dhikr, (a practice of repeating the names of
God, often performed after prayers) and asceticism. Sufism gained adherents among a number of
Muslims as a reaction against the worldliness of the early Umayyad Caliphate (661750 CE). Sufis
have spanned several continents and cultures over a millennium, originally expressing their beliefs in
Arabic, before spreading into Persian, Turkish, Indian languages and a dozen other languages.

63.

Solution: c)

Paul Crutzen pointed out that emissions of nitrous oxide (N2O), a stable, long-lived gas produced by
soil bacteria, from the Earth's surface could affect the amount of nitric oxide (NO) in the stratosphere.
Crutzen showed that nitrous oxide lives long enough to reach the stratosphere, where it is converted
into NO.
Crutzen then noted that increasing use of fertilizers might have led to an increase in nitrous oxide
emissions over the natural background, which would in turn result in an increase in the amount of NO
in the stratosphere. Thus human activity could have an impact on the stratospheric ozone layer. In the
following year,
Crutzen and (independently) Harold Johnston suggested that NO emissions from supersonic
passenger aircraft, which would fly in the lower stratosphere, could also deplete the ozone layer.
Then, more recently in 1995 David W. Fahey, an atmospheric scientist at the National Oceanic and
Atmospheric Administration, found that the drop in ozone would be from 1 to 2% if a fleet of 500
supersonic passenger aircraft were operated.
You can also refer to this very good source about ozone depletion
http://www.ozonedepletion.info/education/part3/ozonesources.html

64.

Solution: d)

Trimurti is considered the most important sculpture of Elephanta and is described as a masterpiece
of Gupta-Chalukyan art. It is placed at the back of the cave facing the entrance, on the north-south
axis and is also known as Maheshmurti or Trimurti Sadashiva. The three-headed image of Shiva is of
a height of 20 ft and represents Panchamukha Shiva. Creation, destruction and protection are the
three essential aspects that are represented by the three heads of Shiva.
http://www.insightsonindia.com INSIGHTS

Page 24


INSIGHTS MOCK TEST SERIES 2015: TEST 28 SOLUTIONS

The Mahayana Buddhism advocated the worship of the image of the Buddha and of the Bodhisattvas.
This brought it nearer to Hindu idol worship. Most interestingly, the Gupta period saw the emergence
of the image of the Buddha as Bhagavan. Hinduism, in its unlimited liberalism, came to accept
Buddha as a God of the Hindus. Buddha came to be worshipped as an incarnation of Vishnu.

65.

Solution: d)

The dew formation is more when the sky is clear and less when it is cloudy.
When the sky is clear and the trees and plants are cooler at nights, there is more evaporation of water
and hence more dew formation.
But when it is cloudy, trees and plants do not get cool in the night and hence there is less dew
formation. As the sun raises high in the sky, these dew drops evaporate into air.

66.

Solution: c)

The NITI Aayog comprises the following:


1. Prime Minister of India as the Chairperson
2. Governing Council comprising the Chief Ministers of all the States and union territories with
legislature and lieutenant governors of other Union Territories
3. Regional Councils
4. Experts, specialists and practitioners with relevant domain knowledge as special invitees
nominated by the Prime Minister
5. Full-time organizational framework (in addition to Prime Minister as the Chairperson)
comprising
1. Vice-Chairperson: Arvind Panagariya
2. Members: Two (2) Full-time: economist Bibek Debroy and former DRDO chief V.K.
Saraswat
3. Part-time members: Maximum of two from leading universities research organizations
and other relevant institutions in an ex-officio capacity. Part-time members will be on a
rotational basis

http://www.insightsonindia.com INSIGHTS

Page 25


INSIGHTS MOCK TEST SERIES 2015: TEST 28 SOLUTIONS

4. Ex Officio members: Maximum of four members of the Union Council of Ministers to be


nominated by the Prime Minister
5. Chief Executive Officer: To be appointed by the Prime Minister for a fixed tenure, in the
rank of Secretary to the Government of India. Sindhushree Khullar appointed as the
Chief Executive Officer.
6. Secretariat as deemed necessary

67.

Solution: a)

Panchayats (Extension to Scheduled Areas) Act, 1996 or PESA is a law enacted by the Government of
India to cover the "Scheduled areas", which are not covered in the 73rd amendment or Panchayati Raj
Act of the Indian Constitution. It was enacted on 24 December 1996 to enable Gram Sabhas to self
govern their natural resources.
How much self-governance will be allowed depends on the state legislative assemblies. For e.g. in
some states like Jharkhand, Gram Sabhas under scheduled areas do not have the power to stop land
alienation, while in some other states they do.

68.

Solution: c)

Grafting describes any of a number of techniques in which a section of a stem with leaf buds is
inserted into the stock of a tree. The upper part of the graft (the scion) becomes the top of the plant,
the lower portion (the understock) becomes the root system or part of the trunk. Although grafting
usually refers to joining only two plants, it may be a combination of several. A third plant added
between two others becomes the trunk or a portion of it. This is called an interstem.
Why grafting?
Some cultivars (varieties) of plants do not come true from seeds. The seed from a Haralson apple will
produce an apple tree, but it will not produce a Haralson apple tree. In other words, fruit trees cannot
be reproduced "true" to the original cultivar from seed. They can only be reproduced by grafting.
Grafting (top working), is a way to change a large tree from old to a new variety. It is also a method of
using a root system better adapted to soil or climate than that produced naturally by an ungrafted
plant. By using special understocks or interstems, grafting is a way to produce dwarf plants.

69.

Solution: a)

http://www.insightsonindia.com INSIGHTS

Page 26


INSIGHTS MOCK TEST SERIES 2015: TEST 28 SOLUTIONS

'Rectilinear propagation' is a wave property which states waves propagate (move or spread out) in
straight lines. This property applies to both transverse and longitudinal waves as well
as electromagnetic waves (light). Even though a wave front may be bent (e.g., the waves created by a
rock hitting a pond) the individual waves are moving in straight lines.
This can be proven by setting up an experiment in which you align three cardboard squares with a
small hole in the centre of each. You then set up a light behind the cardboard and look through all
three holes from the other side to see the light. If you moved any one of the cardboard squares even a
tiny bit, you would no longer be able to see the light. This proves that waves travel in straight lines
and this helps to explain how humans see things, among other uses. It has a number of applications in
real-life as well.

70.

Solution: b)

The Sangam Age constitutes an important chapter in the history of South India. According to Tamil
legends, there existed three Sangams (Academy of Tamil poets) in ancient Tamil Nadu popularly
called Muchchangam. These Sangams flourished under the royal patronage of the Pandyas.
The first Sangam, held at Then Madurai, was attended by gods and legendary sages but no literary
work of this Sangam was available. The second Sangam was held at Kapadapuram but the all the
literary works had perished except Tolkappiyam.
The third Sangam at Madurai was founded by Mudathirumaran. It was attended by a large number of
poets who produced voluminous literature but only a few had survived.

71. Solution: a)
In the second battle of Tarain Rajputs lost to the Muslim invaders. The first Muslim kingdom was
thus firmly established in India at Ajmer and a new era in the history of India began.
After his brilliant victory over Prithiviraj at Tarain, Muhammad Ghori returned to Ghazni leaving
behind his favourite general Qutb-uddin Aibak to make further conquests in India. Aibak
consolidated his position in India by occupying places like Delhi and Meerut.
In 1193 he prepared the ground for another invasion by Muhammad Ghori. This invasion was directed
against the Gahadavala ruler Jayachandra. Muhammad routed Jayachandras forces. Kanauj was
occupied by the Muslims after the battle of Chandawar. Thus, the battles of Tarain and Chandawar
had a very important role in starting the Turkish rule in India.

http://www.insightsonindia.com INSIGHTS

Page 27


INSIGHTS MOCK TEST SERIES 2015: TEST 28 SOLUTIONS

72.

Solution: a)

Astronomers have named a newly discovered galaxy as CR7 (COSMOS Redshift 7) after Portuguese
footballer Cristiano Ronaldo.
CR7 is three times brighter compared to Himiko galaxy which was earlier thought be as brightest
distant galaxy known to mankind. It is part of the early universe which contains the first generation of
stars i.e. known as Population III stars.
CR7 belongs to the age of the ancient Universe which existed 800 million years after the Big Bang.
This period is known as reionization period.
Population III stars are first generation of stars which are born out of the primordial material of the
Big Bang. It is believed that Population III stars are several hundred or even a thousand times more
massive than our Sun and it is more blazing hot, transient, exploding as supernovae after only about 2
million years ago.

73.

Solution: c)

Tropical wet and dry climate occurs north and south of tropical wet type climate regions. It borders
with dry climate on the western part of the continent.
Such climate is extensively found to the north and south of the Amazon forest in Brazil and adjoining
parts of Bolivia and Paraguay in South America, Sudan and south of Central Africa.
The annual rainfall in this climate is considerably less than that in Af and Am climate types and is
variable also. The wet season is shorter and the dry season is longer with the drought being more
severe. Temperature is high throughout the year and diurnal ranges of temperature are the greatest in
the dry season. Deciduous forest and tree-shredded grasslands occur in this climate.

74.

Solution: d)

Mahatma Gandhi on Satyagraha


It is said of passive resistance that it is the weapon of the weak, but the power which is the subject
of this article can be used only by the strong. This power is not passive resistance; indeed it calls for
intense activity. The movement in South Africa was not passive but active
Satyagraha is not physical force. A satyagrahi does not inflict pain on the adversary; he does not seek
his destruction In the use of satyagraha, there is no ill-will whatever.
http://www.insightsonindia.com INSIGHTS

Page 28


INSIGHTS MOCK TEST SERIES 2015: TEST 28 SOLUTIONS

Satyagraha is pure soul-force. Truth is the very substance of the soul. That is why this force is called
satyagraha. The soul is informed with knowledge. In it burns the flame of love. Nonviolence is the
supreme dharma
It is certain that India cannot rival Britain or Europe in force of arms. The British worship the wargod and they can all of them become, as they are becoming, bearers of arms. The hundreds of millions
in India can never carry arms. They have made the religion of non-violence their own ...

75.Solution: a)
When the Simon Commission arrived in India in 1928, it was greeted with the slogan Go back Simon.
All parties, including the Congress and the Muslim League, participated in the demonstrations.
In an effort to win them over, the viceroy, Lord Irwin, announced in October 1929, a vague offer of
dominion status for India in an unspecified future, and a Round Table Conference to discuss a future
constitution. This did not satisfy the Congress leaders. The radicals within the Congress, led by
Jawaharlal Nehru and Subhas Chandra Bose, became more assertive.
The liberals and moderates, who were proposing a constitutional system within the framework of
British dominion, gradually lost their influence. In December 1929, under the presidency of
Jawaharlal Nehru, the Lahore Congress formalised the demand of Purna Swaraj or full independence
for India.
76.

Solution: d)

Parthana Samaj, a Hindu reform society was established in Bombay in the 1860s. In purpose it is
similar to, but not affiliated with, the more widespread Brahmo Samaj and had its greatest sphere of
influence in and around Indias Maharashtra state.
The aim of the society is the promulgation of theistic worship and social reform, and its early goals
were opposition to the caste system, the introduction of widow remarriage, the encouragement of
female education, and the abolition of child marriage.
You can read more at http://www.importantindia.com/11130/prarthana-samaj/

77. Solution: d)
http://www.newworldencyclopedia.org/entry/Partition_of_Bengal_(1905)#Partition
Partition sparked a major political crisis along religious lines. Hindu resistance exploded as the
Indian National Congress began the Swadeshi movement that included boycotting British goods and
http://www.insightsonindia.com INSIGHTS

Page 29


INSIGHTS MOCK TEST SERIES 2015: TEST 28 SOLUTIONS

public institutions, meetings and processions, forming committees, propaganda through press, and
diplomatic pressure. Moreover, revolutionary terrorism was also on the rise. The British needed to
control all this.
Because of the protests and terrorism, British had to finally annul the partition in 1911.
On the other hand, the Muslims in East Bengal hoped that a separate region would give them more
control over education and employment, hence, they opposed those movements.

78.

Solution: a)

It was the first Russian Revolution, in 1905, that fired up the imagination of Indian revolutionaries.
Mohandas Gandhi regarded it as the greatest event of the present century and a great lesson to us.
India was also switching to this Russian remedy against tyranny, Gandhi said.
The revolution made a big impact also on the minds of Indian revolutionaries who, unlike the
Moderates and the Extremists of the Congress party, intended to get absolute independence by
adopting revolutionary methods as practised by Russians.
The Indian Sociologist said in its December 1907 issue: Any agitation in India must be carried out
secretly and the only methods which can bring the English to their senses are the Russian methods
vigorously and incessantly applied until the English relax their tyranny and are driven out of the
country.
You can read more at http://in.rbth.com/blogs/2013/08/15/iday_russias_role_in_the_indian_freedom_movement_28461.html

79.

Solution: c)

Towards the end of 1921, peasant discontent surfaced again in the districts of Hardoi, Bahraich and
Sitapur (UP), with grievances relating to:

High rents- 50 per cent higher than the recorded rates;


Oppression of thikadars in charge of revenue collection; and
Practice of share-rents.

The meetings of the Eka or the Unity Movement involved a symbolic religious ritual in which the
assembled peasants vowed that they would

Pay only the recorded rent but would pay it on time;


Not leave when evicted;

http://www.insightsonindia.com INSIGHTS

Page 30


INSIGHTS MOCK TEST SERIES 2015: TEST 28 SOLUTIONS

Refuse to do forced labour;


Give no help to criminals;
Abide by panchayat decisions.

The grassroot leadership of the Eka Movement came from Madari Pasi and other low-caste leaders,
and many small zamindars.
Congress and Khilafat leaders provided the initial thrust to the peasant grievances and the movement
grew under the name Eka or unity movement. With grass-root leadership not in favour of nonviolence taking over the movement, the authorities succeeded in bringing it to an end. The Kisan
movements were also over shadowed by the Non-Cooperation Movement in UP. By March 1922,
severe repression by authorities brought the movement to an end.

80.

Solution: c)

The major factors of soil formation are the nature of the parent rock and climatic factors. Other
factors are the topography, role of organic material and time taken for the composition of soil
formation. All these differ from place to place.

81.Solution: b)
This question is straight from those corner boxes of NCERT. Here is how to identify several type of
rocks.
http://www.insightsonindia.com INSIGHTS

Page 31


INSIGHTS MOCK TEST SERIES 2015: TEST 28 SOLUTIONS

Bright blue to green: suspect a copper mineral. Dull greens are usually not copper greens, nor
are blues that have a violet cast.

Earth tones are almost always due to iron, either as a principal ingredient or as an impurity or
surface coating.

Bright yellow, orange or red: suspect one of the non-metallic sulfides, then one of the
transition metal radicals (chromate, vanadate, etc.). A few oxides are also brightly colored.
Some uranium minerals are bright yellow or yellow-green.

Pink: if hard, suspect potassium feldspar. The common manganese minerals rhodonite
(silicate) and rhodochrosite (carbonate) are also pink, and manganese can stain other minerals
pink as well. Some lithium silicates are also pink or lavender.

Black or dark green: if hard, suspect a ferromagnesian silicate

Pea green, especially in granite or a metamorphic rock: epidote. Almost always.

82.

Solution: a)

Kanin peninsula belongs to Europe. Guiana highlands are a part of South America.
The other two are important mineral zones in North America.
The mineral deposits in North America are located in three zones: the Canadian region north of the
Great Lakes, the Appalachian region and the mountain ranges of the west. Iron ore, nickel, gold,
uranium and copper are mined in the Canadian Shield Region, coal in the Appalachians region.
Western Cordilleras have vast deposits of copper, lead, zinc, gold and silver.

83.

Solution: a)

Millets are coarse grains and can be grown on less fertile and sandy soils. It is a hardy crop that needs
low rainfall and high to moderate temperature and adequate rainfall. Jowar, bajra and ragi are grown
in India. Other countries are Nigeria, China and Niger.
Maize requires moderate temperature, rainfall and lots of sunshine. It needs well-drained fertile
soils. Maize is grown in North America, Brazil, China, Russia, Canada, India, and Mexico.

84.

Solution: a)

http://www.insightsonindia.com INSIGHTS

Page 32


INSIGHTS MOCK TEST SERIES 2015: TEST 28 SOLUTIONS

Besides the longitudinal divisions, the Himalayas have been divided on the basis of regions from west
to east. These divisions have been demarcated by river valleys. For example, the part of Himalayas
lying between Indus and Satluj has been traditionally known as Punjab Himalaya but it is also known
regionally as Kashmir and Himachal Himalaya from west to east respectively. The part of the
Himalayas lying between Satluj and Kali rivers is known as Kumaon Himalayas. The Kali and Tista
rivers demarcate the Nepal Himalayas and the part lying between Tista and Dihang rivers is known as
Assam Himalayas.

85.

Solution: d)

Rainfall decreases from east to west in Northern plains because of the effect of the North-east
monsoon which decreases from east to west. Also, rainfall in the western India is blocked by Aravali
mountains.
In Coastal areas, both sea and land breeze regulate and moderate temperature throughout the day.
The diurnal ranges are also lesser.
On the contrary, seasonal contrasts are more in the interior due to continentality. Hot air from land
circulates increasing the temperature in surrounding areas as well.

86.

Solution: a)

The target is to provide nearly 20 million houses over seven years.


The components of the scheme are as follows:

Slum rehabilitation of Slum Dwellers with participation of private developers using land as a
resource;

Promotion of affordable housing for weaker section through credit linked subsidy;

Affordable housing in partnership with Public & Private sectors and

Subsidy for beneficiary-led individual house construction or enhancement.

87.

Solution: a)

Regulating the online sale of medicines has become a priority for Indias drug regulators. The
government is considering launching a pilot project to regulate their sale in a state equipped with
proper regulation mechanism.
http://www.insightsonindia.com INSIGHTS

Page 33


INSIGHTS MOCK TEST SERIES 2015: TEST 28 SOLUTIONS

Also, there are a number of issues associated with regulation. Read the article to understand them
http://www.downtoearth.org.in/content/regulate-online-sale-drugs-india-say-experts

88.

Solution: d)

Vaisheshika school is known for its insights in naturalism, and it is a form of atomism in natural
philosophy.
It postulated that all objects in the physical universe are reducible toparamu (atoms), and one's
experiences are derived from the interplay of substance (a function of atoms, their number and their
spatial arrangements), quality, activity, commonness, particularity and inherence.
The school proposed that atoms are indivisible and eternal, can neither be created nor destroyed, and
that each one possesses its own distinct viea (individuality). Knowledge and liberation was
achievable by complete understanding of the world of experience, according to Vaieika school of
Hinduism.

89.

Solution: a)

The rotation of the earth about its axis affects the direction of the wind. This force is called the
Coriolis force after the French physicist who described it in 1844. It deflects the wind to the right
direction in the northern hemisphere and to the left in the southern hemisphere.
The deflection is more when the wind velocity is high. The Coriolis force is directly proportional to the
angle of latitude. It is maximum at the poles and is absent at the equator.
The Coriolis force acts perpendicular to the pressure gradient force. The pressure gradient force is
perpendicular to an isobar. The higher the pressure gradient force, the more is the velocity of the wind
and the larger is the deflection in the direction of wind.

90.

Solution: d)

Domestic borrowers have little connection with Rupees exchange rate.


Exporters will benefit because a depreciated currency will make Indian products cheaper and thus
more competitive in International market.
http://www.insightsonindia.com INSIGHTS

Page 34


INSIGHTS MOCK TEST SERIES 2015: TEST 28 SOLUTIONS

FIIs will suffer. Consider if they invested $20 in Indian shares which amounted to Rs. 1000 of
investment at Rs. 50 per dollar exchange rate. Now, suppose if their stock value goes up to Rs. 1200.
They should ideally get $24 in return if the rupee-dollar exchange rate remains constant.
However, if rupee falls to 60 per dollar, they are again getting back $20 on $20 invested.

91. Solution: d)
India's dependence on the Gulf nations is 61 per cent. This is definitely not good if crisis events start
unfolding in gulf nations as they do.
Moreover, diversifying will not allow any exporting nation to act as a monopoly with India. Hence
India can bargain better and harder with nations.
Read these two articles for a clearer understanding of Indias oil import strategy.
http://www.livemint.com/Politics/dtqX201kKrlYSwlaNuuH5N/India-plans-to-diversify-oilimports.html
http://articles.economictimes.indiatimes.com/2014-07-23/news/51932083_1_crude-oil-imports-oilminister-dharmendra-pradhan-gas-production

92.

Solution: a)

http://www.insightsonindia.com INSIGHTS

Page 35


INSIGHTS MOCK TEST SERIES 2015: TEST 28 SOLUTIONS

93.

Solution: d)

If interest rates are high, the returns on money investments in banks are high for depositors and
investors. So, instead of buying more government bonds they would prefer having a large deposit.
When money supply in the economy reduces, central bank needs to pump more money by buying
bonds from the market.
Selling bonds would only reduce even the present money supply in the market.

94.

Solution: d)

In the context of temple design, Shilpa Shastras were manuals for sculpture and Hindu iconography,
prescribing among other things, the proportions of a sculptured figure, composition, principles,
meaning, as well as rules of architecture.
Sixty-four techniques for such arts or crafts, sometimes called bhya-kal "external or practical arts",
are traditionally enumerated, including carpentry, architecture, jewellery, farriery, acting, dancing,
music, medicine, poetry etc., besides sixty-fourabhyantara-kal or "secret arts"' which include mostly
"erotic arts" such as kissing, embracing, etc. (Monier-Williams s.v. ilpa).
While Shilpa and Vaastu Shastras are related, Shilpa Shastras deal with arts and crafts such as
forming statues, icons, stone murals, painting, carpentry, pottery, jewelry, dying, textiles and
others. Vastu Shastras deal with building architecture - building houses, forts, temples, apartments,
village and town layout, etc.
http://www.insightsonindia.com INSIGHTS

Page 36


INSIGHTS MOCK TEST SERIES 2015: TEST 28 SOLUTIONS

95.

Solution: b)

Takeout financing is an accepted international practice of releasing long-term funds


for financing infrastructure projects. It can be used to effectively address Asset-Liability mismatch of
commercial banks arising out of financing infrastructure projects and also to free up capital for
financing new projects.
http://www.business-standard.com/article/finance/rbi-relaxes-takeout-financing-norms-forexisting-infra-loans-114080701324_1.html

96.

Solution: a)

It is an unconventional monetary policy in which a central bank purchases government securities or


other securities from the market in order to lower interest rates and increase the money supply.
Quantitative easing increases the money supply by flooding financial institutions with capital in an
effort to promote increased lending and liquidity. Quantitative easing is considered when short-term
interest rates are at or approaching zero, and does not involve the printing of new banknotes.
Read this article to understand this concept comprehensively.
http://www.economist.com/blogs/economist-explains/2015/03/economist-explains-5

97.

Solution: a)

It is not that countries which are endowed with a bounty of natural wealth minerals or forests or the
most fertile lands are naturally the richest countries. In fact the resource rich Africa and Latin
America have some of the poorest countries in the world, whereas many prosperous countries have
scarcely any natural wealth.
There was a time when possession of natural resources was the most important consideration but
even then the resource had to be transformed through a production process.
The economic wealth, or well-being, of a country thus does not necessarily depend on the mere
possession of resources; the point is how these resources are used in generating a flow of production
and how, as a consequence, income and wealth are generated from that process.

98.

Solution: a)

http://www.insightsonindia.com INSIGHTS

Page 37


INSIGHTS MOCK TEST SERIES 2015: TEST 28 SOLUTIONS

Of the final goods, we can distinguish between consumption goods and capital goods. Goods like food
and clothing, and services like recreation that are consumed when purchased by their ultimate
consumers are called consumption goods or consumer goods. (This also includes services which are
consumed but for convenience we may refer to them as consumer goods.)
Then there are other goods that are of durable character which are used in the production process.
These are tools, implements and machines. While they make production of other commodities
feasible, they themselves dont get transformed in the production process.
They are also final goods yet they are not final goods to be ultimately consumed. Unlike the final
goods that we have considered above, they are the crucial backbone of any production process, in
aiding and enabling the production to take place. These goods form a part of capital, one of the crucial
factors of production in which a productive enterprise has invested, and they continue to enable the
production process to go on for continuous cycles of production.

99.

Solution: a)

It is essentially wage, interest, rent, and profit payment for the services of scarce resources, or the
factors of production (labour, capital, land, and entrepreneurship), in return for productive services.
Factor payments are frequently categorized according to the services of the productive resource.
Wages are paid for the services of labour, interest is the payment for the services of capital, rent is the
services for land, and profit is the factor payment to entrepreneurship.
In the circular flow of an economy, these are payments made by the business sector for factor services
purchased from the household sector through the financial markets.

100. Solution: b)
The Directive Principles are like the instrument of instructions, which were issued to the GovernorGeneral and to the Governors of the colonies of India by the British Government under the
Government of India Act of 1935. What is called Directive Principles is merely another name for the
instrument of instructions. The only difference now is that they are instructions to the legislature and
the executive.
The phrase Directive Principles of State Policy denotes the ideals that the State should keep in mind
while formulating policies and enacting laws.

http://www.insightsonindia.com INSIGHTS

Page 38


INSIGHTS MOCK TEST SERIES 2015: TEST 28 SOLUTIONS

http://www.insightsonindia.com INSIGHTS

Page 39

INSIGHTS ON INDIA MOCK PRELIMINARY EXAM - 2015


INSIGHTS ON INDIA MOCK TEST - 29
GENERAL STUDIES

PAPER-I
Time Allowed: 2 Hours

Maximum Marks: 200

INSTRUCTIONS
1. IMMEDITELY AFTER THE COMMENCEMENT OF THE EXAMINATION, YOU SHOULD
CHECK THAT THIS TEST BOOKLET DOES NOT HAVE ANY UNPRINTED OR TORN OR MISSING
PAGES OR ITEMS, ETC. IF SO, GET IT REPLACED BY A COMPLETE TEST BOOKLET.
2. You have to enter your Roll Number on the Test
Booklet in the Box provided alongside. DO NOT
Write anything else on the Test Booklet.
4. This Test Booklet contains 100 items (questions). Each item is printed only in English. Each item
comprises four responses (answers). You will select the response which you want to mark on the
Answer Sheet. In case you feel that there is more than one correct response, mark the response which
you consider the best. In any case, choose ONLY ONE response for each item.
5. You have to mark all your responses ONLY on the separate Answer Sheet provided. See directions in
the Answer Sheet.
6. All items carry equal marks.
7. Before you proceed to mark in the Answer Sheet the response to various items in the Test Booklet, you
have to fill in some particulars in the Answer Sheet as per instructions sent to you with your Admission
Certificate.
8. After you have completed filling in all your responses on the Answer Sheet and the examination has
concluded, you should hand over to the Invigilator only the Answer Sheet. You are permitted to take
away with you the Test Booklet.
9. Sheets for rough work are appended in the Test Booklet at the end.
10. Penalty for wrong answers :
THERE WILL BE PENALTY FOR WRONG ANSWERS MARKED BY A CANDIDATE IN THE
OBJECTIVE TYPE QUESTION PAPERS.
(i) There are four alternatives for the answer to every question. For each question for which a
wrong answer has been given by the candidate, one-third of the marks assigned to that
question will be deducted as penalty.
(ii) If a candidate gives more than one answer, it will be treated as a wrong answer even if one of
the given answers happens to be correct and there will be same penalty as above to that
question.
(iii) If a question is left blank, i.e., no answer is given by the candidate, there will be no penalty for
that question.
http://www.insightsonindia.com
INSIGHTS ON INDIA MOCK TEST SERIES FOR CIVIL SERVICES PRELIMINARY EXAM 2015

http://www.insightsonindia.com INSIGHTS

Page 1

1. RBI has recently raised concerns about


Algorithmic trading or High frequency
trading in stock markets disturbing
their stability. What is algorithmic
trading?
a) using a computer program to trade
in the stock market based on a
certain set of instructions
b) using random electronic equity
purchases in the stock market to
make them more volatile
c) done by a big cartel of the stock
market that buy or sell stocks based
on previously agreed set of
instructions
d) purchasing and selling stocks at a
very high frequency

2. The Supreme Court has recently given a


ruling against using Mediation in rape
cases. Consider the following about
mediation an alternative dispute
resolution mechanism.
1. Consent of all parties is required to
start mediation.
2. Decision arrived after mediation
can be enforced in a court of law.
3. Only a court of law can appoint the
mediator.
Choose the correct answer using the codes
below.
a)
b)
c)
d)

1 and 2 only
2 and 3 only
1 and 3 only
All of the above

http://www.insightsonindia.com INSIGHTS

3. The BCIM corridor is one of the major


joint economic initiatives of India and
China. The BCIM corridor will pass
through
1. Kunming
2. Mandalay
3. Xinjiang
4. Dhaka
5. Kolkata
Choose the correct answer using the codes
below.
a)
b)
c)
d)

1, 2, 4 and 5 only
4 and 5 only
1, 2, 3 and 5 only
1, 2, 3 and 4 only

4. The issue of IIM autonomy has been in


news lately. Consider the following
about the governance of IIMs presently.
1. It has been established by executive
order.
2. The MHRD has no say in the
appointments of Directors of the
IIMs.
3. No changes in the educational
curriculum of IIMs can be made
without approval
from the
government.
Choose the correct answer using the codes
below.
a)
b)
c)
d)

1 and 2 only
2 and 3 only
1 only
1 and 3 only
Page 2

5. Which of the following benefits are


being given to Pradhan Mantri Awas
Yojana.
1. Grant for each house constructed
under the scheme
2. Subsidy on the interest rates for
housing loans
3. Scheme is applicable to lower income
groups only
4. Transgender and widows will be
given preference in the allocation of
benefits.
Choose the correct answer using the codes
below.
a)
b)
c)
d)

1, 2 and 3 only
1 and 3 only
2 and 4 only
All of the above

6. What is meant by Competitive


monetary easing policy?
a) Nations competing to attract
investment by lowering interest
rates in the economy
b) Nations competing to liberalize
banking rules in order to attract
greater investment in the financial
sector
c) Competition between states of the
same federation to increase money
supply in the economy
d) Nations competing to attract
investment by giving grater sops to
investors

http://www.insightsonindia.com INSIGHTS

7. Premiers/PMs/Presidents from which of


these countries have visited India in the
past one year?
1. USA
2. Japan
3. China
4. Russia
5. France
Choose the correct answer using the codes
below.
a)
b)
c)
d)

1, 2, 4 and 5 only
2, 3 and 4 only
1 and 3 only
All of the above

8. Mobile number portability will be


available nationally now. How would the
consumers benefit from this move by
TRAI and the government?
1. Consumers can keep their existing
mobile phone numbers while shifting
between States and circles.
2. Consumers can keep existing mobile
phone numbers while changing
between operators.
3. Consumers can keep existing mobile
phone numbers while changing
between operators, states and
circles.
Which of the above is/are true?
a)
b)
c)
d)

1 and 2 only
2 and 3 only
1 and 3 only
All of the above

Page 3

9. In a court proceeding, DNA tests are


allowed to be used as evidence in which
of the following cases?
1. Minor Civil maintenance cases
2. Paternity and maternity disputes
3. Certain criminal cases
Choose the correct answer using the codes
below.
a)
b)
c)
d)

1 and 2 only
1 and 3 only
2 and 3 only
All of the above

10. Green climate fund would be made


available to which of the following
bodies/ organizations/ agencies in India
for mitigating climate change?
1. Governmental development agencies
like NABARD
2. Rural development centred NGOs
3. Private organizations with a high
investment in corporate social
responsibility
4. Grass root agencies affiliated with
local government
Choose the correct answer using the codes
below.
a)
b)
c)
d)

11. Mercury poisoning in humans can


happen in which of the following ways?
1. Consumption of fish
2. Breathing contaminated air
3. Improper disposal of fluorescent
lamps
4. Emission from coal based power
plants
Choose the correct answer using the codes
below.
a)
b)
c)
d)

1 and 2 only
3 and 4 only
2, 3 and 4 only
All of the above

12. To generate awareness about a digital


India across India, the Digital India
week was to be organized at
1. Gram Panchayats
2. Post Offices
3. Community Service Centres
4. Schools
Choose the correct answer using the codes
below.
a)
b)
c)
d)

3 and 4 only
1 and 2 only
1, 2 and 3 only
All of the above

1 and 4 only
1 and 3 only
2 and 4 only
1 only

http://www.insightsonindia.com INSIGHTS

Page 4

13. The Russian city of Ufa was recently in


news because
a) a new Palaeolithic age monument
has been found that shed greater
light on human evolution
b) abundant reserves of shale gas
have been discovered that are very
near to the surface
c) the Shanghai Corporation
Organization (SCO) meeting
where India is attending as a
member
d) the SCO and BRICS summits were
recently hosted here

14. The term troika has been widely used


in the negotiations concerning the
Greece crisis. The troika consists of
1. International Monetary Fund
2. World Bank
3. European Union
4. European Commission
5. European Central Bank
Choose the correct answer using the codes
below.
a)
b)
c)
d)

1, 2 and 3
2, 3 and 5
2, 4 and 5
1, 4 and 5

15. How does the newly established Asian


Infrastructure Investment Bank (AIIB)
differ from the IMF and World Bank
(WB)?
1. No nation holds veto power in the
AIIB, unlike in IMF and WB.

http://www.insightsonindia.com INSIGHTS

2. All member nations will have equal


decision-making power in AIIB,
whereas in IMF and WB its rests on
their economic size and global
economic contributions.
3. Membership of the AIIB is open only
to Asian nations, unlike in IMF and
WB.
Choose the correct answer using the codes
below.
a)
b)
c)
d)

1 and 2 only
1 and 3 only
2 only
None of the above

16. In a recent World Bank report, the


context of poverty has been said to
deplete a persons thinking bandwidth
to make rational decisions. This finding
is rooted in
a) Anthropological studies
b) Genetic studies
c) Behavioural economics
d) Psychological economics

17. Consider the following about the Board


of Control for Cricket in India (BCCI).
1. It is an autonomous society not
controlled by the Government of
India.
2. It is subordinate to International
Cricket Council.
3. It comes under the purview of the
RTI Act.
Choose the correct answer using the codes
below.
Page 5

a)
b)
c)
d)

1 and 2 only
1 and 3 only
2 and 3 only
1 only

18. Consider the following about the


institution of Lokayukta.
1. A state is not legally obliged to create
the institution of Lokayukta.
2. Powers of Lokayukta vary from state
to state.
3. Only a serving or retired Supreme
Court judge can be appointed as a
Lokayukta.
4. A Lokayukta can also investigate
cases pertaining to Central civil
servants.
Choose the correct answer using the codes
below.
a)
b)
c)
d)

1 and 4 only
2 and 3 only
2 only
1, 2 and 3 only

19. Which of the following correctly


understate the difference(s) between the
Indian and American (USA) Supreme
Court?
1. While the Indian SCs jurisdiction
extends to constitutional, civil and
criminal cases; the American SCs
jurisdiction is confined to
constitutional cases only.
2. The American SC does not defend
the fundamental rights of citizens,
unlike the
Indian SC.

http://www.insightsonindia.com INSIGHTS

3. The American SC does not have any


powers of judicial review, unlike with
the
Indian SC.
Choose the correct answer using the codes
below.
a)
b)
c)
d)

1 and 2 only
1 and 3 only
2 and 3 only
1 only

20. The Saraswati River is often called as


antarvahini or gupt. Why?
a) The river does not flow any longer
in India.
b) The river flows underground
invisible to human eyes and does
not come to surface at all.
c) The river surfaces only at one
location and flows underground in
the rest of the territory.
d) The river has been lost since
ancient times and not recovered till
date.

21. Consider the following about Atal


Mission for Rejuvenation and Urban
Transformation (AMRUT) mission.
1. The mission supports cities to
become smart cities.
2. Centre will allocate finances to states
based on a central plan to which
every state should confirm.
3. Urban local bodies implementing the
mission will get funding directly
from the Centre.

Page 6

2. It will allow students movement


from one degree programme to
another.
3. Students can take courses of their
choice, and even acquire more
than the
required credits.
4. Students can choose
interdisciplinary courses.

4. The mission has been merged with


JNNURM, a previous mission for
urban cities.
Choose the correct answer using the codes
below.
a)
b)
c)
d)

1 and 4 only
1 and 3 only
1 only
All of the above

Choose the correct answer using the codes


below.
a)
b)
c)
d)

22. Consider the following statements:


Assertion (A): A national emergency converts
the federal structure into a unitary one without
a formal amendment of the Constitution.
Reason (R): During an Emergency, the Central
government becomes all powerful and the
states go into the total control of the Centre.
In the context of the statements above, which
of these is true?
a) A and R both are true, and R is the
correct explanation for A.
b) A and R both are true, and R is the
NOT the correct explanation for A.
c) A is correct, R is incorrect.
d) A and R both are incorrect.

23. UGC has recently issued guidelines to


implement the choice based credit
system across the country. What
benefits will a student derive from it?
1. It will allow students to transfer
credits earned in one institution
to another.

http://www.insightsonindia.com INSIGHTS

1 and 2 only
1 and 3 only
2 and 4 only
All of the above

24. Consider the following about Centrally


sponsored schemes and Central sector
schemes?
1. Both are formulated by Central
government.
2. Centrally sponsored schemes are
framed from Union list, whereas
Central sector schemes from state
list.
3. Both are implemented by state
governments.
4. Centre provides funding to the states
only in the case of Central sector
schemes.
Choose the correct answer using the codes
below.
a)
b)
c)
d)

1 and 2 only
1 and 3 only
2 and 4 only
All of the above

Page 7

25. The Credit Information Companies


(Regulation) Act, 2005 (Cicra) deals
with which of the following?
1. Banks that default on their loans
2. Companies that analyze credit
history of individuals and companies
3. Foreign institutional investors that
provide credit to Indian companies

b) scholarships to meritorious
students in pursuing higher
education
c) compulsory leadership
development courses in all higher
educational universities
d) growth opportunities for children
from BPL families by way of
Central assistance and mentor
support

Choose the correct answer using the codes


below.
a)
b)
c)
d)

1 and 2 only
1 and 3 only
2 and 3 only
2 only

26. High lead exposure in human beings has


been associated with
1. Learning disabilities
2. Breakdown of nervous system
3. Attention deficit disorder
4. Fertility problems in both men and
women
Choose the correct answer using the codes
below.
a)
b)
c)
d)

1, 2 and 3 only
1 and 4 only
2 and 3 only
All of the above

27. The scheme called Swayam provides for


a) a mass-scale online open
educational course platform

http://www.insightsonindia.com INSIGHTS

28. The government of Greece will be


conducting a referendum concerning the
Greece crisis. What would people of
Greece vote for?
a) Whether to be in the Eurozone or
not
b) Whether to accept the harsh
austerity conditions imposed on
Greece or not
c) Whether to revert back to Greeces
old currency drachma or not
d) All of the above

29. The Union government has come up


with a special scheme called Udaan for
girl students. Consider the following
about it.
1. It is a mentoring and scholarship
scheme.
2. It is aimed at enabling meritorious
girl students to transit from
schools to technical education.
3. The scheme also provides for the
use of free online resources to
enhance teaching
and learning
in senior secondary schools.

Page 8

Choose the correct answer using the codes


below.
a)
b)
c)
d)

1 and 2 only
1 and 3 only
2 and 3 only
All of the above

30. Situated in Antarctica, the Ross Sea is


often called as the last ocean of the
World. Why?
a) It is in a pristine condition
b) It harbours the marine biodiversity that is the last to form on
earth.
c) It is the oldest undivided ocean on
earth which was not lost after its
formation and exists till date.
d) As per Scientific predictions, it
was formed out of the first
molecules of vapour formed on
earth.

31. The U.N. has described Sabar


Shouchagar (Toilets for All) as a unique
model for improved sanitation because
1. The model achieved the first
defecation free district in India.
2. The model involved the community
and faith based organization on
board.
3. The model relied completely on
private support and donations.
Choose the correct answer using the codes
below.
a) 1 and 2 only
b) 1 and 3 only
c) 2 and 3 only
http://www.insightsonindia.com INSIGHTS

d) All of the above

32. Some time ago, the RBI withdrew pre2005 notes from circulation. This was
done because
1. Pre-2005 notes had lesser security
features.
2. Fake currency from circulation
could be removed this way.
3. Black money could be flushed out
of the economy.
4. It would reduce cash hoarding.
Choose the correct answer using the codes
below.
a)
b)
c)
d)

1, 2 and 3 only
1 and 4 only
2, 3 and 4 only
All of the above

33. In South Asia, arrange the following


countries in decreasing order of the
Foreign Direct Investment (FDI) that
they receive.
1. India
2. Iran
3. Bangladesh
4. Pakistan
5. Sri Lanka
Choose the correct order from the codes below.
a)
b)
c)
d)

12345
14325
51234
12435

Page 9

34. Consider the following about Essential


Services Maintenance Act (ESMA).
1. It is a central law.
2. Each state has a separate ESMA.
3. Both Central and State
governments can enforce ESMA.
4. The government cannot order
private operators to restore
essential services.
Choose the correct answer using the codes
below.
a)
b)
c)
d)

1, 2 and 3 only
1 and 4 only
2, 3 and 4 only
1 and 2 only

35. The scheme GIAN Global Initiative for


Academic Network is concerned with
a) forming a network of highly
acclaimed Indian researchers
working abroad
b) intensive collaboration between
the best Indian and foreign
universities for research and
academic exchange
c) bringing world class educators
from across the globe to teach in
India
d) None of the above

36. A European mission Rosetta was


recently in news. Rosetta is
1. The first spacecraft that landed
on a comet
2. The first spacecraft to orbit a
comet

http://www.insightsonindia.com INSIGHTS

3. The first human mission to have


transmitted information from an
asteroid orbiting
the Sun
Choose the correct answer using the codes
below.
a)
b)
c)
d)

1 and 2 only
1 and 3 only
2 and 3 only
All of the above

37. Jayprakash Narayan was associated with


which of the following movements in
India?
1. Nav Nirman Andolan, Gujarat
2. Bihar Movement
3. Non-cooperation Movement
4. Civil Disobedience Movement
5. Champaran Movement
Choose the correct answer using the codes
below.
a)
b)
c)
d)

1 and 5 only
3 and 4 only
1, 2, 3 and 4 only
All of the above

38. The Nabakalebar festival was in news


recently because
a) It is to be promoted through
Incredible India campaign
b) The festival attracted largest
number of devotees from all over
India
c) It is being observed after a period
of almost two decades in Odisha
d) Both (a) and (c)
Page 10

39. Consider the following about Pradhan


Mantri Krishi Sinchayee Yojana
(PMKSY) or PMs Agriculture Irrigation
Scheme.
1. It aims at providing irrigation
facilities to every village in the
country.
2. The scheme will be directly
implemented by the Centre.
3. The scheme will de-incentivize
groundwater irrigation through
financial incentives.
Choose the correct answer using the codes
below.
a)
b)
c)
d)

1 and 2 only
1 and 3 only
2 and 3 only
1 only

40. Consider the following about Euro zone


and European Union
1. Euro zone is an Economic entity,
and EU is a Politico-economic
entity.
2. Euro zone has a common currency,
EU doesnt.
3. Euro zone follows a common
monetary policy, EU doesnt.
4. Both the EU and Euro zone have a
single market.
5. EU has a supra-national
Parliament, Euro zone doesnt.
Choose the correct answer using the codes
below.

c) 1, 2, 3 and 4 only
d) All of the above

41. The 44th constitutional amendment


brought some changes in the powers of
PM in declaring emergency. What was
the change brought?
a) A national emergency can be
declared by the President only on
the concurrence of the cabinet and
not merely on the advice of the
prime minister
b) The President can declare national
emergency only after the
concurrence of the cabinet and
Parliament
c) If a house was not in session, only
then the President could declare
national emergency on the advice
of the Prime Minister
d) The PM could only advice
declaring internal emergency to
the President; for external
emergency the concurrence of the
Parliament was needed

42. World Bank has recently approved an


additional loan of 650 million dollars for
Indias Eastern Dedicated Freight
Corridor (DFC) Project. It passes
through the states of
1. UP
2. Punjab
3. Haryana
4. Bihar
5. Jharkhand
6. West Bengal

a) 1, 2 and 5 only
b) 3 and 4 only
http://www.insightsonindia.com INSIGHTS

Page 11

Choose the correct answer using the codes


below.
a)
b)
c)
d)

1, 2, 4 and 6 only
1, 4, 5 and 6 only
5 and 6 only
All of the above

43. Union Cabinet has recently approved


setting up of an Online National
Agriculture Market. How will this
benefit farmers in the country?
1. Farmers will be able to sell their
produce in mandis as well as
online.
2. Farmers can buy agricultural
produce online.
3. Creation of godowns and facilitate
transportation of the farm produce
after the
online trade
Choose the correct answer using the codes
below.
a)
b)
c)
d)

1 and 2 only
1 and 3 only
2 and 3 only
All of the above

44. A campaign called `Pade Bharat, Badhe


Bharat has been launched by the
Government of India to ensure better
learning levels
a) in primary education
b) in secondary and adult education
c) in adult primary education with
special focus on women
d) in higher secondary school with
special focus on adolescent girl
students
http://www.insightsonindia.com INSIGHTS

45. Consider the following statements.


1. Insulting the Indian National Flag
entails disqualification for
contesting elections to
Parliament and State
Legislatures.
2. A candidate cannot stand for
election from more than one
Parliamentary constituency.
3. All cases of disqualification of MPs
are dealt by the speaker.
Choose the correct answer using the codes
below.
a)
b)
c)
d)

1 and 2 only
1 and 3 only
2 and 3 only
1 only

46. A member of the House in Parliament


can be asked by the speaker to stop
speaking and let another member speak.
This is known as
a) crossing the floor by the MP
b) maintaining the decorum by the
speaker
c) yielding the floor
d) speech adjournment

47. You are a student in a local university


and have filed a nomination paper for
contesting Panchayat elections. The
State Election Commission would deem
your nomination valid only if
1. You have obtained an approval
letter from your university
2. You have a voter ID card
Page 12

3. You have already held some


leadership position in the
college/university.
Choose the correct answer using the codes
below.
a)
b)
c)
d)

1 and 2 only
1 and 3 only
2 and 3 only
2 only

48. Which of the following statements


regarding the advisory jurisdiction of
the Supreme Court are correct?
1. It is binding on the Supreme Court
to give advice to the President
every time it is
referred to.
2. The opinion tendered by the
Supreme Court is binding on the
President.

Choose the correct answer using the codes


below.
a)
b)
c)
d)

1 and 2 only
1 and 3 only
3 only
3 and 4 only

50. The ban on pre-censorship of


newspapers by the country in the
country is based on
a) special provisions made under
Article 19 of the constitution
b) the rule of law in the constitution
of India
c) explicit freedom given to media or
any peoples forum in the
constitution
d) a ruling of Supreme Court which
interpreted Article 19 of the
constitution in a wider sense

Choose the correct answer using the codes


below.
a)
b)
c)
d)

1 only
2 only
Both 1 and 2
None

49. Which of the following come under the


jurisdiction of both the Supreme Court
and High courts?
1. Disputes between the Centre and
the States
2. Disputes between the States
3. Issuing writs to enforce
fundamental rights
4. Judicial review of administrative
acts
http://www.insightsonindia.com INSIGHTS

51. Who among the following is/are


appointed by the Vice-President of
India?
1. Vice-chairperson, NITI Ayog
2. Chief Minister of a Union Territory
3. Members, Finance Commission
Choose the correct answer using the codes
below.
a)
b)
c)
d)

1 and 2 only
1 and 3 only
2 only
None of the above

Page 13

52. In the Kesavananda Bharati case, the


Supreme Court came out with the basic
structure doctrine. What are the
implications of this doctrine for the
constitution and the powers of the
Parliament?
1. Not every provision of the
constitution can be amended by the
Parliament.
2. No constitutional body established
immediately after the
commencement of the
constitution can be scrapped.
3. The powers of Parliament and
courts can neither be expanded nor
contracted.
Choose the correct answer using the codes
below.
a)
b)
c)
d)

1 only
1 and 3 only
2 and 3 only
All of the above

53. Consider the following with reference to


election outcomes in India.
1. Majority of votes by a political
party necessarily mean majority of
seats.
2. Majority of seats garnered by a
political party necessarily mean
majority of votes.
Which of the above is/are true?
a)
b)
c)
d)

1 only
2 only
Both 1 and 2
None

http://www.insightsonindia.com INSIGHTS

54. Who among the following can vote in


the elections to Lok Sabha, Rajya Sabha
as well as State Legislative Council?
a) Registered voters of the country
b) Elected members of State
Legislative Assembly
c) President and Attorney-general
d) Chief Minister of a state

55. Which of the following constitutional


offices cannot be held by the same
individual for successive terms or more
than one term?
1. Office of the President of India
2. Office of the Comptroller and
Auditor General of India
3. Office of the Chairman, Rajya
Sabha
4. Office of the Governor
5. Office of the Chief Election
Commissioner
Choose the correct answer using the codes
below.
a)
b)
c)
d)

1 and 2 only
1, 3 and 5 only
2 and 5 only
2, 3, 4 and 5 only

56. Consider the following statements.


1. The British sovereign appointed
the last Governor General of free
India.
2. The constituent assembly was
composed on a British given
formula.
3. India joining commonwealth
nations after independence meant
Page 14

that India
continued to be a
dominion state of the British.
Choose the correct answer using the codes
below.
a)
b)
c)
d)

1 and 2 only
1and 3 only
2 and 3 only
All of the above

57. Consider the following about Elections


in India.
1. The recommendation for elections
is made by the Government of
India.
2. The announcement of elections is
done by the Election Commission
of India.
3. The model code of conduct is
enforced by the Government of
India.
4. The election conducting machinery
is not controlled by the
government.
Choose the correct answer using the codes
below.
a)
b)
c)
d)

1 and 2 only
1, 3 and 4 only
2 only
1, 2 and 4 only

58. If the Central bank, RBI, reduces the


Cash Reserve Ratio (CRR) as a policy
measure, it means
a) it is trying to counter the growing
fiscal deficit
http://www.insightsonindia.com INSIGHTS

b) it is trying to bring down inflation


in the country
c) it is expanding the money supply
in the economy
d) it is trying to push up the interest
rates in the economy

59. What does the term money multiplier


in banking terminology connote?
a) money injected in the economy
increases with time
b) money injected in the economy
helps in credit creation more than
the amount of money
c) money becomes more and more
liquid with time as it changes
hands in the long-run
d) money coming from foreign
sources has a greater multiplier
effect on the economy than
domestic sources

60. What is known as the Gandhian way of


economic planning or Gandhian
economic philosophy relies on
1. Self-sufficient villages
2. Labour intensive activities
3. Swaraj or Self-rule
Choose the correct answer using the codes
below.
a)
b)
c)
d)

1 and 2 only
1 and 3 only
2 and 3 only
All of the above

Page 15

61. A firm in the economy faces diminishing


marginal product of labour as it tries to
increase output in the short-run. What
does it mean?
a) With fixed set of machines,
employing any additional labour
in the firm is unproductive.
b) The productivity of labour
decreases due to additional
workload
c) The output produced by each
additional worker reduces as the
output is expanded with fixed set
of factors of production
d) None of the above

62. Recently some International firms have


been found to breach anti-trust laws
existing in several countries. What is the
equivalent of an anti-trust law in India?
a) Foreign Exchange Management
Act
b) Foreign Contribution Regulation
Act
c) Competition Act
d) Companies Act

63. Which of the following correctly


understates the difference between a
Consumer Price Index (CPI) and
Wholesale Price Index (WPI)?
1. CPI does not includes all the goods
that WPI includes
2. WPI does not include inflation in
the services sector, CPI does.

http://www.insightsonindia.com INSIGHTS

3. WPI is calculated by RBI, whereas


CPI is calculated by Department of
Statistics.
Choose the correct answer using the codes
below.
a)
b)
c)
d)

1 and 2 only
1 and 3 only
2 and 3 only
2 only

64. Gross Domestic Product of India will


NOT increase in the short-run if
1. the government continues to have
a high fiscal deficit
2. there is inflation in the economy
3. the monetary policy is loose
4. the government liberalises the
external sector
Choose the correct answer using the codes
below.
a)
b)
c)
d)

1 and 2 only
2 and 4 only
All of the above
None of the above

65. The Vedas in India were used for which


of the following purposes?
1. In religious rituals and sacrifices
2. For seeking answers to mythical
questions
3. For deciding appropriate
governance structures in local
kingdoms
Choose the correct answer using the codes
below.
Page 16

a)
b)
c)
d)

1 and 2 only
2 and 3 only
1 and 3 only
All of the above

66. Bhakti trend during the medieval ages


did NOT believe in
1. Worship of humanity
2. Caste system
3. Conceptual erudition
4. Ascetic approach to poetry
Choose the correct answer using the codes
below.
a)
b)
c)
d)

1 and 2 only
2 and 4 only
3 only
2 and 3 only

67. Suppose you have a barometer and you


are moving in a vechile to a nearby
region. You notice that the reading of
the barometer is going down. What all it
can possible indicate?
1. Storm
2. Rainfall
3. Cyclone
4. Increasing temperature
Choose the correct answer using the codes
below.
a)
b)
c)
d)

1 and 2 only
2 and 4 only
2 and 3 only
1 and 4 only

http://www.insightsonindia.com INSIGHTS

68. Which of the following phenomenon can


lead to thunderstorms?
1. When warm moist air collides with
cooler air
2. Heavy orographic rain
3. Rapid upwards movement of
warm, moist air
Choose the correct answer using the codes
below.
a)
b)
c)
d)

1 and 2 only
2 and 3 only
1 and 3 only
All of the above

69. The phenomenon of mirage occurs when


1. when the ground is very hot and
the air is cool
2. when the ground and air both are
very hot
3. When there is refraction of
sunlight
4. When there is presence of a water
body near the mirage
Choose the correct answer using the codes
below.
a)
b)
c)
d)

1 and 3 only
2 and 3 only
1, 3 and 4 only
2 and 4 only

Page 17

70. Consider the following about sunspots.


1. They are temporary dark spots
appearing on the Sun.
2. They correspond to regions of
strong magnetic field intensity.
3. Sunspots are cooler than the
surrounding regions of the Sun.
4. Sunspots can result in change in
weather and climate on earth.

1.
2.
3.
4.

Mimansa
Gnana
Samkhya
Carvaka

Choose the correct answer using the codes


below.
a)
b)
c)
d)

1 only
2 and 3 only
1 and 4 only
3 only

Choose the correct answer using the codes


below.
a)
b)
c)
d)

1 and 3 only
2 and 3 only
2 and 4 only
All of the above

71. The antigens in the human body serve


which of the following purposes?
1. They trigger the immune system
in the body.
2. They act as anti-toxicants in case
you ingest a poison.
3. They kill free radicals in the body
keeping you healthy.
Choose the correct answer using the codes
below.
a)
b)
c)
d)

1 and 3 only
2 and 3 only
1 and 2 only
1 only

72. Karma as a means of liberation has


been depicted in which of these
philosophies?
http://www.insightsonindia.com INSIGHTS

73. Consider the following about Anuvrata


movement.
1. The movement was started by
Buddhist monks after Buddha left
his body.
2. The movement emphasized on
idol worship.
3. The movement propagated the
practice of dharma.
Choose the correct answer using the codes
below.
a)
b)
c)
d)

1 only
2 and 3 only
1 and 2 only
3 only

74. Consider the following about Gamma


and X-rays.
1. Both are electromagnetic
radiation.
2. They differ only in the amount of
energy they carry.
3. While X-ray can penetrate matter,
Gamma ray cannot.
Page 18

4. Radioactive materials that emit


gamma radiation and X-rays
constitute both an external and
internal hazard to humans.
Choose the correct answer using the codes
below.
a)
b)
c)
d)

1 only
2 and 4 only
1, 2 and 4 only
1 and 3 only

75. Consider the following about the use of


musical instruments in Indian music.
1. Sitar is the main instrument of
Carnatic music.
2. Veena is used in Hindustani
music.
3. Flutes are used both in Carnatic
and Hindustani music.
Choose the correct answer using the codes
below.
a)
b)
c)
d)

1 and 2 only
2 and 3 only
3 only
None of the above

76. Which of the following were the


consequences of the Non-cooperation
movement?
1. The British were forced to
negotiate on the power of the
Indian members in the
Central legislative
assembly.
2. It organized Indian struggle for
freedom.

http://www.insightsonindia.com INSIGHTS

3. This movement gave birth to the


mantra of Purna Swaraj.
Choose the correct answer using the codes
below.
a)
b)
c)
d)

1 and 2 only
2 and 3 only
1 and 3 only
2 only

77. A European company was recently fined


heavily for 2010 Mexico oil spill. Which
of the following can be the adverse
consequences of oil spill in oceans?
1. Large scale death of sea birds
2. Dead sea corals
3. Unbalanced food web
4. Decline in the population of
dolphins
Choose the correct answer using the codes
below.
a)
b)
c)
d)

1 and 2 only
2 and 4 only
1 and 3 only
All of the above

78. USA and Cuba have renewed diplomatic


ties recently after almost five decades of
vacuum in relationships. Why were the
ties between US and Cuba severed after
1960s?
1. Cuba was a communist regime.
2. Cuban revolution deteriorated the
ties between USA and Cuba.
3. Cuba did not agree to join the US
led cold war alliance.
Page 19

Choose the correct answer using the codes


below.
a)
b)
c)
d)

1 and 2 only
2 and 3 only
1 and 3 only
All of the above

79. Presently India is a lower middle


income category country as per the
World Bank. The World Bank defines
such economic categories based on
a) Gross National Income in US
Dollars
b) Per capita Gross National Income
in US Dollars
c) Poverty rate, Index of Inequality
and Overall National Income
d) Economic size, Economic
potential and population of a
country

80.The General Services Establishment Act


intensified the discontent already
present in the sepoys which then
became a factor leading to the 1857
mutiny. The Act
a) ordered all recruits of Bengal army
to be ready for service both within
and outside India and even
overseas
b) mandated a solider to be recruited
from every peasant family in the
villages
c) provided for a salary cut for the
sepoys and increased the taxes on
the Indian peasants

http://www.insightsonindia.com INSIGHTS

d) provided for special status to the


Euopean recruits and downgraded
the position of Indian sepoys

81. Consider the following statements about


Swami Dayanand Saraswati.
1. He was a scholar of Sanskrit.
2. He was the first Indian who
preached the gospel of Swadeshi.
3. He started the Suddhi
Movement.
4. As per him, Vedas contained all
truth humanity needed to know.
Choose the correct answer using the codes
below.
a)
b)
c)
d)

1 and 2 only
2 and 4 only
1 and 4 only
All of the above

82. Which of the following factors did NOT


help fuel the nationalist movement in
pre-independent India?
1. Indian press and literature
2. Indian contact with the European
Countries
3. Modern methods of transport and
communication
4. Racial discrimination by the
British
Choose the correct answer using the codes
below.
a)
b)
c)
d)

1 and 2 only
2 and 3 only
1 and 4 only
All factors contributed.
Page 20

83. Which of the following is/are related


with the relationship between the
faction of moderates and extremists as
far as the Indian national movement is
concerned?
1. Surat Split 1907
2. Lucknow Pact 1916
3. August declaration 1917
4. Lahore Session Congress 1929

2. Dandi March
3. Poona Pact
4. Official demand for Pakistan by
Muslim league
Choose the correct answer using the codes
below.
a)
b)
c)
d)

1 and 2 only
2 and 3 only
All of the above
1, 3 and 4 only

Choose the correct answer using the codes


below.
a)
b)
c)
d)

1 and 2 only
2 and 3 only
1 and 4 only
1, 2 and 4 only

84. Which of the following statements with


reference to the Jalianwala Bagh tragedy
1919 is INCORRECT?
a) People gathered at Jalianwala
Bagh to peacefully protest against
the arrest of their leaders.
b) The area was under curfew at that
time.
c) Rabindranath tagore renounced
his knighthood in protest of this
incident.
d) Shooting of the masses without
prior warning by Militay
commanders was permissible
under the law at that time.

85. Chronologically, which of these events


transpired after the First Round Table
Conference?
1. Gandhi-Irwin Pact
http://www.insightsonindia.com INSIGHTS

86. Cripps Mission was sent to India by the


British because
a) they wanted Indian support to
defeat Japan in the Second World
War.
b) after the failure of Third round
table conference, British PM
promised Indian leaders so
c) British wanted to negotiate the
two-nation theory with Indian
leaders
d) British wanted the support of the
Princely state armies to defeat
Burma

87. The system of Mansabdari in medieval


India was concerned with
1. Improving administration in
remote areas
2. Increasing revenue collection for
the Mughals
3. Recruitment to the army
Choose the correct answer using the codes
below.
Page 21

a)
b)
c)
d)

1 and 2 only
2 and 3 only
2 only
3 only

88. Buddhism was patronized more than


Hinduism by some of the Greek rulers in
India because
a) Greeks did not want to conquer
any Indian territories with war
and violence
b) Greeks did not believe in idol
worship, the concepts of penance
and renunciation
c) Greeks did not believe in the
concept of castes and rituals in
Hinduism
d) Greeks found embracing
Buddhism as a simple tool to
assimilate and access the Indian
culture and society prevailing at
that time

89. Which of the following types of microorganisms can NOT be used in Indian
industries?
1. Fungi
2. Yeast
3. Bacteria
4. Micro-algae
Choose the correct answer using the codes
below.
a)
b)
c)
d)

1 only
3 only
2 and 4 only
All can be used.

http://www.insightsonindia.com INSIGHTS

90. Which of the following statements is/are


correct concerning the population of
Asian elephants found in India?
1. They can be found in dry thorn
forests as well as swamps and
grasslands.
2. They are found only in Southern
and Western India.
3. They are an endangered species in
India.
4. They cannot survive in moist and
rainy places.
Choose the correct answer using the codes
below.
a)
b)
c)
d)

1 only
3 only
2 and 4 only
1 and 4 only

91. In Industrial terminology, Sunrise


industries are those which
1. are emerging in nature
2. work for the conservation of
environment
3. focus on technologies related to
solar power generation
4. are heavily subsidized by the
government
5. are populated with people from the
most neglected sections of the
society
Choose the correct answer using the codes
below.
a)
b)
c)
d)

1 only
1, 3 and 4 only
3, 4 and 5 only
2, 3, 4 and 5 only
Page 22

92. The first successful modern textile mill


was established in Mumbai in 1854.
Which of the following geographical and
industrial factors were responsible for
it?
1. Warm climate
2. Abundance of Moisture in
atmosphere
3. Availability of raw material nearby
4. Availability of skilled labour in the
region
Choose the correct answer using the codes
below.
a)
b)
c)
d)

1 and 2 only
3 and 4 only
1, 2 and 3 only
All of the above

93. With reference to the human


population, the natural growth rate of a
country means
a) population growth without
positive or negative intervention
by the government
b) difference between birth and
death rate barring cases of
unnatural deaths
c) population growth that would
have been possible without the use
of family planning devices
d) difference between birth and
death rate of a country

http://www.insightsonindia.com INSIGHTS

94. Why are the oceanic trenches very


helpful in the study of plate movement?
1. They are the deepest part of the
ocean.
2. They are associated with active
volcanoes.
3. Strong earthquakes are more
common in this region than the
surrounding ocean floor.
Choose the correct answer using the codes
below.
a)
b)
c)
d)

1 and 2 only
1 and 3 only
2 and 3 only
All of the above

95. The Tundra region supports only low


growing plants because
1. the growing season is short in
Tundra
2. the regions are mostly water logged
3. duration of day light is little in
some seasons
4. sub soil is permanently frozen
Choose the correct answer using the codes
below.
a)
b)
c)
d)

1, 2 and 4 only
1 and 3 only
2 and 3 only
All of the above

Page 23

96. Mizo hills in the eastern part of India are


a) a northern extension of Arakan
ranges of Myanmar
b) an eastward extension of
Chotanagpur plateau
c) a northward projection of Eastern
Himalays
d) a northward projection of
Bangladeshi mountain ranges

99. Change in which of the following factors


may lead to change in the amount of
insolation received on earth from the
Sun?
1. High increase in cloud cover
2. Wobbling of earth
3. Decrease in land cover on earth
Choose the correct answer using the codes
below.

97. When you enter a car or a bus, during


summers, where windows are closed,
you feel more heat than outside. This is
because of
a) Heating effect of the
electromagnetic portion of Suns
rays
b) Trapping of infrared radiation
inside the Car
c) Increase in ultraviolet radiation
inside the car
d) Concentration of Suns rays
passing from the glass

98. Which of the following factors affect the


spatial and vertical temperature
distribution in ocean waters?
1. Latitude
2. Continentality
3. Direction of Wind
4. Ocean currents
Choose the correct answer using the codes
below.
a)
b)
c)
d)

a)
b)
c)
d)

1 and 2 only
1 and 3 only
2 and 3 only
All of the above

100.
One striking feature of these
paintings is the verdant greenery it
depicts. The style is naturalistic, and
great attention is paid to detail. The
foliage depicted is vast and varied. This
is made noticeable by using multiple
shades of green. These paintings feature
flowering plants and creepers, leafless
trees, rivulets and brooks.
To which of the following paintings does above
description refer?
a)
b)
c)
d)

Kangra School of Painting


Kerala School of Painting
Rajput School of Painting
None of the above

1 and 2 only
2 and 4 only
2, 3 and 4 only
All of the above

http://www.insightsonindia.com INSIGHTS

Page 24


INSIGHTS MOCK TEST SERIES 2015: TEST 29 SOLUTIONS

1. Solution: a)

http://www.thehindu.com/business/bad-loans-may-persist-for-a-while/article7354634.ece
An algorithm is a specific set of clearly defined instructions aimed to carry out a task or process.
Algorithmic trading (automated trading, black-box trading, or simply algo-trading) is the process of
using computers programmed to follow a defined set of instructions for placing a trade in order to
generate profits at a speed and frequency that is impossible for a human trader.
The defined sets of rules are based on timing, price, quantity or any mathematical model. Apart from
profit opportunities for the trader, algo-trading makes markets more liquid and makes trading more
systematic by ruling out emotional human impacts on trading activities.

2. Solution: a)
http://www.thehindu.com/todays-paper/tp-national/sc-has-ruled-out-mediation-in-rapecases/article7351655.ece
The term "mediation" broadly refers to any instance in which a third party helps others reach
agreement. More specifically, mediation has a structure, timetable and dynamics that "ordinary"
negotiation lacks. The process is private and confidential, possibly enforced by law. Participation is
typically voluntary. The mediator acts as a neutral third party and facilitates rather than directs the
process.
These are some of the rules laid down by Delhi HC;
Appointment of mediator/conciliator
(a) Parties to a suit or other proceeding may agree on the name of the sole mediator/conciliator for
mediating between them.
(b) Where, there are two or more sets of parties and are unable to agree on a sole
mediator/conciliator, the Court may ask each party to nominate the mediator/conciliator or may
nominate/appoint the mediator/conciliator, as it deems fit.

3. Solution: a)
Xinjiang province is near the Northern border of India with China. BCIM corridor passes from the
eastern side.

http://www.insightsonindia.com INSIGHTS

Page 1


INSIGHTS MOCK TEST SERIES 2015: TEST 29 SOLUTIONS

One of the strategic factors driving the corridor is reduction of reliance on the Straits of Malacca,
militarily dominated by the U.S.
Mizoram would be connected with Myanmars port of Sittwe, through the Kaladan River, and the
passage will provide all the landlocked north-eastern States access to the sea. Compared with the land
route, Sittwe provides these States access to Kolkata

http://www.thehindu.com/news/national/china-india-fasttrack-bcim-economic-corridorproject/article7355496.ece

4. Solution: c)
As of now, IIMs function as autonomous societies and run as per their Memorandum of Association
(MoA) that is different in case of each institute.
The government does not have absolute say in the appointment of the directors, but normally the
approval of the MHRD is required before appointing director in some of the IIMs.
IIMs are free to change the educational curriculum as per the MOUs. However, if the change is
strategic and very significant, the government may intervene. There is no clear cut role of the
government in the MOUs.
http://www.dnaindia.com/academy/report-bid-to-undermine-faculty-s-role-in-governance-of-iimsdraws-flak-1536273
http://www.insightsonindia.com INSIGHTS

Page 2


INSIGHTS MOCK TEST SERIES 2015: TEST 29 SOLUTIONS

http://www.thehindu.com/opinion/op-ed/let-iims-be-free-of-governmentstranglehold/article7354672.ece?css=print

5. Solution: d)
According to the terms of the Pradhan Mantri Awas Yojana, the government of India will undertake to
construct about two crore houses by the year 2022. Each house provided under the scheme will
involve a central grant of about INR 1 lakh which may go up to INR 2.3 lakhs. This will come as part of
a 6.5 percent interest rate subsidy scheme (previous schemes had an interest rate subsidy of about 1
percent).
This means that the applicants from lower income groups who avail of the housing scheme may apply
for a housing loan with interest subsidy of 6.5 percent. The tenure or term for these housing loans
may go up to 15 years and the total benefit received by such loan subsidy will add up to INR 1 to 2.3
lakh each.
Currently housing loan interest rates are estimated at about 10.5 percent. The subsidy should,
therefore be a major relief to applicants. The Housing for All scheme will replace all previous
government housing schemes such as the Rajiv Awas Yojana.

6. Solution: a)
http://www.thehindubusinessline.com/economy/rajan-world-economy-may-be-slipping-into-1930sdepression-problems/article7357324.ece
http://www.thehindu.com/business/we-are-pushed-to-competitive-monetary-easingrajan/article7224462.ece
In a recent speech Rajan said, I want to focus on unconventional monetary policies (UMP), by which
I mean both policies that hold interest rates at near zero for long, as well as balance sheet policies
such as quantitative easing or exchange intervention, that involve altering central bank balance sheets
in order to affect certain market prices.
Quantitative easing increases the money supply by flooding financial institutions with capital in an
effort to promote increased lending and liquidity.
Such policies only deprive nations of potential growth without any structural improvements in the
economy that attract investment. It is not good for the economy in the long run.

http://www.insightsonindia.com INSIGHTS

Page 3


INSIGHTS MOCK TEST SERIES 2015: TEST 29 SOLUTIONS

7. Solution: c)
We all know about Obamas visit to India where he signed the Nuclear 123 deal with India.
Chinese premier came to India in September 2014. Both countries signed 12 agreements in Delhi, one
of which will see China investing $20bn (12.2bn) in India's infrastructure over five years.
On the other hand, Modi visited Japan, Russia and France, not the other way round. The Japanese
premier did not come to India.
About Russian visit - http://www.firstpost.com/world/expanding-presence-chinas-periphery-modivisit-5-central-asian-nations-russia-trip-july-2270024.html
http://www.thehindu.com/todays-paper/modis-central-asia-visit-will-focus-on-fighting-islamicstate-menace/article7372350.ece
http://www.narendramodi.in/list-of-agreementsinitiativesannouncements-signedagreed-duringvisit-of-prime-minister-to-france-09-12-april-2015

8. Solution: d)
http://www.thehindu.com/todays-paper/mobile-number-portability-from-july-3/article7372758.ece
MNP allows mobile subscribers to retain their existing mobile phone number even when they change
service provider.
At present, mobile subscribers can switch their service provider within the same telecom circle. The
pan-India MNP will allow customers to keep their existing mobile phone numbers while shifting
between States and circles.

9. Solution: d)
http://www.thehindu.com/todays-paper/dna-test-can-be-ordered-even-in-maintenance-cases-sayshc/article7372835.ece
The question has been made in context of a recent SC order allowing DNA tests in maintenance cases.
DNA is a powerful tool for identification purposes and is admissible evidence. DNA or
Deoxyribonucleic Acid is like a blue print of biological guidelines. DNA analysis is commonly known

http://www.insightsonindia.com INSIGHTS

Page 4


INSIGHTS MOCK TEST SERIES 2015: TEST 29 SOLUTIONS

as DNA Finger printing, it gives a unique and specific profile similar to a thumb impression and hence
it is named as DNA Finger Printing.
DNA analysis plays an important role in the case of murder, rape, paternity and maternity disputes
and also in cases of baby exchange.
This is a technique of Justice through advance science.

10.

Solution: d)

The fund would be operationalized soon and would be made available to organisations such as
NABARD that made development investment decisions. NGOs might not qualify.
The GCF is not to be confused with development aid money.
The Green Climate Fund will support projects, programmes, policies and other activities in
developing country Parties using thematic funding windows. It is intended to be the centrepiece of
efforts to raise Climate Finance of $100 billion a year by 2020.
http://www.thehindu.com/todays-paper/tp-national/green-climate-fund-to-become-operationalsoon/article7372378.ece

11. Solution: d)
The question was made in the context of mercury pollution victims (factories) coming out to seek
justice.
http://www.thehindu.com/todays-paper/tp-national/mercury-pollution-victims-seekjustice/article7372374.ece

The consumption of fish is by far the most significant source of ingestion-related mercury
exposure in humans and animals, although plants and livestock also contain mercury due to
bioconcentration of mercury from seawater, freshwater, marine and lacustrine sediments,
soils, and atmosphere, and due to biomagnification by ingesting other mercury-containing
organisms.
Exposure to mercury can occur from breathing contaminated air, from eating foods that have
acquired mercury residues during processing, from exposure to mercury vapor in mercury
amalgam dental restorations, and from improper use or disposal of mercury and mercurycontaining objects, for example, after spills of elemental mercury or improper disposal of
fluorescent lamps.

http://www.insightsonindia.com INSIGHTS

Page 5


INSIGHTS MOCK TEST SERIES 2015: TEST 29 SOLUTIONS

Consumption of whale and dolphin meat, as is the practice in Japan, is a source of high levels
of mercury poisoning. Tetsuya Endo, a professor at the Health Sciences University of
Hokkaido, has tested whale meat purchased in the whaling town of Taiji and found mercury
levels more than 20 times the acceptable Japanese standard.
Human-generated sources, such as coal-burning power plants emit about half of atmospheric
mercury, with natural sources such as volcanoes responsible for the remainder. An estimated
two-thirds of human-generated mercury comes from stationary combustion, mostly of coal.
Other important human-generated sources include gold production, nonferrous metal
production, cement production, waste disposal, human crematoria, caustic soda production,
pig iron and steel production, mercury production (mostly for batteries), and biomass burning.

12. Solution: d)
http://www.thehindu.com/todays-paper/tp-national/digital-india-week-could-provide-investmentboost/article7372373.ece
http://vikaspedia.in/e-governance/digital-india/digital-india-week
Digital India Week was organized throughout India with the objectives:

Inform, educate and engage with citizens through organization of events at large number of
Digital Points of Presence such as CSCs/ Post Offices, Schools, Gram Panchayats etc

Connect all netizens through Digital Media campaigns and events

Inform all about the vision, services and benefits of the Programme

Popularise and expand the reach of existing e-services, plan and launch new services

Educate citizens on functional digital literacy, cyber security, cyber hygiene, ensure better
utilization of Digital infrastructure during the DI Week and beyond

Incentivize, motivate and connect citizens with the Digital India Programme.

13. Solution: d)
http://www.thehindu.com/todays-paper/tp-national/modisharif-meet-unlikely-atufa/article7372368.ece

http://www.insightsonindia.com INSIGHTS

Page 6


INSIGHTS MOCK TEST SERIES 2015: TEST 29 SOLUTIONS

Ufa, the capital of Bashkortostan, will be the venue for the international SCO and BRICS summits for
the first time, which have a huge impact on the economic, social and political development of the
participating countries and the world.
Although Ufa is hosting the recent SCO meet, India is still not a SCO member.
Recently, the Central Bank of Russia issued 3-rouble commemorative silver coins for the July SCO
and BRICS summits in Ufa.

14. Solution: d)
http://www.thehindu.com/todays-paper/tp-opinion/the-woes-of-greece/article7372331.ece
The term troika has been widely used in Greece and Cyprus Ireland, Portugal and Spain to refer to the
presence of the European Commission, European Central Bank, and International Monetary Fund in
these countries since 2010 and the financial measures that these institutions have taken.
These states were unable to repay or refinance their government debt or to bail out over-indebted
banks under their national supervision without the assistance of third parties like the EFSF, the ECB,
or the IMF.
Four eurozone states had to be rescued by sovereign bailout programs, which were provided jointly by
the International Monetary Fund and the European Commission, with additional support at the
technical level from the European Central Bank.

15. Solution: d)
http://www.newindianexpress.com/world/50-Nations-Including-India-Sign-Agreement-on-ChinaLed-AIIB/2015/06/29/article2892704.ece
The AIIB will have authorised capital of USD 100 billion, and Asian countries will contribute to up to
75 per cent of the total capital. Each member will be allocated a share of the quota based on their
economic size (not contribution to authorized capital), according to the agreement.
China, India and Russia are the three largest shareholders, taking a 30.34 per cent, 8.52 per cent, 6.66
per cent stake, respectively. Their voting shares are calculated at 26.06 per cent, 7.5 per cent and 5.92
per cent.
The AIIB is designed to finance infrastructure construction in the continent.

http://www.insightsonindia.com INSIGHTS

Page 7


INSIGHTS MOCK TEST SERIES 2015: TEST 29 SOLUTIONS

In the IMF - Unlike the General Assembly of the United Nations, where each country has one vote,
decision making at the IMF was designed to reflect the relative positions of its member countries in
the global economy.
The United States was a leading force in the establishment of the World Bank in 1944 and remains the
largest shareholder of the World Bank today. As the only World Bank shareholder that retains veto
power over changes in the Banks structure, the United States plays a unique role in influencing and
shaping development priorities.

16. Solution: c)
http://www.thehindu.com/todays-paper/tp-opinion/teaching-the-poor-tobehave/article7368386.ece
Behavioural economics uses insights from psychology, anthropology, sociology and the cognitive
sciences to come up with more realistic models of how people think and make decisions. Where these
decisions tend to be flawed from an economic point of view, governments can intervene with policies
aimed at nudging the targeted citizens towards the right decision.
The report states in all earnestness that poverty shapes mindsets. The context of poverty depletes
a persons bandwidth the mental resources necessary to think properly as a result of which he
or she is, well, a poor decision-maker, especially compared to those who are not in the context of
poverty, such as the rich and the middle classes.

17. Solution: d)
The BCCI was in news recently due to several reasons:
https://en.wikipedia.org/wiki/Board_of_Control_for_Cricket_in_India
http://indianexpress.com/article/opinion/columns/view-from-the-right-clean-up-cricket/\
http://www.thehindu.com/opinion/op-ed/bcci-monopoly-and-judicial-review/article6824141.ece
http://www.cricbuzz.com/cricket-news/73174/icc-and-bcci-need-to-be-more-transparent
http://www.thehindu.com/todays-paper/rebel-cricket-league-will-find-going-tough-says-nsrinivasan/article7368403.ece
The Board of Control for Cricket in India (BCCI) is the national governing body for cricket in India.
The board was formed in December 1928 as a society, registered under the Tamil Nadu Societies
http://www.insightsonindia.com INSIGHTS

Page 8


INSIGHTS MOCK TEST SERIES 2015: TEST 29 SOLUTIONS

Registration Act. It is a consortium of state cricket associations and the state associations select their
representatives who in turn elect the BCCI officials.
The BCCI's constitution provides for annual elections at its Annual General Meeting (AGM) for all
posts, with a bar on re-election of an incumbent president beyond two consecutive years, "provided
that the General Body may in its discretion re-elect the same person as president for the third
consecutive year"
The BCCI has been known to use its power to influence certain ICC decisions. These included
scheduling, player suspension and ICC appointments. As India is a large market in terms of
international cricket revenue, the BCCI's opinions carry weight within the ICC's decision making
process.

18.Solution: c)
http://www.thehindu.com/news/national/corruption-charges-dent-karnataka-lokayuktasimage/article7371954.ece
A mandate for setting up of the institution of Lokayukta through enactment of a law by the State
Legislature within a period of 365 days from the date of commencement of the Act.
The Lokayukta is an anti-corruption authority orombudsman (An ombudsman is an official,
appointed by the government or by parliament to represent the interests of the public). He works
along with the Income Tax Department and the Anti Corruption Bureau. The Lokayukta (sometimes
referred to the institution itself) investigates allegations of corruption and mal-administration against
public servants and is tasked with speedy redressal of public grievances.
The Lokayukta is usually a former High Court Chief Justice or former Supreme Court judge and has a
fixed tenure.
Lokayuktas power varies from State to State. In some States, the Lokayukta inquires into allegations
against public functionaries including Chief Minister, Ministers and MLAs. While some has the power
to investigate into civil servants/bureaucrats, judiciary and police

19. Solution: d)
http://indianexpress.com/article/opinion/columns/an-act-of-moral-will/
The question has been made in the context of the gay marriage judgment given recently by the US
Supreme Court.
http://www.insightsonindia.com INSIGHTS

Page 9


INSIGHTS MOCK TEST SERIES 2015: TEST 29 SOLUTIONS

Here is a table from Laxmikanth comparing the powers of both the courts.

20.

Solution: c)

http://indianexpress.com/article/opinion/editorials/seeking-saraswati-3/
Saraswati River was in news due to NDA government founding a research institute and museum that
will work on Indias mythical river - Saraswati.
The initiative comes from the ministry of tourism, under a scheme for pilgrimage rejuvenation and
spirituality augmentation.
It is also called as antarvahini (roughly, flowing underground), and it surfaces into reality only at
Prayag, where it can safely mingle with the Ganga and Yamuna.
http://www.dailyo.in/politics/saraswati-river-haryana-yamuna-atal-behari-vajpayee-nasa-indusvalley/story/1/3630.html

http://www.insightsonindia.com INSIGHTS

Page 10


INSIGHTS MOCK TEST SERIES 2015: TEST 29 SOLUTIONS

21. Solution: c)
In a bid to recast the countrys urban landscape, the Centre had given nod to the Atal Mission for
Rejuvenation and Urban Transformation (AMRUT), with an outlay of Rs 50,000 crore.
It is the new avatar of the Jawaharlal Nehru National Urban Renewal Mission (JNNURM). But, in a
significant departure from the earlier mission, the Centre will not appraise individual projects.
Details of the Mission:

AMRUT adopts a project approach to ensure basic infrastructure services relating to water
supply, sewerage, storm-water drains, transportation and development of green spaces and
parks with special provision for meeting the needs of children.

Under this mission, 10% of the budget allocation will be given to states and union territories as
incentive based on the achievement of reforms during the previous year.

AMRUT, which seeks to lay a foundation to enable cities and towns to eventually grow into
smart cities, will be implemented in 500 locations with a population of one lakh and above.

Under this mission, states get the flexibility of designing schemes based on the needs of
identified cities and in their execution and monitoring.

States will only submit state annual action Plans to the centre for broad concurrence based on
which funds will be released. But, in a significant departure from JNNURM, the central
government will not appraise individual projects.

Under the mission, states shall transfer funds to urban local bodies within 7 days of transfer by
central government and no diversion of funds to be made failing which penal interest would be
charged besides taking other adverse action by the centre.

22.

Solution: a)

40 years of National emergency of 1975 has made it popular in news items like this one. Hence the
question.
http://indianexpress.com/article/opinion/columns/emergencys-useful-scars/
This kind of transformation of the political system from federal during normal times to unitary during
Emergency is a unique feature of the Indian Constitution.

http://www.insightsonindia.com INSIGHTS

Page 11


INSIGHTS MOCK TEST SERIES 2015: TEST 29 SOLUTIONS

A proclamation of national emergency may be applicable to the entire country or only a part of it. The
42nd Amendment Act of 1976 enabled the president to limit the operation of a National Emergency to
a specified part of India.

23.

Solution: d)

http://indianexpress.com/article/opinion/columns/on-your-marks/

The new system opens up the opportunity for student mobility, allowing students to transfer
credits earned in one institution to another; and for programme portability, allowing
movement from one degree programme to another.
These will be achieved through the introduction of a uniform system of counting credits (which
replaces the papers system), a uniform evaluation system based on grade points (replacing
the marks system), and a uniform semester-based academic year (which replaces the yearlong pattern).
This establishes parity within and across institutions; between Indian higher educational
institutions and many international ones. In principle, this new system should also provide
employers and post-graduate institutions better standards to compare undergraduate students
and their institutions.
The credit based semester system provides flexibility in designing curriculum and assigning
credits based on the course content and hours of teaching.
The choice based credit system provides a cafeteria type approach in which the students can
take courses of their choice, learn at their own pace, undergo additional courses and acquire
more than the required credits, and adopt an interdisciplinary approach to learning, It is
desirable that the HEIs move to CBCS and implement the grading system.

24.

Solution: a)

http://www.livemint.com/Opinion/3yGB9NCwIvxFoiCKMjo42L/Unshackling-centrally-sponsoredschemes.html
In Indias developmental plan exercise we have two types of schemes viz; central sector and centrally
sponsored scheme. The nomenclature is derived from the pattern of funding and the modality for
implementation.

http://www.insightsonindia.com INSIGHTS

Page 12


INSIGHTS MOCK TEST SERIES 2015: TEST 29 SOLUTIONS

Under Central sector schemes, it is 100% funded by the Union government and implemented
by the Central Government machinery. Central sector schemes are mainly formulated on
subjects from the Union List.
In addition, the Central Ministries also implement some schemes directly in States/UTs which
are called Central Sector Schemes but resources under these Schemes are not generally
transferred to States.
Under Centrally Sponsored Scheme (CSS) a certain percentage of the funding is borne by the
States in the ratio of 50:50, 70:30, 75:25 or 90:10 and the implementation is by the State
Governments. Centrally Sponsored Schemes are formulated in subjects from the State List to
encourage States to prioritise in areas that require more attention.
Funds are routed either through consolidated fund of States and or are transferred directly to
State/ District Level Autonomous Bodies/Implementing Agencies.

25.

Solution: d)

http://www.livemint.com/Money/rk6X8CGMHcbEU9PCFv5FhI/Whats-missing-in-the-CicraAct.html
Use of credit information is becoming prevelant. Therefore, it is important that credit agencies
function in a standard way. To an extent, this was achieved when the Credit Information Companies
(Regulation) Act, 2005 (Cicra), came. The legislation was enacted by the government of India to
regulate the credit information companies (CICs).
The Act covers various aspects such as details on eligibility criterion to become a CIC, and the role and
responsibilities of such companies. Data security and penalties for any such lapses have also been
chalked out.

26.

Solution: d)

http://indianexpress.com/article/opinion/columns/the-everywhere-poison/
Though lead is found frequently in our environment, it has no known purpose in our bodies. When
lead gets inside the body, the body confuses it with calcium and other essential nutrients. This
confusion can cause permanent damage to the health of both children and adults.
Lead poisoning can cause a number of adverse human health effects, but is particularly detrimental to
the neurological development of children.
In adults, lead poisoning can cause poor muscle coordination; nerve damage to the sense organs and
nerves controlling the body; increased blood pressure; hearing and vision impairment; reproductive
http://www.insightsonindia.com INSIGHTS

Page 13


INSIGHTS MOCK TEST SERIES 2015: TEST 29 SOLUTIONS

problems (e.g., decreased sperm count); retarded fetal development even at relatively low exposure
levels.

27.

Solution: a)

The human resource development ministry has launched the Study Webs of Active-Learning for
Young Aspiring Minds (SWAYAM), a Web portal where Massive Open On-line Courses (MOOCs) will
be available on all kinds of subjects.
MOOCS or massive open online courses have made access to top university-level content for every
learner possible. By providing free online courses on demand, MOOCS enable learners to learn from
anywhere irrespective of their situation as long as they have internet access.
It will be hosted on a virtual cloud and have provision for the beneficiaries to get certificates following
tests at the end of the courses.
http://edtechreview.in/trends-insights/trends/1598-indian-hrd-ministry-launches-a-mooc-platformswayam

28.

Solution: b)

http://www.bbc.com/news/world-europe-33314442
The government says the EU-IMF bailout terms are unacceptable, but cannot simply be rejected
without the Greek people having their say. Hence the referendum.

http://www.insightsonindia.com INSIGHTS

Page 14


INSIGHTS MOCK TEST SERIES 2015: TEST 29 SOLUTIONS

29.

Solution: d)

This is from PIB features contribution of the new government towards enhancing education in
India.
With the aim to promote educational opportunities among girl students and address the challenge of
lower girl enrolment rates in engineering colleges, the program has been launched.
It is a mentoring and scholarship scheme to enable meritorious girl students to transit from schools to
technical education without much difficulty and also aims to enrich and enhance teaching and
learning of mathematics and science at senior secondary school level by providing free online
resources for all.

http://www.insightsonindia.com INSIGHTS

Page 15


INSIGHTS MOCK TEST SERIES 2015: TEST 29 SOLUTIONS

Therefore it is envisaged as the first step towards achieving this larger goal of addressing the quality
gap between school education and engineering entrance, by addressing the three dimensions of
education - curriculum design, transaction and assessment.
http://www.business-standard.com/article/news-ians/udaan-scheme-for-girl-students-launched114111402061_1.html

30.

Solution: a)

Only 1.5 per cent of the land and less than 10 per cent of the oceans around Antarctica fall under
special protection.
A long-running proposal to designate the Ross Sea (one of the bays surrounding Antarctica),
sometimes called the last ocean of the world because of its pristine condition, as a marine reserve
has so far failed to be negotiated by the convention on the conservation of Antarctic marine living
resources.
The Ross Sea is home to at least 10 mammal species, half a dozen species of birds, 95 species of fish,
and over 1,000 invertebrate species.
http://www.downtoearth.org.in/content/human-activities-taking-toll-antarctic-ecosystems

31. Solution: a)
Nadia district in West Bengal has won the United Nations Public Service Award 2015 in the category
of improving delivery of public services for or eliminating open defecation in the district.
It should be noted that Nadia district is the first open defecation free district in India. The district had
achieved this feat by making toilets available for all under state governments flagship programme
Sabar Shouchagar (Toilets for All).
The model has helped develop generate awareness, improve access to sanitary toilets, and bring
substantial health improvement through improved sanitation. The scheme has mobilised all
stakeholders, especially women and schoolchildren and has seen involvement of faith-based
organisations. The UN has said that this model has the potential to eliminate open defecation from
across the world.

http://www.insightsonindia.com INSIGHTS

Page 16


INSIGHTS MOCK TEST SERIES 2015: TEST 29 SOLUTIONS

32.

Solution: d)

These were the reasons:

Security pre-2005 notes have fewer security features as compared to 2005 and post-2005
currency notes. Thus, lesser will be the chances of its counterfeiting newer currency notes. It
will also remove fake currency notes in circulation which were being pumped in Indian
economy were copies of pre-2005 notes.
Flushing out black money The spillover effect of the RBIs decision will be to flush out black
money. Money has value only as long as it is a medium of exchange and store of value. It loses
its value when it ceases to be a medium of exchange.
A disincentive for cash hoarders With the RBIs announcement, currency hoarders will be left
with no option but to liquidate their unaccounted holdings by spending or exchanging them.

Thus, this was a well thought out exercise by the RBI to capture the money flows into the system
and also help flush out counterfeit notes. International standard practice It is an international
standard practice to remove old series notes.

33.

Solution: d)

India has topped in the regional inflow of Foreign Direct Investment (FDI) in South Asia region
during year 2014, accounting for 34 billion US dollars investments.
It was revealed in United Nations Conference on Trade and Developments (UNCTADs) World
Investment Report 2015. As per the report FDI inflow to India has surged by 22 per cent to about 34
billion US dollars.
India has improved its position to 9th top host country in the world for FDI in 2014. Owing to good
performance of India, the FDI inflows to South Asia region has rose to 41 billion US dollars in 2014.
Top five FDI recipients in South Asia: India, Iran, Pakistan, Bangladesh, Sri Lanka. The report had
divided Asia into three sub-regions viz East and South East Asia, West Asia and South Asia. China is
the worlds largest recipient of FDI with 129 billion dollars investments and has toppled US (92 billion
dollars)

http://www.insightsonindia.com INSIGHTS

Page 17


INSIGHTS MOCK TEST SERIES 2015: TEST 29 SOLUTIONS

34.

Solution: d)

The Delhi Government recently invoked the Essential Services Maintenance Act (ESMA) to end the
two-day-old strike by resident doctors.
The Essential Services Maintenance Act (ESMA) is an act of Parliament of India. It is a central law.

It was established to ensure the delivery of certain services, which if obstructed would affect
the normal life of the people. These include services like public transport (bus services), health
services (doctors and hospitals).

Although it is a very powerful law, its execution rests entirely on the discretion of the State
government. Each state in the union of India, hence has a separate state Essential Services
Maintenance Act with slight variations from the central law in its provisions. This freedom is
accorded by the central law itself.

35.

Solution: c)

http://timesofindia.indiatimes.com/india/GIAN-will-bring-top-scientists-to-IndiaManjul/articleshow/45973668.cms
A new scheme called `Gian` (Global Initiative for Academic Network) has been launched which will
bring world class educators from across the globe to teach in India.
Indian American mathematician, Manjul Bhargava, has agreed to spearhead it. Prof. Bhargava is a
winner of Fields Medal, which is considered as the highest honour for a mathematician in the world.
`Gian` is aimed at tapping the talent pool of scientists and entrepreneurs internationally to encourage
their engagement with the institutes of Higher Education in India so as to augment the country's
existing academic resources, accelerate the pace of quality reform, and elevate India's scientific and
technological capacity to global excellence.

36.

Solution: a)

The European space agency recently announced that its comet-chasing mission Rosetta would be
extended until September 2016 and may end with the dying mother-ship touching down on the
comet.
http://www.insightsonindia.com INSIGHTS

Page 18


INSIGHTS MOCK TEST SERIES 2015: TEST 29 SOLUTIONS

Rosetta was launched in 2004 on an Ariane 5 rocket and reached the comet in 2014, becoming the
first spacecraft to orbit a comet. Rosetta took off from Earth 10 years ago carrying Philae and
traveled 6.4 billion miles before arriving at the comet.
On 12 November 2014, the mission performed the first successful landing on a comet and returned
data from the surface.
Philaes mission was to land successfully on the surface of a comet, attach itself, and transmit data
from the surface about the comets composition. It is a robotic European Space Agency lander.

37.

Solution: c)

The Union Cabinet chaired by the Prime Minister has given its approval to the proposal of the
Ministry of Culture for setting up of a National Memorial at Lala Ka Tola, Sitab, Diara, District
Chhapra (Saran) in Bihar. It is the birth place of Lok Nayak Jai Prakash Narayan.
About Jai Prakash Narayan:

He was an Indian independence activist, social reformer and political leader. He is popularly
called LokNayak.

He participated in Non Cooperation movement (NCM), Civil disobedience movement (CDM)


and Vinoba bhaves Sarvodaya and Bhoodan movement.

In 1999, he was posthumously awarded the Bharat Ratna, Indias highest civilian award, in
recognition of his social work. Other awards include the Magsaysay award for Public Service in
1965.

The Patna airport is also named after him.

Nav Nirman Andolan movement of Gujarat asked Jayaprakash to lead a peaceful agitation. On
8 April 1974, aged 72, he led a silent procession at Patna.

38.

Solution: d)

Since Lord Jagannaths Nabakalebar festival is being observed after a gap of 19 years, the Centre has
decided to promote the Hindu religious festival under the Incredible India Campaign for its
international branding.

The preparations are on in Puri for Nabakalebar festival to be celebrated on 10th July this year.

http://www.insightsonindia.com INSIGHTS

Page 19


INSIGHTS MOCK TEST SERIES 2015: TEST 29 SOLUTIONS

The Nabakalebara is an ancient ritual associated with most of the Jagannath Temples when the
Idols of Lord Jagannath, Balabhadra, Subhadra and Sudarshan are replaced by a new set of
Idols.

A year with an extra Ashadha masa/month as per Hindu Calendar is considered auspicious for
conducting the ceremony. This usually occurs every twelve to nineteen years.

This festival is celebrated at the Jagannath Temple in Puri, Orissa.

39.

Solution: d)

PMKSY is central scheme that aims at providing irrigation facilities to every village in the country by
converging ongoing irrigation schemes implemented by various ministries.
It will focus on end-to-end solution in irrigation supply chain by implementing new programme in a
project mode with decentralised state-level planning and execution.
Pradhan Mantri Krishi Sinchayee Yojana Ensure access to some means of protective irrigation to all
agricultural farms in the country in order to produce per drop more crop to bring desired rural
prosperity.
Flexibility and autonomy: to states in the process of planning and executing irrigation projects in
order to ensure water to every farm.
Irrigation plans: ensure that state and district irrigation plans are prepared on the basis of sources of
availability of water and agro-climatic conditions in that region.
Promoting extension activities: related to on farm water management and crop alignment for
farmers as well as grass root level field functionaries.
Agencies involved: nodal agency for implementation of PMKSY projects will be state agriculture
department. Inter-ministerial National Steering Committee (NSC) will periodically review these
projects.
Budgetary allocation: 1,000 crore rupees for fiscal year 2015-16.
Funding Pattern: Centre- States will share 75: 25 per cent. In case of north-eastern region and hilly
states it will be 90:10.

http://www.insightsonindia.com INSIGHTS

Page 20


INSIGHTS MOCK TEST SERIES 2015: TEST 29 SOLUTIONS

40.

Solution:

The question has been made again with reference to the Greece crisis and its possible exist from
European Union. It is an important topic for this year.
The European Union (EU) is a politico-economic union of 28 member states that are located
primarily in Europe. The EU operates through a system of supranational institutions and
intergovernmental-negotiated decisions by the member states. The institutions are: the European
Commission, the Council of the European Union, the European Council, the Court of Justice of the
European Union, the European Central Bank, the European Court of Auditors, and the European
Parliament. The European Parliament is elected every five years by EU citizens.
The EU has developed a single market through a standardised system of laws that apply in all member
states.
The eurozone, officially called the euro area, is a monetary union of 19 European Union (EU) member
states that have adopted the euro () as their common currency and sole legal tender.
Monetary policy of the zone is the responsibility of the European Central Bank (ECB) which is
governed by a president and a board of the heads of national central banks. The principal task of the
ECB is to keep inflation under control. Though there is no common representation, governance or
fiscal policy for the currency union, some co-operation does take place through the Eurogroup, which
makes political decisions regarding the eurozone and the euro. The Eurogroup is composed of the
finance ministers of eurozone states, but in emergencies, national leaders also form the Eurogroup.

41. Solution: a)
http://indianexpress.com/article/opinion/columns/because-of-the-44th-amendment/
In 1975, the then Prime Minister, Indira Gandhi advised the president to proclaim emergency without
consulting her cabinet. The cabinet was informed of the proclamation after it was made, as a fait
accompli.
The 44th Amendment Act of 1978 introduced a safeguard to eliminate any possibility of the prime
minister alone taking a decision in this regard.
Then, the President could proclaim a national emergency only after receiving a written
recommendation from the cabinet. This means that the emergency can be declared only on the
concurrence of the cabinet and not merely on the advice of the prime minister.

http://www.insightsonindia.com INSIGHTS

Page 21


INSIGHTS MOCK TEST SERIES 2015: TEST 29 SOLUTIONS

42.

Solution: d)

Eastern Dedicated Freight Corridor of Eastern DFC is a proposed freight corridor in India by Indian
Railways. It is going to be a broad gauge corridor. This will have double line and will be electrified.
Due to lack of space the section from Ludhiana in Punjab to Khurja in Uttar Pradesh will be single line
electrified. This corridor will cover total distance of 1839 km. This corridor will also pass through
Dhari (The origin point of Western Dedicated Freight
Corridor) which will serve as a junction.

Apart from faster delivery of goods, this project is also going to help to reduce greenhouse gases
(GHGs) emissions caused by road transportation in this region.

43.

Solution: a)

The scheme will provide more options to farmers for selling their produce. Decision in this regard was
taken at a Union Cabinet meeting presided by Prime Minister.
http://www.insightsonindia.com INSIGHTS

Page 22


INSIGHTS MOCK TEST SERIES 2015: TEST 29 SOLUTIONS

For online national agriculture market, an online platform will be set up. On this platform farmers
will be able to sell and buy vegetables, fruits and other produce from across the country. In this
regard, Union government will also establish an agency to oversee online trading and ensure that
transactions take place smoothly.
This online agriculture market will also focus on creating godowns and facilitate transportation of the
farm produce after the online trade. Presently, farmers are restricted to sell their produce at
traditional mandis or market committees which are charging various taxes on producers.
This platform will also give choice to farmers to sell the farm produce both in traditional mandis or
online platform and boost their incomes and improve availability, moderating price rise.

44.

Solution: a)

The initiative is aimed at class I and II students in reading, writing language comprehension and
numeracy is at par with the world.
The campaign will also seek to ensure that every school provides teaching-learning for 200 days in a
year, with 800 instructional hours.
This would enable students to progress to higher classes with the confidence of good education.
http://www.thehindubusinessline.com/industry-and-economy/education/smriti-irani-launchespadhe-bharat-badhe-bharat-programme/article6353330.ece

45.

Solution: d)

The disqualification of the MPs and MLAs are dealt by either the RPI Act or the anti-defection act.
In case of the anti-defection act, it is the speaker.
In case of the RPI Act, is the President who is advised by the Election Commission.
A MP or MLA can stand from maximum two electoral constituencies. Narendra Modi contested from
both Varansi and Vadodra.

http://www.insightsonindia.com INSIGHTS

Page 23


INSIGHTS MOCK TEST SERIES 2015: TEST 29 SOLUTIONS

46.

Solution: c)

"Crossing the floor"--Passing between the member addressing the House and the Chair which is
considered breach of Parliamentary etiquette.
"Adjournment of Debate"--Adjournment on a motion adopted by the House, of the debate on a
Motion/Resolution/Bill on which the House is then engaged until a future day or sine die as specified
in the motion.
For knowing all important Parliamentary keywords, refer to
http://rajyasabha.nic.in/rsnew/parliamentary_term/important_partliament_term.asp

47.

Solution: d)

The qualifications needed to fight local elections is that

You should be at least 21 years of age


You should be a citizen of India.
You should not have been disqualified under the RPI act.
All other qualifications remaining the same as that of a MP or MLA.

No college verification, approval etc. is needed.

48.

Solution: d)

The Constitution (Article 143) authorises the president to seek the opinion of the Supreme Court in
the two categories of matters:
(a) On any question of law or fact of public importance which has arisen or which is likely to arise.
(b) On any dispute arising out of any pre-constitution treaty, agreement, covenant, engagement,
sanad or other similar instruments.
In the first case, the Supreme Court may tender or may refuse to tender its opinion to the president.
But, in the second case, the Supreme Court must tender its opinion to the president. In both the
cases, the opinion expressed by the Supreme Court is only advisory and not a judicial pronouncement.
Hence, it is not binding on the president; he may follow or may not follow the opinion. However, it
facilitates the government to have an authoritative legal opinion on a matter to be decided by it.

http://www.insightsonindia.com INSIGHTS

Page 24


INSIGHTS MOCK TEST SERIES 2015: TEST 29 SOLUTIONS

49.

Solution: d)

Article 226 of the Constitution empowers a high court to issue writs including habeas corpus,
mandamus, certiorari, prohibition and quo-warrento for the enforcement of the fundamental rights of
the citizens and for any other purpose.
Judicial review is the power of a high court to examine the constitutionality of legislative enactments
and executive orders of both the Central and state governments.
On examination, if they are found to be violative of the Constitution (ultra-vires), they can be
declared as illegal, unconstitutional and invalid (null and void) by the high court. Consequently, they
cannot be enforced by the government.

50.

Solution: d)

Freedom of Speech and Expression under Article 19 implies that every citizen has the right to express
his views, opinions, belief and convictions freely by word of mouth, writing, printing, picturing or in
any other manner. The Supreme Court held that the freedom of speech and expression includes the
following:
o
o
o
o
o
o
o
o
o
o

Right to propagate ones views as well as views of others.


Freedom of the press.
Freedom of commercial advertisements.
Right against tapping of telephonic conversation.
Right to telecast, that is, government has no monopoly on electronic media.
Right against bundh called by a political party or organisation.
Right to know about government activities.
Freedom of silence.
Right against imposition of pre-censorship on a newspaper.
Right to demonstration or picketing but not right to strike.

51. Solution: d)
As per the constitution of India, the Vice-President of India does not make any appointments when he
is acting as the Vice-President of India.
Only when he is in the capacity of the President, can he make the appointments given in the options.
Vice-President of India does not carry any formal authority other than being the Chairman of the
Rajya Sabha.
http://www.insightsonindia.com INSIGHTS

Page 25


INSIGHTS MOCK TEST SERIES 2015: TEST 29 SOLUTIONS

52.

Solution: a)

The reasoning behind the courts judgment was:


Since the Constitution had conferred a limited amending power on the Parliament, the Parliament
cannot under the exercise of that limited power enlarge that very power into an absolute power.
Indeed, a limited amending power is one of the basic features of the Constitution and, therefore, the
limitations on that power cannot be destroyed. In other words, Parliament cannot, under article 368,
expand its amending power so as to acquire for itself the right to repeal or abrogate the Constitution
or to destroy its basic features. The donee of a limited power cannot by the exercise of that power
convert the limited power into an unlimited one.

53.

Solution: d)

A simple case of majority seats not meaning majority votes is Modi governments win by a majority of
seats but only around 38% votes.
On the other hand, in Delhi, AAP secured 54.4% of votes, and won nearly 95% seats.
So majority of seats does not mean necessarily majority of votes.
The opposite is also true. A party can get majority of votes, yet it may not get majority of seats,
because it may not win by a huge margin in all electoral constituencies.

54.

Solution: b)

Voters can not choose members of Rajya Sabha and State Legislative Council. They are indirectly
elected by members of the state LA (almost all in case of RS, and 1/3rd in case of SLC).
Chief Minister of a state cannot vote for Rajya Sabha and State Legislative Council, if he is a member
of State Legislative Council, and not State Legislative Assembly.
Only the Elected members choose 1/3rd of the SLC members, as well as choose RAjya Sabha members,
as well as vote as registered voters for Lok Sabha elections.

http://www.insightsonindia.com INSIGHTS

Page 26


INSIGHTS MOCK TEST SERIES 2015: TEST 29 SOLUTIONS

55.

Solution: c)

India has had the first President running for successive terms. It can be as many number of times. In
the USA, however, there is a ban on more than two terms for a President.
CAG is an important constitutional office maintaining the financial accountability of the executive to
the legislature. A promise of successive terms to a pliant bureaucrat can erode the autonomy of the
institution.
Same is the case with ECI. More than one term may erode the independence of the CEC.
Governor and Chairman, RS do not carry such responsibilities. Hence, they can run for successive
terms.

56.

Solution: a)

The Constituent Assembly was constituted in November 1946 under the scheme formulated by the
Cabinet Mission Plan.
The total strength of the Constituent Assembly was to be 389. Of these, 296 seats were to be allotted
to British India and 93 seats to the Princely States. Out of 296 seats allotted to the British India, 292
members were to be drawn from the eleven governors provinces and four from the four chief
commissioners provinces, one from each.

http://www.insightsonindia.com INSIGHTS

Page 27


INSIGHTS MOCK TEST SERIES 2015: TEST 29 SOLUTIONS

57.Solution: d)
The recommendation for election is made by the government and the notification for election is
issued by the Election Commission.
After the notification, the ECI announces the schedule, and starts accepting nomination papers of
candidates.
After announcing the schedule, the model code of conduct comes automatically in force that prohibits
certain kind of political activities to ensure a level playing field.

58.

Solution: c)

CRR is the amount that the banks have to keep with the RBI. The banks do nto earn any interest rate
on this CRR, which the banks also called dead asset.
Higher is the CRR, lesser money do the banks have to lend to the public.
If CRR is reduced, money supply in the economy is increased. It may lead to inflation and lower
interest rates, not the reverse in the short-run.

59.

Solution: b)

Banks actually create money by lending money.


The money multiplier is the amount of money that banks generate with each dollar of reserves.
Reserves is the amount of deposits that the Federal Reserve requires banks to hold and not lend.
Banking reserves is the ratio of reserves to the total amount of deposits.
The money multiplier is the ratio of deposits to reserves in the banking system.
Imagine that you are the president of a large bank. The Fed requires that you hold 10% of your
deposits in reserves (a reserve ratio of 1/10). This means that for every $1.00 of deposits, you can only
lend out $0.90. The total value of your bank's deposits is $100,000,000. You want to maximize your
bank's profits, so you loan out all of the $90,000,000. All of sudden, you've just increased the supply
of money from $100,000,000 to $190,000,000!
Here's how you did it. Your depositors still have $100,000,000 with you, but only on paper. They can
come in any time and get their money. However, since not everyone wants or needs their money at the
same time, your reserves of $10,000,000 will cover the normal demand for withdrawals.
http://www.insightsonindia.com INSIGHTS

Page 28


INSIGHTS MOCK TEST SERIES 2015: TEST 29 SOLUTIONS

At the same time, you have distributed $90,000,000 in loan funds to your borrowers - that's real
money going out the bank's door.

60.

Solution: d)

Throughout his life, Gandhi sought to develop ways to fight India's extreme poverty, backwardness
and socio-economic challenges as a part of his wider involvement in the Indian independence
movement.
Gandhi's championing of Swadeshi and non-cooperation were centred on the principles of economic
self-sufficiency. Gandhi sought to target European-made clothing and other products as not only a
symbol of British colonialism but also the source of mass unemployment and poverty, as European
industrial goods had left many millions of India's workers, craftsmen and women without a
livelihood.
By championing homespun khadi clothing and Indian-made goods, Gandhi sought to incorporate
peaceful civil resistance as a means of promoting national self-sufficiency.
Gandhi was a self-described philosophical anarchist, and his vision of India meant an India without
an underlying government. He once said that "the ideally nonviolent state would be an ordered
anarchy.

61. Solution: c)
The law of diminishing returns, also referred to as the law of diminishing marginal returns, states that
in a production process, as one input variable is increased, there will be a point at which the marginal
per unit output will start to decrease, holding all other factors constant.
In other words, keeping all other factors constant, the additional output gained by another one unit
increase of the input variable will eventually be smaller than the additional output gained by the
previous increase in input variable. At that point, the diminishing marginal returns take effect.
You can see a live example here in this article http://study.com/academy/lesson/law-of-diminishingreturns-definition-examples-quiz.html

62.

Solution: c)

http://www.reuters.com/article/2015/04/27/us-eu-google-antitrust-idUSKBN0NF1YX20150427
http://www.insightsonindia.com INSIGHTS

Page 29


INSIGHTS MOCK TEST SERIES 2015: TEST 29 SOLUTIONS

Recently Google and some other firms came under the European anti-trust laws that punish anticompetitive practices in markets. Anti-trust laws regulate anti-competitive practices.
The same is done by the Competition Act, 2002 in India which was enacted by the Parliament of India
and governs Indian competition law. It replaced the archaic Monopoly and Restrictive Trade Practices
Act, 1969. Under this legislation, the Competition Commission of India was established to prevent
activities that have an adverse effect on competition in India.
In accordance with the provisions of the Amendment Act, the Competition Commission of India and
the Competition Appellate Tribunal have been established.

63.

Solution: d)

Wholesale Price Index (WPI)


This index is published by office of Economic Adviser (Ministry of Commerce & Industry).The
calculation is done by tracking the average prices of commodities that are traded in bulk i.e.
wholesale.
The commodities under study are divided into three categories: primary articles (food, non-food,
tobacco, fodder and minerals), fuel & power and manufactured products. Currently, the total basket of
products under consideration for WPI is 676 items and the calculation of this index is done by using
Laspeyres formula (it works on averaging principle) and year (2004-2005) commodity prices are
taken as base for reference by equating them to 100 and values of the index are calculated as a
percentage relative to this base.
Consumer Price Index (CPI)
This index is published by Central Statistics Office (CSO) (Ministry of Statistics and Programme
Implementation).In nations like United Kingdom, Poland, Malaysia etc. it is also known as Retail
Price Index. CPI is a method that involves calculation of inflation through time tracking of weighted
averages of 200 consumer goods and services with prices of year 2010 as the base.
So which is the better index for measuring inflation?
The primary disapproval for WPI being used as the key index for estimating inflation is that the
general public doesnt transact on wholesale level on a daily basis. Also, the wholesale doesnt reflect
the pricing trends at retail level and thus, not the definitive ideal indicator of inflation.
On the other hand, many analysts feel that CPI is the accurate method of measuring inflation since it
reflects the ground realities by tracking the costs of the goods that are purchased on daily basis by
general public.
http://www.insightsonindia.com INSIGHTS

Page 30


INSIGHTS MOCK TEST SERIES 2015: TEST 29 SOLUTIONS

The difference between Wholesale Price Index (WPI) and Consumer Price Index (CPI) is that along
with tracking prices of goods, the latter also includes services in the indexing.

64.

Solution: d)

High fiscal deficit does not impeded growth necessarily in the short-run. Only if the fiscal deficit is not
augmented by increased supply will it roadblock growth.
Loose monetary policy gives stimulus to growth.
Liberalising the external sector gives impetus to FDI and FIIs pushing growth in the short-run.
Inflation in the economy is good because it indicates that there is demand for goods greater than its
supply. More demand will lead to more production and more GDP.

65.

Solution: a)

Vedas are a series of sacred texts used in religious rituals and sacrifices. The Vedas are essentially
archetypal poetry of high literary value. They are mythical in nature and their language is symbolic.
Being mythical, they have multiple meanings, and, therefore, the theologian moulds his rites, the
preacher seeks his belief, the philosopher finds the clues for his intellectual speculation and the lawmakers work out the social and political life-style in accordance with the archetypal truths of the
Vedas.
Vedic poets are called the rishis, the seers who visualized the archetypal truths of cosmic functioning
at all levels of existence. Devatas of the vedic poetry symbolise the manifestations of the divine force
of the One Supreme.

66.

Solution: d)

The dominating note in bhakti is ecstasy and total identity with God. It is a poetic approach to religion
and an ascetic approach to poetry. It is poetry of connections connecting the worldly with the
divine, and as a result, the old form of secular love poetry began to have a new meaning in all
languages. The rise of bhakti poetry gave rise to regional languages (Bhasa).
The conception of bhakti did away with the elite tradition of Sanskrit and accepted the more
acceptable language of the common man. Kabir (Hindi) says that Sanskrit is like water of a well
stagnant, Bhasa like flowing water.
http://www.insightsonindia.com INSIGHTS

Page 31


INSIGHTS MOCK TEST SERIES 2015: TEST 29 SOLUTIONS

A seventh century Shaiva Tamil writer Manikkarvachakar has something similar to say about in his
book of poetry Thiruvachakam.
Bhakti also attacked the age-old caste system and devoted itself to the worship of humanity, because
the catch-word of bhakti is that God is there in every human being. The movement was in essence
subaltern, as most of its poets belonged to the so-called lower castes. Bhakti is antitheology and
against any kind of conceptual erudition.

67.

Solution: a)

A barometer measures atmospheric pressure, which can help give you a rough idea of how the
weather will be changing in the next 24 hours.
Sea-level barometer readings usually range from about 29.70 inches of mercury to 30.20 or so, with
29.92 being the average reading.
Pressure also drops with altitude too, so reported pressures are converted to sea-level to factor out the
difference in altitude from individual reports. You can convert your observed pressure to sea-level by
subtracting .06 inches for every 50 feet in altitude.
But it's not so much the actual number reading as the trend the readings are taking. A steadily falling
barometer usually signals a storm is approaching, while a rising barometer normally means sunnier
skies ahead. Although very low readings (under 29.50 inches) can suggest a strong storm
approaching.

68.

Solution: a)

A thunderstorm, also known as an electrical storm, a lightning storm, or a thundershower, is a type of


storm characterized by the presence of lightning and its acoustic effect on the Earth's atmosphere
known as thunder.
While most thunderstorms move with the mean wind flow through the layer of the troposphere that
they occupy, vertical wind shear causes a deviation in their course at a right angle to the wind shear
direction.
Thunderstorms result from the rapid upward movement of warm, moist air.
They can occur inside warm, moist air masses and at fronts. As the warm, moist air moves upward, it
cools, condenses, and forms cumulonimbus clouds that can reach heights of over 20 km. As the rising
air reaches its dew point, water droplets and ice form and begin falling the long distance through the
clouds towards the Earth's surface.
http://www.insightsonindia.com INSIGHTS

Page 32


INSIGHTS MOCK TEST SERIES 2015: TEST 29 SOLUTIONS

As the droplets fall, they collide with other droplets and become larger. The falling droplets create a
downdraft of cold air and moisture that spreads out at the Earth's surface, causing the strong winds
commonly associated with thunderstorms, and occasionally fog.
Thunderstorms can generally form and develop in any particular geographic location, perhaps most
frequently within areas located at mid-latitude when warm moist air collides with cooler air.
Thunderstorms are responsible for the development and formation of many severe weather
phenomena.

69.

Solution: a)

Normally, light waves from the sun travel straight through the atmosphere to your eye. But, light
travels at different speeds through hot air and cold air.
Mirages happen when the ground is very hot and the air is cool. The hot ground warms a layer of air
just above the ground.
When the light moves through the cold air and into the layer of hot air it is refracted (bent).
A layer of very warm air near the ground refracts the light from the sky nearly into a U-shaped bend.
Our brain thinks the light has travelled in a straight line.
Our brain doesn't see the image as bent light from the sky. Instead, our brain thinks the light must
have come from something on the ground.

http://www.insightsonindia.com INSIGHTS

Page 33


INSIGHTS MOCK TEST SERIES 2015: TEST 29 SOLUTIONS

70.

Solution: d)

Sunspots are temporary phenomena on the photosphere of the Sun that appear visibly as dark spots
compared to surrounding regions. They correspond to concentrations of magnetic field that inhibit
convection and result in reduced surface temperature compared to the surrounding photosphere.
Sunspots usually appear as pairs, with each spot having the opposite magnetic polarity of the other.
Although they are at temperatures of roughly 3,0004,500 K (2,7004,200 C), the contrast with the
surrounding material at about 5,780 K (5,500 C) leaves them clearly visible as dark spots.
Manifesting intense magnetic activity, sunspots host secondary phenomena such as coronal loops
(prominences) and reconnection events. Most solar flares and coronal mass ejections originate in
magnetically active regions around visible sunspot groupings. Similar phenomena indirectly observed
on stars other than the sun are commonly called starspots and both light and dark spots have been
measured.
Such activities directly affect the level of solar output and ion concentration on earth and earths
atmosphere, and affect its climate and weather.

71. Solution: d)
An antigen is any substance that causes your immune system to produce antibodies against it. An
antigen may be a foreign substance from the environment, such as chemicals, bacteria, viruses, or
pollen. An antigen may also be formed inside the body, as with bacterial toxins or tissue cells.
Foreign antigens originate from outside the body. Examples include parts of or substances produced
by viruses or microorganisms (such as bacteria and protozoa), as well as substances in snake venom,
certain proteins in foods, and components of serum and red blood cells from other individuals.

72.

Solution: a)

Mms is a Sanskrit word that means "reflection" or "critical investigation".It is also the name of
one of six orthodox (astika) schools of Hinduism. The school is known for its philosophical theories
into the nature of dharma based on hermeneutics of the Vedas.
The school was influential and foundational to Vedanta school of Hinduism, with the difference that
Mimamsa school developed and emphasized karma-kanda (study of ritual actions, early parts of
Vedas), while Vedanta school developed and emphasized jnana-kanda (study of knowledge and
spirituality, later parts of Vedas).
The classical Mimamsa school is sometimes referred to as Purva-Mimamsa or Karma-Mimamsa.
http://www.insightsonindia.com INSIGHTS

Page 34


INSIGHTS MOCK TEST SERIES 2015: TEST 29 SOLUTIONS

73.

Solution: d)

Acharya Tulsi was a prominent Jain religious leader.


In 1949 he launched the Anuvrat Movement (anu = small, vrat=vow, Anuvratas are the limited
version of the Mahavratas for the monks), based on the five Jain principles Truth, Nonviolence, Nonpossession, Non-stealing and Celibacy as applied in their limited version for the lay people.
The movement encouraged people to apply the Anuvratas in their personal lives, even when dealing
with non-religious aspects of the society. The movement also held that Dharma is not for ensuring
happiness in the future lives but also for achieving happiness in the present life.
The movement has continued under the leadership of his disciple Yuvacharya Mahapragya. He was
raised to the rank of the Acharya by Acharya Tulsi himself.

74.

Solution: c)

Characteristics of Gamma Radiation and X-Rays

Gamma radiation and X-rays are electromagnetic radiation like visible light, radio waves, and
ultraviolet light. These electromagnetic radiations differ only in the amount of energy they
have. Gamma rays and X-rays are the most energetic of these.
Gamma radiation is able to travel many meters in air and many centimeters in human tissue. It
readily penetrates most materials and is sometimes called "penetrating radiation."
X-rays are like gamma rays. They, too, are penetrating radiation.
Radioactive materials that emit gamma radiation and X-rays constitute both an external and
internal hazard to humans.
Dense materials are needed for shielding from gamma radiation. Clothing and turnout gear
provide little shielding from penetrating radiation but will prevent contamination of the skin
by radioactive materials.
Gamma radiation is detected with survey instruments, including civil defense instruments.
Low levels can be measured with a standard Geiger counter, such as the CD V-700. High levels
can be measured with an ionization chamber, such as a CD V-715.
Gamma radiation or X-rays frequently accompany the emission of alpha and beta radiation.
Instruments designed solely for alpha detection (such as an alpha scintillation counter) will not
detect gamma radiation.
Pocket chamber (pencil) dosimeters, film badges, thermoluminescent, and other types of
dosimeters can be used to measure accumulated exposure to gamma radiation.

http://www.insightsonindia.com INSIGHTS

Page 35


INSIGHTS MOCK TEST SERIES 2015: TEST 29 SOLUTIONS

75.Solution: c)

The sitar is the main instrument of Hindusthani (North Indian) music and is gourd-shaped with a
long neck. The instrument itself in fact is made out of aged pumpkin shell. It has many strings, the
number varies but 17 is usual. It is usually performed alongside the tabla.
The veena is the sitar's Carnatic counterpart and is also gourd-shaped with a long neck. It is made of
wood. The veena has 4 main strings and 3 drone or sympathetic strings, and is usually paired with the
mridangam which is the South Indian drum.
The Carnatic flute is known as Venu, and the Hindustani flute as Bansuri. This is the distinction
between them in vocabulary. The Venu consists of 8 finger holes, while Bansuri consists of six or
seven finger holes. Also bansuri is used mostly in north indian music while venu in south indian
music.

76.

Solution: d)

The movement aroused tremendous national awakening. While developing complete distrust in
British Government, people revived a sense of self-confidence and self- esteem. For the first time the
Congress movement could become a mass movement. Moreover, the pre-eminent position of Gandhiji
in the National Congress was established.
Mantra of Purna-swaraj was born at the Lahore session of the Congress in 1929.
The British had already made half-hearted concessions by way of the 1919 Act.

77. Solution: d)
http://www.thehindu.com/news/international/bp-settles-2010-us-oil-spill-claims-for-187bln/article7379427.ece
In the British Petroleum (BP) oil spill of 2010, 4.9 million barrels of crude oil were spilt in the Gulf of
Mexico.
According to Time, thousands of dead invertebrates like starfish and coral were found. Unfortunately,
these species play an essential role in the ecosystems to which they belong, thereby impacting many
other marine populations. Similarly, many dolphin offspring were found dead along the Gulf Coast.
Oyster beds were also devastated by the oil spill; in fact, it could take ten years for the population to
reach its former size.

http://www.insightsonindia.com INSIGHTS

Page 36


INSIGHTS MOCK TEST SERIES 2015: TEST 29 SOLUTIONS

You can understand the impact of oil spill in greater detail here https://www.nwf.org/What-WeDo/Protect-Habitat/Gulf-Restoration/Oil-Spill/Effects-on-Wildlife.aspx

78.

Solution: a)

To understand in short, after Cubas independence in 1902, in the following 20 years the United
States repeatedly intervened militarily in Cuban affairs: 1906 1909, 1912 and 1917 1922.
The Cuban economy was highly dominated by US companies.
However, the rise of General Fulgencio Batista in the 1930s to de facto leader and President of Cuba
for two terms (194044 and 195259) led to an era of close co-operation between the governments of
Cuba and the United States.
But, as armed conflict broke out in Cuba between rebels led by Fidel Castro and the Batista
government, the U.S. was urged to end arms sales to Cuban government.
Following the Cuban Revolution of 1959, bilateral relations deteriorated substantially. In 1961, the
U.S. severed diplomatic ties with Cuba and began pursuing covert operations to topple the
Communist regime.
Besides, the U.S. imposed and subsequently tightened a comprehensive set of restrictions and bans
vis-a-visa the Cuban regime as retaliation for the nationalization of U.S. corporations' property by
Cuba, and has stated it will continue it so long as the Cuban government continues to refuse to move
toward democratization and greater respect for human rights.
Read this page to understand the US-Cuba relations in detail. The topic is important for Mains too.
https://en.wikipedia.org/wiki/Cuba%E2%80%93United_States_relations#Relations_1900.E2.80.93
1959

79.

Solution: b)

Economies are currently divided into four income groupings: low, lower-middle, upper-middle, and
high, based on GNI per capita (in U.S. dollars, converted from local currency using the Atlas method.
Indias GDP crossed the $2-trillion mark in 2014, according to data released by the World Bank. After
taking 60 years to reach the $1-trillion mark, India added the next trillion in just seven years.
The World Bank data also show that Indias gross national income per person rose to $1,610 (around
Rs. 1 lakh) a year during 2014 from $1,560 the previous year.
http://www.insightsonindia.com INSIGHTS

Page 37


INSIGHTS MOCK TEST SERIES 2015: TEST 29 SOLUTIONS

It would take India a little more than a decade to rise from its current lower middle income category
to the upper middle income level.
http://www.thehindu.com/business/Economy/india-is-now-a-2trillion-economy-says-world-bankdata/article7380442.ece

80.

Solution: a)

Apart from the social and economic causes of the 1857 revolt, this was one of the chief military causes.
Then the bullet biting episode lead to the popular revolt.

81.Solution: d)

82.

Solution: d)

This is a very good article listing all the factors that were responsible for the growth of nationalism in
India.
http://www.yourarticlelibrary.com/sociology/top-14-factors-for-the-growth-of-nationalism-inindia/32942/
This is equally helpful for your Mains preparation. Do go through it.

83.

Solution: a)

http://www.insightsonindia.com INSIGHTS

Page 38


INSIGHTS MOCK TEST SERIES 2015: TEST 29 SOLUTIONS

84.

Solution: d)

The Jallianwala Bagh massacre, also known as the Amritsar massacre, took place on 13 April 1919
when a crowd of nonviolent protesters, along with Baishakhi pilgrims, who had gathered in the
Jallianwala Bagh garden in Amritsar, Punjab were fired upon by troops of the British Indian Army
under the command of Reginald Dyer.
The crowd had gathered to protest against the arrest of two nationalist leaders, Dr. Satyapal, Dr.
Saifuddin Kitchlew, although a curfew had been recently declared. On Dyer's orders, his troops fired
on the crowd for ten minutes, directing their bullets largely towards the few open gates through which
people were trying to run out.

85.

Solution: d)

Gandi-irwin pact was signed in 1931 after the failure of the first round table conference.
This led to the second round table conference, the failure of which made Gandhi return to India. He
and other important leaders were arrested on return.
When Gandhi was in jail, the British PM Ramsay announced the Communal award to settle which
Gandhi and Ambedkar signed the Poona Pact.
Muslim league made the demand for Pakistan in 1940.
http://www.insightsonindia.com INSIGHTS

Page 39


INSIGHTS MOCK TEST SERIES 2015: TEST 29 SOLUTIONS

86.

Solution: a)

The Cripps mission was an attempt in late March 1942 by the British government to secure full Indian
cooperation and support for their efforts in World War II. The mission was headed by Sir Stafford
Cripps, a senior left-wing politician and government minister in the War Cabinet of Prime Minister
Winston Churchill.
ps worked to keep India loyal to the British war effort in exchange for a promise of full selfgovernment after the war. Cripps promised to give dominion status after the war as well as elections
to be held after the war. Cripps discussed the proposals with the Indian leaders and published them.
Both the major parties, the Congress and the League rejected his proposals and the mission proved a
failure.
Congress moved toward the Quit India movement whereby it refused to cooperate in the war effort,
while the British imprisoned practically the entire Congress leadership for the duration of the war.

87.

Solution: d)

Mansabdari system was the system of providing ranks to the officials of the Mughal Empire. The
mansabdars governed the empire and commanded its armies in the emperor's name. Though they
were usually aristocrats, they did feudal aristocracy, for neither the offices nor the estates that
supported them were hereditary. There were 33 categories of mansabdars
The term is derived from Mansab, meaning 'rank'. Hence, Mansabdar literally means rank-holder.
nstituted by the Mughal emperor Akbar, mansabdari was a system common to both the military and
the Civil department. Basically the Mansabdari system was borrowed from Persia. It was prevalent
during the reign of Babur and Humayun. Akbar made some important changes to the system and
made it more efficient.

88.

Solution: d)

Buddhism prospered under the Indo-Greek kings, and it has been suggested that their invasion of
India was intended to protect the Buddhist faith from the religious persecutions of the ugas (185
73 BCE), who had overthrown the Mauryans.
Kanishkas conquests and patronage of Buddhism played an important role in the development of the
Silk Road, and the transmission of Mahayana Buddhism from Gandhara across the Karakoram range
to China.
http://www.insightsonindia.com INSIGHTS

Page 40


INSIGHTS MOCK TEST SERIES 2015: TEST 29 SOLUTIONS

However, in Buddhist tradition, Kanishka is often described as a violent, faithless ruler before his
conversion to Buddhism.

89.

Solution: d)

The term Industrial Microbiology refers to the use of microorganisms for industrial purposes.
Such things as anticoagulants, antidepressants, vasodilators, herbicides, insecticides, plant hormones,
enzymes, and vitamins have been isolated from microorganisms or produced in large quantities by
genetically engineering the organisms with foreign genes.
In commercial industrial plants, microorganisms are widely used to produce numerous organic
materials that have far-reaching value and application.
Examples can be anti-biotic, wine production, Single-cell protein etc.

90.

Solution: a)

India has the largest population of the Asian elephants. Today, there are just about 20,000 to 25,000
elephants in their natural habitat spreading across the evergreen forests, dry thorn forests, swamps
and grasslands.
Their prime habitats are, however, the moist deciduous forests. Their population in India ranges from
North-West India where they are found in the forest divisions of Dehradun, Bijnor and Nainital
districts of UP to the Western Ghats in the states of Karnataka and Kerala and in Tamil Nadu.
In Central India, their population is distributed in southern Bihar and Orissa. In the East, they are
seen in North-Bengal, Assam and a few other states

91. Solution: a)
Emerging industries are also known as Sunrise Industries. Morover, sunrise industries are expected
to grow fast and become important in the future.
Examples of sunrise industries include hydrogen fuel production, space tourism,
and online encyclopaedias.
Some of the others include Information technology, Wellness, Hospitality and Knowledge.

http://www.insightsonindia.com INSIGHTS

Page 41


INSIGHTS MOCK TEST SERIES 2015: TEST 29 SOLUTIONS

Over a period of years or decades, as an industry grows and matures, it may pass from the sunrise
phase to maturity and, finally, the sunset stage. The compact-disc industry is a typical example of
such a transition. It was a sunrise industry in the 1990s as compact discs replaced vinyl records and
cassette tapes, but the rapid adoption of digital media in the 21st century could mean that the
compact-disc industry's days are numbered.

92.

Solution: d)

Humidity is very essential for textile weaving.


In modern textiles humidity is created artificially. In older days this technology was not available. So,
moist climate was the first need of the mills.
Moreover, cotton requires a warm climate to grow which was present near Mumbai.
For similar reasons, Ahmedabad also became a major hub for textiles in India. The first mill was
established there right after 5 years from the one at Mumbai.

93.

Solution: d)

Births are usually measured using the birth rate i.e. the number of live births per 1,000 people.
Deaths are usually measured using the death rate i.e. the number of deaths per 1,000 people.
Migrations is the movement of people in and out of an area.
Births and deaths are the natural causes of population change. The difference between the birth rate
and the death rate of a country is called the natural growth rate.
The population increase in the world is mainly due to rapid increase in natural growth rate.

94.

Solution: d)

Oceanic Deeps or Trenches are some 3-5 km deeper than the surrounding ocean floor - the deepest
parts of the oceans.
The trenches are relatively steep sided, narrow basins.
They occur at the bases of continental slopes and along island arcs and are associated with active
volcanoes and strong earthquakes.

http://www.insightsonindia.com INSIGHTS

Page 42


INSIGHTS MOCK TEST SERIES 2015: TEST 29 SOLUTIONS

That is why they are very significant in the study of plate movements. As many as 57 deeps have been
explored so far; of which 32 are in the Pacific Ocean; 19 in the Atlantic Ocean and 6 in the Indian
Ocean.

95.

Solution: d)

Polar climates exist poleward beyond 70 latitude. Polar climates consist of two types: (i) Tundra
(ET); (ii) Ice Cap (EF).
Tundra Climate (ET)
The tundra climate (ET) is so called after the types of vegetation, like low growing mosses, lichens and
flowering plants. This is the region of permafrost where the sub soil is permanently frozen. The short
growing season and water logging support only low growing plants. During summer, the tundra
regions have very long duration of day light.
Ice Cap Climate (EF)
The ice cap climate (EF) occurs over interior Greenland and Antartica. Even in summer, the
temperature is below freezing point. This area receives very little precipitation.

96.

Solution: a)

The Lushai Hills is also called Mizo Hills is a part of Patkai range or the purvanchal mountain ranges
of India. Lushai hills are very rich in Flora & Fauna and covered with dense bamboo jungle and
inhabited by the Mizo and Lushais tribes of North East India.
It is a northern extension of the Arakan ranges of Myanmar.
Mizo hills are often confused with Garo Khasi and Jaintia hills.
The Garo,Khasi and Jaintia Hills are the three major hills of Patkai range located in the state of
Meghalaya. The group of mountains are extremely rich in flora & fauna and home to highest
waterfalls in India along with the wettest place on earth Cherrapunji and nearby Mawsynram.

http://www.insightsonindia.com INSIGHTS

Page 43


INSIGHTS MOCK TEST SERIES 2015: TEST 29 SOLUTIONS

97.

Solution: b)

The term greenhouse is derived from the analogy to a greenhouse used in cold areas for preserving
heat. A greenhouse is made up of glass. The glass which is transparent to incoming short wave solar
radiation is opaque to outgoing long wave radiation.
The glass, therefore, allows in more radiation and prevents the long wave radiation going outside the
glass house, causing the temperature inside the glasshouse structure warmer than outside. When you
enter a car or a bus, during summers, where windows are closed, you feel more heat than outside.
Likewise during winter the vehicles with closed doors and windows remain warmer than the
temperature outside. This is another example of the greenhouse effect.

98.

Solution: d)

The temperature of surface water decreases from the equator towards the poles because the amount
of insolation decreases poleward.
The oceans in the northern hemisphere receive more heat due to their contact with larger extent of
land than the oceans in the southern hemisphere.
The winds blowing from the land towards the oceans drive warm surface water away from the coast
resulting in the upwelling of cold water from below. It results into the longitudinal variation in the
temperature. Contrary to this, the onshore winds pile up warm water near the coast and this raises the
temperature.
Warm ocean currents raise the temperature in cold areas while the cold currents decrease the
temperature in warm ocean areas. Gulf stream (warm current) raises the temperature near the
eastern coast of North America and the West Coast of Europe while the Labrador current (cold
current) lowers the temperature near the north-east coast of North America.

99.

Solution: a)

An astronomical theory is Millankovitch oscillations, which infer cycles in the variations in the earths
orbital characteristics around the sun, the wobbling of the earth and the changes in the earths axial
tilt.
All these alter the amount of insolation received from the sun, which in turn, might have a bearing on
the climate.

http://www.insightsonindia.com INSIGHTS

Page 44


INSIGHTS MOCK TEST SERIES 2015: TEST 29 SOLUTIONS

Volcanism is considered as another cause for climate change. Volcanic eruption throws up lots of
aerosols into the atmosphere. These aerosols remain in the atmosphere for a considerable period of
time reducing the suns radiation reaching the Earths surface

100.

Solution: a)

One striking feature of Kangra paintings is the verdant greenery it depicts. The style is naturalistic,
and great attention is paid to detail. The foliage depicted is vast and varied. This is made noticeable by
using multiple shades of green. The Kangra paintings feature flowering plants and creepers, leafless
trees, rivulets and brooks. The Kangra artists adopted various shades of the primary colors and used
delicate and fresher hues. For instance, they used a light pink on the upper hills to indicate distance.
Kangra paintings depict the feminine charm in a very graceful manner. Facial features are soft and
refined. The female figures are exceptionally beautiful. A regal figure seated on a throne, Pahari
school Later Kangra paintings also depicted nocturnal scenes, and storms and lightning. The
paintings were often large and had complex compositions of many figures and elaborate landscapes.
Towns and house clusters were often depicted in the distance. The Kangra painters used colors made
of vegetable and mineral extracts. They employed cool and fresh colors. Kangra paintings are known
for the lyrical blending of form and color. The Kangra Arts Promotion Society - an NGO at
Dharamshala Himachal Pradesh is working for the promotion of this art which is at the verge of
extinction today. This NGO is running a school to train young boys and girls in this art. It also runs a
workshop where genuine Kangra Paintings are made on traditional handmade paper using only
mineral and vegetable colours.

http://www.insightsonindia.com INSIGHTS

Page 45

INSIGHTS MOCK TEST - 30


INSIGHTS ON INDIA MOCK PRELIMINARY EXAM - 2015


INSIGHTS ON INDIA MOCK TEST - 30
GENERAL STUDIES

PAPER-I

Time Allowed: 2 Hours

Maximum Marks: 200

INSTRUCTIONS
1. IMMEDITELY AFTER THE COMMENCEMENT OF THE EXAMINATION, YOU SHOULD
CHECK THAT THIS TEST BOOKLET DOES NOT HAVE ANY UNPRINTED OR TORN OR MISSING
PAGES OR ITEMS, ETC. IF SO, GET IT REPLACED BY A COMPLETE TEST BOOKLET.
2. You have to enter your Roll Number on the Test
Booklet in the Box provided alongside. DO NOT
Write anything else on the Test Booklet.
4. This Test Booklet contains 100 items (questions). Each item is printed only in English. Each item
comprises four responses (answers). You will select the response which you want to mark on the
Answer Sheet. In case you feel that there is more than one correct response, mark the response which
you consider the best. In any case, choose ONLY ONE response for each item.
5. You have to mark all your responses ONLY on the separate Answer Sheet provided. See directions in
the Answer Sheet.
6. All items carry equal marks.
7. Before you proceed to mark in the Answer Sheet the response to various items in the Test Booklet, you
have to fill in some particulars in the Answer Sheet as per instructions sent to you with your
Admission Certificate.
8. After you have completed filling in all your responses on the Answer Sheet and the examination has
concluded, you should hand over to the Invigilator only the Answer Sheet. You are permitted to take
away with you the Test Booklet.
9. Sheets for rough work are appended in the Test Booklet at the end.
10. Penalty for wrong answers :
THERE WILL BE PENALTY FOR WRONG ANSWERS MARKED BY A CANDIDATE IN THE
OBJECTIVE TYPE QUESTION PAPERS.
(i)

There are four alternatives for the answer to every question. For each question for which a
wrong answer has been given by the candidate, one-third of the marks assigned to that
question will be deducted as penalty.

(ii) If a candidate gives more than one answer, it will be treated as a wrong answer even if one of
the given answers happens to be correct and there will be same penalty as above to that
question.
(iii)

If a question is left blank, i.e., no answer is given by the candidate, there will be no penalty
for that question.
http://www.insightsonindia.com

INSIGHTS ON INDIA MOCK TEST SERIES FOR CIVIL SERVICES PRELIMINARY EXAM 2015

http://www.insightsonindia.com INSIGHTS

Page 1

INSIGHTS MOCK TEST - 30


1. Jayaprakash Narayans Concept of Total


Revolution was aimed at
a) Exposing the totalitarian nature of
the incumbent government
b) Removing landlessness from rural
areas completely by Bhoodan and
forced acquisitions
c) Bringing a change in the existing
society in tune with the ideals of the
Sarvodaya
d) Strengthening the local selfgoverning institutions on Gandhian
lines enabling them to counter
increasing centralization in
governance

2. In India the term Buddhist circuit can


be used to refer to
1. Racing track in Noida
2. Sarnath-Gaya-Varansi region
3. Old Mahayana scriptures found in
Veshali
Choose the correct answer using the codes
below.
a)
b)
c)
d)

1 and 2 only
2 and 3 only
1 and 3 only
All of the above

3. Consider the following lines.


Thou hast made me endless, such is thy
pleasure. This frail vessel thou emptiest again
and again, and fillest it ever with fresh life.
http://www.insightsonindia.com INSIGHTS

This little flute of a reed thou hast carried over


hills and dales, and hast breathed through it
melodies eternally new........
Thy infinite gifts come to me only on these
very small hands of mine. Ages pass, and still
thou pourest, and still there is room to fill.
These are from the poems written in
a. Savitri: A Legend and a Symbol - by
Sri Aurbindo
b. Mappings by Vikram Seth
c. Gitanjali - by Rabindranath Tagore
d. The Golden Threshold by Sarojini
Naidu

4. Consider the following about the


recently released Socio-economic Caste
Census (SECC).
1. Majority of households in India are
deprived as per SECC.
2. Majority of households are maleheaded.
3. Over one-third of population living
in rural areas is illiterate.
4. Landlessness is now rare in rural
India.

Choose the correct answer using the codes


below.
a)
b)
c)
d)

1 and 2 only
2 and 3 only
1 and 4 only
3 and 4 only

Page 2

INSIGHTS MOCK TEST - 30


5. There has been a proposal to utilize the


Foreign Exchange reserves for financing
infrastructure projects. Why is there
concern about using Forex for such
projects?
1. Infrastructure projects are long-term
and risky in nature.
2. Enough Forex reserves are needed to
avert a Balance of Payment crisis
situation.
Which of the above is/are correct?
a)
b)
c)
d)

1 only
2 only
Both 1 and 2
None

6. The Scheduled Tribes and Other


Traditional Forest Dwellers
(Recognition of Forest Rights) Act,
2006, also called as the Forest Rights
Act (FRA), 2006 confers which of the
following rights on tribals?
1. Rights over forest land patches
2. Rights over minor produce from
forests
3. Rights over timber produce from
forest
4. Rights over the wildlife in the forests
Choose the correct answer using the codes
below.
a)
b)
c)
d)

1 and 2 only
2, 3 and 4 only
1 and 4 only
All of the above

http://www.insightsonindia.com INSIGHTS

7. Consider the following about the state of


malnourishment in India as per recently
released data by UNICEF.
1. Underweight Children
2. Underweight adolescent girls
3. Stunted Children
Majority of children (girls where applicable) in
India fall under which of the above categories?
a)
b)
c)
d)

1 and 2 only
2 and 3 only
1 and 3 only
2 only

8. When we refer to the stabilizing market


forces we generally refer to
a) Forces of Supply and demand
b) Perfect competition in the market
c) Optimizing behaviour of individuals
d) All of the above
9. Consider the following about
Rakhigarhi.
1. The site is the largest known city of
the Indus civilization.
2. It is older than Harappa and
Mohenjodaro sites.
3. It is situated on the dry bed of the
Sarasvati river.
4. The site does not have urban
features like standards, coins,
drainage system.
Choose the correct answer using the codes
below.
a)
b)
c)
d)

1 and 3 only
2 and 4 only
1, 2 and 3 only
1, 3 and 4 only

Page 3

INSIGHTS MOCK TEST - 30


10. In the recent visit of Prime Minister to


the Central Asia republics, concerns
were raised on terrorist activities from
these republics as
a) Many from the security agencies of
the Central Asian republics are
joining terrorist outfits
b) There is an apprehension of State
sponsored terrorism
c) There is an apprehension of
Military sponsored terrorism
d) The Central Asian republics are the
main base for all the terrorist
activities being run in West Asia
11. Which of these authorities is responsible
for GDP data calculation & which
authority is responsible for revising GDP
calculation methodology?
1. Central government
2. Economic Adviser to Finance
Ministry
3. RBI
4. Central Statistical Office
Choose the correct answer using the codes
below.
a.
b.
c.
d.

3 and 1
3 and 2
4 and 1
4 does both.

12. The Indian constitution protects the


rights of minorities because
1. In an electoral democracy, the will
of the majority prevails over the
minority.
2. All minority classes are socioeconomically vulnerable India.

a.
b.
c.
d.

1 only
2 only
Both 1 and 2
None

13. The recent Chinese stock market crash


was due to
a. Poor state of economic reforms in
China
b. Slack regulation of stock markets by
the Chinese government
c. The contagion of the Greece crisis
d. None of the above

14. Consider the following.


1. Linfa
2. Chan-Hom
3. Nangka
How the above are related to China?
a. These are recently erupted volcanoes
in China.
b. These are typhoons that have
recently either hit China or are
nearing around it.
c. These are parts of the remote
Southern Province of China that
borders Aksai China that were
recently in news due to heightened
cross-border infiltrations.
d. None of the above

Which of the above is/are true?


http://www.insightsonindia.com INSIGHTS

Page 4

INSIGHTS MOCK TEST - 30


15. Why is the Akash missile highly useful


for the Indian army that has recently
inducted it?
1. It is a surface-to-air missile, so it can
hit targets from land.
2. It can cruise at supersonic speeds.
3. It is capable to neutralise aerial
targets like cruise missiles, fighter
jets and air-to- surface missiles.
4. It can engage multiple targets
simultaneously and can travel in
multiple directions.
Choose the correct answer using the codes
below.
a.
b.
c.
d.

1 and 2 only
3 and 4 only
1, 2 and 3 only
All of the above

16. Uttarakhand has become the first Indian


state to install a system to detect
earthquakes and disseminate warnings.
It is able to give advance warning for
earthquakes because
1. The system is capable of detecting
harmless P waves that arrive earlier
than
destructive S waves.
2. It has been deployed at the possible
focuses of past earthquake regions in
Uttarakhand.
Which of the above is/are true?
a.
b.
c.
d.

1 only
2 only
Both 1 and 2
None

http://www.insightsonindia.com INSIGHTS

17. India and United States have signed


inter-governmental agreement to
implement the Foreign Account Tax
Compliance Act (FATCA). It will help
both countries to
1. Promote transparency on tax matters
2. Curb offshore tax evasions
3. Avoid problems related to double
taxation of firms
Choose the correct answer using the codes
below.
a.
b.
c.
d.

1 and 2 only
2 and 3 only
1 and 3 only
All of the above

18. India should be economically interested


in Kazakhastan because
1. It is the Worlds largest producer of
Uranium.
2. It can benefit fertilizer sector in
India due to its abundance of
natural gas.
3. Its hydrocarbon reserves can
supplement the oil economy of
India.
Choose the correct answer using the codes
below.
a.
b.
c.
d.

1 and 2 only
2 and 3 only
1 and 3 only
All of the above

19. Union Government has decided to


develop all villages located within 10
kilometres vicinity of the international
border on a priority basis with the help
of
1. Swachh Bharat Abhiyaan
Page 5

INSIGHTS MOCK TEST - 30


2. Promotion of sports activities in


border areas
3. Promotion of border tourism
4. Protection of heritage sites
Choose the correct answer using the codes
below.
a.
b.
c.
d.

1 and 2 only
2 and 3 only
1 and 4 only
All of the above

20. Japans Meiji-era sites historically


contributed in
a. Japans evolution from feudalism to
an industrialised modern economy
b. Development of fishing industry in
Japan
c. Tourism growth led by heritage
conservation in Japan
d. Sparking the technological
innovation in Japan

21. In the Phillips curve what is the


relationship between rate of
unemployment and inflation in the
short-run?
a. Unemployment and inflation are not
related to each other.
b. Decreasing Unemployment rate
leads to lower inflation in the
economy.
c. Decreasing Unemployment rate
requires higher inflation in the
economy.
d. Inflation can grow even at constant
levels of unemployment in the
economy.
http://www.insightsonindia.com INSIGHTS

22. The Reserve Bank of India exercises


which of the following types of control
over banks?
1. RBI sets the interest rates offered
by all domestic banks in India.
2. RBI can supersede strategic
decisions of the Board of Governors
of Public Sector
Banks in
India.
3. RBI regulates mergers and
acquisition of banks.
Choose the correct answer using the codes
below.
a.
b.
c.
d.

1 and 2 only
2 and 3 only
1 and 3 only
All of the above

23. With regard to the Currency system in


India, which of the following practices is
followed?
a. Level of currency volume is pegged at
the amount of Gold reserves in the
country
b. Level of currency circulation is
delinked from the level of metallic
gold in the country
c. Fresh currency cannot be printed if it
is not backed by a metallic form of
wealth
d. Fresh currency cannot be printed if it
is not backed by foreign exchange
reserves

Page 6

INSIGHTS MOCK TEST - 30


24. The calculation of national income


covers which of the following?
1. Government consumption
expenditure
2. Net exports
3. Gross capital formation
Choose the correct answer using the codes
below.
a.
b.
c.
d.

1 and 2 only
2 and 3 only
1 and 3 only
All of the above

25. Why are subsidies given to general


public by the government inflationary in
nature?
1. It increases the disposable income of
people.
2. It does not tackle the supply side of
goods.
3. It inflates the fiscal deficit of the
government leading to greater
money supply.
4. It distorts market mechanisms of
pricing.
Choose the correct answer using the codes
below.
a.
b.
c.
d.

1 and 4 only
2 and 3 only
1, 3 and 4 only
All of the above

3. NRI deposits
Choose the correct answer using the codes
below.
a.
b.
c.
d.

1 and 2 only
2 and 3 only
1 and 3 only
All of the above

27. What are transfer payments given to


the public by the government?
a. Payment for services rendered in
present times
b. Payment that is not linked to the
services in the present, and is a
reward for services rendered in the
past
c. Payment that is not linked to any
service provided to government
d. Both (b) and (c)

28. The success of the Gold monetization


scheme will depend on
1. The public perception of gold as an
asset
2. Inflation in the economy
3. Ease of conversion of Gold into more
liquid assets
4. Level of financial inclusion in the
economy
Choose the correct answer using the codes
below.

26. The difference between interest rates


between India and other countries can
affect which of the following in India?
1. Currency exchange rate
2. Investment in the country
http://www.insightsonindia.com INSIGHTS

a.
b.
c.
d.

1 and 2 only
2 and 4 only
1, 3 and 4 only
All of the above
Page 7

INSIGHTS MOCK TEST - 30


29. How does the state government exercise


control over the local self-governing
bodies?
1. By enacting laws, rules and
regulations
2. By providing for provisions for
regular elections
3. By releasing finances for
development projects
4. By appointing the chairman and
members of Panchayats and Ward
councils
Choose the correct answer using the codes
below.
a.
b.
c.
d.

1 and 2 only
2 and 3 only
1, 3 and 4 only
1 and 3 only

30. The constituent assembly of India


1. Was elected based on universal adult
franchise
2. Elected and appointed the Prime
Minister and other ministers in the
first government
3. Enacted laws until the first official
Parliament was elected
4. Was restricted in its powers by an
order of the British government
5. Supervised the functioning of the
Supreme Court and other
subordinate courts
Choose the correct answer using the codes
below.
a.
b.
c.
d.

1, 2 and 4 only
2, 3 and 5 only
1 and 3 only
All of the above

http://www.insightsonindia.com INSIGHTS

31. Consider the following.


1. Government resigns if it fails to
enact a public bill in Parliament.
2. Prime Minister is a member of Lok
Sabha.
3. The council of Ministers reflects the
socio-economic diversity of India.
Which of these conventions has/have been
codified into
rules/regulations/laws/provisions?
a.
b.
c.
d.

1 only
2 and 3 only
1 and 3 only
None of the above

32. Abolition of the legislative council in a


state requires
a. the consent of the President and
Governor concerned
b. the consent of the Parliament and
the state legislature concerned
c. the consent of the State government
and State legislature
d. the consent of Parliament, Governor
and State legislature concerned

33. Which of the following can NOT be


forced on Indian citizens?
1. Singing national anthem
2. Showing allegiance to all national
sports teams
3. Healthy habits like regulated diets,
exercise and Yoga
Choose the correct answer using the codes
below.
Page 8

INSIGHTS MOCK TEST - 30


a.
b.
c.
d.

1 only
2 and 3 only
3 only
None of the above

34. Consider the following statements.


Assertion (A): The Constitution of India derives
its authority from the Parliament of India.
Reason (R): It was enacted by the constituent
assembly of India which had legal and
democratic authority.
In the context of the statements above, which
of these is true?
a) A and R both are true, and R is the
correct explanation for A.
b) A and R both are true, and R is the
NOT the correct explanation for A.
c) A is correct, R is incorrect.
d) A is incorrect, R is correct.

35. Consider the following about Public


Interest litigation (PIL).
1. PIL is a legal instrument.
2. It can be entertained by both
administrative and judicial bodies.
3. Representatives of victims can also
file a PIL.
4. PIL can be filed only in social and
environmental cases.
Choose the correct answer using the codes
below.
a.
b.
c.
d.

36. Providing for reservation of women in


which of the following bodies will NOT
require a constitutional amendment?
1. Lok Sabha
2. Rajya Sabha
3. State Legislative Assembly
4. State Legislative Council
5. Political parties
Choose the correct answer using the codes
below.
a.
b.
c.
d.

All of the above


3 and 4 only
5 only
None of the above

37. Match these schedules of Constitution to


what they contain.
1. First Schedule List of names of All
States and Union Territories
2. Second Schedule Powers of
President, Governor and Judges
3. Fourth Schedule Allocation of seats
in Rajya Sabha
4. Seventh Schedule Division of
powers between Legislative,
Executive and Judiciary
Choose the correct answer using the codes
below.
a.
b.
c.
d.

1, 2 and 4 only
2 and 4 only
1 and 3 only
All of the above

1, 2 and 4 only
2 and 3 only
3 only
3 and 4 only

http://www.insightsonindia.com INSIGHTS

Page 9

INSIGHTS MOCK TEST - 30


38. Parliament exercises control over the


administration because
a. the executive is collectively
responsible to the legislature
b. it is the constitutional mandate of the
representatives to command and
control administrative affairs
c. executive branch of the government
is subordinate to the legislative
branch of the government
d. administration is not directly
controlled by the executive

39. Consider the following statements about


the term of Lok Sabha.
1. The President of India can reduce the
term of Lok Sabha.
2. The speaker of Lok Sabha by himself
cannot shorten or extend the term of
a Lok
Sabha.
3. Term of Lok Sabha can be extended
during emergency.
Choose the correct answer using the codes
below.
a.
b.
c.
d.

1 and 2 only
2 and 3 only
1 and 3 only
All of the above

40. Arrange the ranks of the following


functionaries from higher to lower in the
order of precedence.
1. Governor of a state within the state
2. Speaker of Lok Sabha
3. Former President
4. Chief Minister of a State outside a
state
http://www.insightsonindia.com INSIGHTS

Choose the correct order using the codes below.


a.
b.
c.
d.

3124
1324
2134
3241

41. Article 355 of the Indian Constitution


provides that It shall be the duty of the
Union to protect every State against
external aggression and internal
disturbance. This shows the
a. Federal character of the Indian
constitution
b. Quasi-federal Character of Indian
polity
c. Democratic character of Indian
polity
d. Unity and territorial integrity of
India
42. Lok Adalats are a useful to instrument to
reduce the burden on Judiciary in India.
Which of the following types of cases can
be handled by Lok Adalat?
1. Consumer grievance against food
standards in rural processed
products
2. Cases involving property disputes
3. Cases in involving Pension and other
transfer payment rights
4. Cases involving road accidents
Choose the correct answer using the codes
below.
a.
b.
c.
d.

1 and 2 only
2 and 3 only
1 and 4 only
All of the above
Page 10

INSIGHTS MOCK TEST - 30


43. Consider the following about Service


tax.
1. It is an indirect tax.
2. It is levied by the Central
government.
3. It is applicable to manufacturing as
well as services sector in India.

amendments, but the bill is not


passed in case of disagreement
b. it is sent to the President for reconsideration and then transferred to
Lok Sabha for consideration
c. it is sent to a joint select committee
of the Parliament for re-examination
and re-introduction in Lok Sabha
d. None of the above

Choose the correct answer using the codes


below.
a.
b.
c.
d.

1 and 2 only
2 and 3 only
1 and 3 only
2 only

44. Which of the following is fixed by the


Constitution of India?
1. Allowances of the President and
Governor
2. Maximum Size of Council of
Ministers
3. Manner of election of the Sarpanch
of the Gram Panchayat.
Choose the correct answer using the codes
below.
a.
b.
c.
d.

1 and 2 only
2 and 3 only
1 and 3 only
All of the above

45. If the Rajya Sabha does not agree to the


provisions of the Annual Budget then
a. it returns the bill to the Lok Sabha
with proposed amendments, which
then passes it or rejects the proposed
http://www.insightsonindia.com INSIGHTS

46. USA has recently granted Tunisia a


major Non-NATO Ally (MNNA) status.
This would allow both countries to
a. improve economic ties, reinforce
trade and reinvigorate mutual
investment
b. enhance military cooperation
c. negotiate security contracts and
exchange confidential information
d. improve mutual governance related
matters
47. Consider the following.
1. She holds the record for the most
spacewalks by a woman.
2. She holds the world record for the
most spacewalk time by a woman
astronaut.
3. She is the first Indian woman
astronaut to serve NASA.
Which of the above is/are true about Sunita
Williams?
a.
b.
c.
d.

1 and 2 only
2 and 3 only
1 and 3 only
All of the above

Page 11

INSIGHTS MOCK TEST - 30


48. Full Shanghai Corporation Organization


(SCO) membership to India will have
significant benefits in
1. Combating terrorism
2. Opening up trade, energy sector and
strategic transit routes
3. Resolving strictly bilateral land
border disputes
Choose the correct answer using the codes
below.
a.
b.
c.
d.

1 and 2 only
2 and 3 only
1 and 3 only
All of the above

49. Consider the following about the


Regional Anti-terrorist Structure
(RATS) of the Shanghai Corporation
Organization (SCO).
1. It is headquartered in Beijing, China.
2. It is a permanent organ of the
Shanghai Cooperation Organization.
3. It deals with extremism and
separatism too.
4. All member-states contribute to
intelligence gathering in RATS.
Choose the correct answer using the codes
below.
a.
b.
c.
d.

1 and 4 only
2 and 3 only
2, 3 and 4 only
All of the above

http://www.insightsonindia.com INSIGHTS

50. Which of the following crops do NOT


need rich soil for its production?
1. Barley
2. Millets
3. Rice
4. Tea
Choose the correct answer using the codes
below.
a.
b.
c.
d.

1, 2 and 4 only
2 and 3 only
2, 3 and 4 only
1, 3 and 4 only

51. Pillars erected under the regime of


Ashok bear which of the following
characteristics?
1. They contain lessons in Dharma.
2. They are monolithic.
3. All pillars have been made by heavy
metals.
4. They have been erected outside India
too.
Choose the correct answer using the codes
below.
a.
b.
c.
d.

1, 3 and 4 only
2 and 3 only
2, 3 and 4 only
1 and 2 only

52. Consider the following statements.


1. Asia has the highest mean elevation
among all continents.
2. The distance of the deepest point in
the Ocean from mean sea level is
greater than that of the highest point
on land.
Page 12

INSIGHTS MOCK TEST - 30


3. All the open oceans of the world have


the same mean sea level.
Choose the correct answer using the codes
below.
a.
b.
c.
d.

1 and 2 only
2 and 3 only
1 and 3 only
All of the above

53. After the Arab Spring, which of the


following West Asian or Middle-eastern
countries turned democratic?
1. Tunisia
2. Libya
3. Egypt
4. Turkey
5. Lebanon
Choose the correct answer using the codes
below.
a.
b.
c.
d.

1, 2 and 3 only
2 and 3 only
4 and 5 only
2, 3, 4 and 5 only

54. Consider the following objectives.


1. Protection of Intellectual property
rights.
2. Promotion of resource flow between
nations.
3. Ensuring technological
modernization of nations to
specialize world trade.
Which of the above come under the mandate of
the World Trade Organization (WTO)?

b. 2 and 3 only
c. 1 and 3 only
d. 2 only

55. In a conference recently, the PM talked


about technologies being highly
developed in ancient India. Which of
these technologies was the first to be
implemented in ancient India?
a. Technology of mass flight
b. Technology of plastic surgery
c. Technology of surgery without the
use of hands and instruments
d. Transplantation of organs between
members of the same family

56. Consider the following about China type


of climate.
1. High rainfall throughout the year
2. Low temperature variation in
summer and winter
3. Distinct dry period
4. Very cold winters
Choose the correct answer using the codes
below.
a.
b.
c.
d.

1 and 2 only
2 and 3 only
1 and 4 only
3 and 4 only

57. Consider the following about Coke.


1. It is used in coal as well as nuclear
power plants.
2. It can be used to remove silica
content of the iron ore.

a. 1 and 2 only
http://www.insightsonindia.com INSIGHTS

Page 13

INSIGHTS MOCK TEST - 30


3. Coke contains higher percentage of


carbon by weight that coal.
Choose the correct answer using the codes
below.
a.
b.
c.
d.

1 and 2 only
2 and 3 only
1 and 3 only
3 only

58. Which of the following factors


contribute to formation of Roaring
Forties?
1. Air being displaced from
the Equator towards the South Pole
2. Earth's rotation
3. Easterlies
4. Ocean currents
Choose the correct answer using the codes
below.
a.
b.
c.
d.

1 and 2 only
2 and 3 only
1 and 4 only
All of the above

59. Why was Tripura involved in the Indian


freedom struggle?
1. The Tripura tribals were forced to
pay high land rent by the British.
2. Many tribals of Tripura were part of
the Indian British army that were
involved in 1857 mutiny.
3. The princely state of Tripura was
occupied by the British.

a.
b.
c.
d.

1 and 2 only
2 and 3 only
1 and 3 only
None of the above

60. Methane found in mines is dangerous


for the workers because
1. When mixed with air it can become
explosive.
2. It can be poisonous even in open
mines.
3. Methane mixed with mine water can
become toxic for human
consumption.
Choose the correct answer using the codes
below.
a.
b.
c.
d.

1 and 2 only
2 and 3 only
1 only
All of the above

61. Boiler industries do not use hard water


for their operations. Why?
a. Hard water can result in scaling and
cracking of boilers.
b. Hard water blocks boiler operation.
c. Minerals in hard water react with
boiler metal and replace it.
d. It erodes boiler metals at a very fast
pace.

Choose the correct answer using the codes


below.
http://www.insightsonindia.com INSIGHTS

Page 14

INSIGHTS MOCK TEST - 30


62. Keystone species are important in an


ecosystem because
a. They determine the genetic diversity
of an ecological community.
b. They are important in determining
the ability of a large number of other
species to persist in the community.
c. They are found at the edge of an
ecosystem and signal the shift in
ecosystems.
d. They are rare in the ecosystem and
stand at the top of the food web.
63. Anti-cyclonic conditions can be formed
when
1. Atmospheric pressure is high.
2. Temperature is low.
3. Rainfall is high and persistent.
Choose the correct answer using the codes
below.
a.
b.
c.
d.

1 and 2 only
2 and 3 only
1 only
All of the above

64. The principal gases dissolved in Magma


is/are
1. Water vapour
2. Methane
3. Carbon dioxide
4. Sulphur dioxide
5. Ozone
Choose the correct answer using the codes
below.
a.
b.
c.
d.

1 and 2 only
2, 3 and 4 only
1 and 3 only
3, 4 and 5 only

http://www.insightsonindia.com INSIGHTS

65. A rectangular drainage pattern develops


where
a. strongly jointed rock terrain is found
b. soft and hard rocks exist parallel to
each other
c. river channels follow the slope of the
terrain
d. streams flow in two parallel
directions from a central peak

66. The Brahmaputra River carries a large


volume of water and silt in India, but
carries a smaller amount of both water
and silt while coming from Tibet. Why?
1. Tibet is a cold and dry area.
2. Brahmputra forms many riverine
islands in Tibet.
3. In India it passes from the Northeastern region which is a region of
high rainfall.
Choose the correct answer using the codes
below.
a.
b.
c.
d.

1 and 2 only
2 and 3 only
1 and 3 only
All of the above

67. Consider the following statements about


rivers in Peninsular India.
1. The main water divide in Peninsular
India is formed by the Western
Ghats.
2. The drainage basin of Peninsular
Rivers is smaller than Himalayan
Rivers.
3. Peninsular rivers also flow in rift
valleys and form estuaries.
Page 15

INSIGHTS MOCK TEST - 30


Choose the correct answer using the codes


below.
a.
b.
c.
d.

1 and 2 only
2 and 3 only
1 and 3 only
All of the above

68. Pressure and wind system of an area


depends on which of the following
factors?
1. Latitude
2. Altitude
3. Distance from sea
4. Ocean currents
5. Relief features
Choose the correct answer using the codes
below.
a.
b.
c.
d.

1, 2 and 3 only
2 and 3 only
1 and 5 only
All of the above

69. A characteristic feature of the cold


weather season over the northern plains
is the inflow of cyclonic disturbances
from the west and the northwest.
Consider the following about it.
1. These are low-pressure systems
which originate over the
Mediterranean Sea and western Asia.
2. They cause winter rains over the
plains and snowfall in the
Mountains.
3. They help in the cultivation of rabi
crops.

a.
b.
c.
d.

1 and 2 only
2 and 3 only
1 and 3 only
All of the above

70. Why are the western slopes of the


Western Ghats covered with thick
forests and not the eastern slopes?
a. Eastern slopes are regions of high
pressure that repel clouds which
results in lesser rainfall
b. Soil in Eastern slopes is less fertile
than that of western slopes
c. Lesser number of rivers are found on
the eastern side which results in
lesser evaporation and precipitation
d. Eastern slope falls in the rain shadow
area

71. Which of these Indian plants have antimicrobial and anti-biotic properties?
1. Neem
2. Tulsi
3. Arjun
4. Jamun
Choose the correct answer using the codes
below.
a.
b.
c.
d.

1 and 4 only
2 and 3 only
1 and 3 only
1 only

Choose the correct answer using the codes


below.
http://www.insightsonindia.com INSIGHTS

Page 16

INSIGHTS MOCK TEST - 30


72. In Geographical terms a Fault is


1. A linear break in rocks of the earths
crust in Horizontal or vertical
direction
2. A break in rocks in Oblique direction
3. Bend in the rock strata
Choose the correct answer using the codes
below.
a.
b.
c.
d.

1 and 2 only
2 and 3 only
1 and 3 only
1 only

73. Consider the following about the oceanic


resources that lie beyond the Exclusive
Economic Zone (EEZ)
1. It can be exploited only by the
countries that border the EEZ.
2. No individual country can utilize these
resources without the concurrence of
international institutions.
Which of the above is/are correct?
a.
b.
c.
d.

1 only
2 only
Both 1 and 2
None

74. The Sundarban Delta derives its name


from
a. it being the worlds largest delta
b. a local sundari tree which grows in
marshy areas
c. its magnificent beauty that is localy
called sundar
d. it being a large forest nearing a gulf
http://www.insightsonindia.com INSIGHTS

75. Which of the following are examples of


stock resources?
1. Extracting hydrogen from water to
produce energy
2. Extracting shale gas for diversifying
energy basket
3. Employing natural gas to produce
electrical energy
Choose the correct answer using the codes
below.
a.
b.
c.
d.

1 and 2 only
2 and 3 only
1 and 3 only
1 only

76. As the District Agriculture Advisor you


have visited a dry area and find the soil
to be sub-standard for production of
crops. You will recommend which of the
following to improve the productivity of
the soil?
1. Make the soil more acidic
2. Use mulching for improving soil
nutrients
3. Tighten the soil
Choose the correct answer using the codes
below.
a.
b.
c.
d.

1 and 2 only
2 and 3 only
1 and 3 only
2 only

Page 17

INSIGHTS MOCK TEST - 30


77. Developing countries face the problem


of environmental degradation even
more starkly than the developed
countries because of
1. Over-population
2. High incidence of Poverty
3. High consumption per capita
4. Capacity deficit in controlling factors
like forest fires, invasion of unwanted
ecological elements
Choose the correct answer using the codes
below.
a.
b.
c.
d.

1 and 2 only
2 and 3 only
4 only
1, 2 and 4 only

78. Forests and wastelands belonging to


both government and private
individuals and communities are known
as
a. Reserved Forests
b. Protected Forests
c. National classed reserves
d. Unclassed forests

79. Under Gramdan movement spread by


Vinoba Bhave
1. Any land greater than the legal
ceiling was forfeited by Zamindars to
the
government.
2. Villages were distributed to landless
poor people.
3. Villages were instructed to abandon
living spaces demarcated on the basis
of caste.

http://www.insightsonindia.com INSIGHTS

Choose the correct answer using the codes


below.
a.
b.
c.
d.

1 and 2 only
2 and 3 only
1 and 3 only
2 only

80.Ashokas concept of Dhamma included


1. Prohibition of animal sacrifices
2. Efficient organization of
administration for social welfare
3. Avoiding expensive and meaningless
ceremonies
4. Service to father and mother
Choose the correct answer using the codes
below.
a.
b.
c.
d.

1 and 2 only
2, 3 and 4 only
1 and 3 only
All of the above

81. Which of the following distinguish


Mathura school art from Gandhara art?
1. Mathura art developed completely on
indigenous lines unlike Gandhara
art.
2. The Buddha images under Mathura
art exhibited spiritual expressions
which was
largely absent in the
Gandhara school.
3. Mathura school also carved the
images of other deities unlike
Gandhara school.
Choose the correct answer using the codes
below.
Page 18

INSIGHTS MOCK TEST - 30


a.
b.
c.
d.

1 and 2 only
2 and 3 only
1 and 3 only
All of the above

82. Consider the following statements about


administration in Sangam age.
1. Hereditary monarchy was the form
of government.
2. There was no separate military wing
in the administration.
3. The state did not involve itself in
foreign trade.
4. The state did not rely on land
revenue for income.
Choose the correct answer using the codes
below.
a.
b.
c.
d.

1 and 2 only
2 and 4 only
All of the above
1 only

83. The famous Chinese pilgrim, Fahien


visited India during the reign of
Chandragupta II. The main purpose of
his visit was to
a. Understand the functioning of
monarchical government in Gupta
empire
b. Experience the culture and heritage
of India and write a literary piece on
it
c. See the land of the Buddha and to
collect Buddhist manuscripts from
India
d. None of the above

http://www.insightsonindia.com INSIGHTS

84. Champaran movement was started in


1917 in Bihar due to
a. High land rent demanded by the
Britishers from the tenants
b. Lack of security of tenure for tenants
working in British plantations
c. farmers of that region being forced to
grow indigo on their land
d. farmers not being paid rent for using
their land for plantation crops

85. In Hindu temples architecture, Vahana


is that part of the temple which is
a. the mount or vehicle of the temples
main deity
b. portico or colonnaded hall at the
entrance of the temple
c. a mountain-like spire at the front of
the temple
d. rhythmically projecting symmetrical
walls near the Garbhgriha of the
temple
86. Persian invasions by Cyrus and others
had which of the following implications
for India?
1. Boost in Indo-Iranian commerce
2. Kharoshti, an Iranian writing became
popular in north-western India.
3. It helped temporarily avert invasion
by Alexander.
Choose the correct answer using the codes
below.
a.
b.
c.
d.

1 and 2 only
2 and 3 only
1 and 3 only
All of the above
Page 19

INSIGHTS MOCK TEST - 30


87. Consider the following statements.


1. Samudragupta annexed all his
conquered territories of South India.
2. The North Indian conquests of
Samudragupta did not result in the
expansion of the Gupta Empire.
3. Feats of the Gupta Empire are only
recorded in the writings of foreign
visitors to India.
Choose the correct answer using the codes
below.
a.
b.
c.
d.

1 and 2 only
2 and 3 only
1 and 3 only
None of the above

88. Grant of pattas by zamindars to farmers


was a main feature of the Permanent
Settlement system. This grant was
widely not successful because
a. British government later opposed the
move of granting pattas to farmers
b. A very large number of farmers did
not register themselves for the grant
c. There was a nexus between British
officials and Zamindars who allotted
land amongst themselves
d. The zamindars were not officially
accountable to the British
government for grant of pattas

89. The Chalukyan administration was


characterized by
1. High centralization in administration
2. Absence of Village autonomy
3. Maritime power and small standing
army
http://www.insightsonindia.com INSIGHTS

Choose the correct answer using the codes


below.
a.
b.
c.
d.

1 and 2 only
2 and 3 only
1 and 3 only
All of the above

90. The English East India Company wanted


their first trading centre to be set-up at
Surat. The permission was however
refused by the Mughal emperor there
because
a. The Mughal Emperor was against the
trade practices of the British
b. There was portugese pressure on the
Emperor
c. He was not interested in foreign
trade and investment
d. The British did not promise adequate
revenue from the trading centre to
the Emperor

91. In the British-French rivalry to take


control of Indian trade resulted in
French failure. Which of the following
can be the possible causes of the failure
of the French?
1. Commercial and naval superiority of
the British
2. Englands victory in the European
wars
3. Lack of support of French from any
Mughal emperor
Choose the correct answer using the codes
below.
Page 20

INSIGHTS MOCK TEST - 30


a.
b.
c.
d.

1 and 2 only
2 and 3 only
1 and 3 only
All of the above

92. The Poona pact of 1932


1. Was a formal outcome of the Third
Round Table Conference
2. Ensured reservation for depressed
classes in all public institutions
3. Provided for the British government
to take welfare measures for the
depressed classes
Choose the correct answer using the codes
below.
a.
b.
c.
d.

1 and 2 only
2 and 3 only
1 only
None of the above

93. In 1854, Sir Charles Wood sent a


comprehensive dispatch as a grand plan
on education. The plan laid emphasis on
1. Education in vernacular languages
2. Provision of education only by public
institutions
3. Westernization of Indian education
Choose the correct answer using the codes
below.
a.
b.
c.
d.

94. The Vernacular Press Act passed in 1878


ensured that
a. nothing would be published by the
English government in any
vernacular newspaper
b. nothing seditious could be published
that affects the territorial integrity of
British India
c. the vernacular newspapers were not
exposed to pre-censorship by the
British government
d. no Indian national freedom fighter
could establish a vernacular press
company in India

95. Consider the following statements.


Assertion (A): Local self-government bodies
were not recognized officially till Indian
Independence.
Reason (R): The British did not enact any
special legislation for the governance of local
bodies.
In the context of the statements above, which
of these is true?
a) A and R both are true, and R is the
correct explanation for A.
b) A and R both are true, and R is the
NOT the correct explanation for A.
c) A is correct, R is incorrect.
d) A is incorrect, R is correct.

1 and 2 only
2 and 3 only
1 and 3 only
3 only

http://www.insightsonindia.com INSIGHTS

Page 21

INSIGHTS MOCK TEST - 30


96. In the Parliament of Religions held in


Chicago (USA) in 1893, Swami
Vivekananda gave the message of
a. Essential unity of all religions
b. Superiority of Indian culture over all
other cultures in the world
c. A classless society guided by core
human values
d. All of the above

97. Even though the moderates were not


very popular among the extremists, they
succeeded on a number of fronts. They
were successful at
1. popularizing the ideas of democracy,
civil liberties and representative
institutions
2. explained British exploitation of the
Indian economy
3. getting the legislative councils
expanded for Indian benefit
Choose the correct answer using the codes
below.
a.
b.
c.
d.

1 and 2 only
2 and 3 only
1 and 3 only
All of the above

98. The Home Rule Movement of 1916 did


NOT emphasize
1. Revival of Swadeshi
2. Participation of Indians in selfgoverning institutions for India
3. Complete erosion of British
sovereignty over India
4. Separate electorates for all
communities of India
http://www.insightsonindia.com INSIGHTS

Choose the correct answer using the codes


below.
a.
b.
c.
d.

1 and 2 only
2 and 3 only
1 and 3 only
3 and 4 only

99. When Warren Hastings assumed the


administration of Bengal in 1772, he
found it in utter chaos and thus
abolished the dual system. After the
abolition of the dual system
a. The East India Company acted both
as Diwan and collector of revenue by
its own agents
b. The British handed over financial
powers to Mughal administration
and focussed only on revenue
collection
c. The British handed financial
administration to the Mughals and
fixed a share from revenue for every
year
d. None of the above

100.
On what grounds do the National
and State government have control over
religious education imparted in minority
institutions?
a) On grounds of protecting the
culture of minorities
b) On grounds of promoting
scientific temper in society
c) On grounds of national unity and
integrity
d) The government cannot control
religious education imparted in
minority institutions.
Page 22

INSIGHTS MOCK TEST - 30


http://www.insightsonindia.com INSIGHTS

Page 23

INSIGHTS MOCK TEST SERIES 2015: TEST 30 SOLUTIONS


1. Solution: c)
Jaya Prakash once stated that Total Revolution is a combination of seven revolutions, viz.,
political, social, economic, cultural, ideological or intellectual, educational and spiritual; and the
main motive being to bring in a change in the existing society that is in tune with the ideals of the
Sarvodaya.
JP had a very idealistic notion of society and it is in this endeavor, he shifted from Marxism to
Socialism and later towards Sarvodaya.
By the early 1970s, JP completely withdrew from party and power politics, and concentrated more
on social regeneration through peaceful means.
JP Narayanan has been in news for some time.

2. Solution: a)
http://timesofindia.indiatimes.com/city/varanasi/Jaitleys-Buddha-Circuit-plan-is-amasterstroke/articleshow/38184422.cms
The complete map of the Buddhist Circuit comprises Lumbini in Nepal, Bodh Gaya, Vaishali and
Rajgir in Bihar and Sarnath in Varanasi, Shravasti and Kushinagar.
The Buddh International Circuit is also a motor racing circuit in Greater Noida, Uttar Pradesh. The
circuit is best known as the venue for the annual Formula One Indian Grand Prix, which was first
hosted in October 2011 but has been suspended since the 27th of October 2013 due to an ongoing
tax dispute with the Uttar Pradesh government.

3. Solution: c)
This is a centenary (100 years) for Tagores Gitanjali Noble prize award. These lines are the very
first lines of Gitanjali. Hence, such unexpected questions may be expected by UPSC.
http://www.thehindu.com/features/friday-review/music/1000-voices-bengali-choir-celebrate100-years-of-tagores-gitanjali/article7382975.ece
You can read interesting stuff about Gitanjali here (not very useful for the exam though) http://www.sacred-texts.com/hin/tagore/gitnjali.htm

4. Solution: b)

http://www.insightsonindia.com INSIGHTS

Page 1

INSIGHTS MOCK TEST SERIES 2015: TEST 30 SOLUTIONS


As per the census there are 24.39 crore total number of households in the country, of which 17.91
crore live in villages. Of these, 10.69 crore households are considered as deprived. 50 percent or
5.37 crore deprived households in rural areas are landless and depend on manual labour for
livelihood.
House hold size: Average size of a rural Indian household is 4.93. Highest in Uttar Pradesh at 6.26.
Lowest in Andhra Pradesh at 3.86.
Male dominance: majority of 87 per cent households is male-headed. Nearly 13 per cent do have a
female head. In rural areas of Rajasthan, around 91 per cent households are headed by men. In
Kerala 26 per cent are women-headed households (highest among the states).
Literacy: Over one-third of population living in rural areas is illiterate. Higher literacy in terms of
percentage is in Kerala (88.62), Delhi (86.42), Goa (84.58), Sikkim (79.88) and Himachal Pradesh
(77.95). Lowest literacy is in Rajasthan (58), Madhya Pradesh (44.19), Bihar (43.85), Telangana
(40.42) and Chattisgarh (39.59). SC and ST Population:Large proportion of the households across
the country belong to the SC and ST category i.e. about 30 per cent of rural households.
Employment: Rural India is largely self-employed or employed in the unorganised sector. Around
10 per cent households are on salaried jobs of which the majority are in government jobs.
5. Solution: c)
Forex reserves do not serve any useful purpose apart from acting as a hedge against BoP crisis; to
maintain the value of rupee in international market in case of excessive volatility; and to undertake
sterilization in the economy.
Therefore, there was a proposal to use them for infrastructure projects.
While the proposal look simple and straight enough, there are a number of strategic considerations
like nature of financing; risk in projects; BoP status; effect on money supply and inflation etc.
A decision should be made considering all the above factors.
http://www.thehindu.com/news/national/centre-wants-to-break-forexreserves/article7400569.ece
6. Solution: a)
Eligibility to get rights under the Act is confined to those who "primarily reside in forests" and who
depend on forests and forest land for a livelihood. Further, either the claimant must be a member
of the Scheduled Tribes scheduled in that area or must have been residing in the forest for 75 years

Title rights - i.e. ownership - to land that is being farmed by tribals or forest dwellers as on
13 December 2005, subject to a maximum of 4 hectares; ownership is only for land that is
actually being cultivated by the concerned family as on that date, meaning that no new
lands are granted
Use rights - to minor forest produce (also including ownership), to grazing areas, to
pastoralist routes, etc.

http://www.insightsonindia.com INSIGHTS

Page 2

INSIGHTS MOCK TEST SERIES 2015: TEST 30 SOLUTIONS


Relief and development rights - to rehabilitation in case of illegal eviction or forced


displacement; and to basic amenities, subject to restrictions for forest protection
Forest management rights - to protect forests and wildlife

7. Solution: d)
Almost all States have performed poorly in reducing the number of underweight adolescent girls.
Uttar Pradesh still has the highest levels of child stunting, with over 50 per cent of the children
under the age of five underdeveloped, meaning that their height is more than two standard
deviations less than the expected height for their age for that population.
Jharkhand, meanwhile, has the highest number of underweight children under the age of five,
meaning their weight for age is more than two standard deviations less than what would be
expected.
Kerala remains the best performing State in the number of child stunting cases, while Manipur and
Mizoram have the lowest numbers of underweight children.

8. Solution: d)
The stabilizing tendency of the market is often know as the invisible hand.
When there is excess supply or lower demand for a commodity, price of a commodity lowers. This
is turn revives the demand in the market and increases production of the commodity. The reverse
is also true.
Optimizing behaviour of the individuals means that every individual acts rationally in their
maximum self-interest. This is at the core of the principle of market forces.
Perfect competition ensures that all such transactions take place in an unrestricted and nonmonopoly markets.
9. Solution: c)

http://www.insightsonindia.com INSIGHTS

Page 3

INSIGHTS MOCK TEST SERIES 2015: TEST 30 SOLUTIONS


http://timesofindia.indiatimes.com/city/chandigarh/Haryanas-Bhirrana-oldest-Harappan-siteRakhigarhi-Asias-largest-ASI/articleshow/46926693.cms
Rakhigarhi, or Rakhi Garhi is a village in Hisar District in the state of Haryana in India, situated in
the north-west about 150 kilometers from Delhi. In 1963, archaeologists discovered that this place
was the site of the largest known city of the Indus-Sarasvati civilization, much larger and ancient
than Harappa and Mohenjodaro sites.
It is situated on the dry bed of the Sarasvati river, which is believed to have once flown through
this place and dried up by 2000 BC.
According to the archaeologists, Rakhigarhi is an ideal nucleus from where the Harappan
civilisation began in the Ghaggar basin in Haryana and gradually grew from here and slowly
expanded to the Indus valley.

10. Solution: a)
http://www.thehindu.com/opinion/op-ed/prime-minister-narendra-modis-central-asiavisit/article7396013.ece
Here is the text from the article which discusses radicalism in Central Asia:
Prime Minister Narendra Modis eight-day visit to the five Central Asian States Uzbekistan,
Kazakhstan, Kyrgyzstan, Tajikistan and Turkmenistan is taking place at the most opportune
juncture, for Mr. Modi has indicated that he proposes to focus on the radical Islamist threat to the
region. Given the kind of extremist winds sweeping across the region, the Muslim populations of
these states face uncertain times. States such as Tajikistan are especially vulnerable, as many from
the ranks of their security agencies are beginning to join the Islamic State (IS).
The threats that these states face from radical Islamist elements are, indeed, real. At the same
time, it is also significant that the leadership of these Central Asian States should look to India to
provide them with answers on how to insulate their Muslim populations from these kinds of
threats. Indias success, to date, in insulating its own Muslim population from such radicalism has
gained wide acceptance, even as the so-called counter radicalisation programmes followed in the
West are proving to be a failure.
11. Solution: d)
http://www.forbes.com/sites/timworstall/2015/04/18/indias-change-in-gdp-calculation-methodseems-highly-sensible/
The CSO is responsible for coordination of statistical activities in the country, including National
Income Accounting; conduct of Annual Survey of Industries, Economic Censuses, compilation of
Index of Industrial Production, Consumer Price Indices etc. The most important indices used for
compilation of GDP are Index of Industrial Production (IIP), Wholesale Price Index (WPI), and
Consumer Price Indices for Industrial Workers (CPI-IW), Agricultural Labourers (CPI-AL) and
Rural Labourers (CPI-RL).
http://www.insightsonindia.com INSIGHTS

Page 4

INSIGHTS MOCK TEST SERIES 2015: TEST 30 SOLUTIONS


This data is compiled by central and state ministries, independent departments, and other
governmental organisations. For example, the Industrial Statistics Unit at the Department of
Industrial Policy & Promotion, Ministry of Commerce & Industry, provides production data to the
CSO for compiling IIP. Similarly, commodity-wise data on import-export, production, crop, area,
WPI, CPI etc. is collected and analysed by the Price Monitoring Cell, Department of Consumer
Affairs, Ministry of Consumer Affairs, Food and Public Distribution.

12. Solution: a)
Article 29 provides that any section of the citizens residing in any part of India having a distinct
language, script or culture of its own, shall have the right to conserve the same.
Further, no citizen shall be denied admission into any educational institution maintained by the
State or receiving aid out of State funds on grounds only of religion, race, caste, or language.
Their rights are to be protected because their future in a democracy is vulnerable to the will of the
majority.
13. Solution: d)
The exact cause of the stock market crash is not clear. It is still being probed. However, speculative
trading is likely to be blamed.
Read here to understand what is the impact of the crash and what can be the possible causes
http://economictimes.indiatimes.com/markets/stocks/news/chinese-police-find-clues-of-3-2trillion-stock-market-crash/articleshow/48042250.cms
Stocks finally surged few days ago due to several aggressive countermeasures taken by Chinese
authorities.
These countermeasures include an interest rate cut, curbing IPOs, loosening margin requirements,
allowing the use of property as collateral for margin loans, and encouraging brokerage firms to buy
stocks with cash from the Peoples Bank of China. Short sellers have even been threatened with
arrest, which has raised eyebrows around the world.
14. Solution: b)
The powerful Typhoon Linfa has hit the southern China after it made landfall on southern
province of Guangdong.
Typhoon Linfa has hit the southern province by bringing high speed winds along with heavy rain.
However after it had made landfall, the typhoon has lost strength rapidly after it passed near Hong
Kong due to weakened low pressure area.
Typhoon Linfa is the first of two large storms Chan-Hom and Nangka which are going to hit China.
The third storm, Nangka is the most powerful among them and has reached supertyphoon
strength with winds of 250 km/hr. But it has strong presence far out in the Pacific Ocean and not
expected to hit any land with high strength.
http://www.insightsonindia.com INSIGHTS

Page 5

INSIGHTS MOCK TEST SERIES 2015: TEST 30 SOLUTIONS


15. Solution: d)
It was indigenously developed by the DRDO and Bharat Electronics Ltd (BEL) as part of
Integrated Guided Missile Development Programme (IGMDP) of the 1980s.

Manufactured using 92 per cent of indigenous machinery and can be transported anywhere
via road, water or air transport.
It is a potent supersonic mobile multi-target point, multi-directional missile, capable to
engage multiple air targets simultaneously using sophisticated multi-function phased array
radars.
Capable to neutralise aerial targets like cruise missiles, fighter jets and air-to-surface
missiles with strike range of 25km.
It can destroy aerial target upto 30 km away at altitudes upto 18,000 m.
It has supersonic speed i.e. it is three times faster than that of sound and capable to carry
conventional warhead of 60 kg.

16. Solution: a)
The first system has been installed in its capital Dehradun which will issue warnings 1-40 seconds
before earthquakes of magnitude 5 or more occur.
The system is designed and manufactured by Italian firm, Space Dynamics which already has
installed it in Japan, Italy and US.
The system is incorporated with sensors which are capable to detect P (Primary) and S
(Secondary) waves generated during an earthquake. It is capable to detect the harmless P wave,
which travels faster than the S wave for advance warning. The system has the earliest detection
among those currently in use worldwide.
17. Solution: a)
Under the provisions of FATCA, it is mandatory for all financial institutions outside of the US to
periodically transmit information on financial accounts held by US citizens to its US Internal
Revenue Service.
The agreement will promote transparency on tax matters and curb offshore tax evasion by
exchange of information between the two countries. It is obligatory on signatories to exchange a
wide range of financial information among themselves automatically and periodically.
Henceforth, Indian financial institutions will have to reveal information about US tax payers to the
revenue department. This information will be further passed on to the US tax authorities. In turn,
the US will also share similar kind of financial information with India.
18. Solution: d)
India and Kazakhstan have recently signed five agreements in various sectors focusing on
strategic, economic and trade cooperation between both nations.
http://www.insightsonindia.com INSIGHTS

Page 6

INSIGHTS MOCK TEST SERIES 2015: TEST 30 SOLUTIONS


Kazakhstan the worlds largest producer of uranium will supply a total of 5,000 tonnes of the
natural uranium to India during the 2015-19. Thus it will play important role in Indias energy
requirement and energy security.
Apart from signing these agreements, India has also shown interest in fertilizer sector in
Kazakhstan due to abundance availability of Natural Gas; and hydro carbon sectors of Kazakhstan
for drilling and production of crude oil.
19. Solution: d)
These villages will be developed under the Border Area Development Programme (BADP) with
special emphasis on promotion of sports activities and rural tourism.
The funds under BADP for this initiative will be provided to the states as a 100 per cent nonlapsable special central assistance.
It will be supplemented from the budgetary allocation of Rs. 990 crore for the financial year 201516. The development of villages under BADP will include schemes such as Swachh Bharat Abhiyan,
Promotion of sports activities in border areas.
Skill Development Programmes along with promotion of rural tourism (i.e. border tourism) and
protection of heritage sites.
It will also cover construction of helipads in remote and inaccessible hilly areas, which do not have
road connectivity, skill development training to farmers for use of modern or scientific technique
in farming, organic farming etc.
20. Solution: a)
Japans Meiji-era (1868-1912) industrial revolution sites have received World Cultural Heritage
status from United Nations Educational, Scientific and Cultural Organization (UNESCO).
Located southwest of Japan, it includes series of eleven properties that comprises Iron and Steel,
coalmines and shipyards industries. It contributed in Japans evolution from feudalism to a
successful industrialised modern economy.
It traces the development phase of Japans heavy industries and its integration to Western
technology along with fundamental changes in its traditional culture.
21. Solution: c)
In economics, the Phillips curve is a historical inverse relationship between rates of unemployment
and corresponding rates of inflation that result in an economy. Stated simply, decreased
unemployment, (i.e., increased levels of employment) in an economy will correlate with higher
rates of inflation.
The topic has been asked by UPSC a few times in Prelims as well as Mains, hence its important to
know the concept.
http://www.insightsonindia.com INSIGHTS

Page 7

INSIGHTS MOCK TEST SERIES 2015: TEST 30 SOLUTIONS


22. Solution: b)
One of the policy objectives of RBI is to ensure high-quality corporate governance in banks. RBI
has issued guidelines for fit and proper criteria for director of banks. One of these guidelines is
that the directors of the banks should have special knowledge / experience in the various banking
related areas. RBI can also appoint additional directors to the board of a banking company, and
supersede the decisions of the boards.
The interest rates on most of the categories of deposits and lending transactions have been
deregulated and are largely determined by banks. Reserve Bank regulates the interest rates on
savings bank accounts and deposits of non-resident Indians (NRI), small loans up to rupees two
lakh, export credits and a few other categories of advances.

23. Solution: b)
The system of gold pegging was abolished long time ago.
There is some confusion regarding the interchange of the currency with gold, but the system that
India follows is that money cannot be exchanged for gold under any circumstances due to gold's
lack of liquidity; therefore, money cannot be changed into gold by the RBI. India follows the same
principle as Great Britain and the US.
Reserve Bank of India clarifies its position regarding the promissory clause printed on each
banknote:
"As per Section 26 of Reserve Bank of India Act, 1934, the Bank is liable to pay the value of
banknote. This is payable on demand by RBI, being the issuer. The Bank's obligation to pay the
value of banknote does not arise out of a contract but out of statutory provisions.The promissory
clause printed on the banknotes i.e., "I promise to pay the bearer an amount of X" is a statement
which means that the banknote is a legal tender for X amount. The obligation on the part of the
Bank is to exchange a banknote for coins of an equivalent amount.

24. Solution: d)
Apart from the product method, there are two other ways of GDP calculation.
Income Method:
Under this method, national income is measured as a flow of factor incomes. There are generally
four factors of production labour, capital, land and entrepreneurship. Labour gets wages and
salaries, capital gets interest, land gets rent and entrepreneurship gets profit as their
remuneration.

http://www.insightsonindia.com INSIGHTS

Page 8

INSIGHTS MOCK TEST SERIES 2015: TEST 30 SOLUTIONS


Besides, there are some self-employed persons who employ their own labour and capital such as
doctors, advocates, CAs, etc. Their income is called mixed income. The sum-total of all these factor
incomes is called NDP at factor costs.
Expenditure Method:
In this method, national income is measured as a flow of expenditure. GDP is sum-total of private
consumption expenditure. Government consumption expenditure, gross capital formation
(Government and private) and net exports (Export-Import).

25. Solution: d)
This article explains the impact of subsidy very well. Go through the entire article.
http://www.business-standard.com/article/opinion/rajiv-shastri-why-india-s-subsidies-areinflationary-114020200738_1.html
26. Solution: d)
Greater FDI, FII and other capital flows inside the country become more profitable if the interest
rates are low.
For e.g. a foreign investor will be able to earn greater returns if the investment money is available
at a lower interest rate.
Moreover, NRIs are less likely to keep their deposits in India if the interest rate offered is not
competitive as compared to their present residence country.
27. Solution: d)
In economics, a transfer payment (or government transfer or simply transfer) is a redistribution of
income in the market system. These payments are considered to be non-exhaustive because they
do not directly absorb resources or create output.
Transfer payments are made to individuals through programs such as Social Security, Welfare and
Veteran's benefits.
Pensions and other such schemes that benefit the public are also covered under transfer payments.
28. Solution: d)
Refer to this article for understanding the major issues associated with the scheme.
http://www.firstpost.com/business/economy/gold-monetization-scheme-why-offering-higherrate-of-interest-is-critical-for-its-success-2284738.html
Under the GMS, an individual or entity can walk into a test centre and gets the gold melted, purity
assessed and converted into bars, against which the bank will issue a certificate to the holder. On
this deposit, the customer earns an interest decided by the bank, which will be exempted from
http://www.insightsonindia.com INSIGHTS

Page 9

INSIGHTS MOCK TEST SERIES 2015: TEST 30 SOLUTIONS


income, wealth or capital gains taxes. On maturity, the customer can get the gold or cash back plus
interest amount.
High inflation will erode the interest rate offered on the gold and people will again flock back to
Gold.
Also public emotional attachment to gold may not allow them to melt the metal and convert it into
gold bars.
Statement 3 is already taken care in the scheme.

29. Solution: d)
All the members of a municipality shall be elected directly by the people of the municipal area. For
this purpose, each municipal area shall be divided into territorial constituencies to be known as
wards.
The State government does not directly appoint the members. The state legislature may only
provide for the manner of reservation of offices of chairpersons in the municipalities for SCs, STs
and women.
The state legislature may (a) authorise a municipality to levy, collect and appropriate taxes, duties,
tolls and fees; (b) assign to a municipality taxes, duties, tolls and fees levied and collected by state
government; (c) provide for making grants-in-aid to the municipalities from the consolidated fund
of the state; and (d) provide for constitution of funds for crediting all moneys of the municipalities.
30. Solution: c)
The Assembly was made a fully sovereign body, which could frame any Constitution it pleased. The
act empowered the Assembly to abrogate or alter any law made by the British Parliament in
relation to India.
The Assembly also became a legislative body. In other words, two separate functions were assigned
to the Assembly, i.e. making of a constitution for free India and enacting of ordinary laws for the
country.
In the CA, the representatives of each community were to be elected by members of that
community in the provincial legislative assembly and voting was to be by the method of
proportional representation by means of single transferable vote.
The representatives of princely states were to be nominated by the heads of the princely states. It is
thus clear that the Constituent Assembly was to be a partly elected and partly nominated body.
The PM was appointed by the party in majority.
http://www.insightsonindia.com INSIGHTS

Page 10

INSIGHTS MOCK TEST SERIES 2015: TEST 30 SOLUTIONS


31. Solution: d)
The government may resign only if it fails to pass the budget in the Parliament. Failure in passing
regular public bills is not considered a defeat of the government. Passing of a no-confidence
motion does.
PM can be the member of any house. It is not a convention that he must be from the lower house,
neither has it been codified.
Statement 3 is more of a political tool rather than a codified convention.

32. Solution: b)
The very existence of the council depends on the will of the assembly. The council can be abolished
by the Parliament on the recommendation of the assembly.
The position of the council vis-a-vis the assembly is much weaker than the position of the Rajya
Sabha vis-a-vis the Lok Sabha.
The council is subordinate to the assembly in all respects.

33. Solution: d)
This issue was in news because of government planning to introduce Yoga in school curriculums;
and some Indian cricket fans being booked for sedition for supporting Pakistani cricket team.
We enjoy Right to freedom of Speech, Expression, Movement, belief and so on.
All of the activities, if forced, in some way or the other violate these fundamental rights.
A law that makes these compulsory can be challenged in Supreme Court or High courts.

34. Solution: d)
The Preamble in its present form reads:
We, THE PEOPLE OF INDIA, having solemnly resolved to constitute India into a SOVEREIGN
SOCIALIST SECULAR DEMOCRATIC REPUBLIC and to secure to all its citizens.......
This clearly tells that the source of authority of the constitution is the people of India.
A democratic polity, as stipulated in the Preamble, is based on the doctrine of popular sovereignty,
that is, possession of supreme power by the people.

http://www.insightsonindia.com INSIGHTS

Page 11

INSIGHTS MOCK TEST SERIES 2015: TEST 30 SOLUTIONS


Universal adult franchise, periodic elections, rule of law, independence of judiciary, and absence of
discrimination on certain grounds are the manifestations of the democratic character of the Indian
polity.

35. Solution: c)
Public-Interest Litigation is litigation for the protection of the public interest. In Indian law,
Article 32 of the Indian constitution contains a tool which directly joins the public with judiciary.
A PIL may be introduced in a court of law by the court itself (suo motu), rather than the aggrieved
party or another third party. For the exercise of the court's jurisdiction, it is not necessary for the
victim of the violation of his or her rights to personally approach the court. In a PIL, the right to
file suit is given to a member of the public by the courts through judicial activism. The member of
the public may be a non-governmental organization (NGO), an institution or an individual.
The Supreme Court of India, rejecting the criticism of judicial activism, has stated that the
judiciary has stepped in to give direction because due to executive inaction, the laws enacted by
Parliament and the state legislatures for the poor since independence have not been properly
implemented.
Subodh Markandeya well known Senior Advocate of Supreme court of India and Judicial activist
believes that public interest litigation is the principal legal remedy For a common man and it is
main weapon of judicial activist .

36. Solution: c)
Any provision that changes the composition of the legislative bodies will require a constitutional
amendment. It is a major change and hence will require a higher majority in Parliament.
A political party is a self-governing voluntary body and is not a legislative or executive body in that
sense. A change in Societies Act will do for the political parties.
Hence, this is another solution advocated by parties to break the deadlock in Parliament over
women reservation.

http://www.insightsonindia.com INSIGHTS

Page 12

INSIGHTS MOCK TEST SERIES 2015: TEST 30 SOLUTIONS


37. Solution: c)

Third Schedule Forms of Oaths or Affirmations for MPs, Ministers, Constitutional functionaries
etc.
Fourth Schedule - Allocation of seats in the Rajya Sabha to the states and the union territories.
Fifth Schedule - Provisions relating to the administration and control of scheduled areas and
scheduled tribes.
Sixth Schedule - Provisions relating to the administration of tribal areas in the states of Assam,
Meghalaya, Tripura and Mizoram.
Seventh Schedule - Division of powers between the Union and the States in terms of List I
(Union List), List II (State List) and List III (Concurrent List). Presently, the Union List contains
100 subjects (originally 97), the state list contains 61 subjects (originally 66) and the concurrent
list contains 52 subjects (originally 47).
Eighth Schedule - Languages recognized by the Constitution.

38. Solution: a)
None of the options is direct. You can work out by elimination.
Option B is false because the representatives are not supposed to command administration.
Option C is wrong because no branch of the government is subordinate to any other branch.
Option D is wrong because administration is directly controlled by Ministers as head of Ministries
and departments.
Administration is a part of the executive branch. Hence, Option A.

39. Solution: d)
http://www.insightsonindia.com INSIGHTS

Page 13

INSIGHTS MOCK TEST SERIES 2015: TEST 30 SOLUTIONS


Unlike the Rajya Sabha, the Lok Sabha is not a continuing chamber. Its normal term is five years
from the date of its first meeting after the general elections, after which it automatically dissolves.
However, the President is authorised to dissolve the Lok Sabha at any time even before the
completion of five years and this cannot be challenged in a court of law.
Further, the term of the Lok Sabha can be extended during the period of national emergency be a
law of Parliament for one year at a time for any length of time. However, this extension cannot
continue beyond a period of six months after the emergency has ceased to operate.

40. Solution: b)
The Table of Precedence is related to the rank and order of the officials of the Union and State
Governments. The present notification on this subject was issued on 26 July, 1979. This
notification superseded all the previous notifications and was also amended many times. The
updated version of the Table, containing all the amendments made therein so far (2013), is given
below:
1. President
2. Vice-President
3. Prime Minister
4. Governors of states within their respective states
5. Former presidents
5A. Deputy Prime Minister
6. Chief Justice of India
Speaker of Lok Sabha
7. Cabinet Ministers of the Union
Chief Ministers of States within their respective States
Deputy Chairman, Planning Commission
Former Prime Ministers
Leaders of Opposition in Rajya Sabha and Lok Sabha

41. Solution: b)

http://www.insightsonindia.com INSIGHTS

Page 14

INSIGHTS MOCK TEST SERIES 2015: TEST 30 SOLUTIONS


Article 355 imposes two duties on the Centre: (a) to protect every state against external aggression
and internal disturbance; and (b) to ensure that the government of every state is carried on in
accordance with the provisions of the Constitution.
This shows that the Union government has been given prominence over the state government. This
shows the federal character with a Union bias i.e. quasi-federal nature of the Indian polity. The
Union government has a responsibility to ensure the territorial integrity and security of the nation.
42. Solution: d)
This is a very comprehensive article that discussed every aspect of Lok Adalats. Go through the
article selectively.
http://www.legalserviceindia.com/articles/lok_a.htm

43. Solution: d)
Service tax is a tax levied by the government on service providers on certain service transactions,
but is actually borne by the customers. It is categorized under Indirect Tax and came into existence
under the Finance Act, 1994.
It is charged to the individual service providers on cash basis, and to companies on accrual basis.
This tax is payable only when the value of services provided in a financial year is more than Rs 10
lakh. This tax is not applicable in the state of Jammu & Kashmir.
44. Solution: a)
The 2nd schedule of the constitution fixes the allowances, privileges, emoluments of President,
Governor, CJI and other functionaries. It can be revised only after amending this part of the
constitution. These are not subject to the annual vote of the Parliament.
91st amendment fixed the maximum strength of the Council of Ministers at 15% of the strength of
the LS.
While the manner of election of the Panchs is fixed by constitution in form of adult franchise, the
state legislature decides on the manner of election of the Sarpanch.
45. Solution: d)
Annual budget is a money bill.
A Money Bill can be introduced only in the Lok Sabha and not in the Rajya Sabha.
Rajya Sabha cannot amend or reject a Money Bill. It should return the bill to the Lok Sabha within
14 days, either with recommendations or without recommendations.
The Lok Sabha can either accept or reject all or any of the recommendations of the Rajya Sabha. In
both the cases, the money bill is deemed to have been passed by the two Houses.
http://www.insightsonindia.com INSIGHTS

Page 15

INSIGHTS MOCK TEST SERIES 2015: TEST 30 SOLUTIONS


46. Solution: b)
US is hoping to strengthen democracy in Tunisia especially after Beji Caid Essebsi became the first
democratically elected President in 60-year history of Tunisia.
By granting the MNNA status, both countries will enhance military cooperation between
them. The strong relations between both democracies will help to maintain peace and security in
Tunisia as it was the birthplace of Arab Spring (Jasmine revolution) revolts across the Middle East
in 2011.
It will also help Tunisia to counter the rising threat of jihadist groups especially ISIS in the region.
47. Solution: a)
Indian-origin Sunita Williams is among four veteran astronauts chosen by National Aeronautics
and Space Administration (NASA) to fly the first commercial space vehicles.
NASA had chosen Williams for the astronaut programme in 1998. Prior to this she was a US Navy
captain and was a helicopter pilot with more than 3,000 flight hours experience in more than 30
different aircraft. Presently, she ranks 6th on the all-time US endurance list and 2nd all-time for a
female astronaut. She holds the world records for most spacewalk time by a woman astronaut (50
hours, 40 minutes) and most spacewalks by a woman astronaut (seven times).
Kalpana Chawla was the first Indian-American astronaut and first Asian woman in space. She first
flew on Space Shuttle Columbia in 1997 as a mission specialist and primary robotic arm operator.

48. Solution: a)
As Iran has observer status in the SCO, it will serve as a platform for India to boost trade through
the Iranian ports of Bandar Abbas and Chabahar. These ports are considered as Indias gateway to
Central Asia through International North-South Transport Corridor (INSTC).
Thus, it will open up trade, energy sector and strategic transit routes for India between Russia,
Central Asia and China.
SCO will provide platform for the security grouping for India and Pakistan especially on issue to
counter terrorism.
It will also provide a valuable interface to engage with security concern of Afghanistan especially
its members including India due to pullout international troops.

http://www.insightsonindia.com INSIGHTS

Page 16

INSIGHTS MOCK TEST SERIES 2015: TEST 30 SOLUTIONS


SCO may also serve as guarantor for projects such as the Turkmenistan-Afghanistan-PakistanIndia (TAPI) and Iran-Pakistan-India (IPI) pipelines, which are held by India due to security
concerns.
Balance in International Politics: In this politically polarised world, SCO will play an important
role in counter-balancing Indias perceived tilt on security issues towards US and its allies. Thus, it
can help to maintain full balance of Indias relations with the great powers globally.
49. Solution: c)
RATS SCO, headquartered in Tashkent (Uzbekistan), is a permanent organ of the Shanghai
Cooperation Organization which serves to promote cooperation of Member States against
terrorism, separatism, and extremism, which it refers to as "the three evil forces."
RATS SCO includes 8 Member States (the Republic of Kazakhstan, the People's Republic of China,
the Kyrgyz Republic, the Russian Federation, the Republic of Tajikistan, and the Republic of
Uzbekistan, India and Pakistan), three observers, and three dialogue partners.
To ensure the safety of border areas, the border defence cooperation mechanism under the
framework of RATS SCO was established. RATS SCO has also collected and distributed to its
Member States intelligence information regarding the use of the Internet by terrorist groups active
in the region to promote their ideas.
Understand the operations of SCO here http://www.un.org/en/sc/ctc/news/2014-1024_cted_shanghaicoop.html
http://ecrats.org/en/

50. Solution: a)
For Rice, alluvial rich soil is needed. With that warm and moist climate is also required.
For Millets, dry and less fertile soil also will do. That is why it is grown in semi-arid areas.
For Barley too warm and moist climate is needed. It can do without rich soil.
Tea requires cool climate even with a poor soil.

51. Solution: d)
This is the description of the Sarnath Pillar.

http://www.insightsonindia.com INSIGHTS

Page 17

INSIGHTS MOCK TEST SERIES 2015: TEST 30 SOLUTIONS


https://www.khanacademy.org/humanities/art-asia/south-asia/buddhist-art2/a/lion-capitalashokan-pillar-at-sarnath
The pillars have four component parts in two pieces: the three sections of the capitals are made in
a single piece, often of a different stone to that of the monolithic shaft to which they are attached
by a large metal dowel.

52. Solution: b)
Antarctica has the highest mean elevation among all continents.
All open oceans have same mean level because they are all connected with each other.
The deepest part of the ocean is more than 12,000 Km deep (Mariana trench); whereas Mount
Everest is about 8.3 km high.

53. Solution: a)
Turkey and Lebanon are comparatively older democracies than Egypt, Libya or Tunisia which
became democracies right after the Arab Spring.
In 2011, a revolution resulted in the overthrow of the autocratic President Zine El Abidine Ben Ali
followed by Tunisias first free elections. Since then, Tunisia has been consolidating democracy.
On 7 July 2012, Libyans voted in parliamentary elections, the first free elections in almost 40
years.
The Turkish War of Independence (191922), initiated by Mustafa Kemal Atatrk and his
colleagues in Anatolia, resulted in the establishment of the modern Republic of Turkey in 1923,
with Atatrk as its first president.
54. Solution: a)
https://www.wto.org/english/thewto_e/whatis_e/tif_e/org5_e.htm
The WTOs main functions are to do with trade negotiations and the enforcement of negotiated
multilateral trade rules
Much attention is paid to the special needs and problems of developing and transition economies.
The WTO Secretariats Training and Technical Cooperation Institute organizes a number of
programmes to explain how the system works and to help train government officials and
negotiators.
A declaration also recognizes the contribution that trade liberalization makes to the growth and
development of national economies.
55. Solution: b)
http://www.insightsonindia.com INSIGHTS

Page 18

INSIGHTS MOCK TEST SERIES 2015: TEST 30 SOLUTIONS


The Sushruta Samhita is commonly dated to the 6th century B.C., and is attributed to the
physician Sushruta (meaning very famous in Sanskrit). The Sushruta Samhitas most well-known
contribution to plastic surgery is the reconstruction of the nose, known also as rhinoplasty.
It may also be pointed out that the Sushruta Samhita is also one of the foundational texts of the
Ayurveda, the traditional medical system of India. Therefore, the Sushruta Samhita contains more
than just the description of plastic surgery procedures.
http://www.theguardian.com/world/2014/oct/28/indian-prime-minister-genetic-science-existedancient-times

56. Solution: a)
The subtropical humid climate is characterized by hot summers and mild winters, though some of
the regions in this climatic type have rather cold winters. This type of climate is, however, marked
by variations within it.
For example, precipitation regime (seasonal distribution) is not the same everywhere. In some
areas, the summer has the maximum precipitation, while in others there is a distinct winter
precipitation maximum due to cyclonic storms and frontal activity. However, in humid subtropical
climate more or less rain falls throughout the year.
You can read more details at http://www.preservearticles.com/2011111717355/useful-notes-onsubtropical-humid-climate-cf-or-china-type-of-climate.html
57. Solution: d)
Coke is a fuel with few impurities and high carbon content, usually made from coal. It is the solid
carbonaceous material derived from destructive distillation of low-ash, low-sulfur bituminous coal.
Cokes made from coal are grey, hard, and porous. While coke can be formed naturally, the
commonly used form is man-made. The form known as petroleum coke, or pet coke, is derived
from oil refinery coker units or other cracking processes.
Coke is used in preparation of producer gas which is a mixture of carbon monoxide and nitrogen.
Producer gas is produced by passing air over red hot coke. Coke is also used to manufacture water
gas.
58. Solution: a)
The Roaring Forties are strong westerly winds found in the Southern Hemisphere, generally
between the latitudes of 40 and 50 degrees. The strong west-to-east air currents are caused by the
combination of air being displaced from the Equator towards the South Pole and the Earth's
rotation, and there are few landmasses to serve as windbreaks.
The Roaring Forties were a major aid to ships sailing from Europe to the East Indies or Australia
during the Age of Sail, and in modern usage are favoured by yachtsmen on round-the-world
voyages and competitions. The boundaries of the Roaring Forties are not consistent, and shift
http://www.insightsonindia.com INSIGHTS

Page 19

INSIGHTS MOCK TEST SERIES 2015: TEST 30 SOLUTIONS


north or south depending on the season. Similar but stronger conditions occur in more southerly
latitudes and are referred to as the Furious Fifties and Shrieking or Screaming Sixties.
59. Solution: d)
Bengal revolutionaries during the freedom struggle took shelter in Tripura.
Moreover, communism in the state had its beginnings in the pre-independence era, inspired by
freedom struggle activities in Bengal, and culminating in regional parties with communist
leanings.
It capitalised on the tribal dissatisfaction with the mainstream rulers, and has been noted for
connection with the "sub-national or ethnic searches for identity
60. Solution: c)
http://www.nytimes.com/2010/04/07/science/07methane.html?_r=0
Methane has no smell. It is the main ingredient of natural gas, but the smell that consumers
associate with natural gas is actually a chemical added by the gas company to make leaks obvious.
Humans cannot sense methane itself, although at times it is present with another gas, hydrogen
sulfide, that has an evident stink. But because the methane is often pure, miners long ago began
carrying canaries to work with them, knowing that when the birds showed signs of distress it was
time to get out.
The main method for getting methane out of mines is to pump fresh air in, so much that visitors
can often feel a breeze.

61. Solution: a)
Hard water contains considerable amount of dissolved carbonates of magnesium & Calcium in
room temperature.
When they are boiled inside boilers the decomposed calcium & magnesium carbonates (solids)
blocks the pipe lines inside them which leads the pressure inside the boiler to increase rapidly
without control. This may cause the boilers to rupture.
This blocking inside the pipes does not suddenly happen in a day or two. It happens very slowly. It
is expensive task to clean boilers regularly. Therefore, hard water is not used inside boilers.
62. Solution: b)
A keystone species is a species that has a disproportionately large effect on its environment relative
to its abundance.
http://www.insightsonindia.com INSIGHTS

Page 20

INSIGHTS MOCK TEST SERIES 2015: TEST 30 SOLUTIONS


Such species are described as playing a critical role in maintaining the structure of an ecological
community, affecting many other organisms in an ecosystem and helping to determine the types
and numbers of various other species in the community.
The role that a keystone species plays in its ecosystem is analogous to the role of a keystone in an
arch. While the keystone is under the least pressure of any of the stones in an arch, the arch still
collapses without it.

63. Solution: a)
An anti-cyclone -- also known as a high pressure area -- is a large atmospheric circulation system
with the wind flowing clockwise around it in the Northern Hemisphere, and counter-clockwise in
the Southern Hemisphere.
Anticyclones form from air masses cooling more than their surroundings, which causes the air to
contract slightly making the air denser. Since dense air weighs more, the weight of the atmosphere
overlying a location increases, causing increased surface air pressure.
The air mass cooling that results in an anticyclone forming can be caused by either conduction as
the air flows over a relatively cool ocean surface, or through the loss of infrared radiation over land
during the fall, winter, or spring when little sunlight is available to warm the air mass.
The strongest anticyclones occur over snow-covered portions of Asia and North America in the
winter when clear, dry air masses cool from a loss of infrared radiation, while little sunlight is
absorbed to offset that infrared cooling.
64. Solution: c)
The principal components of volcanic gases are water vapor (H2O), carbon dioxide (CO2), sulfur
either as sulfur dioxide (SO2) (high-temperature volcanic gases) or hydrogen sulfide (H2S) (lowtemperature volcanic gases), nitrogen, argon, helium, neon, methane, carbon monoxide and
hydrogen.
Understanding gases dissolved in magma is critical in understanding why volcanoes erupt. Bodies
of magma rise in the crust until they reach a point of neutral buoyancy. The expansion of gases
brings the magma closer to the surface and drives eruptions. The interaction between the viscosity
http://www.insightsonindia.com INSIGHTS

Page 21

INSIGHTS MOCK TEST SERIES 2015: TEST 30 SOLUTIONS


and temperature of the magma and the gas content determines if an eruption will be effusive or
explosive.
65. Solution: a)

The streams within a drainage basin form certain patterns, depending on the slope of land,
underlying rock structure as well as the climatic conditions of the area.
These are dendritic, trellis, rectangular, and radial patterns. The dendritic pattern develops where
the river channel follows the slope of the terrain. The stream with its tributaries resembles the
branches of a tree, thus the name dendritic. A river joined by its tributaries, at approximately right
angles, develops a trellis pattern.
A trellis drainage pattern develops where hard and soft rocks exist parallel to each other. A
rectangular drainage pattern develops on a strongly jointed rocky terrain. The radial pattern
develops when streams flow in different directions from a central peak or dome like structure.
A combination of several patterns may be found in the same drainage basin.

66. Solution: c)
The Brahmaputra rises in Tibet east of Mansarowar lake very close to the sources of the Indus and
the Satluj. It is slightly longer than the Indus, and most of its course lies outside India. It flows
eastwards parallel to the Himalayas. On reaching the Namcha Barwa (7757 m), it takes a U turn
and enters India in Arunachal Pradesh through a gorge.
Here, it is called the Dihang and it is joined by the Dibang, the Lohit, the Kenula and many other
tributaries to form the Brahmaputra in Assam. In Tibet the river carries a smaller volume of water
and less silt as it is a cold and a dry area. In India it passes through a region of high rainfall.
Here the river carries a large volume of water and considerable amount of silt. The Brahmaputra
has a braided channel in its entire length in Assam and forms many riverine islands.
67. Solution: d)
http://www.insightsonindia.com INSIGHTS

Page 22

INSIGHTS MOCK TEST SERIES 2015: TEST 30 SOLUTIONS


Western Ghats runs from north to south close to the western coast.
Most of the major rivers of the Peninsula such as the Mahanadi, the Godavari, the Krishna and the
Kaveri flow eastwards and drain into the Bay of Bengal. These rivers make deltas at their mouths.
There are numerous small streams flowing west of the Western Ghats.
The Narmada and the Tapi are the only long rivers, which flow west and make estuaries. The
drainage basins of the peninsular rivers are comparatively small in size.
68. Solution: d)
Due to the curvature of the earth, the amount of solar energy received varies according to
latitude. As a result, air temperature decreases from the equator towards the poles. As one goes
from the surface of the earth to higher altitudes, the atmosphere becomes less dense and
temperature decreases. The hills are therefore cooler during summers.
The pressure and wind system of any area depend on the latitude and altitude of the place.
Thus it influences the temperature and rainfall pattern. The sea exerts a moderating influence on
climate: As the distance from the sea increases, its moderating influence decreases and the
people experience extreme weather conditions. This condition is known as continentality (i.e. very
hot during summers and very cold during winters).
Ocean currents along with onshore winds affect the climate of the coastal areas, For example,
any coastal area with warm or cold currents flowing past it, will be warmed or cooled if the winds
are onshore.
69. Solution: d)
These low-pressure systems, originate over the Mediterranean Sea and western Asia and move into
India, along with the westerly flow. They cause the much-needed winter rains over the plains and
snowfall in the mountains. Although the total amount of winter rainfall locally known as
mahawat is small, they are of immense importance for the cultivation of rabi crops.
The peninsular region does not have a well-defined cold season. There is hardly any noticeable
seasonal change in temperature pattern during winters due to the moderating influence of the sea.
70. Solution: d)
The reason is orographic rainfall on the western side of the Western Ghats. Clouds condense and
precipitate because of their collision with the western slopes of the mountains.
Moreover, the western slopes are highly continuous. So, few clouds can escape to the other side of
the mountain.
Hence, lesser rainfall and lesser vegetation.
71. Solution: d)
India is known for its herbs and spices from ancient times. Some 2,000 plants have been described
in Ayurveda and at least 500 are in regular use. The World Conservation Unions Red list has
http://www.insightsonindia.com INSIGHTS

Page 23

INSIGHTS MOCK TEST SERIES 2015: TEST 30 SOLUTIONS


named 352 medicinal plants of which 52 are critically threatened and 49 endangered. The
commonly used plants in India are:
Sarpagandha : Used to treat blood pressure; it is found only in India.
Jamun : The juice from ripe fruit is used to prepare vinegar which is carminative and diuretic, and
has digestive properties. The powder of the seed is used for controlling diabetes.
Arjun : The fresh juice of leaves is a cure for earache. It is also used to regulate blood pressure.
Babool : Leaves are used as a cure for eye sores. Its gum is used as a tonic.
Neem : Has high antibiotic and antibacterial properties.
Tulsi Plant : Is used to cure cough and cold.
Kachnar : Is used to cure asthma and ulcers. The buds and roots are good for digestive problems.
72. Solution: a)
Fault is a linear break in rocks of the earths crust along which there has been displacement in a
horizontal, vertical or oblique direction.
Fold is a bend in the rock strata resulting from compression of an area of the earths crust.
Geosyncline is a narrow, shallow, elongated basin with a sinking bottom in which a considerable
thickness of sediments was deposited by the rivers coming from Angara and Gondwanaland.
73. Solution: b)
Technically, all the resources within the territorial extent belong to the nation. The country has
legal powers to acquire even private property for public good.
There are international institutions which regulate some other resources outside territorial extent
of the nation. The oceanic resources beyond 200 km of the Exclusive Economic Zone belong to
open ocean and no individual country can utilise these without the concurrence of international
institutions.
74. Solution: b)
The Sundarban Delta derives its name from its name from the Sundari tree which grows well in
marshland.
Alternatively, it has been proposed that the name is a corruption of Samudraban, Shomudrobn
("Sea Forest"), or Chandra-bandhe (name of a primitive tribe). However, the generally accepted
view is the one associated with Sundari trees.
It is the worlds largest and fastest growing delta. It is also the home of Royal Bengal tiger.
75. Solution: d)

http://www.insightsonindia.com INSIGHTS

Page 24

INSIGHTS MOCK TEST SERIES 2015: TEST 30 SOLUTIONS


Materials in the environment which have the potential to satisfy human needs but human beings
do not have the appropriate technology to access these, are included among stock.
For example, water is a compound of two inflammable gases; hydrogen and oxygen, which can be
used as a rich source of energy.
But we do not have the required technical know how to use them for this purpose. Hence, it can
be considered as stock.

76. Solution: a)
Soils in the drier areas are more alkaline and can be productive after proper treatment and
irrigation.
In some areas the salt content is very high and common salt is obtained by evaporating the water.
Due to the dry climate, high temperature, evaporation is faster and the soil lacks humus and
moisture.
Ploughing the soil in a wrong way i.e. up and down the slope form channels for the quick flow of
water that lead to soil erosion. So soil must be loosened and turned properly.
77. Solution: d)
Habitat destruction, hunting, poaching, over-exploitation, environmental pollution, poisoning and
forest fires are factors, which have led to the decline in Indias biodiversity.
Other important causes of environmental destruction are unequal access, inequitable consumption
of resources and differential sharing of responsibility for environmental well-being.
Over-population in third world countries is often cited as the cause of environmental degradation.
However, an average American consumes 40 times more resources than an average Somalian.
Similarly, the richest five per cent of Indian society probably cause more ecological damage
because of the amount they consume than the poorest 25 per cent. The former shares minimum
responsibilities for environmental well-being.
78. Solution: d)
Reserved forests are forest lands are protected from any further depletion.
Protected forests are regarded as most valuable as far as the conservation of forest and wildlife
resources.
Unclassed forests are other forests and wastelands belonging to both government and private
individuals and communities.
79. Solution: d)

http://www.insightsonindia.com INSIGHTS

Page 25

INSIGHTS MOCK TEST SERIES 2015: TEST 30 SOLUTIONS


After Gandhijis martyrdom, Vinobha Bhave undertook padyatra to spread Gandhijis message
covered almost the entire country.
Once, when he was delivering a lecture at Pochampalli in Andhra Pradesh, some poor landless
villagers demanded some land for their economic well-being. Vinoba Bhave could not promise it to
them immediately but assured them to talk to the Government of India regarding provision of land
for them if they undertook cooperative farming.
Suddenly, Shri Ram Chandra Reddy stood up and offered 80 acres of land to be distributed among
80 land-less villagers. This act was known as Bhoodan. Later he travelled and introduced his
ideas widely all over India. Some zamindars, owners of many villages offered to distribute some
villages among the landless. It was known as Gramdan. However, many land-owners chose to
provide some part of their land to the poor farmers due to the fear of land ceiling act. This
Bhoodan-Gramdan movement initiated by Vinobha Bhave is also known as the Blood-less
Revolution.
80. Solution: d)
His principles of Dhamma were clearly stated in his Edicts. The main features of Asokas Dhamma
as mentioned in his various Edicts may be summed as follows:

Service to father and mother, practice of ahimsa, love of truth, reverence to teachers and
good treatment of relatives.
Prohibition of animal sacrifices and festive gatherings and avoiding expensive and
meaningless ceremonies and rituals.
Efficient organization of administration in the direction of social welfare and maintenance
of constant contact with people through the system of Dhammayatras.
Humane treatment of servants by masters and prisoners by government officials.
Consideration and non-violence to animals and courtesy to relations and liberality to
Brahmins.
Tolerance among all the religious sects.
Conquest through Dhamma instead of through war.

81. Solution: d)
Mathura School of Art developed at Mathura in modern Uttar Pradesh is called the Mathura art. It
flourished in the first century A.D. In its early phase, the Mathura school of art developed on
indigenous lines.
The Buddha images exhibit the spiritual feeling in his face which was largely absent in the
Gandhara school. The Mathura school also carved out the images of Siva and Vishnu along with
their consorts Parvathi and Lakshmi.
The female figures of yakshinis and apsaras of the Mathura school were beautifully carved.

82. Solution: d)
http://www.insightsonindia.com INSIGHTS

Page 26

INSIGHTS MOCK TEST SERIES 2015: TEST 30 SOLUTIONS


The king had also taken the advice of his minister, court-poet and the imperial court or avai.
The king was assisted by a large body of officials who were divided into five councils. They were
ministers (amaichar), priests (anthanar), military commanders (senapathi), envoys (thuthar) and
spies (orrar).
The military administration was also efficiently organized during the Sangam Age. Each ruler had
a regular army and their respective Kodimaram (tutelary tree). Land revenue was the chief source
of states income while custom duty was also imposed on foreign trade.

83. Solution: c)
Out of his nine years stay in India, he spent six years in the Gupta empire. He came to India by the
land route through Khotan, Kashgar, Gandhara and Punjab.
He visited Peshawar, Mathura, Kanauj, Sravasti, Kapilavastu, Kusinagara, Pataliputra, Kasi and
Bodh Gaya among other places. He returned by the sea route, visiting on the way Ceylon and Java.
The main purpose of his visit was to see the land of the Buddha and to collect Buddhist
manuscripts from India. He stayed in Pataliputra for three years studying Sanskrit and copying
Buddhist texts.
Fahien provides valuable information on the religious, social and economic condition of the Gupta
empire.
According to him, Buddhism was in a flourishing condition in the northwestern India but in the
Gangetic valley it was in a state of neglect. He refers to the Gangetic valley as the land of
Brahmanism
84. Solution: c)
During the British period cotton belts of India attracted the British and ultimately cotton was
exported to Britain as a raw material for their textile industries.
Champaran movement was started in 1917 in Bihar. This was started because farmers of that
region were forced to grow indigo on their land because it was necessary for the textile industries
which were located in Britain.
They were unable to grow foodgrains to sustain their families.

85. Solution: a)
The basic form of the Hindu temple comprises the following:
http://www.insightsonindia.com INSIGHTS

Page 27

INSIGHTS MOCK TEST SERIES 2015: TEST 30 SOLUTIONS


(i) a cave-like sanctum (garbhagriha literally womb-house), which, in the early temples, was a
small cubicle with a single entrance and grew into a larger chamber in time. The garbhagriha is
made to house the main icon which is itself the focus of much ritual attention;
(ii) the entrance to the temple which may be a portico or colonnaded hall that incorporates space
for a large number of worshippers and is known as a mandapa;
(iii) from the fifth century CE onwards, freestanding temples tend to have a mountain-like spire,
which can take the shape of a curving shikhar in North India and a pyramidal tower, called a
vimana, in South India;
(iv) the vahan, i.e., the mount or vehicle of the temples main deity along with a standard pillar or
dhvaj is placed axially before the sanctum. Two broad orders of temples in the country are
known Nagara in the north and Dravida in the south.
86. Solution: a)
Cyrus the Great was the greatest conqueror of the Achaemenian Empire. He was the first
conqueror who led an expedition and entered into India.
The Persian invasion provided an impetus to the growth of Indo-Iranian commerce. Also, it
prepared the ground for Alexanders invasion. The use of the Kharoshti script, a form of Iranian
writing became popular in northwestern India and some of Asokas edicts were written in that
script.
We are able to see the influence of Persian art on the art of the Mauryas, particularly the
monolithic pillars of Asoka and the sculptures found on them. The very idea of issuing edicts by
Asoka and the wording used in the edicts are traced to Iranian influence. In short, the Iranian
connection with India proved more fruitful than the short-lived Indo-Macedonian contact.
87. Solution: d)
Samudragupta was the greatest of the rulers of the Gupta dynasty. The Allahabad Pillar inscription
provides a detailed account of his reign. It refers to three stages in his military campaign

Against some rulers of North India


His famous Dakshinapatha expedition against South Indian rulers
A second campaign against some other rulers of North India.

He did not destroy and annex South Indian kingdoms. Instead, he defeated the rulers but gave
them back their kingdoms. He only insisted on them to acknowledge his suzerainty.
88. Solution: d)
According to the Permanent Land revenue settlement the Zamindars were recognised as the
permanent owners of the land. They were given instruction to pay 89% of the annual revenue to
the state and were permitted to enjoy 11% of the revenue as their share. They were left independent
in the internal affairs of their respective districts.

http://www.insightsonindia.com INSIGHTS

Page 28

INSIGHTS MOCK TEST SERIES 2015: TEST 30 SOLUTIONS


The Zamindars were required to issue Patta and Quabuliyats to the cultivators mentioning the area
of their land, and the amount of revenue to be paid by them to the state. The historians have
expressed divergent opinions about its merits and demerits.
By making the Zamindars the owners of the land, the settlement created a class of loyal land lords
who formed a stable element in the state. The permanent settlement secured the political support
of the Zamindars of Bengal who stood loyal during the great mutiny of 1857.
89. Solution: d)
The Chalukya administration was highly centralized unlike that of the Pallavas and the Cholas.
Village autonomy was absent under the Chalukyas.
The Chalukyas had a great maritime power. Pulakesin II had 100 ships in his navy. They also had a
small standing army.
The village administration was carried on by the village headmen in the Rashtrakuta
administration. However, the village assemblies played a significant role in the village
administration.
90. Solution: b)
The English East India Company was established in 1600 and the Charter was issued by Queen
Elizabeth of England.
Captain Hawkins arrived at the royal court of Jahangir in 1609 to seek permission to establish
English trading centre at Surat. But it was refused by the Mughal Emperor due to Portuguese
pressure.
Later in 1612, Jahangir issued a farman (permission letter) to the English and they established a
trading factory at Surat in 1613.
91. Solution: a)
The Third Carnatic War ended with the French defeat. The French agreed to confine its activities
in Pondicherry, Karaikkal, Mahe and Yenam. Thus the Anglo-French rivalry came to a close with
British success and French failure.
The causes for the French failure can be summed up as follows:

Britishers were superior in commerce and naval power.


Lack of support from the French government.
French had support only in the Deccan but the English had a strong base in Bengal.
English had three important ports Calcutta, Bombay and Madras but French had only
Pondicherry.
Difference of opinion between the French Generals.
Englands victory in the European wars decided the destiny of the French in India.

http://www.insightsonindia.com INSIGHTS

Page 29

INSIGHTS MOCK TEST SERIES 2015: TEST 30 SOLUTIONS


92. Solution: d)
Mahatma Gandhi protested against the Communal Award and went on a fast unto death in the
Yeravada jail on 20 September 1932.
Finally, an agreement was reached between Dr Ambedkar and Gandhi. This agreement came to be
called as the Poona Pact.
The British Government also approved of it. Accordingly, 148 seats in different Provincial
Legislatures were reserved for the Depressed Classes in place of 71 as provided in the Communal
Award. The third Round Table Conference came to an end in 1932.
93. Solution: d)
The Woods dispatch - The establishment of departments of public instructions in five provinces
and introduction of the pattern of grants in aid to encourage private participation in the field of
education were recommended.
Besides, the dispatch also laid emphasis on the establishment of schools for technical education,
teacher and women education. Over and above all these, the dispatch recommended the
establishment of one University each in Calcutta, Bombay and Madras, on the model of the
London University.
Consequently, within the next few years, the Indian education became rapidly westernized.
94. Solution: a)
In 1878, the Vernacular Press Act was passed. This Act empowered a Magistrate to secure an
undertaking from the editor, publisher and printer of a vernacular newspaper that nothing would
be published against the English Government. The equipment of the press could be seized if the
offence was committed. This Act crushed the freedom of the Indian press. This created adverse
public opinion against the British Government.
In the same year, the Arms Act was passed. This Act prevented the Indians to keep arms without
appropriate license. Its violation would be a criminal offence. The Europeans and the AngloIndians were exempted from the operation of these legislations.
95. Solution: d)
Ripon believed that self-government is the highest and noblest principles of politics. Therefore,
Ripon helped the growth of local bodies like the Municipal Committees in towns and the local
boards in taluks and villages. The powers of municipalities were increased. Their chairmen were to
be non-officials. They were entrusted the care of local amenities, sanitation, drainage and watersupply and also primary education.
District and taluk boards were created. It was insisted that the majority of the members of these
boards should be elected non-officials. The local bodies were given executive powers with financial
resources of their own. It was perhaps the desire of Ripon that power in India should be gradually
transferred to the educated Indians.
http://www.insightsonindia.com INSIGHTS

Page 30

INSIGHTS MOCK TEST SERIES 2015: TEST 30 SOLUTIONS


96. Solution: a)
Swami Vivekananda participated at the Parliament of Religions held in Chicago (USA) in
September 1893 and raised the prestige of India and Hinduism very high.
He said that all religions teach the same thing and gave the philosophy of vasudhaiv
kutumbakam. It meant that we all belong to the earth and that entire humanity is our family. He
then argued on the essential unity of all religions. He also mentioned that Hinduism captures the
essence of all major religions and is more all-encompassing than other religions.
He asked the people to improve the lives of the poor and depressed classes. He believed that
service to mankind is service to God. Vivekananda preached the message of strength and selfreliance.
97. Solution: d)

The Moderates were able to create a wide national awakening among the people.
They popularized the ideas of democracy, civil liberties and representative institutions.
They explained how the British were exploiting Indians. Particularly, Dadabhai Naoroji in
his famous book Poverty and UnBritish Rule in India wrote his Drain Theory. He showed
how Indias wealth was going away to England in the form of: (a) salaries,(b)savings, (c)
pensions, (d) payments to British troops in India and (e) profits of the British companies. In
fact, the British Government was forced to appoint the Welby Commission, with Dadabhai
as the first Indian as its member, to enquire into the matter.
Some Moderates like Ranade and Gokhale favoured social reforms. They protested against
child marriage and widowhood.
The Moderates had succeeded in getting the expansion of the legislative councils by the
Indian Councils Act of 1892.

98. Solution: d)
Two Home Rule Leagues were established, one by B.G. Tilak at Poona in April 1916 and the other
by Mrs. Annie Besant at Madras in September 1916. The aim of the Movement was to get selfgovernment for India within the British Empire. It believed freedom was the natural right of all
nations. Moreover, the leaders of the Home Movement thought that Indias resources were not
being used for her needs.
The two Leagues cooperated with each other as well with the Congress and the Muslim League in
putting their demand for home rule. While Tilaks Movement concentrated on Maharashtra, Annie
Besants Movement covered the rest of the country. The Home Rule Movement had brought a new
life in the national movement. There was a revival of Swadeshi. Women joined in larger numbers.
99. Solution: a)
Warren Hastings realized the immediate need for introducing reforms by abolishing the dual
system.

http://www.insightsonindia.com INSIGHTS

Page 31

INSIGHTS MOCK TEST SERIES 2015: TEST 30 SOLUTIONS


The East India Company decided to act as Diwan and to undertake the collection of revenue by its
own agents. Hence, the Dual System introduced by Robert Clive was abolished. As a measure After
the abolition of the Dual System, the responsibility of collecting the revenue fell on the shoulders
of the Company.
For that purpose, a Board of Revenue was established at Calcutta to supervise the collection of
revenue. English Collectors were appointed in each district.
100. Solution: d)
http://indiatoday.intoday.in/story/madrasas-derecognised-bjp-devendra-fadnavis-jnurte/1/448785.html
The controversial move by the Maharashtra government to derecognise madrassas has created
furore.
The government argued that madrassas only gave religious instruction and did not impart formal
education in subjects such as science, maths, social science and English. Justifying it, state
Minority Affairs Minister Eknath Khadse said if madrassas started teaching these four subjects,
they would be considered as schools.
Khadse said, "Our Constitution says every child has the right to take formal education that
madrasas do not give. They are just educating children on religion and not giving them any formal
education."

http://www.insightsonindia.com INSIGHTS

Page 32

INSIGHTS MOCK TEST - 31


INSIGHTS ON INDIA MOCK PRELIMINARY EXAM - 2015


INSIGHTS ON INDIA MOCK TEST - 31
GENERAL STUDIES

PAPER-I

Time Allowed: 2 Hours

Maximum Marks: 200

INSTRUCTIONS
1. IMMEDITELY AFTER THE COMMENCEMENT OF THE EXAMINATION, YOU SHOULD
CHECK THAT THIS TEST BOOKLET DOES NOT HAVE ANY UNPRINTED OR TORN OR MISSING
PAGES OR ITEMS, ETC. IF SO, GET IT REPLACED BY A COMPLETE TEST BOOKLET.
2. You have to enter your Roll Number on the Test
Booklet in the Box provided alongside. DO NOT
Write anything else on the Test Booklet.
4. This Test Booklet contains 100 items (questions). Each item is printed only in English. Each item
comprises four responses (answers). You will select the response which you want to mark on the
Answer Sheet. In case you feel that there is more than one correct response, mark the response which
you consider the best. In any case, choose ONLY ONE response for each item.
5. You have to mark all your responses ONLY on the separate Answer Sheet provided. See directions in
the Answer Sheet.
6. All items carry equal marks.
7. Before you proceed to mark in the Answer Sheet the response to various items in the Test Booklet, you
have to fill in some particulars in the Answer Sheet as per instructions sent to you with your
Admission Certificate.
8. After you have completed filling in all your responses on the Answer Sheet and the examination has
concluded, you should hand over to the Invigilator only the Answer Sheet. You are permitted to take
away with you the Test Booklet.
9. Sheets for rough work are appended in the Test Booklet at the end.
10. Penalty for wrong answers :
THERE WILL BE PENALTY FOR WRONG ANSWERS MARKED BY A CANDIDATE IN THE
OBJECTIVE TYPE QUESTION PAPERS.
(i)

There are four alternatives for the answer to every question. For each question for which a
wrong answer has been given by the candidate, one-third of the marks assigned to that
question will be deducted as penalty.

(ii) If a candidate gives more than one answer, it will be treated as a wrong answer even if one of
the given answers happens to be correct and there will be same penalty as above to that
question.
(iii)

If a question is left blank, i.e., no answer is given by the candidate, there will be no penalty
for that question.
http://www.insightsonindia.com

INSIGHTS ON INDIA MOCK TEST SERIES FOR CIVIL SERVICES PRELIMINARY EXAM 20

http://www.insightsonindia.com INSIGHTS

Page 1

INSIGHTS MOCK TEST - 31


1. What is/are the similarities between


phytoplankton in a pond ecosystem; and
lichens in a terrestrial ecosystem?
1. They both can prepare their own
food.
2. They both are the pioneer species
forming the pioneer community.
3. They both are in a symbiotic
relationship with some species in
their respective ecosystem.
Choose the correct answer using the codes
below.
a)
b)
c)
d)

1 and 2 only
1 and 3 only
2 and 3 only
2 only

2. Consider the following about Sickle Cell


Anaemia.
1. It is a hereditary disease
2. It spreads via air and water
3. It leads to leads to jaundice and
frequent ailments in children
4. It can be prevented by increasing
iron intake since childhood
Choose the correct answer using the codes
below.
a)
b)
c)
d)

3 and 4 only
1 and 2 only
1 and 3 only
2 and 4 only

http://www.insightsonindia.com INSIGHTS

3. The Reserve Bank of India (RBI)


recently signed a currency swap
agreement with the Central Bank of Sri
Lanka. This would help both the
countries in avoiding
a) Balance of payment and liquidity
crisis
b) Low Inflation in the economies
c) Adverse trade balance between both
nations
d) Low cross-border capital inflows

4. Which of these are methods of ex-site


conservation?
1. Cryopreservation
2. Seed banks
3. Field Gene banks
4. Home gardens
Choose the correct answer using the codes
below.
a)
b)
c)
d)

3 and 4 only
1 and 2 only
2, 3 and 4 only
All of the above

5. A tiger reserve is generally zoned into


core, buffer and transition areas. In the
Core area, which of these activities are
NOT permitted in India?
1. Tourism
2. Mining
3. Human habitation
4. Industries

Page 2

INSIGHTS MOCK TEST - 31


Choose the correct answer using the codes


below.
a)
b)
c)
d)

1 and 3 only
2 and 4 only
2, 3 and 4 only
All of the above

6. Susceptibility to extinction will be


highest in species which
1. have fixed migratory routes in fixed
seasons
2. have a narrow range of habitat
3. feed at lowest levels in the food web
Choose the correct answer using the codes
below.
a)
b)
c)
d)

1 and 2 only
1 and 3 only
2 and 3 only
All of the above

7. Consider the following minerals and


their uses.
1. Mercury is used in making electric
switches.
2. Phosphorous is used in making
medicines.
3. Potassium and Silver can be used in
photography.
4. Manganese can be used in making
disinfectants.
Choose the correct answer using the codes
below.
a)
b)
c)
d)

3 and 4 only
1 and 2 only
2, 3 and 4 only
All of the above

http://www.insightsonindia.com INSIGHTS

8. Marine algae have been used as manure


in many countries because
a) they contain very high biomass as
compared to their weight
b) they are rich in nutrients
c) they help in decomposing the
detritus present in the farms
d) they produce agar that is highly
fertile and keeps the soil moisture
intact

9. How is social forestry different from


agro-forestry?
1. In agro-forestry, trees are grown
instead of crops on farms.
2. In social forestry, NGOs take
responsibility for afforestation of
arid and fallow lands.
3. While the community owns the
resources in social forestry, agroforestry produce is privately owned.
Choose the correct answer using the codes
below.
a)
b)
c)
d)

1 and 2 only
1 and 3 only
2 only
3 only

10. Consider the following statements about


the water cycle.
Assertion (A): While evaporation is greater
over the oceans, rainfall is greater over the
land.
Reason (R): Trade winds carry moisture laden
winds from the oceans to land.
Page 3

INSIGHTS MOCK TEST - 31


In the context of the statements above, which


of these is true?
a) A and R both are true, and R is the
correct explanation for A.
b) A and R both are true, and R is the
NOT the correct explanation for A.
c) A is correct, R is incorrect.
d) A is incorrect, R is correct.

11. Consider the following functions of


forests.
1. Absorption of atmospheric gases
2. Storage of minerals and water
3. Storage and release of radiant energy
These functions fall under which of the
following categories?
a)
b)
c)
d)

Protective functions
Regulative functions
Productive Functions
Reproductive Functions

12. Which of these processes may lead to a


loss of nutrients from the soil?
1. Nutrient Immobilization
2. Mineralization
3. Leaching
4. Nitrification
Choose the correct answer using the codes
below.
a)
b)
c)
d)

1 and 4 only
2, 3, and 4 only
2 and 3 only
1, 2 and 3 only

http://www.insightsonindia.com INSIGHTS

13. Which of the following is/are free-living


nitrogen fixing bacteria present in the
soil?
1. Rhizobium
2. Clostridium
3. Azotobacter
4. Pseduomonas
Choose the correct answer using the codes
below.
a)
b)
c)
d)

1 and 4 only
2, 3, and 4 only
2 and 3 only
1, 2 and 3 only

14. Consider the following about


Mycorrhizae.
1. It is a symbiotic relation of fungi
with plant roots.
2. It helps in better absorption of
minerals from soil.
3. It can form mutualistic relation with
blue-green algae to improve water
absorption in plants.
Choose the correct answer using the codes
below.
a)
b)
c)
d)

1 and 2 only
1 and 3 only
2 only
3 only

15. Certain plants use C4 pathway of


photosynthesis instead of C3 generally
used by plants. What difference does a
C4 pathway make to the plants?
1. It is a more efficient way of
photosynthesis.
Page 4

INSIGHTS MOCK TEST - 31


2. Plants can survive in low moisture


conditions.
3. Nutrient need from soil is lowered.
4. They can survive in high temperature
conditions.
Choose the correct answer using the codes
below.
a)
b)
c)
d)

1 and 2 only
1 and 3 only
2 and 4 only
All of the above

16. Main inputs of phosphorous as a


nutrient in soil come from
1. Rainfall
2. Phosphorous fixing bacteria
3. Weathering of soil
4. Atmospheric gas exchanges with soil
Choose the correct answer using the codes
below.
a)
b)
c)
d)

1 and 2 only
1 and 3 only
2 and 4 only
All of the above

17. Only 50 per cent of the energy of the


total incident solar radiation is available
to producers for absorption. Why is the
other 50 per cent unavailable?
1. It is lost in respiration and
assimilation by first level organisms.
2. It is absorbed by gases or reflected by
clouds in the atmosphere.
Which of the above is/are correct?

b) 2 only
c) Both 1 and 2
d) None

18. Consider the following statements about


critical link species in a forest
ecosystem.
1. A forest ecosystem has only a few
critical link species.
2. Loss of critical link species has
disproportionate impact on the
ecosystem.
3. Critical link species are always found
at the edge of ecosystems.
4. Critical link species are also known
as Keystone species.
Choose the correct answer using the codes
below.
a)
b)
c)
d)

1 and 2 only
1 and 3 only
2 and 4 only
2 only

19. During PMs recent visit to Seoul, it was


announced that India and Korea will
strengthen their historic connection by
enhancing linkages of Korean people
with Ayodhya. What explains the
linkages?
a) Queen Suriratna, a princess from
Ayodhya had travelled to South
Korea to marry King Kim Suro
b) The land of Ayodhya was donated to
the then King of South Korea as a
goodwill gesture who later returned
the favour by gifting the land back

a) 1 only
http://www.insightsonindia.com INSIGHTS

Page 5

INSIGHTS MOCK TEST - 31


c) King Suro of South Korea helped the


Ayodhyan army to deal with invaders
from the North-western mountains
d) All of the above

20. Consider the following about the


Amaravati, Andhra Pradesh.
1. It was ruled by Satavahanas.
2. Hinduism, Buddhism and Jainism
have all flourished in the region.
3. All major rock-cut architectural
schools were born in this region.
Choose the correct answer using the codes
below.
a)
b)
c)
d)

1 and 2 only
1 and 3 only
2 and 3 only
All of the above

21. Consider the following about Ken and


Betwa rivers.
1. Both are tributaries of Yamuna.
2. Both are major rivers of
Bundelkhand region.
3. Both will be interlinked together as
per a government decision.
Choose the correct answer using the codes
below.
a)
b)
c)
d)

1 and 2 only
1 and 3 only
2 and 3 only
2 only

22. Which of the functions relating to the


Parliament are performed by the
President?
1. Summoning the Houses
http://www.insightsonindia.com INSIGHTS

2.
3.
4.
5.

Proroguing the Houses


Adjournment Sine dine
Dissolving lower house
Addressing both houses

Choose the correct answer using the codes


below.
a)
b)
c)
d)

1, 4 and 5 only
2 and 3 only
1, 2, 4 and 5 only
All of the above

23. The representative of each Union


territory in the Rajya Sabha are
indirectly elected by
a) Members of an electoral college
specially constituted for this purpose
b) Members of Legislative assemblies
c) Members of Local constitutional
bodies
d) MPs, MLAs and ward councillors
jointly from a Union Territory
24. A Member of Parliament ceases to hold
his seat if
1. He owns and runs a private
commercial firm
2. He holds any office of Profit under
the Union or State government
3. He is removed from the political
party on whose ticket he was elected
4. He is expelled from the house.
Choose the correct answer using the codes
below.
a)
b)
c)
d)

2, 3 and 4 only
1 and 3 only
2 and 4 only
All of the above
Page 6

INSIGHTS MOCK TEST - 31


25. The Speaker of Lok Sabha can be


removed on
1. A resolution passed by majority of
the membership of Lok Sabha
2. A resolution passed by special
majority in Lok Sabha
3. An enquiry by the Supreme Court
4. An order of the President
Choose the correct answer using the codes
below.
a)
b)
c)
d)

1 only
2 and 3 only
1 and 3 only
1, 3 and 4 only

26. Apart from the Supreme Court of India,


which of the following is/are authorities
empowered to interpret the
constitution?
1. Chief Justice, High Court
2. Speaker, Lok Sabha
3. Chairman, Rajya Sabha
Choose the correct answer using the codes
below.
a)
b)
c)
d)

1 and 2 only
2 and 3 only
1 only
All of the above

c) Montague Chelmsford Act, 1919


d) Independence of India Act, 1947

28. Rule of law in Article 14 of the


Constitution implies that
1. The state cannot treat citizens
differently with respect to the same
law.
2. The state cannot punish citizens
except for violating the law.
3. The state cannot confer any special
socio-economic privilege on any
individual.
Choose the correct answer using the codes
below.
a)
b)
c)
d)

1 and 2 only
2 and 3 only
2 only
1 only

29. Consider the following about Estimates


Committee.
1. It is exclusively a committee of the
lower house.
2. Chairman of this committee is
invariably from the ruling party.
3. The committee is guided by
Comptroller and Auditor General of
India.
Choose the correct answer using the codes
below.

27. The Institution of Speaker and Deputy


Speaker in India originated under the
provisions of
a) Charter Act 1891
b) Morley-Minto Act, 1909
http://www.insightsonindia.com INSIGHTS

a)
b)
c)
d)

1 and 2 only
2 and 3 only
2 only
1 only
Page 7

INSIGHTS MOCK TEST - 31


30. Administration is held accountable to


the legislature by which of the following
devices?
1. Financial Committees of the
Parliament
2. Committee on delegated legislation
3. Committee on Business advisory
4. Question hour and zero hour in the
Parliament
Choose the correct answer using the codes
below.
a)
b)
c)
d)

1 and 3 only
1, 2 and 4 only
2 and 4 only
All of the above

31. For a no-confidence motion to be


admitted in Lok Sabha
1. Speaker must give consent to it.
2. Support of at least 50 members is
necessary.
3. Leader of Opposition must lead the
motion.
4. Both houses of Parliament must be
in session.
5. Prior permission from the President
must be obtained.
Choose the correct answer using the codes
below.
a)
b)
c)
d)

1 and 2 only
1, 2 and 5 only
2, 3 and 4 only
2 only

http://www.insightsonindia.com INSIGHTS

32. What is the meaning of popular


sovereignty?
a) Views and preferences of the
majority (popular) are considered
sovereign.
b) A nation is sovereign in all respects
by virtue of the sovereignty of its
resident citizens.
c) An area can be declared sovereign by
the wish of the people.
d) People are the ultimate source of the
authority of their government.

33. The Source of power of judicial review


in the constitution flows from
a) The doctrine of separation of power
between legislature and judiciary
b) The authority of the Supreme Court
and High courts to issue writs
(Article 32 and 226)
c) Article 13 of the Constitution
d) The Basic structure of the
Constitution

34. Consider the following about All-India


services.
1. Service conditions are determined by
the President.
2. Services can be abolished only by a
constitutional amendment.
3. UPSC is responsible for
appointments to the posts in AllIndia services.
4. Members of All-India services cannot
be removed from services by Prime
Minister alone.
Page 8

INSIGHTS MOCK TEST - 31


Choose the correct answer using the codes


below.
a)
b)
c)
d)

1, 2 and 4 only
3 only
2, 3 and 4 only
2 and 4 only

35. In the Budgetary process, arrange the


following in correct order.
1. Vote on account
2. Bill sent to Upper House.
3. General Discussion on the Bill.
4. Bill referred to select committee.
Choose the correct answer using the codes
below.
a)
b)
c)
d)

3142
1423
3412
3421

36. Consider the following about the


Economic Survey.
1. It is published by the Ministry of
Finance.
2. It is presented along with the budget
to the Lok Sabha.
3. It is not presented to Rajya Sabha.
Choose the correct answer using the codes
below.
a)
b)
c)
d)

1 and 2 only
3 only
2 and 3 only
1 only

http://www.insightsonindia.com INSIGHTS

37. Consider a situation where the


incumbent executive has been found
guilty of financial irregularities based on
a CAG report as examined by the Public
Accounts Committee. The ultimate
responsibility to take appropriate action
on this report lies on?
a) Prime Minister of India
b) President of India
c) Parliament
d) Supreme Court

38. The International NorthSouth


Transport Corridor will directly connect
India with which of these countries?
1. Russia
2. Europe
3. Central Asia
4. Iran
5. Azerbaijan
Choose the correct answer using the codes
below.
a)
b)
c)
d)

2, 3 and 5 only
1, 3 and 4 only
1 and 4 only
All of the above

39. Consider the following about the project


SETI being led by Stephen Hawking?
1. It will be a human space mission to
search for extra terrestrial
intelligence.
2. The project will rely on sending
decrypted electromagnetic messages
in space.
3. The project has been undertaken in
joint collaboration with NASA.
Page 9

INSIGHTS MOCK TEST - 31


4. It is financial backed by the UK


Government.
Choose the correct answer using the codes
below.
a)
b)
c)
d)

1 only
2 and 3 only
1 and 4 only
None of the above

40. After the death of Aurangzeb in 1707,


Delhi could no longer function as an
effective centre. What can be the
reason(s)?
1. Mughal governors (subedars)
asserted their authority over Mughal
Kingdoms.
2. Zamindars started establishing
regional kingdoms.
3. After his Death, British immediately
adopted the policy of Paramountcy.
Choose the correct answer using the codes
below.
a)
b)
c)
d)

1 and 2 only
1and 3 only
2 and 3 only
All of the above

41. The reason(s) behind Battle of Plassey


between the Nawab of Bengal and the
British was/were
1. British denied stopping trade
relations with India
2. Bengal Nawab did not accept
subsidiary alliance enforced by the
British.
http://www.insightsonindia.com INSIGHTS

3. East India Company had started


interfering in the political matters of
Bengal.
4. British did not pay appropriate
revenue to Bengal Nawab.
5. East India Company did not stop
fortification in Bengal even after the
farman of Bengal Nawab.
Choose the correct answer using the codes
below.
a)
b)
c)
d)

1, 2 and 3 only
4 and 5 only
3, 4 and 5 only
All of the above

42. Consider the following about Gandhian


ideology?
1. Gandhi was opposed to the existing
state because it was forced and based
on centralization of authority.
2. As Per Gandhi, an all powerful state
impeded the progress of an
individual in society.
3. Gandhi held that State represents
violence in organized and
concentrated form.
Choose the correct answer using the codes
below.
a)
b)
c)
d)

1 and 2 only
2 and 3 only
1 and 3 only
All of the above

Page 10

INSIGHTS MOCK TEST - 31


43. Which of these is/are NOT principle(s)


of United Nations?
1. Some sovereigns are more equal than
others.
2. Member nations have rights but no
obligations.
3. All members shall settle their
international disputes by peaceful
means.
4. All members shall settle their
national disputes by peaceful means.
Choose the correct answer using the codes
below.
a)
b)
c)
d)

1 only
2 and 3 only
3 and 4 only
1, 2 and 4 only

44. Which of the following concerning


Wodeyar Dynasty is incorrect?
a) After Tipu Sultan was killed in the
Battle of Seringapatam, Mysore was
placed under Wodeyars the former
ruling dynasty.
b) The kingdom was incorporated into
the Dominion of India after its
independence from British rule.
c) Under British rule, Wodeyar Dynasty
was exempted from Subsidiary
Alliance.
d) None of the above

Reason (R): Suffering is caused by Ignorance


which is due to attachment, greed and
delusion.
In the context of the statements above, which
of these is true?
a) A and R both are true, and R is the
correct explanation for A.
b) A and R both are true, and R is the
NOT the correct explanation for A.
c) A is correct, R is incorrect.
d) A is incorrect, R is correct.

46. In Jainism, Abhayadana was related


with
a) Feeding the poor and homeless
b) Imparting knowledge to the ignorant
c) Giving protection to someone under
threat
d) Selfless service given by the student
to the teacher

47. In the later Vedic period, Bhagadudha


was
a) a local priest who performed
sacrifices for Brahmin households
b) an officer responsible for collection
of taxes
c) the village guard appointed by the
monarchy
d) a jurist who settled civil disputes in
the village

45. Consider the following statements about


the teachings of Buddha.
Assertion (A): Every living being is in the state
of suffering and none is exception to this.
http://www.insightsonindia.com INSIGHTS

Page 11

INSIGHTS MOCK TEST - 31


48. Gandhi suspended which of these


movements calling it a Himalayan
Blunder?
a) Non-cooperation movement
b) Satyagraha movement against
Rowlatt Act
c) Quit India movement
d) Civil Disobedience Movement

49. Consider the following statements about


Vaishnavism.
Assertion (A): Its spread contributed to the
growth of regional literature.
Reason (R): It was adopted and followed by all
then contemporary regional poets and prose
writers.
In the context of the statements above, which
of these is true?
a) A and R both are true, and R is the
correct explanation for A.
b) A and R both are true, and R is the NOT
the correct explanation for A.
c) A is correct, R is incorrect.
d) A is incorrect, R is correct.

50. The book 'One Part Woman' or


Madhurobhagan was recently in news.
Why?
a) Its writer has won the SAARC
Literary Award recently.
b) The book was appreciated recently by
the President of India for its depth
on the issues afflicting transgender in
India.

http://www.insightsonindia.com INSIGHTS

c) It is the first Indian book written on


the plight of transgender in ancient
India.
d) None of the above

51. NASAs New Horizons spacecraft was in


news recently because
a) It is the first spacecraft to
successfully fly by Pluto.
b) It is first spacecraft to land safely on
an asteroid orbiting the Sun.
c) It recently escaped the solar system
to reach Kuiper belt.
d) It spotted an extremely dwarf black
hole in the solar system.

52. The rarest of the rare principle in


handing out the death penalty was given
by the Supreme Court in which case?
a) Shatrughan Chauhan vs Union of
India
b) Bachan Singh vs State of Punjab
c) Mithu vs State of Punjab
d) Ladani vs Union of India

53. Consider the following about the BRICS


New Development Bank.
1. Agreement for establishing NDB was
made at Fortaleza.
2. Bank will be headquartered in
Shanghai.
3. Banks first President will be
Kundapur Vaman Kamath from
India.
4. All member nations will have an
equal say in Banks management.
5. All member nations will contribute
equally to Banks start-up capital.
Page 12

INSIGHTS MOCK TEST - 31


Choose the correct answer using the codes


below.
a)
b)
c)
d)

1 , 4 and 5 only
2 and 3 only
1, 2, 3 and 4 only
All of the above

54. According to a survey of trust in national


governments for 2014 conducted by the
Organisation for Economic Cooperation
and Development (OECD), India has
secured second position amongst 40
countries. According to the survey
change in trust levels can be affected by?
1. Economic outlook
2. Disasters
3. Major scandals
4. Election results
Choose the correct answer using the codes
below.
a)
b)
c)
d)

1 and 4 only
2 and 3 only
1, 2 and 3 only
All of the above

55. You are a District Magistrate. You


denied allotting a huge tender to an
influential local businessman. To seek
revenge, he is posting defamatory and
offensive speeches against you on the
internet. Which legal recourse can you
take against him?
a) Approach the State high court
accusing the businessman of abusing
freedom of speech given under
Article 19
http://www.insightsonindia.com INSIGHTS

b) File a case in local police station


under Section 66A of the
Information Technology Act
c) File a case with the local police
station under the IPC Section
499/500
d) Both (b) and (c)

56. The 12th International Symposium on


Antarctic Earth Sciences (ISAES) was
recently inaugurated at a function in
Goa. The theme(s) of the symposium
is/are
1. Conservation strategies for
Antarctica
2. Plate movements in Antarctica
3. Human habitation in Antarctica
Choose the correct answer using the codes
below.
a)
b)
c)
d)

1 and 2 only
2 and 3 only
1 and 3 only
2 only

57. As per the terms arrived at in the Iran


Nuclear Deal
1. Iran will not enrich any Uranium.
2. Iran will be subject to oversight from
US Nuclear authorities.
3. All previous nuclear installations will
be shutdown in a period of time.
4. Iran will not be able to share nuclear
technology with any nation.
Choose the correct answer using the codes
below.
a) 1 and 4 only
Page 13

INSIGHTS MOCK TEST - 31


b) 1 and 3 only
c) 2 and 3 only
d) None of the above

58. The Defence Acquisition Council (DAC)


that approves major defence related
projects consist of
1. Minister of State for Defence
2. Cabinet Secretary
3. Chiefs of Army, Navy and Air Staff
4. Defence Secretary
5. Director, Intelligence Bureau
6. Director, RAW
7. National Security Advisor
Choose the correct answer using the codes
below.
a)
b)
c)
d)

1, 2, 5, 6 and 7 only
1, 3 and 4 only
2, 3, 4 and 7 only
All of the above

59. While driving in an urban area, you find


out that the Air Quality Index (AQI) for
the area is X coloured. Air of this
quality may cause respiratory impact
even on healthy people, and serious
health impacts on people with
lung/heart disease. Which colour is X
most likely to be?
a) Light Pink
b) Yellow
c) Maroon
d) Blue

http://www.insightsonindia.com INSIGHTS

60. The government has decided to set up a


trade facilitation council. Consider the
following with reference to it.
1. The council will be chaired by the
Union Commerce and Industry
Minister.
2. It will be made a sub-organ of
SAARC.
3. It will provide a single window
mechanism for clearance of major
exports and imports.
4. It will include Ministers from states
also.
Choose the correct answer using the codes
below.
a)
b)
c)
d)

1 and 4 only
1 and 3 only
2 and 3 only
None of the above

61. Ottapayam situated in Kerala was


recently in news. Why?
a) Indias first defence industrial park
will be situated here.
b) Indias biggest disabled friendly IT
Park will be constructed here
supported entirely by World Bank.
c) The first NIMZ in South India will be
built here.
d) It will host the biggest International
Environmental Conservation
Conference held in India till date.

Page 14

INSIGHTS MOCK TEST - 31


62. Consider the following with reference to


the National Pharmaceutical Pricing
Policy (NPPP).
1. Pharmaceutical companies cannot
revise the cost of drugs under the
policy without permission from the
National Pharmaceutical Pricing
Authority.
2. It is applicable to all drugs on the
national list on life-saving medicines.
3. Drug pricing in the policy is fixed
based on a formula.
Which of the above is/are correct?
a)
b)
c)
d)

1 and 2 only
2 and 3 only
1 and 3 only
All of the above

63. A geographical indication (GI) is a name


or sign used on certain products which
corresponds to a specific geographical
location or origin. Which of these are
GIs in India?
1. Mysore Silk
2. Darjeeling Tea
3. Kancheepuram Saree
4. Thanjavur Paintings
Choose the correct answer using the codes
below.
a)
b)
c)
d)

1 and 4 only
1 and 3 only
2 and 3 only
1, 2 and 4 only

http://www.insightsonindia.com INSIGHTS

64. Jansankhya Sthirata Kosh (JSK) or the


National Population Stabilisation Fund
is under
a) Ministry of Planning
b) Ministry of Health and Family
welfare
c) Ministry of Statistics and Program
implementation
d) None of the above

65. As per the Antarctica treaty


1. No commercial activity is allowed in
Antarctica.
2. Each member nation will adopt one
of Antarcticas endangered species.
3. All claims to the Antarctica territory
are void.
4. Any scientific study needs the
approval of Scientific Committee on
Antarctic Research (SCAR).
Choose the correct answer using the codes
below.
a)
b)
c)
d)

1 and 3 only
2 and 4 only
1, 3 and 4 only
All of the above

66. As compared to the unamended


principal land Acquisition act passed in
2013, the Right to Fair Compensation
and Transparency in Land Acquisition,
Rehabilitation and Resettlement
(Amendment) Ordinance, 2015 relaxes
provisions relating to which of the
following categories?
1. Social Impact assessment

Page 15

INSIGHTS MOCK TEST - 31


2. Restrictions on acquisition of multicropped land


3. Consent for public private
partnerships (PPPs) projects
Choose the correct answer using the codes
below.
a)
b)
c)
d)

1 and 2 only
2 and 3 only
1 and 3 only
All of the above

67. Consider the following about the Stone


chariot at Vittala Temple, Hampi.
1. It is made of granite.
2. An image of Garuda was originally
enshrined within its sanctum.
3. It is a monolithic structure.
4. Its base platform is carved with
mythical battle scenes.
5. Two horses are positioned before the
chariot as if they are pulling it.
Choose the correct answer using the codes
below.
a)
b)
c)
d)

2, 3 and 5 only
1, 2, 3 and 4 only
1, 2 and 4 only
1, 3 and 5 only

68. Consider the following about Sun


Temple, Konark.
1. It is an example of Kalinga
architecture.
2. Temple was designed as a gigantic
chariot of Sun God.
3. The alignment of the Sun Temple is
on the east-west direction.
http://www.insightsonindia.com INSIGHTS

4. It is a UNESCO World Heritage Site.


5. The temple was originally built at the
mouth of the river Chandrabhaga.
Choose the correct answer using the codes
below.
a)
b)
c)
d)

2, 3 and 5 only
1, 2, 3 and 4 only
1, 4 and 5 only
All of the above

69. The Basilica of Bom Jesus located in


Goa, India is an important cultural
heritage because
a) It is the oldest church in World.
b) It contains the mortal remains of St.
Francis.
c) It was constructed jointly by all
colonial powers that came to India.
d) While the exterior is baroque
architecture, interior is highly
contrastingly Hindu and Mughal
architecture which is nowhere found
in the world.
70. The famous Padmapani painting in the
Ajanta Caves depicts
a) a bodhisattva with a blue lotus flower
sitting among a crowd of devotees
b) a feminine Buddhist Sangha being
ordained by Gautama Buddha
c) Buddhas Mahaparinirvana being
witnessed by his very close disciples
d) Buddha conquering Mara of all
worldly passions
71. Cloudbursts can be caused by
1. Excess of electrostatic induction in
the clouds
2. High jet stream velocity in the upper
atmosphere
Page 16

INSIGHTS MOCK TEST - 31


3. Mixing of warm air parcel with


cooler air resulting in sudden
condensation
Choose the correct answer using the codes
below.
a)
b)
c)
d)

1 and 2 only
2 and 3 only
1 and 3 only
All of the above

72. The Karnataka High Court has asked the


Karnataka Forest department to
consider banning eucalyptus plantations
across the state. Why?
a) It is not natural to the Karnataka
region and is affecting the
biodiversity of the region.
b) Tall eucalyptus trees do not allow
sufficient sunlight and nutrition to
other forest trees.
c) Large area of forest was already
replaced with monoculture
eucalyptus and planting more trees
would affect the species diversity of
the region.
d) The eucalyptus tree consumes a lot of
water for its growth depleting
groundwater levels.
73. Consider the following about GSLV
Mark III developed by ISRO.
1. It features an Indian cryogenic
technology.
2. It can launch communication
satellites in space.
3. It is presently being used for
commercial satellite launches by
ISRO.

http://www.insightsonindia.com INSIGHTS

Choose the correct answer using the codes


below.
a)
b)
c)
d)

1 and 2 only
2 and 3 only
1 and 3 only
All of the above

74. Presently space activities in India are


guided by
1. Satellite Communications Policy,
2000
2. International Agreements
3. Revised Remote Sensing Policy 2011
Choose the correct answer using the codes
below.
a)
b)
c)
d)

1 and 2 only
2 and 3 only
1 and 3 only
All of the above

75. What is/are the benefit(s) of having a


separate and unified National
Renewable Energy Act (NERA)?
1. The Ministry of New and renewable
energy need not depend on other
ministries for necessary clearances.
2. The law will clarify institutional
mechanisms related with financing,
monitoring, planning and support
for renewable energy generation.
Which of the above is/are correct?
a)
b)
c)
d)

1 only
2 only
Both 1 and 2
None

Page 17

INSIGHTS MOCK TEST - 31


76. The collective self-defence doctrine


presently being pushed by the Japanese
government means
a) Japan is not in favour of independent
armies for all nations
b) Japan may wish to send troops
abroad to rescue allies under attack
c) Japan will intervene constructively in
all international negotiations that
have significant geo-strategic
implications
d) Japan will make military
conscription compulsory for its
citizens in larger national interest
77. Why fossils are generally found in lowlying areas?
a) Faulting and folding of earths crust
puts low-lying areas under very high
temperature and pressure
b) Low-lying areas are at a higher
temperature than areas of higher
altitude, hence more organic matter
is converted to fossil fuels
c) Deposition of sediments is carried by
rivers that also carry organic matter
d) Volcanic magma spreads only on
low-lying areas which is a significant
source of fossils
78. Consider two geo-morphological cases.
1. A continental plate colliding with an
oceanic plate and the latter subsiding
below the former.
2. Two continental plates colliding.
Which of these outcomes is possible in both the
cases?
a) Volcanic mountains may be created
in both the cases.
http://www.insightsonindia.com INSIGHTS

b) Fold Mountains will be created in


both the cases.
c) Fold Mountains may be created in
case 1 and volcanic mountains in
case 2.
d) Volcanic Mountains may be created
in case 1 and Fold Mountains in case
2.
79. Most of the rivers in Africa cannot be
navigated fully. Why?
a) They follow highly tortuous and
narrow beds.
b) A large number of rivers flow
through narrow rift valleys.
c) They form waterfalls.
d) River flow in most of the rivers is too
less to become navigable.

80.Atmosphere is heated mainly from


1. Solar insolation reflected back to
upper troposphere by clouds
2. Convection from heated surface of
the earth
3. Direct solar radiation
Choose the correct answer using the codes
below.
a)
b)
c)
d)

1 and 2 only
2 and 3 only
1 and 3 only
2 only

81. Consider the following about Katabatic


winds.
1. They are generally experienced
during winter.
2. They blow adjacent to Highlands.
Page 18

INSIGHTS MOCK TEST - 31


3. These winds may increase the


temperature and pressure of an area.
4. The winds are very dry.
Choose the correct answer using the codes
below.
a)
b)
c)
d)

1 and 4 only
2 and 3 only
1 and 3 only
All of the above

84. Consider the following statements.


1. All the nine planets (including Pluto)
of the solar system revolve around
the Sun in counter-clockwise
direction.
2. All planets in the solar system rotate
in the same direction in which they
revolve.
Which of the above is/are true?

82. How are tides a great aid in shipping


and navigation?
1. High tide increases the depth of
water near the coasts.
2. They take away the mud brought by
rivers preventing silting of harbours.
3. They make the coastline indented
which supports natural harbours.
Choose the correct answer using the codes
below.
a)
b)
c)
d)

1 and 2 only
2 and 3 only
1 and 3 only
All of the above

83. Some ports of even Polar Regions


remain ice-free during winter. This is
because of the
a) Influence of Warm currents across
the coasts
b) Global warming and its effect on
albedo of earth
c) High salt content in these oceans
that prevent freezing and thawing
d) Dense fog and mist in these regions
that trap solar heat
http://www.insightsonindia.com INSIGHTS

a)
b)
c)
d)

1 only
2 only
Both 1 and 2
None

85. Why is it that we always see one side of


the moon while the other side always
remains away from us?
a) Relative positions of Moon in Earths
orbit around the Sun form a circle.
b) Moon takes the same time to revolve
around the earth and rotate on its
own axis.
c) Frequent eclipsing of the Moon by
the Sun and other planets
d) None of the above

86. Consider the following about latitudes


and longitudes.
1. Equator and Prime meridian are the
largest circles that can be drawn on
earth.
2. All meridians are of equal length.
3. Time zone of a place is determined
both by longitude and latitude.

Page 19

INSIGHTS MOCK TEST - 31


Choose the correct answer using the codes


below.
a)
b)
c)
d)

1 and 2 only
2 and 3 only
1 and 3 only
2 only

87. Vertical sun rays have a greater heating


effect as compared to oblique rays
because
1. They cover shorter distance in
atmosphere to reach earth.
2. They are spread over a larger area on
earth.
Which of the above is/are correct?
a)
b)
c)
d)

1 only
2 only
Both 1 and 2
None

88. Who decides what images appear in the


currency notes that are printed by
Reserve Bank of India for money supply
in the economy?
a) Union Minister of Finance
b) RBI Governor
c) Director-General of Archaeological
Survey of India
d) None of the above

89. Giffen goods are those goods that


a) may show an inverse relationship of
quantity consumed with rising
consumer income
http://www.insightsonindia.com INSIGHTS

b) may show an unclear relationship of


quantity consumed with rising
consumer income
c) are consumed in more quantity
because they are in greater demand
in public
d) are in greater demand because they
are in scarcity

90. In Economic theory and policy making,


Consumer Welfare is deemed to be
maximized when
a) every individual is able to purchase
goods at the exact market price they
were willing to pay
b) a large number of individuals are
able to purchase goods at a much
lower market price that what they
were willing to pay
c) all individuals attain an ideal state of
having no demand for any more
goods and services
d) they are able to purchase goods
beyond their means of earning

91. Real Gross Domestic Product (GDP) of a


country will rise if
1. There is high inflation in the
economy but no growth in
production
2. There is high growth in production
but no inflation in the economy
3. There is very little growth in
production but heavy deflation in the
economy
4. There is very little deflation in the
economy but no growth in
production
Page 20

INSIGHTS MOCK TEST - 31


Choose the correct answer using the codes


below.
a)
b)
c)
d)

1 and 4 only
2 and 3 only
1 and 3 only
2 and 4 only

92. How are employment and interest rates


related in an economy?
a) Generally higher interest rates
impede a higher growth in
employment numbers.
b) Generally lower interest rates impede
a higher growth in employment
numbers.
c) There is no relationship of interest
rate and employment in an economy.
d) Depending on the type of economy,
any of the statements (a), (b) or (c)
can be true.
93. The Reserve Bank of India draws its
authority from
1. Constitution of India
2. Parliamentary Law
3. Conventions and Customs
4. Executive Council of Ministers
Choose the correct answer using the codes
below.
a)
b)
c)
d)

1 and 4 only
2 only
1 and 3 only
1, 2 and 3 only

94. Sterilization operation is undertaken by


the Central bank of a country to prevent
a shock mainly to the
a) Money supply in the economy
http://www.insightsonindia.com INSIGHTS

b) Foreign Exchange reserves


c) Bank deposits of Public
d) Revenues of Government
95. The situation of liquidity trap occurs in
the economy
1. When foreign exchange reserves dry
up.
2. When Bank deposits are low so as to
cause a Bank default on its
customers
3. When interest rates are very low so
that everyone anticipates a rise in
interest rates
4. When people dont wish to hold any
bonds
Choose the correct answer using the codes
below.
a)
b)
c)
d)

1 and 4 only
2 and 3 only
1 and 3 only
3 and 4 only

96. The main objective(s) of the Public


Distribution System (PDS) was/were to
1. Ensure optimum export of
agricultural commodities
2. To prevent any import of agricultural
produce
3. To ensure food security
4. To ensure healthy meals are taken by
each and every family in India
Choose the correct answer using the codes
below.
a)
b)
c)
d)

1, 2 and 4 only
2 and 3 only
3 only
3 and 4 only
Page 21

INSIGHTS MOCK TEST - 31


b) GDP structure is changing to favour


the Services sector more than the
industrial sector
c) Foreign capital inflows in the
economy are not as strong as
domestic investment
d) Financial sector in the country is
becoming increasingly globalized and
risky

97. Consider the following statements.


Assertion (A): Devaluation of currency may
lead to improvement in trade balance.
Reason (R): Devaluation of currency leads to
more competitive export prices.
In the context of the statements above, which
of these is true?
a) A and R both are true, and R is the
correct explanation for A.
b) A and R both are true, and R is the
NOT the correct explanation for A.
c) A is correct, R is incorrect.
d) A is incorrect, R is correct.

98. In which of the following situations an


expansion in the money supply may not
lead to inflation?
1. When economy is operating at a
much lower level of production than
its capacity
2. When supply of goods meets the
demands posed by the extra money
supply

100.
With reference to The Energy and
Resources Institute, consider the
following statements:
1. It was formerly known as Tata
Energy Research Institute
2. It is an autonomous research
institution which works under the
Ministry of Renewable and NonRenewable Energy Sources,
Gvernment of India
Which of the above statements is/are correct?
a)
b)
c)
d)

1 Only
2 Only
Both
None

Which of the above is/are correct?


a)
b)
c)
d)

1 only
2 only
Both 1 and 2
None

99. A country will NOT develop


economically if
a) Increasingly larger share of the GDP
is coming from the Primary sector
http://www.insightsonindia.com INSIGHTS

Page 22


INSIGHTS MOCK TEST SERIES 2015: TEST 31 SOLUTIONS

1. Solution: a)
Lichens are composite, symbiotic organisms
made up from members of as many as three
kingdoms. The dominant partner is a fungus.
Fungi are incapable of making their own food.
They usually provide for themselves as
parasites or decomposers.
The lichen fungi (kingdom Fungi) cultivate
partners that manufacture food by
photosynthesis. Sometimes the partners are
algae (kingdom Protista), other times
cyanobacteria (kingdom Monera), formerly
called blue-green algae. Some enterprising
fungi exploit both at once.
Pioneer species are hardy species which are the
first to colonize previously disrupted or
damaged ecosystems, beginning a chain of
ecological succession that ultimately leads to a
more biodiverse steady-state ecosystem.
Examples can be phytoplankton, zooplankton,
lichens, mosses etc.

2. Solution: c)
Sickle-cell disease (SCD), also known as sicklecell anaemia (SCA) and drepanocytosis, is a
hereditary blood disorder, characterized by an
abnormality in the oxygen-carrying
haemoglobin molecule in red blood cells.
Inherited means that the disease is passed by
genes from parents to their children. SCD is
not contagious. A person cannot catch it, like a
cold or infection, from someone else.
At the present time, hematopoietic stem cell
transplantation (HSCT) is the only cure for
http://www.insightsonindia.com INSIGHTS

SCD. Unfortunately, most people with SCD are


either too old for a transplant or dont have a
relative who is a good enough genetic match for
them to act as a donor. A well-matched donor
is needed to have the best chance for a
successful transplant.
http://www.thehindu.com/todays-paper/tpnational/sickle-cell-anaemia-stalks-adivasissays-study/article7431619.ece

3. Solution: a)
Currency swap agreements exist to assist
countries during periods of tight liquidity or
balance of payments and liquidity crises.
Under this agreement, Sri Lanka can draw a
maximum of $1.1 billion for a period of up to
six months.
The agreement is in addition to the existing
framework on currency swap arrangement for
the SAARC member countries, an arrangement
by which SAARC members can draw currency
$100 million-$400 million, with a total limit of
$2 billion, from an RBI financing facility set up
for this purpose.

4. Solution: d)
Zoos and botanical gardens are the most
conventional methods of ex situ conservation,
all of which house whole, protected specimens
for breeding and reintroduction into the wild
when necessary and possible.
Endangered plants may also be preserved in
part through seed banks or germ plasm banks.
The term seed bank sometimes refers to a
Page 1


INSIGHTS MOCK TEST SERIES 2015: TEST 31 SOLUTIONS

cryogenic laboratory facility in which the seeds


of certain species can be preserved for up to a
century or more without losing their fertility.
It can also be used to refer to a special type of
arboretum where seeds are harvested and the
crop is rotated. For plants that cannot be
preserved in seed banks, the only other option
for preserving germ plasm is in-vitro storage,
where cuttings of plants are kept under strict
conditions in glass tubes and vessels.
Also read the drawbacks of ex-situ
conservation
https://en.wikipedia.org/wiki/Ex_situ_conser
vation#Drawbacks

5. Solution: c)
Under the Wildlife (Protection) Act, the state
governments have to notify the list of core and
buffer areas of tiger reserves in their territory.
Core zone (critical tiger habitats) is where
tigers usually rest, reside, feed and breed.
Buffer zones are areas that lie in the periphery
of the core zone.
Buffer zones constitute the fringe areas i.e. the
outside boundary of tiger reserves up to 10
kms.

6. Solution: a)
Species like Blue whale and Whooping crane
are highly susceptible to extinction because of
the reasons stated in statement 1 and 2.
Island species are also likely to be extinct
because of their narrow range of distribution.
Moreover, species which are higher in food
chain have greater chances to be extinct, not
the ones that lie at the bottom of the food
chain.

7. Solution: d)
Mercury is used in laboratories for making
thermometers, barometers, diffusion pumps,
and many other instruments. It is used for
mercury switches and other electrical
apparatus. It is used as an electrode in some
types of electrolysis and for making batteries
(mercury cells).
http://www.insightsonindia.com INSIGHTS

Page 2


INSIGHTS MOCK TEST SERIES 2015: TEST 31 SOLUTIONS

Potassium dichromate has uses in photography


and in photographic screen printing, where it is
used as an oxidizing agent together with a
strong mineral acid.
The light sensitive silver halides, silver
chloride, silver bromide and silver iodide, are
used to make photographic film and
photographic paper.

8. Solution: b)
Apart from being used in manures seaweeds or
marine algae are a food source for humans
especially in East Asia, it is most commonly
associated with Japanese food. Seaweeds also
are used to make a number of food additives
such as alginates and carrageenan which is
used in cooking and baking as a vegetarian
alternative to gelatine.
Many seaweeds are used as medicine. Alginates
are used in wound dressings and in the
production of dental moulds and agar is used
very widely in Microbiology to help grow
bacterial cultures.

9. Solution: d)
Social forestry is defined as Forestry outside
the conventional forests which primarily aim at
providing continuous flow of goods and
services for the benefit of people. This
definition implies that the production of forest
goods for the needs of the local people is Social
forestry. Thus, social forestry aims at growing
forests of the choice of the local population
On the other hand, agro-forestry includes a
variety of land uses where woody species are
grown in combination with crops. For instance
crops can be grown in between rows of saal,
teak etc.

10.

Solution: d)

About 84% of total evaporation comes from the


oceans. They receive 77% of the total
precipitation.
16% evaporation comes from land. Land
receives 24% precipitation. There is a gain of
7% in land.
This excess is returned by surface run-off and
sub-surface flows to the ocean again.
Thus the water cycle maintains equilibrium.

Seaweeds are ingredients in toothpaste,


cosmetics and paints and are used in industrial
products such as paper coatings, adhesives,
dyes, gels, explosives and many more.
Much of the oil and natural gas we use today
formed from seaweeds which partially
decomposed on the sea floor many millions of
years ago.

http://www.insightsonindia.com INSIGHTS

11. Solution: b)
Production functions include producing furits,
woods, resins, alkaloids, medicines etc.
Page 3


INSIGHTS MOCK TEST SERIES 2015: TEST 31 SOLUTIONS

Protective functions include conservation of


soil and water, prevention of drought, shelter
against cold, wind and radiation, noise etc.
Regulative functions help improve atmosphere
and temperature conditions.
The above functions tell us that the economical
value of forests is very high and they play a very
important role in maintaining the biological
cycle on earth.

Psedomonas is a denitrifying bacterium. It


converts nitrate back to Nitrogen for release in
the atmosphere.

14. Solution: a)

It is a symbiotic relation of fungi with


roots of vascular plants.

Nearly all plants on earth rely on


mycorrhizal fungi for nutrients
and moisture. Many plants are
extremely dependent and can struggle to
survive without the beneficial fungi
(grapes and roses are examples).

Plants and mycorrhizal fungi


operate as a single working unit in
nature. The plant performs
photosynthesis and other above-ground
functions, and the fungi handle
underground nutrition-gathering and
protect the roots. It is not normal to
grow plants without mycorrhizae
on the roots this is often the
cause of disease and insect
problems!

Other benefits include enhanced rooting


of cuttings, increased root generation,
increased drought resistance, increased
salt tolerance, reduced transplant shock,
and enhancement of other valuable
organisms in the soil.

12. Solution: c)
Mineralization in soil science is decomposition
or oxidation of the chemical compounds
(minerals) in organic matter into plantaccessible forms. Mineralization is the opposite
of immobilization.
Immobilization (or demineralisation) in soil
science is the conversion of inorganic
compounds to organic compounds by microorganisms or plants, by which it is prevented
from being accessible to plants. Immobilization
is the opposite of mineralization.

13. Solution: c)
Bacteria that change nitrogen gas from the
atmosphere into solid nitrogen usable by plants
are called nitrogen-fixing bacteria. These
bacteria are found both in the soil and in
symbiotic relationships with plants.
Rhizobium is not a free-living bacterium. It is
tied to the host legume plant in a symbiotic
relationship.

http://www.insightsonindia.com INSIGHTS

Page 4


INSIGHTS MOCK TEST SERIES 2015: TEST 31 SOLUTIONS

Phosphates move quickly through plants and


animals; however, the processes that move
them through the soil or ocean are very slow,
making the phosphorus cycle overall one of the
slowest biogeochemical cycles. Rain water
contains phosphorus and nitrogen from air
pollution.
Initially, phosphate weathers from rocks and
minerals, the most common mineral being
apatite. Overall small losses occur in terrestrial
environments by leaching and erosion, through
the action of rain. In soil, phosphate is
absorbed on iron oxides, aluminium
hydroxides, clay surfaces, and organic matter
particles, and becomes incorporated
(immobilized or fixed). Plants and fungi can
also be active in making P soluble.
15. Solution: d)
It has been explained beautifully here
http://www.majordifferences.com/2014/03/di
fference-between-c3-and-c4plants.html#.VavrYaSqqko
C4 plants have a competitive advantage over
plants possessing the more common C3 carbon
fixation pathway under conditions of drought,
high temperatures, and nitrogen or CO2
limitation.

16. Solution: b)
Unlike many other biogeochemical cycles, the
atmosphere does not play a significant role in
the movement of phosphorus, because
phosphorus and phosphorus-based compounds
are usually solids at the typical ranges of
temperature and pressure found on Earth. P
cannot be found in the air as a gas.
http://www.insightsonindia.com INSIGHTS

17. Solution: b)
50% of available solar radiation is
photosynthetically active radiation. Rest of the
energy is deflected by the atmosphere.
You also need to know that visible light is the
only band of light on the spectrum to be
considered photosynthetically active. It has the
perfect amount of energy to excite the electrons
needed to start photosynthesis and not damage
DNA or break bonds.
Ultraviolet cannot be used for photosynthesis
because it has too much energy. This energy
breaks the bonds in molecules and can destroy
DNA and other important structures in
organism.

18.Solution: d)
Page 5


INSIGHTS MOCK TEST SERIES 2015: TEST 31 SOLUTIONS

that India and Korea will strengthen


their historic connection by enhancing
linkages of Korean people with Ayodhya.

Species that have strong effects on the


composition of communities are called
Keystone Species.
Example: Starfish, Humming Bird, Sea Otter,
African elephants, Beaver, Flying Fox, Prairie
dogs.
In addition to keystone species, there are other
categories of species that are crucial to their
ecosystem's survival.
Critical link Species are species which play an
important role in supporting network species
by functioning as pollinators, nutrient
circulators or absorbers. There are a large
number of critical link species in any
ecosystem.
The species which are found most abundantly
in ecotone boundary are known as edge
species.
Ecotone is defined as the place or area, where
two major communities meet and blend
together. It consists of species of both the
communities.

19. Solution: a)
http://indpaedia.com/ind/index.php/The_Kar
ak_clan_of_Queen_Hwang-ok/_Suri_Ratna

India has begun the work of tracing its


shared heritage with Korea using the
legendary Queen Suriratna, a princess
from Ayodhya who travelled to the
country to marry King Kim Suro in 48
AD, as a pivot.
During Prime Minister Narendra Modis
visit to Seoul in May, it was announced

http://www.insightsonindia.com INSIGHTS

A decision was also taken to upgrade the


monument for Queen Suriratna, also
know as Hur Hwang-ok, in Ayodhya as a
joint project between the two countries.

20.

Solution: d)

http://www.thehindu.com/todayspaper/heres-the-first-look-ofamaravati/article7439266.ece
http://www.deccanchronicle.com/150405/nati
on-current-affairs/article/all-you-need-knowabout-andhra-pradesh%E2%80%99s-capitalcity

21. Solution: c)
The Betwa or Betravati is a river in Northern
India, and a tributary of the Yamuna. Also
known as the Vetravati, the Betwa rises in the
Vindhya Range just north of Hoshangabad in
Madhya Pradesh and flows north-east through
Madhya Pradesh and Orchha to Uttar Pradesh.
Nearly half of its course, which is not
navigable, runs over the Malwa Plateau.
The Ken River is one of the major rivers of the
Bundelkhand region of central India, and flows
through two states, Madhya Pradesh and Uttar
Pradesh. It is a tributary of the Yamuna.
http://www.thehindu.com/todays-paper/tpnational/interlinking-of-rivers-vital-for-waterfood-security-minister/article7439174.ece
Page 6


INSIGHTS MOCK TEST SERIES 2015: TEST 31 SOLUTIONS

populations of other five union territories are


too small to have any representative in the
Rajya Sabha.

22.

Solution: c)

Though the President of India is not a member


of either House of Parliament and does not sit
in the Parliament to attend its meetings, he is
an integral part of the Parliament.
This is because a bill passed by both the
Houses of Parliament cannot become law
without the Presidents assent. He also
performs certain functions relating to the
proceedings of the Parliament, for example, he
summons and prorogues both the Houses,
dissolves the Lok Sabha, addresses both the
Houses, issues ordinances when they are not in
session, and so on.

23.

Solution: a)

The representatives of states in the Rajya


Sabha are elected by the elected members of
state legislative assemblies. The election is held
in accordance with the system of proportional
representation by means of the single
transferable vote.
The representatives of each union territory in
the Rajya Sabha are indirectly elected by
members of an electroral college specially
constituted for the purpose. This election is
also held in accordance with the system of
proportional representation by means of the
single transferable vote. Out of the seven union
territories, only two (Delhi and Puducherry)
have representation in Rajya Sabha. The
http://www.insightsonindia.com INSIGHTS

24.

Solution: c)

Under the Constitution, a person shall be


disqualified for being elected as a Member of
Parliament:

if he holds any office of profit under the


Union or state government (except that
of a minister or any other office
exempted by Parliament).
if he is of unsound mind and stands so
declared by a court.
if he is an undischarged insolvent.
if he voluntary (not expelled) gives
up the membership of the political party
on whose ticket he is elected to the
House
if he is not a citizen of India or has
voluntarily acquired the citizenship of a
foreign state or is under any
acknowledgement of allegiance to a
foreign state; and
if he is so disqualified under any law
made by Parliament.
He must not be a director or managing
agent nor hold an office of profit in a
corporation in which the government
has at least 25 per cent share.
He must not have been dismissed from
government service for corruption or
disloyalty to the State.

25.

Solution: a)
Page 7


INSIGHTS MOCK TEST SERIES 2015: TEST 31 SOLUTIONS

The Speaker is elected by the Lok Sabha from


amongst its members (as soon as may be, after
its first sitting). Whenever the office of the
Speaker falls vacant, the Lok Sabha elects
another member to fill the vacancy. The date of
election of the Speaker is fixed by the
President.

Speaker is the final interpreter of the


provisions of (a) the Constitution of India, (b)
the Rules of Procedure and Conduct of
Business of Lok Sabha, and (c) the
parliamentary precedents, within the House.
Powers of Chairman, Rajya Sabha are the
same.

Usually, the Speaker remains in office during


the life of the Lok Sabha. However, he has to
vacate his office earlier in any of the following
three cases:

Moreover, they derive their powers and duties


from three sources, that is, the Constitution of
India, the Rules of Procedure and Conduct of
Business of Lok Sabha, and Parliamentary
Conventions (residuary powers that are
unwritten or unspecified in the Rules).

o if he ceases to be a member of the


Lok Sabha;
o if he resigns by writing to the
Deputy Speaker; and
o if he is removed by a resolution
passed by a majority of all the
members of the Lok Sabha.
Such a resolution can be moved only after
giving 14 days advance notice.
When a resolution for the removal of the
Speaker is under consideration of the House,
he cannot preside at the sitting of the House,
though he may be present. However, he can
speak and take part in the proceedings of the
House at such a time and vote in the first
instance, though not in the case of an equality
of votes.
It should be noted here that, whenever the Lok
Sabha is dissolved, the Speaker does not vacate
his office and continues till the newly- elected
Lok Sabha meets.

26.

Solution: d)

http://www.insightsonindia.com INSIGHTS

27.

Solution: c)

The institutions of Speaker and Deputy


Speaker originated in India in 1921 under the
provisions of the Government of India Act of
1919 (MontagueChelmsford Reforms). At that
time, the Speaker and the Deputy Speaker were
called the President and Deputy President
respectively and the same nomenclature
continued till 1947.
Before 1921, the Governor- General of India
used to preside over the meetings of the
Central Legislative Council. In 1921, the
Frederick Whyte and Sachidanand Sinha were
appointed by the Governor-General of India as
the first Speaker and the first Deputy Speaker
(respectively) of the central legislative
assembly. In 1925, Vithalbhai J. Patel became
the first Indian and the first elected Speaker of
the central legislative assembly.

Page 8


INSIGHTS MOCK TEST SERIES 2015: TEST 31 SOLUTIONS

The Government of India Act of 1935 changed


the nomenclatures of President and Deputy
President of the Central Legislative Assembly
to the Speaker and Deputy Speaker
respectively.

28.

Solution: c)

The concept of equality before law is an


element of the concept of Rule of Law,
propounded by A.V. Dicey, the British jurist.
His concept has the following three elements or
aspects:

members are from Lok Sabha only. The Rajya


Sabha has no representation in this committee.
These members are elected by the Lok Sabha
every year from amongst its members,
according to the principles of proportional
representation by means of a single
transferable vote.
Thus, all parties get due representation in it.
The term of office is one year. A minister
cannot be elected as a member of the
committee. The chairman of the committee is
appointed by the Speaker from amongst its
members and he is invariably from the ruling
party.

(i) Absence of arbitrary power, that is, no man


can be punished except for a breach of law.
30.

Solution: b)

(ii) Equality before the law, that is, equal


subjection of all citizens (rich or poor, high or
low, official or non-official) to the ordinary law
of the land administered by the ordinary law
courts.

Financial committees of Estimates, PSUs, and


Public Accounts keep track of expenditure, its
usefulness and performance of the
administrative spending.

(iii) The primacy of the rights of the individual,


that is, the constitution is the result of the
rights of the individual as defined and enforced
by the courts of law rather than the
constitution being the source of the individual
rights.

Some laws need to be filled with administrative


details by the instrument of delegated
legislation. The committee on delegated
legislation scrutinizes such rules and
regulations formed by the executive or
bureaucracy.

The first and the second elements are


applicable to the Indian System and not the
third one. In the Indian System, the
constitution is the source of the individual
rights.

Zero hour and question hour are used to raise


any matter in Lok Sabha. The administration
can be held accountable.

31. Solution: d)
29.

Solution: a)

Originally, it had 25 members but in 1956 its


membership was raised to 30. All the thirty
http://www.insightsonindia.com INSIGHTS

Article 75 of the Constitution says that the


council of ministers shall be collectively
responsible to the Lok Sabha. It means that the
Page 9


INSIGHTS MOCK TEST SERIES 2015: TEST 31 SOLUTIONS

ministry stays in office so long as it enjoys


confidence of the majority of the members of
the Lok Sabha.

fundamental rights shall be void. In other


words, it expressively provides for the doctrine
of judicial review.

In other words, the Lok Sabha can remove the


ministry from office by passing a no-confidence
motion. The motion needs the support of 50
members to be admitted.

This power (not source of power, read


carefully) has been conferred on the Supreme
Court (Article 32) and the high courts (Article
226) that can declare a law unconstitutional
and invalid on the ground of contravention of
any of the Fundamental Rights.

32.

Solution: d)

Popular sovereignty is a basic idea of


democracy. Popular sovereignty means that the
people are the ultimate source of the authority
of their government.
Popular sovereignty means that democratic
government is by the people and for the
peoplefor the benefit of the people, not for
the benefit of those who govern in their name.
Government in a democracy is the servant of
the people; it is not their master. Therefore,
those who govern are public servantsthey
hold public office only to serve the
people, not to serve themselves. In a
democracy, political authority flows from the
people to the governmentnot from
government to the People.
Popular sovereignty means that the
government can only exercise authority if it has
been given permission to do so by the
People. Therefore, popular sovereignty limits
the powers of government.

33.

Solution: c)

Article 13 declares that all laws that are


inconsistent with or in derogation of any of the
http://www.insightsonindia.com INSIGHTS

Thus, not only a legislation but any of the


above can be challenged in the courts as
violating a Fundamental Right and hence, can
be declared as void.

34.

Solution: d)

UPSC only certifies eligible candidates.


Appointment is done by DoPT (Government of
India).
Service Conditions of AIS are determined by
the Parliament.
Members of AIS cannot be removed by an
authority subordinate to the one that appointed
it. President appoints members of AIS. So, they
cannot be removed by the PM.

35.

Solution: a)

The Appropriation Bill becomes the


Appropriation Act after it is assented to by the
President. This act authorises (or legalises) the
payments from the Consolidated Fund of India.
This means that the government cannot
withdraw money from the Consolidated Fund
of India till the enactment of the appropriation
Page 10


INSIGHTS MOCK TEST SERIES 2015: TEST 31 SOLUTIONS

bill. This takes time and usually goes on till the


end of April.
But the government needs money to carry on
its normal activities after 31 March (the end of
the financial year). To overcome this functional
difficulty, the Constitution has authorised the
Lok Sabha to make any grant in advance in
respect to the estimated expenditure for a part
of the financial year, pending the completion of
the voting of the demands for grants and the
enactment of the appropriation bill.
This provision is known as the vote on
account. It is passed (or granted) after the
general discussion on budget is over. It is
generally granted for two months for an
amount equivalent to one-sixth of the total
estimate.

36.

Solution: d)

The Finance Ministry of India presents the


Economic Survey in the parliament every year,
just before the Union Budget. It is the
ministry's view on the annual economic
development of the country.
Economic Survey reviews the developments in
the Indian economy over the previous 12
months, summarizes the performance on major
development programs, and highlights the
policy initiatives of the government and the
prospects of the economy in the short to
medium term. This document is presented to
both houses of Parliament during the Budget
Session (not along with the budget).

37.

Solution: c)

http://www.insightsonindia.com INSIGHTS

Every department and official is given


information about the amount of funds they
have and they have to perform within it - if it
goes overboard then they are accountable.
These funds are passed by the
Parliament/Legislature through voting. The
auditing and reporting work is ex post facto as
it is not easy to get the finance back once the
Finance Ministry sanctions funds to the
departments.
So, ultimately Parliament has to hold the
executive financially accountable.

38.

Solution: d)

The International NorthSouth Transport


Corridor is the ship, rail, and road route for
moving freight between India, Russia, Iran,
Europe and Central Asia. The route primarily
involves moving freight from India, Iran,
Azerbaijan and Russia via ship, rail and road.
The objective of the corridor is to increase
trade connectivity between major cities such as
Mumbai, Moscow, Tehran, Baku, Bandar
Abbas, Astrakhan, Bandar Anzali and etc.
Dry runs of two routes were conducted in 2014,
the first was Mumbai to Baku via Bandar Abbas
and the second was Mumbai to Astrakhan via
Bandar Abbas, Tehran and Bandar Anzali.

39.

Solution: d)
Page 11


INSIGHTS MOCK TEST SERIES 2015: TEST 31 SOLUTIONS

British cosmologist Steven Hawking on


Monday launched the biggest-ever search for
intelligent life in the universe in a 10-year,
$100-million (143-million-euro) project to scan
the heavens.

The Breakthrough Listen project,


backed by Russian Silicon Valley
entrepreneur Yuri Milner, will be the
most powerful, comprehensive and
intensive scientific search ever
undertaken for signs of extra-terrestrial
intelligent life.
The project will use some of the biggest
telescopes on Earth, searching far
deeper into the universe than before for
radio spectrum and laser signals.
The initiative is allied with the
Breakthrough Message project, an
international competition to create
digital messages that represent
humanity.

There is no commitment to send any messages


into space, and the project should spark
discussion about whether humans should be
sending messages at all out into the void.

40.

Solution: a)

Aurangzeb was the last of the powerful Mughal


rulers.
He established control over a very large part of
the territory that is now known as India. After
his death in 1707, many Mughal governors
(subadars) and big zamindars began asserting
their authority and establishing regional
kingdoms.
http://www.insightsonindia.com INSIGHTS

As powerful regional kingdoms emerged in


various parts of India, Delhi could no longer
function as an effective centre.

41. Solution: c)
When Alivardi Khan died in 1756,
Sirajuddaulah became the nawab of Bengal.
The Company was worried about his power and
keen on a puppet ruler who would willingly
give trade concessions and other privileges. So
it tried, though without success, to help one of
Sirajuddaulahs rivals become the nawab.
An infuriated Sirajuddaulah asked the
Company to stop meddling in the political
affairs of his dominion, stop fortification, and
pay the revenues. After negotiations failed, the
Nawab marched with 30,000 soldiers to the
English factory at Kassimbazar, captured the
Company officials, locked the warehouse,
disarmed all Englishmen, and blockaded
English ships. Then he marched to Calcutta to
establish control over the Companys fort there.
On hearing the news of the fall of Calcutta,
Company officials in Madras sent forces under
the command of Robert Clive, reinforced by
naval fleets. Prolonged negotiations with the
Nawab followed. Finally, in 1757, Robert Clive
led the Companys army against Sirajuddaulah
at Plassey.

42.

Solution: d)

This page deals brilliantly with Gandhian


ideology of democracy and state.

Page 12


INSIGHTS MOCK TEST SERIES 2015: TEST 31 SOLUTIONS

http://www.mkgandhi.org/articles/indian_de
mocracy.html
Mahatma Gandhis imagination of the
democracy -fully encircled with non-violence exists in no nation of the world as up to now.
Democracy of his imagination happens to be
one, which does not have any provision of
punishment and even an organization like
State happens to be obsolete in it. This is
because Mahatma Gandhi holds, State is
symbolical of centralized and organized
violence. As non-violence is connected with
human soul, man can be non-violent whereas
in opposition to it, State is a soul-less
machine. On this accord, it is impossible to get
rid of violence. Its very existence depends upon
violence. Philosophy of Mahatma Gandhi,
non-violence should be admitted as invariable
part of our life and it is on the basis of this
dictum that modem polities must operate.

from membership, shall fulfill in good


faith the obligations assumed by
them in accordance with the present
Charter.

All Members shall settle their


international disputes by peaceful
means in such a manner that
international peace and security, and
justice, are not endangered.

All Members shall refrain in their


international relations from the threat
or use of force against the territorial
integrity or political independence of
any state, or in any other manner
inconsistent with the Purposes of the
United Nations.

All Members shall give the United


Nations every assistance in any action it
takes in accordance with the present
Charter, and shall refrain from giving
assistance to any state against which the
United Nations is taking preventive or
enforcement action.

The Organization shall ensure that


states which are not Members of the
United Nations act in accordance with
these Principles so far as may be
necessary for the maintenance of
international peace and security.

Nothing contained in the present


Charter shall authorize the United
Nations to intervene in matters which
are essentially within the domestic
jurisdiction of any state or shall require
the Members to submit such matters to
settlement under the present Charter;

While in the present day democracy, there is a


great deal of centralization and inequality. In a
stateless democracy there is decentralization
and equality.

43.

Solution: d)

The Organization and its Members, in pursuit


of the Purposes stated in Article 1, shall act in
accordance with the following Principles.

The Organization is based on the


principle of the sovereign equality of
all its Members.

All Members, in order to ensure to all of


them the rights and benefits resulting

http://www.insightsonindia.com INSIGHTS

Page 13


INSIGHTS MOCK TEST SERIES 2015: TEST 31 SOLUTIONS

but this principle shall not prejudice the


application of enforcement measures
under Chapter Vll.

44.

Solution: c)

The Wadiyar dynasty was an Indian Hindu


dynasty that ruled the Kingdom of Mysore
from 1399 to 1947.
It was in news recently due to the coronation of
the new prince.
http://www.thehindu.com/news/national/kar
nataka/yaduveer-gopal-raj-urs-is-new-heir-ofmysuru-royal-family/article6886404.ece
http://indianexpress.com/article/india/indiaothers/prince-of-mysore-srikantadattanarasimharaja-wodeyar-passes-away/
Four wars were fought with Mysore (1767-69,
1780-84, 1790-92 and 1799). Only in the last
the Battle of Seringapatam did the Company
ultimately win a victory. Tipu Sultan was killed
defending his capital Seringapatam, Mysore
was placed under the former ruling dynasty of
the Wodeyars and a subsidiary alliance was
imposed on the state.

45.

Solution: d)

The Buddha told us, "The nature of Human life


is suffering". (He did not say all living being
suffer).
This suffering has a case which is ignorance of
what is.

http://www.insightsonindia.com INSIGHTS

Suffering can be ended by knowing the four


noble truths and following the eightfold path.
They essentially centre on ending ignorance by
following dharma, right meditation,
contemplation etc.
You can understand their meanings here
http://www.buddhanet.net/elearning/intro_bud.htm

46.

Solution: c)

Dana is a virtue and duty in Jainism. It is


considered an act of compassion, and must be
done with no desire for material gain.
Four types of Dana are discussed in the texts of
Jainism: Ahara-dana (donation of food),
Ausadha-dana (donation of medicine), Jnanadana (donation of knowledge) and and Abhayadana (giving of protection or freedom from
fear, asylum to someone under threat).
Dna is one of ten means to gain positive
karma, in the soteriological theories of Jainism.
Medieval era texts of Jainism dedicate a
substantial portion of their discussions to the
need and virtue of Dna.

47.

Solution: b)

Large Kingdoms and stately cities made their


appearance in the later Vedic Period. In
Taittariya Brahmana you will notice the theory
of the divine origin of kingship.
The government machinery became more
elaborate than before , as a sequel to the
growth of the power of the king . New civil
functionaries, besides the only civil functionary
Page 14


INSIGHTS MOCK TEST SERIES 2015: TEST 31 SOLUTIONS

of the Rigvedic Period the purohita came into


existence. These were : the Bhagadudha (
Collector of taxes), the Suta/ Sarathi (the
Royalherald or Charioteer ), the Khasttri
(Chamberlain), The Akshavapa (Courier).
The military officials of the Rigvedic times, the
Senani (the head of the village ) continued to
function.

48.

Solution: b)

Gandhis entry into public life began with the


Satyagraha in Champaron district of Bihar in
1917. He could mobilize the peasants of this
district against the exploitation of European
indigo planters. In 1918, Gandhiji led a no tax
campaign at Khera in Gujarat where the
peasants were not able to pay the revenue due
to famine.
But, after the Jalianwala Bagh tragedy, the
government expressed no sign of regret but
went ahead with more repression.
Mahatma Gandhi was shocked and suspended
the Satyagraha declaring it as a Himalayan
Blunder.
It was because he had asked those people to
pursue non-violence who could not afford to be
non-violent. The Satyagraha movement failed
in attaining its object as the government did
not withdraw the Rowlatt Act. However, it was
the first experiment of non-violence of
Gandhiji in Indian politics.

49.

Solution: c)

http://www.insightsonindia.com INSIGHTS

Vaishnavism (Vaisnava dharma) is one of the


major branches of Hinduism along with
Shaivism, Smartism, and Shaktism. It is
focused on the veneration of Vishnu.
Vaishnavites, or the followers of the Vishnu,
lead a way of life promoting the central
importance of Vishnu and his ten avatars.
Vaishnavism flourished in predominantly
Shaivite South India during the seventh to
tenth centuries CE and is still commonplace,
especially in Tamil Nadu, as a result of the
twelve Alvars, saints who spread the sect to the
common people with their devotional hymns.
In later years, Vaishnava practices increased in
popularity due to the influence of sages like
Ramanujacharya, Madhvacharya,
Nimbarkacharya, Vallabhacharya, Vedanta
Desika, Manavala Mamunigal, Surdas,
Tulsidas, eknath, Tyagaraja, and many others.
But not everyone adopted and embraced
Vaishnavism

50.

Solution: d)

http://www.thehindu.com/todays-paper/tpopinion/a-battle-withoutwinners/article7449218.ece
The novel Madhorubagan, written by Perumal
Murugan published four years ago, deals with a
ritual that has been practiced in Tiruchengode
a century ago. As per the ritual, a women could
go with any man on the night of the festival. A
child born out of such a relation was treated as
gift of God. In the novel, a couple is childless
and the wife wants to take part in the ritual.
The husband is opposed to it but is forced to be
a silent witness when the wife choses to go her
Page 15


INSIGHTS MOCK TEST SERIES 2015: TEST 31 SOLUTIONS

way. The protest by some Hindu bodies is


against the "sexual permissiveness' and
blasphemy as shown in the book.

In the Bachan singh case it gave the rarest of


rate doctrine according to which death penalty
should be given only in the extreme cases and
where life sentence should not at all be
appropriate.
Last year, while handing down its Shatrughan
Chauhan vs Union of India ruling, the
Supreme Court spelt out clear guidelines on the
legal rights of prisoners on death row.

51. Solution: a)
The spacecraft also made several predictions
about the planet which should be read here. It
is quite contrary to what was understood till
date about Pluto.

53.

Solution: d)

http://www.thehindu.com/todays-paper/tpopinion/historic-flyby/article7449216.ece

Agreement for establishing NDB was signed


during the 6th BRICS Summit being held in
Fortaleza, Brazil in April, 2014. It was formally
launched at the 7th BRICS summit held in Ufa,
Russia in July 2015.

The Kuiper belt sometimes called the


EdgeworthKuiper belt, is a region of the Solar
System beyond the planets, extending from the
orbit of Neptune (at 30 AU) to approximately
50 AU from the Sun.

Purpose: To fund infrastructure projects in the


emerging economies. It is seen as an
alternative institute to west dominated World
Bank and the International Monetary Fund
(IMF).

It is similar to the asteroid belt, but it is far


larger20 times as wide and 20 to 200 times
as massive.

Capital: It will have initial capital of US 50


billion dollars and will be raised to US 100
billion dollars within the next couple of years.

52.

Solution: b)

The Supreme Court in Mithu vs State of Punjab


struck down Section 303 of the Indian Penal
Code, which provided for a mandatory death
sentence for offenders serving a life sentence.
http://www.thehindu.com/opinion/editorial/d
eath-sentences-trial-anderror/article7448579.ece
http://www.insightsonindia.com INSIGHTS

Each members role: They will have an equal


say in the banks management, regardless of
GDP size and contribute an equal share in
establishing a startup capital.
President: Eminent banker Kundapur Vaman
Kamath from India is President of Bank for the
first five years i.e. till 2020.

54.

Solution: d)
Page 16


INSIGHTS MOCK TEST SERIES 2015: TEST 31 SOLUTIONS

According to the OECD Government at a


Glance 2015 report, trust represents the
confidence of citizens and businesses in
government to do what is right and perceived
as fair.
Changes in trust levels could be affected by
many factors, including the economic outlook,
political changes such as elections or other
major events such as disasters or major
scandals including corruption cases. Moreover,
expectations of citizens could grow at a faster
pace than government responses.
This is the only survey that collects data on the
issue of trust in governments. It is a sampling
survey of 1,000 citizens in each country
conducted by World Poll, which has been doing
it since 2005.

55.

Solution: d)

IPC Section 499/500 was in news recently.


The Centre has told the Supreme Court that
defamation should remain a penal offence in
India as the defamer may be too poor to
compensate the victim. It has denied that
criminal defamation had any chilling effect on
free speech. The IPC under Section 499/500
criminalizes defamatory speech. This means
that a person can be imprisoned for a
maximum period of 2 years, if found guilty.
According to the Section 66A of the IT act, any
person who sends, by means of a computer
resource or a communication device,(a) any information that is grossly offensive or
has menacing character; or

http://www.insightsonindia.com INSIGHTS

(b) any information which he knows to be false,


but for the purpose of causing annoyance,
inconvenience, danger, obstruction, insult,
injury, criminal intimidation, enmity, hatred or
ill will, persistently by making use of such
computer resource or a communication device,

56.

Solution: d)

In its 50-year history, Japan has been the only


Asian country to hold it with India being
bestowed the honor to host the 12th such
International symposium now.
The Themes for the Symposium include
Antarctica and Supercontinent Evolution,
Antarctic Surface Processes, Landscapes, and
Links with Cryosphere and Climate, Antarctic
Solid Earth Structure and Interactions with the
Cryosphere: Antarctica, the Southern Ocean,
and Evolution of Climate and the Global
Cryosphere, amongst others.

57.Solution: d)
The deal puts strict limits on Irans
nuclear activities for at least a decade
and calls for stringent U.N. oversight,
with world powers hoping this will make
any dash to make an atomic bomb
virtually impossible.
In return, Iran will get sanctions relief
although the measures can snap back
into place if there are any violations.
The international arms embargo against
Iran will remain for five years but
deliveries would be possible with special
permission of the U.N. Security Council.
Iran has accepted allowing the U.N.
Page 17


INSIGHTS MOCK TEST SERIES 2015: TEST 31 SOLUTIONS

atomic watchdog tightly-controlled


managed access to military bases.
Iran will slash by around two-thirds the
number of centrifuges from around
19,000 to 6,104.
The deal caps uranium enrichment at
3.67% and limits the stockpile to 300 kg,
all for 15 years.
Iran will be required to ship spent fuel
out of the country forever, as well as
allow inspectors from the IAEA
inspectors certain access in perpetuity.
Heightened inspections, including
tracking uranium mining and
monitoring the production and storage
of centrifuges, will last for up to 20
years.

58.

The main aim of the DAC is to fast-track


procurement process of the armed forces by
optimally utilising the available budget.

59.

Solution: c)

Solution: b)

DAC was set up in October 2001 following


recommendations from Group of Ministers
(GoMs) on Reforming the National Security
System.
The need for DAC was felt post-Kargil conflict
and this high-level body is chaired by the
Defence Minister.
Other members include: Minister of State for
Defence, Chief of Army Staff, Chief of Naval
Staff, Chief of Air Staff, Defence Secretary,
Secretary Defence Research & Development,
Secretary Defence Production, Chief of
Integrated Staff Committees (HQ IDS),
Director General (Acquisition) and Deputy
Chief of Integrated Defence Staff.

http://www.insightsonindia.com INSIGHTS

60.
Solution: a)
It will comprise members of the Centre
and states to promote Indias overseas
shipments.

The council will be chaired by the Union


Commerce and Industry Minister and
secretaries of key ministries and state
ministers will be the members.

Page 18


INSIGHTS MOCK TEST SERIES 2015: TEST 31 SOLUTIONS

The main objective of the council will be


facilitating trade from states in a bid to
boost the countrys exports and
rationalising non-essential imports..

The move comes amid declining exports


which fell 5.45 per cent to $25.1 billion
in June, while imports fell 13.46 per cent
to $35.4 billion.

The essential issues that will be taken up


with the state officials include
infrastructure bottlenecks in terms of
road connectivity, power supply, law
and order and overall governance;
regulatory environment; and local
taxation-related matters particularly
where refunds are involved.

62.

61. Solution: a)

The proposed park will be established as


part of the Make in India, Make in
Kerala project
It will have modern common
infrastructure facilities aimed at
attracting component manufacturers in
the defence industry.

http://www.insightsonindia.com INSIGHTS

Solution: c)

The Supreme Court recently directed the


government to have a re-look at the drug
pricing policy to help make life-saving
medicines affordable for the common man.

http://indianexpress.com/article/business/bus
iness-others/with-aim-to-promote-exportsgovt-to-set-up-trading-facilitation-council/

In a major fillip to industrial infrastructure in


the Palakkad region, the Union governments
Department of Industrial Policy and Promotion
(DIPP) has approved a proposal from the
Kerala Industrial Infrastructure Development
Corporation (Kinfra) to set up the countrys
first defence industrial park at Ottappalam.

The Union government has agreed to


bring it under the Modified Industrial
Infrastructure Upgradation Scheme
(MIIUS).

The government had approved the


National Pharmaceutical Pricing Policy
(NPPP) in 2012.
This policy at bringing 348 essential
drugs under price control and also lead
to reduction in prices. With this, the
Govt would control prices of 348
essential drugs.
The policy debars the companies from
using the Wholesale Price Index (WPI)
to increase the prices of the essential
medicines on their own each year. Thus,
the companies had to seek approval
from the National Pharmaceutical
Pricing Authority whenever they wanted
to increase the prices of the items
covered under the Drug Price Control
Order.
It covers only 348 drugs covered under
National list of life saving medicines.

63.

Solution: d)

The GI tag ensures that none other than those


registered as authorised users (or at least those
residing inside the geographic territory) are
Page 19


INSIGHTS MOCK TEST SERIES 2015: TEST 31 SOLUTIONS

allowed to use the popular product name.


Darjeeling tea became the first GI tagged
product in India, in 2004-05, since then by
September 2010, 184 had been added to the
list. These are listed below.
https://en.wikipedia.org/wiki/List_of_Geogra
phical_Indications_in_India

Just scan through the list. Its a long one. Be


aware of popular names.

64.

Solution: d)

The Jansankhya Sthirata Kosh (JSK)


(National Population Stabilisation Fund) has
been registered as an autonomous Society
established under the Societies Registration
Act of 1860.

The Union Health Minister heads the


General Body of JSK and the Ministries
of Health and Family Welfare, Women
and Child Development, Department of
School Education & Literacy, Rural
Development, Planning Commission are
represented by their Secretaries on the
General Body of JSK.
All State Governments are members of
JSK.

65.
Solution: c)
Antarctica is recognized as a heritage of
mankind and as mandated by the
provisions of the Antarctic treaty, signed
by 29 consultative parties, no
commercial activity is permitted in
Antarctica and its environment and all
claims to its territory are frozen.
http://www.insightsonindia.com INSIGHTS

The scientific programs of this natural


polar laboratory are designed and
approved by the Scientific Committee on
Antarctic Research (SCAR) which has a
President and two to three vicepresidents and a small secretariat
located at Cambridge, England.
The science research programs are
proposed and co-ordinated by three
standing scientific groups (SSGs) on
Geosciences (GS), life sciences (LS) and
physical sciences (PS) each having three
chief officers.
India currently has two permanent
stations, Maitri at Schirmacher oasis
and Bharati in the Larsemann Hill area
located ~1000km east of Maitri. The
first Indian station, Dakshin Gangotri,
located on shelf ice is now buried and
lost.

66.

Solution: d)

This Bill amends the principal Act


passed in 2013.

The Bill enables the government to


exempt five categories of projects from
the requirements of: (i) social impact
assessment, (ii) restrictions on
acquisition of multi-cropped land, and
(iii) consent for private projects and
public private partnerships (PPPs)
projects.

The five categories of projects are: (i)


defence, (ii) rural infrastructure, (iii)
affordable housing, (iv) industrial
corridors, and (v) infrastructure
Page 20


INSIGHTS MOCK TEST SERIES 2015: TEST 31 SOLUTIONS

including PPPs where government owns


the land.

The Act would apply retrospectively, if


an award had been made five years
earlier and compensation had not been
paid or possession not taken. The Bill
exempts any period when a court has
given a stay on the acquisition while
computing the five year period.

The Act deemed the head of a


government department guilty for an
offence by the department. The Bill
removes this, and adds the requirement
of prior sanction to prosecute a
government employee.

67.

Solution: c)

The stone chariot located inside the campus of


Vittala temple is almost an iconic structure of
Hampi. The Stone Chariot at the Vijaya Vittala
temple complex in Hampi, Karnataka will now
adorn the new Rs. 10 note.
An image of Garuda (eagle god) was originally
enshrined within its sanctum. Garuda,
according to the Hindu mythology, is the
vehicle of lord Vishnu. Thus the Garuda shrine
facing the temples sanctum is symbolic.
In reality this stone shrine was built with many
giant granite blocks. The joints are smartly
hidden in the carvings and other decorative
features that adorn the Stone Chariot. The
chariot is built on a rectangular platform of a
feet or so high.

http://www.insightsonindia.com INSIGHTS

In front of the chariot two elephants are


positioned as if they are pulling the chariot. In
fact these elephants where brought from
elsewhere and positioned here at a later stage.
Originally two horses were carved in that
position. The tails and the rear legs of the
horses can be still seen just behind these
elephant sculptures.

68.

Solution: d)

It will be adorn a Rs. 50 note as per a


decision taken recently by the government.
Sun Temple is a 13th-century Sun Temple at
Konark in Odisha, India. It is believed that the
temple was built by king Narasimhadeva I of
Eastern Ganga Dynasty around 1250 CE. The
temple is in the shape of a gigantic chariot with
elaborately carved stone wheels, pillars and
Page 21


INSIGHTS MOCK TEST SERIES 2015: TEST 31 SOLUTIONS

walls. A major part of the structure is now in


ruins. The temple is a UNESCO World Heritage
Site. It is also featured on NDTV's list of Seven
Wonders of India and Times of India's list of
Seven Wonders of India.
The temple was originally built at the mouth of
the river Chandrabhaga, but the waterline has
receded since then. The temple has been built
in the form of a giant ornamented chariot of
the Sun god, Surya.

69.

Solution: b)

Goa churches were in news as they will adorn


the new Rs. 500 note. Amongst them most
prominent is this church.
The Basilica of Bom Jesus or Borea Jezuchi
Bajilika is located in Goa, India, and is a
UNESCO World Heritage Site. The basilica
holds the mortal remains of St. Francis Xavier.
The church is located in Old Goa, which was
the capital of Goa in the early days of
Portuguese rule.
'Bom Jesus' (literally, 'Good (or Holy) Jesus') is
the name used for the Ecce Homo in the
countries of Portuguese colonization. The
Jesuit church is Indias first minor basilica, and
is considered to be one of the best examples of
baroque architecture in India.
70.

Solution: a)

It has been choosen to adorn new 1000


currecny notes in India.
It is a beautiful painting of Ajanta caves. This
segment from Gardners Art through the Ages:
Non-Western Perspectives (2009) describes
the scene shown:
The bodhisattva Padmapani sits among a
crowd of devotees, both princesses and
commoners. With long, dark hair handing
down below a jeweled crown, he stands
holding his attribute, a blue lotus flower, in his
right hand. [] The artist has carefully
considered the placement of the painting in the
cave. The bodhisattva gazes downward at
worshipers passing through the entrance to
the shrine on their way to the rock-cut Buddha
image in a cell at the back of the cave.
Also go through this beautiful description of
Ajanta caves.

http://www.insightsonindia.com INSIGHTS

Page 22


INSIGHTS MOCK TEST SERIES 2015: TEST 31 SOLUTIONS

https://www.khanacademy.org/humanities/ar
t-asia/south-asia/buddhist-art2/a/the-cavesof-ajanta

and bursts, bringing rainfall as high as 75


millimeters per hour.

72.
71. Solution: c)
A cloudburst is an extreme amount of
precipitation, sometimes accompanied by hail
and thunder that normally lasts no longer than
a few minutes but is capable of creating flood
conditions.
A cloudburst can suddenly dump 72,300 tons
of water over one acre. However, cloudbursts
are infrequent as they occur only via
orographic lift or occasionally when a warm air
parcel mixes with cooler air, resulting in
sudden condensation.
Cloud burst is a situation when the intermolecular forces between the H2O molecules
get very high due to the rapid decrease in the
temperature or excess of electrostatic induction
in the clouds causing the lighting to remain
inside the cloud only, which causes hyperactive
energy inside the cloud. The water molecules
get denser and denser and get condensed but
do not leave the cloud due to excess of
electroforces.
As the water concentration get higher and
higher and so the weigh gets heavier the water
no longer is able to maintain force with the
clouds and so they fall and it precipitates.
In the Indian subcontinent, a cloudburst
usually occurs when a monsoon cloud drifts
northwards, from the Bay of Bengal or Arabian
Sea across the plains, then onto the Himalaya

http://www.insightsonindia.com INSIGHTS

Solution: d)

The move came after a petitioner said the nonnative eucalyptus tree was responsible for the
lowering groundwater levels in the region.

A eucalyptus tree consumes 90 litres of


water a day

During summers and times of


drought, its roots can go down up to 30ft

It was introduced in Karnataka in 1960s.


In the 1970s, eucalyptus plantations
were spread across 2.1 lakh hectares.

73.

Solution: a)

The GSLV-III or Geosynchronous


Satellite Launch Vehicle Mark III, is a
launch vehicle developed by the Indian
Space Research Organization.

GSLV Mk III is conceived and designed


to make ISRO fully self reliant in
launching heavier communication
satellites of INSAT-4 class, which weigh
4500 to 5000 kg.

It would also enhance the capability of


the country to be a competitive player in
the multimillion dollar commercial
launch market. The vehicle envisages
multi-mission launch capability for
GTO, LEO, Polar and intermediate
circular orbits.
Page 23


INSIGHTS MOCK TEST SERIES 2015: TEST 31 SOLUTIONS

74.

Solution: d)

Indian space industry experts recently opined


that there is a need to have a space law to
protect sovereign, public or commercial
interests in India.
This is to ensure that space assets and
applications are used for the right causes.
There is no single space law in India.
Currently, space activities are guided by a
handful of international space agreements, the
Constitution, national laws, the Satellite
Communications (SatCom) Policy of 2000 and
the revised Remote sensing policy or 2011.
India is among the five countries that do not
have a space law; while 15 others including the
US, Russia, Japan, China, Kazakhstan and
Ukraine, have laws based broadly on the Outer
Space Treaty of 1967.

75.Solution: c)
The government has drafted the National
Renewable Energy Bill, 2015 which aims to
consolidate the renewable energy sector and
give it an institutional structure.

After it is passed by Parliament it would


enable a National Renewable Energy
Policy, Renewable Energy Corporation

http://www.insightsonindia.com INSIGHTS

of India, an advisory group and a


committee on the same.
At present, the renewable energy sector
is governed by the Electricity Act, 2003,
which is also undergoing amendments.
The policy would enable a supportive
system for growth of the sector.
The various segments which are the
focus of the policy are: Renewable
energy resource assessment, technical
and safety standards, monitoring and
verification, manufacturing and skill
development and data management.
Through a separate law, the ministry of
new and renewable energy (MNRE)
would get freedom to execute projects
and not depend on other ministries and
departments for necessary clearances,
said officials.
The law also aims to set up dedicated
renewable electricity investment zones.
The law makes it clear who will finance,
who will plan and monitor and what
support will come from where.

76.

Solution: b)

The security legislation proposed by Japanese


Prime Minister Shinzo Abes administration
moved a step closer to becoming law when
Parliaments lower house approved it. The bills,
which seek to rewrite the countrys post-War
pacifist security policy, are now before the
upper house.
Over a period of seven decades, Japans
security policy, shaped under a war-renouncing
Constitution following the misadventures of

Page 24


INSIGHTS MOCK TEST SERIES 2015: TEST 31 SOLUTIONS

the imperial regime, has been focussed on selfdefence.


But the present bills seek to replace the selfdefence doctrine with collective self-defence,
that would allow Japan to send troops abroad
to rescue allies under attack. This big shift in
approach makes the legislation controversial
and unpopular.

forms parallel to the boundary, to the


mountain range, and to the trench. Powerful
earthquakes shake a wide area on both sides of
the boundary.
This usually happens with an oceanic plate
being subducted under a continental plate.
Mountains are more likely to be formed when
two continental plates converge.

For reasons refer this article:


http://www.thehindu.com/opinion/editorial/j
apans-security-legislation-approved-by-lowerhouse/article7444346.ece
79.
77. Solution: c)
Rocks that carry organic matter are broken in
sediments of different sizes.
A river then carries it from higher altitude in
form of sediment and then deposits it layer
wise from high-lying areas (coarse) to low-lying
areas (fine clay having organic matter).
These sediments get compressed and cemented
together under high temperature and pressure
to become sedimentary rocks. And fossil fuel at
an even greater temperature and pressure
applied over a large period of time.

Rivers are cheap means of transport, compared


to roads and railway. But most rivers in Africa
are not navigable due to water falls, weeds and
being seasonal.
Africa need to combine forces to utilise this
rivers for irrigation, hydro electric power and
combat the big problem of transport by
overcoming weeds and waterfalls.
Navigation can be done only in Zambeji and
Nile river.

80.
78.

Solution: d)

When two plates come together, it is known as


a convergent boundary. The impact of the two
colliding plates buckles the edge of one or both
plates up into a rugged mountain range, and
sometimes bends the other down into a deep
seafloor trench. A chain of volcanoes often
http://www.insightsonindia.com INSIGHTS

Solution: c)

Solution: d)

Heating the atmosphere is secondary. Visible


light from the Sun penetrates the atmosphere
and strikes objects on the Earth's surface.
These objects absorb that energy and re-emit
that energy as infra red energy that heads back
out into space.

Page 25


INSIGHTS MOCK TEST SERIES 2015: TEST 31 SOLUTIONS

But, the atmosphere contains gases like water


vapor and water droplet clouds that absorb that
infra red energy. So the energy is re-emitted
back and forth. This heats the atmosphere.
Basically the atmosphere is heated by the
Earth's surface which is heated by the Sun.

By the rising and falling action of tides, they


take back all the mud deposited by incoming
rivers to the sea. Otherwise the water
availability for the ships would have reduced.

83.
81.Solution: d)
Katabatic wind, also called downslope wind, or
gravity wind, wind that blows down a slope
because of gravity. It occurs at night, when the
highlands radiate heat and are cooled.

Solution: a)

Murmansk in the Arctic and the Baltic ports


near St. Petersburg are both important for
Russian access to the Atlantic. Despite St.
Petersburg being much further south, it is
surrounded by ice for a month or more each
year, while Murmansk is always ice free.

The air in contact with these highlands is thus


also cooled, and it becomes denser than the air
at the same elevation but away from the slope;
it therefore begins to flow downhill. This
process is most pronounced in calm air because
winds mix the air and prevent cold pockets
from forming.

Warm currents increase the temperature of the


sea water. By the action of land and sea breezes
the local area temperature is also increased.
Hence, the ports remain ice-free.

When a katabatic wind is warmed by


compression during its descent into denser air,
it is called a foehn. A large-scale katabatic wind
that descends too rapidly to warm up is called a
fall wind. In areas where fall winds occur,
homes and orchards are situated on hill slopes
above the lowlands where the cold air
accumulates.

Most large objects in orbit around the Sun lie


near the plane of Earth's orbit, known as the
ecliptic. The planets are very close to the
ecliptic, whereas comets and Kuiper belt
objects are frequently at significantly greater
angles to it.

82.

Solution: a)

Statement 4 is not true because indentation of


coasts is a long term natural phenomenon. It is
caused by erosion of coasts and a host of other
factors. It is not caused by tides.

84.

All the planets and most other objects orbit the


Sun in the same direction that the Sun is
rotating (counter-clockwise, as viewed from
above Earth's north pole). There are
exceptions, such as Halley's Comet.
Venus and Uranus rotate in the opposite
direction to what they revolve.

85.
http://www.insightsonindia.com INSIGHTS

Solution: a)

Solution: b)
Page 26


INSIGHTS MOCK TEST SERIES 2015: TEST 31 SOLUTIONS

If the Moon didn't spin at all, then eventually it


would show its far side to the Earth while
moving around our planet in orbit.
However, since the rotational period is exactly
the same as the orbital period, the same
portion of the Moon's sphere is always facing
the Earth.
It can be understood in detail here
http://www.moonconnection.com/moonsame-side.phtml

86.

Solution: d)

o
o
o
o
o

Angle of incidence
Duration of sunshine
Solar constant
Distance between the earth and
the sun
Transparency of the atmosphere.

The vertical rays of the sun heat the minimum


possible area, but on the contrary, the oblique
rays are spread over a relatively larger area, so
that the amount of area over which the
available solar energy has to be distributed in
increased and the energy per unit area on the
earth's surface is decreased.
In addition, the oblique rays have to traverse a
larger distance through the atmosphere before
they strike the surface of the earth. The longer
their path, the larger the amount of energy lost
by various processes of reflection, absorption,
and scattering, etc.

Prime Meridian and all other meridians are


semi-circles, not circles.
Time zone of a place is decided only by its
latitude. This is because the earth rotates
around the Sun from West to East.
The rotation causes the Sun pass overhead
from one place from the east to west. Only
latitude makes the Suns entry timing different
in the regions.

87.

Solution: a)

The actual amount of insolation received at a


place on the earth varies according to the
conditions of the atmosphere as well as the
seasons. The following astronomical and
geographical factors govern the amount of
insolation received at any point on the earth's
surface:
http://www.insightsonindia.com INSIGHTS

88.

Solution: d)

The decision is arrived after a joint meeting


between the Secretary of the Union Finance
Ministry, the Chief General Manager of the
Currency Management Department, Reserve
Bank of India, and the Director-General of
Archaeological Survey of India (ASI).
Recently they agreed to put new images on the
currency notes. For e.g. Stone chariot of Vittal,
Hampi is to be put on a Rs. 10 note.

89.

Solution: a)

Page 27


INSIGHTS MOCK TEST SERIES 2015: TEST 31 SOLUTIONS

A rise in the purchasing power (income) of the


consumer can sometimes induce the consumer
to reduce the consumption of a good.

GDP grows when the present market value of


this years produce is more than that of last
year.

The demand for such a good can be inversely or


positively related to its price depending on the
relative strengths of these two opposing effects.
If the substitution effect is stronger than the
income effect, the demand for the good and the
price of the good would still be inversely
related.

But growth in total market value can also be


because of high inflation and no rise in
production.

However, if the income effect is stronger than


the substitution effect, the demand for the good
would be positively related to its price. Such a
good is called a Giffen good.

At a constant base price (2011-12 year) in India,


increase in total value is calculated. So, real
GDP will only increase if there has been even
slight growth in production irrespective of
inflation or deflation in the economy.

90.

So, real GDP indicator is used to determine


whether there has been an actual growth in
production.

Solution: b)

Consumer welfare refers to the individual


benefits derived from the consumption of
goods and services. In theory, individual
welfare is defined by an individual's own
assessment of his/her satisfaction, given prices
and income. Exact measurement of consumer
welfare therefore requires information about
individual preferences.
The central idea in consumer welfare is to
enhance consumer surplus which is the
difference between what people prefer to pay
and what they actually pay. The greater the
difference, higher is the surplus. It means that
the market is allocating goods most efficiently
(at competitive prices) to people.

91. Solution: b)

http://www.insightsonindia.com INSIGHTS

92.

Solution: a)

To understand the concept properly refer to the


article
http://www.ehow.com/about_6605945_intere
st-rate-employment.html
In brief, low interest rates entice greater
investments in the economy for they promise
better returns on projects.
More investment leads to more employment
creation. However, this relationship need not
hold necessarily in the long run.

93.

Solution: b)

Page 28


INSIGHTS MOCK TEST SERIES 2015: TEST 31 SOLUTIONS

The Reserve Bank of India was established on


April 1, 1935 in accordance with the provisions
of the Reserve Bank of India Act, 1934.
The Central Office of the Reserve Bank was
initially established in Calcutta but was
permanently moved to Mumbai in 1937. The
Central Office is where the Governor sits and
where policies are formulated.
Though originally privately owned, since
nationalisation in 1949, the Reserve Bank is
fully owned by the Government of India.
The constitution makes no mention of the RBI.

94.

Solution: a)

It refers to the process by which the RBI takes


away money from the banking system to
neutralise the fresh money that enters the
system from foreign capital inflows.
It does so by selling or buying government
bonds from public.
If it does not do so, money supply in the
economy will shoot up in a very short period of
time playing havoc with the interest rates and
inflation in the economy.
Refer to the article to better understand
sterilization
http://www.thehindubusinessline.com/iw/200
3/02/02/stories/2003020201001400.htm

95.

Solution: d)

Liquidity trap is a situation when expansionary


monetary policy (increase in money supply)
does not increase the interest rate, income and
hence does not stimulate economic growth.
It is a situation in which the general public is
prepared to hold on to whatever amount of
money is supplied, at a given rate of interest.
They do so because of the fear of adverse events
like deflation, war.
There is a liquidity trap at short term zero
percent interest rate. When interest rate is
zero, public would not want to hold any bond,
since money, which also pays zero percent
interest, has the advantage of being usable in
transactions.
Hence, if the interest is zero, an increase in
quantity of money cannot not induce anyone to
buy bonds and thereby reduce the interest on
bonds below zero.

96.

Solution: c)

The objectives of the Public Distribution


System is to ensure food security and poverty
alleviation by making available the essential
commodities, specially food grains at an
affordable and uniform price at the door steps
of the consumers.
The department of Food and Civil supplies
arranges procurement of PDS items like rice,
wheat/atta, levy sugar, iodised salt
(procurement not being done now), Kerosene

http://www.insightsonindia.com INSIGHTS

Page 29


INSIGHTS MOCK TEST SERIES 2015: TEST 31 SOLUTIONS

oil, LPG under various Govt. of India scheme


under Public Distribution System.

97.

Solution: a)

Devaluation of currency means more of that


currency can be purchased for any given
foreign currency.
This means more of domestic goods can be
purchased from abroad following a fall in
currencys value.
In other words, since our currency becomes
more affordable to the foreigners, so do the
goods that are priced in them.

98.

employment and higher production of goods


and services.
A strong industry favours greater growth in the
services as well as primary sector.
But, if an economy becomes highly dependent
on the primary sector, it will not be able to
produce adequate amount of goods and
services for its population.
On a different note, financial risks have become
a part of the global economic environment.
Better policies and institutional coordination
amongst global and national monetary and
fiscal authorities is required to rationalize this
risk.

100.

Solution: c)

When economy is operating at a much lower


level then there is scope for immediate increase
in production of goods following an increase in
demand.
If economy is already operating at its peak,
then industries will find it difficult to produce
more goods even by adding extra labour since
the productivity of labour will be limited given
the limit of capital goods (machines).

Solution: a)

https://en.wikipedia.org/wiki/The_Energy_an
d_Resources_Institute

Therefore, only when the industry can produce


more (supply) as per the demand, a greater
money supply will not lead to inflation.

99.

Solution: a)

Economic development in the modern world is


fundamentally based on industrialization
which leads to generation of gainful
http://www.insightsonindia.com INSIGHTS

Page 30

INSIGHTS ON INDIA MOCK PRELIMINARY EXAM - 2015


INSIGHTS ON INDIA MOCK TEST - 32
GENERAL STUDIES
PAPER-I
Time Allowed: 2 Hours

Maximum Marks: 200

INSTRUCTIONS
1. IMMEDITELY AFTER THE COMMENCEMENT OF THE EXAMINATION, YOU SHOULD
CHECK THAT THIS TEST BOOKLET DOES NOT HAVE ANY UNPRINTED OR TORN OR
MISSING PAGES OR ITEMS, ETC. IF SO, GET IT REPLACED BY A COMPLETE TEST
BOOKLET.
2. You have to enter your Roll Number on the Test
Booklet in the Box provided alongside. DO NOT
Write anything else on the Test Booklet.
4. This Test Booklet contains 100 items (questions). Each item is printed only in English. Each
item comprises four responses (answers). You will select the response which you want to mark
on the Answer Sheet. In case you feel that there is more than one correct response, mark the
response which you consider the best. In any case, choose ONLY ONE response for each item.
5. You have to mark all your responses ONLY on the separate Answer Sheet provided. See
directions in the Answer Sheet.
6. All items carry equal marks.
7. Before you proceed to mark in the Answer Sheet the response to various items in the Test
Booklet, you have to fill in some particulars in the Answer Sheet as per instructions sent to you
with your Admission Certificate.
8. After you have completed filling in all your responses on the Answer Sheet and the
examination has concluded, you should hand over to the Invigilator only the Answer Sheet.
You are permitted to take away with you the Test Booklet.
9. Sheets for rough work are appended in the Test Booklet at the end.
10. Penalty for wrong answers :
THERE WILL BE PENALTY FOR WRONG ANSWERS MARKED BY A CANDIDATE IN THE
OBJECTIVE TYPE QUESTION PAPERS.
(i) There are four alternatives for the answer to every question. For each question for
which a wrong answer has been given by the candidate, one-third of the marks
assigned to that question will be deducted as penalty.
(ii) If a candidate gives more than one answer, it will be treated as a wrong answer even if
one of the given answers happens to be correct and there will be same penalty as
above to that question.
(iii) If a question is left blank, i.e., no answer is given by the candidate, there will be no
penalty for that question.
http://www.insightsonindia.com
INSIGHTS ON INDIA MOCK TEST SERIES FOR CIVIL SERVICES PRELIMINARY EXAM 2015
http://www.insightsonindia.com INSIGHTS

Page 1

1. The day of the Civil Services Preliminary


Examination, 23rd August, 2015 is the
first day of which month as per the Saka
calendar system?
a) Shravana
b) Bhadra
c) Vaishakha
d) Asvina

2. The recent lowering of the International


Gold prices can be attributed to
1. Rise in interest rates in US
2. Heavy restriction on gold imports in
several nations
3. Highly volatile global exchange rates
Choose the correct answer using the codes
below.
a)
b)
c)
d)

1 and 2 only
1 only
1 and 3 only
3 only

3. Which of these is/are the objectives of


the Minimum Retail Price (MRP) Act?
1. Prevent tax evasion
2. Protect revenue of firms from
fluctuating raw material prices
3. Protect consumers from profiteering
by retailers
Choose the correct answer using the codes
below.

b) 2 and 3 only
c) 1 and 3 only
d) All of the above

4. The Consumer Protection Bill, 2015


recently approved by the Cabinet
intends to
1. Establish consume courts at the
Taluk level
2. Resolve all consumer disputes by
arbitration and mediation
3. Ban all tele-marketing and multilevel marketing in India
Choose the correct answer using the codes
below.
a)
b)
c)
d)

1 and 2 only
2 and 3 only
1 only
None of the above

5. Mercy petition decided by the President


Article 72 of the Constitution of India
are disposed on the advice of the
a) Prime Minister
b) Ministry of Home affairs
c) Parliamentary Committee on Legal
affairs
d) Vice-President

6. A foreign investment in an Indian


company qualifies as Foreign Direct
Investment (FDI) if

a) 1 and 2 only
http://www.insightsonindia.com INSIGHTS

Page 2

a) the investing institution/investor


buys more than 10 per cent stake in
the company
b) the investment stays for a period of
more than two years with native
company irrespective of the level of
investment
c) the investing institution/investor
seek approval via the government
route
d) None of the above is correct.

7. Consider the following with reference to


environment based grouping Conference
of Parties (COP).
1. The parties in COP refers to nations
signatory to UNFCCC.
2. Kyoto Protocol was signed by COP.
3. COP has been successful in
establishing a binding emission
target for all nations.
Choose the correct answer using the codes
below.
a)
b)
c)
d)

1 and 2 only
2 and 3 only
1 and 3 only
All of the above

8. Which of the following government


schemes is/are targeted at Tribals?
1. Development of Particularly
Vulnerable Tribal Groups (PVTGs)
2. Investment in TRIFED
3. Tribal Protection Scheme
4. Higher Education Revival Scheme

a)
b)
c)
d)

1 and 2 only
2 and 4 only
1 and 3 only
3 and 4 only

9. Which of the statements regarding


cricket governance in India is
INCORRECT?
a) State Cricket associations in India
allow people without a background
in national/international cricket to
get membership.
b) Government does not own any State
or National cricket association in
India.
c) A political (peoples) representative
cannot be a member of a Private
State Cricket association.
d) None of the above

10. The International Tribunal for the Law


of the Sea (ITLOS) can settle disputes
relating to
1. Use of resources in International
waters
2. Accident involving two national
vessels in International waters
Which of the above is/are correct?
a)
b)
c)
d)

1 only
2 only
Both 1 and 2
None

Choose the correct answer using the codes


below.
http://www.insightsonindia.com INSIGHTS

Page 3

11. Consider the following about the


International Astronomical Union
(IAU).
1. It is an Intergovernmental
organization.
2. It is an organ of United Nations.
3. It approves all space missions that
extend beyond the orbit of Earth
around the Sun.
4. It is the internationally recognized
authority for assigning designations
to celestial bodies.
Choose the correct answer using the codes
below.
a)
b)
c)
d)

1 and 4 only
1 and 2 only
4 only
2, 3 and 4 only

12. World Student Day as declared by


United Nations is celebrated on the
birthday of
a) Dr. APJ Abdul Kalam
b) Pandit JN Nehru
c) Rabindranath Tagore
d) Dr. Rajendra Prasad

13. Coronary stents recently in news is


related to
a) Treatment of a heart related disease
b) Harmful strains that can be used to
make biological weapons
c) Organ transplant in disabled people
d) a mechanism to reverse the flow of
blood in the body causing death

http://www.insightsonindia.com INSIGHTS

14. Representatives from most of the


princely states joined the Constituent
assembly (CA)
a) immediately after the formation of
the CA
b) after Cabinet Mission Plan was
accepted by Indian National
Congress
c) after the acceptance of Mountbatten
Plan of 1947
d) after the constitution was adopted by
the CA

15. The Constitution of India establishes an


independent judiciary to serve which of
these purposes?
1. To protect the supremacy of the
Constitution
2. To settle any disputes between
Centre and states
3. For the judiciary to be able to
exercise the power of Judicial review
Choose the correct answer using the codes
below.
a)
b)
c)
d)

1 and 2 only
2 and 3 only
1 and 3 only
All of the above

16. In the Table of Precedence


1. Members of Parliament (MPs) are
placed higher than Chief Ministers of
States.
2. Governors (inside their respective
states) are placed higher than former
Presidents.

Page 4

3. Chief Justice of India is placed


higher than Union Cabinet
Ministers.
Choose the correct answer using the codes
below.
a)
b)
c)
d)

1 and 2 only
2 and 3 only
1 and 3 only
All of the above

17. The Foreign policy of India advocates


a) Stopping the arms race leading to
disarmament in both conventional
and nuclear weapons
b) Stopping the arms race in
conventional weapons but retaining
nuclear weapons
c) No particular stand on disarmament
and nuclear weapons stockpile
d) Development of the arms and
nuclear weapons industry only to an
extent of generating gainful
employment

18. Which of the following is/are correct


concerning the elections to Rajya
Sabha?
1. The nominee must be a domicile of
the state from which he wishes to be
elected.
2. Voting is by secret ballot system.
3. The elections are organized by the
Election Commission of India.
Choose the correct answer using the codes
below.

b) 2 and 3 only
c) 1 and 3 only
d) 3 only

19. A political party will be recognized as a


National Party if
a) it is registered with the Election
Commission of India (ECI)
b) it is registered with the ECI and is
recognized as a state party in at least
four states
c) if it has consecutively won at least 2
seats in the Parliament for at least
two general election
d) if the donation collected by the party
registered with ECI exceeds a certain
amount as decided by ECI

20. Clear constitutional recognition to the


existence of political parties in India for
the first time was given by
a) The establishment of Election
Commission of India
b) The Anti-defection provisions in the
Tenth schedule of the Constitution
c) The establishment of the Inter-State
Council of India
d) The 73rd and 74th amendment to the
Constitution of India

21. Which of the following statements is


INCORRECT concerning tribunals in
India?
a) Only the Central government is
authorized by the Constitution to
establish tribunals.

a) 1 and 2 only
http://www.insightsonindia.com INSIGHTS

Page 5

b) Tribunals can be established for


settling disputes other than
administrative disputes.
c) Appeal from the tribunals lie to both
the High Courts and Supreme Court.
d) Central Administrative Tribunal in
India exercises original jurisdiction
in relation to service matters of
Central government employees.

22. What is the objective(s) behind the


specification of regional languages in the
Eighth schedule?
1. These languages are to be used for
enriching Hindi Language
2. Members of these languages are
represented in the Official Languages
Commission.
3. Restricting the choice of state
governments official language
Choose the correct answer using the codes
below.
a)
b)
c)
d)

1 and 2 only
2 and 3 only
1 and 3 only
All of the above

23. The National Human Rights


Commission is authorized to
1. Enquire suo moto in cases of human
rights violation
2. Intervene in a proceeding involving
allegation of human rights pending
before a court of India
3. Take action against those guilty of
human rights violation

http://www.insightsonindia.com INSIGHTS

4. Tender constitutional interpretations


of human rights provisions in the
Constitution when requested by
President
Choose the correct answer using the codes
below.
a)
b)
c)
d)

1 and 2 only
2 and 4 only
1 and 3 only
2 and 3 only

24. Which of these statements regarding the


office of Attorney General of India is
INCORRECT?
a) He has the right to take part in the
proceedings of both houses of
Parliament.
b) Advocate General of India is
subordinate to Attorney General of
India.
c) Attorney General of India forms part
of the Union Executive.
d) His office does not come under the
Union Law ministry.

25. The Comptroller General of India is


responsible to
a) The Parliament
b) The Constitution of India
c) The President of India
d) Public Accounts Committee of Lok
Sabha

Page 6

26. Consider the following statements.

Assertion (A): The Chief Election


Commissioner and other election
commissioners enjoy security of tenure.
Reason (R): Their terms are specified and fixed
by the Constitution of India.
In the context of the statements above, which
of these is true?
a) A and R both are true, and R is the
correct explanation for A.
b) A and R both are true, and R is the
NOT the correct explanation for A.
c) A is correct, R is incorrect.
d) A is incorrect, R is correct

27. The Chief Minister of a Union Territory


is appointed by
a) The Administrator of the UT on the
recommendation of the Central
government
b) The President of India on the
recommendation of the
Administrator of the UT

municipality and the administrative


structure of which resembles that of
a Municipality
b) an area in transition from a rural
area to an urban area
c) a conglomeration of all panchayats
that comes under a common
Metropolitan Planning Committee
d) a panchayat that is temporarily being
governed by a Municipality or
Municipal corporation

29. State Bills relating to which of the


following categories can be reserved for
Presidential assent by the Governor?
1. Bills that impinge on the
fundamental rights of citizens
2. Bills that prima facie appear ultra
vires the constitution
3. Bills that contravene Central laws
Choose the correct answer using the codes
below.
a)
b)
c)
d)

1 and 2 only
2 and 3 only
1 and 3 only
All of the above

c) The President of India based on the


recommendation of the Cabinet
d) The Administrator of the UT based
on his personal discretion

28. As per the 74th Constitutional


Amendment Act, a Nagar Panchayat is
a term used for
a) a village panchayat that has been
recently given the status of a
http://www.insightsonindia.com INSIGHTS

30. Biofertilizers improves the productivity


of crops because
a) it increase the nutrient content of the
soil
b) it makes greater amount of nutrients
available to the plant from the same
soil
c) it inflates the root hairs increasing
their mineral absorption capacity
d) All of the above
Page 7

31. The Coalition Support Fund (CSF)


programme to Pakistan given by the US
is provided for
a) Developing tribal and backward
areas in Pakistan
b) Combating insurgency in
neighbouring Afghanistan
c) Improving defence bases of Pakistani
Army
d) Implementing several socioeconomic programs of USA running
in Pakistan
32. Coastal erosion can be caused by
1. Wave action
2. Tidal currents
3. High Winds

34. 35. Consider the following about Ramon


Magsaysay Award .
1. It is given for exemplary political and
economic work.
2. It is only awarded within Asia.
3. It is awarded by the Noble Prize
committee.
4. It can be awarded to organizations
too.
Choose the correct answer using the codes
below.
a)
b)
c)
d)

1 and 2 only
2 and 4 only
1 and 3 only
All of the above

Choose the correct answer using the codes


below.
a)
b)
c)
d)

1 and 2 only
2 and 3 only
1 and 3 only
All of the above

33. Maritime domain awareness recently in


news is related to
1. Safety and security of the maritime
domain
2. Environment of Oceans
3. Navigable waterways
Choose the correct answer using the codes
below.
a)
b)
c)
d)

35. The Competition Commission of India


(CCI) is
a) an executive body that regulates
mergers and acquisitions between
private bodies
b) a statutory body which monitors
monopolistic and unfair trade
practices in Indian markets
c) an executive body that settles
disputes between firms relating to
mergers and acquisitions
d) a statutory body that ensures that all
budding start-ups in India have
enough competence and resources to
compete with multi-national
companies

1 and 2 only
2 and 3 only
None of the above
All of the above

http://www.insightsonindia.com INSIGHTS

Page 8

36. The newest member of the World Trade


Organization (WTO) is
a) China
b) Russia
c) Kazakhstan
d) Ukraine

37. As per a study conducted by conducted


by Bangladesh-India Joint Tiger Census
Project, what are the reasons for the
declining population of Bengal tiger?
1. Lack of proper forest management
2. Animal-human conflict
3. Loss of habitat
4. Unchecked poaching
Choose the correct answer using the codes
below.
a)
b)
c)
d)

1, 3 and 4 only
2 and 4 only
1 and 3 only
All of the above

38. Tour de France is an annual multiple


stage bicycle race primarily held in
France. The race route passes through
which of these mountain chains?
a) High Atlas
b) Alps
c) Cordillera Central
d) Beaufort Range

39. What does a regional navigational


satellite mean?
a) A satellite which helps other
satellites and space station to
navigate in space
http://www.insightsonindia.com INSIGHTS

b) A satellite which helps provide


accurate earth coordinates for
navigation purposes
c) A satellite that guides rocket
launches till the upper space
d) A satellite that monitors the
navigational activities on earth

40. Union Government recently launched


Deendayal Upadhyaya Gram Jyoti
Yojana (DUGJY). The scheme aims to
provide
a) Uninterrupted electricity supply to
each rural household across the
country
b) Electricity to all Village panchayats
and Public buildings
c) Financial support to all visually
impaired people that need an eye
operation
d) Scholarships to meritorious visually
impaired children from every village
in India

41. Consider the following statements about


a species found in India.
1. It is a critically endangered bird
species as per IUCN.
2. It can be found in Gujarat,
Maharashtra and Rajasthan.
3. An artificial breeding programme for
the species was recently announced
by the government.
4. Greater population of the species is
found in grasslands.
The statements above are related to which of
these species?
Page 9

a)
b)
c)
d)

Spoon Billed Sandpiper


White-bellied heron
Great Indian Bustard
Forest owlet

42. Which of these can be the closest Indian


counterpart to the recently tested Long
March-5 belonging to Chinas space
programme?
a) Indian manned space mission
b) Mars Orbiter Mission
c) Geo-synchronous Satellite Launch
Vehicle
d) GAGAN and IRNSS

43. Consider the following about the All


India Council of Sports constituted
recently by the government.
1. President of India will be its exofficio chairman.
2. It will financially support all states to
establish a council of sports.
3. Its recommendations will not be
binding on the government.
4. It will adjudicate all complaints,
disputes and malpractices associated
with sports.
Choose the correct answer using the codes
below.
a)
b)
c)
d)

1, 3 and 4 only
3 only
1 and 2 only
All of the above

http://www.insightsonindia.com INSIGHTS

44. Surakshit Khadya Abhiyan launched


recently by the Union government aims
at
a) Creating awareness about food
hygiene and food safety
b) Safeguarding all food storage
warehouses and cold storages from
pests and damage
c) Minimizing post harvest losses of
food grains, fruits and vegetables
d) Organizing periodical makeshift food
testing camps in all villages of India

45. Deep Space Climate Observatory


(DSCOVR) is used for
a) purposes of earth observation
b) studying the cosmic rays, particles
and waves in deep space
c) studying cyclical movements of
distant celestial bodies
d) helping satellites launched from
earth to navigate in space and
communicate with earth

46. Estuaries and recesses in coastlines that


form bays can tackle heavy storms,
cyclones and tsunamis. These bays are
formed by
a) Turtle shells
b) Coral reefs
c) Sea grass
d) Mangroves

Page 10

47. Ocean acidification can affect marine life


by
1. Inhibiting shell growth in marine
animals
2. Affecting the growth of
phytoplanktons
3. Reducing the amount of coral reefs
formed
Choose the correct answer using the codes
below.
a)
b)
c)
d)

1 and 2 only
2 and 3 only
1 and 3 only
All of the above

48. Which of the following e-wastes can


NOT be recycled?
1. Lead
2. Copper
3. Gold
4. Silver
5. Platinum
Choose the correct answer using the codes
below.
a)
b)
c)
d)

1 and 4 only
1, 2 and 3 only
3 and 5 only
All of these can be recycled.

c) Bringing back all leading Indian


researchers working abroad
d) None of the above

50. Monetary policy decision is presently


made by
a) Solely by the RBI governor
b) A Monetary policy committee
constituted by the government in
RBI
c) A committee of RBI headed by RBI
governor holding veto power over
decisions
d) The RBI Governor and Deputy
Governors after taking concurrence
from government

51. The GST Bill seeks to establish a GST


Council. It will consist of
1. Union Finance Minister
2. Finance Secretary
3. Ministry of Planning
4. Minister of Finance from each state
government
Choose the correct answer using the codes
below.
a)
b)
c)
d)

1 and 4 only
1, 2 and 3 only
1 and 2 only
3 and 4 only

49. Atal Innovation Mission concerns with


a) Deploying additional funding to
institutes of higher education,
learning and research
b) Providing a platform to academics,
entrepreneurs and researchers to
foster culture of innovation
http://www.insightsonindia.com INSIGHTS

Page 11

52. In India the provision of double


citizenship
a) is provided to a select class of foreign
investors on strategic grounds
b) applies to all Indian diplomats
serving outside India
c) applies to minors of Indian
diplomats born outside India
d) does not exist as Indian laws and
constitution provides for only single
citizenship

c) 1 and 3 only
d) 2 and 4 only

55. Consider the following statements.


Assertion (A): Chalk used in blackboards is
highly soluble in water.
Reason (R): Minerals are generally highly
soluble in water.
In the context of the statements above, which
of these is true?

53. It is difficult to get Vitamin C in our


daily cooked meals because
a) Food containing Vitamin C cannot be
cooked
b) Vitamin C gets destroyed by heat
during cooking
c) If food containing Vitamin C is
heated, then its absorption is the
body is negligible
d) Heating Vitamin C with water makes
it toxic.

54. Consider the following about Jute fibre.


1. It is extracted from the leaves of the
Jute plant.
2. It is cultivated during the rainy
season.
3. It is grown mainly in the Southern
Indian region.
4. Jute is also made from the outer
covering of coconut.

a) A and R both are true, and R is the


correct explanation for A.
b) A and R both are true, and R is the
NOT the correct explanation for A.
c) A is correct, R is incorrect.
d) Both A and R are incorrect.

56. Lemonade is prepared by mixing lemon


juice and sugar in water. You wish to
add ice to cool it. In which case would it
be possible to dissolve more sugar in the
water more quickly?
a) Adding ice cubes before mixing sugar
and lemon
b) Adding ice cubes after adding sugar
and lemon
c) Solubility will remain same
irrespective of the order of adding ice
to the water
d) Dissolving half the sugar before
adding ice cubes and half later

Choose the correct answer using the codes


below.
a) 1 and 2 only
b) 3 and 4 only
http://www.insightsonindia.com INSIGHTS

Page 12

57. The synthesis of food in plants generally


occurs
a) in between leaves and stems
b) in roots and from there it is carried
to leaves which absorb it
c) in the upper part of the brown stems
away from the root
d) in the leaves of the plant

58. Fungi depend on which mode of


nutrition?
1. Autotrophic
2. Heterotrophic
3. Saprotrophic
4. Parasitic
Choose the correct answer using the codes
below.
a)
b)
c)
d)

1 and 4 only
2 and 3 only
1 and 3 only
3 only

59. Sorters disease, an occupational hazard,


is related to the
a) Wool industry
b) Silk Industry
c) Cotton Industry
d) Plastic and Polymer Industry

60. Stainless steel vessels used for cooking


are generally provided with copper at
the bottom. What can be a possible
reason behind it?
a) It is easier to clean copper than the
stainless steel.
http://www.insightsonindia.com INSIGHTS

b) Copper is cheaper than stainless steel


thus reducing the price of the vessel.
c) Copper can conduct heat better than
the stainless steel.
d) It increases the toughness and
durability of the stainless steel vessel
this way.

61. Special type of substances, indicator, is


used to test whether a substance is
acidic or basic. Which of these is/are
natural indicators?
1. Turmeric
2. Lichens
3. Tulsi leaves
4. Skin of Papaya
Choose the correct answer using the codes
below.
a)
b)
c)
d)

1 and 4 only
2 and 3 only
3 and 4 only
1 and 2 only

62. Acid rain can be harmful for


1. Buildings
2. Monuments
3. Plants
4. Animals
Choose the correct answer using the codes
below.
a)
b)
c)
d)

1 and 2 only
2 and 3 only
3 and 4 only
All of the above

Page 13

63. Organic matter can be added to the soil


to
1. Improve Nutrient base of soil
2. Reduce the acidity of soil
3. Increase the activity of
microorganisms in the soil

Choose the correct answer using the codes


below.
a)
b)
c)
d)

1, 2 and 3 only
2, 3 and 4 only
1 and 4 only
All of the above

Choose the correct answer using the codes


below.
a)
b)
c)
d)

1 and 2 only
2 and 3 only
1 and 3 only
All of the above

64. Migratory birds flying for long distances


generally fly high in the sky. What can
be the possible reason(s)?
1. They can escape thunder and rain.
2. Wind flow is helpful at such heights.
3. Conditions are cold which allow
them to disperse heat generated by
flight muscles.
Choose the correct answer using the codes
below.
a)
b)
c)
d)

1 and 2 only
2 and 3 only
1 and 3 only
All of the above

65. While going through a tropical


rainforest, you are likely to find which of
these species?
1. Red-eyed frog
2. New World Monkey
3. Toucan
4. Lion Tailed Macaque
http://www.insightsonindia.com INSIGHTS

66. If you are stuck in a storm that is


accompanied by lightning, what should
you NOT do?
1. Take shelter under an isolated tree
2. Sit near a widow
3. Take cover under a bus
4. Go in an open garage
Choose the correct answer using the codes
below.
a)
b)
c)
d)

1 and 2 only
2, 3 and 4 only
1, 2 and 4 only
1, 3 and 4 only

67. Consider the following statements.


1. The Eastern coast of India has
greater frequency of cyclones than
the Western coast.
2. The Western coast of India has
greater intensity of cyclones than the
Eastern coast.
Which of the above is/are true?
a)
b)
c)
d)

1 only
2 only
Both 1 and 2
None

Page 14

68. Consider the following statements.

Assertion (A): Loamy soil is not suitable for


growing crops.
Reason (R): It cannot hold water which is
essential for plant growth.
In the context of the statements above, which
of these is true?
a) A and R both are true, and R is the
correct explanation for A.
b) A and R both are true, and R is the
NOT the correct explanation for A.
c) A is correct, R is incorrect.
d) Both A and R is incorrect.

b) Country X has necessarily an


increasing population.
c) Country X is necessarily facing a
decline in population.
d) Country X has both high death rates
and birth rates.

71. Cooperative sector industries can be


1. Owned by workers
2. Owned by raw material suppliers
3. Owned by producers of raw materials
4. Operated by raw material suppliers
5. Operated by producers of raw
material
Choose the correct answer using the codes
below.

69. Why do we get muscle cramps after


heavy exercise?
a) Bones and muscles are at different
temperatures.
b) The gel like liquid in muscle cell
becomes hard due to high body
temperature.
c) Muscle cells respire anaerobically to
produce lactic acid due to unfulfilled
heavy demand for oxygen.
d) High blood circulation expands
muscle cells due to heavy demand for
oxygen which stretches the muscles
too much.

70. A country X has an age pyramid which is


broad at its base and keeps narrowing
till it reaches the top. What does it imply
about the population of the country?
a) Country X has a highly unstable
population growth rate.
http://www.insightsonindia.com INSIGHTS

a)
b)
c)
d)

1, 4 and 5 only
2, 3 and 4 only
1, 2 and 3 only
All of the above

72. Which of these factors have helped the


cotton industries to spread
geographically?
1. Humidity, one of the key
requirements of the industry, can be
created artificially
2. Raw cotton is a weight losing raw
material
3. Wide availability of labour
Choose the correct answer using the codes
below.
a)
b)
c)
d)

1 and 2 only
2 and 3 only
1 and 3 only
All of the above
Page 15

73. The conditions for growing rice and


wheat are not same because
1. Rice requires bright sunshine
whereas wheat does not.
2. Wheat requires well drained soil
whereas rice requires soil which can
retain water.
3. Rice requires heavy rainfall whereas
wheat requires moderate rainfall.
Choose the correct answer using the codes
below.
a)
b)
c)
d)

1 and 2 only
2 and 3 only
1 and 3 only
All of the above

74. In India, natural gas can be extracted


from which of these regions?
1. Jaisalmer
2. Tripura
3. Krishna Godavari delta
4. Andhra Pradesh
Choose the correct answer using the codes
below.
a)
b)
c)
d)

1, 2 and 4 only
2, 3 and 4 only
1 and 3 only
All of the above

75. What is the difference between contour


ploughing and terrace farming?
1. Contour ploughing alters the shape
of the slope.

http://www.insightsonindia.com INSIGHTS

2. Terrace farming retains the shape of


the slope.
3. In terrace farming hills are ploughed
up and down unlike contour
ploughing.
Choose the correct answer using the codes
below.
a)
b)
c)
d)

1 and 2 only
2 and 3 only
1 and 3 only
None of the above

76. If International trade is allowed freely


then eventually it will lead to which of
the following?
a) A handful of nations will accumulate
all the wealth of the global economy.
b) Production and distribution of all
commodities will become
concentrated at one geographical
location.
c) Some nations will become
specialized in production of certain
goods and services.
d) All global labour will concentrate at
the nation which has the highest
exports.

77. The difference between Broad money


and Narrow money is based on
a) RBIs authority to issue them freely
in security markets
b) Liquidity of money supply
c) Their use by either government or
household sector
d) The ease with which bonds can be
issued on each of them
Page 16

78. If the currency of India appreciates in


the foreign exchange market, it will
1. Increase Indias foreign exchange
reserves
2. Make imports costly in India
3. Increase the fiscal deficit of India
Choose the correct answer using the codes
below.
a)
b)
c)
d)

1 and 2 only
2 and 3 only
1 and 3 only
None of the above

79. Income tax in India is


a) Levied, collected and appropriated
by Central government
b) Levied, collected by Central
government; and shared with state
governments
c) Levied by Central government, but
collected and appropriated by state
governments
d) Levied, collected and appropriated
by state governments

80.Capital receipts of the Government of


India includes
1. Grants by foreign government
2. Earnings from interest rates on loans
previously forwarded
3. Borrowing from market
Choose the correct answer using the codes
below.
a) 1 and 2 only
http://www.insightsonindia.com INSIGHTS

b) 2 and 3 only
c) 3 only
d) 1 and 3 only

81. The atmosphere is held around the earth


by
a) Primordial heat from inside the earth
b) Magnetic force around the earth that
attracts all charged dust particles
c) Coriolis force and adhesive frictional
force between the earth and
atmosphere
d) Gravitational force of the Earth

82. Arrange these layers of earth in


increasing order of their thickness.
1. Crust
2. Mantle
3. Core
Choose the correct answer using the codes
below.
a)
b)
c)
d)

132
231
123
321

83. Which of the following are responsible


for changing landform on the earth?
1. Volcanoes
2. Sea-waves
3. Glaciers
4. Rivers
5. Wind
Choose the correct answer using the codes
below.
Page 17

a)
b)
c)
d)

1 and 5 only
1, 3, 4 and 5 only
2, 3 and 4 only
All of the above

84. Loess found in desert areas is


a) an accumulation of fine and light
sand spread over large distances
b) a collection of mushroom rocks
narrow at bottom and wide at top
c) a high sand dune that is formed as a
result of heavy desert storms
d) a land near oasis that is rich in
humus and minerals

85. Veblen goods in economic terms are


those goods whose
a) price is directly proportional to their
supply in the market
b) price is directly proportional to their
demand in the market
c) price does not relate to their demand
in the market
d) price and demand are inversely
related

86. The atmospheric layer that helps in


radio transmission is
a) Troposphere
b) Thermosphere
c) Stratosphere
d) Mesosphere

87. Marginal efficiency of capital means


1. Expected rate of return from
additional Investment
http://www.insightsonindia.com INSIGHTS

2. Efficiency of production process as


each factor is increased
Which of the above is/are true?
a)
b)
c)
d)

1 only
2 only
Both 1 and 2
None

88. Consider the following about Nyaya


school of Vedic philosophy.
1. It holds that human suffering results
from mistakes produced by activity
under wrong knowledge.
2. Liberation can be gained through
right knowledge.
3. The school has contributed to the
development of theory of logic.
Choose the correct answer using the codes
below.
a)
b)
c)
d)

1 and 2 only
1 and 3 only
2 and 3 only
All of the above

89. Buddhists teach that suffering can be


ended by direct understanding and
perception of dependent origination.
This points out to
a) The vagueness of the idea of a cause
and effect
b) Interdependence of events in
existence
c) Independence of the existence of the
Self
d) God being the origin of all beings in
existence.
Page 18

90. Subash Chandra Bose resigned from the


congress presidentship and organised
forward Bloc for a more vigorous
national struggle in 1939. This was
because
a) He came to believe that Gandhi's
Non-violent Movement might not
bring independence
b) He did not trust that the Indian
National Congress is fighting for
Indian freedom
c) He was pressurized by the British to
resign from the Congress
d) He wanted to support the activities of
the Congress from outside

c) It was annulled after the First World


War ended.
d) Program of Swadeshi was adopted to
protest against the division.

93. Consider the following with reference to


the Indian freedom struggle during the
First World War.
1. Mainstream political leadership lent
support to the British in the war.
2. No anti-colonial activities were
witnessed during the entire period in
India.
3. The Surat Pact was signed during
this period.
Choose the correct answer using the codes
below.

91. Consider the following about the 1857


mutiny.
1. It was during the reign of Governor
General Canning.
2. Mangal Pandey led the Mutiny.
3. It started from Delhi.
4. The mutiny did not spread to South
India.
Choose the correct answer using the codes
below.
a)
b)
c)
d)

1 and 2 only
1 and 3 only
2 and 3 only
1 and 4 only

92. Which of the following about Partition


of Bengal is INCORRECT?
a) It was ordered by Curzon.
b) All India Muslim League supported
the division of Bengal.
http://www.insightsonindia.com INSIGHTS

a)
b)
c)
d)

1 and 2 only
1 and 3 only
2 and 3 only
1 only

94. In the Government of India Act, 1919


1. Principle of diarchy was introduced
2. Local bodies and provincial
legislatures were given autonomy.
3. Franchise base of Central as well as
provincial legislatures was increased.
Choose the correct answer using the codes
below.
a)
b)
c)
d)

1 and 2 only
1 and 3 only
2 and 3 only
All of the above

Page 19

95. Kuka movement in Punjab, 1872, aimed


at
a) Purifying the Sikh religion by
removing all the abuses,
superstitions and ill-practices
b) Making Sikh land independent from
British control
c) Secure lower land rent and lower
taxes from British officials
d) Achieve justice for the martyrs of the
1857 mutiny

96. Consider the following about C.


Rajagopalachari.
1. He was the last Governor-general of
India.
2. He founded the Swatantra party.
3. He pioneered several temple entry
movements in India.
4. He was described by Gandhi as the
keeper of my conscience.
Choose the correct answer using the codes
below.
a)
b)
c)
d)

1 and 2 only
1 and 3 only
2 and 4 only
All of the above

97. The Karachi session, 1931, of the


Congress was important because
a) The goal of complete independence,
Swaraj, was declared here.
b) Congress spelt the true meaning of
Swaraj in this session.
c) Congress decided to participate in
the Second Round Table Conference.

http://www.insightsonindia.com INSIGHTS

d) Congress launched the second phase


of Civil Disobedience movement in
the session.

98. Which of these statements concerning


Jugantar party formed in 1906 is
correct?
a) The party involved itself in grass root
work of education and uplifting rural
dalits.
b) It was based on a socialistic ideology
and believed in negotiation with the
government.
c) Aurobindo Ghosh was one of the
founder members.
d) All of the above

99. The most significant event between the


7th and 11th century was the Tripartite
struggle centered on Kannauj that lasted
for more than two centuries between the
1. Pala Empire
2. Rashtrakuta Empire
3. Chalukya Empire
4. Chola dynasty
Choose the correct answer using the codes
below.
a)
b)
c)
d)

1 and 2 only
1 and 3 only
2 and 4 only
All of the above

Page 20

100.
Which of the following can be the
possible reason(s) behind the
disintegration of the Gupta Empire?
1. Death of Emperor Harsha
2. Invasion of the Huns
3. Complete drain of economic
resources towards the end of
Harshas tenure
Choose the correct answer using the codes
below.
a)
b)
c)
d)

1 and 2 only
1 and 3 only
2 and 3 only
All of the above

http://www.insightsonindia.com INSIGHTS

Page 21


INSIGHTS MOCK TEST SERIES 2015: TEST 32 SOLUTIONS

1. Solution: b)

The current national calendar of India was set


up in 1957 by the Calendar Reform Committee
that formalized a lunisolar calendar in which
leap years coincide with those of the Gregorian
calendar, and the months are named after the
traditional Indian months. This reformed
Indian calendar began with Saka Era, Chaitra 1,
1879, which corresponds to March 22, 1957.
Names of the 12 months of the Indian Civil
Calendar and Correlation with Gregorian
Calendar:
1. Chaitra (30 / 31* Days) Begins March
22 / 21*
2. Vaisakha (31 Days) Begins April 21
3. Jyaistha (31 Days) Begins May 22
4. Asadha (31 Days) Begins June 22
5. Shravana (31 Days) Begins July 23
6. Bhadra (31 Days) Begins August 23
7. Asvina (30 Days) Begins September 23
8. Kartika (30 Days) Begins October 23
9. Agrahayana (30 Days) Begins
November 22
10. Pausa (30 Days) Begins December 22
11. Magha (30 Days) Begins January 21
12. Phalguna (30 Days) Begins February
20

http://www.insightsonindia.com INSIGHTS

2. Solution: b)
Gold fell back towards a five-year low in global
markets on Wednesday as investors continued
to exit from the precious metal.
A looming rise in U.S. interest rates, the first in
nearly a decade, has dented golds investment
appeal, spurring more sellers in the
international market after Mondays three per
cent rout. Spot gold was down 0.5 per cent at
$1,095.69 an ounce intra-day, while U.S. gold
futures for August delivery were down $8.70 an
ounce at $1,094.80.

3. Solution: c)
http://www.thehindu.com/todays-paper/tpopinion/time-to-abolish-themrp/article7453670.ece
The maximum retail price (MRP) that is
printed on all packaged commodities that
consumers purchase was introduced in 1990 by
the Ministry of Civil Supplies, Department of
Legal Metrology, by making an amendment to
the Standards of Weights and Measures Act
(Packaged Commodities Rules) (1976). It was
meant to prevent tax evasion and protect
consumers from profiteering by retailers.
Before the amendment, manufacturers could
print either the maximum retail price (inclusive
of all taxes) or the retail price (local taxes
extra). When producers opted for the latter
method, it was found that retailers often
charged more than the locally applicable taxes.
Thus, the amendment was made to introduce
the compulsory printing of MRP on all
packaged commodities.
Page 1


INSIGHTS MOCK TEST SERIES 2015: TEST 32 SOLUTIONS

5. Solution: b)

4. Solution: d)
The new bill seeks to replace Consumer
Protection Act, 1986 in order to deal with
consumer protection and safety. It aims at
simplifying the consumer dispute resolution
process along with enhancing the pecuniary
jurisdiction of the consumer grievance
redressal agencies.
Key features
Central Consumer Protection Authority
(CCPA): establishment of CCPA as an apex
regulatory authority with more powers to
protect and enforce the rights of consumers. It
will have powers to recall products and initiate
action suit against defaulting companies
including e-tailers for refunds and return of
products.

Product liability: If product or


services causes personal injury, death or
damage to property, CCPA will have
powers to take action against defaulting
manufacturers or service providers.
Speedy disposal of court cases: In
this regard, provision related to
mediation has been proposed which
will act as an alternative dispute
resolution mechanism. However,
mediation will be under the aegis of
consumer courts.

http://www.insightsonindia.com INSIGHTS

Under the existing rules of procedure


governing mercy petitions, the view of the
Union Ministry of Home Affairs (MHA),
conveyed to the President in writing, is taken
as the view of the Cabinet, and the President
decides a mercy petition accordingly.
http://indianexpress.com/article/explained/ho
w-the-president-decides-matters-of-life-anddeath/#sthash.oNqmc2CR.dpuf
Once a convict has been finally awarded the
death sentence by the Supreme Court,
anybody, including a foreign national, can send
a mercy petition with regard to that person to
the Presidents Office or the MHA. A mercy
plea can also be sent to the Governor of the
state concerned, who then forwards it to the
MHA for further action.

6. Solution: a)
However, all investments lately have all been
merged in one single category. So, until a sector
receives funds upto its composite cap, its fine
with the government.
Read about the change here:
http://indianexpress.com/article/explained/ta
king-a-composite-view-of-foreign-investmentin-india/

Page 2


INSIGHTS MOCK TEST SERIES 2015: TEST 32 SOLUTIONS

7. Solution: a)

http://indianexpress.com/article/explained/m
eaning-cop-21/
The term COP stands for Conference of
Parties. Parties is a reference to the (now) 196
signatories of the United Nations Framework
Convention on Climate Change, or UNFCCC, as
it is called.
The Framework Convention came into force in
1994, two years after its text was finalised at
the Earth Summit in Rio de Janeiro in Brazil.
Every year since 1994, the parties to the
UNFCCC have met at different venues at the
end of the year to discuss a global agreement to
cut emissions of greenhouse gases in the
atmosphere, the main reason why average
global temperatures have been rising.

development, animal husbandry, installation of


non-conventional sources of energy,
construction of link roads or other innovative
activity.

9. Solution: c)
http://indianexpress.com/article/explained/ex
plained-indian-crickets-political-pitch/
In June last year, BJP president Amit Shah
took over as president of the Gujarat Cricket
Association (GCA) from Narendra Modi who
had just been elected the countrys Prime
Minister. Modi had until then controlled GCA
he took over in 2009 while being chief
minister of Gujarat.
Soon after Shah became GCA president, he
was also named BJP president.
Both of them did not have any background in
cricket.

8. Solution: a)
http://www.tribal.nic.in/Content/schemes.asp
x
Union Ministry of Tribal Affairs has revised
Development of Particularly Vulnerable Tribal
Groups (PVTGs)
Under the Scheme, priority has been accorded
for the protection of PVTGs and their
improvement in terms of the social indicators
like education, health, nutrition and livelihood
in order to decrease their vulnerability.
Financial assistance under this scheme will be
utilized for activities like housing, agricultural
development, land distribution and land
http://www.insightsonindia.com INSIGHTS

10.

Solution: c)

The Italian government has approached the


International Tribunal for the Law of the Sea
(ITLOS), one of the four forums available for
international disputes, in the Italian Marines
case.
The International Tribunal for the Law of the
Sea (ITLOS) is an intergovernmental
organization created by the mandate of the
Third United Nations Conference on the Law of
the Sea. It was established by the United
Nations Convention on the Law of the Sea,
signed at Montego Bay, Jamaica, on December
10, 1982.
Page 3


INSIGHTS MOCK TEST SERIES 2015: TEST 32 SOLUTIONS

The Convention entered into force on


November 16, 1994, and established an
international framework for law over "all ocean
space, its uses and resources". The tribunal is
based in Hamburg, Germany. The Convention
also established the International Seabed
Authority, with responsibility for the regulation
of seabed mining beyond the limits of national
jurisdiction, that is beyond the limits of the
territorial sea, the contiguous zone and the
continental shelf.

He was born on 15 October 1931 in


Rameswaram, Ramanathapuram
District of Tamil Nadu.

In 1960, he had joined Defence Research


and Development Organisation (DRDO)
as a scientist and started his career by
designing a small helicopter for the
Indian Army.

In 1969, he had joined India Space


Research Organisation (ISRO) as the
project director for Indias first
indigenous satellite launch vehicle
programme.

Under his leadership and vision, India


entered into the elite space club after
Rohini satellite was place into the
orbit in 1980.

He was a pioneer and instrumental in


development of the Polar Satellite
Launch Vehicle (PSLV), Indias
space workhorse.

In 1980, he was appointed Chairman of


Integrated Guided Missile Development
Program (IGMDP) and was brain
behind developing Agni and Prithvi
missiles. Thus he came to be known as
the Missile Man of India for his work on
the development of ballistic missile

He was appointed CSA to Prime


Minister and Secretary of DRDO in 1992
and remained till 1998.

He was Chief Project Coordinator of


Pokhran II Nuclear Test
i.e. Operation Shakti which was
conducted on 11 May 1998.

11. Solution: c)
It is a collection of professional astronomers, at
the PhD level and beyond, active in
professional research and education in
astronomy. It acts as the internationally
recognized authority for assigning designations
to celestial bodies (stars, planets, asteroids,
etc.) and any surface features on them.
The IAU is a member of the International
Council for Science (ICSU). Its main objective
is to promote and safeguard the science of
astronomy in all its aspects through
international cooperation. The IAU maintains
friendly relations with organizations that
include amateur astronomers in their
membership.

12. Solution: a)
After the expiry of Dr. Kalam, several articled
came in newspapers. Here is all that you need
to know about Dr. Kalam.

http://www.insightsonindia.com INSIGHTS

Page 4


INSIGHTS MOCK TEST SERIES 2015: TEST 32 SOLUTIONS

He was the 11th President of India and


was in office from 2002 to 2007. He was
Indias most popular President and
famously known as Peoples President.

Awards and Honours: Bharat Ratna


(1997), Padma Vibhushan (1990),
Padma Bhushan (1981).

He has unique honour of receiving


honorary doctorates from 30
universities and institutions. United
Nations had recognised his 79th
birthday as World Student Day.

Switzerland has declared 26 May as


Science Day to commemorate visit of A
P J Abdul Kalam to the country in 2005

Coronary stent in India became popular as


Kalam-Raju stent and helped to bring down
cost of the device in the market.
Dr. Soma Raju said Mr. Kalam used to visit the
Nizams Institute of Medical Sciences to call on
colleagues admitted there in 1980s.
During one of the visits, Dr. Soma Raju had a
conversation with him about the high costs of
imported devices when the former President
suggested that indigenous devices should be
made with technology available in the country.

14. Solution: c)
http://www.thehindu.com/todayspaper/abdul-kalam-peoples-president-passesaway/article7471640.ece
http://www.livemint.com/Politics/nodNTgMC
TsS5dfnqajZjpK/Abdul-Kalam-admitted-tohospital.html

13. Solution: a)
It was in news because of Late Dr. Kalam.
A coronary stent is a tube-shaped device placed
in the coronary arteries that supply blood to
the heart, to keep the arteries open in the
treatment of coronary heart disease. It is used
in a procedure called percutaneous coronary
intervention (PCI).

http://www.insightsonindia.com INSIGHTS

The elections to the Constituent Assembly (for


296 seats allotted to the British Indian
Provinces) were held in JulyAugust 1946. The
Indian National Congress won 208 seats, the
Muslim League 73 seats, and the small groups
and independents got the remaining 15 seats.
However, the 93 seats allotted to the princely
states were not filled as they decided to stay
away from the Constituent Assembly.
The representatives of the princely states, who
had stayed away from the Constituent
Assembly, gradually joined it. On April 28,
1947, representatives of the six states5 were
part of the Assembly. After the acceptance of
the Mountbatten Plan of June 3, 1947 for a
partition of the country, the representatives of
most of the other princely states took their
seats in the Assembly. The members of the
Muslim League from the Indian Dominion also
entered the Assembly.
Page 5


INSIGHTS MOCK TEST SERIES 2015: TEST 32 SOLUTIONS

15. Solution: d)
An independent judiciary is needed: one, to
protect the supremacy of the Constitution by
exercising the power of judicial review; and
two, to settle the disputes between the Centre
and the states or between the states.
The Constitution contains various measures
like security of tenure to judges, fixed service
conditions and so on to make the judiciary
independent of the government.
Also, since the judiciary is also administratively
controlled by the higher judiciary,
independence is strengthened.

16. Solution: b)
The order is as follows:
1. President
2. Vice-President
3. Prime Minister
4. Governors of states within their
respective states
5. Former presidents
5A. Deputy Prime Minister
6. Chief Justice of India
Speaker of Lok Sabha
7. Cabinet Ministers of the Union
Chief Ministers of States within their
respective States
http://www.insightsonindia.com INSIGHTS

Deputy Chairman, Planning


Commission
Former Prime Ministers
Leaders of Opposition in Rajya Sabha
and Lok Sabha
MPs are ranked at number 21 after all the
ministers in the Union and states.

17. Solution: a)
The foreign policy of India is opposed to arms
race and advocates disarmament, both
conventional and nuclear. This is aimed at
promoting world peace and security by
reducing or ending tensions between power
blocs and to accelerate economic development
of the country by preventing the unproductive
expenditure on the manufacture of arms.
India has been using the UNO platform to
check the arms race and to achieve
disarmament. India took the initiative of
holding a six-nation summit at New Delhi in
1985 and made concrete proposals for nuclear
disarmament.
By not signing the Nuclear Non-proliferation
Treaty (NPT) of 1968 and the Comprehensive
Test Ban Treaty (CTBT) of 1996, India has kept
its nuclear options open. India opposes NPT
and CTBT due to their discriminatory and
hegemonic nature. They perpetuate an
international system in which only five nations
(USA, Russia, China, UK and France) can
legitimately possess nuclear weapons.

18.Solution: d)
Page 6


INSIGHTS MOCK TEST SERIES 2015: TEST 32 SOLUTIONS

In 2003, the following two changes were


introduced with respect to elections to the
Rajya Sabha:
Domicile or residency requirement of a
candidate contesting an election to the
Rajya Sabha was removed. Prior to this,
a candidate had to be an elector in the
state from where he was to be elected.
Now, it would be sufficient if he is an
elector in any parliamentary
constituency in the country.
Introducing open ballot system, instead
of secret ballot system, for elections to
the Rajya Sabha. This was done to curb
cross-voting and to wipe out the role of
money power during Rajya Sabha
elections. Under the new system, an
elector belonging to a political party has
to show the ballot paper after marking
his vote to a nominated agent of that
political party
19. Solution: b)

At of 2013, a party is recognised as a national


party if any of the following conditions is
fulfilled:

If it secures six per cent of valid votes


polled in any four or more states at a
general election to the Lok Sabha or to
the legislative assembly; and, in
addition, it wins four seats in the Lok
Sabha from any state or states; or
If it wins two per cent of seats in the Lok
Sabha at a general election; and these
candidates are elected from three states;
or
If it is recognised as a state party in four
states.

http://www.insightsonindia.com INSIGHTS

20.

Solution: b)

The 52nd Amendment Act of 1985 provided for


the disqualification of the members of
Parliament and the state legislatures on the
ground of defection from one political party to
another. For this purpose, it made changes in
four Articles of the Constitution and added a
new Schedule (the Tenth Schedule) to the
Constitution. This act is often referred to as the
anti-defection law.
These provisions facilitate democratic
realignment of parties in the legislature by way
of merger of parties. It also reduces corruption
at the political level as well as nondevelopmental expenditure incurred on
irregular elections. And, for the first time, it
gives, a clear-cut constitutional recognition to
the existence of political parties.

21. Solution: a)
Article 323 A empowers the Parliament to
provide for the establishment of administrative
tribunals for the adjudication of disputes
relating to recruitment and conditions of
service of persons appointed to public services
of the Centre, the states, local bodies, public
corporations and other public authorities.
Under Article 323 B, the Parliament and the
state legislatures are authorised to provide for
the establishment of tribunals for the
adjudication of disputes relating to the
following matters: (a) Taxation; (b) Foreign
exchange, import and export;(c) Industrial and
labour; (d) Land reforms; (e) Ceiling on urban
Page 7


INSIGHTS MOCK TEST SERIES 2015: TEST 32 SOLUTIONS

property; (f) Elections to Parliament and state


legislatures; (g) Food stuffs; (h) Rent and
tenancy rights.

22.

Solution: a)

The Constitution imposes a duty upon the


Centre to promote the spread and development
of the Hindi language so that it may become
the lingua franca of the composite culture of
India.
Further, the Centre is directed to secure the
enrichment of Hindi by assimilating the forms,
style and expressions used in hindustani and in
other languages specified in the Eighth
Schedule and by drawing its vocabulary,
primarily on sanskrit and secondarily on other
languages.
Also, the members of these languages are to be
given representation in the Official Language
commission. The Commission consisted of a
chairman and other members representing the
different languages specified in the Eighth
Schedule of the Constitution.

23.

Solution: a)

The functions of the Commission are, inter


alia:

To inquire into any violation of human


rights or negligence in the prevention of
such violation by a public servant, either

http://www.insightsonindia.com INSIGHTS

suo motu or on a petition presented to it


or on an order of a court.
To intervene in any proceeding
involving allegation of violation of
human rights pending before a court.
To visit jails and detention places to
study the living conditions of inmates
and make recommendation thereon.
To review the constitutional and other
legal safeguards for the protection of
human rights and recommend measures
for their effective implementation.
To review the factors including acts of
terrorism that inhibit the enjoyment of
human rights and recommend remedial
measures.
To study treaties and other international
instruments on human rights and make
recommendations for their effective
implementation.

24.

Solution: b)

The Union Executive consists of the President,


Vice-President, PM, Council of Ministers and
Attorney General of India.
The AG is not a member of the Central cabinet.
There is a separate law minister in the Central
cabinet to look after legal matters at the
government level.
In the performance of his official duties, the
Attorney General has the right of audience in
all courts in the territory of India. Further, he
has the right to speak and to take part in the
proceedings of both the Houses of Parliament
or their joint sitting and any committee of the

Page 8


INSIGHTS MOCK TEST SERIES 2015: TEST 32 SOLUTIONS

Parliament of which he may be named a


member, but without a right to vote.

25.

Solution: a)

The role of CAG is to uphold the Constitution


of India and the laws of Parliament in the field
of financial administration.
The accountability of the executive (i.e., council
of ministers) to the Parliament in the sphere of
financial administration is secured through
audit reports of the CAG.
The CAG is an agent of the Parliament and
conducts audit of expenditure on behalf of the
Parliament.
Therefore, he is responsible only to the
Parliament.

26.

Solution: c)

Though the constitution has sought to


safeguard and ensure the independence and
impartiality of the Election Commission, some
flaws can be noted, viz.,

The conditions of service and tenure of office of


the election commissioners and the regional
commissioners shall be determined by the
president.

The Constitution has not prescribed the


qualifications (legal, educational,
administrative or judicial) of the
members of the Election Commission.
The Constitution has not specified the
term of the members of the Election
Commission.
The Constitution has not debarred the
retiring election commissioners from
any further appointment by the
government.

http://www.insightsonindia.com INSIGHTS

27.

Solution: b)

The following best sums up the relationship


between the President, Administrator and the
UT government by giving the example of Delhi.
The chief minister is appointed by the
President (not by the lt. governor). The other
ministers are appointed by the president on the
advice of the chief minister. The ministers hold
office during the pleasure of the president. The
council of ministers is collectively responsible
to the assembly.
The council of ministers headed by the chief
minister aid and advise the lt. governor in the
exercise of his functions except in so far as he is
required to act in his discretion. In the case of
difference of opinion between the lt. governor
and his ministers, the lt. governor is to refer the
matter to the president for decision and act
accordingly.

28.

Solution: b)

74th Amendment Act of 1992 provides for the


constitution of the following three types of
municipalities in every state.

A nagar panchayat for a transitional


area, that is, an area in transition from a
rural area to an urban area.
A municipal council for a smaller urban
area.
Page 9


INSIGHTS MOCK TEST SERIES 2015: TEST 32 SOLUTIONS

A municipal corporation for a larger


urban area

29.

Solution: d)

The controversial Gujarat Control of Terrorism


and Organised Crime (GCTOC) Bill, 2015, has
been sent back to the State following an
objection by the Information Technology (IT)
Ministry.
The Home Affairs Ministry, which sent the Bill
for an inter-ministerial consultation, returned
it with the objections raised by the IT Ministry.
This means the Bill will not be sent for
Presidential assent and cannot become law yet.
Any Bill passed by an Assembly on issues
contravening Central laws needs Presidential
assent.
The Gujarat House passed the Bill again on
March 31 this year, after it was rejected thrice
by two former Presidents the late A.P.J.
Abdul Kalam in 2004 and Pratibha Patil in
2008 and 2009.
The Bill was first introduced as the GUJCOC
Bill in 2003 when Narendra Modi was Chief
Minister with provisions like increasing the
period to file charge sheet from 90 to 180 days
and strict conditions for granting bail to an
accused.

30.

Solution: b)

A Bio fertilizer is a substance which contains


living microorganisms which, when applied to
seed, plant surfaces, or soil, colonizes the
http://www.insightsonindia.com INSIGHTS

rhizosphere or the interior of the plant and


promotes growth by increasing the supply or
availability of primary nutrients to the host
plant.
Bio-fertilizers add nutrients through the
natural processes of nitrogen fixation,
solubilizing phosphorus, and stimulating plant
growth through the synthesis of growthpromoting substances.
Bio-fertilizers can be expected to reduce the
use of chemical fertilizers and pesticides. The
microorganisms in bio-fertilizers restore the
soil's natural nutrient cycle and build soil
organic matter.

31. Solution: b)
Pakistan received $336 million from the United
States for its ongoing role in combating a
Taliban insurgency in neighbouring
Afghanistan.
Regular payments to Pakistan under the
Coalition Support Fund (CSF) programme
began in 2001 when Pakistan joined the US-led
coalition in Afghanistan as a frontline ally.
Pakistan provides use of its air bases and other
facilities in exchange for the reimbursements.
Pakistan has received a total of $13 billion in
CSF payments since the programme began.
http://www.thehindu.com/todays-paper/tpinternational/pak-gets-336m-to-supportforces-in-afghanistan/article7475501.ece

Page 10


INSIGHTS MOCK TEST SERIES 2015: TEST 32 SOLUTIONS

32.

Solution: d)

http://www.thehindu.com/todays-paper/takesteps-to-prevent-sea-erosion-officialstold/article7479929.ece
Coastal erosion is the wearing away of land and
the removal of beach or dune sediments by
wave action, tidal currents, wave currents,
drainage or high winds. Waves, generated by
storms, wind, or fast moving motor craft, cause
coastal erosion, which may take the form of
long-term losses of sediment and rocks, or
merely the temporary redistribution of coastal
sediments; erosion in one location may result
in accretion nearby. The study of erosion and
sediment redistribution is called 'coastal
morphodynamics'. It may be caused by
hydraulic action, abrasion, impact and
corrosion.
Secondary factors that cause it are:

Weathering and transport slope


processes

Slope hydrology

Vegetation

Cliff foot erosion

Cliff foot sediment accumulation

Resistance of cliff foot sediment to


attrition and transport

Human Activity

Tertiary factors

Resource extraction

Coastal management

http://www.insightsonindia.com INSIGHTS

33.

Solution: d)

The maritime domain awareness systems


inaugurated during Mr. Modis visit to
Seychelles, in March 2015, were the outcome of
an older initiative encompassing several Indian
Ocean countries
http://www.thehindu.com/todays-paper/tpopinion/all-form-littlesubstance/article7478857.ece
Maritime domain awareness (MDA) is defined
by the International Maritime Organization as
the effective understanding of anything
associated with the maritime domain that
could impact the security, safety, economy, or
environment. The maritime domain is defined
as all areas and things of, on, under, relating to,
adjacent to, or bordering on a sea, ocean, or
other navigable waterway, including all
maritime-related activities, infrastructure,
people, cargo, and vessels and other
conveyances.

34.

Solution: b)

It is Asias highest honor and often


regarded as the regions equivalent of
the Nobel Prize.

Established: in 1957 in the memory of


Philippines 3rd President Ramon
Magsaysay who had died in air disaster
in March 1957.

Page 11


INSIGHTS MOCK TEST SERIES 2015: TEST 32 SOLUTIONS

It was established by trustees of the New


York City based Rockefeller Brothers
Fund and Philippine government.

It is awarded annually to individuals or


organizations from Asia region for their
altruistic and philanthropic service.

Award carries: Medallion bearing the


likeness of the late President Ramon
Magsaysay, cash prize and a certificate.

35.

Solution: b)

The Competition Commission of India (CCI)


has imposed fine of 420 crore rupees on the
local unit of Hyundai Motor Co.
The fined was imposed after CCI investigation
had found that Hyundai was engaged in anticompetitive practices.

CCI is a quasi-judicial statutory body


whose predecessor was the MRTPC
(Monopolies and Restrictive Trade
Practices Commission).

It was established 2003 as per the


provisions of The Competition Act, 2002
and became fully functional only by
2009.

It was established with an aim to


eliminate anti-competitive practices that
adversely affect competition in different
industries/areas and protect interests of
consumers and ensure freedom of trade
and also to give opinions to statutory
authorities.

http://www.insightsonindia.com INSIGHTS

36.

Solution: c)

Kazakhstan, a member of the Russia-led


economic blocks became 162th member of
World Trade Organization (WTO).
Kazakhstans entry has ended its long wait of
20 years to become a member of WTO as its
accession talks were initiated in 1996.
The process of becoming a WTO member is
unique to each applicant country, and the
terms of accession are dependent upon the
country's stage of economic development and
current trade regime.
The process takes about five years, on average,
but it can last longer if the country is less than
fully committed to the process or if political
issues interfere.
http://economictimes.indiatimes.com/news/in
ternational/business/wto-formally-acceptskazakhstan-as-newmember/articleshow/48241244.cms

37.

Solution: d)

The population of Royal Bengal Tiger in


Sundarbans region of Bangladesh has declined
sharply to 106 from 440 in 2004. It was
revealed in recently released new Tiger census
2015 of Bangladesh.
The Tiger Census was conducted
by Bangladesh-India Joint Tiger Census
Project. It was carried under the initiative
of Strengthening Regional Cooperation for
Wildlife Protection in Asia Project. Financial

Page 12


INSIGHTS MOCK TEST SERIES 2015: TEST 32 SOLUTIONS

appearance of time trials. The race route


passes through the mountain chains of
Pyrenees, Alps and finish on the
Champs-lyses in Paris.

support to this initiative was provided by the


World Bank.
Methodology: Census 2015 was conducted by
using modern technology of hidden cameras to
count tigers. Previous methodology was based
on count of counting pug marks. The camera
tapping method is more accurate than past
method.
Reasons for decline in Population: The rapid
decline in population was attributed to loss of
habitat, lack of proper forest management,
unchecked wildlife poaching and animalhuman conflict in the worlds largest mangrove
forests of Sunderbans.
It should be noted that in Bangladesh,
Sunderbans are spread over 6097 sq.km and it
is only natural habitat of Royal Bengal tigers.
There are around 74 tigers on the Indian side
of the Sundarbans which stretches for nearly
4,000 miles across both countries.

38.

39.

China recently launched two new navigational


satellites into space as part of its domestic
navigation system BeiDou (meaning Compass).
India has also been adding satellites to its
IRNSS.

These navigational satellites are a


constellation of satellite helping provide
accurate location on earth.

All such satellites seek to reduce reliance


on GPS developed by USA.

40.

Tour de France was first organized in 1903 and


currently run by the Amaury Sport
Organisation.

The prestigious cycle race is 3 weeklong, traditionally held every year in the
month of July and covers around 3,500
kms.

Each year the route of race changes but


the format remains same with the

http://www.insightsonindia.com INSIGHTS

Solution: a).

It focuses on feeder separation for rural


households and agricultural purpose.

Seeks to strengthen the distribution and


sub-transmission infrastructure,
including metering at all levels in rural
areas.

Major components of the new


scheme: strengthening of subtransmission and distribution network,
feeder separation, Metering at all levels
including at input points, feeders and
distribution transformers.

It also seeks to strengthen Micro grid


and off grid distribution network of
rural electrification.

Solution: b)

High atlas is in Africa. Cordillera is in Asia.


And Beaufort range is in Canada. Only Alps
range is in Europe covering France.

Solution: b)

Page 13


INSIGHTS MOCK TEST SERIES 2015: TEST 32 SOLUTIONS

The scheme has budgetary outlay of


76,000 crore rupees for implementation
of the projects of which Union
Government will provide grant of
63,000 crore rupees.

41. Solution: c)
Union Ministry of Environment, Forest and
Climate Change (MoEFCC) along with the 3
state governments will soon start an artificial
breeding programme of Great Indian Bustard
(GIB).
Under this integrated collaborative effort, 3
states viz. Rajasthan, Gujarat and Maharashtra
will take part to boost the population of GIB,
one of the critically endangered flying bird
species in the world.
The artificially breeding operations of GIBs will
begin in 2016 across these 3 states and they
will be later released from captivity to semicaptivity after some time of initial operations.
GIB has been exterminated from almost 90 per
cent of their former range even though it has
ability to survive in harsh weather conditions.
The main reasons are Habitat loss and
poaching.

42.

Solution: c)

Long March-5 rocket carrier was recently


tested in China.

http://www.insightsonindia.com INSIGHTS

Long March-5 has the carrying capacity


twice of the payload compared to other
present carriers of China.

It has payload capacity of 14 tonnes to


geostationary transfer orbit and 25
tonnes to low Earth orbits.

It will serve the final chapter in building


Chinas manned space stations and 3
step current program of orbiting,
landing and returning lunar.

ISRO has developed rockets to put satellites in


orbit: the Polar Satellite Launch Vehicle
(PSLV) for launching satellites into polar orbits
and the Geosynchronous Satellite Launch
Vehicle (GSLV) for placing satellites into
geostationary orbits. These rockets have
launched numerous communications satellites
and earth observation satellites.

43.

Solution: b)

Union Government has constituted All India


Council of Sports (AICS), as an advisory body
to deliberate on matters relating to the
development and promotion of sports in the
country.
Government will duly consider the advice
rendered by this Council, but it will not be
binding and obligatory on the government.

Composition: It will be headed by a


president in the rank of Minister of State
(MoS). It will include 4 Members of
Parliament (MPs), coaches, experts,
sports personalities, administrators and
Director General (DG) of Sports
Authority of India (SAI).
Page 14


INSIGHTS MOCK TEST SERIES 2015: TEST 32 SOLUTIONS

Appointments: President and members,


other than ex-officio members will be
appointed by the Union Government.
Meeting: It will meet from time to time
i.e. at least once in a quarter of year to
deliberate on matters related sports and
games in the country.
Functions: Popularize sports amongst
the youth and increase its outreach to
rural and tribal areas along with North
East and Jammu & Kashmir, areas
affected by Left Wing Extremism.

Promote inclusiveness in sports with


special focus on women, tribals and
differently-abled.

Prevent fraud of age and sexual


harassment of women in sports along
with drug abuse.

Bring transparency, professionalism and


good governance in functioning of
National Sports Federations (NFS).

Promote sports sciences and sports


medicine and augment the sports
infrastructure and ensure its proper
utilization.

Tackle the issues arising malpractices in


competitive sports especially match
fixing and early identification of sports
talent and nurturing the identified
talent.

44.

Solution: a)

Union Government has launched a national


campaign Surakshit Khadya Abhiyan in a bid to
http://www.insightsonindia.com INSIGHTS

create awareness and capacity building on Safe


and Hygienic food for all.
It was launched by Union Minister of
Consumer Affairs, Food and Public
Distribution Ram Vilas Paswan in New Delhi.

Objective is building a culture of food


safety in the country.

It will ensure the safe food to consumers


apart from providing them Food security
in order to address the food
requirements.

Initiated under the brand name of Jago


Grahak Jago, which is mass awareness
campaign which creates awareness of
consumer rights and demands.

It is planned by Confederation of Indian


Industry (CII) along with Consumer
Organization VOICE and National
Association of Street Vendors of India
(NASVI).

45.

Solution: a)

Formerly known as Triana, unofficially known


as GoreSat, it is a Earth observation and space
weather satellite launched by SpaceX on a
Falcon 9 launch vehicle on February 11, 2015
from Cape Canaveral.
It was originally developed as a NASA satellite
proposed in 1998 by then-Vice President Al
Gore for the purpose of Earth observation. It is
at the Sun-Earth L1 Lagrangian point,
1,500,000 km (930,000 mi) from Earth, to
monitor variable solar wind condition, provide
early warning of approaching coronal mass
Page 15


INSIGHTS MOCK TEST SERIES 2015: TEST 32 SOLUTIONS

ejections and observe phenomena on Earth


including changes in ozone, aerosols, dust and
volcanic ash, cloud height, vegetation cover and
climate.
At this location it will have a continuous view
of the Sun and the sunlit side of the Earth. It
will take full-Earth pictures about every two
hours and be able to process them faster than
other Earth observation satellites.

46.

Solution: d)

http://www.downtoearth.org.in/news/mangro
ve-forests-help-protect-against-rising-sealevel-says-study-50598
Mangrove forests prevent the effects of
cyclones, storms and other calamities by this
simple mechanim. This is likely due to soil
building up around their mesh-like roots and
acting to reduce energy from waves and tidal
currents, scientists say.
Estuaries and recesses in coastlines that form
bays receive the run-off from erosion on steep
catchments which give them the tendency to fill
in over time.
As they fill in, movement of tidal currents over
the shallow areas create networks of sandbanks
and channels. The sand banks grow upward to
keep pace with water level changes while the
channels get deeper to efficiently drain out the
excess water out to the sea.

47.

Solution: a)

http://www.insightsonindia.com INSIGHTS

Ocean acidification supports coral reef


formation as more carbon is dissolved in water
which helps forming those hard shales.
But, the higher acidity has been proven to
inhibit shell growth in marine animals and is
suspected as a cause of reproductive disorders
in some fish. Unless humans are able to control
and eventually eliminate fossil fuel emissions,
ocean organisms will find themselves under
increasing pressure to adapt to their habitat's
changing chemistry or perish.
Any changes happening in that system would
naturally cause an imbalance in the marine
ecosystem, which would further affect the food
chain.
A study has also found that the rise in ocean
acidification could lead affect phytoplankton,
which forms the base of the marine food chain.
Some researchers also observe that as ocean
acidification prompted some species to grow
faster and others slower, it also changed the
natural competition between species.

48.

Solution: d)

Audiovisual components, televisions, VCRs,


stereo equipment, mobile phones, other
handheld devices, and computer components
contain valuable elements and substances
suitable for reclamation, including lead,
copper, and gold.
One of the major challenges is recycling the
printed circuit boards from the electronic
wastes. The circuit boards contain such
Page 16


INSIGHTS MOCK TEST SERIES 2015: TEST 32 SOLUTIONS

precious metals as gold, silver, platinum, etc.


and such base metals as copper, iron,
aluminum, etc. One way e-waste is processed is
by melting circuit boards, burning cable
sheathing to recover copper wire and open- pit
acid leaching for separating metals of value

49.

Solution: b)

It will be an Innovation Promotion Platform


involving academics, entrepreneurs and
researchers and draw upon national and
international experiences to foster a culture of
innovation, R&D and scientific research in
India.
The platform will also promote a network of
world-class innovation hubs and Grand
Challenges for India.
Initially a sum of Rs.150 crore will be
earmarked for this purpose.

50.

fifth member of the committee. The


Chairperson will nominate one of its
employees as the sixth member.

51. Solution: a)
The GST Bill seeks to establish a GST Council
tasked with optimising tax collection for goods
and services by the State and Centre

Composition: The Council will consist of


the Union Finance Minister (as
Chairman), the Union Minister of State
in charge of revenue or Finance, and the
Minister in charge of Finance or
Taxation or any other, nominated by
each State government.

The GST Council will be the body that


decides which taxes levied by the Centre,
States and local bodies will go into the
GST; which goods and services will be
subjected to GST; and the basis and the
rates at which GST will be applied.

Solution: c)

The Union government has proposed to strip


the Reserve Bank Governors veto vote on
Indias monetary policy. The government has
also proposed to grant itself the power to
appoint four of the six members of the
Monetary Policy Committee, whose remit will
include decisions on setting interest rates to
maintain inflation at the targeted level.

52.

Solution: c)

Although Indian constitution and law do not


provide for double citizenship normally, the
provision of dual citizenship is only applicable
to those children of diplomats who were born
outside India.

Under the revised draft, the nongovernment members of the committee


are to be drawn from the Reserve Bank.

http://articles.economictimes.indiatimes.com/
2014-05-12/news/49795348_1_dualcitizenship-devyani-khobragade-s-indiancitizenship

The Reserve Banks Board will


nominate one of its executives as the

http://www.oneindia.com/india/mha-rejects-

http://www.insightsonindia.com INSIGHTS

Page 17


INSIGHTS MOCK TEST SERIES 2015: TEST 32 SOLUTIONS

khobragade-s-plea-dual-citizenship-kids1814429.html
Children of foreign diplomats, who are born in
India, are also given dual citizenship till the
period of their parents service in India.

53.

Solution: b)

Losses of vitamins depend on cooking time,


temperature, and cooking method. Some
vitamins are quite heat-stable, whereas others
are heat-labile.
Many other factors than heat can destroy
(some) vitamins, such as: solubility in water,
exposure to air (oxidation), exposure to light
(UVs), heat, acid and alkaline solutions,
storage losses, etc.
Especially vitamin C is more vulnerable to
cooking.

54.

Solution: a)

55.

Solution: d)

There are two kinds of chalk. The mineral


Chalk is a porous form of limestone composed
of calcium carbonate. Blackboard chalk is
calcium sulphate.

Jute fibre is obtained from the stem of the jute


plant. It is cultivated during the rainy season.
In India, jute is mainly grown in West Bengal,
Bihar and Assam.

The chemical bonds are too strong to dissolve


in water. It does dissolve in acids. Many
minerals do not dissolve in water.

The jute plant is normally harvested when it is


at flowering stage.

In both versions of chalk there is low solubility


in water.

The stems of the harvested plants are


immersed in water for a few days. The stems
rot and fibres are separated by hand.

56.

Solution: b)

The solubility of a given solute in a given


solvent typically depends on temperature.

http://www.insightsonindia.com INSIGHTS

Page 18


INSIGHTS MOCK TEST SERIES 2015: TEST 32 SOLUTIONS

Depending on the nature of the solute the


solubility may increase or decrease with
temperature. For many solids dissolved in
liquid water, the solubility increases with
temperature up to 100 C.
So, if you add ice first the sugar will take more
time to dissolve. In relatively warm water, it is
possible to dissolve more sugar at a faster pace.

57.Solution: d)
Plants make food in their leaves. The leaves
contain a pigment called chlorophyll, which
colors the leaves green. Chlorophyll can make
food the plant can use from carbon dioxide,
water, nutrients, and energy from
sunlight. This process is called photosynthesis.
Besides leaves, photosynthesis also takes place
in other green parts of the plant in green
stems and green branches. The desert plants
have scale- or spine-like leaves to reduce loss of
water by transpiration. These plants have green
stems which carry out photosynthesis.

58.

Solution: d)

Saprophytic organisms, or saprophytes, derive


their food from dead organisms. They secrete
enzymes that are released on food material
outside their body. These enzymes break down
complex food into simple forms. Common
examples of saprophytes are fungi (moulds,
mushrooms, yeasts) and many bacteria.
Parasitic organisms, or parasites, live on or
inside other living organisms, called hosts, and
obtain their food from them. The host does not
http://www.insightsonindia.com INSIGHTS

get any benefit from the parasite. Different


parasites, like Cuscuta (akash-bel), Cassytha
(amar-bel), hookworms, tapeworms, leeches,
etc., have different modes of feeding,
depending upon habit, habitat and
modifications.

59.

Solution: a)

Wool industry is an important means of


livelihood for many people in our country. But
sorters job is risky as sometimes they get
infected by a bacterium, anthrax, which causes
a fatal blood disease called sorters disease.
Such risks faced by workers in any industry are
called occupational hazards.
Anthrax is a rare infectious disease caused by
the bacterium Bacillus anthracis. Anthrax
occurs naturally around the world in wild and
domestic hoofed animals, especially cattle,
sheep, goats, camels and antelopes. It can also
occur in humans when they are exposed to the
bacterium, usually through handling animals
or animal hides. There are three forms of
anthrax infection: cutaneous (skin), inhalation
(lungs) and gastrointestinal (stomach and
intestine). If people have been intentionally
exposed, as in a bioterrorist release, contact
with skin would be the most likely route of
exposure. Breathing in the spores that have
been spread through the air could cause
inhalation anthrax.
60.

Solution: c)

Copper is a very good conductor of heat and is


shiny in appearance. Because of these

Page 19


INSIGHTS MOCK TEST SERIES 2015: TEST 32 SOLUTIONS

properties it was the preferred material for


kitchen utensils.
But its use is now confined to just outer layer of
kitchen pots that are used in cooking.
Copper toxicity is reduced to zero by using
stainless steel lining on the inner side of the
vessel.

61. Solution: d)
The indicators change their colour when added
to a solution containing an acidic or a basic
substance. Turmeric, litmus, china rose petals
(Gudhal), etc., are some of the naturally
occurring indicators.
The most commonly used natural indicator is
litmus. It is extracted from lichens. It has a
mauve (purple) colour in distilled water. When
added to an acidic solution, it turns red and
when added to a basic solution, it turns blue.
It is available in the form of a solution,
or in the form of strips of paper, known
as litmus paper.
Generally, it is available as red and blue litmus
paper

62.

Solution: d)

As the name indicates the rain containing


excess of acids is called an acid rain. The rain
becomes acidic because carbon dioxide,
sulphur dioxide and nitrogen dioxide (which
are release into the air as pollutants) dissolve
in rain drops to form carbonic acid, sulphuric
acid and nitric acid respectively.
http://www.insightsonindia.com INSIGHTS

Acid rain can also have a damaging effect on


many objects, including buildings, statues,
monuments, and cars. The chemicals found in
acid rain can cause paint to peel and stone
statues to begin to appear old and worn down,
which reduces their value and beauty.
Acid rain causes acidification of lakes and
streams and contributes to the damage of trees
at high elevations (for example, red spruce
trees above 2,000 feet) and many sensitive
forest soils.
You can understand more here
http://epa.gov/acidrain/education/site_stude
nts/whyharmful.html

63.

Solution: c)

Excessive use of chemical fertilisers makes the


soil acidic. Plants do not grow well when the
soil is either too acidic or too basic.
When the soil is too acidic, it is treated with
bases like quick lime (calcium oxide) or slaked
lime (calcium hydroxide).
If the soil is basic, organic matter is added to it.
Organic matter releases acids which neutralises
the basic nature of the soil.

64.

Solution: b)

Some migratory birds travel as much as 15000


km to escape the extreme climatic conditions at
home. Generally they fly high where the wind
flow is helpful and the cold conditions allow
them to disperse the heat generated by their
flight muscles.
Page 20


INSIGHTS MOCK TEST SERIES 2015: TEST 32 SOLUTIONS

But how these birds travel to the same place


year after year is still a mystery. It seems that
these birds have a builtin sense of direction
and know in which direction to travel. Some
birds probably use landmarks to guide them.
Many birds may be guided by the sun during
the day and stars at night. There is some
evidence that birds may use the magnetic field
of the earth to find direction.
And it is not only birds that migrate; mammals,
many types of fish and insects are also known
to migrate seasonally in search of more
hospitable climates.

65.

Solution: d)

All these species have been mentioned with


photographs in your 7th NCERT book. Keep an
eye on such prominent facts and details while
reading NCERTs. They are very likely to come
in exams.

66.

Solution: c)

We must take the following precautions:


Do not take shelter under an isolated tree. If
you are in a forest take shelter under a small
tree. Do not lie on the ground.
Do not take shelter under an umbrella with a
metallic end.
Do not sit near a window. Open garages,
storage sheds, metal sheds are not safe places
to take shelter. Open garages increase the risk
of an electrical shock.
Instead, a car or a bus is a safer place to take
shelter. Electric current is discharged from the
outer surface itself.
If you are in water, get out and go inside a
building.

67.

Solution: a)

The whole coastline of India is vulnerable to


cyclones, particularly the east coast.
The west coast of India is less vulnerable to
cyclonic storms both in terms of intensity and
frequency of the cyclones.
Moreover, tornadoes are also not very frequent.
Most tornadoes are weak and they rarely reach
high speeds upto 300 kmph.
Tornadoes are formed within cyclones.
68.

Solution: d)

The soil is classified on the basis of the


proportion of particles of various sizes. If soil
contains greater proportion of big particles it is
http://www.insightsonindia.com INSIGHTS

Page 21


INSIGHTS MOCK TEST SERIES 2015: TEST 32 SOLUTIONS

called sandy soil. If the proportion of fine


particles is relatively higher, then it is called
clayey soil. If the amount of large and fine
particles is about the same, then the soil is
called loamy.
The best topsoil for growing plants is loam.
Loamy soil is a mixture of sand, clay and
another type of soil particle known as silt. Silt
occurs as a deposit in river beds. The size of the
silt particles is between those of sand and clay.
The loamy soil also has humus in it. It has the
right water holding capacity for the growth of
plants.

69.

Solution: c)

Our muscle cells can also respire anaerobically,


but only for a short time, when there is a
temporary deficiency of oxygen. During heavy
exercise, fast running, cycling, walking for
many hours or heavy weight lifting, the
demand for energy is high. But the supply of
oxygen to produce the energy is limited.
The cramps occur when muscle cells respire
anaerobically. The partial breakdown of
glucose produces lactic acid. The accumulation
of lactic acid causes muscle cramps. We get
relief from cramps after a hot water bath or a
massage.
Hot water bath or massage improves
circulation of blood. As a result, the supply of
oxygen to the muscle cells increases. The
increase in the supply of oxygen results in the
complete breakdown of lactic acid into carbon
dioxide and water.

http://www.insightsonindia.com INSIGHTS

70.

Solution: d)

The shape of the population pyramid tells the


story of the people living in that particular
country. The numbers of children (below 15
years) are shown at the bottom and reflect the
level of births. The size of the top shows the
numbers of aged people (above 65 years) and
reflects the number of deaths.
The population pyramid of a country in which
birth and death rates both are high is broad at
the base and rapidly narrows towards the top.
This is because although, many children are
born, a large percentage of them die in their
infancy, relatively few become adults and there
are very few old people.

71. Solution: d)
Joint sector industries are owned and operated
by the state and individuals or a group of
individuals. Maruti Udyog Limited is an
example of joint sector industry.
Co-operative sector industries are owned and
operated by the producers or suppliers of raw
materials, workers or both.
Anand Milk Union Limited and Sudha Dairy
are success stories of a co-operative venture.

72.

Solution: c)

The first successful modern textile mill was


established in Mumbai in 1854. The warm,
Page 22


INSIGHTS MOCK TEST SERIES 2015: TEST 32 SOLUTIONS

moist climate, port for importing machinery,


availability of raw material and skilled labour
resulted in rapid expansion of the industry in
the region.
Initially this industry flourished in the states of
Maharashtra and Gujarat because of favourable
humid climate. But today, humidity can be
created artificially, and raw cotton is a pure
and not weight losing raw material, so this
industry has spread to other parts of India.
A weight losing material means it loses a lot of
weight after processing. So, if the loss of waste
is larger, it does not make sense to transport it
for long distances. Instead, a weight losing
material will be processed on site and therefore
industries will always be located near farms. It
is not the case with cotton.
Coimbatore, Kanpur, Chennai, Ahmedabad,
Mumbai, Kolkata, Ludhiana, Pondicherry and
Panipat are some of the other important
centres.

sunshine at the time of harvest. It thrives best


in well drained loamy soil.

74.

Solution: d)

Natural gas is found with petroleum deposits


and is released when crude oil is brought to the
surface.
In India Jaisalmer, Krishna Godavari delta,
Tripura and some areas off shore in Mumbai
have natural gas resources.
A huge mass of Indias natural gas production
comes from the western offshore regions,
particularly the Mumbai High complex. The
onshore fields in Assam, Andhra Pradesh, and
Gujarat states are also major producers of
natural gas.

75.Solution: d)
Contour ploughing is ploughing parallel to the
contours of a hill slope to form a natural
barrier for water to flow down the slope.

73.

Solution: b)

Rice is the staple diet of the tropical and subtropical regions. Rice needs high temperature,
high humidity and rainfall. It grows best in
alluvial clayey soil, which can retain water.
In favourable climatic conditions as in West
Bengal and Bangladesh two to three crops a
year are grown.

Contour ploughing follows the natural shape of


the slope without altering it. Terrace farming
alters the shape of the slope to produce flat
areas that provide a catchment for water and a
solid area for crop growth.
In Terrace farming, terraces are made on the
steep slopes so that flat surfaces are available
to grow crops. They can reduce surface run-off
and soil Erosion.

Wheat requires moderate temperature and


rainfall during growing season and bright
76.
http://www.insightsonindia.com INSIGHTS

Solution: c)
Page 23


INSIGHTS MOCK TEST SERIES 2015: TEST 32 SOLUTIONS

Specialization is a method of production where


a business or area focuses on the production of
a limited scope of products or services in order
to gain greater degrees of productive efficiency
within the entire system of businesses or areas.
Many countries specialize in producing the
goods and services that are native to their part
of the world. This specialization is the basis of
global trade as few countries produce enough
goods to be completely self-sufficient.

78.

Solution: d)

When a currency appreciates or strengthens in


relation to other currencies, imports get
cheaper. This means your dollar will buy more
of another foreign currency so that you can
purchase foreign goods.
One of the biggest disadvantages of higher
exchange rates or a strong dollar may be that it
leads to trade deficits. Because strong
currencies lead to cheaper imports, a country
tends to import more than they export. This
causes a trade deficit (not fiscal deficit).

77. Solution: b)
The various components of money can be
aggregated together in order of liquidity. Since
money is primarily used to settle day-to-day
transactions, it needs to be readily usable as a
means of doing so. Clearly, cash and coins are
the most liquid forms of money, since they can
be used instantaneously and universally used
to settle unlimited transactions. Demand
deposits, or accounts with banks, are also quite
liquid, as they can be used to write cheques
against, in settling daily transactions. The sum
of currency in circulation and demand deposits
with banks is called M1, or 'narrow money'.
Time deposits, though not as liquid and
instantly available as transactions settling
medium as M1, are still money, since it will be
available at some point, and very often, as in
the case of fixed deposits, can be converted to
cash for some penalty. Usually, time deposits
are much larger than both currency in
circulation and demand deposits. The sum of
M1 and time deposits is called 'broad money'.

http://www.insightsonindia.com INSIGHTS

79.

Solution: b)

Central Government levies taxes on income


(except tax on agricultural income, which the
State Governments can levy), customs duties,
central excise and service tax. Value Added Tax
(VAT), (Sales tax in States where VAT is not yet
in force), stamp duty, State Excise, land
revenue and tax on professions are levied by
the State Governments.
Some taxes are levied and collected by the
Centre but shared with the States. These
include taxes on income other than agricultural
income and union excise duties on goods
included in Union List, except medicinal and
toilet preparations.
In last 10-15 years, Indian taxation system has
undergone tremendous reforms. The
Constitution (Eightieth Amendment) Act,
2000, significantly changed the manner of
Page 24


INSIGHTS MOCK TEST SERIES 2015: TEST 32 SOLUTIONS

distribution of Central tax collections between


the Central and State Governments. Prior to
this amendment, income tax and Union excise
duties were the only taxes shared with the
States.

80.

Solution: c)

The main difference between revenue receipts


and capital receipts is that revenue receipts are
recurring in nature, which the government can
expect to receive year after year, whereas
capital receipts are a kind of one-time income.
So, earnings from interest rates cannot be
counted as capital receipts as they are recurring
income in nature. They come under revenue
receipts.

atmosphere around it. This is because


atmosphere has a weight.
Primordial heat is responsible for many
tectonic events on the earth like volcanism,
plate movement etc.
Coriolis force is responsible for wind
movement not attraction of atmosphere of
earth.

82.

Solution: c)

The crust forms only 0.5 per cent of the


volume of the earth, 16 per cent consists of the
mantle and 83 per cent makes the core.

Moreover, grants from governments are also


counted in the revenue receipt segment.
The main items of capital receipts are loans
raised by the government from the public
which are called market borrowings, borrowing
by the government from the Reserve Bank and
commercial banks and other financial
institutions through the sale of treasury bills,
loans received from foreign governments and
international organisations, and recoveries of
loans granted by the central government.

81.Solution: d)

83.

Solution: d)

The earth movements are divided on the basis


of the forces which cause them. The forces
which act in the interior of the earth are called
as Endogenic forces and the forces that work
on the surface of the earth are called as
Exogenic forces.

The atmosphere is the thin layer of air (gases,


dust etc.) that surrounds the earth. The
gravitational force of the earth holds the
http://www.insightsonindia.com INSIGHTS

Page 25


INSIGHTS MOCK TEST SERIES 2015: TEST 32 SOLUTIONS

When it stops blowing the sand falls and gets


deposited in low hill like structures. These
are called sand dunes.
When the grains of sand are very fine and light,
the wind can carry it over very long distances.
When such sand is deposited in large areas, it
is called loess. Large deposits of loess are found
in China.

85.

Solution: b)

In economics, Veblen goods are types of


material commodities for which the demand is
proportional to its high price, which is an
apparent contradiction of the law of demand;
Veblen goods also are commodities that
function as positional goods.
Veblen goods are types of luxury goods, such as
expensive wines, jewelry, fashion-designer
handbags, and luxury cars, which are in
demand because of the high prices asked for
them.

All these forces are responsible for shaping the


earth around you.

The high price makes the goods desirable as


symbols of the buyer's high social-status, by
way of conspicuous consumption and
conspicuous leisure; conversely, a decrease of
the prices of Veblen goods would decrease
demand for the products

86.
84.

Solution: a)

When the wind blows, it lifts and transports


sand from one place to another.

http://www.insightsonindia.com INSIGHTS

Solution: b)

In thermosphere temperature rises very rapidly


with increasing height.
Ionosphere is a part of this layer. It extends
between 80-400 kms. This layer helps in radio
transmission. In fact, radio waves transmitted
Page 26


INSIGHTS MOCK TEST SERIES 2015: TEST 32 SOLUTIONS

from the earth are reflected back to the earth


by this layer.
On the other hand, Mesosphere extends only
up to the height of 80 km.

False knowledge is not merely ignorance to


Naiyayikas, it includes delusion. Correct
knowledge is discovering and overcoming one's
delusions, and understanding true nature of
soul, self and reality.

Meteorites burn up in this layer on entering


from the space.
89.
87.

Solution: a)

It is calculated as the profit that a firm is


expected to earn considering the cost of inputs
and the depreciation of capital. It is influenced
by expectations about future input costs and
demand. The MEC and capital outlays are the
elements that a firm takes into account when
deciding about an investment project.
The MEC needs to be higher than the rate of
interest, r, for investment to take place.

Prattyasamutpda, commonly translated as


dependent origination or dependent arising,
states that all dharmas ("things") arise in
dependence upon other dharmas: "if this
exists, that exists; if this ceases to exist, that
also ceases to exist."
It is a pragmatic teaching, which is applied to
dukkha and the cessation of dukkha.
The term is also used to refer to the twelve
links of dependent origination, which describes
the chain of causes which result in rebirth. By
reverting the chain, liberation from rebirth can
be attained.

90.
88.

Solution: d)

In its metaphysics, Nyaya school is closer to


Vaisheshika school of Hinduism than others. It
holds that human suffering results from
mistakes/defects produced by activity under
wrong knowledge (notions and ignorance).
Moksha (liberation), it states, is gained through
right knowledge. This premise led Nyaya to
concern itself with epistemology, that is the
reliable means to gain correct knowledge and
to remove wrong notions.

http://www.insightsonindia.com INSIGHTS

Solution: b)

Solution: a)

The Indian National Army and Netaji Subhash


Chandra Bose were playing their remarkable
part in the freedom struggle of India.
On the eve of the Second World War Bose came
to believe that Gandhi's Non-violent Movement
might not bring independence.
In 1939 he resigned from the congress
presidentship and organised forward Bloc for a
more vigorous national struggle. The
government first imprisoned him, and
thereafter kept him confined to his house
under close watch.
Page 27


INSIGHTS MOCK TEST SERIES 2015: TEST 32 SOLUTIONS

91. Solution: d)
The first war of Indian Independence was
fought in 1857 AD, during the reign of
Governor General of Lord Canning.
The immediate cause of the revolt of 1857 was
that Mangal Pandey killed two British officers
on parade at Barrackpore.
This revolt spread in Lucknow, Ambala and
Meerut cantonments. On May 10, 1857 soldiers
at Meerut refused to touch the new Enfield rifle
cartridges. The soldiers along with other group
of civilians, went on rampage.
They marched to Delhi, seized the city and
proclaimed the Bahadur Shah Zafar, emperor
of India. Due to the lack of unity and
organisation this revolt failed.

92.

Solution: c)

In July 1905, Lord Curzon, the Viceroy and


Governor-General (18991905), ordered the
partition of the province of Bengal supposedly
for improvements in administrative efficiency
in the huge and populous region. It also had
justifications due to increasing conflicts
between Muslims and dominant Hindu
regimes in Bengal.
However, the Indians viewed the partition as
an attempt by the British to disrupt the
growing national movement in Bengal and
divide the Hindus and Muslims of the region.
The Bengali Hindu intelligentsia exerted
considerable influence on local and national
politics.
http://www.insightsonindia.com INSIGHTS

Widespread agitation ensued in the streets and


in the press, and the Congress advocated
boycotting British products under the banner
of swadeshi.

93.

Solution: d)

The First World War began with an


unprecedented outpouring of support towards
Britain from within the mainstream political
leadership, contrary to initial British fears of an
Indian revolt. India contributed massively to
the British war effort by providing men and
resources.
About 1.3 million Indian soldiers and labourers
served in Europe, Africa and the Middle East,
while both the Indian government and the
princes sent large supplies of food, money and
ammunition.
However, Bengal and Punjab remained
hotbeds of anti colonial activities. Nationalism
in Bengal, increasingly closely linked with the
unrests in Punjab, was significant enough to
nearly paralyse the regional administration.

94.

Solution: b)

The British themselves adopted a "carrot and


stick" approach in recognition of India's
support during the war and in response to
renewed nationalist demands. In August 1917,
The means of achieving the proposed measure
were later enshrined in the Government of
India Act 1919, which introduced the principle
of a dual mode of administration, or diarchy, in
Page 28


INSIGHTS MOCK TEST SERIES 2015: TEST 32 SOLUTIONS

which both elected Indian legislators and


appointed British officials shared power. The
act also expanded the central and provincial
legislatures and widened the franchise
considerably.
Diarchy set in motion certain real changes at
the provincial level: a number of noncontroversial or "transferred" portfolios, such
as agriculture, local government, health,
education, and public works, were handed over
to Indians, while more sensitive matters such
as finance, taxation, and maintaining law and
order were retained by the provincial British
administrators.

He also served as leader of the Indian National


Congress, Premier of the Madras Presidency,
Governor of West Bengal, Minister for Home
Affairs of the Indian Union and Chief Minister
of Madras state. Rajagopalachari founded the
Swatantra Party and was one of the first
recipients of India's highest civilian award, the
Bharat Ratna.
He favoured talks with both Muhammad Ali
Jinnah and the Muslim League and proposed
what later came to be known as the C. R.
Formula.

97.
95.

Solution: a)

Gandhis concept of Satyagraha was


from this movement only.
It was originally founded by Bhagat Jawahar
Mal in 1840.His main aim was to purify the
Sikh religion by removing all the abuses,
superstitions and ill-practices from it.
But after the British annexation of Punjab
revival of Sikh power and sovereignty became
the major objective of the Kukas.
This caused a great deal of anxiety in the
British official ranks.
So they took various measures between 1863
and 1872 and were finally able to suppress this
movement.

96.

Solution: b)

Under the presidency of Jawaharlal Nehru at


its historic Lahore session in December 1929,
the Indian National Congress adopted a
resolution calling for complete independence
from the British. It authorised the Working
Committee to launch a civil disobedience
movement throughout the country. It was
decided that 26 January 1930 should be
observed all over India as the Purna Swaraj
(total independence) Day.
A special session was held in Karachi to
endorse the Gandhi-Irwin Pact. Karachi
session became famous because of the two
resolutions that were adopted-one on
Fundamental rights and other on National
Economic program, both drafted by Dr.
Rajendra Prasad. The goal of Purna Swaraj was
reiterated. This was the first time the congress
spelt out what swaraj would mean for the
masses.

Solution: d)

http://www.insightsonindia.com INSIGHTS

Page 29


INSIGHTS MOCK TEST SERIES 2015: TEST 32 SOLUTIONS

98.

Solution: c)

The revolutionary philosophies and movement


made their presence felt during the 1905
Partition of Bengal. Arguably, the initial steps
to organize the revolutionaries were taken by
Aurobindo Ghosh, his brother Barin Ghosh,
Bhupendranath Datta etc. when they formed
the Jugantar party in April 1906.
Jugantar party leaders like Barin Ghosh and
Bagha Jatin initiated making of explosives.
Amongst a number of notable events of
political terrorism were the Alipore bomb case,
the Muzaffarpur killing tried several activists
and many were sentenced to deportation for
life, while Khudiram Bose was hanged.

99.

Solution: d)

Towards the end of the successor of Nagabhata


II, successfully attacked Kannauj and
established control there. This was short-lived
as he was soon after defeated by the Rastrakuta
ruler, Govinda III. However, he was kept busy
in internal politics by an alliance of different
kingdoms in the south. The struggle for
Kannauj became serious after the Pratiharas
exercised control over it.
During the rule of Krishna III, there was
successful campaign against the Cholas. The
Rastrakutas also formed a matrimonial
relationship with the Gangas and defeated the
kingdom of Vengi. By the end of the 9th
Century the power of the Rastrakutas started to
decline along with the Palas.
This was seen as an ideal opportunity by the
feudal king Taila II who defeated the
http://www.insightsonindia.com INSIGHTS

Rastrakuta ruler and declared his kingdom


there. This came to be known the Later
Chalukya dynasty. Their kingdom included the
states of Karnataka, Konkan and northern
Godavari. By the end of the tripartite struggle,
the Pratiharas emerged victorious and
established themselves as the rulers of central
India.

100.

Solution: a)

Harsha died in 647, having ruled for 41 years.


His empire died with him, disintegrating
rapidly into small states. The succeeding period
is very obscure and badly documented, but it
marks the culmination of a process that had
begun with the invasion of the Huns in the last
years of the Gupta Empire.

In 648, Tang Taizong sent Wang Xuance to


India in response to Harsha sending an
ambassador to China. However once in India
he discovered Harsha had died and the new
king attacked Wang and his 30 mounted
subordinates.
This led to Wang Xuance escaping to Tibet and
then mounting a joint of over 7,000 Nepalese
mounted infantry and 1,200 Tibetan infantry
and attack on the Indian state on June 16. The
success of this attack Wang Xuance the
prestigious title of the "Grand Master for the
Closing Court." He also secured a reported
Buddhist relic for China

Page 30

INSIGHTS MOCK TEST - 33


INSIGHTS ON INDIA MOCK PRELIMINARY EXAM - 2015


INSIGHTS ON INDIA MOCK TEST - 33
GENERAL STUDIES

PAPER-I

Time Allowed: 2 Hours

Maximum Marks: 200

INSTRUCTIONS
1. IMMEDITELY AFTER THE COMMENCEMENT OF THE EXAMINATION, YOU SHOULD
CHECK THAT THIS TEST BOOKLET DOES NOT HAVE ANY UNPRINTED OR TORN OR MISSING
PAGES OR ITEMS, ETC. IF SO, GET IT REPLACED BY A COMPLETE TEST BOOKLET.
2. You have to enter your Roll Number on the Test
Booklet in the Box provided alongside. DO NOT
Write anything else on the Test Booklet.
4. This Test Booklet contains 100 items (questions). Each item is printed only in English. Each item
comprises four responses (answers). You will select the response which you want to mark on the
Answer Sheet. In case you feel that there is more than one correct response, mark the response which
you consider the best. In any case, choose ONLY ONE response for each item.
5. You have to mark all your responses ONLY on the separate Answer Sheet provided. See directions in
the Answer Sheet.
6. All items carry equal marks.
7. Before you proceed to mark in the Answer Sheet the response to various items in the Test Booklet, you
have to fill in some particulars in the Answer Sheet as per instructions sent to you with your
Admission Certificate.
8. After you have completed filling in all your responses on the Answer Sheet and the examination has
concluded, you should hand over to the Invigilator only the Answer Sheet. You are permitted to take
away with you the Test Booklet.
9. Sheets for rough work are appended in the Test Booklet at the end.
10. Penalty for wrong answers :
THERE WILL BE PENALTY FOR WRONG ANSWERS MARKED BY A CANDIDATE IN THE
OBJECTIVE TYPE QUESTION PAPERS.
(i)

There are four alternatives for the answer to every question. For each question for which a
wrong answer has been given by the candidate, one-third of the marks assigned to that
question will be deducted as penalty.

(ii) If a candidate gives more than one answer, it will be treated as a wrong answer even if one of
the given answers happens to be correct and there will be same penalty as above to that
question.
(iii)

If a question is left blank, i.e., no answer is given by the candidate, there will be no penalty
for that question.
http://www.insightsonindia.com

INSIGHTS ON INDIA MOCK TEST SERIES FOR CIVIL SERVICES PRELIMINARY EXAM 20

http://www.insightsonindia.com INSIGHTS

Page 1

INSIGHTS MOCK TEST - 33


1. Unnat Bharat Abhiyaan programme


relates to
a) improving the Indian urban
economic growth rate
b) skill development of workers in the
unorganized sector
c) establishment of centres of
excellence throughout the country
d) upliftment of rural India

2. Which of the following correctly


differentiates between Aesthenosphere
and Lithosphere?
1. Aesthenosphere is more solid than
Lithosphere.
2. The Mid-oceanic ridges occur in
Aesthenosphere and not lithosphere.
3. Lithosphere lies above
Aesthenosphere.
Choose the correct answer using the codes
below.
a)
b)
c)
d)

1 and 2 only
2 and 3 only
1 and 3 only
3 only

3. Diastrophism includes all those


processes that involve
1. Uplifting or warping of the large
parts of earths crust
2. Folding of earths crust
3. Generation of earths crust
Choose the correct answer using the codes
below.
http://www.insightsonindia.com INSIGHTS

a)
b)
c)
d)

1 and 2 only
2 and 3 only
1 and 3 only
None of the above

4. Who among the following can appear in


the proceedings of the Parliament but
without a right to vote?
1. President of India
2. CAG
3. Attorney General of India
4. Chairman of Finance Commission
Choose the correct answer using the codes
below.
a)
b)
c)
d)

1, 2 and 4 only
1 and 4 only
3 only
2 and 3 only

5. Nalayira Divyaprabandhanam is known


as
a) a major composition of the Alvar
saints
b) the rulebook of Lingayats guiding
their social and economic conduct
c) the most famous Tamil translation of
Kabirs work
d) the code of conduct of the
Vaishanavas in TamilNadu

6. Some laws made on general matters by


the Autonomous District Councils
established under Sixth Schedule of the
Constitution need the approval of
a) State Legislature
b) Parliament
Page 2

INSIGHTS MOCK TEST - 33


c) Governor of the State


d) Home Minister of the State

7. Which of the following statements


regarding the governance in Union
Territories is INCORRECT?
a) Lt. Governor cannot promulgate an
ordinance in the recess of the
assembly without the prior
permission of the President.
b) Parliament can enact laws for Union
Territories even where an elected
Assembly exists.
c) Governor of a state can be given
additional responsibility of a Union
territory, which he governs without
the aid and advice of the State
Council of Ministers.
d) None of the above

8. Consider the following about the


financial powers of the Governor of a
State.
1. No demand for a grant can be made
except on his recommendation.
2. Money bills can be introduced in the
state legislature only with his prior
recommendation.
3. He can make advances out of the
Contingency Fund of the state to
meet any unforeseen expenditure.
4. He sees that the Annual Financial
Statement (state budget) is laid
before the state legislature.

b) 1 and 2 only
c) 3 and 4 only
d) All of the above

9. To uphold the Sovereignty and integrity


of India is a part of the Oath of the
a) Judges of the Supreme Court
b) Members of the Parliament
c) President of India
d) Both (a) and (b)

10. The Committee on Petitions in the


Parliament pertains to
a) Privilege matters of the Members of
the Parliament
b) Entertaining complaints regarding
political defections
c) Entertaining representation from
public on matters relating to the
Union List
d) Enforcing the code of conduct of the
Members of the Parliament

11. Which of the following statements about


Cabinet Committees is INCORRECT?
a) Non-cabinet ministers are barred
from its membership.
b) They are setup by the Prime
Minister.
c) They are not mentioned in the
constitution.
d) There are both ad hoc and
permanent cabinet committees.

Choose the correct answer using the codes


below.
a) 1, 2 and 4 only
http://www.insightsonindia.com INSIGHTS

12. Consider the following statements.


Page 3

INSIGHTS MOCK TEST - 33


1. No ground has been mentioned in


the constitution for the removal of
the Vice President.
2. He can be elected for any number of
terms.
3. He can act as a President for a
maximum period of six months.
Choose the correct answer using the codes
below.
a)
b)
c)
d)

1 and 2 only
2 and 3 only
1 and 3 only
All of the above

13. The Constitution of Jammu and


Kashmir (J&K) declares
a) The state of J&K to be an integral
part of India.
b) The state of J&K to be a temporary
part of India.
c) The State of J&K to be an
autonomous state supported and
guarded by the Government of
India.
d) J&K to be a sovereign state in
alliance with the Government of
India.

14. Presidents Rule under Article 356 can


be imposed on a state if
1. The state government is pursing
anti-secular politics.
2. The State government has been
accused of corruption and
maladministration.

http://www.insightsonindia.com INSIGHTS

3. The State government disregards


constitutional directions given by the
Central government.
Choose the correct answer using the codes
below.
a)
b)
c)
d)

1 and 2 only
2 and 3 only
1 and 3 only
All of the above

15. The Zonal Councils have been


established by
a) The Constitution of India
b) States Reorganization Act 1956
c) A resolution of the Central Cabinet
d) None of the above

16. Full Faith and Credit Clause laid down


by the Constitution means that
a) The Council of Ministers is able to
govern because it carries the
credibility of a universal adult
franchise.
b) All records and acts of the
Government of India or the states are
to given full faith and credit
throughout the territory of India.
c) Lok Sabha passes the Vote on
Account motion based on the
credibility of the executive even
before the annual budget is
examined thoroughly.
d) None of the above

Page 4

INSIGHTS MOCK TEST - 33


17. The Ajivika sect believes that


a) Everything is pre-determined.
b) The world is a random chaos.
c) The world is a play of four elements
and temporary.
d) Human will is supreme.

18. As per the teachings of Mahavira Jain


1. Monastic existence is necessary for
salvation.
2. Salvation cannot be attained by
women and non-brahmin castes.
3. Observing celibacy is a very
important condition for attaining
salvation.
Choose the correct answer using the codes
below.
a)
b)
c)
d)

1 and 2 only
2 and 3 only
1 and 3 only
All of the above

19. Khanqahs are related to the


a) places where Mughal rulers met their
subjects for grievance redressal
b) places of rest where Sufi mystics met
and discussed spiritual matters with
the visitors
c) art galleries where artists performed
for the general public in Mughal
courts
d) a style of decorative wall embroidery
found mostly on the centre of arches
erected by Mughal rulers

http://www.insightsonindia.com INSIGHTS

20. Earliest sites found to the East or South


of the Indus river system include
1. Kalibangan
2. Lothal
3. Dholavira
Choose the correct answer using the codes
below.
a)
b)
c)
d)

1 and 2 only
2 and 3 only
1 and 3 only
All of the above

21. Instead of spreading paddy seeds on the


field, saplings are sown. Why?
a) It increases survival chances of the
crop.
b) It reduces attack from pests.
c) It allows more use of fertilizers in the
field.
d) If seeds are thrown directly on the
ground, roots may not penetrate
deep.

22. Under Gana or Sangha system of


governance during the period of the
Magadha Kingdom
a) People ruled through their directly
elected representatives
b) Several Rajas ruled together and
took collective decisions.
c) Only one king governed a very large
area.
d) Village assemblies were selfgoverned by popular representation

Page 5

INSIGHTS MOCK TEST - 33


23. As per Buddhist teachings constant


craving could be removed by
a) Renouncing everything
b) Joining the Buddhist sangha
c) Becoming a celibate and doing
penance
d) Following moderation in everything

24. Arthashashtra by Kautilya was written


during the rule of
a) Chandragupta Maurya
b) Bindusara
c) Ashoka
d) Harshavardhana

25. According to Swami Dayanand


Saraswati
1. World is an illusion and one must get
rid of it.
2. Salvation can be attained by
performing the right rituals with the
learned and pious.
3. Worshipping idols is one way of
attaining to the formless ultimate
reality.
Choose the correct answer using the codes
below.
a)
b)
c)
d)

1 and 2 only
2 and 3 only
1 and 3 only
None of the above

26. Consider the following about the Oudh


Kisan Sabha setup in 1920.
1. It was established by Jawaharlal
Nehru.
http://www.insightsonindia.com INSIGHTS

2. It was setup to address the subdued


participation from Awadh in mass
movements.
3. The Sabha adopted strictly
constitutional and legal methods for
achieving its objectives.
Choose the correct answer using the codes
below.
a)
b)
c)
d)

1 and 2 only
2 and 3 only
1 and 3 only
1 only

27. Which of the following statements is


INCORRECT concerning the Civil
Disobedience Movement?
a) Dalit participation in Civil
Disobedience movement was limited.
b) It was launched in response to
Montague statement that did not
satisfy Indian leaders.
c) Gandhi-Irwin pact came amidst the
movement.
d) None of the above

28. Kheda Satyagraha was organized by


Gandhiji in response to
a) British forcing the Kheda peasants to
grow only particular crops on their
farms
b) Exploitative land ownership
structure in rural areas that resulted
in farmer suicides
c) Lack of security of tenure for Kheda
peasants
d) British authorities refusing to grant
remission of revenue for farmers that
suffered crop losses
Page 6

INSIGHTS MOCK TEST - 33


3. Kal
29. The aim of the Home Rule League
Movement launched by Tilak was
a) Demanding purna swaraj from the
British by using both constitutional
and extra-constitutional methods
b) Overthrowing the British
government with support from
Burma
c) Gradual reform of the administration
to introduce self-government by
Indians
d) Organizing mass movements to in
India and abroad to expose the
exploitative character of the British
rule

30. Which of the following statements with


regard to the Zimmerman Plan is
correct?
1. It was formed in Europe.
2. It aimed to mobilize Indian settlers
abroad to help invade British India.
3. It was behind organizing the RIN
mutiny in India.
Choose the correct answer using the codes
below.
a)
b)
c)
d)

1 and 2 only
2 and 3 only
1 and 3 only
All of the above

31. Which of the following newspapers and


journals advocated revolutionary
terrorism in British India?
1. Yugantar
2. Sandhya
http://www.insightsonindia.com INSIGHTS

Choose the correct answer using the codes


below.
a)
b)
c)
d)

1 and 2 only
2 and 3 only
1 and 3 only
All of the above

32. Bande Matram, for the first time, was


the theme song of which movement?
a) Non-cooperation Movement
b) Rowlatt Satyagraha
c) Swadeshi Movement
d) Civil Disobedience Movement

33. Consider the following about the Poona


Sarvajanik Sabha.
1. It was founded by Dadabhai Naoroji.
2. Its objective was to serve as a bridge
between the government and people.
3. It was the first organization to
propose the concept of Swaraj for
India.
Choose the correct answer using the codes
below.
a)
b)
c)
d)

1 and 2 only
2 and 3 only
2 only
1 and 3 only

34. Moral policing with reference to the


Government may refer to
a) State intruding in the cultural liberty
of the individual
Page 7

INSIGHTS MOCK TEST - 33


b) State trying to regulate any area of a


citizens political or social life
c) State using excessive use of police
force even for small illegal activities
d) State mandating moral education to
be compulsory in schools and
colleges

35. 10th August was celebrated recently as


a) World Ecology Day
b) World Bio-Fuel Day
c) Global Commons Day
d) World Conservation Day

36. Consider the following statements about


the recently launched scheme Nayi
manzil.
1. It intends to cover people in 17-35
age groups.
2. It will cover all minority
communities.
3. It will intervene in education and
skill development for improving the
welfare of minorities.
Choose the correct answer using the codes
below.
a)
b)
c)
d)

1 and 2 only
2 and 3 only
2 only
All of the above

37. ISRO has recently started commercial


production at the first indigenous
Titanium Sponge Plant. Titanium
sponge can be useful in applicable in
aerospace and defence areas due to its
http://www.insightsonindia.com INSIGHTS

1. High strength to weight ratio


2. Excellent corrosion resistance
3. Use in Liquid propellant tanks
Choose the correct answer using the codes
below.
a)
b)
c)
d)

1 and 2 only
2 and 3 only
2 only
All of the above

38. A new mobile app Disha has been


launched by the Union Telecom
Minister recently. The app is concerned
with
a) Promoting awareness of government
schemes in rural areas
b) Tracking individuals who are joining
Naxalite cadres and help bring them
back
c) Aiding digital literacy in the State of
Bihar
d) None of the above

39. Aadhar enrolment can be done for


1. Non-resident Indians (NRIs)
2. Overseas Indian Citizens
3. Persons of Indian Origin (PIOs)
Choose the correct answer using the codes
below.
a)
b)
c)
d)

1 and 2 only
2 and 3 only
2 only
All of the above

Page 8

INSIGHTS MOCK TEST - 33


40. Consider the following about Lakshya


developed by DRDO.
1. It is a pilotless target aircraft.
2. Its flight path can be preprogrammed based upon the type of
mission.
3. It is equipped with cruise missile.
Choose the correct answer using the codes
below.
a)
b)
c)
d)

1 and 2 only
2 and 3 only
2 only
All of the above

41. Suez Canal links the


a) Red Sea with Caspian Sea
b) Arabian Sea with Caspian Sea
c) Mediterranean Sea with Red Sea
d) Caspian Sea with Arabian Sea

42. 193 member states of United Nations


have reached an agreement on the
outcome document New Sustainable
Development Agenda for the next 15
years. The new agenda draft document
is called
a) Towards New Development: The
2030 Agenda for Sustainable
Development
b) A Brave New World: The 2030
Agenda for Sustainable Development
c) Transforming our World: The 2030
Agenda for Sustainable Development
d) People, Planet and Peace: The 2030
Agenda for Sustainable Development

http://www.insightsonindia.com INSIGHTS

43. Maitri and Dakshin Gangotri are


a) Indian research stations in
Antarctica
b) Indo-Chinese research base in upper
Himlayas near Siachin
c) Indo-Russian observatory in Arctic
to track climate change
d) Indo-Denmark research observatory
Arctic Ocean

44. Which of these nations do NOT have any


known mineral deposit?
a) Uganda
b) Switzerland
c) Georgia
d) None of the above

45. Which of these crops/plants is/are NOT


native to India and have been brought
here from abroad for cultivation?
1. Lotus
2. Jasmine
3. Mango
Choose the correct answer using the codes
below.
a)
b)
c)
d)

1 and 2 only
2 and 3 only
3 only
All are native to India.

46. Consider the following about Deccan


traps.
1. They are formed from lava flows.
2. They are a flood basalt province.
3. They cover most of Northern
plateaus of India.
Page 9

INSIGHTS MOCK TEST - 33


Choose the correct answer using the codes


below.
a)
b)
c)
d)

1 and 2 only
2 and 3 only
1 and 3 only
All the above

47. Earth is not a perfect sphere; it has a


bulge at the equator. This bulge is due to
a) The revolution of earth around the
Sun
b) The rotation of Earth on its axis
c) Dynamic action of Primordial heat of
earth
d) Lower gravitation force at equator as
compared to the poles

48. Which one of the following is NOT an


evidence to prove Sea floor spreading?
a) The age of rocks from the ocean floor
b) Synchronic reversing of magnetic
field in rocks of ocean floor.
c) The way fossils are distributed across
different continents.
d) Volcanic activity along the midoceanic ridges

49. Lithification is a process by which


a) Aesthenosphere has been formed
b) Lithosphere has been formed
c) Sedimentary rocks are formed
d) Igneous rocks are formed

50. Depth of weathering mantle is more in


tropical forests than in Tundra regions.
The factors contributing to this are
1. Higher moisture and water
availability
2. Higher temperature
3. More active biological weathering
Choose the correct answer using the codes
below.
a)
b)
c)
d)

1 and 2 only
2 and 3 only
1 and 3 only
All the above

51. In a meandering large river, there is


active deposition over convex banks
whereas undercutting dominates over
the concave bank. This is because of
a) Difference in the rush velocity of
water on the convex and concave side
of the bank
b) Formation of ox-bow lakes in the
course of meandering
c) Change in Coriolis force as the river
meanders
d) Low velocity of the river

52. Troposphere is responsible for which of


the following?
1. Temperature variation on earth
2. Pressure variation on surface of
earth
3. Humidity, clouds and precipitation
Choose the correct answer using the codes
below.
a) 1 and 2 only

http://www.insightsonindia.com INSIGHTS

Page 10

INSIGHTS MOCK TEST - 33


b) 2 and 3 only
c) 1 and 3 only
d) All of the above

53. Consider the following statements.


Assertion (A): Generally at the same latitude
rainfall is more over the oceans than on the
continents.
Reason (R): Generally at the same latitude
solar insolation received over the oceans is
more than that on continents.
In the context of the statements above, which
of these is true?
a) A and R both are true, and R is the
correct explanation for A.
b) A and R both are true, and R is the NOT
the correct explanation for A.
c) A is correct, R is incorrect.
d) A is incorrect, R is correct

54. The conditions that support dew


formation are
1. High speed winds in the night
2. Clear sky
3. High relative humidity
4. Cold nights
Choose the correct answer using the codes
below.
a)
b)
c)
d)

1 and 2 only
2 and 4 only
1 and 3 only
2, 3 and 4 only

55. The Central parts of tropical land


generally receive lesser rainfall than the
coastal parts because
a) Westerlies are present at low lying
coastal areas in tropical areas
b) Moisture carried by winds reduces as
they travel inward in a continent
c) Central parts receive higher solar
insolation than the coastal areas
d) Jet streams cannot reach inner
continental areas

56. The Mediterranean climate is


characterised by hot, dry summer and
mild, rainy winter due to the influence
of
1. Influence of sub-tropical high in
summer
2. Westerly wind in winter
Which of the above is/are correct?
a)
b)
c)
d)

1 only
2 only
Both 1 and 2
None

57. Surface salinity of ocean water is


influenced in
1. Coastal regions by the fresh water
flow
2. Polar regions by the processes of
freezing and thawing of ice
3. Movement of ocean currents
Choose the correct answer using the codes
below.
a) 1 and 2 only
b) 2 and 3 only

http://www.insightsonindia.com INSIGHTS

Page 11

INSIGHTS MOCK TEST - 33


c) 1 and 3 only
d) All of the above

58. Apart from Winds, movement of ocean


water is influenced by
1. Temperature differences
2. Density differences
3. External force of Sun
4. External force of Moon
Choose the correct answer using the codes
below.
a)
b)
c)
d)

1 and 4 only
2 and 3 only
1, 3 and 4 only
All of the above

59. Zenkeria Sebastinei is


a) a critically endangered grass in
Agasthiyamalai peak
b) a newly found frog species in
Western Ghats
c) an antidote to cancer found in
Caribbean islands discovered
recently
d) an endemic grass species of Sri
Lanka that has been introduced in
India to improve pastoral lands

60. Which of the following continents does


NOT have any Mega diversity centres
(ecological hotspots as characterized by
IUCN)?
a) Europe
b) North America
c) Australia
d) Africa
http://www.insightsonindia.com INSIGHTS

61. Which of the following factors affect the


recharge or availability of potable
groundwater?
1. Vegetation cover
2. Diversion of water for high water
demanding crops
3. Pollution from industrial effluents
Choose the correct answer using the codes
below.
a)
b)
c)
d)

1 and 2 only
2 and 3 only
1 and 3 only
All of the above

62. The Chipko Andolan (Hug the Trees


movement) was concerned with
a) Local communities losing control in
official management of forest
resources
b) Governments legitimate right to cut
trees in a hilly area
c) Global warming caused due to
deforestation
d) Stopping alienation of people from
their forests

63. Coliform group of bacteria, found in


human intestine, if also present in water
indicates
a) Contamination of water by diseasecausing organisms
b) Cultutal eutrophication of water
c) High pH of water that is unsuitable
for drinking
d) Presence of radiation in water
Page 12

INSIGHTS MOCK TEST - 33


64. What kind of Mirror would be best


suited for use in a solar cooker?
a) Concave mirror
b) Convex mirror
c) Plain mirror
d) A combination of convex and plain
mirror

65. Electricity generation from Ocean


thermal energy is based on the
a) Heat generated from volcanic
eruptions at the bottom of the ocean
b) Hot springs found in the ocean
c) Difference in the temperature of high
and low tides
d) Temperature differential between
surface and Deep Ocean

66. Which of the following gases are


generated in the decomposition process
of slurry in a bio-gas plant?
1. Methane
2. Carbon dioxide
3. Hydrogen
4. Hydrogen Sulphide
Choose the correct answer using the codes
below.
a)
b)
c)
d)

1, 2 and 3 only
2 and 4 only
1 and 3 only
All the above

67. Consider the following statements.

http://www.insightsonindia.com INSIGHTS

Assertion (A): Many thermal power plants are


set up near coal or oil fields.
Reason (R): The transmission of electricity is
more efficient than transporting coal or
petroleum over the same distance.
In the context of the statements above, which
of these is true?
a) A and R both are true, and R is a
possible correct explanation for A.
b) A and R both are true, and R can
NOT be a correct explanation for A.
c) A is correct, R is incorrect.
d) A is incorrect, R is correct

68. Most power stations constructed these


days produce Alternating Current (AC)
instead of Direct Current (DC). What is
the advantage of producing AC over DC?
1. In AC power can be transmitted
across long distances without much
loss of energy.
2. DC power generation is much
costlier than AC generation.
Which of the above is/are true?
a)
b)
c)
d)

1 only
2 only
Both 1 and 2
None

69. Which of the two organs of human body


produce a significant magnetic field?
1. Heart
2. Brain
3. Lungs
4. Legs
Page 13

INSIGHTS MOCK TEST - 33


Choose the correct answer using the codes


below.
a)
b)
c)
d)

1 and 3
2 and 4
1 and 2
3 and 4

70. Sky appears dark to passengers flying at


very high altitudes because
a) Scattering of light is not prominent
at such heights
b) Clouds are absent at such heights
c) The absence of polluting particles at
such heights
d) The low atmospheric pressure
outside the aeroplane
71. In the formation of rainbow, which of
the following phenomenon occurs?
1. Refraction of light
2. Dispersion of light
3. Internal reflection of light
Choose the correct answer using the codes
below.
a)
b)
c)
d)

1 and 2 only
2 and 3 only
1 and 3 only
All of the above

72. When growing plants detect light, a


hormone X, synthesises at the shoot
tip, helps the cells to grow longer in that
direction. The hormone X is
a) Cytokinin
b) Auxin
c) Melanin
d) Abscicic acid
http://www.insightsonindia.com INSIGHTS

73. Rate of breathing in aquatic organisms


is much faster than that seen in
terrestrial organisms. This may be
because
a) Water is a denser medium than air.
b) Aquatic organisms lie at a lower trophic
level than terrestrial organisms.
c) Amount of oxygen dissolved in water is
fairly low as compared to that in air.
d) Aquatic organisms are not biologically
as evolved as the terrestrial organisms.

74. If you observe the bottom of a cooking


vessels getting blackened on a gas stove,
it means that
a) The gas fuel contains very high
amounts of carbon
b) The gas fuel contains lead
c) The air holes of the stove are blocked
and the fuel is getting wasted
d) The gas fuel contains blue carbon

75. Consider the following statements.


Assertion (A): Carbon plays a very important
role in sustaining life on earth.
Reason (R): Carbon is the most abundant
element in the earths crust.
In the context of the statements above, which
of these is true?
a) A and R both are true, and R is a
possible correct explanation for A.
b) A and R both are true, and R can
NOT be a correct explanation for A.
c) A is correct, R is incorrect.
Page 14

INSIGHTS MOCK TEST - 33


d) A is incorrect, R is correct

76. Under the River boards Act, a River


board is established
a) by Central government on the
request of the state governments
concerned to advise them
b) by Supreme Court on a petition by
the state governments concerned to
advise them
c) by the Parliament based on the
exigencies of the situation
d) by the State governments jointly
after seeking permission from the
President

77. Apart from Central government, the


Comptroller and Auditor general of
India presently is authorized to audit the
accounts of
1. State governments
2. Local governments
3. National political parties
Choose the correct answer using the codes
below.
a)
b)
c)
d)

1 and 2 only
2 and 3 only
1 and 3 only
1 only

78. The Preamble to the Indian constitution


is based on
a) Nehru Report 1928
b) Objectives Resolution drafted by
Pandit Nehru
c) Government of India Act, 1935
http://www.insightsonindia.com INSIGHTS

d) The Constitution of the Indian


National Congress in British India

79. Balance of Payments deficit can be


reduced through
1. Export promotion
2. Import control
3. Increasing foreign exchange reserves
4. Devaluation of currency
Choose the correct answer using the codes
below.
a)
b)
c)
d)

1 and 2 only
1, 2 and 3 only
1, 2 and 4 only
3 and 4 only

80.Fiscal policy of the government is


related with
1. Taxation
2. Government spending
3. Government borrowing
4. Public debt
Choose the correct answer using the codes
below.
a)
b)
c)
d)

1 and 2 only
1, 2 and 3 only
3 and 4 only
All of the above

81. As per Keynes, speculative demand for


money arises due to the
a) Predictions in the market about
future bond interest rates
b) Needs for transaction in a market
economy
Page 15

INSIGHTS MOCK TEST - 33


c) Uncertainty of a household
expenditure
d) The fluctuations in the external
sector of the economy

82. Marginal productivity of labour in


agricultural sector of India is zero. This
points to
a) Disguised unemployment in Indian
agriculture
b) Potential for employment in Indian
agriculture
c) Unemployment in Indian agriculture
d) Growing labour demand in the
agricultural sector

83. Liquidity trap is a situation where


a) All potential investors expect the
interest rates to move up in future
b) Interest rates in the market fluctuate
so much that bonds become nonliquid
c) People put all their money in buying
bonds
d) All of the above

84. Since the 1991 economic reforms, Indian


agriculture has been experiencing
slowdown in growth rates. This may be
because of
1. Stagnation in green revolution
technology
2. Falling public investment in
agriculture
3. Low market price of agricultural
produce

http://www.insightsonindia.com INSIGHTS

Choose the correct answer using the codes


below.
a)
b)
c)
d)

1 and 2 only
2 and 3 only
1 and 3 only
All of the above

85. The main objectives of food


management in India is/are
1. Maintenance of buffer stock for
ensuring food security
2. Procurement of food grains from
farmers at remunerative prices
3. Distribution of food grains at
affordable prices
Choose the correct answer using the codes
below.
a)
b)
c)
d)

1 and 2 only
2 and 3 only
1 and 3 only
All of the above

86. Which of these methods can be used in


India to solve Industrial disputes?
1. Decision by labor Unions
2. Collective bargaining
3. Workers participation in
management
Choose the correct answer using the codes
below.
a)
b)
c)
d)

1 and 2 only
2 and 3 only
1 and 3 only
All of the above

Page 16

INSIGHTS MOCK TEST - 33


87. Securities and Exchange Board of India


(SEBI) regulates
1. Money market
2. Capital market
3. Commodity market
4. Futures market
Choose the correct answer using the codes
below.
a)
b)
c)
d)

1 and 2 only
2 and 4 only
1 only
1, 2 and 3 only

88. Price rigging in stock market refers to


a) Insider trading of stocks in securities
markets
b) Manipulation of securities prices so
as to inflate or depress their market
price
c) Speculative and high frequency
trading
d) None of the above
89. Which of the following banking services
are provided to the government by RBI?
1. Maintaining and operating of deposit
accounts of the Central and State
Governments
2. Receipts and collection of payments
to the Central and State
Governments.
3. Making payments on behalf of the
Central and State Governments.
4. Transfer of funds and remittance
facilities to the Central and State
Governments
http://www.insightsonindia.com INSIGHTS

Choose the correct answer using the codes


below.
a)
b)
c)
d)

1 and 2 only
2 and 4 only
1 and 3 only
All of the above

90. Inflation in the country can be managed


by
1. Fiscal policy
2. Monetary policy
3. Supply management through
imports
Choose the correct answer using the codes
below.
a)
b)
c)
d)

1 and 2 only
2 and 3 only
1 and 3 only
All of the above

91. The practice of purchasing a currency in


one market and selling it in another
market is called as
a) Spot trading
b) Futures Trading
c) Arbitrage
d) Speculative trading

92. The free radical theory of aging (FRTA)


states that organisms age because cells
accumulate free radical damage over
time. Free radicals can be tackled by
1. Taking food rich in antioxidants
2. Taking high calories food to remain
energetic throughout the day
Page 17

INSIGHTS MOCK TEST - 33


3. Reducing the intake of watery and


fibrous food
Choose the correct answer using the codes
below.
a)
b)
c)
d)

1 and 2 only
2 and 3 only
1 only
1 and 3 only

93. When a transgene is introduced into the


genome of an organism it can
1. Prevent the expression of an existing
native gene
2. Enhance the expression of a native
gene
Which of the above is/are correct?
a)
b)
c)
d)

1 only
2 only
Both 1 and 2
None

94. Bio-pesticides can be used for the


control of
1. Weeds
2. Insects
3. Pathogens
Choose the correct answer using the codes
below.
a)
b)
c)
d)

1 and 2 only
2 and 3 only
1 and 3 only
All of the above

a) Using different genetic material to


preserve different traits of organisms
b) Changing culture vessels containing
fresh media from time to time to
preserve tissues
c) Growing genetically modified plants
in laboratory
d) None of the above

96. When a plant is introduced from an


outside country Quarantine is
necessary because
1. There is a risk of entry of disease in
the country via plants.
2. Exotic species may come with the
plants that can disrupt ecosystems.
Which of the above is/are correct?
a)
b)
c)
d)

1 only
2 only
Both 1 and 2
None

97. Viruses can spread from host plant to


other plants through
1. Seeds
2. Pollens
3. Insect vectors
4. Fungi
Choose the correct answer using the codes
below.
a)
b)
c)
d)

1 and 2 only
2 and 3 only
1 and 3 only
All of the above

95. Sub-culturing is a process of


http://www.insightsonindia.com INSIGHTS

Page 18

INSIGHTS MOCK TEST - 33


98. Ozone layer is thicker at poles as


compared to tropics. A possible reason
is
a) Lower solar insolation received at
poles to break ozone down
b) Lower breakdown of ozone layer due
to free radicals
c) Greater albedo coming from the ice
sheets at poles
d) None of the above
99. Sea-level rise is projected to have
negative impacts on
1. Human settlements
2. Tourism
3. Freshwater supplies
4. Fisheries
5. Wetlands
Choose the correct answer using the codes
below.
a)
b)
c)
d)

1, 2, 3 and 4 only
2 and 4 only
1, 3 and 5 only
All of the above

100.
Which of the following can help
reduce global warming and its effects?
1. Reducing the use of nitrogenous
fertilizers in agriculture
2. Increasing the vegetation cover
3. Reducing urban heat islands
Choose the correct answer using the codes
below.
a)
b)
c)
d)

1 and 2 only
2 and 3 only
1 and 3 only
All of the above

http://www.insightsonindia.com INSIGHTS

Page 19

INSIGHTS MOCK TEST SERIES 2015: TEST 33 SOLUTIONS


1. Solution: d)

Unnat Bharat Abhiyan is a Ministry of Human Resource Development, Government of India


programme to uplift rural India. The programme is being launched in collaboration with the
Indian Institutes of Technology (IITs), National Institutes of Technology (NITs) and other leading
Government Engineering Institutes like College of Engineering, Pune across the country.
Unnat Bharat Abhiyan is being coordinated and steered by IIT Delhi. The programme involve
engaging with neighbouring communities and using technologies for their upliftment. The
programme is currently being planned and is yet to be launched.

2. Solution: d)
The layer of earth that is just below Lithosphere and goes deeper inside the surface is known as
Asthenosphere. The balance between pressure and temperature is such that the rocks in this layer
of the earth have little strength, and they behave like butter under a knife. This is the part of the
mantle that slows down seismic waves as it consists of molten rocks
If we consider the entire mantle as whole, Asthenosphere comprises just over 6% in volume, but it
is very important in tectonic plate movement as because of the liquidity of this layer, the overlying
layer called Lithosphere is able to move.
What is the difference between Lithosphere and Asthenosphere?
The difference between lithosphere and Asthenosphere pertain to their compositions.
While lithosphere is hard and rigid, Asthenosphere is a layer made up of molten rocks.
Lithosphere stretches from the top of the earths crust down to first 100 kilometers while
Asthenosphere lies beneath lithosphere
Rocks are under severe pressure in Asthenosphere, whereas they face much less pressure in
lithosphere.
The mineral composition of lithosphere is varied as it contains more than 80 minerals whereas
Asthenosphere is mainly comprised of silicates of iron and magnesium.
The depth of the lithosphere is about 100 kilometers, whereas the depth of Asthenosphere is 400700 kilometers.

3. Solution: a)
Diastrophism refers to deformation of the Earth's crust, and more especially to folding and
faulting. Diastrophism can be considered part of geo-tectonics. Diastrophism comes from the
Greek word meaning a twisting.

http://www.insightsonindia.com INSIGHTS

Page 1

INSIGHTS MOCK TEST SERIES 2015: TEST 33 SOLUTIONS


All processes that move, elevate or build up portions of the earths crust come under diastrophism.
They include: (i) orogenic processes involving mountain building through severe folding and
affecting long and narrow belts of the earths crust; (ii) epeirogenic processes involving uplift or
warping of large parts of the earths crust; (iii) earthquakes involving local relatively minor
movements; (iv) plate tectonics involving horizontal movements of crustal plates.

4. Solution: c)
President is a part of the Parliament. But he only addresses the Parliament (does not take part in
the proceedings).
Attorney general can do so. He also enjoys all the privileges and immunities available to the MPs.
CAG only guides the PAC in ensuring the accountability of the executive to the Parliament.

5. Solution: a)
The importance of the traditions of the Alvars and Nayanars was sometimes indicated by the claim
that their compositions were as important as the Vedas.
For instance, one of the major anthologies of compositions by the Alvars, the Nalayira
Divyaprabandham, was frequently described as the Tamil Veda, thus claiming that the text was as
significant as the four Vedas in Sanskrit that were cherished by the Brahmanas.

6. Solution: c)
The Acts of Parliament do not apply fully to these areas. Governor specifies to what extent these
laws are applicable to the 6th schedule areas.
Laws made on matters like land, forests etc. (except on customary or religious matters) require the
assent of the Governor.

7. Solution: d)
The Parliament can make laws on any subject of the three lists (including the State List) for the
union territories. This power of Parliament also extends to Puducherry and Delhi, which have their
own local legislatures.
The Lt. governor is empowered to promulgate ordinances during recess of the assembly. An
ordinance has the same force as an act of the assembly. Every such ordinance must be approved by
the assembly within six weeks from its reassembly. He can also withdraw an ordinance at any time.
http://www.insightsonindia.com INSIGHTS

Page 2

INSIGHTS MOCK TEST SERIES 2015: TEST 33 SOLUTIONS


But, he cannot promulgate an ordinance when the assembly is dissolved or suspended. Further, no
such ordinance can be promulgated or withdrawn without the prior permission of the President.

8. Solution: d)
He constitutes a finance commission after every five years to review the financial position of the
panchayats and the municipalities.
He also lays the reports of the State Finance Commission, the State Public Service Commission
and the Comptroller and Auditor-General relating to the accounts of the state, before the state
legislature.

9. Solution: d)
In his oath, a judge of the Supreme Court swears:

to bear true faith and allegiance to the Constitution of India;


to uphold the sovereignty and integrity of India;
to duly and faithfully and to the best of his ability, knowledge and judgement perform the
duties of the Office without fear or favour, affection or ill-will; and
to uphold the Constitution and the laws.

First three are the same as that of an MP. Fourth one is peculiar to judges.
The President swears:

to faithfully execute the office;


to preserve, protect and defend the Constitution and the law; and
to devote himself to the service and well-being of the people of India.

10.

Solution: c)

Committee on Petitions examines petitions on bills and on matters of general public importance. It
also entertains representations from individuals and associations on matters pertaining to Union
subjects.
The Lok Sabha committee consists of 15 members, while the Rajya Sabha committee consists of 10
members.

11. Solution: a)
They are extra-constitutional in emergence. In other words, they are not mentioned in the
Constitution. However, the Rules of Business provide for their establishment.
http://www.insightsonindia.com INSIGHTS

Page 3

INSIGHTS MOCK TEST SERIES 2015: TEST 33 SOLUTIONS


They are set up by the Prime Minister according to the exigencies of the time and requirements of
the situation. Hence, their number, nomenclature, and composition varies from time to time.
Their membership varies from three to eight. They usually include only Cabinet Ministers.
However, the non-cabinet Ministers are not debarred from their membership.

12. Solution: d)
The Vice-President holds office for a term of five years from the date on which he enters upon his
office. However, he can resign from his office at any time by addressing the resignation letter to the
President. He can also be removed from the office before completion of his term.
A formal impeachment is not required for his removal. He can be removed by a resolution of the
Rajya Sabha passed by an absolute majority (i.e., a majority of the total members of the House)
and agreed to by the Lok Sabha. But, no such resolution can be moved unless at least 14 days
advance notice has been given. Notably, no ground has been mentioned in the Constitution for his
removal.
The Vice-President can hold office beyond his term of five years until his successor assumes
charge. He is also eligible for re-election to that office. He may be elected for any number of terms.

13. Solution: a)
The Constitution of J&K was adopted on 17 November 1957, and came into force on 26 January
1957. Its salient features (as amended from time to time) are as follows:

It declares the State of J&K to be an integral part of India.


It secures justice, liberty, equality and fraternity to the people of the state.
It says that the State of J&K comprises all the territory that was under the ruler of the state
on 15 August 1947. This means that the territory of the state also includes the area which is
under the occupation of Pakistan.

14. Solution: c)
As per Bommai case (1994), secularism is one of the basic features of the Constitution. Hence, a
state government pursuing anti-secular politics is liable to action under Article 356.
However, it would be improper to impose in cases of

Maladministration in the state or allegations of corruption against the ministry or stringent


financial exigencies of the state.
Where the state government is not given prior warning to rectify itself except in case of
extreme urgency leading to disastrous consequences.

http://www.insightsonindia.com INSIGHTS

Page 4

INSIGHTS MOCK TEST SERIES 2015: TEST 33 SOLUTIONS


15. Solution: b)
The Zonal Councils are the statutory (and not the constitutional) bodies. They are established by
an Act of the Parliament, that is, States Reorganisation Act of 1956.
The act divided the country into five zones (Northern, Central, Eastern, Western and Southern)
and provided a zonal council for each zone.
Each zonal council consists of the following members: (a) home minister of Central government.
(b) chief ministers of all the States in the zone. (c) Two other ministers from each state in the zone.
(d) Administrator of each union territory in the zone.

16. Solution: b)
Under the Constitution, the jurisdiction of each state is confined to its own territory. Hence, it is
possible that the acts and records of one state may not be recognised in another state. To remove
any such difficulty, the Constitution contains the Full Faith and Credit clause.
Full faith and credit is to be given throughout the territory of India to public acts, records and
judicial proceedings of the Centre and every state. The expression public acts includes both
legislative and executive acts of the government. The expression public record includes any
official book, register or record made by a public servant in the discharge of his official duties.

17. Solution: a)
Ajivjkas have often been described as fatalists: those who believe that everything is predetermined.
The other school belonging to the tradition of the Lokayatas are usually described as materialists.
They believe the world to be a play of elements which will all pass on the earth and space when one
dies.
Texts from these traditions have not survived, so we know about them only from the works of
other traditions.

18.Solution: c)
Asceticism and penance are required to free oneself from the cycle of karma.
This can be achieved only by renouncing the world; therefore, monastic existence is a necessary
condition of salvation.
Jaina monks and nuns (women) took five vows: to abstain from killing, stealing and lying; to
observe celibacy; and to abstain from possessing property.
http://www.insightsonindia.com INSIGHTS

Page 5

INSIGHTS MOCK TEST SERIES 2015: TEST 33 SOLUTIONS


19. Solution: b)
The khanqah was the centre of social life in Sufism.
We know about Shaikh Nizamuddins hospice (c. Fourteenth century) on the banks of the river
Yamuna in Ghiyaspur, on the outskirts of what was then the city of Delhi.
It comprised several small rooms and a big hall ( jamaat khana) where the inmates and visitors
lived and prayed. The inmates included family members of the Shaikh, his attendants and
disciples.

20.

Solution: d)

21. Solution: a)
People began transplanting paddy since ages. This meant that instead of scattering seed on the
ground, from which plants would sprout, saplings were grown and then planted in the fields.
This led to increased production, as many more plants survived.
However, this also required a large amount of labour and resources.

22.

Solution: b)

http://www.insightsonindia.com INSIGHTS

Page 6

INSIGHTS MOCK TEST SERIES 2015: TEST 33 SOLUTIONS


While Magadha became a powerful kingdom, places like Vajji, with its capital at Vaishali (Bihar),
was under a different form of government, known as gana or sangha.
In a gana or a sangha there were not one, but many rulers. Sometimes, even when thousands of
men ruled together, each one was known as a raja. These rajas performed rituals together. They
also met in assemblies, and decided what had to be done and how, through discussion and debate.
For example, if they were attacked by an enemy, they met to discuss what should be done to meet
the threat. However, women, dasas and kammakaras could not participate in these assemblies.
Both the Buddha and Mahavira belonged to ganas or sanghas.

23.

Solution: a)

The Buddha taught that life is full of suffering and unhappiness. This is caused because we have
cravings and desires (which often cannot be fulfilled).
Sometimes, even if we get what we want, we are not satisfied, and want even more (or want other
things).
The Buddha described this as thirst or tanha. He taught that this constant craving could be
removed by following moderation in everything.

24.

Solution: a)

The empire that Ashoka ruled was founded by his grandfather, Chandragupta Maurya, more than
2300 years ago.
Chandragupta was supported by a wise man named Chanakya or Kautilya. Many of Chanakyas
ideas were written down in a book called the Arthashastra.
The treatise contains details on administration, kingdom economy, politics, ethics and several
other matters that are helpful in governing a kingdom.

25.

Solution: d)

He did not believe in idol worship and all the rituals propagated by the Brahmins.
He believed in the Vedic truth of one God. Salvation for him was to be attained while living in the
world with right conduct.
He rejected that attaning Moksha or liberation was the aim of life.

26.

Solution: d)

http://www.insightsonindia.com INSIGHTS

Page 7

INSIGHTS MOCK TEST SERIES 2015: TEST 33 SOLUTIONS


In June 1920, Jawaharlal Nehru began going around the villages in Awadh, talking to the villagers,
and trying to understand their grievances. By October, the Oudh Kisan Sabha was set up headed by
Jawaharlal Nehru, Baba Ramchandra and a few others.
Within a month, over 300 branches had been set up in the villages around the region. So when the
Non- Cooperation Movement began the following year, the effort of the Congress was to integrate
the Awadh peasant struggle into the wider struggle. The peasant movement, however, developed in
forms that the Congress leadership was unhappy with. As the movement spread in 1921, the
houses of talukdars and merchants were attacked, bazaars were looted, and grain hoards were
taken over .

27.

Solution: b)

Not all social groups were moved by the abstract concept of swaraj. One such group was the
nations untouchables, who from around the 1930s had begun to call themselves dalit or
oppressed. Congress had ignored the dalits for long, for fear of offending the sanatanis, the
conservative high-caste Hindus.
Thus, they began organising themselves, demanding reserved seats in educational institutions, and
a separate electorate that would choose dalit members for legislative councils. Political
empowerment, they believed, would resolve the problems of their social disabilities.
Dalit participation in the Civil Disobedience Movement was therefore limited, particularly in the
Maharashtra and Nagpur region where their organisation was quite strong.

28.

Solution: d)

In Kheda, Gujarat, the peasants were frequently plagued by poverty, famines, scant resource,
untouchability, alcoholism and British discrimination. The famine of Chhappania Akal and some
subsequent famines had destroyed the agrarian economy of the region and the peasants were still
dying out of starvation.
Yet the authorities refused to grant remission of revenue. Gandhiji thus led the movement as the
spiritual leader of the peasants, with Sardar Patel.

29.

Solution: c)

The All India Home Rule League was a national political organisation founded on april 1916 to lead
the national demand for self-government, termed Home Rule, and to obtain the status of a
Dominion within the British Empire as enjoyed by Australia, Canada, South Africa, New Zealand
and Newfoundland at the time.

http://www.insightsonindia.com INSIGHTS

Page 8

INSIGHTS MOCK TEST SERIES 2015: TEST 33 SOLUTIONS


30.

Solution: a)

The "Berlin committee for Indian independence" was established in 1915 by Virendra Nath
Chattopadhya, including Bhupendra Nath Dutt & Lala Hardayal under "Zimmerman plan" with
the full backing of German foreign office.
Their goal was mainly to achieve the following four objectives:
1: Mobilize Indian revolutionaries abroad. 2: Incite rebellion among Indian troops stationed
abroad 3: Send volunteers and arms to India. 4: Even to Organized an armed invasion of British
India to gain India's independence and sent British back to home.

31. Solution: d)
At Barins suggestion Aurobindo Ghosh agreed to the starting of a paper, Yugantar, which was to
preach open revolt and the absolute denial of the British rule and include such items as a series of
articles containing instructions for guerrilla warfare.

32.

Solution: c)

Vande Mataramliterally, "I praise thee, Mother"is a poem from Bankim Chandra Chatterjee's
1882 novel Anandamath.
It was written in Bengali and Sanskrit.
It is a hymn to the Mother Land. It played a vital role in the Indian independence movement, first
sung in a political context by Rabindranath Tagore at the 1896 session of the Indian National
Congress. Thereafter, it became popular as the theme song of the Swdeshi movement. The
movement was also called as the Vandemataram movement.

33.

Solution: c)

During the 1870s young leaders in Bombay also established a number of provincial political
associations, such as the Poona Sarvajanik Sabha (Poona Public Society), founded by Mahadev
Govind Ranade (18421901).
It started with the aim of working as a mediating body between the government and people of
India. It started as an elected body of 95 members elected by 6000 persons on April 2, 1870. The
organisation was a precursor to the Indian National Congress which started with its first session
from Maharashtra itself.

34.

Solution: a)

http://www.insightsonindia.com INSIGHTS

Page 9

INSIGHTS MOCK TEST SERIES 2015: TEST 33 SOLUTIONS


Option A does not cover every area of moral policing. You are to choose the most appropriate
option.
This article clears the idea with reference to pornography. Ban on child pornography was recently
in news.
http://www.thehindu.com/todays-paper/not-for-moral-policing-centre/article7523707.ece

35.

Solution: b)

On this day in 1893, Sir Rudalph Diesel (inventor of the diesel engine) for the first time
successfully ran mechanical engine with Peanut Oil. His research experiment had predicted that
vegetable oil is going to replace the fossil fuels in the next century to fuel different mechanical
engines. Thus to mark this extraordinary achievement, World Biofuel Day is observed every year
on 10th August.
The Ministry of Petroleum and Natural Gas celebrated World Bio Fuel Day on Monday with
Minister Dharmendra Pradhan announcing the availability of high-speed diesel (HSD) blended
with bio-diesel at select outlets around the country.
This is the first time in India that biofuel-blended diesel will be sold in retail outlets.
www.thehindu.com/todays-paper/tp-national/biodiesel-being-sold-at-selectpumps/article7523658.ece

36.

Solution: d)

Aims to provide educational intervention by giving the bridge courses to the trainees and
getting them Certificates for Class XII and X from distance medium educational system.

Aims to provide trade basis skill training in four courses at the same time of formal
education, in field of a) Manufacturing b) Engineering c) Services d) Soft skills.

It intends to cover people in between 17 to 35 age group from all minority communities as
well as Madrasa students.

This scheme will provide avenues for continuing higher education and also open up employment
opportunities in the organised sector.

37.

Solution: d)

Initially ISRO imported the raw material for aerospace grade Titanium alloys with high purity
Titanium sponge from countries like Russia, China and Japan despite the fact that India is
endowed with the third largest reserve of Titanium bearing minerals.
http://www.insightsonindia.com INSIGHTS

Page 10

INSIGHTS MOCK TEST SERIES 2015: TEST 33 SOLUTIONS


38.

Solution: c)

The app was launched in Bihar that aims to help people learn about computers and Internet
through self learning modules. The app envisages offering digital literacy to tribal and Dalit
women in the State which is free and available on Google app store.
Telecom ministry also launched Disha handbook that has pictographic lessons on computers
operations. This initiative was taken by Ministry to spread out digital literacy in women from tribal
and Dalit background in Bihar.

39.

Solution: d)

Union Government has decided to allow Aadhaar enrollment of Persons of Indian Origin (PIOs),
Overseas Citizen of India (OCIs) and Non-Resident Indians (NRIs).
It was announced by Union Planning Minister Rao Inderjit Singh in written reply to Lok Sabha.
This move will help all persons on the Indian soil (including resident Indian, OCI/PIO, NRIs and
visa holding visitors) to authenticate their identity digitally.

40.

Solution: d)

It is remotely piloted high speed target drone system developed by Aeronautical Development
Establishment, a Bengaluru unit of DRDO in 1985. It has two variant commissioned in the armed
forces Lakshya 1 & 2.
Specification:

It can be launches from ground or ship and requires zero length launcher.

Recovered by two stages parachute and sea.

Its flight path can be pre-programmed based upon the type of mission.

It is seven feet long flies with the maximum speed of Mach 0.7.

Has ability to fly at low level and is equipped with cruise missile also.

41. Solution: c)
Egypt has recently launched a major expansion of the Suez Canal.

Suez Canal is an artificial sea-level waterway in Egypt. It links the Mediterranean with the
Red Sea through Isthmus of Suez. It was first opened in 1869 after a decade of construction.

http://www.insightsonindia.com INSIGHTS

Page 11

INSIGHTS MOCK TEST SERIES 2015: TEST 33 SOLUTIONS


Strategically and economically it is one of the most important waterways in the world
providing the shortest sea link between Asia and Europe

The canal was nationalised by Egypt in 1956 after brief war against the UK, France and Israel. The
canal has been a significant income source for Egypt since then

42.

Solution: c)

The draft agreement outlines 17 non-binding goals with 169 specific targets that comprise different
socio-economic issues. These goals will succeed 8 MDGs which was adopted in 2000.
It has broader sustainability agenda dealing with five Ps people, planet, prosperity, peace, and
partnership. It seeks to address the universal need for development that works for all people and
root causes of poverty.
It calls for eradicating hunger and poverty, improving living standards, ensuring quality education,
affordable and reliable energy, achieving gender equality and taking urgent action to combat
climate change etc.

43.

Solution: a)

The South Pole lies almost at the centre of Antarctica.


As it is located in the South Polar Region, it is permanently covered with thick ice sheets. There are
no permanent human settlements.
Many countries have research stations in Antarctica. India also has research stations there. These
are named as Maitri and Dakshin Gangotri.

44.

Solution: d)

In your 7th NCERT books, Switzerland is given as the answer. But it is wrong. Check out this link.
Recently Gold has been discovered. It does not mine Gold though.
http://www.myswitzerland.com/en-in/minerals-paradise-binn-valley.html
http://www.azomining.com/Article.aspx?ArticleID=125

45.

Solution: d)

Mangoes grow on trees that range from 30 to 100 feet. Native to India, mangoes have been grown
since prehistoric times.
http://www.insightsonindia.com INSIGHTS

Page 12

INSIGHTS MOCK TEST SERIES 2015: TEST 33 SOLUTIONS


Lotus are native to India and are, in fact, sacred to a number of Indian religions, including
Buddhism.
Jasmine is a very fragrant, evergreen vine or shrub. Jasmine is native to India and other tropical
areas of Asia, Africa and Australia. There are many varieties of jasmine, with over 150 varieties
known.
46.

Solution: a)

Flood Basalt Provinces are volcanoes outpour highly fluid lava that flows for long distances. Some
parts of the world are covered by thousands of sq. km of thick basalt lava flows. There can be a
series of flows with some flows attaining thickness of more than 50 m. Individual flows may extend
for hundreds of km.
The Deccan Traps from India, presently covering most of the Maharashtra plateau, are a much
larger flood basalt province. It is believed that initially the trap formations covered a much larger
area than the present.

47.

Solution: b)

Wegener suggested that the movement responsible for the drifting of the continents was caused by
pole-fleeing force and tidal force.
The polar-fleeing force relates to the rotation of the earth. You are aware of the fact that the earth
is not a perfect sphere; it has a bulge at the equator. This bulge is due to the rotation of the earth.

48.

Solution: b)

The mapping of the ocean floor and palaeomagnetic studies of rocks from oceanic regions revealed
the following facts:

It was realised that all along the mid-oceanic ridges, volcanic eruptions are common and
they bring huge amounts of lava to the surface in this area.
The rocks equidistant on either sides of the crest of mid-oceanic ridges show remarkable
similarities in terms of period of formation, chemical compositions and magnetic
properties. Rocks closer to the mid-oceanic ridges are normal polarity and are the youngest.
The age of the rocks increases as one moves away from the crest.
The ocean crust rocks are much younger than the continental rocks.

49.

Solution: c)

Rocks (igneous, sedimentary and metamorphic) of the earths surface are exposed to denudational
agents, and are broken up into various sizes of fragments.
http://www.insightsonindia.com INSIGHTS

Page 13

INSIGHTS MOCK TEST SERIES 2015: TEST 33 SOLUTIONS


Such fragments are transported by different exogenous agencies and deposited.


These deposits through compaction turn into rocks. This process is called lithification. In many
sedimentary rocks, the layers of deposits retain their characteristics even after lithification.
Hence, we see a number of layers of varying thickness in sedimentary rocks like sandstone, shale
etc.

50.

Solution: d)

Weathering mantles with different profiles have various mineral and chemical zones that give way
to one another on the vertical, from the slightly altered bedrock to the intensively altered
outcropping rocks, and that are characteristic of the particular mantle.
The formation of the weathering mantle depends on climate, the composition of the bedrock,
hydrogeological conditions, the relief of the terrain, tectonic structure, the duration of
development, the age during which development occurred, and degree of mobility of the earths
crust.
The thickest weathering mantles form in periods of tectonic inactivity in regions with moist, warm
climates. Decomposition of a large mass of organic matter leads to the formation of CO2 and
organic acids that filter from the soil into the weathering mantle, decompose rocks at a deep level,
and leach out the soluble products of weathering.

51. Solution: a)
When river enters a bend, the velocity of water is naturally higher at one end of the bank than the
other end of the bank.
This difference causes erosion of bank at one side due to higher velocity. The other side being at
lower velocity causes deposition of material.

52.

Solution: d)

The troposphere is the lowermost layer of the atmosphere. Its average height is 13 km and extends
roughly to a height of 8 km near the poles and about 18 km at the equator.
This layer contains dust particles and water vapour.
All changes in climate and weather take place in this layer. The temperature in this layer decreases
at the rate of 1C for every 165m of height. This is the most important layer for all biological
activity.

http://www.insightsonindia.com INSIGHTS

Page 14

INSIGHTS MOCK TEST SERIES 2015: TEST 33 SOLUTIONS


53.

Solution: c)

The insolation received at the surface varies from about 320 Watt/m2 in the tropics to about 70
Watt/m2 in the poles.
Maximum insolation is received over the subtropical deserts, where the cloudiness is the least.
Equator receives comparatively less insolation than the tropics.
Generally, at the same latitude the insolation is more over the continent than over the oceans.
In winter, the middle and higher latitudes receive less radiation than in summer.

54.

Solution: d)

When the moisture is deposited in the form of water droplets on cooler surfaces of solid objects
(rather than nuclei in air above the surface) such as stones, grass blades and plant leaves, it is
known as dew. The ideal conditions for its formation are clear sky, calm air, high relative
humidity, and cold and long nights.
For the formation of dew, it is necessary that the dew point is above the freezing point.

55.

Solution: b)

Between the latitudes 35 and 40 N and S of the equator, the rain is heavier on the eastern coasts
and goes on decreasing towards the west. But, between 45 and 65 N and S of equator, due to the
westerlies, the rainfall is first received on the western margins of the continents and it goes on
decreasing towards the east.
The central parts of the tropical land and the eastern and interior parts of the temperate lands
receive rainfall varying between 50 - 100 cm per annum. Here moisture also an important reason.

56.

Solution: c)

Mediterranean climate occurs around Mediterranean sea, along the west coast of continents in
subtropical latitudes between 30 - 40 latitudes e.g. Central California, Central Chile, along the
coast in south eastern and south western Australia.
These areas come under the influence of sub tropical high in summer and westerly wind in winter.
Hence, the climate is characterised by hot, dry summer and mild, rainy winter. Monthly average
temperature in summer is around 25 C and in winter below 10C. The annual precipitation ranges
between 35 - 90 cm.

57.Solution: d)
http://www.insightsonindia.com INSIGHTS

Page 15

INSIGHTS MOCK TEST SERIES 2015: TEST 33 SOLUTIONS


Factors affecting ocean salinity are mentioned below:

The salinity of water in the surface layer of oceans depends mainly on evaporation and
precipitation.
Surface salinity is greatly influenced in coastal regions by the fresh water flow from rivers,
and in polar regions by the processes of freezing and thawing of ice.
Wind, also influences salinity of an area by transferring water to other areas.
The ocean currents contribute to the salinity variations. Salinity, temperature and density of
water are interrelated.
Hence, any change in the temperature or density influences the salinity of an area.

58.

Solution: d)

The vertical motion refers to the rise and fall of water in the oceans and seas. Due to attraction of
the sun and the moon, the ocean water is raised up and falls down twice a day.
More saline water subsides down in the ocean and less dense water comes on the surface. It is a
dynamic process.
The upwelling of cold water from subsurface and the sinking of surface water are also forms of
vertical motion of ocean water.

59.

Solution: a)

This is from your NCERT book. It has been listed by IUCN as critically endangered.

60.

Solution: a)

http://www.insightsonindia.com INSIGHTS

Page 16

INSIGHTS MOCK TEST SERIES 2015: TEST 33 SOLUTIONS


61. Solution: d)
A study of rainfall patterns does not reveal the whole truth behind the water availability in various
regions in India. Rains in India are largely due to the monsoons. This means that most of the rain
falls in a few months of the year.
Despite natures monsoon bounty, failure to sustain water availability underground has resulted
largely from the loss of vegetation cover, diversion for high water demanding crops, and pollution
from industrial effluents and urban wastes. Irrigation methods like dams, tanks and canals have
been used in various parts of India since ancient times.

62.

Solution: d)

The Chipko Andolan was the result of a grassroot level effort to end the alienation of people from
their forests. The movement originated from an incident in a remote village called Reni in
Garhwal, high-up in the Himalayas during the early 1970s.
There was a dispute between the local villagers and a logging contractor who had been allowed to
fell trees in a forest close to the village.

http://www.insightsonindia.com INSIGHTS

Page 17

INSIGHTS MOCK TEST SERIES 2015: TEST 33 SOLUTIONS


Inherent in such a competition to control a natural resource is the conservation of a replenishable


resource. Specifically the method of use was being called into question.
The Chipko movement quickly spread across communities and media, and forced the government,
to whom the forest belongs, to rethink their priorities in the use of forest produce.

63.

Solution: a)

The Ganga Action Plan, a multi-crore project came about in 1985 because the quality of the water
in the Ganga was very poor.
Coliform is a group of bacteria, found in human intestines, whose presence in water indicates
contamination by disease-causing microorganisms.
The coliform level in Ganga increased so much that it indicated the water to be unsafe for humans.

64.

Solution: a)

Concave mirror can direct divergent light beams into a point of light.
This point of light has the concentrated energy of suns rays which increases the energy efficiency
of the solar cooker.

65.

Solution: d)

The water at the surface of the sea or ocean is heated by the Sun while the water in deeper sections
is relatively cold. This difference in temperature is exploited to obtain energy in ocean-thermalenergy conversion plants.
These plants can operate if the temperature difference between the water at the surface and water
at depths up to 2 km is 293 K (20C) or more. The warm surface-water is used to boil a volatile
liquid like ammonia. The vapours of the liquid are then used to run the turbine of generator. The
cold water from the depth of the ocean is pumped up and condense vapour again to liquid.

66.

Solution: d)

In a bio-gas plant, a slurry of cow-dung and water is made in the mixing tank from where it is fed
into the digester. The digester is a sealed chamber in which there is no oxygen.
Anaerobic micro-organisms that do not require oxygen decompose or break down complex
compounds of the cow-dung slurry. It takes a few days for the decomposition process to be
complete and generate gases like methane, carbon dioxide, hydrogen and hydrogen sulphide.

http://www.insightsonindia.com INSIGHTS

Page 18

INSIGHTS MOCK TEST SERIES 2015: TEST 33 SOLUTIONS


67.

Solution: a)

Large amount of fossil fuels are burnt every day in power stations to heat up water to produce
steam which further runs the turbine to generate electricity. The transmission of electricity is more
efficient than transporting coal or petroleum over the same distance. Therefore, many thermal
power plants are set up near coal or oil fields. The term thermal power plant is used since fuel is
burnt to produce heat energy which is converted into electrical energy.
68.

Solution: a)

The difference between the direct and alternating currents is that the direct current always flows in
one direction, whereas the alternating current reverses its direction periodically.
Most power stations constructed these days produce AC. In India, the AC changes direction after
every 1/100 second, that is, the frequency of AC is 50 Hz. An important advantage of AC over DC is
that electric power can be transmitted over long distances without much loss of energy.

69.

Solution: c)

70.

Solution: a)

The molecules of air and other fine particles in the atmosphere have size smaller than the
wavelength of visible light. These are more effective in scattering light of shorter wavelengths at
the blue end than light of longer wavelengths at the red end. The red light has a wavelength about
1.8 times greater than blue light. Thus, when sunlight passes through the atmosphere, the fine
particles in air scatter the blue colour (shorter wavelengths) more strongly than red.
The scattered blue light enters our eyes. If the earth had no atmosphere, there would not have been
any scattering. Then, the sky would have looked dark. The sky appears dark to passengers flying at
very high altitudes, as scattering is not prominent at such heights.

71. Solution: d)
http://www.insightsonindia.com INSIGHTS

Page 19

INSIGHTS MOCK TEST SERIES 2015: TEST 33 SOLUTIONS


A rainbow is a natural spectrum appearing in the sky after a rain shower. It is caused by dispersion
of sunlight by tiny water droplets, present in the atmosphere. A rainbow is always formed in a
direction opposite to that of the Sun.
The water droplets act like small prisms. They refract and disperse the incident sunlight, then
reflect it internally, and finally refract it again when it comes out of the raindrop. Due to the
dispersion of light and internal reflection, different colours reach the observers eye.

72.

Solution: b)

When light is coming from one side of the plant, auxin diffuses towards the shady side of the shoot.
This concentration of auxin stimulates the cells to grow longer on the side of the shoot which is
away from light. Thus, the plant appears to bend towards light.

73.

Solution: c)

Animals have evolved different organs for the uptake of oxygen from the environment and for
getting rid of the carbon dioxide produced. Terrestrial animals can breathe the oxygen in the
atmosphere, but animals that live in water need to use the oxygen dissolved in water.
Since the amount of dissolved oxygen is fairly low compared to the amount of oxygen in the air, the
rate of breathing in aquatic organisms is much faster than that seen in terrestrial organisms.
Fishes take in water through their mouths and force it past the gills where the dissolved oxygen is
taken up by blood.

74.

Solution: c)

Saturated hydrocarbons will generally give a clean flame while unsaturated carbon compounds will
give a yellow flame with lots of black smoke. This results in a sooty deposit on a metal plate.
However, limiting the supply of air leads to incomplete combustion of even saturated
hydrocarbons thus giving a sooty flame.

http://www.insightsonindia.com INSIGHTS

Page 20

INSIGHTS MOCK TEST SERIES 2015: TEST 33 SOLUTIONS


The gas/kerosene stove used at home has inlets for air so that a sufficiently oxygen-rich mixture is
burnt to give a clean blue flame. If you observe the bottoms of cooking vessels getting blackened, it
means that the air holes are blocked and fuel is getting wasted.

75.Solution: c)
All living structures are carbon based. The amount of carbon present in the earths crust and in the
atmosphere is quite meagre. The earths crust has only 0.02% carbon in the form of minerals (like
carbonates, hydrogencarbonates, coal and petroleum) and the atmosphere has 0.03% of carbon
dioxide.
Oxygen and silica are the most abundant on earths crust. In spite of this small amount of carbon
available in nature, the importance of carbon seems to be immense.

76.

Solution: a)

Article 262 of the Constitution provides for the adjudication of inter-state water disputes.
Under this provision, the Parliament has enacted two laws [the River Boards Act (1956) and the
Inter-State Water Disputes Act (1956)].
The River Boards Act provides for the establishment of river boards for the regulation and
development of inter-state river and river valleys. A river board is established by the Central
government on the request of the state governments concerned to advise them.

77. Solution: d)
The Comptroller and Auditor-General of India audits the accounts of not only the Central
government but also those of the states.
But, his appointment and removal is done by the president without consulting the states.
Hence, this office restricts the financial autonomy of the states. The American ComptrollerGeneral, on the contrary, has no role with respect to the accounts of the states.

78.

Solution: b)

The term preamble refers to the introduction or preface to the Constitution. It contains the
summary or essence of the Constitution.
The Preamble to the Indian Constitution is based on the Objectives Resolution, drafted and
moved by Pandit Nehru, and adopted by the Constituent Assembly. It has been amended by the

http://www.insightsonindia.com INSIGHTS

Page 21

INSIGHTS MOCK TEST SERIES 2015: TEST 33 SOLUTIONS


42nd Constitutional Amendment Act (1976), which added three new wordssocialist, secular and
integrity.
79.

Solution: c)

Export promotion and import control will reduce the trade deficit an important component of
BoP deficit.
Currency devaluation will make exports competitive thus increasing exports and reducing the
trade deficit.
Foreign exchange reserves are helpful to bridge (not reduce) the BoP in case there is a shortage of
foreign capital in India. It is also helpful in external crisis.

80.

Solution: d)

Governments revenues and receipts affect government spending. This in turn in affected by
taxation policies; amount of government spending etc.
Borrowing is also an important fiscal policy tool as it increases/decreases money supply in the
market.
All of these combine to form Public Debt which is the centre of fiscal policy.

81.Solution: a)
Speculative demand is the demand for financial assets, such as securities, money, or foreign
currency, that is not dictated by real transactions such as trade or financing.
Speculative demand arises from the need for cash to take advantage of investment opportunities
that may arise.
In economic theory, specifically Keynesian economics, speculative demand is one of the
determinants of demand for money (and credit), the others being transactions demand and
precautionary demand.
Speculative demand refers to real balances held for the purpose of avoiding capital loss from
holding bonds. The net return on bonds is the sum of the interest payments and the capital gains
(or losses) from their varying market value. A rise in interest rate causes aftermarket bond prices
to fall, and that implies a capital loss from holding bonds. Accordingly, the return on bonds can be
negative. Thus, people may hold money to avoid the loss from bonds.

82.

Solution: a)

http://www.insightsonindia.com INSIGHTS

Page 22

INSIGHTS MOCK TEST SERIES 2015: TEST 33 SOLUTIONS


Disguised unemployment is unemployment that does not affect aggregate output. Disguised
unemployment exists where part of the labour force is either left without work or is working in a
redundant manner where worker productivity is essentially zero.
So addition of one more (marginal) worker will add noting to the productivity of the farm (which
means the marginal productivity of the labour is zero). An economy demonstrates disguised
unemployment where productivity is low and where too many workers are filling too few jobs.

83.

Solution: a)

It is a situation where bank cash-holdings are rising and banks cannot find sufficient number of
qualified borrowers even at extraordinary low rates of interest.
It usually arises where people are not buying and firms are not borrowing (for inventory or plant
and equipment) because economic prospects look dim, investors are not investing because
expected returns from investments are low, and/or a recession is beginning. People and businesses
hold on to their cash and thus get trapped in a self-fulfilling prophecy. And everyone expects the
interest rates to rise hoping the situation to become better.

84.

Solution: a)

The pace at which GR unfolded in the 1960s is not present now. New inventions in biotechnology
are being resisted; field testing is low; and consequently very few highly productive GM crop is
being used in farms.
Since 1991 public investment has been very low which is expected to be replaced by private
investment. But private investment has mostly come in producing agricultural pumps; developing
GM seeds and not in the form of increased farmer access to markets; increased investment in land
improvement and like.

85.

Solution: d)

The primary policy objective of the Department of Food & Public Distribution is to ensure food
security for the country through timely and efficient procurement and distribution of foodgrains.
This involves procurement of various foodgrains, building up and maintenance of food stocks,
their storage, movement and delivery to the distributing agencies and monitoring of production,
stock and price levels of foodgrains.

86.

Solution: b)

http://www.insightsonindia.com INSIGHTS

Page 23

INSIGHTS MOCK TEST SERIES 2015: TEST 33 SOLUTIONS


Some of the major preventive machinery for handling industrial disputes in India are as follows: 1.
Workers Participation in Management 2. Collective Bargaining 3. Grievance Procedure 4.
Tripartite Bodies 5. Code of Discipline 6. Standing Orders.
Read more about them here:
http://www.yourarticlelibrary.com/industries/6-preventive-machineries-used-for-handlingindustrial-disputes-in-india/27996/

87.

Solution: d)

http://www.yourarticlelibrary.com/education/sebi-the-purpose-objective-and-functions-ofsebi/8762/
Futures market is regulated by the Forward Markets Commission. There is a proposal by FSRLC to
merge these regulators with IRDA and PFRDA. Read the article above to understand all the details
about SEBI.

88.

Solution: b)

Price rigging refers to manipulating the prices of securities with the main objective of inflating or
depressing the market price of securities. SEBI prohibits such practice because this can defraud
and cheat the investors.
On the other hand, Insider is any person connected with the company such as directors, promoters
etc. These insiders have sensitive information which affects the prices of the securities. This
information is not available to people at large but the insiders get this privileged information by
working inside the company and if they use this information to make profit, then it is known as
insider trading.

89.

Solution: d)

It also provides these services:

Managing the public debt and the issue of new loans and Treasury Bills of the Central
Government.
Providing ways and means advances to the Central and State Governments to bridge the
interval between expenditure and flow of receipts of revenue. Such advances are to be
repaid by the government within three months from the date of borrowal.
Advising the Central and State Governments on financial matters, such as the quantum,
timing and terms of issue of new loans. For ensuring the success of government loan
operations, the RBI plays an active role in the gilt-edged market.

http://www.insightsonindia.com INSIGHTS

Page 24

INSIGHTS MOCK TEST SERIES 2015: TEST 33 SOLUTIONS


The bank also tenders advice to the government on policies concerning banking and
financial issues, planning as well as resource mobilisation.
The Reserve Bank represents the Government of India as member of the International
Monetary Fund and the World Bank.

90.

Solution: d)

In India, to check the rise in prices of food-grains, edible oils, sugar etc., the Government has often
taken steps to increase imports of goods in short supply to enlarge their available supplies.
When inflation is of the type of supply-side inflation, imports are increased to augment the
domestic supplies of goods. To increase imports of goods in short supply the Govern-ment reduces
customs duties on them so that their imports become cheaper and help in containing inflation.
For example in 2008-09 the Indian Government removed customs duties on imports of wheat
and rice and reduced them on oilseeds, steel etc. to increase their supplies in India.

91. Solution: c)
It is a trade that profits by exploiting price differences of identical or similar financial instruments,
on different markets or in different forms. Arbitrage exists as a result of market inefficiencies; it
provides a mechanism to ensure prices do not deviate substantially from fair value for long periods
of time.

92.

Solution: c)

Refer to the link. Also refer to the next sub-heading in the article.
https://en.wikipedia.org/wiki/Free-radical_theory_of_aging#Effects_of_calorie_restriction

93.

Solution: c)

A transgene is a gene or genetic material that has been transferred naturally, or by any of a number
of genetic engineering techniques from one organism to another. The introduction of a transgene
has the potential to change the phenotype of an organism.
In its most precise usage, the term transgene describes a segment of DNA containing a gene
sequence that has been isolated from one organism and is introduced into a different organism.
This non-native segment of DNA may retain the ability to produce RNA or protein in the
transgenic organism, or it may alter the normal function of the transgenic organism's genetic code.

http://www.insightsonindia.com INSIGHTS

Page 25

INSIGHTS MOCK TEST SERIES 2015: TEST 33 SOLUTIONS


94.

Solution: d)

Biopesticides are certain types of pesticides derived from such natural materials as animals, plants,
bacteria, and certain minerals. Biopesticides fall into three major classes:

Microbial pesticides consist of a microorganism (e.g., a bacterium, fungus, virus or


protozoan) as the active ingredient.
Plant-Incorporated-Protectants (PIPs) are pesticidal substances that plants produce from
genetic material that has been added to the plant.
Biochemical pesticides are naturally occurring substances that control pests by non-toxic
mechanisms. Conventional pesticides, by contrast, are generally synthetic materials that
directly kill or inactivate the pest. Biochemical pesticides include substances, such as insect
sex pheromones, that interfere with mating, as well as various scented plant extracts that
attract insect pests to traps.

95.

Solution: b)

In biology, a subculture is a new cell or microbiological culture made by transferring some or all
cells from a previous culture to fresh growth medium.
This action is called subculturing or passaging the cells.
Subculture is used to prolong the life and/or expand the number of cells or microorganisms in the
culture.

96.

Solution: c)

Insects, mites, nematodes, fungi, bacteria, viruses, MLOs and other organisms are known to attack
various crops of economic importance. These pests and pathogens not only reduce the quantity but
also spoil the quality of the produce to a considerable extent. These may be carried with the plants
that enter a national border.
Plant quarantine may, therefore, be defined as 'Rules and regulations promulgated by
governments to regulate the introduction of plants, planting materials, plant products, soil, living
organisms, etc. with a view to prevent inadvertent introduction of exotic pests, weeds and
pathogens harmful to the agriculture or the environment of a country/region, and if introduced, to
prevent their establishment and further spread'. Plant quarantine is thus designed as a safeguard
against harmful pests/pathogens exotic to a country or a region.

97.

Solution: d)

The first two sub-headings and introduction should be read.


http://bugs.bio.usyd.edu.au/learning/resources/PlantPathology/infection/infection_process.html
http://www.insightsonindia.com INSIGHTS

Page 26

INSIGHTS MOCK TEST SERIES 2015: TEST 33 SOLUTIONS


98.

Solution: a)

The ozone layer is typically thicker over the poles than over the equator for three reasons. First,
there is a lack of sunlight during an arctic winter to break it down.
Second, seasonal weather systems and wind patterns in the upper atmosphere push more ozone
toward the poles in winter.
Third, the vertical structure of the atmosphere affects thickness. Near the equator, the troposphere
makes up about 18 kilometres of the atmosphere. At the poles, it thins to only 8-kilometres thick.
The boundary between the layers, called the tropopause, slopes downward from the equator to the
poles like the bottom of a swimming pool. Over the Arctic, at the deepest part of the "pool", the
thicker stratosphere can hold more ozone than over the Tropics or middle latitudes.
The atmosphere over the Arctic also holds more ozone than in the Antarctic, because stratospheric
winds in the Southern Hemisphere are less effective at transporting ozone.

99.

Solution: d)

Several low lying islands like Maldives (tourism hub) may be submerged due to sea-level rise.
Seawater will inundate into fresh groundwater resources because of a higher sea water pressure
due to higher levels.
Inundation of coastal marshes and estuaries may deprive several bird an fish species of their
breeding grounds.

100.

Solution: d)

Nitrogen fertilizers help produce N2O which is a greenhouse gas.


Increasing forest cover will act as a carbon sink reducing the amount of CO2 in the atmosphere.
Urban heat islands preserve the day heat at night too and intensify the heat patterns caused by
global warming. Reducing them will lower the average temperature of cities.

http://www.insightsonindia.com INSIGHTS

Page 27

INSIGHTS MOCK TEST SERIES 2015: TEST 33 SOLUTIONS



http://www.insightsonindia.com INSIGHTS

Page 28

You might also like